Você está na página 1de 1582

AFA 2024

ENGLISH GENERAL PRESENTATION

AULA 00

Teacher Andrea Belo

www.estrategiamilitares.com.br www.militares.estrategia.com
TEACHER ANDREA BELO

SUMÁRIO
APRESENTAÇÃO 3

A PROVA DE INGLÊS DA AFA 4

METODOLOGIA 6

PLANEJAMENTO DAS AULAS 7

INTERPRETAR TEXTOS – PRIMEIROS PASSOS PRÁTICOS 11

SKIMMING 12

SCANNING 13

FALSO COGNATO – SIMPLES DE FATO 14

GRAMÁTICA DE FORMA PRÁTICA 15

O TEMPO VERBAL DE FORMA NATURAL 16

INTERPRETAR IMAGEM COM APRENDIZAGEM 18

EXPRESSÕES IDIOMÁTICAS SIMPÁTICAS 18

QUESTÕES 21

GABARITO 52

QUESTÕES COMENTADAS 53

CONSIDERAÇÕES FINAIS 110

REFERÊNCIAS BIBLIOGRÁFICAS 111

TRADUÇÕES 113

AULA 00 – ENGLISH GENERAL PRESENTATION 2


TEACHER ANDREA BELO

APRESENTAÇÃO
Hello, dear student!!! Welcome to the success!
Bem-vindo ao sucesso? Isso mesmo. Com esse curso de Inglês do Estratégia Militares, a
disciplina de língua inglesa ficará simples e será um diferencial para seus estudos.
Isso porque preparei esse material usando toda minha experiência e background para
desenvolver as melhores aulas que você possa ter.
O meu nome é Andrea Belo e minha formação acadêmica é composta por:
✓ graduação em Letras pela Universidade Federal de Goiás (UFG/GO);
✓ pós-graduação em Linguística Aplicada à Língua Inglesa pela PUC/SP;
✓ pós-graduação em Educação para idiomas pela Cambridge University, na Inglaterra;
✓ especialização em Didática e Prática em língua inglesa pelo LAL/Londres, escola conveniada
à Oxford University, na Inglaterra;
✓ especialização Higher Education Teaching Certificate pela NYU – New York University, em
N.Y./EUA;
✓ certificação Cambridge nos exames FCE, CAE e CPE pela escola de idiomas Cultura Inglesa;
✓ certificação MBA em English Studies – Language program pela FUB – Freie Universität
Berlin, em Berlim/Alemanha;
✓ certificação em Bilingual Education, curso CIP – Cultural Immersion Program, em Fort
Myers, Flórida/EUA;
Além da formação acadêmica, viajei e morei em oito diferentes países, participei de
seminários e workshops em cada país, buscando fluência sempre. Sou também tradutora e
intérprete habilitada e vou ajudá-lo a realizar a prova de Inglês, fazendo o papel de facilitadora,
para que você consiga sua aprovação com esse material, que é prático e intenso, porém eficaz.
Desenvolvi, para você, uma maneira de transformar essa disciplina em algo atrativo e eficaz.
Siga as redes sociais como complementos de estudos e... let’s go!

AULA 00 – ENGLISH GENERAL PRESENTATION 3


TEACHER ANDREA BELO

A PROVA DE INGLÊS DA AFA


A Academia da Força Aérea – AFA, localizada em Pirassununga, Estado de São Paulo, tem
como finalidade a formação de Oficiais Aviadores, Intendentes e de Infantaria da FAB.
A história das escolas de formação de aviadores militares no Brasil remonta ao ano de
1913, quando a Escola Brasileira de Aviação foi fundada, no Campo dos Afonsos, Estado do Rio de
Janeiro.
Essa escola, cujo objetivo era dar formação semelhante àquela disponível nas melhores
escolas européias, operava um conjunto de aviões "Blériot" e "Farman", de fabricação francesa.
Com o advento da Iª Guerra Mundial, no entanto, a escola foi fechada.
À época, tanto o Exército Brasileiro quanto a Marinha do Brasil operavam suas próprias
armas aéreas, a Aviação Militar e a Aviação Naval. Hidroaviões Curtiss "F" foram adquiridos pela
Marinha em maio de 1916 para equipar esta última e, em agosto do mesmo ano, foi criada a
Escola de Aviação Naval.
A Aviação Militar, no entanto, só teve sua Escola de Aviação Militar criada após a guerra,
em 10 de julho de 1919. Até o início da década de 1940, ambas as escolas continuaram as suas
atividades.
O governo brasileiro, no entanto, atento ao desenrolar da guerra aérea na Europa, optou
por amalgamar sob um único comando as atividades aeronáuticas militares, criando o Ministério
da Aeronáutica a 20 de janeiro de 1941 e extinguindo as aviações Militar e Naval, cujos efetivos e
equipamentos passaram a formar a Força Aérea Brasileira.
Por conseguinte, no dia 25 de março de 1941, ambas escolas de formação de pilotos
militares foram extintas, e criou-se a Escola de Aeronáutica, localizada no Campo dos Afonsos,
cujos integrantes passaram a ser designados como Cadetes da Aeronáutica a partir de 1943.
Em 1942, evidenciou-se a necessidade de se transferir a Escola de Aeronáutica para um
local que apresentasse melhores condições climáticas e permitisse a instrução aérea dos futuros
pilotos com a menor interferência possível.
A cidade de Pirassununga foi a escolhida dentre tantas outras que foram avaliadas, e no
ano de 1952 iniciaram-se as obras de construção da nova Escola. A partir de 1960, iniciou-se a
transferência das atividades da Escola, do Campo dos Afonsos para a nova sede, a qual completou-
se em 1971, tendo a instrução aérea dos novos Cadetes sido transferida em 1964. No ano de 1969,
a Escola passou a ser denominada Academia da Força Aérea.
O lema da Academia é a expressão latina: “Macte Animo! Generose Puer, sic itur ad astra”,
extraída do poema Thebaida, de autoria do poeta latino Tatius. Essa exortação pode ser traduzida
por Coragem! É assim, nobre jovem, que se vai aos astros.
A instrução dos Cadetes da Aeronáutica, cujo curso tem a duração de quatro anos, é
centrada nas palavras CORAGEM – LEALDADE – HONRA – DEVER – PÁTRIA; aos futuros Oficiais
são ministradas disciplinas de nível universitário, como Cálculo Diferencial e Integral, Introdução
à Ciência dos Computadores, Mecânica e Línguas Portuguesa e Inglesa, por professores civis,
instrutores militares e monitores.

AULA 00 – ENGLISH GENERAL PRESENTATION 4


TEACHER ANDREA BELO

A instrução militar propriamente dita é dada diariamente, obedecendo a rígidos padrões,


onde os Cadetes são adestrados no paraquedismo, sobrevivência no mar e na selva, entre outras
atividades.
De acordo com a especialização escolhida, o Cadete receberá instrução diferenciada:
• Aviadores: Instrução em manobras de precisão, acrobacias, voos de formatura e por
instrumentos, com 75 horas de voo no avião de treinamento primário/básico T-25
Universal, iniciada no 2º semestre da 1ª série e completada na 3ª série, e 125 horas de voo
de treinamento avançado em aviões T-27 Tucano.
• Intendentes: Treinamento na ciência e tecnologia moderna da gestão econômico-
financeira e serviços de especializados de intendência e suprimento técnico. Apenas esta
especialização está aberta a Cadetes do sexo feminino.
• Infantaria da Aeronáutica: Instrução em métodos de defesa e segurança de instalações
militares, emprego de defesa antiaérea de aeródromos e sítios, comando de frações de
tropas e de equipes contra-incêndio, legislação militar, emprego de armamento, serviço
militar e mobilização.
Nas horas de lazer, os Cadetes participam das atividades de sete clubes diferentes:
Aeromodelismo, Literatura, Informática, Tiro, Centro de Tradições Gaúchas, Clube das Gerais e
Voo a Vela. Os clubes são dirigidos pelos próprios cadetes, sob orientação de oficiais da FAB.
Junto à AFA, estão sediados os Esquadrões de Instrução Aérea, responsáveis pela instrução
de voo dos futuros Oficiais Aviadores, bem como o Esquadrão de Demonstração Aérea –
"Esquadrilha da Fumaça".
As provas de Inglês da AFA têm um foco: avaliar o candidato que saiba compreender e
interpretar textos, com tópicos gramaticais contextualizados. Muitas vezes, há questões mais
aprofundadas na interpretação, necessitando ter um vasto vocabulário.
Nas provas, as questões avaliam a capacidade do candidato em ler, reconhecer palavras
que levam à ideia geral, inferir significados e sentir-se confiante em relação ao vocabulário dentro
do contexto. São diferentes porém, ambas exigentes.
O preparo para uma das provas dessas instituições, leva ao aprimoramento para realizar a
outra. E isso vai ajudar o candidato que fará ambas ou aprimorar os conteúdos para uma delas.
Se você já tem conhecimentos prévios, vai aprimorar vocabulário e revisar os conteúdos,
fazendo uma reciclagem eficaz de tudo que já sabe, ficando ainda mais bem preparado este ano.
Para ser aprovado, o seu objetivo é aprender tópicos que abrangem nossas aulas de Inglês,
usar as estratégias de leitura, e chegar à aprovação final.
E, a prática de resolução de exercícios é fundamental para se acostumar com o tempo.
Quanto menos tempo você gastar em Inglês, mais tempo terá para as outras provas.
Vamos começar? Are you ready? Come on!

AULA 00 – ENGLISH GENERAL PRESENTATION 5


TEACHER ANDREA BELO

METODOLOGIA
Você certamente está pensando: “Que metodologia faz com que Inglês seja simples?” O
método que uso nesse material permitirá que você consiga realizar sua prova de Inglês com
segurança.
Há termos gramaticais no índice das aulas, mas a gramática é explicada contextualizada,
conforme você precisa, com prática de exercícios em diferentes graus de dificuldade, desde
básicos até avançados, com técnicas de leitura essenciais e muito mais.
A metodologia que uso é, curiosamente, uma interdisciplinaridade entre diferentes
metodologias. Tradicional – o professor está no centro do processo educativo, responsável por
transmitir os conhecimentos aos alunos e faço isso às vezes.
Quando o ensino é dinâmico, com aprendizado construído aos poucos, chamamos de
metodologia construtivista, que também uso em nossas aulas. E, na metodologia
sociointeracionista, o professor assume o papel de mediador e estimula avanços frequentes, o
que também vamos fazer.
Com uso de metodologias interdisciplinares e meu método dinâmico, vou esclarecer suas
dúvidas, com explicações detalhadas. O segredo de sua aprovação está na segurança total de
como resolver cada exercício com eficácia, como faremos juntos.
E a maneira como eu explico o conteúdo, conectando diferentes tópicos gramaticais, que
geralmente são ensinados separadamente, será um método único, exclusivo e que vai garantir o
seu sucesso na prova de Inglês.
Outro aspecto muito importante, inclusive, um de nossos diferenciais, é que, você terá
acesso, no final de todas as suas aulas, às traduções de todos os textos usados nas provas
anteriores. Serão traduções das tirinhas, falas de cada personagem das imagens e todas as leituras
necessárias, com palavras e termos já usados em provas anteriores e aqui utilizados para você
praticar.
Apesar de aprender que não precisa traduzir inteiramente os textos para resolver as
questões, as traduções são um complemento para você aprender mais palavras e enriquecer o
seu vocabulário.
Então, use bem o tempo que antecede a prova para estudar com um material eficaz em
mãos. Estude, sempre. Assista jornais importantes, leia textos e artigos que são comuns fontes na
elaboração das questões de Inglês.
Passe suas horas de estudo “afiando” sua mente com nossas aulas de Inglês e seu
constante estudo dia após dia. Resolva provas anteriores. Leia muito. Tenha contato com a língua
sempre que possível.
Toda oportunidade que encontrar para ler e pesquisar as fontes usadas nessas provas, será
uma chance a mais de ser aprovado. Faça todo esforço necessário para alcançar sua meta!

AULA 00 – ENGLISH GENERAL PRESENTATION 6


TEACHER ANDREA BELO

PLANEJAMENTO DAS AULAS


Aulas Conteúdo
Aula 00: Introdução:
English General presentation. ➢ às técnicas de Skimming e Scanning;
➢ à importância dos tempos verbais em
Inglês.;
➢ ao uso dos falsos cognatos;
➢ aos termos gramaticais essenciais;
➢ ao uso de expressões idiomáticas e
➢ às formas de interpretar imagens.
Aula 01: Usando a técnica Skimming em textos para
Skimming and Scanning and other auxiliar na compreensão e leitura e outras
techniques. técnicas.

Identifying False Cognats Usando a técnica Scanning a fim de


compreender e ler diferentes textos e outras
Usando Skimming e Scanning em técnicas.
textos.
Indentificando falsos cognatos.
Aula 02: ➢ Simple Present. (Presente Simples).
➢ Simple Past. (Passado Simples).
Verbs in texts. ➢ Simple Future com will. (Futuro Simples
com will)
➢ Future going to. (Futuro com going to).
➢ Verb to be. (Verbo to be).
➢ Gerund. (Gerúndio).
➢ Present Continuous. (Presente
Contínuo).
➢ Past Continuous. (Passado Contínuo).
➢ Present Perfect. (Presente Perfeito).
➢ Past Perfect. (Passado Perfeito).
➢ Future Perfect (Futuro Perfeito).
➢ Present Perfect Continuous. (Presente
Perfeito Contínuo).
➢ Past Perfect Continuous. (Passado
Perfeito Contínuo).

AULA 00 – ENGLISH GENERAL PRESENTATION 7


TEACHER ANDREA BELO

➢ Future Perfect Continuous. (Futuro


Perfeito Contínuo).
➢ Modal Verbs. (Verbos Modais).
➢ Phrasal verbs prepositions. Preposições
nos phrasal verbs.
Aula 03: Definite articles. Artigos definidos.
Articles. Indefinite articles. Artigos indefinidos.
Nouns. Nouns. Substantivos.
➢ Common noun. Substantivo comum.
➢ Proper noun. Substantivo próprio.
➢ Compound noun. Substantivo composto.
➢ Abstract and concrete nouns
(Substantivos abstratos e concretos).
➢ Collective nouns. (Substantivos
coletivos).
➢ Singular and plural. (Singular e plural)
➢ Countable/Uncountable nouns –
Substantivos contáveis e incontáveis.
Aula 04: Adjectives position. Posição dos adjetivos.
➢ Comparative sentences. Grau
Adjectives. Comparativo.

Adverbs. ➢ Superlative sentences. Grau Superlativo.


Adverbs. Advérbio.
➢ Adverbs of manner. Advérbios de modo.
➢ Adverbs of frequency. Advérbios de
frequência.
➢ Adverbs of time. Advérbios de tempo.
➢ Adverbs of place. Advérbios de lugar.
➢ Adverbs of intensity. Advérbios de
intensidade.
Aula 05: Pronouns. Pronomes:
Pronouns ➢ Personal pronouns. Pronomes pessoais.
➢ Possessive pronouns. Pronomes
Prepositions. possessivos.
➢ Subject prounouns. Pronomes sujeito.

AULA 00 – ENGLISH GENERAL PRESENTATION 8


TEACHER ANDREA BELO

➢ Object pronouns Pronomes objeto.


➢ Adjective pronouns. Pronomes adjetivos.
➢ Reflexive pronouns. Pronomes
reflexivos.
➢ Demonstrative prounouns. Pronomes
demonstrativos.
➢ Indefinite prounouns. Pronomes
indefinidos.
➢ Interrogative pronouns. Pronomes
interrogatives.
Prepositions. Preposições – at, about, above,
across, after, against, along, among, around,
before, behind, below, beside, between,
beyond, by, close to, during, far, for, from, in,
inside, in front of, into, near, next to, on, off,
onto, opposite, out, outside, over, round,
through, throughout, to, towards, under, until,
up

Aula 06: Conjunctions. Conjunções Coordenadas.


Conjunctions and Clauses. Compound sentences. Orações coordenadas.

Direct Speech x Reported Speech Direct Speech. Discurso direto.


Reported Speech. Discurso indireto

Aula 07: Conectives. Conectivos.


Linking words/Conectives. WH questions:
➢ What;
Question Words. ➢ Which;
➢ Where;
➢ When;
➢ Why;
➢ Who;
➢ Whom;
➢ Whose e
➢ How

AULA 00 – ENGLISH GENERAL PRESENTATION 9


TEACHER ANDREA BELO

Aula 08: ➢ Conditionals. Condicionais.


If Clauses. ➢ Zero conditional;
➢ First conditional;
➢ Second conditional
➢ Third conditional.Common
Abbreviations. Abreviações mais
comuns.
➢ Auxiliary verbs abbreviation.
Abreviações em verbos auxiliares.
➢ Modal verbs abbreviation. Abreviações
Abbreviations. em verbos modais.
Contractions. ➢ Conflict abbreviation. Abreviações que
geram conflito:
o (‘s) : is x has;
o (‘s e ‘d) had x would.
➢ Informal contractions. Contrações da
linguagem informal.
Aula 09:
Quantifiers. Determinantes – some/any;
Quantifiers. no/none; lots of/lot of; much/many; few/little;
each/every; neither/either.

Relative clauses. Orações Relativas:


➢ Who
Relative Clauses.
➢ Which
➢ That
➢ Whose
➢ Whom
Aula 10:
Passive Voice. Active/Passive Voice. Voz ativa x voz passiva.
Tag Questions. Tag Questions.
Idioms. Expressões Idiomáticas.

AULA 00 – ENGLISH GENERAL PRESENTATION 10


TEACHER ANDREA BELO

INTERPRETAR TEXTOS – PRIMEIROS PASSOS PRÁTICOS


No momento da sua prova, quando você for resolver a prova de Inglês, você terá que fazer
uma leitura rápida de cada texto, para identificar a ideia central acerca daquele assunto ou
encontrar termos específicos que ajudem a compreender do que se trata.
Fazer isso, com propriedade, até chegar à resposta da questão, é aplicar as técnicas
Skimming e Scanning.
No decorrer das aulas do nosso curso, haverá uma aula exclusiva com detalhes sobre como
usar bem essas técnicas, com vários textos e com outras dicas valiosas para garantir sua
aprovação. Mas, vamos, agora, falar brevemente dessas técnicas nessa aula de apresentação.
Como a própria tradução do verbo “skim” – deslizar os olhos, folhear, desnatar – é
exatamente isso que você vai fazer – passar os olhos pelo texto sem interrupções, mesmo não
entendendo todas as palavras, apenas procurando do que se trata o texto. É simplesmente focar
nas informações necessárias para responder questões que abrangem o texto, como veremos
agora.
O verbo “scan”, escanear, é examinar detalhadamente, codificar a mensagem das frases,
selecionar o vocabulário necessário, encontrar detalhes relevantes à resposta. Veja algumas
características dessas técnicas:

SCANNING SKIMMING
• VOCABULARY SELECTIVITY • FAST READING
• KEEP AN EYE ON THE TEXT • CONCENTRATION/FOCUS
• COGNATS: HELPERS • GENERAL TEXT IDEA
• SPECIFIC DETAILS • TEXT GOALS
• VISUAL CONTACT • SUBJECT INFORMATION

E, se você prestar atenção no contexto e quebrar o hábito de querer traduzir palavra por
palavra, essas técnicas levarão você à resposta com agilidade e sem tradução. Na verdade,
traduzir um texto, no momento da prova, ocupa seu tempo e atrasa a resolução dos exercícios.
Mesmo se você tem Inglês fluente, o ato de traduzir os textos leva tempo enquanto usar
as técnicas aqui ensinadas, poupam seu tempo para resolver todas as questões da prova e
também aprender palavras novas e saber Inglês com a metodologia que uso. Você vai ver.
Vamos aos detalhes de cada umas das técnicas citadas acima.

AULA 00 – ENGLISH GENERAL PRESENTATION 11


TEACHER ANDREA BELO

SKIMMING
Uma boa compreensão do texto que você está lendo, depende da sua capacidade de fazer
deduções, ligar ideias e identificar palavras que determinam o assunto.
E, o que realmente importa a você, é realmente conseguir encontrar as respostas da sua
prova e certificar-se dessas respostas, garantindo boa pontuação na prova de Inglês.
O Skimming é a leitura dinâmica para destacar os aspectos principais do texto, sem se
preocupar com os detalhes. Vejamos um exemplo para você experimentar a técnica Skimming,
primeiramente em português e, em seguida, em Inglês para testar sua capacidade:

Três onças pintadas foram vistas na kapinete ontem. Estavam se


escondendo de possíveis promubinos com suas armas. Os
promubinos não desistiram de trevenar e passaram a noite em claro
com lanternas e espingardas de prontidão. Se tivessem abstoque, não
matavam animais em extinção e sim, protegeriam nossa fauna e flora.

• Onde as onças foram vistas?


• Quem são os possíveis promubinos?
• O que fizeram ao invés de desistir?

Lendo apenas uma vez, você entendeu esse texto? A história fez sentido, mesmo com
palavras desconhecidas ao fazer uma leitura rápida? Você provavelmente atribuiu sentidos às
palavras novas (Kapinete: floresta; promubinos: caçadores; trevenar: procurar; abstoque:
consciência).
Esse é o SKIMMING, é a “chave da questão” em língua inglesa – atribuir significado aos
vocábulos que você não sabe. É conectando ideias e deduzindo o assunto, que se chega ao sentido
geral e coloca você no caminho da resposta.

AULA 00 – ENGLISH GENERAL PRESENTATION 12


TEACHER ANDREA BELO

SCANNING
No momento de resolver a prova de Inglês, você também terá que fazer uma leitura para
procurar uma palavra-chave. Um termo, um tempo verbal, algo sobre o título, sobre a fonte de
referência etc.
Daí você vai praticar a outra técnica, chamada Scanning, que também exploraremos com
detalhes na aula 1, desse curso e agora, veremos algumas considerações para uma introdução à
técnica. Scanning é ter como objetivo achar algo característico, singular, exclusivo para responder
uma determinada questão. Vejamos o uso de Scanning:

Três onças pintadas foram vistas na kapinete ontem. Estavam se


escondendo de possíveis promubinos com suas armas. Os
promubinos não desistiram de trevenar e passaram a noite em claro
com lanternas e espingardas de prontidão. Se tivessem abstoque, não
matavam animais em extinção e sim, protegeriam nossa fauna e flora.

• Onde as onças foram vistas?


• Quem são os possíveis promubinos?
• O que fizeram ao invés de desistir?

Suponhamos que a pergunta fosse “O que as onças estavam fazendo?”


Você teria que voltar ao texto, ler mais uma vez para conferir e se certificar, mesmo que
se lembrasse do contexto.
Ao praticar o Scanning, você leu a informação contida no local em que está exatamente o
que você precisa. Vamos à nossa questão. A frase “Três onças pintadas foram vistas na kapinete
ontem. Estavam se escondendo de possíveis promubinos com suas armas” e já iríamos encontrar
a resposta desejada. Não é mesmo? A técnica leva você às respostas.
Em textos, em imagens ou qualquer forma de leitura, sempre há “vestígios” que nos levam
a perceber sobre o que estamos lendo. São indicativos do assunto com palavras particulares.
Agora, vamos falar um pouco dos falsos cognatos, para não cometer erros na hora da
resolução de exercícios na sua prova.

AULA 00 – ENGLISH GENERAL PRESENTATION 13


TEACHER ANDREA BELO

FALSO COGNATO – SIMPLES DE FATO


Vamos falar agora sobre Falso Cognato. Conforme o título, simples de fato. Você acredita?
Vamos entender o que é um Cognato em Inglês.
Palavras cognatas são aquelas que se assemelham a palavras em Português. E, essas
semelhanças ortográficas, ajudam você a fazer suas leituras.
Veja alguns exemplos para animar você, já que muitas vezes, os cognatos te ajudarão a
resolver questões:

CAMERA TELEPHONE SALAD BLOUSE


CÂMERA TELEFONE SALADA BLUSA

False Cognates ou False Friends, aparecem muito nas provas e são palavras que se diferem
completamente no significado, apesar de serem similares na ortografia. Eu diria que são tricky
words – palavras “enganosas”, “pegadinhas”, pois você acha que é algo quando o significado é
muito diferente do que parece ser.
Em nosso curso, sempre há questões em que aparecem, além de falsos cognatos, palavras
repetidas com objetivo específico, marcas tipográficas, dentre outras particularidades que exigem
atenção na hora da leitura, como veremos na aula 01 com mais detalhes.
É essencial entender por que os falsos cognatos são, um dos sinais mais importantes para
resolver sua prova. Vejamos outros exemplos:

ACTUALLY = de fato/na verdade (não é atualmente, que seria NOWADAYS)

FABRIC = tecido (não é fábrica, que seria FACTORY)

PREJUDICE = preconceito (não é prejudicial, que seria HARMFUL)

COLLEGE = faculdade (não é escola, que seria SCHOOL)

PRETEND = fingir (não é pretender, que seria TO INTEND)

TEMPER = temperamento (não é tempero, que seria SEASONING/FLAVORING/SPICE)

Agora vamos ver um pouco da introdução à gramática de uma forma mais prática.

AULA 00 – ENGLISH GENERAL PRESENTATION 14


TEACHER ANDREA BELO

GRAMÁTICA DE FORMA PRÁTICA


A gramática está presente de várias formas nas questões de Inglês na prova. Na maioria
das vezes, ela vem contextualizada.
Outras vezes, pergunta-se exatamente o termo gramatical, testando seus conhecimentos.
Ou então, são oferecidas opções de escolha de tópicos da gramática que podem ser substituídos
por outros, entre inúmeros exercícios.
Saber a gramática, além de ler e interpretar o texto, é um dos critérios decisivos para que
você tenha êxito. Pensando assim, elaborei explicações objetivas, com o intuito de ajudar você a
resolver a prova de Inglês. E, ao se deparar com tópicos gramaticais mais complexos, seus estudos
exigirão cuidado, atenção e esforço em grandes doses, certo?
No planejamento do nosso curso e na montagem do cronograma, tive a preocupação de
inserir conteúdos que você precisa para estar seguro quanto à gramática.
É importante, primeiramente, saber o que há para estudar da matéria de Inglês a partir do
edital e separei, todos os tópicos presumíveis para a prova.
Vou dar um exemplo básico da gramática com a prática. Quer ver? Se você vai elogiar
alguém, usando o adjetivo brilhante para dizer que você considera esse alguém com essa
característica, a frase seria, em Português: “Que pessoa brilhante!”.
Curiosamente, em Inglês, não é assim. No momento do elogio, o adjetivo, que é a
qualidade usada para, nesse caso, elogiar, vem antes do substantivo e, desde antes, você já sabe
se será um elogio ou crítica por exemplo. Como? Veja: “What a brilliant person!”. Viu? A frase
começa com “What a brilliant…” já manifestando o elogio antes mesmo de falar quem. Se forem
várias pessoas em uma mesma sala, por exemplo, já se sabe que alguém ali é brilhante.
Desta forma, em sua prova, não precisa de pensar que as palavras em Inglês são
“bagunçadas, não tem ordem específica, é difícil...”. Nada disso. Tudo tem uma explicação e, a
cada aula, vou esclarecer e demonstrar com exemplos e com exercícios, que a gramática pode ser
prática, sim!
EM PORTUGUÊS: Que pessoa elegante! > adjetivo após o substantivo “pessoa”
EM INGLÊS: What an elegant person! > adjetivo antes do substantivo “person”
A gramática em Inglês, na hora dos estudos, é considerada algo que dificulta pela
quantidade de regras. Porém, vou simplificar e tornar sua compreensão possível e eficaz. Vamos
focar nos verbos, advérbios, adjetivos e termos gramaticais em geral sempre de maneira
contextualizada, assim como expliquei o uso do adjetivo acima a você.
Em seguida, praticaremos ao máximo a leitura de textos, permitindo você a treinar o que
está aprendendo. Pouco a pouco, vamos avançar para temas mais complexos da gramática para
aprender a analisar a semântica, a sintaxe e a morfologia, também contextualizadas.
Estudaremos classificação, estrutura e a formação de palavras em Inglês, tipos de orações,
funções dos termos dentro dos textos, levando você a interpretar e responder o que se pede. E,
com exercícios de fixação, logo você estará confiante e otimista em relação à prova de Inglês.

AULA 00 – ENGLISH GENERAL PRESENTATION 15


TEACHER ANDREA BELO

O TEMPO VERBAL DE FORMA NATURAL


O tempo verbal de forma natural? Como? Que audácia! Sim, mas você verá que é possível
estudar os verbos em Inglês com os esclarecimentos aqui oferecidos já que o objetivo aqui é que
você possa identificar os verbos com o propósito de acertar as questões interpretando os textos.
Concordo que é necessário paciência para estudar tempos verbais, mas dominá-los é
essencial para se destacar nos estudos e chegar à aprovação. Tenha em mente que o
conhecimento dos verbos entre outros conteúdos aqui explorados, irá trazer a você enormes
benefícios.
Para expressar uma ação no presente, em Português, cada sujeito usado (eu, ela, os
homens etc.) há uma terminação diferente. Usando o verbo trabalhar, que faz parte dos verbos
da primeira conjugação – terminados em -ar, como olhar, falar etc. – e a raiz do verbo trabalhar,
a parte que não muda, é trabalh, certo?
Então, Eu trabalho, termina com a letra o. E, ela trabalha, termina com a letra a. Os homens
trabalham, termina em -am. Portanto, são várias terminações para expressar a ação (verbo)
apenas no tempo presente em Português.
Em Inglês, não é complicado assim.
O verbo fica igual para todo e qualquer sujeito, adicionando apenas a letra -s, -es ou -ies quando
o sujeito é singular, ou seja, quando uma única pessoa pratica a ação. Vejamos com a ajuda de
um esquema:
EM PORTUGUÊS:
EM PORTUGUÊS: EM INGLÊS:
EM INGLÊS: I work, You work, He/She/

Eu trabalho I work

Tu trabalhas You work

Ele/Ela trabalha He/She/It works

Nós trabalhamos We work

Vós trabalhais You work

Eles trabalham They work

Eu gostaria de saber, primeiramente, se você percebeu que o verbo trabalhar (to work)
conjugado no tempo presente é bem mais fácil do que em Português? Não é? Para cada 6
diferentes terminações no fim dos verbos em Português, há apenas 2 variações em inglês – o
verbo “to work” escrito normalmente para os todos os sujeitos exceto singular representado por
“He/She/It”, que acrescentamos “s” no verbo – “He works, She works, It works”, como no
esquema acima.

AULA 00 – ENGLISH GENERAL PRESENTATION 16


TEACHER ANDREA BELO

Por exemplo, o verbo trabalhar no passado, é “worked” qualquer sujeito. Veja abaixo:

EM PORTUGUÊS: EM INGLÊS:
Eu trabalhei I worked
Tu trabalhou You worked
Ele/Ela trabalhou He/She/It worked
Nós trabalhamos We worked
Vós trabalhais You worked
Eles trabalharam They worked

- Mas teacher, já ouvi dizer que há inúmeros verbos irregulares. Com fica?
- Bom, é isso mesmo. Existem verbos irregulares na língua inglesa. Mas, não se assuste.
Eles são minoria, algo em torno de 15% a 20%. Ou seja, dominando os verbos regulares, você já
terá a capacidade de se expressar de forma escrita ou falada com a maioria dos verbos da língua
inglesa.
Por exemplo, o verbo escrever, “write”, que, ao invés de adicionar -ed no final, como a
maioria dos verbos em Inglês, troca-se uma das letras, por ser irregular (teremos uma aula
dedicada exclusivamente aos tempos verbais, com detalhes de como lidar com as regras),
escreve-se “wrote” para qualquer sujeito. Veja outro esquema para ficar ainda mais claro:

EM PORTUGUÊS: EM INGLÊS:
Eu escrevi I wrote
Tu escreveu You wrote
Ele/Ela escreveu He/She/It wrote
Nós escrevemos We wrote
Vós escreveis You wrote
Eles escreveram They wrote

Viu como é simples? E, com naturalidade, você responderá às questões da prova com
segurança, elaborar os parágrafos solicitados e responder o que for solicitado.
Bom, no próximo capítulo continuarei com as dicas sobre como que podemos transformar
a disciplina Inglês em algo simples, falando de expressões idiomáticas, vamos lá?

AULA 00 – ENGLISH GENERAL PRESENTATION 17


TEACHER ANDREA BELO

INTERPRETAR IMAGEM COM APRENDIZAGEM


Agora vamos começar a falar de formas viáveis de interpretar imagens. Claro que teremos
uma aula inteira e completa com os mais variados tipos de figuras encontradas nas provas -
charges, quadrinhos, pinturas, gráficos, fotografias, tirinhas, anúncios de produto, propagandas
diversas e outros, mas, vamos fazer uma introdução ao assunto.
Imagens nunca estão na prova simplesmente para ilustrar, mas, para trazer informações
significativas. Sendo então, indispensável que você também tenha um conhecimento prévio sobre
temas relevantes e conhecimentos gerais.
Você tem que fazer perguntas para construir uma leitura crítica e inteligente.
No momento de resolver uma questão com imagens, você precisa, antes de tudo, saber o
tipo de ilustração da questão e observar os detalhes da imagem e o texto vinculado a ela, como
vimos no exemplo anterior e veremos em inúmeros outros em nossas aulas.
As imagens permitem e trazem consigo atributos, traços únicos que tornam o texto
vinculado a ela, abrangente e repleto de mensagens subliminares.
Você deve aproximar conteúdos e encaixá-los em seus conhecimentos. Vou sempre fazer
referência a assuntos diversos por meio de exercícios, para você aprimorar suas leituras de
imagens de qualquer categoria.
Agora, vamos à introdução de como interpretar expressões idiomáticas.

EXPRESSÕES IDIOMÁTICAS SIMPÁTICAS


As expressões idiomáticas são simpáticas? Podem ser simpáticas na hora da sua prova?
- Sim. Vou provar a você que sim. Em Inglês, as expressões idiomáticas são chamadas de
“Idiom”. É um grupo de palavras com um significado que não há como deduzir a partir das palavras
individuais, restritas, literais. É expressar-se de modo peculiar a alguém.
As expressões idiomáticas – idioms – aparecem com naturalidade e enriquecem a
comunicação textual. São destituídas de tradução e consideradas variações da língua, pois
revelam traços culturais de um povo, de um grupo.
Nos idioms, o significado não corresponde ao que as palavras individuais sugerem pois
trazem consigo metáforas. O mistério para entender expressões idiomáticas em Inglês é não
traduzir as palavras e sim, se familiarizar com elas na medida que se estuda e pratica exercícios.
Por exemplo, se você quer dizer: “um passarinho verde me contou que...”, a expressão
idiomática correta é “I heard it through the grapevine that...”, que significaria, palavra por
palavra, “eu ouvi isso através de um boato”, pois “grapevine”, apesar de ser videira em português,
também possui como possível tradução o termo “boato”.
Veja mais alguns exemplos de idioms abaixo enquanto a aula com outros idioms está por vir.

AULA 00 – ENGLISH GENERAL PRESENTATION 18


TEACHER ANDREA BELO

AULA 00 – ENGLISH GENERAL PRESENTATION 19


TEACHER ANDREA BELO

As expressões idiomáticas acima, estão em meu Instagram com exemplos e as devidas


explicações de cada uma das expressões.
No final dessa aula, vou oferecer o acesso às minhas redes sociais, como complemento de
seus estudos. Além disso, há também dicas de vários outros tópicos gramaticais, necessários a
você.
Agora, vamos aos exercícios para praticar tudo que foi estudado. Preparado?
Serão questões inéditas, exercícios exclusivos das provas anteriores de diferentes instituições
para você estar apto, bem treinado e “afiado” no dia da sua prova. Let’s go!

AULA 00 – ENGLISH GENERAL PRESENTATION 20


TEACHER ANDREA BELO

QUESTÕES
Esse é momento em que vamos praticar tudo o que vimos nessa Aula 00.
Serão questões de várias instituições, para preparar você e colaborar com a sua aprovação.
Começando com questões AFA.
QUESTÕES AFA
Directions: Read the text below and answer questions 01 to 10 according to it.
TEXT Howard Gardner: ‘Multiple intelligences’ are not ‘learning styles’ by Valerie Strauss
The fields of psychology and education were revolutionized 30 years ago when we now
worldrenowned psychologist Howard Gardner published his 1983 book Frames of Mind: The
Theory of Multiple Intelligences, which detailed a new model of human intelligence that went
beyond the traditional view that there was a single kind that could be measured by standardized
tests.
Gardner’s theory initially listed seven intelligences which work together: linguistic, logical-
mathematical, musical, bodily-kinesthetic, interpersonal and intrapersonal; he later added an
eighth, naturalist intelligence and says there may be a few more. The theory became highly
popular with K-12¹ educators around the world seeking ways to reach students who did not
respond to traditional approaches, but over time, ‘multiple intelligences’ somehow became
synonymous with the concept of ‘learning styles’. In this important post, Gardner explains why the
former is not the latter.
It’s been 30 years since I developed the notion of ‘multiple intelligences’. I have been gratified by
the interest shown in this idea and the ways it’s been used in schools, museums, and business
around the world. But one unanticipated consequence has driven me to distraction and that’s the
tendency of many people, including persons whom I cherish, to credit me with the notion of
‘learning styles’ or to collapse ‘multiple intelligences’ with ‘learning styles’. It’s high time to relieve
my pain and to set the record straight.
First a word about ‘MI theory’. On the basis of research in several disciplines, including the study
of how human capacities are represented in the brain, I developed the idea that each of us has a
number of relatively independent mental faculties, which can be termed our ‘multiple
intelligences’. The basic idea is simplicity itself. A belief in a single intelligence assumes that we
have one central, all-purpose computer, and it determines how well we perform in every sector
of life. In contrast, a belief in multiple intelligences assumes that human beings have 7 to 10
distinct intelligences.
Even before I spoke and wrote about ‘MI’, the term ‘learning styles’ was being bandied about in
educational circles. The idea, reasonable enough on the surface, is that all children (indeed all of
us) have distinctive minds and personalities. Accordingly, it makes sense to find out about learners
and to teach and nurture them in ways that are appropriate, that they value, and above all, are
effective.
Two problems: first, the notion of ‘learning styles’ is itself not coherent. Those who use this term
do not define the criteria for a style, nor where styles come from, how they are recognized/

AULA 00 – ENGLISH GENERAL PRESENTATION 21


TEACHER ANDREA BELO

assessed/ exploited. Say that Johnny is said to have a learning style that is ‘impulsive’. Does that
mean that Johnny is ‘impulsive’ about everything? How do we know this? What does this imply
about teaching? Should we teach ‘impulsively’, or should we compensate by ‘teaching
reflectively’? What of learning style is ‘right-brained’ or visual or tactile? Same issues apply.
Problem #2: when researchers have tried to identify learning styles, teach consistently with those
styles, and examine outcomes, there is not persuasive evidence that the learning style analysis
produces more effective outcomes than a ‘one size fits all approach’. Of course, the learning style
analysis might have been inadequate. Or even if it is on the mark, the fact that one intervention
did not work does not mean that the concept of learning styles is fatally imperfect; another
intervention might have proved effective. Absence of evidence does not prove non-existence of a
phenomenon; it signals to educational researchers: ‘back to the drawing boards’.
Here’s my considered judgment about the best way to analyze this lexical terrain: Intelligence: We
all have the multiple intelligences. But we signed out, as a strong intelligence, an area where the
person has considerable computational power. Style or learning style: A hypothesis of how an
individual approaches the range of materials. If an individual has a ‘reflective style’, he/she is
hypothesized to be reflective about the full range of materials. We cannot assume that
reflectiveness in writing necessarily signals reflectiveness in one’s interaction with the others.
Senses: Sometimes people speak about a ‘visual’ learner or an ‘auditory’ learner. The implication
is that some people learn through their eyes, others through their ears. This notion is incoherent.
Both spatial information and reading occur with the eyes, but they make use of entirely different
cognitive faculties. What matters is the power of the mental computer, the intelligence that acts
upon that sensory information once picked up.
These distinctions are consequential. If people want to talk about ‘an impulsive style’ or a ‘visual
learner’, that’s their prerogative. But they should recognize that these labels may be unhelpful, at
best, and ill-conceived at worst.
In contrast, there is strong evidence that human beings have a range of intelligences and that
strength (or weakness) in one intelligence does not predict strength (or weakness) in any other
intelligences. All of us exhibit jagged profiles of intelligences. There are common sense ways of
assessing our own intelligences, and even if it seems appropriate, we can take a more formal test
battery. And then, as teachers, parents, or selfassessors, we can decide how best to make use of
this information.
(Adapted from https://www.washingtonpost.com/news/answer-sheet)

Questão 01 (AFA/2016) – The text


a) aims at highlighting distinctive mind barriers related to learning.
b) provides the reader with a bird’s-eye-view of Gardner’s landmark publication.
c) develops a considerable set of psychological and mental implications.
d) concerns about spending 30 years to measure people’s intelligence.

AULA 00 – ENGLISH GENERAL PRESENTATION 22


TEACHER ANDREA BELO

Questão 02 (AFA/2016) – In the sentence “there was a single kind that could be measured by
standardized tests”, it is possible to find an option to substitute the pronoun accordingly in
a) when.
b) which
c) how.
d) whom.

Questão 03 (AFA/2016) – In the fragment “why the former is not the latter”, the highlighted
words refer to
a) multiple intelligences / learning style.
b) over time / theory.
c) ways to reach students / traditional approaches.
d) traditional approaches / K-12 educators.

Questão 04 (AFA/2016) – In the third paragraph (lines 20 and 29), the author
a) delves into the mind of human beings.
b) reinforces the importance of a thirty-year study.
c) tries to make controversial issues clear.
d) apologizes for the fact that his theory has collapsed.

Questão 05 (AFA/2016) – In the sentence “it’s been 30 years since I developed the notion of
‘multiple intelligences’”, the contraction refers to
a) It has.
b) It been.
c) It is.
d) It was.

Questão 06 (AFA/2016) – Mark the option which shows the appropriate question tag for the
sentence “one unanticipated consequence has driven me to distraction”.
a) Hasn’t driven it?
b) Not has it?
c) Has it?
d) Hasn’t it?

AULA 00 – ENGLISH GENERAL PRESENTATION 23


TEACHER ANDREA BELO

Questão 07 (AFA/2016) – Mark the option that shows synonyms for the underlined expressions
in “it’s high time to relieve my pain and to set the record straight”.
a) An important brake / to register.
b) An ordinary condition / to show the discussion.
c) A belated explanation / to make myself clearly understood.
d) An unusual hour / to comprehend an argument.

Questão 08 (AFA/2016) – Choose the best option to change the sentence “human capacities are
represented in the brain” (4th paragraph), into the active form.
The brain _______________ human capacities.
a) has represented
b) represents
c) has been represented
d) representing

Questão 09 (AFA/2016) – In the fourth paragraph, it’s said that


a) a single intelligence (SI) concept leads to the assumption of computers which control 7 to 10
distinct intelligences.
b) MI theory believes that instead of a central computer mastering various sectors, there are a
larger amount of them relatively autonomous.
c) MI theory estimates the existence of a central computer responsible for 7 to 10 distinct
intelligences.
d) a SI determines people’s performance in different sectors of life through autonomous
computers.

Questão 10 (AFA/2016) – Mark the alternative in which the problems described in paragraphs 6
and 7 are correctly summarized.
a) The idea of teaching distinct leaning styles and their consistence were questionable concepts
when researches started.
b) Educational researchers have found that an impulsive learning style causes problems in its
outcomes.
c) There are proofs that different learning styles exist and produce positive results.
d) The notion of learning styles and the outcomes observed when teaching based on them need
further studies.

AULA 00 – ENGLISH GENERAL PRESENTATION 24


TEACHER ANDREA BELO

QUESTÕES COLÉGIO NAVAL


Read text to do questions 01 to 04 based on it.
Text I
Social media ’destroying how society works;
A former Facebook executive has said social media is doing great harm to society around the
world. The executive is a man called Chamath Palihapitiya. He ___________ Facebook in 2007
and ___________a vice president. He was responsible for increasing the number of users
Facebook had. Mr Palihapitiya said he feels very guilty about getting more people to use social
networks. He said the networks are destroying society because they are changing people's
behavior. Twenty years ago, people talked to each other face to face. Today, people message each
other and do not talk. People also really care about what other people think of them. They post
photos and wait to see how many people like the photo. They get very sad if people do not like
the photo.
Mr. Palihapitiya said people should take a long break from social media so they can experience
real life. He wants people to value each other instead of valuing online "hearts, likes, and thumbs-
up". Palihapitiya also points out how fake news is affecting how we see the world, it is becoming
easier for large websites to spread lies. It is also becoming easier to hurt other people online.
Anyone can hide behind a fake user name and post lies about other people. Palihapitiya said this
was a global problem. He is worried about social media so much that he has banned his children
from using it. However, he did state that Facebook was a good company. He said: "Of course,
it's not all bad. Facebook overwhelmingly does good in the world."

Questão 01 (Colégio Naval/2018) – Read the statements to check if they are TRUE (T) or FALSE
(F).
I- An ex-Facebook boss said social media is damaging society.
II- It is becoming more difficult for big websites to spread fake news.
III- People message each other today instead of talking face to face.
IV- Palihapitiya said social media does not change our behavior.
Choose the option that respectively represents the statements above.
a) F/T/T/F
b) F/F/T/T
c) T/F/T/F
d) T/T/F/T
e) F/F/F/T

AULA 00 – ENGLISH GENERAL PRESENTATION 25


TEACHER ANDREA BELO

Questão 02 (Colégio Naval/2018) – All the underlined words in text I are adjectives, EXCEPT:
(A) social.
(B) global.
(C) long.
(D) executive.
(E) former.

Questão 03 (Colégio Naval/2018) – Which verb forms respectively complete the gaps in text I?
(A) joined/become
(B) joined/became
(C) joins/becomes
(D) joint/became
(E) was joined/ become

Questão 04 (Colégio Naval/2018) – Mark the option in which there is NO Present Continuous
Tense.
(A) A former Facebook executive has said social media is doing great harm to society around the
world.
(B) He was responsible for increasing the number of users Facebookliad.
(C) He said the networks are destroying society because they are changing people's
behavior.
(D) Palihapitiya also points out how fake news is affecting how we see the world.
(E) It is becoming easier for large websites to spread lies.

Read text II to do questions 05 to 10 based on it.


TEXT II
TRAVEL TIPS
How to Plan a Movie-Themed Vacation
It’s easier than you may expect to find, visit, and enjoy the places where your favorite movies
were made.
Lars Leetaru
By Shivani Vora
March 8, 2018
Whether it’s the “Lord of the Rings” trilogy in New Zealand or "Roman Holiday” in Rome, many
noteworthy movies are filmed in appealing locales all over the world that travelers may want to
visit and enjoy.
According to Angela Tillson, a film location manager in Kauai who has worked on the set of films
including "Jurassic Park: The Lost World” and “The Descendants," exploring a beloved movie set

AULA 00 – ENGLISH GENERAL PRESENTATION 26


TEACHER ANDREA BELO

destination through the eyes of the film makes for an enjoyable vacation. "Seeing a place with a
focus on a movie you love will give you a perspective that the average tourist doesn’t usually get.
You’ll certainly have a better impression of the place,” she said. Here are her tips to get started.
Choose Your Destination
If there’s a movie you love, you can find out where it was filmed by looking at the credits at the
end of the film or by going online to The Internet Movie Database, also known as IMDB, which
often lists filming locations. Once you know the locale, you can start planning your trip. Or,
consider doing what Ms. Tillson often does when deciding on where to vacation: pick a spot you’re
interested in visiting, and find out what movies have been filmed there. “It’s fun to sometimes let
a destination determine the movie you're going to live rather than the other way around,”
Ms. Tillson said.
Get in the Mood
Before you head to your destination, be sure to rewatch the movie. A rewatch not only reminds
you of identifiable spots to look out for during your trip, but it also adds to the excitement of your
upcoming exploration.
If the movie is based on a book, consider reading the book, too. It may have details about the
locale that the movie doesn’t touch on. Also, books often have scenes that don’t make it into the
movie adaptations, which gives you a deeper view of the destination.
Ms. Tillson also recommended downloading the movie’s soundtrack or score, and listening to it
throughout your trip.
Book a Themed Trip
Some travel companies sell set itineraries focused on popular movies. Luxury tour operator
Zicasso, for example, has an eight-day trip, all inclusive, to Ireland inspired by "Star Wars: The Last
Jedi” and Wild Frontiers has an eleven-day trip to India inspired by "The Best Exotic Marigold
Hotel." Ms. Tillson suggested doing a web search or checking with a travel agent to find out about
such trips.
Also, in some destinations, local tour operators and hotels sell movie-themed tours. For instance,
The St. Regis Priceville Resort offers a tour that includes a private helicopter ride to
Manawaiopuna Falls, made famous in "Jurassic Park,” and an ATV tour of filming locations of
movies such as “Raiders of the Lost Ark" and “Pirates of the Caribbean.” Lunch is even included.
The cost is $5,674 for two adults.
A more affordable option, in Rome, is the four-hour “Roman Holiday" themed excursion from HR
Tours, where travelers ride a Vespa with a driver and see all the sites from the movie; the cost is
170 euros per person.
Hang Where the Movie Crew Did
When they’re not working, movie crews enjoy hitting local bars and casual restaurants that serve
tasty local cuisine, Ms. Tillson said.
Find out where the behind-the-scenes staff of your film spent their time by asking your
destination’s tourist board or your hotel’s concierge, and check out a few of the spots. “It’s
another way to get involved in the film and spend time in bars and restaurants that you wouldn’t
normally think to hit,” she said.

AULA 00 – ENGLISH GENERAL PRESENTATION 27


TEACHER ANDREA BELO

Questão 05 (Colégio Naval/2018) – Mark the sentence that does NOT contain the use of
comparative adjective.
(A) It's easier than you may expect to find, visit, and enjoy the places where your favorite
movies were made.
(B) You’ll certainly have a better impression of the place.
(C) It’s fun to sometimes let a destination determine the movie you’re going to live rather than
the other way around.
(D) Also, books often have scenes that don’t make it into the movie adaptations, which gives you
a deeper view of the destination.
(E) A more affordable option, in Rome, is the four-hour Roman Holiday themed excursion from
HR Tours.

Questão 06 (Colégio Naval/2018) – What is true about the themed trip inspired by the film
“Star Wars: The Last Jedi"?
(A) It offers no meals.
(B) It lasts 11 days.
(C) It’s very cheap.
(D) It’s located in India.
(E) It’s offered by Zicasso.

Questão 07 (Colégio Naval/2018) – In the first paragraph, the word "appealing” can be replaced
by all these words, EXCEPT for
(A) interesting.
(B) pleasing.
(C) lovable.
(D) repulsive.
(E) attractive.

Questão 08 (Colégio Naval/2018) – Mark the correct question for the following answer.
Angela Tillson is a film location manager in Kauai who has worked on the set of films including
“Jurassic Park: The Lost World” and “The Descendants”.
(A) Who is Ms. Tilson?
(B) Where is Ms. Tilson?
(C) What is Ms. Tilson like?
(D) Why is Ms. Tilson working?
(E) When does Ms. Tilson work?

AULA 00 – ENGLISH GENERAL PRESENTATION 28


TEACHER ANDREA BELO

Questão 09 (Colégio Naval/2018) – What’s the main purpose of text II?


(A) Teach students how to plan where to go on vacation.
(B) inspire the fans of “Lord of the Rings” to visit New Zealand.
(C) Make people feel interested in watching famous movies.
(D) Give some suggestions on traveling to a movie set destination.
(E) Advertise travel companies that sell movie-themed vacations.

Questão 10 (Colégio Naval/2018) – Read the sentence in text II.


"A rewatch not only reminds you of identifiable spots to look out for during your trip, but it also
adds to the excitement of your upcoming exploration."
What does the pronoun it refer to?
(A) Trip.
(B) Spots.
(C) Excitement.
(D) Exploration.
(E) Rewatch.

QUESTÕES EEAR
Text to answer questions 01 and 02
Paul Thompson is a student and he has a part-time job, too. He lives in London and goes to
university there. Paul works at an Italian restaurant called Mamma Mia. He’s a waiter there. He
likes his job very much but he doesn’t like to work on the weekends. When he is free, he likes to
go to the movies. He recently watched “Avengers – Infinity War”, a superhero movie that was a
huge success.

Questão 01 (EEAR/2021 – CFS) – All the words below are synonyms for the word “huge” (line
08), in bold in the text, EXCEPT:
A) insignificant
B) tremendous
C) massive
D) great

Questão 02 (EEAR/2021 – CFS) – According to the text, how long does Paul work?
(A) Part-time.
(B) All day long.
(C) He doesn’t work.
(D) Every other week.

AULA 00 – ENGLISH GENERAL PRESENTATION 29


TEACHER ANDREA BELO

Text to answer question 03


“The Little Prince”, now, is a movie – Alex Weiss
An all-time favorite children’s book, The Little Prince by Antoine de Saint-Exupéry has been turned
into a beautifully animated movie – and it’s finally being released. The inspiring lessons, timeless
story and beautiful quotes from The Little Prince make this a perfect choice for an on-screen
adaptation. At one point to another, your parents read this book to you and when you were a
child and then you picked it up later on in life, realizing how incredibly important this small book
truly is.
(Adapted from bustle.com)

Questão 03 (EEAR/2021 – CFS) – The verbs in bold are, respectively, in the:


(A) present perfect, simple past and simple past.
(B) simple past, simple past and present perfect.
(C) present perfect, simple past and present perfect.
(D) simple past, present perfect and present perfect.

Text to answer questions 04 and 05


Price Tag
Price Tag – Jessie J.
Seems like everybody's got a price
I wonder how they sleep at night
When the sale comes first
And the truth comes second
Just stop for a minute and smile
Why is everybody so serious
Acting so damn mysterious
Got shades on your eyes
And your heels so high
That you can't even have a good time
Everybody look to the left
Everybody look to the right
Can you feel that yeah
We're paying with love tonight
It's not about the money money money
We don't need your money money money
We just wanna make the world dance
Forget about the price tag
Ain't about the uh cha-ching cha-ching

AULA 00 – ENGLISH GENERAL PRESENTATION 30


TEACHER ANDREA BELO

Ain't about the yeah b-bling b-bling


Wanna make the world dance
Forget about the price tag
(letras.mus.br)

Questão 04 (EEAR/2021 – CFS) – The verb WONDER, in bold in the text, can be replaced by
_______ without changing the meaning:
(A) know.
(B) can know.
(C) must know.
(D) want to know.

Questão 05 (EEAR/2021 – CFS) – The underlined words, in the text, are:


(A) adjectives.
(B) adverbs.
(C) nouns.
(D) verbs.

Questão 06 (EEAR/2021 – CFS) – Change the sentence into the interrogative form.
“The cost of living has doubled in relative terms over recent years”
(A) Has the cost of living double in relative terms over recent years?
(B) Has the cost of living doubled in relative terms over recent years?
(C) Have the cost of living double in relative terms over recent years?
(D) Have the cost of living doubled in relative terms over recent years?

Read the text and answer questions 07 and 08.


In The End – Linkin Park
One thing, I don't know why
It doesn't even matter how hard you try
Keep that in mind
I designed this rhyme
To remind myself of a time when
I tried so hard
In spite of the way you were mocking me
Acting like I was part of your property

AULA 00 – ENGLISH GENERAL PRESENTATION 31


TEACHER ANDREA BELO

Remembering all the times you fought with me


I'm surprised it got so
Things aren't the way they were before
You wouldn't even recognize me anymore
Not that you knew me back then
But it all comes back to me in the end
You kept everything inside
And even though I tried, it all fell apart
What it meant to me will eventually be a memory of a time when
I tried so hard
And got so far
But in the end
It doesn't even matter
I had to fall
To lose it all
But in the end
It doesn't even matter
I've put my trust in you
Pushed as far as I can go
For all this
There's only one thing you should know
I've put my trust in you
Pushed as far as I can go
For all this
There's only one thing you should know
https://www.vagalume.com.br/linkin-park/in-the-end.html

Questão 07 (EEAR/2021 – CFS) – The words in bold are, respectively, synonyms for
A) Despite, although, and in the end.
B) Although, in the end, and finally.
C) Despite, though, and although.
D) Although, finally, and despite.

Questão 08 (EEAR/2021 – CFS) – The negative form of the underline sentence, in the text, is
A) I didn’t put my trust in you.
B) I don’t put my trust in you.
C) I do not put my trust in you.
D) I haven’t put my trust in you.

AULA 00 – ENGLISH GENERAL PRESENTATION 32


TEACHER ANDREA BELO

Read the text and answer questions 09 and 10.

www.garfiel.com

Questão 09 (EEAR/2021 – CFS) – According to the text, Garfield:


A) Thinks all days are beautiful.
B) Prefers the kitchen only when it’s raining.
C) Prefers staying outside if it’s a beautiful day.
D) Thinks days are beautiful in the kitchen even when it’s raining.

Questão 10 (EEAR/2021 – CFS) – The sentence “Now it’s raining” is in the:


A) Simple Past
B) Simple Present
C) Past Progressive
D) Present Progressive

QUESTÕES EAM
Questão 01 (EAM/INÉDITA) – Read the dialogue and mark the right option to fill in the gaps
respectively.
A: Good afternoon, Jack!
B: Hey, Jen! How are you doing?
A: I’m alright. What about yourself?
B: I’m doing well.
A: ___ you at Johnnie’s house last weekend?
B: Yes, I ____.
A: Who ____ you go with?
B: I ____ with my wife. What about you? Were you there?
A: No, I ___. I ____ home.
A) Did / sis / went / was / weren’t / was
B) Were / was / were / go / didn’t / went
C) Was / was / did / was / weren’t / was
D) Were / was / did / went / wasn’t / stayed
E) Did / did / did / went / didn’t / went

AULA 00 – ENGLISH GENERAL PRESENTATION 33


TEACHER ANDREA BELO

Questão 02 (EAM/INÉDITA) – Read the sentences and mark the correct option to fill in the
blanks respectively.
Elisa is __ friend. ___ lives near my grandfather’s house. We love riding our bikes. ___ bike is big.
___ is small. We love spending time together!
A) My / she / my / hers
B) Your / I / my / her
C) Her / he / her / mine
D) My / her / my / she
E) His / he / my / his

Questão 03 (EAM/INÉDITA) – Which word is incorrectly used in this meme?

A) Failed
B) Unpossible
C) My
D) In
E) Test

Questão 04 (EAM/INÉDITA) – Use the verbs in the parentheses to complete the following
statements.
I – I usually __ (do) my homework after dinner, but now I __ (do) a diferent task.
II – Eli ___ (eat) a lot. Sometimes, he ___ (want) to eat less
III – Alessandra ___(live) in Boston, United States. During the week, her daughters ___ (work) as
delivery girls for a big tech company.
Now mark the option which completes them respectively.
A) Am doing / am doing / is eating / wants / lives / working
B) Am doing / do / eats / is wanting / is living / works
C) Am doing / do / eat / is wanting / is living / works
D) Do / do / is eating / want / live / is working
E) Do / am doing / eats / wants / lives / work

AULA 00 – ENGLISH GENERAL PRESENTATION 34


TEACHER ANDREA BELO

Questão 05 (EAM/INÉDITA) – Look at the picture below.

What are they doing in the picture?


A) The children play soccer every day.
B) They are playing soccer now.
C) The boys didn’t play soccer yesterday.
D) They never play soccer together.
E) The boys won’t play soccer.

QUESTÕES EFOMM
STRANGE 'ICE VOLCANOES' ERUPT NEAR LAKE SUPERIOR, MICHIGAN
The National Weather Service (NWS) in Grand Rapids, Michigan, captured a photo of two "ice
volcanoes" erupting on Oval Beach in Saugatuck, near Lake Michigan, over the weekend.
"It was a great day to visit the beach and watch the waves interact with the ice," NWS tweeted.
Volcanoes and ice may sound like an unusual combination but according to Michigan Tech, they
are a regular occurrence on the north shore of Lake Superior, north of Lake Michigan, in the winter
months and can range in size from less than a meter to more than eight.
An ice volcano requires three things—high surf, cool temperatures and ice. For an ice volcano to
form, wave activity must be at least a meter tall at their highest. According to Michigan Tech,
anything less is too small to create the volcano's cone.
These volcanoes form at the edge of the ice shelf, caused by the movement of high surf hitting
the face of the formation. Small valleys can cause the energy of the waves to concentrate,
prompting larger waves to form that eject a spatter onto the ice shelf in certain places.
If the cones remain active but become completely enclosed by the ice shelf, wave energy from
below can push water up through any cracks or openings.
Ice volcanoes are typically found in arcs along the shoreline. They are often evenly spaced apart
but can also form individually. Michigan Tech refers to these lone structures as "cold spot" ice
volcanoes, and describes them as "nearly symmetrical single cones that apparently are not related
to shoreline, sand bar or rock reef arcs."

AULA 00 – ENGLISH GENERAL PRESENTATION 35


TEACHER ANDREA BELO

These solitary cones form near the shore and are thought to be a result of a weakness in the ice.
However, they have not yet been observed in an active state.
The phenomena has been observed on the shores of Lake Erie.
In Duluth, Minnesota, a human-made incarnation was created by a spewing fire hydrant.
According to Lake Superior Duluth Streams, the city allowed Orange Street's fire hydrant to spray
water during the winter months to prevent the supply pipe freezing over.This is not the only type
of ice volcano that exists in the solar system. Evidence of ice volcanoes (or cryovolcanoes) have
been found on Ceres, Titan and Pluto. These differ from the volcanoes we have on Earth. Instead
of ejecting molten rock, they emit substances such as water, ammonia and methane.
"To put them in perspective, if Mount Vesuvius had been a cryovolcano, its lava would have frozen
the residents of Pompeii," Rosaly Lopes, a Cassini radar team investigation scientist from NASA,
said in a statement in 2008.
Adapted from https://www.newsweek.com/ice-volcanoes-lake-superior-michigan-1487618

Questão 01 (EFOMM/INÉDITA) – It is possible to infer from the text that


a) The ice volcanoes are dangerous formations to the population of cities around them.
b) The Earth is changing and will be a similar planet to Pluto, Titan and Ceres.
c) In the future, Mount Vesuvius will freeze the population instead of burning it.
d) The ice volcanoes are a common formation that occurs on the north shore of Lake Superior.
e) The investigation scientist from NASA is worried about ice volcanoes.

Text II
Based on the text below, answer questions 02, 03 and 04.
The housing crisis and garden villages
In this piece, we take a look at the potential impact of garden villages, including regional
implications, what the result might be for new buyers, and follow on trends
There are many reports of a UK housing crisis, but what efforts are being put into resolving the
issue?
What do we mean by a garden village?
By definition, it is a piece of brownfield land that is used to develop new areas for families and
businesses. They are usually smaller projects and can contain from 1,500 to 10,000 homes. Often,
garden villages have their own facilities — such as schools, shops and transport stations — which
makes this type of living space perfect for families and first-time buyers looking to lead the
picture-perfect life.
Garden villages allow residents to imprint their own mark on the area, including creating its own
identity and creating rules. However, there are a few ways to identify them. They must be a
settlement outside of an existing town or city and not closely attached. The British government is
currently supporting 17 locations around the country, with £6 million expected to go towards

AULA 00 – ENGLISH GENERAL PRESENTATION 36


TEACHER ANDREA BELO

funding 14 new garden villages and £1.4 million to support three garden towns (which are similar
to garden villages, only larger).
Looking at the potential regional impact:
Regions are set to see more manual work available in these regions as such areas will be creating
more than 50,000 homes. This will help to boost the economy, as it will provide people with more
jobs in the area.
As new homes are built, people will be buying them — this potentially means more residents
within the region. There is a popular misconception that this will put a strain on the resources of
current residents nearby, such as school places for their children and obtaining doctor
appointments. However, this is not the case, as garden villages are built with their own facilities
including schools and general practices. In turn, this will also create more jobs in the area of
development.
However, it’s important to remember that these locations usually have their own transport links.
On the other hand, more cars on the road could cause congestion.
Adapted from https://www.openaccessgovernment.org/housing-crisis-garden-villages/68975/

Questão 02 (EFOMM/INÉDITA) – Read the statements about the text and decide whether they
are TRUE (T) or FALSE (F). Mark the correct option.
I – The Garden Villages will help developing the economy of the country.
II – Garden Villages will not affect traffic in the regions they are built in.
III – Garden Villages are a good solution for the housing crisis, but Garden Towns are a problem
for the government.
IV – A Garden Village must be a settlement outside of an existing town or city and not closely
attached.
V – Garden Villages can cause a strain on the resources of current residents nearby, such as
school places for their children and obtaining doctor appointments.
a) F / T / T / F / F
b) T / F / F / T / F
c) T / T / T / F / T
d) F / F / F / T / F
e) F / T / F / T / T

Questão 03 (EFOMM/INÉDITA) – In the excerpt “Garden villages allow residents to imprint their
own mark on the area, including creating its own identity and creating rules.”, the word in bold
means:
a) Emboss
b) Stymie
c) Destroy
d) Neglect
e) Dislodge

AULA 00 – ENGLISH GENERAL PRESENTATION 37


TEACHER ANDREA BELO

Questão 04 (EFOMM/INÉDITA) – According to the text, Garden Towns


a) Can be placed anywhere.
b) Have around 1,000 homes.
c) Are similar to garden villages, but larger.
d) Are a better version of garden villages.
e) Will be implemented in 17 locations.

Questão 05 (EFOMM/INÉDITA) – Choose the correct option to complete the paragraph below.
MARITIME AUTONOMY: A BRIDGE TOO FAR
Imagine ________ ships en route from departure port ‘A’ to arrive at different ports around the
world, each at a given time. These ships have no captain or crew _______ board and can navigate,
dock, load, unload and refuel on their own and are maintained by sensors, robots and drones. The
ships are navigated and controlled by computers from a fully automated port with no human
intervention or interaction. The operating system of each vessel makes decisions and takes
actions based on the situation it is _______. That’s ‘autonomous shipping’ and it is very different
from automated ships.
Adapted from https://www.oneseaecosystem.net/maritime-autonomy-a-bridge-too-far/

a) very / on / on
b) many / in / at
c) several / on / in
d) much / on / in
e) hundreds of / in / on

QUESTÕES EPCAR
Directions: Answer questions 01 to 08 according to TEXT I.
TEXT I
Over 100,000 Flamingos Reportedly Descend on Mumbai Amid India Strict Coronavirus
Lockdown
As people around the world stay inside to fight the spread of coronavirus, wild animals have begun
to enjoy the newfound space. That appears to be what’s happened in the metropolitan region of
Mumbai, India, where a record-breaking number of flamingos have migrated, painting the
wetlands pink, according to local reports.
The Bombay Natural History Society (BNHS) estimates that around 25% more flamingos have
migrated to the region compared to last year, likely caused by the decrease in human activity, the
Hindustan Times reports. The Science Times calculates around 150,000 flamingos have come to
the area.

AULA 00 – ENGLISH GENERAL PRESENTATION 38


TEACHER ANDREA BELO

India has been under a strict lockdown since March 25, which has required over a billion people
to stay home and shutter all but essential services, in an attempt to stop the spread of COVID-19,
the disease caused by novel coronavirus. On May 1, the Indian government extended the
lockdown through May 18.
Flamingos typically migrate to wetlands of the metropolitan region of Mumbai from November to
May, Rhul Khot, the assistant director of the BNHS, told the Times. But this year’s lockdown “is
giving these birds peace for roosting, no disturbance in their attempt to obtain food and overall
encouraging habitat,” Deepak Apte, the director of the BNHS, explained, per Times. He added that
the increase is also likely tied to a successful breeding season two years ago, as well as the
destruction of wetlands on India’s eastern seafront that could be pushing the birds to the Mumbai
region, per the Times.
Khot also told the Times that an increase in “domestic sewage” from people staying at home
during the lockdown “is helping the undisturbed formation of planktons, algae and microbenthos
formation, which forms the food for flamingos and other wetland birds.”
As the coronavirus pandemic has continued, animals have been spotted throughout the world in
places that are usually dominated by humans. CNN reports that dolphins have swam in India’s
Ganges river for the first time in years, and hundreds of monkeys have “descended” on Delhi as
the city remains under lockdown. Cougars were reportedly spotted prowling the streets in
Santiago, Chile, in April; wild Kashmiri goats meandered through on the town of Llandudno,
Wales, in March.
“Residents are cooped up at home spending their mornings and evenings at their balconies taking
photographs and videos of these relaxed birds,” Sunil Agarwal, a resident of Seawoods in Navi
Mumbai, told the Times about the flamingos. “The lockdown will at least prompt people to focus
on what is around them, which they had been taking for granted, and hopefully this site will be
declared a flamingo sanctuary soon.”
(Adapted from https://time.com/5831198/flamingos-coronavirus/)

Questão 01 (EPCAR/INÉDITA) – Mark the INCORRECT option.


a) Many more flamingos than usual have migrated to the region of Mumbai.
b) Less human activity is likely to encourage wild life to migrate to big cities.
c) Only essential services are open during the strict lockdown in India in order to stop the spread
of COVID-19.
d) They have been successfully destroying the wetlands on India’s eastern seafront.

Questão 02 (EPCAR/INÉDITA) – According to the text, mark the correct alternative.


a) Residents are angry about being at home due to the lockdown imposed in India.
b) 150,000 more flamingos have migrated to the region of Mumbai as a result of the 25%
increase.
c) There has never been so many flamingos migrating to the region of Mumbai.
d) Cougars were reported spotted attacking people in the streets of Santiago, Chile.

AULA 00 – ENGLISH GENERAL PRESENTATION 39


TEACHER ANDREA BELO

Questão 03 (EPCAR/INÉDITA) – In the sentence “the increase is also likely tied to a successful
breeding season two years ago” (paragraph 4) the verb tied means
a) associated.
b) the outcome.
c) permission.
d) avoided.

Questão 04 (EPCAR/INÉDITA) – Mark the correct question to the sentence below.


“around 25% more flamingos have migrated to the region.”
a) How many more flamingos have migrated to the region?
b) How much more flamingos have migrated to the region?
c) How many flamingos have migrated to the region?
d) How much flamingos have migrated to the region?

Questão 05 (EPCAR/INÉDITA) – Mark the alternative that completes the sentence.


One of the reasons why flamingos have migrated in greater number to the region of Mumbai is
a) the fact that the weather is better now due to the lockdown.
b) the destruction of the wetlands on India’s eastern seafront.
c) the increase in domestic sewage from people staying at home during the lockdown.
d) that people we are breeding flamingos in the region of Mumbai.

Questão 06 (EPCAR/INÉDITA) – Mark the alternative that DOESN’T complete the


sentence below.
Wildlife has been seen during the pandemic in
a) Delhi, where monkeys were seen in the city during the lockdown.
b) Ganges river, India, where dolphins were seen for the first time in years.
c) Santiago, where cougars were seen in the forests outside the city.
d) Llandudno, where kashmiri goats were seen rambling through the town.

Questão 07 (EPCAR/INÉDITA) – Read the sentences and mark the correct option.
I. Lockdown has been good to people and animals, who can have the city for themselves.
II. People being cooped up at home is a good thing for animals that can enjoy new spaces.
III. This number of flamingos migrating to the region of Mumbai was only possible due to
humans destroying their original habitat.
The only correct sentence(s) is(are)
a) II.
b) III.
c) I and II.
d) I and III.

AULA 00 – ENGLISH GENERAL PRESENTATION 40


TEACHER ANDREA BELO

Questão 08 (EPCAR/INÉDITA) – Mark the correct alternative to complete the sentence.


According to the text, the lockdown
a) makes people happy because nature is thriving in this situation.
b) has been responsible for significant changes in wildlife behavior.
c) has been responsible for the death of a lot of dolphins around the world.
d) is responsible for this site being considered a flamingo sanctuary.

Directions: Answer questions 09 to 15 according to TEXT II.


TEXT II
It’s boom time for podcasts – but will going mainstream kill the magic?
Fifteen years ago, when the word podcast was added to the dictionary, only the tech-savvy
were listening. Now, as star names pile in, they’re big business. Can the quality survive?
Hello friends! Do you fancy listening to “a new type of time-shifted amateur radio”? No? How
about a brilliant podcast? Of course you do.
Fifteen years ago, Macworld, a magazine for fans of Apple products, announced, with limited
fanfare, that Apple was about to add podcasts to iTunes, its music download offer. Unfortunately,
few readers knew what a podcast was, hence Macworld’s “time-shifted radio” definition. In June
2005, the idea of having thousands of ready-to-hear audio shows, anything from true-crime
documentaries to all-chums-together comedy, to up-to-the-minute news to gripping drama to
revealing interviews, and being able to listen to these shows whenever you want, wherever you
are – well, that wasn’t quite happening. So Apple’s move didn’t seem important. Nor did the fact
that the Oxford English Dictionary added “podcast” to its lexicon in the same year, after tech
journalist Ben Hammersley came up with the term in 2004 (which was also the year the BBC
launched a downloadable version of In Our Time). Podcasts were new. It takes time for the new
to become everyday.
Podcasts were mostly unheard of, except by the tech savvy. They were either downloadable
versions of existing radio shows or they were chatty riffs, made by amateurs who knew how to
upload their aural blogs online. Still, they were interesting. At least to me. Soon after podcasts’
iTunes debut, I started a new job as the Observer’s radio critic. Great job – except there was a
limited choice of programmes for me to review. Radio schedules rarely changed. Presenters
stayed in their jobs for years. The BBC dominated speech radio, aside from phone-ins; hardly any
other broadcaster had the money to make documentaries or drama.
Podcasts rescued me from aural monotony; I wrote my first piece about them in the summer of
2006. Apparently Coke Machine Glow and The Dawn and Drew Show were the ones to look out
for (me neither, now). The podcast I do recall from then is The Ricky Gervais Show; this dominated
the brand new iTunes podcast chart for weeks. Initially free, in early 2006 it switched to a pay-
per-listen model and proved both a forerunner and an outlier: since then, much podcast uptake
has been driven by comedy, but most shows are still free to listeners, paid for by adverts that
appear during episodes.

AULA 00 – ENGLISH GENERAL PRESENTATION 41


TEACHER ANDREA BELO

Today, the iTunes podcast chart is bustling with old hands and new kids on the block. Here are No
Such Thing As a Fish, Fearne Cotton’s Happy Place, That Peter Crouch Podcast, Katherine Ryan:
Telling Everybody Everything. Here are sections for new and noteworthy, cultivating calm, keeping
the kids busy. There are lists of the top 10 episodes, top 10 shows; all aside from the 19 other
regular categories: news, arts, true crime… So many podcasts! There are oodles of shows, too
many to ever get through.
(Adapted from https://www.theguardian.com/tv-and-radio/2020/may/03/its-boom-time-for-podcasts-but-will-going-mainstream-kill-the-magic)

Questão 09 (EPCAR/INÉDITA) – In the underlined sentences in the second paragraph, there are
____ verbs used in the past tense.
a) two
b) five
c) three
d) four

Questão 10 (EPCAR/INÉDITA) – Podcasts


a) became mainstream in 2004.
b) were mostly unknown 15 years ago.
c) were added to Oxford English Dictionary due to their huge popularity in 2004.
d) didn’t took long to become mainstream.

Questão 11 (EPCAR/INÉDITA) – Read the sentence. “Podcasts were mostly unheard of, except
by the tech savvy.” (Paragraph 3)
All the sentences below express a similar idea, EXCEPT
a) Aside tech savvy people, podcasts were mostly unheard of.
b) Aside tech savvy people, few knew podcasts.
c) many people knew podcasts, except by the tech savvy.
d) podcasts were mostly unheard of for those that were not tech savvy.

Questão 12 (EPCAR/INÉDITA) – The word chart (paragraph 4) can be replaced, without change
in meaning, by
a) channel.
b) show.
c) catalogue.
d) concert.

AULA 00 – ENGLISH GENERAL PRESENTATION 42


TEACHER ANDREA BELO

Questão 13 (EPCAR/INÉDITA) – The opposite of savvy (paragraph 3) is


a) ignorant.
b) experienced.
c) versed.
d) adept.

Questão 14 (EPCAR/INÉDITA) – Read the sentence.


“Do you fancy listening to “a new type of time-shifted amateur radio”?” (First paragraph).
The word fancy has the same idea in
a) You have a fancy apartment.
b) Is her purse fancy?
c) They are using some fancy equipment.
d) I quite fancy the idea of going to the movies.

Questão 15 (EPCAR/INÉDITA) – The author considers (that)


a) podcasts will soon replace traditional radio broadcasters.
b) himself an enthusiast of podcasts.
c) there is no difference between radio shows and podcasts nowadays.
d) there aren’t enough podcasts available on iTunes.

QUESTÕES ESA
The Dunning-Kruger Effect
The Dunning-Kruger effect is a type of cognitive bias in which people believe that they are smarter
and more capable than they really are. Essentially, low ability people do not possess the skills
needed to recognize their own incompetence. The combination of poor self-awareness and low
cognitive ability leads them to overestimate their own capabilities.
The term lends a scientific name and explanation to a problem that many people immediately
recognize—that fools are blind to their own foolishness. As Charles Darwin wrote in his book The
Descent of Man, "Ignorance more frequently begets confidence than does knowledge."
(Adapted from https://www.verywellmind.com/an-overview-of-the-dunning-kruger-effect-4160740)

Questão 01 (ESA/INÉDITA) – According to the text, it is correct to infer that:


(A) the Dunning-Kruger effect is a mental disease.
(B) people affected by the Dunning-Kruger effect have low self esteem.
(C) ignorance makes people lack confidence.
(D) Charles Darwin wrote a book about the Dunning-Kruger effect.
(E) the effect makes incompetent people think they are smart.

AULA 00 – ENGLISH GENERAL PRESENTATION 43


TEACHER ANDREA BELO

Questão 02 (ESA/INÉDITA) – The underlined words in the text- “smarter”, “more capable” and
“fools” are, respectively,
(A) adjective (comparative), adjective (comparative), and noun.
(B) adjective (superlative), adjective (comparative), and noun.
(C) noun, adjective (comparative), and adjective (comparative).
(D) adjective (comparative), adjective (superlative), and noun.
(E) adjective (superlative), noun, and adjective (superlative).

Questão 03 (ESA/INÉDITA) – Complete the sentences below using the most appropriate words:
Lockdown is __ measure adopted to make sure people are at home. This is __ only way to oblige
___ people to stay confined.
(A) a lot of / a / the
(B) a / the / a lot of
(C) many / a / a
(D) a / an / a lot of
(E) a / the / a

Questão 04 (ESA/INÉDITA) – Which sentence is grammatically correct?


(A) One of those who is part of your family can be a person affected by the Dunning Kruger
effect.
(B) One of those who are part of your family can be some people affected by the Dunning
Kruger effect.
(C) One of those who are part of your family can be a person affected by the Dunning Kruger
effect.
(D) One of those whose are part of your family can be a person affected by the Dunning Kruger
effect.
(E) One of those whom are part of your family can be a person affected by the Dunning Kruger
effect.

Questão 05 (ESA/INÉDITA) – Complete the sentences below using the most appropriate words:
Technology is __ fastest way to develop things in general. It’s incredible how everything
changed and __ world became different. It’s one of __ most interesting happenings in the
history.
(A) a / the/ a
(B) a / a/ a
(C / a / a/ the
(D) an / a/ the
(E) the / the/ the

AULA 00 – ENGLISH GENERAL PRESENTATION 44


TEACHER ANDREA BELO

QUESTÕES ESCOLA NAVAL


Based on the text below, answer the six questions that follow it. The paragraphs of the text are
numbered.
What Leads Us to this Belief that Connections to and Experiences with the Natural World Are
So Important?
There is an emerging awareness and concern that children are less and less likely to have
experiences that involve the simplest interactions with nature—the plants, animals, and the earth
around them—as a part of their continuous learning process. As we look at the children around
us, we observe them living increasingly unhealthy lifestyles. For many, childhood is spent overly
plugged in and programmed inside their homes, schools, and community settings in humanmade
environments, eliminating the out-of-doors, the benefits of nature, and all that exists in the
natural environment (Wike, 2006).
Even a generation ago, children spent more time outside, because it was the normal thing for
children to do. Adults did not question the value of time spent out-of-doors and had much less
anxiety about the risks involved. Children walked and played outdoors and planted things in the
dirt; they rode their bikes, invented games, and spent the majority of their time in less structured
activities and natural environments. Very young children carried out these activities in their yards
and immediate neighborhoods. Urban environments offered the occasional playground and
vacant lot. Older children roamed beyond their neighborhoods to adjacent lands, streams, woods,
or urban parks. Exposure to the natural world brought opportunities for children to make sense
of their surroundings and to develop their own sense of “place.”
Over a relatively short time, we adults have allowed this connection to the natural world to slip
gradually away from children’s lives. Evidence of this trend surrounds us:
Children now spend nearly 30 hours a week watching a TV or computer screen, listening to
something through headphones or, for older children, using cell phones or media players;
Children experience increasingly timed and structured family lifestyles with less emphasis on
unstructured outside time. Particularly in more densely populated countries, urban growth has
eliminated green spaces and natural environments. Fewer families are vacationing in national
parks (Fish, 2007). Together these changes keep many children separated from nature and
without time for solitude or um-programmed experiences.
Instructional time outside, recess, or unstructured playtime is being eliminated from the school
day (Clements, 2007).
Outdoor play spaces or playgrounds have become safer at some sites due to national playground
standards, but in other locations, play spaces have been eliminated or often lack the natural
elements that encourage a different kind of interaction among children (Moore & Wong, 1997).
The curriculum for children in centers and schools is becoming narrower, with more time spent
on teacher-directed lessons and testing and less time spent investigating and learning through
activities that build on a child’s sense of wonder, curiosity, and the benefit of first-hand
experiences (Hyson, 2003; McMurrer, 2007; Marcon, 1999).
Consequently, we have gradually found our children growing up in a clash of optimal and minimal
learning opportunities. Optimally, technology opens worlds never before so readily available to

AULA 00 – ENGLISH GENERAL PRESENTATION 45


TEACHER ANDREA BELO

children; however, the opening of this side of learning has contributed to shutting the door to
children’s access to the more natural environment that gives a lasting attachment to children’s
sense of place and their awareness of the habitat and environment nearest to them. This lack of
connection can engender both apathy and ignorance in children’s early perceptions of the world
around them and their roles in enjoying, learning from, and protecting it.
(Adapted from: https://www.education.ne.gov/wpcontent/uploads/2017/07/Call_to_Action.pdf)

Questão 01 (ESCOLA NAVAL/INÉDITA) – According to the text, which option completes the
sentence below correctly?
In the past, it was the normal thing for children to ______.
(A) spent more time outside
(B) spend more time indoors
(C) question the value of time spent out-of-doors
(D) spend more time outdoors
(E) spend their times in programmed inside activities

Questão 02 (ESCOLA NAVAL/INÉDITA) – What's the meaning of the word "engender" in


paragraph 4?
(A) Point out.
(B) Disagree.
(C) Give rise to.
(D) Endanger.
(E) Be caused by.

Questão 03 (ESCOLA NAVAL/INÉDITA) – According to the text, which option is correct?


(A) Today children have the maximum and the minimum at the same time when it comes to
learning opportunities.
(B) In the past, education was better because children had access to more outside time at
schools.
(C) Thanks to technology education is better today, since children have instant access to things
other generations didn’t.
(D) Technology access will give children the necessary will to fight for the planet environment in
the future.
(E) The fact the unstructured playtime is being eliminated from the school day is a good thing for
raising awareness of the habitat nearest to the children.

AULA 00 – ENGLISH GENERAL PRESENTATION 46


TEACHER ANDREA BELO

Questão 04 (ESCOLA NAVAL/INÉDITA) – According to the text, which option is correct?


(A) Children now spend around 6 hours a day watching TV or computer screen or doing
something technology-related.
(B) Children spend more and more time in structured family lifestyles emphasizing outdoor
activities.
(C) There is a concern that children have less and less contact with nature and that leads them
to an unhealthy lifestyle.
(D) Children’s structured playtime is being eliminated from their school day.
(E) Children don’t play outside in the nature because National playground standards eliminated
natural spaces from playgrounds.

Questão 05 (ESCOLA NAVAL/INÉDITA) – In paragraph 2, the word "it" refers to


(A) children.
(B) generation.
(C) natural environment.
(D) childhood.
(E) spent more time outside.

QUESTÕES EsPCEx
Leia o texto a seguir e responda às questões 01, 02 e 03
How to Make the Study of Consciousness Scientifically Tractable
Strangely, modern science was long dominated by the idea that to be scientific means to remove
consciousness from our explanations, in order to be “objective.” This was the rationale behind
behaviorism, a now-dead theory of psychology that took this trend to a perverse extreme.
Behaviorists like John Watson and B.F. Skinner scrupulously avoided any discussion of what their
human or animal subjects thought, intended or wanted, and focused instead entirely on behavior.
Erwin Schrödinger labeled this approach in his 1958 book Mind and Matter, the “principle of
objectivation”. Schrödinger did identify both the problem and the solution. He recognized that
“objectivation” is just a simplification that is a temporary step in the progress of science in
understanding nature.
We are now at the point, it seems to a growing number of thinkers who are finally listening to
Schrödinger, where we must abandon, where appropriate, the principle of objectivation. It is time
for us to employ a “principle of subjectivation” and in doing so understand not just half of reality—
the objective world—but the whole, the external and internal worlds.
The science of consciousness has enjoyed a renaissance in the last couple of decades and the
study of our own minds—consciousness/subjectivity—has finally become a respectable pursuit.
It’s still tricky, however, to determine what kinds of data and what kinds of experiments we should
consider legitimate in the study of consciousness.
Adapted from https://blogs.scientificamerican.com/observations/how-to-make-the-study-of-consciousness-scientifically-tractable/

AULA 00 – ENGLISH GENERAL PRESENTATION 47


TEACHER ANDREA BELO

Questão 01 (EsPCEx/INÉDITA) – According to the text, choose the correct statement.


A) the science of consciousness has regained importance over the last years.
B) the rationale behind behaviorism was opposed to being objective.
C) John Watson and B.F. Skinner focused both on behaviour and consciousness.
D) modern science is still dominated by the idea that scientific means objective
E) Schrödinger has been proven wrong by a growing number of thinkers.

Questão 02 (EsPCEx/INÉDITA) – In the sentence “This was the rationale behind behaviorism, a
now-dead theory of psychology that took this trend to a perverse extreme.” (Paragraph 1), the
word rationale means:
A) excuse
B) logic
C) prevention
D) proof
E) fate

Questão 03 (EsPCEx/INÉDITA) – The word did in the sentence “Schrödinger did identify both the
problem and the solution.” (paragraph 2) expresses
A) emphasis
B) irrelevance
C) contrast
D) conclusion
E) denial

Leia o texto a seguir e responda às questões 04, 05 e 06


What will art look like in 20
The future may be uncertain, but some things are undeniable: climate change, shifting
demographics, geopolitics. The only guarantee is that there will be changes, both wonderful and
terrible. It’s worth considering how artists will respond to these changes, as well as what purpose
art serves, now and in the future.
Reports suggest that by 2040 the impacts of human-caused climate change will be unescapable,
making it the big issue at the centre of art and life in 20 years’ time. Artists in the future will wrestle
with the possibilities of the post-human and post-Anthropocene – artificial intelligence, human
colonies in outer space and potential doom.
The identity politics seen in art around the #MeToo and Black Lives Matter movements will grow
as environmentalism, border politics and migration come even more sharply into focus. Art will

AULA 00 – ENGLISH GENERAL PRESENTATION 48


TEACHER ANDREA BELO

become increasingly diverse and might not ‘look like art’ as we expect. In the future, once we’ve
become weary of our lives being visible online for all to see and our privacy has been all but lost,
anonymity may be more desirable than fame. Instead of thousands, or millions, of likes and
followers, we will be starved for authenticity and connection. Art could, in turn, become more
collective and experiential, rather than individual.
Adapted from http://www.bbc.com/culture/story/20190418-what-will-art-look-like-in-20-years

Questão 04 (EsPCEx/INÉDITA) – Which question below has its answer in paragraph 3?


A) What are some undeniable things about the future?
B) Who studied about art in the future?
C) When will the impacts of human-caused climate change be unescapable?
D) What will art look like in the future?
E) What are some possibilities of the post-human and post-Anthropocene?

Questão 05 (EsPCEx/INÉDITA) – Which one from the underlined verbs in the text conveys a verb
form that is different from the others?
A) be (paragraph 1)
B) respond (paragraph 1)
C) wrestle (paragraph 2)
D) seen (paragraph 3)
E) look (paragraph 3)

Questão 06 (EsPCEx/INÉDITA) – Choose the alternative that correctly substitutes the sentences
“The future may be uncertain, but some things are undeniable...” (paragraph 1) with no change
in meaning.
A) The future may be uncertain, so some things are undeniable.
B) The future may be uncertain, because some things are undeniable.
C) The future may be uncertain. Some things are undeniable, as well.
D) The future may be uncertain; moreover, some things are undeniable.
E) The future may be uncertain. Some things are undeniable, though.

Leia o texto a seguir e responda às questões 07, 08 e 09


Internet privacy: the apps that protect you from your apps
Tech companies don’t have favourite songs, but if they __________ (1), they __________ (2) all
pick Radiohead’s Just – “You do it to yourself, you do/ And that’s what really hurts,” they would

AULA 00 – ENGLISH GENERAL PRESENTATION 49


TEACHER ANDREA BELO

croon, staring their users dead in the eye. And strictly speaking, they’d be right: many of the worst
excesses of the industry are, technically, optional. The world isn’t actually a binary choice between
living in a surveillance state and opting out of all technological development since the turn of the
millennium. You can opt out – you just have to know how.
Of course, that knowledge is not always easily acquired, nor is it necessarily easy to apply. So a
new breed of services has arrived to try to help normal users take control of their digital lives.
Companies including Disconnect.
Me and Jumbo act as something like a digital concierge for their users, tweaking privacy settings,
deleting sensitive data and throwing a spanner into the inner workings of surveillance capitalism.
But there’s a Faustian pact involved: to use the privacy apps to their fullest requires handing them
a level of control over your digital life that would be all too easy to abuse – and it’s hard to be
certain that any company can be trusted with information that sensitive.
Most of the companies that track you aren’t open enough for you to even know they’re snooping
on you in the first place.
Adapted from https://www.theguardian.com/technology/2020/feb/16/internet-privacy-settings-apps-to-protect-you-

Questão 07 (EsPCEx/INÉDITA) – According to the text, read the statements and choose the
correct alternative.
A new kind of services has emerged to try to help users take control of their digital lives.
There are only two possible choices in digital life: living in a surveillance state or choosing not to
participate of technology.
Most of the companies that track people are honest about snooping on them.
Using privacy apps requires handing them a safe level of control over users’ digital life.
Companies including Disconnect.Me and Jumbo work as a digital curator for their users, deleting
sensitive data, for example.
A) I and V are correct.
B) All of them are correct.
C) III, IV and V are correct.
D) II, III and IV are correct.
E) I and III are correct.

Questão 08 (EsPCEx/INÉDITA) – In the sentence “Most of the companies that track you aren’t
open enough for you to even know they’re snooping on you in the first place.” (Paragraph 3), the
verb snooping means
A) tolerating
B) preparing
C) spying
D) serving
E) neglecting

AULA 00 – ENGLISH GENERAL PRESENTATION 50


TEACHER ANDREA BELO

Questão 09 (EsPCEx/INÉDITA) – The sentence “Tech companies don’t have favourite songs, but
if they __________ (1), they __________ (2) all pick Radiohead’s Just...” refers to a hypothetical
situation. Choose the alternative containing the correct verb tenses to complete gaps (1) and (2)
in paragraph 1.
A) do, will
B) did, would
C) do, would
D) didn’t, would
E) don’t, won’t

Leia o texto a seguir e responda à questão 10


Brain surgery patient filmed playing violin during operation
A patient at King’s College Hospital in London has played the violin while surgeons operated on
her brain to remove a tumour. The medical team asked Dagmar Turner, 53, to play the violin to
ensure parts of the brain which control delicate hand movements and coordination were not
damaged during the millimetre-precise surgery.
Ms Turner was diagnosed in 2013 with a brain tumour after suffering a seizure during a symphony.
The committed violinist was worried the surgery would mean she would lose the ability to play.
Her tumour was located in the right frontal lobe of her brain, close to an area that controls the
fine movement of her left hand. Professor Keyoumars Ashkan, consultant neurosurgeon at King’s
College Hospital, came up with a plan to operate and reduce the risk.
Prior to surgery, doctors spent two hours carefully mapping her brain to identify areas that were
active when she played the violin and those responsible for controlling language and movement.
Ms Turner played violin while her tumour was removed, while closely monitored by the
anaesthetists and a therapist.
Prof Ashkan said: “We managed to remove over 90 per cent of the tumour, including all the areas
suspicious of aggressive activity, while retaining full function in her left hand.”Ms Turner added:
“The violin is my passion; I’ve been playing since I was 10 years old. The thought of losing my
ability to play was heartbreaking.”
Adapted from https://www.independent.co.uk/news/health/violin-brain-surgery-kings-hospital-
nhs-a9342006.html

Questão 10 (EsPCEx/INÉDITA) – The sentence


Ms Turner was diagnosed in 2013 with a brain tumour by doctors.
is correctly changed into active voice in
A) Doctors diagnose Ms Turner in 2013 with a brain tumour.
B) Doctors diagnosed Ms Turner in 2013 with a brain tumour.
C) Doctors diagnoses Ms Turner in 2013 with a brain tumour.
D) Doctors have diagnosed Ms Turner in 2013 with a brain tumour.
E) Doctors had diagnosed Ms Turner in 2013 with a brain tumour.

AULA 00 – ENGLISH GENERAL PRESENTATION 51


TEACHER ANDREA BELO

GABARITO
GABARITO AFA
01 – B 02 – B 03 – A 04 – Anulada 05 – A
06 – D 07 – C 08 – B 09 – B 10 – D

GABARITO COLÉGIO NAVAL


01 – C 02 – D 03 – B 04 – B 05 – C
06 – E 07 – D 08 – A 09 – D 10 – E

GABARITO EEAR
01 – A 02 – A 03 – A 04 – D 05 – D
06 – B 07 – A 08 – D 09 – D 10 – D

GABARITO EAM
01 – D 02 – A 03 – B 04 – E 05 – B

GABARITO EFOMM
01 – D 02 – B 03 – A 04 – C 05 – C

GABARITO EPCAR
01 – D 02 – C 03 – A 04 – A 05 – B
06 – C 07 – A 08 – B 09 – D 10 – B
11 – C 12 – C 13 – A 14 – D 15 – B

GABARITO ESA
01 – E 02 – A 03 – B 04 – C 05 – E

GABARITO ESCOLA NAVAL


01 – D 02 – C 03 – A 04 – C 05 – E

GABARITO EsPCEx
01 – A 02 – B 03 – A 04 – D 05 – D
06 – E 07 – A 08 – C 09 – B 10 – B

AULA 00 – ENGLISH GENERAL PRESENTATION 52


TEACHER ANDREA BELO

QUESTÕES COMENTADAS
QUESTÕES AFA
Directions: Read the text below and answer questions 01 to 10 according to it.
TEXT Howard Gardner: ‘Multiple intelligences’ are not ‘learning styles’ by Valerie Strauss
The fields of psychology and education were revolutionized 30 years ago when we now
worldrenowned psychologist Howard Gardner published his 1983 book Frames of Mind: The
Theory of Multiple Intelligences, which detailed a new model of human intelligence that went
beyond the traditional view that there was a single kind that could be measured by standardized
tests.
Gardner’s theory initially listed seven intelligences which work together: linguistic, logical-
mathematical, musical, bodily-kinesthetic, interpersonal and intrapersonal; he later added an
eighth, naturalist intelligence and says there may be a few more. The theory became highly
popular with K-12¹ educators around the world seeking ways to reach students who did not
respond to traditional approaches, but over time, ‘multiple intelligences’ somehow became
synonymous with the concept of ‘learning styles’. In this important post, Gardner explains why the
former is not the latter.
It’s been 30 years since I developed the notion of ‘multiple intelligences’. I have been gratified by
the interest shown in this idea and the ways it’s been used in schools, museums, and business
around the world. But one unanticipated consequence has driven me to distraction and that’s the
tendency of many people, including persons whom I cherish, to credit me with the notion of
‘learning styles’ or to collapse ‘multiple intelligences’ with ‘learning styles’. It’s high time to relieve
my pain and to set the record straight.
First a word about ‘MI theory’. On the basis of research in several disciplines, including the study
of how human capacities are represented in the brain, I developed the idea that each of us has a
number of relatively independent mental faculties, which can be termed our ‘multiple
intelligences’. The basic idea is simplicity itself. A belief in a single intelligence assumes that we
have one central, all-purpose computer, and it determines how well we perform in every sector
of life. In contrast, a belief in multiple intelligences assumes that human beings have 7 to 10
distinct intelligences.
Even before I spoke and wrote about ‘MI’, the term ‘learning styles’ was being bandied about in
educational circles. The idea, reasonable enough on the surface, is that all children (indeed all of
us) have distinctive minds and personalities. Accordingly, it makes sense to find out about learners
and to teach and nurture them in ways that are appropriate, that they value, and above all, are
effective.
Two problems: first, the notion of ‘learning styles’ is itself not coherent. Those who use this term
do not define the criteria for a style, nor where styles come from, how they are recognized/
assessed/ exploited. Say that Johnny is said to have a learning style that is ‘impulsive’. Does that
mean that Johnny is ‘impulsive’ about everything? How do we know this? What does this imply
about teaching? Should we teach ‘impulsively’, or should we compensate by ‘teaching
reflectively’? What of learning style is ‘right-brained’ or visual or tactile? Same issues apply.

AULA 00 – ENGLISH GENERAL PRESENTATION 53


TEACHER ANDREA BELO

Problem #2: when researchers have tried to identify learning styles, teach consistently with those
styles, and examine outcomes, there is not persuasive evidence that the learning style analysis
produces more effective outcomes than a ‘one size fits all approach’. Of course, the learning style
analysis might have been inadequate. Or even if it is on the mark, the fact that one intervention
did not work does not mean that the concept of learning styles is fatally imperfect; another
intervention might have proved effective. Absence of evidence does not prove non-existence of a
phenomenon; it signals to educational researchers: ‘back to the drawing boards’.
Here’s my considered judgment about the best way to analyze this lexical terrain: Intelligence: We
all have the multiple intelligences. But we signed out, as a strong intelligence, an area where the
person has considerable computational power. Style or learning style: A hypothesis of how an
individual approaches the range of materials. If an individual has a ‘reflective style’, he/she is
hypothesized to be reflective about the full range of materials. We cannot assume that
reflectiveness in writing necessarily signals reflectiveness in one’s interaction with the others.
Senses: Sometimes people speak about a ‘visual’ learner or an ‘auditory’ learner. The implication
is that some people learn through their eyes, others through their ears. This notion is incoherent.
Both spatial information and reading occur with the eyes, but they make use of entirely different
cognitive faculties. What matters is the power of the mental computer, the intelligence that acts
upon that sensory information once picked up.
These distinctions are consequential. If people want to talk about ‘an impulsive style’ or a ‘visual
learner’, that’s their prerogative. But they should recognize that these labels may be unhelpful, at
best, and ill-conceived at worst.
In contrast, there is strong evidence that human beings have a range of intelligences and that
strength (or weakness) in one intelligence does not predict strength (or weakness) in any other
intelligences. All of us exhibit jagged profiles of intelligences. There are common sense ways of
assessing our own intelligences, and even if it seems appropriate, we can take a more formal test
battery. And then, as teachers, parents, or selfassessors, we can decide how best to make use of
this information.
(Adapted from https://www.washingtonpost.com/news/answer-sheet)

Questão 01 (AFA/2016) – The text


a) aims at highlighting distinctive mind barriers related to learning.
b) provides the reader with a bird’s-eye-view of Gardner’s landmark publication.
c) develops a considerable set of psychological and mental implications.
d) concerns about spending 30 years to measure people’s intelligence.
Comentários: Na letra A, há um erro ao afirmar que o texto tem como objetivo (o verbo aims
indica objetivo) destacar (highlight) diferentes barreiras mentais relacionadas ao aprendizado
(aims at highlighting distinctive mind barriers related to learning.) Na realidade, o texto foca no
trabalho de Howard Gardner.

AULA 00 – ENGLISH GENERAL PRESENTATION 54


TEACHER ANDREA BELO

A letra B representa exatamente a ideia do texto: provides the reader with a bird’s-eye-view of
Gardner’s landmark publication. Ele fornece (provides) ao leitor uma visão geral da publicação
de Gardner. Alternativa correta.
A alternativa C erra ao dizer que o texto desenvolve implicações mentais e psicológicas: develops
a considerable set of psychological and mental implications. Na verdade, o texto trata da diferença
entre múltiplas inteligências (multiple intelligences) e estilos de aprendizado (learning styles).
A alternativa D está incorreta porque diz que há preocupações (concerns) pelo fato de serem
necessários 30 anos para medir a inteligência de alguém: concerns about spending 30 years to
measure people’s intelligence. Na realidade, o trabalho de Gardner foi publicado
aproximadamente 30 anos atrás. Portanto, o texto relaciona “30 anos” com a publicação do
trabalho de Gardner, e não com o tempo de mensuração da inteligência.
OBS.: note que os verbos são utilizados no presente e que todas as alternativas se iniciam com
verbos acrescidos de “S”, pois o sujeito text pode ser substituído pelo termo it, pronome que
estudaremos adiante, numa outra aula para aprimorar esses conceitos também. Sabemos que os
verbos cujos sujeitos sejam HE/SHE/IT são acrescidos de “S” no tempo verbal simple present,
conforme a teoria estudada nessa aula.
GABARITO: B

Questão 02 (AFA/2016) – In the sentence “there was a single kind that could be measured by
standardized tests”, it is possible to find an option to substitute the pronoun accordingly in
a) when.
b) which
c) how.
d) whom.
Comentários: O pronome em questão é that.
A alternativa A está incorreta pois when é usado para designar lugar.
A alternativa B está correta porque a palavra which é um pronome que substitui perfeitamente
o pronome that nessa situação. Pode ser traduzido como “o/a qual”.
A alternativa C está incorreta pois how é usado para designar “como” ou “de que forma”.
A alternativa D está incorreta pois whom é usado como objeto na frase, e não como pronome
relativo.
OBS.: no trecho, encontramos os verbos no tempo passado simples (simple past). There was, could
e measured. É interessante perceber que there was é o passado de there is, could é passado de
can e measured é passado de measure. Dessa forma, vamos trabalhando o assunto da aula e nos
familiarizando com os diferentes tempos verbais.
GABARITO: B

AULA 00 – ENGLISH GENERAL PRESENTATION 55


TEACHER ANDREA BELO

Questão 03 (AFA/2016) – In the fragment “why the former is not the latter”, the highlighted
words refer to
a) multiple intelligences / learning style.
b) over time / theory.
c) ways to reach students / traditional approaches.
d) traditional approaches / K-12 educators.
Comentários: É importante entender que former e latter são palavras usadas para designar
“anterior” e “posterior”, respectivamente. Portanto, o trecho significa: o porquê de o anterior não
ser o posterior. Nesse caso, former e latter se referem a expressões anteriormente citadas no
texto. Former retoma “multiple intelligences” enquanto latter retoma “learning style”. “multiple
intelligences” é citado anteriormente no texto em relação a “learning style”. Isso permite
compreender de forma clara porque former se refere a “multiple intelligences” e latter se refere
a “learning styles”.
A alternativa A está correta. Ela está completamente de acordo com a explicação dada acima.
A alternativa B está incorreta. Former e latter são palavras utilizadas para retomar palavras
anteriormente citadas. Mas essas palavras são, normalmente, substantivos. Portanto, over time
não poderia ser retomado por former nem por latter.
As alternativas C e D são incorretas. Pode-se entender por que essas alternativas estão incorretas
analisando a explicação acima, na qual expliquei como os termos former e latter.
GABARITO: A

Questão 04 (AFA/2016) – In the third paragraph (lines 20 and 29), the author
a) delves into the mind of human beings.
b) reinforces the importance of a thirty-year study.
c) tries to make controversial issues clear.
d) apologizes for the fact that his theory has collapsed.
Comentários: QUESTÃO ANULADA – gerou muitas interpretações e muitas respostas de duplo
sentido.

Questão 05 (AFA/2016) – In the sentence “it’s been 30 years since I developed the notion of
‘multiple intelligences’”, the contraction refers to
a) It has.
b) It been.
c) It is.
d) It was.

AULA 00 – ENGLISH GENERAL PRESENTATION 56


TEACHER ANDREA BELO

Comentários: A alternativa A está correta. Trata-se de um verbo conjugado no tempo “present


perfect”. Ele deve, portanto, ser conjugado com o verbo auxiliar “have”, seguido do verbo
principal conjugado no “past partiple”. Portanto, “it’s been” equivale a “it has been”.
A alternativa B está incorreta. Não se utiliza sujeito + verbo no “past participle”. Tem de haver o
verbo auxiliar “have” para que a estrutura esteja correta.
A alternativa C está incorreta. O uso do verbo “to be” no particípio “been” é o grande indicador
de que temos uma estrutura no “present perfect” e, portanto, faz-se necessário o uso do auxiliar
“have”.
A alternativa D está incorreta. “Was” é um verbo conjugado no “simple past” enquanto o trecho
está conjugado no “present perfect”. Não é possível usar “was” acompanhado de verbo no
particípio.
GABARITO: A

Questão 06 (AFA/2016) – Mark the option which shows the appropriate question tag for the
sentence “one unanticipated consequence has driven me to distraction”.
a) Hasn’t driven it?
b) Not has it?
c) Has it?
d) Hasn’t it?
Comentários: É importante entender que as “question tags” tem uma estrutura específica. Elas
servem para o interlocutor pedir uma confirmação da ideia apresentada. Essa confirmação precisa
ser dada utilizando a mesma estrutura apresentada na afirmação.
Ex.: you are a doctor, aren’t you? (você é um médico, não é?)
A alternativa A está incorreta. O verbo principal “driven” não deveria estar dentro da estrutura da
“question tag”.
A alternativa B está incorreta. A palavra “not” não pode aparecer antes de “has” na estrutura de
uma pergunta. Isso torna a estrutura incorreta.
A alternativa C está incorreta. A “question tag” deve aparecer sempre negando a afirmação. Se a
frase é afirmativa, a “question tag” deve vir na negativa. Já quando a frase é negativa, a “question
tag” deve vir na afirmativa.
A alternativa D está correta. “Hasn’t it” é a “question tag” correta para acompanhar a frase do
enunciado. Podemos substituir o sujeito da oração “one unanticipated consequence” por “it” e,
então, teríamos a frase: It has driven me to distraction. A “question tag” apropriada para esta
frase seria: It has driven me to distraction, hasn’t it?.
GABARITO: D

AULA 00 – ENGLISH GENERAL PRESENTATION 57


TEACHER ANDREA BELO

Questão 07 (AFA/2016) – Mark the option that shows synonyms for the underlined expressions
in “it’s high time to relieve my pain and to set the record straight”.
a) An important brake / to register.
b) An ordinary condition / to show the discussion.
c) A belated explanation / to make myself clearly understood.
d) An unusual hour / to comprehend an argument.
Comentários: A alternativa A está incorreta. O trecho não fala freio(brake) em momento algum.
Nesse caso, “record” não tem a ver com “register” (registrar), e sim com esclarecer, porque
“record” não tem significado isolado na frase. A significação vem de toda a expressão sublinhada.
A alternativa B está incorreta. “high time” não significa condição extraordinária, mas sim uma ideia
oposta a isso. Na segunda parte, a ideia não é mostrar(show) a discussão, mas sim esclarecer
ideias.
A alternativa C está correta. “Belated” é uma palavra que é utilizada para dizer que algo já
passou da hora de ser feito. Isso confere o mesmo significado dado à expressão “high time”. Na
segunda parte, fazer-se entender de forma clara é exatamente a ideia da expressão “to set the
record straight”.
A alternativa D está incorreta. “Uma hora incomum” não define bem a expressão “high time” e,
por isso, não pode ser considerada sinônima desta. Na segunda parte, temos uma definição
parecida, mas ainda diferente. Compreender um argumento é diferente de se fazer
compreendido.
GABARITO: C

Questão 08 (AFA/2016) – Choose the best option to change the sentence “human capacities are
represented in the brain” (4th paragraph), into the active form.
The brain _______________ human capacities.
a) has represented
b) represents
c) has been represented
d) representing
Comentários: O enunciado pede que transformemos uma frase da voz passiva para a voz ativa. A
frase no enunciado diz: As capacidades humanas são representadas no cérebro. Devemos, então,
dizer: O cérebro representa as capacidades humanas.
A alternativa A está incorreta. “Has represented” daria a ideia de “tem representado”, e não de
“representa”. O problema está, portanto, no tempo verbal empregado.
A alternativa B está correta. “Represents” dá a ideia de “representa”, que é exatamente o tempo
verbal adequado para transformar esta frase na voz ativa. O “S” adicionado ao final do verbo é
necessário, pois trata-se de um verbo conjugado na terceira pessoa do singular (IT) do “Simple
Present”.

AULA 00 – ENGLISH GENERAL PRESENTATION 58


TEACHER ANDREA BELO

A alternativa C está incorreta. “Has been represented” tem a ideia de “tem sido representado”, e
não de “representa”. O erro está no tempo verbal empregado.
A alternativa D está incorreta. “Representing” tem a ideia de “representando”, e não de
“representa”. O tempo verbal é inadequado.
GABARITO: B

Questão 09 (AFA/2016) – In the fourth paragraph, it’s said that


a) a single intelligence (SI) concept leads to the assumption of computers which control 7 to 10
distinct intelligences.
b) MI theory believes that instead of a central computer mastering various sectors, there are a
larger amount of them relatively autonomous.
c) MI theory estimates the existence of a central computer responsible for 7 to 10 distinct
intelligences.
d) a SI determines people’s performance in different sectors of life through autonomous
computers.
Comentários: A alternativa A está incorreta. O texto diz que o conceito de inteligência múltipla diz
que humanos tem de 7 a 10 inteligências distintas, não o conceito de inteligência única.
A alternativa B está correta. É exatamente isso que diz o texto, o cérebro não funciona como um
único computador que comanda todas as nossas aptidões e habilidades, mas sim como vários
deles relativamente autônomos comandando as diferentes habilidades que possuímos.
A alternativa C está incorreta. Não há um computador único que comanda todas as inteligências.
De acordo com o parágrafo, essas inteligências são distintas e relativamente independentes.
A alternativa D está incorreta. A alternativa define o conceito de múltiplas inteligências, mas
atribui essa definição ao conceito de inteligência única. Este é, portanto, o equívoco da alternativa.
GABARITO: B

Questão 10 (AFA/2016) – Mark the alternative in which the problems described in paragraphs 6
and 7 are correctly summarized.
a) The idea of teaching distinct leaning styles and their consistence were questionable concepts
when researches started.
b) Educational researchers have found that an impulsive learning style causes problems in its
outcomes.
c) There are proofs that different learning styles exist and produce positive results.
d) The notion of learning styles and the outcomes observed when teaching based on them need
further studies.
Comentários: A alternativa A está incorreta. O conceito de ensino distinto de acordo com o estilo
de aprendizagem e sua consistência serem questionáveis não são os problemas principais listados
nos parágrafos.

AULA 00 – ENGLISH GENERAL PRESENTATION 59


TEACHER ANDREA BELO

A alternativa B está incorreta. Na verdade, os pesquisadores não têm respostas quanto ao modo
adequado de ensinar a cada estilo de aprendizagem. Também não se fala nada sobre problemas
causados pelo estilo de aprendizagem impulsivo.
A alternativa C está incorreta. Os parágrafos falam exatamente o contrário. O texto diz que não há
evidencias de que os estilos de aprendizagem existam, nem que produzem bons resultados.
A alternativa D está correta. O texto diz que os testes já feitos com ensino diferenciado para os
diferentes tipos de aprendizagem não apresentaram evidências de serem mais eficientes, porém
isso não invalida completamente a hipótese. Outros estudos e pesquisas precisam ser feitos
para que se chegue a uma conclusão.
GABARITO: D

QUESTÕES COLÉGIO NAVAL


Read text to do questions 01 to 04 based on it.
Text I
Social media ’destroying how society works;
A former Facebook executive has said social media is doing great harm to society around the
world. The executive is a man called Chamath Palihapitiya. He ___________ Facebook in 2007
and ___________a vice president. He was responsible for increasing the number of users
Facebook had. Mr Palihapitiya said he feels very guilty about getting more people to use social
networks. He said the networks are destroying society because they are changing people's
behavior. Twenty years ago, people talked to each other face to face. Today, people message each
other and do not talk. People also really care about what other people think of them. They post
photos and wait to see how many people like the photo. They get very sad if people do not like
the photo.
Mr. Palihapitiya said people should take a long break from social media so they can experience
real life. He wants people to value each other instead of valuing online "hearts, likes, and thumbs-
up". Palihapitiya also points out how fake news is affecting how we see the world, it is becoming
easier for large websites to spread lies. It is also becoming easier to hurt other people online.
Anyone can hide behind a fake user name and post lies about other people. Palihapitiya said this
was a global problem. He is worried about social media so much that he has banned his children
from using it. However, he did state that Facebook was a good company. He said: "Of course,
it's not all bad. Facebook overwhelmingly does good in the world."

Questão 01 (COLÉGIO NAVAL /2018) – Read the statements to check if they are TRUE (T) or
FALSE (F).
I- An ex-Facebook boss said social media is damaging society.
II- It is becoming more difficult for big websites to spread fake news.
III- People message each other today instead of talking face to face.
IV- Palihapitiya said social media does not change our behavior.

AULA 00 – ENGLISH GENERAL PRESENTATION 60


TEACHER ANDREA BELO

Choose the option that respectively represents the statements above.


a) F/T/T/F
b) F/F/T/T
c) T/F/T/F
d) T/T/F/T
e) F/F/F/T
Comentários: A afirmativa I é verdadeira. O texto diz que ele chegou ao cargo de vice-presidente
da empresa e ele realmente diz que mídias sociais causam danos à sociedade. “A former Facebook
executive has said social media is doing great harm to society around the world”.
A afirmativa II é falsa. O texto diz que está ficando cada vez mais fácil, para os grandes sites,
difundir as fake news. “it is becoming easier for large websites to spread lies”.
A afirmativa III é verdadeira. O texto diz que 20 anos atrás, as pessoas conversavam pessoalmente,
mas que hoje as pessoas usam mensagens para se comunicar. “Twenty years ago, people talked
to each other face to face. Today, people message each other and do not talk”.
A afirmativa IV é falsa. O texto diz que as redes estão destruindo a sociedade porque alteram o
comportamento das pessoas. “He said the networks are destroying society because they are
changing people's behavior”.
GABARITO: C

Questão 02 (COLÉGIO NAVAL /2018) – All the underlined words in text I are adjectives, EXCEPT:
(A) social.
(B) global.
(C) long.
(D) executive.
(E) former.
Comentários: A alternativa A está errada. “Social” funciona como adjetivo caracterizando o termo
“media”.
A alternativa B está errada. “Global” funciona como adjetivo caracterizando o termo “problem”.
A alternativa C está errada. “Long” funciona como adjetivo caracterizando o termo “break”.
A alternativa D está correta. “Executive” funciona como substantivo no trecho (o executivo).
A alternativa E está errada. “Former” funciona como adjetivo caracterizando o termo “executive”.
GABARITO: D

AULA 00 – ENGLISH GENERAL PRESENTATION 61


TEACHER ANDREA BELO

Questão 03 (COLÉGIO NAVAL /2018) – Which verb forms respectively complete the gaps in text I?
(A) joined/become
(B) joined/became
(C) joins/becomes
(D) joint/became
(E) was joined/ become
Comentários: The executive is a man called Chamath Palihapitiya. He _____joined______
Facebook in 2007 and _____became______a vice president.
O texto passa informações que ocorreram no passado (em 2007). Portanto, as palavras que
preencher as lacunas precisam, necessariamente, estar no passado para que o trecho tenha
sentido e correção gramatical. A questão nos apresenta os verbos “to join” e “to become”,
respectivamente, para completar as lacunas. Para resolver a questão, precisamos flexionar ambos
os verbos para o tempo passado (simple past). Temos “joined” como passado de “to join”, e
“became” como passado de “become”. Temos, então, as palavras “joined” e “became” como
respostas corretas para preencher as lacunas.
A alternativa B está correta.
GABARITO: B

Questão 04 (COLÉGIO NAVAL /2018) – Mark the option in which there is NO Present
Continuous Tense.
(A) A former Facebook executive has said social media is doing great harm to society around the
world.
(B) He was responsible for increasing the number of users Facebookliad.
(C) He said the networks are destroying society because they are changing people's
behavior.
(D) Palihapitiya also points out how fake news is affecting how we see the world.
(E) It is becoming easier for large websites to spread lies.
Comentários: A alternativa A está incorreta. Há o “Present Continuous Tense” na sentença. “is
doing”.
A alternativa B está correta. “increasing” não funciona como uma ação em andamento neste caso.
A preposição “for” exige que o verbo que a acompanha flexione para o gerúndio. No entanto, o
termo funciona como infinitivo na frase.
A alternativa C está incorreta. Há o “Present Continuous Tense” na sentença. “are destroying”.
A alternativa D está incorreta. Há o “Present Continuous Tense” na sentença. “is affecting”.
A alternativa E está incorreta. Há o “Present Continuous Tense” na sentença. “is becoming”.
GABARITO: B

AULA 00 – ENGLISH GENERAL PRESENTATION 62


TEACHER ANDREA BELO

Read text II to do questions 05 to 10 based on it.


TEXT II
TRAVEL TIPS
How to Plan a Movie-Themed Vacation
It’s easier than you may expect to find, visit, and enjoy the places where your favorite movies
were made.
Lars Leetaru
By Shivani Vora
March 8, 2018
Whether it’s the “Lord of the Rings” trilogy in New Zealand or "Roman Holiday” in Rome, many
noteworthy movies are filmed in appealing locales all over the world that travelers may want to
visit and enjoy.
According to Angela Tillson, a film location manager in Kauai who has worked on the set of films
including "Jurassic Park: The Lost World” and “The Descendants," exploring a beloved movie set
destination through the eyes of the film makes for an enjoyable vacation. "Seeing a place with a
focus on a movie you love will give you a perspective that the average tourist doesn’t usually get.
You’ll certainly have a better impression of the place,” she said. Here are her tips to get started.
Choose Your Destination
If there’s a movie you love, you can find out where it was filmed by looking at the credits at the
end of the film or by going online to The Internet Movie Database, also known as IMDB, which
often lists filming locations. Once you know the locale, you can start planning your trip. Or,
consider doing what Ms. Tillson often does when deciding on where to vacation: pick a spot you’re
interested in visiting, and find out what movies have been filmed there. “It’s fun to sometimes let
a destination determine the movie you're going to live rather than the other way around,”
Ms. Tillson said.
Get in the Mood
Before you head to your destination, be sure to rewatch the movie. A rewatch not only reminds
you of identifiable spots to look out for during your trip, but it also adds to the excitement of your
upcoming exploration.
If the movie is based on a book, consider reading the book, too. It may have details about the
locale that the movie doesn’t touch on. Also, books often have scenes that don’t make it into the
movie adaptations, which gives you a deeper view of the destination.
Ms. Tillson also recommended downloading the movie’s soundtrack or score, and listening to it
throughout your trip.
Book a Themed Trip
Some travel companies sell set itineraries focused on popular movies. Luxury tour operator
Zicasso, for example, has an eight-day trip, all inclusive, to Ireland inspired by "Star Wars: The Last
Jedi” and Wild Frontiers has an eleven-day trip to India inspired by "The Best Exotic Marigold

AULA 00 – ENGLISH GENERAL PRESENTATION 63


TEACHER ANDREA BELO

Hotel." Ms. Tillson suggested doing a web search or checking with a travel agent to find out about
such trips.
Also, in some destinations, local tour operators and hotels sell movie-themed tours. For instance,
The St. Regis Priceville Resort offers a tour that includes a private helicopter ride to
Manawaiopuna Falls, made famous in "Jurassic Park,” and an ATV tour of filming locations of
movies such as “Raiders of the Lost Ark" and “Pirates of the Caribbean.” Lunch is even included.
The cost is $5,674 for two adults.
A more affordable option, in Rome, is the four-hour “Roman Holiday" themed excursion from HR
Tours, where travelers ride a Vespa with a driver and see all the sites from the movie; the cost is
170 euros per person.
Hang Where the Movie Crew Did
When they’re not working, movie crews enjoy hitting local bars and casual restaurants that serve
tasty local cuisine, Ms. Tillson said.
Find out where the behind-the-scenes staff of your film spent their time by asking your
destination’s tourist board or your hotel’s concierge, and check out a few of the spots. “It’s
another way to get involved in the film and spend time in bars and restaurants that you wouldn’t
normally think to hit,” she said.

Questão 05 (COLÉGIO NAVAL /2018) – Mark the sentence that does NOT contain the use of
comparative adjective.
(A) It's easier than you may expect to find, visit, and enjoy the places where your favorite
movies were made.
(B) You’ll certainly have a better impression of the place.
(C) It’s fun to sometimes let a destination determine the movie you’re going to live rather than
the other way around.
(D) Also, books often have scenes that don’t make it into the movie adaptations, which gives you
a deeper view of the destination.
(E) A more affordable option, in Rome, is the four-hour Roman Holiday themed excursion from
HR Tours.
Comentários: A alternativa A está incorreta. “Easier” é um adjetivo de comparação que significa
“mais fácil”.
A alternativa B está incorreta. “Better” é um adjetivo de comparação que quer dizer “melhor”.
A alternativa C está correta. “Rather” é a palavra que o examinador quer que você pense que é
um adjetivo de comparação por conta de sua terminação em “ER”. Mas “rather” ´significa, na
verdade, “ao invés de”, sendo sinônimo de “instead”.
A alternativa D está incorreta. “Deeper” é um adjetivo de comparação que significa “mais
profundo(a)”.

AULA 00 – ENGLISH GENERAL PRESENTATION 64


TEACHER ANDREA BELO

A alternativa E está incorreta. “More affordable” é um adjetivo de comparação que significa “mais
acessível (financeiramente)”. Esta alternativa poderia gerar dúvida por ser a única que não possue
a terminação em “ER”. É importante ficar atento a isso na hora de resolver sua prova.
GABARITO: C

Questão 06 (COLÉGIO NAVAL /2018) – What is true about the themed trip inspired by the film
“Star Wars: The Last Jedi"?
(A) It offers no meals.
(B) It lasts 11 days.
(C) It’s very cheap.
(D) It’s located in India.
(E) It’s offered by Zicasso.
Comentários: Some travel companies sell set itineraries focused on popular movies. Luxury tour
operator Zicasso, for example, has an eight-day trip, all inclusive, to Ireland inspired by "Star
Wars: The Last Jedi” and Wild Frontiers has an eleven-day trip to India inspired by "The Best
Exotic Marigold Hotel." Ms. Tillson suggested doing a web search or checking with a travel agent
to find out about such trips.
A alternativa A está incorreta. O texto diz que a viagem tem tudo incluso “all inclusive”. Portanto,
é incorreto dizer que a excursão não oferece refeições.
A alternativa B está incorreta. O texto diz que a viagem dura 8 dias “eight-day trip”.
A alternativa C está incorreta. O texto diz que uma agência de turismo de luxo “luxury tour
operator” organiza os passeios. Não é possível, portanto, dizer que se trata de um passeio muito
barato “very cheap”.
A alternativa D está incorreta. O texto diz que o passeio ocorre na Irlanda “Ireland”, e não na Índia.
A alternativa E está correta. O texto diz que a operadora de turismo de luxo Zicasso oferece o
passeio.
GABARITO: E

Questão 07 (COLÉGIO NAVAL /2018) – In the first paragraph, the word "appealing” can be
replaced by all these words, EXCEPT for
(A) interesting.
(B) pleasing.
(C) lovable.
(D) repulsive.
(E) attractive.
Comentários: A alternativa A está incorreta. “Interesting” é um sinônimo de “appealing”.

AULA 00 – ENGLISH GENERAL PRESENTATION 65


TEACHER ANDREA BELO

A alternativa B está incorreta. “Pleasing” pode substituir “appealing” sem prejuízo de significação
do texto.
A alternativa C está incorreta. “Lovable” pode substituir “appealing” nesse contexto sem prejuízo
de sentido.
A alternativa D está correta. “Repulsive” significa repulsivo. Isso é o oposto da ideia transmitida
pela palavra “interesting” que significa interessante.
A alternativa E está incorreta. “Atractive” transmite uma ideia similar a “interesting”.
GABARITO: D

Questão 08 (COLÉGIO NAVAL /2018) – Mark the correct question for the following answer.
Angela Tillson is a film location manager in Kauai who has worked on the set of films including
“Jurassic Park: The Lost World” and “The Descendants”.
(A) Who is Ms. Tilson?
(B) Where is Ms. Tilson?
(C) What is Ms. Tilson like?
(D) Why is Ms. Tilson working?
(E) When does Ms. Tilson work?
Comentários: A alternativa A está correta. O trecho descreve Angela Tillson e, a letra A pergunta
exatamente quem é Ms. Tillson. Portanto, essa é a melhor pergunta para a resposta oferecida no
enunciado.
A alternativa B está incorreta. “Where” é usado para perguntar onde alguma coisa ou alguém
está. A resposta oferecida no enunciado não se encaixa como resposta dessa pergunta.
A alternativa C está incorreta. Esta pergunta é feita quando se quer saber como alguém é. O
trecho não diz como Angela é, mas sim quem ela é.
A alternativa D está incorreta. O trecho diz quem é Angela Tillson, e a alternativa pergunta por
que ela trabalha. Não há nenhuma relação entre a pergunta e a resposta.
A alternativa E está incorreta. A alternativa pergunta quando a Ms. Tillson trabalha, mas o
enunciado diz quem é Angela Tillson. Não há, portanto, relação entre o que é perguntado e o que
é respondido.
GABARITO: A

Questão 09 (COLÉGIO NAVAL /2018) – What’s the main purpose of text II?
(A) Teach students how to plan where to go on vacation.
(B) inspire the fans of “Lord of the Rings” to visit New Zealand.
(C) Make people feel interested in watching famous movies.
(D) Give some suggestions on traveling to a movie set destination.
(E) Advertise travel companies that sell movie-themed vacations.

AULA 00 – ENGLISH GENERAL PRESENTATION 66


TEACHER ANDREA BELO

Comentários: A alternativa A está incorreta. O texto não tem a intenção de ensinar a estudantes
como planejar suas viagens de férias. É um texto aberto a outros nichos que visa ajudar todo tipo
de pessoas com ideias criativas de viagens.
A alternativa B está incorreta. O texto não é direcionado apenas amantes de “Senhor dos anéis”,
mas sim a todos os amantes de filmes e viagens interessantes.
A alternativa C está incorreta. Não é a intenção do texto que as pessoas se interessem por filmes
famosos. O direcionamento dado pelo texto serve para fimes famosos ou não.
A alternativa D está correta. O texto sugere um tipo de viagem ainda pouco explorado, A viagem
a locais onde fimes foram gravados e ter uma perspectiva diferente do local.
A alternativa E está incorreta. A ideia não é fazer propaganda de empresa de turismo. Até porque
o texto cita mais de uma.
GABARITO: D

Questão 10 (COLÉGIO NAVAL /2018) – Read the sentence in text II.


"A rewatch not only reminds you of identifiable spots to look out for during your trip, but it also
adds to the excitement of your upcoming exploration."
What does the pronoun it refer to?
(A) Trip.
(B) Spots.
(C) Excitement.
(D) Exploration.
(E) Rewatch.
Comentários: A alternativa A está incorreta. “Trip” poderia ser substituído no trecho por
“upcoming exploration”, mas não por “it”.
A alternativa B está incorreta. Quem se refere a “spot” no trecho é “identifiable”, e não “it”.
A alternativa C está incorreta. O texto diz que “excitement” é adicionado por “it”. Portanto, “it”
não se refere a “excitement”.
A alternativa D está incorreta. “Exploration” é referência a “trip”, não a “it”.
A alternativa E está correta. “Rewatch” é exatamente o termo retomado por “it” no trecho. O
texto diz que “rewatch” (reassistir) não apenas te lembra dos pontos identificáveis durante a
viagem, mas também adiciona empolgação à viagem que está por vir.
GABARITO: E

AULA 00 – ENGLISH GENERAL PRESENTATION 67


TEACHER ANDREA BELO

QUESTÕES EEAR
Text to answer questions 01 and 02
Paul Thompson is a student and he has a part-time job, too. He lives in London and goes to
university there. Paul works at an Italian restaurant called Mamma Mia. He’s a waiter there. He
likes his job very much but he doesn’t like to work on the weekends. When he is free, he likes to
go to the movies. He recently watched “Avengers – Infinity War”, a superhero movie that was a
huge success.

Questão 01 (EEAR/2021 – CFS) – All the words below are synonyms for the word “huge” (line
08), in bold in the text, EXCEPT:
A) insignificant
B) tremendous
C) massive
D) great
Comentários: A alternativa A está correta. HUGE é enorme e insignificante é o cognato
insignificante. Portanto, não se assemelha à palavra grande, grandioso, como as letras B
(tremeduous) tremendo, letra C (massive) mássico, de impact e letra D (great) grande, grandioso.
GABARITO: A

Questão 02 (EEAR/2021 – CFS) – According to the text, how long does Paul work?
(A) Part-time.
(B) All day long.
(C) He doesn’t work.
(D) Every other week.
Comentários: A alternativa A está correta. Part-time é o período que se fala no texto sobre o
trabalho dele: “Paul Thompson is a student and he has a part-time job, too” (Paul é um aluno e
tem um emprego de meio-período, também).
Portanto, as outra alternativas estão incorretas: ele não trabalha o dia inteiro, como na letra B (all
day long) nem semana sim, semana não, como dito na letra D (every other week) e nem tampouco
podemos afirmar que ele não trabalhar como diz a letra C (He doesn’t work).
GABARITO: A

Text to answer question 03


“The Little Prince”, now, is a movie – Alex Weiss
An all-time favorite children’s book, The Little Prince by Antoine de Saint-Exupéry has been turned
into a beautifully animated movie – and it’s finally being released. The inspiring lessons, timeless
story and beautiful quotes from The Little Prince make this a perfect choice for an on-screen

AULA 00 – ENGLISH GENERAL PRESENTATION 68


TEACHER ANDREA BELO

adaptation. At one point to another, your parents read this book to you and when you were a
child and then you picked it up later on in life, realizing how incredibly important this small book
truly is.
(Adapted from bustle.com)

Questão 03 (EEAR/2021 – CFS) – The verbs in bold are, respectively, in the:


(A) present perfect, simple past and simple past.
(B) simple past, simple past and present perfect.
(C) present perfect, simple past and present perfect.
(D) simple past, present perfect and present perfect.
Comentários: A alternativa A está correta. O primeiro verbo “has been turned”, com a presença
do auxiliar has e verbo no particípio já indica o tempo present perfect.
Assim, já eliminamos as alternativas B e D, que iniciam com simple past.
Daí, o próximo verbo, read no passado, poderia te confundir pois se escreve igual ao presente,
mas tem a presença de “when you were a child” (quando você era criança), indicando que está
no tempo passado.
O último verbo, “picked”, é um verbo apenas na frase portanto não poderia a letra C, que diz ser
present perfect, que precisaria de dois verbos (auxiliar have/has e outro verbo n particípio)
GABARITO: A

Text to answer questions 04 and 05


Price Tag
Price Tag – Jessie J.
Seems like everybody's got a price
I wonder how they sleep at night
When the sale comes first
And the truth comes second
Just stop for a minute and smile
Why is everybody so serious
Acting so damn mysterious
Got shades on your eyes
And your heels so high
That you can't even have a good time
Everybody look to the left
Everybody look to the right
Can you feel that yeah
We're paying with love tonight
It's not about the money money money

AULA 00 – ENGLISH GENERAL PRESENTATION 69


TEACHER ANDREA BELO

We don't need your money money money


We just wanna make the world dance
Forget about the price tag
Ain't about the uh cha-ching cha-ching
Ain't about the yeah b-bling b-bling
Wanna make the world dance
Forget about the price tag
(letras.mus.br)

Questão 04 (EEAR/2021 – CFS) – The verb WONDER, in bold in the text, can be replaced by
_______ without changing the meaning:
(A) know.
(B) can know.
(C) must know.
(D) want to know.
Comentários: A alternativa D está correta. O primeiro verbo “WONDER” significa querer saber,
almejar saber algo. Assim, já eliminamos as alternativas A, B e C, que são os verbos saber (know),
pode saber (can know) e deve saber (must know).
GABARITO: D

Questão 05 (EEAR/2021 – CFS) – The underlined words, in the text, are:


(A) adjectives.
(B) adverbs.
(C) nouns.
(D) verbs.
Comentários: A alternativa A está correta. As palavras serious, mysterious e good são três
adjetivos, que qualificam substantivos.
Assim, já eliminamos as alternativas B, C e D, que diz serem advérbios (adverbs), o que não faz
sentido. Ou substantivos (nouns), que não condiz ou verbos (verbs), que seriam ações e não
qualidades.
GABARITO: D

Questão 06 (EEAR/2021 – CFS) – Change the sentence into the interrogative form.
“The cost of living has doubled in relative terms over recent years”
(A) Has the cost of living double in relative terms over recent years?
(B) Has the cost of living doubled in relative terms over recent years?
(C) Have the cost of living double in relative terms over recent years?
(D) Have the cost of living doubled in relative terms over recent years?

AULA 00 – ENGLISH GENERAL PRESENTATION 70


TEACHER ANDREA BELO

Comentários: A alternativa B está correta. O uso de HAS na frase afirmativa pede Has na pergunta
também. Assim, eliminamos as letras C e D.
Nas perguntas usadas no tempo Present Perfect, há o uso do verbo auxiliar (has ou have) e outro
verbo no particípio (doubled). Portanto, eliminamos a letra A, que o verbo está “normal”, sem
particípio e sim no infinitivo.
GABARITO: B

Read the text and answer questions 07 and 08.


In The End – Linkin Park
One thing, I don't know why
It doesn't even matter how hard you try
Keep that in mind
I designed this rhyme
To remind myself of a time when
I tried so hard
In spite of the way you were mocking me
Acting like I was part of your property
Remembering all the times you fought with me
I'm surprised it got so
Things aren't the way they were before
You wouldn't even recognize me anymore
Not that you knew me back then
But it all comes back to me in the end
You kept everything inside
And even though I tried, it all fell apart
What it meant to me will eventually be a memory of a time when
I tried so hard
And got so far
But in the end
It doesn't even matter
I had to fall
To lose it all
But in the end
It doesn't even matter
I've put my trust in you
Pushed as far as I can go
For all this
There's only one thing you should know
I've put my trust in you

AULA 00 – ENGLISH GENERAL PRESENTATION 71


TEACHER ANDREA BELO

Pushed as far as I can go


For all this
There's only one thing you should know
https://www.vagalume.com.br/linkin-park/in-the-end.html

Questão 07 (EEAR/2021 – CFS) – The words in bold are, respectively, synonyms for
A) Despite, although, and in the end.
B) Although, in the end, and finally.
C) Despite, though, and although.
D) Although, finally, and despite.
Comentários: No enunciado, pede as palavras que são sinônimos de In spite of, even though e
eventually. Na letra A, encontramos a palavra despite, que é um sinônimo de in spite of e
representa a mesma ideia. Portanto, ficamos entre a letra A e a letra C eliminando assim as
alternativas B e D. Continuando nossa análise, o termo even though só pode ser comparado com
although que está na letra A pois, na letra C há though que não completa a ideia.
A palavra eventually tem o sentido de “in the end”. Assim nós confirmamos que o nosso gabarito
é a letra A.
GABARITO: A

Questão 08 (EEAR/2021 – CFS) – The negative form of the underline sentence, in the text, is
A) I didn’t put my trust in you.
B) I don’t put my trust in you.
C) I do not put my trust in you.
D) I haven’t put my trust in you.
Comentários:
A frase que está sublinhada no texto é I've put my trust in you, ou seja, uma estrutura de
Present Perfect, usando o verbo “have” abreviado (I've) e o segundo verbo no particípio (put).
Isso significa que o nosso gabarito só pode ser a letra D pois, as alternativas A, B e C não contém
o auxiliar “have” necessário na estrutura Perfect portanto, nosso gabarito é a letra D.
GABARITO: D

Read the text and answer questions 09 and 10.

www.garfiel.com

AULA 00 – ENGLISH GENERAL PRESENTATION 72


TEACHER ANDREA BELO

Questão 09 (EEAR/2021 – CFS) – According to the text, Garfield:


A) Thinks all days are beautiful.
B) Prefers the kitchen only when it’s raining.
C) Prefers staying outside if it’s a beautiful day.
D) Thinks days are beautiful in the kitchen even when it’s raining.
Comentários: A pergunta é o que podemos retirar de ideia de acordo com o texto.
Na letra A diz que Garfield pensa que todos os dias são bonitos, porém, na tirinha, ele diz que é
bonito na cozinha (in the kitchen).
Na letra B diz que ele prefere a cozinha apenas quando está chovendo e isso não é verdade. Ele
diz que ele prefere a cozinha, representado no segundo e terceiro quadrinho.
Na letra C diz que Garfield prefere ficar de fora da casa e percebemos que ele prefere ficar na
cozinha.
A letra D é o nosso gabarito. Ele acha que os dias são bonitos na cozinha mesmo se estiver
chovendo.
GABARITO: D

Questão 10 (EEAR/2021 – CFS) – The sentence “Now it’s raining” is in the:


A) Simple Past
B) Simple Present
C) Past Progressive
D) Present Progressive
Comentários: A frase “Now it’s raining” está claramente conjugada no Present Continuous ou
também chamado de Present Progressive como diz a letra D, porque há o verb to be ‘is’ e o
próximo verbo principal ‘rain’ com ‘ing’. As alternativas A, B e C dizem tempos verbais que não
estão de acordo.
GABARITO: D
QUESTÕES EAM
Questão 01 (EAM/INÉDITA) – Read the dialogue and mark the right option to fill in the gaps
respectively.
A: Good afternoon, Jack!
B: Hey, Jen! How are you doing?
A: I’m alright. What about yourself?
B: I’m doing well.
A: ___ you at Johnnie’s house last weekend?
B: Yes, I ____.
A: Who ____ you go with?
B: I ____ with my wife. What about you? Were you there?
A: No, I ___. I ____ home.

AULA 00 – ENGLISH GENERAL PRESENTATION 73


TEACHER ANDREA BELO

A) Did / sis / went / was / weren’t / was


B) Were / was / were / go / didn’t / went
C) Was / was / did / was / weren’t / was
D) Were / was / did / went / wasn’t / stayed
E) Did / did / did / went / didn’t / went
Comentários: A: Good afternoon, Jack!
B: Hey, Jen! How are you doing?
A: I’m alright. What about yourself?
B: I’m doing well.
A: __Were___ you at Johnnie’s house last weekend?
B: Yes, I __was__.
A: Who __did__ you go with?
B: I __went__ with my wife. What about you? Were you there?
A: No, I __wasn’t_. I _stayed_ home.
Na primeira lacuna, precisamos usar uma palavra no passado. A expressão last weekend nos dá
essa certeza. A única palavra possível é “were”, pois a pergunta tem como objetivo saber se Jack
foi à casa de Johnnie no fim de semana anterior.
Na segunda lacuna, não poderíamos usar “were” por causa do sujeito “I”, que exige “was”.
Na terceira lacuna, como já sabemos que o diálogo está se desenrolando no passado e, além disso,
podemos ver o verbo “go” no infinitivo. Esses são os indicativos de que “did” é a palavra que
completa a lacuna.
Na quarta lacuna, já sabemos que precisamos nos manter no tempo passado. Por isso, “went” é
a palavra que completa essa lacuna, já que “went” é o passado do verbo “go”.
Na quinta lacuna, sabemos que precisamos manter o diálogo no passado e, que precisamos
completar a lacuna com uma “short answer” negativa porque a frase começa com “no”. Portanto,
“wasn’t” é a palavra a ser encaixada na lacuna, tendo em vista que a pergunta foi feita com a
palavra “were” e, a reposta é dada com “I”.
Na sexta lacuna, sabemos que precisamos de um verbo conjugado no passado, mas essa lacuna
nem seria necessária para resolver a questão, até porque outros verbos conjugados no “simple
past” poderiam se encaixar de forma adequada nessa lacuna.
Temos, portanto, a seguinte sequência: were / was / did / went / wasn’t / stayed.
GABARITO: D

AULA 00 – ENGLISH GENERAL PRESENTATION 74


TEACHER ANDREA BELO

Questão 02 (EAM/INÉDITA) – Read the sentences and mark the correct option to fill in the
blanks respectively.
Elisa is __ friend. ___ lives near my grandfather’s house. We love riding our bikes. ___ bike is big.
___ is small. We love spending time together!
A) My / she / my / hers
B) Your / I / my / her
C) Her / he / her / mine
D) My / her / my / she
E) His / he / my / his
Comentários: Elisa is my friend. She lives near my grandfather’s house. We love riding our bikes.
My bike is big. Hers is small. We love spending time together!
Essa é uma questão interessante, pois não há uma única resposta para cada lacuna. Precisamos
resolver a questão pensando no contexto como um todo. Podemos depreender do trecho que há
duas amigas ou dois amigos que gostam de andar de bicicleta. A bicicleta de uma é grande e a
do(a) outro(a) é pequena. Elas adoram passar tempo juntas.
A primeira lacuna é preenchida adequadamente por “my”, pois percebe-se que a narrador(a) fala
de sua relação com Elisa.
A segunda lacuna é mais fácil, pois precisamos de um “subjective pronoun” que se refira a Elisa
para preenchê-la. “She” é, portanto, a única opção aplicável aqui.
Na terceira lacuna, mais uma vez o contexto geral do trecho é o que nos indica o pronome correto
a ser utilizado. Como é perceptível que o trecho fala da relação entre amigos(as), sendo um deles
a Elisa e o(a) narrador(a) o segundo elemento da relação, o melhor pronome para preencher a
lacuna é “my”.
Na quarta lacuna, após utilizar “my” na lacuna anterior, temos apenas “hers” como opção, já que
já falei da minha bicicleta, agora preciso falar da dela.
Ao analisar as possibilidades para as lacunas, percebemos que elas são várias. É importante,
porém, perceber que a segunda lacuna era a única que não dava possibilidade de nenhum outro
pronome além de “she”, porque tinha de funcionar como sujeito da frase e teria de referi-se a
Elisa. Ao analisarmos as alternativas, vemos que apenas a letra A nos dava o pronome “she” como
opção para a segunda lacuna. Dessa forma, poderíamos resolver a questão ao resolver a segunda
lacuna.
Fica o BIZU para você, meu querido aluno. Preste atenção nas alternativas, pois elas podem lhe
ajudar a resolver uma questão complexa, de maneira simples.
Temos a sequência “my / she / my / hers”
GABARITO: A

AULA 00 – ENGLISH GENERAL PRESENTATION 75


TEACHER ANDREA BELO

Questão 03 (EAM/INÉDITA) – Which word is incorrectly used in this meme?

A) Failed
B) Unpossible
C) My
D) In
E) Test
Comentários: A alternativa A está correta. “Failed” é usado, nesse contexto, para dizer que falhou
ou fracassou no teste. A palavra está sendo empregada corretamente na frase.
A alternativa B está incorreta. O erro está em usar o prefixo “un” ao invés de “im” para dizer
impossível. A forma correta de escrever a palavra seria “impossible”.
A alternativa C está correta. “My” refere-se a “english test” e está sendo empregado de forma
correta.
A alternativa D está correta. “In” está sendo usado de forma correta. “In” é a preposição adequada
para “english test”.
A alternativa E está correta. “Test” está sendo utilizado corretamente, com “english” o
antecedendo.
GABARITO: B

Questão 04 (EAM/INÉDITA) – Use the verbs in the parentheses to complete the following
statements.
I – I usually __ (do) my homework after dinner, but now I __ (do) a diferent task.
II – Eli ___ (eat) a lot. Sometimes, he ___ (want) to eat less
III – Alessandra ___(live) in Boston, United States. During the week, her daughters ___ (work) as
delivery girls for a big tech company.

AULA 00 – ENGLISH GENERAL PRESENTATION 76


TEACHER ANDREA BELO

Now mark the option which completes them respectively.


A) Am doing / am doing / is eating / wants / lives / working
B) Am doing / do / eats / is wanting / is living / works
C) Am doing / do / eat / is wanting / is living / works
D) Do / do / is eating / want / live / is working
E) Do / am doing / eats / wants / lives / work
Comentários: I – I usually _do_ (do) my homework after dinner, but now I am doing__ (do) a
diferent task.
II – Eli eats_ (eat) a lot. Sometimes, he __wants__ (want) to eat less
III – Alessandra _lives_(live) in Boston, United States. During the week, her daughters _work__
(work) as delivery girls for a big tech company.
Na sentença I, a primeira lacuna pede que “do” seja utlizado no presente. Fato confirmado pelo
advérbio “usually” que indica frequência. Portanto, devemos preencher essa lacuna com “do”. Já
a palavra “now” pede que a segunda lacuna seja preenchida no gerúndio, já que dá a ideia de que
está acontecendo naquele momento. A segunda lacuna deve ser preenchida com “am doing”.
Na sentença II, as duas lacunas possuem mais de uma opção que se encaixaria corretamente. Mas
é importante observar que, ao acertar as duas primeiras, a questão já estaria resolvida por conta
das opções apresentadas nas alternativas. Mas, de qualquer forma, a melhor opção para
preencher as lacunas era colocar ambos os verbos no presente simples, porque, nesse contexto,
dão a ideia de hábito, frequência. Portanto, “eats” e “wants” seriam o melhor encaixe para esses
verbos.
Na sentença III, seguindo o mesmo raciocínio da sentença anterior, a melhor opção para ambos
os verbos era colocá-los no presente simples. A melhor forma de encaixá-los seria “lives” e
“work”, respctivamente.
Temos a sequência “do / am doing / eats / wants / lives / work”
GABARITO: E

Questão 05 (EAM/INÉDITA) – Look at the picture below.

AULA 00 – ENGLISH GENERAL PRESENTATION 77


TEACHER ANDREA BELO

What are they doing in the picture?


A) The children play soccer every day.
B) They are playing soccer now.
C) The boys didn’t play soccer yesterday.
D) They never play soccer together.
E) The boys won’t play soccer.
Comentários: A alternativa A está incorreta. A figura apenas mostra duas crianças jogando
futebol. Não é possivel concluir que eles jogam futebol toodos os dias, como sugere a alternativa.
A alternativa B está correta. É possível inferir da figura que os meninos estão jogando futebol
nesse momento.
A alternativa C está incorreta. É impossível afirmar que eles não jogaram futebol ontem com base
na figura.
A alternativa D está incorreta. A própria figura mostra os meninos jogando juntos, o que contraria
a afirmação da alternativa.
A alternativa E está incorreta. Não é possível inferir da figura que os meninos não irão jogar
futebol, principalmente porque eles estão jogando.
GABARITO: B

QUESTÕES EFOMM
STRANGE 'ICE VOLCANOES' ERUPT NEAR LAKE SUPERIOR, MICHIGAN
The National Weather Service (NWS) in Grand Rapids, Michigan, captured a photo of two "ice
volcanoes" erupting on Oval Beach in Saugatuck, near Lake Michigan, over the weekend.
"It was a great day to visit the beach and watch the waves interact with the ice," NWS tweeted.
Volcanoes and ice may sound like an unusual combination but according to Michigan Tech, they
are a regular occurrence on the north shore of Lake Superior, north of Lake Michigan, in the winter
months and can range in size from less than a meter to more than eight.
An ice volcano requires three things—high surf, cool temperatures and ice. For an ice volcano to
form, wave activity must be at least a meter tall at their highest. According to Michigan Tech,
anything less is too small to create the volcano's cone.
These volcanoes form at the edge of the ice shelf, caused by the movement of high surf hitting
the face of the formation. Small valleys can cause the energy of the waves to concentrate,
prompting larger waves to form that eject a spatter onto the ice shelf in certain places.
If the cones remain active but become completely enclosed by the ice shelf, wave energy from
below can push water up through any cracks or openings.
Ice volcanoes are typically found in arcs along the shoreline. They are often evenly spaced apart
but can also form individually. Michigan Tech refers to these lone structures as "cold spot" ice

AULA 00 – ENGLISH GENERAL PRESENTATION 78


TEACHER ANDREA BELO

volcanoes, and describes them as "nearly symmetrical single cones that apparently are not related
to shoreline, sand bar or rock reef arcs."
These solitary cones form near the shore and are thought to be a result of a weakness in the ice.
However, they have not yet been observed in an active state.
The phenomena has been observed on the shores of Lake Erie.
In Duluth, Minnesota, a human-made incarnation was created by a spewing fire hydrant.
According to Lake Superior Duluth Streams, the city allowed Orange Street's fire hydrant to spray
water during the winter months to prevent the supply pipe freezing over.This is not the only type
of ice volcano that exists in the solar system. Evidence of ice volcanoes (or cryovolcanoes) have
been found on Ceres, Titan and Pluto. These differ from the volcanoes we have on Earth. Instead
of ejecting molten rock, they emit substances such as water, ammonia and methane.
"To put them in perspective, if Mount Vesuvius had been a cryovolcano, its lava would have frozen
the residents of Pompeii," Rosaly Lopes, a Cassini radar team investigation scientist from NASA,
said in a statement in 2008.
Adapted from https://www.newsweek.com/ice-volcanoes-lake-superior-michigan-1487618

Questão 01 (EFOMM/INÉDITA) – It is possible to infer from the text that


a) The ice volcanoes are dangerous formations to the population of cities around them.
b) The Earth is changing and will be a similar planet to Pluto, Titan and Ceres.
c) In the future, Mount Vesuvius will freeze the population instead of burning it.
d) The ice volcanoes are a common formation that occurs on the north shore of Lake Superior.
e) The investigation scientist from NASA is worried about ice volcanoes.
Comentários: A alternativa A está incorreta. O texto não fala sobre os “vulcões de gelo” serem
um perigo para a população das cidades ao redor deles.
A alternativa B está incorreta. Em momento algum o texto fala que o nosso planeta está se
transformando para algo similar a qualquer outro planeta. O texto apenas diz que outras
formações parecidas podem ser encontradas em outros planetas.
A alternativa C está incorreta. O texto diz que se o Monte Vesúvio tivesse sido um criovulcão, as
pessoas de Pompéia teriam sido congeladas por sua erupção. O texto não diz que isso vai
acontecer no futuro com o Monte Vesúvio.
A alternativa D está correta. O texto fala que os “vulcões de gelo” são uma formação comum que
ocorre na costa norte do Lake Superior, ao norte do Lake Michigan.
A alternativa E está incorreta. A cientista da NASA não está preocupada com as formações, ela
apenas teceu um comentário sobre o que teria acontecido se o Monte Vesúvio tivesse sido um
vulcão de gelo.
GABARITO: D

AULA 00 – ENGLISH GENERAL PRESENTATION 79


TEACHER ANDREA BELO

Text II
Based on the text below, answer questions 02, 03 and 04.
The housing crisis and garden villages
In this piece, we take a look at the potential impact of garden villages, including regional
implications, what the result might be for new buyers, and follow on trends
There are many reports of a UK housing crisis, but what efforts are being put into resolving the
issue?
What do we mean by a garden village?
By definition, it is a piece of brownfield land that is used to develop new areas for families and
businesses. They are usually smaller projects and can contain from 1,500 to 10,000 homes. Often,
garden villages have their own facilities — such as schools, shops and transport stations — which
makes this type of living space perfect for families and first-time buyers looking to lead the
picture-perfect life.
Garden villages allow residents to imprint their own mark on the area, including creating its own
identity and creating rules. However, there are a few ways to identify them. They must be a
settlement outside of an existing town or city and not closely attached. The British government is
currently supporting 17 locations around the country, with £6 million expected to go towards
funding 14 new garden villages and £1.4 million to support three garden towns (which are similar
to garden villages, only larger).
Looking at the potential regional impact:
Regions are set to see more manual work available in these regions as such areas will be creating
more than 50,000 homes. This will help to boost the economy, as it will provide people with more
jobs in the area.
As new homes are built, people will be buying them — this potentially means more residents
within the region. There is a popular misconception that this will put a strain on the resources of
current residents nearby, such as school places for their children and obtaining doctor
appointments. However, this is not the case, as garden villages are built with their own facilities
including schools and general practices. In turn, this will also create more jobs in the area of
development.
However, it’s important to remember that these locations usually have their own transport links.
On the other hand, more cars on the road could cause congestion.
Adapted from https://www.openaccessgovernment.org/housing-crisis-garden-villages/68975/

Questão 02 (EFOMM/INÉDITA) – Read the statements about the text and decide whether they
are TRUE (T) or FALSE (F). Mark the correct option.
I – The Garden Villages will help developing the economy of the country.
II – Garden Villages will not affect traffic in the regions they are built in.
III – Garden Villages are a good solution for the housing crisis, but Garden Towns are a problem
for the government.
IV – A Garden Village must be a settlement outside of an existing town or city and not closely
attached.
V – Garden Villages can cause a strain on the resources of current residents nearby, such as
school places for their children and obtaining doctor appointments.

AULA 00 – ENGLISH GENERAL PRESENTATION 80


TEACHER ANDREA BELO

a) F / T / T / F / F
b) T / F / F / T / F
c) T / T / T / F / T
d) F / F / F / T / F
e) F / T / F / T / T
Comentários: A sentença I está correta. O texto diz que as Garden Villages vão ajudar a
desenvolver a economia local e, consequentemente, do país.
A sentença II está incorreta. O texto afirma que um possível resultado negativo da implementação
do projeto das Garden Villages é justamente a questão do trânsito, pois haverá mais carros
circulando naquele local depois que todas as casas estiverem ocupadas.
A sentença III está incorreta. O texto não estabelece nenhuma diferença na efetividade das
Garden Villages em relação às Garden Towns. O texto apenas explica que a diferença entre elas é
o tamanho do projeto.
A sentença IV está correta. O texto é bem explícito ao dizer que uma Garden Village precisa ser
um assentamento fora de uma cidade já existente e não pode ser colada nessa cidade, deve haver
uma certa distância.
A sentença V está incorreta. O texto diz que é um erro conceitual achar que as Graden Villages
colocam os recursos já existentes na cidade em demanda excessiva, pois junto com as Garden
Villages surgem escolas, mercados e comércio que atende a necessidades gerais.
Temos a sequência T / F / F / T / F.
GABARITO: B

Questão 03 (EFOMM/INÉDITA) – In the excerpt “Garden villages allow residents to imprint their
own mark on the area, including creating its own identity and creating rules.”, the word in bold
means:
a) Emboss
b) Stymie
c) Destroy
d) Neglect
e) Dislodge
Comentários: A alternativa A está correta. “Imprint” significa fazer uma marca ou fixar
permanentemente. “Emboss” é um sinônimo de “imprint”.
A alternativa B está incorreta. “Stymie” significa bloquear. Não faz sentido que os residentes
sejam permitidos bloquear suas próprias marcas na área.
A alternativa C está incorreta. “Destroy” significa destruir e não tem nenhuma relação com
“imprint” que significa fixar/marcar.

AULA 00 – ENGLISH GENERAL PRESENTATION 81


TEACHER ANDREA BELO

A alternativa D está incorreta. “Neglect” significa negligenciar. Essa palavra não possui nenhuma
relação de significação com a palavra “imprint”.
A alternativa E está incorreta. “Dislodge” significa forçar algo para fora da posição inicial.
“Dislodge” não tem relação alguma com “imprint”.
GABARITO: A

Questão 04 (EFOMM/INÉDITA) – According to the text, Garden Towns


a) Can be placed anywhere.
b) Have around 1,000 homes.
c) Are similar to garden villages, but larger.
d) Are a better version of garden villages.
e) Will be implemented in 17 locations.
Comentários: A alternativa A está incorreta. “Garden Towns” não podem ser postas em qualquer
lugar. Há regras para a implementação desses projetos.
A alternativa B está incorreta. Mil casas é um quantitativo muito pequeno para caracterizar uma
Garden Town. Uma Garden Village pode chegar a ter dez mil casas e uma Garden Town precisa
ser maior do que isso.
A alternativa C está correta. Garden Towns são muito parecidas com Garden Villages, apenas
maiores.
A alternativa D está incorreta. O texto não diz que as Garden Towns são versões melhores das
Garden Villages, mas sim que são apenas versões maiores.
A alternativa E está incorreta. De acordo com o texto, as Garden Villages serão implementadas
em 17 localidades, não as Garden Towns.
GABARITO: C

Questão 05 (EFOMM/INÉDITA) – Choose the correct option to complete the paragraph below.
MARITIME AUTONOMY: A BRIDGE TOO FAR
Imagine ________ ships en route from departure port ‘A’ to arrive at different ports around the
world, each at a given time. These ships have no captain or crew _______ board and can navigate,
dock, load, unload and refuel on their own and are maintained by sensors, robots and drones. The
ships are navigated and controlled by computers from a fully automated port with no human
intervention or interaction. The operating system of each vessel makes decisions and takes
actions based on the situation it is _______. That’s ‘autonomous shipping’ and it is very different
from automated ships.
Adapted from https://www.oneseaecosystem.net/maritime-autonomy-a-bridge-too-far/

AULA 00 – ENGLISH GENERAL PRESENTATION 82


TEACHER ANDREA BELO

a) very / on / on
b) many / in / at
c) several / on / in
d) much / on / in
e) hundreds of / in / on
Comentários: A primeira lacuna tem diferentes opções de preenchimento. Dentre as opções
fornecidas nas alternativas, poderíamos usar “many”, “several” ou “hundreds of”. “Very ships”
não é uma estrutura aceitável e “much ships” também não. Com essa lacuna, podemos eliminar
as alternativas A e D.
A asegunda lacuna deve ser preenchida por “on” por se tratar de uma expressão consolidada na
língua inglesa. Fala-se “on board”. Com isso, podemos eliminar as alternativas B e E.
A terceira lacuna deve ser preenchida pela preposição “in”. Trata-se da Regência do substantivo
“situation”. Diz-se “in a situation”.
Após analisar as lacunas, chegamos à conclusão de que a alternativa C é a que melhor completa o
texto.
GABARITO: C

QUESTÕES EPCAR
Directions: Answer questions 01 to 08 according to TEXT I.
TEXT I
Over 100,000 Flamingos Reportedly Descend on Mumbai Amid India Strict Coronavirus
Lockdown
As people around the world stay inside to fight the spread of coronavirus, wild animals have begun
to enjoy the newfound space. That appears to be what’s happened in the metropolitan region of
Mumbai, India, where a record-breaking number of flamingos have migrated, painting the
wetlands pink, according to local reports.
The Bombay Natural History Society (BNHS) estimates that around 25% more flamingos have
migrated to the region compared to last year, likely caused by the decrease in human activity, the
Hindustan Times reports. The Science Times calculates around 150,000 flamingos have come to
the area.
India has been under a strict lockdown since March 25, which has required over a billion people
to stay home and shutter all but essential services, in an attempt to stop the spread of COVID-19,
the disease caused by novel coronavirus. On May 1, the Indian government extended the
lockdown through May 18.
Flamingos typically migrate to wetlands of the metropolitan region of Mumbai from November to
May, Rhul Khot, the assistant director of the BNHS, told the Times. But this year’s lockdown “is
giving these birds peace for roosting, no disturbance in their attempt to obtain food and overall
encouraging habitat,” Deepak Apte, the director of the BNHS, explained, per Times. He added that

AULA 00 – ENGLISH GENERAL PRESENTATION 83


TEACHER ANDREA BELO

the increase is also likely tied to a successful breeding season two years ago, as well as the
destruction of wetlands on India’s eastern seafront that could be pushing the birds to the Mumbai
region, per the Times.
Khot also told the Times that an increase in “domestic sewage” from people staying at home
during the lockdown “is helping the undisturbed formation of planktons, algae and microbenthos
formation, which forms the food for flamingos and other wetland birds.”
As the coronavirus pandemic has continued, animals have been spotted throughout the world in
places that are usually dominated by humans. CNN reports that dolphins have swam in India’s
Ganges river for the first time in years, and hundreds of monkeys have “descended” on Delhi as
the city remains under lockdown. Cougars were reportedly spotted prowling the streets in
Santiago, Chile, in April; wild Kashmiri goats meandered through on the town of Llandudno,
Wales, in March.
“Residents are cooped up at home spending their mornings and evenings at their balconies taking
photographs and videos of these relaxed birds,” Sunil Agarwal, a resident of Seawoods in Navi
Mumbai, told the Times about the flamingos. “The lockdown will at least prompt people to focus
on what is around them, which they had been taking for granted, and hopefully this site will be
declared a flamingo sanctuary soon.”
(Adapted from https://time.com/5831198/flamingos-coronavirus/)

Questão 01 (EPCAR/INÉDITA) – Mark the INCORRECT option.


a) Many more flamingos than usual have migrated to the region of Mumbai.
b) Less human activity is likely to encourage wild life to migrate to big cities.
c) Only essential services are open during the strict lockdown in India in order to stop the spread
of COVID-19.
d) They have been successfully destroying the wetlands on India’s eastern seafront.
Comentários: A alternativa A está correta. O texto afirma que um número 25% maior de flamingos
migraram para a região em comparação com o ano passado. Podemos confirmar essa informção
no trecho: “around 25% more flamingos have migrated to the region compared to last year”.
A alternativa B está correta. O texto afirma que a diminuição da atividade humana é,
provavelmente, a razão pela qual houve um aumento no número de flamingos migrando para a
região de Mumbai. Vemos essa informação no trecho: “around 25% more flamingos have
migrated to the region compared to last year, likely caused by the decrease in human activity”.
A alternativa C está correta. O texto afirma que, a fim de evitar a disseminação do COVID-19, a
Índia obrigou as pessoas a permanecer em suas casas e fechar tudo, exceto serviços essenciais. O
seguinte trecho confirma isso: “India has been under a strict lockdown since March 25, which has
required over a billion people to stay home and shutter all but essential services”.
A alternativa D está incorreta. Não se pode dizer que a destruição tem sido bem-sucedida, pois
destruir não é intencional, mas sim uma consequência. A destruição vem acontecendo de forma
não intencional e, como consequência disso, as aves estão sendo obrigadas a procurar outros
lugares, indo para a região de Mumbai.
GABARITO: D

AULA 00 – ENGLISH GENERAL PRESENTATION 84


TEACHER ANDREA BELO

Questão 02 (EPCAR/INÉDITA) – According to the text, mark the correct alternative.


a) Residents are angry about being at home due to the lockdown imposed in India.
b) 150,000 more flamingos have migrated to the region of Mumbai as a result of the 25%
increase.
c) There has never been so many flamingos migrating to the region of Mumbai.
d) Cougars were reported spotted attacking people in the streets of Santiago, Chile.
Comentários: A alternativa A está incorreta. O texto usa a expressão “cooped up” que significa
enfiados. Então, podemos dizer que o texto diz que as pessoas estão enfiadas em casa, e não que
elas estão bravas por ter de estar em casa.
A alternativa B está incorreta. O texto não que 150.000 flamingos a mais foram para a região de
Mumbai, já que esse número não representa o aumento de 25% relatado pelo texto. O que o
texto diz é que com o aumento de 25% no número de flamingos migrando para a região, o número
total de aves ultrapassa os 150.000.
A alternativa C está correta. O texto diz que houve um número recorde de flamingos migrando
para região. “a record-breaking number of flamingos have migrated, painting the wetlands pink,
according to local reports”. Portanto, é correto dizer que nunca houve um número tão grande de
flamingos migrando para a região de Mumbai.
A alternativa D está incorreta. O texto não diz que pumas foram vistos atacando as pessoas nas
ruas de Santiago. A palavra que o texto usa é “prowling” que significa rondando, e não atacando.
GABARITO: C

Questão 03 (EPCAR/INÉDITA) – In the sentence “the increase is also likely tied to a successful
breeding season two years ago” (paragraph 4) the verb tied means
a) associated.
b) the outcome.
c) permission.
d) avoided.
Comentários: A alternativa A está correta. “Associated” significa estar relacionado, associado a
algo, e “tied” tem significado muito parecido, sendo usado para dar a ideia de que algo está ligado
ou relacionado a algo.
A alternativa B está incorreta. “the outcome” indica o resultado de algo, e a palavra “tied” não
tem nenhuma relação de significado com o resultado de algo, pois “tied” significa estar
relacionado ou ligado a algo.
A alternativa C está incorreta. “Permission” significa permissão e nada tem a ver com estar ligado
a algo, que é a significação de “tied”.
A alternativa D está incorreta. “Avoided” significa evitar algo, e não guarda nenhuma relação com
a palavra “tied”, que significa estar ligado ou relacionado a algo.
GABARITO: A

AULA 00 – ENGLISH GENERAL PRESENTATION 85


TEACHER ANDREA BELO

Questão 04 (EPCAR/INÉDITA) – Mark the correct question to the sentence below.


“around 25% more flamingos have migrated to the region.”
a) How many more flamingos have migrated to the region?
b) How much more flamingos have migrated to the region?
c) How many flamingos have migrated to the region?
d) How much flamingos have migrated to the region?
Comentários: A alternativa A está correta. O trecho do enunciado está conjugado no present
perfect, portanto, a pergunta precisa ser feita nesse mesmo tempo verbal. A estrutura está
correta com o uso do “How many”, já que “flamingos” é um substantivo contável, e o verbo
“migrate” corretamente conjugado no present perfect como “have migrated”.
A alternativa B está incorreta. O uso do “How much” é o erro da alternativa, pois “flamingos” é
um substantivo contável e, por isso, deveria ser usado “How many” ao invés de” How Much”.
A alternativa C está incorreta. O erro da alternativa está no fato de que essa pegunta nã teria a
frase do enunciado como resposta, já que a pergunta não quer saber a quantidade adicional de
flamingos que migraram para a região, mas sim a quantidade total de flamingos que migraram
para a região.
A alternativa D está incorreta. Mais uma vez, o erro consiste no fato de a alternativa apresentar
o “how much” ao invés do “how many”, pois trata-se de um substantivo contável (flamingos),
além de não estabelecer que a pergunta exige o número adicional de flamingos como resposta,
mas sim o número total.
GABARITO: A

Questão 05 (EPCAR/INÉDITA) – Mark the alternative that completes the sentence.


One of the reasons why flamingos have migrated in greater number to the region of Mumbai is
a) the fact that the weather is better now due to the lockdown.
b) the destruction of the wetlands on India’s eastern seafront.
c) the increase in domestic sewage from people staying at home during the lockdown.
d) that people we are breeding flamingos in the region of Mumbai.
Comentários: A alternativa A está incorreta. O texto não diz, em momento algum, que o clima
está melhor durante o lockdown. Protanto, não se pode afirmar que esse é uma dos fatores que
fez com que mais flamingos migrassem para a região.
A alternativa B está correta. Segundo o texto, um dos fatores determinantes para o aumento do
número de flamingos migrando para a região de Mumbai é a destruição das terras alagadas
orientais da Índia.
A alternativa C está incorreta. O texto não coloca o aumento do esgoto doméstico como um dos
fatores para que os flamingos migrassem para a região. O texto apenas diz que o aumento do
esgoto doméstico contribui para a formação de algas e plânctons que formam a comida dos
flamingos e outras aves de terras alagadas.
A alternativa D está incorreta. O texto não afirma, em momento algum, que pessoas estão
fazendo criação de flamingos na região de Mumbai.
GABARITO: B

AULA 00 – ENGLISH GENERAL PRESENTATION 86


TEACHER ANDREA BELO

Questão 06 (EPCAR/INÉDITA) – Mark the alternative that DOESN’T complete the


sentence below.
Wildlife has been seen during the pandemic in
a) Delhi, where monkeys were seen in the city during the lockdown.
b) Ganges river, India, where dolphins were seen for the first time in years.
c) Santiago, where cougars were seen in the forests outside the city.
d) Llandudno, where kashmiri goats were seen rambling through the town.
Comentários: A alternativa A está incorreta. A alternativa completa a sentença de forma correta,
já que o texto diz que macacos foram vistos em Deli durante o confinamento forçado.
A alternativa B está incorreta. Esta alternativa também completa de forma correta a sentença,
pois o texto afirma que golfinhos foram vistos no rio Ganges pela primeira vez em anos.
A alternativa C está correta. Ela não completa de forma correta pois diz que os pumas foram vistos
em florestas fora da cidade de Santiago, enquanto o texto diz que os pumas foram vistos
rondando as ruas da cidade.
A alternativa D está incorreta. O texto, de fato, afirma que caprinos foram vistos vagando pela
cidade em Llandudno. A diferença é que a alternativa usa a palavra “rambling” ao invés de
“meandering”, o que não acarreta mudança de sentido, já que as duas palavras são sinônimas
nesse contexto.
GABARITO: C

Questão 07 (EPCAR/INÉDITA) – Read the sentences and mark the correct option.
I. Lockdown has been good to people and animals, who can have the city for themselves.
II. People being cooped up at home is a good thing for animals that can enjoy new spaces.
III. This number of flamingos migrating to the region of Mumbai was only possible due to
humans destroying their original habitat.
The only correct sentence(s) is(are)
a) II.
b) III.
c) I and II.
d) I and III.
Comentários: A sentença I está incorreta. O texto afirma que o confinamento das pessoas foi bom
para os animais, mas não afirma que as pessoas também se beneficiam do confinamento.
A sentença II está correta. O fato de as pessoas estarem confinadas em casa é bom para os
animais, pois eles têm muito mais liberdade para explorar novos lugares na cidade sem ter
contato com humanos.
A sentença III está incorreta. O texto não afirma que os flamingos só vão pra região de Mumbai
porque o habitat original deles foi destruído pelos humanos. O texto apenas diz que esse é um
dos fatores que contribuem para o aumento da migração das aves para essa região.
Somente a sentença II está correta.
GABARITO: A

AULA 00 – ENGLISH GENERAL PRESENTATION 87


TEACHER ANDREA BELO

Questão 08 (EPCAR/INÉDITA) – Mark the correct alternative to complete the sentence.


According to the text, the lockdown
a) makes people happy because nature is thriving in this situation.
b) has been responsible for significant changes in wildlife behavior.
c) has been responsible for the death of a lot of dolphins around the world.
d) is responsible for this site being considered a flamingo sanctuary.
Comentários: A alternativa A está incorreta. O texto não afirma, em momento algum, que as
pessoas estão felizes porque a natureza está triunfando nessa situação. As pessoas gostam de ver
a natureza triunfar, mas não gostam do confinamento.
A alternativa B está correta. O texto descreve várias situações atípicas em relação ao
comportamento de animais selvagens durante o confinamento imposto pela disseminação do
novo coronavírus. Portanto, é correto dizer que o confinamento das pessoas tem sido responsável
por mudanças significativas no comportamento dos animais selvagens.
A alternativa C está incorreta. O texto não fala em mortes de golfinhos. O texto apenas cita
golfinhos para dizer que eles foram vistos nadando no rio Ganges pela primeira vez em anos.
A alternativa D está incorreta. O texto não diz que Mumbai é considerado um santuário de
flamingos. O texto diz que um morador da região, ao ser entrevistado, diz que espera que este
local seja declarado um santuário de flamingos em breve.
GABARITO: B

Directions: Answer questions 09 to 15 according to TEXT II.


TEXT II
It’s boom time for podcasts – but will going mainstream kill the magic?
Fifteen years ago, when the word podcast was added to the dictionary, only the tech-savvy
were listening. Now, as star names pile in, they’re big business. Can the quality survive?
Hello friends! Do you fancy listening to “a new type of time-shifted amateur radio”? No? How
about a brilliant podcast? Of course you do.
Fifteen years ago, Macworld, a magazine for fans of Apple products, announced, with limited
fanfare, that Apple was about to add podcasts to iTunes, its music download offer. Unfortunately,
few readers knew what a podcast was, hence Macworld’s “time-shifted radio” definition. In June
2005, the idea of having thousands of ready-to-hear audio shows, anything from true-crime
documentaries to all-chums-together comedy, to up-to-the-minute news to gripping drama to
revealing interviews, and being able to listen to these shows whenever you want, wherever you
are – well, that wasn’t quite happening. So Apple’s move didn’t seem important. Nor did the fact
that the Oxford English Dictionary added “podcast” to its lexicon in the same year, after tech
journalist Ben Hammersley came up with the term in 2004 (which was also the year the BBC
launched a downloadable version of In Our Time). Podcasts were new. It takes time for the new
to become everyday.

AULA 00 – ENGLISH GENERAL PRESENTATION 88


TEACHER ANDREA BELO

Podcasts were mostly unheard of, except by the tech savvy. They were either downloadable
versions of existing radio shows or they were chatty riffs, made by amateurs who knew how to
upload their aural blogs online. Still, they were interesting. At least to me. Soon after podcasts’
iTunes debut, I started a new job as the Observer’s radio critic. Great job – except there was a
limited choice of programmes for me to review. Radio schedules rarely changed. Presenters
stayed in their jobs for years. The BBC dominated speech radio, aside from phone-ins; hardly any
other broadcaster had the money to make documentaries or drama.
Podcasts rescued me from aural monotony; I wrote my first piece about them in the summer of
2006. Apparently Coke Machine Glow and The Dawn and Drew Show were the ones to look out
for (me neither, now). The podcast I do recall from then is The Ricky Gervais Show; this dominated
the brand new iTunes podcast chart for weeks. Initially free, in early 2006 it switched to a pay-
per-listen model and proved both a forerunner and an outlier: since then, much podcast uptake
has been driven by comedy, but most shows are still free to listeners, paid for by adverts that
appear during episodes.
Today, the iTunes podcast chart is bustling with old hands and new kids on the block. Here are No
Such Thing As a Fish, Fearne Cotton’s Happy Place, That Peter Crouch Podcast, Katherine Ryan:
Telling Everybody Everything. Here are sections for new and noteworthy, cultivating calm, keeping
the kids busy. There are lists of the top 10 episodes, top 10 shows; all aside from the 19 other
regular categories: news, arts, true crime… So many podcasts! There are oodles of shows, too
many to ever get through.
(Adapted from https://www.theguardian.com/tv-and-radio/2020/may/03/its-boom-time-for-podcasts-but-will-going-mainstream-kill-the-magic)

Questão 09 (EPCAR/INÉDITA) – In the underlined sentences in the second paragraph, there are
____ verbs used in the past tense.
a) two
b) five
c) three
d) four
Comentários: Ao observar o trecho sublinhado, percebemos o verbo “added” no past simple,
também podemos observar o verbo “came up” que também está no past simple. Após esses dois,
temos “was”, que é o past simple do verbo to be. Além desses, podemos encontrar “launched”,
que é o simple past do verbo launch.Podemos dizer, então, que há quatro verbos sendo usados
no tempo passado no trecho sublinhado do segundo parágrafo.
GABARITO: D

Questão 10 (EPCAR/INÉDITA) – Podcasts


a) became mainstream in 2004.
b) were mostly unknown 15 years ago.
c) were added to Oxford English Dictionary due to their huge popularity in 2004.
d) didn’t took long to become mainstream.

AULA 00 – ENGLISH GENERAL PRESENTATION 89


TEACHER ANDREA BELO

Comentários: A alternativa A está incorreta. O texto não diz que os podcasts se tornaram
mainstream (corrente dominante) em 2004. O texto diz que o termo podcast surgiu em 2004,
quando ainda era pouco conhecido.
A alternativa B está correta. O texto diz que os podcasts eram desconhecidos pela maioria das
pessoas, exceto por aquelas mais esclarecidas em relação a tecnologia. O texto fala isso fazendo
referência ao ano de 2004, quando começaram a surgir os podcasts.
A alternativa C está incorreta. O texto diz que o dicionário de Oxford inseriu o termo podcast
depois que o jornalista de tecnologia Bem Hammersley cunhou o termo em 2004, mas que na
época isso não pareceu importante, pois os podcasts eram novos e desconhecidos.
A alternativa D está incorreta. O texto diz que os podcasts não demoraram para se popularizarem,
mas sim que leva tempo para que coisas novas se tornem cotidianas.
GABARITO: B

Questão 11 (EPCAR/INÉDITA) – Read the sentence.


“Podcasts were mostly unheard of, except by the tech savvy.” (Paragraph 3)
All the sentences below express a similar idea, EXCEPT
a) Aside tech savvy people, podcasts were mostly unheard of.
b) Aside tech savvy people, few knew podcasts.
c) many people knew podcasts, except by the tech savvy.
d) podcasts were mostly unheard of for those that were not tech savvy.
Comentários: A alternativa A está incorreta. A ideia passada pela frase da alternativa é a mesma
da frase do enunciado. A palavra “aside” foi usada no lugar de “except”, o que não muda o sentido
da frase.
A alternativa B está incorreta. A ideia passada pela frase da alternativa é a mesma da frase do
enunciado. É correto dizer que além dos esclarecidos em tecnologia, poucos conheciam os
podcasts, pois o texto escreve a mesma coisa de maneira um pouco diferente.
A alternativa C está correta. O erro da alternativa está no fato de dizer que muitas pessoas
conheciam os podcasts, exceto pelos esclarecidos em tecnologia. Esse trecho dá a entender que
os esclarecidos em tecnologia não conheciam os podcasts, e que o público geral conhecia, mas o
texto diz o contrário disso.
A alternativa D está incorreta. A ideia passada pela frase da alternativa é a mesma da frase do
enunciado. A frase só está em uma ordem diferente, mas diz que os podcasts eram desconhecidos
da maioria que não fazia parte do grupo de esclarecidos em tecnologia.
GABARITO: C

AULA 00 – ENGLISH GENERAL PRESENTATION 90


TEACHER ANDREA BELO

Questão 12 (EPCAR/INÉDITA) – The word chart (paragraph 4) can be replaced, without change
in meaning, by
a) channel.
b) show.
c) catalogue.
d) concert.
Comentários: A alternativa A está incorreta. No contexto apresentado, “chart” quer dizer
catálogo. “Channel” quer dizer canal, emissora. Por isso não se pode substituir um pelo outro sem
alteração de sentido.
A alternativa B está incorreta. “Show” significa atração, programa. Essas definições não são
condizentes com o significado de “chart”, que quer dizer catálogo no contexto apresentado.
A alternativa C está correta. “Catalogue” é exatamente a ideia apresentada por “chart” no
contexto apresentado. Ambas as palavras significam catálogo nesse contexto.
A alternativa D está incorreta. “Concert” significa concerto e nada tem a ver com o significado de
“chart”, que é catálogo.
GABARITO: C

Questão 13 (EPCAR/INÉDITA) – The opposite of savvy (paragraph 3) is


a) ignorant.
b) experienced.
c) versed.
d) adept.
Comentários: A alternativa A está correta. “Ignorant” significa ignorante. Já “savvy” significa
esclarecido, conhecedor. Pode-se, então, dizer que são opostos em significação dentro do
contexto apresentado.
A alternativa B está incorreta. “Experienced” significa experiente. “savvy” significa esclarecido,
conhecedor. Portanto não podemos dizer que são significados opostos.
A alternativa C está incorreta. “Versed” significa versado em algo, que é o mesmo significado de
“savvy” no contexto apresentado. Não se pode dizer que são opostos, mas sim sinônimos.
A alternativa D está incorreta. “Adept” significa adepto, que não traz uma ideia oposta à ideia de
“savvy” no contexto do texto. Não se pode dizer que são opostos.
GABARITO: A

AULA 00 – ENGLISH GENERAL PRESENTATION 91


TEACHER ANDREA BELO

Questão 14 (EPCAR/INÉDITA) – Read the sentence.


“Do you fancy listening to “a new type of time-shifted amateur radio”?” (First paragraph).
The word fancy has the same idea in
a) You have a fancy apartment.
b) Is her purse fancy?
c) They are using some fancy equipment.
d) I quite fancy the idea of going to the movies.
Comentários: A alternativa A está incorreta. A palavra “fancy” está sendo usada na função de
verbo no enunciado, enquanto a alternativa emprega a mesma palavra na função de adjetivo,
mudando seu sentido na frase.
A alternativa B está incorreta. A palavra “fancy”, no enunciado, tem a ideia de gostar, ou de estar
com vontade de fazer algo. Na alternativa, a palavra está sendo usado como uma característica
da bolsa, tendo o mesmo sentido da palavra chique.
A alternativa C está incorreta. A palavra “fancy”, no enunciado, tem a ideia de gostar, ou de estar
com vontade de fazer algo. Na alternativa, a palavra está sendo usado como uma característica
do equipamento, querendo dizer que o equipamento é chique.
A alternativa D está correta. Nessa alternativa, a palavra “fancy” é usada com o mesmo sentido
do enunciado, ou seja, com o sentido de gostar ou querer algo.
GABARITO: D

Questão 15 (EPCAR/INÉDITA) – The author considers (that)


a) podcasts will soon replace traditional radio broadcasters.
b) himself an enthusiast of podcasts.
c) there is no difference between radio shows and podcasts nowadays.
d) there aren’t enough podcasts available on iTunes.
Comentários: A alternativa A está incorreta. O autor não diz, em momento algum, que os podcasts
substituirão as emissoras de rádio tradicionais.
A alternativa B está correta. Ele diz que os podcasts salvaram ele da monotonia auditiva, e diz que
escreveu pela primeira vez sobre eles ainda em 2006. Portanto, é correto dizer que ele se
considera um entusiasta de podcasts.
A alternativa C está incorreta. Ele não aborda as diferenças entre programas de rádio e podcasts
nos dias de hoje. O assunto não é tratado em momento nenhum do texto.
A alternativa D está incorreta. No último parágrafo do texto, o autor diz que há muitos tipos de
podcasts diferentes nos dias de hoje no catálogo do iTunes. Tantos que nem seria possível ouvir
todos.
GABARITO: B

AULA 00 – ENGLISH GENERAL PRESENTATION 92


TEACHER ANDREA BELO

QUESTÕES ESA
The Dunning-Kruger Effect
The Dunning-Kruger effect is a type of cognitive bias in which people believe that they are smarter
and more capable than they really are. Essentially, low ability people do not possess the skills
needed to recognize their own incompetence. The combination of poor self-awareness and low
cognitive ability leads them to overestimate their own capabilities.
The term lends a scientific name and explanation to a problem that many people immediately
recognize—that fools are blind to their own foolishness. As Charles Darwin wrote in his book The
Descent of Man, "Ignorance more frequently begets confidence than does knowledge."
(Adapted from https://www.verywellmind.com/an-overview-of-the-dunning-kruger-effect-4160740)

Questão 01 (ESA/INÉDITA) – According to the text, it is correct to infer that:


(A) the Dunning-Kruger effect is a mental disease.
(B) people affected by the Dunning-Kruger effect have low self esteem.
(C) ignorance makes people lack confidence.
(D) Charles Darwin wrote a book about the Dunning-Kruger effect.
(E) the effect makes incompetent people think they are smart.
Comentários: A alternativa A está incorreta. O texto não diz que o efeito Dunning-Kruger é uma
doença mental, mas sim um viés cognitivo. Portanto, não podemos inferir isso do texto.
A alternativa B está incorreta. O texto não diz que o efeito Dunning-Kruger causa baixa
autoestima. O texto diz o oposto disso, que pessoas afetadas por esse efeito superestimam suas
capacidades.
A alternativa C está incorreta. O texto não diz que a ignorância faz as pessoas não terem confiança.
O que o texto diz é que a ignorância gera confiança com mais frequência que o conhecimento.
A alternativa D está incorreta. O texto não diz que Charles Darwin escreveu um livro sobre o
efeito, mas sim que ele escreveu em seu livro que a ignorância gera confiança com mais
frequência que o conhecimento.
A alternativa E está correta. O texto diz que o efeito Dunning-Kruger é um viés cognitivo que faz
com que pessoas com baixas capacidades e inteligência pensem que são mais inteligentes ou
capazes do que são, de fato.
GABARITO: E

Questão 02 (ESA/INÉDITA) – The underlined words in the text- “smarter”, “more capable” and
“fools” are, respectively,
(A) adjective (comparative), adjective (comparative), and noun.
(B) adjective (superlative), adjective (comparative), and noun.
(C) noun, adjective (comparative), and adjective (comparative).
(D) adjective (comparative), adjective (superlative), and noun.
(E) adjective (superlative), noun, and adjective (superlative).

AULA 00 – ENGLISH GENERAL PRESENTATION 93


TEACHER ANDREA BELO

Comentários: A palavra “smarter” é um adjetivo comparativo, pois é usado na comparação do


quão brilhantes as pessoas acham que são em comparação ao quão brilhantes elas são, de fato.
A expressão “more capable” é um adjetivo comparativo, pois é usado na comparação do quão
capazes as pessoas acham que são em comparação ao quão capazes elas são, de fato.
A palavra “fools” é um substantivo, pois é usado para se referir às pessoas com limitações
intelectuais, não apenas para caracterizá-las. O fato de a palavra estar no plural também é um
bom indicativo de que é um substantivo, e não um adjetivo, pois adjetivos não são flexionados
para o plural em Língua Inglesa.
Temos a sequência: adjective (comparative), adjective (comparative), and noun.
GABARITO: A

Questão 03 (ESA/INÉDITA) – Complete the sentences below using the most appropriate words:
Lockdown is __ measure adopted to make sure people are at home. This is __ only way to oblige
___ people to stay confined.
(A) a lot of / a / the
(B) a / the / a lot of
(C) many / a / a
(D) a / an / a lot of
(E) a / the / a
Comentários: A primeira lacuna deve ser preenchida por “a”. O artigo indefinido é a palavra mais
adequada para preencher a lacuna, pois as outras opções apresentadas pelas alternativas não se
encaixam de forma adequada na lacuna.
A segunda lacuna deve ser preenchida por “the”. A palavra “only” dá a ideia de que há uma única
solução para o problema. Portanto, o artigo a ser usado é o artigo definido “the”.
A terceira lacuna deve ser preenchida por “a lot of”. Dentre as opções apresentadas para a
terceira lacuna, a que melhor se encaixa é “a lot of”, pois as outras opções deixariam a frase sem
sentido.
Temos a sequência: a / the / a lot of
GABARITO: B

Questão 04 (ESA/INÉDITA) – Which sentence is grammatically correct?


(A) One of those who is part of your family can be a person affected by the Dunning Kruger
effect.
(B) One of those who are part of your family can be some people affected by the Dunning
Kruger effect.
(C) One of those who are part of your family can be a person affected by the Dunning Kruger
effect.
(D) One of those whose are part of your family can be a person affected by the Dunning Kruger
effect.
(E) One of those whom are part of your family can be a person affected by the Dunning Kruger
effect.

AULA 00 – ENGLISH GENERAL PRESENTATION 94


TEACHER ANDREA BELO

Comentários: A alternativa A está incorreta. O erro da alternativa está no uso do verbo “is”. O
correto seria o uso do verbo “are”, pois o verbo precisa concordar com “those”, que é plural.
A alternativa B está incorreta. O erro da alternativa está no uso da expressão “some people”. O
correto seria o uso da expressão “a person”, pois a expressão precisa concordar com “One”, que
é que é singular.
A alternativa C está correta. “Are” concorda com “those” e “a person” concorda com “one”.
A alternativa D está incorreta. O erro da alternativa está no uso da palavra “whose” que significa
de quem, e prejudica o sentido da frase. O correto seria usar “who” ou “that”.
A alternativa E está incorreta. O erro da alternativa está no uso da palavra “whom”que deve ser
usada na função de objeto, não na função de sujeito, como é o caso da alternativa.
GABARITO: C

Questão 05 (ESA/INÉDITA) – Complete the sentences below using the most appropriate words:
Technology is __ fastest way to develop things in general. It’s incredible how everything
changed and __ world became different. It’s one of __ most interesting happenings in the
history.
(A) a / the/ a
(B) a / a/ a
(C / a / a/ the
(D) an / a/ the
(E) the / the/ the
Comentários: A primeira lacuna deve ser preenchida por “the”. O artigo indefinido é a palavra
mais adequada para preencher a lacuna com o superlativo (a forma mais rápida de..), pois as
outras opções apresentadas pelas alternativas não se encaixam de forma adequada na lacuna.
A segunda lacuna deve ser preenchida também por “the”. A palavra mundo (world) indica que
está especificando que “o” mundo (the world) ficou diferente e não um mundo qualquer, como
seria “a” world. Portanto, o artigo a ser usado é o artigo definido “the”.
A terceira lacuna deve ser preenchida por “the”. O artigo indefinido é a palavra mais adequada
para preencher a lacuna com o superlativo (um dos mais interessantes acontecimentos) pois as
outras opções apresentadas pelas alternativas não se encaixam de forma adequada na lacuna.
Temos a sequência: the / the / the
GABARITO: E

AULA 00 – ENGLISH GENERAL PRESENTATION 95


TEACHER ANDREA BELO

QUESTÕES ESCOLA NAVAL


Based on the text below, answer the six questions that follow it. The paragraphs of the text are
numbered.
What Leads Us to this Belief that Connections to and Experiences with the Natural World Are So
Important?
There is an emerging awareness and concern that children are less and less likely to have
experiences that involve the simplest interactions with nature—the plants, animals, and the earth
around them—as a part of their continuous learning process. As we look at the children around
us, we observe them living increasingly unhealthy lifestyles. For many, childhood is spent overly
plugged in and programmed inside their homes, schools, and community settings in humanmade
environments, eliminating the out-of-doors, the benefits of nature, and all that exists in the
natural environment (Wike, 2006).
Even a generation ago, children spent more time outside, because it was the normal thing for
children to do. Adults did not question the value of time spent out-of-doors and had much less
anxiety about the risks involved. Children walked and played outdoors and planted things in the
dirt; they rode their bikes, invented games, and spent the majority of their time in less structured
activities and natural environments. Very young children carried out these activities in their yards
and immediate neighborhoods. Urban environments offered the occasional playground and
vacant lot. Older children roamed beyond their neighborhoods to adjacent lands, streams, woods,
or urban parks. Exposure to the natural world brought opportunities for children to make sense
of their surroundings and to develop their own sense of “place.”
Over a relatively short time, we adults have allowed this connection to the natural world to slip
gradually away from children’s lives. Evidence of this trend surrounds us:
Children now spend nearly 30 hours a week watching a TV or computer screen, listening to
something through headphones or, for older children, using cell phones or media players;
Children experience increasingly timed and structured family lifestyles with less emphasis on
unstructured outside time. Particularly in more densely populated countries, urban growth has
eliminated green spaces and natural environments. Fewer families are vacationing in national
parks (Fish, 2007). Together these changes keep many children separated from nature and
without time for solitude or um-programmed experiences.
Instructional time outside, recess, or unstructured playtime is being eliminated from the school
day (Clements, 2007).
Outdoor play spaces or playgrounds have become safer at some sites due to national playground
standards, but in other locations, play spaces have been eliminated or often lack the natural
elements that encourage a different kind of interaction among children (Moore & Wong, 1997).
The curriculum for children in centers and schools is becoming narrower, with more time spent
on teacher-directed lessons and testing and less time spent investigating and learning through
activities that build on a child’s sense of wonder, curiosity, and the benefit of first-hand
experiences (Hyson, 2003; McMurrer, 2007; Marcon, 1999).
Consequently, we have gradually found our children growing up in a clash of optimal and minimal
learning opportunities. Optimally, technology opens worlds never before so readily available to

AULA 00 – ENGLISH GENERAL PRESENTATION 96


TEACHER ANDREA BELO

children; however, the opening of this side of learning has contributed to shutting the door to
children’s access to the more natural environment that gives a lasting attachment to children’s
sense of place and their awareness of the habitat and environment nearest to them. This lack of
connection can engender both apathy and ignorance in children’s early perceptions of the world
around them and their roles in enjoying, learning from, and protecting it.
(Adapted from: https://www.education.ne.gov/wpcontent/uploads/2017/07/Call_to_Action.pdf)

Questão 01 (ESCOLA NAVAL/INÉDITA) – According to the text, which option completes the
sentence below correctly?
In the past, it was the normal thing for children to ______.
(A) spent more time outside
(B) spend more time indoors
(C) question the value of time spent out-of-doors
(D) spend more time outdoors
(E) spend their times in programmed inside activities
Comentários: A alternativa A está incorreta. O erro da alternativa está no uso do verbo conjugado
no Simple Past (spent), quando deveria estar no Simple Present (spend) para que possa se
encaixar corretamente na frase do enunciado.
A alternativa B está incorreta. Não é correto dizer que no passado, era normal que as crianças
passassem mais tempo dentro de casa, já que o texto diz o contrário disso no trecho: “Even a
generation ago, children spent more time outside”.
A alternativa C está incorreta. O texto diz que os ADULTOS NÃO questionavam o valor do tempo
passado fora de casa. Porém o texto não fala nada sobre as crianças questionarem ou não o tempo
passado fora de casa.
A alternativa D está correta. O texto diz exatamente o que é dito pela alternativa, que as crianças,
no passado, passavam mais tempo fora de casa. Isso é confirmado pelo trecho: “Even a generation
ago, children spent more time outside, because it was the normal thing for children to do”.
A alternativa E está incorreta. O texto, de fato, diz que as crianças passam o tempo em atividades
programadas em ambientes internos. Porém, o texto fala que essa é a realidade das crianças nos
dias de hoje, não no passado.
GABARITO: D

Questão 02 (ESCOLA NAVAL/INÉDITA) – What's the meaning of the word "engender" in


paragraph 4?
(A) Point out.
(B) Disagree.
(C) Give rise to.
(D) Endanger.
(E) Be caused by.

AULA 00 – ENGLISH GENERAL PRESENTATION 97


TEACHER ANDREA BELO

Comentários: A alternativa A está incorreta. “Point out” significa apontar/destacar algo,


enquanto “engender” significa causar/ser causa de. Não há, portanto, relação de sentido entre as
expressões.
A alternativa B está incorreta. “Disagree” significa discordar de algo, enquanto “engender”
significa causar/ser causa de. Não há, portanto, relação de sentido entre as expressões.
A alternativa C está correta. “Give rise to” significa dar crescimento a algo/ser a causa de, que é
exatamente o significado de “engender”, ser a causa de algo. Por isso, essa é a alternativa correta.
A alternativa D está incorreta. “Endanger” significa colocar em perigo ou risco, enquanto
“engender” significa causar/ser causa de. Não há, portanto, relação de sentido entre as
expressões.
A alternativa E está incorreta. “Be caused by” significa ser causado por,ou seja, ser a consequência
de algo, não a causa. Já “engender” significa causar/ser causa de. Não há, portanto, relação de
sentido entre as expressões.
GABARITO: C

Questão 03 (ESCOLA NAVAL/INÉDITA) – According to the text, which option is correct?


(A) Today children have the maximum and the minimum at the same time when it comes to
learning opportunities.
(B) In the past, education was better because children had access to more outside time at
schools.
(C) Thanks to technology education is better today, since children have instant access to things
other generations didn’t.
(D) Technology access will give children the necessary will to fight for the planet environment in
the future.
(E) The fact the unstructured playtime is being eliminated from the school day is a good thing for
raising awareness of the habitat nearest to the children.
Comentários: A alternativa A está correta. O texto diz que as crianças de hoje têm o máximo e o
mínimo ao mesmo tempo quando se trata de oportunidades de aprendizagem. Isso ocorre porque
a internet otimiza grandemente o aprendizado, dando acesso às mais diversas informações. Por
outro lado, as atividades externas estão diminuindo drasticamente o que empobrece a
aprendizagem sensorial em relação ao meio ambiente.
A alternativa B está incorreta. Não se pode dizer que antigamente a educação era melhor pelo
fato de que havia mais tempo de atividades externas. O texto não fala que hoje é melhor ou pior,
ele apenas mostra diferenças dizendo o que melhorou e o que piorou, mas não é possível chegar
a uma conclusão final de qual educação é melhor, a de hoje ou a do passado.
A alternativa C está incorreta. O texto fala da prontidão do acesso às informações qua a tecnologia
proporciona como algo bom, porém cita outros pontos que são negativos em relação à educação

AULA 00 – ENGLISH GENERAL PRESENTATION 98


TEACHER ANDREA BELO

de hoje em dia. Portanto, o texto não nos permite definir qual é a melhor educação, a de hoje ou
a do passado.
A alternativa D está incorreta. Não se pode dizer que o acesso à tecnologia vai dar às crianças de
hoje a força de vontade necessária para lutar pelo meio ambiente no futuro. O texto diz que a
falta de atividades sensoriais, de experiências vividas na natureza pode diminuir o senso de
habitat no ambiente que as rodeia.
A alternativa E está incorreta. O texto diz o contrário da alternativa, que a falta de atividade não
estruturada ao ar livre no dia a dia escolar diminui a consciência ambiental das crianças.
GABARITO: A

Questão 04 (ESCOLA NAVAL/INÉDITA) – According to the text, which option is correct?


(A) Children now spend around 6 hours a day watching TV or computer screen or doing
something technology-related.
(B) Children spend more and more time in structured family lifestyles emphasizing outdoor
activities.
(C) There is a concern that children have less and less contact with nature and that leads them
to an unhealthy lifestyle.
(D) Children’s structured playtime is being eliminated from their school day.
(E) Children don’t play outside in the nature because National playground standards eliminated
natural spaces from playgrounds.
Comentários: A alternativa A está incorreta. Essa quantidade de horas está incorreta, já que o
texto diz que as crianças passam por volta de 30 horas por semana vendo TV ou usando
tecnologia. Seria correto dizer quatro horas e meia por dia ao invés de seis horas por dia.
A alternativa B está incorreta. A primeira parte da alternativa está correta, pois o texto diz que as
crianças passam cada vez mais tempo em estilos de vida familiares estruturados. Porém o texto
não diz que esse estilo de vida enfatiza atividades ao ar livre.
A alternativa C está correta. O texto diz que as crianças de hoje brincam cada vez menos ao ar
livre e que isso leva a um estilo de vida sedentário dentro de casa, da escola, em ambientes da
comunidade feitos pelo Homem, eliminando o externo, os benefícios da natureza e tudo que
existe no ambiente natural.
A alternativa D está incorreta. As brincadeiras estruturadas estão cada vez mais presentes no dia
a dia escolar, e não sendo eliminadas como diz a alternativa.
A alternativa E está incorreta. O texto não diz que os padrões nacionais de playground eliminaram
a natureza dos playgrounds, mas sim que os tornaram mais seguros em algumas localidades.
Porém, em muitas outras localidades esses parques foram eliminados ou não possuem elementos
naturais que encorajam um tipo diferente de interação entre crianças.
GABARITO: C

AULA 00 – ENGLISH GENERAL PRESENTATION 99


TEACHER ANDREA BELO

Questão 05 (ESCOLA NAVAL/INÉDITA) – In paragraph 2, the word "it" refers to


(A) children.
(B) generation.
(C) natural environment.
(D) childhood.
(E) spent more time outside.
Comentários: A alternativa A está incorreta. Não se pode refeir a “children” usando o pronome
“it”, já que esse pronome é usado no singular, e children é plural. O pronome adequado seria
“they” para referir-se a “children”.
A alternativa B está incorreta. “generation” significa geração. Não se pode dizer que uma geração
era a coisa normal a se fazer. A frase ficaria sem sentido, por isso sabemos que o pronome não
retoma “generation”.
A alternativa C está incorreta. “Natural environment” é uma coisa, e não se pode dizer que o
ambiente natural era a coisa normal a se fazer. A frase ficaria sem sentido, por isso sabemos que
o pronome não retoma “natural environment”.
A alternativa D está incorreta. “Childhood” significa infância, e não se pode dizer que a infância
era a coisa normal a se fazer, não se trata de uma ação. A frase ficaria sem sentido se usássemos
o pronome para retomar “childhood”.
A alternativa E está correta. Era a coisa normal a se fazer, passar mais tempo ao ar livre. Essa
alternativa está correta, pois “it” retoma a ação de se passar mais tempo do lado de fora.
GABARITO: E

QUESTÕES EsPCEx
Leia o texto a seguir e responda às questões 01, 02 e 03
How to Make the Study of Consciousness Scientifically Tractable
Strangely, modern science was long dominated by the idea that to be scientific means to remove
consciousness from our explanations, in order to be “objective.” This was the rationale behind
behaviorism, a now-dead theory of psychology that took this trend to a perverse extreme.
Behaviorists like John Watson and B.F. Skinner scrupulously avoided any discussion of what their
human or animal subjects thought, intended or wanted, and focused instead entirely on behavior.
Erwin Schrödinger labeled this approach in his 1958 book Mind and Matter, the “principle of
objectivation”. Schrödinger did identify both the problem and the solution. He recognized that
“objectivation” is just a simplification that is a temporary step in the progress of science in
understanding nature.
We are now at the point, it seems to a growing number of thinkers who are finally listening to
Schrödinger, where we must abandon, where appropriate, the principle of objectivation. It is time
for us to employ a “principle of subjectivation” and in doing so understand not just half of reality—
the objective world—but the whole, the external and internal worlds.

AULA 00 – ENGLISH GENERAL PRESENTATION 100


TEACHER ANDREA BELO

The science of consciousness has enjoyed a renaissance in the last couple of decades and the
study of our own minds—consciousness/subjectivity—has finally become a respectable pursuit.
It’s still tricky, however, to determine what kinds of data and what kinds of experiments we should
consider legitimate in the study of consciousness.
Adapted from https://blogs.scientificamerican.com/observations/how-to-make-the-study-of-consciousness-scientifically-tractable/

Questão 01 (EsPCEx/INÉDITA) – According to the text, choose the correct statement.


A) the science of consciousness has regained importance over the last years.
B) the rationale behind behaviorism was opposed to being objective.
C) John Watson and B.F. Skinner focused both on behaviour and consciousness.
D) modern science is still dominated by the idea that scientific means objective
E) Schrödinger has been proven wrong by a growing number of thinkers.
Comentários: A) De acordo com o texto, de fato, o estudo da consciência tem reganhado
importância ao longo dos anos. “The science of consciousness has enjoyed a renaissance in the
last couple of decades and the study of our own minds—consciousness/subjectivity—has finally
become a respectable pursuit.” (4º parágrafo). Alternativa CORRETA.
B) Conforme o texto, a lógica do behaviorismo não era oposta a ser objetivo, pelo contrário. A
lógica dessa teoria da psicologia era justamente remover qualquer tipo de subjetividade do objeto
de estudo. “...to be scientific means to remove consciousness from our explanations, in order to
be ‘objective.’ This was the rationale behind behaviorism...” (1º parágrafo). Alternativa
INCORRETA.
C) Segundo o texto, John Watson e B.F. Skinner não focavam em consciência, mas tão somente no
comportamento. “Behaviorists like John Watson and B.F. Skinner scrupulously avoided any
discussion of what their human or animal subjects thought, intended or wanted, and focused
instead entirely on behavior.” (2º parágrafo). Alternativa INCORRETA.
D) De acordo com o texto, há um número crescente de pensadores que finalmente estão ouvindo
Schrödinger e defendendo que devemos abandonar, quando apropriado, o princípio da
objetivação. “We are now at the point, it seems to a growing number of thinkers who are finally
listening to Schrödinger, where we must abandon, where appropriate, the principle of
objectivation.” (3º parágrafo). Alternativa INCORRETA.
E) O texto não fala que um número crescente de pensadores provou que Schrödinger estava
errado. Como vimos, muitos pensadores, na verdade, começaram a concordar com ele no sentido
de que o objetivismo puro deveria ser abandonado quando possível. “We are now at the point, it
seems to a growing number of thinkers who are finally listening to Schrödinger, where we must
abandon, where appropriate, the principle of objectivation.” (3º parágrafo). Alternativa
INCORRETA.
GABARITO: A

AULA 00 – ENGLISH GENERAL PRESENTATION 101


TEACHER ANDREA BELO

Questão 02 (EsPCEx/INÉDITA) – In the sentence “This was the rationale behind behaviorism, a
now-dead theory of psychology that took this trend to a perverse extreme.” (Paragraph 1), the
word rationale means:
A) excuse
B) logic
C) prevention
D) proof
E) fate
Comentários: A questão quer que você identifique o significado da palavra rationale. No contexto,
o sentido dessa palavra é a lógica, o pensamento, referindo-se ao embasamento do behaviorismo.
Vamos identificar em qual alternativas temos esse significado.
A) Como vimos, rationale, no contexto, significa o pensamento, a lógica, o embasamento de uma
teoria da psicologia. Excuse significa desculpa. Alternativa INCORRETA.
B) Logic (lógica) corresponde ao sentido de rationale no contexto, sendo o nosso gabarito.
Alternativa CORRETA.
C) Prevention significa prevenção, e não lógica, pensamento. Alternativa INCORRETA.
D) Proof significa prova, comprovação, e não lógica, pensamento. Alternativa INCORRETA.
E) Fate significa destino, não se relacionando à ideia de rationale. Alternativa INCORRETA.
GABARITO: B

Questão 03 (EsPCEx/INÉDITA) – The word did in the sentence “Schrödinger did identify both the
problem and the solution.” (paragraph 2) expresses
A) emphasis
B) irrelevance
C) contrast
D) conclusion
E) denial
Comentários: Você provavelmente está mais acostumado com o uso dos auxiliares, como do, does
e did, para fazer interrogativas e negativas. Contudo, também é possível usá-los numa afirmativa,
quando queremos enfatizar, destacar a ideia que estamos transmitindo. Nesse caso, usamos o
auxiliar e o verbo na sua foma base, isto é, sem alterações. Na frase trazida pela questão, a ideia
é que Schrödinger identificou de fato, mesmo, realmente, tanto o problema quanto a solução.
Vamos às alternativas:
A) Did, na frase, expressa ênfase. Alternativa CORRETA.
B) Como vimos, did não expressa irrelevância, mas ênfase. Alternativa INCORRETA.
C) Como vimos, did não expressa contraste, mas ênfase. Alternativa INCORRETA.

AULA 00 – ENGLISH GENERAL PRESENTATION 102


TEACHER ANDREA BELO

D) Did não dá ideia de conclusão, mas ênfase. Alternativa INCORRETA.


E) Denial significa negação. Como vimos, did não expressa negação, mas ênfase. Alternativa
INCORRETA.
GABARITO: A

Leia o texto a seguir e responda às questões 04, 05 e 06


What will art look like in 20
The future may be uncertain, but some things are undeniable: climate change, shifting
demographics, geopolitics. The only guarantee is that there will be changes, both wonderful and
terrible. It’s worth considering how artists will respond to these changes, as well as what purpose
art serves, now and in the future.
Reports suggest that by 2040 the impacts of human-caused climate change will be unescapable,
making it the big issue at the centre of art and life in 20 years’ time. Artists in the future will wrestle
with the possibilities of the post-human and post-Anthropocene – artificial intelligence, human
colonies in outer space and potential doom.
The identity politics seen in art around the #MeToo and Black Lives Matter movements will grow
as environmentalism, border politics and migration come even more sharply into focus. Art will
become increasingly diverse and might not ‘look like art’ as we expect. In the future, once we’ve
become weary of our lives being visible online for all to see and our privacy has been all but lost,
anonymity may be more desirable than fame. Instead of thousands, or millions, of likes and
followers, we will be starved for authenticity and connection. Art could, in turn, become more
collective and experiential, rather than individual.
Adapted from http://www.bbc.com/culture/story/20190418-what-will-art-look-like-in-20-years

Questão 04 (EsPCEx/INÉDITA) – Which question below has its answer in paragraph 3?


A) What are some undeniable things about the future?
B) Who studied about art in the future?
C) When will the impacts of human-caused climate change be unescapable?
D) What will art look like in the future?
E) What are some possibilities of the post-human and post-Anthropocene?
Comentários: A) A pergunta trazida pela letra A não está respondida no parágrafo 3, mas sim no
parágrafo 1. “The future may be uncertain, but some things are undeniable: climate change,
shifting demographics, geopolitics.” Alternativa INCORRETA.
B) O texto, na realidade, não responde à pergunta “Quem estudou sobre a arte no futuro?”
Alternativa INCORRETA.
C) A pergunta trazida pela letra C não está respondida no parágrafo 3, mas sim no parágrafo 2.
“Reports suggest that by 2040 the impacts of human-caused climate change will be
unescapable...” Alternativa INCORRETA.

AULA 00 – ENGLISH GENERAL PRESENTATION 103


TEACHER ANDREA BELO

D) A pergunta trazida pela letra D tem sua resposta no parágrafo 3, como podemos ver no trecho
a seguir: “Art will become increasingly diverse and might not ‘look like art’ as we expect... Art
could, in turn, become more collective and experiential, rather than individual.” Alternativa
CORRETA.
E) A pergunta trazida pela letra E não está respondida no parágrafo 3, mas sim no parágrafo 2.
“Artists in the future will wrestle with the possibilities of the post-human and post-Anthropocene
– artificial intelligence, human colonies in outer space and potential doom.” Alternativa
INCORRETA.
GABARITO: D

Questão 05 (EsPCEx/INÉDITA) – Which one from the underlined verbs in the text conveys a verb
form that is different from the others?
A) be (paragraph 1)
B) respond (paragraph 1)
C) wrestle (paragraph 2)
D) seen (paragraph 3)
E) look (paragraph 3)
Comentários: Ao observarmos os verbos aos quais a questão se refere, podemos constatar que
estão todos na forma verbal do infinitivo, com exceção de seen, que é particípio do verbo see. Os
verbos no infinitivo ficam em sua forma original, ou seja, não sofrem nenhuma alteração em sua
estrutura. É como se o verbo estivesse ali da forma como “nasceu”.
A) A questão pede o verbo que está numa forma diferente. Be está no infinitivo, assim como todos
os demais, com exceção de seen, não sendo, portanto, o nosso gabarito. Alternativa INCORRETA.
B) Respond está no infinitivo, assim como todos os demais, com exceção de seen, não sendo,
portanto, o nosso gabarito. Alternativa INCORRETA.
C) Wrestle está no infinitivo, assim como todos os demais, com exceção de seen, não sendo,
portanto, o nosso gabarito. Alternativa INCORRETA.
D) Seen é o único dos verbos trazidos pela questão que está numa forma diferente, qual seja, o
particípio. Alternativa CORRETA.
E) Look está no infinitivo, assim como todos os demais, com exceção de seen, não sendo, portanto,
o nosso gabarito. Alternativa INCORRETA.
GABARITO: D

Questão 06 (EsPCEx/INÉDITA) – Choose the alternative that correctly substitutes the sentences
“The future may be uncertain, but some things are undeniable...” (paragraph 1) with no change
in meaning.
A) The future may be uncertain, so some things are undeniable.
B) The future may be uncertain, because some things are undeniable.
C) The future may be uncertain. Some things are undeniable, as well.
D) The future may be uncertain; moreover, some things are undeniable.
E) The future may be uncertain. Some things are undeniable, though.

AULA 00 – ENGLISH GENERAL PRESENTATION 104


TEACHER ANDREA BELO

Comentários: A) A conjunção but é adversativa, servindo para contrastar, opor ideias. Temos que
buscar nas alternativas uma “linking word” que desempenhe a mesma função, para que a
afirmativa seja substituída sem que haja alteração de sentido. So (então) introduz uma conclusão.
Dessa forma, a letra A não é o nosso gabarito. Alternativa INCORRETA.
B) Because (porque) introduz uma explicação, uma razão. Como estamos procurando uma
conjunção que oponha ideias, a alternativa B não está correta. Alternativa INCORRETA.
C) A expressão “as well” pode significar também ou assim como. Não estabelece, dessa maneira,
uma oposição, não sendo esse o gabarito da questão. Alternativa INCORRETA.
D) Moreover é usado para acrescentar informações, dando a ideia de além disso. Como estamos
procurando uma conjunção que oponha ideias, a alternativa D não está correta. Alternativa
INCORRETA.
E) Though é uma “linking word” que estabelece justamente um contraste, uma oposição de ideias,
assim como o but. O uso do though é um caso curioso, pois a sua posição na estrutura das orações
pode causar estranhamento. Diferentemente do but, que aparece no meio das sentenças, unindo-
as, a posição mais usual do though é no final, depois das duas sentenças entre as quais deseja-se
estabelecer o contraste. Dessa maneira, “The future may be uncertain. Some things are
undeniable, though.” tem o mesmo sentido das sentenças trazidas pelo enunciado. Alternativa
CORRETA.
GABARITO: E

Leia o texto a seguir e responda às questões 07, 08 e 09


Internet privacy: the apps that protect you from your apps
Tech companies don’t have favourite songs, but if they __________ (1), they __________ (2) all
pick Radiohead’s Just – “You do it to yourself, you do/ And that’s what really hurts,” they would
croon, staring their users dead in the eye. And strictly speaking, they’d be right: many of the worst
excesses of the industry are, technically, optional. The world isn’t actually a binary choice between
living in a surveillance state and opting out of all technological development since the turn of the
millennium. You can opt out – you just have to know how.
Of course, that knowledge is not always easily acquired, nor is it necessarily easy to apply. So a
new breed of services has arrived to try to help normal users take control of their digital lives.
Companies including Disconnect.
Me and Jumbo act as something like a digital concierge for their users, tweaking privacy settings,
deleting sensitive data and throwing a spanner into the inner workings of surveillance capitalism.
But there’s a Faustian pact involved: to use the privacy apps to their fullest requires handing them
a level of control over your digital life that would be all too easy to abuse – and it’s hard to be
certain that any company can be trusted with information that sensitive.
Most of the companies that track you aren’t open enough for you to even know they’re snooping
on you in the first place.
Adapted from https://www.theguardian.com/technology/2020/feb/16/internet-privacy-settings-apps-to-protect-you-

AULA 00 – ENGLISH GENERAL PRESENTATION 105


TEACHER ANDREA BELO

Questão 07 (EsPCEx/INÉDITA) – According to the text, read the statements and choose the
correct alternative.
A new kind of services has emerged to try to help users take control of their digital lives.
There are only two possible choices in digital life: living in a surveillance state or choosing not to
participate of technology.
Most of the companies that track people are honest about snooping on them.
Using privacy apps requires handing them a safe level of control over users’ digital life.
Companies including Disconnect.Me and Jumbo work as a digital curator for their users, deleting
sensitive data, for example.
A) I and V are correct.
B) All of them are correct.
C) III, IV and V are correct.
D) II, III and IV are correct.
E) I and III are correct.
Comentários: I. De acordo com o texto, de fato, um novo tipo de serviço surgiu para ajudar as
pessoas a terem mais controle sobre suas vidas online. “So a new breed of services has arrived to
try to help normal users take control of their digital lives.” (2º parágrafo). Afirmação CORRETA.
II. Segundo o texto, não há apenas duas opções na vida digital, quais sejam, viver em um estado
de vigilância ou optar por não participar da tecnologia. O texto afirma que o mundo não se resume
a essa escolha binária, dual. “The world isn’t actually a binary choice between living in a
surveillance state and opting out of all technological development since the turn of the
millennium.” (1º parágrafo). Afirmação INCORRETA.
III. O texto afirma que a maioria das empresas NÃO são abertas o suficiente sobre o quanto elas
espionam a vida das pessoas. “Most of the companies that track you aren’t open enough for you
to even know they’re snooping on you in the first place.” (3º parágrafo). Afirmação INCORRETA.
IV. O texto afirma que, para usar ao máximo os aplicativos de privacidade, é necessário oferecer a
eles um nível de controle sobre a vida digital que seria muito fácil de abusar. Ou seja, não é um
nível seguro como afirma o item IV. Afirmação INCORRETA.
V. Conforme o texto, as empresas mencionadas, de fato, atuam como concierges, curadoras, para
seus usuários, deletando dados sensíveis, por exemplo. “Companies including Disconnect.Me and
Jumbo act as something like a digital concierge for their users, tweaking privacy settings, deleting
sensitive data and throwing a spanner into the inner workings of surveillance capitalism.” (2º
parágrafo). Afirmação CORRETA.
Como apenas a I e a V estão corretas, o gabarito dessa questão é a letra A.
GABARITO: A

AULA 00 – ENGLISH GENERAL PRESENTATION 106


TEACHER ANDREA BELO

Questão 08 (EsPCEx/INÉDITA) – In the sentence “Most of the companies that track you aren’t
open enough for you to even know they’re snooping on you in the first place.” (Paragraph 3), the
verb snooping means
A) tolerating
B) preparing
C) spying
D) serving
E) neglecting
Comentários: O verbo snoop significa investigar, espionar, na tentativa de descobrir algo,
especialmente informações sobre assuntos pessoais de alguém. No texto, o verbo se refere ao
fato de as empresas que funcionam como curadoras digitais não serem abertas o suficiente para
se ter certeza se elas próprias estão ou não espionando a vida dos usuários do serviço.
A) Como vimos, snoop não significa tolerar, mas investigar, espionar. Alternativa INCORRETA.
B) Snoop não significa preparar, mas espionar. Alternativa INCORRETA.
C) Conforme estudamos, snoop pode perfeitamente ser compreendido como spy, pois transmite
a ideia de espionar, investigar. Assim, no texto, snooping significa spying. Alternativa CORRETA.
D) Snoop não significa servir, mas espionar. Alternativa INCORRETA.
E) Snoop não significa negligenciar, mas espionar. Alternativa INCORRETA.
GABARITO: C

Questão 09 (EsPCEx/INÉDITA) – The sentence “Tech companies don’t have favourite songs, but
if they __________ (1), they __________ (2) all pick Radiohead’s Just...” refers to a hypothetical
situation. Choose the alternative containing the correct verb tenses to complete gaps (1) and (2)
in paragraph 1.
A) do, will
B) did, would
C) do, would
D) didn’t, would
E) don’t, won’t
Comentários: É importante observar que o enunciado da questão afirma que a sentença se refere
a uma situação hipotética. Normalmente, “reduzimos” o tempo de todos os verbos em sentenças
hipotéticas, isto é, “voltamos” o tempo verbal- no caso, de presente para passado. As empresas
de tecnologia não têm (presente) músicas favoritas, mas se elas tivessem, escolheriam... No
contexto, tivessem, em Inglês, seria substituído pelo auxiliar do passado, did. Did entraria no lugar
de toda a expressão “tivessem uma música favorita” e evitaria a repetição desnecessária de
palavras. Would transformaria pick (escolher) em escolheriam.

AULA 00 – ENGLISH GENERAL PRESENTATION 107


TEACHER ANDREA BELO

A) Como estudamos, as formas verbais corretas para tratar de uma situação hipotética, no caso,
seriam did e would (pick). Do, auxiliar do presente, e will, indicativo de futuro, normalmente não
são usados para tratar de situações hipotéticas, não reais. Alternativa INCORRETA.
B) As formas verbais corretas para tratar de uma situação hipotética, no caso, seriam justamente
did e would (pick). Alternativa CORRETA.
C) Como estudamos, as formas verbais corretas para tratar de uma situação hipotética, no caso,
seriam did e would (pick). Do, auxiliar do presente, normalmente não é usado para tratar de
situações hipotéticas, não reais. Alternativa INCORRETA.
D) Como vimos, a ideia seria dizer que as empresas não têm músicas favoritas, mas se tivessem...
Não faria sentido dizer que elas não têm, mas se não tivessem... Alternativa INCORRETA.
E) Como estudamos, as formas verbais corretas para tratar de uma situação hipotética, no caso,
seriam did e would (pick). No caso da letra E, além dos tempos verbais não estarem corretos, usar
a negativa não faria sentido. Como vimos, a ideia seria dizer que as empresas não têm músicas
favoritas, mas se tivessem... Alternativa INCORRETA.
GABARITO: B

Leia o texto a seguir e responda à questão 10


Brain surgery patient filmed playing violin during operation
A patient at King’s College Hospital in London has played the violin while surgeons operated on
her brain to remove a tumour. The medical team asked Dagmar Turner, 53, to play the violin to
ensure parts of the brain which control delicate hand movements and coordination were not
damaged during the millimetre-precise surgery.
Ms Turner was diagnosed in 2013 with a brain tumour after suffering a seizure during a symphony.
The committed violinist was worried the surgery would mean she would lose the ability to play.
Her tumour was located in the right frontal lobe of her brain, close to an area that controls the
fine movement of her left hand. Professor Keyoumars Ashkan, consultant neurosurgeon at King’s
College Hospital, came up with a plan to operate and reduce the risk.
Prior to surgery, doctors spent two hours carefully mapping her brain to identify areas that were
active when she played the violin and those responsible for controlling language and movement.
Ms Turner played violin while her tumour was removed, while closely monitored by the
anaesthetists and a therapist.
Prof Ashkan said: “We managed to remove over 90 per cent of the tumour, including all the areas
suspicious of aggressive activity, while retaining full function in her left hand.”Ms Turner added:
“The violin is my passion; I’ve been playing since I was 10 years old. The thought of losing my
ability to play was heartbreaking.”
Adapted from https://www.independent.co.uk/news/health/violin-brain-surgery-kings-hospital-nhs-a9342006.html

AULA 00 – ENGLISH GENERAL PRESENTATION 108


TEACHER ANDREA BELO

Questão 10 (EsPCEx/INÉDITA) – The sentence


Ms Turner was diagnosed in 2013 with a brain tumour by doctors.
is correctly changed into active voice in
A) Doctors diagnose Ms Turner in 2013 with a brain tumour.
B) Doctors diagnosed Ms Turner in 2013 with a brain tumour.
C) Doctors diagnoses Ms Turner in 2013 with a brain tumour.
D) Doctors have diagnosed Ms Turner in 2013 with a brain tumour.
E) Doctors had diagnosed Ms Turner in 2013 with a brain tumour.
Comentários: Nessa questão, é importante que você se lembre da voz passiva. Resumidamente,
a voz passiva em Inglês é usada quando, ao transmitirmos a mensagem, queremos colocar o foco
no que é feito e não em quem pratica a ação. Quanto à estrutura, vamos sempre precisar de uma
forma do verbo TO BE no tempo verbal que queremos usar e do past participle do verbo que
descreve a ação. Na questão, temos a afirmação já na voz passiva. Assim, para “voltarmos” para a
voz ativa, o sujeito vai virar objeto (Ms Turner) e o que chamamos, em Português, de agente da
passiva vai virar o sujeito (doctors). A forma verbal composta was diagnosed (foi diagnosticada)
voltará para a forma simples diagnosed (diagnosticaram). Assim, a frase na voz ativa ficaria:
Doctors diagnosed Ms Turner in 2013 with a brain tumour.
A) Como vimos, o verbo correto na voz ativa seria diagnosed, preservando o tempo verbal em que
a frase estava na voz passiva, ou seja, o passado. Na letra A, foi usado o verbo no presente.
Alternativa INCORRETA.
B) Vimos que a frase na voz ativa ficaria exatamente assim, como trazido pela alternativa B:
Doctors diagnosed Ms Turner in 2013 with a brain tumour. Alternativa CORRETA.
C) Como vimos, o verbo correto na voz ativa seria diagnosed, preservando o tempo verbal em que
a frase estava na voz passiva. Na letra C, foi usado o verbo no presente e conjugado de forma
incorreta de acordo com o sujeito (doctors= they e não he, she ou it). Alternativa INCORRETA.
D) Conforme estudamos, o verbo correto na voz ativa seria diagnosed, preservando o tempo
verbal em que a frase estava na voz passiva. Na letra D, foi usado o verbo no present perfect.
Alternativa INCORRETA.
E) Conforme vimos, o verbo correto na voz ativa seria diagnosed, mantendo o tempo verbal em
que a frase estava na voz passiva. Na letra E, foi usado o verbo no past perfect. Alternativa
INCORRETA.
GABARITO: B

AULA 00 – ENGLISH GENERAL PRESENTATION 109


TEACHER ANDREA BELO

CONSIDERAÇÕES FINAIS
Parabéns pela nossa primeira aula concluída, um passo a mais até a sua aprovação! Eu sei
que a prova exige que você saiba muitas estruturas, vocábulos e interpretação de textos em
Inglês. Mas, dia após dia, você vai se acostumando com o ritmo das aulas, que preparei de maneira
equilibrada para cada conteúdo a ser estudado.
E, adaptando-se às aulas dinâmicas aqui apresentadas, você ficará cada vez mais confiante
e seguro dos seus resultados. Vai dar certo e levará à sua aprovação!
Outro detalhe importante para seu sucesso nos estudos, é fazer listas de vocabulário das
palavras que você achou difíceis a cada aula, em cada exercício ou lista, a fim de reescrevê-las e
então, recordá-las nos momentos de pausa entre as aulas.
Minha sugestão é que você faça a leitura dessas palavras consideradas “novas” para vê-las
novamente. Isso te ajudará nas questões em que esses vocábulos reaparecem. Acontece muito
com a classe dos verbos, por exemplo.
A cada lista de exercício resolvida ou mesmo a cada exercício que você faça, perceberá
como fica mais fácil identificar um verbo já visto no tempo passado ou particípio. É sua conquista
de etapas e que tornará você, um candidato mais bem preparado e confiante para realizar uma
excelente prova.
É importante lembrar também do nosso Fórum de dúvidas, exclusivo do Estratégia
Militares. Será minha forma de responder, no prazo máximo de 48 horas, o que mais você precise
saber para que os conteúdos fiquem ainda mais claros em seus estudos, certo?

AULA 00 – ENGLISH GENERAL PRESENTATION 110


TEACHER ANDREA BELO

REFERÊNCIAS BIBLIOGRÁFICAS
BARRETO, Tania Pedroza; GARRIDO, Maria Line; SILVA, João Antenor de C., Inglês Instrumental.
Leitura e compreensão de textos. Salvador, Ba UFBA, 1995, p. 64.
BROWN. H. Douglas. Principles of Language Learning and Teaching. Prentice Hall International,
1988.
COMPEDELLI, Samira Yousseff. Português, Literatura, Produção de texto & Gramática – São Paulo:
Ed. Saraiva, 2002.
CORREIA, Clese Mary P. Reading Specific Purposes. Salvador/ Ba: UFBA, 1999.
COSTEIRA, Adriana Araújo de M. Reading Comprehension Skills. João Pessoa/PB: ETFP, 1998.
CRYSTAL David. Cambridge University Press 1997. The Cambridge Encyclopedia of Language.
Cambridge University Press 1997
FREEMAN. Diane Larsen. MURCIA. Marianne Celce. The Grammar Book, 1999.
DYE, Joan., FRANFORT, Nancy. Spectrum II, III A Communicative Course in English. USA, Prentice
Hall, 1994.
FAVERO, Maria de Lourdes Albuquerque (org.). Dicionário de educadores no Brasil: da colônia aos
dias atuais. Rio de Janeiro: UFRJ, MEC, INEP, 1999.
FRANKPORT, Nancy & Dye Hoab. Spectrum II, III Prentice Hall Regents Englewood Cliffs, New Jersy,
1994.
GADELHA, Isabel Maria B. Inglês Instrumental: Leitura, Conscientização e Prática. Teresina:
EDUFFI, 2000.
GUANDALINI, Eiter Otávio. Técnicas de Leitura em Inglês: ESP – English For Specific Purposes:
estagio 1. São Paulo: Texto novo, 2002.
GRELLET, Françoise. Developing Reading Skills. Cambridge University Press, 1995
HOLAENDER, Arnon & Sanders Sidney. A complete English Course. São Paulo. Ed. Moderna, 1995.
HUTCHINSON, Tom & WATERS, Alan. English for Specific Purposes. Cambridge: Cambridge
University Press, 1996
KRASHEN. Stephen D. Second Language Acquisition and Second Language Learning, Prentice-Hall
International, 1988.
LAENG, Mauro. Dicionário de pedagogia. Lisboa: Dom Quixote, 1973.
LEFFA, Vilson J. Metodologia do ensino de línguas. In: BOHN, H.; VANDRESEN, P. (org.). Tópicos de
linguística aplicada: o ensino de línguas estrangeiras. Florianópolis: Editora da UFSC, 1988. p. 211-
231.
LIBERATO, Wilson. Compact English Book Inglês Ensino Médio. São Paulo: FTD, Vol. Único, 1998
Mc ARTHUR. The Oxford Companion to the English Language. Oxford University Press 1992
Fromkin. Victoria. An Introduction to Language
MARQUES, Amadeu. Inglês Série Brasil. ed. Atica. São Paulo: 2004. Vol. Único.

AULA 00 – ENGLISH GENERAL PRESENTATION 111


TEACHER ANDREA BELO

MURPHY, Raymond: Essencial Grammar in Use Oxford. New York Ed. Oxford University, 1997.
OLIVEIRA, Luciano Amaral. English For Tourism Students. Inglês para Estudantes de Turismo: São
Paulo, Rocca, 2001.
OLIVEIRA, Sara Rejane de F. Estratégias de leitura para Inglês Instrumental. Brasília: UNB, 1994.
QUINTANA, et alli. First Certificate. Master Class Oxfor. New York, 2004: Ed. Oxford University.
PAULINO, Berenice F. et all. Leitura em textos em Inglês – Uma Abordagem Instrumental. Belo
Horizonte: Ed. Dos Autores, 1992.
PEREIRA, Edilberto Coelho. Inglês Instrumental. Teresina: ETFPI, 1998.
RODGES, Theodore. Jack C. Richards. Approaches and Methods in Language Teaching. Cambridge
University Press, 2001.
RODMAN Robert. Harcourt Brace 1993. English as a Global Language
STEWART, B., HAINES S. First Certificate, MasterClass. UK – Oxford 2004.
SILVA, João Antenor de C., GARRIDO, Maria Lina, BARRETO, Tânia Pedrosa. Inglês Instrumental:
Leitura e Compreensão de Textos. Salvador: Centro Editorial e Didático, UFBA. 1994
SOARES, Moacir Bretãs. Dicionário de legislação do ensino. 19.ed. Rio de Janeiro: FGV, 1981.
SOUZA, Adriana Srade F. Leitura em Língua Inglesa: Uma abordagem Instrumental. São Paulo:
Disal, 2005.
TUCK, Michael. Oxford Dictionary of Computing for Learners of English. Oxford: Oxford University
Press, 1996.
TOTIS, Verônica Pakrauskas. Língua Inglesa: leitura. São Paulo: Cortez, 1991.
Livros eletrônicos:
Dicionário Houaiss da Língua Portuguesa, Editora Objetiva, 2001.
MOURãO, Janaína Pereira. "Skimming x Scanning"; Brasil Escola. Disponível em
<https://brasilescola.uol.com.br/ingles/skimming-x-scanning.htm>. Acesso em 20 de março de
2019.
www.newsweek.com - Acesso em 18 de março de 2019.
http://www.galaor.com.br/tecnicas-de-leitura/ - Acesso em 19 de março de 2019.
Expressões Idiomáticas (continuação)" em Só Língua Inglesa. Virtuous Tecnologia da
Informação,2008-2019. Consultado em 03/04/2019 às 22:09. Disponível na Internet em
http://www.solinguainglesa.com.br/conteudo/Expressoes5.php

AULA 00 – ENGLISH GENERAL PRESENTATION 112


TEACHER ANDREA BELO

TRADUÇÕES
TEXT Howard Gardner: ‘Multiple intelligences’ are not ‘learning styles’ by Valerie Strauss
The fields of psychology and education were revolutionized 30 years ago when we now
worldrenowned psychologist Howard Gardner published his 1983 book Frames of Mind: The
Theory of Multiple Intelligences, which detailed a new model of human intelligence that went
beyond the traditional view that there was a single kind that could be measured by standardized
tests.
Gardner’s theory initially listed seven intelligences which work together: linguistic, logical-
mathematical, musical, bodily-kinesthetic, interpersonal and intrapersonal; he later added an
eighth, naturalist intelligence and says there may be a few more. The theory became highly
popular with K-12¹ educators around the world seeking ways to reach students who did not
respond to traditional approaches, but over time, ‘multiple intelligences’ somehow became
synonymous with the concept of ‘learning styles’. In this important post, Gardner explains why the
former is not the latter.
It’s been 30 years since I developed the notion of ‘multiple intelligences’. I have been gratified by
the interest shown in this idea and the ways it’s been used in schools, museums, and business
around the world. But one unanticipated consequence has driven me to distraction and that’s the
tendency of many people, including persons whom I cherish, to credit me with the notion of
‘learning styles’ or to collapse ‘multiple intelligences’ with ‘learning styles’. It’s high time to relieve
my pain and to set the record straight.
First a word about ‘MI theory’. On the basis of research in several disciplines, including the study
of how human capacities are represented in the brain, I developed the idea that each of us has a
number of relatively independent mental faculties, which can be termed our ‘multiple
intelligences’. The basic idea is simplicity itself. A belief in a single intelligence assumes that we
have one central, all-purpose computer, and it determines how well we perform in every sector
of life. In contrast, a belief in multiple intelligences assumes that human beings have 7 to 10
distinct intelligences.
Even before I spoke and wrote about ‘MI’, the term ‘learning styles’ was being bandied about in
educational circles. The idea, reasonable enough on the surface, is that all children (indeed all of
us) have distinctive minds and personalities. Accordingly, it makes sense to find out about learners
and to teach and nurture them in ways that are appropriate, that they value, and above all, are
effective.
Two problems: first, the notion of ‘learning styles’ is itself not coherent. Those who use this term
do not define the criteria for a style, nor where styles come from, how they are recognized/
assessed/ exploited. Say that Johnny is said to have a learning style that is ‘impulsive’. Does that
mean that Johnny is ‘impulsive’ about everything? How do we know this? What does this imply
about teaching? Should we teach ‘impulsively’, or should we compensate by ‘teaching
reflectively’? What of learning style is ‘right-brained’ or visual or tactile? Same issues apply.
Problem #2: when researchers have tried to identify learning styles, teach consistently with those
styles, and examine outcomes, there is not persuasive evidence that the learning style analysis
produces more effective outcomes than a ‘one size fits all approach’. Of course, the learning style

AULA 00 – ENGLISH GENERAL PRESENTATION 113


TEACHER ANDREA BELO

analysis might have been inadequate. Or even if it is on the mark, the fact that one intervention
did not work does not mean that the concept of learning styles is fatally imperfect; another
intervention might have proved effective. Absence of evidence does not prove non-existence of a
phenomenon; it signals to educational researchers: ‘back to the drawing boards’.
Here’s my considered judgment about the best way to analyze this lexical terrain: Intelligence: We
all have the multiple intelligences. But we signed out, as a strong intelligence, an area where the
person has considerable computational power. Style or learning style: A hypothesis of how an
individual approaches the range of materials. If an individual has a ‘reflective style’, he/she is
hypothesized to be reflective about the full range of materials. We cannot assume that
reflectiveness in writing necessarily signals reflectiveness in one’s interaction with the others.
Senses: Sometimes people speak about a ‘visual’ learner or an ‘auditory’ learner. The implication
is that some people learn through their eyes, others through their ears. This notion is incoherent.
Both spatial information and reading occur with the eyes, but they make use of entirely different
cognitive faculties. What matters is the power of the mental computer, the intelligence that acts
upon that sensory information once picked up.
These distinctions are consequential. If people want to talk about ‘an impulsive style’ or a ‘visual
learner’, that’s their prerogative. But they should recognize that these labels may be unhelpful, at
best, and ill-conceived at worst.
In contrast, there is strong evidence that human beings have a range of intelligences and that
strength (or weakness) in one intelligence does not predict strength (or weakness) in any other
intelligences. All of us exhibit jagged profiles of intelligences. There are common sense ways of
assessing our own intelligences, and even if it seems appropriate, we can take a more formal test
battery. And then, as teachers, parents, or selfassessors, we can decide how best to make use of
this information.
(Adapted from https://www.washingtonpost.com/news/answer-sheet)

TEXTO Howard Gardner: ‘Inteligências múltiplas’ não são ‘estilos de aprendizagem’ de Valerie
Strauss
Os campos da psicologia e da educação foram revolucionados há 30 anos, quando agora o
psicólogo mundialmente famoso Howard Gardner publicou seu livro de 1983 Frames of Mind: The
Theory of Multiple Intelligences, que detalhava um novo modelo de inteligência humana que ia
além da visão tradicional de que havia um tipo único que pode ser medido por testes
padronizados.
A teoria de Gardner inicialmente listou sete inteligências que trabalham juntas: linguística, lógico-
matemática, musical, corporal-cinestésica, interpessoal e intrapessoal; mais tarde, ele
acrescentou uma oitava inteligência naturalista e diz que pode haver mais algumas. A teoria se
tornou muito popular entre educadores K-12¹ em todo o mundo que buscam maneiras de alcançar
alunos que não respondiam às abordagens tradicionais, mas com o tempo, "inteligências
múltiplas" de alguma forma se tornaram sinônimo do conceito de "estilos de aprendizagem".
Neste importante post, Gardner explica por que o primeiro não é o último.

AULA 00 – ENGLISH GENERAL PRESENTATION 114


TEACHER ANDREA BELO

Já se passaram 30 anos desde que desenvolvi a noção de "inteligências múltiplas". Fiquei satisfeito
com o interesse demonstrado por esta ideia e as formas como ela tem sido usada em escolas,
museus e negócios em todo o mundo. Mas uma consequência imprevista me levou à distração e
essa é a tendência de muitas pessoas, incluindo pessoas que eu prezo, de me creditarem a noção
de "estilos de aprendizagem" ou de colapsar "inteligências múltiplas" com "estilos de
aprendizagem". É hora de aliviar minha dor e esclarecer as coisas.
Primeiro, uma palavra sobre a 'teoria das IM'. Com base na pesquisa em várias disciplinas,
incluindo o estudo de como as capacidades humanas são representadas no cérebro, desenvolvi a
ideia de que cada um de nós tem uma série de faculdades mentais relativamente independentes,
que podem ser denominadas nossas "inteligências múltiplas". A ideia básica é a própria
simplicidade. A crença em uma única inteligência pressupõe que temos um computador central e
multifacetado, e isso determina nosso desempenho em todos os setores da vida. Em contraste, a
crença em inteligências múltiplas pressupõe que os seres humanos tenham 7 a 10 inteligências
distintas.
Mesmo antes de falar e escrever sobre 'MI', o termo 'estilos de aprendizagem' estava sendo
cogitado nos círculos educacionais. A ideia, aparentemente razoável, é que todas as crianças (na
verdade, todos nós) têm mentes e personalidades distintas. Assim, faz sentido descobrir mais
sobre os alunos e ensiná-los e nutri-los de maneiras que sejam adequadas, que valorizem e, acima
de tudo, sejam eficazes.
Dois problemas: primeiro, a noção de "estilos de aprendizagem" em si não é coerente. Aqueles
que usam este termo não definem os critérios para um estilo, nem de onde vêm os estilos, como
são reconhecidos / avaliados / explorados. Digamos que Johnny tenha um estilo de aprendizagem
"impulsivo". Isso significa que Johnny é "impulsivo" em relação a tudo? Como nós sabemos disso?
O que isso significa sobre o ensino? Devemos ensinar ‘impulsivamente’, ou devemos compensar
‘ensinando reflexivamente’? O que dizer do estilo de aprendizagem é "cérebro direito" ou visual
ou tátil? Os mesmos problemas se aplicam.
Problema # 2: quando os pesquisadores tentam identificar estilos de aprendizagem, ensinar de
forma consistente com esses estilos e examinar os resultados, não há evidências persuasivas de
que a análise do estilo de aprendizagem produz resultados mais eficazes do que uma abordagem
de "tamanho único". Claro, a análise do estilo de aprendizagem pode ter sido inadequada. Ou,
mesmo que seja acertado, o fato de uma intervenção não ter funcionado não significa que o
conceito de estilos de aprendizagem seja fatalmente imperfeito; outra intervenção pode ter se
mostrado eficaz. Ausência de evidências não prova a inexistência de um fenômeno; ele sinaliza
para os pesquisadores educacionais: ‘de volta às pranchetas’.
Aqui está meu julgamento ponderado sobre a melhor maneira de analisar este terreno léxico:
Inteligência: todos nós temos as inteligências múltiplas. Mas destacamos, como uma inteligência
forte, uma área em que a pessoa tem considerável poder computacional. Estilo ou estilo de
aprendizagem: Uma hipótese de como um indivíduo aborda a variedade de materiais. Se um
indivíduo tem um "estilo reflexivo", supõe-se que ele / ela seja reflexivo sobre toda a gama de
materiais. Não podemos presumir que a refletividade na escrita necessariamente sinaliza
refletividade na interação de um com os outros.
Sentidos: às vezes, as pessoas falam sobre um aluno "visual" ou "auditivo". A implicação é que
algumas pessoas aprendem com os olhos, outras com os ouvidos. Essa noção é incoerente. Tanto

AULA 00 – ENGLISH GENERAL PRESENTATION 115


TEACHER ANDREA BELO

a informação espacial quanto a leitura ocorrem com os olhos, mas fazem uso de faculdades
cognitivas totalmente diferentes. O que importa é o poder do computador mental, a inteligência
que atua sobre as informações sensoriais uma vez coletadas.
Essas distinções são consequentes. Se as pessoas querem falar sobre 'um estilo impulsivo' ou um
'aprendizado visual', essa é sua prerrogativa. Mas eles deveriam reconhecer que esses rótulos
podem ser inúteis, na melhor das hipóteses, e mal concebidos, na pior.
Em contraste, há fortes evidências de que os seres humanos têm uma gama de inteligências e que
a força (ou fraqueza) em uma inteligência não prediz a força (ou fraqueza) em nenhuma outra
inteligência. Todos nós exibimos perfis irregulares de inteligências. Existem maneiras de bom
senso de avaliar nossas próprias inteligências e, mesmo que pareça apropriado, podemos fazer
uma bateria de testes mais formal. E então, como professores, pais ou autoavaliadores, podemos
decidir a melhor forma de usar essas informações.

Social media ’destroying how society works;


A former Facebook executive has said social media is doing great harm to society around the
world. The executive is a man called Chamath Palihapitiya. He ___________ Facebook in 2007
and ___________a vice president. He was responsible for increasing the number of users
Facebook had. Mr Palihapitiya said he feels very guilty about getting more people to use social
networks. He said the networks are destroying society because they are changing people&apos;s
behavior. Twenty years ago, people talked to each other face to face. Today, people message each
other and do not talk. People also really care about what other people think of them. They post
photos and wait to see how many people like the photo. They get very sad if people do not like
the photo.
Mr. Palihapitiya said people should take a long break from social media so they can experience
real life. He wants people to value each other instead of valuing online "hearts, likes, and thumbs-
up". Palihapitiya also points out how fake news is affecting how we see the world, it is becoming
easier for large websites to spread lies. It is also becoming easier to hurt other people online.
Anyone can hide behind a fake user name and post lies about other people. Palihapitiya said this
was a global problem. He is worried about social media so much that he has banned his children
from using it. However, he did state that Facebook was a good company. He said: "Of course,
it&apos;s not all bad. Facebook overwhelmingly does good in the world."

A mídia social destrói como a sociedade funciona;


Um ex-executivo do Facebook disse que a mídia social está causando um grande dano à sociedade
em todo o mundo. O executivo é um homem chamado Chamath Palihapitiya. Ele ___________ o
Facebook em 2007 e ___________um vice-presidente. Ele foi o responsável por aumentar o
número de usuários do Facebook. Palihapitiya disse que se sente muito culpado por conseguir
que mais pessoas usem as redes sociais. Ele disse que as redes estão destruindo a sociedade
porque estão mudando o comportamento das pessoas. Vinte anos atrás, as pessoas conversavam
cara a cara. Hoje, as pessoas trocam mensagens e não falam. As pessoas também se preocupam

AULA 00 – ENGLISH GENERAL PRESENTATION 116


TEACHER ANDREA BELO

com o que as outras pessoas pensam delas. Eles postam fotos e esperam para ver quantas pessoas
gostaram da foto. Eles ficam muito tristes se as pessoas não gostam da foto.
Palihapitiya disse que as pessoas deveriam fazer uma longa pausa nas redes sociais para que
pudessem experimentar a vida real. Ele quer que as pessoas valorizem umas às outras em vez de
valorizar "corações, curtidas e polegares" online. Palihapitiya também aponta como as notícias
falsas estão afetando a forma como vemos o mundo, está se tornando mais fácil para os grandes
sites espalharem mentiras. Também está se tornando mais fácil machucar outras pessoas online.
Qualquer um pode se esconder atrás de um nome de usuário falso e postar mentiras sobre outras
pessoas. Palihapitiya disse que este era um problema global. Ele está tão preocupado com a mídia
social que proibiu seus filhos de usá-la. No entanto, ele afirmou que o Facebook era uma boa
empresa. Ele disse: "Claro, nem tudo é ruim. O Facebook faz o bem no mundo de forma
esmagadora."

TRAVEL TIPS
How to Plan a Movie-Themed Vacation
It’s easier than you may expect to find, visit, and enjoy the places where your favorite movies
were made.
Lars Leetaru
By Shivani Vora
March 8, 2018
Whether it’s the “Lord of the Rings” trilogy in New Zealand or "Roman Holiday” in Rome, many
noteworthy movies are filmed in appealing locales all over the world that travelers may want to
visit and enjoy.
According to Angela Tillson, a film location manager in Kauai who has worked on the set of films
including "Jurassic Park: The Lost World” and “The Descendants," exploring a beloved movie set
destination through the eyes of the film makes for an enjoyable vacation. "Seeing a place with a
focus on a movie you love will give you a perspective that the average tourist doesn’t usually get.
You’ll certainly have a better impression of the place,” she said. Here are her tips to get started.
Choose Your Destination
If there’s a movie you love, you can find out where it was filmed by looking at the credits at the
end of the film or by going online to The Internet Movie Database, also known as IMDB, which
often lists filming locations. Once you know the locale, you can start planning your trip. Or,
consider doing what Ms. Tillson often does when deciding on where to vacation: pick a spot you’re
interested in visiting, and find out what movies have been filmed there. “It’s fun to sometimes let
a destination determine the movie you&apos;re going to live rather than the other way around,”
Ms. Tillson said.
Get in the Mood
Before you head to your destination, be sure to rewatch the movie. A rewatch not only reminds
you of identifiable spots to look out for during your trip, but it also adds to the excitement of your
upcoming exploration.

AULA 00 – ENGLISH GENERAL PRESENTATION 117


TEACHER ANDREA BELO

If the movie is based on a book, consider reading the book, too. It may have details about the
locale that the movie doesn’t touch on. Also, books often have scenes that don’t make it into the
movie adaptations, which gives you a deeper view of the destination.
Ms. Tillson also recommended downloading the movie’s soundtrack or score, and listening to it
throughout your trip.
Book a Themed Trip
Some travel companies sell set itineraries focused on popular movies. Luxury tour operator
Zicasso, for example, has an eight-day trip, all inclusive, to Ireland inspired by "Star Wars: The Last
Jedi” and Wild Frontiers has an eleven-day trip to India inspired by "The Best Exotic Marigold
Hotel." Ms. Tillson suggested doing a web search or checking with a travel agent to find out about
such trips.
Also, in some destinations, local tour operators and hotels sell movie-themed tours. For instance,
The St. Regis Priceville Resort offers a tour that includes a private helicopter ride to
Manawaiopuna Falls, made famous in "Jurassic Park,” and an ATV tour of filming locations of
movies such as “Raiders of the Lost Ark" and “Pirates of the Caribbean.” Lunch is even included.
The cost is $5,674 for two adults.
A more affordable option, in Rome, is the four-hour “Roman Holiday" themed excursion from HR
Tours, where travelers ride a Vespa with a driver and see all the sites from the movie; the cost is
170 euros per person.
Hang Where the Movie Crew Did
When they’re not working, movie crews enjoy hitting local bars and casual restaurants that serve
tasty local cuisine, Ms. Tillson said.
Find out where the behind-the-scenes staff of your film spent their time by asking your
destination’s tourist board or your hotel’s concierge, and check out a few of the spots. “It’s
another way to get involved in the film and spend time in bars and restaurants that you wouldn’t
normally think to hit,” she said.

DICAS DE VIAGEM
Como planejar férias com tema de filme
É mais fácil do que você espera encontrar, visitar e desfrutar dos lugares onde seus filmes
favoritos foram feitos.
Lars Leetaru
Por Shivani Vora
8 de março de 2018
Quer seja a trilogia "O Senhor dos Anéis" na Nova Zelândia ou "Férias Romanas" em Roma, muitos
filmes notáveis são filmados em locais atraentes em todo o mundo que os viajantes podem querer
visitar e desfrutar.
De acordo com Angela Tillson, gerente de locação de filmes em Kauai que trabalhou no set de
filmes, incluindo "Jurassic Park: The Lost World" e "The Descendants", explorar um destino de set
de filmagem amado pelos olhos do filme é uma experiência agradável férias. “Ver um lugar com
foco em um filme que você adora lhe dará uma perspectiva que o turista comum não costuma
ter. Você certamente terá uma impressão melhor do lugar”, disse ela. Aqui estão suas dicas para
começar .

AULA 00 – ENGLISH GENERAL PRESENTATION 118


TEACHER ANDREA BELO

Escolha o seu destino


Se há um filme que você ama, você pode descobrir onde ele foi filmado olhando os créditos no
final do filme ou acessando o Internet Movie Database, também conhecido como IMDB, que
geralmente lista os locais de filmagem. Depois de saber o local, você pode começar a planejar sua
viagem. Ou considere fazer o que a Sra. Tillson costuma fazer ao decidir onde passar as férias:
escolha um local que você está interessado em visitar e descubra quais filmes foram filmados lá.
“Às vezes é divertido deixar um destino determinar o filme que você vai viver, e não o contrário”,
disse Tillson.
Entre no clima
Antes de se dirigir ao seu destino, certifique-se de assistir novamente ao filme. Uma nova
observação não apenas o lembra de pontos identificáveis a serem observados durante sua
viagem, mas também aumenta a emoção de sua próxima exploração.
Se o filme for baseado em um livro, considere a possibilidade de ler o livro também. Pode haver
detalhes sobre o local que o filme não menciona. Além disso, os livros costumam ter cenas que
não fazem parte das adaptações para o cinema, o que lhe dá uma visão mais aprofundada do
destino.
A Sra. Tillson também recomendou baixar a trilha sonora ou trilha sonora do filme e ouvi-la
durante a viagem.
Reserve uma viagem temática
Algumas agências de viagens vendem itinerários definidos com foco em filmes populares. A
operadora de turismo de luxo Zicasso, por exemplo, tem uma viagem de oito dias, com tudo
incluído, para a Irlanda inspirada em "Star Wars: The Last Jedi" e Wild Frontiers tem uma viagem
de onze dias para a Índia inspirada em "The Best Exotic Marigold Hotel. "A Sra. Tillson sugeriu
fazer uma pesquisa na web ou verificar com um agente de viagens para saber mais sobre essas
viagens.
Além disso, em alguns destinos, os operadores turísticos locais e os hotéis vendem passeios
temáticos de filmes. Por exemplo, o St. Regis Priceville Resort oferece um passeio que inclui um
passeio de helicóptero particular para Manawaiopuna Falls, que ficou famosa em "Jurassic Park",
e um passeio de quadriciclo pelos locais de filmagem de filmes como "Os Caçadores da Arca
Perdida" e " Piratas do Caribe." O almoço está incluído. O custo é de US $ 5.674 para dois adultos.
Uma opção mais acessível, em Roma, é a excursão temática de “Férias Romanas” de quatro horas
da HR Tours, onde os viajantes andam de Vespa com um motorista e veem todos os sites do filme;
o custo é de 170 euros por pessoa.
Fique onde a equipe do filme fez
Quando não estão trabalhando, as equipes de cinema aproveitam os bares locais e restaurantes
casuais que servem a saborosa culinária local, disse Tillson.
Descubra onde a equipe de bastidores de seu filme passou o tempo, perguntando ao conselho de
turismo de seu destino ou ao concierge do seu hotel e confira alguns dos locais. “É outra maneira
de se envolver no filme e passar o tempo em bares e restaurantes que você normalmente não
pensaria em ir”, disse ela.

AULA 00 – ENGLISH GENERAL PRESENTATION 119


TEACHER ANDREA BELO

Paul Thompson is a student and he has a part-time job, too. He lives in London and goes to
university there. Paul works at an Italian restaurant called Mamma Mia. He’s a waiter there. He
likes his job very much but he doesn’t like to work on the weekends. When he is free, he likes to
go to the movies. He recently watched “Avengers – Infinity War”, a superhero movie that was a
huge success.

Paul Thompson é estudante e também tem um emprego de meio período. Ele mora em Londres
e vai para a universidade lá. Paul trabalha em um restaurante italiano chamado Mamma Mia. Ele
é um garçom lá. Ele gosta muito de seu trabalho, mas não gosta de trabalhar nos fins de semana.
Quando está livre, gosta de ir ao cinema. Recentemente, ele assistiu a “Avengers - Infinity War”,
um filme de super-heróis que foi um grande sucesso.

“The Little Prince”, now, is a movie – Alex Weiss


An all-time favorite children’s book, The Little Prince by Antoine de Saint-Exupéry has been turned
into a beautifully animated movie – and it’s finally being released. The inspiring lessons, timeless
story and beautiful quotes from The Little Prince make this a perfect choice for an on-screen
adaptation. At one point to another, your parents read this book to you and when you were a
child and then you picked it up later on in life, realizing how incredibly important this small book
truly is.
(Adapted from bustle.com)

“O Pequeno Príncipe”, agora, é um filme - Alex Weiss


Um livro infantil favorito de todos os tempos, O Pequeno Príncipe, de Antoine de Saint-Exupéry,
foi transformado em um lindo filme de animação - e finalmente está sendo lançado. As lições
inspiradoras, a história atemporal e as belas citações de O Pequeno Príncipe fazem desta uma
escolha perfeita para uma adaptação na tela. De um ponto a outro, seus pais leram este livro para
você e quando você era criança e então você o aprendeu mais tarde na vida, percebendo o quão
incrivelmente importante este pequeno livro realmente é.

Price Tag
Price Tag – Jessie J.
Seems like everybody's got a price
I wonder how they sleep at night
When the sale comes first
And the truth comes second
Just stop for a minute and smile
Why is everybody so serious
Acting so damn mysterious

AULA 00 – ENGLISH GENERAL PRESENTATION 120


TEACHER ANDREA BELO

Got shades on your eyes


And your heels so high
That you can't even have a good time
Everybody look to the left
Everybody look to the right
Can you feel that yeah
We're paying with love tonight
It's not about the money money money
We don't need your money money money
We just wanna make the world dance
Forget about the price tag
Ain't about the uh cha-ching cha-ching
Ain't about the yeah b-bling b-bling
Wanna make the world dance
Forget about the price tag
(letras.mus.br)

Etiqueta de preço
Etiqueta de preço - Jessie J.
parece que todo mundo tem um preço
Eu me pergunto como eles dormem à noite
Quando a venda vem primeiro
E a verdade vem em segundo lugar
Só pare por um minuto e sorria
Porque todos estão tão sérios
Agindo tão misteriosamente
Tem sombras em seus olhos
E seus saltos tão altos
Que você não pode nem mesmo se divertir
Todos olham para a esquerda
Todos olham para a direita
Você pode sentir isso sim
Estamos pagando com amor esta noite
Não se trata de dinheiro, dinheiro, dinheiro
Não precisamos do seu dinheiro, dinheiro, dinheiro
Nós só queremos fazer o mundo dançar
Esqueça o preço
Não é sobre o uh cha-ching cha-ching
Não é sobre o sim b-bling b-bling
Quero fazer o mundo dançar
Esqueça o preço

AULA 00 – ENGLISH GENERAL PRESENTATION 121


TEACHER ANDREA BELO

In The End – Linkin Park


One thing, I don't know why
It doesn't even matter how hard you try
Keep that in mind
I designed this rhyme
To remind myself of a time when
I tried so hard
In spite of the way you were mocking me
Acting like I was part of your property
Remembering all the times you fought with me
I'm surprised it got so
Things aren't the way they were before
You wouldn't even recognize me anymore
Not that you knew me back then
But it all comes back to me in the end
You kept everything inside
And even though I tried, it all fell apart
What it meant to me will eventually be a memory of a time when
I tried so hard
And got so far
But in the end
It doesn't even matter
I had to fall
To lose it all
But in the end
It doesn't even matter
I've put my trust in you
Pushed as far as I can go
For all this
There's only one thing you should know
I've put my trust in you
Pushed as far as I can go
For all this
There's only one thing you should know
https://www.vagalume.com.br/linkin-park/in-the-end.html

AULA 00 – ENGLISH GENERAL PRESENTATION 122


TEACHER ANDREA BELO

No Fim - Linkin Park


Uma coisa nao sei porque
Nem importa o quanto você tente
Tenha isso em mente
Eu desenhei esta rima
Para me lembrar de uma época em que
Eu tentei tanto
Apesar do jeito que você estava zombando de mim
Agindo como se eu fosse parte de sua propriedade
Lembrando de todas as vezes que você lutou comigo
Estou surpreso que tenha ficado tão
As coisas não são como eram antes
Você nem me reconheceria mais
Não que você me conhecesse naquela época
Mas tudo volta para mim no final
Voce manteve tudo dentro
E mesmo que eu tenha tentado, tudo desmoronou
O que isso significou para mim acabará sendo uma lembrança de uma época em que
Eu tentei tanto
E foi tão longe
Mas no final
Nem importa
Eu tive que cair
Para perder tudo
Mas no final
Nem importa
Eu coloquei minha confiança em você
Empurrado o mais longe que posso
Por tudo isso
Só há uma coisa que você deve saber
Eu coloquei minha confiança em você
Empurrado o mais longe que posso
Por tudo isso
Só há uma coisa que você deve saber

AULA 00 – ENGLISH GENERAL PRESENTATION 123


TEACHER ANDREA BELO

STRANGE 'ICE VOLCANOES' ERUPT NEAR LAKE SUPERIOR, MICHIGAN


The National Weather Service (NWS) in Grand Rapids, Michigan, captured a photo of two "ice
volcanoes" erupting on Oval Beach in Saugatuck, near Lake Michigan, over the weekend.
"It was a great day to visit the beach and watch the waves interact with the ice," NWS tweeted.
Volcanoes and ice may sound like an unusual combination but according to Michigan Tech, they
are a regular occurrence on the north shore of Lake Superior, north of Lake Michigan, in the winter
months and can range in size from less than a meter to more than eight.
An ice volcano requires three things—high surf, cool temperatures and ice. For an ice volcano to
form, wave activity must be at least a meter tall at their highest. According to Michigan Tech,
anything less is too small to create the volcano's cone.
These volcanoes form at the edge of the ice shelf, caused by the movement of high surf hitting
the face of the formation. Small valleys can cause the energy of the waves to concentrate,
prompting larger waves to form that eject a spatter onto the ice shelf in certain places.
If the cones remain active but become completely enclosed by the ice shelf, wave energy from
below can push water up through any cracks or openings.
Ice volcanoes are typically found in arcs along the shoreline. They are often evenly spaced apart
but can also form individually. Michigan Tech refers to these lone structures as "cold spot" ice
volcanoes, and describes them as "nearly symmetrical single cones that apparently are not related
to shoreline, sand bar or rock reef arcs."
These solitary cones form near the shore and are thought to be a result of a weakness in the ice.
However, they have not yet been observed in an active state.
The phenomena has been observed on the shores of Lake Erie.
In Duluth, Minnesota, a human-made incarnation was created by a spewing fire hydrant.
According to Lake Superior Duluth Streams, the city allowed Orange Street's fire hydrant to spray
water during the winter months to prevent the supply pipe freezing over.This is not the only type
of ice volcano that exists in the solar system. Evidence of ice volcanoes (or cryovolcanoes) have
been found on Ceres, Titan and Pluto. These differ from the volcanoes we have on Earth. Instead
of ejecting molten rock, they emit substances such as water, ammonia and methane.
"To put them in perspective, if Mount Vesuvius had been a cryovolcano, its lava would have frozen
the residents of Pompeii," Rosaly Lopes, a Cassini radar team investigation scientist from NASA,
said in a statement in 2008.
Adapted from https://www.newsweek.com/ice-volcanoes-lake-superior-michigan-1487618

ESTRANHO 'VULCÃO DE GELO' ERUPTO PERTO DO LAGO SUPERIOR, MICHIGAN


O Serviço Meteorológico Nacional (NWS) em Grand Rapids, Michigan, tirou uma foto de dois
"vulcões de gelo" em erupção na Praia Oval em Saugatuck, perto do Lago Michigan, no fim de
semana.
“Foi um ótimo dia para visitar a praia e ver as ondas interagirem com o gelo”, tuitou NWS.
Vulcões e gelo podem soar como uma combinação incomum, mas de acordo com Michigan Tech,
eles são uma ocorrência regular na costa norte do Lago Superior, ao norte do Lago Michigan, nos
meses de inverno e podem variar em tamanho de menos de um metro a mais de oito.

AULA 00 – ENGLISH GENERAL PRESENTATION 124


TEACHER ANDREA BELO

Um vulcão de gelo requer três coisas - ondas altas, temperaturas frias e gelo. Para que um vulcão
de gelo se forme, a atividade das ondas deve ter pelo menos um metro de altura em seu ponto
mais alto. De acordo com Michigan Tech, qualquer coisa menos é muito pequena para criar o
cone do vulcão.
Esses vulcões se formam na borda da plataforma de gelo, causados pelo movimento das ondas
altas que atingem a face da formação. Vales pequenos podem fazer com que a energia das ondas
se concentre, fazendo com que ondas maiores se formem e ejetem respingos na plataforma de
gelo em certos lugares.
Se os cones permanecerem ativos, mas ficarem completamente fechados pela plataforma de
gelo, a energia das ondas de baixo pode empurrar a água para cima através de quaisquer fendas
ou aberturas.
Vulcões de gelo são normalmente encontrados em arcos ao longo da costa. Freqüentemente, eles
estão uniformemente espaçados, mas também podem se formar individualmente. Michigan Tech
refere-se a essas estruturas solitárias como vulcões de gelo de "ponto frio" e as descreve como
"cones únicos quase simétricos que aparentemente não estão relacionados com a linha da costa,
barra de areia ou arcos de recife de rocha."
Esses cones solitários se formam perto da costa e são considerados o resultado de uma fraqueza
no gelo. No entanto, eles ainda não foram observados em um estado ativo.
O fenômeno foi observado nas margens do Lago Erie.
Em Duluth, Minnesota, uma encarnação de fabricação humana foi criada por um hidrante de
incêndio. De acordo com o Lago Superior Duluth Streams, a cidade permitiu que o hidrante da
Orange Street borrifasse água durante os meses de inverno para evitar que o tubo de
abastecimento congelasse. Este não é o único tipo de vulcão de gelo que existe no sistema solar.
Evidências de vulcões de gelo (ou criovulcões) foram encontradas em Ceres, Titã e Plutão. Eles
diferem dos vulcões que temos na Terra. Em vez de ejetar rocha derretida, eles emitem
substâncias como água, amônia e metano.
"Para colocá-los em perspectiva, se o Monte Vesúvio fosse um criovulcão, sua lava teria congelado
os residentes de Pompéia", disse Rosaly Lopes, uma cientista de investigação da equipe de radar
da Cassini da NASA, em um comunicado em 2008.

The housing crisis and garden villages


In this piece, we take a look at the potential impact of garden villages, including regional
implications, what the result might be for new buyers, and follow on trends
There are many reports of a UK housing crisis, but what efforts are being put into resolving the
issue?
What do we mean by a garden village?
By definition, it is a piece of brownfield land that is used to develop new areas for families and
businesses. They are usually smaller projects and can contain from 1,500 to 10,000 homes. Often,
garden villages have their own facilities — such as schools, shops and transport stations — which

AULA 00 – ENGLISH GENERAL PRESENTATION 125


TEACHER ANDREA BELO

makes this type of living space perfect for families and first-time buyers looking to lead the
picture-perfect life.
Garden villages allow residents to imprint their own mark on the area, including creating its own
identity and creating rules. However, there are a few ways to identify them. They must be a
settlement outside of an existing town or city and not closely attached. The British government is
currently supporting 17 locations around the country, with £6 million expected to go towards
funding 14 new garden villages and £1.4 million to support three garden towns (which are similar
to garden villages, only larger).
Looking at the potential regional impact:
Regions are set to see more manual work available in these regions as such areas will be creating
more than 50,000 homes. This will help to boost the economy, as it will provide people with more
jobs in the area.
As new homes are built, people will be buying them — this potentially means more residents
within the region. There is a popular misconception that this will put a strain on the resources of
current residents nearby, such as school places for their children and obtaining doctor
appointments. However, this is not the case, as garden villages are built with their own facilities
including schools and general practices. In turn, this will also create more jobs in the area of
development.
However, it’s important to remember that these locations usually have their own transport links.
On the other hand, more cars on the road could cause congestion.
Adapted from https://www.openaccessgovernment.org/housing-crisis-garden-villages/68975/

A crise habitacional e as aldeias-jardim


Neste artigo, damos uma olhada no impacto potencial das aldeias-jardim, incluindo
implicações regionais, qual pode ser o resultado para novos compradores e acompanhamos as
tendências
Existem muitos relatos de uma crise imobiliária no Reino Unido, mas que esforços estão sendo
feitos para resolver o problema?
O que queremos dizer com vila-jardim?
Por definição, é um terreno brownfield usado para desenvolver novas áreas para famílias e
empresas. Geralmente são projetos menores e podem conter de 1.500 a 10.000 casas.
Freqüentemente, as aldeias-jardim têm suas próprias instalações - como escolas, lojas e estações
de transporte - o que torna esse tipo de espaço vital perfeito para famílias e compradores de
primeira viagem que procuram levar uma vida perfeita.
As aldeias-jardim permitem que os residentes imprimam suas próprias marcas na área, incluindo
a criação de sua própria identidade e a criação de regras. No entanto, existem algumas maneiras
de identificá-los. Eles devem ser um assentamento fora de uma cidade ou vila existente e não
estreitamente vinculadas. O governo britânico está atualmente apoiando 17 localidades em todo
o país, com £ 6 milhões esperados para financiar 14 novas aldeias-jardim e £ 1,4 milhão para
apoiar três cidades-jardim (que são semelhantes às aldeias-jardim, só que maiores).
Olhando para o impacto regional potencial:

AULA 00 – ENGLISH GENERAL PRESENTATION 126


TEACHER ANDREA BELO

As regiões estão definidas para ver mais trabalho manual disponível nessas regiões, pois essas
áreas criarão mais de 50.000 casas. Isso ajudará a impulsionar a economia, pois proporcionará
mais empregos às pessoas na região.
À medida que novas casas são construídas, as pessoas as compram - isso potencialmente significa
mais residentes na região. Há um equívoco popular de que isso vai sobrecarregar os recursos dos
atuais residentes nas proximidades, como vagas na escola para seus filhos e obtenção de
consultas médicas. No entanto, este não é o caso, uma vez que as aldeias-jardim são construídas
com instalações próprias, incluindo escolas e consultórios gerais. Por sua vez, isso também criará
mais empregos na área de desenvolvimento.
No entanto, é importante lembrar que esses locais geralmente têm suas próprias conexões de
transporte. Por outro lado, mais carros na estrada podem causar congestionamento.

MARITIME AUTONOMY: A BRIDGE TOO FAR


Imagine ________ ships en route from departure port ‘A’ to arrive at different ports around the
world, each at a given time. These ships have no captain or crew _______ board and can navigate,
dock, load, unload and refuel on their own and are maintained by sensors, robots and drones. The
ships are navigated and controlled by computers from a fully automated port with no human
intervention or interaction. The operating system of each vessel makes decisions and takes
actions based on the situation it is _______. That’s ‘autonomous shipping’ and it is very different
from automated ships.
Adapted from https://www.oneseaecosystem.net/maritime-autonomy-a-bridge-too-far/

AUTONOMIA MARÍTIMA: UMA PONTE DEMASIADA


Imagine ________ navios em rota do porto de partida 'A' para chegar a diferentes portos ao redor
do mundo, cada um em um determinado momento. Esses navios não têm capitão ou tripulação
_______ bordo e podem navegar, atracar, carregar, descarregar e reabastecer por conta própria
e são mantidos por sensores, robôs e drones. Os navios são navegados e controlados por
computadores a partir de uma porta totalmente automatizada, sem intervenção ou interação
humana. O sistema operacional de cada embarcação toma decisões e executa ações com base na
situação em que é _______. Isso é ‘transporte autônomo’ e é muito diferente de navios
automatizados.

Over 100,000 Flamingos Reportedly Descend on Mumbai Amid India Strict Coronavirus
Lockdown
As people around the world stay inside to fight the spread of coronavirus, wild animals have begun
to enjoy the newfound space. That appears to be what’s happened in the metropolitan region of
Mumbai, India, where a record-breaking number of flamingos have migrated, painting the
wetlands pink, according to local reports.

AULA 00 – ENGLISH GENERAL PRESENTATION 127


TEACHER ANDREA BELO

The Bombay Natural History Society (BNHS) estimates that around 25% more flamingos have
migrated to the region compared to last year, likely caused by the decrease in human activity, the
Hindustan Times reports. The Science Times calculates around 150,000 flamingos have come to
the area.
India has been under a strict lockdown since March 25, which has required over a billion people
to stay home and shutter all but essential services, in an attempt to stop the spread of COVID-19,
the disease caused by novel coronavirus. On May 1, the Indian government extended the
lockdown through May 18.
Flamingos typically migrate to wetlands of the metropolitan region of Mumbai from November to
May, Rhul Khot, the assistant director of the BNHS, told the Times. But this year’s lockdown “is
giving these birds peace for roosting, no disturbance in their attempt to obtain food and overall
encouraging habitat,” Deepak Apte, the director of the BNHS, explained, per Times. He added that
the increase is also likely tied to a successful breeding season two years ago, as well as the
destruction of wetlands on India’s eastern seafront that could be pushing the birds to the Mumbai
region, per the Times.
Khot also told the Times that an increase in “domestic sewage” from people staying at home
during the lockdown “is helping the undisturbed formation of planktons, algae and microbenthos
formation, which forms the food for flamingos and other wetland birds.”
As the coronavirus pandemic has continued, animals have been spotted throughout the world in
places that are usually dominated by humans. CNN reports that dolphins have swam in India’s
Ganges river for the first time in years, and hundreds of monkeys have “descended” on Delhi as
the city remains under lockdown. Cougars were reportedly spotted prowling the streets in
Santiago, Chile, in April; wild Kashmiri goats meandered through on the town of Llandudno,
Wales, in March.
“Residents are cooped up at home spending their mornings and evenings at their balconies taking
photographs and videos of these relaxed birds,” Sunil Agarwal, a resident of Seawoods in Navi
Mumbai, told the Times about the flamingos. “The lockdown will at least prompt people to focus
on what is around them, which they had been taking for granted, and hopefully this site will be
declared a flamingo sanctuary soon.”
(Adapted from https://time.com/5831198/flamingos-coronavirus/)

Mais de 100.000 flamingos supostamente descem em Mumbai em meio ao bloqueio estrito do


coronavírus na Índia
Enquanto as pessoas ao redor do mundo permanecem lá dentro para lutar contra a disseminação
do coronavírus, os animais selvagens começam a aproveitar o espaço recém-descoberto. Isso
parece ser o que aconteceu na região metropolitana de Mumbai, Índia, para onde um número
recorde de flamingos migrou, pintando os pântanos de rosa, de acordo com relatórios locais.
A Bombay Natural History Society (BNHS) estima que cerca de 25% mais flamingos migraram para
a região em comparação com o ano passado, provavelmente causado pela diminuição da
atividade humana, relata o Hindustan Times. O Science Times calcula que cerca de 150.000
flamingos chegaram à área.

AULA 00 – ENGLISH GENERAL PRESENTATION 128


TEACHER ANDREA BELO

A Índia está sob um bloqueio estrito desde 25 de março, o que exigiu que mais de um bilhão de
pessoas fiquem em casa e fechem todos os serviços, exceto os essenciais, em uma tentativa de
impedir a disseminação da COVID-19, a doença causada pelo novo coronavírus. Em 1º de maio, o
governo indiano estendeu o bloqueio até 18 de maio.
Os flamingos normalmente migram para os pântanos da região metropolitana de Mumbai de
novembro a maio, disse ao Times Rhul Khot, diretor assistente do BNHS. Mas o bloqueio deste
ano "está dando a essas aves paz para empoleirar-se, nenhuma perturbação em sua tentativa de
obter comida e habitat encorajador em geral", explicou Deepak Apte, diretor do BNHS, por Times.
Ele acrescentou que o aumento provavelmente também está relacionado a uma temporada de
reprodução bem-sucedida há dois anos, bem como à destruição de pântanos na orla marítima
oriental da Índia que podem estar empurrando as aves para a região de Mumbai, de acordo com
o Times.
Khot também disse ao Times que um aumento no “esgoto doméstico” de pessoas que ficaram em
casa durante o bloqueio “está ajudando a formação de plânctons, algas e microbentos, que
constituem a comida para flamingos e outras aves pantanosas”.
Como a pandemia de coronavírus continuou, animais foram vistos em todo o mundo em lugares
que geralmente são dominados por humanos. A CNN relata que golfinhos nadaram no rio Ganges,
na Índia, pela primeira vez em anos, e centenas de macacos “desceram” em Delhi enquanto a
cidade permanece sob bloqueio. Segundo consta, pumas foram vistos rondando as ruas de
Santiago, Chile, em abril; cabras selvagens da Caxemira vagavam pela cidade de Llandudno, País
de Gales, em março.
“Os moradores ficam confinados em casa, passando as manhãs e as noites em suas varandas,
tirando fotos e vídeos desses pássaros relaxados”, disse Sunil Agarwal, residente de Seawoods em
Navi Mumbai, ao Times sobre os flamingos. “O bloqueio pelo menos fará com que as pessoas se
concentrem no que está ao seu redor, o que elas consideravam certo e, com sorte, este local será
declarado um santuário de flamingo em breve.”

It’s boom time for podcasts – but will going mainstream kill the magic?
Fifteen years ago, when the word podcast was added to the dictionary, only the tech-savvy
were listening. Now, as star names pile in, they’re big business. Can the quality survive?
Hello friends! Do you fancy listening to “a new type of time-shifted amateur radio”? No? How
about a brilliant podcast? Of course you do.
Fifteen years ago, Macworld, a magazine for fans of Apple products, announced, with limited
fanfare, that Apple was about to add podcasts to iTunes, its music download offer. Unfortunately,
few readers knew what a podcast was, hence Macworld’s “time-shifted radio” definition. In June
2005, the idea of having thousands of ready-to-hear audio shows, anything from true-crime
documentaries to all-chums-together comedy, to up-to-the-minute news to gripping drama to
revealing interviews, and being able to listen to these shows whenever you want, wherever you
are – well, that wasn’t quite happening. So Apple’s move didn’t seem important. Nor did the fact
that the Oxford English Dictionary added “podcast” to its lexicon in the same year, after tech
journalist Ben Hammersley came up with the term in 2004 (which was also the year the BBC

AULA 00 – ENGLISH GENERAL PRESENTATION 129


TEACHER ANDREA BELO

launched a downloadable version of In Our Time). Podcasts were new. It takes time for the new
to become everyday.
Podcasts were mostly unheard of, except by the tech savvy. They were either downloadable
versions of existing radio shows or they were chatty riffs, made by amateurs who knew how to
upload their aural blogs online. Still, they were interesting. At least to me. Soon after podcasts’
iTunes debut, I started a new job as the Observer’s radio critic. Great job – except there was a
limited choice of programmes for me to review. Radio schedules rarely changed. Presenters
stayed in their jobs for years. The BBC dominated speech radio, aside from phone-ins; hardly any
other broadcaster had the money to make documentaries or drama.
Podcasts rescued me from aural monotony; I wrote my first piece about them in the summer of
2006. Apparently Coke Machine Glow and The Dawn and Drew Show were the ones to look out
for (me neither, now). The podcast I do recall from then is The Ricky Gervais Show; this dominated
the brand new iTunes podcast chart for weeks. Initially free, in early 2006 it switched to a pay-
per-listen model and proved both a forerunner and an outlier: since then, much podcast uptake
has been driven by comedy, but most shows are still free to listeners, paid for by adverts that
appear during episodes.
Today, the iTunes podcast chart is bustling with old hands and new kids on the block. Here are No
Such Thing As a Fish, Fearne Cotton’s Happy Place, That Peter Crouch Podcast, Katherine Ryan:
Telling Everybody Everything. Here are sections for new and noteworthy, cultivating calm, keeping
the kids busy. There are lists of the top 10 episodes, top 10 shows; all aside from the 19 other
regular categories: news, arts, true crime… So many podcasts! There are oodles of shows, too
many to ever get through.
(Adapted from https://www.theguardian.com/tv-and-radio/2020/may/03/its-boom-time-for-podcasts-but-will-going-mainstream-kill-the-magic)

É a hora do boom para podcasts – mas tornar-se mainstream vai matar a magia?
Quinze anos atrás, quando a palavra podcast foi adicionada ao dicionário, apenas os
conhecedores de tecnologia estavam ouvindo. Agora, conforme os nomes das estrelas se
acumulam, eles são um grande negócio. A qualidade pode sobreviver?
Olá amigos! Você gosta de ouvir “um novo tipo de rádio amador que mudou no tempo”? Não?
Que tal um podcast brilhante? Claro que você faz.
Quinze anos atrás, a Macworld, uma revista para fãs de produtos da Apple, anunciou, com alarde
limitado, que a Apple estava prestes a adicionar podcasts ao iTunes, sua oferta de download de
músicas. Infelizmente, poucos leitores sabiam o que era um podcast, daí a definição de "rádio
temporário" da Macworld. Em junho de 2005, a ideia de ter milhares de programas de áudio
prontos para ouvir, desde documentários de crimes verdadeiros a comédias para todos, notícias
atualizadas, dramas emocionantes, entrevistas reveladoras e ser capaz de ouvir esses programas
sempre que quiser, onde quer que esteja - bem, isso não estava acontecendo exatamente.
Portanto, a mudança da Apple não parecia importante. Nem o fato de o Oxford English Dictionary
ter adicionado "podcast" ao seu léxico no mesmo ano, depois que o jornalista de tecnologia Ben
Hammersley criou o termo em 2004 (que também foi o ano em que a BBC lançou uma versão
para download de In Our Time) . Os podcasts eram novos. Leva tempo para que o novo se torne
todos os dias.
Podcasts eram praticamente inéditos, exceto pelos conhecedores de tecnologia. Eles eram
versões para download de programas de rádio existentes ou riffs tagarelas, feitos por amadores

AULA 00 – ENGLISH GENERAL PRESENTATION 130


TEACHER ANDREA BELO

que sabiam como fazer upload de seus blogs auditivos online. Ainda assim, eles eram
interessantes. Pelo menos para mim. Logo após a estreia dos podcasts no iTunes, comecei um
novo trabalho como crítico de rádio do Observer. Ótimo trabalho - exceto que havia uma escolha
limitada de programas para eu revisar. Os horários das rádios raramente mudavam. Os
apresentadores permaneceram em seus empregos por anos. A BBC dominou o rádio de fala, além
dos telefonemas; dificilmente qualquer outra emissora tinha dinheiro para fazer documentários
ou dramas.
Os podcasts me resgataram da monotonia auditiva; Escrevi meu primeiro artigo sobre eles no
verão de 2006. Aparentemente, Coke Machine Glow e The Dawn and Drew Show eram os únicos
a se cuidar (eu também não, agora). O podcast de que me lembro então é The Ricky Gervais Show;
isso dominou a nova parada de podcast do iTunes por semanas. Inicialmente gratuito, no início
de 2006, ele mudou para um modelo de pagamento para ouvir e provou ser um precursor e um
outlier: desde então, muito a aceitação de podcast foi impulsionada pela comédia, mas a maioria
dos programas ainda são gratuitos para ouvintes, pagos por anúncios que aparecem durante os
episódios.
Hoje, a parada de podcast do iTunes está repleta de veteranos e novos alunos. Aqui não existe tal
coisa como um peixe, o lugar feliz de Fearne Cotton, aquele podcast de Peter Crouch, Katherine
Ryan: contando tudo a todos. Aqui estão seções para novidades e notáveis, cultivando a calma,
mantendo as crianças ocupadas. Existem listas dos 10 melhores episódios, dos 10 melhores
programas; tudo além das 19 outras categorias regulares: notícias, artes, crime verdadeiro ...
Tantos podcasts! Há uma grande quantidade de programas, muitos para serem exibidos.

The Dunning-Kruger Effect


The Dunning-Kruger effect is a type of cognitive bias in which people believe that they are
smarter and more capable than they really are. Essentially, low ability people do not possess the
skills needed to recognize their own incompetence. The combination of poor self-awareness and
low cognitive ability leads them to overestimate their own capabilities.
The term lends a scientific name and explanation to a problem that many people immediately
recognize—that fools are blind to their own foolishness. As Charles Darwin wrote in his book
The Descent of Man, "Ignorance more frequently begets confidence than does knowledge."
(Adapted from https://www.verywellmind.com/an-overview-of-the-dunning-kruger-effect-4160740)

O efeito Dunning-Kruger
O efeito Dunning-Kruger é um tipo de viés cognitivo em que as pessoas acreditam que são mais
inteligentes e mais capazes do que realmente são. Essencialmente, pessoas com baixa
habilidade não possuem as habilidades necessárias para reconhecer sua própria incompetência.
A combinação de baixa autoconsciência e baixa capacidade cognitiva os leva a superestimar
suas próprias capacidades.
O termo empresta um nome científico e uma explicação para um problema que muitas pessoas
reconhecem imediatamente - que os tolos são cegos para sua própria tolice. Como Charles
Darwin escreveu em seu livro The Descent of Man, "A ignorância gera mais confiança do que o
conhecimento."

AULA 00 – ENGLISH GENERAL PRESENTATION 131


TEACHER ANDREA BELO

What Leads Us to this Belief that Connections to and Experiences with the Natural World Are So Important?
There is an emerging awareness and concern that children are less and less likely to have
experiences that involve the simplest interactions with nature—the plants, animals, and the earth
around them—as a part of their continuous learning process. As we look at the children around
us, we observe them living increasingly unhealthy lifestyles. For many, childhood is spent overly
plugged in and programmed inside their homes, schools, and community settings in humanmade
environments, eliminating the out-of-doors, the benefits of nature, and all that exists in the
natural environment (Wike, 2006).
Even a generation ago, children spent more time outside, because it was the normal thing for
children to do. Adults did not question the value of time spent out-of-doors and had much less
anxiety about the risks involved. Children walked and played outdoors and planted things in the
dirt; they rode their bikes, invented games, and spent the majority of their time in less structured
activities and natural environments. Very young children carried out these activities in their yards
and immediate neighborhoods. Urban environments offered the occasional playground and
vacant lot. Older children roamed beyond their neighborhoods to adjacent lands, streams, woods,
or urban parks. Exposure to the natural world brought opportunities for children to make sense
of their surroundings and to develop their own sense of “place.”
Over a relatively short time, we adults have allowed this connection to the natural world to slip
gradually away from children’s lives. Evidence of this trend surrounds us:
Children now spend nearly 30 hours a week watching a TV or computer screen, listening to
something through headphones or, for older children, using cell phones or media players;
Children experience increasingly timed and structured family lifestyles with less emphasis on
unstructured outside time. Particularly in more densely populated countries, urban growth has
eliminated green spaces and natural environments. Fewer families are vacationing in national
parks (Fish, 2007). Together these changes keep many children separated from nature and
without time for solitude or um-programmed experiences.
Instructional time outside, recess, or unstructured playtime is being eliminated from the school
day (Clements, 2007).
Outdoor play spaces or playgrounds have become safer at some sites due to national playground
standards, but in other locations, play spaces have been eliminated or often lack the natural
elements that encourage a different kind of interaction among children (Moore & Wong, 1997).
The curriculum for children in centers and schools is becoming narrower, with more time spent
on teacher-directed lessons and testing and less time spent investigating and learning through
activities that build on a child’s sense of wonder, curiosity, and the benefit of first-hand
experiences (Hyson, 2003; McMurrer, 2007; Marcon, 1999).
Consequently, we have gradually found our children growing up in a clash of optimal and minimal
learning opportunities. Optimally, technology opens worlds never before so readily available to
children; however, the opening of this side of learning has contributed to shutting the door to
children’s access to the more natural environment that gives a lasting attachment to children’s
sense of place and their awareness of the habitat and environment nearest to them. This lack of
connection can engender both apathy and ignorance in children’s early perceptions of the world
around them and their roles in enjoying, learning from, and protecting it.
(Adapted from: https://www.education.ne.gov/wpcontent/uploads/2017/07/Call_to_Action.pdf)

AULA 00 – ENGLISH GENERAL PRESENTATION 132


TEACHER ANDREA BELO

O que nos leva a essa crença de que as conexões e as experiências com o mundo natural são tão importantes?
Há uma consciência emergente e preocupação de que as crianças são cada vez menos propensas
a ter experiências que envolvem as interações mais simples com a natureza - as plantas, os animais
e a terra ao seu redor - como parte de seu processo de aprendizado contínuo. Ao olharmos para
as crianças ao nosso redor, observamos que vivem estilos de vida cada vez mais prejudiciais à
saúde. Para muitos, a infância é passada excessivamente conectada e programada dentro de suas
casas, escolas e ambientes comunitários em ambientes feitos pelo homem, eliminando o exterior,
os benefícios da natureza e tudo o que existe no ambiente natural (Wike, 2006) .
Mesmo uma geração atrás, as crianças passavam mais tempo fora, porque era a coisa normal que
as crianças fizessem. Os adultos não questionam o valor do tempo passado ao ar livre e têm muito
menos ansiedade quanto aos riscos envolvidos. As crianças caminhavam e brincavam ao ar livre e
plantavam coisas na terra; eles andavam de bicicleta, inventavam jogos e passavam a maior parte
do tempo em atividades menos estruturadas e ambientes naturais. Crianças muito pequenas
realizavam essas atividades em seus quintais e vizinhanças imediatas. Ambientes urbanos
ofereciam parquinho ocasional e terreno baldio. As crianças mais velhas vagavam além de seus
bairros para terras adjacentes, riachos, bosques ou parques urbanos. A exposição ao mundo
natural trouxe oportunidades para as crianças entenderem o que está ao seu redor e
desenvolverem seu próprio senso de "lugar".
Em um período relativamente curto, nós, adultos, permitimos que essa conexão com o mundo
natural se afastasse gradualmente da vida das crianças. A evidência dessa tendência nos cerca:
As crianças agora passam cerca de 30 horas por semana assistindo à TV ou à tela do computador,
ouvindo alguma coisa com fones de ouvido ou, para crianças mais velhas, usando telefones
celulares ou reprodutores de mídia;
As crianças experimentam estilos de vida familiares cada vez mais cronometrados e estruturados,
com menos ênfase no tempo não estruturado fora do tempo. Particularmente em países mais
densamente povoados, o crescimento urbano eliminou espaços verdes e ambientes naturais.
Menos famílias estão de férias em parques nacionais (Fish, 2007). Juntas, essas mudanças mantêm
muitas crianças separadas da natureza e sem tempo para solidão ou experiências um-
programadas.
O tempo de instrução fora, recreio ou brincadeiras não estruturadas está sendo eliminado do dia
escolar (Clements, 2007).
Espaços de recreação ao ar livre ou playgrounds tornaram-se mais seguros em alguns locais devido
aos padrões nacionais de playground, mas em outros locais, os espaços de recreação foram
eliminados ou muitas vezes carecem dos elementos naturais que encorajam um tipo diferente de
interação entre as crianças (Moore & Wong, 1997).
O currículo para crianças em centros e escolas está se tornando mais restrito, com mais tempo
gasto em aulas e testes dirigidos por professores e menos tempo gasto investigando e aprendendo
por meio de atividades que aumentam o senso de admiração, curiosidade e o benefício da
experiência de uma criança experiências (Hyson, 2003; McMurrer, 2007; Marcon, 1999).
Consequentemente, temos gradualmente descoberto nossos filhos crescendo em um conflito de
oportunidades de aprendizagem ideais e mínimas. Idealmente, a tecnologia abre mundos nunca
antes tão disponíveis para as crianças; no entanto, a abertura deste lado da aprendizagem

AULA 00 – ENGLISH GENERAL PRESENTATION 133


TEACHER ANDREA BELO

contribuiu para fechar a porta para o acesso das crianças ao ambiente mais natural que dá um
apego duradouro ao senso de lugar das crianças e sua consciência do habitat e do ambiente mais
próximo a elas. Essa falta de conexão pode gerar apatia e ignorância nas primeiras percepções das
crianças sobre o mundo ao seu redor e seus papéis em desfrutar, aprender e protegê-lo.

How to Make the Study of Consciousness Scientifically Tractable


Strangely, modern science was long dominated by the idea that to be scientific means to remove
consciousness from our explanations, in order to be “objective.” This was the rationale behind
behaviorism, a now-dead theory of psychology that took this trend to a perverse extreme.
Behaviorists like John Watson and B.F. Skinner scrupulously avoided any discussion of what their
human or animal subjects thought, intended or wanted, and focused instead entirely on behavior.
Erwin Schrödinger labeled this approach in his 1958 book Mind and Matter, the “principle of
objectivation”. Schrödinger did identify both the problem and the solution. He recognized that
“objectivation” is just a simplification that is a temporary step in the progress of science in
understanding nature.
We are now at the point, it seems to a growing number of thinkers who are finally listening to
Schrödinger, where we must abandon, where appropriate, the principle of objectivation. It is time
for us to employ a “principle of subjectivation” and in doing so understand not just half of reality—
the objective world—but the whole, the external and internal worlds.
The science of consciousness has enjoyed a renaissance in the last couple of decades and the
study of our own minds—consciousness/subjectivity—has finally become a respectable pursuit.
It’s still tricky, however, to determine what kinds of data and what kinds of experiments we should
consider legitimate in the study of consciousness.
Adapted from https://blogs.scientificamerican.com/observations/how-to-make-the-study-of-consciousness-scientifically-tractable/

Como tornar o estudo da consciência cientificamente rastreável


Estranhamente, a ciência moderna foi dominada por muito tempo pela ideia de que ser científico
significa remover a consciência de nossas explicações, para ser "objetivo". Essa era a razão por
trás do behaviorismo, uma teoria da psicologia agora morta que levou essa tendência a um
extremo perverso.
Behavioristas como John Watson e B.F. Skinner evitavam escrupulosamente qualquer discussão
sobre o que seus sujeitos humanos ou animais pensavam, pretendiam ou desejavam e, em vez
disso, concentravam-se inteiramente no comportamento. Erwin Schrödinger rotulou essa
abordagem em seu livro de 1958 Mind and Matter, o “princípio da objetivação”. Schrödinger
identificou o problema e a solução. Ele reconheceu que a “objetivação” é apenas uma
simplificação que é um passo temporário no progresso da ciência na compreensão da natureza.
Estamos agora no ponto, ao que parece um número crescente de pensadores que estão
finalmente ouvindo Schrödinger, onde devemos abandonar, quando apropriado, o princípio da
objetivação. É hora de empregar um “princípio de subjetivação” e, ao fazê-lo, compreender não
apenas metade da realidade - o mundo objetivo - mas o todo, os mundos externo e interno.

AULA 00 – ENGLISH GENERAL PRESENTATION 134


TEACHER ANDREA BELO

A ciência da consciência passou por um renascimento nas últimas décadas e o estudo de nossas
próprias mentes - consciência / subjetividade - finalmente se tornou uma busca respeitável. Ainda
é complicado, no entanto, determinar quais tipos de dados e quais tipos de experimentos
devemos considerar legítimos no estudo da consciência.

What will art look like in 20


The future may be uncertain, but some things are undeniable: climate change, shifting
demographics, geopolitics. The only guarantee is that there will be changes, both wonderful and
terrible. It’s worth considering how artists will respond to these changes, as well as what purpose
art serves, now and in the future.
Reports suggest that by 2040 the impacts of human-caused climate change will be unescapable,
making it the big issue at the centre of art and life in 20 years’ time. Artists in the future will wrestle
with the possibilities of the post-human and post-Anthropocene – artificial intelligence, human
colonies in outer space and potential doom.
The identity politics seen in art around the #MeToo and Black Lives Matter movements will grow
as environmentalism, border politics and migration come even more sharply into focus. Art will
become increasingly diverse and might not ‘look like art’ as we expect. In the future, once we’ve
become weary of our lives being visible online for all to see and our privacy has been all but lost,
anonymity may be more desirable than fame. Instead of thousands, or millions, of likes and
followers, we will be starved for authenticity and connection. Art could, in turn, become more
collective and experiential, rather than individual.
Adapted from http://www.bbc.com/culture/story/20190418-what-will-art-look-like-in-20-years

Como será a arte em 20


O futuro pode ser incerto, mas algumas coisas são inegáveis: mudanças climáticas, mudanças
demográficas, geopolítica. A única garantia é que haverá mudanças, tanto maravilhosas quanto
terríveis. Vale a pena considerar como os artistas responderão a essas mudanças, bem como a que
propósito a arte serve, agora e no futuro.
Os relatórios sugerem que em 2040 os impactos das mudanças climáticas causadas pelo homem
serão inevitáveis, tornando-se o grande problema no centro da arte e da vida em 20 anos. Os
artistas do futuro lutarão com as possibilidades do pós-humano e do pós-Antropoceno -
inteligência artificial, colônias humanas no espaço sideral e destruição potencial.
A política de identidade vista na arte em torno dos movimentos #MeToo e Black Lives Matter
crescerá à medida que o ambientalismo, a política de fronteira e a migração entrarem em foco
ainda mais nitidamente. A arte se tornará cada vez mais diversa e pode não "parecer arte" como
esperamos. No futuro, quando ficarmos cansados de nossas vidas ficarem visíveis online para
todos verem e nossa privacidade tiver sido perdida, o anonimato pode ser mais desejável do que
a fama. Em vez de milhares ou milhões de gostos e seguidores, ficaremos famintos por
autenticidade e conexão. A arte poderia, por sua vez, tornar-se mais coletiva e experiencial, ao
invés de individual.

AULA 00 – ENGLISH GENERAL PRESENTATION 135


TEACHER ANDREA BELO

Internet privacy: the apps that protect you from your apps
Tech companies don’t have favourite songs, but if they __________ (1), they __________ (2) all
pick Radiohead’s Just – “You do it to yourself, you do/ And that’s what really hurts,” they would
croon, staring their users dead in the eye. And strictly speaking, they’d be right: many of the worst
excesses of the industry are, technically, optional. The world isn’t actually a binary choice between
living in a surveillance state and opting out of all technological development since the turn of the
millennium. You can opt out – you just have to know how.
Of course, that knowledge is not always easily acquired, nor is it necessarily easy to apply. So a
new breed of services has arrived to try to help normal users take control of their digital lives.
Companies including Disconnect.
Me and Jumbo act as something like a digital concierge for their users, tweaking privacy settings,
deleting sensitive data and throwing a spanner into the inner workings of surveillance capitalism.
But there’s a Faustian pact involved: to use the privacy apps to their fullest requires handing them
a level of control over your digital life that would be all too easy to abuse – and it’s hard to be
certain that any company can be trusted with information that sensitive.
Most of the companies that track you aren’t open enough for you to even know they’re snooping
on you in the first place.
Adapted from https://www.theguardian.com/technology/2020/feb/16/internet-privacy-settings-apps-to-protect-you-

Privacidade da Internet: os aplicativos que protegem você de seus aplicativos


Empresas de tecnologia não têm músicas favoritas, mas se __________ (1), __________ (2) todos
escolhem Radiohead's Just - “Você faz isso com você mesmo, você faz / E isso é o que realmente
dói”, eles sussurravam, olhando usuários mortos nos olhos. E, estritamente falando, eles estariam
certos: muitos dos piores excessos da indústria são, tecnicamente, opcionais. O mundo não é
realmente uma escolha binária entre viver em um estado de vigilância e optar por todo o
desenvolvimento tecnológico desde a virada do milênio. Você pode optar por sair - você só precisa
saber como.
Claro, esse conhecimento nem sempre é facilmente adquirido, nem é necessariamente fácil de
aplicar. Assim, uma nova geração de serviços chegou para tentar ajudar os usuários normais a
assumir o controle de suas vidas digitais. Empresas incluindo Disconnect.
Eu e Jumbo agimos como algo como um concierge digital para seus usuários, ajustando as
configurações de privacidade, excluindo dados confidenciais e jogando uma chave no
funcionamento interno do capitalismo de vigilância.
Mas há um pacto faustiano envolvido: para usar os aplicativos de privacidade ao máximo requer
dar a eles um nível de controle sobre sua vida digital que seria muito fácil de abusar - e é difícil ter
certeza de que qualquer empresa pode ser confiável com informações que confidencial.
A maioria das empresas que rastreiam você não é aberta o suficiente para você saber que eles
estão te espionando em primeiro lugar.

AULA 00 – ENGLISH GENERAL PRESENTATION 136


TEACHER ANDREA BELO

Brain surgery patient filmed playing violin during operation


A patient at King’s College Hospital in London has played the violin while surgeons operated on
her brain to remove a tumour. The medical team asked Dagmar Turner, 53, to play the violin to
ensure parts of the brain which control delicate hand movements and coordination were not
damaged during the millimetre-precise surgery.
Ms Turner was diagnosed in 2013 with a brain tumour after suffering a seizure during a symphony.
The committed violinist was worried the surgery would mean she would lose the ability to play.
Her tumour was located in the right frontal lobe of her brain, close to an area that controls the
fine movement of her left hand. Professor Keyoumars Ashkan, consultant neurosurgeon at King’s
College Hospital, came up with a plan to operate and reduce the risk.
Prior to surgery, doctors spent two hours carefully mapping her brain to identify areas that were
active when she played the violin and those responsible for controlling language and movement.
Ms Turner played violin while her tumour was removed, while closely monitored by the
anaesthetists and a therapist.
Prof Ashkan said: “We managed to remove over 90 per cent of the tumour, including all the areas
suspicious of aggressive activity, while retaining full function in her left hand.”Ms Turner added:
“The violin is my passion; I’ve been playing since I was 10 years old. The thought of losing my
ability to play was heartbreaking.”
Adapted from https://www.independent.co.uk/news/health/violin-brain-surgery-kings-hospital-nhs-a9342006.html

Paciente de cirurgia cerebral filmado tocando violino durante a operação


Uma paciente do King’s College Hospital, em Londres, tocou violino enquanto cirurgiões operavam
seu cérebro para remover um tumor. A equipe médica pediu a Dagmar Turner, 53, que tocasse
violino para garantir que partes do cérebro que controlam os movimentos delicados das mãos e a
coordenação não fossem prejudicadas durante a cirurgia de precisão milimétrica.
A Sra. Turner foi diagnosticada em 2013 com um tumor cerebral depois de sofrer uma convulsão
durante uma sinfonia. O violinista comprometido temia que a cirurgia significasse que ela perderia
a habilidade de tocar.
Seu tumor estava localizado no lobo frontal direito de seu cérebro, perto de uma área que controla
o movimento delicado de sua mão esquerda. O professor Keyoumars Ashkan, neurocirurgião
consultor do King’s College Hospital, apresentou um plano para operar e reduzir o risco.
Antes da cirurgia, os médicos passaram duas horas mapeando cuidadosamente seu cérebro para
identificar as áreas que estavam ativas quando ela tocava violino e as responsáveis por controlar
a linguagem e o movimento.
A Sra. Turner tocou violino enquanto seu tumor era removido, monitorada de perto pelos
anestesistas e um terapeuta.
A professora Ashkan disse: “Conseguimos remover mais de 90 por cento do tumor, incluindo todas
as áreas suspeitas de atividade agressiva, mantendo todas as funções em sua mão esquerda.” Ms
Turner acrescentou: “O violino é minha paixão; Jogo desde os 10 anos de idade. A ideia de perder
minha habilidade de jogar foi de partir o coração.”

AULA 00 – ENGLISH GENERAL PRESENTATION 137


AFA 2024

SKIMMING AND SCANNING


AND OTHER TECHNIQUES.
IDENTIFYING FALSE COGNATS.

AULA 01

Teacher Andrea Belo

www.estrategiamilitares.com.br www.militares.estrategia.com
TEACHER ANDREA BELO

SUMÁRIO
INTRODUÇÃO 3

SCANNING: INVESTIGAR E RESPONDER 4

SKIMMING: COMPREENDER E SOLUCIONAR 6

SCANNING & SKIMMING: LER E ACHAR AS RESPOSTAS: “JUST DO IT” 9

CURIOSIDADES SCANNING/SKIMMING 12

SCANNING E SKIMMING EM PEQUENOS TEXTOS 15

SCANNING E SKIMMING EM TEXTOS LONGOS 16

COGNATES AND FALSE COGNATES 17

QUESTÕES 23

GABARITO 57

QUESTÕES COMENTADAS 58

CONSIDERAÇÕES FINAIS 121

REFERÊNCIAS BIBLIOGRÁFICAS 122

TRADUÇÕES 124

AULA 01 – SKIMMING AND SCANNING AND OTHER TECHNIQUES. IDENTIFYING FALSE COGNATS. 2
TEACHER ANDREA BELO

INTRODUÇÃO
Foi apresentada, junto ao cronograma de estudos, de forma sintetizada, uma introdução
às técnicas Skimming e Scanning.
Estudamos e praticamos exercícios fazendo leituras dinâmicas e rápidas, com o objetivo de
se concentrar em palavras chaves, em ideias.
Recordando, Scanning, é a “varredura” do texto. É ler com atenção a primeira página de
um jornal, procurar uma palavra no dicionário, pesquisar na internet um artigo sobre
determinado assunto em busca de informações específicas, necessárias naquele momento, entre
outros.
Durante as leituras dos textos presentes na sua prova, você fará um rastreamento,
procurará algo que realmente interessa e seguir selecionando partes essenciais dos textos até
encontrar a informação desejada.
Você pode voltar no texto time after time com o foco no que precisa para responder aos
exercícios. Concentre-se que vai dar certo! Let’s go!
Em complemento ao Scanning, vimos que a técnica Skimming é uma estratégia que ajuda
você a ler o texto mais rápido também.
Dessa vez, prestando atenção ao layout do texto, título, subtítulo, cognatos, falsos
cognatos, primeiras e/ou últimas linhas de cada parágrafo, informação não verbal (figuras,
tirinhas, anúncios, gráficos, tabelas etc).
Por exemplo, quando você espera para ser atendidos em um consultório médico e fica
folheando aleatoriamente uma revista, você está usando a técnica do Skimming.
Em ambas as técnicas que usaremos bastante agora, você vai, naturalmente e, na
sequência das nossas aulas, obter a ideia geral de cada texto, identificar o assunto e prever
informações essenciais ao texto.
No momento da prova, você vai ler cuidadosamente todos os parágrafos, percebendo a
função e razão de haver certas palavras destacadas, ou em caixa alta, em negrito ou em itálico
etc.
Você deve estar atento aos detalhes dos textos, mas não se esquecer, é claro, que são
colaboradores diretos da sua aprovação os seus conhecimentos gerais, curiosidade ao que está
acontecendo atualmente em nosso país e no mundo e também seu estudo constante de todas as
disciplinas que abrangem os conteúdos da prova de melhores instituições do Brasil.
Vamos dar continuidade ao uso das técnicas, que certamente, serão valiosas e você
também deve estudar vários cognatos e falsos cognatos, que podem aparecer em sua prova.
Come on! Vamos resolver mais exercícios com diferentes tipos de questões e aprimorar
seu vocabulário com cognatos e falsos cognatos e, é claro, com as traduções no fim do material e
assim, day after day, você sentir-se mais preparado e confiante – o melhor candidato!

AULA 01 – SKIMMING AND SCANNING AND OTHER TECHNIQUES. IDENTIFYING FALSE COGNATS. 3
TEACHER ANDREA BELO

SCANNING: INVESTIGAR E RESPONDER


Ler textos presentes na prova exige concentração, capacidade interpretativa e muita,
muita atenção. E, você já sabe que, usando Scanning facilita sua compreensão, encontrando as
palavras-chave, que serão “guia” para encontrar a resposta da questão.
Alguns autores, além de escrever sobre Scanning, chamam essa técnica de Selectivity, por
ser uma forma de leitura seletiva, já que selecionamos os trechos onde se deseja encontrar aquela
determinada informação.
O que realmente importa não é o nome da técnica, mas, se você está analisando, em suas
leituras, as palavras cognatas, o vocabulário essencial e o que mais for questionado nos
enunciados das questões, levando à compreensão mais eficiente.
Na hora de sua prova, a escolha da estratégia de leitura deve estar de acordo com o objetivo
de cada questão e suas possíveis alternativas.
No decorrer das aulas, inúmeras questões serão solucionadas, todas elas detalhadamente,
com ênfase nas melhores formas de encontrar respostas, usando e chegar à sua aprovação.
Escolhendo o Scanning, você sabe exatamente o que está procurando: informações
específicas. Veja um esquema com as principais funções da técnica Scanning:

SCANNING:
✓ ESTRATÉGIA DE LEITURA SELETIVA
✓ FAZER A "VARREDURA" DO TEXTO EM QUESTÃO
✓ ATENTAR-SE EM PALAVRAS, FRASES E TERMOS IMPORTANTES
✓ BUSCAR INFORMAÇÕES OBJETIVAMENTE
✓ NÃO LER TEXTO TODO E SIM, ENCONTRAR RESPOSTAS ESPECÍFICAS À PERGUNTA
✓ CONECTAR PALAVRAS PROCURADAS COM O ENUNCIADO DA PERGUNTA

Scanning é, muitas vezes, chamado de “leitura de passar os olhos” porque você vai
justamente obter as informações fazendo uma rápida visualização. Você está em busca de algo
específico, que poderá facilitar o caminho até a resposta daquele exercício.
Vejamos agora, exemplo de questão que poderia ser da sua prova, explorando Scanning:

AULA 01 – SKIMMING AND SCANNING AND OTHER TECHNIQUES. IDENTIFYING FALSE COGNATS. 4
TEACHER ANDREA BELO

Questão inédita – A opção correta sobre o texto é:


A) O texto trata de uma brincadeira na hora do corte de cabelo.
B) O barbeiro não quer cortar o cabelo de Calvin.
C) Adultos decidem como crianças devem cortar seus cabelos.
D) As crianças, como Calvin, não sabem qual é o corte ideal para a idade delas.
E) O texto trata do corte de cabelo de Calvin, decidido pela sua mãe.
Apenas com a técnica scanning, você “bate os olhos” e percebe que é um corte de cabelo, Calvin
sugere algo, mas a decisão final é da sua mãe, certo? Então, você já acertaria a questão só com a
busca de palavras específicas, tais como “I want...”, sobre o que Calvin quer do corte de cabelo e
a mãe: “the usual “– sem traduzir, mas, escaneando termos importantes.
Na letra A, fala de brincadeira, o que não é visto na examinação de palavras-chave. Falsa.
Quanto à letra B, temos a afirmação de o barbeiro não quer cortar o cabelo de Calvin, mas não
vemos isso no texto, nem lendo e escaneando palavra por palavra. Falsa.
A letra C é falsa porque afirma algo geral e vimos, pela “varredura” do texto, que se trata de Calvin
e sua mãe e não de todos os adultos, como sugerido. Falsa.
A letra D está errada também. Podemos perceber que se trata de Calvin e não de todas as crianças,
como foi generalizado na letra D. Falsa.
A resposta correta é a letra E, pois, foi possível perceber com scanning, como analisamos no início
da resposta.
Sua meta é a aprovação e então, temos um caminho a percorrer juntos.
Vamos à técnica Skimming agora! Come on!

AULA 01 – SKIMMING AND SCANNING AND OTHER TECHNIQUES. IDENTIFYING FALSE COGNATS. 5
TEACHER ANDREA BELO

SKIMMING: COMPREENDER E SOLUCIONAR


Agora é a vez de fazer uso do Skimming, estratégia tal que exige observação das
informações visuais que acompanham o texto (palavras destacadas, título, subtítulo, autor, fonte,
data, layout do texto, tabelas, fotos, referências, enfim, aquela "leitura por cima", para explorar
ao máximo as informações importantes presentes e perceber a ideia geral do texto.

Deve-se, porém, lembrar-se de que essa estratégia, assim como qualquer outra, só
funciona se você estiver concentrado durante a leitura.

Isso porque, além das informações ali contidas, resumirem o texto, cada palavra em
destaque é uma dica do assunto e, muitas vezes, é a própria resposta solicitada.

Skimming, ao começar a ler seu texto, vai proporcionar a você, a compreensão geral -
General comprehension - sobre qual assunto o texto discorre com dicas visuais de fácil percepção
a um futuro aluno de uma excelente universidade, não é verdade?

Veja um esquema com as principais funções da técnica Skimming:

SKIMMING:
✓ ESTRATÉGIA DE LEITURA DE EXAMINAÇÃO

✓ CAPTAR IDEIAS PRINCIPAIS

✓ OBSERVAÇÃO GERAL SEM DETER-SE EM UM PONTO ESPECÍFICO

✓ RECONHECER DICAS SOBRE O ASSUNTO

✓ RECONHECER DICAS SOBRE O ASSUNTO

✓ NÃO LER TEXTO TODO E SIM, EXPLORAR A LEITURA EM GERAL

✓ INVESTIGAÇÃO DA IDEIA CENTRAL DO TEXTO

Para praticar a técnica de Skimming em um texto, que poderia ser, por exemplo, o artigo
abaixo, retirado do jornal The Guardian, observe a questão e os comentários, para que você saiba
como encontrar a resposta:

AULA 01 – SKIMMING AND SCANNING AND OTHER TECHNIQUES. IDENTIFYING FALSE COGNATS. 6
TEACHER ANDREA BELO

AULA 01 – SKIMMING AND SCANNING AND OTHER TECHNIQUES. IDENTIFYING FALSE COGNATS. 7
TEACHER ANDREA BELO

Questão – De acordo com o início do texto:


A) é perguntado se calculamos o tempo que passamos na internet.
B) pede-se para refletirmos sobre o uso da tecnologia em nossas vidas.
C) a pergunta é uma contatação do uso do celular da escritora Adrienne Matei.
D) a pergunta feita é uma reflexão sobre o tempo gasto no celular.
E) pergunta-se o número exato de horas que passamos usando o celular.

Bom, no início do texto, praticando Scanning e Skimming, já eliminamos as alternativas A


e B, que afirmam calcular o tempo na internet e tecnologia (e o texto é especificamente sobre
uso de telefone celular e não sobre internet ou tecnologia de forma geral).
A alternativa C, não tem relação com o que é perguntado: “Do you know how much time
you spend on your phone?” (Você sabe quanto tempo passa em seu telefone?), já que é uma
pergunta direcionada ao leitor e não sobre a escritora mencionada.
E a alternativa E, afirma que se pergunta número exato de horas, mas não há nada de exato
na pergunta e sim uma reflexão de quanto tempo se passa no celular, direcionada, como eu disse,
ao leitor.
A alternativa D é a que contempla, de forma correta essa reflexão: tempo que você, leitor,
passa em seu telefone. Alternativa correta, por análise geral – skimming de informações.

Em cada exercício do nosso material, estamos fazendo uma análise cuidadosa e completa.
Isso vai te oferecer condições seguras para o dia da sua prova.
Independente da universidade escolhida, você precisará encontrar uma forma de ler e
responder as questões para ingressar nessas universidades. Todo exercício é preparo!
Fique atento aos “vestígios” que aparecem, são colaboradores direto do seu acerto nas
questões, junto às técnicas, que vamos treinar em toda essa aula. É importante que você saiba
como selecionar o que lê, onde focar sua atenção no momento de procurar uma informação
específica e essencial à questão.
Podemos excluir algumas opções de resposta a partir de análises. E podemos, também,
excluir alternativas com informações “inventadas”, ou seja, inexistentes no texto, que são
colocadas dentre as alternativas de resposta para gerar dúvida.
Eu costumo nomear essas armadilhas de “tricky” words, ou seja, pegadinhas. Vamos
analisar muitas no decorrer do material, com a prática de vários exercícios na teoria e na lista de
questões comentadas. Fique atento! Very important!

AULA 01 – SKIMMING AND SCANNING AND OTHER TECHNIQUES. IDENTIFYING FALSE COGNATS. 8
TEACHER ANDREA BELO

SCANNING & SKIMMING: LER E ACHAR AS RESPOSTAS: “JUST DO IT”


Já comprovamos que existem várias formas de facilitar a leitura em diferentes situações
da sua prova.

Identificar palavras ou ideias, procurar um verbo, um adjetivo, uma afirmação, uma


negação, entre outras informações essenciais na hora de resolver a prova.

O melhor candidato – você – deve aprender a usar técnicas e, junto aos seus
conhecimentos, chegar ao seu propósito = a sua aprovação.

Skimming e scanning são expressões do Inglês equivalentes a: ler superficialmente e ler


rapidamente, respectivamente.

Essas técnicas, como vimos anteriormente, ajudam você a obter mais rapidamente a
informação dos textos, como eu disse em outros capítulos, não sendo necessário ler cada palavra
contida em seu contexto.

Você vai fazer perguntas ao texto: o que se espera desse texto? Quais são as partes
importantes? Que locais posso reconhecer (se há imagem vinculada), Qual é o assunto central?
Que palavras ajudam a reconhecer do que se trata sua leitura?

E assim, percorrer o caminho ideal para chegar à conclusões, que certamente, levarão você
à aprovação.

O objetivo de compreender os textos vai depender diretamente da sua capacidade em


relacionar ideias, estabelecer referências e fazer deduções lógicas para buscar respostas às
questões.

Com a prática que estamos desenvolvendo nos exercícios, você vai, naturalmente, saber
identificar palavras que sinalizam o que se pede na questão, percebendo os elementos que
colaboram com a resposta correta.

Utilizando a técnica Scanning e também Skimming nas leituras, agora juntas na mesma
questão, será como o título desse capítulo – ler e achar as respostas: Just do it!

Para isso, você deve deixar de lado aquele hábito de ler palavra por palavra, lembrar tudo
o que sabe sobre o assunto e prestar atenção ao contexto em que as questões estão inseridas.

Usando o mesmo texto, que respondemos uma pergunta usando a técnica skimming,
vamos agora usar scanning e, escaneando informações, resolver outra questão, similar às da sua
prova:

AULA 01 – SKIMMING AND SCANNING AND OTHER TECHNIQUES. IDENTIFYING FALSE COGNATS. 9
TEACHER ANDREA BELO

AULA 01 – SKIMMING AND SCANNING AND OTHER TECHNIQUES. IDENTIFYING FALSE COGNATS. 10
TEACHER ANDREA BELO

Em cada exercício do nosso material, estamos fazendo uma análise cuidadosa e completa.
Isso vai te oferecer condições seguras para o dia da sua prova. Veja analisando scanning:
Questão – Na frase do texto “... until you realize it”, o termo sublinhado se refere a:
A) ao tempo gasto com o aparelho celular
B) ao verbo realize, que aparece antes do termo it.
C) aos 20 minutos, antes mencionados.
D) ao celular de Adrienne Matei.
E) aos 35 dias ao telefone.

Bom, escaneando a frase em que o pronome “it” aparece, percebemos, de acordo com
regaras gramaticais existentes, que é, de fato, um pronome que se refere a palavras no singular.
“IT” é o pronome específico de substituição de um objeto ou um animal, por exemplo.
Assim, já eliminamos as alternativas C e E, em que se afirma que “it” se refere a algo no plural:
minutos e dias.
A alternativa A, é, até agora, a melhor opção, pois “until you realize it”, é até você perceber
isso, provavelmente, isso seja o tempo gasto com o celular, mas, continuemos a analisar as
alternativas.
E a alternativa B, afirma que “it” se refere a um verbo, mas, como vimos quando falei a
tradução da frase “até você perceber isso”, é notável que it não está conectado com o verbo e
sim com o tempo gasto com o celular.
A alternativa D diz que se refere ao celular de Adrienne, mas, a pergunta é para o leitor,
você, e não a escritora.
Assim, a melhor opção realmente é a letra A, em que se afirma que “it” se refere ao tempo:
exatamente o que o pronome quer dizer.
Viu como scanning funciona? Você vai direto ao ponto, no local em que a palavra está e o
que deve substitui-la ou encaixar seja qual for a pergunta da prova.
Muitas pessoas consideram scanning e skimming como técnicas preciosas e verdadeiras
estratégias de leitura, já que você consegue ler um grande volume de informação com prática.
Podemos afirmar que o skimming consiste em observar o texto rapidamente, para detectar
o assunto, sem preocupar com os detalhes, permitindo ao leitor identificar rapidamente o sentido
geral do texto.
Geralmente a leitura no skimming é realizada com a velocidade maior que a leitura normal
e assim, é mais abrangente.
E o scanning é uma técnica que consiste em localizar a informação específica desejada com
os olhos. É uma rápida visualização do texto como o nome diz, como um scanner, trabalhando
rapidamente, lendo a informação contida naquele espaço.
Agora, vejamos um esquema em que há informações sobre as técnicas que vimos e
aplicamos nos exercícios. Vamos lá.

AULA 01 – SKIMMING AND SCANNING AND OTHER TECHNIQUES. IDENTIFYING FALSE COGNATS. 11
TEACHER ANDREA BELO

SOLUÇÃO DA QUESTÃO: APROVAÇÃO


✓ Identificação de palavras conhecidas e pouca importância àquelas desconhecidas: leitura
seletiva.
✓ Atenção aos assuntos ligados à atualidade em nosso país e no mundo.
✓ Enriquecimento de vocabulário com exercícios e traduções “bônus” no final.
✓ Estudo constante para sentir-se preparado e capaz de responder as provas.
✓ Técnicas Scanning e Skimming: chegar mais rápido à resposta da questão.

Esquema aprovado? Vamos lá? Rumo à aprovação!

CURIOSIDADES SCANNING/SKIMMING
As estratégias de leitura que estamos usando nessa aula, têm a finalidade de viabilizar a
sua leitura sem que, necessariamente, você aprenda todas as palavras que existem em Inglês.
Seu vocabulário vai se estender, day after day, se você estiver lendo da forma que estou
explicando e mostrando a você através dos exercícios da de provas anteriores, aqui resolvidos e
comentados em detalhes.
O caminho do sucesso é o estudo contínuo e persistência em aprender.
Além disso, as traduções oferecidas no fim do material, como já adiantei, são preciosas
ferramentas de estudo com intuito de enriquecer vocabulário, tanto para ensinar você a
interpretar textos quanto escrever parágrafos inteiros sabendo muitas palavras.
Há curiosidades sobre as técnicas que estamos utilizando? Sim!
Quando pesquisamos sobre preparar-se para provas de mestrado e doutorado, bem como
especializações que exigem proficiência em Inglês, além das provas de várias instituições, a
sugestão e verdadeira instrução é sempre ler textos utilizando das técnicas Scanning e Skimming.
Por quê?
A resposta só pode ser uma: com essas técnicas, conseguimos ler textos com agilidade e
qualidade, já que a assimilação é rápida.
A principal vantagem dessas técnicas é que são um tipo de leitura dinâmica e ensinam você
a reconhecer vocábulos inseridos dentro da frase.
E, reconhecer “dados” incorporados ao texto, é o grande segredo.

AULA 01 – SKIMMING AND SCANNING AND OTHER TECHNIQUES. IDENTIFYING FALSE COGNATS. 12
TEACHER ANDREA BELO

Assim, você enxerga e compreende o assunto através dos blocos de palavras juntas. Seus
olhos fazem “paradas” tão rápidas que não se percebe.
Uma fonte muito usada nas provas são as tirinhas do “Hagar, The Terrible” – Hagar, o
horrível. São curtas, críticas e cheias de assuntos interessantes para as questões.
Escolhi essa tirinha de “Hagar, o Horrível”, por ser um exemplo de fonte explorada com
assuntos explorados em muitas disciplinas. As tirinhas ajudam a ter uma visão ampla de algo com
teor crítico, às vezes sarcástico, propositalmente preparadas para análise mesmo.
Vamos olhar para a história, fazer uma leitura com os olhos rapidamente, tentar entender
a essência e extrair o que se compreende com as palavras que chamam a atenção.

Questão Inédita Teacher Andrea Belo – identificação do assunto/interpretação.


Veja a sugestão de perguntas da forma como você deve iniciar a resolução.
Qual é o assunto que você consegue perceber através da leitura da tirinha do Hagar? Quem
são os personagens? Sobre o que conversam? Como são as reações deles?
Tente responder comprovando com Scanning e Skimming e veja como fica fácil.

https://nebusresearch.files.wordpress.com/2014/06/chris-browne_hagar-the-horrible_19-june-2014.gif

Você deve “ler” os quadrinhos. As expressões faciais. As reações. Os movimentos. Tudo


são indicadores do assunto. É primordial que você leia a figura ou um texto como se estivesse
desvendando um mistério, decifrando um código.
No quadrinho em questão, quando você vê a palavra “beer” no primeiro quadrinho,
pronunciada pela esposa, percebe que ela não está muito satisfeita provavelmente porque Hagar
está bêbado (percebemos pelas estrelinhas ao redor da cabeça dele).

AULA 01 – SKIMMING AND SCANNING AND OTHER TECHNIQUES. IDENTIFYING FALSE COGNATS. 13
TEACHER ANDREA BELO

Como resposta, Hagar pronuncia números indicando que não sabe quantas cervejas bebeu.
A expressão que contém palavras cognatas, “serious problem”, que a mulher dele diz, mostra que
ela está irritada, pois aponta o dedo, demonstração de nervosismo, raiva, fúria.
Ele, “sem noção” do que diz e, perceptivelmente alcoolizado (inclusive com as estrelinhas
ao redor da cabeça), responde com as palavras “math tutor”, que podemos deduzir uma
manifestação de que ele revela não saber matemática e, assim, precisa de um tutor, um professor
de matemática.
Conseguiu identificar o assunto? Encontrou palavras que facilitaram?
As “chaves” da resolução são as técnicas e atenção!
Bom, essa “leitura rápida” foi feita em segundos.
Isso que você está aprendendo e deve fazer, em todas as questões da sua prova. Vamos
para o próximo capítulo. Mas antes, deixo aqui um esquema representativo a seguir.

INTERPRETAÇÃO DE TEXTOS NAS PROVAS


➢ “Ler” as expressões faciais dos personagens.
➢ “Ler” os movimentos dos personagens.
➢ “Ler” os detalhes ao redor dos textos.
➢ “Ler” em conexão ao que você sabe, seu backgroung.
➢ “Ler” as palavras essenciais: palavras-chave.
➢ “Ler” o ambiente/local apresentado na questão.
➢ “Ler” termos em detaque, como negrito, itálico e CAIXA ALTA.
➢ “Ler” as rferências, tais como datas e fonte.
➢ “Ler” e abusar das palavras cognatas – grandes facilitadoras.
➢ “Ler” e identificar os falsos cognatos disfarçados – estudá-los.
➢ “Ler” o título, os enunciados, os comandos – achar o que se pede.
➢ “Ler” e fazer deduções relacionadas ao assunto.
Vejamos outros textos com o uso do Scanning e Skimming.

AULA 01 – SKIMMING AND SCANNING AND OTHER TECHNIQUES. IDENTIFYING FALSE COGNATS. 14
TEACHER ANDREA BELO

SCANNING E SKIMMING EM PEQUENOS TEXTOS


Após utilizarmos as técnicas necessárias com análise de detalhes fundamentais, o caminho
da aprovação está fácil, percebeu? Como você está se saindo?
Você já deve ter notado que usar Skimming e Scanning é basicamente fazer uma leitura
rápida, mas, não estou falando de velocidade, e sim de atingir os objetivos esperados.
Até porque, posteriormente, você pode voltar ao texto – quantas vezes quiser – e retomar
informações sobre o que você procura.
Estamos, em cada exercício, identificando o que é essencial para ter uma noção geral do
que se trata o texto. É uma construção de conhecimento, com foco ao que realmente interessa,
economizando tempo e resolvendo as questões da prova facilmente.
Nesse capítulo, vou exemplificar o uso dessas técnicas juntas em textos pequenos e
posteriormente, em textos mais longos. Veja essas frases:

Após exames, pessoas com níveis alterados de glicemia, a comprovação


de ringope é imediata. Além de necessitar o uso de insulina, é obrigatório evitar
o consumo de lagofe, para não agravar mais ainda as condições do paciente,
que pode fatalmente sabafar, se não tomar os devidos cuidados com a saúde.

Sendo uma questão, você conseguiria ler as palavras que inventei - ringope, lagofe e
sabafar - substuindo-as, de forma automática, pelos termos corretos à compreensão do parágrafo
acima e responder qual é o assunto? Sim, diabetes. E ringope é diabetes, lagofe - açúcar, sabafar
- falecer, não é?
Você percebeu que não demorou tanto tempo para identificar as palavras inventadas e, o
melhor, compreendeu o parágrafo inteiro por dois motivos: você aprendeu a ler de forma
dinâmica, dando importância às palavras peculiares, aquelas que indicam o assunto e/ou o que
precisa saber sobre tal assunto.
E porque o uso das técnicas estudadas em conexão ao vocabulário aprendido a cada
exercício resolvido, proporciona autoconfiança, garantia da sua aprovação.
A falta de vocabulário é considerada uma das causas para o atraso na leitura. Porém,
durante seus estudos, você percebe que as palavras desconhecidas, não devem ser encaradas
como empecilho, mas desafios: você vai procurar o sentido entre as palavras.

AULA 01 – SKIMMING AND SCANNING AND OTHER TECHNIQUES. IDENTIFYING FALSE COGNATS. 15
TEACHER ANDREA BELO

SCANNING E SKIMMING EM TEXTOS LONGOS


A partir de diferentes formas que facilitam suas leituras, você já precisa ter em mente que
vai conseguir ler, utilizando as técnicas estudadas para compreender o texto ou parte dele –
parte essencial para encontrar o que se pede.

E, acelerando sua leitura pouco a pouco, tanto em textos curtos ou em textos maiores, o
tipo de varredura feita pelos seus olhos fará com que seu cérebro responda aos estímulos que
você enviou.

E, dessa forma, dando-lhes significados, que ajudam a encontrar as respostas certas.

Você estará, no decorrer das aulas, ressaltando o que é mais ou o que é menos
interessante em cada texto. O que é mais ou menos relevante. E assim, caminhar para as
soluções das questões.

A leitura dos textos, apesar de muitas vezes apresentarem um conteúdo longo e repleto
de palavras desconhecidas, ficará viável.

Isso porque, com o uso de técnicas, vinculadas às análises cautelosas e com muita atenção
que se deve ter antes de responder uma questão, mesmo se for simples.

Pode ser, inclusive, que você considere simples demais e, no teor de cada pergunta, a
interpretação exija de você um cuidado ainda maior. É necessário muita atenção e o bom uso das
técnicas ensinadas. Don’t forget it!

Vejamos questões inéditas em continuação à resolução de possíveis questões.

Agora, com as questões inéditas e também com as questões de anos anteriores, que vamos
solucionar e comentar, uma a uma.

Aqui, teremos exemplos de textos curtos, longos, imagens, tirinhas, entre outros,
analisando cada possibilidade de exercício que já estiveram ou podem estar presentes em sua
prova.

Vamos explorar, nessa aula, bastante exercícios de escrita, como é cobrado na prova, para
aproveitar as técnicas de scanning e skimming aqui aprendidas.

Mas, como eu disse, as técnicas são excelentes para a prática da leitura rápida, dinâmica,
com agilidade e sabedoria para, no dia da sua prova, você gabaritar a prova de Inglês. Come on!

AULA 01 – SKIMMING AND SCANNING AND OTHER TECHNIQUES. IDENTIFYING FALSE COGNATS. 16
TEACHER ANDREA BELO

COGNATES AND FALSE COGNATES

Eu falei para você, na aula de introdução, a importância de conhecer os falsos cognatos já


que, cognatos, pela semelhança com nossa língua, ajudam na leitura, enquanto os falsos
cognatos, podem atrapalhar quando não se sabe o que realmente significa.

O termo cognato se refere a palavras que têm a mesma origem. Por exemplo, Novembro
(Português) e November (em Inglês) são cognatos.

É normal que as pessoas usem o termo “falsos cognatos” quando se referem a palavras
que são escritas ou faladas de forma parecida e, portanto, nos fazem pensar que as duas palavras
têm o mesmo significado, mas não têm. E aparecem muito nas provas.

Um dos exemplos mais comuns de falsos cognatos, que confundem a mente das pessoas,
é o caso das placas escritas “push” e “pull”. Isso porque, “push”, em Inglês, se assemelha ao
verbo puxar como “puxe”, em Português, mas o significado de “push” é empurrar.

Podemos, inclusive, ver pessoas puxando a porta ao invés de empurrar, não é mesmo?

Apesar de bastante utilizado, este termo não é o mais adequado de todos. Cognatos se
referem, de fato, à origem das palavras, e ao pensar em falsos cognatos não estamos pensando
na origem, e sim nos seus significados em duas línguas diferentes.

E, claro que duas palavras podem ter a mesma origem, mas por diversos acontecimentos
ao longo do tempo estas palavras acabaram adquirindo significados diferentes.

Um termo também comum e melhor para expressar os falsos cognatos é “false friends”,
ou falsos amigos. Veja um exemplo:

Nessa imagem, por exemplo, se for para ler ou


escrever que a mulher está com uma roupa
elegante, diríamos:
“The woman is elegant because of her apparel”
Eu sei que o falso cognato apparel lembra a palavra
aparelho e não parece nada com vestimenta, mas
vestimenta é a tradução de apparel.

Os falsos cognatos, também chamados de falsos amigos são, muitas vezes, considerados
inimigos na hora dos estudos.

AULA 01 – SKIMMING AND SCANNING AND OTHER TECHNIQUES. IDENTIFYING FALSE COGNATS. 17
TEACHER ANDREA BELO

Mas você não deve se apegar aos detalhes, não vai parar sua leitura a cada vírgula e sim,
usará skimming e scanning - ler com os olhos para compreender o assunto e também “escanear”
as informações necessárias e solicitadas nas questões, como eu já te disse antes.
Você vai precisar de uma lista com os falsos cognatos mais comuns em provas. Vou
mostrar, então, uma lista com diversos outros casos que enganam pessoas, para você não se
confundir e sim, aprimorar seus conhecimentos.
Vamos nos preparar bem. Agora, vejamos alguns exemplos de falsos cognatos na lista que
preparei para você, com muitos presentes em provas anteriores diversas.

LISTA DE FALSOS COGNATOS MAIS COMUNS


E FREQUENTES NAS PROVAS
FALSOS COGNATOS E SEUS SIGNIFICADOS
WATCH OUT! CUIDADO!
Abstract – resumo Abstrato – conceptual

Actually - na verdade, de fato Atualmente – These days, nowadays

Accent - sotaque Assento – seat

Adept- especialista, bom conhecedor Adepto - supporter

Agenda - pauta diária Agenda - appointment book

Alias – pseudônimo Aliás – By the way

Alms – esmola Almas – souls

Animus – hostilidade, inimizade Animado – Excited

Annotate - observar Anotar – to take note, to write down

Application– registro, inscrição Aplicação - investment

Appointment - compromisso profissional Apontamento – note

Appreciation – gratidão, reconhecimento Apreciação – judgement

Argument (n) – discussão Argumento - reasoning, point

Arm - braço Arma - gun

Army – exercito Arma – gun

Assist – ajudar, dar assistência Assistir - Watch

AULA 01 – SKIMMING AND SCANNING AND OTHER TECHNIQUES. IDENTIFYING FALSE COGNATS. 18
TEACHER ANDREA BELO

Attend - assistir, participar de Atender - to help; to answer; to see, to

Audience – plateia, público Audiencia – court appearance

Balcony - sacada Balcão - counter

Baton – cassetete Batom – lipstick

Barracks – quartel Barraca – tent

Bond – vínculo, elo Bonde – trolley car

Bonnet – touca, capô de carro Boné – cap

Braces – aparelho dental Braços – arms

Candid – sincero Cândido – innocent, naive

Carton - caixa de papelão Cartão – card

Cartoon – desenho animado Cartão – card

Chef – cozinheiro, mestre cuca Chefe – boss

Cigar - charuto Cigarro - cigarette

College - faculdade Colégio - school

Commodity - artigo, mercadoria Comodidade - comfort

Compromise - entrar em acordo Compromisso - appointment; date

Content - conteúdo Contente – glad, happy

Convict - réu Convicto - Sure

Costume - fantasia (roupa) Costume - custom, habit

Cup - xícara Copo - glass

Curse – maldição, xingamento. Curso – course

Dessert - sobremesa Deserto - desert

Data – dados Data – date

Dependable – confiável Dependente – Dependant

Devolve – transferir Devolver – return, give back, refund

Discussion – debate, opiniões Discussão – argument

AULA 01 – SKIMMING AND SCANNING AND OTHER TECHNIQUES. IDENTIFYING FALSE COGNATS. 19
TEACHER ANDREA BELO

Diversion – desvio, trajeto Diversão – fun

Educated - bom nível de escolaridade Educado - well-mannered, polite

Enroll - registrar-se inscrever-se Enrolar - to roll

Expiation – penitência, castigo Espiar – To spy

Exquisite - belo, refinado Esquisito - strange, odd

Expert – especialista Esperto – clever, smart

Fabric – tecido Fábrica – factory

Fate – destino Fato – fact

File – arquivo Fila – line

Gracious – benéfico Gracioso – graceful

Gratuity – gorjeta Gratuito – For free, gratuit

Gravy – molho, caldo Grave – Serious

Grip - agarrar firme Gripe - cold, flu

Heydey – apogeu Ei, dia – Hey day

Hazard - risco, arriscar Azar - bad luck

Hostage – refém Hóspede – guest

Idiom - expressão idiomática Idioma – language

Injury – ferida, ferimento Injuria – insult, offence

Intend - pretender, ter intenção Entender – understand

Intoxication – embriaguez, drogas Intoxicação – poisoning

Jest – zombo, brincadeira Gesto – gesture

Lecture - palestra, aula Leitura – reading

Legend – lenda Legenda - subtitle

Library - biblioteca Livraria - book shop

Lunch – almoço Lanche – snack

Luxury - luxo Luxúria – lust

AULA 01 – SKIMMING AND SCANNING AND OTHER TECHNIQUES. IDENTIFYING FALSE COGNATS. 20
TEACHER ANDREA BELO

Magazine - revista Magazine - department store

Mayor – prefeito Maior – bigger

Mate – colega, companheiro Matar – to kill

Moisture - umidade Mistura - mix, mixture

Notice - notar, perceber-se Notícia - news

Novel – romance Novela - soap opera

Office – escritório Official – officer

Oration – discurso (formal) Oração – prayer

Orchard – pomar Orquídea - Orchid

Parents - pais Parentes - relatives

Pasta – massa, macarrão Pasta – briefcase

Physician – médico Físico – physicist

Phony – impostor Telephone – Phone, telephone

Policy – Apólice, política (ideal político) Polícia – Police

Pork – carne de porco Porco – pig

Port – porto Porta – door

Prate – tagarelar, falar muito Prato – plate, dish

Preservative – conservante Preservative – condom

Pretend - fingir Pretender - to intend, to plan

Procure - conseguir, adquirir Procurar - to look for

Pull - puxar Pular - to jump

Push - empurrar Puxar - to pull

Realize - perceber, notar Realizar - make come true, to carry out

Recipient – recebedor Recipiente – container

Reclaim – recuperar Reclamar – complain

Record – gravar Recordar, lembrar – remember, remind

AULA 01 – SKIMMING AND SCANNING AND OTHER TECHNIQUES. IDENTIFYING FALSE COGNATS. 21
TEACHER ANDREA BELO

Refrigerant – uma substância Refrigerante - soft drink, soda

Resume - retomar, reiniciar Resumir – sumarize

Retired – aposentado Retirado – removed

Senior – idoso Senhor - gentleman, sir

Sensible – sensato Sensível – sensitive

Service – atendimento Serviço – job

Sort – tipo, espécie Sorte – luck, fate

Stranger - desconhecido Estrangeiro - foreigner

Stupid – burro Estúpido - impolite, rude

Supper – jantar, ceia Super – super

Support - apoiar Suportar (tolerar) - can stand

Sympathetic – compreensivo Simpático - nice

Tax - imposto Taxa - rate; fee

Tent – barraca Tentar – to try

Thicket – moita, mato fechado Ticket, bilhete – ticket

Toss – arremessar Tosse – cough

Turn - vez, volta, virar, girar Turno - shift; round

Ultimately – em última análise Ultimamente – Lately

Valorous – corajoso, destemido Valoroso, de valor, valuable

Vicious – defeituoso, impuro Viciado – addicted

Vine – videira Vinho – wine

Meu conselho para você é o seguinte: leia a lista, estude bastante, sempre.
Quanto mais você ler artigos jornalísticos, reportagens, textos em geral usados como fonte
para elaboração das provas, junto ao material aqui desenvolvido, mais preparado você vai estar.
E agora, vamos aos exercícios de provas de anos anteriores, para praticar questões já
cobradas e aumentar seus conhecimentos sobre como essas questões são elaboradas.

AULA 01 – SKIMMING AND SCANNING AND OTHER TECHNIQUES. IDENTIFYING FALSE COGNATS. 22
TEACHER ANDREA BELO

QUESTÕES
Esse é momento em que vamos praticar tudo o que vimos nessa Aula 01. Serão questões de
várias instituições para preparar você e colaborar com a sua aprovação. Começando com
questões AFA.
QUESTÕES AFA
Directions: Read the text below and answer questions 01 to 10 according to it.
TEXT
The influence of YouTubers on teenagers
A few years ago, the new internet hype called Web 2.0 appeared, resulting in a big shift from using
internet relatively passive and consuming, to a hype of networking as a revolutionary type of
largescale cooperation and economic democracy. This big shift towards user-driven technologies
such as blogs, video-sharing platforms and social networks enabled a revolution of User-
Generated Content (UGC). UGC refers to all the various media content created or produced by the
public, excluding paid professionals. UGC can be described as the process of ‘extracting value by
generating, reviewing, editing and disseminating content’. This revolution of UGC is now being
referred to as social media.
UGC is also stated to be the next iteration of Word-of-Mouth (WOM). Traditional (offline) WOM
has proven to play a major role in customers’ decision-making process. The so-called eWOM is any
positive or negative statement made by (potential) customers about a product or company, which
is made available to a multitude of people and institutions via the Internet. Consumers can share
their own experiences and opinions online, increasing the unbiased product information people
gather from other consumers online.
People are now able to create their own content and participate in all kinds of applications such
as weblogs, social networking and online journalism. Some users create online content on a daily
basis. They share their experiences and opinions about everything; from product reviews, services
and baby advice to boyfriend problems, homework, make up and game tutorials. Often starting
as a hobby, creating online content on a personal blog or channel can become a fulltime job. By
creating content on a daily basis, large communities are built up with thousands of loyal followers.
New followers and subscribers are gained every day. YouTube is one of the most popular platforms
on which reators share their content. With one single video it is possible to reach millions of
people. Other popular channels are Instagram, Snapchat, Vine, Facebook and Twitter. These
platforms are used for real-time contact with YouTube subscribers, as this is not possible on
YouTube itself.
Successful content creators become social influencers with a lot of persuasive influence. Their
influence and fame can be compared with traditional celebrities. The content they share via social
media has influence on the viewers, for example on viewers who are looking for product
information to support their purchasing decisions. Although YouTube is used by all ages, the
platform is very popular among teenagers. According to a research by Defy Media (2015), 96% of
the youngsters aged between 13 and 24 regularly use YouTube. They consume online videos via
social media sites for an average of 11 hours a week, making it their most popular website. This
popularity is reflected again in the amount of YouTubers that focus on teenagers. The influence

AULA 01 – SKIMMING AND SCANNING AND OTHER TECHNIQUES. IDENTIFYING FALSE COGNATS. 23
TEACHER ANDREA BELO

YouTubers have on the behavior of teenagers is increasingly unprecedented. They are seen as role
models and often recognized in the streets by their fans. However, outside this online world,
YouTubers are just as normal as most people.
Parents often do not know about the online behavior of their children and the YouTubers that
children endorse. This results in confusing moments when fans enthusiastically recognize their
favorite YouTuber while other people do not know who this particular YouTuber is. A well-known
Dutch beauty YouTuber explained that when she is in public, fans recognize her from YouTube. Full
of excitement they ask to take a picture together. However, the other people walking by are
confused and do not know why these young teenagers want to take pictures together. This
example shows the unknown impact of social influencers on youngsters of which most adults do
not even know the existence.
Although there already is research available on the influence of traditional media on youngsters,
it is interesting to know what influence YouTubers have on teenagers and whether this influence
is good or bad. Until this point, only little has been studied about the influence of YouTubers on
teenagers. Previous research mostly concerned participatory cultures on YouTube, reviewing
platforms of which YouTube is only a part of several more platforms or the difference between
YouTube and watching traditional television.
Moreover, few studies were conducted in Europe and often limited on the US. Consequently, the
present research takes both the perspective of viewers and YouTubers into account and combines
them within four themes. Therefore, the research question of this research is as following: ‘What
role do YouTubers play in the life of their teenage viewers and how do YouTubers assess their own
influences and experiences?’ The aim of this research is to give an overview of the current YouTube
community, including what influence Dutch YouTubers have on their teenage viewers, what this
means for teenagers and to what extent this influence on their life is good or bad.

QUESTÃO 01 (AFA/INÉDITA) – UGC


(A) relates to media content created by the public, not counting paid professionals.
(B) is an online platform, such as YouTube, focused on creating videos.
(C) includes content produced by paid and nonpaid professionals.
(D) can be described as the process of solely disseminating content.

QUESTÃO 02 (AFA/INÉDITA) – In “Some users create online content on a daily basis.” (Paragraph
3) the underlined expression means
(A) every other day.
(B) very often.
(C) every day.
(D) for a long period.

AULA 01 – SKIMMING AND SCANNING AND OTHER TECHNIQUES. IDENTIFYING FALSE COGNATS. 24
TEACHER ANDREA BELO

QUESTÃO 03 (AFA/INÉDITA) – In “YouTube is one of the most popular platforms on which


creators share their content.” (Paragraph 3), there is an example of
(A) comparative of superiority.
(B) comparative of inferiority.
(C) comparative of equality.
(D) superlative.

QUESTÃO 04 (AFA/INÉDITA) – The fourth paragraph of the text states that


(A) 96% of people regularly use YouTube.
(B) traditional celebrities are still more influential than content creators.
(C) people hardly ever recognize YouTubers in public.
(D) YouTubers influence teenagers on a level never seen before.

QUESTÃO 05 (AFA/INÉDITA) – In the sentence “This big shift towards user-driven technologies …
enabled a revolution of User-Generated Content (UGC).” (Paragraph 1), the underlined word is a
synonym for
(A) increased
(B) allowed
(C) focused
(D) became

QUESTÃO 06 (AFA/INÉDITA) – Mark the alternative that LACKS the correct synonym for the
underlined word.
(A) “… for example on viewers who are looking for product information to support their
purchasing decisions. (Paragraph 4) – buying.
(B) Often starting as a hobby … (paragraph 3) – usually.
(C) … increasing the unbiased product information people gather from other consumers online.
(Paragraph 2) – unfair.
(D) Although YouTube is used by all ages … (paragraph 4) – even though.

QUESTÃO 07 (AFA/INÉDITA) – Mark one of the statements below that DOESN'T agree with the
text.
(A) the research investigates how teenagers identify with YouTubers.
(B) the life of a YouTuber is as beautiful as it looks in their videos.
(C) the research focuses not only on the experience of YouTubers themselves.
(D) teenagers may or may not know how real things are in Youtubers’ videos.

AULA 01 – SKIMMING AND SCANNING AND OTHER TECHNIQUES. IDENTIFYING FALSE COGNATS. 25
TEACHER ANDREA BELO

QUESTÃO 08 (AFA/INÉDITA) – According to the text, parents


(A) many times do not know the YouTubers their kids are fans of.
(B) usually do not endorse their kids spending so much time on YouTube.
(C) enthusiastically recognize their favorite YouTubers in the streets.
(D) are usually aware of the online behavior of their children.

QUESTÃO 09 (AFA/INÉDITA) – According to the text, the word “Moreover” (paragraph 7)


(A) introduces a conclusion.
(B) is used to make an exception.
(C) introduces a contrast.
(D) is used to add information.

QUESTÃO 10 (AFA/INÉDITA) – In the fragment “Consequently, the present research takes both
the perspective of viewers and YouTubers into account and combines them within four themes.”
(Paragraph 7) the underlined pronoun refers to
(A) viewers.
(B) the perspectives of viewers and YouTubers.
(C) YouTubers.
(D) four themes.

QUESTÕES COLÉGIO NAVAL


Texto: TRAVEL TIPS
How to Plan a Movie-Themed Vacation
It’s easier than you may expect to find, visit, and enjoy the places where your favorite movies
were made.
Lars Leetaru
By Shivani Vora
March 8, 2018
Whether it’s the “Lord of the Rings” trilogy in New Zealand or "Roman Holiday” in Rome, many
noteworthy movies are filmed in appealing locales all over the world that travelers may want to
visit and enjoy.
According to Angela Tillson, a film location manager in Kauai who has worked on the set of films
including "Jurassic Park: The Lost World” and “The Descendants," exploring a beloved movie set
destination through the eyes of the film makes for an enjoyable vacation. "Seeing a place with a

AULA 01 – SKIMMING AND SCANNING AND OTHER TECHNIQUES. IDENTIFYING FALSE COGNATS. 26
TEACHER ANDREA BELO

focus on a movie you love will give you a perspective that the average tourist doesn’t usually get.
You’ll certainly have a better impression of the place,” she said. Here are her tips to get started.
Choose Your Destination
If there’s a movie you love, you can find out where it was filmed by looking at the credits at the
end of the film or by going online to The Internet Movie Database, also known as IMDB, which
often lists filming locations. Once you know the locale, you can start planning your trip. Or,
consider doing what Ms. Tillson often does when deciding on where to vacation: pick a spot you’re
interested in visiting, and find out what movies have been filmed there. “It’s fun to sometimes let
a destination determine the movie you're going to live rather than the other way around,” Ms.
Tillson said.
Get in the Mood
Before you head to your destination, be sure to rewatch the movie. A rewatch not only reminds
you of identifiable spots to look out for during your trip, but it also adds to the excitement of your
upcoming exploration.
If the movie is based on a book, consider reading the book, too. It may have details about the
locale that the movie doesn’t touch on. Also, books often have scenes that don’t make it into the
movie adaptations, which gives you a deeper view of the destination. Ms. Tillson also
recommended downloading the movie’s soundtrack or score, and listening to it throughout your
trip.
Book a Themed Trip
Some travel companies sell set itineraries focused on popular movies. Luxury tour operator
Zicasso, for example, has an eight-day trip, all inclusive, to Ireland inspired by "Star Wars: The Last
Jedi” and Wild Frontiers has an eleven-day trip to India inspired by "The Best Exotic Marigold
Hotel." Ms. Tillson suggested doing a web search or checking with a travel agent to find out about
such trips.
Also, in some destinations, local tour operators and hotels sell movie-themed tours. For instance,
The St. Regis Priceville Resort offers a tour that includes a private helicopter ride to
Manawaiopuna Falls, made famous in "Jurassic Park,” and an ATV tour of filming locations of
movies such as “Raiders of the Lost Ark" and “Pirates of the Caribbean.” Lunch is even included.
The cost is $5,674 for two adults.
A more affordable option, in Rome, is the four-hour “Roman Holiday" themed excursion from HR
Tours, where travelers ride a Vespa with a driver and see all the sites from the movie; the cost is
170 euros per person.
Hang Where the Movie Crew Did
When they’re not working, movie crews enjoy hitting local bars and casual restaurants that serve
tasty local cuisine, Ms. Tillson said.
Find out where the behind-the-scenes staff of your film spent their time by asking your
destination’s tourist board or your hotel’s concierge, and check out a few of the spots. “It’s
another way to get involved in the film and spend time in bars and restaurants that you wouldn’t
normally think to hit,” she said.

AULA 01 – SKIMMING AND SCANNING AND OTHER TECHNIQUES. IDENTIFYING FALSE COGNATS. 27
TEACHER ANDREA BELO

QUESTÃO 01 (COLÉGIO NAVAL/2018) – According to Ms. Tillson,


(A) listening to the songs of the movie can get you in the mood of your movie-themed vacation.
(B) you can never choose the destination in the first place. Always base your trip on the movie
you like.
(C) you don’t find any information online about the themed itineraries. You have to check it
exclusively with a travel agent.
(D) it’s difficult to have a different perspective when you explore the destination through the
eyes of the film.
(E) you can visit local bars and restaurants and have the opportunity to meet the movie crew.

Read the text below to do questions 02 to 06.


TEXT III
STEPHEN HAWKING
Stephen Hawking, one of the most famous scientists of the 21st century, died on March 14th,
2018. But his ideas on gravity, black holes and the Big Bang are the greatest legacy he left to the
world.
Early Life and College
On January 8th, 1942, Stephen Hawking was born to a successful family in Oxford, England. He
was not the best student at fundamental or high school, but he was very smart. His dad wanted
him to become a medical doctor, but Stephen really wanted to study math.
Despite his poor grades at school, Stephen aced his exams for college. He was admitted to Oxford
University, but they didn't have math as a major. So, he decided to study physics and chemistry
instead.
Stephen became a member of a rowing team when he was in college. After he graduated, he
decided to continue his education and went to graduate school.
Graduate School, Marriage and Health Problems
While in graduate school, he had some health problems. He began tripping for no reason, and his
speech became hard to understand. His family encouraged him to go to the doctor. Stephen was
diagnosed with a disease called ALS, or Lou Gehrig's disease, which affects the brain and spine.
He was only twenty one, and the doctors initially gave him only a few years to live. Luckily, his
condition progressed more slowly than is often the case.
During this time, Stephen had a relationship with a woman named Jane. He said she and his work
were his inspiration for living. Stephen earned his Ph.D. degree in 1965. He started to get worse,
and eventually became confined to a wheelchair. Nonetheless, he and Jane married in 1965 and
were able to have three children.
He studied how space and time are related, including scientific studies of black holes in space and
how they work in the universe. He also had a lot of success in his work as a college professor.
A New Voice

AULA 01 – SKIMMING AND SCANNING AND OTHER TECHNIQUES. IDENTIFYING FALSE COGNATS. 28
TEACHER ANDREA BELO

In 1985, Stephen got really sick and doctors were able to save him, but he was unable to speak.
He could only use his eyebrows to communicate. Eventually, he was able to use a special voice
synthesizer, allowing him to talk by moving his cheek muscles and using a mouse pad.
Famous Works and Prizes
His most notorious theory is that black holes can emit radiation; also known as Hawking radiation.
He received numerous awards but never won the Nobel Prize.
Stephen always enjoyed writing books. His best seller, “A Brief History in Time” made terms like
the Big Bang and black holes easy to understand. Other famous Stephen Hawking books include:
“A Briefer History in Time", “On the Shoulders of Giants” and "The Universe in a Nutshell". He also
wrote many books for children along with his daughter Lucy. His famous books for children include
"George’s Cosmic Treasure Hunt" and “George and the Big Bang”.
His last work, submitted only two weeks before his death, reveals the universe will come to an
end when stars run out of energy. However, his theory suggests that scientists will be able to find
parallel universes using probes on spaceships.

QUESTÃO 02 (COLÉGIO NAVAL/2018) – Mark the sentence which has an adjective that is NOT in
the superlative degree.
(A) Stephen Hawking, one of the most famous scientists of the 21st century (...).
(B) But his ideas (...) are the greatest legacy he left to the world.
(C) He was not the best student at fundamental or high school.
(D) His most notorious theory is that black holes can emit radiation.
(E) He started to get worse, and, eventually, became confined to a wheelchair.

QUESTÃO 03 (COLÉGIO NAVAL/2018) – Say if the following statements are T (true) or F (false)
about Hawking’s disease. Then mark the correct option, from top to bottom.
( ) It confined him to a wheelchair.
( ) It affected his ability to speak.
( ) It was diagnosed when he was 21 years old.
( ) It progressed more quickly than expected.
( ) It made him sturhble for no reason.
(A) (T) (T) (T) (F) (F)
(B) (T) (F) (T) (F) (F)
(C) (T) (T) (T) (F) (T)
(D) (F) (T) (F) (T) (T)
(E) (F) (T) (F) (T) (F)

AULA 01 – SKIMMING AND SCANNING AND OTHER TECHNIQUES. IDENTIFYING FALSE COGNATS. 29
TEACHER ANDREA BELO

QUESTÃO 04 (COLÉGIO NAVAL/2018) – What statement is correct about Hawking’s last work?
(A) It predicted the end of the universe.
(B) It suggests there are no parallel universes.
(C) It could not be finished because of his death.
(D) It was his bestselling book and received a prize.
(E) It introduced his theory about the Big Bang.

QUESTÃO 05 (COLÉGIO NAVAL/2018) – It can be inferred from the text that Stephen Hawking
(A) received numerous awards, including a Nobel Prize.
(B) was seventy six years old when he died.
(C) married Jane one year after he got his Ph.D degree.
(D) wrote “A Brief History in Time” for kids.
(E) wanted to be a doctor, but studied physics instead.

QUESTÃO 06 (COLÉGIO NAVAL/2018) – In the sentence, “Despite his poor grades at school,
Stephen aced his exams for college.” (Paragraph 3), the underlined phrase means that Stephen
(A) got bad grades in his tests.
(B) passed with average grades.
(C) achieved high marks in the test.
(D) tried hard but did not pass.
(E) had a second change and passed.

QUESTÃO 07 (COLÉGIO NAVAL/2018) – Read the sentence in order to do the question below.
This is my friends’ car and that one is my car. Mark the option which rewrites the sentence using
the correct possessive.
(A) This car is theirs and that one is mine.
(B) This is theirs cars and that is my car.
(C) This is their car and that one is my.
(D) This car is them and that one is my.
(E) This is theirs car and that one is mine.

AULA 01 – SKIMMING AND SCANNING AND OTHER TECHNIQUES. IDENTIFYING FALSE COGNATS. 30
TEACHER ANDREA BELO

QUESTÃO 08 (COLÉGIO NAVAL/2018) – Mark the sentence which is grammatically correct.


(A) How many girls are there at your party last week?
(B) Is there anybody in room 201 at the moment?
(C) Sarah said there is twelve people waiting.
(D) There isn’t an accident in our street last night.
(E) There is exercise bars so you can work out.

QUESTÃO 09 (COLÉGIO NAVAL/2018) – Read the text to do the question below. Complete with
a, the or 0 (no article).
_______________ Brazil is a huge country. In _______________ North, there are
_______________ rain forests and _______________ longest river is also situated there. In
_______________ South, _______________ climate is more European. _______________ Brazil
also has many social differences. _______________ rich own most of _______________ country’s
wealth and _______________ poor often live on minimum wage.
Now mark the option which completes the gaps respectively.
(A) 0 - the - 0 - a - the - the - 0 - a - the - a
(B) The - the - 0 - a - 0 - the - the - The - 0 - the
(C) 0 - the - 0 - the - the - the - 0 - The - the - the
(D) The - 0 - 0 - a - 0-the- The - 0 - a - 0
(E) The - 0 - the - a - 0 - the - The - a - a - a

QUESTÃO 10 (COLÉGIO NAVAL/2018) – Read the comic strip in order to do the question below.
The correct pronoun that completes the third bubble speech is
(A) they.
(B) it.
(C) theirs.
(D) its.
(E) them.

Robotic Cars
The year is 2020, and it’s 7;45 on a rainy Monday morning, and you are in your car on your way
to work. You turn right, and you turn left. A few minutes later, you stop at a traffic light. When
the light turns green and there are no other cars in the intersection, you continue on your way.
Ten minutes later you get to work and stop reading the morning paper. Then, you get out of your
car and you say, “Thank you!". Your car replies, “You’re welcome!’’. This possibie future may

AULA 01 – SKIMMING AND SCANNING AND OTHER TECHNIQUES. IDENTIFYING FALSE COGNATS. 31
TEACHER ANDREA BELO

sound unreai, but in fact many car companies are aiready testing robotic cars, or driverless cars,
on the roads today, although the cars don't speak very much yet.
In the 1980s, Germany and the United States tested the first driverless cars, and by 2020
companies such as Volvo, GM, Nissan and BMW plan to seil driverless cars. Driverless cars are not
really ‘driverless - the drivers are computers that use radar, Computer maps and other modern
technology. They offer many advantages. Perhaps the most important of these is fewer deaths
caused by road accidents. For example, in 1968 more than 53.000 people lost their lives in car
accidents in the USA. This number has fallen to less than 33.000 but it’s still a high number. In
addition, people will spend iess time stuck in traffic jams and there will be no need for people to
have a driving license. One of the major disadvantages of this new technology, however, is the
cost. It’s not free. U$5.000 to U$10.000 is added to the price of the new car. Nevertheless, at
some time in your life, you will probably be sitting in a robotic, ;driverless car on your way to work
or school. The future is almost here. Are you ready for it?
<https://www.aHthinastoDics.eom/uploads/2/3/2/9/23290220/lesson-drivina-robo ticcars2.pdf>

Questão 11 (COLÉGIO NAVAL/2019) – Read the extract from the text


“Ten minutes after you get to work and stop reading the morning paper”
Mark the option that can replace the expression get to
(A) drive to
(B) go to
(C) find
(D) arrive at
(E) come from

Questão 12 (COLÉGIO NAVAL/2019) – Read the extract from the text


“The year is 2020, and it’s 7:45 on a rainy Monday morning (...)”
What’s the correct question referring to the underlined information?
(A) What’s the weather?
(B) What’s the weather like?
(C) How’s the weather?
(D) How’s the climate like?
(E) What’s the climate?

AULA 01 – SKIMMING AND SCANNING AND OTHER TECHNIQUES. IDENTIFYING FALSE COGNATS. 32
TEACHER ANDREA BELO

Questão 13 (COLÉGIO NAVAL/2019) – Read the extract from the text

“They offer many advantages”

The subject pronoun they refers to

(A) Germany and The USA

(B) Volvo, GM, Nissan and BMW

(C) computers with radars

(D) driverless cars

(E) computer maps

Questão 14 (COLÉGIO NAVAL/2019) – Read the sentence below

“That is an intelligent car that works with a computer”

What is the plural of the sentence?

(A) These are intelligents cars that work with computers

(B) Those are intelligent cars that work with computers

(C) Those are intelligents cars that works with computers

(D) These are intelligent cars that works with computers

(E) Those are intelligent cars that works with computers

Questão 15 (COLÉGIO NAVAL/2019) – Read the extract from the text

“When the light turns green and there are no other cars in the intersection (...)”

Mark the option that can replace the underlined sentence

(A) there are some other cars

(B) there aren’t any other cars

(C) there is another car

(D) there are a few other cars

(E) there aren’t many cars

AULA 01 – SKIMMING AND SCANNING AND OTHER TECHNIQUES. IDENTIFYING FALSE COGNATS. 33
TEACHER ANDREA BELO

QUESTÕES EAM
QUESTÃO 01 (EAM/INÉDITA) – Read the dialogue and mark the right option to fill in the gaps
respectively.
A: _________ you go to Anne’s party yesterday?
B: Yes, I ________.
A: Who ________ you go with?
B: I ________ with my girlfriend. Were you there?
A: No, I ________.
a) Did / did / went / was / weren’t
b) Were / was / were / go / didn’t
c) Was / was / did / was / weren’t
d) Were / was / did / went / wasn’t
e) Did / did / did / went / wasn’t

QUESTÃO 02 (EAM/INÉDITA) – Read the sentences and mark the correct option to fill in the
blanks respectively.
Fabi is ______ teammate. We usually create new english tests every week. We live _____ São
Paulo and we love to eat in different restaurants every day. We also love _______ english during
our Podcasts.
a) My / in / speaking
b) Your / at / speaking
c) Her / on / speaks
d) My / in / speak
e) His / in / speak

QUESTÃO 03 (EAM/INÉDITA)

AULA 01 – SKIMMING AND SCANNING AND OTHER TECHNIQUES. IDENTIFYING FALSE COGNATS. 34
TEACHER ANDREA BELO

What’s the main verb tense used in the comic strip?


a) Present Continuous
b) Present Simple
c) Simple Past
d) Future Simple
e) Persent Perfect

QUESTÃO 04 (EAM/INÉDITA) – Use the verbs in the parentheses to complete the following
statements.
I- He always _______(run) early in the morning.
II- We need to ______(take) care of the elderly due to coronavirus outbreak.
III- Jack used to ______(live) in Boston. But now, he lives in Sao paulo.
Now mark the option which completes them respectively.
a) run / take / live
b) runs / take / lived
c) ran / takes / lived
d) runs / take / live
e) run / takes / lives

QUESTÃO 05 (EAM/INÉDITA) – Look at the picture below.

What are they doing in the picture?


a) They are destroying the snowman.
b) They are building a snowman.
c) They are laughing at the snowman.
d) They are throwing snowballs.
e) They don’t like the snowman they built.

AULA 01 – SKIMMING AND SCANNING AND OTHER TECHNIQUES. IDENTIFYING FALSE COGNATS. 35
TEACHER ANDREA BELO

QUESTÕES EEAR
Read the text and answer questions
Diplomat
There are three main aspects to this profession: a diplomat has to keep his country informed
about pertinent internacional events, promote a favorable image of his country and protect his
country’s interests.
Whoever is interested in a diplomatic career has to be extremely familiar with political,
economical, scientific, cultural and administrative issues. To be a diplomat, it is essential to have
a good knowledge of English, not only the conversational language, but also the technical terms
in internacional law and diplomacy itself.
To follow this career, besides being fluently bilingual, one needs a standard college education and
has to take and do well in the Rio Branco Institute examination in Brasilia.
English is so important in this career that in the first part of this selection exam, the applicant has
to demonstrate his or her proficiency in the english language. Then, during the course, foreign
language classes become a priority, giving future diplomats the necessary expertise to deal with
the areas of official correspondences, diplomatic negotiation and internacional media.
(Adapted from Inglês no mundo do trabalho)

QUESTÃO 01 (EEAR/2020 – BCT) – Change the following sentence to the passive voice:
“A diplomat must inform his country about internacional events.”
a) His country must inform a diplomat about internacional events.
b) His country has to be informed by a diplomat about international events.
c) His country will be informed by a diplomat about international events.
d) His country must be informed by a diplomat about internacional events.

QUESTÃO 02 (EEAR/2020 – BCT) – According to the text, we can infer that:


A Diplomat ________.
a) doesn’t need a normal college education
b) must be familiar with English, but only in the conversational language
c) doesn’t have to give much attention to foreign language classes during the course
d) must have a comprehensive knowlegde of political, economical, scientific and administrative
problems

AULA 01 – SKIMMING AND SCANNING AND OTHER TECHNIQUES. IDENTIFYING FALSE COGNATS. 36
TEACHER ANDREA BELO

QUESTÃO 03 (EEAR/2020 – BCT) – In “… giving future diplomats the necessary expertise to deal
with”, the underline word in the text is closest in meaning to, EXCEPT:
a) Skill
b) Knowledge
c) proficiency
d) unawareness

Read the text and answer questions


Oil contaminating Brazil's beaches very likely from Venezuela
Anna Jean Kaiser
Thick crude oil that has stained hundreds of miles of pristine Brazilian beach in recent weeks
probably originated in Venezuela, the Brazilian government has said, in an accusation likely to
further strain relations between the two countries.
Brazilians authorities have been investigating the growing disaster for more than a month, as the
oil has spread to more than 130 beaches across nine states.
Ricardo Salles, the country’s environment minister, told that a study by Petrobrás had concluded
that the oil “is very likely from Venezuela. He said that a foreign ship near Brazil’s coastline
appeared to have caused the spill.
There was no immediate response from Venezuela.
Social media users have shared shocking images of the spill, showing kilometers of white sand
stained with oil blotches and dead, oil-covered turtles and dolphins. One video shows thick black
oil lapping up against a rocky jetty.
Adapted from The Guardian

QUESTÃO 04 (EEAR/2020 – BCT) – According to the text, we can infer that ________.
a) The spill was caused by a domestic ship near Brazilian beaches
b) No dolphins and turtles died according to the images shared by the media
c) A dense and natural oil marked hundreds of miles of clean beaches recently
d) The government is investigating to discover how many beaches are involved in the disaster

QUESTÃO 05 (EEAR/2020 – BCT) – The word “strain”, underline in the text, is closest in meaning
to, EXCEPT:
a) close
b) tense
c) difficult
d) problematic

AULA 01 – SKIMMING AND SCANNING AND OTHER TECHNIQUES. IDENTIFYING FALSE COGNATS. 37
TEACHER ANDREA BELO

QUESTÕES EFOMM
Based on the text below, answer questions 1 to 10.
Residents and businesses of south Florida are experiencing regular episodes of water in the
streets. In the battle against rising seas, the region – which has more to lose than almost
anywhere else in the world – is becoming ground zero.
The first time my father’s basement flooded, it was shortly after he moved in. The building was
an ocean-front high-rise in a small city north of Miami called Sunny Isles Beach. The marble lobby
had a waterfall that never stopped running; crisp-shirted valets parked your car for you. For the
residents who lived in the more lavish flats, these cars were often BMWs and Mercedes. But no
matter their value, the cars all wound up in the same place: the basement.
When I called, I’d ask my dad how the building was doing. “The basement flooded again a couple
weeks ago,” he’d sometimes say. Or: “It’s getting worse.” It’s not only his building: he’s also driven
through a foot of water on a main road a couple of towns over and is used to tiptoeing around
pools in the local supermarket’s car park.
Ask nearly anyone in the Miami area about flooding and they’ll have an anecdote to share. Many
will also tell you that it’s happening more and more frequently. The data backs them up.
It’s easy to think that the only communities suffering from sea level rise are far-flung and remote.
And while places like the Solomon Islands and Kiribati are indeed facing particularly dramatic
challenges, they aren’t the only ones being forced to grapple with the issue. Sea levels are rising
around the world, and in the US, south Florida is ground zero – as much for the adaptation
strategies it is attempting as for the risk that it bears.
One reason is that water levels here are rising especially quickly. The most frequently-used range
of estimates puts the likely range between 15-25cm (6-10in) above 1992 levels by 2030, and 79-
155cm (31-61in) by 2100. With tides higher than they have been in decades – and far higher than
when this swampy, tropical corner of the US began to be drained and built on a century ago –
many of south Florida’s drainage systems and seawalls are no longer enough. That means not only
more flooding, but challenges for the infrastructure that residents depend on every day, from
septic tanks to wells. “The consequences of sea level rise are going to occur way before the high
tide reaches your doorstep,” says William Sweet, an oceanographer at the National Oceanic and
Atmospheric Administration (NOAA).
The flooding would be a challenge for any community, but it poses particular risks here. One
recent report estimated that Miami has the most to lose in terms of financial assets of any coastal
city in the world, just above Guangzhou, China and New York City. This 120-mile (193km) corridor
running up the coast from Homestead to Jupiter – taking in major cities like Miami, Fort
Lauderdale and West Palm Beach – is the eighth most populous metropolitan area in the US. It’s
also booming. In 2015, the US Census Bureau found that the population of all three counties here
was growing – along with the rest of Florida – at around 8%, roughly twice the pace of the US
average. Recent studies have shown that Florida has more residents at risk from climate change
than any other US state.
Adapted from https://www.bbc.com/future/article/20170403-miamis-fight-against-sea-level-rise

AULA 01 – SKIMMING AND SCANNING AND OTHER TECHNIQUES. IDENTIFYING FALSE COGNATS. 38
TEACHER ANDREA BELO

QUESTÃO 01 (EFOMM/INÉDITA) – It is possible to infer from the text that


a) technology will solve the problem Miami is facing due to sea water level rising.
b) the Earth is changing and areas like Miami will disapear soon.
c) in the future, water levels will decrease naturally.
d) buildings near the sea are endangered if sea water level keeps rising at that speed.
e) Miami metropolitan area will be the most populous area in the US in a few years.

QUESTÃO 02 (EFOMM/INÉDITA) – Read the statements about the text and decide whether they
are TRUE (T) or FALSE (F). Mark the correct option.
I – The floods have been taking place regularly in Sunny Isles Beach.
II – There are swimming pools in supermarkets’ car parks in Miami.
III – Residents in Miami area tell a lot of lies about flooding.
IV – Not only remote places are suffering from sea level rise.
V – The BMWs and Mercedes are protected against floodings because they are expensive cars.
a) F / T / T / F / F
b) T / F / F / T / F
c) T / T / T / F / T
d) F / F / F / T / F
e) F / T / F / T / T

QUESTÃO 03 (EFOMM/INÉDITA) – In the excerpt “It’s easy to think that the only communities
suffering from sea level rise are far-flung and remote.”, the word in bold means:
a) Distant
b) Poor
c) Ugly
d) Neglected
e) Near

QUESTÃO 04 (EFOMM/INÉDITA) – According to the text, sea water level


a) reached its maximum level recently.
b) begets consequences that will be seen in many years from now.
c) is considerably higher than it was 100 years ago.
d) is ideal to attract tourists now.
e) doesn’t affect people’s lives in the area.

AULA 01 – SKIMMING AND SCANNING AND OTHER TECHNIQUES. IDENTIFYING FALSE COGNATS. 39
TEACHER ANDREA BELO

QUESTÃO 05 (EFOMM/INÉDITA) – Choose the correct option to complete the paragraph below.
Those who are non-essential workers (and followed the rules) have been _____ home all day,
every day, save for trips to the grocery store or for socially distant walks. And yet, the number of
new cases of COVID-19 _____ the U.S. continues to go up each day, _____ about 2 to 4 percent.
Adapted from https://www.msn.com/en-us/health/other/if-people-are-staying-home-why-is-coronavirus-still-spreading/ar-BB13HSW8?ocid=bingcovid

a) in / on / on
b) on / in / at
c) at / in / by
d) at / on / in
e) in / in / on

QUESTÃO 06 (EFOMM/INÉDITA) – Choose the best alternative to complete the excerpt below.
Finding _____ effective coronavirus vaccine ______ a global priority in ending the pandemic. US
government leaders have put forward the ambitious timeline ______ have one by the end of
2020. It typically takes several years to develop ______ vaccine.
https://www.businessinsider.com/moderna-coronavirus-vaccine-releases-first-human-trial-results-2020-5

a) an / has become / to / a
b) a / has become / to / a
c) an / became / to / the
d) an / has become / of / the
e) a / became / of / a

QUESTÃO 07 (EFOMM/INÉDITA) – Choose the correct alternative to complete the paragraph


below.
In the last few weeks a spate of American stores _______ headlines after putting up signs _______
customers who wear masks they will be denied entry. On Thursday, Vice reported on a Kentucky
convenience store that put up a sign reading: “NO Face Masks ______ in store. Lower your mask
or go somewhere else.
Adapted from https://www.theguardian.com/us-news/2020/may/22/us-stores-against-face-masks

a) have made / telling / allow


b) made / told / allowed
c) have made / telling / allowed
d) made / have told / allow
e) has made / told / allowed

AULA 01 – SKIMMING AND SCANNING AND OTHER TECHNIQUES. IDENTIFYING FALSE COGNATS. 40
TEACHER ANDREA BELO

QUESTÃO 08 (EFOMM/INÉDITA) – Which option is correct?


a) She needs two different equipments to run her business.
b) The pandemy can affect world economy.
c) The social media become very important to businesses.
d) His nose is running because he has a cold.
e) The Real family is very traditional in the UK.

QUESTÃO 09 (EFOMM/INÉDITA) – Mark the option which corresponds to the correct sentences.
I – The person who rules a soccer game is called judge.
II – Songs for children are called nursery rhymes.
III – Mooring is an important process when the ship is sailing.
IV – Your best friends aren’t never far from you.
V – A ship can also be called vessel.
a) I and II
b) II and V
c) III and V
d) II and IV
e) III and IV

QUESTÃO 10 (EFOMM/INÉDITA) – Which option is correct to complete the sentences below?


1 – He heard her ______ the stairs.
2 – He saw her ______ the door.
3 – I watched him ______ the job.
4 – In order to be promoted, you need to be ______ by your boss.
5 – He couldn’t see his child _______ with our dog.
a) climb / slam / do / valued / play
b) climbed / slamming / do / value / play
c) climbing / slam / doing / valuing / playing
d) climbing / slamming / doing / valued / playing
e) climbed / slamming / doing / valued / playing

AULA 01 – SKIMMING AND SCANNING AND OTHER TECHNIQUES. IDENTIFYING FALSE COGNATS. 41
TEACHER ANDREA BELO

QUESTÕES EPCAR
The WHO has redefined burnout as a syndrome linked to chronic work stress. There’s a difference
between a busy workload and something more serious, writes Zaria Gorvett.
If you said you were suffering from ‘burnout’ in the early 1970s, you might have raised some
eyebrows.
At the time, the term was used informally to describe the side effects that heavy drug users
experienced: the general dimming of the mental faculties, for example, as was the case with many
a party animal. However, when German-American psychologist Herbert Freudenberger first
recognised the problem of burnout in New York City in 1974, at a clinic for addicts and homeless
people, Freudenberger wasn’t thinking of drug users.
The clinic’s volunteers were actually struggling, too: their work was intense, and many were
beginning to feel demotivated and emotionally drained. Though they had once found their jobs
rewarding, they had become cynical and depressed; they weren’t giving their patients the
attention they deserved. Freudenberger defined this alarming new condition as a state of
exhaustion caused by prolonged overwork – and borrowed the term ‘burnout’ to describe it.
Its popularity was explosive, and today burnout is a global phenomenon. Although statistics on
the prevalence of burnout specifically are hard to come by, 595,000 people in the UK alone
suffered from workplace stress in 2018.
Sportspeople get it. YouTube stars get it. Entrepreneurs get it. Freudenberger himself eventually
got it. Late last month, the World Health Organization (WHO) announced that the trendy problem
will be recognised in the latest International Classification of Diseases manual, where it is
described as a syndrome “resulting from chronic workplace stress that has not been successfully
managed”.
According to the WHO, burnout has three elements: feelings of exhaustion, mental detachment
from one’s job and poorer performance at work. But waiting until you’re already fully burned out
to do something about it doesn’t help at all –and you wouldn’t wait to treat any other illness until
it was too late.
Feeling the burn
So how can you tell if you’re almost – but not quite – burned out?
“A lot of the signs and symptoms of pre-burnout would be very similar to depression,” says
Siobhán Murray, a psychotherapist based in County Dublin, Ireland, and the author of a book
about burnout, The Burnout Solution. Murray suggests looking out for creeping bad habits, such
as increased alcohol consumpution and relying on sugar to get you through the day. Also watch
out for feelings of tiredness that won’t go away. “So that even if you do sleep well, by
10 in the morning you’re already counting down the hours to bed. Or not having the energy to
exercise or go for a walk.”
As soon as you begin to feel this way, Murray advises going to see your doctor.
“Depression and pre-burnout are very similar, but as much as there was a lot of enthusiasm
recently that burnout has now become a medical condition, it is still not – it is still classified as an
occupational phenomenon.” It’s important to get help from a medical professional who can

AULA 01 – SKIMMING AND SCANNING AND OTHER TECHNIQUES. IDENTIFYING FALSE COGNATS. 42
TEACHER ANDREA BELO

distinguish between the two, because although there are many treatment options for depression,
burnout is still best tackled by making lifestyle changes.
And how do you know if you’re really on the cusp of burnout, or just going through a challenging
month? “Stress is really important, and anxiety is what motivates us to do well,” says Murray. “It’s
when we’re continually exposed to stress and anxiety, that we’re not letting go, that it starts to
turn into burnout.”
Take that big project you’ve been working on. It’s normal to feel a kick of adrenaline when you
think about it, and maybe it’s kept you up at night. But, Murray suggests, if you still feel restless
once it’s over, it’s time to consider if you’re at risk of burnout. “It’s when you’re bringing that with
you into the next stage of your day, and adding to it continually,” she says.
Another classic sign of inching closer to burnout is cynicism: feeling like your work has little value,
avoiding social commitments and becoming more susceptible to disappointment.
“Someone on the brink will probably begin to feel emotionally numbed or mentally distant,” says
Jacky Francis Walker, a psychotherapist based in London who specialises in burnout. “Like they
don’t have the capacity to engage as much in the ordinary things of life.”
She also recommends looking for the final tell-tale sign of burnout, which is the unshakeable
feeling that the quality of your work is beginning to slip. “People say ‘but this isn’t me!’, ‘I’m not
like this’, ‘I can usually do x,y and z’. But obviously if they are in a state of physical depletion, then
they aren’t in their normal range of capabilities,” says Walker.
If this seems less than scientific, look to the Maslach Burnout Inventory (MBI), a test designed to
measure burnout. The most widely used is the MBI-General Survey, which measures things like
exhaustion, cynicism, and some how well you think you’re doing at work.
First published in 1981, it has been cited hundreds of times in studies since. Although it’s typically
used to measure burnout once it's in full swing, there’s no reason you can't apply it to see if you’re
getting close.
(Adapted from https://www.bbc.com/worklife/article/20190610-how-to-tell-if-youve-got-pre-burnout)

Questão 01 (EPCAR/INÉDITA) – Mark the correct option.


a) Burnout is now considered a medical condition.
b) Burnout can improve people’s performance at work.
c) Burnout was widely known in the 1970’s.
d) Herbert Freudenberger first used the term burnout in the 1970’s.

Questão 02 (EPCAR/INÉDITA) – According to the text, mark the correct alternative.


a) The psychologist discovered burnout by accident while analizing drug addicts and homeless.
b) The clinic’s volunteers were having a good time doing their work.
c) Anyone is subject to get it if they reach exhaustion due to prolonged overwork and stress.
d) Drug addicts were the first to suffer from burnout due to their stressful routine.

AULA 01 – SKIMMING AND SCANNING AND OTHER TECHNIQUES. IDENTIFYING FALSE COGNATS. 43
TEACHER ANDREA BELO

Questão 03 (EPCAR/INÉDITA) – In the sentence “the general dimming of the mental faculties”
(paragraph 2) the word dimming means
a) decrease.
b) augmentation.
c) assessment.
d) improvement.

Questão 04 (EPCAR/INÉDITA) – Mark the correct question to the sentence below.


“Freudenberger defined this alarming new condition as a state of exhaustion caused by
prolonged overwork”
a) How did Freudenberger define this alarming new condition?
b) How has Freudenberger defined this alarming new condition?
c) How had Freudenberger defined this alarming new condition?
d) How did Freudenberger defined this alarming new condition?

Questão 05 (EPCAR/INÉDITA) – Mark the alternative that completes the sentence.


Some symptoms the psychologist noticed the volunteers had to deal with were
a) that they were drug abusers and most of them were homeless.
b) that they started to feel demotivated, emotionally drained, cynical and depressed.
c) that they were getting sick very frequently.
d) that they were being diagnosed with generalized anxiety and depression.

Questão 06 (EPCAR/INÉDITA) – Mark the alternative that DOESN’T complete the sentence
below.
The psychologist Herbert Freudenberger
a) was who first used the word burnout to describe this exhaustion condition.
b) recognised the problem of burnout in 1974 in New York City for the first time.
c) created this condition so that he would become known for discovering a disease.
d) defined burnout as a state of exhaustion caused by prolonged overwork.

AULA 01 – SKIMMING AND SCANNING AND OTHER TECHNIQUES. IDENTIFYING FALSE COGNATS. 44
TEACHER ANDREA BELO

Questão 07 (EPCAR/INÉDITA) – Read the sentences and mark the correct option.
I. The WHO has finally recognised burnout as a disease.
II. More than half a million people suffered from workplace stress in 2018 in the UK.
III. Relying on sugar to get you through the day can be a good way of keeping yourself from
burning out.
The only correct sentence(s) is(are)
a) II.
b) III.
c) I and II.
d) I and III.

Questão 08 (EPCAR/INÉDITA) – Mark the correct alternative to complete the sentence.


According to the text, the burnout
a) makes people happy because they feel that they are performing at their best.
b) is best tackled and can be reversed by making lifestyle changes.
c) has been responsible for many deaths around the world since the 1970’s.
d) is responsible for kicks of adrenaline when you think about a project you’ve been working on.

Questão 09 (EPCAR/INÉDITA) – In the underlined sentences in the second paragraph, there are
____ verbs used in the past tense.
a) four
b) one
c) three
d) two

Questão 10 (EPCAR/INÉDITA) – Volunteers


a) never liked their work at all.
b) weren’t giving enough attention to patients due to their burnout condition.
c) accepted being part of a experiment to find out what happens when people get to their limit
at work.
d) would still find their jobs rewarding even though they were burned out.

AULA 01 – SKIMMING AND SCANNING AND OTHER TECHNIQUES. IDENTIFYING FALSE COGNATS. 45
TEACHER ANDREA BELO

QUESTÕES ESA
Brazilian Army Joins Effort to Disinfect Prison from COVID-19
By VOA News
The Brazilian army, in conjunction with the country's prison administration, conducts a large-scale
disinfection operation to slow the spread of the coronavirus at the Gericinó Prison Complex in Rio
de Janeiro. A prison spokesman says soldiers outfitted in full body protection suits and face masks
clean the infirmary, coronavirus isolation cells, administrative areas, and visitor sections of the
prison. Colonel Rego Barros of the army's East Joint Command says the proactive measures are
taken after seeing situations in which inmates become infected by COVID-19 in other states.
So far, Brazil has more than 1,000,000 covid-19 cases and almost 50,000 deaths. Peru has the
second highest coronavirus case total in Latin American behind Brazil. Peru's National Penitentiary
Institute confirms reports of inmates setting fires and demanding to be freed, as more prisoners
become infected with the disease. Reuters news agency says human rights groups are calling on
the Peruvian government to allow house arrest during the pandemic.
(Adapted from https://www.voanews.com/covid-19-pandemic/brazilian-army-joins-effort-disinfect-prison-covid-19)

Questão 01 (ESA/INÉDITA) – Concerning the information in the text, it is correct to state that
a) house arrest is allowed in Peru during the pandemic.
b) the Brazilian army is remotely helping to prevent covid-19 spread in prisons.
c) soldiers are working on a disinfection operation in a prison in Rio de Janeiro.
d) Brazil is only behind Peru in total covid-19 cases in Latin America.
e) Brazil has a total of 1,050,000 covid-19 cases.

Questão 02 (ESA/INÉDITA) – Which option has a correct relation of the underlined terms and
their substitutes?
a) A prison spokesman says... — Him
b) Soldiers clean the coronavirus isolation cells. — Their
c) Brazil has more than 1,000,000 covid-19 cases. — He
d) Human rights groups are calling on the Peruvian government... — Them
e) Inmates become infected by COVID-19 in other states. — They

Questão 03 (ESA/INÉDITA) – “_________ Peru have the _________ covid-19 cases number in
Latin America?” Fill in the blanks with the correct form of the verb and the adjective.
a) Does / worst
b) Does / worse
c) Is / bad
d) Do / worst
e) Do / worse

AULA 01 – SKIMMING AND SCANNING AND OTHER TECHNIQUES. IDENTIFYING FALSE COGNATS. 46
TEACHER ANDREA BELO

Questão 04 (ESA/INÉDITA) – Which sentence is grammatically correct?


a) She needs a new furniture.
b) He reads many books every year.
c) That men is following me.
d) I have a phone new.
e) A car is more fast than a bike.

Texto para questão 05


Seville Hospital uses an experimental treatment against SARS-CoV-2 with successful results,
combining lopinavir and ritonavir antivirals and interferon beta.
By Richard Mann – March 1, 2020
RIO DE JANEIRO, BRAZIL - The application of lopinavir/ritonavir, also used to fight HIV infections,
along with interferon beta, a protein that helps cells protect themselves from infection, confirmed
medical sources to EL PAÍS.
"It's an experimental course of treatment that has yielded positive results against other viruses,"
explains Albert Bosch, president of the Spanish Society of Virology. "One of its greatest advantages
is that it is an approved drug used in other indications, so there is no question about its safety,"
he adds.
(Adapted from https://riotimesonline.com/brazil-news/brazil/brazil-declares-state-of-emergency-for-47-cities-due-to-rains/)

Questão 05 (ESA/INÉDITA) – According to the text, it is incorrect to say that:


a) The results have been satisfactory so far.
b) They are using an approved drug against coronavirus.
c) The experimental treatment is taking place in Seville.
d) Despite the good results, there are still questions about the treatment’s safety.
e) Using this treatment against other viruses has shown positive results, according to the
president of the Spanish Society of Virology.

QUESTÕES ESCOLA NAVAL


The current pandemic is forcing much of the world’s population to stay at home and take it
easy. But we may not be wired that way.
You may be among the more than three million people who’ve seen a short film issued by the
Governor of California’s office. It’s all over social media. In the video, the comedian Larry David,
in his trademark sardonic style, urges people to follow the official advice and stay at home to stop
the spread of Covid-19. What’s the matter with you "idiots", he says, you’re passing up a fantastic
opportunity to sit in an armchair and watch TV all day!

AULA 01 – SKIMMING AND SCANNING AND OTHER TECHNIQUES. IDENTIFYING FALSE COGNATS. 47
TEACHER ANDREA BELO

We’re used to health warnings that urge us to do things that we don’t really have a great urge to
do: to exercise more, to eat five or eight or even 10 portions of fruit and vegetables a day. But for
once the official advice sounds easy; loaf on the sofa, binge-watch box sets, stay at home. This all
sounds as though it should appeal to our lazier sides.
In fact, it is not as simple as that, as you’ve probably already found out, after a few weeks of
lockdown. It turns out we are not biologically programmed to do as little as possible. Indeed, we
thrive on activity. Or at least, a good balance between being busy and being able to rest.
It’s true that we often look for the easy option, the path of least resistance, the shortcut to
success. If you have a remote control, why get up and switch channels on the TV itself? If you have
a car why cycle to the supermarket? If you can get away with doing half as much work than a
colleague, then why not?
Any sort of work or effort involves mental and physical strain, so it makes sense to avoid it where
possible. And sometimes we do just that. This is sometimes known as the principle of least effort
or Zipf’s Law, a law you might think no one is ever tempted to break. Except that we break it all
the time.
Do you ever dream of doing absolutely nothing? Lying in a hammock for a whole afternoon. Just
staring at the ceiling, listening to the silence. It may sound like a lovely idea, but in fact we can
find doing nothing at all – and take sleep out of the equation – very hard to do. In a famous study
conducted a few years ago at the University of Virginia, participants were led one at a time into a
completely bare room with all distractions removed. They had no phone, no books, no screens –
and they weren’t allowed to take a nap. Electrodes were fitted to their ankles and they were left
alone for 15 minutes. It was an opportunity to kick back and relax for a short while.
So, how did it go? Well, before being left alone, participants were shown how to press a computer
key which was wired up to a machine that delivered an electric shock. You might suppose that
having tried it once no one would want to do it again. Wrong. In fact, 71% of the men and 25% of
the women gave themselves at least one electric shock during their time in solitary – and one man
shocked himself a shocking 190 times.
It turns out that having nothing to do was so excruciating, that many of the participants preferred
to, in effect, torture themselves rather than put up with no distractions whatsoever.
This experiment is an extreme example, but we know from everyday life that people constantly
choose to do things they don’t need to do and which are sometimes painful. Think of all of your
friends who run marathons or have punishing regimes at the gym.
They go way beyond what is required for health and fitness. And what about the people who trek
across the ice to reach the poles of the Earth or sail around the world?
Michael Inzlicht from the University of Toronto calls this the paradox of effort. Sometimes we take
the easy route and do as little as we can get away with, but at other times we value situations
more if we have to expend considerable effort. The intrinsic joy of the effort gives us so much
pleasure that we don’t take the short cut. We might spend hours puzzling over a cryptic crossword
instead of using a search engine to find the solution.

AULA 01 – SKIMMING AND SCANNING AND OTHER TECHNIQUES. IDENTIFYING FALSE COGNATS. 48
TEACHER ANDREA BELO

We learn this early in life. As children we are taught through experience and persuasion that effort
leads to reward and over time this conditions us to enjoy effort for its own sake. This is known as
learned industriousness.
All of this means, that while we stay home and self-isolate, lying on the sofa and watching TV will
form only a part of how we pass the time. We might think it is fun to laze around for a few weeks,
but in fact it will drive us to distraction. Enforced and extended rest, unless we are ill and our
bodies demand it, leads not to feelings of being relaxed but of restlessness and irritability.
We need to find ways during lockdown to replicate as far as we can the rhythms and sense of
balance we achieve, at our best, in ordinary life.
So, taking exercise, setting ourselves tasks, doing things that are effortful and difficult are
important. And we should all be looking for activities or experiences that promote what the
psychologist Mihaly Csikszentmihalyi calls flow, in his book Flow: The Psychology of Optimal
Experience. These are tasks, such as painting or gardening or doing jigsaws, which so absorb us
that we don’t notice time passing and we stop worrying about everything else.
In normal times, most of us don’t take rest seriously enough. So during this exceptional period,
we should embrace the opportunity to rest more if we can – and indeed take those more balanced
rhythms of rest and busyness into our lives post shutdown. But during this difficult time, we will
find that we are not instinctively lazy creatures.
Indeed in a weird way we might find that doing less, and resting more, initially requires quite a lot
of effort.
(Adapted from: https://www.bbc.com/future/article/20200602-are-human-beings-naturally-lazy)

Questão 01 (ESCOLA NAVAL/INÉDITA) – According to the text, which option completes the
sentence below correctly?
In normal times, ______.
(A) everybody wants to rest all the time
(B) we try to rest as much as we can
(C) nobody rests properly
(D) the majority of people don’t rest properly
(E) a few people don’t rest as much as they need

Questão 02 (ESCOLA NAVAL/INÉDITA) – What's the meaning of the word "sardonic" in


paragraph 1?
(A) Funny.
(B) Comic.
(C) Sarcastic.
(D) Iconic.
(E) Casual.

AULA 01 – SKIMMING AND SCANNING AND OTHER TECHNIQUES. IDENTIFYING FALSE COGNATS. 49
TEACHER ANDREA BELO

Questão 03 (ESCOLA NAVAL/INÉDITA) – According to the text, which option is correct?


(A) Most people find it difficult to stay home doing nothing for a long period of time.
(B) After a few weeks of lockdown, we realize that we are programmed to do as little as
possible.
(C) Most people are happy to stay at home during this pandemic.
(D) The pandemic is allowing people to find a good balance between being busy and being able
to rest.
(E) We always follow the principle of least effort while doing any activity in our lives.

Questão 04 (ESCOLA NAVAL/INÉDITA) – According to the text, which option is correct?


(A) Doing nothing for a while just staring at the ceiling and listening to the silence is proven to be
a great experience to any person.
(B) The paradox of effort consists in people always choosing the easiest path to do things.
(C) Sometimes we avoid any kind of effort at all costs, and sometimes we choose the harder way
of doing things.
(D) People will always choose to cycle to the supermarket because it’s healthier.
(E) We have always lived in good balance between being busy and being able to rest.

Questão 05 (ESCOLA NAVAL/INÉDITA) – In paragraph 4, the word "that" refers to


(A) People making unnecessary effort.
(B) Cycle to the supermarket.
(C) Get up to switch channels on the TV itself.
(D) Mental and physical strain.
(E) Avoid it.

Questão 06 (ESCOLA NAVAL/INÉDITA) – According to the text which option is correct?


(A) People found out they are lazy because they are enjoying staying at home and doing nothing
for weeks now.
(B) We are never tempted to break Zipf’s law. We only break it when we need to.
(C) A study conducted at University of Virginia showed that many people can’t stand inactivity.
(D) A study conducted at University of Virginia showed we all enjoy having 15 minutes to relax.
(E) Michael Inzlicht was the person responsible for conducting the famous study at the
University of Virginia.

AULA 01 – SKIMMING AND SCANNING AND OTHER TECHNIQUES. IDENTIFYING FALSE COGNATS. 50
TEACHER ANDREA BELO

Questão 07 (ESCOLA NAVAL/INÉDITA) – Which is the correct option to complete the paragraph
below?
Social Integration
Social integration is ______ actual participation in various social relationships, ranging ______
romantic partnerships to friendships.
This integration involves emotions, intimacy, and a sense of belonging ______ different social
groups, such as being part of a family, a partnership, a social activity, or a religious community.
Experts suggest that ______ integrated into such social relationships confers a protective benefit
against maladaptive behaviors and damaging health consequences.
(Adapted from https://www.verywellmind.com/social-support-for-psychological-health-4119970)

(A) the / from / to / being


(B) an / from / at / being
(C) the / to / to / be
(D) the / from / to / be
(E) a / to / at / being

Questão 08 (ESCOLA NAVAL/INÉDITA) – Which option completes the dialogue below correctly?
John: What's the problem?
Mary: My son hasn’t taken a shower yet. He must _______ today.
(A) have done it
(B) have it done
(C) done it
(D) has done it
(E) has it done

Questão 09 (ESCOLA NAVAL/INÉDITA) – Which option completes the paragraph below


correctly?
Latin America's largest country is testing people ______ a rate far lower than any other nation
with at least 40,000 cases. It tests 12 times fewer people ______ Iran, and 32 times fewer ______
the United States. Hospitalized patients aren't being tested. Some medical professionals aren't
being tested. People are dying ______ their homes without being tested.
(Adapted from https://www.washingtonpost.com/world/the_americas/coronavirus-brazil-testing-bolsonaro-cemetery-gravedigger)

(A) at / that / that / on


(B) with/ that / that / on
(C) at / than / than / in
(D) with/ than / that / in
(E) at / than / than / on

AULA 01 – SKIMMING AND SCANNING AND OTHER TECHNIQUES. IDENTIFYING FALSE COGNATS. 51
TEACHER ANDREA BELO

Questão 10 (ESCOLA NAVAL/INÉDITA) – Which word best completes the question below?
How ______ does he go for a run?
It’s really important to exercise in order to keep healthy.
(A) far
(B) old
(C) high
(D) often
(E) many

QUESTÕES EsPCEx
Leia o texto a seguir e responda às questões 01, 02 e 03.
What Happened When Hong Kong’s Schools Went Virtual to Combat the Spread of
Coronavirus
Because of the coronavirus outbreak, all schools in Hong Kong have been closed since January,
and won’t reopen until late April at the earliest. "The exact date of class resumption is subject to
further assessment," announced the Education Bureau, which controls all schools in Hong Kong,
public and private, on February 25. It’s all part of the “social distancing” measures the city has
mandated to slow the virus’s spread, which include closing libraries, museums and recreation
facilities like pools. Students from preschoolers through PhD candidates are now doing all their
education online, a move the Education Bureau calls "suspending classes without suspending
learning."
As coronavirus spreads across the globe, other countries are joining Hong Kong and mainland
China in this massive, unplanned experiment in online learning. According to Unesco, as of Friday,
14 countries have shut schools down nationwide, affecting upwards of 290 million students, while
13 countries, including the United States, have seen localized school closings.
In recent days, schools from Scarsdale, New York, to San Francisco have closed temporarily over
contagion concerns. The University of Washington and Stanford University have turned to online
classes for the remainder of the quarter, and others are following suit for various lengths of time.
Some experts believe more widespread and long-term closures will be necessary in areas with
high levels of community transmission. States are preparing for that possibility by looking at their
own online learning policies.
Adapted from https://www.smithsonianmag.com/innovation/what-can-americans-learn-from-hong-kongs-unplanned-experiment-online-learning-during-coronavirus-180974331/

Questão 01 (EsPCEx/INÉDITA) – What is the main topic of the text?


a) Schools in Hong Kong closed for good because of coronavirus.
b) The changes schools had to implement in education due to coronavirus.
c) School closures in Hong Kong thanks to lack of students.
d) Students in many countries can’t be home schooled.
e) How schools are educating students about coronavirus.

AULA 01 – SKIMMING AND SCANNING AND OTHER TECHNIQUES. IDENTIFYING FALSE COGNATS. 52
TEACHER ANDREA BELO

Questão 02 (EsPCEx/INÉDITA) – In the sentence “The exact date of class resumption is subject to
further assessment” (paragraph 1), the word further expresses
a) addition.
b) distance.
c) contrast.
d) time.
e) conclusion.

Questão 03 (EsPCEx/INÉDITA) – In the sentence “According to Unesco, as of Friday, 14 countries


have shut schools down nationwide” (paragraph 2), the phrasal verb shut down means
a) demolish.
b) decrease.
c) lower.
d) disconnect.
e) close.

Leia o texto a seguir e responda às questões 04, 05 e 06.


The Bod Pod
It’s a small, yet ordinary office space tucked around the corner of a lobby. It’s exactly how you
would think a medical building would look, but in this office, there is a piece of equipment that
looks similar to a space escape pod from a sci-fi movie. Despite its looks, its purpose has nothing
to do with space. The Body Composition Machine, or commonly referred to as the “Bod Pod”, is
an instrument that helps determine total body fat percentage, lean muscle mass as well as resting
metabolic rate and total energy expenditure of an individual.
“It’s state of the art,” said Alan Muriera, 460th Medical Group health promotion manager. “The
Bod Pod results are around 99.5% accurate. It is tested and calibrated everyday. We check the
seals, we check the airway valves, and we check the volume.” Muriera works in the Human
Performance Center on Buckley Air Force Base, Colo., where he administers the Bod Pod
examinations. The Bod Pod is a computerized, egg-shaped chamber which uses the same whole-
body measurement principle as underwater weighing, except it utilizes air to measure your total
body fat. As air is pumped into the chamber, body mass is calculated by the amount of air your
body displaces.
“When people come here, they are either looking to lose weight, or to see muscle gain,” said
Muriera. The Bod Pod is an opportunity for you to acquire baseline data prior to initiating a new
workout or diet regimen. The analysis takes only five minutes to accomplish and is non-invasive.
Muriera explains, “I also like to call it the “Truth Booth”, because I’ve had people come in thinking
they have 15% total body fat, and they are actually 26%.” This machine is accurate and produces
results with a technologically advanced system that will show minute details. “It was definitely a

AULA 01 – SKIMMING AND SCANNING AND OTHER TECHNIQUES. IDENTIFYING FALSE COGNATS. 53
TEACHER ANDREA BELO

humbling experience,” said Chief Master Sgt. Robert Devall, 460th Space Wing command chief. “I
don’t think I’m in bad shape, but I wasn’t quite where I wanted to be after seeing the results.”
Adapted from https://www.buckley.af.mil/News/Article-Display/Article/2105959/the-bod-pod/

Questão 04 (EsPCEx/INÉDITA) – According to the text, read the statements and choose the
correct alternative.
I – The Bod Pod body measurement is somewhat similar to underwater weighing.
II – The text is about a space escape pod.
III – The Bod Pod is a machine that burns body fat.
IV – The Bod Pod can be helpful for people who want to get in shape.
V – Chief Devall is fully satisfied with his Bod Pod examination results.
VI – The Bod Pod can determine the number of calories burned by the individual.
a) I, IV and V are correct.
b) III, V and VI are correct
c) I, II and IV are correct.
d) I, IV and VI are correct.
e) II, III and V are correct.

Questão 05 (EsPCEx/INÉDITA) – the sentence “Despite its looks, its purpose has nothing to do
with space.” (paragraph 1), the word despite can be replaced by
a) in order to
b) in addition to
c) prior to
d) regardless of
e) as a result of

Questão 06 (EsPCEx/2020) - In the sentence “‘It’s state of the art,’ said Alan Muriera, 460th
Medical Group health promotion manager.” (paragraph 2) the expression state of the art refers
to
a) something creative.
b) something old-fashioned.
c) something modern.
d) something handmade.
e) something artistic.

AULA 01 – SKIMMING AND SCANNING AND OTHER TECHNIQUES. IDENTIFYING FALSE COGNATS. 54
TEACHER ANDREA BELO

Leia o texto a seguir e responda às questões 07, 08 e 09.


The US could see more deaths than WWII. It's time to deploy the military to hospitals
We are at war against the coronavirus. If we don’t act rapidly, we’re approaching a US death toll
larger than that of the two world wars combined. To fight an invisible army, we need to be creative.
We must deploy the full might of the US military – in an unconventional way. Unlike most wars,
this time the frontlines are staffed by our nation’s healthcare workers. And while they’re fighting
valiantly so far, the battle to flatten the curve is leaving them overrun.
In the US, we don’t have a centralized economy. Most of our major cities will have their hospital
systems overwhelmed simultaneously. And there will be few doctors to spare. So, what should we
do? To start, we should add medical students and retired doctors to the frontlines. But these
groups will fill only a small fraction of the added medical demand. We must rapidly train and
deploy a much larger workforce.
Let’s use one of America’s most valuable national assets – our top-tier military – to dramatically
increase our healthcare capacity. Members are trained to handle acute stress and are rapidly
deployable. They’re used to being under centralized command and are mentally prepared to be
away from their families. We should rapidly reassign any military doctors and paramedics that can
be spared to serve domestic needs. We don’t need to militarize our hospitals. But our hospitals
will need help from our military. This is an unprecedented crisis, and we need unprecedented
adaptability to save lives.
Adapted from https://www.theguardian.com/commentisfree/2020/mar/19/us-military-coronavirus-hospitals

Questão 07 (EsPCEx/INÉDITA) – Choose the alternative with the correct reference for the
underlined words from the text.
a) they (paragraph 1) = the frontlines.
b) them (paragraph 1) = our nation.
c) their (paragraph 2) = hospital systems.
d) their (paragraph 3) = families.
e) that (paragraph 3) = military doctors and paramedics.

Questão 08 (EsPCEx/INÉDITA) – According to the text, the act of “deploying” could be defined as
a) simulating a movement of troops during a health crisis.
b) coming back home after a mission in another country.
c) relocating military personnel to a specific destination.
d) rescuing civilians threatened by war from conflict zones.
e) hiring medical students to work at hospitals abroad.

AULA 01 – SKIMMING AND SCANNING AND OTHER TECHNIQUES. IDENTIFYING FALSE COGNATS. 55
TEACHER ANDREA BELO

Questão 09 (EsPCEx/INÉDITA) – According to the text, choose the correct statement.


a) coronavirus is likely to kill more people in the US than the two world wars together.
b) deploying the might of the military the traditional way is the solution against the virus.
c) the US must militarize their hospitals to contain the coronavirus spread.
d) most American cities will have their hospitals overwhelmed one at a time.
e) military training does not include stress management skills.

Leia o texto a seguir e responda à questão 10.


Is technology making time go quicker?
It never feels like there are enough hours in the day. If someone __________ (1) you a button to
get things done at twice the speed to free up time, you’d probably jump at it. In November, Netflix
introduced a feature to let users fast-forward their shows, playing at 1.5x speed. People are
already watching university lectures and their favourite YouTubers on double time and those that
speed-listen to podcasts – podfasters – are commonplace.
But fast forward __________ (2) an option since the days of VHS tapes. What’s so different now?
In 2018, Netflix released 1,500 hours of original content and to watch it all, it would take you four
hours of streaming a day for a whole year. With the same number of hours in the day, speed-
watching seems a logical solution, for both viewers and the streaming giants, to the information
and entertainment glut.
‘The brain is very capable of adapting to changes in experience,’ says neuroscientist Prof Edvard
Moser. Yet time is different from other sense data we adapt to, like sight and smell. The body
doesn’t have in-built sensors to measure time the way a clock does. It may __________ (3) like we
can sense time. But that’s a helpful illusion. Regardless of how time feels, there’s nothing stopping
us watching and consuming information at faster speeds than we do currently. ‘But maybe the
pleasure or enjoyment wouldn’t be the same.’
Adapted from https://metro.co.uk/2020/01/15/is-technology-making-time-go-quicker-12057474/

Questão 10 (EsPCEx/INÉDITA) – Choose the alternative containing the correct verb forms to
complete gaps (1), (2) and (3) in paragraphs 1, 2 and 3 respectively.
a) offers, have been, feel
b) offered, has been, feel
c) offered, have been, feels
d) offers, has been, feel
e) offered, has been, feels

AULA 01 – SKIMMING AND SCANNING AND OTHER TECHNIQUES. IDENTIFYING FALSE COGNATS. 56
TEACHER ANDREA BELO

GABARITO
GABARITO AFA
01 – A 02 – C 03 – D 04 – D 05 –B
06 – C 07 – B 08 – A 09 – D 10 – B

GABARITO COLÉGIO NAVAL


01 – A 02 – E 03 – C 04 – A 05 – B
06 – C 07 – A 08 – B 09 – C 10 – E
11 – D 12 – B 13 – D 14 – B 15 – B

GABARITO EAM
01 – E 02 – A 03 – B 04 – D 05 – B

GABARITO EEAR
01 – D 02 – D 03 – D 04 – C 05 – A

GABARITO EFOMM
01 – D 02 – B 03 – A 04 – C 05 – C
06 – A 07 – C 08 – D 09 – B 10 – D

GABARITO EPCAR
01 – D 02 – A 03 – A 04 – A 05 – B
06 – C 07 – A 08 – B 09 – D 10 – B

GABARITO ESA
01 – C 02 – E 03 – A 04 – B 05 – D

GABARITO ESCOLA NAVAL


01 – D 02 – C 03 – A 04 – C 05 – E
06 – C 07 – A 08 – B 09 – C 10 – D

GABARITO EsPCEx
01 – B 02 – A 03 – E 04 – D 05 – D
06 – C 07 – E 08 – C 09 – A 10 – B

AULA 01 – SKIMMING AND SCANNING AND OTHER TECHNIQUES. IDENTIFYING FALSE COGNATS. 57
TEACHER ANDREA BELO

QUESTÕES COMENTADAS
QUESTÕES AFA
Directions: Read the text below and answer questions 01 to 16 according to it.
The influence of YouTubers on teenagers
A few years ago, the new internet hype called Web 2.0 appeared, resulting in a big shift from using
internet relatively passive and consuming, to a hype of networking as a revolutionary type of
largescale cooperation and economic democracy. This big shift towards user-driven technologies
such as blogs, video-sharing platforms and social networks enabled a revolution of User-
Generated Content (UGC). UGC refers to all the various media content created or produced by the
public, excluding paid professionals. UGC can be described as the process of ‘extracting value by
generating, reviewing, editing and disseminating content’. This revolution of UGC is now being
referred to as social media.
UGC is also stated to be the next iteration of Word-of-Mouth (WOM). Traditional (offline) WOM
has proven to play a major role in customers’ decision-making process. The so-called eWOM is any
positive or negative statement made by (potential) customers about a product or company, which
is made available to a multitude of people and institutions via the Internet. Consumers can share
their own experiences and opinions online, increasing the unbiased product information people
gather from other consumers online.
People are now able to create their own content and participate in all kinds of applications such
as weblogs, social networking and online journalism. Some users create online content on a daily
basis. They share their experiences and opinions about everything; from product reviews, services
and baby advice to boyfriend problems, homework, make up and game tutorials. Often starting
as a hobby, creating online content on a personal blog or channel can become a fulltime job. By
creating content on a daily basis, large communities are built up with thousands of loyal followers.
New followers and subscribers are gained every day. YouTube is one of the most popular platforms
on which reators share their content. With one single video it is possible to reach millions of
people. Other popular channels are Instagram, Snapchat, Vine, Facebook and Twitter. These
platforms are used for real-time contact with YouTube subscribers, as this is not possible on
YouTube itself.
Successful content creators become social influencers with a lot of persuasive influence. Their
influence and fame can be compared with traditional celebrities. The content they share via social
media has influence on the viewers, for example on viewers who are looking for product
information to support their purchasing decisions. Although YouTube is used by all ages, the
platform is very popular among teenagers. According to a research by Defy Media (2015), 96% of
the youngsters aged between 13 and 24 regularly use YouTube. They consume online videos via
social media sites for an average of 11 hours a week, making it their most popular website. This
popularity is reflected again in the amount of YouTubers that focus on teenagers. The influence
YouTubers have on the behavior of teenagers is increasingly unprecedented. They are seen as role
models and often recognized in the streets by their fans. However, outside this online world,
YouTubers are just as normal as most people.

AULA 01 – SKIMMING AND SCANNING AND OTHER TECHNIQUES. IDENTIFYING FALSE COGNATS. 58
TEACHER ANDREA BELO

Parents often do not know about the online behavior of their children and the YouTubers that
children endorse. This results in confusing moments when fans enthusiastically recognize their
favorite YouTuber while other people do not know who this particular YouTuber is. A well-known
Dutch beauty YouTuber explained that when she is in public, fans recognize her from YouTube. Full
of excitement they ask to take a picture together. However, the other people walking by are
confused and do not know why these young teenagers want to take pictures together. This
example shows the unknown impact of social influencers on youngsters of which most adults do
not even know the existence.
Although there already is research available on the influence of traditional media on youngsters,
it is interesting to know what influence YouTubers have on teenagers and whether this influence
is good or bad. Until this point, only little has been studied about the influence of YouTubers on
teenagers. Previous research mostly concerned participatory cultures on YouTube, reviewing
platforms of which YouTube is only a part of several more platforms or the difference between
YouTube and watching traditional television.
Moreover, few studies were conducted in Europe and often limited on the US. Consequently, the
present research takes both the perspective of viewers and YouTubers into account and combines
them within four themes. Therefore, the research question of this research is as following: ‘What
role do YouTubers play in the life of their teenage viewers and how do YouTubers assess their own
influences and experiences?’ The aim of this research is to give an overview of the current YouTube
community, including what influence Dutch YouTubers have on their teenage viewers, what this
means for teenagers and to what extent this influence on their life is good or bad.

Questão 01 (AFA/INÉDITA) – UCG


(A) relates to media content created by the public, not counting paid professionals.
(B) is an online platform, such as YouTube, focused on creating videos.
(C) includes content produced by paid and nonpaid professionals.
(D) can be described as the process of solely disseminating content.
Comentários: A alternativa A está correta. UCG se refere a conteúdo de mídia criado pelo público,
sem contar profissionais remunerados. “UGC refers to all the various media content created or
produced by the public, excluding paid professionals.”
A alternativa B está incorreta. UCG não é uma plataforma online, como o YouTube.
A alternativa C está incorreta “UGC refers to all the various media content created or produced by
the public, excluding paid professionals.”
A alternativa D está incorreta. UCG não é somente (solely) o processo de divulgação de conteúdo,
mas também de geração, revisão e disseminação de conteúdo. “UGC can be described as the
process of ‘extracting value by generating, reviewing, editing and disseminating content’.”
GABARITO: A

Questão 02 (AFA/INÉDITA) – In “Some users create online content on a daily basis.” (Paragraph
3) the underlined expression means

AULA 01 – SKIMMING AND SCANNING AND OTHER TECHNIQUES. IDENTIFYING FALSE COGNATS. 59
TEACHER ANDREA BELO

(A) every other day.


(B) very often.
(C) every day.
(D) for a long period.
Comentários: Em “Some users create online content on a daily basis.” (parágrafo 3) a expressão
sublinhada significa diariamente, todo dia.
a) every other day = dia sim, dia não.
b) very often = com muita frequência.
c) every day = todos os dias.
d) for a long period. = por um longo período.
GABARITO: C

Questão 03 (AFA/INÉDITA) – In “YouTube is one of the most popular platforms on which


creators share their content.” (Paragraph 3), there is an example of
(A) comparative of superiority.
(B) comparative of inferiority.
(C) comparative of equality.
(D) superlative.
Comentários: Em “YouTube is one of the most popular platforms on which creators share their
content.” (paragraph 3), há um exemplo de superlative: the most popular – one of the most
popular platforms – uma das plataformas mais populares...
A alternativa A está incorreta. O comparativo é usado para comparar dois substantivos, enquanto
o superlativo é usado
para comparar mais de dois. O comparativo de superioridade é feito com more (adjetivos longos)
ou com o sufixo -er (adjetivos curtos).
A alternativa B está incorreta. O comparativo de inferioridade, geralmente, é feito com less +
adjetivo (less interesting, por exemplo, menos interessante).
A alternativa C está incorreta. O comparativo de igualdade é feito com a estrutura as + adjetivo +
as (tão ... quanto).
A alternativa D está correta, conforme a explicação acima.
GABARITO: D

Questão 04 (AFA/INÉDITA) – The fourth paragraph of the text states that


(A) 96% of people regularly use YouTube.

AULA 01 – SKIMMING AND SCANNING AND OTHER TECHNIQUES. IDENTIFYING FALSE COGNATS. 60
TEACHER ANDREA BELO

(B) traditional celebrities are still more influential than content creators.
(C) people hardly ever recognize YouTubers in public.
(D) YouTubers influence teenagers on a level never seen before.
Comentários: A alternativa A está incorreta. O quarto parágrafo do texto não afirma que 96% das
pessoas usam o YouTube regularmente, mas, sim, que 96% dos jovens entre 13 e 24 anos usam o
YouTube regularmente. “According to a research by Defy Media (2015), 96% of the youngsters
aged between 13 and 24 regularly use YouTube.”
A alternativa B está incorreta. O quarto parágrafo do texto não afirma que as celebridades
tradicionais ainda são mais influentes do que os criadores de conteúdo. “Successful content
creators become social influencers with a lot of persuasive influence. Their influence and fame
can be compared with traditional celebrities.”
A alternativa C está incorreta. O quarto parágrafo do texto não afirma que as pessoas dificilmente
reconhecem os YouTubers em público. “They are seen as role models and often recognized in the
streets by their fans.”
A alternativa D está correta. O quarto parágrafo do texto afirma que os YouTubers influenciam os
adolescentes em um nível nunca visto antes. “The influence YouTubers have on the behavior of
teenagers is increasingly unprecedented.” = A influência dos YouTubers no comportamento dos
adolescentes é cada vez mais sem precedentes.
GABARITO: D

Questão 05 (AFA/INÉDITA) – In the sentence “This big shift towards user-driven technologies …
enabled a revolution of User-Generated Content (UGC).” (Paragraph 1), the underlined word is a
synonym for
(A) increased
(B) allowed
(C) focused
(D) became
Comentários: Em “This big shift towards user-driven technologies … enabled a revolution of User-
Generated Content (UGC).” (parágrafo 1), a palavra sublinhada tem sentido de permitiu,
possibilitou. “Essa grande mudança em direção às tecnologias orientadas ao usuário ... permitiu
uma revolução no conteúdo gerado pelo usuário (UGC).
A alternativa A está incorreta. Increased significa aumentou. A alternativa B está correta. Allowed
é um sinônimo para enabled, também significando permitiu. A alternativa C está incorreta.
Focused significa focou. A alternativa D está incorreta. Became significa se tornou.
GABARITO: B

Questão 06 (AFA/INÉDITA) – Mark the alternative that LACKS the correct synonym for the
underlined word.

AULA 01 – SKIMMING AND SCANNING AND OTHER TECHNIQUES. IDENTIFYING FALSE COGNATS. 61
TEACHER ANDREA BELO

(A) “… for example on viewers who are looking for product information to support their
purchasing decisions. (Paragraph 4) – buying.
(B) Often starting as a hobby … (paragraph 3) – usually.
(C) … increasing the unbiased product information people gather from other consumers online.
(Paragraph 2) – unfair.
(D) Although YouTube is used by all ages … (paragraph 4) – even though.
Comentários: Marque a alternativa que não possui (lack = faltar, não ter) o sinônimo correto para
a palavra sublinhada.
A alternativa A está incorreta, porque buying é, sim, sinônimo de purchasing, ambos os termos se
referindo ao ato de comprar.
A alternativa B está incorreta, porque often é, sim, sinônimo de usually, ambos os termos
significando geralmente, frequentemente, usualmente.
A alternativa C está correta, porque unbiased (imparcial) não é sinônimo de unfair (injusto).
A alternativa D está incorreta, porque although é, sim, sinônimo de even though, ambas as
expressões significando embora, apesar de.
GABARITO: C

Questão 07 (AFA/INÉDITA) – Mark one of the statements below that DOESN'T agree with the
text.
(A) the research investigates how teenagers identify with YouTubers.
(B) the life of a YouTuber is as beautiful as it looks in their videos.
(C) the research focuses not only on the experience of YouTubers themselves.
(D) teenagers may or may not know how real things are in Youtubers’ videos.
Comentários: Marque uma das afirmações abaixo que NÃO está de acordo com o texto.
A alternativa A está incorreta, porque o texto afirma, sim, que a pesquisa investiga como os
adolescentes se identificam com os YouTubers. “The following four themes guide throughout the
research: … (2) identification with YouTubers; how much teenagers identify with their favorite
YouTubers and want to be like them…”
A alternativa B está correta. O texto não afirma que a vida de um YouTuber é tão bonita quanto
parece em seus vídeos, mas, sim, que o estudo busca investigar se a vida de um YouTuber é tão
bonita quanto parece. “Additionally, the interviews investigate whether the life of a YouTuber
really is as beautiful as it looks in their videos and whether teenagers know how real everything
in their videos is.”
A alternativa C está incorreta, porque o texto afirma, sim, que a pesquisa se concentra não apenas
na experiência dos próprios YouTubers, mas dos adolescentes espectadores também. “The
research focuses on both the perception of teenagers and the experience of YouTubers themselves
and combines both results.”

AULA 01 – SKIMMING AND SCANNING AND OTHER TECHNIQUES. IDENTIFYING FALSE COGNATS. 62
TEACHER ANDREA BELO

A alternativa D está incorreta, porque o texto afirma, sim, que os adolescentes podem ou não
saber o quão reais as coisas são nos vídeos dos Youtubers, tanto que a pesquisa investigará esse
aspecto. Se fosse certo que eles sabem, por exemplo, não haveria por que investigar isso em um
estudo. “Additionally, the interviews investigate whether the life of a YouTuber really is as beautiful
as it looks in their videos and whether teenagers know how real everything in their videos is.”
GABARITO: B

Questão 08 (AFA/INÉDITA) – According to the text, parents


(A) many times do not know the YouTubers their kids are fans of.
(B) usually do not endorse their kids spending so much time on YouTube.
(C) enthusiastically recognize their favorite YouTubers in the streets.
(D) are usually aware of the online behavior of their children.
Comentários: De acordo com o texto,
A alternativa A está correta – os pais muitas vezes não conhecem os YouTubers dos quais seus
filhos são fãs. “Parents often do not know about the online behavior of their children and the
YouTubers that children endorse.” = Os pais geralmente não sabem sobre o comportamento online
de seus filhos e os YouTubers que as crianças endossam.
A alternativa B está incorreta – o texto não afirma que os pais não costumam recomendar que
seus filhos passem tanto tempo no YouTube.
A alternativa C está incorreta – o texto não afirma que os pais reconhecem com entusiasmo seus
YouTubers favoritos nas ruas.
A alternativa D está incorreta – o texto não afirma que os pais geralmente estão cientes do
comportamento online de seus filhos. “Parents often do not know about the online behavior of
their children and the YouTubers that children endorse.” = Os pais geralmente não sabem sobre o
comportamento online de seus filhos e os YouTubers que as crianças endossam.
GABARITO: A

Questão 09 (AFA/INÉDITA) – According to the text, the word “Moreover” (paragraph 7)


(A) introduces a conclusion.
(B) is used to make an exception.
(C) introduces a contrast.
(D) is used to add information.
Comentários: De acordo com o texto, a palavra “Moreover” é usada para acrescentar
informações, significando “além disso”.
“Moreover, few studies were conducted in Europe and often limited on the US.” = Além disso,
poucos estudos foram realizados na Europa e muitas vezes limitados nos Estados Unidos. Assim,

AULA 01 – SKIMMING AND SCANNING AND OTHER TECHNIQUES. IDENTIFYING FALSE COGNATS. 63
TEACHER ANDREA BELO

Moreover não introduz uma conclusão (letra A), não é usada para fazer uma exceção (letra B),
nem introduz um contraste (letra C). A letra D está correta.
GABARITO: D

Questão 10 (AFA/INÉDITA) – In the fragment “Consequently, the present research takes both
the perspective of viewers and YouTubers into account and combines them within four themes.”
(Paragraph 7) the underlined pronoun refers to
(A) viewers.
(B) the perspectives of viewers and YouTubers.
(C) YouTubers.
(D) four themes.
Comentários: No fragmento “Consequently, the present research takes both the perspective of
viewers and YouTubers into account and combines them within four themes.” (parágrafo 7) o
pronome sublinhado refere-se às perspectivas tanto dos espectadores e quanto dos YouTubers.
→ Consequentemente, a presente pesquisa leva em consideração tanto a perspectiva dos
telespectadores quanto a dos YouTubers (both = ambas) e as combina (combina essas
perspectivas) em quatro temas.
GABARITO: B

QUESTÕES COLÉGIO NAVAL


Texto: TRAVEL TIPS
How to Plan a Movie-Themed Vacation
It’s easier than you may expect to find, visit, and enjoy the places where your favorite movies
were made.
Lars Leetaru
By Shivani Vora
March 8, 2018
Whether it’s the “Lord of the Rings” trilogy in New Zealand or "Roman Holiday” in Rome, many
noteworthy movies are filmed in appealing locales all over the world that travelers may want to
visit and enjoy.
According to Angela Tillson, a film location manager in Kauai who has worked on the set of films
including "Jurassic Park: The Lost World” and “The Descendants," exploring a beloved movie set
destination through the eyes of the film makes for an enjoyable vacation. "Seeing a place with a
focus on a movie you love will give you a perspective that the average tourist doesn’t usually get.
You’ll certainly have a better impression of the place,” she said. Here are her tips to get started.
Choose Your Destination

AULA 01 – SKIMMING AND SCANNING AND OTHER TECHNIQUES. IDENTIFYING FALSE COGNATS. 64
TEACHER ANDREA BELO

If there’s a movie you love, you can find out where it was filmed by looking at the credits at the
end of the film or by going online to The Internet Movie Database, also known as IMDB, which
often lists filming locations. Once you know the locale, you can start planning your trip. Or,
consider doing what Ms. Tillson often does when deciding on where to vacation: pick a spot you’re
interested in visiting, and find out what movies have been filmed there. “It’s fun to sometimes let
a destination determine the movie you're going to live rather than the other way around,” Ms.
Tillson said.
Get in the Mood
Before you head to your destination, be sure to rewatch the movie. A rewatch not only reminds
you of identifiable spots to look out for during your trip, but it also adds to the excitement of your
upcoming exploration.
If the movie is based on a book, consider reading the book, too. It may have details about the
locale that the movie doesn’t touch on. Also, books often have scenes that don’t make it into the
movie adaptations, which gives you a deeper view of the destination. Ms. Tillson also
recommended downloading the movie’s soundtrack or score, and listening to it throughout your
trip.
Book a Themed Trip
Some travel companies sell set itineraries focused on popular movies. Luxury tour operator
Zicasso, for example, has an eight-day trip, all inclusive, to Ireland inspired by "Star Wars: The Last
Jedi” and Wild Frontiers has an eleven-day trip to India inspired by "The Best Exotic Marigold
Hotel." Ms. Tillson suggested doing a web search or checking with a travel agent to find out about
such trips.
Also, in some destinations, local tour operators and hotels sell movie-themed tours. For instance,
The St. Regis Priceville Resort offers a tour that includes a private helicopter ride to
Manawaiopuna Falls, made famous in "Jurassic Park,” and an ATV tour of filming locations of
movies such as “Raiders of the Lost Ark" and “Pirates of the Caribbean.” Lunch is even included.
The cost is $5,674 for two adults.
A more affordable option, in Rome, is the four-hour “Roman Holiday" themed excursion from HR
Tours, where travelers ride a Vespa with a driver and see all the sites from the movie; the cost is
170 euros per person.
Hang Where the Movie Crew Did
When they’re not working, movie crews enjoy hitting local bars and casual restaurants that serve
tasty local cuisine, Ms. Tillson said.
Find out where the behind-the-scenes staff of your film spent their time by asking your
destination’s tourist board or your hotel’s concierge, and check out a few of the spots. “It’s
another way to get involved in the film and spend time in bars and restaurants that you wouldn’t
normally think to hit,” she said.

Questão 01 (COLÉGIO NAVAL/2018) – According to Ms. Tillson,


(A) listening to the songs of the movie can get you in the mood of your movie-themed vacation.

AULA 01 – SKIMMING AND SCANNING AND OTHER TECHNIQUES. IDENTIFYING FALSE COGNATS. 65
TEACHER ANDREA BELO

(B) you can never choose the destination in the first place. Always base your trip on the movie
you like.
(C) you don’t find any information online about the themed itineraries. You have to check it
exclusively with a travel agent.
(D) it’s difficult to have a different perspective when you explore the destination through the
eyes of the film.
(E) you can visit local bars and restaurants and have the opportunity to meet the movie crew.
Comentários: A alternativa A está correta. Escutar músicas do filme podem te ajudar a “entrar no
clima” da viagem temática. O trecho do texto que fala sobre baixar a trilha sonora do filme está
na parte “get in the mood” do texto e, isso corrobora com a afirmação da alternativa.
A alternativa B está incorreta. A palavra “never” elimina a alternativa, pois ela diz no texto que é
possível escolher o destino baseado no filme, ou escolher o lugar primeiro e procurar por filmes
que tenham sido gravados naquele local.
A alternativa C está incorreta. A afirmação contraria o que é dito no texto. O texto diz que é
possível encontrar informações no IMDB (internet movies data base).
A alternativa D está incorreta. A afirmativa diz que é difícil ter uma perspectiva diferente quando
se explora um lugar através dos olhos de um filme. O texto diz que a perspectiva é diferente da
de um turista comum. São, portanto, afirmações conflitantes.
A alternativa E está incorreta. A afirmativa diz que ao visitar bares e retaurantes locais, você
poderá conhecer a equipe de produção do filme. Isso não é verdade, já que o filme já foi produzido
e lançado. Por causa disso, a equipe não estará lá.
GABARITO: A

Read the text below to do questions 02 to 06.


TEXT III
STEPHEN HAWKING
Stephen Hawking, one of the most famous scientists of the 21st century, died on March 14th,
2018. But his ideas on gravity, black holes and the Big Bang are the greatest legacy he left to the
world.
Early Life and College
On January 8th, 1942, Stephen Hawking was born to a successful family in Oxford, England. He
was not the best student at fundamental or high school, but he was very smart. His dad wanted
him to become a medical doctor, but Stephen really wanted to study math.
Despite his poor grades at school, Stephen aced his exams for college. He was admitted to Oxford
University, but they didn't have math as a major. So, he decided to study physics and chemistry
instead.
Stephen became a member of a rowing team when he was in college. After he graduated, he
decided to continue his education and went to graduate school.

AULA 01 – SKIMMING AND SCANNING AND OTHER TECHNIQUES. IDENTIFYING FALSE COGNATS. 66
TEACHER ANDREA BELO

Graduate School, Marriage and Health Problems


While in graduate school, he had some health problems. He began tripping for no reason, and his
speech became hard to understand. His family encouraged him to go to the doctor. Stephen was
diagnosed with a disease called ALS, or Lou Gehrig's disease, which affects the brain and spine.
He was only twenty one, and the doctors initially gave him only a few years to live. Luckily, his
condition progressed more slowly than is often the case.
During this time, Stephen had a relationship with a woman named Jane. He said she and his work
were his inspiration for living. Stephen earned his Ph.D. degree in 1965. He started to get worse,
and eventually became confined to a wheelchair. Nonetheless, he and Jane married in 1965 and
were able to have three children.
He studied how space and time are related, including scientific studies of black holes in space and
how they work in the universe. He also had a lot of success in his work as a college professor.
A New Voice
In 1985, Stephen got really sick and doctors were able to save him, but he was unable to speak.
He could only use his eyebrows to communicate. Eventually, he was able to use a special voice
synthesizer, allowing him to talk by moving his cheek muscles and using a mouse pad.
Famous Works and Prizes
His most notorious theory is that black holes can emit radiation; also known as Hawking radiation.
He received numerous awards but never won the Nobel Prize.
Stephen always enjoyed writing books. His best seller, “A Brief History in Time” made terms like
the Big Bang and black holes easy to understand. Other famous Stephen Hawking books include:
“A Briefer History in Time", “On the Shoulders of Giants” and "The Universe in a Nutshell". He also
wrote many books for children along with his daughter Lucy. His famous books for children include
"George’s Cosmic Treasure Hunt" and “George and the Big Bang”.
His last work, submitted only two weeks before his death, reveals the universe will come to an
end when stars run out of energy. However, his theory suggests that scientists will be able to find
parallel universes using probes on spaceships.

Questão 02 (COLÉGIO NAVAL/2018) – Mark the sentence which has an adjective that is NOT in
the superlative degree.
(A) Stephen Hawking, one of the most famous scientists of the 21st century (...).
(B) But his ideas (...) are the greatest legacy he left to the world.
(C) He was not the best student at fundamental or high school.
(D) His most notorious theory is that black holes can emit radiation.
(E) He started to get worse, and, eventually, became confined to a wheelchair.
Comentários: A alternativa A está incorreta. “Most famous”, um dos mais famosos. Trata-se de
adjetivo no grau superlativo.

AULA 01 – SKIMMING AND SCANNING AND OTHER TECHNIQUES. IDENTIFYING FALSE COGNATS. 67
TEACHER ANDREA BELO

A alternativa B está incorreta. “Greatest” são o maior legado… Trata-se de adjetivo no grau
superlativo.
A alternativa C está incorreta. “Best” não foi o melhor aluno… Trata-se de adjetivo no grau
superlativo.
A alternativa D está incorreta. “Most notorious” seu trabalho mais notório… Trata-se de adjetivo
no grau superlativo.
A alternativa E está correta. “Worse” ele começou a piorar… Trata-se de adjetivo no grau
comparativo.
GABARITO: E

Questão 03 (COLÉGIO NAVAL/2018) – Say if the following statements are T (true) or F (false)
about Hawking’s disease. Then mark the correct option, from top to bottom.
( ) It confined him to a wheelchair.
( ) It affected his ability to speak.
( ) It was diagnosed when he was 21 years old.
( ) It progressed more quickly than expected.
( ) It made him sturhble for no reason.
(A) (T) (T) (T) (F) (F)
(B) (T) (F) (T) (F) (F)
(C) (T) (T) (T) (F) (T)
(D) (F) (T) (F) (T) (T)
(E) (F) (T) (F) (T) (F)
Comentários: (T) It confined him to a wheelchair. O texto diz que ele acabou ficando preso a uma
cadeira de rodas. A afirmação é, potanto, verdadeira.
(T) It affected his ability to speak. O texto diz claramente que a doença afetou sua capacidade de
falar. Foi por isso que ele passou a usar uma voz artificial. Verdadeira.
(T) It was diagnosed when he was 21 years old. O texto diz que ele foi diagnosticado e, em seguida
diz que ele tinha apenas 21 anos de idade quando isso aconteceu. Verdadeira.
(F) It progressed more quickly than expected. O texto diz o contrário disso. A doença evoluiu mais
devagar que o esperado, não mais rápido. Falsa.
(T) It made him stumble for no reason. Essa afirmativa poderia gerar alguma dúvida, pois ela usa
uma palavra diferente do texto (stumble). “Stumble” é sinônimo de “tripping”. Verdadeira.
GABARITO: C

Questão 04 (COLÉGIO NAVAL/2018) – What statement is correct about Hawking’s last work?
(A) It predicted the end of the universe.

AULA 01 – SKIMMING AND SCANNING AND OTHER TECHNIQUES. IDENTIFYING FALSE COGNATS. 68
TEACHER ANDREA BELO

(B) It suggests there are no parallel universes.


(C) It could not be finished because of his death.
(D) It was his bestselling book and received a prize.
(E) It introduced his theory about the Big Bang.
Comentários: A alternativa A está correta. O texto diz que em seu último trabalho, Hawking prevê
o fim do universo.
A alternativa B está incorreta. O texto diz que Hawking acha que os cientistas estarão aptos a
encontrar universos paralelos.
A alternativa C está incorreta. O trabalho foi finalizado pouco antes do seu falecimento.
A alternativa D está incorreta. O seu livro mais vendido e ganhador de prêmios foi “A Brief History
in Time”.
A alternativa E está incorreta. Sua teoria acerca do Big Bang foi descrita em “A Brief History in
Time”.
GABARITO: A

Questão 05 (COLÉGIO NAVAL/2018) – It can be inferred from the text that Stephen Hawking
(A) received numerous awards, including a Nobel Prize.
(B) was seventy six years old when he died.
(C) married Jane one year after he got his Ph.D degree.
(D) wrote “A Brief History in Time” for kids.
(E) wanted to be a doctor, but studied physics instead.
Comentários: A alternativa A está incorreta. Ele, de fato, recebeu prêmios, mas o prêmio nobel
não está entre eles.
A alternativa B está correta. O texto diz que ele nasceu em janeiro de 1942 e morreu em março
de 2018. Ele tinha 76 quando morreu.
A alternativa C está incorreta. O texto diz que eles se casaram em 1965, mesmo ano que ele
terminou seu Ph.D.
A alternativa D está incorreta. “A Brief History in Time” não foi escrito para crianças. Trata-se de
um trabalho científico.
A alternativa E está incorreta. Ele queria estudar matemática, mas não havia um curso de
matemática na universidade. Ele, então, optou por estudar física. Seu pai queria que ele fosse
médico, não ele.
GABARITO: B

Questão 06 (COLÉGIO NAVAL/2018) – In the sentence, “Despite his poor grades at school,
Stephen aced his exams for college.” (Paragraph 3), the underlined phrase means that Stephen

AULA 01 – SKIMMING AND SCANNING AND OTHER TECHNIQUES. IDENTIFYING FALSE COGNATS. 69
TEACHER ANDREA BELO

(A) got bad grades in his tests.


(B) passed with average grades.
(C) achieved high marks in the test.
(D) tried hard but did not pass.
(E) had a second change and passed.
Comentários: A alternativa A está incorreta. O trecho diz que ele foi bem nos testes para a
faculdade, não que ele tirou notas ruins.
A alternativa B está incorreta. O trecho diz que ele foi bem no teste para a faculdade, não que ele
obteve notas medianas.
A alternativa C está correta. O termo “aced” quer dizer que ele foi muito bem na prova, que ele
alcançou notas altas nos testes.
A alternativa D está incorreta. Se ele tirou notas altas, não é correto afirmar que ele tentou, mas
não passou no teste.
A alternativa E está incorreta. O trecho não fala nada sobre uma segunda chance. Ele passou na
sua primeira tentativa.
GABARITO: C

Questão 07 (COLÉGIO NAVAL/2018) – Read the sentence in order to do the question below.
This is my friends’ car and that one is my car.
Mark the option which rewrites the sentence using the correct possessive.
(A) This car is theirs and that one is mine.
(B) This is theirs cars and that is my car.
(C) This is their car and that one is my.
(D) This car is them and that one is my.
(E) This is theirs car and that one is mine.
Comentários: A alternativa A está correta. Os possessivos estão sendo usados da maneira correta.
Este carro é deles (theirs) e aquele é meu (mine).
A alternativa B está incorreta. “Theirs” está sendo usado de forma incorreta. Ele deveria aparecer
depois de “car”, que também está incorreto, pois deveria ser escrito no singular “car”.
A alternativa C está incorreta. “My” está sendo usado incorretamente. Nessa situação, o pronome
adequado é “mine”.
A alternativa D está incorreta. Esta alternativa está completamente errada quanto ao uso dos
pronomes. “Them” deveria ser substituído por “theirs”, e “my” deveria ser substituído por
“mine”.
A alternativa E está incorreta. Como o pronome está posicionado antes do substantivo (car), ele
deveria ser trocado por “their”.

AULA 01 – SKIMMING AND SCANNING AND OTHER TECHNIQUES. IDENTIFYING FALSE COGNATS. 70
TEACHER ANDREA BELO

GABARITO: A

Questão 08 (COLÉGIO NAVAL/2018) – Mark the sentence which is grammatically correct.


(A) How many girls are there at your party last week?
(B) Is there anybody in room 201 at the moment?
(C) Sarah said there is twelve people waiting.
(D) There isn’t an accident in our street last night.
(E) There is exercise bars so you can work out.
Comentários: A alternativa A está incorreta. A frase pergunta quantas garotas havia na sua festa
semana passada. Por isso, “are” está incorretamente empregado nesse contexto. A conjugação
correta seria “were”.
A alternativa B está correta. A frase está escrita de acordo com a norma padrão da língua inglesa.
A alternativa C está incorreta. Sarah disse que há 12 pessoas esperando. Por se tratar de plural
(twelve people), seria correto escrever “there was” ao invés de “there is”.
A alternativa D está incorreta. Não houve um acidente na nossa rua noite passada. Dever-se-ia
escrever “there wasn’t” ao invé de “there isn’t”.
A alternativa E está incorreta. Há bares de exercício para que você possa malhar.
Por se tratar de plural, deveríamos usar “there are” ao invés de “there is”.
GABARITO: B

Questão 09 (COLÉGIO NAVAL/2018) – Read the text to do the question below. Complete with a,
the or 0 (no article).
__________ Brazil is a huge country. In __________ North, there are __________ rain forests and
__________ longest river is also situated there. In __________ South, __________ climate is more
European. __________ Brazil also has many social differences. __________ rich own most of
__________ country’s wealth and __________ poor often live on minimum wage.
Now mark the option which completes the gaps respectively.
(A) 0 - the - 0 - a - the - the - 0 - a - the - a
(B) The - the - 0 - a - 0 - the - the - The - 0 - the
(C) 0 - the - 0 - the - the - the - 0 - The - the - the
(D) The - 0 - 0 - a - 0-the- The - 0 - a - 0
(E) The - 0 - the - a - 0 - the - The - a - a - a
Comentários: Na primeira lacuna, não necessitamos de artigo antes de “Brazil”.
A segunda lacuna fala “no norte” e não “em um norte”. Portanto, “the” (artigo definido) completa
corretamente a lacuna.

AULA 01 – SKIMMING AND SCANNING AND OTHER TECHNIQUES. IDENTIFYING FALSE COGNATS. 71
TEACHER ANDREA BELO

A terceira lacuna não é preenchida por nenhum artigo.


A quarta lacuna fala “O rio mais longo”. “the” é o artigo definido a ser usado.
A quinta lacuna fala “no sul” e não “em um sul”. Portanto, “the” (artigo definido) completa
corretamente a lacuna.
A sexta lacuna fala “O clima”. “the” é o artigo definido a ser usado.
A sétima lacuna não pede pronome algum antes de “Brazil”.
A oitava lacuna diz “os ricos”. “the” é a opção correta para esta lacuna.
A nona lacuna diz “a maior parte da riqueza do país”. Mais uma vez, “the” é o pronome adequado
a ser utilizado aqui.
A décima lacuna diz “os pobres”. O pronome “the” é o mais adeuqado aqui.
o-the-0-the-the-the-0-the-the-the
GABARITO: C

Questão 10 (COLÉGIO NAVAL/2018) – Read the comic strip in order to do the question below.
The correct pronoun that completes the third bubble speech is
(A) they.
(B) it.
(C) theirs.
(D) its.
(E) them.
Comentários: A alternativa A está incorreta. O pronome a preencher a lacuna deverá retomar
“eyes”. They é um subject pronoun (exerce função de sujeito na frase). A lacuna exerce função de
complemento.
A alternativa B está incorreta. O pronome a preencher a lacuna deverá retomar “eyes”. “It” é um
pronome da terceira pessoa do singular. Por isso não pode retomar um termo no plural.
A alternativa C está incorreta. “Theirs” é pronome possessivo. Portanto, não pode ser usado para
preencher a lacuna.
A alternativa D está incorreta. “Its” também é um pronome possessivo e, potanto, não pode
preencher a lacuna.
A alternativa E está correta. “Them” é um pronome que preenche adequadamente a lacuna por
se tratar de um “object pronoun” da terceira pessoa do plural.
GABARITO: E

Robotic Cars
The year is 2020, and it’s 7;45 on a rainy Monday morning, and you are in your car on your way
to work. You turn right, and you turn left. A few minutes later, you stop at a traffic light. When

AULA 01 – SKIMMING AND SCANNING AND OTHER TECHNIQUES. IDENTIFYING FALSE COGNATS. 72
TEACHER ANDREA BELO

the light turns green and there are no other cars in the intersection, you continue on your way.
Ten minutes later you get to work and stop reading the morning paper. Then, you get out of your
car and you say, “Thank you!". Your car replies, “You’re welcome!’’. This possibie future may
sound unreai, but in fact many car companies are aiready testing robotic cars, or driverless cars,
on the roads today, although the cars don't speak very much yet.
In the 1980s, Germany and the United States tested the first driverless cars, and by 2020
companies such as Volvo, GM, Nissan and BMW plan to seil driverless cars. Driverless cars are not
really ‘driverless - the drivers are computers that use radar, Computer maps and other modern
technology. They offer many advantages. Perhaps the most important of these is fewer deaths
caused by road accidents. For example, in 1968 more than 53.000 people lost their lives in car
accidents in the USA. This number has fallen to less than 33.000 but it’s still a high number. In
addition, people will spend iess time stuck in traffic jams and there will be no need for people to
have a driving license. One of the major disadvantages of this new technology, however, is the
cost. It’s not free. U$5.000 to U$10.000 is added to the price of the new car. Nevertheless, at
some time in your life, you will probably be sitting in a robotic, ;driverless car on your way to work
or school. The future is almost here. Are you ready for it?
<https://www.aHthinastoDics.eom/uploads/2/3/2/9/23290220/lesson-drivina-robo ticcars2.pdf>

Questão 11 (COLÉGIO NAVAL/2019) – Read the extract from the text


“Ten minutes after you get to work and stop reading the morning paper”
Mark the option that can replace the expression get to
(A) drive to
(B) go to
(C) find
(D) arrive at
(E) come from
Comentários: A frase diz que “10 minutos depois você chega no trabalho e para de ler o jornal” e
pede para marcar a opção que melhor substitui a expressão get to que nesta frase possui o sentido
de chegar. Analisemos alternativa por alternativa.
Na letra A, temos “drive to” que significa “dirige para”, dando um sentido de que ainda não
chegou no trabalho 10 minutos depois e que vai começar a dirigir para chegar. Mas a frase original
mostra que depois de 10 minutos você chega no trabalho. A frase fica “10 minutos depois você
dirige para o trabalho e para de ler o jornal”. Alternativa errada.
Na letra B, temos “go to” que significa que a pessoa ainda não chegou no trabalho também e 10
minutos depois e que vai começar a ir. A frase fica “10 minutos depois você vai para o trabalho e
para de ler o jornal”. Veja que você vai, não quer dizer que chega no trabalho. Alternativa errada.
Na letra C, temos “find” que significa “encontra” e esta opção além de não ter um conectivo não
encaixa na frase. Veja como a frase ficaria “10 minutos depois você encontra trabalho e para de
ler jornal”. O sentido mudou totalmente e, portanto, alternativa errada.

AULA 01 – SKIMMING AND SCANNING AND OTHER TECHNIQUES. IDENTIFYING FALSE COGNATS. 73
TEACHER ANDREA BELO

Na letra D, temos “arrive at” que significa “chega no” e este sentido é exatamente o que encaixa
na frase sem que perca seu significado inicial. O verbo to arrive significa chegar, exatamente o
sentido mostrado na frase. Logo, alternativa correta é a letra D.
Na letra E, temos “come from” que significa “vem de” e esta opção não encaixa na frase, pois ela
fica “10 minutos depois você vem do trabalho e para de ler o jornal”. Perceba que o sentido ficou
inverso, como se o personagem saísse do trabalho. Portanto, alternativa errada.
GABARITO: D

Questão 12 (COLÉGIO NAVAL/2019) – Read the extract from the text


“The year is 2020, and it’s 7:45 on a rainy Monday morning (...)”
What’s the correct question referring to the underlined information?
(A) What’s the weather?
(B) What’s the weather like?
(C) How’s the weather?
(D) How’s the climate like?
(E) What’s the climate?
Comentários: A palavra “rainy” significa “chuvoso” e está caracterizando a manhã da segunda
feira do texto. Veja que esta palavra está descrevendo algo. Veja que a palavra “rainy” se refere
a um determinado período, portanto ela descreve o “tempo” (weather).
A palavra “climate” significa clima e se refere a uma condição do estado da atmosfera em um
longo período. Assim a melhor opção é a letra B que faz a pergunta “Como está o tempo?” e a
resposta seria “chuvoso” (rainy). Vejamos as outras alternativas.
Na letra A, temos “What’s the weather?” apesar de usar a palavra weather, a pergunta está
incompleta, pois quando queremos saber como está alguma coisa é necessário o uso da palavra
“like” no final da pergunta. Portanto, alternativa incorreta.
Na letra B, como já mostrado anteriormente alternativa correta é a letra B.
Na letra C, temos “How is the weather?” apesar de usar a palavra weather, a pergunta está
incompleta novamente, pois quando queremos saber como está alguma coisa é necessário o uso
da palavra “like” no final da pergunta. Ficaria certo se fosse “Ho is the weather like?”. Portanto,
alternativa incorreta novamente.
Na letra D, temos “How is the climate like?” está errado ao usar a palavra climate que como
mostrado anteriormente se refere a períodos de longo prazo. Portanto alternativa incorreta.
Na letra E, como já mostrado anteriormente, a palavra climate se refere a períodos de longo
prazo. Além disso falta também o uso da palavra like para se saber como algo está. Portanto
alternativa incorreta.
GABARITO: B

AULA 01 – SKIMMING AND SCANNING AND OTHER TECHNIQUES. IDENTIFYING FALSE COGNATS. 74
TEACHER ANDREA BELO

Questão 13 (COLÉGIO NAVAL/2019) – Read the extract from the text


“They offer many advantages”. The subject pronoun they refers to
(A) Germany and The USA
(B) Volvo, GM, Nissan and BMW
(C) computers with radars
(D) driverless cars
(E) computer maps
Comentários: Na frase “They offer many advantages” a palavra “they” se refere a “driverless cars”
como se pode ver no trecho “Driverless cars are not really ‘driverless’ – the drivers are computers
that use radar, computer maps and other modern technology. They offer many advantages.”
Quando é usado a palavra they para se referir a algo que já foi dito, é importante analisar a frase
que é imediatamente anterior. Veja que no trecho o que foi dito exatamente anterior ao uso da
palavra they é driverless cars.
Não pode ser computers with radar nem computer maps porque não são eles que oferecem
vantagens como descreve em seguida no texto.
Na letra A, temos “Germany and the United States” que não é citado exatamente anterior ao uso
da palavra “they”. Além disso, sabemos que não são eles que oferecem vantagens. Portanto
alternativa incorreta.
Na letra B, temos “Volvo, GM, Nissan and BMW” que não é citado exatamente anterior ao uso da
palavra “they”. Além disso, sabemos que não são eles que oferecem vantagens. Portanto
alternativa incorreta.
Na letra C, temos “Computers with radars” que apesar de ser citado exatamente anterior ao uso
da palavra “they” não são o que oferecem vantagens. Portanto alternativa incorreta.
Na letra D, temos “driverless cars” que é a alternativa correta é a letra D como mostrado
anteriormente.
Na letra E, temos “Computer maps” que apesar de ser citado exatamente anterior ao uso da
palavra “they” não são o que oferecem vantagens. Portanto alternativa incorreta.
GABARITO: D

Questão 14 (COLÉGIO NAVAL/2019) – Read the sentence below


“That is an intelligent car that works with a computer”
What is the plural of the sentence?
(A) These are intelligents cars that work with computers
(B) Those are intelligent cars that work with computers
(C) Those are intelligents cars that works with computers
(D) These are intelligent cars that works with computers
(E) Those are intelligent cars that works with computers

AULA 01 – SKIMMING AND SCANNING AND OTHER TECHNIQUES. IDENTIFYING FALSE COGNATS. 75
TEACHER ANDREA BELO

Comentários: Queremos o plural da frase “That is an intelligent car that works with a computer.”
Na letra A, temos “These are intelligent cars that work with computers” está errado pois o plural
de “that” (aquele) NÃO é “these” (esses) e sim THOSE (aqueles). Portanto alternativa incorreta.
Na letra B, temos “Those are intelligent cars that work with computers” está certo, pois “those”
é plural de “that”, os substantivos aparecem no plural com o uso da letra “s”, está correto o uso
da palavra “are” e os verbos foram usados da maneira correta. Portanto, alternativa correta é a
letra B.
Na letra C, temos “Those are intelligents cars that works with computers” está errado pois o
adjetivo “intelligent” não é flexionado no plural, ou seja, a palavra é a mesma independente se é
plural ou singular, ela é usada sem o sufixo “s”.
Além disso o verbo “work” foi flexionado de maneira errada, já que está se referindo aos carros
(plural) o verbo deveria ser “work” ao invés de “works”. Portanto alternativa incorreta.
Na letra D, temos “These are intelligent cars that works with computers” está errado pois o plural
de “that” NÃO é “these”. Além disso o verbo “work” foi flexionado de maneira errada, já que está
se referindo aos carros (plural) o verbo deveria ser “work” ao invés de “works”. Portanto
alternativa incorreta.
Na letra E, temos “Those are intelligent cars that works with computers” está errado o verbo
“work” foi flexionado de maneira errada, já que está se referindo aos carros (plural) o verbo
deveria ser “work” ao invés de “works”. Portanto alternativa incorreta.
GABARITO: B

Questão 15 (COLÉGIO NAVAL/2019) – Read the extract from the text


“When the light turns green and there are no other cars in the intersection (...)”
Mark the option that can replace the underlined sentence
(A) there are some other cars
(B) there aren’t any other cars
(C) there is another car
(D) there are a few other cars
(E) there aren’t many cars
Comentários: A sentença “there are no other cars” significa que “não tem NENHUM outro carro”,
ou seja, não outros carros. Analisemos as alternativas. Veja que a frase tem um sentido negativo,
que não tem.
Na letra A, temos “there are some other cars” está errado pois a sentença significa “têm alguns
carros”, com sentido positivo e com ideia oposta ao enunciado. Portanto alternativa incorreta.
Na letra B, temos “there aren’t any other cars” está certo pois a sentença significa “não tem outros
carros”, exatamente o mesmo sentido da frase do enunciado. Portanto, alternativa correta é a
letra B.

AULA 01 – SKIMMING AND SCANNING AND OTHER TECHNIQUES. IDENTIFYING FALSE COGNATS. 76
TEACHER ANDREA BELO

Na letra C, temos “there is another cars” está errado pois a sentença significa “tem outro carro”,
com sentido positivo e com ideia oposta ao enunciado. Perceba que não é possível notar um
sentido negativo nesta alternativa. Portanto alternativa incorreta.
Na letra D, temos “there are a few other cars” está errado pois a sentença significa “têm alguns
outros carros”, com sentido positivo e com ideia oposta ao enunciado. Portanto alternativa
incorreta.
Na letra E, temos “there aren’t many cars” está errado pois a sentença significa “não tem muitos
carros”, ou seja, apesar de não ter muitos carros, continua existindo carros e a frase do enunciado
diz que não tem NENHUM carro. Portanto alternativa incorreta.
GABARITO: B

QUESTÕES EAM
Questão 01 (EAM/INÉDITA) – Read the dialogue and mark the right option to fill in the gaps
respectively.
A: _________ you go to Anne’s party yesterday?
B: Yes, I ________.
A: Who ________ you go with?
B: I ________ with my girlfriend. Were you there?
A: No, I ________.
a) Did / did / went / was / weren’t
b) Were / was / were / go / didn’t
c) Was / was / did / was / weren’t
d) Were / was / did / went / wasn’t
e) Did / did / did / went / wasn’t
Comentários: A primeira lacuna deve ser preenchida por “DID”. A presença do verbo “go” no
infinitivo indica que há um auxiliar “do, did, will” no lugar da lacuna. A presença da palavra
“yesterday” no fim da frase indica qu a frase está no passado. O passado exige o auxiliar “did” para
fazer a pergunta.
A segunda lacuna deve ser preenchida por “DID”. A pergunta foi feita com “did”, portanto, a
resposta curta deve ser dada utilizando o mesmo auxiliar (Yes, I did).
A terceira lacuna deve ser preenchida por “DID”. A pergunta se refere à festa do dia anterior e, por
isso, deve-se usar o auxiliar “did” para perguntar com quem a pessoa foi.
A quarta lacuna deve ser preenchida por “WENT”. A conversa está se passando toda no tempo
passado. Por isso, devemos usar “went” ao invés de “go”.
A quinta lacuna deve ser preenchida por “WASN’T”. A pergunta foi feita com o verbo “be” no
passado “were you there?”, logo a resposta deve também utilizar o verbo “be” no passado e na

AULA 01 – SKIMMING AND SCANNING AND OTHER TECHNIQUES. IDENTIFYING FALSE COGNATS. 77
TEACHER ANDREA BELO

negativa, já que a resposta começa com a palavra “no”. O verbo “be” no tempo passado que
concorda com o pronome “I” é “wasn’t”.
GABARITO: E

Questão 02 (EAM/INÉDITA) – Read the sentences and mark the correct option to fill in the
blanks respectively.
Fabi is ______ teammate. We usually create new english tests every week. We live _____ São
Paulo and we love to eat in different restaurants every day. We also love _______ english during
our Podcasts.
a) My / in / speaking
b) Your / at / speaking
c) Her / on / speaks
d) My / in / speak
e) His / in / speak
Comentários: A primeira lacuna deve ser preenchida por “MY”. O início do período seguinte deixa
claro que o texto trata do narrador e Fabi, ao começar com o pronome “we”.
A segunda lacuna deve ser preenchida por “IN”. Por regra, sempre que vai se falar EM alguma
cidade, usa-se a preposição “in”.
A terceira lacuna deve ser preenchida por “SPEAKING”. Trata-se de uma particularidade da língua
inglesa onde verbos como “like”, “love” ou “hate”, quando usados para descrever algo do qual se
gosta ou uma atividade que se gosta de fazer, trazem o verbo seguinte no gerúndio, ainda que
esse verbo não faça parte de um tempo verbal “continuous”. Esse gerúndio vai fazer o papel de
infinitivo.
GABARITO: A

Questão 03 (EAM/INÉDITA)

What’s the main verb tense used in the comic strip?


a) Present Continuous
b) Present Simple

AULA 01 – SKIMMING AND SCANNING AND OTHER TECHNIQUES. IDENTIFYING FALSE COGNATS. 78
TEACHER ANDREA BELO

c) Simple Past
d) Future Simple
e) Persent Perfect
Comentários: A alternativa A está incorreta. Não há nenhuma estrutura no “Present Continuous”
na tirinha. Nem mesmo a palavra “hunting” é “Present Continuous” nesse caso, porque “hunting”
exerce função de adjetivo no contexto apresentado.
A alternativa B está correta. Todas as falas do diálogo estão no “Present Simple”.
A alternativa C está incorreta. Não há nenhum verbo usado na tirinha que esteja no tempo
passado. Isso elimina a alternativa.
A alternativa D está incorreta. Não se pode observar o auxiliar “will” indicativo de futuro, e nem a
expressão “going to” que também serve como indicativo de futuro. Isso elimina a alternativa.
A alternativa E está incorreta. Não se pode observar a presença dos auxiliares “have” ou “has”
acrescidos do verbo conjugado no particípio passado, que é a estrutura do “present perfect”. Isso
elimina a alternativa.
GABARITO: B

Questão 04 (EAM/INÉDITA) – Use the verbs in the parentheses to complete the following
statements.
I – He always _______(run) early in the morning.
II – We need to ______(take) care of the elderly due to coronavirus outbreak.
III – Jack used to ______(live) in Boston. But now, he lives in Sao paulo.
Now mark the option which completes them respectively.
a) run / take / live
b) runs / take / lived
c) ran / takes / lived
d) runs / take / live
e) run / takes / lives
Comentários: A primeira lacuna deve ser preenchida por “runs”. A frase está no presente e o
sujeito é “he”. Portanto, o verbo precisa ser conjugado com a letra “S” no final.
A segunda lacuna deve ser preenchida por “take”. A frase está no presente e o sujeito é “we”.
Portanto, o verbo dave permanecer em sua forma original no infinitivo (take).
A terceira lacuna deve ser preenchida por “live”. A expressão “used to” pode ser traduzida como
“costumava” e, assim como no português, após essa expressão usa-se o verbo no infinitivo, ainda
que esteja falando de um fato passado. Ex: Ele costumava fazer…/He used to do…
GABARITO: D

AULA 01 – SKIMMING AND SCANNING AND OTHER TECHNIQUES. IDENTIFYING FALSE COGNATS. 79
TEACHER ANDREA BELO

Questão 05 (EAM/INÉDITA) – Look at the picture below.

What are they doing in the picture?


a) They are destroying the snowman.
b) They are building a snowman.
c) They are laughing at the snowman.
d) They are throwing snowballs.
e) They don’t like the snowman they built.
Comentários: A alternativa A está incorreta. Ao analisar a imagem, não é possível concluir que
elas estão destruindo o boneco de neve, mas sim construindo-o.
A alternativa B está correta. A imagem nos conduz a acreditar que elas estão, de fato,
construindo um boneco de neve.
A alternativa C está incorreta. Elas não estão rindo do boneco de neve. Elas estão rindo porque
estão se divertindo brincando na neve.
A alternativa D está incorreta. Uma das meninas está com uma bola de neve em uma das mãos,
mas não há nenhum indício, baseado na imagem, de que ela pretende jogar a bola de neve na
outra menina.
A alternativa E está incorreta. Não se pode concluir que elas não gostaram do boneco de neve
que construíram porque elas estão rindo enquanto terminam os últimos detalhes do boneco.
Não há nenhum indício de insatisfação nos rostos delas.
GABARITO: B

QUESTÕES EEAR
Read the text and answer questions

AULA 01 – SKIMMING AND SCANNING AND OTHER TECHNIQUES. IDENTIFYING FALSE COGNATS. 80
TEACHER ANDREA BELO

Diplomat
There are three main aspects to this profession: a diplomat has to keep his country informed
about pertinent internacional events, promote a favorable image of his country and protect his
country’s interests.
Whoever is interested in a diplomatic career has to be extremely familiar with political,
economical, scientific, cultural and administrative issues. To be a diplomat, it is essential to have
a good knowledge of English, not only the conversational language, but also the technical terms
in internacional law and diplomacy itself.
To follow this career, besides being fluently bilingual, one needs a standard college education and
has to take and do well in the Rio Branco Institute examination in Brasilia.
English is so important in this career that in the first part of this selection exam, the applicant has
to demonstrate his or her proficiency in the english language. Then, during the course, foreign
language classes become a priority, giving future diplomats the necessary expertise to deal with
the areas of official correspondences, diplomatic negotiation and internacional media.
(Adapted from Inglês no mundo do trabalho)

Questão 1 (EEAR/2020 – BCT)


Change the following sentence to the passive voice:
“A diplomat must inform his country about internacional events.”
a) His country must inform a diplomat about internacional events.
b) His country has to be informed by a diplomat about international events.
c) His country will be informed by a diplomat about international events.
d) His country must be informed by a diplomat about internacional events.
Comentários: A alternativa A está incorreta. A alternativa continua na voz ativa, mas alterando o
que foi dito pelo trecho do enunciado.
A alternativa B está incorreta. O erro desta alternativa está no uso do verbo “has”, pois o trecho
do enunciado traz o verbo “must”. Não se deve trocar o verbo utilizado pelo enunciado, apenas
usar os mesmos termos que o enunciado usou e estruturar a frase na voz passiva.
A alternativa C está incorreta. O erro desta alternativa está no uso do verbo “will be”, pois o trecho
do enunciado traz o verbo “must”. Não se deve trocar o verbo utilizado pelo enunciado, apenas
usar os mesmos termos que o enunciado usou e estruturar a frase na voz passiva.
A alternativa D está correta. A alternativa faz a correta passagem do trecho da voz ativa para a
voz passiva.
GABARITO: D

Questão 2 (EEAR/2020 – BCT)


According to the text, we can infer that: A Diplomat ________.

AULA 01 – SKIMMING AND SCANNING AND OTHER TECHNIQUES. IDENTIFYING FALSE COGNATS. 81
TEACHER ANDREA BELO

a) doesn’t need a normal college education


b) must be familiar with English, but only in the conversational language
c) doesn’t have to give much attention to foreign language classes during the course
d) must have a comprehensive knowlegde of political, economical, scientific and administrative
problems
Comentários: A alternativa A está incorreta. O texto diz que o candidato a diplomata precisa ter
educação superior padrão, contrariamente ao que diz a alternativa.
A alternativa B está incorreta. O texto diz que o candidato precisa ser familiarizado com a língua
inglesa, não apenas no nível de conversação. A alternativa diz que ele precisa ser familiarizado
apenas com conversação.
A alternativa C está incorreta. O texto diz o contrário da alternativa. Durante o curso, os
candidatos precisam dar muita atenção às aulas de línguas estrangeiras.
A alternativa D está correta. O texto diz que os candidatos precisam ter um abrangente
conhecimento sobre política, economia, além de problemas científicos e administrativos,
exatamente como diz a alternativa.
GABARITO: D

Questão 3 (EEAR/2020 – BCT)


In …” giving future diplomats the necessary expertise to deal with”, the underline word in the
text is closest in meaning to, EXCEPT:
a) Skill
b) Knowledge
c) proficiency
d) unawareness
Comentários: A alternativa A está incorreta. A palavra “expertise” significa habilidade,
conhecimento. A palavra “skill” significa habilidade, aptidão, havendo correlação com a palavra
do enunciado.
A alternativa B está incorreta. A palavra “expertise” significa habilidade, conhecimento. A palavra
“knowledge” significa conhecimento, havendo correlação com a palavra do enunciado.
A alternativa C está incorreta. A palavra “expertise” significa habilidade, conhecimento. A palavra
“proficiency” significa proficiência, havendo correlação com a palavra do enunciado.
A alternativa D está correta. A palavra “expertise” significa habilidade, conhecimento. A palavra
“unawareness” significa o contrário disso, desconhecimento, não ter ciência sobre algo, não
havendo, assim, correlação com a palavra do enunciado.
GABARITO: D

Read the text and answer questions

AULA 01 – SKIMMING AND SCANNING AND OTHER TECHNIQUES. IDENTIFYING FALSE COGNATS. 82
TEACHER ANDREA BELO

Oil contaminating Brazil's beaches very likely from Venezuela


Anna Jean Kaiser
Thick crude oil that has stained hundreds of miles of pristine Brazilian beach in recent weeks
probably originated in Venezuela, the Brazilian government has said, in an accusation likely to
further strain relations between the two countries.
Brazilians authorities have been investigating the growing disaster for more than a month, as the
oil has spread to more than 130 beaches across nine states.
Ricardo Salles, the country’s environment minister, told that a study by Petrobrás had concluded
that the oil “is very likely from Venezuela. He said that a foreign ship near Brazil’s coastline
appeared to have caused the spill.
There was no immediate response from Venezuela.
Social media users have shared shocking images of the spill, showing kilometers of white sand
stained with oil blotches and dead, oil-covered turtles and dolphins. One video shows thick black
oil lapping up against a rocky jetty.
Adapted from The Guardian

Questão 4 (EEAR/2020 – BCT)


According to the text, we can infer that ________.
a) The spill was caused by a domestic ship near Brazilian beaches
b) No dolphins and turtles died according to the images shared by the media
c) A dense and natural oil marked hundreds of miles of clean beaches recently
d) The government is investigating to discover how many beaches are involved in the disaster
Comentários: A alternativa A está incorreta. O texto diz que o vazamento foi causado por um
navio estrangeiro, e não doméstico.
A alternativa B está incorreta. O texto diz que golfinhos e tartarugas foram encontrados mortos e
cobertos de óleo, contrariamente ao que diz a afirmativa.
A alternativa C está correta. O texto diz óleo cru sujou centenas de milhas de praias limpas
recentemente, exatamente como diz a alternativa.
A alternativa D está incorreta. O texto diz que o governo tem investigado o crescente incidente
por mais de um mês, mas não diz que investiga quantas praias estão envolvidas no desastre.
GABARITO: C

Questão 5 (EEAR/2020 – BCT)


The word “strain”, underline in the text, is closest in meaning to, EXCEPT:
a) close
b) tense

AULA 01 – SKIMMING AND SCANNING AND OTHER TECHNIQUES. IDENTIFYING FALSE COGNATS. 83
TEACHER ANDREA BELO

c) difficult
d) problematic
Comentários: A alternativa A está correta. A palavra “strain” significa dificuldade, tensão. A
palavra “close” significa aproximar, estreitar relações, não havendo correlação com a palavra do
enunciado.
A alternativa B está incorreta. A palavra “strain” significa dificuldade, tensão.
A palavra “tense” significa gerar tensão nas relações, havendo correlação com a palavra do
enunciado.
A alternativa C está incorreta. A palavra “strain” significa dificuldade, tensão. A palavra “difficult”
significa dificultar as relações, havendo correlação com a palavra do enunciado.
A alternativa D está incorreta. A palavra “strain” significa dificuldade, tensão. A palavra
“problematic” significa problemático, havendo correlação com a palavra do enunciado.
GABARITO: A

QUESTÕES EFOMM
Based on the text below, answer questions 1 to 10.
Residents and businesses of south Florida are experiencing regular episodes of water in the
streets. In the battle against rising seas, the region – which has more to lose than almost
anywhere else in the world – is becoming ground zero.
The first time my father’s basement flooded, it was shortly after he moved in. The building was
an ocean-front high-rise in a small city north of Miami called Sunny Isles Beach. The marble lobby
had a waterfall that never stopped running; crisp-shirted valets parked your car for you. For the
residents who lived in the more lavish flats, these cars were often BMWs and Mercedes. But no
matter their value, the cars all wound up in the same place: the basement.
When I called, I’d ask my dad how the building was doing. “The basement flooded again a couple
weeks ago,” he’d sometimes say. Or: “It’s getting worse.” It’s not only his building: he’s also driven
through a foot of water on a main road a couple of towns over and is used to tiptoeing around
pools in the local supermarket’s car park.
Ask nearly anyone in the Miami area about flooding and they’ll have an anecdote to share. Many
will also tell you that it’s happening more and more frequently. The data backs them up.
It’s easy to think that the only communities suffering from sea level rise are far-flung and remote.
And while places like the Solomon Islands and Kiribati are indeed facing particularly dramatic
challenges, they aren’t the only ones being forced to grapple with the issue. Sea levels are rising
around the world, and in the US, south Florida is ground zero – as much for the adaptation
strategies it is attempting as for the risk that it bears.
One reason is that water levels here are rising especially quickly. The most frequently-used range
of estimates puts the likely range between 15-25cm (6-10in) above 1992 levels by 2030, and 79-
155cm (31-61in) by 2100. With tides higher than they have been in decades – and far higher than
when this swampy, tropical corner of the US began to be drained and built on a century ago –

AULA 01 – SKIMMING AND SCANNING AND OTHER TECHNIQUES. IDENTIFYING FALSE COGNATS. 84
TEACHER ANDREA BELO

many of south Florida’s drainage systems and seawalls are no longer enough. That means not only
more flooding, but challenges for the infrastructure that residents depend on every day, from
septic tanks to wells. “The consequences of sea level rise are going to occur way before the high
tide reaches your doorstep,” says William Sweet, an oceanographer at the National Oceanic and
Atmospheric Administration (NOAA).
The flooding would be a challenge for any community, but it poses particular risks here. One
recent report estimated that Miami has the most to lose in terms of financial assets of any coastal
city in the world, just above Guangzhou, China and New York City. This 120-mile (193km) corridor
running up the coast from Homestead to Jupiter – taking in major cities like Miami, Fort
Lauderdale and West Palm Beach – is the eighth most populous metropolitan area in the US. It’s
also booming. In 2015, the US Census Bureau found that the population of all three counties here
was growing – along with the rest of Florida – at around 8%, roughly twice the pace of the US
average. Recent studies have shown that Florida has more residents at risk from climate change
than any other US state.
Adapted from https://www.bbc.com/future/article/20170403-miamis-fight-against-sea-level-rise

Questão 01 (EFOMM/INÉDITA) – It is possible to infer from the text that


a) technology will solve the problem Miami is facing due to sea water level rising.
b) the Earth is changing and areas like Miami will disapear soon.
c) in the future, water levels will decrease naturally.
d) buildings near the sea are endangered if sea water level keeps rising at that speed.
e) Miami metropolitan area will be the most populous area in the US in a few years.
Comentários: A alternativa A está incorreta. O texto não fala sobre o problema ser solucionado
com o uso de tecnologia. Portanto, é incorreto afirmar que o uso de tecnologia vai resolver o
problema de Miami.
A alternativa B está incorreta. Em momento algum o texto fala que a area de Miami vai
desaparecer. O texto diz que as edificações perto do mar estão em perigo por causa dos
constantes alagamentos na área.
A alternativa C está incorreta. O texto não diz, nem nos permite concluir que o nível de água irão
diminuir naturalmente. O texto aborda um assunto como um problema crescente, e não algo
temporário.
A alternativa D está correta. Podemos inferir do texto que as edificações perto do mar estão em
perigo caso não haja uma solução para o crescente nível da água do mar.
A alternativa E está incorreta. O texto diz que a região metropolitana de Miami é a oitava mais
populosa dos Estados Unidos e que é uma região cuja população cresce acima da média
americana, mas em momento algum o texto nos dá a entender que a região passará a ser a região
mais populosa do país em alguns anos.
GABARITO: D

AULA 01 – SKIMMING AND SCANNING AND OTHER TECHNIQUES. IDENTIFYING FALSE COGNATS. 85
TEACHER ANDREA BELO

Questão 02 (EFOMM/INÉDITA) – Read the statements about the text and decide whether they
are TRUE (T) or FALSE (F). Mark the correct option.
I – The floods have been taking place regularly in Sunny Isles Beach.
II – There are swimming pools in supermarkets’ car parks in Miami.
III – Residents in Miami area tell a lot of lies about flooding.
IV – Not only remote places are suffering from sea level rise.
V – The BMWs and Mercedes are protected against floodings because they are expensive cars.
a) F / T / T / F / F
b) T / F / F / T / F
c) T / T / T / F / T
d) F / F / F / T / F
e) F / T / F / T / T
Comentários: A sentença I está correta. O texto diz que as pessoas que moram na região relatam
uma quantidade cada vez maior de enchentes por conta do aumento do nível da água do mar. O
texto diz, ainda, que os dados recentes corroboram a versão dos moradores.
A sentença II está incorreta. O texto afirma que há poças de água no estacionamento do
supermercado local, e não que há piscinas nos estacionamentos de Miami.
A sentença III está incorreta. O texto não diz que os moradores da região contam mentiras sobre
os alagamentos. O texto diz que as pessoas estão certas em seus relatos.
A sentença IV está correta. O texto é bem explícito ao dizer que não são apenas lugares remotos
que sofrem com o aumento do nível do mar, mas que regiões populosas como Miami também
sofrem com os efeitos desse fenômeno.
A sentença V está incorreta. O texto diz que, independentemente do valor, todos os carros
acabam no mesmo lugar, o porão. Portanto, é incorreto dizer que BMWs e Mercedes estão
protegidos contra as enchentes.
Temos a sequência T / F / F / T / F.
GABARITO: B

Questão 03 (EFOMM/INÉDITA) – In the excerpt “It’s easy to think that the only communities
suffering from sea level rise are far-flung and remote.”, the word in bold means:
a) Distant
b) Poor
c) Ugly
d) Neglected
e) Near

AULA 01 – SKIMMING AND SCANNING AND OTHER TECHNIQUES. IDENTIFYING FALSE COGNATS. 86
TEACHER ANDREA BELO

Comentários: A alternativa A está correta. “Distant” significa distante. “Far-flung” significa


longínquo e pode ser definido como distante.
A alternativa B está incorreta. “Poor” significa pobre, enquanto“Far-flung” significa longínquo.
Portanto, não podemos dizer que “poor” define “far-flung”.
A alternativa C está incorreta. “Ugly” significa feio, enquanto“Far-flung” significa longínquo.
Portanto, não podemos dizer que “ugly” define “far-flung”.
A alternativa D está incorreta. “Neglected” significa negligenciado, enquanto“Far-flung” significa
longínquo. Portanto, não podemos dizer que “neglected” define “far-flung”.
A alternativa E está incorreta. “Near” significa perto, enquanto“Far-flung” significa longínquo.
Portanto, não podemos dizer que “near” define “far-flung”.
GABARITO: A

Questão 04 (EFOMM/INÉDITA) – According to the text, sea water level


a) reached its maximum level recently.
b) begets consequences that will be seen in many years from now.
c) is considerably higher than it was 100 years ago.
d) is ideal to attract tourists now.
e) doesn’t affect people’s lives in the area.
Comentários: A alternativa A está incorreta. O texto não diz que a água do mar já atingiu seu nível
máximo. Na verdade, o texto diz que o nível ainda deve subir muito até 2100.
A alternativa B está incorreta. O texto afirma que as consequências do aumento do nível do mar
poderão ser vistas muito antes do nível subir até a porta da sua casa, e não que serão vistas daqui
a muitos anos.
A alternativa C está correta. O texto afirma que a maré está muito mais alta do que há cem anos
atrás, quando este canto tropical e pantanoso dos Estados Unidos começou a ser drenado e
construído.
A alternativa D está incorreta. O texto não fala nada sobre o atual nível da água do mar ser ideal
para atrair turistas.
A alternativa E está incorreta. De acordo com o texto, as vidas das pessoas que vivem na área é
afetada pelas enchentes que ocorrem. Não há como ter uma rotina normal com as ruas alagadas
pela água do mar.
GABARITO: C

Questão 05 (EFOMM/INÉDITA) – Choose the correct option to complete the paragraph below.
Those who are non-essential workers (and followed the rules) have been _____ home all day,
every day, save for trips to the grocery store or for socially distant walks. And yet, the number of
new cases of COVID-19 _____ the U.S. continues to go up each day, _____ about 2 to 4 percent.

AULA 01 – SKIMMING AND SCANNING AND OTHER TECHNIQUES. IDENTIFYING FALSE COGNATS. 87
TEACHER ANDREA BELO

Adapted from https://www.msn.com/en-us/health/other/if-people-are-staying-home-why-is-coronavirus-still-spreading/ar-BB13HSW8?ocid=bingcovid

a) in / on / on
b) on / in / at
c) at / in / by
d) at / on / in
e) in / in / on
Comentários: A primeira lacuna deve ser preenchida por “at”. Da mesma forma que, em
português, dizemos que estamos EM casa, em inglês, dizemos que estamos “AT home”. “At” é a
preposição adequada para acompanhar a palavra “home”.
A segunda lacuna deve ser preenchida por “in”. Quando falamos que algo acontece em
determinado país, ou que nós ou outra pessoa está em um determinado país, usamos a
preposição “in”. In the US; in Brazil; in the UK; in Canada; in France etc.
A terceira lacuna deve ser preenchida pela preposição “by”. Essa preposição é ideal aqui porque
ela ajuda a expressar a variação que a palavra “about” propõe, já que não estamos falando de um
número exato.
Temos a sequência: at / in / by
GABARITO: C

Questão 06 (EFOMM/INÉDITA) – Choose the best alternative to complete the excerpt below.
Finding _____ effective coronavirus vaccine ______ a global priority in ending the pandemic. US
government leaders have put forward the ambitious timeline ______ have one by the end of
2020. It typically takes several years to develop ______ vaccine.
https://www.businessinsider.com/moderna-coronavirus-vaccine-releases-first-human-trial-results-2020-5

a) an / has become / to / a
b) a / has become / to / a
c) an / became / to / the
d) an / has become / of / the
e) a / became / of / a
Comentários: A primeira lacuna deve ser preenchida por um artigo, como sugerem as
alternativas. Nesse caso, o artigo correto a completar a lacuna é “an”, pois “effective” inicia com
uma vogal. Isso já elimina as alternativas B e E.
A segunda lacuna deve ser preenchida por “has become”. O present perfect é necessário por se
tratar de uma situação que se inicia no passado e ocorre até o presente. Assim, continuamos com
as alternativas A e D como possíveis respostas.

AULA 01 – SKIMMING AND SCANNING AND OTHER TECHNIQUES. IDENTIFYING FALSE COGNATS. 88
TEACHER ANDREA BELO

A terceira lacuna deve ser preenchida pela preposição “to”. A outra opção de preenchimento
apresentada pela questão é a preposição “of”, mas para usarmos “of” nessa situação, o verbo
“have” deveria estar no gerúndio (having). Dessa forma, já temos a alternativa A como resposta,
mas vamos analisar a última lacuna para nos certificar.
A quarta lacuna pode ser preenchida pelo artigo indefinido “a”. A frase se inicia deixando claro
que não se está falando de nenhuma vacina específica, mas sim de vacinas de forma geral. Dessa
forma, não se pode usar o artigo definido para falar de vacinas de forma geral.
Temos a alternativa A como correta.
GABARITO: A

Questão 07 (EFOMM/INÉDITA) – Choose the correct alternative to complete the paragraph


below.
In the last few weeks a spate of American stores _______ headlines after putting up signs _______
customers who wear masks they will be denied entry. On Thursday, Vice reported on a Kentucky
convenience store that put up a sign reading: “NO Face Masks ______ in store. Lower your mask
or go somewhere else.
Adapted from https://www.theguardian.com/us-news/2020/may/22/us-stores-against-face-masks

a) have made / telling / allow


b) made / told / allowed
c) have made / telling / allowed
d) made / have told / allow
e) has made / told / allowed
Comentários: A primeira lacuna deve ser preenchida por “have made”, pois o texto é escrito no
passado e conta uma história se iniciou e permanece acontecendo no presente.
A segunda lacuna deve ser preenchida por “telling” porque as outras opções disponíveis exigiriam
outra partícula para que o texto fizesse sentido.
A terceira lacuna deve ser preenchida por “allowed”, pois a outra opção não se encaixa de forma
adequada no texto, já que usar “allow” seria como dizer que nenhuma mácara autoriza na loja.
Ficaria completamente sem sentido.
Temos a sequência “have made / telling / allowed”
GABARITO: C

Questão 08 (EFOMM/INÉDITA) – Which option is correct?


a) She needs two different equipments to run her business.
b) The pandemy can affect world economy.
c) The social media become very important to businesses.

AULA 01 – SKIMMING AND SCANNING AND OTHER TECHNIQUES. IDENTIFYING FALSE COGNATS. 89
TEACHER ANDREA BELO

d) His nose is running because he has a cold.


e) The Real family is very traditional in the UK.
Comentários: A alternativa A está incorreta. Não é correto dizer “two different equipments”, seria
correto dizer “two pieces of equipment”, pois equipment é uma palavra que não varia em
número, ou seja, não tem plural.
A alternativa B está incorreta. É incorreto utilizar a palavra “pandemy” para falar de pandemia. A
palavra correta na Língua Inglesa para pandemia é “pandemic”.
A alternativa C está incorreta. O erro desta alternativa está no uso do verbo “become” que está
conjugado de forma incorreta. A forma correta de escrever seria “has become”, pois trata-se de
algo que começou no passado e se estende até o presente, por isso é indicado o tempo verbal
present perfect.
A alternativa D está correta. Para dizer que o nariz está escorrendo, usamos a palavra “running”
como usado na alternativa. Se você fez o simulado anterior da EFOMM e prestou atenção na
correção comentada, não teve dificuldades em acertar essa questão, já que esclareço esse tópico
nos comentários do último simulado.
A alternativa E está incorreta. Não é correto usar “Real” para falar de Real no sentido de realeza.
Em inglês, usamos a palavra Royal para tratar da realeza. Portanto, seria correto escrever “royal
family”.
GABARITO: D

Questão 09 (EFOMM/INÉDITA) – Mark the option which corresponds to the correct sentences.
I – The person who rules a soccer game is called judge.
II – Songs for children are called nursery rhymes.
III – Mooring is an important process when the ship is sailing.
IV – Your best friends aren’t never far from you.
V – A ship can also be called vessel.
a) I and II
b) II and V
c) III and V
d) II and IV
e) III and IV
Comentários: A sentença I está incorreta. A pessoa que comanda um jogo de futebol não é
chamada de juiz, mas sim de árbitro. Em inglês, usamos a palavra “referee” e não a palavra
“judge”, que é usada apenas para juiz de direito.
A sentença II está correta. As músicas infantis, em inglês, são chamadas de “nursery rhymes”.
A sentença III está incorreta. A palavra “mooring” significa amarração. A amarração é o ato de
“amarrar”, ou prender o navio ao porto quando ele está atracando. Portanto, amarração é um

AULA 01 – SKIMMING AND SCANNING AND OTHER TECHNIQUES. IDENTIFYING FALSE COGNATS. 90
TEACHER ANDREA BELO

processo importante na atracação do navio, e não quando ele está navegando, pois quando está
navegando, não é necessário amarrar o navio a lugar algum.
A sentença IV está incorreta. O erro da alternativa está no fato de usar uma dupla negativa. Em
português, usamos a dupla negativa, mas, em inglês, não se usa dupla negativa, pois a segunda
negativa acaba por anular a primeira.
A sentença V está correta. De fato, “ship” ou navio em português, pode ser chamado de “vessel”,
que significa embarcação. Mas é importante notar que nem todo “vessel” pode ser chamado de
“ship”.
Temos as sentenças II e V corretas.
GABARITO: B

Questão 10 (EFOMM/INÉDITA) – Which option is correct to complete the sentences below?


1 – He heard her ______ the stairs.
2 – He saw her ______ the door.
3 – I watched him ______ the job.
4 – In order to be promoted, you need to be ______ by your boss.
5 – He couldn’t see his child _______ with our dog.

a) climb / slam / do / valued / play


b) climbed / slamming / do / value / play
c) climbing / slam / doing / valuing / playing
d) climbing / slamming / doing / valued / playing
e) climbed / slamming / doing / valued / playing
Comentários: A sentença 1 deve ser preenchida por “climbing”. Ele a ouviu subindo as escadas.
Ele a ouviu enquanto ele subia, por isso precisamos usar o verbo no gerúndio “climbing”.
A sentença 2 deve ser preenchida por “slamming”. Não é possível usar “slam” nesse contexto,
pois bater à porta não é algo instantâneo, mas sim um processo, pois a pessoa precisa empurrar
a porta para depois a porta bater no batente. Portanto precisamos usar o gerúndio para
demonstrar que a pessoa assistiu a todo o processo.
A sentença 3 deve ser preenchida por “doing”. A frase dá a entender que a pessoa assistia
enquanto o outro trabalhava, e isso é um processo em andamento. Por isso, devemos usar o
gerúndio nesse caso.
A sentença 4 deve ser preenchida por “valued”. Para ser promovido, você precisa ser valorizado
pelo seu chefe. O uso do verbo “value” no particípio é necessário por conta de a frase estar na
voz passiva.
A sentença 5 deve ser preenchida por “playing”. O uso do gerúndio é necessário porque ele queria
ter visto a criança no ato de brincar com o cachorro. Ele queria ter visto o exato momento em que

AULA 01 – SKIMMING AND SCANNING AND OTHER TECHNIQUES. IDENTIFYING FALSE COGNATS. 91
TEACHER ANDREA BELO

ela brincava com o cachorro. Como ele queria ter acompanhado a ação enquanto ela acontecia,
só podemos usar o verbo no gerúndio.
Temos a sequência: climbing / slamming / doing / valued / playing.
GABARITO: D

QUESTÕES EPCAR
The WHO has redefined burnout as a syndrome linked to chronic work stress. There’s a difference
between a busy workload and something more serious, writes Zaria Gorvett.
If you said you were suffering from ‘burnout’ in the early 1970s, you might have raised some
eyebrows.
At the time, the term was used informally to describe the side effects that heavy drug users
experienced: the general dimming of the mental faculties, for example, as was the case with many
a party animal. However, when German-American psychologist Herbert Freudenberger first
recognised the problem of burnout in New York City in 1974, at a clinic for addicts and homeless
people, Freudenberger wasn’t thinking of drug users.
The clinic’s volunteers were actually struggling, too: their work was intense, and many were
beginning to feel demotivated and emotionally drained. Though they had once found their jobs
rewarding, they had become cynical and depressed; they weren’t giving their patients the
attention they deserved. Freudenberger defined this alarming new condition as a state of
exhaustion caused by prolonged overwork – and borrowed the term ‘burnout’ to describe it.
Its popularity was explosive, and today burnout is a global phenomenon. Although statistics on
the prevalence of burnout specifically are hard to come by, 595,000 people in the UK alone
suffered from workplace stress in 2018.
Sportspeople get it. YouTube stars get it. Entrepreneurs get it. Freudenberger himself eventually
got it. Late last month, the World Health Organization (WHO) announced that the trendy problem
will be recognised in the latest International Classification of Diseases manual, where it is
described as a syndrome “resulting from chronic workplace stress that has not been successfully
managed”.
According to the WHO, burnout has three elements: feelings of exhaustion, mental detachment
from one’s job and poorer performance at work. But waiting until you’re already fully burned out
to do something about it doesn’t help at all –and you wouldn’t wait to treat any other illness until
it was too late.
Feeling the burn
So how can you tell if you’re almost – but not quite – burned out?
“A lot of the signs and symptoms of pre-burnout would be very similar to depression,” says
Siobhán Murray, a psychotherapist based in County Dublin, Ireland, and the author of a book
about burnout, The Burnout Solution. Murray suggests looking out for creeping bad habits, such
as increased alcohol consumpution and relying on sugar to get you through the day. Also watch
out for feelings of tiredness that won’t go away. “So that even if you do sleep well, by

AULA 01 – SKIMMING AND SCANNING AND OTHER TECHNIQUES. IDENTIFYING FALSE COGNATS. 92
TEACHER ANDREA BELO

10 in the morning you’re already counting down the hours to bed. Or not having the energy to
exercise or go for a walk.”
As soon as you begin to feel this way, Murray advises going to see your doctor.
“Depression and pre-burnout are very similar, but as much as there was a lot of enthusiasm
recently that burnout has now become a medical condition, it is still not – it is still classified as an
occupational phenomenon.” It’s important to get help from a medical professional who can
distinguish between the two, because although there are many treatment options for depression,
burnout is still best tackled by making lifestyle changes.
And how do you know if you’re really on the cusp of burnout, or just going through a challenging
month? “Stress is really important, and anxiety is what motivates us to do well,” says Murray. “It’s
when we’re continually exposed to stress and anxiety, that we’re not letting go, that it starts to
turn into burnout.”
Take that big project you’ve been working on. It’s normal to feel a kick of adrenaline when you
think about it, and maybe it’s kept you up at night. But, Murray suggests, if you still feel restless
once it’s over, it’s time to consider if you’re at risk of burnout. “It’s when you’re bringing that with
you into the next stage of your day, and adding to it continually,” she says.
Another classic sign of inching closer to burnout is cynicism: feeling like your work has little value,
avoiding social commitments and becoming more susceptible to disappointment.
“Someone on the brink will probably begin to feel emotionally numbed or mentally distant,” says
Jacky Francis Walker, a psychotherapist based in London who specialises in burnout. “Like they
don’t have the capacity to engage as much in the ordinary things of life.”
She also recommends looking for the final tell-tale sign of burnout, which is the unshakeable
feeling that the quality of your work is beginning to slip. “People say ‘but this isn’t me!’, ‘I’m not
like this’, ‘I can usually do x,y and z’. But obviously if they are in a state of physical depletion, then
they aren’t in their normal range of capabilities,” says Walker.
If this seems less than scientific, look to the Maslach Burnout Inventory (MBI), a test designed to
measure burnout. The most widely used is the MBI-General Survey, which measures things like
exhaustion, cynicism, and some how well you think you’re doing at work.
First published in 1981, it has been cited hundreds of times in studies since. Although it’s typically
used to measure burnout once it's in full swing, there’s no reason you can't apply it to see if you’re
getting close.
(Adapted from https://www.bbc.com/worklife/article/20190610-how-to-tell-if-youve-got-pre-burnout)

Questão 01 (EPCAR/INÉDITA) – Mark the correct option.


a) Burnout is now considered a medical condition.
b) Burnout can improve people’s performance at work.
c) Burnout was widely known in the 1970’s.
d) Herbert Freudenberger first used the term burnout in the 1970’s.

AULA 01 – SKIMMING AND SCANNING AND OTHER TECHNIQUES. IDENTIFYING FALSE COGNATS. 93
TEACHER ANDREA BELO

Comentários: A alternativa A está incorreta. O texto afirma que o burnout ainda não é
considerado uma doença, mas sim um fenômeno ocupacional.
A alternativa B está incorreta. O texto afirma que o burnout gera uma queda de rendimento no
trabalho, e não um aumento de performance como diz a alternativa.
A alternativa C está incorreta. O texto afirma que se você dissesse que estava sofrendo de burnout
na década de 70, as pessoas levantariam as sobrancelhas, pois na época, esse termo era usado
para descrever o efeito colateral que usuários de drogas pesadas experimentavam. Portanto, não
podemos dizer que o fenômeno era amplamente conhecido na época.
A alternativa D está correta. O texto afirma que Herbert Freudenberger pegou “emprestado” a
palavra burnout para descrever o que acontecia com funcionários que trabalhavam sujeitos a
altos níveis de stress e fadiga acumulados. E o texto afirma que isso ocorreu em 1974.
GABARITO: D

Questão 02 (EPCAR/INÉDITA) – According to the text, mark the correct alternative.


a) The psychologist discovered burnout by accident while analizing drug addicts and homeless.
b) The clinic’s volunteers were having a good time doing their work.
c) Anyone is subject to get it if they reach exhaustion due to prolonged overwork and stress.
d) Drug addicts were the first to suffer from burnout due to their stressful routine.
Comentários: A alternativa A está incorreta. O texto não nos permite entender que a descoberta
do psicólogo foi acidental, mas sim proposital, pois ele não estava observando os pacientes da
clínica, mas os voluntários.
A alternativa B está incorreta. O texto diz que os voluntários estavam desmotivados e passando
por dificuldades no trabalho, e não que eles estavam se divertindo ou felizes.
A alternativa C está correta. O texto cita diversos exemplos de pessoas que foram afetadas pelo
burnout, nos dando a entender que toda e qualquer pessoa está sujeita ao burnout se chegarem
à exaustão por conta de stress e excesso de trabalho por períodos prolongados.
A alternativa D está incorreta. O texto diz que foram os voluntários da clínica que foram
observados pelo psicólogo, e não os pacientes. Foi baseado na observação dos voluntários que
ele chegou à definição de burnout.
GABARITO: C

Questão 03 (EPCAR/INÉDITA) – In the sentence “the general dimming of the mental faculties”
(paragraph 2) the word dimming means
a) decrease.
b) augmentation.
c) assessment.
d) improvement.

AULA 01 – SKIMMING AND SCANNING AND OTHER TECHNIQUES. IDENTIFYING FALSE COGNATS. 94
TEACHER ANDREA BELO

Comentários: A alternativa A está correta. “Decrease” significa diminuição, e “dimmimg” tem


significado muito parecido, sendo usado para dar a ideia de que algo está diminuindo ou
perdendo intensidade.
A alternativa B está incorreta. “Augmentation” indica aumento, e a palavra “dimming” não tem
nenhuma relação de significado, pois “dimming” significa diminuir ou perder intensidade.
A alternativa C está incorreta. “Assessment” indica avaliação, e a palavra “dimming” não tem
nenhuma relação de significado, pois “dimming” significa diminuir ou perder intensidade.
A alternativa D está incorreta. “Improvement” significa melhora, e não guarda nenhuma relação
com a palavra “dimming”, que significa diminuir ou perder a intensidade.
GABARITO: A

Questão 04 (EPCAR/INÉDITA) – Mark the correct question to the sentence below.


“Freudenberger defined this alarming new condition as a state of exhaustion caused by
prolonged overwork”
a) How did Freudenberger define this alarming new condition?
b) How has Freudenberger defined this alarming new condition?
c) How had Freudenberger defined this alarming new condition?
d) How did Freudenberger defined this alarming new condition?
Comentários: A alternativa A está correta. O trecho do enunciado está conjugado no past simple,
portanto, a pergunta precisa ser feita nesse mesmo tempo verbal.
A alternativa B está incorreta. O uso do present perfect é o erro da alternativa, pois a afirmativa
do enunciado está no past simple e, por isso, deveria ser usado “did” ao invés de “has defined”.
A alternativa C está incorreta. O uso do past perfect é o erro da alternativa, pois a afirmativa do
enunciado está no past simple e, por isso, deveria ser usado “did” ao invés de “had defined”.
A alternativa D está incorreta. O erro da alternativa está no fato de o verbo “defined” estar
conjugado no passado, pois a presença do auxiliar “did” elimina a necessidade de conjugar o verbo
para configurar o tempo verbal.
GABARITO: A

Questão 05 (EPCAR/INÉDITA) – Mark the alternative that completes the sentence.


Some symptoms the psychologist noticed the volunteers had to deal with were
a) that they were drug abusers and most of them were homeless.
b) that they started to feel demotivated, emotionally drained, cynical and depressed.
c) that they were getting sick very frequently.
d) that they were being diagnosed with generalized anxiety and depression.

AULA 01 – SKIMMING AND SCANNING AND OTHER TECHNIQUES. IDENTIFYING FALSE COGNATS. 95
TEACHER ANDREA BELO

Comentários: A alternativa A está incorreta. Não eram os voluntários que eram viciados em
drogas nem mendigos, mas sim os pacientes.
A alternativa B está correta. Segundo o texto, os voluntários começaram a se sentir desmotivados
e emocionalmente esgotados. Além disso, embora eles achassem seus trabalhos
recompensadores no passado, acabaram se tornando cínicos e depressivos.
A alternativa C está incorreta. O texto não diz, em momento algum, que os voluntários estavam
ficando doentes com frequência. Os sintomas eram mais psicológicos do que físicos.
A alternativa D está incorreta. O texto não afirma que os voluntários eram diagnosticados com
ansiedade generalizada e depressão. O texto diz que os voluntários começaram a ficar
deprimidos, mas isso é colocado de forma comportamental, e não clínica.
GABARITO: B

Questão 06 (EPCAR/INÉDITA) – Mark the alternative that DOESN’T complete the sentence
below.
The psychologist Herbert Freudenberger
a) was who first used the word burnout to describe this exhaustion condition.
b) recognised the problem of burnout in 1974 in New York City for the first time.
c) created this condition so that he would become known for discovering a disease.
d) defined burnout as a state of exhaustion caused by prolonged overwork.
Comentários: A alternativa A está incorreta. A alternativa completa a sentença de forma correta,
já que o texto diz que o psicólogo foi quem primeiro usou a palavra burnout para descrever a
condição de exaustão descrita pelo texto.
A alternativa B está incorreta. Esta alternativa também completa de forma correta a sentença,
pois o texto afirma que ele reconheceu o problema do burnout pela primeira vez em Nova Iorque
no ano de 1974.
A alternativa C está correta. Ela não completa de forma correta pois diz que o psicólogo criou a
condição para que pudesse ficar conhecido por ter descoberto uma doença. O problema é que
ele não criou nada, a condição já existia, ele apenas observou e deu um nome a ela.
A alternativa D está incorreta. Esta alternativa diz exatamente a mesma coisa que é dita no texto.
O psicólogo definiu burnout como um estado de exaustão causado por excesso de trabalho
prolongado.
GABARITO: C

Questão 07 (EPCAR/INÉDITA) – Read the sentences and mark the correct option.
I. The WHO has finally recognised burnout as a disease.
II. More than half a million people suffered from workplace stress in 2018 in the UK.

AULA 01 – SKIMMING AND SCANNING AND OTHER TECHNIQUES. IDENTIFYING FALSE COGNATS. 96
TEACHER ANDREA BELO

III. Relying on sugar to get you through the day can be a good way of keeping yourself from
burning out.
The only correct sentence(s) is(are)
a) II.
b) III.
c) I and II.
d) I and III.
Comentários: A sentença I está incorreta. O texto afirma que a organização mundial da saúde não
reconheceu o burnout como uma doença, mas sim como um fenômeno ocupacional.
A sentença II está correta. O texto afirma que mais 595.000 pessoas sofreram com stress no local
de trabalho somente no Reino Unido no ano de 2018, ou seja, mais de meio milhão de pessoas.
A sentença III está incorreta. O texto diz que usar açúcar para conseguir passar o dia é um dos
maus hábitos que as pessoas devem estar atentas na hora de perceber se estão se aproximando
de um burnout.
Somente a sentença II está correta.
GABARITO: A

Questão 08 (EPCAR/INÉDITA) – Mark the correct alternative to complete the sentence.


According to the text, the burnout
a) makes people happy because they feel that they are performing at their best.
b) is best tackled and can be reversed by making lifestyle changes.
c) has been responsible for many deaths around the world since the 1970’s.
d) is responsible for kicks of adrenaline when you think about a project you’ve been working on.
Comentários: A alternativa A está incorreta. O texto não afirma, em momento algum, que as
pessoas estão felizes porque elas sentem que estão desempenhando sua melhor performance.
Na verdade, o texto afirma que o desempenho das pessoas piora com o burnout.
A alternativa B está correta. O texto diz que o burnout é abordado da melhor forma quando há
mudanças no estilo de vida da pessoa. Essa é a melhor forma de lidar com problema conhecida
até o momento.
A alternativa C está incorreta. O texto não fala em mortes causadas pelo problema do burnout.
A alternativa D está incorreta. O texto não diz que o burnout é responsável por picos de
adrenalina, o texto diz que é normal se sentir assim quando se está envolvido em um projeto
grande, e que só é preocupante caso os sintomas não sumam após a conclusão desse projeto.
GABARITO: B

AULA 01 – SKIMMING AND SCANNING AND OTHER TECHNIQUES. IDENTIFYING FALSE COGNATS. 97
TEACHER ANDREA BELO

Questão 09 (EPCAR/INÉDITA) – In the underlined sentences in the second paragraph, there are
____ verbs used in the past tense.
a) four
b) one
c) three
d) two
Comentários: Ao observar o trecho sublinhado, percebemos o verbo “recognised” no past simple,
também podemos observar o verbo “wasn’t” que também está no past simple. Podemos dizer,
então, que há dois verbos sendo usados no tempo passado no trecho sublinhado do segundo
parágrafo.
GABARITO: D

Questão 10 (EPCAR/INÉDITA) – Volunteers


a) never liked their work at all.
b) weren’t giving enough attention to patients due to their burnout condition.
c) accepted being part of a experiment to find out what happens when people get to their limit
at work.
d) would still find their jobs rewarding even though they were burned out.
Comentários: A alternativa A está incorreta. O texto afirma que os voluntários achavam seus
trabalhos recompensadores antes de sofrer com o burnout.
A alternativa B está correta. O texto diz que os deixaram de dar a devida atenção aos pacientes
por conta de sua condição de burnout.
A alternativa C está incorreta. O texto não fala nada sobre experimento, nem afirma que os
voluntários tinham ciência de que estavam sendo avaliados pelo psicólogo.
A alternativa D está incorreta. O texto diz que os voluntários deixaram de ter a mesma impressão
sobre o próprio trabalho uma vez que foram acometidos pelo burnout e nos permite entender
que deixaram de ver seus trabalhos como recompensadores desde então.
GABARITO: B

QUESTÕES ESA
Brazilian Army Joins Effort to Disinfect Prison from COVID-19
By VOA News
The Brazilian army, in conjunction with the country's prison administration, conducts a large-scale
disinfection operation to slow the spread of the coronavirus at the Gericinó Prison Complex in Rio
de Janeiro. A prison spokesman says soldiers outfitted in full body protection suits and face masks
clean the infirmary, coronavirus isolation cells, administrative areas, and visitor sections of the

AULA 01 – SKIMMING AND SCANNING AND OTHER TECHNIQUES. IDENTIFYING FALSE COGNATS. 98
TEACHER ANDREA BELO

prison. Colonel Rego Barros of the army's East Joint Command says the proactive measures are
taken after seeing situations in which inmates become infected by COVID-19 in other states.
So far, Brazil has more than 1,000,000 covid-19 cases and almost 50,000 deaths. Peru has the
second highest coronavirus case total in Latin American behind Brazil. Peru's National Penitentiary
Institute confirms reports of inmates setting fires and demanding to be freed, as more prisoners
become infected with the disease. Reuters news agency says human rights groups are calling on
the Peruvian government to allow house arrest during the pandemic.
(Adapted from https://www.voanews.com/covid-19-pandemic/brazilian-army-joins-effort-disinfect-prison-covid-19)

Questão 01 (ESA/INÉDITA) – Concerning the information in the text, it is correct to state that
a) house arrest is allowed in Peru during the pandemic.
b) the Brazilian army is remotely helping to prevent covid-19 spread in prisons.
c) soldiers are working on a disinfection operation in a prison in Rio de Janeiro.
d) Brazil is only behind Peru in total covid-19 cases in Latin America.
e) Brazil has a total of 1,050,000 covid-19 cases.
Comentários: Em relação às informações no texto, é correto afirmar que
a) house arrest is allowed in Peru during the pandemic.
a prisão domiciliar é permitida no Peru durante a pandemia- na verdade, o texto afirma que
grupos de direitos humanos estão pedindo ao governo peruano que permita prisão domiciliar
durante a pandemia, mas não se sabe se o pedido foi atendido. “… human rights groups are calling
on the Peruvian government to allow house arrest during the pandemic.” ERRADA.
b) the Brazilian army is remotely helping to prevent covid-19 spread in prisons.
o exército brasileiro está ajudando remotamente a impedir a disseminação do covid-19 nas
prisões- remotely significa remotamente, isto é, à distância. O texto afirma que os membros do
exército brasileiro estão trabalhando diretamente na causa, usando roupas de proteção que
cobrem todo o corpo e máscaras, limpando e desinfectando ambientes na prisão... “A prison
spokesman says soldiers outfitted in full body protection suits and face masks clean the infirmary,
coronavirus isolation cells, administrative areas, and visitor sections of the prison. ERRADA.
c) soldiers are working on a disinfection operation in a prison in Rio de Janeiro.
soldados estão trabalhando em uma operação de desinfecção em uma prisão no Rio de Janeiro-
alternativa de acordo com o texto, conforme vimos acima. “The Brazilian army... conducts a large-
scale disinfection operation to slow the spread of the coronavirus at the Gericinó Prison Complex
in Rio de Janeiro. A prison spokesman says soldiers outfitted in full body protection suits and face
masks clean the infirmary, coronavirus isolation cells, administrative areas, and visitor sections of
the prison.” CORRETA.
d) Brazil is only behind Peru in total covid-19 cases in Latin America.
O Brasil está apenas atrás do Peru no total de casos de covid-19 na América Latina- o texto afirma
que o Brasil é o primeiro país na América Latina em número total de casos, e que o Peru vem em
segundo. “So far, Brazil has more than 1,000,000 covid-19 cases and almost 50,000 deaths. Peru

AULA 01 – SKIMMING AND SCANNING AND OTHER TECHNIQUES. IDENTIFYING FALSE COGNATS. 99
TEACHER ANDREA BELO

has the second highest coronavirus case total in Latin American behind Brazil.” É o contrário do
que diz a alternativa. ERRADA.
e) Brazil has a total of 1,050,000 covid-19 cases.
O Brasil possui um total de 1.050.000 de casos de covid-19- Atenção, pegadinha: o texto afirma
que, até agora, o Brasil tem mais de 1.000.000 de casos de covid-19 e quase 50.000 mortes. Mas
você não pode somar esses números, pois os casos que levaram pessoas à morte estão dentro do
total de mais de um milhão. “So far, Brazil has more than 1,000,000 covid-19 cases and almost
50,000 deaths.” ERRADA.
GABARITO: C

Questão 02 (ESA/INÉDITA) – Which option has a correct relation of the underlined terms and
their substitutes?
a) A prison spokesman says... — Him
b) Soldiers clean the coronavirus isolation cells. — Their
c) Brazil has more than 1,000,000 covid-19 cases. — He
d) Human rights groups are calling on the Peruvian government... — Them
e) Inmates become infected by COVID-19 in other states. — They
Comentários: Qual opção tem uma relação correta dos termos sublinhados e seus substitutos?
a) A prison spokesman says... — Him
O termo sublinhado é sujeito na frase- um porta-voz da prisão diz... Assim, não poderia ser
substituído por him, que é pronome objeto. O correto seria o pronome sujeito he. ERRADA.
b) Soldiers clean the coronavirus isolation cells. — Their
Aqui, o termo sublinhado também é sujeito na frase- soldados limpam as celas de isolamento do
coronavírus. Logo, não poderia ser substituído por their, que é um adjetivo possessivo (deles,
delas). O correto seria o pronome sujeito they. ERRADA.
c) Brazil has more than 1,000,000 covid-19 cases. — He
Brasil, sendo um país, não pode ser substitído por he, que é um pronome sujeito usado
exclusivamente para pessoas do gênero masculino. O correto seria it para se referir a Brasil. It has
more than... ERRADA.
d) Human rights groups are calling on the Peruvian government... — Them
Aqui, o termo sublinhado também é sujeito na frase- grupos de direitos humanos estão pedindo
ao governo peruano... Assim, não poderia ser substituído por them, que é pronome objeto. O
correto seria o pronome sujeito they. ERRADA.
e) Inmates become infected by COVID-19 in other states. — They
O termo sublinhado é sujeito na frase- os presos são infectados pelo COVID-19 em outros estados.
Assim, está correta a relação inmates e they, uma vez que they é pronome sujeito e substitui
corretamente inmates, transmitindo a ideia de “eles, os presos”. CORRETA.

AULA 01 – SKIMMING AND SCANNING AND OTHER TECHNIQUES. IDENTIFYING FALSE COGNATS. 100
TEACHER ANDREA BELO

GABARITO: E

Questão 03 (ESA/INÉDITA) – “_________ Peru have the _________ covid-19 cases number in
Latin America?” Fill in the blanks with the correct form of the verb and the adjective.
a) Does / worst
b) Does / worse
c) Is / bad
d) Do / worst
e) Do / worse
Comentários: a) Does / worst
Peru é o sujeito da frase. É um termo que está na terceira pessoa do singular- ele, o país, Peru.
Assim, no Simple Present, o auxiliar correto para fazer uma pergunta- o Peru tem o pior número
de casos covid-19 da América Latina? - é, de fato, does. → Does Peru have the worst covid-19
cases number in Latin America? Lembre-se que, nesse caso, o verbo não fica alterado, com -s,
ficando em sua forma base, conforme o enunciado trouxe. O adjetivo, por sua vez, está no
superlativo, o que é indicado pela presença do artigo the. A forma correta para “o pior” é the
worst. Bad é um adjetivo irregular, cujo comparativo é worse e o superlativo é the worst. CORRETA.
b) Does / worse
Como vimos acima, does é o auxiliar correto para fazer a pergunta para o sujeito em terceira
pessoa do singular, mas worse não é o superlativo, e, sim, o comparativo. “E como sabíamos que
a questão queria o superlativo, teacher?” Pela presença do artigo definido the, que faz parte da
estrutura do superlativo, transmitindo a ideia de “o pior.” ERRADA.
c) Is / bad
A presença do verbo have na frase trazida pelo enunciado indica que não usaremos o verbo to be,
mas um verbo auxiliar para fazer a pergunta com have- o Peru tem o pior número de casos covid-
19 da América Latina? → Does Peru have the worst covid-19 cases number in Latin America? Para
formarmos o superlativo de bad, não podemos usar the bad. O correto é the worst. ERRADA.
d) Do / worst
Como vimos acima, does é o auxiliar correto para fazer a pergunta para o sujeito em terceira
pessoa do singular, não do. Usamos do para I, you, we e they. Quanto ao adjetivo, a forma correta
para “o pior” é, de fato, the worst, como estudamos acima. ERRADA.
e) Do / worse
Does é o auxiliar correto para fazer a pergunta para o sujeito em terceira pessoa do singular, não
do. Worse não é o superlativo, e, sim, o comparativo do adjetivo bad. ERRADA.
GABARITO: A

Questão 04 (ESA/INÉDITA) – Which sentence is grammatically correct?

AULA 01 – SKIMMING AND SCANNING AND OTHER TECHNIQUES. IDENTIFYING FALSE COGNATS. 101
TEACHER ANDREA BELO

a) She needs a new furniture.


b) He reads many books every year.
c) That men is following me.
d) I have a phone new.
e) A car is more fast than a bike.
Comentários: Essa questão abordou vários tópicos do edital da ESA, como uso de artigos, singular
e plural, ordem de adjetivos, comparativo de adjetivos, concordância verbal no Presente... Fique
atento!
a) She needs a new furniture.
Furniture significa mobília, e é um substantivo incontável. Por isso, não podemos usar o artigo
indefinido a, que transmite a ideia de uma e é usado com substantivos contáveis no singular.
Poderíamos dizer “She needs new furniture” ou “She needs some new furniture”. ERRADA.
b) He reads many books every year.
Essa frase está correta em todos os seus aspectos: concordância do verbo read com sujeito em
terceira pessoa do singular no Simple Present (he reads), uso correto do quantifier many (muitos)
para substantivo contável no plural (books) e expressão adverbial de frequência no final da frase
(every year, todo ano). CORRETA.
c) That men is following me.
Men é plural de man, significando homens. Assim, o pronome that no singular e o verbo to be no
singular estão errados para acompanhar men, no plural. O correto seria “that man is following
me” (no singular) ou “those men are following me” (no plural). ERRADA.
d) I have a phone new.
A posição do adjetivo na frase está errada. Ele deveria estar antes do substantivo: “I have a new
phone.” ERRADA.
e) A car is more fast than a bike.
No caso do adjetivo fast, o comparativo correto seria faster, não “more fast”. Só se usa “more”
antes de adjetivos longos (exemplo: more intelligent, more beautiful). O correto seria “a car is
faster than a bike.” ERRADA.
GABARITO: B

Seville Hospital uses an experimental treatment against SARS-CoV-2 with successful results,
combining lopinavir and ritonavir antivirals and interferon beta.
By Richard Mann – March 1, 2020
RIO DE JANEIRO, BRAZIL - The application of lopinavir/ritonavir, also used to fight HIV infections,
along with interferon beta, a protein that helps cells protect themselves from infection, confirmed
medical sources to EL PAÍS.

AULA 01 – SKIMMING AND SCANNING AND OTHER TECHNIQUES. IDENTIFYING FALSE COGNATS. 102
TEACHER ANDREA BELO

"It's an experimental course of treatment that has yielded positive results against other viruses,"
explains Albert Bosch, president of the Spanish Society of Virology. "One of its greatest advantages
is that it is an approved drug used in other indications, so there is no question about its safety,"
he adds.
(Adapted from https://riotimesonline.com/brazil-news/brazil/brazil-declares-state-of-emergency-for-47-cities-due-to-rains/)

Questão 05 (ESA/INÉDITA) – According to the text, it is incorrect to say that:


a) The results have been satisfactory so far.
b) They are using an approved drug against coronavirus.
c) The experimental treatment is taking place in Seville.
d) Despite the good results, there are still questions about the treatment’s safety.
e) Using this treatment against other viruses has shown positive results, according to the
president of the Spanish Society of Virology.
Comentários: A alternativa A está correta. O hospital reportou que o tratamento contra o vírus
teve sucesso. Portanto, é correto dizer que os resultados tem sido satisfatórios até agora.
A alternativa B está correta. O texto afirma que as drogas utilizadas já são aprovadas para
tratamento de outras doenças, e agora estão sendo utilizadas no tratamento contra o coronavírus.
A alternativa C está correta. O texto afirma que o tratamento experimental está sendo feito num
hospital em Sevilha, na Espanha.
A alternativa D está incorreta. A alternativa erra ao dizer que ainda há questionamentos acerca da
segurança do tratamento. Na verdade, o texto diz que não há nenhuma dúvida quanto à segurança
do tratamento, já que a droga usada já é aprovada para uso em outras indicações.
A alternativa E está correta. O texto diz que este mesmo tratamento já apresentou resultados
positivos contra outros vírus e, de fato, quem diz isso é o presidente Socieadade Espanhola de
Virologia.
GABARITO: D

QUESTÕES ESCOLA NAVAL


The current pandemic is forcing much of the world’s population to stay at home and take it
easy. But we may not be wired that way.
You may be among the more than three million people who’ve seen a short film issued by the
Governor of California’s office. It’s all over social media. In the video, the comedian Larry David,
in his trademark sardonic style, urges people to follow the official advice and stay at home to stop
the spread of Covid-19. What’s the matter with you "idiots", he says, you’re passing up a fantastic
opportunity to sit in an armchair and watch TV all day!
We’re used to health warnings that urge us to do things that we don’t really have a great urge to
do: to exercise more, to eat five or eight or even 10 portions of fruit and vegetables a day. But for

AULA 01 – SKIMMING AND SCANNING AND OTHER TECHNIQUES. IDENTIFYING FALSE COGNATS. 103
TEACHER ANDREA BELO

once the official advice sounds easy; loaf on the sofa, binge-watch box sets, stay at home. This all
sounds as though it should appeal to our lazier sides.
In fact, it is not as simple as that, as you’ve probably already found out, after a few weeks of
lockdown. It turns out we are not biologically programmed to do as little as possible. Indeed, we
thrive on activity. Or at least, a good balance between being busy and being able to rest.
It’s true that we often look for the easy option, the path of least resistance, the shortcut to
success. If you have a remote control, why get up and switch channels on the TV itself? If you have
a car why cycle to the supermarket? If you can get away with doing half as much work than a
colleague, then why not?
Any sort of work or effort involves mental and physical strain, so it makes sense to avoid it where
possible. And sometimes we do just that. This is sometimes known as the principle of least effort
or Zipf’s Law, a law you might think no one is ever tempted to break. Except that we break it all
the time.
Do you ever dream of doing absolutely nothing? Lying in a hammock for a whole afternoon. Just
staring at the ceiling, listening to the silence. It may sound like a lovely idea, but in fact we can
find doing nothing at all – and take sleep out of the equation – very hard to do. In a famous study
conducted a few years ago at the University of Virginia, participants were led one at a time into a
completely bare room with all distractions removed. They had no phone, no books, no screens –
and they weren’t allowed to take a nap. Electrodes were fitted to their ankles and they were left
alone for 15 minutes. It was an opportunity to kick back and relax for a short while.
So, how did it go? Well, before being left alone, participants were shown how to press a computer
key which was wired up to a machine that delivered an electric shock. You might suppose that
having tried it once no one would want to do it again. Wrong. In fact, 71% of the men and 25% of
the women gave themselves at least one electric shock during their time in solitary – and one man
shocked himself a shocking 190 times.
It turns out that having nothing to do was so excruciating, that many of the participants preferred
to, in effect, torture themselves rather than put up with no distractions whatsoever.
This experiment is an extreme example, but we know from everyday life that people constantly
choose to do things they don’t need to do and which are sometimes painful. Think of all of your
friends who run marathons or have punishing regimes at the gym.
They go way beyond what is required for health and fitness. And what about the people who trek
across the ice to reach the poles of the Earth or sail around the world?
Michael Inzlicht from the University of Toronto calls this the paradox of effort. Sometimes we take
the easy route and do as little as we can get away with, but at other times we value situations
more if we have to expend considerable effort. The intrinsic joy of the effort gives us so much
pleasure that we don’t take the short cut. We might spend hours puzzling over a cryptic crossword
instead of using a search engine to find the solution.
We learn this early in life. As children we are taught through experience and persuasion that effort
leads to reward and over time this conditions us to enjoy effort for its own sake. This is known as
learned industriousness.

AULA 01 – SKIMMING AND SCANNING AND OTHER TECHNIQUES. IDENTIFYING FALSE COGNATS. 104
TEACHER ANDREA BELO

All of this means, that while we stay home and self-isolate, lying on the sofa and watching TV will
form only a part of how we pass the time. We might think it is fun to laze around for a few weeks,
but in fact it will drive us to distraction. Enforced and extended rest, unless we are ill and our
bodies demand it, leads not to feelings of being relaxed but of restlessness and irritability.
We need to find ways during lockdown to replicate as far as we can the rhythms and sense of
balance we achieve, at our best, in ordinary life.
So, taking exercise, setting ourselves tasks, doing things that are effortful and difficult are
important. And we should all be looking for activities or experiences that promote what the
psychologist Mihaly Csikszentmihalyi calls flow, in his book Flow: The Psychology of Optimal
Experience. These are tasks, such as painting or gardening or doing jigsaws, which so absorb us
that we don’t notice time passing and we stop worrying about everything else.
In normal times, most of us don’t take rest seriously enough. So during this exceptional period,
we should embrace the opportunity to rest more if we can – and indeed take those more balanced
rhythms of rest and busyness into our lives post shutdown. But during this difficult time, we will
find that we are not instinctively lazy creatures.
Indeed in a weird way we might find that doing less, and resting more, initially requires quite a lot
of effort.
(Adapted from: https://www.bbc.com/future/article/20200602-are-human-beings-naturally-lazy)

Questão 01 (ESCOLA NAVAL/INÉDITA) – According to the text, which option completes the
sentence below correctly?
In normal times, ______.
(A) everybody wants to rest all the time
(B) we try to rest as much as we can
(C) nobody rests properly
(D) the majority of people don’t rest properly
(E) a few people don’t rest as much as they need
Comentários: A alternativa A está incorreta. O texto diz que a maioria das pessoas não leva o
descanso a sério, e não que todo mundo quer descansar o tempo todo.
A alternativa B está incorreta. O texto diz que a maioria das pessoas não leva o descanso a sério,
e não que nós tentamos decansar o máximo que podemos.
A alternativa C está incorreta. O texto diz que a maioria das pessoas não leva o descanso a sério,
e não que ninguém descansa de forma apropriada.
A alternativa D está correta. O texto diz o mesmo que a alternativa, que a maioria das pessoas
não descansa de forma apropriada em tempos de normalidade.
A alternativa E está incorreta. O texto diz que a maioria das pessoas não leva o descanso a sério,
e não que poucas pessoas não descansam o quanto deveriam descansar em tempos normais.
GABARITO: D

AULA 01 – SKIMMING AND SCANNING AND OTHER TECHNIQUES. IDENTIFYING FALSE COGNATS. 105
TEACHER ANDREA BELO

Questão 02 (ESCOLA NAVAL/INÉDITA) – What's the meaning of the word "sardonic" in


paragraph 1?
(A) Funny.
(B) Comic.
(C) Sarcastic.
(D) Iconic.
(E) Casual.
Comentários: A alternativa A está incorreta. “Sardonic” significa algo sardônico, irônico ou
sarcástico, enquanto “funny” significa engraçado. Algo irônico ou sarcástico pode ser engraçado,
mas não necessariamente.
A alternativa B está incorreta. “Sardonic” significa algo sardônico, irônico ou sarcástico, enquanto
“comic” significa cômico. Algo irônico ou sarcástico pode ser cômico, mas não necessariamente.
A alternativa C está correta. “Sardonic” significa algo sardônico, irônico ou sarcástico, exatamente
o significado de “sarcastic” que significa sarcástico. Por isso, essa é a alternativa correta.
A alternativa D está incorreta. “Sardonic” significa algo sardônico, irônico ou sarcástico, enquanto
“iconic” significa icônico. Não há, portanto, relação de sentido entre as palavras.
A alternativa E está incorreta. “Sardonic” significa algo sardônico, irônico ou sarcástico, enquanto
“casual” significa casual. Não há, portanto, relação de sentido entre as expressões.
GABARITO: C

Questão 03 (ESCOLA NAVAL/INÉDITA) – According to the text, which option is correct?


(A) Most people find it difficult to stay home doing nothing for a long period of time.
(B) After a few weeks of lockdown, we realize that we are programmed to do as little as
possible.
(C) Most people are happy to stay at home during this pandemic.
(D) The pandemic is allowing people to find a good balance between being busy and being able
to rest.
(E) We always follow the principle of least effort while doing any activity in our lives.
Comentários: A alternativa A está correta. O texto diz que nós temos dificuldade em ficar em casa
sem fazer nada por um período prolongado.
A alternativa B está incorreta. Não se pode dizer somos programados para fazer o mínimo
possível, pois o texto afirma que nós quebramos a lei do menor esforço o tempo todo.
A alternativa C está incorreta. O texto fala da dificuldade que a maioria das pessoas enfrentam ao
ficar em casa sem poder fazer nada por um período prolongado. O texto não nos permite
entender que a maioria das pessoas está feliz por ficar em casa de maneira forçada durante essa
pandemia.

AULA 01 – SKIMMING AND SCANNING AND OTHER TECHNIQUES. IDENTIFYING FALSE COGNATS. 106
TEACHER ANDREA BELO

A alternativa D está incorreta. Não podemos dizer que a pandemia está nos permitindo encontrar
um equilíbrio entre atividade e descanso. O texto diz que antes da pandemia, as pessoas não
descansavam de maneira adequada, e agora têm de ficar em casa de maneira forçada e
prolongada. Não há equilíbrio nisso.
A alternativa E está incorreta. O texto diz o contrário da alternativa, que estamos o tempo todo
contradizendo e quebrando o princípio do menor esforço.
GABARITO: A

Questão 04 (ESCOLA NAVAL/INÉDITA) – According to the text, which option is correct?


(A) Doing nothing for a while just staring at the ceiling and listening to the silence is proven to be
a great experience to any person.
(B) The paradox of effort consists in people always choosing the easiest path to do things.
(C) Sometimes we avoid any kind of effort at all costs, and sometimes we choose the harder way
of doing things.
(D) People will always choose to cycle to the supermarket because it’s healthier.
(E) We have always lived in good balance between being busy and being able to rest.
Comentários: A alternativa A está incorreta. O texto diz que essa experiência de ficar sem fazer
nada simplesmente olhando para o teto e escutando o silêncio não é uma experiência tão boa
quanto parece.
A alternativa B está incorreta. O texto diz que o paradoxo do esforço consiste no fato de que, às
vezes, nós tomamos a rota mais curta e fácil para fazer o mínimo esforço, mas outras vezes, nós
valorizamos mais determinadas situações se tivermos que fazer um considerável esforço. A
alternativa diz que o paradoxo consiste no fato de que as pessoas sempre escolhem o caminho
mais fácil para fazer as coisas.
A alternativa C está correta. O texto diz que, por vezes, escolhemos poupar todo o esforço possível
ao fazer algo e, por vezes, escolhemos o caminho mais difícil para fazer algo, exatamente o mesmo
que diz a alternativa.
A alternativa D está incorreta. O texto diz que não há por que pedalar até o mercado se você tem
um carro, e não que é melhor ir pedalando porque faz bem para a saúde.
A alternativa E está incorreta. O texto não diz que sempre vivemos em um bom equilíbrio entre
períodos de atividade e descanso, ele diz o contrário disso.
GABARITO: C

Questão 05 (ESCOLA NAVAL/INÉDITA) – In paragraph 4, the word "that" refers to


(A) People making unnecessary effort.
(B) Cycle to the supermarket.
(C) Get up to switch channels on the TV itself.

AULA 01 – SKIMMING AND SCANNING AND OTHER TECHNIQUES. IDENTIFYING FALSE COGNATS. 107
TEACHER ANDREA BELO

(D) Mental and physical strain.


(E) Avoid it.
Comentários: A alternativa A está incorreta. Não se pode dizer que “that” se refere a pessoas
fazendo um esforço desnecessário, pois o trecho que o antecede não fala sobre isso, mas sim
explica que faz sentido evitar trabalhos ou esforços que envolvam tensão física ou mental.
A alternativa B está incorreta. Pedalar até o supermercado está num trecho muito anterior ao
pronome citado. O pronome se refere ao trecho imediatamente anterior a ele.
A alternativa C está incorreta. Levantar-se para trocar os canais na própria televisão está num
trecho muito anterior ao pronome citado. O pronome se refere ao trecho imediatamente anterior
a ele.
A alternativa D está incorreta. Essa alternativa poderia te confundir, pois tensão física e mental
está relacionada ao pronome, mas o pronome “that” retoma, na verdade, o trecho seguinte, que
diz que faz sentido evitar isso (a tensão física e mental).
A alternativa E está correta. “That” retoma o trecho que diz que faz sentido evitar isso quando
possível. Quando o texto diz que às vezes fazemos isso, ele fala sobre evitar isso. E “isso” se refere
à tensão física e mental.
GABARITO: E

Questão 06 (ESCOLA NAVAL/INÉDITA) – According to the text which option is correct?


(A) People found out they are lazy because they are enjoying staying at home and doing nothing
for weeks now.
(B) We are never tempted to break Zipf’s law. We only break it when we need to.
(C) A study conducted at University of Virginia showed that many people can’t stand inactivity.
(D) A study conducted at University of Virginia showed we all enjoy having 15 minutes to relax.
(E) Michael Inzlicht was the person responsible for conducting the famous study at the
University of Virginia.
Comentários: A alternativa A está incorreta. O texto diz que nós não somos naturalmente
preguiçosos e que many people are not enjoying staying at home during this pandemic.
A alternativa B está incorreta. O erro da alternativa está em dizer que nós nunca nos sentimos
tentados a quebrar a lei do menor esforço, porque o texto diz que nós quebramos essa lei o tempo
todo.
A alternativa C está correta. O texto diz que 71% dos homens e 25% das mulheres se aplicaram
choques durante os 15 minutos de experiência em que deveriam permanecer no silêncio e sem
distrações.
A alternativa D está incorreta. O texto diz que muitas pessoas não suportaram 15 minutos de
inatividade sem distrações e se aplicaram choques elétricos durante esses 15 minutos. Por isso,
não se pode dizer que todos aproveitaram seus 15 minutos para relaxar.

AULA 01 – SKIMMING AND SCANNING AND OTHER TECHNIQUES. IDENTIFYING FALSE COGNATS. 108
TEACHER ANDREA BELO

A alternativa E está incorreta. O texto não diz que Michael Inzlicht conduziu o estudo da
Universidade de Virginia. O texto diz que ele é da Universidade de Toronto e chamou a situação
ocorrida no estudo de paradoxo do esforço.
GABARITO: C

Questão 07 (ESCOLA NAVAL/INÉDITA) – Which is the correct option to complete the paragraph
below?
Social Integration
Social integration is ______ actual participation in various social relationships, ranging ______
romantic partnerships to friendships.
This integration involves emotions, intimacy, and a sense of belonging ______ different social
groups, such as being part of a family, a partnership, a social activity, or a religious community.
Experts suggest that ______ integrated into such social relationships confers a protective benefit
against maladaptive behaviors and damaging health consequences.
(Adapted from https://www.verywellmind.com/social-support-for-psychological-health-4119970)

(A) the / from / to / being


(B) an / from / at / being
(C) the / to / to / be
(D) the / from / to / be
(E) a / to / at / being
Comentários: A primeira lacuna deve ser preenchida por “the”. O texto está falando da definição
de integração social e, assim, faz mais sentido que se use um artigo definido para definir alguma
coisa.
A segunda lacuna deve ser preenchida por “from”. Nesse caso, trata-se da regência de “range”.
Something ranges from this to that. Então, as preposições “from” e “to” fazem parte da regência
de “range”.
A terceira lacuna deve ser preenchida por “to”. Mais uma vez, temos uma lacuna que trata de
regência. Agora de “belong”. Who belongs, belongs to...”. Por isso, a necessidade do uso da
preposição “to”.
A quarta lacuna deve ser preenchida por “being”. O uso de “that” imediatamente antes do verbo
exige que o verbo seja escrito no gerúndio.
Temos a sequência: the / from / to / being.
GABARITO: A

AULA 01 – SKIMMING AND SCANNING AND OTHER TECHNIQUES. IDENTIFYING FALSE COGNATS. 109
TEACHER ANDREA BELO

Questão 08 (ESCOLA NAVAL/INÉDITA) – Which option completes the dialogue below correctly?
John: What's the problem? Mary: My son hasn’t taken a shower yet. He must _______ today.
(A) have done it
(B) have it done
(C) done it
(D) has done it
(E) has it done
Comentários: A Alternativa A está incorreta. “I must have done it” não é usado nessa situação,
pois o contexto nos mostra que ele ainda não tomou banho, mas precisa tomar ainda hoje. Essa
alternativa sugere que ele já tomou banho, o que contradiria a frase imediatamente anterior a
essa.
A alternativa B está correta. Essa é a forma de se expressar que algo precisa ser feito com prazo
estipulado. O prazo estipulado é today, e isso quer dizer que ele precisa estar com seu banho
tomado ainda hoje. Traduzindo, temos: Eu tenho que ter isso feito hoje.
A alternativa C está incorreta. O erro dessa alternativa é usar o verbo no particípio. Assim, seria
como dizer: Eu tenho que feito isso hoje. Não faria sentido, não é verdade?
A alternativa D está incorreta. O erro dessa alternativa é usar o auxiliar “has” com o verbo modal
“must”. Esse modal só aceita o auxiliar “have” no passado. Além desse erro, ainda temos o fato
de que essa alternativa sugeriria que ele já tomou banho, o que contradiria a frase imediatamente
anterior a essa.
A alternativa E está incorreta. O erro dessa alternativa é usar o auxiliar “has” com o verbo modal
“must”. Esse modal só aceita o auxiliar “have” no passado.
GABARITO: B

Questão 09 (ESCOLA NAVAL/INÉDITA) – Which option completes the paragraph below


correctly?
Latin America's largest country is testing people ______ a rate far lower than any other nation
with at least 40,000 cases. It tests 12 times fewer people ______ Iran, and 32 times fewer ______
the United States. Hospitalized patients aren't being tested. Some medical professionals aren't
being tested. People are dying ______ their homes without being tested.
(Adapted from https://www.washingtonpost.com/world/the_americas/coronavirus-brazil-testing-bolsonaro-cemetery-gravedigger)

(A) at / that / that / on


(B) with/ that / that / on
(C) at / than / than / in
(D) with/ than / that / in
(E) at / than / than / on

AULA 01 – SKIMMING AND SCANNING AND OTHER TECHNIQUES. IDENTIFYING FALSE COGNATS. 110
TEACHER ANDREA BELO

Comentários: A primeira lacuna deve ser preenchida por “at”. Não se pode usar “with” porque
quando usamos a palavra “rate”, se diz “at a rate” ou “at this rate”, sempre com a preposição
“at”.
A segunda lacuna deve ser preenchida por “than”. A palavra “than” faz parte da estrutura do
comparativo que está sendo usado na frase, dizendo que o país testa 12 vezes menos pessoas que
o Iran.
A terceira lacuna deve ser preenchida por “than”. A palavra “than” faz parte da estrutura do
comparativo que está sendo usado na frase, dizendo que o país testa 32 vezes menos pessoas que
os Estados Unidos.
A quarta lacuna deve ser preenchida por “in”. Trata-se de uma regra. Não se fala “on their homes”,
mas sim “in their homes”, pois a preposição “in” dá a ideia de interior da casa.
Temos a sequência: at / than / than / in.
GABARITO: C

Questão 10 (ESCOLA NAVAL/INÉDITA) – Which word best completes the question below?
How ______ does he go for a run?
It’s really important to exercise in order to keep healthy.
(A) far
(B) old
(C) high
(D) often
(E) many
Comentários: A alternativa A está incorreta. “How far” é uma estrutura usada para perguntar
sobre distância, e não sobre com que frequência se faz algo.
A alternativa B está incorreta. “How old” é uma estrutura usada para perguntar sobre idade, e
não sobre com que frequência se faz algo.
A alternativa C está incorreta. “How high” é uma estrutura usada para perguntar sobre altura de
algo, e não sobre com que frequência se faz algo.
A alternativa D está correta. “How often” é exatamente a estrutura que se utiliza para perguntar
com que frequência se faz algo. Nesse caso, com que frequência ele sai para correr?
A alternativa E está incorreta. “How many” é uma estrutura que se utiliza para perguntar sobre
quantidade, e não sobre frequência.
GABARITO: D

QUESTÕES EsPCEx
Leia o texto a seguir e responda às questões 01, 02 e 03.

AULA 01 – SKIMMING AND SCANNING AND OTHER TECHNIQUES. IDENTIFYING FALSE COGNATS. 111
TEACHER ANDREA BELO

What Happened When Hong Kong’s Schools Went Virtual to Combat the Spread of
Coronavirus
Because of the coronavirus outbreak, all schools in Hong Kong have been closed since January,
and won’t reopen until late April at the earliest. "The exact date of class resumption is subject to
further assessment," announced the Education Bureau, which controls all schools in Hong Kong,
public and private, on February 25. It’s all part of the “social distancing” measures the city has
mandated to slow the virus’s spread, which include closing libraries, museums and recreation
facilities like pools. Students from preschoolers through PhD candidates are now doing all their
education online, a move the Education Bureau calls "suspending classes without suspending
learning."
As coronavirus spreads across the globe, other countries are joining Hong Kong and mainland
China in this massive, unplanned experiment in online learning. According to Unesco, as of Friday,
14 countries have shut schools down nationwide, affecting upwards of 290 million students, while
13 countries, including the United States, have seen localized school closings.
In recent days, schools from Scarsdale, New York, to San Francisco have closed temporarily over
contagion concerns. The University of Washington and Stanford University have turned to online
classes for the remainder of the quarter, and others are following suit for various lengths of time.
Some experts believe more widespread and long-term closures will be necessary in areas with
high levels of community transmission. States are preparing for that possibility by looking at their
own online learning policies.
Adapted from https://www.smithsonianmag.com/innovation/what-can-americans-learn-from-hong-kongs-unplanned-experiment-online-learning-during-coronavirus-180974331/

Questão 01 (EsPCEx/INÉDITA) – What is the main topic of the text?


a) Schools in Hong Kong closed for good because of coronavirus.
b) The changes schools had to implement in education due to coronavirus.
c) School closures in Hong Kong thanks to lack of students.
d) Students in many countries can’t be home schooled.
e) How schools are educating students about coronavirus.
Comentários: a) De acordo com a letra A, o main topic do texto (assunto principal) é que as escolas
em Hong Kong fecharam definitivamente (for good significa para sempre, definitivamente) por
causa do coronavírus. Segundo o texto, por causa do surto de coronavírus, todas as escolas em
Hong Kong estão fechadas desde janeiro e só serão reabertas até o final de abril. Ou seja, o
fechamento é temporário. “Because of the coronavirus outbreak, all schools in Hong Kong have
been closed since January, and won’t reopen until late April at the earliest.” (1º parágrafo).
Alternativa INCORRETA.
b) Segundo a letra B, o main topic do texto são as mudanças que as escolas tiveram que
implementar na educação devido ao coronavírus. De fato, o texto fala sobre como as escolas
tiveram que se adaptar diante do surto de coronavírus, fechando seus espaços físicos e
implementando o ensino online, como alternativa a suspender as aulas. “As coronavirus spreads
across the globe, other countries are joining Hong Kong and mainland China in this massive,
unplanned experiment in online learning.” (2º parágrafo). Alternativa CORRETA.

AULA 01 – SKIMMING AND SCANNING AND OTHER TECHNIQUES. IDENTIFYING FALSE COGNATS. 112
TEACHER ANDREA BELO

c) Conforme a letra C, o main topic do texto é o fechamento de escolas em Hong Kong devido à
falta de alunos. Como vimos, as escolas não fecharam por falta de alunos, mas devido ao surto de
coronavírus. “Because of the coronavirus outbreak, all schools in Hong Kong have been closed
since January, and won’t reopen until late April at the earliest.” (1º parágrafo). Alternativa
INCORRETA.
d) Para a letra D, o main topic do texto é que alunos de muitos países não podem ter aulas em
casa (to be home schooled significa ser ensinado em casa, ter aulas em casa. Conforme vimos, o
assunto do texto é justamente o fato de que muitas escolas ao redor do mundo estão adotando o
ensino online por causa do surto de coronavírus. Alternativa INCORRETA.
e) A letra E afirma que o main topic do texto é como as escolas estão educando os alunos sobre o
coronavírus. O texto não trata desse assunto. Como vimos, o texto fala da necessidade de se
adotar o ensino à distância por causa do surto da doença. Alternativa INCORRETA.
GABARITO: B

Questão 02 (EsPCEx/INÉDITA) – In the sentence “The exact date of class resumption is subject to
further assessment” (paragraph 1), the word further expresses
a) addition.
b) distance.
c) contrast.
d) time.
e) conclusion.
Comentários: A questão quer que você identifique qual ideia a palavra further expressa na frase.
Os dois sentidos mais comuns de further são: mais longe, mais distante (comparativo de far) e
equivalendo a more (mais). No contexto, o sentido dessa palavra é de adição, pois o sentido da
frase é de que a data exata do reinício da aula está sujeita a avaliação adicional, a mais avaliação.
a) Addition significa adição, e é a ideia que further expressa na frase. Alternativa CORRETA.
b) Further pode transmitir a ideia de mais distante, mais longe, mas, no contexto trazido pela
questão, não tem esse sentido, não indica distância. Alternativa INCORRETA.
c) Further não indica contraste na frase trazida pela questão. Alternativa INCORRETA.
d) Further não indica tempo na frase trazida pela questão. Alternativa INCORRETA.
e) Como vimos, further não indica uma conclusão na frase trazida pela questão. Alternativa
INCORRETA.
GABARITO: A

Questão 03 (EsPCEx/INÉDITA) – In the sentence “According to Unesco, as of Friday, 14 countries


have shut schools down nationwide” (paragraph 2), the phrasal verb shut down means
a) demolish.

AULA 01 – SKIMMING AND SCANNING AND OTHER TECHNIQUES. IDENTIFYING FALSE COGNATS. 113
TEACHER ANDREA BELO

b) decrease.
c) lower.
d) disconnect.
e) close.
Comentários: Na frase trazida pela questão, a ideia é que, segundo a Unesco, na sexta-feira, 14
países fecharam as escolas em todo o país. O phrasal verb shut down transmite a ideia, no
contexto, de fechar um estabelecimento, encerrando o seu funcionamento. Vamos às alternativas:
a) Demolish significa demolir. Como vimos, as escolas não foram demolidas, mas sim fechadas.
Alternativa INCORRETA.
b) Decrease significa diminuir. Não é esse o sentido de shut down na frase trazida pela questão.
Alternativa INCORRETA.
c) Lower significa abaixar. Também não é esse o sentido de shut down na frase trazida pela
questão. Alternativa INCORRETA.
d) Shut down pode significar desligar, desconectar, porém, como vimos, não é esse o sentido na
frase trazida pela questão. Alternativa INCORRETA.
e) Close significa fechar, correspondendo à ideia de shut down no contexto. Alternativa CORRETA.
GABARITO: E

Leia o texto a seguir e responda às questões 04, 05 e 06.


The Bod Pod
It’s a small, yet ordinary office space tucked around the corner of a lobby. It’s exactly how you
would think a medical building would look, but in this office, there is a piece of equipment that
looks similar to a space escape pod from a sci-fi movie. Despite its looks, its purpose has nothing
to do with space. The Body Composition Machine, or commonly referred to as the “Bod Pod”, is
an instrument that helps determine total body fat percentage, lean muscle mass as well as resting
metabolic rate and total energy expenditure of an individual.
“It’s state of the art,” said Alan Muriera, 460th Medical Group health promotion manager. “The
Bod Pod results are around 99.5% accurate. It is tested and calibrated everyday. We check the
seals, we check the airway valves, and we check the volume.” Muriera works in the Human
Performance Center on Buckley Air Force Base, Colo., where he administers the Bod Pod
examinations. The Bod Pod is a computerized, egg-shaped chamber which uses the same whole-
body measurement principle as underwater weighing, except it utilizes air to measure your total
body fat. As air is pumped into the chamber, body mass is calculated by the amount of air your
body displaces.
“When people come here, they are either looking to lose weight, or to see muscle gain,” said
Muriera. The Bod Pod is an opportunity for you to acquire baseline data prior to initiating a new
workout or diet regimen. The analysis takes only five minutes to accomplish and is non-invasive.
Muriera explains, “I also like to call it the “Truth Booth”, because I’ve had people come in thinking
they have 15% total body fat, and they are actually 26%.” This machine is accurate and produces

AULA 01 – SKIMMING AND SCANNING AND OTHER TECHNIQUES. IDENTIFYING FALSE COGNATS. 114
TEACHER ANDREA BELO

results with a technologically advanced system that will show minute details. “It was definitely a
humbling experience,” said Chief Master Sgt. Robert Devall, 460th Space Wing command chief. “I
don’t think I’m in bad shape, but I wasn’t quite where I wanted to be after seeing the results.”
Adapted from https://www.buckley.af.mil/News/Article-Display/Article/2105959/the-bod-pod/

Questão 04 (EsPCEx/INÉDITA) – According to the text, read the statements and choose the
correct alternative.
I – The Bod Pod body measurement is somewhat similar to underwater weighing.
II – The text is about a space escape pod.
III – The Bod Pod is a machine that burns body fat.
IV – The Bod Pod can be helpful for people who want to get in shape.
V – Chief Devall is fully satisfied with his Bod Pod examination results.
VI – The Bod Pod can determine the number of calories burned by the individual.
a) I, IV and V are correct.
b) III, V and VI are correct
c) I, II and IV are correct.
d) I, IV and VI are correct.
e) II, III and V are correct.
Comentários: I – De fato, a medição do corpo feita pelo Bod Pod guarda certa similaridade
(somewhat similar= similar em certa medida) quanto à pesagem subaquática, com a diferença que
utiliza ar para medir a gordura corporal total. “The Bod Pod is a computerized, egg-shaped
chamber which uses the same whole-body measurement principle as underwater weighing,
except it utilizes air to measure your total body fat.” (2º parágrafo). Afirmativa VERDADEIRA.
II – O texto não é sobre um dispositivo de fuga espacial. O Bod Pod apenas se parece com um
dispositivo desse tipo. Apesar de sua aparência (despite is looks), seu objetivo não tem nada a ver
com o espaço. “...there is a piece of equipment that looks similar to a space escape pod from a sci-
fi movie. Despite its looks, its purpose has nothing to do with space.” (1º parágrafo). Afirmativa
FALSA.
III – O Bod Pod não é uma máquina que queima gordura. Ele é um dispositivo que utiliza ar para
medir a gordura corporal total, e pode, assim, ajudar quem deseja se planejar para perder peso.
“... it utilizes air to measure your total body fat.” (2º parágrafo). “The Bod Pod is an opportunity
for you to acquire baseline data prior to initiating a new workout or diet regimen.” (3º parágrafo).
Afirmativa FALSA.
IV – De fato, conforme vimos acima, o Bod Pod pode ser útil para pessoas que desejam entrar em
forma. “The Bod Pod is an opportunity for you to acquire baseline data prior to initiating a new
workout or diet regimen.” (3º parágrafo). → O Bod Pod é uma oportunidade para você adquirir
dados de referência antes de iniciar um novo treino ou dieta. (3º parágrafo). Afirmativa
VERDADEIRA.

AULA 01 – SKIMMING AND SCANNING AND OTHER TECHNIQUES. IDENTIFYING FALSE COGNATS. 115
TEACHER ANDREA BELO

V – O comandante Duvall não está totalmente satisfeito com os resultados que obteve no Bod
Pod. Ele diz que foi definitivamente uma lição de humildade e que não acha que está em má forma,
mas não está exatamente onde queria estar depois de ver os resultados. “‘It was definitely a
humbling experience,’ said Chief Master Sgt. Robert Devall, 460th Space Wing command chief. ‘I
don’t think I’m in bad shape, but I wasn’t quite where I wanted to be after seeing the results.’” (3º
parágrafo). Afirmativa FALSA.
VI – O Bod Pod pode, sim, determinar o número de calorias queimadas pelo indivíduo. ... is an
instrument that helps determine total body fat percentage, lean muscle mass as well as resting
metabolic rate and total energy expenditure (gasto total de energia) of an individual. (1º
parágrafo). Afirmativa VERDADEIRA.
d) I, IV e VI são corretas.
GABARITO: D

Questão 05 (EsPCEx/INÉDITA) – the sentence “Despite its looks, its purpose has nothing to do
with space.” (paragraph 1), the word despite can be replaced by
a) in order to
b) in addition to
c) prior to
d) regardless of
e) as a result of
Comentários: Despite significa apesar de. Apesar de sua aparência (despite is looks), seu objetivo
não tem nada a ver com o espaço. Vejamos, nas alternativas, uma outra opção que transmita o
mesmo sentido.
a) in order to= a fim de, para. Alternativa INCORRETA.
b) in addition to= além de. Alternativa INCORRETA.
c) prior to= anteriormente a, antes de. Alternativa INCORRETA.
d) regardless of= apesar de, não obstante. Alternativa CORRETA.
e) as a result of= como resultado de. Alternativa INCORRETA.
GABARITO: D

Questão 06 (EsPCEx/2020) - In the sentence “‘It’s state of the art,’ said Alan Muriera, 460th
Medical Group health promotion manager.” (paragraph 2) the expression state of the art refers
to
a) something creative.
b) something old-fashioned.
c) something modern.
d) something handmade.

AULA 01 – SKIMMING AND SCANNING AND OTHER TECHNIQUES. IDENTIFYING FALSE COGNATS. 116
TEACHER ANDREA BELO

e) something artistic.
Comentários: A expressão state of the art é usada para nos referirmos a algo moderníssimo,
inovador, de ponta, de vanguarda, de última geração. Atenção, pois apesar de, literalmente,
significar estado da arte, não se refere à arte propriamente. No texto, a expressão foi usada para
descrever o moderno dispositivo que mede a gordura corporal, o Bod Pod.
a) O sentido correto da expressão state of the art não é criativo. Ela se refere a algo de ponta, de
última geração, moderno. Alternativa INCORRETA.
b) Old-fashioned significa antiquado, à moda antiga. Como vimos, não é esse o sentido correto da
expressão state of the art. Alternativa INCORRETA.
c) A expressão state of the art é usada para nos referirmos a algo muito moderno- something
modern. Alternativa CORRETA.
d) Handmade significa feito à mão. Como vimos, não é esse o sentido correto da expressão state
of the art. Alternativa INCORRETA.
e) A letra E era a grande pegadinha dessa questão. A expressão state of the art não se refere
diretamente a algo artístico, mas, sim, a algo moderno, de primeira geração, de ponta. Alternativa
INCORRETA.
GABARITO: C

Leia o texto a seguir e responda às questões 07, 08 e 09.


The US could see more deaths than WWII. It's time to deploy the military to hospitals
We are at war against the coronavirus. If we don’t act rapidly, we’re approaching a US death toll
larger than that of the two world wars combined. To fight an invisible army, we need to be creative.
We must deploy the full might of the US military – in an unconventional way. Unlike most wars,
this time the frontlines are staffed by our nation’s healthcare workers. And while they’re fighting
valiantly so far, the battle to flatten the curve is leaving them overrun.
In the US, we don’t have a centralized economy. Most of our major cities will have their hospital
systems overwhelmed simultaneously. And there will be few doctors to spare. So, what should we
do? To start, we should add medical students and retired doctors to the frontlines. But these
groups will fill only a small fraction of the added medical demand. We must rapidly train and
deploy a much larger workforce.
Let’s use one of America’s most valuable national assets – our top-tier military – to dramatically
increase our healthcare capacity. Members are trained to handle acute stress and are rapidly
deployable. They’re used to being under centralized command and are mentally prepared to be
away from their families. We should rapidly reassign any military doctors and paramedics that can
be spared to serve domestic needs. We don’t need to militarize our hospitals. But our hospitals
will need help from our military. This is an unprecedented crisis, and we need unprecedented
adaptability to save lives.
Adapted from https://www.theguardian.com/commentisfree/2020/mar/19/us-military-coronavirus-hospitals

AULA 01 – SKIMMING AND SCANNING AND OTHER TECHNIQUES. IDENTIFYING FALSE COGNATS. 117
TEACHER ANDREA BELO

Questão 07 (EsPCEx/INÉDITA) – Choose the alternative with the correct reference for the
underlined words from the text.
a) they (paragraph 1) = the frontlines.
b) them (paragraph 1) = our nation.
c) their (paragraph 2) = hospital systems.
d) their (paragraph 3) = families.
e) that (paragraph 3) = military doctors and paramedics.
Comentários: Vamos identificar as expressões às quais os termos sublinhados se referem.
And while they’re fighting valiantly so far, the battle to flatten the curve is leaving them overrun.
(1º parágrafo)
they e them – our nation’s healthcare workers.
Most of our major cities will have their hospital systems overwhelmed simultaneously. (2º
parágrafo)
their – most of our major cities.
They’re used to being under centralized command and are mentally prepared to be away from
their families. We should rapidly reassign any military doctors and paramedics that can be spared
to serve domestic needs. (3º parágrafo)
their – (military) members.
that – military doctors and paramedics.
Assim, a alternativa que trouxe a referência correta foi a letra E- that (paragraph 3) = military
doctors and paramedics.
GABARITO: E

Questão 08 (EsPCEx/INÉDITA) – According to the text, the act of “deploying” could be defined as
a) simulating a movement of troops during a health crisis.
b) coming back home after a mission in another country.
c) relocating military personnel to a specific destination.
d) rescuing civilians threatened by war from conflict zones.
e) hiring medical students to work at hospitals abroad.
Comentários: Deployment é um termo frequente no universo militar, que já foi cobrado
anteriormente na prova da EsPCEx. Ele se refere ao deslocamento de pessoal e materiais militares
para um destino específico onde sejam necessários. É o envio de tropas e recursos militares para
que estejam prontos para uma ação rápida. No texto, o ato de “deploying” se refere, mais

AULA 01 – SKIMMING AND SCANNING AND OTHER TECHNIQUES. IDENTIFYING FALSE COGNATS. 118
TEACHER ANDREA BELO

especificamente, a realocar médicos e paramédicos militares para os hospitais para ajudar no


tratamento das pessoas afetadas pelo coronavírus.
a) No texto, o ato de “deploying” não se refere a simular um movimento de tropas durante uma
crise de saúde. Alternativa INCORRETA.
b) Como vimos, o ato de “deploying” não se refere a voltar para casa depois de uma missão em
outro país. Alternativa INCORRETA.
c) No texto, “deploying” se refere justamente a realocar pessoal militar para um destino específico,
no caso, para os hospitais. Alternativa CORRETA.
d) O texto não fala sobre resgatar civis ameaçados pela guerra de zonas de conflito. O texto usa a
expressão war em sentido figurado, no sentido de combate ao coronavírus. “We are at war against
the coronavirus.” Alternativa INCORRETA.
e) No texto, “deploying” não se refere à contratação de estudantes de medicina para trabalhar em
hospitais no exterior. Alternativa INCORRETA.
GABARITO: C

Questão 09 (EsPCEx/INÉDITA) – According to the text, choose the correct statement.


a) coronavirus is likely to kill more people in the US than the two world wars together.
b) deploying the might of the military the traditional way is the solution against the virus.
c) the US must militarize their hospitals to contain the coronavirus spread.
d) most American cities will have their hospitals overwhelmed one at a time.
e) military training does not include stress management skills.
Comentários: a) De acordo com o texto, o coronavírus provavelmente matará mais pessoas nos
EUA do que as duas guerras mundiais juntas. “If we don’t act rapidly, we’re approaching a US death
toll larger than that of the two world wars combined.” (1º parágrafo). Alternativa CORRETA.
b) O texto não fala em deslocar o poder das forças armadas da maneira tradicional, mas sim de
um modo não convencional. “We must deploy the full might of the US military – in an
unconventional way.” (1º parágrafo). Alternativa INCORRETA.
c) Segundo o texto, os EUA não devem militarizar seus hospitais para conter a propagação de
coronavírus, mas os hospitais precisarão de ajuda militar. “We don’t need to militarize our
hospitals. But our hospitals will need help from our military.” (3º parágrafo). Alternativa
INCORRETA.
d) O texto afirma que a maioria das cidades americanas terá seus hospitais sobrecarregados não
um de cada vez, mas simultaneamente. “Most of our major cities will have their hospital systems
overwhelmed simultaneously.” (2º parágrafo). Alternativa INCORRETA.
e) O treinamento militar inclui, sim, habilidades de gerenciamento de estresse, de acordo com o
texto. “Members are trained to handle acute stress and are rapidly deployable.” (3º parágrafo).
Alternativa INCORRETA.
GABARITO: A

AULA 01 – SKIMMING AND SCANNING AND OTHER TECHNIQUES. IDENTIFYING FALSE COGNATS. 119
TEACHER ANDREA BELO

Leia o texto a seguir e responda às questões 10.


Is technology making time go quicker?
It never feels like there are enough hours in the day. If someone __________ (1) you a button to
get things done at twice the speed to free up time, you’d probably jump at it. In November, Netflix
introduced a feature to let users fast-forward their shows, playing at 1.5x speed. People are
already watching university lectures and their favourite YouTubers on double time and those that
speed-listen to podcasts – podfasters – are commonplace.
But fast forward __________ (2) an option since the days of VHS tapes. What’s so different now?
In 2018, Netflix released 1,500 hours of original content and to watch it all, it would take you four
hours of streaming a day for a whole year. With the same number of hours in the day, speed-
watching seems a logical solution, for both viewers and the streaming giants, to the information
and entertainment glut.
‘The brain is very capable of adapting to changes in experience,’ says neuroscientist Prof Edvard
Moser. Yet time is different from other sense data we adapt to, like sight and smell. The body
doesn’t have in-built sensors to measure time the way a clock does. It may __________ (3) like we
can sense time. But that’s a helpful illusion. Regardless of how time feels, there’s nothing stopping
us watching and consuming information at faster speeds than we do currently. ‘But maybe the
pleasure or enjoyment wouldn’t be the same.’
Adapted from https://metro.co.uk/2020/01/15/is-technology-making-time-go-quicker-12057474/

Questão 10 (EsPCEx/INÉDITA) – Choose the alternative containing the correct verb forms to
complete gaps (1), (2) and (3) in paragraphs 1, 2 and 3 respectively.
a) offers, have been, feel
b) offered, has been, feel
c) offered, have been, feels
d) offers, has been, feel
e) offered, has been, feels
Comentários: Na primeira frase- “If someone __________ (1) you a button to get things done at
twice the speed to free up time, you’d probably jump at it”, temos um sentido hipotético, de que
se alguém oferecesse um botão para fazer as coisas duas vezes mais rápido para liberar tempo,
você provavelmente aceitaria a oferta rapidamente. Em Inglês, o verbo no passado offered dá o
sentido de oferecesse.
Na segunda frase- “But fast forward __________ (2) an option since the days of VHS tapes.”, temos
a ideia de que acelerar a velocidade do vídeo tem sido uma opção desde os tempos das fitas de
vídeo cassete. Para concordar com o sujeito fast forward (it), teríamos que usar os verbos has
been.
Na terceira frase- “It may __________ (3) like we can sense time.”, como temos um modal verb
(may), o verbo que o segue fica na sua forma base, sem acréscimo de -s, ou seja, feel.

AULA 01 – SKIMMING AND SCANNING AND OTHER TECHNIQUES. IDENTIFYING FALSE COGNATS. 120
TEACHER ANDREA BELO

GABARITO: B

CONSIDERAÇÕES FINAIS
Outra aula concluída, ufa!!! Mais um passo até a sua aprovação! As técnicas Scanning e
Skimming ficaram mais claras em relação ao seu melhor uso. E os falsos cognatos, analisados com
maior cuidado, não é mesmo?
Continuaremos a estudar os conteúdos de forma minuciosa e prática, com sucesso!
É importante lembrar de fazer listas de vocabulário das palavras que você achou difíceis a
cada aula, em cada exercício ou lista, a fim de reescrevê-las e então, recordá-las nos momentos
de pausa entre as aulas.
Minha sugestão é que você faça a leitura dessas palavras consideradas “novas” para vê-las
novamente. Isso te ajudará nas questões em que esses vocábulos reaparecem. Acontece muito
com a classe dos verbos, por exemplo.
A cada lista de exercício resolvida ou mesmo a cada exercício que você faça, perceberá
como fica mais fácil identificar um verbo já visto no tempo passado ou particípio.
É sua conquista de etapas e que tornará você, um candidato mais bem preparado e
confiante para realizar uma excelente prova.

AULA 01 – SKIMMING AND SCANNING AND OTHER TECHNIQUES. IDENTIFYING FALSE COGNATS. 121
TEACHER ANDREA BELO

É importante lembrar também do nosso Fórum de dúvidas, exclusivo do Estratégia


Militares. Será minha forma de responder, no prazo máximo de 48 horas, o que mais você precise
saber para que os conteúdos fiquem ainda mais claros em seus estudos, certo?

REFERÊNCIAS BIBLIOGRÁFICAS
BARRETO, Tania Pedroza; GARRIDO, Maria Line; SILVA, João Antenor de C., Inglês Instrumental.
Leitura e compreensão de textos. Salvador, Ba UFBA, 1995, p. 64.
BROWN. H. Douglas. Principles of Language Learning and Teaching. Prentice Hall International,
1988.
COMPEDELLI, Samira Yousseff. Português, Literatura, Produção de texto & Gramática – São Paulo:
Ed. Saraiva, 2002.
CORREIA, Clese Mary P. Reading Specific Purposes. Salvador/ Ba: UFBA, 1999.
COSTEIRA, Adriana Araújo de M. Reading Comprehension Skills. João Pessoa/PB: ETFP, 1998.
CRYSTAL David. Cambridge University Press 1997. The Cambridge Encyclopedia of Language.
Cambridge University Press 1997
FREEMAN. Diane Larsen. MURCIA. Marianne Celce. The Grammar Book, 1999.
DYE, Joan., FRANFORT, Nancy. Spectrum II, III A Communicative Course in English. USA, Prentice
Hall, 1994.
FAVERO, Maria de Lourdes Albuquerque (org.). Dicionário de educadores no Brasil: da colônia aos
dias atuais. Rio de Janeiro: UFRJ, MEC, INEP, 1999.

AULA 01 – SKIMMING AND SCANNING AND OTHER TECHNIQUES. IDENTIFYING FALSE COGNATS. 122
TEACHER ANDREA BELO

FRANKPORT, Nancy & Dye Hoab. Spectrum II, III Prentice Hall Regents Englewood Cliffs, New Jersy,
1994.
GADELHA, Isabel Maria B. Inglês Instrumental: Leitura, Conscientização e Prática. Teresina:
EDUFFI, 2000.
GUANDALINI, Eiter Otávio. Técnicas de Leitura em Inglês: ESP – English For Specific Purposes:
estagio 1. São Paulo: Texto novo, 2002.
GRELLET, Françoise. Developing Reading Skills. Cambridge University Press, 1995
HOLAENDER, Arnon & Sanders Sidney. A complete English Course. São Paulo. Ed. Moderna, 1995.
HUTCHINSON, Tom & WATERS, Alan. English for Specific Purposes. Cambridge: Cambridge
University Press, 1996
KRASHEN. Stephen D. Second Language Acquisition and Second Language Learning, Prentice-Hall
International, 1988.
LAENG, Mauro. Dicionário de pedagogia. Lisboa: Dom Quixote, 1973.
LEFFA, Vilson J. Metodologia do ensino de línguas. In: BOHN, H.; VANDRESEN, P. (org.). Tópicos de
linguística aplicada: o ensino de línguas estrangeiras. Florianópolis: Editora da UFSC, 1988. p. 211-
231.
LIBERATO, Wilson. Compact English Book Inglês Ensino Médio. São Paulo: FTD, Vol. Único, 1998
Mc ARTHUR. The Oxford Companion to the English Language. Oxford University Press 1992
Fromkin. Victoria. An Introduction to Language
MARQUES, Amadeu. Inglês Série Brasil. ed. Atica. São Paulo: 2004. Vol. Único.
MURPHY, Raymond: Essencial Grammar in Use Oxford. New York Ed. Oxford University, 1997.
OLIVEIRA, Luciano Amaral. English For Tourism Students. Inglês para Estudantes de Turismo: São
Paulo, Rocca, 2001.
OLIVEIRA, Sara Rejane de F. Estratégias de leitura para Inglês Instrumental. Brasília: UNB, 1994.
QUINTANA, et alli. First Certificate. Master Class Oxford. New York, 2004: Ed. Oxford University.
PAULINO, Berenice F. et all. Leitura em textos em Inglês – Uma Abordagem Instrumental. Belo
Horizonte: Ed. Dos Autores, 1992.
PEREIRA, Edilberto Coelho. Inglês Instrumental. Teresina: ETFPI, 1998.
RODGES, Theodore. Jack C. Richards. Approaches and Methods in Language Teaching. Cambridge
University Press, 2001.
RODMAN Robert. Harcourt Brace 1993. English as a Global Language
STEWART, B., HAINES S. First Certificate, MasterClass. UK – Oxford 2004.
SILVA, João Antenor de C., GARRIDO, Maria Lina, BARRETO, Tânia Pedrosa. Inglês Instrumental:
Leitura e Compreensão de Textos. Salvador: Centro Editorial e Didático, UFBA. 1994
SOARES, Moacir Bretãs. Dicionário de legislação do ensino. 19.ed. Rio de Janeiro: FGV, 1981.

AULA 01 – SKIMMING AND SCANNING AND OTHER TECHNIQUES. IDENTIFYING FALSE COGNATS. 123
TEACHER ANDREA BELO

SOUZA, Adriana Srade F. Leitura em Língua Inglesa: Uma abordagem Instrumental. São Paulo:
Disal, 2005.
TUCK, Michael. Oxford Dictionary of Computing for Learners of English. Oxford: Oxford University
Press, 1996.
TOTIS, Verônica Pakrauskas. Língua Inglesa: leitura. São Paulo: Cortez, 1991.

Livros eletrônicos:
Dicionário Houaiss da Língua Portuguesa, Editora Objetiva, 2001.
MOURãO, Janaína Pereira. "Skimming x Scanning"; Brasil Escola. Disponível em
<https://brasilescola.uol.com.br/ingles/skimming-x-scanning.htm>. Acesso em 20 de março de
2019.
www.newsweek.com - Acesso em 18 de março de 2019.
http://www.galaor.com.br/tecnicas-de-leitura/ - Acesso em 19 de março de 2019.
Expressões Idiomáticas (continuação)" em Só Língua Inglesa. Virtuous Tecnologia da
Informação,2008-2019. Consultado em 03/04/2019 às 22:09. Disponível na Internet em
http://www.solinguainglesa.com.br/conteudo/Expressoes5.php

TRADUÇÕES
The influence of YouTubers on teenagers
A few years ago, the new internet hype called Web 2.0 appeared, resulting in a big shift from
using internet relatively passive and consuming, to a hype of networking as a revolutionary type
of largescale cooperation and economic democracy. This big shift towards user-driven
technologies such as blogs, video-sharing platforms and social networks enabled a revolution of
User-Generated Content (UGC). UGC refers to all the various media content created or produced
by the public, excluding paid professionals. UGC can be described as the process of ‘extracting
value by generating, reviewing, editing and disseminating content’. This revolution of UGC is now
being referred to as social media.
UGC is also stated to be the next iteration of Word-of-Mouth (WOM). Traditional (offline) WOM
has proven to play a major role in customers’ decision-making process. The so-called eWOM is
any positive or negative statement made by (potential) customers about a product or company,
which is made available to a multitude of people and institutions via the Internet. Consumers
can share their own experiences and opinions online, increasing the unbiased product
information people gather from other consumers online.

AULA 01 – SKIMMING AND SCANNING AND OTHER TECHNIQUES. IDENTIFYING FALSE COGNATS. 124
TEACHER ANDREA BELO

People are now able to create their own content and participate in all kinds of applications such
as weblogs, social networking and online journalism. Some users create online content on a daily
basis. They share their experiences and opinions about everything; from product reviews,
services and baby advice to boyfriend problems, homework, make up and game tutorials. Often
starting as a hobby, creating online content on a personal blog or channel can become a fulltime
job. By creating content on a daily basis, large communities are built up with thousands of loyal
followers. New followers and subscribers are gained every day. YouTube is one of the most
popular platforms on which reators share their content. With one single video it is possible to
reach millions of people. Other popular channels are Instagram, Snapchat, Vine, Facebook and
Twitter. These platforms are used for real-time contact with YouTube subscribers, as this is not
possible on YouTube itself.
Successful content creators become social influencers with a lot of persuasive influence. Their
influence and fame can be compared with traditional celebrities. The content they share via
social media has influence on the viewers, for example on viewers who are looking for product
information to support their purchasing decisions. Although YouTube is used by all ages, the
platform is very popular among teenagers. According to a research by Defy Media (2015), 96% of
the youngsters aged between 13 and 24 regularly use YouTube. They consume online videos via
social media sites for an average of 11 hours a week, making it their most popular website. This
popularity is reflected again in the amount of YouTubers that focus on teenagers. The influence
YouTubers have on the behavior of teenagers is increasingly unprecedented. They are seen as
role models and often recognized in the streets by their fans. However, outside this online world,
YouTubers are just as normal as most people.
Parents often do not know about the online behavior of their children and the YouTubers that
children endorse. This results in confusing moments when fans enthusiastically recognize their
favorite YouTuber while other people do not know who this particular YouTuber is. A well-known
Dutch beauty YouTuber explained that when she is in public, fans recognize her from YouTube.
Full of excitement they ask to take a picture together. However, the other people walking by are
confused and do not know why these young teenagers want to take pictures together. This
example shows the unknown impact of social influencers on youngsters of which most adults do
not even know the existence.
Although there already is research available on the influence of traditional media on youngsters,
it is interesting to know what influence YouTubers have on teenagers and whether this influence
is good or bad. Until this point, only little has been studied about the influence of YouTubers on
teenagers. Previous research mostly concerned participatory cultures on YouTube, reviewing
platforms of which YouTube is only a part of several more platforms or the difference between
YouTube and watching traditional television.
Moreover, few studies were conducted in Europe and often limited on the US. Consequently, the
present research takes both the perspective of viewers and YouTubers into account and
combines them within four themes. Therefore, the research question of this research is as
following: ‘What role do YouTubers play in the life of their teenage viewers and how do
YouTubers assess their own influences and experiences?’ The aim of this research is to give an
overview of the current YouTube community, including what influence Dutch YouTubers have on
their teenage viewers, what this means for teenagers and to what extent this influence on their
life is good or bad.

AULA 01 – SKIMMING AND SCANNING AND OTHER TECHNIQUES. IDENTIFYING FALSE COGNATS. 125
TEACHER ANDREA BELO

A influência dos YouTubers sobre os adolescentes


Há alguns anos, surgiu a nova campanha publicitária da Internet chamada Web 2.0, resultando
em uma grande mudança do uso da Internet relativamente passiva e consumidora para uma
campanha publicitária de networking como um tipo revolucionário de cooperação em larga
escala e democracia econômica. Essa grande mudança em direção a tecnologias orientadas ao
usuário, como blogs, plataformas de compartilhamento de vídeo e redes sociais, possibilitou
uma revolução no Conteúdo Gerado pelo Usuário (UGC). UGC refere-se a todos os diversos
conteúdos de mídia criados ou produzidos pelo público, excluindo profissionais pagos. UGC pode
ser descrito como o processo de 'extração de valor gerando, revisando, editando e disseminando
conteúdo'. Essa revolução do UGC agora está sendo chamada de mídia social.
O UGC também é considerado a próxima iteração do Word-of-Mouth (WOM). O WOM
tradicional (offline) provou desempenhar um papel importante no processo de tomada de
decisão dos clientes. A chamada eWOM é qualquer declaração positiva ou negativa feita por
(potenciais) clientes sobre um produto ou empresa, que é disponibilizada a um grande número
de pessoas e instituições através da Internet. Os consumidores podem compartilhar suas
próprias experiências e opiniões online, aumentando as informações imparciais sobre o produto
que as pessoas coletam de outros consumidores online.
As pessoas agora podem criar seu próprio conteúdo e participar de todos os tipos de aplicativos,
como weblogs, redes sociais e jornalismo online. Alguns usuários criam conteúdo online
diariamente. Eles compartilham suas experiências e opiniões sobre tudo; desde análises de
produtos, serviços e conselhos sobre bebês até problemas com namorados, lição de casa,
maquiagem e tutoriais de jogos. Muitas vezes, começando como um hobby, criar conteúdo
online em um blog ou canal pessoal pode se tornar um trabalho de tempo integral. Ao criar
conteúdo diariamente, grandes comunidades são formadas com milhares de seguidores leais.
Novos seguidores e assinantes são conquistados todos os dias. O YouTube é uma das
plataformas mais populares em que os reatores compartilham seu conteúdo. Com um único
vídeo é possível atingir milhões de pessoas. Outros canais populares são Instagram, Snapchat,
Vine, Facebook e Twitter. Essas plataformas são utilizadas para o contato em tempo real com os
assinantes do YouTube, pois isso não é possível no próprio YouTube.
Criadores de conteúdo de sucesso se tornam influenciadores sociais com muita influência
persuasiva. Sua influência e fama podem ser comparadas às celebridades tradicionais. O
conteúdo que compartilham nas redes sociais tem influência sobre os telespectadores, por
exemplo, os espectadores que procuram informações sobre o produto para apoiar suas decisões
de compra. Embora o YouTube seja usado por todas as idades, a plataforma é muito popular
entre os adolescentes. De acordo com uma pesquisa da Defy Media (2015), 96% dos jovens
entre 13 e 24 anos usam o YouTube regularmente. Eles consomem vídeos online por meio de
sites de mídia social por uma média de 11 horas por semana, tornando-o seu site mais popular.
Essa popularidade se reflete novamente na quantidade de YouTubers que se concentram em
adolescentes. A influência dos YouTubers no comportamento dos adolescentes é cada vez mais
sem precedentes. Eles são vistos como modelos e frequentemente reconhecidos nas ruas por
seus fãs. No entanto, fora deste mundo online, os YouTubers são tão normais quanto a maioria
das pessoas.

AULA 01 – SKIMMING AND SCANNING AND OTHER TECHNIQUES. IDENTIFYING FALSE COGNATS. 126
TEACHER ANDREA BELO

Frequentemente, os pais não sabem sobre o comportamento online de seus filhos e sobre os
YouTubers que as crianças endossam. Isso resulta em momentos confusos, quando os fãs
reconhecem com entusiasmo seu YouTuber favorito, enquanto outras pessoas não sabem quem
é esse YouTuber em particular. Uma conhecida beleza holandesa, YouTuber, explicou que,
quando ela está em público, os fãs a reconhecem do YouTube. Cheios de emoção, eles pedem
para tirar uma foto juntos. No entanto, as outras pessoas que passam estão confusas e não
sabem por que esses jovens querem tirar fotos juntos. Este exemplo mostra o impacto
desconhecido dos influenciadores sociais sobre os jovens, dos quais a maioria dos adultos nem
sabe da existência.
Embora já existam pesquisas disponíveis sobre a influência da mídia tradicional sobre os jovens,
é interessante saber qual influência os YouTubers têm sobre os adolescentes e se essa influência
é boa ou ruim. Até o momento, pouco se estudou sobre a influência dos YouTubers sobre os
adolescentes. A pesquisa anterior envolveu principalmente culturas participativas no YouTube,
analisando plataformas das quais o YouTube é apenas uma parte de várias outras plataformas ou
a diferença entre o YouTube e assistir à televisão tradicional.
Além disso, poucos estudos foram conduzidos na Europa e muitas vezes limitados nos Estados
Unidos. Consequentemente, a presente pesquisa leva em consideração a perspectiva dos
telespectadores e dos YouTubers e os combina em quatro temas. Portanto, a questão de
pesquisa desta pesquisa é a seguinte: 'Qual o papel que os YouTubers desempenham na vida de
seus espectadores adolescentes e como os YouTubers avaliam suas próprias influências e
experiências?' Comunidade do YouTube, incluindo a influência que os YouTubers holandeses têm
sobre seus espectadores adolescentes, o que isso meios para os adolescentes e em que medida
essa influência em suas vidas é boa ou ruim.

TRAVEL TIPS
How to Plan a Movie-Themed Vacation
It’s easier than you may expect to find, visit, and enjoy the places where your favorite movies
were made.
Lars Leetaru
By Shivani Vora
March 8, 2018
Whether it’s the “Lord of the Rings” trilogy in New Zealand or "Roman Holiday” in Rome, many
noteworthy movies are filmed in appealing locales all over the world that travelers may want to
visit and enjoy.
According to Angela Tillson, a film location manager in Kauai who has worked on the set of films
including "Jurassic Park: The Lost World” and “The Descendants," exploring a beloved movie set
destination through the eyes of the film makes for an enjoyable vacation. "Seeing a place with a
focus on a movie you love will give you a perspective that the average tourist doesn’t usually
get. You’ll certainly have a better impression of the place,” she said. Here are her tips to get
started.

AULA 01 – SKIMMING AND SCANNING AND OTHER TECHNIQUES. IDENTIFYING FALSE COGNATS. 127
TEACHER ANDREA BELO

Choose Your Destination


If there’s a movie you love, you can find out where it was filmed by looking at the credits at the
end of the film or by going online to The Internet Movie Database, also known as IMDB, which
often lists filming locations. Once you know the locale, you can start planning your trip. Or,
consider doing what Ms. Tillson often does when deciding on where to vacation: pick a spot
you’re interested in visiting, and find out what movies have been filmed there. “It’s fun to
sometimes let a destination determine the movie you're going to live rather than the other way
around,” Ms. Tillson said.
Get in the Mood
Before you head to your destination, be sure to rewatch the movie. A rewatch not only reminds
you of identifiable spots to look out for during your trip, but it also adds to the excitement of
your upcoming exploration.
If the movie is based on a book, consider reading the book, too. It may have details about the
locale that the movie doesn’t touch on. Also, books often have scenes that don’t make it into
the movie adaptations, which gives you a deeper view of the destination. Ms. Tillson also
recommended downloading the movie’s soundtrack or score, and listening to it throughout your
trip.
Book a Themed Trip
Some travel companies sell set itineraries focused on popular movies. Luxury tour operator
Zicasso, for example, has an eight-day trip, all inclusive, to Ireland inspired by "Star Wars: The
Last Jedi” and Wild Frontiers has an eleven-day trip to India inspired by "The Best Exotic
Marigold Hotel." Ms. Tillson suggested doing a web search or checking with a travel agent to
find out about such trips.
Also, in some destinations, local tour operators and hotels sell movie-themed tours. For
instance, The St. Regis Priceville Resort offers a tour that includes a private helicopter ride to
Manawaiopuna Falls, made famous in "Jurassic Park,” and an ATV tour of filming locations of
movies such as “Raiders of the Lost Ark" and “Pirates of the Caribbean.” Lunch is even included.
The cost is $5,674 for two adults.
A more affordable option, in Rome, is the four-hour “Roman Holiday" themed excursion from
HR Tours, where travelers ride a Vespa with a driver and see all the sites from the movie; the
cost is 170 euros per person.
Hang Where the Movie Crew Did
When they’re not working, movie crews enjoy hitting local bars and casual restaurants that
serve tasty local cuisine, Ms. Tillson said.
Find out where the behind-the-scenes staff of your film spent their time by asking your
destination’s tourist board or your hotel’s concierge, and check out a few of the spots. “It’s
another way to get involved in the film and spend time in bars and restaurants that you
wouldn’t normally think to hit,” she said.

DICAS DE VIAGEM

AULA 01 – SKIMMING AND SCANNING AND OTHER TECHNIQUES. IDENTIFYING FALSE COGNATS. 128
TEACHER ANDREA BELO

Como planejar férias com tema de filme


É mais fácil do que você espera encontrar, visitar e desfrutar dos lugares onde seus filmes
favoritos foram feitos.
Lars Leetaru
Por Shivani Vora
8 de março de 2018
Quer seja a trilogia "O Senhor dos Anéis" na Nova Zelândia ou "Férias Romanas" em Roma,
muitos filmes notáveis são filmados em locais atraentes em todo o mundo que os viajantes
podem querer visitar e desfrutar.
De acordo com Angela Tillson, gerente de locação de filmes em Kauai que trabalhou no set de
filmes, incluindo "Jurassic Park: The Lost World" e "The Descendants", explorar um destino de
set de filmagem amado pelos olhos do filme é uma experiência agradável período de férias. “Ver
um lugar com foco em um filme que você adora lhe dará uma perspectiva que o turista comum
não costuma ter. Você certamente terá uma impressão melhor do lugar”, disse ela. Aqui estão
suas dicas para começar .
Escolha o seu destino
Se há um filme que você ama, você pode descobrir onde ele foi filmado olhando os créditos no
final do filme ou acessando o Internet Movie Database, também conhecido como IMDB, que
geralmente lista os locais de filmagem. Depois de saber o local, você pode começar a planejar
sua viagem. Ou considere fazer o que a Sra. Tillson costuma fazer ao decidir onde passar as
férias: escolha um local que você está interessado em visitar e descubra quais filmes foram
filmados lá. “Às vezes é divertido deixar um destino determinar o filme que você vai viver, e não
o contrário”, disse Tillson.
Entre no clima
Antes de se dirigir ao seu destino, certifique-se de assistir novamente ao filme. Uma nova
observação não apenas o lembra de pontos identificáveis a serem observados durante sua
viagem, mas também aumenta a emoção de sua próxima exploração.
Se o filme for baseado em um livro, considere a possibilidade de ler o livro também. Pode haver
detalhes sobre o local que o filme não menciona. Além disso, os livros costumam ter cenas que
não fazem parte das adaptações para o cinema, o que lhe dá uma visão mais aprofundada do
destino. A Sra. Tillson também recomendou baixar a trilha sonora ou trilha sonora do filme e
ouvi-la durante a viagem.
Reserve uma viagem temática
Algumas agências de viagens vendem itinerários definidos com foco em filmes populares. A
operadora de turismo de luxo Zicasso, por exemplo, tem uma viagem de oito dias, com tudo
incluído, para a Irlanda inspirada em "Star Wars: The Last Jedi" e Wild Frontiers tem uma viagem
de onze dias para a Índia inspirada em "The Best Exotic Marigold Hotel. "A Sra. Tillson sugeriu
fazer uma pesquisa na web ou verificar com um agente de viagens para saber mais sobre essas
viagens.

AULA 01 – SKIMMING AND SCANNING AND OTHER TECHNIQUES. IDENTIFYING FALSE COGNATS. 129
TEACHER ANDREA BELO

Além disso, em alguns destinos, os operadores turísticos locais e os hotéis vendem passeios
temáticos de filmes. Por exemplo, o St. Regis Priceville Resort oferece um passeio que inclui um
passeio de helicóptero particular para Manawaiopuna Falls, que ficou famosa em "Jurassic
Park", e um passeio de quadriciclo pelos locais de filmagem de filmes como "Os Caçadores da
Arca Perdida" e " Piratas do Caribe." O almoço está incluído. O custo é de US $ 5.674 para dois
adultos.
Uma opção mais econômica, em Roma, é a excursão temática de “Férias Romanas” de quatro
horas da HR Tours, onde os viajantes andam de Vespa com um motorista e veem todos os sites
do filme; o custo é de 170 euros por pessoa.
Fique onde a equipe do filme fez
Quando não estão trabalhando, as equipes de cinema aproveitam os bares locais e restaurantes
casuais que servem a saborosa culinária local, disse Tillson.
Descubra onde a equipe de bastidores de seu filme passou o tempo, perguntando ao conselho
de turismo de seu destino ou ao concierge do seu hotel e confira alguns dos locais. “É outra
maneira de se envolver no filme e passar o tempo em bares e restaurantes que você
normalmente não pensaria em ir”, disse ela.

STEPHEN HAWKING
Stephen Hawking, one of the most famous scientists of the 21st century, died on March 14th,
2018. But his ideas on gravity, black holes and the Big Bang are the greatest legacy he left to the
world.
Early Life and College
On January 8th, 1942, Stephen Hawking was born to a successful family in Oxford, England. He
was not the best student at fundamental or high school, but he was very smart. His dad wanted
him to become a medical doctor, but Stephen really wanted to study math.
Despite his poor grades at school, Stephen aced his exams for college. He was admitted to
Oxford University, but they didn't have math as a major. So, he decided to study physics and
chemistry instead.
Stephen became a member of a rowing team when he was in college. After he graduated, he
decided to continue his education and went to graduate school.
Graduate School, Marriage and Health Problems

AULA 01 – SKIMMING AND SCANNING AND OTHER TECHNIQUES. IDENTIFYING FALSE COGNATS. 130
TEACHER ANDREA BELO

While in graduate school, he had some health problems. He began tripping for no reason, and
his speech became hard to understand. His family encouraged him to go to the doctor. Stephen
was diagnosed with a disease called ALS, or Lou Gehrig's disease, which affects the brain and
spine. He was only twenty-one, and the doctors initially gave him only a few years to live.
Luckily, his condition progressed more slowly than is often the case.
During this time, Stephen had a relationship with a woman named Jane. He said she and his
work were his inspiration for living. Stephen earned his Ph.D. degree in 1965. He started to get
worse, and eventually became confined to a wheelchair. Nonetheless, he and Jane married in
1965 and were able to have three children.
He studied how space and time are related, including scientific studies of black holes in space
and how they work in the universe. He also had a lot of success in his work as a college
professor.
A New Voice
In 1985, Stephen got really sick and doctors were able to save him, but he was unable to speak.
He could only use his eyebrows to communicate. Eventually, he was able to use a special voice
synthesizer, allowing him to talk by moving his cheek muscles and using a mouse pad.
Famous Works and Prizes
His most notorious theory is that black holes can emit radiation; also known as Hawking
radiation. He received numerous awards but never won the Nobel Prize.
Stephen always enjoyed writing books. His best seller, “A Brief History in Time” made terms like
the Big Bang and black holes easy to understand. Other famous Stephen Hawking books include:
“A Briefer History in Time", “On the Shoulders of Giants” and "The Universe in a Nutshell". He
also wrote many books for children along with his daughter Lucy. His famous books for children
include "George’s Cosmic Treasure Hunt" and “George and the Big Bang”.
His last work, submitted only two weeks before his death, reveals the universe will come to an
end when stars run out of energy. However, his theory suggests that scientists will be able to
find parallel universes using probes on spaceships.

STEPHEN HAWKING
Stephen Hawking, um dos cientistas mais famosos do século 21, morreu em 14 de março de
2018. Mas suas ideias sobre gravidade, buracos negros e o Big Bang são o maior legado que ele
deixou para o mundo.
Juventude e faculdade
Em 8 de janeiro de 1942, Stephen Hawking nasceu em uma família bem-sucedida em Oxford,
Inglaterra. Ele não era o melhor aluno do ensino fundamental ou médio, mas era muito
inteligente. Seu pai queria que ele se tornasse médico, mas Stephen realmente queria estudar
matemática.
Apesar de suas notas baixas na escola, Stephen foi bem-sucedido nos exames para a faculdade.
Ele foi admitido na Universidade de Oxford, mas eles não tinham matemática como
especialização. Então, ele decidiu estudar física e química.

AULA 01 – SKIMMING AND SCANNING AND OTHER TECHNIQUES. IDENTIFYING FALSE COGNATS. 131
TEACHER ANDREA BELO

Stephen tornou-se membro de uma equipe de remo quando estava na faculdade. Depois de se
formar, ele decidiu continuar seus estudos e fez pós-graduação.
Escola de Pós-Graduação, Casamento e Problemas de Saúde
Enquanto estava na pós-graduação, ele teve alguns problemas de saúde. Ele começou a viajar
sem motivo, e sua fala tornou-se difícil de entender. Sua família o incentivou a ir ao médico.
Stephen foi diagnosticado com uma doença chamada ALS, ou doença de Lou Gehrig, que afeta o
cérebro e a coluna. Ele tinha apenas 21 anos e os médicos inicialmente deram-lhe apenas alguns
anos de vida. Felizmente, sua condição progrediu mais lentamente do que costuma acontecer.
Durante esse tempo, Stephen teve um relacionamento com uma mulher chamada Jane. Ele
disse que ela e seu trabalho eram sua inspiração para viver. Stephen obteve seu Ph.D. Ele se
formou em 1965. Ele começou a piorar e acabou ficando confinado a uma cadeira de rodas.
Mesmo assim, ele e Jane se casaram em 1965 e puderam ter três filhos.
Ele estudou como o espaço e o tempo estão relacionados, incluindo estudos científicos sobre
buracos negros no espaço e como eles funcionam no universo. Ele também teve muito sucesso
em seu trabalho como professor universitário.
Uma Nova Voz
Em 1985, Stephen ficou muito doente e os médicos conseguiram salvá-lo, mas ele não
conseguia falar. Ele só podia usar as sobrancelhas para se comunicar. Eventualmente, ele foi
capaz de usar um sintetizador de voz especial, permitindo-lhe falar movendo os músculos da
bochecha e usando um mouse pad.
Obras e prêmios famosos
Sua teoria mais notória é que os buracos negros podem emitir radiação; também conhecida
como radiação Hawking. Ele recebeu vários prêmios, mas nunca ganhou o Prêmio Nobel.
Stephen sempre gostou de escrever livros. Seu best-seller, “A Brief History in Time” tornou
termos como Big Bang e buracos negros fáceis de entender. Outros livros famosos de Stephen
Hawking incluem: "A Briefer History in Time", "On the Shoulders of Giants" e "The Universe in a
Nutshell". Ele também escreveu muitos livros para crianças junto com sua filha Lucy. Seus
famosos livros para crianças incluem "George's Cosmic Treasure Hunt" e "George and the Big
Bang".
Seu último trabalho, apresentado apenas duas semanas antes de sua morte, revela que o
universo chegará ao fim quando as estrelas ficarem sem energia. No entanto, sua teoria sugere
que os cientistas serão capazes de encontrar universos paralelos usando sondas em naves
espaciais.

AULA 01 – SKIMMING AND SCANNING AND OTHER TECHNIQUES. IDENTIFYING FALSE COGNATS. 132
TEACHER ANDREA BELO

Robotic Cars
The year is 2020, and it’s 7;45 on a rainy Monday morning, and you are in your car on your way
to work. You turn right, and you turn left. A few minutes later, you stop at a traffic light. When
the light turns green and there are no other cars in the intersection, you continue on your way.
Ten minutes later you get to work and stop reading the morning paper. Then, you get out of
your car and you say, “Thank you!". Your car replies, “You’re welcome!’’. This possible future
may sound unreal, but in fact many car companies are already testing robotic cars, or driverless
cars, on the roads today, although the cars don't speak very much yet.
In the 1980s, Germany and the United States tested the first driverless cars, and by 2020
companies such as Volvo, GM, Nissan and BMW plan to sail driverless cars. Driverless cars are
not really ‘driverless - the drivers are computers that use radar, Computer maps and other
modern technology. They offer many advantages. Perhaps the most important of these is fewer
deaths caused by road accidents. For example, in 1968 more than 53.000 people lost their lives
in car accidents in the USA. This number has fallen to less than 33.000 but it’s still a high
number. In addition, people will spend less time stuck in traffic jams and there will be no need
for people to have a driving license. One of the major disadvantages of this new technology,
however, is the cost. It’s not free. U$5.000 to U$10.000 is added to the price of the new car.

AULA 01 – SKIMMING AND SCANNING AND OTHER TECHNIQUES. IDENTIFYING FALSE COGNATS. 133
TEACHER ANDREA BELO

Nevertheless, at some time in your life, you will probably be sitting in a robotic, ;driverless car
on your way to work or school. The future is almost here. Are you ready for it?
<https://www.aHthinastoDics.eom/uploads/2/3/2/9/23290220/lesson-drivina-robo ticcars2.pdf>

Carros robóticos
O ano é 2020 e são 7h45 em uma manhã chuvosa de segunda-feira, e você está no carro a
caminho do trabalho. Você vira à direita e vira à esquerda. Poucos minutos depois, você para em
um semáforo. Quando o semáforo ficar verde e não houver outros carros no cruzamento,
continue seu caminho. Dez minutos depois, você começa a trabalhar e para de ler o jornal
matutino. Então, você sai do carro e diz: "Obrigado!". Seu carro responde: "De nada! ''. Esse
futuro possível pode parecer irreal, mas na verdade muitas montadoras estão testando carros
robóticos no ar, ou carros sem motorista, nas estradas hoje, embora os carros ainda não falem
muito.
Na década de 1980, a Alemanha e os Estados Unidos testaram os primeiros carros sem
motorista e, em 2020, empresas como a Volvo, GM, Nissan e BMW planejam vender carros sem
motorista. Carros sem motorista não são realmente 'sem motorista - os motoristas são
computadores que usam radar, mapas de computador e outras tecnologias modernas. Eles
oferecem muitas vantagens. Talvez o mais importante deles seja o menor número de mortes
causadas por acidentes rodoviários. Por exemplo, em 1968, mais de 53.000 pessoas perderam a
vida em acidentes de carro nos EUA. Este número caiu para menos de 33.000, mas ainda é um
número alto. Além disso, as pessoas passarão menos tempo presas em congestionamentos e
não haverá necessidade de carteira de habilitação. Uma das maiores desvantagens dessa nova
tecnologia, entretanto, é o custo. Não é grátis. U $ 5.000 a U $ 10.000 são adicionados ao preço
do carro novo. No entanto, em algum momento da sua vida, você provavelmente estará
sentado em um carro robótico, sem motorista, a caminho do trabalho ou da escola. O futuro
está quase aqui. Você está pronto para isto?
Diplomat
There are three main aspects to this profession: a diplomat has to keep his country informed
about pertinent international events, promote a favorable image of his country and protect his
country’s interests.
Whoever is interested in a diplomatic career has to be extremely familiar with political,
economical, scientific, cultural and administrative issues. To be a diplomat, it is essential to have
a good knowledge of English, not only the conversational language, but also the technical terms
in international law and diplomacy itself.
To follow this career, besides being fluently bilingual, one needs a standard college education
and has to take and do well in the Rio Branco Institute examination in Brasilia.
English is so important in this career that in the first part of this selection exam, the applicant
has to demonstrate his or her proficiency in the English language. Then, during the course,
foreign language classes become a priority, giving future diplomats the necessary expertise to
deal with the areas of official correspondences, diplomatic negotiation and international media.
(Adapted from Inglês no mundo do trabalho)

AULA 01 – SKIMMING AND SCANNING AND OTHER TECHNIQUES. IDENTIFYING FALSE COGNATS. 134
TEACHER ANDREA BELO

Diplomata
Esta profissão tem três aspectos principais: um diplomata deve manter seu país informado
sobre eventos internacionais pertinentes, promover uma imagem favorável de seu país e
proteger os interesses de seu país.
Quem se interessa por uma carreira diplomática deve estar extremamente familiarizado com as
questões políticas, econômicas, científicas, culturais e administrativas. Para ser diplomata, é
fundamental ter bons conhecimentos da língua inglesa, não só da língua de conversação, mas
também dos termos técnicos do direito internacional e da própria diplomacia.
Para seguir essa carreira, além de ser bilíngue fluente, é preciso ter formação superior padrão e
ter aproveitamento no exame do Instituto Rio Branco, em Brasília.
O inglês é tão importante nesta carreira que, na primeira parte desse exame de seleção, o
candidato deve demonstrar sua proficiência na língua inglesa. Depois, no decorrer do curso, as
aulas de língua estrangeira passam a ser uma prioridade, dando aos futuros diplomatas os
conhecimentos necessários para lidar com as áreas de correspondência oficial, negociação
diplomática e mídia internacional.

Oil contaminating Brazil's beaches very likely from Venezuela


Anna Jean Kaiser
Thick crude oil that has stained hundreds of miles of pristine Brazilian beach in recent weeks
probably originated in Venezuela, the Brazilian government has said, in an accusation likely to
further strain relations between the two countries.
Brazilians authorities have been investigating the growing disaster for more than a month, as
the oil has spread to more than 130 beaches across nine states.
Ricardo Salles, the country’s environment minister, told that a study by Petrobrás had concluded
that the oil “is very likely from Venezuela. He said that a foreign ship near Brazil’s coastline
appeared to have caused the spill.
There was no immediate response from Venezuela.
Social media users have shared shocking images of the spill, showing kilometers of white sand
stained with oil blotches and dead, oil-covered turtles and dolphins. One video shows thick black
oil lapping up against a rocky jetty.
Adapted from The Guardian

AULA 01 – SKIMMING AND SCANNING AND OTHER TECHNIQUES. IDENTIFYING FALSE COGNATS. 135
TEACHER ANDREA BELO

Óleo contamina as praias do Brasil muito provavelmente da Venezuela


Anna Jean Kaiser
O óleo cru espesso que manchou centenas de quilômetros de praias primitivas no Brasil nas
últimas semanas provavelmente se originou na Venezuela, disse o governo brasileiro, em uma
acusação que provavelmente prejudicará ainda mais as relações entre os dois países.
As autoridades brasileiras investigam o crescente desastre há mais de um mês, já que o petróleo
se espalhou para mais de 130 praias em nove estados.
Ricardo Salles, ministro do Meio Ambiente do país, disse que um estudo da Petrobrás havia
concluído que o petróleo “é muito provável da Venezuela. Ele disse que um navio estrangeiro
perto da costa do Brasil parece ter causado o vazamento.
Não houve resposta imediata da Venezuela.
Usuários de mídia social compartilharam imagens chocantes do derramamento, mostrando
quilômetros de areia branca manchada com manchas de óleo e tartarugas e golfinhos mortos
cobertos de óleo. Um vídeo mostra óleo preto espesso batendo contra um cais rochoso.

Residents and businesses of south Florida are experiencing regular episodes of water in the
streets. In the battle against rising seas, the region – which has more to lose than almost
anywhere else in the world – is becoming ground zero.
The first time my father’s basement flooded, it was shortly after he moved in. The building was
an ocean-front high-rise in a small city north of Miami called Sunny Isles Beach. The marble
lobby had a waterfall that never stopped running; crisp-shirted valets parked your car for you.
For the residents who lived in the more lavish flats, these cars were often BMWs and Mercedes.
But no matter their value, the cars all wound up in the same place: the basement.
When I called, I’d ask my dad how the building was doing. “The basement flooded again a
couple weeks ago,” he’d sometimes say. Or: “It’s getting worse.” It’s not only his building: he’s
also driven through a foot of water on a main road a couple of towns over and is used to
tiptoeing around pools in the local supermarket’s car park.
Ask nearly anyone in the Miami area about flooding and they’ll have an anecdote to share.
Many will also tell you that it’s happening more and more frequently. The data backs them up.
It’s easy to think that the only communities suffering from sea level rise are far-flung and
remote. And while places like the Solomon Islands and Kiribati are indeed facing particularly

AULA 01 – SKIMMING AND SCANNING AND OTHER TECHNIQUES. IDENTIFYING FALSE COGNATS. 136
TEACHER ANDREA BELO

dramatic challenges, they aren’t the only ones being forced to grapple with the issue. Sea levels
are rising around the world, and in the US, south Florida is ground zero – as much for the
adaptation strategies it is attempting as for the risk that it bears.
One reason is that water levels here are rising especially quickly. The most frequently-used
range of estimates puts the likely range between 15-25cm (6-10in) above 1992 levels by 2030,
and 79-155cm (31-61in) by 2100. With tides higher than they have been in decades – and far
higher than when this swampy, tropical corner of the US began to be drained and built on a
century ago – many of south Florida’s drainage systems and seawalls are no longer enough. That
means not only more flooding, but challenges for the infrastructure that residents depend on
every day, from septic tanks to wells. “The consequences of sea level rise are going to occur way
before the high tide reaches your doorstep,” says William Sweet, an oceanographer at the
National Oceanic and Atmospheric Administration (NOAA).
The flooding would be a challenge for any community, but it poses particular risks here. One
recent report estimated that Miami has the most to lose in terms of financial assets of any
coastal city in the world, just above Guangzhou, China and New York City. This 120-mile (193km)
corridor running up the coast from Homestead to Jupiter – taking in major cities like Miami, Fort
Lauderdale and West Palm Beach – is the eighth most populous metropolitan area in the US. It’s
also booming. In 2015, the US Census Bureau found that the population of all three counties
here was growing – along with the rest of Florida – at around 8%, roughly twice the pace of the
US average. Recent studies have shown that Florida has more residents at risk from climate
change than any other US state.
Adapted from https://www.bbc.com/future/article/20170403-miamis-fight-against-sea-level-rise

Moradores e empresas do sul da Flórida estão enfrentando episódios regulares de água nas
ruas. Na batalha contra a elevação do mar, a região - que tem mais a perder do que qualquer
outro lugar do mundo - está se tornando o marco zero.
A primeira vez que o porão do meu pai inundou, foi logo depois que ele se mudou. O prédio era
um arranha-céu à beira-mar em uma pequena cidade ao norte de Miami chamada Sunny Isles
Beach. O saguão de mármore tinha uma cachoeira que nunca parava de correr; manobristas de
camisa impecável estacionaram seu carro para você. Para os residentes que moravam nos
apartamentos mais luxuosos, esses carros eram geralmente BMWs e Mercedes. Mas,
independentemente do valor, os carros terminaram todos no mesmo lugar: o porão.
Quando liguei, perguntei ao meu pai como estava o prédio. “O porão inundou novamente
algumas semanas atrás”, ele às vezes dizia. Ou: “Está piorando.” Não é apenas seu prédio: ele
também é dirigido por trinta centímetros de água em uma estrada principal, algumas cidades
depois, e está acostumado a andar na ponta dos pés em torno de piscinas no estacionamento
do supermercado local.
Pergunte a quase qualquer pessoa na área de Miami sobre inundações e eles terão uma
anedota para contar. Muitos também dirão que isso está acontecendo com cada vez mais
frequência. Os dados os sustentam.
É fácil pensar que as únicas comunidades que sofrem com a elevação do nível do mar são muito
distantes e remotas. E embora lugares como as Ilhas Salomão e Kiribati estejam de fato

AULA 01 – SKIMMING AND SCANNING AND OTHER TECHNIQUES. IDENTIFYING FALSE COGNATS. 137
TEACHER ANDREA BELO

enfrentando desafios particularmente dramáticos, eles não são os únicos sendo forçados a lidar
com o problema. O nível do mar está subindo em todo o mundo e, nos Estados Unidos, o sul da
Flórida é o marco zero - tanto pelas estratégias de adaptação que tenta quanto pelo risco que
acarreta.
Um dos motivos é que os níveis de água aqui estão subindo com rapidez especial. O intervalo de
estimativas usado com mais frequência coloca o intervalo provável entre 15-25 cm (6-10 pol.)
Acima dos níveis de 1992 em 2030 e 79-155 cm (31-61 pol.) Em 2100. Com as marés mais altas
do que há décadas - e muito mais alto do que quando este canto pantanoso e tropical dos EUA
começou a ser drenado e construído um século atrás - muitos dos sistemas de drenagem e
paredões do sul da Flórida não são mais suficientes. Isso significa não apenas mais enchentes,
mas desafios para a infraestrutura da qual os moradores dependem todos os dias, de fossas
sépticas a poços. “As consequências da elevação do nível do mar ocorrerão muito antes que a
maré alta chegue à sua porta”, diz William Sweet, oceanógrafo da Administração Oceânica e
Atmosférica Nacional (NOAA).
A inundação seria um desafio para qualquer comunidade, mas apresenta riscos específicos aqui.
Um relatório recente estimou que Miami tem mais a perder em termos de ativos financeiros do
que qualquer cidade costeira do mundo, logo acima de Guangzhou, China e Nova York. Este
corredor de 120 milhas (193 km) que sobe a costa de Homestead a Júpiter - passando por
grandes cidades como Miami, Fort Lauderdale e West Palm Beach - é a oitava área
metropolitana mais populosa dos Estados Unidos. Também está crescendo. Em 2015, o US
Census Bureau descobriu que a população de todos os três condados aqui estava crescendo -
junto com o resto da Flórida - em cerca de 8%, quase o dobro do ritmo da média dos EUA.
Estudos recentes mostraram que a Flórida tem mais residentes em risco de mudanças climáticas
do que qualquer outro estado dos EUA.

The WHO has redefined burnout as a syndrome linked to chronic work stress. There’s a
difference between a busy workload and something more serious, writes Zaria Gorvett.
If you said you were suffering from ‘burnout’ in the early 1970s, you might have raised some
eyebrows.
At the time, the term was used informally to describe the side effects that heavy drug users
experienced: the general dimming of the mental faculties, for example, as was the case with
many a party animal. However, when German-American psychologist Herbert Freudenberger
first recognised the problem of burnout in New York City in 1974, at a clinic for addicts and
homeless people, Freudenberger wasn’t thinking of drug users.
The clinic’s volunteers were actually struggling, too: their work was intense, and many were
beginning to feel demotivated and emotionally drained. Though they had once found their jobs
rewarding, they had become cynical and depressed; they weren’t giving their patients the
attention they deserved. Freudenberger defined this alarming new condition as a state of
exhaustion caused by prolonged overwork – and borrowed the term ‘burnout’ to describe it.

AULA 01 – SKIMMING AND SCANNING AND OTHER TECHNIQUES. IDENTIFYING FALSE COGNATS. 138
TEACHER ANDREA BELO

Its popularity was explosive, and today burnout is a global phenomenon. Although statistics on
the prevalence of burnout specifically are hard to come by, 595,000 people in the UK alone
suffered from workplace stress in 2018.
Sportspeople get it. YouTube stars get it. Entrepreneurs get it. Freudenberger himself eventually
got it. Late last month, the World Health Organization (WHO) announced that the trendy
problem will be recognised in the latest International Classification of Diseases manual, where it
is described as a syndrome “resulting from chronic workplace stress that has not been
successfully managed”.
According to the WHO, burnout has three elements: feelings of exhaustion, mental detachment
from one’s job and poorer performance at work. But waiting until you’re already fully burned
out to do something about it doesn’t help at all –and you wouldn’t wait to treat any other illness
until it was too late.
Feeling the burn
So how can you tell if you’re almost – but not quite – burned out?
“A lot of the signs and symptoms of pre-burnout would be very similar to depression,” says
Siobhán Murray, a psychotherapist based in County Dublin, Ireland, and the author of a book
about burnout, The Burnout Solution. Murray suggests looking out for creeping bad habits, such
as increased alcohol consumpution and relying on sugar to get you through the day. Also watch
out for feelings of tiredness that won’t go away. “So that even if you do sleep well, by
10 in the morning you’re already counting down the hours to bed. Or not having the energy to
exercise or go for a walk.”
As soon as you begin to feel this way, Murray advises going to see your doctor.
“Depression and pre-burnout are very similar, but as much as there was a lot of enthusiasm
recently that burnout has now become a medical condition, it is still not – it is still classified as
an occupational phenomenon.” It’s important to get help from a medical professional who can
distinguish between the two, because although there are many treatment options for
depression, burnout is still best tackled by making lifestyle changes.
And how do you know if you’re really on the cusp of burnout, or just going through a challenging
month? “Stress is really important, and anxiety is what motivates us to do well,” says Murray.
“It’s when we’re continually exposed to stress and anxiety, that we’re not letting go, that it
starts to turn into burnout.”
Take that big project you’ve been working on. It’s normal to feel a kick of adrenaline when you
think about it, and maybe it’s kept you up at night. But, Murray suggests, if you still feel restless
once it’s over, it’s time to consider if you’re at risk of burnout. “It’s when you’re bringing that
with you into the next stage of your day, and adding to it continually,” she says.
Another classic sign of inching closer to burnout is cynicism: feeling like your work has little
value, avoiding social commitments and becoming more susceptible to disappointment.
“Someone on the brink will probably begin to feel emotionally numbed or mentally distant,”
says Jacky Francis Walker, a psychotherapist based in London who specialises in burnout. “Like
they don’t have the capacity to engage as much in the ordinary things of life.”

AULA 01 – SKIMMING AND SCANNING AND OTHER TECHNIQUES. IDENTIFYING FALSE COGNATS. 139
TEACHER ANDREA BELO

She also recommends looking for the final tell-tale sign of burnout, which is the unshakeable
feeling that the quality of your work is beginning to slip. “People say ‘but this isn’t me!’, ‘I’m not
like this’, ‘I can usually do x,y and z’. But obviously if they are in a state of physical depletion,
then they aren’t in their normal range of capabilities,” says Walker.
If this seems less than scientific, look to the Maslach Burnout Inventory (MBI), a test designed to
measure burnout. The most widely used is the MBI-General Survey, which measures things like
exhaustion, cynicism, and some how well you think you’re doing at work.
First published in 1981, it has been cited hundreds of times in studies since. Although it’s
typically used to measure burnout once it's in full swing, there’s no reason you can't apply it to
see if you’re getting close.
(Adapted from https://www.bbc.com/worklife/article/20190610-how-to-tell-if-youve-got-pre-burnout)

A OMS redefiniu o burnout como uma síndrome ligada ao estresse laboral crônico. Há uma
diferença entre uma carga de trabalho ocupada e algo mais sério, escreve Zaria Gorvett.
Se você disse que estava sofrendo de "esgotamento" no início da década de 1970, pode ter
levantado algumas sobrancelhas.
Na época, o termo era usado informalmente para descrever os efeitos colaterais que os usuários
pesados de drogas experimentavam: o escurecimento geral das faculdades mentais, por
exemplo, como era o caso de muitos animais festeiros. No entanto, quando o psicólogo
germano-americano Herbert Freudenberger reconheceu pela primeira vez o problema do
esgotamento na cidade de Nova York em 1974, em uma clínica para viciados e sem-teto,
Freudenberger não estava pensando em usuários de drogas.
Os voluntários da clínica também estavam lutando: seu trabalho era intenso e muitos estavam
começando a se sentir desmotivados e emocionalmente esgotados. Embora antes achassem
seus empregos gratificantes, eles se tornaram cínicos e deprimidos; eles não estavam dando aos
seus pacientes a atenção que eles mereciam. Freudenberger definiu essa nova condição
alarmante como um estado de exaustão causado por excesso de trabalho prolongado - e tomou
emprestado o termo "esgotamento" para descrevê-lo.
Sua popularidade foi explosiva e hoje o esgotamento é um fenômeno global. Embora as
estatísticas sobre a prevalência de burnout especificamente sejam difíceis de obter, 595.000
pessoas só no Reino Unido sofreram de estresse no local de trabalho em 2018.
Os desportistas entendem. As estrelas do YouTube entendem. Os empresários entendem. O
próprio Freudenberger finalmente entendeu. No final do mês passado, a Organização Mundial
da Saúde (OMS) anunciou que o problema da moda será reconhecido no mais recente manual
de Classificação Internacional de Doenças, onde é descrito como uma síndrome “resultante de
estresse crônico no local de trabalho que não foi administrado com sucesso”.
De acordo com a OMS, o burnout tem três elementos: sensação de exaustão, desligamento
mental do trabalho e pior desempenho no trabalho. Mas esperar até que você já esteja
totalmente exausto para fazer algo sobre isso não ajuda em nada - e você não esperaria para
tratar qualquer outra doença até que fosse tarde demais.
Sentindo a queimadura

AULA 01 – SKIMMING AND SCANNING AND OTHER TECHNIQUES. IDENTIFYING FALSE COGNATS. 140
TEACHER ANDREA BELO

Então, como você pode saber se você está quase - mas não totalmente - esgotado?
“Muitos dos sinais e sintomas do pré-burnout seriam muito semelhantes à depressão”, diz
Siobhán Murray, psicoterapeuta de County Dublin, Irlanda, e autor de um livro sobre burnout,
The Burnout Solution. Murray sugere que você tome cuidado com os hábitos ruins, como o
aumento do consumo de álcool e depender do açúcar para sobreviver ao dia. Também esteja
atento para a sensação de cansaço que não passa. "Para que, mesmo que você durma bem, por
10 da manhã você já está contando as horas para dormir. Ou não ter energia para fazer
exercícios ou dar um passeio ”.
Assim que você começar a se sentir assim, Murray o aconselha a consultar seu médico.
“Depressão e pré-esgotamento são muito semelhantes, mas por mais que tenha havido muito
entusiasmo recentemente de que o esgotamento se tornou uma condição médica, ainda não é -
ainda é classificado como um fenômeno ocupacional.” É importante obter ajuda de um
profissional médico que possa distinguir entre os dois, porque embora existam muitas opções
de tratamento para a depressão, o burnout ainda é melhor abordado com mudanças no estilo
de vida.
E como saber se você está realmente à beira do esgotamento ou apenas passando por um mês
desafiador? “O estresse é muito importante e a ansiedade é o que nos motiva a nos sair bem”,
diz Murray. “É quando estamos continuamente expostos ao estresse e ansiedade, que não
estamos relaxando, que começa a se transformar em esgotamento.”
Pegue aquele grande projeto em que você está trabalhando. É normal sentir uma descarga de
adrenalina quando você pensa sobre isso, e talvez isso o mantenha acordado à noite. Mas,
Murray sugere, se você ainda se sentir inquieto depois que acabar, é hora de considerar se você
corre o risco de esgotar-se. “É quando você está trazendo isso com você para a próxima fase do
seu dia e aumentando continuamente”, diz ela.
Outro sinal clássico de se aproximar do esgotamento é o cinismo: sentir que seu trabalho tem
pouco valor, evitando compromissos sociais e se tornando mais suscetível a decepções.
“Alguém à beira do abismo provavelmente começará a se sentir emocionalmente entorpecido
ou mentalmente distante”, diz Jacky Francis Walker, psicoterapeuta de Londres especializado
em burnout. “Como se eles não tivessem a capacidade de se envolver tanto nas coisas comuns
da vida.”
Ela também recomenda procurar o sinal revelador final de esgotamento, que é a sensação
inabalável de que a qualidade do seu trabalho está começando a cair. “As pessoas dizem 'mas
este não sou eu!', 'Não sou assim', 'Normalmente consigo fazer x, y e z'. Mas, obviamente, se
eles estão em um estado de esgotamento físico, então eles não estão em sua gama normal de
capacidades ”, diz Walker.
Se isso parecer menos do que científico, consulte o Maslach Burnout Inventory (MBI), um teste
desenvolvido para medir o burnout. O mais amplamente utilizado é o MBI-General Survey, que
mede coisas como exaustão, cinismo, um descubra se você acha que está indo bem no trabalho.
Publicado pela primeira vez em 1981, ele foi citado centenas de vezes em estudos desde então.
Embora seja normalmente usado para medir o desgaste, uma vez que está em pleno

AULA 01 – SKIMMING AND SCANNING AND OTHER TECHNIQUES. IDENTIFYING FALSE COGNATS. 141
TEACHER ANDREA BELO

andamento, não há razão para que você não possa aplicá-lo para ver se você está chegando
perto.

Brazilian Army Joins Effort to Disinfect Prison from COVID-19


By VOA News
The Brazilian army, in conjunction with the country's prison administration, conducts a large-
scale disinfection operation to slow the spread of the coronavirus at the Gericinó Prison Complex
in Rio de Janeiro. A prison spokesman says soldiers outfitted in full body protection suits and
face masks clean the infirmary, coronavirus isolation cells, administrative areas, and visitor
sections of the prison. Colonel Rego Barros of the army's East Joint Command says the proactive
measures are taken after seeing situations in which inmates become infected by COVID-19 in
other states.
So far, Brazil has more than 1,000,000 covid-19 cases and almost 50,000 deaths. Peru has the
second highest coronavirus case total in Latin American behind Brazil. Peru's National

AULA 01 – SKIMMING AND SCANNING AND OTHER TECHNIQUES. IDENTIFYING FALSE COGNATS. 142
TEACHER ANDREA BELO

Penitentiary Institute confirms reports of inmates setting fires and demanding to be freed, as
more prisoners become infected with the disease. Reuters news agency says human rights
groups are calling on the Peruvian government to allow house arrest during the pandemic.
(Adapted from https://www.voanews.com/covid-19-pandemic/brazilian-army-joins-effort-disinfect-prison-covid-19)

Exército Brasileiro une esforços para desinfetar a prisão da COVID-19


Por VOA News
O Exército brasileiro, em conjunto com a administração penitenciária do país, conduz uma
operação de desinfecção em grande escala para retardar a disseminação do coronavírus no
Complexo Penitenciário de Gericinó, no Rio de Janeiro. Um porta-voz da prisão disse que
soldados vestidos com roupas de proteção de corpo inteiro e máscaras limpam a enfermaria, as
células de isolamento do coronavírus, as áreas administrativas e as seções de visitantes da
prisão. O coronel Rego Barros, do Comando Conjunto Leste do Exército, afirma que as medidas
proativas são tomadas após constatar situações em que presos são infectados pelo COVID-19 em
outros estados.
Até agora, o Brasil tem mais de 1.000.000 de casos covid-19 e quase 50.000 mortes. O Peru tem
o segundo maior total de casos de coronavírus na América Latina, atrás apenas do Brasil. O
Instituto Penitenciário Nacional do Peru confirma relatos de presos que atearam fogo e exigiram
sua libertação, conforme mais presos foram infectados com a doença. A agência de notícias
Reuters afirma que grupos de direitos humanos estão pedindo ao governo peruano que permita
a prisão domiciliar durante a pandemia.

Seville Hospital uses an experimental treatment against SARS-CoV-2 with successful results,
combining lopinavir and ritonavir antivirals and interferon beta.
By Richard Mann – March 1, 2020
RIO DE JANEIRO, BRAZIL - The application of lopinavir/ritonavir, also used to fight HIV infections,
along with interferon beta, a protein that helps cells protect themselves from infection,
confirmed medical sources to EL PAÍS.
"It's an experimental course of treatment that has yielded positive results against other viruses,"
explains Albert Bosch, president of the Spanish Society of Virology. "One of its greatest
advantages is that it is an approved drug used in other indications, so there is no question about
its safety," he adds.
(Adapted from https://riotimesonline.com/brazil-news/brazil/brazil-declares-state-of-emergency-for-47-cities-due-to-rains/)

AULA 01 – SKIMMING AND SCANNING AND OTHER TECHNIQUES. IDENTIFYING FALSE COGNATS. 143
TEACHER ANDREA BELO

O Hospital de Sevilha usa um tratamento experimental contra o SARS-CoV-2 com resultados


bem-sucedidos, combinando antivirais lopinavir e ritonavir e interferon beta.
Por Richard Mann - 1º de março de 2020
RIO DE JANEIRO, BRASIL - A aplicação de lopinavir / ritonavir, também usado no combate às
infecções pelo HIV, junto com o interferon beta, proteína que ajuda as células a se protegerem
da infecção, confirmaram fontes médicas do EL PAÍS.
"É um curso experimental de tratamento que produziu resultados positivos contra outros vírus",
explica Albert Bosch, presidente da Sociedade Espanhola de Virologia. “Uma de suas maiores
vantagens é ser um medicamento aprovado e usado em outras indicações, portanto não há
dúvidas sobre sua segurança”, acrescenta.

The current pandemic is forcing much of the world’s population to stay at home and take it
easy. But we may not be wired that way.
You may be among the more than three million people who’ve seen a short film issued by the
Governor of California’s office. It’s all over social media. In the video, the comedian Larry David,
in his trademark sardonic style, urges people to follow the official advice and stay at home to
stop the spread of Covid-19. What’s the matter with you "idiots", he says, you’re passing up a
fantastic opportunity to sit in an armchair and watch TV all day!
We’re used to health warnings that urge us to do things that we don’t really have a great urge to
do: to exercise more, to eat five or eight or even 10 portions of fruit and vegetables a day. But
for once the official advice sounds easy; loaf on the sofa, binge-watch box sets, stay at home.
This all sounds as though it should appeal to our lazier sides.

AULA 01 – SKIMMING AND SCANNING AND OTHER TECHNIQUES. IDENTIFYING FALSE COGNATS. 144
TEACHER ANDREA BELO

In fact, it is not as simple as that, as you’ve probably already found out, after a few weeks of
lockdown. It turns out we are not biologically programmed to do as little as possible. Indeed, we
thrive on activity. Or at least, a good balance between being busy and being able to rest.
It’s true that we often look for the easy option, the path of least resistance, the shortcut to
success. If you have a remote control, why get up and switch channels on the TV itself? If you
have a car why cycle to the supermarket? If you can get away with doing half as much work than
a colleague, then why not?
Any sort of work or effort involves mental and physical strain, so it makes sense to avoid it
where possible. And sometimes we do just that. This is sometimes known as the principle of
least effort or Zipf’s Law, a law you might think no one is ever tempted to break. Except that we
break it all the time.
Do you ever dream of doing absolutely nothing? Lying in a hammock for a whole afternoon. Just
staring at the ceiling, listening to the silence. It may sound like a lovely idea, but in fact we can
find doing nothing at all – and take sleep out of the equation – very hard to do. In a famous
study conducted a few years ago at the University of Virginia, participants were led one at a
time into a completely bare room with all distractions removed. They had no phone, no books,
no screens – and they weren’t allowed to take a nap. Electrodes were fitted to their ankles and
they were left alone for 15 minutes. It was an opportunity to kick back and relax for a short
while.
So, how did it go? Well, before being left alone, participants were shown how to press a
computer key which was wired up to a machine that delivered an electric shock. You might
suppose that having tried it once no one would want to do it again. Wrong. In fact, 71% of the
men and 25% of the women gave themselves at least one electric shock during their time in
solitary – and one man shocked himself a shocking 190 times.
It turns out that having nothing to do was so excruciating, that many of the participants
preferred to, in effect, torture themselves rather than put up with no distractions whatsoever.
This experiment is an extreme example, but we know from everyday life that people constantly
choose to do things they don’t need to do and which are sometimes painful. Think of all of your
friends who run marathons or have punishing regimes at the gym.
They go way beyond what is required for health and fitness. And what about the people who
trek across the ice to reach the poles of the Earth or sail around the world?
Michael Inzlicht from the University of Toronto calls this the paradox of effort. Sometimes we
take the easy route and do as little as we can get away with, but at other times we value
situations more if we have to expend considerable effort. The intrinsic joy of the effort gives us
so much pleasure that we don’t take the short cut. We might spend hours puzzling over a cryptic
crossword instead of using a search engine to find the solution.
We learn this early in life. As children we are taught through experience and persuasion that
effort leads to reward and over time this conditions us to enjoy effort for its own sake. This is
known as learned industriousness.
All of this means, that while we stay home and self-isolate, lying on the sofa and watching TV
will form only a part of how we pass the time. We might think it is fun to laze around for a few

AULA 01 – SKIMMING AND SCANNING AND OTHER TECHNIQUES. IDENTIFYING FALSE COGNATS. 145
TEACHER ANDREA BELO

weeks, but in fact it will drive us to distraction. Enforced and extended rest, unless we are ill and
our bodies demand it, leads not to feelings of being relaxed but of restlessness and irritability.
We need to find ways during lockdown to replicate as far as we can the rhythms and sense of
balance we achieve, at our best, in ordinary life.
So, taking exercise, setting ourselves tasks, doing things that are effortful and difficult are
important. And we should all be looking for activities or experiences that promote what the
psychologist Mihaly Csikszentmihalyi calls flow, in his book Flow: The Psychology of Optimal
Experience. These are tasks, such as painting or gardening or doing jigsaws, which so absorb us
that we don’t notice time passing and we stop worrying about everything else.
In normal times, most of us don’t take rest seriously enough. So during this exceptional period,
we should embrace the opportunity to rest more if we can – and indeed take those more
balanced rhythms of rest and busyness into our lives post shutdown. But during this difficult
time, we will find that we are not instinctively lazy creatures.
Indeed in a weird way we might find that doing less, and resting more, initially requires quite a
lot of effort.
(Adapted from: https://www.bbc.com/future/article/20200602-are-human-beings-naturally-lazy)

A atual pandemia está forçando grande parte da população mundial a ficar em casa e descansar.
Mas podemos não estar conectados dessa forma.
Você pode estar entre os mais de três milhões de pessoas que viram um curta-metragem
lançado pelo gabinete do governador da Califórnia. Está tudo nas redes sociais. No vídeo, o
comediante Larry David, em seu estilo sarcástico, exorta as pessoas a seguirem os conselhos
oficiais e a ficarem em casa para impedir a disseminação do Covid-19. Qual é o problema de
vocês, "idiotas", ele diz, vocês estão perdendo uma oportunidade fantástica de sentar em uma
poltrona e assistir TV o dia todo!
Estamos acostumados a advertências de saúde que nos incitam a fazer coisas que realmente
não temos uma grande vontade de fazer: fazer mais exercícios, comer cinco ou oito ou até dez
porções de frutas e vegetais por dia. Mas, pela primeira vez, o conselho oficial parece fácil; pão
no sofá, jogos de caixinhas de relojoaria, fique em casa. Tudo isso soa como se devesse atrair
nossos lados mais preguiçosos.
Na verdade, não é tão simples assim, como você provavelmente já descobriu, após algumas
semanas de bloqueio. Acontece que não somos biologicamente programados para fazer o
mínimo possível. Na verdade, prosperamos na atividade. Ou, pelo menos, um bom equilíbrio
entre estar ocupado e ser capaz de descansar.
É verdade que muitas vezes procuramos a opção fácil, o caminho de menor resistência, o atalho
para o sucesso. Se você tem um controle remoto, por que mudar de canal na própria TV? Se
você tem um carro, por que ir de bicicleta ao supermercado? Se você consegue fazer metade do
trabalho de um colega, por que não?
Qualquer tipo de trabalho ou esforço envolve desgaste físico e mental, portanto, faz sentido
evitá-lo sempre que possível. E às vezes fazemos exatamente isso. Isso às vezes é conhecido

AULA 01 – SKIMMING AND SCANNING AND OTHER TECHNIQUES. IDENTIFYING FALSE COGNATS. 146
TEACHER ANDREA BELO

como o princípio do menor esforço ou Lei de Zipf, uma lei que você pode pensar que ninguém se
sente tentado a violar. Exceto que quebramos o tempo todo.
Você já sonhou em não fazer absolutamente nada? Deitado em uma rede por uma tarde inteira.
Apenas olhando para o teto, ouvindo o silêncio. Pode parecer uma ideia adorável, mas na
verdade podemos achar que não fazer absolutamente nada - e tirar o sono da equação - é muito
difícil de fazer. Em um famoso estudo realizado há alguns anos na Universidade da Virgínia, os
participantes foram conduzidos um de cada vez a uma sala completamente vazia, sem todas as
distrações. Eles não tinham telefone, nem livros, nem telas - e não tinham permissão para tirar
uma soneca. Eletrodos foram colocados em seus tornozelos e eles foram deixados sozinhos por
15 minutos. Foi uma oportunidade para descontrair e relaxar um pouco.
Então, como foi? Bem, antes de serem deixados sozinhos, os participantes aprenderam a
pressionar uma tecla de computador conectada a uma máquina que aplicou um choque elétrico.
Você pode supor que, depois de tentar uma vez, ninguém iria querer fazê-lo novamente. Errado.
Na verdade, 71% dos homens e 25% das mulheres aplicaram-se a pelo menos um choque
elétrico durante o tempo na solitária - e um homem se chocou 190 vezes.
Acontece que não ter nada para fazer era tão doloroso que muitos dos participantes preferiam,
de fato, torturar-se em vez de tolerar nenhuma distração.
Este experimento é um exemplo extremo, mas sabemos da vida cotidiana que as pessoas
constantemente escolhem fazer coisas que não precisam fazer e que às vezes são dolorosas.
Pense em todos os seus amigos que correm maratonas ou têm regimes punitivos na academia.
Eles vão muito além do que é necessário para a saúde e a boa forma. E as pessoas que viajam
pelo gelo para chegar aos polos da Terra ou velejar ao redor do mundo?
Michael Inzlicht, da Universidade de Toronto, chama isso de paradoxo do esforço. Às vezes,
escolhemos o caminho mais fácil e fazemos o mínimo possível, mas outras vezes valorizamos
mais as situações se tivermos de despender um esforço considerável. A alegria intrínseca do
esforço nos dá tanto prazer que não tomamos o atalho. Podemos passar horas confundindo
palavras cruzadas enigmáticas em vez de usar um mecanismo de busca para encontrar a
solução.
Aprendemos isso cedo na vida. Quando crianças, somos ensinados por meio da experiência e da
persuasão que o esforço leva à recompensa e, com o tempo, isso nos condiciona a desfrutar o
esforço por si mesmo. Isso é conhecido como laboriosidade aprendida.
Tudo isso significa que, enquanto ficarmos em casa e nos isolarmos, deitar no sofá e assistir TV
será apenas uma parte de como passamos o tempo. Podemos pensar que é divertido preguiçar
por algumas semanas, mas na verdade isso nos distrairá. O repouso forçado e prolongado, a
menos que estejamos doentes e o nosso corpo o exija, não conduz à sensação de relaxamento,
mas de inquietação e irritabilidade.
Precisamos encontrar maneiras durante o bloqueio para replicar, tanto quanto pudermos, os
ritmos e o senso de equilíbrio que alcançamos, da melhor maneira possível, na vida cotidiana.
Então, fazer exercícios, definir tarefas para nós mesmos, fazer coisas que são difíceis e difíceis
são importantes. E todos devemos estar procurando por atividades ou experiências que
promovam o que o psicólogo Mihaly Csikszentmihalyi chama de fluxo, em seu livro Flow: The

AULA 01 – SKIMMING AND SCANNING AND OTHER TECHNIQUES. IDENTIFYING FALSE COGNATS. 147
TEACHER ANDREA BELO

Psychology of Optimal Experience. São tarefas, como pintar, fazer jardinagem ou fazer quebra-
cabeças, que nos absorvem tanto que não percebemos o tempo passando e deixamos de nos
preocupar com todo o resto.
Em tempos normais, a maioria de nós não leva o descanso a sério o suficiente. Portanto,
durante esse período excepcional, devemos aproveitar a oportunidade de descansar mais, se
pudermos - e, de fato, levar esses ritmos mais equilibrados de descanso e ocupação para nossas
vidas após o desligamento. Mas durante esse período difícil, descobriremos que não somos
criaturas instintivamente preguiçosas.
De fato, de uma forma estranha, podemos descobrir que fazer menos e descansar mais
inicialmente requer muito esforço.

Social Integration
Social integration is ______ actual participation in various social relationships, ranging ______
romantic partnerships to friendships.
This integration involves emotions, intimacy, and a sense of belonging ______ different social
groups, such as being part of a family, a partnership, a social activity, or a religious community.
Experts suggest that ______ integrated into such social relationships confers a protective
benefit against maladaptive behaviors and damaging health consequences.
(Adapted from https://www.verywellmind.com/social-support-for-psychological-health-4119970)

Integração social

AULA 01 – SKIMMING AND SCANNING AND OTHER TECHNIQUES. IDENTIFYING FALSE COGNATS. 148
TEACHER ANDREA BELO

A integração social é ______ a participação real em várias relações sociais, variando de ______
parcerias românticas a amizades.
Essa integração envolve emoções, intimidade e um sentimento de pertencer a ______ grupos
sociais diferentes, como ser parte de uma família, uma parceria, uma atividade social ou uma
comunidade religiosa. Os especialistas sugerem que ______ integrado em tais relações sociais
confere um benefício protetor contra comportamentos inadequados e consequências
prejudiciais à saúde.

Latin America's largest country is testing people ______ a rate far lower than any other nation
with at least 40,000 cases. It tests 12 times fewer people ______ Iran, and 32 times fewer
______ the United States. Hospitalized patients aren't being tested. Some medical professionals
aren't being tested. People are dying ______ their homes without being tested.
(Adapted from https://www.washingtonpost.com/world/the_americas/coronavirus-brazil-testing-bolsonaro-cemetery-gravedigger)

O maior país da América Latina está testando pessoas ______ uma taxa muito menor do que
qualquer outra nação com pelo menos 40.000 casos. Ele testa 12 vezes menos pessoas ______
no Irã e 32 vezes menos ______ nos Estados Unidos. Pacientes hospitalizados não estão sendo
testados. Alguns profissionais médicos não estão sendo testados. As pessoas estão morrendo
______ em suas casas sem serem testadas.

What Happened When Hong Kong’s Schools Went Virtual to Combat the Spread of
Coronavirus
Because of the coronavirus outbreak, all schools in Hong Kong have been closed since January,
and won’t reopen until late April at the earliest. "The exact date of class resumption is subject to
further assessment," announced the Education Bureau, which controls all schools in Hong Kong,
public and private, on February 25. It’s all part of the “social distancing” measures the city has
mandated to slow the virus’s spread, which include closing libraries, museums and recreation
facilities like pools. Students from preschoolers through PhD candidates are now doing all their
education online, a move the Education Bureau calls "suspending classes without suspending
learning."

AULA 01 – SKIMMING AND SCANNING AND OTHER TECHNIQUES. IDENTIFYING FALSE COGNATS. 149
TEACHER ANDREA BELO

As coronavirus spreads across the globe, other countries are joining Hong Kong and mainland
China in this massive, unplanned experiment in online learning. According to Unesco, as of
Friday, 14 countries have shut schools down nationwide, affecting upwards of 290 million
students, while 13 countries, including the United States, have seen localized school closings.
In recent days, schools from Scarsdale, New York, to San Francisco have closed temporarily over
contagion concerns. The University of Washington and Stanford University have turned to online
classes for the remainder of the quarter, and others are following suit for various lengths of time.
Some experts believe more widespread and long-term closures will be necessary in areas with
high levels of community transmission. States are preparing for that possibility by looking at
their own online learning policies.
Adapted from https://www.smithsonianmag.com/innovation/what-can-americans-learn-from-hong-kongs-unplanned-experiment-online-learning-during-coronavirus-180974331/

O que aconteceu quando as escolas de Hong Kong se tornaram virtuais para combater a propagação do
coronavírus
Por causa do surto de coronavírus, todas as escolas em Hong Kong estão fechadas desde janeiro e não
vão reabrir até o final de abril, no mínimo. "A data exata de retomada das aulas está sujeita a uma
avaliação mais detalhada", anunciou o Education Bureau, que controla todas as escolas em Hong Kong,
públicas e privadas, em 25 de fevereiro. Tudo isso faz parte das medidas de "distanciamento social" que
a cidade determinou desacelerar a propagação do vírus, que inclui o fechamento de bibliotecas, museus
e instalações recreativas como piscinas. Alunos de pré-escolares a candidatos a doutorado agora estão
fazendo toda a sua educação online, um movimento que o Education Bureau chama de "suspender as
aulas sem suspender o aprendizado".
À medida que o coronavírus se espalha pelo globo, outros países estão se juntando a Hong Kong e à
China continental neste experimento massivo e não planejado de aprendizagem online. De acordo com a
Unesco, até sexta-feira, 14 países fecharam escolas em todo o país, afetando mais de 290 milhões de
alunos, enquanto 13 países, incluindo os Estados Unidos, viram escolas fechadas.
Nos últimos dias, escolas de Scarsdale, Nova York, a San Francisco fecharam temporariamente devido a
preocupações com contágio. A University of Washington e a Stanford University optaram por aulas
online pelo restante do trimestre, e outras estão seguindo o exemplo por vários períodos. Alguns
especialistas acreditam que fechamentos mais generalizados e de longo prazo serão necessários em
áreas com altos níveis de transmissão na comunidade. Os estados estão se preparando para essa
possibilidade examinando suas próprias políticas de aprendizagem online.

The Bod Pod


It’s a small, yet ordinary office space tucked around the corner of a lobby. It’s exactly how you
would think a medical building would look, but in this office, there is a piece of equipment that
looks similar to a space escape pod from a sci-fi movie. Despite its looks, its purpose has nothing
to do with space. The Body Composition Machine, or commonly referred to as the “Bod Pod”, is
an instrument that helps determine total body fat percentage, lean muscle mass as well as
resting metabolic rate and total energy expenditure of an individual.
“It’s state of the art,” said Alan Muriera, 460th Medical Group health promotion manager. “The
Bod Pod results are around 99.5% accurate. It is tested and calibrated everyday. We check the
seals, we check the airway valves, and we check the volume.” Muriera works in the Human

AULA 01 – SKIMMING AND SCANNING AND OTHER TECHNIQUES. IDENTIFYING FALSE COGNATS. 150
TEACHER ANDREA BELO

Performance Center on Buckley Air Force Base, Colo., where he administers the Bod Pod
examinations. The Bod Pod is a computerized, egg-shaped chamber which uses the same whole-
body measurement principle as underwater weighing, except it utilizes air to measure your total
body fat. As air is pumped into the chamber, body mass is calculated by the amount of air your
body displaces.
“When people come here, they are either looking to lose weight, or to see muscle gain,” said
Muriera. The Bod Pod is an opportunity for you to acquire baseline data prior to initiating a new
workout or diet regimen. The analysis takes only five minutes to accomplish and is non-invasive.
Muriera explains, “I also like to call it the “Truth Booth”, because I’ve had people come in
thinking they have 15% total body fat, and they are actually 26%.” This machine is accurate and
produces results with a technologically advanced system that will show minute details. “It was
definitely a humbling experience,” said Chief Master Sgt. Robert Devall, 460th Space Wing
command chief. “I don’t think I’m in bad shape, but I wasn’t quite where I wanted to be after
seeing the results.”
Adapted from https://www.buckley.af.mil/News/Article-Display/Article/2105959/the-bod-pod/

The Bod Pod


É um espaço de escritório pequeno, mas comum, dobrado na esquina de um saguão. É
exatamente como você pensaria que um edifício médico seria, mas neste escritório, há um
equipamento que se parece com uma cápsula de fuga espacial de um filme de ficção científica.
Apesar de sua aparência, seu propósito não tem nada a ver com espaço. A Body Composition
Machine, ou comumente conhecida como “Bod Pod”, é um instrumento que ajuda a determinar
o percentual de gordura corporal total, a massa muscular magra, bem como a taxa metabólica
de repouso e o gasto energético total de um indivíduo.
“É o que há de mais moderno”, disse Alan Muriera, gerente de promoção da saúde do 460th
Medical Group. “Os resultados do Bod Pod são cerca de 99,5% precisos. Ele é testado e calibrado
todos os dias. Verificamos os selos, verificamos as válvulas das vias aéreas e verificamos o
volume. ”

Muriera trabalha no Human Performance Center na Buckley Air Force Base, Colorado, onde
administra os exames Bod Pod. O Bod Pod é uma câmara computadorizada em forma de ovo
que usa o mesmo princípio de medição de corpo inteiro da pesagem subaquática, exceto que
utiliza ar para medir sua gordura corporal total. Conforme o ar é bombeado para a câmara, a
massa corporal é calculada pela quantidade de ar que seu corpo desloca.
“Quando as pessoas vêm aqui, elas procuram perder peso ou ver ganho de massa muscular”,
disse Muriera. O Bod Pod é uma oportunidade para você adquirir dados básicos antes de iniciar
um novo treino ou regime de dieta. A análise leva apenas cinco minutos para ser realizada e não
é invasiva. Muriera explica: “Também gosto de chamá-lo de“ Cabine da Verdade ”, porque

AULA 01 – SKIMMING AND SCANNING AND OTHER TECHNIQUES. IDENTIFYING FALSE COGNATS. 151
TEACHER ANDREA BELO

algumas pessoas chegaram pensando que tinham 15% de gordura corporal total e, na verdade,
eram 26%”. Esta máquina é precisa e produz resultados com um sistema tecnologicamente
avançado que mostra os detalhes minuciosos. “Foi definitivamente uma experiência
humilhante”, disse o sargento-chefe. Robert Devall, chefe do comando 460th Space Wing. “Não
acho que estou mal, mas não estava exatamente onde queria estar depois de ver os resultados.”

The US could see more deaths than WWII. It's time to deploy the military to hospitals
We are at war against the coronavirus. If we don’t act rapidly, we’re approaching a US death toll
larger than that of the two world wars combined. To fight an invisible army, we need to be
creative. We must deploy the full might of the US military – in an unconventional way. Unlike
most wars, this time the frontlines are staffed by our nation’s healthcare workers. And while
they’re fighting valiantly so far, the battle to flatten the curve is leaving them overrun.
In the US, we don’t have a centralized economy. Most of our major cities will have their hospital
systems overwhelmed simultaneously. And there will be few doctors to spare. So, what should
we do? To start, we should add medical students and retired doctors to the frontlines. But these

AULA 01 – SKIMMING AND SCANNING AND OTHER TECHNIQUES. IDENTIFYING FALSE COGNATS. 152
TEACHER ANDREA BELO

groups will fill only a small fraction of the added medical demand. We must rapidly train and
deploy a much larger workforce.
Let’s use one of America’s most valuable national assets – our top-tier military – to dramatically
increase our healthcare capacity. Members are trained to handle acute stress and are rapidly
deployable. They’re used to being under centralized command and are mentally prepared to be
away from their families. We should rapidly reassign any military doctors and paramedics that
can be spared to serve domestic needs. We don’t need to militarize our hospitals. But our
hospitals will need help from our military. This is an unprecedented crisis, and we need
unprecedented adaptability to save lives.
Adapted from https://www.theguardian.com/commentisfree/2020/mar/19/us-military-coronavirus-hospitals

Os EUA podem ver mais mortes do que a Segunda Guerra Mundial. É hora de enviar militares
para hospitais
Estamos em guerra contra o coronavírus. Se não agirmos rapidamente, estaremos nos aproximando de
um número de mortos nos Estados Unidos maior do que o das duas guerras mundiais juntas. Para lutar
contra um exército invisível, precisamos ser criativos. Devemos implantar todo o poder dos militares dos
EUA - de uma forma não convencional. Ao contrário da maioria das guerras, desta vez as linhas de frente
são compostas por profissionais de saúde de nosso país. E enquanto eles estão lutando bravamente até
agora, a batalha para aplainar a curva está deixando-os derrotados.
Nos EUA, não temos uma economia centralizada. A maioria de nossas grandes cidades terá seus sistemas
hospitalares sobrecarregados simultaneamente. E haverá poucos médicos disponíveis. Então o que
deveríamos fazer? Para começar, devemos colocar estudantes de medicina e médicos aposentados na
linha de frente. Mas esses grupos atenderão apenas a uma pequena fração da demanda médica
adicional. Devemos treinar e implantar rapidamente uma força de trabalho muito maior.
Vamos usar um dos ativos nacionais mais valiosos da América - nossas forças armadas de alto nível - para
aumentar drasticamente nossa capacidade de saúde. Os membros são treinados para lidar com o
estresse agudo e são rapidamente implantados. Eles estão acostumados a estar sob comando
centralizado e estão mentalmente preparados para ficar longe de suas famílias. Devemos reatribuir
rapidamente quaisquer médicos militares e paramédicos que possam ser dispensados para atender às
necessidades domésticas. Não precisamos militarizar nossos hospitais. Mas nossos hospitais precisarão
da ajuda de nossos militares. Esta é uma crise sem precedentes e precisamos de uma adaptabilidade
sem precedentes para salvar vidas.

Is technology making time go quicker?


It never feels like there are enough hours in the day. If someone __________ (1) you a button to
get things done at twice the speed to free up time, you’d probably jump at it. In November,
Netflix introduced a feature to let users fast-forward their shows, playing at 1.5x speed. People
are already watching university lectures and their favourite YouTubers on double time and those
that speed-listen to podcasts – podfasters – are commonplace.
But fast forward __________ (2) an option since the days of VHS tapes. What’s so different now?
In 2018, Netflix released 1,500 hours of original content and to watch it all, it would take you

AULA 01 – SKIMMING AND SCANNING AND OTHER TECHNIQUES. IDENTIFYING FALSE COGNATS. 153
TEACHER ANDREA BELO

four hours of streaming a day for a whole year. With the same number of hours in the day,
speed-watching seems a logical solution, for both viewers and the streaming giants, to the
information and entertainment glut.
‘The brain is very capable of adapting to changes in experience,’ says neuroscientist Prof Edvard
Moser. Yet time is different from other sense data we adapt to, like sight and smell. The body
doesn’t have in-built sensors to measure time the way a clock does. It may __________ (3) like
we can sense time. But that’s a helpful illusion. Regardless of how time feels, there’s nothing
stopping us watching and consuming information at faster speeds than we do currently. ‘But
maybe the pleasure or enjoyment wouldn’t be the same.’
Adapted from https://metro.co.uk/2020/01/15/is-technology-making-time-go-quicker-12057474/

A tecnologia está fazendo o tempo passar mais rápido?


Nunca parece que há horas suficientes no dia. Se alguém __________ (1) você um botão para
fazer as coisas com o dobro da velocidade para liberar tempo, você provavelmente vai saltar
para ele. Em novembro, a Netflix lançou um recurso para permitir que os usuários avancem
rapidamente em seus programas, tocando a uma velocidade de 1,5x. As pessoas já estão
assistindo a palestras universitárias e seus YouTubers favoritos em tempo dobrado e aqueles que
ouvem podcasts rapidamente - podfasters - são comuns.
Mas avançar __________ (2) uma opção desde os dias das fitas VHS. O que há de tão diferente
agora? Em 2018, a Netflix lançou 1.500 horas de conteúdo original e, para assistir a tudo, levaria
quatro horas de streaming por dia durante um ano inteiro. Com o mesmo número de horas no
dia, assistir a velocidade parece uma solução lógica, tanto para os telespectadores quanto para
os gigantes do streaming, para o excesso de informação e entretenimento.
"O cérebro é muito capaz de se adaptar às mudanças na experiência", diz o neurocientista Prof
Edvard Moser. No entanto, o tempo é diferente de outros dados sensoriais aos quais nos
adaptamos, como a visão e o olfato. O corpo não tem sensores embutidos para medir o tempo
da mesma forma que um relógio. Pode __________ (3) como podemos sentir o tempo. Mas isso
é uma ilusão útil. Independentemente de como o tempo parece, não há nada que nos impeça de
assistir e consumir informações em velocidades mais rápidas do que fazemos atualmente. _Mas
talvez o prazer ou diversão não seja o mesmo.

AULA 01 – SKIMMING AND SCANNING AND OTHER TECHNIQUES. IDENTIFYING FALSE COGNATS. 154
AFA 2024

VERBS IN TEXTS

AULA 02

Teacher Andrea Belo

www.estrategiamilitares.com.br www.militares.estrategia.com
TEACHER ANDREA BELO

SUMÁRIO
INTRODUÇÃO 3

VERBO TO BE 4

SIMPLE PRESENT 6

SIMPLE PAST 8

FUTURE: WILL X GOING TO 11

GERUND 14

PRESENT CONTINUOUS OU PRESENT PROGRESSIVE 16

PAST CONTINUOUS OU PAST PROGRESSIVE 17

PRESENT PERFECT 18

PAST PERFECT 20

FUTURE PERFECT 22

PRESENT PERFECT CONTINUOUS 24

PAST PERFECT CONTINUOUS 26

FUTURE PERFECT CONTINUOUS 28

MODAL VERBS 30

IMPERATIVE TENSE 34

PHRASAL VERBS 36

QUESTÕES 61

GABARITO 61

QUESTÕES COMENTADAS 93

CONSIDERAÇÕES FINAIS 150

REFERÊNCIAS BIBLIOGRÁFICAS 151

TRADUÇÕES 153

AULA 02 – VERBS IN TEXTS 2


TEACHER ANDREA BELO

INTRODUÇÃO
Chegou a vez da nossa aula de verbos, uma das mais importantes de todo o material.
Se você identifica o verbo e consegue entendê-lo no contexto em que ele aparece, os
resultados são garantidos. E isso é muito importante. As interpretações são essenciais, é claro.
O vocabulário também. Mas os verbos são a “alma” da frase, eles apresentam as ideias do
texto e nos levam ao assunto, ao tema, ao que de fato se tratam os textos. Vamos estudá-los!
Verbo é a classe de palavras que exprime ação, que indicam acontecimentos representados
em um determinado tempo. Originada do latim, “verbum” significa, de fato, “palavra”.
Muitas pessoas acreditam que aprender as conjugações dos verbos seja complicado. Mas
não é. Em primeiro lugar, os verbos são essenciais para ajudar na interpretação.
Quanto mais enraizadas são as regras verbais, mais naturalmente você as usará no dia da
prova, extraindo os verbos dos textos e demarcando-os, para realizar uma leitura global do texto
em questão, encontrando as respostas procuradas.
E, saber os verbos, ajudará você a extrapolar o uso da língua inglesa na hora da prova, além
de aumentar a possibilidade de aplicá-los nos contextos exigidos nos exercícios.
Isso porque, ao testar o seu raciocínio e a sua capacidade de compreender textos em
Inglês, os verbos e suas devidas conjugações, em cada tempo verbal, economizam seu tempo e
direcionam a sua atenção ao que deve ser respondido.
Uma dica interessante é reconhecer o verbo assim que você ler cada frase do texto, seja
qual for a forma que a leitura for apresentada.
O verbo vem logo após o sujeito, que executa a ação. Por exemplo, se a frase é “The doctors
work at the hospital”, quem realiza a ação são os médicos (doctors) e a ação realizada é trabalhar
(work), que é o nosso verbo. Certo?
Eis que estamos diante de uma classe de palavras que favorece você a construir seus
pensamentos: os verbos, por excelência!
Ao analisá-los, devido à importância que eles têm, estudaremos exemplos juntos, com suas
peculiaridades e diferentes flexões, ampliando sua competência linguística e fazer bom uso do
aprendizado em sua prova.
Vamos, então, passar por todos os tempos verbais, esclarecendo dúvidas e lembrando que,
alguns tempos verbais são poucos explorados em vestibulares, mas, vamos “passar por eles” para
que o conteúdo fique completo, por inteiro, todos os tempos verbais, desde o famoso verbo to
be até estruturas mais complexas, como Present Perfect e os temidos Phrasal verbs.
Let’s go!

AULA 02 – VERBS IN TEXTS 3


TEACHER ANDREA BELO

VERBO TO BE
O verbo to be é aquele assunto que as pessoas definem como algo que se estuda a vida
inteira e ainda assim não sabe ao certo como se usa. É um verbo ensinado todas as vezes que se
inicia um curso de Inglês e, por esse motivo, muita gente considera “chato” estudar Inglês para
iniciantes.
O verbo to be tem sua importância e vou deixar claro como se usa e o porquê dessa
importância. To be significa ser ou estar. Não existe uma regra para saber se, na frase, é ser ou
estar, depende do contexto e a ideia sobre o que se refere.
Gosto de dizer que o to be não é um verbo difícil e sim exclusivo, já que pode ser usado em
diferentes frases, tanto como verbo principal quanto como verbo auxiliar.
O que diferencia o to be dos demais verbos da língua inglesa, é que em todos os outros,
utilizamos a raiz para fazer frases, o to be muda por inteiro.
Veja – verbo jogar (to play) I play, You play (Eu jogo, você joga) ou o verbo dançar (to dance)
I dance, you dance, they dance (Eu danço, você dança, eles dançam) enquanto o verbo ser – Eu
sou, ele é, eles são fica: I am, he is, they are, sem ao menos usar as letras “be” para iniciar as
conjugações. Você não vai dizer “I be, you be, we be”, como na maioria nos outros verbos, 99%
deles são conjugados através da raiz, do radical.
Usado como verbo ser, as frases geralmente usam adjetivos (Eu sou alto/baixo – I am
tal/short) ou para dizer a profissão (Ele é engenheiro – He is an engineer) entre outros exemplos.
O verbo to be como “estar” expressará sentidos de ação ou de se estar em algum lugar – I
am happy. She is in the supermarket – Eu estou feliz. Ela está no supermercado.
Em todos os tempos verbais que você vai estudar aqui, terá a explicação, seguida das
formas afirmativa, negativa e interrogativa, para que você compreenda melhor o uso dos verbos.
Nas frases afirmativas, o verbo to be é simples, conforme estudamos e, muitas vezes,
somos obrigados a decorá-los (I am, you are, he is, she is, it is, you are, we are, they are).
O motivo pelo qual you are se repete é que as palavras “você” e “vocês”, em Inglês, são
iguais: you. Assim, na conjugação, you are significa você é/você está e também vocês são/vocês
estão dependendo do contexto.
Na forma interrogativa em Inglês, o verbo to be se posiciona no início da frase, antes do
sujeito. A conjugação fica: Am I? Are you? Is he? Is she? Is it? Are you? Are we? Are they?
Na forma negativa, com a adição da partícula de negação “not” nos verbos, a conjugação
fica: I am not, you are not, he is not, she is not, it is not, you are not, we are not, they are not.
Se esses verbos aparecerem na forma abreviada, encontramos I’m not, you aren’t, he isn’t,
she isn’t, it isn’t, you aren’t, we aren’t, they aren’t. E também há as formas no tempo passado do
verbo to be.

AULA 02 – VERBS IN TEXTS 4


TEACHER ANDREA BELO

Preparei um esquema para resumir as três formas do TO BE no presente e no passado –


afirmativa, interrogativa e negativa – e, além do verbo to be, teremos os esquemas de todos os
tempos verbais que vamos estudar em cada capítulo de sua aula. Ficará melhor para você
visualizar e saber ler e encontrar os verbos em suas leituras. Vejamos o to be:
AFFIRMATIVE NEGATIVE INTERROGATIVE
I am ou I'm I am not ou I'm not Am I?
You are ou You're You are not ou You aren't Are You?
He is ou He's He is not ou He isn't Is He?
She is ou She's She is not ou She isn't Is She?
It is ou It's It is not ou It isn't Is It?
You are ou You're You are not ou You aren't Are You?
We are ou We're We are not ou We aren't Are We?
They are ou They're They are not ou They aren't Are They?

Agora, o esquema no passado (Verb to be in the Past), assim como no presente, vejamos:
AFFIRMATIVE NEGATIVE INTERROGATIVE
I was I was not ou I wasn't Was I?
You were You were not ou You weren't Were you?
He was He was not ou He wasn't Was he?
She was She was not ou She wasn't Was she?
It was It was not ou It wasn't Was it?
You were You were not ou You weren't Were you?
We were We were not ou We weren't Were we?
They were They were not ou They weren't Were they?

Para ler, interpretar e encontrar as respostas corretas, é necessário que você saiba, além
do verbo to be, todo o conteúdo que vamos explorar no cronograma de estudos em nosso
material. Cada aula será um complemento para a próxima.
E você também precisa estar atento às notícias do Brasil e do mundo, ler jornais, revistas,
estar com seus estudos em dia e de forma constante.
Sempre digo que, ler textos das fontes usadas pela banca na hora de preparar as provas é
um dos exercícios importantes a se fazer.
Uma vez preparado para interpretar a questão completa, você pode realizar as provas de
qualquer instituição e se sair bem.
Agora, vamos estudar o tempo Present Simple e todos os outros tempos verbais
necessários para resolver sua prova com mérito, contextualizando gramática e vocabulário.
Come on!

AULA 02 – VERBS IN TEXTS 5


TEACHER ANDREA BELO

SIMPLE PRESENT
Em Inglês, o Simple Present, tempo verbal Presente Simple, pode ser usado para expressar
uma ação habitual, aquilo que fazemos com frequência, por exemplo: I study every day (Eu estudo
todos os dias), I sometimes watch TV (Eu assisto TV às vezes), I often use the computer. (Eu uso o
computador com frequência) etc.
Usamos o Present Simple também para exprimir verdades, fatos imutáveis: Birds sing.
(Pássaros cantam), Babies need their moms. (Bebês precisam de suas mães) etc.
Usamos esse tempo também para informar situações, opiniões, fatos em geral: Technology
grows day by day. (A tecnologia cresce dia após dia), I love music. (Eu amo música) etc.
Temos que fazer um esclarecimento para facilitar o estudo de todos os tempos verbais.
Você sabe por que, ao se falar do verbo que será usado em uma frase, tem a preposição “to”
antes dele?
Por exemplo, o sujeito I, o verbo to study e o complemento very much, formam a frase I
study very much – Eu estudo muito, mas o “to” não aparece na frase.
Isso porque, o verbo em sua forma original, no infinitivo, ou seja, sem conjugação, está
acompanhado da preposição “to” enquanto em uma frase, o verbo é conjugado e não usamos
mais o “to” antes dele.
Se você procurar no dicionário os verbos ler, escrever e trabalhar – to read, to write e to
work. Mas, ao escrever as frases “Eu leio, Eu escrevo e Eu trabalho” fica: I read, I write e I work.
Temos três formas em todos os tempos verbais: afirmativa, negativa e interrogativa. Há
dois auxiliares que acompanham as frases interrogativas no presente: Do e Does.
por exemplo, ao dizer “Você trabalha? ou “Ela trabalha?” em Português, apenas colocamos
o ponto de interrogação no fim da frase.
Na forma interrogativa em Inglês, precisamos adicionar “Do” no início da pergunta – Do
you work? (Você trabalha?) e, para sujeitos no singular, classificados como terceira pessoa do
singular (he/she/it), usamos “Does” – Does she work? (Ela trabalha?), demonstrando que as
orações estão no tempo presente.
Na forma negativa, com a adição da partícula de negação “not” nos auxiliares, eles se
tornam do not/don’t e does not/doesn’t, formas abreviadas ou não – I don’t work (Eu não
trabalho), She does not work ou She doesn’t work (Ela não trabalha), He does not work/He doesn’t
work (Ele não trabalha).
Dificilmente você encontra a explicação da existência desses auxiliares. Vou esclarecer e
justificar para você. É simples.
Primeiro, os verbos em Inglês não tem terminações como em Português – Eu estudo, tu
estudas, ele estuda, nós estudamos, vós estudais, eles estudam – sendo apenas “study” para

AULA 02 – VERBS IN TEXTS 6


TEACHER ANDREA BELO

todos os sujeitos e acréscimo de -s, -es ou -ies para terceiras pessoas – I study, you study,
she/he/it studies, we study, they study.
Segundo, se em Português dizemos “Ela estuda” e “Ela estuda?” igual, mudando apenas a
entonação, como saberíamos o tempo da frase se não fosse demarcada pelos auxiliares Do e Does
– Do you study? e Does she study? Faz sentido, não é mesmo?
E, nas negativas, enquanto em Português temos a presença do “não” em todas as frases –
Eu não trabalho, você não trabalha, ela não trabalha etc tanto para presente quanto no passado
ou futuro, veja:
Exemplos: Eu não trabalhei, você não trabalhou, ele não trabalhará etc – como
saberíamos o tempo se não houvesse os auxiliares don’t e doesn’t demonstrando presente? –
Do you work? e Does she work? Entendeu? Got it?
As frases afirmativas são formadas por um sujeito, um verbo principal e o complemento,
que pode ser onde, quando aconteceu, com quem, porque ou qualquer outra informação que
alguém executou.
Lembrando que, ao ser conjugado nas terceiras pessoas do singular (e/she/it), precisamos
acrescentar “s”, “ies” ou “es”, exemplos: I run. She runs. (Eu corro. Ela corre.)
As frases interrogativas são formadas por um auxiliar (Do para sujeito no plural ou Does
para sujeito no singular) no início da frase, um sujeito, um verbo principal e o complemento (onde,
quando aconteceu, com quem, porque ou qualquer outra informação), exemplos: Do you run?
Does he run? (Você corre? Ele corre?)
As frases negativas são formadas por um sujeito, auxiliar don’t ou doesn’t, um verbo
principal e o complemento (onde, quando aconteceu, com quem, porque ou qualquer outra
informação), exemplos: I don’t run. She doesn’t run. (Eu não corro. Ela não corre.)
Vejamos, como exemplo, o verbo estudar – TO STUDY, conjugado em todas as pessoas do
singular e plural nas três formas – afirmativa, negativa e interrogativa no Present Simple:
AFFIRMATIVE NEGATIVE INTERROGATIVE
I study I do not study ou I don't study Do I study?
You study You do not study ou You don't study Do You study?
He studies He does not study ou He doesn't study Does He study?
She studies She does not study ou She doesn't study Does She study?
It studies It does not study ou It doesn't study Does It study?
You study You do not study ou You don't study Do You study?
We study We do not study ou We don't study Do We study?
They study They do not study ou They don't study Do They study?

Agora, vamos aos estudos do tempo Past Simple e suas particularidades.

AULA 02 – VERBS IN TEXTS 7


TEACHER ANDREA BELO

SIMPLE PAST
Em Inglês, o Simple Past, tempo verbal Passado Simples, é usado para demonstrar uma
ação que já aconteceu e ficou no passado, tal como um jogo que acabou, um evento que passou
ou alguém que chegou, por exemplo.

Por isso, as frases no passado simples são geralmente acompanhadas de uma expressão
de tempo definida como yesterday, que significa ontem – I worked yesterday (Eu trabalhei
ontem).

Para narrar ações que já ocorreram, além de yesterday, outras expressões mais comuns
que indicam o passado são: “last” - last night, last Sunday, last week (noite passada, domingo
passado, semana passada.

Outro termo é o “ago” – two years ago, ten minutes ago (dois anos atrás, dez minutos
atrás) etc. É importante salientar que, a palavra “atrás” é usada no tempo passado demonstrando
justamente o tempo. Se fosse a preposição “atrás”, apontando o lugar seria “behind” (He is behind
me – Ele está atrás de mim), ok? Estudaremos sobre isso na aula de preposições.

Temos também as três formas, como nos demais tempos verbais: afirmativa, negativa e
interrogativa.

Há apenas um auxiliar que acompanha as frases interrogativas e negativas: did/didn’t. Por


exemplo, ao dizer “Você trabalhou?”, adicionarmos “Did” no início da pergunta – Did you work?
para qualquer sujeito.

E usamos, em frases negativas: did not/didn’t, forma abreviada ou não – I didn’t work (Eu
não trabalhei), She did not work ou She didn’t work (Ela não trabalhou) etc. Perceba que o verbo
volta à sua forma original “work” tanto na forma interrogativa quanto negativa e, só apresenta
terminações ou diferenças em sua escrita na forma afirmativa. A explicação da existência do
auxiliar did também não é justificada e sim vista como obrigatória no tempo passado.

Mas, assim como no presente, os verbos em Inglês não tem terminações como em
Português – Eu trabalhei, tu trabalhou, ele trabalhou, nós trabalhamos, vós trabalhastes, eles
trabalharam – sendo apenas “worked” para todos os sujeitos e acréscimo de -ed, para qualquer

AULA 02 – VERBS IN TEXTS 8


TEACHER ANDREA BELO

sujeito quando o verbo for regular – I worked, you worked, she/he/it worked, we worked, they
worked.

Explicarei, em seguida, o que acontece quando os verbos são irregulares. Felizmente, são
minoria e isso colabora com seus estudos.

Verbos regulares são aqueles em que cujas terminações no tempo Past Simple apenas
sofrem o acréscimo das partículas “d” e “ed” na maioria dos verbos – She danced rock (Ela dançou
rock – verbo to dance – jogar).

Se o verbo terminar em vogal + a letra “y”, recebem “ed” – He played baseball (Ele jogou
beisebol). Mas, se terminar em consoante + a letra “y”, troca-se o “y” por “ied” – She cried
yesterday. (Ela chorou ontem – verbo to cry – chorar).

Caso o verbo termine com a sequência consoante/vogal/consoante, se dobra a última


consoante e também acrescenta “ed” - I preferred the blue pen. (Eu preferi a caneta azul – verbo
to prefer – preferir). Se o verbo terminar com a vogal “e”, simplesmente recebe “d” – He arrived
yesterday. (Ele chegou ontem – verbo to arrive – chegar).

Verbos irregulares são aqueles em que as conjugações no tempo Past Simple sofrem
diversas alterações, como mudança das letras (por exemplo o verbo to write – escrever – se torna
wrote), acréscimo de letras (por exemplo o verbo to hear – ouvir – se transforma em heard), entre
outras modificações que acontecem.

Alguns verbos mudam completamente, como é o caso do verbo to buy – comprar – se


transforma em bought e o verbo to be – ser/estar – que se transforma em was/were para singular
e plural. Por causa dessas transformações nos verbos irregulares, muitas pessoas acreditam que
eles sejam difíceis ou complicados.

E, na verdade, não há nada de complicado nisso. Com o uso dos verbos irregulares nos
exercícios diversos, eles vão se tornando familiares para você.

E não podemos esquecer que a quantidade de verbos irregulares é bem menor que os
regulares.

AULA 02 – VERBS IN TEXTS 9


TEACHER ANDREA BELO

Como eu já havia dito antes, 90% são os mais fáceis, com -ed acrescido no final deles. Os
verbos diferentes que passam por diversificações, além de ser apenas 10% da língua inglesa, se
repetem nos exercícios de provas de anos anteriores.

E, quanto mais questões você resolver e se dedicar aos estudos com leituras
complementares e muito esforço, passará a conhecê-los e se sentirá confiante com o passar do
tempo.

Vejamos exemplos do verbo TO STUDY, conjugado nas três formas – afirmativa, negativa e
interrogativa no tempo Past Simple:

AFFIRMATIVE NEGATIVE INTERROGATIVE


I studied I did not study ou I didn't study Did I study?
You studied You did not study ou You didn't study Did You study?
He studied He did not study ou He didn't study Did He study?
She studied She did not study ou She didn't study Did She study?
It studied It did not study ou It didn't study Did It study?
You studied You did not study ou You didn't study Did You study?
We studied We did not study ou We didn't study Did We study?
They studied They did not study ou They didn't study Did They study?

AFFIRMATIVE NEGATIVE INTERROGATIVE


I drove I did not drive ou I didn't drive Did I drive?
You drove You did not drive ou You didn't drive Did You drive?
He drove He did not drive ou He didn't drive Did He drive?
She drove She did not drive ou She didn't drive Did She drive?
It drove It did not drive ou It didn't drive Did It drive?
You drove You did not drive ou You didn't drive Did You drive?
We drove We did not drive ou We didn't drive Did We drive?
They drove They did not drive ou They didn't drive Did They drive?

O esquema ajuda a compreender melhor e memorizar cada estrutura, os auxiliares e,


consequentemente, lembrar das formas afirmativas, negativas e interrogativas quando aparecer
nas frases dos textos no dia da prova. Vamos testar seus conhecimentos de tempo futuro agora.
Let’s go!

AULA 02 – VERBS IN TEXTS 10


TEACHER ANDREA BELO

FUTURE: WILL X GOING TO


Quando se fala no tempo futuro, em Inglês, muitas pessoas dizem: futuro é “will” ou então
“going to”.

E as perguntas sobre a diferença entre will e going to são frequentes. As gramáticas, de


uma forma geral, conseguem, incrivelmente, complicar estruturas tão simples! Vamos simplificar
e entender como pode ser simples, de fato.

Portanto, você vai aprender o uso de will e going to de um modo prático para acertar as
questões que envolvem o tempo futuro conectado com um vasto vocabulário e, com certeza,
outros tempos verbais, que estão sendo aprendidos aos poucos.

As frases com o auxiliar “will” são imediatas pois, colocando-se will antes do verbo, pronto
– transformou a ideia de presente em futuro, veja: – I will work (Eu trabalharei), She will dance
(Ela dançará). They will study (Eles estudarão). Mas, se comparado ao “going to”, a dúvida é como
usar um outro no tempo futuro em inglês. Vamos esclarecer isso.

Usamos “will” quando vamos expressar algo no futuro indicando uma certa incerteza e,
muitas vezes, as frases em que o “will” é bem aplicado, encontramos algumas expressões comuns
no tempo futuro, tais como I think (eu acho que…), probably (provavelmente), I guess (eu acho)
maybe (talvez).

Há outras, mas com essas são mais comuns em frases indicativas de futuro nas provas.
Vejamos exemplos com “will” e as expressões expostas acima. Para ficar claro, vamos citar
exemplos tais como “Eu provavelmente viajarei em dezembro” – I will probably travel in
December.

Outro exemplo: “Ela talvez viajará nas férias” – She will maybe travel on vacation. Se você
tem quase certeza do que vai fazer, se já planejou algo – não é uma regra seguida 100% das vezes
– mas é melhor usar o “going to”, como na frase “Planejei a viagem, vou no próximo sábado.” – I
planned the trip. I am going next Saturday.

As frases afirmativas são formadas por um sujeito, o auxiliar will, um verbo principal e o
complemento – “Ela vai dançar amanhã cedo” – She will dance tomorrow in the morning.

AULA 02 – VERBS IN TEXTS 11


TEACHER ANDREA BELO

As frases interrogativas são formadas pelo auxiliar will no início da frase, um sujeito, um
verbo principal e o complemento (onde, quando aconteceu, com quem, porque ou qualquer outra
informação), exemplos: Will you run? Will he run? (Você vai correr? Ele vai correr?)

As frases negativas são formadas por um sujeito, auxiliar will not ou abreviado won’t, um
verbo principal e o complemento (onde, quando aconteceu, com quem, porque ou qualquer outra
informação), exemplos: I won’t run. She won’t run. (Eu não correrei. Ela não correrá.)

Vejamos exemplos do verbo TO STUDY, conjugado no futuro simples com WILL no


“esquema” em todas as formas – afirmativa, negativa e interrogativa para fixar melhor.

AFFIRMATIVE NEGATIVE INTERROGATIVE


I will study I will not study ou I won't study Will I?
You will study You will not study ou You won't study Will You?
He will study He will not study ou He won't study Will He?
She will study She will not study ou She won't study Will She?
It will study It will not study ou It won't study Will It?
You will study You will not study ou You won't study Will You?
We will study We will not study ou We won't study Will We?
They will study They will not study ou They won't study Will They?
Por sua vez, o going to é usado para expressar algo no futuro indicando uma certeza, planos
fixos, já definidos, por exemplo: I’m going to marry in 2021 – Eu vou me casar em 2021.

Nessa frase, a pessoa afirmou que vai se casar no ano de 2021 porque certamente já
planejou o casamento, marcou a data no cartório, preparou-se financeiramente para a festa e
outros elementos necessários para esse evento.

Quando fazemos uma previsão como um palpite, também é aconselhado o uso do going
to, por exemplo: It’s going to be an excellent year for me – Vai ser um excelente ano para mim.

As frases afirmativas são formadas por um sujeito, o verbo to be como auxiliar de cada
sujeito (I am, you are, he is, she is, it is, we are, they are), o verbo principal e o complemento –
“She is going to dance tomorrow. It is the Christmas school presentation. Ela vai dançar amanhã.
É a apresentação natalina na escola.”

AULA 02 – VERBS IN TEXTS 12


TEACHER ANDREA BELO

Percebemos que a pessoa que vai se apresentar foi ou está sendo preparada para o evento
que vai acontecer, houve um planejamento e, apresentações natalinas, acontecem todos os anos
nas escolas, algo que se realiza nas escolas no mês de dezembro.

Nas frases interrogativas, o verbo to be vai para a começo da pergunta.

Em seguida, temos um sujeito, o verbo principal e o complemento – Is she going to dance


at Christmas school presentation tomorrow? Ela vai dançar na apresentação natalina da escola
amanhã?

Desta forma, a pergunta provavelmente foi feita com a certeza da resposta que sim, apenas para
confirmar por causa do uso do going to.

As frases negativas são formadas por um sujeito, o verbo to be seguido do not, o verbo
principal e o complemento – “She is not going to dance tomorrow/ She isn’t going to dance
tomorrow. They are not going to travel – Eles não vão viajar. Assim, as formas abreviadas são: is
not = isn’t (para terceiras pessoas do singular) e are not = aren’t (para plural em geral).

Vejamos exemplos do verbo TO STUDY, conjugado no futuro com GOING TO, em forma de
“esquema” em todas as formas – afirmativa, negativa e interrogativa para você fixar melhor.

AFFIRMATIVE NEGATIVE INTERROGATIVE


I am going to study I am not going to study ou I'm not going to study Am I going to study?
You are going to study You are not going to study ou You aren't going to study Are you going to study?
He is going to study He is not going to study ou He isn't going to study Is he going to study?
She is going to study She is not going to study ou She isn't going to study Is she going to study?
It is going to study It is not going to study ou It isn't going to study Is it going to study?
You are going to study You are not going to study ou You aren't going to study Are you going to study?
We are going to study We are not going to study ou We aren't going to study Are we going to study?
They are goint to study They are not going to study ou They aren't going to study Are they goint to study?

Agora, estudaremos o gerúndio em nossa aula. Esse tempo verbal é muito importante na
construção de outras estruturas em que são necessários, tais como o gerúndio nos tempos da
vertente Continuous (Present e Past Continuous). Let’s go!

AULA 02 – VERBS IN TEXTS 13


TEACHER ANDREA BELO

GERUND
Em Inglês, o gerúndio é um pouco diferente do que conhecemos em Português. Faz parte
de estruturas em que agregamos a partícula -ing no fim dos verbos. Porém, com algumas exceções
que trataremos aqui.
Gerúndio, de um modo geral, pode ser definido com algo que transmite a ideia de ações
prolongadas ou ações ainda em desenvolvimento.
O gerúndio pode, por exemplo, transformar o verbo em substantivo, pode atuar como
sujeito, como predicado, e, na maioria das vezes, é usado para complementar verbos.
O gerúndio foi inserido nesse capítulo antes dos demais tempos verbais justamente porque
agora estudaremos o Present e o Past Continuous, cujas estruturas utilizam o gerúndio, além do
verbo to be, já estudado em nosso material.
Vejamos algumas regras ortográficas que precisam ser observadas ao acrescentar -ing aos
verbos. O gerúndio será sempre utilizado após preposições, por exemplo:
“I have chances of being promoted in this company” (Eu tenho chances de ser promovido nessa
empresa).
Outro exemplo: Margareth has her reasons for behaving different - Margareth tem seus
motivos para comportar-se diferente.
Também usamos o gerúndio os verbos to go – ir e to come – vir, quando fizerem referência
à atividades físicas, tais como:
– go fishing, go bowling, go swimming, go skiing, go riding, go jogging, go shopping, go
hiking, go boating
Veja perguntas: “– I go swimming every Saturday “(Eu nado todos os sábados), “Will you
come fishing with me?” (Você virá pescar comigo?) e I don’t want to go bowling tonight (Eu não
quero ir ao boliche essa noite, por exemplo.
A palavra swimming, no exemplo acima, se refere ao verbo nadar e significa, de fato, nadar.
Mas, há também os casos em que os verbos com –ing no final, serão substantivos.
Por exemplo: Swimming helps me to relax (Nadar me ajuda a relaxar, como se fosse a
natação, o ato de nadar) e Reading is very important to the students (Ler é muito importante para
os alunos, como se fossem as leituras, o ato de ler importante).
As palavras nos exemplos com -ing, na verdade, tornaram-se sujeitos e não verbos. Há
casos em que os verbos, necessitam -ing quando há duas ações, ou seja, dois verbos em uma só
frase.
São esses os exemplos: “To admit, to avoid, to appreciate, to consider, to continue, to delay,
to detest, to deny, to dislike, to enjoy, to escape, to finish, to forgive, to imagine, to include, to
keep, to mention, to miss, to practice, to recommend, to resist, to risk, to suggest, to try, to
understand e to quit.”

AULA 02 – VERBS IN TEXTS 14


TEACHER ANDREA BELO

Esses verbos, quando inseridos em frases, necessitam que o segundo verbo, logo após
deles, tenham o acréscimo de -ing. Com exercícios e muita prática em seus estudos, isso ficará
fácil e natural para você.
Veja alguns exemplos: I admit getting angry sometimes (Eu admito que fico nervoso às
vezes), I enjoy studying English (Eu gosto de estudar Inglês) e They deny doing that (Eles negam
que fizeram aquilo).
Existe uma regra em que os verbos terminados pela letra “e”, perdem o “e”, ao usar -ing.
São exemplos os verbos to drive (dirigir) e to save (economizar), She is driving now (Ela está
dirigindo agora) e He is saving money for his future (Ele está economizando dinheiro para o seu
futuro).
Após algumas expressões em Inglês, precisamos usar o -ing como regra também. São elas:
“can’t stand, it’s worth, be used to, can’t help, feel like, it’s no good, look forward to, what about,
how about, it’s no use, in spite of.”
Vejamos exemplos: “I can’t help laughing now” (Não consigo não rir agora), “I can’t stand
explaining you something thousands of times” (Não aguento explicar a você a mesma coisa mil
vezes) e “It’s worth visiting that museum” (Vale a pena visitar aquele museu).
Vejamos exemplos de alguns verbos, em forma de “esquema” no Gerúndio, conforme as
regras e exemplos de como melhor usar, de acordo com a teoria e explicações estudadas.
GERUND
Acrescenta-se –ING, Acrescenta-se –ING após Acrescenta-se –ING para
retirando a letra “e” do preposições atividades físicas
final dos verbos THE STUDENTS HAVE NO GO SWIMMING
TAKE – TAKING REASONS FOR WORRYING ABOUT GO FISHING
DANCE – DANCING THE TESTS.
GO RIDING A BIKE
LOVE – LOVING Os alunos não têm motivos para
preocupar-se com as provas. GO BOWLING
entre outros entre outros
Acrescenta-se –ING para Acrescenta-se –ING para frases Acrescenta-se –ING após
verbos na função de com dois verbos expressões fixas
sujeito THE DOCTORS LIKED OPERATING I FELL LIKE READING.
SWIMMING IS HEALTHY. THE PATIENT IN THAT HOSPITAL. Estou a fim de ler.
Natação é saudável. Os médicos gostaram de operar o A CAN’T STAND FORGETING
paciente naquele hospital. THINGS.
Não suporto esquecer as
coisas.

Agora, estudaremos os tempos verbais da vertente Continuous (Present e também past


Continuous), que usam o gerúndio em suas estruturas para a elaboração de frases nas formas
afirmativa, negativa e interrogativa.
Assim, já ficará mais simples para compreender tais tempos verbais. Vamos lá!

AULA 02 – VERBS IN TEXTS 15


TEACHER ANDREA BELO

PRESENT CONTINUOUS OU PRESENT PROGRESSIVE


Em continuação ao assunto gerúndio e o uso do -ing nos verbos, vamos falar do Present
Continuous ou Present Progressive, pois esse tempo verbal é conhecido nessas duas
denominações.

O que você precisa saber, essencialmente, é que este tempo verbal é formado pelo verbo
to be e outro verbo, no caso, o verbo principal da frase.

Isto significa que se você souber conjugar o verbo to be e também souber o gerúndio dos
verbos, a estrutura do Present Continuous está formada.

As frases afirmativas são formadas por um sujeito, o verbo to be na afirmativa, o verbo


principal e o complemento – “Ela está estudando agora.” – She is studying now.

Outros exemplos: He is working at this moment. (Ele está trabalhando nesse momento),
They are reading a magazine. (Eles estão lendo uma revista).

As frases interrogativas são formadas pelo verbo to be na forma afirmativa no início da


frase, o verbo principal e o complemento – “Ela está estudando agora?” – Is he working at this
moment? (Ele está trabalhando nesse momento?) e as frases negativas são formadas por um
sujeito, o verbo to be na forma negativa, o verbo principal e o complemento – “Ela não está
estudando agora.” – She is not/isn’t studying now.

Vejamos exemplos do verbo TO STUDY, no Present Continuous, em nosso “esquema” e em


seguida, estudar o Past Continuous:

AFFIRMATIVE NEGATIVE INTERROGATIVE


I am studying ou I'm studying I am not studying ou I'm not studying Am I studying?
You are studying ou You're studying You are not studying ou You aren't studying Are You studying?
He is studying ou He's studying He is not studying ou He isn't studying Is He studying?
She is studying ou She's studying She is not studying ou She isn't studying Is She studying?
It is studying ou It's studying It is not studying ou It isn't studying Is It studying?
You are studying ou You're studying You are not studying ou You aren't studying Are You studying?
We are studying ou We're studying We are not studying ou We aren't studying Are We studying?
They are studying ou They're studying They are not studying ou They aren't studying Are They studying?

AULA 02 – VERBS IN TEXTS 16


TEACHER ANDREA BELO

PAST CONTINUOUS OU PAST PROGRESSIVE


Falar do Past Continuous, também é o mesmo que falar de Past Progressive, pois esse
tempo verbal é conhecido nas duas formas.

O que você precisa saber, essencialmente, é que este tempo verbal é formado pelo verbo
to be, desta vez conjugado no passado e outro verbo, a ação principal da frase.

Mais uma vez, se você souber conjugar o verbo to be no passado e também souber o
gerúndio dos verbos, a estrutura do Past Continuous está formada.

As frases afirmativas são formadas por um sujeito, o verbo to be no passado, na forma


afirmativa, o verbo principal e o complemento – “Ela estava estudando.” – She was studying.

As frases interrogativas são formadas pelo verbo to be no passado e na forma afirmativa


no início da frase, o verbo principal e o complemento – “Ela estava estudando?” – Was she
working? e Were they reading a magazine? (Eles estavam lendo uma revista?).

As frases negativas são formadas por um sujeito, o verbo to be no passado e na forma


negativa, o verbo principal e o complemento – “Ela não estava estudando.” – She was not/ wasn’t
studying.

Vejamos nosso “esquema”, com o verbo TO STUDY, conjugado no Past Continuous:

AFFIRMATIVE NEGATIVE INTERROGATIVE


I was studying I was not studying ou I wasn't studying Was I?
You were studying You were not studying ou You weren't studying Were you?
He was studying He was not studying ou He wasn't studying Was he?
She was studying She was not studying ou She wasn't studying Was she?
It was studying It was not studying ou It wasn't studying Was it?
You were studying You were not studying ou You weren't studying Were you?
We were studying We were not studying ou We weren't studying Were we?
They were studying They were not studying ou They weren't studying Were they?
Agora, estudaremos um tempo verbal muito importante, o Present Perfect.

AULA 02 – VERBS IN TEXTS 17


TEACHER ANDREA BELO

PRESENT PERFECT
O Present Perfect é considerado difícil, é visto como algo complexo, mas, basta entender a
maneira certa de usá-lo e encontrá-lo nas frases, que ele se torna mais simples do que parece.

Vou mostrar, na minha forma de ensinar, como pode ser descomplicado, ok?

Present Perfect é um tempo verbal que descreve uma ação em que estão conectados o
passado e o presente. Ou seja, o Present Perfect conta fatos que ocorreram em um tempo
indefinido do passado e ainda não foram concluídos.

Em Português, não temos um tempo que corresponda a esse. E, por isso, ao invés de dizer
“Tenho estudado para essa prova desde 2017”, as pessoas dizem “Eu estudo para essa prova
desde 2017”, usando o presente para contar algo que já começou e ainda acontece, diferente em
Inglês, que o tempo verbal desse capítulo faz esse papel.

Por esse motivo, o Present Perfect é, muitas vezes, julgado e considerado um tempo verbal
complicado, difícil de aprender. Mas, como eu disse, ao compreender o uso certo, ficará simples.

Vejamos algumas regras de uso correto do Present Perfect. Em primeiro lugar, ações que
“vêm acontecendo recentemente”, por exemplo: I have been sad recently (Ando triste
recentemente/ Tenho estado triste recentemente) ou They have run every day at the park (Eles
correm todos os dias no parque/Eles têm corrido todos os dias no parque).

Outro uso do Present Perfect: ações que acabaram de acontecer - We have just finished our
work. (Nós acabamos de terminar nosso trabalho) e She has just looked that magazine. (Ela
acabou de olhar aquela revista).

E, uma das formas mais comuns de se encontrar o Present Perfect é quando algo aconteceu
em um momento indefinido, como: You have played video game for a long time. (Você joga/tem
jogado vídeo game por muito tempo) e I have helped you a lot. (Eu ajudo/tenho ajudado você
bastante).

As frases afirmativas têm a seguinte estrutura: um sujeito, um verbo auxiliar have/has


(terceiras pessoas do singular) e o verbo principal no particípio passado - She has studied “Ela tem
estudado”, They have worked very much. (Eles têm trabalhado muito).

Para verbos regulares, o particípio passado apenas acrescenta –ed, assim como no Past
Simple de verbos regulares. É igual. E isso é ótimo porque facilita bastante na hora da prova.

AULA 02 – VERBS IN TEXTS 18


TEACHER ANDREA BELO

Se, por sua vez, o verbo for irregular, o particípio não segue nenhuma regra, sendo
geralmente feita troca de letras ou acréscimo de -en no final deles, como por exemplo to write
(escrever), que no passado é wrote, no particípio fica written para qualquer sujeito.

Pelo menos isso, não é? Veja: He has eaten a lot these days – Ele tem comido muito esses
dias. O verbo to eat (comer), que no passado fica ate, por ser irregular, no particípio, muda para
eaten mas o usamos para todos os sujeitos, assim como os regulares.

As frases interrogativas são formadas pelo verbo auxiliar have/has no início das perguntas,
um sujeito e o verbo principal no particípio - Has she studied “Ela tem estudado?”, Have they
worked very much? (Eles têm trabalhado muito?). Aqui também, usa-se o mesmo verbo para
todos os sujeitos.

As frases negativas são formadas por um sujeito, um verbo auxiliar have/has na forma
negativa has not/hasn’t e have not/haven’t e o verbo principal no particípio - She hasn’t studied
“Ela não tem estudado”, They haven’t worked very much. (Eles não têm trabalhado muito).

Vejamos nosso “esquema”, com o verbo TO STUDY, conjugado no Present Perfect:

AFFIRMATIVE NEGATIVE INTERROGATIVE


I have studied I have not studied ou I haven't studied Have I studied?
You have studied You have not studied ou You haven't studied Have You studied?
He has studied He has not studied ou He hasn't studied Has He studied?
She has studied She has not studied ou She hasn't studied Has She studied?
It has studied It has not studied ou It hasn't studied Has It studied?
You have studied You have not studied ou You haven't studied Have You studied?
We have studied We have not studied ou We haven't studied Have We studied?
They have studied They have not studied ou They haven't studied Have They studied?

Agora, estudaremos o Past Perfect em nossa aula. Esse tempo verbal também é geralmente
dito como sendo complicado por fazer arte dos tempos de denominação “Perfect”.

E você verá que é simples e depois vamos exercitar com muitas questões com esses tempos
verbais dentro dos textos. Basta compreender as regras e como usá-los.

AULA 02 – VERBS IN TEXTS 19


TEACHER ANDREA BELO

PAST PERFECT
O Past Perfect é um tempo verbal que descreve uma ação no passado, que ocorreu antes
de outra. Como assim? Bom, o Past Perfect conta fatos correlacionados com outros que
aconteceram.

É simplesmente dizer o que houve em decorrência de outro fator, como por exemplo: “Fui
promovido porque vendi/tive vendido muito esse mês – I got the promotion because I had sold a
lot this month. Ser promovido só aconteceu porque a pessoa vendeu muito, antes de receber a
promoção.

Então, “vender muito” foi expresso, no Past Perfect, como a ação anterior ao ganho da
promoção (had sold = got the promotion), certo?

Assim como o Present Perfect, o Past Perfect tem suas regras de uso e facilitará para você
encontrá-lo nos textos e entender por que foi usado naquele momento.

Em primeiro lugar, ações que aconteceram por causa de outras, como vimos no parágrafo
anterior. Então, para ficar claro, veja: O criminoso fugiu. Então, a polícia chegou. A polícia só
chegou depois que o criminoso fugiu. Logo, o fato de a polícia chegar será expresso no Past Simple
enquanto, o criminoso fugir, que aconteceu antes, estará no Past Perfect: The criminal had run
away when the police arrived. (to run away = fugir/to arrive = chegar).

Usamos Past Perfect para fatos que “tinham/haviam acabado” de acontecer, com o uso do
advérbio just, como também vimos no Present Perfect – We had just left when you called. (Nós
tínhamos acabado de partir quando você ligou).

E, outros advérbios que encontramos no Past Perfect são: already, when, by the time,
never, ever, before, after, para enfatizar a ideia de que a ação estava totalmente acabada antes
da que será descrita: She had already decided not to go. (Ela já tinha decidido não ir.) e I asked my
friend if he had ever gone to London. (Perguntei ao meu amigo se ele já tinha ido em Londres).

As frases afirmativas têm a seguinte estrutura: um sujeito, o verbo auxiliar had (para todos
os sujeitos) e o verbo principal no particípio passado – She had studied “Ela tinha estudado”, They
had worked very much. (Eles tinham trabalhado muito).

AULA 02 – VERBS IN TEXTS 20


TEACHER ANDREA BELO

Aqui também, mesma observação: para verbos regulares, o particípio passado apenas
acrescenta –ed e, quando verbo é irregular, o particípio não segue nenhuma regra, como vimos
os verbos to write (escrever), que no passado é wrote, no particípio fica written.

As frases interrogativas são formadas pelo verbo auxiliar had no início das perguntas, um
sujeito e o verbo principal no particípio – Had she studied “Ela tinha estudado?”, Had they worked
very much? (Eles tinham trabalhado muito?).

As frases negativas são formadas por um sujeito, um verbo auxiliar had na forma negativa
had not/hadn’t e o verbo principal no particípio – She hadn’t studied “Ela não tinha estudado”,
They hadn’t worked very much. (Eles não tinham trabalhado muito).

Vejamos nosso “esquema”, com o verbo TO STUDY, conjugado no Past Perfect:

AFFIRMATIVE NEGATIVE INTERROGATIVE


I had studied I had not studied ou I hadn't studied Has I studied?
You had studied You had not studied ou You hadn't studied Had you studied?
He had studied He had not studied ou He hadn't studied Had he studied?
She had studied She had not studied ou She hadn't studied Had she studied?
It had studied It had not studied ou It hadn't studied Had it studied?
You had studied You had not studied ou You hadn't studied Had you studied?
We had studied We had not studied ou We hadn't studied Had we studied?
They had studied They had not studied ou They hadn't studied Had they studied?

Agora, estudaremos o último tempo verbal Perfect, o Future Perfect.

E você verá que esse tempo verbal também é simples. Basta saber como usá-lo.

Como eu disse antes, algumas estruturas e formas verbais são dificilmente encontradas
nas provas e outras são mais comuns, como o Present Perfect, que acabamos de ver.

Veremos todas para não haver dúvidas.

AULA 02 – VERBS IN TEXTS 21


TEACHER ANDREA BELO

FUTURE PERFECT
O Future Perfect é usado para expressar ações que vão terminar, em um certo tempo no
futuro, como se você já soubesse ou como se estivesse prevendo o que vai acontecer.

Vamos aprender esse tempo verbal através de exemplos.

Em uma frase, se queremos dizer que o voo de uma pessoa será às 20h, por exemplo, não
há tempo de chegar no aeroporto e embarcar às 20h se a pessoa sair de casa às 19:30h. Então,
diremos: Quando ela chegar no aeroporto, o avião já terá partido – When she gets there, the plane
will have left.

Outro exemplo para ficar mais claro: By next month, I will have finished my book. (No
próximo mês, eu terei terminado meu livro).

Percebemos que, o sujeito, que provavelmente seja o escritor do livro, fez uma previsão
para o término de sua ação, dizendo que o livro estará terminado no próximo mês.

Por isso, no Future Perfect, é comum encontrarmos expressões temporais que


acompanhem as frases, tais como: before (antes), by (em, no, na), by the time (quando) etc.

Como eu já disse, o Future Perfect se refere a eventos que serão terminadas em


determinado ponto do futuro, ou seja, em um tempo posterior ao do momento da ação principal
da frase.

As frases afirmativas têm a seguinte estrutura: um sujeito, o verbo auxiliar que representa
o futuro: will, o verbo auxiliar to have (para todas as pessoas como sujeito) e o verbo principal no
particípio – She will have arrived there before you notice it (Ela terá chegado lá antes que você
perceba), demonstrando que ela saiu e vai chegar em um determinado lugar antes que a outra
pessoa perceba.

As frases interrogativas são formadas pelo auxiliar will, dessa vez antes dos sujeitos, logo
no início das frases, o sujeito, o verbo to have e o verbo principal no particípio – Will she have
arrived at the airport by 8pm? (Ela terá chegado no aeroporto às 8pm?).

E as frases negativas são formadas pelo sujeito, auxiliar will, dessa vez na forma negativa,
will not/won’t, o verbo to have e o verbo principal no particípio – She will not have arrived at the
airport by 8pm (Ela não terá chegado no aeroporto às 8pm).

AULA 02 – VERBS IN TEXTS 22


TEACHER ANDREA BELO

Vale ressaltar que, para construir frases no Future Perfect, não importa se as ações serão
realmente concluídas no futuro: o que importa é a projeção que o sujeito vai fazer para usar tal
tempo verbal, veja: The boy will have paid all his debts in December – significando O garoto terá
pago todas as suas dívidas até Dezembro.

Mesmo que ele não pague seus débitos até o fim do ano, é o que se espera que aconteça.
E, por isso, a previsão de que ele, provavelmente terá pago, foi expressada no Future Perfect, ok?

Vejamos o Future Perfect em nosso “esquema”.

AFFIRMATIVE NEGATIVE INTERROGATIVE


I will have studied I will not have studied ou I won't have studied Will I have studied?
You will have studied You will not have studied ou You won't have studied Will You have studied?
He will have studied He will not have studied ou He won't have studied Will He have studied?
She will have studied She will not have studied ou She won't have studied Will She have studied?
It will have studied It will not have studied ou It won't have studied Will It have studied?
You will have studied You will not have studied ou You won't have studied Will You have studied?
We will have studied We will not have studied ou We won't have studied Will We have studied?
They will have studied They will not have studied ou They won't have studied Will They have studied?

Agora, estudaremos os tempos da vertente “Perfect Continuous”, em que há auxiliar to


have, verbos no particípio e também no gerúndio, vamos lá?

E, daqui por diante, vamos estudar e praticar com exercícios variados.

AULA 02 – VERBS IN TEXTS 23


TEACHER ANDREA BELO

PRESENT PERFECT CONTINUOUS


O Present Perfect Continuous é um tempo verbal usado para enfatizar a continuidade de
uma ação que se iniciou no passado e se prolonga até hoje. Pouco usado em provas, mas, como
os textos são de diferentes fontes, pode aparecer ou ajudar na compreensão de alguma
alternativa na hora de sua prova.

A definição se parece com a do Present Perfect, que também é usado para indicar algo que
começou e ainda não terminou, certo? Então, temos que analisar as diferenças para não haver
confusão.

Vou definir as diferenças através de exemplos para que você possa visualizar bem e
encontrar, nos textos, cada tempo verbal no dia da sua prova.

Na seguinte frase: I have studied English for 6 years. (Eu estudo/tenho estudado Inglês há
6 anos), a ação teve início 6 anos atrás e ainda continua até o presente momento – a pessoa ainda
está estudando Inglês.

O Present Perfect Continuous é mais simples do que isso. É quando algo está acontecendo
no mesmo momento em que o sujeito está falando, veja: I have been studying English for 6 years,
seria também “Eu tenho estudado Inglês a 6 anos”, mas o falante, o sujeito da frase, está
estudando Inglês agora, nesse momento, lembrando-se que já estuda esse idioma a 6 anos, ou
seja, estudando e enfatizando a ação de que começou a estudar no passado.

O Present Perfect, por sua vez, revela algo que começou e ainda acontece, mas, não
necessariamente que a pessoa esteja fazendo o que diz.

Outro exemplo para ficar mais clara a diferença para você: Eu estou preparando uma sopa,
seria I am preparing a soup, usando Present Continuous, certo?

Mas, Eu estou preparando uma sopa a 15 minutos, seria I have been preparing a soup for
15 minutes, no Present Perfect Continuous, ou seja, comecei a sopa e ainda estou preparando-a,
a ação não terminou e ainda está sendo feita, ok?

Outro exemplo: alguém está limpando a casa o chão ainda está molhado, usamos o Present
Perfect Continuous – She has been cleaning the house and the floor is still wet – pois ela tem
limpado a casa e não terminou, já que o chão ainda está molhado (wet).

As frases afirmativas têm a seguinte estrutura: um sujeito, o verbo auxiliar to have (ou has
nas terceiras pessoas do singular) o verbo to be no particípio (been) para todos os sujeitos e o
verbo principal no gerúndio (com -ing) – She has been singing (Ela tem cantado), demonstrando
que ela começou a cantar, ainda está cantando.

AULA 02 – VERBS IN TEXTS 24


TEACHER ANDREA BELO

A pessoa que falou isso, o sujeito da frase, provavelmente, está vendo ou ouvindo quem
está cantando, ou com admiração ou fazendo algum tipo de observação sobre o que vê/ouve.

As frases interrogativas são formadas pelo to have (ou has nas terceiras pessoas do
singular) antes do sujeito, o verbo to be no particípio (been) para todos os sujeitos e o verbo
principal no gerúndio (com -ing) – Has she been singing? (Ela tem cantado?).

E as frases negativas são formadas pelo sujeito, o verbo to have not (ou has not nas
terceiras pessoas do singular) o verbo to be no particípio (been) para todos os sujeitos e o verbo
principal no gerúndio (com -ing) – She hasn’t been singing (Ela não tem cantado).

Vejamos uma observação que preparei para facilitar a sua compreensão sobre esses
tempos verbais. Em seguida, o esquema do verbo to study, conjugado no Present Perfect
Continuous.

Não confunda:

• Present Continuous

• Present Perfect Continuous

• Present Perfect

O Present Continuous expressa uma ação que está ocorrendo no momento, agora:

She is dancing now. (Ela está dançando agora.)

O Present Perfect Continuous expressa algo que começou no passado e continua até o presente:

He has been dancing for one hour. (Ele está dançando há uma hora.)

O Present Perfect expressa ações que acabaram em um tempo não definido do passado:

She has danced. (Ela dançou. – pode dançar novamente, todos os dias já que não definiu quando).

AFFIRMATIVE NEGATIVE INTERROGATIVE


I have been studying I have not been studying ou I haven't been studying Have I been studying?
You have been studying You have not been studying ou You haven't been studying Have You been studying?
He has been studying He has not been studying ou He hasn't been studying Has He been studying?
She has been studying She has not been studying ou She hasn't been studying Has She been studying?
It has been studying It has not been studying ou It hasn't been studying Has It been studying?
You have been studying You have not been studying ou You haven't been studying Have You been studying?
We have been studying We have not been studying ou We haven't been studying Have We been studying?
They have been studying They have not been studying ou They haven't been studying Have They been studying?

Vamos ao Past Perfect Continuous agora. Preparado?

AULA 02 – VERBS IN TEXTS 25


TEACHER ANDREA BELO

PAST PERFECT CONTINUOUS


O Past Perfect Continuous, que também pode ser encontrado com o nome Past Perfect
Progressive, como os outros tempos verbais Continuous em geral, é usado para enfatizar ações
anteriores à outras, ambas no passado.

Por exemplo, um aluno estudou por 8 horas seguidas e ficou cansado. A ação de ficar
cansado veio como consequência dos estudos. E, aconteceu depois que ele estudou por longas
horas.

Porém, ambas ações já aconteceram – estudar e ficar cansado – o aluno provavelmente já


descansou e o que aconteceu já passou.

Para descrever esses acontecimentos do passado, usando dois verbos na frase, um deles
será expresso no passado (ficar cansado) e o verbo que descreve o fato anterior ao cansaço, no
Past Perfect Continuous, tempo justamente exclusivo para acontecimentos anteriores a outros.

Vejamos o exemplo acima explicado, agora em Inglês, com o uso do tempo verbal Present
Perfect Continuous: He got tired because he had been studying for 8 hours – to get tired = ficar
cansado, no passado = got tired) e had been studying = tinha estudado ou tinha ficado estudando
8 horas, se for traduzir literalmente. Parece que não é simples, mas é.

Na seguinte frase: I had been writting e-mails all night long before I went to bed (Eu fiquei
escrevendo e-mails a noite inteira antes de ir para cama), ambas ações já aconteceram: escrever
emails a noite inteira e depois ir para a cama dormir.

A última coisa feita foi ir para cama dormir e, por isso, o verbo went to bed. Já a ação
escrever e-mails, o que o sujeito fez antes de dormir, está no Past Perfect Continuous – had been
writing.

É muito comum, no Past Perfect Continuous, o uso de advérbios diversos na elaboração


das sentenças, como when (quando), since (desde), before (antes), after (depois) etc. porque
esses advérbios proporcionam a ligação entre os eventos que aconteceram no passado.

As frases afirmativas do Past Perfect Continuous têm a seguinte estrutura: um sujeito, o


verbo auxiliar to have no passado: had, o verbo to be no particípio (been) para todos os sujeitos e
o verbo principal no gerúndio (com -ing) – She had been singing long hours and she won the music

AULA 02 – VERBS IN TEXTS 26


TEACHER ANDREA BELO

festival. (Ela tinha ficado cantando ou simplesmente ela tinha cantado antes de ganhar o festival
de música. E ganhou: won), demonstrando que praticou e ganhou, duas ações que aconteceram.

As frases interrogativas são formadas pelo had antes do sujeito, no início das frases, o
verbo to be no particípio (been) para todos os sujeitos e o verbo principal no gerúndio (com -ing)
– Had she been singing before she won the festival? (Ela tinha cantado? Ou Ela tinha ficado
cantando antes que ganhasse o festival?).

E as frases negativas, por sua vez, são formadas pelo sujeito, pelo had na negativa: had
not/hadn’t, o verbo to be no particípio (been) para todos os sujeitos e o verbo principal no
gerúndio (com -ing) – She hadn’t been singing before... (Ela não tinha cantado ou não tinha ficado
cantando antes de...)

Vejamos o esquema do verbo to study, conjugado no Past Perfect Continuous:

AFFIRMATIVE NEGATIVE INTERROGATIVE


I had been studying I had not been studying ou I hadn't been studying Had I been studying?
You had been studying You had not been studying ou You hadn't been studying Had You been studying?
He had been studying He had not been studying ou He hadn't been studying Had He been studying?
She had been studying She had not been studying ou She hadn't been studying Had She been studying?
It had been studying It had not been studying ou It hadn't been studying Had It been studying?
You had been studying You had not been studying ou You hadn't been studying Had You been studying?
We had been studying We had not been studying ou We hadn't been studying Had We been studying?
They had been studying They had not been studying ou They hadn't been studying Had They been studying?

Agora, em complemento aos tempos “Perfect Continuous”, vejamos o Future Perfect


Continuous no próximo capítulo e outros tempos verbais e curiosidades adiante. Vamos lá!

AULA 02 – VERBS IN TEXTS 27


TEACHER ANDREA BELO

FUTURE PERFECT CONTINUOUS


O Future Perfect Continuous, que também pode ser encontrado com o nome Future Perfect
Progressive indica uma ação que será completada em algum momento no futuro.

É um tempo verbal pouco usado em Inglês, o menos usado de todos os tempos aqui
estudados.

Porém, você precisa entender bem o uso e regras dele para que, caso apareça em sua
prova, seja simples encontrá-lo e saber como responder as perguntas sobre isso.

Uma das características marcantes desse tempo verbal é que ele expressa algo especial,
pois, quando utilizado, é expressa uma intenção real do que se quer dizer.

Vejamos exemplos: By October of this year, I will have been searching for a job for three
months. (Em Outubro deste ano, fará três meses que estarei procurando um emprego).

O sujeito está dizendo algo que vai acontecer daqui a 3 meses, considerando que ele
estamos em Julho, já que 3 meses contados a partir de Julho, é Outubro. E, se o sujeito não
encontrar o emprego que procura, completará 3 meses a procura de algo que ainda não
encontrou.

Vejamos outro exemplo, com o uso do tempo verbal Future Perfect Continuous: At ten
o’clock, I will have been waiting for you for two hours (Às 22h, terei ficado esperando você por
duas horas). Então, agora são exatamente 20h e daqui 2 horas (22h), a pessoa já terá esperado
outra por 2 horas, ou seja, ficará 2 horas esperando alguém.

Vejamos um exemplo. Na seguinte frase: Next year, the teacher will have been working at
the school for more than 5 years – quer dizer que no ano que vem, o professor terá trabalhado na
escola por mais de 5 anos. Então, o professor já trabalha a quase 4 anos no mesmo lugar e, no
próximo ano, completará 5 ou mais anos trabalhando nessa escola.

As frases afirmativas do Future Perfect Continuous têm a seguinte estrutura: um sujeito, o


verbo auxiliar que representa o futuro: will, o verbo to have (para todos os sujeitos), o verbo to
be no particípio – been – e o verbo principal no gerúndio (com -ing) – She will have been singing
for two hours at 3pm. (Ela terá ficado cantando por 2 horas às 15h), demonstrando que ela está

AULA 02 – VERBS IN TEXTS 28


TEACHER ANDREA BELO

cantando, são 13h e, daqui 2 horas, ela terá ficado fazendo a mesma coisa (cantando) por duas
horas. Certo?

As frases interrogativas são formadas pelo will antes do sujeito, no início das frases, o
sujeito, o verbo auxiliar que representa o futuro: will, o verbo to have (para todos os sujeitos), o
verbo to be no particípio – been – e o verbo principal no gerúndio (com -ing) – Will she have been
singing for two hours at 3pm.

E as frases negativas, por sua vez, são formadas pelo sujeito, o auxiliar will na negativa –
will not/won’t, o verbo to have (para todos os sujeitos), o verbo to be no particípio – been – e o
verbo principal no gerúndio (com -ing) – She will not/won’t have been singing for two hours at
3pm.

Vejamos o esquema do verbo to study, conjugado no Future Perfect Continuous.

AFFIRMATIVE NEGATIVE INTERROGATIVE


I will have been studying I will not have been studying ou I won't have been studying Will I have been studying?
You will have been studying You will not have been studying ou You won't have been studying Will You have been studying?
He will have been studying He will not have been studying ou He won't have been studying Will He have been studying?
She will have been studying She will not have been studying ou She won't have been studying Will She have been studying?
It will have been studying It will not have been studying ou It won't have been studying Will It have been studying?
You will have been studying You will not have been studying ou You won't have been studying Will You have been studying?
We will have been studying We will not have been studying ou We won't have been studying Will We have been studying?
They will have been studying They will not have been studying ou They won't have been studying Will They have been studying?

Agora, estudaremos sobre os verbos modais e suas características, regras, como usar etc.
E vamos acrescentando conteúdo em seus estudos. Vamos lá!

AULA 02 – VERBS IN TEXTS 29


TEACHER ANDREA BELO

MODAL VERBS
Modal verbs são muito simples. Verbo modal é o nome técnico classificado pela Gramática
Normativa como um grupo de palavras em Inglês que possuem suas características próprias.
Como assim? É porque esse grupo de verbos, considerados de certa forma auxiliares, não
seguem as mesmas regras que os outros verbos da língua inglesa.
O uso dos Modal verbs acontece para mudar ou, muitas vezes, complementar o sentido do
verbo principal, expressando ideias variadas, que podem ser: possibilidade, obrigação, dedução,
desejo, proibição, vontade, capacidade, entre outras.
Vamos estudar cada um deles separadamente e conhecer seus significados e maneiras de
usar.
Nas frases afirmativas, eles vêm antes dos verbos principais. Nas interrogativas, no início
das frases e nas negativas, com o acréscimo de “not”, assim como já vimos em outros auxiliares
já estudados.
Agora, um por um para melhor compreensão.

VERBO MODAL CAN


Can é usado para expressar, na maioria das vezes, capacidade ou habilidade. Mas pode
aparecer em frases também demonstrando possibilidade, alguma permissão informal ou fazendo
um pedido informal. Vejamos alguns exemplos:
You can park here. (Você pode estacionar aqui – Permissão)
It can happen to you one day. (Isto pode acontecer com você um dia – Possibilidade)
They can speak French. (Eles sabem/conseguem falar Francês – Habilidade)
I can't have done it! (Não posso ter feito isso! – Capacidade (como fui capaz de fazer isso!)
Can you help me? (Você pode me ajudar? – Pedido informal)

VERBO MODAL COULD


Could é usado quase que nas mesmas situações em que usamos can. Porém, com um “tom”
mais educado. Expressa expressar capacidade, habilidade, possibilidade, permissão formal e
pedido formal. Geralmente está presente em perguntas com um pouco de formalidade, já que
haverá outros modais para casos de formalidade de fato. Vejamos alguns exemplos:
Could you open the door, please? (Você poderia abrir a porta, por favor?)
I could see she was tired. (Eu podia ver que ela estava cansada.)
We could not smoke in that restaurant. (Nós não podíamos fumar naquele restaurante.)
If I win the lottery, I could buy a new house. (Se eu ganhasse na loteria, eu podia/poderia
comprar uma casa nova).

AULA 02 – VERBS IN TEXTS 30


TEACHER ANDREA BELO

VERBO MODAL MAY


May é usado para indicar permissão e possibilidade. Pode também ser usado para
expressar ações e acontecimentos que serão possíveis no futuro e no presente. Pode expressar
deduções, fazer um pedido, pedir ou dar permissão. Pode oferecer ajuda. Vejamos alguns
exemplos:
➢ May I help you? (Posso lhe ajudar? (*Aqui podemos usar can ou may na linguagem oral,
cotidiana, informal, mas, na escrita, o can seria inapropriado e o may, totalmente
adequado)
➢ I may call you later when I leave my job. (Eu posso ligar para você mais tarde quando eu
sair do meu trabalho.)
➢ She may not know what happened. She looks like suspicious. (Ela não deve saber o que
aconteceu. Ela parece suspeita.)
➢ He may be sick, he didn’t come. (Ele deve estar doente, ele não veio. / É possível que esteja
doente.)

VERBO MODAL MIGHT


Might é usado para indicar permissões mais formais, possibilidades remotas. Também
pode ser usado para descrever ações e acontecimentos possíveis em um momento futuro ou
presente, pode expressar deduções, às vezes fazer pedidos ou dar permissão. É importante
destacar que, might passa uma ideia de frases mais polida, ou seja, mais formal e por isso é menos
usado que os demais. Vejamos exemplos.
➢ Might I borrow you pen? Mine is not on my table. (Posso pegar sua caneta emprestada? A
minha não está e minha mesa.)
➢ It might rain later because it is cloudy. (Pode ser que chova mais tarde/ Talvez chova mais
tarde porque está nublado.)
➢ He might tell you why he got disappointed. (Pode ser que ele te conte por que ele ficou
decepcionado)
➢ She might get home because of the traffic. (Ela deve chegar em casa tarde por causa do
trânsito.)
➢ Why did he leave? I don’t know. Maybe might needed. (Por que ele foi embora? Não sei.
Talvez ele precisasse ir.)

AULA 02 – VERBS IN TEXTS 31


TEACHER ANDREA BELO

VERBO MODAL MUST


Must é usado para exprimir obrigações e deduções (se for na afirmativa) e expressar
proibição (na negativa) – must not/mustn’t. Vejamos exemplos:

➢ You must help her, it’s your mom! (Você deve ajudá-la, é sua mãe!)

➢ The doctor said you must stop smoking, or you’ll die. (O médico falou que você deve parar
de fumar ou você vai morrer)

➢ Children must not watch this video. (As crianças não devem assistir esse vídeo.)

Bom, como must não tem forma específica para o tempo passado, usamos had to para
expressar uma obrigação no passado:

➢ Yesterday I had to work up to 10pm. (Ontem tive que trabalhar até às 10 da noite.)

Em frases afirmativas, o must também pode ser substituído por have to para expressar
obrigação. Significarão dever, ter que fazer algo, must e have to com a mesma função e tradução.
Geralmente, na fala, linguagem informal, percebemos que o have to é usado com mais frequência
do que o must:

➢ I must study for my test. (Eu tenho que/devo estudar para minha prova).

➢ I have to study for my test. (Eu tenho que/devo estudar para minha prova).

VERBOS MODAIS SHOULD/OUGHT TO


Should ou Ought to são modais usados na mesma função, com o mesmo significado. São
ambos usados para aconselhar, exprimir expectativas ou obrigações menos intensas. Vejamos
exemplos:

➢ They should/ought to wear seat belt while driving. (Eles deveriam usar cinto de segurança
no carro enquanto está dirigindo)

➢ You should not/ought not to walk alone after 9pm. (Você não deveria andar sozinho após
9h da noite)

➢ You should not/ought not to accept offers from strangers. (Você não deveria aceitar
propostas de estranhos.)

AULA 02 – VERBS IN TEXTS 32


TEACHER ANDREA BELO

VERBO MODAL SHALL


Shall é usado para formar orações que remetem a ações futuras, que ainda vão acontecer.
Shall só é usado na primeira pessoa do singular (I) e do plural (We). Este modal é visto com mais
frequência em perguntas ou quando se oferece algo, sugerindo alguma coisa ou fazendo algum
convite É considerado bem formal, expressa polidez. Vejamos exemplos:

➢ You can count on me. I shall arrive tomorrow. (Você pode contar comigo. Eu chegarei
amanhã.)

➢ We shall arrive tomorrow. (Nós chegaremos amanhã.)

➢ Shall I call her? (Ligo para ela? – como se fosse uma dúvida se ligo ou não para a pessoa,
que pode estar ocupada ou não quer falar.)

➢ Shall I open the window? (Abro a janela? – como se fosse uma dúvida se o clima está quente
ou frio e se a pessoa realmente pode abrir a janela.)

➢ Shall I carry your luggage? (Quer que eu carregue sua bagagem?)

VERBOS MODAIS WILL E WOULD


Will e Would são modais, porém são muito mais usados na função de auxiliar do futuro e
frases condicionais, consecutivamente. Will, acompanhando verbos principais, coloca-os no
tempo futuro, como já vimos no capítulo específico do tempo futuro – I will travel tomorrow – Eu
viajarei amanhã. Would é basicamente a característica da polidez, da delicadeza. E acompanha
verbos principais, colocando-os na função de condicional – I would travel if I had money. (Eu
viajaria se tivesse dinheiro. O would também é usado como o passado de will. Vejamos exemplos:

➢ I will study very much on the weekend. (Vou estudar muito no fim de semana).

➢ Will you cook lunch? (Você vai fazer almoço?)

➢ Would you walk on the cemetery at night? (Você andaria no cemitério a noite?)

Agora, estudaremos o tempo verbal Imperative, que geralmente usamos para dar ordens.
Mas, veremos que há outros usos do imperativo. Vamos lá! Come on!

AULA 02 – VERBS IN TEXTS 33


TEACHER ANDREA BELO

IMPERATIVE TENSE
Imperative Tense é o tempo verbal em Inglês usado para expressar ordens, pedidos,
oferecer instruções e também para aconselhar alguém.

Para elaborar frases no Imperativo, basta usar os verbos em Inglês no infinitivo (sua forma
original sem conjugação) sem a preposição “to”. O “to” é usado para mostrar os verbos separados,
ou seja, fora das frases, não contextualizados.

Lembre-se de que o Imperativo é usado apenas com o verbo em sua forma infinitiva, sem
a preposição “to” e, quase 100% das vezes, no início de frases. Mas o modo Imperativo não é, por
sua vez, conjugado no passado ou na forma contínua. Não há essas variações no Imperativo.

E, para expressar negação, usa-se o Don’t no início das frases, como por exemplo Don’t
repeat that. (Não repita isso).

Quando um verbo é mencionado com “to”, geralmente é antes da frase ser elaborada,
apontando a ação a ser usada em determinada oração: to go (verbo ir) Students go to the
university – Alunos vão à universidade.

O uso do tempo Imperativo costuma ser direto e, às vezes, passa a impressão de que a
pessoa foi rude ou um pouco sem educação.

Isso porque é fácil perceber que, curiosamente, a palavra please, inserida no contexto de
um pedido ou ordem, é bem mais usada em países falantes da língua inglesa do que por nós,
brasileiros.

São diferenças culturais que não devem ser julgadas já que isso não torna ninguém melhor
ou pior do que o outro, mas sim, mostra-nos o quanto há variedades linguísticas e formas diversas
de se expressar como pessoa.

Então, é bom que você esteja atento que, em um pedido ou uma ordem, por exemplo, com
o intuito de amenizar e não parecer ser grosseiro, o sujeito certamente usará a palavra please nos
textos e nos exercícios que analisaremos e já estamos analisando nas aulas.

Vejamos exemplos de frases no modo Imperativo:

Turn on the TV, please. (Ligue a TV, por favor).

Look at the book now, please. (Olhe para o livro agora, por favor).

Hey, John, bring me a cup of water, please. (Ei, John, traga-me um copo de água, por favor).

Go fast! (Vá rápido!)

AULA 02 – VERBS IN TEXTS 34


TEACHER ANDREA BELO

Listen to your teacher. (Ouça sua professora).

Sit down/ Stand up. (Sente-se/Levante-se)

Close the door and the window. (Feche a porta e a janela).

Be careful. (Tome cuidado).

Existem muitas situações em que encontramos frases no Imperativo. Por exemplo, nas
placas de sinalização em nossa cidade: stop (pare), Push (empurre), Insert the coin (insira a
moeda).

Se você analisar, os manuais de instrução de qualquer assunto, tais como a montagem de


um produto novo, um eletrodoméstico que você tenha comprado ou até mesmo as receitas
culinárias, também são cheias deles.

Encontramos frases imperativas em ordem, sequência que devemos seguir para montar
algo ou preparar alguma coisa: “first, you...” (primeiro, você...), “then, you...” (então você...), “so,
you...” (daí, você...) “next, ...” (em seguida, ...), “after, ...” (depois, ...) e assim por diante com
verbos no Imperativo, para seguir os passos e cumprir uma meta. Exemplos:

First, break the egg. And then, join the flour. (Primeiro, quebre o ovo e então, junte à
farinha)

First, connect the cables. Then, plug it. Finally, check your internet connection and...
(Primeiro, ...)

FIRST > Primeiro, ...


NEXT > Em seguida, ...
THEN > Então, ...
AFTER > Depois, ...
SO > Daí, ...
FINALLY > Para concluir, ...
Agora, estudaremos um pouco sobre os Phrasal verbs, já que depois teremos uma aula
exclusiva para explorar a fundo esse tema. Vamos lá!

AULA 02 – VERBS IN TEXTS 35


TEACHER ANDREA BELO

PHRASAL VERBS
É um assunto que necessita de atenção porque é considerado difícil, mas, varemos de
forma prática os que mais caíram em provas anteriores, com desenhos, para ajudar a memorizar.

Phrasal Verbs, definidos de uma maneira mais simples, são verbos que vem acompanhados
por preposições ou advérbios.

Ou seja, é uma combinação de palavras formada por um verbo e uma preposição ou


advérbio.

São também conhecidos como verbos preposicionados ou, em alguns livros e gramáticas,
classificados como expressões verbais, porque esses verbos especiais, quando combinados com
partículas adverbiais ou com preposições, mudam completamente o significado do verbo usado
em sua composição.

Quando você tenta traduzir essas combinações, esses phrasal verbs, palavra por palavra,
elas poderão ficar totalmente sem sentido, já que são verbos interpretados sempre em conjunto.

Para exemplificar, vamos pensar no verbo to call, que, em Português, significa chamar ou
ligar, telefonar (I called you last night = Eu liguei para você ontem a noite).

Esse verbo, quando usado junto às preposições in e off, por exemplo, tornam-se outros
verbos com outros significados, veja:

To call in: convidar – I will probably call my neighbor in to the party – Eu provavelmente
vou convidar meu vizinho para a festa.

To call off: cancelar – I have to call off the meeting with you, I’m sorry – Eu tenho que
cancelar a reunião com você, desculpe-me.

Como afirmei que os phrasal verbs não podem ser traduzidos literalmente, a melhor forma
de aprendê-los é praticando: respondendo exercícios e lendo textos, fontes da sua prova.

Quanto mais intensificado for seu estudo, mais vocabulário, incluindo phrasal verbs, você
aprenderá.

Esse assunto é tão importante em Inglês, que existem vários dicionários de phrasal verbs.

Por isso, trataremos de regras, explicações variadas, maneiras de usá-los, preposições mais
utilizadas na construção dos phrasal verbs, entre outros detalhes essenciais, com a sugestão de
alguns deles que aparecem com frequência nas provas.

AULA 02 – VERBS IN TEXTS 36


TEACHER ANDREA BELO

PREPOSITION AWAY
Vou fazer da seguinte forma. Apontar a lista com todos os phrasal verbs mais usados, que
são formados com a preposição AWAY, as traduções e exemplos.
Assim, você pode estudar, observando-os por ordem alfabética e pelos exemplos, para
fazer sentido para você e facilitar na compreensão e memorização.

ABSTRACT AWAY: ignorar, abstrair, omitir:


“You can abstract away the complexity of life and enjoy it”.
(Você pode ignorar a complexidade da vida e curtir!)

BANG AWAY: dedicar-se muito, “bater na mesma tecla”:


“She has been banging away at English classes”.
(Ela tem se esforçado muito nas aulas de Inglês)

BEAR AWAY: afastar, suportar, “carregar algo (dor)”:


“He had to carry away that idea from his head”.
(Ele teve que afastar/tirar aquela ideia da sua cabeça)

BLAST AWAY: disparar, detonar, explodir.


“Her feelings blasted away and she started to cry”.
(Os sentimentos dispararam/explodiram e ela começou a chorar).

BLOW AWAY: voar/ir com o vento, impressionar:


“The paper plane blew away but I got it”.
(O avião de papel vôou com o vento mas consegui pegá-lo).

AULA 02 – VERBS IN TEXTS 37


TEACHER ANDREA BELO

BOTTLE AWAY: engarrafar, guardar algo para si, resguardar:


“Stop bottling away and tell me what is going on”.
(Pare de guardar para si/esconder e me diga o que está acontecendo)

PREPOSITION ABOUT
BANG ABOUT: executar algo com barulho, fazer barulho:
“The boy banged about in the kitchen last night”.
(O garoto fez muito barulho na cozinha ontem a noite).

BOSS ABOUT: querer mandar, abusar da autoridade:


“He was bossing about and made me nervous”.
(Ele estava abusando/mandando demais e me deixou nervosa)

BRING ABOUT: ocasionar, provocar:


“If you think well, you can bring about some changes”.
(Se você pensar bem, pode ocasionar/trazer algumas mudanças)

COME ABOUT: acontecer, resultar:


“The production has come about with technology”.
(A produção acanteceu/resultou com a technologia)

DOSS ABOUT: enrolar, desperdiçar tempo, retardar


“You did nothing but dossed about”.
(Você não fez nada a não ser enrolar )

AULA 02 – VERBS IN TEXTS 38


TEACHER ANDREA BELO

FALL ABOUT: cair no riso, dar gargalhadas sem parar:


“It was so funny she couldn’t stop falling about”.
(Foi tão engraçado que ela não conseguia parar de rir).

JUMP ABOUT: saltar repentinamente, saltar radicalmente:


“Children were jumping about because of the Christmas gifts”.
(As crianças estavam saltitando empolgadas
por causa dos presentes de Natal).

LAY ABOUT: partir para cima, atacar, bater forte em alguém:


“He bothered her so much that she decided to lay about him”.
(Ele a incomodou tanto que ela decidiu partir para cima dele.)

MILL ABOUT: circular, dar voltas (esperando algo/alguém):


“The gang was milling about all night long”.
(A gangue esteve circulando/agaurdando a noite toda).

PUSH ABOUT: “tirar sarro”, fazer bullying:


“The tall guy was pushing about the boy”.
(O cara alto cometeu bullying com o garoto.)

SET ABOUT: lançar algo, começar, “dar o pontapé inicial”:


“How did you set about solving a puzzle?”.
(Como você começou a resolver o quebra-cabeça?).

AULA 02 – VERBS IN TEXTS 39


TEACHER ANDREA BELO

PREPOSITION BACK
BOUNCE BACK: dar a volta por cima, recuperar-se:
“He bounced back and won the competition”.
(Ele deu a volta por cima e ganhou a competição).

CALL BACK: ligar de volta, retornar à ligação:


“He finally called me back after na hour”
(Ele finalmente me ligou de volta após uma hora).

DIE BACK: perder as folhas (árvore/plantas), morrer:


“The trees always die back after flowering”.
(As árvores sempre perdem as folhas após florir.)

DROP BACK: ficar para trás, cair (preço), ter um declínio:


“Nothing can stop the progress unless the sales to drop back”.
(Nada pode parar o progresso a não ser que as vendas caiam ).

FALL BACK/HANG BACK: recuar, afastar-se, cair, diminuir:


“The enemies fell back because of the police”.
(Os inimigos recuaram por causa da polícia.)

HAND BACK: devolver, entregar de volta (sinônimo de GIVE BACK):


“She got the coins and gave back to him”.
(Ela pegou as moedas e devolveu a ele).

AULA 02 – VERBS IN TEXTS 40


TEACHER ANDREA BELO

LAY BACK/LIE BACK: virar-se de costas, deitar de costas:


“The doctor asked me to lie back for the tests”.
(O médico me pediu para deitar de costas para os exames).

PIN BACK: prender para trás (cabelo, franjas, cortina...):


“He asked me to pin back the curtains.”
(Ele me pediu para prender as cortinas para trás).

PLOW BACK: reinvestir, reabrir:


“He plowed back the company”.
(Ele reabriu/reinvestiu a (na) empresa.)

SHRINK BACK: retrair, recuar, encolher:


“After all, she decided to shrink back”.
(Depois de tudo, ela decidiu recuar).

STRIKE BACK/ TIE BACK: revidar, contra-atacar:


“They were fighting when Tom decided to strike back”.
(Eles estvam brigando quando Tom decidiu revidar)

WIN BACK: reconquistar, recuperar:


“I am trying to win back my losses”.
(Eu estou tentando recuperar minhas perdas).

AULA 02 – VERBS IN TEXTS 41


TEACHER ANDREA BELO

PREPOSITION IN
ASK IN: convidar para entrar:
“She was at the door and asked me in”.
(Ela estava na porta e me convidou para entrar).

BASH IN: destruir, bater forte, atacar:


“The robbed bashed in the window with the crowbar.”
(O ladrão bateu/forçou a janela com o pé de cabra).

BEAR IN: levar em conta, suportar, considerar.


“I think you have to bear it in mind”.
(Eu acho que você deve levar em conta/considerar isso..)

FALL IN: Fall in love = apaixonar-se


“She suddenly fell in love”.
(Ela de repente se apaixonou).

FILL IN: preencher, completar (formulário)


“You should fill in with all information”.
(Você deveria preencher com todas as informações.)

FIT IN: encaixar-se:


“The nail didn’t fit into the hole”.
(O prego não se encaixou no buraco)

AULA 02 – VERBS IN TEXTS 42


TEACHER ANDREA BELO

GIVE IN: entregar/desistir/”deixar de lutar”:


“He gave in his report but his sister gave in doing that”.
(Ele entregou seu relatório mas sua irmã desisiu de fazer).

HANG IN: ser positivo, ser otimista.


“Just hang in, you’ll find a good job!”
(Apenas seja otimista, você vai encontrar um bom emprego).

HAND IN: entregar


“If you hand in the exercise in time, you get an extra point”.
(Se você entregar o exercício a tempo, ganha ponto extra).

HEDGE IN/HEM IN: cercar (algo ou alguém):


“We are all hedged in by rules”
(Estamos todos cercados de regras).

JOIN IN: aderir, reunir-se, juntar-se.


“You joined in the party. It was great”.
(Você aderiu/se juntou à festa. Foi ótimo.)

KEEP IN: manter, guardar (*Keep in touch: manter contato)


“Alright, let’s keep in touch”.
(Tudo bem, vamos manter contato!).

AULA 02 – VERBS IN TEXTS 43


TEACHER ANDREA BELO

KEY IN: teclar


“Wow! You key in too fast!”.
(Uau! Você tecla muito rápido!)

LEAN IN: inclinar, curvar-se, esticar o corpo:


“I had to lean in to hear the teacher”.
(Eu tive que me inclinar para ouvir o professor).

MOVE IN: mudar-se, avançar


“I moved in a few weeks ago”.
(Eu me mudei algumas semanas atrás).

POP IN: “dar uma passadinha”, visitar rapidinho:


“I am leaving, I just poped in to have some coffee”
(Já vou, só passei para um café com você).

PRICE IN: aumentar o preço, subir o valor.


“The store priced in all the products!”
(A loja subiu o preço de todos os produtos!)

PULL IN: encostar, chegar, prender


“You can pull in here”. “The police pulled him in”
(Você pode enconstar aqui). (A polícia o prendeu.)

AULA 02 – VERBS IN TEXTS 44


TEACHER ANDREA BELO

RAKE IN: ganhar, faturar


“Her shop is raking it in a lot”.
(A loja dela está faturando muito!)

SHUT IN: trancafiar, prender alguém/algo em algum lugar:


“He got angry and shut me in there for an hour”
(Ele ficou nervoso e me prendeu lá por uma hora).

SIGN IN: entrar, acessar, ter acesso.


“You can sign in the website anytime”
(Você pode acessar o website a qualquer hora.)

PREPOSITION ON
ADD ON: incluir, adicionar, aumentar,
“He added 10 dollars on for service.”
(Ele incluiu/adicionou 10 dólares pelo serviço).

BEAR ON (ou bear upon): afetar, influenciar, causar impacto.


“The facts bear on a lot for the final decision”.
(Os fatos influenciam muito na decisão final).

BRING ON: fazer acontecer, causar


“I ask myself what I’ve done to bring this on”.
(Eu me pergunto o que fiz para causar isso).

AULA 02 – VERBS IN TEXTS 45


TEACHER ANDREA BELO

CALL ON: pedir, chamar, recorrer.


“We are calling on him for president”
(Estamos recorrendo a ele para presidente).

FIGURE ON: planejar


“He is figuring on his future!”
(Ele está planejando o futuro)

GET ON: embarcar, progredir, continuar, dar-se bem


“I got on the train”. “Stop complaining and get on!”
(Eu embarquei no trem). (Pare de reclamar e continue!)

GO ON: continuar, começar a fazer algo, funcionar, basear-se


“Please, go on!”. “She went on the pills” “There are no clues to go on”
(Por favor, continue!) (Ela começou a tomar o remédio)
(Não há pistas para se basear/funcionar a investigação)

GROW ON: conquistar


“He is not handsome but he grew me on”.
(Ele não é bonito mas ele me conquistou).

HIT ON: “dar em cima”, paquerar”


“Patrick? The girls are all hitting on him”.
(Patrick? As garotas estão todas dando em cima dele).

AULA 02 – VERBS IN TEXTS 46


TEACHER ANDREA BELO

KEEP ON: continuar (sinônimo de go on)


“Keep on walking, como on!”
(Continue nadando, vamos!).

LAY ON: colocar, botar, assentar


“I laid my books down the table”
(Coloquei meus livros na mesa .)

MOVE ON: mudar, prosseguir, mover-se


“You have to move on, let’s go!”
(Você tem que mudar/seguir em frente. Vamos!)

PICK ON: atormentar, importunar


“That boy likes to pick on everybody”
(Aquele garoto gosta de importunar/atormentar a todos)

PUT ON: colocar (roupa, acessório), vestir, aplicar


“She put on a jacket and left”.
(Ela colocou a jaqueta e saiu).

RELY ON: depender de alguém, contar com


“The baby counts on his mom to walk”.
(O bebê conta com a mãe para andar).

AULA 02 – VERBS IN TEXTS 47


TEACHER ANDREA BELO

SWITCH ON: acender, ligar, animar-se


“I witched on all lights”
(Eu liguei/acendi todas as luzes!).

TAKE ON: contratar, assumir


“The game took on a different meaning to me”
(O jogo assumiu um significado diferente para mim .)

TELL ON: denunciar, relatar, dedurar


“You can’t tell on me, please!”
(Você não pode me denuciar/
dedar, por favor!)

TRY ON: experimentar, provar.


“I want try on my new shoes”
(Eu quero experimentar meus sapatos novos)

URGE ON: estimular, encorajar


“She urged his son to swim”.
(Ela encorajou seu filho a nadar).

WAIL ON: lamentar, dramatizar, reclamar


“She was wailing on”
(Ela estava lamentando/reclamando).

AULA 02 – VERBS IN TEXTS 48


TEACHER ANDREA BELO

WALK ON: entrar andando, sem pedir, de repente


“I walked in and saw all his guns”
(Eu entrei de uma vez e vi todas as armas dele.)

PREPOSITION OUT
ASK OUT: convidar para sair
“I want to ask you out but I am afraid you say no”
(Quero te convidar para sair mas tenho medo de você dizer não )

BAIL OUT: tirar a água do barco, resgatar, salvar


“You have to bail out before sailing”.
(Você tem que tirar a água do barco antes de navegar).

BLACK OUT: apagar, escurecer, desmaiar


“He blacked out when crashed the car”.
(Ele apagou/desmaiou quando bateu o carro).

BREAK OUT: começar (algo ruim)


“The fire broke out during the night”
(O fogo começou durante a noite).

BURN OUT: esgotar, estar cansado, exausto


(há a síndrome burn out, estar exausto, geralmente em
decorrência do trabalho, um esgotamento físico e mental).

AULA 02 – VERBS IN TEXTS 49


TEACHER ANDREA BELO

BRING OUT: produzir algo novo, ênfase no verbo trazer


“She is bringing out a new album” “She brought me a gift”
(Ela está produzindo um novo álbum/Ela me trouxe um presente.)

CALL OUT: anunciar, gritar


“He called out the winner”.
(Ele anunciou o vencedor).

CARRY OUT: conduzir, executar


“We carry out this project”.
(Nós conduzimos/executamos esse projeto).

COME OUT: vir para fora, sair, vir à tona


“The real story came out at the trial”
(A história real veio à tona no julgamento).

CRY OUT: clamar, gritar por medo, desespero


“She cried out when she got stuck in the lift”
(Ela gritou de desespero quando ficou presa no elevador.)

CUT OUT: cortar/excluir


“They cut out some movie scenes”
(Eles cortaram/excluíram algumas cenas do filme.)

AULA 02 – VERBS IN TEXTS 50


TEACHER ANDREA BELO

DROP OUT: largar, desistir


“She had to drop out college, it was hard to her”
(Ela teve que largar a faculdade, estava difícil para ela.)

EAT OUT: jantar fora, sair para jantar


“We didn’t eat in but decided to eat out”.
(Nós não jantamos em casa mas decidimos comer fora).

FIGURE OUT: encontrar algo, entender, resolver (um problema)


“I am trying to figure it out”
(Estou tentando entender/resolver isso).

FIND OUT: descobrir, ficar sabendo


“He found out the truth”
(Ele descobriu/ficou sabendo da verdade.)

FREEZE OUT: “dar um gelo” (em alguém), excluir


“She was frozen out of the group”
(Ela foi excluída do grupo.)

FREEK OUT: enlouquecer, pirar, “ter um treco”


“She freaked out at the concert”
(Ela elouqueceu/teve um treco/pirou no show.)

AULA 02 – VERBS IN TEXTS 51


TEACHER ANDREA BELO

GO OUT: sair, passear, viajar


“I would like to go out on vacation”.
(Eu gostaria de passear/viajar nas férias).

GROW OUT: crescer e não caber mais


“You grew up and your shoes don’t fit amore”.
(Você cresceu e seus sapatos não cabem mais).

HAND OUT: distribuir, divulgar


“The sentence was handed out”
(A sentença foi dada/divulgada).

HELP OUT: ajudar alguém com dificuldade


“Her parents helped her out with the payments”
(Os pais dela a ajudaram com os pagamentos.)

IRON OUT: esclarecer, resolver, explicar


“We can iron out the details”
(Nós podemos esclarecer os detalhes.)

LEAVE OUT: “deixar de fora”, excluir


“You can’t leave me out of the trip”
(Você não pode me deixar de fora da viagem.)

AULA 02 – VERBS IN TEXTS 52


TEACHER ANDREA BELO

MAKE OUT: discernir, passar a impressão de


“You made out what he has just said”.
(Você discerniu o que ele acabou de dizer).

PUT OUT: “apagar o fogo”, sanar, causar problemas


“Put the cigarrete out, please”
(Apague o cigarro, por favor).

REACH OUT: oferecer ajuda


“He wanted to reach out the group members”
(Ele queria oferecer ajuda aos membros do grupo.)

RAT OUT: dedurar, desistir


“He ratted out on the opportunity”
(Ele desistiu da oportunidade .)

RING OUT: celebrar o fim, ressoar


“A laughter rang out in the room”.
(Uma risada ressou na sala).

RUN OUT: ficar sem, faltar


“We’ve run out of sugar”
(Ficamos sem açucar.)

AULA 02 – VERBS IN TEXTS 53


TEACHER ANDREA BELO

SEEK OUT: procurar, buscar


“You must seek out new ways to study”
(Você deve buscar novas maneiras de estudar.)

PREPOSITION OVER
BEND OVER: agachar, fazer de tudo
“He bent over to help her”
(Ele fez de tudo para ajudá-la .)

COME OVER: vir, mudar de lado


“He came over”.
(Ele veio para o nosso lado).

GET OVER: superar


“I am sure he will get over it”
(Tenho certeza que ele vai superar isso)

GO OVER: revisar, praticar


“Let’s go over these lines, please”
(Vamos praticar essas falas, por favor.)

HAND OVER: entregar, “passar a bola”


“He passed over his power to her”.
(Ele passou o poder a ela ).

AULA 02 – VERBS IN TEXTS 54


TEACHER ANDREA BELO

PULL OVER: encostar


“The police ordered him to pull over”
(A polícia pediu para ela encostar)

TAKE OVER: assumir, controlar


“Can you take over the project?”.
(Você pode assumir o projeto?)

THINK OVER: pensar sobre, refletir


“What are you thinking over about?”
(No que você está pensando/refletindo?)

TALK OVER: discutir para chegar em um acordo


“We talked over our problems”
(Nós discutimos nossos problemas.)

PREPOSITIONS UP/DOWN
ACT UP: dar problema, parar de funcionar
“My computer is acting up again”
(Meu computador está dando problema de novo.)

BACK UP: fazer uma cópia, apoiar


“You should back up important documents”.
(Você deveria fazer cópia dos documentos importantes).

AULA 02 – VERBS IN TEXTS 55


TEACHER ANDREA BELO

BRING UP: criar(educar), mencionar


“Her parents brought her up well”
(Os pais dela a criaram bem)

END UP: finalizar, chegar em algum lugar


“We ended up the night dancing”
(Nós terminamos a noite dançando.)

GO UP: subir literalmente


“Prices are going up fast”.
(Os preços estão subindo rapidamente).

HANG UP: pendurar (ênfase no verbo)


“I hung up my coat on the hook”
(Eu pendurei meu casaco no cabide)

KEEP UP: continuar, manter


“Keep up working like this”
(Continue trabalhando assim.)

MAKE UP: inventar, constituir-se


“He is making up excuses”
(Ele está inventando desculpas.)

AULA 02 – VERBS IN TEXTS 56


TEACHER ANDREA BELO

LOOK UP: procurar, pesquisar no dicionário


“I looked it up many times”.
(Eu procurei isso no dicionário muitas vezes).

OPEN UP: abrir, abrir-se (ênfase)


“Open up! It’s the police!”.
(Abra! É a polícia!).

BREAK DOWN: descontrolar-se


“I had na emotional break down yesterday”
(Eu tive um descontrole emocional ontem.)

COME DOWN: descer, baixar a posição social


“The man was upset because he had come down”
(O homem estava decepcionado porque baixou sua posição social).

CUT DOWN: reduzir


“They need to cut down drinking”
(Eles precisam reduzir a bebida)

LET DOWN: desapontar


“I can’t let her down, I love her”
(Não posso desapontá-la, eu a amo)

AULA 02 – VERBS IN TEXTS 57


TEACHER ANDREA BELO

PUT DOWN: colocar algo no chão, em lugar baixo


“She put her bag down.”
(Ela colocou a bolsa no chão)

PREPOSITION FOR
ASK FOR/CALL FOR: pedir, solicitar
“I talked to him to ask for a job recommendation”
(Eu falei com ele para pedir uma recomendação de emprego)

FALL FOR: apaixonar-se


“She always falls for intelligent men”.
(Ela sempre se apaixona por homens inteligentes).

LOOK FOR: procurar, querer, desejar


“Some people look for work on internet”
(Algumas pessoas procuram emprego pela internet)

STAND FOR: significar


“He said SY stands for See You”
(Ele disse que SY significa See you.)

WAIT FOR: esperar, aguardar


“Can you wait for me?”.
(Você pode me esperar? )

AULA 02 – VERBS IN TEXTS 58


TEACHER ANDREA BELO

PREPOSITION OFF
BACK OFF: afastar, recuar
“They backed off when they see the police”
(Eles se afastaram ao ver a polícia.)

BUY OFF: subornar, “comprar” a pessoa


“Don’t worry, I bought him off”.
(Não se preocupe, eu o subornei).

CALL OFF: cancelar


“They called the wedding”
(Eles cancelaram o casamento)

FIGHT OFF: combater


“He wants to fight off the attackers”
(Ele quer combater os atacantes.)

LOG OFF: fazer o log off (se desconectar)


“The program is open, you have to log off”.
(O programa está aberto, você tem que fazer o log off)

AULA 02 – VERBS IN TEXTS 59


TEACHER ANDREA BELO

OUTROS PHRASAL VERBS


COME DOWN WITH: adoecer
“My brother came down with pneumonia on the weekend.”
(Meu irmão adoeceu de pneumonia no fim de semana.)

GET ALONG WITH: dar-se bem com


“They get along with each other”.
(Elas se dão muito bem).

LOOK AFTER: cuidar (sinônimo de take care)


“I looked up my nephew last week”
(Eu cuidei do meu sobrinho semana passada)

LOOK FOWARD TO: Esperar muito por algo, estar ansioso por
“I am looking for my vacation”.
(Estou ansiosa por minhas férias)

PUT UP TO: tolerar


“I don’t put up with more frustations”
(Eu não tolero mais frustações.)

Vamos aos exercícios para praticar os phrasal verbs bem como todo o conteúdo que temos
estudado no decorrer de nossas aulas. Let’s go!

AULA 02 – VERBS IN TEXTS 60


TEACHER ANDREA BELO

QUESTÕES
Você vai, agora, responder questões selecionadas de provas já realizadas em anos
anteriores e questões inéditas. Depois, como em todas as nossas aulas, haverá o gabarito e as
questões comentadas.
Vamos começar com questões AFA, de acordo coma sua instituição escolhida e depois,
vamos treinar de outras Carreiras Militares, para adquirir experiência e treinar vocabulário.
QUESTÕES AFA
Directions: Read the text below and answer questions 01 to 10 according to it.
The machine always wins: what drives our addiction to social media
We are swimming in writing. Our lives have become, in the words of the author and academic
Shoshana Zuboff, an “electronic text”. Social media platforms have created a machine for us to
write to. The bait is that we are interacting with other people: our friends, colleagues, celebrities,
politicians, royals, terrorists, porn actors – anyone we like. We are not interacting with them,
however, but with the machine. We write to it, and it passes on the message for us after keeping
a record of the data.
The machine benefits from the “network effect”: the more people write to it, the more benefits
it can offer, until it becomes a disadvantage not to be part of it. Part of what? The world’s first ever
public, live, collective, open-ended writing project. A virtual laboratory. An addiction machine,
which deploys crude techniques of manipulation redolent of the Skinner Box created by
behaviourist BF Skinner to control the behaviour of pigeons and rats with rewards and
punishments. We are users, much as cocaine addicts are users.
What is the incentive to engage in writing like this for hours each day? In a form of mass
casualisation, writers no longer expect to be paid or given employment contracts. What do the
platforms offer us, in lieu of a wage? What gets us hooked? Approval, attention, retweets, shares
and likes.
Meanwhile, hashtags and trending topics underline the extent to which all of these protocols are
organised around the massification of individual voices – a phenomenon cheerfully described by
users with the science-fiction concept of the “hive mind” – and hype. The regular sweet spot
sought after is a brief period of ecstatic collective frenzy around any given topic. It doesn’t
particularly matter to the platforms what the frenzy is about: the point is to generate data, one of
the most profitable raw materials yet discovered. As in the financial markets, volatility adds value.
The more chaos, the better.
Whether or not we think we are addicted, the machine treats us as addicts. Addiction is, quite
deliberately, the template for our relationship to the Twittering Machine. Addiction is all about
attention. For the social media bosses, this is axiomatic.
If social media is an addiction machine, the addictive behaviour it is closest to is gambling: a rigged
lottery. Every gambler trusts in a few abstract symbols – the dots on a dice, numerals, suits, red or
black, the graphemes on a fruit machine – to tell them who they are. In most cases, the answer is
brutal and swift: you are a loser and you are going home with nothing. The true gambler takes a
perverse joy in anteing up, putting their whole being at stake. On social media, you scratch out a

AULA 02 – VERBS IN TEXTS 61


TEACHER ANDREA BELO

few words, a few symbols, and press send, rolling the dice. The internet will tell you who you are
and what your destiny is through arithmetic likes, shares and comments.
The interesting question is what it is that is so addictive. In principle, anyone can win big; in
practice, not everyone is playing with the same odds. Our social media accounts are set up like
enterprises competing for attention. If we are all authors now, we write not for money, but for the
satisfaction of being read. Going viral, or trending, is the equivalent of a windfall. But sometimes,
winning is the worst thing that can happen. The temperate climate of likes and approval is apt to
break, lightning-quick, into sudden storms of fury and disapproval.
(Adapted from https://www.theguardian.com/technology/2019/aug/23/social-media-addiction-gambling)

QUESTÃO 01 (AFA/INÉDITA) – One of the purposes of the text is to show that


a) financial feedback is the major reason why people engage in social media writing.
b) there’s evidence cocaine is more addictive than social media and gambling.
c) anyone can be successful when it comes to social media.
d) social media can be as addictive as drugs.

QUESTÃO 02 (AFA/INÉDITA) – According to the first paragraph, it is correct to state that


a) social media platforms allow people to truly interact with other people.
b) when people communicate through social media, they, as a matter of fact, interact with the
machine.
c) lives nowadays are more based on personal contact.
d) people should avoid interacting on social media platforms.

QUESTÃO 03 (AFA/INÉDITA) – The best definition for “bait”, underlined in the first paragraph is
a) something attractive that is offered to make people act a certain way.
b) something that makes people think about their actions.
c) something unattractive that discourages people from acting a certain way.
d) something that makes people improve their attitude.

QUESTÃO 04 (AFA/INÉDITA) – In the sentence “We are not interacting with them, however, but
with the machine” (paragraph 1), the highlighted word “however” can be replaced, with no
change in meaning, by
a) besides
b) though
c) then
d) since

AULA 02 – VERBS IN TEXTS 62


TEACHER ANDREA BELO

QUESTÃO 05 (AFA/INÉDITA) – Mark the INCORRECT option according to the text.


a) addiction to social media can be compared to gambling addiction.
b) social media platforms have created a machine that treats people as addicts.
c) there are no explanations on why people engage in writing online for hours each day.
d) in practice, not everyone has the same chance of being successful online.

QUESTÃO 06 (AFA/INÉDITA) – Approval, attention, retweets, shares and likes are mentioned in
the text as examples of
a) online communication negative impacts.
b) emergency measures to overcome internet addiction.
c) reasons why people get addicted to social media.
d) reasons why people usually avoid social media.

QUESTÃO 07 (AFA/INÉDITA) – Read the statements below and mark the option that contains the
correct ones according to the text.
I – It doesn’t matter if people consider themselves addicts; the machine treats them this way.
II – Attention is the main key to addiction.
III – Theoretically, anyone can succeed online. In reality, not everybody has the same chances of
succeeding.
a) Only sentences I and II are correct.
b) Only sentences I and III are correct.
c) Only sentences II and III are correct.
d) All sentences are correct.

QUESTÃO 08 (AFA/INÉDITA) – Choose the best option to change the underlined sentence “Social
media platforms have created a machine [...]” (paragraph 1) into the passive form.
A machine ____________________ by social media platforms.
a) has been created
b) has created
c) had been created
d) had created

AULA 02 – VERBS IN TEXTS 63


TEACHER ANDREA BELO

QUESTÃO 09 (AFA/INÉDITA) – The word “meanwhile” (paragraph 4) indicates in the text that
a) some facts happen at different times.
b) a single fact interferes in others.
c) one fact is more important than others.
d) some facts happen at the same time.

QUESTÃO 10 (AFA/INÉDITA) – In the sentence “We are users, much as cocaine addicts are users.”
(paragraph 2), the author DOESN’T
a) make a comparison.
b) treat drug users and internet addicts as different.
c) bring impacting information about social media addiction.
d) describe social media addicts in a certain way.

QUESTÕES COLÉGIO NAVAL


Directions: Answer questions 1 to 10 according to TEXT I.
Many COVID-19 patients lost their sense of smell. Will they get it back?
IN EARLY MARCH, Peter Quagge began experiencing COVID-19 symptoms, such as chills and a low-
grade fever. As he cut pieces of raw chicken to cook for dinner one night, he noticed he couldn’t
smell the meat. “Must be really fresh,” he remembers thinking. But the next morning he couldn’t
smell the Dial soap in the shower or the bleach he used to clean the house. “It sounds crazy, but
I thought the bleach had gone bad,” he says. When Quagge stuck his head into the bottle and
took a long whiff, the bleach burned his eyes and nose, but he couldn’t smell a thing.
The inability to smell, or anosmia, has emerged as a common symptom of COVID-19. Quagge was
diagnosed with COVID-19, though he was not tested, since tests were not widely available at the
time. He sought anosmia treatment with multiple specialists and still has not fully recovered his
sense of smell.
Case reports suggest that anywhere between 34 and 98 percent of hospitalized patients with
COVID-19 will experience anosmia. One study found that COVID-19 patients are 27 times more
likely than others to lose their sense of smell, making anosmia a better predictor of the illness
than fever.
For most COVID-19 patients who suffer anosmia, the sense returns within a few weeks, and
doctors don’t yet know if the virus causes long-term smell loss. While not being able to smell may
sound like a small side effect, the results can be devastating. The sense is intricately tied up in
self-preservation—the ability to smell fire, chemical leaks, or spoiled food—and in our ability to
pick up on complex tastes and enjoy food.
“So many of the ways we connect with each other is over meals or over drinks,” says Steven
Munger, director of the Center for Smell and Taste at the University of Florida. “If you can’t fully
participate in that, it creates a sort of social gap.”

AULA 02 – VERBS IN TEXTS 64


TEACHER ANDREA BELO

Smell even plays a role in our emotional lives, connecting us to loved ones and memories. People
without smell often report feeling isolated and depressed and losing their enjoyment in intimacy.
Now scientists are starting to unravel how COVID-19 affects this critical sense, hoping those
discoveries will help thousands of newly anosmic people looking for answers.
What the nose knows
The olfactory system, which allows humans and other animals to smell, is essentially a way of
decoding chemical information. When someone takes a big sniff, molecules travel up the nose to
the olfactory epithelium, a small piece of tissue at the back of the nasal cavity.
Those molecules bind to olfactory sensory neurons, which then send a signal by way of an axon,
a long tail that threads through the skull and delivers that message to the brain, which registers
the molecules as, say, coffee, leather, or rotting lettuce.
Scientists still don’t fully understand this system, including exactly what happens when it stops
working. And most people don't realize how common smell loss really is, Munger says. “That lack
of public understanding means there’s less attention to try to understand the basic functions of
the system.”
People can lose their sense of smell after suffering a viral infection, like influenza or the common
cold, or after a traumatic brain injury. Some are born without any sense of smell at all or lose it
because of cancer treatments or diseases like Parkinson’s and Alzheimer’s. It may also fade as
people age. While smell disorders aren’t as apparent as hearing loss or vision impairment, data
from the National Institutes of Health (NIH) show that nearly 25 percent of Americans over the
age of 40 report some kind of change in their sense of smell, and over 13 million people have a
measurable disorder like anosmia, the total loss of smell, or hyposmia, a partial loss. Such
conditions can last for years or even be permanent.
It’s not clear if COVID-19 anosmia is different from other instances of smell loss caused by a virus,
but those who experience anosmia due to COVID-19 appear to be unique in a few ways. First, they
notice the loss of the sense immediately because it’s not accompanied by the congestion or
stuffiness that generally characterizes the early stages of virally induced anosmia.
“It’s very dramatic,” says Danielle Reed, associate director of the Monell Chemical Senses Center
in Philadelphia, which studies smell and taste loss. “People just cannot smell anything.”
Another notable difference is that many patients with COVID-19 who report losing their sense of
smell get it back relatively quickly, in just a few weeks, unlike most people who experience
anosmia from other viruses, which can last months or years.
Quagge estimates he’s recovered about 60 percent of his sense of smell so far, but he says in the
early days, without any information about when or if he’d ever get it back, he was scared. An avid
amateur chef, he had to rely on his family to tell him if the milk was bad, and he couldn’t smell his
wife’s perfume. “Stuff that gets to your soul,” he says. “It bummed me out.”
(Adapted from https://www.nationalgeographic.com/science/2020/08/thousands-covid-19-patients-lost-sense-smell-will-get-back-cvd/)

AULA 02 – VERBS IN TEXTS 65


TEACHER ANDREA BELO

QUESTÃO 01 (COLÉGIO NAVAL/INÉDITA) – According to the text, mark the correct alternative.
a) COVID-19 patients lose their sense of smell as a common symptom.
b) Anosmia has been considered a better predictor of the illness than fever.
c) As a consequence, the virus causes long-term smell loss.
d) Anosmia was diagnosed in all COVID-19 patients.

QUESTÃO 02 (COLÉGIO NAVAL/INÉDITA) – Mark the correct option.


a) Doctors claim that the virus causes long-term smell loss.
b) Doctors claim that the virus does not cause long-term smell loss.
c) Doctors affirm that the smell loss is inevitable for those who have been affected by the virus.
d) Doctors still don't know if the virus causes long-term smell loss

QUESTÃO 03 (COLÉGIO NAVAL/INÉDITA) – There are __ verbs used in the past in the underlined
sentence in the tenth paragraph.
a) One.
b) two.
c) three.
d) zero.

QUESTÃO 04 (COLÉGIO NAVAL/INÉDITA) – Mark the option that can replace the word "very" in
the eleventh paragraph without changing its meaning.
a) a lot.
b) really.
c) also.
d) to.

QUESTÃO 05 (COLÉGIO NAVAL/INÉDITA) – Mark the alternative that can answer the question
below according to the text.
Will people get their sense of smell back after recovering from COVID-19?
a) No, it's a long-term smell loss.
b) Maybe, it depends on the patient.
c) Yes, 100% of the patients have recovered their sense of smell after COVID-19.
d) We can't say yet, health authorities still don't know if anosmia could be long term smell loss.

AULA 02 – VERBS IN TEXTS 66


TEACHER ANDREA BELO

QUESTÃO 06 (COLÉGIO NAVAL/INÉDITA) – Read the sentence. "While not being able to smell
may sound like a small side effect, the results can be devastating." Choose the alternative that
expresses a similar idea to the sentence above:
a) The results from anosmia aren't harmful to health.
b) Everyone has hurtful results from the total loss of smell.
c) Even though it may sound like a non-harmful symptom, smell loss can be destructive.
d) Not being able to smell sounds like a small side effect, with good results regardless of the
case.

QUESTÃO 07 (COLÉGIO NAVAL/INÉDITA) – According to the first paragraph, we can assume that:
a) At first, Peter noticed that he was having COVID-19 symptoms.
b) In the first moment, he just felt the loss of smell sense.
c) Peter's first symptoms were chills, anosmia and low-grade fever.
d) He was an asymptomatic case.

QUESTÃO 08 (COLÉGIO NAVAL/INÉDITA) – Mark the INCORRECT alternative, according to the


third paragraph:
a) COVID-19 patients will experience anosmia as a symptom
b) anosmia it's better than fever to diagnose COVID-19.
c) COVID-19 patients tend to have anosmia, differently from others.
d) loss of smell is a common symptom from COVID-19.

QUESTÃO 09 (COLÉGIO NAVAL/INÉDITA) – What does the expression "Bummed me out"


(thirteenth paragraph) mean?
a) makes me feel down.
b) makes me feel happy.
c) makes me feel angry.
d) makes me feel nauseated.

QUESTÃO 10 (COLÉGIO NAVAL/INÉDITA) – Considering the use of the verb tense, mark the
alternative that completes the sentence below correctly: At first, Peter
a) has the common symptoms.
b) was having the common symptoms.
c) have the common symptoms.
d) had the common symptoms.

AULA 02 – VERBS IN TEXTS 67


TEACHER ANDREA BELO

QUESTÕES EAM
Questão 01 (EAM/INÉDITA) – Read the dialogue and mark the right option to fill in the gaps
respectively.
A: __________ you like the President’s speech yesterday?
B: Yes, I __________.
A: Who __________ you watch with?
B: I __________ with my girlfriend. Did you watch it?
A: No, I __________.
a) Did / did / went / watch / wasn’t
b) Were / was / were / watch / didn’t
c) Was / was / did / watched / wasn’t
d) Were / was / did / watched / didn’t
e) Did / did / did / watched / didn’t

Questão 02 (EAM/INÉDITA) – Read the sentences and mark the correct option to fill in the
blanks respectively.
__________ name is Tom. I’m 16 and I live __________ Warsaw which is the capital of Poland.
Actually, I come from another city - Wroclaw, where I __________ for eleven years.
Adapted from (https://sciaga.pl/tekst/2266-3-how_to_write_a_about_myself_an_example)

a) My / in / lived
b) Your / at / lived
c) Her / on / lives
d) My / in / live
e) His / in / live

AULA 02 – VERBS IN TEXTS 68


TEACHER ANDREA BELO

QUESTÃO 03 (EAM/INÉDITA) – What’s the main verb tense used in the comic strip?
a) Present Continuous
b) Present Simple
c) Simple Past
d) Future Simple
e) Present Perfect

QUESTÃO 04 (EAM/INÉDITA) – Use the verbs in the parentheses to complete the following
statements.
I – she __________ (study) law at the University of Warsaw.
II – Some time later, I __________ (notice) that I could do a lot more things with the computer.
III – She used to __________ (do) the dishes at night.
Now mark the option which completes them respectively.
a) study / noticed / do
b) studies / noticed / doing
c) studied / notice / doing
d) studies / noticed / do
e) study / notice / does

QUESTÃO 05 (EAM/INÉDITA) – Look at the picture below.

What are they doing in the picture?


a) They are trying to stop the girl from passing.
b) They are playing jump rope.
c) They are making the girl fall down.
d) They are throwing the rope on the floor.
e) They don’t like to play with the girl.

AULA 02 – VERBS IN TEXTS 69


TEACHER ANDREA BELO

QUESTÕES EEAR
Read the text answer question 01
Douglas Yancey, "Doug" Fannie, is the titular character and chief protagonist of the animated
series Doug. Douglas is a shy, insecure, and somewhat clumsy 11-year-old boy with a wild
imagination and a big heart. He owns a very expressive dog named Porkchop. Doug is a 6th Grade
average kid who always tries to do the right thing, despite his frequent failures. He is naive at
times, and fears that people will think of him as a loser.
Adapted from https://doug.fandom.com/wiki/Doug_Funnie.

QUESTÃO 01 (EEAR – CFS/2020) – In the sentence “He owns a very expressive dog named
“Porkchop”, the verb OWNS can be replaced by __________ without change in meaning.
a) has
b) buys
c) needs
d) wants

Read the text and answer questions 02 and 03.


An e-mail to Mary
Hi, Mary!
How are you? I miss you so much!
I arrived in Rome last week and I’m having a lot of fun. This morning my parents and I woke up at
8:00 and had a great breakfast at the hotel. Afterwards, I walked a little and saw many tourists
(many from Russia and China) and even some famous people!
Now we are going to Venice.
How are things in Las Vegas? Come meet us!
See you soon!
Paul.

QUESTÃO 02 (EEAR – CFS/2020) – All verbs below, underline in the text, are in the Simple Past,
EXCEPT:
a) Arrived
b) Woke
c) Come
d) Had

AULA 02 – VERBS IN TEXTS 70


TEACHER ANDREA BELO

QUESTÃO 03 (EEAR – CFS/2020) – Where did Paul go on his vacation?


a) Rome, Italy
b) Hong Kong, China
c) Miami, The United States of America
d) Las Vegas, The United States of America

Read the text and answer question 04.


The Lazy Song – Bruno Mars
Today I don’t feel like doing anything
I just wanna lay in my bed
Don’t feel like picking up my phone
So leave a message at the tone
‘Cause today I swear I’m not doing anything
https://www.vagalume.com.br/thelazysong-11/brunomars.html

QUESTÃO 04 (EEAR – CFS/2020) – Based on the song extract, the person in the song
a) Doesn’t want to do anything today.
b) Wants to do lots of things today.
c) Is sick and can’t do anything.
d) Has many things to do.

Read the text and answer questions 05 and 06.


Rude
Can I have your daughter for the rest of my life? Say yes, say yes
‘Cause I need to know
You say I’ll never get your blessing till the day I die
Tough luck my friend but the answer is no!
Why you gotta be so rude?
Don’t you know I’m human too
Why you gotta be so rude
I’m gonna marry her anyway
(Marry that girl) Marry her anyway
(Marry that girl) Yeah no matter what you say
(Marry that girl) And we’ll be a family
https://www.vagalume.com.br/magie-11/rude.html

AULA 02 – VERBS IN TEXTS 71


TEACHER ANDREA BELO

QUESTÃO 05 (EEAR – CFS/2020) – What is the correct plural form of the words, in bold type, in
the text?
a) daughter, lives, days, and family.
b) daughters, life, day, and families.
c) daughters, lives, days, and family.
d) daughters, lives, days, and families.

QUESTÃO 06 (EEAR – CFS/2020) – Based on the text, we can infer that the couple
a) is going to wait for the girl’s father to die to get married in peace.
b) doesn’t want to get married without the blessing pf the girl’s father.
c) doesn’t care about the opinion of the girl’s father and is going to get married.
d) Is going to break up because the girl’s father doesn’t accept the marriage.

Read the text and answer question 07.


Alice in Wonderland
“If you _______ know where you want to go, then it _______ matter which path you take”

QUESTÃO 07 (EEAR – CFS/2020) – Complete the text with the appropriate words.
a) don’t / don’t
b) does / don’t
c) don’t / doesn’t
d) doesn’t / doesn’t

QUESTÃO 08 (EEAR – CFS/2020) – Choose the alternative that best completes the dialogue:
Mary: Hi _______ am Mary Smith. _______ am from the USA. What’s your name?
Paul: _______ name is Paul Thompson.
Mary: Nice to meet you. _______ are you from?
Paul: Nice to meet you, too. _______ am from London. And who is _______ woman?
Mary: _______ is Jessica Lopez. _______ is from the USA too.
a) She – She – Her – When – I – that – He – He
b) I – I – My – Where – I – that – She - She
c) I – I – My – When – I – those – He – He
d) I – I – I – Where – He – that – She – She

AULA 02 – VERBS IN TEXTS 72


TEACHER ANDREA BELO

Read the text and answer question 09.

QUESTÃO 09 (EEAR – CFS/2020) – The term “Wake me up”, in the comic strip, is in the
a) Present Continuous
b) Imperative
c) Future
d) Past

Read the text and answer question 10.


All of me
John Legend
‘Cause all of me
Loves all of you
Love your curves and all your edges
All your perfect imperfections
Give your all to me
I’ll give my all to me
You’re my end and my beginning
Even when I lose, I’m winning
‘Cause I give you all of me
And you give me all of you (…)

QUESTÃO 10 (EEAR – CFS/2020) – A synonym for the word “edges” (line 03) is:
a) hair
b) bones
c) center
d) margin

AULA 02 – VERBS IN TEXTS 73


TEACHER ANDREA BELO

QUESTÕES EFOMM
Based on the text below, answer questions 01 to 04.
Nasa SpaceX crew return: Dragon capsule splashes down
Two American astronauts have splashed down, as the first commercial crewed mission to the
International Space
Station returned to Earth.
The SpaceX Dragon Capsule carrying Doug Hurley and Bob Behnken came down in the Gulf of
Mexico just south of Pensacola on Florida's Gulf coast.
They have now emerged from the capsule on to a recovery vessel.
The touchdown marks the first crewed US water landing since the final Apollo moon mission 45
years ago.
Hurley's and Behnken's capsule touched the water at about 14:48 EDT (19:48 BST; 18:48 GMT).
"It's truly our honor and privilege," said Hurley as they arrived.
"On behalf of the SpaceX and Nasa teams, welcome back to Planet Earth. Thanks for flying
SpaceX," the SpaceX mission control said just after splashdown.
President Donald Trump - who attended the capsule's launch two months ago - hailed its safe
return.
"Thank you to all!" he tweeted. "Great to have NASA Astronauts return to Earth after very
successful two month mission."
The successful end to the crew's mission initiates a new era for the American space agency.
All its human transport needs just above the Earth will in future be purchased from private
companies, such as SpaceX.
The government agency says contracting out to service providers in this way will save it billions of
dollars that can be diverted to getting astronauts to the Moon and Mars.
The Dragon capsule launched to the space station at the end of May on a Falcon 9 rocket, also
supplied by SpaceX.
Hurley's and Behnken's mission served as an end-to-end demonstration of the astronaut "taxi
service" the company, owned by tech entrepreneur Elon Musk, will be selling to Nasa from now
on.
The Boeing corporation is also developing a crew capsule solution but has had to delay its
introduction after encountering software problems on its Starliner capsule.
The sight of the vehicle's four main parachutes floating down over the Gulf of Mexico was
confirmation the spacecraft had survived its fiery descent through the atmosphere.
The parachutes then slowed the capsule from about 350mph (560km/h) to just roughly 15mph at
splashdown.
Rigging was then used to hoist the capsule out of the water and on to the recovery vessel.
Technicians monitored "remnant vapours" around the spacecraft before the hatch was opened.

AULA 02 – VERBS IN TEXTS 74


TEACHER ANDREA BELO

The men were to be checked over by medical staff before being flown to shore by helicopter.
The astronauts' Dragon capsule launched to the space station at the end of May on a Falcon 9
rocket, also supplied by SpaceX.
It will now be refurbished to fly again next year.
Nasa Administrator Jim Bridenstine lauded the efforts of everyone involved in Hurley's and
Behnken's mission, and then spoke of his agency's shift in philosophy.
"We don't want to purchase, own and operate the hardware the way we used to," he said.
"We want to be one customer of many customers in a very robust commercial marketplace in
low-Earth orbit. But we also want to have numerous providers that are competing against each
other on cost and innovation and safety, and really create this virtuous cycle of economic
development and capability."
Adapted from https://www.bbc.com/news/science-environment-53621102

QUESTÃO 01 (EFOMM/INÉDITA) – It is possible to infer from the text that


(A) Nasa intends to make its own capsule to satisfy their human transport needs.
(B) it is more expensive to contract other companies to make transport capsules.
(C) Nasa has always contracted private companies to make human transport capsules.
(D) Nasa has no intention to keep operating hardware the same way they used to do.
(E) The SpaceX Dragon Capsule carrying Doug Hurley and Bob Behnken touched the water in the
Atlantic Ocean.

QUESTÃO 02 (EFOMM/INÉDITA) – Read the statements about the text and decide whether they
are TRUE (T) or FALSE (F). Mark the correct option.
I – A crewed spacecraft landed on the water for the first time in 45 years.
II – The parachutes slowed the capsule from about 560km/h to just roughly 15km/h.
III – The Boeing corporation has its crew capsule ready for Nasa’s next mission.
IV – Nasa administrator talked about his agency’s change in philosophy after the splashdown.
V – Nasa will assess the possibility of contracting a private company again.
(A) F / T / T / F / F
(B) T / F / F / T / F
(C) T / T / T / F / T
(D) F / F / F / T / F
(E) F / T / F / T / T

AULA 02 – VERBS IN TEXTS 75


TEACHER ANDREA BELO

QUESTÃO 03 (EFOMM/INÉDITA) – In the excerpt “Technicians monitored "remnant vapours"


around the spacecraft before the hatch was opened.”, the word in bold means:
(A) Residual
(B) Wasting
(C) Debris
(D) Excess
(E) Near

QUESTÃO 04 (EFOMM/INÉDITA) – According to the text, President Donald Trump


(A) attended the capsule's launch in April.
(B) didn’t like the fact that the mission was a success.
(C) celebrated the end of this successful mission.
(D) went to Pensacola, Florida to welcome the crew back to Earth.
(E) wanted the mission to last more time.

QUESTÃO 05 (EFOMM/INÉDITA) – Choose the correct option to complete the paragraph below.
Saving the giant panda is one of the big success stories of conservation.
Decades of efforts to create protected habitat for the iconic mammal has pulled it back from the
brink of extinction. But, according to a new study, while many other animals ______ the same
landscape have benefited from this conservation work, some have lost out. Leopards, snow
leopards, wolves and Asian wild dogs have almost disappeared from the majority of protected
areas. Driven _____ near extinction ______ logging, poaching and disease, their loss could lead to
"major shifts, even collapse, in ecosystems", said researchers in China.
Adapted from https://www.msn.com/en-us/health/other/if-people-are-staying-home-why-is-coronavirus-still-spreading/ar-BB13HSW8?ocid=bingcovid

(A) in / on / by
(B) on / in / at
(C) in / to / by
(D) at / on / in
(E) in / in / on

QUESTÃO 06 (EFOMM/INÉDITA) – Choose the best alternative to complete the excerpt below.
Finding _____ effective coronavirus vaccine ______ a global priority in ending the pandemic. US
government leaders have put forward the ambitious timeline ______ have one by the end of
2020. It typically takes several years to develop ______ vaccine.
https://www.bbc.com/news/science-environment-53616593

AULA 02 – VERBS IN TEXTS 76


TEACHER ANDREA BELO

(A) an / has become / to / a


(B) a / has become / to / a
(C) an / became / to / the
(D) an / has become / of / the
(E) a / became / of / a

QUESTÃO 07 (EFOMM/INÉDITA) – Choose the correct alternative to complete the paragraph


below.
A holistic view of ecosystem management __________ "critically needed to better increase the
resilience and sustainability of the ecosystems not only for giant pandas but also for other wild
species", said Dr Sheng Li of Peking University in Beijing. Giant pandas are __________ as living
proof that conservation works. Their numbers in the wild are finally rebounding after years of
decline, and in 2016 they __________ upgraded from "endangered" to "vulnerable" on the official
extinction Red List.
Adapted from https://www.bbc.com/news/science-environment-53616593

(A) is / saw / have been


(B) are / see / are
(C) was / seen / were
(D) is / seen / were
(E) was / saw / have been

QUESTÃO 08 (EFOMM/INÉDITA) – Which option is correct?


(A) She gave me two precious advices.
(B) Poaching is an essential activity in natural reserves.
(C) Brazil have hosted the olimpics in 2016.
(D) I am used to running 5 kilometers a day.
(E) There was a search to determine Brazil’s population in 2010.

QUESTÃO 09 (EFOMM/INÉDITA) – Mark the option which corresponds to the correct sentences.
I – The person who defends a person in court is called witness.
II – The pregnant woman was in labor because her water broke.
III – Mooring is an important process when the ship is sailing.
IV – That child was birthed in July.
V – The pregnant woman was taken to the delivery room in order to give birth.

AULA 02 – VERBS IN TEXTS 77


TEACHER ANDREA BELO

(A) I and II
(B) II and V
(C) III and V
(D) II and IV
(E) III and IV

QUESTÃO 10 (EFOMM/INÉDITA) – Which option is correct to complete the sentences below?


1 – He heard her __________ the dinner.
2 – He saw her __________ the cake.
3 – I watched him __________ the guitar.
4 – In order to get thin, you need to __________ hard.
5 – He couldn’t see what __________ on in his neighborhood yesterday.
(A) make / bake / played / working out / were going
(B) made / baking / play / work out / has been going
(C) making / bake / playing / working out / had been going
(D) making / baking / playing / work out / was going
(E) make / baked / play / worked out / is going

QUESTÕES EPCAR
Directions: Answer questions 01 to 10 according to TEXT I.
The WHO has redefined burnout as a syndrome linked to chronic work stress. There’s a
difference between a busy workload and something more serious, writes Zaria Gorvett.
If you said you were suffering from ‘burnout’ in the early 1970s, you might have raised some
eyebrows.
At the time, the term was used informally to describe the side effects that heavy drug users
experienced: the general dimming of the mental faculties, for example, as was the case with many
a party animal. However, when German-American psychologist Herbert Freudenberger first
recognised the problem of burnout in New York City in 1974, at a clinic for addicts and homeless
people, Freudenberger wasn’t thinking of drug users.
The clinic’s volunteers were actually struggling, too: their work was intense, and many were
beginning to feel demotivated and emotionally drained. Though they had once found their jobs
rewarding, they had become cynical and depressed; they weren’t giving their patients the
attention they deserved. Freudenberger defined this alarming new condition as a state of
exhaustion caused by prolonged overwork – and borrowed the term ‘burnout’ to describe it.

AULA 02 – VERBS IN TEXTS 78


TEACHER ANDREA BELO

Its popularity was explosive, and today burnout is a global phenomenon. Although statistics on
the prevalence of burnout specifically are hard to come by, 595,000 people in the UK alone
suffered from workplace stress in 2018.
Sportspeople get it. YouTube stars get it. Entrepreneurs get it. Freudenberger himself eventually
got it. Late last month, the World Health Organization (WHO) announced that the trendy problem
will be recognised in the latest International Classification of Diseases manual, where it is
described as a syndrome “resulting from chronic workplace stress that has not been successfully
managed”.
According to the WHO, burnout has three elements: feelings of exhaustion, mental detachment
from one’s job and poorer performance at work. But waiting until you’re already fully burned out
to do something about it doesn’t help at all –and you wouldn’t wait to treat any other illness until
it was too late.
Feeling the burn
So how can you tell if you’re almost – but not quite – burned out?
“A lot of the signs and symptoms of pre-burnout would be very similar to depression,” says
Siobhán Murray, a psychotherapist based in County Dublin, Ireland, and the author of a book
about burnout, The Burnout Solution. Murray suggests looking out for creeping bad habits, such
as increased alcohol consumpution and relying on sugar to get you through the day. Also watch
out for feelings of tiredness that won’t go away. “So that even if you do sleep well, by 10 in the
morning you’re already counting down the hours to bed. Or not having the energy to exercise or
go for a walk.”
As soon as you begin to feel this way, Murray advises going to see your doctor.
“Depression and pre-burnout are very similar, but as much as there was a lot of enthusiasm
recently that burnout has now become a medical condition, it is still not – it is still classified as an
occupational phenomenon.” It’s important to get help from a medical professional who can
distinguish between the two, because although there are many treatment options for depression,
burnout is still best tackled by making lifestyle changes.
And how do you know if you’re really on the cusp of burnout, or just going through a challenging
month? “Stress is really important, and anxiety is what motivates us to do well,” says Murray. “It’s
when we’re continually exposed to stress and anxiety, that we’re not letting go, that it starts to
turn into burnout.”
Take that big project you’ve been working on. It’s normal to feel a kick of adrenaline when you
think about it, and maybe it’s kept you up at night. But, Murray suggests, if you still feel restless
once it’s over, it’s time to consider if you’re at risk of burnout. “It’s when you’re bringing that with
you into the next stage of your day, and adding to it continually,” she says.
Another classic sign of inching closer to burnout is cynicism: feeling like your work has little value,
avoiding social commitments and becoming more susceptible to disappointment.
“Someone on the brink will probably begin to feel emotionally numbed or mentally distant,” says
Jacky Francis Walker, a psychotherapist based in London who specializes in burnout. “Like they
don’t have the capacity to engage as much in the ordinary things of life.”

AULA 02 – VERBS IN TEXTS 79


TEACHER ANDREA BELO

She also recommends looking for the final tell-tale sign of burnout, which is the unshakeable
feeling that the quality of your work is beginning to slip. “People say ‘but this isn’t me!’, ‘I’m not
like this’, ‘I can usually do x,y and z’. But obviously if they are in a state of physical depletion, then
they aren’t in their normal range of capabilities,” says Walker.
If this seems less than scientific, look to the Maslach Burnout Inventory (MBI), a test designed to
measure burnout. The most widely used is the MBI-General Survey, which measures things like
exhaustion, cynicism, and some how well you think you’re doing at work.
First published in 1981, it has been cited hundreds of times in studies since. Although it’s typically
used to measure burnout once it's in full swing, there’s no reason you can't apply it to see if you’re
getting close.
(Adapted from https://www.bbc.com/worklife/article/20190610-how-to-tell-if-youve-got-pre-burnout)

QUESTÃO 01 (EPCAR/INÉDITA) – Mark the correct option.


a) Burnout is now considered a medical condition.
b) Burnout can improve people’s performance at work.
c) Burnout was widely known in the 1970’s.
d) Herbert Freudenberger first used the term burnout in the 1970’s.

QUESTÃO 02 (EPCAR/INÉDITA) – According to the text, mark the correct alternative.


a) The psychologist discovered burnout by accident while analyzing drug addicts and homeless.
b) The clinic’s volunteers were having a good time doing their work.
c) Anyone is subject to get it if they reach exhaustion due to prolonged overwork and stress.
d) Drug addicts were the first to suffer from burnout due to their stressful routine.

QUESTÃO 03 (EPCAR/INÉDITA) – In the sentence “the general dimming of the mental faculties”
(paragraph 2) the word dimming means
a) decrease.
b) augmentation.
c) assessment.
d) improvement.

QUESTÃO 04 (EPCAR/INÉDITA) – Mark the correct question to the sentence below.


“Freudenberger defined this alarming new condition as a state of exhaustion caused by
prolonged overwork”
a) How did Freudenberger define this alarming new condition?
b) How has Freudenberger defined this alarming new condition?
c) How had Freudenberger defined this alarming new condition?
d) How did Freudenberger defined this alarming new condition?

AULA 02 – VERBS IN TEXTS 80


TEACHER ANDREA BELO

QUESTÃO 05 (EPCAR/INÉDITA) – Mark the alternative that completes the sentence.


Some symptoms the psychologist noticed the volunteers had to deal with were
a) that they were drug abusers and most of them were homeless.
b) that they started to feel demotivated, emotionally drained, cynical and depressed.
c) that they were getting sick very frequently.
d) that they were being diagnosed with generalized anxiety and depression.

QUESTÃO 06 (EPCAR/INÉDITA) – Mark the alternative that DOESN’T complete the sentence
below.
The psychologist Herbert Freudenberger
a) was who first used the word burnout to describe this exhaustion condition.
b) recognised the problem of burnout in 1974 in New York City for the first time.
c) created this condition so that he would become known for discovering a disease.
d) defined burnout as a state of exhaustion caused by prolonged overwork.

QUESTÃO 07 (EPCAR/INÉDITA) – Read the sentences and mark the correct option.
I. The WHO has finally recognised burnout as a disease.
II. More than half a million people suffered from workplace stress in 2018 in the UK.
III. Relying on sugar to get you through the day can be a good way of keeping yourself from
burning out.
The only correct sentence(s) is(are)
a) II.
b) III.
c) I and II.
d) I and III.

QUESTÃO 08 (EPCAR/INÉDITA) – Mark the correct alternative to complete the sentence.


According to the text, the burnout
a) makes people happy because they feel that they are performing at their best.
b) is best tackled and can be reversed by making lifestyle changes.
c) has been responsible for many deaths around the world since the 1970’s.
d) is responsible for kicks of adrenaline when you think about a project you’ve been working on.

AULA 02 – VERBS IN TEXTS 81


TEACHER ANDREA BELO

QUESTÃO 09 (EPCAR/INÉDITA) – In the underlined sentences in the second paragraph, there


are __________ verbs used in the past tense.
a) four
b) one
c) three
d) two

QUESTÃO 10 (EPCAR/INÉDITA) – Volunteers


a) never liked their work at all.
b) weren’t giving enough attention to patients due to their burnout condition.
c) accepted being part of an experiment to find out what happens when people get to their limit
at work.
d) would still find their jobs rewarding even though they were burned out.

QUESTÕES ESA
Texto para responder à questão 01
August 08, 2020
By Lise Alves, Senior Contributing Reporter
RIO DE JANEIRO, BRAZIL - In two months, researchers from the Alberto Luiz Coimbra Institute of
Post-Graduate Studies and Engineering Research at the Federal University of Rio de Janeiro
(Coppe/UFRJ), the National Institute of Metrology, Quality and Technology (INMETRO) and the
Pontifical Catholic University of Rio de Janeiro (PUC Rio) will introduce an antiviral mask fabric
that provides greater protection against Covid-19 to healthcare professionals. The announcement
was made yesterday, August 7th, by Professor Renata Simão from the Nanotechnology
Engineering and Metallurgical and Materials Engineering Programs of Coppe.
(Adapted from https://riotimesonline.com)

QUESTÃO 01 (ESA/INÉDITA) – According to the text, it is correct to say that:


A) many researchers will introduce a new protective mask.
B) INMETRO is one of the responsible institution for the protective mask.
C) Coope/UFRJ is the responsible institution for the protective mask.
D) INMETRO has researched about the protective mask.
E) some researchers will introduce a new protective mask.

AULA 02 – VERBS IN TEXTS 82


TEACHER ANDREA BELO

QUESTÃO 02 (ESA/INÉDITA) – Which sentence is grammatically correct?


(A) Yesterday, I went to bed more earlier than the night before.
(B) Fred is more thin than his best friend.
(C) My book is more big than my sister’s dictionary.
(D) Going by car is expensiver than by bus because of the gas.
(E) Elizabeth’s shoes are cheaper than mine.

QUESTÃO 03 (ESA/INÉDITA) – Complete the sentence below using the appropriate words:
“Mr. Tompson __________ traveling by car: He is afraid of planes and __________ like fast trips,
but ___________ driving.”
(A) Likes/don’t/love
(B) Likes/doesn’t/loves
(C) Like/do/loves
(D) Like/don’t/love
(E) Like/does/love

QUESTÃO 04 (ESA/INÉDITA) – “__________ my best friend for real?”


Complete the space with the correct form of the verb and the pronoun.
(A) You is
(B) You are
(C) Are you
(D) Is you
(E) Am you

Texto para responder às questões 05 e 06


August 08, 2020
RIO DE JANEIRO, BRAZIL - On Saturday, August 8th, Brazil reached the number of 100,477 deaths
from Covid-19 since the start of the novel coronavirus pandemic. According to the Ministry of
Health, 905 deaths were recorded over the last 24 hours.
Of the 3,012,412 cases of people infected by the novel coronavirus, 2,094,293 (69.5 percent) have
recovered.
A total of 49,970 new cases have been reported by Health Secretariats since Friday, August 7th.
The official also shows that the number of people under follow-up stands at 817,642 (21.1
percent).
(Adapted from https://riotimesonline.com)

AULA 02 – VERBS IN TEXTS 83


TEACHER ANDREA BELO

QUESTÃO 05 (ESA/INÉDITA) – According to the text, it is correct to say that:


A) the number described is more than a hundred thousand deaths.
B) it is said the number is a hundred thousand deaths.
C) the number described is less than a hundred thousand deaths.
D) more than a hundred thousand people are working on the research.
E) the correct number of deaths is a thousand.

QUESTÃO 06 (ESA/INÉDITA) – The term “quite a few” is the same of


(A) many
(B) much
(C) few
(D) little
(E) several

QUESTÃO 07 (ESA/INÉDITA) – The word “huge” is the same of


(A) very hard
(B) very big
(C) very far
(D) very deep
(E) very cheap

QUESTÃO 08 (ESA/INÉDITA) – “__________ working at the company?”


Complete the space with the correct form of the verb and the pronoun.
(A) Are the girl
(B) She is
(C) Is she
(D) Are she
(E) Am she

QUESTÃO 09 (ESA/INÉDITA) – The word “them” is


(A) a pronoun
(B) a conjunction
(C) a preposition
(D) a noun
(E) an adjective

AULA 02 – VERBS IN TEXTS 84


TEACHER ANDREA BELO

QUESTÃO 10 (ESA/INÉDITA) – The word “useful” is


(A) a verb
(B) a conjunction
(C) a preposition
(D) an adverb
(E) an adjective

QUESTÕES ESCOLA NAVAL


QUESTÃO 01 (ESCOLA NAVAL/INÉDITA) – Which is the correct option to complete the
paragraph below.
What Is Stress?
First, an answer to __________ question, 'What is stress?' __________ term 'stress' refers
__________ the response you have when facing circumstances that force you __________ act,
change, or adjust in some way to maintain your footing or to keep things balanced. (The
circumstances themselves are known as 'stressors', but we'll have more on them later.)
(Adapted from https://www.verywellmind.com/chronic-stress-definition-management-tips-3145241)

(A) the / the / to / to


(B) a / a / at / -
(C) the / a / to / to
(D) the / the / to / -
(E) a / the / at / -

QUESTÃO 02 (ESCOLA NAVAL/INÉDITA) – Which option completes the dialogue below


correctly?
John: What's the matter?
Mary: I haven’t done my exercise yet. I must _______ today.
(A) have done it
(B) have it done
(C) done it
(D) has done it
(E) has it done

AULA 02 – VERBS IN TEXTS 85


TEACHER ANDREA BELO

QUESTÃO 03 (ESCOLA NAVAL/INÉDITA) – Which option completes the paragraph below


correctly?
While quick stress relievers aren't a complete stress management plan in themselves, they are an
excellent first line of defense against the effects of chronic stress. This is because they can help
you turn ______ your body's stress response and respond ______ the stressors you face from a
calm (or calmer), more relaxed place. This helps you to more effectively deal ______ what is
stress, and keep yourself healthier ______ the same time. Common strategies for quick stress
relief include breathing exercises or the use of humor.
(Adapted from https://www.verywellmind.com/chronic-stress-definition-management-tips-3145241)

(A) of / to / with / at
(B) on/ - / to / in
(C) off / to / with / at
(D) off/ to / to / in
(E) on / - / with / on

QUESTÃO 04 (ESCOLA NAVAL/INÉDITA) – Which word best completes the question below?
How ______ does he go when he runs?
It’s really important to exercise in order to keep healthy.
(A) often
(B) old
(C) high
(D) far
(E) many

QUESTÃO 05 (ESCOLA NAVAL/INÉDITA) – Which option completes the text below correctly?
Nowadays we live in a bipolar society. We _______ be able to unite instead of separating
ourselves. Anyone _______ do this. One just have to have a good dose of determination to do so.
(A) can't / can
(B) should / shouldn’t
(C) shouldn't / can't
(D) should / can
(E) shouldn't / should

AULA 02 – VERBS IN TEXTS 86


TEACHER ANDREA BELO

QUESTÕES EsPCEx
Leia o texto a seguir e responda às questões 01, 02 e 03.
Malaria 'completely stopped' by microbe
The team in Kenya and the UK say the finding has "enormous potential" to control the disease.
Malaria is spread by the bite of infected mosquitoes, so protecting them could in turn protect
people. The researchers are now investigating whether they can release infected mosquitoes into
the wild, or use spores to suppress the disease.
The malaria-blocking bug, Microsporidia MB, was discovered by studying mosquitoes on the
shores of Lake Victoria in Kenya. It lives in the gut and genitals of the insects. The researchers
could not find a single mosquito carrying the Microsporidia that was harbouring the malaria
parasite. And lab experiments confirmed the microbe gave the mosquitoes protection.
Microsporidias are fungi, or at least closely related to them, and most are parasites. However, this
new species may be beneficial to the mosquito and was naturally found in around 5% of the
insects studied.
"The data we have so far suggest it is 100% blockage, it's a very severe blockage of malaria," Dr
Jeremy Herren, from the International Centre of Insect Physiology and Ecology in Kenya told the
BBC. He added: "It will come as a quite a surprise. I think people will find that a real big
breakthrough. While huge progress has been made through the use of bed nets and spraying
homes with insecticide, it is widely agreed new tools are needed to tackle malaria.
How does the microbe stop malaria? Microsporidia MB could be priming the mosquito's immune
system, so it is more able to fight off infections. Or the presence of the microbe in the insect could
be having a profound effect on the mosquito's metabolism, making it inhospitable for the malaria
parasite.
Adapted from https://www.bbc.com/news/health-52530828

QUESTÃO 01 (EsPCEx/INÉDITA) – In the sentence “... it is widely agreed new tools are needed to
tackle malaria.” (paragraph 3), the word tackle means
(A) neglect
(B) combat
(C) increase
(D) debate
(E) start

QUESTÃO 02 (EsPCEx/INÉDITA) – Choose the alternative with the correct reference for the
underlined words from the text.
(A) them (paragraph 2) = fungi
(B) them (paragraph 1) = people
(C) they (paragraph 1) = mosquitoes
(D) It (paragraph 2) = malaria
(E) it (paragraph 4) = effect

AULA 02 – VERBS IN TEXTS 87


TEACHER ANDREA BELO

QUESTÃO 03 (EsPCEx/INÉDITA) – According to the text, read the statements and choose the
correct alternative.
I – Malaria is spread by the bite of a bug called Microsporidia MB.
II – Microsporidias are fungi that can be beneficial to the mosquitoes that are malaria vectors.
III – Some mosquitoes carrying the Microsporidia still had the malaria parasite in them.
IV – The use of bed nets and insecticide have been proven to be enough to tackle malaria.
(A) Only I and II are correct.
(B) Only II and IV are correct.
(C) All of them are correct.
(D) Only II, III and IV are correct.
(E) Only II is correct.

Leia o texto a seguir e responda às questões 04, 05 e 06.


(Título omitido propositadamente)
Hoping to stem the torrent of false cures and conspiracy theories about COVID-19, Facebook
announced Thursday it would begin informing users globally who have liked, commented on, or
shared “harmful” misinformation about the coronavirus, pointing them instead in the direction
of a reliable source. Facebook hopes the move will drastically reduce the spread of false
information about the coronavirus online, a growing crisis that the World Health Organization
(WHO) has described as an “infodemic.”
“We want to connect people who may have interacted with harmful misinformation about the
virus with the truth from authoritative sources in case they see or hear these claims again,” said
Guy Rosen, Facebook’s Vice President for Integrity, in a blog post published early Thursday.
The new policy only applies to false claims related to the coronavirus, but campaigners say the
announcement could lay the groundwork for a breakthrough in the battle against political
disinformation online. “Facebook applying this to the pandemic is a good first step but this should
also be applied to political disinformation too, particularly with the 2020 U.S. election
approaching” says Fadi Quran, a campaign director at Avaaz, a global advocacy group that has
lobbied Facebook to “correct the record” on false information since 2018. (In the language of
online security, “disinformation” means the coordinated, purposeful spread of false information,
while “misinformation” refers to accidental inaccuracies.) “I hope this is going to be expanded to
other issues and fast,” Quran tells TIME.
Adapted from https://time.com/5822372/

QUESTÃO 04 (EsPCEx/INÉDITA) – Choose the most appropriate title for the text.
(A) Facebook bans users who have shared coronavirus information.
(B) Facebook notifies users who have interacted with coronavirus misinformation.
(C) Facebook prohibits political disinformation online before U.S. election.
(D) Facebook applies new policies to contain fake news about coronavirus and politics.
(E) Facebook users to face fines if they spread disinformation about COVID-19.

AULA 02 – VERBS IN TEXTS 88


TEACHER ANDREA BELO

QUESTÃO 05 (EsPCEx/INÉDITA) – In the sentence “...but campaigners say the announcement


could lay the groundwork for a breakthrough in the battle against political disinformation
online....” (paragraph 3), the word groundwork means
(A) workout.
(B) breakup.
(C) background.
(D) foundation.
(E) harassment.

QUESTÃO 06 (EsPCEx/INÉDITA) – According to the text, choose the correct statement.


(A) Facebook announced false cures and conspiracy theories about COVID-19.
(B) Facebook developed a new policy to contain fake news during the U.S. election.
(C) Facebook is taking measures aiming to reduce the spread of coronavirus misinformation.
(D) Facebook is notifying users in the U.S. who have interacted with misinformation about the
coronavirus.
(E) Facebook has confirmed that its new policy will be used to contain political disinformation as
well.

Leia o texto a seguir e responda às questões 07, 08 e 09.


The true cost of Saudi Arabia’s $500bn megacity
With a huge artificial moon, glow-in-the-dark beaches, flying drone-powered taxis, robotic butlers
__________ (1) clean the homes of residents and a Jurassic Park-style attraction featuring
animatronic lizards, Neom has been billed __________ (2) humanity’s next chapter. But beneath
the glitzy veneer lies a story of threats, forced eviction and bloodshed.
“The future has a new home,” proclaims the website, as the promotional video tracks colour-
tinted panoramic shots of picturesque desert expanses, and deep azure lagoons. The new city
state of Neom, named __________ (3) a combination of the Greek word for “new” and the Arabic
term for “future”, is intended to cover an area the size of Belgium at the far north of Saudi Arabia’s
Red Sea coastline.
There has been no shortage of outlandish promises for the $500bn (£400bn) city-state. Yet part
of the site is the home of the Huwaitat tribe, who have spanned Saudi Arabia, Jordan, and the
Sinai Peninsula for generations, tracing their lineage back before the founding of the Saudi state.
At least 20,000 members of the tribe now face eviction due to the project, with no information
about where they will live in the future.
“For the Huwaitat tribe, Neom is being built on our blood, on our bones,” says Alia Hayel
Aboutiyah al-Huwaiti, an outspoken activist and member of the tribe. Construction is ploughing

AULA 02 – VERBS IN TEXTS 89


TEACHER ANDREA BELO

ahead despite the challenges of the pandemic and the historic drop in the price of oil, Saudi’s
primary export. For many, though, Neom is less a shining vision of the future than a grim symbol
of Saudi human rights violations, underscored by the treatment of the Huwaitat tribe.
Adapted from https://www.theguardian.com/global-development/2020/may/04/its-being-built-on-our-blood-the-true-cost-of-saudi-arabia-5bn-mega-city-neom

QUESTÃO 07 (EsPCEx/INÉDITA) – Choose the alternative containing the correct words to


respectively complete gaps (1), (2) and (3).
(A) at, of, to
(B) to, on, at
(C) to, as, from
(D) in, with, an
(E) with, of, as

QUESTÃO 08 (EsPCEx/INÉDITA) – Which excerpt from the text refers to the title “The true cost
of Saudi Arabia’s $500bn megacity”?
(A) “‘The future has a new home,’ proclaims the website...” (paragraph 2)
(B) “There has been no shortage of outlandish promises for the $500bn (£400bn) city-state.”
(paragraph 3)
(C) “With a huge artificial moon, glow-in-the-dark beaches, flying drone-powered taxis, robotic
butlers...” (paragraph 1)
(D) “The new city state of Neom... is intended to cover an area the size of Belgium at the far
north of Saudi Arabia’s Red Sea coastline.” (paragraph 2)
(E) “But beneath the glitzy veneer lies a story of threats, forced eviction and bloodshed.”
(paragraph 1)

QUESTÃO 09 (EsPCEx/INÉDITA) – In the sentence “But beneath the glitzy veneer lies a story of
threats, forced eviction and bloodshed.” (paragraph 1), the expression beneath the glitzy veneer
means
(A) under the glamorous appearance.
(B) over the oblivious look.
(C) despite the shiny appearance.
(D) under the fake aspect.
(E) not so important as its cost.

AULA 02 – VERBS IN TEXTS 90


TEACHER ANDREA BELO

Leia o texto a seguir e responda à questão 10.

How surviving a recession prepared me for Covid-19

I was 35 when I lost my job as a legal courier in San Jose, California. It was 2009, and the economy
had lost nearly six million jobs since the Great Recession began in 2007.

Finding work was nearly impossible. At one point, I juggled three jobs that paid a total of less than
$500 per week. It was barely enough to get by. For the next three to four years, I was mostly
unemployed, homeless — sleeping on benches, shelters and sometimes friends’ couches — and
living off the McDonald’s Dollar Menu.

Those were some dark times. But I’ve come a long way since then. In 2017, I moved to Tacoma,
Washington with my best friend Kim, who lent me just enough money to settle down and pursue
my dream of getting into real estate. You’re getting another shot, I told myself. Don’t blow it.

I didn’t. Eight months after I got my broker license, I closed my seventh deal — for a total of $1.6
million in sales. I paid back the money Kim loaned me. For the first time in my life, I had a feeling
of accomplishment and satisfaction. I got serious about saving money and started planning for
the future.

While the underlying cause of today’s economic slowdown is fundamentally different from the
Great Recession, the lessons I learned over the past decade still apply. And they’ve made me more
prepared to handle the hardships that have — and will — come from the coronavirus pandemic.
Adapted from https://www.cnbc.com/2020/04/23/i-was-unemployed-homeless-how-surviving-great-recession-prepared-me-for-coronavirus.html

QUESTÃO 10 (EsPCEx/INÉDITA) – What kind of text is this?

(A) A contract.

(B) An autobiographical text.

(C) A book review.

(D) A fantasy story.

(E) A science fiction narrative.

AULA 02 – VERBS IN TEXTS 91


TEACHER ANDREA BELO

GABARITO
GABARITO AFA
01 – D 02 – B 03 – A 04 – B 05 – C
06 – C 07 – D 08 – A 09 – D 10 – B

GABARITO COLÉGIO NAVAL


01 – B 02 – D 03 – D 04 – B 05 – D
06 – C 07 – C 08 – A 09 – A 10 – D

GABARITO EAM
01 – E 02 – A 03 – B 04 – D 05 – B

GABARITO EEAR
01 – A 02 – C 03 – A 04 – A 05 – D
06 – C 07 – C 08 – B 09 – B 10 – D

GABARITO EFOMM
01 – D 02 – B 03 – A 04 – C 05 – C
06 – A 07 – C 08 – D 09 – B 10 – D

GABARITO EPCAR
01 – D 02 – C 03 – A 04 – A 05 – B
06 – C 07 – A 08 – B 09 – D 10 – B

GABARITO ESA
01 – E 02 – E 03 – B 04 – C 05 – A
06 – A 07 – B 08 – C 09 – A 10 – E

GABARITO ESCOLA NAVAL


01 – A 02 – B 03 – C 04 – D 05 – D

GABARITO EsPCEx
01 – B 02 – A 03 – E 04 – B 05 – D
06 – C 07 – C 08 – E 09 – A 10 – B

AULA 02 – VERBS IN TEXTS 92


TEACHER ANDREA BELO

QUESTÕES COMENTADAS
QUESTÕES AFA
Directions: Read the text below and answer questions 01 to 10 according to it.
The machine always wins: what drives our addiction to social media
We are swimming in writing. Our lives have become, in the words of the author and academic
Shoshana Zuboff, an “electronic text”. Social media platforms have created a machine for us to
write to. The bait is that we are interacting with other people: our friends, colleagues, celebrities,
politicians, royals, terrorists, porn actors – anyone we like. We are not interacting with them,
however, but with the machine. We write to it, and it passes on the message for us after keeping
a record of the data.
The machine benefits from the “network effect”: the more people write to it, the more benefits
it can offer, until it becomes a disadvantage not to be part of it. Part of what? The world’s first ever
public, live, collective, open-ended writing project. A virtual laboratory. An addiction machine,
which deploys crude techniques of manipulation redolent of the Skinner Box created by
behaviourist BF Skinner to control the behaviour of pigeons and rats with rewards and
punishments. We are users, much as cocaine addicts are users.
What is the incentive to engage in writing like this for hours each day? In a form of mass
casualisation, writers no longer expect to be paid or given employment contracts. What do the
platforms offer us, in lieu of a wage? What gets us hooked? Approval, attention, retweets, shares
and likes.
Meanwhile, hashtags and trending topics underline the extent to which all of these protocols are
organised around the massification of individual voices – a phenomenon cheerfully described by
users with the science-fiction concept of the “hive mind” – and hype. The regular sweet spot
sought after is a brief period of ecstatic collective frenzy around any given topic. It doesn’t
particularly matter to the platforms what the frenzy is about: the point is to generate data, one of
the most profitable raw materials yet discovered. As in the financial markets, volatility adds value.
The more chaos, the better.
Whether or not we think we are addicted, the machine treats us as addicts. Addiction is, quite
deliberately, the template for our relationship to the Twittering Machine. Addiction is all about
attention. For the social media bosses, this is axiomatic.
If social media is an addiction machine, the addictive behaviour it is closest to is gambling: a rigged
lottery. Every gambler trusts in a few abstract symbols – the dots on a dice, numerals, suits, red or
black, the graphemes on a fruit machine – to tell them who they are. In most cases, the answer is
brutal and swift: you are a loser and you are going home with nothing. The true gambler takes a
perverse joy in anteing up, putting their whole being at stake. On social media, you scratch out a
few words, a few symbols, and press send, rolling the dice. The internet will tell you who you are
and what your destiny is through arithmetic likes, shares and comments.

AULA 02 – VERBS IN TEXTS 93


TEACHER ANDREA BELO

The interesting question is what it is that is so addictive. In principle, anyone can win big; in
practice, not everyone is playing with the same odds. Our social media accounts are set up like
enterprises competing for attention. If we are all authors now, we write not for money, but for the
satisfaction of being read. Going viral, or trending, is the equivalent of a windfall. But sometimes,
winning is the worst thing that can happen. The temperate climate of likes and approval is apt to
break, lightning-quick, into sudden storms of fury and disapproval.
(Adapted from https://www.theguardian.com/technology/2019/aug/23/social-media-addiction-gambling)

QUESTÃO 01 (AFA/INÉDITA) – One of the purposes of the text is to show that


a) financial feedback is the major reason why people engage in social media writing.
b) there’s evidence cocaine is more addictive than social media and gambling.
c) anyone can be successful when it comes to social media.
d) social media can be as addictive as drugs.
Comentários: de acordo com a letra A, um dos principais objetivos do texto é mostrar que o
retorno financeiro é a principal razão pela qual as pessoas se comprometem a escrever em redes
sociais. Essa informação não está de acordo com o texto. → “In a form of mass casualisation,
writers no longer expect to be paid or given employment contracts …” Em uma forma de
casualização em massa, os escritores não esperam mais ser pagos ou receber contratos de
trabalho… (3º parágrafo). Alternativa INCORRETA.
Conforme a alternativa B, um dos principais objetivos do texto é mostrar que há evidências de
que a cocaína é mais viciante do que as mídias sociais e os jogos de azar. Se voltarmos ao texto,
veremos, que, na verdade, ele equipara o vício em redes sociais aos demais vícios, não afirmando
que a cocaína, por exemplo, é mais viciante. → “We are users, much as cocaine addicts are users”.
Somos usuários, assim como os viciados em cocaína são usuários (2º parágrafo). Alternativa
INCORRETA.
Segundo a letra C, um dos principais objetivos do texto é mostrar que qualquer pessoa pode ser
bem-sucedida quando se trata de redes sociais. A afirmação não está de acordo com o texto, como
se depreende do seguinte trecho → “In principle, anyone can win big; in practice, not everyone is
playing with the same odds.” Em princípio, qualquer um pode ter muito sucesso; na prática, nem
todo mundo está jogando com as mesmas chances. (7º parágrafo). Alternativa INCORRETA.
De acordo com a letra D, um dos principais objetivos do texto é mostrar que as mídias sociais
podem ser tão viciantes quanto as drogas, o que está totalmente de acordo com o texto. Observe
a expressão que o texto usa para se referir às redes sociais: an addiction machine (uma máquina
viciante). Como vimos, o texto afirma que os usuários de redes sociais são tão usuários quanto
viciados em cocaína e têm um padrão de comportamento semelhante ao dos que se viciam em
jogos de azar. Como a afirmação está em harmonia com o texto, é esse o gabarito. Alternativa
CORRETA.
GABARITO: D

AULA 02 – VERBS IN TEXTS 94


TEACHER ANDREA BELO

QUESTÃO 02 (AFA/INÉDITA) – According to the first paragraph, it is correct to state that


a) social media platforms allow people to truly interact with other people.
b) when people communicate through social media, they, as a matter of fact, interact with the
machine.
c) lives nowadays are more based on personal contact.
d) people should avoid interacting on social media platforms.
Comentários: a questão quer que você identifique o que é correto afirmar de acordo com o
primeiro parágrafo, então, foco total nele! Vamos às alternativas:
De acordo com a letra A, as redes sociais permitem (allow) que as pessoas se conectem
verdadeiramente com as outras. Na verdade, segundo o primeiro parágrafo, as pessoas não estão,
na realidade, interagindo umas com as outras, mas com a máquina. → “We are not interacting
with them, however, but with the machine.” Alternativa INCORRETA.
Segundo a letra B, quando as pessoas se comunicam através das redes sociais, elas, na verdade
(as a matter of fact), interagem com a máquina, o que está totalmente de acordo com o trecho
do primeiro parágrafo que vimos acima, na correção da letra A. Alternativa CORRETA.
Conforme a alternativa C, a vida das pessoas hoje em dia (nowadays) se baseia mais num contato
pessoal. Observe que o primeiro parágrafo traz uma afirmação distinta, ao asseverar que a vida
das pessoas virou uma “mensagem de texto”:→ “Our lives have become... an ‘electronic text’.”
Alternativa INCORRETA.
Muito cuidado com alternativas como essa, que extrapolam o que o texto diz. Não há diretamente,
no primeiro parágrafo, uma opinião no sentido de que as pessoas deveriam evitar interagir nas
plataformas de redes sociais. O parágrafo traz fatos sobre essa interação, como, por exemplo, o
fato de que não é uma interação real. Alternativa INCORRETA.
GABARITO: B

QUESTÃO 03 (AFA/INÉDITA) – The best definition for “bait”, underlined in the first paragraph is
a) something attractive that is offered to make people act a certain way.
b) something that makes people think about their actions.
c) something unattractive that discourages people from acting a certain way.
d) something that makes people improve their attitude.
Comentários: você, certamente, já ouviu a expressão “clickbait”, que se refere a uma tática
sensacionalista usada na Internet para “fisgar” usuários para um conteúdo online, atraindo-os
para um vídeo, por exemplo, e gerar visualizações, “clicks”. Se você se lembrasse disso ao resolver
essa questão, poderia chegar ao sentido de “bait” no texto, que transmite a ideia de uma isca,
algo que atrai as pessoas e as faz agir de uma determinada maneira. Vamos às alternativas:
De acordo com a letra A, a melhor definição de “bait” é algo atraente que é oferecido para fazer
as pessoas agirem de uma certa maneira. Se voltamos ao texto, vemos que o que é oferecido às

AULA 02 – VERBS IN TEXTS 95


TEACHER ANDREA BELO

pessoas para atraí-las, “fisgá-las” para as redes sociais, é a ideia de que através das redes elas
podem interagir com qualquer pessoa que quiserem, desde amigos até celebridades e políticos.
Assim, a definição de “bait” está correta. Alternativa CORRETA.
Conforme a alternativa B, “bait” pode ser definido como algo que faz as pessoas pensarem sobre
as suas ações. Como vimos, não é essa a ideia transmitida por “bait” no contexto. Alternativa
INCORRETA.
Segundo a letra C, “bait” pode ser definido como algo não atraente que desestimula as pessoas a
agirem de uma certa maneira. Como vimos, a ideia transmitida por “bait”, no contexto, é
justamente o contrário disso. Alternativa INCORRETA.
Para a letra D, “bait” pode ser definido como algo que faz as pessoas melhorarem o seu
comportamento (attitude). Como vimos, a ideia transmitida por “bait”, no contexto, não é essa,
mas sim de algo que atrai as pessoas, “fisgando”-as. Alternativa INCORRETA
GABARITO: A

QUESTÃO 04 (AFA/INÉDITA) – In the sentence “We are not interacting with them, however, but
with the machine” (paragraph 1), the highlighted word “however” can be replaced, with no
change in meaning, by
a) besides
b) though
c) then
d) since
Comentários: nessa questão, você deve se lembrar que as linking words, as conjunções do Inglês,
servem para estabelecer uma relação entre as ideias expressadas em uma frase. However significa
contudo, entretanto, e é um exemplo de conjunção adversativa. Ele estabelece, no trecho, uma
relação de oposição, contraste entre as ideias. Você deveria identificar nas alternativas qual outra
conjunção poderia substituir however sem alteração de sentido. Vamos às alternativas:
Besides significa além disso, acrescentando ideias. Assim, não estabelece oposição de ideias como
however. Alternativa INCORRETA.
Though significa embora, contudo. Dessa forma, estabelece oposição de ideias como however,
podendo substituí-lo sem alteração de sentido. Alternativa CORRETA.
Then significa então, indicando uma conclusão. Alternativa INCORRETA.
Since pode indicar marco temporal (desde) ou introduzir uma explicação (já que, como).
Alternativa INCORRETA.
GABARITO: B

AULA 02 – VERBS IN TEXTS 96


TEACHER ANDREA BELO

QUESTÃO 05 (AFA/INÉDITA) – Mark the INCORRECT option according to the text.


a) addiction to social media can be compared to gambling addiction.
b) social media platforms have created a machine that treats people as addicts.
c) there are no explanations on why people engage in writing online for hours each day.
d) in practice, not everyone has the same chance of being successful online.
Comentários: muita atenção, pois a questão pede que você identifique a alternativa incorreta de
acordo com o texto. Assim, o nosso gabarito será a afirmação falsa.
De acordo com a letra A, o vício em mídias sociais (addiction=vício) pode ser comparado ao vício
em jogos de azar. → “If social media is an addiction machine, the addictive behaviour it is closest
to is gambling...” Se a mídia social é uma máquina de dependência, o comportamento viciante
mais próximo é o jogo. (6º parágrafo). Essa informação está de acordo com o texto. Como estamos
em busca da incorreta, não é esse o nosso gabarito. Alternativa INCORRETA.
Conforme a alternativa B, as plataformas de mídias sociais criaram uma máquina que trata as
pessoas como viciados. → “Whether or not we think we are addicted, the machine treats us as
addicts.” Independente se nós achamos ou não que somos viciados, a máquina nos trata como tal.
(5º parágrafo). Essa informação está de acordo com o texto. Como estamos em busca da incorreta,
não é o nosso gabarito. Alternativa INCORRETA.
Segundo a letra C, não há explicações sobre porque as pessoas escrevem on-line por horas todos
os dias. Essa informação NÃO está de acordo com o texto. → “What is the incentive to engage in
writing like this for hours each day?... What gets us hooked? Approval, attention, retweets, shares
and likes.” Qual é o incentivo para escrever dessa maneira por horas todos os dias? ... O que nos
deixa viciados? Aprovação, atenção, retweets, compartilhamentos e curtidas. (3º parágrafo).
Como estamos em busca da incorreta, é esse o nosso gabarito. Alternativa CORRETA.
Para a alternativa D, na prática, nem todo mundo tem a mesma chance de ser bem-sucedido na
internet. Essa informação está de acordo com o texto. → “In principle, anyone can win big; in
practice, not everyone is playing with the same odds.” Em princípio, qualquer um pode ter muito
sucesso; na prática, nem todo mundo está jogando com as mesmas chances. (7º parágrafo). Como
estamos em busca da incorreta, não é esse o nosso gabarito. Alternativa INCORRETA.
GABARITO: C

QUESTÃO 06 (AFA/INÉDITA) – Approval, attention, retweets, shares and likes are mentioned in
the text as examples of
a) online communication negative impacts.
b) emergency measures to overcome internet addiction.
c) reasons why people get addicted to social media.
d) reasons why people usually avoid social media.
Comentários: observe que, no texto, approval, attention, retweets, shares e likes são
mencionados como razões pelas quais as pessoas ficam viciadas em redes sociais. → “What is the
incentive to engage in writing like this for hours each day?... What gets us hooked? Approval,

AULA 02 – VERBS IN TEXTS 97


TEACHER ANDREA BELO

attention, retweets, shares and likes.” Qual é o incentivo para escrever dessa maneira por horas
todos os dias? ... O que nos deixa viciados? Aprovação, atenção, retweets, compartilhamentos e
curtidas. (3º parágrafo). Vejamos as alternativas:
Como vimos, aprovação, atenção, retweets, compartilhamentos e curtidas não são mencionados
no texto como impactos negativos da comunicação online, mas sim como razões pelas quais as
pessoas ficam viciadas em redes sociais. Alternativa INCORRETA.
Approval, attention, retweets, shares e likes não são mencionados no texto como medidas
emergenciais para superar o vício em internet. Alternativa INCORRETA.
De acordo com o que estudamos acima, aprovação, atenção, retweets, compartilhamentos e
curtidas são mencionados no texto justamente como razões pelas quais as pessoas ficam viciadas
em redes sociais. Alternativa CORRETA.
Como vimos, approval, attention, retweets, shares e likes não são mencionados no texto como
razões pelas quais as pessoas evitam (avoid) as redes sociais, sendo justamente o motivo pelos
quais elas buscam as redes e se tornam viciadas. Alternativa INCORRETA.
GABARITO: C

QUESTÃO 07 (AFA/INÉDITA) – Read the statements below and mark the option that contains the
correct ones according to the text.
I – It doesn’t matter if people consider themselves addicts; the machine treats them this way.
II – Attention is the main key to addiction.
III – Theoretically, anyone can succeed online. In reality, not everybody has the same chances of
succeeding.
a) Only sentences I and II are correct.
b) Only sentences I and III are correct.
c) Only sentences II and III are correct.
d) All sentences are correct.
Comentários: a questão pede que você marque a alternativa que contém as afirmações corretas
de acordo com o texto. Vamos analisar essas afirmações.
Não importa se as pessoas se consideram viciadas, a máquina as trata dessa forma. → “Whether
or not we think we are addicted, the machine treats us as addicts.” Independente se nós achamos
ou não que somos viciados, a máquina nos trata como tal. (5º parágrafo). Afirmação CORRETA.
Atenção é a chave principal do vício. → “Addiction is all about attention”. (5º parágrafo). Afirmação
CORRETA.
Teoricamente, qualquer um pode ser bem-sucedido online. Na realidade, nem todo mundo tem
as mesmas chances de ter sucesso. → “In principle, anyone can win big; in practice, not everyone
is playing with the same odds.” Em princípio, qualquer um pode ter muito sucesso; na prática,
nem todo mundo está jogando com as mesmas chances. (7º parágrafo). Afirmação CORRETA.

AULA 02 – VERBS IN TEXTS 98


TEACHER ANDREA BELO

Vamos agora às alternativas:


Apenas as afirmações I e II estão corretas. Como vimos, todas estão corretas. Alternativa
INCORRETA.
Apenas as afirmações I e III estão corretas. Como vimos, todas estão corretas. Alternativa
INCORRETA.
Apenas as afirmações II e III estão corretas. Como vimos, todas estão corretas. Alternativa
INCORRETA.
Todas as afirmações estão corretas. Como vimos, todas estão corretas. Alternativa CORRETA.
GABARITO: D

QUESTÃO 08 (AFA/INÉDITA) – Choose the best option to change the underlined sentence “Social
media platforms have created a machine [...]” (paragraph 1) into the passive form.
A machine ____________________ by social media platforms.
a) has been created
b) has created
c) had been created
d) had created
Comentários: nessa questão é importante que você se lembre da voz passiva. Resumidamente, a
voz passiva em Inglês é usada quando, ao transmitirmos a mensagem, queremos colocar o foco
no que é feito e não em quem pratica a ação. Quanto à estrutura, vamos sempre precisar de uma
forma do verbo TO BE no tempo verbal que queremos usar e do past participle do verbo que
descreve a ação. Além disso, o objeto vai virar sujeito e o sujeito vai virar o que chamamos, em
Português, de agente da passiva. Na questão, temos uma ação no tempo verbal Present Perfect-
have created. Para transformar para a voz passiva, vamos usar o verbo TO BE nesse tempo verbal
(has been) e o particípio do verbo create (created).
Social media platforms have created a machine. = A machine has been created by social media
platforms.
Observe que foi necessário usar o auxiliar has, pois, na passiva, o sujeito passou a ser a machine.
Vamos às alternativas: Como vimos, has been created é a forma correta da voz passiva no caso.
Alternativa CORRETA.
Has created não está correto, pois faltou o verbo TO BE. Alternativa INCORRETA.
Had been created não está correto, pois não foi mantido o tempo verbal Presente Perfect no verbo
TO BE, que foi usado no Past Perfect (had been). Alternativa INCORRETA.
Had created também não está correto, pois faltou o verbo TO BE. Além disso, o tempo verbal
também não foi mantido. Alternativa INCORRETA.
GABARITO: A

AULA 02 – VERBS IN TEXTS 99


TEACHER ANDREA BELO

QUESTÃO 09 (AFA/INÉDITA) – The word “meanwhile” (paragraph 4) indicates in the text that
a) some facts happen at different times.
b) a single fact interferes in others.
c) one fact is more important than others.
d) some facts happen at the same time.
Comentários: “Meanwhile” é um conectivo que transmite a ideia de “enquanto isso”, indicando
que algo acontece ao mesmo tempo que outro fato. No texto, temos que “enquanto isso,
hashtags e trending topics destacam até que ponto todos esses protocolos estão organizados em
torno da massificação de vozes individuais...”. De acordo com a letra A, o conectivo transmite a
ideia de que alguns fatos acontecem em momentos diferentes, o que, conforme vimos, não está
correto. Alternativa INCORRETA.
Segundo a alternativa B, o conectivo indica que um único fato interfere em outros. Como vimos,
a noção é temporal (enquanto isso). Alternativa INCORRETA.
Conforme a letra C, o conectivo indica que um fato é mais importante que outros. Como vimos, a
noção é temporal (enquanto isso). Alternativa INCORRETA.
Para a letra D, o conectivo indica que alguns fatos acontecem ao mesmo tempo que outros. Como
vimos, é exatamente essa a ideia transmitida por “meanwhile” (enquanto isso). Alternativa
CORRETA.
GABARITO: D

QUESTÃO 10 (AFA/INÉDITA) – In the sentence “We are users, much as cocaine addicts are users.”
(paragraph 2), the author DOESN’T
a) make a comparison.
b) treat drug users and internet addicts as different.
c) bring impacting information about social media addiction.
d) describe social media addicts in a certain way.
Comentários: a questão quer que você identifique, dentre as alternativas, o que o autor do texto
NÃO FAZ. Segundo a letra A, o autor não faz uma comparação. Isso não está correto, pois a
estrutura much as faz justamente uma comparação entre viciados em internet e viciados em
cocaína, dando a ideia de “assim como”. Alternativa INCORRETA.
Para a alternativa B, o autor não trata usuários de drogas e viciados em internet como diferentes.
Isso está correto, uma vez que, segundo o trecho trazido pela questão, realmente, eles estão
equiparados. “Somos usuários, assim como os viciados em cocaína são usuários.” Alternativa
CORRETA.
Conforme a alternativa C, o autor não traz uma informação impactante sobre o vício em redes
sociais. Certamente, afirmar que o vício em internet se compara ao vício em cocaína é impactante.
Alternativa INCORRETA.

AULA 02 – VERBS IN TEXTS 100


TEACHER ANDREA BELO

De acordo com a letra D, o autor não descreve os viciados em redes sociais em certa medida. Isso
não está correto, pois o autor os descreve, sim, de certa maneira, ao compará-los a usuários de
cocaína. Alternativa INCORRETA.
GABARITO: B

QUESTÕES COLÉGIO NAVAL


Directions: Answer questions 1 to 10 according to TEXT I.
Many COVID-19 patients lost their sense of smell. Will they get it back?
IN EARLY MARCH, Peter Quagge began experiencing COVID-19 symptoms, such as chills and a low-
grade fever. As he cut pieces of raw chicken to cook for dinner one night, he noticed he couldn’t
smell the meat. “Must be really fresh,” he remembers thinking. But the next morning he couldn’t
smell the Dial soap in the shower or the bleach he used to clean the house. “It sounds crazy, but
I thought the bleach had gone bad,” he says. When Quagge stuck his head into the bottle and
took a long whiff, the bleach burned his eyes and nose, but he couldn’t smell a thing.
The inability to smell, or anosmia, has emerged as a common symptom of COVID-19. Quagge was
diagnosed with COVID-19, though he was not tested, since tests were not widely available at the
time. He sought anosmia treatment with multiple specialists and still has not fully recovered his
sense of smell.
Case reports suggest that anywhere between 34 and 98 percent of hospitalized patients with
COVID-19 will experience anosmia. One study found that COVID-19 patients are 27 times more
likely than others to lose their sense of smell, making anosmia a better predictor of the illness
than fever.
For most COVID-19 patients who suffer anosmia, the sense returns within a few weeks, and
doctors don’t yet know if the virus causes long-term smell loss. While not being able to smell may
sound like a small side effect, the results can be devastating. The sense is intricately tied up in
self-preservation—the ability to smell fire, chemical leaks, or spoiled food—and in our ability to
pick up on complex tastes and enjoy food.
“So many of the ways we connect with each other is over meals or over drinks,” says Steven
Munger, director of the Center for Smell and Taste at the University of Florida. “If you can’t fully
participate in that, it creates a sort of social gap.”
Smell even plays a role in our emotional lives, connecting us to loved ones and memories. People
without smell often report feeling isolated and depressed and losing their enjoyment in intimacy.
Now scientists are starting to unravel how COVID-19 affects this critical sense, hoping those
discoveries will help thousands of newly anosmic people looking for answers.
What the nose knows
The olfactory system, which allows humans and other animals to smell, is essentially a way of
decoding chemical information. When someone takes a big sniff, molecules travel up the nose to
the olfactory epithelium, a small piece of tissue at the back of the nasal cavity.

AULA 02 – VERBS IN TEXTS 101


TEACHER ANDREA BELO

Those molecules bind to olfactory sensory neurons, which then send a signal by way of an axon,
a long tail that threads through the skull and delivers that message to the brain, which registers
the molecules as, say, coffee, leather, or rotting lettuce.
Scientists still don’t fully understand this system, including exactly what happens when it stops
working. And most people don't realize how common smell loss really is, Munger says. “That lack
of public understanding means there’s less attention to try to understand the basic functions of
the system.”
People can lose their sense of smell after suffering a viral infection, like influenza or the common
cold, or after a traumatic brain injury. Some are born without any sense of smell at all or lose it
because of cancer treatments or diseases like Parkinson’s and Alzheimer’s. It may also fade as
people age. While smell disorders aren’t as apparent as hearing loss or vision impairment, data
from the National Institutes of Health (NIH) show that nearly 25 percent of Americans over the
age of 40 report some kind of change in their sense of smell, and over 13 million people have a
measurable disorder like anosmia, the total loss of smell, or hyposmia, a partial loss. Such
conditions can last for years or even be permanent.
It’s not clear if COVID-19 anosmia is different from other instances of smell loss caused by a virus,
but those who experience anosmia due to COVID-19 appear to be unique in a few ways. First, they
notice the loss of the sense immediately because it’s not accompanied by the congestion or
stuffiness that generally characterizes the early stages of virally induced anosmia.
“It’s very dramatic,” says Danielle Reed, associate director of the Monell Chemical Senses Center
in Philadelphia, which studies smell and taste loss. “People just cannot smell anything.”
Another notable difference is that many patients with COVID-19 who report losing their sense of
smell get it back relatively quickly, in just a few weeks, unlike most people who experience
anosmia from other viruses, which can last months or years.
Quagge estimates he’s recovered about 60 percent of his sense of smell so far, but he says in the
early days, without any information about when or if he’d ever get it back, he was scared. An avid
amateur chef, he had to rely on his family to tell him if the milk was bad, and he couldn’t smell his
wife’s perfume. “Stuff that gets to your soul,” he says. “It bummed me out.”
(Adapted from https://www.nationalgeographic.com/science/2020/08/thousands-covid-19-patients-lost-sense-smell-will-get-back-cvd/)

QUESTÃO 01 (COLÉGIO NAVAL/INÉDITA) – According to the text, mark the correct alternative.
a) COVID-19 patients lose their sense of smell as a common symptom.
b) Anosmia has been considered a better predictor of the illness than fever.
c) As a consequence, the virus causes long-term smell loss.
d) Anosmia was diagnosed in all COVID-19 patients.
Comentários: A alternativa A está incorreta. O texto não afirma que todos os afetados pela
COVID-19 irão perder o olfato, mas que há essa possibilidade.
A alternativa B está correta. O texto afirma que a anosmia tem sido considerada um melhor
preditor para a COVID-19 do que a febre.

AULA 02 – VERBS IN TEXTS 102


TEACHER ANDREA BELO

A alternativa C está incorreta. O texto afirma que os especialistas ainda não sabem se é um
sintoma que afeta a longo prazo; em nenhum momento afirmam que, como consequência, o vírus
causa uma perda de olfato a longo prazo.
A alternativa D está incorreta. O texto não afirma que a anosmia foi diagnosticada em todos os
pacientes da COVID-19, mas sim que a maioria relatou a anosmia como sintoma.
GABARITO: B

QUESTÃO 02 (COLÉGIO NAVAL/INÉDITA) – Mark the correct option.


a) Doctors claim that the virus causes long-term smell loss.
b) Doctors claim that the virus does not cause long-term smell loss.
c) Doctors affirm that the smell loss is inevitable for those who have been affected by the virus.
d) Doctors still don't know if the virus causes long-term smell loss
Comentários: A alternativa A está incorreta. Os médicos não afirmam que o vírus resulta em uma
perda de olfato a longo prazo, mas sim que ainda não sabem ao certo se o vírus pode causar perda
de olfato a longo prazo.
A alternativa B está incorreta. Os médicos não afirmam que o vírus não resulta em uma perda de
olfato a longo prazo, mas sim que ainda não sabem ao certo se o vírus pode causar perda de olfato
a longo prazo.
A alternativa C está incorreta. Os médicos não afirmam que a perda de olfato é inevitável para os
afetados pelo vírus, mas sim que ainda não sabem ao certo se o vírus pode causar perda de olfato
a longo prazo.
A alternativa D está correta. Os médicos afirmam que ainda não sabem se o vírus pode causar
perda de olfato a longo prazo.
GABARITO: D

QUESTÃO 03 (COLÉGIO NAVAL/INÉDITA) – There are __ verbs used in the past in the underlined
sentence in the tenth paragraph.
a) One.
b) two.
c) three.
d) zero.
Comentários: Ao observar o trecho sublinhado, percebe-se o verbo "caused" no past participle
que pode facilmente ser confundido com o verbo "caused" no simple past, porém, ao analisar a
frase sublinhada, percebe-se que a mesma se encontra no presente e não no passado. Tal
afirmativa pode ser confirmada ao analisarmos o verbo “appear” que aparece no presente.
GABARITO: D

AULA 02 – VERBS IN TEXTS 103


TEACHER ANDREA BELO

QUESTÃO 04 (COLÉGIO NAVAL/INÉDITA) – Mark the option that can replace the word "very" in
the eleventh paragraph without changing its meaning.
a) a lot.
b) really.
c) also.
d) to.
Comentários: A alternativa A está incorreta. Essa expressão não substitui corretamente "very"
pois indica quantidade, e não intensidade.
A alternativa B está correta. A expressão "really" é um sinônimo de "very" e ambos indicam
intensidade.
A alternativa C está incorreta. O advérbio "also" indica aditividade, e não intensidade, assim como
“very”.
A alternativa D está incorreta. "To" é uma preposição e tem como uma de suas funções, a
utilização com um verbo para torná-lo infinitivo, e não um advérbio que indica intensidade, que
é o caso de “very”.
GABARITO: B

QUESTÃO 05 (COLÉGIO NAVAL/INÉDITA) – Mark the alternative that can answer the question
below according to the text.
Will people get their sense of smell back after recovering from COVID-19?
a) No, it's a long-term smell loss.
b) Maybe, it depends on the patient.
c) Yes, 100% of the patients have recovered their sense of smell after COVID-19.
d) We can't say yet, health authorities still don't know if anosmia could be long term smell loss.
Comentários: A alternativa A está incorreta. O texto não afirma que a perda de olfato é,
definitivamente, uma consequência a longo prazo, mas sim que ainda não se pode dizer se a
anosmia pode ser uma perda de olfato a longo prazo.
A alternativa B está incorreta. O texto não afirma que é uma consequência dependente do caso,
mas sim que ainda não se pode dizer se a anosmia pode ser uma perda de olfato a longo prazo.
A alternativa C está incorreta. O texto não afirma que 100% dos pacientes recuperam o olfato
após a COVID-19, mas sim que ainda não se pode dizer se a anosmia pode ser uma perda de olfato
a longo prazo.
A alternativa D está correta. O texto afirma que ainda não se pode dizer se a anosmia pode ser
uma perda de olfato a longo prazo.
GABARITO: D

AULA 02 – VERBS IN TEXTS 104


TEACHER ANDREA BELO

QUESTÃO 06 (COLÉGIO NAVAL/INÉDITA) – Read the sentence. "While not being able to smell
may sound like a small side effect, the results can be devastating." Choose the alternative that
expresses a similar idea to the sentence above:
a) The results from anosmia aren't harmful to health.
b) Everyone has hurtful results from the total loss of smell.
c) Even though it may sound like a non-harmful symptom, smell loss can be destructive.
d) Not being able to smell sounds like a small side effect, with good results regardless of the
case.
Comentários: A alternativa A está incorreta. Não possui uma ideia similar ao trecho dado;
expressa que os resultados da anosmia não são danosos à saúde, ao contrário do trecho dado,
que expressa que apesar de parecer um sintoma não doloroso, ele pode ser destrutivo.
A alternativa B está incorreta. Não possui uma ideia similar ao trecho dado; expressa que todos
têm resultados dolorosos da perda total do olfato, ao contrário do trecho dado, que expressa que
apesar de parecer um sintoma não doloroso, ele pode ser destrutivo.
A alternativa C está correta. Possui uma ideia similar ao trecho dado; expressa que apesar de
parecer um sintoma não doloroso, ele pode ser destrutivo. Assim como o trecho dado expressa.
A alternativa D está incorreta. Não possui uma ideia similar ao trecho dado; expressa que a perda
de olfato parece um sintoma inofensivo, com bons resultados independente do caso, ao contrário
do trecho dado, que expressa que apesar de parecer um sintoma não doloroso, ele pode ser
destrutivo.
GABARITO: C

QUESTÃO 07 (COLÉGIO NAVAL/INÉDITA) – According to the first paragraph, we can assume that:
a) At first, Peter noticed that he was having COVID-19 symptoms.
b) In the first moment, he just felt the loss of smell sense.
c) Peter's first symptoms were chills, anosmia and low-grade fever.
d) He was an asymptomatic case.
Comentários: A alternativa A está incorreta. Em primeiro momento, Peter não notou seus
sintomas da COVID-19; notou a febre e os calafrios, mas não a perda de olfato; e não os relacionou
à COVID-19, imediatamente.
A alternativa B está incorreta. Em primeiro momento, ele não notou a perda de olfato, apenas os
calafrios e febre.
A alternativa C está correta. Assim como o primeiro parágrafo, a alternativa indica que os
primeiros sintomas de Peter foram calafrios, febre e perda de olfato.
A alternativa D está incorreta. Peter não foi um caso assintomático, ele vivenciou calafrios, febre
e perda de olfato como sintomas da COVID-19.
GABARITO: C

AULA 02 – VERBS IN TEXTS 105


TEACHER ANDREA BELO

QUESTÃO 08 (COLÉGIO NAVAL/INÉDITA) – Mark the INCORRECT alternative, according to the


third paragraph:
a) COVID-19 patients will experience anosmia as a symptom
b) anosmia it's better than fever to diagnose COVID-19.
c) COVID-19 patients tend to have anosmia, differently from others.
d) loss of smell is a common symptom from COVID-19.
Comentários: A alternativa A está correta. Pacientes da COVID-19 não irão, necessariamente, ter
a perda de olfato como um sintoma; sendo assim, a opção com informação incorreta.
A alternativa B está incorreta. O texto afirma que a anosmia está sendo considerada um preditor
para a COVID-19 melhor do que a febre, contendo, assim, informações corretas.
A alternativa C está incorreta. O texto afirma que pacientes da COVID-19 tendem a ter a anosmia
como um sintoma, contendo, assim, informações corretas.
A alternativa D está incorreta. O texto afirma que a perda de olfato é um sintoma comum da
COVID-19, contendo, assim, informações corretas.
GABARITO: A

QUESTÃO 09 (COLÉGIO NAVAL/INÉDITA) – What does the expression "Bummed me out"


(thirteenth paragraph) mean?
a) makes me feel down.
b) makes me feel happy.
c) makes me feel angry.
d) makes me feel nauseated.
Comentários: A alternativa A está correta. A expressão "Bummed me out" significa que algo te
deixou triste, assim como "makes me feel down".
A alternativa B está incorreta. A expressão não significa que algo te deixou feliz, mas sim que te
deixou triste.
A alternativa C está incorreta. A expressão não significa que algo te deixou com raiva, mas sim
que te deixou triste.
A alternativa D está incorreta. A expressão não significa que algo te deixou enjoado, mas sim que
te deixou triste.
GABARITO: A

QUESTÃO 10 (COLÉGIO NAVAL/INÉDITA) – Considering the use of the verb tense, mark the
alternative that completes the sentence below correctly: At first, Peter
a) has the common symptoms.
b) was having the common symptoms.
c) have the common symptoms.
d) had the common symptoms.

AULA 02 – VERBS IN TEXTS 106


TEACHER ANDREA BELO

Comentários: A alternativa A está incorreta. O tempo verbal em "has" indica presente, o que não
se encaixa com a frase dada pela questão, que pede um tempo verbal no passado, com a ação
finalizada.
A alternativa B está incorreta. O tempo verbal em "was having" indica continuidade no passado,
o que não se encaixa com a frase dada pela questão, que pede um tempo verbal no passado, com
a ação finalizada.
A alternativa C está incorreta. O tempo verbal em "have" indica presente, o que não se encaixa
com a frase dada pela questão, que pede um tempo verbal no passado, com a ação finalizada.
A alternativa D está correta. O tempo verbal em "had" indica uma ação finalizada no passado, o
que se encaixa com a frase dada pela questão.
GABARITO: D

QUESTÕES EAM
Questão 01 (EAM/INÉDITA) – Read the dialogue and mark the right option to fill in the gaps
respectively.
A: __________ you like the President’s speech yesterday?
B: Yes, I __________.
A: Who __________ you watch with?
B: I __________ with my girlfriend. Did you watch it?
A: No, I __________.
a) Did / did / went / watch / wasn’t
b) Were / was / were / watch / didn’t
c) Was / was / did / watched / wasn’t
d) Were / was / did / watched / didn’t
e) Did / did / did / watched / didn’t
Comentários: A primeira lacuna deve ser preenchida por “did”. A presença do verbo “like” no
infinitivo indica que há um auxiliar “do, did, will” no lugar da lacuna. A presença da palavra
“yesterday” no fim da frase indica que a frase está no passado. O passado exige o auxiliar “did”
para fazer a pergunta.
A segunda lacuna deve ser preenchida por “did”. A pergunta foi feita com “did”, portanto, a
resposta curta deve ser dada utilizando o mesmo auxiliar (Yes, I did).
A terceira lacuna deve ser preenchida por “did”. A pergunta se refere ao discurso do dia anterior
e, por isso, deve-se usar o auxiliar “did” para perguntar com quem a pessoa assistiu.
A quarta lacuna deve ser preenchida por “watched”. A conversa está se passando toda no tempo
passado. Por isso, devemos usar “watched” ao invés de “watch”.

AULA 02 – VERBS IN TEXTS 107


TEACHER ANDREA BELO

A quinta lacuna deve ser preenchida por “didn’t”. A pergunta foi feita com o auxiliar “did”, “did
you watch it?”, logo a resposta deve também utilizar o auxiliar “did” na negativa, já que a resposta
começa com a palavra “no”.
GABARITO: E

Questão 02 (EAM/INÉDITA) – Read the sentences and mark the correct option to fill in the
blanks respectively.
__________ name is Tom. I’m 16 and I live __________ Warsaw which is the capital of Poland.
Actually, I come from another city - Wroclaw, where I __________ for eleven years.
Adapted from (https://sciaga.pl/tekst/2266-3-how_to_write_a_about_myself_an_example)

a) My / in / lived
b) Your / at / lived
c) Her / on / lives
d) My / in / live
e) His / in / live
Comentários: A primeira lacuna deve ser preenchida por “my”. O início do período seguinte deixa
claro que o texto trata do próprio narrador, ao começar com o pronome “I”.
A segunda lacuna deve ser preenchida por “in”. Por regra, sempre que vai se falar EM alguma
cidade, usa-se a preposição “in”.
A terceira lacuna deve ser preenchida por “lived”. No trecho anterior, ele diz que vive em
Varsóvia.
Portanto, se torna claro que o verbo da lacuna deve estar no passado para dizer de onde ele veio
e onde ele viveu.
GABARITO: A

AULA 02 – VERBS IN TEXTS 108


TEACHER ANDREA BELO

QUESTÃO 03 (EAM/INÉDITA) – What’s the main verb tense used in the comic strip?
a) Present Continuous
b) Present Simple
c) Simple Past
d) Future Simple
e) Present Perfect
Comentários: A alternativa A está incorreta. Não há nenhuma estrutura no “Present Continuous”
na tirinha. A estrutura do present continuous exige que o verbo tenha sua terminação em -ing.
A alternativa B está correta. A maioria das falas do diálogo estão no “Present Simple”. Há quatro
falas no present simple. Portanto, podemos afirmar que o present simple é o tempo verbal
principal da tirinha.
A alternativa C está incorreta. Há dois verbos na tirinha que estão conjugados no simple past.
Entretanto, há outros quatro no present simple e, por isso, não se pode dizer que o simple past é
o principal tempo verbal da tirinha.
A alternativa D está incorreta. Não se pode observar o auxiliar “will” indicativo de futuro, e nem
a expressão “going to” que também serve como indicativo de futuro. Isso elimina a alternativa.
A alternativa E está incorreta. Não se pode observar a presença dos auxiliares “have” ou “has”
acrescidos do verbo conjugado no particípio passado, que é a estrutura do “present perfect”. Isso
elimina a alternativa.
GABARITO: B

QUESTÃO 04 (EAM/INÉDITA) – Use the verbs in the parentheses to complete the following
statements.
I – she __________ (study) law at the University of Warsaw.
II – Some time later, I __________ (notice) that I could do a lot more things with the computer.
III – She used to __________ (do) the dishes at night.
Now mark the option which completes them respectively.
a) study / noticed / do
b) studies / noticed / doing
c) studied / notice / doing
d) studies / noticed / do
e) study / notice / does
Comentários: A primeira lacuna deve ser preenchida por “studies”. A frase está no presente e o
sujeito é “she”.
Portanto, o verbo precisa ser conjugado com “-IES” no final.

AULA 02 – VERBS IN TEXTS 109


TEACHER ANDREA BELO

A segunda lacuna deve ser preenchida por “noticed”. A frase está no passado (evidenciado pelo
uso de could logo em seguida). Portanto, o verbo deve ser acrescido de “-D” no final para
demarcar o tempo passado.
A terceira lacuna deve ser preenchida por “do”. A expressão “used to” pode ser traduzida como
“costumava” e, assim como no português, após essa expressão usa-se o verbo no infinitivo, ainda
que esteja falando de um fato passado. Ex: Ele costumava fazer…/He used to do…
GABARITO: D

QUESTÃO 05 (EAM/INÉDITA) – Look at the picture below.

What are they doing in the picture?


a) They are trying to stop the girl from passing.
b) They are playing jump rope.
c) They are making the girl fall down.
d) They are throwing the rope on the floor.
e) They don’t like to play with the girl.
Comentários: A alternativa A está incorreta. Ao analisar a imagem, não é possível concluir que
eles estão tentando impedir a menina de passar, até porque todos estão sorrindo e,
aparentemente, brincando.
A alternativa B está correta. A imagem nos conduz a acreditar que elas estão, de fato, brincando
de pular corda.
A alternativa C está incorreta. Eles não estão tentando fazer a menina cair, eles estão brincando
de pular corda.
A alternativa D está incorreta. A imagem não nos permite concluir que eles estão jogando a corda
no chão, já que ambos estão com a corda na mão e parecem estar segurando firmemente.
A alternativa E está incorreta. Não se pode concluir que eles não gostam de brincar com a menina,
já que os três estão sorrindo e brincando. Não há elementos na imagem que nos permita tirar
essa conclusão.
GABARITO: B

AULA 02 – VERBS IN TEXTS 110


TEACHER ANDREA BELO

QUESTÕES EEAR
Read the text answer question 01
Douglas Yancey, "Doug" Fannie, is the titular character and chief protagonist of the animated
series Doug. Douglas is a shy, insecure, and somewhat clumsy 11-year-old boy with a wild
imagination and a big heart. He owns a very expressive dog named Porkchop. Doug is a 6th Grade
average kid who always tries to do the right thing, despite his frequent failures. He is naive at
times, and fears that people will think of him as a loser.
Adapted from https://doug.fandom.com/wiki/Doug_Funnie.

QUESTÃO 01 (EEAR – CFS/2020) – In the sentence “He owns a very expressive dog named
“Porkchop”, the verb OWNS can be replaced by __________ without change in meaning.
a) has
b) buys
c) needs
d) wants
Comentários: O verbo “owns” é o verbo ‘possuir’ em inglês, portanto, a única alternativa possível
de substituir sem mudar o sentido é a letra A (ter) pois, o verbo ‘ter’ e ‘possuir’ são sinônimos.
As alternativas B, C e D utilizam verbos que não são sinônimos do verbo ‘possuir’ como “buys”
(comprar), “needs” (precisar) e “want” (querer). Portanto, o gabarito é a letra A.
GABARITO: A

Read the text and answer questions 02 and 03.


An e-mail to Mary
Hi, Mary!
How are you? I miss you so much!
I arrived in Rome last week and I’m having a lot of fun. This morning my parents and I woke up at
8:00 and had a great breakfast at the hotel. Afterwards, I walked a little and saw many tourists
(many from Russia and China) and even some famous people!
Now we are going to Venice.
How are things in Las Vegas? Come meet us!
See you soon!
Paul.

QUESTÃO 02 (EEAR – CFS/2020) – All verbs below, underline in the text, are in the Simple Past,
EXCEPT:
a) Arrived
b) Woke
c) Come
d) Had

AULA 02 – VERBS IN TEXTS 111


TEACHER ANDREA BELO

Comentários: Na letra A encontramos o verbo “to arrive” no passado simples (arrived). Na letra
B encontramos o verbo “to wake” também no passado simples (woke). Na letra C, encontramos
o verbo “come” que está no modo imperativo, portanto, é o nosso gabarito. O passado seria
“came”. Na letra D encontramos o verbo “to have” no passado (had).
GABARITO: C

QUESTÃO 03 (EEAR – CFS/2020) – Where did Paul go on his vacation?


a) Rome, Italy
b) Hong Kong, China
c) Miami, The United States of America
d) Las Vegas, The United States of America
Comentários: A pergunta é “where did Paul go on his vacation?”, ou seja, quer saber ele foi nas
últimas férias.
Em nosso texto, encontramos a frase “I arrived in Rome last week” que significa ‘Eu cheguei em
Roma semana passada’. Portanto, ele viajou para Roma, Itália que está na letra A. Nas demais
alternativas encontram-se locais para confundir o candidato, mas que não são locais para onde
ele viajou, nem Hong Kong, nem Miami nem Las Vegas.
GABARITO: A

Read the text and answer question 04.


The Lazy Song – Bruno Mars
Today I don’t feel like doing anything
I just wanna lay in my bed
Don’t feel like picking up my phone
So leave a message at the tone
‘Cause today I swear I’m not doing anything
https://www.vagalume.com.br/thelazysong-11/brunomars.html

QUESTÃO 04 (EEAR – CFS/2020) – Based on the song extract, the person in the song
a) Doesn’t want to do anything today.
b) Wants to do lots of things today.
c) Is sick and can’t do anything.
d) Has many things to do.

AULA 02 – VERBS IN TEXTS 112


TEACHER ANDREA BELO

Comentários: Na música de Bruno Mars, quando ele diz na primeira linha “Today I don’t feel like
doing anything”, precisamos saber que o verbo “feel like” significa ‘estar a fim de fazer algo’.
Como ele diz “I don’t feel like”, isso significa que ele não está a fim de fazer nada.
Portanto, nosso gabarito é a letra A (Doesn’t want to do anything today), pois o verbo ‘querer’ é
o mesmo que dizer que ‘não está a fim’.
As demais alternativas possuem verbos que não representam ‘estar a fim’ como Wants to do lots
of things today (quer fazer muitas coisas), Is sick (está doente) ou Has many things to do (tem
muitas coisas para fazer). Não é o que está sendo dito na primeira linha do texto.
GABARITO: A

Read the text and answer questions 05 and 06.


Rude
Can I have your daughter for the rest of my life? Say yes, say yes
‘Cause I need to know
You say I’ll never get your blessing till the day I die
Tough luck my friend but the answer is no!
Why you gotta be so rude?
Don’t you know I’m human too
Why you gotta be so rude
I’m gonna marry her anyway
(Marry that girl) Marry her anyway
(Marry that girl) Yeah no matter what you say
(Marry that girl) And we’ll be a family
https://www.vagalume.com.br/magie-11/rude.html

QUESTÃO 05 (EEAR – CFS/2020) – What is the correct plural form of the words, in bold type, in
the text?
a) daughter, lives, days, and family.
b) daughters, life, day, and families.
c) daughters, lives, days, and family.
d) daughters, lives, days, and families.
Comentários: A questão pede o plural das palavras em destaque. A primeira palavra, “daughter”,
o plural é regular, apenas com o acréscimo de ‘s’ (daughters).
A segunda palavra tem a regra de que quando termina com as letras ‘fe’ o plural fica com as letras
‘ves’, portanto, o plural de “life” é “lives” assim como diz a alternativa D.

AULA 02 – VERBS IN TEXTS 113


TEACHER ANDREA BELO

As outras duas palavras, “day” e “family” também fazem plurais com o ‘s’. “Day” fica “days” e
“family”, por causa do ‘y’, é retirado o ‘y’ e colocado ‘ies’. Portanto, nosso gabarito é a letra D e
as demais contém plurais incorretos.
GABARITO: D

QUESTÃO 06 (EEAR – CFS/2020) – Based on the text, we can infer that the couple
a) is going to wait for the girl’s father to die to get married in peace.
b) doesn’t want to get married without the blessing pf the girl’s father.
c) doesn’t care about the opinion of the girl’s father and is going to get married.
d) Is going to break up because the girl’s father doesn’t accept the marriage.
Comentários: A pergunta diz que, baseado no texto, o que podemos inferir. Na letra A diz que ele
vai esperar o pai morrer (wait for the girl’s father to die) e isso está incorreto pois, ele diz na
música que quer a benção do pai para se casar com a garota. Na letra B diz que ele não quer se
casar sem a benção (doesn’t want to get married without the blessing) mas ele diz no final da
música (Marry that girl) Marry her anyway (que vai se casar de qualquer jeito), portanto,
incorreta. Na letra C, nosso gabarito, ele diz doesn’t care about the opinion, ou seja, ele não se
preocupa com a opinião e vai se casar de qualquer jeito com a menina, representado pelo termo
‘anyway’. Na letra D diz que ele vai terminar o relacionamento (break up), o que não está correto.
GABARITO: C

Read the text and answer question 07.


Alice in Wonderland
“If you _______ know where you want to go, then it _______ matter which path you take”

QUESTÃO 07 (EEAR – CFS/2020) – Complete the text with the appropriate words.
a) don’t / don’t
b) does / don’t
c) don’t / doesn’t
d) doesn’t / doesn’t
Comentários: Essa questão é uma questão típica de “If Clauses”.
É a Zero Conditional em que o primeiro verbo está no Presente e o segundo também. O primeiro
sujeito é ‘you’. Portanto devemos falar “If you don’t know...”. Enquanto o segundo sujeito é ‘It’.
Para ‘It’ pede-se o auxiliar ‘does’. Então a alternativa correta é a letra C. “If you don’t know where
you want to go, then it doesn’t matter which path you take” e as demais alternativas estão
incorretas.
GABARITO: C

AULA 02 – VERBS IN TEXTS 114


TEACHER ANDREA BELO

QUESTÃO 08 (EEAR – CFS/2020) – Choose the alternative that best completes the dialogue:
Mary: Hi _______ am Mary Smith. _______ am from the USA. What’s your name?
Paul: _______ name is Paul Thompson.
Mary: Nice to meet you. _______ are you from?
Paul: Nice to meet you, too. _______ am from London. And who is _______ woman?
Mary: _______ is Jessica Lopez. _______ is from the USA too.
a) She – She – Her – When – I – that – He – He
b) I – I – My – Where – I – that – She - She
c) I – I – My – When – I – those – He – He
d) I – I – I – Where – He – that – She – She
Comentários: Nessa questão, precisamos analisar todos os pronomes para preencher e assim
vamos eliminando alternativas.
A Mary fala na primeira parte do diálogo Hi I am Mary Smith, representando que é o nome dela,
portanto começa com ‘I’. E ela continua dizendo I am from the USA, precisando de outro pronome
‘I’. Com isso já eliminamos a letra A.
Quando o Paul diz My name is Paul Thompson, dizendo o nome dele, já vemos que agora só é
possível a alternativa B ou C, em que a terceira lacuna é o pronome ‘my’.
Continuando, agora nós conseguimos achar a nossa alternativa correta, porque a Mary pergunta
Where are you from?. Ele vai responder que é de Londres.
E só há como fazer essa pergunta utilizando o WH Questions “WHERE”. Portanto, aqui já
descobrimos que o gabarito é a letra B e as demais alternativas contém palavras que não se
encaixam no texto.
GABARITO: B

Read the text and answer question 09.

AULA 02 – VERBS IN TEXTS 115


TEACHER ANDREA BELO

QUESTÃO 09 (EEAR – CFS/2020) – The term “Wake me up”, in the comic strip, is in the
a) Present Continuous
b) Imperative
c) Future
d) Past
Comentários: A pergunta é em qual tempo está a expressão “Wake me up”. Não podemos dizer
que está em nenhum tempo verbal a não ser o Imperativo da letra B, porque começa com verbo,
característica deste modo. Nas letras A, C e D contém estruturas que não estão no texto. Present
Continuous precisaria do ‘ing’, Future precisaria do ‘will’ ou ‘going to’ e Past precisaria de verbos
no ‘ed’ ou letras diferentes, portanto, o gabarito é a letra B.
GABARITO: B

Read the text and answer question 10.


All of me
John Legend
‘Cause all of me
Loves all of you
Love your curves and all your edges
All your perfect imperfections
Give your all to me
I’ll give my all to me
You’re my end and my beginning
Even when I lose, I’m winning
‘Cause I give you all of me
And you give me all of you (…)

QUESTÃO 10 (EEAR – CFS/2020) – A synonym for the word “edges” (line 03) is:
a) hair
b) bones
c) center
d) margin
Comentários: A pergunta pede um sinônimo da palavra “edges”, que significa ‘bordas’ em inglês.
A única possível é a letra D, “Margin” que significa ‘margem’, ou seja, sinônimo de bordas.
As demais alternativas, “hair”, “bones” e “center” significam palavras que não são similares a
“edges”. Hair (cabelo), Bones (ossos) e Center (centro). Portanto, gabarito letra D.
GABARITO: D

AULA 02 – VERBS IN TEXTS 116


TEACHER ANDREA BELO

QUESTÕES EFOMM
Based on the text below, answer questions 01 to 04.
Nasa SpaceX crew return: Dragon capsule splashes down
Two American astronauts have splashed down, as the first commercial crewed mission to the
International Space
Station returned to Earth.
The SpaceX Dragon Capsule carrying Doug Hurley and Bob Behnken came down in the Gulf of
Mexico just south of Pensacola on Florida's Gulf coast.
They have now emerged from the capsule on to a recovery vessel.
The touchdown marks the first crewed US water landing since the final Apollo moon mission 45
years ago.
Hurley's and Behnken's capsule touched the water at about 14:48 EDT (19:48 BST; 18:48 GMT).
"It's truly our honor and privilege," said Hurley as they arrived.
"On behalf of the SpaceX and Nasa teams, welcome back to Planet Earth. Thanks for flying
SpaceX," the SpaceX mission control said just after splashdown.
President Donald Trump - who attended the capsule's launch two months ago - hailed its safe
return.
"Thank you to all!" he tweeted. "Great to have NASA Astronauts return to Earth after very
successful two month mission."
The successful end to the crew's mission initiates a new era for the American space agency.
All its human transport needs just above the Earth will in future be purchased from private
companies, such as SpaceX.
The government agency says contracting out to service providers in this way will save it billions of
dollars that can be diverted to getting astronauts to the Moon and Mars.
The Dragon capsule launched to the space station at the end of May on a Falcon 9 rocket, also
supplied by SpaceX.
Hurley's and Behnken's mission served as an end-to-end demonstration of the astronaut "taxi
service" the company, owned by tech entrepreneur Elon Musk, will be selling to Nasa from now
on.
The Boeing corporation is also developing a crew capsule solution but has had to delay its
introduction after encountering software problems on its Starliner capsule.
The sight of the vehicle's four main parachutes floating down over the Gulf of Mexico was
confirmation the spacecraft had survived its fiery descent through the atmosphere.
The parachutes then slowed the capsule from about 350mph (560km/h) to just roughly 15mph at
splashdown.
Rigging was then used to hoist the capsule out of the water and on to the recovery vessel.
Technicians monitored "remnant vapours" around the spacecraft before the hatch was opened.

AULA 02 – VERBS IN TEXTS 117


TEACHER ANDREA BELO

The men were to be checked over by medical staff before being flown to shore by helicopter.
The astronauts' Dragon capsule launched to the space station at the end of May on a Falcon 9
rocket, also supplied by SpaceX.
It will now be refurbished to fly again next year.
Nasa Administrator Jim Bridenstine lauded the efforts of everyone involved in Hurley's and
Behnken's mission, and then spoke of his agency's shift in philosophy.
"We don't want to purchase, own and operate the hardware the way we used to," he said.
"We want to be one customer of many customers in a very robust commercial marketplace in
low-Earth orbit. But we also want to have numerous providers that are competing against each
other on cost and innovation and safety, and really create this virtuous cycle of economic
development and capability."
Adapted from https://www.bbc.com/news/science-environment-53621102

QUESTÃO 01 (EFOMM/INÉDITA) – It is possible to infer from the text that


(A) Nasa intends to make its own capsule to satisfy their human transport needs.
(B) it is more expensive to contract other companies to make transport capsules.
(C) Nasa has always contracted private companies to make human transport capsules.
(D) Nasa has no intention to keep operating hardware the same way they used to do.
(E) The SpaceX Dragon Capsule carrying Doug Hurley and Bob Behnken touched the water in the
Atlantic Ocean.
Comentários: A alternativa A está incorreta. O texto diz que a Nasa pretende continuar
contratando empresas privadas para fornecer equipamentos que sirvam para o transporte de
humanos, e não que a Nasa pretende fazer suas próprias cápsulas.
A alternativa B está incorreta. O texto diz que é mais barato para a Nasa contratar outras
empresas para fazer a cápsula do que ela própria fazer. O texto ainda diz que a Nasa deverá
economizar bilhões de dólares nesse processo.
A alternativa C está incorreta. O texto não diz que a Nasa sempre contratou empresas privadas
para produzir as cápsulas. Na verdade, o texto diz que essa é uma nova prática que a Nasa tem a
intenção de manter.
A alternativa D está correta. Podemos entender do texto que a Nasa fez uma mudança em sua
política de contratação de empresas privadas para a produção de cápsulas para o transporte de
humanos. O texto diz claramente que é intenção da Nasa manter essa mudança, e que a Nasa não
tem a intenção de retornar ao seu antigo modo de operar com esses materiais.
A alternativa E está incorreta. O texto diz que a cápsula tocou a água no golfo do México, não no
oceano atlântico.
GABARITO: D

AULA 02 – VERBS IN TEXTS 118


TEACHER ANDREA BELO

QUESTÃO 02 (EFOMM/INÉDITA) – Read the statements about the text and decide whether they
are TRUE (T) or FALSE (F). Mark the correct option.
I – A crewed spacecraft landed on the water for the first time in 45 years.
II – The parachutes slowed the capsule from about 560km/h to just roughly 15km/h.
III – The Boeing corporation has its crew capsule ready for Nasa’s next mission.
IV – Nasa administrator talked about his agency’s change in philosophy after the splashdown.
V – Nasa will assess the possibility of contracting a private company again.
(A) F / T / T / F / F
(B) T / F / F / T / F
(C) T / T / T / F / T
(D) F / F / F / T / F
(E) F / T / F / T / T
Comentários: A sentença I está correta. O texto diz que foi a primeira vez em 45 anos que uma
espaçonave tripulada pousou na água.
A sentença II está incorreta. O texto afirma que os paraquedas reduziram a velocidade da
espaçonave de 560km/h para 15mph, e não 15km/h.
A sentença III está incorreta. O texto diz que a Boeing enfrentou problemas no software do seu
projeto de cápsula. Portanto, não podemos afirmar que a empresa tem sua cápsula tripulada
pronta para a próxima missão da Nasa.
A sentença IV está correta. O texto é bem explícito ao dizer que o administrador da Nasa, ao
louvar os esforços de todos envolvidos na missão que acabara de terminar, aproveitou para falar
sobre a mudança de filosofia da agência.
A sentença V está incorreta. O texto não diz que a Nasa vai avaliar a possibilidade de contratar
empresas privadas novamente. O texto diz que essa é uma nova política já estabelecida na
agência.
GABARITO: B

QUESTÃO 03 (EFOMM/INÉDITA) – In the excerpt “Technicians monitored "remnant vapours"


around the spacecraft before the hatch was opened.”, the word in bold means:
(A) Residual
(B) Wasting
(C) Debris
(D) Excess
(E) Near
Comentários: A alternativa A está correta. “Residual” significa residual, restante. “Remnant”
significa residual, restante.

AULA 02 – VERBS IN TEXTS 119


TEACHER ANDREA BELO

A alternativa B está incorreta. “Wasting” significa desperdiçado. “Remnant” significa residual,


restante.
A alternativa C está incorreta. “Debris” significa detritos, destroços. “Remnant” significa residual,
restante.
A alternativa D está incorreta. “Excess” significa excesso, excedente. “Remnant” significa residual,
restante.
A alternativa E está incorreta. “Near” significa perto, próximo. “Remnant” significa residual,
restante.
GABARITO: A

QUESTÃO 04 (EFOMM/INÉDITA) – According to the text, President Donald Trump


(A) attended the capsule's launch in April.
(B) didn’t like the fact that the mission was a success.
(C) celebrated the end of this successful mission.
(D) went to Pensacola, Florida to welcome the crew back to Earth.
(E) wanted the mission to last more time.
Comentários: A alternativa A está incorreta. O texto não diz que o lançamento foi em abril, mas
sim no fim de maio.
A alternativa B está incorreta. O texto afirma que o presidente usou o Twitter para celebrar o
sucesso da missão. Portanto, não se pode dizer que ele não gostou do fato de a missão ter sido
um sucesso.
A alternativa C está correta. O texto afirma que o presidente usou o Twitter para celebrar o sucesso
da missão, exatamente como diz a alternativa.
A alternativa D está incorreta. O texto não fala nada sobre o presidente ter ido até a Florida para
recepcionar os astronautas.
A alternativa E está incorreta. De acordo com o texto, o presidente não dá nenhuma opinião
quanto à duração da missão. Portanto, não podemos afirmar que ele queria que a missão tivesse
durado mais tempo.
GABARITO: C

QUESTÃO 05 (EFOMM/INÉDITA) – Choose the correct option to complete the paragraph below.
Saving the giant panda is one of the big success stories of conservation.
Decades of efforts to create protected habitat for the iconic mammal has pulled it back from the
brink of extinction. But, according to a new study, while many other animals ______ the same
landscape have benefited from this conservation work, some have lost out. Leopards, snow
leopards, wolves and Asian wild dogs have almost disappeared from the majority of protected

AULA 02 – VERBS IN TEXTS 120


TEACHER ANDREA BELO

areas. Driven _____ near extinction ______ logging, poaching and disease, their loss could lead to
"major shifts, even collapse, in ecosystems", said researchers in China.
Adapted from https://www.msn.com/en-us/health/other/if-people-are-staying-home-why-is-coronavirus-still-spreading/ar-BB13HSW8?ocid=bingcovid

(A) in / on / by
(B) on / in / at
(C) in / to / by
(D) at / on / in
(E) in / in / on
Comentários: A primeira lacuna deve ser preenchida por “in”. A preposição “in” é usada porque
o texto está falando sobre animais que estão dentro do mesmo ambiente (in the same landscape).
A segunda lacuna deve ser preenchida por “to”. A preposição “to” precisa ser usada nesse
contexto porque o verbo no particípio “driven” pede essa preposição. É uma questão de regência.
A terceira lacuna deve ser preenchida pela preposição “by”. Essa preposição é ideal aqui porque
ela expressa a causa, equivalendo à preposição “por” em português.
GABARITO: C

QUESTÃO 06 (EFOMM/INÉDITA) – Choose the best alternative to complete the excerpt below.
Finding _____ effective coronavirus vaccine ______ a global priority in ending the pandemic. US
government leaders have put forward the ambitious timeline ______ have one by the end of
2020. It typically takes several years to develop ______ vaccine.
https://www.bbc.com/news/science-environment-53616593

(A) an / has become / to / a


(B) a / has become / to / a
(C) an / became / to / the
(D) an / has become / of / the
(E) a / became / of / a
Comentários: A primeira lacuna deve ser preenchida por um artigo, como sugerem as
alternativas. Nesse caso, o artigo correto a completar a lacuna é “an”, pois “effective” inicia com
uma vogal. Isso já elimina as alternativas B e E.
A segunda lacuna deve ser preenchida por “has become”. O present perfect é necessário por se
tratar de uma situação que se inicia no passado e ocorre até o presente. Assim, continuamos com
as alternativas A e D como possíveis respostas.
A terceira lacuna deve ser preenchida pela preposição “to”. A outra opção de preenchimento
apresentada pela questão é a preposição “of”, mas para usarmos “of” nessa situação, o verbo
“have” deveria estar no gerúndio (having). Dessa forma, já temos a alternativa A como resposta,
mas vamos analisar a última lacuna para nos certificar.

AULA 02 – VERBS IN TEXTS 121


TEACHER ANDREA BELO

A quarta lacuna pode ser preenchida pelo artigo indefinido “a”. A frase se inicia deixando claro
que não se está falando de nenhuma vacina específica, mas sim de vacinas de forma geral. Dessa
forma, não se pode usar o artigo definido para falar de vacinas de forma geral.
GABARITO: A

QUESTÃO 07 (EFOMM/INÉDITA) – Choose the correct alternative to complete the paragraph


below.
A holistic view of ecosystem management __________ "critically needed to better increase the
resilience and sustainability of the ecosystems not only for giant pandas but also for other wild
species", said Dr Sheng Li of Peking University in Beijing. Giant pandas are __________ as living
proof that conservation works. Their numbers in the wild are finally rebounding after years of
decline, and in 2016 they __________ upgraded from "endangered" to "vulnerable" on the official
extinction Red List.
Adapted from https://www.bbc.com/news/science-environment-53616593

(A) is / saw / have been


(B) are / see / are
(C) was / seen / were
(D) is / seen / were
(E) was / saw / have been
Comentários: A primeira lacuna deve ser preenchida por “was”, pois o texto é escrito no reported
speech, o que exige que o texto seja escrito no tempo passado.
A segunda lacuna deve ser preenchida por “seen” porque a ideia é dizer que os pandas são vistos.
Portanto, precisamos usar o particípio do verbo “see” para satisfazer a exigência da lacuna.
A terceira lacuna deve ser preenchida por “were”, pois deixa claro que isso ocorreu em 2016, o
que indica que nós temos a exata noção de quando o fato ocorreu, e essa é uma situação de uso
do past simple.
GABARITO: C

QUESTÃO 08 (EFOMM/INÉDITA) – Which option is correct?


(A) She gave me two precious advices.
(B) Poaching is an essential activity in natural reserves.
(C) Brazil have hosted the olimpics in 2016.
(D) I am used to running 5 kilometers a day.
(E) There was a search to determine Brazil’s population in 2010.

AULA 02 – VERBS IN TEXTS 122


TEACHER ANDREA BELO

Comentários: A alternativa A está incorreta. Não é correto dizer “two precious advices”, seria
correto dizer “two precious pieces of advice”, pois advice é um substantivo incontável e que,
portanto, não varia em número, ou seja, não tem plural.
A alternativa B está incorreta. A palavra “Poaching” significa caça furtiva, e não faz sentido que
uma atividade de caça seja essencial em reservas naturais, onde a caça é, via de regra, proibida
completamente por serem áreas de conservação ambiental.
A alternativa C está incorreta. O erro desta alternativa está no uso do verbo “have hosted” que
está conjugado de forma incorreta. A forma correta de escrever seria “hosted”, pois trata-se de
uma situação já encerrada no passado e que sabemos exatamente quando ocorreu. Por isso, o
past simple seria o tempo verbal adequado, e não o present perfect.
A alternativa D está correta. A expressão “i am used to” significa que se está acostumado a fazer
algo e, no inglês, essa estrutura exige que o verbo subsequente seja escrito no gerúndio,
exatamente como aparece na alternativa.
A alternativa E está incorreta. Não é correto usar “search” para falar de pesquisa. Portanto, seria
correto escrever “research”.
GABARITO: D

QUESTÃO 09 (EFOMM/INÉDITA) – Mark the option which corresponds to the correct sentences.
I – The person who defends a person in court is called witness.
II – The pregnant woman was in labor because her water broke.
III – Mooring is an important process when the ship is sailing.
IV – That child was birthed in July.
V – The pregnant woman was taken to the delivery room in order to give birth.
(A) I and II
(B) II and V
(C) III and V
(D) II and IV
(E) III and IV
Comentários: A sentença I está incorreta. A pessoa que defende outra num tribunal é chamada
de advogado, e não testemunha. Em inglês, usamos a palavra “lawyer” e não a palavra “witness.
A sentença II está correta. A mulher grávida estava em trabalho de parto porque sua bolsa
estourou.
A sentença III está incorreta. A palavra “mooring” significa amarração. A amarração é o ato de
“amarrar”, ou prender o navio ao porto quando ele está atracando. Portanto, amarração é um
processo importante na atracação do navio, e não quando ele está navegando, pois quando está
navegando, não é necessário amarrar o navio a lugar algum.

AULA 02 – VERBS IN TEXTS 123


TEACHER ANDREA BELO

A sentença IV está incorreta. O erro da alternativa está no uso da palavra “birthed”. A palavra
certa a ser utilizada nessa situação seria “born”.
A sentença V está correta. A mulher grávida foi levada à sala de parto para dar à luz.
GABARITO: B

QUESTÃO 10 (EFOMM/INÉDITA) – Which option is correct to complete the sentences below?


1 – He heard her __________ the dinner.
2 – He saw her __________ the cake.
3 – I watched him __________ the guitar.
4 – In order to get thin, you need to __________ hard.
5 – He couldn’t see what __________ on in his neighborhood yesterday.
(A) make / bake / played / working out / were going
(B) made / baking / play / work out / has been going
(C) making / bake / playing / working out / had been going
(D) making / baking / playing / work out / was going
(E) make / baked / play / worked out / is going
Comentários: A sentença 1 deve ser preenchida por “making”. Ele a ouviu fazendo o jantar. Ele a
ouviu enquanto ele preparava, por isso precisamos usar o verbo no gerúndio “making”.
A sentença 2 deve ser preenchida por “baking”. Não é possível usar “bake” nesse contexto, pois
assar um bolo não é algo instantâneo, mas sim um processo, pois a pessoa precisa colocar o bolo
no forno, esperar o tempo correto e depois retirar do forno. Portanto precisamos usar o gerúndio
para demonstrar que a pessoa assistiu a todo o processo.
A sentença 3 deve ser preenchida por “playing”. A frase dá a entender que a pessoa assistia
enquanto o outro tocava violão, e isso é um processo em andamento. Por isso, devemos usar o
gerúndio nesse caso.
A sentença 4 deve ser preenchida por “work out”. Para emagracer, você precisa se exercitar
muito. O uso do verbo “work out” no presente é necessário por se tratar de um fato atemporal.
A sentença 5 deve ser preenchida por “was going”. O uso da expressão de tempo “yesterday”
indica que a lacuna deve ser preenchida com o tempo verbal past continuous.
GABARITO: D

QUESTÕES EPCAR
Directions: Answer questions 01 to 10 according to TEXT I.
The WHO has redefined burnout as a syndrome linked to chronic work stress. There’s a
difference between a busy workload and something more serious, writes Zaria Gorvett.

AULA 02 – VERBS IN TEXTS 124


TEACHER ANDREA BELO

If you said you were suffering from ‘burnout’ in the early 1970s, you might have raised some
eyebrows.
At the time, the term was used informally to describe the side effects that heavy drug users
experienced: the general dimming of the mental faculties, for example, as was the case with many
a party animal. However, when German-American psychologist Herbert Freudenberger first
recognised the problem of burnout in New York City in 1974, at a clinic for addicts and homeless
people, Freudenberger wasn’t thinking of drug users.
The clinic’s volunteers were actually struggling, too: their work was intense, and many were
beginning to feel demotivated and emotionally drained. Though they had once found their jobs
rewarding, they had become cynical and depressed; they weren’t giving their patients the
attention they deserved. Freudenberger defined this alarming new condition as a state of
exhaustion caused by prolonged overwork – and borrowed the term ‘burnout’ to describe it.
Its popularity was explosive, and today burnout is a global phenomenon. Although statistics on
the prevalence of burnout specifically are hard to come by, 595,000 people in the UK alone
suffered from workplace stress in 2018.
Sportspeople get it. YouTube stars get it. Entrepreneurs get it. Freudenberger himself eventually
got it. Late last month, the World Health Organization (WHO) announced that the trendy problem
will be recognised in the latest International Classification of Diseases manual, where it is
described as a syndrome “resulting from chronic workplace stress that has not been successfully
managed”.
According to the WHO, burnout has three elements: feelings of exhaustion, mental detachment
from one’s job and poorer performance at work. But waiting until you’re already fully burned out
to do something about it doesn’t help at all –and you wouldn’t wait to treat any other illness until
it was too late.
Feeling the burn
So how can you tell if you’re almost – but not quite – burned out?
“A lot of the signs and symptoms of pre-burnout would be very similar to depression,” says
Siobhán Murray, a psychotherapist based in County Dublin, Ireland, and the author of a book
about burnout, The Burnout Solution. Murray suggests looking out for creeping bad habits, such
as increased alcohol consumpution and relying on sugar to get you through the day. Also watch
out for feelings of tiredness that won’t go away. “So that even if you do sleep well, by 10 in the
morning you’re already counting down the hours to bed. Or not having the energy to exercise or
go for a walk.”
As soon as you begin to feel this way, Murray advises going to see your doctor.
“Depression and pre-burnout are very similar, but as much as there was a lot of enthusiasm
recently that burnout has now become a medical condition, it is still not – it is still classified as an
occupational phenomenon.” It’s important to get help from a medical professional who can
distinguish between the two, because although there are many treatment options for depression,
burnout is still best tackled by making lifestyle changes.
And how do you know if you’re really on the cusp of burnout, or just going through a challenging
month? “Stress is really important, and anxiety is what motivates us to do well,” says Murray. “It’s

AULA 02 – VERBS IN TEXTS 125


TEACHER ANDREA BELO

when we’re continually exposed to stress and anxiety, that we’re not letting go, that it starts to
turn into burnout.”
Take that big project you’ve been working on. It’s normal to feel a kick of adrenaline when you
think about it, and maybe it’s kept you up at night. But, Murray suggests, if you still feel restless
once it’s over, it’s time to consider if you’re at risk of burnout. “It’s when you’re bringing that with
you into the next stage of your day, and adding to it continually,” she says.
Another classic sign of inching closer to burnout is cynicism: feeling like your work has little value,
avoiding social commitments and becoming more susceptible to disappointment.
“Someone on the brink will probably begin to feel emotionally numbed or mentally distant,” says
Jacky Francis Walker, a psychotherapist based in London who specializes in burnout. “Like they
don’t have the capacity to engage as much in the ordinary things of life.”
She also recommends looking for the final tell-tale sign of burnout, which is the unshakeable
feeling that the quality of your work is beginning to slip. “People say ‘but this isn’t me!’, ‘I’m not
like this’, ‘I can usually do x,y and z’. But obviously if they are in a state of physical depletion, then
they aren’t in their normal range of capabilities,” says Walker.
If this seems less than scientific, look to the Maslach Burnout Inventory (MBI), a test designed to
measure burnout. The most widely used is the MBI-General Survey, which measures things like
exhaustion, cynicism, and some how well you think you’re doing at work.
First published in 1981, it has been cited hundreds of times in studies since. Although it’s typically
used to measure burnout once it's in full swing, there’s no reason you can't apply it to see if you’re
getting close.
(Adapted from https://www.bbc.com/worklife/article/20190610-how-to-tell-if-youve-got-pre-burnout)

QUESTÃO 01 (EPCAR/INÉDITA) – Mark the correct option.


a) Burnout is now considered a medical condition.
b) Burnout can improve people’s performance at work.
c) Burnout was widely known in the 1970’s.
d) Herbert Freudenberger first used the term burnout in the 1970’s.
Comentários: A alternativa A está incorreta. O texto afirma que o burnout ainda não é
considerado uma doença, mas sim um fenômeno ocupacional.
A alternativa B está incorreta. O texto afirma que o burnout gera uma queda de rendimento no
trabalho, e não um aumento de performance como diz a alternativa.
A alternativa C está incorreta. O texto afirma que se você dissesse que estava sofrendo de burnout
na década de 70, as pessoas levantariam as sobrancelhas, pois na época, esse termo era usado
para descrever o efeito colateral que usuários de drogas pesadas experimentavam. Portanto, não
podemos dizer que o fenômeno era amplamente conhecido na época.
A alternativa D está correta. O texto afirma que Herbert Freudenberger pegou “emprestado” a
palavra burnout para descrever o que acontecia com funcionários que trabalhavam sujeitos a
altos níveis de stress e fadiga acumulados. E o texto afirma que isso ocorreu em 1974.
GABARITO: D

AULA 02 – VERBS IN TEXTS 126


TEACHER ANDREA BELO

QUESTÃO 02 (EPCAR/INÉDITA) – According to the text, mark the correct alternative.


a) The psychologist discovered burnout by accident while analyzing drug addicts and homeless.
b) The clinic’s volunteers were having a good time doing their work.
c) Anyone is subject to get it if they reach exhaustion due to prolonged overwork and stress.
d) Drug addicts were the first to suffer from burnout due to their stressful routine.
Comentários: A alternativa A está incorreta. O texto não nos permite entender que a descoberta
do psicólogo foi acidental, mas sim proposital, pois ele não estava observando os pacientes da
clínica, mas os voluntários.
A alternativa B está incorreta. O texto diz que os voluntários estavam desmotivados e passando
por dificuldades no trabalho, e não que eles estavam se divertindo ou felizes.
A alternativa C está correta. O texto cita diversos exemplos de pessoas que foram afetadas pelo
burnout, nos dando a entender que toda e qualquer pessoa está sujeita ao burnout se chegarem
à exaustão por conta de stress e excesso de trabalho por períodos prolongados.
A alternativa D está incorreta. O texto diz que foram os voluntários da clínica que foram
observados pelo psicólogo, e não os pacientes. Foi baseado na observação dos voluntários que
ele chegou à definição de burnout.
GABARITO: C

QUESTÃO 03 (EPCAR/INÉDITA) – In the sentence “the general dimming of the mental faculties”
(paragraph 2) the word dimming means
a) decrease.
b) augmentation.
c) assessment.
d) improvement.
Comentários: A alternativa A está correta. “Decrease” significa diminuição, e “dimmimg” tem
significado muito parecido, sendo usado para dar a ideia de que algo está diminuindo ou
perdendo intensidade.
A alternativa B está incorreta. “Augmentation” indica aumento, e a palavra “dimming” não tem
nenhuma relação de significado, pois “dimming” significa diminuir ou perder intensidade.
A alternativa C está incorreta. “Assessment” indica avaliação, e a palavra “dimming” não tem
nenhuma relação de significado, pois “dimming” significa diminuir ou perder intensidade.
A alternativa D está incorreta. “Improvement” significa melhora, e não guarda nenhuma relação
com a palavra “dimming”, que significa diminuir ou perder a intensidade.
GABARITO: A

AULA 02 – VERBS IN TEXTS 127


TEACHER ANDREA BELO

QUESTÃO 04 (EPCAR/INÉDITA) – Mark the correct question to the sentence below.


“Freudenberger defined this alarming new condition as a state of exhaustion caused by
prolonged overwork”
a) How did Freudenberger define this alarming new condition?
b) How has Freudenberger defined this alarming new condition?
c) How had Freudenberger defined this alarming new condition?
d) How did Freudenberger defined this alarming new condition?
Comentários: A alternativa A está correta. O trecho do enunciado está conjugado no past simple,
portanto, a pergunta precisa ser feita nesse mesmo tempo verbal.
A alternativa B está incorreta. O uso do present perfect é o erro da alternativa, pois a afirmativa
do enunciado está no past simple e, por isso, deveria ser usado “did” ao invés de “has defined”.
A alternativa C está incorreta. O uso do past perfect é o erro da alternativa, pois a afirmativa do
enunciado está no past simple e, por isso, deveria ser usado “did” ao invés de “had defined”.
A alternativa D está incorreta. O erro da alternativa está no fato de o verbo “defined” estar
conjugado no passado, pois a presença do auxiliar “did” elimina a necessidade de conjugar o verbo
para configurar o tempo verbal.
GABARITO: A

QUESTÃO 05 (EPCAR/INÉDITA) – Mark the alternative that completes the sentence.


Some symptoms the psychologist noticed the volunteers had to deal with were
a) that they were drug abusers and most of them were homeless.
b) that they started to feel demotivated, emotionally drained, cynical and depressed.
c) that they were getting sick very frequently.
d) that they were being diagnosed with generalized anxiety and depression.
Comentários: A alternativa A está incorreta. Não eram os voluntários que eram viciados em
drogas nem mendigos, mas sim os pacientes.
A alternativa B está correta. Segundo o texto, os voluntários começaram a se sentir desmotivados
e emocionalmente esgotados. Além disso, embora eles achassem seus trabalhos
recompensadores no passado, acabaram se tornando cínicos e depressivos.
A alternativa C está incorreta. O texto não diz, em momento algum, que os voluntários estavam
ficando doentes com frequência. Os sintomas eram mais psicológicos do que físicos.
A alternativa D está incorreta. O texto não afirma que os voluntários eram diagnosticados com
ansiedade generalizada e depressão. O texto diz que os voluntários começaram a ficar
deprimidos, mas isso é colocado de forma comportamental, e não clínica.
GABARITO: B

AULA 02 – VERBS IN TEXTS 128


TEACHER ANDREA BELO

QUESTÃO 06 (EPCAR/INÉDITA) – Mark the alternative that DOESN’T complete the sentence
below.
The psychologist Herbert Freudenberger
a) was who first used the word burnout to describe this exhaustion condition.
b) recognised the problem of burnout in 1974 in New York City for the first time.
c) created this condition so that he would become known for discovering a disease.
d) defined burnout as a state of exhaustion caused by prolonged overwork.
Comentários: A alternativa A está incorreta. A alternativa completa a sentença de forma correta,
já que o texto diz que o psicólogo foi quem primeiro usou a palavra burnout para descrever a
condição de exaustão descrita pelo texto.
A alternativa B está incorreta. Esta alternativa também completa de forma correta a sentença,
pois o texto afirma que ele reconheceu o problema do burnout pela primeira vez em Nova Iorque
no ano de 1974.
A alternativa C está correta. Ela não completa de forma correta pois diz que o psicólogo criou a
condição para que pudesse ficar conhecido por ter descoberto uma doença. O problema é que
ele não criou nada, a condição já existia, ele apenas observou e deu um nome a ela.
A alternativa D está incorreta. Esta alternativa diz exatamente a mesma coisa que é dita no texto.
O psicólogo definiu burnout como um estado de exaustão causado por excesso de trabalho
prolongado.
GABARITO: C

QUESTÃO 07 (EPCAR/INÉDITA) – Read the sentences and mark the correct option.
I. The WHO has finally recognised burnout as a disease.
II. More than half a million people suffered from workplace stress in 2018 in the UK.
III. Relying on sugar to get you through the day can be a good way of keeping yourself from
burning out.
The only correct sentence(s) is(are)
a) II.
b) III.
c) I and II.
d) I and III.
Comentários: A sentença I está incorreta. O texto afirma que a organização mundial da saúde não
reconheceu o burnout como uma doença, mas sim como um fenômeno ocupacional.
A sentença II está correta. O texto afirma que mais 595.000 pessoas sofreram com stress no local
de trabalho somente no Reino Unido no ano de 2018, ou seja, mais de meio milhão de pessoas.

AULA 02 – VERBS IN TEXTS 129


TEACHER ANDREA BELO

A sentença III está incorreta. O texto diz que usar açúcar para conseguir passar o dia é um dos
maus hábitos que as pessoas devem estar atentas na hora de perceber se estão se aproximando
de um burnout.
GABARITO: A

QUESTÃO 08 (EPCAR/INÉDITA) – Mark the correct alternative to complete the sentence.


According to the text, the burnout
a) makes people happy because they feel that they are performing at their best.
b) is best tackled and can be reversed by making lifestyle changes.
c) has been responsible for many deaths around the world since the 1970’s.
d) is responsible for kicks of adrenaline when you think about a project you’ve been working on.
Comentários: A alternativa A está incorreta. O texto não afirma, em momento algum, que as
pessoas estão felizes porque elas sentem que estão desempenhando sua melhor performance.
Na verdade, o texto afirma que o desempenho das pessoas piora com o burnout.
A alternativa B está correta. O texto diz que o burnout é abordado da melhor forma quando há
mudanças no estilo de vida da pessoa. Essa é a melhor forma de lidar com problema conhecida
até o momento.
A alternativa C está incorreta. O texto não fala em mortes causadas pelo problema do burnout.
A alternativa D está incorreta. O texto não diz que o burnout é responsável por picos de
adrenalina, o texto diz que é normal se sentir assim quando se está envolvido em um projeto
grande, e que só é preocupante caso os sintomas não sumam após a conclusão desse projeto.
GABARITO: B

QUESTÃO 09 (EPCAR/INÉDITA) – In the underlined sentences in the second paragraph, there


are __________ verbs used in the past tense.
a) four
b) one
c) three
d) two
Comentários: Ao observar o trecho sublinhado, percebemos o verbo “recognised” no past simple,
também podemos observar o verbo “wasn’t” que também está no past simple. Podemos dizer,
então, que há dois verbos sendo usados no tempo passado no trecho sublinhado do segundo
parágrafo.
GABARITO: D

AULA 02 – VERBS IN TEXTS 130


TEACHER ANDREA BELO

QUESTÃO 10 (EPCAR/INÉDITA) – Volunteers


a) never liked their work at all.
b) weren’t giving enough attention to patients due to their burnout condition.
c) accepted being part of an experiment to find out what happens when people get to their limit
at work.
d) would still find their jobs rewarding even though they were burned out.
Comentários: A alternativa A está incorreta. O texto afirma que os voluntários achavam seus
trabalhos recompensadores antes de sofrer com o burnout.
A alternativa B está correta. O texto diz que os deixaram de dar a devida atenção aos pacientes
por conta de sua condição de burnout.
A alternativa C está incorreta. O texto não fala nada sobre experimento, nem afirma que os
voluntários tinham ciência de que estavam sendo avaliados pelo psicólogo.
A alternativa D está incorreta. O texto diz que os voluntários deixaram de ter a mesma impressão
sobre o próprio trabalho uma vez que foram acometidos pelo burnout e nos permite entender
que deixaram de ver seus trabalhos como recompensadores desde então.
GABARITO: B

QUESTÕES ESA
Texto para responder à questão 01
August 08, 2020
By Lise Alves, Senior Contributing Reporter
RIO DE JANEIRO, BRAZIL - In two months, researchers from the Alberto Luiz Coimbra Institute of
Post-Graduate Studies and Engineering Research at the Federal University of Rio de Janeiro
(Coppe/UFRJ), the National Institute of Metrology, Quality and Technology (INMETRO) and the
Pontifical Catholic University of Rio de Janeiro (PUC Rio) will introduce an antiviral mask fabric
that provides greater protection against Covid-19 to healthcare professionals. The announcement
was made yesterday, August 7th, by Professor Renata Simão from the Nanotechnology
Engineering and Metallurgical and Materials Engineering Programs of Coppe.
(Adapted from https://riotimesonline.com)

QUESTÃO 01 (ESA/INÉDITA) – According to the text, it is correct to say that:


A) many researchers will introduce a new protective mask.
B) INMETRO is one of the responsible institution for the protective mask.
C) Coope/UFRJ is the responsible institution for the protective mask.
D) INMETRO has researched about the protective mask.
E) some researchers will introduce a new protective mask.

AULA 02 – VERBS IN TEXTS 131


TEACHER ANDREA BELO

Comentários: A alternativa A está incorreta. O texto afirma que pesquisadores (researchers)


apresentarão uma nova máscara de proteção mas não afirma que são muitos ou a quantidade.
A alternativa B está incorreta, dizendo que o Inmetro é um dos responsáveis pela máscara, como
vimos acima, são pesquisadores de diferentes órgãos.
A alternativa C está incorreta, dizendo que o Coope/UFRJ é responsável pela máscara, como vimos
acima, são pesquisadores de diferentes órgãos.
A alternativa D está incorreta. O Inmetro não está pesquisando sobre as máscaras e sim pessoas
responsáveis (researchers).
A alternativa E está correta, dizendo que alguns pesquisadores (researchers) apresentarão uma
nova máscara de proteção, pois não se afirma quantidade mas a palavra “some” é mais de um.
GABARITO: E

QUESTÃO 02 (ESA/INÉDITA) – Which sentence is grammatically correct?


(A) Yesterday, I went to bed more earlier than the night before.
(B) Fred is more thin than his best friend.
(C) My book is more big than my sister’s dictionary.
(D) Going by car is expensiver than by bus because of the gas.
(E) Elizabeth’s shoes are cheaper than mine.
Comentários: A alternativa A está incorreta. Não se utiliza “more” junto com “earlier”, pois
“earlier” já é uma estrutura de frase comparativa, usando -ier no final do adjetivo considerado
curto. Utiliza-se apenas “earlier”, pois essa palavra, apenas, já oferece o sentido comparativo que
a sentença busca. O uso do “more” é para adjetivos longos, como “more beautiful/more
intelligent etc”.
A alternativa B está incorreta. O modo comparativo de “THIN” é “thinner”. Só se usa “more” antes
de adjetivo quando este adjetivo tiver mais de uma sílaba, ou seja, quando for um adjetivo
considerado longo.
A alternativa C está incorreta. A mesma explicação da alternativa anterior se aplica no caso de
“fast” o correto seria “faster” ou apenas “fast”, mas não “more fast”.
A alternativa D está incorreta. A palavra “expensive” é uma palavra de, pelo menos, três sílabas.
Portanto, é correto dizer “more expensive”, segundo a regra de palavras curtas ou longas e não
podemos colocar -er no fim, formando “expensiver”, que não existe.
A alternativa E está correta. “Cheaper” é a forma correta de fazer o comparativo de “cheap”.
GABARITO: E

AULA 02 – VERBS IN TEXTS 132


TEACHER ANDREA BELO

QUESTÃO 03 (ESA/INÉDITA) – Complete the sentence below using the appropriate words:
“Mr. Tompson __________ traveling by car: He is afraid of planes and __________ like fast trips,
but ___________ driving.”
(A) Likes/don’t/love
(B) Likes/doesn’t/loves
(C) Like/do/loves
(D) Like/don’t/love
(E) Like/does/love
Comentários: Essa é uma questão que exige conhecimento dos tempos verbais e suas
conjugações no Present Simple (presente simples) bem como auxiliares (do/does) e as formas
afirmativa e negativa.
O sujeito “Mr. Tompson” pode ser substituído pelo pronome “he” (ele). Sempre que se conjuga
verbo na terceira pessoa do singular (he/she/it), acrescenta-se “S” ao final do verbo no tempo
Present Simple, como é o caso da frase em destaque em nosso enunciado.
A segunda lacuna deve ser preenchida com o auxiliar “doesn’t”, pois o sujeito está na terceira
pessoa do singular e exige “doesn’t” (does not), segundo a regra dos verbos na terceira pessoa,
modo negativo, ao invés de “don’t”, para sujeitos I, you e todos os demais que se encontram no
plural.
A terceira lacuna deve ser preenchida por “loves”, pois quem ama os trens é ele (he). Portanto,
temos mais uma vez o sujeito na terceira pessoa do singular, que exige o acréscimo da letra “S”
no final do verbo no tempo presente.
GABARITO: B

QUESTÃO 04 (ESA/INÉDITA) – “__________ my best friend for real?”


Complete the space with the correct form of the verb and the pronoun.
(A) You is
(B) You are
(C) Are you
(D) Is you
(E) Am you
Comentários: Essa é uma questão que exige conhecimento das conjugações do verbo to be no
Present Simple (presente simples) bem como as formas afirmativa e interrogativa. Estão em nossa
aula 02, em PDF e videoaulas do Estratégia Militares, da teacher Andrea Belo.
A alternativa A está incorreta. “You” não pode ser seguido de “is”, devido à sua conjugação,
segundo qualquer gramática e a forma do verbo to be no plural é “are” (You are). O verbo deve
estar antes do pronome “you”, pois trata-se de uma pergunta.

AULA 02 – VERBS IN TEXTS 133


TEACHER ANDREA BELO

A alternativa B está incorreta. A ordem de pronome (you) e verbo (are) deveria estar invertida
por ser de uma pergunta, em que o verbo to be passa a ser o primeiro da estrutura da frase.
A alternativa C está correta. Numa frase interrogativa, deve-se inverter a ordem de pronome
sujeito (you) e verbo to be (are). Portanto, “Are you American?” é gramaticalmente correto.
A alternativa D está incorreta. Para o sujeito “You” não se usa “is”, como vimos anteriormente e
sempre “are”.
A alternativa E está incorreta. “You” não é acompanhado de “am”, sempre de “are”.
GABARITO: C

Texto para responder às questões 05 e 06


August 08, 2020
RIO DE JANEIRO, BRAZIL - On Saturday, August 8th, Brazil reached the number of 100,477 deaths
from Covid-19 since the start of the novel coronavirus pandemic. According to the Ministry of
Health, 905 deaths were recorded over the last 24 hours.
Of the 3,012,412 cases of people infected by the novel coronavirus, 2,094,293 (69.5 percent) have
recovered.
A total of 49,970 new cases have been reported by Health Secretariats since Friday, August 7th.
The official also shows that the number of people under follow-up stands at 817,642 (21.1
percent).
(Adapted from https://riotimesonline.com)

QUESTÃO 05 (ESA/INÉDITA) – According to the text, it is correct to say that:


A) the number described is more than a hundred thousand deaths.
B) it is said the number is a hundred thousand deaths.
C) the number described is less than a hundred thousand deaths.
D) more than a hundred thousand people are working on the research.
E) the correct number of deaths is a thousand.
Comentários: A alternativa A afirma que o número de mortes é mais de cem mil – correto
(100,477)
A letra B afirma que que o número de mortes é de cem mil – vimos que é mais de cem mil.
A letra C afirma que que o número de mortes é menos de cem mil – errado.
A letra D afirma que mais de cem mil pessoas estão trabalhando na pesquisa – errado.
A letra E afirma que que o número de mortes é mil – incorreto.
GABARITO: A

AULA 02 – VERBS IN TEXTS 134


TEACHER ANDREA BELO

QUESTÃO 06 (ESA/INÉDITA) – The term “quite a few” is the same of


(A) many
(B) much
(C) few
(D) little
(E) several
Comentários: A alternativa A afirma que “quite a few” é muitos/as e está correto. Apesar de
conter a palavra few em sua estrutura, é o mesmo que vários, muitos, inúmeros etc.
A letra B afirma que é muito, incontável.
A letra C afirma que é pouco mas é o contrário.
A letra D afirma que seria pouco/pequeno.
A letra E afirma que significa duas vezes mas vimos que é muitos/as.
GABARITO: A

QUESTÃO 07 (ESA/INÉDITA) – The word “huge” is the same of


(A) very hard
(B) very big
(C) very far
(D) very deep
(E) very cheap
Comentários: A alternativa A afirma que “huge” é muito difícil, mas huge significa enorme.
A letra B afirma que é muito grande, exatamente – enorme.
A letra C afirma que “huge” é muito longe, mas huge significa enorme.
A letra D afirma que “huge” é muito profundo, mas huge significa enorme.
A letra E afirma que “huge” é muito barato, mas huge significa enorme.
GABARITO: B

QUESTÃO 08 (ESA/INÉDITA) – “__________ working at the company?”


Complete the space with the correct form of the verb and the pronoun.
(A) Are the girl
(B) She is
(C) Is she
(D) Are she
(E) Am she

AULA 02 – VERBS IN TEXTS 135


TEACHER ANDREA BELO

Comentários: Essa é uma questão que exige conhecimento das conjugações do verbo to be no
Present Simple (presente simples) bem como as formas afirmativa e interrogativa. Estão em nossa
aula 02, em PDF e videoaulas do Estratégia Militares, da teacher Andrea Belo.
A alternativa A está incorreta. “SHE” (the girl) não pode ser seguido de “are”, devido à sua
conjugação, segundo qualquer gramática e a forma do verbo to be no singular é “IS” (You are). O
verbo deve estar antes dos pronomes “he/she/it”, pois trata-se de uma pergunta.
A alternativa B está incorreta. A ordem de pronome (he/she/it) e verbo (is) deveria estar invertida
por ser de uma pergunta, em que o verbo to be passa a ser o primeiro da estrutura da frase.
A alternativa C está correta. Numa frase interrogativa, deve-se inverter a ordem de pronome
sujeito (she) e verbo to be (is). Portanto, “Is she working at the company?” é gramaticalmente
correto.
A alternativa D está incorreta. Para o sujeito “She” não se usa “are”, como vimos anteriormente
e sempre “is”.
A alternativa E está incorreta. “She” não é acompanhado de “am”, sempre de “is”.
GABARITO: C

QUESTÃO 09 (ESA/INÉDITA) – The word “them” is


(A) a pronoun
(B) a conjunction
(C) a preposition
(D) a noun
(E) an adjective
Comentários: “Them” significa “eles”, no final de frases – “I am going to the party with them”
(Vou à festa com eles) e é o pronome que se refere ao sujeito “they” (eles/elas)
Nas letras B, C, D e E, as funções gramaticais estão incorretas. Não há como “them” ser uma
conjunção ou preposição ou substantivo ou sequer adjetivo, devido à classificação gramatical que
possui.
GABARITO: A

QUESTÃO 10 (ESA/INÉDITA) – The word “useful” is


(A) a verb
(B) a conjunction
(C) a preposition
(D) an adverb
(E) an adjective

AULA 02 – VERBS IN TEXTS 136


TEACHER ANDREA BELO

Comentários: “Useful” significa “útil”, um adjetivo, que, nas frases, devem vir antes dos
substantivos.
Nas letras A, B, C e D, as funções gramaticais estão incorretas. Não há como “useful” ser um verbo,
uma conjunção ou preposição ou um advérbio.
GABARITO: E

QUESTÕES ESCOLA NAVAL


QUESTÃO 01 (ESCOLA NAVAL/INÉDITA) – Which is the correct option to complete the
paragraph below.
What Is Stress?
First, an answer to __________ question, 'What is stress?' __________ term 'stress' refers
__________ the response you have when facing circumstances that force you __________ act,
change, or adjust in some way to maintain your footing or to keep things balanced. (The
circumstances themselves are known as 'stressors', but we'll have more on them later.)
(Adapted from https://www.verywellmind.com/chronic-stress-definition-management-tips-3145241)

(A) the / the / to / to


(B) a / a / at / -
(C) the / a / to / to
(D) the / the / to / -
(E) a / the / at / -
Comentários: A primeira lacuna deve ser preenchida por “the”. O texto está falando de uma
pergunta específica, que é: “O que é estresse?”. Portanto, o artigo definido é o mais adequado
nesse caso.
A segunda lacuna deve ser preenchida por “the”. O artigo definido é necessário por estarmos
falando de um termo específico, e não de um termo qualquer.
A terceira lacuna deve ser preenchida por “to”. Temos uma lacuna que trata da regência do verbo
“refer”. “Who refers, refers to...”. Por isso, a necessidade do uso da preposição “to”.
A quarta lacuna deve ser preenchida por “to”. Temos uma lacuna que trata da regência do verbo
“force”. “Who force, force someone/something to...”. Por isso, a necessidade do uso da
preposição “to”...
GABARITO: A

AULA 02 – VERBS IN TEXTS 137


TEACHER ANDREA BELO

QUESTÃO 02 (ESCOLA NAVAL/INÉDITA) – Which option completes the dialogue below


correctly?
John: What's the matter?
Mary: I haven’t done my exercise yet. I must _______ today.
(A) have done it
(B) have it done
(C) done it
(D) has done it
(E) has it done
Comentários: A Alternativa A está incorreta. “I must have done it” não é usado nessa situação,
pois o contexto nos mostra que ele ainda não fez exercício, mas precisa fazer ainda hoje. Essa
alternativa sugere que ele já fez exercício, o que contradiria a frase imediatamente anterior a
essa.
A alternativa B está correta. Essa é a forma de se expressar que algo precisa ser feito com prazo
estipulado. O prazo estipulado é today, e isso quer dizer que ele precisa estar com seu exercício
feito ainda hoje. Traduzindo, temos: Eu tenho que ter isso feito hoje.
A alternativa C está incorreta. O erro dessa alternativa é usar o verbo no particípio. Assim, seria
como dizer: Eu tenho que feito isso hoje. Não faria sentido, não é verdade?
A alternativa D está incorreta. O erro dessa alternativa é usar o auxiliar “has” com o verbo modal
“must”. Esse modal só aceita o auxiliar “have” no passado. Além desse erro, ainda temos o fato
de que essa alternativa sugeriria que ele já fez exercício, o que contradiria a frase imediatamente
anterior a essa.
A alternativa E está incorreta. O erro dessa alternativa é usar o auxiliar “has” com o verbo modal
“must”. Esse modal só aceita o auxiliar “have” no passado.
GABARITO: B

QUESTÃO 03 (ESCOLA NAVAL/INÉDITA) – Which option completes the paragraph below


correctly?
While quick stress relievers aren't a complete stress management plan in themselves, they are an
excellent first line of defense against the effects of chronic stress. This is because they can help
you turn ______ your body's stress response and respond ______ the stressors you face from a
calm (or calmer), more relaxed place. This helps you to more effectively deal ______ what is
stress, and keep yourself healthier ______ the same time. Common strategies for quick stress
relief include breathing exercises or the use of humor.
(Adapted from https://www.verywellmind.com/chronic-stress-definition-management-tips-3145241)

AULA 02 – VERBS IN TEXTS 138


TEACHER ANDREA BELO

(A) of / to / with / at
(B) on/ - / to / in
(C) off / to / with / at
(D) off/ to / to / in
(E) on / - / with / on
Comentários: A primeira lacuna deve ser preenchida por “off”. Trata-se de uma expressão fixa,
um phrasal verb. O próprio contexto nos ajuda a perceber que a ajuda consiste em desligar a
resposta do corpo ao estresse.
A segunda lacuna deve ser preenchida por “to”. A palavra “respond” exige que usemos a
preposição “to” por uma questão de regência do verbo.
A terceira lacuna deve ser preenchida por “with”. A palavra “deal” exige que usemos a preposição
“with” por uma questão de regência do verbo.
A quarta lacuna deve ser preenchida por “at”. Trata-se de uma expressão fixa. “At the same time”
significa ao mesmo tempo.
GABARITO: C

QUESTÃO 04 (ESCOLA NAVAL/INÉDITA) – Which word best completes the question below?
How ______ does he go when he runs?
It’s really important to exercise in order to keep healthy.
(A) often
(B) old
(C) high
(D) far
(E) many
Comentários: A alternativa A está incorreta. “How often” é uma estrutura usada para perguntar
sobre frequência, e não sobre distância percorrida.
A alternativa B está incorreta. “How old” é uma estrutura usada para perguntar sobre idade, e
não sobre distância percorrida.
A alternativa C está incorreta. “How high” é uma estrutura usada para perguntar sobre altura, e
não sobre distância percorrida.
A alternativa D está correta. “How far” é exatamente a estrutura que se utiliza para perguntar
sobre distância. Nesse caso, o quão longe ele vai quando sai para correr?
A alternativa E está incorreta. “How many” é uma estrutura usada para perguntar sobre
quantidade, e não sobre distância percorrida.
GABARITO: D

AULA 02 – VERBS IN TEXTS 139


TEACHER ANDREA BELO

QUESTÃO 05 (ESCOLA NAVAL/INÉDITA) – Which option completes the text below correctly?
Nowadays we live in a bipolar society. We _______ be able to unite instead of separating
ourselves. Anyone _______ do this. One just have to have a good dose of determination to do so.
(A) can't / can
(B) should / shouldn’t
(C) shouldn't / can't
(D) should / can
(E) shouldn't / should
Comentários: A primeira lacuna deve ser preenchida por “should”. O trecho inicia dizendo que
nós vivemos em uma sociedade bipolar, então, o trecho seguinte diz que nós deveríamos ser
capazes de nos unir ao invés de nos separar, e o verbo modal adequado para isso é o “should”.
A segunda lacuna deve ser preenchida por “can”. Esse verbo modal é ideal para essa situação,
pois seu uso indica capacidade de fazer algo. O trecho diz que qualquer pessoa pode fazer isso. A
pessoa só precisa de uma boa dose de determinação.
GABARITO: D

QUESTÕES EsPCEx
Leia o texto a seguir e responda às questões 01, 02 e 03.
Malaria 'completely stopped' by microbe
The team in Kenya and the UK say the finding has "enormous potential" to control the disease.
Malaria is spread by the bite of infected mosquitoes, so protecting them could in turn protect
people. The researchers are now investigating whether they can release infected mosquitoes into
the wild, or use spores to suppress the disease.
The malaria-blocking bug, Microsporidia MB, was discovered by studying mosquitoes on the
shores of Lake Victoria in Kenya. It lives in the gut and genitals of the insects. The researchers
could not find a single mosquito carrying the Microsporidia that was harbouring the malaria
parasite. And lab experiments confirmed the microbe gave the mosquitoes protection.
Microsporidias are fungi, or at least closely related to them, and most are parasites. However, this
new species may be beneficial to the mosquito and was naturally found in around 5% of the
insects studied.
"The data we have so far suggest it is 100% blockage, it's a very severe blockage of malaria," Dr
Jeremy Herren, from the International Centre of Insect Physiology and Ecology in Kenya told the
BBC. He added: "It will come as a quite a surprise. I think people will find that a real big
breakthrough. While huge progress has been made through the use of bed nets and spraying
homes with insecticide, it is widely agreed new tools are needed to tackle malaria.
How does the microbe stop malaria? Microsporidia MB could be priming the mosquito's immune
system, so it is more able to fight off infections. Or the presence of the microbe in the insect could
be having a profound effect on the mosquito's metabolism, making it inhospitable for the malaria
parasite.
Adapted from https://www.bbc.com/news/health-52530828

AULA 02 – VERBS IN TEXTS 140


TEACHER ANDREA BELO

QUESTÃO 01 (EsPCEx/INÉDITA) – In the sentence “... it is widely agreed new tools are needed to
tackle malaria.” (paragraph 3), the word tackle means
(A) neglect
(B) combat
(C) increase
(D) debate
(E) start
Comentários: percebemos no contexto em que o verbo tackle foi usado que, embora já tenha
sido feito um grande progresso através do uso de mosquiteiros e pulverização de inseticida em
residências, é amplamente sabido que novas ferramentas são necessárias para combater,
enfrentar a malária. Negligenciar (neglect) não equivale, assim, em sentido, a tackle. ERRADA.
Conforme vimos acima, o verbo tackle transmite, no contexto, justamente o sentido de combater,
enfrentar, lidar com um problema no sentido de solucioná-lo, no caso, a malária. CORRETA.
Increase significa aumentar. A ideia é de combater a doença, não a aumentar. ERRADA.
Debate significa debater. No contexto, a ideia não é de debater a doença, mas sim de combatê-
la. ERRADA.
Start significa começar, iniciar, não equivalendo, conforme estudado acima, em sentido, a tackle.
ERRADA.
GABARITO: B

QUESTÃO 02 (EsPCEx/INÉDITA) – Choose the alternative with the correct reference for the
underlined words from the text.
(A) them (paragraph 2) = fungi
(B) them (paragraph 1) = people
(C) they (paragraph 1) = mosquitoes
(D) It (paragraph 2) = malaria
(E) it (paragraph 4) = effect
Comentários: nesse tipo de questão, é essencial voltar ao texto para identificar corretamente a
quais termos os pronomes se referem nas frases. Vamos fazer isso em cada alternativa.
Atenção: fungi (fungos) é plural de fungus (fungo). Assim, no trecho, them se refere a fungi,
observe: “Microsporidias are fungi, or at least closely related to them, and most are parasites.” =
Microsporídios são fungos, ou pelo menos intimamente relacionados a eles (fungos- fungi), e a
maioria são parasitas. CORRETA.
“Malaria is spread by the bite of infected mosquitoes, so protecting them could in turn protect
people.” = A malária é transmitida pela picada de mosquitos infectados, portanto, protegê-los (os
mosquitos) poderia, por sua vez, proteger as pessoas. Não se refere a pessoas (people). ERRADA.

AULA 02 – VERBS IN TEXTS 141


TEACHER ANDREA BELO

“The researchers are now investigating whether they can release infected mosquitoes into the
wild...” = Os pesquisadores agora estão investigando se podem (eles, os investigadores) liberar
mosquitos infectados na natureza... O pronome não se refere a mosquitos (mosquitoes). ERRADA.
“The malaria-blocking bug... was discovered by studying mosquitoes on the shores of Lake Victoria
in Kenya. It lives in the gut and genitals of the insects.” = O micróbio que bloqueia a malária ... foi
descoberto através do estudo de mosquitos nas margens do lago Victoria, no Quênia. Ele vive no
intestino e nos órgãos genitais dos insetos. O pronome se refere ao micróbio que bloqueia a
malária, e não à malária em si. ERRADA.
“Or the presence of the microbe in the insect could be having a profound effect on the mosquito's
metabolism, making it inhospitable for the malaria parasite.” = Ou a presença do micróbio no
inseto pode estar afetando profundamente o metabolismo do mosquito, tornando-o (tornando o
mosquito) inóspito para o parasita da malária. O pronome não se refere ao efeito, mencionado
anteriormente no trecho. ERRADA.
GABARITO: A

QUESTÃO 03 (EsPCEx/INÉDITA) – According to the text, read the statements and choose the
correct alternative.
I – Malaria is spread by the bite of a bug called Microsporidia MB.
II – Microsporidias are fungi that can be beneficial to the mosquitoes that are malaria vectors.
III – Some mosquitoes carrying the Microsporidia still had the malaria parasite in them.
IV – The use of bed nets and insecticide have been proven to be enough to tackle malaria.
(A) Only I and II are correct.
(B) Only II and IV are correct.
(C) All of them are correct.
(D) Only II, III and IV are correct.
(E) Only II is correct.
Comentários: a malária não é transmitida pela picada do micróbio chamado Microsporidia MB,
mas sim pela picada de mosquitos infectados. O microsporídio, se encontrado no mosquito,
impede que o mosquito transmite a doença. “Malaria is spread by the bite of infected
mosquitoes...” FALSA.
De fato, microsporídios são fungos que podem ser benéficos para os mosquitos vetores da
malária, uma vez que os tornam inóspitos para o parasita da malária. “Microsporidias are fungi...
and most are parasites. However, this new species may be beneficial to the mosquito...”
VERDADEIRA.
O texto afirma que os pesquisadores não conseguiram encontrar um único mosquito carregando
o Microsporídio que estivesse abrigando o parasita da malária. → “The researchers could not find
a single mosquito carrying the Microsporidia that was harbouring the malaria parasite.” Assim,
dizer que alguns mosquitos portadores do Microsporídio ainda tinham o parasita da malária é
incorreto. FALSA.

AULA 02 – VERBS IN TEXTS 142


TEACHER ANDREA BELO

O texto afirma que o uso de mosquiteiros e inseticidas representou significativo progresso no


combate à malária, mas que é essencial desenvolver novas ferramentas para essa missão. Logo,
não se pode dizer que se comprovou que o uso de mosquiteiros e inseticidas foi suficiente. “While
huge progress has been made through the use of bed nets and spraying homes with insecticide,
it is widely agreed new tools are needed to tackle malaria.” FALSA.
GABARITO: E

Leia o texto a seguir e responda às questões 04, 05 e 06.


(Título omitido propositadamente)
Hoping to stem the torrent of false cures and conspiracy theories about COVID-19, Facebook
announced Thursday it would begin informing users globally who have liked, commented on, or
shared “harmful” misinformation about the coronavirus, pointing them instead in the direction
of a reliable source. Facebook hopes the move will drastically reduce the spread of false
information about the coronavirus online, a growing crisis that the World Health Organization
(WHO) has described as an “infodemic.”
“We want to connect people who may have interacted with harmful misinformation about the
virus with the truth from authoritative sources in case they see or hear these claims again,” said
Guy Rosen, Facebook’s Vice President for Integrity, in a blog post published early Thursday.
The new policy only applies to false claims related to the coronavirus, but campaigners say the
announcement could lay the groundwork for a breakthrough in the battle against political
disinformation online. “Facebook applying this to the pandemic is a good first step but this should
also be applied to political disinformation too, particularly with the 2020 U.S. election
approaching” says Fadi Quran, a campaign director at Avaaz, a global advocacy group that has
lobbied Facebook to “correct the record” on false information since 2018. (In the language of
online security, “disinformation” means the coordinated, purposeful spread of false information,
while “misinformation” refers to accidental inaccuracies.) “I hope this is going to be expanded to
other issues and fast,” Quran tells TIME.
Adapted from https://time.com/5822372/

QUESTÃO 04 (EsPCEx/INÉDITA) – Choose the most appropriate title for the text.
(A) Facebook bans users who have shared coronavirus information.
(B) Facebook notifies users who have interacted with coronavirus misinformation.
(C) Facebook prohibits political disinformation online before U.S. election.
(D) Facebook applies new policies to contain fake news about coronavirus and politics.
(E) Facebook users to face fines if they spread disinformation about COVID-19.
Comentários: o texto não afirma que o Facebook irá banir (ban) usuários que tenham
compartilhado quaisquer informações sobre coronavírus, mas sim que irá avisar usuários que
tenham interagido com algum tipo de informação incorreta (misinformation) sobre o vírus na rede
social. ERRADA.

AULA 02 – VERBS IN TEXTS 143


TEACHER ANDREA BELO

Esse seria o título mais apropriado para o texto, por refletir sua ideia central: Facebook notifica
usuários que interagiram com informações incorretas sobre o coronavírus. Veja que logo no início
do texto nos deparamos com essa ideia: “Hoping to stem the torrent of false cures and conspiracy
theories about COVID-19, Facebook announced Thursday it would begin informing users globally
who have liked, commented on, or shared ‘harmful’ misinformation about the coronavirus,
pointing them instead in the direction of a reliable source.” = Na esperança de conter a grande
quantidade de curas falsas e teorias da conspiração sobre o COVID-19, o Facebook anunciou na
quinta-feira que começaria a informar globalmente os usuários que curtiram, comentaram ou
compartilharam informações ‘prejudiciais’ sobre o coronavírus, apontando-os na direção de uma
fonte confiável. CORRETA.
O texto não fala que o Facebook proibiu a desinformação política online antes da eleição nos EUA.
Apenas aponta que adotar a política de conter informações falsas, como está fazendo
especificamente para a situação do coronavírus, poderia ser útil na seara política também,
especialmente com as eleições americanas se aproximando. “Facebook applying this to the
pandemic is a good first step but this should also be applied to political disinformation too,
particularly with the 2020 U.S. election approaching” = “O Facebook aplicar isso à pandemia é um
bom primeiro passo, mas isso também deve ser aplicado à desinformação política, principalmente
com a aproximação das eleições de 2020 nos EUA” ERRADA.
Como vimos acima, o Facebook só adotou, por enquanto, uma política de contenção de “fake
news” quanto ao coronavírus. O texto apenas sugere que seria interessante adotá-la também
para a política. ERRADA.
O texto não fala sobre aplicação de multas (fines) para usuários. ERRADA.
GABARITO: B

QUESTÃO 05 (EsPCEx/INÉDITA) – In the sentence “...but campaigners say the announcement


could lay the groundwork for a breakthrough in the battle against political disinformation
online....” (paragraph 3), the word groundwork means
(A) workout.
(B) breakup.
(C) background.
(D) foundation.
(E) harassment.
Comentários: no trecho trazido pela questão, groundwork se refere à base, a um trabalho
preliminar. → ... mas os ativistas dizem que o anúncio pode lançar as bases para um avanço na
batalha contra a desinformação política online ... Ou seja, a nova política de contenção de
informações falsas que está sendo adotada para o coronavírus pode ser a base para um avanço
nesse mesmo sentido no campo da política. Vamos buscar nas alternativas uma opção que tenha
significado semelhante.
Workout significa treino, sessão de exercícios físicos. ERRADA.

AULA 02 – VERBS IN TEXTS 144


TEACHER ANDREA BELO

Breakup significa rompimento, separação. ERRADA.


Background pode significar fundo, plano de fundo ou ainda experiência (de vida). Não
corresponde exatamente a um trabalho preliminar, de base. ERRADA.
Foundation significa justamente base, fundação. Equivale, no contexto da questão, a groundwork.
CORRETA.
Harassment significa assédio, não tendo relação com o contexto. ERRADA.
GABARITO: D

QUESTÃO 06 (EsPCEx/INÉDITA) – According to the text, choose the correct statement.


(A) Facebook announced false cures and conspiracy theories about COVID-19.
(B) Facebook developed a new policy to contain fake news during the U.S. election.
(C) Facebook is taking measures aiming to reduce the spread of coronavirus misinformation.
(D) Facebook is notifying users in the U.S. who have interacted with misinformation about the
coronavirus.
(E) Facebook has confirmed that its new policy will be used to contain political disinformation as
well.
Comentários: o texto não afirma que o Facebook anunciou notícias falsas sobre a COVID-19. A
plataforma está notificando seus usuários que assim o fizeram ou de alguma forma interagiram
com notícias incorretas sobre a pandemia. “Hoping to stem the torrent of false cures and
conspiracy theories about COVID-19, Facebook announced Thursday it would begin informing
users globally who have liked, commented on, or shared “harmful” misinformation about the
coronavirus, pointing them instead in the direction of a reliable source.” ERRADA.
A nova política do Facebook não foi desenvolvida para conter “fake news” políticas, mas sim sobre
o coronavírus. O texto afirma que poderia ser interessante adotar a mesma política na seara
política. “The new policy only applies to false claims related to the coronavirus, but campaigners
say the announcement could lay the groundwork for a breakthrough in the battle against political
disinformation online.” ERRADA.
De fato, o Facebook está adotando medidas com o objetivo de reduzir a disseminação de
informações errôneas a respeito do coronavírus, como informar seus usuários caso interajam de
alguma forma com tais informações e redirecioná-los para fontes seguras. CORRETA.
O Facebook não está notificando usuários apenas nos Estados Unidos, mas no mundo todo
(globally). “Facebook announced Thursday it would begin informing users globally who have
liked, commented on, or shared ‘harmful’ misinformation about the coronavirus...” ERRADA.
Não há no texto esse dado de que o Facebook confirmou que vai adotar sua nova política também
(as well) para conter desinformação política. ERRADA.
GABARITO: C

AULA 02 – VERBS IN TEXTS 145


TEACHER ANDREA BELO

Leia o texto a seguir e responda às questões 07, 08 e 09.


The true cost of Saudi Arabia’s $500bn megacity
With a huge artificial moon, glow-in-the-dark beaches, flying drone-powered taxis, robotic butlers
__________ (1) clean the homes of residents and a Jurassic Park-style attraction featuring
animatronic lizards, Neom has been billed __________ (2) humanity’s next chapter. But beneath
the glitzy veneer lies a story of threats, forced eviction and bloodshed.
“The future has a new home,” proclaims the website, as the promotional video tracks colour-
tinted panoramic shots of picturesque desert expanses, and deep azure lagoons. The new city
state of Neom, named __________ (3) a combination of the Greek word for “new” and the Arabic
term for “future”, is intended to cover an area the size of Belgium at the far north of Saudi Arabia’s
Red Sea coastline.
There has been no shortage of outlandish promises for the $500bn (£400bn) city-state. Yet part
of the site is the home of the Huwaitat tribe, who have spanned Saudi Arabia, Jordan, and the
Sinai Peninsula for generations, tracing their lineage back before the founding of the Saudi state.
At least 20,000 members of the tribe now face eviction due to the project, with no information
about where they will live in the future.
“For the Huwaitat tribe, Neom is being built on our blood, on our bones,” says Alia Hayel
Aboutiyah al-Huwaiti, an outspoken activist and member of the tribe. Construction is ploughing
ahead despite the challenges of the pandemic and the historic drop in the price of oil, Saudi’s
primary export. For many, though, Neom is less a shining vision of the future than a grim symbol
of Saudi human rights violations, underscored by the treatment of the Huwaitat tribe.
Adapted from https://www.theguardian.com/global-development/2020/may/04/its-being-built-on-our-blood-the-true-cost-of-saudi-arabia-5bn-mega-city-neom

QUESTÃO 07 (EsPCEx/INÉDITA) – Choose the alternative containing the correct words to


respectively complete gaps (1), (2) and (3).
(A) at, of, to
(B) to, on, at
(C) to, as, from
(D) in, with, an
(E) with, of, as
Comentários: vamos analisar cada trecho do texto em que havia lacuna a ser preenchida.
With a huge artificial moon, glow-in-the-dark beaches, flying drone-powered taxis, robotic butlers
_______ (1) clean the homes of residents and a Jurassic Park-style attraction featuring
animatronic lizards... = Nesse primeiro trecho, vemos uma descrição de Neom, a cidade-estado
de que o texto trata. Nessa cidade futurista, haverá mordomos robóticos __________ limpar as
casas dos moradores. Você consegue perceber que esses robôs servirão PARA limpar a casa? Essa
será a finalidade deles. A melhor preposition, no caso, será, então to.
...Neom has been billed __________ (2) humanity’s next chapter. = Continuando a descrição de
Neom, ela foi descrita (billed) COMO o próximo capítulo da humanidade. Com a expressão billed,

AULA 02 – VERBS IN TEXTS 146


TEACHER ANDREA BELO

assim como com sua expressão sinônima described, para transmitir a ideia de “descrita como”,
usamos as.
The new city state of Neom, named __________ (3) a combination of the Greek word for “new”
and the Arabic term for “future” ... = Observe que, nesse trecho, o texto explica a origem do nome,
de onde ele vem. A preposition em Inglês que transmite essa ideia de origem é from- A nova
cidade-estado de Neom, nomeada A PARTIR DE uma combinação da palavra grega para "novo" e
o termo árabe para "futuro"...
GABARITO: C

QUESTÃO 08 (EsPCEx/INÉDITA) – Which excerpt from the text refers to the title “The true cost
of Saudi Arabia’s $500bn megacity”?
(A) “‘The future has a new home,’ proclaims the website...” (paragraph 2)
(B) “There has been no shortage of outlandish promises for the $500bn (£400bn) city-state.”
(paragraph 3)
(C) “With a huge artificial moon, glow-in-the-dark beaches, flying drone-powered taxis, robotic
butlers...” (paragraph 1)
(D) “The new city state of Neom... is intended to cover an area the size of Belgium at the far
north of Saudi Arabia’s Red Sea coastline.” (paragraph 2)
(E) “But beneath the glitzy veneer lies a story of threats, forced eviction and bloodshed.”
(paragraph 1)
Comentários: a questão quer que você identifique qual trecho do texto se refere ao título: "O
verdadeiro custo da megacidade de US$ 500 bilhões da Arábia Saudita". Vamos analisar os trechos
trazidos por cada alternativa.
O futuro tem um novo lar, proclama o site ... Esse trecho não se refere ao verdadeiro custo da
megacidade de Neom. ERRADA.
Não faltaram promessas estranhas para a cidade-estado de US$ 500 bilhões (£400 bilhões). O
título deixa subentendido que há um outro custo por trás da construção da cidade, que vai além
do dinheiro. Assim, esse trecho também não se refere ao verdadeiro custo da megacidade de
Neom. ERRADA.
Com uma enorme lua artificial, praias que brilham no escuro, táxis voadores movidos a drones,
mordomos robóticos ... Esse trecho apenas enumera, de forma descritiva, alguns atrativos que
haverá na megacidade. Não revela o verdadeiro custo de sua construção. ERRADA.
A nova cidade-estado de Neom ... destina-se a cobrir uma área do tamanho da Bélgica no extremo
norte da costa do Mar Vermelho da Arábia Saudita. Esse trecho apenas deixa claro o tamanho da
cidade. Não revela o verdadeiro custo de sua construção. ERRADA.
Mas por baixo da camada glamurosa, há uma história de ameaças, despejo forçado e
derramamento de sangue. Esse trecho, sim, relaciona-se ao título, revelando o verdadeiro custo
da construção da megacidade. O que está por trás de todo o glamour é uma história de
desocupação forçada do terreno e violência. CORRETA.
GABARITO: E

AULA 02 – VERBS IN TEXTS 147


TEACHER ANDREA BELO

QUESTÃO 09 (EsPCEx/INÉDITA) – In the sentence “But beneath the glitzy veneer lies a story of
threats, forced eviction and bloodshed.” (paragraph 1), the expression beneath the glitzy veneer
means
(A) under the glamorous appearance.
(B) over the oblivious look.
(C) despite the shiny appearance.
(D) under the fake aspect.
(E) not so important as its cost.
Comentários: nós vimos o trecho trazido por essa questão na letra E da questão anterior,
remember? Ele significa: Mas por baixo da camada glamurosa, há uma história de ameaças,
despejo forçado e derramamento de sangue. Beneath tem o sentido de por baixo, sob. Observe
como glitzy faz lembrar uma palavra que você certamente conhece: glitter, ambas relacionadas a
glitz, que significa brilho. Glitzy se refere a algo que chama atenção, que brilha, glamuroso. Por
fim, veneer diz respeito à camada superficial de algo, à aparência. Vejamos as alternativas. Já
poderíamos, de imediato, eliminar as letras b, c e e, pois elas não trazem sinônimos de beneath.
Por baixo da aparência glamourosa- como vimos, é exatamente esse o sentido da expressão.
CORRETA.
Sobre o olhar desatento (ou aparência desatenta, a depender do contexto) - over significa sobre,
acima e o restante também não equivale à expressão. ERRADA.
Apesar da aparência brilhante- o “apesar” invalidou a alternativa. ERRADA.
Sob o aspecto falso- como vimos, o texto se refere a uma aparência glamurosa, chamativa, não
necessariamente falsa. ERRADA.
Não é tão importante quanto seu custo- essa alternativa não tem relação com a expressão trazida
pelo texto, conforme explicações dadas acima. ERRADA.
GABARITO: A

Leia o texto a seguir e responda à questão 10.


How surviving a recession prepared me for Covid-19
I was 35 when I lost my job as a legal courier in San Jose, California. It was 2009, and the economy
had lost nearly six million jobs since the Great Recession began in 2007.
Finding work was nearly impossible. At one point, I juggled three jobs that paid a total of less than
$500 per week. It was barely enough to get by. For the next three to four years, I was mostly
unemployed, homeless — sleeping on benches, shelters and sometimes friends’ couches — and
living off the McDonald’s Dollar Menu.

AULA 02 – VERBS IN TEXTS 148


TEACHER ANDREA BELO

Those were some dark times. But I’ve come a long way since then. In 2017, I moved to Tacoma,
Washington with my best friend Kim, who lent me just enough money to settle down and pursue
my dream of getting into real estate. You’re getting another shot, I told myself. Don’t blow it.
I didn’t. Eight months after I got my broker license, I closed my seventh deal — for a total of $1.6
million in sales. I paid back the money Kim loaned me. For the first time in my life, I had a feeling
of accomplishment and satisfaction. I got serious about saving money and started planning for
the future.
While the underlying cause of today’s economic slowdown is fundamentally different from the
Great Recession, the lessons I learned over the past decade still apply. And they’ve made me more
prepared to handle the hardships that have — and will — come from the coronavirus pandemic.
Adapted from https://www.cnbc.com/2020/04/23/i-was-unemployed-homeless-how-surviving-great-recession-prepared-me-for-coronavirus.html

QUESTÃO 10 (EsPCEx/INÉDITA) – What kind of text is this?


(A) A contract.
(B) An autobiographical text.
(C) A book review.
(D) A fantasy story.
(E) A science fiction narrative.
Comentários: como vemos, o texto é todo escrito em primeira pessoa- “I was 35 when I lost my
job as a legal courier in San Jose, California.” O autor narra sua própria experiência e conta como
superou as dificuldades após uma grande crise em sua vida em consequência da grande recessão
dos anos 2000. Os textos que são tipicamente escritos na primeira pessoa e usam o pronome “I”
são do tipo autobiográficos, e permitem ao escritor refletir e fornecer uma perspectiva dos
eventos da sua vida. Vamos ver as alternativas.
O texto não é um contrato. É um relato pessoal, autobiográfico. ERRADA.
Como vimos, o texto é, de fato, autobiográfico. CORRETA.
O texto não é uma crítica literária, não faz menção a nenhum livro. ERRADA.
Uma “fantasy story” é uma história de fantasia, que pode conter monstros, dragões, fadas, elfos...
Não é o caso do nosso texto, que, pelo contrário, retrata a realidade vivida pelo autor. ERRADA.
Assim como a alternativa anterior, a letra E também está incorreta, pois classifica o texto como
ficcional, como uma narrativa de ficção científica. ERRADA.
GABARITO: B

AULA 02 – VERBS IN TEXTS 149


TEACHER ANDREA BELO

CONSIDERAÇÕES FINAIS
Concluímos mais uma aula, outro passo até a sua aprovação!
Nota-se o progresso nos estudos, o ganho de vocabulário e uma maior tranquilidade para
enfrentar os exercícios que surgem.
E você vai se acostumando a equilibrar seus estudos de forma sistematizada, estudando
cada vez mais e com mais dedicação.
Outro detalhe importante para seu sucesso nos estudos, é continuar fazendo listas de
vocabulário das palavras e verbos, principalmente os irregulares, que aparecem em forma de lista
em inúmeras fontes de pesquisa.
Isso te ajudará nas questões futuras e torna você, como eu disse antes, um candidato mais
bem preparado e confiante para realizar uma excelente prova.
É importante lembrar também do nosso Fórum de dúvidas, exclusivo do Estratégia
Militares. Será minha forma de responder o que mais você precise saber para que os conteúdos
fiquem ainda mais claros.
E, caso queira, acesse minhas redes sociais para aprender mais palavras e contar com dicas
importantes, que colaboram diretamente com seus estudos dia após dia.

AULA 02 – VERBS IN TEXTS 150


TEACHER ANDREA BELO

REFERÊNCIAS BIBLIOGRÁFICAS
BARRETO, Tania Pedroza; GARRIDO, Maria Line; SILVA, João Antenor de C., Inglês Instrumental.
Leitura e compreensão de textos. Salvador, Ba UFBA, 1995, p. 64.
BROWN. H. Douglas. Principles of Language Learning and Teaching. Prentice Hall International,
1988.
COMPEDELLI, Samira Yousseff. Português, Literatura, Produção de texto & Gramática – São Paulo:
Ed. Saraiva, 2002.
CORREIA, Clese Mary P. Reading Specific Purposes. Salvador/ Ba: UFBA, 1999.
COSTEIRA, Adriana Araújo de M. Reading Comprehension Skills. João Pessoa/PB: ETFP, 1998.
CRYSTAL David. Cambridge University Press 1997. The Cambridge Encyclopedia of Language.
Cambridge University Press 1997
FREEMAN. Diane Larsen. MURCIA. Marianne Celce. The Grammar Book, 1999.
DYE, Joan., FRANFORT, Nancy. Spectrum II, III A Communicative Course in English. USA, Prentice
Hall, 1994.
FAVERO, Maria de Lourdes Albuquerque (org.). Dicionário de educadores no Brasil: da colônia aos
dias atuais. Rio de Janeiro: UFRJ, MEC, INEP, 1999.
FRANKPORT, Nancy & Dye Hoab. Spectrum II, III Prentice Hall Regents Englewood Cliffs, New Jersy,
1994.
GADELHA, Isabel Maria B. Inglês Instrumental: Leitura, Conscientização e Prática. Teresina:
EDUFFI, 2000.
GUANDALINI, Eiter Otávio. Técnicas de Leitura em Inglês: ESP – English For Specific Purposes:
estagio 1. São Paulo: Texto novo, 2002.
GRELLET, Françoise. Developing Reading Skills. Cambridge University Press, 1995
HOLAENDER, Arnon & Sanders Sidney. A complete English Course. São Paulo. Ed. Moderna, 1995.
HUTCHINSON, Tom & WATERS, Alan. English for Specific Purposes. Cambridge: Cambridge
University Press, 1996
KRASHEN. Stephen D. Second Language Acquisition and Second Language Learning, Prentice-Hall
International, 1988.
LAENG, Mauro. Dicionário de pedagogia. Lisboa: Dom Quixote, 1973.
LEFFA, Vilson J. Metodologia do ensino de línguas. In: BOHN, H.; VANDRESEN, P. (org.). Tópicos de
linguística aplicada: o ensino de línguas estrangeiras. Florianópolis: Editora da UFSC, 1988. p. 211-
231.
LIBERATO, Wilson. Compact English Book Inglês Ensino Médio. São Paulo: FTD, Vol. Único, 1998
Mc ARTHUR. The Oxford Companion to the English Language. Oxford University Press 1992
Fromkin. Victoria. An Introduction to Language
MARQUES, Amadeu. Inglês Série Brasil. ed. Atica. São Paulo: 2004. Vol. Único.

AULA 02 – VERBS IN TEXTS 151


TEACHER ANDREA BELO

MURPHY, Raymond: Essencial Grammar in Use Oxford. New York Ed. Oxford University, 1997.
OLIVEIRA, Luciano Amaral. English For Tourism Students. Inglês para Estudantes de Turismo: São
Paulo, Rocca, 2001.
OLIVEIRA, Sara Rejane de F. Estratégias de leitura para Inglês Instrumental. Brasília: UNB, 1994.
QUINTANA, et alli. First Certificate. Master Class Oxford. New York, 2004: Ed. Oxford University.
PAULINO, Berenice F. et all. Leitura em textos em Inglês – Uma Abordagem Instrumental. Belo
Horizonte: Ed. Dos Autores, 1992.
PEREIRA, Edilberto Coelho. Inglês Instrumental. Teresina: ETFPI, 1998.
RODGES, Theodore. Jack C. Richards. Approaches and Methods in Language Teaching. Cambridge
University Press, 2001.
RODMAN Robert. Harcourt Brace 1993. English as a Global Language
STEWART, B., HAINES S. First Certificate, MasterClass. UK – Oxford 2004.
SILVA, João Antenor de C., GARRIDO, Maria Lina, BARRETO, Tânia Pedrosa. Inglês Instrumental:
Leitura e Compreensão de Textos. Salvador: Centro Editorial e Didático, UFBA. 1994
SOARES, Moacir Bretãs. Dicionário de legislação do ensino. 19.ed. Rio de Janeiro: FGV, 1981.
SOUZA, Adriana Srade F. Leitura em Língua Inglesa: Uma abordagem Instrumental. São Paulo:
Disal, 2005.
TUCK, Michael. Oxford Dictionary of Computing for Learners of English. Oxford: Oxford University
Press, 1996.
TOTIS, Verônica Pakrauskas. Língua Inglesa: leitura. São Paulo: Cortez, 1991.

Livros eletrônicos:
Dicionário Houaiss da Língua Portuguesa, Editora Objetiva, 2001.
MOURãO, Janaína Pereira. "Skimming x Scanning"; Brasil Escola. Disponível em
<https://brasilescola.uol.com.br/ingles/skimming-x-scanning.htm>. Acesso em 20 de março de
2019.
www.newsweek.com - Acesso em 18 de março de 2019.
http://www.galaor.com.br/tecnicas-de-leitura/ - Acesso em 19 de março de 2019.
Expressões Idiomáticas (continuação)" em Só Língua Inglesa. Virtuous Tecnologia da
Informação,2008-2019. Consultado em 03/04/2019 às 22:09. Disponível na Internet em
http://www.solinguainglesa.com.br/conteudo/Expressoes5.php

AULA 02 – VERBS IN TEXTS 152


TEACHER ANDREA BELO

TRADUÇÕES
The machine always wins: what drives our addiction to social media
We are swimming in writing. Our lives have become, in the words of the author and academic
Shoshana Zuboff, an “electronic text”. Social media platforms have created a machine for us to
write to. The bait is that we are interacting with other people: our friends, colleagues, celebrities,
politicians, royals, terrorists, porn actors – anyone we like. We are not interacting with them,
however, but with the machine. We write to it, and it passes on the message for us after keeping
a record of the data.
The machine benefits from the “network effect”: the more people write to it, the more benefits
it can offer, until it becomes a disadvantage not to be part of it. Part of what? The world’s first ever
public, live, collective, open-ended writing project. A virtual laboratory. An addiction machine,
which deploys crude techniques of manipulation redolent of the Skinner Box created by
behaviourist BF Skinner to control the behaviour of pigeons and rats with rewards and
punishments. We are users, much as cocaine addicts are users.
What is the incentive to engage in writing like this for hours each day? In a form of mass
casualisation, writers no longer expect to be paid or given employment contracts. What do the
platforms offer us, in lieu of a wage? What gets us hooked? Approval, attention, retweets, shares
and likes.
Meanwhile, hashtags and trending topics underline the extent to which all of these protocols are
organised around the massification of individual voices – a phenomenon cheerfully described by
users with the science-fiction concept of the “hive mind” – and hype. The regular sweet spot
sought after is a brief period of ecstatic collective frenzy around any given topic. It doesn’t
particularly matter to the platforms what the frenzy is about: the point is to generate data, one of
the most profitable raw materials yet discovered. As in the financial markets, volatility adds value.
The more chaos, the better.
Whether or not we think we are addicted, the machine treats us as addicts. Addiction is, quite
deliberately, the template for our relationship to the Twittering Machine. Addiction is all about
attention. For the social media bosses, this is axiomatic.
If social media is an addiction machine, the addictive behaviour it is closest to is gambling: a rigged
lottery. Every gambler trusts in a few abstract symbols – the dots on a dice, numerals, suits, red or
black, the graphemes on a fruit machine – to tell them who they are. In most cases, the answer is
brutal and swift: you are a loser and you are going home with nothing. The true gambler takes a
perverse joy in anteing up, putting their whole being at stake. On social media, you scratch out a
few words, a few symbols, and press send, rolling the dice. The internet will tell you who you are
and what your destiny is through arithmetic likes, shares and comments.
The interesting question is what it is that is so addictive. In principle, anyone can win big; in
practice, not everyone is playing with the same odds. Our social media accounts are set up like
enterprises competing for attention. If we are all authors now, we write not for money, but for the
satisfaction of being read. Going viral, or trending, is the equivalent of a windfall. But sometimes,
winning is the worst thing that can happen. The temperate climate of likes and approval is apt to
break, lightning-quick, into sudden storms of fury and disapproval.
(Adapted from https://www.theguardian.com/technology/2019/aug/23/social-media-addiction-gambling)

AULA 02 – VERBS IN TEXTS 153


TEACHER ANDREA BELO

A máquina sempre vence: o que impulsiona nosso vício em mídias sociais


Estamos nadando na escrita. Nossas vidas se tornaram, nas palavras da autora e acadêmica
Shoshana Zuboff, um “texto eletrônico”. As plataformas de mídia social criaram uma máquina para
escrevermos. A isca é que estamos interagindo com outras pessoas: nossos amigos, colegas,
celebridades, políticos, membros da realeza, terroristas, atores pornôs - qualquer pessoa de quem
gostemos. Não estamos interagindo com eles, porém, mas com a máquina. Nós escrevemos para
ele e ele passa a mensagem para nós depois de manter um registro dos dados.
A máquina se beneficia do “efeito rede”: quanto mais pessoas escrevem nela, mais benefícios ela
pode oferecer, até que se torne uma desvantagem não fazer parte dela. Parte de quê? O primeiro
projeto de escrita público, ao vivo, coletivo e aberto do mundo. Um laboratório virtual. Uma
máquina de vício, que implanta técnicas rudes de manipulação que lembram a Caixa de Skinner
criada pelo comportamentalista BF Skinner para controlar o comportamento de pombos e ratos
com recompensas e punições. Somos usuários, tanto quanto os viciados em cocaína são usuários.
Qual é o incentivo para escrever assim por horas todos os dias? Em uma forma de precarização
em massa, os escritores não esperam mais ser pagos ou receber contratos de trabalho. O que as
plataformas nos oferecem, em vez de um salário? O que nos vicia? Aprovação, atenção, retuítes,
compartilhamentos e curtidas.
Enquanto isso, hashtags e tópicos de tendência sublinham até que ponto todos esses protocolos
são organizados em torno da massificação de vozes individuais - um fenômeno alegremente
descrito pelos usuários com o conceito de ficção científica da “mente coletiva” - e exagero. O
ponto ideal comum procurado é um breve período de frenesi coletivo extático em torno de
qualquer tópico. Não importa particularmente para as plataformas o que é esse frenesi: o objetivo
é gerar dados, uma das matérias-primas mais lucrativas já descobertas. Como nos mercados
financeiros, a volatilidade agrega valor. Quanto mais caos, melhor.
Quer pensemos ou não que somos viciados, a máquina nos trata como viciados. O vício é,
deliberadamente, o modelo para nosso relacionamento com a Máquina de Twitter. O vício tem
tudo a ver com atenção. Para os chefes das redes sociais, isso é axiomático.
Se a mídia social é uma máquina de vício, o comportamento viciante de que ela está mais próxima
é o jogo: uma loteria fraudada. Todo jogador confia em alguns símbolos abstratos - os pontos em
um dado, numerais, naipes, vermelho ou preto, os grafemas em uma máquina de frutas - para
dizer quem eles são. Na maioria dos casos, a resposta é brutal e rápida: você é um perdedor e está
indo para casa sem nada. O verdadeiro jogador sente uma alegria perversa em apostar, colocando
todo o seu ser em jogo. Nas redes sociais, você risca algumas palavras, alguns símbolos e pressiona
enviar, lançando os dados. A internet vai te dizer quem você é e qual é o seu destino por meio de
curtidas aritméticas, compartilhamentos e comentários.
A questão interessante é o que é tão viciante. Em princípio, qualquer um pode ganhar muito; na
prática, nem todos estão jogando com as mesmas chances. Nossas contas de mídia social são
configuradas como empresas competindo por atenção. Se agora somos todos autores, não
escrevemos por dinheiro, mas pela satisfação de sermos lidos. Tornar-se viral, ou tendência, é o
equivalente a uma sorte inesperada. Mas às vezes, vencer é a pior coisa que pode acontecer. O
clima temperado de gostos e aprovação tende a se transformar, rapidamente, em repentinas
tempestades de fúria e desaprovação.

AULA 02 – VERBS IN TEXTS 154


TEACHER ANDREA BELO

Many COVID-19 patients lost their sense of smell. Will they get it back?
IN EARLY MARCH, Peter Quagge began experiencing COVID-19 symptoms, such as chills and a low-
grade fever. As he cut pieces of raw chicken to cook for dinner one night, he noticed he couldn’t
smell the meat. “Must be really fresh,” he remembers thinking. But the next morning he couldn’t
smell the Dial soap in the shower or the bleach he used to clean the house. “It sounds crazy, but
I thought the bleach had gone bad,” he says. When Quagge stuck his head into the bottle and
took a long whiff, the bleach burned his eyes and nose, but he couldn’t smell a thing.
The inability to smell, or anosmia, has emerged as a common symptom of COVID-19. Quagge was
diagnosed with COVID-19, though he was not tested, since tests were not widely available at the
time. He sought anosmia treatment with multiple specialists and still has not fully recovered his
sense of smell.
Case reports suggest that anywhere between 34 and 98 percent of hospitalized patients with
COVID-19 will experience anosmia. One study found that COVID-19 patients are 27 times more
likely than others to lose their sense of smell, making anosmia a better predictor of the illness
than fever.
For most COVID-19 patients who suffer anosmia, the sense returns within a few weeks, and
doctors don’t yet know if the virus causes long-term smell loss. While not being able to smell may
sound like a small side effect, the results can be devastating. The sense is intricately tied up in
self-preservation—the ability to smell fire, chemical leaks, or spoiled food—and in our ability to
pick up on complex tastes and enjoy food.
“So many of the ways we connect with each other is over meals or over drinks,” says Steven
Munger, director of the Center for Smell and Taste at the University of Florida. “If you can’t fully
participate in that, it creates a sort of social gap.”
Smell even plays a role in our emotional lives, connecting us to loved ones and memories. People
without smell often report feeling isolated and depressed and losing their enjoyment in intimacy.
Now scientists are starting to unravel how COVID-19 affects this critical sense, hoping those
discoveries will help thousands of newly anosmic people looking for answers.
What the nose knows
The olfactory system, which allows humans and other animals to smell, is essentially a way of
decoding chemical information. When someone takes a big sniff, molecules travel up the nose to
the olfactory epithelium, a small piece of tissue at the back of the nasal cavity.
Those molecules bind to olfactory sensory neurons, which then send a signal by way of an axon,
a long tail that threads through the skull and delivers that message to the brain, which registers
the molecules as, say, coffee, leather, or rotting lettuce.
Scientists still don’t fully understand this system, including exactly what happens when it stops
working. And most people don't realize how common smell loss really is, Munger says. “That lack
of public understanding means there’s less attention to try to understand the basic functions of
the system.”
People can lose their sense of smell after suffering a viral infection, like influenza or the common
cold, or after a traumatic brain injury. Some are born without any sense of smell at all or lose it
because of cancer treatments or diseases like Parkinson’s and Alzheimer’s. It may also fade as

AULA 02 – VERBS IN TEXTS 155


TEACHER ANDREA BELO

people age. While smell disorders aren’t as apparent as hearing loss or vision impairment, data
from the National Institutes of Health (NIH) show that nearly 25 percent of Americans over the
age of 40 report some kind of change in their sense of smell, and over 13 million people have a
measurable disorder like anosmia, the total loss of smell, or hyposmia, a partial loss. Such
conditions can last for years or even be permanent.
It’s not clear if COVID-19 anosmia is different from other instances of smell loss caused by a virus,
but those who experience anosmia due to COVID-19 appear to be unique in a few ways. First, they
notice the loss of the sense immediately because it’s not accompanied by the congestion or
stuffiness that generally characterizes the early stages of virally induced anosmia.
“It’s very dramatic,” says Danielle Reed, associate director of the Monell Chemical Senses Center
in Philadelphia, which studies smell and taste loss. “People just cannot smell anything.”
Another notable difference is that many patients with COVID-19 who report losing their sense of
smell get it back relatively quickly, in just a few weeks, unlike most people who experience
anosmia from other viruses, which can last months or years.
Quagge estimates he’s recovered about 60 percent of his sense of smell so far, but he says in the
early days, without any information about when or if he’d ever get it back, he was scared. An avid
amateur chef, he had to rely on his family to tell him if the milk was bad, and he couldn’t smell his
wife’s perfume. “Stuff that gets to your soul,” he says. “It bummed me out.”
(Adapted from https://www.nationalgeographic.com/science/2020/08/thousands-covid-19-patients-lost-sense-smell-will-get-back-cvd/)

Muitos pacientes com COVID-19 perderam o olfato. Eles o receberão de volta?


NO INÍCIO DE MARÇO, Peter Quagge começou a sentir os sintomas da COVID-19, como calafrios
e febre baixa. Enquanto cortava pedaços de frango cru para cozinhar para o jantar uma noite, ele
percebeu que não sentia o cheiro da carne. “Deve ser muito novo”, ele se lembra de ter pensado.
Mas na manhã seguinte ele não sentia o cheiro do sabonete Dial no chuveiro ou da água sanitária
que usava para limpar a casa. “Parece loucura, mas achei que o alvejante tinha estragado”, diz
ele. Quando Quagge enfiou a cabeça na garrafa e deu uma longa baforada, o alvejante queimou
seus olhos e nariz, mas ele não sentiu o cheiro de nada.
A incapacidade de cheirar, ou anosmia, surgiu como um sintoma comum da COVID-19. Quagge
foi diagnosticado com COVID-19, embora não tenha sido testado, uma vez que os testes não
estavam amplamente disponíveis na época. Procurou tratamento de anosmia com vários
especialistas e ainda não recuperou totalmente o olfato.
Relatos de casos sugerem que algo entre 34 e 98 por cento dos pacientes hospitalizados com
COVID-19 terão anosmia. Um estudo descobriu que os pacientes com COVID-19 têm 27 vezes
mais probabilidade do que outros de perder o olfato, tornando a anosmia um melhor indicador
da doença do que a febre.
Para a maioria dos pacientes com COVID-19 que sofrem de anosmia, o sentido retorna dentro de
algumas semanas, e os médicos ainda não sabem se o vírus causa perda de cheiro em longo prazo.
Embora não ser capaz de cheirar possa parecer um pequeno efeito colateral, os resultados podem
ser devastadores. O sentido está intrinsecamente ligado à autopreservação - a capacidade de

AULA 02 – VERBS IN TEXTS 156


TEACHER ANDREA BELO

cheirar fogo, vazamentos de produtos químicos ou comida estragada - e em nossa capacidade de


captar sabores complexos e apreciar a comida.
“Muitas das maneiras de nos conectarmos são durante as refeições ou bebidas”, diz Steven
Munger, diretor do Centro de Olfato e Sabor da Universidade da Flórida. “Se você não pode
participar totalmente disso, isso cria uma espécie de lacuna social.”
O cheiro até desempenha um papel em nossas vidas emocionais, conectando-nos aos entes
queridos e às memórias. Pessoas sem cheiro frequentemente relatam que se sentem isoladas e
deprimidas e perdem o prazer na intimidade. Agora os cientistas estão começando a desvendar
como o COVID-19 afeta esse senso crítico, na esperança de que essas descobertas ajudem
milhares de pessoas recentemente anosmicas em busca de respostas.
O que o nariz sabe
O sistema olfativo, que permite aos humanos e outros animais cheirar, é essencialmente uma
forma de decodificar informações químicas. Quando alguém cheira profundamente, as moléculas
sobem pelo nariz até o epitélio olfatório, um pequeno pedaço de tecido na parte posterior da
cavidade nasal.
Essas moléculas se ligam a neurônios sensoriais olfativos, que então enviam um sinal por meio de
um axônio, uma longa cauda que passa pelo crânio e entrega essa mensagem ao cérebro, que
registra as moléculas como, digamos, café, couro ou alface podre.
Os cientistas ainda não entendem totalmente este sistema, incluindo exatamente o que acontece
quando ele para de funcionar. E a maioria das pessoas não percebe como a perda de cheiro
realmente é comum, diz Munger. “Essa falta de compreensão do público significa que há menos
atenção para tentar entender as funções básicas do sistema.”
As pessoas podem perder o olfato após sofrer uma infecção viral, como gripe ou resfriado comum,
ou após um traumatismo cranioencefálico. Alguns nascem sem qualquer sentido do olfato ou
perdem-no por causa de tratamentos de câncer ou doenças como Parkinson e Alzheimer.
Também pode desaparecer à medida que as pessoas envelhecem. Embora os distúrbios do olfato
não sejam tão aparentes quanto a perda de audição ou visão, dados do National Institutes of
Health (NIH) mostram que quase 25% dos americanos com mais de 40 anos relatam algum tipo
de mudança no olfato, ou mais 13 milhões de pessoas têm um distúrbio mensurável como
anosmia, a perda total do olfato, ou hiposmia, uma perda parcial. Essas condições podem durar
anos ou mesmo ser permanentes.
Não está claro se a anosmia do COVID-19 é diferente de outros casos de perda do olfato causada
por um vírus, mas aqueles que experimentam anosmia devido ao COVID-19 parecem ser únicos
em alguns aspectos. Primeiro, eles notam a perda do sentido imediatamente porque não é
acompanhada pela congestão ou entupimento que geralmente caracteriza os estágios iniciais da
anosmia induzida por vírus.
“É muito dramático”, diz Danielle Reed, diretora associada do Monell Chemical Senses Center, na
Filadélfia, que estuda a perda de cheiro e sabor. “As pessoas simplesmente não conseguem
cheirar nada.”

AULA 02 – VERBS IN TEXTS 157


TEACHER ANDREA BELO

Outra diferença notável é que muitos pacientes com COVID-19 que relatam ter perdido o olfato,
o recuperam com relativa rapidez, em apenas algumas semanas, ao contrário da maioria das
pessoas que experimentam anosmia de outros vírus, que podem durar meses ou anos.
Quagge estima que recuperou cerca de 60 por cento do olfato até agora, mas ele diz que nos
primeiros dias, sem qualquer informação sobre quando ou se o recuperaria, ele estava com medo.
Um chef amador ávido, ele teve que confiar em sua família para lhe dizer se o leite estava ruim e
ele não podia sentir o cheiro do perfume de sua esposa. “Coisas que chegam a sua alma”, diz ele.
"Isso me chateou."

Douglas Yancey, "Doug" Fannie, is the titular character and chief protagonist of the animated
series Doug. Douglas is a shy, insecure, and somewhat clumsy 11-year-old boy with a wild
imagination and a big heart. He owns a very expressive dog named Porkchop. Doug is a 6th Grade
average kid who always tries to do the right thing, despite his frequent failures. He is naive at
times, and fears that people will think of him as a loser.
Adapted from https://doug.fandom.com/wiki/Doug_Funnie.

Douglas Yancey, "Doug" Fannie, é o personagem titular e principal protagonista da série de


animação Doug. Douglas é um menino de 11 anos tímido, inseguro e um tanto desajeitado com
uma imaginação selvagem e um grande coração. Ele tem um cachorro muito expressivo chamado
Porkchop. Doug é um garoto médio da 6ª série que sempre tenta fazer a coisa certa, apesar de
suas falhas frequentes. Ele é ingênuo às vezes e teme que as pessoas pensem que ele é um
perdedor.

An e-mail to Mary
Hi, Mary!
How are you? I miss you so much!
I arrived in Rome last week and I’m having a lot of fun. This morning my parents and I woke up
at 8:00 and had a great breakfast at the hotel. Afterwards, I walked a little and saw many
tourists (many from Russia and China) and even some famous people!
Now we are going to Venice.
How are things in Las Vegas? Come meet us!
See you soon!
Paul.

AULA 02 – VERBS IN TEXTS 158


TEACHER ANDREA BELO

Um e-mail para Mary


Oi Mary!
Como você está? Sinto muito sua falta!
Cheguei a Roma na semana passada e estou me divertindo muito. Esta manhã meus pais e eu
acordamos às 8:00 e tomamos um ótimo café da manhã no hotel. Depois, caminhei um pouco e
vi muitos turistas (muitos da Rússia e da China) e até alguns famosos!
Agora vamos para Veneza.
Como estão as coisas em Las Vegas? Venha nos conhecer!
Vejo você em breve!
Paulo.

The Lazy Song – Bruno Mars


Today I don’t feel like doing anything
I just wanna lay in my bed
Don’t feel like picking up my phone
So leave a message at the tone
‘Cause today I swear I’m not doing anything
https://www.vagalume.com.br/thelazysong-11/brunomars.html

A música do preguiçoso – Bruno Mars


Hoje não tenho vontade de fazer nada
Eu quero apenas deitar em minha cama
Não estou com vontade de pegar meu telefone
Então deixe uma mensagem após o sinal
Porque hoje eu juro que não vou fazer nada

Rude
Can I have your daughter for the rest of my life? Say yes, say yes
‘Cause I need to know
You say I’ll never get your blessing till the day I die
Tough luck my friend but the answer is no!

AULA 02 – VERBS IN TEXTS 159


TEACHER ANDREA BELO

Why you gotta be so rude?


Don’t you know I’m human too
Why you gotta be so rude
I’m gonna marry her anyway
(Marry that girl) Marry her anyway
(Marry that girl) Yeah no matter what you say
(Marry that girl) And we’ll be a family
https://www.vagalume.com.br/magie-11/rude.html

Grosseiro
Posso ter a sua filha pelo resto da minha vida? Diga sim diga sim
Porque eu preciso saber
Você diz que eu nunca vou receber sua bênção até o dia que eu morrer
Que azar meu amigo, mas a resposta é não!
Por que você tem que ser tão rude?
Você não sabe que eu também sou humano
Por que você tem que ser tão rude
Eu vou casar com ela de qualquer maneira
(Case com aquela garota) Case com ela mesmo assim
(Case com aquela garota) Sim, não importa o que você diga
(Case com aquela garota) E nós seremos uma família

All of me
John Legend
‘Cause all of me
Loves all of you
Love your curves and all your edges
All your perfect imperfections
Give your all to me
I’ll give my all to me
You’re my end and my beginning
Even when I lose I’m winning

AULA 02 – VERBS IN TEXTS 160


TEACHER ANDREA BELO

‘Cause I give you all of me


And you give me all of you (…)

Tudo de mim
John Legend
'Porque tudo de mim
Ama tudo de você
Ame suas curvas e todas as suas arestas
Todas as suas perfeitas imperfeições
Dê tudo de si para mim
Eu vou dar tudo de mim para mim
Você é meu fim e meu começo
Mesmo quando eu perco, estou ganhando
Porque eu te dou tudo de mim
E você me dá tudo de você (...)

Nasa SpaceX crew return: Dragon capsule splashes down


Two American astronauts have splashed down, as the first commercial crewed mission to the
International Space
Station returned to Earth.
The SpaceX Dragon Capsule carrying Doug Hurley and Bob Behnken came down in the Gulf of
Mexico just south of Pensacola on Florida's Gulf coast.
They have now emerged from the capsule on to a recovery vessel.
The touchdown marks the first crewed US water landing since the final Apollo moon mission 45
years ago.
Hurley's and Behnken's capsule touched the water at about 14:48 EDT (19:48 BST; 18:48 GMT).
"It's truly our honour and privilege," said Hurley as they arrived.
"On behalf of the SpaceX and Nasa teams, welcome back to Planet Earth. Thanks for flying
SpaceX," the SpaceX mission control said just after splashdown.
President Donald Trump - who attended the capsule's launch two months ago - hailed its safe
return.
"Thank you to all!" he tweeted. "Great to have NASA Astronauts return to Earth after very
successful two month mission."
The successful end to the crew's mission initiates a new era for the American space agency.

AULA 02 – VERBS IN TEXTS 161


TEACHER ANDREA BELO

All its human transport needs just above the Earth will in future be purchased from private
companies, such as SpaceX.
The government agency says contracting out to service providers in this way will save it billions of
dollars that can be diverted to getting astronauts to the Moon and Mars.
The Dragon capsule launched to the space station at the end of May on a Falcon 9 rocket, also
supplied by SpaceX.
Hurley's and Behnken's mission served as an end-to-end demonstration of the astronaut "taxi
service" the company, owned by tech entrepreneur Elon Musk, will be selling to Nasa from now
on.
The Boeing corporation is also developing a crew capsule solution but has had to delay its
introduction after encountering software problems on its Starliner capsule.
The sight of the vehicle's four main parachutes floating down over the Gulf of Mexico was
confirmation the spacecraft had survived its fiery descent through the atmosphere.
The parachutes then slowed the capsule from about 350mph (560km/h) to just roughly 15mph at
splashdown.
Rigging was then used to hoist the capsule out of the water and on to the recovery vessel.
Technicians monitored "remnant vapours" around the spacecraft before the hatch was opened.
The men were to be checked over by medical staff before being flown to shore by helicopter.
The astronauts' Dragon capsule launched to the space station at the end of May on a Falcon 9
rocket, also supplied by SpaceX.
It will now be refurbished to fly again next year.
Nasa Administrator Jim Bridenstine lauded the efforts of everyone involved in Hurley's and
Behnken's mission, and then spoke of his agency's shift in philosophy.
"We don't want to purchase, own and operate the hardware the way we used to," he said.
"We want to be one customer of many customers in a very robust commercial marketplace in
low-Earth orbit. But we also want to have numerous providers that are competing against each
other on cost and innovation and safety, and really create this virtuous cycle of economic
development and capability."
Adapted from https://www.bbc.com/news/science-environment-53621102

Retorno da tripulação da Nasa SpaceX: cápsula do dragão cai


Dois astronautas americanos pousaram na água, como a primeira missão tripulada comercial
ao Espaço Internacional
A estação voltou para a Terra.
A SpaceX Dragon Capsule carregando Doug Hurley e Bob Behnken desceu no Golfo do México, ao
sul de Pensacola, na costa do Golfo da Flórida.
Eles agora emergiram da cápsula para um recipiente de recuperação.

AULA 02 – VERBS IN TEXTS 162


TEACHER ANDREA BELO

O touchdown marca o primeiro pouso na água dos Estados Unidos com tripulação desde a missão
lunar final da Apollo, há 45 anos.
A cápsula de Hurley e Behnken tocou a água por volta das 14:48 EDT (19:48 BST; 18:48 GMT).
"É realmente nossa honra e privilégio", disse Hurley quando eles chegaram.
"Em nome das equipes SpaceX e Nasa, bem-vindos de volta ao Planeta Terra. Obrigado por voar
na SpaceX", disse o controle da missão SpaceX logo após o splashdown.
O presidente Donald Trump - que compareceu ao lançamento da cápsula há dois meses - saudou
seu retorno seguro.
"Obrigado a todos!" ele twittou. "É ótimo ter os astronautas da NASA retornando à Terra após
uma missão de dois meses de muito sucesso."
O fim bem-sucedido da missão da tripulação inicia uma nova era para a agência espacial
americana.
Todas as suas necessidades de transporte humano logo acima da Terra serão no futuro adquiridas
de empresas privadas, como a SpaceX.
A agência governamental afirma que a contratação de provedores de serviços dessa forma
economizará bilhões de dólares, que podem ser desviados para levar astronautas à Lua e Marte.
A cápsula Dragon foi lançada na estação espacial no final de maio em um foguete Falcon 9,
também fornecido pela SpaceX.
A missão de Hurley e Behnken serviu como uma demonstração ponta a ponta do astronauta
"serviço de táxi" que a empresa, de propriedade do empresário de tecnologia Elon Musk, venderá
para a Nasa a partir de agora.
A Boeing Corporation também está desenvolvendo uma solução de cápsula da tripulação, mas
teve que atrasar sua introdução depois de encontrar problemas de software em sua cápsula
Starliner.
A visão dos quatro paraquedas principais do veículo flutuando sobre o Golfo do México foi a
confirmação de que a espaçonave havia sobrevivido à sua descida violenta pela atmosfera.
Os paraquedas, então, reduziram a velocidade da cápsula de cerca de 350 mph (560 km / h) para
apenas cerca de 15 mph no respingo.
Rigging foi então usado para içar a cápsula para fora da água e para o vaso de recuperação. Os
técnicos monitoraram os "vapores remanescentes" ao redor da espaçonave antes que a escotilha
fosse aberta.
Os homens deveriam ser examinados pela equipe médica antes de serem levados de helicóptero
para a costa.
A cápsula Dragon dos astronautas foi lançada na estação espacial no final de maio em um foguete
Falcon 9, também fornecido pela SpaceX.
Agora será reformado para voar novamente no próximo ano.
O administrador da Nasa, Jim Bridenstine, elogiou os esforços de todos os envolvidos na missão
de Hurley e Behnken e, em seguida, falou sobre a mudança de filosofia de sua agência.

AULA 02 – VERBS IN TEXTS 163


TEACHER ANDREA BELO

"Não queremos comprar, possuir e operar o hardware da maneira que costumávamos fazer",
disse ele.
"Queremos ser um cliente de muitos clientes em um mercado comercial muito robusto em órbita
baixa da Terra. Mas também queremos ter vários fornecedores que competem entre si em custo,
inovação e segurança, e realmente criam este ciclo virtuoso de desenvolvimento econômico e
capacidade. "

Saving the giant panda is one of the big success stories of conservation.
Decades of efforts to create protected habitat for the iconic mammal has pulled it back from the
brink of extinction. But, according to a new study, while many other animals ______ the same
landscape have benefited from this conservation work, some have lost out. Leopards, snow
leopards, wolves and Asian wild dogs have almost disappeared from the majority of protected
areas. Driven _____ near extinction ______ logging, poaching and disease, their loss could lead to
"major shifts, even collapse, in ecosystems", said researchers in China.
Adapted from https://www.msn.com/en-us/health/other/if-people-are-staying-home-why-is-coronavirus-still-spreading/ar-BB13HSW8?ocid=bingcovid

Salvar o panda gigante é uma das grandes histórias de sucesso da conservação.


Décadas de esforços para criar um habitat protegido para o mamífero icônico o tirou da beira da
extinção. Mas, de acordo com um novo estudo, embora muitos outros animais ______ da mesma
paisagem tenham se beneficiado desse trabalho de conservação, alguns perderam. Leopardos,
leopardos das neves, lobos e cães selvagens asiáticos quase desapareceram da maioria das áreas
protegidas. Levada _____ à beira da extinção ______ extração de madeira, caça ilegal e doenças,
sua perda pode levar a "grandes mudanças, até mesmo colapso, nos ecossistemas", disseram
pesquisadores na China.

Finding _____ effective coronavirus vaccine ______ a global priority in ending the pandemic. US
government leaders have put forward the ambitious timeline ______ have one by the end of
2020. It typically takes several years to develop ______ vaccine.
https://www.bbc.com/news/science-environment-53616593

Encontrar _____ vacina eficaz contra o coronavírus ______ uma prioridade global para acabar
com a pandemia. Os líderes do governo dos EUA apresentaram um cronograma ambicioso ______
têm um até o final de 2020. Normalmente leva vários anos para desenvolver ______ vacina.

AULA 02 – VERBS IN TEXTS 164


TEACHER ANDREA BELO

A holistic view of ecosystem management __________ "critically needed to better increase the
resilience and sustainability of the ecosystems not only for giant pandas but also for other wild
species", said Dr Sheng Li of Peking University in Beijing. Giant pandas are __________ as living
proof that conservation works. Their numbers in the wild are finally rebounding after years of
decline, and in 2016 they __________ upgraded from "endangered" to "vulnerable" on the official
extinction Red List.
Adapted from https://www.bbc.com/news/science-environment-53616593

Uma visão holística da gestão de ecossistemas __________ "extremamente necessária para


aumentar a resiliência e sustentabilidade dos ecossistemas não apenas para pandas gigantes, mas
também para outras espécies selvagens", disse o Dr. Sheng Li, da Universidade de Pequim, em
Pequim. Os pandas gigantes são __________ uma prova viva de que a conservação funciona. Seus
números na natureza estão finalmente se recuperando após anos de declínio, e em 2016 eles
__________ passaram de "em perigo" para "vulneráveis" na Lista Vermelha de extinção oficial.

The WHO has redefined burnout as a syndrome linked to chronic work stress. There’s a
difference between a busy workload and something more serious, writes Zaria Gorvett.
If you said you were suffering from ‘burnout’ in the early 1970s, you might have raised some
eyebrows.
At the time, the term was used informally to describe the side effects that heavy drug users
experienced: the general dimming of the mental faculties, for example, as was the case with many
a party animal. However, when German-American psychologist Herbert Freudenberger first
recognised the problem of burnout in New York City in 1974, at a clinic for addicts and homeless
people, Freudenberger wasn’t thinking of drug users.
The clinic’s volunteers were actually struggling, too: their work was intense, and many were
beginning to feel demotivated and emotionally drained. Though they had once found their jobs
rewarding, they had become cynical and depressed; they weren’t giving their patients the
attention they deserved. Freudenberger defined this alarming new condition as a state of
exhaustion caused by prolonged overwork – and borrowed the term ‘burnout’ to describe it.
Its popularity was explosive, and today burnout is a global phenomenon. Although statistics on
the prevalence of burnout specifically are hard to come by, 595,000 people in the UK alone
suffered from workplace stress in 2018.
Sportspeople get it. YouTube stars get it. Entrepreneurs get it. Freudenberger himself eventually
got it. Late last month, the World Health Organization (WHO) announced that the trendy problem
will be recognised in the latest International Classification of Diseases manual, where it is
described as a syndrome “resulting from chronic workplace stress that has not been successfully
managed”.
According to the WHO, burnout has three elements: feelings of exhaustion, mental detachment
from one’s job and poorer performance at work. But waiting until you’re already fully burned out

AULA 02 – VERBS IN TEXTS 165


TEACHER ANDREA BELO

to do something about it doesn’t help at all –and you wouldn’t wait to treat any other illness until
it was too late.
Feeling the burn
So how can you tell if you’re almost – but not quite – burned out?
“A lot of the signs and symptoms of pre-burnout would be very similar to depression,” says
Siobhán Murray, a psychotherapist based in County Dublin, Ireland, and the author of a book
about burnout, The Burnout Solution. Murray suggests looking out for creeping bad habits, such
as increased alcohol consumpution and relying on sugar to get you through the day. Also watch
out for feelings of tiredness that won’t go away. “So that even if you do sleep well, by 10 in the
morning you’re already counting down the hours to bed. Or not having the energy to exercise or
go for a walk.”
As soon as you begin to feel this way, Murray advises going to see your doctor.
“Depression and pre-burnout are very similar, but as much as there was a lot of enthusiasm
recently that burnout has now become a medical condition, it is still not – it is still classified as an
occupational phenomenon.” It’s important to get help from a medical professional who can
distinguish between the two, because although there are many treatment options for depression,
burnout is still best tackled by making lifestyle changes.
And how do you know if you’re really on the cusp of burnout, or just going through a challenging
month? “Stress is really important, and anxiety is what motivates us to do well,” says Murray. “It’s
when we’re continually exposed to stress and anxiety, that we’re not letting go, that it starts to
turn into burnout.”
Take that big project you’ve been working on. It’s normal to feel a kick of adrenaline when you
think about it, and maybe it’s kept you up at night. But, Murray suggests, if you still feel restless
once it’s over, it’s time to consider if you’re at risk of burnout. “It’s when you’re bringing that with
you into the next stage of your day, and adding to it continually,” she says.
Another classic sign of inching closer to burnout is cynicism: feeling like your work has little value,
avoiding social commitments and becoming more susceptible to disappointment.
“Someone on the brink will probably begin to feel emotionally numbed or mentally distant,” says
Jacky Francis Walker, a psychotherapist based in London who specializes in burnout. “Like they
don’t have the capacity to engage as much in the ordinary things of life.”
She also recommends looking for the final tell-tale sign of burnout, which is the unshakeable
feeling that the quality of your work is beginning to slip. “People say ‘but this isn’t me!’, ‘I’m not
like this’, ‘I can usually do x,y and z’. But obviously if they are in a state of physical depletion, then
they aren’t in their normal range of capabilities,” says Walker.
If this seems less than scientific, look to the Maslach Burnout Inventory (MBI), a test designed to
measure burnout. The most widely used is the MBI-General Survey, which measures things like
exhaustion, cynicism, and some how well you think you’re doing at work.
First published in 1981, it has been cited hundreds of times in studies since. Although it’s typically
used to measure burnout once it's in full swing, there’s no reason you can't apply it to see if you’re
getting close.
(Adapted from https://www.bbc.com/worklife/article/20190610-how-to-tell-if-youve-got-pre-burnout)

AULA 02 – VERBS IN TEXTS 166


TEACHER ANDREA BELO

A OMS redefiniu o burnout como uma síndrome ligada ao estresse laboral crônico. Há uma
diferença entre uma carga de trabalho ocupada e algo mais sério, escreve Zaria Gorvett.
Se você disse que estava sofrendo de "esgotamento" no início dos anos 1970, pode ter levantado
alguns
sobrancelhas
Na época, o termo era usado informalmente para descrever os efeitos colaterais que os usuários
pesados de drogas experimentavam: o escurecimento geral das faculdades mentais, por exemplo,
como era o caso de muitos animais festeiros. No entanto, quando o psicólogo germano-
americano Herbert Freudenberger reconheceu pela primeira vez o problema do esgotamento na
cidade de Nova York em 1974, em uma clínica para viciados e sem-teto, Freudenberger não estava
pensando em usuários de drogas.
Os voluntários da clínica também estavam lutando: seu trabalho era intenso e muitos estavam
começando a se sentir desmotivados e emocionalmente esgotados. Embora antes achassem seus
empregos gratificantes, eles se tornaram cínicos e deprimidos; eles não estavam dando a seus
pacientes a atenção que eles mereciam. Freudenberger definiu esta nova condição alarmante
como um estado de exaustão causado por excesso de trabalho prolongado - e tomou emprestado
o termo "esgotamento" para descrevê-lo.
Sua popularidade foi explosiva e hoje o esgotamento é um fenômeno global. Embora as
estatísticas sobre a prevalência de burnout especificamente sejam difíceis de obter, 595.000
pessoas só no Reino Unido sofreram de estresse no local de trabalho em 2018.
Os desportistas entendem. As estrelas do YouTube entendem. Os empresários entendem. O
próprio Freudenberger finalmente entendeu. No final do mês passado, a Organização Mundial da
Saúde (OMS) anunciou que o problema da moda será reconhecido no mais recente manual de
Classificação Internacional de Doenças, onde é descrito como uma síndrome “resultante de
estresse crônico no local de trabalho que não foi administrado com sucesso”.
De acordo com a OMS, o burnout tem três elementos: sensação de exaustão, desligamento
mental do trabalho e pior desempenho no trabalho. Mas esperar até que você já esteja
totalmente exausto para fazer algo sobre isso não ajuda em nada - e você não esperaria para
tratar qualquer outra doença até que fosse tarde demais.
Sentindo a queimadura
Então, como você pode saber se você está quase - mas não totalmente - esgotado?
“Muitos dos sinais e sintomas do pré-burnout seriam muito semelhantes à depressão”, diz
Siobhán Murray, psicoterapeuta residente em County Dublin, Irlanda, e autor de um livro sobre
burnout, The Burnout Solution. Murray sugere que você tome cuidado com os hábitos ruins, como
o aumento do consumo de álcool e depender do açúcar para o dia. Também esteja atento para a
sensação de cansaço que não passa. “Para que, mesmo que durma bem, às 10 da manhã você já
estará contando as horas para dormir. Ou não ter energia para fazer exercícios ou dar um
passeio”.
Assim que você começar a se sentir assim, Murray o aconselha a consultar seu médico.
“Depressão e pré-esgotamento são muito semelhantes, mas por mais que tenha havido muito
entusiasmo recentemente de que o esgotamento se tornou uma condição médica, ainda não é -
ainda é classificado como um fenômeno ocupacional.” É importante obter ajuda de um

AULA 02 – VERBS IN TEXTS 167


TEACHER ANDREA BELO

profissional médico que possa distinguir entre os dois, porque embora existam muitas opções de
tratamento para a depressão, o burnout ainda é melhor abordado com mudanças no estilo de
vida.
E como saber se você está realmente à beira do esgotamento ou apenas passando por um mês
desafiador? “O estresse é muito importante e a ansiedade é o que nos motiva a nos sair bem”,
diz Murray. “É quando estamos continuamente expostos ao estresse e ansiedade, que não
estamos relaxando, que começa a se transformar em esgotamento.”
Pegue aquele grande projeto em que você está trabalhando. É normal sentir uma descarga de
adrenalina quando você pensa sobre isso, e talvez isso o mantenha acordado à noite. Mas, Murray
sugere, se você ainda se sentir inquieto depois que acabar, é hora de considerar se você corre o
risco de esgotamento. “É quando você está trazendo isso com você para a próxima fase do seu
dia e aumentando continuamente”, diz ela.
Outro sinal clássico de se aproximar do esgotamento é o cinismo: sentir que seu trabalho tem
pouco valor, evitando compromissos sociais e se tornando mais suscetível a decepções.
“Alguém à beira do abismo provavelmente começará a se sentir emocionalmente entorpecido ou
mentalmente distante”, diz Jacky Francis Walker, psicoterapeuta de Londres especializado em
burnout. “Como se eles não tivessem a capacidade de se envolver tanto nas coisas comuns da
vida.”
Ela também recomenda procurar o sinal revelador final de esgotamento, que é a sensação
inabalável de que a qualidade do seu trabalho está começando a cair. “As pessoas dizem 'mas
este não sou eu!', 'Não sou assim', 'Normalmente consigo fazer x, y e z'. Mas, obviamente, se eles
estão em um estado de esgotamento físico, então eles não estão em sua gama normal de
capacidades”, diz Walker.
Se isso parece menos do que científico, consulte o Maslach Burnout Inventory (MBI), um teste
projetado para medir o burnout. O mais amplamente utilizado é o MBI-General Survey, que mede
coisas como exaustão, cinismo e alguns e quão bem você acha que está indo no trabalho.
Publicado pela primeira vez em 1981, ele foi citado centenas de vezes em estudos desde então.
Embora seja normalmente usado para medir o desgaste, uma vez que está em pleno andamento,
não há razão para que você não possa aplicá-lo para ver se você está chegando perto.

August 08, 2020


By Lise Alves, Senior Contributing Reporter
RIO DE JANEIRO, BRAZIL - In two months, researchers from the Alberto Luiz Coimbra Institute of
Post-Graduate Studies and Engineering Research at the Federal University of Rio de Janeiro
(Coppe/UFRJ), the National Institute of Metrology, Quality and Technology (INMETRO) and the
Pontifical Catholic University of Rio de Janeiro (PUC Rio) will introduce an antiviral mask fabric
that provides greater protection against Covid-19 to healthcare professionals. The announcement
was made yesterday, August 7th, by Professor Renata Simão from the Nanotechnology
Engineering and Metallurgical and Materials Engineering Programs of Coppe.
(Adapted from https://riotimesonline.com)

AULA 02 – VERBS IN TEXTS 168


TEACHER ANDREA BELO

8 de agosto de 2020
Por Lise Alves, repórter sênior contribuinte
RIO DE JANEIRO, BRASIL - Em dois meses, pesquisadores do Instituto Alberto Luiz Coimbra de Pós-
Graduação e Pesquisa em Engenharia da Universidade Federal do Rio de Janeiro (Coppe / UFRJ),
do Instituto Nacional de Metrologia, Qualidade e Tecnologia (INMETRO) e a Pontifícia
Universidade Católica do Rio de Janeiro (PUC Rio) vai apresentar um tecido de máscara antiviral
que oferece maior proteção contra Covid-19 aos profissionais de saúde. O anúncio foi feito ontem,
7 de agosto, pela professora Renata Simão, dos Programas de Engenharia Nanotecnologia e
Engenharia Metalúrgica e de Materiais da Coppe.

August 08, 2020


RIO DE JANEIRO, BRAZIL - On Saturday, August 8th, Brazil reached the number of 100,477 deaths
from Covid-19 since the start of the novel coronavirus pandemic. According to the Ministry of
Health, 905 deaths were recorded over the last 24 hours.
Of the 3,012,412 cases of people infected by the novel coronavirus, 2,094,293 (69.5 percent) have
recovered.
A total of 49,970 new cases have been reported by Health Secretariats since Friday, August 7th.
The official also shows that the number of people under follow-up stands at 817,642 (21.1
percent).
(Adapted from https://riotimesonline.com)

8 de agosto de 2020
RIO DE JANEIRO, BRASIL - No sábado, 8 de agosto, o Brasil atingiu o número de 100.477 mortes
por Covid-19 desde o início da nova pandemia de coronavírus. De acordo com o Ministério da
Saúde, foram registrados 905 óbitos nas últimas 24 horas.
Dos 3.012.412 casos de pessoas infectadas pelo novo coronavírus, 2.094.293 (69,5 por cento) se
recuperaram.
Desde sexta-feira, 7 de agosto, foram notificados 49.970 casos novos pelas Secretarias de Saúde.
O responsável também mostra que o número de pessoas em acompanhamento é de 817.642
(21,1 por cento).

What Is Stress?
First, an answer to __________ question, 'What is stress?' __________ term 'stress' refers
__________ the response you have when facing circumstances that force you __________ act,
change, or adjust in some way to maintain your footing or to keep things balanced. (The
circumstances themselves are known as 'stressors', but we'll have more on them later.)
(Adapted from https://www.verywellmind.com/chronic-stress-definition-management-tips-3145241)

AULA 02 – VERBS IN TEXTS 169


TEACHER ANDREA BELO

Primeiro, uma resposta à pergunta __________, 'O que é estresse?' __________ termo 'estresse'
refere-se __________ a resposta que você tem quando enfrenta circunstâncias que o forçam
__________ a agir, mudar ou ajustar de alguma forma para manter o equilíbrio ou manter o
equilíbrio. (As próprias circunstâncias são conhecidas como 'estressantes', mas falaremos mais
sobre elas mais tarde.)

While quick stress relievers aren't a complete stress management plan in themselves, they are an
excellent first line of defense against the effects of chronic stress. This is because they can help
you turn ______ your body's stress response and respond ______ the stressors you face from a
calm (or calmer), more relaxed place. This helps you to more effectively deal ______ what is
stress, and keep yourself healthier ______ the same time. Common strategies for quick stress
relief include breathing exercises or the use of humor.
(Adapted from https://www.verywellmind.com/chronic-stress-definition-management-tips-3145241)

Embora os apaziguadores rápidos do estresse não sejam um plano completo de gerenciamento


do estresse em si, eles são uma excelente primeira linha de defesa contra os efeitos do estresse
crônico. Isso porque eles podem ajudá-lo a mudar ______ a resposta do seu corpo ao estresse e
a responder ______ aos estressores que você enfrenta de um lugar calmo (ou mais calmo), mais
relaxado. Isso ajuda você a lidar com mais eficiência ______ o que é estresse e a se manter mais
saudável ______ ao mesmo tempo. As estratégias comuns para o alívio rápido do estresse
incluem exercícios respiratórios ou o uso do humor.

Malaria 'completely stopped' by microbe


The team in Kenya and the UK say the finding has "enormous potential" to control the disease.
Malaria is spread by the bite of infected mosquitoes, so protecting them could in turn protect
people. The researchers are now investigating whether they can release infected mosquitoes into
the wild, or use spores to suppress the disease.
The malaria-blocking bug, Microsporidia MB, was discovered by studying mosquitoes on the
shores of Lake Victoria in Kenya. It lives in the gut and genitals of the insects. The researchers
could not find a single mosquito carrying the Microsporidia that was harbouring the malaria
parasite. And lab experiments confirmed the microbe gave the mosquitoes protection.
Microsporidias are fungi, or at least closely related to them, and most are parasites. However, this
new species may be beneficial to the mosquito and was naturally found in around 5% of the
insects studied.
"The data we have so far suggest it is 100% blockage, it's a very severe blockage of malaria," Dr
Jeremy Herren, from the International Centre of Insect Physiology and Ecology in Kenya told the
BBC. He added: "It will come as a quite a surprise. I think people will find that a real big
breakthrough. While huge progress has been made through the use of bed nets and spraying
homes with insecticide, it is widely agreed new tools are needed to tackle malaria.

AULA 02 – VERBS IN TEXTS 170


TEACHER ANDREA BELO

How does the microbe stop malaria? Microsporidia MB could be priming the mosquito's immune
system, so it is more able to fight off infections. Or the presence of the microbe in the insect could
be having a profound effect on the mosquito's metabolism, making it inhospitable for the malaria
parasite.
Adapted from https://www.bbc.com/news/health-52530828

Malária 'completamente parada' por micróbio


A equipe no Quênia e no Reino Unido afirma que a descoberta tem "enorme potencial" para
controlar a doença. A malária é transmitida pela picada de mosquitos infectados, portanto,
protegê-los pode, por sua vez, proteger as pessoas. Os pesquisadores agora estão investigando
se podem liberar mosquitos infectados na natureza ou usar esporos para suprimir a doença.
O vírus bloqueador da malária, Microsporidia MB, foi descoberto estudando mosquitos nas
margens do Lago Vitória, no Quênia. Ele vive no intestino e nos órgãos genitais dos insetos. Os
pesquisadores não conseguiram encontrar um único mosquito portador do Microsporidia que
abrigava o parasita da malária. E experimentos de laboratório confirmaram que o micróbio deu
proteção aos mosquitos. Microsporidias são fungos, ou pelo menos intimamente relacionados a
eles, e a maioria são parasitas. No entanto, essa nova espécie pode ser benéfica para o mosquito
e foi encontrada naturalmente em cerca de 5% dos insetos estudados.
"Os dados que temos até agora sugerem que é um bloqueio de 100%, é um bloqueio muito grave
da malária", disse o Dr. Jeremy Herren, do Centro Internacional de Fisiologia e Ecologia de Insetos
do Quênia, à BBC. Ele acrescentou: "Será uma grande surpresa. Acho que as pessoas vão achar
isso um grande avanço. Embora um grande progresso tenha sido feito com o uso de mosquiteiros
e pulverização de inseticida nas residências, é amplamente aceito que novas ferramentas são
necessárias para combater a malária.
Como o micróbio impede a malária? Microsporidia MB pode estar preparando o sistema
imunológico do mosquito, portanto, é mais capaz de combater infecções. Ou a presença do
micróbio no inseto pode estar tendo um efeito profundo no metabolismo do mosquito, tornando-
o inóspito para o parasita da malária.

(Título omitido propositadamente)


Hoping to stem the torrent of false cures and conspiracy theories about COVID-19, Facebook
announced Thursday it would begin informing users globally who have liked, commented on, or
shared “harmful” misinformation about the coronavirus, pointing them instead in the direction
of a reliable source. Facebook hopes the move will drastically reduce the spread of false
information about the coronavirus online, a growing crisis that the World Health Organization
(WHO) has described as an “infodemic.”
“We want to connect people who may have interacted with harmful misinformation about the
virus with the truth from authoritative sources in case they see or hear these claims again,” said
Guy Rosen, Facebook’s Vice President for Integrity, in a blog post published early Thursday.

AULA 02 – VERBS IN TEXTS 171


TEACHER ANDREA BELO

The new policy only applies to false claims related to the coronavirus, but campaigners say the
announcement could lay the groundwork for a breakthrough in the battle against political
disinformation online. “Facebook applying this to the pandemic is a good first step but this should
also be applied to political disinformation too, particularly with the 2020 U.S. election
approaching” says Fadi Quran, a campaign director at Avaaz, a global advocacy group that has
lobbied Facebook to “correct the record” on false information since 2018. (In the language of
online security, “disinformation” means the coordinated, purposeful spread of false information,
while “misinformation” refers to accidental inaccuracies.) “I hope this is going to be expanded to
other issues and fast,” Quran tells TIME.
Adapted from https://time.com/5822372/

(Título omitido propositadamente)


Na esperança de conter a torrente de curas falsas e teorias de conspiração sobre o COVID-19, o
Facebook anunciou na quinta-feira que começaria a informar os usuários globalmente que
gostaram, comentaram ou compartilharam informações errôneas "prejudiciais" sobre o
coronavírus, apontando-os na direção de um fonte confiável. O Facebook espera que a mudança
reduza drasticamente a disseminação de informações falsas sobre o coronavírus online, uma crise
crescente que a Organização Mundial da Saúde (OMS) descreveu como um “infodêmico”.
“Queremos conectar as pessoas que podem ter interagido com informações errôneas sobre o
vírus com a verdade de fontes confiáveis, caso vejam ou ouçam essas alegações novamente”,
disse Guy Rosen, vice-presidente de integridade do Facebook, em uma postagem de blog
publicada na quinta-feira. .
A nova política só se aplica a falsas alegações relacionadas ao coronavírus, mas os ativistas dizem
que o anúncio pode lançar as bases para um avanço na batalha contra a desinformação política
online. “A aplicação disso pelo Facebook à pandemia é um bom primeiro passo, mas também
deve ser aplicado à desinformação política, especialmente com a aproximação das eleições nos
EUA de 2020”, disse Fadi Quran, diretor de campanha da Avaaz, um grupo de defesa global que
pressionou o Facebook para “Corrigir o registro” de informações falsas desde 2018. (Na linguagem
da segurança online, “desinformação” significa a disseminação coordenada e proposital de
informações falsas, enquanto “desinformação” se refere a imprecisões acidentais.) “Espero que
assim seja expandido para outras questões e rápido ”, disse o Alcorão à TIME.

Leia o texto a seguir e responda às questões 07, 08 e 09.


The true cost of Saudi Arabia’s $500bn megacity
With a huge artificial moon, glow-in-the-dark beaches, flying drone-powered taxis, robotic butlers
__________ (1) clean the homes of residents and a Jurassic Park-style attraction featuring
animatronic lizards, Neom has been billed __________ (2) humanity’s next chapter. But beneath
the glitzy veneer lies a story of threats, forced eviction and bloodshed.
“The future has a new home,” proclaims the website, as the promotional video tracks colour-
tinted panoramic shots of picturesque desert expanses, and deep azure lagoons. The new city

AULA 02 – VERBS IN TEXTS 172


TEACHER ANDREA BELO

state of Neom, named __________ (3) a combination of the Greek word for “new” and the Arabic
term for “future”, is intended to cover an area the size of Belgium at the far north of Saudi Arabia’s
Red Sea coastline.
There has been no shortage of outlandish promises for the $500bn (£400bn) city-state. Yet part
of the site is the home of the Huwaitat tribe, who have spanned Saudi Arabia, Jordan, and the
Sinai Peninsula for generations, tracing their lineage back before the founding of the Saudi state.
At least 20,000 members of the tribe now face eviction due to the project, with no information
about where they will live in the future.
“For the Huwaitat tribe, Neom is being built on our blood, on our bones,” says Alia Hayel
Aboutiyah al-Huwaiti, an outspoken activist and member of the tribe. Construction is ploughing
ahead despite the challenges of the pandemic and the historic drop in the price of oil, Saudi’s
primary export. For many, though, Neom is less a shining vision of the future than a grim symbol
of Saudi human rights violations, underscored by the treatment of the Huwaitat tribe.
Adapted from https://www.theguardian.com/global-development/2020/may/04/its-being-built-on-our-blood-the-true-cost-of-saudi-arabia-5bn-mega-city-neom

O verdadeiro custo da megacidade de US $ 500 bilhões da Arábia Saudita


Com uma enorme lua artificial, praias que brilham no escuro, táxis com drones voadores,
mordomos robóticos __________ (1) limpam as casas dos residentes e uma atração no estilo
Jurassic Park com lagartos animatrônicos, Neom foi faturado __________ (2 ) próximo capítulo
da humanidade. Mas por trás do verniz chamativo está uma história de ameaças, despejo forçado
e derramamento de sangue.
“O futuro tem um novo lar”, proclama o site, enquanto o vídeo promocional rastreia fotos
panorâmicas coloridas de extensões de deserto pitorescas e lagoas azuis profundas. A nova
cidade-estado de Neom, denominada __________ (3) uma combinação da palavra grega para
"novo" e do termo árabe para "futuro", destina-se a cobrir uma área do tamanho da Bélgica no
extremo norte do Mar Vermelho da Arábia Saudita litoral.
Não faltaram promessas bizarras para a cidade-estado de US $ 500 bilhões (£ 400 bilhões). No
entanto, parte do local é o lar da tribo Huwaitat, que abrangeu a Arábia Saudita, a Jordânia e a
Península do Sinai por gerações, traçando sua linhagem antes da fundação do estado saudita. Pelo
menos 20.000 membros da tribo agora enfrentam despejo devido ao projeto, sem nenhuma
informação sobre onde viverão no futuro.
“Para a tribo Huwaitat, Neom está sendo construído em nosso sangue, em nossos ossos”, disse
Alia Hayel Aboutiyah al-Huwaiti, uma ativista declarada e membro da tribo. A construção está
avançando, apesar dos desafios da pandemia e da queda histórica no preço do petróleo, principal
produto de exportação da Arábia Saudita. Para muitos, porém, Neom é menos uma visão
brilhante do futuro do que um símbolo sombrio das violações dos direitos humanos sauditas,
ressaltado pelo tratamento dispensado à tribo Huwaitat.

AULA 02 – VERBS IN TEXTS 173


TEACHER ANDREA BELO

How surviving a recession prepared me for Covid-19


I was 35 when I lost my job as a legal courier in San Jose, California. It was 2009, and the economy
had lost nearly six million jobs since the Great Recession began in 2007.
Finding work was nearly impossible. At one point, I juggled three jobs that paid a total of less than
$500 per week. It was barely enough to get by. For the next three to four years, I was mostly
unemployed, homeless — sleeping on benches, shelters and sometimes friends’ couches — and
living off the McDonald’s Dollar Menu.
Those were some dark times. But I’ve come a long way since then. In 2017, I moved to Tacoma,
Washington with my best friend Kim, who lent me just enough money to settle down and pursue
my dream of getting into real estate. You’re getting another shot, I told myself. Don’t blow it.
I didn’t. Eight months after I got my broker license, I closed my seventh deal — for a total of $1.6
million in sales. I paid back the money Kim loaned me. For the first time in my life, I had a feeling
of accomplishment and satisfaction. I got serious about saving money and started planning for
the future.
While the underlying cause of today’s economic slowdown is fundamentally different from the
Great Recession, the lessons I learned over the past decade still apply. And they’ve made me more
prepared to handle the hardships that have — and will — come from the coronavirus pandemic.
Adapted from https://www.cnbc.com/2020/04/23/i-was-unemployed-homeless-how-surviving-great-recession-prepared-me-for-coronavirus.html

Como sobreviver a uma recessão me preparou para a Covid-19


Eu tinha 35 anos quando perdi meu emprego como mensageiro legal em San Jose, Califórnia. Era
2009 e a economia havia perdido quase seis milhões de empregos desde o início da Grande
Recessão em 2007.
Encontrar trabalho era quase impossível. A certa altura, fiz malabarismos com três empregos que
pagavam um total de menos de US $ 500 por semana. Mal foi o suficiente para sobreviver. Pelos
próximos três a quatro anos, estive principalmente desempregado, sem-teto - dormindo em
bancos, abrigos e às vezes em sofás de amigos - e vivendo do McDonald’s Dollar Menu.
Aqueles foram alguns tempos sombrios. Mas eu já percorri um longo caminho desde então. Em
2017, me mudei para Tacoma, Washington, com minha melhor amiga Kim, que me emprestou
dinheiro suficiente para me estabelecer e realizar meu sonho de entrar no mercado imobiliário.
Você está recebendo outra chance, eu disse a mim mesma. Não estrague tudo.
Eu não fiz. Oito meses depois de obter minha licença de corretor, fechei meu sétimo negócio - por
um total de US $ 1,6 milhão em vendas. Devolvi o dinheiro que Kim me emprestou. Pela primeira
vez na vida, tive um sentimento de realização e satisfação. Comecei a economizar dinheiro e
comecei a planejar o futuro.
Embora a causa subjacente da desaceleração econômica de hoje seja fundamentalmente
diferente da Grande Recessão, as lições que aprendi na última década ainda se aplicam. E eles me
deixaram mais preparado para lidar com as dificuldades que vieram - e virão - com a pandemia
do coronavírus.

AULA 02 – VERBS IN TEXTS 174


AFA 2024

ARTICLES AND NOUNS

AULA 03

Teacher Andrea Belo

www.estrategiamilitares.com.br www.militares.estrategia.com
TEACHER ANDREA BELO

SUMÁRIO
INTRODUÇÃO 3

ARTIGO DEFINIDO THE 4

ARTIGOS INDEFINIDOS A E AN 6

SUBSTANTIVOS 8

CURIOSIDADES SOBRE ARTIGOS 24

CURIOSIDADES SOBRE SUBSTANTIVOS 25

NÚMEROS 26

QUESTÕES 28

GABARITO 55

QUESTÕES COMENTADAS 56

CONSIDERAÇÕES FINAIS 105

REFERÊNCIAS BIBLIOGRÁFICAS 106

TRADUÇÕES 108

AULA 03 – ARTICLES AND NOUNS 2


TEACHER ANDREA BELO

INTRODUÇÃO
Chegou a vez da nossa aula sobre artigos e também sobre substantivos.
Mesmo que haja um vasto conhecimento sobre as regras de gramática em Inglês, sem os
artigos e os substantivos, a comunicação seria impossível.
Isso porque, etimologicamente, substantivo significa, literalmente, aquilo que está
debaixo, a base.
E, se os artigos vêm antes dos substantivos, determinando-os, ambos são fundamentais
aos textos.
Vamos falar um pouco de artigos. Quando eu explico o uso dos artigos em Inglês, sempre
gosto de lembrar que, o artigo definido THE, por exemplo, está na lista das palavras mais usadas
na língua inglesa.
Então, aprender a usar o the e os outros artigos, de forma correta, é de grande importância
para melhorar seu vocabulário em Inglês.
Os artigos estão em todo lugar, em todos as frases dos textos e inseridos nos contextos.
Por isso, é essencial que você tenha conhecimento de como usar os artigos de forma correta,
mesmo sabendo que há regras, muitas regras.
Mas é assim também em Português e em todas as línguas e, saber mais sobre cada item,
que estará presente em sua prova, deixará você devidamente preparado.
Os artigos em Inglês, são classificados como definidos e indefinidos e colaboram
imensamente na interpretação dos textos, pois são os artigos que revelam, que mostram para
você se a palavra que foi mencionada no texto aparece no contexto pela primeira vez ou essa
palavra está retomando alguma ideia já expressada anteriormente, ajudando em sua
compreensão do texto como um todo. Em nossa aula, ficará fácil e claro para você.
O substantivo, por sua vez, é a classe de palavras que nomeia seres, objetos, lugares,
sentimentos, ou seja, denomina e classifica a maioria das palavras constituintes das frases que
usamos para nos comunicar.
A palavra substantivo é derivada da família de substância e substancial, que significam,
fundamental.
Nessa aula, estudaremos todos os artigos e todos os substantivos em Inglês, com exemplos
e explicações completas, já que esses termos estão entre as palavras mais importantes de um
idioma, pois estão presentes em nossos discursos, nos textos que escrevemos e lemos e em
nossos pensamentos quando queremos expressar opiniões.
Vamos estudá-los!

AULA 03 – ARTICLES AND NOUNS 3


TEACHER ANDREA BELO

ARTIGO DEFINIDO THE


Os artigos definidos que temos na língua portuguesa são: o, a, os e as. Em Inglês, eles são
representados por apenas uma palavra: o artigo definido the.
Definite Article, o artigo definido, é o termo usado para se referir a algo específico ou algo
que já foi mencionado anteriormente.
Como já afirmei, existe apenas um artigo definido em Inglês, the, e ele é utilizado diante
de substantivos tanto no singular quanto no plural, pois a única palavra the significa o, a, os e as.
Exemplos simples: The car is blue – O carro é azul. The cars are blue – Os carros são azuis.
The table is big – A mesa é grande. The tables are big – As mesas são grandes.
Observe também outro exemplo = The backpack is red – A mochila é vermelha – A cor
vermelha especifica o substantivo mochila (backpack). Assim, não se trata de uma mochila
qualquer, e sim da mochila vermelha, determinada pelo artigo definido the. Na maioria das vezes,
esse artigo é usado antes dos substantivos, para defini-los.
Vejamos outros casos do correto uso do artigo definido the.
Usamos o artigo definido antes de substantivos já mencionados, como por exemplo: Tom
wrote a book. The book is about art – Tom escreveu um livro. O livro é sobre artes. O artigo the
foi usado para determinar o livro que já havia sido mencionado: o livro que Tom escreveu.
Usamos o artigo definido também para substantivos considerados únicos em sua espécie
como a lua, o sol etc: the moon (a lua), the sun (o sol), the sky (o céu), the planet Earth (o planeta
Terra), the universe (o universo): The sun is shining – O sol está brilhando.
Outro caso em que usamos o artigo definido é quando se fala de algo que se remete a
nomes geográficos de um rio, um mar, um oceano, canais, polos, desertos, golfos, ilhas e
montanhas, veja: the Mississipi River, the Pacif ocean, the English Channel (O Canal da Mancha),
the South Pole (o polo Sul), the Sahara desert, the Gulf of Mexico, the Bahamas, the Alps etc.
Exemplo: The Pacific Ocean is very big. O oceano pacífico é muito grande.
Um caso interessante do uso do artigo definido é quando se fala de adjetivos que estão
sendo usados como substantivos no plural, como: the poor (os pobres), the powerful (os
poderosos), the strong (os fortes), the good (os bons), the bad (os maus) etc. Você deve estar
perguntando: e os substantivos no singular, como por exemplo, o poderoso? Como fica a frase?
Respondo a você que, precisa usar o nome, o substantivo, veja: The powerful man helped the boy
– O poderoso homem ajudou o garoto.
Usamos o artigo definido também para nomes compostos de países, como os Estados
Unidos, o Reino Unido, os Emirados Árabes e a República Dominicana: The United States, The
United Kingdom, The United Arab Emirates, The Dominican Republic. He lives in the USA – Ele
mora nos EUA, sendo in the = nos = em + os, o e artigo definido os = the.
Quando os nomes próprios indicam um grupo familiar, ou seja, o nome de uma família
inteira, usamos também o artigo definido: The Smiths went to the restaurant yesterday – Os Smith
(os membros da família Smith) foram ao restaurante ontem. É muito importante ressaltar que,
para nomes próprios em geral e nomes de pessoas, não se usa artigo definido, nunca.

AULA 03 – ARTICLES AND NOUNS 4


TEACHER ANDREA BELO

Apenas quando estiver se referindo a um nome próprio que já possuam the em sua
estrutura, como The Economist, The New York Times, The Beatles, The Washington Post, The
Empire States, The Louvre, The Kremlin, The Taj Mahal, The Vatican etc.
Então, ao dizer Susan is my friend – Susan é minha amiga, não podemos jamais escrever ou
falar "The Susan is my friend" – nunca, certo?
Usamos o artigo definido também quando se trata do grau superlativo em Inglês (teremos
uma aula destinada aos adjetivos, em que explicarei, com detalhes, os graus comparativo e
superlativo), como por exemplo: Bob is the tallest guy in our classroom – Bob é o cara mais alto
da nossa sala. Kathy is the shortest girl in our classroom – Kathy é a garota mais baixa da nossa
sala.
Agora vejamos os casos em que não devemos usar o artigo definido, como já citei acima,
por exemplo, em nomes próprios. Portanto, não se deve colocar the antes de nomes de cidades,
estados, países e continentes.
Para dizer que a Europa é um grande continente, ficaria: Europe is a big continent, sem o
artigo no início da frase.
E também não usamos o artigo definido the antes de nomes de disciplinas acadêmicas
esportes, ciências, cores, estações do ano, meses e dias da semana. Vejamos exemplos.
Tennis is a great sport. O tênis é um ótimo esporte.
Biology is an important science. A Biologia é uma ciência importante.
Yellow is Paul's favorite color. O amarelo é a cor favorita de Paulo.
Também não devemos usar o artigo definido para substantivos que estiverem no plural,
porém utilizados em sentido genérico, como por exemplo as pessoas (referindo-se a muitas
pessoas, a uma maioria de pessoas): People watch too much TV – As pessoas assistem muita
televisão, sem the para “as” pessoas.
Outro caso em que também não usamos o artigo definido é quando há substantivos
abstratos ou nomes que indicam algum tipo de material: Hope is stronger than fear – A esperança
é mais forte do que o medo. E Silk produces soft clothes – A seda produz roupas macias.
Devemos ficar atentos quando houver referência a cargos e títulos, pois, quando usados
sós na oração, usa-se o artigo definido. Mas, omite-se o artigo definido quando o nome próprio
estiver junto, veja: The president is on TV now – O presidente está na TV agora. Porém, veja essa
outra frase: President Kennedy was murdered – O presidente Kennedy foi assassinado.
Quando vamos construir frases no passado e futuro, usando last e next, não usamos o
artigo the antes deles, pois são expressões temporais: I will travel next month.
Eu vou viajar no próximo mês e não “in the next month” ou em referência ao passado last
month e nunca “in the last month”, isso não existe.
Agora, em complemento ao capítulo que você estudou sobre o artigo definido, vamos
estudar os artigos indefinidos a e an. Come on!

AULA 03 – ARTICLES AND NOUNS 5


TEACHER ANDREA BELO

ARTIGOS INDEFINIDOS A E AN
Os Indefinite articles – artigos indefinidos que temos na língua portuguesa: um, uma, uns,
umas, em Inglês, são representados por apenas duas palavras: a e an e são usados quando nos
referimos a algo em geral, algo não especificado. Eles apenas podem ser usados antes de
substantivos que estão no singular.
Muitas pessoas aprendem erroneamente que a se usa antes de palavras que se inicia com
consoante enquanto an se usa para palavras que se iniciam com vogal. Na verdade, essa regra
funciona quase sempre, mas os artigos indefinidos também dependem do som das letras iniciais
das palavras em que são empregados.
Por exemplo, dizer “um homem velho’ em Inglês é an old man, por causa da letra “o” no
início da palavra old. Mas, dizer “um homem honesto” é an honest man, por causa do som de “o”
da palavra honest. Apesar de começar com a letra “h”, essa letra não é pronunciada na palavra
honest, como na maioria em que o “h” possui o som de “r”, por exemplo, na palavra house. Se o
som for da consoante “r”, aplicamos o artigo indefinido a: He wants to buy a house (Ele quer
comprar uma casa).
Outros casos em que devemos analisar a fonética para usar o artigo correto, é em palavras
cuja vogal inicial tem o som de consoante, que geralmente começam com “u” e “eu”, mas tem o
som de “y”, que é consoante em Inglês: This is a university. (Essa é uma universidade), I am reading
a European Journal. (Estou lendo um jornal europeu.)
Apesar de iniciar com a vogal “o”, a palavra one tem o som da consoante “w” e, por isso,
também pede o uso do artigo indefinido a e não an: She lives in a one-story small house (Ela mora
em uma pequena casa de um andar).
Em regra geral, na maior parte das palavras que começam com vogais, usa-se o artigo
indefinido an e, por sua vez, o artigo indefinido a, é usado para palavras que iniciam com
consoante: She left an hour ago (Ela saiu há uma hora), He is a jornalist (Ele é um jornalista), I will
buy a new skirt (Eu comprarei uma saia nova), He will see an elephant at the zoo (Ele verá um
elefante no zoológico) etc.
O artigo a deve ser usado antes das palavras few e little quando têm sentido positivo,
como: She can see a few buildings far away (Ela consegue avistar alguns prédios de longe), como
se fosse “um pouco de prédios” e por isso a few, mas a melhor tradução é alguns prédios.
Outro exemplo: She wants just a few milk with coffee (Ela quer apenas um pouco de leite
com café).
Agora vejamos os casos em que não devemos usar os artigos indefinidos.
Como eu havia dito antes, os artigos indefinidos a e an não devem ser usados quando a
ideia principal da frase se refere a substantivos no plural: a e an não significam uns/umas nunca
e só podem ser usados exclusivamente no singular. Explicarei o que fazer com palavras no plural.
Vamos lá.
Não devemos usar artigos indefinidos antes de substantivos incontáveis (embora isso
aconteça em Português).

AULA 03 – ARTICLES AND NOUNS 6


TEACHER ANDREA BELO

Então, usamos some, como por exemplo: She gave me some advice (Ela me deu um
conselho) e como conselho em Inglês é incontável – advice – usamos some e não o artigo an.
Em um esquema simples usando os artigos definidos e indefinidos de forma resumida para
ajudar você a se lembrar das regras, poderíamos dizer que:
A: Usamos o artigo indefinido "A" quando as palavras possuem uma consoante como letra
inicial e, além da grafia, sua pronúncia também deve ser de consoante.
AN: Usamos o artigo indefinido "AN" quando as palavras possuem uma vogal como letra
inicial e, além da grafia, sua pronúncia deve ser de uma das vogais também.
THE: Usamos o artigo definido "THE" para palavras contáveis e incontáveis, femininas e
masculinas e, tanto para o singular como para o plural, já que apenas THE significa "o, a, os, as".

A AN
A + Consonant AN + Vowel
lamp apple
door elephant
a + house an + ice-cream
bag orange
tomato umbrella

Outra importante observação a fazer é sobre os artigos indefinidos “a e an”, também


sublinhados no trecho acima.
Poderia ser questionado se todos os artigos indefinidos do trecho acima estão
acompanhados de substantivos ou até mesmo, dos “nomes a que eles se referem” e, você poderia
se sentir confuso, já que esses artigos não determinam os substantivos.
Apesar de não determiná-los, eles acompanham sim, substantivos. Por exemplo, “a glance
away”, é um olhar distante.
Não se sabe de quem é o olhar, mas o artigo acompanhou o substantivo. Assim como “an
instant ago”, não se sabe qual foi o instante atrás a que se referem, mas foi um instante atrás,
que já passou e o artigo acompanhou o substantivo instante.
Agora, vamos aos substantivos, que tanto falamos que os artigos acompanham.
E, apesar de saber que são nomes, há características e dicas que fazem com que você os
identifique e facilite na hora das respostas às questões.
Let’s go!

AULA 03 – ARTICLES AND NOUNS 7


TEACHER ANDREA BELO

SUBSTANTIVOS
Os Nouns, substantivos em Inglês, existem para nomear os seres em geral, representados
por pessoas, lugares, instituições, grupos, elementos da natureza etc.
São aquelas palavras presentes em nossos pensamentos, em nossos discursos, na fala e na
escrita. Os substantivos integram as classes gramaticais e estão relacionados à formação e ao
significado de palavras.
De acordo com uma pesquisa da palavra substantivo, no Dicionário Etimológico,
percebemos que substantivo se origina se substância, parte essencial de alguma coisa. E,
conforme a Nova Gramática do Português Brasileiro, o termo substantivo significa literalmente o
que está debaixo, na base.
A definição de substantivo em Inglês – NOUN – em variados dicionários é praticamente a
mesma: “a noun is a kind of word that is usually the name of something such as a person, place,
thing, quality, or idea.” - um substantivo é um tipo de palavra que geralmente nomeia coisas, tais
como uma pessoa, lugar, qualidade ou ideia. Em Inglês, há mais substantivos do que qualquer
outra classe de palavras, sabia?
Vamos estudar, a parte fundamental dos textos que você vai ler e interpretar em sua prova
que são, de fato, os substantivos.
Os substantivos podem ser: common noun (substantivo comum), proper noun (substantivo
próprio), compound noun (substantivo composto), countable/uncountable nouns – substantivos
contáveis e incontáveis, com as variações de singular/plural (singular e plural) nos substantivos e
certas regras que estudaremos.
Veremos cada um deles com exemplos, já que há peculiaridades e regras que envolvem
todos os tipos de substantivos. Vamos começar pelo substantivo comum.

SUBSTANTIVO COMUM
Os common nouns (substantivos comuns) são aqueles que nomeiam seres da mesma
espécie, fazem referência a uma pessoa, lugar ou coisa de forma geral. São os mais presentes nas
provas, pois estão nos textos que você vai precisar ler no dia da prova.
São escritos com a letra inicial minúscula, exceto se iniciarem a frase. Exemplos: book
(livro), table (mesa), city (cidade) etc.

SUBSTANTIVOS PRÓPRIOS
Por sua vez, os proper nouns (substantivos próprios) são aqueles que representam o
nome de uma determinada pessoa, de uma entidade, de lugares em geral. Também aparecem
bastante nas provas, para nomear pessoas e lugares de referência nos textos variados.
Sempre devem ser escritos com letra maiúscula.

AULA 03 – ARTICLES AND NOUNS 8


TEACHER ANDREA BELO

Além de pessoas e lugares, são substantivos próprios, em Inglês, os dias da semana, dos
meses, de documentos históricos e de instituições.
Exemplos: Robert, Mississipi River, Cambridge University, Monday, January – Roberto, Rio
Mississippi, Universidade Cambridge, segunda-feira, janeiro.
Vamos fazer uma pequena comparação com exemplos para ficar clara a diferença entre
substantivos comuns e próprios, sendo que, as palavras da primeira coluna se relacionam com as
da segunda, assim: para cada exemplo de substantivo comum, há um próprio que o exemplifique,
como school (escola) em relação à Oxford University, que é um nome, um substantivo próprio
nomeando uma universidade, exemplo de escola. E assim por diante. Veja:

COMMON NOUNS PROPER NOUNS


(Substantivos comuns) (Substantivos próprios)
SCHOOL OXFORD
CAR CAMBRIDGE
TEA PORSHE
MAN FERRARI
WEBSITE EARL GREY
MOVIE PAUL SMITH
MOUNTAIN AVENGERS
CITY EVEREST
STATE SÃO PAULO
ANIMAL MICKEY MOUSE
OCEAN GARFIELD
HOUSE ATLANTIC OCEAN
TOWER EIFFEL TOWER
STATUE STATUE OF LIBERTY
RIVER MISSISSIPI RIVER

SUBSTANTIVO COMPOSTO
Quando temos a combinação de dois ou mais substantivos comuns ou próprios formando
um só, são compound nouns (substantivos compostos), tais como driving license (carteira de
habilitação) e washing machine (máquina de lavar roupas).
Às vezes não estão separados por itens, mas são compostos por duas palavras: bedroom
(quarto), motorcycle (motocicleta), haircut (corte de cabelo), policeman (policial ou agente de
polícia) etc.
Em substantivos compostos com preposição ou advérbio, pluralizamos o seu componente
principal (geralmente é a primeira palavra): sister-in-law – sisters-in-law (cunhadas), godmother

AULA 03 – ARTICLES AND NOUNS 9


TEACHER ANDREA BELO

– godmothers (madrinhas), maidservant – maidservants (criadas), runner-up – runners-up (vice-


campeões).
Vejamos uma lista com alguns substantivos compostos que já apareceram e podem
aparecer novamente nas provas:

COMPUND NOUNS
(Substantivos compostos)
ARMCHAIR INTAKE
AIRLINE LADYBUG
AIRPORT LIGHTHOUSE
BACKGROUND MOONLIGHT
BATHROOM MOTHER-IN-LAW
BUS STOP NOTEBOOK
CLASSMATE NEWSPAPER
CUPBOARD OUTDOOR
DAYDREAM OUTSIDE
DOWNSTAIRS ONLINE
EGGPLANT PARTNERSHIP
EVERYBODY PHOTOCOPY
FRIENDSHIP POPCORN
FOOTPRINT POLICEMAN
GENTLEMAN PETSHOP
GOLDFISH RAINBOW
HEADACHE SKATEBOARD
HOMEWORK TIMETABLE
HIGHWAY WEEKEND
HAIRCUT YOURSELF
INSIDE

SUBSTANTIVOS ABSTRATO E CONCRETOS


Assim como em Português, também temos também os substantivos abstratos – abstract
nouns – e os substantivos concretos – concrete nouns.
Os abstratos representam algo imaginado, pois designam coisas intangíveis tais como
ações, sentimentos, ideias, conceitos e qualidades.

AULA 03 – ARTICLES AND NOUNS 10


TEACHER ANDREA BELO

Exemplos de substantivos abstratos: peace, honesty, education, chaos, stress etc – paz,
honestidade, educação, caos, estresse.
Os concretos se referem às coisas que podemos ver e perceber ao nosso redor, já que
designam seres e coisas da realidade física, coisas que podemos ter contato através dos nossos
cinco sentidos: visão, audição, tato, paladar e olfato.
Exemplos de substantivos concretos: roses, sunset, sandwich, bus, voice, school etc. –
rosas, pôr do sol, sanduíche, ônibus, voz, escola etc.

CONCRETE NOUNS are ABSTRACT NOUNS are


words for things that you words for things that you
can experience directly cannot experience directy
through the senses. through the senses.
Substantivos concretos: Substantivos abstratos: não
podemos experimentar, podemos experimentar ou
tocar diretamente através tocar diretamente através
dos nossos sentidos. dos nossos sentidos.
TABLE (mesa) LOVE (amor)
DOG (cachorro) SUCCESS (sucesso)
FRIEND (amigo) FREEDOM (liberdade)
BOOK (livro) HAPPINESS (felicidade)
FLOWER (flor) DEMOCRACY (democracia)
PAPER (papel) VICTORY (vitória)
FEAR (medo)
JUSTICE (justiça)

SUBSTANTIVO COLETIVO
Há também os collective nouns (substantivos coletivos) que denotam um conjunto de
número indeterminado de seres ou coisas, ou seja, designam um grupo de seres da mesma
espécie.
Perceba que, o substantivo coletivo é uma única palavra que, mesmo usada no singular,
indica o agrupamento. Entender e saber aplicar os conceitos de gramática e ortografia da nossa
própria língua é essencial no nosso dia a dia.
A língua portuguesa, quando comparada à língua inglesa, possui mais coletivos inseridos
nos discursos, nos textos em geral.
Mas, podemos notar que, em qualquer língua, a riqueza de vocabulário permite que a
maioria das palavras sejam substituídas por outras, mais simples e poucas pessoas usam os
substantivos coletivos, de fato.

AULA 03 – ARTICLES AND NOUNS 11


TEACHER ANDREA BELO

Em textos científicos, acadêmicos, artigos, reportagens, revistas e jornais que você deve
ler, esses substantivos coletivos podem aparecer, como em Português, usa-se enxame, coletivo
de abelhas, para se referir a esses insetos.
Em Português, temos também a palavra arquipélago, coletivo de ilhas, que é mais comum
aparecer em leituras mais técnicas, assim como as suas fontes de estudo. Outros exemplos são
cardume - coletivo de peixes, banca - para examinadores e elenco - de atores.
Há coletivos bastante incomuns, em todas as línguas. São palavras quase não usadas ou
que não se escuta, como por exemplo em Português, girândola, coletivo de foguetes e também
atilho, coletivo de espigas de milho.
Em Inglês, do mesmo modo, há coletivos básicos, usados no cotidiano, como a palavra
forest (para um grupo de árvores), que muitas pessoas nem sabem que é classificada como um
substantivo coletivo.
Como exemplo de coletivos mais acessíveis, que podem aparecer em provas e são
considerados simples, temos family (família) para classificar parentes de uma mesma linhagem,
temos o coletivo audience (público) para pessoas, entre outros.
Alguns, como em Português, são atípicos, como o coletivo flock (rebanho ou bando de
pássaros) e swarm (enxame de abelhas).
Vejamos, a seguir, uma lista de coletivos como curiosidade, já que raramente são
disponibilizados em materiais que tratam desse assunto, vistos geralmente de forma superficial.
E você estará preparado, caso apareça alguma dessas palavras em sua prova.

SUBSTANTIVOS COLETIVOS EM INGLÊS:


(COLLECTIVE NOUNS)
Agenda = tasks (tarefas agendadas)
Ambush = people (legião)
Ants = colony (colônia de formigas)
Anthology = poems (antologia)
Apes = troop (macacos)
Army = soldiers (soldados)
Audience= spectators (espectadores)
Battery = tests (bateria de exames)
Bats = colony (morcegos)
Beavers = colony (castores)
Bench = magistrates (magistrados)
Bevy = beauties (beldades)
Board = directors (diretores)
Brood = researchers (pesquisadores)
Cast = actors/actresses (elenco)

AULA 03 – ARTICLES AND NOUNS 12


TEACHER ANDREA BELO

Camels = caravan (caravana de camelos)


Chain = islands (arquipélago)
Chorus = angels (coro de anjos)
Clowns = troupe (palhaços)
Clump = trees (arvoredo)
Collection = objects (coleção)
Congregation = workshippers (adoradores)
Crew = sailors (tripulação)
Convoy = lorries/truck (frota de caminhão)
Deck = cards (cartas)
Dogs = pack (matilha de cães)
Drift = lecturesrs (conferencistas)
Elephant = herd (manada)
Faculty = academics (acadêmicos)
Fleet = aircrafts (esquadrilha de aeronaves)
Fleet = ships (esquadra de navios)
Flock = tourists (turistas)
Forest = trees (Floresta)
Galaxy = stars (estrelas)
Grove = trees (bosque de árvores)
Handful = children (crianças)
Huddle = lawyers group (advogados)
Indians = tribe (tribo)
Jury = judges (juízes)
Neverthriving = jugglers (malabaristas, mágicos, ilusicionistas)
Mob = criminals (criminosos)
Orchestra = musicians (músicos)
Tigers = Ambush (alcateia de tigres)
Thieves = gang (ladrões)
Panel = experts (especialistas)
Peloton = cyclists (ciclistas)
People = crowd (multidão)
Shush = librarians (bibliotecários)
Squad = soldiers (esquadrão de soldados)
Staff = employees (funcionários)
Sheep = A flock of sheep (rebanho)
Soldiers = army (exército)
Team = athletes (atletas)

AULA 03 – ARTICLES AND NOUNS 13


TEACHER ANDREA BELO

SUBSTANTIVOS CONTÁVEIS
Temos também substantivos contáveis – countable nouns – são aqueles que podem ser
contados, enumerados.
Os substantivos contáveis possuem forma no plural e no singular, como podemos ver em
one book (um livro), two houses (duas casas) e one hundred years (cem anos).
Por serem classificados como contáveis, às vezes aparecerão precedidos de números ou
precedidos dos artigos definidos a/an, que vimos anteriormente.
Veja alguns exemplos: a pear, an apple, a pair of shoes (uma pera, uma maçã, um par de
sapatos).

COUNTABLE
A CUP OF TEA = uma xícara de chá
Exemplo: I want a cup of tea. – Eu quero uma xícara de chá.

UNCOUNTABLE
TEA = não existe “um chá”
Exemplo: I would like some tea. – Eu gostaria de um chá (quer dizer, um pouco de chá).

AULA 03 – ARTICLES AND NOUNS 14


TEACHER ANDREA BELO

SUBSTANTIVOS INCONTÁVEIS
Os substantivos incontáveis – uncountable nouns – são aqueles que não podem ser
contados ou enumerados. Assim, são usados somente no singular e por isso, não são antecedidos
pelos artigos a/an, não tem plural e não são usados depois de números.
Veja: I have good news for you. (Eu tenho boas notícias para você), She needs some help
(Ela precisa de ajuda).
Alguns exemplos comuns, sempre presentes nos textos das provas são: money (dinheiro),
music (música – estilo musical), bread (pão), entre outros que veremos as regras adiante.
Devemos nos lembrar que, ao contrário do Inglês, usamos, em Português, alguns destes
substantivos como contáveis.
Temos o costume de dizer, por exemplo, Por favor, me dê uma água, que não pode ser
traduzido Please, give me a water, está errada essa construção. O certo é: Please, give me a glass
of water – Por favor, me dê um copo de água ou Please, give me a bottle of water – Por favor, me
dê uma garrafa de água, já que copo e garrafa são contáveis.
Vejamos alguns exemplos de substantivos incontáveis que, você precisa saber o que usar
junto a eles para que seja possível usá-los em frases com sentido de contagem, já que não se pode
usar a ou an.
Um deles é o substantivo salt (sal) – para ser contável, dizemos uma colher de sal, por
exemplo – a teaspoon of salt.
A palavra money (dinheiro) – não existe “um dinheiro” e, para ser contável, usamos o nome
da moeda a que se refere, como por exemplos ten dollars – 10 dólares, five 5 pounds – 5 libras,
one real – 1 real.
Outro substantivo incontável é music. Primeiro, porque a palavra music não se refere
apenas a uma música específica (que é song) como em Português pois music se refere a um estilo
musical – He loves rock music (Ele ama rock, o estilo rock, certo?).
Temos a palavra bread (pão), que, em Português, é usada como contável, mas em Inglês
não se conta o pão e sim as fatias – I want a loaf of bread (Quero uma fatia de pão) e coffee (café),
em que não se conta um café e sim a xícara – I’d like a cup of coffee (Eu gostaria de uma xícara de
café). Para a palavra queijo – cheese, também não se fala: Eu quero um queijo e sim um pedaço,
uma fatia de queijo – I’d like a piece of cheese, a slice of cheese.
E daqui por diante, vamos ver outros possíveis substantivos que podem estar em sua prova.
Arroz, em Inglês – rice – para demonstrar quantidade, é correto usar um saco de arroz, um
grão de arroz: a bag of rice, a grain of rice.
E também papel – paper – não falamos um papel, como é usado em Português. Temos que
dizer uma resma de papel ou uma folha de papel: a ream of paper, a sheet of paper.
Para o substantivo conselho – advice – que, para se falar que vai dar um conselho é I will
give you some advice. O mesmo acontece com informação – uma informação seria some
information pois “one” information ficaria errado e não se usa nem sequer informalmente.

AULA 03 – ARTICLES AND NOUNS 15


TEACHER ANDREA BELO

Vamos ao uso da palavra notícia ou notícias em Inglês – news – que para dizer uma notícia
ou alguma notícia é a piece of news, some news. Esta palavra é sempre usada com verbos no
singular. Por exemplo: The news is fantastic. (As notícias são fantásticas). Mesmo que traduzida
por “notícias”, o verbo to be usado deve ser no singular: is.
Um substantivo bastante interessante é cabelo um Inglês – hair – porque, apesar de ser
incontável: pode ser usado como substantivo contável, significando de fio(s) de cabelo. Veja dois
diferentes exemplos do uso de hair:
My hair is black – Meus cabelos são pretos.
Her hair is brown. When she finds a gray hair, she immediately pulls it out - Os cabelos dela
são marrom. Quando ela acha um fio grisalho, ela o arranca imediatamente.
Vejamos outros substantivos incontáveis, para ficar ainda mais claro, separados por
categoria, facilitando a visualização.
Preparei uma lista, para você, com uncountable nouns divididos em food (comida), feelings
(sentimentos) e everyday stuff (coisas do dia a dia):

FOOD/DRINK FEELINGS EVERYDAY STUFF


Milk Fun Transportation
Water Love Mail
Bread Wisdom Travel
Pasta Knowledge Time
Rice Advice Air
Beef Help Homework
Pork Assistance Education
Fruit Satisfaction Business
Sugar Bravery Garbage
Cheese Curiosity Dust
Butter Attention Land
Tea Beauty Rain
Honey Faith Dirt
Jam Humour Sunshine
Wine Courage Sunrise

Podemos dizer, por exemplo, um suco, enquanto em Inglês, não existe “one juice” e sim
some juice, a cup of juice, a glass of juice ou a liter of juice que seriam a representação de possíveis
traduções: um pouco de suco, um copo de suco ou um litro de suco, por serem substantivos
incontáveis.

AULA 03 – ARTICLES AND NOUNS 16


TEACHER ANDREA BELO

Eis, agora, alguns exemplos ilustrativos com countable and uncountable food and drink
(alimentos e bebidas contáveis e incontáveis) para você comparar e perceber como é diferente
em Inglês comparados ao Português.
Perceba, por exemplo, que a palavra cereal, em Inglês, é incontável enquanto em
Português, dizemos cereais. Veja outros exemplos:

Outros exemplos bastante interessantes são os numbers (números em Inglês), que, no


plural, não sofrem mudança alguma de flexão, em sua maioria.
Em casos onde há substantivos com algum número e hífen, forma-se uma expressão e o
plural fica sem o -s, já que no inglês os adjetivos são invariáveis, como podemos ver em:
I have an 18-hour day (Tenho uma jornada de 18 horas)
He was in a five-star hotel (Ele estava um hotel cinco estrelas)
She works an 8-hour period on Mondays. (Ela trabalha um período de 8 horas na segunda-
feira)
E os numerais seguintes: dozen (dúzia), hundred (cem/centena), thousand (mil) e million
(milhão) e billion (bilhão), permanecem sem o -s: 8 million people - E não 8 millions para 8 milhões.
Veja: 200,000 - two hundred thousand – Three pens (três canetas) Three million dollars (três
milhões de dólares), two billion (dois bilhões).
Se, por acaso, for acrescentado –s ao final dos números, estamos nos referindo a números
indefinidos:
Dozens of clients bought the new product (Dezenas de clientes compraram o novo produto).

AULA 03 – ARTICLES AND NOUNS 17


TEACHER ANDREA BELO

E, por falar em palavras no singular e no plural, vejamos o que fazer para que os
substantivos sejam flexionados de forma correta. Em Português, a maioria das palavras apenas
precisam do acréscimo da letra -s no final delas.
Em Inglês, há outras possibilidades e regras a seguir, de fato.
Plural de palavras terminadas em -ch, -sh, -s, -x, -z e -o, usa-se -es no final delas, veja:
church = churches (igrejas), dish = dishes (pratos), kiss = kisses (beijos), box = boxes (caixas) e
tomato = tomatoes (tomates).
E, para substantivos terminados em -f ou -fe, essas terminações são substituídas por -ves:
shelf = shelves (prateleiras), leaf = leaves (folhas), thief = thieves (ladrões) e wolf = wolves, entre
outras.
Substantivos terminados em -y, colocamos -ies no final das palavras e retiramos o -y que
estava lá como por exemplo family = families e baby = babies.
Em relação a plural irregular, devido a serem palavras que sofrem grandes alterações
quando flexionadas, o melhor jeito de aprender é fazendo exercícios, leituras de variados textos
em Inglês.
É a prática que ensina verdadeiramente.
E, quando estudados com atenção, fica fácil memorizar.
O substantivo pé, que é foot em Inglês, muda para feet, com letras e no lugar de o.
Dizer pessoa, em Inglês, é person, mas pessoas, é people. Até aceita-se dizer persons para
duas pessoas, mas quando se fala de muitas pessoas, mais de três, um grupo de pessoas, é people.
Os substantivos person/people merecem um pouco a mais de atenção pois people, pode
significar também povo, no sentido demográfico e pode significar gente, como eu disse
anteriormente, pessoas.
Assim, o plural de people no sentido de povo tem acréscimo de -s:
The South people enjoys barbecue. (O povo do Sul curte gosta de churrasco).
Como a palavra people está no singular, o verbo to enjoy, correspondente no Present
Simple, vai fazer a concordância, sendo acrescentado de -s para a terceira pessoa – o povo (ele)
gosta.
Agora, nessa frase:
Some Indian peoples have different cultural habits – Alguns povos indianos têm costumes
culturais estranhos, o verbo está fazendo a concordância com o plural peoples.
E ainda existem algumas palavras que tem letra -s no final, mas estão no singular, sendo
iguais quando usadas no plural, como por exemplo headphones (fones de ouvido) e pants (calças).
Há alguns substantivos que têm origem grega ou latina e aparecem com frequência em
textos científicos em Inglês e são diferentes, portanto, merecem atenção.
Veja alguns deles a seguir, que podem estar presentes em uma das questões de sua prova.
São eles:

AULA 03 – ARTICLES AND NOUNS 18


TEACHER ANDREA BELO

➢ alumnus - alumni (ex-alunos, alunos graduados),


➢ axis - axes (eixos),
➢ bacillus - bacilli (bacilos),
➢ bacterium - bacteria (bactérias),
➢ cactus - cacti (cactos),
➢ corpus - corpora (corpora),
➢ criterion - criteria (critérios),
➢ datum - data (dados),
➢ encyclopedia - encyclopedae (enciclopédias),
➢ erratum - errata (erratas),
➢ formula - formulae ou formulas (fórmulas),
➢ fungus - fungi ou funguses (fungos),
➢ medium - media (meios de comunicação),
➢ nucleus - nuclei (núcleos),
➢ phenomenon - phenomena (fenômenos),
➢ stimulus - stimuli (estímulos),
➢ stratum - strata (estratos),
➢ vertebra - vertebrae (vértebras).

Algumas gramáticas ainda apresentam uma categoria especial para classificar o plural com
o nome de parelhas. As parelhas são a demonstração do plural de alguns substantivos que têm
duas partes iguais e uma possível tradução para eles seria “um par de”, como óculos (glasses).
Mas podemos simplesmente chamar de plural irregular, como veremos uma lista adiante,
que preparei como complemento dos seus estudos em relação a esse assunto.
Essas palavras só existem no plural, não possuem singular, apesar de que, em Português,
podemos usá-las no singular, como:
Onde estão os meus óculos de sol? – There are my sunglasses? - E nunca “Where is my
sunglasses?”. Haverá outros exemplos adiante para melhor compreensão.
Não podemos nos esquecer também do que chamamos de falsos plurais. Alguns livros e
estudiosos linguistas afirmam que existem plurais nomeados falsos porque são idênticos na
escrita tanto quando são usados no singular quanto no plural.
Os exemplos de plurais falsos são os nomes de disciplinas, de ciências e outros substantivos
terminados em -ics, que são singulares:
Politics is a complicated thing to talk about – Política é uma coisa complicada de se falar
sobre.

AULA 03 – ARTICLES AND NOUNS 19


TEACHER ANDREA BELO

Temos como outros exemplos, além de politics, acoustics (acústica), athletics (atletismo),
electronics (eletrônica), genetics (genética), linguistics (linguística), mathematics (matemática),
physics (física) e statistics (estatística):
General Statistics can show you the real results of our project. – A estatística geral pode
mostrar a você os resultados reais do nosso projeto.
Mas, há uma importante observação a ser feita: se, por acaso, essas palavras forem usadas
com outro sentido, o verbo que as acompanha poderá ir para o plural, como:
What are your politics? - Quais são suas políticas?
Your family genetics prove the similarities among your relatives. (As genéticas de sua
família provam as semelhanças entre os seus parentes.
Vou apresentar uma lista, de possíveis palavras que possuem plurais irregulares, das
variadas exceções acima e que podem aparecer nos textos das provas:

IRREGULAR PLURAL NOUNS LIST


(LISTA DE SUBSTANTIVOS COM PLURAL IRREGULAR)
alga = algae (algas marinhas) life = lives (vidas)
analysis = analyses (análises) man = men (homens)
appendix = appendices (indices) oasis = oases (oásis)
basis = bases (base) oats = oats (aveia)
binoculars = binoculars (binóculos) ox = oxen (bois)
bus = buses (ônibus) parenthesis = parentheses (parênteses)
child = children (crianças) salmon = salmon (salmões)
clothes = clothes (roupas) scales = scales (balanças)
crisis = crises (crises) scarf = scarves (lenços, cachecóis)
diagnosis = diagnoses (diagnósticos) scissors = scissors (tesouras)
dwarf = dwarves (anões) series = séries (series de TV)
elk = elk (alces) sheep = sheep (ovelhas)
fish = fish (peixes) synthesis = syntheses (sínteses)
foot = feet (pés) synopsis = synopses (sinopses)
gentleman = gentlemen (cavalheiros) surroundings = surroundings (arredores)
glasses = glasses (óculos) species = species (espécie)
half = halves (metades) stairs = stairs (escadas)
hypothesis = hypotheses (hipóteses) thanks = thanks (obrigado(a)
index = indeces/indexes (índices) thesis = theses (teses)
jeans = jeans (calças jeans) tights = tights (meia-calças)
knife = knives (facas) tooth = teeth (dentes)
matrix = matrices/matrixes (matrizes) trout = trout (trutas)
mouse = mice (ratos) woman = women (mulheres)

AULA 03 – ARTICLES AND NOUNS 20


TEACHER ANDREA BELO

PLURAL IRREGULAR E PARTICULARIDADES


01 – Substantivo singular terminado em -y precedido por consoante (letra y)

SINGULAR PLURAL SIGNIFICADO


BABY BABIES BEBÊ
SKY SKIES CÉU
CHERRY CHERRIES CEREJA

Para os substantivos singulares terminados em -y precedidos por vogal, têm o plural regular, ou
seja, recebem apenas o -s final.

02 – Substantivo singular que termina em som sibilante -s, -se, -ss, -sh, -ge, -ch, -dge, -x

SINGULAR PLURAL SIGNIFICADO


BUS BUSES ÔNIBUS
KISS KISSES BEIJO
PHASE PHASES FASE
DISH DISHES PRATO
MASSAGE MASSAGES MASSAGEM
WATCH WATCHES RELÓGIO
JUDGE JUDGES JUIZ
BOX BOXES CAIXA

03 – Substantivo singular terminado em -o e precedido por consoante


SINGULAR PLURAL SIGNIFICADO
POTATO POTATOES BATATA
HERO HEROES (OR HEROS) HERÓI
VOLCANO VOLCANOES (OR VOLCANOS) VULCÃO

04 – Substantivo singular de origem estrangeira terminado em -o e precedido por consoante

SINGULAR PLURAL SIGNIFICADO


PHOTO PHOTOS FOTO
KIMONO KIMONOS QUIMONO
PIANO PIANOS PIANO
PRO PROS PRÓ; PROFISSIONAL

AULA 03 – ARTICLES AND NOUNS 21


TEACHER ANDREA BELO

05 – Substantivo singular terminado em -f ou -fe


Existem palavras em inglês que terminam em -f ou -fe, cujos plurais são formados substituindo-
se estas letras por -ves. São elas:

SINGULAR PLURAL SIGNIFICADO


CALF CALVES BEZERRO, PANTURRILHA
ELF ELVES ELFO
HALF HALVES MEIO, METADE
KNIFE KNIVES FACA
LEAF LEAVES FOLHA
LIFE LIVES VIDA
LOAF LOAVES PÃO DE FORMA
SELF SELVES SI MESMO
SHEAF SHEAVES FEIXE
SHELF SHELVES PRATELEIRA, PLATAFORMA, ESTANTE
THIEF THIEVES LADRÃO
WIFE WIVES ESPOSA
WOLF WOLVES LOBO

IMPORTANTE: existem quatro palavras terminadas em -f que têm duas formas de plural: -ves ou -
fs. São elas:

SINGULAR PLURAL 1 PLURAL 2 SIGNIFICADO


DWARF DWARFS DWARVES ANÃO
HOOF HOOFS HOOVES CASCO DE ANIMAL
SCARF SCARFS SCARVES CACHECOL
WHARF WHARFS WHARVES CAIS

06 – Singular em -oo- > plural em -ee-


Neste grupo, são três as palavras que sofrem esta alteração:

SINGULAR PLURAL SIGNIFICADO


GOOSE GEESE GANSO
FOOT FEET PÉ
TOOTH TEETH DENTE

AULA 03 – ARTICLES AND NOUNS 22


TEACHER ANDREA BELO

07 – Plural em -em
As três palavras abaixo sofrem modificações ou acréscimo e formam o plural com -en.

SINGULAR PLURAL SIGNIFICADO


CHILD CHILDREN CRIANÇA, FILHO/A
MAN MEN HOMEM
OX OXEN BOI
WOMAN WOMEN MULHER

08 – Singular -ouse > Plural -ice


Apenas as duas palavras abaixo sofrem esta mudança:

SINGULAR PLURAL SIGNIFICADO


LOUSE LICE PIOLHO
MOUSE MICE CAMUNDONGO

09 – Quando não há mudanças:


As palavras abaixo têm formas iguais para o singular e plural (ou duas formas aceitáveis):

SINGULAR PLURAL SIGNIFICADO


DEER DEER / DEERS VEADO
DIE DICE DADO
ELK ELK / ELKS ALCE
FISH FISH / FISHES PEIXE
OFFSPRING OFFSPRING / OFFSPRINGS PROLE
SERIES SERIES SÉRIE
SHEEP SHEEP OVELHA
SPECIES SPECIES ESPÉCIE

Agora, para complementar as explicações e exemplos, vamos falar um pouco das inúmeras
curiosidades que envolvem os artigos e os substantivos.

AULA 03 – ARTICLES AND NOUNS 23


TEACHER ANDREA BELO

CURIOSIDADES SOBRE ARTIGOS


Mostrarei a você algumas curiosidades divertidas em Inglês e ao mesmo tempo,
pertinentes. Além de serem fatos interessantes que podem fazer parte de alguma pergunta ou
texto, envolvem o artigo definido the, estudado nessa aula.
Você por acaso sabe o que é pangrama? Tem alguma ideia? Bom, a palavra pangrama tem
origem grega, sendo que pan significa todos, grama significa letra e, um pangrama é uma frase
em que são usadas todas as letras do alfabeto de uma língua, mas essa frase precisa ter sentido
completo.
Os pangramas foram inventados na mesma época em que surgiu a tipografia, com o
propósito de avaliar os efeitos visuais de uma fonte, como uma forma de exercício. A criação de
pangramas é um passatempo divertido, que demanda criatividade e conhecimento da língua.
Se um pangrama deve usar todas as letras com o mínimo de palavras, obviamente, a
combinação de palavras precisa de artigos para conectá-las. E é isso mesmo.
O primeiro pangrama que existiu foi: The quick brown fox jumps over the lazy dog,
traduzido: A rápida raposa marrom pula por cima do cão preguiçoso. Perceba que, na sentença
que gerou o pangrama, a conecção de palavras foi feita com o uso do artigo definido the duas
vezes, no início e quase no final da frase: The quick brown fox jumps over the lazy dog.
Em um artigo chamado Interesting Notes no ano de 1885, essa frase foi uma sugestão da
companhia The Western Union, testando o sistema de telégrafo com transmissões da mensagem
The quick brown fox jumps over the lazy dog. Essa frase aparece, inclusive, como exemplo em
alguns programas da Microsoft.
Em Português, temos pangramas também, veja: “Jane quer LP, fax, CD, giz, TV e bom
whisky”, em que estão presentes todas as 26 letras do nosso alfabeto em uma só frase e, com
sentido. Legal, não é? Você consegue elaborar algum pangrama?
Quer saber outra curiosidade sobre os artigos?
Vamos lá! Foi descoberto que, cerca de 11% de toda a língua inglesa é composta
exclusivamente pela letra “e”. Isso mesmo, a letra “e” está presente em milhares de palavras em
Inglês. Dizem que, uma, em cada oito letras que você vê em uma palavra escrita em Inglês, vai
possuir a letra “e”.
E eu te pergunto, há coincidência com alguma letra que possui no artigo definido que
estudamos?
Então, de todas as palavras vistas em frases onde encontramos a letra “e”, em Inglês, mais de 70%
possuem o artigo THE em sua construção, tanto para conectar substantivos e verbos como
também para dar sentido entre o sujeito, predicado e outros elementos da oração.
Agora, vamos às curiosidades sobre os substantivos.

AULA 03 – ARTICLES AND NOUNS 24


TEACHER ANDREA BELO

CURIOSIDADES SOBRE SUBSTANTIVOS


Agora, vamos à algumas curiosidades sobre os substantivos, que também podem aparecer,
retiradas de fontes que publicam fatos curiosos e pesquisas, como as revistas Time, Forbes e
Newsweek e os jornais The Telegram e The Guardian.
Você sabe qual é a palavra mais antiga que existe em Inglês e que ainda é usada até hoje?
É o substantivo town, que significa cidade. Apesar de existir a palavra city, que também significa
cidade, town é menor do que city. Ao se referir ao termo city, deve-se pensar em uma área mais
populosa, já que antigamente, só havia towns, menores, como se fossem municípios hoje.
Outra curiosidade: além da palavra mais antiga em Inglês ser um substantivo, a palavra
maior que existe a língua inglesa também é um substantivo. Sabe qual é? É o nome de uma doença
pulmonar, que se adquire com a inalação de cinza e pó de carvão. É difícil de escrever e de falar.
Mais uma curiosidade: você sabia que William Shakespeare, além de contribuir com a
língua inglesa em suas obras escritas, ele adorava inventar palavras? Não existe uma lista para
comprovar quantas palavras para saber o número exato de vocábulos que ele inventou. Mas, é
comprovado que, quase todas, são substantivos comuns, próprios, compostos, abstratos, entre
outros. Por exemplo, birthplace (local de nascimento), torture (tortura) e bubble (bolha) foram
criadas por Shakespeare.
E a melhor curiosidade de todas: quantas palavras você acha que precisa saber para falar
Inglês fluentemente? Se você acha que precisa saber todas as palavras que existem no dicionário
ou, no mínimo, 90% delas para falar fluente esse idioma, você está enganado.
É comprovado o fato que, são necessárias apenas 2.000 palavras para se comunicar bem
em Inglês. Pense bem: o Dicionário Oxford, um dos maiores dicionários da língua inglesa, tem
mais de 200 mil verbetes. Isso mesmo, duzentas mil palavras das quais você precisa saber duas
mil para adquirir fluência. E, dessas 2 mil, mais da metade são substantivos.

= 200.000 words (200.000 palavras)


*You need 2.000 to communicate
(Você precisa de 2 mil para se comunicar)

Muitos estudiosos linguistas dizem que a produção oral não precisa de um vocabulário
muito extenso e isso significa que o seu vocabulário ativo (o que você usa para falar). Então, 80%
das palavras que você realmente vai precisar, se resumem em 2000 palavras em Inglês.
Gostou das curiosidades? Percebemos que, a língua inglesa, assim como qualquer outra
língua, sem os substantivos ou verbos, não é possível de ser falada ou escrita.
E você já aprendeu todos os tempos verbais e agora, os artigos e os substantivos, tão
importantes na hora da prova. Vamos aos números.

AULA 03 – ARTICLES AND NOUNS 25


TEACHER ANDREA BELO

NÚMEROS
Em geral, os números em Inglês não são tão complicados como pode parecer, mas você prestar
atenção na escrita e nas terminações.
De 0 até 12, você provavelmente aprendeu na escola ou em algum curso. Vejamos:

Números de 1 a 12
1 – one
2 – two
3 – three
4 – four
5 – five
6 – six
7 – seven
8 – eight
9 – nine
10 – ten
11 – eleven
12 – twelve

A partir do número 13 até o número 19, usamos a terminação “teen”, por isso os adolescentes
são popularmente chamados de “teens” ou teenagers:

13 – thirteen
14 – fourteen
15 – fifteen
16 – sixteen
17 – seventeen
18 – eighteen
19 – nineteen

Em números decimais como 20, 30, 40, 50 até o 90, usamos a terminação é “ty”.

AULA 03 – ARTICLES AND NOUNS 26


TEACHER ANDREA BELO

20 – twenty
21 – twenty-one
22 – twenty-two
23 – twenty-three
24 – twenty-four
Etc.

Até 29 – twenty-nine
30 – thirty
40 – forty
50 – fifty
60 – sixty
70 – seventy
80 – eighty
90 – ninety

A partir do número 100, usamos o número inicial e o termo “hundred”, sendo então 100 = a
hundred/one hundred, o 200 = two hundred e assim por diante.
A partir do número 101, falamos apenas os números sem mudanças de formato. Confira:

101 – one hundred one


102 – one hundred two
103 – one hundred three
Até o 109 – one hundred nine
200 – two hundred
300 – three hundred
400 – four hundred
Até 900 = nine hundred

Acima de 999, usamos o número e o termo “thousand”, sendo então 1000 = a thousand/one
thousand, o 2000 = two thousand, o 3000 = three thousand e assim por diante.
O milhão é “a million” e depois “two million”, “three million” etc.

AULA 03 – ARTICLES AND NOUNS 27


TEACHER ANDREA BELO

QUESTÕES
Você vai, agora, responder questões selecionadas de provas já realizadas em anos
anteriores. Depois, como em todas as nossas aulas, haverá o gabarito e as questões comentadas.
Vamos começar com questões AFA, de acordo coma sua instituição escolhida e depois,
vamos treinar de outras Carreiras Militares, para adquirir experiência e treinar vocabulário.
QUESTÕES AFA
The end of life on Earth?
It weighed about 10,000 tons, entered the atmosphere at a speed of 64,000km/h and exploded
over a city with a blast of 500 kilotons. But on 15 February 2013, we were lucky. The meteorite
that showered pieces of rock over Chelyabinsk, Russia, was relatively small, at only about 17
metres wide. Although many people were injured by falling glass, the damage was nothing
compared to what had happened in Siberia nearly one hundred years ago. Another relatively small
object (approximately 50 metres in diameter) exploded in mid-air over a forest region, flattening
about 80 million trees. If it had exploded over a city such as Moscow or London, millions of people
would have been killed.
By a strange coincidence, the same day that the meteorite terrified the people of Chelyabinsk,
another 50m-wide asteroid passed relatively close to Earth. Scientists were expecting that visit
and know that the asteroid will return to fly close by us in 2046, but the Russian meteorite earlier
in the day had been too small for anyone to spot.
Most scientists agree that comets and asteroids pose the biggest natural threat to human
existence. It was probably a large asteroid or comet colliding with Earth which wiped out the
dinosaurs about 65 million years ago. An enormous object, 10 to 16km in diameter, struck the
Yucatan region of Mexico with the force of 100 megatons. That is the equivalent of one Hiroshima
bomb for every person alive on Earth today.
Many scientists, including the late Stephen Hawking, say that any comet or asteroid greater than
20km in diameter that hits Earth will result in the complete destruction of complex life, including
all animals and most plants. As we have seen, even a much smaller asteroid can cause great
damage.
The Earth has been kept fairly safe for the last 65 million years by good fortune and the massive
gravitational field of the planet Jupiter. Our cosmic guardian, with its stable circular orbit far from
the sun, sweeps up and scatters away most of the dangerous comets and asteroids which might
cross Earth’s orbit. After the Chelyabinsk meteorite, scientists are now monitoring potential
hazards even more carefully but, as far as they know, there is no danger in the foreseeable future.
Types of space rocks
• Comet – a ball of rock and ice that sends out a tail of gas and dust behind it. Bright comets
only appear in our visible night sky about once every ten years.
• Asteroid – a rock a few feet to several kms in diameter. Unlike comets, asteroids have no
tail. Most are too small to cause any damage and burn up in the atmosphere. They appear
to us as ‘shooting stars’.
• Meteoroid – part of an asteroid or comet.
• Meteorite – what a meteoroid is called when it hits Earth.
Taken from: https://learnenglishteens.britishcouncil.org/skills/reading/upper-intermediate-b2-reading/end-life-earth

AULA 03 – ARTICLES AND NOUNS 28


TEACHER ANDREA BELO

Questão 01 (AFA/INÉDITA) – The damage caused by the Russian meteorite


a) could be impossible to calculate.
b) was not as bad as the one in Siberia.
c) was much worse than the one in Siberia.
d) had its impact reduced by the early warning system.

Questão 02 (AFA/INÉDITA) – Without changing that meaning the word “Although” (line 6) could
be substituted for:
a) however
b) therefore
c) despite
d) once

Questão 03 (AFA/INÉDITA) – The statement “many people were injured by falling glass” (line 7)
stands for: Failing glass _______ many people
a) have injured
b) has injured
c) had injured
d) injured

Questão 04 (AFA/INÉDITA) – The passage “the damage was nothing compared to what had
happened in Siberia nearly one hundred years ago” (lines 7 to 8) states that the incident
occurred _______ a country ago.
a) actually
b) precisely
c) approximately
d) exactly

Questão 05 (AFA/INÉDITA) – The Siberian meteorite


a) damaged trees when it exploded.
b) caused glass to shower over people.
c) has the destructive power of a nuclear bomb.
d) exploded over a big city.

AULA 03 – ARTICLES AND NOUNS 29


TEACHER ANDREA BELO

QUESTÕES COLÉGIO NAVAL


Directions: Answer questions 01 to 05 according to TEXT.
Facebook complies with Brazilian judge’s order to block 12 accounts accused of running a fake
news network
Facebook announced Saturday it has obeyed a Brazilian judge’s order for a worldwide block on
the accounts of 12 of President Jair Bolsonaro’s supporters who are under investigation for
allegedly running a fake news network.
Supreme Court Justice Alexandre de Moraes said Friday night that the company had failed to fully
comply with a previous ruling ordering the accounts to be shut down, saying they were still online
and publishing by changing their registration to locations outside Brazil.
Facebook issued a statement saying it complied due to the threat of criminal liability for an
employee in Brazil.
But it called the new order “extreme,” saying it poses a “threat to freedom of expression outside
of Brazil’s jurisdiction and conflicting with laws and jurisdictions worldwide.” The company said it
would appeal to the full court.
Facebook also argued it had complied with the previous order by “restricting the ability for the
target Pages and Profiles to be seen from IP locations in Brazil”.
“People from IP locations in Brazil were not capable of seeing these Pages and Profiles even if the
targets had changed their IP location”, the company said.
Moraes said that Facebook ought to pay $ 367,000 in penalties for not complying with his previous
decision during the last eight days.
He also had ruled Twitter should block the accounts. While Twitter said then the decision was
“disproportionated” under Brazil’s freedom of speech rules and that it would appeal, the targeted
profiles were disabled.
Moraes is overseeing a controversial investigation to determine whether some of Bolsonaro’s
most ardent allies are running a social media network aimed at spreading threats and fake news
against Supreme Court justices.
The probe is one of the main points of confrontation between Bolsonaro and the Supreme Court.
The president himself filed a lawsuit last week demanding the accounts to be unblocked.
(Adapted from https://time.com/5874695/facebook-blocks-accounts-worldwide/)

Questão 01 (COLÉGIO NAVAL/INÉDITA) – Read the extract from the text.


“People from IP locations in Brazil were not capable of seeing these Pages and Profiles even if
the targets had changed their IP location”.
Mark the option that replaces correctly the expression “not capable”.
a) Uncapable
b) Competent
c) Qualified.
d) Efficient
e) Incapable.

AULA 03 – ARTICLES AND NOUNS 30


TEACHER ANDREA BELO

Questão 02 (COLÉGIO NAVAL/INÉDITA) – Read the extract from the text.


“Alexandre de Moraes said Friday night that the company had failed to fully comply with a previous
ruling ordering the accounts to be shut down”.
Choose the correct question for the sentence below.
a) What was the supreme statement on Facebook’s first attitude?
b) At first, did Facebook comply with the order of Alexandre de Moraes?
c) What was the final answer from Facebook to the Brazilian Supreme Court Justice?
d) Were the accounts blocked?
e) Who is Alexandre de Moraes?

Questão 03 (COLÉGIO NAVAL/INÉDITA) – Read the extract from the text.


“Moraes said that Facebook ought to pay $ 367,000 in penalties for not complying with his
previous decision during the last eight days.”
Mark the option that can replace the underlined sentence correctly.
a) For not helping with his latest decision during the last eight days.
b) For cooperating with his previous decision during the last eight days.
c) For assisting with the supreme first decision during the last eight days.
d) For not cooperating with the supreme first decision during the last eight days.
e) For cooperating with his decision.

Questão 04 (COLÉGIO NAVAL/INÉDITA) – According to the seventh paragraph, we can assume


that.
a) Twitter agreed immediately with the decision, without protesting.
b) Even without agreeing, twitter complied with the order.
c) Twitter didn’t respond to the supreme court order.
d) Twitter protested to the order and failed to comply.
e) Twitter was not advertised.

Questão 05 (COLÉGIO NAVAL/INÉDITA) – About the Brazilian President, it is correct to affirm


that.
a) The relationship between him and the supreme court is peaceful.
b) He readily agreed with the supreme decision.
c) He did not comment on the decision.
d) He was against the supreme court decision.
e) He agreed and cooperated with the supreme.

AULA 03 – ARTICLES AND NOUNS 31


TEACHER ANDREA BELO

QUESTÕES EAM
Questão 01 (EAM/INÉDITA) – Read the dialogue and mark the right option to fill in the gaps
respectively.
A: _________ you want to go to the beach yesterday?
B: Yes, I ________.
A: Who ________ you want to go with?
B: I ________ to go with my sister. Did you go?
A: No, I ________.
A) Did / did / went / want / wasn’t
B) Were / was / were / want / didn’t
C) Do / do / did / wanted / wasn’t
D) Do / do / did / wanted / didn’t
E) Did / did / did / wanted / didn’t

Questão 02 (EAM/INÉDITA) – Read the sentences and mark the correct option to fill in the
blanks respectively.
__________ name is Bella. She’s 17 and I live __________ Brasilia which is the capital of Brazil.
Actually she comes from another city – Rio, where she __________ for fifteen years.
A) Her / in / lived
B) Your / at / lived
C) Her / on / lives
D) My / in / live
E) His / in / live

Questão 03 (EAM/INÉDITA)

AULA 03 – ARTICLES AND NOUNS 32


TEACHER ANDREA BELO

What’s the main verb tense used in the comic strip?


A) Present Continuous
B) Present Simple
C) Simple Past
D) Future Simple
E) Present Perfect

Questão 04 (EAM/INÉDITA) – Use the verbs in the parentheses to complete the following
statements.
I – He __________ (study) at the Brazilian Navy Academy.
II – At some point, I finally __________ (realize) that I could pass that test.
III – She loves __________ (run) in the morning.
Now mark the option which completes them respectively.
A) study / realized / running
B) studies / realized / runs
C) studied / realize / run
D) studies / realized / running
E) study / realize / runs

Questão 05 (EAM/INÉDITA) – Look at the picture below.

What are they doing in the picture?


A) They are trying to pull the girl with the ball.
B) They are playing soccer.
C) They are making the girl fall down.
D) They are warning the girl with the ball of some danger.
E) They don’t like to play with the girl with the ball.

AULA 03 – ARTICLES AND NOUNS 33


TEACHER ANDREA BELO

QUESTÕES EEAR
Read the text and answer questions 01 e 02.

Adapted from http://www.englishhact.com.br/2016/10/atividades-com-tirinhas-do-garfield-em.html

Questão 01 (EEAR – CFS/2020) – Which expression is usually used to tell stories in English?
a) There was a house with a man
b) Then the cat ate my breakfast
c) That sounds familiar
d) Once upon a time

Questão 02 (EEAR – CFS/2020) – What is the main verb tense used in the comic strip?
a) Present Perfect
b) Simple Present
c) Past Perfect
d) Simple Past

Read the text and answer question 03.


“The U.S. Constitution doesn't guarantee happiness, only the pursuit of it. Your have to catch up
with it yourself.”
https://www.brainyquote.com/quotes/benjamin_franklin_141100

Questão 03 (EEAR – CFS/2020) – The word in bold, in the text, is na object pronoun. Which
word does it make reference to?
a) Constitution
b) Happiness
c) The U. S.
d) Pursuit

AULA 03 – ARTICLES AND NOUNS 34


TEACHER ANDREA BELO

Read the text and answer question 04.


Being famous
Sandra Rosa is very beautiful, young, and successful. She's a famous actress. She's also very rich.
Her house near the beach is big and beautiful, and her car is very expensive. Her fans love her.
But is she happy?
Sandra says, "yeah, I'm young, rich, beautiful, and famous. People think rich people are happy.
That's not always true!"
Sandra's brother, Mike, is her manager. He says, "Sandra is only 18. She enjoys acting and
entertaining people. But she's not happy. She doesn't like being famous."
"It's true," Sandra says. "I'm never alone. Reporters are everywhere. Wherever I go, they're there.
They're outside my house all the time! That's so annoying!”
Adapted from: https://www.inglesnapontadalingua.com.br/2008/10/texto-de-ingls-alunos-de-nvel-bsico.html

Questão 04 (EEAR – CFS/2020) – According to the text, Sandra is


a) very happy because she is never by herself.
b) very happy because she has a lot of money.
c) unhappy with her carreer because it is annoying.
d) unhappy because wherever she goes, the reporters are there.

Read the text and answer question 05.


Homeless crack addict revitalizes small square in downtown São Paulo
A homeless man has chosen to occupy his free time revitalizing a small square on the corner of
avenues São João and Duque de Caxias, in downtown São Paulo. He planted pau-brasil, palm,
banana and avocado trees. He also planted boldo, sweet potatoes, beans, peppers and
ornamental plants, such as snake plants. Residents noticed the square’s gradual changes and
congratulated the author for the modifications.
Fonte: Folha de São Paulo Internacional – 21/03/2017

Questão 05 (EEAR/2017) – The words “small”, “sweet” e “ornamental”, underlined in the text,
are __________.
a) nouns
b) adverbs
c) pronouns
d) adjectives

AULA 03 – ARTICLES AND NOUNS 35


TEACHER ANDREA BELO

QUESTÕES EFOMM
Based on the text below, answer questions 01 and 02.
Why some people like wearing masks
Some people welcome face coverings for reasons ranging from the convenient and expedient
to the more complex and psychological. But is this a helpful coping mechanism?
Sheltering in place hasn’t been too hard for Jay Lee; watching a film at home and ordering a
takeaway has always been his idea of a good night. Lee, a 32-year-old small business owner in
Leicester, identifies as an introvert. And although 2020 had its hardships – in the spring, he was
made redundant from his job at a large bank – one perk for him has been the widespread adoption
of face masks.
Lee has always dreaded run-ins with old friends and acquaintances around town, finding these
spontaneous interactions “extremely awkward”. He used to time his shopping trips to minimise
the possibility of bumping into someone he knew, waiting until almost closing time before
heading out. “Since I've been wearing the mask, my awkward interactions with friends and family
have significantly reduced,” he says. Now, he goes to the shops whenever he wants, without
worrying about whom he might see. He hopes that, even after the pandemic ends, it will still be
socially acceptable to wear a mask.
Wearing a mask is, for most of us, an annoying but worthwhile sacrifice: it’s one of the most
effective ways to slow the spread of Covid-19. Still, most of us look forward to the day when we
can bare our faces in public again. Face-coverings fog our glasses and clog our pores; they make
it harder to smile at strangers and recognise friends.
Yet some are secretly relishing the new mask-wearing mandates, for reasons ranging from the
convenient and expedient to the more complex and psychological. Some welcome the way face
coverings reduce or change interactions that might otherwise spark social anxiety.
‘Anonymity carries power’
At the lighter end of the scale, some people have found that masking offers a welcome relief from
the pressures to uphold strict standards of grooming and appearance. They have ditched their old
makeup and shaving routines and are saving money, time and stress. Others have discovered that
hiding their mouths affords them unexpected freedoms. Some restaurant servers and retail
workers say they no longer feel obliged to fake-smile at customers, potentially lifting the burden
of emotional labour.
(Adapted from https://www.bbc.com/worklife/article/20210115-why-some-people-like-wearing-masks)

Questão 01 (EFOMM/INÉDITA) – It is possible to infer from the text that


(A) The safety measure related to the masks is well accepted depending on the personality of
the user
(B) Face coverings is an inconvenience for everyone
(C) Introverted people ended up not adapting to the context of a pandemic
(D) Wearing a mask is a pointless sacrifice at the present time
(E) Wearing masks does not interfere with the aesthetic pressure exerted by society

AULA 03 – ARTICLES AND NOUNS 36


TEACHER ANDREA BELO

Questão 02 (EFOMM/INÉDITA) – In the excerpt “And although 2020 had its hardships – in the
spring, he was made redundant from his job at a large bank – one perk for him has been the
widespread adoption of face masks.”, the word in bold means
(A) Advice
(B) Hardship
(C) Happiness
(D) Loss
(E) Advantage

Based on the text below, answer questions 03, 04 and 05.


The World's Biggest Problems Are Interconnected. Here's How We Can Solve Them This
Decade
Two decades ago, people around the world rang in the new millennium with a growing sense of
optimism. The threat posed by the Cold War was fading slowly in the rearview mirror. Leading
thinkers like Francis Fukuyama touted the benefits of globalization, saying it would bring
democracy and prosperity to the developing world. The nascent Internet economy promised to
bring us closer together.
The following 20 years took some of the air out of the assumption of steady progress, but when
future historians assess the 21st century, the year 2020 is likely to serve as the point at which the
optimism bubble burst. The COVID-19 pandemic has exposed a complex web of interlocking
problems that have morphed into full-blown crises. The coronavirus laid bare the dangers of
endemic poverty not only in the developing world but also in rich countries like the U.S., where
millions lack health care and are one paycheck away from living on the street. Around the world,
racial and ethnic minorities have demanded justice after centuries of structural discrimination.
Woven through it all, the earth’s climate is increasingly unstable, posing an existential threat to
human society as we know it. In the next decade, societies will be forced to either confront this
snarl of challenges, or be overwhelmed by them. Our response will define the future for decades
to come.
The recognition that these challenges are fundamentally linked isn’t new. Activists and academics
have for many years pointed to the cascading effects of various social ills. Whether it’s the way
racism contributes to poor health outcomes or gender discrimination harms economic growth,
the examples are seemingly endless. But this understanding has made its way into the
conversation about solutions too.
Notably, for the past five years, the U.N. has touted 17 interrelated sustainable development
goals, objectives for building a more viable world, and called for a push to achieve them by 2030.
The goals, which cover environmental, social and economic progress, are nonbinding but have
become key benchmarks for commitments at a national and corporate level. Countries from China

AULA 03 – ARTICLES AND NOUNS 37


TEACHER ANDREA BELO

to the Maldives, as well as companies like Amazon, Microsoft and PwC, have committed to rolling
out policies over the next decade that will set them on a path to eliminate their carbon footprints.
The understanding that these problems require holistic solutions has only grown amid the
pandemic and its fallout. President Joe Biden has referred to four urgent crises—the pandemic,
the economic crisis, racial injustice and climate change—and promised a push to tackle them all
together. The European Union’s program to propel the bloc out of the COVID-19 crisis targets
climate change, while incorporating equity concerns. As stock markets soared last year,
institutions with trillions of dollars in assets demanded that their investments deliver not only a
good return for their wallets but also a good return for society.
(Adapted from https://time.com/5931603/how-to-solve-worlds-biggest-problems/)

Questão 03 (EFOMM/INÉDITA) – Read the statements about the text and decide whether they
are TRUE (T) or FALSE (F). Mark the correct option
I. The turn of the millennium brought high expectations for the development of the world
II. Covid-19 plays an important role in explaining the interconnection between global problems
III. Despite having played a watershed role, the Covid-19 pandemic mainly affected developing
countries
IV. The intensely discriminatory past has no reflection today
V. Our current actions will affect the future of world society
(A) I – (T) / II – (F) / III – (F) / IV – (F) / V – (T)
(B) I – (T) / II – (T) / III – (F) / IV – (F) / V – (F)
(C) I – (T) / II – (T) / III – (F) / IV – (F) / V – (T)
(D) I – (F) / II – (T) / III – (F) / IV – (F) / V – (T)
(E) I – (F) / II – (T) / III – (T) / IV – (F) / V – (T)

Questão 04 (EFOMM/INÉDITA) – In the excerpt “Woven through it all, the earth’s climate is
increasingly unstable, posing an existential threat to human society as we know it”, the word in
bold means
(A) Unwoven
(B) Separate
(C) In contrast
(D) Sewn
(E) Although

AULA 03 – ARTICLES AND NOUNS 38


TEACHER ANDREA BELO

Questão 05 (EFOMM/INÉDITA) – According to the text, the current problems in the world
(A) Are actually a conspiracy theory that opposes the high expectations of the turn of the
millennium
(B) Must be resolved for the whole society, after their total exposure during the pandemic
(C) Did not worsen during the Covid-19 pandemic
(D) Have a relatively new interconnection
(E) Are not so serious, considering the high development of society

Questão 06 (EFOMM/INÉDITA) – Choose the correct option to complete the paragraph below.
I remember the last time I had _______ choose between optimism and fear, between hope and
the urge to run away. It was right after 9/11. I had babies—one so tiny she was still curled up like
a fern, _______ other toddling around gumming everything she could get her little starfish hands
on. And as their brand-new lungs took _______ the smoke that blew over to Brooklyn _______
the burning towers, I wanted to pack them up and flee to some safer place.
(Adapted from https://time.com/5930399/the-capitol-riots-and-our-fragile-optimism/)
(A) To / the / in / from
(B) The / the / on / into
(C) To / the / on / from
(D) To / the / in / into
(E) The / to / on / from

Questão 07 (EFOMM/INÉDITA) – Which is the correct way to complete the paragraph below?
For the European Union, the rapid rollout of Covid-19 vaccines __________ critical to save lives
and prevent health services from __________ stretched beyond their limits, not to mention
minimising the massive economic damage from lockdowns. Unfortunately, however, though
vaccinations are under way, a rapid near-term increase in infections is likely as the British variant
of the virus __________ across the continent.
(Adapted from https://www.concursosmilitares.com.br/provas-anteriores/marinha/efomm/2020-efomm-oficial-da-marinha-mercante-primeiro-dia.pdf)

(A) Is / being / spreads


(B) Was / be / spreads
(C) Is / be / spreading
(D) Was / being / spreading
(E) Is / being / spreading

AULA 03 – ARTICLES AND NOUNS 39


TEACHER ANDREA BELO

Questão 08 (EFOMM/INÉDITA) – Which of the following sentences expresses probability?


(A) You are not as understanding as you should be
(B) One shouldn’t accelerate in case of a red light
(C) Everyone here must have a critical sense
(D) He studied a lot, so he should be able to pass
(E) You must take good care of your body, even if you don't want to

Questão 09 (EFOMM/INÉDITA) – Choose the correct option to complete the paragraph below.
The U.S. has tragically surpassed 400,000 COVID-19 deaths, and case numbers and
hospitalizations are likewise spiking to record levels around the world. __________ vaccines now
rolling out, there is reason to hope that there is an end in sight. __________, by most estimates,
widespread vaccinations will not be in place until the middle of the year at the earliest.
__________, we have some ways to go yet with social distancing, mask wearing and other
pandemic mitigation behaviors.
(Adapted from https://www.scientificamerican.com/article/how-we-can-deal-with-pandemic-fatigue/)

(A) And / however / and


(B) With / however / so
(C) So / however / even if
(D) With / in agreement / and
(E) With / in contrast / so

Questão 10 (EFOMM/INÉDITA) – Which option is incorrect?


(A) A pandemic is affecting many people
(B) Tell the girls that they are not responsible
(C) A couple of my friends plans to travel this year
(D) I want to finish school
(E) Where are those pants I lent you?

QUESTÕES EPCAR
Directions: Read the text below and answer questions 01 to 15 according to it.
2020: The year in seven fake news stories
How should we deal with fake news? Many western governments are moving towards stricter
regulation of tech giants. But others believe that only a more educated public can stop the blight.
Have you heard that the Covid-19 vaccine will rewrite your DNA? If not, don’t worry. It is fake

AULA 03 – ARTICLES AND NOUNS 40


TEACHER ANDREA BELO

news. What is true, however, is that debate about misinformation online reached a fever pitch
this year.
From politicians calling for fake news to be banned, to others suggesting they should “weaponise”
it, the public’s ability to tell fact from fiction is increasingly contested. Here are seven stories that
show how fake news shaped this year.
First casualty. In January, Iran launched missiles at Iraqi airbases hosting US troops in retaliation
for the American assassination of general Qasem Soleimani. Many feared war. Iranian state TV
announced that 80 “American terrorists” had been killed. The USA said it experienced no
casualties. Some argued that “fake news” allowed both sides to save face and avoid conflict.
Covid-5G. By far the biggest subject for fake news this year was Covid-19. The stories were
numerous and varied, but one popular one was that Covid-19 was caused by 5G radiation. In
Britain, 5G masts were attacked, and blame was often pointed at Bill Gates in what became an
increasingly Byzantine conspiracy theory.
Dolphins of Venice. When Coronavirus lockdowns began, social media was abuzz with animal
news. Nature was healing. Venice, no longer clogged with tourists, now had clean canals down
which Dolphins cruised. Sadly, the heartening video was actually from a different region of Italy.
Viral veganism. In India, a false story that that no-one who eschewed all animal products had
contracted Covid-19 went viral. While this might not seem too harmful, the resulting drop in sales
of meat and eggs hit farmers and butchers hard. Some saw the story as part of the promotion of
Hindutva in India.
Constantine cancelled. The death of George Floyd at the hands of US police, kicked off a summer
of protest against racism. Iconoclasts tore down statues of historical figures they saw as racist.
Rumours abounded about who was next to fall. Perhaps the oddest story was that York Minster
planned to remove a statue of the Roman emperor Constantine because of his support for slavery.
The Church of England was forced to state that they had no such plans.
Rigged election. A daring raid on a CIA server in Germany revealed the plot to rig the US election
against Donald Trump… or so the fake story went. Its popularity is evidence of a stark epistemic
divide among Americans. It is just one of many casting President Trump’s loss this November as
the result of fraud. The president has yet to concede officially, but some of his statements have
been censored by social media companies, already treating him as yesterday’s fake news.
Trained Triceratops. A video of a dinosaur being unloaded from a truck went viral this month in
Indonesia. Many shared it in the belief that it was real. It turned out to be a robot filmed to
promote a theme park. While some fake news stories are all about lies and Russian spies, others
are simply about people seeing what they want to see.
How should we deal with fake news?
Reality check. Regulate! say some. Social media companies should be responsible for information
shared on their platforms. If they do not take action to stop lies being spread, then we __________
ever more anger and division. Algorithms showing people what they want to read will combine
with fake news to reinforce false beliefs. The lack of an objective standard of truth online could
become dangerous.

AULA 03 – ARTICLES AND NOUNS 41


TEACHER ANDREA BELO

Educate! say others. There have always been lies, mistakes and misrepresentations in the media.
Allowing either the government or large companies to decide what is true or false is a recipe for
disaster. What is to stop them from censoring their own critics? What is needed is the ability to
discern reliable sources for oneself. And the best way to do that is to teach people about fake
news.
(Adapted from https://theday.co.uk/stories/2020-the-year-in-seven-fake-news-stories/Acesso em: 14 de jan 2021)

Questão 01 (EPCAR/INÉDITA) – We can deduce from the first paragraph that


(A) strict regulation is the correct way of dealing with fake news.
(B) Covid-19 vaccine will likely rewrite our DNA.
(C) educating people is the only accurate way to stop fake news dissemination.
(D) there is some disagreement about how to handle fake news.

Questão 02 (EPCAR/INÉDITA) – “the public’s ability to tell fact from fiction is increasingly
contested.” (paragraph 2). Mark the correct plural form of the highlighted word.
(A) abilitys
(B) abilities
(C) abilityes
(D) abilitis

Questão 03 (EPCAR/INÉDITA) – “debate about misinformation online reached a fever pitch this
year.” (paragraph 1). Mark the correct option to make the sentence above negative.
(A) debate about misinformation online hasn’t reached a fever pitch this year.
(B) debate about misinformation online didn’t reach a fever pitch this year.
(C) debate about misinformation online did not reached a fever pitch this year.
(D) debate about misinformation online hasn’t reach a fever pitch this year.

Questão 04 (EPCAR/INÉDITA) – According to the third paragraph,


(A) 80 Americans were killed due to war.
(B) the stated fact that Iran launched missiles at Iraqi airbases was false.
(C) the described fake news did not have only downsides for some people.
(D) general Qasem Soleimani was murdered by Iran missiles.

AULA 03 – ARTICLES AND NOUNS 42


TEACHER ANDREA BELO

Questão 05 (EPCAR/INÉDITA) – Mark the option that shows the appropriate question tag for
the sentence.
The stories were numerous and varied, __________? (paragraph 4)
(A) wasn’t it
(B) were they
(C) weren’t they
(D) was it

Questão 06 (EPCAR/INÉDITA) – “Venice, no longer clogged with tourists…” (paragraph 5). It


means that
(A) Venice does not allow tourism anymore.
(B) Venice is not crowded with tourists as it used to be.
(C) Venice does not have a tourism plan any longer.
(D) Venice has been visited by many tourists lately.

Questão 07 (EPCAR/INÉDITA) – The word “eschewed” (paragraph 6) means


(A) chose
(B) consumed
(C) preferred
(D) avoided

Questão 08 (EPCAR/INÉDITA) – According to paragraph 7, The Church of England DIDN’T


(A) confirm the story that York Minster planned to remove a statue of Constantine.
(B) deny that York Minster was planning to remove a statue of Constantine.
(C) talk about the rumor that York Minster was planning to remove a statue of Constantine.
(D) say a word about the story that York Minster planned to remove a statue of Constantine.

Questão 09 (EPCAR/INÉDITA) – Mark the alternative that CANNOT replace the word “sadly”
(paragraph 5) in the text.
(A) happily
(B) unfortunately
(C) lamentably
(D) unluckly

AULA 03 – ARTICLES AND NOUNS 43


TEACHER ANDREA BELO

Questão 10 (EPCAR/INÉDITA) – The sentence “Regulate!” (paragraph 11) in the negative form is
(A) Not regulate!
(B) No regulate!
(C) Don’t regulate!
(D) Do regulate!

Questão 11 (EPCAR/INÉDITA) – Complete the blank in paragraph 11 with the grammatically


correct verb tense.
(A) will see
(B) saw
(C) have seen
(D) sees

Questão 12 (EPCAR/INÉDITA) – “however” (paragraph 1) introduces an idea of


(A) comparison.
(B) contrast.
(C) conclusion.
(D) exemplification.

Questão 13 (EPCAR/INÉDITA) – The sentence “Allowing either the government or large


companies to decide what is true or false is a recipe for disaster.” (paragraph 12) means that
(A) it is advisable to let the government or large companies decide what is true or not.
(B) countries should allow the government to decide what is true or false.
(C) people must trust large companies to decide what is true or not.
(D) it can be dangerous to let the government or large companies decide what is true or false.

Questão 14 (EPCAR/INÉDITA) – Choose the option which contains a superlative


adjective extracted from paragraph 7.
(A) racist
(B) slavery
(C) oddest
(D) iconoclasts

AULA 03 – ARTICLES AND NOUNS 44


TEACHER ANDREA BELO

Questão 15 (EPCAR/INÉDITA) – Mark the sentence below that can answer the following
question correctly. “Have you heard that the Covid-19 vaccine will rewrite your DNA?”
(paragraph 1).
(A) Yes, I’ve.
(B) No, I haven’t.
(C) No, I’ve not.
(D) Yes, I had.

QUESTÕES ESA
Texto para responder à questão 01.
RIO DE JANEIRO, BRAZIL – In the past 24 hours, Brazil recorded 1,113 deaths and 36,653 new cases
related to the novel coronavirus. The data are included in the Ministry of Health's daily balance
released on Tuesday evening, September 15th.
Since the start of the pandemic, 133,119 people have died as a result of Covid-19. On Monday,
the Ministry of Health's data system recorded a total of 132,006 deaths. A further 2,445 are still
under investigation by health authorities.
(Adapted from https://riotimesonline.com - September 15th)

Questão 01 (ESA/INÉDITA) – According to the text, it is correct to say that:


A) more than a thousand hundred deaths were recorded.
B) a thousand and half deaths were registered.
C) many people died becaise of the coronavirus.
D) coronavirus left more than a thousand people sick.
E) more than thirty six thousand deaths were registered.

Questão 02 (ESA/INÉDITA) – Which sentence is grammatically correct?


(A) Last week, I worked out more earlieron Tuesday than the rest of the week.
(B) Albert is more big than his boss.
(C) My apartment is more big than your office downtown.
(D) Traveling to Europe is expensiver than going to the USA.
(E) Anne Claire’s sneakers are cheaper than mine.

AULA 03 – ARTICLES AND NOUNS 45


TEACHER ANDREA BELO

Questão 03 (ESA/INÉDITA) – Complete the sentence below using the appropriate words:
“The old man who works here __________ always complaining: He __________ like children
but __________ talking about them to everyone.”
(A) is /don’t/love
(B) is/doesn’t/loves
(C) isn’t/do/loves
(D) isn’t/don’t/love
(E) is/does/love

Questão 04 (ESA/INÉDITA) – “__________ my new neighbour? Where did you come from?”
Complete the space with the correct form of the verb and the pronoun.
(A) You is
(B) You are
(C) Are you
(D) Is you
(E) Am you

Texto para as questões 05, 06 e 07.


September 16th 2020.
RIO DE JANEIRO, BRAZIL – Eternit, a manufacturer of building materials, has been awarded a
certificate by INMETRO (Product Certification Body) for a concrete roof tile capable of producing
energy. As a result, the company can now market the product. The roof tile collects energy from
the sun (…)
(Adapted from https://riotimesonline.com)

Questão 05 (ESA/INÉDITA) – According to the text, it is correct to say that:


A) Eternit company received an award.
B) it is said Eternit tried to receive an award.
C) the award was specifically to different companies.
D) more than a company got a certificate.
E) the building materials provided an award to the company.

AULA 03 – ARTICLES AND NOUNS 46


TEACHER ANDREA BELO

Questão 06 (ESA/INÉDITA) – The term “roof tile” refers to the


(A) ceiling
(B) ranch
(C) rope
(D) race
(E) couple

Questão 07 (ESA/INÉDITA) – The sentence “...collects energy from the sun” means that:
(A) it refers to a different kind of energy.
(B) it refers to solar energy.
(C) it refers to a brandnew type of energy.
(D) it refers to pools in the garden.
(E) it is made by natural elements.

Questão 08 (ESA/INÉDITA) – “__________ writing you emails? Yes, she is wirting lots of them
day by day and I also answer her with poems on the emails. We are in love.”
Complete the space with the correct form of the verb and the pronoun.
(A) Are the girl
(B) She is
(C) Is she
(D) Are she
(E) Am she

Questão 09 (ESA/INÉDITA) – The word “them” in “She agrees with all of them” is
(A) a pronoun
(B) a conjunction
(C) a preposition
(D) a noun
(E) an adjective

Questão 10 (ESA/INÉDITA) – The word “naive” in “I think he is a little naive” is


(A) a verb
(B) a conjunction
(C) a preposition
(D) a adverb
(E) an adjective

AULA 03 – ARTICLES AND NOUNS 47


TEACHER ANDREA BELO

QUESTÕES ESCOLA NAVAL


Covid babies don't have to be the unlucky generation. But they must be helped
A year into this pandemic, there are babies now learning to walk and talk who have never known
anything but life under the shadow of Covid, and preschoolers who can barely remember a world
before it. Doctors' children have had to learn not to touch Mummy when she gets in from work,
until she's had a shower to wash off any last possible trace of danger. Thanks to popular toddler
demand, you can now buy masks for dolls.
It's perfectly normal for kids to reflect what's happening around them by playing quarantine with
stuffed animals, but normal too for adults to wonder uncomfortably whether all this leaves a
lasting mark. How much will it matter in years to come that, as the minister for loneliness Diana
Barran recently put it, there are toddlers being raised by shielding parents who have never had a
playdate? Will Covid babies grow up solitary creatures, used to entertaining themselves, or warier
of the strangers they so rarely meet and interact with? Taking a tiny bundle out in public used to
mean an endless succession of random older women cooing over the pram, or strangers pulling
faces to entertain a bored baby in a checkout queue. But now passersby daren't get close, and
other shoppers are hidden behind masks.
This week MPs were presented with some early findings from a project led by the First 1001 Days
Movement, an alliance of early-years charities and professionals, tracking the lives of under-twos
growing up through a pandemic. A survey of children's service providers it commissioned found
98% thought the babies and toddlers they worked with had been affected by higher parental
stress and anxiety, while 92% had seen fearful families effectively cutting themselves off from the
outside world, skipping routine appointments or not wanting to leave the house. Nine in 10 had
observed children being played with less, or being less active. Heartbreakingly, more than a
quarter said lockdown left the children they worked with more exposed to domestic conflict,
abuse or neglect.
(Adapted from https://mmm.theguardian.com/commentisfree/2021/jan/22/covid-babies-unlucky-generation-help-lockdown)

Questão 01 (ESCOLA NAVAL/INÉDITA) – According to the text, which option is correct?


(A) There are children who have not yet experienced life outside the pandemic
(B) The pandemic has not affected the lives of any children, as they do not understand what is
happening
(C) The pandemic, in a way, positively impacted the babies' lives
(D) The children's routine has not changed in this pandemic
(E) Characteristics of the pandemic did not impact areas such as sales

Questão 02 (ESCOLA NAVAL/INÉDITA) – What’s the meaning of the word “toddler” in


paragraph 1?
(A) Population
(B) Children
(C) Babies
(D) Adults
(E) Teens

AULA 03 – ARTICLES AND NOUNS 48


TEACHER ANDREA BELO

Questão 03 (ESCOLA NAVAL/INÉDITA) – In paragraph 1, the word “she” refers to


(A) Children
(B) Doctor
(C) Toddler
(D) Mummy
(E) Babies

Questão 04 (ESCOLA NAVAL/INÉDITA) – According to the text, which option completes the
sentence below, correctly? The pandemic __________ an entire generation.
(A) Affects
(B) Affected
(C) Was affecting
(D) Had affected
(E) Is affecting

Questão 05 (ESCOLA NAVAL/INÉDITA) – According to the text, which option is correct?


(A) Children do not realize what happens around them
(B) The impact of the pandemic on children is a normal concern of parents
(C) There is no doubt about this generation that was born in the middle of a pandemic
(D) The relationship with children on the street changed with the pandemic
(E) The impact of the pandemic on children can only be short-term

QUESTÕES EsPCEx
Escolha a única alternativa correta, dentre as opções apresentadas, que responde ou completa
cada questão, assinalando-a.
Leia o texto a seguir e responda às questões 01, 02 e 03.
Hair ice: The strange phenomenon of 'candy floss' on trees
If you (…) for a winter walk, you can be in for a big surprise. People taking a stroll through the trees
in counties Fermanagh and Tyrone have been encountering an unusual sight. At first glance, it
looks like candy floss has appeared on tree branches. On closer inspection, you can see hundreds
of individual strands of what looks like delicate white hair.
As soon as these are touched by human hand or winter sun, they melt away. This strange
phenomenon is called hair ice. The crystals are formed on rotting wood on humid winter nights
when the temperature is just below zero. Scientists have discovered it is caused by a fungus which
enables the ice to form thin hairs with a diameter of about 0.01mm.

AULA 03 – ARTICLES AND NOUNS 49


TEACHER ANDREA BELO

Hair ice - also known as frost flowers - has been spotted in County Fermanagh in Castle Archdale,
Florence Court, Castle Coole, Cladagh Glen and Big Dog Forest. In County Tyrone, it has been
pictured in Omagh and Knockmany, near Augher. So if you're heading out for some daily exercise,
see if you can spot this rare phenomenon before it melts away.
Adapted from https://www.bbc.com/news/uk-northern-ireland-55531529

Questão 01 (EsPCEx/INÉDITA) – Choose the statement which correctly functions as the missing
part of paragraph 1. If you (…) for a winter walk, you can be in for a big surprise.
(A) ’re looking forward
(B) dive into the ocean
(C) go down to the woods
(D) desperately need
(E) went

Questão 02 (EsPCEx/INÉDITA) – Choose the alternative that has the same meaning as the word
glance in the sentence “At first glance, it looks like candy floss has appeared on tree branches.”
(paragraph 1).
(A) research
(B) gender
(C) inquiry
(D) option
(E) look

Questão 03 (EsPCEx/INÉDITA) – According to the text, choose the correct alternative.


(A) Hair ice refers to actual hair.
(B) Scientists are yet to find out what causes hair ice.
(C) Frost flowers are a more common type of hair ice.
(D) Humans can hurt their hands by touching frost flowers.
(E) Frost flowers are formed due to a fungus.

Leia o texto a seguir e responda às questões 04, 05 e 06.


Where is Jack Ma? Chinese tycoon not seen since October
Speculation is mounting over the whereabouts of the Chinese billionaire Jack Ma, who has not
been seen or heard in public for more than two months. Ma, the co-founder and former chairman
of the technology firm Alibaba, has fallen out of favour with China’s leadership. In late October, he

AULA 03 – ARTICLES AND NOUNS 50


TEACHER ANDREA BELO

stood alongside senior officials and delivered a blunt speech criticizing national regulators,
reportedly infuriating China’s president, Xi Jinping.
In the following months, regulators summoned Ma and other executives in for questioning and
halted what would have been the world’s biggest share offering of his company, Ant Group. They
later launched anti-monopoly investigations into Alibaba and its key competitor TenCent, and
called Ant Group in for questioning. In late December, regulators ordered Ma to pare down his
empire.
Ma has not been heard from during the controversies. His last public appearance appears to have
been at a livestreamed event on 31 October, an opening ceremony of the annual Chinese shopping
event Double 11. In November, he reportedly did not appear in his role as a judge for the final
episode of entrepreneurial game show Africa’s Business Heroes, which Alibaba has since blamed
on a scheduling conflict.
Ma was replaced with another Alibaba executive and his photo removed from the website. He
also did not appear at this year’s Shanghai Zhejiang chamber of commerce meeting, where he has
delivered a speech every year since 2016. Bloomberg reported Ma had been warned in early
December not to leave the country. Observers are being cautious in drawing any conclusions, with
so little confirmed information.
Adapted from https://www.theguardian.com/business/2021/jan/05/where-is-jack-ma-chinese-tycoon-not-seen-since-october-alibaba

Questão 04 (EsPCEx/INÉDITA) – In the title, the word tycoon refers to


(A) a powerful company.
(B) a weather phenomenon.
(C) bankruptcy.
(D) a wealthy person.
(E) an underprivileged person.

Questão 05 (EsPCEx/INÉDITA) – In the sentence “Ma, the co-founder and former


chairman of the technology firm Alibaba” (paragraph 1), the prefix co
(A) means sharing a job or responsibility.
(B) gives the opposite meaning of founder.
(C) means Ma does not own Alibaba any longer.
(D) describes something that was poorly executed.
(E) means acting by oneself.

AULA 03 – ARTICLES AND NOUNS 51


TEACHER ANDREA BELO

Questão 06 (EsPCEx/INÉDITA) – Choose the statement in which the word deliver or one of its
forms has been used in the same way as in paragraph 4.
(A) Many goods were not delivered in time for Christmas.
(B) She delivered her first child at home.
(C) Is there anything that can be done to deliver the world from Covid?
(D) The priest delivered a touching sermon against hate.
(E) The store is delivering my new wardrobe anytime now.

Leia o texto a seguir e responda às questões 07, 08 e 09.


‘Bean dad’ apologises after tin can posts cause outcry
A dad in the US who boasted about his parenting skills after telling his hungry nine-year-old
daughter to open a tin of beans or go without food says he is “deeply sorry” about his tweets.
John Roderick, a podcaster and musician, said in a statement that he wanted to “acknowledge and
make amends for the injuries” caused by his posts. He said they were framed in a certain way to
reflect his “comedic persona”.
Twitter users, who nicknamed him “bean dad”, condemned the reported incident, suggesting it
was poor parenting. In an apology posted on his website on Tuesday, Mr Roderick said he had
been “ignorant” and “insensitive” with the way he had described the incident.
Mr Roderick, who has since deactivated his Twitter account, said he was wrong for being “flippant
when confronted” and for “taking my Twitter feed offline yesterday instead of facing the music”.
What was the Twitter story? Mr Roderick shared a story on Saturday on Twitter, beginning with
his daughter asking him to make baked beans. After she brought him a tin-opener and can of
beans, he asked her how she thought a tin-opener worked, he said. When she said she didn't
know, he said he realised “a teaching moment just dropped into my lap”. Explaining that he wanted
his daughter to learn how to open a tin of beans, he said she tried for six hours.
Eventually she opened the tin and ate the beans, he explained, claiming it was a victory for “good
parenting”. But some Twitter users accused Mr Roderick of “child abuse”, suggesting his actions
were “ridiculous” and that it was right for his daughter to ask for “help and support”.
Adapted from https://www.bbc.com/news/world-us-canada-55549536

Questão 07 (EsPCEx/INÉDITA) – According to the text, choose the correct alternative.


(A) Roderick’s daughter never managed to open the can.
(B) Roderick showed no remorse for his behavior.
(C) According to Roderick, he was trying to teach his daughter a life lesson.
(D) Roderick uses “bean dad” as a stage name on his stand-up comedy show.
(E) Roderick’s daughter did not try long to open the tin.

AULA 03 – ARTICLES AND NOUNS 52


TEACHER ANDREA BELO

Questão 08 (EsPCEx/INÉDITA) – Which one from the underlined verbs in the text conveys a
different verb form?
(A) caused
(B) nicknamed
(C) framed
(D) posted
(E) described

Questão 09 (EsPCEx/INÉDITA) – Choose the alternative that has the same meaning as the word
eventually in the sentence “Eventually she opened the tin and ate the beans, he explained,
claiming it was a victory for ‘good parenting.’” (paragraph 5).
(A) occasionally.
(B) in the end.
(C) in the beginning.
(D) comprehensively.
(E) after years.

Leia o texto a seguir e responda à questão 10.


New COVID Vaccines Need Absurd Amounts of Material and Labor
The first doses of mRNA COVID vaccines began arriving at hospitals in the U.S. and several other
countries in December. An overarching question is how fast companies making them can scale up
production to meet global demand. This is the first time mRNA vaccines have been authorized for
use outside of clinical trials. They work by tricking the body’s own cells into making a viral protein
that prompts immune reactions against infection. The U.S. has granted emergency authorization
to two COVID vaccines so far—made by the pharmaceutical companies Pfizer and BioNTech and
the biotechnology firm Moderna, respectively—and both rely on mRNA.
The Trump administration reached a deal with Pfizer in late December to provide 100 million
additional doses to the U.S. by the end of July—resulting in twice the amount the government
originally ordered. Together with its partner BioNTech, Pfizer plans to produce and distribute 1.3
billion doses globally next year. And Moderna intends to produce 500 million to one billion doses,
of which 200 million have already been allocated to the U.S.
Meeting these targets will be no easy feat. Each step in the manufacturing process requires raw
materials that, before COVID, were only produced in the amounts needed for clinical research.
Such mRNA vaccines are created using much faster chemical processes than traditional vaccines
made by growing weakened viruses in chicken eggs. According to a November report by the U.S.
Government Accountability Office (GAO), much of what is needed to produce these vaccines is in
short supply.
Adapted from https://www.scientificamerican.com/article/new-covid-vaccines-need-absurd-amounts-of-material-and-labor/

AULA 03 – ARTICLES AND NOUNS 53


TEACHER ANDREA BELO

Questão 10 (EsPCEx/INÉDITA) – According to the text, read the statements and choose the
correct alternative.

I – mRNA vaccines are usually used besides clinical trials.

II – mRNA vaccines creation process is different from the traditional one.

III – mRNA vaccines stimulate the body’s cells to create a viral protein.

IV – It is known that companies can increase vaccines production to a global scale.

V – Both vaccines the U.S. has authorized up to now are mRNA vaccines.

(A) I, II and III are correct.

(B) II, III and IV are correct.

(C) I, IV and V are correct.

(D) II, IV and V are correct.

(E) II, III and V are correct.

AULA 03 – ARTICLES AND NOUNS 54


TEACHER ANDREA BELO

GABARITO
GABARITO AFA
01 – B 02 – C 03 – D 04 – C 05 – A

GABARITO COLÉGIO NAVAL


01 – E 02 – A 03 – D 04 – B 05 – D

GABARITO EAM
01 – E 02 – A 03 – B 04 – D 05 – D

GABARITO EEAR
01 – D 02 – D 03 – B 04 – D 05 – D

GABARITO EFOMM
01 – A 02 – E 03 – C 04 – D 05 – B
06 – A 07 – A 08 – D 09 – B 10 – C

GABARITO EPCAR
01 – D 02 – B 03 – B 04 – C 05 – C
06 – B 07 – D 08 – A 09 – A 10 – C
11 – A 12 – B 13 – D 14 – C 15 – B

GABARITO ESA
01 – A 02 – E 03 – B 04 – C 05 – A
06 – A 07 – B 08 – C 09 – A 10 – E

GABARITO ESCOLA NAVAL


01 – A 02 – B 03 – D 04 – E 05 – B

GABARITO EsPCEx
01 – C 02 – E 03 – E 04 – D 05 – A
06 – D 07 – C 08 – B 09 – B 10 – E

AULA 03 – ARTICLES AND NOUNS 55


TEACHER ANDREA BELO

QUESTÕES COMENTADAS
QUESTÕES AFA
The end of life on Earth?
It weighed about 10,000 tons, entered the atmosphere at a speed of 64,000km/h and exploded
over a city with a blast of 500 kilotons. But on 15 February 2013, we were lucky. The meteorite
that showered pieces of rock over Chelyabinsk, Russia, was relatively small, at only about 17
metres wide. Although many people were injured by falling glass, the damage was nothing
compared to what had happened in Siberia nearly one hundred years ago. Another relatively small
object (approximately 50 metres in diameter) exploded in mid-air over a forest region, flattening
about 80 million trees. If it had exploded over a city such as Moscow or London, millions of people
would have been killed.
By a strange coincidence, the same day that the meteorite terrified the people of Chelyabinsk,
another 50m-wide asteroid passed relatively close to Earth. Scientists were expecting that visit
and know that the asteroid will return to fly close by us in 2046, but the Russian meteorite earlier
in the day had been too small for anyone to spot.
Most scientists agree that comets and asteroids pose the biggest natural threat to human
existence. It was probably a large asteroid or comet colliding with Earth which wiped out the
dinosaurs about 65 million years ago. An enormous object, 10 to 16km in diameter, struck the
Yucatan region of Mexico with the force of 100 megatons. That is the equivalent of one Hiroshima
bomb for every person alive on Earth today.
Many scientists, including the late Stephen Hawking, say that any comet or asteroid greater than
20km in diameter that hits Earth will result in the complete destruction of complex life, including
all animals and most plants. As we have seen, even a much smaller asteroid can cause great
damage.
The Earth has been kept fairly safe for the last 65 million years by good fortune and the massive
gravitational field of the planet Jupiter. Our cosmic guardian, with its stable circular orbit far from
the sun, sweeps up and scatters away most of the dangerous comets and asteroids which might
cross Earth’s orbit. After the Chelyabinsk meteorite, scientists are now monitoring potential
hazards even more carefully but, as far as they know, there is no danger in the foreseeable future.
Types of space rocks
• Comet – a ball of rock and ice that sends out a tail of gas and dust behind it. Bright comets
only appear in our visible night sky about once every ten years.
• Asteroid – a rock a few feet to several kms in diameter. Unlike comets, asteroids have no
tail. Most are too small to cause any damage and burn up in the atmosphere. They appear
to us as ‘shooting stars’.
• Meteoroid – part of an asteroid or comet.
• Meteorite – what a meteoroid is called when it hits Earth.
Taken from: https://learnenglishteens.britishcouncil.org/skills/reading/upper-intermediate-b2-reading/end-life-earth

AULA 03 – ARTICLES AND NOUNS 56


TEACHER ANDREA BELO

Questão 01 (AFA/INÉDITA) – The damage caused by the Russian meteorite


a) could be impossible to calculate.
b) was not as bad as the one in Siberia.
c) was much worse than the one in Siberia.
d) had its impact reduced by the early warning system.
Comentários: O enunciado pede o que foi causado pelo meteorito russo.
Na letra A, diz que could be impossible, seria impossível calcular, porém o texto usa dados. Então,
seria possível calcular. Alternativa incorreta.
Na letra B, diz que não foi tão ruim quanto o meteorito as Sibéria, not as bad as, exatamente o
que podemos dizer pois, no texto, na frase Although many people [...], confirmamos que ele foi
pequeno em relação ao da Sibéria.
Na letra C, diz que ele foi pior que o da Sibéria, porém, vimos que é ao contrário. O da Sibéria foi
o mais violento com maiores consequências.
Na letra D, diz que ele teve o impacto reduzido por algum tipo de sistema, mas não, porque ele
era pequeno demais para ser detectado pelo sistema. Então, por ele ser minúsculo, ele não foi
detectado e está incorreto, confirmando assim que o nosso gabarito é a alternativa B.
GABARITO: B

Questão 02 (AFA/INÉDITA) – Without changing that meaning the word “Although” (line 6) could
be substituted for:
a) however
b) therefore
c) despite
d) once
Comentários: O enunciado pergunta qual palavra substitui Although sem mudar o sentido
(without changing the meaning).
Na letra A, temos a palavra however que, apesar de significar ‘contudo’, ‘entretanto’ e às vezes ser
usada com sentido de Although, ela não cabe nessa questão, pois ele diz sem mudar o sentido,
without changing that meaning, como está no enunciado. Portanto, however com sentido de
‘entretanto’ não cabe na questão.
Na letra B, therefore tem o sentido de ‘portanto’, ou seja, conclusão. Também não cabe na frase
do texto que foi pedida a ser analisada na linha 6.
Na letra C, despite significa ‘apesar de’, ‘embora’ e, por mais que não seja a melhor alternativa, é
a que mais se aproxima ao que o texto quer dizer na frase que foi destacada.
Na letra D, a palavra once significa ‘já que’, ‘uma vez que’ e não tem sentido com a frase do texto
em que ele pede para substituir. Portanto, nosso gabarito é realmente a letra C.
GABARITO: C

AULA 03 – ARTICLES AND NOUNS 57


TEACHER ANDREA BELO

Questão 03 (AFA/INÉDITA) – The statement “many people were injured by falling glass” (line 7)
stands for: Failing glass _______ many people
a) have injured
b) has injured
c) had injured
d) injured
Comentários: Essa questão é, basicamente, para quem estudou Voz Passiva e suas estruturas. Se
a frase é “many people were injured by falling glass”, então, o falling glass que estava no final fica
automaticamente para o início da frase na Voz Ativa. Se people, que é ‘pessoas’, were injured,
foram prejudicadas, então os vidros prejudicaram, como vimos na estrutura em nossas aulas
(verbo no passado simples na voz ativa, fica com verb to be no passado e o verbo principal no past
simple). Se tem Passado, fica Passado com o verb to be were, porque são pessoas no plural.
Portanto, letra A, have injured, não cabe, pois o have não está nessa questão de estrutura.
Na letra B, has injured também não, porque ‘pessoas’ nem teria que usar have, plural.
Na letra C, had injured não, porque não tem estrutura had nesse tipo de frase, em que se usa past
simple e não estruturas de present perfect.
E a letra D é o nosso gabarito. Se injured está no Passado, teria que ficar “were injured” na Voz
Passiva, assim como está no enunciado.
GABARITO: D

Questão 04 (AFA/INÉDITA) – The passage “the damage was nothing compared to what had
happened in Siberia nearly one hundred years ago” (lines 7 to 8) states that the incident
occurred _______ a country ago.
a) actually
b) precisely
c) approximately
d) exactly
Comentários: O enunciado destaca a frase com a estrutura nearly one hundred years ago, ou seja,
próximo/perto de cem anos atrás. E as alternativas que temos são:
Letra A, actually, que significa ‘de fato’, ‘na verdade’, não cabe na lacuna acima.
Letra B, precisely, seria ‘precisamente’ e também não cabe.
Letra C, approximately, que significa ‘aproximadamente’, exatamente isso, porque ‘perto de cem
anos’ é ‘aproximadamente cem anos’ e, por isso, é o nosso gabarito.
Letra D, exactly, seria ‘exatamente’ cem anos mas vimos que nearly é ‘aproximadamente’ e não
‘exatamente’. Portanto, nosso gabarito é a alternativa C.
GABARITO: C

AULA 03 – ARTICLES AND NOUNS 58


TEACHER ANDREA BELO

Questão 05 (AFA/INÉDITA) – The Siberian meteorite


a) damaged trees when it exploded.
b) caused glass to shower over people.
c) has the destructive power of a nuclear bomb.
d) exploded over a big city.
Comentários: O enunciado pede para confirmar o que aconteceu com o meteorito da Sibéria.
Na letra A, diz que danificou árvores quando explodiu, e podemos confirmar essa informação no
texto quando diz que gerou estrago de oitenta milhões de árvores, como nesse trecho do primeiro
parágrafo: “Another relatively small object (approximately 50 metres in diameter) exploded in mid-
air over a forest region, flattening about 80 million trees...” Portanto, é o nosso gabarito.
Na letra B, diz que foi causada uma chuva de vidro, mas, não foi do meteorito as Sibéria e sim do
outro recente. Por isso, está incorreta.
Na letra C, diz que teve destruição semelhante a uma bomba nuclear, mas não, pois este é um
evento hipotético no texto.
Na letra D, diz que explodiu em uma cidade, porém, o que explodiu ao redor, acima da cidade foi
o outro meteorito de cem anos atrás e não esse da Sibéria. Portanto também está incorreta, e o
nosso gabarito é a letra A.
GABARITO: A

QUESTÕES COLÉGIO NAVAL


Directions: Answer questions 01 to 05 according to TEXT.
Facebook complies with Brazilian judge’s order to block 12 accounts accused of running a fake
news network
Facebook announced Saturday it has obeyed a Brazilian judge’s order for a worldwide block on
the accounts of 12 of President Jair Bolsonaro’s supporters who are under investigation for
allegedly running a fake news network.
Supreme Court Justice Alexandre de Moraes said Friday night that the company had failed to fully
comply with a previous ruling ordering the accounts to be shut down, saying they were still online
and publishing by changing their registration to locations outside Brazil.
Facebook issued a statement saying it complied due to the threat of criminal liability for an
employee in Brazil.
But it called the new order “extreme,” saying it poses a “threat to freedom of expression outside
of Brazil’s jurisdiction and conflicting with laws and jurisdictions worldwide.” The company said it
would appeal to the full court.
Facebook also argued it had complied with the previous order by “restricting the ability for the
target Pages and Profiles to be seen from IP locations in Brazil”.

AULA 03 – ARTICLES AND NOUNS 59


TEACHER ANDREA BELO

“People from IP locations in Brazil were not capable of seeing these Pages and Profiles even if the
targets had changed their IP location”, the company said.
Moraes said that Facebook ought to pay $ 367,000 in penalties for not complying with his previous
decision during the last eight days.
He also had ruled Twitter should block the accounts. While Twitter said then the decision was
“disproportionated” under Brazil’s freedom of speech rules and that it would appeal, the targeted
profiles were disabled.
Moraes is overseeing a controversial investigation to determine whether some of Bolsonaro’s
most ardent allies are running a social media network aimed at spreading threats and fake news
against Supreme Court justices.
The probe is one of the main points of confrontation between Bolsonaro and the Supreme Court.
The president himself filed a lawsuit last week demanding the accounts to be unblocked.
(Adapted from https://time.com/5874695/facebook-blocks-accounts-worldwide/)

Questão 01 (COLÉGIO NAVAL/INÉDITA) – Read the extract from the text.


“People from IP locations in Brazil were not capable of seeing these Pages and Profiles even if
the targets had changed their IP location”.
Mark the option that replaces correctly the expression “not capable”.
a) Uncapable
b) Competent
c) Qualified.
d) Efficient
e) Incapable.
Comentários: A alternativa A está incorreta. A outra forma correta de se falar “not capable” (não
capaz) é “incapable”, e não “uncapable”.
A alternativa B está incorreta. A expressão “competent” significa competente, enquanto “not
capable” significa não capaz.
A alternativa C está incorreta. A expressão “qualified” significa qualificado, enquanto “not
capable” significa não capaz.
A alternativa D está incorreta. A expressão “efficient” significa eficiente, enquanto “not capable”
significa não capaz.
A alternativa E está correta. A expressão “not capable” pode ser escrita da forma “incapable”, que
seu sentifo não irá mudar (incapaz).
GABARITO: E

AULA 03 – ARTICLES AND NOUNS 60


TEACHER ANDREA BELO

Questão 02 (COLÉGIO NAVAL/INÉDITA) – Read the extract from the text.


“Alexandre de Moraes said Friday night that the company had failed to fully comply with a previous
ruling ordering the accounts to be shut down”.
Choose the correct question for the sentence below.
a) What was the supreme statement on Facebook’s first attitude?
b) At first, did Facebook comply with the order of Alexandre de Moraes?
c) What was the final answer from Facebook to the Brazilian Supreme Court Justice?
d) Were the accounts blocked?
e) Who is Alexandre de Moraes?
Comentários: A alternativa A está correta. A frase dada indica que a pergunta se referia ao relato
do Supremo sobre a primeira atitude do Facebook.
A alternativa B está incorreta. A frase dada não indica que a pergunta se referia especificamente
à primeira atitude de Facebook com relação a ordem do supremo, mas sim ao relato do Supremo
sobre a primeira atitude do Facebook.
A alternativa C está incorreta. A frase dada não indica que a pergunta se referia a resposta final do
Facebook ao Supremo, mas sim ao relato do Supremo sobre a primeira atitude do Facebook.
A alternativa D está incorreta. A frase dada não indica que a pergunta se referia ao bloqueamento
das contas, mas sim ao relato do Supremo sobre a primeira atitude do Facebook.
A alternativa E está incorreta. A frase dada não indica que a pergunta se refere a quem é Alexandre
de Moraes, mas sim ao relato do Supremo sobre a primeira atitude do Facebook.
GABARITO: A

Questão 03 (COLÉGIO NAVAL/INÉDITA) – Read the extract from the text.


“Moraes said that Facebook ought to pay $ 367,000 in penalties for not complying with his
previous decision during the last eight days.”
Mark the option that can replace the underlined sentence correctly.
a) For not helping with his latest decision during the last eight days.
b) For cooperating with his previous decision during the last eight days.
c) For assisting with the supreme first decision during the last eight days.
d) For not cooperating with the supreme first decision during the last eight days.
e) For cooperating with his decision.
Comentários: A alternativa A está incorreta. Esta opção expressa que não há colaboração com a
última decisão de Moraes durante os últimos 8 dias, em contrapartida com a frase sublinhada,
que expressa que não há colaboração com a decisão anterior e Moraes durante os últimos 8 dias.

AULA 03 – ARTICLES AND NOUNS 61


TEACHER ANDREA BELO

A alternativa B está incorreta. Esta opção expressa que há colaboração com a decisão anterior de
Moraes durante os últimos 8 dias, em contrapartida com a frase sublinhada, que expressa que
não há colaboração com a decisão anterior e Moraes durante os últimos 8 dias.
A alternativa C está incorreta. Esta opção expressa que há assistência com a decisão anterior de
Moraes durante os últimos 8 dias, em contrapartida com a frase sublinhada, que expressa que
não há colaboração com a decisão anterior e Moraes durante os últimos 8 dias.
A alternativa D está correta. Esta opção expressa que não há colaboração com a decisão anterior,
ou seja, a primeira, de Moraes durante os últimos 8 dias, assim como a frase dada.
A alternativa E está incorreta. Essa opção expressa que há colaboração com a decisão, sem
especificar, em contrapartida com a frase sublinhada, que expressa que não há colaboração com
a decisão anterior e Moraes durante os últimos 8 dias.
GABARITO: D

Questão 04 (COLÉGIO NAVAL/INÉDITA) – According to the seventh paragraph, we can assume


that.
a) Twitter agreed immediately with the decision, without protesting.
b) Even without agreeing, twitter complied with the order.
c) Twitter didn’t respond to the supreme court order.
d) Twitter protested to the order and failed to comply.
e) Twitter was not advertised.
Comentários: A alternativa A está incorreta. O sétimo parágrafo não afirma que o Twitter
concordou imediatamente com a decisão sem protestar, mas sim que concordou com a decisão
contra sua vontade.
A alternativa B está correta. Mesmo sem concordar, o Twitter colaborou com a ordem dada, assim
como afirma o sétimo parágrafo
A alternativa C está incorreta. O sétimo parágrafo não afirma que o Twitter não respondeu a
ordem da Suprema Corte, mas sim que concordou com a decisão contra sua vontade.
A alternativa D está incorreta. O sétimo parágrafo não afirma que o Twitter não colaborou e
protestou a respeito da ordem dada, mas sim que concordou com a decisão contra sua vontade.
A alternativa E está incorreta. O sétimo parágrafo não afirma que o Twitter não foi avisado, mas
sim que concordou com a decisão contra sua vontade.
GABARITO: B

Questão 05 (COLÉGIO NAVAL/INÉDITA) – About the Brazilian President, it is correct to affirm


that.
a) The relationship between him and the supreme court is peaceful.
b) He readily agreed with the supreme decision.
c) He did not comment on the decision.
d) He was against the supreme court decision.
e) He agreed and cooperated with the supreme.

AULA 03 – ARTICLES AND NOUNS 62


TEACHER ANDREA BELO

Comentários: A alternativa A está incorreta. Não é correto afirmar que a relação do Presidente do
Brasil com a Suprem Corte é pacífica, mas sim que era uma relação com confrontos.
A alternativa B está incorreta. Não é correto afirmar que ele concordou imediatamente com a
decisão do Supremo, mas sim que ele foi contra e tentou derrubá-la.
A alternativa C está incorreta. Não é correto afirmar que ele não comentou sobre a decisão, mas
sim que ele foi contra e tentou derrubá-la.
A alternativa D está correta. É correto afirmar que ele foi contra a decisão da Suprem Corte, assim
como o texto afirma.
A alternativa E está incorreta. Não é correto afirmar que ele concordou e colaborou com o
supremo, mas sim que ele foi contra e tentou derrubá-la.
GABARITO: D

QUESTÕES EAM
Questão 01 (EAM/INÉDITA) – Read the dialogue and mark the right option to fill in the gaps
respectively.
A: _________ you want to go to the beach yesterday?
B: Yes, I ________.
A: Who ________ you want to go with?
B: I ________ to go with my sister. Did you go?
A: No, I ________.
A) Did / did / went / want / wasn’t
B) Were / was / were / want / didn’t
C) Do / do / did / wanted / wasn’t
D) Do / do / did / wanted / didn’t
E) Did / did / did / wanted / didn’t
Comentários: A primeira lacuna deve ser preenchida por “did”. A presença do verbo “want” no
infinitivo indica que há um auxiliar “do, did, will” no lugar da lacuna. A presença da palavra
“yesterday” no fim da frase indica que a frase está no passado. O passado exige o auxiliar “did”
para fazer a pergunta.
A segunda lacuna deve ser preenchida por “did”. A pergunta foi feita com “did”, portanto, a
resposta curta deve ser dada utilizando o mesmo auxiliar (Yes, I did).
A terceira lacuna deve ser preenchida por “did”. A pergunta se refere à vontade do dia anterior
e, por isso, deve-se usar o auxiliar “did” para perguntar com quem a pessoa queria ir.
A quarta lacuna deve ser preenchida por “wanted”. A conversa está se passando toda no tempo
passado. Por isso, devemos usar “wanted” ao invés de “want”.

AULA 03 – ARTICLES AND NOUNS 63


TEACHER ANDREA BELO

A quinta lacuna deve ser preenchida por “didn’t”. A pergunta foi feita com o auxiliar “did”, “did
you go?”, logo, a resposta deve também utilizar o auxiliar “did” na negativa, já que a resposta
começa com a palavra “no”.
GABARITO: E

Questão 02 (EAM/INÉDITA) – Read the sentences and mark the correct option to fill in the
blanks respectively.
__________ name is Bella. She’s 17 and I live __________ Brasilia which is the capital of Brazil.
Actually she comes from another city – Rio, where she __________ for fifteen years.
A) Her / in / lived
B) Your / at / lived
C) Her / on / lives
D) My / in / live
E) His / in / live
Comentários: A primeira lacuna deve ser preenchida por “Her”. O início do período seguinte deixa
claro que o texto trata de uma mulher, ao começar com o pronome “She”.
A segunda lacuna deve ser preenchida por “in”. Por regra, sempre que vai se falar EM alguma
cidade, usa-se a preposição “in”.
A terceira lacuna deve ser preenchida por “lived”. No trecho anterior, ele diz que ela vive em
Brasília. Portanto, se torna claro que o verbo da lacuna deve estar no passado para dizer de onde
ela veio e onde ele viveu.
GABARITO: A

Questão 03 (EAM/INÉDITA)

What’s the main verb tense used in the comic strip?


A) Present Continuous
B) Present Simple
C) Simple Past
D) Future Simple
E) Present Perfect

AULA 03 – ARTICLES AND NOUNS 64


TEACHER ANDREA BELO

Comentários: A alternativa A está incorreta. Não há nenhuma estrutura no “Present Continuous”


na tirinha. A estrutura do present continuous exige que o verbo tenha sua terminação em -ing.
A alternativa B está correta. A maioria das falas do diálogo estão no “Present Simple”. Há falas no
present simple em 3 dos 4 quadrinhos. Portanto, podemos afirmar que o present simple é o
tempo verbal principal da tirinha.
A alternativa C está incorreta. Há apenas um verbo na tirinha que está conjugado no simple past.
Por isso, não se pode dizer que o simple past é o principal tempo verbal da tirinha.
A alternativa D está incorreta. Não se pode observar o auxiliar “will” indicativo de futuro, e apenas
uma expressão “going to” que também serve como indicativo de futuro. Isso elimina a alternativa.
A alternativa E está incorreta. Não se pode observar a presença dos auxliares “have” ou “has”
acrescidos do verbo conjugado no particípio passado, que é a estrutura do “present perfect”. Isso
elimina a alternativa.
GABARITO: B

Questão 04 (EAM/INÉDITA) – Use the verbs in the parentheses to complete the following
statements.
I – He __________ (study) at the Brazilian Navy Academy.
II – At some point, I finally __________ (realize) that I could pass that test.
III – She loves __________ (run) in the morning.
Now mark the option which completes them respectively.
A) study / realized / running
B) studies / realized / runs
C) studied / realize / run
D) studies / realized / running
E) study / realize / runs
Comentários: A primeira lacuna deve ser preenchida por “studies”. A frase está no presente e o
sujeito é “he”. Portanto, o verbo precisa ser conjugado com “-IES” no final.
A segunda lacuna deve ser preenchida por “realized”. A frase está no passado (evidenciado pelo
uso de could logo em seguida). Portanto, o verbo deve ser acrescido de “-D” no final para
demarcar o tempo passado.
A terceira lacuna deve ser preenchida por “running”. O verbo “loves” exige que o verbo
subsequente venha no gerúndio.
GABARITO: D

AULA 03 – ARTICLES AND NOUNS 65


TEACHER ANDREA BELO

Questão 05 (EAM/INÉDITA) – Look at the picture below.

What are they doing in the picture?


A) They are trying to pull the girl with the ball.
B) They are playing soccer.
C) They are making the girl fall down.
D) They are warning the girl with the ball of some danger.
E) They don’t like to play with the girl with the ball.
Comentários: A alternaiva A está incorreta. Ao analisar a imagem, não é possível concluir que elas
estão tentando puxar a menina com a bola.
A alternativa B está correta. A imagem nos conduz a acreditar que elas estão, de fato, brincando
de jogar futebol.
A alternativa C está incorreta. Elas não estão tentando fazer a menina com a bola cair, elas estão
brincando de jogar futebol.
A alternativa D está incorreta. A imagem não nos permite concluir que elas estão tentando alertar
a menina com a bola sobre algum perigo, mas sim que estão jogando futebol.
A alternativa E está incorreta. Não se pode concluir que elas não gostam de jogar com a menina,
já que as três parecem estar jogando e brincando. Não há elementos na imagem que nos permita
tirar essa conclusão.
GABARITO: B

QUESTÕES EEAR
Read the text and answer questions 01 e 02.

Adapted from http://www.englishhact.com.br/2016/10/atividades-com-tirinhas-do-garfield-em.html

AULA 03 – ARTICLES AND NOUNS 66


TEACHER ANDREA BELO

Questão 01 (EEAR – CFS/2020) – Which expression is usually used to tell stories in English?
a) There was a house with a man
b) Then the cat ate my breakfast
c) That sounds familiar
d) Once upon a time
Comentários: O enunciado pergunta como se começam as histórias britânicas. Não há como
iniciar histórias com as informações contidas nas alternativas A, B e C, pois faz parte de uma
história específica do Garfield.
A alternativa correta é a letra D, Once upon a time, que significa “Era uma vez”, assim como se
iniciam várias outras histórias.
GABARITO: D

Questão 02 (EEAR – CFS/2020) – What is the main verb tense used in the comic strip?
a) Present Perfect
b) Simple Present
c) Past Perfect
d) Simple Past
Comentários: Nosso gabarito é a alternativa D, pois os verbos predominam no Passado Simples,
ou seja, Simple Past. Os verbos encontrados são: was, passado do verbo to be e ate, passado do
verbo eat.
GABARITO: D

Read the text and answer question 03.


“The U.S. Constitution doesn't guarantee happiness, only the pursuit of it. Your have to catch up
with it yourself.”
https://www.brainyquote.com/quotes/benjamin_franklin_141100

Questão 03 (EEAR – CFS/2020) – The word in bold, in the text, is na object pronoun. Which
word does it make reference to?
a) Constitution
b) Happiness
c) The U. S.
d) Pursuit
Comentários: O pronome “it”, refere-se à felicidade, Happiness, pois, quando se usa um pronome,
temos que observar a última palavra que foi falada. A frase se remete a ele, a palavra dita foi
“happiness” e não as demais.
GABARITO: B

AULA 03 – ARTICLES AND NOUNS 67


TEACHER ANDREA BELO

Read the text and answer question 04.


Being famous
Sandra Rosa is very beautiful, young, and successful. She's a famous actress. She's also very rich.
Her house near the beach is big and beautiful, and her car is very expensive. Her fans love her.
But is she happy?
Sandra says, "yeah, I'm young, rich, beautiful, and famous. People think rich people are happy.
That's not always true!"
Sandra's brother, Mike, is her manager. He says, "Sandra is only 18. She enjoys acting and
entertaining people. But she's not happy. She doesn't like being famous."
"It's true," Sandra says. "I'm never alone. Reporters are everywhere. Wherever I go, they're there.
They're outside my house all the time! That's so annoying!”
Adapted from: https://www.inglesnapontadalingua.com.br/2008/10/texto-de-ingls-alunos-de-nvel-bsico.html

Questão 04 (EEAR – CFS/2020) – According to the text, Sandra is


a) very happy because she is never by herself.
b) very happy because she has a lot of money.
c) unhappy with her carreer because it is annoying.
d) unhappy because wherever she goes, the reporters are there.
Comentários: O enunciado pede o que podemos dizer de acordo com o texto. Já podemos
eliminar, de cara, as alternativas A e B, que dizem que Sandra é feliz, pois ela diz que não é feliz,
apesar de ter um bom emprego e dinheiro.
Na alternativa C, diz que ela não está satisfeita com a carreira, mas isso não é verdade.
Nosso gabarito é a alternativa D, pois diz que há muitos repórteres atrás dela, é famosa e está
sempre cercada de pessoas para entrevistá-la.
GABARITO: D

Read the text and answer question 05.


Homeless crack addict revitalizes small square in downtown São Paulo
A homeless man has chosen to occupy his free time revitalizing a small square on the corner of
avenues São João and Duque de Caxias, in downtown São Paulo. He planted pau-brasil, palm,
banana and avocado trees. He also planted boldo, sweet potatoes, beans, peppers and
ornamental plants, such as snake plants. Residents noticed the square’s gradual changes and
congratulated the author for the modifications.
Fonte: Folha de São Paulo Internacional – 21/03/2017

AULA 03 – ARTICLES AND NOUNS 68


TEACHER ANDREA BELO

Questão 05 (EEAR/2017) – The words “small”, “sweet” e “ornamental”, underlined in the text,
are __________.
a) nouns
b) adverbs
c) pronouns
d) adjectives
Comentários: A afirmativa A está incorreta. “Nouns” são os substantivos, e substantivos são
palavras que designam coisas, pessoas, grupos, lugares.
A afirmativa B está incorreta. “Adverbs” são os advérbios, e advérbios são palavras que designam
ideias de modo, tempo, lugar, dúvida, negação, intensidade, afirmação.
A afirmativa C está incorreta. “Pronouns” são os pronomes, e pronomes são palavras que
substituem, referem-se ou acompanham um nome (substantivo).
A alternativa D está correta. As palavras sublinhadas são adjetivos porque acompanham o
substantivo, caracterizando-o. Dessa forma “small” (pequeno/a) caracteriza “square” (praça),
“sweet” (doce) caracteriza “potatoes” (batatas) e “ornamental” (ornamental/ornamentais)
caracteriza “plants” (plantas).
GABARITO: D

QUESTÕES EFOMM
Based on the text below, answer questions 01 and 02.
Why some people like wearing masks
Some people welcome face coverings for reasons ranging from the convenient and expedient
to the more complex and psychological. But is this a helpful coping mechanism?
Sheltering in place hasn’t been too hard for Jay Lee; watching a film at home and ordering a
takeaway has always been his idea of a good night. Lee, a 32-year-old small business owner in
Leicester, identifies as an introvert. And although 2020 had its hardships – in the spring, he was
made redundant from his job at a large bank – one perk for him has been the widespread adoption
of face masks.
Lee has always dreaded run-ins with old friends and acquaintances around town, finding these
spontaneous interactions “extremely awkward”. He used to time his shopping trips to minimise
the possibility of bumping into someone he knew, waiting until almost closing time before
heading out. “Since I've been wearing the mask, my awkward interactions with friends and family
have significantly reduced,” he says. Now, he goes to the shops whenever he wants, without
worrying about whom he might see. He hopes that, even after the pandemic ends, it will still be
socially acceptable to wear a mask.
Wearing a mask is, for most of us, an annoying but worthwhile sacrifice: it’s one of the most
effective ways to slow the spread of Covid-19. Still, most of us look forward to the day when we
can bare our faces in public again. Face-coverings fog our glasses and clog our pores; they make
it harder to smile at strangers and recognise friends.

AULA 03 – ARTICLES AND NOUNS 69


TEACHER ANDREA BELO

Yet some are secretly relishing the new mask-wearing mandates, for reasons ranging from the
convenient and expedient to the more complex and psychological. Some welcome the way face
coverings reduce or change interactions that might otherwise spark social anxiety.
‘Anonymity carries power’
At the lighter end of the scale, some people have found that masking offers a welcome relief from
the pressures to uphold strict standards of grooming and appearance. They have ditched their old
makeup and shaving routines and are saving money, time and stress. Others have discovered that
hiding their mouths affords them unexpected freedoms. Some restaurant servers and retail
workers say they no longer feel obliged to fake-smile at customers, potentially lifting the burden
of emotional labour.
(Adapted from https://www.bbc.com/worklife/article/20210115-why-some-people-like-wearing-masks)

Questão 01 (EFOMM/INÉDITA) – It is possible to infer from the text that


(A) The safety measure related to the masks is well accepted depending on the personality of
the user
(B) Face coverings is an inconvenience for everyone
(C) Introverted people ended up not adapting to the context of a pandemic
(D) Wearing a mask is a pointless sacrifice at the present time
(E) Wearing masks does not interfere with the aesthetic pressure exerted by society
Comentários: A alternativa A está correta. De acordo com o texto, é correto inferir que a medida
de segurança relacionada às máscaras é bem aceita dependendo da personalidade do usuário,
assim como esta opção indica. Isso pode ser confirmado com o trecho “...identifies as an introvert.
And although 2020 had its hardships – in the spring, he was made redundant from his job at a
large bank – one perk for him has been the widespread adoption of face masks”.
A alternativa B está incorreta. De acordo com o texto, não é correto inferir que coberturas faciais
são um incômodo para todos, mas sim, que, para alguns, foi um alívio. Isso pode ser confirmado
com o trecho “...one perk for him has been the widespread adoption of face masks”.
A alternativa C está incorreta. De acordo com o texto, não é correto inferir que pessoas
introvertidas acabaram não se adaptando ao contexto de uma pandemia, mas sim, que se
adaptaram por não precisarem se expor ao contato com outras pessoas. Isso pode ser confirmado
com o trecho “Since I've been wearing the mask, my awkward interactions with friends and family
have significantly reduced”.
A alternativa D está incorreta. De acordo com o texto, não é correto inferir que usar uma máscara
é um sacrifício inútil no momento, mas sim, que é necessário e útil. Isso pode ser confirmado com
o trecho “Wearing a mask is, for most of us, an annoying but worthwhile sacrifice: it’s one of the
most effective ways to slow the spread of Covid-19”.
A alternativa E está incorreta. De acordo com o texto, não é correto inferir que o uso de máscaras
não interfere na pressão estética exercida pela sociedade, mas sim, que interfere. Isso pode ser
confirmado com o trecho “At the lighter end of the scale, some people have found that masking

AULA 03 – ARTICLES AND NOUNS 70


TEACHER ANDREA BELO

offers a welcome relief from the pressures to uphold strict standards of grooming and
appearance”.
GABARITO: A

Questão 02 (EFOMM/INÉDITA) – In the excerpt “And although 2020 had its hardships – in the
spring, he was made redundant from his job at a large bank – one perk for him has been the
widespread adoption of face masks.”, the word in bold means
(A) Advice
(B) Hardship
(C) Happiness
(D) Loss
(E) Advantage
Comentários: A alternativa A está incorreta. A palavra “perk” significa benefício e não pode ser
comparada com a palavra “advice”, que significa conselho.
A alternativa B está incorreta. A palavra “perk” significa benefício e não pode ser comparada com
a palavra “hardship”, que significa sofrimento.
A alternativa C está incorreta. A palavra “perk” significa benefício e não pode ser comparada com
a palavra “happiness”, que significa felicidade.
A alternativa D está incorreta. A palavra “perk” significa benefício e não pode ser comparada com
a palavra “loss”, que significa perda.
A alternativa E está correta. A palavra “perk” significa benefício e pode ser comparada com a
palavra “advantage”, que significa vantagem.
GABARITO: E

Based on the text below, answer questions 03, 04 and 05.


The World's Biggest Problems Are Interconnected. Here's How We Can Solve Them This
Decade
Two decades ago, people around the world rang in the new millennium with a growing sense of
optimism. The threat posed by the Cold War was fading slowly in the rearview mirror. Leading
thinkers like Francis Fukuyama touted the benefits of globalization, saying it would bring
democracy and prosperity to the developing world. The nascent Internet economy promised to
bring us closer together.
The following 20 years took some of the air out of the assumption of steady progress, but when
future historians assess the 21st century, the year 2020 is likely to serve as the point at which the
optimism bubble burst. The COVID-19 pandemic has exposed a complex web of interlocking
problems that have morphed into full-blown crises. The coronavirus laid bare the dangers of
endemic poverty not only in the developing world but also in rich countries like the U.S., where
millions lack health care and are one paycheck away from living on the street. Around the world,

AULA 03 – ARTICLES AND NOUNS 71


TEACHER ANDREA BELO

racial and ethnic minorities have demanded justice after centuries of structural discrimination.
Woven through it all, the earth’s climate is increasingly unstable, posing an existential threat to
human society as we know it. In the next decade, societies will be forced to either confront this
snarl of challenges, or be overwhelmed by them. Our response will define the future for decades
to come.
The recognition that these challenges are fundamentally linked isn’t new. Activists and academics
have for many years pointed to the cascading effects of various social ills. Whether it’s the way
racism contributes to poor health outcomes or gender discrimination harms economic growth,
the examples are seemingly endless. But this understanding has made its way into the
conversation about solutions too.
Notably, for the past five years, the U.N. has touted 17 interrelated sustainable development
goals, objectives for building a more viable world, and called for a push to achieve them by 2030.
The goals, which cover environmental, social and economic progress, are nonbinding but have
become key benchmarks for commitments at a national and corporate level. Countries from China
to the Maldives, as well as companies like Amazon, Microsoft and PwC, have committed to rolling
out policies over the next decade that will set them on a path to eliminate their carbon footprints.
The understanding that these problems require holistic solutions has only grown amid the
pandemic and its fallout. President Joe Biden has referred to four urgent crises—the pandemic,
the economic crisis, racial injustice and climate change—and promised a push to tackle them all
together. The European Union’s program to propel the bloc out of the COVID-19 crisis targets
climate change, while incorporating equity concerns. As stock markets soared last year,
institutions with trillions of dollars in assets demanded that their investments deliver not only a
good return for their wallets but also a good return for society.
(Adapted from https://time.com/5931603/how-to-solve-worlds-biggest-problems/)

Questão 03 (EFOMM/INÉDITA) – Read the statements about the text and decide whether they
are TRUE (T) or FALSE (F). Mark the correct option
I. The turn of the millennium brought high expectations for the development of the world
II. Covid-19 plays an important role in explaining the interconnection between global problems
III. Despite having played a watershed role, the Covid-19 pandemic mainly affected developing
countries
IV. The intensely discriminatory past has no reflection today
V. Our current actions will affect the future of world society
(A) I – (T) / II – (F) / III – (F) / IV – (F) / V – (T)
(B) I – (T) / II – (T) / III – (F) / IV – (F) / V – (F)
(C) I – (T) / II – (T) / III – (F) / IV – (F) / V – (T)
(D) I – (F) / II – (T) / III – (F) / IV – (F) / V – (T)
(E) I – (F) / II – (T) / III – (T) / IV – (F) / V – (T)

AULA 03 – ARTICLES AND NOUNS 72


TEACHER ANDREA BELO

Comentários: A afirmativa I é verdadeira (T – TRUE). É correto afirmar que a virada do milênio


trouxe grandes expectativas para o desenvolvimento do mundo, assim como esta opção indica.
Isso pode ser confirmado com o trecho “Two decades ago, people around the world rang in the
new millennium with a growing sense of optimism”.
A afirmativa II é verdadeira (T – TRUE). É correto afirmar que a Covid-19 desempenha um papel
importante na explicação da interconexão entre problemas globais, assim como esta opção
indica. Isso pode ser confirmado com o trecho “The COVID-19 pandemic has exposed a complex
web of interlocking problems that have morphed into full-blown crises”.
A afirmativa III é falsa (F – FALSE). Não é correto afirmar que apesar de ter desempenhado um
papel decisivo, a pandemia Covid-19 afetou principalmente os países em desenvolvimento, mas
sim, que afetou a todos os países do mundo. Isso pode ser confirmado com o trecho “The
coronavirus laid bare the dangers of endemic poverty not only in the developing world but also in
rich countries like the U.S., where millions lack health care and are one paycheck away from living
on the street”.
A afirmativa IV é falsa (F – FALSE). Não é correto afirmar que o passado intensamente
discriminatório não tem reflexo hoje, mas sim, que tem. Isso pode ser confirmado com o trecho
“Around the world, racial and ethnic minorities have demanded justice after centuries of structural
discrimination”.
A afirmativa V é verdadeira (T – TRUE). É correto afirmar que nossas ações atuais afetarão o futuro
da sociedade mundial, assim como esta opção indica. Isso pode ser confirmado com o trecho “As
stock markets soared last year, institutions with trillions of dollars in assets demanded that their
investments deliver not only a good return for their wallets but also a good return for society”.
GABARITO: C

Questão 04 (EFOMM/INÉDITA) – In the excerpt “Woven through it all, the earth’s climate is
increasingly unstable, posing an existential threat to human society as we know it”, the word in
bold means
(A) Unwoven
(B) Separate
(C) In contrast
(D) Sewn
(E) Although
Comentários: A alternativa A está incorreta. A palavra “woven” significa tecido/constituído e não
pode ser comparada com a palavra “unwoven”, que significa desfeito.
A alternativa B está incorreta. A palavra “woven” significa tecido/constituído e não pode ser
comparada com a palavra “separate”, que significa separado.
A alternativa C está incorreta. A palavra “woven” significa tecido/constituído e não pode ser
comparada com a palavra “in contrast”, que significa em contrapartida.

AULA 03 – ARTICLES AND NOUNS 73


TEACHER ANDREA BELO

A alternativa D está correta. A palavra “woven” significa tecido/constituído e pode ser comparada
com a palavra “sewn”, que significa costurado/constituído.
A alternativa E está incorreta. A palavra “woven” significa tecido/constituído e não pode ser
comparada com a palavra “although”, que significa apesar.
GABARITO: D

Questão 05 (EFOMM/INÉDITA) – According to the text, the current problems in the world
(A) Are actually a conspiracy theory that opposes the high expectations of the turn of the
millennium
(B) Must be resolved for the whole society, after their total exposure during the pandemic
(C) Did not worsen during the Covid-19 pandemic
(D) Have a relatively new interconnection
(E) Are not so serious, considering the high development of society
Comentários: A alternativa A está incorreta. De acordo com o texto, não é correto afirmar que os
problemas atuais do mundo são, na verdade, uma teoria da conspiração que se opõe às altas
expectativas da virada do milênio, mas sim, que são reais e não correspondem às boas
expectativas da virada do milênio. Isso pode ser confirmado com o trecho “Two decades ago,
people around the world rang in the new millennium with a growing sense of optimism …The
following 20 years took some of the air out of the assumption of steady progress, but when future
historians assess the 21st century, the year 2020 is likely to serve as the point at which the
optimism bubble burst”.
A alternativa B está correta. De acordo com o texto, é correto afirmar que os problemas atuais do
mundo devem ser resolvido para toda a sociedade, após sua exposição total durante a pandemia,
assim como esta opção indica. Isso pode ser confirmado com o trecho “The COVID-19 pandemic
has exposed a complex web of interlocking problems that have morphed into full-blown crises …
As stock markets soared last year, institutions with trillions of dollars in assets demanded that
their investments deliver not only a good return for their wallets but also a good return for society”.
A alternativa C está incorreta. De acordo com o texto, não é correto afirmar que os problemas
atuais do mundo não pioraram durante a pandemia Covid-19, mas sim, que pioraram e se
tornaram mais visíveis. Isso pode ser confirmado com o trecho “The COVID-19 pandemic has
exposed a complex web of interlocking problems that have morphed into full-blown crises”.
A alternativa D está incorreta. De acordo com o texto, não é correto afirmar que os problemas
atuais do mundo tem uma interconexão relativamente nova, mas sim, que não é nova. Isso pode
ser confirmado com o trecho “The recognition that these challenges are fundamentally linked isn’t
new. Activists and academics have for many years pointed to the cascading effects of various social
ills”.
A alternativa E está incorreta. De acordo com o texto, não é correto afirmar que os problemas
atuais do mundo não são tão graves, considerando o alto desenvolvimento da sociedade, mas
sim, que são graves. Isso pode ser confirmado com o trecho “Around the world, racial and ethnic
minorities have demanded justice after centuries of structural discrimination”.
GABARITO: B

AULA 03 – ARTICLES AND NOUNS 74


TEACHER ANDREA BELO

Questão 06 (EFOMM/INÉDITA) – Choose the correct option to complete the paragraph below.
I remember the last time I had _______ choose between optimism and fear, between hope and
the urge to run away. It was right after 9/11. I had babies—one so tiny she was still curled up like
a fern, _______ other toddling around gumming everything she could get her little starfish hands
on. And as their brand-new lungs took _______ the smoke that blew over to Brooklyn _______
the burning towers, I wanted to pack them up and flee to some safer place.
(Adapted from https://time.com/5930399/the-capitol-riots-and-our-fragile-optimism/)
(A) To / the / in / from
(B) The / the / on / into
(C) To / the / on / from
(D) To / the / in / into
(E) The / to / on / from
Comentários: A primeira lacuna deve ser preenchida com “to”, pois a frase nos infere que ele
deveria escolher entre o medo e o otimismo. Isso pode ser confirmado com o trecho “Lembro-me
da última vez que tive de escolher entre otimismo e medo”.
A segunda lacuna deve ser preenchida com “the”, pois a frase se refere especificamente às
segunda filha, ou seja, artigo definido. Isso pode ser confirmado com o trecho “a outra
engatinhando e engolindo tudo o que podia em suas mãos de estrela do mar”.
A terceira lacuna deve ser preenchida com “in”, pois a frase se refere aos pulmões inspirando a
fumaça. Isso pode ser confirmado com o trecho “E enquanto seus pulmões novos absorviam a
fumaça...”.
A quarta lacuna deve ser preenchida com “from”, pois a frase se refere À fumaça que chegava ao
Brooklyn das torres em chamas. Isso pode ser confirmado com o trecho “...chamas para o
Brooklyn, eu queria embalá-los e fugir para um lugar mais seguro”.
GABARITO: A

Questão 07 (EFOMM/INÉDITA) – Which is the correct way to complete the paragraph below?
For the European Union, the rapid rollout of Covid-19 vaccines __________ critical to save lives
and prevent health services from __________ stretched beyond their limits, not to mention
minimising the massive economic damage from lockdowns. Unfortunately, however, though
vaccinations are under way, a rapid near-term increase in infections is likely as the British variant
of the virus __________ across the continent.
(Adapted from https://www.concursosmilitares.com.br/provas-anteriores/marinha/efomm/2020-efomm-oficial-da-marinha-mercante-primeiro-dia.pdf)

(A) Is / being / spreads


(B) Was / be / spreads
(C) Is / be / spreading
(D) Was / being / spreading
(E) Is / being / spreading

AULA 03 – ARTICLES AND NOUNS 75


TEACHER ANDREA BELO

Comentários: A primeira lacuna deve ser preenchida com o verbo to be (ser/estar) no simple
present, ou seja, “is” (é). Isso pode ser confirmado com o trecho “Para a União Europeia, a rápida
distribuição das vacinas Covid-19 é crítica para salvar vidas...”.
A segunda lacuna deve ser preenchida com o verbo to be (ser/estar) no present continuous, ou
seja, “being” (de ser). Isso pode ser confirmado com o trecho “...evitar que os serviços de saúde
sejam esticados além de seus limites...”.
A terceira lacuna deve ser preenchida com o verbo to spread (propagar) no simple present, ou
seja, “spreads” (se propaga). Isso pode ser confirmado com o trecho “...à medida que a variante
britânica do vírus se espalha pelo continente”.
GABARITO: A

Questão 08 (EFOMM/INÉDITA) – Which of the following sentences expresses probability?


(A) You are not as understanding as you should be
(B) One shouldn’t accelerate in case of a red light
(C) Everyone here must have a critical sense
(D) He studied a lot, so he should be able to pass
(E) You must take good care of your body, even if you don't want to
Comentários: A alternativa A está incorreta. A frase “Você não é tão compreensivo quanto deveria
ser” não expressa probabilidade, mas sim, uma afirmação.
A alternativa B está incorreta. A frase “Não se deve acelerar em caso de sinal vermelho” não
expressa probabilidade, mas sim, imperatividade.
A alternativa C está incorreta. A frase “Todos aqui devem ter senso crítico” não expressa
probabilidade, mas sim, uma afirmação.
A alternativa D está correta. A frase “Ele estudou muito, então deveria conseguir passar” expressa
probabilidade.
A alternativa E está incorreta. A frase “Você deve cuidar bem do seu corpo, mesmo que não
queira” não expressa probabilidade, mas sim, uma necessidade.
GABARITO: D

Questão 09 (EFOMM/INÉDITA) – Choose the correct option to complete the paragraph below.
The U.S. has tragically surpassed 400,000 COVID-19 deaths, and case numbers and
hospitalizations are likewise spiking to record levels around the world. __________ vaccines now
rolling out, there is reason to hope that there is an end in sight. __________, by most estimates,
widespread vaccinations will not be in place until the middle of the year at the earliest.
__________, we have some ways to go yet with social distancing, mask wearing and other
pandemic mitigation behaviors.
(Adapted from https://www.scientificamerican.com/article/how-we-can-deal-with-pandemic-fatigue/)

AULA 03 – ARTICLES AND NOUNS 76


TEACHER ANDREA BELO

(A) And / however / and


(B) With / however / so
(C) So / however / even if
(D) With / in agreement / and
(E) With / in contrast / so
Comentários: A primeira lacuna deve ser preenchida com “with” (com), pois a frase se refere ao
momento atual das vacinas circulando. Isso pode ser confirmado com o trecho “Com as vacinas
sendo lançadas, há motivos para esperar que haja um fim à vista”.
A segunda lacuna deve ser preenchida com “however” (contudo), pois a frase, de certa forma, se
opõe à frase anterior. Isso pode ser confirmado com o trecho “Com as vacinas sendo lançadas, há
motivos para esperar que haja um fim à vista. No entanto, pela maioria das estimativas, as
vacinações generalizadas não estarão em vigor até meados do ano, no mínimo”. confirmado com
o trecho “Portanto, ainda temos alguns caminhos a percorrer com o distanciamento social, uso
de máscaras e outros comportamentos de mitigação de pandemia”.
GABARITO: B

Questão 10 (EFOMM/INÉDITA) – Which option is incorrect?


(A) A pandemic is affecting many people
(B) Tell the girls that they are not responsible
(C) A couple of my friends plans to travel this year
(D) I want to finish school
(E) Where are those pants I lent you?
Comentários: A alternativa A está incorreta. A frase “uma pandemia está afetando muitas
pessoas” está totalmente correta.
A alternativa B está incorreta. A frase “diga às meninas que elas não são responsáveis” está
totalmente correta.
A alternativa C está correta. A frase “A couple of my friends plans to travel this year” deveria ser
escrita “A couple of friends of mine are planning to trave this year” (um casal de amigos meus
estão planejando para viajar este ano).
A alternativa D está incorreta. A frase “eu quero terminar a escola” está totalmente correta.
A alternativa E está incorreta. A frase “onde estão aquelas calças que te emprestei?” está
totalmente correta.
GABARITO: C

AULA 03 – ARTICLES AND NOUNS 77


TEACHER ANDREA BELO

QUESTÕES EPCAR
Directions: Read the text below and answer questions 01 to 15 according to it.
2020: The year in seven fake news stories
How should we deal with fake news? Many western governments are moving towards stricter
regulation of tech giants. But others believe that only a more educated public can stop the blight.
Have you heard that the Covid-19 vaccine will rewrite your DNA? If not, don’t worry. It is fake
news. What is true, however, is that debate about misinformation online reached a fever pitch
this year.
From politicians calling for fake news to be banned, to others suggesting they should “weaponise”
it, the public’s ability to tell fact from fiction is increasingly contested. Here are seven stories that
show how fake news shaped this year.
First casualty. In January, Iran launched missiles at Iraqi airbases hosting US troops in retaliation
for the American assassination of general Qasem Soleimani. Many feared war. Iranian state TV
announced that 80 “American terrorists” had been killed. The USA said it experienced no
casualties. Some argued that “fake news” allowed both sides to save face and avoid conflict.
Covid-5G. By far the biggest subject for fake news this year was Covid-19. The stories were
numerous and varied, but one popular one was that Covid-19 was caused by 5G radiation. In
Britain, 5G masts were attacked, and blame was often pointed at Bill Gates in what became an
increasingly Byzantine conspiracy theory.
Dolphins of Venice. When Coronavirus lockdowns began, social media was abuzz with animal
news. Nature was healing. Venice, no longer clogged with tourists, now had clean canals down
which Dolphins cruised. Sadly, the heartening video was actually from a different region of Italy.
Viral veganism. In India, a false story that that no-one who eschewed all animal products had
contracted Covid-19 went viral. While this might not seem too harmful, the resulting drop in sales
of meat and eggs hit farmers and butchers hard. Some saw the story as part of the promotion of
Hindutva in India.
Constantine cancelled. The death of George Floyd at the hands of US police, kicked off a summer
of protest against racism. Iconoclasts tore down statues of historical figures they saw as racist.
Rumours abounded about who was next to fall. Perhaps the oddest story was that York Minster
planned to remove a statue of the Roman emperor Constantine because of his support for slavery.
The Church of England was forced to state that they had no such plans.
Rigged election. A daring raid on a CIA server in Germany revealed the plot to rig the US election
against Donald Trump… or so the fake story went. Its popularity is evidence of a stark epistemic
divide among Americans. It is just one of many casting President Trump’s loss this November as
the result of fraud. The president has yet to concede officially, but some of his statements have
been censored by social media companies, already treating him as yesterday’s fake news.
Trained Triceratops. A video of a dinosaur being unloaded from a truck went viral this month in
Indonesia. Many shared it in the belief that it was real. It turned out to be a robot filmed to
promote a theme park. While some fake news stories are all about lies and Russian spies, others
are simply about people seeing what they want to see.
How should we deal with fake news?

AULA 03 – ARTICLES AND NOUNS 78


TEACHER ANDREA BELO

Reality check. Regulate! say some. Social media companies should be responsible for information
shared on their platforms. If they do not take action to stop lies being spread, then we __________
ever more anger and division. Algorithms showing people what they want to read will combine
with fake news to reinforce false beliefs. The lack of an objective standard of truth online could
become dangerous.
Educate! say others. There have always been lies, mistakes and misrepresentations in the media.
Allowing either the government or large companies to decide what is true or false is a recipe for
disaster. What is to stop them from censoring their own critics? What is needed is the ability to
discern reliable sources for oneself. And the best way to do that is to teach people about fake
news.
(Adapted from https://theday.co.uk/stories/2020-the-year-in-seven-fake-news-stories/Acesso em: 14 de jan 2021)

Questão 01 (EPCAR/INÉDITA) – We can deduce from the first paragraph that


(A) strict regulation is the correct way of dealing with fake news.
(B) Covid-19 vaccine will likely rewrite our DNA.
(C) educating people is the only accurate way to stop fake news dissemination.
(D) there is some disagreement about how to handle fake news.
Comentários: Podemos deduzir do primeiro parágrafo que: A alternativa A está incorreta.
“Regulamentação rígida é a maneira correta de lidar com notícias falsas.” O primeiro parágrafo
afirma que é a forma que alguns governos ocidentais estão lidando com as fake news, mas não
afirma ser necessariamente a forma correta. “How should we deal with fake news? Many western
governments are moving towards stricter regulation of tech giants.” = Como devemos lidar com
notícias falsas? Muitos governos ocidentais estão adotando uma regulamentação mais rígida dos
gigantes da tecnologia.
A alternativa B está incorreta. “A vacina Covid-19 provavelmente reescreverá nosso DNA.” Likely
significa provavelmente, possivelmente. O texto desmente essa informação. “Have you heard that
the Covid-19 vaccine will rewrite your DNA? If not, don’t worry. It is fake news.” = Você já ouviu
falar que a vacina Covid-19 irá reescrever seu DNA? Se não, não se preocupe. São notícias falsas.
A alternativa C está incorreta. Muita atenção, pois o texto não afirma que “educar as pessoas é a
única maneira precisa de impedir a disseminação de notícias falsas.” O primeiro parágrafo apenas
diz que essa é a forma que algumas pessoas acreditam ser a ideal, enquanto outras pensam, por
exemplo, que regulamentação rígida é a única maneira. “How should we deal with fake news?
Many western governments are moving towards stricter regulation of tech giants. But others
believe that only a more educated public can stop the blight.” = Como devemos lidar com notícias
falsas? Muitos governos ocidentais estão adotando uma regulamentação mais rígida dos gigantes
da tecnologia. Mas outros acreditam que apenas um público mais educado pode deter a praga.
A alternativa D está correta. “Há alguma discordância sobre como lidar com notícias falsas.”,
conforme vimos no comentário acima.
GABARITO: D

AULA 03 – ARTICLES AND NOUNS 79


TEACHER ANDREA BELO

Questão 02 (EPCAR/INÉDITA) – “the public’s ability to tell fact from fiction is increasingly
contested.” (paragraph 2). Mark the correct plural form of the highlighted word.
(A) abilitys
(B) abilities
(C) abilityes
(D) abilitis
Comentários: Quando o substantivo termina em -y, tiramos o -y e + ies. Porém, atenção: essa
regra só se se aplica se antes do -y vier uma CONSOANTE, como é o caso na questão. Observe que
é como se você tivesse que “preparar o território” para a chegada do -es. Você, primeiro, elimina
o -y, trocando-o por -i. Com essa alteração feita, acrescenta a terminação -es, indicativa de plural.
Atenção: se antes do -y vier uma vogal, não temos aplicação de nenhuma regra especial. Você
seguirá a regra geral de formação de plural em Inglês, acrescentando o sufixo -s ao substantivo.
Exemplo: day – days.
GABARITO: B

Questão 03 (EPCAR/INÉDITA) – “debate about misinformation online reached a fever pitch this
year.” (paragraph 1). Mark the correct option to make the sentence above negative.
(A) debate about misinformation online hasn’t reached a fever pitch this year.
(B) debate about misinformation online didn’t reach a fever pitch this year.
(C) debate about misinformation online did not reached a fever pitch this year.
(D) debate about misinformation online hasn’t reach a fever pitch this year.
Comentários: A frase está no Simple Past (observe o verbo, reached): debate about
misinformation online reached a fever pitch this year = o debate sobre a desinformação online
atingiu um pico febril este ano. Para formarmos a negativa do Simple Past, devemos usar didn’t
antes do verbo e voltar o verbo à sua forma base, sem -ed: debate about misinformation online
didn’t reach a fever pitch this year.
A alternativa A está incorreta. “hasn’t reached” é a negativa de has reached (’s reached), no
Present Perfect.
A alternativa B está correta, conforme explicado acima.
A alternativa C está incorreta, pois não voltou o verbo à sua forma base, sem -ed.
A alternativa D está incorreta, pois trouxe a forma negativa de has reached (’s reached), no
Present Perfect, e com erro, pois, nesse caso o particípio segue com -ed (hasn’t reached).
GABARITO: B

Questão 04 (EPCAR/INÉDITA) – According to the third paragraph,


(A) 80 Americans were killed due to war.
(B) the stated fact that Iran launched missiles at Iraqi airbases was false.
(C) the described fake news did not have only downsides for some people.
(D) general Qasem Soleimani was murdered by Iran missiles.

AULA 03 – ARTICLES AND NOUNS 80


TEACHER ANDREA BELO

Comentários: De acordo com o terceiro parágrafo, a alternativa A está incorreta. “80 americanos
foram mortos devido à guerra.” = o terceiro parágrafo afirma que isso foi fake news. “Iranian state
TV announced that 80 ‘American terrorists’ had been killed. The USA said it experienced no
casualties. Some argued that ‘fake news’ allowed both sides to save face and avoid conflict.” = A
TV estatal iraniana anunciou que 80 “terroristas americanos” foram mortos. Os EUA disseram que
não houve vítimas. Alguns argumentaram que as “notícias falsas” permitiram que ambos os lados
evitassem a perda do respeito público e evitassem conflitos.
A alternativa B está incorreta. De acordo com o terceiro parágrafo, o fato declarado de que o Irã
lançou mísseis contra bases aéreas iraquianas não foi falso. Isso ocorreu. O que foi falso foi a
notícia de que 80 americanos foram mortos em decorrência disso. “In January, Iran launched
missiles at Iraqi airbases hosting US troops in retaliation for the American assassination of general
Qasem Soleimani.”
A alternativa C está correta. De acordo com o terceiro parágrafo, as notícias falsas descritas não
tiveram apenas desvantagens para algumas pessoas. “Some argued that ‘fake news’ allowed both
sides to save face and avoid conflict.” = Alguns argumentaram que as “notícias falsas” permitiram
que ambos os lados evitassem a perda do respeito público e evitassem conflitos.
A alternativa D está incorreta. O general Qasem Soleimani não foi assassinado por mísseis
iranianos. Os mísseis foram em retaliação a esse assassinato. “In January, Iran launched missiles
at Iraqi airbases hosting US troops in retaliation for the American assassination of general Qasem
Soleimani.” = Em janeiro, o Irã lançou mísseis contra bases aéreas iraquianas que abrigavam
tropas dos EUA em retaliação ao assassinato americano do general Qasem Soleimani.
GABARITO: C

Questão 05 (EPCAR/INÉDITA) – Mark the option that shows the appropriate question tag for
the sentence.
The stories were numerous and varied, __________? (paragraph 4)
(A) wasn’t it
(B) were they
(C) weren’t they
(D) was it
Comentários: As question tags (ou tag questions) são usadas no final das frases para questionar
ou confirmar uma informação. É preciso sempre usar o mesmo sujeito (usa-se, normalmente, o
pronome correspondente) e o mesmo tempo verbal da oração principal na tag question. Assim,
por exemplo, se a oração principal estiver no presente, a tag question também estará no presente.
Outra dica essencial: a tag question vem sempre “com o sinal trocado”, ou seja, se a oração
principal estiver afirmando, a tag question fica na negativa e vice-versa. Assim, em “The stories
were numerous and varied, __________?”, o sujeito é the stories, they, elas, as histórias, e o
tempo verbal é o passado do verbo to be. Como a oração principal está afirmando, a tag question
ficará na negativa.
A alternativa A está incorreta. Essa alternativa trocou o sujeito.

AULA 03 – ARTICLES AND NOUNS 81


TEACHER ANDREA BELO

A alternativa B está incorreta. Essa alternativa não trouxe a tag question “com o sinal trocado”.
A alternativa C está correta. Ela usou o mesmo sujeito (o pronome correspondente, they) e o
mesmo tempo verbal da oração principal na tag question (passado do verbo to be). Além disso, a
tag question veio com “com o sinal trocado” (oração principal afirmando, tag question negando).
A alternativa D está incorreta. Essa alternativa trocou o sujeito e não trouxe a tag question “com
o sinal trocado”.
GABARITO: C

Questão 06 (EPCAR/INÉDITA) – “Venice, no longer clogged with tourists…” (paragraph 5). It


means that
(A) Venice does not allow tourism anymore.
(B) Venice is not crowded with tourists as it used to be.
(C) Venice does not have a tourism plan any longer.
(D) Venice has been visited by many tourists lately.
Comentários: “Venice, no longer clogged with tourists…” (paragraph 5) quer dizer que A
alternativa A está incorreta. A frase trazida pela questão quer dizer que Veneza não está mais
entupida de turistas, como de costume, e não que Veneza não permite mais turismo, como
afirmado pela letra A.
A alternativa B está correta. Como vimos, a frase trazida pela questão quer dizer que Veneza não
está mais entupida de turistas, como de costume, e a frase trazida pela letra B diz que Veneza não
está mais cheia de turistas como costumava ser.
A alternativa C está incorreta. A frase trazida pela questão quer dizer que Veneza não está mais
entupida de turistas, como de costume, e não que Veneza não tem mais um plano de turismo.
A alternativa D está incorreta. A frase trazida pela questão quer dizer que Veneza não está mais
entupida de turistas, como de costume, e não que tem sido visitada por muitos turistas
ultimamente.
GABARITO: B

Questão 07 (EPCAR/INÉDITA) – The word “eschewed” (paragraph 6) means


(A) chose
(B) consumed
(C) preferred
(D) avoided
Comentários: Nesse tipo de questão, é essencial se atentar a todo o contexto, para que você
consiga deduzir o sentido da palavra cobrada, ainda que não a conheça. No parágrafo 6, o texto
informa que na Índia, uma falsa história de que ninguém que “eschewed” todos os produtos de
origem animal haviam contraído Covid-19 se tornou viral. O texto diz ainda que essa história

AULA 03 – ARTICLES AND NOUNS 82


TEACHER ANDREA BELO

provocou uma queda nas vendas de carne e ovos, atingindo duramente os agricultores e
açougueiros.
Perceba, assim, que se a história falsa levou à queda no consumo de carnes e ovos, significa que
ela pregava que quem evitasse produtos de origem animal não contraia Covid. Nas alternativas,
temos (A) escolhia, (B) consumia, (C) preferia e (D) evitava, sendo a D a única que indica a
abstenção de consumo de produtos de origem animal.
GABARITO: D

Questão 08 (EPCAR/INÉDITA) – According to paragraph 7, The Church of England DIDN’T


(A) confirm the story that York Minster planned to remove a statue of Constantine.
(B) deny that York Minster was planning to remove a statue of Constantine.
(C) talk about the rumor that York Minster was planning to remove a statue of Constantine.
(D) say a word about the story that York Minster planned to remove a statue of Constantine.
Comentários: De acordo com o parágrafo 7, a Igreja da Inglaterra foi forçada a declarar que não
tinha planos de remover uma estátua do imperador romano Constantino, contrariando o boato
que estava circulando. “Perhaps the oddest story was that York Minster planned to remove a
statue of the Roman emperor Constantine because of his support for slavery. The Church of
England was forced to state that they had no such plans.” Assim, de acordo com o parágrafo 7,
a Igreja da Inglaterra NÃO
(A) confirmou a história de que York Minster planejava remover uma estátua de Constantino.
CORRETA.
(B) negou que York Minster estava planejando remover uma estátua de Constantino. INCORRETA.
(C) falou sobre o boato de que York Minster estava planejando remover uma estátua de
Constantino. INCORRETA.
(D) disse uma palavra sobre a história de que York Minster planejava remover uma estátua de
Constantino. INCORRETA.
GABARITO: A

Questão 09 (EPCAR/INÉDITA) – Mark the alternative that CANNOT replace the word “sadly”
(paragraph 5) in the text.
(A) happily
(B) unfortunately
(C) lamentably
(D) unluckly
Comentários: No parágrafo 5, sadly significa infelizmente. Entre as alternativas, apenas happily
não pode substituir sadly, pois significa justamente o oposto (felizmente). Unfortunately e

AULA 03 – ARTICLES AND NOUNS 83


TEACHER ANDREA BELO

unluckly significam também infelizmente e lamentably significa lamentavelmente, podendo essas


três substituírem sadly no contexto.
GABARITO: A

Questão 10 (EPCAR/INÉDITA) – The sentence “Regulate!” (paragraph 11) in the negative form is
(A) Not regulate!
(B) No regulate!
(C) Don’t regulate!
(D) Do regulate!
Comentários: A frase “Regulate!” está no modo imperativo, contendo um comando:
regulamente! Para passar uma frase no imperativo para a forma negativa, basta acrescentar
DON’T antes do verbo, “Don’t regulate!”, e assim o comando fica negativo: não regulamente!
Apenas a letra C trouxe a forma correta, estando as demais alternativas incorretas.
GABARITO: C

Questão 11 (EPCAR/INÉDITA) – Complete the blank in paragraph 11 with the grammatically


correct verb tense.
(A) will see
(B) saw
(C) have seen
(D) sees
Comentários: Observe o trecho que contém a lacuna: If they do not take action to stop lies being
spread, then we __________ ever more anger and division. Temos a estrutura de condicional,
com o verbo no presente na if-clause. Se na if-clause o verbo está no presente (como nos indica
o “don’t”), na main clause, isto é, na oração principal, o verbo só pode estar no futuro (First
Conditional) ou também no Presente (Zero Conditional). Assim, eliminamos as letras B e C. A letra
D traz o verbo no presente, mas conjugado de forma errada para o sujeito we. Ficamos, então,
com o futuro trazido pela letra A. If they do not take action to stop lies being spread, then we will
see ever more anger and division. = Se eles não agirem para impedir que mentiras se espalhem,
veremos cada vez mais raiva e divisão.
GABARITO: A

Questão 12 (EPCAR/INÉDITA) – “however” (paragraph 1) introduces an idea of


(A) comparison.
(B) contrast.
(C) conclusion.
(D) exemplification.

AULA 03 – ARTICLES AND NOUNS 84


TEACHER ANDREA BELO

Comentários: “however” (parágrafo 1) apresenta uma ideia de contraste (letra B). “What is true,
however, is that debate about misinformation online reached a fever pitch this year.” = O que é
verdade, contudo, é que o debate sobre a desinformação online atingiu um pico febril este ano.
However não indica comparação (letra A), conclusão (letra C) ou exemplificação (letra D).
GABARITO: B

Questão 13 (EPCAR/INÉDITA) – The sentence “Allowing either the government or large


companies to decide what is true or false is a recipe for disaster.” (paragraph 12) means that
(A) it is advisable to let the government or large companies decide what is true or not.
(B) countries should allow the government to decide what is true or false.
(C) people must trust large companies to decide what is true or not.
(D) it can be dangerous to let the government or large companies decide what is true or false.
Comentários: A frase “Allowing either the government or large companies to decide what is true
or false is a recipe for disaster.” Quer dizer que “Permitir que o governo ou grandes empresas
decidam o que é verdadeiro ou falso é uma receita para o desastre.” Assim, A frase significa que
(A) é aconselhável deixar que o governo ou grandes empresas decidam o que é verdade ou não.
INCORRETA.
(B) os países devem permitir que o governo decida o que é verdadeiro ou falso. INCORRETA.
(C) as pessoas devem confiar nas grandes empresas para decidir o que é verdade ou não.
INCORRETA.
(D) pode ser perigoso deixar o governo ou grandes empresas decidirem o que é verdadeiro ou
falso. CORRETA.
GABARITO: D

Questão 14 (EPCAR/INÉDITA) – Choose the option which contains a superlative


adjective extracted from paragraph 7.
(A) racist
(B) slavery
(C) oddest
(D) iconoclasts
Comentários: A única opção que contém um adjetivo superlativo extraído do parágrafo 7 é a letra
C, oddest (the oddest - o mais estranho, no texto, a mais estranha, pois se refere a uma história).
Racist é um adjetivo, significa racista, mas não está no superlativo (letra A). Slavery é um
substantivo e significa escravidão (letra B). Iconoclasts é um substantivo e se refere àqueles que
destroem imagens religiosas ou se opõem à sua adoração.
GABARITO: C

AULA 03 – ARTICLES AND NOUNS 85


TEACHER ANDREA BELO

Questão 15 (EPCAR/INÉDITA) – Mark the sentence below that can answer the following
question correctly. “Have you heard that the Covid-19 vaccine will rewrite your DNA?”
(paragraph 1).
(A) Yes, I’ve.
(B) No, I haven’t.
(C) No, I’ve not.
(D) Yes, I had.
Comentários: A pergunta está no Present Perfect, com sujeito you. Assim, as únicas respostas
possíveis são Yes, I have ou No, I haven’t.
A letra A está incorreta, pois não fazemos contração em respostas curtas afirmativas.
A letra B está correta, conforme a explicação.
A letra C está incorreta, pois a única contração correta e possível na resposta curta negativa é
unindo have e not (haven’t).
A letra D está incorreta, pois alterou o tempo verbal para o Past Perfect.
GABARITO: B

QUESTÕES ESA
Texto para responder à questão 01.
RIO DE JANEIRO, BRAZIL – In the past 24 hours, Brazil recorded 1,113 deaths and 36,653 new cases
related to the novel coronavirus. The data are included in the Ministry of Health's daily balance
released on Tuesday evening, September 15th.
Since the start of the pandemic, 133,119 people have died as a result of Covid-19. On Monday,
the Ministry of Health's data system recorded a total of 132,006 deaths. A further 2,445 are still
under investigation by health authorities.
(Adapted from https://riotimesonline.com - September 15th)

Questão 01 (ESA/INÉDITA) – According to the text, it is correct to say that:


A) more than a thousand hundred deaths were recorded.
B) a thousand and half deaths were registered.
C) many people died becaise of the coronavirus.
D) coronavirus left more than a thousand people sick.
E) more than thirty six thousand deaths were registered.
Comentários: A pergunta é: de acordo com o texto o que é correto dizer.
Na letra A, temos a afirmativa de que mais de 1100 mortes foram regristradas e é o nosso gabarito
pois, na primeira linha diz que foram 1113, ou seja, mais de 1100.

AULA 03 – ARTICLES AND NOUNS 86


TEACHER ANDREA BELO

Na alternativa B, diz que foram regristradas mil e metade mas não fala qual é essa metade,
portanto, está incorreta.
Na letra C, diz que muitas pessoas morreram, mas está muito vaga essa afirmativa.
Na letra D, diz que o coronavirus deixou mais de mil pessoas doentes mas a quantidade é muito
maior.
Na letra E diz que mais de 36 mil mortes foram regristradas mas não, foram registrados mais de
36 mil novos casos. Portanto, gabarito letra A.
GABARITO: A

Questão 02 (ESA/INÉDITA) – Which sentence is grammatically correct?


(A) Last week, I worked out more earlieron Tuesday than the rest of the week.
(B) Albert is more big than his boss.
(C) My apartment is more big than your office downtown.
(D) Traveling to Europe is expensiver than going to the USA.
(E) Anne Claire’s sneakers are cheaper than mine.
Comentários: A alternativa A está incorreta. Não se utiliza “more” junto com “earlier”, pois
“earlier” já é uma estrutura de frase comparativa, usando -ier no final do adjetivo considerado
curto. Utiliza-se apenas “earlier”, pois essa palavra, apenas, já oferece o sentido comparativo que
a sentença busca. O uso do “more” é para adjetivos longos, como “more beautiful/more
intelligent etc”.
A alternativa B está incorreta. O modo comparativo de “BIG” é “bigger”. Só se usa “more” antes
de adjetivo quando este adjetivo tiver mais de uma sílaba, ou seja, quando for um adjetivo
considerado longo. E não se usa BIG para pessoa e sim TALL, estaria certo taller than e não bigger.
A alternativa C está incorreta. A mesma explicação da alternativa anterior se aplica no caso de
“big” o correto seria “bigger”, como dito anteriormente e não more big.
A alternativa D está incorreta. A palavra “expensive” é uma palavra de, pelo menos, três sílabas.
Portanto, é correto dizer “more expensive”, segundo a regra de palavras curtas ou longas e não
podemos colocar -er no fim, fommando “expensiver”, que não existe.
A alternativa E está correta. “Cheaper” é a forma correta de fazer o comparativo de “cheap”.
GABARITO: E

Questão 03 (ESA/INÉDITA) – Complete the sentence below using the appropriate words:
“The old man who works here __________ always complaining: He __________ like children
but __________ talking about them to everyone.”
(A) is /don’t/love
(B) is/doesn’t/loves
(C) isn’t/do/loves
(D) isn’t/don’t/love
(E) is/does/love

AULA 03 – ARTICLES AND NOUNS 87


TEACHER ANDREA BELO

Comentários: Essa é uma questão que exige conhecimento dos tempos verbais e suas
conjugações no Present Simple (presente simples) para verbos em geral e verbo to be e os
auxiliares (do/does) e as formas afirmativa e negativa.
O sujeito “The old man” pode ser substituído pelo pronome “he” (ele). Sempre que se conjuga
verbo na terceira pessoa do singular (he/she/it), no verbo to be, usa-se IS.
A segunda lacuna deve ser preenchida com o auxiliar “doesn’t”, pois o sujeito está na terceira
pessoa do singular e exige “doesn’t” (does not), segundo a regra dos verbos na terceira pessoa,
modo negativo, ao invés de “don’t”, para sujeitos I, you e todos os demais que se encontram no
plural.
A terceira lacuna deve ser preenchida por “loves”, pois quem ama os trens é ele (he). Portanto,
temos mais uma vez o sujeito na terceira pessoa do singular, que exige o acréscimo da letra “S”
no final do verbo no tempo presente.
Todas essas regras acima estão em nosso PDF na aula 02, explicados com detalhes.
GABARITO: B

Questão 04 (ESA/INÉDITA) – “__________ my new neighbour? Where did you come from?”
Complete the space with the correct form of the verb and the pronoun.
(A) You is
(B) You are
(C) Are you
(D) Is you
(E) Am you
Comentários: Essa é uma questão que exige conhecimento das conjugações do verbo to be no
Present Simple (presente simples) bem como as formas afirmativa e interrogativa. Estão em nossa
aula 02, em PDF e videoaulas do Estratégia Militares, da teacher Andrea Belo.
A alternativa A está incorreta. “You” não pode ser seguido de “is”, devido à sua conjugação,
segundo qualquer gramática e a forma do verbo to be no plural é “are” (You are). O verbo deve
estar antes do pronome “you”, pois trata-se de uma pergunta.
A alternativa B está incorreta. A ordem de pronome (you) e verbo (are) deveria estar invertida
por ser de uma pergunta, em que o verbo to be passa a ser o primeiro da estrutura da frase.
A alternativa C está correta. Numa frase interrogativa, deve-se inverter a ordem de pronome
sujeito (you) e verbo to be (are). Portanto, “Are you my new neighbour?” (Você é meu novo
vizinho?’) é gramaticalmente correto.
A alternativa D está incorreta. Para o sujeito “You” não se usa “is”, como vimos anteriormente e
sempre “are”.
A alternativa E está incorreta. “You” não é acompanhado de “am”, sempre de “are”.
GABARITO: C

AULA 03 – ARTICLES AND NOUNS 88


TEACHER ANDREA BELO

Texto para as questões 05, 06 e 07.


September 16th 2020.
RIO DE JANEIRO, BRAZIL – Eternit, a manufacturer of building materials, has been awarded a
certificate by INMETRO (Product Certification Body) for a concrete roof tile capable of producing
energy. As a result, the company can now market the product. The roof tile collects energy from
the sun (…)
(Adapted from https://riotimesonline.com)

Questão 05 (ESA/INÉDITA) – According to the text, it is correct to say that:


A) Eternit company received an award.
B) it is said Eternit tried to receive an award.
C) the award was specifically to different companies.
D) more than a company got a certificate.
E) the building materials provided an award to the company.
Comentários: Na letra A, diz que a Eternit recebeu um prêmio. É o nosso gabarito pois, na primeira
linha diz Eternit... has been awarded (foi premiada).
Na letra B, diz que a Eternit tentou receber, com o verbo tried, mas ela recebeu.
Na letra C, diz que o prêmio foi para diferentes empresas, plural, mas foi apenas para a Eternit.
Na letra D, diz que mais de uma companhia ganhou o certificado mas foi apenas a Eternit que
ganhou o prêmio.
Na letra E, fala-se sobre materiais providenciarem um prêmio porém essa alternativa está muito
vaga e comprova que o gabarito é a letra A.
GABARITO: A

Questão 06 (ESA/INÉDITA) – The term “roof tile” refers to the


(A) ceiling
(B) ranch
(C) rope
(D) race
(E) couple
Comentários: A alternativa A afirma que “roof tile” (telha em ingles) se refere à ceiling (teto) e
está correto.
A letra B diz que se refere a um rancho, incorreto.
A letra C diz que se refere a uma corda, incorreto.
A letra D que se refere a uma corrida, incorreto.
A letra E diz que se refere a um casal, incorreto.
GABARITO: A

AULA 03 – ARTICLES AND NOUNS 89


TEACHER ANDREA BELO

Questão 07 (ESA/INÉDITA) – The sentence “...collects energy from the sun” means that:
(A) it refers to a different kind of energy.
(B) it refers to solar energy.
(C) it refers to a brandnew type of energy.
(D) it refers to pools in the garden.
(E) it is made by natural elements.
Comentários: Na letra A diz que, a frase do ennunciado collects energy from the sun se refere a
um tipo diferente de energia. Até está correto porém, a alternativa B está mais completa pois,
coletar a energia do sol é a mesma coisa que dizer energia solar. Portanto, gabarito letra B.
A alternativa C diz que se refere a um tipo de energia novidade (brandnew) mas já vimos que a
alternativa B está melhor.
Na letra D diz que se refere a piscinas e não faz sentido.
Na letra E diz que é feito de elementos naturais, poré, já vimos que a alternativa B é a mais
completa, energia solar, comprovando nosso gabarito, letra B.
GABARITO: B

Questão 08 (ESA/INÉDITA) – “__________ writing you emails? Yes, she is wirting lots of them
day by day and I also answer her with poems on the emails. We are in love.”
Complete the space with the correct form of the verb and the pronoun.
(A) Are the girl
(B) She is
(C) Is she
(D) Are she
(E) Am she
Comentários: A frase é “Ela está escrevendo emails a você? Sim, ela está escrevendo muitos dia
após dia e eu respondo com poemas nos emails. Estamos apaixonados.”
A alternativa A está incorreta. “SHE” (the girl) não pode ser seguido de “are”, devido à sua
conjugação, segundo qualquer gramática e a forma do verbo to be no singular é “IS” (You are). O
verbo deve estar antes dos pronomes “he/she/it”, pois trata-se de uma pergunta.
A alternativa B está incorreta. A ordem de pronome (he/she/it) e verbo (is) deveria estar invertida
por ser de uma pergunta, em que o verbo to be passa a ser o primeiro da estrutura da frase.
A alternativa C está correta. Numa frase interrogativa, deve-se inverter a ordem de pronome
sujeito (she) e verbo to be (is). Portanto, “Is she working aat the company?” é gramaticalmente
correto.
A alternativa D está incorreta. Para o sujeito “She” não se usa “are”, como vimos anteriormente
e sempre “is”.
A alternativa E está incorreta. “She” não é acompanhado de “am”, sempre de “is”.
GABARITO: C

AULA 03 – ARTICLES AND NOUNS 90


TEACHER ANDREA BELO

Questão 09 (ESA/INÉDITA) – The word “them” in “She agrees with all of them” is
(A) a pronoun
(B) a conjunction
(C) a preposition
(D) a noun
(E) an adjective
Comentários: “Them” significa “eles/elas”, é o “object pronoun” - pronome objeto, no final de
frases – “She agrees with all of them” (Ela concorda com todos eles) e é o pronome que se refere
ao sujeito “they” (eles/elas)
Nas letras B, C, D e E, as funções gramaticais estão incorretas. Não há como “them” ser uma
conjunção ou preposição ou substantivo ou sequer adjetivo, devido à classificação gramatical que
possui.
GABARITO: A

Questão 10 (ESA/INÉDITA) – The word “naive” in “I think he is a little naive” is


(A) a verb
(B) a conjunction
(C) a preposition
(D) a adverb
(E) an adjective
Comentários: “Naive” significa “inocente”, sem malícias, é um adjetivo, que, nas frases, devem
vir antes dos substantivos. Diferente de “innocent”, que é inocente de quem não é culpado, no
tribunal, por exemplo.
Nas letras A, B, C e D, as funções gramaticais estão incorretas. Não há como “useful” ser um verbo,
uma conjunção ou preposição ou um advérbio.
GABARITO: E

QUESTÕES ESCOLA NAVAL


Covid babies don't have to be the unlucky generation. But they must be helped
A year into this pandemic, there are babies now learning to walk and talk who have never known
anything but life under the shadow of Covid, and preschoolers who can barely remember a world
before it. Doctors' children have had to learn not to touch Mummy when she gets in from work,
until she's had a shower to wash off any last possible trace of danger. Thanks to popular toddler
demand, you can now buy masks for dolls.
It's perfectly normal for kids to reflect what's happening around them by playing quarantine with
stuffed animals, but normal too for adults to wonder uncomfortably whether all this leaves a
lasting mark. How much will it matter in years to come that, as the minister for loneliness Diana

AULA 03 – ARTICLES AND NOUNS 91


TEACHER ANDREA BELO

Barran recently put it, there are toddlers being raised by shielding parents who have never had a
playdate? Will Covid babies grow up solitary creatures, used to entertaining themselves, or warier
of the strangers they so rarely meet and interact with? Taking a tiny bundle out in public used to
mean an endless succession of random older women cooing over the pram, or strangers pulling
faces to entertain a bored baby in a checkout queue. But now passersby daren't get close, and
other shoppers are hidden behind masks.
This week MPs were presented with some early findings from a project led by the First 1001 Days
Movement, an alliance of early-years charities and professionals, tracking the lives of under-twos
growing up through a pandemic. A survey of children's service providers it commissioned found
98% thought the babies and toddlers they worked with had been affected by higher parental
stress and anxiety, while 92% had seen fearful families effectively cutting themselves off from the
outside world, skipping routine appointments or not wanting to leave the house. Nine in 10 had
observed children being played with less, or being less active. Heartbreakingly, more than a
quarter said lockdown left the children they worked with more exposed to domestic conflict,
abuse or neglect.
(Adapted from https://mmm.theguardian.com/commentisfree/2021/jan/22/covid-babies-unlucky-generation-help-lockdown)

Questão 01 (ESCOLA NAVAL/INÉDITA) – According to the text, which option is correct?


(A) There are children who have not yet experienced life outside the pandemic
(B) The pandemic has not affected the lives of any children, as they do not understand what is
happening
(C) The pandemic, in a way, positively impacted the babies' lives
(D) The children's routine has not changed in this pandemic
(E) Characteristics of the pandemic did not impact areas such as sales
Comentários: A alternativa A está correta. De acordo com o texto, é correto afirmar que há
crianças que ainda não experimentaram a vida fora da pandemia, assim como esta opção indica.
Isso pode ser confirmado com o trecho “...there are babies now learning to walk and talk who
have never known anything but life under the shadow of Covid…”.
A alternativa B está incorreta. De acordo com o texto, não é correto afirmar que a pandemia não
afetou a vida de nenhuma criança, pois elas não entendem o que está acontecendo, mas sim, que
afetou. Isso pode ser confirmado com o trecho “Doctors’ children have had to learn not to touch
Mummy when she gets in from work…”.
A alternativa C está incorreta. De acordo com o texto, não é correto afirmar que a pandemia, de
certa forma, impactou positivamente a vida dos bebês, mas sim, que impactou negativamente.
Isso pode ser confirmado com o trecho “Doctors’ children have had to learn not to touch Mummy
when she gets in from work, until she’s had a shower to wash off any last possible trace of
danger”.
A alternativa D está incorreta. De acordo com o texto, não é correto afirmar que a rotina das
crianças não mudou nesta pandemia, mas sim, que mudou. Isso pode ser confirmado com o
trecho “Doctors’ children have had to learn not to touch Mummy when she gets in from work…”.

AULA 03 – ARTICLES AND NOUNS 92


TEACHER ANDREA BELO

A alternativa E está incorreta. De acordo com o texto, não é correto afirmar que as características
da pandemia não impactaram áreas como vendas, mas sim, que impactaram. Isso pode ser
confirmado com o trecho “Thanks to popular toddler demand, you can now buy masks for dolls”.
GABARITO: A

Questão 02 (ESCOLA NAVAL/INÉDITA) – What’s the meaning of the word “toddler” in


paragraph 1?
(A) Population
(B) Children
(C) Babies
(D) Adults
(E) Teens
Comentários: A alternativa A está incorreta. A palavra “toddler” não significa população, mas sim,
“children” (crianças).
A alternativa B está correta. A palavra “toddler” significa crianças, assim como esta opção indica.
A alternativa C está incorreta. A palavra “toddler” não significa bebês, mas sim, “children”
(crianças).
A alternativa D está incorreta. A palavra “toddler” não significa adultos, mas sim, “children”
(crianças).
A alternativa E está incorreta. A palavra “toddler” não significa adolescentes, mas sim, “children”
(crianças).
GABARITO: B

Questão 03 (ESCOLA NAVAL/INÉDITA) – In paragraph 1, the word “she” refers to


(A) Children
(B) Doctor
(C) Toddler
(D) Mummy
(E) Babies
Comentários: A alternativa A está incorreta. A palavra “she” (ela) não se refere às crianças, mas
sim, a mamãe (mummy). Isso pode ser confirmado com o trecho “Doctors’ children have had to
learn not to touch Mummy when she gets in from work…”.
A alternativa B está incorreta. A palavra “she” (ela) não se refere a doutora, mas sim, a mamãe
(mummy). Isso pode ser confirmado com o trecho “Doctors’ children have had to learn not to
touch Mummy when she gets in from work…”.

AULA 03 – ARTICLES AND NOUNS 93


TEACHER ANDREA BELO

A alternativa C está incorreta. A palavra “she” (ela) não se refere às crianças, mas sim, a mamãe
(mummy). Isso pode ser confirmado com o trecho “Doctors’ children have had to learn not to
touch Mummy when she gets in from work…”.
A alternativa D está correta. A palavra “she” (ela) se refere a mamãe (mummy), assim como esta
opção indica. Isso pode ser confirmado com o trecho “Doctors’ children have had to learn not to
touch Mummy when she gets in from work…”.
A alternativa E está incorreta. A palavra “she” (ela) não se refere aos bebês, mas sim, a mamãe
(mummy). Isso pode ser confirmado com o trecho “Doctors’ children have had to learn not to
touch Mummy when she gets in from work…”.
GABARITO: D

Questão 04 (ESCOLA NAVAL/INÉDITA) – According to the text, which option completes the
sentence below, correctly? The pandemic __________ an entire generation.
(A) Affects
(B) Affected
(C) Was affecting
(D) Had affected
(E) Is affecting
Comentários: A alternativa A está incorreta. A melhor forma de completar a frase, de acordo com
o texto, não é com o verbo “to affect” (afetar) no simple present (affects – afeta), mas sim, no
present continuous (is affecting – está afetando). Isso pode ser confirmado com o trecho “...there
are babies now learning to walk and talk who have never known anything but life under the
shadow of Covid…”.
A alternativa B está incorreta. A melhor forma de completar a frase, de acordo com o texto, não
é com o verbo “to affect” (afetar) no simple past (affected – afetou), mas sim, no present
continuous (is affecting – está afetando). Isso pode ser confirmado com o trecho “...there are
babies now learning to walk and talk who have never known anything but life under the shadow
of Covid…”.
A alternativa C está incorreta. A melhor forma de completar a frase, de acordo com o texto, não
é com o verbo “to affect” (afetar) no past continuous (was affecting – estava afetando), mas sim,
no present continuous (is affecting – está afetando). Isso pode ser confirmado com o trecho
“...there are babies now learning to walk and talk who have never known anything but life under
the shadow of Covid…”.
A alternativa D está incorreta. A melhor forma de completar a frase, de acordo com o texto, não
é com o verbo “to affect” (afetar) no past perfect (had affected – afetou), mas sim, no present
continuous (is affecting – está afetando). Isso pode ser confirmado com o trecho “...there are
babies now learning to walk and talk who have never known anything but life under the shadow
of Covid…”.
A alternativa E está correta. A melhor forma de completar a frase, de acordo com o texto, é com
o verbo “to affect” (afetar) no present continuous (is affecting – está afetando), assim como esta

AULA 03 – ARTICLES AND NOUNS 94


TEACHER ANDREA BELO

opção indica. Isso pode ser confirmado com o trecho “...there are babies now learning to walk
and talk who have never known anything but life under the shadow of Covid…”.
GABARITO: E

Questão 05 (ESCOLA NAVAL/INÉDITA) – According to the text, which option is correct?


(A) Children do not realize what happens around them
(B) The impact of the pandemic on children is a normal concern of parents
(C) There is no doubt about this generation that was born in the middle of a pandemic
(D) The relationship with children on the street changed with the pandemic
(E) The impact of the pandemic on children can only be short-term
Comentários: A alternativa A está incorreta. De acordo com o texto, não é correto afirmar que as
crianças não percebem o que acontece ao seu redor, mas sim, que percebem. Isso pode ser
confirmado com o trecho “It’s perfectly normal for kids to reflect what’s happening around
them…”.
A alternativa B está correta. De acordo com o texto, é correto afirmar que o impacto da pandemia
nas crianças é uma preocupação normal dos pais, assim como esta opção indica. Isso pode ser
confirmado com o trecho “...but normal too for adults to wonder uncomfortably whether all this
leaves a lasting mark”.
A alternativa C está incorreta. De acordo com o texto, não é correto afirmar que não há dúvidas
sobre esta geração que nasceu no meio de uma pandemia, mas sim, que existem muitas dúvidas.
Isso pode ser confirmado com o trecho “Will Covid babies grow up solitary creatures, used to
entertaining themselves, or warier of the strangers they so rarely meet and interact with?”.
A alternativa D está incorreta. De acordo com o texto, não é correto afirmar que a relação com as
crianças na rua mudou com a pandemia, mas sim, que mudou. Isso pode ser confirmado com o
trecho “But now passersby daren’t get close, and other shoppers are hidden behind masks”.
A alternativa E está incorreta. De acordo com o texto, não é correto afirmar que o impacto da
pandemia nas crianças só pode ser de curto prazo, mas sim, que pode ser a longo prazo. Isso pode
ser confirmado com o trecho “Will Covid babies grow up solitary creatures, used to entertaining
themselves, or warier of the strangers they so rarely meet and interact with?”.
GABARITO: B

QUESTÕES EsPCEx
Escolha a única alternativa correta, dentre as opções apresentadas, que responde ou completa
cada questão, assinalando-a.
Leia o texto a seguir e responda às questões 01, 02 e 03.
Hair ice: The strange phenomenon of 'candy floss' on trees
If you (…) for a winter walk, you can be in for a big surprise. People taking a stroll through the trees
in counties Fermanagh and Tyrone have been encountering an unusual sight. At first glance, it

AULA 03 – ARTICLES AND NOUNS 95


TEACHER ANDREA BELO

looks like candy floss has appeared on tree branches. On closer inspection, you can see hundreds
of individual strands of what looks like delicate white hair.
As soon as these are touched by human hand or winter sun, they melt away. This strange
phenomenon is called hair ice. The crystals are formed on rotting wood on humid winter nights
when the temperature is just below zero. Scientists have discovered it is caused by a fungus which
enables the ice to form thin hairs with a diameter of about 0.01mm.
Hair ice - also known as frost flowers - has been spotted in County Fermanagh in Castle Archdale,
Florence Court, Castle Coole, Cladagh Glen and Big Dog Forest. In County Tyrone, it has been
pictured in Omagh and Knockmany, near Augher. So if you're heading out for some daily exercise,
see if you can spot this rare phenomenon before it melts away.
Adapted from https://www.bbc.com/news/uk-northern-ireland-55531529

Questão 01 (EsPCEx/INÉDITA) – Choose the statement which correctly functions as the missing
part of paragraph 1. If you (…) for a winter walk, you can be in for a big surprise.
(A) ’re looking forward
(B) dive into the ocean
(C) go down to the woods
(D) desperately need
(E) went
Comentários: A alternativa A está incorreta. A expressão correta é to be looking forward to,
seguida de verbo com -ing, significando que alguém mal vê a hora que algo aconteça, está ansioso,
com boa expectativa quanto a uma situação que vai ou pode acontecer. No trecho, para ficar
correto, teria que estar assim: If you’re looking forward to going for a winter walk... = Se você mal
vê a hora de fazer uma caminhada de inverno...
A alternativa B está incorreta. “dive into the ocean” não faz sentido no contexto, pois significa
“mergulhar no oceano”. Ficaria assim: Se você mergulhar no oceano para uma caminhada de
inverno... (Whaaat?!). O texto fala sobre um fenômeno que acontece nas árvores no inverno.
Atenção ao verbo dive, que significa mergulhar, cuja compreensão era crucial para eliminar a letra
B.
A alternativa C está correta. “go down to the woods” faz perfeito sentido no contexto: If you go
down to the woods for a winter walk, you can be in for a big surprise. = Se você for à floresta para
uma caminhada de inverno, poderá ter uma grande surpresa.
A alternativa D está incorreta. O verbo need não é, em regra, complementado pela preposição for.
Usamos need direto com o complemento, sem preposição. If you need for a winter walk, you can
be in for a big surprise.
A alternativa E está incorreta. Went não completa adequadamente o trecho, pois, geralmente,
quando usamos um verbo no passado na if clause, para falar de hipótese, na outra oração, a que
descreve a consequência caso a situação hipotética se concretizasse, o verbo também estaria no
passado. Exemplo: “If you went for a winter walk, you could be in for a big surprise.” = Se você
fosse à floresta para uma caminhada de inverno, poderia ter uma grande surpresa.
GABARITO: C

AULA 03 – ARTICLES AND NOUNS 96


TEACHER ANDREA BELO

Questão 02 (EsPCEx/INÉDITA) – Choose the alternative that has the same meaning as the word
glance in the sentence “At first glance, it looks like candy floss has appeared on tree branches.”
(paragraph 1).
(A) research
(B) gender
(C) inquiry
(D) option
(E) look
Comentários: A expressão at first glance significa à primeira vista. Glance se refere a uma olhada
rápida.
A alternativa A está incorreta. Research significa pesquisa, não correspondendo ao sentido de
glance.
A alternativa B está incorreta. Gender significa gênero, não correspondendo ao sentido de glance.
A alternativa C está incorreta. Inquiry significa investigação, não correspondendo ao sentido de
glance.
A alternativa D está incorreta. Option significa opção, não correspondendo ao sentido de glance.
A alternativa E está correta. Look, assim como glance, pode significar olhada. Assim, essa
alternativa tem o mesmo significado da palavra glance na frase.
GABARITO: E

Questão 03 (EsPCEx/INÉDITA) – According to the text, choose the correct alternative.


(A) Hair ice refers to actual hair.
(B) Scientists are yet to find out what causes hair ice.
(C) Frost flowers are a more common type of hair ice.
(D) Humans can hurt their hands by touching frost flowers.
(E) Frost flowers are formed due to a fungus.
Comentários: A alternativa A está incorreta. “Hair ice” não se refere a cabelo de verdade (actual).
Esse é apenas o nome dado a cristais de gelo que se formam em árvores e se parecem com fios
de cabelos brancos.
A alternativa B está incorreta. Ela afirma que os cientistas ainda não descobriram o que causa o
“hair ice”, que ainda têm que descobrir, o que não está de acordo com o texto. “Scientists have
discovered it is caused by a fungus which enables the ice to form thin hairs with a diameter of
about 0.01mm.”
A alternativa C está incorreta. “Frost flowers”, de acordo com o texto, não são um tipo mais comum
de “hair ice”, mas apenas uma outra donominação para o mesmo fenômeno. “Hair ice - also
known as frost flowers...”

AULA 03 – ARTICLES AND NOUNS 97


TEACHER ANDREA BELO

A alternativa D está incorreta. O texto não afirma que os humanos podem machucar as mãos ao
tocar em “frost flowers” ou “hair ice”, mas sim que essas delicadas estruturas de gelo derretem
assim que são tocadas. “As soon as these are touched by human hand or winter sun, they melt
away.”
A alternativa E está correta. As “frost flowers” são formadas devido a um fungo. “Scientists have
discovered it is caused by a fungus which enables the ice to form thin hairs with a diameter of
about 0.01mm.”
GABARITO: E

Leia o texto a seguir e responda às questões 04, 05 e 06.


Where is Jack Ma? Chinese tycoon not seen since October
Speculation is mounting over the whereabouts of the Chinese billionaire Jack Ma, who has not
been seen or heard in public for more than two months. Ma, the co-founder and former chairman
of the technology firm Alibaba, has fallen out of favour with China’s leadership. In late October, he
stood alongside senior officials and delivered a blunt speech criticizing national regulators,
reportedly infuriating China’s president, Xi Jinping.
In the following months, regulators summoned Ma and other executives in for questioning and
halted what would have been the world’s biggest share offering of his company, Ant Group. They
later launched anti-monopoly investigations into Alibaba and its key competitor TenCent, and
called Ant Group in for questioning. In late December, regulators ordered Ma to pare down his
empire.
Ma has not been heard from during the controversies. His last public appearance appears to have
been at a livestreamed event on 31 October, an opening ceremony of the annual Chinese shopping
event Double 11. In November, he reportedly did not appear in his role as a judge for the final
episode of entrepreneurial game show Africa’s Business Heroes, which Alibaba has since blamed
on a scheduling conflict.
Ma was replaced with another Alibaba executive and his photo removed from the website. He
also did not appear at this year’s Shanghai Zhejiang chamber of commerce meeting, where he has
delivered a speech every year since 2016. Bloomberg reported Ma had been warned in early
December not to leave the country. Observers are being cautious in drawing any conclusions, with
so little confirmed information.
Adapted from https://www.theguardian.com/business/2021/jan/05/where-is-jack-ma-chinese-tycoon-not-seen-since-october-alibaba

Questão 04 (EsPCEx/INÉDITA) – In the title, the word tycoon refers to


(A) a powerful company.
(B) a weather phenomenon.
(C) bankruptcy.
(D) a wealthy person.
(E) an underprivileged person.

AULA 03 – ARTICLES AND NOUNS 98


TEACHER ANDREA BELO

Comentários: No título, a palavra tycoon se refere a um magnata, uma pessoa muito rica.
A alternativa A está incorreta. No título, a palavra tycoon não se refere a uma empresa poderosa,
mas a uma pessoa muito rica, no caso, um dos cofundadores do Alibaba.
A alternativa B está incorreta. No título, a palavra tycoon não se refere a um fenômeno climático,
mas a um magnata. Atenção: uma palavra semelhante e que se refere a um fenômeno climático é
typhoon, que significa tufão.
A alternativa C está incorreta. No título, a palavra tycoon não se refere a falência, mas a um
magnata.
A alternativa D está correta. No título, a palavra tycoon se refere a um magnata, uma pessoa muito
rica. Fique atento, pois wealthy significa rico.
A alternativa E está incorreta. No título, a palavra tycoon não se refere a uma pessoa
desfavorecida, mas a uma pessoa muito rica, um magnata, no caso, um dos cofundadores do
Alibaba.
GABARITO: D

Questão 05 (EsPCEx/INÉDITA) – In the sentence “Ma, the co-founder and former


chairman of the technology firm Alibaba” (paragraph 1), the prefix co
(A) means sharing a job or responsibility.
(B) gives the opposite meaning of founder.
(C) means Ma does not own Alibaba any longer.
(D) describes something that was poorly executed.
(E) means acting by oneself.
Comentários: A alternativa A está correta. Na frase trazida pela questão, o prefixo co significa
compartilhar um trabalho ou responsabilidade. Co-founder significa cofundador, com o prefixo
indicando que Ma não fundou o Alibaba sozinho.
A alternativa B está incorreta. O prefixo co não dá o significado oposto de fundador.
A alternativa C está incorreta. O prefixo co não significa que Ma não possui mais o Alibaba.
A alternativa D está incorreta. O prefixo co não descreve algo que foi mal executado.
A alternativa E está incorreta. O prefixo co não significa agir por si mesmo.
GABARITO: A

Questão 06 (EsPCEx/INÉDITA) – Choose the statement in which the word deliver or one of its
forms has been used in the same way as in paragraph 4.
(A) Many goods were not delivered in time for Christmas.
(B) She delivered her first child at home.
(C) Is there anything that can be done to deliver the world from Covid?
(D) The priest delivered a touching sermon against hate.
(E) The store is delivering my new wardrobe anytime now.

AULA 03 – ARTICLES AND NOUNS 99


TEACHER ANDREA BELO

Comentários: Vejamos qual é o sentido de deliver no parágrafo 4: “where he has delivered a


speech every year since 2016.” = onde ele fez, deu um discurso todos os anos desde 2016.
A alternativa A está incorreta. O sentido de deliver nessa alternativa é literal, entregar. Muitas
mercadorias (goods) não foram entregues a tempo para o Natal.
A alternativa B está incorreta. Deliver na letra B tem o sentido de dar à luz. Ela deu à luz seu
primeiro filho em casa.
A alternativa C está incorreta. Na letra C, deliver from tem sentido de salvar alguém de alguma
coisa. Há algo que pode ser feito para livrar o mundo da Covid?
A alternativa D está correta. Aqui, deliver tem o mesmo sentido em que foi usado no texto. O
padre fez, deu um sermão comovente contra o ódio.
A alternativa E está incorreta. Como na letra A, o sentido de deliver nessa alternativa é literal,
entregar. A loja vai entregar meu novo guarda-roupa a qualquer momento.
GABARITO: D

Leia o texto a seguir e responda às questões 07, 08 e 09.


‘Bean dad’ apologises after tin can posts cause outcry
A dad in the US who boasted about his parenting skills after telling his hungry nine-year-old
daughter to open a tin of beans or go without food says he is “deeply sorry” about his tweets.
John Roderick, a podcaster and musician, said in a statement that he wanted to “acknowledge and
make amends for the injuries” caused by his posts. He said they were framed in a certain way to
reflect his “comedic persona”.
Twitter users, who nicknamed him “bean dad”, condemned the reported incident, suggesting it
was poor parenting. In an apology posted on his website on Tuesday, Mr Roderick said he had
been “ignorant” and “insensitive” with the way he had described the incident.
Mr Roderick, who has since deactivated his Twitter account, said he was wrong for being “flippant
when confronted” and for “taking my Twitter feed offline yesterday instead of facing the music”.
What was the Twitter story? Mr Roderick shared a story on Saturday on Twitter, beginning with
his daughter asking him to make baked beans. After she brought him a tin-opener and can of
beans, he asked her how she thought a tin-opener worked, he said. When she said she didn't
know, he said he realised “a teaching moment just dropped into my lap”. Explaining that he wanted
his daughter to learn how to open a tin of beans, he said she tried for six hours.
Eventually she opened the tin and ate the beans, he explained, claiming it was a victory for “good
parenting”. But some Twitter users accused Mr Roderick of “child abuse”, suggesting his actions
were “ridiculous” and that it was right for his daughter to ask for “help and support”.
Adapted from https://www.bbc.com/news/world-us-canada-55549536

AULA 03 – ARTICLES AND NOUNS 100


TEACHER ANDREA BELO

Questão 07 (EsPCEx/INÉDITA) – According to the text, choose the correct alternative.


(A) Roderick’s daughter never managed to open the can.
(B) Roderick showed no remorse for his behavior.
(C) According to Roderick, he was trying to teach his daughter a life lesson.
(D) Roderick uses “bean dad” as a stage name on his stand-up comedy show.
(E) Roderick’s daughter did not try long to open the tin.
Comentários: A alternativa A está incorreta. “A filha de Roderick nunca conseguiu abrir a lata.”
não está de acordo com o texto.
“Eventually she opened the tin and ate the beans, he explained, claiming it was a victory for ‘good
parenting’”. “Por fim, ela acabou abrindo a lata e comeu o feijão, explicou ele, alegando que era
uma vitória para bons pais. Atenção: eventually não significa eventualmente, mas, sim, no fim das
contas, por fim.
A alternativa B está incorreta. “Roderick não demonstrou remorso por seu comportamento” não
está de acordo com o texto. “John Roderick, a podcaster and musician, said in a statement that he
wanted to ‘acknowledge and make amends for the injuries’ caused by his posts.”
A alternativa C está correta. De acordo com Roderick, ele estava tentando dar uma lição de vida
para sua filha. “When she said she didn't know, he said he realised ‘a teaching moment just
dropped into my lap’”. / “Explaining that he wanted his daughter to learn how to open a tin of
beans, he said she tried for six hours.”
A alternativa D está incorreta. “Roderick usa ‘bean dad’ como nome artístico em seu show de
comédia stand-up” não está de acordo com o texto. Quem o apelidou assim foram usuários do
Twitter. “Twitter users, who nicknamed him ‘bean dad’, condemned the reported incident,
suggesting it was poor parenting.”
A alternativa E está incorreta. “A filha de Roderick não tentou por muito tempo abrir a lata.” não
está de acordo com o texto. Ela tentou por 6 horas. “Explaining that he wanted his daughter to
learn how to open a tin of beans, he said she tried for six hours.”
GABARITO: C

Questão 08 (EsPCEx/INÉDITA) – Which one from the underlined verbs in the text conveys a
different verb form?
(A) caused
(B) nicknamed
(C) framed
(D) posted
(E) described
Comentários: Atenção: apesar de todos as formas verbais destacadas no texto terminarem em -
ed, uma é diferente das demais. Há quatro particípios (past participles) e um passado (past), que
por ser o diferente, será a nossa resposta. Vamos analisar cada caso:

AULA 03 – ARTICLES AND NOUNS 101


TEACHER ANDREA BELO

A alternativa A está incorreta. “… he wanted to ‘acknowledge and make amends for the injuries’
caused by his posts.” → causados = particípio.
A alternativa B está correta. “Twitter users, who nicknamed him ‘bean dad’…” → apelidaram =
passado.
A alternativa C está incorreta. “… they were framed in a certain way to reflect his ‘comedic
persona”. → enquadrados, exibidos = particípio.
A alternativa D está incorreta. “In an apology posted on his website on Tuesday…” postado →
particípio.
A alternativa E está incorreta. “‘ignorant’ and ‘insensitive’ with the way he had described the
incident. descrito → particípio.
GABARITO: B

Questão 09 (EsPCEx/INÉDITA) – Choose the alternative that has the same meaning as the word
eventually in the sentence “Eventually she opened the tin and ate the beans, he explained,
claiming it was a victory for ‘good parenting.’” (paragraph 5).
(A) occasionally.
(B) in the end.
(C) in the beginning.
(D) comprehensively.
(E) after years.
Comentários: Muita atenção: conforme comentei anteriormente, eventually não é o que parece!
Não tem o sentido de eventualmente, mas, sim, de no fim das contas, por fim, especialmente
depois de muito tempo ou muito esforço, problemas etc.
A alternativa A está incorreta. Occasionally tem sentido de ocasionalmente, acidentalmente. Não
equivale ao sentido de eventually.
A alternativa B está correta. In the end significa por fim, no fim das contas e equivale ao sentido
de eventually, conforme a explicação acima. “Por fim, ela acabou abrindo a lata e comeu o feijão,
explicou ele, alegando que era uma vitória para ‘bons pais’”.
A alternativa C está incorreta. In the beginning significa no início, num primeiro momento, não
equivalendo ao sentido de eventually.
A alternativa D está incorreta. Comprehensively significa de forma abrangente, completa.
A alternativa E está incorreta. After years significa após anos. A pobre garotinha não abriu a lata
após anos, mas, sim, após algumas horas.
GABARITO: B

AULA 03 – ARTICLES AND NOUNS 102


TEACHER ANDREA BELO

Leia o texto a seguir e responda à questão 10.


New COVID Vaccines Need Absurd Amounts of Material and Labor
The first doses of mRNA COVID vaccines began arriving at hospitals in the U.S. and several other
countries in December. An overarching question is how fast companies making them can scale up
production to meet global demand. This is the first time mRNA vaccines have been authorized for
use outside of clinical trials. They work by tricking the body’s own cells into making a viral protein
that prompts immune reactions against infection. The U.S. has granted emergency authorization
to two COVID vaccines so far—made by the pharmaceutical companies Pfizer and BioNTech and
the biotechnology firm Moderna, respectively—and both rely on mRNA.
The Trump administration reached a deal with Pfizer in late December to provide 100 million
additional doses to the U.S. by the end of July—resulting in twice the amount the government
originally ordered. Together with its partner BioNTech, Pfizer plans to produce and distribute 1.3
billion doses globally next year. And Moderna intends to produce 500 million to one billion doses,
of which 200 million have already been allocated to the U.S.
Meeting these targets will be no easy feat. Each step in the manufacturing process requires raw
materials that, before COVID, were only produced in the amounts needed for clinical research.
Such mRNA vaccines are created using much faster chemical processes than traditional vaccines
made by growing weakened viruses in chicken eggs. According to a November report by the U.S.
Government Accountability Office (GAO), much of what is needed to produce these vaccines is in
short supply.
Adapted from https://www.scientificamerican.com/article/new-covid-vaccines-need-absurd-amounts-of-material-and-labor/

Questão 10 (EsPCEx/INÉDITA) – According to the text, read the statements and choose the
correct alternative.
I – mRNA vaccines are usually used besides clinical trials.
II – mRNA vaccines creation process is different from the traditional one.
III – mRNA vaccines stimulate the body’s cells to create a viral protein.
IV – It is known that companies can increase vaccines production to a global scale.
V – Both vaccines the U.S. has authorized up to now are mRNA vaccines.
(A) I, II and III are correct.
(B) II, III and IV are correct.
(C) I, IV and V are correct.
(D) II, IV and V are correct.
(E) II, III and V are correct.

AULA 03 – ARTICLES AND NOUNS 103


TEACHER ANDREA BELO

Comentários: I – A I está incorreta. “Vacinas de mRNA são geralmente usadas além de ensaios
clínicos.” = This is the first time mRNA vaccines have been authorized for use outside of clinical
trials.
II – A II está correta. “O processo de criação de vacinas de mRNA é diferente do tradicional.” = Such
mRNA vaccines are created using much faster chemical processes than traditional vaccines made
by growing weakened viruses in chicken eggs.
III – A III está correta. “Vacinas de mRNA estimulam as células do corpo a criar uma proteína viral.”
= They work by tricking the body’s own cells into making a viral protein that prompts immune
reactions against infection.
IV – A IV está incorreta. Não se sabe ao certo se as empresas conseguirão aumentar a produção
de vacinas em escala global. = “An overarching question is how fast companies making them can
scale up production to meet global demand.”
V – A V está correta. “As duas vacinas que os EUA autorizaram até agora são vacinas de mRNA.” =
The U.S. has granted emergency authorization to two COVID vaccines so far—made by the
pharmaceutical companies Pfizer and BioNTech and the biotechnology firm Moderna,
respectively—and both rely on mRNA.
A alternativa E está correta. II, III e V estão corretas.
GABARITO: E

AULA 03 – ARTICLES AND NOUNS 104


TEACHER ANDREA BELO

CONSIDERAÇÕES FINAIS
Outra aula concluída, com artigos e substantivos. Você está chegando cada dia mais perto
da sua prova e da compreensão geral de tudo o que vai responder. O aprendizado é contínuo.
Seus estudos devem ser também.
É um passo a mais até a sua aprovação! As estruturas de frases com diferentes vocábulos
e interpretação de textos em Inglês dependem dos artigos, dos substantivos e de muito mais
conteúdos que vêm por aí, nas próximas aulas.
E, dia após dia, você se prepara, ficando confiante e seguro dos seus resultados. Vai dar
certo e você sabe disso!
Outro detalhe importante para seu sucesso nos estudos é a leitura de textos
complementares daquelas fontes que usam para elaborar as provas, tais como jornais e revistas
internacionais, com reportagens e artigos diversos para explorar seus conhecimentos. Aqui temos
as traduções como bônus e material complementar também.
Estude! Faça listas de exercícios, de palavras novas, de palavras que você acha difíceis. Até
mesmo os exercícios inéditos ou de anos anteriores aqui presentes, quando resolvidos mais de
uma vez, fica mais fácil identificar algo que antes parecia difícil.
É sua conquista de etapas e que tornará você, um candidato mais bem preparado e
confiante para realizar uma excelente prova.
É importante lembrar também do nosso Fórum de dúvidas, exclusivo do Estratégia
Militares. Será minha forma de responder, no prazo máximo de 48 horas, o que mais você precise
saber para que os conteúdos fiquem ainda mais claros em seus estudos, certo?
E, caso queira, acesse minhas redes sociais para aprender mais palavras e contar com dicas
importantes, que colaboram diretamente com seus estudos dia após dia.

AULA 03 – ARTICLES AND NOUNS 105


TEACHER ANDREA BELO

REFERÊNCIAS BIBLIOGRÁFICAS
BARRETO, Tania Pedroza; GARRIDO, Maria Line; SILVA, João Antenor de C., Inglês Instrumental.
Leitura e compreensão de textos. Salvador, Ba UFBA, 1995, p. 64.
BROWN. H. Douglas. Principles of Language Learning and Teaching. Prentice Hall International,
1988.
COMPEDELLI, Samira Yousseff. Português, Literatura, Produção de texto & Gramática – São Paulo:
Ed. Saraiva, 2002.
CORREIA, Clese Mary P. Reading Specific Purposes. Salvador/ Ba: UFBA, 1999.
COSTEIRA, Adriana Araújo de M. Reading Comprehension Skills. João Pessoa/PB: ETFP, 1998.
CRYSTAL David. Cambridge University Press 1997. The Cambridge Encyclopedia of Language.
Cambridge University Press 1997
FREEMAN. Diane Larsen. MURCIA. Marianne Celce. The Grammar Book, 1999.
DYE, Joan., FRANFORT, Nancy. Spectrum II, III A Communicative Course in English. USA, Prentice
Hall, 1994.
FAVERO, Maria de Lourdes Albuquerque (org.). Dicionário de educadores no Brasil: da colônia aos
dias atuais. Rio de Janeiro: UFRJ, MEC, INEP, 1999.
FRANKPORT, Nancy & Dye Hoab. Spectrum II, III Prentice Hall Regents Englewood Cliffs, New Jersy,
1994.
GADELHA, Isabel Maria B. Inglês Instrumental: Leitura, Conscientização e Prática. Teresina:
EDUFFI, 2000.
GUANDALINI, Eiter Otávio. Técnicas de Leitura em Inglês: ESP – English For Specific Purposes:
estagio 1. São Paulo: Texto novo, 2002.
GRELLET, Françoise. Developing Reading Skills. Cambridge University Press, 1995
HOLAENDER, Arnon & Sanders Sidney. A complete English Course. São Paulo. Ed. Moderna, 1995.
HUTCHINSON, Tom & WATERS, Alan. English for Specific Purposes. Cambridge: Cambridge
University Press, 1996
KRASHEN. Stephen D. Second Language Acquisition and Second Language Learning, Prentice-Hall
International, 1988.
LAENG, Mauro. Dicionário de pedagogia. Lisboa: Dom Quixote, 1973.
LEFFA, Vilson J. Metodologia do ensino de línguas. In: BOHN, H.; VANDRESEN, P. (org.). Tópicos de
linguística aplicada: o ensino de línguas estrangeiras. Florianópolis: Editora da UFSC, 1988. p. 211-
231.
LIBERATO, Wilson. Compact English Book Inglês Ensino Médio. São Paulo: FTD, Vol. Único, 1998
Mc ARTHUR. The Oxford Companion to the English Language. Oxford University Press 1992
Fromkin. Victoria. An Introduction to Language
MARQUES, Amadeu. Inglês Série Brasil. ed. Atica. São Paulo: 2004. Vol. Único.

AULA 03 – ARTICLES AND NOUNS 106


TEACHER ANDREA BELO

MURPHY, Raymond: Essencial Grammar in Use Oxford. New York Ed. Oxford University, 1997.
OLIVEIRA, Luciano Amaral. English For Tourism Students. Inglês para Estudantes de Turismo: São
Paulo, Rocca, 2001.
OLIVEIRA, Sara Rejane de F. Estratégias de leitura para Inglês Instrumental. Brasília: UNB, 1994.
QUINTANA, et alli. First Certificate. Master Class Oxford. New York, 2004: Ed. Oxford University.
PAULINO, Berenice F. et all. Leitura em textos em Inglês – Uma Abordagem Instrumental. Belo
Horizonte: Ed. Dos Autores, 1992.
PEREIRA, Edilberto Coelho. Inglês Instrumental. Teresina: ETFPI, 1998.
RODGES, Theodore. Jack C. Richards. Approaches and Methods in Language Teaching. Cambridge
University Press, 2001.
RODMAN Robert. Harcourt Brace 1993. English as a Global Language
STEWART, B., HAINES S. First Certificate, MasterClass. UK – Oxford 2004.
SILVA, João Antenor de C., GARRIDO, Maria Lina, BARRETO, Tânia Pedrosa. Inglês Instrumental:
Leitura e Compreensão de Textos. Salvador: Centro Editorial e Didático, UFBA. 1994
SOARES, Moacir Bretãs. Dicionário de legislação do ensino. 19.ed. Rio de Janeiro: FGV, 1981.
SOUZA, Adriana Srade F. Leitura em Língua Inglesa: Uma abordagem Instrumental. São Paulo:
Disal, 2005.
TUCK, Michael. Oxford Dictionary of Computing for Learners of English. Oxford: Oxford University
Press, 1996.
TOTIS, Verônica Pakrauskas. Língua Inglesa: leitura. São Paulo: Cortez, 1991.

Livros eletrônicos:
Dicionário Houaiss da Língua Portuguesa, Editora Objetiva, 2001.
MOURãO, Janaína Pereira. "Skimming x Scanning"; Brasil Escola. Disponível em
<https://brasilescola.uol.com.br/ingles/skimming-x-scanning.htm>. Acesso em 20 de março de
2019.
www.newsweek.com - Acesso em 18 de março de 2019.
http://www.galaor.com.br/tecnicas-de-leitura/ - Acesso em 19 de março de 2019.
Expressões Idiomáticas (continuação)" em Só Língua Inglesa. Virtuous Tecnologia da
Informação,2008-2019. Consultado em 03/04/2019 às 22:09. Disponível na Internet em
http://www.solinguainglesa.com.br/conteudo/Expressoes5.php

AULA 03 – ARTICLES AND NOUNS 107


TEACHER ANDREA BELO

TRADUÇÕES
The end of life on Earth?
It weighed about 10,000 tons, entered the atmosphere at a speed of 64,000km/h and exploded
over a city with a blast of 500 kilotons. But on 15 February 2013, we were lucky. The meteorite
that showered pieces of rock over Chelyabinsk, Russia, was relatively small, at only about 17
metres wide. Although many people were injured by falling glass, the damage was nothing
compared to what had happened in Siberia nearly one hundred years ago. Another relatively small
object (approximately 50 metres in diameter) exploded in mid-air over a forest region, flattening
about 80 million trees. If it had exploded over a city such as Moscow or London, millions of people
would have been killed.
By a strange coincidence, the same day that the meteorite terrified the people of Chelyabinsk,
another 50m-wide asteroid passed relatively close to Earth. Scientists were expecting that visit
and know that the asteroid will return to fly close by us in 2046, but the Russian meteorite earlier
in the day had been too small for anyone to spot.
Most scientists agree that comets and asteroids pose the biggest natural threat to human
existence. It was probably a large asteroid or comet colliding with Earth which wiped out the
dinosaurs about 65 million years ago. An enormous object, 10 to 16km in diameter, struck the
Yucatan region of Mexico with the force of 100 megatons. That is the equivalent of one Hiroshima
bomb for every person alive on Earth today.
Many scientists, including the late Stephen Hawking, say that any comet or asteroid greater than
20km in diameter that hits Earth will result in the complete destruction of complex life, including
all animals and most plants. As we have seen, even a much smaller asteroid can cause great
damage.
The Earth has been kept fairly safe for the last 65 million years by good fortune and the massive
gravitational field of the planet Jupiter. Our cosmic guardian, with its stable circular orbit far from
the sun, sweeps up and scatters away most of the dangerous comets and asteroids which might
cross Earth’s orbit. After the Chelyabinsk meteorite, scientists are now monitoring potential
hazards even more carefully but, as far as they know, there is no danger in the foreseeable future.
Types of space rocks
• Comet – a ball of rock and ice that sends out a tail of gas and dust behind it. Bright comets
only appear in our visible night sky about once every ten years.
• Asteroid – a rock a few feet to several kms in diameter. Unlike comets, asteroids have no
tail. Most are too small to cause any damage and burn up in the atmosphere. They appear
to us as ‘shooting stars’.
• Meteoroid – part of an asteroid or comet.
• Meteorite – what a meteoroid is called when it hits Earth.
Taken from: https://learnenglishteens.britishcouncil.org/skills/reading/upper-intermediate-b2-reading/end-life-earth

AULA 03 – ARTICLES AND NOUNS 108


TEACHER ANDREA BELO

O fim da vida na Terra?


Pesava cerca de 10.000 toneladas, entrou na atmosfera a uma velocidade de 64.000 km / he
explodiu sobre uma cidade com uma explosão de 500 quilotons. Mas em 15 de fevereiro de 2013,
tivemos sorte. O meteorito que derramou pedaços de rocha sobre Chelyabinsk, Rússia, era
relativamente pequeno, com apenas cerca de 17 metros de largura. Embora muitas pessoas
tenham se ferido pela queda de vidro, o dano não foi nada comparado ao que aconteceu na Sibéria
quase cem anos atrás. Outro objeto relativamente pequeno (aproximadamente 50 metros de
diâmetro) explodiu no ar sobre uma região de floresta, achatando cerca de 80 milhões de árvores.
Se tivesse explodido sobre uma cidade como Moscou ou Londres, milhões de pessoas teriam sido
mortas.
Por uma estranha coincidência, no mesmo dia em que o meteorito aterrorizou o povo de
Chelyabinsk, outro asteróide de 50 m de largura passou relativamente perto da Terra. Os cientistas
estavam esperando essa visita e sabem que o asteróide voltará a voar perto de nós em 2046, mas
o meteorito russo no início do dia era muito pequeno para que alguém pudesse localizar.
A maioria dos cientistas concorda que os cometas e asteróides representam a maior ameaça
natural à existência humana. Provavelmente foi um grande asteróide ou cometa colidindo com a
Terra que exterminou os dinossauros há cerca de 65 milhões de anos. Um objeto enorme, de 10
a 16 km de diâmetro, atingiu a região de Yucatan, no México, com a força de 100 megatons. Isso
é o equivalente a uma bomba de Hiroshima para cada pessoa viva na Terra hoje.
Muitos cientistas, incluindo o falecido Stephen Hawking, dizem que qualquer cometa ou asteróide
com mais de 20 km de diâmetro que atingir a Terra resultará na destruição completa de vidas
complexas, incluindo todos os animais e a maioria das plantas. Como vimos, mesmo um asteróide
muito menor pode causar grandes danos.
A Terra foi mantida razoavelmente segura nos últimos 65 milhões de anos por boa sorte e pelo
enorme campo gravitacional do planeta Júpiter. Nosso guardião cósmico, com sua órbita circular
estável longe do sol, varre e espalha a maioria dos cometas e asteróides perigosos que podem
cruzar a órbita da Terra. Depois do meteorito de Chelyabinsk, os cientistas agora monitoram os
riscos potenciais com ainda mais cuidado, mas, pelo que sabem, não há perigo no futuro
previsível.
Tipos de rochas espaciais
• Cometa - uma bola de rocha e gelo que envia uma cauda de gás e poeira por trás dela.
Cometas brilhantes só aparecem em nosso céu noturno visível uma vez a cada dez anos.
• Asteróide - uma rocha de alguns pés a vários quilômetros de diâmetro. Ao contrário dos
cometas, os asteróides não têm cauda. A maioria é muito pequena para causar qualquer
dano e queimar na atmosfera. Eles aparecem para nós como ‘estrelas cadentes’.
• Meteoróide - parte de um asteróide ou cometa.
• Meteorito - como um meteoróide é chamado quando atinge a Terra.

AULA 03 – ARTICLES AND NOUNS 109


TEACHER ANDREA BELO

Facebook complies with Brazilian judge’s order to block 12 accounts accused of running a fake news network
Facebook announced Saturday it has obeyed a Brazilian judge’s order for a worldwide block on
the accounts of 12 of President Jair Bolsonaro’s supporters who are under investigation for
allegedly running a fake news network.
Supreme Court Justice Alexandre de Moraes said Friday night that the company had failed to fully
comply with a previous ruling ordering the accounts to be shut down, saying they were still online
and publishing by changing their registration to locations outside Brazil.
Facebook issued a statement saying it complied due to the threat of criminal liability for an
employee in Brazil.
But it called the new order “extreme,” saying it poses a “threat to freedom of expression outside
of Brazil’s jurisdiction and conflicting with laws and jurisdictions worldwide.” The company said it
would appeal to the full court.
Facebook also argued it had complied with the previous order by “restricting the ability for the
target Pages and Profiles to be seen from IP locations in Brazil”.
“People from IP locations in Brazil were not capable of seeing these Pages and Profiles even if the
targets had changed their IP location”, the company said.
Moraes said that Facebook ought to pay $ 367,000 in penalties for not complying with his previous
decision during the last eight days.
He also had ruled Twitter should block the accounts. While Twitter said then the decision was
“disproportionated” under Brazil’s freedom of speech rules and that it would appeal, the targeted
profiles were disabled.
Moraes is overseeing a controversial investigation to determine whether some of Bolsonaro’s
most ardent allies are running a social media network aimed at spreading threats and fake news
against Supreme Court justices.
The probe is one of the main points of confrontation between Bolsonaro and the Supreme Court.
The president himself filed a lawsuit last week demanding the accounts to be unblocked.
(Adapted from https://time.com/5874695/facebook-blocks-accounts-worldwide/)

O Facebook cumpre a ordem do juiz brasileiro para bloquear 12 contas acusadas de


administrar uma falsa rede de notícias
O Facebook anunciou no sábado que obedeceu à ordem de um juiz brasileiro para um bloqueio
mundial das contas de 12 partidários do presidente Jair Bolsonaro que estão sob investigação por
supostamente comandarem uma rede de notícias falsa.
O juiz da Suprema Corte, Alexandre de Moraes, disse na sexta-feira à noite que a empresa não
cumpriu integralmente uma decisão anterior que ordenava o encerramento das contas, alegando
que ainda estavam online e publicando por meio da alteração do registro para locais fora do Brasil.
O Facebook emitiu um comunicado dizendo que cumpriu devido à ameaça de responsabilidade
criminal para um funcionário no Brasil.

AULA 03 – ARTICLES AND NOUNS 110


TEACHER ANDREA BELO

Mas chamou a nova ordem de "extrema", dizendo que ela representa uma "ameaça à liberdade
de expressão fora da jurisdição do Brasil e em conflito com as leis e jurisdições em todo o mundo".
A empresa disse que apelará para o tribunal pleno.
O Facebook também argumentou que cumpriu a ordem anterior ao “restringir a capacidade das
páginas e perfis de destino serem vistos de locais de IP no Brasil”.
“Pessoas de localizações de IP no Brasil não eram capazes de ver essas páginas e perfis mesmo
que os alvos tivessem mudado sua localização de IP”, disse a empresa.
Moraes disse que o Facebook deveria pagar US $ 367.000 em penalidades por não cumprir sua
decisão anterior nos últimos oito dias.
Ele também determinou que o Twitter deveria bloquear as contas. Embora o Twitter tenha dito
então que a decisão foi “desproporcional” sob as regras de liberdade de expressão do Brasil e que
apelaria, os perfis direcionados foram desativados.
Moraes está supervisionando uma investigação polêmica para determinar se alguns dos aliados
mais ardentes de Bolsonaro estão administrando uma rede de mídia social destinada a espalhar
ameaças e notícias falsas contra os juízes da Suprema Corte.
A investigação é um dos principais pontos de confronto entre Bolsonaro e o Supremo Tribunal
Federal.
O próprio presidente entrou com uma ação na semana passada exigindo o desbloqueio das
contas.

Being famous
Sandra Rosa is very beautiful, young, and successful. She's a famous actress. She's also very rich.
Her house near the beach is big and beautiful, and her car is very expensive. Her fans love her.
But is she happy?
Sandra says, "yeah, I'm young, rich, beautiful, and famous. People think rich people are happy.
That's not always true!"
Sandra's brother, Mike, is her manager. He says, "Sandra is only 18. She enjoys acting and
entertaining people. But she's not happy. She doesn't like being famous."
"It's true," Sandra says. "I'm never alone. Reporters are everywhere. Wherever I go, they're there.
They're outside my house all the time! That's so annoying!”
Adapted from: https://www.inglesnapontadalingua.com.br/2008/10/texto-de-ingls-alunos-de-nvel-bsico.html

Sendo famoso
Sandra Rosa é muito bonita, jovem e bem-sucedida. Ela é uma atriz famosa. Ela também é muito
rica. A casa dela perto da praia é grande e bonita e o carro dela é muito caro. Seus fãs amá-la.
Mas ela está feliz?

AULA 03 – ARTICLES AND NOUNS 111


TEACHER ANDREA BELO

Sandra diz: "Sim, sou jovem, rica, bonita e famosa. As pessoas acham que os ricos são felizes. Isso
nem sempre é verdade!"
O irmão de Sandra, Mike, é o empresário dela. Ele diz: "Sandra tem apenas 18 anos. Ela gosta de
atuar e entreter as pessoas. Mas ela não está feliz. Ela não gosta de ser famosa."
"É verdade", diz Sandra. "Nunca estou sozinho. Os repórteres estão por toda parte. Onde quer
que eu vá, eles estão lá. Eles estão fora da minha casa o tempo todo! Isso é tão chato!"

Homeless crack addict revitalizes small square in downtown São Paulo


A homeless man has chosen to occupy his free time revitalizing a small square on the corner of
avenues São João and Duque de Caxias, in downtown São Paulo. He planted pau-brasil, palm,
banana and avocado trees. He also planted boldo, sweet potatoes, beans, peppers and
ornamental plants, such as snake plants. Residents noticed the square’s gradual changes and
congratulated the author for the modifications.
Fonte: Folha de São Paulo Internacional – 21/03/2017

Morador de rua viciado em crack revitaliza pequena praça no centro de São Paulo
Um morador de rua optou por ocupar seu tempo livre revitalizando uma pequena praça na
esquina das avenidas São João e Duque de Caxias, no centro de São Paulo. Ele plantou pau-brasil,
palmeira, banana e abacate. Ele também plantou boldo, batata-doce, feijão, pimentão e plantas
ornamentais, como plantas de cobra. Os moradores perceberam as mudanças graduais da praça
e parabenizaram o autor pelas modificações.

Why some people like wearing masks


Some people welcome face coverings for reasons ranging from the convenient and expedient
to the more complex and psychological. But is this a helpful coping mechanism?
Sheltering in place hasn’t been too hard for Jay Lee; watching a film at home and ordering a
takeaway has always been his idea of a good night. Lee, a 32-year-old small business owner in
Leicester, identifies as an introvert. And although 2020 had its hardships – in the spring, he was
made redundant from his job at a large bank – one perk for him has been the widespread adoption
of face masks.
Lee has always dreaded run-ins with old friends and acquaintances around town, finding these
spontaneous interactions “extremely awkward”. He used to time his shopping trips to minimise
the possibility of bumping into someone he knew, waiting until almost closing time before
heading out. “Since I've been wearing the mask, my awkward interactions with friends and family
have significantly reduced,” he says. Now, he goes to the shops whenever he wants, without
worrying about whom he might see. He hopes that, even after the pandemic ends, it will still be
socially acceptable to wear a mask.

AULA 03 – ARTICLES AND NOUNS 112


TEACHER ANDREA BELO

Wearing a mask is, for most of us, an annoying but worthwhile sacrifice: it’s one of the most
effective ways to slow the spread of Covid-19. Still, most of us look forward to the day when we
can bare our faces in public again. Face-coverings fog our glasses and clog our pores; they make
it harder to smile at strangers and recognise friends.
Yet some are secretly relishing the new mask-wearing mandates, for reasons ranging from the
convenient and expedient to the more complex and psychological. Some welcome the way face
coverings reduce or change interactions that might otherwise spark social anxiety.
‘Anonymity carries power’
At the lighter end of the scale, some people have found that masking offers a welcome relief from
the pressures to uphold strict standards of grooming and appearance. They have ditched their old
makeup and shaving routines and are saving money, time and stress. Others have discovered that
hiding their mouths affords them unexpected freedoms. Some restaurant servers and retail
workers say they no longer feel obliged to fake-smile at customers, potentially lifting the burden
of emotional labour.
(Adapted from https://www.bbc.com/worklife/article/20210115-why-some-people-like-wearing-masks)

Por que algumas pessoas gostam de usar máscaras


Algumas pessoas gostam de coberturas faciais por razões que vão desde as convenientes e
convenientes até as mais complexas e psicológicas. Mas este é um mecanismo de enfrentamento
útil?
O abrigo no local não foi muito difícil para Jay Lee; assistir a um filme em casa e pedir comida para
viagem sempre foi sua ideia de uma boa noite. Lee, um pequeno empresário de 32 anos em
Leicester, se identifica como introvertido. E embora 2020 tenha tido suas dificuldades - na
primavera, ele foi despedido de seu emprego em um grande banco - uma vantagem para ele foi
a adoção generalizada de máscaras faciais.
Lee sempre temeu desentendimentos com velhos amigos e conhecidos pela cidade, achando
essas interações espontâneas “extremamente estranhas”. Ele costumava cronometrar suas idas
às compras para minimizar a possibilidade de esbarrar com alguém que conhecia, esperando
quase até a hora de fechar antes de sair. “Desde que estou usando a máscara, minhas interações
estranhas com amigos e familiares diminuíram significativamente”, diz ele. Agora, ele vai às lojas
quando quer, sem se preocupar com quem vai encontrar. Ele espera que, mesmo após o fim da
pandemia, ainda seja socialmente aceitável usar uma máscara.
Usar uma máscara é, para a maioria de nós, um sacrifício chato, mas vale a pena: é uma das
maneiras mais eficazes de retardar a disseminação de Covid-19. Ainda assim, a maioria de nós
espera o dia em que possamos mostrar nossos rostos em público novamente. As coberturas
faciais embaçam nossos óculos e obstruem nossos poros; eles tornam mais difícil sorrir para
estranhos e reconhecer amigos.
No entanto, alguns estão secretamente saboreando as novas regras de uso de máscaras, por
razões que vão desde as convenientes e convenientes até as mais complexas e psicológicas.
Alguns acolhem bem a maneira como as coberturas faciais reduzem ou mudam as interações que,
de outra forma, poderiam despertar a ansiedade social.

AULA 03 – ARTICLES AND NOUNS 113


TEACHER ANDREA BELO

‘Anonimato traz poder’


Na extremidade mais leve da escala, algumas pessoas descobriram que o mascaramento oferece
um alívio bem-vindo das pressões para manter padrões estritos de aparência e aparência. Eles
abandonaram suas velhas rotinas de maquiagem e barbear e estão economizando dinheiro,
tempo e estresse. Outros descobriram que esconder a boca lhes proporciona liberdades
inesperadas. Alguns atendentes de restaurantes e varejistas dizem que não se sentem mais
obrigados a sorrir falsamente para os clientes, o que pode aliviar o fardo do trabalho emocional.

The World's Biggest Problems Are Interconnected. Here's How We Can Solve Them This Decade
Two decades ago, people around the world rang in the new millennium with a growing sense of
optimism. The threat posed by the Cold War was fading slowly in the rearview mirror. Leading
thinkers like Francis Fukuyama touted the benefits of globalization, saying it would bring
democracy and prosperity to the developing world. The nascent Internet economy promised to
bring us closer together.
The following 20 years took some of the air out of the assumption of steady progress, but when
future historians assess the 21st century, the year 2020 is likely to serve as the point at which the
optimism bubble burst. The COVID-19 pandemic has exposed a complex web of interlocking
problems that have morphed into full-blown crises. The coronavirus laid bare the dangers of
endemic poverty not only in the developing world but also in rich countries like the U.S., where
millions lack health care and are one paycheck away from living on the street. Around the world,
racial and ethnic minorities have demanded justice after centuries of structural discrimination.
Woven through it all, the earth’s climate is increasingly unstable, posing an existential threat to
human society as we know it. In the next decade, societies will be forced to either confront this
snarl of challenges, or be overwhelmed by them. Our response will define the future for decades
to come.
The recognition that these challenges are fundamentally linked isn’t new. Activists and academics
have for many years pointed to the cascading effects of various social ills. Whether it’s the way
racism contributes to poor health outcomes or gender discrimination harms economic growth,
the examples are seemingly endless. But this understanding has made its way into the
conversation about solutions too.
Notably, for the past five years, the U.N. has touted 17 interrelated sustainable development
goals, objectives for building a more viable world, and called for a push to achieve them by 2030.
The goals, which cover environmental, social and economic progress, are nonbinding but have
become key benchmarks for commitments at a national and corporate level. Countries from China
to the Maldives, as well as companies like Amazon, Microsoft and PwC, have committed to rolling
out policies over the next decade that will set them on a path to eliminate their carbon footprints.
The understanding that these problems require holistic solutions has only grown amid the
pandemic and its fallout. President Joe Biden has referred to four urgent crises—the pandemic,
the economic crisis, racial injustice and climate change—and promised a push to tackle them all
together. The European Union’s program to propel the bloc out of the COVID-19 crisis targets
climate change, while incorporating equity concerns. As stock markets soared last year,

AULA 03 – ARTICLES AND NOUNS 114


TEACHER ANDREA BELO

institutions with trillions of dollars in assets demanded that their investments deliver not only a
good return for their wallets but also a good return for society.
(Adapted from https://time.com/5931603/how-to-solve-worlds-biggest-problems/)

Os maiores problemas do mundo estão interligados. Veja como podemos resolvê-los nesta década
Duas décadas atrás, as pessoas em todo o mundo chegavam ao novo milênio com um crescente
senso de otimismo. A ameaça representada pela Guerra Fria estava desaparecendo lentamente
no espelho retrovisor. Pensadores importantes como Francis Fukuyama elogiaram os benefícios
da globalização, dizendo que ela traria democracia e prosperidade ao mundo em
desenvolvimento. A economia nascente da Internet prometia nos aproximar.
Os 20 anos seguintes tiraram um pouco do ar da suposição de progresso constante, mas quando
os futuros historiadores avaliarem o século 21, o ano de 2020 provavelmente servirá como o
ponto em que a bolha de otimismo estourou. A pandemia COVID-19 expôs uma complexa teia de
problemas interligados que se transformaram em crises completas. O coronavírus revelou os
perigos da pobreza endêmica não apenas no mundo em desenvolvimento, mas também em
países ricos como os EUA, onde milhões não têm assistência médica e estão a um salário de viver
nas ruas. Em todo o mundo, as minorias raciais e étnicas exigem justiça após séculos de
discriminação estrutural. Tecido por tudo isso, o clima da Terra está cada vez mais instável,
representando uma ameaça existencial para a sociedade humana como a conhecemos. Na
próxima década, as sociedades serão forçadas a enfrentar esse emaranhado de desafios ou ser
oprimidas por eles. Nossa resposta definirá o futuro nas próximas décadas.
O reconhecimento de que esses desafios estão fundamentalmente ligados não é novo. Por muitos
anos, ativistas e acadêmicos apontaram os efeitos em cascata de vários males sociais. Quer seja
a forma como o racismo contribui para resultados ruins de saúde ou a discriminação de gênero
prejudica o crescimento econômico, os exemplos são aparentemente infinitos. Mas esse
entendimento também entrou na conversa sobre soluções.
Notavelmente, nos últimos cinco anos, a ONU apresentou 17 metas de desenvolvimento
sustentável inter-relacionadas, objetivos para a construção de um mundo mais viável, e pediu um
impulso para alcançá-los até 2030. As metas, que abrangem o progresso ambiental, social e
econômico, são não vinculantes, mas se tornaram referências-chave para compromissos em nível
nacional e corporativo. Países da China às Maldivas, bem como empresas como Amazon,
Microsoft e PwC, se comprometeram a implementar políticas na próxima década que os colocarão
no caminho para eliminar suas pegadas de carbono.
A compreensão de que esses problemas requerem soluções holísticas apenas cresceu em meio à
pandemia e suas consequências. O presidente Joe Biden referiu-se a quatro crises urgentes - a
pandemia, a crise econômica, a injustiça racial e a mudança climática - e prometeu um esforço
para enfrentá-las todas juntas. O programa da União Europeia para tirar o bloco da crise COVID-
19 tem como alvo as mudanças climáticas, ao mesmo tempo que incorpora questões de equidade.
Enquanto os mercados de ações disparavam no ano passado, as instituições com trilhões de
dólares em ativos exigiam que seus investimentos proporcionassem não apenas um bom retorno
para suas carteiras, mas também um bom retorno para a sociedade.

AULA 03 – ARTICLES AND NOUNS 115


TEACHER ANDREA BELO

The U.S. has tragically surpassed 400,000 COVID-19 deaths, and case numbers and
hospitalizations are likewise spiking to record levels around the world. __________ vaccines now
rolling out, there is reason to hope that there is an end in sight. __________, by most estimates,
widespread vaccinations will not be in place until the middle of the year at the earliest.
__________, we have some ways to go yet with social distancing, mask wearing and other
pandemic mitigation behaviors.

Os EUA ultrapassaram tragicamente 400.000 mortes por COVID-19, e o número de casos e


hospitalizações também estão atingindo níveis recordes em todo o mundo. __________ vacinas
agora sendo lançadas, há motivos para esperar que haja um fim à vista. __________, pela maioria
das estimativas, as vacinações generalizadas não estarão em vigor até meados do ano, no mínimo.
__________, ainda temos alguns caminhos a percorrer com o distanciamento social, o uso de
máscaras e outros comportamentos de mitigação de pandemia.

2020: The year in seven fake news stories


How should we deal with fake news? Many western governments are moving towards stricter
regulation of tech giants. But others believe that only a more educated public can stop the blight.
Have you heard that the Covid-19 vaccine will rewrite your DNA? If not, don’t worry. It is fake
news. What is true, however, is that debate about misinformation online reached a fever pitch
this year.
From politicians calling for fake news to be banned, to others suggesting they should “weaponise”
it, the public’s ability to tell fact from fiction is increasingly contested. Here are seven stories that
show how fake news shaped this year.
First casualty. In January, Iran launched missiles at Iraqi airbases hosting US troops in retaliation
for the American assassination of general Qasem Soleimani. Many feared war. Iranian state TV
announced that 80 “American terrorists” had been killed. The USA said it experienced no
casualties. Some argued that “fake news” allowed both sides to save face and avoid conflict.
Covid-5G. By far the biggest subject for fake news this year was Covid-19. The stories were
numerous and varied, but one popular one was that Covid-19 was caused by 5G radiation. In
Britain, 5G masts were attacked, and blame was often pointed at Bill Gates in what became an
increasingly Byzantine conspiracy theory.
Dolphins of Venice. When Coronavirus lockdowns began, social media was abuzz with animal
news. Nature was healing. Venice, no longer clogged with tourists, now had clean canals down
which Dolphins cruised. Sadly, the heartening video was actually from a different region of Italy.
Viral veganism. In India, a false story that that no-one who eschewed all animal products had
contracted Covid-19 went viral. While this might not seem too harmful, the resulting drop in sales
of meat and eggs hit farmers and butchers hard. Some saw the story as part of the promotion of
Hindutva in India.
Constantine cancelled. The death of George Floyd at the hands of US police, kicked off a summer
of protest against racism. Iconoclasts tore down statues of historical figures they saw as racist.

AULA 03 – ARTICLES AND NOUNS 116


TEACHER ANDREA BELO

Rumours abounded about who was next to fall. Perhaps the oddest story was that York Minster
planned to remove a statue of the Roman emperor Constantine because of his support for slavery.
The Church of England was forced to state that they had no such plans.
Rigged election. A daring raid on a CIA server in Germany revealed the plot to rig the US election
against Donald Trump… or so the fake story went. Its popularity is evidence of a stark epistemic
divide among Americans. It is just one of many casting President Trump’s loss this November as
the result of fraud. The president has yet to concede officially, but some of his statements have
been censored by social media companies, already treating him as yesterday’s fake news.
Trained Triceratops. A video of a dinosaur being unloaded from a truck went viral this month in
Indonesia. Many shared it in the belief that it was real. It turned out to be a robot filmed to
promote a theme park. While some fake news stories are all about lies and Russian spies, others
are simply about people seeing what they want to see.
How should we deal with fake news?
Reality check. Regulate! say some. Social media companies should be responsible for information
shared on their platforms. If they do not take action to stop lies being spread, then we __________
ever more anger and division. Algorithms showing people what they want to read will combine
with fake news to reinforce false beliefs. The lack of an objective standard of truth online could
become dangerous.
Educate! say others. There have always been lies, mistakes and misrepresentations in the media.
Allowing either the government or large companies to decide what is true or false is a recipe for
disaster. What is to stop them from censoring their own critics? What is needed is the ability to
discern reliable sources for oneself. And the best way to do that is to teach people about fake
news.
(Adapted from https://theday.co.uk/stories/2020-the-year-in-seven-fake-news-stories/Acesso em: 14 de jan 2021)

2020: o ano em sete notícias falsas


Como devemos lidar com notícias falsas? Muitos governos ocidentais estão adotando uma
regulamentação mais rígida dos gigantes da tecnologia. Mas outros acreditam que apenas um
público mais educado pode deter a praga. Você já ouviu falar que a vacina Covid-19 irá reescrever
seu DNA? Se não, não se preocupe. São notícias falsas. O que é verdade, no entanto, é que o
debate sobre a desinformação online atingiu um pico febril este ano.
De políticos que pedem o banimento de notícias falsas a outros que sugerem que eles deveriam
"transformá-las" em armas, a capacidade do público de distinguir fatos de ficção é cada vez mais
contestada. Aqui estão sete histórias que mostram como as notícias falsas influenciaram este ano.
Primeira vítima. Em janeiro, o Irã lançou mísseis contra bases aéreas iraquianas que hospedavam
tropas americanas em retaliação ao assassinato do general Qasem Soleimani. Muitos temiam a
guerra. A TV estatal iraniana anunciou que 80 “terroristas americanos” foram mortos. Os EUA
disseram que não houve vítimas. Alguns argumentaram que as “notícias falsas” permitiam que
ambos os lados salvassem a face e evitassem conflitos.

AULA 03 – ARTICLES AND NOUNS 117


TEACHER ANDREA BELO

Covid-5G. De longe, o maior assunto para notícias falsas neste ano foi a Covid-19. As histórias
eram numerosas e variadas, mas uma popular era que Covid-19 era causado por radiação 5G. Na
Grã-Bretanha, mastros 5G foram atacados e a culpa foi freqüentemente apontada para Bill Gates
no que se tornou uma teoria da conspiração cada vez mais bizantina.
Golfinhos de Veneza. Quando os bloqueios por Coronavirus começaram, as redes sociais estavam
alvoroçadas com notícias sobre animais. A natureza estava curando. Veneza, não mais
congestionada de turistas, agora tinha canais limpos pelos quais os golfinhos navegavam.
Infelizmente, o vídeo animador era, na verdade, de uma região diferente da Itália.
Veganismo viral. Na Índia, uma falsa história de que ninguém que evitava todos os produtos
animais havia contraído Covid-19 se tornou viral. Embora isso possa não parecer muito prejudicial,
a queda resultante nas vendas de carne e ovos atingiu duramente os agricultores e açougueiros.
Alguns viram a história como parte da promoção do Hindutva na Índia.
Constantine cancelou. A morte de George Floyd nas mãos da polícia dos EUA, deu início a um
verão de protesto contra o racismo. Os iconoclastas derrubaram estátuas de figuras históricas que
consideraram racistas. Rumores abundavam sobre quem seria o próximo a cair. Talvez a história
mais estranha seja que York Minster planejou remover uma estátua do imperador romano
Constantino por causa de seu apoio à escravidão. A Igreja da Inglaterra foi forçada a declarar que
não tinha tais planos.
Eleição forjada. Uma invasão ousada em um servidor da CIA na Alemanha revelou o complô para
fraudar a eleição americana contra Donald Trump ... ou assim dizia a história falsa. Sua
popularidade é evidência de uma grande divisão epistêmica entre os americanos. É apenas uma
das muitas perdas do presidente Trump em novembro como resultado de fraude. O presidente
ainda não cedeu oficialmente, mas algumas de suas declarações foram censuradas por empresas
de mídia social, que já o trataram como uma notícia falsa de ontem.
Triceratops treinados. Um vídeo de um dinossauro sendo descarregado de um caminhão se
tornou viral este mês na Indonésia. Muitos compartilharam acreditando que era real. Era um robô
filmado para promover um parque temático. Enquanto algumas notícias falsas são todas sobre
mentiras e espiões russos, outras são simplesmente sobre pessoas vendo o que querem ver.
Como devemos lidar com notícias falsas?
Verificação da realidade. Regular! diga alguns. As empresas de mídia social devem ser
responsáveis pelas informações compartilhadas em suas plataformas. Se eles não agirem para
impedir que mentiras se espalhem, então nós __________ ainda mais raiva e divisão. Algoritmos
que mostram às pessoas o que elas querem ler se combinam com notícias falsas para reforçar as
crenças falsas. A falta de um padrão objetivo de verdade online pode se tornar perigosa.
Educar! dizem outros. Sempre houve mentiras, erros e deturpações na mídia. Permitir que o
governo ou grandes empresas decidam o que é verdadeiro ou falso é uma receita para o desastre.
O que os impede de censurar seus próprios críticos? O que é necessário é a habilidade de discernir
fontes confiáveis para si mesmo. E a melhor maneira de fazer isso é ensinar as pessoas sobre
notícias falsas.

AULA 03 – ARTICLES AND NOUNS 118


TEACHER ANDREA BELO

RIO DE JANEIRO, BRAZIL - In the past 24 hours, Brazil recorded 1,113 deaths and 36,653 new cases
related to the novel coronavirus. The data are included in the Ministry of Health's daily balance
released on Tuesday evening, September 15th.
Since the start of the pandemic, 133,119 people have died as a result of Covid-19. On Monday,
the Ministry of Health's data system recorded a total of 132,006 deaths. A further 2,445 are still
under investigation by health authorities.
(Adapted from https://riotimesonline.com – September 15th)

RIO DE JANEIRO, BRASIL – Nas últimas 24 horas, o Brasil registrou 1.113 mortes e 36.653 novos
casos relacionados ao novo coronavírus. Os dados constam do balanço diário do Ministério da
Saúde, divulgado na noite de terça-feira, 15 de setembro.
Desde o início da pandemia, 133.119 pessoas morreram como resultado da Covid-19. Na segunda-
feira, o sistema de dados do Ministério da Saúde registrou um total de 132.006 mortes. Outros
2.445 ainda estão sob investigação das autoridades sanitárias.

Covid babies don't have to be the unlucky generation. But they must be helped
A year into this pandemic, there are babies now learning to walk and talk who have never known
anything but life under the shadow of Covid, and preschoolers who can barely remember a world
before it. Doctors' children have had to learn not to touch Mummy when she gets in from work,
until she's had a shower to wash off any last possible trace of danger. Thanks to popular toddler
demand, you can now buy masks for dolls.
It's perfectly normal for kids to reflect what's happening around them by playing quarantine with
stuffed animals, but normal too for adults to wonder uncomfortably whether all this leaves a
lasting mark. How much will it matter in years to come that, as the minister for loneliness Diana
Barran recently put it, there are toddlers being raised by shielding parents who have never had a
playdate? Will Covid babies grow up solitary creatures, used to entertaining themselves, or warier
of the strangers they so rarely meet and interact with? Taking a tiny bundle out in public used to
mean an endless succession of random older women cooing over the pram, or strangers pulling
faces to entertain a bored baby in a checkout queue. But now passersby daren't get close, and
other shoppers are hidden behind masks.
This week MPs were presented with some early findings from a project led by the First 1001 Days
Movement, an alliance of early-years charities and professionals, tracking the lives of under-twos
growing up through a pandemic. A survey of children's service providers it commissioned found
98% thought the babies and toddlers they worked with had been affected by higher parental
stress and anxiety, while 92% had seen fearful families effectively cutting themselves off from the
outside world, skipping routine appointments or not wanting to leave the house. Nine in 10 had
observed children being played with less, or being less active. Heartbreakingly, more than a
quarter said lockdown left the children they worked with more exposed to domestic conflict,
abuse or neglect.
(Adapted from https://mmm.theguardian.com/commentisfree/2021/jan/22/covid-babies-unlucky-generation-help-lockdown)

AULA 03 – ARTICLES AND NOUNS 119


TEACHER ANDREA BELO

Bebês cobiçosos não precisam ser a geração azarada. Mas eles devem ser ajudados
Um ano após o início da pandemia, há bebês aprendendo a andar e a falar que nunca conheceram
nada além da vida sob a sombra de Covid, e crianças em idade pré-escolar que mal conseguem se
lembrar de um mundo antes dela. Os filhos dos médicos tiveram que aprender a não tocar na
mamãe quando ela chega do trabalho, até que ela tome um banho para se livrar do último vestígio
de perigo. Graças à demanda popular de crianças, agora você pode comprar máscaras para
bonecas.
É perfeitamente normal que as crianças reflitam o que está acontecendo ao seu redor brincando
de quarentena com bichos de pelúcia, mas também é normal que os adultos se perguntem,
desconfortavelmente, se tudo isso deixa uma marca duradoura. Quanto importará nos próximos
anos que, como disse recentemente a ministra da solidão Diana Barran, existam crianças sendo
criadas por pais protetores que nunca tiveram um encontro para brincar? Os bebês Covid
crescerão como criaturas solitárias, acostumadas a se divertir ou mais cautelosas com os
estranhos que raramente encontram e com quem interagem? Tirar um pequeno pacote em
público costumava significar uma sucessão interminável de mulheres mais velhas aleatórias
arrulhando no carrinho ou estranhos fazendo caretas para entreter um bebê entediado na fila do
caixa. Mas agora os transeuntes não ousam se aproximar e outros compradores estão escondidos
atrás de máscaras.
Esta semana, os parlamentares foram apresentados a algumas descobertas iniciais de um projeto
liderado pelo Movimento dos Primeiros 1001 Dias, uma aliança de instituições de caridade e
profissionais para os primeiros anos, acompanhando a vida de menores de dois anos crescendo
durante uma pandemia. Uma pesquisa com prestadores de serviços infantis que encomendou
descobriu que 98% achavam que os bebês e crianças com quem trabalhavam haviam sido
afetados por maior estresse e ansiedade dos pais, enquanto 92% viram famílias temerosas se
isolarem efetivamente do mundo exterior, pulando compromissos de rotina ou não querendo sair
de casa. Nove em cada 10 observaram crianças brincando com menos ou sendo menos ativas. De
forma dolorosa, mais de um quarto disse que o confinamento deixou as crianças com quem
trabalharam mais expostas a conflitos domésticos, abusos ou negligência.

Hair ice: The strange phenomenon of 'candy floss' on trees


If you (…) for a winter walk, you can be in for a big surprise. People taking a stroll through the trees
in counties Fermanagh and Tyrone have been encountering an unusual sight. At first glance, it
looks like candy floss has appeared on tree branches. On closer inspection, you can see hundreds
of individual strands of what looks like delicate white hair.
As soon as these are touched by human hand or winter sun, they melt away. This strange
phenomenon is called hair ice. The crystals are formed on rotting wood on humid winter nights
when the temperature is just below zero. Scientists have discovered it is caused by a fungus which
enables the ice to form thin hairs with a diameter of about 0.01mm.
Hair ice - also known as frost flowers - has been spotted in County Fermanagh in Castle Archdale,
Florence Court, Castle Coole, Cladagh Glen and Big Dog Forest. In County Tyrone, it has been

AULA 03 – ARTICLES AND NOUNS 120


TEACHER ANDREA BELO

pictured in Omagh and Knockmany, near Augher. So if you're heading out for some daily exercise,
see if you can spot this rare phenomenon before it melts away.
Adapted from https://www.bbc.com/news/uk-northern-ireland-55531529

Gelo no cabelo: o estranho fenômeno do "algodão doce" nas árvores


Se você (...) para uma caminhada de inverno, pode ter uma grande surpresa. Pessoas que fazem
um passeio por entre as árvores nos condados de Fermanagh e Tyrone têm encontrado uma visão
incomum. À primeira vista, parece que o algodão doce apareceu nos galhos das árvores. Em uma
inspeção mais próxima, você pode ver centenas de fios individuais do que parece ser um cabelo
branco delicado.
Assim que são tocados pela mão humana ou pelo sol de inverno, eles derretem. Este estranho
fenômeno é chamado de gelo no cabelo. Os cristais são formados na madeira apodrecida nas
noites úmidas de inverno, quando a temperatura está logo abaixo de zero. Os cientistas
descobriram que é causado por um fungo que permite ao gelo formar fios finos com um diâmetro
de cerca de 0,01 mm.
O gelo no cabelo - também conhecido como flores de geada - foi detectado no condado de
Fermanagh em Castle Archdale, Florence Court, Castle Coole, Cladagh Glen e Big Dog Forest. No
condado de Tyrone, foi fotografado em Omagh e Knockmany, perto de Augher. Portanto, se você
está saindo para fazer algum exercício diário, veja se consegue identificar esse fenômeno raro
antes que ele desapareça.

Where is Jack Ma? Chinese tycoon not seen since October


Speculation is mounting over the whereabouts of the Chinese billionaire Jack Ma, who has not
been seen or heard in public for more than two months. Ma, the co-founder and former chairman
of the technology firm Alibaba, has fallen out of favour with China’s leadership. In late October, he
stood alongside senior officials and delivered a blunt speech criticizing national regulators,
reportedly infuriating China’s president, Xi Jinping.
In the following months, regulators summoned Ma and other executives in for questioning and
halted what would have been the world’s biggest share offering of his company, Ant Group. They
later launched anti-monopoly investigations into Alibaba and its key competitor TenCent, and
called Ant Group in for questioning. In late December, regulators ordered Ma to pare down his
empire.
Ma has not been heard from during the controversies. His last public appearance appears to have
been at a livestreamed event on 31 October, an opening ceremony of the annual Chinese shopping
event Double 11. In November, he reportedly did not appear in his role as a judge for the final
episode of entrepreneurial game show Africa’s Business Heroes, which Alibaba has since blamed
on a scheduling conflict.
Ma was replaced with another Alibaba executive and his photo removed from the website. He
also did not appear at this year’s Shanghai Zhejiang chamber of commerce meeting, where he has

AULA 03 – ARTICLES AND NOUNS 121


TEACHER ANDREA BELO

delivered a speech every year since 2016. Bloomberg reported Ma had been warned in early
December not to leave the country. Observers are being cautious in drawing any conclusions, with
so little confirmed information.
Adapted from https://www.theguardian.com/business/2021/jan/05/where-is-jack-ma-chinese-tycoon-not-seen-since-october-alibaba

Onde está Jack Ma? Magnata chinês não visto desde outubro
As especulações aumentam sobre o paradeiro do bilionário chinês Jack Ma, que não é visto ou
ouvido em público há mais de dois meses. Ma, o cofundador e ex-presidente da empresa de
tecnologia Alibaba, caiu em desgraça com a liderança da China. No final de outubro, ele ficou ao
lado de altos funcionários e fez um discurso contundente criticando os reguladores nacionais,
supostamente enfurecendo o presidente da China, Xi Jinping.
Nos meses seguintes, os reguladores convocaram Ma e outros executivos para questionar e
suspenderam o que teria sido a maior oferta mundial de ações de sua empresa, o Ant Group.
Posteriormente, eles iniciaram investigações antimonopólio sobre o Alibaba e seu principal
concorrente, o TenCent, e chamaram o Ant Group para interrogatório. No final de dezembro, os
reguladores ordenaram que Ma reduzisse seu império.
Não se ouviu falar de Ma durante as controvérsias. Sua última aparição pública parece ter sido em
um evento transmitido ao vivo em 31 de outubro, uma cerimônia de abertura do evento anual de
compras chinês Double 11. Em novembro, ele não teria aparecido em seu papel de juiz no episódio
final do game show empresarial Africa's Business Heroes, que o Alibaba desde então atribuiu a
um conflito de agendamento.
Ma foi substituído por outro executivo do Alibaba e sua foto removida do site. Ele também não
apareceu na reunião da câmara de comércio de Xangai Zhejiang neste ano, onde fez um discurso
todos os anos desde 2016. A Bloomberg relatou que Ma havia sido avisado no início de dezembro
para não deixar o país. Os observadores estão sendo cautelosos ao tirar quaisquer conclusões,
com tão poucas informações confirmadas.

‘Bean dad’ apologises after tin can posts cause outcry


A dad in the US who boasted about his parenting skills after telling his hungry nine-year-old
daughter to open a tin of beans or go without food says he is “deeply sorry” about his tweets.
John Roderick, a podcaster and musician, said in a statement that he wanted to “acknowledge and
make amends for the injuries” caused by his posts. He said they were framed in a certain way to
reflect his “comedic persona”.
Twitter users, who nicknamed him “bean dad”, condemned the reported incident, suggesting it
was poor parenting. In an apology posted on his website on Tuesday, Mr Roderick said he had
been “ignorant” and “insensitive” with the way he had described the incident.
Mr Roderick, who has since deactivated his Twitter account, said he was wrong for being “flippant
when confronted” and for “taking my Twitter feed offline yesterday instead of facing the music”.
What was the Twitter story? Mr Roderick shared a story on Saturday on Twitter, beginning with
his daughter asking him to make baked beans. After she brought him a tin-opener and can of

AULA 03 – ARTICLES AND NOUNS 122


TEACHER ANDREA BELO

beans, he asked her how she thought a tin-opener worked, he said. When she said she didn't
know, he said he realised “a teaching moment just dropped into my lap”. Explaining that he wanted
his daughter to learn how to open a tin of beans, he said she tried for six hours.
Eventually she opened the tin and ate the beans, he explained, claiming it was a victory for “good
parenting”. But some Twitter users accused Mr Roderick of “child abuse”, suggesting his actions
were “ridiculous” and that it was right for his daughter to ask for “help and support”.
Adapted from https://www.bbc.com/news/world-us-canada-55549536

‘Bean dad’ pede desculpas depois que mensagens de lata causam protestos
Um pai nos Estados Unidos que se gabou de suas habilidades parentais depois de dizer a sua filha
de nove anos com fome para abrir uma lata de feijão ou ficar sem comer diz que “lamenta
profundamente” por seus tweets. John Roderick, um podcaster e músico, disse em um
comunicado que deseja “reconhecer e reparar os ferimentos” causados por suas postagens. Ele
disse que eles foram enquadrados de uma certa maneira para refletir sua “personalidade cômica”.
Os usuários do Twitter, que o apelidaram de “pai feijão”, condenaram o incidente relatado,
sugerindo que foi uma mãe pobre. Em um pedido de desculpas postado em seu site na terça-feira,
Roderick disse que havia sido "ignorante" e "insensível" com a forma como descreveu o incidente.
O Sr. Roderick, que desde então desativou sua conta no Twitter, disse que estava errado por ser
“irreverente quando confrontado” e por “colocar meu feed do Twitter offline ontem em vez de
encarar a música”. Qual foi a história do Twitter? O Sr. Roderick compartilhou uma história no
sábado no Twitter, começando com sua filha pedindo-lhe para fazer feijão. Depois que ela trouxe
para ele um abridor de lata e uma lata de feijão, ele perguntou como ela achava que um abridor
de lata funcionava, ele disse. Quando ela disse que não sabia, ele disse que percebeu que “um
momento de ensino caiu no meu colo”. Explicando que queria que sua filha aprendesse a abrir
uma lata de feijão, ele disse que ela tentou por seis horas.
Por fim, ela abriu a lata e comeu o feijão, explicou ele, alegando que era uma vitória da “boa
paternidade”. Mas alguns usuários do Twitter acusaram Roderick de “abuso infantil”, sugerindo
que suas ações eram “ridículas” e que era certo sua filha pedir “ajuda e apoio”.

New COVID Vaccines Need Absurd Amounts of Material and Labor


The first doses of mRNA COVID vaccines began arriving at hospitals in the U.S. and several other
countries in December. An overarching question is how fast companies making them can scale up
production to meet global demand. This is the first time mRNA vaccines have been authorized for
use outside of clinical trials. They work by tricking the body’s own cells into making a viral protein
that prompts immune reactions against infection. The U.S. has granted emergency authorization
to two COVID vaccines so far—made by the pharmaceutical companies Pfizer and BioNTech and
the biotechnology firm Moderna, respectively—and both rely on mRNA.
The Trump administration reached a deal with Pfizer in late December to provide 100 million
additional doses to the U.S. by the end of July—resulting in twice the amount the government

AULA 03 – ARTICLES AND NOUNS 123


TEACHER ANDREA BELO

originally ordered. Together with its partner BioNTech, Pfizer plans to produce and distribute 1.3
billion doses globally next year. And Moderna intends to produce 500 million to one billion doses,
of which 200 million have already been allocated to the U.S.
Meeting these targets will be no easy feat. Each step in the manufacturing process requires raw
materials that, before COVID, were only produced in the amounts needed for clinical research.
Such mRNA vaccines are created using much faster chemical processes than traditional vaccines
made by growing weakened viruses in chicken eggs. According to a November report by the U.S.
Government Accountability Office (GAO), much of what is needed to produce these vaccines is in
short supply.
Adapted from https://www.scientificamerican.com/article/new-covid-vaccines-need-absurd-amounts-of-material-and-labor/

Novas vacinas COVID precisam de quantidades absurdas de material e mão de obra


As primeiras doses de vacinas de mRNA COVID começaram a chegar a hospitais nos EUA e em
vários outros países em dezembro. Uma questão abrangente é com que rapidez as empresas que
os fabricam podem aumentar a produção para atender à demanda global. Esta é a primeira vez
que vacinas de mRNA foram autorizadas para uso fora dos ensaios clínicos. Eles atuam enganando
as próprias células do corpo para que produzam uma proteína viral que estimula reações
imunológicas contra a infecção. Os EUA concederam autorização de emergência para duas vacinas
COVID até agora - feitas pelas empresas farmacêuticas Pfizer e BioNTech e pela empresa de
biotecnologia Moderna, respectivamente - e ambas dependem de mRNA.
A administração Trump chegou a um acordo com a Pfizer no final de dezembro para fornecer 100
milhões de doses adicionais aos EUA até o final de julho - resultando em duas vezes a quantidade
que o governo originalmente ordenou. Junto com seu parceiro BioNTech, a Pfizer planeja produzir
e distribuir 1,3 bilhão de doses globalmente no próximo ano. E a Moderna pretende produzir de
500 milhões a um bilhão de doses, das quais 200 milhões já foram destinadas aos EUA.
Cumprir essas metas não será uma tarefa fácil. Cada etapa do processo de fabricação requer
matéria-prima que, antes da COVID, era produzida apenas nas quantidades necessárias para a
pesquisa clínica. Essas vacinas de mRNA são criadas usando processos químicos muito mais
rápidos do que as vacinas tradicionais feitas pelo crescimento de vírus enfraquecidos em ovos de
galinha. De acordo com um relatório de novembro do U.S. Government Accountability Office
(GAO), muito do que é necessário para produzir essas vacinas está em falta.

AULA 03 – ARTICLES AND NOUNS 124


AFA 2024

ADJECTVES AND ADVERBS

AULA 04

Teacher Andrea Belo

www.estrategiamilitares.com.br www.militares.estrategia.com
TEACHER ANDREA BELO

SUMÁRIO
INTRODUÇÃO 3

ADJETIVOS 4

ADJETIVOS – GRAU COMPARATIVO 7

ADJETIVOS – GRAU SUPERLATIVO 11

ADVÉRBIOS 15

ADVÉRBIOS: GRAU COMPARATIVO 29

ADVÉRBIOS: GRAU SUPERLATIVO 31

QUESTÕES 33

GABARITO 64

QUESTÕES COMENTADAS 65

CONSIDERAÇÕES FINAIS 125

REFERÊNCIAS BIBLIOGRÁFICAS 126

TRADUÇÕES 128

AULA 04 – ADJECTIVES AND ADVERBS 2


TEACHER ANDREA BELO

INTRODUÇÃO
Identificar em uma frase, os advérbios e os adjetivos, é algo considerado tricky, ou seja,
algo melindroso, arriscado.
Isso porque, em Inglês, como também em outras línguas, essas palavras nos enganam
bastante pela maneira como se apresentam nos textos.
Mas, você verá, nessa aula, como é simples a distinção desses dois termos.
Vamos, então, à nossa aula sobre adjetivos e advérbios, para que você possa aprender as
regras de uso de cada um deles para encontrá-los inseridos em sua prova, levando você a acertar
o que for perguntado acerca desse assunto.
Os adjectives – adjetivos, são palavras que têm o propósito de qualificar ou modificar os
substantivos. Às vezes, modificam pronomes também, como veremos nas explicações.
Adjetivos têm como meta proporcionar detalhes e descrever melhor pessoas, lugares,
objetos etc.
E, com o acréscimo dos adjetivos nas frases, as ideias apresentadas se tornam mais
interessantes e os textos ficam mais interativos e atrativos na hora da leitura.
Os adverbs – advérbios, são palavras que também acrescentam informações importantes
ao texto, porém, geralmente, modificam os verbos ou os próprios adjetivos.
O advérbio é um elemento que indica a circunstância em que se encontra o verbo e assim,
vai ressaltar as circunstâncias, ou seja, o modo, a intensidade, o lugar, uma negação, uma
afirmação, uma dúvida, entre outras situações diversas nos textos.
Tanto adjetivos quanto advérbios são importantes elementos e, devemos sempre nos
lembrar de que, uma determinada palavra em sua prova, pode fazer com que o termo analisado
mude, de uma classe gramatical para outra, a depender do contexto em que está inserida.
Por isso, esse material está dividido em detalhadas etapas, com os tópicos explicitados,
para que você estude cada item, separadamente e com muitos exemplos.
Assim, no momento da prática de exercícios, você perceberá que estão envolvidos, em
uma só questão, diferentes assuntos, que abrangem todo o conteúdo a ser estudado por você –
da mesma forma que são elaboradas as provas.
Vou orientar você, mostrando as normas que precisam ser seguidas em relação aos
adjetivos e aos advérbios, para que você os encontre nos textos e saiba como reconhecê-los nas
alternativas de cada questão.
Vamos lá! Você consegue e será o melhor candidato. Conte comigo!

AULA 04 – ADJECTIVES AND ADVERBS 3


TEACHER ANDREA BELO

ADJETIVOS
Os adjectives (adjetivos) caracterizam os substantivos e expressam qualidades ou
características em relação a pessoas, objetos ou animais.
Essas qualidades podem ser duradouras ou permanentes, características que expressam
estados passageiros e condições e, além de qualidades, as características das ações de pessoas,
objetos ou animais.
A classe de adjetivos está entre as maiores classes gramaticais de palavras, ou seja, possui
um número ilimitado de palavras, que aumenta gradativamente, de acordo com a evolução da
língua.
Dessa forma, não conseguimos fazer uma lista com todos os adjetivos da língua inglesa,
principalmente porque, essa lista se tornaria desatualizada daqui em tempo.
A posição do adjetivo em uma frase pode variar, assim como sua terminação. E veremos
cada caso em particular adiante. Antes de relatar outros detalhes sobre os adjetivos, vou citar
algumas das terminações mais comuns para os adjetivos em geral, com 5 exemplos cada, certo?

TERMINAÇÕES COMUNS NOS ADJETIVOS:


-al = cultural, original, practical, sensational, logical.
-an = Brazilian, American, Canadian, Belgian, European.
-ant = brilliant, elegant, important, significant, relevant.
-ar = bipolar, familiar, nuclear, peculiar, popular.
-ble = acceptable, comfortable, flexible, horrible, possible.
-ed = advanced, haunted, liberated, refined, sophisticated.
-ent = different, efficient, excellent, fluent, violent.
-ful = careful, colorful, helpful, useful, wonderful.
-ic = artistic, dramatic, fantastic, scientific, tragic.
-ing = amazing, darling, interesting, good-looking, outgoing.
-ive = creative, expensive, naive, receptive, talkative.
-less = effectless, faithless, hairless, limitless, useless.
-ous = conscious, dangerous, fabulous, jealous, marvelous.
-y = easy, fancy, friendly, messy, satisfactory.

AULA 04 – ADJECTIVES AND ADVERBS 4


TEACHER ANDREA BELO

Em Inglês, especificamente, os adjetivos são invariáveis em relação ao gênero (masculino


e feminino) e número (singular e plural).
Isso significa que, um mesmo adjetivo é utilizado para caracterizar masculino, feminino,
singular e plural. Por exemplo, o adjetivo new, significa novo, nova, novos e novas, veja:
• I have a new pen. (Tenho uma caneta nova.)
• l have two new pens. (Tenho duas canetas novas.)
• I have a new notebook. (Tenho um caderno novo.)
• I have new notebooks. (Tenho cadernos novos.)
E, como são invariáveis no gênero, veja outros exemplos, com referência a pessoas:
• The boy is blond. (O garoto é loiro.)
• The boys are blond. (Os garotos são loiros.)
• The girl is blond. (A garota é loira.)
• The girls are blond. (As garotas são loiras.)
Comumente, os adjetivos são utilizados depois de verbos de ligação. Esses verbos,
denominados verbos de ligação, não indicam ação, nunca. Eles indicam estado, ligando uma
característica a um sujeito.
Alguns exemplos de verbos de ligação são os verbos ser, estar, permanecer, tornar-se,
sendo em Inglês – to be, to stay, to become etc), ou seja, o adjetivo é ligado ao substantivo por
esse verbo de ligação. Vejamos alguns exemplos:
Bob is a calm buy. (Bob é um garoto calmo). ou Bob is calm. (Bob é calmo).
That song is popular. (Aquela música é pop).
He gets talkative when talking about technology. (Ele fica conversador/ tagarela/
comunicativo quando fala sobre tecnologia).
Uma mudança interessante, que inclusive, eu já havia comentado em nossa primeira aula,
é que, ao contrário do Português, que a ordem é o adjetivo após o substantivo, em Inglês, o
adjetivo vem sempre antes do substantivo. Exemplos:
Um garoto alto = A tall boy. A tall girl – Uma garota alta.
Um garoto baixo = A short boy. A short girl – Uma garota baixa.

Adjectives • TALL (alto) Nouns • BOY (garoto)


(adjetivos) • SHORT (baixo) (substantivos)• GIRL (garota)

AULA 04 – ADJECTIVES AND ADVERBS 5


TEACHER ANDREA BELO

Existem alguns adjetivos com terminações diferenciadas e incomuns, que merecem


atenção. É o caso daqueles terminados em –ing e –ed, que podem, a princípio, fazer com que você
se lembre dos tempos verbais Present Continuous e Past Simple.
Mas, se não há verbo to be que antecede a palavra com terminação -ing, não é Continuous.
E, se também não há alguma expressão que comprove o tempo passado, não se classifica
como Past Simple. Além disso, você sabe diferenciar um adjetivo de um verbo, quer ver?
• Jannet’s job is boring. (O emprego de Janet é chato.)
• Jannet is bored today. (Jannet está entediada hoje.)
• The accident was shocking. (O acidente foi chocante.)
• She is shocked. (Ela está chocada.)
É claro que você sabe, olhando os exemplos acima, que os adjetivos bored e boring não se
parecem, nem de longe, com verbos, ações.
E, como estão qualificando substantivos – Jannet = bored, job = boring, accident = shocking
e she = chocked, são, de fato, adjetivos dentro da frase.
Os adjetivos terminados em -ed indicam como alguém se sente, ao passo que os
terminados em -ing dizem algo sobre alguém.
Quando utilizamos mais de um adjetivo na mesma frase, devemos obedecer a uma ordem
de uso, de acordo com a ideia implícita em cada um:

opinion (opinião) + size/age (tamanho/idade) + shape (forma) + color


(cor) + origin (origem) + material (material) + noun (substantivo)
Exemplo: The expensive big white Canadian wood house. (A grande
casa canadense cara branca de madeira.

Não é necessário utilizar todos os adjetivos na construção de frases e, geralmente, não


aparecem todos em uma só oração.
O exemplo acima foi, intencionalmente, para mostrar a você quais informações podem
estar presentes e fazer parte de uma ideia qualificando alguma coisa usando muitos adjetivos.
Está percebendo como você vai identificar adjetivos nas questões da prova?
Quando perguntar se o termo pode ser substituído por um adjetivo ou se o próprio adjetivo
pode substituir outro, você estará apto a responder com segurança.
Agora vamos às variações de grau dos adjetivos: Comparativo e Superlativo.

AULA 04 – ADJECTIVES AND ADVERBS 6


TEACHER ANDREA BELO

ADJETIVOS – GRAU COMPARATIVO


Os adjetivos podem possuir graus de comparação, divididos em comparativos e
superlativos. Comparative sentences, frases com uso de comparativos, como o próprio nome já
indica, tem como tarefa, comparar dois ou mais substantivos usando um adjetivo para realizar tal
comparação.
Essa relação de comparação pode ser de superioridade (quando um substantivo possui
mais de uma qualidade que o outro), pode ser de igualdade (quando ambos os substantivos
possuem a mesma quantidade de certa qualidade) ou pode ser também de inferioridade (quando
um possui menos daquela qualidade que o outro).
Veremos cada um dos tipos de comparação de adjetivos com exemplos, da mesma maneira
que as bancas utilizam para elaborar as questões da prova.
Você precisa saber identificá-los para que possa substitui-los por outros quando solicitado.
Ou quando se pede para encontrar a alternativa em que não há comparação ou até mesmo
responder se uma frase específica possui elementos comparativos ou não e assim por diante.

COMPARATIVO DE SUPERIORIDADE
Estudar frases comparativas em Inglês, que são muitas, fica simples se você pensar qual é
o real objetivo de uma comparação: estabelecer um paralelo entre uma coisa e outra.
Exatamente isso: usamos o grau comparativo para equiparar uma pessoa ou uma coisa
com outra, para comparar as diferenças entre os dois elementos que se modificam.
A estrutura usada nas frases em que há o comparativo de superioridade é essa: substantivo
1 (sujeito) + verbo + adjetivo comparativo + than (do que) + substantivo 2 (objeto).
Mas, depende do número de letras que cada adjetivo tem. Isso mesmo, se o adjetivo é
curto ou longo, interfere na formação das frases.
E existem outras regras a serem seguidas. Vamos estudar cada uma delas.
Para adjetivos de uma sílaba, ou seja, até 5 letras, com o som de uma só sílaba,
considerados “curtos”, como tall – alto/a, short – baixo/a e outros, acrescentamos a terminação
-er para fazer a frase comparativa:

Bethy is taller than Jane. (Bethy é mais alta do que Jane.)

Se o adjetivo terminar em CVC: consoante + vogal + consoante, como hot, big, sad e wet,
entre outros, a consoante final deve ser duplicada antes da incluir -er + than no final do adjetivo:
Summer is hotter than Spring. (O verão é mais quente do que a primavera.)

AULA 04 – ADJECTIVES AND ADVERBS 7


TEACHER ANDREA BELO

Os adjetivos que possuem cinco ou mais letras, geralmente com duas sílabas, tais como
simple, clever, common e quiet (simples, inteligente, comum e quieto), formam o comparativo
com a adição de -er ou pelo uso da palavra more antes do adjetivo + than. Vale lembrar que ambas
as formas são utilizadas e estão corretas:

Bethy is simpler than Jane. (Bethy é mais simples do que Jane.)


Bethy is more simple than Jane. (Bethy é mais simples do que Jane.)

No caso de outros adjetivos, com mais de 6 letras e considerados longos, tais como
intelligent, interesting, beautiful, comfortable etc (inteligente, interessante, bonito/a, confortável
etc), apenas se faz a comparação acrescentando more antes do adjetivo + than:

Bethy is more intelligent than Jane. (Bethy é mais inteligente do que Jane.)

Por sua vez, adjetivos terminados em y, com até 5 letras, tais como pretty, busy, noisy,
happy etc – lindo/a, ocupado/a, barulhento/a, feliz, e etc, seguem a regra dos outros adjetivos
considerados “curtos”.
Porém, elimina-se a letra y, que é automaticamente substituída pela letra i e, assim, faz-se
o acréscimo de -ier + than:

Bethy is prettier than Jane. (Bethy é mais linda do que Jane.)

Por último, os adjetivos que, quando usados em frases comparativas, mudam


completamente. São chamados de adjetivos irregulares, tais como good, bad e far – bom, mau e
longe, não seguem regras e cada um tem uma grafia diferente quando a comparação é feita, veja:

GOOD: Bethy is better than Jane. (Bethy é melhor do que Jane.)


BAD: Bethy is worse than Jane. (Bethy é pior do que Jane.)
FAR: Bethy’s house is further than Jane’s one. (A casa de Bethy é mais longe do que a casa de Jane.)

Agora, vamos visualizar as formas de se fazer frases comparativas em Inglês, usando


qualquer adjetivo que aparecer em sua prova.
Assim, você poderá identificar o grau comparativo nas frases retiradas dos textos e saberá
responder qualquer questão sobre esse assunto com esse esquema:
• Adjetivos com 1 ou 2 sílabas: -ER (Exemplo: tall)
John is taller than Tom. (John é mais alto do que Tom.)
• Adjetivos com 2 ou mais sílabas: MORE (Exemplo: sociable)
John is more sociable than Tom. (John é mais sociável do que Tom.)
• Adjetivos com final em Y: - IER ou MORE (Exemplo: funny)
John is funnier/more funny than Tom. (John é mais engraçado do que Tom.)
• Adjetivos com final CVC: dobra a letra (Exemplo: thin)

AULA 04 – ADJECTIVES AND ADVERBS 8


TEACHER ANDREA BELO

John is thinner than Tom. (John é mais magro do que Tom.)


• Adjetivos irregulares: se transformam (Exemplo: good)
John is better than Tom when playing golf. (John é melhor do que Tom quando joga golfe.)

Bethy is taller than Jane. – Bethy é mais alta do que Jane.

COMPARATIVO DE IGUALDADE
O grau comparativo de igualdade é usado para comparar dois elementos equivalentes
entre si. Geralmente, a comparação é feita com apenas dois elementos, dois sujeitos em uma
frase.
A estrutura usada nas frases em que há o comparativo de igualdade é essa: substantivo 1
(sujeito) + verbo + as + adjetivo + as + substantivo 2 (objeto).
Essa estrutura é igual para todos os adjetivos da língua inglesa, não depende do número
de letras ou sílabas que cada adjetivo possui. Vejamos exemplos:
Bethy is as intelligent as Jane. (Bethy é tão inteligente quanto Jane.)
Bethy is as pretty as Jane. (Bethy é tão linda quanto Jane.)
Bethy is as good as Jane. (Bethy é tão bondosa quanto Jane.)

Anne is as tall as John – Anne é tão alta quanto John.

AULA 04 – ADJECTIVES AND ADVERBS 9


TEACHER ANDREA BELO

COMPARATIVO DE INFERIORIDADE
Por sua vez, o grau comparativo de inferioridade é usado para comparar dois elementos,
evidenciando que o primeiro é inferior ao segundo.
A estrutura usada nas frases em que há o comparativo de inferioridade é essa: substantivo
1 (sujeito) + verbo + less + adjetivo + substantivo 2 (objeto).
Essa estrutura também é igual para todos os adjetivos da língua inglesa, não depende do
número de letras ou sílabas que cada adjetivo possui. Vejamos exemplos:

Bethy is less tall than Jane. (Bethy é menos alta do que Jane.)
Bethy is less intelligent than Jane. (Bethy é menos inteligente do que Jane.)
Bethy is less pretty than Jane. (Bethy é menos linda do que Jane.)
Bethy is less good than Jane. (Bethy é menos bondosa do que Jane.)

Algumas pessoas preferem usar a negação no comparativo de inferioridade (que é apenas


usar o verbo to be na forma negativa com o acréscimo de not) ao invés do comparativo de
inferioridade, para a frase “soar melhor” e não enfatizar algo inferior, veja exemplos:

Bethy is not as tall as Jane. (Bethy não é tão alta quanto Jane.)
Beth and Jane are not as tall as Susan. (Bethy e Jane não são tão altas quanto Susan.)

Elizabeth is less tall than John. (Anne é menos alta do que John.)

Agora, vamos aos estudos do grau superlativo.

AULA 04 – ADJECTIVES AND ADVERBS 10


TEACHER ANDREA BELO

ADJETIVOS – GRAU SUPERLATIVO


Os superlativos têm a função de intensificar, de forma única, a qualidade de um
substantivo em relação a todos os outros de um mesmo segmento.
Por exemplo, vimos, no capítulo anterior, que dizer que alguém é mais inteligente, tão
inteligente quanto ou tem menos inteligência, temos que falar de outra pessoa para fazer a
comparação.
No superlativo, se diz que alguém é o mais inteligente de todos. E pronto!
Essa relação de intensidade pode ser de superioridade ou de inferioridade que vamos ver
agora com exemplos para ficar claro.

SUPERLATIVO DE SUPERIORIDADE
A estrutura usada nas frases em que há o superlativo de superioridade é essa: substantivo
+ verbo + artigo THE + adjetivo no superlativo (veremos as regras).
Assim como no modo comparativo, analisando o número de letras/sílabas dos adjetivos, o
superlativo é mais simples, mas a formação de frases é diferente para o adjetivo curto ou longo.
Para os adjetivos curtos – de uma sílaba, até 5 letras – como tall – alto/a, short – baixo/a e
outros considerados “curtos”, acrescenta-se a terminação -est no final:
Bethy is the tallest girl in the class. – Bethy é a garota mais alta da sala.

Note que o artigo the é muito importante no superlativo, pois proporcionam ênfase de que
algo/alguém é o mais/a mais sobre o assunto tratado na frase.
Nos adjetivos longos, com mais de 5 letras e mais sílabas, tais como beautiful, interesting,
important etc, acrescentamos o termo the most antes do adjetivo.
Bethy is the most beautiful girl in the class. – O verão é a garota mais bonita da sala.

Por sua vez, adjetivos terminados em y, com até 5 letras, tais como pretty, busy, noisy,
happy etc – lindo/a, ocupado/a, barulhento/a, feliz, e etc, seguem a regra dos outros adjetivos
considerados “curtos”, mas elimina-se a letra y, que é substituída pela letra i e, assim, faz-se o
acréscimo de -iest:
Bethy is the prettiest girl in the class. – Bethy é a garota mais linda da sala.

Os adjetivos que terminam em CVC (consoante/vogal/consoante) apenas seguem regras


de -est nos curtos e the most nos longos, da mesma maneira, dobrando as letras:

Bethy is the thinnest girl in the class. – Bethy é a garota mais magra da sala.

Por último, os adjetivos irregulares que vimos no comparativo, tais como good, bad e far –
bom, mau e longe, também não seguem regras e cada um tem uma grafia diferente quando a
comparação é feita – the best, the worst e the furthest, veja:

AULA 04 – ADJECTIVES AND ADVERBS 11


TEACHER ANDREA BELO

GOOD: Bethy is the best student. – Bethy é a melhor aluna. (de todas)
BAD: Bethy is the worst student. – Bethy é a pior aluna. (de todas)
FAR: Bethy’s house is the furthest in this city. – A casa de Bethy é a mais longe nessa cidade.

Vejamos agora como se usa o superlativo de inferioridade.

SUPERLATIVO DE INFERIORIDADE
A estrutura usada nas frases em que há o superlativo de superioridade é essa: substantivo
+ verbo + artigo THE LEAST + adjetivo – todos os adjetivos são usados com the least antes. A regra
é uma só para todos. Para adjetivos longos, curtos, com CVC ou com final y.

Bethy is the least thin student. – Bethy é a aluna menos magra. (de todas)
Bethy is the least intelligent student. – Bethy é a aluna menos inteligente. (de todas)
Bethy is the least funny student. – Bethy é a aluna menos engraçada. (de todas)

Agora, que estudamos todas as estruturas de comparativo e superlativo, vamos resolver


um exercício do ITA, ano de 2018, com uso do assunto que tratamos.

GOODBYE THINGS, HELLO MINIMALISM: CAN LIVING WITH LESS MAKE YOU HAPPIER?
Fumio Sasaki owns a roll-up mattress, three shirts and four pairs of socks. After deciding to scorn
possessions, he began feeling happier. He explains why.
Let me tell you a bit about myself. I’m 35 years old, male, single, never been married. I work as an
editor at a publishing company. I recently moved from the Nakameguro neighbourhood in Tokyo,
where I lived for a decade, to a neighbourhood called Fudomae in a different part of town. The
rent is cheaper, but the move pretty much wiped out my savings.
Some of you may think that I’m a loser: an unmarried adult with not much money. The old me
would have been way too embarrassed to admit all this. I was filled with useless pride. But I
honestly don’t care about things like that any more. The reason is very simple: I’m perfectly happy
just as I am. The reason? I got rid of most of my material possessions.
Minimalism is a lifestyle in which you reduce your possessions to the least possible. Living with
only the bare essentials has not only provided superficial benefits such as the pleasure of a tidy
room or the simple ease of cleaning, it has also led to a more fundamental shift. It’s given me a
chance to think about what it really means to be happy.
We think that the more we have, the happier we will be. We never know what tomorrow might
bring, so we collect and save as much as we can. This means we need a lot of money, so we
gradually start judging people by how much money they have. You convince yourself that you

AULA 04 – ADJECTIVES AND ADVERBS 12


TEACHER ANDREA BELO

need to make a lot of money so you don’t miss out on success. And for you to make money, you
need everyone else to spend their money. And so it goes.
So I said goodbye to a lot of things, many of which I’d had for years. And yet now I live each day
with a happier spirit. I feel more content now than I ever did in the past.
I wasn’t always a minimalist. I used to buy a lot of things, believing that all those possessions
would ITA (2.o dia) — dezembro/2017 increase my self-worth and lead to a happier life. I loved
collecting a lot of useless stuff, and I couldn’t throw anything away. I was a natural hoarder of
knick-knacks that I thought made me an interesting person.
At the same time, though, I was always comparing myself with other people who had more or
better things, which often made me miserable. I couldn’t focus on anything, and I was always
wasting time. Alcohol was my escape, and I didn’t treat women fairly. I didn’t try to change; I
thought this was all just part of who I was, and I deserved to be unhappy.
My apartment wasn’t horribly messy; if my girlfriend was coming over for the weekend, I could
do enough tidying up to make it look presentable. On a usual day, however, there were books
stacked everywhere because there wasn’t enough room on my bookshelves. Most I had thumbed
through once or twice, thinking that I would read them when I had the time.
The closet was crammed with what used to be my favourite clothes, most of which I’d only worn
a few times. The room was filled with all the things I’d taken up as hobbies and then gotten tired
of. A guitar and amplifier, covered with dust. Conversational English workbooks I’d planned to
study once I had more free time. Even a fabulous antique camera, which of course I had never
once put a roll of film in.
It may sound as if I’m exaggerating when I say I started to become a new person. Someone said
to me: “All you did is throw things away,” which is true. But by having fewer things around, I’ve
started feeling happier each day. I’m slowly beginning to understand what happiness is.
If you are anything like I used to be – miserable, constantly comparing yourself with others, or
just believing your life sucks – I think you should try saying goodbye to some of your things. [...]
Everyone wants to be happy. But trying to buy happiness only makes us happy for a little while.
Fonte: adaptado de adaptado de <https:ltwww.theguardian.com/booksl2017/apr/121goodbyethings-hello-minimalism-can-living-with-Iessmake-you-happier>. Acesso em: 21 maio 2017.

Questão – Todas as frases abaixo usam a forma comparativa do adjetivo, EXCETO:


a) The rent is cheaper, (linha 3)
b) ... you reduce your possessions to the least possible. (linha 9)
c) ...the more we have, the happier we will be. (linha 13)
d) I feel more content now than I ever did in the past. (linha 19)
e) But by having fewer things around, (linha 36)

AULA 04 – ADJECTIVES AND ADVERBS 13


TEACHER ANDREA BELO

Comentários: Já usamos esse texto em outras aulas de nosso material, para responder questões
sobre outros tópicos gramaticais.
E, você pode perceber que, além de usar técnicas aprendidas para fazer a leitura com precisão,
certos exercícios não necessitam da leitura completa de fato para serem solucionados.
Mesmo assim, vale lembrar que, apesar de quase nunca precisar de ler o texto completo nem
traduzir palavra por palavra (e sim usar técnicas aprendidas), conhecer muitos vocábulos é sinal
de que você está preparado, está à frente de outros candidatos, pois possui vocabulário e
conhecimento mais amplo.
Além disso, você já sabe que terá acesso às traduções dos textos no fim do material – um bônus
para aprimorar e conhecer novos termos enquanto estiver estudando.
Veja que, no enunciado dessa questão, já estão as frases que você deve analisar para responder
qual delas não possui forma comparativa. Vamos lá.
Na letra A, o adjetivo cheap está na forma comparativa, pois tem o acréscimo de -er no final por
ser um adjetivo curto, como vimos na teoria acima.
Na letra B, o termo the least possible não está na forma comparativa e é a resposta correta da
questão, pois, é uma estrutura de superlativo de inferioridade, como vimos na teoria está, com o
acréscimo do artigo the least antes do adjetivo possible. E, também porque a terminação -est é
outra característica do grau superlativo.
Na letra C, o adjetivo happy está na forma comparativa, pois tem o acréscimo de -ier no final por
ser um adjetivo curto e terminado com a letra y.
Na letra D, o adjetivo content está na forma comparativa, pois tem o acréscimo de more antes do
adjetivo longo – more content.
Na letra E, o adjetivo few está na forma comparativa, pois tem o acréscimo de -er no final por ser
um adjetivo curto, assim como nas letras A e C.
GABARITO: B

Agora, que você sabe tudo sobre adjetivos – e espero que saiba mesmo – estudaremos a
classe de advérbios detalhadamente. Vamos lá! Come on!

AULA 04 – ADJECTIVES AND ADVERBS 14


TEACHER ANDREA BELO

ADVÉRBIOS
Os advérbios têm quase a mesma função dos adjetivos, contudo, ao invés de agregar
características a substantivos, os advérbios viabilizam qualidades de modo, tempo e lugar aos
verbos e, algumas vezes, aos próprios adjetivos!
Portanto, os advérbios funcionam como modificador de verbos, de adjetivos e, muitas
vezes, de outros advérbios, usados para dizer quando, como ou onde alguma coisa aconteceu e,
geralmente, aparece depois de verbos principais, como veremos.
Vejamos como se dá a formação dos advérbios para ficar mais fácil falar das classificações
deles adiante.
Você deve compreender muito bem para não confundir, já que os advérbios são derivados
de adjetivos e, algumas vezes, possuem a mesma forma do adjetivo.
A maioria dos advérbios são formados pelo acréscimo da terminação -ly, que significa -
mente. Este acréscimo ocorre nos advérbios que indicam modo, que indicam frequência ou
intensidade, entre muitos outros.
ADJETIVO “LENTO” = SLOW
ADVÉRBIO “LENTAMENTE” = SLOWLY
Exemplo: Bethy is slow and she drives slowly. (Bethy é lenta e ela dirige lentamente.)

Adjetivo (qualidade) Advérbio (modo)

Para advérbios de qualquer classificação, acrescenta-se -ly no final dos adjetivos. Para
adjetivos terminados pelo próprio y, trocamos o y por i e acrescentamos normalmente -ly. Veja
alguns exemplos:
crazy (louco) – crazily (loucamente)
easy (fácil) – easily (facilmente)
happy (feliz, alegre) – happily (felizmente, alegremente)
heavy (pesado) – heavily (pesadamente)
lucky (sortudo) – luckily (afortunadamente)

Para os adjetivos terminados em -le, trocando-se o -le por -ly, forma-se o advérbio:
horrible (horrível) – horribly (horrivelmente)
incredible (inacreditável) – incredibly (inacreditavelmente)
probable (provável) – probably (provavelmente)
simple (simples) – simply (simplesmente)
subtle (sutil) – subtly (sutilmente)

AULA 04 – ADJECTIVES AND ADVERBS 15


TEACHER ANDREA BELO

Com alguns adjetivos terminados em -e (sem a letra L antes da letra E), mantemos o -e,
acrescentando, normalmente -ly; com a exceção de dois adjetivos especiais nesse caso: true e
due:
brave (bravo) – bravely (bravamente)
immediate (imediato) – immediately (imediatamente)
Exceções: true (verdadeiro) – truly (verdadeiramente)
due (que se deve, adequado) – duly (pontualmente, a tempo)

Quando os adjetivos terminam em -ic acrescentam -ally e não somente -ly, veja:
automatic (automático) – automatically (automaticamente)
romantic (romântico) – romantically (romanticamente)
specific (específico) – specifically (especificamente)
tragic (trágico) – tragically (tragicamente)

De forma simples e direta, caso o adjetivo já termine em -ly, nada se acrescenta na formação do
advérbio:
Bethy is tired of her daily routine. (Justine está cansada da sua rotina diária.)
Bethy helps her sister friendly. (Bethy ajuda sua irmã amigavelmente.)

Alguns advérbios são usados com a grafia igual aos adjetivos, ou seja, não acrescentamos
nenhuma terminação nem modificamos quaisquer letras. Mas lembre-se de que o sentido muda.
Enquanto o adjetivo bonito – beautiful qualifica o sujeito, o advérbio beautiful indica o modo que
o sujeito executou alguma ação, veja:

Bethy is beautiful. She sings beautiful. (Bethy é bonita. Ela canta bonito – de forma bonita)

Além de beautiful, outros exemplos em que adjetivos e advérbios ficam iguais nas frases
em ambas as funções são fast, hard, right e late. Vejamos frases:
ADJETIVOS ADVÉRBIOS
Bethy é uma nadadora rápida.
Bethy is a fast swimmer. Bethy swims fast.
Bethy nada rápido.
Bethy é uma trabalhadora esforçada.
Bethy is a hard worker. Bethy works hard.
Bethy trabalha duro/com esforço.
Bethy é/está sempre certa.
Bethy is always right. Bethy thinks right.
Bethy pensa certo.
Bethy é/está atrasada.
Bethy is late. Bethy wakes up late sometimes.
Bethy acorda atrasada às vezes.

AULA 04 – ADJECTIVES AND ADVERBS 16


TEACHER ANDREA BELO

Alguns desses advérbios, que coincidem com os adjetivos, quando se coloca a terminação -ly,
muda-se o sentido da palavra e também o significado, como por exemplo hardly e lately:
Bethy works hard. Bethy trabalha duro/com esforço.
Bethy’s sick. She can hardly work. Bethy está doente. Ela mal consegue trabalhar.

Bethy is late. Bethy é/está atrasada.


Bethy’s sick. She didn’t work lately. Bethy está doente. Ela não trabalhou recentemente.

Outra importante observação é que nem todas as palavras terminadas em -ly são advérbios.
Algumas são adjetivos, acredita? Apesar da terminação -ly ser comumente usada em advérbios e
ser uma característica particular deles, lonely, por exemplo, é o adjetivo solitário. Vejamos outros:

Adjetivo lovely: adorável – Bethy is a lovely person. (Bethy é uma pessoa adorável).
Adjetivo friendly: amigável – Bethy is friendly with everybody. (Bethy é amigável com todos).
Adjetivo elderly: velho/idoso – Bethy’s mom is elderly. (A mãe de Bethy é uma idosa).

Um advérbio especial e diferente, que não usa -ly em sua grafia, é well, referente ao
adjetivo good. Uma pessoa pode ser boa/bondosa (como uma qualidade) ou pode ser boa em
alguma coisa que faz com dedicação, demonstrar uma habilidade.
Em Português, usamos apenas uma palavra para representar ambos, “bom de bondoso” e
“bom em algo que faça”. Mas em Inglês, veja:

Bethy is a good swimmer. (Bethy é uma boa nadadora.)


Bethy swims very well. (Bethy nada muito bem. / é boa em natação.)

Já vimos como formar os advérbios e, a maior parte deles sofre o acréscimo de -ly. Para
colocar os advérbios em uma frase, o que é necessário? Quando são usados? Vejamos.
Um advérbio pode ser usado para descrever um verbo, descrevendo quando, por que,
onde ou como uma ação ocorreu. Por exemplo, uma pessoa pode agir rapidamente, calmamente
ou silenciosamente. Na seguinte frase: Bethy falou ao telefone – Bethy talked on the phone, o
verbo falar pode ser modificado por um advérbio como lentamente. Ou, cuidadosamente:
Bethy talked on the phone slowly. (Bethy falou ao telephone lentamente.)
Bethy talked on the phone carefully. (Bethy falou ao telephone cuidadosamente.)

Os advérbios são inúmeros. Eles podem ser de modo, de lugar, de tempo, de frequência,
de intensidade, de certeza ou dúvida e, ainda existem as famosas locuções adverbiais. Eles
aparecem em diversas situações e enriquecem frases em geral.
Vamos, agora, analisar os tipos de advérbios que existem para que as explicações
anteriores sejam melhor compreendidas com exemplos de cada um com detalhes.

AULA 04 – ADJECTIVES AND ADVERBS 17


TEACHER ANDREA BELO

ADVÉRBIOS DE MODO
Como o próprio nome já nos diz, os adverbs of manner (advérbios de modo), indicam a
maneira em que uma ação ocorreu.
Os advérbios são formados a partir de adjetivos, em que se acrescenta o sufixo -ly e assim
deixam de ser adjetivos, passando a ser advérbios, como já vimos anteriormente.
Veja as comparações para que os advérbios de modo fiquem claros:

Bethy is bad (Bethy é má). Bethy sings badly (Bethy canta mal)
Beth is slow (Bethy é lenta/devagar). Bethy sings slowly. (Bethy canta vagarosamente).

Os advérbios de modo são bastante flexíveis e, normalmente, podem aparecer em três


posições:
1 – Antes do sujeito:
Quickly Bethy organized her books – Rapidamente Bethy organizou seus livros.
2 – Entre o sujeito e o verbo:
Bethy quickly organized her books – Bethy rapidamente organizou seus livros.
3 – Após o verbo ou o objeto:
Bethy organized her books quickly – Bethy organizou seus livros rapidamente.
The girl organized the books quickly.

ADVÉRBIOS DE FREQUÊNCIA
Os adverbs of frequency (advérbios de frequência) são, de fato, os mais conhecidos do
Inglês.
Eles são apenas palavras utilizadas para descrever com que frequência alguma atividade é
realizada, como sempre, às vezes, nuca, entre outros.
Essa frequência pode ser demonstrada pelos advérbios: daily (diariamente), weekly
(semanalmente), monthly (mensalmente), yearly (anualmente).

AULA 04 – ADJECTIVES AND ADVERBS 18


TEACHER ANDREA BELO

Bethy goes to the market weekly – Bethy vai ao mercado semanalmente)


Bethy reads her 3 favorite books yearly – Bethy lê seus 3 livros favoritos anualmente)

As formas mais comuns de se ver advérbios de frequência em frases é quando podemos


representar a frequência por porcentagem, como alguns livros fazem – 100% para always
(sempre), 90% para usually e often (frequentemente), 50% para sometimes (às vezes), 30% para
occasionally (ocasionalmente/eventualmente), 10% ou menos para rarely e seldom (raramente)
e 0% para never (nunca). Vejamos em frases:
Bethy always talks on the phone – Bethy sempre fala ao telefone.
Bethy usually talks on the phone – Bethy frequentemente fala ao telefone.
Bethy sometimes talks on the phone – Bethy às vezes fala ao telefone.
Bethy ocasionally talks on the phone – Bethy eventualmente fala ao telefone.
Bethy rarely talks on the phone – Bethy raramente fala ao telefone.
Bethy never talks on the phone – Bethy nunca fala ao telefone.

Como você percebeu, os advérbios de frequência estão antes de um verbo principal. Mas,
eles podem variar a posição em que se encontram e não prejudicar o significado nem o sentido
das frases. Vejamos as frases acima, com troca de posições.

Bethy talks on the phone, always – Bethy fala ao telefone, sempre.


Usually, Bethy talks on the phone – Frequentemente, Bethy fala ao telefone.
Sometimes, Bethy talks on the phone – Às vezes, Bethy fala ao telefone.

Agora, vamos analisar os advérbios de tempo e como usá-los.

ADVÉRBIOS DE TEMPO
Os adverbs of time (advérbios de tempo) indicam quando uma ação ocorreu e não
precisam da terminação -ly.
Geralmente, demonstram e caracterizam algo que aconteceu no tempo passado ou no
tempo futuro, enfatizando frases tanto no Past Simple como do tempo Future Simple.
Os advérbios de tempo, na maioria das vezes, aparecem no final das frases. Alguns deles
são:

last week / last month / last year = semana passada / mês passado / ano passado.
next week / next month / next year = semana que vem / mês que vem / ano que vem.
OU
próxima semana, próximo mês, próximo ano.

AULA 04 – ADJECTIVES AND ADVERBS 19


TEACHER ANDREA BELO

She traveled last week. She will travel next week.


Ela viajou semana passada. Ela vai viajar na próxima semana.

ADVÉRBIOS DE LUGAR
Os adverbs of place (advérbios de lugar) indicam, como o nome diz, a localização onde uma
ação aconteceu e eles não precisam da terminação -ly.
Podem aparecer após o verbo, na frente das orações, antes do sujeito, entre outras
posições. Vejamos exemplos pois há muitos casos e vou explicando a você, um a um.
Bethy walks everywhere – Bethy anda por todos os lugares/por toda parte.

Os advérbios here e there (aqui e lá) são muito comuns em frases para indicar lugar.
Come here! (Venha aqui!), para dizer “Venha até mim.”
The table is in here. (A mesa está aqui), para dizer “Quero mostrar a você que a mesa está aqui.”
Put it there. (Coloque isso lá!), para dizer “Leve isso para lá, para longe daqui.”
The table is in there. (A mesa está lá.) para dizer “Pode ir, a mesa já está lá.”

AULA 04 – ADJECTIVES AND ADVERBS 20


TEACHER ANDREA BELO

Alguns advérbios são, ao mesmo tempo, advérbios e preposições. Vejamos exemplos:

ADVÉRBIO USADO COMO ADVÉRBIO: ADVÉRBIO USADO COMO PREPOSIÇÃO:

The necklace rolled around in my fingers. I am wearing a necklace around my neck.


around
O colar se enrolou entre os meus dedos. Estou usando um colar em volta do pescoço.

Hurry! You are getting behind. Let's hide behind the door.
behind
Depressa! Você está ficando para trás. Vamos nos esconder atrás da porta.

Bethy fell down. Bethy made her way down the hill.
down
Bethy caiu. (para baixo) Bethy fez o caminho morro abaixo.

We decided to drop in on Bethy. I dropped the letter in the box.


in
Decidimos visitar Bethy sem avisar. Eu coloquei a carta dentro da caixa.

Let's get off at the next stop. She put the flowers off the vase.
off
Vamos descer na próxima parada. Ela colocou as flores fora do vaso.

She rode on for two more hours. Put the books on the table.
on
Ela rodou por mais duas horas. Coloque os livros na mesa.

She turned over and went back to sleep. I think I will hang the picture over my table.
over
Ela se virou e voltou a dormir. Acho que vou colocar o quadro acima da mesa.

Há advérbios de lugar que terminam em -where, para indicar um lugar específico, como:
I would like to go somewhere hot. (Eu gostaria de ir em algum lugar quente.)
Is there anywhere I can find a drugstore? (Há algum lugar em que eu encontre uma drogaria?
I have nowhere to go. (Eu não tenho lugar nenhum para ir.)
I keep looking for you everywhere! (Eu continuo procurando por você em todos os lugares.)

Outros advérbios de lugar terminam em -wards, para expressar movimentos em um lugar


em particular, um lugar, uma direção específica:
Cats don't usually walk backwards. (Gatos geralmente não andam para trás.)
The ship sailed westwards. (O navio foi para o oeste/direção ocidental.)
The balloon drifted upwards. (O balão foi levado para cima.)
Let’s walk homewards until we arrive. (Vamos andar de volta para casa.)

AULA 04 – ADJECTIVES AND ADVERBS 21


TEACHER ANDREA BELO

The balloon drifted upwards.


O balão foi levado para cima.

Agora, vamos estudar alguns advérbios classificados como de dúvida ou de certeza.

ADVÉRBIOS DE DÚVIDA E/OU CERTEZA(a)


Os nomes dos advérbios já indicam, em todos os subtítulos, sobre o que se tratam. Assim,
os adverbs of doubt or certainty (advérbios de dúvida ou certeza), vão justamente indicar o grau
de dúvida ou de certeza de algo que aconteceu, acontece ou acontecerá.
Esses advérbios aparecem com a terminação -ly. Exemplos:
Bethy maybe dance tonight – Bethy talvez dance essa noite.

Bethy will clearly dance tonight – Bethy vai, sem dúvida, dançar essa noite.

Além de maybe, outros advérbios nessa categoria:


assuredly (indubitavelmente, sem dúvidas)
certainly (certamente, seguramente, evidentemente)
clearly (claramente, sem dúvidas, evidentemente)
definitely (definitivamente)
maybe (talvez)
perhaps (talvez – no início ou no final da frase)
possibly (possivelmente)
probably (provavelmente)
Entre os advérbios de dúvida e certeza, estão os que demonstram um ponto de vista. São
os advérbios que expressam a opinião do sujeito, o que ele pensa sobre a situação da frase. Esses
advérbios, por sua vez, também aparecem com a terminação -ly, comum na classe adverbial.
Bethy unluckily lost the game – Bethy por azar perdeu o jogo.
O sujeito da frase está expressando, através do advérbio unluckly, a ideia da falta de
sorte de Bethy em relação ao jogo. E, assim, a tradução do advérbio é “por azar”.

AULA 04 – ADJECTIVES AND ADVERBS 22


TEACHER ANDREA BELO

Outros advérbios que evidenciam pontos de vista:


bravely (corajosamente)
carelessly (de forma desleixada, negligente)
cleverly (inteligentemente)
confidentially (confidencialmente)
disappointingly (de modo decepcionante, desapontador)
foolishly (de forma tola, insensata)
happily (por sorte, felizmente)
kindly (gentilmente, de bom grado)
luckily (por sorte)
naturally (naturalmente)
obviously (obviamente)
personally (pessoalmente)
presumably (presumivelmente)
rightly (com razão)
seriously (seriamente)
stupidly (estupidamente, de modo imbecil)
theoretically (teoricamente)
truthfully (na verdade)
unbelievably (inacreditavelmente)
unfortunately (infelizmente)
unluckily (por azar)
wisely (sabiamente)
presumably (presumivelmente)
technically (tecnicamente)

Já que esses advérbios exibem pontos de vista, a característica principal deles é que seja
um comentário com caráter opinativo.
Eles modificam a oração inteira, e não apenas palavras isoladamente:
Fortunately, Bethy decided to help us.
(Felizmente, Bethy decidiu nos ajudar), em que fortunately: ponto de vista.
Stupidly, I forgot my keys.
(Estupidamente/de forma estúpida, eu esqueci minhas chaves), em que stupidly: ponto de vista.

AULA 04 – ADJECTIVES AND ADVERBS 23


TEACHER ANDREA BELO

Fortunately, my friend decided to help me when I fell down in front of everybody.


(Felizmente, meu amigo decidiu me ajudar quando caí em frente todos.)

ADVÉRBIOS DE INTENSIDADE
De uma forma geral, os adjetivos e advérbios possuem funções parecidas. Mas, enquanto
os adjetivos tratam de substantivos (como pessoas e coisas são), os advérbios tratam, quase
sempre, dos verbos (como as ações são feitas).
No entanto, é possível utilizar os advérbios para modificar os adjetivos. São os chamados
modifiers (modificadores) e são utilizados antes dos adjetivos para enfatizar a intensidade em
relação àquela qualidade:
absolutely [absolutamente]
extremely [extremamente]
really [realmente]
so [tão]
such [tão]
too/very [muito, demais]

This cake was very delicious but extremely expensive.


Esse bolo estava muito delicioso, mas extremamente caro.

Os advérbios intensificadores, além de permitir que o sujeito se expresse com mais clareza,
também proporciona às frases uma sensação mais natural. Esses advérbios são uma ferramenta
importante, já que são uma forma essencial de mostrar emoção ou alcance de uma emoção.
Esses advérbios modificadores de frases, fornecem contexto emocional a elas e fortalecem
os seus significados, já que diz que algo foi feito extremante, incrivelmente etc.
Alguns desses advérbios podem aumentam a intensidade negativamente também. As
frases soam negativas, mas elas existem: She made the report awfully - Ela fez o relatório
terrivelmente ou insanely (insanamente) e terribly (terrivelmente).
Esses modificadores, por sua vez, dão força às palavras que elas modificam, só que com
conotação negativa, para intencionalmente, dar a ideia de gravidade em algo que talvez poderia
ser entendido como normal ou cotidiano.

AULA 04 – ADJECTIVES AND ADVERBS 24


TEACHER ANDREA BELO

Bethy is dreadfully sorry. (Bethy está terrivelmente arrependida)


Perceba que, nesse exemplo, o advérbio de intensidade reforça a ideia de que
quer fazer um pedido de desculpas sobre algo que aconteceu. Com o acréscimo do
advérbio de intensidade dreadfully, a frase ficou muito mais forte e impactante.

Voltando a falar de advérbios de intensidade em geral, veja outros que já apareceram e


podem aparecer novamente em textos das provas:
Outros intensificadores comuns:
amazingly (surpreendentemente)
at all (em absoluto)
especially (especialmente)
extraordinarily (extraordinariamente)
outrageously (escandalosamente)
phenomenally (fenomenalmente)
remarkably (notavelmente)
terribly (terrivelmente)
totally (totalmente)
unusually (incomumente)

She is remarkable the champion of the competition.


(Ela é notavelmente a campeã da competição.)

AULA 04 – ADJECTIVES AND ADVERBS 25


TEACHER ANDREA BELO

LOCUÇÕES ADVERBIAIS
O que é uma locução adverbial? Locução adverbial é um conjunto de duas ou mais palavras
que desempenham a função de advérbio. Essas locuções adverbiais se formam com uma
preposição, às vezes uma preposição + substantivo ou preposição + adjetivo ou ainda preposição
+ advérbio.
As locuções adverbiais são expressões em que, quando o conjunto de duas ou mais
palavras são agrupadas, desempenham a função de um advérbio e pode alterar o sentido desse
verbo, ou de um adjetivo ou até mesmo de outro advérbio. Vejamos exemplos:
Algumas locuções adverbiais que aparecem nos textos das provas:
a só/a sós – lonely
à toa – occasionally
à vontade – at will, freely
absolutamente – not at all
ao acaso – without consideration
ao contrário – in contrary
às avessas – just the opposite
às claras – openly, directly
às direitas – straightforward
às pressas – fast
de bom grado – of good will
de cor – by heart
de fato, na verdade – in fact
de má vontade – unwillingly
é claro – of course
em geral – generally
em silêncio – silently
por via das dúvidas – just in case
sem dúvida – no doubt
again and again – repedidamente
arm in arm – de braços dados
at first – a principio
at most – no máximo
at once – imediatamente
at random – ao acaso
by and large – em geral
day by day – dia após dia
fairly well – razoavelmente
far and near/far and wide – em toda parte
from now on – de hoje em diante
hand in hand -de mãos dadas
hardly ever – quase nunca
head over hills – de cabeça para baixo
in full – por extenso
likely – muito provável
little by little – pouco a pouco
once in a while – de vez em quando
sooner or late – mais cedo ou mais tarde
through and through– por complete
to a certain extent – até certo ponto

AULA 04 – ADJECTIVES AND ADVERBS 26


TEACHER ANDREA BELO

Agora, veremos uma questão com vários e diferentes advérbios.


Note que, algumas vezes, ao olhar apenas uma vez no texto, fazer uma leitura dinâmica,
com agilidade para poupar seu tempo, advérbios confundem-nos com adjetivos e vice-versa.
Por esse motivo, essa aula explora os dois assuntos: para que você compreenda claramente
para não ter dúvidas na hora da prova. Vamos lá!

FRAYING AT THE EDGES: A LIFE-CHANGING DIAGNOSIS


IT BEGAN WITH what she saw in the bathroom mirror. On a dull morning, Geri Taylor padded
into the shiny bathroom of her Manhattan apartment. She casually checked her reflection in the
mirror, doing her daily inventory. Immediately, she stiffened with fright.
Huh? What?
She didn‘t recognize herself.
She gazed saucer-eyed at her image, thinking: Oh, is this what I look like? No, that‘s not me.
Who‘s that in my mirror?
This was in late 2012. She was 69, in her early months getting familiar with retirement. For some
time she had experienced the sensation of clouds coming over her, mantling thought. There had
been a few hiccups at her job. She had been a nurse who climbed the rungs to health care
executive. Once, she was leading a staff meeting when she had no idea what she was talking
about, her mind like a stalled engine that wouldn‘t turn over.
"Fortunately I was the boss and I just said, Enough of that; Sally, tell me what you‘re up to," she
would say of the episode.
Certain mundane tasks stumped her. She told her husband, Jim Taylor, that the blind in the
bedroom was broken. He showed her she was pulling the wrong cord. Kept happening. Finally,
nothing else working, he scribbled on the adjacent wall which cord was which.
Then there was the day she got off the subway at 14th Street and Seventh Avenue unable to figure
out why she was there.
So, yes, she had had inklings that something was going wrong with her mind. She held tight to
these thoughts. She even hid her suspicions from Mr. Taylor, who chalked up her thinning
memory to the infirmities of age.
"I thought she was getting like me," he said. "I had been forgetful for 10 years".
But to not recognize her own face! To Ms. Taylor, this was the "drop-dead moment" when she
had to accept a terrible truth. She wasn‘t just seeing the twitches of aging but the early fumes of
the disease.
She had no further issues with mirrors, but there was no ignoring that something important had
happened. She confided her fears to her husband and made an appointment with a neurologist.

AULA 04 – ADJECTIVES AND ADVERBS 27


TEACHER ANDREA BELO

"Before then I thought I could fake it,' she would explain. "This convinced me I had to come
clean." In November 2012, she saw the neurologist who was treating her migraines. He listened
to her symptoms, took blood, gave her the Mini Mental State Examination, a standard cognitive
test made up of a set of unremarkable questions and commands. (For instance, she was asked to
count backward from 100 in intervals of seven; she had to say the phrase: "No ifs, ands or buts";
she was told to pick up a piece of paper, fold it in half and place it on the floor beside her.)
He told her three common words, said he was going to ask her them in a little bit. He emphasized
this by pointing a finger at his head — remember those words. That simple. Yet when he called
for them, she knew only one: Beach. In her mind, she would go on to associate it with the doctor,
thinking of him as Dr. Beach.
He gave a diagnosis of mild cognitive impairment, a common precursor to Alzheimer‘s disease.
The first label put on what she had. Even then, she understood it was the footfall of what would
come. Alzheimer‘s had struck her father, a paternal aunt and a cousin. She long suspected it would
eventually find her.
Fonte: http://www.nytimes.com/interactive/2016/05/01/nyregion/living-with-alzheimers.html?action=click&contentCollection=Americas&module= Trending&version=Full®ion= Marginalia&pgtype=article. (acesso em 1/05/2016).

Questão – Marque a opção em que o item sublinhado NÃO é um advérbio.


a) She casually checked her reflection in the mirror, [...] (linha 2)
b) “Fortunately, I was the boss and I just said, [...] (linha 13)
c) Finally, nothing else working, he scribbled on the adjacent wall which cord was which.
(linhas 16-17)
d) She wasn’t just seeing the twitches of aging but the early fumes of the disease. (linha 25)
e) She long suspected it would eventually find her. (linha 40)
Comentários: Na letra A, casually possui -ly em sua estrutura, terminação de advérbio e está
exercendo a função de advérbio na frase: o modo casual: casualmente.
Na letra B, fortunately também possui -ly em sua estrutura, terminação de advérbio e está
exercendo a função de advérbio na frase: felizmente.
Na letra C, finally conta com -ly em sua estrutura, terminação de advérbio e está exercendo a
função de advérbio na frase: finalmente.
Na letra D, early possui -ly como terminação, mas é a palavra cedo, em Inglês, contrário de late.
Até pode ser usado como advérbio de tempo (alguém chegou cedo ou tarde) mas aqui, é um
adjetivo: “She wasn’t just seeing the twitches of aging but the early fumes of the disease”: “Ela
não estava apenas vendo as contrações do envelhecimento, mas as emanações precoces da
doença”. Alternativa que não é advérbio: letra D
Na letra E, eventually possui -ly em sua estrutura, terminação de advérbio e está exercendo a
função de advérbio na frase: eventualmente.

AULA 04 – ADJECTIVES AND ADVERBS 28


TEACHER ANDREA BELO

ADVÉRBIOS: GRAU COMPARATIVO


Assim como os adjetivos, os advérbios possuem grau comparativo e superlativo. Vejamos,
primeiramente, como se faz o grau comparativo, mas, desde já digo a você: é mais simples nos
advérbios do que nos adjetivos – e que bom, então, certo?

COMPARATIVO DE IGUALDADE
O comparative of equality – comparativo de igualdade nos advérbios tem a estrutura
seguinte: as + advérbio + as, significando: tanto/tão...quanto/como. Veja:
Bethy drives as carefully as Tom. – Bethy dirige tão cuidadosamente quanto Tom.
.
Does Bethy study as frequently as Tom? – A Bethy estuda tão frequentemente quanto Tom?

Na forma negativa, apenas acrescenta-se not, ficando: not as/not so + advérbio + as,
significando – não tão...como/quanto:
Bethy is not as carefully as Tom. – Bethy não é tão cuidadosamente quanto Tom.
.
E, para frases em outros tempos verbais, sem utilizar o verbo to be, conjuga-se a forma
negativa normalmente – como você aprendeu na aula de verbos – e a estrutura as/as:
Bethy doesn’t drive as carefully as Tom. – Bethy não dirige tão cuidadosamente quanto Tom.
.

COMPARATIVO DE SUPERIORIDADE
O comparative of superiority – comparativo de superioridade tem diferentes estruturas
quando elaborado com advérbios considerados curtos ou longos de acordo com as letras/sílabas,
lembra?

AULA 04 – ADJECTIVES AND ADVERBS 29


TEACHER ANDREA BELO

Eles têm esse formato: more + advérbio + than, significando: mais/do que. Veja:
Bethy travels more frequently than Tom. – Bethy viaja mais frequentemente do que Tom.
.
Does Bethy study more frequently than Tom? – Bethy estuda mais frequentemente do que Tom?

Na forma negativa, apenas acrescenta-se not no verbo to be ou conjuga-se a forma


negativa do Present Simple normalmente – como você aprendeu na aula de verbos:
Bethy doesn’t swim more easily than Tom. – Bethy não nada mais facilmente do que Tom.
.
Se o advérbio for um daqueles que fica igual, idêntico ao adjetivo em sua estrutura, como
vimos o adjetivo e o advérbio alto – high, por exemplo, segue a regra do adjetivo: -er no final do
advérbio + than:
Peter talks higher than Tom. – Peter fala mais alto do que Tom.
.

COMPARATIVO DE INFERIORIDADE
O comparative of inferiority – também tem diferentes estruturas quando elaborado com
advérbios considerados curtos ou longos de acordo com as letras/sílabas, lembra?
Eles têm esse formato: less + advérbio + than, significando: mais/do que. Veja:
Bethy travels less frequently than Tom. – Bethy viaja menos frequentemente do que Tom.
.
Does Bethy study less frequently than Tom? - Bethy estuda menos frequentemente do que Tom?

Na forma negativa, apenas acrescenta-se not no verbo to be ou conjuga-se a forma


negativa do Present Simple normalmente – como você aprendeu na aula de verbos:
Bethy doesn’t swim less easily than Tom. – Bethy não nada menos facilmente do que Tom.
.

AULA 04 – ADJECTIVES AND ADVERBS 30


TEACHER ANDREA BELO

As traduções ficam “estranhas”, não ficam? Por esse motivo, prefere-se usar o comparativo
de igualdade na forma negativa para, ao invés de “desmerecer” algo ou alguém, se diz que
algo/alguém não pratica uma ação tão bem quanto outro.
Assim, fica mais “educado” e mais adequado. Veja a última comparação acima, usada com
comparativo de igualdade no lugar de inferioridade:
Bethy doesn’t swim as easily as Jane. – Bethy não nada tão facilmente quanto Jane.
.

Vejamos agora, o grau superlativo nos advérbios

ADVÉRBIOS: GRAU SUPERLATIVO


O grau superlativo é usado para demonstrar que algo dentro de um grupo se destaca, isto
é, alcança o grau máximo no aspecto em que é comparado.
No caso do grau superlativo para advérbios, temos as estruturas que usam expressões tais
como “o(a) mais” e também “o (a) menos”, para superioridade ou inferioridade.

SUPERLATIVO DE SUPERIORIDADE
Para o superlativo de superioridade, usamos the most + advérbio ou -est para advérbios
curtos, que coincidem com os adjetivos, aqueles em que se usa a mesma palavra para ambas as
classes gramaticais como vimos antes, lembra? Vejamos exemplos de ambos os casos:
Bethy does her homework the most quickly. – Bethy é a faz a tarefa de casa mais rapidamente. (que todos).

Bethy talks the loudest in her classroom. – Bethy é a que fala mais alto em sua sala.
.
Em ambas as frases, percebemos que poderíamos traduzir assim: “mais (...) de todos”,
porque o superlativo é justamente isso: mostrar que algo/alguém é mais, o máximo em alguma
coisa, incomparável a outros.

AULA 04 – ADJECTIVES AND ADVERBS 31


TEACHER ANDREA BELO

She talks the loudest in her classroom. – Ela é a que fala mais alto em sua sala

SUPERLATIVO DE INFERIORIDADE
Para o superlativo de inferioridade dos advérbios, no lugar de the most, usa-se the least
antes do advérbio da frase.
Bethy drives the least carefully. – Bethy é a que dirige menos cuidadosamente.
.
No superlativo de advérbios, há algumas formas comparativas e superlativas irregulares,
cujos advérbios não seguem as regras apresentadas. Os exemplos que aparecem nas provas são
the best, the worst e, o que usamos, the furthest (o melhor, o pior e o mais longe) – sempre com
o sentido de ser “o mais alguma coisa de todos.”
Vimos no comparativo. Agora, vejamos no superlativo:
I walked seven blocks, my friends walked two, some of them four but I walked the furthest.
Eu caminhei sete quarteirões, meus amigos caminharam dois, alguns quatro quarteirões,
mas eu (fui o que) caminhei mais longe. (significando mais longe de todos).

She runs the furthest on the competition. – Ela é a que corre mais longe na competição.

AULA 04 – ADJECTIVES AND ADVERBS 32


TEACHER ANDREA BELO

QUESTÕES
Você vai, agora, responder questões selecionadas de provas já realizadas em anos
anteriores. Depois, como em todas as nossas aulas, haverá o gabarito e as questões comentadas.
Vamos começar com questões AFA, de acordo coma sua instituição escolhida e depois,
vamos treinar de outras Carreiras Militares, para adquirir experiência e treinar vocabulário.
QUESTÕES AFA
Directions: Read the text below and answer questions 01 to 15 according to it.
How to cope with parent guilt, during the pandemic and beyond
As a mother, I’m vulnerable to the influence of our cultural messaging, and I have fallen prey to the
“We can never give enough, do enough or be enough” narrative. As a psychologist, though, I know
how risky this constant thread of guilt is for our well-being. Guilt can be helpful as an uncomfortable
emotion that motivates us to make amends and change hurtful behaviors, but in the case of parent
guilt, much of the time we are only hurting ourselves.
Enter a global pandemic that has shattered our already precarious parenting lives and provided ample
evidence of our systems’ failures. Employers are expecting our remote-learning children to behave
during our work day, and women are leaving jobs in record numbers for their suddenly homebound
children, because they “just can’t do it all anymore.” Ilyse DiMarco, a clinical psychologist and the
author of the upcoming book “Mom Brain,” sums up the problem: “The issue with guilt right now is
there is potential guilt with anything you do.”
Stress levels have risen, with parents outpacing nonparents in surveys, creating fertile ground for
mental health problems. “We’re feeling like we’re not measuring up in some area or maybe we’re not
making the right choices,” DiMarco says. “Not surprisingly, if we feel like we’re failing, we feel
depressed; if we’re worried about not making the right decision, we feel anxious.”
Daily life in a pandemic has given us a host of new reasons to feel guilty. In addition to the impossibility
of simultaneously working, parenting and, in some cases, teaching our children, we face daily
decisions around health and safety.
Do we allow play dates? What if all of my child’s friends are playing basketball and I say no? What if
we choose inperson school and our child gets covid-19, or we choose remote school and our child
becomes depressed?
Working moms are not okay. No good choices and no good answers, but guaranteed guilt, with a
sprinkling of judgment caused by social pressures.
“Guilt is a useful emotion when it tells us we’ve done something wrong,” says clinical psychologist Jill
Stoddard, author of “Be Mighty.” “What’s happening now is we feel like we’ve done something wrong
even when we’re doing the very best we can.”
So, what can we do about the guilt hanging heavy around our collective necks? Experts suggest a
combination of mindfulness, meaningful self-care and shifting our perspective.
Mindfulness
Do not ignore the guilt; acknowledging it means you can do something about it. Even if our pandemic-
related guilt may be misplaced, Stoddard says we can still use the feeling for its positive function and

AULA 04 – ADJECTIVES AND ADVERBS 33


TEACHER ANDREA BELO

make amends. She has told her children, “I really wish I could spend more time with you guys, and it’s
been so hard on all of us. Things will be different someday. I’m sorry I can’t be there for you more.”
She adds: “You’re not saying, ‘I screwed up.’ You’re saying, ‘It hurts my heart I can’t do this the way I
wish I could right now.’ ”
Remember you are not alone. Feelings of failure are a universal experience of parents during the
coronavirus pandemic. Seeking social support is critical for mental health in times of stress. It can be
as simple as texting a friend to share “Mom fails” from your day. Even this brief connection offers
support and solidarity, and it can decrease our sense of self-blame and inadequacy when we hear
others’ similar feelings and experiences.
Practice self-kindness. Talk to yourself as you would to your friend: “You are doing your very best in
this moment.” This practice helps shift thinking patterns from self-criticism to self-compassion, which
is known to increase positive emotions and decrease negative ones, including guilt.
Real self-care
Prioritize the daily demands on your time and energy. DiMarco advises doing this by asking, “What’s
most important here, on a day-to-day or hour-to-hour basis?” Focus on what is highest on the priority
list, and allow yourself to let go of the other potential tasks instead of feeling guilty for, inevitably,
not getting everything done.
Prioritize yourself. “An important piece, and counterintuitive piece, to manage guilt about not being
there for everybody else, is to also be there for yourself, and make sure you’re on the list somewhere,”
DiMarco says. Plan ahead to ensure time for you in the daily routine, even if it’s just 15 to 30 minutes.
This may require an agreement with your partner and/or kids about when you will not be available
the next day. Be realistic about the amount of time you need and how you will spend it (e.g., 15
minutes on a meditation app). Then, stay committed to making it happen.
Remember the basics. We make sure our kids are fed, but are we sleeping, drinking enough water,
and eating well enough to meet our own basic needs? These stress management essentials
strengthen us, which will help us ward off the guilt.
Shift focus
Adjust expectations. “If you expect yourself to stay calm and never lose it, you’re setting yourself up
for failure,” says child and adolescent psychologist Emily W. King. “Own it that you’re more
emotionally fragile right now, notice it and walk outside for 10 minutes, or whatever you need, instead
of getting in a negative feedback loop, and then you’re emotionally exhausted.”
Consider your successes. Flip the script so you’re not focusing only on the negative. What do you feel
proud of? What can your children do now they couldn’t do a year ago? Redefine success for you and
your children dur ing a global pandemic. “Success is not about grades but about independence or
chores,” says King. For parents, success can be getting through each day with everyone sheltered, fed
and in bed safely.
Invent new narratives. Instead of looking at yourself as being never enough, how about
acknowledging that you have been — and continue to be — enough to endure a global pandemic
while parenting? What recent generation of parents has accomplished this? In fact, we are not just
enough, we are so much greater. Remind yourself of this when parent guilt attempts to tread that
familiar path, both now and after the pandemic.
(Adapted from https://www.washingtonpost.com/lifestyle/2021/02/25/how-cope-with-parent-guilt-during-pandemic-beyond/)

AULA 04 – ADJECTIVES AND ADVERBS 34


TEACHER ANDREA BELO

Questão 01 (AFA/INÉDITA) – The text


a) aims at highlighting how to deal with remote-learning children during the pandemic.
b) provides the reader with a heartfelt narrative of a father about his experience during the
pandemic.
c) focus on a considerable set of psychological implications of pandemic on children.
d) concerns mainly about the mental state of mothers during the pandemic due to the guilt they
feel.

Questão 02 (AFA/INÉDITA) – In the sentence “Guilt can be helpful as an uncomfortable emotion


that motivates us to make amends and change hurtful behaviors,” (paragraph 01), it is possible
to find an option to substitute the highlighted pronoun accordingly in
a) which.
b) who.
c) how.
d) when.

Questão 03 (AFA/INÉDITA) – In the fragment “because they ‘just can’t do it all anymore.’”
(paragraph 02), the highlighted word refers to
a) children
b) jobs
c) women
d) numbers

Questão 04 (AFA/INÉDITA) – In the eighth paragraph, the author


a) blames the guilt working moms feel on fathers and employers.
b) starts reinforcing the importance of denying guilt.
c) tries to make controversial issues about guilt clear.
d) introduces the topic of what can be done about the guilt moms feel.

Questão 05 (AFA/INÉDITA) – Mark the option which shows the best answer for the question
“So, what can we do about the guilt hanging heavy around our collective necks?” (paragraph 8)
a) nothing more than mindfulness.
b) a merging of different strategies.
c) ignoring the guilt.
d) only meaningful self-care.

AULA 04 – ADJECTIVES AND ADVERBS 35


TEACHER ANDREA BELO

Questão 06 (AFA/INÉDITA) – In the sentence “She has told her children, ‘I really wish I could
spend more time with you guys, and it’s been so hard on all of us.’” (paragraph 09), the
contraction refers to
a) It is.
b) It has.
c) It was.
d) It will.

Questão 07 (AFA/INÉDITA) – Mark the option which shows the appropriate question tag for the
sentence “I’m vulnerable to the influence of our cultural messaging” (paragraph 01).
a) I’m not?
b) Am I?
c) Aren’t I?
d) Am I not?

Questão 08 (AFA/INÉDITA) – Mark the option that shows a synonym for the underlined
expression in “These stress management essentials strengthen us, which will help us ward off
the guilt.” (paragraph 14).
a) repel
b) contradict
c) forbid
d) accept

Questão 09 (AFA/INÉDITA) – Choose the best option to change the sentence “we are … hurting
ourselves.” (paragraph 01), into the passive form. Ourselves _______________ by us.
a) are being hurt
b) are hurting
c) have been hurt
d) is been hurt

Questão 10 (AFA/INÉDITA) – In the third paragraph, it’s said that


a) parents are feeling as stressed as nonparents according to surveys.
b) stress levels have risen more in nonparents compared to parents.
c) nonparents are outpacing parents in stress levels in surveys.
d) surveys show parents are now more affected by stress than nonparents.

AULA 04 – ADJECTIVES AND ADVERBS 36


TEACHER ANDREA BELO

Questão 11 (AFA/INÉDITA) – Mark the alternative in which the problems described in paragraph
2 are correctly summarized.
a) employers are expecting remote-learning children to behave during work day.
b) right now, there is potential guilt with anything moms do.
c) women are leaving jobs in record numbers.
d) parenting lives were not precarious before the pandemic.

Questão 12 (AFA/INÉDITA) – Mark the option that contains the correct negative form for the
sentence “Stress levels have risen” (paragraph 3).
a) Stress levels not have risen.
b) Stress levels do not have risen.
c) Stress levels have not risen.
d) Stress levels have risen not.

Questão 13 (AFA/INÉDITA) – The underlined expression “Then, stay committed to making it


happen.” (paragraph 13) means
a) stand still.
b) create a commission.
c) keep dedicated.
d) let go.

Questão 14 (AFA/INÉDITA) – Mark the option which shows the appropriate plural form for the
word “child” (paragraph 5).
a) childs.
b) childrens.
c) childen.
d) children.

Questão 15 (AFA/INÉDITA) – The expression “our already precarious parenting lives”


(paragraph 2) suggests that parenting lives
a) were unstable even before the pandemic.
b) became precarious after the pandemic.
c) were unlikely to collapse before the pandemic.
d) were steady before the pandemic.

AULA 04 – ADJECTIVES AND ADVERBS 37


TEACHER ANDREA BELO

QUESTÕES COLÉGIO NAVAL


Read Text I to do questions 01 to 04 based on it.
A wristband that tells your boss if you are unhappy
At first glance the silicone wristband could be mistaken for one that tracks your heart rate when
you are doing exercise.
However, the wearable technology, called a Moodbeam, isn't here to monitor your physical health.
Instead it allows your employer to track your emotional state.
The gadget, which links to a mobile phone app and web interface, has two buttons, one yellow
and one blue. The idea is that you press the yellow one if you are feeling happy, and the blue one
if you are sad.
Aimed at companies who wish to monitor the wellbeing of staff who are working from home, the
idea is that employees are encouraged to wear the wristband (they can say no), and press the
relevant button as they see fit throughout the working week.
Managers can then view an online dashboard to see how workers are feeling and coping. With
bosses no longer able to check in physically with their team, Moodbeam hopes to bridge the gap.
"Businesses are trying to get on top of staying connected with staff working from home. Here they
can ask 500 members: 'You ok?' without picking up the phone," says Moodbeam co-founder
Christina Colmer McHugh.
She originally came up with the idea for the product after she discovered that her daughter was
struggling at school, and she wanted a way for her child to let her know how she was feeling. The
wristband was launched commercially in 2016.
(Adapted from https://www.bbc.com/news/business-55637328)

Questão 01 (COLÉGIO NAVAL/INÉDITA) – Read the extract from the text.


“At first glance the silicone wristband could be mistaken for one that tracks your heart rate when
you are doing exercise”.
Mark the alternative that can replace the underlined word without changing its meaning.
(A) Peek
(B) Ignore
(C) Touch
(D) Stare
(E) Eye

Questão 02 (COLÉGIO NAVAL/INÉDITA) – About the initial purpose of the wristband, it’s correct
to affirm that
(A) Fulfilled the final results
(B) Had a personal character
(C) Aimed at analyzing the mental health of the entire population
(D) Criticized the current teaching method of schools
(E) Had no purpose, considering it was a sudden discovery

AULA 04 – ADJECTIVES AND ADVERBS 38


TEACHER ANDREA BELO

Questão 03 (COLÉGIO NAVAL/INÉDITA) – Read the extract from the text


“The wristband was launched commercially in 2016”
The sentence above means that
(A) The gadget was created in 2016
(B) The gadget has been on the market for over a decade
(C) The gadget was not launched on the market
(D) The gadget won over consumers in 2016
(E) The gadget was effectively placed on the market in 2016

Questão 04 (COLÉGIO NAVAL/INÉDITA) – What can we infer from the text? Mark the correct
option
(A) The final use of the gadget was for the parents’ emotional control over their children
(B) The gadget enabled remote team control
(C) The gadget system is very complex
(D) Companies tend not to care about the emotional health of employees
(E) The wristband is a gadget that does not adapt to the context of a pandemic

Read the text II to do items 05 to 10.


Vegan Restaurant Gets Michelin Star in France, a First
Lemongrass, seaweed and fir — yes, the tree — are not the sorts of ingredients that once earned
French chefs plaudits in the Michelin Guide, but on Monday the thick red bible of gastronomy
announced that it was giving its first star to a fully vegan restaurant in France.
The bestowal of the star to ONA, a restaurant near Bordeaux, is more evidence that a country long
renown for classics like coq au vin, blanquette de veau and boeuf bourguignon has opened up to
animal-free cuisine. A growing number of chefs are cutting meat from their menus, sometimes
entirely.
“It’s a new movement in France, where diets are still very meat-based,” said Claire Vallée, the chef
at ONA, which opened five years ago in Arès, a small Atlantic Coast town about 25 miles west of
Bordeaux.
“Each has its place,” Ms. Vallée said. “We want to show that you can eat differently.”
The restaurant’s very name speaks to France’s shifting culinary landscape: It stands for origine
non-animale.
Vegan establishments have already received Michelin stars in the United States, Spain and
Germany. But this is a first for France.
ONA shuns all animal-based products, Ms. Vallée said, even in its decorations and furnishings. It
does not, for instance, use wool or leather.

AULA 04 – ADJECTIVES AND ADVERBS 39


TEACHER ANDREA BELO

Last fall, its seven-course menu featured dishes with intriguing combinations of fir, boletus
mushroom and sake, or dulse seaweed, lemongrass and galangal, a relative of ginger. The
restaurant is currently closed because of the coronavirus pandemic.
The Michelin Guide, in a statement this week, said Ms. Vallée had given vegan cuisine its “letters
of nobility.”
Gwendal Poullenec, the international head of the Michelin Guides, noted that the move away
from meat was not entirely new.
Alain Passard, the owner and chef at L’Arpège in Paris, removed meat from his menu two decades
ago, and Alain Ducasse, perhaps France’s biggest culinary superstar, also decided to drastically
reduce the amount of meat used at his flagship Parisian restaurant.
But awarding a star to a restaurant that is not just meatless but avowedly vegan has the potential
to shake things up even further, Mr. Poullenec said.
“The general public might not associate pure veganism with a gastronomical experience,” he said.
A Michelin star might “liberate” chefs who are still reluctant to explore plant-based cooking, he
said.
(Adapted from https://www.nytimes.com/2021/01/19/world/europe/ona-michelin-star-france.html?action=click&module=News&pgtype=Homepage)

Questão 05 (COLÉGIO NAVAL/INÉDITA) – According to the text, mark the INCORRECT option
(A) Lemongrass is an example of ingredient that normally does not predominantly compose
Michelin-starred restaurant dishes
(B) There is an entirely vegan restaurant, starring with the Michelin award, in France
(C) The majority of the French population is adept at the vegan movement
(D) There are chefs who have a meatless menu
(E) France isn’t the only country with Michelin-starred vegan restaurants

Questão 06 (COLÉGIO NAVAL/INÉDITA) – Read the extract from the text


“ONA shuns all animal-based products, Ms. Vallée said, even in its decorations and furnishings. It
does not, for instance, use wool or leather”
Mark the option that can replace the underlined sentence without changing its meaning
(A) ONA uses all animal-based products
(B) ONA sells all animal-based products
(C) ONA is indifferent to all animal-based products
(D) ONA despises all animal-based products
(E) ONA approves all animal-based products

AULA 04 – ADJECTIVES AND ADVERBS 40


TEACHER ANDREA BELO

Questão 07 (COLÉGIO NAVAL/INÉDITA) – Read the extract from the text


“It does not, for instance, use wool or leather” (Paragraph 7)
The word “It” refers to
(A) ONA
(B) Wool
(C) Leather
(D) Animal-based products
(E) Vegan establishments

Questão 08 (COLÉGIO NAVAL/INÉDITA) – Claire Vallée


(A) Does not identify herself with veganism
(B) Thinks that the vegan movement in France opposes the reality of French food
(C) Does not want to diversify food in France
(D) Has an animal-free menu, only
(E) Has no decor in her restaurant

Questão 09 (COLÉGIO NAVAL/INÉDITA) – The expression “this is a first”, in paragraph 6, means


(A) That something or someone won first place
(B) That something is indifferent to someone
(C) That something ordinary happened
(D) That something unexpected happened
(E) That something totally new has happened

Questão 10 (COLÉGIO NAVAL/INÉDITA) – The word “bestowal” (paragraph 2), can be replaced
by the word __________ without changing its meaning
(A) Withdrawal
(B) Grant
(C) Break
(D) Disagreement
(E) Give

AULA 04 – ADJECTIVES AND ADVERBS 41


TEACHER ANDREA BELO

QUESTÕES EAM
Read text I and answer questions 01 and 02
Why you’re more creative in coffee shops
If we’re already working in isolation at home, why do we miss working with our heads
similarly down in a public setting?
Some of the most successful people in history have done their best work in coffee shops.
Pablo Picasso, JK Rowling, Simone de Beauvoir and Jean-Paul Sartre, Bob Dylan – whether they’re
painters, singersongwriters, philosophers or writers, people across nations and centuries have
tapped into their creativity working away at a table in a café.
Of course, Covid-19 has put the kibosh on lingering for hours in cozy rooms packed with people
sipping lattes. As we begin another year living amid a pandemic, many of us continue to work
remotely on our own. And if remote work becomes permanent for some – as many experts predict
– we might ask ourselves why, when things settle down, we should bother going back out to work
in public, only to ostensibly isolate ourselves with our heads down – something we’re already
doing at home.
But putting on your noise-cancelling headphones to toil away at your desk is actually different
than doing the same surrounded by other people buzzing over your shoulder. There are many
ways coffee shops trigger our creativity in a way offices and homes don’t. Research shows that
the stimuli in these places make them effective environments to work; the combination of noise,
casual crowds and visual variety can give us just the right amount of distraction to help us be our
sharpest and most creative. (So, no, it’s not just that double espresso.)
A sweet spot of noise and crowds
Some of us stick in our earbuds as soon as we sit down to work in a public setting. But scientists
have known for years that background noise can benefit our creative thinking.
A 2012 study published in the Journal of Consumer Research showed that a low-to-moderate level
of ambient noise in a place like a cafeteria can actually boost your creative output. The idea is that
if you’re very slightly distracted from the task at hand by ambient stimuli, it boosts your abstract
thinking ability, which can lead to more creative idea generation.
(Adapted from https://www.bbc.com/worklife/article/20210114-why-youre-more-creative-in-coffee-shops)

Questão 01 (EAM/INÉDITA) – Say if the following statements are T (true) or F (false) about
coffee shops and the current world context. Then, mark the correct option, from top to bottom.
( ) Coffee shops are a good place to work
( ) Cafeterias are suitable workplaces for a certain type of profession
( ) Covid-19 did not change the work routine of most workers
( ) Remote work is a reality for many people, considering the current context
( ) It is possible that there is a general post-pandemic doubt among workers regarding the
workplace

AULA 04 – ADJECTIVES AND ADVERBS 42


TEACHER ANDREA BELO

(A) T – F – F – T – F
(B) T – F – F – F – F
(C) F – F – F – T – T
(D) F – F – F – F – T
(E) T – F – F – T – T

Questão 02 (EAM/INÉDITA) – About the benefits resulting from the cafeteria as a workplace, it
is correct to say that
(A) The place with a lot of noise ends up affecting the creative process of the worker
(B) Background noise pollution contributes to the development of work
(C) According to studies, noise pollution is one of the main reasons for the creative block
(D) The ability of abstract thinking is not affected by noise pollution
(E) The double espresso is the main reason why the coffee shop is a good place to work

Read text II and answer questions 03 and 04


Electric cars are not the only green solution
Chris Barker says more cars of any kind are bad news for pollution, while Dr Robin Russell-
Jones believes British entrepreneurs can rise to the challenge of ‘green growth’
It is indeed good news that the advance in battery technology brings down the costs and
inconveniences associated with electric cars. But it does not begin to solve the other problems
associated with the volume of traffic on the roads. In fact, if electric cars become cheaper and
continue to benefit from tax relief, the total number of cars may well rise. Electric cars produce
no toxic emissions at the point of use but, as with all cars, embodied carbon in the production of
vehicles and batteries generate greenhouse gas emissions. In addition, research by the
government in 2019 found that two-thirds of the contamination from conventional vehicles arises
from tyre, brake and road dust.
There are also other problems associated with too many cars of any type. The greater the level of
traffic, the greater the danger of death or injury and the need for more and wider roads, creating
no-go areas for people and cutting off communities from each other. Too much reliance on cars
promotes a sedentary lifestyle and brings health risks.
For a better life and to counteract the climate emergency it is necessary to reduce traffic. This
cannot be done by promoting electric cars. It needs the promotion of walking, cycling and public
transport, and of measures to reduce the need to travel.
(Adapted from https://www.theguardian.com/environment/2021/jan/25/electric-cars-are-not-the-only-green-solution)

AULA 04 – ADJECTIVES AND ADVERBS 43


TEACHER ANDREA BELO

Questão 03 (EAM/INÉDITA) – In the sentence “But it does not begin to solve the other problems
associated with the volume of traffic on the roads” (paragraph 1), the pronoun “it” refers to
(A) Advance in battery technology
(B) Electric cars
(C) Costs associated with electric cars
(D) Volume of traffic on the roads
(E) Tax relief

Questão 04 (EAM/INÉDITA) – It is FALSE to say that


(A) Battery technology lowers electric car costs
(B) Decreasing costs can increase the number of cars on the roads
(C) Electric cars have no benefits, considering the arguments of the text
(D) The most effective way to decrease carbon emissions is by decreasing traffic
(E) Other modes of locomotion should be motivated

Questão 05 (EAM/INÉDITA) –) Complete the paragraph below, about Britain’s class system, with
the missing prepositions Britain’s class system determines who is wealthy, who has power,
_______ even who lives. Before Covid-19 arrived, _______ you were a man in one of England’s
poorest communities, you could expect _______ live nine years fewer than someone in one of
its most affluent areas.
(Adapted from https://www.theguardian.com/commentisfree/2021/jan/26/combination-covid-class-devastating-britain-poorest)

(A) And / if / to
(B) With / if / for
(C) And / if / for
(D) And / and / for
(E) And / X / to

QUESTÕES EEAR
Read the text and answer questions 01, 02, 03 and 04
Sinovac: Brazil results show Chinese vaccine 50.4% effective
A coronavirus vaccine developed by China's Sinovac has been found to be 50.4% effective in
Brazilian clinical trials, according to the latest results released by researchers.
It shows the vaccine is significantly less effective than previous data suggested - barely over the
50% needed for regulatory approval.

AULA 04 – ADJECTIVES AND ADVERBS 44


TEACHER ANDREA BELO

The Chinese vaccine is one of two that the Brazilian government has lined up. Brazil has been one
of the countries worst affected by Covid-19.
Sinovac, a Beijing-based biopharmaceutical company, is behind CoronaVac, an inactivated
vaccine. It works by using killed viral particles to expose the body's immune system to the virus
without risking a serious disease response.
Several countries, including Indonesia, Turkey and Singapore, have placed orders for the vaccine.
Last week researchers at the Butantan Institute, which has been conducting the trials in Brazil,
announced that the vaccine had a 78% efficacy against "mild-to-severe" Covid-19 cases.
But on Tuesday they revealed that calculations for this figure did not include data from a group of
"very mild infections" among those who received the vaccine that did not require clinical
assistance.
With the inclusion of this data, the efficacy rate is now 50.4%, said researchers.
But Butantan stressed that the vaccine is 78% effective in preventing mild cases that needed
treatment and 100% effective in staving off moderate to serious cases.
The Sinovac trials have yielded different results across different countries.
Last month Turkish researchers said the Sinovac vaccine was 91.25% effective, while Indonesia,
which rolled out its mass vaccination programme on Wednesday, said it was 65.3% effective. Both
were interim results from late-stage trials.
(Adapted from https://www.bbc.com/news/world-latin-america-55642648)

Questão 01 (EEAR/INÉDITA) – According to the text, we can infer that


(A) The Chinese vaccine has a high rate of effectiveness, according to Brazilian studies
(B) CoronaVac is the only vaccine option of the Brazilian government
(C) CoronaVac's origins is unknown
(D) The first calculation of the total effectiveness of CoronaVac did not include a significant
group of infected

Questão 02 (EEAR/INÉDITA) – The words DEVELOPED and ANNOUNCED are


(A) Regular verbs
(B) Not verbs
(C) Irregular verbs
(D) Modal verbs

Questão 03 (EEAR/INÉDITA) – to the text, choose the best response


(A) The first effectiveness rate announced by Butantan Institute was wrong
(B) The first effectiveness rate announced by Butantan Institute was miscalculated
(C) The first effectiveness rate announced by Butantan Institute was lower than the second
(D) The first effectiveness rate announced by Butantan Institute was the right one

AULA 04 – ADJECTIVES AND ADVERBS 45


TEACHER ANDREA BELO

Questão 04 (EEAR/INÉDITA) – According to the text, the effectiveness of the Sinovac vaccine
(A) Is the same in all countries that adopted this vaccine
(B) Is not very reliable
(C) Is the best among all the other vaccines
(D) Is variable according to the research

Read the comic strip and answer questions 05 and 06

(Adapted from https://www.gocomics.com/pickles/2021/01/12)

Questão 05 (EEAR/INÉDITA) – Choose the best alternative to complete the blank


(A) Is keeping
(B) Keeps
(C) Kept
(D) Was keeping

Questão 06 (EEAR/INÉDITA) – Choose the best alternative according to the comic strip
(A) The man’s snores did not bother the woman
(B) The woman prefers to continue listening to snoring than having more work in the morning
(C) The man does not consider sleeping in separate beds, an option
(D) The woman was so irritated by snoring that she prefers to sleep in separate beds

Read the comic strip and answer questions 07 and 08

(Adapted from https://www.gocomics.com/pickles/2020/12/02)

AULA 04 – ADJECTIVES AND ADVERBS 46


TEACHER ANDREA BELO

Questão 07 (EEAR/INÉDITA) – Choose the best alternative to complete the blank


(A) Used
(B) Using
(C) Use
(D) Have been using

Questão 08 (EEAR/INÉDITA) – According to what the woman said, she


(A) Doesn’t like cats at all
(B) Doesn’t care about the man’s opinion
(C) Likes cats so much that she uses them as decoration
(D) Put her pet as a Christmas tree decoration

Read the lyrics and answer questions 09 and 10


Best part - Daniel Caesar / H.E.R
[...]
You don't know, babe
When you hold me
And kiss me slowly
It's the sweetest thing
And it don't change
If I had it my way
You would know that you are
You're the coffee that I need in the morning
You're my sunshine in the rain when it's pouring
Won't you give yourself to me
Give it all, oh
I just wanna see
I just wanna see how beautiful you are
You know that I see it
I know you're a star
Where you go I follow
No matter how far
If life is a movie
Oh you're the best part, oh oh oh

AULA 04 – ADJECTIVES AND ADVERBS 47


TEACHER ANDREA BELO

You're the best part, oh oh oh


Best part
It's the sunrise
And those brown eyes, yes
You're the one that I desire
When we wake up
And then we make love
It makes me feel so nice
You're my water when I'm stuck in the desert
You're the Tylenol I take when my head hurts
You're the sunshine on my life
I just wanna see how beautiful you are
You know that I see it
I know you're a star
Where you go I follow
No matter how far
If life is a movie
Then you're the best part, oh oh oh
You're the best part, oh oh oh
Best part
[...]
(Adapted from LyricFind)

Questão 09 (EEAR/INÉDITA) – According to the lyrics


(A) The song portrays the reality of an abusive relationship
(B) The song expresses the feeling of intense passion of a loving relationship
(C) The song criticizes the idea of an extreme emotional dependence
(D) The song plunges into the idea of a three-way relationship

Questão 10 (EEAR/INÉDITA) – The underlined expression “the sweetest thing”, in the text,
means that the action
(A) Was sensitive and selfless
(B) Was actually sweet
(C) Was rude and selfish
(D) Was indifferent

AULA 04 – ADJECTIVES AND ADVERBS 48


TEACHER ANDREA BELO

QUESTÕES EFOMM
Based on the text below, answer questions 01 and 02.
Why some people like wearing masks
Some people welcome face coverings for reasons ranging from the convenient and expedient
to the more complex and psychological. But is this a helpful coping mechanism?
Sheltering in place hasn’t been too hard for Jay Lee; watching a film at home and ordering a takeaway
has always been his idea of a good night. Lee, a 32-year-old small business owner in Leicester,
identifies as an introvert. And although 2020 had its hardships – in the spring, he was made redundant
from his job at a large bank – one perk for him has been the widespread adoption of face masks.
Lee has always dreaded run-ins with old friends and acquaintances around town, finding these
spontaneous interactions “extremely awkward”. He used to time his shopping trips to minimise the
possibility of bumping into someone he knew, waiting until almost closing time before heading out.
“Since I've been wearing the mask, my awkward interactions with friends and family have significantly
reduced,” he says. Now, he goes to the shops whenever he wants, without worrying about whom he
might see. He hopes that, even after the pandemic ends, it will still be socially acceptable to wear a
mask.
Wearing a mask is, for most of us, an annoying but worthwhile sacrifice: it’s one of the most effective
ways to slow the spread of Covid-19. Still, most of us look forward to the day when we can bare our
faces in public again. Face-coverings fog our glasses and clog our pores; they make it harder to smile
at strangers and recognise friends.
Yet some are secretly relishing the new mask-wearing mandates, for reasons ranging from the
convenient and expedient to the more complex and psychological. Some welcome the way face
coverings reduce or change interactions that might otherwise spark social anxiety.
‘Anonymity carries power’
At the lighter end of the scale, some people have found that masking offers a welcome relief from the
pressures to uphold strict standards of grooming and appearance. They have ditched their old makeup
and shaving routines and are saving money, time and stress. Others have discovered that hiding their
mouths affords them unexpected freedoms. Some restaurant servers and retail workers say they no
longer feel obliged to fake-smile at customers, potentially lifting the burden of emotional labour.
(Adapted from https://www.bbc.com/worklife/article/20210115-why-some-people-like-wearing-masks)

Questão 01 (EFOMM/INÉDITA) – It is possible to infer from the text that


(A) The safety measure related to the masks is well accepted depending on the personality of
the user
(B) Face coverings is an inconvenience for everyone
(C) Introverted people ended up not adapting to the context of a pandemic
(D) Wearing a mask is a pointless sacrifice at the present time
(E) Wearing masks does not interfere with the aesthetic pressure exerted by society

AULA 04 – ADJECTIVES AND ADVERBS 49


TEACHER ANDREA BELO

Questão 02 (EFOMM/INÉDITA) – In the excerpt “And although 2020 had its hardships – in the
spring, he was made redundant from his job at a large bank – one perk for him has been the
widespread adoption of face masks.”, the word in bold means
(A) Advice
(B) Hardship
(C) Happiness
(D) Loss
(E) Advantage

Based on the text below, answer questions 03, 04 and 05.


The World's Biggest Problems Are Interconnected. Here's How We Can Solve Them This
Decade
Two decades ago, people around the world rang in the new millennium with a growing sense of
optimism. The threat posed by the Cold War was fading slowly in the rearview mirror. Leading
thinkers like Francis Fukuyama touted the benefits of globalization, saying it would bring
democracy and prosperity to the developing world. The nascent Internet economy promised to
bring us closer together.
The following 20 years took some of the air out of the assumption of steady progress, but when
future historians assess the 21st century, the year 2020 is likely to serve as the point at which the
optimism bubble burst. The COVID-19 pandemic has exposed a complex web of interlocking
problems that have morphed into full-blown crises. The coronavirus laid bare the dangers of
endemic poverty not only in the developing world but also in rich countries like the U.S., where
millions lack health care and are one paycheck away from living on the street. Around the world,
racial and ethnic minorities have demanded justice after centuries of structural discrimination.
Woven through it all, the earth’s climate is increasingly unstable, posing an existential threat to
human society as we know it. In the next decade, societies will be forced to either confront this
snarl of challenges, or be overwhelmed by them. Our response will define the future for decades
to come.
The recognition that these challenges are fundamentally linked isn’t new. Activists and academics
have for many years pointed to the cascading effects of various social ills. Whether it’s the way
racism contributes to poor health outcomes or gender discrimination harms economic growth,
the examples are seemingly endless. But this understanding has made its way into the
conversation about solutions too.
Notably, for the past five years, the U.N. has touted 17 interrelated sustainable development
goals, objectives for building a more viable world, and called for a push to achieve them by 2030.
The goals, which cover environmental, social and economic progress, are nonbinding but have
become key benchmarks for commitments at a national and corporate level. Countries from China
to the Maldives, as well as companies like Amazon, Microsoft and PwC, have committed to rolling
out policies over the next decade that will set them on a path to eliminate their carbon footprints.

AULA 04 – ADJECTIVES AND ADVERBS 50


TEACHER ANDREA BELO

The understanding that these problems require holistic solutions has only grown amid the
pandemic and its fallout. President Joe Biden has referred to four urgent crises—the pandemic,
the economic crisis, racial injustice and climate change—and promised a push to tackle them all
together. The European Union’s program to propel the bloc out of the COVID-19 crisis targets
climate change, while incorporating equity concerns. As stock markets soared last year,
institutions with trillions of dollars in assets demanded that their investments deliver not only a
good return for their wallets but also a good return for society.
(Adapted from https://time.com/5931603/how-to-solve-worlds-biggest-problems/)

Questão 03 (EFOMM/INÉDITA) – Read the statements about the text and decide whether they
are TRUE (T) or FALSE (F). Mark the correct option
I. The turn of the millennium brought high expectations for the development of the world
II. Covid-19 plays an important role in explaining the interconnection between global problems
III. Despite having played a watershed role, the Covid-19 pandemic mainly affected developing
countries
IV. The intensely discriminatory past has no reflection today
V. Our current actions will affect the future of world society
(A) I – (T) / II – (F) / III – (F) / IV – (F) / V – (T)
(B) I – (T) / II – (T) / III – (F) / IV – (F) / V – (F)
(C) I – (T) / II – (T) / III – (F) / IV – (F) / V – (T)
(D) I – (F) / II – (T) / III – (F) / IV – (F) / V – (T)
(E) I – (F) / II – (T) / III – (T) / IV – (F) / V – (T)

Questão 04 (EFOMM/INÉDITA) – In the excerpt “Woven through it all, the earth’s climate is
increasingly unstable, posing an existential threat to human society as we know it”, the word in
bold means
(A) Unwoven
(B) Separate
(C) In contrast
(D) Sewn
(E) Although

Questão 05 (EFOMM/INÉDITA) – According to the text, the current problems in the world
(A) Are actually a conspiracy theory that opposes the high expectations of the turn of the
millennium
(B) Must be resolved for the whole society, after their total exposure during the pandemic
(C) Did not worsen during the Covid-19 pandemic
(D) Have a relatively new interconnection
(E) Are not so serious, considering the high development of society

AULA 04 – ADJECTIVES AND ADVERBS 51


TEACHER ANDREA BELO

Questão 06 (EFOMM/INÉDITA) – Choose the correct option to complete the paragraph below.
I remember the last time I had _______ choose between optimism and fear, between hope and
the urge to run away. It was right after 9/11. I had babies—one so tiny she was still curled up like
a fern, _______ other toddling around gumming everything she could get her little starfish hands
on. And as their brand-new lungs took _______ the smoke that blew over to Brooklyn _______
the burning towers, I wanted to pack them up and flee to some safer place.
(Adapted from https://time.com/5930399/the-capitol-riots-and-our-fragile-optimism/)

(A) To / the / in / from


(B) The / the / on / into
(C) To / the / on / from
(D) To / the / in / into
(E) The / to / on / from

Questão 07 (EFOMM/INÉDITA) – Which is the correct way to complete the paragraph below?
For the European Union, the rapid rollout of Covid-19 vaccines __________ critical to save lives
and prevent health services from ____________ stretched beyond their limits, not to mention
minimising the massive economic damage from lockdowns. Unfortunately, however, though
vaccinations are under way, a rapid near-term increase in infections is likely as the British variant
of the virus __________ across the continent.
(Adapted from https://www.concursosmilitares.com.br/provas-anteriores/marinha/efomm/2020-efomm-oficial-da-marinha-mercante-primeiro-dia.pdf)

(A) Is / being / spreads


(B) Was / be / spreads
(C) Is / be / spreading
(D) Was / being / spreading
(E) Is / being / spreading

Questão 08 (EFOMM/INÉDITA) – Which of the following sentences expresses probability?


(A) You are not as understanding as you should be
(B) One shouldn’t accelerate in case of a red light
(C) Everyone here must have a critical sense
(D) He studied a lot, so he should be able to pass
(E) You must take good care of your body, even if you don't want to

AULA 04 – ADJECTIVES AND ADVERBS 52


TEACHER ANDREA BELO

Questão 09 (EFOMM/INÉDITA) – Choose the correct option to complete the paragraph below.
The U.S. has tragically surpassed 400,000 COVID-19 deaths, and case numbers and hospitalizations
are likewise spiking to record levels around the world. __________ vaccines now rolling out, there is
reason to hope that there is an end in sight. __________ , by most estimates, widespread vaccinations
will not be in place until the middle of the year at the earliest. __________ , we have some ways to
go yet with social distancing, mask wearing and other pandemic mitigation behaviors.
(Adapted from https://www.scientificamerican.com/article/how-we-can-deal-with-pandemic-fatigue/)

(A) And / however / and


(B) With / however / so
(C) So / however / even if
(D) With / in agreement / and
(E) With / in contrast / so

Questão 10 (EFOMM/INÉDITA) – Which option is incorrect?


(A) A pandemic is affecting many people
(B) Tell the girls that they are not responsible
(C) A couple of my friends plans to travel this year
(D) I want to finish school
(E) Where are those pants I lent you?

QUESTÕES EPCAR
Directions: Answer questions 01 to 10 according to TEXT I
Facebook building a version of Instagram for children under 13
Social media giant says it’s exploring introducing a parent-controlled experience that allows
kids to ‘safely’ use the photo sharing platform
Facebook is considering launching a version of its popular photo social media platform, Instagram,
for children under the age of 13.
BuzzFeed News first reported Facebook announced in an internal company post that the company
would begin building a version of Instagram for people under the age of 13 years to allow them
to “safely” use Instagram for the first time. Currently the company does not allow people who are
under this age to create an account on the platform.
A spokesperson for Facebook told the Guardian the company was exploring a parent-controlled
version of Instagram, similar to the Messenger Kids app that is for kids between six and 12.
“Increasingly kids are asking their parents if they can join apps that help them keep up with their
friends. Right now there aren’t many options for parents, so we’re working on building additional
products … that are suitable for kids, managed by parents,” the spokesperson said.

AULA 04 – ADJECTIVES AND ADVERBS 53


TEACHER ANDREA BELO

“We’re exploring bringing a parent-controlled experience to Instagram to help kids keep up with
their friends, discover new hobbies and interests, and more.”
In a blog post earlier this week, which did not mention the proposed new Instagram service, the
company noted that although people were asked to enter their age when signing up for
Instagram, there was nothing to prevent people from lying about it at registration.
Facebook said it would overcome that by using machine learning in combination with the
registration age to determine people’s ages on the platform.
The company also announced plans to roll out new safety features, including preventing adults
from messaging people under the age of 18 who do not follow them, safety notices for teens
when messaged by an adult sending a large amount of friend requests or messages to people
under 18, and make it more difficult for adults to find and follow teens using the search function
in Instagram.
Teens will also be encouraged to put their profiles on private at the point of registration.
A study of Australian teens’ internet usage published by the Australian eSafety commissioner in
February found 57% of Australian teenagers use Instagram, while 30% reported being contacted
by a stranger, and 20% reported being sent inappropriate unwanted content on the social media
sites they used.
(Adapted from https://www.theguardian.com/technology/2021/mar/19/facebook-building-a-version-of-instagram-for-children-under-13)

Questão 01 (EPCAR/INÉDITA) – Mark the option that can replace the word “allows” in the title
without changing its meaning
(A) Grants
(B) Prevents
(C) Prohibits
(D) Disapproves

Questão 02 (EPCAR/INÉDITA) – We can deduce from the second paragraph that


(A) Facebook developed a platform for children under 13
(B) The possible development of the new social media was not first disclosed by Facebook
(C) The new social media will target all age groups
(D) Instagram allows any and all types of people to sign up

Questão 03 (EPCAR/INÉDITA) – Mark the option with the suitable question to answer the
underlined fragment below “Facebook is considering launching a version of its popular photo
social media platform, Instagram, for children under the age of 13”
(A) What were the latest Facebook launches?
(B) What will be the changes in the current Facebook photo social media platform?
(C) How was the launch of the new Facebook photo social media platform?
(D) What is the next possible Facebook launch?

AULA 04 – ADJECTIVES AND ADVERBS 54


TEACHER ANDREA BELO

Questão 04 (EPCAR/INÉDITA) – Mark the option that can replace the underlined word keeping
the same meaning “A spokesperson for Facebook told the Guardian the company was exploring
a parent-controlled version of Instagram…”
(A) Neglecting
(B) Analyzing
(C) Overlooking
(D) Finding.

Questão 05 (EPCAR/INÉDITA) – What can be said about the new version of Instagram?
(A) The platform will be for children between 6 and 12 years old
(B) It will be for children under 13 years of age to use the platform without the help of adults
(C) It’s a platform that, in some aspects, resembles another one that already exists
(D) Nothing is known at all about the new Facebook platform

Questão 06 (EPCAR/INÉDITA) – According to the fourth paragraph, kids


(A) Are increasingly interested in becoming part of social media
(B) Are not very well inserted in the virtual world, despite having many options
(C) Are less and less interested in apps that connect them with their friends
(D) Prefer to keep in touch in person, disregarding the internet

Questão 07 (EPCAR/INÉDITA) – Mark the correct option


(A) Facebook has already developed an accessible version of Instagram for children under 13
(B) Facebook is developing a version of Instagram for children under 13 to use the app
completely autonomously
(C) Facebook is developing a version of Instagram so that children under 13 just chat with their
friends
(D) Facebook is researching an accessible and secure Instagram platform for children under 13

Questão 08 (EPCAR/INÉDITA) – Considering the use of verb tenses, mark the alternative that
completes the sentence below correctly Facebook
(A) fights false ages in the system using technology
(B) is fighting false ages in the system using technology
(C) will fight false ages in the system using technology
(D) has been fighting false ages in the system using technology.

AULA 04 – ADJECTIVES AND ADVERBS 55


TEACHER ANDREA BELO

Questão 09 (EPCAR/INÉDITA) – We can deduce from the eighth paragraph that


(A) Instagram has no security issues
(B) Security issues are worrying to the point that Instagram needs new security measures
(C) People over 18 are prohibited from sending messages to minors
(D) The new security measures aim to protect female users

Questão 10 (EPCAR/INÉDITA) – The word “them” (paragraph 8) refers to


(A) Adults
(B) People under the age of 18
(C) New safety features
(D) The company

QUESTÕES ESA
COVID-19 Conspiracy Theories Are Spreading Rapidly—and They're a Public Health Risk All
Their Own
Public health crises have spawned conspiracy theories as far back as when the Black Death
ravaged Europe in the 1300s, as people desperately try to make sense of the chaotic forces
disrupting their lives. While modern science offers a better understanding of how diseases infect
people and how to contain them, COVID-19 conspiracy theories are spreading rapidly via social
media, unreliable news outlets and from our own political leaders, including U.S. President Donald
Trump. The result: many Americans now believe pandemic -related conspiracy theories—and,
alarmingly, those same people are less likely to take steps to prevent the virus from spreading.
In a University of Pennsylvania Annenberg Public Policy Center study published Monday in Social
Science &
Medicine, researchers surveyed a group of 840 U.S. adults—first in late March, and then again in
mid-July—to determine how Americans’ beliefs and actions regarding the pandemic changed over
time. Overall, they found that COVID-19 conspiracy theories are not only commonplace, they’re
gaining traction. Back in March, 28% of people believed a debunked rumor that the Chinese
government created the coronavirus as a bioweapon; that number rose to 37% by July. About
24% believed that the U.S. Centers for Disease Control and Prevention exaggerated the virus’
danger to hurt Trump politically despi te a lack of evidence; by July, that figure rose to 32%. And
in March, about 15% of respondents said they believed that the pharmaceutical industry created
the virus to boost drug and vaccine sales—another unfounded theory—compared to 17% in July.
(Adapted from https://time.com/5891333/covid-19-conspiracy-theories/)

AULA 04 – ADJECTIVES AND ADVERBS 56


TEACHER ANDREA BELO

Questão 01 (ESA/INÉDITA) – Concerning the information in the text, is correct to state that
(A) Adherence to conspiracy theories regarding the COVID-19 pandemic has declined over time
in the USA
(B) Social networks are not a potential disseminator of misinformation that collaborates with
conspiracy theories
(C) The artificial creation of the coronavirus is scientifically proven
(D) Modern science does not affect the understanding of diseases
(E) Conspiracy theories may affect the adherence of security measures by part of the population

Questão 02 (ESA/INÉDITA) – Which option has a correct relation of the underlined terms and
their substitutes?
(A) A government spokesman says… – Her
(B) Prisoners are considered a COVID-19 risk group. – They
(C) Me and the girls were at the party and ... – They
(D) The US has thousands of cases of COVID-19. – He
(E) Feminist groups still fight for women's rights – Them

Questão 03 (ESA/INÉDITA) – “__________ the US still have the __________ number of COVID-
19 cases?”. Fill in the blanks with the correct form of the verb and the adjective
(A) Does / worst
(B) Does / better
(C) Is / best
(D) Is / worse
(E) Do / worst

Questão 04 (ESA/INÉDITA) – Which sentence is grammatically correct?


(A) That women is in love with me
(B) A motorcycle is more slow than a car
(C) She practices physical exercise every week
(D) I have a sofa new
(E) He need to be loved

AULA 04 – ADJECTIVES AND ADVERBS 57


TEACHER ANDREA BELO

Questão 05 (ESA/2019) – “__________ American?” Complete the space with the correct form of
the verb and the pronoun.
(A) You is
(B) You are
(C) Are you
(D) Is you
(E) Am you

QUESTÕES ESCOLA NAVAL


Based on the text below, answer the six questions that follow it.
Covid babies don't have to be the unlucky generation. But they must be helped
A year into this pandemic, there are babies now learning to walk and talk who have never known
anything but life under the shadow of Covid, and preschoolers who can barely remember a world
before it. Doctors’ children have had to learn not to touch Mummy when she gets in from work,
until she’s had a shower to wash off any last possible trace of danger.
Thanks to popular toddler demand, you can now buy masks for dolls.
It’s perfectly normal for kids to reflect what’s happening around them by playing quarantine with
stuffed animals, but normal too for adults to wonder uncomfortably whether all this leaves a
lasting mark. How much will it matter in years to come that, as the minister for loneliness Diana
Barran recently put it, there are toddlers being raised by shielding parents who have never had a
playdate? Will Covid babies grow up solitary creatures, used to entertaining themselves, or warier
of the strangers they so rarely meet and interact with? Taking a tiny bundle out in public used to
mean an endless succession of random older women cooing over the pram, or strangers pulling
faces to entertain a bored baby in a checkout queue.
But now passersby daren’t get close, and other shoppers are hidden behind masks.
This week MPs were presented with some early findings from a project led by the First 1001 Days
Movement, an alliance of early-years charities and professionals, tracking the lives of under-twos
growing up through a pandemic. A survey of children’s service providers it commissioned found
98% thought the babies and toddlers they worked with had been affected by higher parental
stress and anxiety, while 92% had seen fearful families effectively cutting themselves off from the
outside world, skipping routine appointments or not wanting to leave the house. Nine in 10 had
observed children being played with less, or being less active. Heartbreakingly, more than a
quarter said lockdown left the children they worked with more exposed to domestic conflict,
abuse or neglect.
(Adapted from https://www.theguardian.com/commentisfree/2021/jan/22/covid-babies-unlucky-generation-help-lockdown)

AULA 04 – ADJECTIVES AND ADVERBS 58


TEACHER ANDREA BELO

Questão 01 (ESCOLA NAVAL/INÉDITA) – According to the text, which option is correct?


(A) There are children who have not yet experienced life outside the pandemic
(B) The pandemic has not affected the lives of any children, as they do not understand what is
happening
(C) The pandemic, in a way, positively impacted the babies' lives
(D) The children's routine has not changed in this pandemic
(E) Characteristics of the pandemic did not impact areas such as sales

Questão 02 (ESCOLA NAVAL/INÉDITA) – What’s the meaning of the word “toddler” in paragraph
1?
(A) Population
(B) Children
(C) Babies
(D) Adults
(E) Teens

Questão 03 (ESCOLA NAVAL/INÉDITA) – In paragraph 1, the word “she” refers to


(A) Children
(B) Doctor
(C) Toddler
(D) Mummy
(E) Babies

Questão 04 (ESCOLA NAVAL/INÉDITA) – According to the text, which option completes the
sentence below, correctly?
The pandemic __________ an entire generation.
(A) Affects
(B) Affected
(C) Was affecting
(D) Had affected
(E) Is affecting

AULA 04 – ADJECTIVES AND ADVERBS 59


TEACHER ANDREA BELO

Questão 05 (ESCOLA NAVAL/INÉDITA) – According to the text, which option is correct?


(A) Children do not realize what happens around them
(B) The impact of the pandemic on children is a normal concern of parents
(C) There is no doubt about this generation that was born in the middle of a pandemic
(D) The relationship with children on the street changed with the pandemic
(E) The impact of the pandemic on children can only be short-term

QUESTÕES EsPCEx
Leia o texto a seguir e responda às questões 01, 02 e 03.
Bill Gates: ‘Carbon neutrality in a decade is a fairytale. Why peddle fantasies?’
Bill Gates appears via video conference – Microsoft Teams, not Zoom, obviously – from his office
in Seattle, a large space with floor-to-ceiling windows overlooking Lake Washington. It’s a gloomy
day outside and Gates is, somewhat eccentrically, positioned a long way from the camera, behind
a large, kidney-shaped desk; his communications manager sits off to one side. “As a way to start,”
says Gates’ aide, “would it be helpful for Bill to make a couple of comments about why he wrote
his new book?”
Unlike the Elon Musks or Larry Ellisons of this world, however, Gates is perceived to be sensible,
uxorious, modest, vowing not to ruin his children with boundless inheritance or to waste energy
trying to send things to Mars. Gates’ new book, How To Avoid A Climate Disaster, grew out of two
things: his interest in the sciences and what struck him as an irresistible challenge – the fiendishly
difficult problem of how to further global development while reducing emissions.
There’s another, greater obstacle to reaching zero emissions, which is the political challenge – part
of which involves climate activists limiting their exposure to accusations of hypocrisy. Gates loves
private jets; he calls them his “guilty pleasure”.
The depressing part of the book is its account of the challenge ahead, which Gates presents as
extremely urgent – He points to a headline figure: 51bn. This is the amount of greenhouses gas,
in tons, emitted globally each year, which we have to get down to net zero by 2050. The first step
towards this is understanding what we’re dealing with.
Adapted from https://www.theguardian.com/technology/2021/feb/15/bill-gates-carbon-neutrality-in-a-decade-is-afairytale-why-peddle-fantasies

Questão 01 (EsPCEx/INÉDITA) – According to the text, choose the correct alternative.


(A) Gates usually invests in sending things to Mars.
(B) Gates wrote an e-book about world climate.
(C) Bill Gates made a video conference via Zoom.
(D) Bill Gates currently lives in Seattle.
(E) For Gates, it will take more than a decade to reach carbon neutrality.

AULA 04 – ADJECTIVES AND ADVERBS 60


TEACHER ANDREA BELO

Questão 02 (EsPCEx/INÉDITA) – In the sentence “... Gates is perceived to be sensible ...”


(paragraph 2), the word sensible means
(A) reasonable
(B) delicate
(C) gentle
(D) tender
(E) polite

Questão 03 (EsPCEx/INÉDITA) – In the sentence “… positioned a long way from the camera,
behind a large, kidney-shaped desk … (paragraph 1), the expression kidney-shaped refers to
(A) the size of the desk.
(B) the outline of the desk.
(C) the weight of the desk.
(D) the height of the desk.
(E) the color of the desk.

Leia o texto a seguir e responda às questões 04, 05, 06 e 07.


How a Spanish town pioneered dolls with Down's syndrome
The first time Kelle Hampton glimpsed a doll with Down’s syndrome, anger boiled up inside her. Its
exaggerated features bore little resemblance to the sweet facial characteristics that she loved about her
daughter Nella, who was born with the genetic disorder.
The experience set the US blogger and author firmly against such dolls. But to her surprise, years later she
found herself smitten with another doll. This time it had been carefully crafted to subtly capture the
characteristics that made Nella unique. “This one was simply a beautiful doll any child would want to play
with,” she said.
The doll hailed from a small town in eastern Spain that was put on the map after enterprising potters
began transforming clay from the nearby mountains into children’s dolls, giving rise to what has been
described as the country’s first doll factory. More than a century later the spotlight is again on the
Valencian town of Onil – this time for its singular combination of inclusivity and artisan dollmaking.
The town, home to around 7,500 people and 38 toymakers, made headlines across Spain last year after a
local collection of four dolls with Down’s syndrome won the country’s much-coveted “toy of the year”
prize.
The dolls – two boys and two girls of varying skin colours – were rolled out by toymaker Miniland. “We
were apprehensive at the beginning,” said Victoria Orruño, the company’s marketing director. “But the
reaction surprised us. It was very positive.”
Adapted from https://www.theguardian.com/society/2021/feb/15/dolls-and-diversity-how-toys-pass-on-the-power-of-being-see

AULA 04 – ADJECTIVES AND ADVERBS 61


TEACHER ANDREA BELO

Questão 04 (EsPCEx/INÉDITA) – Choose the statement in which the verb to boil has been used in
a figurative way just like in paragraph 1.
(A) Could you boil some water up for me?
(B) I've boiled some potatoes for lunch.
(C) I can hardly boil an egg.
(D) He was boiling with rage.
(E) Liquid nitrogen boils at a very low temperature.

Questão 05 (EsPCEx/INÉDITA) – In “Its exaggerated features bore little resemblance to the sweet
facial characteristics that she loved about her daughter …” (paragraph 1), its refers to
(A) Kelle Hampton
(B) her daughter
(C) Down’s syndrome
(D) anger
(E) a doll

Questão 06 (EsPCEx/INÉDITA) – Choose the alternative that has the same meaning as the word
spotlight in the sentence “More than a century later the spotlight is again on the Valencian town
of Onil…” (paragraph 3).
(A) polemic
(B) anger
(C) attention
(D) mistake
(E) force

Questão 07 (EsPCEx/INÉDITA) – About the underlined words her, she and herself, it is correct to
say that
(A) they are adjectives.
(B) they are verbs.
(C) they are pronouns.
(D) they are nouns.
(E) they are adverbs.

AULA 04 – ADJECTIVES AND ADVERBS 62


TEACHER ANDREA BELO

Leia o texto a seguir e responda às questões 08, 09, e 10.


Professor lectures for two hours online – on mute
A university professor in Singapore gave a two-hour online lecture but didn't realize he was on mute.
Professor Wang, who teaches maths, was not aware that throughout his online presentation, the
microphone on his computer was switched off. This meant that none of the students attending his online
class heard what professor Wang was talking about. His university switched his classes online to help
reduce the spread of COVID-19. His lecture started well but then it froze. The video came back but the
mic was off and professor Wang did not notice. One of his students says it might have been because the
professor was using an iPad and not a computer.
Students tried many times to contact professor Wang during the lesson but could not get through to
him. They waved their arms and even tried calling him on his personal phone. A video of the moment he
understood his mic was off has gone viral on the Internet. It has received more than 653,000 views. After
realizing what happened, professor Wang appeared shocked and stressed. He said he was upset at the
thought of wasting two hours of his students' time. He has offered to redo the lecture at a different time
so his students do not miss that class. One of his students posted on social media about the professor's
mishap. She wrote: “I took a class under him before and he teaches well. I feel bad for him.”
Adapted from https://www.dailymail.co.uk/news/article-9247009/Maths-professor-Singapore-delivers-two-hour-lecture-Zoom-realisemute.html.

Questão 08 (EsPCEx/INÉDITA) – According to the text, choose the correct alternative.


(A) The professor said he would not redo the lecture.
(B) A video of the professor has gone viral on the Internet.
(C) The professor is sad because students made fun of him.
(D) The problem might have been caused by the computer the professor used.
(E) Students refused to warn the professor about what was happening.

Questão 09 (EsPCEx/INÉDITA) – In the sentence “...One of his students posted on social media
about the professor's mishap....” (paragraph 2), the prefix mis means
(A) wrongly
(B) against
(C) without
(D) self
(E) beyond

Questão 10 (EsPCEx/INÉDITA) – In “His lecture started well but then it froze.” (paragraph 1), the
word then
(A) introduces a conclusion.
(B) indicates a sequence of facts.
(C) makes a comparison.
(D) indicates a place.
(E) reinforces irony.

AULA 04 – ADJECTIVES AND ADVERBS 63


TEACHER ANDREA BELO

GABARITO
GABARITO AFA
01 – D 02 – A 03 – C 04 – D 05 – B
06 – B 07 – C 08 – A 09 – A 10 – D
11 – B 12 – C 13 – C 14 – D 15 – A

GABARITO COLÉGIO NAVAL


01 – A 02 – B 03 – E 04 – B 05 – C
06 – D 07 – A 08 – B 09 – E 10 – B

GABARITO EAM
01 – E 02 – B 03 – A 04 – C 05 – A

GABARITO EEAR
01 – D 02 – A 03 – B 04 – D 05 – C
06 – B 07 – A 08 – C 09 – B 10 – A

GABARITO EFOMM
01 – A 02 – E 03 – C 04 – D 05 – B
06 – A 07 – A 08 – D 09 – B 10 – C

GABARITO EPCAR
01 – A 02 – B 03 – D 04 – B 05 – E
06 – A 07 – D 08 – C 09 – B 10 – A

GABARITO ESA
01 – E 02 – B 03 – A 04 – C 05 – C

GABARITO ESCOLA NAVAL


01 – A 02 – B 03 – D 04 – E 05 – B

GABARITO EsPCEx
01 – E 02 – A 03 – B 04 – D 05 – E
06 – C 07 – C 08 – B 09 – A 10 – B

AULA 04 – ADJECTIVES AND ADVERBS 64


TEACHER ANDREA BELO

QUESTÕES COMENTADAS
QUESTÕES AFA
Directions: Read the text below and answer questions 01 to 15 according to it.
How to cope with parent guilt, during the pandemic and beyond
As a mother, I’m vulnerable to the influence of our cultural messaging, and I have fallen prey to
the “We can never give enough, do enough or be enough” narrative. As a psychologist, though, I
know how risky this constant thread of guilt is for our well-being. Guilt can be helpful as an
uncomfortable emotion that motivates us to make amends and change hurtful behaviors, but in
the case of parent guilt, much of the time we are only hurting ourselves.
Enter a global pandemic that has shattered our already precarious parenting lives and provided
ample evidence of our systems’ failures. Employers are expecting our remote-learning children to
behave during our work day, and women are leaving jobs in record numbers for their suddenly
homebound children, because they “just can’t do it all anymore.” Ilyse DiMarco, a clinical
psychologist and the author of the upcoming book “Mom Brain,” sums up the problem: “The issue
with guilt right now is there is potential guilt with anything you do.”
Stress levels have risen, with parents outpacing nonparents in surveys, creating fertile ground for
mental health problems. “We’re feeling like we’re not measuring up in some area or maybe we’re
not making the right choices,” DiMarco says. “Not surprisingly, if we feel like we’re failing, we feel
depressed; if we’re worried about not making the right decision, we feel anxious.”
Daily life in a pandemic has given us a host of new reasons to feel guilty. In addition to the
impossibility of simultaneously working, parenting and, in some cases, teaching our children, we
face daily decisions around health and safety.
Do we allow play dates? What if all of my child’s friends are playing basketball and I say no? What
if we choose inperson school and our child gets covid-19, or we choose remote school and our
child becomes depressed?
Working moms are not okay. No good choices and no good answers, but guaranteed guilt, with a
sprinkling of judgment caused by social pressures.
“Guilt is a useful emotion when it tells us we’ve done something wrong,” says clinical psychologist
Jill Stoddard, author of “Be Mighty.” “What’s happening now is we feel like we’ve done something
wrong even when we’re doing the very best we can.”
So, what can we do about the guilt hanging heavy around our collective necks? Experts suggest a
combination of mindfulness, meaningful self-care and shifting our perspective.
Mindfulness
Do not ignore the guilt; acknowledging it means you can do something about it. Even if our
pandemic-related guilt may be misplaced, Stoddard says we can still use the feeling for its positive
function and make amends. She has told her children, “I really wish I could spend more time with
you guys, and it’s been so hard on all of us. Things will be different someday. I’m sorry I can’t be

AULA 04 – ADJECTIVES AND ADVERBS 65


TEACHER ANDREA BELO

there for you more.” She adds: “You’re not saying, ‘I screwed up.’ You’re saying, ‘It hurts my heart
I can’t do this the way I wish I could right now.’ ”
Remember you are not alone. Feelings of failure are a universal experience of parents during the
coronavirus pandemic. Seeking social support is critical for mental health in times of stress. It can
be as simple as texting a friend to share “Mom fails” from your day. Even this brief connection
offers support and solidarity, and it can decrease our sense of self-blame and inadequacy when
we hear others’ similar feelings and experiences.
Practice self-kindness. Talk to yourself as you would to your friend: “You are doing your very best
in this moment.” This practice helps shift thinking patterns from self-criticism to self-compassion,
which is known to increase positive emotions and decrease negative ones, including guilt.
Real self-care
Prioritize the daily demands on your time and energy. DiMarco advises doing this by asking,
“What’s most important here, on a day-to-day or hour-to-hour basis?” Focus on what is highest
on the priority list, and allow yourself to let go of the other potential tasks instead of feeling guilty
for, inevitably, not getting everything done.
Prioritize yourself. “An important piece, and counterintuitive piece, to manage guilt about not
being there for everybody else, is to also be there for yourself, and make sure you’re on the list
somewhere,” DiMarco says. Plan ahead to ensure time for you in the daily routine, even if it’s just
15 to 30 minutes. This may require an agreement with your partner and/or kids about when you
will not be available the next day. Be realistic about the amount of time you need and how you
will spend it (e.g., 15 minutes on a meditation app). Then, stay committed to making it happen.
Remember the basics. We make sure our kids are fed, but are we sleeping, drinking enough water,
and eating well enough to meet our own basic needs? These stress management essentials
strengthen us, which will help us ward off the guilt.
Shift focus
Adjust expectations. “If you expect yourself to stay calm and never lose it, you’re setting yourself
up for failure,” says child and adolescent psychologist Emily W. King. “Own it that you’re more
emotionally fragile right now, notice it and walk outside for 10 minutes, or whatever you need,
instead of getting in a negative feedback loop, and then you’re emotionally exhausted.”
Consider your successes. Flip the script so you’re not focusing only on the negative. What do you
feel proud of? What can your children do now they couldn’t do a year ago? Redefine success for
you and your children dur ing a global pandemic. “Success is not about grades but about
independence or chores,” says King. For parents, success can be getting through each day with
everyone sheltered, fed and in bed safely.
Invent new narratives. Instead of looking at yourself as being never enough, how about
acknowledging that you have been — and continue to be — enough to endure a global pandemic
while parenting? What recent generation of parents has accomplished this? In fact, we are not
just enough, we are so much greater. Remind yourself of this when parent guilt attempts to tread
that familiar path, both now and after the pandemic.
(Adapted from https://www.washingtonpost.com/lifestyle/2021/02/25/how-cope-with-parent-guilt-during-pandemic-beyond/)

AULA 04 – ADJECTIVES AND ADVERBS 66


TEACHER ANDREA BELO

Questão 01 (AFA/INÉDITA) – The text


a) aims at highlighting how to deal with remote-learning children during the pandemic.
b) provides the reader with a heartfelt narrative of a father about his experience during the
pandemic.
c) focus on a considerable set of psychological implications of pandemic on children.
d) concerns mainly about the mental state of mothers during the pandemic due to the guilt they
feel.
Comentários: O texto
a) visa destacar como lidar com crianças com aprendizagem remota durante a pandemia –
alternativa incorreta, uma vez que o texto visa destacar como as mães devem lidar com a
sensação de culpa e de insuficiência quanto aos filhos durante a pandemia.
b) fornece ao leitor uma narrativa sincera de um pai sobre sua experiência durante a pandemia –
alternativa incorreta, o texto contém a narrativa de uma mãe. “As a mother, I’m vulnerable to the
influence of our cultural messaging,”
c) enfocar um conjunto considerável de implicações psicológicas da pandemia em crianças –
alternativa incorreta, já que o texto enfoca implicações psicológicas da pandemia nas mães.
d) preocupação principalmente com o estado mental das mães durante a pandemia devido à
culpa que sentem – alternativa CORRETA. “How to cope with parent guilt, during the pandemic
and beyond / As a mother, I’m vulnerable to the influence of our cultural messaging, and I have
fallen prey to the ‘We can never give enough, do enough or be enough’ narrative.” = Como lidar
com a culpa dos pais, durante a pandemia e depois / Como mãe, sou vulnerável à influência de
nossa mensagem cultural e sou vítima da narrativa “Nunca podemos dar o suficiente, fazer o
suficiente ou ser o suficiente”.
GABARITO: D

Questão 02 (AFA/INÉDITA) – In the sentence “Guilt can be helpful as an uncomfortable emotion


that motivates us to make amends and change hurtful behaviors,” (paragraph 01), it is possible
to find an option to substitute the highlighted pronoun accordingly in
a) which.
b) who.
c) how.
d) when.
Comentários: Na frase trazida pela questão, é possível encontrar uma opção para substituir
corretamente o pronome (that) na letra A (which), pois that, no caso, está retomando uma coisa
(emotion – uma emoção que, a qual). Para nos referirmos a coisas, podemos usar that ou which.
Para pessoas, that ou who. Nas demais alternativas, temos quem (pessoas), como e quando.
GABARITO: A

AULA 04 – ADJECTIVES AND ADVERBS 67


TEACHER ANDREA BELO

Questão 03 (AFA/INÉDITA) – In the fragment “because they ‘just can’t do it all anymore.’”
(paragraph 02), the highlighted word refers to
a) children
b) jobs
c) women
d) numbers
Comentários: No trecho completo de onde foi extraído o fragmento trazido pela questão,
percebemos que they se refere a women, veja: “Employers are expecting our remote-learning
children to behave during our work day, and women are leaving jobs in record numbers for their
suddenly homebound children, because they ‘just can’t do it all anymore.’” = Os empregadores
estão esperando que nossos filhos com aprendizagem remota se comportem durante nosso dia
de trabalho, e as mulheres estão deixando o emprego em número recorde por causa de seus filhos
repentinamente presos em casa, porque elas “simplesmente não dão conta mais de fazer tudo”.
GABARITO: C

Questão 04 (AFA/INÉDITA) – In the eighth paragraph, the author


a) blames the guilt working moms feel on fathers and employers.
b) starts reinforcing the importance of denying guilt.
c) tries to make controversial issues about guilt clear.
d) introduces the topic of what can be done about the guilt moms feel.
Comentários: No oitavo parágrafo, o autor
a) culpa os pais e empregadores pela culpa que as mães que trabalham sentem.
b) passa a reforçar a importância de negar a culpa.
c) tenta esclarecer questões controversas sobre a culpa.
d) apresenta o tópico sobre o que pode ser feito sobre a culpa que as mães sentem – “So, what
can we do about the guilt hanging heavy around our collective necks? Experts suggest a
combination of mindfulness, meaningful self-care and shifting our perspective.” = Então, o que
podemos fazer sobre a culpa pesando em torno de nossos pescoços coletivos? Os especialistas
sugerem uma combinação de atenção plena, autocuidado significativo e mudança de perspectiva.
GABARITO: D

Questão 05 (AFA/INÉDITA) – Mark the option which shows the best answer for the question
“So, what can we do about the guilt hanging heavy around our collective necks?” (paragraph 8)
a) nothing more than mindfulness.
b) a merging of different strategies.
c) ignoring the guilt.
d) only meaningful self-care.

AULA 04 – ADJECTIVES AND ADVERBS 68


TEACHER ANDREA BELO

Comentários: Marque a opção que mostra a melhor resposta para a pergunta “Então, o que
podemos fazer com a culpa que pesa sobre nossos pescoços coletivos?” (parágrafo 8)
a) nada mais do que atenção plena.
b) uma combinação de diferentes estratégias.
c) ignorar a culpa.
d) somente autocuidado significativo.
“So, what can we do about the guilt hanging heavy around our collective necks? Experts suggest
a combination of mindfulness, meaningful self-care and shifting our perspective.” = Então, o que
podemos fazer sobre a culpa pesando em torno de nossos pescoços coletivos? Os especialistas
sugerem uma combinação de atenção plena, autocuidado significativo e mudança de perspectiva.
GABARITO: B

Questão 06 (AFA/INÉDITA) – In the sentence “She has told her children, ‘I really wish I could
spend more time with you guys, and it’s been so hard on all of us.’” (paragraph 09), the
contraction refers to
a) It is.
b) It has.
c) It was.
d) It will.
Comentários: Na frase trazida pela questão, a contração ’s se refere ao auxiliar do Present Perfect,
has = it has been so hard... → tem sido tão difícil ...
GABARITO: B

Questão 07 (AFA/INÉDITA) – Mark the option which shows the appropriate question tag for the
sentence “I’m vulnerable to the influence of our cultural messaging” (paragraph 01).
a) I’m not?
b) Am I?
c) Aren’t I?
d) Am I not?
Comentários: A opção que mostra a tag question apropriada para a frase é a letra C. A tag
question para a afirmativa do presente do verbo to be com sujeito I é um pouco atípica, pois o
verbo am não é mantido. Usamos obrigatoriamente aren’t.
GABARITO: C

AULA 04 – ADJECTIVES AND ADVERBS 69


TEACHER ANDREA BELO

Questão 08 (AFA/INÉDITA) – Mark the option that shows a synonym for the underlined
expression in “These stress management essentials strengthen us, which will help us ward off
the guilt.” (paragraph 14).
a) repel
b) contradict
c) forbid
d) accept
Comentários: Marque a opção que mostra um sinônimo para a expressão sublinhada (ward off).
Ward off significa repelir, afastar. Assim,
a) repelir
b) contradizer
c) proibir
d) aceitar
GABARITO: A

Questão 09 (AFA/INÉDITA) – Choose the best option to change the sentence “we are … hurting
ourselves.” (paragraph 01), into the passive form. Ourselves _______________ by us.
a) are being hurt
b) are hurting
c) have been hurt
d) is been hurt
Comentários: Escolha a melhor opção para passar a frase para a voz passiva. Ourselves
_______________ by us.
• Devemos manter o tempo verbal (Present Continuous).
• O objeto da frase original vira o sujeito na voz passiva (ourselves).
• Além disso, o sujeito da frase original vira o que equivale ao agente da passiva (we = by us). we
are … hurting ourselves → ourselves are being hurt by us.
GABARITO: A

Questão 10 (AFA/INÉDITA) – In the third paragraph, it’s said that


a) parents are feeling as stressed as nonparents according to surveys.
b) stress levels have risen more in nonparents compared to parents.
c) nonparents are outpacing parents in stress levels in surveys.
d) surveys show parents are now more affected by stress than nonparents.
Comentários: No terceiro parágrafo, é dito que
a) os pais estão se sentindo tão estressados quanto os não pais, de acordo com pesquisas.

AULA 04 – ADJECTIVES AND ADVERBS 70


TEACHER ANDREA BELO

b) os níveis de estresse aumentaram mais em não pais em comparação com os pais.


c) os não pais estão ultrapassando os pais nos níveis de estresse nas pesquisas.
d) pesquisas mostram que os pais são agora mais afetados pelo estresse do que os não pais. Stress
levels have risen, with parents outpacing nonparents in surveys, creating fertile ground for mental
health problems. = Os níveis de estresse aumentaram, com os pais ultrapassando os não-pais nas
pesquisas, criando um terreno fértil para problemas de saúde mental.
GABARITO: D

Questão 11 (AFA/INÉDITA) – Mark the alternative in which the problems described in paragraph
2 are correctly summarized.
a) employers are expecting remote-learning children to behave during work day.
b) right now, there is potential guilt with anything moms do.
c) women are leaving jobs in record numbers.
d) parenting lives were not precarious before the pandemic.
Comentários: Assinale a alternativa em que os problemas descritos no parágrafo 2 estão
corretamente resumidos.
a) os empregadores esperam que as crianças com aprendizagem remota se comportem durante
o dia de trabalho.
b) agora, há culpa potencial com qualquer coisa que as mães façam.
c) as mulheres estão deixando o emprego em número recorde.
d) a vida dos pais não era cautelosa antes da pandemia.
“Ilyse DiMarco, a clinical psychologist and the author of the upcoming book ‘Mom Brain’, sums
up the problem: ‘The issue with guilt right now is there is potential guilt with anything you do’.” =
Ilyse DiMarco, psicóloga clínica e autora do próximo livro “Mom Brain”, resume o problema: “O
problema com a culpa agora é que há culpa potencial em qualquer coisa que você faça”.
GABARITO: B

Questão 12 (AFA/INÉDITA) – Mark the option that contains the correct negative form for the
sentence “Stress levels have risen” (paragraph 3).
a) Stress levels not have risen.
b) Stress levels do not have risen.
c) Stress levels have not risen.
d) Stress levels have risen not.
Comentários: A forma correta de fazer a negativa da frase, que está no Present Perfect, é
acrescentando not após o auxiliar have = have not risen (poderia ser feita a contração também =
haven’t risen).
GABARITO: C

AULA 04 – ADJECTIVES AND ADVERBS 71


TEACHER ANDREA BELO

Questão 13 (AFA/INÉDITA) – The underlined expression “Then, stay committed to making it


happen.” (paragraph 13) means
a) stand still.
b) create a commission.
c) keep dedicated.
d) let go.
Comentários: A expressão sublinhada (parágrafo 13) significa manter-se comprometido = “Então,
mantenha o compromisso de fazer acontecer.”
a) ficar parado.
b) criar uma comissão.
c) manter-se dedicado.
d) deixar pra lá.
GABARITO: C

Questão 14 (AFA/INÉDITA) – Mark the option which shows the appropriate plural form for the
word “child” (paragraph 5).
a) childs.
b) childrens.
c) childen.
d) children.
Comentários: A opção que mostra a forma plural apropriada para a palavra child (criança ou filho)
é a letra D. O plural de child é irregular, não sendo formado com o mero acréscimo de -s.
Gabarito: D

Questão 15 (AFA/INÉDITA) – The expression “our already precarious parenting lives”


(paragraph 2) suggests that parenting lives
a) were unstable even before the pandemic.
b) became precarious after the pandemic.
c) were unlikely to collapse before the pandemic.
d) were steady before the pandemic.
Comentários: A expressão “nossa já precária vida de pais” (parágrafo 2) sugere que a vida de pais
a) eram instáveis mesmo antes da pandemia.
b) tornaram-se precárias após a pandemia.
c) eram improváveis de entrar em colapso antes da pandemia.
d) eram estáveis antes da pandemia.
GABARITO: A

AULA 04 – ADJECTIVES AND ADVERBS 72


TEACHER ANDREA BELO

QUESTÕES COLÉGIO NAVAL


Read Text I to do questions 01 to 04 based on it.
A wristband that tells your boss if you are unhappy
At first glance the silicone wristband could be mistaken for one that tracks your heart rate when
you are doing exercise.
However, the wearable technology, called a Moodbeam, isn't here to monitor your physical health.
Instead it allows your employer to track your emotional state.
The gadget, which links to a mobile phone app and web interface, has two buttons, one yellow
and one blue. The idea is that you press the yellow one if you are feeling happy, and the blue one
if you are sad.
Aimed at companies who wish to monitor the wellbeing of staff who are working from home, the
idea is that employees are encouraged to wear the wristband (they can say no), and press the
relevant button as they see fit throughout the working week.
Managers can then view an online dashboard to see how workers are feeling and coping. With
bosses no longer able to check in physically with their team, Moodbeam hopes to bridge the gap.
"Businesses are trying to get on top of staying connected with staff working from home. Here they
can ask 500 members: 'You ok?' without picking up the phone," says Moodbeam co-founder
Christina Colmer McHugh.
She originally came up with the idea for the product after she discovered that her daughter was
struggling at school, and she wanted a way for her child to let her know how she was feeling. The
wristband was launched commercially in 2016.
(Adapted from https://www.bbc.com/news/business-55637328)

Questão 01 (COLÉGIO NAVAL/INÉDITA) – Read the extract from the text.


“At first glance the silicone wristband could be mistaken for one that tracks your heart rate when
you are doing exercise”.
Mark the alternative that can replace the underlined word without changing its meaning.
(A) Peek
(B) Ignore
(C) Touch
(D) Stare
(E) Eye
Comentários: A alternativa A está correta. A palavra “glance” significa relance e pode ser
substituída pela palavra “peek”, que significa olhadinha.
A alternativa B está incorreta. A palavra “glance” significa relance e não pode ser substituída pela
palavra “ignore”, que significa ignorar.

AULA 04 – ADJECTIVES AND ADVERBS 73


TEACHER ANDREA BELO

A alternativa C está incorreta. A palavra “glance” significa relance e não pode ser substituída pela
palavra “touch”, que significa tocar.
A alternativa D está incorreta. A palavra “glance” significa relance e não pode ser substituída pela
palavra “stare”, que significa olhar fixamente.
A alternativa E está incorreta. A palavra “glance” significa relance e não pode ser substituída pela
palavra “eye”, que significa olho.
GABARITO: A

Questão 02 (COLÉGIO NAVAL/INÉDITA) – About the initial purpose of the wristband, it’s correct
to affirm that
(A) Fulfilled the final results
(B) Had a personal character
(C) Aimed at analyzing the mental health of the entire population
(D) Criticized the current teaching method of schools
(E) Had no purpose, considering it was a sudden discovery
Comentários: A alternativa A está incorreta. Sobre o objetivo inicial da pulseira, não é correto
afirmar que cumpriu com os resultados finais pois acabou tendo um desfecho diferente do
esperado. Isso pode ser confirmado com o trecho “She originally came up with the idea for the
product after she discovered that her daughter was struggling at school…”.
A alternativa B está correta. Sobre o objetivo inicial da pulseira, é correto afirmar que tinha um
caráter pessoal, considerando que a criadora confeccionou a pulseira para saber como sua filha
estava indo na escola, assim como esta opção indica. Isso pode ser confirmado com o trecho “She
originally came up with the idea for the product after she discovered that her daughter was
struggling at school, and she wanted a way for her child to let her know how she was feeling”.
A alternativa C está incorreta. Sobre o objetivo inicial da pulseira, não é correto afirmar que visava
analisar a saúde mental de toda a população, mas sim, que o objetivo original era descobrir os
sentimentos da filha e porque ela estava indo mal na escola. Isso pode ser confirmado com o
trecho “She originally came up with the idea for the product after she discovered that her
daughter was struggling at school, and she wanted a way for her child to let her know how she
was feeling”.
A alternativa D está incorreta. Sobre o objetivo inicial da pulseira, não é correto afirmar que
criticava o método de ensino atual das escolas, mas sim, que o objetivo original era descobrir os
sentimentos da filha e porque ela estava indo mal na escola. Isso pode ser confirmado com o
trecho “She originally came up with the idea for the product after she discovered that her
daughter was struggling at school, and she wanted a way for her child to let her know how she
was feeling”.
A alternativa E está incorreta. Sobre o objetivo inicial da pulseira, não é correto afirmar que não
possuía objetivo, considerando a descoberta repentina, mas sim, que o objetivo original era
descobrir os sentimentos da filha e porque ela estava indo mal na escola. Isso pode ser confirmado
com o trecho “She originally came up with the idea for the product after she discovered that her

AULA 04 – ADJECTIVES AND ADVERBS 74


TEACHER ANDREA BELO

daughter was struggling at school, and she wanted a way for her child to let her know how she
was feeling”.
GABARITO: B

Questão 03 (COLÉGIO NAVAL/INÉDITA) – Read the extract from the text


“The wristband was launched commercially in 2016”
The sentence above means that
(A) The gadget was created in 2016
(B) The gadget has been on the market for over a decade
(C) The gadget was not launched on the market
(D) The gadget won over consumers in 2016
(E) The gadget was effectively placed on the market in 2016
Comentários: A alternativa A está incorreta. A frase “A pulseira foi lançada comercialmente em
2016” não significa que o dispositivo foi criado em 2016, mas sim, que foi efetivamente colocado
no mercado em 2016.
A alternativa B está incorreta. A frase “A pulseira foi lançada comercialmente em 2016” não
significa que o dispositivo está no mercado há mais de uma década, mas sim, que foi efetivamente
colocado no mercado em 2016.
A alternativa C está incorreta. A frase “A pulseira foi lançada comercialmente em 2016” não
significa que o dispositivo não foi lançado no mercado, mas sim, que foi efetivamente colocado no
mercado em 2016.
A alternativa D está incorreta. A frase “A pulseira foi lançada comercialmente em 2016” não
significa que o dispositivo conquistou os consumidores em 2016, mas sim, que foi efetivamente
colocado no mercado em 2016.
A alternativa E está correta. A frase “A pulseira foi lançada comercialmente em 2016” significa que
o dispositivo foi efetivamente colocado no mercado em 2016, assim como esta opção indica.
GABARITO: E

Questão 04 (COLÉGIO NAVAL/INÉDITA) – What can we infer from the text? Mark the correct
option
(A) The final use of the gadget was for the parents’ emotional control over their children
(B) The gadget enabled remote team control
(C) The gadget system is very complex
(D) Companies tend not to care about the emotional health of employees
(E) The wristband is a gadget that does not adapt to the context of a pandemic

AULA 04 – ADJECTIVES AND ADVERBS 75


TEACHER ANDREA BELO

Comentários: A alternativa A está incorreta. Não é correto afirmar que o uso final do dispositivo
foi para o controle emocional dos pais sobre seus filhos, mas sim, que o uso final foi por empresas
para controlarem os sentimentos de seus empregados. Isso pode ser confirmado com o trecho
“Aimed at companies who wish to monitor the wellbeing of staff who are working from home, the
idea is that employees are encouraged to wear the wristband (they can say no), and press the
relevant button as they see fit throughout the working week”.
A alternativa B está correta. É correto afirmar que o dispositivo possibilitou um controle de equipe
remoto, assim como esta opção indica. Isso pode ser confirmado com o trecho “Aimed at
companies who wish to monitor the wellbeing of staff who are working from home, the idea is
that employees are encouraged to wear the wristband (they can say no), and press the relevant
button as they see fit throughout the working week”.
A alternativa C está incorreta. Não é correto afirmar que o sistema do dispositivo é muito
complexo, mas sim, que é bem simples, com dois botões para feliz ou triste. Isso pode ser
confirmado com o trecho “The gadget, which links to a mobile phone app and web interface, has
two buttons, one yellow and one blue. The idea is that you press the yellow one if you are feeling
happy, and the blue one if you are sad”.
A alternativa D está incorreta. Não é correto afirmar que as empresas tendem a não se importar
com a saúde emocional de seus empregados, mas sim, que elas se importam. Isso pode ser
confirmado com o trecho “Aimed at companies who wish to monitor the wellbeing of staff who
are working from home…”.
A alternativa E está incorreta. Não é correto afirmar que a pulseira é um dispositivo que não se
adapta ao contexto de pandemia, mas sim, que se adapta bem ao contexto remoto. Isso pode ser
confirmado com o trecho “Aimed at companies who wish to monitor the wellbeing of staff who
are working from home…”.
GABARITO: B

Read the text II to do items 05 to 10.


Vegan Restaurant Gets Michelin Star in France, a First
Lemongrass, seaweed and fir — yes, the tree — are not the sorts of ingredients that once earned
French chefs plaudits in the Michelin Guide, but on Monday the thick red bible of gastronomy
announced that it was giving its first star to a fully vegan restaurant in France.
The bestowal of the star to ONA, a restaurant near Bordeaux, is more evidence that a country long
renown for classics like coq au vin, blanquette de veau and boeuf bourguignon has opened up to
animal-free cuisine. A growing number of chefs are cutting meat from their menus, sometimes
entirely.
“It’s a new movement in France, where diets are still very meat-based,” said Claire Vallée, the chef
at ONA, which opened five years ago in Arès, a small Atlantic Coast town about 25 miles west of
Bordeaux.
“Each has its place,” Ms. Vallée said. “We want to show that you can eat differently.”

AULA 04 – ADJECTIVES AND ADVERBS 76


TEACHER ANDREA BELO

The restaurant’s very name speaks to France’s shifting culinary landscape: It stands for origine
non-animale.
Vegan establishments have already received Michelin stars in the United States, Spain and
Germany. But this is a first for France.
ONA shuns all animal-based products, Ms. Vallée said, even in its decorations and furnishings. It
does not, for instance, use wool or leather.
Last fall, its seven-course menu featured dishes with intriguing combinations of fir, boletus
mushroom and sake, or dulse seaweed, lemongrass and galangal, a relative of ginger. The
restaurant is currently closed because of the coronavirus pandemic.
The Michelin Guide, in a statement this week, said Ms. Vallée had given vegan cuisine its “letters
of nobility.”
Gwendal Poullenec, the international head of the Michelin Guides, noted that the move away
from meat was not entirely new.
Alain Passard, the owner and chef at L’Arpège in Paris, removed meat from his menu two decades
ago, and Alain Ducasse, perhaps France’s biggest culinary superstar, also decided to drastically
reduce the amount of meat used at his flagship Parisian restaurant.
But awarding a star to a restaurant that is not just meatless but avowedly vegan has the potential
to shake things up even further, Mr. Poullenec said.
“The general public might not associate pure veganism with a gastronomical experience,” he said.
A Michelin star might “liberate” chefs who are still reluctant to explore plant-based cooking, he
said.
(Adapted from https://www.nytimes.com/2021/01/19/world/europe/ona-michelin-star-france.html?action=click&module=News&pgtype=Homepage)

Questão 05 (COLÉGIO NAVAL/INÉDITA) – According to the text, mark the INCORRECT option
(A) Lemongrass is an example of ingredient that normally does not predominantly compose
Michelin-starred restaurant dishes
(B) There is an entirely vegan restaurant, starring with the Michelin award, in France
(C) The majority of the French population is adept at the vegan movement
(D) There are chefs who have a meatless menu
(E) France isn’t the only country with Michelin-starred vegan restaurants
Comentários: A alternativa A está incorreta. É correto afirmar que o capim-limão é um exemplo
de ingrediente que normalmente não compõe predominantemente um prato de restaurante com
estrela Michelin, o que faz com que esta seja uma opção com informações corretas. Isso pode ser
confirmado com o trecho “Lemongrass, seaweed and fir — yes, the tree — are not the sorts of
ingredients that once earned French chefs plaudits in the Michelin Guide…”.
A alternativa B está incorreta. É correto afirmar que existe um restaurante totalmente vegano,
estrelado pelo prêmio Michelin, na França, o que faz com que esta seja uma opção com
informações corretas. Isso pode ser confirmado com o trecho “...but on Monday the thick red bible
of gastronomy announced that it was giving its first star to a fully vegan restaurant in France”.

AULA 04 – ADJECTIVES AND ADVERBS 77


TEACHER ANDREA BELO

A alternativa C está correta. Não é correto afirmar que a maioria da população francesa é adepta
ao movimento vegano, mas sim, que esse é um país de carnívoros, o que faz com que esta seja
uma opção com informações incorretas. Isso pode ser confirmado com o trecho “It’s a new
movement in France, where diets are still very meat-based”.
A alternativa D está incorreta. É correto afirmar que existem chefs que possuem cardápios sem
carne, o que faz com que esta seja uma opção com informações corretas. Isso pode ser confirmado
com o trecho “ONA shuns all animal-based products, Ms. Vallée said, even in its decorations and
furnishings. It does not, for instance, use wool or leather”.
A alternativa E está incorreta. É correto afirmar que a França não é o único país com restaurantes
veganos que são premaiados com estrelas Michelin, o que faz com que esta seja uma opção com
informações corretas. Isso pode ser confirmado com o trecho “Vegan establishments have already
received Michelin stars in the United States, Spain and Germany. But this is a first for France”.
GABARITO: C

Questão 06 (COLÉGIO NAVAL/INÉDITA) – Read the extract from the text


“ONA shuns all animal-based products, Ms. Vallée said, even in its decorations and furnishings. It
does not, for instance, use wool or leather”
Mark the option that can replace the underlined sentence without changing its meaning
(A) ONA uses all animal-based products
(B) ONA sells all animal-based products
(C) ONA is indifferent to all animal-based products
(D) ONA despises all animal-based products
(E) ONA approves all animal-based products
Comentários: A alternativa A está incorreta. A frase “ONA evita todos os produtos com base
animal” não pode ser substituída pela frase “ONA usa todos os produtos com base animal”, mas
sim, pela frase “ONA despreza todos os produtos com base animal”.
A alternativa B está incorreta. A frase “ONA evita todos os produtos com base animal” não pode
ser substituída pela frase “ONA vende todos os produtos com base animal”, mas sim, pela frase
“ONA despreza todos os produtos com base animal”.
A alternativa C está incorreta. A frase “ONA evita todos os produtos com base animal” não pode
ser substituída pela frase “ONA é indiferente à todos os produtos com base animal”, mas sim, pela
frase “ONA despreza todos os produtos com base animal”.
A alternativa D está correta. A frase “ONA evita todos os produtos com base animal” pode ser
substituída pela frase “ONA despreza todos os produtos com base animal”, assim como esta opção
indica.
A alternativa E está incorreta. A frase “ONA evita todos os produtos com base animal” não pode
ser substituída pela frase “ONA aprova todos os produtos com base animal”, mas sim, pela frase
“ONA despreza todos os produtos com base animal”.
GABARITO: D

AULA 04 – ADJECTIVES AND ADVERBS 78


TEACHER ANDREA BELO

Questão 07 (COLÉGIO NAVAL/INÉDITA) – Read the extract from the text


“It does not, for instance, use wool or leather” (Paragraph 7)
The word “It” refers to
(A) ONA
(B) Wool
(C) Leather
(D) Animal-based products
(E) Vegan establishments
Comentários: A alternativa A está correta. A palavra “it” se refere à ONA, que é citada
anteriormente na sentença, assim como esta opção indica. Isso pode ser confirmado com o trecho
“ONA shuns all animal-based products, Ms. Vallée said, even in its decorations and furnishings. It
does not, for instance, use wool or leather”.
A alternativa B está incorreta. A palavra “it” não se refere a lã, mas sim, à ONA, que é citada
anteriormente na sentença. Isso pode ser confirmado com o trecho “ONA shuns all animal-based
products, Ms. Vallée said, even in its decorations and furnishings. It does not, for instance, use
wool or leather”.
A alternativa C está incorreta. A palavra “it” não se refere a couro, mas sim, à ONA, que é citada
anteriormente na sentença. Isso pode ser confirmado com o trecho “ONA shuns all animal-based
products, Ms. Vallée said, even in its decorations and furnishings. It does not, for instance, use
wool or leather”.
A alternativa D está incorreta. A palavra “it” não se refere a produtos com base animal, mas sim,
à ONA, que é citada anteriormente na sentença. Isso pode ser confirmado com o trecho “ONA
shuns all animal-based products, Ms. Vallée said, even in its decorations and furnishings. It does
not, for instance, use wool or leather”.
A alternativa E está incorreta. A palavra “it” não se refere a estabelecimentos veganos, mas sim, à
ONA, que é citada anteriormente na sentença. Isso pode ser confirmado com o trecho “ONA shuns
all animal-based products, Ms. Vallée said, even in its decorations and furnishings. It does not, for
instance, use wool or leather”.
GABARITO: A

Questão 08 (COLÉGIO NAVAL/INÉDITA) – Claire Vallée


(A) Does not identify herself with veganism
(B) Thinks that the vegan movement in France opposes the reality of French food
(C) Does not want to diversify food in France
(D) Has an animal-free menu, only
(E) Has no decor in her restaurant

AULA 04 – ADJECTIVES AND ADVERBS 79


TEACHER ANDREA BELO

Comentários: A alternativa A está incorreta. Não é correto afirmar que Claire Vallée não se
identifica com o veganismo, mas sim, que se identifica e tem um restaurante vegano. Isso pode
ser confirmado com o trecho “ONA shuns all animal-based products, Ms. Vallée said”.
A alternativa B está correta. É correto afirmar que Claire Vallée pensa que o movimento vegano se
opõe a realidade da alimentação francesa, assim como esta opção indica. Isso pode ser
confirmado com o trecho “It’s a new movement in France, where diets are still very meat-based”.
A alternativa C está incorreta. Não é correto afirmar que Claire Vallée não quer diversificar
alimentos na França, mas sim, que quer. Isso pode ser confirmado com o trecho “We want to show
that you can eat differently”.
A alternativa D está incorreta. Não é correto afirmar que Claire Vallée possui um cardápio livre de
animais, apenas, mas sim, que todo o restaurante é vegano. Isso pode ser confirmado com o
trecho “ONA shuns all animal-based products, Ms. Vallée said, even in its decorations and
furnishings. It does not, for instance, use wool or leather”.
A alternativa E está incorreta. Não é correto afirmar que Claire Vallée não tem decoração em seu
restaurante, mas sim, que ela tem um decoração vegana. Isso pode ser confirmado com o trecho
“ONA shuns all animal-based products, Ms. Vallée said, even in its decorations and furnishings. It
does not, for instance, use wool or leather”.
GABARITO: B

Questão 09 (COLÉGIO NAVAL/INÉDITA) – The expression “this is a first”, in paragraph 6, means


(A) That something or someone won first place
(B) That something is indifferent to someone
(C) That something ordinary happened
(D) That something unexpected happened
(E) That something totally new has happened
Comentários: A alternativa A está incorreta. A expressão “esta é a primeira vez” não significa que
algo ou alguém ganhou em primeiro lugar, mas sim, que algo totalmente novo aconteceu.
A alternativa B está incorreta. A expressão “esta é a primeira vez” não significa que algo é
indiferente para alguém, mas sim, que algo totalmente novo aconteceu.
A alternativa C está incorreta. A expressão “esta é a primeira vez” não significa que algo corriqueiro
aconteceu, mas sim, que algo totalmente novo aconteceu.
A alternativa D está incorreta. A expressão “esta é a primeira vez” não significa que algo
inesperado aconteceu, mas sim, que algo totalmente novo aconteceu.
A alternativa E está correta. A expressão “esta é a primeira vez” significa que algo totalmente novo
aconteceu, assim como esta opção indica.
GABARITO: E

AULA 04 – ADJECTIVES AND ADVERBS 80


TEACHER ANDREA BELO

Questão 10 (COLÉGIO NAVAL/INÉDITA) – The word “bestowal” (paragraph 2), can be replaced
by the word __________ without changing its meaning
(A) Withdrawal
(B) Grant
(C) Break
(D) Disagreement
(E) Give
Comentários: A alternativa A está incorreta. A palavra “bestowal” significa concessão e não pode
ser substituída pela palavra “withdrawal”, que significa retirada.
A alternativa B está correta. A palavra “bestowal” significa concessão e pode ser substituída pela
palavra “grant”, que significa concessão.
A alternativa C está incorreta. A palavra “bestowal” significa concessão e não pode ser substituída
pela palavra “break”, que significa quebra.
A alternativa D está incorreta. A palavra “bestowal” significa concessão e não pode ser substituída
pela palavra “disagreement”, que significa desacordo.
A alternativa E está incorreta. A palavra “bestowal” significa concessão e não pode ser substituída
pela palavra “give”, que significa dar.
GABARITO: B

QUESTÕES EAM
Read text I and answer questions 01 and 02
Why you’re more creative in coffee shops
If we’re already working in isolation at home, why do we miss working with our heads
similarly down in a public setting?
Some of the most successful people in history have done their best work in coffee shops.
Pablo Picasso, JK Rowling, Simone de Beauvoir and Jean-Paul Sartre, Bob Dylan – whether they’re
painters, singersongwriters, philosophers or writers, people across nations and centuries have
tapped into their creativity working away at a table in a café.
Of course, Covid-19 has put the kibosh on lingering for hours in cozy rooms packed with people
sipping lattes. As we begin another year living amid a pandemic, many of us continue to work
remotely on our own. And if remote work becomes permanent for some – as many experts predict
– we might ask ourselves why, when things settle down, we should bother going back out to work
in public, only to ostensibly isolate ourselves with our heads down – something we’re already
doing at home.
But putting on your noise-cancelling headphones to toil away at your desk is actually different
than doing the same surrounded by other people buzzing over your shoulder. There are many
ways coffee shops trigger our creativity in a way offices and homes don’t. Research shows that
the stimuli in these places make them effective environments to work; the combination of noise,

AULA 04 – ADJECTIVES AND ADVERBS 81


TEACHER ANDREA BELO

casual crowds and visual variety can give us just the right amount of distraction to help us be our
sharpest and most creative. (So, no, it’s not just that double espresso.)
A sweet spot of noise and crowds
Some of us stick in our earbuds as soon as we sit down to work in a public setting. But scientists
have known for years that background noise can benefit our creative thinking.
A 2012 study published in the Journal of Consumer Research showed that a low-to-moderate level
of ambient noise in a place like a cafeteria can actually boost your creative output. The idea is that
if you’re very slightly distracted from the task at hand by ambient stimuli, it boosts your abstract
thinking ability, which can lead to more creative idea generation.
(Adapted from https://www.bbc.com/worklife/article/20210114-why-youre-more-creative-in-coffee-shops)

Questão 01 (EAM/INÉDITA) – Say if the following statements are T (true) or F (false) about
coffee shops and the current world context. Then, mark the correct option, from top to bottom.
( ) Coffee shops are a good place to work
( ) Cafeterias are suitable workplaces for a certain type of profession
( ) Covid-19 did not change the work routine of most workers
( ) Remote work is a reality for many people, considering the current context
( ) It is possible that there is a general post-pandemic doubt among workers regarding the
workplace
(A) T – F – F – T – F
(B) T – F – F – F – F
(C) F – F – F – T – T
(D) F – F – F – F – T
(E) T – F – F – T – T
Comentários: A primeira afirmativa é verdadeira (T – true). É correto afirmar que cafeterias são
um bom lugar pra trabalhar. Isso pode ser confirmado com o trecho “...people across nations and
centuries have tapped into their creativity working away at a table in a café”.
A segunda afirmativa é falsa (F – false). Não é correto afirmar que as cafeterias são um bom local
de trabalho para um certo tipo de profissão, mas sim, que diversos tipos de pessoas se tornaram
mais criativas em uma cafeterias. Isso pode ser confirmado com o trecho “…whether they’re
painters, singer-songwriters, philosophers or writers, people across nations and centuries have
tapped into their creativity working away at a table in a café”.
A terceira afirmativa é falsa (F – false). Não é correto afirmar que a Covid-19 não mudou a rotina
de trabalho da maioria dos trabalhadores, mas sim, que mudou. Isso pode ser confirmado com o
trecho “...Covid-19 has put the kibosh on lingering for hours in cozy rooms packed with people
sipping lattes”.
A quarta afirmativa é verdadeira (T – true). É correto afirmar que o trabalho remoto é a realidade
para muitas pessoas, considerando o contexto atual. Isso pode ser confirmado com o trecho “And

AULA 04 – ADJECTIVES AND ADVERBS 82


TEACHER ANDREA BELO

if remote work becomes permanent for some – as many experts predict – we might ask ourselves
why, when things settle down…”.
A quinta afirmativa é verdadeira (T – true). É correto afirmar que é possível que haja uma dúvida
pós-pandêmica geral entre os trabalhadores em relação ao local de trabalho. Isso pode ser
confirmado com o trecho “...we should bother going back out to work in public, only to ostensibly
isolate ourselves with our heads down – something we’re already doing at home”.
GABARITO: E

Questão 02 (EAM/INÉDITA) – About the benefits resulting from the cafeteria as a workplace, it
is correct to say that
(A) The place with a lot of noise ends up affecting the creative process of the worker
(B) Background noise pollution contributes to the development of work
(C) According to studies, noise pollution is one of the main reasons for the creative block
(D) The ability of abstract thinking is not affected by noise pollution
(E) The double espresso is the main reason why the coffee shop is a good place to work
Comentários: A alternativa A está incorreta. Sobre os benefícios resultantes da cafeteria como
um bom local de trabalho, não é correto afirmar que um lugar com muito barulho acaba afetando
o processo criativo do trabalhador, mas sim, que ajuda no processo criativo. Isso pode ser
confirmado com o trecho “But scientists have known for years that background noise can benefit
our creative thinking”.
A alternativa B está correta. Sobre os benefícios resultantes da cafeteria como um bom local de
trabalho, é correto afirmar que a poluição sonora de fundo contribui para o desenvolvimento do
trabalho, assim como esta opção indica. Isso pode ser confirmado com o trecho “But scientists
have known for years that background noise can benefit our creative thinking”.
A alternativa C está incorreta. Sobre os benefícios resultantes da cafeteria como um bom local de
trabalho, não é correto afirmar que segundo estudos, a poluição sonora é um dos principais
motivos do bloqueio criativo, mas sim, que a criatividade pode ser acelerada por esses locais. Isso
pode ser confirmado com o trecho “A 2012 study published in the Journal of Consumer Research
showed that a low-to-moderate level of ambient noise in a place like a cafeteria can actually boost
your creative output”.
A alternativa D está incorreta. Sobre os benefícios resultantes da cafeteria como um bom local de
trabalho, não é correto afirmar que a capacidade de pensamento abstrato não é afetada pela
poluição sonora, mas sim, que é positivamente afetada.
Isso pode ser confirmado com o trecho “...it boosts your abstract thinking ability, which can lead
to more creative idea generation”.
A alternativa E está incorreta. Sobre os benefícios resultantes da cafeteria como um bom local de
trabalho, não é correto afirmar que o expresso duplo é o principal motivo pelo qual a cafeteria é
um bom lugar para se trabalhar, mas sim, que não é. Isso pode ser confirmado com o trecho “So,
no, it’s not just that double espresso”.
GABARITO: B

AULA 04 – ADJECTIVES AND ADVERBS 83


TEACHER ANDREA BELO

Read text II and answer questions 03 and 04


Electric cars are not the only green solution
Chris Barker says more cars of any kind are bad news for pollution, while Dr Robin Russell-
Jones believes British entrepreneurs can rise to the challenge of ‘green growth’
It is indeed good news that the advance in battery technology brings down the costs and
inconveniences associated with electric cars. But it does not begin to solve the other problems
associated with the volume of traffic on the roads. In fact, if electric cars become cheaper and
continue to benefit from tax relief, the total number of cars may well rise. Electric cars produce
no toxic emissions at the point of use but, as with all cars, embodied carbon in the production of
vehicles and batteries generate greenhouse gas emissions. In addition, research by the
government in 2019 found that two-thirds of the contamination from conventional vehicles arises
from tyre, brake and road dust.
There are also other problems associated with too many cars of any type. The greater the level of
traffic, the greater the danger of death or injury and the need for more and wider roads, creating
no-go areas for people and cutting off communities from each other. Too much reliance on cars
promotes a sedentary lifestyle and brings health risks.
For a better life and to counteract the climate emergency it is necessary to reduce traffic. This
cannot be done by promoting electric cars. It needs the promotion of walking, cycling and public
transport, and of measures to reduce the need to travel.
(Adapted from https://www.theguardian.com/environment/2021/jan/25/electric-cars-are-not-the-only-green-solution)

Questão 03 (EAM/INÉDITA) – In the sentence “But it does not begin to solve the other problems
associated with the volume of traffic on the roads” (paragraph 1), the pronoun “it” refers to
(A) Advance in battery technology
(B) Electric cars
(C) Costs associated with electric cars
(D) Volume of traffic on the roads
(E) Tax relief
Comentários: A alternativa A está correta. O pronome “it”, neste caso, se refere ao avanço em
tecnologia de bateria, assim como esta opção indica. Isso pode ser confirmado com o trecho “It is
indeed good news that the advance in battery technology brings down the costs and
inconveniences associated with electric cars. But it does not begin to solve the other problems
associated with the volume of traffic on the roads” (Na verdade, é uma boa notícia que o avanço
na tecnologia de baterias reduza os custos e os inconvenientes associados aos carros elétricos.
Mas não começa a resolver os outros problemas associados ao volume de tráfego nas estradas).
A alternativa B está incorreta. O pronome “it”, neste caso, não se refere aos carros elétricos, mas
sim, ao avanço em tecnologia de bateria. Isso pode ser confirmado com o trecho “It is indeed
good news that the advance in battery technology brings down the costs and inconveniences
associated with electric cars. But it does not begin to solve the other problems associated with
the volume of traffic on the roads” (Na verdade, é uma boa notícia que o avanço na tecnologia de

AULA 04 – ADJECTIVES AND ADVERBS 84


TEACHER ANDREA BELO

baterias reduza os custos e os inconvenientes associados aos carros elétricos. Mas não começa a
resolver os outros problemas associados ao volume de tráfego nas estradas).
A alternativa C está incorreta. O pronome “it”, neste caso, não se refere aos custos associado aos
carros elétricos, mas sim, ao avanço em tecnologia de bateria. Isso pode ser confirmado com o
trecho “It is indeed good news that the advance in battery technology brings down the costs and
inconveniences associated with electric cars. But it does not begin to solve the other problems
associated with the volume of traffic on the roads” (Na verdade, é uma boa notícia que o avanço
na tecnologia de baterias reduza os custos e os inconvenientes associados aos carros elétricos.
Mas não começa a resolver os outros problemas associados ao volume de tráfego nas estradas).
A alternativa D está incorreta. O pronome “it”, neste caso, não se refere ao volume de tráfego nas
estradas, mas sim, ao avanço em tecnologia de bateria. Isso pode ser confirmado com o trecho
“It is indeed good news that the advance in battery technology brings down the costs and
inconveniences associated with electric cars. But it does not begin to solve the other problems
associated with the volume of traffic on the roads” (Na verdade, é uma boa notícia que o avanço
na tecnologia de baterias reduza os custos e os inconvenientes associados aos carros elétricos.
Mas não começa a resolver os outros problemas associados ao volume de tráfego nas estradas).
A alternativa E está incorreta. O pronome “it”, neste caso, não se refere a redução de impostos,
mas sim, ao avanço em tecnologia de bateria. Isso pode ser confirmado com o trecho “It is indeed
good news that the advance in battery technology brings down the costs and inconveniences
associated with electric cars. But it does not begin to solve the other problems associated with
the volume of traffic on the roads” (Na verdade, é uma boa notícia que o avanço na tecnologia de
baterias reduza os custos e os inconvenientes associados aos carros elétricos. Mas não começa a
resolver os outros problemas associados ao volume de tráfego nas estradas).
GABARITO: A

Questão 04 (EAM/INÉDITA) – It is FALSE to say that


(A) Battery technology lowers electric car costs
(B) Decreasing costs can increase the number of cars on the roads
(C) Electric cars have no benefits, considering the arguments of the text
(D) The most effective way to decrease carbon emissions is by decreasing traffic
(E) Other modes of locomotion should be motivated
Comentários: A alternativa A está incorreta. É correto afirmar que a tecnologia de bateria diminui
os custos do carro elétrico, o que faz com que esta seja uma opção com informações corretas.
Isso pode ser confirmado com o trecho “It is indeed good News that the advance in battery
technology brings down the costs and inconveniences associated with electric cars”.
A alternativa B está incorreta. É correto afirmar que diminuir os custos pode aumentar o número
de carros nas estradas, o que faz com que esta seja uma opção com informações corretas. Isso
pode ser confirmado com o trecho “In fact, if electric cars become cheaper and continue to benefit
from tax relief, the total number of cars may well rise”.

AULA 04 – ADJECTIVES AND ADVERBS 85


TEACHER ANDREA BELO

A alternativa C está correta. Não é correto afirmar que os carros elétricos não possuem benefícios,
de acordo com os argumentos do texto, mas sim, que possui alguns benefícios mas muitos
malefícios, o que faz com que esta seja uma opção com informações incorretas. Isso pode ser
confirmado com o trecho “Electric cars produce no toxic emissions at the point of use but, as with
all cars, embodied carbon in the production of vehicles and batteries generate greenhouse gas
emissions”.
A alternativa D está incorreta. É correto afirmar que o modo mais efetivo de diminuir a emissão
de gases é diminuindo o tráfego, o que faz com que esta seja uma opção com informações
corretas. Isso pode ser confirmado com o trecho “For a better life and to counteract the climate
emergency it is necessary to reduce traffic”.
A alternativa E está incorreta. É correto afirmar que outros modos de locomoção devem ser
motivados, o que faz com que esta seja uma opção com informações corretas. Isso pode ser
confirmado com o trecho “It needs the promotion of walking, cycling and public transport, and of
measures to reduce the need to travel”.
GABARITO: C

Questão 05 (EAM/INÉDITA) –) Complete the paragraph below, about Britain’s class system, with
the missing prepositions Britain’s class system determines who is wealthy, who has power,
_______ even who lives. Before Covid-19 arrived, _______ you were a man in one of England’s
poorest communities, you could expect _______ live nine years fewer than someone in one of
its most affluent areas.
(Adapted from https://www.theguardian.com/commentisfree/2021/jan/26/combination-covid-class-devastating-britain-poorest)

(A) And / if / to
(B) With / if / for
(C) And / if / for
(D) And / and / for
(E) And / X / to
Comentários: A primeira lacuna deve ser preenchida com “and”, pois a frase nos indica uma
adição a ser feita à frase anterior, ou seja, “Britain’s class system determines who is wealthy, who
has power, and even who lives” (O sistema de classes da Grã-Bretanha determina quem é rico,
quem tem poder e até quem vive).
A segunda lacuna deve ser preenchida com “if”, pois a frase nos infere uma possibilidade, ou seja,
“Before Covid-19 arrived, if you were a man in one of England’s poorest communities...” (Antes
da chegada de Covid-19, se você fosse um homem em uma das comunidades mais pobres...).
A terceira lacuna deve ser preenchida com “to”, pois a frase nos indica que você poderia esperar
viver muito menos do que um rico, se fosse pobre na Inglaterra; ou seja, “...you could expect to
live nine years fewer than someone in one of its most affluent areas” (...poderia esperar viver
nove anos a menos do que alguém em uma de suas áreas mais ricas).
GABARITO: A

AULA 04 – ADJECTIVES AND ADVERBS 86


TEACHER ANDREA BELO

QUESTÕES EEAR
Read the text and answer questions 01, 02, 03 and 04
Sinovac: Brazil results show Chinese vaccine 50.4% effective
A coronavirus vaccine developed by China's Sinovac has been found to be 50.4% effective in
Brazilian clinical trials, according to the latest results released by researchers.
It shows the vaccine is significantly less effective than previous data suggested - barely over the
50% needed for regulatory approval.
The Chinese vaccine is one of two that the Brazilian government has lined up. Brazil has been one
of the countries worst affected by Covid-19.
Sinovac, a Beijing-based biopharmaceutical company, is behind CoronaVac, an inactivated
vaccine. It works by using killed viral particles to expose the body's immune system to the virus
without risking a serious disease response.
Several countries, including Indonesia, Turkey and Singapore, have placed orders for the vaccine.
Last week researchers at the Butantan Institute, which has been conducting the trials in Brazil,
announced that the vaccine had a 78% efficacy against "mild-to-severe" Covid-19 cases.
But on Tuesday they revealed that calculations for this figure did not include data from a group of
"very mild infections" among those who received the vaccine that did not require clinical
assistance.
With the inclusion of this data, the efficacy rate is now 50.4%, said researchers.
But Butantan stressed that the vaccine is 78% effective in preventing mild cases that needed
treatment and 100% effective in staving off moderate to serious cases.
The Sinovac trials have yielded different results across different countries.
Last month Turkish researchers said the Sinovac vaccine was 91.25% effective, while Indonesia,
which rolled out its mass vaccination programme on Wednesday, said it was 65.3% effective. Both
were interim results from late-stage trials.
(Adapted from https://www.bbc.com/news/world-latin-america-55642648)

Questão 01 (EEAR/INÉDITA) – According to the text, we can infer that


(A) The Chinese vaccine has a high rate of effectiveness, according to Brazilian studies
(B) CoronaVac is the only vaccine option of the Brazilian government
(C) CoronaVac's origins is unknown
(D) The first calculation of the total effectiveness of CoronaVac did not include a significant
group of infected
Comentários: A alternativa A está incorreta. De acordo com o texto, não é correto afirmar que a
vacina Chinesa tem uma alta taxa de eficácia, de acordo com estudos brasileiros, mas sim, que
possui 50,4% de eficácia. Isso pode ser confirmado com o trecho “A coronavírus vaccine
developed by China’s Sinovac has been found to be 50,4% effecctive in Brazilian clinial trials…”.

AULA 04 – ADJECTIVES AND ADVERBS 87


TEACHER ANDREA BELO

A alternativa B está incorreta. De acordo com o texto, não é correto afirmar que a CoronaVac é a
única opção de vacina do governo brasileiro, mas sim, que o mesmo possui duas opções. Isso pode
ser confirmado com o trecho “The Chinese vaccine is one of two that the Brazilian government
has lined up”.
A alternativa C está incorreta. De acordo com o texto, não é correto afirmar que a origem da
CoronaVac é desconhecida, mas sim, que é uma vacina inativada, feita pela Sinovac. Isso pode ser
confirmado com o trecho “Sinovac, a Beijing-based biopharmaceutical company, is behind
CoronaVac, an inactivated vaccine”. A alternativa D está correta. De acordo com o texto, é correto
afirmar que o primeiro cálculo de eficácia total da CoronaVac não incluiu um grupo importante
de infectados, assim como esta opção indica. Isso pode ser confirmado com o trecho “But on
Tuesday they revealed that calculations for this figure did not include data from a group of "very
mild infections" among those who received the vaccine that did not require clinical assistance”.
GABARITO: D

Questão 02 (EEAR/INÉDITA) – The words DEVELOPED and ANNOUNCED are


(A) Regular verbs
(B) Not verbs
(C) Irregular verbs
(D) Modal verbs
Comentários: A alternativa A está correta. As palavras “developed” (desenvolveu) e “announced”
(anunciou) são verbos regulares, que utilizam o -ed ao final d verbo em sua forma no passado,
assim como esta opção indica.
A alternativa B está incorreta. As palavras “developed” (desenvolveu) e “announced” (anunciou)
são verbos, e regulares, que utilizam o -ed ao final d verbo em sua forma no passado.
A alternativa C está incorreta. As palavras “developed” (desenvolveu) e “announced” (anunciou)
não são verbos irregulares, mas sim, verbos regulares, que utilizam o -ed ao final d verbo em sua
forma no passado.
A alternativa D está incorreta. As palavras “developed” (desenvolveu) e “announced” (anunciou)
não são verbos modais, mas sim, verbos regulares, que utilizam o -ed ao final d verbo em sua
forma no passado.
GABARITO: A

Questão 03 (EEAR/INÉDITA) – to the text, choose the best response


(A) The first effectiveness rate announced by Butantan Institute was wrong
(B) The first effectiveness rate announced by Butantan Institute was miscalculated
(C) The first effectiveness rate announced by Butantan Institute was lower than the second
(D) The first effectiveness rate announced by Butantan Institute was the right one

AULA 04 – ADJECTIVES AND ADVERBS 88


TEACHER ANDREA BELO

Comentários: A alternativa A está incorreta. De acordo com o texto, a primeira taxa de eficácia
anunciada pelo instituto butantan não estava errada, mas sim, que foi mal calculada. Isso pode
ser confirmado com o trecho “But on Tuesday they revealed that calculations for this figure did
not include data from a group of ‘very mild infections’ among those who received the vaccine that
did not require clinical assistance”.
A alternativa B está correta. De acordo com o texto, a primeira taxa de eficácia anunciada pelo
instituto butantan foi mal calculada, assim como esta opção indica. Isso pode ser confirmado com
o trecho “But on Tuesday they revealed that calculations for this figure did not include data from
a group of ‘very mild infections’ among those who received the vaccine that did not require clinical
assistance”.
A alternativa C está incorreta. De acordo com o texto, a primeira taxa de eficácia anunciada pelo
instituto butantan não foi mais baixa do que a segunda, mas sim, que foi mal calculada. Isso pode
ser confirmado com o trecho “But on Tuesday they revealed that calculations for this figure did
not include data from a group of ‘very mild infections’ among those who received the vaccine that
did not require clinical assistance”.
A alternativa D está incorreta. De acordo com o texto, a primeira taxa de eficácia anunciada pelo
instituto butantan não era a certa, mas sim, que foi mal calculada. Isso pode ser confirmado com
o trecho “But on Tuesday they revealed that calculations for this figure did not include data from
a group of ‘very mild infections’ among those who received the vaccine that did not require clinical
assistance”.
GABARITO: B

Questão 04 (EEAR/INÉDITA) – According to the text, the effectiveness of the Sinovac vaccine
(A) Is the same in all countries that adopted this vaccine
(B) Is not very reliable
(C) Is the best among all the other vaccines
(D) Is variable according to the research
Comentários: A alternativa A está incorreta. De acordo com o texto, não é correto afirmar que a
eficácia da vacina da Sinovac é a mesma em todos os países que adotaram a mesma, mas sim,
que a eficácia varia de acordo com a pesquisa. Isso pode ser confirmado com o trecho “The
Sinovac trials have yielded different results across different countries. Last month Turkish
researchers said the Sinovac vaccine was 91.25% effective, while Indonesia, which rolled out its
mass vaccination programme on Wednesday…”.
A alternativa B está incorreta. De acordo com o texto, não é correto afirmar que a eficácia da
vacina da Sinovac não é muito confiável, mas sim, que a eficácia varia de acordo com a pesquisa.
Isso pode ser confirmado com o trecho “The Sinovac trials have yielded different results across
different countries. Last month Turkish researchers said the Sinovac vaccine was 91.25% effective,
while Indonesia, which rolled out its mass vaccination programme on Wednesday…”.
A alternativa C está incorreta. De acordo com o texto, não é correto afirmar que a eficácia da
vacina da Sinovac é a melhor entre todas as outras vacinas, mas sim, que a eficácia varia de acordo

AULA 04 – ADJECTIVES AND ADVERBS 89


TEACHER ANDREA BELO

com a pesquisa. Isso pode ser confirmado com o trecho “The Sinovac trials have yielded different
results across different countries. Last month Turkish researchers said the Sinovac vaccine was
91.25% effective, while Indonesia, which rolled out its mass vaccination programme on
Wednesday…”.
A alternativa D está correta. De acordo com o texto, é correto afirmar que a eficácia da vacina da
Sinovac varia de acordo com a pesquisa, assim como esta opção indica. Isso pode ser confirmado
com o trecho “The Sinovac trials have yielded different results across different countries. Last
month Turkish researchers said the Sinovac vaccine was 91.25% effective, while Indonesia, which
rolled out its mass vaccination programme on Wednesday…”.
GABARITO: D

Read the comic strip and answer questions 05 and 06

(Adapted from https://www.gocomics.com/pickles/2021/01/12)

Questão 05 (EEAR/INÉDITA) – Choose the best alternative to complete the blank


(A) Is keeping
(B) Keeps
(C) Kept
(D) Was keeping
Comentários: A alternativa A está incorreta. A forma correta do verbo “to keep” (manter) para
preencher a lacuna não é “is keeping” (está mantendo) no present continuous, mas sim, “kept”
(manteve) no simple past. Isso pode ser confirmado com o trecho “Your snoring kept me awake
half the night” (Seu ronco me manteve acordada metade da noite).
A alternativa B está incorreta. A forma correta do verbo “to keep” (manter) para preencher a
lacuna não é “keeps” (mantém) no present simple, mas sim, “kept” (manteve) no simple past. Isso
pode ser confirmado com o trecho “Your snoring kept me awake half the night” (Seu ronco me
manteve acordada metade da noite).
A alternativa C está correta. A forma correta do verbo “to keep” (manter) para preencher a lacuna
é “kept” (manteve) no simple past, assim como esta opção indica. Isso pode ser confirmado com

AULA 04 – ADJECTIVES AND ADVERBS 90


TEACHER ANDREA BELO

o trecho “Your snoring kept me awake half the night” (Seu ronco me manteve acordada metade
da noite).
A alternativa D está incorreta. A forma correta do verbo “to keep” (manter) para preencher a
lacuna não é “was keeping” (estava mantendo) no past continuous, mas sim, “kept” (manteve)
no simple past. Isso pode ser confirmado com o trecho “Your snoring kept me awake half the
night” (Seu ronco me manteve acordada metade da noite).
GABARITO: C

Questão 06 (EEAR/INÉDITA) – Choose the best alternative according to the comic strip
(A) The man’s snores did not bother the woman
(B) The woman prefers to continue listening to snoring than having more work in the morning
(C) The man does not consider sleeping in separate beds, an option
(D) The woman was so irritated by snoring that she prefers to sleep in separate beds
Comentários: A alternativa A está incorreta. De acordo com a tirinha, não é correto afirmar que
os roncos do homem não incomodam a mulher, mas sim, que incomoda. Isso pode ser confirmado
com o trecho “Your snoring kept me awake half the night”.
A alternativa B está correta. De acordo com a tirinha, é correto afirmar que a mulher prefere
continuar escutando aos roncos do que ter mais trabalho pela manhã, assim como esta opção
indica. Isso pode ser confirmado com o trecho “No, I can’t stand the thought of having to make
up two beds every day”.
A alternativa C está incorreta. De acordo com a tirinha, não é correto afirmar que o homem não
considera dormir em camas separadas, uma opção, mas sim, que ele considera. Isso pode ser
confirmado com o trecho “Maybe it’s to get separate beds”.
A alternativa D está incorreta. De acordo com a tirinha, não é correto afirmar que a mulher estava
tão irritada com os roncos que ela prefere dormir em camas separadas, mas sim, que ela prefere
dormir na mesma cama. Isso pode ser confirmado com o trecho “Maybe it’s to get separate beds”.
GABARITO: B

Read the comic strip and answer questions 07 and 08

(Adapted from https://www.gocomics.com/pickles/2020/12/02)

AULA 04 – ADJECTIVES AND ADVERBS 91


TEACHER ANDREA BELO

Questão 07 (EEAR/INÉDITA) – Choose the best alternative to complete the blank


(A) Used
(B) Using
(C) Use
(D) Have been using
Comentários: A alternativa A está correta. A melhor forma do verbo to use, neste caso, é no
simple past “used” (usei), assim como esta opção indica. Isso pode ser confirmado com o trecho
“...I decorated the tree, Earl? I used all cats ornaments this year”.
A alternativa B está incorreta. A melhor forma do verbo to use, neste caso, não é no present
continuous “using” (usando), mas sim, no simple past “used” (usei). Isso pode ser confirmado com
o trecho “...I decorated the tree, Earl? I used all cats ornaments this year”.
A alternativa C está incorreta. A melhor forma do verbo to use, neste caso, não é no simple
present “use” (uso), mas sim, no simple past “used” (usei). Isso pode ser confirmado com o trecho
“...I decorated the tree, Earl? I used all cats ornaments this year”.
A alternativa D está incorreta. A melhor forma do verbo to use, neste caso, não é no present
perfect continuous “have been using” (tenho usado), mas sim, no simple past “used” (usei). Isso
pode ser confirmado com o trecho “...I decorated the tree, Earl? I used all cats ornaments this
year”.
GABARITO: A

Questão 08 (EEAR/INÉDITA) – According to what the woman said, she


(A) Doesn’t like cats at all
(B) Doesn’t care about the man’s opinion
(C) Likes cats so much that she uses them as decoration
(D) Put her pet as a Christmas tree decoration
Comentários: A alternativa A está incorreta. De acordo com o texto, não é correto afirmar que a
mulher não gosta de gatos, mas sim, que ela gosta. Isso pode ser confirmado com o trecho “I used
all cat ornaments this year”.
A alternativa B está incorreta. De acordo com o texto, não é correto afirmar que a mulher não se
importa com a opinião do homem, mas sim, que ela se importa. Isso pode ser confirmado com o
trecho “How do you like the way I decorated the tree, Earl?”.
A alternativa C está correta. De acordo com o texto, é correto afirmar que a mulher gosta tanto
de gatos que os utiliza como decoração, assim como esta opção indica. Isso pode ser confirmado
com o trecho “I used all cat ornaments this year”.
A alternativa D está incorreta. De acordo com o texto, não é correto afirmar que a mulher colocou
seu animal de estimação como uma decoração de árvore de natal, mas sim, que o gato dela estava
lá por engano. Isso pode ser confirmado com o trecho “Muffin! Get out of that tree!”.
GABARITO: C

AULA 04 – ADJECTIVES AND ADVERBS 92


TEACHER ANDREA BELO

Read the lyrics and answer questions 09 and 10


Best part - Daniel Caesar / H.E.R
[...]
You don't know, babe
When you hold me
And kiss me slowly
It's the sweetest thing
And it don't change
If I had it my way
You would know that you are
You're the coffee that I need in the morning
You're my sunshine in the rain when it's pouring
Won't you give yourself to me
Give it all, oh
I just wanna see
I just wanna see how beautiful you are
You know that I see it
I know you're a star
Where you go I follow
No matter how far
If life is a movie
Oh you're the best part, oh oh oh
You're the best part, oh oh oh
Best part
It's the sunrise
And those brown eyes, yes
You're the one that I desire
When we wake up
And then we make love
It makes me feel so nice
You're my water when I'm stuck in the desert
You're the Tylenol I take when my head hurts
You're the sunshine on my life

AULA 04 – ADJECTIVES AND ADVERBS 93


TEACHER ANDREA BELO

I just wanna see how beautiful you are


You know that I see it
I know you're a star
Where you go I follow
No matter how far
If life is a movie
Then you're the best part, oh oh oh
You're the best part, oh oh oh
Best part
[...]
(Adapted from LyricFind)

Questão 09 (EEAR/INÉDITA) – According to the lyrics


(A) The song portrays the reality of an abusive relationship
(B) The song expresses the feeling of intense passion of a loving relationship
(C) The song criticizes the idea of an extreme emotional dependence
(D) The song plunges into the idea of a three-way relationship
Comentários: A alternativa A está incorreta. De acordo com a letra, não é correto afirmar que a
música retrata a realidade de um relacionamento abusivo, mas sim, que a música expressa o
sentimento de paixão intensa de um relacionamento amoroso.
Isso pode ser confirmado com o trecho “You're the coffee that I need in the morning / You're my
sunshine in the rain when it's pouring / Won't you give yourself to me…”.
A alternativa B está correta. De acordo com a letra, é correto afirmar que a expressa o sentimento
de paixão intensa de um relacionamento amoroso, assim como esta opção indica. Isso pode ser
confirmado com o trecho “You're the coffee that I need in the morning / You're my sunshine in
the rain when it's pouring / Won't you give yourself to me…”.
A alternativa C está incorreta. De acordo com a letra, não é correto afirmar que a música critica a
ideia de uma dependência emocional extrema, mas sim, que a música expressa o sentimento de
paixão intensa de um relacionamento amoroso. Isso pode ser confirmado com o trecho “You're
the coffee that I need in the morning / You're my sunshine in the rain when it's pouring / Won't
you give yourself to me…”.
A alternativa D está incorreta. De acordo com a letra, não é correto afirmar que a música mergulha
na ideia de um relacionamento a três, mas sim, que a música expressa o sentimento de paixão
intensa de um relacionamento amoroso. Isso pode ser confirmado com o trecho “You're the
coffee that I need in the morning / You're my sunshine in the rain when it's pouring / Won't you
give yourself to me…”.
GABARITO: B

AULA 04 – ADJECTIVES AND ADVERBS 94


TEACHER ANDREA BELO

Questão 10 (EEAR/INÉDITA) – The underlined expression “the sweetest thing”, in the text,
means that the action
(A) Was sensitive and selfless
(B) Was actually sweet
(C) Was rude and selfish
(D) Was indifferent
Comentários: A alternativa A está correta. A expressão “a coisa mais doce” significa que a ação
foi sensível e altruísta, assim como esta opção indica. Isso pode ser confirmado com o trecho
“…When you hold me / And kiss me slowly / It’s the sweetest thing…”.
A alternativa B está incorreta. A expressão “a coisa mais doce” não significa que a ação foi
realmente doce, mas sim, que foi sensível e altruísta. Isso pode ser confirmado com o trecho
“…When you hold me / And kiss me slowly / It’s the sweetest thing…”.
A alternativa C está incorreta. A expressão “a coisa mais doce” não significa que a ação foi rude e
egoísta, mas sim, que foi sensível e altruísta. Isso pode ser confirmado com o trecho “…When you
hold me / And kiss me slowly / It’s the sweetest thing…”.
A alternativa D está incorreta. A expressão “a coisa mais doce” não significa que a ação foi
indiferente, mas sim, que foi sensível e altruísta. Isso pode ser confirmado com o trecho “…When
you hold me / And kiss me slowly / It’s the sweetest thing…”.
GABARITO: A

QUESTÕES EFOMM
Based on the text below, answer questions 01 and 02.
Why some people like wearing masks
Some people welcome face coverings for reasons ranging from the convenient and expedient
to the more complex and psychological. But is this a helpful coping mechanism?
Sheltering in place hasn’t been too hard for Jay Lee; watching a film at home and ordering a
takeaway has always been his idea of a good night. Lee, a 32-year-old small business owner in
Leicester, identifies as an introvert. And although 2020 had its hardships – in the spring, he was
made redundant from his job at a large bank – one perk for him has been the widespread adoption
of face masks.
Lee has always dreaded run-ins with old friends and acquaintances around town, finding these
spontaneous interactions “extremely awkward”. He used to time his shopping trips to minimise
the possibility of bumping into someone he knew, waiting until almost closing time before
heading out. “Since I've been wearing the mask, my awkward interactions with friends and family
have significantly reduced,” he says. Now, he goes to the shops whenever he wants, without
worrying about whom he might see. He hopes that, even after the pandemic ends, it will still be
socially acceptable to wear a mask.
Wearing a mask is, for most of us, an annoying but worthwhile sacrifice: it’s one of the most
effective ways to slow the spread of Covid-19. Still, most of us look forward to the day when we

AULA 04 – ADJECTIVES AND ADVERBS 95


TEACHER ANDREA BELO

can bare our faces in public again. Face-coverings fog our glasses and clog our pores; they make
it harder to smile at strangers and recognise friends.
Yet some are secretly relishing the new mask-wearing mandates, for reasons ranging from the
convenient and expedient to the more complex and psychological. Some welcome the way face
coverings reduce or change interactions that might otherwise spark social anxiety.
‘Anonymity carries power’
At the lighter end of the scale, some people have found that masking offers a welcome relief from
the pressures to uphold strict standards of grooming and appearance. They have ditched their old
makeup and shaving routines and are saving money, time and stress. Others have discovered that
hiding their mouths affords them unexpected freedoms. Some restaurant servers and retail
workers say they no longer feel obliged to fake-smile at customers, potentially lifting the burden
of emotional labour.
(Adapted from https://www.bbc.com/worklife/article/20210115-why-some-people-like-wearing-masks)

Questão 01 (EFOMM/INÉDITA) – It is possible to infer from the text that


(A) The safety measure related to the masks is well accepted depending on the personality of
the user
(B) Face coverings is an inconvenience for everyone
(C) Introverted people ended up not adapting to the context of a pandemic
(D) Wearing a mask is a pointless sacrifice at the present time
(E) Wearing masks does not interfere with the aesthetic pressure exerted by society
Comentários: A alternativa A está correta. De acordo com o texto, é correto inferir que a medida
de segurança relacionada às máscaras é bem aceita dependendo da personalidade do usuário,
assim como esta opção indica. Isso pode ser confirmado com o trecho “...identifies as an introvert.
And although 2020 had its hardships – in the spring, he was made redundant from his job at a
large bank – one perk for him has been the widespread adoption of face masks”.
A alternativa B está incorreta. De acordo com o texto, não é correto inferir que coberturas faciais
são um incômodo para todos, mas sim, que, para alguns, foi um alívio. Isso pode ser confirmado
com o trecho “...one perk for him has been the widespread adoption of face masks”.
A alternativa C está incorreta. De acordo com o texto, não é correto inferir que pessoas
introvertidas acabaram não se adaptando ao contexto de uma pandemia, mas sim, que se
adaptaram por não precisarem se expor ao contato com outras pessoas. Isso pode ser confirmado
com o trecho “Since I've been wearing the mask, my awkward interactions with friends and family
have significantly reduced”.
A alternativa D está incorreta. De acordo com o texto, não é correto inferir que usar uma máscara
é um sacrifício inútil no momento, mas sim, que é necessário e útil. Isso pode ser confirmado com
o trecho “Wearing a mask is, for most of us, an annoying but worthwhile sacrifice: it’s one of the
most effective ways to slow the spread of Covid-19”.

AULA 04 – ADJECTIVES AND ADVERBS 96


TEACHER ANDREA BELO

A alternativa E está incorreta. De acordo com o texto, não é correto inferir que o uso de máscaras
não interfere na pressão estética exercida pela sociedade, mas sim, que interfere. Isso pode ser
confirmado com o trecho “At the lighter end of the scale, some people have found that masking
offers a welcome relief from the pressures to uphold strict standards of grooming and
appearance”.
GABARITO: A

Questão 02 (EFOMM/INÉDITA) – In the excerpt “And although 2020 had its hardships – in the
spring, he was made redundant from his job at a large bank – one perk for him has been the
widespread adoption of face masks.”, the word in bold means
(A) Advice
(B) Hardship
(C) Happiness
(D) Loss
(E) Advantage
Comentários: A alternativa A está incorreta. A palavra “perk” significa benefício e não pode ser
comparada com a palavra “advice”, que significa conselho.
A alternativa B está incorreta. A palavra “perk” significa benefício e não pode ser comparada com
a palavra “hardship”, que significa sofrimento.
A alternativa C está incorreta. A palavra “perk” significa benefício e não pode ser comparada com
a palavra “happiness”, que significa felicidade.
A alternativa D está incorreta. A palavra “perk” significa benefício e não pode ser comparada com
a palavra “loss”, que significa perda.
A alternativa E está correta. A palavra “perk” significa benefício e pode ser comparada com a
palavra “advantage”, que significa vantagem.
GABARITO: E

Based on the text below, answer questions 03, 04 and 05.


The World's Biggest Problems Are Interconnected. Here's How We Can Solve Them This
Decade
Two decades ago, people around the world rang in the new millennium with a growing sense of
optimism. The threat posed by the Cold War was fading slowly in the rearview mirror. Leading
thinkers like Francis Fukuyama touted the benefits of globalization, saying it would bring
democracy and prosperity to the developing world. The nascent Internet economy promised to
bring us closer together.
The following 20 years took some of the air out of the assumption of steady progress, but when
future historians assess the 21st century, the year 2020 is likely to serve as the point at which the
optimism bubble burst. The COVID-19 pandemic has exposed a complex web of interlocking

AULA 04 – ADJECTIVES AND ADVERBS 97


TEACHER ANDREA BELO

problems that have morphed into full-blown crises. The coronavirus laid bare the dangers of
endemic poverty not only in the developing world but also in rich countries like the U.S., where
millions lack health care and are one paycheck away from living on the street. Around the world,
racial and ethnic minorities have demanded justice after centuries of structural discrimination.
Woven through it all, the earth’s climate is increasingly unstable, posing an existential threat to
human society as we know it. In the next decade, societies will be forced to either confront this
snarl of challenges, or be overwhelmed by them. Our response will define the future for decades
to come.
The recognition that these challenges are fundamentally linked isn’t new. Activists and academics
have for many years pointed to the cascading effects of various social ills. Whether it’s the way
racism contributes to poor health outcomes or gender discrimination harms economic growth,
the examples are seemingly endless. But this understanding has made its way into the
conversation about solutions too.
Notably, for the past five years, the U.N. has touted 17 interrelated sustainable development
goals, objectives for building a more viable world, and called for a push to achieve them by 2030.
The goals, which cover environmental, social and economic progress, are nonbinding but have
become key benchmarks for commitments at a national and corporate level. Countries from China
to the Maldives, as well as companies like Amazon, Microsoft and PwC, have committed to rolling
out policies over the next decade that will set them on a path to eliminate their carbon footprints.
The understanding that these problems require holistic solutions has only grown amid the
pandemic and its fallout. President Joe Biden has referred to four urgent crises—the pandemic,
the economic crisis, racial injustice and climate change—and promised a push to tackle them all
together. The European Union’s program to propel the bloc out of the COVID-19 crisis targets
climate change, while incorporating equity concerns. As stock markets soared last year,
institutions with trillions of dollars in assets demanded that their investments deliver not only a
good return for their wallets but also a good return for society.
(Adapted from https://time.com/5931603/how-to-solve-worlds-biggest-problems/)

Questão 03 (EFOMM/INÉDITA) – Read the statements about the text and decide whether they
are TRUE (T) or FALSE (F). Mark the correct option
I. The turn of the millennium brought high expectations for the development of the world
II. Covid-19 plays an important role in explaining the interconnection between global problems
III. Despite having played a watershed role, the Covid-19 pandemic mainly affected developing
countries
IV. The intensely discriminatory past has no reflection today
V. Our current actions will affect the future of world society
(A) I – (T) / II – (F) / III – (F) / IV – (F) / V – (T)
(B) I – (T) / II – (T) / III – (F) / IV – (F) / V – (F)
(C) I – (T) / II – (T) / III – (F) / IV – (F) / V – (T)
(D) I – (F) / II – (T) / III – (F) / IV – (F) / V – (T)
(E) I – (F) / II – (T) / III – (T) / IV – (F) / V – (T)

AULA 04 – ADJECTIVES AND ADVERBS 98


TEACHER ANDREA BELO

Comentários: A afirmativa I é verdadeira (T – true). É correto afirmar que a virada do milênio


trouxe grandes expectativas para o desenvolvimento do mundo, assim como esta opção indica.
Isso pode ser confirmado com o trecho “Two decades ago, people around the world rang in the
new millennium with a growing sense of optimism”.
A afirmativa II é verdadeira (T – true). É correto afirmar que a Covid-19 desempenha um papel
importante na explicação da interconexão entre problemas globais, assim como esta opção
indica. Isso pode ser confirmado com o trecho “The COVID-19 pandemic has exposed a complex
web of interlocking problems that have morphed into full-blown crises”.
A afirmativa III é falsa (F – false). Não é correto afirmar que apesar de ter desempenhado um
papel decisivo, a pandemia Covid-19 afetou principalmente os países em desenvolvimento, mas
sim, que afetou a todos os países do mundo. Isso pode ser confirmado com o trecho “The
coronavirus laid bare the dangers of endemic poverty not only in the developing world but also in
rich countries like the U.S., where millions lack health care and are one paycheck away from living
on the street”.
A afirmativa IV é falsa (F – false). Não é correto afirmar que o passado intensamente
discriminatório não tem reflexo hoje, mas sim, que tem. Isso pode ser confirmado com o trecho
“Around the world, racial and ethnic minorities have demanded justice after centuries of structural
discrimination”.
A afirmativa V é verdadeira (T – true). É correto afirmar que nossas ações atuais afetarão o futuro
da sociedade mundial, assim como esta opção indica. Isso pode ser confirmado com o trecho “As
stock markets soared last year, institutions with trillions of dollars in assets demanded that their
investments deliver not only a good return for their wallets but also a good return for society”.
GABARITO: C

Questão 04 (EFOMM/INÉDITA) – In the excerpt “Woven through it all, the earth’s climate is
increasingly unstable, posing an existential threat to human society as we know it”, the word in
bold means
(A) Unwoven
(B) Separate
(C) In contrast
(D) Sewn
(E) Although
Comentários: A alternativa A está incorreta. A palavra “woven” significa tecido/constituído e não
pode ser comparada com a palavra “unwoven”, que significa desfeito.
A alternativa B está incorreta. A palavra “woven” significa tecido/constituído e não pode ser
comparada com a palavra “separate”, que significa separado.
A alternativa C está incorreta. A palavra “woven” significa tecido/constituído e não pode ser
comparada com a palavra “in contrast”, que significa em contrapartida.

AULA 04 – ADJECTIVES AND ADVERBS 99


TEACHER ANDREA BELO

A alternativa D está correta. A palavra “woven” significa tecido/constituído e pode ser comparada
com a palavra “sewn”, que significa costurado/constituído.
A alternativa E está incorreta. A palavra “woven” significa tecido/constituído e não pode ser
comparada com a palavra “although”, que significa apesar.
GABARITO: D

Questão 05 (EFOMM/INÉDITA) – According to the text, the current problems in the world
(A) Are actually a conspiracy theory that opposes the high expectations of the turn of the
millennium
(B) Must be resolved for the whole society, after their total exposure during the pandemic
(C) Did not worsen during the Covid-19 pandemic
(D) Have a relatively new interconnection
(E) Are not so serious, considering the high development of society
Comentários: A alternativa A está incorreta. De acordo com o texto, não é correto afirmar que os
problemas atuais do mundo são, na verdade, uma teoria da conspiração que se opõe às altas
expectativas da virada do milênio, mas sim, que são reais e não correspondem às boas expectativas
da virada do milênio. Isso pode ser confirmado com o trecho “Two decades ago, people around the
world rang in the new millennium with a growing sense of optimism …The following 20 years took
some of the air out of the assumption of steady progress, but when future historians assess the 21st
century, the year 2020 is likely to serve as the point at which the optimism bubble burst”.
A alternativa B está correta. De acordo com o texto, é correto afirmar que os problemas atuais do
mundo devem ser resolvido para toda a sociedade, após sua exposição total durante a pandemia,
assim como esta opção indica. Isso pode ser confirmado com o trecho “The COVID-19 pandemic has
exposed a complex web of interlocking problems that have morphed into full-blown crises … As stock
markets soared last year, institutions with trillions of dollars in assets demanded that their
investments deliver not only a good return for their wallets but also a good return for society”.
A alternativa C está incorreta. De acordo com o texto, não é correto afirmar que os problemas atuais
do mundo não pioraram durante a pandemia Covid-19, mas sim, que pioraram e se tornaram mais
visíveis. Isso pode ser confirmado com o trecho “The COVID-19 pandemic has exposed a complex web
of interlocking problems that have morphed into full-blown crises”.
A alternativa D está incorreta. De acordo com o texto, não é correto afirmar que os problemas atuais
do mundo tem uma interconexão relativamente nova, mas sim, que não é nova. Isso pode ser
confirmado com o trecho “The recognition that these challenges are fundamentally linked isn’t new.
Activists and academics have for many years pointed to the cascading effects of various social ills”.
A alternativa E está incorreta. De acordo com o texto, não é correto afirmar que os problemas atuais
do mundo não são tão graves, considerando o alto desenvolvimento da sociedade, mas sim, que são
graves. Isso pode ser confirmado com o trecho “Around the world, racial and ethnic minorities have
demanded justice after centuries of structural discrimination”.
GABARITO: B

AULA 04 – ADJECTIVES AND ADVERBS 100


TEACHER ANDREA BELO

Questão 06 (EFOMM/INÉDITA) – Choose the correct option to complete the paragraph below.
I remember the last time I had _______ choose between optimism and fear, between hope and
the urge to run away. It was right after 9/11. I had babies—one so tiny she was still curled up like
a fern, _______ other toddling around gumming everything she could get her little starfish hands
on. And as their brand-new lungs took _______ the smoke that blew over to Brooklyn _______
the burning towers, I wanted to pack them up and flee to some safer place.
(Adapted from https://time.com/5930399/the-capitol-riots-and-our-fragile-optimism/)

(A) To / the / in / from


(B) The / the / on / into
(C) To / the / on / from
(D) To / the / in / into
(E) The / to / on / from
Comentários: A primeira lacuna deve ser preenchida com “to”, pois a frase nos infere que ele
deveria escolher entre o medo e o otimismo. Isso pode ser confirmado com o trecho “Lembro-me
da última vez que tive de escolher entre otimismo e medo”.
A segunda lacuna deve ser preenchida com “the”, pois a frase se refere especificamente às
segunda filha, ou seja, artigo definido. Isso pode ser confirmado com o trecho “a outra
engatinhando e engolindo tudo o que podia em suas mãos de estrela do mar”.
A terceira lacuna deve ser preenchida com “in”, pois a frase se refere aos pulmões inspirando a
fumaça. Isso pode ser confirmado com o trecho “E enquanto seus pulmões novos absorviam a
fumaça...”.
A quarta lacuna deve ser preenchida com “from”, pois a frase se refere À fumaça que chegava ao
Brooklyn das torres em chamas. Isso pode ser confirmado com o trecho “...chamas para o
Brooklyn, eu queria embalá-los e fugir para um lugar mais seguro”.
GABARITO: A

Questão 07 (EFOMM/INÉDITA) – Which is the correct way to complete the paragraph below?
For the European Union, the rapid rollout of Covid-19 vaccines __________ critical to save lives
and prevent health services from ____________ stretched beyond their limits, not to mention
minimising the massive economic damage from lockdowns. Unfortunately, however, though
vaccinations are under way, a rapid near-term increase in infections is likely as the British variant
of the virus __________ across the continent.
(Adapted from https://www.concursosmilitares.com.br/provas-anteriores/marinha/efomm/2020-efomm-oficial-da-marinha-mercante-primeiro-dia.pdf)

(A) Is / being / spreads


(B) Was / be / spreads
(C) Is / be / spreading
(D) Was / being / spreading
(E) Is / being / spreading

AULA 04 – ADJECTIVES AND ADVERBS 101


TEACHER ANDREA BELO

Comentários: A primeira lacuna deve ser preenchida com o verbo to be (ser/estar) no simple
present, ou seja, “is” (é). Isso pode ser confirmado com o trecho “Para a União Europeia, a rápida
distribuição das vacinas Covid-19 é crítica para salvar vidas...”.
A segunda lacuna deve ser preenchida com o verbo to be (ser/estar) no present continuous, ou
seja, “being” (de ser). Isso pode ser confirmado com o trecho “...evitar que os serviços de saúde
sejam esticados além de seus limites...”.
A terceira lacuna deve ser preenchida com o verbo to spread (propagar) no simple present, ou
seja, “spreads” (se propaga). Isso pode ser confirmado com o trecho “...à medida que a variante
britânica do vírus se espalha pelo continente”.
GABARITO: A

Questão 08 (EFOMM/INÉDITA) – Which of the following sentences expresses probability?


(A) You are not as understanding as you should be
(B) One shouldn’t accelerate in case of a red light
(C) Everyone here must have a critical sense
(D) He studied a lot, so he should be able to pass
(E) You must take good care of your body, even if you don't want to
Comentários: A alternativa A está incorreta. A frase “Você não é tão compreensivo quanto deveria
ser” não expressa probabilidade, mas sim, uma afirmação.
A alternativa B está incorreta. A frase “Não se deve acelerar em caso de sinal vermelho” não
expressa probabilidade, mas sim, imperatividade.
A alternativa C está incorreta. A frase “Todos aqui devem ter senso crítico” não expressa
probabilidade, mas sim, uma afirmação.
A alternativa D está correta. A frase “Ele estudou muito, então deveria conseguir passar” expressa
probabilidade.
A alternativa E está incorreta. A frase “Você deve cuidar bem do seu corpo, mesmo que não
queira” não expressa probabilidade, mas sim, uma necessidade.
GABARITO: D

Questão 09 (EFOMM/INÉDITA) – Choose the correct option to complete the paragraph below.
The U.S. has tragically surpassed 400,000 COVID-19 deaths, and case numbers and
hospitalizations are likewise spiking to record levels around the world. __________ vaccines now
rolling out, there is reason to hope that there is an end in sight. __________ , by most estimates,
widespread vaccinations will not be in place until the middle of the year at the earliest.
__________ , we have some ways to go yet with social distancing, mask wearing and other
pandemic mitigation behaviors.
(Adapted from https://www.scientificamerican.com/article/how-we-can-deal-with-pandemic-fatigue/)

AULA 04 – ADJECTIVES AND ADVERBS 102


TEACHER ANDREA BELO

(A) And / however / and


(B) With / however / so
(C) So / however / even if
(D) With / in agreement / and
(E) With / in contrast / so
Comentários: A primeira lacuna deve ser preenchida com “with” (com), pois a frase se refere ao
momento atual das vacinas circulando. Isso pode ser confirmado com o trecho “Com as vacinas
sendo lançadas, há motivos para esperar que haja um fim à vista”.
A segunda lacuna deve ser preenchida com “however” (contudo), pois a frase, de certa forma, se
opõe à frase anterior. Isso pode ser confirmado com o trecho “Com as vacinas sendo lançadas, há
motivos para esperar que haja um fim à vista. No entanto, pela maioria das estimativas, as
vacinações generalizadas não estarão em vigor até meados do ano, no mínimo”.
A terceira lacuna deve ser preenchida com “so” (então/portanto), pois a frase nos infere uma
conclusão. Isso pode ser confirmado com o trecho “Portanto, ainda temos alguns caminhos a
percorrer com o distanciamento social, uso de máscaras e outros comportamentos de mitigação
de pandemia”.
GABARITO: B

Questão 10 (EFOMM/INÉDITA) – Which option is incorrect?


(A) A pandemic is affecting many people
(B) Tell the girls that they are not responsible
(C) A couple of my friends plans to travel this year
(D) I want to finish school
(E) Where are those pants I lent you?
Comentários: A alternativa A está incorreta. A frase “uma pandemia está afetando muitas
pessoas” está totalmente correta.
A alternativa B está incorreta. A frase “diga às meninas que elas não são responsáveis” está
totalmente correta.
A alternativa C está correta. A frase “A couple of my friends plans to travel this year” deveria ser
escrita “A couple of friends of mine are planning to trave this year” (um casal de amigos meus
estão planejando para viajar este ano).
A alternativa D está incorreta. A frase “eu quero terminar a escola” está totalmente correta.
A alternativa E está incorreta. A frase “onde estão aquelas calças que te emprestei?” está
totalmente correta.
GABARITO: C

AULA 04 – ADJECTIVES AND ADVERBS 103


TEACHER ANDREA BELO

QUESTÕES EPCAR
Directions: Answer questions 01 to 10 according to TEXT I
Facebook building a version of Instagram for children under 13
Social media giant says it’s exploring introducing a parent-controlled experience that allows
kids to ‘safely’ use the photo sharing platform
Facebook is considering launching a version of its popular photo social media platform, Instagram,
for children under the age of 13.
BuzzFeed News first reported Facebook announced in an internal company post that the company
would begin building a version of Instagram for people under the age of 13 years to allow them
to “safely” use Instagram for the first time. Currently the company does not allow people who are
under this age to create an account on the platform.
A spokesperson for Facebook told the Guardian the company was exploring a parent-controlled
version of Instagram, similar to the Messenger Kids app that is for kids between six and 12.
“Increasingly kids are asking their parents if they can join apps that help them keep up with their
friends. Right now there aren’t many options for parents, so we’re working on building additional
products … that are suitable for kids, managed by parents,” the spokesperson said.
“We’re exploring bringing a parent-controlled experience to Instagram to help kids keep up with
their friends, discover new hobbies and interests, and more.”
In a blog post earlier this week, which did not mention the proposed new Instagram service, the
company noted that although people were asked to enter their age when signing up for
Instagram, there was nothing to prevent people from lying about it at registration.
Facebook said it would overcome that by using machine learning in combination with the
registration age to determine people’s ages on the platform.
The company also announced plans to roll out new safety features, including preventing adults
from messaging people under the age of 18 who do not follow them, safety notices for teens
when messaged by an adult sending a large amount of friend requests or messages to people
under 18, and make it more difficult for adults to find and follow teens using the search function
in Instagram.
Teens will also be encouraged to put their profiles on private at the point of registration.
A study of Australian teens’ internet usage published by the Australian eSafety commissioner in
February found 57% of Australian teenagers use Instagram, while 30% reported being contacted
by a stranger, and 20% reported being sent inappropriate unwanted content on the social media
sites they used.
(Adapted from https://www.theguardian.com/technology/2021/mar/19/facebook-building-a-version-of-instagram-for-children-under-13)

AULA 04 – ADJECTIVES AND ADVERBS 104


TEACHER ANDREA BELO

Questão 01 (EPCAR/INÉDITA) – Mark the option that can replace the word “allows” in the title
without changing its meaning
(A) Grants
(B) Prevents
(C) Prohibits
(D) Disapproves
Comentários: A alternativa A está correta. A palavra “allows” significa permite e pode ser
substituída pela palavra “grants”, que significa concede/permite.
A alternativa B está incorreta. A palavra “allows” significa permite e não pode ser substituída pela
palavra “prevents”, que significa previne.
A alternativa C está incorreta. A palavra “allows” significa permite e não pode ser substituída pela
palavra “prohibits”, que significa proíbe.
A alternativa D está incorreta. A palavra “allows” significa permite e não pode ser substituída pela
palavra “disapproves”, que significa desaprova.
GABARITO: A

Questão 02 (EPCAR/INÉDITA) – We can deduce from the second paragraph that


(A) Facebook developed a platform for children under 13
(B) The possible development of the new social media was not first disclosed by Facebook
(C) The new social media will target all age groups
(D) Instagram allows any and all types of people to sign up
Comentários: A alternativa A está incorreta. De acordo com o segundo parágrafo, não é correto
afirmar que o Facebook desenvolveu uma plataforma para crianças menores de 13 anos, mas sim,
que a empresa ainda estava começando a desenvolver a plataforma. Isso pode ser confirmado
com o trecho “...Facebook announced in an internal company post that the company would begin
building a version of Instagram for people under the age of 13 years…”.
A alternativa B está correta. De acordo com o segundo parágrafo, é correto afirmar que o possível
desenvolvimento da nova mídia social não foi divulgado pela primeira vez pelo Facebook, assim
como esta opção indica. Isso pode ser confirmado com o trecho “BuzzFeed News first reported
Facebook announced in an internal company post that the company would begin building a
version of Instagram for people under the age of 13 years…”.
A alternativa C está incorreta. De acordo com o segundo parágrafo, não é correto afirmar que a
nova mídia social terá como alvo todas as faixas etárias, mas sim, que o alvo será pessoas abaixo
de 13 anos de idade. Isso pode ser confirmado com o trecho “...Facebook announced in an internal
company post that the company would begin building a version of Instagram for people under the
age of 13 years…”.
A alternativa D está incorreta. De acordo com o segundo parágrafo, não é correto afirmar que o
Instagram permite que todo e qualquer tipo de pessoa se inscreva, mas sim, que menores de 13

AULA 04 – ADJECTIVES AND ADVERBS 105


TEACHER ANDREA BELO

anos não podem se inscrever na mídia. Isso pode ser confirmado com o trecho “Currently the
company does not allow people who are under this age to create an account on the platform”.
GABARITO: B

Questão 03 (EPCAR/INÉDITA) – Mark the option with the suitable question to answer the
underlined fragment below “Facebook is considering launching a version of its popular photo
social media platform, Instagram, for children under the age of 13”
(A) What were the latest Facebook launches?
(B) What will be the changes in the current Facebook photo social media platform?
(C) How was the launch of the new Facebook photo social media platform?
(D) What is the next possible Facebook launch?
Comentários: A alternativa A está incorreta. A frase “O Facebook está considerando lançar uma
versão de sua popular plataforma de mídia social de fotos...” não responde à pergunta “Quais
foram os últimos lançamentos do Facebook?”, mas sim, à pergunta “Qual é o próximo lançamento
possível do Facebook?”.
A alternativa B está incorreta. A frase “O Facebook está considerando lançar uma versão de sua
popular plataforma de mídia social de fotos...” não responde à pergunta “Quais serão as
mudanças na atual plataforma de mídia social de fotos do Facebook?”, mas sim, à pergunta “Qual
é o próximo lançamento possível do Facebook?”.
A alternativa C está incorreta. A frase “O Facebook está considerando lançar uma versão de sua
popular plataforma de mídia social de fotos...” não responde à pergunta “Como foi o lançamento
da nova plataforma de mídia social de fotos do Facebook?”, mas sim, à pergunta “Qual é o
próximo lançamento possível do Facebook?”.
A alternativa D está correta. A frase “O Facebook está considerando lançar uma versão de sua
popular plataforma de mídia social de fotos...” responde à pergunta “Qual é o próximo
lançamento possível do Facebook?”, assim como esta opção indica.
GABARITO: D

Questão 04 (EPCAR/INÉDITA) – Mark the option that can replace the underlined word keeping
the same meaning “A spokesperson for Facebook told the Guardian the company was exploring
a parent-controlled version of Instagram…”
(A) Neglecting
(B) Analyzing
(C) Overlooking
(D) Finding.
Comentários: A alternativa A está incorreta. A palavra “exploring” significa explorando e não pode
ser substituída pela palavra “neglecting”, que significa negligenciando.

AULA 04 – ADJECTIVES AND ADVERBS 106


TEACHER ANDREA BELO

A alternativa B está correta. A palavra “exploring” significa explorando e pode ser substituída pela
palavra “analyzing”, que significa analisando.
A alternativa C está incorreta. A palavra “exploring” significa explorando e não pode ser
substituída pela palavra “overlooking”, que significa negligenciando.
A alternativa D está incorreta. A palavra “exploring” significa explorando e não pode ser
substituída pela palavra “finding”, que significa encontrando.
GABARITO: B

Questão 05 (EPCAR/INÉDITA) – What can be said about the new version of Instagram?
(A) The platform will be for children between 6 and 12 years old
(B) It will be for children under 13 years of age to use the platform without the help of adults
(C) It’s a platform that, in some aspects, resembles another one that already exists
(D) Nothing is known at all about the new Facebook platform
Comentários: A alternativa A está incorreta. Sobre a nova versão do Instagram, não é correto
afirmar que a plataforma será para crianças entre 6 e 12 anos, mas sim, que o Messenger Kids é
para essa faixa etária. Isso pode ser confirmado com o trecho “...similar to the Messenger Kids
app that is for kids between six and 12”.
A alternativa B está incorreta. Sobre a nova versão do Instagram, não é correto afirmar que caberá
a menores de 13 anos utilizar a plataforma sem a ajuda de adultos, mas sim, que será uma
plataforma para menores de 13 anos e controlada pelos pais/parentes. Isso pode ser confirmado
com o trecho “A spokesperson for Facebook told the Guardian the company was exploring a
parent-controlled version of Instagram…”.
A alternativa C está correta. Sobre a nova versão do Instagram, é correto afirmar que é uma
plataforma que, em alguns aspectos, se assemelha a outra já existente, assim como esta opção
indica. Isso pode ser confirmado com o trecho “A spokesperson for Facebook told the Guardian
the company was exploring a parent-controlled version of Instagram, similar to the Messenger
Kids app that is for kids between six and 12”.
A alternativa D está incorreta. Sobre a nova versão do Instagram, não é correto afirmar que nada
se sabe sobre a nova plataforma do Facebook, mas sim, que, provavelmente, será uma plataforma
controlada por pais que é uma nova versão do Instagram. Isso pode ser confirmado com o trecho
“A spokesperson for Facebook told the Guardian the company was exploring a parent-controlled
version of Instagram…” .
GABARITO: C

Questão 06 (EPCAR/INÉDITA) – According to the fourth paragraph, kids


(A) Are increasingly interested in becoming part of social media
(B) Are not very well inserted in the virtual world, despite having many options
(C) Are less and less interested in apps that connect them with their friends
(D) Prefer to keep in touch in person, disregarding the internet

AULA 04 – ADJECTIVES AND ADVERBS 107


TEACHER ANDREA BELO

Comentários: A alternativa A está correta. De acordo com o quarto parágrafo, é correto afirmar
que as crianças estão cada vez mais interessados em fazer parte das redes sociais, assim como
esta opção indica. Isso pode ser confirmado com o trecho “Increasingly kids are asking their
parents if they can join apps that help them keep up with their friends”.
A alternativa B está incorreta. De acordo com o quarto parágrafo, não é correto afirmar que as
crianças não estão muito bem inseridos no mundo virtual, apesar de terem muitas opções, mas
sim, que elas não tem muitas opções. Isso pode ser confirmado com o trecho “Right now there
aren’t many options for parents, so we’re working on building additional products…”.
A alternativa C está incorreta. De acordo com o quarto parágrafo, não é correto afirmar que as
crianças estão cada vez menos interessadas em aplicativos que os conectam com seus amigos,
mas sim, que estão cada vez mais interessadas. Isso pode ser confirmado com o trecho
“Increasingly kids are asking their parents if they can join apps that help them keep up with their
friends”.
A alternativa D está incorreta. De acordo com o quarto parágrafo, não é correto afirmar que as
crianças preferem o contato presencial, desconsiderando a internet, pois o parágrafo não afirma
nada deste tipo.
GABARITO: A

Questão 07 (EPCAR/INÉDITA) – Mark the correct option


(A) Facebook has already developed an accessible version of Instagram for children under 13
(B) Facebook is developing a version of Instagram for children under 13 to use the app
completely autonomously
(C) Facebook is developing a version of Instagram so that children under 13 just chat with their
friends
(D) Facebook is researching an accessible and secure Instagram platform for children under 13
Comentários: A alternativa A está incorreta. Não é correto afirmar que o Facebook já desenvolveu
uma versão acessível do Instagram para crianças menores de 13 anos, mas sim, que o mesmo está
explorando ideias para criar esta plataforma. Isso pode ser confirmado com o trecho “We’re
exploring bringing a parent-controlled experience to Instagram to help kids…”.
A alternativa B está incorreta. Não é correto afirmar que o Facebook está desenvolvendo uma
versão do Instagram para menores de 13 anos, para usarem o aplicativo de forma totalmente
autônoma, mas sim, que será controlada pelos pais. Isso pode ser confirmado com o trecho
“We’re exploring bringing a parent-controlled experience...”.
A alternativa C está incorreta. Não é correto afirmar que o Facebook está desenvolvendo uma
versão do Instagram para que crianças menores de 13 anos apenas conversem com seus amigos,
mas sim, para que as crianças encontrem novos hobbies e interesses, conversem com amigos, e
etc. Isso pode ser confirmado com o trecho “..to Instagram to help kids keep up with their friends,
discover new hobbies and interests, and more”.
A alternativa D está correta. É correto afirmar que o Facebook está pesquisando uma plataforma
de Instagram acessível e segura para crianças menores de 13 anos, assim como esta opção indica.

AULA 04 – ADJECTIVES AND ADVERBS 108


TEACHER ANDREA BELO

Isso pode ser confirmado com o trecho “We’re exploring bringing a parent-controlled experience
to Instagram to help kids keep up with their friends, discover new hobbies and interests, and
more”.
GABARITO: D

Questão 08 (EPCAR/INÉDITA) – Considering the use of verb tenses, mark the alternative that
completes the sentence below correctly Facebook
(A) fights false ages in the system using technology
(B) is fighting false ages in the system using technology
(C) will fight false ages in the system using technology
(D) has been fighting false ages in the system using technology.
Comentários: A alternativa A está incorreta. A frase dada não pede um verbo no simple present
(fights – luta), mas sim, um verbo no simple future (will fight – vai lutar/combater). Isso pode ser
confirmado com o trecho que indica que isso é algo que o Facebook almeja combater, ou seja, no
futuro; “Facebook said it would overcome that by using machine learning in combination with the
registration age to determine people’s ages on the platform”.
A alternativa B está incorreta. A frase dada não pede um verbo no presente continuous (is fighting
– está lutando), mas sim, um verbo no simple future (will fight – vai lutar/combater). Isso pode
ser confirmado com o trecho que indica que isso é algo que o Facebook almeja combater, ou seja,
no futuro; “Facebook said it would overcome that by using machine learning in combination with
the registration age to determine people’s ages on the platform”.
A alternativa C está correta. A frase dada pede um verbo no simple future (will fight – vai
lutar/combater), assim como esta opção indica. Isso pode ser confirmado com o trecho que indica
que isso é algo que o Facebook almeja combater, ou seja, no futuro; “Facebook said it would
overcome that by using machine learning in combination with the registration age to determine
people’s ages on the platform”.
A alternativa D está incorreta. A frase dada não pede um verbo no presente perfect continuous
(has been fighting – tem lutado), mas sim, um verbo no simple future (will fight – vai
lutar/combater). Isso pode ser confirmado com o trecho que indica que isso é algo que o Facebook
almeja combater, ou seja, no futuro; “Facebook said it would overcome that by using machine
learning in combination with the registration age to determine people’s ages on the platform”.
GABARITO: C

Questão 09 (EPCAR/INÉDITA) – We can deduce from the eighth paragraph that


(A) Instagram has no security issues
(B) Security issues are worrying to the point that Instagram needs new security measures
(C) People over 18 are prohibited from sending messages to minors
(D) The new security measures aim to protect female users

AULA 04 – ADJECTIVES AND ADVERBS 109


TEACHER ANDREA BELO

Comentários: A alternativa A está incorreta. De acordo com o oitavo parágrafo, não é correto
afirmar que o Instagram não tem problemas de segurança, mas sim, que tem e novas medidas de
segurança estão à vista. Isso pode ser confirmado com o trecho “The company also announced
plans to roll out new safety features, including preventing adults from messaging people under
the age of 18 who do not follow them…”.
A alternativa B está correta. De acordo com o oitavo parágrafo, é correto afirmar que os
problemas de segurança são preocupantes a ponto de o Instagram precisar de novas medidas de
segurança, assim como esta opção indica. Isso pode ser confirmado com o trecho “The company
also announced plans to roll out new safety features, including preventing adults from messaging
people under the age of 18 who do not follow them, safety notices for teens when messaged…”.
A alternativa C está incorreta. De acordo com o oitavo parágrafo, não é correto afirmar que
pessoas com mais de 18 anos estão proibidas de enviar mensagens a menores, mas sim, existem
medidas para dificultar isso considerando possíveis crimes. Isso pode ser confirmado com o trecho
“The company also announced plans to roll out new safetuy features, including preventing adults
from messaging people under the age of 18 who do not follow them, safety notices for teens
when messaged…”.
A alternativa D está incorreta. De acordo com o oitavo parágrafo, não é correto afirmar que as
novas medidas de segurança visam proteger as usuárias mulheres, mas sim, que as medidas
citadas visam proteger menores de idade. Isso pode ser confirmado com o trecho “…safety notices
for teens when messaged by an adult sending a large amount of friend requests or messages to
people under 18…”..
GABARITO: B

Questão 10 (EPCAR/INÉDITA) – The word “them” (paragraph 8) refers to


(A) Adults
(B) People under the age of 18
(C) New safety features
(D) The company
Comentários: A alternativa A está correta. A palavra “them” (eles) se refere aos adultos, assim
como esta opção indica. Isso pode ser confirmado com o trecho “...incluindo a prevenção de
adultos de enviar mensagens para pessoas com menos de 18 anos que não os seguem...”.
A alternativa B está incorreta. A palavra “them” (eles) não se refere às pessoas com idade menor
de 18, mas sim, aos adultos. Isso pode ser confirmado com o trecho “...incluindo a prevenção de
adultos de enviar mensagens para pessoas com menos de 18 anos que não os seguem...”.
A alternativa C está incorreta. A palavra “them” (eles) não se refere aos novos recursos de
segurança, mas sim, aos adultos. Isso pode ser confirmado com o trecho “...incluindo a prevenção
de adultos de enviar mensagens para pessoas com menos de 18 anos que não os seguem...”.
A alternativa D está incorreta. A palavra “them” (eles) não se refere a empresa, mas sim, aos
adultos. Isso pode ser confirmado com o trecho “...incluindo a prevenção de adultos de enviar
mensagens para pessoas com menos de 18 anos que não os seguem...”
GABARITO: A

AULA 04 – ADJECTIVES AND ADVERBS 110


TEACHER ANDREA BELO

QUESTÕES ESA
COVID-19 Conspiracy Theories Are Spreading Rapidly—and They're a Public Health Risk All
Their Own
Public health crises have spawned conspiracy theories as far back as when the Black Death
ravaged Europe in the 1300s, as people desperately try to make sense of the chaotic forces
disrupting their lives. While modern science offers a better understanding of how diseases infect
people and how to contain them, COVID-19 conspiracy theories are spreading rapidly via social
media, unreliable news outlets and from our own political leaders, including U.S. President Donald
Trump. The result: many Americans now believe pandemic -related conspiracy theories—and,
alarmingly, those same people are less likely to take steps to prevent the virus from spreading.
In a University of Pennsylvania Annenberg Public Policy Center study published Monday in Social
Science &
Medicine, researchers surveyed a group of 840 U.S. adults—first in late March, and then again in
mid-July—to determine how Americans’ beliefs and actions regarding the pandemic changed over
time. Overall, they found that COVID-19 conspiracy theories are not only commonplace, they’re
gaining traction. Back in March, 28% of people believed a debunked rumor that the Chinese
government created the coronavirus as a bioweapon; that number rose to 37% by July. About
24% believed that the U.S. Centers for Disease Control and Prevention exaggerated the virus’
danger to hurt Trump politically despi te a lack of evidence; by July, that figure rose to 32%. And
in March, about 15% of respondents said they believed that the pharmaceutical industry created
the virus to boost drug and vaccine sales—another unfounded theory—compared to 17% in July.
(Adapted from https://time.com/5891333/covid-19-conspiracy-theories/)

Questão 01 (ESA/INÉDITA) – Concerning the information in the text, is correct to state that
(A) Adherence to conspiracy theories regarding the COVID-19 pandemic has declined over time
in the USA
(B) Social networks are not a potential disseminator of misinformation that collaborates with
conspiracy theories
(C) The artificial creation of the coronavirus is scientifically proven
(D) Modern science does not affect the understanding of diseases
(E) Conspiracy theories may affect the adherence of security measures by part of the population
Comentários: A alternativa A está incorreta. De acordo com as informações do texto, não é
correto afirmar que a adesão a teorias da conspiração em relação à pandemia COVID-19 diminuiu
ao longo do tempo nos EUA, mas sim, que aumentou com o tempo. Isso pode ser confirmado com
o trecho “Overall, they found that COVID-19 conspiracy theories are not only commonplace,
they’re gaining traction”.
A alternativa B está incorreta. De acordo com as informações do texto, não é correto afirmar que
as redes sociais não são um potencial disseminador de desinformação que colabora com as teorias

AULA 04 – ADJECTIVES AND ADVERBS 111


TEACHER ANDREA BELO

da conspiração, mas sim, que são um disseminador de fake News. Isso pode ser confirmado com
o trecho “...OVID-19 conspiracy theories are spreading rapidly via social media...”.
A alternativa C está incorreta. De acordo com as informações do texto, não é correto afirmar que
a criação artificial do coronavírus é comprovada cientificamente, mas sim, que isso é uma teoria
da conspiração. Isso pode ser confirmado com o trecho “...28% of people believed a debunked
rumor that the Chinese government created the coronavirus as a bioweapon; that number rose
to 37% by July”.
A alternativa D está incorreta. De acordo com as informações do texto, não é correto afirmar que
a ciência moderna não afeta a compreensão das doenças, mas sim, que a ciência colabora com
esses compreendimentos. Isso pode ser confirmado com o trecho “While modern science offers
a better understanding of how diseases infect people and how to contain them…”.
A alternativa E está correta. De acordo com as informações do texto, é correto afirmar que teorias
de conspiração podem afetar a adesão de medidas de segurança por parte da população, assim
como esta opção indica. Isso pode ser confirmado com o trecho “The result: many Americans now
believe pandemic-related conspiracy theories—and, alarmingly, those same people are less likely
to take steps to prevent the virus from spreading”.
GABARITO: E

Questão 02 (ESA/INÉDITA) – Which option has a correct relation of the underlined terms and
their substitutes?
(A) A government spokesman says… – Her
(B) Prisoners are considered a COVID-19 risk group. – They
(C) Me and the girls were at the party and ... – They
(D) The US has thousands of cases of COVID-19. – He
(E) Feminist groups still fight for women's rights – Them
Comentários: A alternativa A está incorreta. O substituto correto para “a government
spokesman” (um porta-voz do governo) não é “her” (dela), mas sim, “he” (ele).
A alternativa B está correta. O substituto correto para “prisioners” (prisioneiros) é “they” (eles),
assim como esta opção indica.
A alternativa C está incorreta. O substituto correto para “me and the girls” (eu e as meninas) não
é “they” (elas), mas sim, “we” (nós).
A alternativa D está incorreta. O substituto correto para “the US” (os EUA) não é “he” (ele), mas
sim, “it”.
A alternativa E está incorreta. O substituto correto para “feminist groups” (grupos feministas) não
é “them” (deles), mas sim, “they” (eles).
GABARITO: B

AULA 04 – ADJECTIVES AND ADVERBS 112


TEACHER ANDREA BELO

Questão 03 (ESA/INÉDITA) – “__________ the US still have the __________ number of COVID-
19 cases?”. Fill in the blanks with the correct form of the verb and the adjective
(A) Does / worst
(B) Does / better
(C) Is / best
(D) Is / worse
(E) Do / worst
Comentários: A primeira lacuna deve ser preenchida com “Does” pois esta forma verbal se
encaixa em uma pergunta funcionando como um auxiliar, onde as ações principais permanecem
no infinitivo.
A segunda lacuna deve ser preenchida com “worst” (pior) pois é o melhor adjetivo que se refere
ao número de casos da COVID-19.
GABARITO: A

Questão 04 (ESA/INÉDITA) – Which sentence is grammatically correct?


(A) That women is in love with me
(B) A motorcycle is more slow than a car
(C) She practices physical exercise every week
(D) I have a sofa new
(E) He need to be loved
Comentários: A alternativa A está incorreta. A palavra “women” está no plural, ou seja, mulheres;
o correto seria “woman” (mulher). Ou seja “aquela mulher está apaixonada por mim”.
A alternativa B está incorreta. Ao comparar algo, neste caso, deve ser utilizado o sufixo -er ao fim
da palavra, ou seja, o correto seria “slower” (mais lento). Ou seja “Uma motocicleta é mais lenta
que um carro”.
A alternativa C está correta. A frase "Ela pratica exercícios físicos todas as semanas” está
totalmente correta.
A alternativa D está incorreta. Neste caso, o correto seria “I have a new sofa” (eu tenho um sofá
novo), a ordem está errada.
A alternativa E está incorreta. Neste caso, o verbo “to need” deveria estar no simple present, ou
seja, “he needs” (ele precisa). Ou seja, “ele precisa ser amado”.
GABARITO: C

AULA 04 – ADJECTIVES AND ADVERBS 113


TEACHER ANDREA BELO

Questão 05 (ESA/2019) – “__________ American?” Complete the space with the correct form of
the verb and the pronoun.
(A) You is
(B) You are
(C) Are you
(D) Is you
(E) Am you
Comentários: Questão que necessita apenas de conhecimento sobre o verbo to be.
A alternativa A está incorreta. “You” nunca deve ser seguido de “is”, e sim “are”. O verbo deve
estar antes do pronome “you”, pois trata-se de uma pergunta.
A alternativa B está incorreta. A ordem de pronome (you) e verbo (are) deveria estar invertida
por se tratar de uma pergunta.
A alternativa C está correta. Numa frase interrogativa, deve-se inverter a ordem de pronome e
verbo. Portanto, “Are you American?” é gramaticalmente correto.
A alternativa D está incorreta. “You” nunca é acompanhado de “is”, sempre de “are”.
A alternativa E está incorreta. “You” nunca é acompanhado de “am”, sempre de “are”.
GABARITO: C

QUESTÕES ESCOLA NAVAL


Based on the text below, answer the six questions that follow it.
Covid babies don't have to be the unlucky generation. But they must be helped
A year into this pandemic, there are babies now learning to walk and talk who have never known
anything but life under the shadow of Covid, and preschoolers who can barely remember a world
before it. Doctors’ children have had to learn not to touch Mummy when she gets in from work,
until she’s had a shower to wash off any last possible trace of danger.
Thanks to popular toddler demand, you can now buy masks for dolls.
It’s perfectly normal for kids to reflect what’s happening around them by playing quarantine with
stuffed animals, but normal too for adults to wonder uncomfortably whether all this leaves a
lasting mark. How much will it matter in years to come that, as the minister for loneliness Diana
Barran recently put it, there are toddlers being raised by shielding parents who have never had a
playdate? Will Covid babies grow up solitary creatures, used to entertaining themselves, or warier
of the strangers they so rarely meet and interact with? Taking a tiny bundle out in public used to
mean an endless succession of random older women cooing over the pram, or strangers pulling
faces to entertain a bored baby in a checkout queue.
But now passersby daren’t get close, and other shoppers are hidden behind masks.
This week MPs were presented with some early findings from a project led by the First 1001 Days
Movement, an alliance of early-years charities and professionals, tracking the lives of under-twos

AULA 04 – ADJECTIVES AND ADVERBS 114


TEACHER ANDREA BELO

growing up through a pandemic. A survey of children’s service providers it commissioned found


98% thought the babies and toddlers they worked with had been affected by higher parental
stress and anxiety, while 92% had seen fearful families effectively cutting themselves off from the
outside world, skipping routine appointments or not wanting to leave the house. Nine in 10 had
observed children being played with less, or being less active. Heartbreakingly, more than a
quarter said lockdown left the children they worked with more exposed to domestic conflict,
abuse or neglect.
(Adapted from https://www.theguardian.com/commentisfree/2021/jan/22/covid-babies-unlucky-generation-help-lockdown)

Questão 01 (ESCOLA NAVAL/INÉDITA) – According to the text, which option is correct?


(A) There are children who have not yet experienced life outside the pandemic
(B) The pandemic has not affected the lives of any children, as they do not understand what is
happening
(C) The pandemic, in a way, positively impacted the babies' lives
(D) The children's routine has not changed in this pandemic
(E) Characteristics of the pandemic did not impact areas such as sales
Comentários: A alternativa A está correta. De acordo com o texto, é correto afirmar que há
crianças que ainda não experimentaram a vida fora da pandemia, assim como esta opção indica.
Isso pode ser confirmado com o trecho “...there are babies now learning to walk and talk who
have never known anything but life under the shadow of Covid…”.
A alternativa B está incorreta. De acordo com o texto, não é correto afirmar que a pandemia não
afetou a vida de nenhuma criança, pois elas não entendem o que está acontecendo, mas sim, que
afetou. Isso pode ser confirmado com o trecho “Doctors’ children have had to learn not to touch
Mummy when she gets in from work…”.
A alternativa C está incorreta. De acordo com o texto, não é correto afirmar que a pandemia, de
certa forma, impactou positivamente a vida dos bebês, mas sim, que impactou negativamente.
Isso pode ser confirmado com o trecho “Doctors’ children have had to learn not to touch Mummy
when she gets in from work, until she’s had a shower to wash off any last possible trace of danger”.
A alternativa D está incorreta. De acordo com o texto, não é correto afirmar que a rotina das
crianças não mudou nesta pandemia, mas sim, que mudou. Isso pode ser confirmado com o
trecho “Doctors’ children have had to learn not to touch Mummy when she gets in from work…”.
A alternativa E está incorreta. De acordo com o texto, não é correto afirmar que as características
da pandemia não impactaram áreas como vendas, mas sim, que impactaram. Isso pode ser
confirmado com o trecho “Thanks to popular toddler demand, you can now buy masks for dolls”.
GABARITO: A

AULA 04 – ADJECTIVES AND ADVERBS 115


TEACHER ANDREA BELO

Questão 02 (ESCOLA NAVAL/INÉDITA) – What’s the meaning of the word “toddler” in paragraph
1?
(A) Population
(B) Children
(C) Babies
(D) Adults
(E) Teens
Comentários: A alternativa A está incorreta. A palavra “toddler” não significa população, mas sim,
“children” (crianças).
A alternativa B está correta. A palavra “toddler” significa crianças, assim como esta opção indica.
A alternativa C está incorreta. A palavra “toddler” não significa bebês, mas sim, “children”
(crianças).
A alternativa D está incorreta. A palavra “toddler” não significa adultos, mas sim, “children”
(crianças).
A alternativa E está incorreta. A palavra “toddler” não significa adolescentes, mas sim, “children”
(crianças).
GABARITO: B

Questão 03 (ESCOLA NAVAL/INÉDITA) – In paragraph 1, the word “she” refers to


(A) Children
(B) Doctor
(C) Toddler
(D) Mummy
(E) Babies
Comentários: A alternativa A está incorreta. A palavra “she” (ela) não se refere às crianças, mas
sim, a mamãe (mummy). Isso pode ser confirmado com o trecho “Doctors’ children have had to
learn not to touch Mummy when she gets in from work…”.
A alternativa B está incorreta. A palavra “she” (ela) não se refere a doutora, mas sim, a mamãe
(mummy). Isso pode ser confirmado com o trecho “Doctors’ children have had to learn not to
touch Mummy when she gets in from work…”.
A alternativa C está incorreta. A palavra “she” (ela) não se refere às crianças, mas sim, a mamãe
(mummy). Isso pode ser confirmado com o trecho “Doctors’ children have had to learn not to
touch Mummy when she gets in from work…”.
A alternativa D está correta. A palavra “she” (ela) se refere a mamãe (mummy), assim como esta
opção indica. Isso pode ser confirmado com o trecho “Doctors’ children have had to learn not to
touch Mummy when she gets in from work…”.

AULA 04 – ADJECTIVES AND ADVERBS 116


TEACHER ANDREA BELO

A alternativa E está incorreta. A palavra “she” (ela) não se refere aos bebês, mas sim, a mamãe
(mummy). Isso pode ser confirmado com o trecho “Doctors’ children have had to learn not to
touch Mummy when she gets in from work…”.
GABARITO: D

Questão 04 (ESCOLA NAVAL/INÉDITA) – According to the text, which option completes the
sentence below, correctly?
The pandemic __________ an entire generation.
(A) Affects
(B) Affected
(C) Was affecting
(D) Had affected
(E) Is affecting
Comentários: A alternativa A está incorreta. A melhor forma de completar a frase, de acordo com
o texto, não é com o verbo “to affect” (afetar) no simple present (affects – afeta), mas sim, no
present continuous (is affecting – está afetando). Isso pode ser confirmado com o trecho “...there
are babies now learning to walk and talk who have never known anything but life under the
shadow of Covid…”.
A alternativa B está incorreta. A melhor forma de completar a frase, de acordo com o texto, não
é com o verbo “to affect” (afetar) no simple past (affected – afetou), mas sim, no present
continuous (is affecting – está afetando). Isso pode ser confirmado com o trecho “...there are
babies now learning to walk and talk who have never known anything but life under the shadow
of Covid…”.
A alternativa C está incorreta. A melhor forma de completar a frase, de acordo com o texto, não
é com o verbo “to affect” (afetar) no past continuous (was affecting – estava afetando), mas sim,
no present continuous (is affecting – está afetando). Isso pode ser confirmado com o trecho
“...there are babies now learning to walk and talk who have never known anything but life under
the shadow of Covid…”.
A alternativa D está incorreta. A melhor forma de completar a frase, de acordo com o texto, não
é com o verbo “to affect” (afetar) no past perfect (had affected – afetou), mas sim, no present
continuous (is affecting – está afetando). Isso pode ser confirmado com o trecho “...there are
babies now learning to walk and talk who have never known anything but life under the shadow
of Covid…”.
A alternativa E está correta. A melhor forma de completar a frase, de acordo com o texto, é com
o verbo “to affect” (afetar) no present continuous (is affecting – está afetando), assim como esta
opção indica. Isso pode ser confirmado com o trecho “...there are babies now learning to walk
and talk who have never known anything but life under the shadow of Covid…”.
GABARITO: E

AULA 04 – ADJECTIVES AND ADVERBS 117


TEACHER ANDREA BELO

Questão 05 (ESCOLA NAVAL/INÉDITA) – According to the text, which option is correct?


(A) Children do not realize what happens around them
(B) The impact of the pandemic on children is a normal concern of parents
(C) There is no doubt about this generation that was born in the middle of a pandemic
(D) The relationship with children on the street changed with the pandemic
(E) The impact of the pandemic on children can only be short-term
Comentários: A alternativa A está incorreta. De acordo com o texto, não é correto afirmar que as
crianças não percebem o que acontece ao seu redor, mas sim, que percebem. Isso pode ser
confirmado com o trecho “It’s perfectly normal for kids to reflect what’s happening around
them…”.
A alternativa B está correta. De acordo com o texto, é correto afirmar que o impacto da pandemia
nas crianças é uma preocupação normal dos pais, assim como esta opção indica. Isso pode ser
confirmado com o trecho “...but normal too for adults to wonder uncomfortably whether all this
leaves a lasting mark”.
A alternativa C está incorreta. De acordo com o texto, não é correto afirmar que não há dúvidas
sobre esta geração que nasceu no meio de uma pandemia, mas sim, que existem muitas dúvidas.
Isso pode ser confirmado com o trecho “Will Covid babies grow up solitary creatures, used to
entertaining themselves, or warier of the strangers they so rarely meet and interact with?”.
A alternativa D está incorreta. De acordo com o texto, não é correto afirmar que a relação com as
crianças na rua mudou com a pandemia, mas sim, que mudou. Isso pode ser confirmado com o
trecho “But now passersby daren’t get close, and other shoppers are hidden behind masks”.
A alternativa E está incorreta. De acordo com o texto, não é correto afirmar que o impacto da
pandemia nas crianças só pode ser de curto prazo, mas sim, que pode ser a longo prazo. Isso pode
ser confirmado com o trecho “Will Covid babies grow up solitary creatures, used to entertaining
themselves, or warier of the strangers they so rarely meet and interact with?”.
GABARITO: B

QUESTÕES EsPCEx
Leia o texto a seguir e responda às questões 01, 02 e 03.
Bill Gates: ‘Carbon neutrality in a decade is a fairytale. Why peddle fantasies?’
Bill Gates appears via video conference – Microsoft Teams, not Zoom, obviously – from his office
in Seattle, a large space with floor-to-ceiling windows overlooking Lake Washington. It’s a gloomy
day outside and Gates is, somewhat eccentrically, positioned a long way from the camera, behind
a large, kidney-shaped desk; his communications manager sits off to one side. “As a way to start,”
says Gates’ aide, “would it be helpful for Bill to make a couple of comments about why he wrote
his new book?”
Unlike the Elon Musks or Larry Ellisons of this world, however, Gates is perceived to be sensible,
uxorious, modest, vowing not to ruin his children with boundless inheritance or to waste energy
trying to send things to Mars. Gates’ new book, How To Avoid A Climate Disaster, grew out of two

AULA 04 – ADJECTIVES AND ADVERBS 118


TEACHER ANDREA BELO

things: his interest in the sciences and what struck him as an irresistible challenge – the fiendishly
difficult problem of how to further global development while reducing emissions.
There’s another, greater obstacle to reaching zero emissions, which is the political challenge – part
of which involves climate activists limiting their exposure to accusations of hypocrisy. Gates loves
private jets; he calls them his “guilty pleasure”.
The depressing part of the book is its account of the challenge ahead, which Gates presents as
extremely urgent – He points to a headline figure: 51bn. This is the amount of greenhouses gas,
in tons, emitted globally each year, which we have to get down to net zero by 2050. The first step
towards this is understanding what we’re dealing with.
Adapted from https://www.theguardian.com/technology/2021/feb/15/bill-gates-carbon-neutrality-in-a-decade-is-afairytale-why-peddle-fantasies

Questão 01 (EsPCEx/INÉDITA) – According to the text, choose the correct alternative.


(A) Gates usually invests in sending things to Mars.
(B) Gates wrote an e-book about world climate.
(C) Bill Gates made a video conference via Zoom.
(D) Bill Gates currently lives in Seattle.
(E) For Gates, it will take more than a decade to reach carbon neutrality.
Comentários: De acordo com o texto, escolha a alternativa correta.
A alternativa A está incorreta. “Gates geralmente investe no envio de coisas para Marte.” →
“Unlike the Elon Musks or Larry Ellisons of this world, however, Gates is perceived to be sensible,
uxorious, modest, vowing not to ruin his children with boundless inheritance or to waste energy
trying to send things to Mars.” = Ao contrário dos Elon Musks ou Larry Ellisons deste mundo, no
entanto, Gates é visto como sensato, zeloso, modesto, jurando não arruinar seus filhos com
herança ilimitada ou desperdiçar energia tentando enviar coisas para Marte.
A alternativa B está incorreta. “Gates escreveu um e-book sobre o clima mundial.” → O texto não
afirma que é um e-book, isto é, um livro eletrônico. A alternativa C está incorreta. “Bill Gates fez
uma videoconferência via Zoom.” → “Bill Gates appears via video conference – Microsoft Teams,
not Zoom, obviously – from his office in Seattle…”
A alternativa D está incorreta. O texto não afirma que “Bill Gates atualmente mora em Seattle.”,
dizendo apenas que ele estava em seu escritório em Seattle. → “Bill Gates appears via video
conference – Microsoft Teams, not Zoom, obviously – from his office in Seattle…”
A alternativa E está correta. “Para Gates, levará mais de uma década para alcançar a neutralidade
de carbono.” → Bill Gates: ‘Carbon neutrality in a decade is a fairytale. Why peddle fantasies?’ =
Bill Gates: ‘A neutralidade do carbono em uma década é um conto de fadas. Por que vender
fantasias?’ / “This is the amount of greenhouses gas, in tons, emitted globally each year, which we
have to get down to net zero by 2050.” = Esta é a quantidade de gases de efeito estufa, em
toneladas, emitida globalmente a cada ano, que temos que reduzir para zero líquido até 2050.
GABARITO: E

AULA 04 – ADJECTIVES AND ADVERBS 119


TEACHER ANDREA BELO

Questão 02 (EsPCEx/INÉDITA) – In the sentence “... Gates is perceived to be sensible ...”


(paragraph 2), the word sensible means
(A) reasonable
(B) delicate
(C) gentle
(D) tender
(E) polite
Comentários: Atenção, pois a compreensão dos termos sensible / sensitive já foi exigida pela
EsPCEx. Na frase trazida pela questão, a palavra “sensible” significa sensato, razoável. Não significa
sensível. O termo que significa sensível é “sensitive”.
(A) razoável
(B) delicado
(C) gentil
(D) suave, sensível, delicado
(E) educado
GABARITO: A

Questão 03 (EsPCEx/INÉDITA) – In the sentence “… positioned a long way from the camera,
behind a large, kidney-shaped desk … (paragraph 1), the expression kidney-shaped refers to
(A) the size of the desk.
(B) the outline of the desk.
(C) the weight of the desk.
(D) the height of the desk.
(E) the color of the desk.
Comentários: Na frase “… posicionado longe da câmera, atrás de uma grande mesa em forma de
rim… (parágrafo 1), a expressão kidney-shaped refere-se a
(A) o tamanho da mesa.
(B) o contorno, o formato da mesa.
(C) o peso da mesa.
(D) a altura da mesa.
(E) a cor da mesa.
GABARITO: B

AULA 04 – ADJECTIVES AND ADVERBS 120


TEACHER ANDREA BELO

Leia o texto a seguir e responda às questões 04, 05, 06 e 07.


How a Spanish town pioneered dolls with Down's syndrome
The first time Kelle Hampton glimpsed a doll with Down’s syndrome, anger boiled up inside her. Its
exaggerated features bore little resemblance to the sweet facial characteristics that she loved about her
daughter Nella, who was born with the genetic disorder.
The experience set the US blogger and author firmly against such dolls. But to her surprise, years later she
found herself smitten with another doll. This time it had been carefully crafted to subtly capture the
characteristics that made Nella unique. “This one was simply a beautiful doll any child would want to play
with,” she said.
The doll hailed from a small town in eastern Spain that was put on the map after enterprising potters
began transforming clay from the nearby mountains into children’s dolls, giving rise to what has been
described as the country’s first doll factory. More than a century later the spotlight is again on the
Valencian town of Onil – this time for its singular combination of inclusivity and artisan dollmaking.
The town, home to around 7,500 people and 38 toymakers, made headlines across Spain last year after a
local collection of four dolls with Down’s syndrome won the country’s much-coveted “toy of the year”
prize.
The dolls – two boys and two girls of varying skin colours – were rolled out by toymaker Miniland. “We
were apprehensive at the beginning,” said Victoria Orruño, the company’s marketing director. “But the
reaction surprised us. It was very positive.”
Adapted from https://www.theguardian.com/society/2021/feb/15/dolls-and-diversity-how-toys-pass-on-the-power-of-being-see

Questão 04 (EsPCEx/INÉDITA) – Choose the statement in which the verb to boil has been used in
a figurative way just like in paragraph 1.
(A) Could you boil some water up for me?
(B) I've boiled some potatoes for lunch.
(C) I can hardly boil an egg.
(D) He was boiling with rage.
(E) Liquid nitrogen boils at a very low temperature.
Comentários: No texto, o verbo to boil, ferver, foi usado em sentido figurado, veja: The first time
Kelle Hampton glimpsed a doll with Down’s syndrome, anger boiled up inside her. = A primeira vez
que Kelle Hampton viu uma boneca com síndrome de Down, a raiva “ferveu” dentro dela. Vamos
buscar nas alternativas aquela em que o verbo também foi usado em sentido figurado, isto é, não
literal.
(A) Você poderia ferver um pouco de água para mim?
(B) Fervi algumas batatas para o almoço.
(C) Mal consigo cozinhar um ovo.
(D) Ele estava fervendo de raiva.
(E) O nitrogênio líquido ferve a uma temperatura muito baixa.
GABARITO: D

AULA 04 – ADJECTIVES AND ADVERBS 121


TEACHER ANDREA BELO

Questão 05 (EsPCEx/INÉDITA) – In “Its exaggerated features bore little resemblance to the sweet
facial characteristics that she loved about her daughter …” (paragraph 1), its refers to
(A) Kelle Hampton
(B) her daughter
(C) Down’s syndrome
(D) anger
(E) a doll
Comentários: No trecho trazido pela questão, o possessive adjective its se refere à boneca, veja:
“The first time Kelle Hampton glimpsed a doll with Down’s syndrome, anger boiled up inside her.
Its exaggerated features bore little resemblance to the sweet facial characteristics that she loved
about her daughter Nella… = A primeira vez que Kelle Hampton viu uma boneca com síndrome de
Down, a raiva ferveu dentro dela. Seus traços exagerados (Traços exagerados de quem? Da
boneca) tinham pouca semelhança com as doces características faciais que ela amava em sua filha
Nella...
GABARITO: E

Questão 06 (EsPCEx/INÉDITA) – Choose the alternative that has the same meaning as the word
spotlight in the sentence “More than a century later the spotlight is again on the Valencian town
of Onil…” (paragraph 3).
(A) polemic
(B) anger
(C) attention
(D) mistake
(E) force
Comentários: A palavra spotlight significa holofote. No trecho trazido pela questão, tem o sentido
de destaque, ênfase, atenção (como se jogássemos um holofote sobre algo ou alguém, chamando
a atenção para aquilo). “More than a century later the spotlight is again on the Valencian town of
Onil…” = “Mais de um século depois, os holofotes estão novamente na cidade valenciana de
Onil…” (a atenção está toda voltada para lá).
(A) polêmica
(B) raiva
(C) atenção
(D) erro
(E) força
GABARITO: C

AULA 04 – ADJECTIVES AND ADVERBS 122


TEACHER ANDREA BELO

Questão 07 (EsPCEx/INÉDITA) – About the underlined words her, she and herself, it is correct to
say that
(A) they are adjectives.
(B) they are verbs.
(C) they are pronouns.
(D) they are nouns.
(E) they are adverbs.
Comentários: Her, she e herself são pronomes no texto, respectivamente: object pronoun, subject
pronoun e reflexive pronoun. Não são adjetivos, verbos, substantivos (nouns) ou advérbios.
GABARITO: C

Leia o texto a seguir e responda às questões 08, 09, e 10.


Professor lectures for two hours online – on mute
A university professor in Singapore gave a two-hour online lecture but didn't realize he was on
mute. Professor Wang, who teaches maths, was not aware that throughout his online
presentation, the microphone on his computer was switched off. This meant that none of the
students attending his online class heard what professor Wang was talking about. His university
switched his classes online to help reduce the spread of COVID-19. His lecture started well but
then it froze. The video came back but the mic was off and professor Wang did not notice. One of
his students says it might have been because the professor was using an iPad and not a computer.
Students tried many times to contact professor Wang during the lesson but could not get through
to him. They waved their arms and even tried calling him on his personal phone. A video of the
moment he understood his mic was off has gone viral on the Internet. It has received more than
653,000 views. After realizing what happened, professor Wang appeared shocked and stressed.
He said he was upset at the thought of wasting two hours of his students' time. He has offered to
redo the lecture at a different time so his students do not miss that class. One of his students
posted on social media about the professor's mishap. She wrote: “I took a class under him before
and he teaches well. I feel bad for him.”
Adapted from https://www.dailymail.co.uk/news/article-9247009/Maths-professor-Singapore-delivers-two-hour-lecture-Zoom-realisemute.html.

Questão 08 (EsPCEx/INÉDITA) – According to the text, choose the correct alternative.


(A) The professor said he would not redo the lecture.
(B) A video of the professor has gone viral on the Internet.
(C) The professor is sad because students made fun of him.
(D) The problem might have been caused by the computer the professor used.
(E) Students refused to warn the professor about what was happening.
Comentários: De acordo com o texto, escolha a alternativa correta.
A alternativa A está incorreta. “O professor disse que não iria refazer a palestra.” → “He has offered
to redo the lecture at a different time so his students do not miss that class.”
A alternativa B está correta. “Um vídeo do professor se tornou viral na Internet.” → A video of the
moment he understood his mic was off has gone viral on the Internet.
A alternativa C está incorreta. O texto não afirma que “O professor está triste porque os alunos
zombaram dele.” → “He said he was upset at the thought of wasting two hours of his students'

AULA 04 – ADJECTIVES AND ADVERBS 123


TEACHER ANDREA BELO

time.” = Ele disse que estava chateado com a ideia de desperdiçar duas horas do tempo de seus
alunos.
A alternativa D está incorreta. “O problema pode ter sido causado pelo computador utilizado pelo
professor.” → “One of his students says it might have been because the professor was using an
iPad and not a computer.”
A alternativa E está incorreta. “Os alunos se recusaram a alertar o professor sobre o que estava
acontecendo.” → Students tried many times to contact professor Wang during the lesson but
could not get through to him.
GABARITO: B

Questão 09 (EsPCEx/INÉDITA) – In the sentence “...One of his students posted on social media
about the professor's mishap....” (paragraph 2), the prefix mis means
(A) wrongly
(B) against
(C) without
(D) self
(E) beyond
Comentários: Na frase trazida pela questão, o prefixo mis, em mishap, indica um erro, que algo
não aconteceu como deveria, assim como acontece em mistake (erro) e misunderstood (mal-
entendido). Mishap significa um acidente, um contratempo, um infortúnio.
(A) erroneamente
(B) contra
(C) sem
(D) próprio
(E) além
GABARITO: A

Questão 10 (EsPCEx/INÉDITA) – In “His lecture started well but then it froze.” (paragraph 1), the
word then
(A) introduces a conclusion.
(B) indicates a sequence of facts.
(C) makes a comparison.
(D) indicates a place.
(E) reinforces irony.
Comentários: Em “A palestra dele começou bem, mas depois congelou”. (parágrafo 1), a palavra
then
(A) apresenta uma conclusão – then pode significar então, mas não neste caso.
(B) indica uma sequência de fatos.
(C) faz uma comparação – quem faz comparação é than com a.
(D) indica um lugar.
(E) reforça a ironia.
GABARITO: B

AULA 04 – ADJECTIVES AND ADVERBS 124


TEACHER ANDREA BELO

CONSIDERAÇÕES FINAIS
Concluímos mais uma aula, outro passo até a sua aprovação!
Desta vez, com adjetivos e advérbios. E estamos caminhando para maior vocabulário e
mais aprendizado de fato.
Nota-se o progresso em seus estudos e, provavelmente, uma maior tranquilidade para
enfrentar os exercícios que surgem.
E você vai se acostumando a equilibrar seus estudos de forma sistematizada, estudando
cada vez mais e com mais dedicação.
Outro detalhe importante para seu sucesso nos estudos é continuar fazendo listas de
palavras e verbos, participar das aulas complementares, fazer simulados e pedir ajuda quando
precisar.
Isso te ajudará nas questões futuras e torna você, como eu disse antes, um candidato mais
bem preparado e confiante para realizar uma excelente prova.
É importante lembrar também do nosso Fórum de dúvidas, exclusivo do Estratégia
Militares. Será minha forma de responder, no prazo máximo de 48 horas, o que mais você precise
saber para que os conteúdos fiquem ainda mais claros em seus estudos, certo?

AULA 04 – ADJECTIVES AND ADVERBS 125


TEACHER ANDREA BELO

REFERÊNCIAS BIBLIOGRÁFICAS
BARRETO, Tania Pedroza; GARRIDO, Maria Line; SILVA, João Antenor de C., Inglês Instrumental.
Leitura e compreensão de textos. Salvador, Ba UFBA, 1995, p. 64.
BROWN. H. Douglas. Principles of Language Learning and Teaching. Prentice Hall International,
1988.
COMPEDELLI, Samira Yousseff. Português, Literatura, Produção de texto & Gramática – São Paulo:
Ed. Saraiva, 2002.
CORREIA, Clese Mary P. Reading Specific Purposes. Salvador/ Ba: UFBA, 1999.
COSTEIRA, Adriana Araújo de M. Reading Comprehension Skills. João Pessoa/PB: ETFP, 1998.
CRYSTAL David. Cambridge University Press 1997. The Cambridge Encyclopedia of Language.
Cambridge University Press 1997
FREEMAN. Diane Larsen. MURCIA. Marianne Celce. The Grammar Book, 1999.
DYE, Joan., FRANFORT, Nancy. Spectrum II, III A Communicative Course in English. USA, Prentice
Hall, 1994.
FAVERO, Maria de Lourdes Albuquerque (org.). Dicionário de educadores no Brasil: da colônia aos
dias atuais. Rio de Janeiro: UFRJ, MEC, INEP, 1999.
FRANKPORT, Nancy & Dye Hoab. Spectrum II, III Prentice Hall Regents Englewood Cliffs, New Jersy,
1994.
GADELHA, Isabel Maria B. Inglês Instrumental: Leitura, Conscientização e Prática. Teresina:
EDUFFI, 2000.
GUANDALINI, Eiter Otávio. Técnicas de Leitura em Inglês: ESP – English For Specific Purposes:
estagio 1. São Paulo: Texto novo, 2002.
GRELLET, Françoise. Developing Reading Skills. Cambridge University Press, 1995
HOLAENDER, Arnon & Sanders Sidney. A complete English Course. São Paulo. Ed. Moderna, 1995.
HUTCHINSON, Tom & WATERS, Alan. English for Specific Purposes. Cambridge: Cambridge
University Press, 1996
KRASHEN. Stephen D. Second Language Acquisition and Second Language Learning, Prentice-Hall
International, 1988.
LAENG, Mauro. Dicionário de pedagogia. Lisboa: Dom Quixote, 1973.
LEFFA, Vilson J. Metodologia do ensino de línguas. In: BOHN, H.; VANDRESEN, P. (org.). Tópicos de
linguística aplicada: o ensino de línguas estrangeiras. Florianópolis: Editora da UFSC, 1988. p. 211-
231.
LIBERATO, Wilson. Compact English Book Inglês Ensino Médio. São Paulo: FTD, Vol. Único, 1998
Mc ARTHUR. The Oxford Companion to the English Language. Oxford University Press 1992
Fromkin. Victoria. An Introduction to Language
MARQUES, Amadeu. Inglês Série Brasil. ed. Atica. São Paulo: 2004. Vol. Único.

AULA 04 – ADJECTIVES AND ADVERBS 126


TEACHER ANDREA BELO

MURPHY, Raymond: Essencial Grammar in Use Oxford. New York Ed. Oxford University, 1997.
OLIVEIRA, Luciano Amaral. English For Tourism Students. Inglês para Estudantes de Turismo: São
Paulo, Rocca, 2001.
OLIVEIRA, Sara Rejane de F. Estratégias de leitura para Inglês Instrumental. Brasília: UNB, 1994.
QUINTANA, et alli. First Certificate. Master Class Oxford. New York, 2004: Ed. Oxford University.
PAULINO, Berenice F. et all. Leitura em textos em Inglês – Uma Abordagem Instrumental. Belo
Horizonte: Ed. Dos Autores, 1992.
PEREIRA, Edilberto Coelho. Inglês Instrumental. Teresina: ETFPI, 1998.
RODGES, Theodore. Jack C. Richards. Approaches and Methods in Language Teaching. Cambridge
University Press, 2001.
RODMAN Robert. Harcourt Brace 1993. English as a Global Language
STEWART, B., HAINES S. First Certificate, MasterClass. UK – Oxford 2004.
SILVA, João Antenor de C., GARRIDO, Maria Lina, BARRETO, Tânia Pedrosa. Inglês Instrumental:
Leitura e Compreensão de Textos. Salvador: Centro Editorial e Didático, UFBA. 1994
SOARES, Moacir Bretãs. Dicionário de legislação do ensino. 19.ed. Rio de Janeiro: FGV, 1981.
SOUZA, Adriana Srade F. Leitura em Língua Inglesa: Uma abordagem Instrumental. São Paulo:
Disal, 2005.
TUCK, Michael. Oxford Dictionary of Computing for Learners of English. Oxford: Oxford University
Press, 1996.
TOTIS, Verônica Pakrauskas. Língua Inglesa: leitura. São Paulo: Cortez, 1991.

Livros eletrônicos:
Dicionário Houaiss da Língua Portuguesa, Editora Objetiva, 2001.
MOURãO, Janaína Pereira. "Skimming x Scanning"; Brasil Escola. Disponível em
<https://brasilescola.uol.com.br/ingles/skimming-x-scanning.htm>. Acesso em 20 de março de
2019.
www.newsweek.com - Acesso em 18 de março de 2019.
http://www.galaor.com.br/tecnicas-de-leitura/ - Acesso em 19 de março de 2019.
Expressões Idiomáticas (continuação)" em Só Língua Inglesa. Virtuous Tecnologia da
Informação,2008-2019. Consultado em 03/04/2019 às 22:09. Disponível na Internet em
http://www.solinguainglesa.com.br/conteudo/Expressoes5.php

AULA 04 – ADJECTIVES AND ADVERBS 127


TEACHER ANDREA BELO

TRADUÇÕES
How to cope with parent guilt, during the pandemic and beyond
As a mother, I’m vulnerable to the influence of our cultural messaging, and I have fallen prey to the
“We can never give enough, do enough or be enough” narrative. As a psychologist, though, I know
how risky this constant thread of guilt is for our well-being. Guilt can be helpful as an uncomfortable
emotion that motivates us to make amends and change hurtful behaviors, but in the case of parent
guilt, much of the time we are only hurting ourselves.
Enter a global pandemic that has shattered our already precarious parenting lives and provided ample
evidence of our systems’ failures. Employers are expecting our remote-learning children to behave
during our work day, and women are leaving jobs in record numbers for their suddenly homebound
children, because they “just can’t do it all anymore.” Ilyse DiMarco, a clinical psychologist and the
author of the upcoming book “Mom Brain,” sums up the problem: “The issue with guilt right now is
there is potential guilt with anything you do.”
Stress levels have risen, with parents outpacing nonparents in surveys, creating fertile ground for
mental health problems. “We’re feeling like we’re not measuring up in some area or maybe we’re not
making the right choices,” DiMarco says. “Not surprisingly, if we feel like we’re failing, we feel
depressed; if we’re worried about not making the right decision, we feel anxious.”
Daily life in a pandemic has given us a host of new reasons to feel guilty. In addition to the impossibility
of simultaneously working, parenting and, in some cases, teaching our children, we face daily
decisions around health and safety.
Do we allow play dates? What if all of my child’s friends are playing basketball and I say no? What if
we choose inperson school and our child gets covid-19, or we choose remote school and our child
becomes depressed?
Working moms are not okay. No good choices and no good answers, but guaranteed guilt, with a
sprinkling of judgment caused by social pressures.
“Guilt is a useful emotion when it tells us we’ve done something wrong,” says clinical psychologist Jill
Stoddard, author of “Be Mighty.” “What’s happening now is we feel like we’ve done something wrong
even when we’re doing the very best we can.”
So, what can we do about the guilt hanging heavy around our collective necks? Experts suggest a
combination of mindfulness, meaningful self-care and shifting our perspective.
Mindfulness
Do not ignore the guilt; acknowledging it means you can do something about it. Even if our pandemic-
related guilt may be misplaced, Stoddard says we can still use the feeling for its positive function and
make amends. She has told her children, “I really wish I could spend more time with you guys, and it’s
been so hard on all of us. Things will be different someday. I’m sorry I can’t be there for you more.”
She adds: “You’re not saying, ‘I screwed up.’ You’re saying, ‘It hurts my heart I can’t do this the way I
wish I could right now.’ ”
Remember you are not alone. Feelings of failure are a universal experience of parents during the
coronavirus pandemic. Seeking social support is critical for mental health in times of stress. It can be
as simple as texting a friend to share “Mom fails” from your day. Even this brief connection offers

AULA 04 – ADJECTIVES AND ADVERBS 128


TEACHER ANDREA BELO

support and solidarity, and it can decrease our sense of self-blame and inadequacy when we hear
others’ similar feelings and experiences.
Practice self-kindness. Talk to yourself as you would to your friend: “You are doing your very best in
this moment.” This practice helps shift thinking patterns from self-criticism to self-compassion, which
is known to increase positive emotions and decrease negative ones, including guilt.
Real self-care
Prioritize the daily demands on your time and energy. DiMarco advises doing this by asking, “What’s
most important here, on a day-to-day or hour-to-hour basis?” Focus on what is highest on the priority
list, and allow yourself to let go of the other potential tasks instead of feeling guilty for, inevitably,
not getting everything done.
Prioritize yourself. “An important piece, and counterintuitive piece, to manage guilt about not being
there for everybody else, is to also be there for yourself, and make sure you’re on the list somewhere,”
DiMarco says. Plan ahead to ensure time for you in the daily routine, even if it’s just 15 to 30 minutes.
This may require an agreement with your partner and/or kids about when you will not be available
the next day. Be realistic about the amount of time you need and how you will spend it (e.g., 15
minutes on a meditation app). Then, stay committed to making it happen.
Remember the basics. We make sure our kids are fed, but are we sleeping, drinking enough water,
and eating well enough to meet our own basic needs? These stress management essentials
strengthen us, which will help us ward off the guilt.
Shift focus
Adjust expectations. “If you expect yourself to stay calm and never lose it, you’re setting yourself up
for failure,” says child and adolescent psychologist Emily W. King. “Own it that you’re more
emotionally fragile right now, notice it and walk outside for 10 minutes, or whatever you need, instead
of getting in a negative feedback loop, and then you’re emotionally exhausted.”
Consider your successes. Flip the script so you’re not focusing only on the negative. What do you feel
proud of? What can your children do now they couldn’t do a year ago? Redefine success for you and
your children dur ing a global pandemic. “Success is not about grades but about independence or
chores,” says King. For parents, success can be getting through each day with everyone sheltered, fed
and in bed safely.
Invent new narratives. Instead of looking at yourself as being never enough, how about
acknowledging that you have been — and continue to be — enough to endure a global pandemic
while parenting? What recent generation of parents has accomplished this? In fact, we are not just
enough, we are so much greater. Remind yourself of this when parent guilt attempts to tread that
familiar path, both now and after the pandemic.
(Adapted from https://www.washingtonpost.com/lifestyle/2021/02/25/how-cope-with-parent-guilt-during-pandemic-beyond/)

Como lidar com a culpa dos pais, durante a pandemia e além


Como mãe, sou vulnerável à influência de nossa mensagem cultural e sou vítima da narrativa
"Nunca podemos dar o suficiente, fazer o suficiente ou ser o suficiente". Como psicólogo, porém,
sei como esse fio constante de culpa é arriscado para o nosso bem-estar. A culpa pode ser útil
como uma emoção desconfortável que nos motiva a fazer reparações e mudar comportamentos

AULA 04 – ADJECTIVES AND ADVERBS 129


TEACHER ANDREA BELO

prejudiciais, mas no caso da culpa dos pais, na maior parte do tempo, estamos apenas nos
prejudicando.
Entre em uma pandemia global que destruiu nossa já precária vida de pais e forneceu ampla
evidência das falhas de nossos sistemas. Os empregadores estão esperando que nossos filhos com
aprendizagem remota se comportem durante o nosso dia de trabalho, e as mulheres estão
deixando o emprego em números recordes para seus filhos repentinamente presos em casa,
porque "simplesmente não podem mais fazer tudo". Ilyse DiMarco, psicóloga clínica e autora do
próximo livro “Mom Brain”, resume o problema: “O problema com a culpa agora é que há culpa
potencial em qualquer coisa que você faça”.
Os níveis de estresse aumentaram, com os pais ultrapassando os não pais nas pesquisas, criando
um terreno fértil para problemas de saúde mental. “Sentimos que não estamos medindo em
alguma área ou talvez não estejamos fazendo as escolhas certas”, diz DiMarco. “Não é de
surpreender que, se sentimos que estamos fracassando, nos sentimos deprimidos; se estamos
preocupados em não tomar a decisão certa, ficamos ansiosos.”
A vida diária em uma pandemia nos deu uma série de novos motivos para nos sentirmos culpados.
Além da impossibilidade de trabalhar simultaneamente, cuidar dos filhos e, em alguns casos,
ensinar nossos filhos, enfrentamos diariamente decisões em torno da saúde e segurança.
Nós permitimos datas de jogo? E se todos os amigos do meu filho estiverem jogando basquete e
eu disser não? E se escolhermos a escola presencial e nosso filho obtiver 19 anos ou escolher uma
escola remota e nosso filho ficar deprimido?
Mães que trabalham não estão bem. Sem boas escolhas e sem boas respostas, mas culpa
garantida, com uma pitada de julgamento causado por pressões sociais.
“A culpa é uma emoção útil quando nos diz que fizemos algo errado”, diz a psicóloga clínica Jill
Stoddard, autora de “Be Mighty”. “O que está acontecendo agora é que sentimos que fizemos algo
errado, mesmo quando estamos fazendo o melhor que podemos.”
Então, o que podemos fazer sobre a culpa pesando em torno de nossos pescoços coletivos? Os
especialistas sugerem uma combinação de atenção plena, autocuidado significativo e mudança
de perspectiva.
Atenção plena
Não ignore a culpa; reconhecê-lo significa que você pode fazer algo a respeito. Mesmo que nossa
culpa relacionada à pandemia possa ser deslocada, Stoddard diz que ainda podemos usar o
sentimento para sua função positiva e fazer as pazes. Ela disse a seus filhos: “Eu realmente gostaria
de passar mais tempo com vocês, e tem sido tão difícil para todos nós. As coisas serão diferentes
algum dia. Lamento não poder estar mais ao seu lado.” Ela acrescenta: “Você não está dizendo,
‘Eu estraguei tudo’. Você está dizendo: ‘Dói meu coração, não posso fazer isso do jeito que gostaria
de poder agora.’”
Lembre-se de que você não está sozinho. Sentimentos de fracasso são uma experiência universal
dos pais durante a pandemia do coronavírus. Buscar apoio social é fundamental para a saúde
mental em tempos de estresse. Pode ser tão simples quanto enviar uma mensagem de texto para
um amigo para compartilhar "Mamãe falha" do seu dia. Mesmo esta breve conexão oferece apoio

AULA 04 – ADJECTIVES AND ADVERBS 130


TEACHER ANDREA BELO

e solidariedade, e pode diminuir nosso senso de autoculpa e inadequação quando ouvimos


sentimentos e experiências semelhantes de outras pessoas.
Pratique a bondade consigo mesmo. Fale consigo mesmo como faria com seu amigo: "Você está
fazendo o seu melhor neste momento." Essa prática ajuda a mudar os padrões de pensamento da
autocrítica para a autocompaixão, que é conhecida por aumentar as emoções positivas e diminuir
as negativas, incluindo a culpa.
Autocuidado verdadeiro
Priorize as demandas diárias de seu tempo e energia. DiMarco aconselha fazer isso perguntando:
"O que é mais importante aqui, no dia a dia ou de hora em hora?" Concentre-se no que é mais
importante na lista de prioridades e permita-se deixar de lado as outras tarefas potenciais, em vez
de se sentir culpado por, inevitavelmente, não realizar tudo.
Priorize você mesmo. “Uma peça importante, e contra-intuitiva, para gerenciar a culpa por não
estar lá para todos os outros, é também estar lá para você mesmo e certificar-se de que está na
lista em algum lugar”, diz DiMarco. Planeje com antecedência para garantir tempo para você na
rotina diária, mesmo que seja apenas de 15 a 30 minutos. Isso pode exigir um acordo com seu
parceiro e / ou filhos sobre quando você não estará disponível no dia seguinte. Seja realista sobre
a quantidade de tempo que você precisa e como irá gastá-lo (por exemplo, 15 minutos em um
aplicativo de meditação). Então, mantenha o compromisso de fazer acontecer.
Lembre-se do básico. Certificamo-nos de que nossos filhos estão alimentados, mas estamos
dormindo, bebendo água o suficiente e comendo bem o suficiente para atender às nossas
necessidades básicas? Esses princípios básicos de gerenciamento de estresse nos fortalecem, o
que nos ajudará a afastar a culpa.
Mudar o foco
Ajuste as expectativas. “Se você espera ficar calmo e nunca perder veja, você está se preparando
para o fracasso”, diz a psicóloga infantil e adolescente Emily W. King. “Reconheça que você está
mais frágil emocionalmente agora, observe e caminhe para fora por 10 minutos, ou o que você
precisar, em vez de entrar em um ciclo de feedback negativo, e então você está emocionalmente
exausto.”
Considere seus sucessos. Inverta o script para que você não se concentre apenas no negativo. Do
que você se orgulha? O que seus filhos podem fazer agora que não podiam fazer um ano atrás?
Redefina o sucesso para você e seus filhos durante uma pandemia global. “O sucesso não tem a
ver com notas, mas com independência ou tarefas domésticas”, diz King. Para os pais, o sucesso
pode ser passar cada dia com todos abrigados, alimentados e na cama com segurança.
Invente novas narrativas. Em vez de se ver como se nunca fosse o suficiente, que tal reconhecer
que você foi - e continua sendo - o suficiente para suportar uma pandemia global enquanto criava
filhos? Que geração recente de pais conseguiu isso? Na verdade, não somos apenas suficientes,
somos muito maiores. Lembre-se disso quando a culpa dos pais tentar trilhar esse caminho
familiar, agora e depois da pandemia.

AULA 04 – ADJECTIVES AND ADVERBS 131


TEACHER ANDREA BELO

A wristband that tells your boss if you are unhappy


At first glance the silicone wristband could be mistaken for one that tracks your heart rate when you
are doing exercise.
However, the wearable technology, called a Moodbeam, isn't here to monitor your physical health.
Instead it allows your employer to track your emotional state.
The gadget, which links to a mobile phone app and web interface, has two buttons, one yellow and
one blue. The idea is that you press the yellow one if you are feeling happy, and the blue one if you
are sad.
Aimed at companies who wish to monitor the wellbeing of staff who are working from home, the idea
is that employees are encouraged to wear the wristband (they can say no), and press the relevant
button as they see fit throughout the working week.
Managers can then view an online dashboard to see how workers are feeling and coping. With bosses
no longer able to check in physically with their team, Moodbeam hopes to bridge the gap.
"Businesses are trying to get on top of staying connected with staff working from home. Here they can
ask 500 members: 'You ok?' without picking up the phone," says Moodbeam co-founder Christina
Colmer McHugh.
She originally came up with the idea for the product after she discovered that her daughter was
struggling at school, and she wanted a way for her child to let her know how she was feeling. The
wristband was launched commercially in 2016.
(Adapted from https://www.bbc.com/news/business-55637328)

Uma pulseira que diz ao seu chefe se você está infeliz


À primeira vista, a pulseira de silicone pode ser confundida com uma que monitora sua freqüência
cardíaca quando você está fazendo exercícios.
No entanto, a tecnologia vestível, chamada Moodbeam, não está aqui para monitorar sua saúde física.
Em vez disso, permite que seu empregador rastreie seu estado emocional.
O gadget, que se vincula a um aplicativo de celular e interface da web, tem dois botões, um amarelo
e um azul. A ideia é que você aperte o amarelo se estiver se sentindo feliz e o azul se estiver triste.
Destinado a empresas que pretendem monitorizar o bem-estar dos colaboradores que trabalham a
partir de casa, a ideia é que os colaboradores sejam incentivados a usar a pulseira (podem dizer não)
e a premir o botão correspondente à medida que quiserem ao longo da semana de trabalho.
Os gerentes podem então visualizar um painel online para ver como os trabalhadores estão se
sentindo e lidando com a situação. Com os chefes não sendo mais capazes de fazer o check-in
fisicamente com sua equipe, a Moodbeam espera preencher a lacuna.
“As empresas estão tentando se manter conectadas aos funcionários que trabalham em casa. Aqui,
eles podem perguntar a 500 membros: 'Você está bem?' sem atender o telefone ", diz a co-fundadora
da Moodbeam, Christina Colmer McHugh.
Ela originalmente teve a ideia do produto depois de descobrir que sua filha estava passando por
dificuldades na escola e queria uma maneira de seu filho dizer a ela como estava se sentindo. A
pulseira foi lançada comercialmente em 2016.

AULA 04 – ADJECTIVES AND ADVERBS 132


TEACHER ANDREA BELO

Vegan Restaurant Gets Michelin Star in France, a First


Lemongrass, seaweed and fir — yes, the tree — are not the sorts of ingredients that once earned
French chefs plaudits in the Michelin Guide, but on Monday the thick red bible of gastronomy
announced that it was giving its first star to a fully vegan restaurant in France.
The bestowal of the star to ONA, a restaurant near Bordeaux, is more evidence that a country long
renown for classics like coq au vin, blanquette de veau and boeuf bourguignon has opened up to
animal-free cuisine. A growing number of chefs are cutting meat from their menus, sometimes
entirely.
“It’s a new movement in France, where diets are still very meat-based,” said Claire Vallée, the chef
at ONA, which opened five years ago in Arès, a small Atlantic Coast town about 25 miles west of
Bordeaux.
“Each has its place,” Ms. Vallée said. “We want to show that you can eat differently.”
The restaurant’s very name speaks to France’s shifting culinary landscape: It stands for origine
non-animale.
Vegan establishments have already received Michelin stars in the United States, Spain and
Germany. But this is a first for France.
ONA shuns all animal-based products, Ms. Vallée said, even in its decorations and furnishings. It
does not, for instance, use wool or leather.
Last fall, its seven-course menu featured dishes with intriguing combinations of fir, boletus
mushroom and sake, or dulse seaweed, lemongrass and galangal, a relative of ginger. The
restaurant is currently closed because of the coronavirus pandemic.
The Michelin Guide, in a statement this week, said Ms. Vallée had given vegan cuisine its “letters
of nobility.”
Gwendal Poullenec, the international head of the Michelin Guides, noted that the move away
from meat was not entirely new.
Alain Passard, the owner and chef at L’Arpège in Paris, removed meat from his menu two decades
ago, and Alain Ducasse, perhaps France’s biggest culinary superstar, also decided to drastically
reduce the amount of meat used at his flagship Parisian restaurant.
But awarding a star to a restaurant that is not just meatless but avowedly vegan has the potential
to shake things up even further, Mr. Poullenec said.
“The general public might not associate pure veganism with a gastronomical experience,” he said.
A Michelin star might “liberate” chefs who are still reluctant to explore plant-based cooking, he
said.
(Adapted from https://www.nytimes.com/2021/01/19/world/europe/ona-michelin-star-france.html?action=click&module=News&pgtype=Homepage)

Vegan Restaurant recebe estrela Michelin na França, a primeira


Capim-limão, algas marinhas e abeto - sim, a árvore - não são os tipos de ingredientes que outrora
mereceram elogios de chefs franceses no Guia Michelin, mas na segunda-feira a espessa bíblia
vermelha da gastronomia anunciou que estava dando sua primeira estrela a um restaurante
totalmente vegano na França.

AULA 04 – ADJECTIVES AND ADVERBS 133


TEACHER ANDREA BELO

A concessão da estrela ao ONA, um restaurante perto de Bordeaux, é mais uma evidência de que
um país conhecido por clássicos como coq au vin, blanquette de veau e boeuf bourguignon se
abriu para uma culinária sem animais. Um número crescente de chefs está cortando carne de seus
menus, às vezes inteiramente.
“É um novo movimento na França, onde as dietas ainda são muito baseadas em carne”, disse Claire
Vallée, a chef da ONA, que abriu há cinco anos em Arès, uma pequena cidade da costa atlântica a
cerca de 40 quilômetros a oeste de Bordeaux.
“Cada um tem seu lugar”, disse Vallée. “Queremos mostrar que você pode comer de forma
diferente.”
O próprio nome do restaurante fala com a paisagem culinária em mudança da França: significa
origine non-animale.
Os estabelecimentos veganos já receberam estrelas Michelin nos Estados Unidos, Espanha e
Alemanha. Mas esta é a primeira vez na França.
A ONA rejeita todos os produtos de origem animal, disse Vallée, até mesmo em suas decorações
e móveis. Não usa, por exemplo, lã ou couro.
No outono passado, seu menu de sete pratos apresentava pratos com combinações intrigantes de
abeto, cogumelo boletus e saquê, ou algas marinhas, capim-limão e galanga, um parente do
gengibre. O restaurante está fechado atualmente por causa da pandemia do coronavírus.
O Guia Michelin, em um comunicado esta semana, disse que Vallée deu à cozinha vegana suas
"cartas de nobreza".
Gwendal Poullenec, chefe internacional dos Guias Michelin, observou que o afastamento da carne
não foi totalmente novo.
Alain Passard, o proprietário e chef do L'Arpège em Paris, retirou a carne de seu menu há duas
décadas, e Alain Ducasse, talvez a maior estrela da culinária da França, também decidiu reduzir
drasticamente a quantidade de carne usada em seu restaurante parisiense emblemático.
Mas premiar um restaurante que não é apenas sem carne, mas declaradamente vegano, tem o
potencial de agitar ainda mais as coisas, disse Poullenec.
“O público em geral pode não associar o veganismo puro com uma experiência gastronômica”,
disse ele. Uma estrela Michelin pode “liberar” chefs que ainda relutam em explorar a culinária
baseada em vegetais, disse ele.

Why you’re more creative in coffee shops


If we’re already working in isolation at home, why do we miss working with our heads
similarly down in a public setting?
Some of the most successful people in history have done their best work in coffee shops.
Pablo Picasso, JK Rowling, Simone de Beauvoir and Jean-Paul Sartre, Bob Dylan – whether they’re
painters, singersongwriters, philosophers or writers, people across nations and centuries have
tapped into their creativity working away at a table in a café.

AULA 04 – ADJECTIVES AND ADVERBS 134


TEACHER ANDREA BELO

Of course, Covid-19 has put the kibosh on lingering for hours in cozy rooms packed with people
sipping lattes. As we begin another year living amid a pandemic, many of us continue to work
remotely on our own. And if remote work becomes permanent for some – as many experts predict
– we might ask ourselves why, when things settle down, we should bother going back out to work
in public, only to ostensibly isolate ourselves with our heads down – something we’re already
doing at home.
But putting on your noise-cancelling headphones to toil away at your desk is actually different
than doing the same surrounded by other people buzzing over your shoulder. There are many
ways coffee shops trigger our creativity in a way offices and homes don’t. Research shows that
the stimuli in these places make them effective environments to work; the combination of noise,
casual crowds and visual variety can give us just the right amount of distraction to help us be our
sharpest and most creative. (So, no, it’s not just that double espresso.)
A sweet spot of noise and crowds
Some of us stick in our earbuds as soon as we sit down to work in a public setting. But scientists
have known for years that background noise can benefit our creative thinking.
A 2012 study published in the Journal of Consumer Research showed that a low-to-moderate level
of ambient noise in a place like a cafeteria can actually boost your creative output. The idea is that
if you’re very slightly distracted from the task at hand by ambient stimuli, it boosts your abstract
thinking ability, which can lead to more creative idea generation.
(Adapted from https://www.bbc.com/worklife/article/20210114-why-youre-more-creative-in-coffee-shops)

Por que você é mais criativo em cafeterias


Se já estamos trabalhando isolados em casa, por que sentimos falta de trabalhar com a cabeça
baixa em um ambiente público?
Algumas das pessoas mais bem-sucedidas da história fizeram seu melhor trabalho em cafeterias.
Pablo Picasso, JK Rowling, Simone de Beauvoir e Jean-Paul Sartre, Bob Dylan - sejam eles
pintores, cantores, escritores, filósofos ou escritores, pessoas de várias nações e séculos usaram
sua criatividade trabalhando à mesa de um café.
Claro, Covid-19 colocou o kibosh em permanecer por horas em quartos aconchegantes lotados
de pessoas tomando café com leite. Ao começarmos mais um ano vivendo em meio a uma
pandemia, muitos de nós continuamos a trabalhar remotamente por conta própria. E se o
trabalho remoto se tornar permanente para alguns - como muitos especialistas prevêem -
podemos nos perguntar por que, quando as coisas se acalmam, devemos nos preocupar em
voltar a trabalhar em público, apenas para nos isolarmos ostensivamente de cabeça baixa - algo
que estamos já está fazendo em casa.
Mas colocar seus fones de ouvido com cancelamento de ruído para trabalhar duro em sua mesa
é, na verdade, diferente de fazer o mesmo rodeado por outras pessoas zumbindo por cima do
seu ombro. Há muitas maneiras pelas quais as cafeterias acionam nossa criatividade de uma
maneira que os escritórios e residências não fazem. A pesquisa mostra que os estímulos nesses
locais os tornam ambientes eficazes para trabalhar; a combinação de ruído, multidões casuais e

AULA 04 – ADJECTIVES AND ADVERBS 135


TEACHER ANDREA BELO

variedade visual pode nos dar a quantidade certa de distração para nos ajudar a ser mais nítidos
e criativos. (Então, não, não é apenas aquele expresso duplo.)
Um ótimo local de barulho e multidões
Alguns de nós colocam os fones de ouvido assim que nos sentamos para trabalhar em um
ambiente público. Mas os cientistas sabem há anos que o ruído de fundo pode beneficiar nosso
pensamento criativo.
Um estudo de 2012 publicado no Journal of Consumer Research mostrou que um nível baixo a
moderado de ruído ambiente em um lugar como uma cafeteria pode realmente aumentar sua
produção criativa. A ideia é que se você estiver ligeiramente distraído da tarefa em mãos por
estímulos ambientais, isso aumenta sua capacidade de pensamento abstrato, o que pode levar a
uma geração de ideias mais criativas.

Electric cars are not the only green solution


Chris Barker says more cars of any kind are bad news for pollution, while Dr Robin Russell-
Jones believes British entrepreneurs can rise to the challenge of ‘green growth’
It is indeed good news that the advance in battery technology brings down the costs and
inconveniences associated with electric cars. But it does not begin to solve the other problems
associated with the volume of traffic on the roads. In fact, if electric cars become cheaper and
continue to benefit from tax relief, the total number of cars may well rise. Electric cars produce
no toxic emissions at the point of use but, as with all cars, embodied carbon in the production of
vehicles and batteries generate greenhouse gas emissions. In addition, research by the
government in 2019 found that two-thirds of the contamination from conventional vehicles arises
from tyre, brake and road dust.
There are also other problems associated with too many cars of any type. The greater the level of
traffic, the greater the danger of death or injury and the need for more and wider roads, creating
no-go areas for people and cutting off communities from each other. Too much reliance on cars
promotes a sedentary lifestyle and brings health risks.
For a better life and to counteract the climate emergency it is necessary to reduce traffic. This
cannot be done by promoting electric cars. It needs the promotion of walking, cycling and public
transport, and of measures to reduce the need to travel.
(Adapted from https://www.theguardian.com/environment/2021/jan/25/electric-cars-are-not-the-only-green-solution)

Os carros elétricos não são a única solução verde


Chris Barker diz que mais carros de qualquer tipo são más notícias para a poluição, enquanto o
Dr. Robin Russell-Jones acredita que os empresários britânicos podem enfrentar o desafio do
‘crescimento verde’
Na verdade, é uma boa notícia que o avanço na tecnologia de baterias reduza os custos e os
inconvenientes associados aos carros elétricos. Mas não começa a resolver os outros problemas
associados ao volume de tráfego nas estradas. Na verdade, se os carros elétricos ficarem mais

AULA 04 – ADJECTIVES AND ADVERBS 136


TEACHER ANDREA BELO

baratos e continuarem a se beneficiar da redução de impostos, o número total de carros poderá


aumentar. Os carros elétricos não produzem emissões tóxicas no ponto de uso, mas, como
acontece com todos os carros, o carbono incorporado na produção de veículos e baterias gera
emissões de gases de efeito estufa. Além disso, uma pesquisa do governo em 2019 descobriu
que dois terços da contaminação de veículos convencionais são provenientes de pneus, freios e
poeira da estrada.
Existem também outros problemas associados a muitos carros de qualquer tipo. Quanto maior o
nível de tráfego, maior o perigo de morte ou ferimentos e a necessidade de mais estradas e
mais largas, criando áreas proibidas para as pessoas e isolando as comunidades umas das
outras. Depender demais de carros promove um estilo de vida sedentário e traz riscos à saúde.
Para uma vida melhor e para enfrentar a emergência climática é necessário reduzir o trânsito.
Isso não pode ser feito promovendo carros elétricos. Precisa da promoção de caminhadas,
ciclismo e transporte público, e de medidas para reduzir a necessidade de viagens.

Sinovac: Brazil results show Chinese vaccine 50.4% effective


A coronavirus vaccine developed by China's Sinovac has been found to be 50.4% effective in
Brazilian clinical trials, according to the latest results released by researchers.
It shows the vaccine is significantly less effective than previous data suggested - barely over the
50% needed for regulatory approval.
The Chinese vaccine is one of two that the Brazilian government has lined up. Brazil has been one
of the countries worst affected by Covid-19.
Sinovac, a Beijing-based biopharmaceutical company, is behind CoronaVac, an inactivated
vaccine. It works by using killed viral particles to expose the body's immune system to the virus
without risking a serious disease response.
Several countries, including Indonesia, Turkey and Singapore, have placed orders for the vaccine.
Last week researchers at the Butantan Institute, which has been conducting the trials in Brazil,
announced that the vaccine had a 78% efficacy against "mild-to-severe" Covid-19 cases.
But on Tuesday they revealed that calculations for this figure did not include data from a group of
"very mild infections" among those who received the vaccine that did not require clinical
assistance.
With the inclusion of this data, the efficacy rate is now 50.4%, said researchers.
But Butantan stressed that the vaccine is 78% effective in preventing mild cases that needed
treatment and 100% effective in staving off moderate to serious cases.
The Sinovac trials have yielded different results across different countries.
Last month Turkish researchers said the Sinovac vaccine was 91.25% effective, while Indonesia,
which rolled out its mass vaccination programme on Wednesday, said it was 65.3% effective. Both
were interim results from late-stage trials.
(Adapted from https://www.bbc.com/news/world-latin-america-55642648)

AULA 04 – ADJECTIVES AND ADVERBS 137


TEACHER ANDREA BELO

Sinovac: Resultados do Brasil mostram vacina chinesa 50,4% eficaz


Uma vacina contra o coronavírus desenvolvida pela Sinovac da China demonstrou ser 50,4% eficaz
em testes clínicos brasileiros, de acordo com os últimos resultados divulgados por pesquisadores.
Isso mostra que a vacina é significativamente menos eficaz do que os dados anteriores sugeridos
- pouco mais que 50% necessários para a aprovação regulatória.
A vacina chinesa é uma das duas que o governo brasileiro formou. O Brasil foi um dos países mais
afetados pela Covid-19.
A Sinovac, uma empresa biofarmacêutica com sede em Pequim, está por trás do CoronaVac, uma
vacina inativada. Ele age usando partículas virais mortas para expor o sistema imunológico do
corpo ao vírus, sem arriscar uma resposta de doença grave.
Vários países, incluindo Indonésia, Turquia e Cingapura, fizeram pedidos para a vacina. Na semana
passada, pesquisadores do Instituto Butantan, que vem realizando os testes no Brasil, anunciaram
que a vacina tinha 78% de eficácia contra os casos "leves a graves" de Covid-19.
Mas na terça-feira, eles revelaram que os cálculos para este número não incluem dados de um
grupo de "infecções muito leves" entre aqueles que receberam a vacina que não necessitou de
assistência clínica.
Com a inclusão desses dados, a taxa de eficácia passou a ser de 50,4%, disseram os pesquisadores.
Mas o Butantan ressaltou que a vacina é 78% eficaz na prevenção de casos leves que precisam de
tratamento e 100% eficaz na prevenção de casos moderados a graves.
Os testes Sinovac produziram resultados diferentes em diferentes países.
No mês passado, pesquisadores turcos disseram que a vacina Sinovac foi 91,25% eficaz, enquanto
a Indonésia, que lançou seu programa de vacinação em massa na quarta-feira, disse que era 65,3%
eficaz. Ambos foram resultados provisórios de estudos em estágio final.

Best part - Daniel Caesar / H.E.R


[...]
You don't know, babe
When you hold me
And kiss me slowly
It's the sweetest thing
And it don't change
If I had it my way
You would know that you are
You're the coffee that I need in the morning
You're my sunshine in the rain when it's pouring
Won't you give yourself to me
Give it all, oh
I just wanna see

AULA 04 – ADJECTIVES AND ADVERBS 138


TEACHER ANDREA BELO

I just wanna see how beautiful you are


You know that I see it
I know you're a star
Where you go I follow
No matter how far
If life is a movie
Oh you're the best part, oh oh oh
You're the best part, oh oh oh
Best part
It's the sunrise
And those brown eyes, yes
You're the one that I desire
When we wake up
And then we make love
It makes me feel so nice
You're my water when I'm stuck in the desert
You're the Tylenol I take when my head hurts
You're the sunshine on my life
I just wanna see how beautiful you are
You know that I see it
I know you're a star
Where you go I follow
No matter how far
If life is a movie
Then you're the best part, oh oh oh
You're the best part, oh oh oh
Best part
[...]
(Adapted from LyricFind)

Melhor parte - Daniel Caesar / H.E.R


[...]
Você nao sabe gata
Quando você me abraça
E me beije devagar
É a coisa mais doce
E isso não muda
Se eu tivesse do meu jeito
Você saberia que você é
Você é o café que eu preciso de manhã
Você é meu raio de sol na chuva quando está chovendo
Você não vai se dar para mim
Dê tudo, oh

AULA 04 – ADJECTIVES AND ADVERBS 139


TEACHER ANDREA BELO

Eu só quero ver
Eu só quero ver como você é linda
Você sabe que eu vejo isso
Eu sei que você é uma estrela
Onde você for eu sigo
Não importa a distância
Se a vida é um filme
Oh, você é a melhor parte, oh oh oh
Você é a melhor parte, oh oh oh
Melhor parte
É o nascer do sol
E aqueles olhos castanhos sim
Você é aquele que eu desejo
Quando acordamos
E então fazemos amor
Isso me faz sentir tão bem
Você é minha água quando estou preso no deserto
Você é o Tylenol que eu tomo quando minha cabeça dói
Você é o raio de sol na minha vida
Eu só quero ver como você é linda
Você sabe que eu vejo isso
Eu sei que você é uma estrela
Onde você for eu sigo
Não importa a distância
Se a vida é um filme
Então você é a melhor parte, oh oh oh
Você é a melhor parte, oh oh oh
Melhor parte
[...]

Why some people like wearing masks


Some people welcome face coverings for reasons ranging from the convenient and expedient
to the more complex and psychological. But is this a helpful coping mechanism?
Sheltering in place hasn’t been too hard for Jay Lee; watching a film at home and ordering a takeaway
has always been his idea of a good night. Lee, a 32-year-old small business owner in Leicester,
identifies as an introvert. And although 2020 had its hardships – in the spring, he was made redundant
from his job at a large bank – one perk for him has been the widespread adoption of face masks.
Lee has always dreaded run-ins with old friends and acquaintances around town, finding these
spontaneous interactions “extremely awkward”. He used to time his shopping trips to minimise the
possibility of bumping into someone he knew, waiting until almost closing time before heading out.
“Since I've been wearing the mask, my awkward interactions with friends and family have significantly
reduced,” he says. Now, he goes to the shops whenever he wants, without worrying about whom he

AULA 04 – ADJECTIVES AND ADVERBS 140


TEACHER ANDREA BELO

might see. He hopes that, even after the pandemic ends, it will still be socially acceptable to wear a
mask.
Wearing a mask is, for most of us, an annoying but worthwhile sacrifice: it’s one of the most effective
ways to slow the spread of Covid-19. Still, most of us look forward to the day when we can bare our
faces in public again. Face-coverings fog our glasses and clog our pores; they make it harder to smile
at strangers and recognise friends.
Yet some are secretly relishing the new mask-wearing mandates, for reasons ranging from the
convenient and expedient to the more complex and psychological. Some welcome the way face
coverings reduce or change interactions that might otherwise spark social anxiety.
‘Anonymity carries power’
At the lighter end of the scale, some people have found that masking offers a welcome relief from the
pressures to uphold strict standards of grooming and appearance. They have ditched their old makeup
and shaving routines and are saving money, time and stress. Others have discovered that hiding their
mouths affords them unexpected freedoms. Some restaurant servers and retail workers say they no
longer feel obliged to fake-smile at customers, potentially lifting the burden of emotional labour.
(Adapted from https://www.bbc.com/worklife/article/20210115-why-some-people-like-wearing-masks)

Por que algumas pessoas gostam de usar máscaras


Algumas pessoas gostam de coberturas faciais por razões que vão desde as convenientes e
convenientes até as mais complexas e psicológicas. Mas este é um mecanismo de
enfrentamento útil?
O abrigo no local não foi muito difícil para Jay Lee; assistir a um filme em casa e pedir comida para
viagem sempre foi sua ideia de uma boa noite. Lee, um pequeno empresário de 32 anos em
Leicester, se identifica como introvertido. E embora 2020 tenha tido suas dificuldades - na
primavera, ele foi despedido de seu emprego em um grande banco - uma vantagem para ele foi
a adoção generalizada de máscaras faciais.
Lee sempre temeu desentendimentos com velhos amigos e conhecidos pela cidade, achando
essas interações espontâneas “extremamente estranhas”. Ele costumava cronometrar suas idas
às compras para minimizar a possibilidade de esbarrar com alguém que conhecia, esperando
quase até a hora de fechar antes de sair. “Desde que estou usando a máscara, minhas interações
estranhas com amigos e familiares diminuíram significativamente”, diz ele. Agora, ele vai às lojas
quando quer, sem se preocupar com quem vai encontrar. Ele espera que, mesmo após o fim da
pandemia, ainda seja socialmente aceitável usar uma máscara.
Usar uma máscara é, para a maioria de nós, um sacrifício chato, mas vale a pena: é uma das
maneiras mais eficazes de retardar a disseminação de Covid-19. Ainda assim, a maioria de nós
espera o dia em que possamos mostrar nossos rostos em público novamente. As coberturas
faciais embaçam nossos óculos e obstruem nossos poros; eles tornam mais difícil sorrir para
estranhos e reconhecer amigos.
No entanto, alguns estão secretamente saboreando as novas regras de uso de máscaras, por
razões que vão desde as convenientes e convenientes até as mais complexas e psicológicas.

AULA 04 – ADJECTIVES AND ADVERBS 141


TEACHER ANDREA BELO

Alguns acolhem bem a maneira como as coberturas faciais reduzem ou mudam as interações que,
de outra forma, poderiam desencadear a ansiedade social.
‘Anonimato traz poder’
Na extremidade mais leve da escala, algumas pessoas descobriram que o mascaramento oferece
um alívio bem-vindo das pressões para manter padrões estritos de aparência e aparência. Eles
abandonaram suas velhas rotinas de maquiagem e barbear e estão economizando dinheiro,
tempo e estresse. Outros descobriram que esconder a boca lhes proporciona liberdades
inesperadas. Alguns atendentes de restaurantes e varejistas dizem que não se sentem mais
obrigados a sorrir falsamente para os clientes, o que pode aliviar o fardo do trabalho emocional.

The World's Biggest Problems Are Interconnected. Here's How We Can Solve Them This
Decade
Two decades ago, people around the world rang in the new millennium with a growing sense of
optimism. The threat posed by the Cold War was fading slowly in the rearview mirror. Leading
thinkers like Francis Fukuyama touted the benefits of globalization, saying it would bring
democracy and prosperity to the developing world. The nascent Internet economy promised to
bring us closer together.
The following 20 years took some of the air out of the assumption of steady progress, but when
future historians assess the 21st century, the year 2020 is likely to serve as the point at which the
optimism bubble burst. The COVID-19 pandemic has exposed a complex web of interlocking
problems that have morphed into full-blown crises. The coronavirus laid bare the dangers of
endemic poverty not only in the developing world but also in rich countries like the U.S., where
millions lack health care and are one paycheck away from living on the street. Around the world,
racial and ethnic minorities have demanded justice after centuries of structural discrimination.
Woven through it all, the earth’s climate is increasingly unstable, posing an existential threat to
human society as we know it. In the next decade, societies will be forced to either confront this
snarl of challenges, or be overwhelmed by them. Our response will define the future for decades
to come.
The recognition that these challenges are fundamentally linked isn’t new. Activists and academics
have for many years pointed to the cascading effects of various social ills. Whether it’s the way
racism contributes to poor health outcomes or gender discrimination harms economic growth,
the examples are seemingly endless. But this understanding has made its way into the
conversation about solutions too.
Notably, for the past five years, the U.N. has touted 17 interrelated sustainable development
goals, objectives for building a more viable world, and called for a push to achieve them by 2030.
The goals, which cover environmental, social and economic progress, are nonbinding but have
become key benchmarks for commitments at a national and corporate level. Countries from China
to the Maldives, as well as companies like Amazon, Microsoft and PwC, have committed to rolling
out policies over the next decade that will set them on a path to eliminate their carbon footprints.

AULA 04 – ADJECTIVES AND ADVERBS 142


TEACHER ANDREA BELO

The understanding that these problems require holistic solutions has only grown amid the
pandemic and its fallout. President Joe Biden has referred to four urgent crises—the pandemic,
the economic crisis, racial injustice and climate change—and promised a push to tackle them all
together. The European Union’s program to propel the bloc out of the COVID-19 crisis targets
climate change, while incorporating equity concerns. As stock markets soared last year,
institutions with trillions of dollars in assets demanded that their investments deliver not only a
good return for their wallets but also a good return for society.
(Adapted from https://time.com/5931603/how-to-solve-worlds-biggest-problems/)

Os maiores problemas do mundo estão interligados. Veja como podemos resolvê-los nesta
década
Duas décadas atrás, as pessoas em todo o mundo chegavam ao novo milênio com um crescente
senso de otimismo. A ameaça representada pela Guerra Fria estava desaparecendo lentamente
no espelho retrovisor. Pensadores importantes como Francis Fukuyama elogiaram os benefícios
da globalização, dizendo que ela traria democracia e prosperidade ao mundo em
desenvolvimento. A economia nascente da Internet prometia nos aproximar.
Os 20 anos seguintes tiraram um pouco do ar da suposição de progresso constante, mas quando
os futuros historiadores avaliarem o século 21, o ano de 2020 provavelmente servirá como o
ponto em que a bolha de otimismo estourou. A pandemia COVID-19 expôs uma complexa teia de
problemas interligados que se transformaram em crises completas. O coronavírus revelou os
perigos da pobreza endêmica não apenas no mundo em desenvolvimento, mas também em
países ricos como os EUA, onde milhões não têm assistência médica e estão a um salário de viver
nas ruas. Em todo o mundo, as minorias raciais e étnicas exigem justiça após séculos de
discriminação estrutural. Tecido por tudo isso, o clima da Terra está cada vez mais instável,
representando uma ameaça existencial para a sociedade humana como a conhecemos. Na
próxima década, as sociedades serão forçadas a enfrentar esse emaranhado de desafios ou ser
oprimidas por eles. Nossa resposta definirá o futuro nas próximas décadas.
O reconhecimento de que esses desafios estão fundamentalmente ligados não é novo. Por muitos
anos, ativistas e acadêmicos apontaram os efeitos em cascata de vários males sociais. Quer seja
a forma como o racismo contribui para resultados ruins de saúde ou a discriminação de gênero
prejudica o crescimento econômico, os exemplos são aparentemente infinitos. Mas esse
entendimento também entrou na conversa sobre soluções.
Notavelmente, nos últimos cinco anos, a ONU apresentou 17 metas de desenvolvimento
sustentável inter-relacionadas, objetivos para a construção de um mundo mais viável, e pediu um
impulso para alcançá-los até 2030. As metas, que abrangem o progresso ambiental, social e
econômico, são não vinculantes, mas se tornaram referências-chave para compromissos em nível
nacional e corporativo. Países da China às Maldivas, bem como empresas como Amazon,
Microsoft e PwC, se comprometeram a implementar políticas na próxima década que os colocarão
no caminho para eliminar suas pegadas de carbono.
A compreensão de que esses problemas exigem soluções holísticas apenas cresceu em meio à
pandemia e suas consequências. O presidente Joe Biden se referiu a quatro crises urgentes - a
pandemia, a crise econômica, a injustiça racial e a mudança climática - e prometeu um esforço

AULA 04 – ADJECTIVES AND ADVERBS 143


TEACHER ANDREA BELO

para enfrentá-las todas juntas. O programa da União Europeia para tirar o bloco da crise COVID-
19 visa as mudanças climáticas, ao mesmo tempo em que incorpora questões de equidade.
Enquanto os mercados de ações disparavam no ano passado, as instituições com trilhões de
dólares em ativos exigiam que seus investimentos proporcionassem não apenas um bom retorno
para suas carteiras, mas também um bom retorno para a sociedade.

I remember the last time I had _______ choose between optimism and fear, between hope and
the urge to run away. It was right after 9/11. I had babies—one so tiny she was still curled up like
a fern, _______ other toddling around gumming everything she could get her little starfish hands
on. And as their brand-new lungs took _______ the smoke that blew over to Brooklyn _______
the burning towers, I wanted to pack them up and flee to some safer place.
(Adapted from https://time.com/5930399/the-capitol-riots-and-our-fragile-optimism/)

Lembro-me da última vez em que _______ escolhi entre otimismo e medo, entre esperança e
vontade de fugir. Foi logo depois do 11 de setembro. Eu tive bebês - um tão pequeno que ela
ainda estava enrolado como uma samambaia, _______ o outro engatinhando em torno de goma
de tudo que ela podia colocar suas pequenas mãos de estrela do mar. E enquanto seus pulmões
novos levavam _______ a fumaça que soprava para o Brooklyn _______ as torres em chamas, eu
queria embalá-los e fugir para um lugar mais seguro.

For the European Union, the rapid rollout of Covid-19 vaccines __________ critical to save lives
and prevent health services from ____________ stretched beyond their limits, not to mention
minimising the massive economic damage from lockdowns. Unfortunately, however, though
vaccinations are under way, a rapid near-term increase in infections is likely as the British variant
of the virus __________ across the continent.
(Adapted from https://www.concursosmilitares.com.br/provas-anteriores/marinha/efomm/2020-efomm-oficial-da-marinha-mercante-primeiro-dia.pdf)

Para a União Europeia, a rápida distribuição das vacinas Covid-19 __________ crítica para salvar
vidas e evitar que os serviços de saúde ____________ se estendeu além de seus limites, sem
mencionar a minimização dos enormes danos econômicos dos bloqueios. Infelizmente, no
entanto, embora as vacinações estejam em andamento, um rápido aumento de infecções em
curto prazo é provável, visto que a variante britânica do vírus __________ em todo o continente.

The U.S. has tragically surpassed 400,000 COVID-19 deaths, and case numbers and hospitalizations
are likewise spiking to record levels around the world. __________ vaccines now rolling out, there is
reason to hope that there is an end in sight. __________ , by most estimates, widespread vaccinations

AULA 04 – ADJECTIVES AND ADVERBS 144


TEACHER ANDREA BELO

will not be in place until the middle of the year at the earliest. __________ , we have some ways to
go yet with social distancing, mask wearing and other pandemic mitigation behaviors.
(Adapted from https://www.scientificamerican.com/article/how-we-can-deal-with-pandemic-fatigue/)

Os EUA ultrapassaram tragicamente 400.000 mortes por COVID-19, e o número de casos e


hospitalizações também estão atingindo níveis recordes em todo o mundo. __________ vacinas
agora sendo lançadas, há motivos para esperar que haja um fim à vista. __________, pela maioria
das estimativas, as vacinações generalizadas não estarão em vigor até meados do ano, no mínimo.
__________, ainda temos alguns caminhos a percorrer com o distanciamento social, o uso de
máscaras e outros comportamentos de mitigação de pandemia.

Facebook building a version of Instagram for children under 13


Social media giant says it’s exploring introducing a parent-controlled experience that allows
kids to ‘safely’ use the photo sharing platform
Facebook is considering launching a version of its popular photo social media platform, Instagram,
for children under the age of 13.
BuzzFeed News first reported Facebook announced in an internal company post that the company
would begin building a version of Instagram for people under the age of 13 years to allow them
to “safely” use Instagram for the first time. Currently the company does not allow people who are
under this age to create an account on the platform.
A spokesperson for Facebook told the Guardian the company was exploring a parent-controlled
version of Instagram, similar to the Messenger Kids app that is for kids between six and 12.
“Increasingly kids are asking their parents if they can join apps that help them keep up with their
friends. Right now there aren’t many options for parents, so we’re working on building additional
products … that are suitable for kids, managed by parents,” the spokesperson said.
“We’re exploring bringing a parent-controlled experience to Instagram to help kids keep up with
their friends, discover new hobbies and interests, and more.”
In a blog post earlier this week, which did not mention the proposed new Instagram service, the
company noted that although people were asked to enter their age when signing up for
Instagram, there was nothing to prevent people from lying about it at registration.
Facebook said it would overcome that by using machine learning in combination with the
registration age to determine people’s ages on the platform.
The company also announced plans to roll out new safety features, including preventing adults
from messaging people under the age of 18 who do not follow them, safety notices for teens
when messaged by an adult sending a large amount of friend requests or messages to people
under 18, and make it more difficult for adults to find and follow teens using the search function
in Instagram.
Teens will also be encouraged to put their profiles on private at the point of registration.

AULA 04 – ADJECTIVES AND ADVERBS 145


TEACHER ANDREA BELO

A study of Australian teens’ internet usage published by the Australian eSafety commissioner in
February found 57% of Australian teenagers use Instagram, while 30% reported being contacted
by a stranger, and 20% reported being sent inappropriate unwanted content on the social media
sites they used.
(Adapted from https://www.theguardian.com/technology/2021/mar/19/facebook-building-a-version-of-instagram-for-children-under-13)

Facebook criando uma versão do Instagram para crianças menores de 13 anos


A gigante da mídia social diz que está explorando a introdução de uma experiência controlada
pelos pais que permite que as crianças usem "com segurança" a plataforma de
compartilhamento de fotos
O Facebook está considerando lançar uma versão de sua popular plataforma de mídia social de
fotos, Instagram, para crianças menores de 13 anos.
O BuzzFeed News relatou pela primeira vez que o Facebook anunciou em um post interno da
empresa que a empresa começaria a construir uma versão do Instagram para menores de 13 anos
para permitir que usassem o Instagram “com segurança” pela primeira vez. Atualmente a
empresa não permite que menores de idade criem uma conta na plataforma.
Um porta-voz do Facebook disse ao Guardian que a empresa estava explorando uma versão do
Instagram controlada pelos pais, semelhante ao aplicativo Messenger Kids que é para crianças
entre seis e 12 anos.
“Cada vez mais as crianças estão perguntando a seus pais se podem aderir a aplicativos que os
ajudem a manter o contato com seus amigos. No momento não há muitas opções para os pais,
então estamos trabalhando na construção de produtos adicionais ... que sejam adequados para
crianças, gerenciados pelos pais ”, disse o porta-voz.
“Estamos explorando trazer uma experiência controlada pelos pais para o Instagram para ajudar
as crianças a acompanhar seus amigos, descobrir novos hobbies e interesses e muito mais.”
Em uma postagem no blog no início desta semana, que não mencionou a proposta do novo serviço
Instagram, a empresa observou que, embora as pessoas tenham sido solicitadas a inserir sua
idade ao se inscrever no Instagram, não havia nada que as impedisse de mentir sobre isso no
momento do registro.
O Facebook disse que iria superar isso usando o aprendizado de máquina em combinação com a
idade do registro para determinar a idade das pessoas na plataforma.
A empresa também anunciou planos para lançar novos recursos de segurança, incluindo a
prevenção de adultos de enviar mensagens para pessoas menores de 18 anos que não os seguem,
avisos de segurança para adolescentes quando mensagens de um adulto enviam uma grande
quantidade de pedidos de amizade ou mensagens para pessoas com menos de 18, e torna mais
difícil para os adultos encontrar e seguir adolescentes usando a função de pesquisa do Instagram.
Os adolescentes também serão incentivados a colocar seus perfis em privado no momento do
registro.
Um estudo sobre o uso da Internet por adolescentes australianos publicado pelo comissário de
eSafety australiano em fevereiro revelou que 57% dos adolescentes australianos usam o

AULA 04 – ADJECTIVES AND ADVERBS 146


TEACHER ANDREA BELO

Instagram, enquanto 30% relataram ter sido contatados por um estranho e 20% relataram ter
recebido conteúdo impróprio e indesejado nos sites de mídia social que eles usado.

COVID-19 Conspiracy Theories Are Spreading Rapidly—and They're a Public Health Risk All
Their Own
Public health crises have spawned conspiracy theories as far back as when the Black Death
ravaged Europe in the 1300s, as people desperately try to make sense of the chaotic forces
disrupting their lives. While modern science offers a better understanding of how diseases infect
people and how to contain them, COVID-19 conspiracy theories are spreading rapidly via social
media, unreliable news outlets and from our own political leaders, including U.S. President Donald
Trump. The result: many Americans now believe pandemic -related conspiracy theories—and,
alarmingly, those same people are less likely to take steps to prevent the virus from spreading.
In a University of Pennsylvania Annenberg Public Policy Center study published Monday in Social
Science &
Medicine, researchers surveyed a group of 840 U.S. adults—first in late March, and then again in
mid-July—to determine how Americans’ beliefs and actions regarding the pandemic changed over
time. Overall, they found that COVID-19 conspiracy theories are not only commonplace, they’re
gaining traction. Back in March, 28% of people believed a debunked rumor that the Chinese
government created the coronavirus as a bioweapon; that number rose to 37% by July. About
24% believed that the U.S. Centers for Disease Control and Prevention exaggerated the virus’
danger to hurt Trump politically despi te a lack of evidence; by July, that figure rose to 32%. And
in March, about 15% of respondents said they believed that the pharmaceutical industry created
the virus to boost drug and vaccine sales—another unfounded theory—compared to 17% in July.
(Adapted from https://time.com/5891333/covid-19-conspiracy-theories/)

As teorias da conspiração COVID-19 estão se espalhando rapidamente - e são um risco para a


saúde pública por conta própria
As crises de saúde pública geraram teorias da conspiração desde quando a Peste Negra devastou
a Europa nos anos 1300, enquanto as pessoas tentam desesperadamente entender as forças
caóticas que perturbam suas vidas. Enquanto a ciência moderna oferece uma melhor
compreensão de como as doenças infectam as pessoas e como contê-las, as teorias da
conspiração COVID-19 estão se espalhando rapidamente por meio da mídia social, de meios de
comunicação não confiáveis e de nossos próprios líderes políticos, incluindo o presidente dos
EUA, Donald Trump. Resultado: muitos americanos agora acreditam em teorias de conspiração
relacionadas à pandemia - e, de forma alarmante, essas mesmas pessoas têm menos
probabilidade de tomar medidas para evitar que o vírus se espalhe.
Em um estudo do Annenberg Public Policy Center da Universidade da Pensilvânia, publicado na
segunda-feira na Social Science &
Em medicina, os pesquisadores entrevistaram um grupo de 840 adultos norte-americanos -
primeiro no final de março e depois novamente em meados de julho - para determinar como as

AULA 04 – ADJECTIVES AND ADVERBS 147


TEACHER ANDREA BELO

crenças e ações dos americanos em relação à pandemia mudaram ao longo do tempo. No geral,
eles descobriram que as teorias da conspiração COVID-19 não são apenas comuns, elas estão
ganhando força. Em março, 28% das pessoas acreditavam em um boato desmentido de que o
governo chinês criou o coronavírus como uma arma biológica; esse número subiu para 37% em
julho. Cerca de 24% acreditam que os Centros de Controle e Prevenção de Doenças dos Estados
Unidos exageram o perigo do vírus de ferir Trump politicamente, desrespeitando a falta de
evidências; em julho, esse número subiu para 32%. E em março, cerca de 15% dos entrevistados
disseram acreditar que a indústria farmacêutica criou o vírus para impulsionar as vendas de
medicamentos e vacinas - outra teoria infundada - em comparação com 17% em julho.

Covid babies don't have to be the unlucky generation. But they must be helped
A year into this pandemic, there are babies now learning to walk and talk who have never known
anything but life under the shadow of Covid, and preschoolers who can barely remember a world
before it. Doctors’ children have had to learn not to touch Mummy when she gets in from work,
until she’s had a shower to wash off any last possible trace of danger.
Thanks to popular toddler demand, you can now buy masks for dolls.
It’s perfectly normal for kids to reflect what’s happening around them by playing quarantine with
stuffed animals, but normal too for adults to wonder uncomfortably whether all this leaves a
lasting mark. How much will it matter in years to come that, as the minister for loneliness Diana
Barran recently put it, there are toddlers being raised by shielding parents who have never had a
playdate? Will Covid babies grow up solitary creatures, used to entertaining themselves, or warier
of the strangers they so rarely meet and interact with? Taking a tiny bundle out in public used to
mean an endless succession of random older women cooing over the pram, or strangers pulling
faces to entertain a bored baby in a checkout queue.
But now passersby daren’t get close, and other shoppers are hidden behind masks.
This week MPs were presented with some early findings from a project led by the First 1001 Days
Movement, an alliance of early-years charities and professionals, tracking the lives of under-twos
growing up through a pandemic. A survey of children’s service providers it commissioned found
98% thought the babies and toddlers they worked with had been affected by higher parental
stress and anxiety, while 92% had seen fearful families effectively cutting themselves off from the
outside world, skipping routine appointments or not wanting to leave the house. Nine in 10 had
observed children being played with less, or being less active. Heartbreakingly, more than a
quarter said lockdown left the children they worked with more exposed to domestic conflict,
abuse or neglect.
(Adapted from https://www.theguardian.com/commentisfree/2021/jan/22/covid-babies-unlucky-generation-help-lockdown)

Bebês cobiçosos não precisam ser a geração azarada. Mas eles devem ser ajudados
Um ano após o início da pandemia, há bebês aprendendo a andar e a falar que nunca conheceram
nada além da vida sob a sombra de Covid, e crianças em idade pré-escolar que mal conseguem se
lembrar de um mundo antes dela. Os filhos dos médicos tiveram que aprender a não tocar na

AULA 04 – ADJECTIVES AND ADVERBS 148


TEACHER ANDREA BELO

mamãe quando ela chega do trabalho, até que ela tome um banho para lavar qualquer último
vestígio de perigo.
Graças à demanda popular de crianças, agora você pode comprar máscaras para bonecas.
É perfeitamente normal que as crianças reflitam o que está acontecendo ao seu redor brincando
de quarentena com bichos de pelúcia, mas também é normal que os adultos se perguntem
desconfortavelmente se tudo isso deixa uma marca duradoura. Quanto importará nos próximos
anos que, como disse recentemente a ministra da solidão Diana Barran, existam crianças sendo
criadas por pais protetores que nunca tiveram um encontro para brincar? Os bebês Covid
crescerão como criaturas solitárias, acostumadas a se divertir ou mais cautelosas com os
estranhos que raramente encontram e com quem interagem? Tirar um pequeno pacote em
público costumava significar uma sucessão interminável de mulheres mais velhas aleatórias
arrulhando no carrinho ou estranhos fazendo caretas para entreter um bebê entediado na fila do
caixa.
Mas agora os transeuntes não se atrevem a chegar perto, e outros compradores estão escondidos
atrás de máscaras.
Esta semana, os parlamentares foram apresentados a algumas descobertas iniciais de um projeto
liderado pelo Movimento dos Primeiros 1001 Dias, uma aliança de instituições de caridade e
profissionais para os primeiros anos, acompanhando a vida de menores de dois anos crescendo
durante uma pandemia. Uma pesquisa com prestadores de serviços infantis que encomendou
descobriu que 98% achavam que os bebês e crianças com quem trabalhavam haviam sido
afetados por maior estresse e ansiedade dos pais, enquanto 92% viram famílias temerosas se
isolarem efetivamente do mundo exterior, pulando compromissos de rotina ou não querendo sair
de casa. Nove em cada 10 observaram crianças brincando com menos ou sendo menos ativas. De
forma dolorosa, mais de um quarto disse que o confinamento deixou as crianças com quem
trabalharam mais expostas a conflitos domésticos, abusos ou negligência.

Bill Gates: ‘Carbon neutrality in a decade is a fairytale. Why peddle fantasies?’


Bill Gates appears via video conference – Microsoft Teams, not Zoom, obviously – from his office in
Seattle, a large space with floor-to-ceiling windows overlooking Lake Washington. It’s a gloomy day
outside and Gates is, somewhat eccentrically, positioned a long way from the camera, behind a large,
kidney-shaped desk; his communications manager sits off to one side. “As a way to start,” says Gates’
aide, “would it be helpful for Bill to make a couple of comments about why he wrote his new book?”
Unlike the Elon Musks or Larry Ellisons of this world, however, Gates is perceived to be sensible,
uxorious, modest, vowing not to ruin his children with boundless inheritance or to waste energy trying
to send things to Mars. Gates’ new book, How To Avoid A Climate Disaster, grew out of two things: his
interest in the sciences and what struck him as an irresistible challenge – the fiendishly difficult
problem of how to further global development while reducing emissions.
There’s another, greater obstacle to reaching zero emissions, which is the political challenge – part of
which involves climate activists limiting their exposure to accusations of hypocrisy. Gates loves private
jets; he calls them his “guilty pleasure”.

AULA 04 – ADJECTIVES AND ADVERBS 149


TEACHER ANDREA BELO

The depressing part of the book is its account of the challenge ahead, which Gates presents as
extremely urgent – He points to a headline figure: 51bn. This is the amount of greenhouses gas, in
tons, emitted globally each year, which we have to get down to net zero by 2050. The first step towards
this is understanding what we’re dealing with.
Adapted from https://www.theguardian.com/technology/2021/feb/15/bill-gates-carbon-neutrality-
in-a-decade-is-afairytale-why-peddle-fantasies

Bill Gates: ‘A neutralidade do carbono em uma década é um conto de fadas. Por que vender
fantasias?'
Bill Gates aparece por meio de videoconferência - Microsoft Teams, não Zoom, obviamente - de seu
escritório em Seattle, um grande espaço com janelas do chão ao teto com vista para o Lago
Washington. É um dia sombrio lá fora e Gates está, um tanto excentricamente, posicionado bem longe
da câmera, atrás de uma grande mesa em formato de rim; seu gerente de comunicações fica de lado.
“Para começar”, diz o assessor de Gates, “seria útil que Bill fizesse alguns comentários sobre por que
escreveu seu novo livro?”
Ao contrário dos Elon Musks ou Larry Ellisons deste mundo, no entanto, Gates é visto como sensato,
uxório, modesto, jurando não arruinar seus filhos com herança ilimitada ou desperdiçar energia
tentando enviar coisas a Marte. O novo livro de Gates, How To Avoid A Climate Disaster, surgiu de
duas coisas: seu interesse pelas ciências e o que o considerou um desafio irresistível - o problema
terrivelmente difícil de como promover o desenvolvimento global e, ao mesmo tempo, reduzir as
emissões.
Há outro obstáculo maior para chegar a emissões zero, que é o desafio político - parte do qual envolve
ativistas climáticos que limitam sua exposição a acusações de hipocrisia. Gates adora jatos
particulares; ele os chama de “prazer culpado”.
A parte deprimente do livro é seu relato do desafio à frente, que Gates apresenta como extremamente
urgente - ele aponta para um número manchete: 51 bilhões. Esta é a quantidade de gases de efeito
estufa, em toneladas, emitida globalmente a cada ano, que temos que reduzir para zero líquido até
2050. O primeiro passo para isso é entender com o que estamos lidando.

How a Spanish town pioneered dolls with Down's syndrome


The first time Kelle Hampton glimpsed a doll with Down’s syndrome, anger boiled up inside her.
Its exaggerated features bore little resemblance to the sweet facial characteristics that she loved
about her daughter Nella, who was born with the genetic disorder.
The experience set the US blogger and author firmly against such dolls. But to her surprise, years
later she found herself smitten with another doll. This time it had been carefully crafted to subtly
capture the characteristics that made Nella unique. “This one was simply a beautiful doll any child
would want to play with,” she said.
The doll hailed from a small town in eastern Spain that was put on the map after enterprising
potters began transforming clay from the nearby mountains into children’s dolls, giving rise to
what has been described as the country’s first doll factory. More than a century later the spotlight
is again on the Valencian town of Onil – this time for its singular combination of inclusivity and
artisan dollmaking.

AULA 04 – ADJECTIVES AND ADVERBS 150


TEACHER ANDREA BELO

The town, home to around 7,500 people and 38 toymakers, made headlines across Spain last year
after a local collection of four dolls with Down’s syndrome won the country’s much-coveted “toy
of the year” prize.
The dolls – two boys and two girls of varying skin colours – were rolled out by toymaker Miniland.
“We were apprehensive at the beginning,” said Victoria Orruño, the company’s marketing director.
“But the reaction surprised us. It was very positive.”
Adapted from https://www.theguardian.com/society/2021/feb/15/dolls-and-diversity-how-toys-pass-on-the-power-of-being-see

Como uma cidade espanhola foi pioneira em bonecos com síndrome de Down
A primeira vez que Kelle Hampton viu uma boneca com síndrome de Down, a raiva ferveu dentro
dela. Seus traços exagerados tinham pouca semelhança com as doces características faciais que
ela amava em sua filha Nella, que nasceu com o distúrbio genético.
A experiência colocou o blogueiro e o autor norte-americano firmemente contra essas bonecas.
Mas, para sua surpresa, anos depois ela se viu apaixonada por outra boneca. Desta vez, ele foi
cuidadosamente elaborado para capturar sutilmente as características que tornavam Nella única.
“Esta era simplesmente uma linda boneca com a qual qualquer criança gostaria de brincar”, disse
ela.
A boneca veio de uma pequena cidade no leste da Espanha que foi colocada no mapa depois que
oleiros empreendedores começaram a transformar a argila das montanhas próximas em bonecas
infantis, dando origem ao que foi descrito como a primeira fábrica de bonecas do país. Mais de
um século depois, os holofotes estão novamente na cidade valenciana de Onil - desta vez por sua
combinação singular de inclusão e fabricação artesanal de bonecos.
A cidade, que abriga cerca de 7.500 pessoas e 38 fabricantes de brinquedos, ganhou as manchetes
em toda a Espanha no ano passado depois que uma coleção local de quatro bonecos com
síndrome de Down ganhou o cobiçado prêmio de "brinquedo do ano" do país.
Os bonecos - dois meninos e duas meninas de várias cores de pele - foram desenrolados pelo
fabricante de brinquedos Miniland. “Estávamos apreensivos no início”, disse Victoria Orruño,
diretora de marketing da empresa. “Mas a reação nos surpreendeu. Foi muito positivo.”

Professor lectures for two hours online – on mute


A university professor in Singapore gave a two-hour online lecture but didn't realize he was on
mute. Professor Wang, who teaches maths, was not aware that throughout his online
presentation, the microphone on his computer was switched off. This meant that none of the
students attending his online class heard what professor Wang was talking about. His university
switched his classes online to help reduce the spread of COVID-19. His lecture started well but
then it froze. The video came back but the mic was off and professor Wang did not notice. One of
his students says it might have been because the professor was using an iPad and not a computer.
Students tried many times to contact professor Wang during the lesson but could not get through
to him. They waved their arms and even tried calling him on his personal phone. A video of the

AULA 04 – ADJECTIVES AND ADVERBS 151


TEACHER ANDREA BELO

moment he understood his mic was off has gone viral on the Internet. It has received more than
653,000 views. After realizing what happened, professor Wang appeared shocked and stressed.
He said he was upset at the thought of wasting two hours of his students' time. He has offered to
redo the lecture at a different time so his students do not miss that class. One of his students
posted on social media about the professor's mishap. She wrote: “I took a class under him before
and he teaches well. I feel bad for him.”
Adapted from https://www.dailymail.co.uk/news/article-9247009/Maths-professor-Singapore-delivers-two-hour-lecture-Zoom-realisemute.html.

O professor dá palestras por duas horas online – no modo mudo


Um professor universitário em Cingapura deu uma palestra online de duas horas, mas não
percebeu que estava no mudo. O professor Wang, que ensina matemática, não sabia que, durante
sua apresentação online, o microfone de seu computador estava desligado. Isso significa que
nenhum dos alunos de sua aula online ouviu o que o professor Wang estava falando. Sua
universidade mudou suas aulas online para ajudar a reduzir a disseminação do COVID-19. Sua
palestra começou bem, mas depois congelou. O vídeo voltou, mas o microfone estava desligado e
o professor Wang não percebeu. Um de seus alunos diz que pode ter sido porque o professor
estava usando um iPad e não um computador.
Os alunos tentaram várias vezes entrar em contato com o professor Wang durante a aula, mas não
conseguiam falar com ele. Eles acenaram com os braços e até tentaram ligar para ele em seu
telefone pessoal. Um vídeo do momento em que ele percebeu que seu microfone estava desligado
se tornou viral na Internet. Recebeu mais de 653.000 visualizações. Depois de perceber o que
aconteceu, o professor Wang pareceu chocado e estressado. Ele disse que estava chateado com a
ideia de perder duas horas do tempo de seus alunos. Ele se ofereceu para refazer a palestra em
um horário diferente para que seus alunos não perdessem aquela aula. Um de seus alunos postou
nas redes sociais sobre o acidente do professor. Ela escreveu: “Eu tive aulas com ele antes e ele
ensina bem. Eu me sinto mal por ele."

AULA 04 – ADJECTIVES AND ADVERBS 152


AFA 2024

PRONOUNS AND PREPOSITIONS

AULA 05

Teacher Andrea Belo

www.estrategiamilitares.com.br www.militares.estrategia.com
TEACHER ANDREA BELO

SUMÁRIO
INTRODUÇÃO 3

PRONOUNS 4

PREPOSITIONS 18

QUESTÕES 23

GABARITO 57

QUESTÕES COMENTADAS 58

CONSIDERAÇÕES FINAIS 122

REFERÊNCIAS BIBLIOGRÁFICAS 123

TRADUÇÕES 125

AULA 05 – PRONOUNS AND PREPOSITIONS 2


TEACHER ANDREA BELO

INTRODUÇÃO
Vamos, então, à nossa aula sobre outros importantes tópicos gramaticais: os pronomes e
suas classes; os determinantes e suas propriedades; as preposições e suas variedades e,
também, as conjunções e suas famosas orações coordenadas e subordinadas, entre outras.
Na língua inglesa, existem muitas classes gramaticais, as quais estamos estudando no
decorrer das nossas aulas. Mas, a que substitui e determina os substantivos é, de fato, a classe
dos pronomes (pronouns). E, além de acompanhar os substantivos, define-lhes os limites de
significação. Veremos muitos tipos de pronomes nessa aula.
Quantifiers – os determinantes, são expressões usadas para indicar informações a respeito
da quantidade de algo. Veremos muitos tipos deles.
Prepositions – as famosas preposições – as milhares de preposições que existem em Inglês,
são palavras que ligam termos e estabelece relação entre eles. Vamos estudar todas as possíveis
preposições que podem aparecer nas provas.
As Conjunctions – conjunções – são palavras que ligam duas palavras, ligando, assim, duas
frases, das quais veremos muitas e inúmeros exemplos para que fique claro o uso delas nas
abundantes orações em que aparecem.
As línguas, de forma geral, são sistemas que devemos internalizar naturalmente, mas,
como o conhecimento da língua consiste em uma fragmentação de conteúdos em regras (e essas
cheias de exceções), fica complicado aprender e usar bem todos esses tópicos gramaticais, todo
esse conteúdo proposto em nossas aulas, eu sei disso.
Até porque uma língua é parte integrante de uma cultura e de suas relações sociais e, como
brasileiro, você estuda a língua estrangeira como um elemento externo, para capacitar você a
conseguir ler e responder as questões no dia da sua prova.
Mas, se seu objetivo é ser aprovado, imagine que por trás de cada texto, há formas de
desvendar os elementos ali presentes através das análises linguísticas e das técnicas, que tanto
explico e ensino você, como ajuda para sua aprovação.
Entender todos os itens das aulas, dia após dia, é abrir uma porta, aliás, abrir portas para
um mundo novo: o conhecimento e propriedade para utilizar no dia da sua prova.
E você vai conseguir. Vou fornecer ferramentas específicas para simplificar o estudo de
pronomes, de determinantes, de preposições, de conjunções, assim como de todos os assuntos
que abrangem as melhores instituições do Brasil, e, uma vez preparado para uma dessas provas,
da maneira como estou explicando e contextualizando para você, o caminho para a aprovação
ficará menor de ser percorrido.
Vamos lá! Você consegue e será o melhor candidato!

AULA 05 – PRONOUNS AND PREPOSITIONS 3


TEACHER ANDREA BELO

PRONOUNS
Os prounouns – pronomes – são palavras que acompanham os substantivos e podem
substituí-los (direta ou indiretamente), retomá-los ou se referir a eles.
Estudaremos todos os tipos de pronomes, tais como: personal, possessive, subject, object,
adjective, reflexive, demonstrative, indefinite e interrogative pronouns. Vou explicar todos eles
usando a metodologia que eu desenvolvi, especialmente para o curso do Estratégia Militares, uma
maneira facilitadora de compreender aquilo que a gramática complica.
Os pronomes servem para remeter, retomar ou qualificar palavras do texto. Sem os
pronomes, as orações ficariam confusas e longas demais, como por exemplo, ao citar os nomes
de 3 pessoas em uma frase, podemos usar o pronome sujeito they (eles/elas), remetendo àquelas
pessoas, veja: Elizabeth, Lourdes and Alexander play golf together. They play golf together.
Os pronomes em Inglês, de acordo com a função que exercem, são classificados em:
 Pronomes pessoais (personal pronouns: subject e object pronouns)
 Pronomes possessivos (possessive pronouns: possessive adjectives and possessive
prounouns)
 Pronomes demonstrativos (demonstrative pronouns)
 Pronomes reflexivos (reflexive pronouns)
 Pronomes indefinidos (indefinite pronouns)
 Pronomes relativos (relative pronouns)
 Pronomes interrogativos (interrogative pronouns)

PERSONAL PRONOUNS
Os personal pronouns – pronomes pessoais, podem indicar pessoas, lugares e objetos. São
classificados em:
 Subject Pronouns: os pronomes que funcionam como sujeitos, como por exemplo:

Lavinia is beautiful – She is beautiful.

noun subject pronoun

 Object Pronouns: os pronomes que funcionam como objetos, por exemplo:

I want to go to the park with Lavinia – I want to go to the park with her.

Noun object pronoun

AULA 05 – PRONOUNS AND PREPOSITIONS 4


TEACHER ANDREA BELO

Com meu método de explicação, o resumo é o seguinte:


Os pronomes sujeitos, subject prounouns – vêm no início das frases, em sua maioria. Isso
porque, o sujeito de uma frase é a pessoa que realiza a ação, certo?
Pode até ser que o subject pronoun apareça no meio de uma frase mas têm a função de
sujeito, de quem realiza a ação, como por exemplo:

He works at the office every day Every day, he works at the office.
(Ele trabalha no escritório todos os dias.) (Todos os dias, ele trabalha no escritório.)

Já os object pronouns sempre aparecem no fim das frases, na função de objeto mesmo,
que seriam os objetos direto e indireto, mas isso não precisa ser lembrado para entender e
aprender os pronomes que estamos estudando.
O que você precisa saber (e que nenhum livro diz) é que os pronomes sujeitos e objetos
têm o mesmo significado, porém colocados em lugares diferentes da frase. Se eu falar “Ela
estuda” é “She studies” mas, “Eu estudo com ela”, com a palavra ela no final da frase, ela vira um
pronome objeto e fica: “I study with her”, entendeu o bizu?
Como esses pronomes estão correlacionados, fiz uma tabela para você. Veja qual pronome
sujeito se refere a cada pronome objeto abaixo, veja os dois tipos de personal pronouns:
SUBJECT OBJECT
EXAMPLES
PRONOUNS PRONOUNS
I ME I am the person. It’s me. (Eu sou a pessoa. Sou eu).
YOU YOU You are the person. It’s you. (Você é a pessoa. É você).
HE HIM He is the person. It’s him. (Ele é a pessoa. É ele).
SHE HER She is the person. It’s her. (Ela é a pessoa. É ela).
IT (neutro) IT (neutro) It is the object. It’s it. (Ele é o objeto. É ele).
WE US We are the people. It’s us. (Nós somos as pessoas. Somos nós).
YOU YOU You are the person. It’s you. (Vocês são as pessoas. São vocês).
THEY THEM They are the people. It’s them. (Eles são as pessoas. São eles).

POSSESSIVE PRONOUNS
Os pronomes a serem apresentados, em cada subcapítulo do nosso material, serão
explicados de forma prática e compreensível, com exemplos e quadros ilustrativos, propiciando
uma melhor compreensão. Vamos lá.
Sobre os possessive pronouns, são aqueles que indicam que algo pertence a alguém. São
classificados em possessive adjectives e possessive pronouns (pronomes adjetivos e pronomes
substantivos).
Veremos cada um deles separadamente, para que você possa diferenciá-los e encontrá-los
na prova, caso seja solicitado uma substituição ou análise de frases com eles.

AULA 05 – PRONOUNS AND PREPOSITIONS 5


TEACHER ANDREA BELO

Possessive Adjectives: Esses pronomes são sempre são acompanhados por substantivos,
modificando-os. E eles não têm singular e plural, por exemplo: my significa meu, minha, meus e
minhas, assim como your significa seu, sua, seus, suas e assim por diante, veja:

Before leaving, I got my wallet, my keys, my laptop and my books.


(Antes de sair, eu peguei minha carteira, minhas chaves, meu laptop e meus livros.)

POSSESSIVE
EXAMPLES
ADJECTIVES
MY My car is black. (Meu carro é preto).
YOUR Where is your car? (Onde está o seu carro?)
HIS His car is black. (O carro dele é preto).
HER Where is her car? (Onde está o carro dela?)
ITS (netro) Where is its car? (Onde está o carro dela (loja/concessionária?)
OUR Where is our car? (Onde está o nosso carro?)
YOUR Your car is black. (Seu carro é preto).
THEIR Their car is black. (O carro deles/delas é preto).

Possessive Pronouns: esses pronomes têm a função de substituir o substantivo,


diferentemente dos pronomes adjetivos, que sempre estão ligados a ele. Eles também não sofrem
flexão de grau - singular e plural.
O mais importante para você saber é que, esses pronomes sempre estarão no final das
frases, sempre! Assim fica mais fácil de encontrá-los nos textos das provas e também saber suas
funções corretas, já que são similares e não podem ser confundidos.
Na hora da sua prova, em sua análise das alternativas de escolha de cada pergunta, você
precisa usar as técnicas certas – inclusive aprendida em nossa primeira aula – para ler os textos
com maior agilidade e compreender termos que levarão à resolução com segurança.
Veja um exemplo e o quadro ilustrativo:

Whose umbrella is this? It’s mine. And the keys? They are mine.
(De quem é esse guarda-chuva? É meu. E as chaves? São minhas.)

POSSESSIVE
EXAMPLES
PRONOUNS
MINE Whose books are these? De quem são esses livros? They are mine. (São meus).
YOURS Whose books are these? De quem são esses livros? They are yours. (São seus).
HIS Whose books are these? De quem são esses livros? They are his. (São dele).
HERS Whose books are these? De quem são esses livros? They are hers. (São dela).
ITS (neutro) Whose books are these? De quem...livros? They are its. (São dela - loja/biblioteca).
OURS Whose books are these? De quem são esses livros? They are ours. (São nossos).
YOURS Whose books are these? De quem são esses livros? They are yours. (São seus).
THEIRS Whose books are these? De quem são esses livros? They are theirs. (São meus).

AULA 05 – PRONOUNS AND PREPOSITIONS 6


TEACHER ANDREA BELO

Agora, depois de tantos pronomes, vamos visualizá-los em textos de exercícios de muitas


provas. Vou mostrar os textos desses e de outroas instituições importantes e explicar a que se
referem, para comprovar nossos estudos. E logo, mais pronomes de outras categorias. Let’s go!

Christoph Oswald has no problem approaching women.


As he makes his way through the crowd at his favorite
Frankfurt club, his cell phone scans a 10-meter radius
for “his type”: tall, slim, sporty, in her 30s-and, most
important, looking for him, a handsome 36-year-old
software consultant who loves ski holidays. Before he
reaches the bar, his phone starts vibrating and an
attractive blonde appears on its screen. “Hi, I’m Susan,”
she says. “Come find me!” Christoph picks her out of
the crowd, and soon they’re laughing over a drink. (…)
(NEWSWEEK, JUNE 7 / JUNE 14, 2004)

Nesse trecho de um dos textos que vamos resolver a lista de exercícios adiante, sublinhei alguns
pronomes para você visualizar.
Veja que há o pronome - subject pronoun - HE (ele) - e, em seguida, o possessive adjective HIS,
que se refere à Christoph Oswald (HE – ele).
Veja também que há o pronome - subject pronoun - SHE - e o possessive adjective SHE, que se
refere à mulher (SHE – ela).
Como o texto se refere às duas pessoas da imagem, um homem e uma mulher, os pronomes
usados foram ele/ela e dele/dela.
Se a questão abordasse os pronomes sublinhados, poderia ser assim:
Questão – De acordo com o primeiro parágrafo,
(A) Ele faz o caminho dele até o seu clube favorito.
(B) Ele fez o caminho que a mulher explicou por telefone.
(C) Ele faz o caminho dela até o clube favorito dela.
(D) Ele fará o caminho dele até o seu clube favorito.
(E) Ele tem o próprio caminho que prefere até o seu clube favorito.
Comentários: A resposta seria a letra A porque é a única que demonstra o pronome correto e
exato para ele – dele.
Na frase “He makes his way...at his favorite...” significa Ele faz o caminho dele... até o clube
favorito dele..”, que muitas vezes, as pessoas traduzem como “seu clube favorito mas remete a
ele, o personagem homem.
Observe que as provas não trazem os nomes dos conceitos no enunciado – “Encontre o pronome
sujeito, identifique o pronome objeto, qual é o pronome...” entre outros.
Mas você deve conhecer os tópicos gramaticais para resolver as questões. Assim, identifica do
que se trata a questão e acerta as respostas com segurança.

AULA 05 – PRONOUNS AND PREPOSITIONS 7


TEACHER ANDREA BELO

Fonte: <http://www.cartoonistgroup.com/subject/The-Snert-Comics-and-Cartoons-by-Hagar+The+Horrible.php>. Acesso em: 23 mai. 2017.

Nessa charge, da prova ITA/2018, sobre os pronomes sublinhados, os dois primeiros são
possessive adjectives, your se refere à Helga (your husband – seu marido), uma frase sobre o
marido ser preguiçoso (lazy). Ela responde que o trabalho dele (his job) é difícil (hard).
Depois, um object pronoun: é dito que todos servem a ele (him), significando ele, como um
sujeito, lembra que expliquei isso? E o último é também um possessive adjective: ... a pilllow for
my back (um travesseiro para minhas costas).

COULD THIS CATERPILLAR HELP SOLVE THE WORLD’S PLASTIC BAG PROBLEM?
A developmental biologist and amateur beekeeper has come up with a new way to get rid of used
plastic bags: Make waxworms eat them. The larvae of the greater wax moth (Galleria mellonella),
these caterpillars thrive on beeswax. While cleaning out empty hive boxes that were infested with
these caterpillars, Federica Bertocchini of the Institute of Biomedicine and Biotechnology of Cantabria
in Spain put them in a plastic grocery bag. To her surprise the waxworms quickly ate their way out,
leaving the bag riddled with holes. It turns out the caterpillars can break down the bag's polyethylene
into ethylene glycol, which can be readily converted into useful substances such as antifreeze, the
team reports today in Current Biology. Polyethylene is very hard to break down making the 80 million
tons produced a year a big recycling challenge. Only recently have researchers begun to make
progress doing so, and this caterpillar may be another solution. PENNISI, Elizabeth. Adapted from:
Science. Could this caterpillar help solve the world’s plastic bag problem?
In: Science, 2017. Disponível em: <http://www.sciencemag.org/news/2017/04/could-caterpillar-help-solve-world-s-plastic-bagproblem.>. Acesso em: 26/06/2017.

AULA 05 – PRONOUNS AND PREPOSITIONS 8


TEACHER ANDREA BELO

Nesse texto, da prova IME/2018, sobre os pronomes sublinhados, os dois primeiros são object
pronouns, them e se referem, respectivamente às minhocas – waxworms: elas, pronome them e
às larvas – caterpillars, também elas, pronome them.
O terceiro, her, é um possessive adjective, que se refere à Federica Bertocchini, portanto,
dela. O último sublinhado – their – é outro possessive adjective, que se refere às waxworms,
portanto, delas. Você não precisa decorar esses nomes para o dia da prova. Porém, precisa saber
onde cada pronome se encaixa, entendê-los e identificar como são usados nas frases, é suficiente.
E agora, um quadro só, com os dois tipos de possessive pronouns, para ficar mais fácil
ainda de memorizar e comparar ambos pronomes. E, não se esqueça dos bizus de como
memorizá-los com mais facilidade. Vamos lá!

POSSESSIVE PRONOUNS AND POSSESSIVE ADJECTIVES


POSSESSIVE ADJECTIVE POSSESSIVE PRONOUN
MY BOOK (meu livro) MINE (o meu / a minha)
YOUR PENCIL (seu lápis) YOURS (o seu / a sua)
HIS BACKPACK (mochila dele) HIS (a dele / o dele)
HER DRESS (vestido dela) HERS (a dela / o dela)
ITS HOUSE (casa dele[a] - neutro e usado para animais) ITS (o dele / a dele / o dela / a dela - neutro e usado para animais
OUR TV (nossa televisão) OURS (o nosso / a nossa
YOUR CARS (seus carros) YOURS (os seus / as suas)
THEIR PENS (canetas deles[as]) THEIRS (os deles / as deles / os delas / as delas)

REFLEXIVE PRONOUNS
Os reflexive pronouns – pronomes reflexivos, são aqueles que praticam a ação sobre si
mesmo (eu mesmo, você mesmo etc) e aparecem após os verbos. São usados de acordo com o
sujeito da oração, sempre. São palavras que têm os sufixos “-self” (singular) e “-selves” (plural).
Os reflexive pronouns possuem 3 funções:
 Função reflexiva: é quando há concordância com o sujeito e aparece depois dele:
Lisa was looking herself in the mirror. (Lisa estava se olhando no espelho).

 Função enfática: é quando o pronome concorda com o sujeito e se posiciona depois


desse sujeito ou então do objeto.
Lisa herself prepared the sandwich. (Lisa mesma preparou o sanduíche).

 Função idiomática: é quando o pronome vem precedido da preposição by, para


indicar que o sujeito praticou a ação sozinho.
Lisa likes studying by herself. (Lisa gosta de estudar sozinha. – só ela.)

AULA 05 – PRONOUNS AND PREPOSITIONS 9


TEACHER ANDREA BELO

Veja um exemplo e o quadro ilustrativo:


REFLEXIVE
POSSESSIVE PRONOUN
PRONOUNS
I - MYSELF I cut myself with a knife. Eu me cortei com a faca.
YOU - YOURSELF You cut yourself with a knife. Você se cortou com a faca.
HE - HIMSELF He cut himself with a knife. Ele se cortou com a faca.
SHE - HERSELF She cut herself with a knife. Ela se cortou com a faca.
IT - ITSELF It cut itself with a knife. Ele se cortou com a faca.
WE - OURSELVES We cut ourselves with a knife. Nós nos cortamos com a faca.
YOU - YOURSELVES You cut yourselves with a knife. Vocês se cortaram com a faca.
THEY - THEMSELVES They cut themselves with a knife. Eles se cortaram com a faca.

Vejamos uma tabela dos pronomes que estudamos até agora, com as devidas
classificações.
PERSONAL OBJECT POSSESSIVE POSSESSIVE REFLEXIVE
PRONOUNS PRONOUNS PRONOUNS PRONOUNS PRONOUNS
I ME MY MINE MYSELF
YOU YOU YOUR YOURS YOURSELF
HE HIM HIS HIS HIMSELF
SHE HER HER HERS HERSELF
IT IT ITS ITS ITSELF
WE US OUR OURS OURSELVES
YOU YOU YOUR YOURS YOURSELVES
THEY THEM THEIR THEIRS THEMSELVES

Aconselho que você tente escrever exemplos para cada pronome acima com o intuito de
treinar. E, quando aparecerem pronomes nas questões, sublinhe, circule, encontre uma forma de
destacá-los e descobrir que pronome é para aprimorar os conhecimentos.
Já estudamos muitos pronomes: os pessoais, possessivos, e reflexivos. Ainda faltam os
pronomes demonstrativos, indefinidos, relativos e interrogativos.
Você já percebeu que os pronomes, em geral, são aquelas palavras que substituem ou
acompanham outras, principalmente os substantivos, certo?
Afinal, se analisarmos a própria palavra pronome, “pro” significa “em função de”. Logo, é
possível concluir que “pronome” é sinônimo de algo que está “em função do nome”.
Por isso, os pronomes existem para remeter, retomar ou qualificar outras palavras
expressas nos textos das provas.
E assim, se torna tão importante identificá-los, para ter a certeza de que cada alternativa
faz sentindo, analisando cada termo que faz parte dos textos.
Mas ainda há muito o que estudar. Preparado? Então, vamos aos pronomes
demonstrativos.

AULA 05 – PRONOUNS AND PREPOSITIONS 10


TEACHER ANDREA BELO

DEMONSTRATIVE PRONOUNS
Os demonstrative pronouns – pronomes demonstrativos, são apenas 4 e são simples de
entender. São usados para situar a posição de algo no espaço em que se encontra.
São aqueles pronomes que mostram, que indicam algo que pode estar perto ou longe. Veja
uma tabela e uma ilustração com esses pronomes.
DEMONSTRATIVE
(Em Português)
PRONOUNS
THIS este, esta, isso (singular)
THESE estes, estas (plural)
THAT aquele, aquela, aquilo (singular)
THOSE aqueles, aquelas (plural)

Pela simplicidade, os pronomes demonstrativos podem ser reconhecidos nos textos em


Inglês e assim, facilitar na busca pelas respostas das questões que lá estiverem. Vamos estudar os
pronomes indefinidos e dar continuidade ao seu aprendizado.

INDEFINITE PRONOUNS
Os indefinite pronouns – pronomes indefinidos, recebem esse nome porque substituem ou
acompanham o substantivo, porém, de forma indeterminada, como?
Eles oferecem a ideia, por exemplo, de algum lugar, alguma coisa, alguém, qualquer lugar,
qualquer pessoa, entre outros nomes imprecisos dentro das orações.

AULA 05 – PRONOUNS AND PREPOSITIONS 11


TEACHER ANDREA BELO

Os pronomes indefinidos começam com as palavras some, any, no e every. E terminam com
os sufixos -body, -one, -thing, -where, entre outros, a depender do contexto.
Em geral, usa-se -some, -every e -no em frases afirmativas, -any em frases negativas e
interrogativas.
Aqui, vale lembrar algumas diferenças entre some e any, só para ilustrar: some para frases
afirmativas e any para negativas e interrogativas, ambas palavras significam algum, alguma,
alguns, algumas e, dependendo do contexto, nas negativas, significam nenhum/nenhuma, ok?
Vejamos alguns exemplos para ilustrar:
Do you have any money? Do you play any sports?
Yes, I have some money. Yes, I play some sports.
No, I don’t have any money. No, I don’t play any sports.

São muitos os pronomes indefinidos e, por isso, precisamos de analisar diferentes


exemplos que possam estar nos textos no dia da sua prova.
Vou mostrar variados exemplos abaixo, nos quadros que seguem:
SOMEBODY alguém I think somebody has arrived. Eu acho que alguém chegou.
SOMEONE alguém I believe someone forgot the book. Eu acredito que alguém esqueceu o livro.
SOMETHING algo She said something is wrong. Ela disse que alguma coisa está errada.
SOMEWHERE em algum lugar He is somewhere in Europe. Ele está em algum lugar na Europa.
SOMEWAY de algum maneira I have to get there someway. Eu tenho que chegar lá de alguma maneira.

ANYBODY ninguém I can’t see anybody in the room. Não consigo ver ninguém na sala.
ANYONE qualquer um, ninguém He didn’t see anyone there. Ele não viu ninguém lá.
ANYTHING nada I won’t do anything at the moment. Não consigo fazer nada nesse momento.
ANYWHERE qualquer lugar I can’t go anywhere. Não posso ir em lugar nenhum/qualquer lugar.
ANYWAY de qualquer forma, jeito She is going home anyway. Ela está indo para casa de qualquer forma/jeito.

NOBODY ninguém She said nobody won the medal. Ela disse que ninguém ganhou a medalha.
NO ONE ninguém No one came to college yesterday. Ninguém veio à faculdade ontem.
NONE nenhum(a) None of the exercises are right. Nenhum dos exercícios está certo.
NOTHING nada There is nothing I can do. Não há nada que eu possa fazer.
NOWHERE em nenhum lugar She is going nowhere. Ela não está indo para lugar nenhum.
NOWAY de jeito nenhum I will not it this. No way!" (Eu não vou comer isso. De jeito nenhum!)

EVERYBODY toda a gente, todo mundo Everybody is going to the party. Todos vão para a festa.
EVERYONE todos, todo mundo I talked to everyone at the party. Eu falei com todos da festa.
EVERYTHING tudo She is doing everything she can. Ela está fazendo tudo o que pode.
EVERYWHERE em todos os lugares I’ve been everywhere in this city. Já estive em todos os lugares dessa cidade.
EVERY WAY de todo jeito, todos os sentidos He drove every way at the race. Ele dirigiu em todos os sentidos na corrida.

AULA 05 – PRONOUNS AND PREPOSITIONS 12


TEACHER ANDREA BELO

Os pronomes vistos acima, com algum, alguma,


os respectivos exemplos, são muito ONE a gente, certo, SUCH tal, tais
cobrados nas provas. E, podem um certo
tudo, todo(s),
aparecer outros, que estão no quadro ALL
toda(s)
OTHER outro(s), outra(s)
aqui ao lado para você, já com os MUCH muito(a) ANOTHER outro(a)
significados. MANY muitos(as) EITHER
um ou outro, uma
ou outra, cada
Agora, a parte mais importante: nem um(a) nem
o exemplo dos pronomes estudados LITTLE pouco(a) NEITHER
outro(a),
dentro dos textos, tanto na prova do nenhum(a) dos(as)
dois(duas)
ITA quanto do IME, não é mesmo? E FEW poucos(as) BOTH ambos(as)
também Fuvest e Unesp, entre várias FEWER menos ENOUGH bastante, suficiente
outras no Brasil inteiro. EACH cada SEVERAL vários(as)

Assim, você pode visualizar de fato, em que situações eles aparecem e com dar sentido a
eles na hora em que você estiver lendo os textos da prova.

"If we lived on a planet where nothing ever changed, there would be little to do. There
would be nothing to figure out. There would be no impetus for science. And if we lived in an
unpredictable world, where things changed in random or very complex ways, we would not be
able to figure things out. ___________________________. If I throw a stick up in the air, it always
falls down. If the sun sets in the west, it always rises again the next morning in the east. And so it
becomes possible to figure things out. We can do science, and with it we can improve our lives."
Carl Sagan, http://todayinsci.com/S/Sagan_Carl/SaganCarl-Quotations.htm Acessado em 14 de Abril de 2015.

Nesse texto, explorado na prova do IME/2016, os pronomes indefinidos, como eu disse


antes, representam quantidades indefinidas.
Veja os primeiros pronomes sublinhados – nothing, que se refere ao verbo change (mudar),
afirmando que, se nada mudasse (nothing ever changed), haveria pouco a se fazer (little to do).
Perceba as quantidades indefinidas: nada e pouco.
Depois, nothing de novo, para afirmar que não haveria nada para resolver (nothing to
figure out) e, em seguida, diz que não haveria nenhum ímpeto para a ciência (no impetus for
science).

Let me tell you a bit about myself. I’m 35 years old, male, single, never been married. I work
as an editor at a publishing company. I recently moved from the Nakameguro neighbourhood in
Tokyo, where I lived for a decade, to a neighbourhood called Fudomae in a different part of town.
The rent is cheaper, but the move pretty much wiped out my savings.
Some of you may think that I’m a loser: an unmarried adult with not much money. The old
me would have been way too embarrassed to admit all this. I was filled with useless pride. But I
honestly don’t care about things like that anymore. The reason is very simple: I’m perfectly happy
just as I am. The reason? I got rid of most of my material possessions.

AULA 05 – PRONOUNS AND PREPOSITIONS 13


TEACHER ANDREA BELO

Minimalism is a lifestyle in which you reduce your possessions to the least possible. Living
with only the bare essentials has not only provided superficial benefits such as the pleasure of a
tidy room or the simple ease of cleaning, it has also led to a more fundamental shift. It’s given me
a chance to think about what it really means to be happy.
We think that the more we have, the happier we will be. We never know what tomorrow
might bring, so we collect and save as much as we can. This means we need a lot of money, so we
gradually start judging people by how much money they have. You convince yourself that you need
to make a lot of money so you don’t miss out on success. And for you to make money, you need
everyone else to spend their money. And so it goes.
So I said goodbye to a lot of things, many of which I’d had for years. And yet now I live each
day with a happier spirit. I feel more content now than I ever did in the past.

No trecho apresentado acima, retirado de um texto que foi explorado na prova do


ITA/2018, o pronome indefinido much aparece muitas vezes, representando a quantidade
“muito(a)”:

Nas linhas 3 e 4, temos a frase “The rent is cheaper, but the move pretty much…” (muito)
e, na frase do parágrafo seguinte, “an unmarried adult with not much money (não tanto dinheiro),
mais adiante temos outra frase em que aparece much, na linha 14: “.. we collect and save as much
as we can (o máximo que pudermos) e, por último, na linha 15: “people by how much money we
have (quanto de dinheiro nós temos).
Também aparecem os pronomes some e any, nas linhas 5, 6 e 7: “Some of you...” (alguns
de vocês) e “... about things like that any more” (não mais), se referindo à negativa “don’t care”,
que aparece anteriormente (como eu disse antes: any em frases negativas, viu?)
E ainda temos outros pronomes, que fazem parte do quadro que apresentei a você
anteriormente. No texto, encontramos: “such” (tal/tais) e “each” (cada), que também
demonstram características dos pronomes indefinidos: a ideia de quantidade.
Na linha 16, há o pronome everyone (todos) e, eu apenas retirei um trecho do texto do
ITA/2018. Imagina se fôssemos analisar todo o texto, ou melhor, todos os textos da prova!
E não devemos nos esquecer de que há perguntas sobre o sentido da frase de acordo com
o contexto, interpretação, substituição, entre outras, em que os pronomes estão ali, no meio do
vocabulário.
Uma vez dominado essas palavras, a compreensão e busca pelas respostas fica mais
acessível, não é mesmo? Por isso, eu sempre gosto de enfatizar a importância de se conhecer os
termos gramaticais, sejam eles pronomes, verbos, preposições, artigos, ou melhor, como seria
bom conhecer e dominar todos eles para realizar uma excelente prova. E, basta estudar! Conte
comigo.
Em continuidade à aula, vamos estudar os pronomes relativos.

AULA 05 – PRONOUNS AND PREPOSITIONS 14


TEACHER ANDREA BELO

RELATIVE PRONOUNS
Os relative pronouns – pronomes relativos, são palavras que exercem a função de sujeito
ou de objeto nas frases.
Quando for sujeito, haverá um substantivo antes desse pronome. Mas, quando aparecer
após um verbo, com ou sem preposição, então, estamos falando de um pronome relativo com
função de objeto.
Em ambas situações, o que importa saber é a estrutura e entender para que e onde são
usados tais pronomes nos textos, independente da função que exercem.
Vejamos um quadro com os nomes dos pronomes e vamos falar dos detalhes de cada um
deles para compreender melhor.

PARA PESSOAS WHO WHO ou WHOM


PARA COISAS WHICH WHICH
PARA PESSOAS OU COISAS THAT THAT

Na função de sujeito, os pronomes relativos são: who (para pessoas), which (para
objetos/animais) e that (para pessoas e objetos: neutro).

The man who arrived is charming. (O homem que chegou é charmoso).

O pronome se refere ao homem que chegou: o sujeito

Na função de objeto, temos os pronomes relativos who/whom (para pessoas) e também


which (para objetos/animais) e that (para pessoas e objetos: neutro).

She didn’t say who arrived. (Ela não disse quem chegou).

O pronome se refere ao homem que chegou (objeto) e não a ela (sujeito)

Vejamos outros exemplos com o uso de diferentes pronomes relativos

The girl who disappeared studied with me. (A garota que desapareceu estudava comigo).
O pronome se refere à garota que desapareceu: o sujeito

They didn’t show who the magician was. (Eles não mostraram quem era o mágico).

O pronome se refere ao mágico que chegou: o sujeito

AULA 05 – PRONOUNS AND PREPOSITIONS 15


TEACHER ANDREA BELO

Veja o quadro ilustrativo:


RELATIVE
Tradução
PRONOUNS
WHO quem, o qual
WHOSE cujo, cuja, cujos, cujas
WHICH que, o qual, o que
WHERE onde, em que, no que, no qual, na qual, nos quais, nas quais
WHEN quando, em que, no qual, na qual, nos quais, nas quais
THAT que
WHAT o que
Os pronomes vistos acima, com os respectivos exemplos, também são muito cobrados nas
provas.
E, independente da maneira como esses pronomes vierem nos textos, se você entender o
funcionamento de cada um deles dentro do contexto, saberá responder as questões da prova.
Assim, você pode visualizar de fato, em que situações eles aparecem e com dar sentido a
eles na hora em que você estiver lendo os textos da prova.

INTERROGATIVE PRONOUNS
Os interrogative pronouns – pronomes interrogativos, também chamados de Question
Words, são aquelas palavras utilizadas para fazer perguntas, termos que representam aquilo que
queremos saber: o lugar, o motivo, a hora, o dia, entre outros.
Vejamos alguns exemplos:

What is your favorite fruit? Grapes. (Qual é a sua fruta favorita?) Uvas.

Which fruit do you prefer, apple or pear? Pear. (Qual fruta você prefere, maçã ou pera?) Pera.

Where do you go on the weekends? To the club. (Onde você vai nos fins de semana?) Ao clube.

When is Christmas? It’s on December 24th. (Quando é o Natal? É 24 de dezembro).

How do you go to work? By bus. (Como você vai para o trabalho?) De ônibus.

Who is that boy over there? It’s my brother. (Quem é aquele garoto lá?) É meu irmão.

To whom did he give the present? To me. (Para quem ele deu o presente?) Para mim.

AULA 05 – PRONOUNS AND PREPOSITIONS 16


TEACHER ANDREA BELO

Whose house is that? It’s mine. (De quem é aquela casa?) Minha.

Why do you study English? Because I need. (Por que você estuda alemão?) Porque eu preciso.

Observação: Você percebeu que usamos dois pronomes interrogativos para fazer perguntas com
a palavra qual – WHAT/WHICH – sendo What para perguntas gerais “O quê/Qual” mas, se for uma
escolha – “Qual você prefere, esse ou aquele?”, usamos “Which”.
‘What’ e ‘Which’, apesar de terem a mesma tradução, o momento de uso é diferente para
cada um deles, já que ‘what’ não apresenta restrições como o pronome interrogativo ‘which’ é
usado somente quando há um número limitado de opções.
Vejamos o quadro ilustrativo:
INTERROGATIVE
Tradução
PRONOUNS
WHAT O que, Que
WHICH Qual, Quais
WHERE Onde
WHEN Quando
HOW Como
WHO Quem (função de sujeito)
WHOM Quem (função de objeto)
WHOSE De quem
WHY Por que

Os pronomes interrogativos, nos textos, são usados quando há perguntas dentro de um


contexto e, na maioria das vezes, a resposta logo em seguida.
Ou, às vezes, são leituras mais técnicas, reflexivas, artigos científicos, entre outros, em que
há perguntas sem resposta dentro dos textos. E precisam de respostas mais amplas do que um
“sim” ou um “não”. Por isso aparecem nas alternativas de resposta algumas opções como um
lugar ou horário e, se você souber qual é o pronome, acerta a questão.
Veremos questões com esses pronomes adiante para praticar.

AULA 05 – PRONOUNS AND PREPOSITIONS 17


TEACHER ANDREA BELO

PREPOSITIONS
As prepositions (preposições) são palavras ou grupos de palavras que ligam e estabelecem
relações dentro das frases.
Nessas relações, um termo explica ou completa o sentido do outro. Vejamos as
preposições mais importantes e mais usadas nas provas: preposições de lugar e de tempo.
Vou mostrar uma imagem com as preposições principais e comentar sobre cada uma delas
em seguida. Isso porque, algumas preposições podem ser de lugar e de tempo, dependendo do
contexto. E vamos analisar para que você entenda bem. Vamos lá.

PREPOSITIONS OF PLACE

 In - dependendo do contexto pode significar: dentro de; em; de; no e na.


 On - dependendo do contexto pode significar: sobre a; em cima de; acima de; em; no; na.
 At - dependendo do contexto pode significar: à; em; na; no.
 To - dependendo do contexto pode significar: para; a.
 For - dependendo do contexto pode significar: para; durante; por.

IN

AULA 05 – PRONOUNS AND PREPOSITIONS 18


TEACHER ANDREA BELO

A preposição in é utilizada nos seguintes casos:


1) Para indicar tempo, seja o ano, o mês, as estações do ano ou uma parte do dia.
Exemplos:
 I study in the evening. (Eu estudo a noite.)
 He plays volleyball in the afternoon. (Ele joga vôlei de tarde.)
 My birthday is in April. (Meu aniversário dela é em abril.)
 My brother was born in 2012. (Meu irmão nasceu em 2012.)
 We always visit New York in the summer. (Nós sempre visitamos Nova Iorque no verão).
Nessa regra existe uma exceção em relação ao termo “night”. Nesse caso, a preposição utilizada
é o “at”, por exemplo: at night (à noite – madrugada, após dormir).

2) Para indicar lugar, seja uma cidade, um país ou qualquer local específico.
Exemplos:
 She lives in Italy. (Ela mora na Itália.)
 He works in São Paulo. (Ele trabalha em São Paulo.)
 Some people like to have a TV in the bedroom. (Algumas pessoas gostam de ter uma TV
no quarto.)
 They left the dog in the house. (Eles deixaram o cachorro na casa.)
 He is waiting for me in the living room. (Ele está me esperando na sala.)

ON
A preposição on é utilizada nos seguintes casos:
1) Para indicar tempo. No entanto, diferentemente do “in” ela é usada para datas específicas.
Exemplos:
 I was born on March 10th. (Eu nasci no em 10 de março.)
 He studies English on Tuesdays and Thursdays (Ele estuda Inglês às terças e quintas.)
 Joanne bought a new car on November 17th. (Joanne comprou um carro novo dia 17 de
novembro.)
 They always go out on Saturdays. (Eles sempre saem aos sábados.)
 I will travel on December 18th. (Vou viajar dia 18 de dezembro.)
 She will go to the park on Sunday. (Ela vai ao parque no domingo.)
2) Para indicar lugares e objetos. Todavia, diferentemente do “in” ela é usada para locais e objetos
que possuem uma superfície. Nesse caso, ela significa em cima de.

AULA 05 – PRONOUNS AND PREPOSITIONS 19


TEACHER ANDREA BELO

Exemplos:
 The book is on the bed. (O livro está em cima da cama).
 The cushion is on the floor. (A almofada está no chão.)
 My dog sleeps on the couch. (Meu cachorro dorme em cima do/no sofá).
 I will put the paper on my desk. (Vou colocar o papel em cima da/na mesinha.)
 She left her blouse on that chair. (Ela deixou a blusa dela em cima daquela/naquela
cadeira.)

3) Para referir meios eletrônicos onde informações são disponibilizadas.


Exemplos:
 I checked that information on the company’s website. (Eu chequei aquela informação no
site da empresa.)
 Don’t believe everything you see on the Internet. (Não acredite em tudo que você lê na
Internet.)
 The principal can’t talk to you now because he is on the phone. (O diretor não pode falar
com você agora pois está ao telefone.)
 They watched the concert live on TV. (Eles assistiram o show ao vivo na TV.)

4) Para indicar nomes de ruas ou avenidas.


Exemplos:
 I live on Alvaro Luiz Street. (Eu moro na rua Álvaro Luiz.)
 She wrote a book about a guy who lived on Seventh Street. (Ela escreveu um livro sobre
um cara que morava na Seventh Street.)
 There are many famous places on Oxford Street. (Há muitos lugares famosos na rua
Oxford.)
 They've got a company on Madison Avenue. (Eles têm uma empresa na Madison
Avenue).
Entretanto, se ao endereço for acrescido o número, utiliza-se o "at":
Exemplos:
 I live at 300 Karl Street. (Eu moro na rua Karl, nº 300.)
 He wrote a book about a man who lived at 25 Baker Street. (Ele escreveu um livro sobre
um homem que morava na Baker Street, nº 25.)

AULA 05 – PRONOUNS AND PREPOSITIONS 20


TEACHER ANDREA BELO

AT
A preposição at é utilizada nos seguintes casos:
1) Para indicar horários
Exemplo:
 I woke up at 6 am. (Acordei às 06:00h.)
 The concert is at 8 pm. (O show é às 20h.)
 Our flight will leave at 3 o'clock. (Nosso voo sairá às três horas.)
 My friends usually eat dinner at 10 pm. (Meus amigos geralmente jantam às 10h.)
 She finished the exam at 2 pm. (Ela terminou a prova às 14h.)

2) Para indicar locais específicos.


Exemplos:
 She has snacks at work every day. (Ela lancha no trabalho todos os dias.)
 I saw him reading the newspaper at the café. (Eu o vi lendo o jornal no café.)
 She is at the hospital to visit her mom. (Ela está no hospital para visitar sua mãe.)
 She is waiting for her dad at the airport. (Ela está esperando o pai no aeroporto.)

TO
A preposição to é utilizada nos seguintes casos:
1) Para indicar movimento, posição, destino ou direção.
We are going to Greece. (Nós vamos para a Grécia).
2) Para indicar duração de tempo (início e fim de um período).
I studied English from 2015 to 2018. (Eu estudei Inglês de 2015 a 2018).
3) Para indicar distância.
It’s about 2 blocks from the supermarket to my house. (São cerca de 2 quarteirões do
supermercado até minha casa).
4) Para indicar comparação entre coisas.
I prefer going to the movies to watch TV (Eu prefiro ir ao cinema a assistir TV).
5) Para indicar o motivo ou propósito. Nesse caso, a preposição é seguida de verbo.
We go out to relax and have fun. (Nós saímos para relaxar e nos divertir).

AULA 05 – PRONOUNS AND PREPOSITIONS 21


TEACHER ANDREA BELO

FOR
A preposição for é utilizada nos seguintes casos:
1) Para indicar duração de tempo.
I’ve worked at school for six years. (Trabalhei na escola por seis anos).
2) Para indicar finalidade. Nesse caso, é seguido de gerúndio.
A tape is used for fixing things. (Uma fita é usada para consertar as coisas).
3) Para indicar benefício ou favor.
Working out is very good for the health. (Fazer exercícios é muito bom para a saúde).
4) Para indicar motivo ou propósito. No entanto, diferentemente do “to” ele é seguido de
substantivo: This space is for guests only. (Este espaço é só para convidados).

Vou mostrar a você um quadro ilustrativo com as preposições que mais apareceram em
exercícios de variadas provas e em seguida, questões.

PREPOSITIONS OF TIME (Preposições de tempo)


DAYS OF THE WEEK ON MONDAY
ON
(Dias da semana) (Na segunda-feira)
IN MAY (Em maio)
MONTHS, TIME and YEARS
IN IN WINTER (No inverno)
(Meses, horas e ano)
IN 2019 (Em 2019)
EXACTLY TIME: AT NOUGHT, FOR A
WEEKEND, CERTAIN SPECIFIC TIME. AT NIGHT (À noite)
AT
(Tempo exato: à noite, no final de semana, AT SEVEN (Às sete)
algum tempo específico.)

FROM A CERTAIN POINT OF TIME


(PAST TILL NOW). SINCE 1999 (Desde 1999)
SINCE
[Em um certo tempo SINCE I WAS BORN (Desde que eu nasci)
(do passado até agora).]

OVER A CERTAIN PERIOD OF TIME


(PAST TILL NOW). FOR 3 YEARS (Por 3 anos)
FOR
[Depois de algum período do tempo FOR A LONG TIME (Por muito tempo)
(do passado até agora).]

A CERTAIN TIME IN THE PAST.


AGO 10 YEARS AGO (10 anos atrás)
(Um certo tempo no passado.)

BEFORE YOU WERE BORN


EARLIER THAN A CERTAIN POINT OF TIME.
BEFORE (Antes de você nascer)
(Antes de algum tempo no passado.)
BEFORE THAT (Antes disso)

AULA 05 – PRONOUNS AND PREPOSITIONS 22


TEACHER ANDREA BELO

QUESTÕES
Você vai, agora, responder questões selecionadas de provas já realizadas em anos
anteriores. Depois, como em todas as nossas aulas, haverá o gabarito e as questões comentadas.
Vamos começar com questões AFA, de acordo coma sua instituição escolhida e depois,
vamos treinar de outras Carreiras Militares, para adquirir experiência e treinar vocabulário.
QUESTÕES AFA
Directions: Read the text below and answer questions 01 to 10 according to it.

Covid won't be the last pandemic. Will we be better prepared for the next one?
Wouldn’t it be wonderful if we never had to face another pandemic? As we reflect on the past
year, the words that come to mind are “never again”. Never again should more than 120,000 lives
be lost to a contagious virus, a number that would have been unthinkable almost a year ago, when
government advisers thought that losing 20,000 people would be a good outcome. Nor should we
endure another year of lockdowns, and the pain of job losses and unemployment that followed.
And nobody ever wants to see children taken out of school for 21 weeks of the year again, unable
to learn in classrooms or see their friends.

But scientists are clear that we will face another pandemic in the future. Most agree the question
is when, not if. In 2018, I gave a talk at the Hay festival outlining a scenario that was loosely based
on recent news reports: a farmer becomes infected with an antibiotic-resistant bacterium from a
pig in China, spreading this infection through their community and then boarding a plane to the
UK. The worrying thing about this scenario is that such bacteria, which evade our antibiotics of
last resort, are already upon us. They could yet be the source of a future pandemic.

Or perhaps the next pandemic will be caused by a virus spilling over from animals to humans,
much like Covid-19. Recent reports that the first humans have been infected with avian influenza
at a poultry plant in Russia give a worrying glimpse of how close we live to such risks. Every time
a virus circulates among animals, particularly bats, rodents, livestock and birds, and then comes
into contact with humans, there’s a chance it will infect people and lead to human transmission.

Wherever the next pandemic comes from, Britain must think long term about its ability to respond
and must ensure it is prepared. Key to this will be working together with other governments to
establish robust response systems. Over the past year, the UK has already built many of the
structures that will be needed to address a future pandemic. We now have a huge testing
capacity, processes in place for fast-tracking vaccine development and protocols for undertaking
rapid research in hospitals. But more needs to be done at a global level to ensure we’re truly
prepared.
(Adapted from https://www.theguardian.com/commentisfree/2021/mar/24/covid-pandemic-prepared-investment-science)

AULA 05 – PRONOUNS AND PREPOSITIONS 23


TEACHER ANDREA BELO

Questão 01 (AFA/INÉDITA) – The first paragraph of the text states that


a) Considering the recent pandemic experience, it would be peaceful and acceptable to go
through another situation like this
b) Although extensive, the pandemic has not reached many deadly victims
c) Unemployment is not relevant in this pandemic context
d) It is common knowledge that the experiences lived in the last year were not interesting for
the population

Questão 02 (AFA/INÉDITA) – In the phrase “In 2018, I gave a talk at the Hay festival outlining a
scenario that was loosely based on recent news reports…” (paragraph 2), the underlined word is
a synonym for
a) Informing
b) Describing
c) Summarizing
d) Criticizing

Questão 03 (AFA/INÉDITA) – The passage “But scientists are clear that we will face another
pandemic in the future. Most agree the question is when, not if” (paragraph 2) suggests that
a) The exact timing of the next pandemic is still unknown
b) Scientists cannot say whether there will be another pandemic, they still wonder that
c) Scientists know that another pandemic will happen in the future, most already know when
d) The pandemic is such an unknown context that scientists are still wondering whether it will
happen and, if so, when

Questão 04 (AFA/INÉDITA) – The second paragraph of the text states that


a) The antibiotics available to the population cover all types of existing bacteria
b) The aforementioned bacterium does not have a transmissible profile
c) The bacterium can cause much greater damage
d) The bacterium has no potential to cause a pandemic like the one we are living in

Questão 05 (AFA/INÉDITA) – In the phrase “Or perhaps the next pandemic will be caused by a
virus spilling over from animals to humans, much like Covid-19” (paragraph 3), the underlined
expression is a synonym for
a) Gathering
b) Discharging
c) Hiding
d) Sorting out

AULA 05 – PRONOUNS AND PREPOSITIONS 24


TEACHER ANDREA BELO

Questão 06 (AFA/INÉDITA) – Mark the alternative which rewrites the sentence below correctly,
in the passive voice “Nor should we endure another year of lockdowns…” (paragraph 1)
a) Nor another year of lockdowns should be endured by us
b) We shouldn’t face another year of lockdowns
c) We need to face another year of lockdowns
d) Another year of lockdowns should not be faced

Questão 07 (AFA/INÉDITA) – The passage “Key to this will be working together with other
governments to establish robust response systems” (paragraph 4) suggests that
a) Problem response systems must be developed individually by each government
b) Governments working together does not aim at any major objective
c) The best solution to the problem will be to work together among the countries
d) An individualistic attitude would collaborate to solve the problem

Questão 08 (AFA/INÉDITA) – The text


a) Focuses on explaining the starting point of a pandemic
b) Alerts the population about an upcoming pandemic and how it can occur and be resolved
c) Criticizes the individualistic attitude of some countries when dealing with the current
pandemic context
d) Develops the idea that the UK is the best prepared country for a pandemic

Should you be grateful for a job?


It's become a common refrain: “I’m just grateful to have a job”.
The last year has wreaked undeniable havoc on the working world. Globally, the working hours
and income lost in 2020 added up to the equivalent of 255 million full-time jobs. Workplace
closures, layoffs and a steep rise in unemployment are enough to make anyone who’s managed
to hold onto their job feel some measure of gratitude – or, at least, pressure to be grateful.
That pressure pre-dates the pandemic. One of the most pervasive conversations around jobs is
that we should be thankful to be hired, especially when competition for a position is fierce.
Candidates are even expected to express the sentiment if they want to be hired in the first place:
it’s hard to imagine leaving an interview without saying how much you appreciate being
considered, or sending a thank-you email.
But it’s possible some of that gratitude is misplaced. Perhaps it’s not quite appropriate to be
thankful that an employer is ‘letting you’ work for them. And while gratitude can be objectively
good for you – research consistently associates giving thanks with increased happiness – it also
has a darker side that can make you more willing to put up with a situation that makes you
unhappy.

AULA 05 – PRONOUNS AND PREPOSITIONS 25


TEACHER ANDREA BELO

Differing obligations
Some workers may be much more inclined to feel grateful for their jobs than others.
Workers who expect to be hired or promoted may express less gratitude than those without
systemic advantages. This is often the case for white men, who experience more upward mobility
than other groups, and less bias that prevents them from securing jobs, or getting interviews in
the first place. For instance, multiple studies have shown résumés with “white-sounding” names,
and those that downplay racial cues, are significantly more likely to garner a response.
Imposter syndrome may also play a part: workers who aren’t confident they deserve their roles
may develop feelings of unworthiness, despite being qualified or skilled. Women are particularly
vulnerable to imposter syndrome, and may find themselves giving outsize thanks for their jobs.
And, in recent months, Latino and black Americans were significantly more likely to be affected
by pandemic-related lay-offs than white Americans. Those among these groups who have kept
their jobs are likely feeling pressure to express gratitude – even if they have to force it, and even
if their workplace doesn’t inspire much to be thankful for.
(Adapted from https://www.bbc.com/worklife/article/20210329-should-you-be-grateful-for-a-job)

Questão 09 (AFA/INÉDITA) – In the phrase “The last year has wreaked undeniable havoc on the
working world” (paragraph 2), the underlined word is a synonym for
a) Harmony
b) Gains
c) Losses
d) Damage

Questão 10 (AFA/INÉDITA) – The second paragraph of the text states that


a) A large part of the population has suffered the consequences of the pandemic in the
workplace
b) The pressure to feel grateful to have a job did not arise in the pandemic
c) 2020 was a difficult and exhausting year, but it provided many gains for the working world
d) The feeling of gratitude is always something genuine on the part of the worker, towards his
employer

QUESTÕES COLÉGIO NAVAL


Read Text I to do questions 01 to 04 based on it.
Covid: 'Israel may be reaching herd immunity'
This happens when enough of a population has protection against an infection that it stops being
able to spread – and even people who don't themselves have immunity are indirectly protected.
For Covid the estimated threshold for herd immunity is at least 65%-70%.

AULA 05 – PRONOUNS AND PREPOSITIONS 26


TEACHER ANDREA BELO

But scientists in the UK are more cautious.


Dr Sarah Pitt, a virologist at the University of Brighton, urged "extreme caution" in concluding that
herd immunity had been reached – something she believes will be difficult even at high
vaccination rates.
She said it was still too early to tell: "We need to see whether the cases in Israel continue to fall
and stay at low levels."
Reaching this level of population immunity is important to protect people who can't be vaccinated
or whose immune system is too weak to produce a good, protective response.
In Israel, more than half (5.3 million) its residents have been vaccinated and an additional 830,000
people have tested positive for the virus in the past, which should give them some natural
immunity.
That works out as roughly 68% of the population who are likely to have antibodies in their blood
which can fight off the virus.
Prof Eyal Leshem, a director at Israel's largest hospital, the Sheba Medical Center, said herd
immunity was the "only explanation" for the fact that cases continued to fall even as more
restrictions were lifted.
"There is a continuous decline despite returning to near normalcy," he said.
"This tells us that even if a person is infected, most people they meet walking around won't be
infected by them."
And cases are falling in all age groups including children, even though under-16s are not generally
being vaccinated.
(Adapted from https://www.bbc.com/news/health-56722186)

Questão 01 (COLÉGIO NAVAL/INÉDITA) – Read the extract from the text. For Covid the
estimated threshold for herd immunity is at least 65%-70%. Mark the alternative that can
replace the underlined word without changing its meaning.
(A) Brink
(B) Middle
(C) Conclusion
(D) Disadvantage
(E) End

Questão 02 (COLÉGIO NAVAL/INÉDITA) – About herd immunity, it’s correct to affirm that
(A) Is only achieved when the entire population is vaccinated
(B) Not necessarily the whole population needs to be vaccinated to reach it
(C) It only protects those who have already been immunized
(D) For the case of COVID-19, the vaccination threshold is very low
(E) It has already been achieved against COVID-19

AULA 05 – PRONOUNS AND PREPOSITIONS 27


TEACHER ANDREA BELO

Questão 03 (COLÉGIO NAVAL/INÉDITA) – Read the extract from the text: “Reaching this level of
population immunity is important to protect people who can't be vaccinated…”. The sentence
above means that
(A) People who cannot be vaccinated, unfortunately, cannot be protected in any other way
(B) Reaching a certain number of vaccinees does not interfere with the protection of those who
have not been vaccinated
(C) Upon reaching a certain number of vaccinees, those who cannot be vaccinated are protected,
but not as much as those who have been vaccinated
(D) Reaching a certain number of vaccinees, even those who are not vaccinated are protected
(E) Achieving any level of vaccinees ensures the protection of those who have not been
vaccinated

Questão 04 (COLÉGIO NAVAL/INÉDITA) – What can we infer from the text? Mark the correct
option
(A) The entire Israeli population is already immune to COVID-19
(B) Although many have already been vaccinated, COVID-19 cases continue to increase in Israel
(C) 68% of the Israeli population has already been vaccinated
(D) Israel has already achieved herd immunity against COVID-19
(E) It is possible that the majority of Israel's population is already protected from COVID-19

Read the text II to do items 05 to 10.


My Family Wants to Visit This Summer. Is Travel Safe Yet?
Navigating our semi-vaccinated world is full of tough questions like this one. Luckily, the U.S.
Centers for Disease Control and Prevention (CDC) has issued guidance that may help.
According to the CDC, fully vaccinated people can visit indoors and unmasked with low-risk
unvaccinated people from a single household. Recent research suggests getting both doses of the
provides Moderna or Pfizer-BioNTech vaccines about 90% protection against COVID-19 infections.
That means a person fully vaccinated with these shots has a pretty slim chance of infecting an
unvaccinated person, or vice versa.
But it’s important to read the fine print there. Since no vaccine is 100% perfect, you should only
visit with low-risk unvaccinated people—i.e., those who do not have medical conditions or other
factors that would increase their chances of having a severe case of COVID-19 if they somehow
did get infected. And you should only visit with one household of unvaccinated people at a time;
your vaccination status wouldn’t make it any safer for a bunch of unvaccinated people to spend
time together inside and unmasked.
Let’s assume the family members who want to visit meet both of those standards. Great! But
there’s still the question of travel. Right now, the CDC does not recommend non-essential travel
for unvaccinated people.

AULA 05 – PRONOUNS AND PREPOSITIONS 28


TEACHER ANDREA BELO

Even though airplanes have pretty good filtration systems, travelers encounter lots of other people
in fairly tight quarters during the average trip, says Cindy Prins, an associate professor of
epidemiology at the University of Florida. Each of those interactions raises the chances of COVID-
19 exposure. Plus, if your family members were unknowingly infected while traveling, or got
infected during the trip, they could potentially seed new cases in your area.
(Adapted from https://time.com/5954515/summer-travel-covid-vaccination/)

Questão 05 (COLÉGIO NAVAL/INÉDITA) – According to the text, mark the INCORRECT option
(A) The vaccine allows meetings, but with some conditions
(B) Considering the low level of protection provided by vaccines, coexistence must still be
controlled
(C) In addition to protecting against COVID-19, the vaccine prevents the spread of the virus
(D) Vaccines against COVID-19 do not give full protection against the virus
(E) Vaccinated persons should still exercise caution when in contact with other people, especially
those in a high-risk group

Questão 06 (COLÉGIO NAVAL/INÉDITA) – Read the extract from the text


“But it’s important to read the fine print there. Since no vaccine is 100% perfect, you should only
visit with low-riskunvaccinated people…”
Mark the option that can replace the underlined sentence without changing its meaning
(A) People from unvaccinated risk groups should not be visited, considering that the vaccine
cannot be trusted 100%
(B) Even with the full effectiveness of all vaccines, you can only visit unvaccinated people who
are not at risk groups
(C) The total effectiveness of all vaccines allows you to visit anyone, even if not vaccinated
(D) Although vaccines are not 100% effective, you can visit anyone after being vaccinated
(E) Since no vaccine is 100% effective, you should not visit anyone

Questão 07 (COLÉGIO NAVAL/INÉDITA) – Read the extract from the text


“Plus, if your family members were unknowingly infected while traveling, or got infected during
the trip, they could potentially seed new cases in your area” (paragraph 5). The word “they”
refers to
(A) New cases
(B) Airplanes
(C) Family members
(D) Interactions
(E) Filtration systems

AULA 05 – PRONOUNS AND PREPOSITIONS 29


TEACHER ANDREA BELO

Questão 08 (COLÉGIO NAVAL/INÉDITA) – Moderna and Pfizer-BioNTech vacines


(A) Offer a very low level of protection
(B) Offer a high level of protection against COVID-19 when taken in two doses
(C) Do not have any type of measurement regarding the level of protection offered
(D) Offer a high level of protection even with just one dose
(E) Offer total protection against COVID-19

Questão 09 (COLÉGIO NAVAL/INÉDITA) – The expression “though”, in paragraph 1, means


(A) Easy
(B) Fragile
(C) Hard
(D) Contradictory
(E) Stressful

Questão 10 (COLÉGIO NAVAL/INÉDITA) – The word “slim” (paragraph 2), can be replaced by the
word __________ without changing its meaning
(A) Thin
(B) Large
(C) Medium
(D) Delicate
(E) Small

QUESTÕES EAM
Read text I and answer questions 01 and 02
Which processed foods are better than natural?
The language used to describe the foods we eat can have a huge effect on how we perceive them:
"organic", "artisan", "homemade" and "handpicked" foods sound slightly more tempting than the
prosaic "tinned", "rehydrated" or "freeze-dried".
Another adjective that can whet our appetites is "natural", while we tend to associate "processed"
food with long lists of ingredients we can't pronounce. But when it comes to our health – is natural
always better than processed?
Actually, naturalness doesn't automatically mean a food is healthy, says Christina Sadler, manager
at the European Food Information Council and researcher at the University of Surrey.
In fact, natural foods can contain toxins, and minimal processing can in fact make them safer.

AULA 05 – PRONOUNS AND PREPOSITIONS 30


TEACHER ANDREA BELO

Kidney beans, for instance, contain lectins, which can cause vomiting and diarrhoea. They're
removed by soaking the beans in water overnight and then cooking them in boiling water.
Processing also makes cow's milk safe to consume. Milk has been pasteurised since the late 1800s,
in order to kill harmful bacteria. Before this time, it was distributed locally, because there wasn't
good refrigeration in houses.
"Cows in cities were milked every day, and people would bring milk in carts back to their
neighbourhoods to sell it," says John Lucey, food science professor at the University of Wisconsin-
Madison.
"As cities got bigger, milk got further away and took longer to get to the consumer, which meant
pathogens could multiply."
Mounting evidence suggesting that some organisms in milk could be harmful led to the
development of heating devices for milk and the invention of pasteurisation, which was soon
adopted across Europe, and later in the US.
"It's one of the major public health success stories of the last century," Lucey says. "Just before
World War Two, around a quarter of all food and waterborne diseases came from milk. Now it's
less than 1%."
Processing can also help to retain nutrients in food we eat. For example, freezing, which is
classified as minimal processing, allows fruit and vegetables to retain nutrients that can otherwise
degrade while sitting in a fridge.
(Adapted from https://www.bbc.com/future/article/20210521-which-processed-foods-are-better-than-natural)

Questão 01 (EAM/INÉDITA) – Say if the following statements are T (true) or F (false) about
processed and natural foods. Then, mark the correct option, from top to bottom.
( ) The language used to describe the types of food has no relevant impact on their
understanding
( ) "Organic" and "homemade" are repulsive expressions
( ) Sometimes we have a misconception about "processed" food
( ) Healthy food is not necessarily "natural"
( ) Food processing can be beneficial
(A) F – F – T – T – T
(B) F – F – T – T – F
(C) F – T – T – T – T
(D) T – F – T – T – F
(E) T – F – T – T – T

AULA 05 – PRONOUNS AND PREPOSITIONS 31


TEACHER ANDREA BELO

Questão 02 (EAM/INÉDITA) – About the example of milk given by the text, it is correct to say
that
(A) The increase in cities has not changed the consequences of the consumption of pure milk
(B) Pasteurization of milk did not have as an incentive the pathogens that the product contained
(C) It was safer to consume milk before the invention of processing this food
(D) Milk processing has made it a healthier and more accessible product
(E) Heating devices and pasteurization did not result in better preservation of milk

Read text II and answer questions 03 and 04


CDC studying reports of heart inflammation in young Covid vaccine recipients
Some teenagers and young adults who received Covid-19 vaccines have experienced heart
inflammation, a US Centers for Disease Control and Prevention advisory group said,
recommending further study of the rare condition.
In a statement dated 17 May, the CDC’s advisory committee on immunisation practices said it had
looked into reports that a few young vaccine recipients, predominantly male adolescents and
young adults, developed myocarditis, an inflammation of the heart muscle.
The condition often goes away without complications and can be caused by a variety of viruses,
the CDC group said.
CDC monitoring systems had not found more cases than would be expected in the population,
but members of the committee on vaccinations felt that healthcare providers should be made
aware of the reports of the “potential adverse event”, the committee said in the statement.
It did not say how many people had been affected and recommended further investigation.
(Adapted from https://www.theguardian.com/world/2021/may/23/cdc-studying-reports-of-heart-inflammation-in-young-covid-vaccine-recipients)

Questão 03 (EAM/INÉDITA) – In the sentence “It did not say how many people had been
affected and recommended further investigation” (paragraph 5), the pronoun “It” refers to
(A) CDC monitoring systems
(B) The committee on vaccinations
(C) CDC studying reports
(D) The statement
(E) Healthcare providers

AULA 05 – PRONOUNS AND PREPOSITIONS 32


TEACHER ANDREA BELO

Questão 04 (EAM/INÉDITA) – It is FALSE to say that


(A) Some people are known to have developed heart inflammation after taking the vaccine
against COVID-19
(B) Heart inflammation is a rare condition in these circumstances
(C) The cause of the inflammation can have several sources
(D) Heart inflammation may be an adverse event related to the COVID-19 vaccine
(E) A considerable number of people who received the vaccine against COVID-19, developed
heart inflammation

Questão 05 (EAM/INÉDITA) – Complete the paragraph below, about the pandemic situation in
Nepal, with the missing prepositions
“As I write this, my country is battling __________ new and brutal wave of the Covid-19 pandemic.
The rise in the number __________ infections poses a serious challenge __________ our brave
doctors, nurses, other care providers, citizen volunteers and the entire health service system”.
(Adapted from https://www.theguardian.com/commentisfree/2021/may/10/nepal-covid-uk-g7)

(A) A / with / for


(B) A / of / to
(C) An / of / to
(D) An / at / for
(E) A / at / to

QUESTÕES EEAR
Read the text I to do questions 01 to 10 based on it.
Menthol Cigarettes Kill Smokers, Most of Them Black.
Tobacco companies gain goodwill by advertising to black readers — and by donating money to
civil rights organizations.
Menthol creates a cooling sensation in tobacco products. Some studies have shown that menthol
also acts as a mild anesthetic.
There is a movement to ban menthol cigarettes. Why? The mint-flavored products target black
Americans.
On the whole, black Americans smokers smoke less than white people do. But black smokers die
of causes linked to tobacco use at higher rates than white smokers. About 85 percent of black
smokers use Newport, Kool, and other menthol brands.

AULA 05 – PRONOUNS AND PREPOSITIONS 33


TEACHER ANDREA BELO

It is easy to become addicted to these brands. They also are harder to quit than tobacco without
menthol.
The African American Tobacco Control Leadership Council is for menthol bans. It says menthol
products are the “main vectors” of disease and death among black Americans.
__________ (9) white health advocates oppose flavored e-cigarettes, which include menthol.
There is concern about the use of menthol cigars and cigarillos among black teenagers. Most black
high school students smoke menthol tobacco products.
Tobacco companies target black communities with menthol cigarettes. They give away free
samples and offer discounts.
They sponsor concerts and special events. They advertise in newspapers and magazines geared
to black readership. They donate money to civil rights organizations.
The companies have also been donors to black political candidates. They have been supporters of
the Congressional Black Caucus.
The tobacco companies say they want to be good citizens. But it is hard for them to resist the
chance to make a profit.
_______ (11) friends in Congress work to defeat efforts to ban flavored tobacco products.
A lobbyist said, a menthol ban would infringe on the rights of adults who preferred it to plain
tobacco.
The tobacco industry has joined forces with civil rights activists. Among them is the Rev. Al
Sharpton. Sharpton visited black communities in California. He raised the fear that a menthol ban
would give the police an excuse to stop and frisk blacks.
Mr. Sharpton also helped to defeat a menthol ban in New York.
Observers point out that society is making marijuana legal. In what direction should public policy
go? The tobacco industry also will not go quietly. It is creating new products that are less harmful
to users.
Americans consume alcohol, tobacco and all manner of drugs, some legal, some not. The banning
of one type of cigarette is not efficient. It may not work.
Source: The New York Times March 22, 2021

Questão 01 (EEAR/INÉDITA) – The passage “There is a movement to ban menthol cigarettes.”


states that menthol cigarettes must be ____________ .
a) forbidden.
b) allowed.
c) accepted.
d) recollected.

AULA 05 – PRONOUNS AND PREPOSITIONS 34


TEACHER ANDREA BELO

Questão 02 (EEAR/INÉDITA) – sentence in bold in the text is in the:


a) Past continuous.
b) Present Continuous.
c) Simple Present.
d) Simple Past.

Questão 03 (EEAR/INÉDITA) – Choose the alternative in which you can find the Present
Continuous.
a) Tobacco companies gain goodwill by advertising to black readers.
b) Menthol creates a cooling sensation in tobacco products.
c) The banning of one type of cigarette may not work.
d) Observers point out that society is making marijuana legal.

Questão 04 (EEAR/INÉDITA) – In “They sponsor concerts and special events”, the word
“sponsor” means:
a) to get something by paying money for it.
b) to pay for someone to do something or for something to happen.
c) to get or receive something from someone with the intention of giving it back after a period
of time.
d) to make something clear or easy to understand by describing or giving information about it.

Questão 05 (EEAR/INÉDITA) – Choose the answer in which the verb "do", in bold, is being used
in the same way as the sentence below. “black Americans smokers smoke less than white
people do”
a) Do you think she will recognize me? She might do.
b) I’m going to do some work in the garden this weekend.
c) I don’t want to wait for a bus. Let’s get a taxi.
d) I do like your new jacket!

Questão 06 (EEAR/INÉDITA) – All alternatives are not in the comparative, except:


a) reader.
b) smoker.
c) higher.
d) manner.

AULA 05 – PRONOUNS AND PREPOSITIONS 35


TEACHER ANDREA BELO

Questão 07 (EEAR/INÉDITA) – The adjective form “good” and “easy”, underlined in the text,
have as their comparative forms, respectively:
a) gooder and easyer.
b) better and easier.
c) best and the easiest.
d) better and the easyer.

Questão 08 (EEAR/INÉDITA) – In “But it is hard for them to resist the chance to make a profit”,
the word “them” refers to
a) citizens.
b) rights organizations.
c) Congressional Black Caucus.
d) tobacco companies.

Questão 09 (EEAR/INÉDITA) – Fill in the blank with the suitable option:


a) Much.
b) Many.
c) A lots of.
d) Lots.

Questão 10 (EEAR/INÉDITA) – In “It is easy to become addicted to these brands.”, the


underlined word has the same use as in
a) But it is hard for them to resist the chance to make a profit.
b) It says menthol products are the “main vectors” of disease.
c) A lobbyist said, a menthol ban would infringe on the rights of adults who preferred it to plain
tobacco.
d) It is creating new products that are less harmful to users.

QUESTÕES EFOMM
Based on the text below, answer questions 01 and 02.
Does motherhood belong on a resume?
Many have advocated that motherhood is a legitimate job that builds employable skills. Does
the title belong on mums' CVs?
Mums multi-task. They plan. They research, organise, negotiate, manage time and lead.
Although mothers’ juggling hasn’t ever been a secret, their role has, perhaps, never been more
obvious than during the pandemic. As schools transitioned to remote classrooms, and women

AULA 05 – PRONOUNS AND PREPOSITIONS 36


TEACHER ANDREA BELO

took on more of both the physical and mental load of home life than before, the skills required to
keep the trains on the tracks have been on full display.
As a result, the question increasingly floating to the surface is whether or not these skills have a
place on mothers’ CVs.
There’s long been a push to recognise motherhood as a legitimate job that trains workers in
legitimate skills, valuable to employers. And some voices are getting louder. One of the newest
leaders is HeyMama, a US-based community for working mums, who’ve launched a campaign
called Motherhood on the Resume. It’s quite literal, says Katya Libin, HeyMama’s co-founder and
CEO – the organisation is advocating for mothers to update their titles on LinkedIn, or even add
the position on a resume, like any other ‘recognised’ job in, say, sales or engineering.
Whether motherhood ‘belongs’ on a resume is, of course, subjective. The question, instead, lies
in whether mothers can reap tangible benefits for the addition of the title – or whether some
systemically entrenched biases around mums could produce the opposite effect.
(Adapted from https://www.bbc.com/worklife/article/20210617-does-motherhood-belong-on-a-resume)

Questão 01 (EFOMM/INÉDITA) – It is possible to infer from the text that


A) Mothers are limited to one task at a time because of their children
B) The pandemic made maternal tasks less clear
C) Maternal skills should not be included in mothers' CVs
D) The struggle for recognition of motherhood as a legitimate job is still very superficial
E) It is possible that adding motherhood to the curriculum could have positive or negative
effects

Questão 02 (EFOMM/INÉDITA) – In the excerpt “...or whether some systemically entrenched


biases around mums could produce the opposite effect”, the word in bold means
A) Changeable
B) Ingrained
C) Obvious
D) Strong
E) Secured

Based on the text below, answer questions 03, 04 and 05.


U.S. birthrates are plummeting. Increasing legal immigration can help.
THE DECADES-LONG decline in the U.S. birthrate accelerated in 2020, as the average number of
babies born to American women over the course of their lifetimes fell to its lowest level since
government record-keeping began nearly a century ago. After a decade in which the population

AULA 05 – PRONOUNS AND PREPOSITIONS 37


TEACHER ANDREA BELO

grew at the most sluggish pace since the 1930s, last year’s slowdown in births, which intensified
as the pandemic took hold, suggests a new demographic normal — one that poses daunting
economic and geopolitical challenges.
In a country where fertility is now well below the replacement rate required to compensate for
deaths, an obvious question arises: As the United States ages, how will a dwindling cohort of
younger, working-age Americans sustain the expensive social services that their parents and
grandparents are counting on in retirement?
The rational answer is a robust immigration system, one that affords the nation a ready supply of
scrappy, striving employees in jobs for which there are insufficient numbers of native-born
Americans, as well as a steady stream of well-educated professionals to fill engineering, scientific,
technology and medical jobs, among others. Immigrants are twice as likely to start new businesses
as native-born Americans and buttress economic growth — think of Tesla, Google and PayPal, all
started by entrepreneurs born elsewhere.
Yet as the 2020 Census demonstrated, immigration flattened out following the Great Recession
in 2008, was actively impeded by the Trump administration and fell once the coronavirus
tightened its grip. That reversed a trend toward rising numbers of immigrants since the 1960s,
when a half-century boom in foreign-born arrivals, the vast majority of them legal, helped fuel
economic growth.
Granted, no celestial mandate dictates that the United States must grow at a faster clip than other
developed countries, as it has for most of its history. But population stagnation may mean a very
different future, and probably a less vital one, than many Americans might imagine. In the short
run, it might mean jobs that go begging for workers as caretakers for the elderly, truck drivers,
computer programmers and other occupations. In the long run, it could impede the country’s
ability to maintain its status as a superpower, project influence and compete with China.
Among the many theories to explain the falling birthrate are women’s increasing labor-force
participation; the daunting cost of living, particularly housing, in job-rich urban areas; a dramatic
drop-off in teen pregnancy; and a social media-distracted younger generation that may be having
less sex. Whatever combination of factors is driving the baby bust, what’s clear is that there will
be no easy answer to reversing it, though family-friendly government and corporate policies could
help.
By contrast, increasing the level of legal immigration is a policy choice that comes with a direct
positive impact. Making that choice would entail forging a political consensus on a uniquely
divisive issue that President Donald Trump, among others, has helped turn into a question of tribal
identity after years in which it was a matter of bipartisan consensus. But not making it is likely
tantamount to acquiescing to an era of demographic stagnation and, over time, diminished
national stature.
(Adapted from https://www.washingtonpost.com/opinions/us-birthrates-are-plummeting-increasing-
legal-immigration-can-help/2021/06/25/b3e28236-aded-11eb-b476-c3b287e52a01_story.html)

AULA 05 – PRONOUNS AND PREPOSITIONS 38


TEACHER ANDREA BELO

Questão 03 (EFOMM/INÉDITA) – Read the statements about the text and decide whether they
are TRUE (T) or FALSE (F). Mark the correct option
I. The decline in the U.S. birthrate started in the pandemic
II. In the US, the birthrate is not high enough to offset the deathrate
III. The legal immigration system does not serve as a solution to sustain the needs of the older
population
IV. The legal immigration system is an immediate way to reach young people of working age
V. The new US demographic normal could have profound impacts on the country's economy
I – (F) / II – (T) / III – (F) / IV – (F) / V – (T)
I – (F) / II – (T) / III – (T) / IV – (T) / V – (T)
I – (T) / II – (T) / III – (F) / IV – (T) / V – (T)
I – (F) / II – (T) / III – (F) / IV – (T) / V – (T)
I – (T) / II – (T) / III – (T) / IV – (F) / V – (T)

Questão 04 (EFOMM/INÉDITA) – In the excerpt “…suggests a new demographic normal — one


that poses daunting economic and geopolitical challenges”, the word in bold means
A) Frightening
B) Comforting
C) Ending
D) Constructive
E) Impossible

Questão 05 (EFOMM/INÉDITA) – According to the text, the falling birthrate in the US


A) Collaborated with the high cost of living in job-rich urban areas
B) May be related to the insertion of women in the labor market
C) Is directly related to the increase in teenage pregnancy
D) Has slowed down with the pandemic
E) Can be easily reversed

Questão 06 (EFOMM/INÉDITA) – Choose the correct option to complete the paragraph below.
“More _______ us are starting to pick back up the strands of our pre-pandemic social lives. As we
figure out who the first people we want to meet up _______ are, we’re recognising there are
friendships _______ the ‘before times’ we didn’t keep up during lockdown – and aren’t
particularly excited _______ re-ignite now that we can”.
(Adapted from https://www.bbc.com/worklife/article/20210623-why-its-ok-to-let-friendships-fade-out)

AULA 05 – PRONOUNS AND PREPOSITIONS 39


TEACHER ANDREA BELO

A) Of / at / from / to
B) X / with / in / to
C) Of / with / from / to
D) Of / with / in / to
E) X / at / in / to

Questão 07 (EFOMM/INÉDITA) – Which is the correct way to complete the paragraph below?
“In June 2021, the US Food and Drug Administration (FDA) __________ the first Alzheimer’s drug
in 18 years: aducanumab (also known by its brand name Aduhelm). At the time of writing, the
drug __________ also under review in the EU, Japan, and several other countries. For the roughly
30 million people worldwide who __________ with Alzheimer’s, this is unprecedented news, and
must seem like cause for optimism”.
(Adapted from https://www.theguardian.com/commentisfree/2021/jun/28/alzheimers-drug-aducanumab-approval-dementia)

A) Approved / is / live
B) Approve / is / live
C) Approve / is / lives
D) Approved / was / live
E) Approved / was / lives

Questão 08 (EFOMM/INÉDITA) – Which of the following sentences expresses probability?


A) You must eat more vegetables
B) She needs to learn to make her own decisions
C) My father is a very determined person
D) You are much smarter than you think
E) She left two hours ago so she should be getting there by now

Questão 09 (EFOMM/INÉDITA) – Choose the correct option to complete the paragraph below.
“In the early days of the pandemic, economist Jeanet Bentzen of the University of Copenhagen
examined Google searches for the word ‘prayer’ in 95 countries. She identified that they hit an
all-time global high in March 2020 and increases occurred in lockstep with the number of COVID-
19 cases identified in each country. Stateside, __________ the Pew Research Center, 55 percent
of Americans prayed to end the spread of the novel coronavirus in March 2020, __________
nearly one quarter reported that their faith increased the following month, __________ limited
access to houses of worship”.
(Adapted from https://www.scientificamerican.com/article/psychiatry-needs-to-get-right-with-god/)

AULA 05 – PRONOUNS AND PREPOSITIONS 40


TEACHER ANDREA BELO

A) In contrast to / and / despite


B) According to / and / despite
C) According to / on the other hand / accordingly
D) However / and / despite
E) According to / on the other hand / despite

Questão 10 (EFOMM/INÉDITA) – Which option is incorrect?


A) I want to have lunch before going back to work
B) When can you give me back the money I loaned you?
C) Did you pass the exam?
D) I want they to be happy
E) I need to buy new clothes

QUESTÕES EPCAR
Directions: Read the text below and answer questions 01 to 10 according to it.
Hugs are coming back. Not everyone is thrilled.
When Stevi Stephens was 5 years old, her grandmother bent down for a hug, and Stephens
wondered if stepping on her foot would make her stop.
As a baby, her mother told her, Stephens cried when anyone held her; later, as a married woman,
she used to get up and change sides of the bed multiple times each night when her husband would
scoot over in his sleep to put an arm around her. “He was like a heat-seeking missile,” she says.
Don’t misunderstand: “I had a great sex life,” Stephens, now 76, clarifies. But Stephens’s attitude
toward being hugged hasn’t changed. With another person wrapped around her, she feels
restrained, uncomfortable.
Stephens, a retired anthropologist, lives in a small co-op of restored fishermen’s huts on
Vancouver Island with about nine other people. “Nobody’s ever tried to hug me here,” she says.
And for Stephens, the past 14 months of social distancing — and the freedom from almost any
hugs at all, even from her two grown children — has been blissful.
Ever since coronavirus vaccines became widely available to the general population, it’s been
evident in parks, restaurants and homes throughout America: Arms across your back are back.
Grandparents are hugging grandkids again.
Friends are hugging friends. Even epidemiologists, a notably cautious bunch, are hugging. For
many, the return of hugs has been a welcome step toward the return of normalcy.
Others, though, have been dreading this moment for a long time. In the frenzied, joyous rush to
make up for a year of lost embraces, it’s easy to lose sight of people like Stephens, who cringe at

AULA 05 – PRONOUNS AND PREPOSITIONS 41


TEACHER ANDREA BELO

the thought of having to endure a whole separate human body enveloping them with little to no
prior notice. Personal-space enthusiasts are sad to see their year of living huglessly come to an
end — even as they hold onto hope that some pandemic distancing habits might stick.
Stevi Stephens lives in a small co-op of restored fishermen’s huts on Vancouver Island. “Nobody’s
ever tried to hug me here,” she says. (Katie Rose MacKenzie)
There are reasons so many humans feel comforted by hugging. Like a massage, it “involves
stimulation and pressure receptors, and when that happens, the whole nervous system slows
down and stress hormone is reduced,” says Tiffany Field, director of the Touch Research Institute
at the University of Miami School of Medicine.
Still, for some, hugs induce stress more than they relieve it. When Sam Zelinka, a federal-
government research scientist based in Madison, Wis., gets a hug from anyone other than his wife
or kids, he gets the same feeling as when a stranger stands too close to him. And while Zelinka,
38, isn’t about to “spray paint a T-shirt and march in the streets yelling about no hugs,” he’s not
exactly looking forward to one from anyone but his parents.
Brooke Todd, a 24-year-old social worker in Stroudsburg, Pa., tends to tense up when someone
moves to hug her, and she didn’t totally make sense of how much calmer her social life had
become during the pandemic until last summer. “It took a few months, but suddenly I was like,
‘Oh, this is nice. I don’t look like a jerk all the time for not wanting to hug someone.’”
Certain kinds of people tend to dislike being touched or embraced, Field notes: some children and
adults with autism, for example, as well as many survivors of sexual assault, such as Todd. Plus,
Field says, the #MeToo movement prompted lots of people to raise a quizzical eyebrow at hugs
in the workplace. “I think that even before covid, hugging was already going out.”
(Adapted from https://www.washingtonpost.com/lifestyle/2021/05/15/hugs-greeting-pandemic/)

Questão 01 (EPCAR/INÉDITA) – Mark the option that can replace the word “thrilled” in the title
without changing its meaning
A) Excited
B) Bored
C) Calm
D) Blessed

Questão 02 (EPCAR/INÉDITA) – We can deduce from the first paragraph that


A) Stevi came to dislike hugs when she became an adult
B) Stevi did not like hugs at age 5
C) Stevi was a very aggressive child
D) Stevi didn't like her grandmother

AULA 05 – PRONOUNS AND PREPOSITIONS 42


TEACHER ANDREA BELO

Questão 03 (EPCAR/INÉDITA) – Mark the option with the suitable question to answer the
underlined fragment below: “But Stephens’s attitude toward being hugged hasn’t changed.
With another person wrapped around her, she feels restrained, uncomfortable”
A) Does Stephens like to be hugged?
B) How does Stephens feel about being hugged?
C) What is Stephens' attitude when hugged?
D) What happened to Stephens' attitude towards being hugged?

Questão 04 (EPCAR/INÉDITA) – Mark the option that can replace the underlined word keeping
the same meaning
“With another person wrapped around her, she feels restrained, uncomfortable”
A) Friendly
B) Curbed
C) Bold
D) Controlled

Questão 05 (EPCAR/INÉDITA) – Mark the alternative that can answer the question below
according to the text: How does Stephens feel about social distancing?
A) She feels restrained and uncomfortable
B) She feels indifferent
C) She feels good and comfortable
D) She feels needy

Questão 06 (EPCAR/INÉDITA) – According to the fifth paragraph, coronavirus vaccines


A) Have made it possible to start returning to social life
B) Are still restricted to a small portion of the population
C) Have not yet made physical contact possible
D) Demonstrate their consequences very subtly across America

Questão 07 (EPCAR/INÉDITA) – Mark the correct option


A) Epidemiologists are the most cautious group regarding COVID-19
B) Many people believe that being able to hug is a step back to normal
C) There are no people who fear the return to normality
D) Stephens is one of the few people who do not want an end to social distancing

AULA 05 – PRONOUNS AND PREPOSITIONS 43


TEACHER ANDREA BELO

Questão 08 (EPCAR/INÉDITA) – Considering the use of verb tenses, mark the alternative that
completes the sentence below correctly: Personal-space enthusiasts
A) Expected that some habits created in the pandemic will continue to prevail
B) Have expected that some habits created in the pandemic will continue to prevail
C) Will expect that some habits created in the pandemic will continue to prevail
D) Expect that some habits created in the pandemic will continue to prevail

Questão 09 (EPCAR/INÉDITA) – We can deduce from the eighth paragraph that


A) Humans are comfortable with hugs just for psychological reasons
B) Few humans are comforted by hugs, according to scientific data
C) The comfort that a hug brings is scientifically proven
D) It is scientifically proven that hugging stimulates stress

Questão 10 (EPCAR/INÉDITA) – The word “they” (paragraph 9) refers to


A) Some
B) Hugs
C) Kids
D) Humans

QUESTÕES ESA
Read the text and answer questions 01, 02 and 03
What You Need to Know About COVID-19 and Flu
Every winter is a bit of a roulette wheel when it comes to influenza. Flu vaccines work, but aren’t
100% effective in preventing disease, so it’s always a challenge convincing people to get their flu
shots. And while the symptoms are generally bearable, infections can become more severe and
even deadly among people who are older or who have underlying health conditions. Last flu
season, even though experts considered it a relatively mild year, about 400,000 people in the U.S.
were hospitalized and 22,000 people died from the flu.
This winter, the influenza virus has a rival—the coronavirus fueling the COVID-19 pandemic—and
health officials are anticipating a showdown that could have dire consequences for the health of
millions. Both diseases are caused by viruses that spread with abandon from person to person
through sneezes, coughs, and respiratory droplets during close contact.
But while researchers know quite a bit about the influenza virus, the coronavirus, SARS-CoV-2, is
very much a black box, and they can only guess what will happen when the two pathogens collide
throughout the world.
Why is it dangerous to have flu and COVID-19 viruses around at the same time?

AULA 05 – PRONOUNS AND PREPOSITIONS 44


TEACHER ANDREA BELO

A double whammy of flu and SARS-CoV-2 infections this winter could be devastating for public
health, as the colder weather brings people into closer contact with each other in confined spaces
indoors. “What I worry about is that both viruses can cause serious respiratory illness,” says Dr.
David Chokshi, health commissioner for New York City. “In the same way that COVID-19 often gets
to the point where someone has to be hospitalized to support their breathing, or needs to be in
the ICU with a breathing tube or ventilator, unfortunately we see that in the most severe cases of
influenza as well. What we worry about is that happening at the same time and really stressing
the capacity of our hospitals.”
Public health experts are concerned that too many people who are seriously ill with influenza or
COVID-19 could flood hospitals and stretch already worn out health care workers, and health care
systems, to their limits.
(Adapted from https://time.com/5917061/covid-19-flu-season/)

Questão 01 (ESA/INÉDITA) – Concerning the information in the text, is correct to state that
(A) The flu vaccine is not homogeneously accepted, considering its effectiveness.
(B) Flu symptoms are usually severe and unbearable.
(C) The flu always develops to a more severe condition, in all those infected.
(D) The numbers of hospitalizations and deaths from influenza are minimal.
(E) Those who are older or with health problems are less susceptible to getting the flu.

Questão 02 (ESA/INÉDITA) – In the sentence “This winter, the influenza virus has a rival—the
coronavirus fueling the COVID-19 pandemic—and health officials are anticipating a showdown
that could have dire consequences for the health of millions” (paragraph 2), the word dire
means
(A) Good.
(B) Mild.
(C) Drastic.
(D) Nice.
(E) Partial.

Questão 03 (ESA/INÉDITA) – The words CAUSED and SUPPORT are


(A) Irregular verbs.
(B) Modal verbs.
(C) Not verbs.
(D) Regular verbs.
(E) From different grammatical classes.

AULA 05 – PRONOUNS AND PREPOSITIONS 45


TEACHER ANDREA BELO

Read the text and answer questions 04 and 05


Nine Tools for Better, Longer Sleep
For many of us, a restful night of sleep is hard to come by under the best of circumstances. But
against the backdrop of a long, cold winter and the ongoing pandemic, it can feel almost
impossible. While there will always be newfangled gadgets that claim to solve your sleep
problems with advanced technology — yes, there are such things as sleep robots and sleep-
tracking rings — newer doesn’t always mean better. Wirecutter, the New York Times Company
__________ (1) reviews and recommends products, has tested countless items __________ (2)
find real, proven solutions, from blackout curtains and sleep masks to better pillows and white-
noise machines.
In addition to the products they test for work, Wirecutter writers and editors have found their
own sleep solutions for these overworked, overstressed times. These aren’t necessarily products
we’ve rigorously tested (unless we’re talking about meditation apps), but they are ___(3) things
Wirecutter staffers are finding useful at home for falling, and staying, asleep.
(Adapted from https://www.nytimes.com/2021/03/09/realestate/nine-tools-for-better-longer-sleep.html)

Questão 04 (ESA/INÉDITA) – Concerning the information in the text, is correct to say that
(A) A good night's sleep has been recurring in this pandemic.
(B) Pandemic and winter are factors that contribute to a good night's sleep.
(C) There is no technology developed to provide a good night's sleep.
(D) Meditation apps have not been tested by The New York Times team.
(E) The purpose of the text is to say that a team from The New York Times evaluated services
that help us to have a better night's sleep.

Questão 05 (ESA/INÉDITA) – In the sentence “While there will always be newfangled gadgets
that claim to solve your sleep problems with advanced technology…” (paragraph 1), the word
claim means
(A) Allege.
(B) Justify.
(C) Deny.
(D) Insist.
(E) Abandon.

AULA 05 – PRONOUNS AND PREPOSITIONS 46


TEACHER ANDREA BELO

QUESTÕES ESCOLA NAVAL


Based on the text below, answer questions 01 to 08.
If you bought a dog during lockdown, they'll need help coming out of it
Despite the lows of the past year, for many, lockdown has been the perfect opportunity to
welcome a dog into their lives. The demand for dogs during the pandemic has been huge, with a
60% increase in calls from people seeking to adopt from the Dogs Trust charity, and with many
other rescue organisations reporting similar findings. Google searches for “buy a puppy”
increased by 115% after the UK first went into lockdown in March 2020, with prices for some of
the most soughtafter breeds reaching record levels.
Dog ownership is a wonderful thing, but it is also a huge responsibility and a commitment that
spans way beyond lockdown: as the saying goes, “A dog is for life”. As restrictions ease and the
resumption of normality begins, it’s important we consider the implications for our canine
companions and give them a hand to help them adjust.
Having a dog around has helped many people cope with lockdown. Our dogs mostly love us being
around too: going for longer walks, having more playtime, and resting by our side. Nevertheless,
it’s safe to say life has not been normal for our dogs for most of the past year. Few have met other
dogs, and if they have seen them, it would have been from afar or on a lead, meaning that they
were unable to interact or play. There have also been fewer visitors coming into the home, but
probably more deliveries, with people coming to the door carrying parcels and going away again.
This is all particularly concerning for puppies acquired during the pandemic, as their expectation
of “normal” is lockdown life, and they may never have seen visitors inside the house or have been
left home alone.
We are all longing for a great British summer in which we can go on dog-date walks with a friend
and their dog, have family round for garden barbecues and take our pooches to the pub or cafe,
and of course, we need our dogs to be able to cope calmly with all of that. A return to normal is
something humans are able to process, understand and prepare for. But our dogs – especially
young ones – won’t understand why everything has changed. As far as our dogs know, normality
for them has been enjoying time with family only – so to be expected to cope with groups of
people, children and other dogs, both in and out of the home, could be overwhelming for them.
A big worry for dog owners is the long-term impact of lockdown on their ability to cope with being
left at home on their own. Dogs who had separation anxiety before the lockdown are likely to get
worse when left again as owners head back to work – but we also expect to see new cases
developing, because other dogs, and particularly puppies, have learned to expect company all
day.
One of the biggest reasons why dogs are rehomed is because of behaviour-related issues. A rise
in problematic behaviours after lockdown could mean families have no other option but to give
up their dog. And, really sadly, most of these problems can be prevented with the right early
experiences.
Our message to owners is to start preparing now, rather than waiting until things return more to
normal. It’s easy to do: start building up experiences of all the things we will expect them to do

AULA 05 – PRONOUNS AND PREPOSITIONS 47


TEACHER ANDREA BELO

once the lockdown eases. For instance, start building in minimal periods apart, initially just being
briefly separated from you by a door or child gate. If they stay calm, build up the time separated
really gradually, so they start to adjust to not being with you all the time. If your dog gets worried
when separated – barking, whining, panting, or scratching at the door – you have progressed too
fast. Go back to a shorter period to help them adjust. By gradually increasing your time apart, you
can ensure they are able to settle on their own and help them prepare for the time when you
need to return to work or study.
Our dogs will also need help when it comes to seeing friends and family, both outdoors and
indoors. Teaching your dog how to greet new people calmly, how to settle when guests visit or
when you’re in a cafe are key skills. Coming back when called, walking on a loose lead and not
barking when the doorbell goes are also vital skills that will set them up for success.
It is much easier to prevent problems than treat them, and it’s not too late to help prepare your
dogs for the changes coming. To support dog owners, there is online training, so dogs and their
owners can equip themselves with the skills they can put into practice as normality resumes.
When people take on the responsibilities of dog ownership, they do so with the best intentions
to care for them in the long term. But the pandemic will have devastating effects on some people’s
lives, including their ability to care for the ir dogs. While we provide lots of support to help keep
dogs and owners together, we’re also here for when things aren’t going so well, and owners may
be having trouble seeing a future with their dog. If anyone does find themselves struggling, and
may be considering having to rehome their dog, please contact Dogs Trust and we will do
everything we can to help.
(Adapted from https://www.theguardian.com/commentisfree/2021/apr/05/bought-dog-lockdown)

Questão 01 (ESCOLA NAVAL/INÉDITA) – According to the text, which option is correct?


(A) the demand for new dogs as pets shrank over the past year.
(B) for the author, lockdown has been the ideal moment for getting a new dog.
(C) a report shows that people prefer buying breed dogs to adopting stray dogs.
(D) coming out of lockdown life may have serious consequences for dogs.
(E) Google searches by people wishing to adopt dogs have increased.

Questão 02 (ESCOLA NAVAL/INÉDITA) – What’s the meaning of the word “seeking” in


paragraph 1?
(A) deciding
(B) dreaming of
(C) suggesting
(D) asking for
(E) looking for

AULA 05 – PRONOUNS AND PREPOSITIONS 48


TEACHER ANDREA BELO

Questão 03 (ESCOLA NAVAL/INÉDITA) – According to the contents of paragraph 3, among other


reasons, life for dogs has not been ordinary over the past year because they:
(A) could meet other dogs and play with them.
(B) had to get used to more people visiting them.
(C) haven’t been able to interact with other dogs.
(D) were able to rest by their owners’ side.
(E) were left home alone more often than usual.

Questão 04 (ESCOLA NAVAL/INÉDITA) – In paragraph 2, the words “them” refer to


(A) canine companions.
(B) restrictions.
(C) implications.
(D) dog owners.
(E) people.

Questão 05 (ESCOLA NAVAL/INÉDITA) – According to paragraph 4, it is correct to say that dogs


might have trouble understanding why life has changed because:
(A) they expect to interact with more groups of people.
(B) normality for them has been staying home with family.
(C) they don’ want to process the return to normality.
(D) their owners will stop going on dog-date walks with friends.
(E) they’re looking forward to meeting children again.

Questão 06 (ESCOLA NAVAL/INÉDITA) – In the sentence “A big worry for dog owners is the
longterm impact of lockdown on their ability to cope with being left at home on their own.”
(paragraph 5), the underlined words mean __________.
(A) understand.
(B) handle.
(C) cooperate.
(D) forget.
(E) deny.

AULA 05 – PRONOUNS AND PREPOSITIONS 49


TEACHER ANDREA BELO

Questão 07 (ESCOLA NAVAL/INÉDITA) – What does the quantifier “many” refer to in the
excerpt “Despite the lows of the past year, for many, lockdown has been the perfect
opportunity to welcome a dog into their lives.” (paragraph 1)?
(A) lows.
(B) people.
(C) dogs.
(D) lives.
(E) opportunities.

Questão 08 (ESCOLA NAVAL/INÉDITA) – According to the text, which option is correct?


(A) now it is too late to prepare dogs for the changes ahead.
(B) barking when the doorbell goes is a desirable dog reaction.
(C) when people decide to have a dog, they have mean intentions.
(D) it is not possible to avoid dog behavioral problems.
(E) dogs must be taught how to deal with the changes they will face.

Questão 09 (ESCOLA NAVAL/INÉDITA) – HOW __________ TIMES A WEEK SHOULD YOU EXERCISE
AND HOW __________ FOR? It's crucial to factor in rest days, too.
(https://www.independent.co.uk/life-style/health-and-families/)

Which words best complete the question above?


(A) many / long
(B) much / long
(C) many / far
(D) often / high
(E) often / far

Questão 10 (ESCOLA NAVAL/INÉDITA) – Which option completes the text below correctly?
__________ active is crucial for __________ healthy and, considering 41 per cent of Britons aged
40-60 fail to walk for even ten minutes a month, according to a 2017 study, you might be inclined
to up your activity levels.
(https://www.independent.co.uk/life-style/health-and-families/)

AULA 05 – PRONOUNS AND PREPOSITIONS 50


TEACHER ANDREA BELO

(A) To keep / stay


(B) To keeping / staying
(C) Keeping / staying
(D) Keeping / stay
(E) To keep / to stay

QUESTÕES EsPCEx
Leia o texto a seguir e responda às questões 01, 02 e 03.
Eight States Are Seeding Clouds to Overcome Megadrought

The mountaintops rumble to life unnaturally each year as snow clouds darken the sky across the
West. Open flames burst from the throats of metal chimneys, mounted on squat towers nestled
among the peaks. With a low hiss, puffs of particles belch from their mouths into the air, where
the wind catches them and whisks them away. These aren’t ordinary particles. They’re tiny bits of
crushed-up silver iodide, a crystal-like photosensitive substance once used in photography. But it’s
not used to take pictures out in the mountains. It’s meant to make snow.
As the wind whips the particles across the mountaintops, drafts of air sweep them higher into the
sky—so high that some of them eventually touch the clouds. There, an elegant transformation
takes place. The crystalline iodide particles have a structure similar to ice—and inside a cloud, like
attracts like. Water droplets begin to cluster around the particles, freezing solid as they gather
together.

These frozen clusters eventually grow too heavy to stay in the air. They fall from the cloud and drift
gently toward the Earth, dusting the mountaintops with fresh snow. This is not a page from a
science fiction novel. “Cloud seeding” is a real practice—in fact, it’s been around for decades.
It’s used today to boost precipitation in at least eight states across the western U.S. and dozens of
countries around the world. Interest in cloud seeding is growing as temperatures steadily rise,
increasing drought risks in places like the Mountain West. But there’s a catch. Scientists aren’t sure
how well cloud seeding works today, let alone in a warmer climate.
Amid growing concerns about water resources in the western U.S., scientists are working to
answer those questions.

Today, cloud seeding research represents the cutting edge of weather and climate science—a
convergence of questions about the influence of warming on our dwindling water resources and
our ability to control those consequences.
Adapted from https://www.scientificamerican.com/article/eight-states-are-seeding-clouds-to-overcome-megadrought/

AULA 05 – PRONOUNS AND PREPOSITIONS 51


TEACHER ANDREA BELO

Questão 01 (EsPCEx/INÉDITA) – Which text structure is mainly used in paragraphs 1, 2 and 3 to


create a scenario?
(A) persuasion
(B) literary text
(C) first person perspective
(D) description
(E) fiction

Questão 02 (EsPCEx/INÉDITA) – Choose the alternative that correctly substitutes the word
toward in the sentence “They fall from the cloud and drift gently toward the Earth, dusting the
mountaintops with fresh snow.” (paragraph 3).
(A) across from
(B) in the direction of
(C) next to
(D) alongside
(E) within

Questão 03 (EsPCEx/INÉDITA) – Choose the alternative that correctly substitutes the word
toward in the sentence “They fall from the cloud and drift gently toward the Earth, dusting the
mountaintops with fresh snow.” (paragraph 3).
(A) silver iodide is a photosensitive substance only used in photography.
(B) the crystalline iodide particles and ice have an exactly alike structure.
(C) “cloud seeding” might become reality anytime soon.
(D) “cloud seeding” is used to shrink precipitation in some countries around the world.
(E) scientists don’t know how exactly well cloud seeding works nowadays.

Leia o texto a seguir e responda às questões 04, 05 e 06.


Impact of Covid-19 Pandemic on Mental Health
Aside from killing nearly 400,000 Americans to date and wreaking havoc on the country’s
economy, the COVID-19 pandemic is also taking a heavy toll on mental health. That’s according to
data compiled by the U.S. Census Bureau and the National Center for Health Statistics, showing
that more than 4 in 10 U.S. adults had developed symptoms of depression or anxiety by the end
of 2020, a sharp increase over the results of a comparable survey conducted in the first half of
2019.
As the survey shows, the share of respondents showing signs of anxiety or depression has nearly
quadrupled compared to results obtained before the pandemic. As hundreds of thousands have

AULA 05 – PRONOUNS AND PREPOSITIONS 52


TEACHER ANDREA BELO

died and millions have lost their jobs, Americans are facing a plethora of uncertainties with respect
to their and their families’ health and financial wellbeing, worries which are only exacerbated
when dealt with alone amid a time of social distancing.
Adapted from https://www.statista.com/chart/21878/.

Questão 04 (EsPCEx/INÉDITA) – According to the text, the pandemic


(A) a) makes a mild difference in mental health.
(B) b) caused a spike in anxiety and depression.
(C) c) is no longer related to mental health.
(D) d) decreased the share of Americans showing signs of anxiety or depression.
(E) e) has a meaningless impact on mental health.

Questão 05 (EsPCEx/INÉDITA) – Choose the alternative containing the excerpt from the text that
best summarizes it.
(A) a) “… a comparable survey conducted in the first half of 2019”.
(B) b) “… when dealt with alone amid a time of social distancing”.
(C) c) “That’s according to data compiled by the U.S. Census Bureau and the National Center for
Health Statistics…”
(D) d) “As hundreds of thousands have died and millions have lost their jobs…”.
(E) e) “…the COVID-19 pandemic is also taking a heavy toll on mental health”.

Questão 06 (EsPCEx/INÉDITA) – In the sentence “Americans are facing a plethora of


uncertainties with respect to their and their families’ health…” (paragraph 2), the word plethora
means
(A) apparition
(B) variant
(C) wealth
(D) abundance
(E) stillness

Leia o texto a seguir e responda às questões 07, 08 e 09.


‘Black Mirror’ Finds Terror, and Soul, in the Machine
“Black Mirror” is hands down the most relevant program of our time, if for no other reason than
how often it can make you wonder if we’re all living in an episode of it. This prescient and
mordantly funny science-fiction anthology is smart enough to be just barely ahead of its time. It

AULA 05 – PRONOUNS AND PREPOSITIONS 53


TEACHER ANDREA BELO

doesn’t imagine interstellar civilizations or postapocalyptic scenarios. Instead, it depicts variations


on a near future transformed by information technology — our world, just a little worse.
“Black Mirror,” created for British television by Charlie Brooker, is a product of the 21st century
and its digital, virtual breakthroughs. It speaks to a culture of people who live virtual second lives
on social platforms. So, it’s concerned not with body snatchers but with the internet hive mind;
not nuclear winter but artificial intelligence; not the complications of time travel but the
implications of _______ able to offload human consciousness onto devices. Its view of technology
is not cold and robotic but deeply emotional, because — as with our smartphones — we’ve made
the machines extensions of our bodies and souls.
Typical of the Netflix large-portions ethos, a few new episodes are too long, and feel diluted
compared with the lapidary early seasons. Still, “Black Mirror” hasn’t lost its currency. Its title
refers to the glass screens of computers, tablets and phones, but the machines are not the danger
here: it’s the anonymous, antiseptic monstrousness they can empower.
The brilliance of “Black Mirror” is that it’s not about how technology imperils our humanity. It’s
about the all-too-human faces reflected in our own black mirrors, staring back at us.
Adapted from https://www.nytimes.com/2016/10/21/arts/television/review-black-mirror-finds-terror-and-soul-in-the-machine.html.

Questão 07 (EsPCEx/INÉDITA) – Choose the alternative containing the correct verb form to
complete the gap in the text.
(A) be
(B) been
(C) being
(D) was
(E) were

Questão 08 (EsPCEx/INÉDITA) – According to the text, choose the correct statement.


(A) despite “Black Mirror” being a program about technology, it can’t be described as
emotionless.
(B) the program title shows no relation whatsoever to the show’s main theme.
(C) the word “prescient” (first paragraph) is synonymous with “fictional”.
(D) the presence of interstellar civilizations and postapocalyptic scenarios in “Black Mirror”
makes it a typical sci-fi tv show.
(E) the possessive “its”, in bold in the second paragraph, refers to “Black Mirror”

AULA 05 – PRONOUNS AND PREPOSITIONS 54


TEACHER ANDREA BELO

Questão 09 (EsPCEx/INÉDITA) – In the sentence “‘Black Mirror’ is hands down the most relevant
program of our time, if for no other reason than how often it can make you wonder if we’re all
living in an episode of it.”, the underlined expression means
(A) thoroughly
(B) possibly
(C) undoubtedly
(D) mordantly
(E) equally.

Leia o texto a seguir e responda à questão 10.


Whoooaaa Duuuuude: Why We Stretch Words in Tweets and Texts
On twitter, when a simple ha won’t do, there’s always hahahaaaa, haaaahaaaa, or even
hahahahahahahahahahahahaha, indicating you’ve just read the funniest thing you’ve ever seen.
(Or that you’re a sarcastic talking raccoon.) These are known as stretchable or lengthened words,
and now researchers from the University of Vermont have figured out just how pervasive they are
on Twitter, uncovering fascinating patterns about their use.
Stretchability is a powerful linguistic device that visually punches up a written word, imparting a
wide range of emotions. That goes for the gooooooaaaaaaal of a soccer announcer, a teenager’s
exasperated finallyyyyy, and a surfer’s aweeeeeesome. And booooy are they popular on Twitter.
Writing today in the journal PLOS One, the researchers detail how they combed through 100 billion
tweets, mapping how often these words are stretched, and how far they are elongated—haha
versus hahahahaaaa, for example.
Consider dude and its many formulations. “That can convey basically anything, like ‘Duuuuude,
that's awful,’” says University of Vermont applied mathematician Peter Sheridan Dodds, one of
the study’s coauthors. On the other hand, “Dude!” is different. “It could be excitement; it could
be joy,” says Dodds.
“I hate using exclamation marks because they just don't fit my personality,” I tell Dodds and his
coauthor, Chris Danforth, also an applied mathematician at University of Vermont. But I do stretch
words: “I’ve found myself recently in texts to friends or messages to coworkers doing thaaanks
with three As, to signify some sort of excitement and appreciation without having to use a stupid
exclamation mark.” “Just three?” asks Danforth. “That's restraint. Because two would not work.
Two is like, this person doesn't know how to spell. They've made a mistake.”
All right, sooooo, we use stretchable words all the time to convey extra meaning—sadness, anger,
excitement. And that can be particularly powerful on a platform like Twitter, whose inherent
brevity doesn’t exactly encourage nuanced communication. Those extra letters add some oomph
to a brief message, making it more attention-grabbing. “You're taking what we would think of as
the dictionary text and you're turning it into something visual,” says Danforth. “It can't be ignored
when you see 20 As in a row.”
Adapted from https://www.wired.com/story/whoooaaa-duuuuude-stretch-words/

AULA 05 – PRONOUNS AND PREPOSITIONS 55


TEACHER ANDREA BELO

Questão 10 (EsPCEx/INÉDITA) – When the text brings expressions like whooooole, booooy and
sooooo, it means that
(A) prolonging words in social networks and in text messages is being ironized.
(B) prolonging words when writing them in informal communication is not impacting.
(C) some language codes should be avoided in formal messages.
(D) prolonging words in text messages can create ambiguity.
(E) metalanguage is being applied, by using the code itself to explain it.

AULA 05 – PRONOUNS AND PREPOSITIONS 56


TEACHER ANDREA BELO

GABARITO
GABARITO AFA
01 – D 02 – B 03 – A 04 – C 05 – B
06 – A 07 – C 08 – B 09 – D 10 – A

GABARITO COLÉGIO NAVAL


01 – A 02 – B 03 – D 04 – E 05 – B
06 – A 07 – C 08 – B 09 – C 10 – E

GABARITO EAM
01 – A 02 – D 03 – B 04 – E 05 – B

GABARITO EEAR
01 – A 02 – C 03 – D 04 – B 05 – A
06 – C 07 – B 08 – D 09 – B 10 – A

GABARITO EFOMM
01 – E 02 – B 03 – D 04 – A 05 – B
06 – C 07 – A 08 – E 09 – B 10 – D

GABARITO EPCAR
01 – A 02 – B 03 – D 04 – B 05 – C
06 – A 07 – B 08 – D 09 – C 10 – B

GABARITO ESA
01 – A 02 – C 03 – D 04 – E 05 – A

GABARITO ESCOLA NAVAL


01 – D 02 – E 03 – C 04 – A 05 – B
06 – B 07 – B 08 – E 09 – A 10 – B

GABARITO EsPCEx
01 – D 02 – B 03 – E 04 – B 05 – E
06 – D 07 – C 08 – A 09 – C 10 – E

AULA 05 – PRONOUNS AND PREPOSITIONS 57


TEACHER ANDREA BELO

QUESTÕES COMENTADAS
QUESTÕES AFA
Directions: Read the text below and answer questions 01 to 10 according to it.
Covid won't be the last pandemic. Will we be better prepared for the next one?
Wouldn’t it be wonderful if we never had to face another pandemic? As we reflect on the past
year, the words that come to mind are “never again”. Never again should more than 120,000 lives
be lost to a contagious virus, a number that would have been unthinkable almost a year ago, when
government advisers thought that losing 20,000 people would be a good outcome. Nor should we
endure another year of lockdowns, and the pain of job losses and unemployment that followed.
And nobody ever wants to see children taken out of school for 21 weeks of the year again, unable
to learn in classrooms or see their friends.
But scientists are clear that we will face another pandemic in the future. Most agree the question
is when, not if. In 2018, I gave a talk at the Hay festival outlining a scenario that was loosely based
on recent news reports: a farmer becomes infected with an antibiotic-resistant bacterium from a
pig in China, spreading this infection through their community and then boarding a plane to the
UK. The worrying thing about this scenario is that such bacteria, which evade our antibiotics of
last resort, are already upon us. They could yet be the source of a future pandemic.
Or perhaps the next pandemic will be caused by a virus spilling over from animals to humans,
much like Covid-19. Recent reports that the first humans have been infected with avian influenza
at a poultry plant in Russia give a worrying glimpse of how close we live to such risks. Every time
a virus circulates among animals, particularly bats, rodents, livestock and birds, and then comes
into contact with humans, there’s a chance it will infect people and lead to human transmission.
Wherever the next pandemic comes from, Britain must think long term about its ability to respond
and must ensure it is prepared. Key to this will be working together with other governments to
establish robust response systems. Over the past year, the UK has already built many of the
structures that will be needed to address a future pandemic. We now have a huge testing
capacity, processes in place for fast-tracking vaccine development and protocols for undertaking
rapid research in hospitals. But more needs to be done at a global level to ensure we’re truly
prepared.
(Adapted from https://www.theguardian.com/commentisfree/2021/mar/24/covid-pandemic-prepared-investment-science)

Questão 01 (AFA/INÉDITA) – The first paragraph of the text states that


a) Considering the recent pandemic experience, it would be peaceful and acceptable to go
through another situation like this
b) Although extensive, the pandemic has not reached many deadly victims
c) Unemployment is not relevant in this pandemic context
d) It is common knowledge that the experiences lived in the last year were not interesting for
the population

AULA 05 – PRONOUNS AND PREPOSITIONS 58


TEACHER ANDREA BELO

Comentários: A alternativa A está incorreta. De acordo com o primeiro parágrafo, não é correto
afirmar que considerando a experiência recente da pandemia, seria pacífico e aceitável passar por
outra situação como esta, mas sim, que não seria nem um pouco aceitável para a população em
geral. Isso pode ser confirmado com o trecho “As we reflect on the past year, the words that come
to mind are ‘never again’”.
A alternativa B está incorreta. De acordo com o primeiro parágrafo, não é correto afirmar que
embora extensa, a pandemia não atingiu muitas vítimas mortais, mas sim, que foram, no mínimo,
120 mil mortes pela COVID-19. Isso pode ser confirmado com o trecho “Never again should more
than 120,000 lives be lost to a contagious virus…”.
A alternativa C está incorreta. De acordo com o primeiro parágrafo, não é correto afirmar que o
desemprego não é relevante neste contexto pandêmico, mas sim, que é relevante. Isso pode ser
confirmado com o trecho “Nor should we endure another year of lockdowns, and the pain of job
losses and unemployment that followed”.
A alternativa D está correta. De acordo com o primeiro parágrafo, é correto afirmar que o é de
conhecimento geral que as experiências vividas no último ano não foram interessantes para a
população, assim como esta opção indica. Isso pode ser confirmado com o trecho “And nobody
ever wants to see children taken out of school for 21 weeks of the year again..”.
GABARITO: D

Questão 02 (AFA/INÉDITA) – In the phrase “In 2018, I gave a talk at the Hay festival outlining a
scenario that was loosely based on recent news reports…” (paragraph 2), the underlined word is
a synonym for
a) Informing
b) Describing
c) Summarizing
d) Criticizing
Comentários: A alternativa A está incorreta. A palavra “outlining” significa
delineando/descrevendo e, nesse caso, não é um sinônimo para “informing”, que significa
informando.
A alternativa B está correta. A palavra “outlining” significa delineando/descrevendo e, nesse caso,
é um sinônimo para “describing”, que significa descrevendo.
A alternativa C está incorreta. A palavra “outlining” significa delineando/descrevendo e, nesse
caso, não é um sinônimo para “summarizing”, que significa resumindo.
A alternativa D está incorreta. A palavra “outlining” significa delineando/descrevendo e, nesse
caso, não é um sinônimo para “criticizing”, que significa criticando
GABARITO: B

AULA 05 – PRONOUNS AND PREPOSITIONS 59


TEACHER ANDREA BELO

Questão 03 (AFA/INÉDITA) – The passage “But scientists are clear that we will face another
pandemic in the future. Most agree the question is when, not if” (paragraph 2) suggests that
a) The exact timing of the next pandemic is still unknown
b) Scientists cannot say whether there will be another pandemic, they still wonder that
c) Scientists know that another pandemic will happen in the future, most already know when
d) The pandemic is such an unknown context that scientists are still wondering whether it will
happen and, if so, when
Comentários: A alternativa A está correta. A passagem “Mas os cientistas têm certeza de que
enfrentaremos outra pandemia no futuro. A maioria concorda que a questão é quando, não se”
sugere que o momento exato da próxima pandemia ainda é desconhecido, assim como esta opção
indica.
A alternativa B está incorreta. A passagem “Mas os cientistas têm certeza de que enfrentaremos
outra pandemia no futuro. A maioria concorda que a questão é quando, não se” não sugere que
os cientistas não podem dizer se haverá outra pandemia, eles ainda se perguntam isso, pois os
cientistas estão certos de que isso irá acontecer.
A alternativa C está incorreta. A passagem “Mas os cientistas têm certeza de que enfrentaremos
outra pandemia no futuro. A maioria concorda que a questão é quando, não se” não sugere que
os cientistas sabem que outra pandemia vai acontecer no futuro, a maioria já sabe quando, pois
os cientistas ainda não sabem quando isso irá acontecer.
A alternativa D está incorreta. A passagem “Mas os cientistas têm certeza de que enfrentaremos
outra pandemia no futuro. A maioria concorda que a questão é quando, não se” não sugere que
a pandemia é um contexto tão desconhecido que os cientistas ainda estão se perguntando se isso
vai acontecer e, em caso afirmativo, quando, pois os cientistas estão certos de que irá acontecer,
só não sabem quando.
GABARITO: A

Questão 04 (AFA/INÉDITA) – The second paragraph of the text states that


a) The antibiotics available to the population cover all types of existing bacteria
b) The aforementioned bacterium does not have a transmissible profile
c) The bacterium can cause much greater damage
d) The bacterium has no potential to cause a pandemic like the one we are living in
Comentários: A alternativa A está incorreta. De acordo com o segundo parágrafo, não é correto
afirmar que os antibióticos disponíveis para a população cobrem todos os tipos de bactérias
existentes, mas sim, que não cobrem, e o parágrafo deu um exemplo. Isso pode ser confirmado
com o trecho “...an antibiotic-resistant bacterium from a pig in China...”.
A alternativa B está incorreta. De acordo com o segundo parágrafo, não é correto afirmar que a
bactéria citada não possui perfil transmissível, mas sim, que possui. Isso pode ser confirmado com
o trecho “...a farmer becomes infected with an antibiotic-resistant bacterium from a pig in China,
spreading this infection through their community and then boarding a plane to the UK”.

AULA 05 – PRONOUNS AND PREPOSITIONS 60


TEACHER ANDREA BELO

A alternativa C está correta. De acordo com o segundo parágrafo, é correto afirmar que a bactéria
pode causar danos muito maiores, assim como esta opção indica. Isso pode ser confirmado com
o trecho “They could yet be the source of a future pandemic”.
A alternativa D está incorreta. De acordo com o segundo parágrafo, não é correto afirmar que a
bactéria não tem potencial para causar uma pandemia como a que vivemos, mas sim, que pode
ter. Isso pode ser confirmado com o trecho “They could yet be the source of a future pandemic”.
GABARITO: C

Questão 05 (AFA/INÉDITA) – In the phrase “Or perhaps the next pandemic will be caused by a
virus spilling over from animals to humans, much like Covid-19” (paragraph 3), the underlined
expression is a synonym for
a) Gathering
b) Discharging
c) Hiding
d) Sorting out
Comentários: A alternativa A está incorreta. A expressão “spilling over” significa espalhando e
não é sinônimo para “gathering”, que significa reunindo.
A alternativa B está correta. A expressão “spilling over” significa espalhando e é sinônimo para
“discharging”, que significa descarregando/espalhando.
A alternativa C está incorreta. A expressão “spilling over” significa espalhando e não é sinônimo
para “hiding”, que significa escondendo.
A alternativa D está incorreta. A expressão “spilling over” significa espalhando e não é sinônimo
para “sorting out”, que significa separando.
GABARITO: B

Questão 06 (AFA/INÉDITA) – Mark the alternative which rewrites the sentence below correctly,
in the passive voice “Nor should we endure another year of lockdowns…” (paragraph 1)
a) Nor another year of lockdowns should be endured by us
b) We shouldn’t face another year of lockdowns
c) We need to face another year of lockdowns
d) Another year of lockdowns should not be faced
Comentários: A alternativa A está correta. A opção reescreve a frase dada (Nem devemos
suportar mais um ano de bloqueios) corretamente, na voz passiva; ou seja, “Nem mais um ano de
bloqueios deve ser suportado por nós”.
A alternativa B está incorreta. A opção não reescreve a frase dada (Nem devemos suportar mais
um ano de bloqueios) corretamente, na voz passiva, mas sim, reescreve a frase dada ainda na voz
ativa; ou seja, “Não devemos enfrentar outro ano de bloqueios”.

AULA 05 – PRONOUNS AND PREPOSITIONS 61


TEACHER ANDREA BELO

A alternativa C está incorreta. A opção não reescreve a frase dada (Nem devemos suportar mais
um ano de bloqueios) corretamente, na voz passiva, mas sim, reescreve, incorretamente, a frase
dada ainda na voz ativa; ou seja, “Precisamos enfrentar mais um ano de bloqueios”.
A alternativa D está incorreta. A opção não reescreve a frase dada (Nem devemos suportar mais
um ano de bloqueios) corretamente, na voz passiva. Nesse caso, a opção diz “Mais um ano de
bloqueios não deve ser enfrentado”, sem especificar por quem ele não deve ser enfrentado, assim
como a frase da questão faz.
GABARITO: A

Questão 07 (AFA/INÉDITA) – The passage “Key to this will be working together with other
governments to establish robust response systems” (paragraph 4) suggests that
a) Problem response systems must be developed individually by each government
b) Governments working together does not aim at any major objective
c) The best solution to the problem will be to work together among the countries
d) An individualistic attitude would collaborate to solve the problem
Comentários: A alternativa A está incorreta. A passagem “A chave para isso será trabalhar em
conjunto com outros governos para estabelecer sistemas de resposta robustos” não sugere que
os sistemas de resposta a problemas devem ser desenvolvidos individualmente por cada governo,
pois os mesmos devem trabalhar em conjunto para estabelecer um sistema mais robusto.
A alternativa B está incorreta. A passagem “A chave para isso será trabalhar em conjunto com
outros governos para estabelecer sistemas de resposta robustos” não sugere que governos
trabalhando juntos não visam a nenhum objetivo principal, mas sim, que esses governos
trabalhariam juntos para alcançar um objetivo que seria estabelecer sistemas de resposta mais
robustos.
A alternativa C está correta. A passagem “A chave para isso será trabalhar em conjunto com
outros governos para estabelecer sistemas de resposta robustos” sugere que a melhor solução
para o problema será trabalhar em conjunto entre os países, assim como esta opção indica.
A alternativa D está incorreta. A passagem “A chave para isso será trabalhar em conjunto com
outros governos para estabelecer sistemas de resposta robustos” não sugere que uma atitude
individualista colaboraria para resolver o problema, mas sim, que uma atitude altruísta e coletiva
seria a chave para o problema.
GABARITO: C

Questão 08 (AFA/INÉDITA) – The text


a) Focuses on explaining the starting point of a pandemic
b) Alerts the population about an upcoming pandemic and how it can occur and be resolved
c) Criticizes the individualistic attitude of some countries when dealing with the current
pandemic context
d) Develops the idea that the UK is the best prepared country for a pandemic

AULA 05 – PRONOUNS AND PREPOSITIONS 62


TEACHER ANDREA BELO

Comentários: A alternativa A está incorreta. Não é correto afirmar que o texto concentra-se em
explicar o ponto de partida de uma pandemia, mas sim, que o texto alerta a população sobre uma
próxima pandemia e como ela pode ocorrer e ser resolvida.
Isso pode ser confirmado com o trecho “Wherever the next pandemic comes from, Britain must
think long term about its ability to respond and must ensure it is prepared. Key to this will be
working together with other governments to establish robust response systems”.
A alternativa B está correta. É correto afirmar que o texto alerta a população sobre uma próxima
pandemia e como ela pode ocorrer e ser resolvida, assim como esta opção indica. Isso pode ser
confirmado com o trecho “Wherever the next pandemic comes from, Britain must think long term
about its ability to respond and must ensure it is prepared. Key to this will be working together
with other governments to establish robust response systems”.
A alternativa C está incorreta. Não é correto afirmar que o texto critica a atitude individualista de
alguns países ao lidar com o atual contexto pandêmico, mas sim, que o texto alerta a população
sobre uma próxima pandemia e como ela pode ocorrer e ser resolvida. Isso pode ser confirmado
com o trecho “Wherever the next pandemic comes from, Britain must think long term about its
ability to respond and must ensure it is prepared. Key to this will be working together with other
governments to establish robust response systems”.
A alternativa D está incorreta. Não é correto afirmar que o texto desenvolve a ideia de que o Reino
Unido é o país mais bem preparado para uma pandemia, mas sim, que o texto alerta a população
sobre uma próxima pandemia e como ela pode ocorrer e ser resolvida. Isso pode ser confirmado
com o trecho “Wherever the next pandemic comes from, Britain must think long term about its
ability to respond and must ensure it is prepared. Key to this will be working together with other
governments to establish robust response systems”.
GABARITO: B

Should you be grateful for a job?


It's become a common refrain: “I’m just grateful to have a job”.
The last year has wreaked undeniable havoc on the working world. Globally, the working hours
and income lost in 2020 added up to the equivalent of 255 million full-time jobs. Workplace
closures, layoffs and a steep rise in unemployment are enough to make anyone who’s managed
to hold onto their job feel some measure of gratitude – or, at least, pressure to be grateful.
That pressure pre-dates the pandemic. One of the most pervasive conversations around jobs is
that we should be thankful to be hired, especially when competition for a position is fierce.
Candidates are even expected to express the sentiment if they want to be hired in the first place:
it’s hard to imagine leaving an interview without saying how much you appreciate being
considered, or sending a thank-you email.
But it’s possible some of that gratitude is misplaced. Perhaps it’s not quite appropriate to be
thankful that an employer is ‘letting you’ work for them. And while gratitude can be objectively
good for you – research consistently associates giving thanks with increased happiness – it also
has a darker side that can make you more willing to put up with a situation that makes you
unhappy.

AULA 05 – PRONOUNS AND PREPOSITIONS 63


TEACHER ANDREA BELO

Differing obligations
Some workers may be much more inclined to feel grateful for their jobs than others.
Workers who expect to be hired or promoted may express less gratitude than those without
systemic advantages. This is often the case for white men, who experience more upward mobility
than other groups, and less bias that prevents them from securing jobs, or getting interviews in
the first place. For instance, multiple studies have shown résumés with “white-sounding” names,
and those that downplay racial cues, are significantly more likely to garner a response.
Imposter syndrome may also play a part: workers who aren’t confident they deserve their roles
may develop feelings of unworthiness, despite being qualified or skilled. Women are particularly
vulnerable to imposter syndrome, and may find themselves giving outsize thanks for their jobs.
And, in recent months, Latino and black Americans were significantly more likely to be affected
by pandemic-related lay-offs than white Americans. Those among these groups who have kept
their jobs are likely feeling pressure to express gratitude – even if they have to force it, and even
if their workplace doesn’t inspire much to be thankful for.
(Adapted from https://www.bbc.com/worklife/article/20210329-should-you-be-grateful-for-a-job)

Questão 09 (AFA/INÉDITA) – In the phrase “The last year has wreaked undeniable havoc on the
working world” (paragraph 2), the underlined word is a synonym for
a) Harmony
b) Gains
c) Losses
d) Damage
Comentários: A alternativa A está incorreta. A palavra “havoc” significa destruição/estrago e,
nesse caso, não é sinônimo de “harmony”, que significa harmonia.
A alternativa B está incorreta. A palavra “havoc” significa destruição/estrago e, nesse caso, não é
sinônimo de “gains”, que significa ganhos.
A alternativa C está incorreta. A palavra “havoc” significa destruição/estrago e, nesse caso, não é
sinônimo de “losses”, que significa perdas.
A alternativa D está correta. A palavra “havoc” significa destruição/estrago e, nesse caso, é
sinônimo de “damage”, que significa danos/estrago.
GABARITO: D

Questão 10 (AFA/INÉDITA) – The second paragraph of the text states that


a) A large part of the population has suffered the consequences of the pandemic in the
workplace
b) The pressure to feel grateful to have a job did not arise in the pandemic
c) 2020 was a difficult and exhausting year, but it provided many gains for the working world
d) The feeling of gratitude is always something genuine on the part of the worker, towards his
employer

AULA 05 – PRONOUNS AND PREPOSITIONS 64


TEACHER ANDREA BELO

Comentários: A alternativa A está correta. De acordo com o segundo parágrafo, é correto afirmar
que grande parte da população sofreu as consequências da pandemia no local de trabalho, assim
como esta opção indica. Isso pode ser confirmado com o trecho “The last year has wreaked
undeniable havoc on the working world. Globally, the working hours and income lost in 2020
added up to the equivalent of 255 million full-time jobs”.
A alternativa B está incorreta. De acordo com o segundo parágrafo, não é correto afirmar que a
pressão para se sentir grato por ter um emprego não surgiu na pandemia, pois esta afirmação
aparece no terceiro parágrafo. Isso pode ser confirmado com o trecho “That pressure pre-dates
the pandemic”.
A alternativa C está incorreta. De acordo com o segundo parágrafo, não é correto afirmar que
2020 foi um ano difícil e exaustivo, mas proporcionou muitos ganhos para o mundo do trabalho,
mas sim, que causou muitos estragos. Isso pode ser confirmado com o trecho “The last year has
wreaked undeniable havoc on the working world”.
A alternativa D está incorreta. De acordo com o segundo parágrafo, não é correto afirmar que o
sentimento de gratidão é sempre algo genuíno por parte do trabalhador, para com o seu
empregador, mas sim, que pode ocorrer uma pressão para isso ocorrer. Isso pode ser confirmado
com o trecho “...to make anyone who’s managed to hold onto their job feel some measure of
gratitude – or, at least, pressure to be grateful”.
GABARITO: A

QUESTÕES COLÉGIO NAVAL


Read Text I to do questions 01 to 04 based on it.
Covid: 'Israel may be reaching herd immunity'
This happens when enough of a population has protection against an infection that it stops being
able to spread – and even people who don't themselves have immunity are indirectly protected.
For Covid the estimated threshold for herd immunity is at least 65%-70%.
But scientists in the UK are more cautious.
Dr Sarah Pitt, a virologist at the University of Brighton, urged "extreme caution" in concluding that
herd immunity had been reached – something she believes will be difficult even at high
vaccination rates.
She said it was still too early to tell: "We need to see whether the cases in Israel continue to fall
and stay at low levels."
Reaching this level of population immunity is important to protect people who can't be vaccinated
or whose immune system is too weak to produce a good, protective response.
In Israel, more than half (5.3 million) its residents have been vaccinated and an additional 830,000
people have tested positive for the virus in the past, which should give them some natural
immunity.
That works out as roughly 68% of the population who are likely to have antibodies in their blood
which can fight off the virus.

AULA 05 – PRONOUNS AND PREPOSITIONS 65


TEACHER ANDREA BELO

Prof Eyal Leshem, a director at Israel's largest hospital, the Sheba Medical Center, said herd
immunity was the "only explanation" for the fact that cases continued to fall even as more
restrictions were lifted.
"There is a continuous decline despite returning to near normalcy," he said.
"This tells us that even if a person is infected, most people they meet walking around won't be
infected by them."
And cases are falling in all age groups including children, even though under-16s are not generally
being vaccinated.
(Adapted from https://www.bbc.com/news/health-56722186)

Questão 01 (COLÉGIO NAVAL/INÉDITA) – Read the extract from the text. For Covid the
estimated threshold for herd immunity is at least 65%-70%. Mark the alternative that can
replace the underlined word without changing its meaning.
(A) Brink
(B) Middle
(C) Conclusion
(D) Disadvantage
(E) End
Comentários: A alternativa A está correta. A palavra “threshold” significa limiar/limite e pode ser
substituída pela palavra “brink”, que significa beira/limiar.
A alternativa B está incorreta. A palavra “threshold” significa limiar/limite e não pode ser
substituída pela palavra “middle”, que significa meio.
A alternativa C está incorreta. A palavra “threshold” significa limiar/limite e não pode ser
substituída pela palavra “conclusion”, que significa conclusão.
A alternativa D está incorreta. A palavra “threshold” significa limiar/limite e não pode ser
substituída pela palavra “disadvantage”, que significa desvantagem.
A alternativa E está incorreta. A palavra “threshold” significa limiar/limite e não pode ser
substituída pela palavra “end”, que significa fim.
GABARITO: A

Questão 02 (COLÉGIO NAVAL/INÉDITA) – About herd immunity, it’s correct to affirm that
(A) Is only achieved when the entire population is vaccinated
(B) Not necessarily the whole population needs to be vaccinated to reach it
(C) It only protects those who have already been immunized
(D) For the case of COVID-19, the vaccination threshold is very low
(E) It has already been achieved against COVID-19

AULA 05 – PRONOUNS AND PREPOSITIONS 66


TEACHER ANDREA BELO

Comentários: A alternativa A está incorreta. Sobre a imunidade de rebanho, não é correto afirmar
que só é alcançada quando toda a população é vacinada, mas sim, que acontece quando uma
quantidade suficiente da população é vacinada, capaz de parar com a disseminação do vírus. Isso
pode ser confirmado com o trecho “This happens when enough of a population has protection
against an infection that it stops being able to spread…”.
A alternativa B está correta. Sobre a imunidade de rebanho, é correto afirmar que não
necessariamente toda a população precisa ser vacinada para alcançá-la, assim como esta opção
indica. Isso pode ser confirmado com o trecho “This happens when enough of a population has
protection against an infection that it stops being able to spread…”.
A alternativa C está incorreta. Sobre a imunidade de rebanho, não é correto afirmar que só protege
quem já foi imunizado, mas sim, que protege até aqueles que não foram imunizados, se houver
um número necessário de pessoas imunizadas. Isso pode ser confirmado com o trecho “...and
even people who don't themselves have immunity are indirectly protected”.
A alternativa D está incorreta. Sobre a imunidade de rebanho, não é correto afirmar que para o
caso de COVID-19, o limiar de vacinação é muito baixo, mas sim, que está entre 65 e 70%. Isso
pode ser confirmado com o trecho “For Covid the estimated threshold for herd immunity is at least
65%-70%”.
A alternativa E está incorreta. Sobre a imunidade de rebanho, não é correto afirmar que já foi
alcançado contra COVID-19, mas sim, que ainda não foi. Isso pode ser confirmado com o trecho
“Dr Sarah Pitt, a virologist at the University of Brighton, urged "extreme caution" in concluding
that herd immunity had been reached - something she believes will be difficult even at high
vaccination rates”.
GABARITO: B

Questão 03 (COLÉGIO NAVAL/INÉDITA) – Read the extract from the text: “Reaching this level of
population immunity is important to protect people who can't be vaccinated…”. The sentence
above means that
(A) People who cannot be vaccinated, unfortunately, cannot be protected in any other way
(B) Reaching a certain number of vaccinees does not interfere with the protection of those who
have not been vaccinated
(C) Upon reaching a certain number of vaccinees, those who cannot be vaccinated are protected,
but not as much as those who have been vaccinated
(D) Reaching a certain number of vaccinees, even those who are not vaccinated are protected
(E) Achieving any level of vaccinees ensures the protection of those who have not been
vaccinated
Comentários: A alternativa A está incorreta. A frase “Alcançar esse nível de imunidade
populacional é importante para proteger as pessoas que não podem ser vacinadas” não significa
que pessoas que não podem ser vacinadas, infelizmente, não podem ser protegidas de outra
forma, mas sim, que atingindo um determinado número de vacinados, mesmo aqueles que não
são vacinados ficam protegidos.

AULA 05 – PRONOUNS AND PREPOSITIONS 67


TEACHER ANDREA BELO

A alternativa B está incorreta. A frase “Alcançar esse nível de imunidade populacional é


importante para proteger as pessoas que não podem ser vacinadas” não significa que atingir um
determinado número de vacinados não interfere na proteção daqueles que não foram vacinados,
mas sim, que atingindo um determinado número de vacinados, mesmo aqueles que não são
vacinados ficam protegidos.
A alternativa C está incorreta. A frase “Alcançar esse nível de imunidade populacional é importante
para proteger as pessoas que não podem ser vacinadas” não significa que ao atingir um
determinado número de vacinados, os que não podem ser vacinados ficam protegidos, mas não
tanto quanto os que foram vacinados, mas sim, que atingindo um determinado número de
vacinados, mesmo aqueles que não são vacinados ficam protegidos.
A alternativa D está correta. A frase “Alcançar esse nível de imunidade populacional é importante
para proteger as pessoas que não podem ser vacinadas” significa que atingindo um determinado
número de vacinados, mesmo aqueles que não são vacinados ficam protegidos, assim como esta
opção indica.
A alternativa E está incorreta. A frase “Alcançar esse nível de imunidade populacional é importante
para proteger as pessoas que não podem ser vacinadas” não significa que alcançar qualquer nível
de vacinados garante a proteção daqueles que não foram vacinados, mas sim, que atingindo um
determinado número de vacinados, mesmo aqueles que não são vacinados ficam protegidos.
GABARITO: D

Questão 04 (COLÉGIO NAVAL/INÉDITA) – What can we infer from the text? Mark the correct
option
(A) The entire Israeli population is already immune to COVID-19
(B) Although many have already been vaccinated, COVID-19 cases continue to increase in Israel
(C) 68% of the Israeli population has already been vaccinated
(D) Israel has already achieved herd immunity against COVID-19
(E) It is possible that the majority of Israel's population is already protected from COVID-19
Comentários: A alternativa A está incorreta. De acordo com o texto, não podemos inferir que toda
a população israelense já está imune ao COVID-19, mas sim, que pelo menos metade da população
já está imune. Isso pode ser confirmado com o trecho “ In Israel, more than half (5.3 million) its
residents have been vaccinated…”.
A alternativa B está incorreta. De acordo com o texto, não podemos inferir que embora muitos já
tenham sido vacinados, os casos de COVID-19 continuam a aumentar em Israel, mas sim, que o
número de casos está diminuindo. Isso pode ser confirmado com o trecho “...cases continued to
fall even as more restrictions were lifted”.
A alternativa C está incorreta. De acordo com o texto, não podemos inferir que 68% da população
israelense já foi vacinada, mas sim, que 68% da população está propensa a já ter anticorpos contra

AULA 05 – PRONOUNS AND PREPOSITIONS 68


TEACHER ANDREA BELO

o corona vírus. Isso pode ser confirmado com o trecho “That works out as roughly 68% of the
population who are likely to have antibodies in their blood which can fight off the virus”.
A alternativa D está incorreta. De acordo com o texto, não podemos inferir que Israel já alcançou
imunidade coletiva contra COVID-19, mas sim, que ainda é muito cedo para afirmar isso. Isso pode
ser confirmado com o trecho “She said it was still too early to tell: ‘We need to see whether the
cases in Israel continue to fall…’”.
A alternativa E está correta. De acordo com o texto, podemos inferir que é possível que a maioria
da população de Israel já esteja protegida do COVID-19, assim como esta opção indica. Isso pode
ser confirmado com o trecho “That works out as roughly 68% of the population who are likely to
have antibodies in their blood which can fight off the virus”.
GABARITO: E

Read the text II to do items 05 to 10.


My Family Wants to Visit This Summer. Is Travel Safe Yet?
Navigating our semi-vaccinated world is full of tough questions like this one. Luckily, the U.S.
Centers for Disease Control and Prevention (CDC) has issued guidance that may help.
According to the CDC, fully vaccinated people can visit indoors and unmasked with low-risk
unvaccinated people from a single household. Recent research suggests getting both doses of the
provides Moderna or Pfizer-BioNTech vaccines about 90% protection against COVID-19 infections.
That means a person fully vaccinated with these shots has a pretty slim chance of infecting an
unvaccinated person, or vice versa.
But it’s important to read the fine print there. Since no vaccine is 100% perfect, you should only
visit with low-risk unvaccinated people—i.e., those who do not have medical conditions or other
factors that would increase their chances of having a severe case of COVID-19 if they somehow
did get infected. And you should only visit with one household of unvaccinated people at a time;
your vaccination status wouldn’t make it any safer for a bunch of unvaccinated people to spend
time together inside and unmasked.
Let’s assume the family members who want to visit meet both of those standards. Great! But
there’s still the question of travel. Right now, the CDC does not recommend non-essential travel
for unvaccinated people.
Even though airplanes have pretty good filtration systems, travelers encounter lots of other people
in fairly tight quarters during the average trip, says Cindy Prins, an associate professor of
epidemiology at the University of Florida. Each of those interactions raises the chances of COVID-
19 exposure. Plus, if your family members were unknowingly infected while traveling, or got
infected during the trip, they could potentially seed new cases in your area.
(Adapted from https://time.com/5954515/summer-travel-covid-vaccination/)

AULA 05 – PRONOUNS AND PREPOSITIONS 69


TEACHER ANDREA BELO

Questão 05 (COLÉGIO NAVAL/INÉDITA) – According to the text, mark the INCORRECT option
(A) The vaccine allows meetings, but with some conditions
(B) Considering the low level of protection provided by vaccines, coexistence must still be
controlled
(C) In addition to protecting against COVID-19, the vaccine prevents the spread of the virus
(D) Vaccines against COVID-19 do not give full protection against the virus
(E) Vaccinated persons should still exercise caution when in contact with other people, especially
those in a high-risk group
Comentários: A alternativa A está incorreta. De acordo com o texto, é correto afirmar que a vacina
permite reuniões, mas com algumas condições; sendo assim uma opção com informações
corretas. Isso pode ser confirmado com o trecho “...fully vaccinated people can visit indoors and
unmasked with low-risk unvaccinated people from a single household”.
A alternativa B está correta. De acordo com o texto, não é correto afirmar que considerando o
baixo nível de proteção proporcionado pelas vacinas, a coexistência ainda deve ser controlada,
mas sim, que as vacinas dão um alto nível de proteção. Isso pode ser confirmado com o trecho
“Recent research suggests getting both doses of the Moderna or Pfizer-BioNTech vaccines provides
about 90% protection against COVID-19 infections”.
A alternativa C está incorreta. De acordo com o texto, é correto afirmar que além de proteger
contra COVID-19, a vacina previne a disseminação do vírus; sendo assim uma opção com
informações corretas. Isso pode ser confirmado com o trecho “That means a person fully
vaccinated with these shots has a pretty slim chance of infecting an unvaccinated person, or vice
versa”.
A alternativa D está incorreta. De acordo com o texto, é correto afirmar que vacinas contra COVID-
19 não conferem proteção total contra o vírus; sendo assim uma opção com informações corretas.
Isso pode ser confirmado com o trecho “But it’s important to read the fine print there. Since no
vaccine is 100% perfect...”.
A alternativa E está incorreta. De acordo com o texto, é correto afirmar que pessoas vacinadas
ainda devem ter cuidado quando em contato com outras pessoas, especialmente aquelas em um
grupo de alto risco; sendo assim uma opção com informações corretas. Isso pode ser confirmado
com o trecho “...you should only visit with low-risk unvaccinated people—i.e., those who do not
have medical conditions or other factors that would increase their chances of having a severe case
of COVID-19 if they somehow did get infected”.
GABARITO: B

Questão 06 (COLÉGIO NAVAL/INÉDITA) – Read the extract from the text


“But it’s important to read the fine print there. Since no vaccine is 100% perfect, you should only
visit with low-riskunvaccinated people…”
Mark the option that can replace the underlined sentence without changing its meaning

AULA 05 – PRONOUNS AND PREPOSITIONS 70


TEACHER ANDREA BELO

(A) People from unvaccinated risk groups should not be visited, considering that the vaccine
cannot be trusted 100%
(B) Even with the full effectiveness of all vaccines, you can only visit unvaccinated people who
are not at risk groups
(C) The total effectiveness of all vaccines allows you to visit anyone, even if not vaccinated
(D) Although vaccines are not 100% effective, you can visit anyone after being vaccinated
(E) Since no vaccine is 100% effective, you should not visit anyone
Comentários: A alternativa A está correta. A frase “Uma vez que nenhuma vacina é 100% perfeita,
você só deve visitar pessoas não vacinadas de baixo risco” pode ser substituída pela frase “Pessoas
de grupos de risco não vacinadas não devem ser visitadas, visto que a vacina não é 100%
confiável”, assim como esta opção indica.
A alternativa B está incorreta. A frase “Uma vez que nenhuma vacina é 100% perfeita, você só
deve visitar pessoas não vacinadas de baixo risco” não pode ser substituída pela frase “Mesmo
com a eficácia total de todas as vacinas, você só pode visitar pessoas não vacinadas que não estão
em grupos de risco”, mas sim, pela frase “Pessoas de grupos de risco não vacinadas não devem
ser visitadas, visto que a vacina não é 100% confiável”.
A alternativa C está incorreta. A frase “Uma vez que nenhuma vacina é 100% perfeita, você só
deve visitar pessoas não vacinadas de baixo risco” não pode ser substituída pela frase “A eficácia
total de todas as vacinas permite que você visite qualquer pessoa, mesmo que não esteja
vacinada”, mas sim, pela frase “Pessoas de grupos de risco não vacinadas não devem ser visitadas,
visto que a vacina não é 100% confiável”.
A alternativa D está incorreta. A frase “Uma vez que nenhuma vacina é 100% perfeita, você só
deve visitar pessoas não vacinadas de baixo risco” não pode ser substituída pela frase “Embora as
vacinas não sejam 100% eficazes, você pode visitar qualquer pessoa após ser vacinado”, mas sim,
pela frase “Pessoas de grupos de risco não vacinadas não devem ser visitadas, visto que a vacina
não é 100% confiável”.
A alternativa E está incorreta. A frase “Uma vez que nenhuma vacina é 100% perfeita, você só
deve visitar pessoas não vacinadas de baixo risco” não pode ser substituída pela frase “Uma vez
que nenhuma vacina é 100% eficaz, você não deve visitar ninguém”, mas sim, pela frase “Pessoas
de grupos de risco não vacinadas não devem ser visitadas, visto que a vacina não é 100%
confiável”.
GABARITO: A

Questão 07 (COLÉGIO NAVAL/INÉDITA) – Read the extract from the text


“Plus, if your family members were unknowingly infected while traveling, or got infected during
the trip, they could potentially seed new cases in your area” (paragraph 5)
The word “they” refers to
(A) New cases
(B) Airplanes
(C) Family members
(D) Interactions
(E) Filtration systems

AULA 05 – PRONOUNS AND PREPOSITIONS 71


TEACHER ANDREA BELO

Comentários: A alternativa A está incorreta. A palavra “They” (eles) não se refere aos novos casos,
mas sim, aos membros da família, que são citados anteriormente na frase. Isso pode ser
confirmado com o trecho “Além disso, se os membros da sua família foram infectados
inadvertidamente durante uma viagem, ou foram infectados durante a viagem, eles podem
potencialmente semear novos casos na sua área”.
A alternativa B está incorreta. A palavra “They” (eles) não se refere aos aviões, mas sim, aos
membros da família, que são citados anteriormente na frase. Isso pode ser confirmado com o
trecho “Além disso, se os membros da sua família foram infectados inadvertidamente durante
uma viagem, ou foram infectados durante a viagem, eles podem potencialmente semear novos
casos na sua área”.
A alternativa C está correta. A palavra “They” (eles) se refere aos membros da família, que são
citados anteriormente na frase, assim como esta opção indica. Isso pode ser confirmado com o
trecho “Além disso, se os membros da sua família foram infectados inadvertidamente durante
uma viagem, ou foram infectados durante a viagem, eles podem potencialmente semear novos
casos na sua área”.
A alternativa D está incorreta. A palavra “They” (eles) não se refere às interações, mas sim, aos
membros da família, que são citados anteriormente na frase. Isso pode ser confirmado com o
trecho “Além disso, se os membros da sua família foram infectados inadvertidamente durante
uma viagem, ou foram infectados durante a viagem, eles podem potencialmente semear novos
casos na sua área”.
A alternativa E está incorreta. A palavra “They” (eles) não se refere aos sistemas de filtração, mas
sim, aos membros da família, que são citados anteriormente na frase. Isso pode ser confirmado
com o trecho “Além disso, se os membros da sua família foram infectados inadvertidamente
durante uma viagem, ou foram infectados durante a viagem, eles podem potencialmente semear
novos casos na sua área”.
GABARITO: C

Questão 08 (COLÉGIO NAVAL/INÉDITA) – Moderna and Pfizer-BioNTech vacines


(A) Offer a very low level of protection
(B) Offer a high level of protection against COVID-19 when taken in two doses
(C) Do not have any type of measurement regarding the level of protection offered
(D) Offer a high level of protection even with just one dose
(E) Offer total protection against COVID-19
Comentários: A alternativa A está incorreta. Sobre as vacinas Moderna e Pfizer-BioNTech, não é
correto afirmar que oferecem um nível de proteção muito baixo, mas sim, que quando tomadas
as duas doses, elas oferecem 90% de proteção contra a COVID-19. Isso pode ser confirmado com
o trecho “Recent research suggests getting both doses of the provides Moderna or Pfizer-BioNTech
vaccines about 90% protection against COVID-19 infections”.
A alternativa B está correta. Sobre as vacinas Moderna e Pfizer-BioNTech, é correto afirmar que
oferecem um alto nível de proteção contra COVID-19 quando tomado em duas doses, assim como

AULA 05 – PRONOUNS AND PREPOSITIONS 72


TEACHER ANDREA BELO

esta opção indica. Isso pode ser confirmado com o trecho “Recent research suggests getting both
doses of the provides Moderna or Pfizer-BioNTech vaccines about 90% protection against COVID-
19 infections”.
A alternativa C está incorreta. Sobre as vacinas Moderna e Pfizer-BioNTech, não é correto afirmar
que não possui nenhum tipo de medição quanto ao nível de proteção oferecido, mas sim, que
quando tomadas as duas doses, elas oferecem 90% de proteção contra a COVID-19. Isso pode ser
confirmado com o trecho “Recent research suggests getting both doses of the provides Moderna
or Pfizer-BioNTech vaccines about 90% protection against COVID-19 infections”.
A alternativa D está incorreta. Sobre as vacinas Moderna e Pfizer-BioNTech, não é correto afirmar
que oferecem um alto nível de proteção, mesmo com apenas uma dose, mas sim, que quando
tomadas as duas doses, elas oferecem 90% de proteção contra a COVID-19. Isso pode ser
confirmado com o trecho “Recent research suggests getting both doses of the provides Moderna
or Pfizer-BioNTech vaccines about 90% protection against COVID-19 infections”.
A alternativa E está incorreta. Sobre as vacinas Moderna e Pfizer-BioNTech, não é correto afirmar
que oferecem proteção total contra COVID-19, mas sim, que quando tomadas as duas doses, elas
oferecem 90% de proteção contra a COVID-19. Isso pode ser confirmado com o trecho “Recent
research suggests getting both doses of the provides Moderna or Pfizer-BioNTech vaccines about
90% protection against COVID-19 infections”.
GABARITO: B

Questão 09 (COLÉGIO NAVAL/INÉDITA) – The expression “though”, in paragraph 1, means


(A) Easy
(B) Fragile
(C) Hard
(D) Contradictory
(E) Stressful
Comentários: A alternativa A está incorreta. A palavra “though” (difícil/complicado) não significa
“easy” (fácil).
A alternativa B está incorreta. A palavra “though” (difícil/complicado) não significa “fragile”
(frágil).
A alternativa C está correta. A palavra “though” (difícil/complicado) significa “hard” (difícil).
A alternativa D está incorreta. A palavra “though” (difícil/complicado) não significa “contradictory”
(contraditório).
A alternativa E está incorreta. A palavra “though” (difícil/complicado) não significa “stressful”
(estressante).
GABARITO: C

AULA 05 – PRONOUNS AND PREPOSITIONS 73


TEACHER ANDREA BELO

Questão 10 (COLÉGIO NAVAL/INÉDITA) – The word “slim” (paragraph 2), can be replaced by the
word __________ without changing its meaning
(A) Thin
(B) Large
(C) Medium
(D) Delicate
(E) Small
Comentários: A alternativa A está incorreta. A palavra “slim”, nesse caso, significa pequena e não
pode ser substituída pela palavra “thin”, que significa magro(a).
A alternativa B está incorreta. A palavra “slim”, nesse caso, significa pequena e não pode ser
substituída pela palavra “large”, que significa amplo.
A alternativa C está incorreta. A palavra “slim”, nesse caso, significa pequena e não pode ser
substituída pela palavra “medium”, que significa médio.
A alternativa D está incorreta. A palavra “slim”, nesse caso, significa pequena e não pode ser
substituída pela palavra “delicate”, que significa delicado.
A alternativa E está correta. A palavra “slim”, nesse caso, significa pequena e pode ser substituída
pela palavra “small”, que significa pequena.
GABARITO: E

QUESTÕES EAM
Read text I and answer questions 01 and 02
Which processed foods are better than natural?
The language used to describe the foods we eat can have a huge effect on how we perceive them:
"organic", "artisan", "homemade" and "handpicked" foods sound slightly more tempting than the
prosaic "tinned", "rehydrated" or "freeze-dried".
Another adjective that can whet our appetites is "natural", while we tend to associate "processed"
food with long lists of ingredients we can't pronounce. But when it comes to our health – is natural
always better than processed?
Actually, naturalness doesn't automatically mean a food is healthy, says Christina Sadler, manager
at the European Food Information Council and researcher at the University of Surrey.
In fact, natural foods can contain toxins, and minimal processing can in fact make them safer.
Kidney beans, for instance, contain lectins, which can cause vomiting and diarrhoea. They're
removed by soaking the beans in water overnight and then cooking them in boiling water.
Processing also makes cow's milk safe to consume. Milk has been pasteurised since the late 1800s,
in order to kill harmful bacteria. Before this time, it was distributed locally, because there wasn't
good refrigeration in houses.

AULA 05 – PRONOUNS AND PREPOSITIONS 74


TEACHER ANDREA BELO

"Cows in cities were milked every day, and people would bring milk in carts back to their
neighbourhoods to sell it," says John Lucey, food science professor at the University of Wisconsin-
Madison.
"As cities got bigger, milk got further away and took longer to get to the consumer, which meant
pathogens could multiply."
Mounting evidence suggesting that some organisms in milk could be harmful led to the
development of heating devices for milk and the invention of pasteurisation, which was soon
adopted across Europe, and later in the US.
"It's one of the major public health success stories of the last century," Lucey says. "Just before
World War Two, around a quarter of all food and waterborne diseases came from milk. Now it's
less than 1%."
Processing can also help to retain nutrients in food we eat. For example, freezing, which is
classified as minimal processing, allows fruit and vegetables to retain nutrients that can otherwise
degrade while sitting in a fridge.
(Adapted from https://www.bbc.com/future/article/20210521-which-processed-foods-are-better-than-natural)

Questão 01 (EAM/INÉDITA) – Say if the following statements are T (true) or F (false) about
processed and natural foods. Then, mark the correct option, from top to bottom.
( ) The language used to describe the types of food has no relevant impact on their
understanding
( ) "Organic" and "homemade" are repulsive expressions
( ) Sometimes we have a misconception about "processed" food
( ) Healthy food is not necessarily "natural"
( ) Food processing can be beneficial
(A) F – F – T – T – T
(B) F – F – T – T – F
(C) F – T – T – T – T
(D) T – F – T – T – F
(E) T – F – T – T – T
Comentários: A primeira afirmação é falsa (false – F). Não é correto afirmar que a linguagem
utilizada para descrever os tipos de alimentos não tem impacto relevante no seu entendimento,
mas sim, que tem um enorme efeito na percepção do alimento. Isso pode ser confirmado com o
trecho “The language used to describe the foods we eat can have a huge effect on how we
perceive them…”.
A segunda afirmação é falsa (false – F). Não é correto afirmar que "orgânico" e "caseiro" são
expressões repulsivas, mas sim, que são expressões mais tentadoras do que outras relacionadas
a comida processada. Isso pode ser confirmado com o trecho “…’organic’, ‘artisan’, ‘homemade’

AULA 05 – PRONOUNS AND PREPOSITIONS 75


TEACHER ANDREA BELO

and ‘handpicked’ foods sound slightly more tempting than the prosaic ‘tinned’, ‘rehydrated’ or
‘freeze-dried’”.
A terceira afirmação é verdadeira (true – T). É correto afirmar que às vezes temos um conceito
errado sobre alimentos "processados". Isso pode ser confirmado com o trecho “...while we tend
to associate ‘processed’ food with long lists of ingredients we can't pronounce”.
A quarta afirmação é verdadeira (true – T). É correto afirmar que alimentação saudável não é
necessariamente “natural”. Isso pode ser confirmado com o trecho “...naturalness doesn't
automatically mean a food is healthy…”.
A quinta afirmação é verdadeira (true – T). É correto afirmar que o processamento de alimentos
pode ser benéfico. Isso pode ser confirmado com o trecho “...natural foods can contain toxins,
and minimal processing can in fact make them safer”.
GABARITO: A

Questão 02 (EAM/INÉDITA) – About the example of milk given by the text, it is correct to say
that
(A) The increase in cities has not changed the consequences of the consumption of pure milk
(B) Pasteurization of milk did not have as an incentive the pathogens that the product contained
(C) It was safer to consume milk before the invention of processing this food
(D) Milk processing has made it a healthier and more accessible product
(E) Heating devices and pasteurization did not result in better preservation of milk
Comentários: A alternativa A está incorreta. De acordo com o texto e o exemplo sobre leite dado,
não é correto afirmar que o aumento das cidades não mudou as consequências do consumo de
leite puro, mas sim, que mudou pois, desta forma, poderiam haver mais patógenos no alimento.
Isso pode ser confirmado com o trecho “As cities got bigger, milk got further away and took longer
to get to the consumer, which meant pathogens could multiply”.
A alternativa B está incorreta. De acordo com o texto e o exemplo sobre leite dado, não é correto
afirmar que a pasteurização do leite não teve como incentivo os patógenos que o produto
continha, mas sim, que pode ter tido. Isso pode ser confirmado com o trecho “Processing also
makes cow's milk safe to consume. Milk has been pasteurised since the late 1800s, in order to kill
harmful bacteria”.
A alternativa C está incorreta. De acordo com o texto e o exemplo sobre leite dado, não é correto
afirmar que era mais seguro consumir leite antes da invenção do processamento deste alimento,
mas sim, que após essa invenção, consumir o mesmo tornou-se mais seguro. Isso pode ser
confirmado com o trecho “Processing also makes cow's milk safe to consume. Milk has been
pasteurised since the late 1800s, in order to kill harmful bacteria”.
A alternativa D está correta. De acordo com o texto e o exemplo sobre leite dado, é correto
afirmar que o processamento do leite o tornou um produto mais saudável e acessível, assim como
esta opção indica. Isso pode ser confirmado com o trecho “Processing also makes cow's milk safe
to consume. Milk has been pasteurised since the late 1800s, in order to kill harmful bacteria …
around a quarter of all food and waterborne diseases came from milk. Now it's less than 1%…”.

AULA 05 – PRONOUNS AND PREPOSITIONS 76


TEACHER ANDREA BELO

A alternativa E está incorreta. De acordo com o texto e o exemplo sobre leite dado, não é correto
afirmar que dispositivos de aquecimento e pasteurização não resultaram em melhor preservação
do leite, mas sim, que resultaram. Isso pode ser confirmado com o trecho “Processing also makes
cow's milk safe to consume. Milk has been pasteurised since the late 1800s, in order to kill harmful
bacteria. Before this time, it was distributed locally, because there wasn't good refrigeration in
houses”.
GABARITO: D

Read text II and answer questions 03 and 04


CDC studying reports of heart inflammation in young Covid vaccine recipients
Some teenagers and young adults who received Covid-19 vaccines have experienced heart
inflammation, a US Centers for Disease Control and Prevention advisory group said,
recommending further study of the rare condition.
In a statement dated 17 May, the CDC’s advisory committee on immunisation practices said it had
looked into reports that a few young vaccine recipients, predominantly male adolescents and
young adults, developed myocarditis, an inflammation of the heart muscle.
The condition often goes away without complications and can be caused by a variety of viruses,
the CDC group said.
CDC monitoring systems had not found more cases than would be expected in the population,
but members of the committee on vaccinations felt that healthcare providers should be made
aware of the reports of the “potential adverse event”, the committee said in the statement.
It did not say how many people had been affected and recommended further investigation.
(Adapted from https://www.theguardian.com/world/2021/may/23/cdc-studying-reports-of-heart-inflammation-in-young-covid-vaccine-recipients)

Questão 03 (EAM/INÉDITA) – In the sentence “It did not say how many people had been
affected and recommended further investigation” (paragraph 5), the pronoun “It” refers to
(A) CDC monitoring systems
(B) The committee on vaccinations
(C) CDC studying reports
(D) The statement
(E) Healthcare providers
Comentários: A alternativa A está incorreta. O pronome “it” não se refere aos sistemas de
monitoramento da CDC, mas sim, ao comitê de vacinação, citado no parágrafo anterior. Portanto,
“…but members of the committee on vaccinations felt that healthcare providers should be made
aware of the reports of the ‘potential adverse event’, the committee said in the statement … It
did not say how many people had been affected and recommended further investigation”.
A alternativa B está correta. O pronome “it” se refere ao comitê de vacinação, citado no parágrafo
anterior, assim como esta opção indica. Portanto, “…but members of the committee on

AULA 05 – PRONOUNS AND PREPOSITIONS 77


TEACHER ANDREA BELO

vaccinations felt that healthcare providers should be made aware of the reports of the ‘potential
adverse event’, the committee said in the statement … It did not say how many people had been
affected and recommended further investigation”.
A alternativa C está incorreta. O pronome “it” não se refere aos relatórios de estudo da CDC, mas
sim, ao comitê de vacinação, citado no parágrafo anterior. Portanto, “…but members of the
committee on vaccinations felt that healthcare providers should be made aware of the reports of
the ‘potential adverse event’, the committee said in the statement … It did not say how many
people had been affected and recommended further investigation”.
A alternativa D está incorreta. O pronome “it” não se refere a declaração, mas sim, ao comitê de
vacinação, citado no parágrafo anterior. Portanto, “…but members of the committee on
vaccinations felt that healthcare providers should be made aware of the reports of the ‘potential
adverse event’, the committee said in the statement … It did not say how many people had been
affected and recommended further investigation”.
A alternativa E está incorreta. O pronome “it” não se refere aos prestadores de cuidados de saúde,
mas sim, ao comitê de vacinação, citado no parágrafo anterior. Portanto, “…but members of the
committee on vaccinations felt that healthcare providers should be made aware of the reports of
the ‘potential adverse event’, the committee said in the statement … It did not say how many
people had been affected and recommended further investigation”.
GABARITO: B

Questão 04 (EAM/INÉDITA) – It is FALSE to say that


(A) Some people are known to have developed heart inflammation after taking the vaccine
against COVID-19
(B) Heart inflammation is a rare condition in these circumstances
(C) The cause of the inflammation can have several sources
(D) Heart inflammation may be an adverse event related to the COVID-19 vaccine
(E) A considerable number of people who received the vaccine against COVID-19, developed
heart inflammation
Comentários: A alternativa A está incorreta. De acordo com o texto, é correto afirmar que sabe-
se que algumas pessoas desenvolveram inflamação do coração após tomar a vacina contra COVID-
19. Isso pode ser confirmado com o trecho “Some teenagers and young adults who received
Covid-19 vaccines have experienced heart inflammation, a US Centers for Disease Control and
Prevention advisory group said…”.
A alternativa B está incorreta. De acordo com o texto, é correto afirmar que a inflamação do
coração é uma condição rara nessas circunstâncias. Isso pode ser confirmado com o trecho “...a
US Centers for Disease Control and Prevention advisory group said, recommending further study
of the rare condition”.

AULA 05 – PRONOUNS AND PREPOSITIONS 78


TEACHER ANDREA BELO

A alternativa C está incorreta. De acordo com o texto, é correto afirmar que a causa da inflamação
pode ter várias origens. Isso pode ser confirmado com o trecho “The condition often goes away
without complications and can be caused by a variety of viruses…”.
A alternativa D está incorreta. De acordo com o texto, é correto afirmar que a inflamação do
coração pode ser um evento adverso relacionado à vacina COVID-19. Isso pode ser confirmado
com o trecho “…but members of the committee on vaccinations felt that healthcare providers
should be made aware of the reports of the ‘potential adverse event’…”.
A alternativa E está correta. De acordo com o texto, não é correto afirmar que um número
considerável de pessoas que receberam a vacina contra COVID-19 desenvolveu inflamação
cardíaca, mas sim, que foram apenas alguns adolescentes e jovens adultos. Isso pode ser
confirmado com o trecho “Some teenagers and young adults who received Covid-19 vaccines
have experienced heart inflammation…”.
GABARITO: E

Questão 05 (EAM/INÉDITA) – Complete the paragraph below, about the pandemic situation in
Nepal, with the missing prepositions
“As I write this, my country is battling __________ new and brutal wave of the Covid-19 pandemic.
The rise in the number __________ infections poses a serious challenge __________ our brave
doctors, nurses, other care providers, citizen volunteers and the entire health service system”.
(Adapted from https://www.theguardian.com/commentisfree/2021/may/10/nepal-covid-uk-g7)

(A) A / with / for


(B) A / of / to
(C) An / of / to
(D) An / at / for
(E) A / at / to
Comentários: A primeira lacuna deve ser preenchida com “a”, pois a frase se refere a uma nova
e brutal onda da pandemia da COVID-19. Portanto, “...meu país está lutando contra uma nova e
brutal onda da pandemia Covid-19”.
A segunda lacuna deve ser preenchida com “of”, pois a frase se refere ao número de infecções.
Portanto, “O aumento no número de infecções representa...”.
A terceira lacuna deve ser preenchida com “to”, pois a frase se refere ao desafio que vai ser para
os médicos e etc.
Portanto, “...um sério desafio para nossos bravos médicos, enfermeiras, outros prestadores de
cuidados, cidadãos voluntários e todo o sistema de serviços de saúde”.
GABARITO: B

AULA 05 – PRONOUNS AND PREPOSITIONS 79


TEACHER ANDREA BELO

QUESTÕES EEAR
Read the text I to do questions 01 to 10 based on it.
Menthol Cigarettes Kill Smokers, Most of Them Black.
Tobacco companies gain goodwill by advertising to black readers — and by donating money to
civil rights organizations.
Menthol creates a cooling sensation in tobacco products. Some studies have shown that menthol
also acts as a mild anesthetic.
There is a movement to ban menthol cigarettes. Why? The mint-flavored products target black
Americans.
On the whole, black Americans smokers smoke less than white people do. But black smokers die
of causes linked to tobacco use at higher rates than white smokers. About 85 percent of black
smokers use Newport, Kool, and other menthol brands.
It is easy to become addicted to these brands. They also are harder to quit than tobacco without
menthol.
The African American Tobacco Control Leadership Council is for menthol bans. It says menthol
products are the “main vectors” of disease and death among black Americans.
__________ (9) white health advocates oppose flavored e-cigarettes, which include menthol.
There is concern about the use of menthol cigars and cigarillos among black teenagers. Most black
high school students smoke menthol tobacco products.
Tobacco companies target black communities with menthol cigarettes. They give away free
samples and offer discounts.
They sponsor concerts and special events. They advertise in newspapers and magazines geared
to black readership. They donate money to civil rights organizations.
The companies have also been donors to black political candidates. They have been supporters of
the Congressional Black Caucus.
The tobacco companies say they want to be good citizens. But it is hard for them to resist the
chance to make a profit.
_______ (11) friends in Congress work to defeat efforts to ban flavored tobacco products.
A lobbyist said, a menthol ban would infringe on the rights of adults who preferred it to plain
tobacco.
The tobacco industry has joined forces with civil rights activists. Among them is the Rev. Al
Sharpton. Sharpton visited black communities in California. He raised the fear that a menthol ban
would give the police an excuse to stop and frisk blacks.
Mr. Sharpton also helped to defeat a menthol ban in New York.
Observers point out that society is making marijuana legal. In what direction should public policy
go? The tobacco industry also will not go quietly. It is creating new products that are less harmful
to users.
Americans consume alcohol, tobacco and all manner of drugs, some legal, some not. The banning
of one type of cigarette is not efficient. It may not work.
Source: The New York Times March 22, 2021

AULA 05 – PRONOUNS AND PREPOSITIONS 80


TEACHER ANDREA BELO

Questão 01 (EEAR/INÉDITA) – The passage “There is a movement to ban menthol cigarettes.”


states that menthol cigarettes must be ____________ .
a) forbidden.
b) allowed.
c) accepted.
d) recollected.
Comentários: A alternativa A está correta. forbidden = proibido. O verbo “to ban” significa “banir”
portanto na questão diz que cigarros de menta devem ser banidos, ou seja, proibidos.
A alternativa B está incorreta. allowed = permitido.
A alternativa C está incorreta. accepted = aceito.
A alternativa D está incorreta. recollected = recolhido.
GABARITO: A

Questão 02 (EEAR/INÉDITA) – sentence in bold in the text is in the:


a) Past continuous.
b) Present Continuous.
c) Simple Present.
d) Simple Past.
Comentários: A alternativa C está correta. “They donate money to civil rights organizations.” (Eles
doam dinheiro para organizações de direitos civis.). O Simple Present, ou Presente Simples, é
comum ser utilizado para falar de fatos, hábitos ou opiniões.
GABARITO: C

Questão 03 (EEAR/INÉDITA) – Choose the alternative in which you can find the Present
Continuous.
a) Tobacco companies gain goodwill by advertising to black readers.
b) Menthol creates a cooling sensation in tobacco products.
c) The banning of one type of cigarette may not work.
d) Observers point out that society is making marijuana legal.
Comentários: A alternativa A está incorreta. Apesar de aparecer uma palavra com ING, não se
trata de um caso de present continuous. “Advertising” = propaganda / publicidade.
A alternativa B está incorreta. Apesar de aparecer uma palavra com ING, não se trata de um caso
de present continuous. “Cooling” = refriamento.
A alternativa C está incorreta. Apesar de aparecer uma palavra com ING, não se trata de um caso
de present continuous. “Banning” = banimento
A alternativa D está correta. Present Continuous = Verbo To Be + Verbo com ING. “Observers point
out that society IS makING marijuana legal.
GABARITO: D

AULA 05 – PRONOUNS AND PREPOSITIONS 81


TEACHER ANDREA BELO

Questão 04 (EEAR/INÉDITA) – In “They sponsor concerts and special events”, the word
“sponsor” means:
a) to get something by paying money for it.
b) to pay for someone to do something or for something to happen.
c) to get or receive something from someone with the intention of giving it back after a period
of time.
d) to make something clear or easy to understand by describing or giving information about it.
Comentários: A alternativa A está incorreta. “to get something by paying money for it” (obter algo
pagando por isso) = comprar.
A alternativa B está correta. “to pay for someone to do something or for something to happen”
(pagar para que alguém faça algo ou para que algo aconteça) = Patrocinar. O texto diz que
empresas de tabaco patrocinam eventos.
A alternativa C está incorreta. “to get or receive something from someone with the intention of
giving it back after a period of time” (obter ou receber algo de alguém com a intenção de devolvê-
lo após um certo período de tempo) = Pegar emprestado
A alternativa D está incorreta. “to make something clear or easy to understand by describing or
giving information about it” (tornar algo claro ou fácil de entender, descrevendo ou dando
informações a respeito) = Explicar.
GABARITO: B

Questão 05 (EEAR/INÉDITA) – Choose the answer in which the verb "do", in bold, is being used
in the same way as the sentence below. “black Americans smokers smoke less than white
people do”
a) Do you think she will recognize me? She might do.
b) I’m going to do some work in the garden this weekend.
c) I don’t want to wait for a bus. Let’s get a taxi.
d) I do like your new jacket!
Comentários: A alternativa A está correta. “black Americans smokers smoke less than white
people do” (Fumantes americanos negros fumam menos do que os brancos (fumam)). Nesse caso,
o verbo auxiliar está sendo utilizado para substituir um verbo que já apareceu anteriormente.
Observe que o mesmo acontece na alternativa A. “Do you think she will recognize me? She might
do.” (Você acha que ela vai me reconhecer? Ela pode (reconhecer).
A alternativa B está incorreta. Na alternativa B, “do” é o verbo fazer.
A alternativa C está incorreta. Na alternativa C, “do” é o verbo auxiliar do simple present que está
sendo utilizado para formar a frase negativa.
A alternativa D está incorreta. Na alternativa B, “do” está sendo utilizado para expressar uma
ênfase na frase. “I do like your new jacket!” (Eu (realmente) gosto do seu novo casaco!”
GABARITO: A

AULA 05 – PRONOUNS AND PREPOSITIONS 82


TEACHER ANDREA BELO

Questão 06 (EEAR/INÉDITA) – All alternatives are not in the comparative, except:


a) reader.
b) smoker.
c) higher.
d) manner.
Comentários: A alternativa A está incorreta. reader = leitor.
A alternativa B está incorreta. smoker = fumante.
A alternativa C está correta. higher = mais alto. Os Adjetivos que possuem uma sílaba e os que são
dissílabos terminados em -le, -ow e -er formam o comparativo com o acréscimo de -er ao final do
adjetivo.
A alternativa D está incorreta. manner = maneira / modo.
GABARITO: C

Questão 07 (EEAR/INÉDITA) – The adjective form “good” and “easy”, underlined in the text,
have as their comparative forms, respectively:
a) gooder and easyer.
b) better and easier.
c) best and the easiest.
d) better and the easyer.
Comentários: A alternativa B está correta. Good é um caso especial de comparativo e portanto
não segue a regra normal. O comparativo de good é better. Já o adjetivo easy terá como
comparativo easier pois se entrará na seguinte regra: Os Adjetivos que possuem uma sílaba e os
que são dissílabos terminados em -le, -ow e -er formam o comparativo com o acréscimo de -er ao
final do adjetivo. Observe que também que os adjetivos terminados em y precedido de consoante
trocam o y por i ao receber –er.
GABARITO: B

Questão 08 (EEAR/INÉDITA) – In “But it is hard for them to resist the chance to make a profit”,
the word “them” refers to
a) citizens.
b) rights organizations.
c) Congressional Black Caucus.
d) tobacco companies.
Comentários: A alternativa D está correta. “The tobacco companies say they want to be good
citizens. But it is hard for them to resist the chance to make a profit. (As empresas de tabaco
dizem que querem ser bons cidadãos. Mas é difícil para elas (as empresas de tabaco) resistirem à
chance de lucrar. Portanto o pronome them refere-se a “tobacco companies”.
GABARITO: D

AULA 05 – PRONOUNS AND PREPOSITIONS 83


TEACHER ANDREA BELO

Questão 09 (EEAR/INÉDITA) – Fill in the blank with the suitable option:


a) Much.
b) Many.
c) A lots of.
d) Lots.
Comentários: A alternativa A está incorreta. Much deve ser utilizado junto com substantivos
incontáveis singulares.
A alternativa B está correta. Many deve ser utilizado junto com substantivos contáveis no plural.
A alternativa C está incorreta. “A lots of” não existe, o correto seria “a Lot of” ou “lots of”.
A alternativa D está incorreta. “Lots” não existe, o correto seria “a Lot of” ou “lots of”.
GABARITO: B

Questão 10 (EEAR/INÉDITA) – In “It is easy to become addicted to these brands.”, the


underlined word has the same use as in
a) But it is hard for them to resist the chance to make a profit.
b) It says menthol products are the “main vectors” of disease.
c) A lobbyist said, a menthol ban would infringe on the rights of adults who preferred it to plain
tobacco.
d) It is creating new products that are less harmful to users.
Comentários: A alternativa A está correta. No inglês as sentenças têm de ter um sujeito. Quando
em português esse sujeito é impessoal ou inexistente, em inglês devemos usar o “it”. Esse “it”
não tem conteúdo semântico, isto é, ele não significa nada, e não deve ser traduzidoem
português. “It is easy to become addicted to these brands.” = “É fácil ficar viciado nessas marcas.”
“But it is hard for them to resist the chance to make a profit.” = Mas é difícil para eles resistir à
chance de ter lucro.
A alternativa B está incorreta. Nesse caso o “It” é um pronome sujeito e faz referência a “The
African American Tobacco Control Leadership Council” (O Conselho de Liderança para o Controle
do Tabaco Afro-americano).
A alternativa C está incorreta. Nesse caso o “It” é um pronome objeto e faz referência “menthol
cigarette” (cigarro de menta).
A alternativa D está incorreta. Nesse caso o “It” é um pronome sujeito e faz referência a “tobacco
industry” (indústria do tabaco).
GABARITO: A

AULA 05 – PRONOUNS AND PREPOSITIONS 84


TEACHER ANDREA BELO

QUESTÕES EFOMM
Based on the text below, answer questions 01 and 02.
Does motherhood belong on a resume?
Many have advocated that motherhood is a legitimate job that builds employable skills. Does
the title belong on mums' CVs?
Mums multi-task. They plan. They research, organise, negotiate, manage time and lead.
Although mothers’ juggling hasn’t ever been a secret, their role has, perhaps, never been more
obvious than during the pandemic. As schools transitioned to remote classrooms, and women
took on more of both the physical and mental load of home life than before, the skills required to
keep the trains on the tracks have been on full display.
As a result, the question increasingly floating to the surface is whether or not these skills have a
place on mothers’ CVs.
There’s long been a push to recognise motherhood as a legitimate job that trains workers in
legitimate skills, valuable to employers. And some voices are getting louder. One of the newest
leaders is HeyMama, a US-based community for working mums, who’ve launched a campaign
called Motherhood on the Resume. It’s quite literal, says Katya Libin, HeyMama’s co-founder and
CEO – the organisation is advocating for mothers to update their titles on LinkedIn, or even add
the position on a resume, like any other ‘recognised’ job in, say, sales or engineering.
Whether motherhood ‘belongs’ on a resume is, of course, subjective. The question, instead, lies
in whether mothers can reap tangible benefits for the addition of the title – or whether some
systemically entrenched biases around mums could produce the opposite effect.
(Adapted from https://www.bbc.com/worklife/article/20210617-does-motherhood-belong-on-a-resume)

Questão 01 (EFOMM/INÉDITA) – It is possible to infer from the text that


A) Mothers are limited to one task at a time because of their children
B) The pandemic made maternal tasks less clear
C) Maternal skills should not be included in mothers' CVs
D) The struggle for recognition of motherhood as a legitimate job is still very superficial
E) It is possible that adding motherhood to the curriculum could have positive or negative
effects
Comentários: A alternativa A está incorreta. De acordo com o texto, não é correto inferir que as
mães estão limitadas a uma tarefa de cada vez por causa de seus filhos, mas sim, que as mães
realizam diversas tarefas ao mesmo tempo. Isso pode ser confirmado com o trecho “Mums multi-
task. They plan. They research, organise, negotiate, manage time and lead”.
A alternativa B está incorreta. De acordo com o texto, não é correto inferir que a pandemia tornou
as tarefas maternas menos claras, mas sim, que as tornou mais óbvias ainda. Isso pode ser

AULA 05 – PRONOUNS AND PREPOSITIONS 85


TEACHER ANDREA BELO

confirmado com o trecho “Although mothers’ juggling hasn’t ever been a secret, their role has,
perhaps, never been more obvious than during the pandemic”.
A alternativa C está incorreta. De acordo com o texto, não é correto inferir que habilidades
maternas não devem ser incluídas no currículo das mães, mas sim, que estão lutando para que
isso aconteça. Isso pode ser confirmado com o trecho “...the organisation is advocating for
mothers to update their titles on LinkedIn, or even add the position on a resume…”.
A alternativa D está incorreta. De acordo com o texto, não é correto inferir que a luta pelo
reconhecimento da maternidade como trabalho legítimo ainda é muito superficial, mas sim, que
já existe uma comunidade que colabora com essa luta. Isso pode ser confirmado com o trecho
“And some voices are getting louder. One of the newest leaders is HeyMama, a US-based
community for working mums, who’ve launched a campaign called Motherhood on the Resume”.
A alternativa E está correta. De acordo com o texto, é correto inferir que é possível que adicionar
a maternidade ao currículo possa ter efeitos positivos ou negativos, assim como esta opção indica.
Isso pode ser confirmado com o trecho “The question, instead, lies in whether mothers can reap
tangible benefits for the addition of the title – or whether some systemically entrenched biases
around mums could produce the opposite effect”.
GABARITO: E

Questão 02 (EFOMM/INÉDITA) – In the excerpt “...or whether some systemically entrenched


biases around mums could produce the opposite effect”, the word in bold means
A) Changeable
B) Ingrained
C) Obvious
D) Strong
E) Secured
Comentários: A alternativa A está incorreta. A palavra “entrenched” significa arraigado
(enraizado) e não pode ser comparada a palavra “changeable”, que significa mutável.
A alternativa B está correta. A palavra “entrenched” significa arraigado (enraizado) e pode ser
comparada a palavra “ingrained”, que significa arraigado.
A alternativa C está incorreta. A palavra “entrenched” significa arraigado (enraizado) e não pode
ser comparada a palavra “obvious”, que significa óbvio.
A alternativa D está incorreta. A palavra “entrenched” significa arraigado (enraizado) e não pode
ser comparada a palavra “strong”, que significa forte.
A alternativa E está incorreta. A palavra “entrenched” significa arraigado (enraizado) e não pode
ser comparada a palavra “secured”, que significa protegido.
GABARITO: B

AULA 05 – PRONOUNS AND PREPOSITIONS 86


TEACHER ANDREA BELO

Based on the text below, answer questions 03, 04 and 05.


U.S. birthrates are plummeting. Increasing legal immigration can help.
THE DECADES-LONG decline in the U.S. birthrate accelerated in 2020, as the average number of
babies born to American women over the course of their lifetimes fell to its lowest level since
government record-keeping began nearly a century ago. After a decade in which the population
grew at the most sluggish pace since the 1930s, last year’s slowdown in births, which intensified
as the pandemic took hold, suggests a new demographic normal — one that poses daunting
economic and geopolitical challenges.
In a country where fertility is now well below the replacement rate required to compensate for
deaths, an obvious question arises: As the United States ages, how will a dwindling cohort of
younger, working-age Americans sustain the expensive social services that their parents and
grandparents are counting on in retirement?
The rational answer is a robust immigration system, one that affords the nation a ready supply of
scrappy, striving employees in jobs for which there are insufficient numbers of native-born
Americans, as well as a steady stream of well-educated professionals to fill engineering, scientific,
technology and medical jobs, among others. Immigrants are twice as likely to start new businesses
as native-born Americans and buttress economic growth — think of Tesla, Google and PayPal, all
started by entrepreneurs born elsewhere.
Yet as the 2020 Census demonstrated, immigration flattened out following the Great Recession
in 2008, was actively impeded by the Trump administration and fell once the coronavirus
tightened its grip. That reversed a trend toward rising numbers of immigrants since the 1960s,
when a half-century boom in foreign-born arrivals, the vast majority of them legal, helped fuel
economic growth.
Granted, no celestial mandate dictates that the United States must grow at a faster clip than other
developed countries, as it has for most of its history. But population stagnation may mean a very
different future, and probably a less vital one, than many Americans might imagine. In the short
run, it might mean jobs that go begging for workers as caretakers for the elderly, truck drivers,
computer programmers and other occupations. In the long run, it could impede the country’s
ability to maintain its status as a superpower, project influence and compete with China.
Among the many theories to explain the falling birthrate are women’s increasing labor-force
participation; the daunting cost of living, particularly housing, in job-rich urban areas; a dramatic
drop-off in teen pregnancy; and a social media-distracted younger generation that may be having
less sex. Whatever combination of factors is driving the baby bust, what’s clear is that there will
be no easy answer to reversing it, though family-friendly government and corporate policies could
help.
By contrast, increasing the level of legal immigration is a policy choice that comes with a direct
positive impact. Making that choice would entail forging a political consensus on a uniquely
divisive issue that President Donald Trump, among others, has helped turn into a question of tribal
identity after years in which it was a matter of bipartisan consensus. But not making it is likely
tantamount to acquiescing to an era of demographic stagnation and, over time, diminished
national stature.
(Adapted from https://www.washingtonpost.com/opinions/us-birthrates-are-plummeting-increasing-legal-immigration-can-help/2021/06/25/b3e28236-aded-11eb-b476-c3b287e52a01_story.html)

AULA 05 – PRONOUNS AND PREPOSITIONS 87


TEACHER ANDREA BELO

Questão 03 (EFOMM/INÉDITA) – Read the statements about the text and decide whether they
are TRUE (T) or FALSE (F). Mark the correct option
I. The decline in the U.S. birthrate started in the pandemic
II. In the US, the birthrate is not high enough to offset the deathrate
III. The legal immigration system does not serve as a solution to sustain the needs of the older
population
IV. The legal immigration system is an immediate way to reach young people of working age
V. The new US demographic normal could have profound impacts on the country's economy
I – (F) / II – (T) / III – (F) / IV – (F) / V – (T)
I – (F) / II – (T) / III – (T) / IV – (T) / V – (T)
I – (T) / II – (T) / III – (F) / IV – (T) / V – (T)
I – (F) / II – (T) / III – (F) / IV – (T) / V – (T)
I – (T) / II – (T) / III – (T) / IV – (F) / V – (T)
Comentários: A afirmativa I é falsa (F – false). Não é correto afirmar que o declínio na taxa de
natalidade dos EUA começou na pandemia, mas sim, que esse declínio já dura décadas e acelerou
em 2020. Isso pode ser confirmado com o trecho “THE DECADES-LONG decline in the U.S.
birthrate accelerated in 2020…”.
A afirmativa II é verdadeira (T – true). É correto afirmar que nos EUA, a taxa de natalidade não é
alta o suficiente para compensar a taxa de mortalidade. Isso pode ser confirmado com o trecho
“In a country where fertility is now well below the replacement rate required to compensate for
deaths…”.
A afirmativa III é falsa (F – false). Não é correto afirmar que o sistema de imigração legal não serve
como solução para atender às necessidades da população idosa, mas sim, que pode servir. Isso
pode ser confirmado com o trecho “The rational answer is a robust immigration system, one that
affords the nation a ready supply of scrappy, striving employees…”.
A afirmativa IV é verdadeira (T – true). É correto afirmar que o sistema de imigração legal é uma
forma imediata de chegar aos jovens em idade produtiva. Isso pode ser confirmado com o trecho
“...a robust immigration system, one that affords the nation a ready supply of scrappy, striving
employees in jobs for which there are insufficient numbers of nativeborn Americans…”.
A afirmativa V é verdadeira (T – true). É correto afirmar que a nova normalidade demográfica dos
EUA pode ter impactos profundos na economia do país. Isso pode ser confirmado com o trecho
“In the long run, it could impede the country’s ability to maintain its status as a superpower,
project influence and compete with China”.
GABARITO: D

AULA 05 – PRONOUNS AND PREPOSITIONS 88


TEACHER ANDREA BELO

Questão 04 (EFOMM/INÉDITA) – In the excerpt “…suggests a new demographic normal — one


that poses daunting economic and geopolitical challenges”, the word in bold means
A) Frightening
B) Comforting
C) Ending
D) Constructive
E) Impossible
Comentários: A alternativa A está correta. A palavra “daunting” significa “assustador” e pode ser
comparada com a palavra “frightening”, que significa “assustador”.
A alternativa B está incorreta. A palavra “daunting” significa “assustador” e não pode ser
comparada com a palavra “comforting”, que significa “reconfortante”.
A alternativa C está incorreta. A palavra “daunting” significa “assustador” e não pode ser
comparada com a palavra “ending”, que significa “final”.
A alternativa D está incorreta. A palavra “daunting” significa “assustador” e não pode ser
comparada com a palavra “constructive”, que significa “construtivo”.
A alternativa E está incorreta. A palavra “daunting” significa “assustador” e não pode ser
comparada com a palavra “impossible”, que significa “imopssível”.
GABARITO: A

Questão 05 (EFOMM/INÉDITA) – According to the text, the falling birthrate in the US


A) Collaborated with the high cost of living in job-rich urban areas
B) May be related to the insertion of women in the labor market
C) Is directly related to the increase in teenage pregnancy
D) Has slowed down with the pandemic
E) Can be easily reversed
Comentários: A alternativa A está incorreta. De acordo com o texto, não é correto afirmar que a
queda na taxa de natalidade nos EUA colaborou com o alto custo de vida em áreas urbanas ricas
em empregos, mas sim, que pode ser explicada por esse motivo. Isso pode ser confirmado com o
trecho “Among the many theories to explain the falling birthrate are … the daunting cost of living,
particularly housing, in job-rich urban areas…”.
A alternativa B está correta. De acordo com o texto, é correto afirmar que a queda na taxa de
natalidade nos EUA pode estar relacionada à inserção da mulher no mercado de trabalho, assim
como esta opção indica. Isso pode ser confirmado com o trecho “Among the many theories to
explain the falling birthrate are women’s increasing labor-force participation…”.
A alternativa C está incorreta. De acordo com o texto, não é correto afirmar que a queda na taxa
de natalidade nos EUA está diretamente relacionada ao aumento da gravidez na adolescência,

AULA 05 – PRONOUNS AND PREPOSITIONS 89


TEACHER ANDREA BELO

mas sim, com a diminuição deste fator. Isso pode ser confirmado com o trecho “Among the many
theories to explain the falling birthrate are … a dramatic drop-off in teen pregnancy…”.
A alternativa D está incorreta. De acordo com o texto, não é correto afirmar que a queda na taxa
de natalidade nos EUA diminuiu com a pandemia, mas sim, que acelerou com a pandemia. Isso
pode ser confirmado com o trecho “THE DECADESLONG decline in the U.S. birthrate accelerated
in 2020…”.
A alternativa E está incorreta. De acordo com o texto, não é correto afirmar que a queda na taxa
de natalidade nos EUA pode ser facilmente revertida, mas sim, que não será fácil reverter isso.
Isso pode ser confirmado com o trecho “...what’s clear is that there will be no easy answer to
reversing it…”.
GABARITO: B

Questão 06 (EFOMM/INÉDITA) – Choose the correct option to complete the paragraph below.
“More _______ us are starting to pick back up the strands of our pre-pandemic social lives. As we
figure out who the first people we want to meet up _______ are, we’re recognising there are
friendships _______ the ‘before times’ we didn’t keep up during lockdown – and aren’t
particularly excited _______ re-ignite now that we can”.
(Adapted from https://www.bbc.com/worklife/article/20210623-why-its-ok-to-let-friendships-fade-out)

A) Of / at / from / to
B) X / with / in / to
C) Of / with / from / to
D) Of / with / in / to
E) X / at / in / to
Comentários: A primeira lacuna deve ser preenchida com “of”, pois a frase se refere a nós como
população, ou seja, “Mais de nós estão começando a retomar as vertentes de nossas vidas sociais
pré-pandemia...”.
A segunda lacuna deve ser preenchida com “with”, pois a frase se refere as pessoas com quem
vamos querer nos encontrar no pós-pandemia, ou seja, “À medida que descobrimos quem são as
primeiras pessoas com quem queremos nos encontrar...”.
A terceira lacuna deve ser preenchida com “from”, pois a frase indica a procedência das amizades
em questão, ou seja, “...estamos reconhecendo que existem amizades dos ‘tempos anteriores’...”.
A quarta lacuna deve ser preenchida com “to”, pois a frase se refere ao fato de não estarmos
animados para nos reencontrar com certas pessoas, agora que podemos, ou seja, “...e não
estamos particularmente animados para reencontrar agora que podemos”.
GABARITO: C

AULA 05 – PRONOUNS AND PREPOSITIONS 90


TEACHER ANDREA BELO

Questão 07 (EFOMM/INÉDITA) – Which is the correct way to complete the paragraph below?
“In June 2021, the US Food and Drug Administration (FDA) __________ the first Alzheimer’s drug
in 18 years: aducanumab (also known by its brand name Aduhelm). At the time of writing, the
drug __________ also under review in the EU, Japan, and several other countries. For the roughly
30 million people worldwide who __________ with Alzheimer’s, this is unprecedented news, and
must seem like cause for optimism”.
(Adapted from https://www.theguardian.com/commentisfree/2021/jun/28/alzheimers-drug-aducanumab-approval-dementia)

A) Approved / is / live
B) Approve / is / live
C) Approve / is / lives
D) Approved / was / live
E) Approved / was / lives
Comentários: A primeira lacuna deve ser preenchida com o verbo to approve no simple past
(approved – aprovou), pois o texto indica algo que aconteceu no passado e em um tempo
específico. Isso pode ser confirmado com o trecho “In June 2021, the US Food and Drug
Administration (FDA) approved the first Alzheimer’s drug…”.
A segunda lacuna deve ser preenchida com o verbo to be no simple present (is – é), pois o
medicamento está, no presente momento, em análise em outros países. Isso pode ser confirmado
com o trecho “At the time of writing, the drug is also under review in the EU, Japan, and several
other countries”.
A terceira lacuna deve ser preenchida com o verbo to live no simple present referindo-se ao
pronome “they” (live – vivem), pois a frase se refere as pessoas que vivem com o Alzheimer. Isso
pode ser confirmado com o trecho “For the roughly 30 million people worldwide who live with
Alzheimer’s…”.
GABARITO: A

Questão 08 (EFOMM/INÉDITA) – Which of the following sentences expresses probability?


A) You must eat more vegetables
B) She needs to learn to make her own decisions
C) My father is a very determined person
D) You are much smarter than you think
E) She left two hours ago so she should be getting there by now
Comentários: A alternativa A está incorreta. A frase “Você deve comer mais vegetais” não
expressa probabilidade, mas sim, uma ordem/necessidade.
A alternativa B está incorreta. A frase “Ela precisa aprender a tomar suas próprias decisões” não
expressa probabilidade, mas sim, uma necessidade.

AULA 05 – PRONOUNS AND PREPOSITIONS 91


TEACHER ANDREA BELO

A alternativa C está incorreta. A frase “Meu pai é uma pessoa muito determinada” não expressa
probabilidade, mas sim, uma afirmação.
A alternativa D está incorreta. A frase “Você é muito mais inteligente do que pensa” não expressa
probabilidade, mas sim, uma afirmação.
A alternativa E está correta. A frase “Ela saiu há duas horas, então ela já deve estar chegando lá”
expressa probabilidade.
GABARITO: E

Questão 09 (EFOMM/INÉDITA) – Choose the correct option to complete the paragraph below.
“In the early days of the pandemic, economist Jeanet Bentzen of the University of Copenhagen
examined Google searches for the word ‘prayer’ in 95 countries. She identified that they hit an
all-time global high in March 2020 and increases occurred in lockstep with the number of COVID-
19 cases identified in each country. Stateside, __________ the Pew Research Center, 55 percent
of Americans prayed to end the spread of the novel coronavirus in March 2020, __________
nearly one quarter reported that their faith increased the following month, __________ limited
access to houses of worship”.
(Adapted from https://www.scientificamerican.com/article/psychiatry-needs-to-get-right-with-god/)

A) In contrast to / and / despite


B) According to / and / despite
C) According to / on the other hand / accordingly
D) However / and / despite
E) According to / on the other hand / despite
Comentários: A primeira lacuna deve ser preenchida com “according to” (de acordo com), pois o
texto está indicando uma fonte que deu um dado demonstrado logo em seguida. Isso pode ser
confirmado com o trecho “Nos Estados Unidos, de acordo com o Pew Research Center, 55 por
cento dos americanos oraram...”.
A segunda lacuna deve ser preenchida com “and” (e), pois o texto está adicionando uma
informação relacionada a anterior. Isso pode ser confirmado com o trecho “...55 por cento dos
americanos oraram para acabar com a disseminação do novo coronavírus em março de 2020, e
quase um quarto relatou...”.
A terceira lacuna deve ser preenchida com “despite” (apesar), pois o texto expressa uma ideia
que se contradiz com a informação anterior. Isso pode ser confirmado com o trecho “...quase um
quarto relatou que sua fé aumentou no mês seguinte, apesar do acesso limitado a templos
religiosos”.
GABARITO: B

AULA 05 – PRONOUNS AND PREPOSITIONS 92


TEACHER ANDREA BELO

Questão 10 (EFOMM/INÉDITA) – Which option is incorrect?


A) I want to have lunch before going back to work
B) When can you give me back the money I loaned you?
C) Did you pass the exam?
D) I want they to be happy
E) I need to buy new clothes
Comentários: A alternativa A está incorreta. A frase “I want to have lunch before going back to
work” (Quero almoçar antes de voltar ao trabalho) está totalmente correta.
A alternativa B está incorreta. A frase “When can you give me back the money I loaned you?”
(Quando você pode me devolver o dinheiro que eu lhe emprestei?) está totalmente correta.
A alternativa C está incorreta. A frase “Did you pass the exam?” (Você passou no teste?) está
totalmente correta.
A alternativa D está correta. A frase “I want they to be happy” não está correta porque o pronome
correto é “them”, pois a ação recai sobre os indivíduos, não são eles que estão realizando a ação,
ou seja, o correto é “I want them to be happy” (Eu quero que eles sejam felizes).
A alternativa E está incorreta. A frase “I need to buy new clothes” (Preciso comprar roupas novas)
está totalmente correta.
GABARITO: D

QUESTÕES EPCAR
Directions: Read the text below and answer questions 01 to 10 according to it.
Hugs are coming back. Not everyone is thrilled.
When Stevi Stephens was 5 years old, her grandmother bent down for a hug, and Stephens
wondered if stepping on her foot would make her stop.
As a baby, her mother told her, Stephens cried when anyone held her; later, as a married woman,
she used to get up and change sides of the bed multiple times each night when her husband would
scoot over in his sleep to put an arm around her. “He was like a heat-seeking missile,” she says.
Don’t misunderstand: “I had a great sex life,” Stephens, now 76, clarifies. But Stephens’s attitude
toward being hugged hasn’t changed. With another person wrapped around her, she feels
restrained, uncomfortable.
Stephens, a retired anthropologist, lives in a small co-op of restored fishermen’s huts on
Vancouver Island with about nine other people. “Nobody’s ever tried to hug me here,” she says.
And for Stephens, the past 14 months of social distancing — and the freedom from almost any
hugs at all, even from her two grown children — has been blissful.
Ever since coronavirus vaccines became widely available to the general population, it’s been
evident in parks, restaurants and homes throughout America: Arms across your back are back.
Grandparents are hugging grandkids again.

AULA 05 – PRONOUNS AND PREPOSITIONS 93


TEACHER ANDREA BELO

Friends are hugging friends. Even epidemiologists, a notably cautious bunch, are hugging. For
many, the return of hugs has been a welcome step toward the return of normalcy.
Others, though, have been dreading this moment for a long time. In the frenzied, joyous rush to
make up for a year of lost embraces, it’s easy to lose sight of people like Stephens, who cringe at
the thought of having to endure a whole separate human body enveloping them with little to no
prior notice. Personal-space enthusiasts are sad to see their year of living huglessly come to an
end — even as they hold onto hope that some pandemic distancing habits might stick.
Stevi Stephens lives in a small co-op of restored fishermen’s huts on Vancouver Island. “Nobody’s
ever tried to hug me here,” she says. (Katie Rose MacKenzie)
There are reasons so many humans feel comforted by hugging. Like a massage, it “involves
stimulation and pressure receptors, and when that happens, the whole nervous system slows
down and stress hormone is reduced,” says Tiffany Field, director of the Touch Research Institute
at the University of Miami School of Medicine.
Still, for some, hugs induce stress more than they relieve it. When Sam Zelinka, a federal-
government research scientist based in Madison, Wis., gets a hug from anyone other than his wife
or kids, he gets the same feeling as when a stranger stands too close to him. And while Zelinka,
38, isn’t about to “spray paint a T-shirt and march in the streets yelling about no hugs,” he’s not
exactly looking forward to one from anyone but his parents.
Brooke Todd, a 24-year-old social worker in Stroudsburg, Pa., tends to tense up when someone
moves to hug her, and she didn’t totally make sense of how much calmer her social life had
become during the pandemic until last summer. “It took a few months, but suddenly I was like,
‘Oh, this is nice. I don’t look like a jerk all the time for not wanting to hug someone.’”
Certain kinds of people tend to dislike being touched or embraced, Field notes: some children and
adults with autism, for example, as well as many survivors of sexual assault, such as Todd. Plus,
Field says, the #MeToo movement prompted lots of people to raise a quizzical eyebrow at hugs
in the workplace. “I think that even before covid, hugging was already going out.”
(Adapted from https://www.washingtonpost.com/lifestyle/2021/05/15/hugs-greeting-pandemic/)

Questão 01 (EPCAR/INÉDITA) – Mark the option that can replace the word “thrilled” in the title
without changing its meaning
A) Excited
B) Bored
C) Calm
D) Blessed
Comentários: A alternativa A está correta. A palavra “thrilled” significa entusiasmado e pode ser
substituída pela palavra “excited”, que significa animado.
A alternativa B está incorreta. A palavra “thrilled” significa entusiasmado e não pode ser
substituída pela palavra “bored”, que significa entediado.

AULA 05 – PRONOUNS AND PREPOSITIONS 94


TEACHER ANDREA BELO

A alternativa C está incorreta. A palavra “thrilled” significa entusiasmado e não pode ser
substituída pela palavra “calm”, que significa calmo.
A alternativa D está incorreta. A palavra “thrilled” significa entusiasmado e não pode ser
substituída pela palavra “blessed”, que significa abençoado.
GABARITO: A

Questão 02 (EPCAR/INÉDITA) – We can deduce from the first paragraph that


A) Stevi came to dislike hugs when she became an adult
B) Stevi did not like hugs at age 5
C) Stevi was a very aggressive child
D) Stevi didn't like her grandmother
Comentários: A alternativa A está incorreta. De acordo com o primeiro parágrafo, não podemos
deduzir que Stevi passou a não gostar de abraços quando se tornou adulta, mas sim, que ela não
gostava de abraços aos 5 anos. Isso pode ser confirmado com o trecho “...Stevi Stephens was 5
years old, her grandmother bent down for a hug, and Stephens wondered if stepping on her foot
would make her stop”.
A alternativa B está correta. De acordo com o primeiro parágrafo, podemos deduzir que Stevi não
gostava de abraços aos 5 anos, assim como esta opção indica. Isso pode ser confirmado com o
trecho “...Stevi Stephens was 5 years old, her grandmother bent down for a hug, and Stephens
wondered if stepping on her foot would make her stop”.
A alternativa C está incorreta. De acordo com o primeiro parágrafo, não podemos deduzir que
Stevi era uma criança muito agressiva, mas sim, que ela não gostava de abraços aos 5 anos, o que
não é suficiente para afirmar que ela era agressiva. Isso pode ser confirmado com o trecho “...Stevi
Stephens was 5 years old, her grandmother bent down for a hug, and Stephens wondered if
stepping on her foot would make her stop”.
A alternativa D está incorreta. De acordo com o primeiro parágrafo, não podemos deduzir que
Stevi não gostava da avó, mas sim, que ela não gostava de abraços aos 5 anos e tentou desviar de
um abraço de sua avó, o que não é suficiente para afirmar que ela não gostava de sua avó. Isso
pode ser confirmado com o trecho “...Stevi Stephens was 5 years old, her grandmother bent down
for a hug, and Stephens wondered if stepping on her foot would make her stop”.
GABARITO: B

Questão 03 (EPCAR/INÉDITA) – Mark the option with the suitable question to answer the
underlined fragment below
“But Stephens’s attitude toward being hugged hasn’t changed. With another person wrapped
around her, she feels restrained, uncomfortable”
A) Does Stephens like to be hugged?
B) How does Stephens feel about being hugged?
C) What is Stephens' attitude when hugged?
D) What happened to Stephens' attitude towards being hugged?

AULA 05 – PRONOUNS AND PREPOSITIONS 95


TEACHER ANDREA BELO

Comentários: A alternativa A está incorreta. A frase “A atitude de Stephens em relação a ser


abraçada não mudou” não responde à pergunta “Stephens gosta de ser abraçada?”, mas sim, à
pergunta “O que aconteceu com a atitude de Stephens em relação a ser abraçada?”.
A alternativa B está incorreta. A frase “A atitude de Stephens em relação a ser abraçada não
mudou” não responde à pergunta “Como Stephens se sente ao ser abraçada?”, mas sim, à
pergunta “O que aconteceu com a atitude de Stephens em relação a ser abraçada?”.
A alternativa C está incorreta. A frase “A atitude de Stephens em relação a ser abraçada não
mudou” não responde à pergunta “Qual é a atitude de Stephens ao ser abraçada?”, mas sim, à
pergunta “O que aconteceu com a atitude de Stephens em relação a ser abraçada?”.
A alternativa D está correta. A frase “A atitude de Stephens em relação a ser abraçada não mudou”
responde à pergunta “O que aconteceu com a atitude de Stephens em relação a ser abraçada?”,
assim como esta opção indica.
GABARITO: D

Questão 04 (EPCAR/INÉDITA) – Mark the option that can replace the underlined word keeping
the same meaning
“With another person wrapped around her, she feels restrained, uncomfortable”
A) Friendly
B) Curbed
C) Bold
D) Controlled
Comentários: A alternativa A está incorreta. A palavra “restrained” significa contida e não pode
ser substituída pela palavra “friendly”, que significa amigável.
A alternativa B está correta. A palavra “restrained” significa contida e pode ser substituída pela
palavra “curbed”, que significa contida.
A alternativa C está incorreta. A palavra “restrained” significa contida e não pode ser substituída
pela palavra “bold”, que significa corajoso.
A alternativa D está incorreta. A palavra “restrained” significa contida e não pode ser substituída
pela palavra “controlled”, que significa controlada.
GABARITO: B

Questão 05 (EPCAR/INÉDITA) – Mark the alternative that can answer the question below
according to the text: How does Stephens feel about social distancing?
A) She feels restrained and uncomfortable
B) She feels indifferent
C) She feels good and comfortable
D) She feels needy

AULA 05 – PRONOUNS AND PREPOSITIONS 96


TEACHER ANDREA BELO

Comentários: A alternativa A está incorreta. A pergunta “Como Stephens se sente sobre o


distanciamento social?” não é respondida com “Ela se sente contida e desconfortável”, mas sim,
com “Ela se sente bem e confortável”, pois, para ela, esses meses de confinamento foram
maravilhosos. Isso pode ser confirmado com o trecho “…for Stephens, the past 14 months of social
distancing — and the freedom from almost any hugs at all, even from her two grown children —
has been blissful”.
A alternativa B está incorreta. A pergunta “Como Stephens se sente sobre o distanciamento
social?” não é respondida com “Ela se sente indiferente”, mas sim, com “Ela se sente bem e
confortável”, pois, para ela, esses meses de confinamento foram maravilhosos. Isso pode ser
confirmado com o trecho “…for Stephens, the past 14 months of social distancing — and the
freedom from almost any hugs at all, even from her two grown children — has been blissful”.
A alternativa C está correta. A pergunta “Como Stephens se sente sobre o distanciamento social?”
é respondida com “Ela se sente bem e confortável”, pois, para ela, esses meses de confinamento
foram maravilhosos, assim como esta opção indica. Isso pode ser confirmado com o trecho “…for
Stephens, the past 14 months of social distancing — and the freedom from almost any hugs at all,
even from her two grown children — has been blissful”.
A alternativa D está incorreta. A pergunta “Como Stephens se sente sobre o distanciamento
social?” não é respondida com “Ela se sente carente”, mas sim, com “Ela se sente bem e
confortável”, pois, para ela, esses meses de confinamento foram maravilhosos. Isso pode ser
confirmado com o trecho “…for Stephens, the past 14 months of social distancing — and the
freedom from almost any hugs at all, even from her two grown children — has been blissful”.
GABARITO: C

Questão 06 (EPCAR/INÉDITA) – According to the fifth paragraph, coronavirus vaccines


A) Have made it possible to start returning to social life
B) Are still restricted to a small portion of the population
C) Have not yet made physical contact possible
D) Demonstrate their consequences very subtly across America
Comentários: A alternativa A está correta. De acordo com o quinto parágrafo, é correto afirmar
que vacinas para o coronavírus têm possibilitado o retorno à vida social, assim como esta opção
indica. Isso pode ser confirmado com o trecho “...since coronavirus vaccines became widely
available to the general population, it’s been evident in parks, restaurants and homes throughout
America: Arms across your back are back”.
A alternativa B está incorreta. De acordo com o quinto parágrafo, não é correto afirmar que
vacinas para o coronavírus ainda estão restritas a uma pequena parcela da população, mas sim,
que, nesse caso, já estão amplamente disponíveis para a população geral. Isso pode ser
confirmado com o trecho “...since coronavirus vaccines became widely available to the general
population…”.
A alternativa C está incorreta. De acordo com o quinto parágrafo, não é correto afirmar que
vacinas para o coronavírus ainda não possibilitaram o contato físico, mas sim, que têm

AULA 05 – PRONOUNS AND PREPOSITIONS 97


TEACHER ANDREA BELO

possibilitado cada vez mais. Isso pode ser confirmado com o trecho “Grandparents are hugging
grandkids again. Friends are hugging friends. Even epidemiologists, a notably cautious bunch, are
hugging”.
A alternativa D está incorreta. De acordo com o quinto parágrafo, não é correto afirmar que
vacinas para o coronavírus demonstram suas consequências de maneira muito sutil em toda a
América, mas sim, que já é evidente que o contato físico está voltando à rotina. Isso pode ser
confirmado com o trecho “Ever since coronavirus vaccines became widely available to the general
population, it’s been evident in parks, restaurants and homes throughout America: Arms across
your back are back”.
GABARITO: A

Questão 07 (EPCAR/INÉDITA) – Mark the correct option


A) Epidemiologists are the most cautious group regarding COVID-19
B) Many people believe that being able to hug is a step back to normal
C) There are no people who fear the return to normality
D) Stephens is one of the few people who do not want an end to social distancing
Comentários: A alternativa A está incorreta. De acordo com o texto, não é correto afirmar que os
epidemiologistas são o grupo mais cauteloso em relação ao COVID-19, mas sim, que eles são um
grupo notavelmente cauteloso. Isso pode ser confirmado com o trecho “Even epidemiologists, a
notably cautious bunch, are hugging”.
A alternativa B está correta. De acordo com o texto, é correto afirmar que muitas pessoas
acreditam que ser capaz de abraçar é um passo de volta ao normal, assim como esta opção indica.
Isso pode ser confirmado com o trecho “For many, the return of hugs has been a welcome step
toward the return of normalcy”.
A alternativa C está incorreta. De acordo com o texto, não é correto afirmar que não há quem
tema o retorno à normalidade, mas sim, que existem pessoas que temem. Isso pode ser
confirmado com o trecho “Others, though, have been dreading this moment for a long time”.
A alternativa D está incorreta. De acordo com o texto, não é correto afirmar que Stephens é uma
das poucas pessoas que não quer o fim do distanciamento social, mas sim, que existem muitas
pessoas que não querem a volta à rotina de abraços. Isso pode ser confirmado com o trecho “...it’s
easy to lose sight of people like Stephens, who cringe at the thought of having to endure a whole
separate human body enveloping them with little to no prior notice”.
GABARITO: B

Questão 08 (EPCAR/INÉDITA) – Considering the use of verb tenses, mark the alternative that
completes the sentence below correctly: Personal-space enthusiasts
A) Expected that some habits created in the pandemic will continue to prevail
B) Have expected that some habits created in the pandemic will continue to prevail
C) Will expect that some habits created in the pandemic will continue to prevail
D) Expect that some habits created in the pandemic will continue to prevail

AULA 05 – PRONOUNS AND PREPOSITIONS 98


TEACHER ANDREA BELO

Comentários: A alternativa A está incorreta. A frase dada não pede um verbo no simple past
(expected – esperavam), mas sim, um no simple present (expect – esperam), ou seja, “Personal
space enthusiasts expect that some habits created in the Pandemic will continue to prevail” (Os
entusiastas do espaço pessoal esperam que alguns hábitos criados na pandemia continuem a
prevalecer). Isso pode ser confirmado com o trecho “Personal-space enthusiasts are sad to see
their year of living huglessly come to an end — even as they hold onto hope that some pandemic
distancing habits might stick”.
A alternativa B está incorreta. A frase dada não pede um verbo no present perfct (have expected
– esperavam), mas sim, um no simple present (expect – esperam), ou seja, “Personal space
enthusiasts expect that some habits created in the pandemic will continue to prevail” (Os
entusiastas do espaço pessoal esperam que alguns hábitos criados na pandemia continuem a
prevalecer). Isso pode ser confirmado com o trecho “Personal-space enthusiasts are sad to see
their year of living huglessly come to an end — even as they hold onto hope that some pandemic
distancing habits might stick”.
A alternativa C está incorreta. A frase dada não pede um verbo no simple future (will expect– vão
esperar), mas sim, um no simple present (expect – esperam), ou seja, “Personal space enthusiasts
expect that some habits created in the pandemic will continue to prevail” (Os entusiastas do
espaço pessoal esperam que alguns hábitos criados na pandemia continuem a prevalecer). Isso
pode ser confirmado com o trecho “Personal-space enthusiasts are sad to see their year of living
huglessly come to an end — even as they hold onto hope that some pandemic distancing habits
might stick”.
A alternativa D está correta. A frase dada pede um verbo simple present (expect – esperam), ou
seja, “Personal space enthusiasts expect that some habits created in the pandemic will continue
to prevail” (Os entusiastas do espaço pessoal esperam que alguns hábitos criados na pandemia
continuem a prevalecer), assim como esta opção indica. Isso pode ser confirmado com o trecho
“Personal-space enthusiasts are sad to see their year of living huglessly come to an end — even as
they hold onto hope that some pandemic distancing habits might stick”.
GABARITO: D

Questão 09 (EPCAR/INÉDITA) – We can deduce from the eighth paragraph that


A) Humans are comfortable with hugs just for psychological reasons
B) Few humans are comforted by hugs, according to scientific data
C) The comfort that a hug brings is scientifically proven
D) It is scientifically proven that hugging stimulates stress
Comentários: A alternativa A está incorreta. De acordo com o oitavo parágrafo, não podemos
deduzir que os humanos se sentem confortáveis com abraços apenas por razões psicológicas, mas
sim, que existem razões hormonais e físicas para isso. Isso pode ser confirmado com o trecho
“There are reasons so many humans feel comforted by hugging. Like a massage, it ‘involves
stimulation and pressure receptors…’”.
A alternativa B está incorreta. De acordo com o oitavo parágrafo, não podemos deduzir que
poucos humanos são confortados por abraços, de acordo com dados científicos, mas sim, que

AULA 05 – PRONOUNS AND PREPOSITIONS 99


TEACHER ANDREA BELO

existem razões científicas para tantos humanos se sentirem confortáveis com um abraço. Isso
pode ser confirmado com o trecho “There are reasons so many humans feel comforted by
hugging. Like a massage, it ‘involves stimulation and pressure receptors…’”.
A alternativa C está correta. De acordo com o oitavo parágrafo, podemos deduzir que o conforto
que um abraço traz é comprovado cientificamente, assim como esta opção indica. Isso pode ser
confirmado com o trecho “involves stimulation and pressure receptors, and when that happens,
the whole nervous system slows down and stress hormone is reduced”.
A alternativa D está incorreta. De acordo com o oitavo parágrafo, não podemos deduzir que está
cientificamente comprovado que abraçar estimula o estresse, mas sim, que reduz o estresse. Isso
pode ser confirmado com o trecho “involves stimulation and pressure receptors, and when that
happens, the whole nervous system slows down and stress hormone is reduced”.
GABARITO: C

Questão 10 (EPCAR/INÉDITA) – The word “they” (paragraph 9) refers to


A) Some
B) Hugs
C) Kids
D) Humans
Comentários: A alternativa A está incorreta. A palavra “they” (eles/elas) não se refere à alguns,
mas sim, aos abraços que são citados anteriormente na mesma frase. Isso pode ser confirmado
com o trecho “Still, for some, hugs induce stress more than they relieve it”.
A alternativa B está correta. A palavra “they” (eles/elas) se refere aos abraços que são citados
anteriormente na mesma frase. Isso pode ser confirmado com o trecho “Still, for some, hugs
induce stress more than they relieve it”.
A alternativa C está incorreta. A palavra “they” (eles/elas) não se refere às crianças, mas sim, aos
abraços que são citados anteriormente na mesma frase. Isso pode ser confirmado com o trecho
“Still, for some, hugs induce stress more than they relieve it”.
A alternativa D está incorreta. A palavra “they” (eles/elas) não se refere aos humanos, mas sim,
aos abraços que são citados anteriormente na mesma frase. Isso pode ser confirmado com o
trecho “Still, for some, hugs induce stress more than they relieve it”.
GABARITO: B

QUESTÕES ESA
Read the text and answer questions 01, 02 and 03
What You Need to Know About COVID-19 and Flu
Every winter is a bit of a roulette wheel when it comes to influenza. Flu vaccines work, but aren’t
100% effective in preventing disease, so it’s always a challenge convincing people to get their flu
shots. And while the symptoms are generally bearable, infections can become more severe and

AULA 05 – PRONOUNS AND PREPOSITIONS 100


TEACHER ANDREA BELO

even deadly among people who are older or who have underlying health conditions. Last flu
season, even though experts considered it a relatively mild year, about 400,000 people in the U.S.
were hospitalized and 22,000 people died from the flu.
This winter, the influenza virus has a rival—the coronavirus fueling the COVID-19 pandemic—and
health officials are anticipating a showdown that could have dire consequences for the health of
millions. Both diseases are caused by viruses that spread with abandon from person to person
through sneezes, coughs, and respiratory droplets during close contact.
But while researchers know quite a bit about the influenza virus, the coronavirus, SARS-CoV-2, is
very much a black box, and they can only guess what will happen when the two pathogens collide
throughout the world.
Why is it dangerous to have flu and COVID-19 viruses around at the same time?
A double whammy of flu and SARS-CoV-2 infections this winter could be devastating for public
health, as the colder weather brings people into closer contact with each other in confined spaces
indoors. “What I worry about is that both viruses can cause serious respiratory illness,” says Dr.
David Chokshi, health commissioner for New York City. “In the same way that COVID-19 often gets
to the point where someone has to be hospitalized to support their breathing, or needs to be in
the ICU with a breathing tube or ventilator, unfortunately we see that in the most severe cases of
influenza as well. What we worry about is that happening at the same time and really stressing
the capacity of our hospitals.”
Public health experts are concerned that too many people who are seriously ill with influenza or
COVID-19 could flood hospitals and stretch already worn out health care workers, and health care
systems, to their limits.
(Adapted from https://time.com/5917061/covid-19-flu-season/)

Questão 01 (ESA/INÉDITA) – Concerning the information in the text, is correct to state that
(A) The flu vaccine is not homogeneously accepted, considering its effectiveness.
(B) Flu symptoms are usually severe and unbearable.
(C) The flu always develops to a more severe condition, in all those infected.
(D) The numbers of hospitalizations and deaths from influenza are minimal.
(E) Those who are older or with health problems are less susceptible to getting the flu.
Comentários: A alternativa A está correta. De acordo com o texto, é correto afirmar que a vacina
contra gripe não é aceita de forma homogênea, considerando sua eficácia, assim como esta opção
indica. Isso pode ser confirmado com o trecho “Flu vaccines work, but aren’t 100% effective in
preventing disease, so it’s always a challenge convincing people to get their flu shots”.
A alternativa B está incorreta. De acordo com o texto, não é correto afirmar que os sintomas da
gripe são geralmente graves e insuportáveis, mas sim, que eles geralmente são suportáveis. Isso
pode ser confirmado com o trecho “And while the symptoms are generally bearable…”.
A alternativa C está incorreta. De acordo com o texto, não é correto afirmar que a gripe sempre
evolui para uma condição mais grave, em todos os infectados, mas sim, que entre pacientes mais
velhos ou com problemas de saúde, a infecção pode se tornar mais severa. Isso pode ser

AULA 05 – PRONOUNS AND PREPOSITIONS 101


TEACHER ANDREA BELO

confirmado com o trecho “...infections can become more severe and even deadly among people
who are older or who have underlying health conditions”.
A alternativa D está incorreta. De acordo com o texto, não é correto afirmar que os números de
hospitalizações e mortes por influenza são mínimos, mas sim, que são números altos. Isso pode
ser confirmado com o trecho “...about 400,000 people in the U.S. were hospitalized and 22,000
people died from the flu”.
A alternativa E está incorreta. De acordo com o texto, não é correto afirmar que pessoas mais
velhas ou com problemas de saúde são menos suscetíveis a pegar gripe, mas sim, que essas
pessoas são mais suscetíveis a desenvolver um quadro grave de gripe. Isso pode ser confirmado
com o trecho “...infections can become more severe and even deadly among people who are older
or who have underlying health conditions”.
GABARITO: A

Questão 02 (ESA/INÉDITA) – In the sentence “This winter, the influenza virus has a rival—the
coronavirus fueling the COVID-19 pandemic—and health officials are anticipating a showdown
that could have dire consequences for the health of millions” (paragraph 2), the word dire
means
(A) Good.
(B) Mild.
(C) Drastic.
(D) Nice.
(E) Partial.
Comentários: A alternativa A está incorreta. A palavra “dire” significa terrível e não pode ser
comparada com a palavra “good”, que significa bom.
A alternativa B está incorreta. A palavra “dire” significa terrível e não pode ser comparada com a
palavra “mild”, que significa suave.
A alternativa C está correta. A palavra “dire” significa terrível e pode ser comparada com a palavra
“drastic”, que significa drástico.
A alternativa D está incorreta. A palavra “dire” significa terrível e não pode ser comparada com a
palavra “nice”, que significa legal.
A alternativa E está incorreta. A palavra “dire” significa terrível e não pode ser comparada com a
palavra “partial”, que significa parcial.
GABARITO: C

Questão 03 (ESA/INÉDITA) – The words CAUSED and SUPPORT are


(A) Irregular verbs.
(B) Modal verbs.
(C) Not verbs.
(D) Regular verbs.
(E) From different grammatical classes.

AULA 05 – PRONOUNS AND PREPOSITIONS 102


TEACHER ANDREA BELO

Comentários: A alternativa A está incorreta. As palavras “caused” (causou) e “support”


(apoiar/suportar) não são verbos irregulares, mas sim, verbos regulares; ou seja, quando
colocados no passado, fazem uso da terminação -ed, por exemplo.
A alternativa B está incorreta. As palavras “caused” (causou) e “support” (apoiar/suportar) não
são verbos modais, mas sim, verbos regulares; ou seja, quando colocados no passado, fazem uso
da terminação -ed, por exemplo.
A alternativa C está incorreta. As palavras “caused” (causou) e “support” (apoiar/suportar) são
verbos regulares; ou seja, quando colocados no passado, fazem uso da terminação -ed, por
exemplo.
A alternativa D está correta. As palavras “caused” (causou) e “support” (apoiar/suportar) são
regulares, assim como esta opção indica; ou seja, quando colocados no passado, fazem uso da
terminação -ed, por exemplo.
A alternativa E está incorreta. As palavras “caused” (causou) e “support” (apoiar/suportar) são
ambas verbos regulares; ou seja, quando colocados no passado, fazem uso da terminação -ed,
por exemplo.
GABARITO: D

Read the text and answer questions 04 and 05


Nine Tools for Better, Longer Sleep
For many of us, a restful night of sleep is hard to come by under the best of circumstances. But
against the backdrop of a long, cold winter and the ongoing pandemic, it can feel almost
impossible. While there will always be newfangled gadgets that claim to solve your sleep
problems with advanced technology — yes, there are such things as sleep robots and sleep-
tracking rings — newer doesn’t always mean better. Wirecutter, the New York Times Company
__________ (1) reviews and recommends products, has tested countless items __________ (2)
find real, proven solutions, from blackout curtains and sleep masks to better pillows and white-
noise machines.
In addition to the products they test for work, Wirecutter writers and editors have found their
own sleep solutions for these overworked, overstressed times. These aren’t necessarily products
we’ve rigorously tested (unless we’re talking about meditation apps), but they are ___(3) things
Wirecutter staffers are finding useful at home for falling, and staying, asleep.
(Adapted from https://www.nytimes.com/2021/03/09/realestate/nine-tools-for-better-longersleep.html?action=click&algo=bandit-
allsurfaces&block=trending_recirc&fellback=false&imp_id=219715208&impression_id=06846758-81e0-11eb-
b4b15521553e2d9f&index=8&pgtype=Article&region=footer&req_id=115710805&surface=most-popularstory&variant=1_bandit-all-surfaces_daysback_4)

Questão 04 (ESA/INÉDITA) – Concerning the information in the text, is correct to say that
(A) A good night's sleep has been recurring in this pandemic.
(B) Pandemic and winter are factors that contribute to a good night's sleep.
(C) There is no technology developed to provide a good night's sleep.
(D) Meditation apps have not been tested by The New York Times team.
(E) The purpose of the text is to say that a team from The New York Times evaluated services
that help us to have a better night's sleep.

AULA 05 – PRONOUNS AND PREPOSITIONS 103


TEACHER ANDREA BELO

Comentários: A alternativa A está incorreta. De acordo com o texto, não é correto afirmar que
uma boa noite de sono tem sido recorrente nesta pandemia, mas sim, que tem sido quase
impossível. Isso pode ser confirmado com o trecho “For many of us, a restful night of sleep is hard
to come by under the best of circumstances. But against the backdrop of a long, cold winter and
the ongoing pandemic, it can feel almost impossible”.
A alternativa B está incorreta. De acordo com o texto, não é correto afirmar que pandemia e
inverno são fatores que contribuem para uma boa noite de sono, mas sim, que a tornam quase
impossível. Isso pode ser confirmado com o trecho “For many of us, a restful night of sleep is hard
to come by under the best of circumstances. But against the backdrop of a long, cold winter and
the ongoing pandemic, it can feel almost impossible”.
A alternativa C está incorreta. De acordo com o texto, não é correto afirmar que não existe
tecnologia desenvolvida para proporcionar uma boa noite de sono, mas sim, que existe. Isso pode
ser confirmado com o trecho “While there will Always be newfangled gadgets that claim to solve
your sleep problems with advanced technology…”.
A alternativa D está incorreta. De acordo com o texto, não é correto afirmar que aplicativos de
meditação não foram testados pela equipe do The New York Times, mas sim, que foram testados.
Isso pode ser confirmado com o trecho “These aren’t necessarily products we’ve rigorously tested
(unless we’re talking about meditation apps)…”.
A alternativa E está correta. De acordo com o texto, é correto afirmar que o objetivo do texto é
dizer que uma equipe do The New York Times avaliou serviços que nos ajudam a ter uma noite de
sono melhor, assim como esta opção indica. Isso pode ser confirmado com o trecho “In addition
to the products they test for work, Wirecutter writers and editors have found their own sleep
solutions for these overworked…”.
GABARITO: E

Questão 05 (ESA/INÉDITA) – In the sentence “While there will always be newfangled gadgets
that claim to solve your sleep problems with advanced technology…” (paragraph 1), the word
claim means
(A) Allege.
(B) Justify.
(C) Deny.
(D) Insist.
(E) Abandon.
Comentários: A alternativa A está correta. A palavra “claim” significa pretender/alegar e pode ser
comparada com a palavra “allege”, que significa alegar.
A alternativa B está incorreta. A palavra “claim” significa pretender/alegar e não pode ser
comparada com a palavra “justify”, que significa justificar.
A alternativa C está incorreta. A palavra “claim” significa pretender/alegar e não pode ser
comparada com a palavra “deny”, que significa negar.

AULA 05 – PRONOUNS AND PREPOSITIONS 104


TEACHER ANDREA BELO

A alternativa D está incorreta. A palavra “claim” significa pretender/alegar e não pode ser
comparada com a palavra “insist”, que significa insistir.
A alternativa E está incorreta. A palavra “claim” significa pretender/alegar e não pode ser
comparada com a palavra”abandon”, que significa abandonar.
GABARITO: A

QUESTÕES ESCOLA NAVAL


Based on the text below, answer questions 01 to 08.
If you bought a dog during lockdown, they'll need help coming out of it
Despite the lows of the past year, for many, lockdown has been the perfect opportunity to
welcome a dog into their lives. The demand for dogs during the pandemic has been huge, with a
60% increase in calls from people seeking to adopt from the Dogs Trust charity, and with many
other rescue organisations reporting similar findings. Google searches for “buy a puppy”
increased by 115% after the UK first went into lockdown in March 2020, with prices for some of
the most soughtafter breeds reaching record levels.
Dog ownership is a wonderful thing, but it is also a huge responsibility and a commitment that
spans way beyond lockdown: as the saying goes, “A dog is for life”. As restrictions ease and the
resumption of normality begins, it’s important we consider the implications for our canine
companions and give them a hand to help them adjust.
Having a dog around has helped many people cope with lockdown. Our dogs mostly love us being
around too: going for longer walks, having more playtime, and resting by our side. Nevertheless,
it’s safe to say life has not been normal for our dogs for most of the past year. Few have met other
dogs, and if they have seen them, it would have been from afar or on a lead, meaning that they
were unable to interact or play. There have also been fewer visitors coming into the home, but
probably more deliveries, with people coming to the door carrying parcels and going away again.
This is all particularly concerning for puppies acquired during the pandemic, as their expectation
of “normal” is lockdown life, and they may never have seen visitors inside the house or have been
left home alone.
We are all longing for a great British summer in which we can go on dog-date walks with a friend
and their dog, have family round for garden barbecues and take our pooches to the pub or cafe,
and of course, we need our dogs to be able to cope calmly with all of that. A return to normal is
something humans are able to process, understand and prepare for. But our dogs – especially
young ones – won’t understand why everything has changed. As far as our dogs know, normality
for them has been enjoying time with family only – so to be expected to cope with groups of
people, children and other dogs, both in and out of the home, could be overwhelming for them.
A big worry for dog owners is the long-term impact of lockdown on their ability to cope with being
left at home on their own. Dogs who had separation anxiety before the lockdown are likely to get
worse when left again as owners head back to work – but we also expect to see new cases
developing, because other dogs, and particularly puppies, have learned to expect company all
day.

AULA 05 – PRONOUNS AND PREPOSITIONS 105


TEACHER ANDREA BELO

One of the biggest reasons why dogs are rehomed is because of behaviour-related issues. A rise
in problematic behaviours after lockdown could mean families have no other option but to give
up their dog. And, really sadly, most of these problems can be prevented with the right early
experiences.
Our message to owners is to start preparing now, rather than waiting until things return more to
normal. It’s easy to do: start building up experiences of all the things we will expect them to do
once the lockdown eases. For instance, start building in minimal periods apart, initially just being
briefly separated from you by a door or child gate. If they stay calm, build up the time separated
really gradually, so they start to adjust to not being with you all the time. If your dog gets worried
when separated – barking, whining, panting, or scratching at the door – you have progressed too
fast. Go back to a shorter period to help them adjust. By gradually increasing your time apart, you
can ensure they are able to settle on their own and help them prepare for the time when you
need to return to work or study.
Our dogs will also need help when it comes to seeing friends and family, both outdoors and
indoors. Teaching your dog how to greet new people calmly, how to settle when guests visit or
when you’re in a cafe are key skills. Coming back when called, walking on a loose lead and not
barking when the doorbell goes are also vital skills that will set them up for success.
It is much easier to prevent problems than treat them, and it’s not too late to help prepare your
dogs for the changes coming. To support dog owners, there is online training, so dogs and their
owners can equip themselves with the skills they can put into practice as normality resumes.
When people take on the responsibilities of dog ownership, they do so with the best intentions
to care for them in the long term. But the pandemic will have devastating effects on some people’s
lives, including their ability to care for the ir dogs. While we provide lots of support to help keep
dogs and owners together, we’re also here for when things aren’t going so well, and owners may
be having trouble seeing a future with their dog. If anyone does find themselves struggling, and
may be considering having to rehome their dog, please contact Dogs Trust and we will do
everything we can to help.
(Adapted from https://www.theguardian.com/commentisfree/2021/apr/05/bought-dog-lockdown)

Questão 01 (ESCOLA NAVAL/INÉDITA) – According to the text, which option is correct?


(A) the demand for new dogs as pets shrank over the past year.
(B) for the author, lockdown has been the ideal moment for getting a new dog.
(C) a report shows that people prefer buying breed dogs to adopting stray dogs.
(D) coming out of lockdown life may have serious consequences for dogs.
(E) Google searches by people wishing to adopt dogs have increased.
Comentários: De acordo com o texto, qual opção é a correta?
a) a demanda por novos cães como animais de estimação diminuiu no último ano – o texto diz
que a demanda aumentou. “The demand for dogs during the pandemic has been huge, with a
60% increase in calls from people seeking to adopt… Google searches for “buy a puppy” increased
by 115%...” = A demanda por cães durante a pandemia tem sido enorme, com um aumento de

AULA 05 – PRONOUNS AND PREPOSITIONS 106


TEACHER ANDREA BELO

60% nas ligações de pessoas que buscam adotar... As pesquisas no Google por “comprar um
cachorro” aumentaram 115%... ERRADA.
b) para o autor, o lockdown tem sido o momento ideal para arranjar um novo cachorro – o autor
não afirma isso, como sua opinião pessoal. Ele diz que para muitas pessoas o lockdown tem sido
o momento ideal para arranjar um novo cachorro. “Despite the lows of the past year, for many,
lockdown has been the perfect opportunity to welcome a dog into their lives.” = Apesar dos baixos
do ano passado, para muitos, o confinamento tem sido a oportunidade perfeita para dar as boas-
vindas a um cão em suas vidas. ERRADA.
c) um relatório mostra que as pessoas preferem comprar cães de raça a adotar cães vira-latas – o
texto não afirma isso. Apenas menciona, como vimos na letra A, que as buscas por adoção e
compra de cachorros aumentaram, sem afirmar que as pessoas preferem adotar. ERRADA.
d) sair da vida de lockdown pode ter consequências graves para os cães – “As restrictions ease
and the resumption of normality begins, it’s important we consider the implications for our canine
companions and give them a hand to help them adjust.” = À medida que as restrições diminuem
e a retomada da normalidade começa, é importante considerarmos as implicações para nossos
companheiros caninos e ajudá-los a se ajustar. CORRETA.
e) as pesquisas no Google por pessoas que desejam adotar cães aumentaram – o texto afirma que
as pesquisas no Google por “comprar um cachorro” aumentaram 115%... ERRADA.
GABARITO: D

Questão 02 (ESCOLA NAVAL/INÉDITA) – What’s the meaning of the word “seeking” in


paragraph 1?
(A) deciding
(B) dreaming of
(C) suggesting
(D) asking for
(E) looking for
Comentários: Qual é o significado da palavra “seeking” no parágrafo 1? Seeking significa, no
contexto, buscando, procurando. Vejamos as alternativas:
a) decidindo
b) sonhando com
c) sugerindo
d) pedindo por
e) procurando.
GABARITO: E

AULA 05 – PRONOUNS AND PREPOSITIONS 107


TEACHER ANDREA BELO

Questão 03 (ESCOLA NAVAL/INÉDITA) – According to the contents of paragraph 3, among other


reasons, life for dogs has not been ordinary over the past year because they:
(A) could meet other dogs and play with them.
(B) had to get used to more people visiting them.
(C) haven’t been able to interact with other dogs.
(D) were able to rest by their owners’ side.
(E) were left home alone more often than usual.
Comentários: De acordo com o conteúdo do parágrafo 3, entre outras razões, a vida dos cães não
foi normal no ano passado porque eles:
a) puderam encontrar outros cães e brincar com eles – na verdade, não puderam, por conta das
restrições devido ao lockdown. ERRADA.
b) tiveram que se acostumar com a visita de mais pessoas – na realidade, as pessoas praticamente
não recebiam visitas, por conta da pandemia. ERRADA.
c) não tiveram a possibilidade de interagir com outros cães – de fato, por conta da pandemia.
“Nevertheless, it’s safe to say life has not been normal for our dogs for most of the past year. Few
have met other dogs, and if they have seen them, it would have been from afar or on a lead,
meaning that they were unable to interact or play.” = No entanto, é seguro dizer que a vida não
foi normal para nossos cães durante a maior parte do ano passado. Poucos conheceram outros
cães e, se os viram, teria sido de longe ou com trela, o que significa que não puderam interagir ou
brincar. CORRETA.
d) puderam descansar ao lado de seus donos – não é que o texto afirma. “Our dogs mostly love
us being around too: going for longer walks, having more playtime, and resting by our side.” =
Nossos cães também adoram que fiquemos por perto: dar mais passeios, ter mais tempo para
brincar e descansar ao nosso lado. ERRADA.
e) foram deixados sozinhos em casa com mais frequência do que o normal – o texto afirma que
os cães ficaram mais em casa com a família. “A big worry for dog owners is the long-term impact
of lockdown on their ability to cope with being left at home on their own. Dogs who had
separation anxiety before the lockdown are likely to get worse when left again as owners head
back to work…” = Uma grande preocupação para os donos de cães é o impacto a longo prazo do
lockdown em sua capacidade de lidar com o fato de ficarem sozinhos em casa. Os cães que tinham
ansiedade de separação antes do lockdown tendem a piorar quando deixados novamente, pois
os donos voltam ao trabalho... ERRADA.
GABARITO: C

Questão 04 (ESCOLA NAVAL/INÉDITA) – In paragraph 2, the words “them” refer to


(A) canine companions.
(B) restrictions.
(C) implications.
(D) dog owners.
(E) people.

AULA 05 – PRONOUNS AND PREPOSITIONS 108


TEACHER ANDREA BELO

Comentários: No parágrafo 2, as palavras “them” referem-se a


a) companheiros caninos.
b) restrições.
c) implicações.
d) donos de cães.
e) pessoas.
“As restrictions ease and the resumption of normality begins, it’s important we consider the
implications for our canine companions and give them a hand to help them adjust.” = À medida
que as restrições diminuem e a retomada da normalidade começa, é importante considerarmos
as implicações para nossos companheiros caninos e ajudá-los para que eles se adaptem.
GABARITO: A

Questão 05 (ESCOLA NAVAL/INÉDITA) – According to paragraph 4, it is correct to say that dogs


might have trouble understanding why life has changed because:
(A) they expect to interact with more groups of people.
(B) normality for them has been staying home with family.
(C) they don’ want to process the return to normality.
(D) their owners will stop going on dog-date walks with friends.
(E) they’re looking forward to meeting children again.
Comentários: De acordo com o parágrafo 4, é correto dizer que os cães podem ter problemas
para entender por que a vida mudou porque:
a) eles esperam interagir com mais grupos de pessoas.
b) a normalidade para eles tem sido ficar em casa com a família.
c) eles não querem processar o retorno à normalidade.
d) seus donos deixarão de fazer encontros de cachorros com amigos.
e) eles estão ansiosos para encontrar crianças novamente.
“But our dogs – especially young ones – won’t understand why everything has changed. As far as
our dogs know, normality for them has been enjoying time with family only – so to be expected
to cope with groups of people, children and other dogs, both in and out of the home, could be
overwhelming for them.” = Mas nossos cães - especialmente os mais jovens - não vão entender
por que tudo mudou. Até onde nossos cães sabem, a normalidade para eles é aproveitar o tempo
apenas com a família - então, esperar que lidem com grupos de pessoas, crianças e outros cães,
dentro e fora de casa, pode ser opressor para eles.
GABARITO: B

AULA 05 – PRONOUNS AND PREPOSITIONS 109


TEACHER ANDREA BELO

Questão 06 (ESCOLA NAVAL/INÉDITA) – In the sentence “A big worry for dog owners is the
longterm impact of lockdown on their ability to cope with being left at home on their own.”
(paragraph 5), the underlined words mean __________.
(A) understand.
(B) handle.
(C) cooperate.
(D) forget.
(E) deny.
Comentários: Na frase “Uma grande preocupação para os donos de cães é o impacto de longo
prazo do lockdown em sua capacidade de lidar com (cope with) o fato de ficarem em casa
sozinhos.” (parágrafo 5), as palavras sublinhadas significam __________.
a) entender.
b) lidar.
c) cooperar.
d) esquecer.
e) negar.
GABARITO: B

Questão 07 (ESCOLA NAVAL/INÉDITA) – What does the quantifier “many” refer to in the
excerpt “Despite the lows of the past year, for many, lockdown has been the perfect
opportunity to welcome a dog into their lives.” (paragraph 1)?
(A) lows.
(B) people.
(C) dogs.
(D) lives.
(E) opportunities.
Comentários: A que se refere o quantificador “muitos” no trecho “Apesar dos baixos do ano
passado, para muitos (= para muitas pessoas), o lockdown tem sido a oportunidade perfeita para
dar as boas-vindas a um cão em suas vidas”. (parágrafo 1)?
a) baixos.
b) pessoas.
c) cães.
d) vidas.
e) oportunidades.
GABARITO: B

AULA 05 – PRONOUNS AND PREPOSITIONS 110


TEACHER ANDREA BELO

Questão 08 (ESCOLA NAVAL/INÉDITA) – According to the text, which option is correct?


(A) now it is too late to prepare dogs for the changes ahead.
(B) barking when the doorbell goes is a desirable dog reaction.
(C) when people decide to have a dog, they have mean intentions.
(D) it is not possible to avoid dog behavioral problems.
(E) dogs must be taught how to deal with the changes they will face.
Comentários: De acordo com o texto, qual opção é a correta?
a) agora é tarde demais para preparar os cães para as mudanças que virão – “It is much easier to
prevent problems than treat them, and it’s not too late to help prepare your dogs for the changes
coming.” = É muito mais fácil prevenir problemas do que tratá-los, e não é tarde demais para
ajudar a preparar seus cães para as mudanças que estão por vir. ERRADA.
b) latir quando a campainha toca é uma reação desejável do cão – “Coming back when called,
walking on a loose lead and not barking when the doorbell goes are also vital skills that will set
them up for success.” = Voltar quando chamado, andar com a guia solta e não latir quando a
campainha toca também são habilidades vitais que os prepararão para o sucesso. ERRADA.
c) quando as pessoas decidem ter um cachorro, elas têm más intenções – “When people take on
the responsibilities of dog ownership, they do so with the best intentions to care for them in the
long term.” = Quando as pessoas assumem as responsabilidades de ter um cão, o fazem com as
melhores intenções de cuidar deles a longo prazo. ERRADA.
d) não é possível evitar problemas de comportamento canino – “It is much easier to prevent
problems than treat them, and it’s not too late to help prepare your dogs for the changes coming.”
= É muito mais fácil prevenir problemas do que tratálos, e não é tarde demais para ajudar a
preparar seus cães para as mudanças que estão por vir. ERRADA.
e) os cães devem ser ensinados a lidar com as mudanças que irão enfrentar – “It is much easier
to prevent problems than treat them, and it’s not too late to help prepare your dogs for the
changes coming.” = É muito mais fácil prevenir problemas do que tratá-los, e não é tarde demais
para ajudar a preparar seus cães para as mudanças que estão por vir. CORRETA.
GABARITO: E

Questão 09 (ESCOLA NAVAL/INÉDITA) – HOW __________ TIMES A WEEK SHOULD YOU EXERCISE
AND HOW __________ FOR? It's crucial to factor in rest days, too.
(https://www.independent.co.uk/life-style/health-and-families/)

Which words best complete the question above?


(A) many / long
(B) much / long
(C) many / far
(D) often / high
(E) often / far

AULA 05 – PRONOUNS AND PREPOSITIONS 111


TEACHER ANDREA BELO

Comentários: Como times é um substantivo contável, para perguntarmos “quantas vezes”


(quantas vezes por semana você deveria se exercitar...), devemos usar many: how many.
Na segunda parte, usaremos long (how long for) dando o sentido de “e por quanto tempo” →
quantas vezes por semana você deveria se exercitar e por quanto tempo?.
GABARITO: A

Questão 10 (ESCOLA NAVAL/INÉDITA) – Which option completes the text below correctly?
__________ active is crucial for __________ healthy and, considering 41 per cent of Britons aged
40-60 fail to walk for even ten minutes a month, according to a 2017 study, you might be inclined
to up your activity levels.
(https://www.independent.co.uk/life-style/health-and-families/)

(A) To keep / stay


(B) To keeping / staying
(C) Keeping / staying
(D) Keeping / stay
(E) To keep / to stay
Comentários: Nas lacunas, usaremos verbos com -ing com sentido de infinitivo, desempenhando
as funções de sujeito e de objeto: Keeping active is crucial for staying healthy and, considering 41
per cent of Britons aged 40-60 fail to walk for even ten minutes a month, according to a 2017
study, you might be inclined to up your activity levels. = Manter-se ativo é crucial para permanecer
saudável e, considerando que 41 por cento dos britânicos com idades entre 40-60 não conseguem
andar até mesmo dez minutos por mês, de acordo com um estudo de 2017, você pode estar
inclinado a aumentar seus níveis de atividade.
GABARITO: B

QUESTÕES EsPCEx
Leia o texto a seguir e responda às questões 01, 02 e 03.
Eight States Are Seeding Clouds to Overcome Megadrought
The mountaintops rumble to life unnaturally each year as snow clouds darken the sky across the
West. Open flames burst from the throats of metal chimneys, mounted on squat towers nestled
among the peaks. With a low hiss, puffs of particles belch from their mouths into the air, where
the wind catches them and whisks them away. These aren’t ordinary particles. They’re tiny bits of
crushed-up silver iodide, a crystal-like photosensitive substance once used in photography. But it’s
not used to take pictures out in the mountains. It’s meant to make snow.
As the wind whips the particles across the mountaintops, drafts of air sweep them higher into the
sky—so high that some of them eventually touch the clouds. There, an elegant transformation
takes place. The crystalline iodide particles have a structure similar to ice—and inside a cloud, like

AULA 05 – PRONOUNS AND PREPOSITIONS 112


TEACHER ANDREA BELO

attracts like. Water droplets begin to cluster around the particles, freezing solid as they gather
together.
These frozen clusters eventually grow too heavy to stay in the air. They fall from the cloud and drift
gently toward the Earth, dusting the mountaintops with fresh snow. This is not a page from a
science fiction novel. “Cloud seeding” is a real practice—in fact, it’s been around for decades.
It’s used today to boost precipitation in at least eight states across the western U.S. and dozens of
countries around the world. Interest in cloud seeding is growing as temperatures steadily rise,
increasing drought risks in places like the Mountain West. But there’s a catch. Scientists aren’t sure
how well cloud seeding works today, let alone in a warmer climate.
Amid growing concerns about water resources in the western U.S., scientists are working to
answer those questions.
Today, cloud seeding research represents the cutting edge of weather and climate science—a
convergence of questions about the influence of warming on our dwindling water resources and
our ability to control those consequences.
Adapted from https://www.scientificamerican.com/article/eight-states-are-seeding-clouds-to-overcome-megadrought/

Questão 01 (EsPCEx/INÉDITA) – Which text structure is mainly used in paragraphs 1, 2 and 3 to


create a scenario?
(A) persuasion
(B) literary text
(C) first person perspective
(D) description
(E) fiction
Comentários: Qual estrutura de texto é usada principalmente nos parágrafos 1, 2 e 3 para criar
um cenário?
a) persuasão
b) texto literário
c) perspectiva de primeira pessoa
d) descrição
e) ficção
Os três primeiros parágrafos são essencialmente descritivos, para que o leitor possa imaginar
exatamente como ocorre todo o processo de “cloud seeding” (semeadura de nuvens). Veja um
trecho que ilustra bem a estrutura descritiva do texto nos primeiros parágrafos: “Open flames
burst from the throats of metal chimneys, mounted on squat towers nestled among the peaks.
With a low hiss, puffs of particles belch from their mouths into the air, where the wind catches
them and whisks them away.” = Chamas abertas saem das gargantas das chaminés de metal,
montadas em torres baixas aninhadas entre os picos. Com um silvo baixo, lufadas de partículas
saem de suas bocas para o ar, onde o vento as agarra e leva embora.
GABARITO: D

AULA 05 – PRONOUNS AND PREPOSITIONS 113


TEACHER ANDREA BELO

Questão 02 (EsPCEx/INÉDITA) – Choose the alternative that correctly substitutes the word
toward in the sentence “They fall from the cloud and drift gently toward the Earth, dusting the
mountaintops with fresh snow.” (paragraph 3).
(A) across from
(B) in the direction of
(C) next to
(D) alongside
(E) within
Comentários: Escolha a alternativa que substitui corretamente a palavra toward na frase “They
fall from the cloud and drift gently toward the Earth, dusting the mountaintops with fresh snow.”
Na frase trazida por essa questão, toward significa em direção a, veja: “Eles caem da nuvem e
vagam suavemente em direção à Terra, cobrindo o topo das montanhas com neve fresca”.
a) em frente a
b) em direção a
c) próximo a
d) ao lado
e) dentro
GABARITO: B

Questão 03 (EsPCEx/INÉDITA) – Choose the alternative that correctly substitutes the word
toward in the sentence “They fall from the cloud and drift gently toward the Earth, dusting the
mountaintops with fresh snow.” (paragraph 3).
(A) silver iodide is a photosensitive substance only used in photography.
(B) the crystalline iodide particles and ice have an exactly alike structure.
(C) “cloud seeding” might become reality anytime soon.
(D) “cloud seeding” is used to shrink precipitation in some countries around the world.
(E) scientists don’t know how exactly well cloud seeding works nowadays.
Comentários: De acordo com o texto, escolha a afirmação correta.
a) o iodeto de prata é uma substância fotossensível usada apenas em fotografia – “They’re tiny
bits of crushed-up silver iodide, a crystal-like photosensitive substance once used in photography.
But it’s not used to take pictures out in the mountains. It’s meant to make snow.” = Eles são
pequenos pedaços de iodeto de prata esmagado, uma substância fotossensível semelhante a um
cristal que já foi usada em fotografia. Mas não é usado para tirar fotos nas montanhas. É para fazer
neve. ERRADA.
b) as partículas cristalinas de iodeto e gelo têm uma estrutura exatamente igual – “The crystalline
iodide particles have a structure similar to ice...” = As partículas cristalinas de iodeto têm uma
estrutura semelhante à do gelo... ERRADA.

AULA 05 – PRONOUNS AND PREPOSITIONS 114


TEACHER ANDREA BELO

c) a “semeadura de nuvens” pode se tornar realidade em breve. – “This is not a page from a
science fiction novel. ‘Cloud seeding’ is a real practice—in fact, it’s been around for decades.” =
Esta não é uma página de um romance de ficção científica. A “semeadura de nuvens” é uma
prática real - na verdade, existe há décadas. ERRADA.
d) a “semeadura de nuvens” é usada para reduzir a precipitação em alguns países ao redor do
mundo – “It’s used today to boost precipitation in at least eight states across the western U.S. and
dozens of countries around the world.” = É usada hoje para aumentar a precipitação em pelo
menos oito estados no oeste dos EUA e dezenas de países ao redor do mundo. ERRADA.
e) os cientistas não sabem o quão exatamente bem funciona a semeadura de nuvens hoje em dia.
– “Scientists aren’t sure how well cloud seeding works today, let alone in a warmer climate.” = Os
cientistas não têm certeza do quanto funciona a semeadura de nuvens hoje, muito menos em um
clima mais quente. CORRETA.
GABARITO: E

Leia o texto a seguir e responda às questões 04, 05 e 06.


Impact of Covid-19 Pandemic on Mental Health
Aside from killing nearly 400,000 Americans to date and wreaking havoc on the country’s
economy, the COVID-19 pandemic is also taking a heavy toll on mental health. That’s according to
data compiled by the U.S. Census Bureau and the National Center for Health Statistics, showing
that more than 4 in 10 U.S. adults had developed symptoms of depression or anxiety by the end
of 2020, a sharp increase over the results of a comparable survey conducted in the first half of
2019.
As the survey shows, the share of respondents showing signs of anxiety or depression has nearly
quadrupled compared to results obtained before the pandemic. As hundreds of thousands have
died and millions have lost their jobs, Americans are facing a plethora of uncertainties with respect
to their and their families’ health and financial wellbeing, worries which are only exacerbated
when dealt with alone amid a time of social distancing.
Adapted from https://www.statista.com/chart/21878/.

Questão 04 (EsPCEx/INÉDITA) – According to the text, the pandemic


(A) a) makes a mild difference in mental health.
(B) b) caused a spike in anxiety and depression.
(C) c) is no longer related to mental health.
(D) d) decreased the share of Americans showing signs of anxiety or depression.
(E) e) has a meaningless impact on mental health.
Comentários: De acordo com o texto, a pandemia
a) faz uma pequena (mild = suave, pequena, insignificante) diferença na saúde mental – o texto
afirma que a pandemia fez muita diferença na saúde mental, aumentado os casos de depressão e

AULA 05 – PRONOUNS AND PREPOSITIONS 115


TEACHER ANDREA BELO

ansiedade. “… the COVID-19 pandemic is also taking a heavy toll on mental health … more than 4
in 10 U.S. adults had developed symptoms of depression or anxiety by the end of 2020, a sharp
increase over the results of a comparable survey conducted in the first half of 2019.” = ... a
pandemia de COVID-19 também está afetando a saúde mental ... mais de 4 em cada 10 adultos
norte-americanos desenvolveram sintomas de depressão ou ansiedade no final de 2020, um
aumento acentuado em relação aos resultados de uma pesquisa comparável realizada no primeiro
semestre de 2019. ERRADA.
b) causou um aumento na ansiedade e depressão – spike significa um aumento brusco, acentuado.
CORRETA, conforme vimos na alternativa anterior.
c) não está mais relacionada à saúde mental – o texto mostra que a pandemia ainda está, sim,
relacionada à saúde mental, impactando-a negativamente. ERRADA.
d) diminuiu a proporção de americanos que apresentam sinais de ansiedade ou depressão – como
vimos, essa proporção aumentou. “As the survey shows, the share of respondents showing signs
of anxiety or depression has nearly quadrupled compared to results obtained before the
pandemic.” = Como mostra a pesquisa, a proporção de entrevistados que mostram sinais de
ansiedade ou depressão quase quadruplicou em comparação com os resultados obtidos antes da
pandemia. ERRADA.
e) tem um impacto inexpressivo (meaningless) na saúde mental – ERRADA, conforme já explicado.
GABARITO: B

Questão 05 (EsPCEx/INÉDITA) – Choose the alternative containing the excerpt from the text that
best summarizes it.
(A) a) “… a comparable survey conducted in the first half of 2019”.
(B) b) “… when dealt with alone amid a time of social distancing”.
(C) c) “That’s according to data compiled by the U.S. Census Bureau and the National Center for
Health Statistics…”
(D) d) “As hundreds of thousands have died and millions have lost their jobs…”.
(E) e) “…the COVID-19 pandemic is also taking a heavy toll on mental health”.
Comentários: Escolha a alternativa que contém o trecho do texto que melhor o sintetiza.
a) “… uma pesquisa comparável realizada no primeiro semestre de 2019”.
b) “… quando se lida sozinho em um momento de distanciamento social”.
c) “Isso está de acordo com os dados compilados pelo U.S. Census Bureau e pelo National Center
for Health Statistics ...”
d) “Como centenas de milhares morreram e milhões perderam seus empregos…”.
e) “… a pandemia COVID-19 também está afetando fortemente a saúde mental”.
Como vimos acima, o texto trata essencialmente do impacto da pandemia na saúde mental das
pessoas e de como ela fez aumentar os casos de depressão e ansiedade. Assim, a alternativa que
contém o trecho do texto que melhor o sintetiza é a letra E. → “… a pandemia de COVID-19

AULA 05 – PRONOUNS AND PREPOSITIONS 116


TEACHER ANDREA BELO

também está afetando fortemente a saúde mental”. Os demais trechos estão relacionados ao
assunto, mas não sintetizam o texto.
GABARITO: E

Questão 06 (EsPCEx/INÉDITA) – In the sentence “Americans are facing a plethora of


uncertainties with respect to their and their families’ health…” (paragraph 2), the word plethora
means
(A) apparition
(B) variant
(C) wealth
(D) abundance
(E) stillness
Comentários: O trecho indicado pela questão quer dizer que “Os americanos estão enfrentando
uma infinidade, uma abundância, uma quantidade excessiva de incertezas com relação à saúde
deles e de suas famílias ...”. Plethora, assim, significa uma quantidade grande ou excessiva de
(algo), infinidade, abundância. Assim,
Na frase, a palavra plethora significa
a) aparição
b) variante
c) riqueza
d) abundância
e) quietude
GABARITO: D

Leia o texto a seguir e responda às questões 07, 08 e 09.


‘Black Mirror’ Finds Terror, and Soul, in the Machine
“Black Mirror” is hands down the most relevant program of our time, if for no other reason than
how often it can make you wonder if we’re all living in an episode of it. This prescient and
mordantly funny science-fiction anthology is smart enough to be just barely ahead of its time. It
doesn’t imagine interstellar civilizations or postapocalyptic scenarios. Instead, it depicts variations
on a near future transformed by information technology — our world, just a little worse.
“Black Mirror,” created for British television by Charlie Brooker, is a product of the 21st century
and its digital, virtual breakthroughs. It speaks to a culture of people who live virtual second lives
on social platforms. So, it’s concerned not with body snatchers but with the internet hive mind;
not nuclear winter but artificial intelligence; not the complications of time travel but the
implications of _______ able to offload human consciousness onto devices. Its view of technology

AULA 05 – PRONOUNS AND PREPOSITIONS 117


TEACHER ANDREA BELO

is not cold and robotic but deeply emotional, because — as with our smartphones — we’ve made
the machines extensions of our bodies and souls.
Typical of the Netflix large-portions ethos, a few new episodes are too long, and feel diluted
compared with the lapidary early seasons. Still, “Black Mirror” hasn’t lost its currency. Its title
refers to the glass screens of computers, tablets and phones, but the machines are not the danger
here: it’s the anonymous, antiseptic monstrousness they can empower.
The brilliance of “Black Mirror” is that it’s not about how technology imperils our humanity. It’s
about the all-too-human faces reflected in our own black mirrors, staring back at us.
Adapted from https://www.nytimes.com/2016/10/21/arts/television/review-black-mirror-finds-terror-and-soul-in-the-machine.html.

Questão 07 (EsPCEx/INÉDITA) – Choose the alternative containing the correct verb form to
complete the gap in the text.
(A) be
(B) been
(C) being
(D) was
(E) were
Comentários: Observe que temos, antes da lacuna, uma preposição, of. De modo geral, após
preposições, usamos um verbo com -ing, transmitindo a ideia de infinitivo. “… not the
complications of time travel but the implications of being able to offload human consciousness
onto devices.” = não as complicações da viagem no tempo, mas as implicações de ser capaz de
descarregar a consciência humana em dispositivos.
GABARITO: C

Questão 08 (EsPCEx/INÉDITA) – According to the text, choose the correct statement.


(A) despite “Black Mirror” being a program about technology, it can’t be described as
emotionless.
(B) the program title shows no relation whatsoever to the show’s main theme.
(C) the word “prescient” (first paragraph) is synonymous with “fictional”.
(D) the presence of interstellar civilizations and postapocalyptic scenarios in “Black Mirror”
makes it a typical sci-fi tv show.
(E) the possessive “its”, in bold in the second paragraph, refers to “Black Mirror”
Comentários: De acordo com o texto, escolha a afirmação correta.
a) apesar de “Black Mirror” ser um programa sobre tecnologia, não pode ser descrito como sem
emoção – segundo o texto, de fato, “Black Mirror” não pode ser descrito como um programa
sem apelo emocional, apesar de ser sobre tecnologia. “Its view of technology is not cold and
robotic but deeply emotional...”. = Sua visão da tecnologia não é fria e robótica, mas
profundamente emocional ... CORRETA.

AULA 05 – PRONOUNS AND PREPOSITIONS 118


TEACHER ANDREA BELO

b) o título do programa não mostra qualquer relação com o tema principal do programa – a
alternativa afirma que o título do programa não mostra nenhuma relação com o tema principal
do programa (whatsoever = de jeito nenhum). Na verdade, o título do programa faz referência à
tela “preta” espelhada do celular, de tablets e computadores, quando os dispositivos estão em
repouso, estando, assim, diretamente ligado à tecnologia. “Its title refers to the glass screens of
computers, tablets and phones, but the machines are not the danger here: it’s the anonymous,
antiseptic monstrousness they can empower.” = Seu título se refere às telas de vidro de
computadores, tablets e telefones, mas as máquinas não são o perigo aqui: é a monstruosidade
anônima e anti-séptica que elas podem fortalecer. ERRADA.
c) a palavra “presciente” (primeiro parágrafo) é sinônimo de “ficcional” – a palavra prescient
significa presciente (observe a formação da palavra: pré + ciente). Presciente é um adjetivo que se
refere ao que prevê o futuro, que “vê antes”, que tem ciência de algo antecipadamente. Não é
sinônimo de ficcional. ERRADA.
d) a presença de civilizações interestelares e cenários pós-apocalípticos em “Black Mirror” o torna
um típico programa de ficção científica – na verdade, o texto afirma que a falta de civilizações
interestelares e cenários pós-apocalípticos em “Black Mirror” ilustra que o programa é diferente
dos programas de ficção científica típicos, estando à frente de seu tempo. “This... anthology is
smart enough to be just barely ahead of its time. It doesn’t imagine interstellar civilizations or
postapocalypt ic scenarios. Instead, it depicts variations on a near future transformed by
information technology...” ERRADA.
e) o possessivo “its”, em negrito no segundo parágrafo, refere-se a “Black Mirror” – O possessivo
se refere ao século XXI, e não a “Black Mirror”. “Black Mirror, criado para a televisão britânica por
Charlie Brooker, é um produto do século XXI e seus avanços digitais e virtuais.” ERRADA.
GABARITO: A

Questão 09 (EsPCEx/INÉDITA) – In the sentence “‘Black Mirror’ is hands down the most relevant
program of our time, if for no other reason than how often it can make you wonder if we’re all
living in an episode of it.”, the underlined expression means
(A) thoroughly
(B) possibly
(C) undoubtedly
(D) mordantly
(E) equally.
Comentários: No trecho, hands down significa sem sombra de dúvidas, indubitavelmente,
indiscutivelmente. “‘Black Mirror’ is hands down the most relevant program of our time, if for no
other reason than how often it can make you wonder if we’re all living in an episode of it.” =
“‘Black Mirror ’é, sem dúvida, o programa mais relevante do nosso tempo, no mínimo pela
frequência com que pode fazer você se perguntar se estamos todos vivendo em um episódio
dele.” Assim, a expressão sublinhada significa

AULA 05 – PRONOUNS AND PREPOSITIONS 119


TEACHER ANDREA BELO

a) completamente
b) possivelmente
c) sem dúvida, indubitavelmente
d) mordazmente
e) igualmente
GABARITO: C

Leia o texto a seguir e responda à questão 10.


Whoooaaa Duuuuude: Why We Stretch Words in Tweets and Texts
On twitter, when a simple ha won’t do, there’s always hahahaaaa, haaaahaaaa, or even
hahahahahahahahahahahahaha, indicating you’ve just read the funniest thing you’ve ever seen.
(Or that you’re a sarcastic talking raccoon.) These are known as stretchable or lengthened words,
and now researchers from the University of Vermont have figured out just how pervasive they are
on Twitter, uncovering fascinating patterns about their use.
Stretchability is a powerful linguistic device that visually punches up a written word, imparting a
wide range of emotions. That goes for the gooooooaaaaaaal of a soccer announcer, a teenager’s
exasperated finallyyyyy, and a surfer’s aweeeeeesome. And booooy are they popular on Twitter.
Writing today in the journal PLOS One, the researchers detail how they combed through 100 billion
tweets, mapping how often these words are stretched, and how far they are elongated—haha
versus hahahahaaaa, for example.
Consider dude and its many formulations. “That can convey basically anything, like ‘Duuuuude,
that's awful,’” says University of Vermont applied mathematician Peter Sheridan Dodds, one of
the study’s coauthors. On the other hand, “Dude!” is different. “It could be excitement; it could
be joy,” says Dodds.
“I hate using exclamation marks because they just don't fit my personality,” I tell Dodds and his
coauthor, Chris Danforth, also an applied mathematician at University of Vermont. But I do stretch
words: “I’ve found myself recently in texts to friends or messages to coworkers doing thaaanks
with three As, to signify some sort of excitement and appreciation without having to use a stupid
exclamation mark.” “Just three?” asks Danforth. “That's restraint. Because two would not work.
Two is like, this person doesn't know how to spell. They've made a mistake.”
All right, sooooo, we use stretchable words all the time to convey extra meaning—sadness, anger,
excitement. And that can be particularly powerful on a platform like Twitter, whose inherent
brevity doesn’t exactly encourage nuanced communication. Those extra letters add some oomph
to a brief message, making it more attention-grabbing. “You're taking what we would think of as
the dictionary text and you're turning it into something visual,” says Danforth. “It can't be ignored
when you see 20 As in a row.”
Adapted from https://www.wired.com/story/whoooaaa-duuuuude-stretch-words/

AULA 05 – PRONOUNS AND PREPOSITIONS 120


TEACHER ANDREA BELO

Questão 10 (EsPCEx/INÉDITA) – When the text brings expressions like whooooole, booooy and
sooooo, it means that
(A) prolonging words in social networks and in text messages is being ironized.
(B) prolonging words when writing them in informal communication is not impacting.
(C) some language codes should be avoided in formal messages.
(D) prolonging words in text messages can create ambiguity.
(E) metalanguage is being applied, by using the code itself to explain it.
Comentários: Quando o texto traz expressões como whooooole, booooy e sooooo, ele quer dizer
que
a) o prolongamento das palavras nas redes sociais e nas mensagens de texto está sendo ironizado
– Ironizar é o ato de usar palavras ou expressões com um sentido contrário ao que carregam
normalmente, ou seja, diz-se uma coisa querendo dizer outra. Ao usar as palavras “esticadas”,
com letras a mais, o texto não ironizou o processo de prolongamento de palavras, mas, sim,
utilizou o próprio recurso para ilustrar seu uso. ERRADA.
b) prolongar as palavras ao escrevê-las em comunicação informal não causa impacto – É
justamente o contrário: o texto deixa evidente e visualmente perceptível o quanto esse recurso
de prolongar palavras é impactante na linguagem escrita. Aposto que, ao “bater o olho” no texto,
as palavras com várias letras extras foram a primeira coisa que chamou a sua atenção. ERRADA.
c) alguns códigos de linguagem devem ser evitados em mensagens formais – O texto não usa as
palavras prolongadas para dizer que devem ser evitadas. Apenas usa o próprio código de
linguagem para ilustrá-lo. ERRADA.
d) prolongar palavras em mensagens de texto pode criar ambiguidade – O texto, em nenhum
momento, associa ambiguity ao recurso de prolongar as palavras. Ele diz que prolongar palavras,
na verdade, deixa emoções e sentimentos mais claros, de forma visual. ERRADA.
e) a metalinguagem está sendo aplicada, usando o próprio código para explicá-lo – De fato,
quando o autor prolonga as palavras no corpo do texto, ele está fazendo uso da metalinguagem,
pois ele usa as próprias palavras “esticadas” para explicar e ilustrar o recurso linguístico de
“esticar” as palavras com o intuito de enfatizar um sentimento ou emoção que se deseja
comunicar. CORRETA.
GABARITO: E

AULA 05 – PRONOUNS AND PREPOSITIONS 121


TEACHER ANDREA BELO

CONSIDERAÇÕES FINAIS
Outra aula alcançada com sucesso – outro passo até a sua aprovação!
E, dia após dia, os tópicos aprendidos aumentam, seu conhecimento fica mais amplo, o
vocabulário que você conhece se estende e a tendência é melhorar e ser capaz de alcançar a
aprovação de fato.
Nota-se o avanço em seus estudos e, provavelmente, uma maior tranquilidade para
enfrentar os exercícios que surgem. E você vai se acostumando a equilibrar seus estudos de forma
sistematizada, estudando cada vez mais e com mais dedicação.
Outro detalhe importante para seu sucesso nos estudos é continuar fazendo listas de
palavras e verbos, participar das aulas complementares, fazer simulados e pedir ajuda quando
precisar.
Isso te ajudará nas questões futuras e torna você, como eu disse antes, um candidato mais bem
preparado e confiante para realizar uma excelente prova de vestibular.
É importante lembrar também do nosso Fórum de dúvidas, exclusivo do Estratégia
Militares. Será minha forma de responder você, esclarecer o que mais você precise saber para
que os conteúdos fiquem ainda mais claros em seus estudos, certo?
E, caso queira, acesse minhas redes sociais (Teacher Andrea Belo) para aprender mais
palavras e contar com dicas importantes, que colaboram diretamente com seus estudos dia após
dia.

AULA 05 – PRONOUNS AND PREPOSITIONS 122


TEACHER ANDREA BELO

REFERÊNCIAS BIBLIOGRÁFICAS
BARRETO, Tania Pedroza; GARRIDO, Maria Line; SILVA, João Antenor de C., Inglês Instrumental.
Leitura e compreensão de textos. Salvador, Ba UFBA, 1995, p. 64.
BROWN. H. Douglas. Principles of Language Learning and Teaching. Prentice Hall International,
1988.
COMPEDELLI, Samira Yousseff. Português, Literatura, Produção de texto & Gramática – São Paulo:
Ed. Saraiva, 2002.
CORREIA, Clese Mary P. Reading Specific Purposes. Salvador/ Ba: UFBA, 1999.
COSTEIRA, Adriana Araújo de M. Reading Comprehension Skills. João Pessoa/PB: ETFP, 1998.
CRYSTAL David. Cambridge University Press 1997. The Cambridge Encyclopedia of Language.
Cambridge University Press 1997
FREEMAN. Diane Larsen. MURCIA. Marianne Celce. The Grammar Book, 1999.
DYE, Joan., FRANFORT, Nancy. Spectrum II, III A Communicative Course in English. USA, Prentice
Hall, 1994.
FAVERO, Maria de Lourdes Albuquerque (org.). Dicionário de educadores no Brasil: da colônia aos
dias atuais. Rio de Janeiro: UFRJ, MEC, INEP, 1999.
FRANKPORT, Nancy & Dye Hoab. Spectrum II, III Prentice Hall Regents Englewood Cliffs, New Jersy,
1994.
GADELHA, Isabel Maria B. Inglês Instrumental: Leitura, Conscientização e Prática. Teresina:
EDUFFI, 2000.
GUANDALINI, Eiter Otávio. Técnicas de Leitura em Inglês: ESP – English For Specific Purposes:
estagio 1. São Paulo: Texto novo, 2002.
GRELLET, Françoise. Developing Reading Skills. Cambridge University Press, 1995
HOLAENDER, Arnon & Sanders Sidney. A complete English Course. São Paulo. Ed. Moderna, 1995.
HUTCHINSON, Tom & WATERS, Alan. English for Specific Purposes. Cambridge: Cambridge
University Press, 1996
KRASHEN. Stephen D. Second Language Acquisition and Second Language Learning, Prentice-Hall
International, 1988.
LAENG, Mauro. Dicionário de pedagogia. Lisboa: Dom Quixote, 1973.
LEFFA, Vilson J. Metodologia do ensino de línguas. In: BOHN, H.; VANDRESEN, P. (org.). Tópicos de
linguística aplicada: o ensino de línguas estrangeiras. Florianópolis: Editora da UFSC, 1988. p. 211-
231.
LIBERATO, Wilson. Compact English Book Inglês Ensino Médio. São Paulo: FTD, Vol. Único, 1998
Mc ARTHUR. The Oxford Companion to the English Language. Oxford University Press 1992
Fromkin. Victoria. An Introduction to Language
MARQUES, Amadeu. Inglês Série Brasil. ed. Atica. São Paulo: 2004. Vol. Único.

AULA 05 – PRONOUNS AND PREPOSITIONS 123


TEACHER ANDREA BELO

MURPHY, Raymond: Essencial Grammar in Use Oxford. New York Ed. Oxford University, 1997.
OLIVEIRA, Luciano Amaral. English For Tourism Students. Inglês para Estudantes de Turismo: São
Paulo, Rocca, 2001.
OLIVEIRA, Sara Rejane de F. Estratégias de leitura para Inglês Instrumental. Brasília: UNB, 1994.
QUINTANA, et alli. First Certificate. Master Class Oxford. New York, 2004: Ed. Oxford University.
PAULINO, Berenice F. et all. Leitura em textos em Inglês – Uma Abordagem Instrumental. Belo
Horizonte: Ed. Dos Autores, 1992.
PEREIRA, Edilberto Coelho. Inglês Instrumental. Teresina: ETFPI, 1998.
RODGES, Theodore. Jack C. Richards. Approaches and Methods in Language Teaching. Cambridge
University Press, 2001.
RODMAN Robert. Harcourt Brace 1993. English as a Global Language
STEWART, B., HAINES S. First Certificate, MasterClass. UK – Oxford 2004.
SILVA, João Antenor de C., GARRIDO, Maria Lina, BARRETO, Tânia Pedrosa. Inglês Instrumental:
Leitura e Compreensão de Textos. Salvador: Centro Editorial e Didático, UFBA. 1994
SOARES, Moacir Bretãs. Dicionário de legislação do ensino. 19.ed. Rio de Janeiro: FGV, 1981.
SOUZA, Adriana Srade F. Leitura em Língua Inglesa: Uma abordagem Instrumental. São Paulo:
Disal, 2005.
TUCK, Michael. Oxford Dictionary of Computing for Learners of English. Oxford: Oxford University
Press, 1996.
TOTIS, Verônica Pakrauskas. Língua Inglesa: leitura. São Paulo: Cortez, 1991.

Livros eletrônicos:
Dicionário Houaiss da Língua Portuguesa, Editora Objetiva, 2001.
MOURãO, Janaína Pereira. "Skimming x Scanning"; Brasil Escola. Disponível em
<https://brasilescola.uol.com.br/ingles/skimming-x-scanning.htm>. Acesso em 20 de março de
2019.
www.newsweek.com - Acesso em 18 de março de 2019.
http://www.galaor.com.br/tecnicas-de-leitura/ - Acesso em 19 de março de 2019.
Expressões Idiomáticas (continuação)" em Só Língua Inglesa. Virtuous Tecnologia da
Informação,2008-2019. Consultado em 03/04/2019 às 22:09. Disponível na Internet em
http://www.solinguainglesa.com.br/conteudo/Expressoes5.php

AULA 05 – PRONOUNS AND PREPOSITIONS 124


TEACHER ANDREA BELO

TRADUÇÕES
Covid won't be the last pandemic. Will we be better prepared for the next one?
Wouldn’t it be wonderful if we never had to face another pandemic? As we reflect on the past
year, the words that come to mind are “never again”. Never again should more than 120,000 lives
be lost to a contagious virus, a number that would have been unthinkable almost a year ago, when
government advisers thought that losing 20,000 people would be a good outcome. Nor should we
endure another year of lockdowns, and the pain of job losses and unemployment that followed.
And nobody ever wants to see children taken out of school for 21 weeks of the year again, unable
to learn in classrooms or see their friends.
But scientists are clear that we will face another pandemic in the future. Most agree the question
is when, not if. In 2018, I gave a talk at the Hay festival outlining a scenario that was loosely based
on recent news reports: a farmer becomes infected with an antibiotic-resistant bacterium from a
pig in China, spreading this infection through their community and then boarding a plane to the
UK. The worrying thing about this scenario is that such bacteria, which evade our antibiotics of
last resort, are already upon us. They could yet be the source of a future pandemic.
Or perhaps the next pandemic will be caused by a virus spilling over from animals to humans,
much like Covid-19. Recent reports that the first humans have been infected with avian influenza
at a poultry plant in Russia give a worrying glimpse of how close we live to such risks. Every time
a virus circulates among animals, particularly bats, rodents, livestock and birds, and then comes
into contact with humans, there’s a chance it will infect people and lead to human transmission.
Wherever the next pandemic comes from, Britain must think long term about its ability to respond
and must ensure it is prepared. Key to this will be working together with other governments to
establish robust response systems. Over the past year, the UK has already built many of the
structures that will be needed to address a future pandemic. We now have a huge testing
capacity, processes in place for fast-tracking vaccine development and protocols for undertaking
rapid research in hospitals. But more needs to be done at a global level to ensure we’re truly
prepared.
(Adapted from https://www.theguardian.com/commentisfree/2021/mar/24/covid-pandemic-prepared-investment-science)

Covid não será a última pandemia. Estaremos melhor preparados para o próximo?
Não seria maravilhoso se nunca tivéssemos que enfrentar outra pandemia? Ao refletirmos sobre
o ano passado, as palavras que vêm à mente são “nunca mais”. Nunca mais se deveriam perder
mais de 120.000 vidas por causa de um vírus contagioso, um número que seria impensável quase
um ano atrás, quando os conselheiros do governo pensaram que perder 20.000 pessoas seria um
bom resultado. Nem devemos suportar mais um ano de bloqueios e a dor da perda de empregos
e do desemprego que se seguiram. E ninguém quer ver as crianças fora da escola durante 21
semanas do ano novamente, incapazes de aprender nas salas de aula ou de ver os amigos.
Mas os cientistas têm certeza de que enfrentaremos outra pandemia no futuro. A maioria
concorda que a questão é quando, não se. Em 2018, dei uma palestra no festival Hay descrevendo
um cenário que foi vagamente baseado em notícias recentes: um fazendeiro é infectado por uma
bactéria resistente a antibióticos de um porco na China, espalhando a infecção por sua

AULA 05 – PRONOUNS AND PREPOSITIONS 125


TEACHER ANDREA BELO

comunidade e, em seguida, embarca em um avião para o Reino Unido. O preocupante neste


cenário é que essas bactérias, que fogem aos nossos antibióticos de último recurso, já estão sobre
nós. Eles ainda podem ser a fonte de uma futura pandemia.
Ou talvez a próxima pandemia seja causada por um vírus que se espalha dos animais para os
humanos, como o Covid-19. Relatórios recentes de que os primeiros humanos foram infectados
com a gripe aviária em uma avicultura na Rússia dão uma ideia preocupante de quão próximos
vivemos de tais riscos. Cada vez que um vírus circula entre animais, especialmente morcegos,
roedores, animais domésticos e pássaros, e então entra em contato com humanos, há uma
chance de infectar pessoas e levar à transmissão humana.
De onde quer que venha a próxima pandemia, a Grã-Bretanha deve pensar a longo prazo sobre
sua capacidade de resposta e deve garantir que está preparada. A chave para isso será trabalhar
em conjunto com outros governos para estabelecer sistemas de resposta robustos. No ano
passado, o Reino Unido já construiu muitas das estruturas que serão necessárias para lidar com
uma futura pandemia. Agora temos uma enorme capacidade de testes, processos em
funcionamento para o desenvolvimento rápido de vacinas e protocolos para a realização de
pesquisas rápidas em hospitais. Mas é preciso fazer mais em nível global para garantir que
estejamos realmente preparados.

Should you be grateful for a job?


It's become a common refrain: “I’m just grateful to have a job”.
The last year has wreaked undeniable havoc on the working world. Globally, the working hours
and income lost in 2020 added up to the equivalent of 255 million full-time jobs. Workplace
closures, layoffs and a steep rise in unemployment are enough to make anyone who’s managed
to hold onto their job feel some measure of gratitude – or, at least, pressure to be grateful.
That pressure pre-dates the pandemic. One of the most pervasive conversations around jobs is
that we should be thankful to be hired, especially when competition for a position is fierce.
Candidates are even expected to express the sentiment if they want to be hired in the first place:
it’s hard to imagine leaving an interview without saying how much you appreciate being
considered, or sending a thank-you email.
But it’s possible some of that gratitude is misplaced. Perhaps it’s not quite appropriate to be
thankful that an employer is ‘letting you’ work for them. And while gratitude can be objectively
good for you – research consistently associates giving thanks with increased happiness – it also
has a darker side that can make you more willing to put up with a situation that makes you
unhappy.
Differing obligations
Some workers may be much more inclined to feel grateful for their jobs than others.
Workers who expect to be hired or promoted may express less gratitude than those without
systemic advantages. This is often the case for white men, who experience more upward mobility
than other groups, and less bias that prevents them from securing jobs, or getting interviews in

AULA 05 – PRONOUNS AND PREPOSITIONS 126


TEACHER ANDREA BELO

the first place. For instance, multiple studies have shown résumés with “white-sounding” names,
and those that downplay racial cues, are significantly more likely to garner a response.
Imposter syndrome may also play a part: workers who aren’t confident they deserve their roles
may develop feelings of unworthiness, despite being qualified or skilled. Women are particularly
vulnerable to imposter syndrome, and may find themselves giving outsize thanks for their jobs.
And, in recent months, Latino and black Americans were significantly more likely to be affected
by pandemic-related lay-offs than white Americans. Those among these groups who have kept
their jobs are likely feeling pressure to express gratitude – even if they have to force it, and even
if their workplace doesn’t inspire much to be thankful for.
(Adapted from https://www.bbc.com/worklife/article/20210329-should-you-be-grateful-for-a-job)

Você deveria ser grato por um trabalho?


Tornou-se um refrão comum: “Estou muito grato por ter um emprego”.
O ano passado causou estragos inegáveis no mundo do trabalho. Globalmente, as horas de
trabalho e a renda perdidas em 2020 somaram o equivalente a 255 milhões de empregos em
tempo integral. Fechamentos de locais de trabalho, demissões e um aumento acentuado no
desemprego são o suficiente para fazer qualquer um que conseguiu manter seu emprego sentir
um pouco de gratidão - ou, pelo menos, pressão para ser grato.
Essa pressão é anterior à pandemia. Uma das conversas mais difundidas sobre empregos é que
devemos ser gratos por termos sido contratados, especialmente quando a competição por uma
posição é feroz. Espera-se até que os candidatos expressem o sentimento se quiserem ser
contratados: é difícil imaginar sair de uma entrevista sem dizer o quanto você aprecia ser
considerado ou enviar um e-mail de agradecimento.
Mas é possível que parte dessa gratidão esteja perdida. Talvez não seja muito apropriado ser grato
por um empregador "deixar você" trabalhar para eles. E embora a gratidão possa ser
objetivamente boa para você - pesquise consistentemente associando agradecimento a maior
felicidade - ela também tem um lado mais sombrio que pode torná-lo mais disposto a suportar
uma situação que o deixa infeliz.
Obrigações divergentes
Alguns trabalhadores podem estar muito mais inclinados a se sentir gratos por seus empregos do
que outros.
Os trabalhadores que esperam ser contratados ou promovidos podem expressar menos gratidão
do que aqueles sem vantagens sistêmicas. Esse costuma ser o caso dos homens brancos, que
experimentam mais mobilidade ascendente do que outros grupos, e menos preconceito que os
impede de conseguir empregos ou de conseguir entrevistas. Por exemplo, vários estudos têm
mostrado currículos com nomes “que parecem brancos” e aqueles que minimizam as pistas raciais
têm uma probabilidade significativamente maior de obter uma resposta.
A síndrome do impostor também pode desempenhar um papel: os trabalhadores que não estão
confiantes de que merecem seus papéis podem desenvolver sentimentos de indignidade, apesar
de serem qualificados ou qualificados. As mulheres são particularmente vulneráveis à síndrome
do impostor e podem se dar conta de que estão dando grandes agradecimentos por seus

AULA 05 – PRONOUNS AND PREPOSITIONS 127


TEACHER ANDREA BELO

empregos. E, nos últimos meses, os americanos latinos e negros tinham uma probabilidade
significativamente maior de serem afetados por demissões relacionadas à pandemia do que os
americanos brancos. Aqueles entre esses grupos que mantiveram seus empregos provavelmente
estão se sentindo pressionados a expressar gratidão - mesmo que tenham que forçá-lo, e mesmo
que seu local de trabalho não inspire muito a agradecer.

Covid: 'Israel may be reaching herd immunity'


This happens when enough of a population has protection against an infection that it stops being
able to spread – and even people who don't themselves have immunity are indirectly protected.
For Covid the estimated threshold for herd immunity is at least 65%-70%.
But scientists in the UK are more cautious.
Dr Sarah Pitt, a virologist at the University of Brighton, urged "extreme caution" in concluding that
herd immunity had been reached – something she believes will be difficult even at high
vaccination rates.
She said it was still too early to tell: "We need to see whether the cases in Israel continue to fall
and stay at low levels."
Reaching this level of population immunity is important to protect people who can't be vaccinated
or whose immune system is too weak to produce a good, protective response.
In Israel, more than half (5.3 million) its residents have been vaccinated and an additional 830,000
people have tested positive for the virus in the past, which should give them some natural
immunity.
That works out as roughly 68% of the population who are likely to have antibodies in their blood
which can fight off the virus.
Prof Eyal Leshem, a director at Israel's largest hospital, the Sheba Medical Center, said herd
immunity was the "only explanation" for the fact that cases continued to fall even as more
restrictions were lifted.
"There is a continuous decline despite returning to near normalcy," he said.
"This tells us that even if a person is infected, most people they meet walking around won't be
infected by them."
And cases are falling in all age groups including children, even though under-16s are not generally
being vaccinated.
(Adapted from https://www.bbc.com/news/health-56722186)

Covid: 'Israel pode estar alcançando imunidade de rebanho'


Isso acontece quando um número suficiente de uma população tem proteção contra uma infecção
que ela para de se espalhar - e mesmo as pessoas que não têm imunidade são protegidas
indiretamente.

AULA 05 – PRONOUNS AND PREPOSITIONS 128


TEACHER ANDREA BELO

Para a Covid, o limite estimado para imunidade de rebanho é de pelo menos 65% -70%.
Mas os cientistas do Reino Unido são mais cautelosos.
A Dra. Sarah Pitt, virologista da Universidade de Brighton, pediu "extrema cautela" ao concluir que
a imunidade coletiva foi alcançada - algo que ela acredita que será difícil mesmo com altas taxas
de vacinação.
Ela disse que ainda é muito cedo para dizer: "Precisamos ver se os casos em Israel continuam
caindo e se mantêm em níveis baixos."
Alcançar esse nível de imunidade populacional é importante para proteger as pessoas que não
podem ser vacinadas ou cujo sistema imunológico é muito fraco para produzir uma resposta
protetora boa.
Em Israel, mais da metade (5,3 milhões) de seus residentes foram vacinados e mais 830.000
pessoas testaram positivo para o vírus no passado, o que deve lhes dar alguma imunidade natural.
Isso equivale a cerca de 68% da população que provavelmente tem anticorpos no sangue que
podem combater o vírus.
O professor Eyal Leshem, diretor do maior hospital de Israel, o Sheba Medical Center, disse que a
imunidade coletiva era a "única explicação" para o fato de que os casos continuaram diminuindo,
mesmo quando mais restrições foram suspensas.
"Há um declínio contínuo, apesar de voltar à normalidade", disse ele.
"Isso nos diz que mesmo se uma pessoa estiver infectada, a maioria das pessoas que encontra
andando por aí não será infectada por ela."
E os casos estão caindo em todas as faixas etárias, incluindo crianças, embora os menores de 16
anos geralmente não sejam vacinados.

My Family Wants to Visit This Summer. Is Travel Safe Yet?


Navigating our semi-vaccinated world is full of tough questions like this one. Luckily, the U.S.
Centers for Disease Control and Prevention (CDC) has issued guidance that may help.
According to the CDC, fully vaccinated people can visit indoors and unmasked with low-risk
unvaccinated people from a single household. Recent research suggests getting both doses of the
provides Moderna or Pfizer-BioNTech vaccines about 90% protection against COVID-19 infections.
That means a person fully vaccinated with these shots has a pretty slim chance of infecting an
unvaccinated person, or vice versa.
But it’s important to read the fine print there. Since no vaccine is 100% perfect, you should only
visit with low-risk unvaccinated people—i.e., those who do not have medical conditions or other
factors that would increase their chances of having a severe case of COVID-19 if they somehow
did get infected. And you should only visit with one household of unvaccinated people at a time;
your vaccination status wouldn’t make it any safer for a bunch of unvaccinated people to spend
time together inside and unmasked.

AULA 05 – PRONOUNS AND PREPOSITIONS 129


TEACHER ANDREA BELO

Let’s assume the family members who want to visit meet both of those standards. Great! But
there’s still the question of travel. Right now, the CDC does not recommend non-essential travel
for unvaccinated people.
Even though airplanes have pretty good filtration systems, travelers encounter lots of other people
in fairly tight quarters during the average trip, says Cindy Prins, an associate professor of
epidemiology at the University of Florida. Each of those interactions raises the chances of COVID-
19 exposure. Plus, if your family members were unknowingly infected while traveling, or got
infected during the trip, they could potentially seed new cases in your area.
(Adapted from https://time.com/5954515/summer-travel-covid-vaccination/)

Minha família quer visitar este verão. A viagem é segura ainda?


Navegar em nosso mundo semivacinado está cheio de questões difíceis como esta. Felizmente, os
EUA.
Os Centros de Controle e Prevenção de Doenças (CDC) emitiram orientações que podem ajudar.
De acordo com o CDC, pessoas totalmente vacinadas podem visitar locais fechados e
desmascaradas com pessoas não vacinadas de baixo risco de uma única casa. Pesquisas recentes
sugerem a obtenção de ambas as doses das vacinas Moderna ou Pfizer-BioNTech com cerca de
90% de proteção contra infecções por COVID-19. Isso significa que uma pessoa totalmente
vacinada com essas injeções tem uma chance muito pequena de infectar uma pessoa não
vacinada, ou vice-versa.
Mas é importante ler as letras miúdas lá. Uma vez que nenhuma vacina é 100% perfeita, você só
deve visitar pessoas não vacinadas de baixo risco, ou seja, aqueles que não têm condições médicas
ou outros fatores que aumentariam suas chances de ter um caso grave de COVID-19 se de alguma
forma eles pegassem infetado. E você só deve visitar uma família de pessoas não vacinadas de
cada vez; seu status de vacinação não tornaria mais seguro para um grupo de pessoas não
vacinadas passarem algum tempo juntas dentro de casa e sem máscara.
Suponhamos que os membros da família que desejam visitar atendam a esses dois padrões.
Excelente! Mas ainda há a questão das viagens. No momento, o CDC não recomenda viagens não
essenciais para pessoas não vacinadas.
Embora os aviões tenham sistemas de filtragem muito bons, os viajantes encontram muitas outras
pessoas em locais bastante apertados durante a viagem normal, diz Cindy Prins, professora
associada de epidemiologia da Universidade da Flórida. Cada uma dessas interações aumenta as
chances de exposição ao COVID-19. Além disso, se os membros da sua família foram infectados
inconscientemente durante uma viagem ou foram infectados durante a viagem, eles podem
potencialmente semear novos casos na sua área.

AULA 05 – PRONOUNS AND PREPOSITIONS 130


TEACHER ANDREA BELO

Which processed foods are better than natural?


The language used to describe the foods we eat can have a huge effect on how we perceive them:
"organic", "artisan", "homemade" and "handpicked" foods sound slightly more tempting than the
prosaic "tinned", "rehydrated" or "freeze-dried".
Another adjective that can whet our appetites is "natural", while we tend to associate "processed"
food with long lists of ingredients we can't pronounce. But when it comes to our health – is natural
always better than processed?
Actually, naturalness doesn't automatically mean a food is healthy, says Christina Sadler, manager
at the European Food Information Council and researcher at the University of Surrey.
In fact, natural foods can contain toxins, and minimal processing can in fact make them safer.
Kidney beans, for instance, contain lectins, which can cause vomiting and diarrhoea. They're
removed by soaking the beans in water overnight and then cooking them in boiling water.
Processing also makes cow's milk safe to consume. Milk has been pasteurised since the late 1800s,
in order to kill harmful bacteria. Before this time, it was distributed locally, because there wasn't
good refrigeration in houses.
"Cows in cities were milked every day, and people would bring milk in carts back to their
neighbourhoods to sell it," says John Lucey, food science professor at the University of Wisconsin-
Madison.
"As cities got bigger, milk got further away and took longer to get to the consumer, which meant
pathogens could multiply."
Mounting evidence suggesting that some organisms in milk could be harmful led to the
development of heating devices for milk and the invention of pasteurisation, which was soon
adopted across Europe, and later in the US.
"It's one of the major public health success stories of the last century," Lucey says. "Just before
World War Two, around a quarter of all food and waterborne diseases came from milk. Now it's
less than 1%."
Processing can also help to retain nutrients in food we eat. For example, freezing, which is
classified as minimal processing, allows fruit and vegetables to retain nutrients that can otherwise
degrade while sitting in a fridge.
(Adapted from https://www.bbc.com/future/article/20210521-which-processed-foods-are-better-than-natural)

Quais alimentos processados são melhores do que naturais?


A linguagem usada para descrever os alimentos que comemos pode ter um efeito enorme em
como os percebemos: alimentos "orgânicos", "artesanais", "caseiros" e "escolhidos a dedo" soam
um pouco mais tentadores do que os prosaicos "enlatados", "reidratados" ou "liofilizado".
Outro adjetivo que pode abrir nosso apetite é "natural", enquanto tendemos a associar alimentos
"processados" a longas listas de ingredientes que não podemos pronunciar. Mas no que diz
respeito à nossa saúde - é natural sempre melhor do que processado?

AULA 05 – PRONOUNS AND PREPOSITIONS 131


TEACHER ANDREA BELO

Na verdade, naturalidade não significa automaticamente que um alimento é saudável, diz


Christina Sadler, gerente do Conselho Europeu de Informação sobre Alimentos e pesquisadora da
Universidade de Surrey.
Na verdade, os alimentos naturais podem conter toxinas e o processamento mínimo pode torná-
los mais seguros.
O feijão vermelho, por exemplo, contém lectinas, que podem causar vômitos e diarreia. Eles são
removidos deixando-os de molho em água durante a noite e depois cozinhando-os em água
fervente.
O processamento também torna o leite de vaca seguro para consumo. O leite foi pasteurizado
desde o final dos anos 1800, a fim de matar bactérias nocivas. Antes era distribuído localmente,
porque não havia boa refrigeração nas residências.
"Vacas nas cidades eram ordenhadas todos os dias e as pessoas traziam leite em carrinhos de
volta para seus bairros para vendê-lo", diz John Lucey, professor de ciência de alimentos da
Universidade de Wisconsin-Madison.
"À medida que as cidades cresciam, o leite ficava mais longe e demorava mais para chegar ao
consumidor, o que significava que os patógenos poderiam se multiplicar."
Evidências crescentes que sugerem que alguns organismos no leite podem ser prejudiciais
levaram ao desenvolvimento de dispositivos de aquecimento do leite e à invenção da
pasteurização, que logo foi adotada em toda a Europa e, mais tarde, nos Estados Unidos.
“É uma das maiores histórias de sucesso de saúde pública do século passado”, diz Lucey. "Pouco
antes da Segunda Guerra Mundial, cerca de um quarto de todas as doenças transmitidas por
alimentos e água vinham do leite. Agora é menos de 1%."
O processamento também pode ajudar a reter nutrientes nos alimentos que comemos. Por
exemplo, o congelamento, que é classificado como processamento mínimo, permite que frutas e
vegetais retenham nutrientes que, de outra forma, podem se degradar enquanto permanecem
na geladeira.

CDC studying reports of heart inflammation in young Covid vaccine recipients


Some teenagers and young adults who received Covid-19 vaccines have experienced heart
inflammation, a US Centers for Disease Control and Prevention advisory group said,
recommending further study of the rare condition.
In a statement dated 17 May, the CDC’s advisory committee on immunisation practices said it had
looked into reports that a few young vaccine recipients, predominantly male adolescents and
young adults, developed myocarditis, an inflammation of the heart muscle.
The condition often goes away without complications and can be caused by a variety of viruses,
the CDC group said.

AULA 05 – PRONOUNS AND PREPOSITIONS 132


TEACHER ANDREA BELO

CDC monitoring systems had not found more cases than would be expected in the population,
but members of the committee on vaccinations felt that healthcare providers should be made
aware of the reports of the “potential adverse event”, the committee said in the statement.
It did not say how many people had been affected and recommended further investigation.
(Adapted from https://www.theguardian.com/world/2021/may/23/cdc-studying-reports-of-heart-inflammation-in-young-covid-vaccine-recipients)

CDC estudando relatórios de inflamação do coração em jovens recipientes da vacina Covid


Alguns adolescentes e jovens adultos que receberam as vacinas Covid-19 tiveram inflamação do
coração, disse um grupo consultivo do Centro de Controle e Prevenção de Doenças dos EUA,
recomendando estudos adicionais sobre a rara doença.
Em uma declaração datada de 17 de maio, o comitê consultivo do CDC sobre as práticas de
imunização disse que examinou relatos de que alguns jovens receptores da vacina,
predominantemente adolescentes do sexo masculino e adultos jovens, desenvolveram
miocardite, uma inflamação do músculo cardíaco.
A condição geralmente desaparece sem complicações e pode ser causada por uma variedade de
vírus, disse o grupo do CDC.
Os sistemas de monitoramento do CDC não encontraram mais casos do que o esperado na
população, mas os membros do comitê de vacinação consideram que os profissionais de saúde
devem ser informados sobre os relatórios de “evento adverso potencial”, disse o comitê no
comunicado.
Não foi informado quantas pessoas foram afetadas e recomendou uma investigação mais
aprofundada.

Menthol Cigarettes Kill Smokers, Most of Them Black.


Tobacco companies gain goodwill by advertising to black readers — and by donating money to
civil rights organizations.
Menthol creates a cooling sensation in tobacco products. Some studies have shown that menthol
also acts as a mild anesthetic.
There is a movement to ban menthol cigarettes. Why? The mint-flavored products target black
Americans.
On the whole, black Americans smokers smoke less than white people do. But black smokers die
of causes linked to tobacco use at higher rates than white smokers. About 85 percent of black
smokers use Newport, Kool, and other menthol brands.
It is easy to become addicted to these brands. They also are harder to quit than tobacco without
menthol.
The African American Tobacco Control Leadership Council is for menthol bans. It says menthol
products are the “main vectors” of disease and death among black Americans.

AULA 05 – PRONOUNS AND PREPOSITIONS 133


TEACHER ANDREA BELO

__________ (9) white health advocates oppose flavored e-cigarettes, which include menthol.
There is concern about the use of menthol cigars and cigarillos among black teenagers. Most black
high school students smoke menthol tobacco products.
Tobacco companies target black communities with menthol cigarettes. They give away free
samples and offer discounts.
They sponsor concerts and special events. They advertise in newspapers and magazines geared
to black readership. They donate money to civil rights organizations.
The companies have also been donors to black political candidates. They have been supporters of
the Congressional Black Caucus.
The tobacco companies say they want to be good citizens. But it is hard for them to resist the
chance to make a profit.
_______ (11) friends in Congress work to defeat efforts to ban flavored tobacco products.
A lobbyist said, a menthol ban would infringe on the rights of adults who preferred it to plain
tobacco.
The tobacco industry has joined forces with civil rights activists. Among them is the Rev. Al
Sharpton. Sharpton visited black communities in California. He raised the fear that a menthol ban
would give the police an excuse to stop and frisk blacks.
Mr. Sharpton also helped to defeat a menthol ban in New York.
Observers point out that society is making marijuana legal. In what direction should public policy
go? The tobacco industry also will not go quietly. It is creating new products that are less harmful
to users.
Americans consume alcohol, tobacco and all manner of drugs, some legal, some not. The banning
of one type of cigarette is not efficient. It may not work.
Source: The New York Times March 22, 2021

Os cigarros mentolados matam os fumantes, a maioria deles negros.


As empresas de tabaco ganham boa vontade anunciando para leitores negros - e doando dinheiro
para organizações de direitos civis.
O mentol cria uma sensação de resfriamento nos produtos do tabaco. Alguns estudos mostraram
que o mentol também atua como um anestésico leve.
Existe um movimento para proibir os cigarros mentolados. Porque? Os produtos com sabor de
menta têm como alvo os negros americanos.
No geral, os fumantes americanos negros fumam menos do que os brancos. Mas os fumantes
negros morrem de causas ligadas ao uso do tabaco em taxas mais elevadas do que os fumantes
brancos. Cerca de 85% dos fumantes negros usam Newport, Kool e outras marcas de mentol.
É fácil ficar viciado nessas marcas. Eles também são mais difíceis de parar do que o tabaco sem
mentol.

AULA 05 – PRONOUNS AND PREPOSITIONS 134


TEACHER ANDREA BELO

O Conselho de Liderança para o Controle do Tabaco Afro-americano é a favor da proibição do


mentol. Ele afirma que os produtos mentolados são os “principais vetores” de doenças e morte
entre os negros americanos.
__________ (9) os defensores da saúde branca se opõem aos cigarros eletrônicos com sabor, que
incluem mentol.
Há preocupação com o uso de charutos e cigarrilhas de mentol entre adolescentes negros. A
maioria dos estudantes negros do ensino médio fuma produtos de tabaco mentol.
As empresas de tabaco têm como alvo as comunidades negras com cigarros mentolados. Eles
distribuem amostras grátis e oferecem descontos.
Eles patrocinam shows e eventos especiais. Eles anunciam em jornais e revistas voltados para o
público negro. Eles doam dinheiro para organizações de direitos civis.
As empresas também têm sido doadoras a candidatos políticos negros. Eles têm apoiado o
Congressional Black Caucus.
As empresas de tabaco dizem que querem ser bons cidadãos. Mas é difícil para eles resistir à
chance de lucrar.
_______ (11) amigos no Congresso trabalham para derrotar os esforços para proibir produtos de
tabaco aromatizados.
Um lobista disse que a proibição do mentol infringiria os direitos dos adultos que preferissem o
tabaco puro.
A indústria do tabaco uniu forças com ativistas dos direitos civis. Entre eles está o Rev. Al
Sharpton. Sharpton visitou comunidades negras na Califórnia. Ele levantou o temor de que a
proibição do mentol desse à polícia uma desculpa para parar e revistar os negros.
O Sr. Sharpton também ajudou a derrotar a proibição do mentol em Nova York.
Os observadores apontam que a sociedade está tornando a maconha legal. Em que direção as
políticas públicas devem ir? A indústria do tabaco também não vai ficar quieta. Ela está criando
novos produtos que são menos prejudiciais aos usuários.
Os americanos consomem álcool, tabaco e todos os tipos de drogas, algumas legais, outras não.
A proibição de um tipo de cigarro não é eficiente. Pode não funcionar.

Does motherhood belong on a resume?


Many have advocated that motherhood is a legitimate job that builds employable skills. Does
the title belong on mums' CVs?
Mums multi-task. They plan. They research, organise, negotiate, manage time and lead.
Although mothers’ juggling hasn’t ever been a secret, their role has, perhaps, never been more
obvious than during the pandemic. As schools transitioned to remote classrooms, and women
took on more of both the physical and mental load of home life than before, the skills required to
keep the trains on the tracks have been on full display.

AULA 05 – PRONOUNS AND PREPOSITIONS 135


TEACHER ANDREA BELO

As a result, the question increasingly floating to the surface is whether or not these skills have a
place on mothers’ CVs.
There’s long been a push to recognise motherhood as a legitimate job that trains workers in
legitimate skills, valuable to employers. And some voices are getting louder. One of the newest
leaders is HeyMama, a US-based community for working mums, who’ve launched a campaign
called Motherhood on the Resume. It’s quite literal, says Katya Libin, HeyMama’s co-founder and
CEO – the organisation is advocating for mothers to update their titles on LinkedIn, or even add
the position on a resume, like any other ‘recognised’ job in, say, sales or engineering.
Whether motherhood ‘belongs’ on a resume is, of course, subjective. The question, instead, lies
in whether mothers can reap tangible benefits for the addition of the title – or whether some
systemically entrenched biases around mums could produce the opposite effect.
(Adapted from https://www.bbc.com/worklife/article/20210617-does-motherhood-belong-on-a-resume)

A maternidade pertence a um currículo?


Muitos têm defendido que a maternidade é um trabalho legítimo que desenvolve habilidades
empregáveis. O título pertence aos currículos das mães?
Mums multi-tarefa. Eles planejam. Eles pesquisam, organizam, negociam, administram o tempo
e lideram.
Embora o malabarismo das mães nunca tenha sido um segredo, seu papel, talvez, nunca tenha
sido mais óbvio do que durante a pandemia. Com a transição das escolas para salas de aula
remotas e as mulheres assumindo mais carga física e mental da vida doméstica do que antes, as
habilidades necessárias para manter os trens nos trilhos ficaram em plena exibição.
Como resultado, a questão cada vez mais flutuando à superfície é se essas habilidades têm ou não
um lugar no currículo das mães.
Há muito tempo há um esforço para reconhecer a maternidade como um trabalho legítimo que
treina as trabalhadoras em habilidades legítimas, valiosas para os empregadores. E algumas vozes
estão ficando mais altas. Um dos mais novos líderes é HeyMama, uma comunidade sediada nos
Estados Unidos para mães que trabalham, que lançou uma campanha chamada Maternidade em
Movimento. É bastante literal, diz Katya Libin, cofundadora e CEO da HeyMama - a organização
está defendendo que as mães atualizem seus cargos no LinkedIn, ou mesmo adicionem a posição
em um currículo, como qualquer outro trabalho 'reconhecido' em, digamos, vendas ou
Engenharia.
Se a maternidade "pertence" a um currículo é, obviamente, subjetivo. A questão, em vez disso,
está em se as mães podem colher benefícios tangíveis com a adição do título - ou se alguns
preconceitos sistemicamente arraigados em torno das mães poderiam produzir o efeito oposto.

AULA 05 – PRONOUNS AND PREPOSITIONS 136


TEACHER ANDREA BELO

U.S. birthrates are plummeting. Increasing legal immigration can help.


THE DECADES-LONG decline in the U.S. birthrate accelerated in 2020, as the average number of
babies born to American women over the course of their lifetimes fell to its lowest level since
government record-keeping began nearly a century ago. After a decade in which the population
grew at the most sluggish pace since the 1930s, last year’s slowdown in births, which intensified
as the pandemic took hold, suggests a new demographic normal — one that poses daunting
economic and geopolitical challenges.
In a country where fertility is now well below the replacement rate required to compensate for
deaths, an obvious question arises: As the United States ages, how will a dwindling cohort of
younger, working-age Americans sustain the expensive social services that their parents and
grandparents are counting on in retirement?
The rational answer is a robust immigration system, one that affords the nation a ready supply of
scrappy, striving employees in jobs for which there are insufficient numbers of native-born
Americans, as well as a steady stream of well-educated professionals to fill engineering, scientific,
technology and medical jobs, among others. Immigrants are twice as likely to start new businesses
as native-born Americans and buttress economic growth — think of Tesla, Google and PayPal, all
started by entrepreneurs born elsewhere.
Yet as the 2020 Census demonstrated, immigration flattened out following the Great Recession
in 2008, was actively impeded by the Trump administration and fell once the coronavirus
tightened its grip. That reversed a trend toward rising numbers of immigrants since the 1960s,
when a half-century boom in foreign-born arrivals, the vast majority of them legal, helped fuel
economic growth.
Granted, no celestial mandate dictates that the United States must grow at a faster clip than other
developed countries, as it has for most of its history. But population stagnation may mean a very
different future, and probably a less vital one, than many Americans might imagine. In the short
run, it might mean jobs that go begging for workers as caretakers for the elderly, truck drivers,
computer programmers and other occupations. In the long run, it could impede the country’s
ability to maintain its status as a superpower, project influence and compete with China.
Among the many theories to explain the falling birthrate are women’s increasing labor-force
participation; the daunting cost of living, particularly housing, in job-rich urban areas; a dramatic
drop-off in teen pregnancy; and a social media-distracted younger generation that may be having
less sex. Whatever combination of factors is driving the baby bust, what’s clear is that there will
be no easy answer to reversing it, though family-friendly government and corporate policies could
help.
By contrast, increasing the level of legal immigration is a policy choice that comes with a direct
positive impact. Making that choice would entail forging a political consensus on a uniquely
divisive issue that President Donald Trump, among others, has helped turn into a question of tribal
identity after years in which it was a matter of bipartisan consensus. But not making it is likely
tantamount to acquiescing to an era of demographic stagnation and, over time, diminished
national stature.
(Adapted from https://www.washingtonpost.com/opinions/us-birthrates-are-plummeting-increasing-
legal-immigration-can-help/2021/06/25/b3e28236-aded-11eb-b476-c3b287e52a01_story.html)

AULA 05 – PRONOUNS AND PREPOSITIONS 137


TEACHER ANDREA BELO

As taxas de natalidade dos EUA estão despencando. O aumento da imigração legal pode ajudar.
O declínio DE LONGO DÉCADAS na taxa de natalidade dos EUA acelerou em 2020, quando o
número médio de bebês nascidos de mulheres americanas ao longo de suas vidas caiu para seu
nível mais baixo desde que os registros do governo começaram há quase um século. Depois de
uma década em que a população cresceu no ritmo mais lento desde 1930, a desaceleração do
ano passado nos nascimentos, que se intensificou com a chegada da pandemia, sugere um novo
normal demográfico - que apresenta desafios econômicos e geopolíticos assustadores.
Em um país onde a fertilidade está agora bem abaixo da taxa de reposição necessária para
compensar as mortes, surge uma questão óbvia: à medida que os Estados Unidos envelhecem,
como uma coorte cada vez menor de americanos mais jovens em idade produtiva manterá os
caros serviços sociais que seus pais e os avós estão contando na aposentadoria?
A resposta racional é um sistema de imigração robusto, que proporcione à nação um suprimento
imediato de funcionários problemáticos e esforçados em empregos para os quais não há número
suficiente de americanos nativos, bem como um fluxo constante de profissionais bem-educados
para preencher a engenharia , trabalhos científicos, tecnológicos e médicos, entre outros. Os
imigrantes têm duas vezes mais chances de abrir novos negócios do que os americanos nativos e
fomentar o crescimento econômico - pense em Tesla, Google e PayPal, todos iniciados por
empreendedores nascidos em outros lugares.
Ainda assim, como o Censo de 2020 demonstrou, a imigração se achatou após a Grande Recessão
em 2008, foi ativamente impedida pelo governo Trump e caiu quando o coronavírus aumentou
seu controle. Isso reverteu a tendência de aumento do número de imigrantes desde a década de
1960, quando um boom de meio século nas chegadas de estrangeiros, a grande maioria delas
legais, ajudou a impulsionar o crescimento econômico.
É verdade que nenhum mandato celestial impõe que os Estados Unidos cresçam mais rápido do
que outros países desenvolvidos, como aconteceu durante a maior parte de sua história. Mas a
estagnação da população pode significar um futuro muito diferente, e provavelmente menos
vital, do que muitos americanos podem imaginar. No curto prazo, pode significar empregos que
imploram por trabalhadores como zeladores de idosos, motoristas de caminhão, programadores
de computador e outras ocupações. No longo prazo, isso poderia impedir a capacidade do país de
manter seu status de superpotência, projetar influência e competir com a China.
Entre as muitas teorias para explicar a queda da taxa de natalidade estão a crescente participação
das mulheres na força de trabalho; o custo de vida assustador, especialmente a habitação, em
áreas urbanas ricas em empregos; uma queda dramática na gravidez na adolescência; e uma
geração mais jovem distraída com a mídia social que pode estar fazendo menos sexo. Seja qual
for a combinação de fatores que está causando a queda do bebê, o que está claro é que não
haverá uma resposta fácil para revertê-la, embora o governo favorável à família e as políticas
corporativas possam ajudar.
Em contraste, aumentar o nível de imigração legal é uma escolha de política que tem um impacto
positivo direto. Fazer essa escolha implicaria em forjar um consenso político sobre uma questão
exclusivamente divisiva que o presidente Donald Trump, entre outros, ajudou a transformar em
uma questão de identidade tribal depois de anos em que era uma questão de consenso
bipartidário. Mas não fazer isso provavelmente equivale a concordar com uma era de estagnação
demográfica e, com o tempo, diminuição da estatura nacional.

AULA 05 – PRONOUNS AND PREPOSITIONS 138


TEACHER ANDREA BELO

“More _______ us are starting to pick back up the strands of our pre-pandemic social lives. As we
figure out who the first people we want to meet up _______ are, we’re recognising there are
friendships _______ the ‘before times’ we didn’t keep up during lockdown – and aren’t
particularly excited _______ re-ignite now that we can”.
(Adapted from https://www.bbc.com/worklife/article/20210623-why-its-ok-to-let-friendships-fade-out)

“Mais _______ nós estamos começando a retomar as vertentes de nossa vida social pré-
pandêmica. À medida que descobrimos quem são as primeiras pessoas que queremos encontrar
_______, estamos reconhecendo que existem amizades _______ os 'tempos anteriores' que não
mantivemos durante o bloqueio - e não estamos particularmente animados _______ reacendem
agora que nós podemos".

“In June 2021, the US Food and Drug Administration (FDA) __________ the first Alzheimer’s drug
in 18 years: aducanumab (also known by its brand name Aduhelm). At the time of writing, the
drug __________ also under review in the EU, Japan, and several other countries. For the roughly
30 million people worldwide who __________ with Alzheimer’s, this is unprecedented news, and
must seem like cause for optimism”.
(Adapted from https://www.theguardian.com/commentisfree/2021/jun/28/alzheimers-drug-aducanumab-approval-dementia)

“Em junho de 2021, a Food and Drug Administration (FDA) dos EUA __________ o primeiro
medicamento para Alzheimer em 18 anos: aducanumabe (também conhecido por sua marca
Aduhelm). No momento em que este artigo foi escrito, a droga __________ também estava em
análise na UE, no Japão e em vários outros países. Para os cerca de 30 milhões de pessoas em
todo o mundo que __________ com Alzheimer, esta é uma notícia sem precedentes e deve
parecer motivo de otimismo ”.

“In the early days of the pandemic, economist Jeanet Bentzen of the University of Copenhagen
examined Google searches for the word ‘prayer’ in 95 countries. She identified that they hit an
all-time global high in March 2020 and increases occurred in lockstep with the number of COVID-
19 cases identified in each country. Stateside, __________ the Pew Research Center, 55 percent
of Americans prayed to end the spread of the novel coronavirus in March 2020, __________
nearly one quarter reported that their faith increased the following month, __________ limited
access to houses of worship”.
(Adapted from https://www.scientificamerican.com/article/psychiatry-needs-to-get-right-with-god/)

“Nos primeiros dias da pandemia, a economista Jeanet Bentzen, da Universidade de Copenhague,


examinou as pesquisas do Google pela palavra‘ oração ’em 95 países. Ela identificou que eles
atingiram um recorde global em março de 2020 e os aumentos ocorreram em sincronia com o

AULA 05 – PRONOUNS AND PREPOSITIONS 139


TEACHER ANDREA BELO

número de casos COVID-19 identificados em cada país. Nos Estados Unidos, __________ o Pew
Research Center, 55 por cento dos americanos oraram para acabar com a propagação do novo
coronavírus em março de 2020, __________ quase um quarto relatou que sua fé aumentou no
mês seguinte, __________ acesso limitado às casas de culto ”.

Hugs are coming back. Not everyone is thrilled.


When Stevi Stephens was 5 years old, her grandmother bent down for a hug, and Stephens
wondered if stepping on her foot would make her stop.
As a baby, her mother told her, Stephens cried when anyone held her; later, as a married woman,
she used to get up and change sides of the bed multiple times each night when her husband would
scoot over in his sleep to put an arm around her. “He was like a heat-seeking missile,” she says.
Don’t misunderstand: “I had a great sex life,” Stephens, now 76, clarifies. But Stephens’s attitude
toward being hugged hasn’t changed. With another person wrapped around her, she feels
restrained, uncomfortable.
Stephens, a retired anthropologist, lives in a small co-op of restored fishermen’s huts on
Vancouver Island with about nine other people. “Nobody’s ever tried to hug me here,” she says.
And for Stephens, the past 14 months of social distancing — and the freedom from almost any
hugs at all, even from her two grown children — has been blissful.
Ever since coronavirus vaccines became widely available to the general population, it’s been
evident in parks, restaurants and homes throughout America: Arms across your back are back.
Grandparents are hugging grandkids again.
Friends are hugging friends. Even epidemiologists, a notably cautious bunch, are hugging. For
many, the return of hugs has been a welcome step toward the return of normalcy.
Others, though, have been dreading this moment for a long time. In the frenzied, joyous rush to
make up for a year of lost embraces, it’s easy to lose sight of people like Stephens, who cringe at
the thought of having to endure a whole separate human body enveloping them with little to no
prior notice. Personal-space enthusiasts are sad to see their year of living huglessly come to an
end — even as they hold onto hope that some pandemic distancing habits might stick.
Stevi Stephens lives in a small co-op of restored fishermen’s huts on Vancouver Island. “Nobody’s
ever tried to hug me here,” she says. (Katie Rose MacKenzie)
There are reasons so many humans feel comforted by hugging. Like a massage, it “involves
stimulation and pressure receptors, and when that happens, the whole nervous system slows
down and stress hormone is reduced,” says Tiffany Field, director of the Touch Research Institute
at the University of Miami School of Medicine.
Still, for some, hugs induce stress more than they relieve it. When Sam Zelinka, a federal-
government research scientist based in Madison, Wis., gets a hug from anyone other than his wife
or kids, he gets the same feeling as when a stranger stands too close to him. And while Zelinka,
38, isn’t about to “spray paint a T-shirt and march in the streets yelling about no hugs,” he’s not
exactly looking forward to one from anyone but his parents.

AULA 05 – PRONOUNS AND PREPOSITIONS 140


TEACHER ANDREA BELO

Brooke Todd, a 24-year-old social worker in Stroudsburg, Pa., tends to tense up when someone
moves to hug her, and she didn’t totally make sense of how much calmer her social life had
become during the pandemic until last summer. “It took a few months, but suddenly I was like,
‘Oh, this is nice. I don’t look like a jerk all the time for not wanting to hug someone.’”
Certain kinds of people tend to dislike being touched or embraced, Field notes: some children and
adults with autism, for example, as well as many survivors of sexual assault, such as Todd. Plus,
Field says, the #MeToo movement prompted lots of people to raise a quizzical eyebrow at hugs
in the workplace. “I think that even before covid, hugging was already going out.”
(Adapted from https://www.washingtonpost.com/lifestyle/2021/05/15/hugs-greeting-pandemic/)

Os abraços estão voltando. Nem todo mundo está entusiasmado.


Quando Stevi Stephens tinha 5 anos, sua avó se abaixou para um abraço, e Stephens se perguntou
se pisar em seu pé a faria parar.
Quando bebê, sua mãe disse a ela, Stephens chorava quando alguém a segurava; mais tarde,
como uma mulher casada, ela costumava se levantar e mudar de lado da cama várias vezes todas
as noites, quando o marido se agitava durante o sono para colocar um braço em volta dela. “Ele
era como um míssil de busca de calor”, diz ela.
Não entenda mal: "Eu tive uma ótima vida sexual", esclarece Stephens, agora com 76 anos. Mas
a atitude de Stephens em relação a ser abraçado não mudou. Com outra pessoa enrolada em
torno dela, ela se sente contida, desconfortável.
Stephens, um antropólogo aposentado, vive em uma pequena cooperativa de cabanas de
pescadores restauradas na Ilha de Vancouver com cerca de outras nove pessoas. “Ninguém nunca
tentou me abraçar aqui”, diz ela. E para Stephens, os últimos 14 meses de distanciamento social
- e a liberdade de quase todos os abraços, mesmo de seus dois filhos adultos - foram maravilhosos.
Desde que as vacinas contra o coronavírus se tornaram amplamente disponíveis para a população
em geral, isso ficou evidente em parques, restaurantes e residências em toda a América: os braços
nas costas estão de volta. Os avós estão abraçando os netos novamente.
Amigos estão abraçando amigos. Até mesmo epidemiologistas, um grupo notavelmente
cauteloso, estão se abraçando. Para muitos, o retorno dos abraços foi um passo bem-vindo em
direção ao retorno da normalidade.
Outros, entretanto, há muito temem esse momento. Na corrida frenética e alegre para
compensar um ano de abraços perdidos, é fácil perder de vista pessoas como Stephens, que
estremece com a ideia de ter que suportar um corpo humano separado inteiro envolvendo-os
com pouco ou nenhum aviso prévio. Os entusiastas do espaço pessoal estão tristes ao ver o fim
de seu ano de vida imensamente infinita - mesmo que tenham esperança de que alguns hábitos
de distanciamento pandêmico possam persistir.
Stevi Stephens vive em uma pequena cooperativa de cabanas de pescadores restauradas na Ilha
de Vancouver. “Ninguém nunca tentou me abraçar aqui”, diz ela. (Katie Rose MacKenzie)
Existem razões pelas quais tantos humanos se sentem consolados com abraços. Como uma
massagem, "envolve receptores de estimulação e pressão e, quando isso acontece, todo o sistema

AULA 05 – PRONOUNS AND PREPOSITIONS 141


TEACHER ANDREA BELO

nervoso fica lento e o hormônio do estresse é reduzido", diz Tiffany Field, diretora do Touch
Research Institute da University of Miami School of Medicine.
Ainda assim, para alguns, os abraços induzem mais estresse do que o aliviam. Quando Sam
Zelinka, um cientista pesquisador do governo federal baseado em Madison, Wisconsin, recebe
um abraço de alguém que não seja sua esposa ou filhos, ele tem a mesma sensação de quando
um estranho fica muito perto dele. E embora Zelinka, 38, não esteja prestes a "pintar com spray
uma camiseta e marchar nas ruas gritando sobre não abraços", ele não está exatamente ansioso
para receber um de ninguém além de seus pais.
Brooke Todd, uma assistente social de 24 anos de Stroudsburg, Pensilvânia, tende a ficar tensa
quando alguém se move para abraçá-la, e ela não entendeu totalmente como sua vida social se
tornou mais calma durante a pandemia até o final verão. “Demorou alguns meses, mas de repente
eu estava tipo, 'Oh, isso é bom. Não pareço um idiota o tempo todo por não querer abraçar
alguém. '”
Certos tipos de pessoas tendem a não gostar de ser tocados ou abraçados, Field observa: algumas
crianças e adultos com autismo, por exemplo, bem como muitos sobreviventes de agressão
sexual, como Todd. Além disso, diz Field, o movimento #MeToo levou muitas pessoas a levantar
uma sobrancelha intrigada com os abraços no local de trabalho. “Acho que antes mesmo de ser
cobiçoso, abraçar já estava saindo.”

What You Need to Know About COVID-19 and Flu


Every winter is a bit of a roulette wheel when it comes to influenza. Flu vaccines work, but aren’t
100% effective in preventing disease, so it’s always a challenge convincing people to get their flu
shots. And while the symptoms are generally bearable, infections can become more severe and
even deadly among people who are older or who have underlying health conditions. Last flu
season, even though experts considered it a relatively mild year, about 400,000 people in the U.S.
were hospitalized and 22,000 people died from the flu.
This winter, the influenza virus has a rival—the coronavirus fueling the COVID-19 pandemic—and
health officials are anticipating a showdown that could have dire consequences for the health of
millions. Both diseases are caused by viruses that spread with abandon from person to person
through sneezes, coughs, and respiratory droplets during close contact.
But while researchers know quite a bit about the influenza virus, the coronavirus, SARS-CoV-2, is
very much a black box, and they can only guess what will happen when the two pathogens collide
throughout the world.
Why is it dangerous to have flu and COVID-19 viruses around at the same time?
A double whammy of flu and SARS-CoV-2 infections this winter could be devastating for public
health, as the colder weather brings people into closer contact with each other in confined spaces
indoors. “What I worry about is that both viruses can cause serious respiratory illness,” says Dr.
David Chokshi, health commissioner for New York City. “In the same way that COVID-19 often gets
to the point where someone has to be hospitalized to support their breathing, or needs to be in

AULA 05 – PRONOUNS AND PREPOSITIONS 142


TEACHER ANDREA BELO

the ICU with a breathing tube or ventilator, unfortunately we see that in the most severe cases of
influenza as well. What we worry about is that happening at the same time and really stressing
the capacity of our hospitals.”
Public health experts are concerned that too many people who are seriously ill with influenza or
COVID-19 could flood hospitals and stretch already worn out health care workers, and health care
systems, to their limits.
(Adapted from https://time.com/5917061/covid-19-flu-season/)

O que você precisa saber sobre COVID-19 e gripe


Todo inverno é uma espécie de roleta no que diz respeito à gripe. As vacinas contra a gripe
funcionam, mas não são 100% eficazes na prevenção de doenças, por isso é sempre um desafio
convencer as pessoas a tomar a vacina contra a gripe. E embora os sintomas sejam geralmente
suportáveis, as infecções podem se tornar mais graves e até mortais entre as pessoas mais velhas
ou com problemas de saúde subjacentes. Na última temporada de gripe, embora os especialistas
considerem um ano relativamente ameno, cerca de 400.000 pessoas nos EUA foram
hospitalizadas e 22.000 pessoas morreram de gripe.
Neste inverno, o vírus da gripe tem um rival - o coronavírus que alimenta a pandemia COVID-19 -
e as autoridades de saúde estão prevendo um confronto que pode ter consequências terríveis
para a saúde de milhões de pessoas. Ambas as doenças são causadas por vírus que se espalham
livremente de pessoa para pessoa por meio de espirros, tosses e gotículas respiratórias durante
o contato próximo.
Mas enquanto os pesquisadores sabem um pouco sobre o vírus da gripe, o coronavírus, SARS-
CoV-2, é uma caixa preta e eles só podem adivinhar o que acontecerá quando os dois patógenos
colidirem em todo o mundo.
Por que é perigoso ter os vírus da gripe e COVID-19 ao mesmo tempo?
Um golpe duplo de gripe e infecções por SARS-CoV-2 neste inverno pode ser devastador para a
saúde pública, já que o tempo mais frio aproxima as pessoas em espaços confinados dentro de
casa. “O que me preocupa é que ambos os vírus podem causar doenças respiratórias graves”,
disse o Dr. David Chokshi, comissário de saúde da cidade de Nova York. “Da mesma forma que o
COVID-19 muitas vezes chega ao ponto em que alguém tem que ser hospitalizado para suportar
sua respiração, ou precisa estar na UTI com um tubo de respiração ou ventilador, infelizmente
vemos isso nos casos mais graves de gripe também. O que nos preocupa é que isso aconteça ao
mesmo tempo e realmente estresse a capacidade de nossos hospitais. ”
Especialistas em saúde pública estão preocupados com o fato de que muitas pessoas gravemente
doentes com influenza ou COVID-19 possam inundar hospitais e levar os já desgastados
profissionais e sistemas de saúde aos seus limites.

AULA 05 – PRONOUNS AND PREPOSITIONS 143


TEACHER ANDREA BELO

Nine Tools for Better, Longer Sleep


For many of us, a restful night of sleep is hard to come by under the best of circumstances. But
against the backdrop of a long, cold winter and the ongoing pandemic, it can feel almost
impossible. While there will always be newfangled gadgets that claim to solve your sleep
problems with advanced technology — yes, there are such things as sleep robots and sleep-
tracking rings — newer doesn’t always mean better. Wirecutter, the New York Times Company
__________ (1) reviews and recommends products, has tested countless items __________ (2)
find real, proven solutions, from blackout curtains and sleep masks to better pillows and white-
noise machines.
In addition to the products they test for work, Wirecutter writers and editors have found their
own sleep solutions for these overworked, overstressed times. These aren’t necessarily products
we’ve rigorously tested (unless we’re talking about meditation apps), but they are ___(3) things
Wirecutter staffers are finding useful at home for falling, and staying, asleep.
(Adapted from https://www.nytimes.com/2021/03/09/realestate/nine-tools-for-better-longer-sleep.html)

Nove ferramentas para um sono melhor e mais longo


Para muitos de nós, uma boa noite de sono é difícil de conseguir, nas melhores circunstâncias.
Mas, tendo como pano de fundo um inverno longo e frio e a pandemia em curso, pode parecer
quase impossível. Embora sempre existam aparelhos inovadores que pretendem resolver seus
problemas de sono com tecnologia avançada - sim, existem coisas como robôs do sono e anéis de
rastreamento do sono - mais recente nem sempre significa melhor. Wirecutter, a New York Times
Company __________ (1) analisa e recomenda produtos, testou inúmeros itens __________ (2)
encontrou soluções reais e comprovadas, de cortinas blackout e máscaras de dormir a melhores
travesseiros e máquinas de ruído branco.
Além dos produtos que testam para o trabalho, os redatores e editores do Wirecutter
encontraram suas próprias soluções de sono para esses tempos de sobrecarga e sobrecarga de
trabalho. Estes não são necessariamente produtos que testamos rigorosamente (a menos que
estejamos falando sobre aplicativos de meditação), mas são ___ (3) coisas que os funcionários do
Wirecutter estão achando úteis em casa para cair e ficar dormindo.

If you bought a dog during lockdown, they'll need help coming out of it
Despite the lows of the past year, for many, lockdown has been the perfect opportunity to
welcome a dog into their lives. The demand for dogs during the pandemic has been huge, with a
60% increase in calls from people seeking to adopt from the Dogs Trust charity, and with many
other rescue organisations reporting similar findings. Google searches for “buy a puppy”
increased by 115% after the UK first went into lockdown in March 2020, with prices for some of
the most soughtafter breeds reaching record levels.
Dog ownership is a wonderful thing, but it is also a huge responsibility and a commitment that
spans way beyond lockdown: as the saying goes, “A dog is for life”. As restrictions ease and the

AULA 05 – PRONOUNS AND PREPOSITIONS 144


TEACHER ANDREA BELO

resumption of normality begins, it’s important we consider the implications for our canine
companions and give them a hand to help them adjust.
Having a dog around has helped many people cope with lockdown. Our dogs mostly love us being
around too: going for longer walks, having more playtime, and resting by our side. Nevertheless,
it’s safe to say life has not been normal for our dogs for most of the past year. Few have met other
dogs, and if they have seen them, it would have been from afar or on a lead, meaning that they
were unable to interact or play. There have also been fewer visitors coming into the home, but
probably more deliveries, with people coming to the door carrying parcels and going away again.
This is all particularly concerning for puppies acquired during the pandemic, as their expectation
of “normal” is lockdown life, and they may never have seen visitors inside the house or have been
left home alone.
We are all longing for a great British summer in which we can go on dog-date walks with a friend
and their dog, have family round for garden barbecues and take our pooches to the pub or cafe,
and of course, we need our dogs to be able to cope calmly with all of that. A return to normal is
something humans are able to process, understand and prepare for. But our dogs – especially
young ones – won’t understand why everything has changed. As far as our dogs know, normality
for them has been enjoying time with family only – so to be expected to cope with groups of
people, children and other dogs, both in and out of the home, could be overwhelming for them.
A big worry for dog owners is the long-term impact of lockdown on their ability to cope with being
left at home on their own. Dogs who had separation anxiety before the lockdown are likely to get
worse when left again as owners head back to work – but we also expect to see new cases
developing, because other dogs, and particularly puppies, have learned to expect company all
day.
One of the biggest reasons why dogs are rehomed is because of behaviour-related issues. A rise
in problematic behaviours after lockdown could mean families have no other option but to give
up their dog. And, really sadly, most of these problems can be prevented with the right early
experiences.
Our message to owners is to start preparing now, rather than waiting until things return more to
normal. It’s easy to do: start building up experiences of all the things we will expect them to do
once the lockdown eases. For instance, start building in minimal periods apart, initially just being
briefly separated from you by a door or child gate. If they stay calm, build up the time separated
really gradually, so they start to adjust to not being with you all the time. If your dog gets worried
when separated – barking, whining, panting, or scratching at the door – you have progressed too
fast. Go back to a shorter period to help them adjust. By gradually increasing your time apart, you
can ensure they are able to settle on their own and help them prepare for the time when you
need to return to work or study.
Our dogs will also need help when it comes to seeing friends and family, both outdoors and
indoors. Teaching your dog how to greet new people calmly, how to settle when guests visit or
when you’re in a cafe are key skills. Coming back when called, walking on a loose lead and not
barking when the doorbell goes are also vital skills that will set them up for success.
It is much easier to prevent problems than treat them, and it’s not too late to help prepare your
dogs for the changes coming. To support dog owners, there is online training, so dogs and their
owners can equip themselves with the skills they can put into practice as normality resumes.

AULA 05 – PRONOUNS AND PREPOSITIONS 145


TEACHER ANDREA BELO

When people take on the responsibilities of dog ownership, they do so with the best intentions
to care for them in the long term. But the pandemic will have devastating effects on some people’s
lives, including their ability to care for the ir dogs. While we provide lots of support to help keep
dogs and owners together, we’re also here for when things aren’t going so well, and owners may
be having trouble seeing a future with their dog. If anyone does find themselves struggling, and
may be considering having to rehome their dog, please contact Dogs Trust and we will do
everything we can to help.
(Adapted from https://www.theguardian.com/commentisfree/2021/apr/05/bought-dog-lockdown)

Se você comprou um cachorro durante o confinamento, eles precisarão de ajuda para resolvê-lo
Apesar das baixas do ano passado, para muitos, o confinamento tem sido a oportunidade perfeita
para dar as boas-vindas a um cão em suas vidas. A demanda por cães durante a pandemia tem
sido enorme, com um aumento de 60% nas ligações de pessoas que buscam adotar a instituição
de caridade Dogs Trust, e com muitas outras organizações de resgate relatando descobertas
semelhantes. As pesquisas no Google por “comprar um filhote” aumentaram 115% depois que o
Reino Unido entrou em bloqueio pela primeira vez em março de 2020, com os preços de algumas
das raças mais procuradas atingindo níveis recordes.
Ser dono de um cachorro é uma coisa maravilhosa, mas também é uma responsabilidade enorme
e um compromisso que vai além do bloqueio: como diz o ditado, “Um cachorro é para a vida”. À
medida que as restrições diminuem e a retomada da normalidade começa, é importante
considerarmos as implicações para nossos companheiros caninos e ajudá-los a se ajustar.
Ter um cachorro por perto tem ajudado muitas pessoas a lidar com o bloqueio. Nossos cães
também adoram que fiquemos por perto: dar mais passeios, ter mais tempo para brincar e
descansar ao nosso lado. No entanto, é seguro dizer que a vida não tem sido normal para nossos
cães durante a maior parte do ano passado. Poucos conheceram outros cães e, se os viram, teria
sido de longe ou com trela, o que significa que não puderam interagir ou brincar. Também houve
menos visitantes entrando em casa, mas provavelmente mais entregas, com pessoas chegando à
porta carregando pacotes e saindo novamente. Tudo isso é particularmente preocupante para
cachorros adquiridos durante a pandemia, já que sua expectativa de vida “normal” é uma vida
trancada e eles podem nunca ter visto visitantes dentro de casa ou ter sido deixados em casa
sozinhos.
Todos ansiamos por um grande verão britânico, no qual possamos passear com um amigo e o
cachorro deles, receber a família para churrascos no jardim e levar nossos cachorros ao bar ou
café e, claro, precisamos dos nossos cachorros ser capaz de lidar com tudo isso com calma. O
retorno ao normal é algo para o qual os humanos são capazes de processar, compreender e se
preparar. Mas nossos cães - especialmente os mais jovens - não vão entender por que tudo
mudou. Até onde nossos cães sabem, a normalidade para eles é aproveitar o tempo apenas com
a família - então, esperar que lidem com grupos de pessoas, crianças e outros cães, dentro e fora
de casa, pode ser opressor para eles.
Uma grande preocupação para os donos de cães é o impacto a longo prazo do bloqueio em sua
capacidade de lidar com o fato de ficarem sozinhos em casa. Os cães que tinham ansiedade de
separação antes do bloqueio tendem a piorar quando deixados novamente, à medida que os

AULA 05 – PRONOUNS AND PREPOSITIONS 146


TEACHER ANDREA BELO

donos voltam ao trabalho - mas também esperamos ver novos casos se desenvolvendo, porque
outros cães, principalmente cachorros, aprenderam a esperar companhia o dia todo.
Uma das principais razões pelas quais os cães são realojados é por causa de questões relacionadas
ao comportamento. Um aumento de comportamentos problemáticos após o confinamento pode
significar que as famílias não têm outra opção a não ser desistir de seus cães. E, realmente triste,
a maioria desses problemas pode ser evitada com as primeiras experiências corretas.
Nossa mensagem aos proprietários é para começar a se preparar agora, em vez de esperar até
que as coisas voltem ao normal. É fácil de fazer: comece a construir experiências de todas as coisas
que esperamos que eles façam assim que o bloqueio diminuir. Por exemplo, comece a construir
em períodos mínimos de intervalo, inicialmente apenas brevemente separado de você por uma
porta ou portão infantil. Se eles ficarem calmos, aumente o tempo separados realmente
gradualmente, para que eles comecem a se ajustar a não estar com você o tempo todo. Se o seu
cão fica preocupado quando se separa - latindo, ganindo, ofegando ou arranhando a porta - você
progrediu rápido demais. Volte para um período mais curto para ajudá-los a se ajustar.
Aumentando gradualmente o seu tempo separados, você pode garantir que eles sejam capazes
de se estabelecer por conta própria e ajudá-los a se preparar para o momento em que você precisa
voltar ao trabalho ou aos estudos.
Nossos cães também precisam de ajuda quando se trata de ver amigos e familiares, tanto ao ar
livre quanto dentro de casa. Ensinar seu cão a cumprimentar pessoas novas com calma, como se
acomodar quando os convidados o visitam ou quando você está em um café são habilidades
essenciais. Voltar quando chamado, andar com a guia solta e não latir quando a campainha toca
também são habilidades vitais que os prepararão para o sucesso.
É muito mais fácil prevenir problemas do que tratá-los, e não é tarde demais para ajudar a
preparar seus cães para as mudanças que estão por vir. Para apoiar os donos de cães, há
treinamento online, para que os cães e seus donos possam se equipar com as habilidades que
podem colocar em prática quando a normalidade for retomada.
Quando as pessoas assumem as responsabilidades de ter um cão, o fazem com as melhores
intenções de cuidar deles a longo prazo. Mas a pandemia terá efeitos devastadores na vida de
algumas pessoas, incluindo a capacidade de cuidar de seus cães. Embora forneçamos muito apoio
para ajudar a manter os cães e os donos juntos, também estamos aqui para quando as coisas não
estiverem indo tão bem e os donos possam ter problemas para ver um futuro com seus cães. Se
alguém se encontrar truggling e pode estar considerando ter que realojar seu cachorro, entre em
contato com a Dogs Trust e faremos tudo o que pudermos para ajudar.

Eight States Are Seeding Clouds to Overcome Megadrought


The mountaintops rumble to life unnaturally each year as snow clouds darken the sky across the
West. Open flames burst from the throats of metal chimneys, mounted on squat towers nestled
among the peaks. With a low hiss, puffs of particles belch from their mouths into the air, where
the wind catches them and whisks them away. These aren’t ordinary particles. They’re tiny bits of
crushed-up silver iodide, a crystal-like photosensitive substance once used in photography. But it’s
not used to take pictures out in the mountains. It’s meant to make snow.

AULA 05 – PRONOUNS AND PREPOSITIONS 147


TEACHER ANDREA BELO

As the wind whips the particles across the mountaintops, drafts of air sweep them higher into the
sky—so high that some of them eventually touch the clouds. There, an elegant transformation
takes place. The crystalline iodide particles have a structure similar to ice—and inside a cloud, like
attracts like. Water droplets begin to cluster around the particles, freezing solid as they gather
together.
These frozen clusters eventually grow too heavy to stay in the air. They fall from the cloud and drift
gently toward the Earth, dusting the mountaintops with fresh snow. This is not a page from a
science fiction novel. “Cloud seeding” is a real practice—in fact, it’s been around for decades.
It’s used today to boost precipitation in at least eight states across the western U.S. and dozens of
countries around the world. Interest in cloud seeding is growing as temperatures steadily rise,
increasing drought risks in places like the Mountain West. But there’s a catch. Scientists aren’t sure
how well cloud seeding works today, let alone in a warmer climate.
Amid growing concerns about water resources in the western U.S., scientists are working to
answer those questions.
Today, cloud seeding research represents the cutting edge of weather and climate science—a
convergence of questions about the influence of warming on our dwindling water resources and
our ability to control those consequences.
Adapted from https://www.scientificamerican.com/article/eight-states-are-seeding-clouds-to-overcome-megadrought/

Oito estados estão semeando nuvens para superar o megadrought


Os topos das montanhas ganham vida anormalmente a cada ano, enquanto as nuvens de neve
escurecem o céu no oeste. Chamas abertas saíram das gargantas das chaminés de metal,
montadas em torres baixas aninhadas entre os picos. Com um silvo baixo, lufadas de partículas
saem de suas bocas para o ar, onde o vento as agarra e leva embora. Estas não são partículas
comuns. Eles são pequenos pedaços de iodeto de prata esmagado, uma substância fotossensível
semelhante a um cristal que já foi usada em fotografia. Mas não é usado para tirar fotos nas
montanhas. É para fazer neve.
À medida que o vento chicoteia as partículas no topo das montanhas, rajadas de ar as levam para
o céu - tão alto que algumas delas acabam tocando as nuvens. Lá, ocorre uma transformação
elegante. As partículas cristalinas de iodeto têm uma estrutura semelhante à do gelo - e dentro
de uma nuvem, semelhante atrai semelhante. Gotículas de água começam a se agrupar em torno
das partículas, congelando-se à medida que se juntam.
Esses aglomerados congelados eventualmente ficam muito pesados para permanecer no ar. Eles
caem da nuvem e vagam suavemente em direção à Terra, cobrindo o topo das montanhas com
neve fresca. Esta não é uma página de um romance de ficção científica. "Semear na nuvem" é uma
prática real - na verdade, existe há décadas.
É usado hoje para aumentar a precipitação em pelo menos oito estados no oeste dos EUA e
dezenas de países ao redor do mundo. O interesse na semeadura de nuvens está crescendo à
medida que as temperaturas aumentam continuamente, aumentando os riscos de seca em
lugares como Mountain West. Mas há um problema. Os cientistas não têm certeza de como
funciona a semeadura de nuvens hoje, muito menos em um clima mais quente.

AULA 05 – PRONOUNS AND PREPOSITIONS 148


TEACHER ANDREA BELO

Em meio às crescentes preocupações com os recursos hídricos no oeste dos EUA, os cientistas
estão trabalhando para responder a essas perguntas.
Hoje, a pesquisa de semeadura de nuvens representa a vanguarda da ciência do tempo e do clima
- uma convergência de questões sobre a influência do aquecimento em nossos recursos hídricos
cada vez menores e nossa capacidade de controlar essas consequências.

Impact of Covid-19 Pandemic on Mental Health


Aside from killing nearly 400,000 Americans to date and wreaking havoc on the country’s
economy, the COVID-19 pandemic is also taking a heavy toll on mental health. That’s according to
data compiled by the U.S. Census Bureau and the National Center for Health Statistics, showing
that more than 4 in 10 U.S. adults had developed symptoms of depression or anxiety by the end
of 2020, a sharp increase over the results of a comparable survey conducted in the first half of
2019.
As the survey shows, the share of respondents showing signs of anxiety or depression has nearly
quadrupled compared to results obtained before the pandemic. As hundreds of thousands have
died and millions have lost their jobs, Americans are facing a plethora of uncertainties with respect
to their and their families’ health and financial wellbeing, worries which are only exacerbated
when dealt with alone amid a time of social distancing.
Adapted from https://www.statista.com/chart/21878/.

Impacto da pandemia de Covid-19 na saúde mental


Além de matar quase 400.000 americanos até o momento e causar estragos na economia do país,
a pandemia COVID-19 também está afetando fortemente a saúde mental. Isso está de acordo com
dados compilados pelo US Census Bureau e pelo National Center for Health Statistics, mostrando
que mais de 4 em cada 10 adultos norte-americanos desenvolveram sintomas de depressão ou
ansiedade até o final de 2020, um aumento acentuado em relação aos resultados de uma pesquisa
comparável realizado no primeiro semestre de 2019.
Como mostra a pesquisa, a proporção de entrevistados que mostram sinais de ansiedade ou
depressão quase quadruplicou em comparação com os resultados obtidos antes da pandemia.
Enquanto centenas de milhares morreram e milhões perderam seus empregos, os americanos
enfrentam uma infinidade de incertezas com relação à saúde e ao bem-estar financeiro de suas
famílias, preocupações que só são exacerbadas quando tratadas sozinhos em um momento de
distanciamento social.

‘Black Mirror’ Finds Terror, and Soul, in the Machine


“Black Mirror” is hands down the most relevant program of our time, if for no other reason than
how often it can make you wonder if we’re all living in an episode of it. This prescient and
mordantly funny science-fiction anthology is smart enough to be just barely ahead of its time. It

AULA 05 – PRONOUNS AND PREPOSITIONS 149


TEACHER ANDREA BELO

doesn’t imagine interstellar civilizations or postapocalyptic scenarios. Instead, it depicts variations


on a near future transformed by information technology — our world, just a little worse.
“Black Mirror,” created for British television by Charlie Brooker, is a product of the 21st century
and its digital, virtual breakthroughs. It speaks to a culture of people who live virtual second lives
on social platforms. So, it’s concerned not with body snatchers but with the internet hive mind;
not nuclear winter but artificial intelligence; not the complications of time travel but the
implications of _______ able to offload human consciousness onto devices. Its view of technology
is not cold and robotic but deeply emotional, because — as with our smartphones — we’ve made
the machines extensions of our bodies and souls.
Typical of the Netflix large-portions ethos, a few new episodes are too long, and feel diluted
compared with the lapidary early seasons. Still, “Black Mirror” hasn’t lost its currency. Its title
refers to the glass screens of computers, tablets and phones, but the machines are not the danger
here: it’s the anonymous, antiseptic monstrousness they can empower.
The brilliance of “Black Mirror” is that it’s not about how technology imperils our humanity. It’s
about the all-too-human faces reflected in our own black mirrors, staring back at us.
Adapted from https://www.nytimes.com/2016/10/21/arts/television/review-black-mirror-finds-terror-and-soul-in-the-machine.html.

‘Black Mirror’ Encontra Terror e Alma na Máquina


“Black Mirror” é sem dúvida o programa mais relevante de nosso tempo, nem que seja por outro
motivo, senão a frequência com que pode fazer você se perguntar se estamos todos vivendo em
um episódio dele. Esta antologia de ficção científica presciente e mordazmente engraçada é
inteligente o suficiente para estar um pouco à frente de seu tempo. Não imagina civilizações
interestelares ou cenários pós-apocalípticos. Em vez disso, ele descreve variações em um futuro
próximo transformado pela tecnologia da informação - nosso mundo, apenas um pouco pior.
“Black Mirror”, criado para a televisão britânica por Charlie Brooker, é um produto do século 21 e
seus avanços digitais virtuais. Ele fala a uma cultura de pessoas que vivem uma segunda vida
virtual em plataformas sociais. Então, não está preocupado com ladrões de corpo, mas com a
mente coletiva da internet; não inverno nuclear, mas inteligência artificial; não as complicações
da viagem no tempo, mas as implicações da capacidade de _______ de descarregar a consciência
humana em dispositivos. Sua visão da tecnologia não é fria e robótica, mas profundamente
emocional, porque - como com nossos smartphones - fizemos as máquinas extensões de nossos
corpos e almas.
Típico do ethos de grandes porções da Netflix, alguns novos episódios são muito longos e parecem
diluídos em comparação com as temporadas precárias. Ainda assim, “Black Mirror” não perdeu
sua moeda. Seu título se refere às telas de vidro de computadores, tablets e telefones, mas as
máquinas não são o perigo aqui: é a monstruosidade anônima e anti-séptica que elas podem
fortalecer.
O brilhantismo de "Black Mirror" é que não se trata de como a tecnologia ameaça nossa
humanidade. É sobre os rostos humanos refletidos em nossos próprios espelhos negros, olhando
para nós.

AULA 05 – PRONOUNS AND PREPOSITIONS 150


TEACHER ANDREA BELO

Whoooaaa Duuuuude: Why We Stretch Words in Tweets and Texts


On twitter, when a simple ha won’t do, there’s always hahahaaaa, haaaahaaaa, or even
hahahahahahahahahahahahaha, indicating you’ve just read the funniest thing you’ve ever seen.
(Or that you’re a sarcastic talking raccoon.) These are known as stretchable or lengthened words,
and now researchers from the University of Vermont have figured out just how pervasive they are
on Twitter, uncovering fascinating patterns about their use.
Stretchability is a powerful linguistic device that visually punches up a written word, imparting a
wide range of emotions. That goes for the gooooooaaaaaaal of a soccer announcer, a teenager’s
exasperated finallyyyyy, and a surfer’s aweeeeeesome. And booooy are they popular on Twitter.
Writing today in the journal PLOS One, the researchers detail how they combed through 100 billion
tweets, mapping how often these words are stretched, and how far they are elongated—haha
versus hahahahaaaa, for example.
Consider dude and its many formulations. “That can convey basically anything, like ‘Duuuuude,
that's awful,’” says University of Vermont applied mathematician Peter Sheridan Dodds, one of
the study’s coauthors. On the other hand, “Dude!” is different. “It could be excitement; it could
be joy,” says Dodds.
“I hate using exclamation marks because they just don't fit my personality,” I tell Dodds and his
coauthor, Chris Danforth, also an applied mathematician at University of Vermont. But I do stretch
words: “I’ve found myself recently in texts to friends or messages to coworkers doing thaaanks
with three As, to signify some sort of excitement and appreciation without having to use a stupid
exclamation mark.” “Just three?” asks Danforth. “That's restraint. Because two would not work.
Two is like, this person doesn't know how to spell. They've made a mistake.”
All right, sooooo, we use stretchable words all the time to convey extra meaning—sadness, anger,
excitement. And that can be particularly powerful on a platform like Twitter, whose inherent
brevity doesn’t exactly encourage nuanced communication. Those extra letters add some oomph
to a brief message, making it more attention-grabbing. “You're taking what we would think of as
the dictionary text and you're turning it into something visual,” says Danforth. “It can't be ignored
when you see 20 As in a row.”
Adapted from https://www.wired.com/story/whoooaaa-duuuuude-stretch-words/

Whoooaaa Duuuuude: Por que esticamos as palavras em tweets e textos


No twitter, quando um simples ha não serve, sempre há hahahaaaa, haaaahaaaa, ou mesmo
hahahahahahahahahahahahaha, indicando que você acabou de ler a coisa mais engraçada que já
viu. (Ou que você é um guaxinim falante sarcástico.) Essas palavras são conhecidas como palavras
extensíveis ou alongadas, e agora os pesquisadores da Universidade de Vermont descobriram o
quão difundidas elas são no Twitter, descobrindo padrões fascinantes sobre seu uso.
A capacidade de alongamento é um poderoso dispositivo linguístico que perfura visualmente uma
palavra escrita, transmitindo uma ampla gama de emoções. Isso vale para o gooooooaaaaaaal de
um locutor de futebol, um adolescente exasperado finalmenteyyyy, e um surfista
aweeeeeeeeeeeeeeeeeeeeee. E booooy, eles são populares no Twitter. Escrevendo hoje na revista
PLOS One, os pesquisadores detalham como vasculharam 100 bilhões de tweets, mapeando com

AULA 05 – PRONOUNS AND PREPOSITIONS 151


TEACHER ANDREA BELO

que frequência essas palavras são alongadas e até que ponto elas são alongadas - haha versus
hahahahaaaa, por exemplo.
Considere dude e suas muitas formulações. “Isso pode transmitir basicamente qualquer coisa,
como‘ Duuuuude, isso é horrível ’”, diz o matemático Peter Sheridan Dodds da Universidade de
Vermont, um dos co-autores do estudo. Por outro lado, “Cara!” é diferente. “Pode ser excitação;
pode ser uma alegria ”, diz Dodds.
“Odeio usar pontos de exclamação porque eles simplesmente não combinam com minha
personalidade”, digo a Dodds e seu co-autor, Chris Danforth, também matemático aplicado na
Universidade de Vermont. Mas eu estico as palavras: "Eu me peguei recentemente em textos para
amigos ou mensagens para colegas de trabalho fazendo thaaanks com três As, para significar
algum tipo de empolgação e apreciação sem ter que usar um ponto de exclamação estúpido."
"Apenas três?" pergunta Danforth. “Isso é restrição. Porque dois não funcionariam. Dois é tipo,
essa pessoa não sabe soletrar. Eles cometeram um erro. ”
Tudo bem, entããão, usamos palavras extensíveis o tempo todo para transmitir um significado
extra - tristeza, raiva, excitação. E isso pode ser particularmente poderoso em uma plataforma
como o Twitter, cuja brevidade inerente não incentiva exatamente a comunicação diferenciada.
Essas letras extras adicionam algum vigor a uma mensagem breve, tornando-a mais atraente.
“Você está pegando o que pensaríamos como o texto do dicionário e transformando-o em algo
visual”, diz Danforth. “Não pode ser ignorado quando você vê 20 As em uma fileira.”

AULA 05 – PRONOUNS AND PREPOSITIONS 152


AFA 2024

CONJUNCTIONS, DIRECT SPEECH


AND REPORTED SPEECH

AULA 06

Teacher Andrea Belo

www.estrategiamilitares.com.br www.militares.estrategia.com
TEACHER ANDREA BELO

SUMÁRIO
INTRODUÇÃO 3

DIRECT SPEECH 4

INDIRECT SPEECH/REPORTED SPEECH 5

CONJUNCTIONS 10

QUESTÕES 17

GABARITO 48

QUESTÕES COMENTADAS 49

CONSIDERAÇÕES FINAIS 110

REFERÊNCIAS BIBLIOGRÁFICAS 111

TRADUÇÕES 113

AULA 06 – CONJUNCTIONS, DIRECT SPEECH AND REPORTED SPEECH 2


TEACHER ANDREA BELO

INTRODUÇÃO
Vamos, então, à nossa aula sobre alguns tópicos considerados complexos: direct and
indirect ou reported speech (discursos direto e indireto) e, também, as conjunções.
Os discursos direto e indireto são usados quando queremos expressar as informações que
alguém nos relatou, nos contou. O discurso direto (direct speech) – ao relatar o que alguém disse,
usando as mesmas palavras que a pessoa utilizou, como mostrarei a você na teoria dessa aula.
E, por sua vez, o discurso indireto (indirect speech ou reported speech) – ao relatar o que
foi dito, porém, usando as nossas próprias palavras, que também explicarei com detalhes adiante.
As Conjunctions – conjunções – são palavras que ligam duas palavras, ligando, assim, duas
frases, das quais veremos muitas e inúmeros exemplos para que fique claro o uso delas nas
abundantes orações em que aparecem.
As línguas, de forma geral, são sistemas que devemos internalizar naturalmente, mas, como o
conhecimento da língua consiste em uma fragmentação de conteúdos em regras (e essas cheias
de exceções), fica complicado aprender e usar bem todos esses tópicos gramaticais, todo esse
conteúdo proposto em nossas aulas, eu sei disso.
Esses itens que selecionei para nossa aula 06 e que estudaremos agora, são de extrema
importância para solucionar questões em que o vestibular exige mais de um tempo verbal na
mesma frase e assim, tenta confundir você ou apenas, descobrir se você aprendeu bem os tempos
verbais que já foram estudados para que você tenha domínio de identificá-los, como você fará!
É claro que seu objetivo é ser aprovado. E, alcançar a aprovação depende de alguns passos,
tais como adotar uma postura positiva, estudar muito e dar o seu melhor. Assim, mais cedo ou
mais tarde vai alcançar sua vaga.
No caminho à aprovação, você vai resolver, durante a teoria e no fim do material,
exercícios de vestibulares anteriores bem como exercícios inéditos e, essas questões irão ajudar
você a colocar em prática o que aprende a cada dia.
Além disso, você avaliará seu conhecimento. Vamos lá! Você consegue e será o melhor
candidato!

AULA 06 – CONJUNCTIONS, DIRECT SPEECH AND REPORTED SPEECH 3


TEACHER ANDREA BELO

DIRECT SPEECH
O discurso direto em Inglês – direct speech – é usado quando queremos reproduzir
qualquer tipo de informação que nos é relatada. Podemos dizer que, em suma, o discurso direto
é utilizado para repetir o que uma outra pessoa disse do jeito exatamente que foi dito, sem
alteração. Veja:

She said: “I study every day because I want to be approved”.


(Ela disse: “Eu estudo todos os dias porque quero ser aprovada”.)

E sabe por que é importante estudar o discurso direto em relação ao vestibular?


Simplesmente pelo fato que é possível encontrar (e aparece muitas vezes) o discurso direto em
jornais e portais de notícias, a fim de enfatizar e, consequentemente, deixar a notícia mais direta
ou, às vezes, mais dramática, vejamos:

“I won’t resign”, says the president during the meeting.


(Não irei renunciar”, diz o presidente durante a reunião.)
“Our diplomatic relations are over”, affirm both candidates.
(Nossas relações diplomáticas acabaram”, afirmam ambos os candidatos.)

O discurso direto pode também ser encontrado em diálogos de narrativas ficcionais, pois
permite que traços da fala e de personalidade dos personagens envolvidos tenham destaque,
atraindo a atenção de seus leitores. Nos textos de vestibular podem aparecer por esse motivo:
enfatizar alguma parte do texto.

“I could tell you my adventures—beginning from this morning,” said Alice a little timidly;
“but it’s no use going back to yesterday, because I was a different person then.”
(“Eu poderia lhes contar minhas aventuras – começando por esta manhã", disse Alice um
pouco tímida; “mas não adianta voltar a ontem, porque eu era uma pessoa diferente.”)
Lewis Carrol, Alice no País das Maravilhas

Agora, vamos estudar o discurso indireto – indirect speech, também chamado de reported
speech, que, além de ter muitas possibilidades de uso, é mais recorrente ainda no vestibular.

AULA 06 – CONJUNCTIONS, DIRECT SPEECH AND REPORTED SPEECH 4


TEACHER ANDREA BELO

INDIRECT SPEECH/REPORTED SPEECH


O indirect speech/reported speech, ou discurso indireto, é, por sua vez, uma maneira de
falar sobre o que alguém disse, para repassar uma notícia, uma história.
A principal característica é que no discurso indireto, se fala na voz de quem está contando
a ação e não de quem a viveu.
Por esse motivo, existem algumas regras básicas para se usar bem o discurso indireto,
como por exemplo, a mudança dos tempos verbais.
Se você diz, por exemplo, que você quer um carro novo e alguém vai me contar, a
sequência é a seguinte:

- Eu quero um carro novo.


- Ele disse que queria um carro novo.

Em Inglês, é a mesma coisa. Observe que o verbo querer estava no presente quando você
falou (eu quero).
E foi automaticamente para o passado quando alguém contou o que você falou (ele/ela
disse que queria).
Veja em Inglês o exemplo do carro e outro, ilustrado:

(Você dizendo): – I want a new car.


(Alguém dizendo/contando o que ouviu): – He/She said that he/she wanted a new car.

(O pai dizendo): – I want to see your grades (Eu quero ver suas notas).
(A mãe dizendo à filha o que foi dito): – He said that he wanted to see your grades.
(Ele disse que ele queria ver as suas
notas).

AULA 06 – CONJUNCTIONS, DIRECT SPEECH AND REPORTED SPEECH 5


TEACHER ANDREA BELO

Veja algumas mudanças que acontecem com os verbos quando o discurso direto é
transformado em discurso indireto, com exemplos abaixo de cada tempo verbal:

VERB CHANGES
DIRECT SPEECH REPORTED SPEECH

Simple Present Simple Past

I study with you. He said that she studied with me.

Simple Past Past Perfect

I wrote the email. He said that he had written the email.

Present Continuous Past Continuous

I am working. He said he was working.

Past Continuous Past Perfect Continuous

I was shopping. He said that he had been shopping.

Present Perfect Past Perfect

I have eaten fast food. He said that he had been eaten fast food.

Will Would

I will visit you tomorrow. He said that he would visit me the next day.

Can Could

I can help you. He said he could me.

Você percebeu que em todas as frases eu usei “He said”, que pode ser acompanhado ou
não de “that”. Mas, além do verbo to say (passado said), podemos também usar o verbo to tell
(passado told) em frases com indirect/reported speech.
Vejamos exemplos:
I want a glass of water now. (Eu quero um copo de água agora.)
He told me (that) he wanted this glass of water. (Ele me disse que ele queria esse copo de água.)
I don’t want to work out today. (Eu não quero malhar.)
He told me (that) he didn’t want to work out yesterday. (Ele me disse que não queria malhar ontem.)

AULA 06 – CONJUNCTIONS, DIRECT SPEECH AND REPORTED SPEECH 6


TEACHER ANDREA BELO

A partir dos exemplos acima, podemos notar que algumas outras palavras, além dos
tempos verbais, se alteram com o discurso indireto. Se fosse, por exemplo, em Português:

- Eu comprei esse boné. (discurso direto)


- Ele disse que comprou aquele boné. (discurso indireto)
Em Inglês, fica:
- I have bought this cap. He said he had bought that cap.
- Eu comprei esse boné. Ele disse que ele tinha comprado aquele boné.

Veja as possíveis modificações no discurso indireto, que podem perguntar no vestibular.

TIME EXPRESSION CHANGES


DIRECT SPEECH INDIRECT SPEECH

Today That day

Yesterday The day before

Last night The night before

Now Then

Here There

Tomorrow The next day

This That (quando em expressão de tempo)

This, That The (quando adjetivos)

This, These It, Them (quando pronomes)

Pode ainda, haver outras alterações. Alguns lugares, além dos pronomes e das indicações
de tempo acima, também podem mudar com a passagem do discurso direto para o indireto.

They are meeting at my house.


(Eles estão se encontrando na minha casa.)
He said that they were meeting at his house.
(Ele disse que eles estavam se encontrando na casa dele.)

AULA 06 – CONJUNCTIONS, DIRECT SPEECH AND REPORTED SPEECH 7


TEACHER ANDREA BELO

I got here by train. (Eu cheguei aqui de trem.)


He said he had got there by train. (Ele disse que ele chegou lá de trem.)

E os verbos modais, já apresentados na aula 2, também mudam. Se você diz que pode fazer
algo, no momento que vou contar a alguém, digo: Ele(a) disse que podia fazer algo.
Assim como o posso se torna podia, em Inglês seria o can se tornar could, entendeu?
Vejamos a seguir a tabela e os exemplos para facilitar.

MODAL VERBS CHANGES


DIRECT SPEECH INDIRECT SPEECH

Can Could

May Might

Must Had to

Should Should

Ought to Ought to

I can dance rock. (Eu consigo dançar rock.)


He said he could dance rock. (Ele disse que ele conseguia dançar rock.)

She must study a lot. (Ela deve estudar muito.)


She said she had to study a lot. (Ela disse que ela teve que estudar muito.)

No texto abaixo do jornal The Economist, vou mostrar como uma das frases em discurso
direto poderia ter sido explorada para testar seus conhecimentos, como já foi feito em outras
provas:
“CHINA has begun to enter the age of mass car consumption. This is a great and historic advance.”
So proclaimed the state-run news agency, Xinhua, last year. Environmentalists may feel a twinge
of fear at this burgeoning romance with motoring. But a rapid social and economic transformation
is under way in urban China, and the car is steering it.
In 2002 demand for cars in China soared by 56%, far more than even the rosiest projections. The
next year growth quickened to 75%, before slowing in 2004 (when the government tightened
rules on credit for car purchases) to around 15%. But in a sluggish global market, China’s demand
remains mesmerising. Few expect this year’s growth to dip below 10%. As long as the economy

AULA 06 – CONJUNCTIONS, DIRECT SPEECH AND REPORTED SPEECH 8


TEACHER ANDREA BELO

goes on galloping at its current high-singledigit clip, many expect car sales to increase by 10-20%
annually for several years to come.
The Economist June 4th 2005

Na frase “Environmentalists may feel a twinge of fear…” (Ambientalistas podem sentir uma
pontada de medo...), no discurso direto, poderia ser questionado como ficaria no discurso indireto
ou se o verbo modal “may” mudaria na modificação de discurso direto para indireto, veja:
Questão: A frase “Environmentalists may feel a twinge of fear…” devidamente
transformada em discurso indireto, ficaria:
A) He said that environmentalists may felt a twinge of fear.
B) He said that environmentalists can felt a twinge of fear.
C) He said that environmentalists may be felling a twinge of fear.
D) He said that environmentalists may have felt a twinge of fear.
E) He said that environmentalists might feel a twinge of fear.
Comentários: Aqui, devemos analisar cada alternativa, para encontrar a melhor alternativa
para a sentença transformada em discurso indireto, de acordo com as regras vistas acima, na
explicação com quadros/tabelas que preparei para você.
A primeira coisa a se observar é que o verbo modal “may”, ao direcionar-se à uma frase do
discurso indireto, passa de may para might, lembra?
Na letra A, afirma-se que o verbo feel vai para o passado – felt, mas, ao analisar a teoria
acima, podemos perceber que, como eu disse anteriormente, may se transforma em might. Falsa.
Na letra B, afirma-se que o verbo modal may muda para can e que o verbo feel vai para o
passado – felt, mas, ao analisar a teoria acima, podemos perceber que, como eu disse
anteriormente, may se transforma em might e jamais seria trocado um verbo modal por outro,
pois, como vimos na aula de verbos, o sentido também muda de acordo com a situação em que
é encaixado. Falsa.
Na letra C, afirma-se que a estrutura inteira da frase muda de may feel para may be feeling,
no gerúndio, mas, ao analisar a teoria acima, podemos perceber que, como eu disse
anteriormente, may se transforma em might. Falsa.
Na letra D, afirma-se que estrutura inteira da frase muda de may feel para may have felt,
no present perfect, mas, ao analisar a teoria acima, podemos perceber que, como eu disse
anteriormente, may se transforma em might. Falsa.
A letra E, afirma-se que o verbo feel continua feel e o modal may muda para might,
exatamente como vimos na tabela ilustrativa. E podemos perceber que, como eu disse
anteriormente, may transformado em might está correto para discurso indireto com verbos
modais.
Agora, vamos aos estudos das conjunções. Let’s go! Veremos questões com esses
pronomes adiante para praticar.

AULA 06 – CONJUNCTIONS, DIRECT SPEECH AND REPORTED SPEECH 9


TEACHER ANDREA BELO

CONJUNCTIONS
As conjunções (Conjunctions), chamadas também de linking words, connectors ou
conectivos são palavras que ligam duas partes de uma oração, para que a sentença possa fazer
sentido. Observe um exemplo, através dessas duas orações:
➢ I wanted to text you. (Eu queria te mandar uma mensagem.)
➢ I don’t have your number. (Eu não tenho o número do seu celular.)
Elas têm uma relação, mas falta algo para unir essas frases e fazê-las ter um sentido maior:
➢ I wanted to text you, but I don’t have your number. (Eu queria te mandar uma
mensagem, mas eu não tenho o número do seu celular.)
Essa palavra but exerceu o papel que precisávamos: uniu as duas sentenças e estabeleceu
uma lógica entre elas. Portanto, o but é uma conjunção.

TIPOS DE CONJUNÇÕES
Há três tipos de conjunções em inglês:
✓ Conjunções Coordenadas (Coordinating Conjunctions)
✓ Conjunções Subordinadas (Subordinating Conjunctions)
✓ Conjunções Correlativas (Correlative Conjunctions)

CONJUNÇÕES COORDENADAS
As conjunções coordenadas são as mais comuns e as que geralmente usamos ou
identificamos quando pensamos em linking words. Elas têm o papel de juntar orações.
Elas podem juntar orações independentes (ou seja, orações que possuem sentido
completo por si próprias, sem precisar de outra oração para fazer sentido), frases ou apenas
palavras.
Na língua inglesa há sete conjunções coordenadas importantes:

For, And, Nor, But, Or, Yet, So

Vejamos as particularidades de cada uma delas.

AULA 06 – CONJUNCTIONS, DIRECT SPEECH AND REPORTED SPEECH 10


TEACHER ANDREA BELO

For [por]– Explica o motivo ou a proposta de algo (equivalente ao porquê).

I go to the park every week, for I love running.


[Eu vou ao parque toda semana, por amar correr.]
Peter though he had a great chance to be accepted at Oxford,
for his grandfather was the Dean of that university.
[Paul achava que tinha uma grande chance de ser aceito em Oxford,
por seu avô ser o reitor daquela Universidade.]

And [e] – Adiciona uma coisa à outra.

Daniel goes to the beach to surf and relax.


[O Daniel vai à praia para surfar e relaxar.]
I love red and white wine.
[Eu gosto de vinho branco e tinto.]

Nor [nem] – Utilizado para apresentar uma alternativa com ideia negativa à uma outra
ideia também negativa que já foi afirmada anteriormente.

The virus cannot live in immunized individuals, nor in the air.


[O vírus não pode viver em indivíduos imunizados, nem no ar.]
The guy didn’t have the chest of a body builder, nor did he have the six-pack abs.
[O cara não tinha o peitoral de um fisiculturista, nem tinha o abdômen tanquinho.]

But [mas] – Mostra contraste.

The game in the park is entertaining in the winter, but it’s better in the heat of summer.
[O jogo no parque é divertido no inverno, mas é melhor no calor do verão.]
She is very old but still sensual.
[Ela é muito velha, mas ainda é sensual.]

Or [ou] – Apresenta uma alternativa ou uma escolha.

Those men play on teams: shirts or skins.


[Aqueles homens jogam em times: com camiseta ou sem camiseta.]
Do you want a boy or a girl, Mom?
[Você quer um menino ou uma menina, mamãe?]

AULA 06 – CONJUNCTIONS, DIRECT SPEECH AND REPORTED SPEECH 11


TEACHER ANDREA BELO

Yet – Introduz uma ideia constratante que segue logicamente a ideia precedente, similar
ao “mas”.

I often take a book to read, yet I never seem to turn a single page.
[Eu frequentemente levo um livro para ler, mas parece que nunca viro uma só página.]
Dorian was the oldest of the girls, yet her accent was the most prominent.
[A Doriana era a mais velha das irmãs, mas seu sotaque era o mais proeminente.]

So [então, logo] – Indica efeito, resultado ou consequência.

I’ve started dating one of the soccer players, so now I have an


excuse to often watch the game.
[Eu comecei a namorar um dos jogadores de futebol, então agora
eu tenho uma desculpa para assistir aos jogos frequentemente.]
This is the easiest way to get there, so don’t say anything.
[Este é o caminho mais fácil para chegar lá, então não diga nada.]

Observe que as conjunções coordenadas geralmente ficam no meio de uma sentença e


uma vírgula é utilizada antes do linking word.
Com exceção se ambas as orações sejam muito curtas, neste caso a vírgula não será
utilizada.
Quando uma conjunção coordenada conecta duas orações independentes (ou seja, que
cada uma possui sentido sozinha, sem precisar da outra oração para fazer sentido), ela é
acompanhada da vírgula.
A vírgula será utilizada quando but expressar contraste.

CONJUNÇÕES SUBORDINADAS
Dos três tipos que há de conjunções, as subordinadas são as mais complexas de se
reconhecer, mas intuitivas de dominar.
As conjunções subordinadas introduzem as orações dependentes (ou seja, orações que
não possuem sentido completo por si próprias, elas precisam de outra oração para fazer sentido)
prendendo-as a uma oração independente (a que possui sentido completo por si só).
As conjunções subordinadas estabelecem uma relação de sentido entre a oração
dependente com o resto da frase. Há inúmeras conjunções subordinadas em inglês, as mais
comuns são:

AULA 06 – CONJUNCTIONS, DIRECT SPEECH AND REPORTED SPEECH 12


TEACHER ANDREA BELO

COMPARAÇÃO
POSSIBILIDADE LUGAR
CONTRASTE CAUSA E EFEITO TEMPO
CONDIÇÃO MODO
CONCESSÃO

ALTHOUGH BECAUSE
AFTER AS IF AS IF
(Apesar, (Porque – nas
(Depois) (Como se) (Como se)
Embora) respostas)

ASSUMING
EVEN THOUGH IN ORDER – THAT AS SOON AS THAT AS THOUGH
(Apesar de) (Para – que) (Assim que) (Assumindo (Como se)
que)

RATHER THAN SINCE BEFORE EVEN IF HOW


(Ao invés de) (Desde) (Antes) (Mesmo se) (Como)

BY THE TIME
THAN SO THAT IF NEXT
(No momento
(Do que) (De modo que) (Se) (Próximo)
que)

IN CASE –
THOUGH WHY NOW THAT THAT WHERE
(Embora) (Por quê) (Agora que) (Em caso – (Onde)
que)

WHERE AS WHEREVER
ONCE ONLY IF
(Enquanto - (Onde quer
(Uma vez) (Somente se)
que) que)

PROVIDED
WHETHER SINCE
- THAT -
(Se) (Desde)
(Devido a)

WHILE UNTIL UNLESS


- -
(Enquanto) (Até) (A menos que)

WHEN UNTIL
- - -
(Quando) (Até)

WHETHER
- - - -
(Se)

AULA 06 – CONJUNCTIONS, DIRECT SPEECH AND REPORTED SPEECH 13


TEACHER ANDREA BELO

Exemplos:

Although I’ve been here before, he’s just too hard to forget.
Embora eu já tenha vindo aqui antes, ele é muito difícil de esquecer.

I guess I’ll never be the same since I won this medal.


Eu acho que eu nunca mais serei o mesmo desde que eu ganhei essa medalha.

They are watching TV while my mom is making dinner.


Eles estão assistindo TV enquanto a minha mãe está fazendo o jantar.

If you leave her, she will die.


Se você deixá-la, ela morrerá.

I’m proud of you because you passed the exam.


Eu estou orgulhoso de você porque você passou no exame.

Talk to me before you leave.


Fale comigo antes de partir.

Once you go there, you never forget it.


Uma vez que você vai lá, você nunca esquece.

When I see you smile, I can do anything.


Quando eu vejo você sorrindo, eu consigo fazer qualquer coisa.

As orações podem ir em qualquer ordem, ou seja, tanto uma oração dependente como
uma independente podem começar a frase, mas o que nunca muda é que a conjunção
subordinada é a primeira palavra da oração dependente.

CONJUNÇÕES CORRELATIVAS
As conjunções correlativas estão sempre em grupo. Elas vêm em pares e você precisa
utilizar ambas em lugares diferentes em uma oração para fazer sentido.
Por esse motivo, elas têm esse nome justamente pelo fato delas trabalharem juntas (co-)
e por relacionar um elemento de uma sentença com outro (relação).

AULA 06 – CONJUNCTIONS, DIRECT SPEECH AND REPORTED SPEECH 14


TEACHER ANDREA BELO

Sua correlação sempre denota igualdade, e mostra a relação entre as ideias expressas em
diferentes partes da sentença:

as . . . as [como . . . como]
both . . . and [ambos . . . e]
either . . . or [ou . . . ou]
hardly . . . when [dificilmente. . . quando]
if . . . then [se . . . então]
just as . . . so [assim como . . . assim]
neither . . . nor [nem . . . nem]
no sooner . . . than [não antes . . . do que]
not . . . but [não . . . mas]
not only . . . but also [não somente . . . mas também]
rather . . . than [em vez . . . do que]
scarcely . . . when [mal . . . quando]
what with . . . and [o que com . . . e]
whether . . . or [se. . . ou]

Vejamos alguns exemplos:

I didn’t know whether you’d want the pizza or hamburger, so I got you both.
Eu não sabia se você iria querer pizza ou hamburger, então eu peguei os dois para você.

I want either the pizza or the hamburger.


Eu quero ou a pizza ou o hamburger.

I’ll eat them both – not only the pizza but also the hamburger.
Eu comerei os dois – não só a pizza, mas também o hamburger.

I’ll have both the pizza and the hamburger.


Eu vou querer ambos a pizza e o hamburger.

Há ainda uma outra conjunção, chamada de conjunção adverbial.


Ela estabelece uma ideia de conjunção que une duas orações, entretanto, por ter valor de
advérbio, não é comum que ela apareça junto às demais conjunções.

AULA 06 – CONJUNCTIONS, DIRECT SPEECH AND REPORTED SPEECH 15


TEACHER ANDREA BELO

As conjunções adverbiais mais comuns (apesar de pouco usadas) e que podem aparecer
em alguma prova de vestibular são:

SIMILARES A AND SIMILARES A BUT SIMILARES A SO

ALSO CONVERSELY ACCORDINGLY


(Também) (Inversamente) (Consequentemente)

BESIDES HOWEVER CONSEQUENTLY


(Além de) (Contudo) (Por conseguinte)

FURTHERMORE INTEAD OF HENCE


(Além disso) (Ao invés) (Portanto)

LIKEWISE NEVERTHELESS MEANWHILE


(Além disso) (Não obstante) (Enquanto isso)

OTHERWISE THEREFORE
(Caso contrário) (Portanto)

RATHER THUS
(Em vez) (Assim)

STILL
(Apesar)

Vamos aos exercícios de várias instituições para treinar.

AULA 06 – CONJUNCTIONS, DIRECT SPEECH AND REPORTED SPEECH 16


TEACHER ANDREA BELO

QUESTÕES
Você vai, agora, responder questões selecionadas de provas já realizadas em anos
anteriores. Depois, como em todas as nossas aulas, haverá o gabarito e as questões comentadas.
Vamos começar com questões AFA, de acordo coma sua instituição escolhida e depois,
vamos treinar de outras Carreiras Militares, para adquirir experiência e treinar vocabulário.
QUESTÕES AFA
Directions: Read the text below and answer questions 01 to 08 according to it.
Ignore the pessimism: Covid vaccines are quietly prevailing
Nightmare scenarios involving deadly new variants are making us all too gloomy – but there’s
a scientific case for optimism
It can be quite easy, reading the press, to believe that the pandemic will never end. Even when
good news about vaccines started to arrive in the autumn, this grim narrative managed to harden.
In the past month, you could read “five reasons that herd immunity is probably impossible”, even
with mass vaccination; breathless reports about yetuncharacterised but potentially ruinous
variants, such as the “double mutant” variant in India, or two concerning variants potentially
swapping mutations and teaming up in a “nightmare scenario” in California; get ready, some
analysts said, for the “permanent pandemic”.
Among many people I know, a sort of low-grade doom has set in. They think the vaccines are a
mere sliver of hope, only holding back the virus for a short time before being worn down by a rush
of ever-cleverer variants that will slosh around us, perhaps forever. Things might briefly get better,
they believe, but only by a little, and even that is tenuous.
However despite such dark talk, and the potential difficulties along the way of vaccine rollout, I
still remain optimistic. Since about the midway point of last year I have believed that extremely
potent vaccines are going to end the pandemic. They’ll do so by either driving the disease down
to near-extinction, or so constraining its force and spread that it becomes a manageable concern,
like measles or mumps. I actually think this will happen fairly soon, as long as we get everyone –
the whole world, not just the rich – vaccinated.
The scientific case for optimism is straightforward. The vaccines we have are beyond very good,
they’re among the most effective ever created. They appear to be potent in real-life situations,
and results so far show that protection is longlasting. Crucially, new results in the US show that
the mRNA vaccines used there effectively prevented coronavirus infections – not just serious
symptoms – in results similar to those previously reported by a UK-based study. And another
study in the UK suggested that vaccinated groups were less likely to spread coronavirus infection
overall. This is exactly what we need to choke out the pandemic: vaccines that don’t just protect,
but actually halt the virus infecting people and spreading.
When it comes to variants, it is clear that some are more infectious, and some are more deadly.
But their interaction with vaccines isn’t yet clear. Some lab-based results show that certain viral
mutations may make some immune responses less potent. And one study suggested the

AULA 06 – CONJUNCTIONS, DIRECT SPEECH AND REPORTED SPEECH 17


TEACHER ANDREA BELO

Oxford/AstraZeneca vaccine might be less potent against the South African variant. But the
majority of scientists believe that vaccines have so far held the line, and will continue to do so. If
variants continue to make small advances, vaccines can be updated. A doomsday strain may be
possible, but exceedingly hard to predict. Evolution isn’t an on-demand miracle worker for viral
supremacy; even over decades most viruses don’t escape vaccine protection.
(Adapted from https://www.theguardian.com/commentisfree/2021/apr/12/ignore-scare-stories-covid-vaccines-variants-scientific-optimism)

Questão 01 (AFA/INÉDITA) – The first paragraph of the text states that


A) Vaccination made everyone optimistic about the pandemic scenario
B) The media can make us more and more pessimistic about the end of the pandemic
C) The pandemic is less intense with each passing day
D) Most analysts see the pandemic as a permanent scenario

Questão 02 (AFA/INÉDITA) – In the phrase “Among many people I know, a sort of low-grade
doom has set in” (paragraph 2), the underlined word is a synonym for
A) Pessimism
B) Sadness
C) Disaster
D) Boon

Questão 03 (AFA/INÉDITA) – The passage “Things might briefly get better, they believe, but only
by a little, and even that is tenuous” (paragraph 2) suggests that
A) They believe that it’s possible that, in a short and fast way, things will improve
B) They believe that things will get better little by little
C) They believe that things tend to get worse, with no prospect of improvement
D) They believe that things will get better, even if only briefly

Questão 04 (AFA/INÉDITA) – The second paragraph of the text states that


A) Some people are overwhelmed by sadness in this pandemic context
B) Most people have a lot of confidence in the fact that the vaccine will permanently end the
pandemic
C) Some people believe that the pandemic is already coming to an end
D) Some people are being pessimistic about the pandemic even with the arrival of vaccination

AULA 06 – CONJUNCTIONS, DIRECT SPEECH AND REPORTED SPEECH 18


TEACHER ANDREA BELO

Questão 05 (AFA/INÉDITA) – In the phrase “...or so constraining its force and spread that it
becomes a manageable concern…” (paragraph 3), the underlined word is a synonym for
A) Restricting
B) Forbidding
C) Encouraging
D) Allowing

Questão 06 (AFA/INÉDITA) – The pronoun “they” (paragraph 4) refers to


A) Real-life situations
B) The whole world
C) Vaccines
D) Scientific case

Questão 07 (AFA/INÉDITA) – The passage “...despite such dark talk, and the potential difficulties
along the way of vaccine rollout, I still remain optimistic” (paragraph 3) suggests that
A) Bad thoughts and difficulties make us stronger in the pandemic
B) Optimism still remains despite all obstacles to vaccination
C) Optimism is unnecessary in the face of dark talks and difficulties
D) Optimism arises from the obstacles to vaccination

Questão 08 (AFA/INÉDITA) – The text


A) Expresses the duality present in the current pandemic situation
B) Criticizes pessimism about the end of the pandemic
C) Develops arguments against mass vaccination
D) Warns about fake news and its possible impact on vaccination

Directions: Read the text below and answer questions 09 and 10 according to it.
Sinopharm: Chinese Covid vaccine gets WHO emergency approval
The World Health Organization (WHO) has granted emergency approval for the Covid vaccine
made by Chinese state-owned company Sinopharm.
It is the first vaccine developed by a non-Western country to get WHO backing. The vaccine has
already been given to millions of people in China and elsewhere.
The WHO had previously only approved the vaccines made by Pfizer, AstraZeneca, Johnson &
Johnson and Moderna.

AULA 06 – CONJUNCTIONS, DIRECT SPEECH AND REPORTED SPEECH 19


TEACHER ANDREA BELO

But individual health regulators in various countries - especially poorer ones in Africa, Latin
America and Asia - have approved Chinese jabs for emergency use.
With little data released internationally early on, the effectiveness of the various Chinese vaccines
has long been uncertain.
But the WHO on Friday said it had validated the "safety, efficacy and quality" of the Sinopharm
jab.
The WHO said the addition of the vaccine had "the potential to rapidly accelerate Covid-19 vaccine
access for countries seeking to protect health workers and populations at risk".
It is recommending that the vaccine be administered in two doses to those aged 18 and over.
A decision is expected in the coming days on another Chinese vaccine developed by Sinovac, while
Russia's Sputnik vaccine is under assessment.
Why does WHO backing matter?
The green light from the global health body is a guideline for national regulators that a vaccine is
safe and effective.
WHO director-general Tedros Adhanom Ghebreyesus said it would give countries "confidence to
expedite their own regulatory approval".
It also means that the vaccine can be used in the global Covax programme, which was set up last
year to try to ensure fair access to vaccines among rich and poor nations.
The decision to list the Chinese vaccine for emergency use is expected to give a substantial boost
to the scheme, which has been struggling with supply problems.
(Adapted from https://www.bbc.com/news/world-asia-china-56967973)

Questão 09 (AFA/INÉDITA) – In the phrase “…the potential to rapidly accelerate Covid-19


vaccine access for countries seeking to protect health workers and populations at risk”
(paragraph 6), the underlined word is a synonym for
A) Hoping
B) Begging
C) Pursuing
D) Expecting

Questão 10 (AFA/INÉDITA) – The first paragraph of the text states that


A) Other vaccines from Eastern countries have already been approved by WHO
B) Before WHO approval, the vaccine use was restricted to China
C) The first non-Western vaccine was approved by WHO still in the testing phase
D) The vaccine was already being used even before WHO approval

AULA 06 – CONJUNCTIONS, DIRECT SPEECH AND REPORTED SPEECH 20


TEACHER ANDREA BELO

QUESTÕES COLÉGIO NAVAL


Read Text I to do questions 01 to 05 based on it.
The American dream
__________ (1) unitary definition does not exist–the American dream probably has a different
meaning to every US citizen. For some it is __________ (2) dream of freedom and equality, for
others it is the dream of a fulfilled life or even the dream of fame and wealth.
In general, the American dream can be defined as being the opportunity and freedom for all
citizens to achieve their goals and become rich and famous if only they work hard enough.
The first settlers hoped for freedom of religion; in their home countries they had been persecuted
because of __________ (3) religious and political views. Bad living conditions in Europe and the
hope for better living standards in America attracted more and more settlers to migrate to the
new world. The booming US industry during the first half of the 20th century caused the myth
“from rags to riches”.
Today’s role of the American dream is a matter of discussion. Some think that the American dream
is still a viable element today, for others it is only an illusion.
The American dream focusses on the success of the individuum (not the wellbeing of the whole
population). On the one hand, this has advantages, e.g. lower taxes, little interference of the state.
On the other hand, the same advantages are disadvantageous for society: there is no social fond
for the poor and there is no serious interest in important issues like environmental protection etc.
Adapted from: https://www.ego4u.com/en/read-on/countries/usa/american-dream

Questão 01 (COLÉGIO NAVAL/INÉDITA) – Choose the alternative containing the correct words to
respectively complete gaps (1) and (2)
(A) An / the
(B) The / a
(C) A / a
(D) The / The
(E) A / the

Questão 02 (COLÉGIO NAVAL/INÉDITA) – According to the text, the American dream


(A) has a clear and unique definition for all Americans.
(B) is only linked to making money and being famous.
(C) is not about freedom, but about the possibility of making a lot of money.
(D) has to do with freedom and the possibility that Americans can achieve their dreams.
(E) is something unique for all Americans.

AULA 06 – CONJUNCTIONS, DIRECT SPEECH AND REPORTED SPEECH 21


TEACHER ANDREA BELO

Questão 03 (COLÉGIO NAVAL/INÉDITA) – Choose the alternative containing the correct words to
respectively complete gap (3)
(A) your
(B) them
(C) their
(D) his
(E) yours

Questão 04 (COLÉGIO NAVAL/INÉDITA) – According to the text


(A) the american dream remains a very important thing for all Americans.
(B) the American dream has more to do with individual success than with the collective success
of the nation as a whole.
(C) the first settlers persecuted the Americans because of their religion.
(D) The American dream shows us only advantages, such as the state's little interference in the
economy.
(E) The American dream, despite being clear, is not an easy thing to achieve.

Questão 05 (COLÉGIO NAVAL/INÉDITA) – “…the American dream can be defined as being the
opportunity…”. The underlined verb expresses
(A) possibility.
(B) obligation.
(C) prohibition.
(D) permission.
(E) hability.

Questão 06 (COLÉGIO NAVAL/INÉDITA) – Read the sentences below


I. He had a bit of knowledge about the subject.
II. A lot of news was published.
III. They left an item of furniture at the apartment.
IV. I had an excellent idea.
Choose the option which defines the underlined nouns as U (uncountable) ou C (countable).
(A) U – U – U – C
(B) U – C – U – C
(C) U – U – C – U
(D) U – C – U – C
(E) C – U – C – U

AULA 06 – CONJUNCTIONS, DIRECT SPEECH AND REPORTED SPEECH 22


TEACHER ANDREA BELO

Questão 07 (COLÉGIO NAVAL/INÉDITA) – Mach the questions to the answers


I. How long does it take to take a shower?
II. Are you Thiago’s sister?
III. Where are you from?
IV. How far is our classroom from the bathroom?
( ) No, I’m not.
( ) It takes 15 minutes.
( ) It’s about 30 meters.
( ) I’m from London.
(A) I – II – IV – III
(B) II – I – IV – III
(C) I – II – III – IV
(D) II – IV – III – II
(E) IV – II – III – I

Questão 08 (COLÉGIO NAVAL/INÉDITA) – Mark the option in which all the verbs are written in
the Simple Past.
(A) began – hit – known – left – spoken.
(B) won – wore – threw – stolen – read.
(C) sank – put – spent – spoke – left
(D) went – was – wanted – rise – lose
(E) had – hit – bought – cut – done.

Questão 09 (COLÉGIO NAVAL/INÉDITA) – Choose the alternative in which you can find the
Present Continuous.
(A) Practicing exercises is a great way to avoid health problems.
(B) I am interested in studying English in England.
(C) I’ll never get used to working until late.
(D) She is American, but she's living in Canada nowadays.
(E) He never stopped smoking.

AULA 06 – CONJUNCTIONS, DIRECT SPEECH AND REPORTED SPEECH 23


TEACHER ANDREA BELO

Questão 10 (COLÉGIO NAVAL/INÉDITA) – All alternatives are not in the comparative, except:
(A) reader.
(B) smoker.
(C) matter.
(D) manner.
(E) higher.

QUESTÕES EAM
Read text I and answer questions 01, 02 and 03
In the first half of 2021, more than 8,100 people died in the US because of gun violence. It is
___________ (1) number in the last 20 years.
A report said that nearly 54 Americans died every day in shootings. It was 14 more people than in
the same time in the past six years. Last weekend, more than 120 people died in shootings.
Experts said that there were more reasons why gun violence was growing. More people owned a
gun, and there were a lot of problems between police and communities. Also, there were big
differences in the economy and healthcare, and there were racial problems in American society
which have lasted a very long time.
Adapted from: https://www.newsinlevels.com/products/guns-in-the-us-level-2/

Questão 01 (EAM/INÉDITA) – Choose the alternative containing the correct words to complete
gap (1)
(A) the highest
(B) higher than
(C) the most high
(D) more high
(E) the most highest

Questão 02 (EAM/INÉDITA) – In “A report said that nearly 54 Americans died every day in
shootings”, the word “nearly”, is closest in meaning to
(A) hardly.
(B) almost.
(C) near.
(D) close.
(E) always.

AULA 06 – CONJUNCTIONS, DIRECT SPEECH AND REPORTED SPEECH 24


TEACHER ANDREA BELO

Questão 03 (EAM/INÉDITA) – In “and there were a lot of problems between police and
communities”. The expression “a lot of” can be replaced by
(A) lots.
(B) much.
(C) few.
(D) little.
(E) many.

Read the following text to answer questions 04 and 05.


The construction of the Empire State Building began in 1930. It was built by 3,400 workers who
were mostly poor immigrants from Europe. They worked in any weather condition and with
simple tools. It was extremely dangerous work, but they made good money.
The building was constructed under great time pressure. They had to finish the work by May 1,
1931. The workers built 4 new floors every week and they finished the work in time.
The building is 381 meters high and has 102 floors. It has 6,500 windows and 73 elevators. Over
60,000 tons of steel were used for the construction. At the time, it was the tallest skyscraper in
the world.
Today, the building is still a symbol of New York. Thousands of people visit it every day. There are
also offices of many companies in the building and 20,000 people work there.
https://www.daysinlevels.com/article/the-day-the-empire-state-building-opened-level-2/

Questão 04 (EAM/INÉDITA) – In “They had to finish the work by May 1, 1931”. The word “they”
refers to
(A) the Empire State Building.
(B) good money.
(C) poor immigrants.
(D) workers.
(E) dangerous work.

Questão 05 (EAM/INÉDITA) – According to the text


(A) The Empire State Building was built by more than six thousand five hundred workers
(B) The Empire State Building was designed by a group of European entrepreneurs.
(C) The Empire State Building has long been considered the tallest building in the world. Today it
remains a symbol of New York City.
(D) The workers who were involved in the construction of the Empire State Building besides
working in any weather conditions and with simple tools they still earned little money.
(E) Despite having to finish the work by May 1, 1931, the Empire State Building was built without
haste as everything was under control.

AULA 06 – CONJUNCTIONS, DIRECT SPEECH AND REPORTED SPEECH 25


TEACHER ANDREA BELO

QUESTÕES EEAR
Read the text and answer questions 01 to 04.
Man living alone on Italian island to leave after 32 years
An Italian man is to leave the island he has lived on for more than 30 years after pressure from
authorities.
Mauro Morandi, 81, moved to Budelli off northern Sardinia in 1989.
Last year, however, Mr Morandi said the owners of the island, which is famed for its pink beach,
wanted him to relocate.
"I will leave hoping that in the future Budelli will be protected as I have protected it for 32 years,"
he wrote on Facebook on Sunday.
Speaking to BBC Outlook in 2018, the former physical education teacher said he had "always been
a bit of a rebel".
"I was quite fed up with a lot of things about our society: consumerism and the political situation
in Italy," Mr Morandi said. "I decided to move to a desert island in Polynesia, away from all
civilisation. I wanted to start a new life close to nature."
After beginning their journey by sea, he and a number of friends landed on the Italian archipelago
of La Maddalena, where they planned to work and make money to fund the rest of their travels.
But after arriving on Budelli and meeting the island's guardian, who was about to retire, Mr
Morandi decided to stay - and has remained ever since, looking after the island and speaking to
tourists.
However, he has faced a number of eviction threats over the years.
In 2020 the president of La Maddalena's National Park, Fabrizio Fonnesu, told CNN that Mr
Morandi had carried out illegal alterations to his hut, a former radio station dating back to World
War Two.
A petition calling on the Italian government to allow him to stay gathered more than 70,000
signatures.
But on Sunday, Mr Morandi announced his decision to leave, telling the The Guardian newspaper
that he would move to a small flat on the nearby island of La Maddalena: "My life won't change
too much, I'll still see the sea."
(Adapted from https://www.bbc.com/news/world-europe-56885716)

Questão 01 (EEAR/INÉDITA) – According to the text, we can infer that ________


(A) Mauro's departure will occur on his own free will
(B) Despite leaving his home, Mauro still wants the best for the island
(C) The owners of the island are famous for having a pink beach
(D) Mauro began to become rebellious in recent years

AULA 06 – CONJUNCTIONS, DIRECT SPEECH AND REPORTED SPEECH 26


TEACHER ANDREA BELO

Questão 02 (EEAR/INÉDITA) – The passage “...he and a number of friends landed on the Italian
archipelago of La Maddalena, where they planned to work and make money to fund the rest of
their travels” states that the stay should be
(A) Permanent
(B) Pleasant
(C) Temporary
(D) Uncomfortable

Questão 03 (EEAR/INÉDITA) – The sentence in bold in the text is in the


(A) Simple present
(B) Past continuous
(C) Present perfect
(D) Simple past

Questão 04 (EEAR/INÉDITA) – In “In 2020 the president of La Maddalena's National Park,


Fabrizio Fonnesu, told CNN that Mr Morandi had carried out illegal alterations to his hut, a
former radio station dating back to World War Two”, the word “his” refers to
(A) Mr Morandi
(B) Fabrizio Fonnesu
(C) Hut
(D) CNN

Read the comic strip and answer questions 05 and 06.

(Adapted from https://comics.azcentral.com/slideshow?comic=dt)

AULA 06 – CONJUNCTIONS, DIRECT SPEECH AND REPORTED SPEECH 27


TEACHER ANDREA BELO

Questão 05 (EEAR/INÉDITA) – Choose the best alternative to complete the blank


(A) Reduce
(B) Reduced
(C) Are reducing
(D) Have reduced

Questão 06 (EEAR/INÉDITA) – Choose the best alternative according to the comic strip
(A) The company's occupational accidents were completely extinguished last year
(B) A small percentage of accidents was reduced last year
(C) The security guard was the only one that didn’t allow a total decrease in workplace injuries
(D) The security guard was an honest worker

Read the comic strip and answer questions 07 and 08.

(Adapted from https://www.gocomics.com/pickles/2021/04/24)

Questão 07 (EEAR/INÉDITA) – Choose the best alternative to complete the blank


(A) Wanted
(B) Want
(C) Have wanted
(D) Are wanting

Questão 08 (EEAR/INÉDITA) – According to what Sylvia said, she __________.


(A) Thinks her father's hair is messy and that he stinks
(B) Thinks her father's hair stinks
(C) Doubts that her father wants to look good to her mother
(D) Thinks her father is not taking very good care of his appearance

AULA 06 – CONJUNCTIONS, DIRECT SPEECH AND REPORTED SPEECH 28


TEACHER ANDREA BELO

Read the lyrics and answer questions 09 and 10.


As I Am – Justin Bieber
By myself sometimes
To give my mind some space
Yeah, I know, yeah, I know that it hurts
When I push your love away, I hate myself
I wanna tell you lies
So your heart won't break
Yeah, I know, yeah, I know that I made my fair share of mistakes
Sometimes, I don't know why you love me
Sometimes, I don't know why you care
Take me with the good and the ugly
Say, "I'm not goin' anywhere"
Take me as I am, swear I'll do the best I can
Say, "I'm not goin' anywhere"
Take me as I am, swear I'll do the best I can
Say, "I'm not goin' anywhere"
Thousand miles apart, yeah, you're the one that holds my heart
It's no surprise
And I tell you all the time
And when life gets way too hard
I'll meet you in the stars, you know I will
I'll keep you near (mmm)
As I wipe away your tears…
(Adapted from https://genius.com/Justin-bieber-as-i-am-lyrics)

Questão 09 (EEAR/INÉDITA) – According to the lyrics


(A) The song expresses a relationship based on love but, at the same time, insecurity
(B) The song portrays an abusive relationship
(C) The song criticizes the emotional dependence that is developed in some relationships
(D) The song delves into the idea of self-love as the basis for other relationships

AULA 06 – CONJUNCTIONS, DIRECT SPEECH AND REPORTED SPEECH 29


TEACHER ANDREA BELO

Questão 10 (EEAR/INÉDITA) – The underlined phrase “you're the one that holds my heart”, in
the text, means that the person
(A) Is the one that really holds the other person’s heart
(B) Is the one that has all the love and appreciation of the other person
(C) Is the one who has full responsibility for the other person's mental health
(D) Is the one that keeps the other person alive

QUESTÕES EFOMM
Royal Navy flotilla reports Covid outbreak with 100 cases despite all sailors being vaccinated
The outbreak has not prompted a rethink of the Royal Navy’s voyage to 40 countries
More than 100 cases of coronavirus have been reported aboard the Royal Navy’s new aircraft
carrier and an escorting group of warships.
The vessels affected include the Royal Navy’s flagship HMS Queen Elizabeth carrier, with
unconfirmed media reports saying the group of warships recently stopped off for a brief spell of
shore leave in Limassol, Cyprus.
The virus has spread through the strike group’s 3,700 strength crew despite them being fully
vaccinated. “As part of routine testing, a small number of crew from the Carrier Strike Group have
tested positive for Covid-19,” a Royal Navy spokesman confirmed in a statement.
The incident comes despite a series of measures deployed on board Navy warships in light of the
pandemic. These include the use of masks, social distancing and a track and trace system.
The outbreak has not yet prompted a rethink of the strike group’s planned voyage to 40 countries,
said a Royal Navy spokesperson. “The Carrier Strike Group will continue to deliver their
operational tasks and there are no effects on the deployment.”
The aircraft carrier was about a quarter of the way through its 28-week deployment when the
Covid cases were detected. It is leading a strike group that includes 10 Marine Corps F35-B fighters
and is currently in the Indo-Pacific.
It is the largest combined naval and air task force assembled under British command since the
Falklands War, and was expected to pass through the South China Sea.
Ben Wallace, the defence secretary, said during a press briefing on Tuesday that the first cases
involving the strike group were reported on 4 July.
“Our crew are double vaccinated so you’ll be glad to know there is no serious effects on any of
the crew and we will manage it,” he said, adding that going forwards he will offer support to the
captain of the ship in “whatever decision he makes”.
https://www.independent.co.uk/news/uk/home-news/hms-queen-elizabeth-covid-royal-navy-b1883699.html

AULA 06 – CONJUNCTIONS, DIRECT SPEECH AND REPORTED SPEECH 30


TEACHER ANDREA BELO

Questão 01 (EFOMM/INÉDITA) – In the extract from the text: “[...] the Royal Navy’s new aircraft
carrier and an escorting group of warships.” (paragraph 1). The word in bold is closest in
meaning to
A) enemy
B) convoy
C) foe
D) outfit
E) gear

Questão 02 (EFOMM/INÉDITA) – In the extract from the text: “The outbreak has not yet
prompted a rethink of the strike group’s planned voyage to 40 countries” (paragraph 5), the
verb “to prompt” means
A) to arouse
B) to derogate
C) to prevent
D) to keep
E) to hinder

Questão 03 (EFOMM/INÉDITA) – What is true according to the text?


A) Although most of the crew were fully vaccinated, there was an outbreak of Covid 19 inside the
English ship with another 100 confirmed cases.
B) Despite the Covid 19 outbreak, only the Royal Navy's HMS Queen Elizabeth aircraft carrier was
affected by the virus, thanks to security measures adopted on other ships.
C) Although the crew of the British ship was completely immunized against Covid 19, there was
still an outbreak of the virus with dozens of confirmed cases.
D) The Covid 19 outbreak shows that the Royal Navy's HMS Queen Elizabeth aircraft carrier was
not taking the necessary measures to control the virus, such as wearing masks and social distance.
E) The Covid 19 outbreak caused the British Navy to think twice about the voyage that had been
planned for the ship.

Questão 04 (EFOMM/INÉDITA) – In the extract from the text: “[...] there is no serious effects on
any of the crew and we will manage it” (paragraph 5), the word “it” refers to
A) the largest combined naval and air task force.
B) the strike group.
C) the Falklands War
D) the outbreak of covid 19.
E) the captain of the ship

AULA 06 – CONJUNCTIONS, DIRECT SPEECH AND REPORTED SPEECH 31


TEACHER ANDREA BELO

Read the text below to answer question 05


Lewis Hamilton hopes for diversity legacy
Sir Lewis Hamilton has said he hopes improving the diversity of Formula One will be part of his
legacy.
The seven-times world champion ____________ a report which he hopes will encourage the
industry to become more inclusive and inspire young people from ethnic minorities to pursue a
career in the sport.
https://www.bbc.com/news/av/uk-57814103

Questão 05 (EFOMM/INÉDITA) – Fill in the blank with the suitable verb


A) have commissioned
B) had commissioned
C) have been commissioned
D) has been commissioned
E) has commissioned

Read the text below to answer questions 06 and 07


A pastor in Sierra Leone came across a jaw-dropping stone – a 706-carat (141 grams) diamond. It
is the largest diamond to be found in Sierra Leone in over 40 years.
When the pastor __________ it, he decided __________ it over to the government. __________
that it may boost the development of the West African nation.
The President of Sierra Leone said that he believed that a diamond like this should be publicly
__________ in the country, so that people will know the value of it and who the buyer is.
https://www.newsinlevels.com/products/big-diamond-in-sierra-leone

Questão 06 (EFOMM/INÉDITA) – Which is the correct way to complete the text above?
A) found / to handing / Hope / sold.
B) find / hand / Hope / selling.
C) found / to hand / Hoping / sold.
D) find / to handing / Hoping / selling
E) found / to hand / Hope / sell

AULA 06 – CONJUNCTIONS, DIRECT SPEECH AND REPORTED SPEECH 32


TEACHER ANDREA BELO

Questão 07 (EFOMM/INÉDITA) – In “A pastor in Sierra Leone came across a jaw-dropping


stone”, the expression in bold means
A) huge
B) ugly.
C) breathtaking
D) creepy
E) grisly

Questão 08 (EFOMM/INÉDITA) – Which is the correct way to complete the paragraph below?
Fast food is one of those things that the USA __________ to the world. But while hamburgers and
fries __________ worldwide as __________ typically American, in actual fact Americans did not
invent fast food - they just made it into a lifestyle.
Adapted from: https://linguapress.com/advanced/american-food.htm
A) had given / is seen / to be.
B) have given / are seen / been.
C) has given / see / being.
D) have given / see / been.
E) has given / are seen / being.

Read the text below to answer questions 09 and 10


_____ the night of April 4th 1968, someone was waiting opposite the windows of the Lorraine
Motel, _____ downtown Memphis.
_____ front of the motel, a big white Cadillac was parked; it was the car _____ which the Rev.
Martin Luther King was being driven round, as he traveled through the southern states, speaking
to audiences _____ towns and cities, promoting the cause of non-violence and civil rights.
When King stepped out onto the balcony, to take a breath of fresh air after eating his dinner, a
shot rang out. The civil rights leader and Nobel-prizewinner, the man who preached non-violence,
fell to the ground, fatally wounded. Within minutes, he was dead.
https://linguapress.com/advanced/martin-luther-king.htm

Questão 09 (EFOMM/INÉDITA) – Fill in the blank with the suitable prepositions


A) At / on / In / in / in
B) On / in / At / on / on
C) At / on / In / on / in
D) At / on / At / in / at
E) On / in / In / in / in

AULA 06 – CONJUNCTIONS, DIRECT SPEECH AND REPORTED SPEECH 33


TEACHER ANDREA BELO

Questão 10 (EFOMM/INÉDITA) – In “When King stepped out onto the balcony, to take a breath
of fresh air after eating his dinner”, the expression in bold means
A) to jump from somewhere high.
B) to stay somewhere watching something specific.
C) to hide from someone who is chasing you.
D) to leave a place for a short time.
E) to find someone by chance.

QUESTÕES EPCAR
Directions: Answer questions 01 to 10 according to text.
France to drop testing requirements for vaccinated tourists from some countries
France is making itself available as a destination for international tourists who have had
coronavirus jabs.
The government has announced that it is removing the need for coronavirus tests for vaccinated
Europeans and also allowing vaccinated tourists from most of the rest of the world, including the
US, to visit, provided they have a negative test.
Associated Press report that the relaxed rules will kick in from Wednesday. Tourism will not be
possible, however, from countries with virus surges and or prevalent variants. This “red list” for
the moment has 16 countries, including India, South Africa, and Brazil.
Outside of Europe, most of the rest of the world is classed as “orange” in the new travel rulebook
released today.
Vaccinated visitors from “orange” countries — including the US and Britain — will no longer need
to quarantine on arrival and will no longer have to justify the reasons for their trip to France. They
will, however, still be asked for a negative PCR test no older than 72 hours or a negative antigenic
test of no more than 48 hours.
Unvaccinated children will be allowed in with vaccinated adults.
European visitors and those from seven countries classed as “green” — Australia, South Korea,
Israel, Japan, Lebanon, New Zealand, and Singapore — will no longer need to undergo testing if
they are vaccinated.
(Adapted from https://www.theguardian.com/world/live/2021/jun/04/coronavirus-live-news-japan-taiwan-vaccine-fauci-
calls-wuhan-lab-records?page=with:block-60b9f3308f082ac930ae9fb8#block-60b9f3308f082ac930ae9fb8)

Questão 01 (EPCAR/INÉDITA) – Mark the option that can replace the word “drop” in the title
without changing its meaning
A) Abandon
B) Keep
C) Allow
D) Return

AULA 06 – CONJUNCTIONS, DIRECT SPEECH AND REPORTED SPEECH 34


TEACHER ANDREA BELO

Questão 02 (EPCAR/INÉDITA) – We can deduce from the first paragraph that


A) In France, tests are not required for any vaccinated tourist from any country
B) Europeans are the only ones who will still need negative tests to enter France
C) Vaccinated tourists from any country will be allowed to enter France, some may depend on a
negative test
D) Despite the advance, France's borders are not yet fully open

Questão 03 (EPCAR/INÉDITA) – Mark the option with the suitable question to answer the
underlined fragment below “Associated Press report that the relaxed rules will kick in from
Wednesday. Tourism will not be possible, however, from countries with virus surges and or
prevalent variants”.
A) Tourists from which countries will be able to enter France?
B) When will the announced rules take effect?
C) What were the relaxed rules announced by France?
D) When were the relaxed rules announced by the government?

Questão 04 (EPCAR/INÉDITA) – Mark the option that can replace the underlined word keeping
the same meaning “Vaccinated visitors from ‘orange’ countries — including the US and Britain —
will no longer need to quarantine on arrival and will no longer have to justify the reasons for
their trip to France”
A) Deny
B) Accuse
C) Legitimize
D) Legalize

Questão 05 (EPCAR/INÉDITA) – Mark the alternative that can answer the question below
according to the text. In addition to Europe, how is the pandemic situation in the rest of the
world?
A) All countries are classified as "orange"
B) These other countries are ranked between "orange" and "red", where “red” harbors a worse-
off minority
C) Visitors from all other countries are eligible to enter France testing negative for Covid-19
D) The pandemic situation in the rest of the world is still very serious, so the French borders
remain completely closed

AULA 06 – CONJUNCTIONS, DIRECT SPEECH AND REPORTED SPEECH 35


TEACHER ANDREA BELO

Questão 06 (EPCAR/INÉDITA) – According to the fourth paragraph, vaccinated visitors from


“orange” countries
A) Will face fewer barriers to enter France
B) Never needed quarantine on arrival or justify going to France
C) Will not need to present anything other than a passport to enter France
D) Are not American or British, for example

Questão 07 (EPCAR/INÉDITA) – Mark the correct option


A) Only vaccinated children can enter France
B) There is no information about the entry of children into France
C) Unvaccinated children will not be allowed into France under any circumstances
D) Vaccinated adults can enter with children, even if they are not vaccinated

Questão 08 (EPCAR/INÉDITA) – Considering the use of verb tenses, mark the alternative that
completes the sentence below correctly. France
A) Was facilitating the entry of certain tourists
B) Facilitated the entry of certain tourists
C) Will facilitate the entry of certain tourists
D) Facilitates the entry of certain tourists

Questão 09 (EPCAR/INÉDITA) – We can deduce from the sixth paragraph that


A) Visitors from countries classified as "green" will not need to be vaccinated to enter France
B) It is not just Europeans who will be able to enter France if they are vaccinated
C) Australia and Japan are examples of countries on the "red list"
D) Not all Europeans will be able to enter France without testing negative, even if they are
vaccinated

Questão 10 (EPCAR/INÉDITA) – The word “they” (paragraph 4) refers to


A) Vaccinated visitors from “orange” countries
B) “Orange” countries
C) US and Britain
D) PCR test and antigenic test

AULA 06 – CONJUNCTIONS, DIRECT SPEECH AND REPORTED SPEECH 36


TEACHER ANDREA BELO

QUESTÕES ESA
Read the text and answer questions 01, 02 and 03
If you bought a dog during lockdown, they'll need help coming out of it
Behavioural problems are a common reason for rehoming, so prepare them for when you’re
not there 24/7
Despite the lows of the past year, for many lockdown has been the perfect opportunity to
welcome a dog into their lives. The demand for dogs during the pandemic has been huge, with a
60% increase in calls from people seeking to adopt from the Dogs Trust charity, and with many
other rescue organisations reporting similar findings. Google searches for “buy a puppy”
increased by 115% after the UK first went into lockdown in March 2020, with prices for some of
the most soughtafter breeds reaching record levels.
Dog ownership is a wonderful thing, but it is also a huge responsibility and a commitment that
spans way beyond lockdown: as the saying goes, “A dog is for life”. As restrictions ease and the
resumption of normality begins, it’s important we consider the implications for our canine
companions and give them a hand to help them adjust.
Having a dog around has helped many people cope with lockdown. Our dogs mostly love us being
around too: going for longer walks, having more playtime, and resting by our side. Nevertheless,
it’s safe to say life has not been normal for our dogs for most of the past year. Few have met other
dogs, and if they have seen them, it would have been from afar or on a lead, meaning that they
were unable to interact or play. There have also been fewer visitors coming into the home, but
probably more deliveries, with people coming to the door carrying parcels and going away again.
This is all particularly concerning for puppies acquired during the pandemic, as their expectation
of “normal” is lockdown life, and they may never have seen visitors inside the house or have been
left home alone.
We are all longing for a great British summer in which we can go on dog-date walks with a friend
and their dog, have family round for garden barbecues and take our pooches to the pub or cafe,
and of course, we need our dogs to be able to cope calmly with all of that. A return to normal is
something humans are able to process, understand and prepare for. But our dogs – especially
young ones – won’t understand why everything has changed. As far as our dogs know, normality
for them has been enjoying time with family only – so to be expected to cope with groups of
people, children and other dogs, both in and out of the home, could be overwhelming for them.
(Adapted from https://www.theguardian.com/commentisfree/2021/apr/05/bought-dog-lockdown)

Questão 01 (ESA/INÉDITA) – Concerning the information in the text, is correct to state that
(A) Some problems about the dog's behavior, most of the time, have nothing to do with
relocating the animal
(B) Last year's context ended up making it difficult to adopt animals
(C) Despite the high adoption of dogs taking place during the lockdown period, the responsibility
for them goes beyond this context
(D) Although many dogs know only the context of the pandemic, they will be able to adapt to
other contexts with ease
(E) The cooperative relationship between owner and dog is not mutual in a pandemic context

AULA 06 – CONJUNCTIONS, DIRECT SPEECH AND REPORTED SPEECH 37


TEACHER ANDREA BELO

Questão 02 (ESA/INÉDITA) – In the sentence “As far as our dogs know, normality for them has
been enjoying time with family only – so to be expected to cope with groups of people, children
and other dogs, both in and out of the home, could be overwhelming for them” (paragraph 4),
the word “overwhelming” means
(A) Crushing
(B) Amazing
(C) Comfortable
(D) Awkward
(E) Indifferent

Questão 03 (ESA/INÉDITA) – The words COPE and GO are


(A) Regular verbs
(B) Irregular verbs
(C) Not verbs
(D) Verbs
(E) From different grammatical classes

Read the text and answer questions 04 and 05


Is the U.S. COVID-19 Vaccine Rollout Working? We're Not Sure Yet
After four months and 171 million doses of COVID-19 vaccines administered across the U.S., more
than a few of us are eager to know: are the shots working?
Thus far, available evidence can half-answer that question: The vaccines are working well for
those who can get them. As soon as the rollout kicked off, a variety of researchers began
conducting what might be considered __________ (1) unofficial phase 4 clinical trial, monitoring
early vaccine recipients among the general public. Several of their studies have found a significant
reduction in both the rates of infection and severity of symptoms among vaccinated populations
compared __________ (2) similar demographics who are still awaiting a dose. On an individual
level, the vaccines appear to be working as well in real life as they did in controlled trials.
But there’s another part of that question: will mass vaccination hasten the end of __________ (3)
pandemic? On first pass, this might seem to be happening. As of April 7, nearly 20% of Americans
have received a completed regimen of either the two-dose Pfizer-BioNTech and Moderna
vaccines, or the one-shot Janssen variety. Meanwhile, the daily number of deaths in the U.S.
attributed to COVID-19, on a rolling seven-day average, is currently 611—a ghastly figure, but a
tremendous drop from the all-time post-holiday peak of 3,428 on Jan. 11. The last time the figure
was this low was on Jul. 4, 2020, just before deaths climbed again in the wake of a second wave
over the summer, from which the country never fully recovered before the much more
catastrophic third wave spurred by the holidays.
(Adapted from https://time.com/5953007/covid-19-mass-vaccination/)

AULA 06 – CONJUNCTIONS, DIRECT SPEECH AND REPORTED SPEECH 38


TEACHER ANDREA BELO

Questão 04 (ESA/INÉDITA) – Concerning the information in the text, is correct to say that
(A) All the available evidence is able to show in the best way whether the vaccines are working
or not
(B) So far, vaccines have shown better results in trials than in actual individual vaccination
(C) Mass vaccination will certainly anticipate an end to the pandemic
(D) Most Americans have already received two doses of the vaccine against COVID-19
(E) It is possible to affirm that the vaccination is working, at least, partially

Questão 05 (ESA/INÉDITA) – In the sentence “Several of their studies have found a significant
reduction in both the rates of infection and severity of symptoms among vaccinated populations
compared…” (paragraph 2), the word “several” means
(A) Many
(B) None
(C) A few
(D) Some
(E) A part

QUESTÕES ESCOLA NAVAL


Based on the text below, answer the questions that follow it. The paragraphs of the text are
numbered.
The machine always wins: what drives our addiction to social media
[1] We are swimming in writing. Our lives have become, in the words of the author and academic
Shoshana Zuboff, an “electronic text”. Social media platforms have created a machine for us to
write to. The bait is that we are interacting with other people: our friends, colleagues, celebrities,
politicians, royals, terrorists, porn actors – anyone we like. We are not interacting with them,
however, but with the machine. We write to it, and it passes on the message for us after keeping
a record of the data.
[2] The machine benefits from the “network effect”: the more people write to it, the more benefits
it can offer, until it becomes a disadvantage not to be part of it. Part of what? The world’s first ever
public, live, collective, open-ended writing project. A virtual laboratory. An addiction machine,
which deploys crude techniques of manipulation redolent of the Skinner Box created by
behaviourist BF Skinner to control the behaviour of pigeons and rats with rewards and
punishments. We are users, much as cocaine addicts are users.
[3] What is the incentive to engage in writing like this for hours each day? In a form of mass
casualisation, writers no longer expect to be paid or given employment contracts. What do the
platforms offer us, in lieu of a wage? What gets us hooked? Approval, attention, retweets, shares
and likes.

AULA 06 – CONJUNCTIONS, DIRECT SPEECH AND REPORTED SPEECH 39


TEACHER ANDREA BELO

[4] Meanwhile, hashtags and trending topics underline the extent to which all of these protocols
are organized around the massification of individual voices – a phenomenon cheerfully described
by users with the science-fiction concept of the “hive mind” – and hype. The regular sweet spot
sought after is a brief period of ecstatic collective frenzy around any given topic. It doesn’t
particularly matter to the platforms what the frenzy is about: the point is to generate data, one of
the most profitable raw materials yet discovered. As in the financial markets, volatility adds value.
The more chaos, the better.
[5] Whether or not we think we are addicted, the machine treats us as addicts. Addiction is, quite
deliberately, the template for our relationship to the Twittering Machine. Addiction is all about
attention. For the social media bosses, this is axiomatic.
[6] If social media is an addiction machine, the addictive behaviour it is closest to is gambling: a
rigged lottery. Every gambler trusts in a few abstract symbols – the dots on a dice, numerals, suits,
red or black, the graphemes on a fruit machine – to tell them who they are. In most cases, the
answer is brutal and swift: you are a loser and you are going home with nothing. The true gambler
takes a perverse joy in anteing up, putting their whole being at stake. On social media, you scratch
out a few words, a few symbols, and press send, rolling the dice. The internet will tell you who you
are and what your destiny is through arithmetic likes, shares and comments.
[7] The interesting question is what it is that is so addictive. In principle, anyone can win big; in
practice, not everyone is playing with the same odds. Our social media accounts are set up like
enterprises competing for attention. If we are all authors now, we write not for money, but for the
satisfaction of being read. Going viral, or trending, is the equivalent of a windfall. But sometimes,
winning is the worst thing that can happen. The temperate climate of likes and approval is apt to
break, lightning-quick, into sudden storms of fury and disapproval.
(Adapted from https://www.theguardian.com/technology/2019/aug/23/social-media-addiction-gambling)

Questão 01 (ESCOLA NAVAL/INÉDITA) – According to the first paragraph, which option is


correct?
(A) people should avoid interacting on social media platforms.
(B) when people communicate through social media, they make real friends.
(C) lives nowadays are more based on personal contact.
(D) social media platforms allow people to truly interact with other people.
(E) when people communicate through social media, they, as a matter of fact, interact with the
machine.

Questão 02 (ESCOLA NAVAL/INÉDITA) – Which word can replace “however” in paragraph 1 with
no change in meaning?
(A) though
(B) since
(C) then
(D) besides
(E) therefore

AULA 06 – CONJUNCTIONS, DIRECT SPEECH AND REPORTED SPEECH 40


TEACHER ANDREA BELO

Questão 03 (ESCOLA NAVAL/INÉDITA) – Which option shows the main message of the text?
(A) everybody can go viral on social platforms.
(B) social media can be as addictive as drugs.
(C) financial feedback is the major reason why people engage in social media writing.
(D) there’s evidence cocaine is more addictive than social media and gambling.
(E) anyone can be successful when it comes to social media.

Questão 04 (ESCOLA NAVAL/INÉDITA) – According to the text, which option is incorrect?


(A) in practice, not everyone has the same chance of being successful online.
(B) every gambler trusts in a few abstract symbols, such as the dots on a dice.
(C) social media platforms have created a machine that treats people as addicts.
(D) there are no explanations on why people engage in writing online for hours each day.
(E) addiction to social media can be compared to gambling addiction.

Questão 05 (ESCOLA NAVAL/INÉDITA) – The word “meanwhile” in paragraph 4 indicates that


(A) some facts happen at the same time.
(B) one fact is more important than others.
(C) a single fact interferes in others.
(D) some facts happen at different times.
(E) some facts happen for no reason.

Questão 06 (ESCOLA NAVAL/INÉDITA) – Which is the best definition for “bait”, in paragraph 1?
(A) something that makes people improve their attitude.
(B) something unattractive that discourages people from acting a certain way.
(C) something attractive that is offered to make people act a certain way.
(D) something that makes people think about their actions.
(E) something attractive that discourages people from acting a certain way.

Questão 07 (ESCOLA NAVAL/INÉDITA) – Which word or expression does the object pronoun
“them”, in paragraph 1, refer to?
(A) internet addicts.
(B) the machine.
(C) platforms.
(D) social media.
(E) other people.

AULA 06 – CONJUNCTIONS, DIRECT SPEECH AND REPORTED SPEECH 41


TEACHER ANDREA BELO

Questão 08 (ESCOLA NAVAL/INÉDITA) – In the sentence “We are users, much as cocaine addicts
are users.” (paragraph 2), the author DOESN’T
(A) describe social media addicts in a certain way.
(B) make a comparison.
(C) treat drug users and internet addicts as different.
(D) compare drug users and internet addicts.
(E) bring impacting information about social media addiction.

Questão 09 (ESCOLA NAVAL/INÉDITA) – Approval, attention, retweets, shares and likes are
mentioned in the text as examples of
(A) emergency measures to overcome internet addiction.
(B) reasons why people get addicted to social media.
(C) reason why people do not like social media.
(D) online communication negative impacts.
(E) reasons why people usually avoid social media.

Questão 10 (ESCOLA NAVAL/INÉDITA) – Which is the best option to change the sentence “Social
media platforms have created a machine [...]” (paragraph 1) into the passive form?
A machine _______________ by social media platforms.
(A) has created.
(B) had created.
(C) had been created.
(D) has been created.
(E) have been created.

QUESTÕES EsPCEx
Leia o texto a seguir e responda às questões 01, 02 e 03.
Homelessness in the Living Rooms of the Rich
When photographer Jana Sophia Nolle moved to San Francisco three years ago, she met a lot of
people. Some inhabited multimillion-dollar houses with pristine Victorian architecture and fancy
furniture. Others lived in cardboard boxes.
The contrast between their dwellings plagued Nolle, who hails from Kassel, Germany, where
income inequality is less stark. While San Francisco boasts the highest density of billionaires per

AULA 06 – CONJUNCTIONS, DIRECT SPEECH AND REPORTED SPEECH 42


TEACHER ANDREA BELO

capita in the world, the Bay Area hosts the country’s third-largest population of people
experiencing homelessness. The government shelters only a third of them.
“I’d never seen so many people living on the street in a country as rich as America,” Nolle says. “I
was shocked.” So, when an unhoused man she knew suggested, jokingly, that she invite him into
one of her wealthy friends’ homes, it sparked an idea: What if, instead, she pitched his tent there?
This provocative vision inspired her series Living Room, sending Nolle on a quest to photograph
the scrappy DIY shelters of the poor inside the immaculately styled parlors of the rich. “They’re
implants in rooms where they don’t belong,” she says.
Adapted from https://www.wired.com/story/san-francisco-shelters-living-room/

Questão 01 (EsPCEx/INÉDITA) – In the excerpt “While San Francisco boasts the highest density
of billionaires per capita in the world, the Bay Area hosts the country’s third-largest population
of people experiencing homelessness.” (paragraph 2), the word while is being used to
(A) give an idea of time.
(B) add information.
(C) give an example.
(D) express emphasis.
(E) contrast ideas.

Questão 02 (EsPCEx/INÉDITA) – According to the text, read the statements and choose the
correct alternative.
I- the income inequality is less severe in Kassel, Germany.
II- the photographer had never seen so many homeless people in a wealthy country.
III- a San Francisco region is home to one of the largest homeless groups in the United States.
IV- the photographer invited a homeless person to live in the home of one of her wealthy
friends.
V- when the photographer moved to San Francisco, she witnessed a situation of extreme social
inequality.
(A) only I, III and V are correct.
(B) only I, II, III and IV are correct.
(C) only I, II, III and V are correct.
(D) only I, II, and IV are correct.
(E) All of them are correct.

AULA 06 – CONJUNCTIONS, DIRECT SPEECH AND REPORTED SPEECH 43


TEACHER ANDREA BELO

Questão 03 (EsPCEx/INÉDITA) – Which definition below corresponds to dwellings in the


sentence “The contrast between their dwellings plagued Nolle, who hails from Kassel, Germany,
where income inequality is less stark.” (paragraph 2)?
(A) living rooms.
(B) cardboard boxes.
(C) fancy furniture.
(D) inhabited houses.
(E) places of residence.

Leia o texto a seguir e responda às questões 04, 05 e 06.


‘Tradwives’: the new trend for submissive women has a dark heart and history
A thrilling new trend _________ (1) emerged, and it’s as seismic as the New Look __________ (2)
in 40s fashion, or the emergence of Mediterranean cooking in 90s Britain: it’s called being a
housewife.
So a “tradwife” is a woman who doesn’t work so as to look after their children, their husband,
their home and then talk non-stop about how great this is on social media. Who knew being so
traditional was also so modern? And so busy! Last week alone, there were interviews with
tradwives in the Daily Mail, the Times and on the BBC, This Morning, Victoria Derbyshire and, for
all I know, piped 24/7 across all channels. I’m afraid that – being both non-trad and a non-wife – I
am less plugged in than these women.
The tradwives have been keenly giving interviews about how they are the true feminists in
choosing not to work… as much as the tradwives think they are being renegade rebels by not
working, their rebellion is based on their husband earning enough to support a whole household.
Whoa there, little rebels!
Adapted from https://www.theguardian.com/fashion/2020/jan/27/tradwives-new-trend-submissive-women-dark-heart-history

Questão 04 (EsPCEx/INÉDITA) – In the sentence “Whoa there, little rebels!” (paragraph 3), the
author
(A) aligns her thinking with that of the so-called “tradwives”.
(B) is ironic about the supposed rebelliousness of the “tradwives”.
(C) reinforces that the new housewives are the real feminists.
(D) defines the degree of rebelliousness of “tradwives” as high.
(E) demonstrates that “tradwives” are renegade by their husbands.

AULA 06 – CONJUNCTIONS, DIRECT SPEECH AND REPORTED SPEECH 44


TEACHER ANDREA BELO

Questão 05 (EsPCEx/INÉDITA) – Choose the alternative containing the correct verbs to complete
gaps (1) and (2) in paragraph 1, respectively.
(A) have, was
(B) have, is
(C) has, were
(D) has, was
(E) is, was

Questão 06 (EsPCEx/INÉDITA) – In the sentence “The tradwives have been keenly giving
interviews about how they are the true feminists in choosing not to work…” (paragraph 3), the
word keenly means
(A) enthusiastically
(B) generously
(C) reluctantly
(D) repeatedly
(E) promptly

Leia o texto a seguir e responda às questões 07, 08 e 09.


Pharmacogenomics can show what your body makes of a drug
Doctors have long appreciated that the same dose of medicine will not necessarily have the same
effect on different patients. One of the ways in which people differ biologically is in how they
metabolise drugs, a process largely dependent on enzymes that can vary a lot from person to
person. Differences in enzymes can lead two patients with the same disease, and the same
treatment, to end up with a five-fold difference in the amount of working drug molecules in their
blood.
Sometimes such differences will mean that the drug is not circulating at a high enough level to
work. Sometimes it is circulating at too high a level and causing toxic side-effects. Though side-
effects sound peripheral they are a huge medical problem. The study of how genes affect an
individual’s response to drugs is known as pharmacogenetics, and it is flourishing.
One example of the technique’s potential is found with warfarin, a commonly used blood thinner
that has what doctors call a “narrow therapeutic window”: the distance between a level in the
blood too low to do good and a level too high for safety is small. Variations in an enzyme gene
called CYP2C9, which allow warfarin to stay in the blood for longer, are quite common and this
makes it harder to find the best therapeutic dose. A test produced by a genetic-testing company
makes it possible to look at the genes for such enzymes and appears to limit side-effects and
improve clinical outcomes.

AULA 06 – CONJUNCTIONS, DIRECT SPEECH AND REPORTED SPEECH 45


TEACHER ANDREA BELO

In chemotherapy, too, genetics can guide the choice of treatment in ways that other indicators
cannot. That matters since the drugs are so powerful. Some are so toxic that four patients in 100
will die from treatment, says David Kerr, a professor at Oxford University.
Adapted from https://www.economist.com/technology-quarterly/2020/03/12/pharmacogenomics-can-show-what-your-body-makes-of-a-drug.

Questão 07 (EsPCEx/INÉDITA) – According to the text, a relevant effect of the variation of


enzymes in the body from one person to another is that it
(A) equals the amount of medicine working in the blood of patients with the same disease.
(B) affects the way an individual metabolizes drugs and responds to them.
(C) increases the chances of healing a patient with more enzymes in the body by five times.
(D) causes the same dose of the drug to have the same effect in different patients.
(E) is biologically dissociated from the way people metabolize a drug in the body.

Questão 08 (EsPCEx/INÉDITA) – According to the text, the study of how genes affect an
individual's response to drugs is
(A) failing.
(B) concluded.
(C) fragmented.
(D) stagnant.
(E) thriving.

Questão 09 (EsPCEx/INÉDITA) – Among the excerpts from the text, which one best corroborates
the beneficial potential of pharmacogenetics?
(A) “genetics can guide the choice of treatment in ways that other indicators cannot.” (paragraph
4)
(B) “the distance between a level in the blood too low to do good and a level too high for safety
is small.” (paragraph 3)
(C) “a process largely dependent on enzymes that can vary a lot from person to person.”
(paragraph 1)
(D) “which allow warfarin to stay in the blood for longer” (paragraph 3)
(E) “a commonly used blood thinner that has what doctors call a ‘narrow therapeutic window’”
(paragraph 3)

Leia o texto a seguir e responda à questão 10.


America's Long Overdue Awakening to Systemic Racism
Just before 7 p.m. on June 1, a deployment of local, state and federal forces, armored head to toe
in riot gear, unleashed rubber bullets and sprayed tear gas onto a crowd of peaceful demonstrators

AULA 06 – CONJUNCTIONS, DIRECT SPEECH AND REPORTED SPEECH 46


TEACHER ANDREA BELO

gathered in the park to protest under the mantra “Black Lives Matter.” Since then, the debate over
systemic racism has spread across the nation and around the world.
Trump’s Administration has repeatedly denied that discrimination against black Americans is
embedded in the political, economic, and social structure of the country. Trump believes there are
“injustices in society,” but the notion that antiblackness is intrinsic to U.S. law enforcement has
been declared untrue. Attorney General William Barr warned against “automatically assuming
that the actions of an individual necessarily mean that their organization is rotten.”
But, for all that’s good about America, something is rotten. The protesters in Lafayette Park on
June 1 may have been galvanized by the disturbing video of the murder of George Floyd,
suffocated to death beneath the knee of a Minneapolis police officer. But at the core of their
movement is much more than the outrage over the latest instances of police brutality.
Centuries of racist policy have left black Americans in the dust. The U.S. may think it has brushed
chattel slavery into the dustbin of history after the Civil War, but the country never did a very good
job incinerating its traumatic remains, instead leaving embers that still burn today: an education
system that fails black Americans, substandard health care that makes them more vulnerable to
death and disease, and an economy that leaves millions without access to a living wage.
Adapted from https://time.com/5851855/systemic-racism-america/

Questão 10 (EsPCEx/INÉDITA) – According to the text, the June 1 protest


(A) was ended in a peaceful way.
(B) was violently developing until the police arrived.
(C) targeted gender inequality.
(D) had few participants.
(E) sparked a wide-ranging debate on racism.

AULA 06 – CONJUNCTIONS, DIRECT SPEECH AND REPORTED SPEECH 47


TEACHER ANDREA BELO

GABARITO
GABARITO AFA
01 – B 02 – C 03 – A 04 – D 05 – A
06 – C 07 – B 08 – A 09 – C 10 – D

GABARITO COLÉGIO NAVAL


01 – E 02 – D 03 – C 04 – B 05 – A
06 – A 07 – B 08 – C 09 – D 10 – E

GABARITO EAM
01 – A 02 – B 03 – E 04 – D 05 – C

GABARITO EEAR
01 – B 02 – C 03 – D 04 – A 05 – B
06 – C 07 – B 08 – D 09 – A 10 – B

GABARITO EFOMM
01 – B 02 – A 03 – C 04 – D 05 – E
06 – C 07 – C 08 – E 09 – E 10 – D

GABARITO EPCAR
01 – A 02 – D 03 – B 04 – C 05 – B
06 – A 07 – D 08 – C 09 – B 10 – A

GABARITO ESA
01 – C 02 – A 03 – D 04 – E 05 – A

GABARITO ESCOLA NAVAL


01 – E 02 – A 03 – B 04 – D 05 – A
06 – C 07 – E 08 – C 09 – B 10 – D

GABARITO EsPCEx
01 – E 02 – C 03 – E 04 – B 05 – D
06 – A 07 – B 08 – E 09 – A 10 – E

AULA 06 – CONJUNCTIONS, DIRECT SPEECH AND REPORTED SPEECH 48


TEACHER ANDREA BELO

QUESTÕES COMENTADAS
QUESTÕES AFA
Directions: Read the text below and answer questions 01 to 08 according to it.
Ignore the pessimism: Covid vaccines are quietly prevailing
Nightmare scenarios involving deadly new variants are making us all too gloomy – but there’s
a scientific case for optimism
It can be quite easy, reading the press, to believe that the pandemic will never end. Even when
good news about vaccines started to arrive in the autumn, this grim narrative managed to harden.
In the past month, you could read “five reasons that herd immunity is probably impossible”, even
with mass vaccination; breathless reports about yetuncharacterised but potentially ruinous
variants, such as the “double mutant” variant in India, or two concerning variants potentially
swapping mutations and teaming up in a “nightmare scenario” in California; get ready, some
analysts said, for the “permanent pandemic”.
Among many people I know, a sort of low-grade doom has set in. They think the vaccines are a
mere sliver of hope, only holding back the virus for a short time before being worn down by a rush
of ever-cleverer variants that will slosh around us, perhaps forever. Things might briefly get better,
they believe, but only by a little, and even that is tenuous.
However despite such dark talk, and the potential difficulties along the way of vaccine rollout, I
still remain optimistic. Since about the midway point of last year I have believed that extremely
potent vaccines are going to end the pandemic. They’ll do so by either driving the disease down
to near-extinction, or so constraining its force and spread that it becomes a manageable concern,
like measles or mumps. I actually think this will happen fairly soon, as long as we get everyone –
the whole world, not just the rich – vaccinated.
The scientific case for optimism is straightforward. The vaccines we have are beyond very good,
they’re among the most effective ever created. They appear to be potent in real-life situations,
and results so far show that protection is longlasting. Crucially, new results in the US show that
the mRNA vaccines used there effectively prevented coronavirus infections – not just serious
symptoms – in results similar to those previously reported by a UK-based study. And another
study in the UK suggested that vaccinated groups were less likely to spread coronavirus infection
overall. This is exactly what we need to choke out the pandemic: vaccines that don’t just protect,
but actually halt the virus infecting people and spreading.
When it comes to variants, it is clear that some are more infectious, and some are more deadly.
But their interaction with vaccines isn’t yet clear. Some lab-based results show that certain viral
mutations may make some immune responses less potent. And one study suggested the
Oxford/AstraZeneca vaccine might be less potent against the South African variant. But the
majority of scientists believe that vaccines have so far held the line, and will continue to do so. If
variants continue to make small advances, vaccines can be updated. A doomsday strain may be
possible, but exceedingly hard to predict. Evolution isn’t an on-demand miracle worker for viral
supremacy; even over decades most viruses don’t escape vaccine protection.
(Adapted from https://www.theguardian.com/commentisfree/2021/apr/12/ignore-scare-stories-covid-vaccines-variants-scientific-optimism)

AULA 06 – CONJUNCTIONS, DIRECT SPEECH AND REPORTED SPEECH 49


TEACHER ANDREA BELO

Questão 01 (AFA/INÉDITA) – The first paragraph of the text states that


A) Vaccination made everyone optimistic about the pandemic scenario
B) The media can make us more and more pessimistic about the end of the pandemic
C) The pandemic is less intense with each passing day
D) Most analysts see the pandemic as a permanent scenario
Comentário: A alternativa A está incorreta. De acordo com o primeiro parágrafo, não é correto
afirmar que a vacinação deixou todos otimistas com o cenário pandêmico, mas sim, que mesmo
com a vacinação em massa, o pessimismo sobre o fim da pandemia ainda existe. Isso pode ser
confirmado com o trecho “...you could read ‘five reasons that herd immunity is probably
impossible’, even with mass vaccination…”.
A alternativa B está correta. De acordo com o primeiro parágrafo, é correto afirmar que a mídia
pode nos tornar cada vez mais pessimistas sobre o fim da pandemia, assim como esta opção
indica. Isso pode ser confirmado com o trecho “It can be quite easy, reading the press, to believe
that the pandemic will never end. Even when good news about vaccines started to arrive in the
autumn, this grim narrative managed to harden”.
A alternativa C está incorreta. De acordo com o primeiro parágrafo, não é correto afirmar que a
pandemia é menos intensa a cada dia que passa, mas sim, que ela parece estar mais intensa a
cada dia que passa. Isso pode ser confirmado com o trecho “...breathless reports about yet-
uncharacterised but potentially ruinous variants, such as the ‘double mutant’ variant in India, or
two concerning variants potentially swapping mutations and teaming up in a ‘nightmare scenario’
in California…”.
A alternativa D está incorreta. De acordo com o primeiro parágrafo, não é correto afirmar que a
maioria dos analistas veem a pandemia como um cenário permanente, mas sim, que alguns
previram uma pandemia permanente. Isso pode ser confirmado com o trecho “...get ready, some
analysts said, for the ‘permanent pandemic’…”.
GABARITO: B

Questão 02 (AFA/INÉDITA) – In the phrase “Among many people I know, a sort of low-grade
doom has set in” (paragraph 2), the underlined word is a synonym for
A) Pessimism
B) Sadness
C) Disaster
D) Boon
Comentário: A alternativa A está incorreta. A palavra “doom” significa ruína/destruição e não é
sinônimo de “pessimism”, que significa pessimismo.
A alternativa B está incorreta. A palavra “doom” significa ruína/destruição e não é sinônimo de
“sadness”, que significa tristeza.

AULA 06 – CONJUNCTIONS, DIRECT SPEECH AND REPORTED SPEECH 50


TEACHER ANDREA BELO

A alternativa C está correta. A palavra “doom” significa ruína/destruição e é sinônimo de


“disaster”, que significa disastre.
A alternativa D está incorreta. A palavra “doom” significa ruína/destruição e não é sinônimo de
“boon”, que significa benção.
GABARITO: C

Questão 03 (AFA/INÉDITA) – The passage “Things might briefly get better, they believe, but only
by a little, and even that is tenuous” (paragraph 2) suggests that
A) They believe that it’s possible that, in a short and fast way, things will improve
B) They believe that things will get better little by little
C) They believe that things tend to get worse, with no prospect of improvement
D) They believe that things will get better, even if only briefly
Comentário: A alternativa A está correta. A passagem “As coisas podem melhorar brevemente,
eles acreditam, mas apenas um pouco, e mesmo isso é tênue” sugere que eles acreditam que é
possível que, de uma forma rápida e curta, as coisas melhorem, assim como esta opção indica.
A alternativa B está incorreta. A passagem “As coisas podem melhorar brevemente, eles
acreditam, mas apenas um pouco, e mesmo isso é tênue” não sugere que eles acreditam que as
coisas vão melhorar aos poucos, mas sim, que as coisas podem melhorar, mas de forma breve.
A alternativa C está incorreta. A passagem “As coisas podem melhorar brevemente, eles
acreditam, mas apenas um pouco, e mesmo isso é tênue” não sugere que eles acreditam que as
coisas tendem a piorar, sem perspectiva de melhora, mas sim, que as coisas podem melhorar,
mas de forma breve.
A alternativa D está incorreta. A passagem “As coisas podem melhorar brevemente, eles
acreditam, mas apenas um pouco, e mesmo isso é tênue” não sugere que eles acreditam que as
coisas vão melhorar, mesmo que apenas brevemente, mas sim, que é possível que isso aconteça,
mas se acontecer, será de forma breve.
GABARITO: A

Questão 04 (AFA/INÉDITA) – The second paragraph of the text states that


A) Some people are overwhelmed by sadness in this pandemic context
B) Most people have a lot of confidence in the fact that the vaccine will permanently end the
pandemic
C) Some people believe that the pandemic is already coming to an end
D) Some people are being pessimistic about the pandemic even with the arrival of vaccination
Comentário: A alternativa A está incorreta. De acordo com o segundo parágrafo, não é correto
afirmar que algumas pessoas estão sobrecarregadas de tristeza neste contexto de pandemia, mas

AULA 06 – CONJUNCTIONS, DIRECT SPEECH AND REPORTED SPEECH 51


TEACHER ANDREA BELO

sim, que um desastre se instalou entre algumas pessoas. Isso pode ser confirmado com o trecho
“Among many people I know, a sort of low-grade doom has set in”.
A alternativa B está incorreta. De acordo com o segundo parágrafo, não é correto afirmar que a
maioria das pessoas tem muita confiança no fato de que a vacina acabará permanentemente com
a pandemia, mas sim, que algumas pessoas pensam que a vacina vai segurar o vírus por um curto
espaço de tempo. Isso pode ser confirmado com o trecho “They think the vaccines are a mere
sliver of hope, only holding back the virus for a short time before being worn down by a rush of
evercleverer variants that will slosh around us, perhaps forever”.
A alternativa C está incorreta. De acordo com o segundo parágrafo, não é correto afirmar que
algumas pessoas acreditam que a pandemia já está chegando ao fim, mas sim, que algumas
pessoas acreditam que o vírus vai continuar entre nós para sempre. Isso pode ser confirmado com
o trecho “They think the vaccines are a mere sliver of hope, Only holding back the virus for a short
time before being worn down by a rush of ever-cleverer variants that will slosh around us, perhaps
forever”.
A alternativa D está correta. De acordo com o segundo parágrafo, é correto afirmar que algumas
pessoas estão pessimistas em relação à pandemia mesmo com a chegada da vacinação, assim
como esta opção indica. Isso pode ser confirmado com o trecho “Things might briefly get better,
they believe, but only by a little, and even that is tenuous”.
GABARITO: D

Questão 05 (AFA/INÉDITA) – In the phrase “...or so constraining its force and spread that it
becomes a manageable concern…” (paragraph 3), the underlined word is a synonym for
A) Restricting
B) Forbidding
C) Encouraging
D) Allowing
Comentário: A alternativa A está correta. A palavra “constraining” significa, nesse caso,
restringindo e é um sinônimo de “restricting”, que significa restringindo.
A alternativa B está incorreta. A palavra “constraining” significa, nesse caso, restringindo e não é
um sinônimo de “forbidding”, que significa proibindo.
A alternativa C está incorreta. A palavra “constraining” significa, nesse caso, restringindo e não é
um sinônimo de “encouraging”, que significa encorajando.
A alternativa D está incorreta. A palavra “constraining” significa, nesse caso, restringindo e não é
um sinônimo de “allowing”, que significa permitindo.
GABARITO: A

AULA 06 – CONJUNCTIONS, DIRECT SPEECH AND REPORTED SPEECH 52


TEACHER ANDREA BELO

Questão 06 (AFA/INÉDITA) – The pronoun “they” (paragraph 4) refers to


A) Real-life situations
B) The whole world
C) Vaccines
D) Scientific case
Comentário: A alternativa A está incorreta. O pronome “they” (eles/elas) não se refere às
situações da vida real, mas sim, às vacinas. Isso pode ser confirmado com o trecho “The vaccines
we have are beyond very good, they’re among the most effective ever created”.
A alternativa B está incorreta. O pronome “they” (eles/elas) não se refere ao mundo todo, mas
sim, às vacinas. Isso pode ser confirmado com o trecho “The vaccines we have are beyond very
good, they’re among the most effective ever created”.
A alternativa C está correta. O pronome “they” (eles/elas) não se refere às vacinas, assim como
esta opção indica. Isso pode ser confirmado com o trecho “The vaccines we have are beyond very
good, they’re among the most effective ever created”.
A alternativa D está incorreta. O pronome “they” (eles/elas) não se refere ao caso científico, mas
sim, às vacinas. Isso pode ser confirmado com o trecho “The vaccines we have are beyond very
good, they’re among the most effective ever created”.
GABARITO: C

Questão 07 (AFA/INÉDITA) – The passage “...despite such dark talk, and the potential difficulties
along the way of vaccine rollout, I still remain optimistic” (paragraph 3) suggests that
A) Bad thoughts and difficulties make us stronger in the pandemic
B) Optimism still remains despite all obstacles to vaccination
C) Optimism is unnecessary in the face of dark talks and difficulties
D) Optimism arises from the obstacles to vaccination
Comentário: A alternativa A está incorreta. A passagem “...apesar de toda essa conversa sombria
e das dificuldades potenciais ao longo do caminho para o lançamento da vacina, ainda continuo
otimista” não sugere que pensamentos ruins e dificuldades nos tornam mais fortes na pandemia,
mas sim, que o otimismo ainda permanece, apesar de todos os obstáculos à vacinação.
A alternativa B está correta. A passagem “...apesar de toda essa conversa sombria e das
dificuldades potenciais ao longo do caminho para o lançamento da vacina, ainda continuo
otimista” sugere que o otimismo ainda permanece, apesar de todos os obstáculos à vacinação,
assim como esta opção indica.
A alternativa C está incorreta. A passagem “...apesar de toda essa conversa sombria e das
dificuldades potenciais ao longo do caminho para o lançamento da vacina, ainda continuo
otimista” não sugere que o otimismo é desnecessário em face de conversas sombrias e

AULA 06 – CONJUNCTIONS, DIRECT SPEECH AND REPORTED SPEECH 53


TEACHER ANDREA BELO

dificuldades, mas sim, que o otimismo ainda permanece, apesar de todos os obstáculos à
vacinação.
A alternativa D está incorreta. A passagem “...apesar de toda essa conversa sombria e das
dificuldades potenciais ao longo do caminho para o lançamento da vacina, ainda continuo
otimista” não sugere que o otimismo surge dos obstáculos à vacinação, mas sim, que o otimismo
ainda permanece, apesar de todos os obstáculos à vacinação.
GABARITO: B

Questão 08 (AFA/INÉDITA) – The text


A) Expresses the duality present in the current pandemic situation
B) Criticizes pessimism about the end of the pandemic
C) Develops arguments against mass vaccination
D) Warns about fake news and its possible impact on vaccination
Comentário: A alternativa A está correta. É correto afirmar que o texto expressa a dualidade
presente na atual situação de pandemia, indicando o pessimismo e o otimismo acerca do assunto,
assim como esta opção indica. Isso pode ser confirmado com o trecho “It can be quite easy,
reading the press, to believe that the pandemic will never end … However despite such dark talk,
and the potential difficulties along the way of vaccine rollout, I still remain optimistic”.
A alternativa B está incorreta. Não é correto afirmar que o texto critica o pessimismo sobre o fim
da pandemia, mas sim, que ele expressa a dualidade presente na atual situação de pandemia,
indicando o pessimismo e o otimismo acerca do assunto. Isso pode ser confirmado com o trecho
“It can be quite easy, reading the press, to believe that the pandemic will never end … However
despite such dark talk, and the potential difficulties along the way of vaccine rollout, I still remain
optimistic”.
A alternativa C está incorreta. Não é correto afirmar que o texto desenvolve argumentos contra
a vacinação em massa, mas sim, que ele expressa a dualidade presente na atual situação de
pandemia, indicando o pessimismo e o otimismo acerca do assunto. Isso pode ser confirmado
com o trecho “It can be quite easy, reading the press, to believe that the pandemic will never end
… However despite such dark talk, and the potential difficulties along the way of vaccine rollout, I
still remain optimistic”.
A alternativa D está incorreta. Não é correto afirmar que o texto avisa sobre notícias falsas e seu
possível impacto na vacinação, mas sim, que ele expressa a dualidade presente na atual situação
de pandemia, indicando o pessimismo e o otimismo acerca do assunto. Isso pode ser confirmado
com o trecho “It can be quite easy, reading the press, to believe that the pandemic will never end
… However despite such dark talk, and the potential difficulties along the way of vaccine rollout, I
still remain optimistic”.
GABARITO: A

AULA 06 – CONJUNCTIONS, DIRECT SPEECH AND REPORTED SPEECH 54


TEACHER ANDREA BELO

Directions: Read the text below and answer questions 09 and 10 according to it.
Sinopharm: Chinese Covid vaccine gets WHO emergency approval
The World Health Organization (WHO) has granted emergency approval for the Covid vaccine
made by Chinese state-owned company Sinopharm.
It is the first vaccine developed by a non-Western country to get WHO backing. The vaccine has
already been given to millions of people in China and elsewhere.
The WHO had previously only approved the vaccines made by Pfizer, AstraZeneca, Johnson &
Johnson and Moderna.
But individual health regulators in various countries - especially poorer ones in Africa, Latin
America and Asia - have approved Chinese jabs for emergency use.
With little data released internationally early on, the effectiveness of the various Chinese vaccines
has long been uncertain.
But the WHO on Friday said it had validated the "safety, efficacy and quality" of the Sinopharm
jab.
The WHO said the addition of the vaccine had "the potential to rapidly accelerate Covid-19 vaccine
access for countries seeking to protect health workers and populations at risk".
It is recommending that the vaccine be administered in two doses to those aged 18 and over.
A decision is expected in the coming days on another Chinese vaccine developed by Sinovac, while
Russia's Sputnik vaccine is under assessment.
Why does WHO backing matter?
The green light from the global health body is a guideline for national regulators that a vaccine is
safe and effective.
WHO director-general Tedros Adhanom Ghebreyesus said it would give countries "confidence to
expedite their own regulatory approval".
It also means that the vaccine can be used in the global Covax programme, which was set up last
year to try to ensure fair access to vaccines among rich and poor nations.
The decision to list the Chinese vaccine for emergency use is expected to give a substantial boost
to the scheme, which has been struggling with supply problems.
(Adapted from https://www.bbc.com/news/world-asia-china-56967973)

Questão 09 (AFA/INÉDITA) – In the phrase “…the potential to rapidly accelerate Covid-19


vaccine access for countries seeking to protect health workers and populations at risk”
(paragraph 6), the underlined word is a synonym for
A) Hoping
B) Begging
C) Pursuing
D) Expecting

AULA 06 – CONJUNCTIONS, DIRECT SPEECH AND REPORTED SPEECH 55


TEACHER ANDREA BELO

Comentário: A alternativa A está incorreta. A palavra “seeking” significa buscam e não é um


sinônimo para “hoping”, que significa na esperança.
A alternativa B está incorreta. A palavra “seeking” significa buscam e não é um sinônimo para
“begging”, que significa implorando.
A alternativa C está correta. A palavra “seeking” significa buscam e é um sinônimo para
“pursuing”, que significa na buscando/procurando.
A alternativa D está incorreta. A palavra “seeking” significa buscam e não é um sinônimo para
“expecting”, que significa esperando.
GABARITO: C

Questão 10 (AFA/INÉDITA) – The first paragraph of the text states that


A) Other vaccines from Eastern countries have already been approved by WHO
B) Before WHO approval, the vaccine use was restricted to China
C) The first non-Western vaccine was approved by WHO still in the testing phase
D) The vaccine was already being used even before WHO approval
Comentário: A alternativa A está incorreta. De acordo com o primeiro parágrafo, não é correto
afirmar que outras vacinas de países orientais já foram aprovadas pela OMS, mas sim, que essa
foi a primeira. Isso pode ser confirmado com o trecho “It is the first vaccine developed by a non-
Western country to get WHO backing”.
A alternativa B está incorreta. De acordo com o primeiro parágrafo, não é correto afirmar que
antes da aprovação da OMS, o uso da vacina era restrito à China, mas sim, que a vacina já estava
sendo utilizada na China e em outros lugares. Isso pode ser confirmado com o trecho “The vaccine
has already been given to millions of people in China and elsewhere”.
A alternativa C está incorreta. De acordo com o primeiro parágrafo, não é correto afirmar que a
primeira vacina não ocidental foi aprovada pela OMS ainda em fase de testes, mas sim, que ela já
estava sendo aplicada oficialmente. Isso pode ser confirmado com o trecho “The vaccine has
already been given to millions of people in China and elsewhere”.
A alternativa D está correta. De acordo com o primeiro parágrafo, é correto afirmar que a vacina
já estava sendo usada antes mesmo da aprovação da OMS, assim como esta opção indica. Isso
pode ser confirmado com o trecho “It is the first vaccine developed by a non-Western country to
get WHO backing. The vaccine has already been given to millions of people in China and
elsewhere”.
GABARITO: D

AULA 06 – CONJUNCTIONS, DIRECT SPEECH AND REPORTED SPEECH 56


TEACHER ANDREA BELO

QUESTÕES COLÉGIO NAVAL


Read Text I to do questions 01 to 05 based on it.
The American dream
__________ (1) unitary definition does not exist–the American dream probably has a different
meaning to every US citizen. For some it is __________ (2) dream of freedom and equality, for
others it is the dream of a fulfilled life or even the dream of fame and wealth.
In general, the American dream can be defined as being the opportunity and freedom for all
citizens to achieve their goals and become rich and famous if only they work hard enough.
The first settlers hoped for freedom of religion; in their home countries they had been persecuted
because of __________ (3) religious and political views. Bad living conditions in Europe and the
hope for better living standards in America attracted more and more settlers to migrate to the
new world. The booming US industry during the first half of the 20th century caused the myth
“from rags to riches”.
Today’s role of the American dream is a matter of discussion. Some think that the American dream
is still a viable element today, for others it is only an illusion.
The American dream focusses on the success of the individuum (not the wellbeing of the whole
population). On the one hand, this has advantages, e.g. lower taxes, little interference of the state.
On the other hand, the same advantages are disadvantageous for society: there is no social fond
for the poor and there is no serious interest in important issues like environmental protection etc.
Adapted from: https://www.ego4u.com/en/read-on/countries/usa/american-dream

Questão 01 (COLÉGIO NAVAL/INÉDITA) – Choose the alternative containing the correct words to
respectively complete gaps (1) and (2)
(A) An / the
(B) The / a
(C) A / a
(D) The / The
(E) A / the
Comentários: A alternativa E está correta.
Lacuna 1 – Lembre-se de que o artigo indefinido “a” (um, uma) é utilizado antes de palavras que
iniciem por som de consoante, ou seja, antes de consoantes, da semivogal Y e do H
sonoro/audível, portanto, a pesar da palavra “unitary” começar com “u” que é uma vogal, ela deve
ser precedida do artigo indefinido “a” e não “an" pois “uni” possui som de “y ” que é uma
consoante.
Lacuna 2 – Observe que o substantivo “dream” já foi citado anteriormente e por isso usaremos o
artigo definido “the”.
GABARITO: E

AULA 06 – CONJUNCTIONS, DIRECT SPEECH AND REPORTED SPEECH 57


TEACHER ANDREA BELO

Questão 02 (COLÉGIO NAVAL/INÉDITA) – According to the text, the American dream


(A) has a clear and unique definition for all Americans.
(B) is only linked to making money and being famous.
(C) is not about freedom, but about the possibility of making a lot of money.
(D) has to do with freedom and the possibility that Americans can achieve their dreams.
(E) is something unique for all Americans.
Comentários: A alternativa A está incorreta. O texto não diz que o sonho americano possui uma
definição clara e única. Ele deixa isso bem claro nas primeiras linhas ao dizer que não existe uma
definição unitária de sonho americano e que provavelmente tem um significado diferente para
cada cidadão americano.
A alternativa B está incorreta. O texto não diz que o sonho americano está somente ligado a ganhar
dinheiro e ser famoso, segundo o texto o sonho americano também tem a ver com liberdade e
igualdade.
A alternativa C está incorreta. Segundo o texto, o sonho americano também tem a ver com
liberdade e não somente com a possibilidade de se ganhar dinheiro.
A alternativa D está correta. De acordo com o texto, o sonho americano tem a ver com igualdade,
liberdade e a possibilidade de realizar seus sonhos, seja ele ganhar dinheiro, ser famoso ou ser
bem-sucedido.
A alternativa E está incorreta. De acordo com o texto, não existe uma definição unitária de sonho
americano e que provavelmente tem um significado diferente para cada cidadão americano.
GABARITO: D

Questão 03 (COLÉGIO NAVAL/INÉDITA) – Choose the alternative containing the correct words to
respectively complete gap (3)
(A) your
(B) them
(C) their
(D) his
(E) yours
Comentários: A alternativa C está correta. “The first settlers hoped for freedom of religion; in their
home countries they had been persecuted because of their religious and political views” (Os
primeiros colonos esperavam liberdade de religião; em seus países de origem, foram perseguidos
por causa de suas opiniões religiosas e políticas). Observe que o após a lacuna temos “religious”
e “political” que são adjetivos que modificam o substantivo “views”. Portanto a lacuna está ligada
ao substantivo “views” (opiniões), logo devemos colocar um pronome possessivo adjetivo. Como
as opiniões religiosas e políticas são dos primeiros colonos (terceira pessoa do plural) então o
pronome que devemos utilizar é o pronome “their” (possessivo adjetivo que faz referência a
terceira pessoal do plural).
GABARITO: C

AULA 06 – CONJUNCTIONS, DIRECT SPEECH AND REPORTED SPEECH 58


TEACHER ANDREA BELO

Questão 04 (COLÉGIO NAVAL/INÉDITA) – According to the text


(A) the american dream remains a very important thing for all Americans.
(B) the American dream has more to do with individual success than with the collective success
of the nation as a whole.
(C) the first settlers persecuted the Americans because of their religion.
(D) The American dream shows us only advantages, such as the state's little interference in the
economy.
(E) The American dream, despite being clear, is not an easy thing to achieve.
Comentários: A alternativa A está incorreta. Segundo o texto, hoje em dia, para muitos, o sonho
americano é apenas uma ilusão.
A alternativa B está correta. De acordo como texto, o sonho americano foca no sucesso do
indivíduo e não no bem-estar da população como um todo.
A alternativa C está incorreta. O texto não fala que os primeiros colonos perseguiram os
americanos por causa de religião.
A alternativa D está incorreta. Segundo o texto, o sonho americano traz vantagens, como por
exemplo impostos mais baixos e pouca interferência do estado e desvantagens como a falta de
afeto social pelos pobres e a falta de interesse sério em questões importantes como proteção
ambiental.
A alternativa E está incorreta. Segundo o texto, o sonho americano não possui uma definição clara
única e pode ser interpretada de diversas maneiras.
GABARITO: B

Questão 05 (COLÉGIO NAVAL/INÉDITA) – “…the American dream can be defined as being the
opportunity…”. The underlined verb expresses
(A) possibility.
(B) obligation.
(C) prohibition.
(D) permission.
(E) hability.
Comentários: A alternativa A está correta. “...the American dream can be defined as being the
opportunity...” (...o sonho americano pode ser definido como sendo a oportunidade…). Portanto
nesse caso o verbo modal “can” expressa a ideia de possibilidade.
GABARITO: A

AULA 06 – CONJUNCTIONS, DIRECT SPEECH AND REPORTED SPEECH 59


TEACHER ANDREA BELO

Questão 06 (COLÉGIO NAVAL/INÉDITA) – Read the sentences below


I. He had a bit of knowledge about the subject.
II. A lot of news was published.
III. They left an item of furniture at the apartment.
IV. I had an excellent idea.
Choose the option which defines the underlined nouns as U (uncountable) ou C (countable).
(A) U – U – U – C
(B) U – C – U – C
(C) U – U – C – U
(D) U – C – U – C
(E) C – U – C – U
Comentários: A alternativa A está correta.
knowledge = conhecimento (substantivo incontável).
News = notícia (substantivo incontável).
Furniture = mobília (substantivo incontável).
Idea = ideia (substantivo contável que está no singular).
Observe que antes do substantivo “idea” temos um artigo indefinido “an” que são proibidos antes
de substantivo incontáveis e por isso “idea” só poderia ser um substantivo contável.
GABARITO: A

Questão 07 (COLÉGIO NAVAL/INÉDITA) – Mach the questions to the answers


I. How long does it take to take a shower?
II. Are you Thiago’s sister?
III. Where are you from?
IV. How far is our classroom from the bathroom?
( ) No, I’m not.
( ) It takes 15 minutes.
( ) It’s about 30 meters.
( ) I’m from London.
(A) I – II – IV – III
(B) II – I – IV – III
(C) I – II – III – IV
(D) II – IV – III – II
(E) IV – II – III – I

AULA 06 – CONJUNCTIONS, DIRECT SPEECH AND REPORTED SPEECH 60


TEACHER ANDREA BELO

Comentários: A alternativa B está correta.


I – A expressão “How long does it take” pergunta por período de tempo e por isso a resposta
adequada seria “it takes 15 minutes”.
II – A pergunta com o verbo “to be” terá a resposta com o verbo “to be” também. “Are you Thiago’s
sister?” (Você é irmã do Thiago?), uma possível resposta é “no, I’m not” (não, não sou).
III – A preposição from transmite a ideia de origem. “Where are you from?” (De onde você é?).
Portanto a resposta será “I’m from London” (eu sou de Londres).
IV – A expressão “How far” pergunta por distâncias e por isso a resposta adequada seria “It’s about
30 meters”.
GABARITO: B

Questão 08 (COLÉGIO NAVAL/INÉDITA) – Mark the option in which all the verbs are written in
the Simple Past.
(A) began – hit – known – left – spoken.
(B) won – wore – threw – stolen – read.
(C) sank – put – spent – spoke – left
(D) went – was – wanted – rise – lose
(E) had – hit – bought – cut – done.
Comentários: A alternativa A está incorreta. Known e spoken são os particípios dos verbos know
e speak, respectivamente. Todos os outros verbos estão no simple past.
A alternativa B está incorreta. stolen é o particípio do verbo steal. Todos os outros verbos estão
no simple past.
A alternativa C está correta. Todos os verbos contidos na alternativa C estão no simple past.
A alternativa D está incorreta. rise e lose são as formas básicas (infinitivo sem to) desses verbos.
Não estão nem no passado e nem no particípio. Todos os outros verbos estão no simple past.
A alternativa E está incorreta. done é o particípio do verbo do. Todos os outros verbos estão no
simple past.
GABARITO: C

Questão 09 (COLÉGIO NAVAL/INÉDITA) – Choose the alternative in which you can find the
Present Continuous.
(A) Practicing exercises is a great way to avoid health problems.
(B) I am interested in studying English in England.
(C) I’ll never get used to working until late.
(D) She is American, but she's living in Canada nowadays.
(E) He never stopped smoking.

AULA 06 – CONJUNCTIONS, DIRECT SPEECH AND REPORTED SPEECH 61


TEACHER ANDREA BELO

Comentários: A alternativa D está correta. O present continuous é caracterizado por sujeito +


verbo to be (no presente) + verbo com – in. A única opção que apresenta essa estrutura é a
alternativa D. (she's living in Canada). She (sujeito) is (verbo to be no presente) living (verbo com
–ING)
GABARITO: D

Questão 10 (COLÉGIO NAVAL/INÉDITA) – All alternatives are not in the comparative, except:
(A) reader.
(B) smoker.
(C) matter.
(D) manner.
(E) higher.
Comentários: A alternativa A está incorreta. reader = leitor.
A alternativa B está incorreta. smoker = fumante.
A alternativa C está incorreta. matter = matéria / assunto / importar.
A alternativa D está incorreta. manner = maneira / modo.
A alternativa E está correta. higher = mais alto. Cuidado, pois, apesar das outras palavras também
terminarem com –er, elas não formam o comparativo (comparativo = adjetivo / advérbio + er).
GABARITO: E

QUESTÕES EAM
Read text I and answer questions 01, 02 and 03
In the first half of 2021, more than 8,100 people died in the US because of gun violence. It is
___________ (1) number in the last 20 years.
A report said that nearly 54 Americans died every day in shootings. It was 14 more people than in
the same time in the past six years. Last weekend, more than 120 people died in shootings.
Experts said that there were more reasons why gun violence was growing. More people owned a
gun, and there were a lot of problems between police and communities. Also, there were big
differences in the economy and healthcare, and there were racial problems in American society
which have lasted a very long time.
Adapted from: https://www.newsinlevels.com/products/guns-in-the-us-level-2/

AULA 06 – CONJUNCTIONS, DIRECT SPEECH AND REPORTED SPEECH 62


TEACHER ANDREA BELO

Questão 01 (EAM/INÉDITA) – Choose the alternative containing the correct words to complete
gap (1)
(A) the highest
(B) higher than
(C) the most high
(D) more high
(E) the most highest
Comentários: A alternativa A está correta. O superlativo de adjetivos monossílabos é feito
adicionando – EST ao final da palavra. Observe que o adjetivo high é monossílabo e portanto, seu
superlativo é high + est = highest.
GABARITO: A

Questão 02 (EAM/INÉDITA) – In “A report said that nearly 54 Americans died every day in
shootings”, the word “nearly”, is closest in meaning to
(A) hardly.
(B) almost.
(C) near.
(D) close.
(E) always.
Comentários: A alternativa A está incorreta. Hardly = quase não.
A alternativa B está correta. Almost = nearly = quase.
A alternativa C está incorreta. Near = perto.
A alternativa D está incorreta. Close = perto.
A alternativa E está incorreta. Always = sempre.
GABARITO: B

Questão 03 (EAM/INÉDITA) – In “and there were a lot of problems between police and
communities”. The expression “a lot of” can be replaced by
(A) lots.
(B) much.
(C) few.
(D) little.
(E) many.
Comentários: A alternativa A está incorreta. Lots está errado, o correto seria ou “a lot of” ou “lots
of”.

AULA 06 – CONJUNCTIONS, DIRECT SPEECH AND REPORTED SPEECH 63


TEACHER ANDREA BELO

A alternativa B está incorreta. Much é usado com substantivos incontáveis e singulares.


A alternativa C está incorreta. Few deve ser usado com substantivos contáveis e plurais.
Entretanto “few” significa “poucos” e, nesse caso, “a lot of” significa muitos por concordar com o
substantivo “problems” que é plural.
A alternativa D está incorreta. Little é usado com substantivos incontáveis e singulares.
A alternativa E está correta. Many deve ser usado com substantivos contáveis e plurais. Observe
que “a lot of” está ao lado do substantivo “problems” que é contável e plural.
GABARITO: E

Read the following text to answer questions 04 and 05.


The construction of the Empire State Building began in 1930. It was built by 3,400 workers who
were mostly poor immigrants from Europe. They worked in any weather condition and with
simple tools. It was extremely dangerous work, but they made good money.
The building was constructed under great time pressure. They had to finish the work by May 1,
1931. The workers built 4 new floors every week and they finished the work in time.
The building is 381 meters high and has 102 floors. It has 6,500 windows and 73 elevators. Over
60,000 tons of steel were used for the construction. At the time, it was the tallest skyscraper in
the world.
Today, the building is still a symbol of New York. Thousands of people visit it every day. There are
also offices of many companies in the building and 20,000 people work there.
https://www.daysinlevels.com/article/the-day-the-empire-state-building-opened-level-2/

Questão 04 (EAM/INÉDITA) – In “They had to finish the work by May 1, 1931”. The word “they”
refers to
(A) the Empire State Building.
(B) good money.
(C) poor immigrants.
(D) workers.
(E) dangerous work.
Comentários: A alternativa D está correta. "They had to finish the work by May 1, 1931” (Eles
tiveram que terminar o trabalho em 1° de maio de 1931), segundo o texto, os trabalhadores
tiveram que terminar a obra até primeiro de maio.
GABARITO: D

AULA 06 – CONJUNCTIONS, DIRECT SPEECH AND REPORTED SPEECH 64


TEACHER ANDREA BELO

Questão 05 (EAM/INÉDITA) – According to the text


(A) The Empire State Building was built by more than six thousand five hundred workers
(B) The Empire State Building was designed by a group of European entrepreneurs.
(C) The Empire State Building has long been considered the tallest building in the world. Today it
remains a symbol of New York City.
(D) The workers who were involved in the construction of the Empire State Building besides
working in any weather conditions and with simple tools they still earned little money.
(E) Despite having to finish the work by May 1, 1931, the Empire State Building was built without
haste as everything was under control.
Comentários: A alternativa A está incorreta. De acordo com o texto o Empire State Building foi
construído por 3.400 trabalhadores, ele possui 6.500 janelas.
A alternativa B está incorreta. O texto não fala que o Empire State Building foi planejado por um
grupo de empreendedores europeus e sim que ele foi construído por um grupo de trabalhadores
que em sua maioria eram imigrantes pobres da Europa.
A alternativa C está correta. De fato, o Empire State Building foi o prédio mais alto do mundo e
apesar de não possuir mais esse marco, ele continua sendo um ponto importante da cidade de
Nova York.
A alternativa D está incorreta. Segundo o texto, de fato, os operários trabalhavam em qualquer
condição climática e com ferramentas simples, entretanto eles ganhavam um bom dinheiro.
A alternativa E está incorreta. De acordo com o texto, o prédio foi construído sob grande pressão
de tempo. Eles tinham que terminar a obra até 1º de maio de 1931 e por isso os trabalhadores
construíam quatro novos andares todas as semanas para poderem terminar a obra a tempo.
GABARITO: C

QUESTÕES EEAR
Read the text and answer questions 01 to 04.
Man living alone on Italian island to leave after 32 years
An Italian man is to leave the island he has lived on for more than 30 years after pressure from
authorities.
Mauro Morandi, 81, moved to Budelli off northern Sardinia in 1989.
Last year, however, Mr Morandi said the owners of the island, which is famed for its pink beach,
wanted him to relocate.
"I will leave hoping that in the future Budelli will be protected as I have protected it for 32 years,"
he wrote on Facebook on Sunday.
Speaking to BBC Outlook in 2018, the former physical education teacher said he had "always been
a bit of a rebel".

AULA 06 – CONJUNCTIONS, DIRECT SPEECH AND REPORTED SPEECH 65


TEACHER ANDREA BELO

"I was quite fed up with a lot of things about our society: consumerism and the political situation
in Italy," Mr Morandi said. "I decided to move to a desert island in Polynesia, away from all
civilisation. I wanted to start a new life close to nature."
After beginning their journey by sea, he and a number of friends landed on the Italian archipelago
of La Maddalena, where they planned to work and make money to fund the rest of their travels.
But after arriving on Budelli and meeting the island's guardian, who was about to retire, Mr
Morandi decided to stay - and has remained ever since, looking after the island and speaking to
tourists.
However, he has faced a number of eviction threats over the years.
In 2020 the president of La Maddalena's National Park, Fabrizio Fonnesu, told CNN that Mr
Morandi had carried out illegal alterations to his hut, a former radio station dating back to World
War Two.
A petition calling on the Italian government to allow him to stay gathered more than 70,000
signatures.
But on Sunday, Mr Morandi announced his decision to leave, telling the The Guardian newspaper
that he would move to a small flat on the nearby island of La Maddalena: "My life won't change
too much, I'll still see the sea."
(Adapted from https://www.bbc.com/news/world-europe-56885716)

Questão 01 (EEAR/INÉDITA) – According to the text, we can infer that ________


(A) Mauro's departure will occur on his own free will
(B) Despite leaving his home, Mauro still wants the best for the island
(C) The owners of the island are famous for having a pink beach
(D) Mauro began to become rebellious in recent years
Comentários: A alternativa A está incorreta. De acordo com o texto, não podemos inferir que a
saída de Mauro ocorrerá por livre e espontânea vontade, mas sim, que ocorrerá por pressão a
partir de autoridades. Isso pode ser confirmado com o trecho “An Italian man is to leave the island
he has lived on for more than 30 years after pressure from authorities”.
A alternativa B está correta. De acordo com o texto, podemos inferir que apesar de estar saindo
de casa, Mauro ainda quer o melhor para a ilha, assim como esta opção indica. Isso pode ser
confirmado com o trecho “I will leave hoping that in the future Budelli will be protected as I have
protected it for 32 years”.
A alternativa C está incorreta. De acordo com o texto, não podemos inferir que os donos da ilha
são famosos por terem uma praia rosa, mas sim, que a ilha é famosa por ter uma praia rosa. Isso
pode ser confirmado com o trecho “Last year, however, Mr Morandi said the owners of the island,
which is famed for its pink beach, wanted him to relocate”.
A alternativa D está incorreta. De acordo com o texto, não podemos inferir que Mauro começou
a se rebelar nos últimos anos, mas sim, que ele sempre foi um pouco rebelde. Isso pode ser
confirmado com o trecho “...said he had ‘always been a bit of a rebel’”.
GABARITO: B

AULA 06 – CONJUNCTIONS, DIRECT SPEECH AND REPORTED SPEECH 66


TEACHER ANDREA BELO

Questão 02 (EEAR/INÉDITA) – The passage “...he and a number of friends landed on the Italian
archipelago of La Maddalena, where they planned to work and make money to fund the rest of
their travels” states that the stay should be
(A) Permanent
(B) Pleasant
(C) Temporary
(D) Uncomfortable
Comentários: A alternativa A está incorreta. A passagem “...ele e vários amigos desembarcaram
no arquipélago italiano de La Maddalena, onde planejavam trabalhar e ganhar dinheiro para
financiar o resto de suas viagens” não afirma que a estadia deveria ser permanente, mas sim, que
deveria ser temporária.
A alternativa B está incorreta. A passagem “...ele e vários amigos desembarcaram no arquipélago
italiano de La Maddalena, onde planejavam trabalhar e ganhar dinheiro para financiar o resto de
suas viagens” não afirma que a estadia deveria ser agradável, mas sim, que deveria ser
temporária.
A alternativa C está correta. A passagem “...ele e vários amigos desembarcaram no arquipélago
italiano de La Maddalena, onde planejavam trabalhar e ganhar dinheiro para financiar o resto de
suas viagens” afirma que a estadia deveria ser temporária, assim como esta opção indica.
A alternativa D está incorreta. A passagem “...ele e vários amigos desembarcaram no arquipélago
italiano de La Maddalena, onde planejavam trabalhar e ganhar dinheiro para financiar o resto de
suas viagens” não afirma que a estadia deveria ser desconfortável, mas sim, que deveria ser
temporária.
GABARITO: C

Questão 03 (EEAR/INÉDITA) – The sentence in bold in the text is in the


(A) Simple present
(B) Past continuous
(C) Present perfect
(D) Simple past
Comentários: A alternativa A está incorreta. A frase “Eu estava farto de muitas coisas sobre a
nossa sociedade” não está no presente simples, mas sim, no passado simples (simple past),
indicando algo que aconteceu e terminou em algum momento do passado.
A alternativa B está incorreta. A frase “Eu estava farto de muitas coisas sobre a nossa sociedade”
não está no passado contínuo, mas sim, no passado simples (simple past), indicando algo que
aconteceu e terminou em algum momento do passado.

AULA 06 – CONJUNCTIONS, DIRECT SPEECH AND REPORTED SPEECH 67


TEACHER ANDREA BELO

A alternativa C está incorreta. A frase “Eu estava farto de muitas coisas sobre a nossa sociedade”
não está no presente perfeito, mas sim, no passado simples (simple past), indicando algo que
aconteceu e terminou em algum momento do passado.
A alternativa D está correta. A frase “Eu estava farto de muitas coisas sobre a nossa sociedade”
está no passado simples (simple past), indicando algo que aconteceu e terminou em algum
momento do passado, assim como esta opção indica.
GABARITO: D

Questão 04 (EEAR/INÉDITA) – In “In 2020 the president of La Maddalena's National Park,


Fabrizio Fonnesu, told CNN that Mr Morandi had carried out illegal alterations to his hut, a
former radio station dating back to World War Two”, the word “his” refers to
(A) Mr Morandi
(B) Fabrizio Fonnesu
(C) Hut
(D) CNN
Comentários: A alternativa A está correta. A palavra “his” (dele) se refere à Mauro Morandi, que
é citado anteriormente na frase, assim como esta opção indica. Isso pode ser confirmado com o
trecho “Em 2020, o presidente do Parque Nacional de La Maddalena, Fabrizio Fonnesu, disse à
CNN que Morandi havia feito alterações ilegais em sua cabana, uma antiga estação de rádio que
datava da Segunda Guerra Mundial”.
A alternativa B está incorreta. A palavra “his” (dele) não se refere à Fabrizio Fonnesu, mas sim, à
Mauro Morandi, que é citado anteriormente na frase. Isso pode ser confirmado com o trecho “Em
2020, o presidente do Parque Nacional de La Maddalena, Fabrizio Fonnesu, disse à CNN que
Morandi havia feito alterações ilegais em sua cabana, uma antiga estação de rádio que datava da
Segunda Guerra Mundial”.
A alternativa C está incorreta. A palavra “his” (dele) não se refere à cabana, mas sim, à Mauro
Morandi, que é citado anteriormente na frase. Isso pode ser confirmado com o trecho “Em 2020,
o presidente do Parque Nacional de La Maddalena, Fabrizio Fonnesu, disse à CNN que Morandi
havia feito alterações ilegais em sua cabana, uma antiga estação de rádio que datava da Segunda
Guerra Mundial”.
A alternativa D está incorreta. A palavra “his” (dele) não se refere à CNN, mas sim, à Mauro
Morandi, que é citado anteriormente na frase. Isso pode ser confirmado com o trecho “Em 2020,
o presidente do Parque Nacional de La Maddalena, Fabrizio Fonnesu, disse à CNN que Morandi
havia feito alterações ilegais em sua cabana, uma antiga estação de rádio que datava da Segunda
Guerra Mundial”.
GABARITO: A

AULA 06 – CONJUNCTIONS, DIRECT SPEECH AND REPORTED SPEECH 68


TEACHER ANDREA BELO

Read the comic strip and answer questions 05 and 06.

(Adapted from https://comics.azcentral.com/slideshow?comic=dt)

Questão 05 (EEAR/INÉDITA) – Choose the best alternative to complete the blank


(A) Reduce
(B) Reduced
(C) Are reducing
(D) Have reduced
Comentários: A alternativa A está incorreta. O tempo verbal correto do verbo to reduce nesse
caso não é no simple present, mas sim, no simple past (reduced), considerando que a tirinha
indica algo que aconteceu no último ano.
A alternativa B está correta. O tempo verbal correto do verbo to reduce nesse caso é no simple
past (reduced), considerando que a tirinha indica algo que aconteceu no último ano, assim como
esta opção indica.
A alternativa C está incorreta. O tempo verbal correto do verbo to reduce nesse caso não é no
presente continuous, mas sim, no simple past (reduced), considerando que a tirinha indica algo
que aconteceu no último ano.
A alternativa D está incorreta. O tempo verbal correto do verbo to reduce nesse caso não é no
present perfect, mas sim, no simple past (reduced), considerando que a tirinha indica algo que
aconteceu no último ano.
GABARITO: B

Questão 06 (EEAR/INÉDITA) – Choose the best alternative according to the comic strip
(A) The company's occupational accidents were completely extinguished last year
(B) A small percentage of accidents was reduced last year
(C) The security guard was the only one that didn’t allow a total decrease in workplace injuries
(D) The security guard was an honest worker

AULA 06 – CONJUNCTIONS, DIRECT SPEECH AND REPORTED SPEECH 69


TEACHER ANDREA BELO

Comentários: A alternativa A está incorreta. De acordo com a tirinha, não é correto afirmar que
os acidentes de trabalho da empresa foram totalmente extintos no ano passado, mas sim, que
foram reduzidos em 76%. Isso pode ser confirmado com o trecho “...reduced workplace injuries
by 76% this past year”.
A alternativa B está incorreta. De acordo com a tirinha, não é correto afirmar que uma pequena
porcentagem de acidentes foi reduzida no ano passado, mas sim, que foram reduzidos em 76%.
Isso pode ser confirmado com o trecho “...reduced workplace injuries by 76% this past year”.
A alternativa C está correta. De acordo com a tirinha, é correto afirmar que o segurança foi o único
que não permitiu uma diminuição total dos acidentes de trabalho, assim como esta opção indica;
considerando que estavam todos trabalhando de casa menos o segurança. Isso pode ser
confirmado com o trecho “Our security guard kept hurting his back stealing office equipment”.
A alternativa D está incorreta. De acordo com a tirinha, não é correto afirmar que o segurança era
um trabalhador honesto, mas sim, que ele não era pois ele roubava equipamento do escritório.
Isso pode ser confirmado com o trecho “Our security guard kept hurting his back stealing office
equipment”.
GABARITO: C

Read the comic strip and answer questions 07 and 08.

(Adapted from https://www.gocomics.com/pickles/2021/04/24)

Questão 07 (EEAR/INÉDITA) – Choose the best alternative to complete the blank


(A) Wanted
(B) Want
(C) Have wanted
(D) Are wanting
Comentários: A alternativa A está incorreta. O tempo verbal correto do verbo to want nesse
caso não é no simple past, mas sim, no simple present (want - quer), considerando que a tirinha
indica algo está acontecendo agora.

AULA 06 – CONJUNCTIONS, DIRECT SPEECH AND REPORTED SPEECH 70


TEACHER ANDREA BELO

A alternativa B está correta. O tempo verbal correto do verbo to want nesse caso é no simple
present (want - quer), considerando que a tirinha indica algo está acontecendo agora, assim
como esta opção indica.
A alternativa C está incorreta. O tempo verbal correto do verbo to want nesse caso não é no
present perfect, mas sim, no simple present (want - quer), considerando que a tirinha indica
algo está acontecendo agora.
A alternativa D está incorreta. O tempo verbal correto do verbo to want nesse caso não é no
present continuous, mas sim, no simple present (want - quer), considerando que a tirinha indica
algo está acontecendo agora.
GABARITO: B

Questão 08 (EEAR/INÉDITA) – According to what Sylvia said, she __________.


(A) Thinks her father's hair is messy and that he stinks
(B) Thinks her father's hair stinks
(C) Doubts that her father wants to look good to her mother
(D) Thinks her father is not taking very good care of his appearance
Comentários: A alternativa A está incorreta. De acordo com o que Sylvia disse, não é correto
afirmar que ela acha que o cabelo do pai está bagunçado e que ele fede, mas sim, que ela só acha
que seu pai não está cuidando de seu cabelo. Isso pode ser confirmado com o trecho “...doesn’t
mean you don’t need to take care of your hair anymore”.
A alternativa B está incorreta. De acordo com o que Sylvia disse, não é correto afirmar que ela
acha que o cabelo do pai fede, mas sim, que ele não está cuidando de seu cabelo. Isso pode ser
confirmado com o trecho “...doesn’t mean you don’t need to take care of your hair anymore”.
A alternativa C está incorreta. De acordo com o que Sylvia disse, não é correto afirmar que ela
duvida que seu pai queira ter uma boa aparência para sua mãe, mas sim, que ela quer que ele
pareça bem para sua mãe. Isso pode ser confirmado com o trecho “...you want to look good for
mom, don’t you?”; esse trecho não indica uma dúvida dela, mas sim, um desejo.
A alternativa D está correta. De acordo com o que Sylvia disse, é correto afirmar que ela acha que
o pai dela não está cuidando muito bem da aparência, assim como esta opção indica. Isso pode
ser confirmado com o trecho “...doesn’t mean you don’t need to take care of your hair anymore”.
GABARITO: D

Read the lyrics and answer questions 09 and 10.


As I Am – Justin Bieber
By myself sometimes
To give my mind some space
Yeah, I know, yeah, I know that it hurts

AULA 06 – CONJUNCTIONS, DIRECT SPEECH AND REPORTED SPEECH 71


TEACHER ANDREA BELO

When I push your love away, I hate myself


I wanna tell you lies
So your heart won't break
Yeah, I know, yeah, I know that I made my fair share of mistakes
Sometimes, I don't know why you love me
Sometimes, I don't know why you care
Take me with the good and the ugly
Say, "I'm not goin' anywhere"
Take me as I am, swear I'll do the best I can
Say, "I'm not goin' anywhere"
Take me as I am, swear I'll do the best I can
Say, "I'm not goin' anywhere"
Thousand miles apart, yeah, you're the one that holds my heart
It's no surprise
And I tell you all the time
And when life gets way too hard
I'll meet you in the stars, you know I will
I'll keep you near (mmm)
As I wipe away your tears…
(Adapted from https://genius.com/Justin-bieber-as-i-am-lyrics)

Questão 09 (EEAR/INÉDITA) – According to the lyrics


(A) The song expresses a relationship based on love but, at the same time, insecurity
(B) The song portrays an abusive relationship
(C) The song criticizes the emotional dependence that is developed in some relationships
(D) The song delves into the idea of self-love as the basis for other relationships
Comentários: A alternativa A está correta. De acordo com a letra, é correto afirmar que a música
expressa uma relação de amor, mas, ao mesmo tempo, de insegurança, assim como esta opção
indica. Isso pode ser confirmado com o trecho “When I push your love away, I hate myself / I
wanna tell you lies / So your heart won't break”.
A alternativa B está incorreta. De acordo com a letra, não é correto afirmar que a música retrata
um relacionamento abusivo, mas sim, que ela expressa uma relação de amor, mas, ao mesmo
tempo, de insegurança. Isso pode ser confirmado com o trecho “When I push your love away, I
hate myself / I wanna tell you lies / So your heart won't break”.

AULA 06 – CONJUNCTIONS, DIRECT SPEECH AND REPORTED SPEECH 72


TEACHER ANDREA BELO

A alternativa C está incorreta. De acordo com a letra, não é correto afirmar que a música critica a
dependência emocional que se desenvolve em alguns relacionamentos, mas sim, que ela expressa
uma relação de amor, mas, ao mesmo tempo, de insegurança. Isso pode ser confirmado com o
trecho “When I push your love away, I hate myself / I wanna tell you lies / So your heart won't
break”.
A alternativa D está incorreta. De acordo com a letra, não é correto afirmar que a música investiga
a ideia de amor-próprio como base para outros relacionamentos, mas sim, que ela expressa uma
relação de amor, mas, ao mesmo tempo, de insegurança. Isso pode ser confirmado com o trecho
“When I push your love away, I hate myself / I wanna tell you lies / So your heart won't break”.
GABARITO: A

Questão 10 (EEAR/INÉDITA) – The underlined phrase “you're the one that holds my heart”, in
the text, means that the person
(A) Is the one that really holds the other person’s heart
(B) Is the one that has all the love and appreciation of the other person
(C) Is the one who has full responsibility for the other person's mental health
(D) Is the one that keeps the other person alive
Comentários: A alternativa A está incorreta. A frase “você é aquele que segura meu coração” não
significa que a pessoa é aquela que realmente segura o coração da outra, mas sim, que a pessoa
é aquela que tem todo o amor e apreço da outra pessoa.
A alternativa B está correta. A frase “você é aquele que segura meu coração” significa que a
pessoa é aquela que tem todo o amor e apreço da outra pessoa, assim como esta opção indica.
A alternativa C está incorreta. A frase “você é aquele que segura meu coração” não significa que
a pessoa é aquela que tem total responsabilidade pela saúde mental da outra pessoa, mas sim,
que a pessoa é aquela que tem todo o amor e apreço da outra pessoa.
A alternativa D está incorreta. A frase “você é aquele que segura meu coração” não significa que
a pessoa é aquela que mantém a outra pessoa viva, mas sim, que a pessoa é aquela que tem todo
o amor e apreço da outra pessoa.
GABARITO: B

QUESTÕES EFOMM
Royal Navy flotilla reports Covid outbreak with 100 cases despite all sailors being vaccinated
The outbreak has not prompted a rethink of the Royal Navy’s voyage to 40 countries
More than 100 cases of coronavirus have been reported aboard the Royal Navy’s new aircraft
carrier and an escorting group of warships.
The vessels affected include the Royal Navy’s flagship HMS Queen Elizabeth carrier, with
unconfirmed media reports saying the group of warships recently stopped off for a brief spell of
shore leave in Limassol, Cyprus.

AULA 06 – CONJUNCTIONS, DIRECT SPEECH AND REPORTED SPEECH 73


TEACHER ANDREA BELO

The virus has spread through the strike group’s 3,700 strength crew despite them being fully
vaccinated. “As part of routine testing, a small number of crew from the Carrier Strike Group have
tested positive for Covid-19,” a Royal Navy spokesman confirmed in a statement.
The incident comes despite a series of measures deployed on board Navy warships in light of the
pandemic. These include the use of masks, social distancing and a track and trace system.
The outbreak has not yet prompted a rethink of the strike group’s planned voyage to 40 countries,
said a Royal Navy spokesperson. “The Carrier Strike Group will continue to deliver their
operational tasks and there are no effects on the deployment.”
The aircraft carrier was about a quarter of the way through its 28-week deployment when the
Covid cases were detected. It is leading a strike group that includes 10 Marine Corps F35-B fighters
and is currently in the Indo-Pacific.
It is the largest combined naval and air task force assembled under British command since the
Falklands War, and was expected to pass through the South China Sea.
Ben Wallace, the defence secretary, said during a press briefing on Tuesday that the first cases
involving the strike group were reported on 4 July.
“Our crew are double vaccinated so you’ll be glad to know there is no serious effects on any of
the crew and we will manage it,” he said, adding that going forwards he will offer support to the
captain of the ship in “whatever decision he makes”.
https://www.independent.co.uk/news/uk/home-news/hms-queen-elizabeth-covid-royal-navy-b1883699.html

Questão 01 (EFOMM/INÉDITA) – In the extract from the text: “[...] the Royal Navy’s new aircraft
carrier and an escorting group of warships.” (paragraph 1). The word in bold is closest in
meaning to
A) enemy
B) convoy
C) foe
D) outfit
E) gear
Comentários: A alternativa A está incorreta. Enemy = inimigo.
A alternativa B está correta. Convoy = comboio / escolta. Convoy é sinônimo de escorting que
significa “escolta”.
A alternativa C está incorreta. Foe = inimigo.
A alternativa D está incorreta. Outfit = equipamento.
A alternativa E está incorreta. Gear = engrenagem.
GABARITO: B

AULA 06 – CONJUNCTIONS, DIRECT SPEECH AND REPORTED SPEECH 74


TEACHER ANDREA BELO

Questão 02 (EFOMM/INÉDITA) – In the extract from the text: “The outbreak has not yet
prompted a rethink of the strike group’s planned voyage to 40 countries” (paragraph 5), the
verb “to prompt” means
A) to arouse
B) to derogate
C) to prevent
D) to keep
E) to hinder
Comentários: A alternativa A está correta. To arouse = despertar / provocar / estimular. “The
outbreak has not yet prompted a rethink of the strike group’s planned voyage to 40 countries” (O
surto ainda não levou (provocou / despertou) a um repensar da viagem planejada do grupo de
greve para 40 países).
A alternativa B está incorreta. To derogate = derrogar / diminuir / abolir
A alternativa C está incorreta. To prevent = prevenir.
A alternativa D está incorreta. To keep = manter.
A alternativa E está incorreta. To hinder = impedir.
GABARITO: A

Questão 03 (EFOMM/INÉDITA) – What is true according to the text?


A) Although most of the crew were fully vaccinated, there was an outbreak of Covid 19 inside the
English ship with another 100 confirmed cases.
B) Despite the Covid 19 outbreak, only the Royal Navy's HMS Queen Elizabeth aircraft carrier was
affected by the virus, thanks to security measures adopted on other ships.
C) Although the crew of the British ship was completely immunized against Covid 19, there was
still an outbreak of the virus with dozens of confirmed cases.
D) The Covid 19 outbreak shows that the Royal Navy's HMS Queen Elizabeth aircraft carrier was
not taking the necessary measures to control the virus, such as wearing masks and social distance.
E) The Covid 19 outbreak caused the British Navy to think twice about the voyage that had been
planned for the ship.
Comentários: A alternativa A está incorreta. De acordo com o texto, toda a tripulação foi
vacinada, e não a maioria dela.
A alternativa B está incorreta. Segundo o texto, mais de 100 casos de coronavírus foram relatados
a bordo do novo porta-aviões da Marinha Real e um grupo de escolta de navios de guerra. Ou
seja, o porta-aviões HMS Queen Elizabeth não foi o único afetado.
A alternativa C está correta. De acordo com o texto, a flotilha da Marinha Real relatou um surto
de Covid com mais de 100 casos, apesar de todos os marinheiros terem sido vacinados.

AULA 06 – CONJUNCTIONS, DIRECT SPEECH AND REPORTED SPEECH 75


TEACHER ANDREA BELO

A alternativa D está incorreta. De acordo com o texto, o incidente ocorreu apesar de uma série
de medidas implantadas a bordo de navios de guerra da Marinha em função da pandemia. Isso
inclui o uso de máscaras, distanciamento social e um sistema de rastreio e rastreamento.
A alternativa E está incorreta. De acordo com o texto, o surto não levou a um repensar da viagem
da Marinha Real a 40 países.
GABARITO: C

Questão 04 (EFOMM/INÉDITA) – In the extract from the text: “[...] there is no serious effects on
any of the crew and we will manage it” (paragraph 5), the word “it” refers to
A) the largest combined naval and air task force.
B) the strike group.
C) the Falklands War
D) the outbreak of covid 19.
E) the captain of the ship
Comentários: A alternativa D está correta. “Our crew are double vaccinated so you’ll be glad to
know there is no serious effects on any of the crew and we will manage it” (Nossa tripulação tem
vacinação dupla, então você ficará feliz em saber que não há efeitos sérios em qualquer membro
da tripulação e nós cuidaremos disso (da situação do contágio de covid 19).
GABARITO: D.

Read the text below to answer question 05


Lewis Hamilton hopes for diversity legacy
Sir Lewis Hamilton has said he hopes improving the diversity of Formula One will be part of his
legacy.
The seven-times world champion ____________ a report which he hopes will encourage the
industry to become more inclusive and inspire young people from ethnic minorities to pursue a
career in the sport.
https://www.bbc.com/news/av/uk-57814103

Questão 05 (EFOMM/INÉDITA) – Fill in the blank with the suitable verb


A) have commissioned
B) had commissioned
C) have been commissioned
D) has been commissioned
E) has commissioned

AULA 06 – CONJUNCTIONS, DIRECT SPEECH AND REPORTED SPEECH 76


TEACHER ANDREA BELO

Comentários: A alternativa E está correta. Usaremos o present perfect por se tratar de uma ação
que ocorreu em um passado indeterminado. Como o sujeito da oração é “The seven-times world
champion” (O heptacampeão mundial), no caso, Lewis Hamilton que é terceira pessoa do singular,
portanto usaremos o verbo auxiliar “has” para formar o present perfect.
Present perfect = Have / has + verbo no particípio.
A alternativa D está incorreta por não se tratar de uma voz passiva.
GABARITO: E.

Read the text below to answer questions 06 and 07


A pastor in Sierra Leone came across a jaw-dropping stone – a 706-carat (141 grams) diamond. It
is the largest diamond to be found in Sierra Leone in over 40 years.
When the pastor __________ it, he decided __________ it over to the government. __________
that it may boost the development of the West African nation.
The President of Sierra Leone said that he believed that a diamond like this should be publicly
__________ in the country, so that people will know the value of it and who the buyer is.
https://www.newsinlevels.com/products/big-diamond-in-sierra-leone

Questão 06 (EFOMM/INÉDITA) – Which is the correct way to complete the text above?
A) found / to handing / Hope / sold.
B) find / hand / Hope / selling.
C) found / to hand / Hoping / sold.
D) find / to handing / Hoping / selling
E) found / to hand / Hope / sell
Comentários: A alternativa C está correta.
Lacuna 1: Trata-se de uma oração no passado. Quando fatos ou ações passadas são apresentados
em ordem cronológica, usa-se o verbo no simple past depois da conjunção when. O passado do
verbo find é found.
Lacuna 2: Após o verbo “to decide” usamos o verbo no infinitivo. (to hand).
Lacuna 3: Hoping = Esperando / Na esperança. “Hoping that it may boost the development of the
West African nation” (Na esperança de que isso possa impulsionar o desenvolvimento da nação
da África Ocidental).
Lacuna 4: Trata-se de um caso de voz passiva “should + be + particípio” (should be sold).
GABARITO: C

AULA 06 – CONJUNCTIONS, DIRECT SPEECH AND REPORTED SPEECH 77


TEACHER ANDREA BELO

Questão 07 (EFOMM/INÉDITA) – In “A pastor in Sierra Leone came across a jaw-dropping


stone”, the expression in bold means
A) huge
B) ugly.
C) breathtaking
D) creepy
E) grisly
Comentários: A alternativa A está incorreta. Huge = grande / enorme.
A alternativa B está incorreta. Ugly = feio.
A alternativa C está correta. Breathtaking = de tirar o fôlego. A expressão “jaw-dropping” significa
“de cair o queixo”. Ou seja, significa que algo é surpreendente ou “de tirar o fôlego”.
A alternativa D está incorreta. Creepy = assustador
A alternativa E está incorreta. Grisly = terrível.
GABARITO: C

Questão 08 (EFOMM/INÉDITA) – Which is the correct way to complete the paragraph below?
Fast food is one of those things that the USA __________ to the world. But while hamburgers and
fries __________ worldwide as __________ typically American, in actual fact Americans did not
invent fast food - they just made it into a lifestyle.
Adapted from: https://linguapress.com/advanced/american-food.htm
A) had given / is seen / to be.
B) have given / are seen / been.
C) has given / see / being.
D) have given / see / been.
E) has given / are seen / being.
Comentários: A alternativa E está correta.
Lacuna 1: Usamos o present perfect para expressar ações que ocorreram em um passado
indeterminado. Como o sujeito é “the USA” usaremos o verbo auxiliar “has”. Portanto fica “has
given”.
Lacuna 2: Encontra-se na voz passiva (to be + particípio). Como o sujeito está no plural, usaremos
“are + particípio”, ou seja, “are seen”.
Lacuna 3: As being = como sendo. “Fast food is one of those things that the USA has given to the
world. But while hamburgers and fries are seen worldwide as being typically American” (Fast food
é uma daquelas coisas que os EUA deram ao mundo. Mas enquanto hambúrgueres e batatas fritas
são vistos em todo o mundo como sendo tipicamente americanos).
GABARITO: E

AULA 06 – CONJUNCTIONS, DIRECT SPEECH AND REPORTED SPEECH 78


TEACHER ANDREA BELO

Read the text below to answer questions 09 and 10


_____ the night of April 4th 1968, someone was waiting opposite the windows of the Lorraine
Motel, _____ downtown Memphis.
_____ front of the motel, a big white Cadillac was parked; it was the car _____ which the Rev.
Martin Luther King was being driven round, as he traveled through the southern states, speaking
to audiences _____ towns and cities, promoting the cause of non-violence and civil rights.
When King stepped out onto the balcony, to take a breath of fresh air after eating his dinner, a
shot rang out. The civil rights leader and Nobel-prizewinner, the man who preached non-violence,
fell to the ground, fatally wounded. Within minutes, he was dead.
https://linguapress.com/advanced/martin-luther-king.htm

Questão 09 (EFOMM/INÉDITA) – Fill in the blank with the suitable prepositions


A) At / on / In / in / in
B) On / in / At / on / on
C) At / on / In / on / in
D) At / on / At / in / at
E) On / in / In / in / in
Comentários: A alternativa E está correta.
Lacuna 1: at night é utilizado para se referir a dado momento do dia – como se ele fosse fatiado
em: manhã, tarde e noite. Entretanto, é cabível, também, o uso da preposição on para se referir
a uma noite específica que é o caso da lacuna 1.
Lacuna 2: Usamos a preposição in para descrever lugares maiores como bairros, cidades, países.
“downtown Memphis” = centro de Memphis (lugar grande).
Lacuna 3: In front of é uma preposição de lugar (em frente).
Lacuna 4: In which = em que / no qual.
Lacuna 5: Usamos a preposição in para descrever lugares maiores como bairros, cidades, países.
GABARITO: E.

Questão 10 (EFOMM/INÉDITA) – In “When King stepped out onto the balcony, to take a breath
of fresh air after eating his dinner”, the expression in bold means
A) to jump from somewhere high.
B) to stay somewhere watching something specific.
C) to hide from someone who is chasing you.
D) to leave a place for a short time.
E) to find someone by chance.

AULA 06 – CONJUNCTIONS, DIRECT SPEECH AND REPORTED SPEECH 79


TEACHER ANDREA BELO

Comentários: A alternativa A está incorreta. to jump from somewhere high = pular de algum lugar
alto.
A alternativa B está incorreta. to stay somewhere watching something specific = ficar em algum
lugar assistindo a algo específico.
A alternativa C está incorreta. to hide from someone who is chasing you = se esconder de alguém
que está perseguindo você.
A alternativa D está correta. to leave a place for a short time = deixar um local por um curto
período de tempo. Essa é exatamente a definição da expressão "step out”. “When King stepped
out onto the balcony, to take a breath of fresh air after eating his dinner” (Quando King saiu
para a varanda, para tomar um fôlego de ar fresco depois de comer seu jantar).
A alternativa E está incorreta. to find someone by chance = encontrar alguém por acaso.
GABARITO: D

QUESTÕES EPCAR
Directions: Answer questions 01 to 10 according to text.
France to drop testing requirements for vaccinated tourists from some countries
France is making itself available as a destination for international tourists who have had
coronavirus jabs.
The government has announced that it is removing the need for coronavirus tests for vaccinated
Europeans and also allowing vaccinated tourists from most of the rest of the world, including the
US, to visit, provided they have a negative test.
Associated Press report that the relaxed rules will kick in from Wednesday. Tourism will not be
possible, however, from countries with virus surges and or prevalent variants. This “red list” for
the moment has 16 countries, including India, South Africa, and Brazil.
Outside of Europe, most of the rest of the world is classed as “orange” in the new travel rulebook
released today.
Vaccinated visitors from “orange” countries — including the US and Britain — will no longer need
to quarantine on arrival and will no longer have to justify the reasons for their trip to France. They
will, however, still be asked for a negative PCR test no older than 72 hours or a negative antigenic
test of no more than 48 hours.
Unvaccinated children will be allowed in with vaccinated adults.
European visitors and those from seven countries classed as “green” — Australia, South Korea,
Israel, Japan, Lebanon, New Zealand, and Singapore — will no longer need to undergo testing if
they are vaccinated.
(Adapted from https://www.theguardian.com/world/live/2021/jun/04/coronavirus-live-news-japan-taiwan-vaccine-
fauci-calls-wuhan-lab-records?page=with:block-60b9f3308f082ac930ae9fb8#block-60b9f3308f082ac930ae9fb8)

AULA 06 – CONJUNCTIONS, DIRECT SPEECH AND REPORTED SPEECH 80


TEACHER ANDREA BELO

Questão 01 (EPCAR/INÉDITA) – Mark the option that can replace the word “drop” in the title
without changing its meaning
A) Abandon
B) Keep
C) Allow
D) Return
Comentários: A alternativa A está correta. A palavra “drop”, nesse caso, significa
retirar/abandonar, e pode ser substituída pela palavra “abandon”, que significa abandonar.
A alternativa B está incorreta. A palavra “drop”, nesse caso, significa retirar/abandonar, e não
pode ser substituída pela palavra “keep”, que significa manter.
A alternativa C está incorreta. A palavra “drop”, nesse caso, significa retirar/abandonar, e não
pode ser substituída pela palavra “allow”, que significa permitir.
A alternativa D está incorreta. A palavra “drop”, nesse caso, significa retirar/abandonar, e não
pode ser substituída pela palavra “return”, que significa retornar.
GABARITO: A

Questão 02 (EPCAR/INÉDITA) – We can deduce from the first paragraph that


A) In France, tests are not required for any vaccinated tourist from any country
B) Europeans are the only ones who will still need negative tests to enter France
C) Vaccinated tourists from any country will be allowed to enter France, some may depend on a
negative test
D) Despite the advance, France's borders are not yet fully open
Comentários: A alternativa A está incorreta. De acordo com o primeiro parágrafo, não podemos
deduzir que na França, os testes não são exigidos para nenhum turista vacinado de qualquer país,
mas sim, que Europeus vacinados não precisarão mostrar teste negativo para entrar no país,
enquanto outros turistas de outros países precisarão estar vacinados e mostrar teste negativo.
Isso pode ser confirmado com o trecho “...it is removing the need for coronavirus tests for
vaccinated Europeans and also allowing vaccinated tourists from most of the rest of the world,
including the US, to visit, provided they have a negative test”.
A alternativa B está incorreta. De acordo com o primeiro parágrafo, não podemos deduzir que os
europeus são os únicos que ainda precisarão de exames negativos para entrar na França, mas sim,
que eles são os únicos que poderão entrar na França com vacina e sem testes. Isso pode ser
confirmado com o trecho “...it is removing the need for coronavírus tests for vaccinated
Europeans...”.
A alternativa C está incorreta. De acordo com o primeiro parágrafo, não podemos deduzir que
turistas vacinados de qualquer país terão permissão para entrar na França, alguns podem
depender de um teste negativo, mas sim, que apenas turistas vacinados da maioria dos outros

AULA 06 – CONJUNCTIONS, DIRECT SPEECH AND REPORTED SPEECH 81


TEACHER ANDREA BELO

países poderão entrar com teste negativo, isso não inclui certos países. Isso pode ser confirmado
com o trecho “...and also allowing vaccinated tourists from most of the rest of the world…”.
A alternativa D está correta. De acordo com o primeiro parágrafo, podemos deduzir que apesar
do avanço, as fronteiras da França ainda não estão totalmente abertas, assim como esta opção
indica. Isso pode ser confirmado com o trecho “The government has announced that it is removing
the need for coronavirus tests for vaccinated Europeans and also allowing vaccinated tourists from
most of the rest of the world…”.
GABARITO: D

Questão 03 (EPCAR/INÉDITA) – Mark the option with the suitable question to answer the
underlined fragment below “Associated Press report that the relaxed rules will kick in from
Wednesday. Tourism will not be possible, however, from countries with virus surges and or
prevalent variants”.
A) Tourists from which countries will be able to enter France?
B) When will the announced rules take effect?
C) What were the relaxed rules announced by France?
D) When were the relaxed rules announced by the government?
Comentários: A alternativa A está incorreta. A frase “A Associated Press informa que as regras
relaxadas entrarão em vigor na quarta-feira” não responde à pergunta “Turistas de quais países
poderão entrar na França?”, mas sim, à pergunta “Quando as regras anunciadas entrarão em
vigor?”.
A alternativa B está correta. A frase “A Associated Press informa que as regras relaxadas entrarão
em vigor na quarta-feira” responde à pergunta “Quando as regras anunciadas entrarão em
vigor?”, assim como esta opção indica.
A alternativa C está incorreta. A frase “A Associated Press informa que as regras relaxadas
entrarão em vigor na quarta-feira” não responde à pergunta “Quais foram as regras relaxadas
anunciadas pela França?”, mas sim, à pergunta “Quando as regras anunciadas entrarão em
vigor?”.
A alternativa D está incorreta. A frase “A Associated Press informa que as regras relaxadas
entrarão em vigor na quarta-feira” não responde à pergunta “Quando as regras relaxadas foram
anunciadas pelo governo?”, mas sim, à pergunta “Quando as regras anunciadas entrarão em
vigor?”.
GABARITO: B

AULA 06 – CONJUNCTIONS, DIRECT SPEECH AND REPORTED SPEECH 82


TEACHER ANDREA BELO

Questão 04 (EPCAR/INÉDITA) – Mark the option that can replace the underlined word keeping
the same meaning “Vaccinated visitors from ‘orange’ countries — including the US and Britain —
will no longer need to quarantine on arrival and will no longer have to justify the reasons for
their trip to France”
A) Deny
B) Accuse
C) Legitimize
D) Legalize
Comentários: A alternativa A está incorreta. A palavra “justify” significa justificar e não pode ser
substituída pela palavra “deny”, que significa negar.
A alternativa B está incorreta. A palavra “justify” significa justificar e não pode ser substituída pela
palavra “accuse”, que significa acusar.
A alternativa C está correta. A palavra “justify” significa justificar e pode ser substituída pela
palavra “legitimize”, que significa legitimar.
A alternativa D está incorreta. A palavra “justify” significa justificar e não pode ser substituída pela
palavra “legalize”, que significa legalizar.
GABARITO: C

Questão 05 (EPCAR/INÉDITA) – Mark the alternative that can answer the question below
according to the text. In addition to Europe, how is the pandemic situation in the rest of the
world?
A) All countries are classified as "orange"
B) These other countries are ranked between "orange" and "red", where “red” harbors a worse-
off minority
C) Visitors from all other countries are eligible to enter France testing negative for Covid-19
D) The pandemic situation in the rest of the world is still very serious, so the French borders
remain completely closed
Comentários: A alternativa A está incorreta. A pergunta “Além da Europa, como está a situação
da pandemia no resto do mundo?” não é bem respondida com “Todos os países são classificados
como ‘laranja’”, pois a maioria dos países fora da Europa estão em situação “laranja”, mas não
todos. Isso pode ser confirmado com o trecho “Tourism will not be possible, however, from
countries with virus surges and or prevalent variants. This ‘red list’ … Outside of Europe, most of
the rest of the world is classed as ‘orange’…”.
A alternativa B está correta. A pergunta “Além da Europa, como está a situação da pandemia no
resto do mundo?” é bem respondida com “Esses outros países estão classificados entre ‘laranja’
e ‘vermelho’, onde ‘vermelho’ abriga uma minoria em pior situação”, assim como esta opção
indica. Isso pode ser confirmado com o trecho “Tourism will not be possible, however, from
countries with virus surges and or prevalent variants. This ‘red list’ for the moment has 16
countries … Outside of Europe, most of the rest of the world is classed as ‘orange’…”.

AULA 06 – CONJUNCTIONS, DIRECT SPEECH AND REPORTED SPEECH 83


TEACHER ANDREA BELO

A alternativa C está incorreta. A pergunta “Além da Europa, como está a situação da pandemia no
resto do mundo?” não é bem respondida com “Visitantes de todos os outros países são elegíveis
para entrar na França com teste negativo para Covid-19”, pois visitantes da “lista vermelha” não
podem entrar no país de forma alguma. Isso pode ser confirmado com o trecho “Tourism will not
be possible, however, from countries with virus surges and or prevalent variants. This ‘red list’ for
the moment has 16 countries, including India, South Africa, and Brazil”.
A alternativa D está incorreta. A pergunta “Além da Europa, como está a situação da pandemia
no resto do mundo?” não é bem respondida com a situação de pandemia no resto do mundo
ainda é muito grave, por isso as fronteiras francesas permanecem completamente fechadas, pois
visitantes de países “laranja” podem entrar no país, se estiverem vacinados e apresentarem teste
negativo. Isso pode ser confirmado com o trecho “Vaccinated visitors from ‘orange’ countries —
including the US and Britain — will no longer need to quarantine on arrival and will no longer have
to justify the reasons for their trip to France. They will, however, still be asked for a negative PCR
test no older than 72 hours or a negative antigenic test of no more than 48 hours”.
GABARITO: B

Questão 06 (EPCAR/INÉDITA) – According to the fourth paragraph, vaccinated visitors from


“orange” countries
A) Will face fewer barriers to enter France
B) Never needed quarantine on arrival or justify going to France
C) Will not need to present anything other than a passport to enter France
D) Are not American or British, for example
Comentários: A alternativa A está correta. De acordo com o quarto parágrafo, é correto afirmar
que os visitantes vacinados de países “laranja” terão de enfrentar menos barreiras para entrar na
França, assim como esta opção indica. Isso pode ser confirmado com o trecho “...will no longer
need to quarantine on arrival and will no longer have to justify the reasons for their trip to France”.
A alternativa B está incorreta. De acordo com o quarto parágrafo, não é correto afirmar que os
visitantes vacinados de países “laranja” nunca precisaram de quarentena na chegada ou justifique
a ida para a França, mas sim, que precisavam, mas não precisam mais. Isso pode ser confirmado
com o trecho “...will no longer need to quarantine on arrival and will no longer have to justify the
reasons for their trip to France”.
A alternativa C está incorreta. De acordo com o quarto parágrafo, não é correto afirmar que os
visitantes vacinados de países “laranja” não precisarão apresentar nada além de um passaporte
para entrar na França, mas sim, que deverão apresentar testes de PCR ou de antígeno negativos
para entrar. Isso pode ser confirmado com o trecho “They will, however, still be asked for a
negative PCR test no older than 72 hours or a negative antigenic test of no more than 48 hours”.
A alternativa D está incorreta. De acordo com o quarto parágrafo, não é correto afirmar que os
visitantes vacinados de países “laranja” não são americanos ou britânicos, por exemplo, mas sim,
que podem ser. Isso pode ser confirmado com o trecho “Vaccinated visitors from ‘orange’
countries — including the US and Britain…”.
GABARITO: A

AULA 06 – CONJUNCTIONS, DIRECT SPEECH AND REPORTED SPEECH 84


TEACHER ANDREA BELO

Questão 07 (EPCAR/INÉDITA) – Mark the correct option


A) Only vaccinated children can enter France
B) There is no information about the entry of children into France
C) Unvaccinated children will not be allowed into France under any circumstances
D) Vaccinated adults can enter with children, even if they are not vaccinated
Comentário: A alternativa A está incorreta. Não é correto afirmar que apenas crianças vacinadas
podem entrar na França, mas sim, que crianças não vacinadas poderão entrar se estiverem com
adultos vacinados. Isso pode ser confirmado com o trecho “Unvaccinated children will be allowed
in with vaccinated adults”.
A alternativa B está incorreta. Não é correto afirmar que não há informações sobre a entrada de
crianças na França, mas sim, que crianças não vacinadas poderão entrar se estiverem com adultos
vacinados. Isso pode ser confirmado com o trecho “Unvaccinated children will be allowed in with
vaccinated adults”.
A alternativa C está incorreta. Não é correto afirmar que crianças não vacinadas não serão
permitidas na França em nenhuma circunstância, mas sim, que crianças não vacinadas poderão
entrar se estiverem com adultos vacinados. Isso pode ser confirmado com o trecho “Unvaccinated
children will be allowed in with vaccinated adults”.
A alternativa D está correta. É correto afirmar que crianças não vacinadas poderão entrar se
estiverem com adultos vacinados, assim como esta opção indica. Isso pode ser confirmado com o
trecho “Unvaccinated children will be allowed in with vaccinated adults”.
GABARITO: D

Questão 08 (EPCAR/INÉDITA) – Considering the use of verb tenses, mark the alternative that
completes the sentence below correctly. France
A) Was facilitating the entry of certain tourists
B) Facilitated the entry of certain tourists
C) Will facilitate the entry of certain tourists
D) Facilitates the entry of certain tourists
Comentários: A alternativa A está incorreta. A frase dada não pede um verbo no past continuous
(was facilitating – estava facilitando), mas sim, um verbo no simple future (will facilitate – facilitar).
Pois o texto se refere a regras que a França irá implementar, em breve. Isso pode ser confirmado
com o trecho “Associated Press report that the relaxed rules will kick in from Wednesday”.
A alternativa B está incorreta. A frase dada não pede um verbo no simple past (facilitated –
facilitou), mas sim, um verbo no simple future (will facilitate – facilitar). Pois o texto se refere a
regras que a França irá implementar, em breve. Isso pode ser confirmado com o trecho
“Associated Press report that the relaxed rules will kick in from Wednesday”.
A alternativa C está correta. A frase dada pede um verbo no simple future (will facilitate – facilitar),
assim como esta opção indica. Pois o texto se refere a regras que a França irá implementar, em

AULA 06 – CONJUNCTIONS, DIRECT SPEECH AND REPORTED SPEECH 85


TEACHER ANDREA BELO

breve. Isso pode ser confirmado com o trecho “Associated Press report that the relaxed rules will
kick in from Wednesday”.
A alternativa D está incorreta. A frase dada não pede um verbo no simple present (facilitates –
facilita), mas sim, um verbo no simple future (will facilitate – facilitar). Pois o texto se refere a
regras que a França irá implementar, em breve. Isso pode ser confirmado com o trecho
“Associated Press report that the relaxed rules will kick in from Wednesday”.
GABARITO: C

Questão 09 (EPCAR/INÉDITA) – We can deduce from the sixth paragraph that


A) Visitors from countries classified as "green" will not need to be vaccinated to enter France
B) It is not just Europeans who will be able to enter France if they are vaccinated
C) Australia and Japan are examples of countries on the "red list"
D) Not all Europeans will be able to enter France without testing negative, even if they are
vaccinated
Comentários: A alternativa A está incorreta. De acordo com o sexto parágrafo, não é correto
afirmar que visitantes de países classificados como "verdes" não precisarão ser vacinados para
entrar na França, mas sim, que precisarão estar vacinados. Isso pode ser confirmado com o trecho
“European visitors and those from seven countries … will no longer need to undergo testing if they
are vaccinated”.
A alternativa B está correta. De acordo com o sexto parágrafo, é correto afirmar que não são
apenas os europeus que poderão entrar na França se forem vacinados, assim como esta opção
indica. Isso pode ser confirmado com o trecho “European visitors and those from seven countries
classed as ‘green’ … will no longer need to undergo testing if they are vaccinated”.
A alternativa C está incorreta. De acordo com o sexto parágrafo, não é correto afirmar que
Austrália e Japão são exemplos de países na "lista vermelha", mas sim, que estes são países
classificados como “verde”. Isso pode ser confirmado com o trecho “European visitors and those
from seven countries classed as ‘green’ — Australia, South Korea, Israel, Japan...”.
A alternativa D está incorreta. De acordo com o sexto parágrafo, não é correto afirmar que nem
todos os europeus poderão entrar na França sem teste negativo, mesmo se estiverem vacinados,
mas sim, que todos os europeus vacinados poderão entrar sem apresentar teste negativo. Isso
pode ser confirmado com o trecho “European visitors and those from...”.
GABARITO: B

Questão 10 (EPCAR/INÉDITA) – The word “they” (paragraph 4) refers to


A) Vaccinated visitors from “orange” countries
B) “Orange” countries
C) US and Britain
D) PCR test and antigenic test

AULA 06 – CONJUNCTIONS, DIRECT SPEECH AND REPORTED SPEECH 86


TEACHER ANDREA BELO

Comentário: A alternativa A está correta. A palavra “they” (eles) se refere aos visitantes vacinados
de países “laranja”, assim como esta opção indica. Isso pode ser confirmado com o trecho
“Vaccinated visitors from “orange” countries … and will no longer have to justify the reasons for
their trip to France. They will…”.
A alternativa B está incorreta. A palavra “they” (eles) não se refere aos países “laranja”, mas sim,
aos visitantes vacinados de países “laranja”. Isso pode ser confirmado com o trecho “Vaccinated
visitors from “orange” countries … and will no longer have to justify the reasons for their trip to
France. They will…”.
A alternativa C está incorreta. A palavra “they” (eles) não se refere aos EUA e Grã-Bretanha, mas
sim, aos visitantes vacinados de países “laranja”. Isso pode ser confirmado com o trecho
“Vaccinated visitors from “orange” countries … and will no longer have to justify the reasons for
their trip to France. They will…”.
A alternativa D está incorreta. A palavra “they” (eles) não se refere ao teste de PCR e ao teste de
antígeno, mas sim, aos visitantes vacinados de países “laranja”. Isso pode ser confirmado com o
trecho “Vaccinated visitors from “orange” countries … and will no longer have to justify the
reasons for their trip to France. They will…”.
GABARITO: A

QUESTÕES ESA
Read the text and answer questions 01, 02 and 03
If you bought a dog during lockdown, they'll need help coming out of it
Behavioural problems are a common reason for rehoming, so prepare them for when you’re
not there 24/7
Despite the lows of the past year, for many lockdown has been the perfect opportunity to
welcome a dog into their lives. The demand for dogs during the pandemic has been huge, with a
60% increase in calls from people seeking to adopt from the Dogs Trust charity, and with many
other rescue organisations reporting similar findings. Google searches for “buy a puppy”
increased by 115% after the UK first went into lockdown in March 2020, with prices for some of
the most soughtafter breeds reaching record levels.
Dog ownership is a wonderful thing, but it is also a huge responsibility and a commitment that
spans way beyond lockdown: as the saying goes, “A dog is for life”. As restrictions ease and the
resumption of normality begins, it’s important we consider the implications for our canine
companions and give them a hand to help them adjust.
Having a dog around has helped many people cope with lockdown. Our dogs mostly love us being
around too: going for longer walks, having more playtime, and resting by our side. Nevertheless,
it’s safe to say life has not been normal for our dogs for most of the past year. Few have met other
dogs, and if they have seen them, it would have been from afar or on a lead, meaning that they
were unable to interact or play. There have also been fewer visitors coming into the home, but
probably more deliveries, with people coming to the door carrying parcels and going away again.
This is all particularly concerning for puppies acquired during the pandemic, as their expectation

AULA 06 – CONJUNCTIONS, DIRECT SPEECH AND REPORTED SPEECH 87


TEACHER ANDREA BELO

of “normal” is lockdown life, and they may never have seen visitors inside the house or have been
left home alone.
We are all longing for a great British summer in which we can go on dog-date walks with a friend
and their dog, have family round for garden barbecues and take our pooches to the pub or cafe,
and of course, we need our dogs to be able to cope calmly with all of that. A return to normal is
something humans are able to process, understand and prepare for. But our dogs – especially
young ones – won’t understand why everything has changed. As far as our dogs know, normality
for them has been enjoying time with family only – so to be expected to cope with groups of
people, children and other dogs, both in and out of the home, could be overwhelming for them.
(Adapted from https://www.theguardian.com/commentisfree/2021/apr/05/bought-dog-lockdown)

Questão 01 (ESA/INÉDITA) – Concerning the information in the text, is correct to state that
(A) Some problems about the dog's behavior, most of the time, have nothing to do with
relocating the animal
(B) Last year's context ended up making it difficult to adopt animals
(C) Despite the high adoption of dogs taking place during the lockdown period, the responsibility
for them goes beyond this context
(D) Although many dogs know only the context of the pandemic, they will be able to adapt to
other contexts with ease
(E) The cooperative relationship between owner and dog is not mutual in a pandemic context
Comentários: A alternativa A está incorreta. De acordo com o texto, não é correto afirmar que
alguns problemas com o comportamento do cão, na maioria das vezes, não têm nada a ver com
a realocação do animal, mas sim, que problemas com comportamento são comumente
relacionados à realocação do animal. Isso pode ser confirmado com o trecho “Behavioural
problems are a common reason for rehoming…”.
A alternativa B está incorreta. De acordo com o texto, não é correto afirmar que o contexto do
ano passado acabou dificultando a adoção de animais, mas sim, que acabou sendo o período
perfeito para a adoção de animais, para algumas pessoas. Isso pode ser confirmado com o trecho
“Despite the lows of the past year, for many lockdown has been the perfect opportunity to
welcome a dog into their lives”.
A alternativa C está correta. De acordo com o texto, é correto afirmar que apesar da alta na
adoção de cães ocorrendo durante o período de lockdown, a responsabilidade por eles vai além
deste contexto, assim como esta opção indica. Isso pode ser confirmado com o trecho “Dog
ownership is a wonderful thing, but it is also a huge responsibility and a commitment that spans
way beyond lockdown”.
A alternativa D está incorreta. De acordo com o texto, não é correto afirmar que embora muitos
cães conheçam apenas o contexto da pandemia, eles serão capazes de se adaptar a outros
contextos com facilidade, mas sim, que eles devem ser ajudados pelos donos. Isso pode ser
confirmado com o trecho “As restrictions ease and the resumption of normality begins, it’s

AULA 06 – CONJUNCTIONS, DIRECT SPEECH AND REPORTED SPEECH 88


TEACHER ANDREA BELO

important we consider the implications for our canine companions and give them a hand to help
them adjust”.
A alternativa E está incorreta. De acordo com o texto, não é correto afirmar que a relação de
cooperação entre o dono e o cão não é mútua em um contexto de pandemia, mas sim, que acaba
sendo mútua. Isso pode ser confirmado com o trecho “Having a dog around has helped many
people cope with lockdown. Our dogs mostly love us being around too…”.
GABARITO: C

Questão 02 (ESA/INÉDITA) – In the sentence “As far as our dogs know, normality for them has
been enjoying time with family only – so to be expected to cope with groups of people, children
and other dogs, both in and out of the home, could be overwhelming for them” (paragraph 4),
the word “overwhelming” means
(A) Crushing
(B) Amazing
(C) Comfortable
(D) Awkward
(E) Indifferent
Comentários: A alternativa A está correta. A palavra “overwhelming” significa
opressor/desgastante e pode ser comparada com a palavra “crushing”, que significa
esmagador/desgastante.
A alternativa B está incorreta. A palavra “overwhelming” significa opressor/desgastante e não
pode ser comparada com a palavra “amazing”, que significa incrível.
A alternativa C está incorreta. A palavra “overwhelming” significa opressor/desgastante e não
pode ser comparada com a palavra “comfortable”, que significa confortável.
A alternativa D está incorreta. A palavra “overwhelming” significa opressor/desgastante e não
pode ser comparada com a palavra “awkward”, que significa estranho.
A alternativa E está incorreta. A palavra “overwhelming” significa opressor/desgastante e não
pode ser comparada com a palavra “indifferent”, que significa indiferente.
GABARITO: A

Questão 03 (ESA/INÉDITA) – The words COPE and GO are


(A) Regular verbs
(B) Irregular verbs
(C) Not verbs
(D) Verbs
(E) From different grammatical classes

AULA 06 – CONJUNCTIONS, DIRECT SPEECH AND REPORTED SPEECH 89


TEACHER ANDREA BELO

Comentários: A alternativa A está incorreta. As palavras “cope” (lidar) e “go” (ir) são verbos;
porém, “cope” é um verbo regular pois utiliza a terminação -ed no seu modo passado, enquanto
“go” é um verbo irregular por ter um modo passado diferente, “went” (fui/foi).
A alternativa B está incorreta. As palavras “cope” (lidar) e “go” (ir) são verbos; porém, “cope” é
um verbo regular pois utiliza a terminação -ed no seu modo passado, enquanto “go” é um verbo
irregular por ter um modo passado diferente, “went” (fui/foi).
A alternativa C está incorreta. As palavras “cope” (lidar) e “go” (ir) são verbos; “cope” é um verbo
regular pois utiliza a terminação -ed no seu modo passado, enquanto “go” é um verbo irregular
por ter um modo passado diferente, “went” (fui/foi).
A alternativa D está correta. As palavras “cope” (lidar) e “go” (ir) são verbos; “cope” é um verbo
regular pois utiliza a terminação -ed no seu modo passado, enquanto “go” é um verbo irregular
por ter um modo passado diferente, “went” (fui/foi), assim como esta opção indica.
A alternativa E está incorreta. As palavras “cope” (lidar) e “go” (ir) são, ambos, verbos; porém,
“cope” é um verbo regular pois utiliza a terminação -ed no seu modo passado, enquanto “go” é
um verbo irregular por ter um modo passado diferente, “went” (fui/foi).
GABARITO: D

Read the text and answer questions 04 and 05


Is the U.S. COVID-19 Vaccine Rollout Working? We're Not Sure Yet
After four months and 171 million doses of COVID-19 vaccines administered across the U.S., more
than a few of us are eager to know: are the shots working?
Thus far, available evidence can half-answer that question: The vaccines are working well for
those who can get them. As soon as the rollout kicked off, a variety of researchers began
conducting what might be considered __________ (1) unofficial phase 4 clinical trial, monitoring
early vaccine recipients among the general public. Several of their studies have found a significant
reduction in both the rates of infection and severity of symptoms among vaccinated populations
compared __________ (2) similar demographics who are still awaiting a dose. On an individual
level, the vaccines appear to be working as well in real life as they did in controlled trials.
But there’s another part of that question: will mass vaccination hasten the end of __________ (3)
pandemic? On first pass, this might seem to be happening. As of April 7, nearly 20% of Americans
have received a completed regimen of either the two-dose Pfizer-BioNTech and Moderna
vaccines, or the one-shot Janssen variety. Meanwhile, the daily number of deaths in the U.S.
attributed to COVID-19, on a rolling seven-day average, is currently 611—a ghastly figure, but a
tremendous drop from the all-time post-holiday peak of 3,428 on Jan. 11. The last time the figure
was this low was on Jul. 4, 2020, just before deaths climbed again in the wake of a second wave
over the summer, from which the country never fully recovered before the much more
catastrophic third wave spurred by the holidays.
(Adapted from https://time.com/5953007/covid-19-mass-vaccination/)

AULA 06 – CONJUNCTIONS, DIRECT SPEECH AND REPORTED SPEECH 90


TEACHER ANDREA BELO

Questão 04 (ESA/INÉDITA) – Concerning the information in the text, is correct to say that
(A) All the available evidence is able to show in the best way whether the vaccines are working
or not
(B) So far, vaccines have shown better results in trials than in actual individual vaccination
(C) Mass vaccination will certainly anticipate an end to the pandemic
(D) Most Americans have already received two doses of the vaccine against COVID-19
(E) It is possible to affirm that the vaccination is working, at least, partially
Comentários: A alternativa A está incorreta. De acordo com o texto, não é correto afirmar que
todas as evidências disponíveis são capazes de mostrar da melhor forma se as vacinas estão
funcionando ou não, mas sim, que as evidências só conseguem mostrar se as vacinas estão
funcionando para quem já as tomou. Isso pode ser confirmado com o trecho “Thus far, available
evidence can half-answer that question: The vaccines are working well for those who can get
them”.
A alternativa B está incorreta. De acordo com o texto, não é correto afirmar que até agora, as
vacinas têm mostrado melhores resultados em ensaios do que na vacinação individual real, mas
sim, que as vacinas parecem estar funcionando na vida real tão bem quanto funcionaram nos
testes. Isso pode ser confirmado com o trecho “On an individual level, the vaccines appear to be
working as well in real life as they did in controlled trials”.
A alternativa C está incorreta. De acordo com o texto, não é correto afirmar que a vacinação em
massa certamente antecipará o fim da pandemia, mas sim, que isso parece estar acontecendo,
mas não se pode afirmar. Isso pode ser confirmado com o trecho “...will mass vaccination hasten
the end of __________ (3) pandemic? On first pass, this might seem to be happening”.
A alternativa D está incorreta. De acordo com o texto, não é correto afirmar que a maioria dos
americanos já recebeu duas doses da vacina contra COVID-19, mas sim, que 20% dos americanos
tomaram as doses suficientes para imunizá-los completamente. Isso pode ser confirmado com o
trecho “...nearly 20% of Americans have received a completed regimen of either the two-dose
Pfizer-BioNTech and Moderna vaccines, or the one-shot Janssen variety”.
A alternativa E está correta. De acordo com o texto, é correto afirmar que a vacinação está
funcionando, pelo menos, parcialmente, assim como esta opção indica. Isso pode ser confirmado
com o trecho “Thus far, available evidence can halfanswer that question: The vaccines are working
well for those who can get them…”.
GABARITO: E

Questão 05 (ESA/INÉDITA) – In the sentence “Several of their studies have found a significant
reduction in both the rates of infection and severity of symptoms among vaccinated populations
compared…” (paragraph 2), the word “several” means
(A) Many
(B) None
(C) A few
(D) Some
(E) A part

AULA 06 – CONJUNCTIONS, DIRECT SPEECH AND REPORTED SPEECH 91


TEACHER ANDREA BELO

Comentários: A alternativa A está correta. A palavra “several”, nesse caso, significa vários, e
pode ser comparada com a palavra “many”, que significa muitos.
A alternativa B está incorreta. A palavra “several”, nesse caso, significa vários, e não pode ser
comparada com a palavra “none”, que significa nenhum.
A alternativa C está incorreta. A palavra “several”, nesse caso, significa vários, e não pode ser
comparada com a palavra “a few”, que significa alguns.
A alternativa D está incorreta. A palavra “several”, nesse caso, significa vários, e não pode ser
comparada com a palavra “some”, que significa alguns.
A alternativa E está incorreta. A palavra “several”, nesse caso, significa vários, e não pode ser
comparada com a palavra “a part”, que significa uma parte, por não especificar muito bem a
quantidade.
GABARITO: A

QUESTÕES ESCOLA NAVAL


Based on the text below, answer the questions that follow it. The paragraphs of the text are
numbered.
The machine always wins: what drives our addiction to social media
[1] We are swimming in writing. Our lives have become, in the words of the author and academic
Shoshana Zuboff, an “electronic text”. Social media platforms have created a machine for us to
write to. The bait is that we are interacting with other people: our friends, colleagues, celebrities,
politicians, royals, terrorists, porn actors – anyone we like. We are not interacting with them,
however, but with the machine. We write to it, and it passes on the message for us after keeping
a record of the data.
[2] The machine benefits from the “network effect”: the more people write to it, the more benefits
it can offer, until it becomes a disadvantage not to be part of it. Part of what? The world’s first ever
public, live, collective, open-ended writing project. A virtual laboratory. An addiction machine,
which deploys crude techniques of manipulation redolent of the Skinner Box created by
behaviourist BF Skinner to control the behaviour of pigeons and rats with rewards and
punishments. We are users, much as cocaine addicts are users.
[3] What is the incentive to engage in writing like this for hours each day? In a form of mass
casualisation, writers no longer expect to be paid or given employment contracts. What do the
platforms offer us, in lieu of a wage? What gets us hooked? Approval, attention, retweets, shares
and likes.
[4] Meanwhile, hashtags and trending topics underline the extent to which all of these protocols
are organized around the massification of individual voices – a phenomenon cheerfully described
by users with the science-fiction concept of the “hive mind” – and hype. The regular sweet spot
sought after is a brief period of ecstatic collective frenzy around any given topic. It doesn’t
particularly matter to the platforms what the frenzy is about: the point is to generate data, one of
the most profitable raw materials yet discovered. As in the financial markets, volatility adds value.
The more chaos, the better.

AULA 06 – CONJUNCTIONS, DIRECT SPEECH AND REPORTED SPEECH 92


TEACHER ANDREA BELO

[5] Whether or not we think we are addicted, the machine treats us as addicts. Addiction is, quite
deliberately, the template for our relationship to the Twittering Machine. Addiction is all about
attention. For the social media bosses, this is axiomatic.
[6] If social media is an addiction machine, the addictive behaviour it is closest to is gambling: a
rigged lottery. Every gambler trusts in a few abstract symbols – the dots on a dice, numerals, suits,
red or black, the graphemes on a fruit machine – to tell them who they are. In most cases, the
answer is brutal and swift: you are a loser and you are going home with nothing. The true gambler
takes a perverse joy in anteing up, putting their whole being at stake. On social media, you scratch
out a few words, a few symbols, and press send, rolling the dice. The internet will tell you who you
are and what your destiny is through arithmetic likes, shares and comments.
[7] The interesting question is what it is that is so addictive. In principle, anyone can win big; in
practice, not everyone is playing with the same odds. Our social media accounts are set up like
enterprises competing for attention. If we are all authors now, we write not for money, but for the
satisfaction of being read. Going viral, or trending, is the equivalent of a windfall. But sometimes,
winning is the worst thing that can happen. The temperate climate of likes and approval is apt to
break, lightning-quick, into sudden storms of fury and disapproval.
(Adapted from https://www.theguardian.com/technology/2019/aug/23/social-media-addiction-gambling)

Questão 01 (ESCOLA NAVAL/INÉDITA) – According to the first paragraph, which option is


correct?
(A) people should avoid interacting on social media platforms.
(B) when people communicate through social media, they make real friends.
(C) lives nowadays are more based on personal contact.
(D) social media platforms allow people to truly interact with other people.
(E) when people communicate through social media, they, as a matter of fact, interact with the
machine.
Comentários: Essa questão quer que você identifique o que é correto afirmar de acordo com o
primeiro parágrafo, então, foco total nele! Vamos às alternativas:
a) Muito cuidado com alternativas como essa, que extrapolam o que o texto diz. Não há
diretamente, no primeiro parágrafo, uma opinião no sentido de que as pessoas deveriam evitar
interagir nas plataformas de redes sociais. O parágrafo traz fatos sobre essa interação, como, por
exemplo, o fato de que não é uma interação real. Alternativa INCORRETA.
b) Não é correto, conforme o texto, dizer que quando as pessoas se comunicam pelas redes sociais,
elas fazem amigos de verdade. INCORRETA.
c) Conforme a alternativa C, a vida das pessoas hoje em dia (nowadays) se baseia mais num
contato pessoal. Observe que o primeiro parágrafo traz uma afirmação distinta, ao asseverar que
a vida das pessoas virou uma “mensagem de texto”: → “Our lives have become... an ‘electronic
text’.” Alternativa INCORRETA.
d) De acordo com essa alternativa, as redes sociais permitem (allow) que as pessoas se conectem
verdadeiramente com as outras. Na verdade, segundo o primeiro parágrafo, as pessoas não estão,

AULA 06 – CONJUNCTIONS, DIRECT SPEECH AND REPORTED SPEECH 93


TEACHER ANDREA BELO

na realidade, interagindo umas com as outras, mas com a máquina. → “We are not interacting
with them, however, but with the machine.” Alternativa INCORRETA.
e) Segundo a letra E, quando as pessoas se comunicam através das redes sociais, elas, na verdade
(as a matter of fact), interagem com a máquina, o que está totalmente de acordo com o trecho do
primeiro parágrafo que vimos acima, na correção da letra A. Alternativa CORRETA.
GABARITO: E

Questão 02 (ESCOLA NAVAL/INÉDITA) – Which word can replace “however” in paragraph 1 with
no change in meaning?
(A) though
(B) since
(C) then
(D) besides
(E) therefore
Comentários: Nessa questão, você deve se lembrar que as linking words, as conjunções do Inglês,
servem para estabelecer uma relação entre as ideias expressadas em uma frase. However significa
contudo, entretanto, e é um exemplo de conjunção adversativa. Ele estabelece, no trecho, uma
relação de oposição, contraste entre as ideias. Você deveria identificar nas alternativas qual outra
conjunção poderia substituir however sem alteração de sentido. Vamos às alternativas:
a) Though significa embora, contudo. Dessa forma, estabelece oposição de ideias como however,
podendo substituí-lo sem alteração de sentido. Alternativa CORRETA.
b) Since pode indicar marco temporal (desde) ou introduzir uma explicação (já que, como).
Alternativa INCORRETA.
c) Then significa então, indicando uma conclusão. Alternativa INCORRETA.
d) Besides significa além disso, acrescentando ideias. Assim, não estabelece oposição de ideias
como however. Alternativa INCORRETA.
e) Therefore significa portanto, indicando uma conclusão. Alternativa INCORRETA.
GABARITO: A

Questão 03 (ESCOLA NAVAL/INÉDITA) – Which option shows the main message of the text?
(A) everybody can go viral on social platforms.
(B) social media can be as addictive as drugs.
(C) financial feedback is the major reason why people engage in social media writing.
(D) there’s evidence cocaine is more addictive than social media and gambling.
(E) anyone can be successful when it comes to social media.

AULA 06 – CONJUNCTIONS, DIRECT SPEECH AND REPORTED SPEECH 94


TEACHER ANDREA BELO

Comentários: a) De acordo com a letra A, todo mundo pode viralizar em plataformas sociais, o
que não está de acordo com o texto. INCORRETA.
b) De acordo com essa letra, a principal mensagem do texto é mostrar que as mídias sociais podem
ser tão viciantes quanto as drogas, o que está totalmente de acordo com o texto. Observe a
expressão que o texto usa para se referir às redes sociais: an addiction machine (uma máquina
viciante). Como vimos, o texto afirma que os usuários de redes sociais são tão usuários quanto
viciados em cocaína e têm um padrão de comportamento semelhante ao dos que se viciam em
jogos de azar. Como a afirmação está em harmonia com o texto, é esse o gabarito. Alternativa
CORRETA.
c) Conforme essa alternativa, o retorno financeiro é a principal razão pela qual as pessoas se
comprometem a escrever em redes sociais. Essa informação não está de acordo com o texto. →
“In a form of mass casualisation, writers no longer expect to be paid or given employment
contracts …” Em uma forma de casualização em massa, os escritores não esperam mais ser pagos
ou receber contratos de trabalho… (3º parágrafo). Alternativa INCORRETA.
d) De acordo com a letra D, o texto busca mostrar que há evidências de que a cocaína é mais
viciante do que as mídias sociais e os jogos de azar. Se voltarmos ao texto, veremos, que, na
verdade, ele equipara o vício em redes sociais aos demais vícios, não afirmando que a cocaína,
por exemplo, é mais viciante. → “We are users, much as cocaine addicts are users”. Somos
usuários, assim como os viciados em cocaína são usuários (2º parágrafo). Alternativa INCORRETA.
e) Segundo a letra E, o texto busca mostrar que qualquer pessoa pode ser bem-sucedida quando
se trata de redes sociais. A afirmação não está de acordo com o texto, como se depreende do
seguinte trecho → “In principle, anyone can win big; in practice, not everyone is playing with the
same odds.” Em princípio, qualquer um pode ter muito sucesso; na prática, nem todo mundo está
jogando com as mesmas chances. (7º parágrafo). Alternativa INCORRETA.
GABARITO: B

Questão 04 (ESCOLA NAVAL/INÉDITA) – According to the text, which option is incorrect?


(A) in practice, not everyone has the same chance of being successful online.
(B) every gambler trusts in a few abstract symbols, such as the dots on a dice.
(C) social media platforms have created a machine that treats people as addicts.
(D) there are no explanations on why people engage in writing online for hours each day.
(E) addiction to social media can be compared to gambling addiction.
Comentários: Muita atenção, pois a questão pede que você identifique a alternativa incorreta de
acordo com o texto. Assim, o nosso gabarito será a afirmação falsa.
a) Para essa alternativa, na prática, nem todo mundo tem a mesma chance de ser bem-sucedido
na internet. Essa informação está de acordo com o texto. → “In principle, anyone can win big; in
practice, not everyone is playing with the same odds.” Em princípio, qualquer um pode ter muito
sucesso; na prática, nem todo mundo está jogando com as mesmas chances. (7º parágrafo). Como
estamos em busca da incorreta, não é esse o nosso gabarito. Alternativa INCORRETA.

AULA 06 – CONJUNCTIONS, DIRECT SPEECH AND REPORTED SPEECH 95


TEACHER ANDREA BELO

b) Essa alternativa afirma que todo jogador confia em alguns símbolos abstratos, como os pontos
em um dado, o que está de acordo com o texto. “Every gambler trusts in a few abstract symbols –
the dots on a dice, numerals, suits, red or black, the graphemes on a fruit machine…” Como
estamos em busca da incorreta, não é esse o nosso gabarito. Alternativa INCORRETA.
c) Conforme essa alternativa, as plataformas de mídias sociais criaram uma máquina que trata as
pessoas como viciados. → “Whether or not we think we are addicted, the machine treats us as
addicts.” Independente se nós achamos ou não que somos viciados, a máquina nos trata como
tal. (5º parágrafo). Essa informação está de acordo com o texto. Como estamos em busca da
incorreta, não é o nosso gabarito. Alternativa INCORRETA.
d) Segundo essa letra, não há explicações sobre porque as pessoas escrevem on-line por horas
todos os dias. Essa informação NÃO está de acordo com o texto. → “What is the incentive to
engage in writing like this for hours each day?... What gets us hooked? Approval, attention,
retweets, shares and likes.” Qual é o incentivo para escrever dessa maneira por horas todos os
dias? ... O que nos deixa viciados? Aprovação, atenção, retweets, compartilhamentos e curtidas.
(3º parágrafo). Como estamos em busca da incorreta, é esse o nosso gabarito. Alternativa
CORRETA.
e) De acordo com a letra E, o vício em mídias sociais (addiction=vício) pode ser comparado ao vício
em jogos de azar. → “If social media is an addiction machine, the addictive behaviour it is closest
to is gambling...” Se a mídia social é uma máquina de dependência, o comportamento viciante
mais próximo é o jogo. (6º parágrafo). Essa informação está de acordo com o texto. Como estamos
em busca da incorreta, não é esse o nosso gabarito. Alternativa INCORRETA.
GABARITO: D

Questão 05 (ESCOLA NAVAL/INÉDITA) – The word “meanwhile” in paragraph 4 indicates that


(A) some facts happen at the same time.
(B) one fact is more important than others.
(C) a single fact interferes in others.
(D) some facts happen at different times.
(E) some facts happen for no reason.
Comentários: “Meanwhile” é um conectivo que transmite a ideia de “enquanto isso”, indicando
que algo acontece ao mesmo tempo que outro fato. No texto, temos que “enquanto isso, hashtags
e trending topics destacam até que ponto todos esses protocolos estão organizados em torno da
massificação de vozes individuais...”.
a) Para a letra A, o conectivo indica que alguns fatos acontecem ao mesmo tempo que outros.
Como vimos, é exatamente essa a ideia transmitida por “meanwhile” (enquanto isso). Alternativa
CORRETA.
b) Conforme a letra B, o conectivo indica que um fato é mais importante que outros. Como vimos,
a noção é temporal (enquanto isso). Alternativa INCORRETA.
c) Segundo a alternativa C, o conectivo indica que um único fato interfere em outros. Como vimos,
a noção é temporal (enquanto isso). Alternativa INCORRETA.

AULA 06 – CONJUNCTIONS, DIRECT SPEECH AND REPORTED SPEECH 96


TEACHER ANDREA BELO

d) De acordo com a letra D, o conectivo transmite a ideia de que alguns fatos acontecem em
momentos diferentes, o que, conforme vimos, não está correto. Alternativa INCORRETA.
e) Para a letra E, o conectivo indica que alguns fatos acontecem sem razão, o que não está correto.
INCORRETA.
GABARITO: A

Questão 06 (ESCOLA NAVAL/INÉDITA) – Which is the best definition for “bait”, in paragraph 1?
(A) something that makes people improve their attitude.
(B) something unattractive that discourages people from acting a certain way.
(C) something attractive that is offered to make people act a certain way.
(D) something that makes people think about their actions.
(E) something attractive that discourages people from acting a certain way.
Comentários: Você, certamente, já ouviu a expressão “clickbait”, que se refere a uma tática
sensacionalista usada na Internet para “fisgar” usuários para um conteúdo online, atraindo-os
para um vídeo, por exemplo, e gerar visualizações, “clicks”. Se você se lembrasse disso ao resolver
essa questão, poderia chegar ao sentido de “bait” no texto, que transmite a ideia de uma isca,
algo que atrai as pessoas e as faz agir de uma determinada maneira. Vamos às alternativas:
a) Para a letra A, “bait” pode ser definido como algo que faz as pessoas melhorarem o seu
comportamento (attitude). Como vimos, a ideia transmitida por “bait”, no contexto, não é essa,
mas sim de algo que atrai as pessoas, “fisgando”-as. Alternativa INCORRETA.
b) Segundo a letra B, “bait” pode ser definido como algo não atraente que desestimula as pessoas
a agirem de uma certa maneira. Como vimos, a ideia transmitida por “bait”, no contexto, é
justamente o contrário disso. Alternativa INCORRETA.
c) De acordo com a letra C, a melhor definição de “bait” é algo atraente que é oferecido para fazer
as pessoas agirem de uma certa maneira. Se voltamos ao texto, vemos que o que é oferecido às
pessoas para atraí-las, “fisgá-las” para as redes sociais, é a ideia de que através das redes elas
podem interagir com qualquer pessoa que quiserem, desde amigos até celebridades e políticos.
Assim, a definição de “bait” está correta. Alternativa CORRETA.
d) Conforme a alternativa D, “bait” pode ser definido como algo que faz as pessoas pensarem
sobre as suas ações. Como vimos, não é essa a ideia transmitida por “bait” no contexto.
Alternativa INCORRETA.
e) De acordo com a letra E, “bait” pode ser definido como algo atraente que desestimula as
pessoas de agir de determinada maneira. Como vimos, não é essa a ideia transmitida por “bait”
no contexto. Alternativa INCORRETA.
GABARITO: C

AULA 06 – CONJUNCTIONS, DIRECT SPEECH AND REPORTED SPEECH 97


TEACHER ANDREA BELO

Questão 07 (ESCOLA NAVAL/INÉDITA) – Which word or expression does the object pronoun
“them”, in paragraph 1, refer to?
(A) internet addicts.
(B) the machine.
(C) platforms.
(D) social media.
(E) other people.
Comentários: Nesse tipo de questão, é muito importante que você volte ao texto para identificar
a que o pronome se refere. Voltando ao primeiro parágrafo, percebemos a que “them” se refere:
→ “The bait is that we are interacting with other people... We are not interacting with them,
however, but with the machine.” O pronome them (eles, elas) está retomando a expressão other
people.
GABARITO: E

Questão 08 (ESCOLA NAVAL/INÉDITA) – In the sentence “We are users, much as cocaine addicts
are users.” (paragraph 2), the author DOESN’T
(A) describe social media addicts in a certain way.
(B) make a comparison.
(C) treat drug users and internet addicts as different.
(D) compare drug users and internet addicts.
(E) bring impacting information about social media addiction.
Comentários: A questão quer que você identifique, dentre as alternativas, o que o autor do texto
NÃO FAZ.
a) De acordo com a letra A, o autor não descreve os viciados em redes sociais em certa medida.
Isso não está correto, pois o autor os descreve, sim, de certa maneira, ao compará-los a usuários
de cocaína. Alternativa INCORRETA.
b) Segundo a letra B, o autor não faz uma comparação. Isso não está correto, pois a estrutura
much as faz justamente uma comparação entre viciados em internet e viciados em cocaína, dando
a ideia de “assim como”. Alternativa INCORRETA.
c) Para a alternativa C, o autor não trata usuários de drogas e viciados em internet como
diferentes. Isso está correto, uma vez que, segundo o trecho trazido pela questão, realmente, eles
estão equiparados. “Somos usuários, assim como os viciados em cocaína são usuários.”
Alternativa CORRETA.
d) Para a alternativa D, o autor não compara usuários de drogas e dependentes de internet, o que,
como vimos acima, não é correto, uma vez que, sim, ele os compara. Alternativa INCORRETA.
e) Conforme a alternativa E, o autor não traz uma informação impactante sobre o vício em redes
sociais. Certamente, afirmar que o vício em internet se compara ao vício em cocaína é impactante.
Alternativa INCORRETA.
GABARITO: C

AULA 06 – CONJUNCTIONS, DIRECT SPEECH AND REPORTED SPEECH 98


TEACHER ANDREA BELO

Questão 09 (ESCOLA NAVAL/INÉDITA) – Approval, attention, retweets, shares and likes are
mentioned in the text as examples of
(A) emergency measures to overcome internet addiction.
(B) reasons why people get addicted to social media.
(C) reason why people do not like social media.
(D) online communication negative impacts.
(E) reasons why people usually avoid social media.
Comentários: Observe que, no texto, approval, attention, retweets, shares e likes são
mencionados como razões pelas quais as pessoas ficam viciadas em redes sociais. → “What is the
incentive to engage in writing like this for hours each day?... What gets us hooked? Approval,
attention, retweets, shares and likes.” Qual é o incentivo para escrever dessa maneira por horas
todos os dias? ... O que nos deixa viciados? Aprovação, atenção, retweets, compartilhamentos e
curtidas. (3º parágrafo). Vejamos as alternativas:
a) Approval, attention, retweets, shares e likes não são mencionados no texto como medidas
emergenciais para superar o vício em internet. Alternativa INCORRETA.
b) De acordo com o que estudamos acima, aprovação, atenção, retweets, compartilhamentos e
curtidas são mencionados no texto justamente como razões pelas quais as pessoas ficam viciadas
em redes sociais. Alternativa CORRETA.
c) Approval, attention, retweets, shares e likes não são mencionados no texto como o motivo pelo
qual as pessoas não gostam de mídias sociais. Alternativa INCORRETA.
d) Como vimos, aprovação, atenção, retweets, compartilhamentos e curtidas não são
mencionados no texto como impactos negativos da comunicação online, mas sim como razões
pelas quais as pessoas ficam viciadas em redes sociais. Alternativa INCORRETA.
e) Como vimos, approval, attention, retweets, shares e likes não são mencionados no texto como
razões pelas quais as pessoas evitam (avoid) as redes sociais, sendo justamente o motivo pelos
quais elas buscam as redes e se tornam viciadas. Alternativa INCORRETA.
Gabarito: B

Questão 10 (ESCOLA NAVAL/INÉDITA) – Which is the best option to change the sentence “Social
media platforms have created a machine [...]” (paragraph 1) into the passive form?
A machine _______________ by social media platforms.
(A) has created.
(B) had created.
(C) had been created.
(D) has been created.
(E) have been created.

AULA 06 – CONJUNCTIONS, DIRECT SPEECH AND REPORTED SPEECH 99


TEACHER ANDREA BELO

Comentários: Nessa questão, é importante que você se lembre da voz passiva. Resumidamente,
a voz passiva em Inglês é usada quando, ao transmitirmos a mensagem, queremos colocar o
foco no que é feito e não em quem pratica a ação. Quanto à estrutura, vamos sempre precisar
de uma forma do verbo TO BE no tempo verbal que queremos usar e do past participle do verbo
que descreve a ação. Além disso, o objeto vai virar sujeito e o sujeito vai virar o que chamamos,
em Português, de agente da passiva. Na questão, temos uma ação no tempo verbal Present
Perfect- have created. Para transformar para a voz passiva, vamos usar o verbo TO BE nesse
tempo verbal (has been) e o partícipio do verbo create (created). Social media platforms have
created a machine. = A machine has been created by social media platforms. Observe que foi
necessário usar o auxiliar has, pois, na passiva, o sujeito passou a ser a machine.
GABARITO: D

QUESTÕES EsPCEx
Leia o texto a seguir e responda às questões 01, 02 e 03.
Homelessness in the Living Rooms of the Rich
When photographer Jana Sophia Nolle moved to San Francisco three years ago, she met a lot of
people. Some inhabited multimillion-dollar houses with pristine Victorian architecture and fancy
furniture. Others lived in cardboard boxes.
The contrast between their dwellings plagued Nolle, who hails from Kassel, Germany, where
income inequality is less stark. While San Francisco boasts the highest density of billionaires per
capita in the world, the Bay Area hosts the country’s third-largest population of people
experiencing homelessness. The government shelters only a third of them.
“I’d never seen so many people living on the street in a country as rich as America,” Nolle says. “I
was shocked.” So, when an unhoused man she knew suggested, jokingly, that she invite him into
one of her wealthy friends’ homes, it sparked an idea: What if, instead, she pitched his tent there?
This provocative vision inspired her series Living Room, sending Nolle on a quest to photograph
the scrappy DIY shelters of the poor inside the immaculately styled parlors of the rich. “They’re
implants in rooms where they don’t belong,” she says.
Adapted from https://www.wired.com/story/san-francisco-shelters-living-room/

Questão 01 (EsPCEx/INÉDITA) – In the excerpt “While San Francisco boasts the highest density
of billionaires per capita in the world, the Bay Area hosts the country’s third-largest population
of people experiencing homelessness.” (paragraph 2), the word while is being used to
(A) give an idea of time.
(B) add information.
(C) give an example.
(D) express emphasis.
(E) contrast ideas.

AULA 06 – CONJUNCTIONS, DIRECT SPEECH AND REPORTED SPEECH 100


TEACHER ANDREA BELO

Comentários: No trecho “Enquanto São Francisco ostenta a maior densidade de bilionários per
capita do mundo, a área da baía abriga a terceira maior população de pessoas sem teto do país”.
(parágrafo 2), a palavra while está sendo usada para
a) dar uma ideia de tempo.
b) adicionar informações.
c) dar um exemplo.
d) expressar ênfase.
e) contrastar ideias.
No trecho “While San Francisco boasts the highest density of billionaires per capita in the world,
the Bay Area hosts the country’s third-largest population of people experiencing homelessness.”,
WHILE é um conectivo que estabelece uma relação de contraste entre as orações. O sentido é de
que ENQUANTO São Francisco possui a maior quantidade de bilionários per capita do mundo, a
Bay Area abriga a terceira maior população do país de pessoas sem-teto, restando evidente o
contraste entre esses fatos. Assim, a conjunção desempenha a função de contrastar ideias,
conforme afirma a letra E.
Atenção: essa alternativa pode ser considerada a pegadinha da questão. While pode também
transmitir ideia de tempo, no sentido de enquanto, indicando que uma ação acontece ao mesmo
tempo que outra. Mas não é esse o sentido no trecho trazido pela questão. Alternativa
INCORRETA.
No trecho, while serve para conectar as ideias contrastantes de “maior quantidade de bilionários
per capita do mundo” e “terceira maior população do país de pessoas sem-teto” na mesma cidade.
O conectivo não desempenha a função de acrescentar informações. Alternativa INCORRETA.
Como vimos, while, no trecho, não desempenha a função de exemplificar, mas sim de opor ideias.
Alternativa INCORRETA
A conjunção while não expressa ênfase no trecho, mas uma oposição de ideias. Alternativa
INCORRETA CORRETA, conforme a explicação.
GABARITO: E

Questão 02 (EsPCEx/INÉDITA) – According to the text, read the statements and choose the
correct alternative.
I- the income inequality is less severe in Kassel, Germany.
II- the photographer had never seen so many homeless people in a wealthy country.
III- a San Francisco region is home to one of the largest homeless groups in the United States.
IV- the photographer invited a homeless person to live in the home of one of her wealthy
friends.
V- when the photographer moved to San Francisco, she witnessed a situation of extreme social
inequality.

AULA 06 – CONJUNCTIONS, DIRECT SPEECH AND REPORTED SPEECH 101


TEACHER ANDREA BELO

(A) only I, III and V are correct.


(B) only I, II, III and IV are correct.
(C) only I, II, III and V are correct.
(D) only I, II, and IV are correct.
(E) All of them are correct.
Comentários: I) a desigualdade de renda é menos severa em Kassel, Alemanha – de fato, de
acordo com o texto, a desigualdade de renda é menos severa em Kassel, na Alemanha. (parágrafo
2). “...Kassel, Germany, where income inequality is less stark”. (...Kassel, Alemanha, onde a
desigualdade de renda é menos acentuada). CORRETA.
II) a fotógrafa nunca havia visto tantos moradores de rua em um país rico – De fato, a fotógrafa
nunca havia visto tantas pessoas desabrigadas em um país rico. (parágrafo 3). “I’d never seen so
many people living on the street in a country as rich as America...” (Nunca havia visto tantas
pessoas morando na rua em um país tão rico quanto a América...). CORRETA.
III) uma região de São Francisco é o lar de um dos maiores grupos de sem-teto dos Estados Unidos
– alternativa de acordo com o texto: “Enquanto São Francisco ostenta a maior densidade de
bilionários per capita do mundo, a área da baía abriga a terceira maior população de pessoas sem
teto do país”. (parágrafo 2). CORRETA.
IV) a fotógrafa convidou um morador de rua para morar na casa de um de seus amigos ricos – não
é isso que afirma o texto: “So, when an unhoused man she knew suggested, jokingly, that she
invite him into one of her wealthy friends’ homes, it sparked an idea: What if, instead, she pitched
his tent there?” (parágrafo 3) = Então, quando um homem sem casa que ela conhecia sugeriu, de
brincadeira, que ela o convidasse para a casa de um de seus amigos ricos, ela teve uma ideia: e
se, em vez disso, ela montasse a barraca dele lá? INCORRETA.
V) quando a fotógrafa se mudou para São Francisco, ela presenciou uma situação de extrema
desigualdade social – alternativa de acordo com o texto: “When photographer Jana Sophia Nolle
moved to San Francisco three years ago, she met a lot of people. Some inhabited multimillion-
dollar houses with pristine Victorian architecture and fancy furniture. Others lived in cardboard
boxes.” (parágrafo 1) = Quando a fotógrafa Jana Sophia Nolle se mudou para São Francisco há três
anos, ela conheceu muitas pessoas. Alguns habitavam casas multimilionárias com arquitetura
vitoriana imaculada e móveis elegantes. Outros viviam em caixas de papelão. CORRETA.
c) only I, II, III and V are correct.
GABARITO: C

Questão 03 (EsPCEx/INÉDITA) – Which definition below corresponds to dwellings in the


sentence “The contrast between their dwellings plagued Nolle, who hails from Kassel, Germany,
where income inequality is less stark.” (paragraph 2)?
(A) living rooms.
(B) cardboard boxes.
(C) fancy furniture.
(D) inhabited houses.
(E) places of residence.

AULA 06 – CONJUNCTIONS, DIRECT SPEECH AND REPORTED SPEECH 102


TEACHER ANDREA BELO

Comentários: Qual definição abaixo corresponde a dwellings na frase “O contraste entre as suas
moradias (dwellings = moradias) atormentou Nolle, que vem de Kassel, Alemanha, onde a
desigualdade de renda é menos acentuada.” (parágrafo 2)?
a) salas de estar.
b) caixas de papelão.
c) móveis elegantes.
d) casas habitadas.
e) locais de residência (moradias).
GABARITO: E

Leia o texto a seguir e responda às questões 04, 05 e 06.


‘Tradwives’: the new trend for submissive women has a dark heart and history
A thrilling new trend _________ (1) emerged, and it’s as seismic as the New Look __________ (2)
in 40s fashion, or the emergence of Mediterranean cooking in 90s Britain: it’s called being a
housewife.
So a “tradwife” is a woman who doesn’t work so as to look after their children, their husband,
their home and then talk non-stop about how great this is on social media. Who knew being so
traditional was also so modern? And so busy! Last week alone, there were interviews with
tradwives in the Daily Mail, the Times and on the BBC, This Morning, Victoria Derbyshire and, for
all I know, piped 24/7 across all channels. I’m afraid that – being both non-trad and a non-wife – I
am less plugged in than these women.
The tradwives have been keenly giving interviews about how they are the true feminists in
choosing not to work… as much as the tradwives think they are being renegade rebels by not
working, their rebellion is based on their husband earning enough to support a whole household.
Whoa there, little rebels!
Adapted from https://www.theguardian.com/fashion/2020/jan/27/tradwives-new-trend-submissive-women-dark-heart-history

Questão 04 (EsPCEx/INÉDITA) – In the sentence “Whoa there, little rebels!” (paragraph 3), the
author
(A) aligns her thinking with that of the so-called “tradwives”.
(B) is ironic about the supposed rebelliousness of the “tradwives”.
(C) reinforces that the new housewives are the real feminists.
(D) defines the degree of rebelliousness of “tradwives” as high.
(E) demonstrates that “tradwives” are renegade by their husbands.
Comentários: Podemos perceber, ao longo do texto, que a autora é irônica ao falar sobre as
“tradwives”. O termo é um neologismo formado a partir do adjetivo traditional (tradicional) e do
substantivo wives (esposas) para designar um certo tipo de mulheres que opta por não trabalhar.
Ao utilizar a expressão “Whoa there, little rebels!”, algo como “Nossa, q ue rebeldes elas!”, a

AULA 06 – CONJUNCTIONS, DIRECT SPEECH AND REPORTED SPEECH 103


TEACHER ANDREA BELO

autora do texto está ironizando a suposta rebeldia dessas mulheres. Assim, Na frase “Whoa there,
little rebels!” (parágrafo 3), a autora a) alinha seu pensamento com o das chamadas “tradwives”
– conforme explicado, ela não alinha seu pensamento com o delas. INCORRETA.
b) é irônica sobre a suposta rebeldia das “tradwives” – CORRETA, conforme explicado. Como
vimos, a expressão ironiza a suposta rebeldia das “tradwives”. Para a autora, por mais que as
donas-de-casa pensem que estão sendo rebeldes por não trabalharem, a rebeldia delas se baseia
no fato de o marido ganhar o suficiente para sustentar a família inteira. 3º parágrafo. “...the
tradwives think they are being renegade rebels by not working, their rebellion is based on their
husband earning enough to support a whole household.”
c) reforça que as novas donas de casa são as verdadeiras feministas – a autora não reforça que as
novas donas de casa são as verdadeiras feministas. Como vimos, na verdade, ela ironiza essa
afirmação, relativizando a rebeldia e o feminismo delas. INCORRETA.
d) define o grau de rebeldia das “tradwives” como alto – ao utilizar a expressão “Whoa there, little
rebels!’, a autora do texto NÃO define o grau de rebeldia das “tradwives” como elevado. Pelo
contrário, ela ironiza essa suposta rebeldia. INCORRETA.
e) demonstra que as “tradwives” são renegadas por seus maridos – a autora não demonstra que
as “tradwives” são renegadas pelos maridos. Quando ela usa a expressão renegade rebels no
trecho “...the tradwives think they are being renegade rebels by not working...”, a autora diz que
as “tradwives” se acham rebeldes renegadas por não trabalharem. INCORRETA.
GABARITO: B

Questão 05 (EsPCEx/INÉDITA) – Choose the alternative containing the correct verbs to complete
gaps (1) and (2) in paragraph 1, respectively.
(A) have, was
(B) have, is
(C) has, were
(D) has, was
(E) is, was
Comentários: Escolha a alternativa que contém os verbos corretos para preencher as lacunas (1)
e (2) no parágrafo 1, respectivamente.
A thrilling new trend _______ (1) emerged, → “A thrilling new trend” (uma nova tendência
empolgante) é o sujeito (ela, a tendência, it). Percebemos que teremos, no caso, uma forma verbal
composta, pois há uma lacuna e, após ela, um particípio. Para transmitirmos a ideia de que “uma
nova tendência empolgante surgiu”, usaremos o auxiliar has (para o sujeito it, it has emerged),
formando o Present Perfect.
and it’s as seismic as the New Look _______ (2) in 40s fashion, or the emergence of Mediterranean
cooking in 90s Britain: it’s called being a housewife → e é tão sísmica quanto o New Look era na
moda dos anos 40. Como a frase se refere aos anos 40, teremos que usar o verbo to be no passado
(was), concordando com o sujeito New Look (it).
d) has, was
GABARITO: D

AULA 06 – CONJUNCTIONS, DIRECT SPEECH AND REPORTED SPEECH 104


TEACHER ANDREA BELO

Questão 06 (EsPCEx/INÉDITA) – In the sentence “The tradwives have been keenly giving
interviews about how they are the true feminists in choosing not to work…” (paragraph 3), the
word keenly means
(A) enthusiastically
(B) generously
(C) reluctantly
(D) repeatedly
(E) promptly
Comentários: Na frase “As tradwives têm dado entrevistas entusiasmadamente sobre como elas
são as verdadeiras feministas ao escolherem não trabalhar ...” (parágrafo 3), a palavra keenly
significa
a) com entusiasmo
b) generosamente
c) relutantemente
d) repetidamente
e) prontamente
GABARITO: A

Leia o texto a seguir e responda às questões 07, 08 e 09.


Pharmacogenomics can show what your body makes of a drug
Doctors have long appreciated that the same dose of medicine will not necessarily have the same
effect on different patients. One of the ways in which people differ biologically is in how they
metabolise drugs, a process largely dependent on enzymes that can vary a lot from person to
person. Differences in enzymes can lead two patients with the same disease, and the same
treatment, to end up with a five-fold difference in the amount of working drug molecules in their
blood.
Sometimes such differences will mean that the drug is not circulating at a high enough level to
work. Sometimes it is circulating at too high a level and causing toxic side-effects. Though side-
effects sound peripheral they are a huge medical problem. The study of how genes affect an
individual’s response to drugs is known as pharmacogenetics, and it is flourishing.
One example of the technique’s potential is found with warfarin, a commonly used blood thinner
that has what doctors call a “narrow therapeutic window”: the distance between a level in the
blood too low to do good and a level too high for safety is small. Variations in an enzyme gene
called CYP2C9, which allow warfarin to stay in the blood for longer, are quite common and this
makes it harder to find the best therapeutic dose. A test produced by a genetic-testing company
makes it possible to look at the genes for such enzymes and appears to limit side-effects and
improve clinical outcomes.

AULA 06 – CONJUNCTIONS, DIRECT SPEECH AND REPORTED SPEECH 105


TEACHER ANDREA BELO

In chemotherapy, too, genetics can guide the choice of treatment in ways that other indicators
cannot. That matters since the drugs are so powerful. Some are so toxic that four patients in 100
will die from treatment, says David Kerr, a professor at Oxford University.
Adapted from https://www.economist.com/technology-quarterly/2020/03/12/pharmacogenomics-can-show-what-your-body-makes-of-a-drug.

Questão 07 (EsPCEx/INÉDITA) – According to the text, a relevant effect of the variation of


enzymes in the body from one person to another is that it
(A) equals the amount of medicine working in the blood of patients with the same disease.
(B) affects the way an individual metabolizes drugs and responds to them.
(C) increases the chances of healing a patient with more enzymes in the body by five times.
(D) causes the same dose of the drug to have the same effect in different patients.
(E) is biologically dissociated from the way people metabolize a drug in the body.
Comentários: Conforme o texto, um efeito relevante da variação de enzimas no corpo de uma
pessoa para outra é que a) iguala a quantidade de medicamento em funcionamento no sangue de
pacientes com a mesma doença – Não iguala. Na verdade, segundo o texto, as diferenças nas
enzimas podem levar dois pacientes com a mesma doença e o mesmo tratamento a uma diferença
de cinco vezes na quantidade de moléculas de medicamento em funcionamento no sangue.
“Differences in enzymes can lead two patients with the same disease, and the same treatment, to
end up with a five-fold difference in the amount of working drug molecules in their blood.”
INCORRETA.
b) afeta a maneira como um indivíduo metaboliza os medicamentos e responde a eles – De fato,
segundo o texto, uma das maneiras pelas quais as pessoas diferem biologicamente é na maneira
como metabolizam os medicamentos, um processo amplamente dependente de enzimas que
podem variar muito de pessoa para pessoa. “One of the ways in which people differ biologically is
in how they metabolise drugs, a process largely dependent on enzymes that can vary a lot from
person to person.” CORRETA.
c) aumenta em cinco vezes as chances de cura de um paciente com mais enzimas no organismo –
Quando o texto fala em “cinco vezes”, não se refere a chances de cura, mas sim a uma diferença
de cinco vezes na quantidade de moléculas de medicamento em funcionamento no sangue de
pacientes com a mesma doença e submetidos ao mesmo tratamento, porém com variações
quanto a enzimas. “Differences in enzymes can lead two patients with the same disease, and the
same treatment, to end up with a five-fold difference in the amount of working drug molecules in
their blood.” INCORRETA.
d) faz com que a mesma dose do medicamento tenha o mesmo efeito em diferentes pacientes –
O texto afirma justamente o contrário, isto é, que a mesma dose pode pode ter efeitos diferentes
por conta das diferenças nas enzimas de um organismo para o outro. INCORRETA.
e) é biologicamente dissociada da forma como as pessoas metabolizam no organismo um
medicamento – O assunto central do texto é justamente o quanto a variação de enzimas no
organismo de uma pessoa para outra interfere na forma como elas metabolizam no organismo um
medicamento. “One of the ways in which people differ biologically is in how they metabolise
drugs, a process largely dependent on enzymes that can vary a lot from person to person.”
INCORRETA.
GABARITO: B

AULA 06 – CONJUNCTIONS, DIRECT SPEECH AND REPORTED SPEECH 106


TEACHER ANDREA BELO

Questão 08 (EsPCEx/INÉDITA) – According to the text, the study of how genes affect an
individual's response to drugs is
(A) failing.
(B) concluded.
(C) fragmented.
(D) stagnant.
(E) thriving.
Comentários: De acordo com o texto, o estudo de como os genes afetam a resposta de um
indivíduo às drogas está
a) falhando.
b) concluído.
c) fragmentado.
d) estagnado.
e) prosperando.
“The study of how genes affect an individual’s response to drugs is known as pharmacogenetics,
and it is flourishing.” = O estudo de como os genes afetam a resposta de um indivíduo às drogas é
conhecido como farmacogenética e está florescendo (prosperando). Veja que o texto, ao
comentar sobre a farmacogenética (que é o estudo de como os genes afetam a resposta de um
indivíduo aos medicamentos a que a questão se refere) utiliza o verbo “to flourish”, transmitindo
a ideia de que o estudo está se desenvolvendo, prosperando, florescendo.
GABARITO: E

Questão 09 (EsPCEx/INÉDITA) – Among the excerpts from the text, which one best corroborates
the beneficial potential of pharmacogenetics?
(A) “genetics can guide the choice of treatment in ways that other indicators cannot.” (paragraph
4)
(B) “the distance between a level in the blood too low to do good and a level too high for safety
is small.” (paragraph 3)
(C) “a process largely dependent on enzymes that can vary a lot from person to person.”
(paragraph 1)
(D) “which allow warfarin to stay in the blood for longer” (paragraph 3)
(E) “a commonly used blood thinner that has what doctors call a ‘narrow therapeutic window’”
(paragraph 3)
Comentários: Dentre os trechos do texto, qual melhor corrobora o potencial benéfico da
farmacogenética?
a) “genetics can guide the choice of treatment in ways that other indicators cannot.” (4º parágrafo)
– a genética pode

AULA 06 – CONJUNCTIONS, DIRECT SPEECH AND REPORTED SPEECH 107


TEACHER ANDREA BELO

orientar a escolha do tratamento de maneiras que outros indicadores não podem. – Esse trecho
corrobora, confirma o potencial benéfico da farmacogenética, pois reforça que ela é capaz de
orientar escolhas nos tratamentos de doenças de formas que outros indicadores não conseguem.
CORRETA.
b) “the distance between a level in the blood too low to do good and a level too high for safety is
small.” (3º parágrafo) – a distância entre um nível no sangue muito baixo para fazer o bem e um
nível muito alto para segurança é pequena. – Esse trecho contém apenas a definição de um termo
médico trazido pelo texto, qual seja, “narrow therapeutic window” (janela terapêutica estreita).
Não confirma diretamente o potencial benéfico da farmacogenética. INCORRETA.
c) “a process largely dependent on enzymes that can vary a lot from person to person.” (1º
parágrafo) – um processo amplamente dependente de enzimas que podem variar muito de pessoa
para pessoa – Esse excerto traz apenas uma informação complementar a respeito do processo de
metabolizar medicamentos no organismo. Não confirma diretamente o potencial benéfico da
farmacogenética. INCORRETA.
d) “which allow warfarin to stay in the blood for longer” (3º parágrafo) – que permitem que a
varfarina permaneça no sangue por mais tempo –Se voltamos ao texto e analisamos o início desse
trecho, percebemos que ele adiciona informação sobre o que foi dito antes, ou seja, sobre as
variações em uma enzima específica. → “Variations in an enzyme gene called CYP2C9, which allow
warfarin to stay in the blood for longer...” Não corrobora diretamente o potencial benéfico da
farmacogenética. INCORRETA.
e) “a commonly used blood thinner that has what doctors call a ‘narrow therapeutic window’” (3º
parágrafo) – um anticoagulante comumente usado que possui o que os médicos chamam de
terapêutica; – Percebemos no 3º parágrafo que essa é uma explicação sobre a varfarina, um
anticoagulante. O trecho não corrobora o potencial benéfico da farmacogenética. INCORRETA.
GABARITO: A

Leia o texto a seguir e responda à questão 10.


America's Long Overdue Awakening to Systemic Racism
Just before 7 p.m. on June 1, a deployment of local, state and federal forces, armored head to toe
in riot gear, unleashed rubber bullets and sprayed tear gas onto a crowd of peaceful demonstrators
gathered in the park to protest under the mantra “Black Lives Matter.” Since then, the debate over
systemic racism has spread across the nation and around the world.
Trump’s Administration has repeatedly denied that discrimination against black Americans is
embedded in the political, economic, and social structure of the country. Trump believes there are
“injustices in society,” but the notion that antiblackness is intrinsic to U.S. law enforcement has
been declared untrue. Attorney General William Barr warned against “automatically assuming
that the actions of an individual necessarily mean that their organization is rotten.”
But, for all that’s good about America, something is rotten. The protesters in Lafayette Park on
June 1 may have been galvanized by the disturbing video of the murder of George Floyd,
suffocated to death beneath the knee of a Minneapolis police officer. But at the core of their
movement is much more than the outrage over the latest instances of police brutality.

AULA 06 – CONJUNCTIONS, DIRECT SPEECH AND REPORTED SPEECH 108


TEACHER ANDREA BELO

Centuries of racist policy have left black Americans in the dust. The U.S. may think it has brushed
chattel slavery into the dustbin of history after the Civil War, but the country never did a very good
job incinerating its traumatic remains, instead leaving embers that still burn today: an education
system that fails black Americans, substandard health care that makes them more vulnerable to
death and disease, and an economy that leaves millions without access to a living wage.
Adapted from https://time.com/5851855/systemic-racism-america/

Questão 10 (EsPCEx/INÉDITA) – According to the text, the June 1 protest


(A) was ended in a peaceful way.
(B) was violently developing until the police arrived.
(C) targeted gender inequality.
(D) had few participants.
(E) sparked a wide-ranging debate on racism.
Comentários: De acordo com o texto, o protesto descrito
a) foi encerrado de forma pacífica – De acordo com o texto, pouco antes das 19h em 1º de
junho, um destacamento de forças locais, estaduais e federais, blindados da cabeça aos pés em
equipamento anti-motim, disparou balas de borracha e jogou gás lacrimogêneo em uma
multidão de manifestantes pacíficos reunidos no parque para protestar sob o mantra “Black
Lives Matter”. “Just before 7 p.m. on June 1, a deployment of local, state and federal forces,
armored head to toe in riot gear, unleashed rubber bullets and sprayed tear gas onto a crowd of
peaceful demonstrators gathered in the park to protest under the mantra ‘Black Lives Matter’”.
INCORRETA.
b) estava se desenvolvendo de forma violenta até a polícia chegar. – Conforme o trecho que
vimos acima, foi com a chegada da força policial que o protesto ficou violento. Até então, estava
pacífico. “unleashed rubber bullets and sprayed tear gas onto a crowd of peaceful
demonstrators” = disparou balas de borracha e jogou gás lacrimogêneo em uma multidão de
manifestantes pacíficos. INCORRETA.
c) tinha como alvo a desigualdade de gênero – O texto não fala em desigualdade de gênero,
mas, sim, de raças. “gathered in the park to protest under the mantra ‘Black Lives Matter’”. =
reunidos no parque para protestar sob o mantra “Black Lives Matter”. INCORRETA.
d) contava com poucos participantes – Havia, na verdade, uma multidão de pessoas → crowd.
“unleashed rubber bullets and sprayed tear gas onto a crowd of peaceful demonstrators
gathered in the park to protest under the mantra ‘Black Lives Matter’”. = disparou balas de
borracha e jogou gás lacrimogêneo em uma multidão de manifestantes pacíficos reunidos no
parque para protestar sob o mantra “Black Lives Matter”. INCORRETA.
a) desencadeou um amplo debate sobre racismo – De acordo com o texto, o protesto, seguido
da violenta repressão policial, desencadeou, de fato, um amplo debate sobre racismo. “Since
then, the debate over systemic racism has spread across the nation and around the world.” =
Desde então (isto é, desde o episódio do protesto), o debate sobre o racismo sistêmico se
espalhou por todo o país e ao redor do mundo. CORRETA.
GABARITO: E

AULA 06 – CONJUNCTIONS, DIRECT SPEECH AND REPORTED SPEECH 109


TEACHER ANDREA BELO

CONSIDERAÇÕES FINAIS
Concluímos mais uma aula, outro passo até a sua aprovação!
E estamos caminhando para maior vocabulário e mais aprendizado de fato.
Nota-se o progresso em seus estudos e, provavelmente, uma maior tranquilidade para
enfrentar os exercícios que surgem.
E você vai se acostumando a equilibrar seus estudos de forma sistematizada, estudando
cada vez mais e com mais dedicação.
Outro detalhe importante para seu sucesso nos estudos, é continuar fazendo listas de
vocabulário das palavras e verbos, principalmente os irregulares, que aparecem em forma de lista
em inúmeras fontes de pesquisa.
Isso te ajudará nas questões futuras e torna você, como eu disse antes, um candidato mais
bem preparado e confiante para realizar uma excelente prova.
É importante lembrar também do nosso Fórum de dúvidas, exclusivo do Estratégia
Militares. Será minha forma de responder, no prazo máximo de 48 horas, o que mais você precise
saber para que os conteúdos fiquem ainda mais claros em seus estudos, certo?

AULA 06 – CONJUNCTIONS, DIRECT SPEECH AND REPORTED SPEECH 110


TEACHER ANDREA BELO

REFERÊNCIAS BIBLIOGRÁFICAS
BARRETO, Tania Pedroza; GARRIDO, Maria Line; SILVA, João Antenor de C., Inglês Instrumental.
Leitura e compreensão de textos. Salvador, Ba UFBA, 1995, p. 64.
BROWN. H. Douglas. Principles of Language Learning and Teaching. Prentice Hall International,
1988.
COMPEDELLI, Samira Yousseff. Português, Literatura, Produção de texto & Gramática – São Paulo:
Ed. Saraiva, 2002.
CORREIA, Clese Mary P. Reading Specific Purposes. Salvador/ Ba: UFBA, 1999.
COSTEIRA, Adriana Araújo de M. Reading Comprehension Skills. João Pessoa/PB: ETFP, 1998.
CRYSTAL David. Cambridge University Press 1997. The Cambridge Encyclopedia of Language.
Cambridge University Press 1997
FREEMAN. Diane Larsen. MURCIA. Marianne Celce. The Grammar Book, 1999.
DYE, Joan., FRANFORT, Nancy. Spectrum II, III A Communicative Course in English. USA, Prentice
Hall, 1994.
FAVERO, Maria de Lourdes Albuquerque (org.). Dicionário de educadores no Brasil: da colônia aos
dias atuais. Rio de Janeiro: UFRJ, MEC, INEP, 1999.
FRANKPORT, Nancy & Dye Hoab. Spectrum II, III Prentice Hall Regents Englewood Cliffs, New Jersy,
1994.
GADELHA, Isabel Maria B. Inglês Instrumental: Leitura, Conscientização e Prática. Teresina:
EDUFFI, 2000.
GUANDALINI, Eiter Otávio. Técnicas de Leitura em Inglês: ESP – English For Specific Purposes:
estágio 1. São Paulo: Texto novo, 2002.
GRELLET, Françoise. Developing Reading Skills. Cambridge University Press, 1995
HOLAENDER, Arnon & Sanders Sidney. A complete English Course. São Paulo. Ed. Moderna, 1995.
HUTCHINSON, Tom & WATERS, Alan. English for Specific Purposes. Cambridge: Cambridge
University Press, 1996
KRASHEN. Stephen D. Second Language Acquisition and Second Language Learning, Prentice-Hall
International, 1988.
LAENG, Mauro. Dicionário de pedagogia. Lisboa: Dom Quixote, 1973.
LEFFA, Vilson J. Metodologia do ensino de línguas. In: BOHN, H.; VANDRESEN, P. (org.). Tópicos de
linguística aplicada: o ensino de línguas estrangeiras. Florianópolis: Editora da UFSC, 1988. p. 211-
231.
LIBERATO, Wilson. Compact English Book Inglês Ensino Médio. São Paulo: FTD, Vol. Único, 1998
Mc ARTHUR. The Oxford Companion to the English Language. Oxford University Press 1992

AULA 06 – CONJUNCTIONS, DIRECT SPEECH AND REPORTED SPEECH 111


TEACHER ANDREA BELO

Fromkin. Victoria. An Introduction to Language


MARQUES, Amadeu. Inglês Série Brasil. ed. Atica. São Paulo: 2004. Vol. Único.
MURPHY, Raymond: Essencial Grammar in Use Oxford. New York Ed. Oxford University, 1997.
OLIVEIRA, Luciano Amaral. English For Tourism Students. Inglês para Estudantes de Turismo: São
Paulo, Rocca, 2001.
OLIVEIRA, Sara Rejane de F. Estratégias de leitura para Inglês Instrumental. Brasília: UNB, 1994.
QUINTANA, et alli. First Certificate. Master Class Oxford. New York, 2004: Ed. Oxford University.
PAULINO, Berenice F. et all. Leitura em textos em Inglês – Uma Abordagem Instrumental. Belo
Horizonte: Ed. Dos Autores, 1992.
PEREIRA, Edilberto Coelho. Inglês Instrumental. Teresina: ETFPI, 1998.
RODGES, Theodore. Jack C. Richards. Approaches and Methods in Language Teaching. Cambridge
University Press, 2001.
RODMAN Robert. Harcourt Brace 1993. English as a Global Language
STEWART, B., HAINES S. First Certificate, MasterClass. UK – Oxford 2004.
SILVA, João Antenor de C., GARRIDO, Maria Lina, BARRETO, Tânia Pedrosa. Inglês Instrumental:
Leitura e Compreensão de Textos. Salvador: Centro Editorial e Didático, UFBA. 1994
SOARES, Moacir Bretãs. Dicionário de legislação do ensino. 19.ed. Rio de Janeiro: FGV, 1981.
SOUZA, Adriana Srade F. Leitura em Língua Inglesa: Uma abordagem Instrumental. São Paulo:
Disal, 2005.
TUCK, Michael. Oxford Dictionary of Computing for Learners of English. Oxford: Oxford University
Press, 1996.
TOTIS, Verônica Pakrauskas. Língua Inglesa: leitura. São Paulo: Cortez, 1991.

Livros eletrônicos:
Dicionário Houaiss da Língua Portuguesa, Editora Objetiva, 2001.
MOURãO, Janaína Pereira. "Skimming x Scanning"; Brasil Escola. Disponível em
<https://brasilescola.uol.com.br/ingles/skimming-x-scanning.htm>. Acesso em 20 de março de
2019.
www.newsweek.com - Acesso em 18 de março de 2019.
http://www.galaor.com.br/tecnicas-de-leitura/ - Acesso em 19 de março de 2019.
Expressões Idiomáticas (continuação)" em Só Língua Inglesa. Virtuous Tecnologia da
Informação,2008-2019. Consultado em 03/04/2019 às 22:09. Disponível na Internet em
http://www.solinguainglesa.com.br/conteudo/Expressoes5.php

AULA 06 – CONJUNCTIONS, DIRECT SPEECH AND REPORTED SPEECH 112


TEACHER ANDREA BELO

TRADUÇÕES
Ignore the pessimism: Covid vaccines are quietly prevailing
Nightmare scenarios involving deadly new variants are making us all too gloomy – but there’s
a scientific case for optimism
It can be quite easy, reading the press, to believe that the pandemic will never end. Even when
good news about vaccines started to arrive in the autumn, this grim narrative managed to harden.
In the past month, you could read “five reasons that herd immunity is probably impossible”, even
with mass vaccination; breathless reports about yetuncharacterised but potentially ruinous
variants, such as the “double mutant” variant in India, or two concerning variants potentially
swapping mutations and teaming up in a “nightmare scenario” in California; get ready, some
analysts said, for the “permanent pandemic”.
Among many people I know, a sort of low-grade doom has set in. They think the vaccines are a
mere sliver of hope, only holding back the virus for a short time before being worn down by a rush
of ever-cleverer variants that will slosh around us, perhaps forever. Things might briefly get better,
they believe, but only by a little, and even that is tenuous.
However despite such dark talk, and the potential difficulties along the way of vaccine rollout, I
still remain optimistic. Since about the midway point of last year I have believed that extremely
potent vaccines are going to end the pandemic. They’ll do so by either driving the disease down
to near-extinction, or so constraining its force and spread that it becomes a manageable concern,
like measles or mumps. I actually think this will happen fairly soon, as long as we get everyone –
the whole world, not just the rich – vaccinated.
The scientific case for optimism is straightforward. The vaccines we have are beyond very good,
they’re among the most effective ever created. They appear to be potent in real-life situations,
and results so far show that protection is longlasting. Crucially, new results in the US show that
the mRNA vaccines used there effectively prevented coronavirus infections – not just serious
symptoms – in results similar to those previously reported by a UK-based study. And another
study in the UK suggested that vaccinated groups were less likely to spread coronavirus infection
overall. This is exactly what we need to choke out the pandemic: vaccines that don’t just protect,
but actually halt the virus infecting people and spreading.
When it comes to variants, it is clear that some are more infectious, and some are more deadly.
But their interaction with vaccines isn’t yet clear. Some lab-based results show that certain viral
mutations may make some immune responses less potent. And one study suggested the
Oxford/AstraZeneca vaccine might be less potent against the South African variant. But the
majority of scientists believe that vaccines have so far held the line, and will continue to do so. If
variants continue to make small advances, vaccines can be updated. A doomsday strain may be
possible, but exceedingly hard to predict. Evolution isn’t an on-demand miracle worker for viral
supremacy; even over decades most viruses don’t escape vaccine protection.
(Adapted from https://www.theguardian.com/commentisfree/2021/apr/12/ignore-scare-stories-covid-vaccines-variants-scientific-optimism)

AULA 06 – CONJUNCTIONS, DIRECT SPEECH AND REPORTED SPEECH 113


TEACHER ANDREA BELO

Ignore o pessimismo: as vacinas Covid estão prevalecendo silenciosamente


Cenários de pesadelo envolvendo novas variantes mortais estão nos deixando sombrios
demais – mas há um caso científico para otimismo
Pode ser muito fácil, lendo a imprensa, acreditar que a pandemia nunca vai acabar. Mesmo
quando boas notícias sobre vacinas começaram a chegar no outono, essa narrativa sombria
conseguiu endurecer. No mês passado, você poderia ler “cinco razões pelas quais a imunidade de
rebanho é provavelmente impossível”, mesmo com a vacinação em massa; relatos sem fôlego
sobre variantes ainda não caracterizadas, mas potencialmente destrutivas, como a variante
“dupla mutante” na Índia, ou duas sobre variantes potencialmente trocando mutações e se
unindo em um “cenário de pesadelo” na Califórnia; prepare-se, disseram alguns analistas, para a
“pandemia permanente”.
Entre muitas pessoas que conheço, uma espécie de desgraça de baixo grau se instalou. Eles
pensam que as vacinas são um mero fragmento de esperança, apenas segurando o vírus por um
curto período de tempo antes de ser desgastado por uma corrida de variantes cada vez mais
inteligentes que vai se espalhar ao nosso redor, talvez para sempre. As coisas podem melhorar
brevemente, eles acreditam, mas apenas um pouco, e mesmo isso é tênue.
No entanto, apesar de toda essa conversa sombria e das dificuldades potenciais ao longo do
caminho para o lançamento da vacina, ainda continuo otimista. Desde meados do ano passado,
tenho acreditado que vacinas extremamente potentes vão acabar com a pandemia. Eles farão
isso reduzindo a doença à quase extinção ou restringindo sua força e disseminação de forma que
se torne uma preocupação administrável, como sarampo ou caxumba. Na verdade, acho que isso
vai acontecer em breve, desde que todos - o mundo inteiro, não apenas os ricos - sejam vacinados.
O caso científico de otimismo é direto. As vacinas que temos são muito boas, estão entre as mais
eficazes já criadas. Eles parecem ser potentes em situações da vida real e os resultados até agora
mostram que a proteção é duradoura. Crucialmente, novos resultados nos EUA mostram que as
vacinas de mRNA usadas lá preveniram eficazmente infecções por coronavírus - não apenas
sintomas graves - em resultados semelhantes aos relatados anteriormente por um estudo
baseado no Reino Unido. E outro estudo no Reino Unido sugeriu que os grupos vacinados eram
menos propensos a espalhar a infecção por coronavírus em geral. Isso é exatamente o que
precisamos para sufocar a pandemia: vacinas que não apenas protegem, mas na verdade
interrompem o vírus que infecta as pessoas e se espalha.
Quando se trata de variantes, é claro que algumas são mais infecciosas e outras são mais mortais.
Mas sua interação com as vacinas ainda não está clara. Alguns resultados de laboratório mostram
que certas mutações virais podem tornar algumas respostas imunológicas menos potentes. E um
estudo sugeriu que a vacina Oxford / AstraZeneca pode ser menos potente contra a variante sul-
africana. Mas a maioria dos cientistas acredita que as vacinas até agora se mantiveram na linha e
continuarão a fazê-lo. Se as variantes continuarem a fazer pequenos avanços, as vacinas podem
ser atualizadas. Uma tensão do Juízo Final pode ser possível, mas extremamente difícil de prever.
A evolução não é um fazedor de milagres sob demanda para a supremacia viral; mesmo ao longo
de décadas, a maioria dos vírus não escapa da proteção da vacina.

AULA 06 – CONJUNCTIONS, DIRECT SPEECH AND REPORTED SPEECH 114


TEACHER ANDREA BELO

Sinopharm: Chinese Covid vaccine gets WHO emergency approval


The World Health Organization (WHO) has granted emergency approval for the Covid vaccine
made by Chinese state-owned company Sinopharm.
It is the first vaccine developed by a non-Western country to get WHO backing. The vaccine has
already been given to millions of people in China and elsewhere.
The WHO had previously only approved the vaccines made by Pfizer, AstraZeneca, Johnson &
Johnson and Moderna.
But individual health regulators in various countries - especially poorer ones in Africa, Latin
America and Asia - have approved Chinese jabs for emergency use.
With little data released internationally early on, the effectiveness of the various Chinese vaccines
has long been uncertain.
But the WHO on Friday said it had validated the "safety, efficacy and quality" of the Sinopharm
jab.
The WHO said the addition of the vaccine had "the potential to rapidly accelerate Covid-19 vaccine
access for countries seeking to protect health workers and populations at risk".
It is recommending that the vaccine be administered in two doses to those aged 18 and over.
A decision is expected in the coming days on another Chinese vaccine developed by Sinovac, while
Russia's Sputnik vaccine is under assessment.
Why does WHO backing matter?
The green light from the global health body is a guideline for national regulators that a vaccine is
safe and effective.
WHO director-general Tedros Adhanom Ghebreyesus said it would give countries "confidence to
expedite their own regulatory approval".
It also means that the vaccine can be used in the global Covax programme, which was set up last
year to try to ensure fair access to vaccines among rich and poor nations.
The decision to list the Chinese vaccine for emergency use is expected to give a substantial boost
to the scheme, which has been struggling with supply problems.
(Adapted from https://www.bbc.com/news/world-asia-china-56967973)

Sinopharm: vacina chinesa Covid obtém aprovação de emergência da OMS


A Organização Mundial da Saúde (OMS) concedeu aprovação de emergência para a vacina
Covid, fabricada pela empresa estatal chinesa Sinopharm.
É a primeira vacina desenvolvida por um país não ocidental para obter o apoio da OMS. A vacina
já foi dada a milhões de pessoas na China e em outros lugares.
A OMS havia anteriormente aprovado apenas as vacinas da Pfizer, AstraZeneca, Johnson &
Johnson e Moderna.
Mas reguladores de saúde individuais em vários países - especialmente os mais pobres na África,
América Latina e Ásia - aprovaram jabs chineses para uso emergencial.

AULA 06 – CONJUNCTIONS, DIRECT SPEECH AND REPORTED SPEECH 115


TEACHER ANDREA BELO

Com poucos dados divulgados internacionalmente no início, a eficácia das várias vacinas chinesas
há muito é incerta.
Mas a OMS disse na sexta-feira que validou a "segurança, eficácia e qualidade" do Sinopharm jab.
A OMS disse que a adição da vacina tem "o potencial de acelerar rapidamente o acesso à vacina
Covid-19 para países que buscam proteger profissionais de saúde e populações em risco".
Recomenda-se que a vacina seja administrada em duas doses em maiores de 18 anos.
Uma decisão é esperada nos próximos dias sobre outra vacina chinesa desenvolvida pela Sinovac,
enquanto a vacina russa Sputnik está sendo avaliada.
Por que o apoio da OMS é importante?
O sinal verde do órgão de saúde global é uma diretriz para os reguladores nacionais de que uma
vacina é segura e eficaz.
O diretor-geral da OMS, Tedros Adhanom Ghebreyesus, disse que isso daria aos países "confiança
para agilizar sua própria aprovação regulatória".
Isso também significa que a vacina pode ser usada no programa global Covax, que foi criado no
ano passado para tentar garantir o acesso justo às vacinas entre as nações ricas e pobres.
A decisão de listar a vacina chinesa para uso emergencial deve dar um impulso substancial ao
esquema, que tem lutado com problemas de abastecimento.

The American dream


__________ (1) unitary definition does not exist–the American dream probably has a different
meaning to every US citizen. For some it is __________ (2) dream of freedom and equality, for
others it is the dream of a fulfilled life or even the dream of fame and wealth.
In general, the American dream can be defined as being the opportunity and freedom for all
citizens to achieve their goals and become rich and famous if only they work hard enough.
The first settlers hoped for freedom of religion; in their home countries they had been persecuted
because of __________ (3) religious and political views. Bad living conditions in Europe and the
hope for better living standards in America attracted more and more settlers to migrate to the
new world. The booming US industry during the first half of the 20th century caused the myth
“from rags to riches”.
Today’s role of the American dream is a matter of discussion. Some think that the American dream
is still a viable element today, for others it is only an illusion.
The American dream focusses on the success of the individuum (not the wellbeing of the whole
population). On the one hand, this has advantages, e.g. lower taxes, little interference of the state.
On the other hand, the same advantages are disadvantageous for society: there is no social fond
for the poor and there is no serious interest in important issues like environmental protection etc.
Adapted from: https://www.ego4u.com/en/read-on/countries/usa/american-dream

AULA 06 – CONJUNCTIONS, DIRECT SPEECH AND REPORTED SPEECH 116


TEACHER ANDREA BELO

O sonho americano
__________ (1) definição unitária não existe – o sonho americano provavelmente tem um
significado diferente para cada cidadão dos EUA. Para alguns é __________ (2) sonho de liberdade
e igualdade, para outros é o sonho de uma vida plena ou mesmo o sonho de fama e riqueza.
Em geral, o sonho americano pode ser definido como sendo a oportunidade e a liberdade para
todos os cidadãos alcançarem seus objetivos e se tornarem ricos e famosos se trabalharem duro
o suficiente.
Os primeiros colonos esperavam a liberdade de religião; em seus países de origem eles foram
perseguidos por causa de __________ (3) pontos de vista religiosos e políticos. As más condições
de vida na Europa e a esperança de melhores padrões de vida na América atraíram cada vez mais
colonos a migrar para o novo mundo. A indústria em expansão dos EUA durante a primeira
metade do século 20 causou o mito “dos trapos à riqueza”.
O papel atual do sonho americano é uma questão de discussão. Alguns pensam que o sonho
americano ainda é um elemento viável hoje, para outros é apenas uma ilusão.
O sonho americano se concentra no sucesso do indivíduo (não no bem-estar de toda a população).
Por um lado, isto tem vantagens, e. impostos mais baixos, pouca interferência do Estado. Por
outro lado, as mesmas vantagens são desvantajosas para a sociedade: não há apego social aos
pobres e não há interesse sério em questões importantes como proteção ambiental etc.

In the first half of 2021, more than 8,100 people died in the US because of gun violence. It is
___________ (1) number in the last 20 years.
A report said that nearly 54 Americans died every day in shootings. It was 14 more people than in
the same time in the past six years. Last weekend, more than 120 people died in shootings.
Experts said that there were more reasons why gun violence was growing. More people owned a
gun, and there were a lot of problems between police and communities. Also, there were big
differences in the economy and healthcare, and there were racial problems in American society
which have lasted a very long time.
Adapted from: https://www.newsinlevels.com/products/guns-in-the-us-level-2/

No primeiro semestre de 2021, mais de 8.100 pessoas morreram nos Estados Unidos por causa
da violência armada. É ___________ (1) número nos últimos 20 anos.
Um relatório disse que quase 54 americanos morreram todos os dias em tiroteios. Foram 14
pessoas a mais do que na mesma época nos últimos seis anos. No último fim de semana, mais de
120 pessoas morreram em tiroteios.
Especialistas dizem que há mais razões para o crescimento da violência armada. Mais pessoas
tinham uma arma e havia muitos problemas entre a polícia e as comunidades. Além disso, havia
grandes diferenças na economia e na saúde, e havia problemas raciais na sociedade americana
que duram muito tempo.

AULA 06 – CONJUNCTIONS, DIRECT SPEECH AND REPORTED SPEECH 117


TEACHER ANDREA BELO

The construction of the Empire State Building began in 1930. It was built by 3,400 workers who
were mostly poor immigrants from Europe. They worked in any weather condition and with
simple tools. It was extremely dangerous work, but they made good money.
The building was constructed under great time pressure. They had to finish the work by May 1,
1931. The workers built 4 new floors every week and they finished the work in time.
The building is 381 meters high and has 102 floors. It has 6,500 windows and 73 elevators. Over
60,000 tons of steel were used for the construction. At the time, it was the tallest skyscraper in
the world.
Today, the building is still a symbol of New York. Thousands of people visit it every day. There are
also offices of many companies in the building and 20,000 people work there.
https://www.daysinlevels.com/article/the-day-the-empire-state-building-opened-level-2/

A construção do Empire State Building começou em 1930. Ele foi construído por 3.400
trabalhadores, em sua maioria imigrantes pobres da Europa. Eles trabalharam em qualquer
condição climática e com ferramentas simples. Era um trabalho extremamente perigoso, mas eles
ganhavam um bom dinheiro.
O prédio foi construído sob grande pressão de tempo. Eles tinham que terminar a obra até 1º de
maio de 1931. Os trabalhadores construíram 4 novos andares todas as semanas e terminaram a
obra a tempo.
O prédio tem 381 metros de altura e 102 andares. Possui 6.500 janelas e 73 elevadores. Mais de
60.000 toneladas de aço foram utilizadas para a construção. Na época, era o arranha-céu mais
alto do mundo.
Hoje, o prédio ainda é um símbolo de Nova York. Milhares de pessoas o visitam todos os dias. Há
também escritórios de várias empresas no prédio e 20.000 pessoas trabalham lá.

Man living alone on Italian island to leave after 32 years


An Italian man is to leave the island he has lived on for more than 30 years after pressure from
authorities.
Mauro Morandi, 81, moved to Budelli off northern Sardinia in 1989.
Last year, however, Mr Morandi said the owners of the island, which is famed for its pink beach,
wanted him to relocate.
"I will leave hoping that in the future Budelli will be protected as I have protected it for 32 years,"
he wrote on Facebook on Sunday.
Speaking to BBC Outlook in 2018, the former physical education teacher said he had "always been
a bit of a rebel".
"I was quite fed up with a lot of things about our society: consumerism and the political situation
in Italy," Mr Morandi said. "I decided to move to a desert island in Polynesia, away from all
civilisation. I wanted to start a new life close to nature."

AULA 06 – CONJUNCTIONS, DIRECT SPEECH AND REPORTED SPEECH 118


TEACHER ANDREA BELO

After beginning their journey by sea, he and a number of friends landed on the Italian archipelago
of La Maddalena, where they planned to work and make money to fund the rest of their travels.
But after arriving on Budelli and meeting the island's guardian, who was about to retire, Mr
Morandi decided to stay - and has remained ever since, looking after the island and speaking to
tourists.
However, he has faced a number of eviction threats over the years.
In 2020 the president of La Maddalena's National Park, Fabrizio Fonnesu, told CNN that Mr
Morandi had carried out illegal alterations to his hut, a former radio station dating back to World
War Two.
A petition calling on the Italian government to allow him to stay gathered more than 70,000
signatures.
But on Sunday, Mr Morandi announced his decision to leave, telling the The Guardian newspaper
that he would move to a small flat on the nearby island of La Maddalena: "My life won't change
too much, I'll still see the sea."
(Adapted from https://www.bbc.com/news/world-europe-56885716)

Homem morando sozinho na ilha italiana para partir depois de 32 anos


Um italiano deve deixar a ilha onde vive há mais de 30 anos, após pressão das autoridades.
Mauro Morandi, 81, mudou-se para Budelli, no norte da Sardenha, em 1989.
No ano passado, porém, Morandi disse que os proprietários da ilha, famosa por sua praia rosa,
queriam que ele se mudasse.
“Vou partir com a esperança de que no futuro a Budelli esteja protegida como eu a protegi por
32 anos”, escreveu ele no Facebook no domingo.
Em declarações à BBC Outlook em 2018, o ex-professor de educação física disse que "sempre foi
um pouco rebelde".
"Eu estava farto de muitas coisas sobre nossa sociedade: o consumismo e a situação política na
Itália", disse Morandi. "Decidi me mudar para uma ilha deserta na Polinésia, longe de todas as
civilizações. Queria começar uma nova vida perto da natureza."
Depois de iniciar a viagem por mar, ele e vários amigos desembarcaram no arquipélago italiano
de La Maddalena, onde planejavam trabalhar e ganhar dinheiro para custear o resto de suas
viagens.
Mas depois de chegar a Budelli e conhecer o guardião da ilha, que estava prestes a se aposentar,
Morandi decidiu ficar - e tem permanecido desde então, cuidando da ilha e conversando com os
turistas.
No entanto, ele enfrentou várias ameaças de despejo ao longo dos anos.
Em 2020, o presidente do Parque Nacional de La Maddalena, Fabrizio Fonnesu, disse à CNN que
Morandi havia feito alterações ilegais em sua cabana, uma antiga estação de rádio que datava da
Segunda Guerra Mundial.

AULA 06 – CONJUNCTIONS, DIRECT SPEECH AND REPORTED SPEECH 119


TEACHER ANDREA BELO

Uma petição apelando ao governo italiano para permitir que ele ficasse reuniu mais de 70.000
assinaturas.
Mas no domingo, o Sr. Morandi anunciou sua decisão de partir, dizendo ao jornal The Guardian
que se mudaria para um pequeno apartamento na ilha vizinha de La Maddalena: "Minha vida não
vai mudar muito, ainda vou ver o mar . "

As I Am – Justin Bieber
By myself sometimes
To give my mind some space
Yeah, I know, yeah, I know that it hurts
When I push your love away, I hate myself
I wanna tell you lies
So your heart won't break
Yeah, I know, yeah, I know that I made my fair share of mistakes
Sometimes, I don't know why you love me
Sometimes, I don't know why you care
Take me with the good and the ugly
Say, "I'm not goin' anywhere"
Take me as I am, swear I'll do the best I can
Say, "I'm not goin' anywhere"
Take me as I am, swear I'll do the best I can
Say, "I'm not goin' anywhere"
Thousand miles apart, yeah, you're the one that holds my heart
It's no surprise
And I tell you all the time
And when life gets way too hard
I'll meet you in the stars, you know I will
I'll keep you near (mmm)
As I wipe away your tears…
(Adapted from https://genius.com/Justin-bieber-as-i-am-lyrics)

As I Am - Justin Bieber
Sozinho às vezes
Para dar algum espaço à minha mente
Sim, eu sei, sim, eu sei que dói

AULA 06 – CONJUNCTIONS, DIRECT SPEECH AND REPORTED SPEECH 120


TEACHER ANDREA BELO

Quando eu afasto seu amor, eu me odeio


Eu quero te contar mentiras
Então seu coração não vai quebrar
Sim, eu sei, sim, eu sei que cometi meu quinhão de erros
Às vezes eu não sei porque você me ama
Às vezes, eu não sei porque você se importa
Leve-me com o bom e o feio
Diga: "Eu não vou a lugar nenhum"
Me aceite como eu sou, juro que farei o melhor que puder
Diga: "Eu não vou a lugar nenhum"
Leve-me como eu sou, juro que farei o melhor que puder
Diga: "Eu não vou a lugar nenhum"
A milhares de quilômetros de distância, sim, você é aquele que segura meu coração
Não é surpresa
E eu te digo o tempo todo
E quando a vida fica muito difícil
Eu vou te encontrar nas estrelas, você sabe que eu vou
Vou mantê-lo por perto (mmm)
Enquanto eu enxugo suas lágrimas ...

Royal Navy flotilla reports Covid outbreak with 100 cases despite all sailors being vaccinated
The outbreak has not prompted a rethink of the Royal Navy’s voyage to 40 countries
More than 100 cases of coronavirus have been reported aboard the Royal Navy’s new aircraft
carrier and an escorting group of warships.
The vessels affected include the Royal Navy’s flagship HMS Queen Elizabeth carrier, with
unconfirmed media reports saying the group of warships recently stopped off for a brief spell of
shore leave in Limassol, Cyprus.
The virus has spread through the strike group’s 3,700 strength crew despite them being fully
vaccinated. “As part of routine testing, a small number of crew from the Carrier Strike Group have
tested positive for Covid-19,” a Royal Navy spokesman confirmed in a statement.
The incident comes despite a series of measures deployed on board Navy warships in light of the
pandemic. These include the use of masks, social distancing and a track and trace system.

AULA 06 – CONJUNCTIONS, DIRECT SPEECH AND REPORTED SPEECH 121


TEACHER ANDREA BELO

The outbreak has not yet prompted a rethink of the strike group’s planned voyage to 40 countries,
said a Royal Navy spokesperson. “The Carrier Strike Group will continue to deliver their
operational tasks and there are no effects on the deployment.”
The aircraft carrier was about a quarter of the way through its 28-week deployment when the
Covid cases were detected. It is leading a strike group that includes 10 Marine Corps F35-B fighters
and is currently in the Indo-Pacific.
It is the largest combined naval and air task force assembled under British command since the
Falklands War, and was expected to pass through the South China Sea.
Ben Wallace, the defence secretary, said during a press briefing on Tuesday that the first cases
involving the strike group were reported on 4 July.
“Our crew are double vaccinated so you’ll be glad to know there is no serious effects on any of
the crew and we will manage it,” he said, adding that going forwards he will offer support to the
captain of the ship in “whatever decision he makes”.
https://www.independent.co.uk/news/uk/home-news/hms-queen-elizabeth-covid-royal-navy-b1883699.html

Flotilha da Marinha Real relata surto de Covid com 100 casos, apesar de todos os marinheiros
terem sido vacinados
O surto não levou a um repensar da viagem da Marinha Real a 40 países
Mais de 100 casos de coronavírus foram relatados a bordo do novo porta-aviões da Marinha Real
e um grupo de escolta de navios de guerra.
Os navios afetados incluem o porta-aviões HMS Queen Elizabeth da Marinha Real, com relatos da
mídia não confirmados dizendo que o grupo de navios de guerra parou recentemente para um
breve período de licença em Limassol, Chipre.
O vírus se espalhou através da equipe de 3.700 força do grupo de ataque, apesar de eles estarem
totalmente vacinados. “Como parte dos testes de rotina, um pequeno número de tripulantes do
Carrier Strike Group testou positivo para Covid-19”, confirmou um porta-voz da Marinha Real em
um comunicado.
O incidente ocorre apesar de uma série de medidas implantadas a bordo de navios de guerra da
Marinha em função da pandemia. Isso inclui o uso de máscaras, distanciamento social e um
sistema de rastreio e rastreamento.
O surto ainda não levou a um repensar da viagem planejada do grupo de ataque a 40 países, disse
um porta-voz da Marinha Real. “O Carrier Strike Group continuará entregando suas tarefas
operacionais e não haverá efeitos na implantação.”
O porta-aviões estava a cerca de um quarto de sua implantação de 28 semanas quando os casos
Covid foram detectados. Ele está liderando um grupo de ataque que inclui 10 caças F35-B do
Corpo de Fuzileiros Navais e está atualmente no Indo-Pacífico.
É a maior força-tarefa combinada naval e aérea montada sob o comando britânico desde a Guerra
das Malvinas, e esperava-se que passasse pelo Mar da China Meridional.

AULA 06 – CONJUNCTIONS, DIRECT SPEECH AND REPORTED SPEECH 122


TEACHER ANDREA BELO

Ben Wallace, o secretário de defesa, disse durante uma coletiva de imprensa na terça-feira que
os primeiros casos envolvendo o grupo de ataque foram relatados em 4 de julho.
“Nossa tripulação está duplamente vacinada, então você ficará feliz em saber que não há efeitos
graves em nenhum membro da tripulação e nós vamos administrar isso”, disse ele, acrescentando
que, daqui para frente, oferecerá apoio ao capitão do navio em “ qualquer decisão que ele tomar
”.

Lewis Hamilton hopes for diversity legacy


Sir Lewis Hamilton has said he hopes improving the diversity of Formula One will be part of his
legacy.
The seven-times world champion ____________ a report which he hopes will encourage the
industry to become more inclusive and inspire young people from ethnic minorities to pursue a
career in the sport.
https://www.bbc.com/news/av/uk-57814103

Lewis Hamilton espera um legado de diversidade


Sir Lewis Hamilton disse que espera que melhorar a diversidade da Fórmula 1 faça parte de seu
legado.
O heptacampeão mundial ____________ um relatório que ele espera que incentive a indústria a
se tornar mais inclusiva e inspire jovens de minorias étnicas a seguirem uma carreira no esporte.

A pastor in Sierra Leone came across a jaw-dropping stone – a 706-carat (141 grams) diamond. It
is the largest diamond to be found in Sierra Leone in over 40 years.
When the pastor __________ it, he decided __________ it over to the government. __________
that it may boost the development of the West African nation.
The President of Sierra Leone said that he believed that a diamond like this should be publicly
__________ in the country, so that people will know the value of it and who the buyer is.
https://www.newsinlevels.com/products/big-diamond-in-sierra-leone

Um pastor em Serra Leoa encontrou uma pedra de cair o queixo - um diamante de 706 quilates
(141 gramas). É o maior diamante encontrado em Serra Leoa em mais de 40 anos.
Quando o pastor o __________, ele decidiu __________ para o governo. __________ que pode
impulsionar o desenvolvimento da nação da África Ocidental.
O Presidente da Serra Leoa disse acreditar que um diamante como este deve ser __________
publicamente no país, para que as pessoas saibam o seu valor e quem é o comprador.

AULA 06 – CONJUNCTIONS, DIRECT SPEECH AND REPORTED SPEECH 123


TEACHER ANDREA BELO

Fast food is one of those things that the USA __________ to the world. But while hamburgers and
fries __________ worldwide as __________ typically American, in actual fact Americans did not
invent fast food - they just made it into a lifestyle.
Adapted from: https://linguapress.com/advanced/american-food.htm

Fast food é uma daquelas coisas que os EUA __________ para o mundo. Mas enquanto
hambúrgueres e batatas fritas __________ no mundo todo como __________ tipicamente
americanos, na verdade os americanos não inventaram o fast food - eles apenas o transformaram
em um estilo de vida.

_____ the night of April 4th 1968, someone was waiting opposite the windows of the Lorraine
Motel, _____ downtown Memphis.
_____ front of the motel, a big white Cadillac was parked; it was the car _____ which the Rev.
Martin Luther King was being driven round, as he traveled through the southern states, speaking
to audiences _____ towns and cities, promoting the cause of non-violence and civil rights.
When King stepped out onto the balcony, to take a breath of fresh air after eating his dinner, a
shot rang out. The civil rights leader and Nobel-prizewinner, the man who preached non-violence,
fell to the ground, fatally wounded. Within minutes, he was dead.
https://linguapress.com/advanced/martin-luther-king.htm

_____ na noite de 4 de abril de 1968, alguém estava esperando em frente às janelas do Lorraine
Motel, _____ no centro de Memphis.
_____ frente ao motel, um grande Cadillac branco estava estacionado; era o carro _____ que o
reverendo Martin Luther King estava dirigindo, enquanto viajava pelos estados do sul, falando
para o público _____ cidades e vilas, promovendo a causa da não-violência e dos direitos civis.
Quando King saiu para a varanda, para tomar um fôlego de ar fresco depois de comer seu jantar,
um tiro foi disparado. O líder dos direitos civis e ganhador do Prêmio Nobel, o homem que pregava
a não-violência, caiu no chão, mortalmente ferido. Em poucos minutos, ele estava morto.

France to drop testing requirements for vaccinated tourists from some countries
France is making itself available as a destination for international tourists who have had
coronavirus jabs.
The government has announced that it is removing the need for coronavirus tests for vaccinated
Europeans and also allowing vaccinated tourists from most of the rest of the world, including the
US, to visit, provided they have a negative test.
Associated Press report that the relaxed rules will kick in from Wednesday. Tourism will not be
possible, however, from countries with virus surges and or prevalent variants. This “red list” for
the moment has 16 countries, including India, South Africa, and Brazil.

AULA 06 – CONJUNCTIONS, DIRECT SPEECH AND REPORTED SPEECH 124


TEACHER ANDREA BELO

Outside of Europe, most of the rest of the world is classed as “orange” in the new travel rulebook
released today.
Vaccinated visitors from “orange” countries — including the US and Britain — will no longer need
to quarantine on arrival and will no longer have to justify the reasons for their trip to France. They
will, however, still be asked for a negative PCR test no older than 72 hours or a negative antigenic
test of no more than 48 hours.
Unvaccinated children will be allowed in with vaccinated adults.
European visitors and those from seven countries classed as “green” — Australia, South Korea,
Israel, Japan, Lebanon, New Zealand, and Singapore — will no longer need to undergo testing if
they are vaccinated.
(Adapted from https://www.theguardian.com/world/live/2021/jun/04/coronavirus-live-news-japan-taiwan-vaccine-
fauci-calls-wuhan-lab-records?page=with:block-60b9f3308f082ac930ae9fb8#block-60b9f3308f082ac930ae9fb8)

França vai retirar os requisitos de teste para turistas vacinados de alguns países
A França está se tornando um destino para turistas internacionais que já receberam vacina contra
o coronavírus.
O governo anunciou que está eliminando a necessidade de testes de coronavírus para europeus
vacinados e também permitindo que turistas vacinados da maior parte do resto do mundo,
incluindo os EUA, visitem, desde que tenham um teste negativo.
A Associated Press informou que as regras relaxadas entrarão em vigor na quarta-feira. O turismo
não será possível, no entanto, em países com surtos de vírus e / ou variantes predominantes. Essa
“lista vermelha” no momento tem 16 países, incluindo Índia, África do Sul e Brasil.
Fora da Europa, a maior parte do resto do mundo é classificada como “laranja” no novo livro de
regras de viagens lançado hoje.
Visitantes vacinados de países “laranja” - incluindo os EUA e a Grã-Bretanha - não precisarão mais
ficar em quarentena na chegada e não terão mais que justificar os motivos de sua viagem à França.
No entanto, eles ainda serão solicitados a fazer um teste de PCR negativo com no máximo 72
horas ou um teste antigênico negativo com no máximo 48 horas.
Crianças não vacinadas serão permitidas com adultos vacinados.
Visitantes europeus e de sete países classificados como “verdes” - Austrália, Coréia do Sul, Israel,
Japão, Líbano, Nova Zelândia e Cingapura - não precisarão mais se submeter a testes se forem
vacinados.

If you bought a dog during lockdown, they'll need help coming out of it
Behavioural problems are a common reason for rehoming, so prepare them for when you’re
not there 24/7
Despite the lows of the past year, for many lockdown has been the perfect opportunity to
welcome a dog into their lives. The demand for dogs during the pandemic has been huge, with a
60% increase in calls from people seeking to adopt from the Dogs Trust charity, and with many

AULA 06 – CONJUNCTIONS, DIRECT SPEECH AND REPORTED SPEECH 125


TEACHER ANDREA BELO

other rescue organisations reporting similar findings. Google searches for “buy a puppy”
increased by 115% after the UK first went into lockdown in March 2020, with prices for some of
the most soughtafter breeds reaching record levels.
Dog ownership is a wonderful thing, but it is also a huge responsibility and a commitment that
spans way beyond lockdown: as the saying goes, “A dog is for life”. As restrictions ease and the
resumption of normality begins, it’s important we consider the implications for our canine
companions and give them a hand to help them adjust.
Having a dog around has helped many people cope with lockdown. Our dogs mostly love us being
around too: going for longer walks, having more playtime, and resting by our side. Nevertheless,
it’s safe to say life has not been normal for our dogs for most of the past year. Few have met other
dogs, and if they have seen them, it would have been from afar or on a lead, meaning that they
were unable to interact or play. There have also been fewer visitors coming into the home, but
probably more deliveries, with people coming to the door carrying parcels and going away again.
This is all particularly concerning for puppies acquired during the pandemic, as their expectation
of “normal” is lockdown life, and they may never have seen visitors inside the house or have been
left home alone.
We are all longing for a great British summer in which we can go on dog-date walks with a friend
and their dog, have family round for garden barbecues and take our pooches to the pub or cafe,
and of course, we need our dogs to be able to cope calmly with all of that. A return to normal is
something humans are able to process, understand and prepare for. But our dogs – especially
young ones – won’t understand why everything has changed. As far as our dogs know, normality
for them has been enjoying time with family only – so to be expected to cope with groups of
people, children and other dogs, both in and out of the home, could be overwhelming for them.
(Adapted from https://www.theguardian.com/commentisfree/2021/apr/05/bought-dog-lockdown)

Se você comprou um cachorro durante o confinamento, eles precisarão de ajuda para resolvê-lo
Problemas comportamentais são um motivo comum para realocação, então prepare-os para
quando você não estiver lá 24 horas por dia, 7 dias por semana
Apesar das baixas do ano passado, para muitos o confinamento tem sido a oportunidade perfeita
de dar as boas-vindas a um cão em suas vidas. A demanda por cães durante a pandemia tem sido
enorme, com um aumento de 60% nas ligações de pessoas que buscam adotar da instituição de
caridade Dogs Trust, e com muitas outras organizações de resgate relatando descobertas
semelhantes. As pesquisas no Google por “comprar um cachorro” aumentaram 115% depois que
o Reino Unido foi bloqueado pela primeira vez em março de 2020, com os preços de algumas das
raças mais procuradas atingindo níveis recordes.
Ser dono de um cachorro é uma coisa maravilhosa, mas também é uma responsabilidade enorme
e um compromisso que vai além do bloqueio: como diz o ditado, “Um cachorro é para a vida”. À
medida que as restrições diminuem e a retomada da normalidade começa, é importante
considerarmos as implicações para nossos companheiros caninos e ajudá-los a se ajustar.
Ter um cachorro por perto tem ajudado muitas pessoas a lidar com o bloqueio. Nossos cães
também adoram que fiquemos por perto: dar mais passeios, ter mais tempo para brincar e
descansar ao nosso lado. No entanto, é seguro dizer que a vida não tem sido normal para nossos
cães durante a maior parte do ano passado. Poucos conheceram outros cães e, se os viram, teria

AULA 06 – CONJUNCTIONS, DIRECT SPEECH AND REPORTED SPEECH 126


TEACHER ANDREA BELO

sido de longe ou com trela, o que significa que não puderam interagir ou brincar. Também houve
menos visitantes entrando em casa, mas provavelmente mais entregas, com pessoas chegando à
porta carregando pacotes e saindo novamente. Tudo isso é particularmente preocupante para
cachorros adquiridos durante a pandemia, já que sua expectativa de vida “normal” é uma vida
fechada e eles podem nunca ter visto visitantes dentro de casa ou ter sido deixados em casa
sozinhos.
Todos ansiamos por um ótimo verão britânico no qual possamos passear com um amigo e o
cachorro deles, receber a família para churrascos no jardim e levar nossos cachorros ao bar ou
café e, claro, precisamos dos nossos cachorros ser capaz de lidar com tudo isso com calma. O
retorno ao normal é algo para o qual os humanos são capazes de processar, compreender e se
preparar. Mas nossos cães - especialmente os mais jovens - não vão entender por que tudo
mudou. Até onde nossos cães sabem, a normalidade para eles é aproveitar o tempo apenas com
a família - então, esperar que lidem com grupos de pessoas, crianças e outros cães, dentro e fora
de casa, pode ser opressor para eles.

Is the U.S. COVID-19 Vaccine Rollout Working? We're Not Sure Yet
After four months and 171 million doses of COVID-19 vaccines administered across the U.S., more
than a few of us are eager to know: are the shots working?
Thus far, available evidence can half-answer that question: The vaccines are working well for
those who can get them. As soon as the rollout kicked off, a variety of researchers began
conducting what might be considered __________ (1) unofficial phase 4 clinical trial, monitoring
early vaccine recipients among the general public. Several of their studies have found a significant
reduction in both the rates of infection and severity of symptoms among vaccinated populations
compared __________ (2) similar demographics who are still awaiting a dose. On an individual
level, the vaccines appear to be working as well in real life as they did in controlled trials.
But there’s another part of that question: will mass vaccination hasten the end of __________ (3)
pandemic? On first pass, this might seem to be happening. As of April 7, nearly 20% of Americans
have received a completed regimen of either the two-dose Pfizer-BioNTech and Moderna
vaccines, or the one-shot Janssen variety. Meanwhile, the daily number of deaths in the U.S.
attributed to COVID-19, on a rolling seven-day average, is currently 611—a ghastly figure, but a
tremendous drop from the all-time post-holiday peak of 3,428 on Jan. 11. The last time the figure
was this low was on Jul. 4, 2020, just before deaths climbed again in the wake of a second wave
over the summer, from which the country never fully recovered before the much more
catastrophic third wave spurred by the holidays.
(Adapted from https://time.com/5953007/covid-19-mass-vaccination/)

A implementação da vacina COVID-19 nos EUA está funcionando? Ainda não temos certeza
Depois de quatro meses e 171 milhões de doses de vacinas COVID-19 administradas nos EUA,
muitos de nós estão ansiosos para saber: as injeções estão funcionando?
Até agora, as evidências disponíveis podem responder pela metade a essa pergunta: as vacinas
estão funcionando bem para aqueles que podem tomá-las. Assim que a implantação começou,

AULA 06 – CONJUNCTIONS, DIRECT SPEECH AND REPORTED SPEECH 127


TEACHER ANDREA BELO

uma variedade de pesquisadores começou a conduzir o que poderia ser considerado __________
(1) ensaio clínico não oficial de fase 4, monitorando os primeiros receptores da vacina entre o
público em geral. Vários de seus estudos encontraram uma redução significativa nas taxas de
infecção e na gravidade dos sintomas entre as populações vacinadas em comparação com
__________ (2) dados demográficos semelhantes que ainda aguardam uma dose. A nível
individual, as vacinas parecem funcionar tão bem na vida real como nos ensaios controlados.
Mas há outra parte dessa questão: a vacinação em massa apressará o fim da __________ (3)
pandemia? Na primeira passagem, isso pode parecer estar acontecendo. Desde 7 de abril, quase
20% dos americanos receberam um regime completo das vacinas Pfizer-BioNTech e Moderna de
duas doses ou da variedade Janssen de uma dose. Enquanto isso, o número diário de mortes nos
Estados Unidos atribuídas ao COVID-19, em uma média contínua de sete dias, é atualmente 611
- um número medonho, mas uma queda tremenda em relação ao pico pós-feriado de todos os
tempos de 3.428 em janeiro 11. A última vez que o número foi tão baixo foi em 4 de julho de
2020, pouco antes de as mortes subirem novamente na sequência de uma segunda onda durante
o verão, da qual o país nunca se recuperou totalmente antes da muito mais catastrófica terceira
onda estimulada por os feriados.

The machine always wins: what drives our addiction to social media
[1] We are swimming in writing. Our lives have become, in the words of the author and academic
Shoshana Zuboff, an “electronic text”. Social media platforms have created a machine for us to
write to. The bait is that we are interacting with other people: our friends, colleagues, celebrities,
politicians, royals, terrorists, porn actors – anyone we like. We are not interacting with them,
however, but with the machine. We write to it, and it passes on the message for us after keeping
a record of the data.
[2] The machine benefits from the “network effect”: the more people write to it, the more benefits
it can offer, until it becomes a disadvantage not to be part of it. Part of what? The world’s first ever
public, live, collective, open-ended writing project. A virtual laboratory. An addiction machine,
which deploys crude techniques of manipulation redolent of the Skinner Box created by
behaviourist BF Skinner to control the behaviour of pigeons and rats with rewards and
punishments. We are users, much as cocaine addicts are users.
[3] What is the incentive to engage in writing like this for hours each day? In a form of mass
casualisation, writers no longer expect to be paid or given employment contracts. What do the
platforms offer us, in lieu of a wage? What gets us hooked? Approval, attention, retweets, shares
and likes.
[4] Meanwhile, hashtags and trending topics underline the extent to which all of these protocols
are organized around the massification of individual voices – a phenomenon cheerfully described
by users with the science-fiction concept of the “hive mind” – and hype. The regular sweet spot
sought after is a brief period of ecstatic collective frenzy around any given topic. It doesn’t
particularly matter to the platforms what the frenzy is about: the point is to generate data, one of
the most profitable raw materials yet discovered. As in the financial markets, volatility adds value.
The more chaos, the better.

AULA 06 – CONJUNCTIONS, DIRECT SPEECH AND REPORTED SPEECH 128


TEACHER ANDREA BELO

[5] Whether or not we think we are addicted, the machine treats us as addicts. Addiction is, quite
deliberately, the template for our relationship to the Twittering Machine. Addiction is all about
attention. For the social media bosses, this is axiomatic.
[6] If social media is an addiction machine, the addictive behaviour it is closest to is gambling: a
rigged lottery. Every gambler trusts in a few abstract symbols – the dots on a dice, numerals, suits,
red or black, the graphemes on a fruit machine – to tell them who they are. In most cases, the
answer is brutal and swift: you are a loser and you are going home with nothing. The true gambler
takes a perverse joy in anteing up, putting their whole being at stake. On social media, you scratch
out a few words, a few symbols, and press send, rolling the dice. The internet will tell you who you
are and what your destiny is through arithmetic likes, shares and comments.
[7] The interesting question is what it is that is so addictive. In principle, anyone can win big; in
practice, not everyone is playing with the same odds. Our social media accounts are set up like
enterprises competing for attention. If we are all authors now, we write not for money, but for the
satisfaction of being read. Going viral, or trending, is the equivalent of a windfall. But sometimes,
winning is the worst thing that can happen. The temperate climate of likes and approval is apt to
break, lightning-quick, into sudden storms of fury and disapproval.
(Adapted from https://www.theguardian.com/technology/2019/aug/23/social-media-addiction-gambling)

A máquina sempre vence: o que impulsiona nosso vício em mídias sociais


[1] Estamos nadando na escrita. Nossas vidas se tornaram, nas palavras da autora e acadêmica
Shoshana Zuboff, um “texto eletrônico”. As plataformas de mídia social criaram uma máquina para
escrevermos. A isca é que estamos interagindo com outras pessoas: nossos amigos, colegas,
celebridades, políticos, membros da realeza, terroristas, atores pornôs - qualquer pessoa de quem
gostemos. Não estamos interagindo com eles, porém, mas com a máquina. Nós escrevemos para
ele e ele passa a mensagem para nós depois de manter um registro dos dados.
[2] A máquina se beneficia do “efeito rede”: quanto mais pessoas escrevem nela, mais benefícios
ela pode oferecer, até que se torne uma desvantagem não fazer parte dela. Parte de quê? O
primeiro projeto de escrita público, ao vivo, coletivo e aberto do mundo. Um laboratório virtual.
Uma máquina de vício, que implanta técnicas rudes de manipulação que lembram a Caixa de
Skinner criada pelo comportamentalista BF Skinner para controlar o comportamento de pombos
e ratos com recompensas e punições. Somos usuários, tanto quanto os viciados em cocaína são
usuários.
[3] Qual é o incentivo para escrever assim por horas todos os dias? Em uma forma de precarização
em massa, os escritores não esperam mais ser pagos ou receber contratos de trabalho. O que as
plataformas nos oferecem, em vez de um salário? O que nos vicia? Aprovação, atenção, retuítes,
compartilhamentos e curtidas.
[4] Enquanto isso, hashtags e tópicos de tendência sublinham a extensão em que todos esses
protocolos são organizados em torno da massificação de vozes individuais - um fenômeno
alegremente descrito pelos usuários com o conceito de ficção científica da "mente coletiva" - e
exagero. O ponto ideal comum procurado é um breve período de frenesi coletivo extático em
torno de qualquer tópico. Não importa particularmente para as plataformas o que é esse frenesi:

AULA 06 – CONJUNCTIONS, DIRECT SPEECH AND REPORTED SPEECH 129


TEACHER ANDREA BELO

o objetivo é gerar dados, uma das matérias-primas mais lucrativas já descobertas. Como nos
mercados financeiros, a volatilidade agrega valor. Quanto mais caos, melhor.
[5] Quer pensemos ou não que somos viciados, a máquina nos trata como viciados. O vício é,
deliberadamente, o modelo para nosso relacionamento com a Máquina de Twitter. O vício tem
tudo a ver com atenção. Para os chefes das redes sociais, isso é axiomático.
[6] Se a mídia social é uma máquina de vício, o comportamento viciante do qual ela está mais
próxima é o jogo: uma loteria fraudada. Todo jogador confia em alguns símbolos abstratos - os
pontos em um dado, numerais, naipes, vermelho ou preto, os grafemas em uma máquina de frutas
- para dizer quem eles são. Na maioria dos casos, a resposta é brutal e rápida: você é um perdedor
e está indo para casa sem nada. O verdadeiro jogador sente uma alegria perversa em apostar,
colocando todo o seu ser em jogo. Nas redes sociais, você risca algumas palavras, alguns símbolos
e pressiona enviar, lançando os dados. A internet vai te dizer quem você é e qual é o seu destino
por meio de curtidas aritméticas, compartilhamentos e comentários.
[7] A questão interessante é o que é tão viciante. Em princípio, qualquer um pode ganhar muito;
na prática, nem todos estão jogando com as mesmas chances. Nossas contas de mídia social são
configuradas como empresas competindo por atenção. Se agora somos todos autores, não
escrevemos por dinheiro, mas pela satisfação de sermos lidos. Tornar-se viral, ou tendência, é o
equivalente a uma sorte inesperada. Mas às vezes, vencer é a pior coisa que pode acontecer. O
clima temperado de gostos e aprovação tende a se transformar, rapidamente, em repentinas
tempestades de fúria e desaprovação.

Homelessness in the Living Rooms of the Rich


When photographer Jana Sophia Nolle moved to San Francisco three years ago, she met a lot of
people. Some inhabited multimillion-dollar houses with pristine Victorian architecture and fancy
furniture. Others lived in cardboard boxes.
The contrast between their dwellings plagued Nolle, who hails from Kassel, Germany, where
income inequality is less stark. While San Francisco boasts the highest density of billionaires per
capita in the world, the Bay Area hosts the country’s third-largest population of people
experiencing homelessness. The government shelters only a third of them.
“I’d never seen so many people living on the street in a country as rich as America,” Nolle says. “I
was shocked.” So, when an unhoused man she knew suggested, jokingly, that she invite him into
one of her wealthy friends’ homes, it sparked an idea: What if, instead, she pitched his tent there?
This provocative vision inspired her series Living Room, sending Nolle on a quest to photograph
the scrappy DIY shelters of the poor inside the immaculately styled parlors of the rich. “They’re
implants in rooms where they don’t belong,” she says.
Adapted from https://www.wired.com/story/san-francisco-shelters-living-room/

AULA 06 – CONJUNCTIONS, DIRECT SPEECH AND REPORTED SPEECH 130


TEACHER ANDREA BELO

Sem-teto nas salas dos ricos


Quando a fotógrafa Jana Sophia Nolle se mudou para São Francisco há três anos, ela conheceu
muitas pessoas. Alguns habitavam casas multimilionárias com arquitetura vitoriana imaculada e
móveis elegantes. Outros viviam em caixas de papelão.
O contraste entre suas moradias atormentou Nolle, que nasceu em Kassel, na Alemanha, onde a
desigualdade de renda é menos acentuada. Enquanto São Francisco ostenta a maior densidade
de bilionários per capita do mundo, a área da baía abriga a terceira maior população de pessoas
sem teto do país. O governo abriga apenas um terço deles.
“Nunca tinha visto tantas pessoas morando nas ruas de um país tão rico como os Estados
Unidos”, diz Nolle. "Fiquei chocado." Então, quando um homem sem casa que ela conhecia
sugeriu, brincando, que ela o convidasse para uma das casas de uma de suas amigas ricas, teve
uma ideia: e se, em vez disso, ela armou a barraca dele lá? Esta visão provocativa inspirou sua
série Living Room, enviando Nolle em uma missão para fotografar os abrigos DIY para os pobres
dentro dos salões de estilo imaculado dos ricos. “Eles são implantes em salas onde não
pertencem”, diz ela.

‘Tradwives’: the new trend for submissive women has a dark heart and history
A thrilling new trend _________ (1) emerged, and it’s as seismic as the New Look __________ (2)
in 40s fashion, or the emergence of Mediterranean cooking in 90s Britain: it’s called being a
housewife.
So a “tradwife” is a woman who doesn’t work so as to look after their children, their husband,
their home and then talk non-stop about how great this is on social media. Who knew being so
traditional was also so modern? And so busy! Last week alone, there were interviews with
tradwives in the Daily Mail, the Times and on the BBC, This Morning, Victoria Derbyshire and, for
all I know, piped 24/7 across all channels. I’m afraid that – being both non-trad and a non-wife – I
am less plugged in than these women.
The tradwives have been keenly giving interviews about how they are the true feminists in
choosing not to work… as much as the tradwives think they are being renegade rebels by not
working, their rebellion is based on their husband earning enough to support a whole household.
Whoa there, little rebels!
Adapted from https://www.theguardian.com/fashion/2020/jan/27/tradwives-new-trend-submissive-women-dark-heart-history

‘Tradwives’: a nova tendência para mulheres submissas tem um coração sombrio e uma
história
Uma nova tendência emocionante _________ (1) surgiu e é tão sísmica quanto o New Look
__________ (2) na moda dos anos 40, ou o surgimento da culinária mediterrânea na Grã-Bretanha
dos anos 90: é chamado de dona de casa.
Portanto, uma "esposa comercial" é uma mulher que não trabalha para cuidar dos filhos, do
marido, da casa e depois falar sem parar sobre como isso é bom nas redes sociais. Quem diria que

AULA 06 – CONJUNCTIONS, DIRECT SPEECH AND REPORTED SPEECH 131


TEACHER ANDREA BELO

ser tão tradicional também era tão moderno? E tão ocupado! Só na semana passada, houve
entrevistas com esposas comerciais no Daily Mail, no Times e na BBC, This Morning, Victoria
Derbyshire e, pelo que sei, transmitidas 24 horas por dia, 7 dias por semana, em todos os canais.
Eu temo que - sendo não-comerciante e não-esposa - estou menos conectado do que essas
mulheres.
As tradwives têm dado entrevistas agudamente sobre como são as verdadeiras feministas ao
escolher não trabalhar ... por mais que as tradwives pensem que são rebeldes renegadas por não
trabalharem, sua rebelião se baseia no fato de o marido ganhar o suficiente para sustentar uma
família inteira. Uau, pequenos rebeldes!

Pharmacogenomics can show what your body makes of a drug


Doctors have long appreciated that the same dose of medicine will not necessarily have the same
effect on different patients. One of the ways in which people differ biologically is in how they
metabolise drugs, a process largely dependent on enzymes that can vary a lot from person to
person. Differences in enzymes can lead two patients with the same disease, and the same
treatment, to end up with a five-fold difference in the amount of working drug molecules in their
blood.
Sometimes such differences will mean that the drug is not circulating at a high enough level to
work. Sometimes it is circulating at too high a level and causing toxic side-effects. Though side-
effects sound peripheral they are a huge medical problem. The study of how genes affect an
individual’s response to drugs is known as pharmacogenetics, and it is flourishing.
One example of the technique’s potential is found with warfarin, a commonly used blood thinner
that has what doctors call a “narrow therapeutic window”: the distance between a level in the
blood too low to do good and a level too high for safety is small. Variations in an enzyme gene
called CYP2C9, which allow warfarin to stay in the blood for longer, are quite common and this
makes it harder to find the best therapeutic dose. A test produced by a genetic-testing company
makes it possible to look at the genes for such enzymes and appears to limit side-effects and
improve clinical outcomes.
In chemotherapy, too, genetics can guide the choice of treatment in ways that other indicators
cannot. That matters since the drugs are so powerful. Some are so toxic that four patients in 100
will die from treatment, says David Kerr, a professor at Oxford University.
Adapted from https://www.economist.com/technology-quarterly/2020/03/12/pharmacogenomics-can-show-what-your-body-makes-of-a-drug.

A farmacogenômica pode mostrar o que seu corpo faz de uma droga


Os médicos há muito reconhecem que a mesma dose de medicamento não terá necessariamente
o mesmo efeito em pacientes diferentes. Uma das maneiras pelas quais as pessoas diferem
biologicamente é na forma como metabolizam as drogas, um processo amplamente dependente
de enzimas que podem variar muito de pessoa para pessoa. Diferenças nas enzimas podem levar
dois pacientes com a mesma doença, e o mesmo tratamento, a uma diferença de cinco vezes na
quantidade de moléculas de droga em funcionamento em seu sangue.

AULA 06 – CONJUNCTIONS, DIRECT SPEECH AND REPORTED SPEECH 132


TEACHER ANDREA BELO

Às vezes, essas diferenças significam que o medicamento não está circulando em um nível alto o
suficiente para funcionar. Às vezes, está circulando em um nível muito alto e causando efeitos
colaterais tóxicos. Embora os efeitos colaterais pareçam periféricos, eles são um grande problema
médico. O estudo de como os genes afetam a resposta de um indivíduo às drogas é conhecido
como farmacogenética e está florescendo.
Um exemplo do potencial da técnica é encontrado com varfarina, um anticoagulante comumente
usado que tem o que os médicos chamam de "janela terapêutica estreita": a distância entre um
nível no sangue muito baixo para fazer o bem e um nível muito alto para segurança é pequena .
Variações em um gene enzimático chamado CYP2C9, que permitem que a varfarina permaneça
no sangue por mais tempo, são bastante comuns e isso torna mais difícil encontrar a melhor dose
terapêutica. Um teste produzido por uma empresa de testes genéticos torna possível observar os
genes dessas enzimas e parece limitar os efeitos colaterais e melhorar os resultados clínicos.
Também na quimioterapia, a genética pode orientar a escolha do tratamento de uma forma que
outros indicadores não podem. Isso é importante, pois as drogas são muito poderosas. Alguns são
tão tóxicos que quatro em cada 100 pacientes morrem devido ao tratamento, diz David Kerr,
professor da Universidade de Oxford.

America's Long Overdue Awakening to Systemic Racism


Just before 7 p.m. on June 1, a deployment of local, state and federal forces, armored head to toe
in riot gear, unleashed rubber bullets and sprayed tear gas onto a crowd of peaceful demonstrators
gathered in the park to protest under the mantra “Black Lives Matter.” Since then, the debate over
systemic racism has spread across the nation and around the world.
Trump’s Administration has repeatedly denied that discrimination against black Americans is
embedded in the political, economic, and social structure of the country. Trump believes there are
“injustices in society,” but the notion that antiblackness is intrinsic to U.S. law enforcement has
been declared untrue. Attorney General William Barr warned against “automatically assuming
that the actions of an individual necessarily mean that their organization is rotten.”
But, for all that’s good about America, something is rotten. The protesters in Lafayette Park on
June 1 may have been galvanized by the disturbing video of the murder of George Floyd,
suffocated to death beneath the knee of a Minneapolis police officer. But at the core of their
movement is much more than the outrage over the latest instances of police brutality.
Centuries of racist policy have left black Americans in the dust. The U.S. may think it has brushed
chattel slavery into the dustbin of history after the Civil War, but the country never did a very good
job incinerating its traumatic remains, instead leaving embers that still burn today: an education
system that fails black Americans, substandard health care that makes them more vulnerable to
death and disease, and an economy that leaves millions without access to a living wage.
Adapted from https://time.com/5851855/systemic-racism-america/

AULA 06 – CONJUNCTIONS, DIRECT SPEECH AND REPORTED SPEECH 133


TEACHER ANDREA BELO

O despertar da América para o racismo sistêmico há muito tempo


Pouco antes das 19 horas em 1º de junho, uma implantação de forças locais, estaduais e federais,
blindadas da cabeça aos pés em equipamento anti-motim, dispararam balas de borracha e gás
lacrimogêneo em uma multidão de manifestantes pacíficos reunidos no parque para protestar sob
o mantra "Black Lives Matter". Desde então, o debate sobre o racismo sistêmico se espalhou por
todo o país e pelo mundo.
A administração de Trump negou repetidamente que a discriminação contra os negros americanos
está embutida na estrutura política, econômica e social do país. Trump acredita que há "injustiças
na sociedade", mas a noção de que o anti-negritude é intrínseco à aplicação da lei dos EUA foi
declarada falsa. O procurador-geral William Barr alertou contra "assumir automaticamente que as
ações de um indivíduo necessariamente significam que sua organização está podre."
Mas, apesar de tudo o que há de bom na América, algo está podre. Os manifestantes em Lafayette
Park em 1º de junho podem ter sido galvanizados pelo vídeo perturbador do assassinato de
George Floyd, sufocado até a morte sob o joelho de um policial de Minneapolis. Mas no centro de
seu movimento está muito mais do que a indignação com os últimos casos de brutalidade policial.
Séculos de política racista deixaram os negros americanos comendo poeira. Os Estados Unidos
podem pensar que jogaram a escravidão na lata de lixo da história após a Guerra Civil, mas o país
nunca fez um trabalho muito bom incinerando seus restos traumáticos, em vez de deixar brasas
que ainda queimam hoje: um sistema educacional que prejudica os negros americanos, abaixo do
padrão cuidados de saúde que os tornam mais vulneráveis à morte e doenças e uma economia
que deixa milhões sem acesso a um salário-mínimo.

AULA 06 – CONJUNCTIONS, DIRECT SPEECH AND REPORTED SPEECH 134


AFA 2024

LINKING WORDS (CONECTIVES) AND


QUESTION WORDS (WH QUESTIONS)

AULA 07

Teacher Andrea Belo

www.estrategiamilitares.com.br www.militares.estrategia.com
TEACHER ANDREA BELO

SUMÁRIO
INTRODUÇÃO 3

LINKING WORDS 4

CONJUNCTIONS 12

QUESTION WORDS/WH QUESTIONS 18

QUESTÕES 25

GABARITO 58

QUESTÕES COMENTADAS 59

CONSIDERAÇÕES FINAIS 117

REFERÊNCIAS BIBLIOGRÁFICAS 118

TRADUÇÕES 120

AULA 07 – LINKING WORDS (CONECTIVES) AND QUESTION WORDS (WH QUESTIONS) 2


TEACHER ANDREA BELO

INTRODUÇÃO
Desta vez, vamos à nossa aula sobre os tópicos que proporcionam ligação de ideias:
linking words ou connectors (termos de ligação/conectivos) e palavras especiais que levam à
possibilidade de fazer perguntas: questions words, que também são chamados de Wh questions.
Sobre linking words, é muito importante lembrar que, na aula 05, quando foram explicadas
as conjunções inseridas nas compound sentences, foram apresentadas a você inúmeras
conjunções com a função de linking words.
Em se tratando dos question words, muitos textos das provas de vestibular trazem
questionamentos sobre temas polêmicos, descobertas, assuntos para reflexão, entre outros,
utilizando esses termos nos textos propriamente ditos e, são essenciais à compreensão do que se
pergunta – se refere a um lugar, ao tempo, a uma pessoa, ao motivo ou a qualquer tipo de
informação que se pergunta.
Vamos estudar cada um deles e a situação em que são inseridos nas leituras que você vai
fazer e precisa conhecê-los.
Os tópicos que vamos estudar hoje são palavras necessárias para o vocabulário de quem
busca acertar o máximo possível de questões do vestibular.
Elas têm a função de unir, explicar, contrastar ou exemplificar frases e, são muito
importantes porque, com o uso de variados termos como os que estudaremos, as ideias são
expressas com mais exatidão nos textos, do jeito que o vestibular exige de você.
Como eu sempre digo, o seu objetivo é ser aprovado e para conseguir a sua aprovação, os
estudos e aprimoramento de conteúdo e assuntos se faz necessário e válido, sempre.
Vamos então estudar, praticar exercícios, tanto durante a teoria como também no fim do
material, treinando tudo o que aprendeu, em junção ao aprendizado de cada aula.

AULA 07 – LINKING WORDS (CONECTIVES) AND QUESTION WORDS (WH QUESTIONS) 3


TEACHER ANDREA BELO

LINKING WORDS
Os conectivos – linking words – são aquelas palavras ou expressões que servem para
estabelecer uma conexão lógica entre frases e elementos de um texto.
Muitas delas são conjunções ou locuções conjuntivas, mas há também muitos advérbios,
preposições, entre outros, que estão presentes nos textos no dia do vestibular. Veja um exemplo
com “according to”:

“I am not sure but, according to Peter, everybody is going to the party”.


“Não tenho certeza mas, de acordo com Peter, todos vão para a festa”

Esses termos podem ser identificados por diversos nomes: linking words, words of
transition, connectives, words of connection, logical connectors, transition devices, cohesive
devices, discourse markers ou até connective adjuncts.
São também conhecidos como articuladores, já que ligam ideias dentro de textos e, no
vestibular, conectam aquilo que as questões exigem que você descubra, através das análises que
sempre fazemos nas aulas e o conhecimento necessário que você precisa e estou proporcionando
mais para o seu sucesso.
O papel dos famosos linking words é estabelecer relações entre contextos – uma ideia
anterior e uma ideia posterior. Essas relações podem ser de muitos tipos, tais como exclusão,
concessão, adição, condição etc. O uso delas confere ao texto coerência e coesão.
É um assunto muito importante pois, uma vez que linking words são fundamentais para a
construção de sentenças, é, consequentemente, essencial na produção dos textos de jornais e
revistas, que são usados nas provas de vestibular, já que são termos que colaboram diretamente
com uma boa comunicação.
Lembrando que as linking words não sofre flexão de grau (aumentativo ou diminutivo),
número (singular ou plural) ou gênero (feminino ou masculino). Ou seja, são palavras invariáveis.
Linking words são fundamentais na língua inglesa, porque são como peças-chave na hora
da leitura dos textos para colaborar com o raciocínio da mensagem que as frases querem
proporcionar.
Para identificar os linking words nos textos que você vai ler, você precisa ter muito claro
qual é a ideia que o narrador quer expressar.
Como eu disse antes, a maioria desses termos ligam duas ideias numa sequência, como
também pode ligar duas ideias diferentes ou tentar dizer que uma coisa depende de outra.
Vejamos alguns exemplos com o que desejam expressar dentro do texto. Contudo, é
preciso entender que a lista de linking words é extensa e você deve, aula após aula, adicionar as
que aparecerem em uma lista de estudos e assim, aprender cada dia mais.

AULA 07 – LINKING WORDS (CONECTIVES) AND QUESTION WORDS (WH QUESTIONS) 4


TEACHER ANDREA BELO

EXEMPLIFICAÇÃO
Para exemplificar, uma expressão muito comum é o tão usado “for example” (por
exemplo), que também pode ser substituído por “for instance” e aparece em muitos textos do
vestibular.
There are many topics to study, for example, countable and uncountable nouns.
Há muitos tópicos para estudar, por exemplo, substantivos contáveis e incontáveis, que já estudamos.

Ou então, poderíamos dizer:


There are many topics to study, for instance, countable and uncountable nouns.
Há muitos tópicos para estudar, por exemplo, substantivos contáveis e incontáveis, que já estudamos.

Para exemplificar e dar ênfase em palavras, em ideias, também são usados outros linking
words, vistos no quadro abaixo e um exercício para treinar e visualizar melhor no contexto das
provas típicas de vestibular:
EXAMPLES / SUPPORT / EMPHASIS
in other words important to realize notably markedly in fact such as
to put it differently another key point including especially in general for example
for one thing first thing to remember like specifically in particular for instance
as an illustration most compelling evidence to be sure expressively in detail to point out
in this case must be remebered namely surprisingly to demonstrate with this in mind
for this reason point often overlooked chiefly frequently to emphasize
to put it another way on the negative side truly significantly to repeat
that is to say on the positive side indeed to clarify
with attention to certainly to explain
by all means surely to enumerate

Para ilustrar os linking words de exemplificação, vejamos um texto retirado de uma prova,
em que aparece “for instance”.
Se a questão abordasse os exemplos decorrentes do parágrafo em que a expressão em
questão aparece, poderia ser assim:

AULA 07 – LINKING WORDS (CONECTIVES) AND QUESTION WORDS (WH QUESTIONS) 5


TEACHER ANDREA BELO

Two in every three people on the planet–some 4 billion in total–are “excluded from the rule
of law.” In many cases, this begins with the lack of official recognition of their birth: around 40%
of the developing world’s five-year- old children are not registered as even existing.
Later, people will find that the home they live in, the land they farm, or the business that
they start, is not protected by legally enforceable property rights. Even in the rare cases when they
can afford to go to court, the service is poor. India, for example, has only 11 judges for every
1million people.
These alarming statistics are contained in a report from a commission on the legal
empowerment of the poor, released on June 3rd at the United Nations. It argues that not only are
such statistics evidence of grave injustice, they also reflect one of the main reasons why so much
of humanity remains mired in poverty. Because they are outside the rule of law, the vast majority
of poor people are obliged to work (if they work at all) in the informal economy, which is less
productive than the formal, legal part of the economy.
The Economist, June 7th 2008.

Questão – According to the sentence “... India, for example, has only 11 judges for every 1 million
people”, the underlined term refers to
(A) judges
(B) India
(C) 1 million people
(D) India population
(E) Every 1 million people
A resposta seria a letra A porque é a única que demonstra o exemplo exatos do que se trata a
referência de “for example” (por exemplo), logo após falar da India e, afirmar que há apenas 11
juízes para cada 1 milhão de pessoas.
Agora vejamos linking words de contraste a seguir.

CONTRASTE
Para contrastar, o linking word comum é o “but” (mas), que também pode ser substituído
por “however” e aparece em muitos textos do vestibular, inclusive, perguntando se pode haver a
devida substituição.

She loves going to the beach but never on Saturdays, it’s crowded.
Ela ama ir à praia, mas nunca aos sábados, é muito cheio.

Ou então, poderíamos dizer:

She loves going to the beach however she never goes on Saturdays, it’s crowded.
Ela ama ir à praia mas ela nunca vai aos sábados, é muito cheio.

AULA 07 – LINKING WORDS (CONECTIVES) AND QUESTION WORDS (WH QUESTIONS) 6


TEACHER ANDREA BELO

Podemos usar, também para contrastar, “despite” e “in spite of”, que são seguidas por
substantivos ou gerúndios.

Despite losing the match, the players celebrated their efforts.


Apesar de perder o jogo, os jogadores comemoraram o esforço.

Ou então, poderíamos dizer:

In spite of the lost match, the players celebrated their efforts.


Apesar da perda do jogo, os jogadores comemoraram o esforço.

Para exemplificar, com ideia de contraste, também muito usadas, vejamos o quadro:
OPPOSITION / LIMITATION / CONTRADICTION
although this may be true in reality although
in contrast after all instead
different from but whereas
of course ..., but (abd) still despite
on the other hand unlike conversely
on the contrary or otherwise
at the same time (and) yet however
in spite of while rather
even so / though albeit nevertheless
be that as it may besides nonetheless
then again as much as regardeless
above all even though notwithstanding

Para construções seguidas por sujeito e um verbo, precisa-se adicionar “the fact that”:

Despite the fact that they lost the match, the players celebrated their efforts.
Apesar do fato de perder o jogo, os jogadores comemoraram o esforço.

Ou

In spite of the fact that they lost the match, the players celebrated their efforts.
Apesar do fato de perder o jogo, os jogadores comemoraram o esforço.

Agora vejamos linking words com a função de resumir a seguir.

AULA 07 – LINKING WORDS (CONECTIVES) AND QUESTION WORDS (WH QUESTIONS) 7


TEACHER ANDREA BELO

RESUMO
Para resumir, há linking words comuns, tais como é o “in conclusion, in summary” (em
síntese/ em conclusão a, em suma), que são expressões geralmente usadas no começo das frases,
indicando que vamos resumir a ideia principal do que acabou de ser apresentada.

In conclusion, the meeting was very productive, and the ideas were well presented.
Em síntese, a reunião foi produtiva e as ideias foram bem apresentadas.

Ou então, poderíamos dizer:

In summary, the meeting was very productive, and the ideas were well presented.
Em síntese, a reunião foi produtiva e as ideias foram bem apresentadas.

ADIÇÃO
Para adicionar, os linking words comuns são “and” e “also” e, outras expressões
geralmente usadas para adicionar ideias de maneira mais formal seriam “furthermore” e
“moreover”, usadas bastante em textos de vestibular.

The meeting was very productive. Moreover lots of ideas were presented.
A reunião foi muito produtiva. Além disso, muitas ideias foram apresentadas.

Ou então, poderíamos dizer:

The meeting was very productive and lots of ideas were presented.
A reunião foi muito produtiva e muitas ideias foram apresentadas.

Para exemplificar, desta vez com ideia de adição, também muito usadas nas provas de vestibular,
vejamos o quadro abaixo e um exercício para treinar e visualizar melhor como nas provas:
AGREEMENT / ADDITION / SIMILARITY
in the first place by the same token too
not only ... But also again moreover
as a matter of fact to say nothing of as well as
in like manner and together with
in addition also of course
coupled with then likewise
in the same fashion / way equally comparatively
first, second, third identically correspondingly
in the light of uniquely similary
not to mention like furthermore
to say nothing of as a matter of fact additionally
equally important

AULA 07 – LINKING WORDS (CONECTIVES) AND QUESTION WORDS (WH QUESTIONS) 8


TEACHER ANDREA BELO

Para ilustrar mais uma vez os linking words de adição, vejamos um texto retirado do jornal
The Guardian, fonte de várias provas, em que aparece “and” muitas vezes no texto. E a questão
poderia ser assim:
Missing Out: In Praise of the Unlived Life is Adam Phillips's 17th book and is a characteristic blend
of literary criticism and philosophical reflection packaged around a central idea. The theme here
is missed opportunities, roads not taken, alternative versions of our lives and ourselves, all of
which, Phillips argues, exert a powerful hold over our imaginations. Using a series of examples and
close readings of authors including Philip Larkin and Shakespeare, the book suggests that a
broader understanding of life's inevitable disappointments and thwarted desires can enable us to
live fuller, richer lives. Good things come to those who wait.
Does he see himself as a champion of frustration? "I'm not on the side of frustration exactly, so
much as the idea that one has to be able to bear frustration in order for satisfaction to be realistic.
I'm interested in how the culture of consumer capitalism depends on the idea that we cant bear
frustration, so that every time we feel a bit restless or bored or irritable, we eat, or we shop.”
The Guardian guardian.co.uk, 1 June 2012. Adaptado.

Questão – Percebe-se a ideia de adição no primeiro parágrafo, em


(A) book, literary criticism, philosophical reflection.
(B) literary criticism, philosophical reflection, imaginations.
(C) book, literary criticism, alternative versions.
(D) book, literary criticism, central idea.
(E) literary criticism, philosophical reflection.
Comentários: A resposta seria a letra E porque a adição, representada desta vez pelo linking word
and, inúmeras vezes, mostra ideias de algo sendo adicionada a cada vez que aparece no texto,
conforme vimos exemplos acima. Perceba que, a única opção em que a ordem das palavras está
de acordo com o que é adicionado, ou seja, logo após book e o linking word “and”, é a letra B de
fato: “book... and ... a characteristic blend of literary criticismand philosophical reflection...”
Agora vejamos linking words com a função de dar a ideia de sequência a seguir.

SEQUÊNCIA
Para oferecer a ideia de sequência, há linking words fundamentais, tais como é o “first,
second, after, then, so”, entre outros, que são expressões geralmente usadas no começo das
frases, indicando que vamos resumir a ideia principal do que acabou de ser apresentada.

First, he decided to study. Then, he bought a good material and then dedicated a lot.
Primeiro, ele decidiu estudar. Daí, ele comprou um bom material e então dedicou-se muito.

AULA 07 – LINKING WORDS (CONECTIVES) AND QUESTION WORDS (WH QUESTIONS) 9


TEACHER ANDREA BELO

Veja uma imagem com a sequência lógica muito usada nos textos, de uma forma geral e
que, uma vez conhecendo-os, você conseguirá identificar ideias questionadas no dia da sua prova.

Em narrativas, os linking words organizam a história numa sequência de eventos,


facilitando para você na compreensão do texto durante sua leitura e busca de respostas às
perguntas apresentadas na prova de vestibular.
Os linking words funcionam como conectivos em todas as frases. Assim, elas
desempenham o papel de conectar ideias unindo termos ou mesmo orações. Mas, quando
oferecem a sequência, melhor ainda para localizar você no assunto e as partes dele enquanto está
sendo apresentado.
Por esse motivo, são elementos essenciais tanto na comunicação quanto na escrita, visto
que eles colaboram com a coesão e coerência textuais.
Há ainda, linking words que explicam a razão, a condição, a causa de algo, como podemos
ver no quadro abaixo e outros exemplos de outros linking words adiante:

Due to the heavy rain the flight was cancelled.


Devido/em decorrência da chuva forte, o vôo foi cancelado.

Ou então, poderíamos dizer:

Because of the heavy rain the flight was cancelled.


Por causa/em decorrência da chuva forte, o vôo foi cancelado.

AULA 07 – LINKING WORDS (CONECTIVES) AND QUESTION WORDS (WH QUESTIONS) 10


TEACHER ANDREA BELO

CAUSES / CONDITION / PURPOSE


in the event that if when because of in case in as much as
granted (that) then whenever as provided that
as / so long as unless since since given that
on (the) condition (that) while while only / even if
for the purpose of lest so that
with this intention so as to
with this in mind owing to
in the hope that due to
for fear that
in order to
seeing / being that
in view of

CONCLUSION / SUMMARY / RESTATEMENT


as can be seen after all overall
generally speaking in fact ordinarily
in the final analysis in summary usually
all things considered in conclusion by and large
as shown above in short to sum up
in the long run in brief on the whole
given these points in essence in any event
as has been noted to summarize in either case
in a word on balance all in all
for the most part altogether

TIME / CHRONOLOGY / SEQUENCE


at the present time immediately when formerly by the time instantuly
from time to time quickly onde suddenly whenever presently
sooner or later finally about shortly occasionally
at the same time after next hencefoth
up to the present time later now whenever
to begin with last now that eventually
in due time until meanwhile
until now till further
as soon as since during
as long as then first, second
in the meantime before in time
in a moment hence prior to
without delay forthwith
in the first place straightaway
all of a sudden
as this instant

AULA 07 – LINKING WORDS (CONECTIVES) AND QUESTION WORDS (WH QUESTIONS) 11


TEACHER ANDREA BELO

CONJUNCTIONS
Vimos, na aula anterior, uma introdução às conjunções, pois, na maioria delas, o papel
desempenhado é de conectar frases.
As conjunções, chamadas também de linking words, connectors ou conectivos são palavras
que ligam duas partes de uma oração, para que a sentença possa fazer sentido. Observe:
✓ I wanted to text you. (Eu queria te mandar uma mensagem.)
✓ I don’t have your number. (Eu não tenho o número do seu celular.)
Elas têm uma relação, mas falta algo para unir essas frases e fazê-las ter um sentido maior:
I wanted to text you, but I don’t have your number.
(Eu queria te mandar uma mensagem, mas eu não tenho o número do seu celular.)
Essa palavra but exerceu o papel que precisávamos: uniu as duas sentenças e estabeleceu
uma lógica entre elas. Portanto, o but é uma conjunção.

TIPOS DE CONJUNÇÕES
Há três tipos de conjunções em inglês:
✓ Conjunções Coordenadas (Coordinating Conjunctions)
✓ Conjunções Subordinadas (Subordinating Conjunctions)
✓ Conjunções Correlativas (Correlative Conjunctions)

CONJUNÇÕES COORDENADAS
As conjunções coordenadas são as mais comuns e as que geralmente usamos ou
identificamos quando pensamos em linking words. Elas têm o papel de juntar orações.
Elas podem juntar orações independentes (ou seja, orações que possuem sentido
completo por si próprias, sem precisar de outra oração para fazer sentido), frases ou apenas
palavras.
Na língua inglesa há sete conjunções coordenadas importantes:

For, And, Nor, But, Or, Yet, So

Vejamos as particularidades de cada uma delas.

AULA 07 – LINKING WORDS (CONECTIVES) AND QUESTION WORDS (WH QUESTIONS) 12


TEACHER ANDREA BELO

For [por]– Explica o motivo ou a proposta de algo (equivalente ao porquê).

I go to the park every week, for I love running.


[Eu vou ao parque toda semana, por amar correr.]
Peter though he had a great chance to be accepted at Oxford,
for his grandfather was the Dean of that university.
[Paul achava que tinha uma grande chance de ser aceito em
Oxford, por seu avô ser o reitor daquela Universidade.]

And [e] – Adiciona uma coisa à outra.

Daniel goes to the beach to surf and relax.


[O Daniel vai à praia para surfar e relaxar.]
I love red and white wine.
[Eu gosto de vinho branco e tinto.]

Nor [nem] – Utilizado para apresentar uma alternativa com ideia negativa à uma outra
ideia também negativa que já foi afirmada anteriormente.

The virus cannot live in immunized individuals, nor in the air.


[O vírus não pode viver em indivíduos imunizados, nem no ar.]
The guy didn’t have the chest of a body builder, nor did he have the six-pack abs.
[O cara não tinha o peitoral de um fisiculturista, nem tinha o abdômen tanquinho.]

But [mas] – Mostra contraste.

The game in the park is entertaining in the winter, but it’s better in the heat of summer.
[O jogo no parque é divertido no inverno, mas é melhor no calor do verão.]
She is very old but still works.
[Ela é muito velha, mas ainda trabalha.]

Or [ou] – Apresenta uma alternativa ou uma escolha.

Those men play on teams: shirts or skins.


[Aqueles homens jogam em times: com camiseta ou sem camiseta.]
Do you want a boy or a girl, Mom?
[Você quer um menino ou uma menina, mamãe?]

AULA 07 – LINKING WORDS (CONECTIVES) AND QUESTION WORDS (WH QUESTIONS) 13


TEACHER ANDREA BELO

Yet – Introduz uma ideia constratante que segue logicamente a ideia precedente, similar
ao “mas”.

I often take a book to read, yet I never seem to turn a single page.
[Eu frequentemente levo um livro para ler, mas parece que nunca viro uma só página.]
Dorian was the oldest of the girls, yet her accent was the most prominent.
[A Doriana era a mais velha das irmãs, mas seu sotaque era o mais proeminente.]

So [então, logo] – Indica efeito, resultado ou consequência.

I’ve started dating one of the soccer players, so now I have an


excuse to often watch the game.
[Eu comecei a namorar um dos jogadores de futebol, então agora
eu tenho uma desculpa para assistir aos jogos frequentemente.]
This is the easiest way to get there, so don’t say anything.
[Este é o caminho mais fácil para chegar lá, então não diga nada.]

Observe que as conjunções coordenadas geralmente ficam no meio de uma sentença e


uma vírgula é utilizada antes do linking word.
Com exceção se ambas as orações sejam muito curtas, neste caso a vírgula não será
utilizada.
Quando uma conjunção coordenada conecta duas orações independentes (ou seja, que
cada uma possui sentido sozinha, sem precisar da outra oração para fazer sentido), ela é
acompanhada da vírgula.
A vírgula será utilizada quando but expressar contraste.

CONJUNÇÕES SUBORDINADAS
Dos três tipos que há de conjunções, as subordinadas são as mais complexas de se
reconhecer, mas intuitivas de dominar.
As conjunções subordinadas introduzem as orações dependentes (ou seja, orações que
não possuem sentido completo por si próprias, elas precisam de outra oração para fazer sentido)
prendendo-as a uma oração independente (a que possui sentido completo por si só).
As conjunções subordinadas estabelecem uma relação de sentido entre a oração
dependente com o resto da frase. Há inúmeras conjunções subordinadas em inglês, as mais
comuns são:

AULA 07 – LINKING WORDS (CONECTIVES) AND QUESTION WORDS (WH QUESTIONS) 14


TEACHER ANDREA BELO

COMPARAÇÃO
POSSIBILIDADE LUGAR
CONTRASTE CAUSA E EFEITO TEMPO
CONDIÇÃO MODO
CONCESSÃO
ALTHOUGH BECAUSE AFTER AS IF AS IF
(Apesar, Embora) (Porque – nas respostas) (Depois) (Como se) (Como se)
EVEN THOUGH IN ORDER – THAT AS SOON AS ASSUMING THAT AS THOUGH
(Apesar de) (Para – que) (Assim que) (Assumindo que) (Como se)
RATHER THAN SINCE BEFORE EVEN IF HOW
(Ao invés de) (Desde) (Antes) (Mesmo se) (Como)
THAN SO THAT BY THE TIME IF NEXT
(Do que) (De modo que) (No momento que) (Se) (Próximo)
THOUGH WHY NOW THAT IN CASE – THAT WHERE
(Embora) (Por quê) (Agora que) (Em caso – que) (Onde)
WHERE AS ONCE ONLY IF WHEREVER
-
(Enquanto que) (Uma vez) (Somente se) (Onde quer que)
WHETHER SINCE PROVIDED THAT
- -
(Se) (Desde) (Devido a)
WHILE UNTIL UNLESS
- -
(Enquanto) (Até) (A menos que)
WHEN UNTIL
- - -
(Quando) (Até)
WHETHER
- - - -
(Se)

Exemplos:

Although I’ve been here before, he’s just too hard to forget.
Embora eu já tenha vindo aqui antes, ele é muito difícil de esquecer.

I guess I’ll never be the same since I won this medal.


Eu acho que eu nunca mais serei o mesmo desde que eu ganhei essa medalha.

They are watching TV while my mom is making dinner.


Eles estão assistindo TV enquanto a minha mãe está fazendo o jantar.

If you leave her, she will die.


Se você deixá-la, ela morrerá.

I’m proud of you because you passed the exam.


Eu estou orgulhoso de você porque você passou no exame.

Talk to me before you leave.


Fale comigo antes de partir.

Once you go there, you never forget it.


Uma vez que você vai lá, você nunca esquece.

When I see you smile, I can do anything.


Quando eu vejo você sorrindo, eu consigo fazer qualquer coisa.

AULA 07 – LINKING WORDS (CONECTIVES) AND QUESTION WORDS (WH QUESTIONS) 15


TEACHER ANDREA BELO

As orações podem ir em qualquer ordem, ou seja, tanto uma oração dependente como
uma independente podem começar a frase, mas o que nunca muda é que a conjunção
subordinada é a primeira palavra da oração dependente.

CONJUNÇÕES CORRELATIVAS
As conjunções correlativas estão sempre em grupo. Elas vêm em pares e você precisa
utilizar ambas em lugares diferentes em uma oração para fazer sentido.
Por esse motivo, elas têm esse nome justamente pelo fato delas trabalharem juntas (co-)
e por relacionar um elemento de uma sentença com outro (relação).
Sua correlação sempre denota igualdade, e mostra a relação entre as ideias expressas em
diferentes partes da sentença:

as . . . as [como . . . como]
both . . . and [ambos . . . e]
either . . . or [ou . . . ou]
hardly . . . when [dificilmente. . . quando]
if . . . then [se . . . então]
just as . . . so [assim como . . . assim]
neither . . . nor [nem . . . nem]
no sooner . . . than [não antes . . . do que]
not . . . but [não . . . mas]
not only . . . but also [não somente . . . mas também]
rather . . . than [em vez . . . do que]
scarcely . . . when [mal . . . quando]
what with . . . and [o que com . . . e]
whether . . . or [se. . . ou]

Vejamos alguns exemplos:

I didn’t know whether you’d want the pizza or hamburger, so I got you both.
Eu não sabia se você iria querer pizza ou hamburger, então eu peguei os dois para você.

I want either the pizza or the hamburger.


Eu quero ou a pizza ou o hamburger.

AULA 07 – LINKING WORDS (CONECTIVES) AND QUESTION WORDS (WH QUESTIONS) 16


TEACHER ANDREA BELO

Há ainda uma outra conjunção, chamada de conjunção adverbial.


Ela estabelece uma ideia de conjunção que une duas orações, entretanto, por ter valor de
advérbio, não é comum que ela apareça junto às demais conjunções.
As conjunções adverbiais mais comuns (apesar de pouco usadas) e que podem aparecer
em alguma prova de vestibular são:
SIMILARES A AND SIMILARES A BUT SIMILARES A SO
ALSO CONVERSELY ACCORDINGLY
(Também) (Inversamente) (Consequentemente)
BESIDES HOWEVER CONSEQUENTLY
(Além de) (Contudo) (Por conseguinte)
FURTHERMORE INTEAD OF HENCE
(Além disso) (Ao invés) (Portanto)
LIKEWISE NEVERTHELESS MEANWHILE
(Além disso) (Não obstante) (Enquanto isso)
OTHERWISE THEREFORE
(Caso contrário) (Portanto)
RATHER THUS
(Em vez) (Assim)
STILL
(Apesar)

AULA 07 – LINKING WORDS (CONECTIVES) AND QUESTION WORDS (WH QUESTIONS) 17


TEACHER ANDREA BELO

QUESTION WORDS/WH QUESTIONS


Os Question Words são pronomes interrogativos utilizados para elaborar perguntas em
Inglês. Eles são empregados antes dos verbos auxiliares e modais para se questionar algo.
Vale lembrar que, em Português, podemos transformar qualquer afirmação em pergunta
somente mudando a entonação, o que é diferente em Inglês, como você tem visto em nossas
aulas e praticamos bem isso na aula de verbos, com os devidos auxiliares de cada tempo verbal.
Muitas vezes, os question words são chamados Wh Questions porque eles contêm as letras
“W” e “H” em sua estrutura, veja: What (O quê/qual), Which (O quê/qual), When (Quando), Who
(Quem), When (Quando), Why (Por quê), HoW (Como), entre outros que estudaremos.

WHAT
What significa o quê/qual e é usado para perguntar sobre objetos, situações, assuntos
diversos e tudo aquilo que não sabemos. É o Wh question mais genérico de todos e,
consequentemente, o mais usado em textos dos mais variados tipos.
A pergunta feita com WHAT é geral, como abaixo, qual é o seu nome, endereço, o que você
gosta, que horas são e o que você faz, a resposta pode ser qualquer uma e não possui escolhas,
como entre duas ou mais coisas que você goste.
Se perguntar “o que você gosta?” – What do you like, a resposta pode ser o que vier em
sua mente, diferente de WHICH, que veremos a seguir.

WHAT'S
YOUR
NAME?

WHAT'S
WHAT DO
YOUR
YOU DO?
ADDRESS?

WHAT

WHAT WHAT DO
TIME IS IT? YOU LIKE?

AULA 07 – LINKING WORDS (CONECTIVES) AND QUESTION WORDS (WH QUESTIONS) 18


TEACHER ANDREA BELO

WHICH
Which também significa o quê/qual porém, é usado quando temos opções limitadas,
escolhas. Enquanto what é usado para perguntas de um modo geral, o which é usado quando são
oferecidas opções de respostas.
Por exemplo, a pergunta sobre o que você gosta – “What do you like?”, usando o which,
você provavelmente precisaria escolher algo que gosta, como: “Which do you like, pizza or ice-
cream?”, em que sua resposta tem que ser uma das duas ou mais opções.
Veja outros exemplos:

WHAT DO YOU EAT IN


THE MORNING? WHICH DO YOU EAT IN THE
MORNING, BREAD OR TOAST?

WHICH NAME
DO YOU
PREFER, TOM
OR JOHN?

WHICH JOB IS WHICH CITY


BETTER, WOULD YOU
BEING A LIVE, NEW
TEACHER OR YORK OR
A LAWYER? LONDON?

WHICH

WHICH KIND WHAT DO


OF MOVIE DO YOU LIKE IN
YOU LIKE THE
THE MOST, MORNING,
SUSPENSE OR MILK OR
DRAMA? JUICE?

AULA 07 – LINKING WORDS (CONECTIVES) AND QUESTION WORDS (WH QUESTIONS) 19


TEACHER ANDREA BELO

WHEN
When significa o quando e é usado para saber sobre tempo/período ou ocasião - quando
algo aconteceu, acontece ou irá acontecer.

WHEN IS
YOUR
BIRTHDAY?

WHEN ARE WHEN DID


YOU GOING YOU
TO DECIDE? GRADUATE?

WHEN?

WHEN WILL
WHEN DO
YOU
YOU GO TO
TRAVEL
THE GYM?
AGAIN?

WHERE
Where significa onde e é usado para saber sobre local, localização.

WHERE IS
YOUR
SCHOOL?

WHERE CAN WHERE DO


YOU LEAVE YOU WORK
YOUR CAR? OUT?

WHERE?

WHERE WHERE DID


WILL YOU YOU GO
GRADUATE? YESTERDAY?

AULA 07 – LINKING WORDS (CONECTIVES) AND QUESTION WORDS (WH QUESTIONS) 20


TEACHER ANDREA BELO

WHY
Why significa porquê e é usado para saber o motivo, a razão de algo acontecer, ter
acontecido ou porque vai acontecer. A resposta é sempre because – why para perguntar e
because para responder.

WHY ARE
YOU TIRED?
BECAUSE I
STUDIED A
LOT.

WHY ARE WHY DO YOU


YOU STUDY?
LEAVING? BECAUSE I
BECAUSE I WANT TO BE
HAVE TO GO. APPROVED.

WHY?

WHY WILL WHY DID


YOU YOU GET
TRAVEL? LATE?
BECAUSE IT'S BECAUSE OF
MY THE
VACATION. TRAFFIC.

WHO/WHOM
Who e Whom significam quem, para saber sobre pessoas mas, são usados em diferentes
situações – enquanto who tem a função de sujeito, whom tem a função de objeto, como vou
mostrar abaixo.
Se a pergunta for “Quem é o ator principal desse filme?”, é “Who is this movie main actor?”
(a resposta do who será o ator, que é o sujeito da pergunta).
Mas, se a pergunta for “Sobre quem é esse filme?”, é “Whom is this movie about?” (a
resposta será sobre quem é o filme, sendo o sujeito da frase o filme e não sobre quem é). Sobre
quem é o objeto. I o whom faz exatamente esse papel: de objeto direto ou indireto nas frases.
E ainda podem aparecer, nos textos do vestibular, perguntas com a preposição “to” no
final, como por exemplo: “To whom was she talking?” (Com quem ela estava falando?), também
na função de objeto. Vejamos exemplos.

AULA 07 – LINKING WORDS (CONECTIVES) AND QUESTION WORDS (WH QUESTIONS) 21


TEACHER ANDREA BELO

WHO ARE
YOU
TALKING
TO?

WHO IS THE
AUTHOR? WHO IS
WHOM IS YOUR
THIS STORY TEACHER?
ABOUT? WHO?
WHOM?

WITH
TO WHOM
WHOM DID
SHOULD I
YOU GO
TALK?
OUT?

WHOSE
Whose significa de quem e é usado para saber quem é o dono de algo, para saber a quem
pertence alguma coisa. É sempre seguido de um nome e um verbo.

WHOSE
PEN IS
THIS?

WHOSE
WHOSE
TEACHE
BOOK IS
R IS THE
THIS?
BEST?
WHOSE?

WHOSE WHOSE
CARDS ARE
ARE THOSE
THOSE? BOOKS?

AULA 07 – LINKING WORDS (CONECTIVES) AND QUESTION WORDS (WH QUESTIONS) 22


TEACHER ANDREA BELO

HOW
Usamos a Question Word HOW (como) quando queremos descrever a forma como algo é
feito e a condição ou qualidade. Veja alguns exemplos abaixo e outros para melhor compreensão.

HOW ARE
YOU DOING?

HOW MANY
HOW FAST IS STUDENTS?
THAT BUS? HOW MUCH
MONEY?

HOW HIGH IS
HOW OLD
THE
ARE YOU?
MONUMENT?

HOW?

HOW OFTEN HOW FAR IS


DO YOU YOUR
STUDY? WORK?

HOW BIG IS
HOW TALL
YOUR
ARE YOU?
HOUSE?

Outros exemplos:
How was your class? – Como está sua aula?
How do you cook vegetables? – Como você cozinha verduras?
How do you know his name? – Como você sabe o nome dele?
How can I speak English fast? – Como eu posso falar Inglês rápido?

How old: usado para perguntar a idade de alguém ou algo.


How old is you mother? Quantos anos tem sua mãe?
How old is this building? Quantos anos tem esse edifício?

AULA 07 – LINKING WORDS (CONECTIVES) AND QUESTION WORDS (WH QUESTIONS) 23


TEACHER ANDREA BELO

How long: usando para perguntar há quanto tempo, quanto tempo.


How long have you been studying? Há quanto tempo você tem estudado?

How far: usando para perguntar a distância entre uma coisa e outra.
How far is the hotel from the school? Qual é a distância entre o hotel e a escola?

How many: quantos – usado para substantivos contáveis, quando o plural é possível.
How many students can you see? Quantos alunos você consegue ver?

How much: quanto – usado para substantivos incontáveis, quando o plural não é possível.
How much money do you need? Quanto de dinheiro você precisa?

Preparei uma tabela para você:


HOW MANY ...? HOW HIGH ...?
(Quanto ...?) (Qual a altura ...?) > para objetos, seres inanimados
HOW MUCH ...? HOW DEEP ...?
(Quantos ...?) (Qual a profundidade ...?)
HOW OLD ...? HOW FAST ...?
(Quanto anos ...?) (Qual a velocidade ...?)
HOW FAR ...? HOW WELL ...?
(Qual a distância ...?) (Quão bem ...?)
HOW LONG ...? HOW BIG ...?
(Quanto tempo ...?) (Qual o tamanho ...?)
HOW OFTEN ...? HOW COME ...?
(Qual a frequência ...?) (Como ...?) > informal
HOW TALL ...?
(Qual a altura ...?) > para pessoas

AULA 07 – LINKING WORDS (CONECTIVES) AND QUESTION WORDS (WH QUESTIONS) 24


TEACHER ANDREA BELO

QUESTÕES
Você vai, agora, responder questões selecionadas de provas já realizadas em anos
anteriores. Depois, como em todas as nossas aulas, haverá o gabarito e as questões comentadas.
Vamos começar com questões AFA, de acordo coma sua instituição escolhida e depois,
vamos treinar de outras Carreiras Militares, para adquirir experiência e treinar vocabulário.
QUESTÕES AFA
Read Text I to do questions 01, 02 and 03 based on it
Text I
February’s Gonna Be a Big Month for Mars On the 9th, the first of three spacecraft will arrive
at the Red Planet and inaugurate a new era of Martian exploration.

_______ FEBRUARY 9, the United Arab Emirates’ Hope spacecraft is expected to enter orbit
around Mars after a six-month, 300-million-mile journey from Earth. It will mark the beginning of
a historic month for the Red Planet, which will see three separate national missions enter orbit or
touch down on the surface. Two of the countries behind these missions, the UAE and China, will
be visiting Mars for the first time; they will become the fifth and sixth countries to pull off that
feat, respectively. The third mission, launched _______ NASA, is expected to become the United
States’ 15th mission to successfully orbit or land on Mars.
The UAE is the only country that will not attempt a soft landing during the February Mars invasion.
Instead, its Hope orbiter will study the Martian atmosphere from more than 12,000 miles above
the surface. Planetary scientists hope that the UAE’s robo-meteorologist will fill in gaps in our
understanding of the Martian climate and help validate environmental data captured by rovers
and landers on the ground. For the country’s first foray into deep-space exploration, the UAE
space agency worked with an international team of researchers at the University of Colorado,
Boulder, to help plan the mission and build the spacecraft.
“There’s really no point in exploring outer space without adding to knowledge, and we’ve never
run a science mission,” Sarah bint Yousef Al Amiri, the UAE minister of state for advanced sciences
and science lead for the Emirates Mars Mission, said during a press conference last week. “It
wasn’t an easy journey, but it was such an enjoyment to rethink how you develop a planetary
exploration mission.”

AULA 07 – LINKING WORDS (CONECTIVES) AND QUESTION WORDS (WH QUESTIONS) 25


TEACHER ANDREA BELO

The Hope spacecraft will be the first new orbiter around Mars since the European Space Agency’s
ExoMars spacecraft arrived in 2016, but it won’t be the newcomer for long. China’s Tianwen-1
mission—which is a lander, rover, and orbiter rolled up into one—is expected to arrive less than
a day later. China’s space agency has been quiet about its plans for visiting the Red Planet, but
the craft is expected to attempt a landing shortly after it achieves orbit.
Unlike NASA’s car-sized Mars rovers Curiosity and Opportunity, China’s Tianwen-1 rover is small
enough to stow away inside the stationary lander that will carry it to the surface. Once it has safely
touched down, the six-wheeled rover will detach itself from the lander and spend the next three
months exploring its landing site, Utopia Planitia, the planet’s largest impact crater. The rover and
lander will both relay data _______ the surface to the Tianwen-1 orbiter, which will send it back
to Earth. Although the Chinese National Space Administration hasn’t provided a lot of details
about the exact scientific goals of its mission, a paper about it published last year in Nature
Astronomy says the agency’s goal is to “perform a global and extensive survey of the entire
planet.”
On February 18, a little more than a week after this robotic delegation arrives, NASA’s
Perseverance rover is expected to touch down. This will involve a harrowing descent to the
surface, during which the rover must reduce its speed from more than 10,000 miles an hour to
just a few feet per second over the course of 15 minutes. The descent will end with some aerial
acrobatics, during which a rocket-powered sky crane will gently deposit the rover on the surface
while hovering a few dozen feet above the ground.
“Don’t let anybody tell you different—landing _______ Mars is hard to do,” John McNamee,
project manager for the Perseverance mission at NASA’s Jet Propulsion Laboratory, said in a
statement. “But the women and men on this team are the best in the world at what they do.
When our spacecraft hits the top of the Mars atmosphere at about three and a half miles per
second, we’ll be ready.”
Perseverance is essentially a nuclear-powered self-driving car, and its primary mission is to collect
samples that will be picked up by another spacecraft later this decade and returned to Earth. With
any luck, this red dust will contain evidence that Mars once hosted microbial life. But whether
scientists will recognize extraterrestrial life when they see it remains an open question. Aside from
hunting for aliens, Perseverance will also enable a first-of-its-kind technology demonstration
involving a small helicopter called Ingenuity. A few days after landing, Perseverance will jettison
the helicopter in a clearing where it will attempt several short flights. If it works, it will be the first
time an aircraft has flown on another planet.
The arrival of three national missions on Mars within two weeks of one another is a historic
moment in the history of space exploration. It underscores the rapid development of space
capabilities around the world and the true internationalization of planetary exploration. Mars was
once the exclusive stomping grounds of the United States and the Soviet Union, but it is now also
an accessible destination for the European Union, Japan, India, the UAE, and China. Getting to
Mars is still a major challenge—historically only 40 percent of Mars missions have been
successful—and there’s no guarantee that all three missions will succeed in their objectives. But
launching a trio of spacecraft to our closest planetary neighbor is a major achievement and bodes
well for the future of space exploration.
(Adapted from https://www.wired.com/story/februarys-gonna-be-a-big-month-for-mars/)

AULA 07 – LINKING WORDS (CONECTIVES) AND QUESTION WORDS (WH QUESTIONS) 26


TEACHER ANDREA BELO

QUESTÃO 01 (AFA/INÉDITA) – Which is the correct option to complete the gaps in the text?
(A) On – by – from – on
(B) At – to – from – by
(C) On – by – to – by
(D) In – by – on – by
(E) In – to – to – on

QUESTÃO 02 (AFA/INÉDITA) – According to the text, which option is correct?


(A) The UAE and China had already visited Mars before, five or six times.
(B) The UAE is not going to land on Mars. Rather than, its Hope orbiter will study the Martian
atmosphere from space.
(C) The United Arab Emirates’ Hope spacecraft will be studying mars soil for 6 months.
(D) About six countries have already accomplished the feat of going to mars.
(E) The arrival of five national missions to Mars two weeks apart is a historic moment in the
history of space exploration.

QUESTÃO 03 (AFA/INÉDITA) – “Although the Chinese National Space Administration hasn’t


provided a lot of details about […]”. The highlighted word is closest in meaning to
(A) However
(B) Despite
(C) Since
(D) Even though
(E) Besides

Read Text ll to do question 04 based on it


Text ll
IN LOS ANGELES, the corner of Melrose and Harper has become a tourist destination to rival the
Eiffel Tower, or the graffitied remains of the Berlin Wall. Rather than an architectural marvel or a
piece of living history, people line up (or did, in pre-Covid times) to visit the bright pink exterior
wall of Paul Smith, a clothing retailer. The wall—repainted every three months in the Pantone
shade “Pink Ladies”—is the background to hundreds of thousands of photos, making it one of the
most Instagrammed places in Los Angeles, and even the world.
(Adapted from https://www.wired.com/story/fake-famous-review-instagram-influencers-documentary/)

AULA 07 – LINKING WORDS (CONECTIVES) AND QUESTION WORDS (WH QUESTIONS) 27


TEACHER ANDREA BELO

QUESTÃO 04 (AFA/INÉDITA) – Mark the option that correctly substitutes the expression rather
than (line 02).
(A) Instead of.
(B) Despite of.
(C) As well as.
(D) In addition to.
(E) At last.

Read Text lll to do questions 05 and 06 based on it


Text lll
Russian Fabergé exhibition contains 'at least 20 fakes', expert says
Andre Ruzhnikov accuses director Mikhail Piotrovsky of ‘destroying the authority of the museum’
A Russian museum __________ to close a Fabergé exhibition that contains items loaned from the
personal collection of a billionaire, after a prominent expert said it contained more than a dozen
fakes.
In a letter, the art dealer Andre Ruzhnikov accused the Hermitage Museum’s director, Mikhail
Piotrovsky, of “destroying the authority of the museum” by hosting the Fabergé: Jeweller to the
Imperial Court show, which runs until 14 March.
Ruzhnikov told the Guardian that the exhibition included at least 20 fakes, and that he thought
the exhibition, which is the first big Fabergé event at the St Petersburg institution since 1993,
should close immediately. He said: “I want the shame to end. I want this show to be closed and
forgotten, and that’s it. You cannot subject the Hermitage to such shame.”
The Hermitage and Alexander Ivanov have denied the claims and the billionaire produced
documents that support the authenticity of the items that were loaned from the Fabergé Museum
in Baden-Baden, which he established in 2009.
Piotrovsky and the Hermitage did not respond to a request for comment.
(Adapted from https://www.theguardian.com/artanddesign/2021/feb/01/russian-faberge-exhibition-contains-fakes-expert-says)

QUESTÃO 05 (AFA/INÉDITA) – Which is the correct option to complete the gap in the text?
(A) has asked
(B) had been asked
(C) had asked
(D) has been asked
(E) was asked

AULA 07 – LINKING WORDS (CONECTIVES) AND QUESTION WORDS (WH QUESTIONS) 28


TEACHER ANDREA BELO

QUESTÃO 06 (AFA/INÉDITA) – In the phrase “… a Fabergé exhibition that contains items loaned
from the personal collection of a billionaire” (paragraph 1), the underlined word is a synonym
for
(A) donated
(B) purchased
(C) acquired
(D) borrowed
(E) lent

Read Text lV to do questions 07, 08, 09 and 10 based on it


Text lV
When will offices be full again?
Maybe never, some executives say.
By Lauren Hirsch
Many companies do not expect their workers to return to offices until next summer, and even
then things may never be the same as before, judging by the comments executives made this
week, highlighted in today’s DealBook newsletter.
On earnings calls, executives from Goldman Sachs said that about a third of workers in New York
and London were coming in regularly; at JPMorgan Chase, it’s around 20 percent in both cities;
and Citigroup said “a small percentage” of employees in North America had returned.
“Being together enables greater collaboration, which is key to our culture,” said David M.
Solomon, Goldman’s chief. But Jamie Dimon of JPMorgan acknowledged that some working
habits may have changed permanently, which “will ultimately reduce the space you need for your
employees.” Terrance R. Dolan, the finance chief at U.S. Bancorp, told analysts that the bank will
most likely “consolidate” its corporate real estate to reflect “the new horizon.”
Is that a problem? Steven J. Goulart, the chief investment officer at MetLife, said at a regulatory
round table that the “pressure to de-densify” offices to support social distancing could support
demand for real estate even if buildings aren’t as full as before.
And as executives conduct more business remotely, going back to in-person meetings and pitches
seems less urgent. Natarajan Chandrasekaran, the chairman of Indian conglomerate Tata Sons,
said in an interview with The New York Times that he used to fly from India to the United States
to pitch a $50,000 project. But recently, he said, his firm’s consultancy business closed $2 billion
worth of deals in “five or six Zoom calls.”
There are other perks from working at home. BlackRock’s Laurence D. Fink is excited about what
employees could do with the time they save on daily commutes. “They could spend two hours
improving their health by exercising,” he said on a conference call. “They could spend two hours
more in building a deeper, stronger, more resilient family.”
Paul Draovitch of Duke Energy said at an investor event that working from home was “not without
risks,” but also brought certain benefits: “When my Pomeranians walk into the room, it's really a
pleasure.”
Adapted from: https://www.nytimes.com/2020/10/16/business/when-will-offices-be-full-again-maybe-never-some-executives-say.html

AULA 07 – LINKING WORDS (CONECTIVES) AND QUESTION WORDS (WH QUESTIONS) 29


TEACHER ANDREA BELO

QUESTÃO 07 (AFA/INÉDITA) – The correct form of the sentence “They could spend two hours
more in building a deeper, stronger, more resilient family.” in the indirect speech is:
(A) It said that they could have spent two hours more in building a deeper, stronger, more resilient
family.
(B) It said that they could spent two hours more in building a deeper, stronger, more resilient family.
(C) It said that they will spend two hours more in building a deeper, stronger, more resilient family.
(D) It said that they spend two hours more in building a deeper, stronger, more resilient family.
(E) It said that they have spended two hours more in building a deeper, stronger, more resilient
family.

QUESTÃO 08 (AFA/INÉDITA) – In “There are other perks from working at home” the underlined
word is closest in meaning to:
(A) goods
(B) benefits
(C) harms
(D) losses
(E) ways

QUESTÃO 09 (AFA/INÉDITA) – In the sentence “They could spend two hours improving their
health by exercising,” extracted from the text the pronoun they refers to:
(A) daily commutes
(B) BlackRock’s Laurence D. Fink
(C) Employees
(D) Many companies
(E) Family

QUESTÃO 10 (AFA/INÉDITA) – “They could spend two hours improving their health by
exercising,” The underlined word can be replaced by:
(A) coming up with
(B) puting up with
(C) puting away
(D) turning down
(E) brushing up on

AULA 07 – LINKING WORDS (CONECTIVES) AND QUESTION WORDS (WH QUESTIONS) 30


TEACHER ANDREA BELO

QUESTÕES COLÉGIO NAVAL


Read the text and answer the following questions 01 – 02.
Inflation: Used cars and food push US prices higher
Consumer prices jumped 5.4% in the 12 months to the end of June, up from 5% _______ previous
month.
It marks the biggest 12-month increase _______ August 2008, according to the US Labor
Department.
Inflation, which measures the rate at which cost of living increases, has been rising as the economy
reopens from coronavirus lockdowns.
It has sparked fears that prices _______ increasing too quickly, which could prompt the Federal
Reserve to push up interest rates or pull back on pandemic support earlier than expected.
However, some economists and the Federal Reserve say that the inflationary pressures will be
temporary.
Used vehicles accounted for one-third of the increase in the Consumer Price Index (CPI) _______
June, the Labor Department said _______ Tuesday.
But prices also reflected a broader surge in consumer demand as restrictions eased, with the costs
of meals in restaurants and cafes, hotel stays and airline tickets all rising last month.
(https://www.bbc.com/news/business-57573387)

QUESTÃO 01 (COLÉGIO NAVAL/INÉDITA) – In the title we can affirm that the prices are
(A) increasing
(B) dropping down
(C) decreasing
(D) at the same level
(E) in equal measures

QUESTÃO 02 (COLÉGIO NAVAL/INÉDITA) – Compete the gap in the sentence: “5% ____ previous
month.”
(A) a
(B) an
(C) the
(D) of
(E) no article

AULA 07 – LINKING WORDS (CONECTIVES) AND QUESTION WORDS (WH QUESTIONS) 31


TEACHER ANDREA BELO

QUESTÃO 03 (COLÉGIO NAVAL/INÉDITA) – Complete the blank space: “_____ August 2008”
(A) from
(B) with
(C) for
(D) since
(E) about

QUESTÃO 04 (COLÉGIO NAVAL/INÉDITA) – In the sentence, “Inflation, which measures the rate
cause
(…)”, the word in bold refers to
(A) rate
(B) cause
(C) the
(D) measures
(E) inflation

QUESTÃO 05 (COLÉGIO NAVAL/INÉDITA) – In, “which could prompt the Federal Reserve to push
up interest rates”. The underline verb gives an idea of
(A) obligation
(B) possibility
(C) ability
(D) advice
(E) permission

QUESTÃO 06 (COLÉGIO NAVAL/INÉDITA) – The correct statement according to the text is


(A) The cost of living is not a currently problem in the US.
(B) Specialists say that inflation will be temporary.
(C) Used vehicles were not affected by this situation.
(D) Food department didn’t have any inflation grow.
(E) The prices are getting down now.

AULA 07 – LINKING WORDS (CONECTIVES) AND QUESTION WORDS (WH QUESTIONS) 32


TEACHER ANDREA BELO

QUESTÃO 07 (COLÉGIO NAVAL/INÉDITA) – The word “however”, in bold in the text can be
replaced by
(A) also
(B) otherwise
(C) nevertheless
(D) so
(E) then

QUESTÃO 08 (COLÉGIO NAVAL/INÉDITA) – “It has sparked fears that prices ____ increasing too
quickly.” The correct verb form to fill in the blank.
(A) is
(B) to be
(C) will be
(D) are
(E) do

QUESTÃO 09 (COLÉGIO NAVAL/INÉDITA) – The expression, “coronavirus lockdowns”, means


I. Restriction of going and getting back. (Restrições de ir e vir)
II. The virus itself will lock people at home. (O virus por si mesmo trancou as pessoas em casa)
III. People are with some situations restricted by the global disease. (As pessoas estão com
alguma restrições pela doença global)
(A) All of them are correct.
(B) I and III are incorrect.
(C) II and III are correct.
(D) II is correct, but III is incorrect.
(E) I and III are correct.

QUESTÃO 10 (COLÉGIO NAVAL/INÉDITA) – “___ June, the Labor Department said ___ Tuesday.”
Complete the spaces with the suitable prepostitions
(A) in / on
(B) in / in
(C) on in
(D) at / in
(E) in / at

AULA 07 – LINKING WORDS (CONECTIVES) AND QUESTION WORDS (WH QUESTIONS) 33


TEACHER ANDREA BELO

QUESTÕES EAM
Read the following text to answer questions 01 to 07.
Lost Dog
Sam saw a dog on the sidewalk. The dog looked lost. Sam approached the dog and patted its head.
The dog wagged his tail. The dog had a collar. Sam looked at the collar and it read the dog's name.
Its name was Spike. Spike was a small dog and looked well-groomed. “Come on, Spike,” he said to
the dog. The dog followed him home. He brought the dog upstairs to his room. He had a jar of
dog food because he used to have a dog. He poured the dog food into a bowl and placed it in front
of Spike. Spike started eating. “I'll take care of you, Spike,” he said to the dog. He knew the dog
did not belong to him. He printed posters of Spike that said “Found Dog”. He went around hanging
the poster but nobody came to find Spike.
(Adapted from <https://www.eslfast.com/begin2/b2/b2014.htm>)

QUESTÃO 01 (EAM/INÉDITA) – Where did Sam most likely find Spike?


(A) At an animal shelter.
(B) At his home.
(C) On his front porch.
(D) On the street.
(E) At a market.

QUESTÃO 02 (EAM/INÉDITA) – Spike looked like


(A) a stray dog.
(B) it was not well-cared.
(C) a dog of a big breed.
(D) it was not lost.
(E) it had an owner.

QUESTÃO 03 (EAM/INÉDITA) – How many people reached Sam looking for Spike?
(A) zero
(B) many.
(C) two.
(D) some.
(E) much.

AULA 07 – LINKING WORDS (CONECTIVES) AND QUESTION WORDS (WH QUESTIONS) 34


TEACHER ANDREA BELO

QUESTÃO 04 (EAM/INÉDITA) – Did Sam feed the dog?


(A) No, he didn’t.
(B) No, he did.
(C) Yes, he did.
(D) Yes, he fed.
(E) Yes, he’s feeding.

QUESTÃO 05 (EAM/INÉDITA) – Read the following sentence. Sam looked at the collar and it
read the dog's name. The pronoun IT refers to:
(A) Sam.
(B) the dog.
(C) the collar.
(D) name.
(E) the dog’s name.

QUESTÃO 06 (EAM/INÉDITA) – Mark the option that is FALSE about the text.
(A) Sam was Spike’s dog.
(B) Spike was not Sam’s dog.
(C) Spike seemed lost.
(D) Spike was little.
(E) Spike went to Sam’s home.

QUESTÃO 07 (EAM/INÉDITA) – Read the following sentence. Its name was Spike. The possessive
adjective ITS refers to:
(A) the man.
(B) the dog.
(C) the dog’s owner.
(D) the collar.
(E) Sam.

QUESTÃO 08 (EAM/INÉDITA) – Which items of clothing can you see in the picture below?

AULA 07 – LINKING WORDS (CONECTIVES) AND QUESTION WORDS (WH QUESTIONS) 35


TEACHER ANDREA BELO

(A) shoes, t-shirt, dress, shorts.


(B) sneakers, shirt, dress, shorts.
(C) shoes, suit, dress, trousers.
(D) sneakers, tuxedo, blouse, shorts.
(E) shoes, shirt, skirt, pants.

QUESTÃO 09 (EAM/INÉDITA) – Mark the option that completes the sentences with the right
form of the verbs in parenthesis, respectively.
Lucy __________ (to go) to the gym every day, but she __________ (to go) this morning,
because she was __________ (to feel) sick.
(A) goes / went / feeling
(B) gos / didn’t go / felt
(C) going / went / feeling
(D) goes / didn’t go / feeling
(E) go / didn’t go / feels

QUESTÃO 10 (EAM/INÉDITA) – Mark the option that completes the blanks with the right
preposition of time or place.
I arrived __________ New York __________ noon.
(A) in / on
(B) at / at
(C) at / in
(D) on / at
(E) in / at

QUESTÕES EEAR
Read Text I to do questions 01, 02 and 03 based on it.
Text I
We must not let hope abandon us, because God, with his love, walks with us. “I hope, because
God is beside me”: we can all say this. Each one of us can say: “I hope, I have hope, because God
walks with me.” He walks and he holds my hand. God does not leave us to ourselves. The Lord
Jesus has conquered evil and has opened the path of life for us. Let us allow the Lord to teach us
what it means to hope. ___________, let us listen to the words of Sacred Scripture, beginning
with the prophet Isaiah, the great messenger of hope.
By Pope Francis

AULA 07 – LINKING WORDS (CONECTIVES) AND QUESTION WORDS (WH QUESTIONS) 36


TEACHER ANDREA BELO

QUESTÃO 01 (EEAR/INÉDITA) – “We must not let hope abandon us” The underlined modal verb
expresses the idea of:
(A) prohibition
(B) obligation
(C) possibility
(D) suggestion
(E) deduction

QUESTÃO 02 (EEAR/INÉDITA) – The pronoun his, (line 1), is classified as:


(A) object pronoun
(B) possessive pronoun
(C) possessive adjective
(D) subject pronoun
(E) reflexive pronoun

QUESTÃO 03 (EEAR/INÉDITA) – Fill in the blank with the suitable option:


(A) Besides
(B) Because
(C) Although
(D) Therefore
(E) However

Read text II and answer questions 04 and 05 based on it.


Text II

AULA 07 – LINKING WORDS (CONECTIVES) AND QUESTION WORDS (WH QUESTIONS) 37


TEACHER ANDREA BELO

QUESTÃO 04 (EEAR/INÉDITA) – What is the main verb tense used in the text above?
(A) Simple Past
(B) Past Perfect
(C) Simple Present
(D) Past Perfect Continuous
(E) Present Perfect Continuous

QUESTÃO 05 (EEAR/INÉDITA) – Choose the best alternative according to the dialogue in the
cartoon
(A) The boy does not seem irritated by the attitude of his cat and dog.
(B) According to the boy, Garfield was happy and thin because he had had a good breakfast.
(C) We can see in the text that the boy was being ironic and he was irritated by the attitude of
the dog and the cat.
(D) Garfiel fails to realize that he had done something wrong.
(E) The dog ate the boy's breakfast while garfield chewed up his slipper.

QUESTÃO 06 (EEAR/INÉDITA) – Choose the option with the correct tag questions for the
sentences below.
1) Listen to me, _________________?
2) Let's go to the movies, _______________?
3) I’m right, ________________?
(A) won’t you / won’t we / aren’t.
(B) won’t you / shall we / aren’t I.
(C) will you /shall we / aren’t I.
(D) will you / won’t we / am I not.
(E) will you / shan’t we / am I not.

Read text III and answer questions 07 and 08 based on it.


Text III
Adele – Someone Like You
I heard that you're settled down
That you found a girl and you're married now

AULA 07 – LINKING WORDS (CONECTIVES) AND QUESTION WORDS (WH QUESTIONS) 38


TEACHER ANDREA BELO

I heard that your dreams came true


Guess she gave you things, I didn't give to you
Old friend, why are you so shy?
Ain't like you to hold back or hide from the light
I hate to turn up out of the blue, uninvited
But I couldn't stay away, I couldn't fight it
I had hoped you'd see my face
And that you'd be reminded that for me, it isn't over
(Adapted from https://www.letras.mus.br/adele/1778689/)

QUESTÃO 07 (EEAR/INÉDITA) – What is the main verb tense used in the first paragraph of the
text above?
(A) Present Perfect.
(B) Past Perfect.
(C) Simple Present.
(D) Past Perfect Continuous.
(E) Simple Past.

QUESTÃO 08 (EEAR/INÉDITA) – In the excerpt “I hate to turn up out of the blue, uninvited” the
expression in bold means
(A) blue as the sky.
(B) unexpectedly.
(C) once in a blue moon.
(D) rarely.
(E) promptly.

QUESTÃO 09 (EEAR/INÉDITA) – Choose the option that contains only uncountable nouns
(A) beauty, iron, information, newspaper, baggage.
(B) beauty, bee, information, newspaper, wool.
(C) song, iron, information, paper, wool.
(D) song, bee, information, newspaper, baggage.
(E) beauty, iron, information, paper, wool.

AULA 07 – LINKING WORDS (CONECTIVES) AND QUESTION WORDS (WH QUESTIONS) 39


TEACHER ANDREA BELO

Read text IV and answer question 10 based on it.


Text IV
There’s no end to the benefits of donating blood for those who need it.
According to the American Red Cross, one donation can save as many as three lives, and someone
in the United States needs blood every two seconds.
It turns out that donating blood doesn’t just benefit recipients. There are health benefits for
donors, too, on top of the benefits that come from helping others.
Donating blood has benefits for your emotional and physical health. According to a report by the
Mental Health Foundation, helping others can: reduce stress, improve your emotional well-being,
benefit your physical health, help get rid of negative feelings, provide a sense of belonging and
reduce isolation.
Research has found further evidence of the health benefits that come specifically from donating
blood.

QUESTÃO 10 (EEAR/INÉDITA) – “According to a report by the Mental Health Foundation,


helping others can: reduce stress, […], help get rid of negative feelings […]”. The expression “get
rid of” in bold in the text means
(A) put away
(B) throw away
(C) put up with
(D) find out
(E) work out

QUESTÕES EFOMM
Online school to continue into next academic year amid risk of further disruption
A virtual school will stay open for at least most of the next academic year amid a rise in the number
of pupils selfisolating.
Oak National Academy has been used by pupils and teachers to support remote learning - which
has taken place during lockdowns and - during the Covid pandemic.
The Reach Foundation, which the school is part of, said it will keep operating at no cost for at least
the next two terms.
The charity said this will help teachers to plan for the start of the next academic year by being
able to access Oak National Academy’s online lessons and learning resources.
The announcement comes against a backdrop of rising Covid infections in the UK and the large
numbers of children self-isolating as the school year drew to a close.

AULA 07 – LINKING WORDS (CONECTIVES) AND QUESTION WORDS (WH QUESTIONS) 40


TEACHER ANDREA BELO

More than one million children were out of school last week due to Covid-related reasons - such
as a positive test, suspected Covid case or being told to isolate - just before schools broke up for
the summer holiday.
In the final weeks of term, over 300,000 pupils used Oak National Academy’s online learning
lessons and resources.
Schools are already considering what measures they may need to put in place should pupils test
positive, or they are asked to self-isolate in the next academic year.
The extension of the virtual school into the autumn and spring terms is backed by £2.1m in funding
from the Department for Education (DfE).
Ed Vainker from the Reach Foundation - which has incubated Oak since its launch - said the charity
understood teachers wanted the online school to continue.
“We are also mindful that Covid-19 will be with us in some form for the foreseeable future, and
the autumn and winter may bring further disruption,” the chief executive said.
“I’m therefore pleased the DfE will continue to support Oak, so it remains open and free as part
of resilience planning for the next two terms.”
Nick Gibb, the schools minister, said: “From being set up by a small group of teachers in under
two weeks, Oak National Academy has become one of the great success stories as we tackle the
pandemic.”
He said the online school helped millions of pupils and teachers while most children were not
allowed to set foot in school during a lockdown last spring and one at the start of this year.
But even since after all students were allowed back on since in March, Oak National Academy said
30,000 teachers have used its services a week, with demand rising amid the spread of the Delta
variant.
“The impact Oak has made and the good it has done for the sector and children is immeasurable,
and we will now look for the best way to harness that for the future,” Mr Gibb added.
https://www.independent.co.uk/news/education/education-news/uk-online-school-two-terms-b1888491.html

QUESTÃO 01 (EFOMM/INÉDITA) – In the extract from the text: “The announcement comes
against a backdrop of rising Covid infections in the UK [...]” (paragraph 5). The word in bold is
closest in meaning to
A) Scenery
B) Decrease
C) Increase
D) Scene
E) View

AULA 07 – LINKING WORDS (CONECTIVES) AND QUESTION WORDS (WH QUESTIONS) 41


TEACHER ANDREA BELO

QUESTÃO 02 (EFOMM/INÉDITA) – In the extract from the text: “We are also mindful that Covid-
19 will be with us in some form for the foreseeable future, and the autumn and winter may
bring further disruption” (paragraph 12), the verb “to bring” means
A) To refuse
B) To carry
C) To hold
D) To serve
E) To consider

QUESTÃO 03 (EFOMM/INÉDITA) – What is true according to the text?


A) Despite the increase in the number of self-isolated students, a virtual school has decided to
close its doors
B) Oak National Academy lost its popularity during the period of social isolation resulting from
the Covid-19 pandemic
C) More than a million students were out of school last week because of the summer break
D) The next school year will not need to have protective measures against Covid-19
E) Given the pandemic conditions, Oak will continue to act as a great help to students and
teachers

QUESTÃO 04 (EFOMM/INÉDITA) – In the extract from the text: “[…] which has incubated Oak
since its launch […] (paragraph 10), the word “which” refers to
A) Reach foundation
B) Oak
C) Ed Vainker
D) Teachers
E) Department for Education

Read the text below to answer question 05:


Cannabis part of the future says tobacco giant
The UK's largest tobacco firm says it __________ cannabis as part of its future as it tries to move
away from selling traditional cigarettes. British American Tobacco said it wanted to "accelerate"
its transformation by reducing the health impact of its products.
https://www.bbc.com/news/business-57995285

AULA 07 – LINKING WORDS (CONECTIVES) AND QUESTION WORDS (WH QUESTIONS) 42


TEACHER ANDREA BELO

QUESTÃO 05 (EFOMM/INÉDITA) – Fill in the blank with the suitable verb


A) Is seeing
B) Saw
C) Sees
D) Was seeing
E) Will see

Read the text below to answer questions 06 and 07:


A Belgian chocolate company is now ____ 3D printers which allow the company to create more
intricate, difficultto-mould chocolates. The chocolates are intended for people who seek original
designs.
The chocolate is melted before _____ poured into a syringe which is attached to the printer and
since the chocolates are hard to transport, the company _____ to open other shops around the
world.
https://www.newsinlevels.com/products/chocolate-3d-printing-level-3/

QUESTÃO 06 (EFOMM/INÉDITA) – Which is the correct way to complete the text above?
A) Using / being / hopes
B) Uses / being / hoped
C) Uses / being / is hoping
D) Using / being / was hoping
E) Using / being / had hoped

QUESTÃO 07 (EFOMM/INÉDITA) – In “…which allow the company to create more intricate…”,


the word in bold means
A) Hard
B) Intriguing
C) Fancy
D) Complex
E) Perplexing

QUESTÃO 08 (EFOMM/INÉDITA) – Which is the correct way to complete the paragraph below?
The small town of Pelhrimov in the Czech Republic _______ thousands of strange records and
hosted a festival to show them off. Some of the records were racing with folding ladders and
folding pink scarves. The president of the festival _______ that he was happy to have people come
and understand the ‘essence’ of what the people in this town _______. He wants to show that life
is more than being on Facebook or sitting at a computer. It is good to do something!
Adapted from: https://www.newsinlevels.com/products/town-has-strange-records-level-3/

AULA 07 – LINKING WORDS (CONECTIVES) AND QUESTION WORDS (WH QUESTIONS) 43


TEACHER ANDREA BELO

A) Holds / said / do
B) Holds / says / do
C) Hold / said / do
D) Holds / said / does
E) Holds / says / does

Read the text below to answer questions 09 and 10:


Jeff Bezos, _______ richest man ________ __ Earth, visited space on Tuesday. He and three other
crew members soared about 107 kilometers above the Texas desert aboard his company Blue
Origin´s New Shepard rocket.
_______ historic unpiloted sub-orbital flight helped to inaugurate a new era of private commercial
space tourism. The trip to the edge of space lasted about 10 minutes and 20 seconds, including a
few minutes of weightlessness. The crew included his brother Mark, Wally Funk, an 82-year-old
pioneering female aviator, and Oliver Daemen, an 18-year-old Dutch physics student.
Bezos, who recently stepped down as Amazon´s CEO, thanked employees and customers _______
the company that he founded, saying that their hard work and Amazon purchases had made his
space flight possible. His words sparked criticism, with some politicians saying that Amazon
abused its market power to hurt small businesses. They also said Bezos should have spent
_______ money to pay his workers fairly and protect their health.
https://www.newsinlevels.com/products/the-worlds-richest-man-visits-space-level-3/

QUESTÃO 09 (EFOMM/INÉDITA) – Fill in the blank with the suitable prepositions


A) A / in / a / of / the
B) The / on / an / of / a
C) The / in / a / of / the
D) A / on / a / of / the
E) The / on / a / of / the

QUESTÃO 10 (EFOMM/INÉDITA) – In “…an 82-year-old pioneering female aviator…”, the


expression in bold means
A) The first person in a row
B) The first person to do something
C) The last person to do something
D) The most important person to do something
E) The least important person to so something

AULA 07 – LINKING WORDS (CONECTIVES) AND QUESTION WORDS (WH QUESTIONS) 44


TEACHER ANDREA BELO

QUESTÕES EPCAR
Directions: Answer questions 01 to 05 according to TEXT
Why We Must Improve Vaccine Manufacturing Before the Next Pandemic
It should worry everyone that experts surveyed by TIME regarded both increasing funding in a
post-COVID-19 world for vaccine development and scaling up of manufacturing capacity
feasible—but improving equitable vaccine distribution was not.
To stop the next pandemic in its tracks we need to ensure that people all over the world are
protected quickly, and that will entail having all these pieces in place. The good news is, all these
elements are feasible, and indeed starting to work today.
On vaccine R&D, the Coalition for Epidemic Preparedness Innovations (CEPI), was set up with the
precise purpose of identifying and investing in R&D for vaccines against emerging infectious
diseases with epidemic potential. So, when it came to COVID-19, with CEPI’s and other R&D
support, as well as industry engagement, the scientific and vaccine manufacturing community
rallied, producing the first safe and effective vaccine in record time—just 327 days. Today we have
not just one but 15 in widespread use.
Increased investment now could get us there even faster the next time, particularly given the
potential of the relatively new RNA vaccine technologies that have proved so effective with
COVID-19. These plug-and-play vaccine technologies not only make it possible to identify and
develop antigens rapidly, but much of the regulatory testing and approval can be done in advance,
even before we know what the threat is.
As for manufacturing, it may be difficult to immediately discern when there are severe supply
shortages, but the world has actually rapidly built up manufacturing capacity during COVID-19.
Waiving intellectual property has been talked about a lot as a potential solution for boosting
production. But the growth we have seen in the past year has been achieved through technology
transfers, where both the intellectual property and the vital know-how needed to make vaccines
is shared between manufacturers.
However, we need to do more. Given the extremely large number of doses needed during a
pandemic, export bans of vaccines and essential components and supply bottlenecks have led to
a vaccine divide. Currently, more than a third of adults in high-income countries have now been
vaccinated, while less than 1% of those in low-income countries have had their first jab.
To prevent this kind of scenario from happening the next time round and ensure that those most
at risk are prioritized wherever they are, it is not distribution channels we are lacking, but global
manufacturing capacity. We already have highly effective distribution channels, through COVAX
and its partners, and we already have access to doses, enough to protect 1.8 billion people in
lower-income economies by early next year, enough to protect almost 30% of people in these
countries.
But through investments now to increase global manufacturing capacity, particularly in emerging
economies, and support of technology transfers, the next time a pandemic strikes we can get
there sooner.
(Adapted from https://time.com/6072101/improve-vaccine-manufacturing/)

AULA 07 – LINKING WORDS (CONECTIVES) AND QUESTION WORDS (WH QUESTIONS) 45


TEACHER ANDREA BELO

QUESTÃO 01 (EPCAR/INÉDITA) – Mark the option that can replace the word “improve” in the
title without changing its meaning
A) Blow
B) Help
C) Recuperate
D) Enhance

QUESTÃO 02 (EPCAR/INÉDITA) – We can deduce from the first paragraph that


A) Experts do not believe that increasing manufacturing capacity is feasible
B) Improving an equitable distribution of vaccines should be considered feasible
C) Investment in vaccine development is not appropriate in the post-pandemic world
D) There was an equitable distribution of vaccines in the COVID-19 pandemic

QUESTÃO 03 (EPCAR/INÉDITA) – Mark the option with the suitable question to answer the
underlined fragment below
“To stop the next pandemic in its tracks we need to ensure that people all over the world are
protected quickly, and that will entail having all these pieces in place”.
A) How will mass vaccination work in a possible next pandemic?
B) How can the next pandemic occur?
C) What can be done to stop a possible next pandemic from going forward?
D) What are the advantages of vaccination?

QUESTÃO 04 (EPCAR/INÉDITA) – Mark the option that can replace the underlined word keeping
the same meaning
“To stop the next pandemic in its tracks we need to ensure that people all over the world are
protected quickly, and that will entail having all these pieces in place”.
A) Exclude
B) Encompass
C) Indeed
D) Rid

QUESTÃO 05 (EPCAR/INÉDITA) – Mark the alternative that can answer the question below
according to the text
How was the COVID-19 vaccine development process?
A) It was unusually fast and effective
B) It was fast, like all vaccine development
C) It was time-consuming but effective
D) It was time-consuming and ineffective

AULA 07 – LINKING WORDS (CONECTIVES) AND QUESTION WORDS (WH QUESTIONS) 46


TEACHER ANDREA BELO

QUESTÕES ESA
Read the text and answer questions 01, 02 and 03
Children who ate ‘cannabis sweets’ risked serious harm, say Surrey police
Investigation begins after two boys, 12 and 13, and a girl, 12, were taken to hospital after falling unconscious
Four children who were taken to hospital after eating sweets they thought contained cannabis
are not believed to have suffered long-term effects, police have said.
The youngsters were found vomiting uncontrollably and falling in and out of consciousness on
Saturday afternoon on Pound Lane in Epsom, according to Surrey police. A 12-year-old girl, 12-
year-old boy and 13-year-old boy were taken to hospital by the South East Coast ambulance
service and are lucky not to have been more seriously affected by whatever they ate, the police
said.
They suffered “a violent reaction from eating the jelly sweets that they believed contained
cannabis”, police said.
Another 12-year-old boy was taken to hospital by his family because he was also believed to have
eaten the sweets.
One of the 12-year-old boys was discharged from hospital late on Saturday and the other three
were expected to leave on Sunday. The source of the sweets is still being investigated and police
do not have a clear idea of what they look like, although a similar report from Friday night involved
“jelly apple rings” which look like green jelly circles.
DS Lisa Betchley said: “These children were incredibly lucky that they were not more seriously
affected by whatever it was that they ate – and this is thanks, in great part, to the prompt actions
of two medical students who happened to be nearby and assisted in the early stages, as well as
the South East Coast ambulance service and hospital staff for their rapid response and treatment.
(Adapted from https://www.theguardian.com/uk-news/2021/may/02/children-who-ate-cannabis-sweets-risked-serious-harm-say-surrey-police)

QUESTÃO 01 (ESA/INÉDITA) – Concerning the information in the text, is correct to state that
(A) The impacts caused by the food eaten by the children are likely to be short-term
(B) The children's symptoms were severe headaches and nausea
(C) The content of the sweets is still completely unknown
(D) What happened to the children reached the worst possible scenario
(E) The children were saved, mainly, by the small amount of toxic food ingested

QUESTÃO 02 (ESA/INÉDITA) – In the sentence “One of the 12-year-old boys was discharged
from hospital late on Saturday and the other three were expected to leave on Sunday”
(paragraph 4), the word “discharged” means
(A) Accomplished
(B) Done
(C) Dismissed
(D) Retired
(E) Hospitalized

AULA 07 – LINKING WORDS (CONECTIVES) AND QUESTION WORDS (WH QUESTIONS) 47


TEACHER ANDREA BELO

QUESTÃO 03 (ESA/INÉDITA) – The words ATE and WERE are


(A) Regular verbs
(B) Not verbs
(C) Modal verbs
(D) From different grammatical classes
(E) Irregular verbs

Read the text and answer questions 04 and 05


Will Covid-19 vaccines reduce virus transmission?
Vaccinated people can still get infected, but they are less likely to pass it on
There are two ways that getting vaccinated can slow the spread of the virus. First, _______ (1)
can help prevent you getting infected. Second, even if you are unlucky and catch the virus, it may
reduce the risk of passing it on. It is crucial to understand how big these benefits are.
Two huge new studies have taken advantage of the successful UK vaccine rollout. An Oxford-ONS
analysis _______ (2) more than 370,000 survey participants found infections were reduced by
65% after a single dose. For protection against the virus, one dose was similar _______ (3) having
had a prior infection. There was no major difference between the two available vaccines.
Curiously, infection rates were lower up to three weeks before the jab. Did the virus have magical
pre-cognition and keep away? More plausible is the idea of “reverse causation”. People can have
the vaccination only if they have not tested positive or shown recent symptoms, so it is inevitable
there were fewer recorded infections before vaccinations took place. Statistics can be tricky
things.
Most important, the studies showed that if you are infected after vaccination, it tends to be much
milder, both in terms of self-reported symptoms and viral load.
If vaccinated people develop a weaker infection, then they might be less likely to pass on the virus.
(Adapted from https://www.theguardian.com/theobserver/commentisfree/2021/may/02/vaccinated-people-less-likely-to-pass-covid-on)

QUESTÃO 04 (ESA/INÉDITA) – Concerning the information in the text, is correct to say that
(A) The vaccine completely prevents the organism from being infected by the virus
(B) Vaccines have the potential to significantly reduce the rate of virus infection
(C) The first dose of the vaccine starts to take effect when the second dose is applied
(D) The two vaccines studied and cited in the text had significant differences in their results
(E) The drop in the rate of infections in the period prior to vaccine doses occurred by pure
coincidence

AULA 07 – LINKING WORDS (CONECTIVES) AND QUESTION WORDS (WH QUESTIONS) 48


TEACHER ANDREA BELO

QUESTÃO 05 (ESA/INÉDITA) – In the sentence “Did the virus have magical precognition and
keep away? More plausible is the idea of ‘reverse causation’” (paragraph 3), the word
“plausible” means
(A) Reasonable
(B) Improbable
(C) Irrational
(D) Partial
(E) Impartial

QUESTÕES ESCOLA NAVAL


Based on the text below, answer the six questions that follow it.
Any amount of alcohol consumption harmful to the brain, finds study
UK study of 25,000 people finds even moderate drinking is linked to lower grey matter density
There is no safe amount of alcohol consumption for the brain, with even “moderate” drinking
adversely affecting nearly every part of it, a study of more than 25,000 people in the UK has found.
The study, which is still to be peer-reviewed, suggests that the more alcohol consumed, the lower
the brain volume. In effect, the more you drink, the worse off your brain.
“There’s no threshold drinking for harm – any alcohol is worse. Pretty much the whole brain seems
to be affected – not just specific areas, as previously thought,” said the lead author, Anya Topiwala,
a senior clinical lecturer at the University of Oxford.
Using the UK Biobank, a substantial database designed to help researchers decode the genetic
and environmental factors that lead some people to develop diseases while others do not,
researchers in this study analysed data from 25,378 participants such as age, sex, education, self-
reported alcohol consumption, brain size and health from MRI scans, information about hospital
and outpatient visits, and memory tests.
Higher volume of alcohol consumption per week was associated with lower grey matter density –
the researchers found, with alcohol explaining up to a 0.8% change in grey matter volume, even
after accounting for individual biological and behavioural characteristics.
This might seem like a small figure, but it is a larger contribution than any other modifiable risk
factors. For example, it is four times the contribution of smoking or BMI, said Topiwala.
Widespread negative associations were also seen between alcohol consumption and integrity of
white matter, the brain fibres that scaffold the billions of neurons that make up grey matter. In
addition, an individual’s underlying conditions such as high blood pressure and high BMI made the
negative association between alcohol and brain health stronger, the researchers found.
Contrary to previous research that suggested there is a benefit to drinking wine in moderation
compared with beer or spirits, the study found no evidence to suggest alcoholic beverage type
conferred differences in risks to the brain.
(Adapted from https://www.theguardian.com/society/2021/may/18/any-amount-of-alcohol-consumption-harmful-to-the-brain-finds-study)

AULA 07 – LINKING WORDS (CONECTIVES) AND QUESTION WORDS (WH QUESTIONS) 49


TEACHER ANDREA BELO

QUESTÃO 01 (ESCOLA NAVAL/INÉDITA) – According to the text, which option is correct?


(A) Consumption of small amounts of alcohol is safe and beneficial to the brain
(B) Brain volume increases with increasing alcohol consumption
(C) Alcohol consumption, even in large quantities, affects only some parts of the brain
(D) There is a minimum limit of alcohol consumption so that it can cause damage to health
(E) Moderate alcohol consumption affects a good part of the brain

QUESTÃO 02 (ESCOLA NAVAL/INÉDITA) – What’s the meaning of the word “threshold” in


paragraph 3?
(A) Conclusion
(B) Brink
(C) End
(D) Point
(E) Disadvantage

QUESTÃO 03 (ESCOLA NAVAL/INÉDITA) – In paragraph 1, the word “it” refers to


(A) Moderate drinking
(B) Alcohol consumption
(C) Study
(D) Brain
(E) UK

QUESTÃO 04 (ESCOLA NAVAL/INÉDITA) – According to the text, which option completes the
sentence below correctly?
Alcohol consumption __________ to affect only some parts of the brain
(A) Was thought
(B) Is thought
(C) Has been thought
(D) Will be thought
(E) Is being thought

QUESTÃO 05 (ESCOLA NAVAL/INÉDITA) – According to the text, which option is correct?


(A) Smoking is what most negatively affects the brain
(B) Consuming more alcohol means increasing brain density
(C) The study presented in the text was based on data analysis
(D) An individual's underlying conditions do not interfere with the health impact of alcohol
(E) There are certain types of alcoholic beverages that affect the brain

AULA 07 – LINKING WORDS (CONECTIVES) AND QUESTION WORDS (WH QUESTIONS) 50


TEACHER ANDREA BELO

QUESTÃO 06 (ESCOLA NAVAL/INÉDITA) – What’s the meaning of the word “previous” in


paragraph 8?
(A) Prior
(B) First
(C) Current
(D) Future
(E) Timely

QUESTÃO 07 (ESCOLA NAVAL/INÉDITA) – Which is the correct option to complete the paragraph
below?
“___ Covid-19 crisis has been difficult on people across the globe, including India. In the past few
days there has been intense discussion ___ the decision of our government and Indian vaccine
manufacturers, including SII, ___ export vaccines”.
(Adapted from https://www.theguardian.com/world/2021/may/19/poorer-countries-face-long-delays-receiving-covid-vaccines)

(A) X / on / to
(B) The / in / to
(C) The / on / at
(D) X / on / at
(E) The / on / to

QUESTÃO 08 (ESCOLA NAVAL/INÉDITA) – Which option completes the dialogue below correctly?
Ed: _____ at the party yesterday?
Jack: Yes, didn't you see me?
(A) Was you
(B) Did you go
(C) Were you
(D) Do you go
(E) You were

QUESTÃO 09 (ESCOLA NAVAL/INÉDITA) – Which is the correct option to complete the paragraph
below?
“A few weeks ago, I __________ my first shot of a vaccine against Covid-19. As the newly
vaccinated exited the clinic, there was a mix of relief and elation on people’s faces. We exchanged
little smiles of solidarity. If we could have burst into spontaneous applause, I __________ sure we
would have done”.
(Adapted from https://www.theguardian.com/commentisfree/2021/may/17/animals-overlooked-allies-fight-against-covid-vaccines)

AULA 07 – LINKING WORDS (CONECTIVES) AND QUESTION WORDS (WH QUESTIONS) 51


TEACHER ANDREA BELO

(A) Received / am
(B) Received / was
(C) Receive / am
(D) Receive / was
(E) Received / will be

QUESTÃO 10 (ESCOLA NAVAL/INÉDITA) – Which word best completes the question below?
How _______ hours does it take to walk around the entire city?
(A) Much
(B) Many
(C) Often
(D) Very
(E) Lot of

QUESTÕES EsPCEx
Leia o texto a seguir e responda às questões 01, 02 e 03
Can New Zealand's tourism industry make a sustainable return?
New Zealand is hopeful that a recently opened travel bubble with Australia will rekindle its
pandemic-battered tourism industry. However, many are also seeing an opportunity to rethink
how to make the sector more climate friendly.
New Zealand's Queenstown - __________ (1) popular tourist spot - is throbbing with activity.
"To see and hear all the boats and the screaming and the complete joy… it puts a smile on all of
our faces," said Jolanda Cave, the general manager at Shotover Jet - one of the most established
adventure companies there.
It's a busy time __________ (2) the company, named after the river, where for more than half a
decade, boats have been whizzing, spinning and splashing to the delight of tourists.
But even so, the numbers it is seeing are a fraction of what it was used to before the pandemic
closed the country's borders. It used to operate eight boat rides an hour. Sometimes, that was
down to just one.
"It's been real eye opener for us to go from 1,200 (visitors) a day to 200 a day," Ms Cave said.
Like many tourism operators across New Zealand Ms Cave is excited about the recently opened
travel bubble with Australia, its biggest market.
"It's given people hope. Australians represent a huge part of our business. (The bubble) will mean
that we will see some growth. I think a lot of Queenstown will," she says.
In 2019, international tourism was worth $12.6bn (£9bn) in total, __________ (3) Australians
contributing $1.94bn.

AULA 07 – LINKING WORDS (CONECTIVES) AND QUESTION WORDS (WH QUESTIONS) 52


TEACHER ANDREA BELO

Between 1.18 and 1.5 million Australians came to New Zealand annually, accounting for 40% of
the country's overseas visitors.
Those numbers dropped to zero when New Zealand closed its borders.
(Adapted from https://www.bbc.com/news/business-56967636)

QUESTÃO 01 (EsPCEx/INÉDITA) – Choose the alternative containing the correct words to


respectively complete gaps (1), (2) and (3)
A) A, at, with
B) An, for, with
C) A, at, in
D) An, for, in
E) A, for, with

QUESTÃO 02 (EsPCEx/INÉDITA) – According to the text, is correct to state that


A) Despite being agitated, the Shotover jet routine was much more intense before the pandemic
B) New Zealand recently closed its borders with Australia
C) The opening of borders with Australia still represents a small part of the return to New
Zealand's tourist routine
D) Australians account for more than half of New Zealand tourists
E) New Zealand did not close its borders during the pandemic

QUESTÃO 03 (EsPCEx/INÉDITA) – In the sentence “To see and hear all the boats and the
screaming and the complete joy… it puts a smile on all of our faces” (paragraph 2), the word joy
means
A) Melancholy
B) Happiness
C) Sorrow
D) Courage
E) Calm

Leia o texto a seguir e responda às questões 04, 05 e 06


Rich countries close their eyes to the global Covid surge at their own peril
The pandemic’s death toll is now being felt most gravely in developing nations. This virus is not done yet
Is there one pandemic, or two? That ___ (1) a question being asked a year ago, when wealthy
countries accounting for only 15% of the global population had 80% of the Covid deaths. Could it
be that the rich world was more vulnerable, somehow, because its populations ___ (2) older, or
more individualistic, or had forgotten to be scared of infectious disease?

AULA 07 – LINKING WORDS (CONECTIVES) AND QUESTION WORDS (WH QUESTIONS) 53


TEACHER ANDREA BELO

Even then, some were warning that the worst was yet to come, once the disease took hold in
poorer countries. World Bank analysts Philip Schellekens and Diego Sourrouille, for example,
predicted a “massive shift” in disease burden to the developing world. Just in terms of
demography, they said, you’d expect those countries to account for around 70% of deaths.
As things stand they account for a little over half of it, which is probably an underestimate due to
variations in data quality – and the pandemic is far from over.
Last week saw more than 5.8 million new cases of Covid globally, the highest number yet. More
than 3 million people have now died from Covid, according to the World Health Organization
(WHO), which also reports that infections and hospitalisations in those aged at 25 to 59 ___ (3)
increasing an alarming rate. “It took nine months to reach 1 million deaths, four months to reach
2 million, and three months to reach 3 million,” WHO director general, Tedros Adhanom
Ghebreyesus, said last week.
(Adapted from https://www.theguardian.com/commentisfree/2021/apr/27/rich-countries-covid-pandemic-death-developing-nations)

QUESTÃO 04 (EsPCEx/INÉDITA) – In the sentence “Could it be that the rich world was more
vulnerable, somehow, because its populations were older, or more individualistic, or had
forgotten to be scared of infectious disease?” (paragraph 1), the word scared means
A) Bold
B) Cautious
C) Brave
D) Afraid
E) Partial

QUESTÃO 05 (EsPCEx/INÉDITA) – Choose the alternative containing the correct verb forms to
complete the gaps (1), (2) and (3) in paragraphs 1, 1 and 3 respectively
A) Is, was, are
B) Was, were, are
C) Is, were, are
D) Was, was, are
E) Was, were, were

QUESTÃO 06 (EsPCEx/INÉDITA) – According to the text, choose the correct statement


The COVID-19 pandemic has always affected all countries homogeneously
A) Wealthier countries were affected first by the pandemic because of their greater vulnerability
B) At the beginning of the pandemic, some people believed that the worst would come with the
involvement of
C) COVID-19 in poor countries
D) After about a year, the end of the pandemic is near
E) Last week, there were drops in the rates of coronavirus infections

AULA 07 – LINKING WORDS (CONECTIVES) AND QUESTION WORDS (WH QUESTIONS) 54


TEACHER ANDREA BELO

Leia o texto a seguir e responda às questões 07, 08 e 09


(Título omitido propositalmente)
The panel, set up by the World Health Organization, said the combined response of the WHO and
global governments was a "toxic cocktail".
The WHO should have declared a global emergency earlier than it did, its report said, adding that
without urgent change the world was vulnerable to another major disease outbreak.
More than 3.3 million people around the world have now died of Covid.
While the US and Europe are beginning to ease restrictions and resume some aspects of pre-
pandemic life, the virus is still devastating parts of Asia.
India in particular is seeing record-breaking numbers of new cases and deaths, with severe oxygen
shortages in hospitals across the country.
Countries neighbouring India, such as Nepal, are also seeing surges of the virus.
What did the report say?
Covid-19: Make it the Last Pandemic, was compiled by the Independent Panel for Pandemic
Preparedness and Response.
Its aim was to find answers as to how the virus had killed more than 3.3 million people and infected
more than 159 million.
"The situation we find ourselves in today could have been prevented," co-chair Ellen Johnson
Sirleaf, a former president of Liberia, told reporters.
"It is due to a myriad of failures, gaps and delays in preparedness and response."
The panel argued that the WHO's Emergency Committee should have declared the outbreak in
China an international emergency a week earlier than it did.
It should have done so at its first meeting on 22 January last year, the report said, instead of waiting
until 30 January.
The month following the WHO's declaration was "lost" as countries failed to take appropriate
measures to halt the spread of the virus.
The WHO was then hindered by its own regulations that travel restrictions should be a last resort,
the panel said, adding that Europe and the US wasted the entire month of February and acted
only when their hospitals began to fill up.
(Adapted from https://www.bbc.com/news/world-57085505)

QUESTÃO 07 (EsPCEx/INÉDITA) – Choose the most appropriate title for the text
A) Covid-19 pandemic was preventable, report says
B) The return to the pre-pandemic routine in the USA and Europe
C) More recent data shows an increase in the number of deaths due to COVID-19
D) Why is Covid-19 more intense in poorer countries?
E) Why the pandemic has intensified socioeconomic inequalities around the world

AULA 07 – LINKING WORDS (CONECTIVES) AND QUESTION WORDS (WH QUESTIONS) 55


TEACHER ANDREA BELO

QUESTÃO 08 (EsPCEx/INÉDITA) – What kind of text is this?


A) Letter
B) Essay
C) Opinion article
D) Textbook
E) News report

QUESTÃO 09 (EsPCEx/INÉDITA) – In the sentence “The WHO was then hindered by its own
regulations that travel restrictions should be a last resort…” (paragraph 14), the word hindered
means
A) Encouraged
B) Burdened
C) Approved
D) Neutralized
E) Supported

Leia o texto a seguir e responda à questão 10


Teens Aged 12 to 15 Can Now Get Pfizer-BioNTech COVID-19 Vaccine in the U.S.
___ (1) May 10, the U.S. Food and Drug Administration (FDA) extended authorization of the Pfizer-
BioNTech COVID-19 vaccine to 12- to 15-year-olds. It’s the first COVID-19 shot authorized for this
younger population.
“I cannot tell you how many people have been anxiously awaiting this day to get their kids
vaccinated,” says Dr. Flor Munoz, a pediatric infectious disease specialist ___ (2) Texas Children’s
Hospital and Baylor College of Medicine, and a member of the infectious disease committee of
the American Academy of Pediatrics.
The green light was based on review of data the two companies released in March showing that
two doses of their vaccine provided similar protection from COVID-19 disease among this age
group as it did for adults. Among the more than 2,200 teens in the study group the FDA reviewed,
16 developed COVID-19—none of them were in the vaccinated group.
The companies’ studies found that levels of virus-fighting antibodies were on average higher
among the 12- to 15-year-olds that got the vaccine than among vaccinated 16- to 25-year-olds in
previous studies.

AULA 07 – LINKING WORDS (CONECTIVES) AND QUESTION WORDS (WH QUESTIONS) 56


TEACHER ANDREA BELO

“This is part of the totality of getting our country protected against COVID-19, which is just waiting
around ___ (3) corner to come have another wave if we don’t get a sufficient degree of
vaccination,” said Dr. Peter Marks, director of the Center for Biologics Evaluation and Research at
the FDA, during a briefing discussing the decision.
The authorization was based on a so-called “bridging” study, in which researchers used data from
the previous study in adults to set thresholds for safety and efficacy. Building off of the success of
the adult studies enabled the scientists to enroll some 2,000 12- to 15-year-olds, rather than the
tens of thousands of adults that the earlier studies required.
(Adapted from https://time.com/6047384/teens-pfizer-covid-vaccine/)

QUESTÃO 10 (EsPCEx/INÉDITA) – Choose the alternative containing the correct words to


complete the gaps (1), (2) and (3) in paragraphs 1, 2 and 4 respectively
A) In, at, X
B) On, in, the
C) On, in, X
D) On, at, the
E) In, at, the

AULA 07 – LINKING WORDS (CONECTIVES) AND QUESTION WORDS (WH QUESTIONS) 57


TEACHER ANDREA BELO

GABARITO
GABARITO AFA
01 – A 02 – B 03 – D 04 – A 05 – D
06 – D 07 – A 08 – B 09 – C 10 – E

GABARITO COLÉGIO NAVAL


01 – A 02 – C 03 – D 04 – E 05 – B
06 – B 07 – C 08 – D 09 – E 10 – A

GABARITO EAM
01 – D 02 – E 03 – A 04 – C 05 – C
06 – A 07 – B 08 – B 09 – D 10 – E

GABARITO EEAR
01 – A 02 – C 03 – D 04 – A 05 – C
06 – C 07 – E 08 – B 09 – E 10 – B

GABARITO EFOMM
01 – A 02 – B 03 – E 04 – A 05 – C
06 – A 07 – D 08 – A 09 – E 10 – B

GABARITO EPCAR
01 – D 02 – B 03 – C 04 – B 05 – A

GABARITO ESA
01 – A 02 – C 03 – E 04 – B 05 – A

GABARITO ESCOLA NAVAL


01 – E 02 – B 03 – D 04 – A 05 – C
06 – A 07 – E 08 – C 09 – A 10 – B

GABARITO EsPCEx
01 – E 02 – A 03 – B 04 – D 05 – B
06 – C 07 – A 08 – E 09 – B 10 – D

AULA 07 – LINKING WORDS (CONECTIVES) AND QUESTION WORDS (WH QUESTIONS) 58


TEACHER ANDREA BELO

QUESTÕES COMENTADAS
QUESTÕES AFA
Read Text I to do questions 01, 02 and 03 based on it
Text I
February’s Gonna Be a Big Month for Mars On the 9th, the first of three spacecraft will arrive
at the Red Planet and inaugurate a new era of Martian exploration.

_______ FEBRUARY 9, the United Arab Emirates’ Hope spacecraft is expected to enter orbit
around Mars after a six-month, 300-million-mile journey from Earth. It will mark the beginning of
a historic month for the Red Planet, which will see three separate national missions enter orbit or
touch down on the surface. Two of the countries behind these missions, the UAE and China, will
be visiting Mars for the first time; they will become the fifth and sixth countries to pull off that
feat, respectively. The third mission, launched _______ NASA, is expected to become the United
States’ 15th mission to successfully orbit or land on Mars.
The UAE is the only country that will not attempt a soft landing during the February Mars invasion.
Instead, its Hope orbiter will study the Martian atmosphere from more than 12,000 miles above
the surface. Planetary scientists hope that the UAE’s robo-meteorologist will fill in gaps in our
understanding of the Martian climate and help validate environmental data captured by rovers
and landers on the ground. For the country’s first foray into deep-space exploration, the UAE
space agency worked with an international team of researchers at the University of Colorado,
Boulder, to help plan the mission and build the spacecraft.
“There’s really no point in exploring outer space without adding to knowledge, and we’ve never
run a science mission,” Sarah bint Yousef Al Amiri, the UAE minister of state for advanced sciences
and science lead for the Emirates Mars Mission, said during a press conference last week. “It
wasn’t an easy journey, but it was such an enjoyment to rethink how you develop a planetary
exploration mission.”

AULA 07 – LINKING WORDS (CONECTIVES) AND QUESTION WORDS (WH QUESTIONS) 59


TEACHER ANDREA BELO

The Hope spacecraft will be the first new orbiter around Mars since the European Space Agency’s
ExoMars spacecraft arrived in 2016, but it won’t be the newcomer for long. China’s Tianwen-1
mission—which is a lander, rover, and orbiter rolled up into one—is expected to arrive less than
a day later. China’s space agency has been quiet about its plans for visiting the Red Planet, but
the craft is expected to attempt a landing shortly after it achieves orbit.
Unlike NASA’s car-sized Mars rovers Curiosity and Opportunity, China’s Tianwen-1 rover is small
enough to stow away inside the stationary lander that will carry it to the surface. Once it has safely
touched down, the six-wheeled rover will detach itself from the lander and spend the next three
months exploring its landing site, Utopia Planitia, the planet’s largest impact crater. The rover and
lander will both relay data _______ the surface to the Tianwen-1 orbiter, which will send it back
to Earth. Although the Chinese National Space Administration hasn’t provided a lot of details
about the exact scientific goals of its mission, a paper about it published last year in Nature
Astronomy says the agency’s goal is to “perform a global and extensive survey of the entire
planet.”
On February 18, a little more than a week after this robotic delegation arrives, NASA’s
Perseverance rover is expected to touch down. This will involve a harrowing descent to the
surface, during which the rover must reduce its speed from more than 10,000 miles an hour to
just a few feet per second over the course of 15 minutes. The descent will end with some aerial
acrobatics, during which a rocket-powered sky crane will gently deposit the rover on the surface
while hovering a few dozen feet above the ground.
“Don’t let anybody tell you different—landing _______ Mars is hard to do,” John McNamee,
project manager for the Perseverance mission at NASA’s Jet Propulsion Laboratory, said in a
statement. “But the women and men on this team are the best in the world at what they do.
When our spacecraft hits the top of the Mars atmosphere at about three and a half miles per
second, we’ll be ready.”
Perseverance is essentially a nuclear-powered self-driving car, and its primary mission is to collect
samples that will be picked up by another spacecraft later this decade and returned to Earth. With
any luck, this red dust will contain evidence that Mars once hosted microbial life. But whether
scientists will recognize extraterrestrial life when they see it remains an open question. Aside from
hunting for aliens, Perseverance will also enable a first-of-its-kind technology demonstration
involving a small helicopter called Ingenuity. A few days after landing, Perseverance will jettison
the helicopter in a clearing where it will attempt several short flights. If it works, it will be the first
time an aircraft has flown on another planet.
The arrival of three national missions on Mars within two weeks of one another is a historic
moment in the history of space exploration. It underscores the rapid development of space
capabilities around the world and the true internationalization of planetary exploration. Mars was
once the exclusive stomping grounds of the United States and the Soviet Union, but it is now also
an accessible destination for the European Union, Japan, India, the UAE, and China. Getting to
Mars is still a major challenge—historically only 40 percent of Mars missions have been
successful—and there’s no guarantee that all three missions will succeed in their objectives. But
launching a trio of spacecraft to our closest planetary neighbor is a major achievement and bodes
well for the future of space exploration.
(Adapted from https://www.wired.com/story/februarys-gonna-be-a-big-month-for-mars/)

AULA 07 – LINKING WORDS (CONECTIVES) AND QUESTION WORDS (WH QUESTIONS) 60


TEACHER ANDREA BELO

QUESTÃO 01 (AFA/INÉDITA) – Which is the correct option to complete the gaps in the text?
(A) On – by – from – on
(B) At – to – from – by
(C) On – by – to – by
(D) In – by – on – by
(E) In – to – to – on
Comentários: A alternativa A está correta.
Lacuna 1: Deve-se usar a preposição “on” para dia do mês (data completa – dia e mês). Cuidado,
pois caso apareça somente o mês não devemos usar a preposição “on” e sim a preposição “in”.
Lacuna 2: Na frase em questão trata-se de uma voz passiva e o sujeito da voz ativa torna-se agente
da passiva precedido por by. “The third mission, launched by NASA” (A Terceira missão, lançada
pela NASA.)
Lacuna 3: A preposição que melhor se encaixa nessa lacuna é “from”, pois indica procedência,
origem de alguém ou algo. “The rover and lander will both relay data from the surface to the
Tianwen-1 orbiter” (O veículo e o módulo de pouso transmitirão dados da superfície para o
orbitador Tianwen-1). Neste caso, os dados têm como origem a superfície de Marte.
Lacuna 4: Land on = pousar (A preposição “on” é usada para indicar que alguma coisa está em
uma superfície)
A alternativa B está incorreta. A sequência correta é On – by – from – on.
A alternativa C está incorreta. A sequência correta é On – by – from – on.
A alternativa D está incorreta. A sequência correta é On – by – from – on.
A alternativa E está incorreta. A sequência correta é On – by – from – on.
GABARITO: A

QUESTÃO 02 (AFA/INÉDITA) – According to the text, which option is correct?


(A) The UAE and China had already visited Mars before, five or six times.
(B) The UAE is not going to land on Mars. Rather than, its Hope orbiter will study the Martian
atmosphere from space.
(C) The United Arab Emirates’ Hope spacecraft will be studying mars soil for 6 months.
(D) About six countries have already accomplished the feat of going to mars.
(E) The arrival of five national missions to Mars two weeks apart is a historic moment in the
history of space exploration.
Comentários: A alternativa A está incorreta. De acordo com o texto, é a primeira vez que os
Emirados Árabes e a China realizarão uma expedição para Marte. “Two of the countries behind
these missions, the UAE and China, will be visiting Mars for the first time” (Dois dos países por
trás dessas missões, os Emirados Árabes Unidos e a China, visitarão Marte pela primeira vez).

AULA 07 – LINKING WORDS (CONECTIVES) AND QUESTION WORDS (WH QUESTIONS) 61


TEACHER ANDREA BELO

A alternativa B está correta. De acordo com o texto, Os Emirados Árabes Unidos não farão um
pouso suave durante a invasão de Marte em fevereiro. Em vez disso, seu orbitador Hope estudará
a atmosfera marciana de mais de 12.000 milhas acima da superfície.
A alternativa C está incorreta. De acordo com o texto, seis meses é o tempo que a espaçonave
Hope levará para chegar em Marte e ela não pousará para estudar o solo.
A alternativa D está incorreta. De acordo com o texto, os Emirados Árabes e a china serão,
respectivamente, o quinto e o sexto países a realizarem uma expedição para Marte.
A alternativa E está incorreta. De acordo com o texto, três missões espaciais chegarão a Marte e
não cinco.
GABARITO: B

QUESTÃO 03 (AFA/INÉDITA) – “Although the Chinese National Space Administration hasn’t


provided a lot of details about […]”. The highlighted word is closest in meaning to
(A) However
(B) Despite
(C) Since
(D) Even though
(E) Besides
Comentários: A alternativa A está incorreta. However = porém.
A alternativa B está incorreta. Despite = Apesar de.
A alternativa C está incorreta. Since = Visto que.
A alternativa D está correta. Although = Even though = Embora. “Although the Chinese National
Space Administration hasn’t provided a lot of details about […]” (Embora a Administração Espacial
Nacional da China não tenha fornecido muitos detalhes sobre [...]).
A alternativa E está incorreta. Besides = Além de.
GABARITO: D

Read Text ll to do question 04 based on it


Text ll
IN LOS ANGELES, the corner of Melrose and Harper has become a tourist destination to rival the
Eiffel Tower, or the graffitied remains of the Berlin Wall. Rather than an architectural marvel or a
piece of living history, people line up (or did, in pre-Covid times) to visit the bright pink exterior
wall of Paul Smith, a clothing retailer. The wall—repainted every three months in the Pantone
shade “Pink Ladies”—is the background to hundreds of thousands of photos, making it one of the
most Instagrammed places in Los Angeles, and even the world.
(Adapted from https://www.wired.com/story/fake-famous-review-instagram-influencers-documentary/)

AULA 07 – LINKING WORDS (CONECTIVES) AND QUESTION WORDS (WH QUESTIONS) 62


TEACHER ANDREA BELO

QUESTÃO 04 (AFA/INÉDITA) – Mark the option that correctly substitutes the expression rather
than (line 02).
(A) Instead of.
(B) Despite of.
(C) As well as.
(D) In addition to.
(E) At last.
Comentários: A alternativa A está correta. rather than = instead of = ao invés de.
A alternativa B está incorreta. Despite of = apesar de.
A alternativa C está incorreta. As well as = assim como, também.
A alternativa D está incorreta. In addition to = além de.
A alternativa E está incorreta. At last = afinal.
GABARITO: A

Read Text lll to do questions 05 and 06 based on it


Text lll
Russian Fabergé exhibition contains 'at least 20 fakes', expert says
Andre Ruzhnikov accuses director Mikhail Piotrovsky of ‘destroying the authority of the museum’
A Russian museum __________ to close a Fabergé exhibition that contains items loaned from the
personal collection of a billionaire, after a prominent expert said it contained more than a dozen
fakes.
In a letter, the art dealer Andre Ruzhnikov accused the Hermitage Museum’s director, Mikhail
Piotrovsky, of “destroying the authority of the museum” by hosting the Fabergé: Jeweller to the
Imperial Court show, which runs until 14 March.
Ruzhnikov told the Guardian that the exhibition included at least 20 fakes, and that he thought
the exhibition, which is the first big Fabergé event at the St Petersburg institution since 1993,
should close immediately. He said: “I want the shame to end. I want this show to be closed and
forgotten, and that’s it. You cannot subject the Hermitage to such shame.”
The Hermitage and Alexander Ivanov have denied the claims and the billionaire produced
documents that support the authenticity of the items that were loaned from the Fabergé Museum
in Baden-Baden, which he established in 2009.
Piotrovsky and the Hermitage did not respond to a request for comment.
(Adapted from https://www.theguardian.com/artanddesign/2021/feb/01/russian-faberge-exhibition-contains-fakes-expert-says)

AULA 07 – LINKING WORDS (CONECTIVES) AND QUESTION WORDS (WH QUESTIONS) 63


TEACHER ANDREA BELO

QUESTÃO 05 (AFA/INÉDITA) – Which is the correct option to complete the gap in the text?
(A) has asked
(B) had been asked
(C) had asked
(D) has been asked
(E) was asked
Comentários: A alternativa A está incorreta. Esta alternativa não se encontra na voz passiva e sim
na voz ativa e por isso não preenche corretamente a lacuna do texto.
A alternativa B está incorreta. Apesar de estar na voz passiva, o tempo verbal utilizada na
alternativa não está correto. Deveria ser utilizado o present perfect e não past perfect, por se
tratar de uma oração que expressa uma ação que o correu num passado indeterminado.
A alternativa C está incorreta. Esta alternativa não se encontra na voz passiva e sim na voz ativa e
por isso não preenche corretamente a lacuna do texto.
A alternativa D está correta. Nessa questão trata-se de uma voz passiva. “A Russian museum has
been asked to close a Fabergé exhibition” (Um museu russo foi convidado a encerrar uma
exposição de Fabergé.). O tempo verbal a ser utilizado na oração deve ser o present perfect por
se tratar de uma ação que ocorreu num passado indeterminado.
A alternativa E está incorreta. Apesar de estar na voz passiva, o tempo verbal utilizada na
alternativa não está correto. Deveria ser utilizado o present perfect e não o simple past, por se
tratar de uma oração que expressa uma ação que o correu num passado indeterminado.
GABARITO: D

QUESTÃO 06 (AFA/INÉDITA) – In the phrase “…a Fabergé exhibition that contains items loaned
from the personal collection of a billionaire” (paragraph 1), the underlined word is a synonym
for
(A) donated
(B) purchased
(C) acquired
(D) borrowed
(E) lent
Comentários: A alternativa A está incorreta. Os itens da exposição foram emprestados e não
doados.
A alternativa B está incorreta. Os itens da exposição foram emprestados e não comprados.
Purchase é sinônimo de buy (comprar).
A alternativa C está incorreta. Os itens da exposição foram emprestados e não adquiridos.
Adquirir significa passar a ser o dono de algo, e no caso, o museu não passou a ser dono dos itens,
eles foram apenas emprestados pelo bilionário.

AULA 07 – LINKING WORDS (CONECTIVES) AND QUESTION WORDS (WH QUESTIONS) 64


TEACHER ANDREA BELO

A alternativa D está correta. De acordo com o texto, um museu russo foi convidado a fechar uma
exposição de Fabergé que contém itens emprestados da coleção pessoal de um bilionário. Os itens
da exposição foram pegos emprestados de um bilionário. A palavra borrow tem o sentido de
“pegar emprestado” e portanto preenche corretamente a lacuna.
A alternativa E está incorreta. Cuidado, preste atenção na diferença entre borrow e lend. Lend é
usado quando você “empresta algo para alguém”. Diferente disso, borrow tem o sentido de
“pegar emprestado”. O museu pegou empresta (borrow) já o bilionário emprestou (lend).
GABARITO: D

Read Text lV to do questions 07, 08, 09 and 10 based on it


Text lV
When will offices be full again?
Maybe never, some executives say.
By Lauren Hirsch
Many companies do not expect their workers to return to offices until next summer, and even
then things may never be the same as before, judging by the comments executives made this
week, highlighted in today’s DealBook newsletter.
On earnings calls, executives from Goldman Sachs said that about a third of workers in New York
and London were coming in regularly; at JPMorgan Chase, it’s around 20 percent in both cities;
and Citigroup said “a small percentage” of employees in North America had returned.
“Being together enables greater collaboration, which is key to our culture,” said David M.
Solomon, Goldman’s chief. But Jamie Dimon of JPMorgan acknowledged that some working
habits may have changed permanently, which “will ultimately reduce the space you need for your
employees.” Terrance R. Dolan, the finance chief at U.S. Bancorp, told analysts that the bank will
most likely “consolidate” its corporate real estate to reflect “the new horizon.”
Is that a problem? Steven J. Goulart, the chief investment officer at MetLife, said at a regulatory
round table that the “pressure to de-densify” offices to support social distancing could support
demand for real estate even if buildings aren’t as full as before.
And as executives conduct more business remotely, going back to in-person meetings and pitches
seems less urgent. Natarajan Chandrasekaran, the chairman of Indian conglomerate Tata Sons,
said in an interview with The New York Times that he used to fly from India to the United States
to pitch a $50,000 project. But recently, he said, his firm’s consultancy business closed $2 billion
worth of deals in “five or six Zoom calls.”
There are other perks from working at home. BlackRock’s Laurence D. Fink is excited about what
employees could do with the time they save on daily commutes. “They could spend two hours
improving their health by exercising,” he said on a conference call. “They could spend two hours
more in building a deeper, stronger, more resilient family.”
Paul Draovitch of Duke Energy said at an investor event that working from home was “not without
risks,” but also brought certain benefits: “When my Pomeranians walk into the room, it's really a
pleasure.”
Adapted from: https://www.nytimes.com/2020/10/16/business/when-will-offices-be-full-again-maybe-never-some-executives-say.html

AULA 07 – LINKING WORDS (CONECTIVES) AND QUESTION WORDS (WH QUESTIONS) 65


TEACHER ANDREA BELO

QUESTÃO 07 (AFA/INÉDITA) – The correct form of the sentence “They could spend two hours
more in building a deeper, stronger, more resilient family.” in the indirect speech is:
(A) It said that they could have spent two hours more in building a deeper, stronger, more
resilient family.
(B) It said that they could spent two hours more in building a deeper, stronger, more resilient
family.
(C) It said that they will spend two hours more in building a deeper, stronger, more resilient
family.
(D) It said that they spend two hours more in building a deeper, stronger, more resilient family.
(E) It said that they have spended two hours more in building a deeper, stronger, more resilient
family.
Comentários: A alternativa A está correta. O discurso indireto do modal could é: could + have +
verbo no particípio portanto a única opção que atende esse requisito é a alternativa A.
A alternativa B está incorreta. Could + spent está errado, após verbol modais o verbo deve estar
na forma infinitivo sem to.
A alternativa C está incorreta. Will não é a forma de discurso indireto de could.
A alternativa D está incorreta. O discurso indireto do modal could é could + have + verbo no
particípio portanto essa alternativa não atende aos requisitos necessários.
A alternativa E está incorreta. O discurso indireto do modal could é could + have + verbo no
particípio portanto essa alternativa não atende aos requisitos necessários.
GABARITO: A

QUESTÃO 08 (AFA/INÉDITA) – In “There are other perks from working at home” the underlined
word is closest in meaning to:
(A) goods
(B) benefits
(C) harms
(D) losses
(E) ways
Comentários: A alternativa A está incorreta. Goods = bens, mercadorias.
A alternativa B está correta. Perks = benefícios (benefits). O texto fala dos benefícios de se
trabalhar em casa.
A alternativa C está incorreta. Harm = malefícios, danos.
A alternativa D está incorreta. Losses = perdas, prejuízos.
A alternativa E está incorreta. Ways = maneiras, meios.
GABARITO: B

AULA 07 – LINKING WORDS (CONECTIVES) AND QUESTION WORDS (WH QUESTIONS) 66


TEACHER ANDREA BELO

QUESTÃO 09 (AFA/INÉDITA) – In the sentence “They could spend two hours improving their
health by exercising,” extracted from the text the pronoun they refers to:
(A) daily commutes
(B) BlackRock’s Laurence D. Fink
(C) Employees
(D) Many companies
(E) Family
Comentários: A alternativa A está incorreta. O pronome they não faz referência a “viagens
diárias”.
A alternativa B está incorreta. O pronome they não faz referência a “BlackRock’s Laurence D.
Fink”.
A alternativa C está correta. De acordo com o texto as pessoas que poderiam melhorar sua saúde
através de exercícios eram os empregados. Portanto o pronome they faz referência a employees.
A alternativa D está incorreta. O pronome they não faz referência a “many companies”.
A alternativa E está incorreta. O pronome they não faz referência a “family”. Dica: O pronome
they é um pronome de 3ª pessoas do plural, logo não irá fazer referência a singulares.
GABARITO: C

QUESTÃO 10 (AFA/INÉDITA) – “They could spend two hours improving their health by
exercising,” The underlined word can be replaced by:
(A) coming up with
(B) puting up with
(C) puting away
(D) turning down
(E) brushing up on
Comentários: A alternativa A está incorreta. Come up with significa pensar/falar algo novo ou
aparecer com algo e portanto não se encaixa no contexto.
A alternativa B está incorreta. Put up with = tolerar, aguentar algo ou alguém.
A alternativa C está incorreta. Put away = guardar.
A alternativa D está incorreta. Turn down = recusar.
A alternativa E está correta. Brush up on = improve = get better (melhorar). É Exatamente o que
o texto diz, melhor sua saúde através de exercícios.
GABARITO: E

AULA 07 – LINKING WORDS (CONECTIVES) AND QUESTION WORDS (WH QUESTIONS) 67


TEACHER ANDREA BELO

QUESTÕES COLÉGIO NAVAL


Read the text and answer the following questions 01 – 02.
Inflation: Used cars and food push US prices higher
Consumer prices jumped 5.4% in the 12 months to the end of June, up from 5% _______ previous
month.
It marks the biggest 12-month increase _______ August 2008, according to the US Labor
Department.
Inflation, which measures the rate at which cost of living increases, has been rising as the economy
reopens from coronavirus lockdowns.
It has sparked fears that prices _______ increasing too quickly, which could prompt the Federal
Reserve to push up interest rates or pull back on pandemic support earlier than expected.
However, some economists and the Federal Reserve say that the inflationary pressures will be
temporary.
Used vehicles accounted for one-third of the increase in the Consumer Price Index (CPI) _______
June, the Labor Department said _______ Tuesday.
But prices also reflected a broader surge in consumer demand as restrictions eased, with the costs
of meals in restaurants and cafes, hotel stays and airline tickets all rising last month.
(https://www.bbc.com/news/business-57573387)

QUESTÃO 01 (COLÉGIO NAVAL/INÉDITA) – In the title we can affirm that the prices are
(A) increasing
(B) dropping down
(C) decreasing
(D) at the same level
(E) in equal measures
Comentários: A letra A está correta. O título diz que: Inflação: Carros usados e alimentação puxam
os preços para cima. Portanto, a expressão “higher”, coloca como aumento. Sinônimo de
“increasing”, que significa “aumento”.
A letra B está incorreta. dropping down = queda.
A letra C está incorreta. decreasing = decrescer.
A letra D está incorreta. at the same level = no mesmo nível.
A letra E está incorreta. in equal measures = em níveis/medidas iguais.
Portanto, as expressões das outras opções fogem do contexto do título.
GABARITO: A

AULA 07 – LINKING WORDS (CONECTIVES) AND QUESTION WORDS (WH QUESTIONS) 68


TEACHER ANDREA BELO

QUESTÃO 02 (COLÉGIO NAVAL/INÉDITA) – Compete the gap in the sentence: “5% ____ previous
month.”
(A) a
(B) an
(C) the
(D) of
(E) no article
Comentários: A letra A está incorreta. “a” é artigo indefinido e somente poderia ser mencionado
em casos genéricos. Na sentença é um mês específico. “O mês anterior.”
A letra B está incorreta. “an" é artigo indefinido e somente poderia ser mencionado em casos
genéricos diante de sons de vogais. Na sentença é um mês específico. “O mês anterior.”
A letra C está correta. “The” é artigo definido; usado para coisas específicas e já mencionadas
anteriormente. Logo, é o gabarito de acordo com a gramática.
A letra D está incorreta. Of = de/do/da; é uma preposição, a qual não deve ser usada nesse
contexto.
A letra E está incorreta. O uso de “não artigo” não se perfaz, tendo em vista as explicações
anteriores.
GABARITO: C

QUESTÃO 03 (COLÉGIO NAVAL/INÉDITA) – Complete the blank space: “_____ August 2008”
(A) from
(B) with
(C) for
(D) since
(E) about
Comentários: A letra A está incorreta. from = de (origem). Não se encaixa no contexto “desde
Agosto de 2008.”
A letra B está incorreta. with = com. Não se encaixa no contexto “desde Agosto de 2008.”
A letra C está incorreta. for = por, durante (duração da ação). Não se encaixa no contexto “desde
Agosto de 2008.”
A letra D está correta. since = desde (início da ação). Encaixa no contexto “desde Agosto de 2008.”
A letra E está incorreta. about = sobre. Não se encaixa no contexto “desde Agosto de 2008.”
GABARITO: D

AULA 07 – LINKING WORDS (CONECTIVES) AND QUESTION WORDS (WH QUESTIONS) 69


TEACHER ANDREA BELO

QUESTÃO 04 (COLÉGIO NAVAL/INÉDITA) – In the sentence, “Inflation, which measures the rate
cause
(…)”, the word in bold refers to
(A) rate
(B) cause
(C) the
(D) measures
(E) inflation
Comentários: A questão versa sobre o uso do relativo “which”, o qual retoma coisas, objetos,
animais etc. (não sendo pessoa). Conectando-se a substantivos (nouns).
A letra A está incorreta. “Rate” não é o antecedente. Está após a menção do relativo.
A letra B está incorreta. “Cause” não é o antecedente. Está após a menção do relativo.
A letra C está incorreta. “The” não é o antecedente. Também não é substantivo. Está após a
menção do relativo.
A letra D está incorreta. “measures” é verbo. Medir. Não podendo ser relativo de “which”.
A letra E está correta. Inflation = inflação; é substantivo e antecedente de “which”.
GABARITO: E

QUESTÃO 05 (COLÉGIO NAVAL/INÉDITA) – In, “which could prompt the Federal Reserve to push
up interest rates”. The underline verb gives an idea of
(A) obligation
(B) possibility
(C) ability
(D) advice
(E) permission
Comentários: A questão trata sobre modais. O CN diz sobre CAN, mas Could é o passado de CAN.
Podendo ser cobrado.
A letra A está incorreta. Could não expressa obrigação. E sim: must, have to.
A letra B está correta. Could nesse contexto expressa “possibilidade”. “no qual poderá...”
A letra C está incorreta. Could poderia até expressar uma habilidade, mas não se aplica nesse
contexto.
A letra D está incorreta. Could não expressa “conselho”. E sim: should / ought to.
A letra E está incorreta. Could poderia até expressar uma “permissão, mas não nesse contexto.
GABARITO: B

AULA 07 – LINKING WORDS (CONECTIVES) AND QUESTION WORDS (WH QUESTIONS) 70


TEACHER ANDREA BELO

QUESTÃO 06 (COLÉGIO NAVAL/INÉDITA) – The correct statement according to the text is


(A) The cost of living is not a currently problem in the US.
(B) Specialists say that inflation will be temporary.
(C) Used vehicles were not affected by this situation.
(D) Food department didn’t have any inflation grow.
(E) The prices are getting down now.
Comentários: A letra A está incorreta. O custo de vida não é um problema atual nos EUA. (falso)
A letra B está correta. Especialistas dizem que a inflação será temporária. (verdadeiro); O texto
todo fala sobre a alta inflacionária.
A letra C está incorreta. Veículos usados não foram afetados por essa situação. (falso)
A letra D está incorreta. O departamento de alimentos não teve nenhum crescimento da inflação.
(falso)
A letra E está incorreta. Os preços estão caindo agora. (falso)
GABARITO: B

QUESTÃO 07 (COLÉGIO NAVAL/INÉDITA) – The word “however”, in bold in the text can be
replaced by
(A) also
(B) otherwise
(C) nevertheless
(D) so
(E) then
Comentários: A conjunção “however” = mas, porém. (adversativa).
A letra A está incorreta. also = também (conjunção aditiva)
A letra B está incorreta. otherwise = por outro modo (conjunção alternativa)
A letra C está correta. nevertheless = mas, porém (adversativa). Logo, sinônimo de “however”.
A letra D está incorreta. so = então (conjunção conclusiva)
A letra E está incorreta. then = então (conjunção conclusiva)
GABARITO: C

AULA 07 – LINKING WORDS (CONECTIVES) AND QUESTION WORDS (WH QUESTIONS) 71


TEACHER ANDREA BELO

QUESTÃO 08 (COLÉGIO NAVAL/INÉDITA) – “It has sparked fears that prices ____ increasing too
quickly.” The correct verb form to fill in the blank.
(A) is
(B) to be
(C) will be
(D) are
(E) do
Comentários: A questão trata de verbos. Verbo to be = ser/estar
A letra A está incorreta. “is” não conjuga com “prices”
A letra B está incorreta. to be = verbo sem estar conjugado
A letra C está incorreta. will be = futuro (não encaixa nesse contexto)
A letra D está correta. prices (they) are. Conjugação correta.
A letra E está incorreta. do = fazer; não encaixa gramaticalmente no contexto.
GABARITO: D

QUESTÃO 09 (COLÉGIO NAVAL/INÉDITA) – The expression, “coronavirus lockdowns”, means


I. Restriction of going and getting back. (Restrições de ir e vir)
II. The virus itself will lock people at home. (O virus por si mesmo trancou as pessoas em casa)
III. People are with some situations restricted by the global disease. (As pessoas estão com
alguma restrições pela doença global)
(A) All of them are correct.
(B) I and III are incorrect.
(C) II and III are correct.
(D) II is correct, but III is incorrect.
(E) I and III are correct.
Comentários: Traduções ao lado das sentenças.
A letra A está incorreta. Todas não estão corretas, pois a II está errada.
A letra B está incorreta. I e III não estão incorretas. E sim, certas.
A letra C está incorreta. A II não está correta. Somente a III.
A letra D está incorreta. II não está correta e sim a III.
A letra E está correta. I e III estão corretas.
GABARITO: E

AULA 07 – LINKING WORDS (CONECTIVES) AND QUESTION WORDS (WH QUESTIONS) 72


TEACHER ANDREA BELO

QUESTÃO 10 (COLÉGIO NAVAL/INÉDITA) – “___ June, the Labor Department said ___ Tuesday.”
Complete the spaces with the suitable prepostitions
(A) in / on
(B) in / in
(C) on in
(D) at / in
(E) in / at
Comentários: A questão versa sobre o uso adequado das preposições.
A letra A está correta. In = mês / On = dias da semana.
A letra B está incorreta. In não pode ser usado para dias da semana.
A letra C está incorreta. Está invertido o uso correto.
A letra D está incorreta. At não se aplica no uso e in também não.
A letra E está incorreta. In está correto, mas At não.
GABARITO: A

QUESTÕES EAM
Read the following text to answer questions 01 to 07.
Lost Dog
Sam saw a dog on the sidewalk. The dog looked lost. Sam approached the dog and patted its head.
The dog wagged his tail. The dog had a collar. Sam looked at the collar and it read the dog's name.
Its name was Spike. Spike was a small dog and looked well-groomed. “Come on, Spike,” he said to
the dog. The dog followed him home. He brought the dog upstairs to his room. He had a jar of
dog food because he used to have a dog. He poured the dog food into a bowl and placed it in front
of Spike. Spike started eating. “I'll take care of you, Spike,” he said to the dog. He knew the dog
did not belong to him. He printed posters of Spike that said “Found Dog”. He went around hanging
the poster but nobody came to find Spike.
(Adapted from <https://www.eslfast.com/begin2/b2/b2014.htm>)

QUESTÃO 01 (EAM/INÉDITA) – Where did Sam most likely find Spike?


(A) At an animal shelter.
(B) At his home.
(C) On his front porch.
(D) On the street.
(E) At a market.
Comentários: Onde é mais provável que o Sam tenha encontrado o Spike?

AULA 07 – LINKING WORDS (CONECTIVES) AND QUESTION WORDS (WH QUESTIONS) 73


TEACHER ANDREA BELO

a) Em um abrigo de animais.
b) Em sua casa.
c) Na varanda da frente.
d) Na rua.
e) Em um mercado.
“Sam saw a dog on the sidewalk. The dog looked lost.” = Sam viu um cachorro na calçada. O
cachorro parecia perdido. Assim, é mais provável que o Sam tenha encontrado o Spike na rua,
pois o cachorrinho estava na calçada, parecendo perdido.
GABARITO: D

QUESTÃO 02 (EAM/INÉDITA) – Spike looked like


(A) a stray dog.
(B) it was not well-cared.
(C) a dog of a big breed.
(D) it was not lost.
(E) it had an owner.
Comentários: O Spike parecia
a) um cachorro vira-lata.
b) que não era bem cuidado.
c) um cão de raça grande.
d) que não estava perdido.
e) que tinha dono.
“The dog had a collar. Sam looked at the collar and it read the dog's name. Its name was Spike.
Spike was a small dog and looked well-groomed.” = O cachorro tinha uma coleira. Sam olhou para
a coleira e leu o nome do cachorro. Seu nome era Spike. Spike era um cachorro pequeno e parecia
bem cuidado. Assim, o Spike parecia que tinha dono (estava de coleira, bem cuidado...).
GABARITO: E

QUESTÃO 03 (EAM/INÉDITA) – How many people reached Sam looking for Spike?
(A) zero
(B) many.
(C) two.
(D) some.
(E) much.

AULA 07 – LINKING WORDS (CONECTIVES) AND QUESTION WORDS (WH QUESTIONS) 74


TEACHER ANDREA BELO

Comentários: Quantas pessoas entraram em contato com Sam procurando pelo Spike?
a) zero
b) muitas.
c) duas.
d) algumas.
e) “muitas” (mas lembre-se que much só é usado com substantivos incontáveis, e people, pessoas,
é contável). “He went around hanging the poster but nobody came to find Spike.” = Ele andou
pendurando o pôster, mas ninguém apareceu para encontrar o Spike.
GABARITO: A

QUESTÃO 04 (EAM/INÉDITA) – Did Sam feed the dog?


(A) No, he didn’t.
(B) No, he did.
(C) Yes, he did.
(D) Yes, he fed.
(E) Yes, he’s feeding.
Comentários: O Sam alimentou o cachorro? Sim, ele o alimentou, conforme o texto: “He had a jar
of dog because he used to have a dog. He poured the dog food into a bowl and placed it in front
of Spike. Spike started eating.” E a resposta curta correta para essa pergunta no Simple Past é Yes,
he did, com o auxiliar de passado did (em respostas curtas afirmativas não usamos o verbo
principal, apenas o auxiliar).
GABARITO: C

QUESTÃO 05 (EAM/INÉDITA) – Read the following sentence.


Sam looked at the collar and it read the dog's name.
The pronoun IT refers to:
(A) Sam.
(B) the dog.
(C) the collar.
(D) name.
(E) the dog’s name.
Comentários: Na frase trazida pela questão, it se refere à coleira do cachorro (the collar), veja: O
Sam olhou para a coleira e nela (na coleira) lia-se o nome do cachorro.
GABARITO: C

AULA 07 – LINKING WORDS (CONECTIVES) AND QUESTION WORDS (WH QUESTIONS) 75


TEACHER ANDREA BELO

QUESTÃO 06 (EAM/INÉDITA) – Mark the option that is FALSE about the text.
(A) Sam was Spike’s dog.
(B) Spike was not Sam’s dog.
(C) Spike seemed lost.
(D) Spike was little.
(E) Spike went to Sam’s home.
Comentários: Marque a opção que é FALSA sobre o texto.
a) Sam era o cachorro de Spike. – Sam era o homem e Spike era o cachorro. FALSA.
b) Spike não era o cachorro de Sam. – De fato, Sam o encontrou na rua.
c) Spike parecia perdido. – “The dog looked lost”. Verdadeira.
d) Spike era pequeno. – “Spike was a small dog…” Verdadeira.
e) O Spike foi para a casa de Sam. – “The dog followed him home. He brought the dog upstairs to
his room”. Verdadeira.
GABARITO: A

QUESTÃO 07 (EAM/INÉDITA) – Read the following sentence.


Its name was Spike.
The possessive adjective ITS refers to:
(A) the man.
(B) the dog.
(C) the dog’s owner.
(D) the collar.
(E) Sam.
Comentários: Em “Its name was Spike.”, its se refere ao cachorro, indicando posse, a quem
pertence o nome: o nome dele (do cachorro) era Spike.
GABARITO: B

QUESTÃO 08 (EAM/INÉDITA) – Which items of clothing can you see in the picture below?

AULA 07 – LINKING WORDS (CONECTIVES) AND QUESTION WORDS (WH QUESTIONS) 76


TEACHER ANDREA BELO

(A) shoes, t-shirt, dress, shorts.


(B) sneakers, shirt, dress, shorts.
(C) shoes, suit, dress, trousers.
(D) sneakers, tuxedo, blouse, shorts.
(E) shoes, shirt, skirt, pants.
Comentários: Quais peças de vestuário você pode ver na imagem abaixo? Vemos na imagem:
sneakers (tênis), shirt (camisa), dress (vestido), shorts (short).
Outros termos que aparecem nas alternativas:
shoes = sapatos, de modo geral.
t-shirt = camiseta (camisa de malha, em forma de T, sem botões)
suit = terno
trousers = calça
tuxedo = smoking
blouse = blusa
skirt = saia
pants = calça
GABARITO: B

QUESTÃO 09 (EAM/INÉDITA) – Mark the option that completes the sentences with the right
form of the verbs in parenthesis, respectively.
Lucy __________ (to go) to the gym every day, but she __________ (to go) this morning,
because she was __________ (to feel) sick.
(A) goes / went / feeling
(B) gos / didn’t go / felt
(C) going / went / feeling
(D) goes / didn’t go / feeling
(E) go / didn’t go / feels
Comentários: Lucy goes to the gym every day = Simple Present, por conta da expressão every day
(todo dia) e conjugado na terceira pessoa do singular – Lucy vai à academia todo dia. but she
didn’t go this morning = mas ela não foi essa manhã (but indicando contraste de ideias) because
she was feeling sick. = porque ela estava se sentindo mal. (algo contínuo, mas temporário – Past
Continuous)
GABARITO: D

AULA 07 – LINKING WORDS (CONECTIVES) AND QUESTION WORDS (WH QUESTIONS) 77


TEACHER ANDREA BELO

QUESTÃO 10 (EAM/INÉDITA) – Mark the option that completes the blanks with the right
preposition of time or place.
I arrived __________ New York __________ noon.
(A) in / on
(B) at / at
(C) at / in
(D) on / at
(E) in / at
Comentários: Para cidades usamos a preposição in, e para horários (noon = meio-dia) a
preposition at.
GABARITO: E

QUESTÕES EEAR
Read Text I to do questions 01, 02 and 03 based on it.
Text I
We must not let hope abandon us, because God, with his love, walks with us. “I hope, because
God is beside me”: we can all say this. Each one of us can say: “I hope, I have hope, because God
walks with me.” He walks and he holds my hand. God does not leave us to ourselves. The Lord
Jesus has conquered evil and has opened the path of life for us. Let us allow the Lord to teach us
what it means to hope. ___________, let us listen to the words of Sacred Scripture, beginning
with the prophet Isaiah, the great messenger of hope.
By Pope Francis

QUESTÃO 01 (EEAR/INÉDITA) – “We must not let hope abandon us” The underlined modal verb
expresses the idea of:
(A) prohibition
(B) obligation
(C) possibility
(D) suggestion
(E) deduction
Comentários: A alternativa A está correta. Must not = prohibition (proibição)
A alternativa B está incorreta. Must até pode ter ideia de obrigação, em alguns casos, mas must
not é proibição.
A alternativa C está incorreta. Must not não expressa a ideia de possibilidade.
A alternativa D está incorreta. Must not não expressa a ideia de sugestão.

AULA 07 – LINKING WORDS (CONECTIVES) AND QUESTION WORDS (WH QUESTIONS) 78


TEACHER ANDREA BELO

A alternativa E está incorreta. Must até pode ter ideia de dedução, em alguns casos, mas must
not é proibição.
GABARITO: A

QUESTÃO 02 (EEAR/INÉDITA) – The pronoun his, (line 1), is classified as:


(A) object pronoun
(B) possessive pronoun
(C) possessive adjective
(D) subject pronoun
(E) reflexive pronoun
Comentários: A alternativa A está incorreta. O pronome “his” não é um object pronoun.
A alternativa B está incorreta. O pronome “his”, em alguns casos, pode ser um possessive
pronoun, todavia, como ele está ao lado de um substantivo ele é um pronome possessivo adjetivo
(possessive adjective).
A alternativa C está correta. Observe que o pronome “his” está ao lado do substantivo “love” e
portanto ele é um pronome possessivo adjetivo (possessive adjective).
A alternativa D está incorreta. O pronome “his” não é um subject pronoun.
A alternativa E está incorreta. O pronome “his” não é um pronome reflexivo.
GABARITO: C

QUESTÃO 03 (EEAR/INÉDITA) – Fill in the blank with the suitable option:


(A) Besides
(B) Because
(C) Although
(D) Therefore
(E) However
Comentários: A alternativa A está incorreta. Besides = Além de. Conjunção aditiva.
A alternativa B está incorreta. Because = Porque. Conjunção explicativa.
A alternativa C está incorreta. Although = Embora. Conjunção concessiva
A alternativa D está correta. Therefore = Portanto. Conjunção Conclusiva. “Therefore” é usado
para expressar a consequência de algo dito anteriormente. “Let us allow the Lord to teach us what
it means to hope. Therefore, let us listen to the words of Sacred Scripture.” (Vamos permitir que
o Senhor nos ensine o que significa ter esperança. Portanto, ouçamos as palavras da Sagrada
Escritura.)
A alternativa E está incorreta. However = Porém. Conjunção adversativa.
GABARITO: D

AULA 07 – LINKING WORDS (CONECTIVES) AND QUESTION WORDS (WH QUESTIONS) 79


TEACHER ANDREA BELO

Read text II and answer questions 04 and 05 based on it.


Text II

QUESTÃO 04 (EEAR/INÉDITA) – What is the main verb tense used in the text above?
(A) Simple Past
(B) Past Perfect
(C) Simple Present
(D) Past Perfect Continuous
(E) Present Perfect Continuous
Comentários: A alternativa A está correta. There was = havia, was = estava, ate = comeu, chewed
= mastigou. Todos os verbos citados anteriormente encontram-se no simple past.
A alternativa B está incorreta. O tempo verbal principal utilizado no texto acima é o simple past.
A alternativa C está incorreta. O tempo verbal principal utilizado no texto acima é o simple past.
A alternativa D está incorreta. O tempo verbal principal utilizado no texto acima é o simple past.
A alternativa E está incorreta. O tempo verbal principal utilizado no texto acima é o simple past.
GABARITO: A

QUESTÃO 05 (EEAR/INÉDITA) – Choose the best alternative according to the dialogue in the
cartoon
(A) The boy does not seem irritated by the attitude of his cat and dog.
(B) According to the boy, Garfield was happy and thin because he had had a good breakfast.
(C) We can see in the text that the boy was being ironic and he was irritated by the attitude of
the dog and the cat.
(D) Garfiel fails to realize that he had done something wrong.
(E) The dog ate the boy's breakfast while garfield chewed up his slipper.

AULA 07 – LINKING WORDS (CONECTIVES) AND QUESTION WORDS (WH QUESTIONS) 80


TEACHER ANDREA BELO

Comentários: A alternativa A está incorreta. De acordo com o cartoon, o garoto estava claramente
irritado com a atitude do Garfield e do cachorro.
A alternativa B está incorreta. De acordo com o cartoon, o gato era gordo (“the cat was fat”) e o
cacho que estava feliz.
A alternativa C está correta. De acordo com o cartoon, o garoto estava sendo irônico ao narrar
uma suposta história em quadrinhos mas na verdade estava relatando um fato ocorrido e
mostrava-se irritado com a atitude de Garfield e do cachorro.
A alternativa D está incorreta. No final do cartoon, Garfield diz que a história parecia familiar, e
portanto mostrando que tinha noção de que havia feito algo que tinha desagradado o garoto.
A alternativa E está incorreta. De acordo com o cartoon, Garfield comeu o café da manhã do
menino e o cachorro é quem rasgou o chinelo dele.
GABARITO: C

QUESTÃO 06 (EEAR/INÉDITA) – Choose the option with the correct tag questions for the
sentences below.
1) Listen to me, _________________?
2) Let's go to the movies, _______________?
3) I’m right, ________________?
(A) won’t you / won’t we / aren’t.
(B) won’t you / shall we / aren’t I.
(C) will you /shall we / aren’t I.
(D) will you / won’t we / am I not.
(E) will you / shan’t we / am I not.
Comentários: A alternativa A está incorreta. A sequência correta é will you /shall we / aren’t I.
A alternativa B está incorreta. A sequência correta é will you /shall we / aren’t I.
A alternativa C está correta. A sequência correta é will you /shall we / aren’t I.
1. Observe que a oração “Listen to me” encontra-se no imperativo e a question tag de verbos no
imperativo é sempre “will you”
2. A question tag de “let’s” é sempre “shall we”.
3. Cuidado com essa opção pois a question tag de “I am” não é “am not I” e sim “aren’t I”.
A alternativa D está incorreta. A sequência correta é will you /shall we / aren’t I.
A alternativa E está incorreta. A sequência correta é will you /shall we / aren’t I.
GABARITO: C

AULA 07 – LINKING WORDS (CONECTIVES) AND QUESTION WORDS (WH QUESTIONS) 81


TEACHER ANDREA BELO

Read text III and answer questions 07 and 08 based on it.


Text III
Adele – Someone Like You
I heard that you're settled down
That you found a girl and you're married now
I heard that your dreams came true
Guess she gave you things, I didn't give to you
Old friend, why are you so shy?
Ain't like you to hold back or hide from the light
I hate to turn up out of the blue, uninvited
But I couldn't stay away, I couldn't fight it
I had hoped you'd see my face
And that you'd be reminded that for me, it isn't over
(Adapted from https://www.letras.mus.br/adele/1778689/)

QUESTÃO 07 (EEAR/INÉDITA) – What is the main verb tense used in the first paragraph of the
text above?
(A) Present Perfect.
(B) Past Perfect.
(C) Simple Present.
(D) Past Perfect Continuous.
(E) Simple Past.
Comentários: A alternativa A está incorreta. O tempo verbal predominante no primeiro parágrafo
do texto é o simple past.
A alternativa B está incorreta. O tempo verbal predominante no primeiro parágrafo do texto é o
simple past.
A alternativa C está incorreta. O tempo verbal predominante no primeiro parágrafo do texto é o
simple past.
A alternativa D está incorreta. O tempo verbal predominante no primeiro parágrafo do texto é o
simple past.
A alternativa E está correta. “Heard” é o passado do verbo hear (ouvir), “settled down” é o
passado de settle down (sossegar), “found” é o passado de find (encontrar), “came true” é o
passado de come true (tornar-se realidade), gave é o passado de give (dar). Todos os verbos
citados anteriormente estão no simple past.
GABARITO: E

AULA 07 – LINKING WORDS (CONECTIVES) AND QUESTION WORDS (WH QUESTIONS) 82


TEACHER ANDREA BELO

QUESTÃO 08 (EEAR/INÉDITA) – In the excerpt “I hate to turn up out of the blue, uninvited” the
expression in bold means
(A) blue as the sky.
(B) unexpectedly.
(C) once in a blue moon.
(D) rarely.
(E) promptly.
Comentários: A alternativa A está incorreta. A expressão “out of the blue” não significa blue as
the sky (azul como o céu).
A alternativa B está correta. A expressão “out of the blue” significa aparecer do nada, sem aviso.
E portanto, é sinônimo de inesperadamente (unexpectedly)
A alternativa C está incorreta. A expressão “once in a blue moon” significa raramente e portanto,
não é sinônimo de “out of the blue” que significa inesperadamente.
A alternativa D está incorreta. O advérbio “raraly” (raramente) não é sinônimo de “out of the
blue” que significa inesperadamente.
A alternativa E está incorreta. O advérbio “promptly” (prontamente) não é sinônimo de “out of
the blue” que significa inesperadamente.
GABARITO: B

QUESTÃO 09 (EEAR/INÉDITA) – Choose the option that contains only uncountable nouns
(A) beauty, iron, information, newspaper, baggage.
(B) beauty, bee, information, newspaper, wool.
(C) song, iron, information, paper, wool.
(D) song, bee, information, newspaper, baggage.
(E) beauty, iron, information, paper, wool.
Comentários: A alternativa A está incorreta. Newspaper = jornal (contável)
A alternativa B está incorreta. Bee = abelha e Newspaper = jornal (contáveis)
A alternativa C está incorreta. Song = música (contável). Cuidado, pois o substantivo “music” é
incontável.
A alternativa D está incorreta. Song = música, Bee = abelha e Newspaper = jornal (contáveis)
A alternativa E está correta. Todos os substantivos da lista são incontáveis.
GABARITO: E

AULA 07 – LINKING WORDS (CONECTIVES) AND QUESTION WORDS (WH QUESTIONS) 83


TEACHER ANDREA BELO

Read text IV and answer question 10 based on it.


Text IV
There’s no end to the benefits of donating blood for those who need it.
According to the American Red Cross, one donation can save as many as three lives, and someone
in the United States needs blood every two seconds.
It turns out that donating blood doesn’t just benefit recipients. There are health benefits for
donors, too, on top of the benefits that come from helping others.
Donating blood has benefits for your emotional and physical health. According to a report by the
Mental Health Foundation, helping others can: reduce stress, improve your emotional well-being,
benefit your physical health, help get rid of negative feelings, provide a sense of belonging and
reduce isolation.
Research has found further evidence of the health benefits that come specifically from donating
blood.

QUESTÃO 10 (EEAR/INÉDITA) – “According to a report by the Mental Health Foundation,


helping others can: reduce stress, […], help get rid of negative feelings […]”. The expression “get
rid of” in bold in the text means
(A) put away
(B) throw away
(C) put up with
(D) find out
(E) work out
Comentários: A alternativa A está incorreta. put away = guardar, prender.
A alternativa B está correta. A expressão “to get rid of” significa “livrar-se de” e portanto é
sinônimo de “to throw away” que significa “jogar fora”.
A alternativa C está incorreta. put up with = tolerar.
A alternativa D está incorreta. find out = descobrir.
A alternativa E está incorreta. work out = malhar (academia).
GABARITO: B

QUESTÕES EFOMM
Online school to continue into next academic year amid risk of further disruption
A virtual school will stay open for at least most of the next academic year amid a rise in the number
of pupils selfisolating.
Oak National Academy has been used by pupils and teachers to support remote learning - which
has taken place during lockdowns and - during the Covid pandemic.

AULA 07 – LINKING WORDS (CONECTIVES) AND QUESTION WORDS (WH QUESTIONS) 84


TEACHER ANDREA BELO

The Reach Foundation, which the school is part of, said it will keep operating at no cost for at least
the next two terms.
The charity said this will help teachers to plan for the start of the next academic year by being
able to access Oak National Academy’s online lessons and learning resources.
The announcement comes against a backdrop of rising Covid infections in the UK and the large
numbers of children self-isolating as the school year drew to a close.
More than one million children were out of school last week due to Covid-related reasons - such
as a positive test, suspected Covid case or being told to isolate - just before schools broke up for
the summer holiday.
In the final weeks of term, over 300,000 pupils used Oak National Academy’s online learning
lessons and resources.
Schools are already considering what measures they may need to put in place should pupils test
positive, or they are asked to self-isolate in the next academic year.
The extension of the virtual school into the autumn and spring terms is backed by £2.1m in funding
from the Department for Education (DfE).
Ed Vainker from the Reach Foundation - which has incubated Oak since its launch - said the charity
understood teachers wanted the online school to continue.
“We are also mindful that Covid-19 will be with us in some form for the foreseeable future, and
the autumn and winter may bring further disruption,” the chief executive said.
“I’m therefore pleased the DfE will continue to support Oak, so it remains open and free as part
of resilience planning for the next two terms.”
Nick Gibb, the schools minister, said: “From being set up by a small group of teachers in under
two weeks, Oak National Academy has become one of the great success stories as we tackle the
pandemic.”
He said the online school helped millions of pupils and teachers while most children were not
allowed to set foot in school during a lockdown last spring and one at the start of this year.
But even since after all students were allowed back on since in March, Oak National Academy said
30,000 teachers have used its services a week, with demand rising amid the spread of the Delta
variant.
“The impact Oak has made and the good it has done for the sector and children is immeasurable,
and we will now look for the best way to harness that for the future,” Mr Gibb added.
https://www.independent.co.uk/news/education/education-news/uk-online-school-two-terms-b1888491.html

AULA 07 – LINKING WORDS (CONECTIVES) AND QUESTION WORDS (WH QUESTIONS) 85


TEACHER ANDREA BELO

QUESTÃO 01 (EFOMM/INÉDITA) – In the extract from the text: “The announcement comes
against a backdrop of rising Covid infections in the UK [...]” (paragraph 5). The word in bold is
closest in meaning to
A) Scenery
B) Decrease
C) Increase
D) Scene
E) View
Comentários: A alternativa A está correta. A palavra “backdrop” significa “cenário” e pode ser
comparada com a palavra “scenery”, que significa “cenário”.
A alternativa B está incorreta. A palavra “backdrop” significa “cenário” e não pode ser comparada
com a palavra “decrease”, que significa “diminuição”.
A alternativa C está incorreta. A palavra “backdrop” significa “cenário” e não pode ser comparada
com a palavra “increase”, que significa “aumento”.
A alternativa D está incorreta. A palavra “backdrop” significa “cenário” e não pode ser comparada
com a palavra “scene”, que significa “cena”.
A alternativa E está incorreta. A palavra “backdrop” significa “cenário” e não pode ser comparada
com a palavra “view”, que significa “vista”.
GABARITO: A

QUESTÃO 02 (EFOMM/INÉDITA) – In the extract from the text: “We are also mindful that Covid-
19 will be with us in some form for the foreseeable future, and the autumn and winter may
bring further disruption” (paragraph 12), the verb “to bring” means
A) To refuse
B) To carry
C) To hold
D) To serve
E) To consider
Comentários: A alternativa A está incorreta. O verbo “to bring” significa “trazer” e não pode ser
comparado ao verbo “to refuse”, que significa “recusar”.
A alternativa B está correta. O verbo “to bring” significa “trazer” e pode ser comparado ao verbo
“to carry”, que significa “carregar/trazer”.
A alternativa C está incorreta. O verbo “to bring” significa “trazer” e não pode ser comparado ao
verbo “to hold”, que significa “segurar”.
A alternativa D está incorreta. O verbo “to bring” significa “trazer” e não pode ser comparado ao
verbo “to serve”, que significa “servir”.
A alternativa E está incorreta. O verbo “to bring” significa “trazer” e não pode ser comparado ao
verbo “to consider”, que significa “considerar”.
GABARITO: B

AULA 07 – LINKING WORDS (CONECTIVES) AND QUESTION WORDS (WH QUESTIONS) 86


TEACHER ANDREA BELO

QUESTÃO 03 (EFOMM/INÉDITA) – What is true according to the text?


A) Despite the increase in the number of self-isolated students, a virtual school has decided to
close its doors
B) Oak National Academy lost its popularity during the period of social isolation resulting from
the Covid-19 pandemic
C) More than a million students were out of school last week because of the summer break
D) The next school year will not need to have protective measures against Covid-19
E) Given the pandemic conditions, Oak will continue to act as a great help to students and
teachers
Comentários: A alternativa A está incorreta. De acordo com o texto, não é correto afirmar que
apesar do aumento do número de alunos isolados, uma escola virtual decidiu fechar suas portas,
mas sim, que uma escola virtual vai continuar aberta. Isso pode ser confirmado com o trecho “A
virtual school will stay open for at least most of the next academic year amid a rise in the number
of pupils self-isolating”.
A alternativa B está incorreta. De acordo com o texto, não é correto afirmar que Oak National
Academy perdeu sua popularidade durante o período de isolamento social resultante da
pandemia Covid-19, mas sim, que ela tem sido muito usada por estudantes e professores para
ajudar no ensino remoto, ou seja, ganhou popularidade. Isso pode ser confirmado com o trecho
“Oak National Academy has been used by pupils and teachers to support remote learning - which
has taken place during lockdowns and - during the Covid pandemic”.
A alternativa C está incorreta. De acordo com o texto, não é correto afirmar que mais de um
milhão de alunos estiveram fora da escola na semana passada por causa das férias de verão, mas
sim, que esses alunos estiveram fora por conta de razões relacionadas à Covid-19, isso logo antes
das férias de verão. Isso pode ser confirmado com o trecho “More than one million children were
out of school last week due to Covid-related reasons … just before schools broke up for the
summer holiday”.
A alternativa D está incorreta. De acordo com o texto, não é correto afirmar que o próximo ano
letivo não precisará ter medidas de proteção contra Covid-19, mas sim, que é possível que sejam
necessárias. Isso pode ser confirmado com o trecho “Schools are already considering what
measures they may need to put in place should pupils test positive, or they are asked to self-
isolate in the next academic year”.
A alternativa E está correta. De acordo com o texto, é correto afirmar que dadas as condições
pandêmicas, Oak continuará a atuar como uma grande ajuda para alunos e professores, assim
como esta opção indica. Isso pode ser confirmado com o trecho “...it will keep operating at no
cost for at least the next two terms … this will help teachers to plan for the start of the next
academic year by being able to access Oak National Academy’s online lessons and learning
resources”.
GABARITO: E

AULA 07 – LINKING WORDS (CONECTIVES) AND QUESTION WORDS (WH QUESTIONS) 87


TEACHER ANDREA BELO

QUESTÃO 04 (EFOMM/INÉDITA) – In the extract from the text: “[…] which has incubated Oak
since its launch […] (paragraph 10), the word “which” refers to
A) Reach foundation
B) Oak
C) Ed Vainker
D) Teachers
E) Department for Education
Comentários: A palavra “which” em “Ed Vainker da Reach Foundation - que incubou Oak desde
seu lançamento - disse que a instituição de caridade entendeu que os professores queriam que a
escola online continuasse” se refere à Reach Foundation, citada anteriormente na frase.
GABARITO: A

Read the text below to answer question 05:


Cannabis part of the future says tobacco giant
The UK's largest tobacco firm says it __________ cannabis as part of its future as it tries to move
away from selling traditional cigarettes. British American Tobacco said it wanted to "accelerate"
its transformation by reducing the health impact of its products.
https://www.bbc.com/news/business-57995285

QUESTÃO 05 (EFOMM/INÉDITA) – Fill in the blank with the suitable verb


A) Is seeing
B) Saw
C) Sees
D) Was seeing
E) Will see
Comentários: A lacuna deve ser preenchida com o verbo “to see” (ver) no simple present
referindo-se ao pronome “it”, ou seja, “sees” (vê); indo de acordo com o tempo verbal do resto
da frase: “A maior empresa de tabaco do Reino Unido diz que vê a cannabis como parte de seu
futuro, enquanto tenta deixar de vender cigarros tradicionais”.
GABARITO: C

Read the text below to answer questions 06 and 07:


A Belgian chocolate company is now ____ 3D printers which allow the company to create more
intricate, difficultto-mould chocolates. The chocolates are intended for people who seek original
designs.

AULA 07 – LINKING WORDS (CONECTIVES) AND QUESTION WORDS (WH QUESTIONS) 88


TEACHER ANDREA BELO

The chocolate is melted before _____ poured into a syringe which is attached to the printer and
since the chocolates are hard to transport, the company _____ to open other shops around the
world.
https://www.newsinlevels.com/products/chocolate-3d-printing-level-3/

QUESTÃO 06 (EFOMM/INÉDITA) – Which is the correct way to complete the text above?
A) Using / being / hopes
B) Uses / being / hoped
C) Uses / being / is hoping
D) Using / being / was hoping
E) Using / being / had hoped
Comentários: A primeira lacuna deve ser preenchida com o verbo “to use” (usar) no present
continuous “is [now] using”, indo de acordo com o tempo verbal indicado anteriormente na frase.
A segunda lacuna deve ser preenchida com o verbo “to be” (ser/estar) no present participle
“being” (ser), ou seja, “O chocolate é derretido antes de ser colocado em uma seringa...”.
A terceira lacuna deve ser preenchida com o verbo “to hope” (esperar) no simple present “hopes”
(espera), ou seja, “...a empresa espera abrir outras lojas ao redor do mundo”.
GABARITO: A

QUESTÃO 07 (EFOMM/INÉDITA) – In “…which allow the company to create more intricate…”,


the word in bold means
A) Hard
B) Intriguing
C) Fancy
D) Complex
E) Perplexing
Comentários: A alternativa A está incorreta. A palavra “intricate” significa “intrincado/complexo”
e não pode ser comparada com a palavra “hard”, que significa “difícil/duro”.
A alternativa B está incorreta. A palavra “intricate” significa “intrincado/complexo” e não pode
ser comparada com a palavra “intringuing”, que significa “intrigante”.
A alternativa C está incorreta. A palavra “intricate” significa “intrincado/complexo” e não pode
ser comparada com a palavra “fancy”, que significa “chique”.
A alternativa D está correta. A palavra “intricate” significa “intrincado/complexo” e pode ser
comparada com a palavra “complex”, que significa “complexo”.
A alternativa E está incorreta. A palavra “intricate” significa “intrincado/complexo” e não pode
ser comparada com a palavra “perplexing”, que significa “desconcertante”.
GABARITO: D

AULA 07 – LINKING WORDS (CONECTIVES) AND QUESTION WORDS (WH QUESTIONS) 89


TEACHER ANDREA BELO

QUESTÃO 08 (EFOMM/INÉDITA) – Which is the correct way to complete the paragraph below?
The small town of Pelhrimov in the Czech Republic _______ thousands of strange records and
hosted a festival to show them off. Some of the records were racing with folding ladders and
folding pink scarves. The president of the festival _______ that he was happy to have people come
and understand the ‘essence’ of what the people in this town _______. He wants to show that life
is more than being on Facebook or sitting at a computer. It is good to do something!
Adapted from: https://www.newsinlevels.com/products/town-has-strange-records-level-3/

A) Holds / said / do
B) Holds / says / do
C) Hold / said / do
D) Holds / said / does
E) Holds / says / does
Comentários: A primeira lacuna deve ser preenchida com o verbo “to hold” (segurar) no simple
present referindo-se ao pronome “it”, ou seja, “holds” (segura/possui); “A pequena cidade de
Pelhrimov na República Tcheca possui milhares de registros estranhos...”.
A segunda lacuna deve ser preenchida com o verbo “to say” (dizer) no simple past referindo-se
ao pronome “he”, ou seja, “said” (disse); “O presidente do festival disse que estava feliz...”.
A terceira lacuna deve ser preenchida com o verbo “to do” (fazer) no simple present referindo-se
ao pronome “they”, ou seja, “do” (fazem); “...do que as pessoas nesta cidade fazem”.
GABARITO: A

Read the text below to answer questions 09 and 10:


Jeff Bezos, _______ richest man ________ __ Earth, visited space on Tuesday. He and three other
crew members soared about 107 kilometers above the Texas desert aboard his company Blue
Origin´s New Shepard rocket.
_______ historic unpiloted sub-orbital flight helped to inaugurate a new era of private commercial
space tourism. The trip to the edge of space lasted about 10 minutes and 20 seconds, including a
few minutes of weightlessness. The crew included his brother Mark, Wally Funk, an 82-year-old
pioneering female aviator, and Oliver Daemen, an 18-year-old Dutch physics student.
Bezos, who recently stepped down as Amazon´s CEO, thanked employees and customers _______
the company that he founded, saying that their hard work and Amazon purchases had made his
space flight possible. His words sparked criticism, with some politicians saying that Amazon
abused its market power to hurt small businesses. They also said Bezos should have spent
_______ money to pay his workers fairly and protect their health.
https://www.newsinlevels.com/products/the-worlds-richest-man-visits-space-level-3/

AULA 07 – LINKING WORDS (CONECTIVES) AND QUESTION WORDS (WH QUESTIONS) 90


TEACHER ANDREA BELO

QUESTÃO 09 (EFOMM/INÉDITA) – Fill in the blank with the suitable prepositions


A) A / in / a / of / the
B) The / on / an / of / a
C) The / in / a / of / the
D) A / on / a / of / the
E) The / on / a / of / the
Comentários: A primeira lacuna deve ser preenchida com a preposição “the” (o), pois está se
referindo a uma pessoa específica, “...the richest man...”.
A segunda lacuna deve ser preenchida com a preposição “on” (na), pois se refere ao homem mais
rico na Terra.
A terceira lacuna deve ser preenchida com a preposição “a” (uma), pois antecede “historic”.
A quarta lacuna deve ser preenchida com a preposição “of” (da), ou seja, “...agradeceu aos
funcionários e clientes da empresa que fundou...”.
A quinta lacuna deve ser preenchida com “the” (o), pois se refere à um dinheiro específico.
GABARITO: E

QUESTÃO 10 (EFOMM/INÉDITA) – In “…an 82-year-old pioneering female aviator…”, the


expression in bold means
A) The first person in a row
B) The first person to do something
C) The last person to do something
D) The most important person to do something
E) The least important person to so something
Comentários: A alternativa A está incorreta. A expressão “pioneering” não significa a primeira
pessoa em uma fila, mas sim, a primeira pessoa a fazer algo.
A alternativa B está correta. A expressão “pioneering” significa a primeira pessoa a fazer algo,
assim como esta opção indica.
A alternativa C está incorreta. A expressão “pioneering” não significa a última pessoa a fazer algo,
mas sim, a primeira pessoa a fazer algo.
A alternativa D está incorreta. A expressão “pioneering” não significa a pessoa mais importante a
fazer algo, mas sim, a primeira pessoa a fazer algo.
A alternativa E está incorreta. A expressão “pioneering” não significa a pessoa menos importante
a fazer algo, mas sim, a primeira pessoa a fazer algo.
GABARITO: B

AULA 07 – LINKING WORDS (CONECTIVES) AND QUESTION WORDS (WH QUESTIONS) 91


TEACHER ANDREA BELO

QUESTÕES EPCAR
Directions: Answer questions 01 to 05 according to TEXT
Why We Must Improve Vaccine Manufacturing Before the Next Pandemic
It should worry everyone that experts surveyed by TIME regarded both increasing funding in a
post-COVID-19 world for vaccine development and scaling up of manufacturing capacity
feasible—but improving equitable vaccine distribution was not.
To stop the next pandemic in its tracks we need to ensure that people all over the world are
protected quickly, and that will entail having all these pieces in place. The good news is, all these
elements are feasible, and indeed starting to work today.
On vaccine R&D, the Coalition for Epidemic Preparedness Innovations (CEPI), was set up with the
precise purpose of identifying and investing in R&D for vaccines against emerging infectious
diseases with epidemic potential. So, when it came to COVID-19, with CEPI’s and other R&D
support, as well as industry engagement, the scientific and vaccine manufacturing community
rallied, producing the first safe and effective vaccine in record time—just 327 days. Today we have
not just one but 15 in widespread use.
Increased investment now could get us there even faster the next time, particularly given the
potential of the relatively new RNA vaccine technologies that have proved so effective with
COVID-19. These plug-and-play vaccine technologies not only make it possible to identify and
develop antigens rapidly, but much of the regulatory testing and approval can be done in advance,
even before we know what the threat is.
As for manufacturing, it may be difficult to immediately discern when there are severe supply
shortages, but the world has actually rapidly built up manufacturing capacity during COVID-19.
Waiving intellectual property has been talked about a lot as a potential solution for boosting
production. But the growth we have seen in the past year has been achieved through technology
transfers, where both the intellectual property and the vital know-how needed to make vaccines
is shared between manufacturers.
However, we need to do more. Given the extremely large number of doses needed during a
pandemic, export bans of vaccines and essential components and supply bottlenecks have led to
a vaccine divide. Currently, more than a third of adults in high-income countries have now been
vaccinated, while less than 1% of those in low-income countries have had their first jab.
To prevent this kind of scenario from happening the next time round and ensure that those most
at risk are prioritized wherever they are, it is not distribution channels we are lacking, but global
manufacturing capacity. We already have highly effective distribution channels, through COVAX
and its partners, and we already have access to doses, enough to protect 1.8 billion people in
lower-income economies by early next year, enough to protect almost 30% of people in these
countries.
But through investments now to increase global manufacturing capacity, particularly in emerging
economies, and support of technology transfers, the next time a pandemic strikes we can get
there sooner.
(Adapted from https://time.com/6072101/improve-vaccine-manufacturing/)

AULA 07 – LINKING WORDS (CONECTIVES) AND QUESTION WORDS (WH QUESTIONS) 92


TEACHER ANDREA BELO

QUESTÃO 01 (EPCAR/INÉDITA) – Mark the option that can replace the word “improve” in the
title without changing its meaning
A) Blow
B) Help
C) Recuperate
D) Enhance
Comentário: A alternativa A está incorreta. A palavra “improve” significa “melhorar” e não pode
ser substituída pela palavra “blow”, que significa “explodir”.
A alternativa B está incorreta. A palavra “improve” significa “melhorar” e não pode ser substituída
pela palavra “help”, que significa “ajudar”.
A alternativa C está incorreta. A palavra “improve” significa “melhorar” e não pode ser substituída
pela palavra “recuperate”, que significa “recuperar”.
A alternativa D está correta. A palavra “improve” significa “melhorar” e pode ser substituída pela
palavra “enhance”, que significa “aprimorar”.
GABARITO: D

QUESTÃO 02 (EPCAR/INÉDITA) – We can deduce from the first paragraph that


A) Experts do not believe that increasing manufacturing capacity is feasible
B) Improving an equitable distribution of vaccines should be considered feasible
C) Investment in vaccine development is not appropriate in the post-pandemic world
D) There was an equitable distribution of vaccines in the COVID-19 pandemic
Comentário: A alternativa A está incorreta. De acordo com o primeiro parágrafo, não se pode
deduzir que os especialistas não acreditam que o aumento da capacidade de fabricação seja
viável, mas sim, que eles acreditam que é viável. Isso pode ser confirmado com o trecho “...experts
surveyed by TIME regarded … scaling up of manufacturing capacity feasible…”.
A alternativa B está correta. De acordo com o primeiro parágrafo, pode-se deduzir que melhorar
uma distribuição equitativa de vacinas deve ser considerado viável, assim como esta opção indica.
Isso pode ser confirmado com o trecho “It should worry everyone that experts surveyed by TIME
regarded both increasing … and scaling up of manufacturing capacity feasible—but improving
equitable vaccine distribution was not”.
A alternativa C está incorreta. De acordo com o primeiro parágrafo, não se pode deduzir que o
investimento no desenvolvimento de vacinas não é apropriado no mundo pós-pandêmico, mas
sim, que os especialistas consideram isso viável, ou seja, também é apropriado. Isso pode ser
confirmado com o trecho “...experts surveyed by TIME regarded both increasing funding in a post-
COVID-19 world for vaccine development and scaling up of manufacturing capacity feasible…”.
A alternativa D está incorreta. De acordo com o primeiro parágrafo, não se pode deduzir que
houve uma distribuição equitativa de vacinas na pandemia de COVID-19, pois o parágrafo nos
insinua que não houve essa distribuição. Isso pode ser confirmado com o trecho “It should worry

AULA 07 – LINKING WORDS (CONECTIVES) AND QUESTION WORDS (WH QUESTIONS) 93


TEACHER ANDREA BELO

everyone that experts surveyed by TIME regarded both increasing … and scaling up of
manufacturing capacity feasible—but improving equitable vaccine distribution was not”.
GABARITO: B

QUESTÃO 03 (EPCAR/INÉDITA) – Mark the option with the suitable question to answer the
underlined fragment below
“To stop the next pandemic in its tracks we need to ensure that people all over the world are
protected quickly, and that will entail having all these pieces in place”.
A) How will mass vaccination work in a possible next pandemic?
B) How can the next pandemic occur?
C) What can be done to stop a possible next pandemic from going forward?
D) What are the advantages of vaccination?
Comentário: A alternativa A está incorreta. A frase “Para impedir a próxima pandemia em
andamento, precisamos garantir que as pessoas em todo o mundo sejam protegidas
rapidamente” não responde à pergunta “Como funcionará a vacinação em massa em uma possível
próxima pandemia?”, mas sim, à pergunta “O que pode ser feito para impedir o avanço de uma
possível próxima pandemia?”.
A alternativa B está incorreta. A frase “Para impedir a próxima pandemia em andamento,
precisamos garantir que as pessoas em todo o mundo sejam protegidas rapidamente” não
responde à pergunta “Como pode ocorrer a próxima pandemia?”, mas sim, à pergunta “O que
pode ser feito para impedir o avanço de uma possível próxima pandemia?”.
A alternativa C está correta. A frase “Para impedir a próxima pandemia em andamento,
precisamos garantir que as pessoas em todo o mundo sejam protegidas rapidamente” responde
à pergunta “O que pode ser feito para impedir o avanço de uma possível próxima pandemia?”,
assim como esta opção indica.
A alternativa D está incorreta. A frase “Para impedir a próxima pandemia em andamento,
precisamos garantir que as pessoas em todo o mundo sejam protegidas rapidamente” não
responde à pergunta “Quais são as vantagens da vacinação?”, mas sim, à pergunta “O que pode
ser feito para impedir o avanço de uma possível próxima pandemia?”.
GABARITO: C

QUESTÃO 04 (EPCAR/INÉDITA) – Mark the option that can replace the underlined word keeping
the same meaning
“To stop the next pandemic in its tracks we need to ensure that people all over the world are
protected quickly, and that will entail having all these pieces in place”.
A) Exclude
B) Encompass
C) Indeed
D) Rid

AULA 07 – LINKING WORDS (CONECTIVES) AND QUESTION WORDS (WH QUESTIONS) 94


TEACHER ANDREA BELO

Comentário: A alternativa A está incorreta. A palavra “entail” significa implicar e não pode ser
substituída pela palavra “exclude”, que significa excluir.
A alternativa B está correta. A palavra “entail” significa implicar e pode ser substituída pela palavra
“encompass”, que significa englobar.
A alternativa C está incorreta. A palavra “entail” significa implicar e não pode ser substituída pela
palavra “indeed”, que significa de fato.
A alternativa D está incorreta. A palavra “entail” significa implicar e não pode ser substituída pela
palavra “rid”, que significa livrar/livrar-se.
GABARITO: B

QUESTÃO 05 (EPCAR/INÉDITA) – Mark the alternative that can answer the question below
according to the text
How was the COVID-19 vaccine development process?
A) It was unusually fast and effective
B) It was fast, like all vaccine development
C) It was time-consuming but effective
D) It was time-consuming and ineffective
Comentário: A alternativa A está correta. A pergunta “Como foi o processo de desenvolvimento
da vacina COVID-19?” é bem respondida com “Foi excepcionalmente rápido e eficaz”, assim como
esta opção indica. Isso pode ser confirmado com o trecho “...producing the first safe and effective
vaccine in record time—just 327 days”.
A alternativa B está incorreta. A pergunta “Como foi o processo de desenvolvimento da vacina
COVID-19?” não é bem respondida com “Foi rápido, como todo desenvolvimento de vacina”, mas
sim, com “Foi excepcionalmente rápido e eficaz”. Isso pode ser confirmado com o trecho
“...producing the first safe and effective vaccine in record time—just 327 days”.
respondida com “Foi demorado, mas eficaz”, mas sim, com “Foi excepcionalmente rápido e
eficaz”. Isso pode ser confirmado com o trecho “...producing the first safe and effective vaccine in
record time—just 327 days”.
A alternativa D está incorreta. A pergunta “Como foi o processo de desenvolvimento da vacina
COVID-19?” não é bem respondida com “Foi demorado e ineficaz”, mas sim, com “Foi
excepcionalmente rápido e eficaz”. Isso pode ser confirmado com o trecho “...producing the first
safe and effective vaccine in record time—just 327 days”.
GABARITO: A

AULA 07 – LINKING WORDS (CONECTIVES) AND QUESTION WORDS (WH QUESTIONS) 95


TEACHER ANDREA BELO

QUESTÕES ESA
Read the text and answer questions 01, 02 and 03
Children who ate ‘cannabis sweets’ risked serious harm, say Surrey police
Investigation begins after two boys, 12 and 13, and a girl, 12, were taken to hospital after falling unconscious
Four children who were taken to hospital after eating sweets they thought contained cannabis
are not believed to have suffered long-term effects, police have said.
The youngsters were found vomiting uncontrollably and falling in and out of consciousness on
Saturday afternoon on Pound Lane in Epsom, according to Surrey police. A 12-year-old girl, 12-
year-old boy and 13-year-old boy were taken to hospital by the South East Coast ambulance
service and are lucky not to have been more seriously affected by whatever they ate, the police
said.
They suffered “a violent reaction from eating the jelly sweets that they believed contained
cannabis”, police said.
Another 12-year-old boy was taken to hospital by his family because he was also believed to have
eaten the sweets.
One of the 12-year-old boys was discharged from hospital late on Saturday and the other three
were expected to leave on Sunday. The source of the sweets is still being investigated and police
do not have a clear idea of what they look like, although a similar report from Friday night involved
“jelly apple rings” which look like green jelly circles.
DS Lisa Betchley said: “These children were incredibly lucky that they were not more seriously
affected by whatever it was that they ate – and this is thanks, in great part, to the prompt actions
of two medical students who happened to be nearby and assisted in the early stages, as well as
the South East Coast ambulance service and hospital staff for their rapid response and treatment.
(Adapted from https://www.theguardian.com/uk-news/2021/may/02/children-who-ate-cannabis-sweets-risked-serious-harm-say-surrey-police)

QUESTÃO 01 (ESA/INÉDITA) – Concerning the information in the text, is correct to state that
(A) The impacts caused by the food eaten by the children are likely to be short-term
(B) The children's symptoms were severe headaches and nausea
(C) The content of the sweets is still completely unknown
(D) What happened to the children reached the worst possible scenario
(E) The children were saved, mainly, by the small amount of toxic food ingested
Comentários: A alternativa A está correta. De acordo com o texto, é correto afirmar que os
impactos causados pelos alimentos ingeridos pelas crianças são provavelmente de curto prazo,
assim como esta opção indica. Isso pode ser confirmado com o trecho “Four children who were
taken to hospital after eating sweets they thought contained cannabis are not believed to have
suffered long-term effects…”.
A alternativa B está incorreta. De acordo com o texto, não é correto afirmar que os sintomas das
crianças eram fortes dores de cabeça e náuseas, mas sim, que eram vômitos intensos e perda de
consciência. Isso pode ser confirmado com o trecho “The youngsters were found vomiting

AULA 07 – LINKING WORDS (CONECTIVES) AND QUESTION WORDS (WH QUESTIONS) 96


TEACHER ANDREA BELO

uncontrollably and falling in and out of consciousness on Saturday afternoon on Pound Lane in
Epsom, according to Surrey police”.
A alternativa C está incorreta. De acordo com o texto, não é correto afirmar que o conteúdo dos
doces ainda é completamente desconhecido, mas sim, que é possível que tivesse maconha na
composição. Isso pode ser confirmado com o trecho “Four children who were taken to hospital
after eating sweets they thought contained cannabis are not believed to have suffered long-term
effects…”.
A alternativa D está incorreta. De acordo com o texto, não é correto afirmar que o que aconteceu
com as crianças atingiu o pior cenário possível, mas sim, que elas tiveram sorte de não ter sido
pior. Isso pode ser confirmado com o trecho “These children were incredibly lucky that they were
not more seriously affected by whatever it was that they ate…”.
A alternativa E está incorreta. De acordo com o texto, não é correto afirmar que as crianças foram
salvas, principalmente, pela pequena quantidade de alimentos tóxicos ingeridos, mas sim, que foi
principalmente pelo trabalho dos outros. Isso pode ser confirmado com o trecho “...and this is
thanks, in great part, to the prompt actions of two medical students who happened to be nearby
and assisted in the early stages, as well as the South East Coast ambulance service and hospital
staff for their rapid response and treatment”.
GABARITO: A

QUESTÃO 02 (ESA/INÉDITA) – In the sentence “One of the 12-year-old boys was discharged
from hospital late on Saturday and the other three were expected to leave on Sunday”
(paragraph 4), the word “discharged” means
(A) Accomplished
(B) Done
(C) Dismissed
(D) Retired
(E) Hospitalized
Comentários: A alternativa A está incorreta. A palavra “discharged” significa dispensado e não
pode ser comparada com a palavra “accomplished”, que significa realizado.
A alternativa B está incorreta. A palavra “discharged” significa dispensado e não pode ser
comparada com a palavra “done”, que significa feito.
A alternativa C está correta. A palavra “discharged” significa dispensado e pode ser comparada
com a palavra “dismissed”, que significa dispensado.
A alternativa D está incorreta. A palavra “discharged” significa dispensado e não pode ser
comparada com a palavra “retired”, que significa aposentado.
A alternativa E está incorreta. A palavra “discharged” significa dispensado e não pode ser
comparada com a palavra “hospitalized”, que significa internado/hospitalizado.
GABARITO: C

AULA 07 – LINKING WORDS (CONECTIVES) AND QUESTION WORDS (WH QUESTIONS) 97


TEACHER ANDREA BELO

QUESTÃO 03 (ESA/INÉDITA) – The words ATE and WERE are


(A) Regular verbs
(B) Not verbs
(C) Modal verbs
(D) From different grammatical classes
(E) Irregular verbs
Comentários: A alternativa A está incorreta. As palavras “ate” (comeram) e “were” (foram) não
são verbos regulares, mas sim, verbos irregulares por terem sua forma passada única sem seguir
o sufixo -ed.
A alternativa B está incorreta. As palavras “ate” (comeram) e “were” (foram) são verbos
irregulares por terem sua forma passada única sem seguir o sufixo -ed.
A alternativa C está incorreta. As palavras “ate” (comeram) e “were” (foram) não são verbos
modais, mas sim, verbos irregulares por terem sua forma passada única sem seguir o sufixo -ed.
A alternativa D está incorreta. As palavras “ate” (comeram) e “were” (foram) não são de
diferentes classes gramaticais, mas sim, ambos verbos irregulares por terem sua forma passada
única sem seguir o sufixo -ed.
A alternativa E está correta. As palavras “ate” (comeram) e “were” (foram) são verbos irregulares
por terem sua forma passada única sem seguir o sufixo -ed, assim como esta opção indica.
GABARITO: E

Read the text and answer questions 04 and 05


Will Covid-19 vaccines reduce virus transmission?
Vaccinated people can still get infected, but they are less likely to pass it on
There are two ways that getting vaccinated can slow the spread of the virus. First, _______ (1)
can help prevent you getting infected. Second, even if you are unlucky and catch the virus, it may
reduce the risk of passing it on. It is crucial to understand how big these benefits are.
Two huge new studies have taken advantage of the successful UK vaccine rollout. An Oxford-ONS
analysis _______ (2) more than 370,000 survey participants found infections were reduced by
65% after a single dose. For protection against the virus, one dose was similar _______ (3) having
had a prior infection. There was no major difference between the two available vaccines.
Curiously, infection rates were lower up to three weeks before the jab. Did the virus have magical
pre-cognition and keep away? More plausible is the idea of “reverse causation”. People can have
the vaccination only if they have not tested positive or shown recent symptoms, so it is inevitable
there were fewer recorded infections before vaccinations took place. Statistics can be tricky
things.
Most important, the studies showed that if you are infected after vaccination, it tends to be much
milder, both in terms of self-reported symptoms and viral load.
If vaccinated people develop a weaker infection, then they might be less likely to pass on the virus.
(Adapted from https://www.theguardian.com/theobserver/commentisfree/2021/may/02/vaccinated-people-less-likely-to-pass-covid-on)

AULA 07 – LINKING WORDS (CONECTIVES) AND QUESTION WORDS (WH QUESTIONS) 98


TEACHER ANDREA BELO

QUESTÃO 04 (ESA/INÉDITA) – Concerning the information in the text, is correct to say that
(A) The vaccine completely prevents the organism from being infected by the virus
(B) Vaccines have the potential to significantly reduce the rate of virus infection
(C) The first dose of the vaccine starts to take effect when the second dose is applied
(D) The two vaccines studied and cited in the text had significant differences in their results
(E) The drop in the rate of infections in the period prior to vaccine doses occurred by pure
coincidence
Comentários: A alternativa A está incorreta. De acordo com o texto, não é correto afirmar que a
vacina evita completamente que o organismo seja infectado pelo vírus, mas sim, que previne a
infecção, não impede. Isso pode ser confirmado com o trecho “There are two ways that getting
vaccinated can slow the spread of the virus. First, it can help prevent you getting infected…”.
A alternativa B está correta. De acordo com o texto, é correto afirmar que as vacinas têm o
potencial de reduzir significativamente a taxa de infecção do vírus, assim como esta opção indica.
Isso pode ser confirmado com o trecho “There are two ways that getting vaccinated can slow the
spread of the virus. First, it can help prevent you getting infected. Second, even if you are unlucky
and catch the virus, it may reduce the risk of passing it on”.
A alternativa C está incorreta. De acordo com o texto, não é correto afirmar que a primeira dose
da vacina começa a fazer efeito quando a segunda dose é aplicada, mas sim, que as infecções
diminuíram 65% em voluntários após a primeira dose. Isso pode ser confirmado com o trecho “An
Oxford-ONS analysis of more than 370,000 survey participants found infections were reduced by
65% after a single dose”.
A alternativa D está incorreta. De acordo com o texto, não é correto afirmar que as duas vacinas
estudadas e citadas no texto tiveram diferenças significativas em seus resultados, mas sim, que
não houve grandes diferenças entre as duas vacinas. Isso pode ser confirmado com o trecho
“There was no major difference between the two available vaccines”.
A alternativa E está incorreta. De acordo com o texto, não é correto afirmar que a queda na taxa
de infecções no período anterior às doses da vacina ocorreu por pura coincidência, mas sim, que
provavelmente havia um motivo. Isso pode ser confirmado com o trecho “More plausible is the
idea of “reverse causation”. People can have the vaccination only if they have not tested positive
or shown recent symptoms, so it is inevitable there were fewer recorded infections before
vaccinations took place”.
GABARITO: B

QUESTÃO 05 (ESA/INÉDITA) – In the sentence “Did the virus have magical precognition and
keep away? More plausible is the idea of ‘reverse causation’” (paragraph 3), the word
“plausible” means
(A) Reasonable
(B) Improbable
(C) Irrational
(D) Partial
(E) Impartial

AULA 07 – LINKING WORDS (CONECTIVES) AND QUESTION WORDS (WH QUESTIONS) 99


TEACHER ANDREA BELO

Comentários: A alternativa A está correta. A palavra “plausible” significa plausível e pode ser
comparada com a palavra “reasonable”, que significa razoável.
A alternativa B está incorreta. A palavra “plausible” significa plausível e não pode ser comparada
com a palavra “improbable”, que significa improvável.
A alternativa C está incorreta. A palavra “plausible” significa plausível e não pode ser comparada
com a palavra “irrational”, que significa irracional.
A alternativa D está incorreta. A palavra “plausible” significa plausível e não pode ser comparada
com a palavra “partial”, que significa parcial.
A alternativa E está incorreta. A palavra “plausible” significa plausível e não pode ser comparada
com a palavra “impartial”, que significa imparcial.
GABARITO: A

QUESTÕES ESCOLA NAVAL


Based on the text below, answer the six questions that follow it.
Any amount of alcohol consumption harmful to the brain, finds study
UK study of 25,000 people finds even moderate drinking is linked to lower grey matter density
There is no safe amount of alcohol consumption for the brain, with even “moderate” drinking
adversely affecting nearly every part of it, a study of more than 25,000 people in the UK has found.
The study, which is still to be peer-reviewed, suggests that the more alcohol consumed, the lower
the brain volume. In effect, the more you drink, the worse off your brain.
“There’s no threshold drinking for harm – any alcohol is worse. Pretty much the whole brain seems
to be affected – not just specific areas, as previously thought,” said the lead author, Anya Topiwala,
a senior clinical lecturer at the University of Oxford.
Using the UK Biobank, a substantial database designed to help researchers decode the genetic
and environmental factors that lead some people to develop diseases while others do not,
researchers in this study analysed data from 25,378 participants such as age, sex, education, self-
reported alcohol consumption, brain size and health from MRI scans, information about hospital
and outpatient visits, and memory tests.
Higher volume of alcohol consumption per week was associated with lower grey matter density –
the researchers found, with alcohol explaining up to a 0.8% change in grey matter volume, even
after accounting for individual biological and behavioural characteristics.
This might seem like a small figure, but it is a larger contribution than any other modifiable risk
factors. For example, it is four times the contribution of smoking or BMI, said Topiwala.
Widespread negative associations were also seen between alcohol consumption and integrity of
white matter, the brain fibres that scaffold the billions of neurons that make up grey matter. In
addition, an individual’s underlying conditions such as high blood pressure and high BMI made the
negative association between alcohol and brain health stronger, the researchers found.
Contrary to previous research that suggested there is a benefit to drinking wine in moderation
compared with beer or spirits, the study found no evidence to suggest alcoholic beverage type
conferred differences in risks to the brain.
(Adapted from https://www.theguardian.com/society/2021/may/18/any-amount-of-alcohol-consumption-harmful-to-the-brain-finds-study)

AULA 07 – LINKING WORDS (CONECTIVES) AND QUESTION WORDS (WH QUESTIONS) 100
TEACHER ANDREA BELO

QUESTÃO 01 (ESCOLA NAVAL/INÉDITA) – According to the text, which option is correct?


(A) Consumption of small amounts of alcohol is safe and beneficial to the brain
(B) Brain volume increases with increasing alcohol consumption
(C) Alcohol consumption, even in large quantities, affects only some parts of the brain
(D) There is a minimum limit of alcohol consumption so that it can cause damage to health
(E) Moderate alcohol consumption affects a good part of the brain
Comentários: A alternativa A está incorreta. De acordo com o texto, não é correto afirmar que o
consumo de pequenas quantidades de álcool é seguro e benéfico para o cérebro, mas sim, que
afeta o cérebro. Isso pode ser confirmado com o trecho “There is no safe amount of alcohol
consumption for the brain, with even ‘moderate’ drinking adversely affecting nearly every part of
it…”.
A alternativa B está incorreta. De acordo com o texto, não é correto afirmar que o volume do
cérebro aumenta com o aumento do consumo de álcool, mas sim, que quanto mais álcool, menor
será o volume do cérebro. Isso pode ser confirmado com o trecho “…suggests that the more
alcohol consumed, the lower the brain volume”.
A alternativa C está incorreta. De acordo com o texto, não é correto afirmar que o consumo de
álcool, mesmo em grandes quantidades, afeta apenas algumas partes do cérebro, mas sim, que
qualquer consumo afeta quase todo o cérebro. Isso pode ser confirmado com o trecho “Pretty
much the whole brain seems to be affected – not just specific areas, as previously thought”.
A alternativa D está incorreta. De acordo com o texto, não é correto afirmar que existe um limite
mínimo de consumo de álcool para que possa causar danos à saúde, mas sim, que qualquer
volume afeta a saúde. Isso pode ser confirmado com o trecho “There’s no threshold drinking for
harm – any alcohol is worse”.
A alternativa E está correta. De acordo com o texto, é correto afirmar que o consumo moderado
de álcool afeta boa parte do cérebro, assim como esta opção indica. Isso pode ser confirmado com
o trecho “There is no safe amount of alcohol consumption for the brain, with even ‘moderate’
drinking adversely affecting nearly every part of it…”.
GABARITO: E

QUESTÃO 02 (ESCOLA NAVAL/INÉDITA) – What’s the meaning of the word “threshold” in


paragraph 3?
(A) Conclusion
(B) Brink
(C) End
(D) Point
(E) Disadvantage

AULA 07 – LINKING WORDS (CONECTIVES) AND QUESTION WORDS (WH QUESTIONS) 101
TEACHER ANDREA BELO

Comentários: A alternativa A está incorreta. A palavra “thresold” não significa conclusão, mas sim,
limite/limiar, assim como a palavra “brink”, que significa limite.
A alternativa B está correta. A palavra “thresold” significa limite/limiar, assim como a palavra
“brink”, que significa limite.
A alternativa C está incorreta. A palavra “thresold” não significa fim, mas sim, limite/limiar, assim
como a palavra “brink”, que significa limite.
A alternativa D está incorreta. A palavra “thresold” não significa ponto, mas sim, limite/limiar,
assim como a palavra “brink”, que significa limite.
A alternativa E está incorreta. A palavra “thresold” não significa desvantagem, mas sim,
limite/limiar, assim como a palavra “brink”, que significa limite.
GABARITO: B

QUESTÃO 03 (ESCOLA NAVAL/INÉDITA) – In paragraph 1, the word “it” refers to


(A) Moderate drinking
(B) Alcohol consumption
(C) Study
(D) Brain
(E) UK
Comentários: A alternativa A está incorreta. A palavra “it” não se refere a beber moderadamente,
mas sim, ao cérebro, ou seja, “Não há uma quantidade segura de consumo de álcool para o
cérebro, mesmo com o consumo ‘moderado’ afetando adversamente quase todas as partes
dele...”.
A alternativa B está incorreta. A palavra “it” não se refere ao consumo de álcool, mas sim, ao
cérebro, ou seja, “Não há uma quantidade segura de consumo de álcool para o cérebro, mesmo
com o consumo ‘moderado’ afetando adversamente quase todas as partes dele...”.
A alternativa C está incorreta. A palavra “it” não se refere ao estudo, mas sim, ao cérebro, ou seja,
“Não há uma quantidade segura de consumo de álcool para o cérebro, mesmo com o consumo
‘moderado’ afetando adversamente quase todas as partes dele...”.
A alternativa D está correta. A palavra “it” se refere ao cérebro, ou seja, “Não há uma quantidade
segura de consumo de álcool para o cérebro, mesmo com o consumo ‘moderado’ afetando
adversamente quase todas as partes dele...”, assim como esta opção indica.
A alternativa E está incorreta. A palavra “it” não se refere ao Reino Unido, mas sim, ao cérebro, ou
seja, “Não há uma quantidade segura de consumo de álcool para o cérebro, mesmo com o
consumo ‘moderado’ afetando adversamente quase todas as partes dele...”.
GABARITO: D

AULA 07 – LINKING WORDS (CONECTIVES) AND QUESTION WORDS (WH QUESTIONS) 102
TEACHER ANDREA BELO

QUESTÃO 04 (ESCOLA NAVAL/INÉDITA) – According to the text, which option completes the
sentence below correctly?
Alcohol consumption __________ to affect only some parts of the brain
(A) Was thought
(B) Is thought
(C) Has been thought
(D) Will be thought
(E) Is being thought
Comentários: A alternativa A está correta. O tempo verbal correto do verbo to be, nesse caso, é o
simple past; ou seja, “Acreditava-se que o consumo de álcool afetava apenas uma parte do
cérebro”. Isso pode ser confirmado com o trecho “There’s no threshold drinking for harm – any
alcohol is worse. Pretty much the whole brain seems to be affected – not just specific areas, as
previously thought”.
A alternativa B está incorreta. O tempo verbal correto do verbo to be, nesse caso, não é o simple
present, mas sim, o simple past; ou seja, “Acreditava-se que o consumo de álcool afetava apenas
uma parte do cérebro”. Isso pode ser confirmado com o trecho “There’s no threshold drinking for
harm – any alcohol is worse. Pretty much the whole brain seems to be affected – not just specific
areas, as previously thought”.
A alternativa C está incorreta. O tempo verbal correto do verbo to be, nesse caso, não é o presente
perfect, mas sim, o simple past; ou seja, “Acreditava-se que o consumo de álcool afetava apenas
uma parte do cérebro”. Isso pode ser confirmado com o trecho “There’s no threshold drinking for
harm – any alcohol is worse. Pretty much the whole brain seems to be affected – not just specific
areas, as previously thought”.
A alternativa D está incorreta. O tempo verbal correto do verbo to be, nesse caso, não é o simple
future, mas sim, o simple past; ou seja, “Acreditava-se que o consumo de álcool afetava apenas
uma parte do cérebro”. Isso pode ser confirmado com o trecho “There’s no threshold drinking for
harm – any alcohol is worse. Pretty much the whole brain seems to be affected – not just specific
areas, as previously thought”.
A alternativa E está incorreta. O tempo verbal correto do verbo to be, nesse caso, não é o presente
continuous, mas sim, o simple past; ou seja, “Acreditava-se que o consumo de álcool afetava
apenas uma parte do cérebro”. Isso pode ser confirmado com o trecho “There’s no threshold
drinking for harm – any alcohol is worse. Pretty much the whole brain seems to be affected – not
just specific areas, as previously thought”.
GABARITO: A

AULA 07 – LINKING WORDS (CONECTIVES) AND QUESTION WORDS (WH QUESTIONS) 103
TEACHER ANDREA BELO

QUESTÃO 05 (ESCOLA NAVAL/INÉDITA) – According to the text, which option is correct?


(A) Smoking is what most negatively affects the brain
(B) Consuming more alcohol means increasing brain density
(C) The study presented in the text was based on data analysis
(D) An individual's underlying conditions do not interfere with the health impact of alcohol
(E) There are certain types of alcoholic beverages that affect the brain
Comentários: A alternativa A está incorreta. De acordo com o texto, não é correto afirmar que
fumar é o que mais afeta negativamente o cérebro, mas sim, que beber afeta mais do que fumar.
Isso pode ser confirmado com o trecho “This might seem like a small figure, but it is a larger
contribution than any other modifiable risk factors. For example, it is four times the contribution
of smoking or BMI…”.
A alternativa B está incorreta. De acordo com o texto, não é correto afirmar que consumir mais
álcool significa aumentar a densidade do cérebro, mas sim, que significa diminuir essa densidade.
Isso pode ser confirmado com o trecho “Higher volume of alcohol consumption per week was
associated with lower grey matter density”.
A alternativa C está correta. De acordo com o texto, é correto afirmar que o estudo apresentado
no texto se baseou na análise dos dados, assim como esta opção indica. Isso pode ser confirmado
com o trecho “Using the UK Biobank, a substantial database designed to help researchers decode
the genetic and environmental factors that lead some people to develop diseases while others do
not, researchers in this study analysed data…”.
A alternativa D está incorreta. De acordo com o texto, não é correto afirmar que as condições
subjacentes de um indivíduo não interferem no impacto do álcool na saúde, mas sim, que
interferem. Isso pode ser confirmado com o trecho “...an individual’s underlying conditions such
as high blood pressure and high BMI made the negative association between alcohol and brain
health stronger…”.
A alternativa E está incorreta. De acordo com o texto, não é correto afirmar que existem certos
tipos de bebidas alcoólicas que afetam o cérebro, mas sim, que o tipo de bebida não altera o
impacto causado. Isso pode ser confirmado com o trecho “...the study found no evidence to
suggest alcoholic beverage type conferred differences in risks to the brain”.
GABARITO: C

QUESTÃO 06 (ESCOLA NAVAL/INÉDITA) – What’s the meaning of the word “previous” in


paragraph 8?
(A) Prior
(B) First
(C) Current
(D) Future
(E) Timely

AULA 07 – LINKING WORDS (CONECTIVES) AND QUESTION WORDS (WH QUESTIONS) 104
TEACHER ANDREA BELO

Comentários: A alternativa A está correta. A palavra “previous” significa anterior e pode ser
comparada com a palavra “prior”, que significa anterior.
A alternativa B está incorreta. A palavra “previous” significa anterior e não pode ser comparada
com a palavra “first”, que significa primeira.
A alternativa C está incorreta. A palavra “previous” significa anterior e não pode ser comparada
com a palavra “current”, que significa atual.
A alternativa D está incorreta. A palavra “previous” significa anterior e não pode ser comparada
com a palavra “future”, que significa futura.
A alternativa E está incorreta. A palavra “previous” significa anterior e não pode ser comparada
com a palavra “timely”, que significa oportuno.
GABARITO: A

QUESTÃO 07 (ESCOLA NAVAL/INÉDITA) – Which is the correct option to complete the paragraph
below?
“___ Covid-19 crisis has been difficult on people across the globe, including India. In the past few
days there has been intense discussion ___ the decision of our government and Indian vaccine
manufacturers, including SII, ___ export vaccines”.
(Adapted from https://www.theguardian.com/world/2021/may/19/poorer-countries-face-long-delays-receiving-covid-vaccines)

(A) X / on / to
(B) The / in / to
(C) The / on / at
(D) X / on / at
(E) The / on / to
Comentários: A primeira lacuna deve ser preenchida com “the” pois a frase se refere a uma crise
específica, ou seja, “A crise da Covid-19 tem sido difícil para as pessoas em todo o mundo,
incluindo a Índia”.
A segunda lacuna deve ser preenchida com “on” pois a frase a uma discussão acerca de um
assunto, ou seja, “Nos últimos dias, tem havido intensa discussão sobre a decisão do nosso
governo...”.
A terceira lacuna deve ser preenchida com “to” pois a frase se refere a decisão de exportar vacinas,
ou seja, “Nos últimos dias, tem havido intensa discussão sobre a decisão do nosso governo e dos
fabricantes indianos de vacinas, incluindo SII, para exportar vacinas”.
GABARITO: E

AULA 07 – LINKING WORDS (CONECTIVES) AND QUESTION WORDS (WH QUESTIONS) 105
TEACHER ANDREA BELO

QUESTÃO 08 (ESCOLA NAVAL/INÉDITA) – Which option completes the dialogue below correctly?
Ed: _____ at the party yesterday?
Jack: Yes, didn't you see me?
(A) Was you
(B) Did you go
(C) Were you
(D) Do you go
(E) You were
Comentários: A lacuna em questão deve ser preenchida com “were you”, considerando que segue
o pronome necessário, a situação de interrogação e o tempo verbal (passado), considerando o
indicador “yesterday” (ontem).
GABARITO: C

QUESTÃO 09 (ESCOLA NAVAL/INÉDITA) – Which is the correct option to complete the paragraph
below?
“A few weeks ago, I __________ my first shot of a vaccine against Covid-19. As the newly
vaccinated exited the clinic, there was a mix of relief and elation on people’s faces. We exchanged
little smiles of solidarity. If we could have burst into spontaneous applause, I __________ sure we
would have done”.
(Adapted from https://www.theguardian.com/commentisfree/2021/may/17/animals-overlooked-allies-fight-against-covid-vaccines)

(A) Received / am
(B) Received / was
(C) Receive / am
(D) Receive / was
(E) Received / will be
Comentários: A primeira lacuna deve ser preenchida com o verbo to receive no simple past,
seguindo o tempo da frase, tal qual indica algo que acontece há algumas semanas, ou seja,
“Algumas semanas atrás, recebi minha primeira injeção de vacina contra Covid-19”.
A segunda lacuna deve ser preenchida com o verbo to be no simple present pois, apesar de se
tratar de algo que poderia ter acontecido no passado, o autor, nesta parte, indica um sentimento
atual, ou seja, “Se pudéssemos explodir em aplausos espontâneos, tenho certeza de que teríamos
feito”.
GABARITO: A

AULA 07 – LINKING WORDS (CONECTIVES) AND QUESTION WORDS (WH QUESTIONS) 106
TEACHER ANDREA BELO

QUESTÃO 10 (ESCOLA NAVAL/INÉDITA) – Which word best completes the question below?
How _______ hours does it take to walk around the entire city?
(A) Much
(B) Many
(C) Often
(D) Very
(E) Lot of
Comentários: A alternativa A está incorreta. A palavra que melhor completa a frase não é “much”
(quanto/muito), mas sim, “many” (quanto/muito), por se encaixar com substantivos contáveis,
que é o caso de “horas”. Isso pode ser confirmado com a frase “How many hours does it take to
walk around the entire city?” (Quantas horas leva para passear pela cidade inteira?).
A alternativa B está correta. A palavra que melhor completa a frase é “many” (quanto/muito), por
se encaixar com substantivos contáveis, que é o caso de “horas”, assim como esta opção indica.
Isso pode ser confirmado com a frase “How many hours does it take to walk around the entire
city?” (Quantas horas leva para passear pela cidade inteira?).
A alternativa C está incorreta. A palavra que melhor completa a frase não é “often”
(frequentemente), mas sim, “many” (quanto/muito), por se encaixar com substantivos contáveis,
que é o caso de “horas”. Isso pode ser confirmado com a frase “How many hours does it take to
walk around the entire city?” (Quantas horas leva para passear pela cidade inteira?).
A alternativa D está incorreta. A palavra que melhor completa a frase não é “very” (muito -
intensidade), mas sim, “many” (quanto/muito), por se encaixar com substantivos contáveis, que
é o caso de “horas”. Isso pode ser confirmado com a frase “How many hours does it take to walk
around the entire city?” (Quantas horas leva para passear pela cidade inteira?).
A alternativa E está incorreta. A palavra que melhor completa a frase não é “lot of” (muitos), mas
sim, “many” (quanto/muito), por se encaixar com substantivos contáveis, que é o caso de “horas”.
Isso pode ser confirmado com a frase “How many hours does it take to walk around the entire
city?” (Quantas horas leva para passear pela cidade inteira?).
GABARITO: B

QUESTÕES EsPCEx
Leia o texto a seguir e responda às questões 01, 02 e 03
Can New Zealand's tourism industry make a sustainable return?
New Zealand is hopeful that a recently opened travel bubble with Australia will rekindle its
pandemic-battered tourism industry. However, many are also seeing an opportunity to rethink
how to make the sector more climate friendly.
New Zealand's Queenstown - __________ (1) popular tourist spot - is throbbing with activity.
"To see and hear all the boats and the screaming and the complete joy… it puts a smile on all of
our faces," said Jolanda Cave, the general manager at Shotover Jet - one of the most established
adventure companies there.
It's a busy time __________ (2) the company, named after the river, where for more than half a
decade, boats have been whizzing, spinning and splashing to the delight of tourists.

AULA 07 – LINKING WORDS (CONECTIVES) AND QUESTION WORDS (WH QUESTIONS) 107
TEACHER ANDREA BELO

But even so, the numbers it is seeing are a fraction of what it was used to before the pandemic
closed the country's borders. It used to operate eight boat rides an hour. Sometimes, that was
down to just one.
"It's been real eye opener for us to go from 1,200 (visitors) a day to 200 a day," Ms Cave said.
Like many tourism operators across New Zealand Ms Cave is excited about the recently opened
travel bubble with Australia, its biggest market.
"It's given people hope. Australians represent a huge part of our business. (The bubble) will mean
that we will see some growth. I think a lot of Queenstown will," she says.
In 2019, international tourism was worth $12.6bn (£9bn) in total, __________ (3) Australians
contributing $1.94bn.
Between 1.18 and 1.5 million Australians came to New Zealand annually, accounting for 40% of
the country's overseas visitors.
Those numbers dropped to zero when New Zealand closed its borders.
(Adapted from https://www.bbc.com/news/business-56967636)

QUESTÃO 01 (EsPCEx/INÉDITA) – Choose the alternative containing the correct words to


respectively complete gaps (1), (2) and (3)
A) A, at, with
B) An, for, with
C) A, at, in
D) An, for, in
E) A, for, with
Comentários: A primeira lacuna deve ser preenchida com “a”, pois a frase se refere à um ponto
turístico popular.
A segunda lacuna deve ser preenchida com “for”, pois a frase se refere a algo que é para a
companhia, ou seja, “É um momento agitado para a empresa”.
A terceira lacuna deve ser preenchida com “with”, pois a frase se refere à contribuição dos
australianos, ou seja, “Em 2019, o turismo internacional valia US $ 12,6 bilhões (£ 9 bilhões) no
total, com os australianos contribuindo com US $ 1,94 bilhão”.
GABARITO: E

QUESTÃO 02 (EsPCEx/INÉDITA) – According to the text, is correct to state that


A) Despite being agitated, the Shotover jet routine was much more intense before the pandemic
B) New Zealand recently closed its borders with Australia
C) The opening of borders with Australia still represents a small part of the return to New
Zealand's tourist routine
D) Australians account for more than half of New Zealand tourists
E) New Zealand did not close its borders during the pandemic
Comentários: A alternativa A está correta. De acordo com o texto, é correto afirmar que apesar
de agitada, a rotina do jato Shotover era muito mais intensa antes da pandemia, assim como esta

AULA 07 – LINKING WORDS (CONECTIVES) AND QUESTION WORDS (WH QUESTIONS) 108
TEACHER ANDREA BELO

opção indica. Isso pode ser confirmado com o trecho “It's a busy time ___(2) the company ... But
even so, the numbers it is seeing are a fraction of what it was used to before the pandemic closed
the country's borders. It used to operate eight boat rides an hour. Sometimes, that was down to
just one”.
A alternativa B está incorreta. De acordo com o texto, não é correto afirmar que a Nova Zelândia
fechou recentemente suas fronteiras com a Austrália, mas sim, que abriu recentemente suas
fronteiras com a Austrália. Isso pode ser confirmado com o trecho “Like many tourism operators
across New Zealand Ms Cave is excited about the recently opened travel bubble with Australia…”.
A alternativa C está incorreta. De acordo com o texto, não é correto afirmar que a abertura das
fronteiras com a Austrália ainda representa uma pequena parte do retorno à rotina turística da
Nova Zelândia, mas sim, que representa a abertura de fronteiras com a maior parte do turismo do
país. Isso pode ser confirmado com o trecho “It's given people hope. Australians represent a huge
part of our business…”.
A alternativa D está incorreta. De acordo com o texto, não é correto afirmar que os australianos
representam mais da metade dos turistas da Nova Zelândia, mas sim, que representam 40% dos
turistas de lá. Isso pode ser confirmado com o trecho “Between 1.18 and 1.5 million Australians
came to New Zealand annually, accounting for 40% of the country's overseas visitors”.
A alternativa E está incorreta. De acordo com o texto, não é correto afirmar que a Nova Zelândia
não fechou suas fronteiras durante a pandemia, mas sim, que fechou. Isso pode ser confirmado
com o trecho “Those numbers dropped to zero when New Zealand closed its borders”.
GABARITO: A

QUESTÃO 03 (EsPCEx/INÉDITA) – In the sentence “To see and hear all the boats and the
screaming and the complete joy… it puts a smile on all of our faces” (paragraph 2), the word joy
means
A) Melancholy
B) Happiness
C) Sorrow
D) Courage
E) Calm
Comentários: A alternativa A está incorreta. A palavra “joy” significa alegria e não pode ser
comparada com a palavra “melancholy”, que significa melancolia.
A alternativa B está correta. A palavra “joy” significa alegria e pode ser comparada com a palavra
“happiness”, que significa alegria.
A alternativa C está incorreta. A palavra “joy” significa alegria e não pode ser comparada com a
palavra “sorrow”, que significa tristeza.
A alternativa D está incorreta. A palavra “joy” significa alegria e não pode ser comparada com a
palavra “courage”, que significa coragem.
A alternativa E está incorreta. A palavra “joy” significa alegria e não pode ser comparada com a
palavra “calm”, que significa calma.
GABARITO: B

AULA 07 – LINKING WORDS (CONECTIVES) AND QUESTION WORDS (WH QUESTIONS) 109
TEACHER ANDREA BELO

Leia o texto a seguir e responda às questões 04, 05 e 06


Rich countries close their eyes to the global Covid surge at their own peril
The pandemic’s death toll is now being felt most gravely in developing nations. This virus is
not done yet
Is there one pandemic, or two? That ___ (1) a question being asked a year ago, when wealthy
countries accounting for only 15% of the global population had 80% of the Covid deaths. Could it
be that the rich world was more vulnerable, somehow, because its populations ___ (2) older, or
more individualistic, or had forgotten to be scared of infectious disease?
Even then, some were warning that the worst was yet to come, once the disease took hold in
poorer countries. World Bank analysts Philip Schellekens and Diego Sourrouille, for example,
predicted a “massive shift” in disease burden to the developing world. Just in terms of
demography, they said, you’d expect those countries to account for around 70% of deaths.
As things stand they account for a little over half of it, which is probably an underestimate due to
variations in data quality – and the pandemic is far from over.
Last week saw more than 5.8 million new cases of Covid globally, the highest number yet. More
than 3 million people have now died from Covid, according to the World Health Organization
(WHO), which also reports that infections and hospitalisations in those aged at 25 to 59 ___ (3)
increasing an alarming rate. “It took nine months to reach 1 million deaths, four months to reach
2 million, and three months to reach 3 million,” WHO director general, Tedros Adhanom
Ghebreyesus, said last week.
(Adapted from https://www.theguardian.com/commentisfree/2021/apr/27/rich-countries-covid-pandemic-death-developing-nations)

QUESTÃO 04 (EsPCEx/INÉDITA) – In the sentence “Could it be that the rich world was more
vulnerable, somehow, because its populations were older, or more individualistic, or had
forgotten to be scared of infectious disease?” (paragraph 1), the word scared means
A) Bold
B) Cautious
C) Brave
D) Afraid
E) Partial
Comentários: A alternativa A está incorreta. A palavra “scared” significa assustado/com medo e
não pode ser comparada com a palavra “bold”, que significa corajoso.
A alternativa B está incorreta. A palavra “scared” significa assustado/com medo e não pode ser
comparada com a palavra “cautious”, que significa cauteloso.
A alternativa C está incorreta. A palavra “scared” significa assustado/com medo e não pode ser
comparada com a palavra “brave”, que significa corajoso.
A alternativa D está correta. A palavra “scared” significa assustado/com medo e pode ser
comparada com a palavra “afraid”, que significa com medo.

AULA 07 – LINKING WORDS (CONECTIVES) AND QUESTION WORDS (WH QUESTIONS) 110
TEACHER ANDREA BELO

A alternativa E está incorreta. A palavra “scared” significa assustado/com medo e não pode ser
comparada com a palavra “partial”, que significa parcial.
GABARITO: D

QUESTÃO 05 (EsPCEx/INÉDITA) – Choose the alternative containing the correct verb forms to
complete the gaps (1), (2) and (3) in paragraphs 1, 1 and 3 respectively
A) Is, was, are
B) Was, were, are
C) Is, were, are
D) Was, was, are
E) Was, were, were
Comentários: A primeira lacuna deve ser preenchida com “was”, pois a frase se refere à uma
pergunta feita no ano passado, ou seja, “Existe uma pandemia ou duas? Essa foi uma pergunta
que foi feita há um ano...”.
A segunda lacuna deve ser preenchida com “were”, pois a frase se refere à algo que aconteceu no
passado, ou seja, “Será que o mundo rico era mais vulnerável, de alguma forma, porque suas
populações eram mais velhas...”.
A terceira lacuna deve ser preenchida com “are”, pois a frase se refere a algo que está acontecendo
agora, ou seja, “...que também relata que infecções e hospitalizações em pessoas com idade entre
25 e 59 anos estão aumentando a uma taxa alarmante”.
GABARITO: B

QUESTÃO 06 (EsPCEx/INÉDITA) – According to the text, choose the correct statement


The COVID-19 pandemic has always affected all countries homogeneously
A) Wealthier countries were affected first by the pandemic because of their greater vulnerability
B) At the beginning of the pandemic, some people believed that the worst would come with the
involvement of
C) COVID-19 in poor countries
D) After about a year, the end of the pandemic is near
E) Last week, there were drops in the rates of coronavirus infections
Comentários: A alternativa A está incorreta. De acordo com o texto, não é correto afirmar que a
pandemia COVID-19 sempre afetou todos os países de maneira homogênea, mas sim, que no
início, afetou mais os países ricos. Isso pode ser confirmado com o trecho “...a question being
asked a year ago, when wealthy countries accounting for only 15% of the global population had
80% of the Covid deaths”.
A alternativa B está incorreta. De acordo com o texto, não é correto afirmar que os países mais
ricos foram afetados primeiro pela pandemia devido à sua maior vulnerabilidade, pois o texto não

AULA 07 – LINKING WORDS (CONECTIVES) AND QUESTION WORDS (WH QUESTIONS) 111
TEACHER ANDREA BELO

afirma isso, apenas questiona. Isso pode ser confirmado com o trecho “Could it be that the rich
world was more vulnerable, somehow…”.
A alternativa C está correta. De acordo com o texto, é correto afirmar que no início da pandemia,
algumas pessoas acreditavam que o pior viria com o envolvimento da COVID-19 nos países pobres,
assim como esta opção indica. Isso pode ser confirmado com o trecho “Even then, some were
warning that the worst was yet to come, once the disease took hold in poorer countries”.
A alternativa D está incorreta. De acordo com o texto, não é correto afirmar que após cerca de um
ano, o fim da pandemia está próximo, mas sim, que ainda está longe de terminar. Isso pode ser
confirmado com o trecho “...the Pandemic is far from over”.
A alternativa E está incorreta. De acordo com o texto, não é correto afirmar que na semana
passada, houve queda nas taxas de infecções por coronavírus, mas sim, que foi o maior número
de novos casos registrado. Isso pode ser confirmado com o trecho “Last week saw more than 5.8
million new cases of Covid globally, the highest number yet”.
GABARITO: C

Leia o texto a seguir e responda às questões 07, 08 e 09


(Título omitido propositalmente)
The panel, set up by the World Health Organization, said the combined response of the WHO and
global governments was a "toxic cocktail".
The WHO should have declared a global emergency earlier than it did, its report said, adding that
without urgent change the world was vulnerable to another major disease outbreak.
More than 3.3 million people around the world have now died of Covid.
While the US and Europe are beginning to ease restrictions and resume some aspects of pre-
pandemic life, the virus is still devastating parts of Asia.
India in particular is seeing record-breaking numbers of new cases and deaths, with severe oxygen
shortages in hospitals across the country.
Countries neighbouring India, such as Nepal, are also seeing surges of the virus.
What did the report say?
Covid-19: Make it the Last Pandemic, was compiled by the Independent Panel for Pandemic
Preparedness and Response.
Its aim was to find answers as to how the virus had killed more than 3.3 million people and infected
more than 159 million.
"The situation we find ourselves in today could have been prevented," co-chair Ellen Johnson
Sirleaf, a former president of Liberia, told reporters.
"It is due to a myriad of failures, gaps and delays in preparedness and response."
The panel argued that the WHO's Emergency Committee should have declared the outbreak in
China an international emergency a week earlier than it did.

AULA 07 – LINKING WORDS (CONECTIVES) AND QUESTION WORDS (WH QUESTIONS) 112
TEACHER ANDREA BELO

It should have done so at its first meeting on 22 January last year, the report said, instead of waiting
until 30 January.
The month following the WHO's declaration was "lost" as countries failed to take appropriate
measures to halt the spread of the virus.
The WHO was then hindered by its own regulations that travel restrictions should be a last resort,
the panel said, adding that Europe and the US wasted the entire month of February and acted
only when their hospitals began to fill up.
(Adapted from https://www.bbc.com/news/world-57085505)

QUESTÃO 07 (EsPCEx/INÉDITA) – Choose the most appropriate title for the text
A) Covid-19 pandemic was preventable, report says
B) The return to the pre-pandemic routine in the USA and Europe
C) More recent data shows an increase in the number of deaths due to COVID-19
D) Why is Covid-19 more intense in poorer countries?
E) Why the pandemic has intensified socioeconomic inequalities around the world
Comentários: A alternativa A está correta. O título mais apropriado para o texto é “A pandemia de
Covid-19 era evitável, diz o relatório”, assim como esta opção indica. Isso pode ser confirmado
com o trecho “The situation we find ourselves in today could have been prevented”.
A alternativa B está incorreta. O título mais apropriado para o texto não é “O retorno à rotina pré-
pandêmica nos EUA e na Europa”, mas sim, “A pandemia de Covid-19 era evitável, diz o relatório”.
Isso pode ser confirmado com o trecho “The situation we find ourselves in today could have been
prevented”.
A alternativa C está incorreta. O título mais apropriado para o texto não é “Dados mais recentes
mostram aumento no número de óbitos por COVID-19”, mas sim, “A pandemia de Covid-19 era
evitável, diz o relatório”. Isso pode ser confirmado com o trecho “The situation we find ourselves
in today could have been prevented”.
A alternativa D está incorreta. O título mais apropriado para o texto não é “Por que a Covid-19 é
mais intensa nos países mais pobres?”, mas sim, “A pandemia de Covid-19 era evitável, diz o
relatório”. Isso pode ser confirmado com o trecho “The situation we find ourselves in today could
have been prevented”.
A alternativa E está incorreta. O título mais apropriado para o texto não é “Por que a pandemia
intensificou as desigualdades socioeconômicas em todo o mundo”, mas sim, “A pandemia de
Covid-19 era evitável, diz o relatório”. Isso pode ser confirmado com o trecho “The situation we
find ourselves in today could have been prevented”.
GABARITO: A

AULA 07 – LINKING WORDS (CONECTIVES) AND QUESTION WORDS (WH QUESTIONS) 113
TEACHER ANDREA BELO

QUESTÃO 08 (EsPCEx/INÉDITA) – What kind of text is this?


A) Letter
B) Essay
C) Opinion article
D) Textbook
E) News report
Comentários: A alternativa A está incorreta. O texto dado não é uma carta, mas sim, um relatório
de notícia (news report).
A alternativa B está incorreta. O texto dado não é uma redação, mas sim, um relatório de notícia
(news report).
A alternativa C está incorreta. O texto dado não é um artigo de opinião, mas sim, um relatório de
notícia (news report).
A alternativa D está incorreta. O texto dado não é um livro didático, mas sim, um relatório de
notícia (news report).
A alternativa E está correta. O texto dado é um relatório de notícia, assim como esta opção indica.
GABARITO: E

QUESTÃO 09 (EsPCEx/INÉDITA) – In the sentence “The WHO was then hindered by its own
regulations that travel restrictions should be a last resort…” (paragraph 14), the word hindered
means
A) Encouraged
B) Burdened
C) Approved
D) Neutralized
E) Supported
Comentários: A alternativa A está incorreta. A palavra “hindered” significa impedido e não pode
ser comparada com a palavra “encouraged”, que significa encorajado.
A alternativa B está correta. A palavra “hindered” significa impedido e pode ser comparada com a
palavra “burdened”, que significa sobrecarregado/impedido.
A alternativa C está incorreta. A palavra “hindered” significa impedido e não pode ser comparada
com a palavra “approved”, que significa aprovado.
A alternativa D está incorreta. A palavra “hindered” significa impedido e não pode ser comparada
com a palavra “neutralized”, que significa neutralizado.
A alternativa E está incorreta. A palavra “hindered” significa impedido e não pode ser comparada
com a palavra “supported”, que significa apoiado.
GABARITO: B

AULA 07 – LINKING WORDS (CONECTIVES) AND QUESTION WORDS (WH QUESTIONS) 114
TEACHER ANDREA BELO

Leia o texto a seguir e responda à questão 10


Teens Aged 12 to 15 Can Now Get Pfizer-BioNTech COVID-19 Vaccine in the U.S.
___ (1) May 10, the U.S. Food and Drug Administration (FDA) extended authorization of the Pfizer-
BioNTech COVID-19 vaccine to 12- to 15-year-olds. It’s the first COVID-19 shot authorized for this
younger population.
“I cannot tell you how many people have been anxiously awaiting this day to get their kids
vaccinated,” says Dr. Flor Munoz, a pediatric infectious disease specialist ___ (2) Texas Children’s
Hospital and Baylor College of Medicine, and a member of the infectious disease committee of
the American Academy of Pediatrics.
The green light was based on review of data the two companies released in March showing that
two doses of their vaccine provided similar protection from COVID-19 disease among this age
group as it did for adults. Among the more than 2,200 teens in the study group the FDA reviewed,
16 developed COVID-19—none of them were in the vaccinated group.
The companies’ studies found that levels of virus-fighting antibodies were on average higher
among the 12- to 15-year-olds that got the vaccine than among vaccinated 16- to 25-year-olds in
previous studies.
“This is part of the totality of getting our country protected against COVID-19, which is just waiting
around ___ (3) corner to come have another wave if we don’t get a sufficient degree of
vaccination,” said Dr. Peter Marks, director of the Center for Biologics Evaluation and Research at
the FDA, during a briefing discussing the decision.
The authorization was based on a so-called “bridging” study, in which researchers used data from
the previous study in adults to set thresholds for safety and efficacy. Building off of the success of
the adult studies enabled the scientists to enroll some 2,000 12- to 15-year-olds, rather than the
tens of thousands of adults that the earlier studies required.
(Adapted from https://time.com/6047384/teens-pfizer-covid-vaccine/)

QUESTÃO 10 (EsPCEx/INÉDITA) – Choose the alternative containing the correct words to


complete the gaps (1), (2) and (3) in paragraphs 1, 2 and 4 respectively
A) In, at, X
B) On, in, the
C) On, in, X
D) On, at, the
E) In, at, the

AULA 07 – LINKING WORDS (CONECTIVES) AND QUESTION WORDS (WH QUESTIONS) 115
TEACHER ANDREA BELO

Comentários: A primeira lacuna deve ser preenchida com “on”, pois se refere ao dia em que
ocorreu a autorização, ou seja, “Em 10 de maio, a Food and Drug Administration (FDA) dos EUA
estendeu a autorização da vacina Pfizer-BioNTech COVID-19 para crianças de 12 a 15 anos de
idade”.
A segunda lacuna deve ser preenchida com “at”, pois a frase se refere ao local onde Flor Munoz
exerce sua função, ou seja, “...diz o Dr. Flor Munoz, especialista em doenças infecciosas pediátricas
do Texas Children’s Hospital e Baylor College of Medicine...”.
A terceira lacuna deve ser preenchida com “the”, pois se refere à expressão “around the corner”,
ou seja, “Isso faz parte da totalidade de proteger nosso país contra a COVID-19, que está
esperando virando a esquina para ter outra onda...”.
GABARITO: D

AULA 07 – LINKING WORDS (CONECTIVES) AND QUESTION WORDS (WH QUESTIONS) 116
TEACHER ANDREA BELO

CONSIDERAÇÕES FINAIS
Outra aula alcançada com sucesso – outro passo até a sua aprovação!
E, dia após dia, os tópicos aprendidos aumentam, seu conhecimento fica mais amplo, o
vocabulário que você conhece se estende e a tendência é melhorar e ser capaz de alcançar a
aprovação de fato.
Nota-se o avanço em seus estudos e, provavelmente, uma maior tranquilidade para
enfrentar os exercícios que surgem. E você vai se acostumando a equilibrar seus estudos de forma
sistematizada, estudando cada vez mais e com mais dedicação.
Outro detalhe importante para seu sucesso nos estudos é continuar fazendo listas de
palavras e verbos, participar das aulas complementares, fazer simulados e pedir ajuda quando
precisar.
Isso te ajudará nas questões futuras e torna você, como eu disse antes, um candidato mais bem
preparado e confiante para realizar uma excelente prova de vestibular.
É importante lembrar também do nosso Fórum de dúvidas, exclusivo do Estratégia
Militares. Será minha forma de responder você, esclarecer o que mais você precise saber para
que os conteúdos fiquem ainda mais claros em seus estudos, certo?
E, caso queira, acesse minhas redes sociais (Teacher Andrea Belo) para aprender mais
palavras e contar com dicas importantes, que colaboram diretamente com seus estudos dia após
dia.

AULA 07 – LINKING WORDS (CONECTIVES) AND QUESTION WORDS (WH QUESTIONS) 117
TEACHER ANDREA BELO

REFERÊNCIAS BIBLIOGRÁFICAS
BARRETO, Tania Pedroza; GARRIDO, Maria Line; SILVA, João Antenor de C., Inglês Instrumental.
Leitura e compreensão de textos. Salvador, Ba UFBA, 1995, p. 64.
BROWN. H. Douglas. Principles of Language Learning and Teaching. Prentice Hall International,
1988.
COMPEDELLI, Samira Yousseff. Português, Literatura, Produção de texto & Gramática – São Paulo:
Ed. Saraiva, 2002.
CORREIA, Clese Mary P. Reading Specific Purposes. Salvador/ Ba: UFBA, 1999.
COSTEIRA, Adriana Araújo de M. Reading Comprehension Skills. João Pessoa/PB: ETFP, 1998.
CRYSTAL David. Cambridge University Press 1997. The Cambridge Encyclopedia of Language.
Cambridge University Press 1997
FREEMAN. Diane Larsen. MURCIA. Marianne Celce. The Grammar Book, 1999.
DYE, Joan., FRANFORT, Nancy. Spectrum II, III A Communicative Course in English. USA, Prentice
Hall, 1994.
FAVERO, Maria de Lourdes Albuquerque (org.). Dicionário de educadores no Brasil: da colônia aos
dias atuais. Rio de Janeiro: UFRJ, MEC, INEP, 1999.
FRANKPORT, Nancy & Dye Hoab. Spectrum II, III Prentice Hall Regents Englewood Cliffs, New Jersy,
1994.
GADELHA, Isabel Maria B. Inglês Instrumental: Leitura, Conscientização e Prática. Teresina:
EDUFFI, 2000.
GUANDALINI, Eiter Otávio. Técnicas de Leitura em Inglês: ESP – English For Specific Purposes:
estágio 1. São Paulo: Texto novo, 2002.
GRELLET, Françoise. Developing Reading Skills. Cambridge University Press, 1995
HOLAENDER, Arnon & Sanders Sidney. A complete English Course. São Paulo. Ed. Moderna, 1995.
HUTCHINSON, Tom & WATERS, Alan. English for Specific Purposes. Cambridge: Cambridge
University Press, 1996
KRASHEN. Stephen D. Second Language Acquisition and Second Language Learning, Prentice-Hall
International, 1988.
LAENG, Mauro. Dicionário de pedagogia. Lisboa: Dom Quixote, 1973.
LEFFA, Vilson J. Metodologia do ensino de línguas. In: BOHN, H.; VANDRESEN, P. (org.). Tópicos de
linguística aplicada: o ensino de línguas estrangeiras. Florianópolis: Editora da UFSC, 1988. p. 211-
231.
LIBERATO, Wilson. Compact English Book Inglês Ensino Médio. São Paulo: FTD, Vol. Único, 1998
Mc ARTHUR. The Oxford Companion to the English Language. Oxford University Press 1992
Fromkin. Victoria. An Introduction to Language
MARQUES, Amadeu. Inglês Série Brasil. ed. Atica. São Paulo: 2004. Vol. Único.

AULA 07 – LINKING WORDS (CONECTIVES) AND QUESTION WORDS (WH QUESTIONS) 118
TEACHER ANDREA BELO

MURPHY, Raymond: Essencial Grammar in Use Oxford. New York Ed. Oxford University, 1997.
OLIVEIRA, Luciano Amaral. English For Tourism Students. Inglês para Estudantes de Turismo: São
Paulo, Rocca, 2001.
OLIVEIRA, Sara Rejane de F. Estratégias de leitura para Inglês Instrumental. Brasília: UNB, 1994.
QUINTANA, et alli. First Certificate. Master Class Oxford. New York, 2004: Ed. Oxford University.
PAULINO, Berenice F. et all. Leitura em textos em Inglês – Uma Abordagem Instrumental. Belo
Horizonte: Ed. Dos Autores, 1992.
PEREIRA, Edilberto Coelho. Inglês Instrumental. Teresina: ETFPI, 1998.
RODGES, Theodore. Jack C. Richards. Approaches and Methods in Language Teaching. Cambridge
University Press, 2001.
RODMAN Robert. Harcourt Brace 1993. English as a Global Language
STEWART, B., HAINES S. First Certificate, MasterClass. UK – Oxford 2004.
SILVA, João Antenor de C., GARRIDO, Maria Lina, BARRETO, Tânia Pedrosa. Inglês Instrumental:
Leitura e Compreensão de Textos. Salvador: Centro Editorial e Didático, UFBA. 1994
SOARES, Moacir Bretãs. Dicionário de legislação do ensino. 19.ed. Rio de Janeiro: FGV, 1981.
SOUZA, Adriana Srade F. Leitura em Língua Inglesa: Uma abordagem Instrumental. São Paulo:
Disal, 2005.
TUCK, Michael. Oxford Dictionary of Computing for Learners of English. Oxford: Oxford University
Press, 1996.
TOTIS, Verônica Pakrauskas. Língua Inglesa: leitura. São Paulo: Cortez, 1991.

Livros eletrônicos:
Dicionário Houaiss da Língua Portuguesa, Editora Objetiva, 2001.
MOURãO, Janaína Pereira. "Skimming x Scanning"; Brasil Escola. Disponível em
<https://brasilescola.uol.com.br/ingles/skimming-x-scanning.htm>. Acesso em 20 de março de
2019.
www.newsweek.com - Acesso em 18 de março de 2019.
http://www.galaor.com.br/tecnicas-de-leitura/ - Acesso em 19 de março de 2019.
Expressões Idiomáticas (continuação)" em Só Língua Inglesa. Virtuous Tecnologia da
Informação,2008-2019. Consultado em 03/04/2019 às 22:09. Disponível na Internet em
http://www.solinguainglesa.com.br/conteudo/Expressoes5.php

AULA 07 – LINKING WORDS (CONECTIVES) AND QUESTION WORDS (WH QUESTIONS) 119
TEACHER ANDREA BELO

TRADUÇÕES
February’s Gonna Be a Big Month for Mars On the 9th, the first of three spacecraft will arrive
at the Red Planet and inaugurate a new era of Martian exploration.

_______ FEBRUARY 9, the United Arab Emirates’ Hope spacecraft is expected to enter orbit
around Mars after a six-month, 300-million-mile journey from Earth. It will mark the beginning of
a historic month for the Red Planet, which will see three separate national missions enter orbit or
touch down on the surface. Two of the countries behind these missions, the UAE and China, will
be visiting Mars for the first time; they will become the fifth and sixth countries to pull off that
feat, respectively. The third mission, launched _______ NASA, is expected to become the United
States’ 15th mission to successfully orbit or land on Mars.
The UAE is the only country that will not attempt a soft landing during the February Mars invasion.
Instead, its Hope orbiter will study the Martian atmosphere from more than 12,000 miles above
the surface. Planetary scientists hope that the UAE’s robo-meteorologist will fill in gaps in our
understanding of the Martian climate and help validate environmental data captured by rovers
and landers on the ground. For the country’s first foray into deep-space exploration, the UAE
space agency worked with an international team of researchers at the University of Colorado,
Boulder, to help plan the mission and build the spacecraft.
“There’s really no point in exploring outer space without adding to knowledge, and we’ve never
run a science mission,” Sarah bint Yousef Al Amiri, the UAE minister of state for advanced sciences
and science lead for the Emirates Mars Mission, said during a press conference last week. “It
wasn’t an easy journey, but it was such an enjoyment to rethink how you develop a planetary
exploration mission.”

AULA 07 – LINKING WORDS (CONECTIVES) AND QUESTION WORDS (WH QUESTIONS) 120
TEACHER ANDREA BELO

The Hope spacecraft will be the first new orbiter around Mars since the European Space Agency’s
ExoMars spacecraft arrived in 2016, but it won’t be the newcomer for long. China’s Tianwen-1
mission—which is a lander, rover, and orbiter rolled up into one—is expected to arrive less than
a day later. China’s space agency has been quiet about its plans for visiting the Red Planet, but
the craft is expected to attempt a landing shortly after it achieves orbit.
Unlike NASA’s car-sized Mars rovers Curiosity and Opportunity, China’s Tianwen-1 rover is small
enough to stow away inside the stationary lander that will carry it to the surface. Once it has safely
touched down, the six-wheeled rover will detach itself from the lander and spend the next three
months exploring its landing site, Utopia Planitia, the planet’s largest impact crater. The rover and
lander will both relay data _______ the surface to the Tianwen-1 orbiter, which will send it back
to Earth. Although the Chinese National Space Administration hasn’t provided a lot of details
about the exact scientific goals of its mission, a paper about it published last year in Nature
Astronomy says the agency’s goal is to “perform a global and extensive survey of the entire
planet.”
On February 18, a little more than a week after this robotic delegation arrives, NASA’s
Perseverance rover is expected to touch down. This will involve a harrowing descent to the
surface, during which the rover must reduce its speed from more than 10,000 miles an hour to
just a few feet per second over the course of 15 minutes. The descent will end with some aerial
acrobatics, during which a rocket-powered sky crane will gently deposit the rover on the surface
while hovering a few dozen feet above the ground.
“Don’t let anybody tell you different—landing _______ Mars is hard to do,” John McNamee,
project manager for the Perseverance mission at NASA’s Jet Propulsion Laboratory, said in a
statement. “But the women and men on this team are the best in the world at what they do.
When our spacecraft hits the top of the Mars atmosphere at about three and a half miles per
second, we’ll be ready.”
Perseverance is essentially a nuclear-powered self-driving car, and its primary mission is to collect
samples that will be picked up by another spacecraft later this decade and returned to Earth. With
any luck, this red dust will contain evidence that Mars once hosted microbial life. But whether
scientists will recognize extraterrestrial life when they see it remains an open question. Aside from
hunting for aliens, Perseverance will also enable a first-of-its-kind technology demonstration
involving a small helicopter called Ingenuity. A few days after landing, Perseverance will jettison
the helicopter in a clearing where it will attempt several short flights. If it works, it will be the first
time an aircraft has flown on another planet.
The arrival of three national missions on Mars within two weeks of one another is a historic
moment in the history of space exploration. It underscores the rapid development of space
capabilities around the world and the true internationalization of planetary exploration. Mars was
once the exclusive stomping grounds of the United States and the Soviet Union, but it is now also
an accessible destination for the European Union, Japan, India, the UAE, and China. Getting to
Mars is still a major challenge—historically only 40 percent of Mars missions have been
successful—and there’s no guarantee that all three missions will succeed in their objectives. But
launching a trio of spacecraft to our closest planetary neighbor is a major achievement and bodes
well for the future of space exploration.
(Adapted from https://www.wired.com/story/februarys-gonna-be-a-big-month-for-mars/)

AULA 07 – LINKING WORDS (CONECTIVES) AND QUESTION WORDS (WH QUESTIONS) 121
TEACHER ANDREA BELO

Fevereiro vai ser um grande mês para Marte No dia 9, a primeira das três espaçonaves
chegará ao Planeta Vermelho e inaugurará uma nova era de exploração marciana.

_______ 9 de fevereiro, espera-se que a espaçonave Hope dos Emirados Árabes Unidos entre em
órbita ao redor de Marte após uma viagem de 300 milhões de milhas da Terra de seis meses. Isso
marcará o início de um mês histórico para o Planeta Vermelho, que verá três missões nacionais
separadas entrarem em órbita ou tocarem na superfície. Dois dos países por trás dessas missões,
os Emirados Árabes Unidos e a China, visitarão Marte pela primeira vez; eles se tornarão o quinto
e o sexto países a realizarem essa façanha, respectivamente. A terceira missão, lançada _______
da NASA, deverá se tornar a 15ª missão dos Estados Unidos a orbitar ou pousar em Marte com
sucesso.
Os Emirados Árabes Unidos são o único país que não fará um pouso suave durante a invasão de
Marte em fevereiro. Em vez disso, seu orbitador Hope estudará a atmosfera marciana a mais de
12.000 milhas acima da superfície. Cientistas planetários esperam que o robô meteorologista dos
Emirados Árabes Unidos preencha as lacunas em nossa compreensão do clima marciano e ajude
a validar os dados ambientais capturados por rovers e pousadores no solo. Para a primeira
incursão do país na exploração do espaço profundo, a agência espacial dos Emirados Árabes
Unidos trabalhou com uma equipe internacional de pesquisadores da Universidade do Colorado,
em Boulder, para ajudar a planejar a missão e construir a espaçonave.
“Não há realmente nenhum ponto em explorar o espaço sideral sem adicionar conhecimento, e
nunca dirigimos uma missão científica”, disse Sarah bint Yousef Al Amiri, a ministra de estado dos
Emirados Árabes Unidos para ciências avançadas e liderança científica da Missão dos Emirados
Marte, durante uma conferência de imprensa na semana passada. “Não foi uma jornada fácil, mas
foi um prazer repensar como você desenvolve uma missão de exploração planetária.”
A espaçonave Hope será a primeira nova órbita em torno de Marte desde que a espaçonave
ExoMars da Agência Espacial Europeia chegou em 2016, mas não será a nova nave por muito
tempo. A missão Tianwen-1 da China - que é um módulo de pouso, rover e orbital reunidos em

AULA 07 – LINKING WORDS (CONECTIVES) AND QUESTION WORDS (WH QUESTIONS) 122
TEACHER ANDREA BELO

um só - deve chegar menos de um dia depois. A agência espacial da China não disse nada sobre
seus planos de visitar o Planeta Vermelho, mas a nave deve tentar pousar logo após entrar em
órbita.
Ao contrário dos rovers de Marte Curiosity and Opportunity do tamanho de um carro da NASA, o
rover chinês Tianwen-1 é pequeno o suficiente para ser guardado dentro do módulo de pouso
estacionário que o levará para a superfície. Assim que pousar com segurança, o rover de seis rodas
se separará da sonda e passará os próximos três meses explorando seu local de pouso, Utopia
Planitia, a maior cratera de impacto do planeta. O rover e o módulo de pouso irão retransmitir
dados _______ da superfície para o orbitador Tianwen-1, que os enviará de volta à Terra. Embora
a Administração Espacial Nacional chinesa não tenha fornecido muitos detalhes sobre os objetivos
científicos exatos de sua missão, um artigo sobre o assunto publicado no ano passado na Nature
Astronomy afirma que o objetivo da agência é “realizar uma pesquisa global e extensa em todo o
planeta . ”
Em 18 de fevereiro, um pouco mais de uma semana após a chegada desta delegação robótica, o
rover Perseverance da NASA deve pousar. Isso envolverá uma descida angustiante à superfície,
durante a qual o rover deve reduzir sua velocidade de mais de 16 mil quilômetros por hora para
apenas alguns pés por segundo ao longo de 15 minutos. A descida terminará com algumas
acrobacias aéreas, durante as quais um guindaste do céu movido a foguete depositará
suavemente o rover na superfície enquanto paira algumas dezenas de metros acima do solo.
“Não deixe ninguém lhe dizer algo diferente - pousar _______ em Marte é difícil”, disse John
McNamee, gerente de projeto da missão Perseverance no Laboratório de Propulsão a Jato da
NASA, em um comunicado. “Mas as mulheres e os homens desta equipe são os melhores do
mundo no que fazem. Quando nossa espaçonave atingir o topo da atmosfera de Marte a cerca de
três milhas e meia por segundo, estaremos prontos. ”
O Perseverance é essencialmente um carro autônomo movido a energia nuclear, e sua missão
principal é coletar amostras que serão coletadas por outra espaçonave no final desta década e
devolvidas à Terra. Com alguma sorte, essa poeira vermelha conterá evidências de que Marte já
hospedou vida microbiana. Mas se os cientistas reconhecerão a vida extraterrestre quando a
virem, permanece uma questão em aberto. Além de caçar alienígenas, o Perseverance também
permitirá uma demonstração de tecnologia inédita envolvendo um pequeno helicóptero
chamado Ingenuity. Poucos dias após o pouso, o Perseverance lançará o helicóptero em uma
clareira onde tentará vários voos curtos. Se funcionar, será a primeira vez que uma aeronave
voará em outro planeta.
A chegada de três missões nacionais a Marte com duas semanas de diferença é um momento
histórico na história da exploração espacial. Ele ressalta o rápido desenvolvimento das
capacidades espaciais em todo o mundo e a verdadeira internacionalização da exploração
planetária ação. Marte já foi o reduto exclusivo dos Estados Unidos e da União Soviética, mas
agora também é um destino acessível para a União Europeia, Japão, Índia, Emirados Árabes
Unidos e China. Chegar a Marte ainda é um grande desafio - historicamente, apenas 40 por cento
das missões a Marte foram bem-sucedidas - e não há garantia de que todas as três missões terão
sucesso em seus objetivos. Mas lançar um trio de naves espaciais ao nosso vizinho planetário mais
próximo é uma grande conquista e um bom presságio para o futuro da exploração espacial.

AULA 07 – LINKING WORDS (CONECTIVES) AND QUESTION WORDS (WH QUESTIONS) 123
TEACHER ANDREA BELO

IN LOS ANGELES, the corner of Melrose and Harper has become a tourist destination to rival the
Eiffel Tower, or the graffitied remains of the Berlin Wall. Rather than an architectural marvel or a
piece of living history, people line up (or did, in pre-Covid times) to visit the bright pink exterior
wall of Paul Smith, a clothing retailer. The wall—repainted every three months in the Pantone
shade “Pink Ladies”—is the background to hundreds of thousands of photos, making it one of the
most Instagrammed places in Los Angeles, and even the world.
(Adapted from https://www.wired.com/story/fake-famous-review-instagram-influencers-documentary/)

EM LOS ANGELES, a esquina da Melrose com a Harper tornou-se um destino turístico que rivaliza
com a Torre Eiffel ou com os restos grafitados do Muro de Berlim. Em vez de uma maravilha
arquitetônica ou um pedaço da história viva, as pessoas fazem fila (ou faziam, em tempos pré-
Covid) para visitar a parede externa rosa brilhante de Paul Smith, um varejista de roupas. A parede
– repintada a cada três meses no tom Pantone “Pink Ladies” - é o plano de fundo para centenas
de milhares de fotos, tornando-se um dos lugares mais Instagrammed em Los Angeles, e até
mesmo no mundo.

Russian Fabergé exhibition contains 'at least 20 fakes', expert says


Andre Ruzhnikov accuses director Mikhail Piotrovsky of ‘destroying the authority of the museum’
A Russian museum __________ to close a Fabergé exhibition that contains items loaned from the
personal collection of a billionaire, after a prominent expert said it contained more than a dozen fakes.
In a letter, the art dealer Andre Ruzhnikov accused the Hermitage Museum’s director, Mikhail
Piotrovsky, of “destroying the authority of the museum” by hosting the Fabergé: Jeweller to the
Imperial Court show, which runs until 14 March.
Ruzhnikov told the Guardian that the exhibition included at least 20 fakes, and that he thought the
exhibition, which is the first big Fabergé event at the St Petersburg institution since 1993, should close
immediately. He said: “I want the shame to end. I want this show to be closed and forgotten, and
that’s it. You cannot subject the Hermitage to such shame.”
The Hermitage and Alexander Ivanov have denied the claims and the billionaire produced documents
that support the authenticity of the items that were loaned from the Fabergé Museum in Baden-
Baden, which he established in 2009.
Piotrovsky and the Hermitage did not respond to a request for comment.
(Adapted from https://www.theguardian.com/artanddesign/2021/feb/01/russian-faberge-exhibition-contains-fakes-expert-says)

A exposição russa Fabergé contém 'pelo menos 20 falsificações', diz especialista


Andre Ruzhnikov acusa o diretor Mikhail Piotrovsky de "destruir a autoridade do museu"
Um museu russo __________ para fechar uma exposição de Fabergé que contém itens
emprestados da coleção pessoal de um bilionário, depois que um importante especialista disse
que ela continha mais de uma dúzia de falsificações.

AULA 07 – LINKING WORDS (CONECTIVES) AND QUESTION WORDS (WH QUESTIONS) 124
TEACHER ANDREA BELO

Em uma carta, o negociante de arte Andre Ruzhnikov acusou o diretor do Museu Hermitage,
Mikhail Piotrovsky, de "destruir a autoridade do museu" ao hospedar a mostra Fabergé: Joalheiro
da Corte Imperial, que vai até 14 de março.
Ruzhnikov disse ao Guardian que a exposição incluía pelo menos 20 falsificações e que ele achava
que a exposição, que é o primeiro grande evento Fabergé na instituição de São Petersburgo desde
1993, deveria fechar imediatamente. Ele disse: “Quero que a vergonha acabe. Eu quero que esse
show seja encerrado e esquecido, e é isso. Você não pode sujeitar l'Hermitage a tal vergonha. ”
O Hermitage e Alexander Ivanov negaram as reivindicações e o bilionário produziu documentos
que comprovam a autenticidade dos itens que foram emprestados do Museu Fabergé em Baden-
Baden, que ele fundou em 2009.
Piotrovsky e o Hermitage não responderam a um pedido de comentário.

When will offices be full again?


Maybe never, some executives say.
By Lauren Hirsch
Many companies do not expect their workers to return to offices until next summer, and even
then things may never be the same as before, judging by the comments executives made this
week, highlighted in today’s DealBook newsletter.
On earnings calls, executives from Goldman Sachs said that about a third of workers in New York
and London were coming in regularly; at JPMorgan Chase, it’s around 20 percent in both cities;
and Citigroup said “a small percentage” of employees in North America had returned.
“Being together enables greater collaboration, which is key to our culture,” said David M.
Solomon, Goldman’s chief. But Jamie Dimon of JPMorgan acknowledged that some working
habits may have changed permanently, which “will ultimately reduce the space you need for your
employees.” Terrance R. Dolan, the finance chief at U.S. Bancorp, told analysts that the bank will
most likely “consolidate” its corporate real estate to reflect “the new horizon.”
Is that a problem? Steven J. Goulart, the chief investment officer at MetLife, said at a regulatory
round table that the “pressure to de-densify” offices to support social distancing could support
demand for real estate even if buildings aren’t as full as before.
And as executives conduct more business remotely, going back to in-person meetings and pitches
seems less urgent. Natarajan Chandrasekaran, the chairman of Indian conglomerate Tata Sons,
said in an interview with The New York Times that he used to fly from India to the United States
to pitch a $50,000 project. But recently, he said, his firm’s consultancy business closed $2 billion
worth of deals in “five or six Zoom calls.”
There are other perks from working at home. BlackRock’s Laurence D. Fink is excited about what
employees could do with the time they save on daily commutes. “They could spend two hours
improving their health by exercising,” he said on a conference call. “They could spend two hours
more in building a deeper, stronger, more resilient family.”

AULA 07 – LINKING WORDS (CONECTIVES) AND QUESTION WORDS (WH QUESTIONS) 125
TEACHER ANDREA BELO

Paul Draovitch of Duke Energy said at an investor event that working from home was “not without
risks,” but also brought certain benefits: “When my Pomeranians walk into the room, it's really a
pleasure.”
Adapted from: https://www.nytimes.com/2020/10/16/business/when-will-offices-be-full-again-maybe-never-some-executives-say.html

Quando os escritórios estarão cheios novamente?


Talvez nunca, dizem alguns executivos.
Por Lauren Hirsch
Muitas empresas não esperam que seus funcionários voltem aos escritórios até o próximo verão
e, mesmo assim, as coisas podem nunca mais ser as mesmas de antes, a julgar pelos comentários
que os executivos fizeram esta semana, destacados no boletim informativo DealBook de hoje.
Em chamadas de lucros, executivos da Goldman Sachs disseram que cerca de um terço dos
trabalhadores em Nova York e Londres vinham regularmente; no JPMorgan Chase, é cerca de 20%
nas duas cidades; e o Citigroup disse que “uma pequena porcentagem” de funcionários na
América do Norte havia retornado.
“Estarmos juntos permite uma maior colaboração, que é a chave para nossa cultura”, disse David
M. Solomon, chefe do Goldman. Mas Jamie Dimon, do JPMorgan, reconheceu que alguns hábitos
de trabalho podem ter mudado permanentemente, o que “acabará reduzindo o espaço de que
você precisa para seus funcionários”. Terrance R. Dolan, chefe financeiro do U.S. Bancorp, disse a
analistas que o banco provavelmente "consolidará" seu patrimônio corporativo para refletir "o
novo horizonte".
Isso é um problema? Steven J. Goulart, o diretor de investimentos da MetLife, disse em uma mesa
redonda regulatória que a "pressão para diminuir a densidade" de escritórios para apoiar o
distanciamento social poderia apoiar a demanda por imóveis, mesmo que os prédios não
estivessem tão cheios como antes.
E, à medida que os executivos conduzem mais negócios remotamente, voltar às reuniões e
apresentações pessoais parece menos urgente. Natarajan Chandrasekaran, presidente do
conglomerado indiano Tata Sons, disse em uma entrevista ao The New York Times que costumava
voar da Índia para os Estados Unidos para lançar um projeto de US $ 50.000. Mas recentemente,
disse ele, o negócio de consultoria de sua empresa fechou negócios no valor de US $ 2 bilhões em
"cinco ou seis ligações da Zoom".
Existem outras vantagens de trabalhar em casa. Laurence D. Fink, da BlackRock, está animado
com o que os funcionários podem fazer com o tempo que economizam no deslocamento diário.
“Eles poderiam passar duas horas melhorando sua saúde se exercitando”, disse ele em uma
teleconferência. “Eles poderiam gastar mais duas horas construindo uma família mais profunda,
mais forte e mais resiliente.”
Paul Draovitch, da Duke Energy, disse em um evento para investidores que trabalhar em casa
“não era isento de riscos”, mas também trouxe alguns benefícios: “Quando meus pomeranos
entram na sala, é realmente um prazer”.

AULA 07 – LINKING WORDS (CONECTIVES) AND QUESTION WORDS (WH QUESTIONS) 126
TEACHER ANDREA BELO

Inflation: Used cars and food push US prices higher


Consumer prices jumped 5.4% in the 12 months to the end of June, up from 5% _______ previous
month.
It marks the biggest 12-month increase _______ August 2008, according to the US Labor
Department.
Inflation, which measures the rate at which cost of living increases, has been rising as the economy
reopens from coronavirus lockdowns.
It has sparked fears that prices _______ increasing too quickly, which could prompt the Federal
Reserve to push up interest rates or pull back on pandemic support earlier than expected.
However, some economists and the Federal Reserve say that the inflationary pressures will be
temporary.
Used vehicles accounted for one-third of the increase in the Consumer Price Index (CPI) _______
June, the Labor Department said _______ Tuesday.
But prices also reflected a broader surge in consumer demand as restrictions eased, with the costs
of meals in restaurants and cafes, hotel stays and airline tickets all rising last month.
(https://www.bbc.com/news/business-57573387)

Inflação: carros usados e alimentos elevam os preços nos EUA


Os preços ao consumidor subiram 5,4% nos 12 meses até o final de junho, ante 5% _______ do
mês anterior.
É o maior aumento em 12 meses _______ de agosto de 2008, de acordo com o Departamento do
Trabalho dos EUA.
A inflação, que mede a taxa de aumento do custo de vida, tem aumentado à medida que a
economia se reabre devido ao bloqueio do coronavírus.
Isso gerou temores de que os preços _______ subam muito rapidamente, o que poderia levar o
Federal Reserve a elevar as taxas de juros ou reduzir o apoio à pandemia mais cedo do que o
esperado.
No entanto, alguns economistas e o Federal Reserve dizem que as pressões inflacionárias serão
temporárias.
Os veículos usados foram responsáveis por um terço do aumento no Índice de Preços ao
Consumidor (IPC) de _______ junho, o Departamento do Trabalho disse _______ terça-feira.
Mas os preços também refletiram um aumento mais amplo na demanda do consumidor à medida
que as restrições diminuíram, com os custos de refeições em restaurantes e cafés, estadias em
hotéis e passagens aéreas aumentando no mês passado.

AULA 07 – LINKING WORDS (CONECTIVES) AND QUESTION WORDS (WH QUESTIONS) 127
TEACHER ANDREA BELO

Lost Dog
Sam saw a dog on the sidewalk. The dog looked lost. Sam approached the dog and patted its head.
The dog wagged his tail. The dog had a collar. Sam looked at the collar and it read the dog's name.
Its name was Spike. Spike was a small dog and looked well-groomed. “Come on, Spike,” he said to
the dog. The dog followed him home. He brought the dog upstairs to his room. He had a jar of
dog food because he used to have a dog. He poured the dog food into a bowl and placed it in front
of Spike. Spike started eating. “I'll take care of you, Spike,” he said to the dog. He knew the dog
did not belong to him. He printed posters of Spike that said “Found Dog”. He went around hanging
the poster but nobody came to find Spike.
(Adapted from <https://www.eslfast.com/begin2/b2/b2014.htm>)

Cachorro perdido
Sam viu um cachorro na calçada. O cachorro parecia perdido. Sam se aproximou do cachorro e
acariciou sua cabeça. O cachorro abanou o rabo. O cachorro tinha uma coleira. Sam olhou para a
coleira e leu o nome do cachorro. Seu nome era Spike. Spike era um cachorro pequeno e parecia
bem cuidado. “Vamos, Spike”, ele disse ao cachorro. O cachorro o seguiu para casa. Ele levou o
cachorro para seu quarto no andar de cima. Ele tinha um pote de comida de cachorro porque
costumava ter um cachorro. Ele despejou a comida de cachorro em uma tigela e colocou na frente
de Spike. Spike começou a comer. “Vou cuidar de você, Spike”, disse ele ao cachorro. Ele sabia
que o cachorro não pertencia a ele. Ele imprimiu pôsteres de Spike que diziam “Cachorro
Encontrado”. Ele andou pendurando o pôster, mas ninguém apareceu para encontrar Spike.

We must not let hope abandon us, because God, with his love, walks with us. “I hope, because
God is beside me”: we can all say this. Each one of us can say: “I hope, I have hope, because God
walks with me.” He walks and he holds my hand. God does not leave us to ourselves. The Lord
Jesus has conquered evil and has opened the path of life for us. Let us allow the Lord to teach us
what it means to hope. ___________, let us listen to the words of Sacred Scripture, beginning
with the prophet Isaiah, the great messenger of hope.
By Pope Francis

Não devemos deixar a esperança nos abandonar, porque Deus, com seu amor, caminha conosco.
“Espero, porque Deus está ao meu lado”: todos podemos dizer isso. Cada um de nós pode dizer:
“Espero, tenho esperança, porque Deus caminha comigo”. Ele caminha e segura minha mão. Deus
não nos deixa sozinhos. O Senhor Jesus venceu o mal e abriu o caminho da vida para nós. Vamos
permitir que o Senhor nos ensine o que significa ter esperança. ___________, ouçamos as
palavras da Sagrada Escritura, começando pelo profeta Isaías, o grande mensageiro da esperança.

AULA 07 – LINKING WORDS (CONECTIVES) AND QUESTION WORDS (WH QUESTIONS) 128
TEACHER ANDREA BELO

Adele – Someone Like You


I heard that you're settled down
That you found a girl and you're married now
I heard that your dreams came true
Guess she gave you things, I didn't give to you
Old friend, why are you so shy?
Ain't like you to hold back or hide from the light
I hate to turn up out of the blue, uninvited
But I couldn't stay away, I couldn't fight it
I had hoped you'd see my face
And that you'd be reminded that for me, it isn't over
(Adapted from https://www.letras.mus.br/adele/1778689/)

Adele - Alguém como você


Ouvi dizer que você sossegou
Que você encontrou uma garota e está casado agora
Ouvi dizer que seus sonhos se tornaram realidade
Acho que ela deu coisas pra você, eu não dei pra você
Velho amigo, por que você é tão tímido?
Não é como se você se contivesse ou se escondesse da luz
Eu odeio aparecer do nada, sem ser convidado
Mas eu não pude ficar longe, não pude lutar contra isso
Eu esperava que você visse meu rosto
E que você seja lembrado de que, para mim, ainda não acabou

There’s no end to the benefits of donating blood for those who need it.
According to the American Red Cross, one donation can save as many as three lives, and someone
in the United States needs blood every two seconds.
It turns out that donating blood doesn’t just benefit recipients. There are health benefits for
donors, too, on top of the benefits that come from helping others.
Donating blood has benefits for your emotional and physical health. According to a report by the
Mental Health Foundation, helping others can: reduce stress, improve your emotional well-being,

AULA 07 – LINKING WORDS (CONECTIVES) AND QUESTION WORDS (WH QUESTIONS) 129
TEACHER ANDREA BELO

benefit your physical health, help get rid of negative feelings, provide a sense of belonging and
reduce isolation.
Research has found further evidence of the health benefits that come specifically from donating
blood.

Não há fim para os benefícios de doar sangue para quem precisa.


De acordo com a Cruz Vermelha americana, uma doação pode salvar até três vidas, e alguém nos
Estados Unidos precisa de sangue a cada dois segundos.
Acontece que doar sangue não beneficia apenas os destinatários. Também existem benefícios
para a saúde para os doadores, além dos benefícios que advêm de ajudar os outros.
Doar sangue traz benefícios para sua saúde física e emocional. De acordo com um relatório da
Fundação de Saúde Mental, ajudar os outros pode: reduzir o estresse, melhorar seu bem-estar
emocional, beneficiar sua saúde física, ajudar a livrar-se de sentimentos negativos, proporcionar
um sentimento de pertencimento e reduzir o isolamento.
Pesquisas encontraram evidências adicionais dos benefícios à saúde que vêm especificamente da
doação de sangue.

Online school to continue into next academic year amid risk of further disruption
A virtual school will stay open for at least most of the next academic year amid a rise in the number
of pupils selfisolating.
Oak National Academy has been used by pupils and teachers to support remote learning - which
has taken place during lockdowns and - during the Covid pandemic.
The Reach Foundation, which the school is part of, said it will keep operating at no cost for at least
the next two terms.
The charity said this will help teachers to plan for the start of the next academic year by being
able to access Oak National Academy’s online lessons and learning resources.
The announcement comes against a backdrop of rising Covid infections in the UK and the large
numbers of children self-isolating as the school year drew to a close.
More than one million children were out of school last week due to Covid-related reasons - such
as a positive test, suspected Covid case or being told to isolate - just before schools broke up for
the summer holiday.
In the final weeks of term, over 300,000 pupils used Oak National Academy’s online learning
lessons and resources.
Schools are already considering what measures they may need to put in place should pupils test
positive, or they are asked to self-isolate in the next academic year.
The extension of the virtual school into the autumn and spring terms is backed by £2.1m in funding
from the Department for Education (DfE).

AULA 07 – LINKING WORDS (CONECTIVES) AND QUESTION WORDS (WH QUESTIONS) 130
TEACHER ANDREA BELO

Ed Vainker from the Reach Foundation - which has incubated Oak since its launch - said the charity
understood teachers wanted the online school to continue.
“We are also mindful that Covid-19 will be with us in some form for the foreseeable future, and
the autumn and winter may bring further disruption,” the chief executive said.
“I’m therefore pleased the DfE will continue to support Oak, so it remains open and free as part
of resilience planning for the next two terms.”
Nick Gibb, the schools minister, said: “From being set up by a small group of teachers in under
two weeks, Oak National Academy has become one of the great success stories as we tackle the
pandemic.”
He said the online school helped millions of pupils and teachers while most children were not
allowed to set foot in school during a lockdown last spring and one at the start of this year.
But even since after all students were allowed back on since in March, Oak National Academy said
30,000 teachers have used its services a week, with demand rising amid the spread of the Delta
variant.
“The impact Oak has made and the good it has done for the sector and children is immeasurable,
and we will now look for the best way to harness that for the future,” Mr Gibb added.
https://www.independent.co.uk/news/education/education-news/uk-online-school-two-terms-b1888491.html

Escola online para continuar no próximo ano letivo em meio ao risco de mais interrupções
Uma escola virtual permanecerá aberta pelo menos durante a maior parte do próximo ano letivo
em meio a um aumento no número de alunos que se auto-isolam.
A Oak National Academy tem sido usada por alunos e professores para apoiar o aprendizado
remoto - que ocorreu durante bloqueios e - durante a pandemia de Covid.
A Fundação Reach, da qual a escola faz parte, disse que continuará operando sem custo pelo
menos nos próximos dois períodos.
A instituição de caridade disse que isso ajudará os professores a planejarem o início do próximo
ano acadêmico, podendo acessar as aulas online e os recursos de aprendizagem da Oak National
Academy.
O anúncio vem em um cenário de aumento de infecções por Covid no Reino Unido e o grande
número de crianças se isolando enquanto o ano letivo se aproximava.
Mais de um milhão de crianças estiveram fora da escola na semana passada devido a motivos
relacionados à Covid - como um teste positivo, suspeita de caso Covid ou ser informada para se
isolar - pouco antes do término das aulas para as férias de verão.
Nas últimas semanas do semestre, mais de 300.000 alunos usaram as lições e recursos de
aprendizagem online da Oak National Academy.
As escolas já estão considerando quais medidas podem precisar implementar caso os alunos
tenham resultados positivos, ou se eles forem solicitados a isolar-se no próximo ano letivo.
A extensão da escola virtual para os períodos de outono e primavera é apoiada por £ 2,1 milhões
em financiamento do Departamento de Educação (DfE).

AULA 07 – LINKING WORDS (CONECTIVES) AND QUESTION WORDS (WH QUESTIONS) 131
TEACHER ANDREA BELO

Ed Vainker da Reach Foundation - que incubou Oak desde seu lançamento - disse que a instituição
de caridade entendeu que os professores queriam que a escola online continuasse.
“Também estamos cientes de que a Covid-19 estará conosco de alguma forma no futuro
previsível, e o outono e o inverno podem trazer mais perturbações”, disse o presidente-executivo.
“Estou, portanto, satisfeito que o DfE continuará a apoiar o Oak, então ele permanece aberto e
gratuito como parte do planejamento de resiliência para os próximos dois mandatos.”
Nick Gibb, o ministro das escolas, disse: “Tendo sido criada por um pequeno grupo de professores
em menos de duas semanas, a Oak National Academy tornou-se uma das grandes histórias de
sucesso no combate à pandemia.”
Ele disse que a escola online ajudou milhões de alunos e professores, enquanto a maioria das
crianças não teve permissão para colocar os pés na escola durante um bloqueio na última
primavera e outro no início deste ano.
Mas mesmo depois que todos os alunos foram autorizados a voltar em março, a Oak National
Academy disse que 30.000 professores usaram seus serviços por semana, com a demanda
crescendo em meio à disseminação da variante Delta.
“O impacto que a Oak fez e o bem que fez para o setor e as crianças é incomensurável, e agora
vamos procurar a melhor maneira de aproveitar isso para o futuro”, acrescentou o Sr. Gibb.

Cannabis part of the future says tobacco giant


The UK's largest tobacco firm says it __________ cannabis as part of its future as it tries to move
away from selling traditional cigarettes. British American Tobacco said it wanted to "accelerate"
its transformation by reducing the health impact of its products.
https://www.bbc.com/news/business-57995285

Cannabis faz parte do futuro, diz gigante do tabaco


A maior empresa de tabaco do Reino Unido afirma que __________ cannabis como parte de seu
futuro, enquanto tenta deixar de vender cigarros tradicionais. A British American Tobacco disse
que deseja "acelerar" sua transformação reduzindo o impacto de seus produtos na saúde.

A Belgian chocolate company is now ____ 3D printers which allow the company to create more
intricate, difficultto-mould chocolates. The chocolates are intended for people who seek original
designs.
The chocolate is melted before _____ poured into a syringe which is attached to the printer and
since the chocolates are hard to transport, the company _____ to open other shops around the
world.
https://www.newsinlevels.com/products/chocolate-3d-printing-level-3/

AULA 07 – LINKING WORDS (CONECTIVES) AND QUESTION WORDS (WH QUESTIONS) 132
TEACHER ANDREA BELO

Uma empresa de chocolate belga agora ____ impressoras 3D, o que permite à empresa criar
chocolates mais complexos e difíceis de moldar. Os chocolates são destinados a pessoas que
buscam designs originais.
O chocolate é derretido antes de _____ despejar em uma seringa acoplada à impressora e como
os chocolates são difíceis de transportar, a empresa _____ abrirá outras lojas ao redor do mundo.

The small town of Pelhrimov in the Czech Republic _______ thousands of strange records and
hosted a festival to show them off. Some of the records were racing with folding ladders and
folding pink scarves. The president of the festival _______ that he was happy to have people come
and understand the ‘essence’ of what the people in this town _______. He wants to show that life
is more than being on Facebook or sitting at a computer. It is good to do something!
Adapted from: https://www.newsinlevels.com/products/town-has-strange-records-level-3/

A pequena cidade de Pelhrimov, na República Tcheca, _______ milhares de registros estranhos e


sediou um festival para exibi-los. Alguns dos recordes estavam correndo com escadas dobráveis
e lenços rosa dobráveis. O presidente do festival _______ que ele estava feliz por ter gente vindo
e entendendo a 'essência' do que as pessoas nesta cidade _______. Ele quer mostrar que a vida
é mais do que estar no Facebook ou sentado em um computador. É bom fazer alguma coisa!

Jeff Bezos, _______ richest man __________ Earth, visited space on Tuesday. He and three other
crew members soared about 107 kilometers above the Texas desert aboard his company Blue
Origin´s New Shepard rocket.
_______ historic unpiloted sub-orbital flight helped to inaugurate a new era of private commercial
space tourism. The trip to the edge of space lasted about 10 minutes and 20 seconds, including a
few minutes of weightlessness. The crew included his brother Mark, Wally Funk, an 82-year-old
pioneering female aviator, and Oliver Daemen, an 18-year-old Dutch physics student.
Bezos, who recently stepped down as Amazon´s CEO, thanked employees and customers _______
the company that he founded, saying that their hard work and Amazon purchases had made his
space flight possible. His words sparked criticism, with some politicians saying that Amazon
abused its market power to hurt small businesses. They also said Bezos should have spent
_______ money to pay his workers fairly and protect their health.
https://www.newsinlevels.com/products/the-worlds-richest-man-visits-space-level-3/

Jeff Bezos, _______ homem mais rico __________ Terra, visitou o espaço na terça-feira. Ele e três
outros membros da tripulação voaram cerca de 107 quilômetros acima do deserto do Texas a
bordo do foguete New Shepard de sua empresa, Blue Origin.

AULA 07 – LINKING WORDS (CONECTIVES) AND QUESTION WORDS (WH QUESTIONS) 133
TEACHER ANDREA BELO

_______ o histórico voo suborbital não-piloto ajudou a inaugurar uma nova era de turismo
espacial comercial privado. A viagem até a borda do espaço durou cerca de 10 minutos e 20
segundos, incluindo alguns minutos sem gravidade. A tripulação incluía seu irmão Mark, Wally
Funk, uma aviadora pioneira de 82 anos, e Oliver Daemen, um estudante holandês de física de 18
anos.
Bezos, que recentemente deixou o cargo de CEO da Amazon, agradeceu aos funcionários e
clientes _______ a empresa que fundou, dizendo que seu trabalho árduo e as compras da Amazon
tornaram seu voo espacial possível. Suas palavras geraram críticas, com alguns políticos dizendo
que a Amazon abusou de seu poder de mercado para prejudicar pequenos negócios. Eles também
disseram que Bezos deveria ter gasto _______ dinheiro para pagar seus trabalhadores de forma
justa e proteger sua saúde.

Why We Must Improve Vaccine Manufacturing Before the Next Pandemic


It should worry everyone that experts surveyed by TIME regarded both increasing funding in a
post-COVID-19 world for vaccine development and scaling up of manufacturing capacity
feasible—but improving equitable vaccine distribution was not.
To stop the next pandemic in its tracks we need to ensure that people all over the world are
protected quickly, and that will entail having all these pieces in place. The good news is, all these
elements are feasible, and indeed starting to work today.
On vaccine R&D, the Coalition for Epidemic Preparedness Innovations (CEPI), was set up with the
precise purpose of identifying and investing in R&D for vaccines against emerging infectious
diseases with epidemic potential. So, when it came to COVID-19, with CEPI’s and other R&D
support, as well as industry engagement, the scientific and vaccine manufacturing community
rallied, producing the first safe and effective vaccine in record time—just 327 days. Today we have
not just one but 15 in widespread use.
Increased investment now could get us there even faster the next time, particularly given the
potential of the relatively new RNA vaccine technologies that have proved so effective with
COVID-19. These plug-and-play vaccine technologies not only make it possible to identify and
develop antigens rapidly, but much of the regulatory testing and approval can be done in advance,
even before we know what the threat is.
As for manufacturing, it may be difficult to immediately discern when there are severe supply
shortages, but the world has actually rapidly built-up manufacturing capacity during COVID-19.
Waiving intellectual property has been talked about a lot as a potential solution for boosting
production. But the growth we have seen in the past year has been achieved through technology
transfers, where both the intellectual property and the vital know-how needed to make vaccines
is shared between manufacturers.
However, we need to do more. Given the extremely large number of doses needed during a
pandemic, export bans of vaccines and essential components and supply bottlenecks have led to
a vaccine divide. Currently, more than a third of adults in high-income countries have now been
vaccinated, while less than 1% of those in low-income countries have had their first jab.

AULA 07 – LINKING WORDS (CONECTIVES) AND QUESTION WORDS (WH QUESTIONS) 134
TEACHER ANDREA BELO

To prevent this kind of scenario from happening the next time round and ensure that those most
at risk are prioritized wherever they are, it is not distribution channels we are lacking, but global
manufacturing capacity. We already have highly effective distribution channels, through COVAX
and its partners, and we already have access to doses, enough to protect 1.8 billion people in
lower-income economies by early next year, enough to protect almost 30% of people in these
countries.
But through investments now to increase global manufacturing capacity, particularly in emerging
economies, and support of technology transfers, the next time a pandemic strikes we can get
there sooner.
(Adapted from https://time.com/6072101/improve-vaccine-manufacturing/)

Por que devemos melhorar a fabricação de vacinas antes da próxima pandemia


Deveria preocupar a todos que os especialistas pesquisados pela TIME considerassem o aumento
do financiamento em um mundo pós-COVID-19 para o desenvolvimento de vacinas e o aumento
da capacidade de fabricação viável - mas não era possível melhorar a distribuição equitativa da
vacina.
Para interromper a próxima pandemia em seu caminho, precisamos garantir que as pessoas em
todo o mundo sejam protegidas rapidamente, e isso implicará em ter todas essas peças no lugar.
A boa notícia é que todos esses elementos são viáveis e, de fato, estão começando a funcionar
hoje.
Em P&D de vacinas, foi criada a Coalition for Epidemic Preparedness Innovations (CEPI) com o
objetivo preciso de identificar e investir em P&D de vacinas contra doenças infecciosas
emergentes com potencial epidêmico. Então, quando se tratou do COVID-19, com a CEPI e outros
apoios de P&D, bem como o envolvimento da indústria, a comunidade científica e de fabricação
de vacinas se reuniu, produzindo a primeira vacina segura e eficaz em tempo recorde - apenas
327 dias. Hoje temos não apenas um, mas 15 em uso generalizado.
O aumento do investimento agora pode nos levar lá ainda mais rápido da próxima vez,
especialmente devido ao potencial das tecnologias de vacina de RNA relativamente novas que se
mostraram tão eficazes com COVID-19. Essas tecnologias de vacinas plug-and-play não apenas
possibilitam identificar e desenvolver antígenos rapidamente, mas muitos dos testes regulatórios
e da aprovação podem ser feitos com antecedência, mesmo antes de sabermos qual é a ameaça.
Quanto à manufatura, pode ser difícil discernir imediatamente quando há severa escassez de
suprimentos, mas o mundo, na verdade, aumentou rapidamente a capacidade de manufatura
durante o COVID-19. Tem-se falado muito sobre a renúncia à propriedade intelectual como uma
solução potencial para aumentar a produção. Mas o crescimento que vimos no ano passado foi
alcançado por meio de transferências de tecnologia, onde tanto a propriedade intelectual quanto
o know-how vital necessário para fazer vacinas são compartilhados entre os fabricantes.
No entanto, precisamos fazer mais. Dado o número extremamente grande de doses necessárias
durante uma pandemia, as proibições de exportação de vacinas e componentes essenciais e os
gargalos no fornecimento levaram a uma divisão de vacinas. Atualmente, mais de um terço dos

AULA 07 – LINKING WORDS (CONECTIVES) AND QUESTION WORDS (WH QUESTIONS) 135
TEACHER ANDREA BELO

adultos em países de alta renda já foram vacinados, enquanto menos de 1% daqueles em países
de baixa renda receberam sua primeira vacina.
Para evitar que esse tipo de cenário aconteça na próxima vez e garantir que aqueles que estão
em maior risco sejam priorizados onde quer que estejam, não são os canais de distribuição que
faltam, mas a capacidade de produção global. Já temos canais de distribuição altamente eficazes,
por meio da COVAX e seus parceiros, e já temos acesso a doses, o suficiente para proteger 1,8
bilhão de pessoas em economias de baixa renda até o início do próximo ano, o suficiente para
proteger quase 30% das pessoas nesses países.
Mas, por meio de investimentos agora para aumentar a capacidade de manufatura global,
especialmente nas economias emergentes, e do apoio a transferências de tecnologia, na próxima
vez que ocorrer uma pandemia, poderemos chegar lá mais cedo.

“To stop the next pandemic in its tracks we need to ensure that people all over the world are
protected quickly, and that will entail having all these pieces in place”.

“Para parar a próxima pandemia em seu caminho, precisamos garantir que as pessoas em todo
o mundo sejam protegidas rapidamente, e isso implicará em ter todas essas peças no lugar”.

Children who ate ‘cannabis sweets’ risked serious harm, say Surrey police
Investigation begins after two boys, 12 and 13, and a girl, 12, were taken to hospital after
falling unconscious
Four children who were taken to hospital after eating sweets they thought contained cannabis
are not believed to have suffered long-term effects, police have said.
The youngsters were found vomiting uncontrollably and falling in and out of consciousness on
Saturday afternoon on Pound Lane in Epsom, according to Surrey police. A 12-year-old girl, 12-
year-old boy and 13-year-old boy were taken to hospital by the South East Coast ambulance
service and are lucky not to have been more seriously affected by whatever they ate, the police
said.
They suffered “a violent reaction from eating the jelly sweets that they believed contained
cannabis”, police said.
Another 12-year-old boy was taken to hospital by his family because he was also believed to have
eaten the sweets.
One of the 12-year-old boys was discharged from hospital late on Saturday and the other three
were expected to leave on Sunday. The source of the sweets is still being investigated and police
do not have a clear idea of what they look like, although a similar report from Friday night involved
“jelly apple rings” which look like green jelly circles.

AULA 07 – LINKING WORDS (CONECTIVES) AND QUESTION WORDS (WH QUESTIONS) 136
TEACHER ANDREA BELO

DS Lisa Betchley said: “These children were incredibly lucky that they were not more seriously
affected by whatever it was that they ate – and this is thanks, in great part, to the prompt actions
of two medical students who happened to be nearby and assisted in the early stages, as well as
the South East Coast ambulance service and hospital staff for their rapid response and treatment.
(Adapted from https://www.theguardian.com/uk-news/2021/may/02/children-who-ate-cannabis-sweets-risked-serious-harm-say-surrey-police)

Crianças que comeram "doces de cannabis" correram risco de sofrer sérios danos, diz a polícia
de Surrey
A investigação começa depois que dois meninos, de 12 e 13 anos, e uma menina de 12 anos,
foram levados ao hospital após ficarem inconscientes
Segundo a polícia, quatro crianças que foram levadas ao hospital após comerem doces que
pensavam conter cannabis não teriam sofrido efeitos de longo prazo.
Os jovens foram encontrados vomitando incontrolavelmente e perdendo e perdendo a
consciência na tarde de sábado em Pound Lane, em Epsom, de acordo com a polícia de Surrey.
Uma menina de 12 anos, um menino de 12 anos e um menino de 13 anos foram levados ao
hospital pelo serviço de ambulância da Costa Sudeste e têm sorte de não ter sido mais seriamente
afetados por tudo o que comeram, disse a polícia. .
Eles sofreram “uma reação violenta ao comer os doces de geléia que eles acreditavam conter
cannabis”, disse a polícia.
Outro menino de 12 anos foi levado ao hospital por sua família porque ele também teria comido
os doces.
Um dos meninos de 12 anos recebeu alta do hospital na noite de sábado e os outros três deveriam
sair no domingo. A origem dos doces ainda está sendo investigada e a polícia não tem uma ideia
clara de como eles se parecem, embora um relatório semelhante na sexta-feira à noite envolvesse
“anéis de gelatina de maçã” que se parecem com círculos de gelatina verdes.
A DS Lisa Betchley disse: “Essas crianças tiveram uma sorte incrível de não terem sido afetadas
mais seriamente pelo que quer que comessem - e isso se deve, em grande parte, às ações
imediatas de dois estudantes de medicina que por acaso estavam por perto e ajudaram nos
estágios iniciais, bem como ao serviço de ambulância da Costa Sudeste e à equipe do hospital por
sua rápida resposta e tratamento.

Will Covid-19 vaccines reduce virus transmission?


Vaccinated people can still get infected, but they are less likely to pass it on
There are two ways that getting vaccinated can slow the spread of the virus. First, _______ (1)
can help prevent you getting infected. Second, even if you are unlucky and catch the virus, it may
reduce the risk of passing it on. It is crucial to understand how big these benefits are.
Two huge new studies have taken advantage of the successful UK vaccine rollout. An Oxford-ONS
analysis _______ (2) more than 370,000 survey participants found infections were reduced by

AULA 07 – LINKING WORDS (CONECTIVES) AND QUESTION WORDS (WH QUESTIONS) 137
TEACHER ANDREA BELO

65% after a single dose. For protection against the virus, one dose was similar _______ (3) having
had a prior infection. There was no major difference between the two available vaccines.
Curiously, infection rates were lower up to three weeks before the jab. Did the virus have magical
pre-cognition and keep away? More plausible is the idea of “reverse causation”. People can have
the vaccination only if they have not tested positive or shown recent symptoms, so it is inevitable
there were fewer recorded infections before vaccinations took place. Statistics can be tricky
things.
Most important, the studies showed that if you are infected after vaccination, it tends to be much
milder, both in terms of self-reported symptoms and viral load.
If vaccinated people develop a weaker infection, then they might be less likely to pass on the virus.
(Adapted from https://www.theguardian.com/theobserver/commentisfree/2021/may/02/vaccinated-people-less-likely-to-pass-covid-on)

As vacinas da Covid-19 reduzirão a transmissão do vírus?


As pessoas vacinadas ainda podem ser infectadas, mas são menos propensas a transmiti-lo
A vacinação pode retardar a propagação do vírus de duas maneiras. Em primeiro lugar, _______
(1) pode ajudar a prevenir a infecção. Em segundo lugar, mesmo que você tenha azar e pegue o
vírus, isso pode reduzir o risco de transmiti-lo. É fundamental entender o quão grandes são esses
benefícios.
Dois novos grandes estudos tiraram proveito do lançamento bem-sucedido da vacina no Reino
Unido. Uma análise Oxford-ONS _______ (2) mais de 370.000 participantes da pesquisa
descobriram que as infecções foram reduzidas em 65% após uma única dose. Para proteção
contra o vírus, uma dose foi semelhante _______ (3) tendo tido uma infecção anterior. Não houve
grande diferença entre as duas vacinas disponíveis.
Curiosamente, as taxas de infecção foram menores até três semanas antes da injeção. O vírus
teve pré-cognição mágica e se manteve afastado? Mais plausível é a ideia de “causalidade
reversa”. As pessoas só podem ser vacinadas se não tiverem testado positivo ou se apresentarem
sintomas recentes, portanto, é inevitável que houvesse menos infecções registradas antes da
vacinação. As estatísticas podem ser complicadas.
Mais importante, os estudos mostraram que, se você for infectado após a vacinação, tende a ser
muito mais brando, tanto em termos de sintomas autorrelatados quanto de carga viral.
Se as pessoas vacinadas desenvolverem uma infecção mais fraca, é menos provável que
transmitam o vírus.

Any amount of alcohol consumption harmful to the brain, finds study


UK study of 25,000 people finds even moderate drinking is linked to lower grey matter density
There is no safe amount of alcohol consumption for the brain, with even “moderate” drinking
adversely affecting nearly every part of it, a study of more than 25,000 people in the UK has found.

AULA 07 – LINKING WORDS (CONECTIVES) AND QUESTION WORDS (WH QUESTIONS) 138
TEACHER ANDREA BELO

The study, which is still to be peer-reviewed, suggests that the more alcohol consumed, the lower
the brain volume. In effect, the more you drink, the worse off your brain.
“There’s no threshold drinking for harm – any alcohol is worse. Pretty much the whole brain seems
to be affected – not just specific areas, as previously thought,” said the lead author, Anya Topiwala,
a senior clinical lecturer at the University of Oxford.
Using the UK Biobank, a substantial database designed to help researchers decode the genetic
and environmental factors that lead some people to develop diseases while others do not,
researchers in this study analysed data from 25,378 participants such as age, sex, education, self-
reported alcohol consumption, brain size and health from MRI scans, information about hospital
and outpatient visits, and memory tests.
Higher volume of alcohol consumption per week was associated with lower grey matter density –
the researchers found, with alcohol explaining up to a 0.8% change in grey matter volume, even
after accounting for individual biological and behavioural characteristics.
This might seem like a small figure, but it is a larger contribution than any other modifiable risk
factors. For example, it is four times the contribution of smoking or BMI, said Topiwala.
Widespread negative associations were also seen between alcohol consumption and integrity of
white matter, the brain fibres that scaffold the billions of neurons that make up grey matter. In
addition, an individual’s underlying conditions such as high blood pressure and high BMI made the
negative association between alcohol and brain health stronger, the researchers found.
Contrary to previous research that suggested there is a benefit to drinking wine in moderation
compared with beer or spirits, the study found no evidence to suggest alcoholic beverage type
conferred differences in risks to the brain.
(Adapted from https://www.theguardian.com/society/2021/may/18/any-amount-of-alcohol-consumption-harmful-to-the-brain-finds-study)

Qualquer quantidade de consumo de álcool prejudicial ao cérebro, conclui estudo


Estudo do Reino Unido com 25.000 pessoas mostra que até mesmo o consumo moderado de
álcool está relacionado à menor densidade de massa cinzenta
Não existe uma quantidade segura de consumo de álcool para o cérebro, mesmo com o consumo
“moderado” afetando adversamente quase todas as partes dele, concluiu um estudo com mais de
25.000 pessoas no Reino Unido.
O estudo, que ainda precisa ser revisado por especialistas, sugere que quanto mais álcool é
consumido, menor é o volume do cérebro. Na verdade, quanto mais você bebe, pior fica o seu
cérebro.
“Não há limite para o mal - qualquer álcool é pior. Quase todo o cérebro parece ser afetado - não
apenas áreas específicas, como se pensava anteriormente ”, disse a autora principal, Anya
Topiwala, professora clínica sênior da Universidade de Oxford.
Usando o UK Biobank, um banco de dados substancial projetado para ajudar os pesquisadores a
decodificar os fatores genéticos e ambientais que levam algumas pessoas a desenvolver doenças
enquanto outras não, os pesquisadores neste estudo analisaram dados de 25.378 participantes,
como idade, sexo, educação, auto-relato consumo de álcool, tamanho do cérebro e saúde a partir

AULA 07 – LINKING WORDS (CONECTIVES) AND QUESTION WORDS (WH QUESTIONS) 139
TEACHER ANDREA BELO

de exames de ressonância magnética, informações sobre visitas hospitalares e ambulatoriais e


testes de memória.
Maior volume de consumo de álcool por semana foi associado a menor densidade de substância
cinzenta - descobriram os pesquisadores, com o álcool explicando uma mudança de até 0,8% no
volume de substância cinzenta, mesmo depois de levar em conta as características biológicas e
comportamentais individuais.
Isso pode parecer um número pequeno, mas é uma contribuição maior do que qualquer outro
fator de risco modificável. Por exemplo, é quatro vezes a contribuição do tabagismo ou do IMC,
disse Topiwala.
Associações negativas generalizadas também foram observadas entre o consumo de álcool e a
integridade da substância branca, as fibras cerebrais que sustentam os bilhões de neurônios que
constituem a substância cinzenta. Além disso, as condições subjacentes de um indivíduo, como
pressão alta e IMC alto, tornam a associação negativa entre o álcool e a saúde do cérebro mais
forte, descobriram os pesquisadores.
Ao contrário de pesquisas anteriores que sugeriam que há um benefício em beber vinho com
moderação em comparação com cerveja ou destilados, o estudo não encontrou evidências que
sugiram que o tipo de bebida alcoólica confere diferenças nos riscos para o cérebro.

“___ Covid-19 crisis has been difficult on people across the globe, including India. In the past few
days there has been intense discussion ___ the decision of our government and Indian vaccine
manufacturers, including SII, ___ export vaccines”.
(Adapted from https://www.theguardian.com/world/2021/may/19/poorer-countries-face-long-delays-receiving-covid-vaccines)

“___ A crise da Covid-19 tem sido difícil para as pessoas em todo o mundo, incluindo a Índia. Nos
últimos dias, tem havido intensa discussão ___ sobre a decisão de nosso governo e dos fabricantes
de vacinas indianos, incluindo SII, ___ exportar vacinas ”.

“A few weeks ago, I __________ my first shot of a vaccine against Covid-19. As the newly
vaccinated exited the clinic, there was a mix of relief and elation on people’s faces. We exchanged
little smiles of solidarity. If we could have burst into spontaneous applause, I __________ sure we
would have done”.
(Adapted from https://www.theguardian.com/commentisfree/2021/may/17/animals-overlooked-allies-fight-against-covid-vaccines)

“Há algumas semanas, __________ minha primeira injeção de vacina contra a Covid-19. Quando
os recém-vacinados saíram da clínica, havia uma mistura de alívio e alegria nos rostos das pessoas.
Trocamos pequenos sorrisos de solidariedade. Se pudéssemos explodir em aplausos espontâneos,
eu __________ certeza que teríamos feito ”.

AULA 07 – LINKING WORDS (CONECTIVES) AND QUESTION WORDS (WH QUESTIONS) 140
TEACHER ANDREA BELO

Can New Zealand's tourism industry make a sustainable return?


New Zealand is hopeful that a recently opened travel bubble with Australia will rekindle its
pandemic-battered tourism industry. However, many are also seeing an opportunity to rethink
how to make the sector more climate friendly.
New Zealand's Queenstown - __________ (1) popular tourist spot - is throbbing with activity.
"To see and hear all the boats and the screaming and the complete joy… it puts a smile on all of
our faces," said Jolanda Cave, the general manager at Shotover Jet - one of the most established
adventure companies there.
It's a busy time __________ (2) the company, named after the river, where for more than half a
decade, boats have been whizzing, spinning and splashing to the delight of tourists.
But even so, the numbers it is seeing are a fraction of what it was used to before the pandemic
closed the country's borders. It used to operate eight boat rides an hour. Sometimes, that was
down to just one.
"It's been real eye opener for us to go from 1,200 (visitors) a day to 200 a day," Ms Cave said.
Like many tourism operators across New Zealand Ms Cave is excited about the recently opened
travel bubble with Australia, its biggest market.
"It's given people hope. Australians represent a huge part of our business. (The bubble) will mean
that we will see some growth. I think a lot of Queenstown will," she says.
In 2019, international tourism was worth $12.6bn (£9bn) in total, __________ (3) Australians
contributing $1.94bn.
Between 1.18 and 1.5 million Australians came to New Zealand annually, accounting for 40% of
the country's overseas visitors.
Those numbers dropped to zero when New Zealand closed its borders.
(Adapted from https://www.bbc.com/news/business-56967636)

A indústria do turismo da Nova Zelândia pode ter um retorno sustentável?


A Nova Zelândia está esperançosa de que uma bolha de viagens recentemente aberta com a
Austrália reacenda sua pandêmica indústria de turismo. No entanto, muitos também estão
vendo uma oportunidade de repensar como tornar o setor mais favorável ao clima.
Queenstown na Nova Zelândia - __________ (1) ponto turístico popular - está fervilhando de
atividade.
“Ver e ouvir todos os barcos e os gritos e a alegria completa… coloca um sorriso em todos os
nossos rostos”, disse Jolanda Cave, gerente geral da Shotover Jet - uma das empresas de aventura
mais estabelecidas lá.
É uma época movimentada __________ (2) a empresa, que leva o nome do rio, onde há mais de
meia década os barcos zunem, giram e espirram para o deleite dos turistas.

AULA 07 – LINKING WORDS (CONECTIVES) AND QUESTION WORDS (WH QUESTIONS) 141
TEACHER ANDREA BELO

Mesmo assim, os números que ele está vendo são uma fração do que era antes de a pandemia
fechar as fronteiras do país. Costumava operar oito viagens de barco por hora. Às vezes, isso era
reduzido para apenas um.
"Foi uma verdadeira revelação para nós passar de 1.200 (visitantes) por dia para 200 por dia",
disse a Sra. Cave.
Como muitas operadoras de turismo na Nova Zelândia, a Sra. Cave está animada com a bolha de
viagens recentemente aberta com a Austrália, seu maior mercado.
"Isso dá esperança às pessoas. Os australianos representam uma grande parte do nosso negócio.
(A bolha) significará que veremos algum crescimento. Acho que muito em Queenstown, sim", diz
ela.
Em 2019, o turismo internacional valia $ 12,6 bilhões (£ 9 bilhões) no total, __________ (3)
australianos contribuindo com $ 1,94 bilhões.
Entre 1,18 e 1,5 milhão de australianos vieram para a Nova Zelândia anualmente, representando
40% dos visitantes estrangeiros do país.
Esses números caíram para zero quando a Nova Zelândia fechou suas fronteiras.

Rich countries close their eyes to the global Covid surge at their own peril
The pandemic’s death toll is now being felt most gravely in developing nations. This virus is
not done yet
Is there one pandemic, or two? That ___ (1) a question being asked a year ago, when wealthy
countries accounting for only 15% of the global population had 80% of the Covid deaths. Could it
be that the rich world was more vulnerable, somehow, because its populations ___ (2) older, or
more individualistic, or had forgotten to be scared of infectious disease?
Even then, some were warning that the worst was yet to come, once the disease took hold in
poorer countries. World Bank analysts Philip Schellekens and Diego Sourrouille, for example,
predicted a “massive shift” in disease burden to the developing world. Just in terms of
demography, they said, you’d expect those countries to account for around 70% of deaths.
As things stand they account for a little over half of it, which is probably an underestimate due to
variations in data quality – and the pandemic is far from over.
Last week saw more than 5.8 million new cases of Covid globally, the highest number yet. More
than 3 million people have now died from Covid, according to the World Health Organization
(WHO), which also reports that infections and hospitalisations in those aged at 25 to 59 ___ (3)
increasing an alarming rate. “It took nine months to reach 1 million deaths, four months to reach
2 million, and three months to reach 3 million,” WHO director general, Tedros Adhanom
Ghebreyesus, said last week.
(Adapted from https://www.theguardian.com/commentisfree/2021/apr/27/rich-countries-covid-pandemic-death-developing-nations)

AULA 07 – LINKING WORDS (CONECTIVES) AND QUESTION WORDS (WH QUESTIONS) 142
TEACHER ANDREA BELO

Os países ricos fecham os olhos para o aumento global da Covid por sua própria conta e risco
O número de mortes da pandemia agora está sendo sentido de forma mais grave nas nações
em desenvolvimento. Este vírus ainda não acabou
Existe uma pandemia ou duas? Isso ___ (1) uma pergunta feita há um ano, quando os países ricos,
responsáveis por apenas 15% da população global, tinham 80% das mortes de Covid. Será que o
mundo rico era mais vulnerável, de alguma forma, porque suas populações ___ (2) mais velhas,
ou mais individualistas, ou se esqueceram de ter medo de doenças infecciosas?
Mesmo assim, alguns alertavam que o pior ainda estava por vir, uma vez que a doença se alastrou
nos países mais pobres. Os analistas do Banco Mundial Philip Schellekens e Diego Sourrouille, por
exemplo, previram uma “mudança massiva” na carga de doenças para o mundo em
desenvolvimento. Apenas em termos de demografia, eles disseram, você espera que esses países
sejam responsáveis por cerca de 70% das mortes.
Do jeito que as coisas estão, eles respondem por pouco mais da metade, o que provavelmente é
uma estimativa subestimada devido às variações na qualidade dos dados - e a pandemia está
longe de terminar.
Na semana passada, assistimos a mais de 5,8 milhões de novos casos de Covid em todo o mundo,
o maior número até agora. Mais de 3 milhões de pessoas morreram de Covid, de acordo com a
Organização Mundial da Saúde (OMS), que também relata que infecções e hospitalizações em
pessoas com idade entre 25 e 59 anos ___ (3) aumentam em uma taxa alarmante. “Demorou nove
meses para chegar a 1 milhão de mortes, quatro meses para chegar a 2 milhões e três meses para
chegar a 3 milhões”, disse o diretor geral da OMS, Tedros Adhanom Ghebreyesus, na semana
passada.

The panel, set up by the World Health Organization, said the combined response of the WHO and
global governments was a "toxic cocktail".
The WHO should have declared a global emergency earlier than it did, its report said, adding that
without urgent change the world was vulnerable to another major disease outbreak.
More than 3.3 million people around the world have now died of Covid.
While the US and Europe are beginning to ease restrictions and resume some aspects of pre-
pandemic life, the virus is still devastating parts of Asia.
India in particular is seeing record-breaking numbers of new cases and deaths, with severe oxygen
shortages in hospitals across the country.
Countries neighbouring India, such as Nepal, are also seeing surges of the virus.
What did the report say?
Covid-19: Make it the Last Pandemic, was compiled by the Independent Panel for Pandemic
Preparedness and Response.
Its aim was to find answers as to how the virus had killed more than 3.3 million people and infected
more than 159 million.

AULA 07 – LINKING WORDS (CONECTIVES) AND QUESTION WORDS (WH QUESTIONS) 143
TEACHER ANDREA BELO

"The situation we find ourselves in today could have been prevented," co-chair Ellen Johnson
Sirleaf, a former president of Liberia, told reporters.
"It is due to a myriad of failures, gaps and delays in preparedness and response."
The panel argued that the WHO's Emergency Committee should have declared the outbreak in
China an international emergency a week earlier than it did.
It should have done so at its first meeting on 22 January last year, the report said, instead of waiting
until 30 January.
The month following the WHO's declaration was "lost" as countries failed to take appropriate
measures to halt the spread of the virus.
The WHO was then hindered by its own regulations that travel restrictions should be a last resort,
the panel said, adding that Europe and the US wasted the entire month of February and acted
only when their hospitals began to fill up.
(Adapted from https://www.bbc.com/news/world-57085505)

O painel, organizado pela Organização Mundial da Saúde, disse que a resposta combinada da OMS
e dos governos globais foi um "coquetel tóxico".
A OMS deveria ter declarado uma emergência global mais cedo do que o fez, disse seu relatório,
acrescentando que, sem uma mudança urgente, o mundo estava vulnerável a outro grande surto
de doença.
Mais de 3,3 milhões de pessoas em todo o mundo já morreram de Covid.
Embora os Estados Unidos e a Europa estejam começando a reduzir as restrições e a retomar
alguns aspectos da vida pré-pandêmica, o vírus ainda está devastando partes da Ásia.
A Índia, em particular, está registrando um número recorde de novos casos e mortes, com grave
escassez de oxigênio em hospitais de todo o país.
Países vizinhos à Índia, como o Nepal, também estão observando surtos do vírus.
O que disse o relatório?
Covid-19: Make it the Last Pandemic, foi compilado pelo Independent Panel for Pandemic
Preparedness and Response.
Seu objetivo era encontrar respostas sobre como o vírus matou mais de 3,3 milhões de pessoas e
infectou mais de 159 milhões.
"A situação em que nos encontramos hoje poderia ter sido evitada", disse a repórteres a co-
presidente Ellen Johnson Sirleaf, ex-presidente da Libéria.
"É devido a uma miríade de falhas, lacunas e atrasos na preparação e resposta."
O painel argumentou que o Comitê de Emergência da OMS deveria ter declarado o surto na China
uma emergência internacional uma semana antes.
Deveria ter feito isso em sua primeira reunião em 22 de janeiro do ano passado, disse o relatório,
em vez de esperar até 30 de janeiro.

AULA 07 – LINKING WORDS (CONECTIVES) AND QUESTION WORDS (WH QUESTIONS) 144
TEACHER ANDREA BELO

O mês seguinte à declaração da OMS foi "perdido", pois os países não tomaram as medidas
adequadas para conter a propagação do vírus.
A OMS foi então impedida por seus próprios regulamentos de que as restrições às viagens
deveriam ser o último recurso, disse o painel, acrescentando que a Europa e os EUA
desperdiçaram todo o mês de fevereiro e agiram apenas quando seus hospitais começaram a
encher.

Teens Aged 12 to 15 Can Now Get Pfizer-BioNTech COVID-19 Vaccine in the U.S.
___ (1) May 10, the U.S. Food and Drug Administration (FDA) extended authorization of the Pfizer-
BioNTech COVID-19 vaccine to 12- to 15-year-olds. It’s the first COVID-19 shot authorized for this
younger population.
“I cannot tell you how many people have been anxiously awaiting this day to get their kids
vaccinated,” says Dr. Flor Munoz, a pediatric infectious disease specialist ___ (2) Texas Children’s
Hospital and Baylor College of Medicine, and a member of the infectious disease committee of
the American Academy of Pediatrics.
The green light was based on review of data the two companies released in March showing that
two doses of their vaccine provided similar protection from COVID-19 disease among this age
group as it did for adults. Among the more than 2,200 teens in the study group the FDA reviewed,
16 developed COVID-19—none of them were in the vaccinated group.
The companies’ studies found that levels of virus-fighting antibodies were on average higher
among the 12- to 15-year-olds that got the vaccine than among vaccinated 16- to 25-year-olds in
previous studies.
“This is part of the totality of getting our country protected against COVID-19, which is just waiting
around ___ (3) corner to come have another wave if we don’t get a sufficient degree of
vaccination,” said Dr. Peter Marks, director of the Center for Biologics Evaluation and Research at
the FDA, during a briefing discussing the decision.
The authorization was based on a so-called “bridging” study, in which researchers used data from
the previous study in adults to set thresholds for safety and efficacy. Building off of the success of
the adult studies enabled the scientists to enroll some 2,000 12- to 15-year-olds, rather than the
tens of thousands of adults that the earlier studies required.
(Adapted from https://time.com/6047384/teens-pfizer-covid-vaccine/)

Adolescentes de 12 a 15 anos agora podem receber a vacina Pfizer-BioNTech COVID-19 nos EUA.
___ (1) Em 10 de maio, a Food and Drug Administration (FDA) dos EUA estendeu a autorização da
vacina Pfizer-BioNTech COVID-19 para crianças de 12 a 15 anos. É a primeira injeção COVID-19
autorizada para esta população mais jovem.
“Eu não posso dizer quantas pessoas estão esperando ansiosamente por este dia para vacinar seus
filhos”, disse a Dra. Flor Munoz, uma especialista em doenças infecciosas pediátricas ___ (2) Texas

AULA 07 – LINKING WORDS (CONECTIVES) AND QUESTION WORDS (WH QUESTIONS) 145
TEACHER ANDREA BELO

Children's Hospital e Baylor College of Medicine, e membro do comitê de doenças infecciosas da


Academia Americana de Pediatria.
A luz verde foi baseada na revisão de dados que as duas empresas divulgaram em março,
mostrando que duas doses de sua vacina forneciam proteção semelhante contra a doença COVID-
19 nessa faixa etária, assim como nos adultos. Entre os mais de 2.200 adolescentes no grupo de
estudo que o FDA revisou, 16 desenvolveram COVID-19 - nenhum deles estava no grupo vacinado.
Os estudos das empresas descobriram que os níveis de anticorpos anti-vírus eram, em média,
maiores entre os jovens de 12 a 15 anos que receberam a vacina do que entre os vacinados de 16
a 25 anos em estudos anteriores.
“Isso é parte da totalidade de proteger nosso país contra COVID-19, que está apenas esperando
na esquina ___ (3) para vir ter outra onda se não conseguirmos um grau suficiente de vacinação”,
disse o Dr. Peter Marks , diretor do Centro de Avaliação e Pesquisa Biológica do FDA, durante um
briefing discutindo a decisão.
A autorização baseou-se no chamado estudo “ponte”, no qual os pesquisadores usaram dados do
estudo anterior em adultos para definir limites de segurança e eficácia. A construção do sucesso
dos estudos de adultos permitiu aos cientistas inscrever cerca de 2.000 jovens de 12 a 15 anos,
em vez das dezenas de milhares de adultos que os estudos anteriores exigiam.

AULA 07 – LINKING WORDS (CONECTIVES) AND QUESTION WORDS (WH QUESTIONS) 146
AFA 2024

ABBREVIATIONS, CONTRACTIONS
AND IF CLAUSES

AULA 08

Teacher Andrea Belo

www.estrategiamilitares.com.br www.militares.estrategia.com
TEACHER ANDREA BELO

SUMÁRIO
INTRODUÇÃO 3

ABBREVIATIONS 4

CONTRACTIONS 13

IF CLAUSES 20

QUESTÕES 23

GABARITO 55

QUESTÕES COMENTADAS 56

CONSIDERAÇÕES FINAIS 117

REFERÊNCIAS BIBLIOGRÁFICAS 118

TRADUÇÕES 120

AULA 08 – ABBREVIATIONS, CONTRACTIONS AND IF CLAUSES 2


TEACHER ANDREA BELO

INTRODUÇÃO
Teremos, agora, uma aula inteira para falar de abreviações presentes nos textos das
provas e que, muitas vezes, podem gerar conflitos de significado, tanto pela similaridade com
certas palavras, quanto pela complexidade de certas abreviações.
O que é uma abreviatura? Um recurso da língua escrita usado para representar, de forma
reduzida, uma palavra, uma frase ou até mesmo uma expressão. As abreviações são
simplesmente formas diretas de diminuir as palavras a fim de facilitar e tornar a comunicação
mais ágil.
As orações condicionais (if clauses) são utilizadas para falar de planos futuros, de situações
prováveis ou improváveis e de escolhas que foram feitas no passado e possuem o nome if clauses
porque todas têm if em sua estrutura como mostrarei a você na aula.
Diante das provas de diferentes bancas que elaboram as provas, faz-se necessário o uso de
uma ortografia dentro da norma padrão da língua, em que há regras que devem ser respeitadas.
Do latim, tem como raiz o termo "breve", que significa “de pouca extensão”, “de pouca
duração”. Considerando o fator economia vocabular, além da escrita, a tendência hoje é que se
fale cada vez mais abreviaturas.
E essa tendência se reflete por meio das mudanças linguísticas e as inúmeras abreviações
estão dentro de um contexto, em que a necessidade de uma comunicação rápida se processa com
uma dimensão inimaginável.
Por um lado, a abreviação facilita a comunicação em relação à quantidade de informação.
Claro que encontramos problemas de comunicação por causa das abreviações.
A sensibilidade para escrever as palavras seguindo regras gramaticais é essencial no dia da
sua prova mas, em alguns gêneros textuais, elas aparecem – em tirinhas, textos com linguagem
informal, charges, às vezes em letras de música, poemas ou até em um texto qualquer, que tenha
a intenção de explorar as abreviações.
Abreviar, portanto, é inevitavelmente uma tendência da fala e da escrita.
Vamos então estudar as particularidades de abreviações e das If Clauses em Inglês, praticar
exercícios, tanto durante a teoria como também na lista de questões, treinando tudo o que
aprendeu, em junção ao aprendizado de cada aula.
Vamos lá e conte comigo!

AULA 08 – ABBREVIATIONS, CONTRACTIONS AND IF CLAUSES 3


TEACHER ANDREA BELO

ABBREVIATIONS
Considerando que abbreviations – as abreviações, como analisamos anteriormente, são,
de certa forma, parte fundamental e funcional da fala, o contexto social em que elas estão nos
faz acreditar que novas palavras podem ser formadas em um futuro bem próximo.
Isso porque o condicionamento da escrita depende da sua essência de sentido, que é, de
fato, a fala. Apesar de ser considerado informal um texto repleto de abreviações, eles são
inevitáveis. A polêmica sobre o uso de abreviações persiste, mas, a realidade é que, abreviar é
uma “arte da fala”, que acaba por ser respeitada pela escrita, que a utiliza quando convém.
As abreviaturas demonstram o ritmo acelerado do hoje, da atualidade e que faz com que
haja economia de palavras para realizar a comunicação em um tempo menor, em qualquer língua.
É comprovado que a linguagem nunca foi tão abreviada como atualmente. A revolução eletrônica
fez e faz com que as pessoas sintam a necessidade de abreviar a escrita para ganhar tempo nos
diferentes diálogos do mundo cibernético, principalmente em Inglês.
Mas, quem pensa que as abreviaturas do nosso cotidiano surgiram apenas por causa dos
recursos tecnológicos, está enganado. É inegável que elas se acentuaram após a globalização,
porém, quando ainda éramos colônia de Portugal, algumas abreviações já eram manifestadas.
Resultante da braquigrafia, originária do grego braqui = reduzir e grafhein = escrever, na
época do Brasil-Colônia, o uso de abreviaturas nos documentos era uma constante. Entre os
fatores que influenciavam na proliferação desta prática estava a distância entre Brasil e Portugal.
Com a vinda da Família Real portuguesa para o Brasil, no século XVIII, é que se iniciou o uso
de documentos impressos, mas, mesmo assim as abreviaturas ainda estavam presentes devido
aos altos preços dos materiais utilizados para impressão.
Contextualizando-nos nos dias de hoje, percebemos que muito tempo se passou, contudo,
as abreviaturas continuam sendo usadas e, de modo ainda mais intenso. Entretanto, abreviar
causa uma influência direta no desempenho linguístico, já que nosso sistema é regido por um
padrão formal da linguagem, apesar de que as abreviaturas nunca deixarão de existir.
Vivemos em uma sociedade dinâmica, e então, cabe a nós, adequarmos em tal dinamismo.
Então, muito mais do que reduzir palavras, devemos fazer considerações sobre as diversas
situações comunicativas.
E eu não estou falando de gírias ou expressões do dia a dia e sim abreviações aceitas, como
por exemplo, falamos “foto” no lugar de fotografia. O americano diz “photo” ao invés de
“photograph” também entre outras.
Apesar da não aceitação dessa tendência por parte de muitas pessoas, os que acreditam
que as abreviações empobrecem a escrita, não há como deixar de perceber que as redes sociais
proporcionam o compartilhamento de diferentes interesses com a exploração de abreviações e,
palavras em Inglês, por via de regra, prevalecem.
Apesar de algumas abreviações não serem reconhecidas pela norma culta, existem, sim,
algumas regras que regem as abreviaturas e siglas.
A abreviatura é, conceitualmente, uma grafia que permite economizar espaço ou tempo
necessários para a escrita de uma palavra, mediante a omissão de certas letras.

AULA 08 – ABBREVIATIONS, CONTRACTIONS AND IF CLAUSES 4


TEACHER ANDREA BELO

Um exemplo típico é “dr”, para doctor ou “st” para street. Ou a expressão “o mais
rápido/breve possível” – em Inglês “as soon as possible”, reduzido para asap e, muito utilizada,
não só na linguagem oral mas, em mensagens e às vezes, em textos mesmo.
Elas são reduções de parte de uma palavra escrita que a resume por meio, na maioria das
vezes, de suas letras iniciais – ou sílabas iniciais, mas não é regra para todas as abreviaturas que
existem.
É importante ressaltar que, em alguns casos, as abreviações não obedecem a nenhuma
regra em particular.
E ainda há os acrônimos, também chamados de siglas e, apesar de serem um tipo de
abreviação, são verdadeiras palavras formadas pela junção de sílabas iniciais de vocábulos em
Inglês ou em qualquer outra língua.
São tipos de abreviaturas utilizadas para reduzir o nome de alguma associação, empresa,
instituição, país, organização e afins.
Veja alguns dos inúmeros exemplos que existem, muitos comuns e algumas pessoas nem
sabem que são abreviações de outras palavras:
“jpeg” – joint photographic experts group
“htpp” – hyper text transfer protocol
“NASA” – National Aeronautics and Space Administration
“scuba” – self-contained underwater breathing apparatus
“gif” – graphics interchange format
“laser” – light amplification by stimulated emission of radiation
“radar” – radio detection and ranging
“CD rom” – compact disk read only memory
“HIV” – human immunodeficiency virus
“pin number” – personal identification number
“S.O.S” – Save our souls
“U.S.A” – United States of America
“ZIP code” – zone improvement plan

Veja também, outras abreviações interessantes e curiosas que podem aparecer, por
exemplo, no meio de um texto, demonstrando uma fala informal de um personagem a que a
história remete, entre outras funções, que podem ser exploradas nas provas.

AULA 08 – ABBREVIATIONS, CONTRACTIONS AND IF CLAUSES 5


TEACHER ANDREA BELO

2F4U Too Fast For You N/A Not Available/Applicable


4YEO FYEO For Your Eyes Only NNTR No Need To Reply
AAMOF As A Matter Of Fact NRN No Reply Necessary
ACK Acknowledgment OMG Oh My God
AKA Also Known As OP Original Poster, Original Post
BTT Back To Topic OTOH On The Other Hand
BTW By The Way ROTFL Rolling On The Floor Laughing
B/C Because SFLR Sorry, For Late Reply
C&P Copy And Paste SPOC Single Point Of Contact
CYS Check Your Settings TBA To Be Announced
EOD End Od Discussion TBC To Be Continued / To Be Confirmed
EOM End Of Message TGIF Thanks God, Its Fridat
FAQ Frequently Asked Questions THX / TNX Thanks
FACK Full Acknowledge TYT Take Your Time
FKA Formerly Known As TTYL Talk To You Later
HTH Hope This Helps WRT With Regard To
IOW In Other Words YMMD You Made My Dad
LOL Laughing Out Loud YAM Yet Another Meeting
DGMW Don't Get Me Wrong ICYMI In Case You Missed It
MMW Mark My Words

Algumas são diferentes, não são? Fica como curiosidade, caso algum desses termos esteja
inserido em um contexto para testar seus conhecimentos da variedade linguística.
Vejamos, a partir de agora, algumas abreviações muito usadas, divididas por assunto, para
ficar mais bem exemplificado e para, é claro, remeter a exemplos que aparecem nas provas.

ACADEMIC TERMS
Já falamos acima que, hoje, com o ritmo acelerado da comunicação, as pessoas utilizam
muitas abreviações na fala e escrita.
E aparecem em textos, também, em diferentes estilos textuais, charges, artigos, jornais,
entre outros que veremos exemplos.
Economizar na fala e na escrita passou a ser uma tendência comum e, as abreviaturas
podem não possuem regras, como também pontuamos anteriormente.
Há, em alguns textos, termos acadêmicos (academic terms), que são importantes de se
conhecer para que você leia o texto melhor e compreenda com maior facilidade.
Muitas vezes, as abreviações são apresentadas ao final do texto mas, já aconteceu de não
haver as devidas referências, porque algumas fontes como The Economist e The Guardian utilizam
certos vocábulos com frequência e já fazem parte dos artigos, de uma forma geral.

AULA 08 – ABBREVIATIONS, CONTRACTIONS AND IF CLAUSES 6


TEACHER ANDREA BELO

Nesse caso, é bom que você conheça, pelo menos, as abreviações mais recorrentes e
básicas para estar mais bem preparado. Segue uma lista das palavras que são mais comuns em
textos da sua prova, porque são assuntos geralmente explorados.
ADHD – Attention Deficit Hyperactive Condition
AUG – Acceptable Use Guideline
BLT – Building Leadership Teams
CEO – Chief Executive Officer
CTE – Career Related Experiences
CPR – Cardiopulmonary Resuscitation
DBQ – Document Based Question
EL – Extended Learning
FLIP – Families Learning in Partnership
LD – Learning Disability
MD – Managing Director
OT – Occupational Therapy
PA – Personal Assistant
SVP – Senior Vice President
T&L – Teaching and Learning Department
VP – Vice President
YM – Youth Mentors

EVERY DAY ABBREVIATION


Para exemplificar outras abreviações que também são usadas na prova, vejamos uma lista abaixo,
inclusive porque são exemplos retirados de provas anteriores e selecionados para nossa aula,
considerados “every day use” (uso no dia a dia).
abbrev. – abbreviation (abreviação)
abstr. – abstract (resumo de tese/trabalho)
adj. – adjective (adjetivo)
Admin. – administration, administrative (administração, administrativo)
adv. – adverb (advérbio)

AULA 08 – ABBREVIATIONS, CONTRACTIONS AND IF CLAUSES 7


TEACHER ANDREA BELO

adv. – advanced, adventure, advices (avançado/aventura/conselhos)


agst. – against (contra)
alg. – algebra (algebra)
alph. – alphabet (alfabeto)
app. Apêndix (apêndice)
approx. - approximately (aproximadamente)
appt. – appointment (compromisso)
apt. – apartment (apartamento)
betw. – between (entre duas coisas)
c/o – care of (usado ao enviar e-mail para alguém que não está no
endereço habitual).
dept. – department (departamento)
min. – minute/ minimum (minute/mínimo)
misc. – miscellaneous (diversos)
Mr. – Mister (senhor)
Mrs. – Mistress (senhora)
no. – number (número)
R.S.V.P. – Répondez, s'il vous plait (termo que vem do vocabulário
Francês para "por favor responda" e é usado em convites para festas e
eventos e deve ser respondido com um "sim, participaremos" ou "não,
não participaremos"
tel. – telefone (telefone)
temp. – temperature or temporary (temperatura ou temporário)
vet. – veteran or veterinarian (veterano ou veterinário)
vs. – versus (versus)

AULA 08 – ABBREVIATIONS, CONTRACTIONS AND IF CLAUSES 8


TEACHER ANDREA BELO

SOCIAL MEDIA
Outras abreviações que também estão presentes em textos típicos da prova, são “social media”,
termos informais, usados em “chats” e, acima de tudo, usados hoje em mensagens, usados para
expressar a comunicação “moderna”, uma forma de evolução, em que há novos termos e suas
devidas abreviações. Vejamos os mais recorrentes em textos:
AMBW – all my best wishes
AAMOF – as a matter of fact
ACE – a cool experience
AYS – Are you serious?
AFAIK – as far as I know
AFK – away from keyboard
ATM – at the moment
BRB – be right back
CUL – see you later
CWYL – chat with you later
FAWC – for anyone who cares
IDC – I don’t care
IIRC – if I recall/remember correctly
IBRB – I’ll be right back
IMU – I miss you
IRL – in real life
J4F – just for fun
JIC – just in case
JK – just kidding
JSYK – just so you know
LOL – laugh out loud
NBD – Not big deal
NP – no problem
NSFW – Not safe for work
POV – Point of View
RBTL – Read between the lines

AULA 08 – ABBREVIATIONS, CONTRACTIONS AND IF CLAUSES 9


TEACHER ANDREA BELO

RUOK – Are you OK?


ROFL – rolling on the floor laughing
4AO – For adults only
OP – Original poster
OIB – Oh, I'm back
PTB – Please text back
SSDD – Same stuff, different day
SRSLY – Seriously
TY – thank you
TTYL – Talk to you later
TNTL – Trying not to laugh
TL; DR – Too long; didn’t read
VN – Very nice
WYWH – Wish you were here
WTG – Way to go

MAP LOCATION
Há, também, algumas abreviações que aparecem em textos relacionadas a algum lugar, endereço,
direções, ou seja, remetendo a localizações – “map location”. Vejamos algumas.
Ave. – Avenue
Blvd. – Boulevard
BRG – Bridge
Cyn. – Canyon
DRCTN – Direction
Dr. – Drive
Ln. – Lane
Rd. – Road
St. – Street
E – East
N – North
NE – Northeast

AULA 08 – ABBREVIATIONS, CONTRACTIONS AND IF CLAUSES 10


TEACHER ANDREA BELO

NW – Northwest
S – South
SE – Southeast
SW – Southwest
W – West
XRD – crossroad

Agora, a própria palavra “map”, pode ser sigla de inúmeras palavras, de diferentes
categorias. Essas, quando aparecem nos textos, geralmente vem com a referência do que seja no
rodapé ou durante a leitura mesmo. Mostrarei algumas por curiosidade.
MAP Messaging Application Protocol MAP Memory and Place (project Australia)
MAP Magnetic Acoustic-Pressure MAP Mesh Access Point
MAP Magnetically Accelerated Projectile (Star Wars) MAP Message Access Protocol
MAP Main Audio Program MAP Message Application Part (Spent)
MAP Mainframe Acquisition Project MAP Message Application Programming
MAP Maintenance & Administration Panel MAP Microprocessor Applications Project
MAP Maintenance Activation Plan MAP Michigan Advocacy Project
MAP Maintenance Activity Pirmasens MAP Microsoft Assessment and Planning Toolkit
MAP Maintenance Analysis Procedure MAP Microwave Anisotropy Probe (NASA)
MAP Maintenance and Administrative Position (Nolen) MAP Midrange Alliance Program
MAP Major Accounts Processing MAP Military Aeronautical Pentathlon
MAP Management Accounting Principles MAP Military Assistance Program (US)
MAP Management Achievement Plan MAP Milstar Advanced Processor
MAP Management Assistance Program MAP Minnesota AIDS Project
MAP Management Process MAP Mitogen-Activated Protein
MAP Manifold Absolute Pressure MAP Mitsubishi Assistance Package
MAP Manufacturing Assembly Procedure MAP Mobile Application Part
MAP Manufacturing Automated Protocol MAP Mobile Application Part (GSM)
MAP Market Access Program MAP Modified American Plan (hospitality industry)
MAP Marketing and Publications MAP Modified Atmosphere Packaging
MAP Materiel Acquisition Plan MAP Modular Architecture Platform
MAP Math and Physics MAP Modular Arithmetic Coprocessor
MAP Maximum Performance (various organizations) MAP Modular Avionics Package
MAP Mazda Advancement Plan MAP Moisture, Ash and Protein
MAP Mean Annual Precipitation MAP Morbidly and Performance Assessments
MAP Mean Arterial Pressure MAP Mortgagor Assistance Program (mortgage industry CAM)
MAP Measurement Adoption Process MAP Move Assistance Program
MAP Media Access Procedure MAP Multicultural Achievers Program
MAP Media Access Project MAP Multimodal Application Platform (LiteScape)
MAP Media Awareness Project MAP Multiple Aim Point System
MAP Medical Assistance Program MAP Multiplexed Access Point
MAP Mediterranean Action Plan MAP Multiservice Access Platform
MAP Membership Action Plan (NATO) MAP Mutualized Access Point
MAP Memory Address Print

AULA 08 – ABBREVIATIONS, CONTRACTIONS AND IF CLAUSES 11


TEACHER ANDREA BELO

Agora, vamos às abreviações quando há termos relacionados à cozinha, uma receita na


prova, algo que remeta à “cooking vocabulary”.

COOKING AND BAKING VOCABULARY


Como eu disse anteriormente, pode aparecer, em sua prova, um texto retirado de uma das
fontes mais comuns em provas e, conter parte de uma receita ou algum assunto relacionado à
culinária, a chefs de cozinha, entre outros.
Se aparecer algum desses termos, mesmo que haja legenda, é bom você já ter visto pois,
como eu sempre digo, poupa seu tempo, que é valioso no dia da prova. Veja alguns mais comuns:
doz – dozen = 12 pieces
tsp or t – teaspoon/teaspoons
tbs, tbsp or T – tablespoon/tablespoons
c – cup/cups
gm – gram
mg – milligram
fg – few grains
gal – gallon
lb – pound/pounds
pt – pint = 568ml
qt – quart = 2 pints
min – minutes
hr – hour
b.p. – baking powder

Agora vamos estudar algumas contrações típicas em provas, tendo em vista palavras que
normalmente aceitam abreviações em forma de contração, sem perder o sentido ou desencadear
informalidade.
Muitas vezes, não são usadas mas, a depender do texto, podem estar sim, presentes.
Vamos lá.
Let’s go!

AULA 08 – ABBREVIATIONS, CONTRACTIONS AND IF CLAUSES 12


TEACHER ANDREA BELO

CONTRACTIONS
Contractions – as contrações em Inglês – são termos que, muitas vezes, deixam as pessoas
em dúvida porque, apesar de ser um recurso considerado facilitador da comunicação, pode gerar
confusão na hora da leitura e, é claro, na comunicação em geral.
Uma contração pode ser definida como uma forma reduzida de uma palavra ou uma
combinação de palavras.
As contrações são utilizadas quando não queremos usar a forma completa de determinado
termo ou, veremos que podem aparecer com outros propósitos também.
As contrações são formas encurtadas de certas palavras, com a omissão de algumas letras.
E, essas letras que faltam, são automaticamente substituídas por um apóstrofo.
Nas provas de Inglês, as contrações podem ser divididas em dois grupos: as mais comuns
e utilizadas com mais frequência, aceitas em textos formais e informais ou aquelas que fazem
parte do cotidiano mas podem aparecer no meio de um texto para expressar uma opinião, podem
estar presentes em charges ou qualquer outro tipo de leitura.
Uma das intenções pelas quais essas abreviações aparecem é mostrar de fato a abreviação
e depois fazer perguntas sobre a formação da palavra abreviada que exijam seu conhecimento.
Quando um falante da língua inglesa se expressa (seja um americano ou um britânico ou
de qualquer lugar em que se fala Inglês como língua oficial), é típico que essas pessoas, já
familiarizadas com sua língua materna, falem um pouco mais rápido e usem abreviações e
contrações, assim como nós, brasileiros, usamos algumas também quando falamos e escrevemos.
As contrações são usadas, na maioria das vezes, apenas em diálogos informais e,
normalmente não estão presentes em textos escritos mas os textos usados nas provas podem
explorar qualquer estilo textual e pedir, inclusive, para que você faça a verificação de frases com
contrações.
Mas, algumas contrações são usadas com outro propósito, que não é a informalidade ou
gíria mas, por sua vez, utilizadas por causa de regras específicas pelo que representam, como
veremos adiante.
As provas trazem, em suas questões, textos longos e por isso, suscetíveis a demonstrar
palavras com diferentes tipos de contrações, que, depois de observadas por você, podem ser
exigidos questionamentos sobre tais contractions.
Como já falei anteriormente, as abreviações acontecem quando, basicamente, há uma
omissão de algumas letras e a adição do apóstrofo para sinalizar e mostrar que ali há uma palavra
contraída. Ou, para mostrar posse, como estudaremos, entre outras funções.
Uma vez que você já conhece muitos termos com diversas contrações, já ficará mais seguro
se precisar se a pergunta envolver alguma informação que você já leu em nosso material e ajudará
você a responder com maior segurança, nosso objetivo de aula após aula.
Vejamos as contrações mais comuns em provas.

AULA 08 – ABBREVIATIONS, CONTRACTIONS AND IF CLAUSES 13


TEACHER ANDREA BELO

CONTRACTIONS WITH LETTER “S”


Quando pensamos em contractions, a primeira coisa que vem à nossa mente são palavras
que, possuem contrações e apóstrofos em sua composição porque não estão sendo usadas de
acordo com as regras que as regem.
E, ainda se imagina que, palavras com apóstrofos, estão abreviadas porque estão sendo
usadas informalmente mas, não é sempre assim.
No caso de palavras com contrações que utilizam o apóstrofo e a letra “s”, pode ser que
estejamos diante de um caso de posse, ou seja, demonstrando que algo pertence a alguém. Veja:

Roger has a computer. (Roger tem um computador).


Roger’s computer is modern. (O computador de Roger é moderno).

Se você quiser usar a preposição “of” e dizer: “The computer of Roger is modern”, que
também poderia ser traduzido como “O computador de Roger é moderno”, apesar de não ser
considerado totalmente errado, não é usual. Não se vê essa construção no Inglês oral nem escrito,
tampouco em textos da sua prova.
Isso porque, como eu disse antes, o apóstrofo em união à letra “s” no final de substantivos,
tem a função de posse em relação ao sujeito da frase.
Outro caso em que usamos essa estrutura é quando queremos nos referir a um grupo ou
uma família, por exemplo, dizer que a casa de praia é da família Smith – “This is the Smith’s beach
house (Essa é a casa de praia dos “Smith”). Veja outro exemplo:

This is the Hangton’s dog. (Este é o cachorro dos Hangton – dessa família).

É importante lembrar que, palavras já terminadas em “s”, quando vamos demonstrar posse, usa-
se apenas o apóstrofo, sem a letra “s”, veja:

No reply is my favorite Beatles’s song. (No reply é a minha música dos Beatles favorita).

Como o nome da banda é The Beatles e já possui a letra “s” no fim da palavra, apenas se
acrescenta o apóstrofo e a condição de posse foi estabelecida – seguindo regras. Por isso, cuidado
se, na prova houver alternativas dizendo que as contrações são formas de abreviar sempre com
a intenção de diminuir palavras ou poupar tempo.
Vimos que, algumas vezes, a estrutura com “‘s” são exemplificações de frases que indicam
posse. E o apóstrofo, junto à letra “s”, também pode ser o verbo to be, como veremos agora.

AULA 08 – ABBREVIATIONS, CONTRACTIONS AND IF CLAUSES 14


TEACHER ANDREA BELO

CONTRACTION: VERB TO BE
Já estudamos o verbo to be na aula sobre tempos verbais. E vimos as possíveis contrações
mas, não falamos das particularidades que possam aparecer nas provas em relação à letra “s”,
quando você precisa definir se é verbo to be ou verbo have, ambos na terceira pessoa, usados
com apóstrofo e a letra s.
Primeiro, vamos relembrar a conjugação com as devidas abreviações no presente e
passado:
PRESENT – AFIRMATIVO PRESENTE – NEGATIVO PASSADO – NEGATIVO
I am = I'm I am not = I'm not I was not = I wasn't
You are = You're You are not = You aren't You were not = You weren't
He/She/It is = He/She/It's He/She/It is not = He/She/It isn't He was not = wasn't
We are = We're We are not = We aren't We were not = We weren't
You are = You're You are not = You aren't You were not = You weren't
They are = They're They are not = They aren't They were not = They weren't

Observe que, para os sujeitos He, She e It, no presente, foi mostrada a forma da contração
apenas representada, como eu disse, pelo apóstrofo e a letra “s”. Mas, ao falar “Ela é” ou “Ela
tem”, ambas contrações são iguais, veja:

HE IS = HE’S
HE HAS = HE’S
HE IS INTERESTING (Ele é interessante) – A contração fica: HE’S INTERESTING.
HE HAS A BIKE. (Ele tem uma bicicleta) – A contração fica: HE’S INTERESTING.

Como saber se, no texto, a contração usada se refere ao verbo to be ou ao verbo to have?
Como todas as palavras iguais ou similares, não só em Inglês mas em qualquer língua, temos que
analisar o contexto.
Quando se trata de uma qualidade ou um estado (cansado, satisfeito, doente etc.)
geralmente a contração é verbo to be mas, quando há descrições ou remete a algo que o sujeito
possua, é o verbo to have. Veremos nos textos ainda nessa aula.
Agora, vejamos outras contrações comuns em provas.

CONTRACTION: DO AND DID


Também já estudamos os auxiliares do e did na aula sobre tempos verbais. Estão aqui
devidamente citados para relembrar a estrutura de cada um deles. Mas, há uma contração que
pode aparecer na prova, que é formada pela negação do don’t.
Primeiro, vamos relembrar a conjugação com as devidas abreviações:

AULA 08 – ABBREVIATIONS, CONTRACTIONS AND IF CLAUSES 15


TEACHER ANDREA BELO

DO + NOT DID + NOT


I do not = I don't I did not = I didn't
You do not = You don't You did not = You didn't
He/She/It does not = He/She/It doesn't He/She/It did not = He/She/It didn't
We do not = We don't We did not = We didn't
You do not = You don't You did not = You didn't
They do not = They don't They did not = They didn't

O auxiliar do, usado na negativa e formando don’t, na frase “Eu não sei” é “I don’t know”.
Porém, pode aparecer abreviado em alguns textos e assim, ao invés de “I don’t know”, você pode
encontrar “I dunno”, uma contração e ao mesmo tempo abreviação do auxiliar na forma negativa
e o verbo saber (to know), formando “dunno”. #gettheteachersclue (#ficaadicadateacher)

CONTRACTION: WILL
O auxiliar will, também já estudado na aula de tempos verbais, como uma das formas de
se demonstrar o que ainda vai acontecer, possui as seguintes formas com contrações:
WILL WILL + NOT
I will = I'll I will not = I won't
You will = You'll You will not = You won't
He/She/It will = He/She/It'll He/She/It will not = He/She/It won't
We will = We'll We will not = We won't
You will = You'll You will not = You won't
They will = They'll They will not = They won't

A dica aqui é apenas prestar atenção para não confundir palavras terminadas em “ll” (bell,
well, doll, bull, entre outras) com palavras abreviadas e que usam o apóstrofo e duas letras “s”
(He’ll.., she’ll..., they’ll...) certo?

CONTRACTION: VERB TO HAVE IN THE PRESENT


Em relação ao verbo ter (to have), devemos ficar atentos sobre as contrações com
apóstrofo, pois, nas terceiras pessoas do singular (he, she, it), a estrutura ficará igual às contrações
do verbo to be, apenas com a letra “s”.
Para não confundir você, primeiramente, tente substituir a contração pelo verbo ter, para
ver se faz sentido.
Em seguida, observe se há um verbo no particípio após o apóstrofo com “s” pois, como
estudamos nos tempos verbais, se, logo após o “has”, conjugado para as terceiras pessoas, houver
um verbo no particípio, é a estrutura convencional de Present Perfect.

AULA 08 – ABBREVIATIONS, CONTRACTIONS AND IF CLAUSES 16


TEACHER ANDREA BELO

E, analisando dessa forma, confirma-se que a contração encontrada é o apóstrofo com s


(‘s) de “has” e não de “is”, certo? Veja as contrações para todos os sujeitos e exemplos:
HAVE HAVE + NOT
I have = I've I have not = I haven't
You have = You've You have not = You haven't
He/She/It has = He/She/It's He/She/It have not = He/She/It hasn't
We have = We've We have not = We haven't
You have = You've You have not = You haven't
They have = They've They have not = They haven't

She’s studied a lot = She has studied a lot. (Ela tem estudado muito/Ela estuda muito)
She’s a good student = She is a good student. (Ela é uma boa aluna.)

Assim como o verbo to have no presente pode gerar confusão em relação à abreviação do
verbo to be quando é apóstrofo e a letra s, o verbo to have no passado também gera dúvidas em
relação a um auxiliar que já estudamos.
Vejamos agora para deixar claras as diferenças.

CONTRACTION: VERB TO HAVE IN THE PAST


Em relação ao verbo ter (to have) conjugado no passado, devemos ficar atentos sobre as
contrações com apóstrofo e a letra “d”, pois a estrutura ficará igual às frases em que se utilizam
contrações do auxiliar would, apenas com a letra “d”.
Para não confundir você, primeiramente, tente substituir a contração pelo verbo ter, para
ver faz sentido (teve, tinha, tinham, tivesse, tivessem, entre outros).
Em seguida, observe se há um verbo no particípio após o apóstrofo com “d” pois, como
estudamos, se, logo após o “had”, houver um verbo no particípio, é a estrutura convencional de
Past Perfect.
E, analisando dessa forma, se o próximo verbo não estiver no particípio e sim no infinitivo
(forma original), a contração é would (‘d). Veja as contrações para todos os sujeitos e exemplos:
HAD WOULD
I had = I'd I would = I'd
You had = You'd You would = You'd
He/She/It had = He/She/It'd He/She/It would = He/She/It'd
We had = We'd We would = We'd
You had = You'd You would = You'd
They had = They'd They would = They'd

AULA 08 – ABBREVIATIONS, CONTRACTIONS AND IF CLAUSES 17


TEACHER ANDREA BELO

She’d studied a lot = She had studied a lot. (Ela tinha estudado muito/Ela estudou muito)
She’d study if she had time = She would study if she had time. (Ela estudaria se tivesse tempo.)

Agora, vejamos contrações com os verbos modais.

CONTRACTION: MODAL VERBS


Os verbos modais, quando usados com as contrações, em suas formas negativas, ficam
conforme o quadro abaixo.
MODAL CAN MODAL COULD MODAL SHOULD
I can not = I can't I could not = I couldn't I should not = I shouldn't
You can not = You can't You could not = You couldn't You should not = You shouldn't
He/She/It can not = He/She/It can't He could not = He/She/It couldn't He/She/It should not = He/She/It shouldn't
We can not = We can't We could not = We couldn't We should not = We shouldn't
You can not = You can't You could not = You couldn't You should not = You shouldn't
They can not = They can't They could not = They couldn't They should not = You shouldn't

Geralmente, não há confusões em relação às contrações no que diz respeito aos verbos
modais e sim, são fáceis de serem identificadas no dia da prova, pois, muitas vezes, ao invés de
aparecer as formas abreviadas, é mais comum, em textos, aparecer o “not” junto ao verbo modal.
E, no caso do “can”, o not vem “colado” palavra com palavra, veja exemplos de alguns
modais mais comuns em textos:

She cannot go = She can’t go. (Ela não pode/não consegue ir)
She should not go = She can’t go. (Ela não deveria ir)
She must not go = She can’t go. (Ela não deve ir)

Agora, vejamos um exemplo de contraction que não é muito comum mas pode aparecer
em diálogos informais ou charges ou como demonstração de frases informais.

CONTRACTION: AIN’T
A contração “ain’t” pode significar muitas palavras negativas.
É um termo que pode substituir e representar as contrações am not, is not, are not, don’t,
doesn’t, didn’t, there isn’t, there aren’t, has not e have not. Veja exemplos:

AULA 08 – ABBREVIATIONS, CONTRACTIONS AND IF CLAUSES 18


TEACHER ANDREA BELO

I ain’t working now = I’m not working now. (Eu não estou trabalhando agora).

He ain’t worried about it = He isn’t worried about it. (Ele não está preocupado com isso).

She ain’t good at Geography = She isn’t good at Geography. (Ela não é boa em Geografia).

They ain’t the best students = They aren’t the best students. (Eles não são os melhores alunos).

I ain’t got a car = I don’t have a car. (Eu não tenho um carro).

I ain’t know that = I didn’t know that. (Eu não sabia disso).

You ain’t seen everything! = You have not seen everything! (Você não viu tudo!)

Ain’t no one like you= There isn’t no one like you. (Não há ninguém como você).

Ain’t no friends like you, guys. = There aren’t friends like you, guys. (Eu não tenho um carro).

He ain’t heard anything = He hasn’t heard anything. (Ele não ouviu nada).

They ain’t written the emails = They haven’t written the emails. (Eles não escreveram os emails).

Essa forma “ain’t” de negação não é comum nos textos das provas mas, como eu disse
sobre todas as abreviações e contrações que usamos, podem aparecer palavras em tirinhas,
charges ou no meio de um texto, para fazer algum tipo de referência.
Agora, vamos estudar as IF CLAUSES, frases condicionais. Let’s go!!!!

AULA 08 – ABBREVIATIONS, CONTRACTIONS AND IF CLAUSES 19


TEACHER ANDREA BELO

IF CLAUSES
As if clauses são formadas por duas partes: a if clause (que é a oração que expressa uma
condição e por isso, é chamada de condicional) e a main clause (que é a oração principal).
Ou seja, a estrutura é uma oração subordinada e outra principal.
Elas são classificadas de quatro formas:

ZERO CONDITIONAL
FIRST CONDITIONAL
SECOND CONDITIONAL
THIRD CONDITIONAL

Na verdade, são, consecutivamente chamadas de zero, primeira, segunda e terceira


condicional, porque o grau de complexidade aumenta a cada número que elas têm.
Vejamos cada uma delas e a forma como se apresentam:

ZERO CONDITIONALS
As frases classificadas como Zero Conditionals indicam fatos verdadeiros, que, devido a
alguma coisa, acontecerá outra com certeza.
Geralmente também expressam fatos que ocorrem com frequência, e algumas vezes, são
empregadas para dar ordens.
A composição da zero conditional é a seguinte:
Frase 1: If + simple present/ Frase 2: simple present
Exemplos:

If I study a lot, I am approved. (Se eu estudar muito, sou aprovado.)


If you feel bad, talk to the doctor. (Se você se sentir mal, fale com o médico.)
If I win the lottery, I get rich. (Se eu ganhar na loteria, eu fico rico.)

AULA 08 – ABBREVIATIONS, CONTRACTIONS AND IF CLAUSES 20


TEACHER ANDREA BELO

FIRST CONDITIONALS
As frases classificadas como First conditionals indicam possibilidades ou prováveis ações
futuras.
É como afirmar algo e, em seguida, dizer o que vai acontecer em decorrência do que você
afirmou, por exemplo se você comer muito, logo, ficará satisfeito.
O verbo comer no presente e o verbo ficar (ficará) no futuro, vejamos o esquema da
estrutura:
Frase 1: If + simple present/ Frase 2: simple future
Exemplos:

If you buy one of the book now, I will give you a discount.
(Se você comprar o livro agora, eu te darei um desconto).

If I go to Europe, I can visit many places.


(Se eu for para a Europa, eu posso visitar muitos lugares).

Poderíamos, por exemplo, usar a mesma figura que foi utilizada para falar da zero
conditional sobre ganhar na loteria para aqui, ao invés de mostrar possibilidade, como ganhar na
loteria e ficar rico, a first conditional apresenta algo que vai acontecer caso você ganhe na loteria,
como por exemplo você vai comprar muitas coisas, certo?
If I win the lottery, I will buy a lot of things.
(Se eu ganhar na loteria, eu comprarei muitas coisas – pode acontecer um dia.)

SECOND CONDITIONALS
As frases classificadas como Second Conditionals indicam situações pouco prováveis e, na
maioria das vezes, irreais no futuro, já que expressa que se tivesse acontecido algo, outra coisa
também teria acontecido e isso é muito hipotético, veja a estrutura e o exemplo:
Frase 1: If+ simple past/ Frase 2: auxiliaries would, could, might, should + verb

If I had lots of money, I could buy a big house and a boat.


(Se eu tivesse muito dinheiro, eu poderia comprar uma casa grande e um barco).
(não tenho o dinheiro) (poderia comprar se a situação fosse diferente, é quase irreal)

AULA 08 – ABBREVIATIONS, CONTRACTIONS AND IF CLAUSES 21


TEACHER ANDREA BELO

Se dessa vez, usássemos aquela figura utilizada anteriormente para zero conditional (fato,
coisas possíveis) e first conditional (possibilidade grande), desta vez na second conditional, seria
dizer que, se eu tivesse ganhado na loteria, compraria muitas coisas, veja:
If I won the lottery, I would buy a lot of things.
(Se eu ganhasse na loteria, eu compraria muitas coisas – dificilmente vai acontecer, já que a 1ª situação não aconteceu)

THIRD CONDITIONAL
As frases classificadas como Third Conditionals, por sua vez, indicam algo que não ocorreu no
passado e, assim, expressa o sentimento de arrependimento:
Frase 1: If+ past perfect/ Frase 2: auxiliaries would have or any conditional + verb in the participle

If I had saved money, I would have bought lots of things in the past.
(Se eu tivesse economizado dinheiro, eu teria comprado muitas coisas no passado).
(não economizei) (poderia ter comprado e demonstra arrependimento)

If I had won the lottery, I would have traveled around the world.
(Se eu tivesse ganhado na loteria, eu teria viajado pelo mundo.

Agora, vamos resolver exercícios de anos anteriores para você treinar, como sempre
fazemos em nossas aulas. Vamos lá!

AULA 08 – ABBREVIATIONS, CONTRACTIONS AND IF CLAUSES 22


TEACHER ANDREA BELO

QUESTÕES
Você vai, agora, responder questões selecionadas de provas já realizadas em anos
anteriores. Depois, como em todas as nossas aulas, haverá o gabarito e as questões comentadas.
Vamos começar com questões AFA, de acordo coma sua instituição escolhida e depois,
vamos treinar de outras Carreiras Militares, para adquirir experiência e treinar vocabulário.
QUESTÕES AFA
Hanging out
R. Jordania
In American cities, teenagers like to spend time together – “hang out”, as they say– at drugstore,
luncheonettes, or ice cream parlors.
Often, they don’t even meet inside, but gather on the sidewalk in front of the store. From time to
time they go in for coffee, milk, ice cream. They also like to play the pinball machines.
Most parents disapprove of their children’s “hanging out” that way. They consider it a waste of
time, which could be better used doing homework, working at a part-time job, or helping in the
house.
Adapted from life in the USA

QUESTÃO 01 (AFA/INÉDITA) – According to the text,


A) Teenagers hang out only after they have done their housework
B) Every parent wants that their kid works a part-time job
C) Most parents disapprove of their kid’s time management
D) Teenagers do not like to stay at the sidewalks
E) Teenagers never stay on the sidewalks

QUESTÃO 02 (AFA/INÉDITA) – The term “hanging out” states the idea of


A) Socializing
B) Breaking up
C) Hugging
D) Loving
E) Chatting

Read the following text and answer questions 03, 04 e 05


Is it a good idea for a student to have a job? Why or why not?
Brandon Smith
I’m a junior in high school, and I have a part-time job in a restaurant. I wash dishes on Saturdays
and Sundays from 8:00 until 4:00. I earn $5.50 an hour. It isn’t much money, but I save almost

AULA 08 – ABBREVIATIONS, CONTRACTIONS AND IF CLAUSES 23


TEACHER ANDREA BELO

every penny! I want to go to a good university, and the cost goes up every year. Of course, I spend
some money when I go out on Saturday nights.
Lauren Russel
I’m a senior in high school. I have a job as a cashier in a grocery store. The job pays well – about
$6.75 an hour. I work every weeknight after school from 4:00 until 8:00. I don’t have time for
homework, and my grades aren’t very good this year. But I have to work, or I can’t buy nice clothes
and I can’t go out on Saturday nights. Also, a car costs a lot of money.
Erica Davis
I’m a freshman in college. College is very expensive, so I work in a law office for three hours every
weekday afternoon. I make photocopies, file papers, and sort mail for $8.25 an hour. The job gives
me good experience because I want to be a lawyer someday. But I don’t want to work every
semester. I need time to study.
Adapted from Interchange

QUESTÃO 03 (AFA/INÉDITA) – According to the text,


A) Brandon works to earn money for college
B) Brandon works to buy nice clothes
C) Lauren works to get job experience
D) Lauren works to pay for a motorcycle
E) Lauren bought a motorcycle.

QUESTÃO 04 (AFA/INÉDITA) – The text states that Erica Davis


A) Has graduated from high school
B) Works as a cashier in a grocery store
C) Wants to buy a car
D) Earns less then six dollars an hour
E) Earns six dollars an hour

QUESTÃO 05 (AFA/INÉDITA) – According to the text,


A) Brandon is the oldest of them
B) Lauren is the youngest of them
C) Brandon is paid the less amount of money per hour
D) Lauren is paid the most amount of money per hour
E) Lauren and Brendon paid the same amount

AULA 08 – ABBREVIATIONS, CONTRACTIONS AND IF CLAUSES 24


TEACHER ANDREA BELO

Read the following song and answer questions 06, 07 and 08.
SUNDAY MORNING
Sunday morning, rain is falling
Steal some covers, share some skin
Clouds are shrouding us in moments unforgettable
You twist to fit the mold that I _____ in
But things just get so crazy
Living life gets hard to do
And I would gladly hit the road, get up and go if I knew
That someday it would lead me back to you
That someday it would lead me back to you
Music Sunday morning by Marron 5

QUESTÃO 06 (AFA/INÉDITA) – The world “gladly”, underlined in the text, suggests an idea of
A) joyfully
B) sadly
C) joylessly
D) quickly
E) angrily

QUESTÃO 07 (AFA/INÉDITA) – According to the text,


A) The author of the song is breaking up with his girlfriend.
B) The author of the song is missing his home.
C) The author of the song wants to see someone he loves.
D) The author of the song is writting a sad song about Sundays.
E) The author of the song is writting a sad song about weekends.

QUESTÃO 08 (AFA/INÉDITA) – Mark the correct answer to complete the blank:


A) Am
B) Was
C) Were
D) Are
E) Do

AULA 08 – ABBREVIATIONS, CONTRACTIONS AND IF CLAUSES 25


TEACHER ANDREA BELO

Read the following song and answer questions 09 and 10


MIRRORS
Aren't you something to admire,
Because your shine is something like a mirror
And I can’t help but notice, you reflect in this heart of mine
If you ever feel alone and the glare makes me hard to find
Just know that I'm always parallel out on the other side
Beause with your hand in my hand and a pocket full of soul
I can tell you theres no place we couldn't go
Just put your hand on the glass
I'm here trying to pull you through you just gotta be strong
Because I don't wanna lose you now
I'm lookin' right at the other half of me
The vacancy that sat in my heart
Is a space, but now you're home
Music Mirrors by Justin Timberlake

QUESTÃO 09 (AFA/INÉDITA) – According to the text


A) The author is writing about an object he wants.
B) The author is sad because of a past accident
C) The author is writing about a girl he loves.
D) The author misses his hometown.
E) The author misses his mother.

QUESTÃO 10 (AFA/INÉDITA) – The world “glare”, underlined in the text, suggest the idea of
A) blaze
B) dirty
C) stinky
D) nasty
E) clean

AULA 08 – ABBREVIATIONS, CONTRACTIONS AND IF CLAUSES 26


TEACHER ANDREA BELO

QUESTÕES COLÉGIO NAVAL


Read Text I to do questions 01 to 04 based on it.
The delta variant is surging. How should that change how we live?
It’s been a week since the Centers for Disease Control and Prevention issued new masking
guidance based on evidence that vaccinated people can become infected and transmit the more
contagious delta variant to others. Many people who thought the vaccines allowed them to return
to pre-pandemic normal are now asking whether they need to change how they go about their
daily lives.
While delta does change the risk calculus, it doesn’t mean that we have go to back to hunkering
down at home. When deciding which activities to engage in, vaccinated people should consider
two factors: the medical risk of your household and the value of the activities to you.
Someone who is vaccinated, generally healthy, and either living alone or sharing a household with
others who are also vaccinated and healthy might decide that they are protected enough that they
won’t change anything. According to CDC data, the vaccinated are approximately eight times less
likely to become infected than the unvaccinated. Even if they contract covid-19, chances are
excellent that a vaccinated person will experience symptoms akin to the common cold; after all,
the vaccines reduce the chance of severe illness by a whopping 25-fold.
(Adapted from https://www.washingtonpost.com/opinions/2021/08/04/delta-variant-risk-assessment-leana-wen/)

QUESTÃO 01 (COLÉGIO NAVAL/INÉDITA) – Read the extract from the text


Even if they contract covid-19, chances are excellent that a vaccinated person will experience
symptoms akin to the common cold…

Mark the alternative that can replace the underlined word without changing its meaning.
(A) Equal
(B) Analogous
(C) Disconnected
(D) Unrelated
(E) Uniform

QUESTÃO 02 (COLÉGIO NAVAL/INÉDITA) – About the delta variant, it’s correct to affirm that
(A) It is less contagious than the others
(B) It does not affect the daily life of those who are already vaccinated
(C) It did not generate new doubts in the population
(D) It makes the vaccinated as vulnerable as the unvaccinated
(E) It can affect even those who are vaccinated

AULA 08 – ABBREVIATIONS, CONTRACTIONS AND IF CLAUSES 27


TEACHER ANDREA BELO

QUESTÃO 03 (COLÉGIO NAVAL/INÉDITA) – Read the extract from the text


“Even if they contract covid-19, chances are excellent that a vaccinated person will experience
symptoms akin to the common cold” (paragraph 3)

The word “they” refers to


(A) Unvaccinated people
(B) Symptoms
(C) Vaccinated people
(D) Vaccines
(E) CDC data

QUESTÃO 04 (COLÉGIO NAVAL/INÉDITA) – Read the extract from the text


“…after all, the vaccines reduce the chance of severe illness by a whopping 25-fold”.

The sentence above means that


(A) The vaccines prevent 25 different diseases
(B) The vaccines only prevent serious diseases
(C) The vaccines inhibit the onset of severe illness
(D) The vaccines significantly reduce the chance of severe illness
(E) The vaccines reduce the chance of serious illness after 25 days

Read the text II to do items 05 to 10.


Therapy via Zoom should make mental healthcare available for all – it hasn’t
Whether with a private therapist on Zoom, through an app that daily reminds us to log our
emotions, or in a back-andforth with a chatbot, teletherapy is often proffered as a catch-all salve
for our current mental healthcare crises. Remote treatment is touted as an efficient way to reach
more patients in a time of extreme difficulty, an intimate intervention that can scale.
During the on-and-off mandated social distancing that has marked the past 18 months of the
pandemic, teletherapy has shed its status as a minor form of care to become, at times, the only
thing on offer. The popularity of remote therapeutic sessions has soared in the US and the “users”
we once called patients are increasingly comfortable with such practices, and in some cases even
prefer them.
Having a therapist see a patient on Zoom or condensing treatment to self-tracking and AI
interfaces may be recent innovations, but the broad notion that technology and distanced
processes will solve our woes is nothing new. We have been turning to forms of technology to
deliver mental health services for more than 100 years. From 19th-century written cures delivered

AULA 08 – ABBREVIATIONS, CONTRACTIONS AND IF CLAUSES 28


TEACHER ANDREA BELO

by post and ad hoc telephone hotlines to the continuing elusive work to create an AI shrink, there
have been numerous mediated, networked, and remote relationships used in attempts to fix
longstanding problems with therapeutic provision. While those problems have obviously evolved
over time, they have also stayed relatively the same: good care is expensive and in short supply,
and barely begins to meet an overwhelming demand.
(Adapted from https://www.theguardian.com/commentisfree/2021/aug/03/therapy-zoom-mental-healthcare-remote-treatment)

QUESTÃO 05 (COLÉGIO NAVAL/INÉDITA) – According to the text, mark the INCORRECT option
(A) Remote treatment is, in fact, an efficient way to reach more people who need to take care of
their mental health
(B) During the pandemic, teletherapy gained prominence
(C) Social distancing ended up affecting the way people treat their mental health
(D) People who use teletherapy are already used to the new reality
(E) There are recent innovations being applied to the moment of therapy

QUESTÃO 06 (COLÉGIO NAVAL/INÉDITA) – Read the extract from the text


“Remote treatment is touted as an efficient way to reach more patients in a time of extreme
difficulty, an intimate intervention that can scale”

Mark the option that can replace the underlined sentence without changing its meaning
(A) Remote treatment is an efficient way to reach more people
(B) Remote treatment is publicized as an effective way to reach more people
(C) Remote treatment is criticized for being an efficient way to reach more patients
(D) Remote treatment is not considered an efficient way to reach more patients
(E) Remote treatment is not an efficient way to reach more people

QUESTÃO 07 (COLÉGIO NAVAL/INÉDITA) – Read the extract from the text


“…and in some cases even prefer them” (paragraph 2).
The word “them” refers to
(A) Users
(B) Patients
(C) Remote therapeutic sessions
(D) Some cases
(E) The US

AULA 08 – ABBREVIATIONS, CONTRACTIONS AND IF CLAUSES 29


TEACHER ANDREA BELO

QUESTÃO 08 (COLÉGIO NAVAL/INÉDITA) – Remote therapeutic sessions


(A) Have not gained popularity among the US population
(B) Are not relying on the help of recent innovations
(C) Were never the only therapeutic treatment option during the pandemic
(D) Have already gained the preference of some users/patients
(E) Were less sought after during the period of social isolation

QUESTÃO 09 (COLÉGIO NAVAL/INÉDITA) – The expression “on-and-off”, in paragraph 2, means


(A) Continuous
(B) Unique
(C) Endless
(D) Frequent
(E) Intermittent

QUESTÃO 10 (COLÉGIO NAVAL/INÉDITA) – The word “current” (paragraph 1), can be replaced by
the word ___ without changing its meaning
(A) Ongoing
(B) Actual
(C) Uncommon
(D) Temporary
(E) Permanent

QUESTÕES EAM
Read text I and answer questions 01 and 02
Parosmia: 'The smells and tastes we still miss, long after Covid'
Back in November I realised my chicken pasta tasted like washing-up liquid. I haven't eaten meat
since - mac and cheese, green grapes and baby rusks have become my staples.
I don't know whether I will ever be able to enjoy a Nando's medium-spiced chicken butterfly again
as now it smells and tastes foul, like something alien. I have struggled to come to terms with this.
If my smell goes back to normal, I'll never ever take a Nando's with friends for granted again.

Chanay Knight, 21, Birmingham


Caught Covid in October, developed parosmia in November I miss a simple pleasure - breakfast in
bed brought to me every morning by the husband I love. Deeply aromatic coffee with hot, frothy,

AULA 08 – ABBREVIATIONS, CONTRACTIONS AND IF CLAUSES 30


TEACHER ANDREA BELO

milk. Good bread, crisp and deep gold, slightly charred at the edges with butter or tangy
marmalade.
In spring we both caught Covid and he was hospitalised. I struggled down to the kitchen to make
coffee and toast for myself. Exhausted by such a simple task, I clung to the ritual and pictured him
beside me.
We both recovered, but coffee and toast is now repulsive to me - like a field just sprayed with
manure… unpleasant with a sweet fermented smell on top.
I've lost something that meant so much more to me than just breakfast.
Wendy Thompson, 59, Tameside
Caught Covid in May, developed parosmia in October
(Adapted from https://www.bbc.com/news/stories-55936729)

QUESTÃO 01 (EAM/INÉDITA) – About the reports given in the text, it is not to say that
(A) COVID-19 has no long-term impacts
(B) COVID-19 can change a person's lifestyle in the long run
(C) Some simple pleasures were maintained even with the loss of smell and taste
(D) COVID-19 is only a transient and extreme disease, which is asymptomatic or causes death
(E) The reports were not based on individual experiences

QUESTÃO 02 (EAM/INÉDITA) – In the sentence “In spring we both caught Covid and he was
hospitalised. I struggled down to the kitchen to make coffee and toast for myself” (paragraph 5),
the pronoun “we” refers to
(A) Wendy Thompson
(B) Chanay Knight
(C) Wendy and Chanay
(D) Wendy and her family
(E) Wendy and her husband

Read text II and answer questions 03 and 04


What can we take with us from lockdown to make life better than before?
Spring has sprung, albeit a little early. The weekend brought blue skies, daffodils bursting into
flower, the drone of lawnmowers coming back to life. The natural world is waking up fast, in
uncanny synchronicity with a nation now readying itself to emerge from Covid hibernation. Like
all jaded hacks, I rolled my eyes when the prime minister started waxing lyrical last week about
the crocus of hope pushing up through the frost of lockdown. But dammit, the crocuses are out
now, and if this strangely unfamiliar emotion is not hope, then it’s hard to know what else it is.

AULA 08 – ABBREVIATIONS, CONTRACTIONS AND IF CLAUSES 31


TEACHER ANDREA BELO

The coming of spring always paves the way, on some atavistic level, for the imagining of summer.
But this time the vaccine (touch wood, cross fingers, pray that the missing unknown carrier of the
Brazilian variant is tracked down before spreading it) provides some more rational grounds for
daring to imagine new beginnings.
The only way to keep one’s sanity in lockdown is to take life one day at a time, like an alcoholic in
recovery, and never think about how much longer there is to go. But suddenly it seems possible
to stop dwelling on everything that’s out of bounds and imagine having choices again. Travel
agents are reporting a rush of possibly over-optimistic bookings; fashion magazines are suddenly
full of heels and frivolousness, rather than clothes for collapsing on the sofa in. And for parents
who’ve been working from home, hunting down the PE kit ahead of next week’s return to school
is a reminder that one day our own presence may be required in a formal office again.
(Adapted from https://www.theguardian.com/commentisfree/2021/mar/01/covid-life-flexible-working-lockdown)

QUESTÃO 03 (EAM/INÉDITA) – Say if the following statements are T (true) or F (false) about
lockdown and its developments. Then, mark the correct option, from top to bottom.
( ) Imagining new beginnings is more complex in the context of a pandemic
( ) Superstitions are not accepted as a way to alleviate the near future
( ) The vaccine makes us imagine new beginnings
( ) A long-term vision for an end to the pandemic is the only way to tackle it
( ) "Normal life" is still out of the question in our imagination
(A) T – F – T – F – F
(B) T – F – T – T – F
(C) F – F – T – F – F
(D) T – T – T – F – T
(E) F – T – F – T – F

QUESTÃO 04 (EAM/INÉDITA) – “And for parents who’ve been working from home…”
The word “who” refers to:
(A) Children
(B) Someone unknown
(C) Travel agents
(D) Parents
(E) Fashion magazines

AULA 08 – ABBREVIATIONS, CONTRACTIONS AND IF CLAUSES 32


TEACHER ANDREA BELO

QUESTÃO 05 (EAM/INÉDITA) – Read the sentences and mark the correct options to fill in the
blanks respectively
Pedro is _______ older brother. _______ lives on the same street as us. We love to walk
together. _______ sneakers are blue like mine.

(A) Mine / He / His


(B) My / He / His
(C) My / They / Mine
(D) My / He / Mine
(E) Mine / We / His

QUESTÃO 06 (EAM/INÉDITA) – What sports can you see in the picture below?

(Adapted from https://pt.dreamstime.com/ilustra%C3%A7%C3%A3o-stock-crian%C3%A7as-e-projeto-dos-esportes-image90259764)

(A) Golf, cycling, and baseball


(B) Basketball, tennis and canoeing
(C) Soccer, volleyball and basketball
(D) Badminton, volleyball and golf
(E) Canoeing, skating and tennis

QUESTÃO 07 (EAM/INÉDITA) – Look at the picture below

(Adapted from https://br.freepik.com/vetores-premium/dois-garotinhos-com-mascaras-medicas-jogando-futebol-protecao-contra-virus-


conceito-dealergias-criancas-no-campo-de-futebol-ilustracao-plana-personagem-de-desenho-animado-esporte-e-recreacao_8207330.htm)

AULA 08 – ABBREVIATIONS, CONTRACTIONS AND IF CLAUSES 33


TEACHER ANDREA BELO

What are they doing in the picture?


(A) The children are playing soccer now
(B) The children play soccer everyday
(C) The children were playing soccer yesterday
(D) The children are not playing soccer
(E) The children didn’t play soccer yesterday

QUESTÃO 08 (EAM/INÉDITA) – Read the text below

(Adapted from http://ritasesl4u.pbworks.com/w/page/34378444/Grammar-Present%20Tenses)

In the cartoon, the future tense is used to express


(A) A plan
(B) A confirmation
(C) An ad
(D) A possibility
(E) A prediction

Read text II and answer questions 09 and 10


Coronavirus crisis unlikely to be over by the end of the year, WHO warns
Despite the spread of Covid-19 being slowed in some countries due to lockdowns and vaccination
programs, it is “premature” and “unrealistic” to the think the pandemic will be over by the end of
the year, the World Health Organization’s executive director of emergency services has said.
Speaking at a press briefing Geneva, Dr Michael Ryan said while vaccinating the most vulnerable
people, including healthcare workers, would help remove the “tragedy and fear” from the
situation, and would help to ease pressure on hospitals, the “virus is very much in control”.

AULA 08 – ABBREVIATIONS, CONTRACTIONS AND IF CLAUSES 34


TEACHER ANDREA BELO

“It will be very premature, and I think unrealistic, to think that we’re going to finish with this virus
by the end of the year,” Ryan said.
“If the vaccines begin to impact not only on death and not only on hospitalisation, but have a
significant impact on transmission dynamics and transmission risk, then I believe we will
accelerate toward controlling this pandemic.”
(Adapted from https://www.theguardian.com/world/2021/mar/02/coronavirus-crisis-unlikely-to-be-over-by-the-end-of-the-year-who-warns)

QUESTÃO 09 (EAM/INÉDITA) – In the sentence “It will be very premature, and I think unrealistic,
to think that we’re going to finish with this virus by the end of the year” (paragraph 2), the
pronoun “I” refers to
(A) Michel Ryan
(B) The world population
(C) The World Health Organization
(D) Vaccination
(E) Pandemic

QUESTÃO 10 (EAM/INÉDITA) – It is FALSE to say that


(A) The spread of coronavirus is being partially contained in some places
(B) Considering vaccination and lockdown, the pandemic is likely to end in a few months
(C) Most vulnerable people should be a priority in vaccination
(D) It is misleading to think that the pandemic will end by the end of the year
(E) Decreasing numbers of death and hospitalization are not the only thing to think about to end
the pandemic

QUESTÕES EEAR
Read the text and answer questions 01 to 05.
‘We are on the eve of a genocide’: Brazil urged to save Amazon from Covid-19
Brazil’s leaders must take immediate action to save the country’s indigenous peoples from a
Covid-19 “genocide”, a global coalition of artists, celebrities, scientists, and intellectuals has said.
In an open letter to Brazil, figures including Madonna, Oprah Winfrey, Brad Pitt, David Hockney
and Paul McCartney warned the pandemic meant indigenous communities in the Amazon faced
“an extreme threat to their very survival”.
“Five centuries ago, these ethnic groups were decimated by diseases brought by European
colonisers … Now, with this new scourge spreading rapidly across Brazil … [they] may disappear
completely since they have no means of combating Covid-19,” they wrote.
The organiser of the petition, the Brazilian photojournalist Sebastião Salgado, said trespassers
including wildcat gold miners and illegal loggers must be expelled immediately from indigenous

AULA 08 – ABBREVIATIONS, CONTRACTIONS AND IF CLAUSES 35


TEACHER ANDREA BELO

lands to stop them importing an illness that has killed more than 240,000 people around the
world, including 6,750 in Brazil. “We are on the eve of a genocide,” Salgado, who has spent nearly
four decades documenting the Amazon and its inhabitants, told the Guardian.
Even before Covid-19, Brazil’s indigenous peoples were locked in what activists call a historic
struggle for survival. Fears Covid-19 could devastate indigenous communities grew last month
when the death of a Yanomami teenager revived horrific memories of epidemics caused by
roadbuilders and gold prospectors in the 1970s and 80s. The Brazilian city so far worst hit by
coronavirus is Manaus, the capital of Amazonas, the state where part of the Yanomami reserve is
located.
Salgado – who is calling for the creation of an army-led taskforce to evict intruders from protected
areas – believes international pressure could force the government to act.
Adapted from https://www.theguardian.com/world/2020/may/03/eve-of-genocide-brazil-urged-save-amazon-tribes-covid-19-sebastiao-salgado

QUESTÃO 01 (EEAR/INÉDITA) – What’s the active voice for “... these ethnic groups were
decimated by diseases brought by European colonisers …” (paragraph 2)?
(A) Diseases brought by European colonisers have decimated these ethnic groups.
(B) Diseases brought by European colonisers are decimating these ethnic groups.
(C) Diseases brought by European colonisers will decimate these ethnic groups.
(D) Diseases brought by European colonisers were decimating these ethnic groups.
(E) Diseases brought by European colonisers decimated these ethnic groups.

QUESTÃO 02 (EEAR/INÉDITA) – In “‘Now, with this new scourge spreading rapidly across Brazil
… [they] may disappear completely since they have no means of combating Covid-19’, they
wrote.”, the underlined words respectively refer to
(A) these ethnic groups / international figures.
(B) diseases / international figures.
(C) these ethnic groups / these ethnic groups.
(D) European colonisers / these ethnic groups.
(E) European colonisers / international figures.

QUESTÃO 03 (EEAR/INÉDITA) – In the sentence “We are on the eve of a genocide...” (paragraph
3)., the expression, in bold type, is closest in meaning to _____________.
(A) much ahead of.
(B) a little after.
(C) about to have.
(D) at the night of.
(E) in the middle of.

AULA 08 – ABBREVIATIONS, CONTRACTIONS AND IF CLAUSES 36


TEACHER ANDREA BELO

QUESTÃO 04 (EEAR/INÉDITA) – The words “is calling”, underlined in the text, form a verb in the
__________.
(A) simple past
(B) future tense
(C) simple present
(D) present progressive
(E) present perfect

QUESTÃO 05 (EEAR/INÉDITA) – According to the text, Brazilian indigenous peoples are _______.
(A) endangered.
(B) decimated.
(C) secure.
(D) immune.
(E) unharmed.

Read the cartoon and answer questions 06 to 08.

Adapted from https://www.dailyrepublic.com/comics/beetle-bailey-245/attachment/bbt20130313/.

QUESTÃO 06 (EEAR/INÉDITA) – In the cartoon, the sentence “Clean this office right” refers to
(A) the future tense.
(B) the imperative mood.
(C) the indicative mood.
(D) the present continuous tense.
(E) the past tense.

AULA 08 – ABBREVIATIONS, CONTRACTIONS AND IF CLAUSES 37


TEACHER ANDREA BELO

QUESTÃO 07 (EEAR/INÉDITA) – Choose the best alternative to complete the blank in the text.
(A) didn’t cleaned
(B) didn’t clean
(C) did cleaned
(D) don’t cleaned
(E) doesn’t cleaned

QUESTÃO 08 (EEAR/INÉDITA) – The word “him” is a / an __________ pronoun.


(A) subject
(B) reflexive
(C) demonstrative
(D) possessive
(E) object

Read the text and answer questions 09 and 10.


The US Navy recorded three videos that show UFOs. A UFO is a flying object in the sky that
scientists cannot easily explain. Sometimes, UFOs are clouds or airplanes; however, many people
believe that they are spacecraft from other planets.
A UFO research group published the videos in 2018, and the US Navy said that the videos were
real. However, some people did not believe them. US officials decided to publish the videos again
because they wanted to show that the videos were real. The officials said that they could not
explain the origin of the objects in the videos. They consider the objects as unidentified.
Adapted from https://www.newsinlevels.com/.

QUESTÃO 09 (EEAR/INÉDITA) – According to the text,


(A) The videos the US Navy published were fake.
(B) The objects in the videos are clouds and airplanes.
(C) UFOs are unidentified flying objects.
(D) Everybody believed the videos the first time they were published.
(E) Many people believe UFOs are spacecraft from the Earth.

AULA 08 – ABBREVIATIONS, CONTRACTIONS AND IF CLAUSES 38


TEACHER ANDREA BELO

QUESTÃO 10 (EEAR/INÉDITA) – In the sentence “A UFO is a flying object in the sky that
scientists cannot easily explain”, cannot is used to express that scientists
(A) are prohibited to explain UFOs.
(B) are capable of explaining UFOs.
(C) are not allowed to explain UFOs.
(D) are unable to explain UFOs.
(E) are not confident about UFOs.

QUESTÕES EFOMM
Read Text I to do questions 01 to 04 based on it.
Vaccine passports are less a threat to liberty than a mark of solidarity
Anti-vaxxers in France and elsewhere claim personal freedom. But what of brotherhood?
In France over the past few weeks, the topic of vaccine passports has induced an avalanche of
outrage. Opposition to the measure has united both the hard left and right, with more than
200,000 people taking to the streets to express their contempt. In the kaleidoscope of disparate
groups involved, the only unifying banner is the assertion that Emmanuel Macron’s policy is an
infringement of the French tenet of liberté.
Nor is France unique in facing such resistance. In the United States, mask and vaccine mandates
have generated passionate opprobrium and legal action.
Those resolutely opposed to anything styled as a vaccination passport tend to frame the issue as
a solely personal choice. That can seem superficially reasonable, but it highlights a crucial
misunderstanding – a presumption that vaccination is solely an individual boon. On the contrary,
immunisation is, at heart, a public health measure, implemented to reduce incidence and burden
of disease at a population level. That it has huge individual benefit is undeniable but viewing
vaccination through this reductive, individualistic lens fundamentally distorts the reality that it is
about much more than protecting oneself.
Immunisation collectively reduces reservoirs for disease, providing a firewall that protects
vulnerable members of society. While a Covid infection might not do a young, healthy person
lasting harm, their passing on that infection could inflict substantial, even fatal, damage to
vulnerable people.
This is a consideration frequently missed in the arguments about proof of vaccination in public
spaces. Those decrying it as an infringement of their liberties fail to realise that others have a
reasonable expectation that they should not be needlessly exposed to a potentially deadly virus
if it can be avoided.
(Adapted from https://www.theguardian.com/commentisfree/2021/aug/15/vaccine-passports-are-less-a-threat-to-liberty-than-a-mark-of-solidarity)

AULA 08 – ABBREVIATIONS, CONTRACTIONS AND IF CLAUSES 39


TEACHER ANDREA BELO

QUESTÃO 01 (EFOMM/INÉDITA) – Read the extract from the text


“In France over the past few weeks, the topic of vaccine passports has induced an avalanche of outrage”
Mark the alternative that can replace the underlined word without changing its meaning.
(A) Indignation
(B) Happiness
(C) Craziness
(D) Comprehension
(E) Despair

QUESTÃO 02 (EFOMM/INÉDITA) – About vaccine passports and their consequences, it’s correct
to affirm that
(A) The measure was fully accepted by the French population
(B) France is the only country that is dealing with opposition to measures to combat the
pandemic
(C) Vaccination is in fact a personal act and with individual goals
(D) The function of the vaccine is to protect only the individual who takes it
(E) Mass vaccination protects those who cannot be vaccinated

QUESTÃO 03 (EFOMM/INÉDITA) – Read the extract from the text


“Opposition to the measure has united both the hard left and right, with more than 200,000
people taking to the streets to express their contempt” (paragraph 1)
The word “their” refers to
(A) Measure
(B) More than 200,000 people
(C) Vaccine passports
(D) Streets
(E) Contempt

QUESTÃO 04 (EFOMM/INÉDITA) – Read the extract from the text


“That can seem superficially reasonable but it highlights a crucial misunderstanding…”
The sentence above means that
(A) That may seem mildly understandable, but it reinforces an important misunderstanding
(B) That's completely understandable, though it highlights a critical misunderstanding
(C) That is completely incomprehensible and highlights a critical misunderstanding
(D) That's almost entirely understandable and highlights an important misunderstanding
(E) That may sound understandable, but it doesn't give enough importance to an important
misunderstanding

AULA 08 – ABBREVIATIONS, CONTRACTIONS AND IF CLAUSES 40


TEACHER ANDREA BELO

Read the text II to do items 05 to 10.


Israel extends Covid restrictions to three-year-olds as cases surge
Israel is now requiring anyone over the age of three to show proof of vaccination or a negative
Covid-19 test before entering many indoor spaces, as it tackles a sharp rise in infections.
Restaurants, cafes, museums, libraries, gyms and pools are among the venues covered by the
"Green Pass" system.
However, proof of immunity is not needed to go into shops or malls.
The country's Covid-19 tsar said it was "at war" with the virus despite its world-leading vaccination
programme.
"Our morbidity is rising day by day," Salman Zarka told a parliamentary committee on Wednesday,
according to the Jerusalem Post.
The next two weeks leading up to the Jewish New Year festival of Rosh Hashanah on 6 September
would be "critical", he warned.
Israel has seen a surge of infections driven by the more contagious Delta variant since late June.
The health ministry reported some 7,870 new Covid-19 cases on Tuesday, which was slightly down
on Monday's sixmonth daily record of 8,752.
More than 120 people have died after contracting the virus in the past week - double the monthly
total recorded in July - and 600 people are in a serious or critical condition in hospital.
The government has sought to combat the surge by reinstating the restrictions it lifted in mid-
June and by bringing back the Green Pass, which shows whether someone has been fully
vaccinated, has recently recovered from Covid-19, or tested negative in the previous 24 hours.
Before Wednesday only children aged 12 and over, who have been eligible to get a vaccine since
June, and adults were required to present a Green Pass.
It will now also apply to children between the ages of three and 11. Their tests will be funded by
the government as they are ineligible for vaccination, unless they are five or older and are
considered are at significant risk from Covid-19.
The approximately one million residents - about 11% of the population - who have not been
vaccinated despite being eligible must pay for their own tests.
Israel has also begun giving third doses of the Pfizer-BioNTech vaccine to people over 50, medical
workers and those with underlying health conditions.
So far, some 1.1 million eligible people have received their booster shots.
Israeli healthcare provider Maccabi, which covers about a quarter of the population, reported on
Wednesday that a third Pfizer dose was 86% effective at preventing Covid-19 infection in people
over 60.
Mr Zarka also noted that no-one who was currently in a critical condition in hospital had received
a booster shot.
(Adapted from https://www.bbc.com/news/world-middle-east-58245285)

AULA 08 – ABBREVIATIONS, CONTRACTIONS AND IF CLAUSES 41


TEACHER ANDREA BELO

QUESTÃO 05 (EFOMM/INÉDITA) – According to the text, mark the INCORRECT option


(A) In Israel, the Green Pass isn’t required to enter malls
(B) Israel's vaccination program sets an example for the rest of the world
(C) Success in Israel's vaccination program is ensuring victory in the battle against the virus
(D) Delta variant is making an impact in Israel
(E) The Green Pass helps to contain the spread of the virus

QUESTÃO 06 (EFOMM/INÉDITA) – Read the extract from the text


“Restaurants, cafes, museums, libraries, gyms and pools are among the venues covered by the
‘Green Pass’ system. However, proof of immunity is not needed to go into shops or malls”.
Mark the option that can replace the underlined sentence without changing its meaning
(A) Likewise, proof of immunity is not required to enter shops or malls
(B) On the other hand, proof of immunity is not required to enter stores or malls
(C) However, proof of immunity to enter stores or malls is mandatory
(D) In addition, proof of immunity is needed to enter shops or malls
(E) Nevertheless, proof of immunity is essential to enter shops or malls

QUESTÃO 07 (EFOMM/INÉDITA) – Read the extract from the text


“The government has sought to combat the surge by reinstating the restrictions it lifted in mid-
June and by bringing back the Green Pass, which shows whether someone…” (paragraph 8)
The word “which” refers to
(A) Government
(B) The surge
(C) Restrictions
(D) Someone fully vaccinated
(E) Green Pass

QUESTÃO 08 (EFOMM/INÉDITA) – The Green Pass


(A) Is intended for people aged 12 and over
(B) Is only for children under 3 years old
(C) Allows everyone to take tests for free
(D) Is now intended for all people aged 3 years and over
(E) Will be used for the first time in Israel

AULA 08 – ABBREVIATIONS, CONTRACTIONS AND IF CLAUSES 42


TEACHER ANDREA BELO

QUESTÃO 09 (EFOMM/INÉDITA) – The expression “day by day”, in paragraph 3, means


(A) Gradually over many days
(B) Abruptly
(C) Each month
(D) During all day long
(E) One day every two weeks

QUESTÃO 10 (EFOMM/INÉDITA) – The word “eligible” (paragraph 9), can be replaced by the
word ___ without changing its meaning
(A) Improper
(B) Good
(C) Privileged
(D) Suitable
(E) Awkward

QUESTÕES EPCAR
Directions: Answer questions 01 to 05 according to text.
What It Will Take to Get Life Back to Normal
Vaccines have brought the United States tantalizingly close to crushing the coronavirus within its
borders. After months of hiccups, some 1.4 million people are now being vaccinated every day,
and many more shots are coming through the pipeline. The Food and Drug Administration has
just authorized a third vaccine — a single-dose shot made by Johnson & Johnson — while Pfizer
and Moderna are promising to greatly expand the supply of their shots, to roughly 100 million
total doses per month, by early spring.
If those vaccines make their way into arms quickly, the nation could be on its way to a relatively
pleasant summer and something approaching normal by autumn. Imagine schools running at full
capacity in September and families gathering for Thanksgiving.
But turning that “if” into a “when” will require clearing additional hurdles so that everyone who
needs to be vaccinated gets vaccinated. This is especially true for racial minorities, who are being
disproportionately missed by the vaccination effort.
There’s plenty of disagreement among experts as to why America is still having problems with
vaccine uptake. Some officials have suggested that the main cause is that too many people are
hesitant to get the vaccine. Others point the finger at overcautious public health officials who they
say have undersold the promise of the vaccines. Still others point to long lines at clinics as proof
that far more people want the vaccine than can actually get it.
There is probably some truth to all of these hypotheses, and the underlying problems are not
new. Vaccine hesitancy had been growing steadily in America long before the current pandemic,

AULA 08 – ABBREVIATIONS, CONTRACTIONS AND IF CLAUSES 43


TEACHER ANDREA BELO

so much so that in 2019 the World Health Organization ranked it as one of the leading global
health threats. At the same time, poor health care access and other logistical constraints, such as
a lack of public transportation and limited internet access, have long impeded public health efforts
in low-income communities.
To maximize the number of Americans getting vaccinations, policymakers need to tackle each of
these crises with greater urgency than they have so far.
As supply increases, health officials should mount ambitious vaccination campaigns modeled on
ones that have worked to curb diseases in other countries. That will mean not relying solely on
web portals for scheduling vaccine appointments. It will mean going block by block and door to
door, through high-risk communities especially. It will mean setting up employee vaccination sites
at schools, grocery stores, transit hubs and meatpacking plants, and community clinics at houses
of worship, with local leaders promoting and running them.
“The easier you can make it for people to get vaccinated, the more likely your program will be to
succeed,” said Dr. Walter Orenstein, a former director of the national immunization program at
the Centers for Disease Control and Prevention. “It’s really that simple.”
Outreach efforts cost money. But they’re far less expensive than allowing the pandemic to fester.
Congress has appropriated some money to help states with vaccine rollout. It should offer more,
and states should put as much of those resources as possible toward vaccination efforts that meet
people where they are.
Health officials should also recognize that vaccine hesitancy has many root causes — deliberate
disinformation campaigns, mistrust of medical authorities in marginalized communities, ill-
considered messaging by health officials. The best way to counter that is with campaigns that are
locally led, that clearly outline the benefits of vaccination and that frame getting the shot as not
just a personal choice but a collective responsibility.
Doctors and scientists can help those pro-vaccine messages stick by minding their own public
communications. It’s crucial to be transparent about what vaccines will and won’t do for society
— overselling now will only sow more mistrust later.
That said, underselling is its own problem. It’s true that these vaccines will not immediately
restore the world to total normalcy. But they will eventually allow people to hug their loved ones,
to return to their offices — and to be protected from dying from or becoming seriously ill with
Covid-19. Health officials should be clear about that.
Policymakers at the highest levels of government should press social media companies and e-
commerce sites to curb the most aggressive purveyors of vaccine disinformation.
To not only quell this pandemic but to try to prevent the next one, America will need to improve
its health system and its public health apparatus, both of which have significant holes. “The
problem with a lot of the response is that it was predicated on the idea that we have a good
system in place for doing adult immunizations across the country,” said Dr. Peter Hotez, a vaccine
expert at Baylor College of Medicine. “The fact is, we really don’t.”
In the end, lawmakers and the people who vote them into office will have to address the much
broader problems that this pandemic has exposed.
(Adapted from https://www.nytimes.com/2021/02/26/opinion/vaccine-covid-coronavirus.html?action=click&module=Opinion&pgtype=Homepage)

AULA 08 – ABBREVIATIONS, CONTRACTIONS AND IF CLAUSES 44


TEACHER ANDREA BELO

QUESTÃO 01 (EPCAR/INÉDITA) – Mark the option that can replace the word “pleasant” in the
second paragraph without changing its meaning
(A) Pretty
(B) Gloomy
(C) Cordial
(D) Boring
(E) Harsh

QUESTÃO 02 (EPCAR/INÉDITA) – Mark the option with the suitable question to answer the
underlined fragment below
“After months of hiccups, some 1.4 million people are now being vaccinated every day, and
many more shots are coming through the pipeline” (paragraph 1)

(A) What are the expectations for the future of vaccination?


(B) How many people have already received the vaccine?
(C) How long was the vaccine developed in?
(D) Where are the vaccines coming from?
(E) How effective is this vaccine?

QUESTÃO 03 (EPCAR/INÉDITA) – Mark the statement that is NOT mentioned in the text
(A) Vaccines are helpoing the US to achieve a post-pandemic future
(B) US vaccination is slow and has no foreseeable future
(C) If vaccination in the US is successful, the country may be close to a better daily life
(D) A post-pandemic reality depends on some factors
(E) Vaccination in the USA is not impartial, there are privileged groups

QUESTÃO 04 (EPCAR/INÉDITA) – Mark the alternative that complete the sentence below
correctly
Vaccine acceptance _______________ a problem in the US
(A) Is
(B) Was
(C) Was being
(D) Has been
(E) Will be

AULA 08 – ABBREVIATIONS, CONTRACTIONS AND IF CLAUSES 45


TEACHER ANDREA BELO

QUESTÃO 05 (EPCAR/INÉDITA) – Mark the option that can replace the underlined word keeping
the same meaning
“Some officials have suggested that the main cause is that too many people are hesitant to get
the vaccine” (paragraph 4)
(A) Confident
(B) Skeptical
(C) Unbased
(D) Impartial
(E) Lazy

QUESTÕES ESA
Brazil, officially the Federative Republic of Brazil, is the largest country in both South America and
Latin America. At 8.5 million square kilometers (3.2 million square miles) and with over 211 million
people, Brazil is the world's fifth-largest country by area and the sixth most populous. Its capital
is Brasília, and its most populous city is São Paulo. _______ federation is composed of the union
of the 26 states and the Federal District. It is the largest country to have Portuguese as an official
language and the only one in the Americas; it is also one of the most multicultural and ethnically
diverse nations, due to over _______ century of mass immigration from around the world.
Bounded by the Atlantic Ocean on the east, Brazil has a coastline of 7,491 kilometers (4,655 mi).
It borders all other countries in South America except Ecuador and Chile and covers 47.3% of the
continent's land area. Its Amazon River basin includes _______ enormous tropical forest, home
to diverse wildlife, a variety of ecological systems, and extensive natural resources spanning
numerous protected habitats.
Adapted from https://en.wikipedia.org/wiki/Brazil.

QUESTÃO 01 (ESA/INÉDITA) – According to the text, it is incorrect to say that:


(A) Brazil is the largest country in South America.
(B) Brazil is the world's fifth most populous country.
(C) Brazil does not border Ecuador and Chile.
(D) Brazil’s official name is the Federative Republic of Brazil.
(E) Brazil is the only country in the Americas to have Portuguese as an official language.

QUESTÃO 02 (ESA/INÉDITA) – In the sentence “Its capital is Brasília, and its most populous city
is São Paulo.” (paragraph 1), the possessive adjectives “its” refer to
(A) Brazil.
(B) Latin America.
(C) South America.
(D) The Federal District.
(E) city.

AULA 08 – ABBREVIATIONS, CONTRACTIONS AND IF CLAUSES 46


TEACHER ANDREA BELO

QUESTÃO 03 (ESA/INÉDITA) – Choose the correct option to fill in the blanks with the missing
articles, from top to bottom:
(A) the / a / an
(B) a / a / a
(C) the / an / a
(D) an / an / an
(E) the / the / a

QUESTÃO 04 (ESA/INÉDITA) – “Officially” is


(A) an adjective
(B) a pronoun
(C) an adverb
(D) an article
(E) a verb

QUESTÃO 05 (ESA/INÉDITA) – “Reading makes life a lot easier”. In this sentence, the word
“makes” indicates
(A) a plural noun.
(B) third person plural.
(C) the Imperative form.
(D) the Simple Present Tense.
(E) the Present Continuous.

QUESTÃO 06 (ESA/INÉDITA) – “Reading makes life a lot easier”. “Easier”, in this sentence,
represents
(A) a time expression.
(B) a demonstrative pronoun.
(C) the Simple Present form of a verb.
(D) the superlative form of an adjective.
(E) the comparative form of an adjective.

QUESTÃO 07 (ESA/INÉDITA) – Choose the sentence that indicates future:


(A) I’m working now.
(B) Sam’s sleeping at the moment.
(C) He’s doing the laundry right now.
(D) Are you studying at this moment?
(E) They’re visiting their friends next weekend.

AULA 08 – ABBREVIATIONS, CONTRACTIONS AND IF CLAUSES 47


TEACHER ANDREA BELO

QUESTÃO 08 (ESA/INÉDITA) – Which alternative does not have the superlative form of the
adjective?
(A) the best
(B) the worst
(C) the bigger
(D) the greatest
(E) the most exquisite

QUESTÃO 09 (ESA/INÉDITA) – Which sentence is grammatically correct?


(A) Do you has a sister?
(B) Does she has a car?
(C) Do you have a brother?
(D) Do they has a job?
(E) Does he haves a bike?

QUESTÃO 10 (ESA/INÉDITA)
“A: Is John your friend?
B: Yes, _________.”
Fill in the blank with the correct form of pronoun and verb.
(A) he’s.
(B) he is.
(C) you are.
(D) she is.
(E) he isn’t.

QUESTÕES ESCOLA NAVAL


Can covid make your ears ring? What we know about its possible connection to tinnitus.
The recent death ____ Texas Roadhouse CEO Kent Taylor is bringing more attention to what
experts say is another troublesome physical ailment that may be associated with the coronavirus
pandemic: tinnitus, or the perception of a loud ringing or buzzing sound in the ears.
Taylor, 65, died ____ suicide last week, and his family told news outlets, including The Washington
Post, that he had been battling “post-Covid related symptoms, including severe tinnitus” at the
time of his death.
[Texas Roadhouse CEO dies by suicide while battling ‘unbearable’ post-covid-19 symptoms, family
says] Though early research and anecdotal reports have documented tinnitus and sudden hearing

AULA 08 – ABBREVIATIONS, CONTRACTIONS AND IF CLAUSES 48


TEACHER ANDREA BELO

loss in some covid-19 patients, audiologists emphasized that there is no conclusive evidence
connecting the virus to the onset or worsening of tinnitus.
The condition is “one of those things that is just so variable in every single person,” said Eldré
Beukes, ____ audiologist and research fellow at Anglia Ruskin University in ____ United Kingdom,
who has published research on how individuals with tinnitus have been affected by the pandemic.
In the United States, more than 50 million people experience some form of tinnitus, which is often
linked to hearing loss. About 20 million struggle with a chronic condition, while 2 million have
extreme and debilitating cases, according to the American Tinnitus Association.
(Adapted from https://www.washingtonpost.com/lifestyle/wellness/tinnitus-covid-treatment-ear-hearing/2021/03/23/aca1811c-8b32-11eb-9423-04079921c915_story.html)

QUESTÃO 01 (ESCOLA NAVAL/INÉDITA) – Which is the correct option to complete the gaps in
the text?
(A) Of / by / an / the
(B) Of / with / an / X
(C) Of / by / an / X
(D) Of / of / an / X
(E) Of / by / a / the

QUESTÃO 02 (ESCOLA NAVAL/INÉDITA) – According to the text, which option is correct?


(A) Kent Taylor's death is not associated with post-COVID-19 symptoms
(B) Tinnitus was not reported as a post-COVID-19 symptom for Kent
(C) Tinnitus is a post-COVID-19 symptom proven by science
(D) Most COVID-19 patients have tinnitus as a symptom of the disease
(E) Tinnitus is a condition that varies in intensity according to the individual

QUESTÃO 03 (ESCOLA NAVAL/INÉDITA) – What’s the meaning of the word “sudden” in


paragraph 4?
(A) Gradual
(B) Abrupt
(C) Expected
(D) Fast
(E) Lazy

Don’t fear the AstraZeneca jab, the risks are minimal


Statistics about the Oxford/AstraZeneca vaccine have dominated the news, with concern over
blood clots leading many European countries to suspend its use.
Then, on Thursday, the European Medicines Agency (EMA) and the UK Medicines and Healthcare
products Regulatory Agency (MRHA) declared there was no general increased risk of

AULA 08 – ABBREVIATIONS, CONTRACTIONS AND IF CLAUSES 49


TEACHER ANDREA BELO

thromboses, but they were continuing to look at a rare type of clotting linked with low platelets,
particularly in the brain.
From nearly 12m Oxford/AZ vaccinations up to 7 March, the UK “yellow card” system has
recorded 61,000 reports detailing 228,000 reactions, around double the rate for the
Pfizer/BioNTech vaccine. There have been huge numbers of immediate side-effects such as pain,
nausea, fatigue, headache, and fever, with other notable reports including palpitations (1,318),
“feeling jittery” (10), “screaming” (4), chilblains (10), alcohol poisoning (2), libido increased (1),
libido decreased (1), and one remarkable report of a pregnancy following vaccination. But some
have been serious events, including 289 deaths soon after the jab, and all of these will have
been examined carefully.
Every decision has trade-offs. Both UK vaccines have had more than 200 severe allergic reactions
(anaphylaxis), which is why we have to sit for 15 minutes after the jab. So it’s reasonable that,
rather than saying the vaccine is “safe”, the EMA and MRHA emphasise that the benefits of the
vaccine outweigh the risks of side-effects.
In the UK there has been one of these rare blood reactions in 2m jabs. But for every 2 million
people in their 50s getting a jab, the current group being vaccinated, we would currently expect
to prevent around five deaths a week. The ratio of benefit to harms is high, and even higher on
mainland Europe as their third wave starts.
The “cautionary” pause in many countries may mean increased vaccination hesitancy. France has
moved from initially not approving the Oxford/AZ vaccine for over-65s, to pausing it for
everyone, to now not approving it for under-55s. It would not be surprising if people are
confused by this. Confidence can shatter like glass and be hard to remake.
(Adapted from https://www.theguardian.com/theobserver/commentisfree/2021/mar/21/do-not-fear-the-astrazeneca-covid-jab-the-risks-areminimal)

QUESTÃO 04 (ESCOLA NAVAL/INÉDITA) – It is possible to infer from the text that


(A) There was never any concern about the side effects of the Oxford/AstraZeneca vaccine
(B) The Pfizer/BioNTech vaccine had a higher number of reactions than the Oxford/AstraZeneca
vaccine
(C) A vaccine is only safe when the rate of side effects is minimal or zero
(D) The mistrust and hesitation about the Oxford/AstraZeneca vaccine are predictable,
considering the current scenario
(E) The harms are greater than the benefits, in the Oxford/AstraZeneca vaccine

QUESTÃO 05 (ESCOLA NAVAL/INÉDITA) – In the excerpt “...with other notable reports including
palpitations (1,318), ‘feeling jittery’…” (paragraph 3), the word in bold means
(A) Calm
(B) Seasick
(C) Indecisive
(D) Headache
(E) Anxious

AULA 08 – ABBREVIATIONS, CONTRACTIONS AND IF CLAUSES 50


TEACHER ANDREA BELO

QUESTÃO 06 (ESCOLA NAVAL/INÉDITA) – Choose the correct alternative to complete the


paragraph below
It is profoundly difficult ____ grapple with risks whose stakes may include the global collapse of
civilisation, or even the extinction ____ humanity. ____ pandemic has shattered our illusions of
safety and reminded us that despite all the progress made in science and technology, we remain
vulnerable ____ catastrophes that can overturn our entire way of life. These are live possibilities,
not mere hypotheses, and our governments will have to confront them.
(Adapted from https://www.theguardian.com/commentisfree/2021/mar/23/covid-19-humanity-resilience-climate-ai-pandemic)

(A) To / of / the / to
(B) To / of / X / at
(C) X / of / the / for
(D) To / of / the / for
(E) At / of / the / for

We can’t get back to normal until everyone else does


In the United States, despite worrying upticks in cases in some states, we’re well on the way to
vaccinating ourselves out of the catastrophic domestic covid-19 crisis. To get fully “back to
normal,” though, we’re also going to have to — gulp — help vaccinate the rest of the world.
The Biden administration has so turbocharged the process of getting vaccines into arms that,
recently, we’ve been administering as many as 3 million shots per day, and sometimes more.
According to President Biden, by the end of May we will have obtained enough vaccine doses “for
every adult in America.”
True, we should expect setbacks. It is maddening to see the images of huge spring-break crowds
in Miami, jammed together with few participants wearing masks properly. Those young people
are only helping the covid-19 virus survive and potentially to mutate. But they are unlikely to
become seriously ill if infected — and if they do take the virus home with them, at least there is a
growing chance their parents and grandparents will already be immunized.
I keep thinking, though, about other crowds that were part of the old normal. I think of the masses
of foreign tourists who gathered in the summer on the National Mall, waiting their turn to enter
the Smithsonian’s National Air and Space Museum.
One of the languages I used to pick out most frequently in tourist crowds was Portuguese, spoken
with various Brazilian accents. Before covid-19, almost 2 million Brazilians visited the United States
each year. But Brazil has seen more deaths from the virus than any country except the United
States; anyone who has been in Brazil over the past 14 days is forbidden to enter the country.
We’ll never really get back to normal as long as international borders are essentially closed. And I
don’t see how those borders can fully open until we can be sure that visitors are not bringing with
them covid-19 — perhaps in the form of variant strains that are more infectious, more deadly or
more resistant to our vaccines.
(Adapted from https://www.washingtonpost.com/opinions/the-only-way-for-the-us-to-truly-get-back-to-normal-is-to-vaccinate-theworld/2021/03/22/de1a8aa8-8b3d-11eb-a730-1b4ed9656258_story.html)

AULA 08 – ABBREVIATIONS, CONTRACTIONS AND IF CLAUSES 51


TEACHER ANDREA BELO

QUESTÃO 07 (ESCOLA NAVAL/INÉDITA) – In the phrase “But they are unlikely to become
seriously ill if infected…” (paragraph 3), the underlined word is a synonym for
(A) Strong
(B) Diseased
(C) Bad
(D) Sad
(E) Affected

QUESTÃO 08 (ESCOLA NAVAL/INÉDITA) – The first paragraph of the text states that
(A) All US states have stabilized the number of cases
(B) In addition to the high cases, the US is doing poorly in its vaccination
(C) To return to normal, national vaccination is not enough, but global vaccination
(D) Vaccination in each country must be a national problem for each
(E) Some states in the US have a stable number of cases, all others are up in cases

QUESTÃO 09 (ESCOLA NAVAL/INÉDITA) – In the phrase “It is maddening to see the images
of huge spring-break crowds in Miami, jammed together with few participants wearing masks
properly” (paragraph 3), the underlined word is a synonym for
(A) Soothing
(B) Deranging
(C) Indifferent
(D) Coherent
(E) Weird

QUESTÃO 10 (ESCOLA NAVAL/INÉDITA) – The text


(A) Develops the idea that vaccines should be state-owned
(B) Accuses vaccine-related nationalism
(C) Aims to explain how the entire population's vaccination process works
(D) Focuses on the idea that the pandemic will only end when everyone overcomes it
(E) Advocates the administration of vaccines individually by countries

AULA 08 – ABBREVIATIONS, CONTRACTIONS AND IF CLAUSES 52


TEACHER ANDREA BELO

QUESTÕES EsPCEx
Read the text and answer the following questions 01 – 05.
THE HISTORY OF PIZZA
The modern pizza was originally invented in Naples, Italy but the word pizza is Greek in origin,
derived from the Greek word pēktos meaning solid or clotted. The ancient Greeks covered their
bread with oils, herbs and cheese. The first major innovation that led to flat bread pizza was the
use of tomato as a topping. It was common for the poor of the area around Naples to add tomato
to their yeast-based flat bread, and so the pizza began.
While it is difficult to say for sure ____ invented the pizza, it is however believed that modern pizza
was first made by baker Raffaele Esposito of Naples. In fact, a popular urban legend holds that the
archetypal pizza, Pizza Margherita, was invented in 1889, when the Royal Palace of Capodimonte
commissioned the Neapolitan pizzaiolo Raffaele Esposito to create a pizza in honor of the visiting
Queen Margherita. Of the three different pizzas he created, the Queen strongly preferred a pie
swathed in the colors of the Italian flag: red (tomato), green (basil), and white (mozzarella).
Supposedly, this kind of pizza was then named after the Queen as Pizza Margherita.
Later, the dish has become popular in many parts of the world:
• The first pizzeria, Antica Pizzeria Port'Alba, was opened in 1830 in Naples.

• In North America, The first pizzeria was opened in 1905 by Gennaro Lombardi at 53 1/3 Spring
Street in New York City.
• The first Pizza Hut, the chain of pizza restaurants appeared in the United States during the 1930s.

• Nowadays, many varieties of pizza exist worldwide, along with several dish variants based upon
pizza.

QUESTÃO 01 (EsPCEx/INÉDITA) – Check the correct statements according to the text:


I – The first pizza was invented in the 30’s.
II – The flat pizza covered with tomato was an innovation.
III – The first pizzeria in the USA was in New York City.
A) All of them are correct.
B) III is correct, but II is incorrect.
C) I and II are correct.
D) I and III are correct.
E) II and III are correct.

AULA 08 – ABBREVIATIONS, CONTRACTIONS AND IF CLAUSES 53


TEACHER ANDREA BELO

QUESTÃO 02 (EsPCEx/INÉDITA) – The blank space in the text can be completed


A) who
B) what
C) how
D) when
E) which

QUESTÃO 03 (EsPCEx/INÉDITA) – In the active voice the sentence: “The modern pizza was
originally invented in Naples”.
A) Someone invents the modern pizza in Naples.
B) Someone invented the modern pizza in Naples.
C) Someone has invented the modern pizza in Naples.
D) Someone was inventing the modern pizza in Naples.
E) Someome had invented the modern pizza in Naples.

QUESTÃO 04 (EsPCEx/INÉDITA) – The sentence in bold in the text “and so the pizza began”,
expresses an idea of
A) comparison
B) exclusion
C) result
D) contrast
E) time

QUESTÃO 05 (EsPCEx/INÉDITA) – The expression, “in fact”, in bold can be replaced by


A) otherwise
B) furthermore
C) however
D) indeed
E) instead of

AULA 08 – ABBREVIATIONS, CONTRACTIONS AND IF CLAUSES 54


TEACHER ANDREA BELO

GABARITO
GABARITO AFA
01 – C 02 – A 03 – A 04 – A 05 – C
06 – A 07 – C 08 – A 09 – C 10 – A

GABARITO COLÉGIO NAVAL


01 – B 02 – E 03 – C 04 – D 05 – A
06 – B 07 – C 08 – D 09 – E 10 – A

GABARITO EAM
01 – B 02 – E 03 – A 04 – D 05 – B
06 – C 07 – A 08 – E 09 – A 10 – B

GABARITO EEAR
01 – E 02 – A 03 – C 04 – D 05 – A
06 – B 07 – B 08 – E 09 – C 10 – D

GABARITO EFOMM
01 – A 02 – E 03 – B 04 – A 05 – C
06 – B 07 – E 08 – D 09 – A 10 – D

GABARITO EPCAR
01 – C 02 – A 03 – B 04 – D 05 – B

GABARITO ESA
01 – B 02 – A 03 – A 04 – C 05 – D
06 – E 07 – C 08 – C 09 – C 10 – B

GABARITO ESCOLA NAVAL


01 – A 02 – E 03 – B 04 – D 05 – E
06 – A 07 – B 08 – C 09 – B 10 – D

GABARITO EsPCEx
01 – E 02 – A 03 – B 04 – C 05 – D

AULA 08 – ABBREVIATIONS, CONTRACTIONS AND IF CLAUSES 55


TEACHER ANDREA BELO

QUESTÕES COMENTADAS
QUESTÕES AFA
Hanging out
R. Jordania
In American cities, teenagers like to spend time together – “hang out”, as they say– at drugstore,
luncheonettes, or ice cream parlors.
Often, they don’t even meet inside, but gather on the sidewalk in front of the store. From time to
time they go in for coffee, milk, ice cream. They also like to play the pinball machines.
Most parents disapprove of their children’s “hanging out” that way. They consider it a waste of
time, which could be better used doing homework, working at a part-time job, or helping in the
house.
Adapted from life in the USA

QUESTÃO 01 (AFA/INÉDITA) – According to the text,


A) Teenagers hang out only after they have done their housework
B) Every parent wants that their kid works a part-time job
C) Most parents disapprove of their kid’s time management
D) Teenagers do not like to stay at the sidewalks
E) Teenagers never stay on the sidewalks
Comentários: A alternativa A está errada. O trecho “They consider it a waste of time, which could
be better used … helping in the house” afirma que os pais consideram a resenha uma perda de
tempo e que esse tempo poderia ser usado para as tarefas de casa. Logo, pode-se inferir que os
adolescentes não fazem suas tarefas de casa antes de saírem para resenhar.
A alternativa B está errada. O trecho “Most parents disapprove of their children’s “hanging out”
that way. They consider it a waste of time, which could be better used … working at a part-time”
afirma que muitos pais acham que o tempo de seus filhos seriam melhor gasto se eles
trabalhassem meio período, mas não todos eles.
A alternativa C está correta. O trecho “Most parents disapprove of their children’s “hanging out”
that way. They consider it a waste of time …” afirma que muitos pais acham não gostam da
resenha de seus filhos e acham que eles poderiam investir seu tempo em outras coisas, logo,
pode-se inferir que eles não aprovam a maneira como seus filhos usam seu tempo.
A alternativa D está errada. O trecho “Often, they don’t even meet inside, but gather on the
sidewalk in front of the store” afirma que, muitas vezes, eles se encontram na calçada. Logo, não
se pode afirmar que eles não gostam de ficar na calçada.
A alternativa E está errada. O trecho “Often, they don’t even meet inside, but gather on the
sidewalk in front of the store” afirma que, muitas vezes, eles se encontram na calçada. Logo, não
se pode afirmar que eles nunca ficam na calçada.
GABARITO: C

AULA 08 – ABBREVIATIONS, CONTRACTIONS AND IF CLAUSES 56


TEACHER ANDREA BELO

QUESTÃO 02 (AFA/INÉDITA) – The term “hanging out” states the idea of


A) Socializing
B) Breaking up
C) Hugging
D) Loving
E) Chatting
Comentários: O trecho “In American cities, teenagers like to spend time together – “hang out, as
they say” afirma que, nas cidades americana, adolescentes gostam de passer tempo junto e que
eles chamam isso de “hanging out”, então, essa expressão pode ser entendida como socializar,
que, em inglês, é “socializing”. Logo, a alternativa A está correta.
A alternativa B está errada. A expressão “breaking up” significa “terminar”, geralmente, um
relacionamento. Logo, não tem o mesmo sentido do que a expressão em análise.
A alternativa C está errada. A palavra “hugging” significa “abraçar”. Logo, não tem o mesmo
sentido do que a expressão em análise.
A alternativa D está errada. A palavra “loving” significa “amar”. Logo, não tem o mesmo sentido
do que a expressão em análise.
A alternativa R está errada. A palavra “chatting” significa “conversar online/bater papo pela
internet”. Logo, não tem o mesmo sentido do que a expressão em análise.
GABARITO: A

Read the following text and answer questions 03, 04 e 05


Is it a good idea for a student to have a job? Why or why not?
Brandon Smith
I’m a junior in high school, and I have a part-time job in a restaurant. I wash dishes on Saturdays
and Sundays from 8:00 until 4:00. I earn $5.50 an hour. It isn’t much money, but I save almost
every penny! I want to go to a good university, and the cost goes up every year. Of course, I spend
some money when I go out on Saturday nights.
Lauren Russel
I’m a senior in high school. I have a job as a cashier in a grocery store. The job pays well – about
$6.75 an hour. I work every weeknight after school from 4:00 until 8:00. I don’t have time for
homework, and my grades aren’t very good this year. But I have to work, or I can’t buy nice clothes
and I can’t go out on Saturday nights. Also, a car costs a lot of money.
Erica Davis
I’m a freshman in college. College is very expensive, so I work in a law office for three hours every
weekday afternoon. I make photocopies, file papers, and sort mail for $8.25 an hour. The job gives
me good experience because I want to be a lawyer someday. But I don’t want to work every
semester. I need time to study.
Adapted from Interchange

AULA 08 – ABBREVIATIONS, CONTRACTIONS AND IF CLAUSES 57


TEACHER ANDREA BELO

QUESTÃO 03 (AFA/INÉDITA) – According to the text,


A) Brandon works to earn money for college
B) Brandon works to buy nice clothes
C) Lauren works to get job experience
D) Lauren works to pay for a motorcycle
E) Lauren bought a motorcycle.
Comentários: Brandon disse “I want to go to a good university, and the cost goes up every year”,
que significa “Eu quero ir para uma boa faculdade, e os custos aumentam a cada ano”. Logo,
Brandon trabalha principalmente para juntar dinheiro para ir para a faculdade. Logo, a alternativa
A está correta e a alternativa B está errada.
Lauren disse “But I have to work, or I can’t buy nice clothes and I can’t go out on Saturday nights.
Also, a car costs a lot of money”, que significa “Mas eu tenho que trabalhar, ou eu não vou
conseguir comprar roupas bonitas e eu não vou poder sair nas noites de sábado. Além disso, um
carro custa muito dinheiro”. Então, ela não trabalha para conseguir experiencia profissional e nem
quer comprar uma moto. Logo, as alternativas C, D e E estão erradas.
GABARITO: A

QUESTÃO 04 (AFA/INÉDITA) – The text states that Erica Davis


A) Has graduated from high school
B) Works as a cashier in a grocery store
C) Wants to buy a car
D) Earns less then six dollars an hour
E) Earns six dollars an hour
Comentários: A alternativa A está correta. Erica disse “I’m a freshman in college”, que significa
que ela já está na faculdade. Logo, é possível inferir que ela já se formou no Ensino Médio.
A alternativa B está errada. O trecho “I work in a law office”, prova que ela trabalha num escritório
de direito.
A alternativa C está errada. O trecho “Also, a car costs a lot of money” que deixa implícito querer
comprar um carro, foi dito por Lauren e não por Erica.
As alternativas D e E estão erradas. O trecho “for $8.25 an hour” prova que ela recebe 8.25 dólares
por hora.
GABARITO: A

AULA 08 – ABBREVIATIONS, CONTRACTIONS AND IF CLAUSES 58


TEACHER ANDREA BELO

QUESTÃO 05 (AFA/INÉDITA) – According to the text,


A) Brandon is the oldest of them
B) Lauren is the youngest of them
C) Brandon is paid the less amount of money per hour
D) Lauren is paid the most amount of money per hour
E) Lauren and Brendon paid the same amount
Comentários: Brandon está no primeiro ano do ensino médio, Lauren no último ano do ensino
médio e Erica está na faculdade. Então, pode-se inferir que Brandon é o mais novo e Erica a mais
velha. Logo, as alternativas A e B estão erradas.
Brandon recebe 5.50 dólares por hora, Lauren recebe 6.75 e Erica recebe 8.5. Logo, Brandon é
quem recebe o menor salário e Erica é quem é melhor paga. Logo, a alternativa C está correta e
as alternativas D e E estão erradas.
GABARITO: C

Read the following song and answer questions 06, 07 and 08.
SUNDAY MORNING
Sunday morning, rain is falling
Steal some covers, share some skin
Clouds are shrouding us in moments unforgettable
You twist to fit the mold that I _____ in
But things just get so crazy
Living life gets hard to do
And I would gladly hit the road, get up and go if I knew
That someday it would lead me back to you
That someday it would lead me back to you
Music Sunday morning by Marron 5

QUESTÃO 06 (AFA/INÉDITA) – The world “gladly”, underlined in the text, suggests an idea of
A) joyfully
B) sadly
C) joylessly
D) quickly
E) angrily

AULA 08 – ABBREVIATIONS, CONTRACTIONS AND IF CLAUSES 59


TEACHER ANDREA BELO

Comentários: O trecho que deve ser analisado é “And I would gladly hit the road, get up and go if
I knew that someday it would lead me back to you”, o qual pode ser traduzido como “E eu ficaria
alegre em pegar a estrada, levantar e ir se soubesse que um dia isso me levaria de volta a você”.
A alternativa A está correta, pois o sinônimo do adverbio “joyfully” é de modo alegre.
A alternativa B está errada, pois “sadly” é sinônimo de ‘tristemente’.
A alternativa C está errada, pois o adverbio “joylessly” ´transmite a ideia, também, de tristemente.
A alternativa D está errada, pois “quickly” é sinônimo de ‘rapidamente’.
A alternativa E está errada, pois “angrily” é sinônimo de ‘’com raiva/de forma nervosa”.
GABARITO: A

QUESTÃO 07 (AFA/INÉDITA) – According to the text,


A) The author of the song is breaking up with his girlfriend.
B) The author of the song is missing his home.
C) The author of the song wants to see someone he loves.
D) The author of the song is writting a sad song about Sundays.
E) The author of the song is writting a sad song about weekends.
Comentários: No trecho “And I would gladly hit the road, get up and go if I knew that someday it
would lead me back to you”, o qual pode ser traduzido como “E eu ficaria alegre em pegar a
estrada, levantar e ir se soubesse que um dia isso me levaria de volta a você”, pode ser notado
que o autor sente falta de alguém amada(o). Por isso, a alternativa C é a verdadeira.
A alternativa A está errada, pois afirma que o autor está terminando com sua namorada.
A alternativa B está errada, pois afirma que o autor está sentindo falta de sua casa.
A alternativa D está errada, pois afirma que o autor está escrevendo uma música triste sobre os
dias de domingo.
A alternativa E está errada, pois afirma que o autor está escrevendo uma música triste sobre os
fins de semana e vimos que não é isso em nossa análise.
GABARITO: C

QUESTÃO 08 (AFA/INÉDITA) – Mark the correct answer to complete the blank:


A) Am
B) Was
C) Were
D) Are
E) Do
Comentários: A alternativa A está correta, primordialmente, pois na primeira estrofe da letra
todos os verbos estão no presente, indicando que a frase deve ser completada também nesse

AULA 08 – ABBREVIATIONS, CONTRACTIONS AND IF CLAUSES 60


TEACHER ANDREA BELO

tempo verbal. A frase “You twist to fit the mold that I am” in” significa “Você se contorce para se
encaixar no molde em que estou”.
A alternativa B está errada pois o verbo “twist” está no presente, então o verbo “to be” também
deve estar no presente.
A alternativa C está errada pois o verbo “twist” está no presente, então o verbo “to be” também
deve estar no presente.
A alternativa D está errada pois o sujeito é “I”, então a conjução “are” para o verbo “to be” estaria
errada.
A alternativa E está errada pois o “I” não está enfatizando uma ideia nem fazendo uma pergunta,
então o auxiliar “do” estaria errado na frase.
GABARITO: A

Read the following song and answer questions 09 and 10


MIRRORS
Aren't you something to admire,
Because your shine is something like a mirror
And I can’t help but notice, you reflect in this heart of mine
If you ever feel alone and the glare makes me hard to find
Just know that I'm always parallel out on the other side
Beause with your hand in my hand and a pocket full of soul
I can tell you theres no place we couldn't go
Just put your hand on the glass
I'm here trying to pull you through you just gotta be strong
Because I don't wanna lose you now
I'm lookin' right at the other half of me
The vacancy that sat in my heart
Is a space, but now you're home
Music Mirrors by Justin Timberlake

QUESTÃO 09 (AFA/INÉDITA) – According to the text


A) The author is writing about an object he wants.
B) The author is sad because of a past accident
C) The author is writing about a girl he loves.
D) The author misses his hometown.
E) The author misses his mother.

AULA 08 – ABBREVIATIONS, CONTRACTIONS AND IF CLAUSES 61


TEACHER ANDREA BELO

Comentários: Para uma clara interpretação pode ser utilizado o trecho “Because I don't wanna
lose you now I'm lookin' right at the other half of me”, o qual pode ser traduzido como “Porque
eu não quero perder você agora, estou olhando diretamente para a outra metade mim.”.
Claramente pode ser observado que o autor da canção está se declarando para alguém amado(a),
tornando a alternativa C correta.
A alternativa A está errada, pois afirma que o autor está escrevendo sobre um objeto de desejo.
A alternativa B está errada, pois afirma que o autor está triste sobre um acidente passado.
A alternativa D está errada, pois afirma que o autor está sentindo saudade da sua cidade natal.
A alternativa E está errada, pois afirma que o autor está sentindo saudade da sua mãe.
GABARITO: C

QUESTÃO 10 (AFA/INÉDITA) – The world “glare”, underlined in the text, suggest the idea of
A) blaze
B) dirty
C) stinky
D) nasty
E) clean
Comentários: A palavra “glare” significa ‘clarão’ ou ‘brilho intenso’. Assim, a alternativa A está
correta, pois “blaze” significa “brilho” ou ‘chama’, os quais são sinônimos adequados para o
contexto.
A alternativa B está errada, pois “dirty” significa ‘sujo’.
A alternativa C está errada, pois “stinky” significa “mal-cheiroso/fédido”.
A alternativa D está errada, pois “nasty” significa ‘nojento ou desagradável’.
A alternativa E está errada, pois “clean” significa ‘limpo’ e não é sinônimo de brilho/clarão.
GABARITO: A

QUESTÕES COLÉGIO NAVAL


Read Text I to do questions 01 to 04 based on it.
The delta variant is surging. How should that change how we live?
It’s been a week since the Centers for Disease Control and Prevention issued new masking
guidance based on evidence that vaccinated people can become infected and transmit the more
contagious delta variant to others. Many people who thought the vaccines allowed them to return
to pre-pandemic normal are now asking whether they need to change how they go about their
daily lives.
While delta does change the risk calculus, it doesn’t mean that we have go to back to hunkering
down at home. When deciding which activities to engage in, vaccinated people should consider
two factors: the medical risk of your household and the value of the activities to you.

AULA 08 – ABBREVIATIONS, CONTRACTIONS AND IF CLAUSES 62


TEACHER ANDREA BELO

Someone who is vaccinated, generally healthy, and either living alone or sharing a household with
others who are also vaccinated and healthy might decide that they are protected enough that they
won’t change anything. According to CDC data, the vaccinated are approximately eight times less
likely to become infected than the unvaccinated. Even if they contract covid-19, chances are
excellent that a vaccinated person will experience symptoms akin to the common cold; after all,
the vaccines reduce the chance of severe illness by a whopping 25-fold.
(Adapted from https://www.washingtonpost.com/opinions/2021/08/04/delta-variant-risk-assessment-leana-wen/)

QUESTÃO 01 (COLÉGIO NAVAL/INÉDITA) – Read the extract from the text


Even if they contract covid-19, chances are excellent that a vaccinated person will experience
symptoms akin to the common cold…
Mark the alternative that can replace the underlined word without changing its meaning.
(A) Equal
(B) Analogous
(C) Disconnected
(D) Unrelated
(E) Uniform
Comentários: A alternativa A está incorreta. A palavra “akin” significa “semelhante” e não pode
ser substituída pela palavra “equal”, que significa “igual”.
A alternativa B está correta. A palavra “akin” significa “semelhante” e pode ser substituída pela
palavra “analogous”, que significa “análogo”.
A alternativa C está incorreta. A palavra “akin” significa “semelhante” e não pode ser substituída
pela palavra “disconnected”, que significa “desconectado”.
A alternativa D está incorreta. A palavra “akin” significa “semelhante” e não pode ser substituída
pela palavra “unrelated”, que significa “não relacionado”.
A alternativa E está incorreta. A palavra “akin” significa “semelhante” e não pode ser substituída
pela palavra “uniform”, que significa “uniforme”.
GABARITO: B

QUESTÃO 02 (COLÉGIO NAVAL/INÉDITA) – About the delta variant, it’s correct to affirm that
(A) It is less contagious than the others
(B) It does not affect the daily life of those who are already vaccinated
(C) It did not generate new doubts in the population
(D) It makes the vaccinated as vulnerable as the unvaccinated
(E) It can affect even those who are vaccinated

AULA 08 – ABBREVIATIONS, CONTRACTIONS AND IF CLAUSES 63


TEACHER ANDREA BELO

Comentários: A alternativa A está incorreta. Sobre a variante delta, não é correto afirmar que é
menos contagiosa que as demais, mas sim, que é mais contagiosa do que as demais. Isso pode ser
confirmado com o trecho “...vaccinated people can become infected and transmit the more
contagious delta variant to others”.
A alternativa B está incorreta. Sobre a variante delta, não é correto afirmar que não afeta o dia a
dia de quem já está vacinado, mas sim, que pode afetar. Isso pode ser confirmado com o trecho
“Many people who thought the vaccines allowed them to return to pre-pandemic normal are now
asking whether they need to change how they go about their daily lives”.
A alternativa C está incorreta. Sobre a variante delta, não é correto afirmar que não gerou novas
dúvidas na população, mas sim, que gerou. Isso pode ser confirmado com o trecho “Many people
who thought the vaccines allowed them to return to pre-pandemic normal are now asking whether
they need to change how they go about their daily lives”.
A alternativa D está incorreta. Sobre a variante delta, não é correto afirmar que torna os vacinados
tão vulneráveis quanto os não vacinados, pois os vacinados estão menos sucessivos a se
infectarem e a variante não muda essa estatística. Isso pode ser confirmado com o trecho
“According to CDC data, the vaccinated are approximately eight times less likely to become infected
than the unvaccinated”.
A alternativa E está correta. Sobre a variante delta, é correto afirmar que pode afetar até quem
está vacinado, assim como esta opção indica. Isso pode ser confirmado com o trecho “...vaccinated
people should consider two factors: the medical risk of your household and the value of the
activities to you”.
GABARITO: E

QUESTÃO 03 (COLÉGIO NAVAL/INÉDITA) – Read the extract from the text


“Even if they contract covid-19, chances are excellent that a vaccinated person will experience
symptoms akin to the common cold” (paragraph 3)
The word “they” refers to
(A) Unvaccinated people
(B) Symptoms
(C) Vaccinated people
(D) Vaccines
(E) CDC data
Comentários: A alternativa A está incorreta. A palavra “they” (eles/elas) não se refere às pessoas
não vacinadas, mas sim, às pessoas vacinadas. Isso pode ser confirmado com o trecho “...the
vaccinated are approximately eight times less likely to become infected than the unvaccinated.
Even if they contract covid-19, chances are excellent that a vaccinated person will experience
symptoms akin to the common cold…”.
A alternativa B está incorreta. A palavra “they” (eles/elas) não se refere aos sintomas, mas sim, às
pessoas vacinadas. Isso pode ser confirmado com o trecho “...the vaccinated are approximately

AULA 08 – ABBREVIATIONS, CONTRACTIONS AND IF CLAUSES 64


TEACHER ANDREA BELO

eight times less likely to become infected than the unvaccinated. Even if they contract covid-19,
chances are excellent that a vaccinated person will experience symptoms akin to the common
cold…”.
A alternativa C está correta. A palavra “they” (eles/elas) se refere às pessoas vacinadas, assim
como esta opção indica. Isso pode ser confirmado com o trecho “...the vaccinated are
approximately eight times less likely to become infected than the unvaccinated. Even if they
contract covid-19, chances are excellent that a vaccinated person will experience symptoms akin
to the common cold…”.
A alternativa D está incorreta. A palavra “they” (eles/elas) não se refere às vacinas, mas sim, às
pessoas vacinadas. Isso pode ser confirmado com o trecho “...the vaccinated are approximately
eight times less likely to become infected than the unvaccinated. Even if they contract covid-19,
chances are excellent that a vaccinated person will experience symptoms akin to the common
cold…”.
A alternativa E está incorreta. A palavra “they” (eles/elas) não se refere aos dados da CDC, mas
sim, às pessoas vacinadas. Isso pode ser confirmado com o trecho “...the vaccinated are
approximately eight times less likely to become infected than the unvaccinated. Even if they
contract covid-19, chances are excellent that a vaccinated person will experience symptoms akin
to the common cold…”.
GABARITO: C

QUESTÃO 04 (COLÉGIO NAVAL/INÉDITA) – Read the extract from the text


“…after all, the vaccines reduce the chance of severe illness by a whopping 25-fold”.

The sentence above means that


(A) The vaccines prevent 25 different diseases
(B) The vaccines only prevent serious diseases
(C) The vaccines inhibit the onset of severe illness
(D) The vaccines significantly reduce the chance of severe illness
(E) The vaccines reduce the chance of serious illness after 25 days
Comentários: A alternativa A está incorreta. A frase “Afinal, as vacinas reduzem a chance de
doenças graves em 25 vezes” não significa que as vacinas previnem 25 doenças diferentes, mas
sim, que as vacinas reduzem significativamente a chance de doenças graves.
A alternativa B está incorreta. A frase “Afinal, as vacinas reduzem a chance de doenças graves em
25 vezes” não significa que as vacinas só previnem doenças graves, mas sim, que as vacinas
reduzem significativamente a chance de doenças graves.
A alternativa C está incorreta. A frase “Afinal, as vacinas reduzem a chance de doenças graves em
25 vezes” não significa que as vacinas inibem o aparecimento de doenças graves, mas sim, que as
vacinas reduzem significativamente a chance de doenças graves.

AULA 08 – ABBREVIATIONS, CONTRACTIONS AND IF CLAUSES 65


TEACHER ANDREA BELO

A alternativa D está correta. A frase “Afinal, as vacinas reduzem a chance de doenças graves em
25 vezes” significa que as vacinas reduzem significativamente a chance de doenças graves, assim
como esta opção indica.
A alternativa E está incorreta. A frase “Afinal, as vacinas reduzem a chance de doenças graves em
25 vezes” não significa que as vacinas reduzem a chance de doenças graves após 25 dias, mas sim,
que as vacinas reduzem significativamente a chance de doenças graves.
GABARITO: D

Read the text II to do items 05 to 10.


Therapy via Zoom should make mental healthcare available for all – it hasn’t
Whether with a private therapist on Zoom, through an app that daily reminds us to log our
emotions, or in a back-andforth with a chatbot, teletherapy is often proffered as a catch-all salve
for our current mental healthcare crises. Remote treatment is touted as an efficient way to reach
more patients in a time of extreme difficulty, an intimate intervention that can scale.
During the on-and-off mandated social distancing that has marked the past 18 months of the
pandemic, teletherapy has shed its status as a minor form of care to become, at times, the only
thing on offer. The popularity of remote therapeutic sessions has soared in the US and the “users”
we once called patients are increasingly comfortable with such practices, and in some cases even
prefer them.
Having a therapist see a patient on Zoom or condensing treatment to self-tracking and AI
interfaces may be recent innovations, but the broad notion that technology and distanced
processes will solve our woes is nothing new. We have been turning to forms of technology to
deliver mental health services for more than 100 years. From 19th-century written cures delivered
by post and ad hoc telephone hotlines to the continuing elusive work to create an AI shrink, there
have been numerous mediated, networked, and remote relationships used in attempts to fix
longstanding problems with therapeutic provision. While those problems have obviously evolved
over time, they have also stayed relatively the same: good care is expensive and in short supply,
and barely begins to meet an overwhelming demand.
(Adapted from https://www.theguardian.com/commentisfree/2021/aug/03/therapy-zoom-mental-healthcare-remote-treatment)

QUESTÃO 05 (COLÉGIO NAVAL/INÉDITA) – According to the text, mark the INCORRECT option
(A) Remote treatment is, in fact, an efficient way to reach more people who need to take care of
their mental health
(B) During the pandemic, teletherapy gained prominence
(C) Social distancing ended up affecting the way people treat their mental health
(D) People who use teletherapy are already used to the new reality
(E) There are recent innovations being applied to the moment of therapy
Comentários: A alternativa A está correta. De acordo com o texto, é incorreto afirmar que o
tratamento remoto é, de fato, uma forma eficiente de atingir mais pessoas que precisam cuidar

AULA 08 – ABBREVIATIONS, CONTRACTIONS AND IF CLAUSES 66


TEACHER ANDREA BELO

de sua saúde mental, mas sim, que ele é apontado como uma forma eficiente para isso, não
necessariamente é. Isso pode ser confirmado com o trecho “Remote treatment is touted as an
efficient way to reach more patients in a time of extreme difficulty...”, “...good care is expensive
and in short supply, and barely begins to meet an overwhelming demand”.
A alternativa B está incorreta. De acordo com o texto, é correto afirmar que durante a pandemia,
a teleterapia ganhou destaque. Isso pode ser confirmado com o trecho “During the on-and-off
mandated social distancing that has marked the past 18 months of the pandemic, teletherapy has
shed its status as a minor form of care to become, at times, the only thing on offer”.
A alternativa C está incorreta. De acordo com o texto, é correto afirmar que o distanciamento
social acabou afetando a forma como as pessoas tratam sua saúde mental. Isso pode ser
confirmado com o trecho “...teletherapy has shed its status as a minor form of care to become, at
times, the only thing on offer”.
A alternativa D está incorreta. De acordo com o texto, é correto afirmar que pessoas que usam
teleterapia já estão acostumadas com a nova realidade. Isso pode ser confirmado com o trecho
“The popularity of remote therapeutic sessions has soared in the US and the “users” we once called
patients are increasingly comfortable with such practices, and in some cases even prefer them”.
A alternativa E está incorreta. De acordo com o texto, é correto afirmar que existem inovações
recentes sendo aplicadas ao momento da terapia. Isso pode ser confirmado com o trecho “Having
a therapist see a patient on Zoom or condensing treatment to self-tracking and AI interfaces may
be recent innovations…”.
GABARITO: A

QUESTÃO 06 (COLÉGIO NAVAL/INÉDITA) – Read the extract from the text


“Remote treatment is touted as an efficient way to reach more patients in a time of extreme
difficulty, an intimate intervention that can scale”

Mark the option that can replace the underlined sentence without changing its meaning
(A) Remote treatment is an efficient way to reach more people
(B) Remote treatment is publicized as an effective way to reach more people
(C) Remote treatment is criticized for being an efficient way to reach more patients
(D) Remote treatment is not considered an efficient way to reach more patients
(E) Remote treatment is not an efficient way to reach more people
Comentários: A alternativa A está incorreta. A frase “O tratamento remoto é apontado como uma
forma eficiente de alcançar mais pacientes” não pode ser substituída por “O tratamento remoto
é uma forma eficiente de alcançar mais pessoas”, mas sim, por “O tratamento remoto é divulgado
como uma forma eficaz de alcançar mais pessoas”.
A alternativa B está correta. A frase “O tratamento remoto é apontado como uma forma eficiente
de alcançar mais pacientes” pode ser substituída por “O tratamento remoto é divulgado como
uma forma eficaz de alcançar mais pessoas”, assim como esta opção indica.

AULA 08 – ABBREVIATIONS, CONTRACTIONS AND IF CLAUSES 67


TEACHER ANDREA BELO

A alternativa C está incorreta. A frase “O tratamento remoto é apontado como uma forma
eficiente de alcançar mais pacientes” não pode ser substituída por “O tratamento remoto é
criticado por ser uma forma eficiente de atingir mais pacientes”, mas sim, por “O tratamento
remoto é divulgado como uma forma eficaz de alcançar mais pessoas”.
A alternativa D está incorreta. A frase “O tratamento remoto é apontado como uma forma
eficiente de alcançar mais pacientes” não pode ser substituída por “O tratamento remoto não é
considerado uma forma eficiente de atingir mais pacientes”, mas sim, por “O tratamento remoto
é divulgado como uma forma eficaz de alcançar mais pessoas”.
A alternativa E está incorreta. A frase “O tratamento remoto é apontado como uma forma eficiente
de alcançar mais pacientes” não pode ser substituída por “O tratamento remoto não é uma forma
eficiente de atingir mais pessoas”, mas sim, por “O tratamento remoto é divulgado como uma
forma eficaz de alcançar mais pessoas”.
GABARITO: B

QUESTÃO 07 (COLÉGIO NAVAL/INÉDITA) – Read the extract from the text


“…and in some cases even prefer them” (paragraph 2).

The word “them” refers to


(A) Users
(B) Patients
(C) Remote therapeutic sessions
(D) Some cases
(E) The US
Comentários: O pronome “them” se refere às sessões terapêuticas remotas, que são citadas
anteriormente na frase, ou seja, “The popularity of remote therapeutic sessions has soared in the
US and the ‘users’ we once called patients are increasingly comfortable with such practices, and
in some cases even prefer them”.
GABARITO: C

QUESTÃO 08 (COLÉGIO NAVAL/INÉDITA) – Remote therapeutic sessions


(A) Have not gained popularity among the US population
(B) Are not relying on the help of recent innovations
(C) Were never the only therapeutic treatment option during the pandemic
(D) Have already gained the preference of some users/patients
(E) Were less sought after during the period of social isolation
Comentários: A alternativa A está incorreta. Não é correto afirmar que as sessões terapêuticas
remotas não ganharam popularidade entre a população dos EUA, mas sim, que ganharam. Isso

AULA 08 – ABBREVIATIONS, CONTRACTIONS AND IF CLAUSES 68


TEACHER ANDREA BELO

pode ser confirmado com o trecho “The popularity of remote therapeutic sessions has soared in
the US and the ‘users’ we once called patients…”.
A alternativa B está incorreta. Não é correto afirmar que as sessões terapêuticas remotas não
estão contando com a ajuda de inovações recentes, mas sim, que estão. Isso pode ser confirmado
com o trecho “Having a therapist see a patient on Zoom or condensing treatment to self-tracking
and AI interfaces may be recent innovations…”.
A alternativa C está incorreta. Não é correto afirmar que as sessões terapêuticas remotas nunca
foram a única opção de tratamento terapêutico durante a pandemia, mas sim, que, algumas vezes,
foram sim. Isso pode ser confirmado com o trecho “...teletherapy has shed its status as a minor
form of care to become, at times, the only thing on offer”.
A alternativa D está correta. É correto afirmar que as sessões terapêuticas remotas já
conquistaram a preferência de alguns usuários / pacientes, assim como esta opção indica. Isso
pode ser confirmado com o trecho “…are increasingly comfortable with such practices, and in
some cases even prefer them”.
A alternativa E está incorreta. Não é correto afirmar que as sessões terapêuticas remotas foram
menos procuradas durante o período de isolamento social, mas sim, que ganharam popularidade.
Isso pode ser confirmado com o trecho “The popularity of remote therapeutic sessions has soared
in the US and the ‘users’ we once called patients…”.
GABARITO: D

QUESTÃO 09 (COLÉGIO NAVAL/INÉDITA) – The expression “on-and-off”, in paragraph 2, means


(A) Continuous
(B) Unique
(C) Endless
(D) Frequent
(E) Intermittent
Comentários: A alternativa A está incorreta. A expressão “on-and-off” não significa algo contínuo,
mas sim, algo intermitente.
A alternativa B está incorreta. A expressão “on-and-off” não significa algo único, mas sim, algo
intermitente.
A alternativa C está incorreta. A expressão “on-and-off” não significa sem fim, mas sim, algo
intermitente.
A alternativa D está incorreta. A expressão “on-and-off” não significa algo frequente, mas sim,
algo intermitente.
A alternativa E está correta. A expressão “on-and-off” significa algo intermitente, assim como esta
opção indica.
GABARITO: E

AULA 08 – ABBREVIATIONS, CONTRACTIONS AND IF CLAUSES 69


TEACHER ANDREA BELO

QUESTÃO 10 (COLÉGIO NAVAL/INÉDITA) – The word “current” (paragraph 1), can be replaced by
the word ___ without changing its meaning
(A) Ongoing
(B) Actual
(C) Uncommon
(D) Temporary
(E) Permanent
Comentários: A alternativa A está correta. A palavra “current” significa “atual” e pode ser
substituída pela palavra “ongoing”, que significa “em andamento/atual”.
A alternativa B está incorreta. A palavra “current” significa “atual” e não pode ser substituída pela
palavra “actual”, que significa “real”.
A alternativa C está incorreta. A palavra “current” significa “atual” e não pode ser substituída pela
palavra “uncommon”, que significa “incomum”.
A alternativa D está incorreta. A palavra “current” significa “atual” e não pode ser substituída pela
palavra “temporary”, que significa “temporário”.
A alternativa E está incorreta. A palavra “current” significa “atual” e não pode ser substituída pela
palavra “permanent”, que significa “permanente”.
GABARITO: A

QUESTÕES EAM
Read text I and answer questions 01 and 02
Parosmia: 'The smells and tastes we still miss, long after Covid'
Back in November I realised my chicken pasta tasted like washing-up liquid. I haven't eaten meat
since - mac and cheese, green grapes and baby rusks have become my staples.
I don't know whether I will ever be able to enjoy a Nando's medium-spiced chicken butterfly again
as now it smells and tastes foul, like something alien. I have struggled to come to terms with this.
If my smell goes back to normal, I'll never ever take a Nando's with friends for granted again.

Chanay Knight, 21, Birmingham


Caught Covid in October, developed parosmia in November I miss a simple pleasure - breakfast in
bed brought to me every morning by the husband I love. Deeply aromatic coffee with hot, frothy,
milk. Good bread, crisp and deep gold, slightly charred at the edges with butter or tangy
marmalade.
In spring we both caught Covid and he was hospitalised. I struggled down to the kitchen to make
coffee and toast for myself. Exhausted by such a simple task, I clung to the ritual and pictured him
beside me.

AULA 08 – ABBREVIATIONS, CONTRACTIONS AND IF CLAUSES 70


TEACHER ANDREA BELO

We both recovered, but coffee and toast is now repulsive to me - like a field just sprayed with
manure… unpleasant with a sweet fermented smell on top.
I've lost something that meant so much more to me than just breakfast.
Wendy Thompson, 59, Tameside
Caught Covid in May, developed parosmia in October
(Adapted from https://www.bbc.com/news/stories-55936729)

QUESTÃO 01 (EAM/INÉDITA) – About the reports given in the text, it is not to say that
(A) COVID-19 has no long-term impacts
(B) COVID-19 can change a person's lifestyle in the long run
(C) Some simple pleasures were maintained even with the loss of smell and taste
(D) COVID-19 is only a transient and extreme disease, which is asymptomatic or causes death
(E) The reports were not based on individual experiences
Comentários: A alternativa A está incorreta. Sobre os relatos dados no texto, não é correto
afirmar que a COVID-19 não tem impactos de longo prazo, mas sim, que tem. Isso pode ser
confirmado com o trecho “If my smell goes back to normal, I'll never ever take a Nando's with
friends for granted again”.
A alternativa B está correta. Sobre os relatos dados no texto, é correto afirmar que a COVID-19
pode mudar o estilo de vida de uma pessoa a longo prazo, assim como esta opção indica. Isso
pode ser confirmado com o trecho “I've lost something that meant so much more to me than just
breakfast”.
A alternativa C está incorreta. Sobre os relatos dados no texto, não é correto afirmar que alguns
prazeres simples foram mantidos mesmo com a perda do olfato e do paladar, mas sim, que não
foram mantidos. Isso pode ser confirmado com o trecho “I miss a simple pleasure - breakfast in
bed brought…”.
A alternativa D está incorreta. Sobre os relatos dados no texto, não é correto afirmar que a COVID-
19 é apenas uma doença transitória e extrema, que é assintomática ou causa a morte, mas sim,
que pode causar sequelas extremas, como a perda de olfato. Isso pode ser confirmado com o
trecho “If my smell goes back to normal, I'll never ever take a Nando's with friends for granted
again”.
A alternativa E está incorreta. Sobre os relatos dados no texto, não é correto afirmar que os relatos
não foram baseados em experiências individuais, mas sim, que foram. Isso pode ser confirmado
com o trecho “In spring we both caught Covid and he was hospitalised. I struggled down…”.
GABARITO: B

AULA 08 – ABBREVIATIONS, CONTRACTIONS AND IF CLAUSES 71


TEACHER ANDREA BELO

QUESTÃO 02 (EAM/INÉDITA) – In the sentence “In spring we both caught Covid and he was
hospitalised. I struggled down to the kitchen to make coffee and toast for myself” (paragraph 5),
the pronoun “we” refers to
(A) Wendy Thompson
(B) Chanay Knight
(C) Wendy and Chanay
(D) Wendy and her family
(E) Wendy and her husband
Comentários: A alternativa A está incorreta. O pronome “we” (nós) não se refere à Wendy
Thompson, mas sim, à Wendy e seu marido, que é citado anteriormente. Isso pode ser confirmado
com o trecho “...by the husband I love ... In spring we both caught Covid and…”.
A alternativa B está incorreta. O pronome “we” (nós) não se refere à Chanay Knight, mas sim, à
Wendy e seu marido, que é citado anteriormente. Isso pode ser confirmado com o trecho “...by
the husband I love ... In spring we both caught Covid and…”.
A alternativa C está incorreta. O pronome “we” (nós) não se refere à Wendy e Chanay, mas sim,
à Wendy e seu marido, que é citado anteriormente. Isso pode ser confirmado com o trecho “...by
the husband I love ... In spring we both caught Covid and…”.
A alternativa D está incorreta. O pronome “we” (nós) não se refere à Wendy e sua família, mas
sim, à Wendy e seu marido, que é citado anteriormente. Isso pode ser confirmado com o trecho
“...by the husband I love ... In spring we both caught Covid and…”.
A alternativa E está correta. O pronome “we” (nós) se refere à Wendy e seu marido, que é citado
anteriormente, assim como esta opção indica. Isso pode ser confirmado com o trecho “...by the
husband I love ... In spring we both caught Covid and…”.
GABARITO: E

Read text II and answer questions 03 and 04


What can we take with us from lockdown to make life better than before?
Spring has sprung, albeit a little early. The weekend brought blue skies, daffodils bursting into
flower, the drone of lawnmowers coming back to life. The natural world is waking up fast, in
uncanny synchronicity with a nation now readying itself to emerge from Covid hibernation. Like
all jaded hacks, I rolled my eyes when the prime minister started waxing lyrical last week about
the crocus of hope pushing up through the frost of lockdown. But dammit, the crocuses are out
now, and if this strangely unfamiliar emotion is not hope, then it’s hard to know what else it is.
The coming of spring always paves the way, on some atavistic level, for the imagining of summer.
But this time the vaccine (touch wood, cross fingers, pray that the missing unknown carrier of the
Brazilian variant is tracked down before spreading it) provides some more rational grounds for
daring to imagine new beginnings.
The only way to keep one’s sanity in lockdown is to take life one day at a time, like an alcoholic in
recovery, and never think about how much longer there is to go. But suddenly it seems possible

AULA 08 – ABBREVIATIONS, CONTRACTIONS AND IF CLAUSES 72


TEACHER ANDREA BELO

to stop dwelling on everything that’s out of bounds and imagine having choices again. Travel
agents are reporting a rush of possibly over-optimistic bookings; fashion magazines are suddenly
full of heels and frivolousness, rather than clothes for collapsing on the sofa in. And for parents
who’ve been working from home, hunting down the PE kit ahead of next week’s return to school
is a reminder that one day our own presence may be required in a formal office again.
(Adapted from https://www.theguardian.com/commentisfree/2021/mar/01/covid-life-flexible-working-lockdown)

QUESTÃO 03 (EAM/INÉDITA) – Say if the following statements are T (true) or F (false) about
lockdown and its developments. Then, mark the correct option, from top to bottom.
( ) Imagining new beginnings is more complex in the context of a pandemic
( ) Superstitions are not accepted as a way to alleviate the near future
( ) The vaccine makes us imagine new beginnings
( ) A long-term vision for an end to the pandemic is the only way to tackle it
( ) "Normal life" is still out of the question in our imagination
(A) T – F – T – F – F
(B) T – F – T – T – F
(C) F – F – T – F – F
(D) T – T – T – F – T
(E) F – T – F – T – F
Comentários: A primeira afirmativa é verdadeira (true – T). Sobre o lockdown e seus
desdobramentos, é correto afirmar que imaginar novos começos é mais complexo no contexto
de uma pandemia. Isso pode ser confirmado com o trecho “…this time the vaccine provides some
more rational grounds for daring to imagine new beginnings…”.
A segunda afirmativa é falsa (false – F). Sobre o lockdown e seus desdobramentos, não é correto
afirmar que superstições não são aceitas como forma de amenizar o futuro próximo, mas sim,
que são. Isso pode ser confirmado com o trecho “But this time the vaccine (touch wood, cross
fingers, pray that the missing unknown carrier of the Brazilian variant is tracked down before
spreading it) …”.
A terceira afirmativa é verdadeira (true – T). Sobre o lockdown e seus desdobramentos, é correto
afirmar que a vacina nos faz imaginar novos começos. Isso pode ser confirmado com o trecho
“…this time the vaccine provides some more rational grounds for daring to imagine new
beginnings…”.
A quarta afirmativa é falsa (false – F). Sobre o lockdown e seus desdobramentos, não é correto
afirmar que uma visão de longo prazo para o fim da pandemia é a única maneira de enfrentá-la,
mas sim, que a única forma de a enfrentar, é um dia de cada vez. Isso pode ser confirmado com
o trecho “The only way to keep one’s sanity in lockdown is to take life one day at a time…”.
A quinta afirmativa é falsa (false – F). Sobre o lockdown e seus desdobramentos, não é correto
afirmar que "Vida normal" ainda está fora de questão em nossa imaginação, mas sim, que já é

AULA 08 – ABBREVIATIONS, CONTRACTIONS AND IF CLAUSES 73


TEACHER ANDREA BELO

possível imaginar novamente. Isso pode ser confirmado com o trecho “But suddenly it seems
possible to stop dwelling on everything that’s out of bounds, and imagine having choices again”.
GABARITO: A

QUESTÃO 04 (EAM/INÉDITA) – “And for parents who’ve been working from home…”
The word “who” refers to:
(A) Children
(B) Someone unknown
(C) Travel agents
(D) Parents
(E) Fashion magazines
Comentários: A alternativa A está incorreta. A palavra “who” não se refere as crianças, mas sim,
aos pais (parentes). Isso pode ser confirmado com o trecho dado pela questão, “E para os pais
que trabalham em casa...”.
A alternativa B está incorreta. A palavra “who” não se refere a alguém desconhecido, mas sim,
aos pais (parentes). Isso pode ser confirmado com o trecho dado pela questão, “E para os pais
que trabalham em casa...”.
A alternativa C está incorreta. A palavra “who” não se refere a agentes de viagem, mas sim, aos
pais (parentes). Isso pode ser confirmado com o trecho dado pela questão, “E para os pais que
trabalham em casa...”.
A alternativa D está correta. A palavra “who” se refere aos pais (parentes), assim como esta opção
indica. Isso pode ser confirmado com o trecho dado pela questão, “E para os pais que trabalham
em casa...”.
A alternativa E está incorreta. A palavra “who” não se refere as revistas de moda, mas sim, aos
pais (parentes). Isso pode ser confirmado com o trecho dado pela questão, “E para os pais que
trabalham em casa...”.
GABARITO: D

QUESTÃO 05 (EAM/INÉDITA) – Read the sentences and mark the correct options to fill in the
blanks respectively
Pedro is _______ older brother. _______ lives on the same street as us. We love to walk
together. _______ sneakers are blue like mine.
(A) Mine / He / His
(B) My / He / His
(C) My / They / Mine
(D) My / He / Mine
(E) Mine / We / His

AULA 08 – ABBREVIATIONS, CONTRACTIONS AND IF CLAUSES 74


TEACHER ANDREA BELO

Comentários: A primeira lacuna deve ser preenchida com “my” (meu), pois a frase se refere ao
autor indicando que Pedro é seu irmão, ou seja, “Pedro é meu irmão mais velho” (Pedro is my
older brother).
A segunda lacuna deve ser preenchida com “he” (ele), pois a frase se refere ao Pedro que mora
na mesma rua que o autor e sua família, ou seja, “He lives on the same...” (Ele mora na mesma...).
A terceira lacuna deve ser preenchida com “his” (dele), pois a frase se refere ao tênis de Pedro,
ou seja, “His sneakers are blue...” (O tênis dele é azul...).
GABARITO: B

QUESTÃO 06 (EAM/INÉDITA) – What sports can you see in the picture below?

(Adapted from https://pt.dreamstime.com/ilustra%C3%A7%C3%A3o-stock-crian%C3%A7as-e-projeto-dos-esportes-image90259764)

(A) Golf, cycling, and baseball


(B) Basketball, tennis and canoeing
(C) Soccer, volleyball and basketball
(D) Badminton, volleyball and golf
(E) Canoeing, skating and tennis
Comentários: A alternativa A está incorreta. Golf não é praticado na imagem, ciclismo é praticado
na imagem e baseball não é praticado na imagem.
A alternativa B está incorreta. Basquete é praticado na imagem, tênis não é praticado na imagem
e canoagem não é praticado na imagem.
A alternativa C está correta. Futebol é praticado na imagem, vôlei é praticado na imagem e
basquete é praticado na imagem.
A alternativa D está incorreta. Badminton não é praticado na imagem, vôlei é praticado na
imagem e golf não é praticado na imagem.
A alternativa E está incorreta. Canoagem não é praticado na imagem, patinação é praticado na
imagem e tênis não é praticado na imagem.
GABARITO: C

AULA 08 – ABBREVIATIONS, CONTRACTIONS AND IF CLAUSES 75


TEACHER ANDREA BELO

QUESTÃO 07 (EAM/INÉDITA) – Look at the picture below

(Adapted from https://br.freepik.com/vetores-premium/dois-garotinhos-com-mascaras-medicas-jogando-futebol-protecao-contra-virus-


conceito-dealergias-criancas-no-campo-de-futebol-ilustracao-plana-personagem-de-desenho-animado-esporte-e-recreacao_8207330.htm)

What are they doing in the picture?


(A) The children are playing soccer now
(B) The children play soccer everyday
(C) The children were playing soccer yesterday
(D) The children are not playing soccer
(E) The children didn’t play soccer yesterday
Comentários: A alternativa A está correta. É correto afirmar que as crianças estão jogando futebol
agora, assim como esta opção indica.
A alternativa B está incorreta. Não é correto afirmar que as crianças jogam futebol todos os dias,
mas sim, que as crianças estão jogando futebol agora.
A alternativa C está incorreta. Não é correto afirmar que as crianças estavam jogando futebol
ontem, mas sim, que as crianças estão jogando futebol agora.
A alternativa D está incorreta. Não é correto afirmar que as crianças não estão jogando futebol,
mas sim, que as crianças estão jogando futebol agora.
A alternativa E está incorreta. Não é correto afirmar que as crianças não jogaram futebol ontem,
mas sim, que as crianças estão jogando futebol agora.
GABARITO: A

AULA 08 – ABBREVIATIONS, CONTRACTIONS AND IF CLAUSES 76


TEACHER ANDREA BELO

QUESTÃO 08 (EAM/INÉDITA) – Read the text below

(Adapted from http://ritasesl4u.pbworks.com/w/page/34378444/Grammar-Present%20Tenses)

In the cartoon, the future tense is used to express


(A) A plan
(B) A confirmation
(C) An ad
(D) A possibility
(E) A prediction
Comentários: A alternativa A está incorreta. A frase “o mundo vai acabar amanhã” não expressa
um plano, mas sim, uma previsão, que pode ser considerada errônea já que ele está segurando o
cartaz por 15 anos.
A alternativa B está incorreta. A frase “o mundo vai acabar amanhã” não expressa uma
confirmação, mas sim, uma previsão, que pode ser considerada errônea já que ele está segurando
o cartaz por 15 anos.
A alternativa C está incorreta. A frase “o mundo vai acabar amanhã” não expressa um anúncio,
mas sim, uma previsão, que pode ser considerada errônea já que ele está segurando o cartaz por
15 anos.
A alternativa D está incorreta. A frase “o mundo vai acabar amanhã” não expressa uma
possibilidade, pois não tem como afirmar que há uma possibilidade; mas sim, uma previsão, que
pode ser considerada errônea já que ele está segurando o cartaz por 15 anos.
A alternativa E está correta. A frase “o mundo vai acabar amanhã” expressa uma previsão, que
pode ser considerada errônea já que ele está segurando o cartaz por 15 anos; assim como esta
opção indica.
GABARITO: E

AULA 08 – ABBREVIATIONS, CONTRACTIONS AND IF CLAUSES 77


TEACHER ANDREA BELO

Read text II and answer questions 09 and 10


Coronavirus crisis unlikely to be over by the end of the year, WHO warns
Despite the spread of Covid-19 being slowed in some countries due to lockdowns and vaccination
programs, it is “premature” and “unrealistic” to the think the pandemic will be over by the end of
the year, the World Health Organization’s executive director of emergency services has said.
Speaking at a press briefing Geneva, Dr Michael Ryan said while vaccinating the most vulnerable
people, including healthcare workers, would help remove the “tragedy and fear” from the
situation, and would help to ease pressure on hospitals, the “virus is very much in control”.
“It will be very premature, and I think unrealistic, to think that we’re going to finish with this virus
by the end of the year,” Ryan said.
“If the vaccines begin to impact not only on death and not only on hospitalisation, but have a
significant impact on transmission dynamics and transmission risk, then I believe we will
accelerate toward controlling this pandemic.”
(Adapted from https://www.theguardian.com/world/2021/mar/02/coronavirus-crisis-unlikely-to-be-over-by-the-end-of-the-year-who-warns)

QUESTÃO 09 (EAM/INÉDITA) – In the sentence “It will be very premature, and I think unrealistic,
to think that we’re going to finish with this virus by the end of the year” (paragraph 2), the
pronoun “I” refers to
(A) Michel Ryan
(B) The world population
(C) The World Health Organization
(D) Vaccination
(E) Pandemic
Comentários: A alternativa A está correta. O pronome “I” (eu) se refere a Michel Ryan, assim
como esta opção indica. Isso pode ser confirmado com o trecho “’It will be very premature, and I
think unrealistic, to think that we’re going to finish with this virus by the end of the year,’ Ryan
said”.
A alternativa B está incorreta. O pronome “I” (eu) não se refere à população munidal, mas sim, a
Michel Ryan. Isso pode ser confirmado com o trecho “’It will be very premature, and I think
unrealistic, to think that we’re going to finish with this virus by the end of the year,’ Ryan said”.
A alternativa C está incorreta. O pronome “I” (eu) não se refere à Organização Mundial da Saúde,
mas sim, a Michel Ryan. Isso pode ser confirmado com o trecho “’It will be very premature, and I
think unrealistic, to think that we’re going to finish with this virus by the end of the year,’ Ryan
said”.
A alternativa D está incorreta. O pronome “I” (eu) não se refere a vacinação, mas sim, a Michel
Ryan. Isso pode ser confirmado com o trecho “’It will be very premature, and I think unrealistic,
to think that we’re going to finish with this virus by the end of the year,’ Ryan said”.
A alternativa E está incorreta. O pronome “I” (eu) não se refere a pandemia, mas sim, a Michel
Ryan. Isso pode ser confirmado com o trecho “’It will be very premature, and I think unrealistic,
to think that we’re going to finish with this virus by the end of the year,’ Ryan said”.
GABARITO: A

AULA 08 – ABBREVIATIONS, CONTRACTIONS AND IF CLAUSES 78


TEACHER ANDREA BELO

QUESTÃO 10 (EAM/INÉDITA) – It is FALSE to say that


(A) The spread of coronavirus is being partially contained in some places
(B) Considering vaccination and lockdown, the pandemic is likely to end in a few months
(C) Most vulnerable people should be a priority in vaccination
(D) It is misleading to think that the pandemic will end by the end of the year
(E) Decreasing numbers of death and hospitalization are not the only thing to think about to end
the pandemic
Comentários: A alternativa A está incorreta. É verdadeiro dizer que a disseminação do coronavírus
está sendo parcialmente contida em alguns lugares. Isso pode ser confirmado com o trecho
“Despite the spread of Covid-19 being slowed in some countries due to lockdowns and vaccination
programs ...”.
A alternativa B está correta. É falso dizer que considerando a vacinação e o bloqueio, a pandemia
provavelmente terminará em alguns meses, mas sim, que é irreal pensar que a pandemia vai
acabar até o fim do ano. Isso pode ser confirmado com o trecho “It will be very premature, and I
think unrealistic, to think that we’re going to finish with this virus by the end of the year”.
A alternativa C está incorreta. É verdadeiro dizer que pessoas mais vulneráveis devem ser uma
prioridade na vacinação. Isso pode ser confirmado com o trecho “...said while vaccinating the
most vulnerable people, including healthcare workers, would help remove the…”.
A alternativa D está incorreta. É verdadeiro dizer que é enganoso pensar que a pandemia
terminará no final do ano. Isso pode ser confirmado com o trecho “…it is ‘premature’ and
‘unrealistic’ to the think the pandemic will be over by the end of the year…”.
A alternativa E está incorreta. É verdadeiro dizer que a diminuição do número de mortes e
hospitalizações não é a única coisa a se pensar para acabar com a pandemia. Isso pode ser
confirmado com o trecho “If the vaccines begin to impact not only on death and not only on
hospitalisation but have a significant impact on transmission dynamics and transmission risk, then
I believe we will accelerate toward controlling this pandemic”.
GABARITO: B

QUESTÕES EEAR
Read the text and answer questions 01 to 05.
‘We are on the eve of a genocide’: Brazil urged to save Amazon from Covid-19
Brazil’s leaders must take immediate action to save the country’s indigenous peoples from a
Covid-19 “genocide”, a global coalition of artists, celebrities, scientists, and intellectuals has said.
In an open letter to Brazil, figures including Madonna, Oprah Winfrey, Brad Pitt, David Hockney
and Paul McCartney warned the pandemic meant indigenous communities in the Amazon faced
“an extreme threat to their very survival”.
“Five centuries ago, these ethnic groups were decimated by diseases brought by European
colonisers … Now, with this new scourge spreading rapidly across Brazil … [they] may disappear
completely since they have no means of combating Covid-19,” they wrote.

AULA 08 – ABBREVIATIONS, CONTRACTIONS AND IF CLAUSES 79


TEACHER ANDREA BELO

The organiser of the petition, the Brazilian photojournalist Sebastião Salgado, said trespassers
including wildcat gold miners and illegal loggers must be expelled immediately from indigenous
lands to stop them importing an illness that has killed more than 240,000 people around the
world, including 6,750 in Brazil. “We are on the eve of a genocide,” Salgado, who has spent nearly
four decades documenting the Amazon and its inhabitants, told the Guardian.
Even before Covid-19, Brazil’s indigenous peoples were locked in what activists call a historic
struggle for survival. Fears Covid-19 could devastate indigenous communities grew last month
when the death of a Yanomami teenager revived horrific memories of epidemics caused by
roadbuilders and gold prospectors in the 1970s and 80s. The Brazilian city so far worst hit by
coronavirus is Manaus, the capital of Amazonas, the state where part of the Yanomami reserve is
located.
Salgado – who is calling for the creation of an army-led taskforce to evict intruders from protected
areas – believes international pressure could force the government to act.
Adapted from https://www.theguardian.com/world/2020/may/03/eve-of-genocide-brazil-urged-save-amazon-tribes-covid-19-sebastiao-salgado

QUESTÃO 01 (EEAR/INÉDITA) – What’s the active voice for “... these ethnic groups were
decimated by diseases brought by European colonisers …” (paragraph 2)?
(A) Diseases brought by European colonisers have decimated these ethnic groups.
(B) Diseases brought by European colonisers are decimating these ethnic groups.
(C) Diseases brought by European colonisers will decimate these ethnic groups.
(D) Diseases brought by European colonisers were decimating these ethnic groups.
(E) Diseases brought by European colonisers decimated these ethnic groups.
Comentários: Ao passar da voz passiva para a voz passiva (ou vice-versa), é muito importante
preservar o tempo verbal. No trecho trazido pela questão, temos a estrutura verbal “were
decimated” - foram dizimados, no passado. Assim, temos que buscar nas alternativas a que esteja
com a voz ativa no Simple Past.
a) Diseases brought by European colonisers have decimated these ethnic groups. Have
decimated- Present Perfect. ERRADA.
b) Diseases brought by European colonisers are decimating these ethnic groups. Are decimating-
Present Continuous. ERRADA.
c) Diseases brought by European colonisers will decimate these ethnic groups. Will decimate-
Future. ERRADA.
d) Diseases brought by European colonisers were decimating these ethnic groups. Were
decimating- Past Continuous. ERRADA.
e) Diseases brought by European colonisers decimated these ethnic groups. Decimated- Simple
Past. CORRETA.
GABARITO: E

AULA 08 – ABBREVIATIONS, CONTRACTIONS AND IF CLAUSES 80


TEACHER ANDREA BELO

QUESTÃO 02 (EEAR/INÉDITA) – In “‘Now, with this new scourge spreading rapidly across Brazil
… [they] may disappear completely since they have no means of combating Covid-19’, they
wrote.”, the underlined words respectively refer to
(A) these ethnic groups / international figures.
(B) diseases / international figures.
(C) these ethnic groups / these ethnic groups.
(D) European colonisers / these ethnic groups.
(E) European colonisers / international figures.
Comentários: Nesse tipo de questão, é essencial voltar ao texto para entender a quais palavras
ou expressões os termos sublinhados se referem.
“Five centuries ago, these ethnic groups were decimated by diseases brought by European
colonisers … Now, with this new scourge spreading rapidly across Brazil … [they] may disappear
completely since they have no means of combating Covid-19,” they wrote.
Cinco séculos atrás, esses grupos étnicos foram dizimados por doenças trazidas pelos
colonizadores europeus ... Agora, com esse novo flagelo se espalhando rapidamente pelo Brasil
... [eles- os grupos índigenas mencionados anteriormente] podem desaparecer completamente,
pois não têm como combater o Covid-19”, escreveram eles- as figuras internacionais
mencionadas no parágrafo anterior que escreveram uma carta aberta destinada ao Brasil e seus
líderes.
Assim, o primeiro they = these ethinc groups e o segundo they = international figures.
a) these ethnic groups / international figures. Conforme análise acima, CORRETA.
b) diseases / international figures. Conforme análise acima, ERRADA.
c) these ethnic groups / these ethnic groups. Apenas o primeiro they se refere a these ethnic
groups. ERRADA.
d) European colonisers / these ethnic groups. Conforme análise acima, ERRADA.
e) European colonisers / international figures. Conforme análise acima, ERRADA.
GABARITO: A

QUESTÃO 03 (EEAR/INÉDITA) – In the sentence “We are on the eve of a genocide...” (paragraph
3)., the expression, in bold type, is closest in meaning to _____________.
(A) much ahead of.
(B) a little after.
(C) about to have.
(D) at the night of.
(E) in the middle of.
Comentários: a) much ahead of. Essa expressão transmite a ideia de que estamos muito à frente
de um genocídio. ERRADA.

AULA 08 – ABBREVIATIONS, CONTRACTIONS AND IF CLAUSES 81


TEACHER ANDREA BELO

b) a little after. A little after significa um pouco depois. ERRADA.


c) about to have. A expressão “on the eve of” significa na véspera de. Ou seja, de acordo com o
trecho do texto em que foi usada, afirma-se que estamos na véspera de um genocídio, ou seja,
um genocídio está prestes a acontecer. Dizer que we are about to have a genocide transmite
exatamente essa ideia, de que algo está prestes a acontecer (about to). CORRETA.
d) at the night of. Apesar de em outros contextos comumente eve ser traduzida como noite-
como, por exemplo, em Christmas Eve e em New Year’s Eve, na verdade, porque a palavra se
refere à noite de véspera, no contexto da questão não tem esse sentido literal de noite, fazendo
apenas uma referência a algo que está prestes a acontecer. ERRADA.
e) in the middle of. No meio de- como vimos, está prestes a acontecer, não no meio. ERRADA.
GABARITO: C

QUESTÃO 04 (EEAR/INÉDITA) – The words “is calling”, underlined in the text, form a verb in the
__________.
(A) simple past
(B) future tense
(C) simple present
(D) present progressive
(E) present perfect
Comentários: As palavras “is calling (verbo to be no presente + verbo-ING)”, sublinhadas no texto,
formam um verbo no Present Continuous ou Present Progressive, indicando que a ação está
acontecendo de forma contínua, progressiva, nesse momento.
“Salgado – who is calling for the creation of an army-led taskforce to evict intruders from
protected areas – believes international pressure could force the government to act.” = Salgado -
que está pedindo a criação de uma força-tarefa liderada pelo exército para expulsar intrusos de
áreas protegidas - acredita que a pressão internacional pode forçar o governo a agir.
GABARITO: D

QUESTÃO 05 (EEAR/INÉDITA) – According to the text, Brazilian indigenous peoples are _______.
(A) endangered.
(B) decimated.
(C) secure.
(D) immune.
(E) unharmed.

AULA 08 – ABBREVIATIONS, CONTRACTIONS AND IF CLAUSES 82


TEACHER ANDREA BELO

Comentários: Segundo o texto, os povos indígenas brasileiros estão _______.


a) em perigo.
b) dizimados.
c) seguros.
d) imunes.
e) ilesos.
“Brazil’s leaders must take immediate action to save the country’s indigenous peoples from a
Covid-19 ‘genocide’” = Os líderes do Brasil devem tomar medidas imediatas para salvar os povos
indígenas do país de um “genocídio” por Covid-19.
GABARITO: A

Read the cartoon and answer questions 06 to 08.

Adapted from https://www.dailyrepublic.com/comics/beetle-bailey-245/attachment/bbt20130313/.

QUESTÃO 06 (EEAR/INÉDITA) – In the cartoon, the sentence “Clean this office right” refers to
(A) the future tense.
(B) the imperative mood.
(C) the indicative mood.
(D) the present continuous tense.
(E) the past tense.
Comentários: a) the future tense. “Clean this office right” é um comando, uma ordem. Significa
“limpe esse escritório direito”. Está no modo imperativo, não no tempo futuro. ERRADA.
b) the imperative mood. O modo imperativo é uma forma verbal que dá um comando. A frase
trazida pela questão se encaixa nesse padrão, como vimos acima. Uma dica para você identificar

AULA 08 – ABBREVIATIONS, CONTRACTIONS AND IF CLAUSES 83


TEACHER ANDREA BELO

o imperativo facilmente é que ele não tem um sujeito explícito, o que é muito raro em Inglês.
O comando começa direto no verbo → CLEAN this office right. Se for uma ordem negativa,
vai começar com don’t + verbo. (Exemplo: don’t clean this office). CORRETA
c) the indicative mood. O modo indicativo é um modo verbal que expressa um fato, uma certeza.
Como vimo, a frase em destaque não indica um fato, mas, sim, uma ordem um comando, e se
refere ao modo imperativo. ERRADA.
d) the present continuous tense. O tempo Present Continuous indica que uma ação está
acontecendo, de forma contínua, no momento da fala. Sua característica marcante é a presença
do verbo to be no presente e do verbo com -ing. Não é o que temos no caso. ERRADA.
e) the past tense. A frase também não está no tempo passado. O verbo no passado pode ser
irregular ou, sendo regular, termina com -ed, geralmente. Como vimos, a frase está no modo
imperativo. ERRADA.
GABARITO: B

QUESTÃO 07 (EEAR/INÉDITA) – Choose the best alternative to complete the blank in the text.
(A) didn’t cleaned
(B) didn’t clean
(C) did cleaned
(D) don’t cleaned
(E) doesn’t cleaned
Comentários: a) didn’t cleaned. A letra A não está correta para preencher a lacuna. Quando
fazemos a negativa do Simple Past, usamos didn’t + o verbo em sua forma base, sem alterações.
Assim, o verbo não fica com a terminação -ed. ERRADA.
b) didn’t clean. Essa alternativa está correta, conforme a explicação acima. → Cookie sent him
here because he didn’t clean the mess hall right. = Cookie o mandou para cá porque ele não
limpou o refeitório corretamente. CORRETA.
c) did cleaned. No contexto da tirinha, faria sentido “ele não limpou”, conforme vimos na letra C.
Mas, de toda maneira, se quiséssemos afirmar, de forma enfática, no passado, e dizer que ele
limpou sim, poderíamos usar o auxiliar did, mas o verbo ficaria na sua forma base, sem -ed.
ERRADA.
d) don’t cleaned. Usamos don’t para negar no presente com os sujeitos I, you, we e they. No
passado, usamos didn’t + o verbo em sua forma base, sem alterações. “Don’t cleaned” é uma
estrutura que não existe. ERRADA.
e) doesn’t cleaned. Usamos doesn’t para negar no presente com os sujeitos he, she, e it. No
passado, usamos didn’t + o verbo em sua forma base, sem alterações. “Doesn’t cleaned” é uma
estrutura que não existe. ERRADA.
GABARITO: B

AULA 08 – ABBREVIATIONS, CONTRACTIONS AND IF CLAUSES 84


TEACHER ANDREA BELO

QUESTÃO 08 (EEAR/INÉDITA) – The word “him” is a / an __________ pronoun.


(A) subject
(B) reflexive
(C) demonstrative
(D) possessive
(E) object
Comentários: No trecho “Cookie sent him here...”, him funciona como objeto do verbo send =
Cookie o enviou aqui... Dicas sobre as alternativas:
a) Subject pronouns: I, he, she, it, we, you, they.
b) Reflexive pronouns: terminação -self (ou -selves no plural).
c) Demonstrative pronouns: this, that, these, those.
d) Possessive pronouns: mine, his, hers, its, ours, yours, theirs.
e) Object pronouns: me, him, her, it, us, you, them.
GABARITO: E

Read the text and answer questions 09 and 10.


The US Navy recorded three videos that show UFOs. A UFO is a flying object in the sky that
scientists cannot easily explain. Sometimes, UFOs are clouds or airplanes; however, many people
believe that they are spacecraft from other planets.
A UFO research group published the videos in 2018, and the US Navy said that the videos were
real. However, some people did not believe them. US officials decided to publish the videos again
because they wanted to show that the videos were real. The officials said that they could not
explain the origin of the objects in the videos. They consider the objects as unidentified.
Adapted from https://www.newsinlevels.com/.

QUESTÃO 09 (EEAR/INÉDITA) – According to the text,


(A) The videos the US Navy published were fake.
(B) The objects in the videos are clouds and airplanes.
(C) UFOs are unidentified flying objects.
(D) Everybody believed the videos the first time they were published.
(E) Many people believe UFOs are spacecraft from the Earth.

AULA 08 – ABBREVIATIONS, CONTRACTIONS AND IF CLAUSES 85


TEACHER ANDREA BELO

Comentários: De acordo com o texto,


a) The videos the US Navy published were fake. Algumas pessoas acharam que os vídeos eram
falsos, mas não eram- “US officials decided to publish the vídeos again because they wanted to
show that the videos were real.” = As autoridades americanas decidiram publicar os vídeos
novamente porque queriam mostrar que os vídeos eram reais.
b) The objects in the videos are clouds and airplanes. Os objetos nos vídeos foram declarados
como não identificados, e não como nuvens ou aviões. O texto fala que às vezes ocorre que o que
se acreditou ser um OVNI era uma nuvem ou um avião, mas não afirma que isso foi provado
para os Óvnis dos vídeos. “The officials said that they could not explain the origin of the objects
in the videos. They consider the objects as unidentified.”
c) UFOs are unidentified flying object. De fato, UFOs são objetos voadores não identificados (Óvnis
em Português). “A UFO is a flying object in the sky that scientists cannot easily explain.” = Um
OVNI é um objeto voador no céu que os cientistas não conseguem explicar facilmente.
d) everybody believed the videos the first time they were published. Nem todo mundo acreditou
nos vídeos na primeira vez em que eles foram publicados. “However, some people did not believe
them. US officials decided to publish the videos again because they wanted to show that the
videos were real.”
e) Many people believe UFOs are spacecraft from the Earth. De acordo com o texto, “many people
believe that they are spacecraft from other planets.” = muitas pessoas acreditam que são
espaçonaves de outros planetas (não da Terra, como diz a alternativa).
GABARITO: C

QUESTÃO 10 (EEAR/INÉDITA) – In the sentence “A UFO is a flying object in the sky that
scientists cannot easily explain”, cannot is used to express that scientists
(A) are prohibited to explain UFOs.
(B) are capable of explaining UFOs.
(C) are not allowed to explain UFOs.
(D) are unable to explain UFOs.
(E) are not confident about UFOs.
Comentários: a) are prohibited to explain UFOs. Essa alternativa diz que eles são proibidos de
explicar OVNIs. ERRADA.
b) are capable of explaining UFOs. Essa alternativa diz que eles são capazes, capable. ERRADA.
c) are not allowed to explain UFOs. Essa alternativa diz que eles não têm permissão para explicar
OVNIs. Cannot até pode transmitir a ideia de falta de permissão, mas não foi usado nesse sentido
no contexto. A ideia é de que eles não conseguem. ERRADA
d) are unable to explain UFOs. Cannot, que é o mesmo que can’t (can not) foi usado no trecho
para dizer que os cientistas são incapazes de explicar os OVNIs, não conseguem fazê-lo facilmente.
Unable = incapazes. CORRETA.

AULA 08 – ABBREVIATIONS, CONTRACTIONS AND IF CLAUSES 86


TEACHER ANDREA BELO

e) are not confident about UFOs. Essa alternativa diz que eles não estão confiantes sobre OVNIs.
Como vimos, não é essa a ideia que o cannot expressa. Não tem a ver com confiança, mas sim
com capacidade. ERRADA.
GABARITO: D

QUESTÕES EFOMM
Read Text I to do questions 01 to 04 based on it.
Vaccine passports are less a threat to liberty than a mark of solidarity
Anti-vaxxers in France and elsewhere claim personal freedom. But what of brotherhood?
In France over the past few weeks, the topic of vaccine passports has induced an avalanche of
outrage. Opposition to the measure has united both the hard left and right, with more than
200,000 people taking to the streets to express their contempt. In the kaleidoscope of disparate
groups involved, the only unifying banner is the assertion that Emmanuel Macron’s policy is an
infringement of the French tenet of liberté.
Nor is France unique in facing such resistance. In the United States, mask and vaccine mandates
have generated passionate opprobrium and legal action.
Those resolutely opposed to anything styled as a vaccination passport tend to frame the issue as
a solely personal choice. That can seem superficially reasonable, but it highlights a crucial
misunderstanding – a presumption that vaccination is solely an individual boon. On the contrary,
immunisation is, at heart, a public health measure, implemented to reduce incidence and burden
of disease at a population level. That it has huge individual benefit is undeniable but viewing
vaccination through this reductive, individualistic lens fundamentally distorts the reality that it is
about much more than protecting oneself.
Immunisation collectively reduces reservoirs for disease, providing a firewall that protects
vulnerable members of society. While a Covid infection might not do a young, healthy person
lasting harm, their passing on that infection could inflict substantial, even fatal, damage to
vulnerable people.
This is a consideration frequently missed in the arguments about proof of vaccination in public
spaces. Those decrying it as an infringement of their liberties fail to realise that others have a
reasonable expectation that they should not be needlessly exposed to a potentially deadly virus
if it can be avoided.
(Adapted from https://www.theguardian.com/commentisfree/2021/aug/15/vaccine-passports-are-less-a-threat-to-liberty-than-a-mark-of-solidarity)

QUESTÃO 01 (EFOMM/INÉDITA) – Read the extract from the text


“In France over the past few weeks, the topic of vaccine passports has induced an avalanche of outrage”
Mark the alternative that can replace the underlined word without changing its meaning.
(A) Indignation
(B) Happiness
(C) Craziness
(D) Comprehension
(E) Despair

AULA 08 – ABBREVIATIONS, CONTRACTIONS AND IF CLAUSES 87


TEACHER ANDREA BELO

Comentários: A alternativa A está correta. A palavra “outrage” significa “raiva/choque” e pode


ser comparada com a palavra “indignation”, que significa “indignação”.
A alternativa B está incorreta. A palavra “outrage” significa “raiva/choque” e não pode ser
comparada com a palavra “happiness”, que significa “felicidade”.
A alternativa C está incorreta. A palavra “outrage” significa “raiva/choque” e não pode ser
comparada com a palavra “craziness”, que significa “loucura”.
A alternativa D está incorreta. A palavra “outrage” significa “raiva/choque” e não pode ser
comparada com a palavra “comprehension”, que significa “compreensão”.
A alternativa E está incorreta. A palavra “outrage” significa “raiva/choque” e não pode ser
comparada com a palavra “despair”, que significa “desespero”.
GABARITO: A

QUESTÃO 02 (EFOMM/INÉDITA) – About vaccine passports and their consequences, it’s correct
to affirm that
(A) The measure was fully accepted by the French population
(B) France is the only country that is dealing with opposition to measures to combat the
pandemic
(C) Vaccination is in fact a personal act and with individual goals
(D) The function of the vaccine is to protect only the individual who takes it
(E) Mass vaccination protects those who cannot be vaccinated
Comentários: A alternativa A está incorreta. Sobre passaportes de vacinas e suas consequências,
não é correto afirmar que a medida foi totalmente aceita pela população francesa, mas sim, que
houve indignação com a nova medida. Isso pode ser confirmado com o trecho “Opposition to the
measure has united both the hard left and right, with more than 200,000 people taking to the
streets to express their contempt”.
A alternativa B está incorreta. Sobre passaportes de vacinas e suas consequências, não é correto
afirmar que a França é o único país que enfrenta oposição às medidas de combate à pandemia,
mas sim, que os EUA também enfrentam esse tipo de coisa. Isso pode ser confirmado com o
trecho “In the United States, mask and vaccine mandates have generated passionate opprobrium
and legal action”.
A alternativa C está incorreta. Sobre passaportes de vacinas e suas consequências, não é correto
afirmar que a vacinação é de fato um ato pessoal e com objetivos individuais, mas sim, que é uma
medida de saúde pública. Isso pode ser confirmado com o trecho “On the contrary; immunisation
is, at heart, a public health measure, implemented to reduce incidence and burden of disease at a
population level”.
A alternativa D está incorreta. Sobre passaportes de vacinas e suas consequências, não é correto
afirmar que a função da vacina é proteger apenas o indivíduo que a toma, mas sim, que a vacina
protege todos, se utilizada corretamente em nível mundial. Isso pode ser confirmado com o

AULA 08 – ABBREVIATIONS, CONTRACTIONS AND IF CLAUSES 88


TEACHER ANDREA BELO

trecho “Immunisation collectively reduces reservoirs for disease, providing a firewall that protects
vulnerable members of society”.
A alternativa E está correta. Sobre passaportes de vacinas e suas consequências, é correto afirmar
que a vacinação em massa protege aqueles que não podem se vacinar, assim como esta opção
indica. Isso pode ser confirmado com o trecho “Immunisation collectively reduces reservoirs for
disease, providing a firewall that protects vulnerable members of society. While a Covid infection
might not do a young, healthy person lasting harm, their passing on that infection could inflict
substantial, even fatal, damage to vulnerable people”.
GABARITO: E

QUESTÃO 03 (EFOMM/INÉDITA) – Read the extract from the text


“Opposition to the measure has united both the hard left and right, with more than 200,000
people taking to the streets to express their contempt” (paragraph 1)

The word “their” refers to


(A) Measure
(B) More than 200,000 people
(C) Vaccine passports
(D) Streets
(E) Contempt
Comentários: A alternativa A está incorreta. A palavra “their” (seu) não se refere a medida, mas
sim, as mais de 200 mil pessoas. Isso pode ser confirmado com a frase “A oposição à medida uniu
duras esquerda e direita, com mais de 200.000 pessoas tomando as ruas para expressar seu
desprezo”.
A alternativa B está correta. A palavra “their” (seu) se refere as mais de 200 mil pessoas, assim
como esta opção indica. Isso pode ser confirmado com a frase “A oposição à medida uniu duras
esquerda e direita, com mais de 200.000 pessoas tomando as ruas para expressar seu desprezo”.
A alternativa C está incorreta. A palavra “their” (seu) não se refere aos passaportes de vacina, mas
sim, as mais de 200 mil pessoas. Isso pode ser confirmado com a frase “A oposição à medida uniu
duras esquerda e direita, com mais de 200.000 pessoas tomando as ruas para expressar seu
desprezo”.
A alternativa D está incorreta. A palavra “their” (seu) não se refere as ruas, mas sim, as mais de
200 mil pessoas. Isso pode ser confirmado com a frase “A oposição à medida uniu duras esquerda
e direita, com mais de 200.000 pessoas tomando as ruas para expressar seu desprezo”.
A alternativa E está incorreta. A palavra “their” (seu) não se refere ao desprezo, mas sim, as mais
de 200 mil pessoas. Isso pode ser confirmado com a frase “A oposição à medida uniu duras
esquerda e direita, com mais de 200.000 pessoas tomando as ruas para expressar seu desprezo”.
GABARITO: B

AULA 08 – ABBREVIATIONS, CONTRACTIONS AND IF CLAUSES 89


TEACHER ANDREA BELO

QUESTÃO 04 (EFOMM/INÉDITA) – Read the extract from the text


“That can seem superficially reasonable but it highlights a crucial misunderstanding…”
The sentence above means that
(A) That may seem mildly understandable, but it reinforces an important misunderstanding
(B) That's completely understandable, though it highlights a critical misunderstanding
(C) That is completely incomprehensible and highlights a critical misunderstanding
(D) That's almost entirely understandable and highlights an important misunderstanding
(E) That may sound understandable, but it doesn't give enough importance to an important
misunderstanding
Comentários: A alternativa A está correta. A frase “Isso pode parecer superficialmente razoável,
mas destaca um mal-entendido crucial” significa “Isso pode parecer ligeiramente compreensível,
mas reforça um importante mal-entendido”, assim como esta opção indica.
A alternativa B está incorreta. A frase “Isso pode parecer superficialmente razoável, mas destaca
um mal-entendido crucial” não significa “Isso é completamente compreensível, embora destaque
um mal-entendido crítico”.
A alternativa C está incorreta. A frase “Isso pode parecer superficialmente razoável, mas destaca
um mal-entendido crucial” não significa “Isso é completamente incompreensível e destaca um
mal-entendido crítico”.
A alternativa D está incorreta. A frase “Isso pode parecer superficialmente razoável, mas destaca
um mal-entendido crucial” não significa “Isso é quase totalmente compreensível e destaca um
importante mal-entendido”.
A alternativa E está incorreta. A frase “Isso pode parecer superficialmente razoável, mas destaca
um mal-entendido crucial” não significa “Isso pode parecer compreensível, mas não dá
importância suficiente a um mal-entendido importante”.
GABARITO: A

Read the text II to do items 05 to 10.


Israel extends Covid restrictions to three-year-olds as cases surge
Israel is now requiring anyone over the age of three to show proof of vaccination or a negative
Covid-19 test before entering many indoor spaces, as it tackles a sharp rise in infections.
Restaurants, cafes, museums, libraries, gyms and pools are among the venues covered by the
"Green Pass" system.
However, proof of immunity is not needed to go into shops or malls.
The country's Covid-19 tsar said it was "at war" with the virus despite its world-leading vaccination
programme.
"Our morbidity is rising day by day," Salman Zarka told a parliamentary committee on Wednesday,
according to the Jerusalem Post.

AULA 08 – ABBREVIATIONS, CONTRACTIONS AND IF CLAUSES 90


TEACHER ANDREA BELO

The next two weeks leading up to the Jewish New Year festival of Rosh Hashanah on 6 September
would be "critical", he warned.
Israel has seen a surge of infections driven by the more contagious Delta variant since late June.
The health ministry reported some 7,870 new Covid-19 cases on Tuesday, which was slightly down
on Monday's sixmonth daily record of 8,752.
More than 120 people have died after contracting the virus in the past week - double the monthly
total recorded in July - and 600 people are in a serious or critical condition in hospital.
The government has sought to combat the surge by reinstating the restrictions it lifted in mid-
June and by bringing back the Green Pass, which shows whether someone has been fully
vaccinated, has recently recovered from Covid-19, or tested negative in the previous 24 hours.
Before Wednesday only children aged 12 and over, who have been eligible to get a vaccine since
June, and adults were required to present a Green Pass.
It will now also apply to children between the ages of three and 11. Their tests will be funded by
the government as they are ineligible for vaccination, unless they are five or older and are
considered are at significant risk from Covid-19.
The approximately one million residents - about 11% of the population - who have not been
vaccinated despite being eligible must pay for their own tests.
Israel has also begun giving third doses of the Pfizer-BioNTech vaccine to people over 50, medical
workers and those with underlying health conditions.
So far, some 1.1 million eligible people have received their booster shots.
Israeli healthcare provider Maccabi, which covers about a quarter of the population, reported on
Wednesday that a third Pfizer dose was 86% effective at preventing Covid-19 infection in people
over 60.
Mr Zarka also noted that no-one who was currently in a critical condition in hospital had received
a booster shot.
(Adapted from https://www.bbc.com/news/world-middle-east-58245285)

QUESTÃO 05 (EFOMM/INÉDITA) – According to the text, mark the INCORRECT option


(A) In Israel, the Green Pass isn’t required to enter malls
(B) Israel's vaccination program sets an example for the rest of the world
(C) Success in Israel's vaccination program is ensuring victory in the battle against the virus
(D) Delta variant is making an impact in Israel
(E) The Green Pass helps to contain the spread of the virus
Comentários: A alternativa A está incorreta. De acordo com o texto, é correto afirmar que em
Israel, o Green Pass não é obrigatório para entrar em shoppings. Isso pode ser confirmado com o
trecho “However, proof of immunity is not needed to go into shops or malls”.

AULA 08 – ABBREVIATIONS, CONTRACTIONS AND IF CLAUSES 91


TEACHER ANDREA BELO

A alternativa B está incorreta. De acordo com o texto, é correto afirmar que o programa de
vacinação de Israel é um exemplo para o resto do mundo. Isso pode ser confirmado com o trecho
“...despite its world-leading vaccination programme”.
A alternativa C está correta. De acordo com o texto, não é correto afirmar que o sucesso no
programa de vacinação de Israel está garantindo a vitória na batalha contra o vírus, mas sim, que
está havendo alta no número de casos e mortes pelo vírus, apesar da vacinação exitosa. Isso pode
ser confirmado com o trecho “...said it was ‘at war’ with the virus despite its world-leading
vaccination programme”.
A alternativa D está incorreta. De acordo com o texto, é correto afirmar que a variante Delta está
causando impacto em Israel. Isso pode ser confirmado com o trecho “Israel has seen a surge of
infections driven by the more contagious Delta variant since late June”.
A alternativa E está incorreta. De acordo com o texto, é correto afirmar que o Green Pass ajuda a
conter a propagação do vírus. Isso pode ser confirmado com o trecho “The government has sought
to combat the surge by reinstating the restrictions it lifted in mid-June and by bringing back the
Green Pass…”.
GABARITO: C

QUESTÃO 06 (EFOMM/INÉDITA) – Read the extract from the text


“Restaurants, cafes, museums, libraries, gyms and pools are among the venues covered by the
‘Green Pass’ system. However, proof of immunity is not needed to go into shops or malls”.
Mark the option that can replace the underlined sentence without changing its meaning
(A) Likewise, proof of immunity is not required to enter shops or malls
(B) On the other hand, proof of immunity is not required to enter stores or malls
(C) However, proof of immunity to enter stores or malls is mandatory
(D) In addition, proof of immunity is needed to enter shops or malls
(E) Nevertheless, proof of immunity is essential to enter shops or malls
Comentários: A alternativa A está incorreta. A frase “No entanto, não é necessária prova de
imunidade para entrar em lojas ou shoppings” não pode ser substituída pela frase “Da mesma
forma, não é exigida prova de imunidade para entrar em lojas ou shoppings”.
A alternativa B está correta. A frase “No entanto, não é necessária prova de imunidade para entrar
em lojas ou shoppings” pode ser substituída pela frase “Por outro lado, não é exigida prova de
imunidade para entrar em lojas ou shoppings”.
A alternativa C está incorreta. A frase “No entanto, não é necessária prova de imunidade para
entrar em lojas ou shoppings” não pode ser substituída pela frase “No entanto, a prova de
imunidade para entrar em lojas ou shoppings é obrigatória”.
A alternativa D está incorreta. A frase “No entanto, não é necessária prova de imunidade para
entrar em lojas ou shoppings” não pode ser substituída pela frase “Além disso, é necessária prova
de imunidade para entrar em lojas ou shoppings”.

AULA 08 – ABBREVIATIONS, CONTRACTIONS AND IF CLAUSES 92


TEACHER ANDREA BELO

A alternativa E está incorreta. A frase “No entanto, não é necessária prova de imunidade para
entrar em lojas ou shoppings” não pode ser substituída pela frase “No entanto, a comprovação
de imunidade é imprescindível para entrar em lojas ou shoppings”.
GABARITO: B

QUESTÃO 07 (EFOMM/INÉDITA) – Read the extract from the text


“The government has sought to combat the surge by reinstating the restrictions it lifted in mid-
June and by bringing back the Green Pass, which shows whether someone…” (paragraph 8)
The word “which” refers to
(A) Government
(B) The surge
(C) Restrictions
(D) Someone fully vaccinated
(E) Green Pass
Comentários: A palavra “which” (que) se refere ao Green Pass citado anteriormente na frase. Isso
pode ser confirmado com a frase “O governo tentou combater o aumento restabelecendo as
restrições levantadas em meados de junho e trazendo de volta o Green Pass, que mostra se
alguém...”.
GABARITO: E

QUESTÃO 08 (EFOMM/INÉDITA) – The Green Pass


(A) Is intended for people aged 12 and over
(B) Is only for children under 3 years old
(C) Allows everyone to take tests for free
(D) Is now intended for all people aged 3 years and over
(E) Will be used for the first time in Israel
Comentários: A alternativa A está incorreta. De acordo com o texto, não é correto afirmar que o
Green Pass se destina a pessoas com 12 anos ou mais, mas sim, que agora é para todos com 3
anos ou mais, em Israel. Isso pode ser confirmado com o trecho “It will now also apply to children
between the ages of three and 11”.
A alternativa B está incorreta. De acordo com o texto, não é correto afirmar que o Green Pass é
apenas para crianças menores de 3 anos, mas sim, que é para todos com 3 anos ou mais, em
Israel. Isso pode ser confirmado com o trecho “It will now also apply to children between the ages
of three and 11”.
A alternativa C está incorreta. De acordo com o texto, não é correto afirmar que o Green Pass
permite que todos façam testes gratuitamente, mas sim, que o governo irá pagar os testes de
certas crianças. Isso pode ser confirmado com o trecho “Their tests will be funded by the

AULA 08 – ABBREVIATIONS, CONTRACTIONS AND IF CLAUSES 93


TEACHER ANDREA BELO

government as they are ineligible for vaccination, unless they are five or older and are considered
are at significant risk from Covid-19”.
A alternativa D está correta. De acordo com o texto, é correto afirmar que o Green Pass agora é
destinado a todas as pessoas com 3 anos ou mais, assim como esta opção indica. Isso pode ser
confirmado com o trecho “It will now also apply to children between the ages of three and 11”.
A alternativa E está incorreta. De acordo com o texto, não é correto afirmar que o Green Pass será
usado pela primeira vez em Israel, mas sim, que ele já foi utilizado anteriormente. Isso pode ser
confirmado com o trecho “The government has sought to combat the surge by reinstating the
restrictions it lifted in mid-June and by bringing back the Green Pass, which shows…”.
GABARITO: D

QUESTÃO 09 (EFOMM/INÉDITA) – The expression “day by day”, in paragraph 3, means


(A) Gradually over many days
(B) Abruptly
(C) Each month
(D) During all day long
(E) One day every two weeks
Comentários: A alternativa A está correta. A expressão “day by day” significa “gradualmente ao
longo de muitos dias”, assim como esta opção indica.
A alternativa B está incorreta. A expressão “day by day” não significa “abruptamente”, mas sim,
“gradualmente ao longo de muitos dias”.
A alternativa C está incorreta. A expressão “day by day” não significa “cada mês”, mas sim,
“gradualmente ao longo de muitos dias”.
A alternativa D está incorreta. A expressão “day by day” não significa “durante todo o dia”, mas
sim, “gradualmente ao longo de muitos dias”.
A alternativa E está incorreta. A expressão “day by day” não significa “um dia a cada duas
semanas”, mas sim, “gradualmente ao longo de muitos dias”.
GABARITO: A

QUESTÃO 10 (EFOMM/INÉDITA) – The word “eligible” (paragraph 9), can be replaced by the
word ___ without changing its meaning
(A) Improper
(B) Good
(C) Privileged
(D) Suitable
(E) Awkward

AULA 08 – ABBREVIATIONS, CONTRACTIONS AND IF CLAUSES 94


TEACHER ANDREA BELO

Comentários: A alternativa A está incorreta. A palavra “eligible” significa “elegível” e não pode
ser substituída pela palavra “improper”, que significa “impróprio”.
A alternativa B está incorreta. A palavra “eligible” significa “elegível” e não pode ser substituída
pela palavra “good”, que significa “bom”.
A alternativa C está incorreta. A palavra “eligible” significa “elegível” e não pode ser substituída
pela palavra “privileged”, que significa “privilegiado”.
A alternativa D está correta. A palavra “eligible” significa “elegível” e pode ser substituída pela
palavra “suitable”, que significa “adequado”.
A alternativa E está incorreta. A palavra “eligible” significa “elegível” e não pode ser substituída
pela palavra “awkward”, que significa “estranho”.
GABARITO: D

QUESTÕES EPCAR
Directions: Answer questions 01 to 05 according to text.
What It Will Take to Get Life Back to Normal
Vaccines have brought the United States tantalizingly close to crushing the coronavirus within its
borders. After months of hiccups, some 1.4 million people are now being vaccinated every day,
and many more shots are coming through the pipeline. The Food and Drug Administration has
just authorized a third vaccine — a single-dose shot made by Johnson & Johnson — while Pfizer
and Moderna are promising to greatly expand the supply of their shots, to roughly 100 million
total doses per month, by early spring.
If those vaccines make their way into arms quickly, the nation could be on its way to a relatively
pleasant summer and something approaching normal by autumn. Imagine schools running at full
capacity in September and families gathering for Thanksgiving.
But turning that “if” into a “when” will require clearing additional hurdles so that everyone who
needs to be vaccinated gets vaccinated. This is especially true for racial minorities, who are being
disproportionately missed by the vaccination effort.
There’s plenty of disagreement among experts as to why America is still having problems with
vaccine uptake. Some officials have suggested that the main cause is that too many people are
hesitant to get the vaccine. Others point the finger at overcautious public health officials who they
say have undersold the promise of the vaccines. Still others point to long lines at clinics as proof
that far more people want the vaccine than can actually get it.
There is probably some truth to all of these hypotheses, and the underlying problems are not
new. Vaccine hesitancy had been growing steadily in America long before the current pandemic,
so much so that in 2019 the World Health Organization ranked it as one of the leading global
health threats. At the same time, poor health care access and other logistical constraints, such as
a lack of public transportation and limited internet access, have long impeded public health efforts
in low-income communities.

AULA 08 – ABBREVIATIONS, CONTRACTIONS AND IF CLAUSES 95


TEACHER ANDREA BELO

To maximize the number of Americans getting vaccinations, policymakers need to tackle each of
these crises with greater urgency than they have so far.
As supply increases, health officials should mount ambitious vaccination campaigns modeled on
ones that have worked to curb diseases in other countries. That will mean not relying solely on
web portals for scheduling vaccine appointments. It will mean going block by block and door to
door, through high-risk communities especially. It will mean setting up employee vaccination sites
at schools, grocery stores, transit hubs and meatpacking plants, and community clinics at houses
of worship, with local leaders promoting and running them.
“The easier you can make it for people to get vaccinated, the more likely your program will be to
succeed,” said Dr. Walter Orenstein, a former director of the national immunization program at
the Centers for Disease Control and Prevention. “It’s really that simple.”
Outreach efforts cost money. But they’re far less expensive than allowing the pandemic to fester.
Congress has appropriated some money to help states with vaccine rollout. It should offer more,
and states should put as much of those resources as possible toward vaccination efforts that meet
people where they are.
Health officials should also recognize that vaccine hesitancy has many root causes — deliberate
disinformation campaigns, mistrust of medical authorities in marginalized communities, ill-
considered messaging by health officials. The best way to counter that is with campaigns that are
locally led, that clearly outline the benefits of vaccination and that frame getting the shot as not
just a personal choice but a collective responsibility.
Doctors and scientists can help those pro-vaccine messages stick by minding their own public
communications. It’s crucial to be transparent about what vaccines will and won’t do for society
— overselling now will only sow more mistrust later.
That said, underselling is its own problem. It’s true that these vaccines will not immediately
restore the world to total normalcy. But they will eventually allow people to hug their loved ones,
to return to their offices — and to be protected from dying from or becoming seriously ill with
Covid-19. Health officials should be clear about that.
Policymakers at the highest levels of government should press social media companies and e-
commerce sites to curb the most aggressive purveyors of vaccine disinformation.
To not only quell this pandemic but to try to prevent the next one, America will need to improve
its health system and its public health apparatus, both of which have significant holes. “The
problem with a lot of the response is that it was predicated on the idea that we have a good
system in place for doing adult immunizations across the country,” said Dr. Peter Hotez, a vaccine
expert at Baylor College of Medicine. “The fact is, we really don’t.”
In the end, lawmakers and the people who vote them into office will have to address the much
broader problems that this pandemic has exposed.
(Adapted from https://www.nytimes.com/2021/02/26/opinion/vaccine-covid-coronavirus.html?action=click&module=Opinion&pgtype=Homepage)

AULA 08 – ABBREVIATIONS, CONTRACTIONS AND IF CLAUSES 96


TEACHER ANDREA BELO

QUESTÃO 01 (EPCAR/INÉDITA) – Mark the option that can replace the word “pleasant” in the
second paragraph without changing its meaning
(A) Pretty
(B) Gloomy
(C) Cordial
(D) Boring
(E) Harsh
Comentários: A alternativa A está incorreta. A palavra “pleasant” significa agradável e não pode
ser substituída pela palavra “pretty”, que significa bonito.
A alternativa B está incorreta. A palavra “pleasant” significa agradável e não pode ser substituída
pela palavra “gloomy”, que significa sombrio.
A alternativa C está correta. A palavra “pleasant” significa agradável e pode ser substituída pela
palavra “cordial”, que significa cordial/agradável.
A alternativa D está incorreta. A palavra “pleasant” significa agradável e não pode ser substituída
pela palavra “boring”, que significa entediante.
A alternativa E está incorreta. A palavra “pleasant” significa agradável e não pode ser substituída
pela palavra “harsh”, que significa severo/difícil.
GABARITO: C

QUESTÃO 02 (EPCAR/INÉDITA) – Mark the option with the suitable question to answer the
underlined fragment below
“After months of hiccups, some 1.4 million people are now being vaccinated every day, and
many more shots are coming through the pipeline” (paragraph 1)
(A) What are the expectations for the future of vaccination?
(B) How many people have already received the vaccine?
(C) How long was the vaccine developed in?
(D) Where are the vaccines coming from?
(E) How effective is this vaccine?
Comentários: A alternativa A está correta. A frase “muitas outras vacinas estão chegando pelo
duto” responde à pergunta “Quais são as expectativas para o futuro da vacinação?”, assim como
esta opção indica.
A alternativa B está incorreta. A frase “muitas outras vacinas estão chegando pelo duto” não
responde à pergunta “Quantas pessoas já receberam a vacina?”, mas sim, à pergunta “Quais são
as expectativas para o futuro da vacinação?”.
A alternativa C está incorreta. A frase “muitas outras vacinas estão chegando pelo duto” não
responde à pergunta “Por quanto tempo a vacina foi desenvolvida?”, mas sim, à pergunta “Quais
são as expectativas para o futuro da vacinação?”.

AULA 08 – ABBREVIATIONS, CONTRACTIONS AND IF CLAUSES 97


TEACHER ANDREA BELO

A alternativa D está incorreta. A frase “muitas outras vacinas estão chegando pelo duto” não
responde à pergunta “De onde vêm as vacinas?”, mas sim, à pergunta “Quais são as expectativas
para o futuro da vacinação?”.
A alternativa E está incorreta. A frase “muitas outras vacinas estão chegando pelo duto” não
responde à pergunta “Quão eficaz é esta vacina?”, mas sim, à pergunta “Quais são as expectativas
para o futuro da vacinação?”.
GABARITO: A

QUESTÃO 03 (EPCAR/INÉDITA) – Mark the statement that is NOT mentioned in the text
(A) Vaccines are helpoing the US to achieve a post-pandemic future
(B) US vaccination is slow and has no foreseeable future
(C) If vaccination in the US is successful, the country may be close to a better daily life
(D) A post-pandemic reality depends on some factors
(E) Vaccination in the USA is not impartial, there are privileged groups
Comentários: A alternativa A está incorreta. O texto menciona que as vacinas estão ajudando os
EUA a alcançar um futuro pós pandêmico. Isso pode ser confirmado com o trecho “Vaccines have
brought the United States tantalizingly close to crushing the coronavirus within its borders”.
A alternativa B está correta. O texto não menciona que a vacinação dos EUA é lenta e não tem
futuro previsível, mas sim, que não é lenta e tem um futuro previsível. Isso pode ser confirmado
com o trecho “...some 1.4 million people are now being vaccinated every day, and many more
shots are coming through the pipeline. The Food and Drug Administration has just authorized a
third vaccine — a single-dose shot made by Johnson & Johnson — while Pfizer and Moderna are
promising to greatly expand the supply of their shots, to roughly 100 million total doses per
month, by early spring”.
A alternativa C está incorreta. O texto menciona que se a vacinação nos EUA for bem-sucedida, o
país pode estar próximo de uma vida diária melhor. Isso pode ser confirmado com o trecho “If
those vaccines make their way into arms quickly, the nation could be on its way to a relatively
pleasant summer and something approaching normal by autumn”.
A alternativa D está incorreta. O texto menciona que uma realidade pós-pandêmica depende de
alguns fatores. Isso pode ser confirmado com o trecho “But turning that “if” into a “when” will
require clearing additional hurdles so that everyone who needs to be vaccinated gets vaccinated”.
A alternativa E está incorreta. O texto menciona que a vacinação nos EUA não é imparcial, existem
grupos privilegiados. Isso pode ser confirmado com o trecho “This is especially true for racial
minorities, who are being disproportionately missed by the vaccination effort”.
GABARITO: B

AULA 08 – ABBREVIATIONS, CONTRACTIONS AND IF CLAUSES 98


TEACHER ANDREA BELO

QUESTÃO 04 (EPCAR/INÉDITA) – Mark the alternative that complete the sentence below
correctly
Vaccine acceptance _______________ a problem in the US
(A) Is
(B) Was
(C) Was being
(D) Has been
(E) Will be
Comentários: A alternativa A está incorreta. O tempo verbal correto para completar não é no
simple present (is - é), mas sim, no present perfect (has been – tem sido), por indicar algo que
começou no passado e continua no presente. Isso pode ser confirmado com o trecho “There’s
plenty of disagreement among experts as to why America is still having problems with vaccine
uptake”.
A alternativa B está incorreta. O tempo verbal correto para completar não é no simple past (was
- era), mas sim, no present perfect (has been – tem sido), por indicar algo que começou no
passado e continua no presente. Isso pode ser confirmado com o trecho “There’s plenty of
disagreement among experts as to why America is still having problems with vaccine uptake”.
A alternativa C está incorreta. O tempo verbal correto para completar não é no past continuous
(was being – estava sendo), mas sim, no present perfect (has been – tem sido), por indicar algo
que começou no passado e continua no presente. Isso pode ser confirmado com o trecho “There’s
plenty of disagreement among experts as to why America is still having problems with vaccine
uptake”.
A alternativa D está correta. O tempo verbal correto para completar é no no present perfect (has
been – tem sido), por indicar algo que começou no passado e continua no presente, assim como
esta opção indica. Isso pode ser confirmado com o trecho “There’s plenty of disagreement among
experts as to why America is still having problems with vaccine uptake”.
A alternativa E está incorreta. O tempo verbal correto para completar não é no simple future (will
be - será), mas sim, no present perfect (has been – tem sido), por indicar algo que começou no
passado e continua no presente. Isso pode ser confirmado com o trecho “There’s plenty of
disagreement among experts as to why America is still having problems with vaccine uptake”.
GABARITO: D

QUESTÃO 05 (EPCAR/INÉDITA) – Mark the option that can replace the underlined word keeping
the same meaning
“Some officials have suggested that the main cause is that too many people are hesitant to get
the vaccine” (paragraph 4)
(A) Confident
(B) Skeptical
(C) Unbased
(D) Impartial
(E) Lazy

AULA 08 – ABBREVIATIONS, CONTRACTIONS AND IF CLAUSES 99


TEACHER ANDREA BELO

Comentários: A alternativa A está incorreta. A palavra “hesitant” significa hesitante e não pode
ser substuída pela palavra “confidente”, que significa confiante.
A alternativa B está correta. A palavra “hesitant” significa hesitante e pode ser substuída pela
palavra “skeptical”, que significa cético/hesitante.
A alternativa C está incorreta. A palavra “hesitant” significa hesitante e não pode ser substuída
pela palavra “unbased”, que significa sem base.
A alternativa D está incorreta. A palavra “hesitant” significa hesitante e não pode ser substuída
pela palavra “imapartial”, que significa imparcial.
A alternativa E está incorreta. A palavra “hesitant” significa hesitante e não pode ser substuída
pela palavra “lazy”, que significa preguiçoso.
GABARITO: B

QUESTÕES ESA
Brazil, officially the Federative Republic of Brazil, is the largest country in both South America and
Latin America. At 8.5 million square kilometers (3.2 million square miles) and with over 211 million
people, Brazil is the world's fifth-largest country by area and the sixth most populous. Its capital
is Brasília, and its most populous city is São Paulo. _______ federation is composed of the union
of the 26 states and the Federal District. It is the largest country to have Portuguese as an official
language and the only one in the Americas; it is also one of the most multicultural and ethnically
diverse nations, due to over _______ century of mass immigration from around the world.
Bounded by the Atlantic Ocean on the east, Brazil has a coastline of 7,491 kilometers (4,655 mi).
It borders all other countries in South America except Ecuador and Chile and covers 47.3% of the
continent's land area. Its Amazon River basin includes _______ enormous tropical forest, home
to diverse wildlife, a variety of ecological systems, and extensive natural resources spanning
numerous protected habitats.
Adapted from https://en.wikipedia.org/wiki/Brazil.

QUESTÃO 01 (ESA/INÉDITA) – According to the text, it is incorrect to say that:


(A) Brazil is the largest country in South America.
(B) Brazil is the world's fifth most populous country.
(C) Brazil does not border Ecuador and Chile.
(D) Brazil’s official name is the Federative Republic of Brazil.
(E) Brazil is the only country in the Americas to have Portuguese as an official language.
Comentários: A questão quer que você assinale a alternativa incorreta.
a) Brazil is the largest country in South America. “Brazil, officially the Federative Republic of Brazil,
is the largest country in both South America and Latin America.” Essa alternativa está de acordo
com o texto. ERRADA.

AULA 08 – ABBREVIATIONS, CONTRACTIONS AND IF CLAUSES 100


TEACHER ANDREA BELO

b) Brazil is the world's fifth most populous country. Brazil is the world's fifth-largest country by
area and the sixth most populous. O Brasil é o quinto maior país do mundo em área e o sexto mais
populoso (não o quinto, como afirma a alternativa). Alternativa incorreta, sendo, portanto, o
nosso gabarito. CORRETA.
c) Brazil does not border Ecuador and Chile. “It borders all other countries in South America except
Ecuador and Chile...” De acordo com o texto. ERRADA.
d) Brazil’s official name is the Federative Republic of Brazil. “Brazil, officially the Federative
Republic of Brazil...” De acordo com o texto. ERRADA.
e) Brazil is the only country in the Americas to have Portuguese as an official language. “It is the
largest country to have Portuguese as an official language and the only one in the Americas;” De
acordo com o texto. ERRADA.
GABARITO: B

QUESTÃO 02 (ESA/INÉDITA) – In the sentence “Its capital is Brasília, and its most populous city
is São Paulo.” (paragraph 1), the possessive adjectives “its” refer to
(A) Brazil.
(B) Latin America.
(C) South America.
(D) The Federal District.
(E) city.
Comentários: Nesse tipo de questão, é essencial voltar ao texto e ler todo o trecho onde os
possessivos aparecem, para entender a que se referem. Quando você se deparar com esse tipo
de questão, perguntando a que um adjetivo possessivo se refere (que é o caso de its), você deve
identificar a quem o substantivo pertence e marcar a alternativa que contenha essa informação.
... Brazil is the world's fifth-largest country by area and the sixth most populous. Its capital is
Brasília, and its most populous city is São Paulo. → ... O Brasil é o quinto maior país do mundo em
área e o sexto mais populoso. Sua capital (capital dele, do Brasil) é Brasília e sua cidade mais
populosa (cidade mais populosa dele, do Brasil) é São Paulo. Assim, os dois its se referem a Brazil.
a) Brazil. Como comprovado acima, alternativa CORRETA.
b) Latin America. Conforme analisado acima, ERRADA.
c) South America. Conforme analisado acima, ERRADA.
d) The Federal District. Conforme analisado acima, ERRADA.
e) city. Conforme analisado acima, ERRADA.
GABARITO: A

AULA 08 – ABBREVIATIONS, CONTRACTIONS AND IF CLAUSES 101


TEACHER ANDREA BELO

QUESTÃO 03 (ESA/INÉDITA) – Choose the correct option to fill in the blanks with the missing
articles, from top to bottom:
(A) the / a / an
(B) a / a / a
(C) the / an / a
(D) an / an / an
(E) the / the / a
Comentários: THE federation is composed of the union of the 26 states and the Federal District.
→ o trecho se refere especificamente à federação brasileira, não a uma federação qualquer. Por
isso, o artigo definido é o mais adequado.
... due to over A century of mass immigration from around the world. → devido a mais de um
século de imigração em massa de todo o mundo. No caso, o artigo indefinido é que expressa a
ideia de um (um século). Como century começa com som de consoante, devemos usar A (e não
AN).
Its Amazon River basin includes AN enormous tropical forest ... → Sua bacia do rio Amazonas inclui
uma enorme floresta tropical. O trecho se refere a uma floresta tropical (não é a única- é uma,
dentre outras florestas que existem), daí o uso do artigo indefinido. Além disso, foi necessário
usar AN, para concordar com o som de vogal do adjetivo imediatamente após o artigo, enormous.
Assim, temos a seguinte sequência correta: the / a / an
a) the / a / an. Como comprovado acima, alternativa CORRETA.
b) a / a / a. Conforme analisado acima, ERRADA.
c) the / an / a. Conforme analisado acima, ERRADA.
d) an / an / an. Conforme estudado acima, ERRADA.
e) the / the / a. Conforme estudado acima, ERRADA.
GABARITO: A

QUESTÃO 04 (ESA/INÉDITA) – “Officially” is


(A) an adjective
(B) a pronoun
(C) an adverb
(D) an article
(E) a verb
Comentários: “Officially” significa oficialmente, sendo um advérbio. A terminação -ly costuma
equivaler ao sufixo-mente em Português, sendo característico de advérbios de modo. As demais
alternativas trouxeram como opções adjetivo, pronome, artigo e verbo.
GABARITO: C

AULA 08 – ABBREVIATIONS, CONTRACTIONS AND IF CLAUSES 102


TEACHER ANDREA BELO

QUESTÃO 05 (ESA/INÉDITA) – “Reading makes life a lot easier”. In this sentence, the word
“makes” indicates
(A) a plural noun.
(B) third person plural.
(C) the Imperative form.
(D) the Simple Present Tense.
(E) the Present Continuous.
Comentários: A frase “Reading makes life a lot easier” significa ler torna a vida muito mais fácil.
Makes é um verbo e está conjugado na terceira pessoa do singular, no Simple Present, por isso
a terminação-s.
a) plural noun. Como vimos, makes é um verbo e está conjugado na terceira pessoa do singular,
no Simple Present, por isso a terminação-s. Não é um substantivo no plural (a plural noun).
ERRADA.
b) third person plural. A terminação -s indica que o verbo está conjugado na terceira pessoa do
singular, no Simple Present, não no plural. ERRADA.
c) the Imperative form. O imperativo é formado apenas pelo verbo, ou, na negativa, pelo don’t
mais o verbo. Exemplo: vá lá- go there, não vá lá- don’t go there. Não é o caso do verbo make na
frase. ERRADA.
d) the Simple Present Tense. Makes é um verbo e está conjugado na terceira pessoa do singular,
por isso a terminação-s, indicando Simple Present. CORRETA.
e) the Present Continuous. Como vimos, makes está no Simple Present (por isso a terminação-s),
não no Present Continuous. Lembre-se que esse tempo verbal é formado pelo verbo to be no
presente e outro verbo, o verbo principal da frase, acrescido de -ing. ERRADA.
GABARITO: D

QUESTÃO 06 (ESA/INÉDITA) – “Reading makes life a lot easier”. “Easier”, in this sentence,
represents
(A) a time expression.
(B) a demonstrative pronoun.
(C) the Simple Present form of a verb.
(D) the superlative form of an adjective.
(E) the comparative form of an adjective.
Comentários: Easier representa a forma comparativa do adjetivo easy, significando mais fácil.
Como easy é um adjetivo curto terminado em -y, tiramos o -y e eacrescentamos -ier para formar
o comparativo.
a) a time expression. Easier não é expressão de tempo. ERRADA
b) a demonstrative pronoun. Os pronomes demonstrativos são, tradicionalmente, this, these, that
e those. ERRADA.

AULA 08 – ABBREVIATIONS, CONTRACTIONS AND IF CLAUSES 103


TEACHER ANDREA BELO

c) the Simple Present form of a verb. Easier é comparativo de um adjetivo, não verbo. ERRADA.
d) the superlative form of an adjective. Easier representa a forma comparativa do adjetivo easy,
não o superlativo, que seria the easiest. ERRADA.
e) the comparative form of an adjective. Alternativa correta, conforme explicado acima.
CORRETA.
GABARITO: E

QUESTÃO 07 (ESA/INÉDITA) – Choose the sentence that indicates future:


(A) I’m working now.
(B) Sam’s sleeping at the moment.
(C) He’s doing the laundry right now.
(D) Are you studying at this moment?
(E) They’re visiting their friends next weekend.
Comentários: Escolha a frase que indica futuro:
a) Estou trabalhando agora – presente (verbo no Presente Contínuo).
b) Sam está dormindo no momento – (presente verbo no Presente Contínuo).
c) Ele está lavando a roupa agora. – presente (verbo no Presente Contínuo).
d) Você está estudando neste momento? – presente (verbo no Presente Contínuo).
e) Eles visitarão seus amigos no próximo fim de semana. – futuro (verbo no Presente Contínuo,
indicando, porém, futuro. Quem deixa isso isso claro é a expressão de futuro “next weekend”).
GABARITO: E

QUESTÃO 08 (ESA/INÉDITA) – Which alternative does not have the superlative form of the
adjective?
(A) the best
(B) the worst
(C) the bigger
(D) the greatest
(E) the most exquisite
Comentários: Qual alternativa não possui a forma superlativa do adjetivo?
a) The best é o superlativo de good.
b) The worst é o superlativo de bad.
c) Bigger é comparativo de big.
d) The greatest é o superlativo de great.
e) The most exquisite é o superlativo de exquisite.
GABARITO: C

AULA 08 – ABBREVIATIONS, CONTRACTIONS AND IF CLAUSES 104


TEACHER ANDREA BELO

QUESTÃO 09 (ESA/INÉDITA) – Which sentence is grammatically correct?


(A) Do you has a sister?
(B) Does she has a car?
(C) Do you have a brother?
(D) Do they has a job?
(E) Does he haves a bike?
Comentários: Qual frase está gramaticalmente correta? Observe que todas as alternativas trazem
perguntas no Simple Present. Neste tempo verbal, usamos o auxiliar do para os sujeitos I, you, we
e they e does para os sujeitos he, she, it. Em qualquer caso, na pergunta no Simple Present, o
verbo fica na sua forma base, sem alteração, “normal”.
a) O correto seria “Do you have a sister?” (verbo na sua forma base na interrogativa).
b) O correto seria “Does she have a car?” (verbo na sua forma base na interrogativa).
c) Auxiliar correto para o sujeito you e verbo na sua forma base na interrogativa. Alternativa
CORRETA.
d) O correto seria “Do they have a job?” (verbo na sua forma base na interrogativa).
e) O correto seria “Does he have a bike?” (verbo na sua forma base na interrogativa).
GABARITO: C

QUESTÃO 10 (ESA/INÉDITA)
“A: Is John your friend?
B: Yes, _________.”
Fill in the blank with the correct form of pronoun and verb.
(A) he’s.
(B) he is.
(C) you are.
(D) she is.
(E) he isn’t.
Comentários: Temos uma pergunta em que foi utilizado o presente do verbo to be – O John é seu
amigo? Na resposta curta afirmativa, devemos usar o sujeito (he) com o verbo correspondente
(is), sem contração → Yes, he is (Sim, ele é).
GABARITO: B

AULA 08 – ABBREVIATIONS, CONTRACTIONS AND IF CLAUSES 105


TEACHER ANDREA BELO

QUESTÕES ESCOLA NAVAL


Can covid make your ears ring? What we know about its possible connection to tinnitus.
The recent death ____ Texas Roadhouse CEO Kent Taylor is bringing more attention to what
experts say is another troublesome physical ailment that may be associated with the coronavirus
pandemic: tinnitus, or the perception of a loud ringing or buzzing sound in the ears.
Taylor, 65, died ____ suicide last week, and his family told news outlets, including The Washington
Post, that he had been battling “post-Covid related symptoms, including severe tinnitus” at the
time of his death.
[Texas Roadhouse CEO dies by suicide while battling ‘unbearable’ post-covid-19 symptoms, family
says] Though early research and anecdotal reports have documented tinnitus and sudden hearing
loss in some covid-19 patients, audiologists emphasized that there is no conclusive evidence
connecting the virus to the onset or worsening of tinnitus.
The condition is “one of those things that is just so variable in every single person,” said Eldré
Beukes, ____ audiologist and research fellow at Anglia Ruskin University in ____ United Kingdom,
who has published research on how individuals with tinnitus have been affected by the pandemic.
In the United States, more than 50 million people experience some form of tinnitus, which is often
linked to hearing loss. About 20 million struggle with a chronic condition, while 2 million have
extreme and debilitating cases, according to the American Tinnitus Association.
(Adapted from https://www.washingtonpost.com/lifestyle/wellness/tinnitus-covid-treatment-ear-hearing/2021/03/23/aca1811c-8b32-11eb-9423-04079921c915_story.html)

QUESTÃO 01 (ESCOLA NAVAL/INÉDITA) – Which is the correct option to complete the gaps in
the text?
(A) Of / by / an / the
(B) Of / with / an / X
(C) Of / by / an / X
(D) Of / of / an / X
(E) Of / by / a / the
Comentários: A primeira lacuna deve ser preenchida com “of”, pois a frase se refere à morte de
Kent Taylor, ou seja, “A morte recente do CEO da Texas Roadhouse, Kent Taylor...”.
A segunda lacuna deve ser preenchida com “by”, pois a frase se refere ao motivo da morte de
Taylor, ou seja, “Taylor, 65, morreu por suicídio na semana passada...”.
A terceira lacuna deve ser preenchida com “an”, pois a frase se refere a um audiologista, ou seja,
“...Eldré Beukes, audiologista e pesquisador...”.
A quarta lacuna deve ser preenchida com “the”, pois a frase se refere ao Reino Unido, uma exceção
à regra de que países não precisam do artigo definido, ou seja, “...na Anglia Ruskin University no
Reino Unido...”.
GABARITO: A

AULA 08 – ABBREVIATIONS, CONTRACTIONS AND IF CLAUSES 106


TEACHER ANDREA BELO

QUESTÃO 02 (ESCOLA NAVAL/INÉDITA) – According to the text, which option is correct?


(A) Kent Taylor's death is not associated with post-COVID-19 symptoms
(B) Tinnitus was not reported as a post-COVID-19 symptom for Kent
(C) Tinnitus is a post-COVID-19 symptom proven by science
(D) Most COVID-19 patients have tinnitus as a symptom of the disease
(E) Tinnitus is a condition that varies in intensity according to the individual
Comentários: A alternativa A está incorreta. De acordo com o texto, não é correto afirmar que a
morte de Kent Taylor não está associada a sintomas pós-COVID-19, mas sim, que o suicídio de Kent
pode ou não estar associado ao zumbido como um sintoma pós-COVID-19, uma vez que não é
possível afirmar que esse é, de fato, um sintoma posterior à COVID-19; e que não é possível
afirmar as motivações que levou Kent a cometer suicídio. Isso pode ser confirmado com o trecho
“...audiologists emphasized that there is no conclusive evidence connecting the virus to the onset
or worsening of tinnitus”.
A alternativa B está incorreta. De acordo com o texto, não é correto afirmar que o zumbido não
foi relatado como um sintoma pós-COVID-19 para Kent, mas sim, que a família dele relatou o
zumbido como um dos sintomas pós-COVID que ele estava enfrentando. Isso pode ser confirmado
com o trecho “…his family told news outlets, including The Washington Post, that he had been
battling ‘post-Covid related symptoms, including severe tinnitus’…”.
A alternativa C está incorreta. De acordo com o texto, não é correto afirmar que o zumbido é um
sintoma pós-COVID-19 comprovado pela ciência, mas sim, que não existem evidências para
comprovar isso. Isso pode ser confirmado com o trecho “...audiologists emphasized that there is
no conclusive evidence connecting the virus to the onset or worsening of tinnitus”.
A alternativa D está incorreta. De acordo com o texto, não é correto afirmar que a maioria dos
pacientes com COVID-19 tem zumbido como sintoma da doença, mas sim, que o zumbido e a
perda repentina de audição formam relatados por alguns pacientes da COVID-19. Isso pode ser
confirmado com o trecho “Though early research and anecdotal reports have documented
tinnitus and sudden hearing loss in some covid-19 patients…”.
A alternativa E está correta. De acordo com o texto, é correto afirmar que o zumbido é uma
condição que varia de intensidade de acordo com o indivíduo, assim como esta opção indica. Isso
pode ser confirmado com o trecho “The condition is ‘one of those things that is just so variable in
every single person,’…”.
GABARITO: E

QUESTÃO 03 (ESCOLA NAVAL/INÉDITA) – What’s the meaning of the word “sudden” in


paragraph 4?
(A) Gradual
(B) Abrupt
(C) Expected
(D) Fast
(E) Lazy

AULA 08 – ABBREVIATIONS, CONTRACTIONS AND IF CLAUSES 107


TEACHER ANDREA BELO

Comentários: A alternativa A está incorreta. A palavra “sudden” significa súbito/repentino e não


pode ser comparada com a palavra “gradual”, que significa gradual.
A alternativa B está correta. A palavra “sudden” significa súbito/repentino e pode ser comparada
com a palavra “abrupt”, que significa abrupto/inesperado.
A alternativa C está incorreta. A palavra “sudden” significa súbito/repentino e não pode ser
comparada com a palavra “expected”, que significa esperado.
A alternativa D está incorreta. A palavra “sudden” significa súbito/repentino e não pode ser
comparada com a palavra “fast”, que significa rápido.
A alternativa E está incorreta. A palavra “sudden” significa súbito/repentino e não pode ser
comparada com a palavra “lazy”, que significa preguiçoso.
GABARITO: B

Don’t fear the AstraZeneca jab, the risks are minimal


Statistics about the Oxford/AstraZeneca vaccine have dominated the news, with concern over
blood clots leading many European countries to suspend its use.
Then, on Thursday, the European Medicines Agency (EMA) and the UK Medicines and Healthcare
products Regulatory Agency (MRHA) declared there was no general increased risk of thromboses,
but they were continuing to look at a rare type of clotting linked with low platelets, particularly in
the brain.
From nearly 12m Oxford/AZ vaccinations up to 7 March, the UK “yellow card” system has recorded
61,000 reports detailing 228,000 reactions, around double the rate for the Pfizer/BioNTech
vaccine. There have been huge numbers of immediate side-effects such as pain, nausea, fatigue,
headache, and fever, with other notable reports including palpitations (1,318), “feeling jittery”
(10), “screaming” (4), chilblains (10), alcohol poisoning (2), libido increased (1), libido decreased
(1), and one remarkable report of a pregnancy following vaccination. But some have been serious
events, including 289 deaths soon after the jab, and all of these will have been examined carefully.
Every decision has trade-offs. Both UK vaccines have had more than 200 severe allergic reactions
(anaphylaxis), which is why we have to sit for 15 minutes after the jab. So it’s reasonable that,
rather than saying the vaccine is “safe”, the EMA and MRHA emphasise that the benefits of the
vaccine outweigh the risks of side-effects.
In the UK there has been one of these rare blood reactions in 2m jabs. But for every 2 million
people in their 50s getting a jab, the current group being vaccinated, we would currently expect
to prevent around five deaths a week. The ratio of benefit to harms is high, and even higher on
mainland Europe as their third wave starts.
The “cautionary” pause in many countries may mean increased vaccination hesitancy. France has
moved from initially not approving the Oxford/AZ vaccine for over-65s, to pausing it for everyone,
to now not approving it for under-55s. It would not be surprising if people are confused by this.
Confidence can shatter like glass and be hard to remake.
(Adapted from https://www.theguardian.com/theobserver/commentisfree/2021/mar/21/do-not-fear-the-astrazeneca-covid-jab-the-risks-areminimal)

AULA 08 – ABBREVIATIONS, CONTRACTIONS AND IF CLAUSES 108


TEACHER ANDREA BELO

QUESTÃO 04 (ESCOLA NAVAL/INÉDITA) – It is possible to infer from the text that


(A) There was never any concern about the side effects of the Oxford/AstraZeneca vaccine
(B) The Pfizer/BioNTech vaccine had a higher number of reactions than the Oxford/AstraZeneca
vaccine
(C) A vaccine is only safe when the rate of side effects is minimal or zero
(D) The mistrust and hesitation about the Oxford/AstraZeneca vaccine are predictable,
considering the current scenario
(E) The harms are greater than the benefits, in the Oxford/AstraZeneca vaccine
Comentários: A alternativa A está incorreta. De acordo com o texto, não é possível inferir que
nunca houve qualquer preocupação com os efeitos colaterais da vacina Oxford/AstraZeneca, mas
sim, que existem preocupações acerca da formação de coágulos. Isso pode ser confirmado com o
trecho “Statistics about the Oxford/AstraZeneca vaccine have dominated the news, with concern
over blood clots…”.
A alternativa B está incorreta. De acordo com o texto, não é possível inferir que a vacina
Pfizer/BioNTech teve um número maior de reações do que a vacina Oxford/AstraZeneca, mas sim,
o contrário. Isso pode ser confirmado com o trecho “From nearly 12m Oxford/AZ vaccinations up
to 7 March, the UK ‘yellow card’ system has recorded 61,000 reports detailing 228,000 reactions,
around double the rate for the Pfizer/BioNTech vaccine”.
A alternativa C está incorreta. De acordo com o texto, não é possível inferir que uma vacina só é
segura quando a taxa de efeitos colaterais é mínima ou zero, pois o texto não afirma isso em
momento algum. Isso pode ser confirmado com o trecho “...rather than saying the vaccine is
“safe”, the EMA and MRHA emphasise that the benefits of the vaccine outweigh the risks of side-
effects.” (...em vez de dizer que a vacina é "segura", a EMA e a MRHA enfatizam que os benefícios
da vacina superam os riscos de efeitos colaterais).
A alternativa D está correta. De acordo com o texto, é possível inferir que a desconfiança e
hesitação sobre a vacina Oxford/AstraZeneca são previsíveis, considerando o cenário atual, assim
como esta opção indica. Isso pode ser confirmado com o trecho “It would not be surprising if
people are confused by this”.
A alternativa E está incorreta. De acordo com o texto, não é possível inferir que os danos são
maiores do que os benefícios, na vacina Oxford/AstraZeneca, mas sim, o contrário. Isso pode ser
confirmado com o trecho “The ratio of benefit to harms is high…”.
GABARITO: D

QUESTÃO 05 (ESCOLA NAVAL/INÉDITA) – In the excerpt “...with other notable reports including
palpitations (1,318), ‘feeling jittery’…” (paragraph 3), the word in bold means
(A) Calm
(B) Seasick
(C) Indecisive
(D) Headache
(E) Anxious

AULA 08 – ABBREVIATIONS, CONTRACTIONS AND IF CLAUSES 109


TEACHER ANDREA BELO

Comentários: A alternativa A está incorreta. A palavra “jittery” significa ansioso e não pode ser
comparada com a palavra “calm”, que significa calmo.
A alternativa B está incorreta. A palavra “jittery” significa ansioso e não pode ser comparada com
a palavra “seasick”, que significa enjoado.
A alternativa C está incorreta. A palavra “jittery” significa ansioso e não pode ser comparada com
a palavra “indecisive”, que significa indeciso.
A alternativa D está incorreta. A palavra “jittery” significa ansioso e não pode ser comparada com
a palavra “headache”, que significa dor de cabeça.
A alternativa E está correta. A palavra “jittery” significa ansioso e pode ser comparada com a
palavra “anxious”, que significa ansioso.
GABARITO: E

QUESTÃO 06 (ESCOLA NAVAL/INÉDITA) – Choose the correct alternative to complete the


paragraph below
It is profoundly difficult ____ grapple with risks whose stakes may include the global collapse of
civilisation, or even the extinction ____ humanity. ____ pandemic has shattered our illusions of
safety and reminded us that despite all the progress made in science and technology, we remain
vulnerable ____ catastrophes that can overturn our entire way of life. These are live possibilities,
not mere hypotheses, and our governments will have to confront them.
(Adapted from https://www.theguardian.com/commentisfree/2021/mar/23/covid-19-humanity-resilience-climate-ai-pandemic)

(A) To / of / the / to
(B) To / of / X / at
(C) X / of / the / for
(D) To / of / the / for
(E) At / of / the / for
Comentários: A primeira lacuna deve ser preenchida com “to”, para complementar o verbo
“grapple” (lidar), ou seja, “É profundamente difícil lidar com riscos...”.
A segunda lacuna deve ser preenchida com “of”, pois a frase se refere à extinção da humanidade,
ou seja, “...ou mesmo a extinção da humanidade”.
A terceira lacuna deve ser preenchida com “the”, pois a frase se refere a uma pandemia específica,
a pandemia que estamos vivendo; ou seja, “A pandemia destruiu nossas ilusões...”.
A quarta lacuna deve ser preenchida com “to”, pois a frase se refere a nós continuarmos
vulneráveis às catástrofes, ou seja, “...continuamos vulneráveis a catástrofes...”.
GABARITO: A

AULA 08 – ABBREVIATIONS, CONTRACTIONS AND IF CLAUSES 110


TEACHER ANDREA BELO

We can’t get back to normal until everyone else does


In the United States, despite worrying upticks in cases in some states, we’re well on the way to
vaccinating ourselves out of the catastrophic domestic covid-19 crisis. To get fully “back to
normal,” though, we’re also going to have to — gulp — help vaccinate the rest of the world.
The Biden administration has so turbocharged the process of getting vaccines into arms that,
recently, we’ve been administering as many as 3 million shots per day, and sometimes more.
According to President Biden, by the end of May we will have obtained enough vaccine doses “for
every adult in America.”
True, we should expect setbacks. It is maddening to see the images of huge spring-break crowds
in Miami, jammed together with few participants wearing masks properly. Those young people
are only helping the covid-19 virus survive and potentially to mutate. But they are unlikely to
become seriously ill if infected — and if they do take the virus home with them, at least there is a
growing chance their parents and grandparents will already be immunized.
I keep thinking, though, about other crowds that were part of the old normal. I think of the masses
of foreign tourists who gathered in the summer on the National Mall, waiting their turn to enter
the Smithsonian’s National Air and Space Museum.
One of the languages I used to pick out most frequently in tourist crowds was Portuguese, spoken
with various Brazilian accents. Before covid-19, almost 2 million Brazilians visited the United States
each year. But Brazil has seen more deaths from the virus than any country except the United
States; anyone who has been in Brazil over the past 14 days is forbidden to enter the country.
We’ll never really get back to normal as long as international borders are essentially closed. And I
don’t see how those borders can fully open until we can be sure that visitors are not bringing with
them covid-19 — perhaps in the form of variant strains that are more infectious, more deadly or
more resistant to our vaccines.
(Adapted from https://www.washingtonpost.com/opinions/the-only-way-for-the-us-to-truly-get-back-to-normal-is-to-vaccinate-theworld/2021/03/22/de1a8aa8-8b3d-11eb-a730-1b4ed9656258_story.html)

QUESTÃO 07 (ESCOLA NAVAL/INÉDITA) – In the phrase “But they are unlikely to become
seriously ill if infected…” (paragraph 3), the underlined word is a synonym for
(A) Strong
(B) Diseased
(C) Bad
(D) Sad
(E) Affected
Comentários: A alternativa A está incorreta. A palavra “ill” significa doença e não sinônimo para a
palavra “strong”, que significa forte.
A alternativa B está correta. A palavra “ill” significa doença e sinônimo para a palavra “diseased”,
que significa doente.
A alternativa C está incorreta. A palavra “ill” significa doença e não sinônimo para a palavra “bad”,
que significa mal.

AULA 08 – ABBREVIATIONS, CONTRACTIONS AND IF CLAUSES 111


TEACHER ANDREA BELO

A alternativa D está incorreta. A palavra “ill” significa doença e não sinônimo para a palavra “sad”,
que significa triste.
A alternativa E está incorreta. A palavra “ill” significa doença e não sinônimo para a palavra
“affected”, que significa afetado.
GABARITO: B

QUESTÃO 08 (ESCOLA NAVAL/INÉDITA) – The first paragraph of the text states that
(A) All US states have stabilized the number of cases
(B) In addition to the high cases, the US is doing poorly in its vaccination
(C) To return to normal, national vaccination is not enough, but global vaccination
(D) Vaccination in each country must be a national problem for each
(E) Some states in the US have a stable number of cases, all others are up in cases
Comentários: A alternativa A está incorreta. De acordo com o texto, não é correto afirmar que
todos os estados dos EUA estabilizaram o número de casos, mas sim, que alguns estão em alta no
número de casos. Isso pode ser confirmado com o trecho “In the United States, despite worrying
upticks in cases in some states…”.
A alternativa B está incorreta. De acordo com o texto, não é correto afirmar que além dos casos
elevados, os EUA estão mal na vacinação, mas sim, que alguns estados estão em alta, porém a
vacinação está correndo bem no país. Isso pode ser confirmado com o trecho “...we’re well on the
way to vaccinating ourselves out of the catastrophic domestic covid-19 crisis”.
A alternativa C está correta. De acordo com o texto, é correto afirmar que para voltar ao normal,
não basta a vacinação nacional, mas a vacinação global, assim como esta opção indica. Isso pode
ser confirmado com o trecho “To get fully ‘back to normal,’ though, we’re also going to have to —
gulp — help vaccinate the rest of the world”.
A alternativa D está incorreta. De acordo com o texto, não é correto afirmar que a vacinação em
cada país deve ser um problema nacional para cada, mas sim, que os países devem se ajudar para
combater a pandemia. Isso pode ser confirmado com o trecho “To get fully ‘back to normal,’
though, we’re also going to have to — gulp — help vaccinate the rest of the world”.
A alternativa E está incorreta. De acordo com o texto, não é correto afirmar que alguns estados
dos EUA têm um número estável de casos, todos os outros estão altos, mas sim, o contrário. Isso
pode ser confirmado com o trecho “In the United States, despite worrying upticks in cases in some
states, we’re well on the way to vaccinating ourselves out of the catastrophic domestic covid-19
crisis”.
GABARITO: C

AULA 08 – ABBREVIATIONS, CONTRACTIONS AND IF CLAUSES 112


TEACHER ANDREA BELO

QUESTÃO 09 (ESCOLA NAVAL/INÉDITA) – In the phrase “It is maddening to see the images
of huge spring-break crowds in Miami, jammed together with few participants wearing masks
properly” (paragraph 3), the underlined word is a synonym for
(A) Soothing
(B) Deranging
(C) Indifferent
(D) Coherent
(E) Weird
Comentários: A alternativa A está incorreta. A palavra “maddening” significa enlouquecedor e não
é sinônimo da palavra “soothing”, que significa reconfortante.
A alternativa B está correta. A palavra “maddening” significa enlouquecedor e é sinônimo da
palavra “deranging”, que significa pertubador.
A alternativa C está incorreta. A palavra “maddening” significa enlouquecedor e não é sinônimo
da palavra “indifferent”, que significa indiferente.
A alternativa D está incorreta. A palavra “maddening” significa enlouquecedor e não é sinônimo
da palavra “coherent”, que significa coerente.
A alternativa E está incorreta. A palavra “maddening” significa enlouquecedor e não é sinônimo
da palavra “weird”, que significa estranho.
GABARITO: B

QUESTÃO 10 (ESCOLA NAVAL/INÉDITA) – The text


(A) Develops the idea that vaccines should be state-owned
(B) Accuses vaccine-related nationalism
(C) Aims to explain how the entire population's vaccination process works
(D) Focuses on the idea that the pandemic will only end when everyone overcomes it
(E) Advocates the administration of vaccines individually by countries
Comentários: A alternativa A está incorreta. Não é correto afirmar que o texto desenvolve a ideia
de que as vacinas devem ser estatais, mas sim, que o texto se concentra na ideia de que a
pandemia só terminará quando todos a superarem. Isso pode ser confirmado com o trecho “To
get fully ‘back to normal,’ though, we’re also going to have to — gulp — help vaccinate the rest of
the world”.
A alternativa B está incorreta. Não é correto afirmar que o texto acusa nacionalismo relacionado
à vacina, mas sim, que o texto se concentra na ideia de que a pandemia só terminará quando
todos a superarem. Isso pode ser confirmado com o trecho “To get fully ‘back to normal,’ though,
we’re also going to have to — gulp — help vaccinate the rest of the world”.
A alternativa C está incorreta. Não é correto afirmar que o texto visa explicar como funciona todo
o processo de vacinação de toda a população, mas sim, que o texto se concentra na ideia de que

AULA 08 – ABBREVIATIONS, CONTRACTIONS AND IF CLAUSES 113


TEACHER ANDREA BELO

a pandemia só terminará quando todos a superarem. Isso pode ser confirmado com o trecho “To
get fully ‘back to normal,’ though, we’re also going to have to — gulp — help vaccinate the rest of
the world”.
A alternativa D está correta. É correto afirmar que o texto se concentra na ideia de que a pandemia
só terminará quando todos a superarem, assim como esta opção indica. Isso pode ser confirmado
com o trecho “To get fully ‘back to normal,’ though, we’re also going to have to — gulp — help
vaccinate the rest of the world”.
A alternativa E está incorreta. Não é correto afirmar que o texto advoga a administração de vacinas
individualmente pelos países, mas sim, que o texto se concentra na ideia de que a pandemia só
terminará quando todos a superarem. Isso pode ser confirmado com o trecho “To get fully ‘back
to normal,’ though, we’re also going to have to — gulp — help vaccinate the rest of the world”.
GABARITO: D

QUESTÕES EsPCEx
Read the text and answer the following questions 01 – 05.
THE HISTORY OF PIZZA
The modern pizza was originally invented in Naples, Italy but the word pizza is Greek in origin,
derived from the Greek word pēktos meaning solid or clotted. The ancient Greeks covered their
bread with oils, herbs and cheese. The first major innovation that led to flat bread pizza was the
use of tomato as a topping. It was common for the poor of the area around Naples to add tomato
to their yeast-based flat bread, and so the pizza began.
While it is difficult to say for sure ____ invented the pizza, it is however believed that modern pizza
was first made by baker Raffaele Esposito of Naples. In fact, a popular urban legend holds that the
archetypal pizza, Pizza Margherita, was invented in 1889, when the Royal Palace of Capodimonte
commissioned the Neapolitan pizzaiolo Raffaele Esposito to create a pizza in honor of the visiting
Queen Margherita. Of the three different pizzas he created, the Queen strongly preferred a pie
swathed in the colors of the Italian flag: red (tomato), green (basil), and white (mozzarella).
Supposedly, this kind of pizza was then named after the Queen as Pizza Margherita.
Later, the dish has become popular in many parts of the world:
• The first pizzeria, Antica Pizzeria Port'Alba, was opened in 1830 in Naples.

• In North America, The first pizzeria was opened in 1905 by Gennaro Lombardi at 53 1/3 Spring
Street in New York City.
• The first Pizza Hut, the chain of pizza restaurants appeared in the United States during the 1930s.

• Nowadays, many varieties of pizza exist worldwide, along with several dish variants based upon
pizza.

AULA 08 – ABBREVIATIONS, CONTRACTIONS AND IF CLAUSES 114


TEACHER ANDREA BELO

QUESTÃO 01 (EsPCEx/INÉDITA) – Check the correct statements according to the text:


I – The first pizza was invented in the 30’s.
II – The flat pizza covered with tomato was an innovation.
III – The first pizzeria in the USA was in New York City.
A) All of them are correct.
B) III is correct, but II is incorrect.
C) I and II are correct.
D) I and III are correct.
E) II and III are correct.
Comentários: A letra A está incorreta
A letra B está incorreta.
A letra C está incorreta.
A letra D está incorreta.
A letra E está correta. As opções II e III que estão corretas. II) A primeira pizza plana com tomate
foi uma inovação e III) A primeira pizzaria nos EUA foi em Nova Iorque.
GABARITO: E

QUESTÃO 02 (EsPCEx/INÉDITA) – The blank space in the text can be completed


A) who
B) what
C) how
D) when
E) which
Comentários: A letra A está correta. who = quem / a quem (pronome relativo usado para pessoas);
“é difícil saber QUEM inventou a pizza.
A letra B está incorreta. what = o que / qual.
A letra C está incorreta. how = como.
A letra D está incorreta. when = quando.
A letra E está incorreta. which = que / qual.
GABARITO: A

QUESTÃO 03 (EsPCEx/INÉDITA) – In the active voice the sentence: “The modern pizza was
originally invented in Naples”.
A) Someone invents the modern pizza in Naples.
B) Someone invented the modern pizza in Naples.
C) Someone has invented the modern pizza in Naples.
D) Someone was inventing the modern pizza in Naples.
E) Someome had invented the modern pizza in Naples.

AULA 08 – ABBREVIATIONS, CONTRACTIONS AND IF CLAUSES 115


TEACHER ANDREA BELO

Comentários: A letra A está incorreta.


A letra B está correta. A frase na voz passiva encontra-se com o verbo to be (auxiliar) no passado.
Portanto, a voz ativa será levada para o passado também. (was invented > invented)
A letra C está incorreta.
A letra D está incorreta.
A letra E está incorreta.
GABARITO: B

QUESTÃO 04 (EsPCEx/INÉDITA) – The sentence in bold in the text “and so the pizza began”,
expresses an idea of
A) comparison
B) exclusion
C) result
D) contrast
E) time
Comentários: A letra A está incorreta. Comparação.
A letra B está incorreta. Exclusão.
A letra C está correta. And so = e então; ideia de resultado ou conclusão.
A letra D está incorreta. Constraste.
A letra E está incorreta. Tempo.
GABARITO: C

QUESTÃO 05 (EsPCEx/INÉDITA) – The expression, “in fact”, in bold can be replaced by


A) otherwise
B) furthermore
C) however
D) indeed
E) instead of
Comentários: A letra A está incorreta. Otherwise = de outro modo.
A letra B está incorreta. Furthermore = além disso.
A letra C está incorreta. However = contudo.
A letra D está correta. Indeed = de fato, verdadeiramente; portanto sinônimo de “in fact”.
A letra E está incorreta. Instead of = ao invés de
GABARITO: D

AULA 08 – ABBREVIATIONS, CONTRACTIONS AND IF CLAUSES 116


TEACHER ANDREA BELO

CONSIDERAÇÕES FINAIS
Outra aula alcançada com sucesso – outro passo até a sua aprovação!
E, dia após dia, os tópicos aprendidos aumentam, seu conhecimento fica mais amplo, o
vocabulário que você conhece se estende e a tendência é melhorar e ser capaz de alcançar a
aprovação de fato.
Nota-se o avanço em seus estudos e, provavelmente, uma maior tranquilidade para
enfrentar os exercícios que surgem. E você vai se acostumando a equilibrar seus estudos de forma
sistematizada, estudando cada vez mais e com mais dedicação.
Outro detalhe importante para seu sucesso nos estudos é continuar fazendo listas de
palavras e verbos, participar das aulas complementares, fazer simulados e pedir ajuda quando
precisar.
Isso te ajudará nas questões futuras e torna você, como eu disse antes, um candidato mais bem
preparado e confiante para realizar uma excelente prova de vestibular.
É importante lembrar também do nosso Fórum de dúvidas, exclusivo do Estratégia
Militares. Será minha forma de responder você, esclarecer o que mais você precise saber para
que os conteúdos fiquem ainda mais claros em seus estudos, certo?
E, caso queira, acesse minhas redes sociais (Teacher Andrea Belo) para aprender mais
palavras e contar com dicas importantes, que colaboram diretamente com seus estudos dia após
dia.

AULA 08 – ABBREVIATIONS, CONTRACTIONS AND IF CLAUSES 117


TEACHER ANDREA BELO

REFERÊNCIAS BIBLIOGRÁFICAS
BARRETO, Tania Pedroza; GARRIDO, Maria Line; SILVA, João Antenor de C., Inglês Instrumental.
Leitura e compreensão de textos. Salvador, Ba UFBA, 1995, p. 64.
BROWN. H. Douglas. Principles of Language Learning and Teaching. Prentice Hall International,
1988.
COMPEDELLI, Samira Yousseff. Português, Literatura, Produção de texto & Gramática – São Paulo:
Ed. Saraiva, 2002.
CORREIA, Clese Mary P. Reading Specific Purposes. Salvador/ Ba: UFBA, 1999.
COSTEIRA, Adriana Araújo de M. Reading Comprehension Skills. João Pessoa/PB: ETFP, 1998.
CRYSTAL David. Cambridge University Press 1997. The Cambridge Encyclopedia of Language.
Cambridge University Press 1997
FREEMAN. Diane Larsen. MURCIA. Marianne Celce. The Grammar Book, 1999.
DYE, Joan., FRANFORT, Nancy. Spectrum II, III A Communicative Course in English. USA, Prentice
Hall, 1994.
FAVERO, Maria de Lourdes Albuquerque (org.). Dicionário de educadores no Brasil: da colônia aos
dias atuais. Rio de Janeiro: UFRJ, MEC, INEP, 1999.
FRANKPORT, Nancy & Dye Hoab. Spectrum II, III Prentice Hall Regents Englewood Cliffs, New Jersy,
1994.
GADELHA, Isabel Maria B. Inglês Instrumental: Leitura, Conscientização e Prática. Teresina:
EDUFFI, 2000.
GUANDALINI, Eiter Otávio. Técnicas de Leitura em Inglês: ESP – English For Specific Purposes:
estágio 1. São Paulo: Texto novo, 2002.
GRELLET, Françoise. Developing Reading Skills. Cambridge University Press, 1995
HOLAENDER, Arnon & Sanders Sidney. A complete English Course. São Paulo. Ed. Moderna, 1995.
HUTCHINSON, Tom & WATERS, Alan. English for Specific Purposes. Cambridge: Cambridge
University Press, 1996
KRASHEN. Stephen D. Second Language Acquisition and Second Language Learning, Prentice-Hall
International, 1988.
LAENG, Mauro. Dicionário de pedagogia. Lisboa: Dom Quixote, 1973.
LEFFA, Vilson J. Metodologia do ensino de línguas. In: BOHN, H.; VANDRESEN, P. (org.). Tópicos de
linguística aplicada: o ensino de línguas estrangeiras. Florianópolis: Editora da UFSC, 1988. p. 211-
231.
LIBERATO, Wilson. Compact English Book Inglês Ensino Médio. São Paulo: FTD, Vol. Único, 1998
Mc ARTHUR. The Oxford Companion to the English Language. Oxford University Press 1992
Fromkin. Victoria. An Introduction to Language
MARQUES, Amadeu. Inglês Série Brasil. ed. Atica. São Paulo: 2004. Vol. Único.

AULA 08 – ABBREVIATIONS, CONTRACTIONS AND IF CLAUSES 118


TEACHER ANDREA BELO

MURPHY, Raymond: Essencial Grammar in Use Oxford. New York Ed. Oxford University, 1997.
OLIVEIRA, Luciano Amaral. English For Tourism Students. Inglês para Estudantes de Turismo: São
Paulo, Rocca, 2001.
OLIVEIRA, Sara Rejane de F. Estratégias de leitura para Inglês Instrumental. Brasília: UNB, 1994.
QUINTANA, et alli. First Certificate. Master Class Oxford. New York, 2004: Ed. Oxford University.
PAULINO, Berenice F. et all. Leitura em textos em Inglês – Uma Abordagem Instrumental. Belo
Horizonte: Ed. Dos Autores, 1992.
PEREIRA, Edilberto Coelho. Inglês Instrumental. Teresina: ETFPI, 1998.
RODGES, Theodore. Jack C. Richards. Approaches and Methods in Language Teaching. Cambridge
University Press, 2001.
RODMAN Robert. Harcourt Brace 1993. English as a Global Language
STEWART, B., HAINES S. First Certificate, MasterClass. UK – Oxford 2004.
SILVA, João Antenor de C., GARRIDO, Maria Lina, BARRETO, Tânia Pedrosa. Inglês Instrumental:
Leitura e Compreensão de Textos. Salvador: Centro Editorial e Didático, UFBA. 1994
SOARES, Moacir Bretãs. Dicionário de legislação do ensino. 19.ed. Rio de Janeiro: FGV, 1981.
SOUZA, Adriana Srade F. Leitura em Língua Inglesa: Uma abordagem Instrumental. São Paulo:
Disal, 2005.
TUCK, Michael. Oxford Dictionary of Computing for Learners of English. Oxford: Oxford University
Press, 1996.
TOTIS, Verônica Pakrauskas. Língua Inglesa: leitura. São Paulo: Cortez, 1991.

Livros eletrônicos:
Dicionário Houaiss da Língua Portuguesa, Editora Objetiva, 2001.
MOURãO, Janaína Pereira. "Skimming x Scanning"; Brasil Escola. Disponível em
<https://brasilescola.uol.com.br/ingles/skimming-x-scanning.htm>. Acesso em 20 de março de
2019.
www.newsweek.com - Acesso em 18 de março de 2019.
http://www.galaor.com.br/tecnicas-de-leitura/ - Acesso em 19 de março de 2019.
Expressões Idiomáticas (continuação)" em Só Língua Inglesa. Virtuous Tecnologia da
Informação,2008-2019. Consultado em 03/04/2019 às 22:09. Disponível na Internet em
http://www.solinguainglesa.com.br/conteudo/Expressoes5.php

AULA 08 – ABBREVIATIONS, CONTRACTIONS AND IF CLAUSES 119


TEACHER ANDREA BELO

TRADUÇÕES
Hanging out
R. Jordania
In American cities, teenagers like to spend time together – “hang out”, as they say– at drugstore,
luncheonettes, or ice cream parlors.
Often, they don’t even meet inside, but gather on the sidewalk in front of the store. From time to
time they go in for coffee, milk, ice cream. They also like to play the pinball machines.
Most parents disapprove of their children’s “hanging out” that way. They consider it a waste of
time, which could be better used doing homework, working at a part-time job, or helping in the
house.
Adapted from life in the USA

Saindo
R. Jordania
Nas cidades americanas, os adolescentes gostam de passar tempo juntos - “passear”, como se
costuma dizer - em drogarias, lanchonetes ou sorveterias.
Muitas vezes, eles nem mesmo se encontram dentro, mas se reúnem na calçada em frente à loja.
De vez em quando vão tomar café, leite, sorvete. Eles também gostam de jogar nas máquinas de
pinball.
A maioria dos pais desaprova que seus filhos "saiam" dessa maneira. Eles consideram uma perda
de tempo, que poderia ser mais bem aproveitada fazendo o dever de casa, trabalhando meio
período ou ajudando na casa.

Is it a good idea for a student to have a job? Why or why not?


Brandon Smith
I’m a junior in high school, and I have a part-time job in a restaurant. I wash dishes on Saturdays
and Sundays from 8:00 until 4:00. I earn $5.50 an hour. It isn’t much money, but I save almost
every penny! I want to go to a good university, and the cost goes up every year. Of course, I spend
some money when I go out on Saturday nights.
Lauren Russel
I’m a senior in high school. I have a job as a cashier in a grocery store. The job pays well – about
$6.75 an hour. I work every weeknight after school from 4:00 until 8:00. I don’t have time for
homework, and my grades aren’t very good this year. But I have to work, or I can’t buy nice clothes
and I can’t go out on Saturday nights. Also, a car costs a lot of money.
Erica Davis

AULA 08 – ABBREVIATIONS, CONTRACTIONS AND IF CLAUSES 120


TEACHER ANDREA BELO

I’m a freshman in college. College is very expensive, so I work in a law office for three hours every
weekday afternoon. I make photocopies, file papers, and sort mail for $8.25 an hour. The job gives
me good experience because I want to be a lawyer someday. But I don’t want to work every
semester. I need time to study.
Adapted from Interchange

É uma boa ideia um aluno ter um emprego? Por que ou por que não?
Brandon Smith
Eu sou um júnior no ensino médio e tenho um emprego de meio período em um restaurante. Eu
lavo pratos aos sábados e domingos das 8:00 às 4:00. Eu ganho $ 5,50 por hora. Não é muito
dinheiro, mas economizo quase cada centavo! Quero ir para uma boa universidade e o custo
aumenta a cada ano. Claro, gasto algum dinheiro quando saio nas noites de sábado.
Lauren Russel
Estou no último ano do ensino médio. Eu trabalho como caixa em uma mercearia. O trabalho paga
bem - cerca de US $ 6,75 por hora. Eu trabalho todas as noites da semana depois da escola das
4:00 às 8:00. Não tenho tempo para o dever de casa e minhas notas não estão muito boas este
ano. Mas eu tenho que trabalhar, ou não posso comprar roupas bonitas e não posso sair nas
noites de sábado. Além disso, um carro custa muito dinheiro.
Erica Davis
Eu sou um calouro na faculdade. A faculdade é muito cara, então eu trabalho em um escritório
de advocacia por três horas todas as tardes dos dias de semana. Eu faço fotocópias, arquivo papéis
e classifico correspondências por US $ 8,25 a hora. O trabalho me dá uma boa experiência porque
quero ser advogado um dia. Mas eu não quero trabalhar todo semestre. Preciso de tempo para
estudar.

SUNDAY MORNING
Sunday morning, rain is falling
Steal some covers, share some skin
Clouds are shrouding us in moments unforgettable
You twist to fit the mold that I _____ in
But things just get so crazy
Living life gets hard to do
And I would gladly hit the road, get up and go if I knew
That someday it would lead me back to you
That someday it would lead me back to you
Music Sunday morning by Marron 5

AULA 08 – ABBREVIATIONS, CONTRACTIONS AND IF CLAUSES 121


TEACHER ANDREA BELO

DOMINGO DE MANHÃ
Domingo de manhã, chuva está caindo
Roube algumas capas, compartilhe um pouco de pele
Nuvens nos envolvem em momentos inesquecíveis
Você torce para se encaixar no molde em que eu _____
Mas as coisas ficam tão loucas
Viver a vida fica difícil de fazer
E eu ficaria feliz em pegar a estrada, levantar e ir se eu soubesse
Que um dia isso me levaria de volta a você
Que um dia isso me levaria de volta a você

MIRRORS
Aren't you something to admire,
Because your shine is something like a mirror
And I can’t help but notice, you reflect in this heart of mine
If you ever feel alone and the glare makes me hard to find
Just know that I'm always parallel out on the other side
Beause with your hand in my hand and a pocket full of soul
I can tell you theres no place we couldn't go
Just put your hand on the glass
I'm here trying to pull you through you just gotta be strong
Because I don't wanna lose you now
I'm lookin' right at the other half of me
The vacancy that sat in my heart
Is a space, but now you're home
Music Mirrors by Justin Timberlake

ESPELHOS
Você não é algo para admirar,
Porque seu brilho é algo como um espelho
E eu não posso deixar de notar, você reflete neste meu coração
Se você se sentir sozinho e o brilho me tornar difícil de encontrar

AULA 08 – ABBREVIATIONS, CONTRACTIONS AND IF CLAUSES 122


TEACHER ANDREA BELO

Apenas saiba que estou sempre paralelo do outro lado


Porque com sua mão na minha mão e um bolso cheio de alma
Eu posso te dizer que não há nenhum lugar onde não possamos ir
É só colocar a mão no vidro
Estou aqui tentando puxar você só tem que ser forte
Porque eu não quero te perder agora
Estou olhando bem para a outra metade de mim
A vaga que sentou em meu coração
É um espaço, mas agora você está em casa

The delta variant is surging. How should that change how we live?
It’s been a week since the Centers for Disease Control and Prevention issued new masking
guidance based on evidence that vaccinated people can become infected and transmit the more
contagious delta variant to others. Many people who thought the vaccines allowed them to return
to pre-pandemic normal are now asking whether they need to change how they go about their
daily lives.
While delta does change the risk calculus, it doesn’t mean that we have go to back to hunkering
down at home. When deciding which activities to engage in, vaccinated people should consider
two factors: the medical risk of your household and the value of the activities to you.
Someone who is vaccinated, generally healthy, and either living alone or sharing a household with
others who are also vaccinated and healthy might decide that they are protected enough that they
won’t change anything. According to CDC data, the vaccinated are approximately eight times less
likely to become infected than the unvaccinated. Even if they contract covid-19, chances are
excellent that a vaccinated person will experience symptoms akin to the common cold; after all,
the vaccines reduce the chance of severe illness by a whopping 25-fold.
(Adapted from https://www.washingtonpost.com/opinions/2021/08/04/delta-variant-risk-assessment-leana-wen/)

A variante delta está aumentando. Como isso deve mudar a forma como vivemos?
Já se passou uma semana desde que os Centros de Controle e Prevenção de Doenças emitiram
uma nova orientação de mascaramento com base em evidências de que as pessoas vacinadas
podem ser infectadas e transmitir a variante delta mais contagiosa a outras pessoas. Muitas
pessoas que pensavam que as vacinas lhes permitiam voltar ao estado normal pré-pandêmico
agora estão se perguntando se precisam mudar a forma como vivem suas vidas diárias.
Embora o delta mude o cálculo de risco, isso não significa que devemos voltar a nos acomodar em
casa. Ao decidir quais atividades participar, as pessoas vacinadas devem levar em consideração
dois fatores: o risco médico de sua família e o valor das atividades para você.

AULA 08 – ABBREVIATIONS, CONTRACTIONS AND IF CLAUSES 123


TEACHER ANDREA BELO

Alguém que é vacinado, geralmente saudável, e vive sozinho ou compartilha uma casa com outras
pessoas que também estão vacinadas e saudáveis pode decidir que está protegido o suficiente
para não mudar nada. De acordo com dados do CDC, os vacinados têm aproximadamente oito
vezes menos probabilidade de serem infectados do que os não vacinados. Mesmo que contraiam
covid-19, as chances são excelentes de que uma pessoa vacinada apresente sintomas semelhantes
aos do resfriado comum; afinal, as vacinas reduzem a chance de doenças graves em 25 vezes.

Therapy via Zoom should make mental healthcare available for all – it hasn’t
Whether with a private therapist on Zoom, through an app that daily reminds us to log our
emotions, or in a back-andforth with a chatbot, teletherapy is often proffered as a catch-all salve
for our current mental healthcare crises. Remote treatment is touted as an efficient way to reach
more patients in a time of extreme difficulty, an intimate intervention that can scale.
During the on-and-off mandated social distancing that has marked the past 18 months of the
pandemic, teletherapy has shed its status as a minor form of care to become, at times, the only
thing on offer. The popularity of remote therapeutic sessions has soared in the US and the “users”
we once called patients are increasingly comfortable with such practices, and in some cases even
prefer them.
Having a therapist see a patient on Zoom or condensing treatment to self-tracking and AI
interfaces may be recent innovations, but the broad notion that technology and distanced
processes will solve our woes is nothing new. We have been turning to forms of technology to
deliver mental health services for more than 100 years. From 19th-century written cures delivered
by post and ad hoc telephone hotlines to the continuing elusive work to create an AI shrink, there
have been numerous mediated, networked, and remote relationships used in attempts to fix
longstanding problems with therapeutic provision. While those problems have obviously evolved
over time, they have also stayed relatively the same: good care is expensive and in short supply,
and barely begins to meet an overwhelming demand.
(Adapted from https://www.theguardian.com/commentisfree/2021/aug/03/therapy-zoom-mental-healthcare-remote-treatment)

A terapia via Zoom deve tornar a saúde mental disponível para todos – não
Seja com um terapeuta particular no Zoom, por meio de um aplicativo que nos lembra diariamente
de registrar nossas emoções ou nas idas e vindas com um chatbot, a teleterapia é frequentemente
oferecida como um bálsamo abrangente para nossas crises atuais de saúde mental. O tratamento
remoto é apontado como uma forma eficiente de atingir mais pacientes em um momento de
extrema dificuldade, uma intervenção íntima que pode escalar.
Durante o distanciamento social obrigatório intermitente que marcou os últimos 18 meses da
pandemia, a teleterapia deixou de ser uma forma secundária de cuidado para se tornar, às vezes,
a única coisa oferecida. A popularidade das sessões terapêuticas remotas disparou nos Estados
Unidos e os “usuários” que antes chamávamos de pacientes estão cada vez mais confortáveis com
essas práticas e, em alguns casos, até as preferem.
Fazer com que um terapeuta veja um paciente no Zoom ou condensar o tratamento em interfaces
de auto-rastreamento e IA podem ser inovações recentes, mas a noção ampla de que a tecnologia

AULA 08 – ABBREVIATIONS, CONTRACTIONS AND IF CLAUSES 124


TEACHER ANDREA BELO

e os processos distantes resolverão nossos problemas não é nada novo. Temos recorrido a formas
de tecnologia para fornecer serviços de saúde mental há mais de 100 anos. De curas escritas do
século 19 entregues por linhas diretas post e ad hoc ao trabalho elusivo contínuo para criar um
analista de IA, tem havido numerosos relacionamentos mediados, em rede e remotos usados nas
tentativas de resolver problemas de longa data com provisão terapêutica. Embora esses
problemas tenham obviamente evoluído com o tempo, eles também permaneceram
relativamente os mesmos: um bom atendimento é caro e escasso, e mal começa a atender a uma
demanda esmagadora.

Parosmia: 'The smells and tastes we still miss, long after Covid'
Back in November I realised my chicken pasta tasted like washing-up liquid. I haven't eaten meat
since - mac and cheese, green grapes and baby rusks have become my staples.
I don't know whether I will ever be able to enjoy a Nando's medium-spiced chicken butterfly again
as now it smells and tastes foul, like something alien. I have struggled to come to terms with this.
If my smell goes back to normal, I'll never ever take a Nando's with friends for granted again.
Chanay Knight, 21, Birmingham
Caught Covid in October, developed parosmia in November I miss a simple pleasure - breakfast in
bed brought to me every morning by the husband I love. Deeply aromatic coffee with hot, frothy,
milk. Good bread, crisp and deep gold, slightly charred at the edges with butter or tangy
marmalade.
In spring we both caught Covid and he was hospitalised. I struggled down to the kitchen to make
coffee and toast for myself. Exhausted by such a simple task, I clung to the ritual and pictured him
beside me.
We both recovered, but coffee and toast is now repulsive to me - like a field just sprayed with
manure… unpleasant with a sweet fermented smell on top.
I've lost something that meant so much more to me than just breakfast.
Wendy Thompson, 59, Tameside
Caught Covid in May, developed parosmia in October
(Adapted from https://www.bbc.com/news/stories-55936729)

Parosmia: 'Os cheiros e sabores que ainda sentimos falta, muito depois de Covid'
Em novembro, percebi que meu macarrão com frango tinha gosto de detergente. Não comi carne
desde então - macarrão com queijo, uvas verdes e biscoitos de bebê se tornaram meus produtos
básicos.
Não sei se algum dia poderei saborear uma borboleta de frango de tempero médio do Nando
novamente, pois agora tem um cheiro e um gosto horrível, como algo estranho. Tenho lutado
para chegar a um acordo com isso.
Se meu cheiro voltar ao normal, nunca mais vou tomar um Nando's com amigos como garantido.
Chanay Knight, 21, Birmingham

AULA 08 – ABBREVIATIONS, CONTRACTIONS AND IF CLAUSES 125


TEACHER ANDREA BELO

Peguei Covid em outubro, desenvolveu parosmia em novembro. Sinto falta de um prazer simples
- o café da manhã na cama trazido para mim todas as manhãs pelo marido que amo. Café
profundamente aromático com leite quente e espumoso. Pão bom, crocante e dourado profundo,
ligeiramente chamuscado nas pontas com manteiga ou marmelada picante.
Na primavera, nós dois pegamos Covid e ele foi hospitalizado. Lutei para baixo até a cozinha para
fazer café e torradas para mim. Exausto por uma tarefa tão simples, agarrei-me ao ritual e
imaginei-o ao meu lado.
Nós dois nos recuperamos, mas café e torradas agora são repulsivos para mim - como um campo
apenas borrifado com estrume ... desagradável com um cheiro doce de fermentado por cima.
Perdi algo que significava muito mais para mim do que apenas o café da manhã.
Wendy Thompson, 59, Tameside
Peguei Covid em maio, desenvolveu parosmia em outubro

What can we take with us from lockdown to make life better than before?
Spring has sprung, albeit a little early. The weekend brought blue skies, daffodils bursting into
flower, the drone of lawnmowers coming back to life. The natural world is waking up fast, in
uncanny synchronicity with a nation now readying itself to emerge from Covid hibernation. Like
all jaded hacks, I rolled my eyes when the prime minister started waxing lyrical last week about
the crocus of hope pushing up through the frost of lockdown. But dammit, the crocuses are out
now, and if this strangely unfamiliar emotion is not hope, then it’s hard to know what else it is.
The coming of spring always paves the way, on some atavistic level, for the imagining of summer.
But this time the vaccine (touch wood, cross fingers, pray that the missing unknown carrier of the
Brazilian variant is tracked down before spreading it) provides some more rational grounds for
daring to imagine new beginnings.
The only way to keep one’s sanity in lockdown is to take life one day at a time, like an alcoholic in
recovery, and never think about how much longer there is to go. But suddenly it seems possible
to stop dwelling on everything that’s out of bounds and imagine having choices again. Travel
agents are reporting a rush of possibly over-optimistic bookings; fashion magazines are suddenly
full of heels and frivolousness, rather than clothes for collapsing on the sofa in. And for parents
who’ve been working from home, hunting down the PE kit ahead of next week’s return to school
is a reminder that one day our own presence may be required in a formal office again.
(Adapted from https://www.theguardian.com/commentisfree/2021/mar/01/covid-life-flexible-working-lockdown)

O que podemos levar conosco do bloqueio para tornar a vida melhor do que antes?
A primavera chegou, embora um pouco cedo. O fim de semana trouxe céu azul, narcisos
explodindo em flores, o zumbido dos cortadores de grama voltando à vida. O mundo natural está
despertando rápido, em sincronicidade incrível com uma nação que agora se prepara para sair da
hibernação de Covid. Como todos os hacks cansados, revirei os olhos quando o primeiro-ministro
começou a ficar lírico na semana passada sobre o açafrão de esperança que se erguia através da

AULA 08 – ABBREVIATIONS, CONTRACTIONS AND IF CLAUSES 126


TEACHER ANDREA BELO

geada do bloqueio. Mas, caramba, os açafrões estão fora agora, e se essa emoção estranhamente
desconhecida não é esperança, então é difícil saber o que mais é.
A chegada da primavera sempre abre caminho, em algum nível atávico, para a imaginação do
verão. Mas desta vez a vacina (toque na madeira, cruze os dedos, reze para que o desconhecido
portador da variante brasileira seja rastreado antes de espalhá-la) fornece alguns fundamentos
mais racionais para ousar imaginar novos começos.
A única maneira de manter a sanidade em bloqueio é tirar a vida um dia de cada vez, como um
alcoólatra em recuperação, e nunca pensar em quanto tempo ainda falta. Mas, de repente, parece
possível parar de pensar em tudo que está fora dos limites e imaginar ter opções novamente. Os
agentes de viagens estão relatando uma onda de reservas possivelmente otimistas demais;
revistas de moda de repente estão cheias de saltos altos e frivolidade, em vez de roupas para
desabar no sofá. E para os pais que têm trabalhado em casa, procurar o kit de educação física
antes do retorno às aulas na semana que vem é um lembrete de que um dia nosso a própria
presença pode ser exigida em um escritório formal novamente.

Coronavirus crisis unlikely to be over by the end of the year, WHO warns
Despite the spread of Covid-19 being slowed in some countries due to lockdowns and vaccination
programs, it is “premature” and “unrealistic” to the think the pandemic will be over by the end of
the year, the World Health Organization’s executive director of emergency services has said.
Speaking at a press briefing Geneva, Dr Michael Ryan said while vaccinating the most vulnerable
people, including healthcare workers, would help remove the “tragedy and fear” from the
situation, and would help to ease pressure on hospitals, the “virus is very much in control”.
“It will be very premature, and I think unrealistic, to think that we’re going to finish with this virus
by the end of the year,” Ryan said.
“If the vaccines begin to impact not only on death and not only on hospitalisation, but have a
significant impact on transmission dynamics and transmission risk, then I believe we will
accelerate toward controlling this pandemic.”
(Adapted from https://www.theguardian.com/world/2021/mar/02/coronavirus-crisis-unlikely-to-be-over-by-the-end-of-the-year-who-warns)

A crise do coronavírus dificilmente terminará até o final do ano, alerta a OMS


Apesar de a disseminação da Covid-19 ter sido desacelerada em alguns países devido a bloqueios
e programas de vacinação, é “prematuro” e “irreal” pensar que a pandemia acabará até o final do
ano, o diretor executivo da Organização Mundial de Saúde da serviços de emergência disse.
Falando em uma coletiva de imprensa em Genebra, o Dr. Michael Ryan disse que vacinar as
pessoas mais vulneráveis, incluindo profissionais de saúde, ajudaria a remover a "tragédia e o
medo" da situação e ajudaria a aliviar a pressão sobre os hospitais, o "vírus é muito importante
no controle".
“Será muito prematuro, e acho irreal, pensar que vamos acabar com esse vírus até o final do ano”,
disse Ryan.

AULA 08 – ABBREVIATIONS, CONTRACTIONS AND IF CLAUSES 127


TEACHER ANDREA BELO

“Se as vacinas começarem a ter impacto não apenas na morte e não apenas na hospitalização,
mas tiverem um impacto significativo na dinâmica de transmissão e no risco de transmissão,
então acredito que iremos acelerar o controle desta pandemia.”

‘We are on the eve of a genocide’: Brazil urged to save Amazon from Covid-19
Brazil’s leaders must take immediate action to save the country’s indigenous peoples from a
Covid-19 “genocide”, a global coalition of artists, celebrities, scientists, and intellectuals has said.
In an open letter to Brazil, figures including Madonna, Oprah Winfrey, Brad Pitt, David Hockney
and Paul McCartney warned the pandemic meant indigenous communities in the Amazon faced
“an extreme threat to their very survival”.
“Five centuries ago, these ethnic groups were decimated by diseases brought by European
colonisers … Now, with this new scourge spreading rapidly across Brazil … [they] may disappear
completely since they have no means of combating Covid-19,” they wrote.
The organiser of the petition, the Brazilian photojournalist Sebastião Salgado, said trespassers
including wildcat gold miners and illegal loggers must be expelled immediately from indigenous
lands to stop them importing an illness that has killed more than 240,000 people around the
world, including 6,750 in Brazil. “We are on the eve of a genocide,” Salgado, who has spent nearly
four decades documenting the Amazon and its inhabitants, told the Guardian.
Even before Covid-19, Brazil’s indigenous peoples were locked in what activists call a historic
struggle for survival. Fears Covid-19 could devastate indigenous communities grew last month
when the death of a Yanomami teenager revived horrific memories of epidemics caused by
roadbuilders and gold prospectors in the 1970s and 80s. The Brazilian city so far worst hit by
coronavirus is Manaus, the capital of Amazonas, the state where part of the Yanomami reserve is
located.
Salgado – who is calling for the creation of an army-led taskforce to evict intruders from protected
areas – believes international pressure could force the government to act.
Adapted from https://www.theguardian.com/world/2020/may/03/eve-of-genocide-brazil-urged-save-amazon-tribes-covid-19-sebastiao-salgado

‘Estamos às vésperas de um genocídio’: o Brasil pede para salvar a Amazônia de Covid-19


Os líderes do Brasil devem tomar medidas imediatas para salvar os povos indígenas do país de um
"genocídio" Covid-19, disse uma coalizão global de artistas, celebridades, cientistas e intelectuais.
Em uma carta aberta ao Brasil, figuras como Madonna, Oprah Winfrey, Brad Pitt, David Hockney
e Paul McCartney alertaram que a pandemia significa que as comunidades indígenas na Amazônia
enfrentam “uma ameaça extrema à sua própria sobrevivência”.
“Há cinco séculos essas etnias foram dizimadas por doenças trazidas pelos colonizadores
europeus ... Agora, com esse novo flagelo se espalhando rapidamente pelo Brasil ... [elas] podem
desaparecer por completo, pois não têm como combater a Covid-19”, escreveram.
O organizador da petição, o fotojornalista brasileiro Sebastião Salgado, disse que invasores,
incluindo garimpeiros e madeireiros ilegais, devem ser expulsos imediatamente de terras

AULA 08 – ABBREVIATIONS, CONTRACTIONS AND IF CLAUSES 128


TEACHER ANDREA BELO

indígenas para impedi-los de importar uma doença que já matou mais de 240.000 pessoas em
todo o mundo, incluindo 6.750 no Brasil. “Estamos às vésperas de um genocídio”, disse Salgado,
que passou quase quatro décadas documentando a Amazônia e seus habitantes, ao Guardian.
Mesmo antes da Covid-19, os povos indígenas do Brasil estavam travados no que os ativistas
chamam de uma luta histórica pela sobrevivência. Os temores de que a Covid-19 pudesse devastar
comunidades indígenas aumentaram no mês passado, quando a morte de um adolescente
Yanomami reviveu memórias horríveis de epidemias causadas por construtores de estradas e
garimpeiros de ouro nas décadas de 1970 e 80. A cidade brasileira mais atingida pelo coronavírus
é Manaus, capital do Amazonas, estado onde está localizada parte da reserva Yanomami.
Salgado - que pede a criação de uma força-tarefa comandada pelo exército para expulsar intrusos
de áreas protegidas - acredita que a pressão internacional pode forçar o governo a agir.

The US Navy recorded three videos that show UFOs. A UFO is a flying object in the sky that
scientists cannot easily explain. Sometimes, UFOs are clouds or airplanes; however, many people
believe that they are spacecraft from other planets.
A UFO research group published the videos in 2018, and the US Navy said that the videos were
real. However, some people did not believe them. US officials decided to publish the videos again
because they wanted to show that the videos were real. The officials said that they could not
explain the origin of the objects in the videos. They consider the objects as unidentified.
Adapted from https://www.newsinlevels.com/.

A Marinha dos Estados Unidos gravou três vídeos que mostram OVNIs. Um OVNI é um objeto
voador no céu que os cientistas não conseguem explicar facilmente. Às vezes, os OVNIs são
nuvens ou aviões; no entanto, muitas pessoas acreditam que são espaçonaves de outros planetas.
Um grupo de pesquisa de OVNIs publicou os vídeos em 2018, e a Marinha dos Estados Unidos
disse que os vídeos eram reais. No entanto, algumas pessoas não acreditaram neles. As
autoridades americanas decidiram publicar os vídeos novamente porque queriam mostrar que os
vídeos eram reais. Os funcionários disseram que não sabiam explicar a origem dos objetos dos
vídeos. Eles consideram os objetos como não identificados.

Vaccine passports are less a threat to liberty than a mark of solidarity


Anti-vaxxers in France and elsewhere claim personal freedom. But what of brotherhood?
In France over the past few weeks, the topic of vaccine passports has induced an avalanche of
outrage. Opposition to the measure has united both the hard left and right, with more than
200,000 people taking to the streets to express their contempt. In the kaleidoscope of disparate
groups involved, the only unifying banner is the assertion that Emmanuel Macron’s policy is an
infringement of the French tenet of liberté.

AULA 08 – ABBREVIATIONS, CONTRACTIONS AND IF CLAUSES 129


TEACHER ANDREA BELO

Nor is France unique in facing such resistance. In the United States, mask and vaccine mandates
have generated passionate opprobrium and legal action.
Those resolutely opposed to anything styled as a vaccination passport tend to frame the issue as
a solely personal choice. That can seem superficially reasonable, but it highlights a crucial
misunderstanding – a presumption that vaccination is solely an individual boon. On the contrary,
immunisation is, at heart, a public health measure, implemented to reduce incidence and burden
of disease at a population level. That it has huge individual benefit is undeniable but viewing
vaccination through this reductive, individualistic lens fundamentally distorts the reality that it is
about much more than protecting oneself.
Immunisation collectively reduces reservoirs for disease, providing a firewall that protects
vulnerable members of society. While a Covid infection might not do a young, healthy person
lasting harm, their passing on that infection could inflict substantial, even fatal, damage to
vulnerable people.
This is a consideration frequently missed in the arguments about proof of vaccination in public
spaces. Those decrying it as an infringement of their liberties fail to realise that others have a
reasonable expectation that they should not be needlessly exposed to a potentially deadly virus
if it can be avoided.
(Adapted from https://www.theguardian.com/commentisfree/2021/aug/15/vaccine-passports-are-less-a-threat-to-liberty-than-a-mark-of-solidarity)

Passaportes de vacinas são menos uma ameaça à liberdade do que uma marca de solidariedade
Os antivaxxers na França e em outros lugares reivindicam liberdade pessoal. Mas e a irmandade?
Na França, nas últimas semanas, o tema dos passaportes para vacinas provocou uma avalanche
de indignação. A oposição à medida uniu a extrema direita e esquerda, com mais de 200.000
pessoas tomando as ruas para expressar seu desprezo. No caleidoscópio de grupos díspares
envolvidos, a única bandeira unificadora é a afirmação de que a política de Emmanuel Macron é
uma violação do princípio francês de liberté.
A França também não é a única a enfrentar tal resistência. Nos Estados Unidos, os mandatos de
máscaras e vacinas geraram opróbrio apaixonado e ações legais.
Aqueles que se opõem resolutamente a qualquer coisa denominada passaporte de vacinação
tendem a enquadrar a questão como uma escolha exclusivamente pessoal. Isso pode parecer
superficialmente razoável, mas destaca um mal-entendido crucial - a presunção de que a
vacinação é apenas um benefício individual. Pelo contrário, a imunização é, no fundo, uma medida
de saúde pública, implementada para reduzir a incidência e o peso da doença a nível da
população. Que tem um enorme benefício individual é inegável, mas ver a vacinação por meio
dessa lente redutora e individualista distorce fundamentalmente a realidade de que se trata de
muito mais do que proteger a si mesmo.
A imunização reduz coletivamente os reservatórios de doenças, fornecendo um firewall que
protege os membros vulneráveis da sociedade. Embora uma infecção por Covid possa não causar
danos duradouros a uma pessoa jovem e saudável, a transmissão dessa infecção pode causar
danos substanciais, até mesmo fatais, a pessoas vulneráveis.

AULA 08 – ABBREVIATIONS, CONTRACTIONS AND IF CLAUSES 130


TEACHER ANDREA BELO

Essa é uma consideração frequentemente omitida nos argumentos sobre a prova de vacinação
em espaços públicos. Aqueles que o denunciam como uma violação de suas liberdades deixam de
perceber que os outros têm uma expectativa razoável de que não devem ser expostos
desnecessariamente a um vírus potencialmente mortal se ele puder ser evitado.

Israel extends Covid restrictions to three-year-olds as cases surge


Israel is now requiring anyone over the age of three to show proof of vaccination or a negative
Covid-19 test before entering many indoor spaces, as it tackles a sharp rise in infections.
Restaurants, cafes, museums, libraries, gyms and pools are among the venues covered by the
"Green Pass" system.
However, proof of immunity is not needed to go into shops or malls.
The country's Covid-19 tsar said it was "at war" with the virus despite its world-leading vaccination
programme.
"Our morbidity is rising day by day," Salman Zarka told a parliamentary committee on Wednesday,
according to the Jerusalem Post.
The next two weeks leading up to the Jewish New Year festival of Rosh Hashanah on 6 September
would be "critical", he warned.
Israel has seen a surge of infections driven by the more contagious Delta variant since late June.
The health ministry reported some 7,870 new Covid-19 cases on Tuesday, which was slightly down
on Monday's sixmonth daily record of 8,752.
More than 120 people have died after contracting the virus in the past week - double the monthly
total recorded in July - and 600 people are in a serious or critical condition in hospital.
The government has sought to combat the surge by reinstating the restrictions it lifted in mid-
June and by bringing back the Green Pass, which shows whether someone has been fully
vaccinated, has recently recovered from Covid-19, or tested negative in the previous 24 hours.
Before Wednesday only children aged 12 and over, who have been eligible to get a vaccine since
June, and adults were required to present a Green Pass.
It will now also apply to children between the ages of three and 11. Their tests will be funded by
the government as they are ineligible for vaccination, unless they are five or older and are
considered are at significant risk from Covid-19.
The approximately one million residents - about 11% of the population - who have not been
vaccinated despite being eligible must pay for their own tests.
Israel has also begun giving third doses of the Pfizer-BioNTech vaccine to people over 50, medical
workers and those with underlying health conditions.
So far, some 1.1 million eligible people have received their booster shots.
Israeli healthcare provider Maccabi, which covers about a quarter of the population, reported on
Wednesday that a third Pfizer dose was 86% effective at preventing Covid-19 infection in people
over 60.
Mr Zarka also noted that no-one who was currently in a critical condition in hospital had received
a booster shot.
(Adapted from https://www.bbc.com/news/world-middle-east-58245285)

AULA 08 – ABBREVIATIONS, CONTRACTIONS AND IF CLAUSES 131


TEACHER ANDREA BELO

Israel estende as restrições da Covid a crianças de três anos conforme os casos aumentam
Israel agora exige que qualquer pessoa com mais de três anos de idade mostre prova de
vacinação ou um teste negativo de Covid-19 antes de entrar em muitos espaços internos, já
que isso enfrenta um aumento acentuado nas infecções.
Restaurantes, cafés, museus, bibliotecas, ginásios e piscinas estão entre os locais cobertos pelo
sistema "Green Pass".
No entanto, a prova de imunidade não é necessária para entrar em lojas ou shoppings.
O czar Covid-19 do país disse que está "em guerra" com o vírus, apesar de seu programa de
vacinação líder mundial.
"Nossa morbidade está aumentando dia a dia", disse Salman Zarka a um comitê parlamentar na
quarta-feira, de acordo com o Jerusalem Post.
As próximas duas semanas que antecederam o festival judaico do Ano Novo de Rosh Hashanah
em 6 de setembro serão "críticas", ele advertiu.
Israel viu um surto de infecções causadas pela variante Delta, mais contagiosa, desde o final de
junho.
O ministério da saúde relatou cerca de 7.870 novos casos de Covid-19 na terça-feira, o que
diminuiu ligeiramente em relação ao recorde diário de seis meses de segunda-feira de 8.752.
Mais de 120 pessoas morreram após contrair o vírus na semana passada - o dobro do total mensal
registrado em julho - e 600 pessoas estão em estado grave ou crítico no hospital.
O governo tentou combater o aumento restabelecendo as restrições levantadas em meados de
junho e trazendo de volta o Green Pass, que mostra se alguém foi totalmente vacinado, se
recuperou recentemente do Covid-19 ou se deu negativo nos 24 anteriores. horas.
Antes da quarta-feira, apenas crianças com 12 anos ou mais, que eram elegíveis para receber a
vacina desde junho, e adultos eram obrigados a apresentar o Green Pass.
Agora também se aplica a crianças entre três e 11 anos de idade. Seus testes serão financiados
pelo governo, pois não são elegíveis para vacinação, a menos que tenham cinco anos ou mais e
sejam considerados em risco significativo de Covid-19.
Os cerca de um milhão de residentes - cerca de 11% da população - que não foram vacinados,
apesar de serem elegíveis, devem pagar pelos seus próprios exames.
Israel também começou a dar terceiras doses da vacina Pfizer-BioNTech para pessoas com mais
de 50 anos, trabalhadores médicos e pessoas com problemas de saúde subjacentes.
Até agora, cerca de 1,1 milhão de pessoas elegíveis receberam suas doses de reforço.
O provedor de saúde israelense Maccabi, que cobre cerca de um quarto da população, informou
na quarta-feira que uma terceira dose da Pfizer foi 86% eficaz na prevenção da infecção por Covid-
19 em pessoas com mais de 60 anos.
O Sr. Zarka também observou que ninguém que estava atualmente em estado crítico no hospital
recebeu uma injeção de reforço.

AULA 08 – ABBREVIATIONS, CONTRACTIONS AND IF CLAUSES 132


TEACHER ANDREA BELO

What It Will Take to Get Life Back to Normal


Vaccines have brought the United States tantalizingly close to crushing the coronavirus within its
borders. After months of hiccups, some 1.4 million people are now being vaccinated every day,
and many more shots are coming through the pipeline. The Food and Drug Administration has
just authorized a third vaccine — a single-dose shot made by Johnson & Johnson — while Pfizer
and Moderna are promising to greatly expand the supply of their shots, to roughly 100 million
total doses per month, by early spring.
If those vaccines make their way into arms quickly, the nation could be on its way to a relatively
pleasant summer and something approaching normal by autumn. Imagine schools running at full
capacity in September and families gathering for Thanksgiving.
But turning that “if” into a “when” will require clearing additional hurdles so that everyone who
needs to be vaccinated gets vaccinated. This is especially true for racial minorities, who are being
disproportionately missed by the vaccination effort.
There’s plenty of disagreement among experts as to why America is still having problems with
vaccine uptake. Some officials have suggested that the main cause is that too many people are
hesitant to get the vaccine. Others point the finger at overcautious public health officials who they
say have undersold the promise of the vaccines. Still others point to long lines at clinics as proof
that far more people want the vaccine than can actually get it.
There is probably some truth to all of these hypotheses, and the underlying problems are not
new. Vaccine hesitancy had been growing steadily in America long before the current pandemic,
so much so that in 2019 the World Health Organization ranked it as one of the leading global
health threats. At the same time, poor health care access and other logistical constraints, such as
a lack of public transportation and limited internet access, have long impeded public health efforts
in low-income communities.
To maximize the number of Americans getting vaccinations, policymakers need to tackle each of
these crises with greater urgency than they have so far.
As supply increases, health officials should mount ambitious vaccination campaigns modeled on
ones that have worked to curb diseases in other countries. That will mean not relying solely on
web portals for scheduling vaccine appointments. It will mean going block by block and door to
door, through high-risk communities especially. It will mean setting up employee vaccination sites
at schools, grocery stores, transit hubs and meatpacking plants, and community clinics at houses
of worship, with local leaders promoting and running them.
“The easier you can make it for people to get vaccinated, the more likely your program will be to
succeed,” said Dr. Walter Orenstein, a former director of the national immunization program at
the Centers for Disease Control and Prevention. “It’s really that simple.”
Outreach efforts cost money. But they’re far less expensive than allowing the pandemic to fester.
Congress has appropriated some money to help states with vaccine rollout. It should offer more,
and states should put as much of those resources as possible toward vaccination efforts that meet
people where they are.
Health officials should also recognize that vaccine hesitancy has many root causes — deliberate
disinformation campaigns, mistrust of medical authorities in marginalized communities, ill-

AULA 08 – ABBREVIATIONS, CONTRACTIONS AND IF CLAUSES 133


TEACHER ANDREA BELO

considered messaging by health officials. The best way to counter that is with campaigns that are
locally led, that clearly outline the benefits of vaccination and that frame getting the shot as not
just a personal choice but a collective responsibility.
Doctors and scientists can help those pro-vaccine messages stick by minding their own public
communications. It’s crucial to be transparent about what vaccines will and won’t do for society
— overselling now will only sow more mistrust later.
That said, underselling is its own problem. It’s true that these vaccines will not immediately
restore the world to total normalcy. But they will eventually allow people to hug their loved ones,
to return to their offices — and to be protected from dying from or becoming seriously ill with
Covid-19. Health officials should be clear about that.
Policymakers at the highest levels of government should press social media companies and e-
commerce sites to curb the most aggressive purveyors of vaccine disinformation.
To not only quell this pandemic but to try to prevent the next one, America will need to improve
its health system and its public health apparatus, both of which have significant holes. “The
problem with a lot of the response is that it was predicated on the idea that we have a good
system in place for doing adult immunizations across the country,” said Dr. Peter Hotez, a vaccine
expert at Baylor College of Medicine. “The fact is, we really don’t.”
In the end, lawmakers and the people who vote them into office will have to address the much
broader problems that this pandemic has exposed.
(Adapted from https://www.nytimes.com/2021/02/26/opinion/vaccine-covid-coronavirus.html?action=click&module=Opinion&pgtype=Homepage)

O que é necessário para fazer a vida voltar ao normal


As vacinas trouxeram os Estados Unidos tentadoramente perto de esmagar o coronavírus dentro
de suas fronteiras. Após meses de soluços, cerca de 1,4 milhão de pessoas estão sendo vacinadas
todos os dias e muitas outras vacinas estão sendo tomadas. A Food and Drug Administration acaba
de autorizar uma terceira vacina - uma dose única feita pela Johnson & Johnson - enquanto a
Pfizer e a Moderna prometem expandir muito o fornecimento de suas vacinas, para cerca de 100
milhões de doses totais por mês, no início da primavera .
Se essas vacinas chegarem às armas rapidamente, a nação poderá estar a caminho de um verão
relativamente agradável e algo quase normal no outono. Imagine escolas funcionando com
capacidade total em setembro e famílias se reunindo para o Dia de Ação de Graças.
Mas transformar esse “se” em um “quando” exigirá a eliminação de obstáculos adicionais para
que todos que precisam ser vacinados sejam vacinados. Isso é especialmente verdadeiro para as
minorias raciais, que estão sendo desproporcionalmente perdidas pelo esforço de vacinação.
Há muita discordância entre os especialistas sobre por que os Estados Unidos ainda estão tendo
problemas com a absorção da vacina. Alguns funcionários sugeriram que a principal causa é que
muitas pessoas hesitam em tomar a vacina. Outros apontam o dedo para as autoridades de saúde
pública excessivamente cautelosas, que dizem ter menosprezado a promessa das vacinas. Outros
ainda apontam as longas filas nas clínicas como prova de que muito mais pessoas querem a vacina
do que podem realmente obtê-la.

AULA 08 – ABBREVIATIONS, CONTRACTIONS AND IF CLAUSES 134


TEACHER ANDREA BELO

Provavelmente há alguma verdade em todas essas hipóteses, e os problemas subjacentes não são
novos. A hesitação vacinal vinha crescendo continuamente nos Estados Unidos muito antes da
atual pandemia, tanto que em 2019 a Organização Mundial da Saúde a classificou como uma das
principais ameaças à saúde global. Ao mesmo tempo, o acesso precário aos cuidados de saúde e
outras restrições logísticas, como a falta de transporte público e o acesso limitado à Internet, há
muito impedem os esforços de saúde pública em comunidades de baixa renda.
Para maximizar o número de americanos vacinados, os formuladores de políticas precisam
enfrentar cada uma dessas crises com maior urgência do que até agora.
À medida que a oferta aumenta, as autoridades de saúde devem montar campanhas de vacinação
ambiciosas, baseadas nas que têm trabalhado para conter doenças em outros países. Isso
significará não depender apenas de portais da web para o agendamento de vacinas. Isso
significará ir quarteirão por quarteirão e de porta em porta, especialmente em comunidades de
alto risco. Isso significará estabelecer locais de vacinação para funcionários em escolas,
supermercados, centros de trânsito e frigoríficos, e clínicas comunitárias em templos religiosos,
com líderes locais promovendo e administrando-os.
“Quanto mais fácil for para que as pessoas sejam vacinadas, maior será a probabilidade de sucesso
do seu programa”, disse o Dr. Walter Orenstein, ex-diretor do programa nacional de imunização
dos Centros de Controle e Prevenção de Doenças. “É realmente tão simples.”
Os esforços de divulgação custam dinheiro. Mas eles são muito mais baratos do que permitir que
a pandemia se espalhe. O Congresso destinou algum dinheiro para ajudar os estados com a
implantação de vacinas. Deve oferecer mais, e os estados devem destinar o máximo possível
desses recursos para esforços de vacinação que atendam as pessoas onde elas estão.
As autoridades de saúde também devem reconhecer que a hesitação da vacina tem muitas causas
profundas - campanhas deliberadas de desinformação, desconfiança nas autoridades médicas em
comunidades marginalizadas, mensagens mal-consideradas por autoridades de saúde. A melhor
maneira de contra-atacar isso é com campanhas conduzidas localmente, que delineiam
claramente os benefícios da vacinação e que o enquadramento seja uma escolha não apenas
pessoal, mas também uma responsabilidade coletiva.
Médicos e cientistas podem ajudar a manter essas mensagens pró-vacina cuidando de suas
próprias comunicações públicas. É crucial ser transparente sobre o que as vacinas farão ou não
pela sociedade - exagerar agora só vai semear mais desconfiança mais tarde.
Dito isso, a venda a preços inferiores é um problema seu. É verdade que essas vacinas não
restaurarão imediatamente o mundo à total normalidade. Mas eles vão permitir que as pessoas
abracem seus entes queridos, voltem para seus escritórios - e sejam protegidas de morrer ou ficar
gravemente doente com Covid-19. As autoridades de saúde devem ser claras sobre isso.
Os formuladores de políticas nos mais altos escalões do governo devem pressionar as empresas
de mídia social e sites de comércio eletrônico para conter os fornecedores mais agressivos de
desinformação de vacinas.
Para não apenas conter essa pandemia, mas também tentar prevenir a próxima, os Estados
Unidos precisarão melhorar seu sistema de saúde e seu aparato de saúde pública, ambos com
lacunas significativas. “O problema com grande parte da resposta é que foi baseada na ideia de
que temos um bom sistema para fazer imunizações de adultos em todo o país”, disse o Dr. Peter

AULA 08 – ABBREVIATIONS, CONTRACTIONS AND IF CLAUSES 135


TEACHER ANDREA BELO

Hotez, especialista em vacinas do Baylor College of Medicine. "O fato é que realmente não
queremos."
No final, os legisladores e as pessoas que votam nele terão de enfrentar os problemas muito mais
amplos que essa pandemia expôs.

Brazil, officially the Federative Republic of Brazil, is the largest country in both South America and
Latin America. At 8.5 million square kilometers (3.2 million square miles) and with over 211 million
people, Brazil is the world's fifth-largest country by area and the sixth most populous. Its capital
is Brasília, and its most populous city is São Paulo. _______ federation is composed of the union
of the 26 states and the Federal District. It is the largest country to have Portuguese as an official
language and the only one in the Americas; it is also one of the most multicultural and ethnically
diverse nations, due to over _______ century of mass immigration from around the world.
Bounded by the Atlantic Ocean on the east, Brazil has a coastline of 7,491 kilometers (4,655 mi).
It borders all other countries in South America except Ecuador and Chile and covers 47.3% of the
continent's land area. Its Amazon River basin includes _______ enormous tropical forest, home
to diverse wildlife, a variety of ecological systems, and extensive natural resources spanning
numerous protected habitats.
Adapted from https://en.wikipedia.org/wiki/Brazil.

O Brasil, oficialmente República Federativa do Brasil, é o maior país da América do Sul e da


América Latina. Com 8,5 milhões de quilômetros quadrados (3,2 milhões de milhas quadradas) e
mais de 211 milhões de habitantes, o Brasil é o quinto maior país do mundo em área e o sexto
mais populoso. Sua capital é Brasília e sua cidade mais populosa é São Paulo. A federação _______
é composta pela união dos 26 estados e do Distrito Federal. É o maior país a ter o português como
língua oficial e a única das Américas; é também uma das nações mais multiculturais e etnicamente
diversas, devido a mais de _______ século de imigração em massa de todo o mundo.
Delimitado pelo Oceano Atlântico a leste, o Brasil tem uma costa de 7.491 quilômetros (4.655
milhas). Faz fronteira com todos os outros países da América do Sul, exceto Equador e Chile, e
cobre 47,3% da área terrestre do continente. A bacia do rio Amazonas inclui _______ uma enorme
floresta tropical, lar de uma vida selvagem diversa, uma variedade de sistemas ecológicos e
extensos recursos naturais abrangendo vários habitats protegidos.

Can covid make your ears ring? What we know about its possible connection to tinnitus.
The recent death ____ Texas Roadhouse CEO Kent Taylor is bringing more attention to what
experts say is another troublesome physical ailment that may be associated with the coronavirus
pandemic: tinnitus, or the perception of a loud ringing or buzzing sound in the ears.

AULA 08 – ABBREVIATIONS, CONTRACTIONS AND IF CLAUSES 136


TEACHER ANDREA BELO

Taylor, 65, died ____ suicide last week, and his family told news outlets, including The Washington
Post, that he had been battling “post-Covid related symptoms, including severe tinnitus” at the
time of his death.
[Texas Roadhouse CEO dies by suicide while battling ‘unbearable’ post-covid-19 symptoms, family
says] Though early research and anecdotal reports have documented tinnitus and sudden hearing
loss in some covid-19 patients, audiologists emphasized that there is no conclusive evidence
connecting the virus to the onset or worsening of tinnitus.
The condition is “one of those things that is just so variable in every single person,” said Eldré
Beukes, ____ audiologist and research fellow at Anglia Ruskin University in ____ United Kingdom,
who has published research on how individuals with tinnitus have been affected by the pandemic.
In the United States, more than 50 million people experience some form of tinnitus, which is often
linked to hearing loss. About 20 million struggle with a chronic condition, while 2 million have
extreme and debilitating cases, according to the American Tinnitus Association.
(Adapted from https://www.washingtonpost.com/lifestyle/wellness/tinnitus-covid-treatment-ear-hearing/2021/03/23/aca1811c-8b32-11eb-9423-04079921c915_story.html)

Covid pode fazer seus ouvidos zumbirem? O que sabemos sobre sua possível conexão com o zumbido.
A morte recente ____ O CEO da Texas Roadhouse, Kent Taylor, está chamando mais atenção para
o que os especialistas dizem ser outra doença física problemática que pode estar associada à
pandemia de coronavírus: zumbido ou a percepção de um zumbido alto ou zumbido nos ouvidos.
Taylor, 65, morreu ____ suicídio na semana passada, e sua família disse a meios de comunicação,
incluindo o The Washington Post, que ele estava lutando contra “sintomas relacionados pós-Covid,
incluindo zumbido severo” no momento de sua morte.
[CEO do Texas Roadhouse morre por suicídio enquanto lutava contra os sintomas "insuportáveis"
pós-covid-19, a família diz] Embora pesquisas iniciais e relatos anedóticos tenham documentado
zumbido e perda súbita de audição em alguns pacientes covid-19, os audiologistas enfatizaram
que não há evidências conclusivas conectar o vírus ao início ou agravamento do zumbido.
A condição é "uma daquelas coisas que variam muito em cada pessoa", disse Eldré Beukes, ____
audiologista e pesquisador da Anglia Ruskin University no ____ Reino Unido, que publicou
pesquisas sobre como indivíduos com zumbido foram afetados por a pandemia. Nos Estados
Unidos, mais de 50 milhões de pessoas apresentam algum tipo de zumbido, que geralmente está
relacionado à perda auditiva. Cerca de 20 milhões lutam com uma condição crônica, enquanto 2
milhões têm casos extremos e debilitantes, de acordo com a American Tinnitus Association.

Don’t fear the AstraZeneca jab, the risks are minimal


Statistics about the Oxford/AstraZeneca vaccine have dominated the news, with concern over
blood clots leading many European countries to suspend its use.
Then, on Thursday, the European Medicines Agency (EMA) and the UK Medicines and Healthcare
products Regulatory Agency (MRHA) declared there was no general increased risk of

AULA 08 – ABBREVIATIONS, CONTRACTIONS AND IF CLAUSES 137


TEACHER ANDREA BELO

thromboses, but they were continuing to look at a rare type of clotting linked with low platelets,
particularly in the brain.
From nearly 12m Oxford/AZ vaccinations up to 7 March, the UK “yellow card” system has
recorded 61,000 reports detailing 228,000 reactions, around double the rate for the
Pfizer/BioNTech vaccine. There have been huge numbers of immediate side-effects such as pain,
nausea, fatigue, headache, and fever, with other notable reports including palpitations (1,318),
“feeling jittery” (10), “screaming” (4), chilblains (10), alcohol poisoning (2), libido increased (1),
libido decreased (1), and one remarkable report of a pregnancy following vaccination. But some
have been serious events, including 289 deaths soon after the jab, and all of these will have
been examined carefully.
Every decision has trade-offs. Both UK vaccines have had more than 200 severe allergic reactions
(anaphylaxis), which is why we have to sit for 15 minutes after the jab. So it’s reasonable that,
rather than saying the vaccine is “safe”, the EMA and MRHA emphasise that the benefits of the
vaccine outweigh the risks of side-effects.
In the UK there has been one of these rare blood reactions in 2m jabs. But for every 2 million
people in their 50s getting a jab, the current group being vaccinated, we would currently expect
to prevent around five deaths a week. The ratio of benefit to harms is high, and even higher on
mainland Europe as their third wave starts.
The “cautionary” pause in many countries may mean increased vaccination hesitancy. France has
moved from initially not approving the Oxford/AZ vaccine for over-65s, to pausing it for
everyone, to now not approving it for under-55s. It would not be surprising if people are
confused by this. Confidence can shatter like glass and be hard to remake.
(Adapted from https://www.theguardian.com/theobserver/commentisfree/2021/mar/21/do-not-fear-the-astrazeneca-covid-jab-the-risks-areminimal)

Não tema o jab AstraZeneca, os riscos são mínimos


As estatísticas sobre a vacina Oxford / AstraZeneca dominaram as notícias, com a preocupação
com os coágulos sanguíneos levando muitos países europeus a suspender seu uso.
Então, na quinta-feira, a Agência Europeia de Medicamentos (EMA) e a Agência Reguladora de
Medicamentos e Produtos de Saúde (MRHA) do Reino Unido declararam que não havia risco geral
de aumento de tromboses, mas continuavam observando um tipo raro de coagulação associada a
plaquetas baixas , particularmente no cérebro.
De quase 12 milhões de vacinações Oxford / AZ até 7 de março, o sistema de “cartão amarelo” do
Reino Unido registrou 61.000 relatórios detalhando 228.000 reações, cerca do dobro da taxa para
a vacina Pfizer / BioNTech. Tem havido um grande número de efeitos colaterais imediatos, como
dor, náusea, fadiga, dor de cabeça e febre, com outros relatos notáveis, incluindo palpitações
(1.318), "sensação de nervosismo" (10), "gritos" (4), frieiras ( 10), envenenamento por álcool (2),
aumento da libido (1), diminuição da libido (1) e um notável relato de gravidez após a vacinação.
Mas alguns foram eventos sérios, incluindo 289 mortes logo após o jab, e todos eles foram
examinados com cuidado.
Cada decisão tem vantagens e desvantagens. Ambas as vacinas do Reino Unido tiveram mais de
200 reações alérgicas graves (anafilaxia), razão pela qual temos que sentar por 15 minutos após a

AULA 08 – ABBREVIATIONS, CONTRACTIONS AND IF CLAUSES 138


TEACHER ANDREA BELO

injeção. Portanto, é razoável que, em vez de dizer que a vacina é "segura", a EMA e a MRHA
enfatizem que os benefícios da vacina superam os riscos de efeitos colaterais.
No Reino Unido, ocorreu uma dessas raras reações sanguíneas em injecções de 2 milhões. Mas
para cada 2 milhões de pessoas em seus 50 anos recebendo uma vacina, o grupo atual sendo
vacinado, esperamos evitar cerca de cinco mortes por semana. A proporção de benefícios para
danos é alta, e ainda mais alta na Europa continental, quando a terceira onda começa.
A pausa “preventiva” em muitos países pode significar um aumento da hesitação à vacinação. A
França passou de não aprovar inicialmente a vacina Oxford / AZ para maiores de 65 anos, para
pausá-la para todos, e agora não aprová-la para menores de 55 anos. Não seria surpreendente se
as pessoas ficassem confusas com isso. A confiança pode se estilhaçar como vidro e ser difícil de
refazer.

It is profoundly difficult ____ grapple with risks whose stakes may include the global collapse of
civilisation, or even the extinction ____ humanity. ____ pandemic has shattered our illusions of
safety and reminded us that despite all the progress made in science and technology, we remain
vulnerable ____ catastrophes that can overturn our entire way of life. These are live possibilities,
not mere hypotheses, and our governments will have to confront them.
(Adapted from https://www.theguardian.com/commentisfree/2021/mar/23/covid-19-humanity-resilience-climate-ai-pandemic)

É profundamente difícil ____ lidar com riscos cujas apostas podem incluir o colapso global da
civilização, ou mesmo a extinção ____ da humanidade. A pandemia de ____ destruiu nossas
ilusões de segurança e nos lembrou que, apesar de todo o progresso feito na ciência e tecnologia,
continuamos vulneráveis a ____ catástrofes que podem destruir todo o nosso estilo de vida. São
possibilidades vivas, não meras hipóteses, e nossos governos terão que enfrentá-las.

We can’t get back to normal until everyone else does


In the United States, despite worrying upticks in cases in some states, we’re well on the way to
vaccinating ourselves out of the catastrophic domestic covid-19 crisis. To get fully “back to
normal,” though, we’re also going to have to — gulp — help vaccinate the rest of the world.
The Biden administration has so turbocharged the process of getting vaccines into arms that,
recently, we’ve been administering as many as 3 million shots per day, and sometimes more.
According to President Biden, by the end of May we will have obtained enough vaccine doses “for
every adult in America.”
True, we should expect setbacks. It is maddening to see the images of huge spring-break crowds
in Miami, jammed together with few participants wearing masks properly. Those young people
are only helping the covid-19 virus survive and potentially to mutate. But they are unlikely to
become seriously ill if infected — and if they do take the virus home with them, at least there is a
growing chance their parents and grandparents will already be immunized.

AULA 08 – ABBREVIATIONS, CONTRACTIONS AND IF CLAUSES 139


TEACHER ANDREA BELO

I keep thinking, though, about other crowds that were part of the old normal. I think of the masses
of foreign tourists who gathered in the summer on the National Mall, waiting their turn to enter
the Smithsonian’s National Air and Space Museum.
One of the languages I used to pick out most frequently in tourist crowds was Portuguese, spoken
with various Brazilian accents. Before covid-19, almost 2 million Brazilians visited the United States
each year. But Brazil has seen more deaths from the virus than any country except the United
States; anyone who has been in Brazil over the past 14 days is forbidden to enter the country.
We’ll never really get back to normal as long as international borders are essentially closed. And I
don’t see how those borders can fully open until we can be sure that visitors are not bringing with
them covid-19 — perhaps in the form of variant strains that are more infectious, more deadly or
more resistant to our vaccines.
(Adapted from https://www.washingtonpost.com/opinions/the-only-way-for-the-us-to-truly-get-back-to-normal-is-to-vaccinate-theworld/2021/03/22/de1a8aa8-8b3d-11eb-a730-1b4ed9656258_story.html)

Não podemos voltar ao normal até que todos o façam


Nos Estados Unidos, apesar do preocupante aumento de casos em alguns estados, estamos no
bom caminho para nos vacinarmos da catastrófica crise doméstica covid-19. Para voltar
totalmente "ao normal", porém, também vamos ter que - engolir em seco - ajudar a vacinar o
resto do mundo.
A administração Biden turbinou tanto o processo de colocar as vacinas nas armas que,
recentemente, temos administrado até 3 milhões de injeções por dia, e às vezes mais. De acordo
com o presidente Biden, até o final de maio teremos obtido doses de vacina suficientes “para cada
adulto na América”.
É verdade que devemos esperar contratempos. É enlouquecedor ver as imagens de grandes
multidões nas férias de primavera em Miami, amontoadas com poucos participantes usando
máscaras adequadas. Esses jovens estão apenas ajudando o vírus covid-19 a sobreviver e,
potencialmente, sofrer mutação. Mas é improvável que fiquem gravemente doentes se infectados
- e se levarem o vírus para casa, pelo menos há uma chance crescente de que seus pais e avós já
estejam imunizados.
Eu fico pensando, porém, em outras multidões que faziam parte do velho normal. Penso nas
massas de turistas estrangeiros que se reuniram no verão no National Mall, esperando a vez de
entrar no Museu Nacional Aéreo e Espacial do Smithsonian.
Uma das línguas que escolhi com mais frequência na multidão de turistas era o português, falado
com vários sotaques brasileiros. Antes de covid-19, quase 2 milhões de brasileiros visitavam os
Estados Unidos a cada ano. Mas o Brasil viu mais mortes pelo vírus do que qualquer outro país,
exceto os Estados Unidos; quem está no Brasil há mais de 14 dias está proibido de entrar no país.
Nunca voltaremos ao normal enquanto as fronteiras internacionais estiverem essencialmente
fechadas. E não vejo como essas fronteiras podem se abrir totalmente até que possamos ter
certeza de que os visitantes não estão trazendo com eles covid-19 - talvez na forma de cepas
variantes que são mais infecciosas, mais mortais ou mais resistentes às nossas vacinas.

AULA 08 – ABBREVIATIONS, CONTRACTIONS AND IF CLAUSES 140


TEACHER ANDREA BELO

THE HISTORY OF PIZZA


The modern pizza was originally invented in Naples, Italy but the word pizza is Greek in origin,
derived from the Greek word pēktos meaning solid or clotted. The ancient Greeks covered their
bread with oils, herbs and cheese. The first major innovation that led to flat bread pizza was the
use of tomato as a topping. It was common for the poor of the area around Naples to add tomato
to their yeast-based flat bread, and so the pizza began.
While it is difficult to say for sure ____ invented the pizza, it is however believed that modern pizza
was first made by baker Raffaele Esposito of Naples. In fact, a popular urban legend holds that the
archetypal pizza, Pizza Margherita, was invented in 1889, when the Royal Palace of Capodimonte
commissioned the Neapolitan pizzaiolo Raffaele Esposito to create a pizza in honor of the visiting
Queen Margherita. Of the three different pizzas he created, the Queen strongly preferred a pie
swathed in the colors of the Italian flag: red (tomato), green (basil), and white (mozzarella).
Supposedly, this kind of pizza was then named after the Queen as Pizza Margherita.
Later, the dish has become popular in many parts of the world:
• The first pizzeria, Antica Pizzeria Port'Alba, was opened in 1830 in Naples.
• In North America, The first pizzeria was opened in 1905 by Gennaro Lombardi at 53 1/3 Spring
Street in New York City.
• The first Pizza Hut, the chain of pizza restaurants appeared in the United States during the 1930s.
• Nowadays, many varieties of pizza exist worldwide, along with several dish variants based upon
pizza.

A HISTÓRIA DA PIZZA
A pizza moderna foi originalmente inventada em Nápoles, Itália, mas a palavra pizza é de origem
grega, derivada da palavra grega pēktos que significa sólido ou coagulado. Os antigos gregos
cobriam o pão com óleos, ervas e queijo. A primeira grande inovação que levou à pizza de pão
achatado foi o uso de tomate como cobertura. Era comum que os pobres da área ao redor de
Nápoles acrescentassem tomate ao pão achatado à base de fermento, e assim a pizza começou.
Embora seja difícil dizer com certeza que ____ inventou a pizza, porém, acredita-se que a pizza
moderna foi feita pela primeira vez pelo padeiro Raffaele Esposito, de Nápoles. Na verdade, uma
lenda urbana popular afirma que a pizza arquetípica, Pizza Margherita, foi inventada em 1889,
quando o Palácio Real de Capodimonte encomendou ao pizzaiolo napolitano Raffaele Esposito a
criação de uma pizza em homenagem à visitante Rainha Margherita. Das três diferentes pizzas que
criou, a Rainha preferiu fortemente uma torta envolta nas cores da bandeira italiana: vermelha
(tomate), verde (manjericão) e branca (mussarela). Supostamente, esse tipo de pizza recebeu o
nome da Rainha de Pizza Margherita.
Mais tarde, o prato se tornou popular em muitas partes do mundo:
• A primeira pizzaria, Antica Pizzeria Port'Alba, foi inaugurada em 1830 em Nápoles.
• Na América do Norte, a primeira pizzaria foi inaugurada em 1905 por Gennaro Lombardi na 53
1/3 Spring Street na cidade de Nova York.
• A primeira Pizza Hut, rede de pizzarias surgiu nos Estados Unidos durante a década de 1930.
• Hoje em dia, existem muitas variedades de pizza em todo o mundo, juntamente com diversas
variantes de pratos baseados em pizza.

AULA 08 – ABBREVIATIONS, CONTRACTIONS AND IF CLAUSES 141


AFA 2024

RELATIVE CLAUSES AND QUANTIFIERS

AULA 09

Teacher Andrea Belo

www.estrategiamilitares.com.br www.militares.estrategia.com
TEACHER ANDREA BELO

SUMÁRIO
INTRODUÇÃO 3

RELATIVE CLAUSES 4

QUANTIFIERS 12

QUESTÕES 14

GABARITO 46

QUESTÕES COMENTADAS 47

CONSIDERAÇÕES FINAIS 106

REFERÊNCIAS BIBLIOGRÁFICAS 107

TRADUÇÕES 109

AULA 09 – RELATIVE CLAUSES AND QUANTIFIERS 2


TEACHER ANDREA BELO

INTRODUÇÃO
Desta vez, vamos à nossa aula sobre Relative Clauses (Orações Relativas), também
presentes nos textos das provas de vestibular e que, têm suas particularidades, as quais vamos
estudar a teoria e praticar com muitos exercícios no decorrer do material.
O que são Relatives Clauses? Relative Clauses são sentenças ou frases que oferecem uma
informação sobre algo ou alguém.
A palavra “relative” deve ser entendida como “ligado a” ou “relacionado a” e, para facilitar,
devemos lembrar dos Relative Pronouns, que estudamos na aula destinada aos pronomes e suas
classificações.
As orações relativas são construídas similares às adverbiais, que estudamos anteriormente.
Então, possuem uma oração independente e outra dependente.
Assim, nos textos de vestibular, não seria diferente – aparecem, nos textos, as orações dos
mais variados tipos, com a exploração do vocabulário, dos termos de diferentes classificações
gramaticais, do sentido que cada frase tem, inseridas nos vários assuntos trabalhados.
Diante das provas de diferentes bancas de vestibulares, faz-se necessário conhecer muitos
tipos de frases para sentir-se mais seguro na hora de analisar cada texto e responder às questões.
O estudo das orações, como fazemos dia a dia em nosso material, abrange tópicos
gramaticais presentes nas entrelinhas do que se pergunta no vestibular.
E isso vai ampliar os conhecimentos que você já possui, pois a língua escrita possui prestígio
maior do que a língua falada, visto que é considerada um referencial de uso correto porque segue
regras e vai aparecer no vestibular para interpretação e busca de respostas do que é questionado.
Vamos estudar também os Quantifiers – os determinantes, são expressões usadas para
indicar informações a respeito da quantidade de algo. Veremos muitos tipos deles.
Assim, meu objetivo, como sua professora e orientadora, é dar a você condições para que
você aprenda os tópicos do vestibular, assim com as orações relativas e Quantifiers de hoje e que
você reconheça as variedades da língua em suas diversas situações de uso.
Vamos então estudar as particularidades de todas as orações relativas, praticar exercícios,
tanto durante a teoria como também na lista de questões, treinando tudo o que aprendeu, em
junção ao aprendizado de cada aula.
Vamos lá e conte comigo!

AULA 09 – RELATIVE CLAUSES AND QUANTIFIERS 3


TEACHER ANDREA BELO

RELATIVE CLAUSES
As Relative Clauses são orações que exercem a função de adjetivos. Por esse motivo, em
algumas gramáticas, também são chamadas de Adjectives Clauses. E são, em Português, orações
subordinadas.
Elas são empregadas com o intuito de informar sobre uma pessoa ou determinada coisa e
assim, funcionam como complementos de um substantivo ou um pronome.
Essas orações – relative clauses – que vamos estudar cada particularidade, para
compreender bem, são construídas com a adição de partículas que têm o objetivo de adicionar
informações à oração principal. Essas partículas são os pronomes relativos.
Em Inglês, os pronomes que já estudamos em aulas anteriores e que são utilizados nas
orações relativas são: who, whom, which, whose e that.
Outros pronomes relativos, menos usados, mas que estudaremos também são when,
where e why. Eles são responsáveis por unir uma frase à outra bem como fornecer informações
adicionais à frase anterior.
Cada pronome relativo é utilizado para complementar uma oração de forma diferente.
Vejamos, em suma, como é a classificação dessas orações, para depois estudar cada uma delas
separadamente com exemplos.
Os pronomes relativos podem exercer diferentes funções na frase (sujeito, objeto ou
posse) e, determinam qual será a relative clause.
As Relative Clauses podem ser Defining Relative Clauses ou Non-defining Relative Clauses,
veja:
Defining relative clauses (orações restritivas): Essas orações são usadas para definir sobre
quem ou sobre o que estamos falando. Não exigem o uso de vírgulas e os pronomes relativos
utilizados nelas são: who, whom, which, whose, where, when, why e that, como estudaremos
adiante, um a um.
Non-defining relative clauses (orações explicativas): Um pouco diferente das defining
relative clauses, por sua vez, as non-defining relative clauses não fornecem informações essenciais
sobre o que antecede a frase. Aqui, as informações adicionadas geralmente aparecem entre
vírgulas. E os pronomes mais utilizados são: who, whom, whose e which.
As defining clauses, independente do pronome relativo a ser usado, aparecem de forma
mais direta nas frases, ou seja, como eu afirmei anteriormente, sem estar entre parênteses ou
entre vírgulas. Elas destacam pessoas ou coisas à qual se referem.
As non-defining clauses são orações que trazem informações extras sobre o sujeito ou
objeto das frases. Geralmente, são colocadas entre vírgulas ou parênteses.
Se tentarmos retirá-las da oração principal, ela dá a impressão de que está incompleta,
apesar de manter o sentido.
Resumidamente, eis as características principais das Defining e Non-defining clauses:

AULA 09 – RELATIVE CLAUSES AND QUANTIFIERS 4


TEACHER ANDREA BELO

DEFINING RELATIVE CLAUSE NON-DEFINING RELATIVE CLAUSE


O seu antecedente é indefinido. O seu antecedente é definido.
Traz informação essencial para restringir o Traz informação adicional, mas não essencial
significado do seu antecedente. ao seu antecedente.
Nunca é colocada entre vírgulas. É sempre colocada entre vírgulas.
Começa normalmente por um pronome relativo, Começa sempre por um pronome relativo, ex.
ex. who(m), which, whose. who(m), which, whose.
O pronome relativo that pode ser empregue em O pronome relativo that não pode ser
vez de who(m) e which. empregue em caso nenhum.
Como complemento do verbo, o pronome
O pronome relativo não pode ser omitido
relativo pode ser omitido: who(m), which e that.

Veremos cada pronome para saber qual dos dois tipos podem ser usadas com cada um
deles.

RELATIVE PRONOUN THAT


O pronome relativo that é o que mais parece em frases de forma geral, em diálogos
informais, em filmes, em séries, em textos variados, entre outros, já que pode ser usado quando
se trata de pessoas ou de objetos. Atenção: é usado apenas em Defining Relative Clauses.
Vou introduzir oferecendo dois exemplos para você compreender melhor, desde o início
da explicação. É como se fôssemos dizer: “Eu tenho um amigo que eu admiro muito.” ou “Eu
tenho um cachorro que sabe nadar”, em que se fala de coisas/animais referentes à pessoa ou se
fala da própria pessoa, usando o mesmo pronome: that.
As frases acima seriam: “I have a friend that I admire a lot.” E “I have a dog that knows
how to swim”.
No caso de THAT, foi escolhido para ser o primeiro da lista de todos que explicarei nessa
aula, justamente porque serve para todas as orações relativas.
Eu costumo chamar esse pronome de pronome carta coringa, que pode ser utilizado em
qualquer oração, quando se há alguma dúvida em relação a qual usar para desenvolver as frases
desejadas.
É importante observar que, se vier um sujeito após o pronome that, ele pode ser omitido
sem prejudicar o sentido da frase. E às vezes pergunta-se isso em questões de vestibular.
Vejamos outros exemplos:

These are the flowers that I bought for you.


(Esses são as flores que eu comprei para você.)

Aqui, por causa do sujeito “I”, após o pronome that, ele poderia ser omitido:
These are the flowers I bought for you.

AULA 09 – RELATIVE CLAUSES AND QUANTIFIERS 5


TEACHER ANDREA BELO

These are the boys that I was talking about.


(Esses são os garotos que eu estava falando.)

Aqui também, por causa do sujeito “I”, após o pronome that, ele poderia ser omitido:
These are the boys I was talking about.

He spoke so well that everybody was surprised.


(Ele falou tão bem que todos ficaram surpresos.)

Aqui também, por causa do sujeito “everybody”, após that, ele poderia ser omitido:
He spoke so well everybody was surprised.

Agora vejamos exemplos em que o pronome that não pode ser omitido, já que, não há
sujeitos após os mesmos e sim verbos, que “pedem” que o pronome esteja presente:

The cat that was hidden is mine.


(O gato que estava escondido é meu).

Aqui, por causa do verbo “was hidden” e não um sujeito, após


that, ele não poderia ser omitido: The cat that was hidden is mine.

The girl that won the competition is the best.


(A garota que ganhou a competição é a melhor.)

Aqui, também por causa do verbo “won” (Past Simple) e não um sujeito,
após that, ele não poderia ser omitido: The girl that won the game is the
best.

Vejamos o pronome that em um dos textos da Newsweek, fonte usada para elaborar as
provas, para exemplificar:
Christoph Oswald has no problem approaching
women. As he makes his way through the crowd at his favorite
Frankfurt club, his cell phone scans a 10-meter radius for "his
type": tall, slim, sporty, in her 30s--and, most important,
looking for him, a handsome 36-year-old software consultant
who loves ski holidays. Before he reaches the bar, his phone
starts vibrating and an attractive blonde appears on its screen.
"Hi, I'm Susan," she says. "Come find me!" Christoph picks her
out of the crowd, and soon they're laughing over a drink.

AULA 09 – RELATIVE CLAUSES AND QUANTIFIERS 6


TEACHER ANDREA BELO

Both Christoph and Susan have phones equipped with Symbian Dater, a program that
promises to turn the cell phone into a matchmaker. By downloading Symbian, they installed a 20-
character encrypted code that includes details of who they are and what they're looking for in a
mate. Whenever they go out, their matchmaking phones sniff out other Symbian Daters over the
unlicensed, and therefore free, Bluetooth radio frequency. If profiles match up, the phones beep
wildly and send out short video messages.
(NEWSWEEK, JUNE 7 / JUNE 14, 2004)

O texto está falando de Christoph Oswald e suas aventuras com as mulheres e, no segundo
parágrafo, foi usado o pronome that, se referindo a um programa de celular que promete algo:
“... Symbian Dater, a program that promises to turn the cell phone into …”.
Assim como estudamos, o that pode se referir a pessoas ou coisas e oferecer informações
sobre elas. No caso, foi informado que é um programa que promete algo.
Vamos ao estudo do pronome which.

RELATIVE PRONOUN WHICH


O pronome relativo which é usado quando se trata de coisas/objetos e animais e nunca
usado para pessoas. Pode ser também traduzido como que.
Diferente de that, é usado em defining ou non-defining relative clauses.
No caso de WHICH, é válido observar que, muitas vezes, pergunta-se em questões de
vestibular se which pode ser substituído por that ou vice-versa e, devemos analisar
cuidadosamente se tais pronomes, nessas frases, referem-se ao sujeito ou ao objeto.
Como uma observação, lembre-se de que, se vier um sujeito após o pronome which, assim
como acontece com o pronome that, ele também pode ser omitido sem prejudicar o sentido da
frase, que pode ser uma provável pergunta do vestibular. Vejamos exemplos:

Orca whale, which is big, is very dangerous.


(A baleia orca, que é grande, é muito perigosa.)

Aqui, por causa do verbo “is”, após o pronome which, ele não pode ser omitido.

This is the document which you need to sign.


(Este é o documento que você precisa assinar.)

Aqui também, por causa do sujeito “you”, após o pronome which, ele pode ser omitido:
This is the document you need to sign.

AULA 09 – RELATIVE CLAUSES AND QUANTIFIERS 7


TEACHER ANDREA BELO

RELATIVE PRONOUNS WHO AND WHOM


Os pronomes relativos who e whom se referem apenas a pessoas e nunca objetos ou
animais. A diferença entre who e whom é bastante sutil e deve-se prestar atenção à explicação e
aos exemplos para não gerar confusão na hora de sua prova de vestibular.
Observe: enquanto who se refere à uma pessoa que executa a ação da oração, whom é,
por sua vez, utilizado quando se trata de alguém que recebe uma ação. Exemplos:

The charming guy who works with me has asked me out.


(O homem charmoso que trabalha comigo me chamou para sair.)

Aqui, você pode perceber que o pronome who faz referência ao homem charmoso,
ou seja, the charming guy, que foi quem chamou a pessoa para sair, executou a ação.

George Lee, who was the school’s president, discoursed yesterday.


(George Lee, que era o presidente da escola, discursou ontem.)

No exemplo acima, você pode perceber mais uma vez que o pronome who faz referência a
George Lee, ou seja, o presidente da escola, que foi quem discursou, executou também a ação.

Você percebeu que, usei exemplos com frases entre vírgulas ou não. Ambos os casos se
usam o pronome relativo who.
Agora, vejamos exemplos com o pronome relativo whom, para que a diferença entre o uso
de who e whom fique clara a você.

I don’t know the name of the boy whom she is going out with.
(Eu não sei o nome do garoto quem/com quem ela está saindo).

Aqui, você pode perceber que o pronome whom faz referência a quem recebeu a
ação: ela. O sujeito da oração é “I” e o garoto de que trata a frase é aquele com quem
ela está saindo, como um adjetivo para a pessoa com quem alguém está saindo.

The boy to whom you gave the present is my brother.


(O garoto para quem você deu o presente é meu irmão).

Aqui também, você pode perceber que o pronome


whom faz referência a quem recebeu a ação: meu
irmão.

AULA 09 – RELATIVE CLAUSES AND QUANTIFIERS 8


TEACHER ANDREA BELO

Vejamos o pronome who em um dos textos do New York Times, fonte muito usada para
preparar provas de vestibular, para exemplificar:
LOS ANGELES − Come summer 2006, Warner Brothers Pictures hopes to usher “Superman”
into thousands of theaters after a 19-year absence. But given the tortured history surrounding
that studio’s attempts to revive “Superman,” the forerunner of Hollywood’s now-ubiquitous
comic-book blockbusters, the Man of Steel’s arrival would be nothing short of a miracle.
Since Warner began developing a remake of the successful comic-book franchise in 1993,
it has spent nearly $10 million in development, employed no fewer than 10 writers, hired four
directors and met with scores of Clark Kent hopefuls without settling on one. The latest director
⎯ Bryan Singer, who directed “X-Men” and its sequel, was named on July 18 to replace Joseph
McGinty Nichol, known as McG, who left the project after refusing to board a plane to Australia,
where the studio was determined to make the film.
(THE NEW YORK TIMES, JULY 22, 2004)

O texto está falando da Warner Bros Pictures, de cinema, de filmes e, no segundo


parágrafo, foi usado o pronome who, duas vezes. Conforme estudamos, o pronome who, sempre
se refere a pessoas, certo?
Na primeira vez que who aparece, ele está se referindo a Bryan Singer: “... The last director
– Bryan Singer, who directed X-Men and its sequel, was named…”, corretamente como
estudamos.
Na segunda vez que who está na frase, está também se referindo a pessoas – desta vez, a
Joseph McGinty Nichol: “... replace Joseph McGinty Nichol, known as McG, who left the project
after refusing to board…”, explicando algo sobre Joseph, como estudamos.
Agora, vamos ao pronome whose. Come on!

RELATIVE PRONOUN WHOSE


O pronome relativo whose é usado para indicar posse, para indicar que algo pertence a
alguém. A tradução que melhor representa o whose seria o cujo/cuja em Português. Vejamos
exemplos:

That bag, whose owner in unknown, is in the lost and found department.
(Aquela mala, cujo dono é desconhecido, está no departamento de achados e perdidos.)

That teacher, whose class is attractive, is preparing a speech for tomorrow.


(Aquela professora, cuja aula é atrativa, está preparando um discurso para amanhã.)

Vamos ao pronome relativo where.

AULA 09 – RELATIVE CLAUSES AND QUANTIFIERS 9


TEACHER ANDREA BELO

RELATIVE PRONOUN WHERE


O pronome relativo where é usado para indicar o lugar que a frase está fazendo referência,
o local do que se trata a informação principal da oração com esse pronome relativo. É um dos
pronomes relativos mais simples, pois tem o mesmo significado de quando é pronome
interrogativo: onde e se encaixa de forma prática nas frases. Vejamos exemplos:

The house where I live is big, beautiful and comfortable.


(A casa onde/em que eu moro é grande, bonita e confortável.)

The college where she studies is modern.


(A faculdade onde/em que ela estuda é moderna.)

Vamos ao pronome relativo when.

RELATIVE PRONOUN WHEN


O pronome relativo when é usado para indicar tempo: o dia, a semana, o mês, enfim, o
período do que se trata a informação principal da oração com esse pronome relativo.
É um pronome relativo simples também, pois, assim como o where, tem o mesmo
significado como pronome interrogativo: quando. Vejamos exemplos:

That was the day when I started working in this company.


(Aquele foi o dia quando/em que eu comecei a trabalhar nessa empresa.)

January is the month when I graduate.


(Janeiro é o mês quando/em que eu me formo.)

Vejamos um texto, que usaremos em nossa lista de exercícios adiante, com o pronome
when para ilustrar e exemplificar:
The role of women in Spanish society has changed fast since the country became a
democracy after General Franco died in 1975. He had swept away liberal reforms introduced in
the 1930s, when Spain was a republic. For women specifically, these included a benevolent
divorce law and certain property rights. In the 1930s many women played a big part on the left,
often fighting side by side with men in the pro-Republic militias during the 1936-39 civil war. But
after it the new regime, for the most part applauded by the church, put them back in the home
as wives and mothers, with divorce forbidden and working outside frowned on.

AULA 09 – RELATIVE CLAUSES AND QUANTIFIERS 10


TEACHER ANDREA BELO

Change began in the 1960s when Spain opened up to tourists. Faced with competition from
sexually liberated north Europeans, Spanish women “declared war on them, on men and on their
elders”, in the words of Lucia Graves, author of “A Woman Unknown”, which recounts her life as
an Englishwoman married to a Spaniard at the time. That aggressive self-assertion continues.
Not wholly successfully. At universities, women students now outnumber men. A typical
couple has one or two children these days, a far cry from the days when families of eight or ten
were common. But Spanish women still face the problems of their sisters in northern Europe.
Their progress at work is often blocked, their pay often lower than men’s.
The Economist August 11th 2001

Percebemos que, nas três vezes em que o pronome when aparece no texto, refere-se a um
período de tempo, contextualiza o moment, veja: “... when Spain was a republic” (quando a
Espanha era república), “... when Spain opened up to tourists” (quando a Espanha abriu-se aos
turistas) e “... when families of eight or tem were common” (quando famílias com oito ou dez
filhos era comum). Todos os períodos do que se trata o assunto do texto.
Vamos ao último pronome relativo: why.

RELATIVE PRONOUN WHY


O pronome relativo why é usado para indicar a razão, o motivo do que se trata a
informação principal da oração.
É um pronome relativo que também significa porque, como na hora de fazer perguntas,
em que está na função de pronome interrogativo. Podemos também traduzir como “por causa
de”. Vejamos exemplos:

There are three different ways to go. That’s the reason why I got confused.
(Há três diferentes caminhos para ir lá. Essa é a razão por quê/pela qual eu fiquei confuso.)

I don’t know the reason why she is angry.


(Eu não sei a razão por quê/pela qual ela está nervosa.)

Vamos ao estudo dos quantifiers. Let’s go!

AULA 09 – RELATIVE CLAUSES AND QUANTIFIERS 11


TEACHER ANDREA BELO

QUANTIFIERS
Os determinantes – quantifiers, são expressões usadas para indicar informações a respeito
da quantidade de alguma coisa. Para estudar e entender bem os tipos de quantifiers, é importante
lembrar de quando estudamos os substantivos.
Você se lembra que havia substantivos classificados com contáveis – countable – aqueles
que podemos contar, não necessitando de nenhuma unidade de medida, permitindo a forma
singular e plural. E os substantivos incontáveis – uncountable – que não podemos contar,
tornando-se necessária uma unidade de medida. Esses substantivos só apresentam a forma
singular. Agora vejamos os quantifiers que mais aparecem no vestibular:

MUCH, MANY E A LOT OF


As três palavras indicam a mesma coisa: uma grande quantidade. No entanto, o uso de
cada um depende exatamente do tipo de substantivo que há na frase:

MUCH é usado nos casos de substantivos não-contáveis.


Example: How much sugar do you need? A few.
(Quanto de açúcar você precisa? Um pouco.)

MANY é usado nos casos de substantives contáveis.


Example: How many students are there? Six.
(Quantos estudantes estão lá? Seis.)

A LOT OF é usado em ambos os casos.


Example: I have a lot of things to do.
Eu tenho muitas coisas para fazer.

TOO E SO
Esses dois pronomes são usados para intensificar algo. A diferença é que “too” dá a
impressão de exagero, às vezes com uma ideia negativa, enquanto “so” indica uma grande
quantidade, sempre com ideia positiva. Examples:

There is too much traffic jam in SP. (Há congestionamento demais em SP.)
I love you so much. (Eu te amo muito.)

AULA 09 – RELATIVE CLAUSES AND QUANTIFIERS 12


TEACHER ANDREA BELO

LITTLE, FEW E ENOUGH


Os quantifiers “little” e “few” significam ambos uma pequena quantidade de alguma coisa.
A diferença entre os dois é que enquanto “little” se refere aos substantivos não-contáveis, “few”
se refere aos contáveis. Examples:

There are a few people in the classroom. (Há poucas pessoas na sala de aula.)
Add a little sugar, please. (Adicione um pouco de açúcar, por favor.)

Por sua vez, “enough” se refere a algo suficiente, podendo ser usado tanto para
substantivos contáveis como incontáveis. Examples:

There isn’t enough food. (Essa comida não é suficiente/não é o bastante.)


I drove fast enough to get there in time.
(Eu dirigi rápido o suficiente/ o bastante para chegar lá na hora.)

Então, “enough” significa suficiente ou bastante, certo? Mas não basta saber a tradução.
preciso saber como usar enough em Inglês de forma correta. Afinal, enough pode ser usado em
várias classes gramaticais em inglês.
Mas, o que importa mesmo é aprender como identificar enough nos textos do vestibular.
Em geral, podemos dizer que o uso mais frequente de enough é como vimos acima, antes ou
depois de substantivos.
Ainda podemos usar enough em algumas expressões fixas, veja:
➢ Enough is enough! I don’t want to argue anymore. (Já chega! Não quero mais discutir.)
➢ I’m just annoyed with her because she’s behaved so badly. Fair enough! (Eu só estou
chateada com ela porque ela se comportou muito mal. Nada mais justo!)
➢ I can’t eat anymore. I’ve had enough. (Não aguento mais comer. Estou satisfeito/cheio.)
➢ I’ve had enough of your excuses (Já estou cheio de suas desculpas/cansado de suas
desculpas.)
➢ The doctors tried everything they could, but it was not enough to save him. (Os médicos
tentaram tudo que podiam, mas não foi suficiente para salvá-lo.)
➢ You’ve been practicing the flute all afternoon. Enough! (Você tocou flauta a tarde inteira.
Basta! Chega!)

Vamos aos exercícios para praticar os pronomes relativos inseridos nos textos de vestibular
de anos anteriores. Let’s go!

AULA 09 – RELATIVE CLAUSES AND QUANTIFIERS 13


TEACHER ANDREA BELO

QUESTÕES
Você vai, agora, responder questões selecionadas de provas já realizadas em anos
anteriores. Depois, como em todas as nossas aulas, haverá o gabarito e as questões comentadas.
Vamos começar com questões AFA, de acordo coma sua instituição escolhida e depois,
vamos treinar de outras Carreiras Militares, para adquirir experiência e treinar vocabulário.
QUESTÕES AFA
Read the text and answer questions 01, 02 and 03.

'America Is On the Move Again.' Joe Biden Delivers His First Major Presidential Speech

President Joe Biden has attended dozens of presidential addresses to Congress since he first came
to Washington in 1973. But after nearly half a century of being an audience member, he finally
got the speaking slot Wednesday night to lay out his own vision for the country.

“America is ___ the move again,” he said during his roughly hour-long remarks. “Turning peril into
possibility. Crisis into opportunity. Setback into strength.”

Biden seized the opportunity to boast about his Administration’s accomplishments, highlighting
how Americans have received over 220 million shots of the COVID-19 vaccines in his first 100 days
in office, which he called “one of the greatest logistical achievements this country has ever seen.”

Like the rest of Biden’s presidential milestones, this one was altered because ___ the pandemic.
The audience in the House chamber, usually more than 1,000 people, including all members of
Congress and multiple Supreme Court justices, was limited to just 200. Attendees were seated
apart from each other to accommodate social distancing, with a maximum of three people in one
row.

His speech largely focused on economic priorities, and he outlined the American Families Plan, his
$1.8 trillion proposal ___ raise taxes on the wealthy to pay for affordable childcare, paid leave,
and access to education. If implemented, the plan would offer universal preschool for three- and
four-year-olds and two years of free community college and create a national paid leave program.
This proposal is the third prong of Biden’s $6 trillion plan to bolster the post-pandemic economy
by combating inequality and climate change. He signed the first part, the $1.9 trillion American
Rescue Plan, into law on March 11. In the past month, has also called for a $2 trillion bill that
would create millions of jobs and repair the country’s infrastructure, hailing it in his address to
Congress as a “blue-collar blueprint to build America.”

In total, these proposals would be the biggest investment in clean energy in U.S. history and the
largest federal investment in the U.S. economy since Lyndon Johnson’s Great Society.

(Adapted from https://time.com/5974129/joe-biden-joint-address-congress/)

AULA 09 – RELATIVE CLAUSES AND QUANTIFIERS 14


TEACHER ANDREA BELO

QUESTÃO 01 (AFA/INÉDITA) – According to the text, which option is correct?


(A) Joe Biden has delivered dozens of presidential speeches since taking office
(B) Biden is taking a very positive and optimistic view of the current situation in the United
States
(C) The President of the United States did not have good logistical planning for vaccination
against COVID-19
(D) Joe Biden's presidential events have not been changed due to the pandemic
(E) Biden plans to tax the entire population equally in order to offer more free services

QUESTÃO 02 (AFA/INÉDITA) – Which is the correct option to complete the gaps in the text?
(A) In / of / to
(B) On / X / to
(C) In / X / in
(D) On / of / in
(E) On / of / to

QUESTÃO 03 (AFA/INÉDITA) – What’s the meaning of the word “bolster” in paragraph 5?


(A) Block
(B) Maintain
(C) Enhance
(D) Hinder
(E) Inspire

Read the text and answer questions 04, 05 and 06.


The lockdown habit that’s hardest to break: using an iPad as a babysitter
How am I supposed to wean my children off screens when they know I’m addicted too?
It started, last March, as an act of necessity – sticking the kids on iPads all day so I could make a
hard deadline that fell six weeks into lockdown. There was no way around this; at five years old,
my kids couldn’t manage their Zoom schedules or self-entertain for long without fighting, and I
couldn’t break off every two minutes to help them.
Overnight, kindergarten and after-school disappeared, to be replaced with the sedative of kids
YouTube, and when the appeal of that started to wane, the more addictive and ruinous content
on TikTok. If it was hideous, I told myself, it was an emergency. It wouldn’t be like this for ever.
A year later and in New York, at least, we’re in a radically different place. Next week my kids’
elementary school goes back full-time and although it lets out early – at 2pm – half of the city is

AULA 09 – RELATIVE CLAUSES AND QUANTIFIERS 15


TEACHER ANDREA BELO

now at least partially vaccinated, so most parents are comfortable hiring a sitter. (If they can find
one, that is; indoor after-school programmes still seem risky to many, so the scramble is on for
that already mythical creature, the 22-year-old sitter only looking for two hours work a day.)
Things are, on the surface, starting to look vaguely like normal.
Except, of course, they’re not. Among the many new habits formed during the pandemic, a
reliance on screens as babysitters may be one of the toughest to break. Over the course of the
year, I have grown accustomed to cleaning the house, finishing work, folding the laundry and even
– if my kids hit a solid addictive high on Minecraft – taking a nap, all without the cost or logistical
planning of babysitting. I can put in a solid few hours at the park, safe in the knowledge that when
we get home help in the form of two hours of back-to-back three-minute videos will keep
everyone happy until dinner.
(Adapted from https://www.theguardian.com/commentisfree/2021/apr/26/lockdown-ipad-babysitter-children-addicted)

QUESTÃO 04 (AFA/INÉDITA) – It is possible to infer from the text that


(A) Although they are beginning to see a light at the end of the tunnel, mothers still see the
internet as the solution to isolated life
(B) The use of the internet as the only entertainment for children in isolated life, started out of
pure convenience
(C) The prospect was that that day-to-day life based on electronics would be the reality forever
(D) New York City is in a place very similar to last year
(E) The progress of vaccination in New York does not allow any hope for improvement

QUESTÃO 05 (AFA/INÉDITA) – In the excerpt “How am I supposed to wean my children off


screens when they know I’m addicted too?” (subtitle), the word in bold means
(A) Encourage
(B) Incite
(C) Help
(D) Detach
(E) Destroy

QUESTÃO 06 (AFA/INÉDITA) – Choose the correct alternative to complete the paragraph below
Dusk is falling in ___ Indian capital, and the acrid smell ___ burning bodies fills the air. It’s the
evening of April 26, and ___ a tiny crematorium in a Delhi suburb, seven funeral pyres are still
burning. “I have lived here all my life and pass through this area twice ___ day,” says local resident
Gaurav Singh. “I have never seen so many bodies burning together.”
(Adapted from https://time.com/5964796/india-covid-19-failure/)

AULA 09 – RELATIVE CLAUSES AND QUANTIFIERS 16


TEACHER ANDREA BELO

(A) The / of / at / a
(B) The / of / in / a
(C) The / with / in / an
(D) A / of / at / a
(E) A / of / in / a

Read the text and answer questions 07, 08, 09 and 10.
Australians stuck overseas 'abandoned' by their own country
Australian citizen Mandeep Sharma feels utterly deserted by his government.
He is one of the 9,000 Australian nationals stranded in India right now, left to fend for themselves
after Canberra this week banned all flights from the pandemic-ravaged nation until mid-May.
He has a wife and two daughters in Adelaide and fears their separation could drag on for months.
Catching Covid in India is also a real risk.
The Australian government's flight ban was the latest hardline action taken to keep the virus out
of the country. Infection rates are near zero, and Australia has had far fewer fatalities than most
countries, thanks to strict border controls and quarantine measures.
Still, those policies have left many Australians locked out. The ban on Indian arrivals marked an
escalation – the first time the country has stopped evacuations and blocked citizens from
returning home altogether. It's intensified calls for more to be done to get Australians home.
Why can't citizens get in?
Prior to the pandemic, there were estimated to be about one million Australians living overseas.
Many have come home in the past year, but many more have struggled to return.
Currently, around 36,000 nationals are registered for government help to fly home, but the true
number of those who wish to come back is estimated to be much higher.
Australia became one of the first nations to close its borders in March 2020. Only returning
nationals and residents were allowed in. Some exemptions were granted, including celebrities,
sports stars and contract workers. Since April, it has also allowed travellers from virus-free New
Zealand.
Those who returned were forced to undertake – and fund – a two-week quarantine stay at hotels
in state capitals. In the early pandemic rush, state governments agreed to take on the burden of
running quarantine instead of the federal government, given the lack of national facilities.
About a month in, a problem emerged. The number of people returning home - mostly from New
Zealand, the US and the UK- threatened to overwhelm the system. There were only so many hotel
rooms available.
But instead of expanding the system - for example, adding a purpose-built quarantine centre –
the government drastically cut back the number of plane arrivals allowed in each week.
(Adapted from https://www.bbc.com/news/world-australia-56924188)

AULA 09 – RELATIVE CLAUSES AND QUANTIFIERS 17


TEACHER ANDREA BELO

QUESTÃO 07 (AFA/INÉDITA) – In the phrase “He is one of the 9,000 Australian nationals
stranded in India right now…” (paragraph 1), the underlined word is a synonym for
(A) Marooned
(B) Imprisoned
(C) Found
(D) Homeless
(E) Stablished

QUESTÃO 08 (AFA/INÉDITA) – third paragraph of the text states that


(A) The ban on flights to Australia was the first effective measure to prevent the spread of the
virus in the country
(B) Banning flights was a government strategy to keep the virus in the country
(C) There are no new cases of coronavirus in Australia
(D) Australia has a terrible mortality rate from coronavirus
(E) Australia's prosperous results are due to the way the country handled the pandemic

QUESTÃO 09 (AFA/INÉDITA) – In the phrase “The number of people returning home - mostly
from New Zealand, the US and the UK- threatened to overwhelm the system” (paragraph 9), the
underlined word is a synonym for
(A) Enlighten
(B) Win
(C) Surrender
(D) Disturb
(E) Defeat

QUESTÃO 10 (AFA/INÉDITA) – The text


(A) Addresses an issue caused by disorganization, or neglect, of the Australian government
(B) Develops the idea that inhabitants should have the right to come and go in their home
country
(C) Aims to give visibility to those who are most affected by the pandemic in Australia
(D) Criticizes all the Australian government's disorganization regarding the pandemic situation
(E) Focuses on the idea that Australia has excellent results from restrictive measures against
COVID-19

AULA 09 – RELATIVE CLAUSES AND QUANTIFIERS 18


TEACHER ANDREA BELO

QUESTÕES COLÉGIO NAVAL


Read Text I to do questions 01 to 04 based on it.
Texas passes law banning abortion after six weeks
A law banning abortion from as early as six weeks into pregnancy has come into effect in the
US state of Texas.
It bans abortions after the detection of what anti-abortion campaigners call a fetal heartbeat,
something medical authorities say is misleading.
The law, one of the most restrictive in the country, took effect after the Supreme Court did not
respond to an emergency appeal by abortion providers.
Doctors and women's rights groups have heavily criticized the law.
It gives any individual the right to sue doctors who perform an abortion past the six-week point.
The so-called "Heartbeat Act" was signed into law by Texas Governor Greg Abbott in May.
But rights groups, including Planned Parenthood and the American Civil Liberties Union (ACLU),
then requested that the Supreme Court block the legislation.
In the early hours of Wednesday, the ACLU confirmed that the court had "not responded to our
request", adding: "Access to almost all abortion has just been cut off for millions of people."
The group, which says that up to 90% of abortions in Texas take place after six weeks of pregnancy,
described the development as "blatantly unconstitutional".

QUESTÃO 01 (COLÉGIO NAVAL/INÉDITA) – Read the extract from the text


“…described the development as ‘blatantly unconstitutional’”.
Mark the alternative that can replace the underlined word without changing its meaning.
(A) Extremely
(B) Painfully
(C) Completely
(D) Specifically
(E) Brazenly

QUESTÃO 02 (COLÉGIO NAVAL/INÉDITA) – About the law that took effect in Texas, it’s correct to
affirm that
(A) It makes abortion more viable for everyone
(B) There are groups that are against the implementation of the law and that were ignored by
the Supreme Court
(C) Prohibits doctors from performing abortions after the six-week period
(D) It is, among the common sense of all, constitutional
(E) Despite being one of the most restrictive, it won't affect many people

AULA 09 – RELATIVE CLAUSES AND QUANTIFIERS 19


TEACHER ANDREA BELO

QUESTÃO 03 (COLÉGIO NAVAL/INÉDITA) – Read the extract from the text


“It gives any individual the right to sue doctors who perform an abortion past the six-week point”
(paragraph 4)
The word “who” refers to
(A) Doctors
(B) Any individual
(C) The law
(D) Abortion
(E) Women’s rights groups

QUESTÃO 04 (COLÉGIO NAVAL/INÉDITA) – Read the extract from the text


“Access to almost all abortion has just been cut off for millions of people” (paragraph 7)
The sentence above means that
(A) Millions of people have been deprived of the right to have nearly all abortions
(B) Abortion has been banned by law for millions of people
(C) Access to almost all abortions has been made easier for millions of people
(D) Millions of people have renounced the right to have almost all abortions
(E) Almost all abortions are available to millions of people

Read the text II to do item 05.


Nuclear energy is key in fight for climate
Ignore the myths about nuclear power, writes Rob Loveday of Generation Atomic – it is an
essential source of clean energy
The letter on nuclear energy (25 August) sadly could not be more wrong – nuclear is one of the
cleanest fuels we have and has always been so. The carbon and material footprints of nuclear –
for its entire lifetime, including mining and decommissioning – are lower than solar and on a par
with wind, according to the Intergovernmental Panel on Climate Change.
Moreover, the nuclear industry manages its waste stream – that is more than can be said for the
solar industry, which is set to produce millions of tons of toxic waste. Besides, spent nuclear fuel
is not “waste” but a valuable source of low-carbon energy that can be recycled via reprocessing
or proposed breeder reactors, thereby neutralizing the vast majority of the radioactivity.
At a time when we need every tool in the box to fight the climate crisis – and nuclear is one of the
most effective – isn’t it time we started to look at the facts rather than repeat myths about nuclear
waste?
(Adapted from https://www.theguardian.com/environment/2021/aug/31/nuclear-energy-is-key-in-fight-for-climate)

AULA 09 – RELATIVE CLAUSES AND QUANTIFIERS 20


TEACHER ANDREA BELO

QUESTÃO 05 (COLÉGIO NAVAL/INÉDITA) – According to the text, mark the INCORRECT option
(A) Nuclear energy is clean energy
(B) Common sense about nuclear energy does not match its characteristics
(C) Despite being clean, nuclear energy is second to solar in terms of waste production
(D) In fact, there is radioactivity in the nuclear power generation process
(E) Nuclear energy is a way to fight the climate crisis

QUESTÕES EAM
QUESTÃO 01 (EAM/INÉDITA) – Read the dialogue and mark the right option to fill in the gaps
respectively.
A: Hello, Denver!
B: Hey, Thomas! How are you and your wife?
A: We are ok. What about you?
B: I’m doing well. Hard times, man!
A: _________ you get the vaccine at the hospital you work?
B: Yes, I ________. Thank God!
A: You _______ lucky. I have to go! See you!
B: See you, take care!

(A) Did / did / went


(B) Were / was / is
(C) Was / was / did
(D) Were / was / did
(E) Did / did / are

QUESTÃO 02 (EAM/INÉDITA) – Read the sentences and mark the correct option to fill in the
blanks respectively.
Doctor Sam is ______ uncle. ________ lives near my house. All my family and friends call him
when they feel sick.
(A) Her/ you
(B) Your / I
(C) Her / She
(D) My / He
(E) His / He

AULA 09 – RELATIVE CLAUSES AND QUANTIFIERS 21


TEACHER ANDREA BELO

QUESTÃO 03 (EAM/INÉDITA) – Which grammarly correct form should be used in this album
cover to avoid abbreviation and get into formal language?

Which grammarly correct form should be used in this album cover to avoid abbreviation and get
into formal language?
(A) Going down slowly
(B) Goin’ down slowly
(C) Goin dow slowly
(D) Goin’ down slow
(E) Going down slow

QUESTÃO 04 (EAM/INÉDITA) – Use the verbs in the parentheses to complete the following
statements.
I – Catia and her sister sometimes ______ (do) their homework together after lunch, but now they
______ (do) another thing to make it different.
II – Carol_______ (eat) pizza very Fridays. Sometimes, she _______ (want) to eat vegetables but
she doesn’t like it.
III – Anne _________(live) in Africa. During the week, her daughters _______ (work) as tourist
guides tehre and they make a good money.

Now mark the option which completes them respectively.


(A) Are doing / am doing / is eating / wants / lives / working
(B) Am doing / do / eats / is wanting / is living / works
(C) Am doing / do / eat / is wanting / is living / works
(D) Do / do / is eating / want / live / is working
(E) Do / are doing / eats / wants / lives / work

AULA 09 – RELATIVE CLAUSES AND QUANTIFIERS 22


TEACHER ANDREA BELO

QUESTÃO 05 (EAM/INÉDITA) – Look at the picture below.

What is he doing in the picture?


(A) The man plays the guitar every day.
(B) He is playing the guitar now.
(C) The man didn’t play the guitar yesterday.
(D) He never plays the guitar.
(E) The man won’t play the guitar.

QUESTÃO 06 (EAM/INÉDITA)

Which missing word is in Stephen King’s quote?


(A) O verbo GO, após os verbos modais can e should.
(B) A palavra BUT, de contraste, no lugar de AND.
(C) O intensificador VERY, antes do adjetivo BRAVE.
(D) O verbo start no futuro, com uso do WILL antes dele.
(E) A preposição TO após o adjetico BRAVE, par a frase fazer sentido.

AULA 09 – RELATIVE CLAUSES AND QUANTIFIERS 23


TEACHER ANDREA BELO

QUESTÃO 07 (EAM/INÉDITA) – Which correctly form should be used for:


They arrived too early, _________ they?
(A) does
(B) didn’t
(C) aren’t
(D) are
(E) don’t

QUESTÃO 08 (EAM/INÉDITA) – Use the verbs in the parentheses to complete the following
statements.
I – I usually ______ (do) exercises by myself, but now I ______ (do) with my sister.
II – Richard_______ (complain) all the time. When angry, he _______ (want) to argue with
everybody around him.
III – Barbara _________(live) in Panamá. During the afternoon, her young brother and sister
_______ (go) to the beach while she is working.

Now mark the option which completes them respectively.


(A) Am doing / am doing / is complaining / wants / lives / going
(B) Am doing / do / complains / is wanting / is living / goes
(C) Am doing / do /complain / is wanting / is living / goes
(D) Do / do / is complaining / want / live / is going
(E) Do / am doing / complains / wants / lives / go

QUESTÃO 09 (EAM/INÉDITA) – Read the sentences and mark the correct option to fill in the
blanks respectively.
Sephora isn’t _____ best friend. _____ is only a classmate but _____ is already part of my life.

(A) Her/ you/you


(B) Your / I /you
(C) Her / she/ she
(D) My / she /she
(E) His / he /she

AULA 09 – RELATIVE CLAUSES AND QUANTIFIERS 24


TEACHER ANDREA BELO

QUESTÃO 10 (EAM/INÉDITA) – Read the sentences and mark the correct complement: Roberto
Carlos _____ a great singer.
(A) isn’t
(B) wasn’t
(C) were
(D) are
(E) is

QUESTÕES EEAR
Read the cartoon and answer questions 01, 02 and 03.

Adapted from https://www.gocomics.com/garfield/2021/03/21.

QUESTÃO 01 (EEAR/INÉDITA) – According to the cartoon, we can say that _______________.


(A) Odie’s daddy has just got home.
(B) Odie loves when “Daddy’s home” is on.
(C) “Daddy’s home” is a TV show.
(D) the man and his pets are excitedly watching TV.
(E) only Odie is happy when Daddy finally gets home.

QUESTÃO 02 (EEAR/INÉDITA) – “Now it’s time for ‘Daddy’s home’!”.


The reported form of the sentence is: it’s been announced on TV that
(A) then it was time for “Daddy’s home”.
(B) now it is time for “Daddy’s home”.
(C) now it was time for “Daddy’s home”.
(D) then it is time for “Daddy’s home”.
(E) then is it time for “Daddy’s home”.

AULA 09 – RELATIVE CLAUSES AND QUANTIFIERS 25


TEACHER ANDREA BELO

QUESTÃO 03 (EEAR/INÉDITA) – In “I’m right here!”, the underlined word


(A) works as the opposite of left.
(B) can’t be removed without changing the sentence’s meaning.
(C) shows that the man is at the correct place.
(D) emphasizes “here”.
(E) is incorrectly positioned in the sentence.

Read the text and answer questions 04 to 10.


The Titanic
The Titanic was built by the White Star Line. The owners of the company thought that if ocean
liners were big and luxurious enough more people ________ travel with them. The Titanic was
designed to be the largest in a series of three ships made by the White Star line. It was 268 meters
long, 28 meters wide, and weighed 45 000 tons. It produced enough power to travel at a speed
of 24 knots (about 40km per hour).
The bulk of the ship was divided into compartments. They were separated by steel doors that did
not let any water through. The ship could still move and float if 3 or 4 of the 16 compartments
were filled with water.
The Titanic was more like a floating hotel than a ship. It cost $7.5 million, and it was unlike any
other ship that had ever been built. Palm trees and other expensive plants decorated the luxurious
hallways and corridors. The ship could carry 2 600 passengers and a crew of 900.
On April 10, 1912 over 2200 passengers boarded the Titanic on its maiden voyage to New York.
Many of them were immigrants who saved all their money for the journey. First class passengers
had to pay between $2 500 and $4 500 for a private room and a bath, third class passengers had
to share rooms and paid $35 each. Although the ship’s owners said the Titanic was unsinkable,
many problems before the first voyage were overlooked.
Safety regulations at that time were not very strict. The ship only had 16 lifeboats, enough for
about 1 500 passengers. It was only tested for a few hours and never went at full speed. The
telegraph system on board was new and not many people knew how to operate it.
During the night of April 14, 1912, the waters of the North Atlantic had a temperature of about -
2° C. At noon on that day the radio operators got messages from other ships about icebergs that
were nearby. The Titanic’s captain, Edward Smith, did not care about these warnings. He was
captain of a steel giant that could not sink. The only thing he cared about was setting up a new
world speed record. The Titanic was to be the fastest ship that ever sailed from Southampton to
New York.
The night was clear, and the Titanic sped on. When a big iceberg was sighted, the first officer shut
down all the engines. But it would have taken the ship about half a mile to come to a full stop.
Even though, on the surface, the ship stayed clear of the iceberg, it ripped a big hole in the hull.
At once the compartments began to flood with cold, icy water.

AULA 09 – RELATIVE CLAUSES AND QUANTIFIERS 26


TEACHER ANDREA BELO

The bulkheads were lowered but it was too late. Water flooded at least five compartments. The
collision with the iceberg was so slight that the passengers hardly heard it. Most of them didn’t
take any notice and continued dancing and having fun. Some passengers were asleep in their
cabins.
The bow of the ship dipped under the water’s surface and the back part of the ship began to rise.
After a short time, the Titanic broke into two pieces. When Captain Smith realized that the Titanic
was sinking, he had a distress signal sent out, but the nearest ship was a hundred kilometers away.
As time went on, chaos emerged, and passengers rushed to the boat deck. Women and children
were allowed on the lifeboats first. Lights flickered and electricity was finally gone. At 2:20 a.m.
the Titanic disappeared into the Atlantic Ocean.
The Carpathia, which was the nearest ship, came to the scene about two hours later and picked
up the freezing passengers in their lifeboats. By early morning, the news of the disaster had gone
around the world. The world’s largest ocean liner, the Titanic, had sunk on its maiden voyage,
killing 1513 people.
Adapted from https://www.english-online.at/history/titanic/titanic.htm.

QUESTÃO 04 (EEAR/INÉDITA) – According to the text, the Titanic was _______________.


(A) bought by a company named the White Star Line.
(B) the biggest of a series of three ships.
(C) 268 metres wide.
(D) 28 metres long.
(E) the second largest ship of the White Star Line.

QUESTÃO 05 (EEAR/INÉDITA) – In “The telegraph system on board was new and not many
people knew how to operate it.”, the underlined pronoun refers to:
(A) on board.
(B) many people.
(C) Titanic.
(D) full speed.
(E) the system.

QUESTÃO 06 (EEAR/INÉDITA) – Choose the appropriate word to fill in the blank in the text.
(A) will.
(B) were going.
(C) can.
(D) are.
(E) would.

AULA 09 – RELATIVE CLAUSES AND QUANTIFIERS 27


TEACHER ANDREA BELO

QUESTÃO 07 (EEAR/INÉDITA) – In “The ship could still move and float if 3 or 4 of


the 16 compartments were filled with water.”, the underlined modal verb indicates
(A) prohibition.
(B) permission.
(C) advice.
(D) ability.
(E) obligation.

QUESTÃO 08 (EEAR/INÉDITA) – In “On April 10, 1912 over 2200 passengers boarded the Titanic
on its maiden voyage to New York.”, the underlined word is closest in meaning to
______________.
(A) marvelous.
(B) ordinary.
(C) initial.
(D) incredible.
(E) famous.

Read the extract below.


“The collision with the iceberg was so slight that the passengers hardly heard it. Most of them
didn’t take any notice and continued dancing and having fun. Some passengers were asleep in
their cabins.”

QUESTÃO 09 (EEAR/INÉDITA) – We can infer from the extract that ___________.


(A) the passengers barely heard the collision when it happened.
(B) the passengers knew right away what was going on.
(C) the collision with the iceberg was strong and loud.
(D) all passengers were having fun when Titanic struck an iceberg.
(E) the collision frightened the passengers as soon as it happened.

QUESTÃO 10 (EEAR/INÉDITA) – In “As time went on, chaos emerged, and passengers rushed to
the boat deck.”, the ___________ is being used.
(A) Simple Present.
(B) Simple Past.
(C) Past Participle.
(D) Imperative.
(E) Present Perfect.

AULA 09 – RELATIVE CLAUSES AND QUANTIFIERS 28


TEACHER ANDREA BELO

QUESTÕES EFOMM
Read the text and answer questions 01, 02 and 03.
Oxygen shortages are killing thousands. Why aren’t we doing more about this?
As with vaccines and drugs, the pandemic has shown how access to this life-saving resource is deeply unequal
The basics ___ caring for acutely unwell patients are simple: air needs to go in and out and blood
needs to go round and round. Across the world, the pandemic has consistently shown how poorly
equipped healthcare systems are for addressing these needs. Much attention has been paid to
vaccines, drug therapies and ventilators ___ recent months, while relatively little has been said
about the most basic human requirement of all – oxygen.
Oxygen is all around us, and yet there are acute shortages of it in many healthcare settings. This
is because the infrastructure needed to supply oxygen to patients, such as large vacuum-insulated
evaporators (which are like giant, very cold vacuum flasks), is relatively expensive and needs
regular maintenance and top-ups of liquid oxygen. Where this isn’t available, hospitals might use
concentrators ___ extract oxygen from the surrounding environment. These require electricity
and compressed air – which, again, are scarce in many places. Or they might use cylinders, which
can store and deliver short-term oxygen therapy. But they need regularly refilling, which depends
on secure supply lines.
I am a doctor who specializes in respiratory and intensive care medicine. Thankfully, I have never
been in a situation where I needed to provide a patient with oxygen therapy when none was
available. But this is exactly the situation facing healthcare workers in many other countries.
During the pandemic, systems for delivering oxygen came under pressure almost everywhere. In
the UK, hospitals that were decades old struggled to cope with surges of patients needing the
type of high oxygen flows used in modern therapy. In some low- and middle-income countries,
the infrastructure to deliver oxygen to patients was nonexistent.
(Adapted from https://www.theguardian.com/commentisfree/2021/jun/08/oxygen-shortages-killing-vaccines-drugs)

QUESTÃO 01 (EFOMM/INÉDITA) – According to the text, which option is correct?


(A) The pandemic clarified the fact that basic needs are often not well met by health systems
(B) Oxygen has received as much attention as vaccines at this time of pandemic
(C) Oxygen scarcity is minimal as it is an element that we have in abundance around us
(D) The pandemic did not overload oxygen delivery systems
(E) The supply of oxygen occurred in all countries in the world, even if it was scarcely

QUESTÃO 02 (EFOMM/INÉDITA) – Which is the correct option to complete the gaps in the text?
(A) At / on / that
(B) At / in / that
(C) Of / in / that
(D) Of / on / that
(E) Of / in / with

AULA 09 – RELATIVE CLAUSES AND QUANTIFIERS 29


TEACHER ANDREA BELO

QUESTÃO 03 (EFOMM/INÉDITA) – What’s the meaning of the word “struggled” in paragraph 3?


(A) Forgot
(B) Battled
(C) Tried
(D) Competed
(E) Contested

Read the text and answer questions 04, 05 and 06.


If we loosen restrictions too early, there is a real risk of a third wave in the UK
Not enough people are vaccinated against Covid as a new variant spreads, requiring us to
remain vigilant for a little longer
Just when it felt like we could begin to relax again, Covid-19 has thrown us yet another curveball.
While we’ve been debating in the UK where to go for a holiday and booking long-overdue nights
out with friends, the virus has been causing havoc across the world.
The Kent variant, B.1.1.7, found its way into countries such as Taiwan and Singapore; P.1 is causing
another wave in Brazil; and most devastatingly, the highly transmissible variant B.1.617 (which
has three subtypes) is causing a public health crisis in India. The country has seen cases surging
with estimated deaths to be several times greater than that being reported by the government,
which continues to downplay the epidemic.
The UK has done well, largely through strong public compliance, in bringing its case numbers
down, but Boris Johnson has repeated his mistake from last summer and left the borders open.
The Sunday Times estimates that at least 20,000 passengers from India were allowed to enter the
UK while Johnson delayed imposing a travel ban because he didn’t want to upset the Indian prime
minister, Narendra Modi, while negotiating a trade deal. This is at a time when other countries
such as New Zealand and Hong Kong completely stopped all flights.
This puts us now in a precarious situation. The World Health Organization has identified B.1.617
as a “variant of concern” because it is at least as transmissible as the Kent variant, potentially even
more so. This means that the virus can jump faster among those susceptible and accelerate out
of control. The UK has not fully vaccinated enough of the population to avoid a third wave among
younger people that could still strain the NHS.
(Adapted from https://www.theguardian.com/commentisfree/2021/may/17/third-wave-risk-uk-vaccinated-covid-variant)

QUESTÃO 04 (EFOMM/INÉDITA) – It is possible to infer from the text that


(A) Brazil and India are the countries facing the best phase of the pandemic, with a decrease in
cases and deaths
(B) The UK failed to reduce the number of cases
(C) The UK is in a very comfortable situation regarding the pandemic
(D) The situation in the UK would be better if tourists had not been allowed in
(E) It is impossible for a third wave to happen in the UK because of the advance in vaccination

AULA 09 – RELATIVE CLAUSES AND QUANTIFIERS 30


TEACHER ANDREA BELO

QUESTÃO 05 (EFOMM/INÉDITA) – In the excerpt “...the virus has been causing havoc across the
world” (paragraph 1), the word in bold means
(A) Waste
(B) Miracle
(C) Calm
(D) Misunderstanding
(E) Chaos

QUESTÃO 06 (EFOMM/INÉDITA) – the correct alternative to complete the paragraph below


“Covid-19 has killed millions around the world, but ___ some who are lucky enough to survive the
infection, ___ nightmare is not over: adding insult to injury are deadly fungal infections that follow
in the wake of the virus. Making matters worse, inequities ___ long predated the pandemic have
left some countries without the capacity to combat these serious infections”.
(Adapted from https://www.theguardian.com/commentisfree/2021/jun/02/black-fungus-covid-india-mucormycosis)

(A) To / the / that


(B) For / the / that
(C) For / the / in
(D) To / the / in
(E) For / a / that

Read the text and answer questions 07, 08, 09 and 10.
After a year at home, children with disabilities deserve priority vaccination
They disappeared more than a year ago and many are still out of sight. When the pandemic hit,
53,000 under-18s in England with disabilities that made them vulnerable to coronavirus began to
shield away at home.
Kept off school long after their classmates went back, and away from friends, they have found
their childhoods put on hold. And while the vaccine rollout gave high-risk adults some reprieve in
the new year, those aged under 16, who have not been eligible for any vaccine, are still living in
limbo.
Yet we have barely heard a thing about disabled children’s plight through the coronavirus crisis –
a silence that has not been helped by the faux-reassuring narrative that “no healthy child” has
died from the virus. Now that the UK regulator has at last approved the Pfizer vaccine for 12- to
15-year-olds, this blind spot is becoming even more glaring.
(Adapted from https://www.theguardian.com/commentisfree/2021/jun/10/children-disabilities-priority-vaccination-england-education-shielding)

AULA 09 – RELATIVE CLAUSES AND QUANTIFIERS 31


TEACHER ANDREA BELO

QUESTÃO 07 (EFOMM/INÉDITA) – In the phrase “...53,000 under-18s in England with disabilities


that made them vulnerable to coronavirus began to shield away at home” (paragraph 1), the
underlined word is a synonym for
(A) Defenseless
(B) Safe
(C) Unsusceptible
(D) Closed
(E) Open

QUESTÃO 08 (EFOMM/INÉDITA) – The second paragraph of the text states that


(A) Children with disabilities followed the same safety rules in the pandemic as children without
disabilities
(B) The childhood of children with disabilities was not affected by the pandemic
(C) There are still people who cannot be vaccinated
(D) Vaccination did not help high-risk adults to calm down on the holiday season
(E) No children have returned to face-to-face classes yet

QUESTÃO 09 (EFOMM/INÉDITA) – In the phrase “And while the vaccine rollout gave high-risk
adults some reprieve in the new year…” (paragraph 2), the underlined word is a synonym for
(A) Continuation
(B) Clemency
(C) Immunity
(D) Order
(E) Recover

QUESTÃO 10 (EFOMM/INÉDITA) – The text


(A) Develops the idea that children without disabilities have many privileges
(B) Focuses on the fact that high-risk adults should not have been given priority in vaccination
(C) Advocate for the vaccination of all children as soon as possible
(D) Aims to explain the importance of social interaction for children and their development
(E) Explains the experience of children with disabilities during the pandemic and defends their
priority in vaccination

AULA 09 – RELATIVE CLAUSES AND QUANTIFIERS 32


TEACHER ANDREA BELO

QUESTÕES EPCAR
Directions: Read the text below and answer questions according to the text.
TEXT
Rare purple textiles from the time of biblical kings found for the first time in Israel
By Harry Baker – Staff Writer
The color was a favorite of the biblical kings David and Solomon
A patch of 3,000-year-old wool dyed true purple, recently discovered in Timna Valley in Israel.
Archaeologists have uncovered rare fragments of 3,000-year-old textiles stained purple — a color
considered the height of royal fashion at the time — in southern Israel.
The archaeologists discovered the purple textiles — which included bits of woven fabric, a tassel
and a bundle of wool fibers — at Slaves' Hill in Timna Valley, an ancient copper production district
in the Arava desert, the team wrote in a new study describing the findings.
Radiocarbon dating of the fabrics revealed that they were woven around 1000 B.C., placing them
in the time of the biblical kings David and Solomon — who ruled from 1010–970 B.C. and 970–
931 B.C., respectively — in Jerusalem. The Bible mentions the kings and other important figures
wearing the color during this time, according to the researchers.
Related: 10 fascinating biblical-era discoveries from 2018
The dye used to stain the fabrics was made from mollusks found hundreds of miles away in the
Mediterranean and was extremely valuable as a result. However, until now, no physical evidence
of its use had ever been recovered in Israel or the rest of the southern Levant (an area
encompassing the eastern Mediterranean).
"For the first time, we have direct evidence of the dyed fabrics themselves, preserved for some
3,000 years," Naama Sukenik, curator of organic finds at the Israel Antiquities Authority and lead
author of the new paper describing the textiles, told Live Science in an email. "Each fragment gives
us new information, tells us a new story about the site and important information about the
people that lived there."
(Adapted from https://www.livescience.com/59693-could-earth-turn-into-venus. – Access on 07/28/20)

QUESTÃO 01 (EPCAR/INÉDITA) – The text states that


(A) there are registers of discoveries with evidence.
(B) There are purple registers.
(C) Most of discoveries were in Israel.
(D) it’s possible that Israel succeeds because of the discoveries.
(E) There are all kinds of discoveries.

AULA 09 – RELATIVE CLAUSES AND QUANTIFIERS 33


TEACHER ANDREA BELO

QUESTÃO 02 (EPCAR/INÉDITA) – “Archaeologists have uncovered rare fragments of 3,000-year-


old textiles stained purple” (paragraph 1). The underlined word is a synonym for
(A) grubby
(B) blemish
(C) grimy
(D) smeared
(E) dirty

QUESTÃO 03 (EPCAR/INÉDITA) – Mark the alternative in which the highlighted word is used
with the same meaning as in the sentence below.
“…a color considered the height of royal fashion at the time – in southern Israel” (paragraph 1).

(A) I needed to study a lot for a long time.


(B) We had to understand that all the times you got late.
(C) He made me confess two times.
(D) She was cooking when you called.
(E) I arrived in time.

QUESTÃO 04 (EPCAR/INÉDITA) – Mark the alternative that is grammatically correct about the
text.
(A) The archaeologists found purple textiles
(B) The archaeologists has found purple textiles
(C) The archaeologists haven’t found purple textiles
(D) The archaeologists has been found purple textiles
(E) The archaeologists will find purple textiles

QUESTÃO 05 (EPCAR/INÉDITA) – Mark the statement that is in DISAGREEMENT with the text.
(A) bits of woven fabric, a tassel and a bundle of wool fibers were included.
(B) bits of woven fabric, a tassel and a bundle of wool fibers were not included
(C) fabric, a tassel and a bundle of wool fibers was included.
(D) fabric, a tassel and a bundle of wool fibers were included.
(E) fabrics were not included.

AULA 09 – RELATIVE CLAUSES AND QUANTIFIERS 34


TEACHER ANDREA BELO

QUESTÃO 06 (EPCAR/INÉDITA) – Mark the alternative that the question is grammatically


INCORRECT.
“… the team wrote in a new study describing the findings.” (paragraph 2)

(A) Did the team write in a new study describing the findings?
(B) Has the team written in a new study describing the findings?
(C) Did the team wrote in a new study describing the findings?
(D) Has the team written something describing the findings?
(E) Did the team writing in a new study describing the findings?

QUESTÃO 07 (EPCAR/INÉDITA) – Mark the statement that is NOT mentioned in the text.
(A) The fabrics they found were from kings David and Solomon’s period.
(B) The fabrics can be placed at kings David and Solomon’s period.
(C) The fabrics refused kings David and Solomon’s period.
(D) The fabrics are part of kings David and Solomon’s period.
(E) The purple fabric is part of kings David and Solomon’s period.

QUESTÃO 08 (EPCAR/INÉDITA) – According to the text,


(A) The fabrics dye was made from mollusks
(B) The fabrics dye was produced with all kinds of mollusks
(C) The fabrics dye was made from mollusks waste
(D) The fabrics dye was made from mollusks reinforcement
(E) The fabrics dye was made from mollusks rest

QUESTÃO 09 (EPCAR/INÉDITA) – “However, until now, no physical evidence of its use had ever
been recovered in Israel or the rest of the southern Levant” (paragraph 4). The underlined word
is a synonym for
(A) Otherwise
(B) On the other hand
(C) Even so
(D) By the way
(E) Then

AULA 09 – RELATIVE CLAUSES AND QUANTIFIERS 35


TEACHER ANDREA BELO

QUESTÃO 10 (EPCAR/INÉDITA) – “… Naama Sukenik, curator of organic finds at the Israel


Antiquities Authority…” The highlighted word can be substituted for ___.
(A) guardian
(B) burglar
(C) mugger
(D) poacher
(E) doctor

QUESTÕES ESA
Texto para responder à questão 01
RIO DE JANEIRO, BRAZIL - In the past 24 hours, Brazil recorded 1,113 deaths and 36,653 new cases
related to the novel coronavirus. The data are included in the Ministry of Health's daily balance
released on Tuesday evening, September 15th.
Since the start of the pandemic, 133,119 people have died as a result of Covid-19. On Monday,
the Ministry of Health's data system recorded a total of 132,006 deaths. A further 2,445 are still
under investigation by health authorities.
(Adapted from https://riotimesonline.com - September 15th)

QUESTÃO 01 (ESA/INÉDITA) – According to the text, it is correct to say that:


(A) lot of researchers will introduce a new protective mask.
(B) one specific researcher is one of the responsible institution for the protective mask.
(C) Coope/UFRJ is the responsible main responsable for the protective mask.
(D) INMETRO has researched about the protective mask.
(E) researchers will introduce a new protective mask.

QUESTÃO 02 (ESA/INÉDITA) – Which sentence is grammatically correct?


(A) Yesterday, I finished my activities more earlier than the night before.
(B) Cleyton is more thin than his best friend.
(C) My towel is more big than my sister’s one.
(D) Going by train is expensiver than by bus because of the tickets.
(E) Robert’s shoes are bigger than mine.

AULA 09 – RELATIVE CLAUSES AND QUANTIFIERS 36


TEACHER ANDREA BELO

QUESTÃO 03 (ESA/INÉDITA) – Complete the sentence below using the appropriate words:
“Mr. Copolla _______ selling cars: He ______ stops different car sales and he makes a lot of money.”

(A) Likes/don’t
(B) Likes/doesn’t
(C) Like/do
(D) Like/don’t
(E) Like/does

QUESTÃO 04 (ESA/INÉDITA) – “___ the new manager in this store? Where did you come from?”
Complete the space with the correct form of the verb and the pronoun.
(A) You is
(B) You are
(C) Are you
(D) Is you
(E) Am you

Texto para a questão 05


August 08, 2020
RIO DE JANEIRO, BRAZIL - On Saturday, August 8th, Brazil reached the number of 100,477 deaths
from Covid-19 since the start of the novel coronavirus pandemic. According to the Ministry of
Health, 905 deaths were recorded over the last 24 hours.
Of the 3,012,412 cases of people infected by the novel coronavirus, 2,094,293 (69.5 percent) have
recovered.
A total of 49,970 new cases have been reported by Health Secretariats since Friday, August 7th.
The official also shows that the number of people under follow-up stands at 817,642 (21.1
percent).
(Adapted from https://riotimesonline.com)

QUESTÃO 05 (ESA/INÉDITA) – According to the text, it is correct to say that:


(A) more than a hundred thousand deaths is the number described.
(B) A hundred thousand deaths is the number showed.
(C) Less than a hundred thousand deaths is the number described.
(D) more than a hundred thousand people are working on the research.
(E) the correct number of deaths is a thousand.

AULA 09 – RELATIVE CLAUSES AND QUANTIFIERS 37


TEACHER ANDREA BELO

QUESTÕES ESCOLA NAVAL


How to defeat AIDS, malaria and tuberculosis

At the turn of the millennium, it was clear a new approach was required in the war against three
of the biggest threats to human life and development. There was need for a trustworthy
international organization that could solicit donations from rich countries and wealthy
organizations and spend that money on combating those threats in collaboration with the
governments of afflicted poor countries, but with appropriate oversight to ensure effectiveness
and avoid theft. The result was the Global Fund to Fight aids, Tuberculosis and Malaria.
And it worked. Though it is impossible to say what would have happened without the Global Fund,
as it is now formally known, the fund’s officials claim to have saved 32m lives since it opened in
2002. (…)
The Economist – October 10th

QUESTÃO 01 (ESCOLA NAVAL/INÉDITA) – The text shows that


a) Some diseases should be taken as relevant because they are considered threats.
b) Some diseases are being researched in a clear way since 2002.
c) It is important to know how to defend yourself related to some diseases.
d) It is necessary to spend some money at the turn of the millenium.
e) The turn of the millenium required some effectiveness.

QUESTÃO 02 (ESCOLA NAVAL/INÉDITA) – Mark the incorrect definition of “Trustworthy


international organization”
a) An example of steadfast international organization.
b) A kind of devoted international organization.
c) A kind of resolute international organization.
d) A dutiful international organization.
e) An unreliable international organization.

AULA 09 – RELATIVE CLAUSES AND QUANTIFIERS 38


TEACHER ANDREA BELO

QUESTÃO 03 (ESCOLA NAVAL/INÉDITA) – According to the text, the adjective “wealthy” means
a) rich
b) delightful
c) subtle
d) swell
e) winsome

QUESTÃO 04 (ESCOLA NAVAL/INÉDITA) – The text


a) points out several diseases and their heal.
b) shows some diseases and uphold them.
c) highlights the cruelest diseases and settle them.
d) states the right way to cure those diseases.
e) demonstrates some diseases names and their cure importance.

QUESTÃO 05 (ESCOLA NAVAL/INÉDITA) – The text conclusion seems to be


a) logical and straight.
b) nonsense and worn out.
c) controversial and useful.
d) excessive and repeated.
e) confused and nuclear.

Texto para questões 06 a 08


Taal: The 'very small but dangerous volcano'
BBC News (Jan/2020)

Over the past few days, it's begun spewing lava, triggering earthquakes, and emitting huge plumes
of ash that have spread across the island of Luzon and beyond.

AULA 09 – RELATIVE CLAUSES AND QUANTIFIERS 39


TEACHER ANDREA BELO

Scientists fear a bigger "hazardous eruption" is imminent. Taal is tiny, as volcanoes go, but it has
been deadly before. And according to Renato Solidum, the head of the Philippines' Institute of
Volcanology and Seismology (Philvolcs), it is "very small but a dangerous volcano". "Taal volcano
is a baby volcano sitting within a much bigger caldera volcano," said Ben Kennedy, associate
professor of physical volcanology at the University of Canterbury in New Zealand. The entire
Volcano Island has been marked as a permanent danger zone by Phivolcs.

QUESTÃO 06 (ESCOLA NAVAL/INÉDITA) – According to the text, which option is correct about
Taal volcano
a) Taal is a wee and dangerous volcano.
b) Taal is a huge and menacing volcano.
c) Taal is a tiny and sheltered volcano.
d) Taal is a huge and hazardous volcano.
e) Taal is a big and treacherous volcano.

QUESTÃO 07 (ESCOLA NAVAL/INÉDITA) – What is the meaning of the verb “triggering” in the
sentence “… it's begun spewing lava, triggering earthquakes.”
a) to raze
b) to defeat
c) to ruin
d) to tear down
e) to incite

QUESTÃO 08 (ESCOLA NAVAL/INÉDITA) – Mark the incorrect option according to the text
a) People are afraid about the eruption.
b) Taal is not big but it can cause big consequences.
c) Scientists fear that the eruption can come and destroy a lot.
d) One of the volcano consequences can be earthquakes.
e) Taal is an unsafe and risky volcano.

Texto para questões de 09 e 10


Massive and malodorous – world's biggest flower found

AULA 09 – RELATIVE CLAUSES AND QUANTIFIERS 40


TEACHER ANDREA BELO

The largest single flower ever recorded was found recently in Sumatra, Indonesia, measuring a
reported 111cm (3.64ft) across. This was a specimen of Rafflesia tuan-mudae and beat the
previous largest flower record of 107cm for Rafflesia arnoldii, also in Sumatra.
Rafflesia is not only a giant flower, but it has no leaves, stems or proper roots. It cannot
photosynthesise and instead sucks the food and water out of a particular vine using long thin
filaments that look like fungal cells. It gorges itself on the vine for a few years before bursting out
into a flower bud, swells for several months before blooming into a flower that looks like a bright
red bucket with big thick lobes. It gives off a whiff of rotting meat that, together with its gigantic
size, helps attract pollinating flies. Rafflesia also steals some of the DNA from the vine it lives on,
using it for its own genetic code for reasons that are not clear.

QUESTÃO 09 (ESCOLA NAVAL/INÉDITA) – According to the text


a) A flor raflesia é gigante mas a raiz e folhas são pequenas.
b) Não há fotossíntese por causa dos longos filamentos e fungos.
c) A flor incha por horas e demora para voltar a seu tamanho normal.
d) Ao desabrochar, se parece com um balde vermelho gigante brilhante.
e) Quando desabrocha, se parece com um buquê grande e vermelho cheio de flores.

QUESTÃO 10 (ESCOLA NAVAL/INÉDITA) – In the text, the sentence “It gives off a whiff of rotting
meat that, together with its gigantic size, helps attract pollinating flies.…” the underlined term
means
a) amiss smelling
b) neat smelling
c) special smelling
d) balm smelling.
e) reserved smelling.

QUESTÕES EsPCEx
Leia o texto a seguir e responda às questões 01, 02, 03, 04, 05 e 06.
India monsoon: Rescuers search for survivors after heavy rains
Rescuers are scouring devastated parts of western India for survivors after heavy rains caused deadly floods.
Tens of thousands of people have been moved out of affected areas, with record-breaking rainfall
reported along parts of the coast.
The states of Goa and Maharashtra have been badly affected, with many feared missing near the
financial hub Mumbai.
At least 136 people have died in Maharashtra, while in neighbouring Goa hundreds of homes have
been damaged.

AULA 09 – RELATIVE CLAUSES AND QUANTIFIERS 41


TEACHER ANDREA BELO

Many factors contribute to flooding, but experts say climate change caused by global warming
makes extreme rainfall more likely.
Heavy rains and flooding have also hit western Europe and parts of China in recent weeks, while
North America has grappled with scorching heatwaves.
The monsoon season in India lasts from June to September each year.
Rescuers have struggled to reach affected residents. Landslides have blocked roads, including the
main highway between Mumbai and Goa, and officials have deployed helicopters, divers and naval
rescue teams.
Goa's Chief Minister Pramod Sawant said flooding there was the worst in decades, and had caused
"widespread damage".
In the state of Maharashtra, major rivers are at risk of bursting their banks. Some 90,000 people
have been evacuated so far in the state.
The village of Taliye, southeast of Mumbai, saw landslides flatten most of its homes. An official
told Reuters news agency at least 42 people __________ there.
Maharashtra's chief minister, Udhav Thackeray, plans to visit Taliye on Saturday.
Weather experts say heavy rains will continue to lash the city over the next few days.
Indian Prime Minister Narendra Modi said he was "anguished" by the loss of lives.
"The situation in Maharashtra due to heavy rains is being closely monitored and assistance is being
provided to the affected," he tweeted.
https://www.bbc.com/news/world-asia-india-57952521

QUESTÃO 01 (EsPCEx/INÉDITA) – Choose the alternative containing the correct word to


complete the gap.
A) had died.
B) will have died.
C) have died.
D) has died.
E) have been died.

QUESTÃO 02 (EsPCEx/INÉDITA) – According to the text, choose the correct statement.


A) While in Goa hundreds of houses were destroyed, in Maharashtra 136 people are still missing.
B) According to specialists, one of the main factors causing drastic climate change is global
warming.
C) Despite record rains reported in parts of the coast of India, few families needed to be
removed from their homes.
D) Heavy rains hit not only India but also several other countries in the world such as the United
States and Canada.
E) India has suffered not only from record rains in parts of the coast but also from terrible heat
waves in its central region.

AULA 09 – RELATIVE CLAUSES AND QUANTIFIERS 42


TEACHER ANDREA BELO

QUESTÃO 03 (EsPCEx/INÉDITA) – In the sentence “[…] major rivers are at risk of bursting their
banks. Some 90,000 people have been […]”, the verb to burst is closest in meaning to
A) to wave.
B) to rock.
C) to crack.
D) to agree.
E) to mend.

QUESTÃO 04 (EsPCEx/INÉDITA) – "[...] in Maharashtra due to heavy rains is being closely


monitored [...]" (last paragraph). The expression due to can be correctly replaced without
changing the meaning by:
A) for instance.
B) at last.
C) rather than.
D) because of.
E) had better.

QUESTÃO 05 (EsPCEx/INÉDITA) – In the text “[...] southeast of Mumbai, saw landslides flatten
most of its homes.”, the word its refers to
A) southeast of Mumbai
B) flatten
C) Reuters news agency
D) the state of Maharashtra
E) The village of Taliye

QUESTÃO 06 (EsPCEx/INÉDITA) – Choose the alternative that has the sentence “The situation in
Maharashtra due to heavy rains is being closely monitored and assistance is being provided to
the affected," he tweeted” (last paragraph) correctly changed into reported speech.
A) He tweeted that the situation in Maharashtra due to heavy rains had been closely monitored
and assistance had been provided to the affected.
B) He tweeted that the situation in Maharashtra due to heavy rains has been closely monitored
and assistance has been provided to the affected.
C) He tweeted that the situation in Maharashtra due to heavy rains will be closely monitored and
assistance will be provided to the affected.
D) He tweeted that the situation in Maharashtra due to heavy rains was being closely monitored
and assistance was being provided to the affected.
E) He tweeted that the situation in Maharashtra due to heavy rains is closely monitored and
assistance was provided to the affected.

AULA 09 – RELATIVE CLAUSES AND QUANTIFIERS 43


TEACHER ANDREA BELO

Leia o texto a seguir e responda às questões 07, 08, 09 e 10.


Serial killer on death row Rodney Alcala dies of natural causes
A man sentenced to death in the US state of California for murdering a 12-year-old girl and four
other women ____________ of natural causes, officials say.
Rodney Alcala, 77, died at a hospital near California's Corcoran state prison in the early hours of
Saturday.
Alcala, who was known as the "Dating Game Killer" after taking part in a US TV show, was convicted
in 2010.
As well as the California killings, he had also pleaded guilty to the murders of two other women in
New York.
Alcala was first handed the death penalty in Orange County in 1980 for the kidnap and murder of
Robin Samsoe, 12, in Los Angeles the previous year.
However, his sentence was overturned by the California Supreme Court and he was granted a new
trial.
He later received the same penalty in the second trial, but this was again overturned in 2003.
In the years that followed, investigators discovered forensic evidence linking Alcala to the other
California murders.
At the 2010 trial, he was found guilty of killing Samsoe as well as four other women aged between
18 and 32 in the years between 1977 and 1979.
In September 1978, Alcala took part in The Dating Game, an American TV show in which a single
woman questioned three single men hidden from her view before selecting one based on their
answers.
Alcala was selected at the end of show, but the woman later said she decided not to date him after
a conversation with him backstage, describing him as "creepy".
In 2012, Alcala was extradited to New York after he was charged over two additional murders from
1971 and 1977. He pleaded guilty and was sentenced in New York to 25 years to life.
California state prison officials said in a statement on Saturday that Alcala was suspected of being
involved in a number of additional killings.
No further details were given about his death.
There are currently some 700 inmates on death row in California, but Governor Gavin Newsom
has ordered a moratorium on executions.
https://www.bbc.com/news/world-us-canada-57955961

AULA 09 – RELATIVE CLAUSES AND QUANTIFIERS 44


TEACHER ANDREA BELO

QUESTÃO 07 (EsPCEx/INÉDITA) – According to the text, choose the correct statement.


A) A man sentenced to death for murder in the US state of California is murdered in jail.
B) In addition to the crimes Rodney Alcala had been convicted of, he was suspected of a number
of additional crimes according to California state prison officials.
C) Rodney Alcala was wrongfully sentenced to death, which can be seen in the failure of the
California Supreme Court to have twice overturned his sentence.
D) Rodney Alcala was sentenced to life in prison by the state of New York and to the death
penalty by the state of California where he was executed.
E) Several times Rodney Alcala had his sentence overturned by the California Supreme Court due
to lack of forensic evidence in the investigations.

QUESTÃO 08 (EsPCEx/INÉDITA) – Choose the alternative containing the correct word to


complete the gap.
A) has died.
B) have died.
C) had died.
D) had been died.
E) have been dying.

QUESTÃO 09 (EsPCEx/INÉDITA) – In the sentence “As well as the California killings, he had also
pleaded guilty to the murders of two other women in New York”, the expression As well as is
closest in meaning to
A) At least.
B) Since.
C) Hardly.
D) Eventually.
E) Besides.

QUESTÃO 10 (EsPCEx/INÉDITA) – "However, his sentence was overturned by the California


Supreme Court and he was granted a[...]" (paragraph 6). The word “However” can be correctly
replaced without changing the meaning by:
A) Despite.
B) Whereas.
C) Yet.
D) Even though.
E) Thus.

AULA 09 – RELATIVE CLAUSES AND QUANTIFIERS 45


TEACHER ANDREA BELO

GABARITO
GABARITO AFA
01 – B 02 – E 03 – C 04 – A 05 – D
06 – A 07 – B 08 – E 09 – D 10 – A

GABARITO COLÉGIO NAVAL


01 – E 02 – B 03 – A 04 – A 05 – C

GABARITO EAM
01 – E 02 – D 03 – E 04 – E 05 – B
06 – E 07 – B 08 – E 09 – D 10 – E

GABARITO EEAR
01 – C 02 – A 03 – D 04 – B 05 – E
06 – E 07 – D 08 – C 09 – A 10 – B

GABARITO EFOMM
01 – A 02 – C 03 – B 04 – D 05 – E
06 – B 07 – A 08 – C 09 – B 10 – E

GABARITO EPCAR
01 – A 02 – B 03 – D 04 – A 05 – B
06 – C 07 – C 08 – A 09 – C 10 – A

GABARITO ESA
01 – E 02 – E 03 – B 04 – C 05 – A

GABARITO ESCOLA NAVAL


01 – A 02 – E 03 – A 04 – E 05 – E
06 – A 07 – E 08 – A 09 – D 10 – A

GABARITO EsPCEx
01 – A 02 – B 03 – C 04 – D 05 – E
06 – D 07 – B 08 – A 09 – E 10 – C

AULA 09 – RELATIVE CLAUSES AND QUANTIFIERS 46


TEACHER ANDREA BELO

QUESTÕES COMENTADAS
QUESTÕES AFA
Read the text and answer questions 01, 02 and 03.
'America Is On the Move Again.' Joe Biden Delivers His First Major Presidential Speech
President Joe Biden has attended dozens of presidential addresses to Congress since he first came
to Washington in 1973. But after nearly half a century of being an audience member, he finally
got the speaking slot Wednesday night to lay out his own vision for the country.
“America is ___ the move again,” he said during his roughly hour-long remarks. “Turning peril into
possibility. Crisis into opportunity. Setback into strength.”
Biden seized the opportunity to boast about his Administration’s accomplishments, highlighting
how Americans have received over 220 million shots of the COVID-19 vaccines in his first 100 days
in office, which he called “one of the greatest logistical achievements this country has ever seen.”
Like the rest of Biden’s presidential milestones, this one was altered because ___ the pandemic.
The audience in the House chamber, usually more than 1,000 people, including all members of
Congress and multiple Supreme Court justices, was limited to just 200. Attendees were seated
apart from each other to accommodate social distancing, with a maximum of three people in one
row.
His speech largely focused on economic priorities, and he outlined the American Families Plan, his
$1.8 trillion proposal ___ raise taxes on the wealthy to pay for affordable childcare, paid leave,
and access to education. If implemented, the plan would offer universal preschool for three- and
four-year-olds and two years of free community college and create a national paid leave program.
This proposal is the third prong of Biden’s $6 trillion plan to bolster the post-pandemic economy
by combating inequality and climate change. He signed the first part, the $1.9 trillion American
Rescue Plan, into law on March 11. In the past month, has also called for a $2 trillion bill that
would create millions of jobs and repair the country’s infrastructure, hailing it in his address to
Congress as a “blue-collar blueprint to build America.”
In total, these proposals would be the biggest investment in clean energy in U.S. history and the
largest federal investment in the U.S. economy since Lyndon Johnson’s Great Society.
(Adapted from https://time.com/5974129/joe-biden-joint-address-congress/)

QUESTÃO 01 (AFA/INÉDITA) – According to the text, which option is correct?


(A) Joe Biden has delivered dozens of presidential speeches since taking office
(B) Biden is taking a very positive and optimistic view of the current situation in the United
States
(C) The President of the United States did not have good logistical planning for vaccination
against COVID-19
(D) Joe Biden's presidential events have not been changed due to the pandemic
(E) Biden plans to tax the entire population equally in order to offer more free services

AULA 09 – RELATIVE CLAUSES AND QUANTIFIERS 47


TEACHER ANDREA BELO

Comentários: A alternativa A está incorreta. De acordo com o texto, não é correto afirmar que
Joe Biden fez dezenas de discursos presidenciais desde que assumiu o cargo, mas sim, que ele já
compareceu a dezenas de discursos presidenciais desde chegou em Washington. Isso pode ser
confirmado com o trecho “President Joe Biden has attended dozens of presidential addresses to
Congress since he first came to Washington in 1973”.
A alternativa B está correta. De acordo com o texto, é correto afirmar que Biden está tendo uma
visão muito positiva e otimista da situação atual nos Estados Unidos, assim como esta opção
indica. Isso pode ser confirmado com o trecho “Turning peril into possibility. Crisis into
opportunity. Setback into strength”.
A alternativa C está incorreta. De acordo com o texto, não é correto afirmar que o Presidente dos
Estados Unidos não tinha um bom planejamento logístico para a vacinação contra COVID-19, mas
sim, que, segundo ele, esse foi uma das maiores conquistas logísticas do país. Isso pode ser
confirmado com o trecho “…Americans have received over 220 million shots of the COVID-19
vaccines in his first 100 days in office, which he called ‘one of the greatest logistical achievements
this country has ever seen’”.
A alternativa D está incorreta. De acordo com o texto, não é correto afirmar que os eventos
presidenciais de Joe Biden não foram alterados devido à pandemia, mas sim, que foram. Isso pode
ser confirmado com o trecho “Like the rest of Biden’s presidential milestones, this one was altered
because of the pandemic”.
A alternativa E está incorreta. De acordo com o texto, não é correto afirmar que Biden planeja
taxar igualmente toda a população, a fim de oferecer mais serviços gratuitos, mas sim, que ele
planeja taxar mais os mais ricos para fazer isso. Isso pode ser confirmado com o trecho “...proposal
to raise taxes on the wealthy to pay for affordable childcare, paid leave, and access to education”.
GABARITO: B

QUESTÃO 02 (AFA/INÉDITA) – Which is the correct option to complete the gaps in the text?
(A) In / of / to
(B) On / X / to
(C) In / X / in
(D) On / of / in
(E) On / of / to
Comentários: A primeira lacuna deve ser preenchida com “on”, pois a frase faz uso da expressão
“on the move” (significa progredindo).
A segunda lacuna deve ser preenchida com “of”, pois a frase se refere ao discurso de Biden que
foi alterado por conta da pandemia, ou seja, “Como o resto dos marcos presidenciais de Biden,
este foi alterado por causa da pandemia”.
A terceira lacuna deve ser preenchida com “to”, pois a proposta de Biden, ou seja, “Seu discurso
se concentrou amplamente nas prioridades econômicas e ele delineou o Plano de Famílias
Americanas, sua proposta de US $ 1,8 trilhão para aumentar os impostos dos ricos para pagar por
creches acessíveis...”.
GABARITO: E

AULA 09 – RELATIVE CLAUSES AND QUANTIFIERS 48


TEACHER ANDREA BELO

QUESTÃO 03 (AFA/INÉDITA) – What’s the meaning of the word “bolster” in paragraph 5?


(A) Block
(B) Maintain
(C) Enhance
(D) Hinder
(E) Inspire
Comentários: A alternativa A está incorreta. A palavra “bolster” significa impulsionar e não pode
ser comparada com a palavra “block”, que significa bloquear.
A alternativa B está incorreta. A palavra “bolster” significa impulsionar e não pode ser comparada
com a palavra “maintain”, que significa manter.
A alternativa C está correta. A palavra “bolster” significa impulsionar e pode ser comparada com
a palavra “enhance”, que significa melhorar/realçar.
A alternativa D está incorreta. A palavra “bolster” significa impulsionar e não pode ser comparada
com a palavra “hinder”, que significa impedir.
A alternativa E está incorreta. A palavra “bolster” significa impulsionar e não pode ser comparada
com a palavra “inspire”, que significa inspirar.
GABARITO: C

Read the text and answer questions 04, 05 and 06.


The lockdown habit that’s hardest to break: using an iPad as a babysitter
How am I supposed to wean my children off screens when they know I’m addicted too?
It started, last March, as an act of necessity – sticking the kids on iPads all day so I could make a
hard deadline that fell six weeks into lockdown. There was no way around this; at five years old,
my kids couldn’t manage their Zoom schedules or self-entertain for long without fighting, and I
couldn’t break off every two minutes to help them.
Overnight, kindergarten and after-school disappeared, to be replaced with the sedative of kids
YouTube, and when the appeal of that started to wane, the more addictive and ruinous content
on TikTok. If it was hideous, I told myself, it was an emergency. It wouldn’t be like this for ever.
A year later and in New York, at least, we’re in a radically different place. Next week my kids’
elementary school goes back full-time and although it lets out early – at 2pm – half of the city is
now at least partially vaccinated, so most parents are comfortable hiring a sitter. (If they can find
one, that is; indoor after-school programmes still seem risky to many, so the scramble is on for
that already mythical creature, the 22-year-old sitter only looking for two hours work a day.)
Things are, on the surface, starting to look vaguely like normal.
Except, of course, they’re not. Among the many new habits formed during the pandemic, a
reliance on screens as babysitters may be one of the toughest to break. Over the course of the
year, I have grown accustomed to cleaning the house, finishing work, folding the laundry and even
– if my kids hit a solid addictive high on Minecraft – taking a nap, all without the cost or logistical

AULA 09 – RELATIVE CLAUSES AND QUANTIFIERS 49


TEACHER ANDREA BELO

planning of babysitting. I can put in a solid few hours at the park, safe in the knowledge that when
we get home help in the form of two hours of back-to-back three-minute videos will keep
everyone happy until dinner.
(Adapted from https://www.theguardian.com/commentisfree/2021/apr/26/lockdown-ipad-babysitter-children-addicted)

QUESTÃO 04 (AFA/INÉDITA) – It is possible to infer from the text that


(A) Although they are beginning to see a light at the end of the tunnel, mothers still see the
internet as the solution to isolated life
(B) The use of the internet as the only entertainment for children in isolated life, started out of
pure convenience
(C) The prospect was that that day-to-day life based on electronics would be the reality forever
(D) New York City is in a place very similar to last year
(E) The progress of vaccination in New York does not allow any hope for improvement
Comentários: A alternativa A está correta. De acordo com o texto, é correto inferir que embora
comecem a ver uma luz no fim do túnel, as mães ainda veem a internet como a solução para a
vida isolada, assim como esta opção indica. Isso pode ser confirmado com o trecho “I can put in a
solid few hours at the park, safe in the knowledge that when we get home help in the form of two
hours of back-to-back three-minute videos will keep everyone happy until dinner”.
A alternativa B está incorreta. De acordo com o texto, não é correto inferir que o uso da internet
como único entretenimento para crianças em vida isolada, nasceu por pura conveniência, mas
sim, que começou como um ato de necessidade. Isso pode ser confirmado com o trecho “It
started, last March, as an act of necessity – sticking the kids on iPads all day so I could make a
hard deadline that fell six weeks into lockdown”.
A alternativa C está incorreta. De acordo com o texto, não é correto inferir que a perspectiva era
que aquele dia a dia baseado em eletrônicos seria a realidade para sempre, mas sim, que havia
uma perspectiva otimista a respeito do futuro, por parte de uma mãe. Isso pode ser confirmado
com o trecho “If it was hideous, I told myself, it was an emergency. It wouldn’t be like this for
ever”.
A alternativa D está incorreta. De acordo com o texto, não é correto inferir que a cidade de Nova
York está em um lugar muito parecido com o ano passado, mas sim, que a situação já mudou
muito. Isso pode ser confirmado com o trecho “A year later and in New York, at least, we’re in a
radically different place. Next week my kids’ elementary school goes back full-time…”.
A alternativa E está incorreta. De acordo com o texto, não é correto inferir que o progresso da
vacinação em Nova York não permite qualquer esperança de melhora, mas sim, que permite, pois
metade da cidade já está, pelo menos, a meio caminho da imunização completa. Isso pode ser
confirmado com o trecho “...half of the city is now at least partially vaccinated…”.
GABARITO: A

AULA 09 – RELATIVE CLAUSES AND QUANTIFIERS 50


TEACHER ANDREA BELO

QUESTÃO 05 (AFA/INÉDITA) – In the excerpt “How am I supposed to wean my children off


screens when they know I’m addicted too?” (subtitle), the word in bold means
(A) Encourage
(B) Incite
(C) Help
(D) Detach
(E) Destroy
Comentários: A alternativa A está incorreta. A palavra “wean” significa, neste caso, “afastar” e
não pode ser comparada com a palavra “encourage”, que significa “encorajar”.
A alternativa B está incorreta. A palavra “wean” significa, neste caso, “afastar” e não pode ser
comparada com a palavra “incite”, que significa “incitar”.
A alternativa C está incorreta. A palavra “wean” significa, neste caso, “afastar” e não pode ser
comparada com a palavra “help”, que significa “ajudar”.
A alternativa D está correta. A palavra “wean” significa, neste caso, “afastar” e pode ser
comparada com a palavra “detach”, que significa “separar”.
A alternativa E está incorreta. A palavra “wean” significa, neste caso, “afastar” e não pode ser
comparada com a palavra “destroy”, que significa “destruir”.
GABARITO: D

QUESTÃO 06 (AFA/INÉDITA) – Choose the correct alternative to complete the paragraph below
Dusk is falling in ___ Indian capital, and the acrid smell ___ burning bodies fills the air. It’s the
evening of April 26, and ___ a tiny crematorium in a Delhi suburb, seven funeral pyres are still
burning. “I have lived here all my life and pass through this area twice ___ day,” says local resident
Gaurav Singh. “I have never seen so many bodies burning together.”
(Adapted from https://time.com/5964796/india-covid-19-failure/)

(A) The / of / at / a
(B) The / of / in / a
(C) The / with / in / an
(D) A / of / at / a
(E) A / of / in / a
Comentários: A primeira lacuna deve ser preenchida com “the”, pois a frase se refere
especificamente à capital indiana, ou seja, só existe uma. Sendo assim, “Anoitece na capital
indiana...”.
A segunda lacuna deve ser preenchida com “of”, pois a frase se refere ao cheiro de corpos sendo
cremados. Ou seja, “...e o cheiro acre de corpos queimados preenche o ar...”.

AULA 09 – RELATIVE CLAUSES AND QUANTIFIERS 51


TEACHER ANDREA BELO

A terceira lacuna deve ser preenchida com “at”, pois a frase se refere a um lugar específico. Ou
seja, “...e em um minúsculo crematório em um subúrbio de Delhi, sete piras funerárias ainda estão
queimando”.
A quarta lacuna deve ser preenchida com o trecho “a”, pois a frase se refere a expressão “... a
day”, ou seja, por dia. Sendo assim, “Vivi aqui toda a minha vida e passo por esta área duas vezes
por dia”.
GABARITO: A

Read the text and answer questions 07, 08, 09 and 10.
Australians stuck overseas 'abandoned' by their own country
Australian citizen Mandeep Sharma feels utterly deserted by his government.
He is one of the 9,000 Australian nationals stranded in India right now, left to fend for themselves
after Canberra this week banned all flights from the pandemic-ravaged nation until mid-May.
He has a wife and two daughters in Adelaide and fears their separation could drag on for months.
Catching Covid in India is also a real risk.
The Australian government's flight ban was the latest hardline action taken to keep the virus out
of the country. Infection rates are near zero, and Australia has had far fewer fatalities than most
countries, thanks to strict border controls and quarantine measures.
Still, those policies have left many Australians locked out. The ban on Indian arrivals marked an
escalation – the first time the country has stopped evacuations and blocked citizens from
returning home altogether. It's intensified calls for more to be done to get Australians home.
Why can't citizens get in?
Prior to the pandemic, there were estimated to be about one million Australians living overseas.
Many have come home in the past year, but many more have struggled to return.
Currently, around 36,000 nationals are registered for government help to fly home, but the true
number of those who wish to come back is estimated to be much higher.
Australia became one of the first nations to close its borders in March 2020. Only returning
nationals and residents were allowed in. Some exemptions were granted, including celebrities,
sports stars and contract workers. Since April, it has also allowed travellers from virus-free New
Zealand.
Those who returned were forced to undertake – and fund – a two-week quarantine stay at hotels
in state capitals. In the early pandemic rush, state governments agreed to take on the burden of
running quarantine instead of the federal government, given the lack of national facilities.
About a month in, a problem emerged. The number of people returning home - mostly from New
Zealand, the US and the UK- threatened to overwhelm the system. There were only so many hotel
rooms available.
But instead of expanding the system - for example, adding a purpose-built quarantine centre –
the government drastically cut back the number of plane arrivals allowed in each week.
(Adapted from https://www.bbc.com/news/world-australia-56924188)

AULA 09 – RELATIVE CLAUSES AND QUANTIFIERS 52


TEACHER ANDREA BELO

QUESTÃO 07 (AFA/INÉDITA) – In the phrase “He is one of the 9,000 Australian nationals
stranded in India right now…” (paragraph 1), the underlined word is a synonym for
(A) Marooned
(B) Imprisoned
(C) Found
(D) Homeless
(E) Stablished
Comentários: A alternativa A está incorreta. A palavra “stranded” significa preso/encalhado e,
nesse caso, não é sinônimo para “marooned”, que significa perdido/abandonado.
A alternativa B está correta. A palavra “stranded” significa preso/encalhado e, nesse caso, é
sinônimo para “imprisioned”, que significa preso.
A alternativa C está incorreta. A palavra “stranded” significa preso/encalhado e, nesse caso, não
é sinônimo para “found”, que significa encontrado.
A alternativa D está incorreta. A palavra “stranded” significa preso/encalhado e, nesse caso, não
é sinônimo para “homeless”, que significa sem-teto.
A alternativa E está incorreta. A palavra “stranded” significa preso/encalhado e, nesse caso, não
é sinônimo para “stablished”, que significa estabelecido.
GABARITO: B

QUESTÃO 08 (AFA/INÉDITA) – third paragraph of the text states that


(A) The ban on flights to Australia was the first effective measure to prevent the spread of the
virus in the country
(B) Banning flights was a government strategy to keep the virus in the country
(C) There are no new cases of coronavirus in Australia
(D) Australia has a terrible mortality rate from coronavirus
(E) Australia's prosperous results are due to the way the country handled the pandemic
Comentários: A alternativa A está incorreta. O terceiro parágrafo não afirma que a proibição de
voos para a Austrália foi a primeira medida eficaz para prevenir a propagação do vírus no país,
mas sim, que foi a última medida tomada para evitar a propagação do vírus no país. Isso pode ser
confirmado com o trecho “The Australian government's flight ban was the latest hardline action
taken to keep the virus out of the country”.
A alternativa B está incorreta. O terceiro parágrafo não afirma que proibir voos era uma estratégia
governamental para manter o vírus no país, mas sim, que tinha o intuito de manter o vírus fora
do país. Isso pode ser confirmado com o trecho “The Australian government's flight ban was the
latest hardline action taken to keep the virus out of the country”.

AULA 09 – RELATIVE CLAUSES AND QUANTIFIERS 53


TEACHER ANDREA BELO

A alternativa C está incorreta. O terceiro parágrafo não afirma que não há novos casos de
coronavírus na Austrália, mas sim, que as taxas de infecção estão perto de zero, e não zeradas.
Isso pode ser confirmado com o trecho “Infection rates are near zero...”.
A alternativa D está incorreta. O terceiro parágrafo não afirma que a Austrália tem uma taxa de
mortalidade terrível por coronavírus, mas sim, que o país tem menos fatalidades do que a maioria
dos países. Isso pode ser confirmado com o trecho “...Australia has had far fewer fatalities than
most countries…”.
A alternativa E está correta. O terceiro parágrafo afirma que os resultados prósperos da Austrália
se devem à forma como o país lidou com a pandemia, assim com esta opção indica. Isso pode ser
confirmado com o trecho “...Australia has had far fewer fatalities than most countries, thanks to
strict border controls and quarantine measures”.
GABARITO: E

QUESTÃO 09 (AFA/INÉDITA) – In the phrase “The number of people returning home - mostly
from New Zealand, the US and the UK- threatened to overwhelm the system” (paragraph 9), the
underlined word is a synonym for
(A) Enlighten
(B) Win
(C) Surrender
(D) Disturb
(E) Defeat
Comentários: A alternativa A está incorreta. A palavra “overwhelm” significa sobrecarregar e,
nesse caso, não é um sinônimo para a palavra “enlighten”, que significa iluminar.
A alternativa B está incorreta. A palavra “overwhelm” significa sobrecarregar e, nesse caso, não é
um sinônimo para a palavra “win”, que significa ganhar.
A alternativa C está incorreta. A palavra “overwhelm” significa sobrecarregar e, nesse caso, não é
um sinônimo para a palavra “surrender”, que significa render.
A alternativa D está correta. A palavra “overwhelm” significa sobrecarregar e, nesse caso, é um
sinônimo para a palavra “disturb”, que significa pertubar.
A alternativa E está incorreta. A palavra “overwhelm” significa sobrecarregar e, nesse caso, não é
um sinônimo para a palavra “defeat”, que significa derrotar.
GABARITO: D

AULA 09 – RELATIVE CLAUSES AND QUANTIFIERS 54


TEACHER ANDREA BELO

QUESTÃO 10 (AFA/INÉDITA) – The text


(A) Addresses an issue caused by disorganization, or neglect, of the Australian government
(B) Develops the idea that inhabitants should have the right to come and go in their home
country
(C) Aims to give visibility to those who are most affected by the pandemic in Australia
(D) Criticizes all the Australian government's disorganization regarding the pandemic situation
(E) Focuses on the idea that Australia has excellent results from restrictive measures against
COVID-19
Comentários: A alternativa A está correta. É correto afirmar que o texto aborda um problema
causado pela desorganização ou negligência do governo australiano, assim como esta opção
indica. Isso pode ser confirmado com o trecho “But instead of expanding the system - for example,
adding a purpose-built quarantine centre - the government drastically cut back the number of
plane arrivals allowed in each week”.
A alternativa B está incorreta. Não é correto afirmar que o texto desenvolve a ideia de que os
habitantes devem ter o direito de ir e vir em seu país de origem, mas sim, que ele aborda um
problema causado pela desorganização ou negligência do governo australiano. Isso pode ser
confirmado com o trecho “But instead of expanding the system - for example, adding a purpose-
built quarantine centre - the government drastically cut back the number of plane arrivals allowed
in each week”.
A alternativa C está incorreta. Não é correto afirmar que o texto visa dar visibilidade àqueles que
são mais afetados pela pandemia na Austrália, mas sim, que ele aborda um problema causado
pela desorganização ou negligência do governo australiano. Isso pode ser confirmado com o
trecho “But instead of expanding the system – for example, adding a purposebuilt quarantine
centre – the government drastically cut back the number of plane arrivals allowed in each week”.
A alternativa D está incorreta. Não é correto afirmar que o texto critica toda a desorganização do
governo australiano em relação à situação de pandemia, mas sim, que ele aborda um problema
causado pela desorganização ou negligência do governo australiano. Isso pode ser confirmado
com o trecho “But instead of expanding the system - for example, adding a purpose-built
quarantine centre - the government drastically cut back the number of plane arrivals allowed in
each week”.
A alternativa E está incorreta. Não é correto afirmar que o texto se concentra na ideia de que a
Austrália tem excelentes resultados de medidas restritivas contra COVID-19, mas sim, que ele
aborda um problema causado pela desorganização ou negligência do governo australiano. Isso
pode ser confirmado com o trecho “But instead of expanding the system – for example, adding a
purpose-built quarantine centre – the government drastically cut back the number of plane
arrivals allowed in each week”.
GABARITO: A

AULA 09 – RELATIVE CLAUSES AND QUANTIFIERS 55


TEACHER ANDREA BELO

QUESTÕES COLÉGIO NAVAL


Read Text I to do questions 01 to 04 based on it.
Texas passes law banning abortion after six weeks
A law banning abortion from as early as six weeks into pregnancy has come into effect in the
US state of Texas.
It bans abortions after the detection of what anti-abortion campaigners call a fetal heartbeat,
something medical authorities say is misleading.
The law, one of the most restrictive in the country, took effect after the Supreme Court did not
respond to an emergency appeal by abortion providers.
Doctors and women's rights groups have heavily criticized the law.
It gives any individual the right to sue doctors who perform an abortion past the six-week point.
The so-called "Heartbeat Act" was signed into law by Texas Governor Greg Abbott in May.
But rights groups, including Planned Parenthood and the American Civil Liberties Union (ACLU),
then requested that the Supreme Court block the legislation.
In the early hours of Wednesday, the ACLU confirmed that the court had "not responded to our
request", adding: "Access to almost all abortion has just been cut off for millions of people."
The group, which says that up to 90% of abortions in Texas take place after six weeks of pregnancy,
described the development as "blatantly unconstitutional".

QUESTÃO 01 (COLÉGIO NAVAL/INÉDITA) – Read the extract from the text


“…described the development as ‘blatantly unconstitutional’”.
Mark the alternative that can replace the underlined word without changing its meaning.
(A) Extremely
(B) Painfully
(C) Completely
(D) Specifically
(E) Brazenly
Comentários: A alternativa A está incorreta. A palavra “blatantly” significa “descaradamente” e
não pode ser comparada com a palavra “extremely”, que significa “extremamente”.
A alternativa B está incorreta. A palavra “blatantly” significa “descaradamente” e não pode ser
comparada com a palavra “painfully”, que significa “dolorosamente”.
A alternativa C está incorreta. A palavra “blatantly” significa “descaradamente” e não pode ser
comparada com a palavra “completely”, que significa “completamente”.
A alternativa D está incorreta. A palavra “blatantly” significa “descaradamente” e não pode ser
comparada com a palavra “specifically”, que significa “extremamente”.
A alternativa E está correta. A palavra “blatantly” significa “descaradamente” e pode ser
comparada com a palavra “brazenly”, que significa “descaradamente”.
GABARITO: E

AULA 09 – RELATIVE CLAUSES AND QUANTIFIERS 56


TEACHER ANDREA BELO

QUESTÃO 02 (COLÉGIO NAVAL/INÉDITA) – About the law that took effect in Texas, it’s correct to
affirm that
(A) It makes abortion more viable for everyone
(B) There are groups that are against the implementation of the law and that were ignored by
the Supreme Court
(C) Prohibits doctors from performing abortions after the six-week period
(D) It is, among the common sense of all, constitutional
(E) Despite being one of the most restrictive, it won't affect many people
Comentários: A alternativa A está incorreta. Sobre a lei que entrou em vigor no Texas, não é
correto afirmar que torna o aborto mais viável para todos, mas sim, que restringe o acesso ao
aborto para muitos. Isso pode ser confirmado com o trecho “Access to almost all abortion has just
been cut off for millions of people”.
A alternativa B está correta. Sobre a lei que entrou em vigor no Texas, é correto afirmar que
existem grupos que são contra a implementação da lei e que foram ignorados pelo Supremo
Tribunal, assim como esta opção indica. Isso pode ser confirmado com o trecho “Doctors and
women's rights groups have heavily criticised the law … the ACLU confirmed that the court had
‘not responded to our request’…”.
A alternativa C está incorreta. Sobre a lei que entrou em vigor no Texas, não é correto afirmar que
proíbe os médicos de realizar abortos após o período de seis semanas, mas sim, que dá o direito
a qualquer um de processar um médico que realize um aborto depois do período de seis semanas.
Isso pode ser confirmado com o trecho “It gives any individual the right to sue doctors who
perform an abortion past the six-week point”.
A alternativa D está incorreta. Sobre a lei que entrou em vigor no Texas, não é correto afirmar que
é, no senso comum de todos, constitucional, mas sim, que é considerada descaradamente
inconstitucional por alguns. Isso pode ser confirmado com o trecho “...described the development
as ‘blatantly unconstitutional’”.
A alternativa E está incorreta. Sobre a lei que entrou em vigor no Texas, não é correto afirmar que
apesar de ser uma das mais restritivas, não afetará muitas pessoas, mas sim, que afetará. Isso
pode ser confirmado com o trecho “...which says that up to 90% of abortions in Texas take place
after six weeks of pregnancy…”.
GABARITO: B

QUESTÃO 03 (COLÉGIO NAVAL/INÉDITA) – Read the extract from the text


“It gives any individual the right to sue doctors who perform an abortion past the six-week point”
(paragraph 4)
The word “who” refers to
(A) Doctors
(B) Any individual
(C) The law
(D) Abortion
(E) Women’s rights groups

AULA 09 – RELATIVE CLAUSES AND QUANTIFIERS 57


TEACHER ANDREA BELO

Comentários: A alternativa A está correta. A palavra “who” se refere aos médicos, citados
anteriormente na frase, assim como esta opção indica. Isso pode ser confirmado com o trecho
“Dá a qualquer indivíduo o direito de processar os médicos que realizam um aborto após o período
de seis semanas”.
A alternativa B está incorreta. A palavra “who” não se refere a qualquer indivíduo, mas sim, aos
médicos, citados anteriormente na frase. Isso pode ser confirmado com o trecho “Dá a qualquer
indivíduo o direito de processar os médicos que realizam um aborto após o período de seis
semanas”.
A alternativa C está incorreta. A palavra “who” não se refere a lei, mas sim, aos médicos, citados
anteriormente na frase. Isso pode ser confirmado com o trecho “Dá a qualquer indivíduo o direito
de processar os médicos que realizam um aborto após o período de seis semanas”.
A alternativa D está incorreta. A palavra “who” não se refere ao aborto, mas sim, aos médicos,
citados anteriormente na frase. Isso pode ser confirmado com o trecho “Dá a qualquer indivíduo
o direito de processar os médicos que realizam um aborto após o período de seis semanas”.
A alternativa E está incorreta. A palavra “who” não se refere aos grupos de direitos das mulheres,
mas sim, aos médicos, citados anteriormente na frase. Isso pode ser confirmado com o trecho “Dá
a qualquer indivíduo o direito de processar os médicos que realizam um aborto após o período de
seis semanas”.
GABARITO: A

QUESTÃO 04 (COLÉGIO NAVAL/INÉDITA) – Read the extract from the text


“Access to almost all abortion has just been cut off for millions of people” (paragraph 7)
The sentence above means that
(A) Millions of people have been deprived of the right to have nearly all abortions
(B) Abortion has been banned by law for millions of people
(C) Access to almost all abortions has been made easier for millions of people
(D) Millions of people have renounced the right to have almost all abortions
(E) Almost all abortions are available to millions of people
Comentários: A alternativa A está correta. A frase “O acesso a quase todos os abortos acaba de
ser cortado para milhões de pessoas” significa que “Milhões de pessoas foram privadas do direito
de realizar quase todos os abortos”, assim como esta opção indica.
A alternativa B está incorreta. A frase “O acesso a quase todos os abortos acaba de ser cortado
para milhões de pessoas” não significa que “O aborto foi proibido por lei para milhões de pessoas”.
A alternativa C está incorreta. A frase “O acesso a quase todos os abortos acaba de ser cortado
para milhões de pessoas” não significa que “O acesso a quase todos os abortos se tornou mais
fácil para milhões de pessoas”.

AULA 09 – RELATIVE CLAUSES AND QUANTIFIERS 58


TEACHER ANDREA BELO

A alternativa D está incorreta. A frase “O acesso a quase todos os abortos acaba de ser cortado
para milhões de pessoas” não significa que “Milhões de pessoas renunciaram ao direito de fazer
quase todos os abortos”.
A alternativa E está incorreta. A frase “O acesso a quase todos os abortos acaba de ser cortado
para milhões de pessoas” não significa que “Quase todos os abortos estão disponíveis para
milhões de pessoas”.
GABARITO: A

Read the text II to do item 05.


Nuclear energy is key in fight for climate
Ignore the myths about nuclear power, writes Rob Loveday of Generation Atomic – it is an
essential source of clean energy
The letter on nuclear energy (25 August) sadly could not be more wrong – nuclear is one of the
cleanest fuels we have and has always been so. The carbon and material footprints of nuclear –
for its entire lifetime, including mining and decommissioning – are lower than solar and on a par
with wind, according to the Intergovernmental Panel on Climate Change.
Moreover, the nuclear industry manages its waste stream – that is more than can be said for the
solar industry, which is set to produce millions of tons of toxic waste. Besides, spent nuclear fuel
is not “waste” but a valuable source of low-carbon energy that can be recycled via reprocessing
or proposed breeder reactors, thereby neutralizing the vast majority of the radioactivity.
At a time when we need every tool in the box to fight the climate crisis – and nuclear is one of the
most effective – isn’t it time we started to look at the facts rather than repeat myths about nuclear
waste?
(Adapted from https://www.theguardian.com/environment/2021/aug/31/nuclear-energy-is-key-in-fight-for-climate)

QUESTÃO 05 (COLÉGIO NAVAL/INÉDITA) – According to the text, mark the INCORRECT option
(A) Nuclear energy is clean energy
(B) Common sense about nuclear energy does not match its characteristics
(C) Despite being clean, nuclear energy is second to solar in terms of waste production
(D) In fact, there is radioactivity in the nuclear power generation process
(E) Nuclear energy is a way to fight the climate crisis
Comentários: A alternativa A está incorreta. De acordo com o texto, é correto afirmar que energia
nuclear é energia limpa. Isso pode ser confirmado com o trecho “…nuclear is one of the cleanest
fuels we have and has always been so”.
A alternativa B está incorreta. De acordo com o texto, é correto afirmar que o senso comum sobre
a energia nuclear não condiz com suas características. Isso pode ser confirmado com o trecho “The
letter on nuclear energy (25 August) sadly could not be more wrong – nuclear is one of the cleanest
fuels we have and has always been so”.

AULA 09 – RELATIVE CLAUSES AND QUANTIFIERS 59


TEACHER ANDREA BELO

A alternativa C está correta. De acordo com o texto, não é correto afirmar que apesar de ser limpa,
a energia nuclear é secundária à solar em termos de produção de resíduos, mas sim, que a energia
solar é pior nesse quesito. Isso pode ser confirmado com o trecho “...the nuclear industry manages
its waste stream – that is more than can be said for the solar industry, which is set to produce
millions of tons of toxic waste”.
A alternativa D está incorreta. De acordo com o texto, é correto afirmar que de fato, há
radioatividade no processo de geração de energia nuclear. Isso pode ser confirmado com o trecho
“…can be recycled via reprocessing or proposed breeder reactors, thereby neutralising the vast
majority of the radioactivity”.
A alternativa E está incorreta. De acordo com o texto, é correto afirmar que a energia nuclear é
uma forma de combater a crise climática. Isso pode ser confirmado com o trecho “At a time when
we need every tool in the box to fight the climate crisis – and nuclear is one of the most effective…”.
GABARITO: C

QUESTÕES EAM
QUESTÃO 01 (EAM/INÉDITA) – Read the dialogue and mark the right option to fill in the gaps
respectively.
A: Hello, Denver!
B: Hey, Thomas! How are you and your wife?
A: We are ok. What about you?
B: I’m doing well. Hard times, man!
A: _________ you get the vaccine at the hospital you work?
B: Yes, I ________. Thank God!
A: You _______ lucky. I have to go! See you!
B: See you, take care!

(A) Did / did / went


(B) Were / was / is
(C) Was / was / did
(D) Were / was / did
(E) Did / did / are
Comentários: Na primeira lacuna, precisamos usar uma palavra no passado. A pergunta não seria
“Você toma a vacina no hospital que você trabalha e a resposta “sim, eu tomo”. A opção possível
é “DID”, pois é a pergunta no passado “Você tomou a vacina...?”
Na segunda lacuna, já sabemos que precisamos responder - e dar uma resposta curta – Yes, I did,
para o passado – pergunta com DID, resposta com did.

AULA 09 – RELATIVE CLAUSES AND QUANTIFIERS 60


TEACHER ANDREA BELO

Na terceira lacuna, como já sabemos que o diálogo está se desenrolando entre amigos e, por
causa do amigo que trabalha ter tomado a vacina, o outro diz: ‘Você é sortudo” usando, então,
“are” – You are lucky!”
GABARITO: E

QUESTÃO 02 (EAM/INÉDITA) – Read the sentences and mark the correct option to fill in the
blanks respectively.
Doctor Sam is ______ uncle. ________ lives near my house. All my family and friends call him
when they feel sick.

(A) Her/ you


(B) Your / I
(C) Her / She
(D) My / He
(E) His / He
Comentários: Essa é uma questão de pronomes e que devemos prestar muita atenção, pois não
há uma única resposta para cada lacuna. Precisamos resolver a questão pensando no contexto
como um todo.
A primeira lacuna é preenchida adequadamente por “my”, pois percebe-se que a narrador(a) fala
de sua relação com seu tio Sam – “Dr. Sam é meu tio – my uncle”
A segunda lacuna é mais fácil, pois precisamos de um sujeito para a frase “Ele mora perto da
minha casa. - “He” é, portanto, a única opção aplicável aqui, já que a frase é completa com: Toda
minha família e amigos ligam para ele quando se sentem doentes.
Temos a sequência “my / He”
GABARITO: D

QUESTÃO 03 (EAM/INÉDITA) – Which grammarly correct form should be used in this album
cover to avoid abbreviation and get into formal language?

AULA 09 – RELATIVE CLAUSES AND QUANTIFIERS 61


TEACHER ANDREA BELO

Which grammarly correct form should be used in this album cover to avoid abbreviation and get
into formal language?
(A) Going down slowly
(B) Goin’ down slowly
(C) Goin dow slowly
(D) Goin’ down slow
(E) Going down slow
Comentários: As alternativas A, B, C e D estão incorretas. A pergunta é qual frase seria
gramaticalmente correta evitando abreviações. A palavra abreviada é o verbo go, no gerúndio
(going = goin’) e portanto, temos as alternativas A e E inicialmente. Mas, na letra A, há o acréscimo
de -ly no adjetivo slow, sem necessidade, já que slow é lento e tem sentido completo, sem precisar
usar advérbio, é qualidade e não modo. A alternativa que nos atende é a letra E.
GABARITO: E

QUESTÃO 04 (EAM/INÉDITA) – Use the verbs in the parentheses to complete the following
statements.
I – Catia and her sister sometimes ______ (do) their homework together after lunch, but now they
______ (do) another thing to make it different.
II – Carol_______ (eat) pizza very Fridays. Sometimes, she _______ (want) to eat vegetables but
she doesn’t like it.
III – Anne _________(live) in Africa. During the week, her daughters _______ (work) as tourist
guides tehre and they make a good money.

Now mark the option which completes them respectively.


(A) Are doing / am doing / is eating / wants / lives / working
(B) Am doing / do / eats / is wanting / is living / works
(C) Am doing / do / eat / is wanting / is living / works
(D) Do / do / is eating / want / live / is working
(E) Do / are doing / eats / wants / lives / work
Comentários: Na sentença I, a primeira lacuna pede que “do” seja utilizado no presente. Fato
confirmado pelo advérbio “sometimes” que indica frequência. Portanto, devemos preencher essa
lacuna com “do”. Já a palavra “now” pede que a segunda lacuna seja preenchida no gerúndio, já
que dá a ideia de que está acontecendo naquele momento. A segunda lacuna deve ser preenchida
com “are doing”.
Na sentença II, as duas lacunas possuem mais de uma opção que se encaixaria corretamente. Mas
é importante observar que, ao acertar as duas primeiras, a questão já estaria resolvida por conta
das opções apresentadas nas alternativas. Mas, de qualquer forma, a melhor opção para

AULA 09 – RELATIVE CLAUSES AND QUANTIFIERS 62


TEACHER ANDREA BELO

preencher as lacunas era colocar ambos os verbos no presente simples, porque, nesse contexto,
dão a ideia de hábito, frequência. Portanto, “eats” e “wants” seriam o melhor encaixe para esses
verbos.
Na sentença III, seguindo o mesmo raciocínio da sentença anterior, a melhor opção para ambos
os verbos era colocá-los no presente simples. A melhor forma de encaixá-los seria “lives” e
“work”, respectivamente.
Temos a sequência “do / am doing / eats / wants / lives / work”
GABARITO: E

QUESTÃO 05 (EAM/INÉDITA) – Look at the picture below.

What is he doing in the picture?


(A) The man plays the guitar every day.
(B) He is playing the guitar now.
(C) The man didn’t play the guitar yesterday.
(D) He never plays the guitar.
(E) The man won’t play the guitar.
Comentários: A alternativa A está incorreta. A figura apenas um homem tocando violão. Não é
possivel concluir que ele toca violão todos os dias, como sugere a alternativa.
A alternativa B está correta. É possível inferir da figura que há um homem tocando violão nesse
momento.
A alternativa C está incorreta. É impossível afirmar que um homem tocou violão ontem com base
na figura.
A alternativa D está incorreta. A própria figura mostra um homem tocando violão, o que contraria
a afirmação da alternativa.

AULA 09 – RELATIVE CLAUSES AND QUANTIFIERS 63


TEACHER ANDREA BELO

A alternativa E está incorreta. Não é possível inferir da figura que um homem não tocará violão,
principalmente porque ele está tocando.
GABARITO: B

QUESTÃO 06 (EAM/INÉDITA)

Which missing word is in Stephen King’s quote?


(A) O verbo GO, após os verbos modais can e should.
(B) A palavra BUT, de contraste, no lugar de AND.
(C) O intensificador VERY, antes do adjetivo BRAVE.
(D) O verbo start no futuro, com uso do WILL antes dele.
(E) A preposição TO após o adjetico BRAVE, par a frase fazer sentido.
Comentários: A alternativa A está incorreta. O verbo go não é necessário na frase.
A alternativa B está incorreta. A frase com and está certa, é uma sentença motivadora, com
acréscimo de informações e não contraste.
A alternativa C está incorreta. Não precisamos de “very” na frase para ter sentido.
A alternativa D está incorreta. O verbo start no futuro ficaria errado na frase.
A alternativa E está correta. A frase é “Você pode, você deveria e se você é corajodo o suficiente
para começar, você vai” e precisa da preposição TO (para) antes do verbo começar (to start).
GABARITO: E

QUESTÃO 07 (EAM/INÉDITA) – Which correctly form should be used for:


They arrived too early, _________ they?
(A) does
(B) didn’t
(C) aren’t
(D) are
(E) don’t

AULA 09 – RELATIVE CLAUSES AND QUANTIFIERS 64


TEACHER ANDREA BELO

Comentários: As alternativas A, C, D e E estão incorretas. O erro está no question tag, que, quando
o verbo está no passado, deve-se negar com didn’t, auxiliar do passado. As outras usam presente
simples.
GABARITO: B

QUESTÃO 08 (EAM/INÉDITA) – Use the verbs in the parentheses to complete the following
statements.
I – I usually ______ (do) exercises by myself, but now I ______ (do) with my sister.
II – Richard_______ (complain) all the time. When angry, he _______ (want) to argue with
everybody around him.
III – Barbara _________(live) in Panamá. During the afternoon, her young brother and sister
_______ (go) to the beach while she is working.

Now mark the option which completes them respectively.


(A) Am doing / am doing / is complaining / wants / lives / going
(B) Am doing / do / complains / is wanting / is living / goes
(C) Am doing / do /complain / is wanting / is living / goes
(D) Do / do / is complaining / want / live / is going
(E) Do / am doing / complains / wants / lives / go
Comentários: Na sentença I, a primeira lacuna pede que “do” seja utilizado no presente. Fato
confirmado pelo advérbio “usually” que indica frequência. Portanto, devemos preencher essa
lacuna com “do”. Já a palavra “now” pede que a segunda lacuna seja preenchida no gerúndio, já
que dá a ideia de que está acontecendo naquele momento.
A segunda lacuna deve ser preenchida com “am doing”.
Na sentença II, as duas lacunas possuem mais de uma opção que se encaixaria corretamente. Mas
é importante observar que, ao acertar as duas primeiras, a questão já estaria resolvida por conta
das opções apresentadas nas alternativas. Mas, de qualquer forma, a melhor opção para
preencher as lacunas era colocar ambos os verbos no presente simples, porque, nesse contexto,
dão a ideia de hábito, frequência. Portanto, “complains” e “wants” seriam o melhor encaixe para
esses verbos.
Na sentença III, seguindo o mesmo raciocínio da sentença anterior, a melhor opção para ambos
os verbos era colocá-los no presente simples. A melhor forma de encaixá-los seria “lives” (3ª
pessoa) e “go” (plural – brother and sister), respectivamente.
Temos a sequência “do / am doing / complains / wants / lives / go”.
GABARITO: E

AULA 09 – RELATIVE CLAUSES AND QUANTIFIERS 65


TEACHER ANDREA BELO

QUESTÃO 09 (EAM/INÉDITA) – Read the sentences and mark the correct option to fill in the
blanks respectively.
Sephora isn’t _____ best friend. _____ is only a classmate but _____ is already part of my life.

(A) Her/ you/you


(B) Your / I /you
(C) Her / she/ she
(D) My / she /she
(E) His / he /she
Comentários: Essa é uma questão de pronomes e que devemos prestar muita atenção, pois não
há uma única resposta para cada lacuna. Precisamos resolver a questão pensando no contexto
como um todo.
A primeira lacuna é preenchida adequadamente por “my”, pois percebe-se que a narrador(a) fala
de sua relação com Sephora – Ela não é minha melhor amiga – é apenas uma colega de sala...
A segunda lacuna é mais dedutiva, pois precisamos de um sujeito para a frase “Ela é apenas minha
colega de sala”. - “She” é, portanto, a única opção aplicável aqui. Até agora, temos a sequência
“my / she”
O último espaço é mas “Ela não é minha melhor amiga – é apenas uma colega de sala... ela já é
parte da minha vida – conclusão: a terceira coluna é o pronome SHE.
GABARITO: D

QUESTÃO 10 (EAM/INÉDITA) – Read the sentences and mark the correct complement: Roberto
Carlos _____ a great singer.
(A) isn’t
(B) wasn’t
(C) were
(D) are
(E) is
Comentários: Essa é uma questão de pronomes e que se deve prestar muita atenção, pois há uma
única opção: se o Roberto Carlos é considerado um ótimo cantor (great singer) para a maioria das
pessoas, a resposta é IS. As alternativas que não se encaixam na frase que analisamos.
GABARITO: E

AULA 09 – RELATIVE CLAUSES AND QUANTIFIERS 66


TEACHER ANDREA BELO

QUESTÕES EEAR
Read the cartoon and answer questions 01, 02 and 03.

Adapted from https://www.gocomics.com/garfield/2021/03/21.

QUESTÃO 01 (EEAR/INÉDITA) – According to the cartoon, we can say that _______________.


(A) Odie’s daddy has just got home.
(B) Odie loves when “Daddy’s home” is on.
(C) “Daddy’s home” is a TV show.
(D) the man and his pets are excitedly watching TV.
(E) only Odie is happy when Daddy finally gets home.
Comentários: Vemos no cartum que toda vez que é anunciado na TV um programa chamado
“Daddy’s home” (algo como “O papai está em casa / O papai chegou”), o cachorro acha que é o
“pai” dele dizendo que chegou e corre para a porta para esperá-lo, sem se dar conta de que o seu
“pai” está sentado no sofá com ele. Assim, De acordo com o cartum, podemos dizer que
_______________.
a) O pai de Odie acabou de chegar em casa.
b) O Odie ama quando está passando “Daddy’s home”.
c) “Daddy’s home” é um programa de TV.
d) o homem e seus animais de estimação estão assistindo à TV com entusiasmo.
e) só Odie fica feliz quando o papai finalmente chega em casa.
GABARITO: C

AULA 09 – RELATIVE CLAUSES AND QUANTIFIERS 67


TEACHER ANDREA BELO

QUESTÃO 02 (EEAR/INÉDITA) – “Now it’s time for ‘Daddy’s home’!”.


The reported form of the sentence is: it’s been announced on TV that
(A) then it was time for “Daddy’s home”.
(B) now it is time for “Daddy’s home”.
(C) now it was time for “Daddy’s home”.
(D) then it is time for “Daddy’s home”.
(E) then is it time for “Daddy’s home”.
Comentários: “Agora é hora de ‘Daddy’s home’!”. A forma relatada da frase (discurso indireto /
reported speech) é: foi anunciado na TV que naquele momento era hora de “Daddy’s home”.
now → then ou at that moment
is → was
It’s been announced on TV that then it was time for “Daddy’s home”.
GABARITO: A

QUESTÃO 03 (EEAR/INÉDITA) – In “I’m right here!”, the underlined word


(A) works as the opposite of left.
(B) can’t be removed without changing the sentence’s meaning.
(C) shows that the man is at the correct place.
(D) emphasizes “here”.
(E) is incorrectly positioned in the sentence.
Comentários: Em “I’m right here!”, a palavra sublinhada é enfática, e reforça “here” (aqui) = Eu
estou bem aqui, exatamente aqui! Assim, Em “Estou bem aqui!”, a palavra sublinhada (right)
a) funciona como o oposto de esquerda (left).
b) não pode ser removida sem alterar o significado da frase.
c) mostra que o homem está no lugar correto.
d) enfatiza “aqui”.
e) está posicionada incorretamente na frase.
GABARITO: D

Read the text and answer questions 04 to 10.


The Titanic
The Titanic was built by the White Star Line. The owners of the company thought that if ocean
liners were big and luxurious enough more people ________ travel with them. The Titanic was
designed to be the largest in a series of three ships made by the White Star line. It was 268 meters

AULA 09 – RELATIVE CLAUSES AND QUANTIFIERS 68


TEACHER ANDREA BELO

long, 28 meters wide, and weighed 45 000 tons. It produced enough power to travel at a speed
of 24 knots (about 40km per hour).
The bulk of the ship was divided into compartments. They were separated by steel doors that did
not let any water through. The ship could still move and float if 3 or 4 of the 16 compartments
were filled with water.
The Titanic was more like a floating hotel than a ship. It cost $7.5 million, and it was unlike any
other ship that had ever been built. Palm trees and other expensive plants decorated the luxurious
hallways and corridors. The ship could carry 2 600 passengers and a crew of 900.
On April 10, 1912 over 2200 passengers boarded the Titanic on its maiden voyage to New York.
Many of them were immigrants who saved all their money for the journey. First class passengers
had to pay between $2 500 and $4 500 for a private room and a bath, third class passengers had
to share rooms and paid $35 each. Although the ship’s owners said the Titanic was unsinkable,
many problems before the first voyage were overlooked.
Safety regulations at that time were not very strict. The ship only had 16 lifeboats, enough for
about 1 500 passengers. It was only tested for a few hours and never went at full speed. The
telegraph system on board was new and not many people knew how to operate it.
During the night of April 14, 1912, the waters of the North Atlantic had a temperature of about -
2° C. At noon on that day the radio operators got messages from other ships about icebergs that
were nearby. The Titanic’s captain, Edward Smith, did not care about these warnings. He was
captain of a steel giant that could not sink. The only thing he cared about was setting up a new
world speed record. The Titanic was to be the fastest ship that ever sailed from Southampton to
New York.
The night was clear, and the Titanic sped on. When a big iceberg was sighted, the first officer shut
down all the engines. But it would have taken the ship about half a mile to come to a full stop.
Even though, on the surface, the ship stayed clear of the iceberg, it ripped a big hole in the hull.
At once the compartments began to flood with cold, icy water.
The bulkheads were lowered but it was too late. Water flooded at least five compartments. The
collision with the iceberg was so slight that the passengers hardly heard it. Most of them didn’t
take any notice and continued dancing and having fun. Some passengers were asleep in their
cabins.
The bow of the ship dipped under the water’s surface and the back part of the ship began to rise.
After a short time, the Titanic broke into two pieces. When Captain Smith realized that the Titanic
was sinking, he had a distress signal sent out, but the nearest ship was a hundred kilometers away.
As time went on, chaos emerged, and passengers rushed to the boat deck. Women and children
were allowed on the lifeboats first. Lights flickered and electricity was finally gone. At 2:20 a.m.
the Titanic disappeared into the Atlantic Ocean.
The Carpathia, which was the nearest ship, came to the scene about two hours later and picked
up the freezing passengers in their lifeboats. By early morning, the news of the disaster had gone
around the world. The world’s largest ocean liner, the Titanic, had sunk on its maiden voyage,
killing 1513 people.
Adapted from https://www.english-online.at/history/titanic/titanic.htm.

AULA 09 – RELATIVE CLAUSES AND QUANTIFIERS 69


TEACHER ANDREA BELO

QUESTÃO 04 (EEAR/INÉDITA) – According to the text, the Titanic was _______________.


(A) bought by a company named the White Star Line.
(B) the biggest of a series of three ships.
(C) 268 metres wide.
(D) 28 metres long.
(E) the second largest ship of the White Star Line.
Comentários: De acordo com o texto, o Titanic era / foi / tinha _______________.
a) comprado por uma empresa chamada White Star Line.
b) o maior de uma série de três navios.
c) 268 metros de largura.
d) 28 metros de comprimento.
e) o segundo maior navio da White Star Line.
The Titanic was built by the White Star Line… The Titanic was designed to be the largest in a series
of three ships made by the White Star line. It was 268 metres long, 28 metres wide, and weighed
45 000 tons. = O Titanic foi construído pela White Star Line (construído, não comprado, letra A
errada) ... O Titanic foi projetado para ser o maior de uma série de três navios feitos pela linha
White Star. (letra B correta, letra E incorreta). Tinha 268 metros de comprimento, 28 metros de
largura (letras C e D inverteram as informações) e pesava 45.000 toneladas.
GABARITO: B

QUESTÃO 05 (EEAR/INÉDITA) – In “The telegraph system on board was new and not many
people knew how to operate it.”, the underlined pronoun refers to:
(A) on board.
(B) many people.
(C) Titanic.
(D) full speed.
(E) the system.
Comentários: Em “O sistema telegráfico a bordo era novo e poucas pessoas sabiam como operá-
lo (operar o sistema mencionado).”, o pronome sublinhado refere-se a:
a) a bordo.
b) muitas pessoas.
c) Titanic.
d) velocidade total.
e) o sistema.
GABARITO: E

AULA 09 – RELATIVE CLAUSES AND QUANTIFIERS 70


TEACHER ANDREA BELO

QUESTÃO 06 (EEAR/INÉDITA) – Choose the appropriate word to fill in the blank in the text.
(A) will.
(B) were going.
(C) can.
(D) are.
(E) would.
Comentários: The owners of the company thought that if ocean liners were big and luxurious
enough more people ________ travel with them. = Os proprietários da empresa achavam que, se
os transatlânticos fossem grandes e luxuosos o suficiente, mais pessoas viajariam (would travel)
com eles.
Temos no caso o second ou unreal conditional (observe que na if clause o verbo está no passado
– se os transatlânticos fossem grandes e luxuosos o suficiente...). Assim, a lacuna deve ser
preenchida com verbo que indique hipótese, consequência hipotética caso a condição fosse
cumprida (viajariam = would travel).
GABARITO: E

QUESTÃO 07 (EEAR/INÉDITA) – In “The ship could still move and float if 3 or 4 of


the 16 compartments were filled with water.”, the underlined modal verb indicates
(A) prohibition.
(B) permission.
(C) advice.
(D) ability.
(E) obligation.
Comentários: Em “O navio ainda poderia se mover e flutuar (no sentido de que estaria apto a
fazê-lo) se 3 ou 4 dos 16 compartimentos estivessem inundados com água.”, o verbo modal
sublinhado indica
a) proibição.
b) permissão.
c) conselho.
d) habilidade.
e) obrigação.
GABARITO: D

AULA 09 – RELATIVE CLAUSES AND QUANTIFIERS 71


TEACHER ANDREA BELO

QUESTÃO 08 (EEAR/INÉDITA) – In “On April 10, 1912 over 2200 passengers boarded the Titanic
on its maiden voyage to New York.”, the underlined word is closest in meaning to
______________.
(A) marvelous.
(B) ordinary.
(C) initial.
(D) incredible.
(E) famous.
Comentários: Em “Em 10 de abril de 1912, mais de 2.200 passageiros embarcaram no Titanic em
sua viagem inaugural para Nova York.”, A palavra sublinhada tem o significado mais próximo de
______________.
a) maravilhoso.
b) comum.
c) inicial.
d) incrível.
e) famoso.
GABARITO: C

Read the extract below.


“The collision with the iceberg was so slight that the passengers hardly heard it. Most of them
didn’t take any notice and continued dancing and having fun. Some passengers were asleep in
their cabins.”

QUESTÃO 09 (EEAR/INÉDITA) – We can infer from the extract that ___________.


(A) the passengers barely heard the collision when it happened.
(B) the passengers knew right away what was going on.
(C) the collision with the iceberg was strong and loud.
(D) all passengers were having fun when Titanic struck an iceberg.
(E) the collision frightened the passengers as soon as it happened.
Comentários: Leia o extrato abaixo.
“A colisão com o iceberg foi tão leve que os passageiros quase não ouviram. A maioria deles não
percebeu e continuou dançando e se divertindo. Alguns passageiros estavam dormindo em suas
cabines.”
Podemos inferir do trecho que ___________.
a) os passageiros mal ouviram a colisão quando ela aconteceu.

AULA 09 – RELATIVE CLAUSES AND QUANTIFIERS 72


TEACHER ANDREA BELO

b) os passageiros souberam imediatamente o que estava acontecendo. (a maioria não percebeu).


c) a colisão com o iceberg foi forte e alta. (foi imperceptível para a maioria).
d) todos os passageiros estavam se divertindo quando o Titanic atingiu um iceberg (alguns
estavam dormindo).
e) a colisão assustou os passageiros assim que aconteceu. (foi imperceptível para a maioria).
GABARITO: A

QUESTÃO 10 (EEAR/INÉDITA) – In “As time went on, chaos emerged, and passengers rushed to
the boat deck.”, the ___________ is being used.
(A) Simple Present.
(B) Simple Past.
(C) Past Participle.
(D) Imperative.
(E) Present Perfect.
Comentários: Em “À medida que o tempo passou (went on), surgiu (emerged) o caos e os
passageiros correram (rushed) para o convés do barco.”, o Simple Past está sendo usado.
a) Presente Simples.
b) Passado Simples.
c) Particípio Passado.
d) Imperativo.
e) Presente Perfeito.
GABARITO: B

QUESTÕES EFOMM
Read the text and answer questions 01, 02 and 03.
Oxygen shortages are killing thousands. Why aren’t we doing more about this?
As with vaccines and drugs, the pandemic has shown how access to this life-saving resource is deeply unequal
The basics ___ caring for acutely unwell patients are simple: air needs to go in and out and blood
needs to go round and round. Across the world, the pandemic has consistently shown how poorly
equipped healthcare systems are for addressing these needs. Much attention has been paid to
vaccines, drug therapies and ventilators ___ recent months, while relatively little has been said
about the most basic human requirement of all – oxygen.
Oxygen is all around us, and yet there are acute shortages of it in many healthcare settings. This
is because the infrastructure needed to supply oxygen to patients, such as large vacuum-insulated
evaporators (which are like giant, very cold vacuum flasks), is relatively expensive and needs
regular maintenance and top-ups of liquid oxygen. Where this isn’t available, hospitals might use

AULA 09 – RELATIVE CLAUSES AND QUANTIFIERS 73


TEACHER ANDREA BELO

concentrators ___ extract oxygen from the surrounding environment. These require electricity
and compressed air – which, again, are scarce in many places. Or they might use cylinders, which
can store and deliver short-term oxygen therapy. But they need regularly refilling, which depends
on secure supply lines.
I am a doctor who specializes in respiratory and intensive care medicine. Thankfully, I have never
been in a situation where I needed to provide a patient with oxygen therapy when none was
available. But this is exactly the situation facing healthcare workers in many other countries.
During the pandemic, systems for delivering oxygen came under pressure almost everywhere. In
the UK, hospitals that were decades old struggled to cope with surges of patients needing the
type of high oxygen flows used in modern therapy. In some low- and middle-income countries,
the infrastructure to deliver oxygen to patients was nonexistent.
(Adapted from https://www.theguardian.com/commentisfree/2021/jun/08/oxygen-shortages-killing-vaccines-drugs)

QUESTÃO 01 (EFOMM/INÉDITA) – According to the text, which option is correct?


(A) The pandemic clarified the fact that basic needs are often not well met by health systems
(B) Oxygen has received as much attention as vaccines at this time of pandemic
(C) Oxygen scarcity is minimal as it is an element that we have in abundance around us
(D) The pandemic did not overload oxygen delivery systems
(E) The supply of oxygen occurred in all countries in the world, even if it was scarcely
Comentários: A alternativa A está correta. De acordo com o texto, é correto afirmar que a
pandemia esclareceu o fato de que as necessidades básicas muitas vezes não são bem atendidas
pelos sistemas de saúde, assim como esta opção indica. Isso pode ser confirmado com o trecho
“...the pandemic has consistently shown how poorly equipped healthcare systems are for
addressing these needs”.
A alternativa B está incorreta. De acordo com o texto, não é correto afirmar que o oxigênio tem
recebido tanta atenção quanto as vacinas neste momento de pandemia, mas sim, que pouca
atenção é dada ao oxigênio, ao contrário do que é feito com as vacinas. Isso pode ser confirmado
com o trecho “Much attention has been paid to vaccines … while relatively little has been said
about the most basic human requirement of all – oxygen”.
A alternativa C está incorreta. De acordo com o texto, não é correto afirmar que a escassez de
oxigênio é mínima, pois é um elemento que temos em abundância ao nosso redor, mas sim, que
mesmo com a abundância, existem muitos lugares com escassez do oxigênio. Isso pode ser
confirmado com o trecho “Oxygen is all around us, and yet there are acute shortages of it in many
healthcare settings”.
A alternativa D está incorreta. De acordo com o texto, não é correto afirmar que a pandemia não
sobrecarregou os sistemas de fornecimento de oxigênio, mas sim, que sobrecarregou. Isso pode
ser confirmado com o trecho “During the pandemic, systems for delivering oxygen came under
pressure almost everywhere”.
A alternativa E está incorreta. De acordo com o texto, não é correto afirmar que o fornecimento
de oxigênio ocorreu em todos os países do mundo, mesmo que dificilmente, mas sim, que houve

AULA 09 – RELATIVE CLAUSES AND QUANTIFIERS 74


TEACHER ANDREA BELO

países que essa infraestrutura não existia. Isso pode ser confirmado com o trecho “In some low-
and middle-income countries, the infrastructure to deliver oxygen to patients was nonexistent”.
GABARITO: A

QUESTÃO 02 (EFOMM/INÉDITA) – Which is the correct option to complete the gaps in the text?
(A) At / on / that
(B) At / in / that
(C) Of / in / that
(D) Of / on / that
(E) Of / in / with
Comentários: A primeira lacuna deve ser preenchida com “of”, pois a frase se refere aos princípios
básicos de cuidar de pacientes, ou seja, “Os princípios básicos de cuidar de pacientes agudamente
indispostos são simples”.
A segunda lacuna deve ser preenchida com “in”, pois a frase se refere aos meses, ou seja, “Muita
atenção tem sido dada a vacinas, terapias medicamentosas e ventiladores nos últimos meses...”.
A terceira lacuna deve ser preenchida com “that”, pois a frase se refere ao que os concentradores
fazem, ou seja, “Onde isso não estiver disponível, os hospitais podem usar concentradores que
extraem oxigênio do ambiente circundante”.
GABARITO: C
QUESTÃO 03 (EFOMM/INÉDITA) – What’s the meaning of the word “struggled” in paragraph 3?
(A) Forgot
(B) Battled
(C) Tried
(D) Competed
(E) Contested
Comentários: A alternativa A está incorreta. A palavra “struggled” significa lutaram e não pode
ser comparada com a palavra “forgot”, que significa esqueceram.
A alternativa B está correta. A palavra “struggled” significa lutaram e pode ser comparada com a
palavra “battled”, que significa batalharam.
A alternativa C está incorreta. A palavra “struggled” significa lutaram e não pode ser comparada
com a palavra “tried”, que significa tentaram.
A alternativa D está incorreta. A palavra “struggled” significa lutaram e não pode ser comparada
com a palavra “competed”, que significa competiram.
A alternativa E está incorreta. A palavra “struggled” significa lutaram e não pode ser comparada
com a palavra “contested”, que significa contestaram.
GABARITO: B

AULA 09 – RELATIVE CLAUSES AND QUANTIFIERS 75


TEACHER ANDREA BELO

Read the text and answer questions 04, 05 and 06.


If we loosen restrictions too early, there is a real risk of a third wave in the UK
Not enough people are vaccinated against Covid as a new variant spreads, requiring us to
remain vigilant for a little longer
Just when it felt like we could begin to relax again, Covid-19 has thrown us yet another curveball.
While we’ve been debating in the UK where to go for a holiday and booking long-overdue nights
out with friends, the virus has been causing havoc across the world.
The Kent variant, B.1.1.7, found its way into countries such as Taiwan and Singapore; P.1 is causing
another wave in Brazil; and most devastatingly, the highly transmissible variant B.1.617 (which
has three subtypes) is causing a public health crisis in India. The country has seen cases surging
with estimated deaths to be several times greater than that being reported by the government,
which continues to downplay the epidemic.
The UK has done well, largely through strong public compliance, in bringing its case numbers
down, but Boris Johnson has repeated his mistake from last summer and left the borders open.
The Sunday Times estimates that at least 20,000 passengers from India were allowed to enter the
UK while Johnson delayed imposing a travel ban because he didn’t want to upset the Indian prime
minister, Narendra Modi, while negotiating a trade deal. This is at a time when other countries
such as New Zealand and Hong Kong completely stopped all flights.
This puts us now in a precarious situation. The World Health Organization has identified B.1.617
as a “variant of concern” because it is at least as transmissible as the Kent variant, potentially even
more so. This means that the virus can jump faster among those susceptible and accelerate out
of control. The UK has not fully vaccinated enough of the population to avoid a third wave among
younger people that could still strain the NHS.
(Adapted from https://www.theguardian.com/commentisfree/2021/may/17/third-wave-risk-uk-vaccinated-covid-variant)

QUESTÃO 04 (EFOMM/INÉDITA) – It is possible to infer from the text that


(A) Brazil and India are the countries facing the best phase of the pandemic, with a decrease in
cases and deaths
(B) The UK failed to reduce the number of cases
(C) The UK is in a very comfortable situation regarding the pandemic
(D) The situation in the UK would be better if tourists had not been allowed in
(E) It is impossible for a third wave to happen in the UK because of the advance in vaccination
Comentários: A alternativa A está incorreta. De acordo com o texto, não é possível inferir que
Brasil e Índia são os países que enfrentam a melhor fase da pandemia, com diminuição de casos
e mortes, mas sim, que são países onda e crises na saúde pública por conta da pandemia. Isso
pode ser confirmado com o trecho “...P.1 is causing another wave in Brazil; and most
devastatingly, the highly transmissible variant B.1.617 (which has three subtypes) is causing a
public health crisis in India”.

AULA 09 – RELATIVE CLAUSES AND QUANTIFIERS 76


TEACHER ANDREA BELO

A alternativa B está incorreta. De acordo com o texto, não é possível inferir que o Reino Unido
não conseguiu reduzir o número de casos, mas sim, que o país tem feito bem. Isso pode ser
confirmado com o trecho “The UK has done well, largely through strong public compliance, in
bringing its case numbers down…”.
A alternativa C está incorreta. De acordo com o texto, não é possível inferir que o Reino Unido
está em uma situação muito confortável em relação à pandemia, mas sim, que não está por conta
da abertura de fronteiras sem que toda a população estivesse vacinada. Isso pode ser confirmado
com o trecho “...but Boris Johnson has repeated his mistake from last summer and left the borders
open … This puts us now in a precarious situation”.
A alternativa D está correta. De acordo com o texto, é possível inferir que a situação no Reino
Unido seria melhor se os turistas não tivessem sido permitidos, assim como esta opção indica.
Isso pode ser confirmado com o trecho “The Sunday Times estimates that at least 20,000
passengers from India were allowed to enter the UK … This puts us now in a precarious situation”.
A alternativa E está incorreta. De acordo com o texto, não é possível inferir que é impossível
ocorrer uma terceira onda no Reino Unido devido ao avanço da vacinação, mas sim, que ainda
não foi toda a população que foi vacinada, tornando possível uma terceira onda entre os mais
jovens. Isso pode ser confirmado com o trecho “The UK has not fully vaccinated enough of the
population to avoid a third wave among younger people that could still strain the NHS”.
GABARITO: D

QUESTÃO 05 (EFOMM/INÉDITA) – In the excerpt “...the virus has been causing havoc across the
world” (paragraph 1), the word in bold means
(A) Waste
(B) Miracle
(C) Calm
(D) Misunderstanding
(E) Chaos
Comentários: A alternativa A está incorreta. A palavra “havoc” significa “destruição/caos” e não
pode ser comparada com a palavra “waste”, que significa “desperdício”.
A alternativa B está incorreta. A palavra “havoc” significa “destruição/caos” e não pode ser
comparada com a palavra “miracle”, que significa “milagre”.
A alternativa C está incorreta. A palavra “havoc” significa “destruição/caos” e não pode ser
comparada com a palavra “calm”, que significa “calma”.
A alternativa D está incorreta. A palavra “havoc” significa “destruição/caos” e não pode ser
comparada com a palavra “misunderstanding”, que significa “mal-entendido”.
A alternativa E está correta. A palavra “havoc” significa “destruição/caos” e pode ser comparada
com a palavra “chaos”, que significa “caos”.
GABARITO: E

AULA 09 – RELATIVE CLAUSES AND QUANTIFIERS 77


TEACHER ANDREA BELO

QUESTÃO 06 (EFOMM/INÉDITA) – the correct alternative to complete the paragraph below


“Covid-19 has killed millions around the world, but ___ some who are lucky enough to survive the
infection, ___ nightmare is not over: adding insult to injury are deadly fungal infections that follow
in the wake of the virus. Making matters worse, inequities ___ long predated the pandemic have
left some countries without the capacity to combat these serious infections”.
(Adapted from https://www.theguardian.com/commentisfree/2021/jun/02/black-fungus-covid-india-mucormycosis)

(A) To / the / that


(B) For / the / that
(C) For / the / in
(D) To / the / in
(E) For / a / that
Comentários: A primeira lacuna deve ser preenchida com “for”, pois se refere ao fato de que o
pesadelo da covid não acabou para alguns que sobreviveram, ou seja, “...mas para alguns que
têm a sorte de sobreviver à infecção, o pesadelo ainda não acabou...”.
A segunda lacuna deve ser preenchida com “the”, pois a frase se refere a um pesadelo específico,
o pesadelo da Covid-19, ou seja, “Covid-19 já matou milhões em todo o mundo, mas para alguns
que têm a sorte de sobreviver à infecção, o pesadelo ainda não acabou...”.
A terceira lacuna deve ser preenchida com “that”, pois a frase se refere as desigualdades que
antecederam a pandemia, ou seja, “Para piorar as coisas, as desigualdades que antecederam a
pandemia...”.
GABARITO: B

Read the text and answer questions 07, 08, 09 and 10.
After a year at home, children with disabilities deserve priority vaccination
They disappeared more than a year ago and many are still out of sight. When the pandemic hit,
53,000 under-18s in England with disabilities that made them vulnerable to coronavirus began to
shield away at home.
Kept off school long after their classmates went back, and away from friends, they have found
their childhoods put on hold. And while the vaccine rollout gave high-risk adults some reprieve in
the new year, those aged under 16, who have not been eligible for any vaccine, are still living in
limbo.
Yet we have barely heard a thing about disabled children’s plight through the coronavirus crisis –
a silence that has not been helped by the faux-reassuring narrative that “no healthy child” has
died from the virus. Now that the UK regulator has at last approved the Pfizer vaccine for 12- to
15-year-olds, this blind spot is becoming even more glaring.
(Adapted from https://www.theguardian.com/commentisfree/2021/jun/10/children-disabilities-priority-vaccination-england-education-shielding)

AULA 09 – RELATIVE CLAUSES AND QUANTIFIERS 78


TEACHER ANDREA BELO

QUESTÃO 07 (EFOMM/INÉDITA) – In the phrase “...53,000 under-18s in England with disabilities


that made them vulnerable to coronavirus began to shield away at home” (paragraph 1), the
underlined word is a synonym for
(A) Defenseless
(B) Safe
(C) Unsusceptible
(D) Closed
(E) Open
Comentários: A alternativa A está correta. A palavra “vulnerable” significa vulnerável e é sinônimo
de “defenseless”, que significa indefeso.
A alternativa B está incorreta. A palavra “vulnerable” significa vulnerável e não é sinônimo de
“safe”, que significa seguro.
A alternativa C está incorreta. A palavra “vulnerable” significa vulnerável e não é sinônimo de
“unsusceptible”, que significa insuscetível.
A alternativa D está incorreta. A palavra “vulnerable” significa vulnerável e não é sinônimo de
“closed”, que significa fechado.
A alternativa E está incorreta. A palavra “vulnerable” significa vulnerável e não é sinônimo de
“open”, que significa aberto.
GABARITO: A

QUESTÃO 08 (EFOMM/INÉDITA) – The second paragraph of the text states that


(A) Children with disabilities followed the same safety rules in the pandemic as children without
disabilities
(B) The childhood of children with disabilities was not affected by the pandemic
(C) There are still people who cannot be vaccinated
(D) Vaccination did not help high-risk adults to calm down on the holiday season
(E) No children have returned to face-to-face classes yet
Comentários: A alternativa A está incorreta. De acordo com o segundo parágrafo, não é correto
afirmar que crianças com deficiência seguiram as mesmas regras de segurança na pandemia que
crianças sem deficiência, mas sim, que as regras não foram as mesmas. Isso pode ser confirmado
com o trecho “Kept off school long after their classmates went back…”.
A alternativa B está incorreta. De acordo com o segundo parágrafo, não é correto afirmar que a
infância das crianças com deficiência não foi afetada pela pandemia, mas sim, que foi deixada de
lado. Isso pode ser confirmado com o trecho “Kept off school long after their classmates went
back, and away from friends, they have found their childhoods put on hold”.
A alternativa C está correta. De acordo com o segundo parágrafo, é correto afirmar que ainda há
pessoas que não podem ser vacinadas, assim como esta opção indica. Isso pode ser confirmado

AULA 09 – RELATIVE CLAUSES AND QUANTIFIERS 79


TEACHER ANDREA BELO

com o trecho “...those aged under 16, who have not been eligible for any vaccine, are still living
in limbo”.
A alternativa D está incorreta. De acordo com o segundo parágrafo, não é correto afirmar que a
vacinação não ajudou adultos de alto risco a se acalmarem na época de festas, mas sim, que
ajudou. Isso pode ser confirmado com o trecho “And while the vaccine rollout gave high-risk
adults some reprieve in the new year…”.
A alternativa E está incorreta. De acordo com o segundo parágrafo, não é correto afirmar que
nenhuma criança voltou às aulas presenciais ainda, mas sim, que já voltaram. Isso pode ser
confirmado com o trecho “Kept off school long after their classmates went back…”.
GABARITO: C

QUESTÃO 09 (EFOMM/INÉDITA) – In the phrase “And while the vaccine rollout gave high-risk
adults some reprieve in the new year…” (paragraph 2), the underlined word is a synonym for
(A) Continuation
(B) Clemency
(C) Immunity
(D) Order
(E) Recover
Comentários: A alternativa A está incorreta. A palavra “reprieve” significa um alívio de
responsabilidade e não é sinônimo de “continuation”, que significa continuação.
A alternativa B está correta. A palavra “reprieve” significa um alívio de responsabilidade e é
sinônimo de “clemency”, que significa clemência.
A alternativa C está incorreta. A palavra “reprieve” significa um alívio de responsabilidade e não
é sinônimo de “immunity”, que significa imunidade.
A alternativa D está incorreta. A palavra “reprieve” significa um alívio de responsabilidade e não
é sinônimo de “order”, que significa ordem.
A alternativa E está incorreta. A palavra “reprieve” significa um alívio de responsabilidade e não
é sinônimo de “recover”, que significa recuperar.
GABARITO: B

QUESTÃO 10 (EFOMM/INÉDITA) – The text


(A) Develops the idea that children without disabilities have many privileges
(B) Focuses on the fact that high-risk adults should not have been given priority in vaccination
(C) Advocate for the vaccination of all children as soon as possible
(D) Aims to explain the importance of social interaction for children and their development
(E) Explains the experience of children with disabilities during the pandemic and defends their
priority in vaccination

AULA 09 – RELATIVE CLAUSES AND QUANTIFIERS 80


TEACHER ANDREA BELO

Comentários: A alternativa A está incorreta. Não é correto afirmar que o texto desenvolve a ideia
de que crianças sem deficiência têm muitos privilégios, mas sim, que ele explica a experiência de
crianças com deficiência durante a pandemia e defende sua prioridade na vacinação. Isso pode
ser confirmado com os trechos “Kept off school long after their classmates went back, and away
from friends, they have found their childhoods put on hold…” e “children with disabilities deserve
priority vaccination”.
A alternativa B está incorreta. Não é correto afirmar que o texto enfoca o fato de que adultos de
alto risco não deveriam ter recebido prioridade na vacinação, mas sim, que ele explica a
experiência de crianças com deficiência durante a pandemia e defende sua prioridade na
vacinação. Isso pode ser confirmado com os trechos “Kept off school long after their classmates
went back, and away from friends, they have found their childhoods put on hold…” e “children
with disabilities deserve priority vaccination”.
A alternativa C está incorreta. Não é correto afirmar que o texto advoga pela vacinação de todas
as crianças o mais rápido possível, mas sim, que ele explica a experiência de crianças com
deficiência durante a pandemia e defende sua prioridade na vacinação. Isso pode ser confirmado
com os trechos “Kept off school long after their classmates went back, and away from friends,
they have found their childhoods put on hold…” e “children with disabilities deserve priority
vaccination”.
A alternativa D está incorreta. Não é correto afirmar que o texto visa explicar a importância da
interação social para as crianças e seu desenvolvimento, mas sim, que ele explica a experiência
de crianças com deficiência durante a pandemia e defende sua prioridade na vacinação. Isso pode
ser confirmado com os trechos “Kept off school long after their classmates went back, and away
from friends, they have found their childhoods put on hold…” e “children with disabilities deserve
priority vaccination”.
A alternativa E está correta. É correto afirmar que o texto explica a experiência de crianças com
deficiência durante a pandemia e defende sua prioridade na vacinação, assim como esta opção
indica. Isso pode ser confirmado com os trechos “Kept off school long after their classmates went
back, and away from friends, they have found their childhoods put on hold…” e “children with
disabilities deserve priority vaccination”.
GABARITO: E

QUESTÕES EPCAR
Directions: Read the text below and answer questions according to the text.
TEXT
Rare purple textiles from the time of biblical kings found for the first time in Israel
By Harry Baker – Staff Writer
The color was a favorite of the biblical kings David and Solomon
A patch of 3,000-year-old wool dyed true purple, recently discovered in Timna Valley in Israel.
Archaeologists have uncovered rare fragments of 3,000-year-old textiles stained purple — a color
considered the height of royal fashion at the time — in southern Israel.

AULA 09 – RELATIVE CLAUSES AND QUANTIFIERS 81


TEACHER ANDREA BELO

The archaeologists discovered the purple textiles — which included bits of woven fabric, a tassel
and a bundle of wool fibers — at Slaves' Hill in Timna Valley, an ancient copper production district
in the Arava desert, the team wrote in a new study describing the findings.
Radiocarbon dating of the fabrics revealed that they were woven around 1000 B.C., placing them
in the time of the biblical kings David and Solomon — who ruled from 1010–970 B.C. and 970–
931 B.C., respectively — in Jerusalem. The Bible mentions the kings and other important figures
wearing the color during this time, according to the researchers.
Related: 10 fascinating biblical-era discoveries from 2018
The dye used to stain the fabrics was made from mollusks found hundreds of miles away in the
Mediterranean and was extremely valuable as a result. However, until now, no physical evidence
of its use had ever been recovered in Israel or the rest of the southern Levant (an area
encompassing the eastern Mediterranean).
"For the first time, we have direct evidence of the dyed fabrics themselves, preserved for some
3,000 years," Naama Sukenik, curator of organic finds at the Israel Antiquities Authority and lead
author of the new paper describing the textiles, told Live Science in an email. "Each fragment gives
us new information, tells us a new story about the site and important information about the
people that lived there."
(Adapted from https://www.livescience.com/59693-could-earth-turn-into-venus. – Access on 07/28/20)

QUESTÃO 01 (EPCAR/INÉDITA) – The text states that


(A) there are registers of discoveries with evidence.
(B) There are purple registers.
(C) Most of discoveries were in Israel.
(D) it’s possible that Israel succeeds because of the discoveries.
(E) There are all kinds of discoveries.
Comentários: De acordo com o texto, podemos afirmar que são registros de descobertas com
evidências, assim como diz a alternativa A.
A alternativa B está incorreta, pois afirma que foram encontrados “registros roxos”. Mas, no texto
temos a informação de que foram encontradas roupas roxas, entre outras.
A alternativa C afirma que a maioria das descobertas foram feitas em Israel, com o termo “most
of”, porém o texto se restringe às descobertas que nele se encontram e não a maioria das
descobertas em geral, como parece ser.
Na alternativa D, temos a afirmação de que Israel pode ter sucesso por causa das descobertas,
mas não temos essa informação no texto.
Na alternativa E, afirma-se todos os tipos de descoberta mas é uma resposta muito geral.
GABARITO: A

AULA 09 – RELATIVE CLAUSES AND QUANTIFIERS 82


TEACHER ANDREA BELO

QUESTÃO 02 (EPCAR/INÉDITA) – “Archaeologists have uncovered rare fragments of 3,000-year-


old textiles stained purple” (paragraph 1). The underlined word is a synonym for
(A) grubby
(B) blemish
(C) grimy
(D) smeared
(E) dirty
Comentários: As alternativas A (grubby), C (grimy) e D (smeared) são sinônimos do adjetivo sujo
(dirty), nossa letra E, que está incorreta e não tem similaridade com mancha/manchado.
A alternativa correta é a letra B, pois blemish é sinônimo de stained, que também pode significar
“mancha/manchado”.
GABARITO: B

QUESTÃO 03 (EPCAR/INÉDITA) – Mark the alternative in which the highlighted word is used
with the same meaning as in the sentence below.
“…a color considered the height of royal fashion at the time – in southern Israel” (paragraph 1).

(A) I needed to study a lot for a long time.


(B) We had to understand that all the times you got late.
(C) He made me confess two times.
(D) She was cooking when you called.
(E) I arrived in time.
Comentários: O enunciado pede para marcar a alternativa que tem o mesmo sentido da
expressão destacada at the time, que significa “naquele tempo” ou “naquela hora”.
A alternativa A afirma algo sobre long time (longo tempo), mas at the time se refere a um
momento específico.
As alternativas B e C, usam o termo “times”, para se referir a “vezes”, visto que a palavra times,
no plural, significa vezes (I repeted four times = Eu repeti quatro vezes). Portanto, alternativas
incorretas.
Na letra E, “in time” seria “na hora” e não é o sentido que queremos.
O nosso gabarito é a alternativa D, pois o termo when significa “quando”, podendo substituir a
expressão at the time sem prejuízo ao significado.
GABARITO: D

AULA 09 – RELATIVE CLAUSES AND QUANTIFIERS 83


TEACHER ANDREA BELO

QUESTÃO 04 (EPCAR/INÉDITA) – Mark the alternative that is grammatically correct about the
text.
(A) The archaeologists found purple textiles
(B) The archaeologists has found purple textiles
(C) The archaeologists haven’t found purple textiles
(D) The archaeologists has been found purple textiles
(E) The archaeologists will find purple textiles
Comentários: O enunciado pede para marcar a alternativa que está gramaticalmente correta
sobre o assunto do texto.
A alternativa A é o nosso gabarito. O verbo to find, no passado é found e está sendo usado
corretamente para o sujeito “archaeologists”.
As alternativas B, D e E estão incorretas. Has é um verbo auxiliar usado para o singular. Como se
refere aos arqueólogos (plural), deveria ter sido usado o verbo auxiliar have. E na E está futuro
mas o texto contou no passado.
A alternativa C está incorreta, pois afirma que os arqueólogos não encontraram roupas roxas, mas
de acordo com o texto, vimos que encontraram de fato.
GABARITO: A

QUESTÃO 05 (EPCAR/INÉDITA) – Mark the statement that is in DISAGREEMENT with the text.
(A) bits of woven fabric, a tassel and a bundle of wool fibers were included.
(B) bits of woven fabric, a tassel and a bundle of wool fibers were not included
(C) fabric, a tassel and a bundle of wool fibers was included.
(D) fabric, a tassel and a bundle of wool fibers were included.
(E) fabrics were not included.
Comentários: O enunciado pede para marcar a alternativa que está em desacordo com o texto.
De acordo com o texto, foram incluídos pedaços de tecido, uma borla e um feixe de fibras de lã,
assim como afirmam as alternativas A, C, D e E, mesmo que resumidas.
Por outro lado, a alternativa B, nosso gabarito, utiliza a palavra not, dizendo que não foram
incluídos, mas vimos que foram incluídos de fato.
GABARITO: B

QUESTÃO 06 (EPCAR/INÉDITA) – Mark the alternative that the question is grammatically


INCORRECT.
“… the team wrote in a new study describing the findings.” (paragraph 2)

AULA 09 – RELATIVE CLAUSES AND QUANTIFIERS 84


TEACHER ANDREA BELO

(A) Did the team write in a new study describing the findings?
(B) Has the team written in a new study describing the findings?
(C) Did the team wrote in a new study describing the findings?
(D) Has the team written something describing the findings?
(E) Did the team writing in a new study describing the findings?
Comentários: A única alternativa incorreta gramaticalmente é a letra C, o nosso gabarito. Para
fazer perguntas no passado simples, o verbo principal volta para o infinitivo, ou seja, sua forma
original, pois o verbo auxiliar Did já indica passado.
As demais alternativas estão gramaticalmente corretas. A letra A está com o verbo principal em
sua forma infinitiva e no gerúndio e as letras B e D utilizam a estrutura do present Perfect,
seguindo as regras impostas pela língua.
GABARITO: C

QUESTÃO 07 (EPCAR/INÉDITA) – Mark the statement that is NOT mentioned in the text.
(A) The fabrics they found were from kings David and Solomon’s period.
(B) The fabrics can be placed at kings David and Solomon’s period.
(C) The fabrics refused kings David and Solomon’s period.
(D) The fabrics are part of kings David and Solomon’s period.
(E) The purple fabric is part of kings David and Solomon’s period.
Comentários: O enunciado pede para marcar a alternativa que não é mencionada no texto.
A única opção possível é a alternativa C, onde afirma que os tecidos recusaram (refused) o período
dos reis Davi e Salomão e não vemos essa informação no texto.
As demais alternativas, A, B, D e E trazem afirmações que estão de acordo com o texto.
GABARITO: C

QUESTÃO 08 (EPCAR/INÉDITA) – According to the text,


(A) The fabrics dye was made from mollusks
(B) The fabrics dye was produced with all kinds of mollusks
(C) The fabrics dye was made from mollusks waste
(D) The fabrics dye was made from mollusks reinforcement
(E) The fabrics dye was made from mollusks rest
Comentários: De acordo com o texto, a alternativa correta é a letra A, em que afirma que os
tecidos foram feitos de moluscos.
As demais alternativas contêm informações desnecessárias e incorretas sobre a produção dos
tecidos, como all kinds of mollusks (todos os tipos de moluscos), mollusks waste (resíduos/restos
de moluscos), mollusks reinforcement (reforço de moluscos) e restos de moluscos.
GABARITO: A

AULA 09 – RELATIVE CLAUSES AND QUANTIFIERS 85


TEACHER ANDREA BELO

QUESTÃO 09 (EPCAR/INÉDITA) – “However, until now, no physical evidence of its use had ever
been recovered in Israel or the rest of the southern Levant” (paragraph 4). The underlined word
is a synonym for
(A) Otherwise
(B) On the other hand
(C) Even so
(D) By the way
(E) Then
Comentários: O enunciado pede o sinônimo do termo however, que significa,
“contudo/entretanto”.
A única alternativa possível é a letra C, pois a expressão even so é sinônimo de however e significa
“mesmo assim”, causando contraste.
As demais alternativas têm significados variados e que não substituem however: otherwise
(senão), on the other hand (por outro lado), by the way (a propósito) e then (então).
GABARITO: C

QUESTÃO 10 (EPCAR/INÉDITA) – “… Naama Sukenik, curator of organic finds at the Israel


Antiquities Authority…” The highlighted word can be substituted for ___.
(A) guardian
(B) burglar
(C) mugger
(D) poacher
(E) doctor
Comentários: O termo curator se refere a um curador, alguém encarregado de cuidar de algum
interesse, como se fosse um guardião.
Assim, pode ser substituído, sem prejuízo, por guardian, alternativa A.
As demais alternativas, B, C e D contêm sinônimos da palavra “ladrão”. E a letra E, doctor, é
médico, diferente de curator.
GABARITO: A

QUESTÕES ESA
Texto para responder à questão 01
RIO DE JANEIRO, BRAZIL - In the past 24 hours, Brazil recorded 1,113 deaths and 36,653 new cases
related to the novel coronavirus. The data are included in the Ministry of Health's daily balance
released on Tuesday evening, September 15th.

AULA 09 – RELATIVE CLAUSES AND QUANTIFIERS 86


TEACHER ANDREA BELO

Since the start of the pandemic, 133,119 people have died as a result of Covid-19. On Monday,
the Ministry of Health's data system recorded a total of 132,006 deaths. A further 2,445 are still
under investigation by health authorities.
(Adapted from https://riotimesonline.com - September 15th)

QUESTÃO 01 (ESA/INÉDITA) – According to the text, it is correct to say that:


(A) lot of researchers will introduce a new protective mask.
(B) one specific researcher is one of the responsible institution for the protective mask.
(C) Coope/UFRJ is the responsible main responsable for the protective mask.
(D) INMETRO has researched about the protective mask.
(E) researchers will introduce a new protective mask.
Comentários: A alternativa A está incorreta. O texto afirma que pesquisadores (researchers)
apresentarão uma nova máscara de proteção mas não afirma que são muitos ou a quantidade.
A alternativa B está incorreta, dizendo que um pesquisador específico é um dos responsáveis pela
máscara, como vimos acima, são pesquisadores de diferentes órgãos.
A alternativa C está incorreta, dizendo que o Coope/UFRJ é responsável principal pela máscara,
como vimos acima, são pesquisadores de diferentes órgãos.
A alternativa D está incorreta. O Inmetro não está pesquisando sobre as máscaras e sim pessoas
responsáveis (researchers).
A alternativa E está correta, dizendo que pesquisadores (researchers) apresentarão uma nova
máscara de proteção, pois não se afirma quantidade mas a palavra “some” é mais de um.
GABARITO: E

QUESTÃO 02 (ESA/INÉDITA) – Which sentence is grammatically correct?


(A) Yesterday, I finished my activities more earlier than the night before.
(B) Cleyton is more thin than his best friend.
(C) My towel is more big than my sister’s one.
(D) Going by train is expensiver than by bus because of the tickets.
(E) Robert’s shoes are bigger than mine.
Comentários: A alternativa A está incorreta. Não se utiliza “more” junto com “earlier”, pois
“earlier” já é uma estrutura de frase comparativa, usando -ier no final do adjetivo considerado
curto. Utiliza-se apenas “earlier”, pois essa palavra, apenas, já oferece o sentido comparativo que
a sentença busca. O uso do “more” é para adjetivos longos, como “more beautiful/more
intelligent etc.”.
A alternativa B está incorreta. O modo comparativo de “THIN” é “thinner”. Só se usa “more” antes
de adjetivo quando este adjetivo tiver mais de uma sílaba, ou seja, quando for um adjetivo
considerado longo.

AULA 09 – RELATIVE CLAUSES AND QUANTIFIERS 87


TEACHER ANDREA BELO

A alternativa C está incorreta. A mesma explicação da alternativa anterior se aplica no caso de


“fast” o correto seria “faster” ou apenas “fast”, mas não “more fast”.
A alternativa D está incorreta. A palavra “expensive” é uma palavra de, pelo menos, três sílabas.
Portanto, é correto dizer “more expensive”, segundo a regra de palavras curtas ou longas e não
podemos colocar -er no fim, formando “expensiver”, que não existe.
A alternativa E está correta. “bigger” é a forma correta de fazer o comparativo de “big”.
GABARITO: E

QUESTÃO 03 (ESA/INÉDITA) – Complete the sentence below using the appropriate words:
“Mr. Copolla _______ selling cars: He ______ stops different car sales and he makes a lot of money.”

(A) Likes/don’t
(B) Likes/doesn’t
(C) Like/do
(D) Like/don’t
(E) Like/does
Comentários: Essa é uma questão que exige conhecimento dos tempos verbais e suas
conjugações no Present Simple (presente simples) bem como auxiliares (do/does) e as formas
afirmativa e negativa.
O sujeito “Mr. Copolla” pode ser substituído pelo pronome “he” (ele). Sempre que se conjuga
verbo na terceira pessoa do singular (he/she/it), acrescenta-se “S” ao final do verbo no tempo
Present Simple, como é o caso da frase em destaque em nosso enunciado.
A segunda lacuna deve ser preenchida com o auxiliar “doesn’t”, pois o sujeito está na terceira
pessoa do singular e exige “doesn’t” (does not), segundo a regra dos verbos na terceira pessoa,
modo negativo, ao invés de “don’t”, para sujeitos I, you e todos os demais que se encontram no
plural.
Todas essas regras acima estão em nosso PDF na aula 02, explicados com detalhes.
Temos então a sequência: likes / doesn’t.
GABARITO: B

QUESTÃO 04 (ESA/INÉDITA) – “___ the new manager in this store? Where did you come from?”
Complete the space with the correct form of the verb and the pronoun.
(A) You is
(B) You are
(C) Are you
(D) Is you
(E) Am you

AULA 09 – RELATIVE CLAUSES AND QUANTIFIERS 88


TEACHER ANDREA BELO

Comentários: Essa é uma questão que exige conhecimento das conjugações do verbo to be no
Present Simple (presente simples) bem como as formas afirmativa e interrogativa. Estão em nossa
aula 02, em PDF e videoaulas do Estratégia Militares, da teacher Andrea Belo.
A alternativa A está incorreta. “You” não pode ser seguido de “is”, devido à sua conjugação,
segundo qualquer gramática e a forma do verbo to be no plural é “are” (You are). O verbo deve
estar antes do pronome “you”, pois trata-se de uma pergunta.
A alternativa B está incorreta. A ordem de pronome (you) e verbo (are) deveria estar invertida
por ser de uma pergunta, em que o verbo to be passa a ser o primeiro da estrutura da frase.
A alternativa C está correta. Numa frase interrogativa, deve-se inverter a ordem de pronome
sujeito (you) e verbo to be (are). Portanto, “Are you part of my life?” é gramaticalmente correto.
A alternativa D está incorreta. Para o sujeito “You” não se usa “is”, como vimos anteriormente e
sempre “are”.
A alternativa E está incorreta. “You” não é acompanhado de “am”, sempre de “are”.
GABARITO: C

Texto para a questão 05


August 08, 2020
RIO DE JANEIRO, BRAZIL - On Saturday, August 8th, Brazil reached the number of 100,477 deaths
from Covid-19 since the start of the novel coronavirus pandemic. According to the Ministry of
Health, 905 deaths were recorded over the last 24 hours.
Of the 3,012,412 cases of people infected by the novel coronavirus, 2,094,293 (69.5 percent) have
recovered.
A total of 49,970 new cases have been reported by Health Secretariats since Friday, August 7th.
The official also shows that the number of people under follow-up stands at 817,642 (21.1
percent).
(Adapted from https://riotimesonline.com)

QUESTÃO 05 (ESA/INÉDITA) – According to the text, it is correct to say that:


(A) more than a hundred thousand deaths is the number described.
(B) A hundred thousand deaths is the number showed.
(C) Less than a hundred thousand deaths is the number described.
(D) more than a hundred thousand people are working on the research.
(E) the correct number of deaths is a thousand.
Comentários: A alternativa A afirma que o número de mortes é mais de cem mil – correto
(100,477).
A letra B diz que que o número de mortes é de cem mil – vimos que é mais de cem mil.
A letra C diz que que o número de mortes é menos de cem mil – errado.
A letra D fala que mais de cem mil pessoas estão trabalhando na pesquisa – errado.
A letra E diz que que o número de mortes é mil – errado.
GABARITO: A

AULA 09 – RELATIVE CLAUSES AND QUANTIFIERS 89


TEACHER ANDREA BELO

QUESTÕES ESCOLA NAVAL


How to defeat AIDS, malaria and tuberculosis

At the turn of the millennium, it was clear a new approach was required in the war against three
of the biggest threats to human life and development. There was need for a trustworthy
international organization that could solicit donations from rich countries and wealthy
organizations and spend that money on combating those threats in collaboration with the
governments of afflicted poor countries, but with appropriate oversight to ensure effectiveness
and avoid theft. The result was the Global Fund to Fight aids, Tuberculosis and Malaria.
And it worked. Though it is impossible to say what would have happened without the Global Fund,
as it is now formally known, the fund’s officials claim to have saved 32m lives since it opened in
2002. (…)
The Economist – October 10th

QUESTÃO 01 (ESCOLA NAVAL/INÉDITA) – The text shows that


a) Some diseases should be taken as relevant because they are considered threats.
b) Some diseases are being researched in a clear way since 2002.
c) It is important to know how to defend yourself related to some diseases.
d) It is necessary to spend some money at the turn of the millenium.
e) The turn of the millenium required some effectiveness.
Comentários: A questão quer saber qual mensagem o texto mostra. Logo no título do texto, já é
possível termos uma ótima noção da principal mensagem do artigo: "How to defeat AIDS, malaria
and tuberculosis"- Como combater a AIDS, a malária e a tuberculose. Olhos de águia (ou seriam
olhos de coruja?!) no verbo DEFEAT, que significa combater, derrotar. Saindo do título e
escaneando com os olhos o texto, já percebemos as três das maiores ameaças são as doenças
mencionadas no título. Atenção à palavra "approach", que significa abordagem, uma forma de
lidar com uma situação. Mais adiante, percebemos que essa abordagem foi a criação de um fundo
global de combate às referidas doenças. Agora, vamos às alternativas:
a) Conforme a análise que fizemos do título do texto e da primeira frase, você já pode perceber
que esta é a alternativa correta. De fato, o texto mostra que algumas doenças devem ser tidas
como relevantes porque são consideradas ameaças. Alternativa CORRETA.

AULA 09 – RELATIVE CLAUSES AND QUANTIFIERS 90


TEACHER ANDREA BELO

b) O texto não mostra que algumas doenças estão sendo pesquisadas de maneira clara desde
2002, mas sim que desde que o Fundo Global de Combate às referidas doenças foi criado, em
2002, milhões de vidas foram salvas. Alternativa INCORRETA
c) Certamente, o texto não mostra que é importante saber se defender em relação a algumas
doenças, mas sim que era necessária uma nova abordagem no combate a algumas doenças e essa
nova abordagem foi justamente a criação de um fundo global específico com esse propósito.
Alternativa INCORRETA
d) O texto mostra que era necessária uma nova abordagem na virada do milênio na guerra contra
a Aids, a malária e a tuberculose. Essa nova abordagem foi a criação de um fundo global que fosse
eficaz nesse propósito, a partir das doações de organizações e países ricos, abastados. O texto não
mostra que era necessário pura e simplesmente gastar dinheiro na virada do milênio. Era
necessário adotar uma nova maneira de lidar com o combate às doenças, maneira esta que foi
justamente criar o referido fundo global e gastar o dinheiro arrecadado de forma eficaz e sem
desvios no combate às doenças. Alternativa INCORRETA
e) Vimos que, na verdade, a virada do milênio exigia uma nova abordagem no combate às três
doenças mencionadas. O que tinha que ter "effectiveness"- eficácia era a gestão do fundo e a
correta aplicação dos recursos arrecadados para o consequente combate eficaz das doenças.
Alternativa INCORRETA.
GABARITO: A

QUESTÃO 02 (ESCOLA NAVAL/INÉDITA) – Mark the incorrect definition of “Trustworthy


international organization”
a) An example of steadfast international organization.
b) A kind of devoted international organization.
c) A kind of resolute international organization.
d) A dutiful international organization.
e) An unreliable international organization.
Comentários: Muita atenção ao enunciado, pois a questão pede a alternativa INCORRETA. Você
tem que marcar a definição incorreta de "Trustworthy international inorganization". Keep in mind:
"trustworthy" significa "digno de confiança", ou seja, algo em que se pode confiar. Observe o
processo de formação da palavra: trust- confiar, e worthymerecedor, digno= digno de confiança.
Vamos observar as alternativas em busca da definição incorreta, ou seja, algo que não se aproxime
dessa ideia de uma organização internacional fidedigna, confiável. Se você fizer uma leitura
dinâmica, vai bater o olho na letra E e ver a palavra UNRELIABLE, que tem o prefixo de negação
UN. "Reliable" pode ser um termo com o qual você já está mais familiarizado e significa
CONFIÁVEL. Se não conhecia, no problem, vamos memorizar agora: RELIABLE- CONFIÁVEL e
UNRELIABLE- NÃO CONFIÁVEL. Temos aí a nossa alternativa a ser marcada, pois traz justamente a
ideia contrária de "trustworthy", que, como vimos, quer dizer confiável. Só para garantir, vamos
ver as demais alternativas.

AULA 09 – RELATIVE CLAUSES AND QUANTIFIERS 91


TEACHER ANDREA BELO

a) Temos aqui a palavra "steadfast", que significa firme, estável, inabalável. Assim, é compatível
com a definição de uma organização internacional confiável. Como estamos em busca da definição
incorreta, este não é o nosso gabarito. Alternativa INCORRETA
b) O adjetivo utilizado nesta alternativa, "devoted", quer dizer confiável, leal, verdadeiro. Este
sentido está em harmonia com a definição de "trustworthy", não sendo, portanto, o nosso
gabarito. Alternativa INCORRETA.
c) Aqui temos a palavra "resolute", que também quer dizer firme, inabalável, logo, confiável. É
compatível com a definição de "trustworthy", não sendo o nosso gabarito. Alternativa INCORRETA.
d) Seguindo o mesmo raciocínio, "dutiful" é um adjetivo que se refere a algo leal, confiável, que
obedece ao que deve ser feito. Sendo semelhante à noção de "trustworthy", não é a alternativa a
ser assinalada. Alternativa INCORRETA.
e) Como vimos, aqui temos o nosso gabarito, pois UNRELIABLE é justamente o contrário de
"trustworthy", significando NÃO CONFIÁVEL. Alternativa CORRETA.
GABARITO: E

QUESTÃO 03 (ESCOLA NAVAL/INÉDITA) – According to the text, the adjective “wealthy” means
a) rich
b) delightful
c) subtle
d) swell
e) winsome
Comentários: Esta questão você deveria conhecer o significado do adjetivo "wealthy". Você se
lembra que o texto fala a respeito da criação de um fundo global para auxiliar no combate à AIDS,
à malária e à tuberculose? Como você pode ver no texto, esse fundo foi criado a partir de doações
de RICH countries and WEALTHY organizations. Acompanhe o raciocínio: por uma questão de
paralelismo de ideias, para fazer sentido no contexto, a noção de WEALTHY deve se aproximar de
RICH. Não faria o menor sentido que as doações para compor o fundo global viessem de países
ricos e organizações não abastadas, ou qualquer outra ideia associada às associações que se
afastasse do sentido de ter dinheiro para doar para o fundo. Pensando assim, você poderia acertar
a questão e marcar a letra A, mesmo sem necessariamente saber de cor o adjetivo WEALTHY.
Apenas para termos certeza, vejamos as outras também:
a) RICH- Eis o nosso gabarito, pois WEALTHY significa rico, abastado, "cheio da grana". Alternativa
CORRETA.
b) DELIGHTFUL significa agradável, delicioso, e já vimos que não é esse o sentido de "wealthy".
Alternativa INCORRETA.
c) SUBTLE quer dizer sutil, não sendo, portanto, o sentido de "wealthy", que, como vimos, significa
"rico". Alternativa INCORRETA.
d) SWELL, como adjetivo, dá a ideia de algo excelente, muito bom, não sendo o nosso gabarito.
Dica extra: como verbo, significa "inchar" (uma parte do corpo). Alternativa INCORRETA.

AULA 09 – RELATIVE CLAUSES AND QUANTIFIERS 92


TEACHER ANDREA BELO

e) WINSOME é um adjetivo usado para descrever algo que encanta, cativa, não sendo, assim, a
resposta para a nossa questão. Alternativa INCORRETA.
GABARITO: A

QUESTÃO 04 (ESCOLA NAVAL/INÉDITA) – The text


a) points out several diseases and their heal.
b) shows some diseases and uphold them.
c) highlights the cruelest diseases and settle them.
d) states the right way to cure those diseases.
e) demonstrates some diseases names and their cure importance.
Comentários: A questão quer que você perceba a exata abordagem que o texto faz das doenças
nele referidas. Qual é esse "approach"? Vamos ver? Check it out:
a) De acordo com essa alternativa, o texto aponta várias doenças (SEVERAL diseases) e a sua cura,
tratamento (heal). Como vimos, o artigo menciona, na verdade, TRÊS doenças como sérias
ameaças à vida humana- a AIDS, a malária e a tuberculose, e não SEVERAL- e trata da importância
do combate às mesmas. Alternativa INCORRETA.
b) Atenção ao interessante verbo "uphold", que significa defender, apoiar, confirmar. É isso que o
texto faz em relação às doenças que menciona? Certamente que não. O texto cita as três doenças
que representam maior ameaça e fala sobre uma medida efetiva para auxiliar no combate às
mesmas. Alternativa INCORRETA.
c) De acordo com essa alternativa, o texto destaca, enfatiza (highlights) as doenças mais cruéis e
as resolve. Cuidado para não marcar letras como essa, que podem parecer certas numa primeira
análise. O texto não afirma que as três doenças são as mais cruéis, mas sim que estão dentre as
maiores ameaças à vida humana e ao desenvolvimento. Você não pode dar como correta uma
alternativa que vai além do que foi dito, deal? Alternativa INCORRETA.
d) A letra D diz que o texto declara a maneira correta de curar as referidas doenças. Essa alternativa
também claramente extrapola o que o texto diz, uma vez que ele não fala sobre a forma certa de
curar as doenças mencionadas, mas sim sobre a importância de uma abordagem efetiva que
auxilie no combate às mesmas. Alternativa INCORRETA.
e) De acordo com a letra E, o texto demonstra algumas doenças (SOME diseases names) e a
importância da cura delas. É exatamente esta a abordagem que o texto faz das doenças nele
referidas. Alternativa CORRETA.
GABARITO: E

AULA 09 – RELATIVE CLAUSES AND QUANTIFIERS 93


TEACHER ANDREA BELO

QUESTÃO 05 (ESCOLA NAVAL/INÉDITA) – The text conclusion seems to be


a) logical and straight.
b) nonsense and worn out.
c) controversial and useful.
d) excessive and repeated.
e) confused and nuclear.
Comentários: Você deve ter uma boa compreensão da conclusão do texto e perceber de que
forma tal conclusão se relaciona com o texto de uma forma geral. O texto, ao final, traz a seguinte
conclusão: " E deu certo. Apesar de ser impossível dizer o que teria acontecido sem o fundo
global,..., os representantes do fundo alegam que salvaram 32 milhões de vidas desde que ele foi
aberto em 2002." Vamos ver as alternativas:
a) De acordo com essa letra, a conclusão é lógica e direta (straight). Percebemos que sim, a
conclusão de que vimos acima é uma decorrência lógica e direta do tema central do artigo, que
ressalta a importância da criação do fundo global como medida de combate às doenças
mencionadas. Alternativa CORRETA.
b) Para a letra b, a conclusão do texto é sem sentido e desgastada. Conforme vimos, a conclusão,
pelo contrário, tem tudo a ver com a ideia central do texto e vai direto ao ponto, indicando que a
necessária criação do fundo global como medida de combate às doenças citadas já deu resultados
muito positivos desde a sua criação. Alternativa INCORRETA.
c) Para essa letra, a conclusão seria controversa e útil. Só no "controversa" já eliminaríamos a
alternativa, uma vez que a conclusão não foi controversa, mas, sim, em harmonia com todo o
contexto. Alternativa INCORRETA.
d) Para essa alternativa, a conclusão seria excessiva e repetida. E não, não percebemos isso. A
conclusão traz informações novas a respeito do que o texto trata é na conclusão que ficamos
sabendo dos resultados positivos da criação do fundo global no combate às doenças, inclusive
com números) logo, não é excessiva nem repetida. Alternativa INCORRETA.
e) A conclusão não é confusa e obscura. Pelo contrário! Ela é direta, objetiva, e deixa bastante
clara a eficácia do fundo global na guerra contra a AIDS, a malária e a tuberculose. Alternativa
CORRETA.
GABARITO: E

Texto para questões 06 a 08


Taal: The 'very small but dangerous volcano'
BBC News (Jan/2020)

AULA 09 – RELATIVE CLAUSES AND QUANTIFIERS 94


TEACHER ANDREA BELO

Over the past few days, it's begun spewing lava, triggering earthquakes, and emitting huge plumes
of ash that have spread across the island of Luzon and beyond.
Scientists fear a bigger "hazardous eruption" is imminent. Taal is tiny, as volcanoes go, but it has
been deadly before. And according to Renato Solidum, the head of the Philippines' Institute of
Volcanology and Seismology (Philvolcs), it is "very small but a dangerous volcano". "Taal volcano
is a baby volcano sitting within a much bigger caldera volcano," said Ben Kennedy, associate
professor of physical volcanology at the University of Canterbury in New Zealand. The entire
Volcano Island has been marked as a permanent danger zone by Phivolcs.

QUESTÃO 06 (ESCOLA NAVAL/INÉDITA) – According to the text, which option is correct about
Taal volcano
a) Taal is a wee and dangerous volcano.
b) Taal is a huge and menacing volcano.
c) Taal is a tiny and sheltered volcano.
d) Taal is a huge and hazardous volcano.
e) Taal is a big and treacherous volcano.
Comentários: A questão pede que você identifique a opção correta sobre o vulcão Taal.
a) De acordo com essa opção, o vulcão Taal é pequeno e perigoso. Olha só como no título
da notícia já percebemos as características mais marcantes do vulcão- Taal: The 'very small but
dangerous volcano'. Reparou as palavras-chave? Você deve focar nos adjectives presentes no
título: wee – pequenino (a) e dangerous - perigoso. Temos na letra A o nosso gabarito. Alternativa
CORRETA mas, vejamos as outras.
b) A alternativa B associa os adjetivos "huge" e "menacing" ao vulcão Taal. De cara, você já poderia
eliminar essa alternativa se lembrasse que HUGE quer dizer, very, very big, enorme, imenso. Então,
keep that in mind: HUGE= VERY, VERY BIG. Isso já bastaria para você "cortar" essa letra. Mas para
que você vá aumentando o seu vocabulário, anota aí: "menacing" significa alarmante, ameaçador.
Essa característica seria compatível com a descrição do vulcão, mas "huge" tornou a opção
incorreta. Alternativa INCORRETA.
c) De fato, conforme o texto, o vulcão é "tiny", ou seja, minúsculo, muito pequeno, para os padrões
de um vulcão. Observe o trecho: "Taal is tiny, as volcanoes go...". É minúsculo, para um vulcão.
Sobre ser "sheltered", isto é, protegido, abrigado, não é a ideia que o texto transmite, uma vez
que o vulcão começou a dar sinais de erupção. Alternativa INCORRETA.
d) Já vimos que o Taal não é HUGE, mas sim small, tiny, o que já nos permitiria eliminar essa
alternativa. "Hazardous" é uma outra maneira de dizer perigoso. O texto nos permite sim concluir
que o vulcão é perigoso, apesar de ter associado este adjetivo especificamente a uma possível
erupção iminente. Alternativa INCORRETA.
e) O adjetivo BIG já torna a alternativa incorreta, por passar a ideia oposta. Como vimos, o vulcão
não é grande, mas sim pequeno para os padrões de um vulcão. Apenas para enriquecer o seu
vocabulário, anota aí: "treacherous" significa traiçoeiro. Alternativa INCORRETA.
GABARITO: A

AULA 09 – RELATIVE CLAUSES AND QUANTIFIERS 95


TEACHER ANDREA BELO

QUESTÃO 07 (ESCOLA NAVAL/INÉDITA) – What is the meaning of the verb “triggering” in the
sentence “… it's begun spewing lava, triggering earthquakes.”
a) to raze
b) to defeat
c) to ruin
d) to tear down
e) to incite
Comentários: A questão 17 pede que você entenda o significado do verbo "triggering" no trecho
"it's begun spewing lava, triggering earthquakes." A ideia é de que o vulcão começou a expelir
lava, PROVOCANDO, desencadeando terremotos. "Trigger" como substantivo significa gatilho, e o
seu sentido como verbo está também relacionado a essa ideia: causar algo, servir de gatilho para
que aconteça alguma coisa. Vamos às alternativas, então:
a) "Raze" significa demolir, destruir, não sendo, portanto, esse o sentido de "triggering" no trecho,
que, como vimos, quer dizer desencadear, dar causa, provocar. Alternativa INCORRETA
b) "Defeat" quer dizer derrotar, não sendo também, portanto, o nosso gabarito. Alternativa
INCORRETA
c) "Ruin" significa arruinar, e não provocar, dar causa. #dicadateacher: temos o verbo RUIN
"dentro" do verbo em português arRUINar. Macete legal para você guardar! Alternativa
INCORRETA.
d) "Tear down" é um phrasal verb que significa demolir, destruir. Não tem o sentido de provocar,
dar causa, que é o que estamos buscando. Alternativa INCORRETA.
e) "Incite" é um verbo cognato e tem o sentido de incitar. E o que é incitar, my student? Justamente
o que estamos procurando! Incitar é dar causa, estando em harmonia com o sentido de
"triggering" no trecho em questão. Alternativa CORRETA.
GABARITO: E

QUESTÃO 08 (ESCOLA NAVAL/INÉDITA) – Mark the incorrect option according to the text
a) People are afraid about the eruption.
b) Taal is not big but it can cause big consequences.
c) Scientists fear that the eruption can come and destroy a lot.
d) One of the volcano consequences can be earthquakes.
e) Taal is an unsafe and risky volcano.
Comentários: A questão pede que você marque a alternativa INCORRETA de acordo com o texto.
a) Segundo o texto, não são as pessoas em geral que estão temendo a erupção do vulcão, mas os
cientistas- Scientists fear a bigger "hazardous eruption" is imminent. Assim, essa alternativa
parece extrapolar o que o texto diz, estando incorreta. Como a questão pede justamente a
incorreta, a letra A é o nosso gabarito. Alternativa CORRETA.

AULA 09 – RELATIVE CLAUSES AND QUANTIFIERS 96


TEACHER ANDREA BELO

b) De acordo com o texto, de fato, o vulcão Taal não é grande (para os padrões de um vulcão), mas
é perigoso, podendo causar grandes consequências- it is "very small but a dangerous volcano".
Alternativa de acordo com o texto, mas lembre-se de que buscamos a incorreta, logo, não é o
nosso gabarito. Alternativa INCORRETA.
c) Na notícia, temos a seguinte informação: Scientists fear a bigger "hazardous eruption" is
imminent. Assim, de fato os cientistas temem que o vulcão possa entrar em erupção e causar
muita destruição, pois seria uma erupção muito perigosa, arriscada. Alternativa em sintonia com
o texto, mas lembre-se de que buscamos a incorreta, logo, não é o nosso gabarito. Alternativa
INCORRETA.
d) Segundo o texto, realmente uma das consequências do vulcão podem ser terremotosit's begun
spewing lava, triggering earthquakes. Alternativa conforme o texto, mas buscamos a incorreta.
Alternativa INCORRETA.
e) Conforme o texto, de fato o vulcão Taal não é seguro e traz riscos- ele já começou a expelir lava,
provocando terremotos, é "pequeno" mas perigoso, toda a sua área foi marcada como zona de
perigo... Vemos essas ideias por todo o texto. Alternativa de acordo com a notícia, mas buscamos
a incorreta. Alternativa INCORRETA.
GABARITO: A

Texto para questões de 09 e 10


Massive and malodorous – world's biggest flower found

The largest single flower ever recorded was found recently in Sumatra, Indonesia, measuring a
reported 111cm (3.64ft) across. This was a specimen of Rafflesia tuan-mudae and beat the
previous largest flower record of 107cm for Rafflesia arnoldii, also in Sumatra.
Rafflesia is not only a giant flower, but it has no leaves, stems or proper roots. It cannot
photosynthesise and instead sucks the food and water out of a particular vine using long thin
filaments that look like fungal cells. It gorges itself on the vine for a few years before bursting out
into a flower bud, swells for several months before blooming into a flower that looks like a bright
red bucket with big thick lobes. It gives off a whiff of rotting meat that, together with its gigantic
size, helps attract pollinating flies. Rafflesia also steals some of the DNA from the vine it lives on,
using it for its own genetic code for reasons that are not clear.

AULA 09 – RELATIVE CLAUSES AND QUANTIFIERS 97


TEACHER ANDREA BELO

QUESTÃO 09 (ESCOLA NAVAL/INÉDITA) – According to the text


a) A flor raflesia é gigante mas a raiz e folhas são pequenas.
b) Não há fotossíntese por causa dos longos filamentos e fungos.
c) A flor incha por horas e demora para voltar a seu tamanho normal.
d) Ao desabrochar, se parece com um balde vermelho gigante brilhante.
e) Quando desabrocha, se parece com um buquê grande e vermelho cheio de flores.
Comentários: A questão 19 pede que você identifique o que está de acordo com o texto. Vamos
às alternativas:
a) Na verdade, segundo o texto, a flor referida é sim gigante, mas NÃO tem folhas nem raízes
próprias, como você pode observar no trecho- Rafflesia is not only a giant flower, but it has no
leaves, stems os proper roots= A raflesia não é apenas uma flor gigante, mas não tem folhas, caules
e raízes próprias. Alternativa INCORRETA.
b) Alternativa tricky, atenção! O texto menciona que a flor não faz fotossíntese, mas não
estabelece relação de causa e consequência entre este fato e os longos filamentos e fungos. Na
verdade, a flor não tem capacidade de fazer a fotossíntese e, em vez disso, ela suga a comida e a
água através de longos filamentos que se parecem com células de fungos. Alternativa INCORRETA.
c) Atenção ao verbo inchar- SWELL! Na verdade, segundo o texto, a flor incha por MESESMONTHS
(não horas) antes de desabrochar. Alternativa INCORRETA.
d) É exatamente o que captamos do seguinte trecho: "...before blooming into a flower that looks
like a bright red bucket..." = antes de desabrochar em uma flor que se parece com um balde
vermelho brilhante. BLOOM significa florescer, desabrochar e BUCKET significa balde. Keep that in
mind! Alternativa CORRETA.
e) Outra alternativa tricky, olha a pegadinha. Como vimos acima, bucket não significa buquê, mas
sim balde. Alternativa INCORRETA.
GABARITO: D

QUESTÃO 10 (ESCOLA NAVAL/INÉDITA) – In the text, the sentence “It gives off a whiff of rotting
meat that, together with its gigantic size, helps attract pollinating flies.…” the underlined term
means
a) amiss smelling
b) neat smelling
c) special smelling
d) balm smelling.
e) reserved smelling.
Comentários: A questão 20 pede o significado do termo sublinhado ROTTING. Keep that in mind:
rotting significa PODRE. Ou seja, a flor exala um cheiro ruim, de carne apodrecendo, se
deteriorando. Tínhamos outra pista no título, com a palavra "malodorous", que claramente nos
faz pensar em algo que tem um mau odor. Vejamos as alternativas:

AULA 09 – RELATIVE CLAUSES AND QUANTIFIERS 98


TEACHER ANDREA BELO

a) Amiss smelling, ou seja, cheiro ruim. Porque “amiss” é sinônimo de errado, ruim. Vimos que a
flor exala um cheiro podre, logo, essa letra está correta. Alternativa CORRETA.
b) Neat smelling, isto é, cheiro bom. A palavra “neat” é arrumado, correto, bom. Vimos que é
justamente o oposto, a flor exala um mau cheiro. Alternativa INCORRETA.
c) Special smelling quer dizer cheiro especial. Não é o que o texto diz, pois ele se refere ao cheiro
da flor como um cheiro de algo que está se decompondo. Alternativa INCORRETA.
d) Balm pode significar bálsamo ou se referir a um tipo de erva cítrica. Não é este o cheiro que a
flor exala. Alternativa INCORRETA.
e) A flor não exala um cheiro "reservado". O cheiro fétido e o seu tamanho gigante ajudam a atrair
moscas polinizadoras, como percebemos no trecho do texto trazido pela questão. "Flies" neste
contexto está sendo usado como o substantivo moscas, right? Alternativa INCORRETA.
GABARITO: A

QUESTÕES EsPCEx
Leia o texto a seguir e responda às questões 01, 02, 03, 04, 05 e 06.
India monsoon: Rescuers search for survivors after heavy rains
Rescuers are scouring devastated parts of western India for survivors after heavy rains caused deadly floods.
Tens of thousands of people have been moved out of affected areas, with record-breaking rainfall
reported along parts of the coast.
The states of Goa and Maharashtra have been badly affected, with many feared missing near the
financial hub Mumbai.
At least 136 people have died in Maharashtra, while in neighbouring Goa hundreds of homes have
been damaged.
Many factors contribute to flooding, but experts say climate change caused by global warming
makes extreme rainfall more likely.
Heavy rains and flooding have also hit western Europe and parts of China in recent weeks, while
North America has grappled with scorching heatwaves.
The monsoon season in India lasts from June to September each year.
Rescuers have struggled to reach affected residents. Landslides have blocked roads, including the
main highway between Mumbai and Goa, and officials have deployed helicopters, divers and naval
rescue teams.
Goa's Chief Minister Pramod Sawant said flooding there was the worst in decades, and had caused
"widespread damage".
In the state of Maharashtra, major rivers are at risk of bursting their banks. Some 90,000 people
have been evacuated so far in the state.
The village of Taliye, southeast of Mumbai, saw landslides flatten most of its homes. An official
told Reuters news agency at least 42 people __________ there.

AULA 09 – RELATIVE CLAUSES AND QUANTIFIERS 99


TEACHER ANDREA BELO

Maharashtra's chief minister, Udhav Thackeray, plans to visit Taliye on Saturday.


Weather experts say heavy rains will continue to lash the city over the next few days.
Indian Prime Minister Narendra Modi said he was "anguished" by the loss of lives.
"The situation in Maharashtra due to heavy rains is being closely monitored and assistance is being
provided to the affected," he tweeted.
https://www.bbc.com/news/world-asia-india-57952521

QUESTÃO 01 (EsPCEx/INÉDITA) – Choose the alternative containing the correct word to


complete the gap.
A) had died.
B) will have died.
C) have died.
D) has died.
E) have been died.
Comentários: A alternativa A está correta. “An official told Reuters news agency at least 42 people
had died there”. (Um funcionário disse à agência de notícias Reuters que pelo menos 42 pessoas
morreram lá). Observe que se trata de uma ação que ocorreu em um passado indeterminado e
portanto, poderia parecer que o tempo verbal adequado era o present perfect entretanto, como
trata-se de um discurso indireto o verbo deverá estar no past perfect e não no present perfect.
GABARITO: A

QUESTÃO 02 (EsPCEx/INÉDITA) – According to the text, choose the correct statement.


A) While in Goa hundreds of houses were destroyed, in Maharashtra 136 people are still missing.
B) According to specialists, one of the main factors causing drastic climate change is global
warming.
C) Despite record rains reported in parts of the coast of India, few families needed to be
removed from their homes.
D) Heavy rains hit not only India but also several other countries in the world such as the United
States and Canada.
E) India has suffered not only from record rains in parts of the coast but also from terrible heat
waves in its central region.
Comentários: A alternativa A está incorreta. De acordo com o texto, pelo menos 136 pessoas
morreram em Maharashtra, enquanto na vizinha Goa centenas de casas foram danificadas.
A alternativa B está correta. De acordo com o texto, muitos fatores contribuem para as
inundações, mas os especialistas dizem que as mudanças climáticas causadas pelo aquecimento
global tornam as chuvas extremas mais prováveis.

AULA 09 – RELATIVE CLAUSES AND QUANTIFIERS 100


TEACHER ANDREA BELO

A alternativa C está incorreta. De acordo com o texto, dezenas de milhares de pessoas foram
removidas das áreas afetadas, com chuvas recordes relatadas em partes da costa.
A alternativa D está incorreta. O texto não fala que chuvas fortes atingiram os Estados Unidos.
Segundo o texto, fortes chuvas e inundações também atingiram a Europa Ocidental e partes da
China nas últimas semanas, enquanto a América do Norte enfrentou ondas de calor escaldantes.
A alternativa E está incorreta. O texto não fala que a Índia tem sofrido com fortes ondas de calor.
GABARITO: B

QUESTÃO 03 (EsPCEx/INÉDITA) – In the sentence “[…] major rivers are at risk of bursting their
banks. Some 90,000 people have been […]”, the verb to burst is closest in meaning to
A) to wave.
B) to rock.
C) to crack.
D) to agree.
E) to mend.
Comentários: A alternativa A está incorreta. To wave = acenar / agitar.
A alternativa B está incorreta. To rock = balançar.
A alternativa C está correta. To crack = quebrar / rachar. “In the state of Maharashtra, major rivers
are at risk of bursting their banks” (No estado de Maharashtra, grandes rios correm o risco de
estourar / romper / quebrar suas margens).
A alternativa D está incorreta. Too agree = concordar
A alternativa E está incorreta. To mend = consertar.
GABARITO: C

QUESTÃO 04 (EsPCEx/INÉDITA) – "[...] in Maharashtra due to heavy rains is being closely


monitored [...]" (last paragraph). The expression due to can be correctly replaced without
changing the meaning by:
A) for instance.
B) at last.
C) rather than.
D) because of.
E) had better.
Comentários: A alternativa A está incorreta. For instance = por exemplo.
A alternativa B está incorreta. At last = finalmente.
A alternativa C está incorreta. Rather than = ao invés de.

AULA 09 – RELATIVE CLAUSES AND QUANTIFIERS 101


TEACHER ANDREA BELO

A alternativa D está correta. Because of = devido a. ““The situation in Maharashtra due to heavy
rains is being closely monitored and assistance is being provided to the affected," he tweeted.” (A
situação em Maharashtra devido às fortes chuvas está sendo monitorada de perto e assistência
está sendo fornecida aos afetados”, ele tuitou.)
A alternativa E está incorreta. Had better = should = deveria (conselho).
GABARITO: D

QUESTÃO 05 (EsPCEx/INÉDITA) – In the text “[...] southeast of Mumbai, saw landslides flatten
most of its homes.”, the word its refers to
A) southeast of Mumbai
B) flatten
C) Reuters news agency
D) the state of Maharashtra
E) The village of Taliye
Comentários: A alternativa E está correta. “The village of Taliye, southeast of Mumbai, saw
landslides flatten most of its homes.” (A vila de Taliye, a sudeste de Mumbai, viu deslizamentos de
terra destruir a maioria de suas casas.) “Its” é um pronome possessivo adjetivo e irá fazer
referência a “The village of Taliye”. As casas que foram destruídas são da vila de Taliye.
GABARITO: E

QUESTÃO 06 (EsPCEx/INÉDITA) – Choose the alternative that has the sentence “The situation in
Maharashtra due to heavy rains is being closely monitored and assistance is being provided to
the affected," he tweeted” (last paragraph) correctly changed into reported speech.
A) He tweeted that the situation in Maharashtra due to heavy rains had been closely monitored
and assistance had been provided to the affected.
B) He tweeted that the situation in Maharashtra due to heavy rains has been closely monitored
and assistance has been provided to the affected.
C) He tweeted that the situation in Maharashtra due to heavy rains will be closely monitored and
assistance will be provided to the affected.
D) He tweeted that the situation in Maharashtra due to heavy rains was being closely monitored
and assistance was being provided to the affected.
E) He tweeted that the situation in Maharashtra due to heavy rains is closely monitored and
assistance was provided to the affected.
Comentários: A alternativa D está correta. Para colocarmos uma frase do discurso direto para o
discurso indireto devemos colocar os verbos da oração do discurso direto para o passado. Ou seja,
como a oração está no present continuous (to be no presente (am - is - are) + ING ----> is being)
no discurso indireto ela irá ficar no past continuous (to be no passado (was - were) + ING ----> was
being). Portanto, a única oração em que os verbos estão no past continuous é a alternativa D. “He

AULA 09 – RELATIVE CLAUSES AND QUANTIFIERS 102


TEACHER ANDREA BELO

tweeted that the situation in Maharashtra due to heavy rains was being closely monitored and
assistance was being provided to the affected.” (Ele tweetou que a situação em Maharashtra
devido às fortes chuvas estava sendo monitorada de perto e que assistência estava sendo
fornecida aos afetados.).
GABARITO: D

Leia o texto a seguir e responda às questões 07, 08, 09 e 10.


Serial killer on death row Rodney Alcala dies of natural causes
A man sentenced to death in the US state of California for murdering a 12-year-old girl and four
other women ____________ of natural causes, officials say.
Rodney Alcala, 77, died at a hospital near California's Corcoran state prison in the early hours of
Saturday.
Alcala, who was known as the "Dating Game Killer" after taking part in a US TV show, was convicted
in 2010.
As well as the California killings, he had also pleaded guilty to the murders of two other women in
New York.
Alcala was first handed the death penalty in Orange County in 1980 for the kidnap and murder of
Robin Samsoe, 12, in Los Angeles the previous year.
However, his sentence was overturned by the California Supreme Court and he was granted a new
trial.
He later received the same penalty in the second trial, but this was again overturned in 2003.
In the years that followed, investigators discovered forensic evidence linking Alcala to the other
California murders.
At the 2010 trial, he was found guilty of killing Samsoe as well as four other women aged between
18 and 32 in the years between 1977 and 1979.
In September 1978, Alcala took part in The Dating Game, an American TV show in which a single
woman questioned three single men hidden from her view before selecting one based on their
answers.
Alcala was selected at the end of show, but the woman later said she decided not to date him after
a conversation with him backstage, describing him as "creepy".
In 2012, Alcala was extradited to New York after he was charged over two additional murders from
1971 and 1977. He pleaded guilty and was sentenced in New York to 25 years to life.
California state prison officials said in a statement on Saturday that Alcala was suspected of being
involved in a number of additional killings.
No further details were given about his death.
There are currently some 700 inmates on death row in California, but Governor Gavin Newsom
has ordered a moratorium on executions.
https://www.bbc.com/news/world-us-canada-57955961

AULA 09 – RELATIVE CLAUSES AND QUANTIFIERS 103


TEACHER ANDREA BELO

QUESTÃO 07 (EsPCEx/INÉDITA) – According to the text, choose the correct statement.


A) A man sentenced to death for murder in the US state of California is murdered in jail.
B) In addition to the crimes Rodney Alcala had been convicted of, he was suspected of a number
of additional crimes according to California state prison officials.
C) Rodney Alcala was wrongfully sentenced to death, which can be seen in the failure of the
California Supreme Court to have twice overturned his sentence.
D) Rodney Alcala was sentenced to life in prison by the state of New York and to the death
penalty by the state of California where he was executed.
E) Several times Rodney Alcala had his sentence overturned by the California Supreme Court due
to lack of forensic evidence in the investigations.
Comentários: A alternativa A está incorreta. De acordo com o texto, o homem condenado à morte
no estado americano da Califórnia por assassinato morreu de causas naturais.
A alternativa B está correta. De acordo com o texto, funcionários da prisão estadual da Califórnia
disseram em um comunicado no sábado que Alcala era suspeito de estar envolvido em uma série
de assassinatos adicionais.
A alternativa C está incorreta. O texto não fala que Rodney Alcala foi condenado injustamente à
sentença de morte.
A alternativa D está incorreta. Segundo o texto, ele morreu de causas naturais e não pela sentença
de morte.
A alternativa E está incorreta. O texto não fala que por diversas vezes, Rodney Alcala teve sua
sentença anulada pela suprema corte da Califórnia devido à falta de provas forenses nas
investigações.
GABARITO: B

QUESTÃO 08 (EsPCEx/INÉDITA) – Choose the alternative containing the correct word to


complete the gap.
A) has died.
B) have died.
C) had died.
D) had been died.
E) have been dying.
Comentários: A alternativa A está correta. Usamos o present perfect para expressar ações que
ocorrem em um passado indeterminado. Ou seja, uma ação que ocorreu no passado mas que não
é dito quando ocorreu, o fato é na ação em si. “A man sentenced to death in the US state of
California for murdering a 12-year-old girl and four other women has died of natural causes” (Um
homem condenado à morte no estado da Califórnia, EUA, por assassinar uma menina de 12 anos
e quatro outras mulheres morreu de causas naturais). Estrutura do present perfect: Have / Has +

AULA 09 – RELATIVE CLAUSES AND QUANTIFIERS 104


TEACHER ANDREA BELO

verbo no particípio. Como o sujeito é “A man” (terceira pessoa do singular) então usaremos o
verbo auxiliar “Has”. Portanto o verbo que completa corretamente a lacuna é “has died”.
GABARITO: A

QUESTÃO 09 (EsPCEx/INÉDITA) – In the sentence “As well as the California killings, he had also
pleaded guilty to the murders of two other women in New York”, the expression As well as is
closest in meaning to
A) At least.
B) Since.
C) Hardly.
D) Eventually.
E) Besides.
Comentários: A alternativa A está incorreta. At least = pelo menos.
A alternativa B está incorreta. Since = desde / já que.
A alternativa C está incorreta. Hardly = quase não.
A alternativa D está incorreta. Eventually = finalmente.
A alternativa E está correta. Besides = além de. “As well as the California killings, he had also
pleaded guilty to the murders of two other women in New York” (Além dos assassinatos na
Califórnia, ele também se confessou culpado dos assassinatos de duas outras mulheres em Nova
York”.
GABARITO: E

QUESTÃO 10 (EsPCEx/INÉDITA) – "However, his sentence was overturned by the California


Supreme Court and he was granted a[...]" (paragraph 6). The word “However” can be correctly
replaced without changing the meaning by:
A) Despite.
B) Whereas.
C) Yet.
D) Even though.
E) Thus.
Comentários: A alternativa A está incorreta. Despite = Apesar de. (concessiva)
A alternativa B está incorreta. Whereas = Enquanto (Usamos “whereas” para mostrar a diferença
entre duas coisas ou fatos
A alternativa C está correta. Yet = However = Entretanto / Contudo / Todavia.
A alternativa D está incorreta. Even though = Embora (concessiva)
A alternativa E está incorreta. Thus = Portanto (explicativa)
GABARITO: C

AULA 09 – RELATIVE CLAUSES AND QUANTIFIERS 105


TEACHER ANDREA BELO

CONSIDERAÇÕES FINAIS
Outra aula alcançada com sucesso – outro passo até a sua aprovação!
E, dia após dia, os tópicos aprendidos aumentam, seu conhecimento fica mais amplo, o
vocabulário que você conhece se estende e a tendência é melhorar e ser capaz de alcançar a
aprovação de fato.
Nota-se o avanço em seus estudos e, provavelmente, uma maior tranquilidade para
enfrentar os exercícios que surgem. E você vai se acostumando a equilibrar seus estudos de forma
sistematizada, estudando cada vez mais e com mais dedicação.
Outro detalhe importante para seu sucesso nos estudos é continuar fazendo listas de
palavras e verbos, participar das aulas complementares, fazer simulados e pedir ajuda quando
precisar.
Isso te ajudará nas questões futuras e torna você, como eu disse antes, um candidato mais bem
preparado e confiante para realizar uma excelente prova de vestibular.
É importante lembrar também do nosso Fórum de dúvidas, exclusivo do Estratégia
Militares. Será minha forma de responder você, esclarecer o que mais você precise saber para
que os conteúdos fiquem ainda mais claros em seus estudos, certo?
E, caso queira, acesse minhas redes sociais (Teacher Andrea Belo) para aprender mais
palavras e contar com dicas importantes, que colaboram diretamente com seus estudos dia após
dia.

AULA 09 – RELATIVE CLAUSES AND QUANTIFIERS 106


TEACHER ANDREA BELO

REFERÊNCIAS BIBLIOGRÁFICAS
BARRETO, Tania Pedroza; GARRIDO, Maria Line; SILVA, João Antenor de C., Inglês Instrumental.
Leitura e compreensão de textos. Salvador, Ba UFBA, 1995, p. 64.
BROWN. H. Douglas. Principles of Language Learning and Teaching. Prentice Hall International,
1988.
COMPEDELLI, Samira Yousseff. Português, Literatura, Produção de texto & Gramática – São Paulo:
Ed. Saraiva, 2002.
CORREIA, Clese Mary P. Reading Specific Purposes. Salvador/ Ba: UFBA, 1999.
COSTEIRA, Adriana Araújo de M. Reading Comprehension Skills. João Pessoa/PB: ETFP, 1998.
CRYSTAL David. Cambridge University Press 1997. The Cambridge Encyclopedia of Language.
Cambridge University Press 1997
FREEMAN. Diane Larsen. MURCIA. Marianne Celce. The Grammar Book, 1999.
DYE, Joan., FRANFORT, Nancy. Spectrum II, III A Communicative Course in English. USA, Prentice
Hall, 1994.
FAVERO, Maria de Lourdes Albuquerque (org.). Dicionário de educadores no Brasil: da colônia aos
dias atuais. Rio de Janeiro: UFRJ, MEC, INEP, 1999.
FRANKPORT, Nancy & Dye Hoab. Spectrum II, III Prentice Hall Regents Englewood Cliffs, New Jersy,
1994.
GADELHA, Isabel Maria B. Inglês Instrumental: Leitura, Conscientização e Prática. Teresina:
EDUFFI, 2000.
GUANDALINI, Eiter Otávio. Técnicas de Leitura em Inglês: ESP – English For Specific Purposes:
estágio 1. São Paulo: Texto novo, 2002.
GRELLET, Françoise. Developing Reading Skills. Cambridge University Press, 1995
HOLAENDER, Arnon & Sanders Sidney. A complete English Course. São Paulo. Ed. Moderna, 1995.
HUTCHINSON, Tom & WATERS, Alan. English for Specific Purposes. Cambridge: Cambridge
University Press, 1996
KRASHEN. Stephen D. Second Language Acquisition and Second Language Learning, Prentice-Hall
International, 1988.
LAENG, Mauro. Dicionário de pedagogia. Lisboa: Dom Quixote, 1973.
LEFFA, Vilson J. Metodologia do ensino de línguas. In: BOHN, H.; VANDRESEN, P. (org.). Tópicos de
linguística aplicada: o ensino de línguas estrangeiras. Florianópolis: Editora da UFSC, 1988. p. 211-
231.
LIBERATO, Wilson. Compact English Book Inglês Ensino Médio. São Paulo: FTD, Vol. Único, 1998
Mc ARTHUR. The Oxford Companion to the English Language. Oxford University Press 1992
Fromkin. Victoria. An Introduction to Language
MARQUES, Amadeu. Inglês Série Brasil. ed. Atica. São Paulo: 2004. Vol. Único.

AULA 09 – RELATIVE CLAUSES AND QUANTIFIERS 107


TEACHER ANDREA BELO

MURPHY, Raymond: Essencial Grammar in Use Oxford. New York Ed. Oxford University, 1997.
OLIVEIRA, Luciano Amaral. English For Tourism Students. Inglês para Estudantes de Turismo: São
Paulo, Rocca, 2001.
OLIVEIRA, Sara Rejane de F. Estratégias de leitura para Inglês Instrumental. Brasília: UNB, 1994.
QUINTANA, et alli. First Certificate. Master Class Oxford. New York, 2004: Ed. Oxford University.
PAULINO, Berenice F. et all. Leitura em textos em Inglês – Uma Abordagem Instrumental. Belo
Horizonte: Ed. Dos Autores, 1992.
PEREIRA, Edilberto Coelho. Inglês Instrumental. Teresina: ETFPI, 1998.
RODGES, Theodore. Jack C. Richards. Approaches and Methods in Language Teaching. Cambridge
University Press, 2001.
RODMAN Robert. Harcourt Brace 1993. English as a Global Language
STEWART, B., HAINES S. First Certificate, MasterClass. UK – Oxford 2004.
SILVA, João Antenor de C., GARRIDO, Maria Lina, BARRETO, Tânia Pedrosa. Inglês Instrumental:
Leitura e Compreensão de Textos. Salvador: Centro Editorial e Didático, UFBA. 1994
SOARES, Moacir Bretãs. Dicionário de legislação do ensino. 19.ed. Rio de Janeiro: FGV, 1981.
SOUZA, Adriana Srade F. Leitura em Língua Inglesa: Uma abordagem Instrumental. São Paulo:
Disal, 2005.
TUCK, Michael. Oxford Dictionary of Computing for Learners of English. Oxford: Oxford University
Press, 1996.
TOTIS, Verônica Pakrauskas. Língua Inglesa: leitura. São Paulo: Cortez, 1991.

Livros eletrônicos:
Dicionário Houaiss da Língua Portuguesa, Editora Objetiva, 2001.
MOURãO, Janaína Pereira. "Skimming x Scanning"; Brasil Escola. Disponível em
<https://brasilescola.uol.com.br/ingles/skimming-x-scanning.htm>. Acesso em 20 de março de
2019.
www.newsweek.com - Acesso em 18 de março de 2019.
http://www.galaor.com.br/tecnicas-de-leitura/ - Acesso em 19 de março de 2019.
Expressões Idiomáticas (continuação)" em Só Língua Inglesa. Virtuous Tecnologia da
Informação,2008-2019. Consultado em 03/04/2019 às 22:09. Disponível na Internet em
http://www.solinguainglesa.com.br/conteudo/Expressoes5.php

AULA 09 – RELATIVE CLAUSES AND QUANTIFIERS 108


TEACHER ANDREA BELO

TRADUÇÕES
'America Is On the Move Again.' Joe Biden Delivers His First Major Presidential Speech
President Joe Biden has attended dozens of presidential addresses to Congress since he first came
to Washington in 1973. But after nearly half a century of being an audience member, he finally
got the speaking slot Wednesday night to lay out his own vision for the country.
“America is ___ the move again,” he said during his roughly hour-long remarks. “Turning peril into
possibility. Crisis into opportunity. Setback into strength.”
Biden seized the opportunity to boast about his Administration’s accomplishments, highlighting
how Americans have received over 220 million shots of the COVID-19 vaccines in his first 100 days
in office, which he called “one of the greatest logistical achievements this country has ever seen.”
Like the rest of Biden’s presidential milestones, this one was altered because ___ the pandemic.
The audience in the House chamber, usually more than 1,000 people, including all members of
Congress and multiple Supreme Court justices, was limited to just 200. Attendees were seated
apart from each other to accommodate social distancing, with a maximum of three people in one
row.
His speech largely focused on economic priorities, and he outlined the American Families Plan, his
$1.8 trillion proposal ___ raise taxes on the wealthy to pay for affordable childcare, paid leave,
and access to education. If implemented, the plan would offer universal preschool for three- and
four-year-olds and two years of free community college and create a national paid leave program.
This proposal is the third prong of Biden’s $6 trillion plan to bolster the post-pandemic economy
by combating inequality and climate change. He signed the first part, the $1.9 trillion American
Rescue Plan, into law on March 11. In the past month, has also called for a $2 trillion bill that
would create millions of jobs and repair the country’s infrastructure, hailing it in his address to
Congress as a “blue-collar blueprint to build America.”
In total, these proposals would be the biggest investment in clean energy in U.S. history and the
largest federal investment in the U.S. economy since Lyndon Johnson’s Great Society.
(Adapted from https://time.com/5974129/joe-biden-joint-address-congress/)

'A América está em movimento novamente.' Joe Biden profere seu primeiro discurso presidencial importante
O presidente Joe Biden participou de dezenas de discursos presidenciais no Congresso desde que
veio pela primeira vez a Washington em 1973. Mas depois de quase meio século como membro
da audiência, ele finalmente conseguiu falar na quarta-feira à noite para expor sua própria visão
para o país.
“A América é ___ a mudança de novo”, disse ele durante seus comentários de aproximadamente
uma hora. “Transformando o perigo em possibilidade. Crise em oportunidade. Retroceda em
força.”
Biden aproveitou a oportunidade para se gabar das realizações de seu governo, destacando como
os americanos receberam mais de 220 milhões de injeções da vacina COVID-19 em seus primeiros
100 dias de mandato, o que ele chamou de “uma das maiores conquistas logísticas que este país
já viu.”

AULA 09 – RELATIVE CLAUSES AND QUANTIFIERS 109


TEACHER ANDREA BELO

Como o resto dos marcos presidenciais de Biden, este foi alterado devido ___ a pandemia. A
audiência na Câmara da Câmara, geralmente mais de 1.000 pessoas, incluindo todos os membros
do Congresso e vários juízes da Suprema Corte, foi limitada a apenas 200. Os participantes
sentaram-se separados uns dos outros para acomodar o distanciamento social, com no máximo
três pessoas em um fileira.
Seu discurso se concentrou amplamente nas prioridades econômicas e ele delineou o Plano de
Famílias Americanas, sua proposta de US $ 1,8 trilhão ___ aumentar os impostos sobre os ricos
para pagar por creches a preços acessíveis, licenças pagas e acesso à educação. Se implementado,
o plano ofereceria pré-escola universal para crianças de três e quatro anos e dois anos de
faculdade comunitária gratuita e criaria um programa nacional de licença remunerada. Esta
proposta é a terceira ponta do plano de US $ 6 trilhões de Biden para impulsionar a economia
pós-pandemia, combatendo a desigualdade e as mudanças climáticas. Ele assinou a primeira
parte, o Plano de Resgate Americano de US $ 1,9 trilhão, em lei em 11 de março. No mês passado,
também pediu uma conta de US $ 2 trilhões que criaria milhões de empregos e repararia a
infraestrutura do país, saudando-o em seu discurso para O Congresso como um “projeto de
colarinho azul para construir a América”.
No total, essas propostas seriam o maior investimento em energia limpa na história dos EUA e o
maior investimento federal na economia dos EUA desde a Grande Sociedade de Lyndon Johnson.

The lockdown habit that’s hardest to break: using an iPad as a babysitter


How am I supposed to wean my children off screens when they know I’m addicted too?
It started, last March, as an act of necessity – sticking the kids on iPads all day so I could make a
hard deadline that fell six weeks into lockdown. There was no way around this; at five years old,
my kids couldn’t manage their Zoom schedules or self-entertain for long without fighting, and I
couldn’t break off every two minutes to help them.
Overnight, kindergarten and after-school disappeared, to be replaced with the sedative of kids
YouTube, and when the appeal of that started to wane, the more addictive and ruinous content
on TikTok. If it was hideous, I told myself, it was an emergency. It wouldn’t be like this for ever.
A year later and in New York, at least, we’re in a radically different place. Next week my kids’
elementary school goes back full-time and although it lets out early – at 2pm – half of the city is
now at least partially vaccinated, so most parents are comfortable hiring a sitter. (If they can find
one, that is; indoor after-school programmes still seem risky to many, so the scramble is on for
that already mythical creature, the 22-year-old sitter only looking for two hours work a day.)
Things are, on the surface, starting to look vaguely like normal.
Except, of course, they’re not. Among the many new habits formed during the pandemic, a
reliance on screens as babysitters may be one of the toughest to break. Over the course of the
year, I have grown accustomed to cleaning the house, finishing work, folding the laundry and even
– if my kids hit a solid addictive high on Minecraft – taking a nap, all without the cost or logistical
planning of babysitting. I can put in a solid few hours at the park, safe in the knowledge that when
we get home help in the form of two hours of back-to-back three-minute videos will keep
everyone happy until dinner.
(Adapted from https://www.theguardian.com/commentisfree/2021/apr/26/lockdown-ipad-babysitter-children-addicted)

AULA 09 – RELATIVE CLAUSES AND QUANTIFIERS 110


TEACHER ANDREA BELO

O hábito de bloqueio mais difícil de quebrar: usar um iPad como babá


Como devo afastar meus filhos das telas quando eles sabem que também sou viciado?
Tudo começou, em março passado, como um ato de necessidade - colocar as crianças em iPads o
dia todo para que eu pudesse cumprir um prazo difícil que caiu em seis semanas. Não havia
maneira de contornar isso; aos cinco anos de idade, meus filhos não conseguiam gerenciar suas
programações do Zoom ou se divertir por muito tempo sem lutar, e eu não conseguia interromper
a cada dois minutos para ajudá-los.
Da noite para o dia, o jardim de infância e o período pós-escola desapareceram, para serem
substituídos pelo sedativo infantil do YouTube, e quando o apelo disso começou a diminuir, o
conteúdo mais viciante e destruidor do TikTok. Se era horrível, disse a mim mesmo, era uma
emergência. Não seria assim para sempre.
Um ano depois e em Nova York, pelo menos, estamos em um lugar radicalmente diferente. Na
próxima semana, a escola primária dos meus filhos volta em tempo integral e, embora saia mais
cedo - às 14h - metade da cidade agora está pelo menos parcialmente vacinada, então a maioria
dos pais se sente confortável em contratar uma babá. (Isto é, se eles conseguirem encontrar um;
programas pós-escola fechados ainda parecem arriscados para muitos, então a disputa começou
por aquela criatura já mítica, a babá de 22 anos que procura apenas duas horas de trabalho por
dia.) Coisas estão, na superfície, começando a parecer vagamente normais.
Exceto, é claro, eles não são. Entre os muitos novos hábitos formados durante a pandemia, a
dependência de telas como babás pode ser um dos mais difíceis de romper. Ao longo do ano,
acostumei-me a limpar a casa, terminar o trabalho, dobrar a roupa e até mesmo - se meus filhos
obtiverem uma alta e viciante de Minecraft - tirar uma soneca, tudo sem o custo ou planejamento
logístico de babá. Posso passar algumas horas no parque, com a certeza de que, quando
chegarmos em casa, a ajuda na forma de duas horas de vídeos consecutivos de três minutos
manterá todos felizes até o jantar.

Australians stuck overseas 'abandoned' by their own country


Australian citizen Mandeep Sharma feels utterly deserted by his government.
He is one of the 9,000 Australian nationals stranded in India right now, left to fend for themselves
after Canberra this week banned all flights from the pandemic-ravaged nation until mid-May.
He has a wife and two daughters in Adelaide and fears their separation could drag on for months.
Catching Covid in India is also a real risk.
The Australian government's flight ban was the latest hardline action taken to keep the virus out
of the country. Infection rates are near zero, and Australia has had far fewer fatalities than most
countries, thanks to strict border controls and quarantine measures.
Still, those policies have left many Australians locked out. The ban on Indian arrivals marked an
escalation – the first time the country has stopped evacuations and blocked citizens from
returning home altogether. It's intensified calls for more to be done to get Australians home.
Why can't citizens get in?

AULA 09 – RELATIVE CLAUSES AND QUANTIFIERS 111


TEACHER ANDREA BELO

Prior to the pandemic, there were estimated to be about one million Australians living overseas.
Many have come home in the past year, but many more have struggled to return.
Currently, around 36,000 nationals are registered for government help to fly home, but the true
number of those who wish to come back is estimated to be much higher.
Australia became one of the first nations to close its borders in March 2020. Only returning
nationals and residents were allowed in. Some exemptions were granted, including celebrities,
sports stars and contract workers. Since April, it has also allowed travellers from virus-free New
Zealand.
Those who returned were forced to undertake – and fund – a two-week quarantine stay at hotels
in state capitals. In the early pandemic rush, state governments agreed to take on the burden of
running quarantine instead of the federal government, given the lack of national facilities.
About a month in, a problem emerged. The number of people returning home - mostly from New
Zealand, the US and the UK- threatened to overwhelm the system. There were only so many hotel
rooms available.
But instead of expanding the system - for example, adding a purpose-built quarantine centre –
the government drastically cut back the number of plane arrivals allowed in each week.
(Adapted from https://www.bbc.com/news/world-australia-56924188)

Australianos presos no exterior "abandonados" por seu próprio país


O cidadão australiano Mandeep Sharma sente-se totalmente abandonado por seu governo.
Ele é um dos 9.000 australianos presos na Índia agora, abandonados à própria sorte depois que
Canberra proibiu nesta semana todos os voos do país devastado pela pandemia até meados de
maio.
Ele tem mulher e duas filhas em Adelaide e teme que a separação possa se arrastar por meses.
Capturar Covid na Índia também é um risco real.
A proibição de voos do governo australiano foi a última ação da linha dura para manter o vírus
fora do país. As taxas de infecção são próximas de zero e a Austrália teve muito menos mortes do
que a maioria dos países, graças a controles rígidos de fronteira e medidas de quarentena.
Ainda assim, essas políticas deixaram muitos australianos sem acesso. A proibição de chegadas
de índios marcou uma escalada - a primeira vez que o país interrompeu as evacuações e bloqueou
totalmente os cidadãos de voltarem para casa. É intensificado o apelo para que mais seja feito
para levar os australianos para casa.
Por que os cidadãos não podem entrar?
Antes da pandemia, estimava-se que havia cerca de um milhão de australianos vivendo no
exterior. Muitos voltaram para casa no ano passado, mas muitos outros tiveram dificuldade para
voltar.
Atualmente, cerca de 36.000 cidadãos estão registrados para receber ajuda do governo para voar
para casa, mas estima-se que o número real de pessoas que desejam voltar seja muito maior.

AULA 09 – RELATIVE CLAUSES AND QUANTIFIERS 112


TEACHER ANDREA BELO

A Austrália se tornou uma das primeiras nações a fechar suas fronteiras em março de 2020.
Apenas cidadãos e residentes que retornavam eram permitidos. Algumas isenções foram
concedidas, incluindo celebridades, estrelas do esporte e trabalhadores contratados. Desde abril,
também permite viajantes da Nova Zelândia livre de vírus.
Os que voltaram foram forçados a empreender - e financiar - uma estadia de quarentena de duas
semanas em hotéis nas capitais dos estados. No início da onda de pandemia, os governos
estaduais concordaram em assumir o fardo da quarentena em vez do governo federal, devido à
falta de instalações nacionais.
Cerca de um mês depois, surgiu um problema. O número de pessoas voltando para casa -
principalmente da Nova Zelândia, Estados Unidos e Reino Unido - ameaçou sobrecarregar o
sistema. Havia tantos quartos de hotel disponíveis.
Mas, em vez de expandir o sistema - por exemplo, adicionar um centro de quarentena construído
para esse fim - o governo reduziu drasticamente o número permitido de chegadas de aviões a
cada semana.

Texas passes law banning abortion after six weeks


A law banning abortion from as early as six weeks into pregnancy has come into effect in the
US state of Texas.
It bans abortions after the detection of what anti-abortion campaigners call a fetal heartbeat,
something medical authorities say is misleading.
The law, one of the most restrictive in the country, took effect after the Supreme Court did not
respond to an emergency appeal by abortion providers.
Doctors and women's rights groups have heavily criticized the law.
It gives any individual the right to sue doctors who perform an abortion past the six-week point.
The so-called "Heartbeat Act" was signed into law by Texas Governor Greg Abbott in May.
But rights groups, including Planned Parenthood and the American Civil Liberties Union (ACLU),
then requested that the Supreme Court block the legislation.
In the early hours of Wednesday, the ACLU confirmed that the court had "not responded to our
request", adding: "Access to almost all abortion has just been cut off for millions of people."
The group, which says that up to 90% of abortions in Texas take place after six weeks of pregnancy,
described the development as "blatantly unconstitutional".

Texas aprova lei que proíbe o aborto após seis semanas


Uma lei que proíbe o aborto a partir das seis semanas de gravidez entrou em vigor no estado
americano do Texas.
Ele proíbe o aborto após a detecção do que os ativistas antiaborto chamam de batimento cardíaco
fetal, algo que as autoridades médicas dizem ser enganoso.
A lei, uma das mais restritivas do país, entrou em vigor depois que a Suprema Corte não respondeu
a um apelo de emergência dos provedores de aborto.

AULA 09 – RELATIVE CLAUSES AND QUANTIFIERS 113


TEACHER ANDREA BELO

Médicos e grupos de direitos das mulheres criticaram fortemente a lei.


Dá a qualquer indivíduo o direito de processar os médicos que realizam um aborto além do ponto
de seis semanas.
O chamado "Heartbeat Act" foi transformado em lei pelo governador do Texas, Greg Abbott, em
maio.
Mas grupos de direitos humanos, incluindo Planned Parenthood e American Civil Liberties Union
(ACLU), solicitaram que a Suprema Corte bloqueasse a legislação.
Na madrugada desta quarta-feira, a ACLU confirmou que o tribunal "não respondeu ao nosso
pedido", acrescentando: "O acesso a quase todo o aborto acaba de ser cortado para milhões de
pessoas".
O grupo, que afirma que até 90% dos abortos no Texas acontecem após seis semanas de gravidez,
descreveu o desenvolvimento como "flagrantemente inconstitucional".

Nuclear energy is key in fight for climate


Ignore the myths about nuclear power, writes Rob Loveday of Generation Atomic – it is an
essential source of clean energy
The letter on nuclear energy (25 August) sadly could not be more wrong – nuclear is one of the
cleanest fuels we have and has always been so. The carbon and material footprints of nuclear –
for its entire lifetime, including mining and decommissioning – are lower than solar and on a par
with wind, according to the Intergovernmental Panel on Climate Change.
Moreover, the nuclear industry manages its waste stream – that is more than can be said for the
solar industry, which is set to produce millions of tons of toxic waste. Besides, spent nuclear fuel
is not “waste” but a valuable source of low-carbon energy that can be recycled via reprocessing
or proposed breeder reactors, thereby neutralizing the vast majority of the radioactivity.
At a time when we need every tool in the box to fight the climate crisis – and nuclear is one of the
most effective – isn’t it time we started to look at the facts rather than repeat myths about nuclear
waste?
(Adapted from https://www.theguardian.com/environment/2021/aug/31/nuclear-energy-is-key-in-fight-for-climate)

A energia nuclear é a chave na luta pelo clima


Ignore os mitos sobre a energia nuclear, escreve Rob Loveday da Generation Atomic - é uma
fonte essencial de energia limpa
A carta sobre energia nuclear (25 de agosto), infelizmente, não poderia estar mais errada - o
nuclear é um dos combustíveis mais limpos que temos e sempre foi. As pegadas de carbono e
materiais da energia nuclear - por toda a sua vida útil, incluindo mineração e descomissionamento
- são menores do que a solar e no mesmo nível do vento, de acordo com o Painel
Intergovernamental sobre Mudanças Climáticas.
Além disso, a indústria nuclear gerencia seu fluxo de resíduos - isso é mais do que pode ser dito
da indústria solar, que deve produzir milhões de toneladas de resíduos tóxicos. Além disso, o

AULA 09 – RELATIVE CLAUSES AND QUANTIFIERS 114


TEACHER ANDREA BELO

combustível nuclear usado não é um “resíduo”, mas uma fonte valiosa de energia de baixo
carbono que pode ser reciclada por meio de reprocessamento ou reatores de regeneração
propostos, neutralizando assim a grande maioria da radioatividade.
Em um momento em que precisamos de todas as ferramentas da caixa para combater a crise
climática - e a nuclear é uma das mais eficazes - não é hora de começarmos a olhar para os fatos
em vez de repetir mitos sobre o lixo nuclear?

The Titanic
The Titanic was built by the White Star Line. The owners of the company thought that if ocean
liners were big and luxurious enough more people ________ travel with them. The Titanic was
designed to be the largest in a series of three ships made by the White Star line. It was 268 meters
long, 28 meters wide, and weighed 45 000 tons. It produced enough power to travel at a speed
of 24 knots (about 40km per hour).
The bulk of the ship was divided into compartments. They were separated by steel doors that did
not let any water through. The ship could still move and float if 3 or 4 of the 16 compartments
were filled with water.
The Titanic was more like a floating hotel than a ship. It cost $7.5 million, and it was unlike any
other ship that had ever been built. Palm trees and other expensive plants decorated the luxurious
hallways and corridors. The ship could carry 2 600 passengers and a crew of 900.
On April 10, 1912 over 2200 passengers boarded the Titanic on its maiden voyage to New York.
Many of them were immigrants who saved all their money for the journey. First class passengers
had to pay between $2 500 and $4 500 for a private room and a bath, third class passengers had
to share rooms and paid $35 each. Although the ship’s owners said the Titanic was unsinkable,
many problems before the first voyage were overlooked.
Safety regulations at that time were not very strict. The ship only had 16 lifeboats, enough for
about 1 500 passengers. It was only tested for a few hours and never went at full speed. The
telegraph system on board was new and not many people knew how to operate it.
During the night of April 14, 1912, the waters of the North Atlantic had a temperature of about -
2° C. At noon on that day the radio operators got messages from other ships about icebergs that
were nearby. The Titanic’s captain, Edward Smith, did not care about these warnings. He was
captain of a steel giant that could not sink. The only thing he cared about was setting up a new
world speed record. The Titanic was to be the fastest ship that ever sailed from Southampton to
New York.
The night was clear, and the Titanic sped on. When a big iceberg was sighted, the first officer shut
down all the engines. But it would have taken the ship about half a mile to come to a full stop.
Even though, on the surface, the ship stayed clear of the iceberg, it ripped a big hole in the hull.
At once the compartments began to flood with cold, icy water.
The bulkheads were lowered but it was too late. Water flooded at least five compartments. The
collision with the iceberg was so slight that the passengers hardly heard it. Most of them didn’t

AULA 09 – RELATIVE CLAUSES AND QUANTIFIERS 115


TEACHER ANDREA BELO

take any notice and continued dancing and having fun. Some passengers were asleep in their
cabins.
The bow of the ship dipped under the water’s surface and the back part of the ship began to rise.
After a short time, the Titanic broke into two pieces. When Captain Smith realized that the Titanic
was sinking, he had a distress signal sent out, but the nearest ship was a hundred kilometers away.
As time went on, chaos emerged, and passengers rushed to the boat deck. Women and children
were allowed on the lifeboats first. Lights flickered and electricity was finally gone. At 2:20 a.m.
the Titanic disappeared into the Atlantic Ocean.
The Carpathia, which was the nearest ship, came to the scene about two hours later and picked
up the freezing passengers in their lifeboats. By early morning, the news of the disaster had gone
around the world. The world’s largest ocean liner, the Titanic, had sunk on its maiden voyage,
killing 1513 people.
Adapted from https://www.english-online.at/history/titanic/titanic.htm.

O Titanic
O Titanic foi construído pela White Star Line. Os proprietários da empresa achavam que, se os
transatlânticos fossem grandes e luxuosos o suficiente, mais pessoas ________ viajariam com
eles. O Titanic foi projetado para ser o maior de uma série de três navios feitos pela linha White
Star. Tinha 268 metros de comprimento, 28 metros de largura e pesava 45.000 toneladas. Ele
produziu energia suficiente para viajar a uma velocidade de 24 nós (cerca de 40 km por hora).
A maior parte do navio foi dividido em compartimentos. Eles estavam separados por portas de
aço que não deixavam a água passar. O navio ainda poderia se mover e flutuar se 3 ou 4 dos 16
compartimentos estivessem cheios de água.
O Titanic parecia mais um hotel flutuante do que um navio. Custou US $ 7,5 milhões e era
diferente de qualquer outro navio já construído. Palmeiras e outras plantas caras decoravam os
corredores e corredores luxuosos. O navio podia transportar 2 600 passageiros e uma tripulação
de 900.
Em 10 de abril de 1912, mais de 2.200 passageiros embarcaram no Titanic em sua viagem
inaugural para Nova York. Muitos deles eram imigrantes que economizaram todo o seu dinheiro
para a viagem. Os passageiros da primeira classe tiveram que pagar entre US $ 2.500 e US $ 4.500
por um quarto privativo e um banheiro, os passageiros da terceira classe tiveram que dividir os
quartos e pagaram US $ 35 cada. Embora os proprietários do navio tenham dito que o Titanic era
inafundável, muitos problemas antes da primeira viagem foram esquecidos.
Os regulamentos de segurança naquela época não eram muito rígidos. O navio tinha apenas 16
botes salva-vidas, o suficiente para cerca de 1.500 passageiros. Ele foi testado por apenas algumas
horas e nunca foi em velocidade máxima. O sistema telegráfico a bordo era novo e poucas pessoas
sabiam como operá-lo.
Durante a noite de 14 de abril de 1912, as águas do Atlântico Norte apresentavam uma
temperatura de cerca de -2 ° C. Ao meio-dia desse dia os operadores de rádio receberam
mensagens de outros navios sobre icebergs que estavam próximos. O capitão do Titanic, Edward
Smith, não se importou com esses avisos. Ele era o capitão de um gigante de aço que não podia

AULA 09 – RELATIVE CLAUSES AND QUANTIFIERS 116


TEACHER ANDREA BELO

afundar. A única coisa com que se importava era estabelecer um novo recorde mundial de
velocidade. O Titanic era para ser o navio mais rápido que já navegou de Southampton para Nova
York.
A noite estava clara e o Titanic acelerou. Quando um grande iceberg foi avistado, o primeiro oficial
desligou todos os motores. Mas o navio levaria cerca de meia milha para parar completamente.
Mesmo que, na superfície, o navio tenha ficado longe do iceberg, ele abriu um grande buraco no
casco. Imediatamente os compartimentos começaram a inundar com água fria e gelada.
As anteparas foram baixadas, mas era tarde demais. A água inundou pelo menos cinco
compartimentos. A colisão com o iceberg foi tão leve que os passageiros quase não ouviram. A
maioria deles não percebeu e continuou dançando e se divertindo. Alguns passageiros dormiam
em suas cabines.
A proa do navio mergulhou sob a superfície da água e a parte traseira do navio começou a subir.
Após um curto período de tempo, o Titanic se partiu em duas partes. Quando o capitão Smith
percebeu que o Titanic estava afundando, ele mandou um sinal de socorro, mas o navio mais
próximo estava a cem quilômetros de distância.
Com o passar do tempo, o caos surgiu e os passageiros correram para o convés do barco.
Mulheres e crianças foram permitidas nos botes salva-vidas primeiro. As luzes piscaram e a
eletricidade finalmente acabou. Às 02h20, o Titanic desapareceu no Oceano Atlântico.
O Carpathia, que era o navio mais próximo, entrou em cena cerca de duas horas depois e recolheu
os passageiros congelados em seus botes salva-vidas. No início da manhã, a notícia do desastre
se espalhou pelo mundo. O maior transatlântico do mundo, o Titanic, havia afundado em sua
viagem inaugural, matando 1.513 pessoas.

Oxygen shortages are killing thousands. Why aren’t we doing more about this?
As with vaccines and drugs, the pandemic has shown how access to this life-saving resource is deeply unequal
The basics ___ caring for acutely unwell patients are simple: air needs to go in and out and blood
needs to go round and round. Across the world, the pandemic has consistently shown how poorly
equipped healthcare systems are for addressing these needs. Much attention has been paid to
vaccines, drug therapies and ventilators ___ recent months, while relatively little has been said
about the most basic human requirement of all – oxygen.
Oxygen is all around us, and yet there are acute shortages of it in many healthcare settings. This
is because the infrastructure needed to supply oxygen to patients, such as large vacuum-insulated
evaporators (which are like giant, very cold vacuum flasks), is relatively expensive and needs
regular maintenance and top-ups of liquid oxygen. Where this isn’t available, hospitals might use
concentrators ___ extract oxygen from the surrounding environment. These require electricity
and compressed air – which, again, are scarce in many places. Or they might use cylinders, which
can store and deliver short-term oxygen therapy. But they need regularly refilling, which depends
on secure supply lines.
I am a doctor who specializes in respiratory and intensive care medicine. Thankfully, I have never
been in a situation where I needed to provide a patient with oxygen therapy when none was

AULA 09 – RELATIVE CLAUSES AND QUANTIFIERS 117


TEACHER ANDREA BELO

available. But this is exactly the situation facing healthcare workers in many other countries.
During the pandemic, systems for delivering oxygen came under pressure almost everywhere. In
the UK, hospitals that were decades old struggled to cope with surges of patients needing the
type of high oxygen flows used in modern therapy. In some low- and middle-income countries,
the infrastructure to deliver oxygen to patients was nonexistent.
(Adapted from https://www.theguardian.com/commentisfree/2021/jun/08/oxygen-shortages-killing-vaccines-drugs)

A escassez de oxigênio está matando milhares. Por que não estamos fazendo mais sobre isso?
Tal como acontece com vacinas e medicamentos, a pandemia mostrou como o acesso a este
recurso que salva vidas é profundamente desigual
O básico ___ cuidar de pacientes com indisposição aguda é simples: o ar precisa entrar e sair e o
sangue precisa girar e girar. Em todo o mundo, a pandemia tem mostrado de forma consistente
como os sistemas de saúde são mal equipados para atender a essas necessidades. Muita atenção
tem sido dada às vacinas, terapias medicamentosas e ventiladores ___ nos últimos meses,
enquanto relativamente pouco foi dito sobre a necessidade humana mais básica de todas - o
oxigênio.
O oxigênio está ao nosso redor e, ainda assim, há uma escassez aguda dele em muitos ambientes
de saúde. Isso ocorre porque a infraestrutura necessária para fornecer oxigênio aos pacientes,
como grandes evaporadores isolados a vácuo (que são como frascos de vácuo gigantes e muito
frios), é relativamente cara e requer manutenção regular e recargas de oxigênio líquido. Onde
não estiver disponível, os hospitais podem usar concentradores ___ para extrair oxigênio do
ambiente circundante. Isso requer eletricidade e ar comprimido - que, novamente, são escassos
em muitos lugares. Ou podem usar cilindros, que podem armazenar e administrar oxigenoterapia
de curto prazo. Mas eles precisam ser reabastecidos regularmente, o que depende de linhas de
abastecimento seguras.
Sou médico especialista em medicina respiratória e de terapia intensiva. Felizmente, nunca estive
em uma situação em que precisasse fornecer oxigenoterapia a um paciente quando não havia
nenhuma disponível. Mas esta é exatamente a situação que os profissionais de saúde enfrentam
em muitos outros países. Durante a pandemia, os sistemas de fornecimento de oxigênio ficaram
sob pressão em quase todos os lugares. No Reino Unido, hospitais com décadas de existência
lutavam para lidar com o aumento repentino de pacientes que precisavam do tipo de alto fluxo
de oxigênio usado na terapia moderna. Em alguns países de baixa e média renda, a infraestrutura
para fornecer oxigênio aos pacientes era inexistente.

If we loosen restrictions too early, there is a real risk of a third wave in the UK
Not enough people are vaccinated against Covid as a new variant spreads, requiring us to
remain vigilant for a little longer
Just when it felt like we could begin to relax again, Covid-19 has thrown us yet another curveball.
While we’ve been debating in the UK where to go for a holiday and booking long-overdue nights
out with friends, the virus has been causing havoc across the world.

AULA 09 – RELATIVE CLAUSES AND QUANTIFIERS 118


TEACHER ANDREA BELO

The Kent variant, B.1.1.7, found its way into countries such as Taiwan and Singapore; P.1 is causing
another wave in Brazil; and most devastatingly, the highly transmissible variant B.1.617 (which
has three subtypes) is causing a public health crisis in India. The country has seen cases surging
with estimated deaths to be several times greater than that being reported by the government,
which continues to downplay the epidemic.
The UK has done well, largely through strong public compliance, in bringing its case numbers
down, but Boris Johnson has repeated his mistake from last summer and left the borders open.
The Sunday Times estimates that at least 20,000 passengers from India were allowed to enter the
UK while Johnson delayed imposing a travel ban because he didn’t want to upset the Indian prime
minister, Narendra Modi, while negotiating a trade deal. This is at a time when other countries
such as New Zealand and Hong Kong completely stopped all flights.
This puts us now in a precarious situation. The World Health Organization has identified B.1.617
as a “variant of concern” because it is at least as transmissible as the Kent variant, potentially even
more so. This means that the virus can jump faster among those susceptible and accelerate out
of control. The UK has not fully vaccinated enough of the population to avoid a third wave among
younger people that could still strain the NHS.
(Adapted from https://www.theguardian.com/commentisfree/2021/may/17/third-wave-risk-uk-vaccinated-covid-variant)

Se afrouxarmos as restrições muito cedo, há um risco real de uma terceira onda no Reino Unido
Não são vacinadas pessoas suficientes contra a Covid à medida que uma nova variante se espalha,
exigindo que permaneçamos vigilantes por mais algum tempo
Exatamente quando parecia que poderíamos começar a relaxar novamente, a Covid-19 nos
lançou mais um obstáculo. Enquanto estivemos debatendo no Reino Unido onde ir passar as férias
e reservando noites há muito esperadas com amigos, o vírus tem causado estragos em todo o
mundo.
A variante Kent, B.1.1.7, encontrou seu caminho em países como Taiwan e Cingapura; P.1 está
causando outra onda no Brasil; e mais devastadoramente, a variante B.1.617 altamente
transmissível (que tem três subtipos) está causando uma crise de saúde pública na Índia. O país
tem visto casos crescentes, com mortes estimadas várias vezes maiores do que as relatadas pelo
governo, que continua a minimizar a epidemia.
O Reino Unido se saiu bem, em grande parte por meio de forte concordância pública, em reduzir
o número de casos, mas Boris Johnson repetiu seu erro do verão passado e deixou as fronteiras
abertas. O Sunday Times estima que pelo menos 20.000 passageiros da Índia foram autorizados
a entrar no Reino Unido, enquanto Johnson atrasou a imposição de uma proibição de viagens
porque não queria incomodar o primeiro-ministro indiano, Narendra Modi, enquanto negociava
um acordo comercial. Isso ocorre em um momento em que outros países, como Nova Zelândia e
Hong Kong, interromperam completamente todos os voos.
Isso nos coloca agora em uma situação precária. A Organização Mundial de Saúde identificou
B.1.617 como uma “variante preocupante” porque é pelo menos tão transmissível quanto a
variante Kent, potencialmente ainda mais. Isso significa que o vírus pode saltar mais rápido entre
os suscetíveis e acelerar fora de controle. O Reino Unido não vacinou totalmente a população o
suficiente para evitar uma terceira onda entre os mais jovens que ainda poderia prejudicar o NHS.

AULA 09 – RELATIVE CLAUSES AND QUANTIFIERS 119


TEACHER ANDREA BELO

“Covid-19 has killed millions around the world, but ___ some who are lucky enough to survive the
infection, ___ nightmare is not over: adding insult to injury are deadly fungal infections that follow
in the wake of the virus. Making matters worse, inequities ___ long predated the pandemic have
left some countries without the capacity to combat these serious infections”.
(Adapted from https://www.theguardian.com/commentisfree/2021/jun/02/black-fungus-covid-india-mucormycosis)

“Covid-19 já matou milhões em todo o mundo, mas ___ alguns que têm a sorte de sobreviver à
infecção, ___ pesadelo ainda não acabou: agravando a situação, estão as infecções fúngicas
mortais que se seguem na esteira do vírus. Para piorar as coisas, as desigualdades ___ muito
anteriores à pandemia deixaram alguns países sem a capacidade de combater essas infecções
graves ”.

After a year at home, children with disabilities deserve priority vaccination


They disappeared more than a year ago and many are still out of sight. When the pandemic hit,
53,000 under-18s in England with disabilities that made them vulnerable to coronavirus began to
shield away at home.
Kept off school long after their classmates went back, and away from friends, they have found
their childhoods put on hold. And while the vaccine rollout gave high-risk adults some reprieve in
the new year, those aged under 16, who have not been eligible for any vaccine, are still living in
limbo.
Yet we have barely heard a thing about disabled children’s plight through the coronavirus crisis –
a silence that has not been helped by the faux-reassuring narrative that “no healthy child” has
died from the virus. Now that the UK regulator has at last approved the Pfizer vaccine for 12- to
15-year-olds, this blind spot is becoming even more glaring.
(Adapted from https://www.theguardian.com/commentisfree/2021/jun/10/children-disabilities-priority-vaccination-england-education-shielding)

Depois de um ano em casa, crianças com deficiência merecem vacinação prioritária


Eles desapareceram há mais de um ano e muitos ainda estão fora de vista. Quando a pandemia
atingiu, 53.000 menores de 18 anos na Inglaterra com deficiências que os tornavam vulneráveis
ao coronavírus começaram a se proteger em casa.
Continuaram fora da escola muito depois que seus colegas voltaram, e longe dos amigos, eles
descobriram que sua infância foi adiada. E embora o lançamento da vacina tenha dado aos adultos
de alto risco alguma folga no ano novo, aqueles com menos de 16 anos, que não tinham direito a
nenhuma vacina, ainda vivem no limbo.
No entanto, quase não ouvimos nada sobre a situação das crianças deficientes durante a crise do
coronavírus - um silêncio que não foi ajudado pela narrativa falsa e tranquilizadora de que
"nenhuma criança saudável" morreu do vírus. Agora que o regulador do Reino Unido finalmente
aprovou a vacina da Pfizer para crianças de 12 a 15 anos, esse ponto cego está se tornando ainda
mais evidente.

AULA 09 – RELATIVE CLAUSES AND QUANTIFIERS 120


TEACHER ANDREA BELO

Rare purple textiles from the time of biblical kings found for the first time in Israel
By Harry Baker – Staff Writer
The color was a favorite of the biblical kings David and Solomon
A patch of 3,000-year-old wool dyed true purple, recently discovered in Timna Valley in Israel.
Archaeologists have uncovered rare fragments of 3,000-year-old textiles stained purple — a color
considered the height of royal fashion at the time — in southern Israel.
The archaeologists discovered the purple textiles — which included bits of woven fabric, a tassel
and a bundle of wool fibers — at Slaves' Hill in Timna Valley, an ancient copper production district
in the Arava desert, the team wrote in a new study describing the findings.
Radiocarbon dating of the fabrics revealed that they were woven around 1000 B.C., placing them
in the time of the biblical kings David and Solomon — who ruled from 1010–970 B.C. and 970–
931 B.C., respectively — in Jerusalem. The Bible mentions the kings and other important figures
wearing the color during this time, according to the researchers.
Related: 10 fascinating biblical-era discoveries from 2018
The dye used to stain the fabrics was made from mollusks found hundreds of miles away in the
Mediterranean and was extremely valuable as a result. However, until now, no physical evidence
of its use had ever been recovered in Israel or the rest of the southern Levant (an area
encompassing the eastern Mediterranean).
"For the first time, we have direct evidence of the dyed fabrics themselves, preserved for some
3,000 years," Naama Sukenik, curator of organic finds at the Israel Antiquities Authority and lead
author of the new paper describing the textiles, told Live Science in an email. "Each fragment gives
us new information, tells us a new story about the site and important information about the
people that lived there."
(Adapted from https://www.livescience.com/59693-could-earth-turn-into-venus. – Access on 07/28/20)

Têxteis roxos raros da época dos reis bíblicos encontrados pela primeira vez em Israel
Por Harry Baker – redator da equipe
A cor era uma das favoritas dos reis bíblicos Davi e Salomão
Um pedaço de lã de 3.000 anos tingido de roxo verdadeiro, recentemente descoberto no vale de
Timna, em Israel.
Arqueólogos descobriram fragmentos raros de tecidos de 3.000 anos manchados de roxo - uma
cor considerada o auge da moda real na época - no sul de Israel.
Os arqueólogos descobriram os tecidos roxos - que incluíam pedaços de tecido, uma borla e um
feixe de fibras de lã - na Colina dos Escravos no Vale de Timna, um antigo distrito de produção de
cobre no deserto de Arava, escreveu a equipe em um novo estudo que descreve o descobertas.
A datação por radiocarbono dos tecidos revelou que eles foram tecidos por volta de 1000 a.C.,
colocando-os na época dos reis bíblicos Davi e Salomão - que governaram de 1010–970 a.C. e
970–931 a.C., respectivamente - em Jerusalém. A Bíblia menciona os reis e outras figuras
importantes vestindo a cor durante esse período, de acordo com os pesquisadores.
Relacionado: 10 descobertas fascinantes da era bíblica de 2018

AULA 09 – RELATIVE CLAUSES AND QUANTIFIERS 121


TEACHER ANDREA BELO

A tinta usada para manchar os tecidos era feita de moluscos encontrados a centenas de
quilômetros de distância no Mediterrâneo e era extremamente valiosa como resultado. No
entanto, até agora, nenhuma evidência física de seu uso havia sido recuperada em Israel ou no
resto do sul do Levante (uma área que abrange o Mediterrâneo oriental).
"Pela primeira vez, temos evidências diretas dos próprios tecidos tingidos, preservados por cerca
de 3.000 anos", disse Naama Sukenik, curador de achados orgânicos da Autoridade de
Antiguidades de Israel e principal autor do novo artigo que descreve os têxteis, ao Live Science
em um email. "Cada fragmento nos dá novas informações, conta uma nova história sobre o site e
informações importantes sobre as pessoas que moraram lá."

RIO DE JANEIRO, BRAZIL - In the past 24 hours, Brazil recorded 1,113 deaths and 36,653 new cases related
to the novel coronavirus. The data are included in the Ministry of Health's daily balance released on
Tuesday evening, September 15th.
Since the start of the pandemic, 133,119 people have died as a result of Covid-19. On Monday, the
Ministry of Health's data system recorded a total of 132,006 deaths. A further 2,445 are still under
investigation by health authorities.
(Adapted from https://riotimesonline.com - September 15th)

RIO DE JANEIRO, BRASIL - Nas últimas 24 horas, o Brasil registrou 1.113 mortes e 36.653 novos casos
relacionados ao novo coronavírus. Os dados constam do balanço diário do Ministério da Saúde, divulgado
na noite de terça-feira, 15 de setembro.
Desde o início da pandemia, 133.119 pessoas morreram como resultado da Covid-19. Na segunda-feira, o
sistema de dados do Ministério da Saúde registrou um total de 132.006 mortes. Outros 2.445 ainda estão
sob investigação das autoridades sanitárias.
August 08, 2020

RIO DE JANEIRO, BRAZIL - On Saturday, August 8th, Brazil reached the number of 100,477 deaths
from Covid-19 since the start of the novel coronavirus pandemic. According to the Ministry of
Health, 905 deaths were recorded over the last 24 hours.
Of the 3,012,412 cases of people infected by the novel coronavirus, 2,094,293 (69.5 percent) have
recovered.
A total of 49,970 new cases have been reported by Health Secretariats since Friday, August 7th.
The official also shows that the number of people under follow-up stands at 817,642 (21.1
percent).
(Adapted from https://riotimesonline.com)

8 de agosto de 2020
RIO DE JANEIRO, BRASIL - No sábado, 8 de agosto, o Brasil atingiu o número de 100.477 mortes
por Covid-19 desde o início da nova pandemia de coronavírus. De acordo com o Ministério da
Saúde, foram registrados 905 óbitos nas últimas 24 horas.

AULA 09 – RELATIVE CLAUSES AND QUANTIFIERS 122


TEACHER ANDREA BELO

Dos 3.012.412 casos de pessoas infectadas pelo novo coronavírus, 2.094.293 (69,5 por cento) se
recuperaram.
Desde sexta-feira, 7 de agosto, foram notificados 49.970 casos novos pelas Secretarias de Saúde.
O responsável também mostra que o número de pessoas em acompanhamento é de 817.642
(21,1 por cento).

How to defeat AIDS, malaria and tuberculosis

At the turn of the millennium, it was clear a new approach was required in the war against three
of the biggest threats to human life and development. There was need for a trustworthy
international organization that could solicit donations from rich countries and wealthy
organizations and spend that money on combating those threats in collaboration with the
governments of afflicted poor countries, but with appropriate oversight to ensure effectiveness
and avoid theft. The result was the Global Fund to Fight aids, Tuberculosis and Malaria.
And it worked. Though it is impossible to say what would have happened without the Global Fund,
as it is now formally known, the fund’s officials claim to have saved 32m lives since it opened in
2002. (…)
The Economist – October 10th

Como derrotar AIDS, malária e tuberculose

Na virada do milênio, ficou claro que uma nova abordagem era necessária na guerra contra três
das maiores ameaças à vida humana e ao desenvolvimento. Havia necessidade de uma
organização internacional confiável que pudesse solicitar doações de países ricos e organizações

AULA 09 – RELATIVE CLAUSES AND QUANTIFIERS 123


TEACHER ANDREA BELO

ricas e gastar esse dinheiro no combate a essas ameaças em colaboração com os governos dos
países pobres atingidos, mas com supervisão adequada para garantir a eficácia e evitar roubos. O
resultado foi o Fundo Global de Combate à Aids, Tuberculose e Malária.
E funcionou. Embora seja impossível dizer o que teria acontecido sem o Fundo Global, como agora
é formalmente conhecido, os funcionários do fundo afirmam ter salvado 32 milhões de vidas
desde que foi inaugurado em 2002. (...)

Taal: The 'very small but dangerous volcano'


BBC News (Jan/2020)

Over the past few days, it's begun spewing lava, triggering earthquakes, and emitting huge plumes
of ash that have spread across the island of Luzon and beyond.
Scientists fear a bigger "hazardous eruption" is imminent. Taal is tiny, as volcanoes go, but it has
been deadly before. And according to Renato Solidum, the head of the Philippines' Institute of
Volcanology and Seismology (Philvolcs), it is "very small but a dangerous volcano". "Taal volcano
is a baby volcano sitting within a much bigger caldera volcano," said Ben Kennedy, associate
professor of physical volcanology at the University of Canterbury in New Zealand. The entire
Volcano Island has been marked as a permanent danger zone by Phivolcs.

Taal: O 'vulcão muito pequeno, mas perigoso'

Nos últimos dias, começou a cuspir lava, provocando terremotos e emitindo enormes nuvens de
cinzas que se espalharam pela ilha de Luzon e além.

AULA 09 – RELATIVE CLAUSES AND QUANTIFIERS 124


TEACHER ANDREA BELO

Os cientistas temem que uma "erupção perigosa" maior seja iminente. Taal é minúsculo, como os
vulcões, mas já foi mortal antes. E de acordo com Renato Solidum, chefe do Instituto de
Vulcanologia e Sismologia das Filipinas (Philvolcs), é "muito pequeno, mas um vulcão perigoso".
"O vulcão Taal é um vulcão bebê localizado dentro de uma caldeira vulcânica muito maior", disse
Ben Kennedy, professor associado de vulcanologia física da Universidade de Canterbury, na Nova
Zelândia. Toda a Ilha do Vulcão foi marcada como zona de perigo permanente por Phivolcs.

Massive and malodorous – world's biggest flower found

The largest single flower ever recorded was found recently in Sumatra, Indonesia, measuring a
reported 111cm (3.64ft) across. This was a specimen of Rafflesia tuan-mudae and beat the
previous largest flower record of 107cm for Rafflesia arnoldii, also in Sumatra.
Rafflesia is not only a giant flower, but it has no leaves, stems or proper roots. It cannot
photosynthesise and instead sucks the food and water out of a particular vine using long thin
filaments that look like fungal cells. It gorges itself on the vine for a few years before bursting out
into a flower bud, swells for several months before blooming into a flower that looks like a bright
red bucket with big thick lobes. It gives off a whiff of rotting meat that, together with its gigantic
size, helps attract pollinating flies. Rafflesia also steals some of the DNA from the vine it lives on,
using it for its own genetic code for reasons that are not clear.

Maciço e fedorento - a maior flor do mundo encontrada

A maior flor isolada já registrada foi encontrada recentemente em Sumatra, Indonésia, medindo
111 cm (3,64 pés) de diâmetro. Este era um espécime de Rafflesia tuan-mudae e bateu o maior
recorde de flor anterior de 107 cm para Rafflesia arnoldii, também em Sumatra.

AULA 09 – RELATIVE CLAUSES AND QUANTIFIERS 125


TEACHER ANDREA BELO

Rafflesia não é apenas uma flor gigante, mas não tem folhas, caules ou raízes adequadas. Ele não
pode fotossintetizar e, em vez disso, suga a comida e a água de uma determinada videira usando
filamentos longos e finos que parecem células de fungos. Ele se empanturra com a videira por
alguns anos antes de estourar em um botão de flor, incha por vários meses antes de desabrochar
em uma flor que parece um balde vermelho brilhante com grandes lóbulos grossos. Emite um
cheiro a carne podre que, junto com seu tamanho gigantesco, ajuda a atrair as moscas
polinizadoras. Rafflesia também rouba parte do DNA da videira em que vive, usando-o como seu
próprio código genético por motivos que não são claros.

India monsoon: Rescuers search for survivors after heavy rains


Rescuers are scouring devastated parts of western India for survivors after heavy rains caused deadly floods.
Tens of thousands of people have been moved out of affected areas, with record-breaking rainfall
reported along parts of the coast.
The states of Goa and Maharashtra have been badly affected, with many feared missing near the
financial hub Mumbai.
At least 136 people have died in Maharashtra, while in neighbouring Goa hundreds of homes have
been damaged.
Many factors contribute to flooding, but experts say climate change caused by global warming
makes extreme rainfall more likely.
Heavy rains and flooding have also hit western Europe and parts of China in recent weeks, while
North America has grappled with scorching heatwaves.
The monsoon season in India lasts from June to September each year.
Rescuers have struggled to reach affected residents. Landslides have blocked roads, including the
main highway between Mumbai and Goa, and officials have deployed helicopters, divers and naval
rescue teams.
Goa's Chief Minister Pramod Sawant said flooding there was the worst in decades, and had caused
"widespread damage".
In the state of Maharashtra, major rivers are at risk of bursting their banks. Some 90,000 people
have been evacuated so far in the state.
The village of Taliye, southeast of Mumbai, saw landslides flatten most of its homes. An official
told Reuters news agency at least 42 people __________ there.
Maharashtra's chief minister, Udhav Thackeray, plans to visit Taliye on Saturday.
Weather experts say heavy rains will continue to lash the city over the next few days.
Indian Prime Minister Narendra Modi said he was "anguished" by the loss of lives.
"The situation in Maharashtra due to heavy rains is being closely monitored and assistance is being
provided to the affected," he tweeted.
https://www.bbc.com/news/world-asia-india-57952521

AULA 09 – RELATIVE CLAUSES AND QUANTIFIERS 126


TEACHER ANDREA BELO

Serial killer on death row Rodney Alcala dies of natural causes


A man sentenced to death in the US state of California for murdering a 12-year-old girl and four
other women ____________ of natural causes, officials say.
Rodney Alcala, 77, died at a hospital near California's Corcoran state prison in the early hours of
Saturday.
Alcala, who was known as the "Dating Game Killer" after taking part in a US TV show, was convicted
in 2010.
As well as the California killings, he had also pleaded guilty to the murders of two other women in
New York.
Alcala was first handed the death penalty in Orange County in 1980 for the kidnap and murder of
Robin Samsoe, 12, in Los Angeles the previous year.
However, his sentence was overturned by the California Supreme Court and he was granted a new
trial.
He later received the same penalty in the second trial, but this was again overturned in 2003.
In the years that followed, investigators discovered forensic evidence linking Alcala to the other
California murders.
At the 2010 trial, he was found guilty of killing Samsoe as well as four other women aged between
18 and 32 in the years between 1977 and 1979.
In September 1978, Alcala took part in The Dating Game, an American TV show in which a single
woman questioned three single men hidden from her view before selecting one based on their
answers.
Alcala was selected at the end of show, but the woman later said she decided not to date him after
a conversation with him backstage, describing him as "creepy".
In 2012, Alcala was extradited to New York after he was charged over two additional murders from
1971 and 1977. He pleaded guilty and was sentenced in New York to 25 years to life.
California state prison officials said in a statement on Saturday that Alcala was suspected of being
involved in a number of additional killings.
No further details were given about his death.
There are currently some 700 inmates on death row in California, but Governor Gavin Newsom
has ordered a moratorium on executions.
https://www.bbc.com/news/world-us-canada-57955961

O assassino em série no corredor da morte Rodney Alcala morre de causas naturais


Um homem condenado à morte no estado americano da Califórnia por assassinar uma menina de
12 anos e quatro outras mulheres ____________ de causas naturais, dizem as autoridades.
Rodney Alcala, 77, morreu em um hospital perto da prisão estadual de Corcoran, na Califórnia, na
madrugada de sábado.
Alcala, que ficou conhecido como o "Dating Game Killer" depois de participar de um programa de
TV nos Estados Unidos, foi condenado em 2010.
Além dos assassinatos na Califórnia, ele também se confessou culpado dos assassinatos de duas
outras mulheres em Nova York.

AULA 09 – RELATIVE CLAUSES AND QUANTIFIERS 127


TEACHER ANDREA BELO

Alcala foi condenado pela primeira vez à pena de morte em Orange County em 1980 pelo
sequestro e assassinato de Robin Samsoe, 12, em Los Angeles no ano anterior.
No entanto, sua sentença foi anulada pela Suprema Corte da Califórnia e ele foi concedido um
novo julgamento.
Posteriormente, ele recebeu a mesma pena no segundo julgamento, mas foi novamente anulada
em 2003.
Nos anos que se seguiram, os investigadores descobriram evidências forenses ligando Alcala aos
outros assassinatos na Califórnia.
No julgamento de 2010, ele foi considerado culpado de matar Samsoe, bem como quatro outras
mulheres com idades entre 18 e 32 anos entre 1977 e 1979.
Em setembro de 1978, Alcala participou do The Dating Game, um programa de TV americano no
qual uma mulher solteira questionava três homens solteiros escondidos de sua vista antes de
selecionar um com base em suas respostas.
Alcala foi selecionado no final do show, mas a mulher disse mais tarde que decidiu não sair com
ele após uma conversa com ele nos bastidores, descrevendo-o como "assustador".
Em 2012, Alcala foi extraditado para Nova York depois de ser acusado de dois assassinatos
adicionais de 1971 e 1977. Ele se declarou culpado e foi condenado em Nova York a 25 anos de
prisão perpétua.
Funcionários da prisão estadual da Califórnia disseram em um comunicado no sábado que Alcala
era suspeito de estar envolvido em uma série de assassinatos adicionais.
Nenhum outro detalhe foi dado sobre sua morte.
Existem atualmente cerca de 700 presos no corredor da morte na Califórnia, mas o governador
Gavin Newsom ordenou uma moratória nas execuções.

AULA 09 – RELATIVE CLAUSES AND QUANTIFIERS 128


AFA 2024

PASSIVE VOICE, TAG


QUESTIONS AND IDIOMS

AULA 10

Teacher Andrea Belo

www.estrategiamilitares.com.br www.militares.estrategia.com
TEACHER ANDREA BELO

SUMÁRIO
INTRODUÇÃO 3

PASSIVE VOICE 4

TAG QUESTIONS 9

IDIOMS 12

QUESTÕES 23

GABARITO 55

QUESTÕES COMENTADAS 56

CONSIDERAÇÕES FINAIS 116

REFERÊNCIAS BIBLIOGRÁFICAS 117

TRADUÇÕES 119

AULA 10 – PASSIVE VOICE, TAG QUESTIONS AND IDIOMS 2


TEACHER ANDREA BELO

INTRODUÇÃO
Vamos, então, à nossa aula sobre alguns tópicos considerados complexos: passive voice
(voz passiva) e, também, Tag Questions. E ainda Idioms, as famosas expressões idiomáticas. A voz
passiva (passive voice) é usada de várias maneiras, quando queremos enfatizar a ação no lugar de
quem a praticou e assim, ter a possibilidade de omitir o autor. Geralmente, não sabemos quem é
o autor e queremos, desta forma, utilizar um tom mais impessoal. Você entenderá melhor no
decorrer da aula.
As Tag Questions, por sua vez, são aquelas perguntas do fim das frases, o famoso “né”, que
usamos para confirmar algo que estamos falando, que veremos um por um, já que em Inglês, há
regras específicas que, quando estudadas e compreendidas, fica mais fácil de entender para usar
e, claro, identificar e responder questões na prova.
Vamos, então, falar de expressões idiomáticas? Yessssssss, vamos à nossa aula sobre os
IDIOMS – um falso cognato que não significa idiomas e sim expressões idiomáticas ou, também
chamadas de provérbios, dizeres populares, presentes nos textos das provas, conforme veremos
nessa aula.
O idioms são conjuntos de palavras ou de frases cujo significado extrapola o significado
literal de cada termo separado. Podemos dizer que as expressões idiomáticas são muito mais do
que apenas a interpretação das palavras que as integram, já que são utilizadas para demonstrar
algo de caráter cultural.
As expressões idiomáticas estão arraigadas na cultura linguística, demonstram
características de diferentes formas de expressar de um determinado povo, da linguagem de um
determinado país, do uso de termos regionais e as variadas formas de verbalização de uma
sociedade.
E, se as expressões idiomáticas retratam traços culturais de certo grupo, de um povo, são
informações muito específicas e, por isso, de quase impossível tradução. Podemos até conseguir
equipará-las com algumas expressões equivalentes em nossa língua materna, mas uma expressão
idiomática não tem o mesmo sentido quando traduzida.
No caminho à aprovação, você vai resolver, durante a teoria e também no fim do material,
exercícios de vestibulares anteriores bem como exercícios inéditos e, essas questões irão ajudar
você a colocar em prática o que aprende a cada dia. Além disso, você avaliará seu conhecimento.
Vamos lá! Você consegue e estamos juntos!

AULA 10 – PASSIVE VOICE, TAG QUESTIONS AND IDIOMS 3


TEACHER ANDREA BELO

PASSIVE VOICE
Quando falamos em voz passiva, é preciso entender a voz ativa e, para estudar ambas,
estamos nos referindo à estrutura de frases, ou seja, a ordem das palavras.
Frases na active voice (voz ativa) são aquelas em que o sujeito que pratica alguma ação
está em evidência, enquanto na passive voice (voz passiva) o objeto que recebe a ação é que está
em evidência.
A voz ativa é o modo mais utilizado para ser claro e direto. Como é um sujeito que realiza
uma ação, o destaque é dado para o sujeito. Uma frase na voz ativa é composta por sujeito +
verbo auxiliar + verbo principal + objeto. Por exemplo, na frase:

Pedro washes the car – Pedro lava o carro.


Pedro é o sujeito da frase, to wash é o verbo principal, com o acréscimo de -es na terceira
pessoa do singular e the car é o objeto.
Como a frase está na voz ativa, queremos dar ênfase na ação de Pedro ao lavar o carro,
tornando-o como agente principal da frase.
Já na voz passiva, queremos dar ênfase para o objeto que está sofrendo a ação, e não para
o sujeito. Veja:

The car is washed by Pedro – O carro é lavado por Pedro.


Para formar a voz passiva em inglês, o objeto da voz ativa passa a ser o sujeito da voz
passiva, e o sujeito da voz ativa passa a ser o agente da voz passiva.
Como a frase está na voz passiva, desta vez queremos dar ênfase na ação de as roupas
estarem sendo lavadas.
A passive voice (voz passiva) é um tipo de construção frasal onde se expressa o que
acontece com o sujeito, sem obrigatoriamente enfatizá-lo.
Nós nos expressamos usando a voz ativa e, ao transformar em voz passiva, é que as frases
ficam repletas de novas estruturas, como veremos.
A voz passiva é geralmente utilizada em textos técnicos, artigos, anúncios e, como nos
vestibulares essas fontes são comuns, a voz passiva é facilmente encontrada em textos explorados
nas provas de vestibular.
VOZ ATIVA: I cleaned the garage this morning. – Eu limpei a garagem esta manhã.
VOZ PASSIVA: The garage was cleaned this morning by me – A garagem foi limpa esta manhã por mim.

A voz passiva pode ser utilizada em frases afirmativas, negativas e interrogativas. Sua
formação é expressa por um objeto (um substantivo) + verbo to be + particípio passado do verbo
principal + complemento da frase.
Veja, agora, cada uma delas em diferentes tempos verbais. Let’s go!!!

AULA 10 – PASSIVE VOICE, TAG QUESTIONS AND IDIOMS 4


TEACHER ANDREA BELO

PASSIVE VOICE VERB TENSES TRANSFORMATION


Simple Present: am/is/are + particípio
Affirmative Form: The house is painted by Tom.
Negative Form: The house isn’t painted by Tom.
Interrogative Form: Is the house painted by Tom?

Present Continuous: am/is/are being + particípio


Affirmative Form: The house is being painted by Tom.
Negative Form: The house isn’t being painted by Tom.
Interrogative Form: Is the house being painted by Tom?

Present Perfect: has/have been + particípio


Affirmative Form: The house has been painted by Tom.
Negative Form: The house hasn’t been painted by Tom.
Interrogative Form: Has the house been painted by Tom?

Simple Past: was/were + particípio


Affirmative Form: The house was painted by Tom.
Negative Form: The house wasn’t painted by Tom.
Interrogative Form: Was the house painted by Tom?

Past Continuous: was/were being + particípio


Affirmative Form: The house was being painted by Tom.
Negative Form: The house wasn’t being painted by Tom.
Interrogative Form: Was the house being painted by Tom?

Past Perfect: had been + particípio


Affirmative Form: The house had been painted by Tom.
Negative Form: The house hadn’t been painted by Tom.
Interrogative Form: Had the house been painted by Tom?

AULA 10 – PASSIVE VOICE, TAG QUESTIONS AND IDIOMS 5


TEACHER ANDREA BELO

Simple Future: will be + particípio


Affirmative Form: The house will be painted by Tom.
Negative Form: The house won’t be painted by Tom.
Interrogative Form: Will the house be painted by Tom?

Future Perfect: will have been + particípio


Affirmative Form: By next week, the house will have been painted by Tom.
Negative Form: By next week, the house won’t have been painted by Tom.
Interrogative Form: Will the house have been painted by next week by Tom?

Modal Verbs:
Modal verb + be + particípio

Teacher Andrea won the competition. / The competition was won by teacher Andrea.

Comparando as frases para facilitar a compreensão, diferentemente da voz ativa, com a


ênfase está no sujeito, que praticou a ação, na voz passiva se enfatiza o objeto, aquele que sofre
a ação expressa pelo verbo. Exatamente isso:
VOZ ATIVA: Teacher Andrea won the competition.
VOZ PASSIVA: The competition was won by teacher Andrea.

AULA 10 – PASSIVE VOICE, TAG QUESTIONS AND IDIOMS 6


TEACHER ANDREA BELO

Agora, usando o verbo to clean (limpar), conjugado no particípio, conforme pede a voz
passiva, veja frases em todos os tempos verbais com os devidos complementos.
Preparei para você uma tabela com exemplos para ficar claro o que falamos acima.

SUJEITO + "TO BE" (conjugado) + PARTICÍPIO + RESTO DA ORAÇÃO

SIMPLE PRESENT The house is cleaned every day.

PRESENT CONTINUOUS The house is being cleaned at the moment.

SIMPLE PAST The house was cleaned yesterday.

PAST CONTINUOUS The house was being cleaned last week.

PRESENT PERFECT The house has been cleaned since you left.

PAST PERFECT The house had been cleaned before they arrived.

FUTURE The house will be cleaned next week.

FUTURE CONTINUOUS The house will be being cleaned tomorrow.

PRESENT CONDITIONAL The house would be cleaned if the had visitors.

PAST CONDITIONAL The house would have been cleaned if it had been city.

INFINITIVE The house must be cleaned before we arrived.

A melhor forma de fixar os conteúdos, não só a voz passiva, como qualquer tópico
estudado, é fazendo exercícios em que o conteúdo está inserido. Então, vamos lá.
Em outro texto, de uma fonte usada em muitos vestibulares, vou mostrar como há uma
frase que remete à voz passiva e como isso poderia ter sido questionado em sua prova:
Australians are not known for their love of boat people.
They famously turned away a small group of Afghan refugees at the
height of the war and rather amusingly, ran a scare campaign featuring
crocodiles and sharks to deter would-be immigrants. But if global warming
continues at its current rate, neighbouring Pacific islands could be lost to
floods and Australia will be facing a new kind of intruder: climate refugees.
Although the Red Cross produced a report four years ago estimating
that 58 per cent of refugees are caused by environmental factors, no one has
made any attempt to tackle the issue. Oxford University’s Norman Myers
recently claimed that there could be an estimated 150 million environmental refugees within the next
50 years, and half of these could land on Australia’s doorstep. But the UN refuses to grant them
refugee status, and aid groups and environmentalists squabble over whose responsibility they are.
DAZED & CONFUSED July 2005

AULA 10 – PASSIVE VOICE, TAG QUESTIONS AND IDIOMS 7


TEACHER ANDREA BELO

Na frase “... the Red Cross produced a report…” na voz passiva, seria “A report was produced
by the Red Cross” e poderia ser uma das perguntas do seu vestibular, veja:
Questão: A frase “the Red Cross produced a report …” na posição de voz passiva, seria:

A ( ) A report had produced by the Red Cross.


B ( ) A report was producing by the Red Cross.
C ( ) A report will be produced by the Red Cross.
D ( ) A report were produced by the Red Cross.
E ( ) A report was produced by the Red Cross.

Comentários: Como sempre, devemos analisar cada alternativa, para encontrar o que
melhor se encaixa.
Nesse caso, para a sentença transformada em voz passiva, de acordo com o que vimos no
capítulo agora estudado, para verbos no passado simples (Past Simple), a frase é preenchida pelo
verbo to be relativo ao sujeito e o verbo principal no particípio, lembra?
Na letra A, afirma-se que report, antes no fim da frase como objeto, foi para o começo
como sujeito, o que está correto.
Porém, foi colocado o verbo have no passado (had) antes do particípio de produce
(produced), enquanto deveria ter sido usado, de acordo com a regra que estudamos, o verbo to
be no passado, que seria “was” antes de “produced”. Falsa.
Na letra B, afirma-se que report, antes no fim da frase como objeto, foi para o começo
como sujeito, o que está correto. Porém, foi colocado, junto ao verbo to be conjugado no passado
– was – o verbo produce no gerúndio (producing), enquanto deveria ter sido usado, de acordo
com a regra que estudamos, o verbo to produce no particípio - produced. Falsa.
Na letra C, afirma-se que report, antes no fim da frase como objeto, foi para o começo
como sujeito, o que está correto. Porém, foi colocada a forma de futuro para falar de algo que já
aconteceu, portanto, está no passado e assim, precisamos do verbo to be no passado e o verbo
principal produce no particípio - produced. E por isso é falsa.
Na letra D, afirma-se que report, antes no fim da frase como objeto, foi para o começo
como sujeito, o que está correto. Porém, foi colocado, o verbo to be conjugado no passado no
plural – were – mas a palavra report é singular (relatório) e então, deveria-se usar was ao invés
de were.
A letra E, afirma-se que report, antes no fim da frase como objeto, foi para o começo como
sujeito, o que está correto. E, em seguida, foi colocado, o verbo to be conjugado no passado no
singular – was – concordando com a palavra report é singular (relatório) e então, a estrutura está
correta, a concordância também. Alternativa certa.
Vamos, agora, aos estudos das Tag Questions.

AULA 10 – PASSIVE VOICE, TAG QUESTIONS AND IDIOMS 8


TEACHER ANDREA BELO

TAG QUESTIONS
Bom, você sabe o que é uma Tag Question? É uma pergunta curta no fim de uma frase,
feita para se certificar de que o que foi dito está certo. Em Português, costumamos perguntar no
fim da frase: “não é?”, “certo?” ou simplesmente “né?” para fazer essa confirmação.
Então, vamos aprender como formar essas perguntas curtas em inglês, que são tão usadas
no dia a dia para verificar se uma informação está correta e sempre estão nas provas.
A primeira coisa importante a se lembrar é que uma Tag Question é formada com o mesmo
verbo auxiliar ou modal da frase principal e segue o mesmo tempo verbal dela.
Então a estrutura das Tag Questions é a seguinte:
Verbo auxiliar ou verbo modal + not (se for negativa) + sujeito + ?
Veja alguns exemplos para cada verbo/estrutura:

VERBO TO BE:
She is intelligent, isn’t she?
Ela é inteligente, não é?
They are from Brazil, aren’t they?
Eles são do Brasil, não são?
You were in my house earlier, weren’t you?
Você estava na minha casa mais cedo, não estava?
He was a great father, wasn’t he?
Ele foi um ótimo pai, não foi?
O pronome I é uma exceção, conforme será explicado no fim deste capítulo.

HAVE:
You have met her before, haven’t you?
Você já a encontrou antes, não encontrou?
It has been raining a lot since last month, hasn’t it?
Tem chovido muito desde mês passado, não tem?
He had gone to another city, hadn’t he?
Ele havia ido para outra cidade, não é?
We had been good friends, hadn’t we?
Nós havíamos sido bons amigos, não é?

AULA 10 – PASSIVE VOICE, TAG QUESTIONS AND IDIOMS 9


TEACHER ANDREA BELO

WILL:
You will study Medicine, won’t you?
Você vai estudar Medicina, não vai?
We will go to the beach on Sunday, won’t we?
Nós iremos à praia no domingo, não iremos?

CAN:
She can speak Spanish, can’t she?
Ela consegue falar ESpanhol, não consegue?
The kids can play on the street, can’t they?
As crianças podem brincar na rua, não podem?

SHOULD:
He should be an actor, shouldn’t he?
Ele deveria ser ator, não deveria?
We should go out more often, shouldn’t we?
Nós deveríamos sair mais vezes, não deveríamos?

COULD:
I could help you, couldn’t I?
Eu poderia te ajudar, não poderia?
Those guys could leave us alone, couldn’t they?
Aqueles caras poderiam nos deixar em paz, não poderiam?

WOULD:
She would be an excellent mother, wouldn’t she?
Ela seria uma mãe excelente, não seria?
You would like a cup of coffee, wouldn’t you?
Você gostaria de uma xícara de café, não gostaria?

Atenção: quando não houver nenhum verbo auxiliar ou modal na frase e ela estiver no presente,
use DO ou DOES. Se estiver no passado, use DID, veja:
You work in a school, don’t you?
Você trabalha em uma escola, não trabalha?
He eats French fries, doesn’t he?
Ele come batata frita, não come?

AULA 10 – PASSIVE VOICE, TAG QUESTIONS AND IDIOMS 10


TEACHER ANDREA BELO

I said I love you, didn’t I?


Eu disse que te amo, não disse?
They went to the cinema, didn’t they?
Eles foram ao cinema, não foram?

Outro fator importante é que, assim como em Português, a tag question sempre será oposta à
frase principal. Então, se a primeira frase for positiva, a tag question será negativa. Mas se a
primeira frase for negativa, a tag question será positiva. Por exemplo:
You like chocolate, don’t you?
Você gosta de chocolate, não gosta?
(Frase principal positiva / tag question negativa.)
You don’t like broccoli, do you?
Você não gosta de brócolis, gosta?
(Frase principal negativa / tag question positiva.)

Você aprendeu agora que as tag questions utilizam o mesmo verbo auxiliar da frase principal,
certo? Existe uma exceção para esta regra, referente ao pronome I. Quando a frase for na primeira
pessoa do singular, a tag question será feita com o verbo to be “are”, não com o “am”.
Isto porque o am não pode ser contraído com o not. Então fica assim, veja:
I’m your best friend, aren’t I?
Eu sou seu melhor amigo, não sou?
I am late, aren’t I?
Eu estou atrasado, não estou?

Existem também as tag questions imperativas. Você deve se lembrar que uma frase no imperativo
é uma ordem e pode soar um tanto rude. Uma tag question pode então ser adicionada ao fim da
frase no imperativo para “amenizá-la”. O mais comum é usar WILL, não importando se a frase for
positiva ou negativa. Nesse caso, não exige uma resposta de fato, ela está ali só para deixar a frase
mais “educada”, mais polida. Veja exemplos:
Open the door, will you?
Abra a porta, sim?
Don’t forget to call me later, will you?
Não se esqueça de me ligar mais tarde, sim?
Turn off the lights, will you?
Apague as luzes, sim?
Don’t smoke in this room, will you?
Não fume neste local, sim?
Don’t tell anyone, will you?
Não conte para ninguém, sim?

AULA 10 – PASSIVE VOICE, TAG QUESTIONS AND IDIOMS 11


TEACHER ANDREA BELO

IDIOMS
Eu gosto sempre de dizer que aprender uma língua é muito mais do aprender um conjunto
de palavras. Esse é um dos motivos pelos quais, quando se estuda Inglês, é tão importante
abandonar a vontade que se tem de traduzir tudo o que lê.
Até porque eu já ofereci técnicas que ajudam você a entender o “todo” sem precisar
traduzir de fato.
Um exemplo importante, que comprova o quanto a tradução pode atrapalhar você, é
justamente o exemplo das expressões idiomáticas, que possuem sentido diferente do sentido das
palavras analisadas individualmente.
É correto afirmar que, uma expressão idiomática aparece quando uma frase assume
significado diferente daquele que as palavras teriam, se fossem analisadas isoladamente.
As provas se utilizam de artigos, notícias dos mais diversos tipos e, as expressões
idiomáticas se encontram, tanto no linguajar diário quanto no noticiário, nos anúncios de jornais,
no rádio, na TV, em textos de cunho político, científico, em filmes, em músicas, na literatura,
enfim, podem estar presentes em suas leituras no dia da sua prova.
As expressões idiomáticas são uma parte importante da comunicação formal ou informal,
escrita ou falada, e, o motivo que leva um falante ou um escritor, um autor de um texto a usar
uma expressão idiomática é o desejo de acrescentar, na interpretação daquela frase, algo que a
linguagem convencional não poderia suprir.
Também chamadas de provérbios, as expressões idiomáticas surgem com frequência e,
muitas vezes, é divertido comparamos as expressões de língua inglesa àquelas em nosso idioma.
Aprender a utilizar expressões idiomáticas é importante e, no seu caso, mais importante
ainda é aprendê-las e identificá-las nos textos da prova, pois você estará garantindo acertar certas
questões, que poucas pessoas acertariam quando envolve termos dessa natureza.
Uma expressão idiomática tem por função enriquecer a frase, ela pode reforçar ideias,
pode enfatizar um sentimento de alguém e pode, ainda, diminuir, amenizar o impacto que algum
termo possa causar, seja com humor ou com ironia. As expressões idiomáticas expressam ideias
de diferentes maneiras dentro de cada contexto.
Vale afirmar que, pelo sentido exclusivo que possuem, não há, para as expressões
idiomáticas, um significado concreto, como a maioria dos vocábulos que há em nosso vocabulário.
Para que as expressões sejam estudadas com eficácia, é necessário considerar o contexto
em que são produzidas, já que sempre estão associadas a situações que se relacionam à valores
culturais, conforme já expliquei.
Vamos estudá-las agora? Come on!
Alguns desses IDIOMS, são expressões idiomáticas que usam, em sua estrutura, palavras
similares às que usamos em nossas expressões em Português.
Bom, na maioria das vezes, as expressões são compostas por palavras diferentes e, quando
tentamos traduzir, fica totalmente sem sentido.

AULA 10 – PASSIVE VOICE, TAG QUESTIONS AND IDIOMS 12


TEACHER ANDREA BELO

Por exemplo, desejar boa sorte a alguém, antes de uma apresentação, além de “Good
luck”, podemos também dizer “break a leg”, a expressão idiomática que tem esse sentido.
Se você traduzir, significa “quebrar uma perna”, mas, é muito usado com essa intenção de
salientar que você deseja boa sorte.
Entre as expressões que assemelham à nossa língua, estão alguns exemplos:

A expressão “Antes tarde do que nunca”, usa as palavras “tarde”


e “nunca”, o que facilita para você se lembrar. A palavra “antes”,
em Inglês, que é “before”, não é usada no início da frase, mas, a
tradução de “Better late than never” - “Melhor tarde do que
nunca” proporciona a dedução equivalente: “Antes tarde do que
nunca”.

A expressão “De uma vez por todas”, usa o termo “por todas”, (for
all), o que também facilita para você fazer a conecção. “Once” é
uma vez, mas, ligando as ideias, “Uma vez por todas” nos leva a
compreender que se trata “De uma vez por todos(as)”.

A expressão “Para um bom entendedor, meia palavra basta”, o


idiom é quase igual escrito em outra ordem: “A word to the wise
is enough” – “Uma palavra para um sábio é suficiente”. Expressão
boa para se lembrar, caso apareça em sua prova.

AULA 10 – PASSIVE VOICE, TAG QUESTIONS AND IDIOMS 13


TEACHER ANDREA BELO

Para compreender bem a expressão “Estamos quites”, é preciso


saber como se fala “par ou ímpar” em Inglês – “Even or odd”.
Assim, a expressão é formada pela palavra “par” do par ou ímpar,
como se fosse “Estamos pares”. Interessante, não é?

A expressão “Fica para a próxima”, em Inglês, não usa as mesmas


palavras, mas usa termos que equivalem ao mesmo sentido:
“Maybe some other time” significa Talvez alguma outra hora” e
por isso também é uma expressão mais simples de ser
interpretada.

A expressão “Entre a cruz e a espada”, usa a preposição


“between”, (entre duas coisas), já que entre muitos, seria a
preposição “among”. Apesar disso, em Português, temos as
palavras cruz e espada. Em Inglês, a expressão se compões com
as palavras rocha (rock) e um lugar duro, como uma parede (hard
place). Diferente, não é?

Na expressão “Pavio curto”, há outras palavras para representar


o que seria um pavio curto, alguém que explode fácil, se enfurece
rapidamente: um temperamento curto – short temper. Também
poderia ser traduzido como pouca disposição, já que a palavra
“temper” pode ser temperamento, disposição, humor.

AULA 10 – PASSIVE VOICE, TAG QUESTIONS AND IDIOMS 14


TEACHER ANDREA BELO

A expressão “Quem não arrisca não petisca”, em Inglês, significa


nada arriscado, nada ganho – “Nothing ventured, nothing
gained”. E assim, apesar de estranha a tradução, representa que,
se não for arriscado, não se obtém o que se deseja.

Na expressão “É hora de encarar os fatos”, usa-se a palavra


música no lugar dos fatos, que seria “facts” e assim, fica: é hora
de encarar a música.

A expressão “No dia de São Nunca”, ou seja, um dia que não vai
acontecer, algo que não vai acontecer, em Inglês, é quando o
inferno congelar – “when hell freezes over”. Acredita-se que o
inferno seja um lugar quente e com bastante fogo e por isso, seria
difícil congelar onde há fogo, assim como fazer algo no dia de São
Nunca, que não existe.

A expressão “mamão com açúcar”, em Inglês, é um “pedaço de


bolo” – piece of cake, representando algo muito fácil de fazer,
uma tarefa muito fácil de realizar.

AULA 10 – PASSIVE VOICE, TAG QUESTIONS AND IDIOMS 15


TEACHER ANDREA BELO

A expressão “Uma vez na vida, outra na morte”, para representar


que algo é feito raramente ou quase nunca, em Inglês, é uma das
mais diferentes de todas: “Once in a blue moon”, que traduzida,
seria “uma vez na lua azul”. Isso porque, o efeito “lua azul” é raro
de se observar e só ocorre uma vez a cada dois anos e meio.
O termo é usado para descrever um acontecimento incomum.

A expressão “Chorar o leite derramado”, usa a palavra “leite”,


(milk), mas diz chorar (verbo cry) o leite derramado (spiled), como
se a pessoa ficasse tão triste que desperdiçou, que chorou. Viu
como essas diferenças culturais são interessantes?

A expressão “Acordar com o pé esquerdo” tem o verbo


“acordar/levantar”, (get up), mas não é com o pé esquerdo e sim
“do lado errado da cama (wrong side of the bed), querendo dizer
que a pessoa dormiu do lado em que não está acostumada e pode
ter acordado de mau humor por isso. Daí, se equivale a acordar
com o pé esquerdo.

A expressão “Cada macaco no seu galho”, em Inglês, “Every Jack


to his trade”, significa “Cada Jack em seu comércio”, ou seja, cada
pessoa em sua função, cada um encaixado naquilo que sabe fazer.
Interessante saber que em Português usa-se o macaco e em Inglês
o nome de uma pessoa: Jack.

AULA 10 – PASSIVE VOICE, TAG QUESTIONS AND IDIOMS 16


TEACHER ANDREA BELO

Para dizer “Isso são águas passadas”, para expressar algo que
ficou no passado, que não vale a pena falar sobre, que é melhor
ser esquecido, em Inglês, se diz são “apenas águas embaixo da
ponte”: just water under the bridge”, significando algo que foi
levado pela correnteza, que deveria ter sido deixado para trás.

A expressão “varar a noite” ou, algumas pessoas dizem “virar a


noite” ou então “passar a noite em claro, demonstrando que
alguém não dormiu, seja para trabalhar ou fazer qualquer
atividade realizada durante toda a noite, é queimar o óleo da
meia-noite: “burn the midnight oil” – porque antigamente, usava-
se lamparinas, reservatórios com um líquido combustível, no qual
se mergulhava um pavio que traspassava uma rodela de madeira
para acender e gerar luz.

Agora, veremos outras expressões que, são realmente totalmente diferentes na tradução,
nas palavras usadas, no sentido e, de difícil dedução do que possa ser, mas, caso esteja em sua
prova, você estudou e aprimorou seu vocabulário. Armazene as informações, ok? Attention!!!
Essas expressões idiomáticas que veremos agora são muito interessantes. São, as que mais
representam uma determinada cultura local, costumes e vocabulário utilizado por falantes da
língua inglesa em cada país em que vivem.
Vejamos exemplos dessas expressões e, em seguida, vamos visualizar expressões
idiomáticas em textos de provas anteriores para praticar, enriquecer vocabulário e saber como
identificar tais termos no dia da sua prova.

Para a expressão “Vire essa boca pra lá”, em Inglês, há duas


frases: morda sua língua - “bite your tongue” e suma com esse
pensamento/desapareça com essa ideia, algo assim: “perish your
thought”.

AULA 10 – PASSIVE VOICE, TAG QUESTIONS AND IDIOMS 17


TEACHER ANDREA BELO

A expressão “Dia sim, dia não” também é expressa de forma bem


diferente, já que “Every other day” significa “cada outro dia”
literalmente. Isso porque, em Inglês, para dizer, por exemplo, de
15 em 15 dias, se diz “every 15 days” – então o “every” é utilizado
nessas expressões de tempo. Got it?

A expressão “Beco sem saída” - It’s a catch 22, teve origem


baseada no famoso livro “Catch-22” (1961), de Joseph Heller, em
que escreveu sobre a Segunda Guerra, relatando que os pilotos
enfrentavam um dilema: Alegar insanidade ou não para recusar
as missões de bombardeio, de acordo com o regulamento 22:
“THERE IS A CATCH: "An airman would have to be crazy to fly more
missions, and if he was crazy he would be unfit to fly. Yet, if an
airman would refuse to fly more missions, this would indicate that
he is sane, which would mean that he would be fit to fly the
missions". Eis o impasse: Um piloto considerado louco, insano,
estava inapto para voar nas missões de bombardeio, mas, se recusasse as missões com sanidade,
teria que, obrigatoriamente, realizá-las.

IDIOMS IN YOUR TEST


Veremos agora várias expressões idiomáticas que já apareceram em textos de provas de
anos anteriores, destacadas e comentadas para aprimorar seus estudos. E, além de expressões
idiomáticas propriamente ditas, mostrarei também termos que, se tornaram, de certa forma,
tipos de expressões, derivações de verbos e outras classificações gramaticais, que não são gírias
e sim, adaptações da língua, formando locuções frasais.
Em uma prova de vestibular, por exemplo, apareceu a expressão idiomática “walking in
another’s shoes”, cuja tradução literal é “andar no sapato do outro” e equivale à expressão
“colocar-se no lugar do outro” em Português, idiom que já esteve presente em outras provas de
vestibulares variados, observe:
THURSDAY, DECEMBER 16, 2010. Newsweek Article: Bullying and Empathy (Kate Altman, M.S.)

AULA 10 – PASSIVE VOICE, TAG QUESTIONS AND IDIOMS 18


TEACHER ANDREA BELO

The effective_______ of such programs is unclear at this point, and experts are divided on
whether it makes more sense to offer the programs to young children (elementary school age) or
older children (middle school age) (both, is probably the answer). High school kids are simply
difficult to reach logistically, since they all have different schedules all day. Unsurprisingly, some
experts have found that the most important component to empathy training is to include the
parents.
In assessing these programs and the broader issues of empathy-training and bullying, there
are multiple factors to consider and no clear answers. First of all, empathy is one of the most
difficult and least-understood skills we can develop – adults and kids alike. Empathy is the process
of viewing and understanding the world through another’s experience, and it is often confused
with sympathy, which is, essentially, compassion and lacks the “walking in another’s shoes”
component (which is not to say it is not an admirable trait, it’s just different from empathy).
Developmentally, children may not be able to truly understand and practice empathy until they
are closer to the pre-teen years, but introducing the concept early and often is a good primer for
its later development.
Another big question to consider: are programs focused on empathy simply band-aids on
much larger, more systemic problems? Why are kids bullying other kids in the first place? What
family issues, societal issues, educational issues, are contributing to the need/urge to humiliate
and attack other children for some sort of personal gain and satisfaction? My guess is that for
many kids, participating in a brief (or even a few brief) empathy-skills seminars simply is not
enough, and will not get at the root(s) of the problem(s), no matter how young they are when the
programs begin.
Newsweek offers an article on how schools are using empathy-training programs in an
effort to reduce bullying in schools: http://www.newsweek.com/2010/12/15/canschools-teach-
kids-not-to-bully.html
WALK IN ANOTHER’S SHOES
O texto está falando sobre bullying e, na linha em que há a expressão idiomática,
“...compassion and lacks the walking in another’s shoes component...”, diz que uma das coisas
que estão faltando é o componente “colocar-se no lugar do outro”.
Agora, veremos o índice da revista Time, explorado em muitas provas, com a expressão
“make a comeback”, que, ao invés de ser “fazer um retorno”, como parece, é “dar a volta por
cima”, que também pode aparecer.
No lugar em que a expressão está encaixada, que diz: “Charles Dickens is making a
comeback – as a fictional character”, demonstra que, o artigo da página 52 será sobre o
personagem dando a volta por cima, veja:
MAKE A COMEBACK

AULA 10 – PASSIVE VOICE, TAG QUESTIONS AND IDIOMS 19


TEACHER ANDREA BELO

O próximo texto que vou mostrar a você, também é da revista Time, explorado com a
expressão “From that point on”, que parece ser algo sem sentido como “deste ponto dentro”,
mas significa, de fato “daqui por diante”.
Na linha 13, onde a expressão está encaixada, que diz: “From that point on, there’s no
reason to think computers would stop getting more powerful.” – Daqui por diante, não há razões
para pensar que os computadores parariam de se tornar mais poderosos.
FROM THAT POINT ON

Thursday, Feb. 10, 2011 2045:


The Year Man Becomes Immortal
By Lev Grossman
(...), Kurzweil believes that we’re approaching a moment when computers will become
intelligent, and not just intelligent but more intelligent than humans. When that happens,
humanity – our bodies, our minds, our civilization – will be completely and irreversibly
transformed. He believes that this moment is not only inevitable but imminent. According to his
calculations, the end of human civilization as we know it is about 35 years away.

AULA 10 – PASSIVE VOICE, TAG QUESTIONS AND IDIOMS 20


TEACHER ANDREA BELO

Computers are getting faster. Everybody knows that. Also, computers are getting faster
faster – that is, the rate at which they’re getting faster is increasing.
True? True.
So if computers are getting so much faster, so incredibly fast, there might conceivably come
a moment when they are capable of something comparable to human intelligence. Artificial
intelligence. All that horsepower could be put in the service of emulating whatever it is our brains
are doing when they create consciousness – not just doing arithmetic very quickly or composing
piano music but also driving cars, writing books, making ethical decisions, appreciating fancy
paintings, making witty observations at cocktail parties.
If you can swallow that idea, and Kurzweil and a lot of other very smart people can, then
all bets are off. From that point on, there’s no reason to think computers would stop getting more
powerful. They would keep on developing until they were far more intelligent than we are. Their
rate of development would also continue to increase, because they would take over their own
development from their slowerthinking human creators. Imagine a computer scientist that was
itself a super-intelligent computer. It would work incredibility quickly. It could draw on huge
amounts of data effortlessly. It wouldn't even take breaks to play Farmville. (...)
http://www.time.com/printout/0,8816,2048138,00.html. Acesso em 07/04/2011. Adaptado.

Sobre o próximo texto, da fonte The Guardian, bastante usada em provas, foi usada a
expressão “by accident”, na linha 19 e não tem relação nenhuma com acidente e significa “por
acaso”. Inclusive, muito usada em textos, de forma geral.
BY ACCIDENT

[…] A picture of Brighton beach in 1976, featured in the Guardian a few weeks ago,
appeared to show an alien race. Almost everyone was slim. I mentioned it on social media, then
went on holiday. When I returned, I found that people were still debating it. The heated discussion
prompted me to read more. How have we grown so fat, so fast? To my astonishment, almost
every explanation proposed in the thread turned out to be untrue. […] The obvious explanation,
many on social media insisted, is that we’re eating more. […]
So here’s the first big surprise: we ate more in 1976. According to government figures, we
currently consume an average of 2,130 kilocalories a day, a figure that appears to include sweets
and alcohol. But in 1976, we consumed 2,280 kcal excluding alcohol and sweets, or 2,590 kcal
when they’re included. I have found no reason to disbelieve the figures. […]
So what has happened? The light begins to dawn when you look at the nutrition figures in
more detail. Yes, we ate more in 1976, but differently. Today, we buy half as much fresh milk per
person, but five times more yoghurt, three times more ice cream and – wait for it – 39 times as
many dairy desserts. We buy half as many eggs as in 1976, but a third more breakfast cereals and
twice the cereal snacks; half the total potatoes, but three times the crisps. While our direct
purchases of sugar have sharply declined, the sugar we consume in drinks and confectionery is
likely to have rocketed (there are purchase numbers only from 1992, at which point they were
rising rapidly. Perhaps, as we consumed just 9kcal a day in the form of drinks in 1976, no one

AULA 10 – PASSIVE VOICE, TAG QUESTIONS AND IDIOMS 21


TEACHER ANDREA BELO

thought the numbers were worth collecting.) In other words, the opportunities to load our food
with sugar have boomed. As some experts have long proposed, this seems to be the issue.
The shift has not happened by accident. As Jacques Peretti argued in his film The Men Who
Made Us Fat, food companies have invested heavily in designing products that use sugar to bypass
our natural appetite control mechanisms, and in packaging and promoting these products to
break down what remains of our defenses, including through the use of subliminal scents. They
employ an army of food scientists and psychologists to trick us into eating more than we need,
while their advertisers use the latest findings in neuroscience to overcome our resistance.
They hire biddable scientists and thinktanks to confuse us about the causes of obesity.
Above all, just as the tobacco companies did with smoking, they promote the idea that weight is
a question of “personal responsibility”. After spending billions on overriding our willpower, they
blame us for failing to exercise it.
To judge by the debate the 1976 photograph triggered, it works. “There are no excuses.
Take responsibility for your own lives, people!” “No one force feeds you junk food, it’s personal
choice. We’re not lemmings.” “Sometimes I think having free healthcare is a mistake. It’s
everyone’s right to be lazy and fat because there is a sense of entitlement about getting fixed.”
The thrill of disapproval chimes disastrously with industry propaganda. We delight in blaming the
victims.
More alarmingly, according to a paper in the Lancet, more than 90% of policymakers
believe that “personal motivation” is “a strong or very strong influence on the rise of obesity”.
Such people propose no mechanism by which the 61% of English people who are overweight or
obese have lost their willpower. But this improbable explanation seems immune to evidence.
Perhaps this is because obesophobia is often a fatly-disguised form of snobbery. In most
rich nations, obesity rates are much higher at the bottom of the socioeconomic scale. They
correlate strongly with inequality, which helps to explain why the UK’s incidence is greater than
in most European and OECD nations. The scientific literature shows how the lower spending
power, stress, anxiety and depression associated with low social status makes people more
vulnerable to bad diets.
Just as jobless people are blamed for structural unemployment, and indebted people are
blamed for impossible housing costs, fat people are blamed for a societal problem. But yes,
willpower needs to be exercised – by governments.
Yes, we need personal responsibility – on the part of policymakers. And yes, control needs
to be exerted – over those who have discovered our weaknesses and ruthlessly exploit them.
Adaptado de: <https://www.theguardian.com/commentisfree/2018/aug/15/age-of-obesity-shaming-overweight-people/>. Acesso em: ago. 2018.

No parágrafo em que está inserida, diz “The shift has not happened by accident” – A
transição não aconteceu por acaso e em seguida, dará a explicação do motivo pelo qual não pode
se dizer que foi por acaso: que alguém perguntou algo e investiu etc. Vamos aos exercícios para
praticar exercícios variados inseridos nos textos de vestibular de anos anteriores. Let’s go!

AULA 10 – PASSIVE VOICE, TAG QUESTIONS AND IDIOMS 22


TEACHER ANDREA BELO

QUESTÕES
Você vai, agora, responder questões selecionadas de provas já realizadas em anos
anteriores. Depois, como em todas as nossas aulas, haverá o gabarito e as questões comentadas.
Vamos começar com questões AFA, de acordo coma sua instituição escolhida e depois,
vamos treinar de outras Carreiras Militares, para adquirir experiência e treinar vocabulário.
QUESTÕES AFA
Industries shut, Ganga water quality improves
With industries shutdown and people confined to their homes for days following the imposition
of the 21-day lockdown, the water quality of Ganga river has significantly improved, according to
a IITBHU professor.
Dr P K Mishra, a professor at the institute's Chemical Engineering department, attributed the
improvement to the lack of industrial waste polluting the waters.
"One-tenth of the pollution in Ganga river comes from industries. As industries are shut due to
lockdown, situation has become better. We have seen 40-50 per cent improvement in the Ganga.
It is a significant development," Mishra told news agency ANI. With rainfall on March 15 and 16,
the Ganga's water level has also increased, he said, adding that this meant the river's cleaning
capacity has also increased.
"There is a lot of difference between when we see the water of the Ganga river today and what
used to be earlier. Today, the water looks clean," ANI quoted a local as saying. He added that
people are also not taking bath at the ghats — a common ritual among locals there. "If this is the
condition in 10 days, then I believe Ganga river will be like it used to be earlier," he added. Another
local said nobody would have thought that the lockdown would have such an impact on
environment.
With several countries under lockdown to contain the spread of coronavirus, the global pollution
levels have also dropped significantly. In India, pollution levels have dipped as the lockdown in
megacities has kept cars off the road and closed factories. New Delhi, which regularly has
unhealthy air conditions, has now seen its AQI falling below 95 — a big reduction from its monthly
average of 161 from March 2019. Mumbai, too, has witnessed a similar reduction in pollution.
Due to the improvement in air quality following in India, Dhauladhar range, which is part of a
Himalayan chain of mountains in Himachal Pradesh, has now become visible from Jalandhar in
Punjab, The mountain rises from Kangra and Mandi.
Adapted from (https://www.msn.com/en-in/news/environment/industries-shut-ganga-water-quality-improves/ar-BB12beCN)

QUESTÃO 01 (AFA/INÉDITA) – What’s the meaning of the word “chain” in the last paragraph?
A) Lots
B) Rise
C) Some
D) String
E) Valley

AULA 10 – PASSIVE VOICE, TAG QUESTIONS AND IDIOMS 23


TEACHER ANDREA BELO

QUESTÃO 02 (AFA/INÉDITA) – According to the text, which option is correct?


A) Industries are responsible for 40-50% of Ganga river pollution.
B) The river is cleaner just because people stopped bathing at the ghats.
C) The Ganga river water is 40-50% better as a result of the lockdown.
D) Since Ganga river water is better, coronavirus was a good thing after all.
E) India will now keep industries shut permanently so the river water keep clean.

QUESTÃO 03 (AFA/INÉDITA) – According to the text, which option completes the sentence
below correctly?
The text says it was a good thing for the river water _____________.
A) to shut industries
B) to have the economy paralized
C) that coronavirus started infecting people in India
D) that New Delhi has better air conditions
E) that Dhauladhar range has now become visible from Jalandhar in Punjab

QUESTÃO 04 (AFA/INÉDITA) – “a big reduction from its monthly average of 161 from March
2019” in the second last paragraph, the word in bold refers to
A) AQI
B) Air conditions
C) New Delhi
D) Factories
E) Mumbai

QUESTÃO 05 (AFA/INÉDITA) – Which option completes the paragraph below correctly?


Around the 18th and 19th centuries two main ideas about dreams _______ (become) popular.
One was the idea that the things we see in our dreams are things our conscious mind is hiding
from us. However, the opposite idea _______ (say) that while we’re asleep, the brain _______
(organise) memories and events from the day. Dreams are just a random collection of these
thoughts, but we _______(try) to make a story from them when we wake up.
A) become / said / organises / try
B) had become / says / organised / tried
C) became / said / organised / tried
D) become / says / organises / try
E) became / said / organises / try

AULA 10 – PASSIVE VOICE, TAG QUESTIONS AND IDIOMS 24


TEACHER ANDREA BELO

QUESTÃO 06 (AFA/INÉDITA) – Mark the CORRECT alternative.


“With several countries under lockdown to contain the spread of coronavirus” (second last
paragraph)
If a country is under lockdown, it means that
A) it is locked.
B) it is in a state of isolation as a security measure.
C) everyone in that country is a prisoner.
D) this country is under a dictatorship.
E) there’s no way people can talk to each other.

QUESTÃO 07 (AFA/INÉDITA) – Mark the alternative which has the sentence below correctly
reported.
Dr P K Mishra attributed the improvement to the lack of industrial waste polluting the waters.
The author
A) replied: “Dr P K Mishra attributed the improvement to the lack of industrial waste polluting
the waters.”
B) said that Dr P K Mishra had attributed the improvement to the lack of industrial waste
polluting the waters.
C) asked the readers if Dr P K Mishra attributed the improvement to the lack of industrial waste
polluting the waters.
D) asked if Dr P K Mishra attributed the improvement to the lack of industrial waste polluting
the waters.
E) said Dr P K Mishra attributed the improvement to the lack of industrial waste polluting the
waters.

QUESTÃO 08 (AFA/INÉDITA) – In the third paragraph, the text DOESN’T


A) say how much industries activities affect the quality of Ganga river water.
B) say why the industries are shut and that the river water has been better since then.
C) say how better the river water is using numbers for that.
D) explain the process through which rainfall increases the cleaning capacity of the river water.
E) say rainfall boosted the river water improvement.

AULA 10 – PASSIVE VOICE, TAG QUESTIONS AND IDIOMS 25


TEACHER ANDREA BELO

QUESTÃO 09 (AFA/INÉDITA) – In the sentence “Mumbai, too, has witnessed a similar reduction
in pollution” (second last paragraph), the underlined word means
A) produced.
B) was a victim of.
C) manufactured.
D) was responsible for.
E) saw an event take place.

QUESTÃO 10 (AFA/INÉDITA) – “Due to the improvement in air quality following in India,


Dhauladhar range, which is part of a Himalayan chain of mountains in Himachal Pradesh, has
now become visible from Jalandhar in Punjab, The mountain rises from Kangra and Mandi.” (last
paragraph). The highlighted expression is closest in meaning to
A) otherwise.
B) thereafter.
C) beacause of.
D) moreover.
E) wherever.

QUESTÕES COLÉGIO NAVAL


Who was Steve Jobs?
Do you have an iPad, iPod, iPhone, a smartwatch or a Mac computer? If
you don’t, you probably know someone who has one (or wants one)!
Steve Jobs made the company – Apple – that created these things that are
now such an important part of the lives of millions of people.
He was born on February 24, 1955, in San Francisco, California. When he
was a boy, he had a special hobby: he liked to take apart televisions and
put them back together again. He was a very good student in school and
even skipped a grade. After he finished grade four, he went into grade six,
and in 1972, when he was seventeen, he graduated from high school. He
then began his studies in Reed College, in Portland, Oregon, but dropped
out after six months. Deciding to quit was not at all easy (...)
After Steve Jobs returned to the United States, he found a job as a video
game designer at a company called Atari, and two years after that, in 1976, when he was only
twenty-one years old, he created his own company – Apple Computer – with his friend, Steve
Wozniak. Steve Jobs later got married in 1991 and had four children. Unfortunately, doctors
discovered that he had cancer in 2003 and in 2011 he died at the age of only 56.
https://www.allthingstopics.com/uploads/2/3/2/9/23290220/7300865_orig.png

AULA 10 – PASSIVE VOICE, TAG QUESTIONS AND IDIOMS 26


TEACHER ANDREA BELO

QUESTÃO 01 (COLÉGIO NAVAL/INÉDITA) – Which Read the extract from the text:
“If you don’t, you probably know someone who has one (or wants one)”
Mark the option that can replace the term one.
a) i-phone
b) computer
c) someone
d) you
e) company

QUESTÃO 02 (COLÉGIO NAVAL/INÉDITA) – What’s the correct question referring to the


underlined information?
“these things that are now such an important part of the lives of millions of people”
a) How important these things are for people’s lives?
b) How important are those things for people’s lives?
c) What’s the importance of the things mentioned?
d) What’s the importance of all the things at all?
e) How important are things and people’s lives?

QUESTÃO 03 (COLÉGIO NAVAL/INÉDITA) – Read the extract from the text


“He liked to take apart televisions and put them back together again”
The pronoun “them” refers to:
a) equipments in general
b) Apple equipments
c) Steve Jobs’ hobbies
d) televisions
e) things to take apart

QUESTÃO 04 (COLÉGIO NAVAL/INÉDITA) – Read the first sentence of the text


“Do you have an iPad, iPod, iPhone, a smartwatch or a Mac computer?”
What’s the plural form of the sentence?
a) Do you have any iPad, iPod, iPhone, smartwatches or any Mac computer?
b) Do you have iPads, iPods, iPhones, smartwatches or Mac computers?
c) Do you have any iPads, iPods, iPhones, smartwatches or Mac computers?
d) Do you have some iPads, iPods, iPhones, smartwatches or Mac computers?
e) Do you have iPads, iPods, iPhones, a smartwatchs or a Mac computers?

AULA 10 – PASSIVE VOICE, TAG QUESTIONS AND IDIOMS 27


TEACHER ANDREA BELO

Texto para questões 05 a 08


FIVE WAYS TO GET MORE FIBRE IN YOUR DIET
Roughage helps reduce the risk of heart disease and bowel
cancer, yet few of us eat enough of it. Here’s how to up your
intake.
Fibre, or roughage, refers to indigestible carbohydrates. A fibre-
rich diet is linked to health benefits including a reduced risk of
heart disease and bowel cancer. While UK guidelines say adults
should get 30g a day, fewer than one in 10 meet this goal.
Popular low-carb diets may be a reason why. Understanding
what is in your food _____ help: a typical apple contains 2-3g of fibre, a sesame bagel about 4g.
Jo Greening, a spokesperson for the British Dietetic Association (BDA), says it is worth checking
the labels, as different brands have different levels of fibre.

QUESTÃO 05 (COLÉGIO NAVAL/INÉDITA) – According to the text, a fibre diet


a) is linked to a lot of benefits
b) is linked to heart disease risk reduce
c) is linked to a few benefits
d) is linked to all health benefits
e) is linked to cancer disease prevention

QUESTÃO 06 (COLÉGIO NAVAL/INÉDITA) – “… yet few of us eat enough of it. Here’s how to up
your intake.” The underlined word can be substituted for
a) even
b) but
c) besides
d) otherwise
e) then

QUESTÃO 07 (COLÉGIO NAVAL/INÉDITA) – The word bowel (line 2) means in the text
a) gut
b) chin
c) elbow
d) bladder
e) thigh

AULA 10 – PASSIVE VOICE, TAG QUESTIONS AND IDIOMS 28


TEACHER ANDREA BELO

QUESTÃO 08 (COLÉGIO NAVAL/INÉDITA) – The space in the text, “Understanding what is in your
food _____ help.” Can be completed with
a) should
b) could
c) can
d) ought to
e) must

Texto para questões de 09 e 10

A solicitor engaged by former members of the Supreme Horse Racing Club


said on Thursday evening he was “very optimistic” the 29 horses it has
hitherto owned, including the top-class chaser Kemboy, will be able to race
this season.
The Thurles-based Patrick Kennedy was speaking after the ruling body of
Irish horse racing in effect kicked SHRC out of the sport by declaring it was
no longer “permitted to be owners or part-owners of racehorses”.
The announcement from Horse Racing Ireland represents the moment
when its patience expired over the club’s repeated failure to provide answers to its questions.
Following complaints from club members about alleged irregularities in the way SHRC was run,
HRI got involved this summer and its concerns have not been allayed.

QUESTÃO 09 (COLÉGIO NAVAL/INÉDITA) – Mark the incorrect statement according to the text
a) Um advogado entrou para o Supreme Horse Racing Club.
b) Um membro do Supreme Horse Racing Club está muito engajado no projeto.
c) Foi incluído um advgado para o Supreme Horse Racing Club.
d) O Supreme Horse Racing Club incluiu um advogado.
e) Um advogado foi contratado por membros do Supreme Horse Racing Club.

QUESTÃO 10 (COLÉGIO NAVAL/INÉDITA) – In the third paragraph“… Following complaints from


club members about alleged irregularities in the way SHRC was run, …”
The verb “run” refers to:
a) in a hurry irregularities
b) in a hurry action
c) something urgent
d) a kind of function
e) the behave of something

AULA 10 – PASSIVE VOICE, TAG QUESTIONS AND IDIOMS 29


TEACHER ANDREA BELO

QUESTÕES EAM
QUESTÃO 01 (EAM/INÉDITA) – Use the verbs in parentheses to complete the following
statements.
I. __________ (go) there! It’s dangerous!
II. Are you ready __________ (go) home?
III. She never __________ (go) to the beach in the summer.
IV. I guess they __________ (go) away tomorrow.
V. Mr. Allen __________ (go) to a party two days ago.
VI. He __________ (go) to school right now.
Now mark the option which completes them respectively.
a) Not go / go / go / ’ll go / goes / going
b) Don’t go / to go / goes / ’ll go / went / ’s going
c) Don’t go / to go / gos / go / went / is going
d) Not go / to go / goes / will go / will go / is going
e) Don’t go / go / goes / went / went / going

QUESTÃO 02 (EAM/INÉDITA) – Look at the picture below.

Which option correctly describes the picture above? Consider that the action takes place in the
future.
a) Mark and Chris went to the beach.
b) Mark and Chris will go skiing.
c) Mark and Chris are going to go to the beach.
d) Mark and Chris go to the beach.
e) Mark and Chris went skiing.

AULA 10 – PASSIVE VOICE, TAG QUESTIONS AND IDIOMS 30


TEACHER ANDREA BELO

QUESTÃO 03 (EAM/INÉDITA) – Fill in the blanks with the correct prepositions of time
respectively.
I. I never go to bed late __________ night.
II. They have karate classes __________ Wednesdays.
III. My birthday is __________ April.
IV. My birthday is __________ April 7.
a) at / on / in / on
b) at / on / in / in
c) at / at / on / on
d) in / at / in / in
e) on / on / in / in

QUESTÃO 04 (EAM/INÉDITA) – Read the dialog and mark the right option to fill the gaps
respectively.
A: Rick, look! Is __________ wallet here with me __________?
B: Uh... No, __________ is not __________. Maybe you should ask Lisa. It definitely looks like
__________ wallet.
a) this / your / this / mine / hers
b) these / yours / that / my / her
c) that / yours / that / mine / her
d) this / yours / this / my / hers
e) this / yours / that / mine / her

QUESTÃO 05 (EAM/INÉDITA) – Look at the picture below.

Adapted from https://englishoutofthebox.blogspot.com/2019/01/what-do-you-know-about-modal-verbs.html

AULA 10 – PASSIVE VOICE, TAG QUESTIONS AND IDIOMS 31


TEACHER ANDREA BELO

In the picture, “can” is used to express


a) an ability.
b) a request.
c) an order.
d) a possibility.
e) a prediction.

QUESTÕES EEAR
Texto para questões de 01 a 04
(Título omitido propositalmente)

QUESTÃO 01 (EEAR/INÉDITA) – Choose the most appropriate title for the text:
a) How to teach English.
b) Teaching English in Brazil.
c) Oportunities and flexibility.
d) Professional oportunities.
e) How to deal with English as a language.

QUESTÃO 02 (EEAR/INÉDITA) – In the sentence “Teaching English is a rewarding profession that


can enable…”, rewarding means all those words, except:
a) profitable.
b) rad.
c) fruitful.
d) tip top.
e) vain.

AULA 10 – PASSIVE VOICE, TAG QUESTIONS AND IDIOMS 32


TEACHER ANDREA BELO

QUESTÃO 03 (EEAR/INÉDITA) – Read the extract from the text


“It will give you the opportunity to learn valuable skills…”
The pronoun “it” refers to the:
a) rewarding profession.
b) fascinating places.
c) problem solving.
d) English.
e) the world.

QUESTÃO 04 (EEAR/INÉDITA) – According to the text, choose the correct statement


a) Flexibility is one of the advantages of teaching English.
b) Communication is one of the advantages of trading lore in English.
c) Problem solving is one of the advantages of trading lore in English.
d) Experience is one of the advantages of teaching English.
e) Flexibility is one of the advantages of learning English.

Texto para questões 05 a 08


Taal: The 'very small but dangerous volcano'

Over the past few days, it's begun spewing lava, triggering earthquakes and emitting huge plumes
of ash that have spread across the island of Luzon and beyond.
Scientists fear a bigger "hazardous eruption" is imminent. Taal is tiny, as volcanoes go, but it has
been deadly before. And according to Renato Solidum, the head of the Philippines' Institute of
Volcanology and Seismology (Philvolcs), it is "very small but a dangerous volcano". "Taal volcano
is a baby volcano sitting within a much bigger caldera volcano," said Ben Kennedy, associate
professor of physical volcanology at the University of Canterbury in New Zealand. The entire
Volcano Island has been marked as a permanent danger zone by Phivolcs.
BBC News (Jan/2020)

AULA 10 – PASSIVE VOICE, TAG QUESTIONS AND IDIOMS 33


TEACHER ANDREA BELO

QUESTÃO 05 (EEAR/INÉDITA) – According to the text,


a) Taal is small and very dangerous.
b) Taal is huge and menacing.
c) Taal is tiny and sheltered.
d) Taal is huge and hazardous.
e) Taal is huge and treacherous.

QUESTÃO 06 (EEAR/INÉDITA) – “… it's begun spewing lava, triggering earthquakes.”


The underlined words mean
a) deliberate and raze.
b) deliberate and defeat.
c) deliberate and ruin.
d) deliberate and tear down.
e) deliberate and incite.

QUESTÃO 07 (EEAR/INÉDITA) – What is the question the author refers when he says: “… but it
has been deadly before”
a) Was it always dangerous?
b) Is it considered really dangerous?
c) Is it dangerous?
d) Has it always been dangerous?
e) Has it caused any danger before?

QUESTÃO 08 (EEAR/INÉDITA) – What kind of text is it?


a) Jornalistic biography.
b) Journalistic letter.
c) News nature tale.
d) Nature book review.
e) An article.

AULA 10 – PASSIVE VOICE, TAG QUESTIONS AND IDIOMS 34


TEACHER ANDREA BELO

Texto para questões de 09 e 10


A solicitor engaged by former members of the Supreme Horse Racing
Club said on Thursday evening he was “very optimistic” the 29 horses
it has hitherto owned, including the top-class chaser Kemboy, will be
able to race this season.
The Thurles-based Patrick Kennedy was speaking after the ruling body
of Irish horse racing in effect kicked SHRC out of the sport by declaring
it was no longer “permitted to be owners or part-owners of
racehorses”.
The announcement from Horse Racing Ireland represents the moment when its patience expired
over the club’s repeated failure to provide answers to its questions.
The Guardian – November 14th

QUESTÃO 09 (EEAR/INÉDITA) – According to the text, the term solicitor means


a) A lawyer for the Supreme Horse Racing Club.
b) A new member to manage the Horse Racing Club.
c) Supreme Horse Racing Club’s new principal.
d) A candidate for the next Supreme Horse Racing Club elections.
e) A administrator for the Supreme Horse Racing Club.

QUESTÃO 10 (EEAR/INÉDITA) – In the text, the sentence “… was speaking after the ruling body
of Irish horse racing in effect, …” means
a) after confirming horses in force.
b) after making sure all horses were effective.
c) after getting new information about the horses.
d) after reading the horses legislation.
e) after making sure abou the racing effects.

QUESTÕES EFOMM
Read the text and answer questions 01, 02 and 03.
Netflix edits Squid Game phone number after woman deluged with calls
Popular drama’s use of real South Korean number leads to thousands of prank calls and texts
Netflix has edited out a phone number that appears ___ its hit series Squid Game after a South
Korean woman and others who use similar combinations were deluged with calls – with some
callers even asking to join the show’s life-or-death games.

AULA 10 – PASSIVE VOICE, TAG QUESTIONS AND IDIOMS 35


TEACHER ANDREA BELO

The South Korea-made production has topped Netflix popularity charts in 90 countries since its
launch last month and is on track to become its most watched series ever.
The nine-episode drama, which addresses widening economic inequality, involves hundreds of
cash-strapped people competing ___ children’s games to win the final reward of 45.6bn Korean
won (£28m), with the losers killed in uncompromising and violent ways.
To take part, contestants have to call a number on a business card printed with symbols. But while
film and television makers usually use fake numbers in such circumstances, adding 010 – the
standard prefix ___ South Korean mobiles – to the eight digits on the card generated a real phone
number.
It belongs to a South Korean woman who said she received thousands of calls and text messages
to her phone “to the point that it’s hard for me to go on with daily life”.
(Adapted from https://www.theguardian.com/media/2021/oct/07/netflix-edits-squid-game-phone-number-woman-deluged-calls)

QUESTÃO 01 (EFOMM/INÉDITA) – According to the text, which option is correct?


(A) Netflix edited the phone number that appears in Squid Game to be a real number
(B) The new South Korean production has achieved stratospheric success
(C) Usually, film and television producers use real phone numbers
(D) The owner of the number accidentally released in the series did not suffer any kind of impact
(E) Only one woman suffered from the exposure of her real phone number

QUESTÃO 02 (EFOMM/INÉDITA) – Which is the correct option to complete the gaps in the text?
(A) In / in / for
(B) In / on / for
(C) On / in / for
(D) In / in / at
(E) In / on / at

QUESTÃO 03 (EFOMM/INÉDITA) – What’s the meaning of the word “deluged” in the title?
(A) Deluded
(B) Shocked
(C) Overwhelmed
(D) Rested
(E) Empty

AULA 10 – PASSIVE VOICE, TAG QUESTIONS AND IDIOMS 36


TEACHER ANDREA BELO

Read the text and answer questions 04 and 05.


New tool identifies groups most at-risk from Covid after vaccination
While the risk of severe Covid-19 after vaccination remains low, some people remain more
vulnerable than others, research shows
The immunosuppressed and those with dementia, Parkinson’s or chronic disorders such as kidney
disease are still at a greater risk of hospitalisation or death from Covid after vaccination compared
to the rest of the population, new research shows.
The QCovid tool developed by scientists at the University of Oxford shows that while the risk of
severe Covid-19 after vaccination remains low, some people remain more vulnerable than others.
Age continues to be a “major risk factor”, the experts said, with older vaccinated individuals more
likely to end up ill from Covid compared to their younger counterparts.
Those from Indian and Pakistani backgrounds who are vaccinated face the highest risk among
ethnic groups.
People living with chronic conditions such as Down’s syndrome, kidney disease, sickle cell disease,
HIV/Aids and liver cirrhosis are also at more risk of severe disease or hospital admission after
vaccination.
The new algorithm behind the QCovid tool, which was first developed in 2020 and helped
influence policy on shielding, predicts those most at risk of serious Covid-19 outcomes from 14 or
more days after second vaccination dose, when substantial immunity is expected to have
developed. The findings do not take into account the different types of vaccines received by
people.
(Adapted from https://www.independent.co.uk/news/health/covid-vaccine-at-risk-latest-b1922196.html)

QUESTÃO 04 (EFOMM/INÉDITA) – It is possible to infer from the text that


(A) After complete vaccination, there are no more risk groups
(B) The risk of developing a severe response to Covid-19 after full vaccination is greater
(C) Indians and Pakistanis are at increased risk against Covid-19 because of their genetic
conditions
(D) The QCovid tool considers the post-vaccination immunization period to achieve its results
(E) The QCovid tool compares the effectiveness of different vaccines for Covid-19

QUESTÃO 05 (EFOMM/INÉDITA) – In the excerpt “Those from Indian and Pakistani backgrounds
who are vaccinated face the highest risk among ethnic groups” (paragraph 4), the word in bold
means
(A) Hide
(B) Finish
(C) Encounter
(D) Avoid
(E) Support

AULA 10 – PASSIVE VOICE, TAG QUESTIONS AND IDIOMS 37


TEACHER ANDREA BELO

QUESTÃO 06 (EFOMM/INÉDITA) – Choose the correct alternative to complete the paragraph


below
“Across the UK, children are preparing to return to school after almost two months ___ summer
holidays. But experts have warned that the number of coronavirus cases may surge ___ the
coming weeks, as more children and adults mix inside the classroom. Professor Jason Leitch,
Scotland’s national clinical director, told Sky News on Tuesday that the UK could be facing a ‘fragile
moment’ ___ its response to the ongoing pandemic, as the Delta variant continues to spread, and
lockdown rules remain relaxed”.
(Adapted from https://www.independent.co.uk/life-style/health-and-families/coronavirus-children-school-testing-b1912550.html)

(A) At / in /on
(B) At / on / in
(C) Of / in / on
(D) Of / at / in
(E) Of / in / in

Read the text and answer questions 07, 08, 09 and 10.
Malaria vaccine: When will it be available?
Researchers and health professionals have been celebrating after the World Health
Organization (WHO) approved the widespread use of the world's first malaria vaccine.
With more than 260,000 children under five dying from malaria each year in sub-Saharan Africa,
this development, decades in the making, could save tens of thousands of lives, the WHO says.
But when will people start benefitting from the vaccine known as RTS,S?
We have been looking at that and some other key questions.
How effective and safe is it?
The vaccine was proven effective six years ago, preventing 40% of malaria cases and 30% of severe
cases.
Since 2019, researchers have been carrying out wider pilot immunisation programmes in Ghana,
Kenya, and Malawi.
More than 800,000 children have received at least one dose and the WHO says there are no safety
concerns.
Does it matter that the protection is relatively low?
It would obviously be better if it was higher, but what many would say is you need to think about
the scale of the problem – with hundreds of millions of cases, a 40% reduction is still a huge
number of lives saved.

AULA 10 – PASSIVE VOICE, TAG QUESTIONS AND IDIOMS 38


TEACHER ANDREA BELO

"This is a moderately efficacious vaccine… [but] saving, preserving, avoiding 30-40% of cases and
deaths can bring a major benefit to the population," the WHO's Pedro Alonso told BBC Focus on
Africa.
Health authorities are also keen to stress this is a new weapon in the fight against malaria to be
used alongside other preventative measures, such as treated bed nets and drugs that target the
malaria parasite.
How does the vaccine work?
Malaria is a parasite that invades and destroys our blood cells in order to reproduce, and it's
spread by the bite of blood-sucking mosquitoes.
The vaccine targets the most deadly and common parasite in Africa: Plasmodium falciparum.
It tries to deal with the form of the parasite which enters the victim's blood shortly after being
bitten, by partially blocking access into human cells and therefore preventing disease, Dr Alonso
said.
It needs four doses to be effective. The first three are given a month apart at the age of five, six
and seven months, and a final booster is needed at around 18 months.
Children are considered to be the most at risk from dying from malaria as, unlike adults, they have
not had a chance to build up immunity.
(Adapted from https://www.bbc.com/news/world-africa-58833382)

QUESTÃO 07 (EFOMM/INÉDITA) – In the phrase “…a 40% reduction is still a huge number of
lives saved” (paragraph 7), the underlined word is a synonym for
(A) Enormous
(B) Limited
(C) Narrow
(D) Good
(E) Paltry

QUESTÃO 08 (EFOMM/INÉDITA) – The seventh paragraph of the text states that


(A) Malaria vaccine has achieved the highest rate of protection that a vaccine can provide
(B) The problem caused by Malaria is relatively small
(C) Despite the low rate of protection, the Malaria vaccine is extremely important and necessary
(D) 40% reduction in Malaria cases is not a significant number
(E) It is better that the malaria vaccine protection rate is low

AULA 10 – PASSIVE VOICE, TAG QUESTIONS AND IDIOMS 39


TEACHER ANDREA BELO

QUESTÃO 09 (EFOMM/INÉDITA) – In the phrase “Health authorities are also keen to stress this
is a new weapon in the fight against malaria…” (paragraph 9), the underlined word is a synonym
for
(A) Apathetic
(B) Indifferent
(C) Reluctant
(D) Anxious
(E) Strong

QUESTÃO 10 (EFOMM/INÉDITA) – The text


(A) Explains why the new malaria vaccine should not be distributed for use in the population
(B) Aims to lecture on the new vaccine against Malaria and highlight its importance
(C) Develops the idea that the Malaria vaccine should also be aimed at adults
(D) Focuses on advocating the non-use of Malaria vaccine in African children
(E) Criticizes the use of a vaccine that does not have a high protection rate

QUESTÕES EPCAR
Answer questions 01 to 10 according to the text.
The Brazilian army is turning into a de facto police force
Few places illustrate the modern role of the Brazilian army better than Tabatinga, a city of 62,000
on the shared border point between Brazil, Colombia, and Peru. The frontier has not budged since
the Portuguese built a now-ruined fort there in the 1700s. But Júlio Nagy, a local commander, has
his sights trained on unconventional threats. In February and March his troops intercepted 3.7
tonnes of cannabis. Last year they destroyed an airstrip built by illegal gold miners. Inside a small
army-run zoo, garish macaws rescued from animal traffickers squawk intermittently.
The country’s official defence review states that “at present, Brazil has no enemies”. Brazilian
strategists say that an insufficiency of military adversaries _______ _______ justify skimping on
defence. Criminal gangs operating in border areas can overwhelm civilian police, and in the future
Brazil hopes to deter foreigners covetous of its natural resources. New threats require new
responses. With 334,000 troops at its disposal, the government has had to find ways to deploy
them.
The army’s remit has expanded to police work. Most Brazilians seem unfazed by this trend. Unlike
politicians and police officers, servicemen are seen as honest, competent, and kind. Despite the
shadow of the dictatorship, confidence rankings of institutions often put the army at the top.
Soldiers are trying to adapt to their new role. At a training centre in Campinas, near São Paulo,
they are subjected to tear-gas and stun grenades, so they know what such weapons feel like
before unleashing them on civilians.
(Adapted from https://www.economist.com/the-americas/2017/07/06/the-brazilian-army-is-turning-into-a-de-facto-police-force)

AULA 10 – PASSIVE VOICE, TAG QUESTIONS AND IDIOMS 40


TEACHER ANDREA BELO

QUESTÃO 01 (EPCAR/INÉDITA) – According to the text, it is incorrect to say that:


(A) The Brazilian army is performing police work.
(B) Soldiers are training to adapt to their new role.
(C) The Brazilian army deals with unusual threats.
(D) Politicians, police officers and servicemen are seen as honest.
(E) Brazil does not have traditional military adversaries now.

QUESTÃO 02 (EPCAR/INÉDITA) – the sentence “Few places illustrate the modern role of the
Brazilian army better than Tabatinga...” (paragraph 1), the underlined word indicates
(A) a large quantity.
(B) a huge quantity.
(C) an exact number.
(D) a definite amount.
(E) a small quantity.

QUESTÃO 03 (EPCAR/INÉDITA) – Brazilian strategists say that an insufficiency of military


adversaries _______ _______ justify skimping on defence. (paragraph 2)
To form a negative present statement, the correct words to respectively fill in the blanks are
(A) do not
(B) are not
(C) is not
(D) does not
(E) not does

QUESTÃO 04 (EPCAR/INÉDITA) – In “... garish macaws rescued from animal traffickers squawk
intermittently.” (paragraph 1), what type of adverb is intermittently?
(A) an adverb of manner.
(B) an adverb of time.
(C) an adverb of place.
(D) an adverb of degree.
(E) an adverb of quantity.

AULA 10 – PASSIVE VOICE, TAG QUESTIONS AND IDIOMS 41


TEACHER ANDREA BELO

QUESTÃO 05 (EPCAR/INÉDITA) – Which sentence expresses something that is happening at the


moment of speaking?
(A) ... so they know what such weapons feel like before unleashing them on civilians.
(B) ... at present, Brazil has no enemies.
(C) Soldiers are trying to adapt to their new role.
(D) ... in the future Brazil hopes to deter foreigners covetous ...
(E) ... confidence rankings of institutions often put the army at the top.

QUESTÃO 06 (EPCAR/INÉDITA) – Which noun below conveys a plural form?


(A) centre.
(B) dictatorship.
(C) airstrip.
(D) servicemen.
(E) commander.

QUESTÃO 07 (EPCAR/INÉDITA) – Criminal gangs operating in border áreas can overwhelm civilian
police, and in the future Brazil hopes to deter foreigners covetous of its natural resources.
With 334,000 troops at its disposal, the government has had to find ways to deploy them.
In paragraph 2, the underlined words refer, respectively, to
(A) civilian police / troops.
(B) civilian police / the government.
(C) Brazil / troops.
(D) natural resources / disposal.
(E) Brazil / the government.

QUESTÃO 08 (EPCAR/INÉDITA) – In “But Júlio Nagy, a local commander, has his sights trained
on unconventional threats. (paragraph 1), the opposite of unconventional is
(A) unusual.
(B) ordinary.
(C) atypical.
(D) funny.
(E) strange.

AULA 10 – PASSIVE VOICE, TAG QUESTIONS AND IDIOMS 42


TEACHER ANDREA BELO

QUESTÃO 09 (EPCAR/INÉDITA) – What is the interrogative form of the sentence “New threats
require new responses.”?
(A) Are new threats require new responses?
(B) Does new threats require new responses?
(C) Do new threats require new responses?
(D) Do new threats requires new responses?
(E) Is new threats require new responses?

QUESTÃO 10 (EPCAR/INÉDITA) – “_______ illegal gold miners”


All the following terms can be used to correctly fill in the blank, except
(A) an.
(B) the.
(C) some.
(D) many.
(E) a lot of.

QUESTÕES ESA
Dogs are just like ‘play, eat, sleep’ – they bring me back into the moment. I think we can all learn
something from that,” says Carole Henderson, who has been taking her “furry backup” to the
office for the last few months.
They are not so good at making the tea and things get a bit rowdy when the delivery man comes
round, but Henderson’s labradors Barney and Rusty, and labradoodle Lily, have been her sidekicks
for the last decade. As well as being excellent foot-warmers, they have helped her emotionally
with getting through solo months in the office.
After the latest lockdown left her with no choice but to part-time furlough the rest of her team,
including her husband, the empty office felt like a cavern. “I felt a bit sorry for myself and thought:
‘I’m taking the dogs with me, because at least then I can have a doggy cuddle when things aren’t
going to plan,’” she says.
As a teacher of grief recovery method, Henderson’s work has felt more important than ever,
particularly with reaching out to those grieving in isolation, and without her colleagues there in-
person, her dogs have provided immense emotional support. “Things do get to me. I’m human,
doing this work. Someone will tell me a really sad story and there’s nothing better than getting a
cuddle from one of my dogs,” she says. “Stroking a lovely bit of fur brings you right back down,
then I’m ready to help the next person.”
(https://www.theguardian.com/lifeandstyle/2021/apr/06/the-dogs-keeping-office-workers-company-through-lockdown)

AULA 10 – PASSIVE VOICE, TAG QUESTIONS AND IDIOMS 43


TEACHER ANDREA BELO

QUESTÃO 01 (ESA/INÉDITA) – According to the text, it is correct to say that:


(A) We are going to learn with pets.
(B) We learn every day with all kind of pets.
(C) We can learn with our pets.
(D) We are supposed to learn as much as possible.
(E) We are supposed to learn im the future.

QUESTÃO 02 (ESA/INÉDITA) – The sentence says "who has been taking her “furry backup” to
the office for the last few months." means that
(A) Pets are at the office.
(B) Pets sometimes go to the office.
(C) Pets go to the office with their owners every day.
(D) Pets seldom go to the office with theirs owners.
(E) Pets are everywhere.

QUESTÃO 03 (ESA/INÉDITA) – The word foot-warmers is


(A) Uncaring.
(B) Stormy.
(C) Aloof.
(D) Balmy.
(E) Comfortable.

QUESTÃO 04 (ESA/INÉDITA) – In this part "I felt a bit sorry for myself and thought: I’m taking
the dogs with me, because at least then I can have ...". We can say that
(A) Having a dog at home assists the work at home office.
(B) Pets have helped a lot during lockdown period.
(C) Having a dog at home develops the work of their owners.
(D) Dogs are always used to aid in grieving recovery.
(E) Dogs are always used in tests.

AULA 10 – PASSIVE VOICE, TAG QUESTIONS AND IDIOMS 44


TEACHER ANDREA BELO

QUESTÃO 05 (ESA/INÉDITA) – The word cuddle in "because at least then I can have a doggy
cuddle when things aren’t going to plan" can be replaced for:
(A) Push.
(B) Release.
(C) Impact.
(D) Hug.
(E) Stop.

QUESTÕES ESCOLA NAVAL


Limits on coronavirus testing in Brazil are hiding the true dimensions of Latin America’s largest
outbreak
Atop a shaded hill at the edge of São Paulo, the gravedigger thinks he knows the truth. No matter
how bad it appears in Brazil — the country hit hardest by the coronavirus in the Southern
Hemisphere — the reality is significantly worse.
Manoel Norberto Pereira watched another body being wheeled in, accompanied by what has by
now become a familiar set of details. Sex: female. Age: 77 years. Cause of death: insufficient
respiration.
Every day brings more. The cemetery now receives around 50 bodies every day — double the
average in normal times. Many are marked as confirmed cases of covid-19, the disease caused by
the virus. But many more cite only an unidentified respiratory ailment. To Pereira, they’re the
unseen toll of the coronavirus in Brazil, which has officially infected 51,000 people and killed 3,400
— but unofficially many times more than that.
Imprecise and insufficient testing is a global problem, but in Brazil, it's on an entirely different
scale. Latin America's largest country is testing people at a rate far lower than any other nation
with at least 40,000 cases. It tests 12 times fewer people than Iran, and 32 times fewer than the
United States. Hospitalized patients aren't being tested. Some medical professionals aren't being
tested. People are dying in their homes without being tested.
Researchers at the Federal University of Minas Gerais have suggested Brazil has eight times more
coronavirus cases than the official numbers indicate. A research team at the University of São
Paulo thinks it has 16 times more — more than 800,000 cases.
According to government statistics, nearly 37,300 people have been hospitalized this year with
respiratory ailments — four times the number at this point last year — but only half have received
test results. In São Paulo, nearly 1,300 people have died of unidentified respiratory problems.
Amazonas state has reported 193 deaths from covid-19. But its capital city is burying so many
people — three times the average — that it’s digging trenches for mass graves.
“The big conclusion is that we don’t know what the real scenario is,” said Leonardo Costa Ribeiro,
an economist at the Federal University of Minas Gerais. “It gives the sensation that it is more
controlled, when the reality is very different.”

AULA 10 – PASSIVE VOICE, TAG QUESTIONS AND IDIOMS 45


TEACHER ANDREA BELO

Domingos Alves, a data scientist at the University of São Paulo, did little to hide his anger at the
chasm between how government officials talk about the virus and what the data shows.
“As a researcher, I look at the data and make analyses for the government,” he said. “But as a
citizen, I’m frustrated. The government is trying to control the epidemic without the elements of
how to control it, because of their lack of knowledge about how serious it is.
“It will get much worse. There will be lines at the hospitals. There will be lines at the cemeteries.
The next few weeks will be very dark.”
The new health minister, Nelson Teich, named by Bolsonaro to replace Mandetta, has made
increased testing a pillar of the country’s response to the pandemic. He said this week that the
government intends to purchase 46 million tests, enough for just over a fifth of the population.
He hasn’t specified when the tests will arrive or when they’ll be used.
“We are not talking about testing the entire country,” Teich said. “We are going to use the tests so
that people tested will reflect the Brazilian population.”
But the obstacles will be enormous — and could portend the struggles that await much of the
developing world as the coronavirus moves deeper into Latin America and Africa. A variable as
basic as reliable data could become a distinguishing factor between poor and wealthy countries.
Analysts say Brazil has neither the manufacturing nor the purchasing capacity to meet the demand
for tests. Its laboratories, already overwhelmed by a testing backlog, aren’t equipped to process
them at scale. And in the international scrum for supplies and testing equipment, the country is
losing out to wealthier nations that can pay more or leverage closer ties with China.
“Brazil could be thought of as a barometer of what could be expected in other countries,” said
Marcelo Gomes, a researcher at the Oswaldo Cruz Foundation, a scientific institution.
“Information is paramount. The more we have, the more we can make the population aware of
what it is exactly that we are facing.”
A gravedigger at São Paulo’s Vila Formosa Cemetery, one of Latin America’s largest, said he started
noticing strange deaths in early March — weeks before Brazil’s first confirmed coronavirus death.
The causes of death — pneumonia, respiratory failure — were what he would expect to see during
the Southern Hemisphere’s winter, not peak summer.
“One day, I saw on the news that in all of São Paulo there had been 20 burials of coronavirus
victims,” said the man, who spoke on the condition of anonymity because he didn’t have
permission to give an interview. “But I knew at my cemetery alone, there had been 27.”
Pereira feels the same frustration. Some days, he can’t believe the number of people out in the
street. If they were seeing what he saw, no way would they be outside. No way would there be a
national debate over reopening the economy. No one would be listening to claims that the
pandemic had been overblown.
Bodies don’t lie, he said. And here came another one.
Adapted from (https://www.washingtonpost.com/world/the_americas/coronavirus-brazil-testing-bolsonaro-cemetery-gravedigger/2020/04/22/fe757ee4-83cc-11ea-878a-86477a724bdb_story.html)

AULA 10 – PASSIVE VOICE, TAG QUESTIONS AND IDIOMS 46


TEACHER ANDREA BELO

QUESTÃO 01 (ESCOLA NAVAL/INÉDITA) – What’s the meaning of the word “overblown” in the
second last paragraph?
A) Blown up
B) Risen
C) Made up
D) Exaggerated
E) Invented

QUESTÃO 02 (ESCOLA NAVAL/INÉDITA) – According to the text, which option is correct?


A) The figures of coronavirus in Brazil are as close to reality as they can possibly get.
B) The cemetery used to receive around 40 bodies every day in normal times.
C) People dying due to an unidentified respiratory ailment are the unseen toll of COVID-19 in
Brazil.
D) Brazil has around 60,000 people unofficially infected by the coronavirus.
E) Researchers at Federal University of Minas Gerais suggested Brazil has more than 800,000
cases.

QUESTÃO 03 (ESCOLA NAVAL/INÉDITA) – According to the text, which option completes the
sentence below correctly?
The text says that gravediggers in Brazil _____________.
A) See a much worse reality than it is disclosed officially.
B) Are working as they usually do, with no changes in cemetery routine.
C) Are responsible for informing the government what are the real numbers related to the
pandemic.
D) Like the fact that there are more people dying, because this way they can secure their jobs.
E) Suggest that people watch the news, since it is a reliable source of accurate information.

QUESTÃO 04 (ESCOLA NAVAL/INÉDITA) – “It tests 12 times fewer people than Iran, and 32 times
fewer than the United States.” in paragraph 4, the word in bold refers to
A) Imprecise and insuficient testing
B) Global problem
C) Brazil
D) People
E) Nation

AULA 10 – PASSIVE VOICE, TAG QUESTIONS AND IDIOMS 47


TEACHER ANDREA BELO

QUESTÃO 05 (ESCOLA NAVAL/INÉDITA) – Which option completes the paragraph below


correctly?
Brazil ______(become) one of the global epicenters of the pandemic, recently surpassing China in
the number of reported cases. President Jair Bolsonaro ______(be) notoriously dismissive of the
virus, ______(say) earlier this week the number of deaths ______(be) declining, when in fact it
was increasing.
A) becomes / has been / saying / were
B) had become / was / said / were
C) has become / has been / said / were
D) becomes / was / saying / was
E) has become / has been / saying / was

QUESTÃO 06 (ESCOLA NAVAL/INÉDITA) – Mark the CORRECT alternative.


“Its laboratories, already overwhelmed by a testing backlog, aren’t equipped to process them at
scale.” (paragraph 14)
If laboratories are overwhelmed, it means that
A) They are comfortable with the demand.
B) The demand is over their capacity.
C) The demand matches their capacity.
D) They are experienced in this kind of situation.
E) The demand is not a concern for them.

QUESTÃO 07 (ESCOLA NAVAL/INÉDITA) – Mark the alternative which has the sentence below
correctly reported.
Analysts say Brazil has neither the manufacturing nor the purchasing capacity to meet the
demand for tests.
The author
A) replied: “Analysts say Brazil has neither the manufacturing nor the purchasing capacity to
meet the demand for tests.”
B) said that Analysts said Brazil had neither the manufacturing nor the purchasing capacity to
meet the demand for tests.
C) asked the readers if analysts say Brazil has neither the manufacturing nor the purchasing
capacity to meet the demand for tests.
D) asked analysts if they say Brazil has neither the manufacturing nor the purchasing capacity to
meet the demand for tests.

AULA 10 – PASSIVE VOICE, TAG QUESTIONS AND IDIOMS 48


TEACHER ANDREA BELO

E) said analysts say Brazil has neither the manufacturing nor the purchasing capacity to meet the
demand for tests.

QUESTÃO 08 (ESCOLA NAVAL/INÉDITA) – In the fourth paragraph, the text DOESN’T


A) explain that testing has been a problem for most countries in the world.
B) say that Brazil’s testing rate is far from ideal.
C) say testing rates in Iran and U.S. are examples of countries that test many more people than
Brazil.
D) say that Brazil tests a lot of people compared to countries with less than 40,000 cases.
E) say that Brazil is not testing health care professionals and that people are dying without being
tested.

QUESTÃO 09 (ESCOLA NAVAL/INÉDITA) – In the sentence “Information is paramount.”


(paragraph 15), the underlined word means
A) production.
B) power.
C) cure.
D) immunity.
E) primordial.

QUESTÃO 10 (ESCOLA NAVAL/INÉDITA) – “because of their lack of knowledge about how serious
it is.” (paragraph 9). The highlighted expression is closest in meaning to
A) otherwise.
B) thereafter.
C) due to.
D) moreover.
E) wherever.

QUESTÕES EsPCEx
Leia o texto a seguir e responda às questões 01, 02 e 03.
Afghanistan: US takes control of Kabul airport to evacuate staff
There have been scenes of panic at Kabul airport as desperate residents try to flee following the
seizure of the Afghan capital by the Taliban.
The US army say soldiers shot two armed men, while three people are reported to have died after
falling ___ (1) the underside of a plane they were clinging to shortly after take-off.

AULA 10 – PASSIVE VOICE, TAG QUESTIONS AND IDIOMS 49


TEACHER ANDREA BELO

The airport was closed earlier for soldiers to try and clear the runways.
US military planes are now landing, including one carrying US marines.
Marines are being flown in to help the evacuation effort. The US and other countries are rushing
to remove staff and allies from the country.
A German evacuation plane also landed at Kabul airport on Monday, sources told Reuters news
agency.
___ (2) Sunday the Taliban declared victory after Afghan President Ashraf Ghani fled abroad and
his government collapsed.
The militants' return to rule brings an end to almost 20 years of a US-led coalition's presence in
the country.
Kabul was the last major city in Afghanistan to fall to a Taliban offensive that began months ago
but accelerated in recent days as they gained control of territories, shocking many observers.
The Islamist group was able to seize control after most foreign troops pulled out.
Following the Taliban's seizure ___ (3) Kabul, many people headed to the airport.
Evacuations of foreigners and some Afghans with links to foreign governments and organisations
have been taking place, but passengers said rumours spread that even those without visas were
being allowed to travel.
(Adapted from https://www.bbc.com/news/world-asia-58227029)

QUESTÃO 01 (EsPCEx/INÉDITA) – Choose the alternative containing the correct words to


respectively complete gaps (1), (2) and (3)
A) From, on, of
B) At, on, of
C) From, on, at
D) From, in, at
E) At, on, of

QUESTÃO 02 (EsPCEx/INÉDITA) – According to the text, is correct to state that


A) The Taliban's victory was peacefully accepted
B) Only the US is withdrawing its troops from Kabul
C) With the return of the Taliban, the US seized power from Kabul
D) Lack of foreign troops facilitated Taliban victory
E) No one is allowed to leave Kabul

AULA 10 – PASSIVE VOICE, TAG QUESTIONS AND IDIOMS 50


TEACHER ANDREA BELO

QUESTÃO 03 (EsPCEx/INÉDITA) – In the sentence “The Islamist group was able to seize control
after most foreign troops pulled out” (paragraph 10), the word “seize” means to
A) Steal
B) Take
C) Give
D) Release
E) Remove

Leia o texto a seguir e responda às questões 04, 05 e 06


New Zealand enters nationwide lockdown over one Covid case
New Zealand has announced a snap lockdown after a man tested positive for Covid, the first
case in six months.
The case ___ (1) detected in Auckland, which will be in lockdown for a week, while the rest of the
country will be in lockdown for three days.
Authorities say they are working on the assumption that the new case was the Delta variant.
Just around 20% of its population has been fully vaccinated.
Coromandel, a coastal town where the infected person had visited, will be in lockdown for seven
days too.
Prime Minister Jacinda Ardern ___ (2) the toughest "level 4" rules were required - closing schools,
offices and all businesses with only essential services remaining operational.
"I want to assure New Zealand that we have planned for this eventuality. Going hard and early has
worked for us before," she said.
The patient is a 58-year-old man, who is believed to have been infectious since last Thursday.
There ___ (3) reportedly a rush at supermarkets in Auckland, as locals anticipated a snap
lockdown.
Officials said there was a need for strong response because of the fear of the Delta variant, and
because there was no clear link between the new case and the border or quarantine facilities.
Data released by New Zealand's Ministry of Health on Monday showed that all Covid-19 cases
detected at the country's border in recent weeks had been Delta.
"We have seen what can happen elsewhere if we fail to get on top of it. We only get one chance,"
Ms Ardern said in a televised national address, calling the Delta strain "a game changer".
New Zealand has been successful in eliminating the virus from within its borders, although its
international borders remain largely closed.
However, its vaccination programme has rolled out at a slow pace, with only around 20% of people
fully vaccinated and 33% of people having received one dose, according to Our World in Data.
(Adapted from https://www.bbc.com/news/world-asia-58241619)

AULA 10 – PASSIVE VOICE, TAG QUESTIONS AND IDIOMS 51


TEACHER ANDREA BELO

QUESTÃO 04 (EsPCEx/INÉDITA) – In the sentence “...there was no clear link between the new
case and the border or quarantine facilities” (paragraph 9), the word “clear” means
A) Blocked
B) Free
C) Evident
D) Complicated
E) Hidden

QUESTÃO 05 (EsPCEx/INÉDITA) – Choose the alternative containing the correct verb forms to
complete the gaps (1), (2) and (3) in paragraphs 1, 5 and 8 respectively
A) Will be, said, was
B) Was, says, was
C) Will be, said, were
D) Was, said, were
E) Was, said, was

QUESTÃO 06 (EsPCEx/INÉDITA) – According to the text, choose the correct statement


A) All of New Zealand will be in lockdown for 7 days
B) Despite coping well with the pandemic, New Zealand is not promoting rapid vaccination
C) Fortunately, the infected person has only been to one city
D) New Zealand's approach to the pandemic has never been effective
E) New Zealand has already provided vaccine for all its inhabitants

Leia o texto a seguir e responda às questões 07, 08 e 09


(Título omitido propositalmente)
Born in 1981, Britney Spears is an American singer, songwriter, and actress who influenced the
revival of teen pop during the 1990s and 2000s. People often referred to her as the Princess of
Pop.
In 2005 and 2006, she gave birth to her sons Sean and Federline; however, in 2006, she filed for
divorce from her husband Kevin Federline. They reached an agreement to share joint custody of
their sons.
Then, Spears was admitted to several drug rehabilitation facilities, and in 2007, she lost custody
of her sons to Federline. In 2008, a court placed Spears under conservatorship of her father James
Spears, giving him complete control of her life and assets for 13 years.

AULA 10 – PASSIVE VOICE, TAG QUESTIONS AND IDIOMS 52


TEACHER ANDREA BELO

Now, Britney´s father is stepping down from the conservatorship. According to his lawyer, there
are no actual reasons to suspend or remove him; however, James Spears is responding to the
public battle and a petition for his suspension.
(Adapted from https://www.newsinlevels.com/products/britney-spears-free-again-level-3/)

QUESTÃO 07 (EsPCEx/INÉDITA) – Choose the most appropriate title for the text
A) Britney Spears free again
B) Britney Spears Maternity
C) How Rehab Affected Britney Spears
D) What was the role of Britney Spears' father in her rehabilitation?
E) How are Britney Spears' kids?

QUESTÃO 08 (EsPCEx/INÉDITA) – What kind of text is this?


A) Opinion article
B) News report
C) Letter
D) Essay
E) Textbook

QUESTÃO 09 (EsPCEx/INÉDITA) – In the sentence “According to his lawyer, there are no actual
reasons to suspend or remove him…” (paragraph 4), the word “actual” means
A) Confused
B) Hypothetical
C) Doubtful
D) Problematic
E) Real

Leia o texto a seguir e responda à questão 10


Afghan women to have rights within Islamic law, Taliban say
The rights of women in Afghanistan will be respected "within the framework of Islamic law",
the Taliban say.
In the group's first news conference since taking control of the country ___ (1) Sunday, a
spokesman said women would be free to work but gave little detail about other rules and
restrictions.
Zabihullah Mujahid repeated that all Afghans must live "within the framework of Islam".

AULA 10 – PASSIVE VOICE, TAG QUESTIONS AND IDIOMS 53


TEACHER ANDREA BELO

Rights groups fear women's freedoms could be eroded under the Taliban.
The militant group introduced or supported punishments in line with their strict interpretation of
Islam's legal system, Sharia law, when they controlled Afghanistan between 1996 and 2001.
Women had to wear the all-covering burka, and the Taliban also disapproved of girls aged 10 and
over going to school.
In the news briefing on Tuesday, Mr Mujahid fielded several questions from the international
media about what women's rights could look like under a Taliban government.
"We are going to allow women to work and study within our frameworks," he said. "Women are
going to be very active within our society."
But he did not expand when asked ___ (2) dress codes and what roles women would be able to
have within ___ (3) country's workforce.
(Adapted from https://www.bbc.com/news/world-asia-58249952)

QUESTÃO 10 (EsPCEx/INÉDITA) – Choose the alternative containing the correct words to


complete the gaps (1), (2) and (3) in paragraphs 1, 7 and 7 respectively
A) On, about, X
B) In, about, the
C) In, from, the
D) On, about, the
E) On, about, a

AULA 10 – PASSIVE VOICE, TAG QUESTIONS AND IDIOMS 54


TEACHER ANDREA BELO

GABARITO
GABARITO AFA
01 – D 02 – C 03 – A 04 – C 05 – E
06 – B 07 – B 08 – D 09 – E 10 – C

GABARITO COLÉGIO NAVAL


01 – A 02 – B 03 – D 04 – D 05 – B
06 – B 07 – A 08 – A 09 – E 10 – E

GABARITO EAM
01 – B 02 – C 03 – A 04 – E 05 – B

GABARITO EEAR
01 – D 02 – E 03 – A 04 – A 05 – A
06 – E 07 – E 08 – E 09 – A 10 – D

GABARITO EFOMM
01 – B 02 – A 03 – C 04 – D 05 – C
06 – E 07 – A 08 – C 09 – D 10 – B

GABARITO EPCAR
01 – D 02 – E 03 – D 04 – A 05 – C
06 – D 07 – E 08 – B 09 – C 10 – A

GABARITO ESA
01 – C 02 – A 03 – D 04 – B 05 – D

GABARITO ESCOLA NAVAL


01 – D 02 – C 03 – A 04 – C 05 – E
06 – B 07 – B 08 – D 09 – E 10 – C

GABARITO EsPCEx
01 – A 02 – D 03 – B 04 – C 05 – E
06 – B 07 – A 08 – B 09 – E 10 – D

AULA 10 – PASSIVE VOICE, TAG QUESTIONS AND IDIOMS 55


TEACHER ANDREA BELO

QUESTÕES COMENTADAS
QUESTÕES AFA
Industries shut, Ganga water quality improves
With industries shutdown and people confined to their homes for days following the imposition
of the 21-day lockdown, the water quality of Ganga river has significantly improved, according to
a IITBHU professor.
Dr P K Mishra, a professor at the institute's Chemical Engineering department, attributed the
improvement to the lack of industrial waste polluting the waters.
"One-tenth of the pollution in Ganga river comes from industries. As industries are shut due to
lockdown, situation has become better. We have seen 40-50 per cent improvement in the Ganga.
It is a significant development," Mishra told news agency ANI. With rainfall on March 15 and 16,
the Ganga's water level has also increased, he said, adding that this meant the river's cleaning
capacity has also increased.
"There is a lot of difference between when we see the water of the Ganga river today and what
used to be earlier. Today, the water looks clean," ANI quoted a local as saying. He added that
people are also not taking bath at the ghats — a common ritual among locals there. "If this is the
condition in 10 days, then I believe Ganga river will be like it used to be earlier," he added. Another
local said nobody would have thought that the lockdown would have such an impact on
environment.
With several countries under lockdown to contain the spread of coronavirus, the global pollution
levels have also dropped significantly. In India, pollution levels have dipped as the lockdown in
megacities has kept cars off the road and closed factories. New Delhi, which regularly has
unhealthy air conditions, has now seen its AQI falling below 95 — a big reduction from its monthly
average of 161 from March 2019. Mumbai, too, has witnessed a similar reduction in pollution.
Due to the improvement in air quality following in India, Dhauladhar range, which is part of a
Himalayan chain of mountains in Himachal Pradesh, has now become visible from Jalandhar in
Punjab, The mountain rises from Kangra and Mandi.
Adapted from (https://www.msn.com/en-in/news/environment/industries-shut-ganga-water-quality-improves/ar-BB12beCN)

QUESTÃO 01 (AFA/INÉDITA) – What’s the meaning of the word “chain” in the last paragraph?
A) Lots
B) Rise
C) Some
D) String
E) Valley
Comentário: A alternativa A está incorreta. “Lots” dá ideia de quantidade, não de
posicionamento. Seria como dizer muitas montanhas, enquanto “chain of mountains” quer dizer
cadeia de montanhas.

AULA 10 – PASSIVE VOICE, TAG QUESTIONS AND IDIOMS 56


TEACHER ANDREA BELO

A alternativa B está incorreta. Usar “rise” nesse contexto seria como dizer que a montanha sobe,
não que elas se encadeiam, como sugere a palavra “chain”.
A alternativa C está incorreta. “Some” é uma palavra que dá ideia de quantidade (alguns), e não
de posicionamento geográfico, como é o caso de “chain”.
A alternativa D está correta. “String of mountains” significa exatamente o mesmo que “chain of
mountains”, ou seja, cadeia de montanhas. A palavra “string” traz a ideia de sequência.
A alternativa E está incorreta. “Valley of mountains” significa o vale de montanhas, e o trecho
não fala sobre o vale da montanha, mas sim da cadeia montanhosa em si.
GABARITO: D

QUESTÃO 02 (AFA/INÉDITA) – According to the text, which option is correct?


A) Industries are responsible for 40-50% of Ganga river pollution.
B) The river is cleaner just because people stopped bathing at the ghats.
C) The Ganga river water is 40-50% better as a result of the lockdown.
D) Since Ganga river water is better, coronavirus was a good thing after all.
E) India will now keep industries shut permanently so the river water keep clean.
Comentário: A alternativa A está incorreta. Segundo o texto, as indústrias são responsáveis por
um décimo da poluição do rio Ganga, não 40-50% como dito pela alternativa.
A alternativa B está incorreta. As pessoas terem parado de se banhar na água do rio é um dos
fatores que contribuem para a limpeza da água, de acordo com o texto. Mas a palavra “just” torna
a alternativa incorreta, pois seu uso faz com que achemos que a melhora da água do rio se deu
simplesmente por esse fator.
A alternativa C está correta. O texto afirma que a água do rio está 40- 50% melhor como resultado
do lockdown (estado de isolamento ou acesso restrito instituído como medida de segurança).
A alternativa D está incorreta. Apesar de o “lockdown” ter sido uma medida adotada por conta
do surto do coronavírus, o texto não fala sobre o referido vírus, mas sim sobre a qualidade da
água do rio. Portanto, não se pode afirmar que o coronavírus foi algo bom com base no que é dito
no texto.
A alternativa E está incorreta. O texto diz que o fato de as indústrias estarem fechadas contribuiu
para a melhoria da água do rio, mas o texto não diz nada sobre manter as indústrias fechadas para
manter a água do rio limpa.
GABARITO: C

AULA 10 – PASSIVE VOICE, TAG QUESTIONS AND IDIOMS 57


TEACHER ANDREA BELO

QUESTÃO 03 (AFA/INÉDITA) – According to the text, which option completes the sentence
below correctly?
The text says it was a good thing for the river water _____________.
A) to shut industries
B) to have the economy paralized
C) that coronavirus started infecting people in India
D) that New Delhi has better air conditions
E) that Dhauladhar range has now become visible from Jalandhar in Punjab
Comentário: A alternativa A está correta. O texto afirma que fechar as indústrias foi um fator
determinante para a melhora da água do rio, já que as indústrias são responsáveis por 10% da
poluição do rio Ganga.
A alternativa B está incorreta. O texto não fala nada sobre a economia do país e, portanto, essa
é uma informação que não se pode confirmar com base no texto.
A alternativa C está incorreta. O texto não fala nada sobre o coronavírus e, portanto, não se pode
dizer que foi algo bom para a água do rio que as pessoas começaram a ser infectadas pelo vírus
na Índia.
A alternativa D está incorreta. As melhores condições da qualidade do ar em Nova Deli também
são consequência diminuição da poluição, mas essa informação não tem relação com a qualidade
da água do rio Ganga, de acordo com o texto.
A alternativa E está incorreta. A melhora das condições de visibilidade tem a ver com a
diminuição da poluição de maneira geral, mas não tem relação com a melhora da qualidade da
água do rio Ganga.
GABARITO: A

QUESTÃO 04 (AFA/INÉDITA) – “a big reduction from its monthly average of 161 from March
2019” in the second last paragraph, the word in bold refers to
A) AQI
B) Air conditions
C) New Delhi
D) Factories
E) Mumbai
Comentário: “Its” é um adjetivo possessivo (possessive adjective) do pronome “it”, ou seja, é uma
palavra que indica posse por parte de alguma coisa. O texto diz que há uma medição de qualidade
do ar (AQI), e que Nova Deli tem uma média mensal de 161 de março de 2019 nesse índice. O
texto diz que o valor dessa medição caiu para um valor menor que 95. Ao entender esse contexto,
podemos perceber que o trecho destacado no enunciado diz: uma grande redução em relação à
sua média mensal de 161 de março de 2019. A palavra “its” é representada pela palavra “sua” na
nossa tradução. Depois de analisar o contexto completo e traduzir o trecho para facilitar o

AULA 10 – PASSIVE VOICE, TAG QUESTIONS AND IDIOMS 58


TEACHER ANDREA BELO

raciocínio, fica mais claro que esses números têm relação com a cidade de Nova Deli e seus dados
de poluição do ar. Portanto, é correto dizer que “its” se refere à cidade de Nova Deli. Isso nos leva
ao gabarito de letra C.
GABARITO: C

QUESTÃO 05 (AFA/INÉDITA) – Which option completes the paragraph below correctly?


Around the 18th and 19th centuries two main ideas about dreams _______ (become) popular.
One was the idea that the things we see in our dreams are things our conscious mind is hiding
from us. However, the opposite idea _______ (say) that while we’re asleep, the brain _______
(organise) memories and events from the day. Dreams are just a random collection of these
thoughts, but we _______(try) to make a story from them when we wake up.
A) become / said / organises / try
B) had become / says / organised / tried
C) became / said / organised / tried
D) become / says / organises / try
E) became / said / organises / try
Comentário: A primeira lacuna deve ser preenchida por “became”. O texto já começa
oferecendo referências de séculos passados, o que já nos dá a dica que precisamos em relação ao
tempo verbal a ser usado. O tempo verbal ideal para essa situação é o Simple Past, já que estamos
falando de algo inniciado e encerrado no passado.
A segunda lacuna deve ser preenchida por “said”. O texto fala de duas ideias que surgiram em
séculos passados, e para isso usa verbos no tempo verbal Simple Past. Logo, devemos manter o
uso desse tempo verbal para continuar falando da segunda ideia que surgiu.
A terceira lacuna deve ser preenchida por “organises”. Na frase imediatamente anterior, o texto
muda o tempo verbal do Simple Past para o Simple Present quando começa a falar do processo
que ocorre no cérebro enquanto dormimos. Por isso essa lacuna deve ser preenchida pelo verbo
no Simple Present.
A quarta lacuna deve ser preenchida por “try”. Nesse momento do texto, o Simpple Present já
está estabelecido como tempo verbal usado no trecho. Por isso, o mais adeuqado é que nós
mantenhamos o tempo verbal em uso no trecho e utilizemos o verbo no Simple Present.
Temos a sequência: became / said / organises / try
GABARITO: E

AULA 10 – PASSIVE VOICE, TAG QUESTIONS AND IDIOMS 59


TEACHER ANDREA BELO

QUESTÃO 06 (AFA/INÉDITA) – Mark the CORRECT alternative.


“With several countries under lockdown to contain the spread of coronavirus” (second last
paragraph)
If a country is under lockdown, it means that
A) it is locked.
B) it is in a state of isolation as a security measure.
C) everyone in that country is a prisoner.
D) this country is under a dictatorship.
E) there’s no way people can talk to each other.
Comentário: A alternativa A está incorreta. A alternativa diz que o país estaria trancado (locked).
Essa não seria a melhor forma de descrever “lockdown”.
A alternativa B está correta. Essa é a definição de “lockdown”. Portanto, é correto dizer que se
um país está em um “lockdown”, isso significa que ele está em um estado de isolamento como
uma medida de segurança.
A alternativa C está incorreta. Essa alternativa poderia confundir alguns alunos, já que uma das
possíveis significações de “lockdown” é o ato de confinar prisioneiros em suas celas. Mas nesse
contexto, esse significado não tem sentido, já que não é coerente pensar que todos os habitantes
de um determinado país é um prisioneiro.
A alternativa D está incorreta. Um país normalmente se isola dos demais quando está sob uma
ditadura, mas não é esse o caso aqui. O contexto apresentado é muito específico no que diz
respeito ao motivo pelo qual o “lockdown” foi imposto, que é a contenção do contágio do
coronavírus na Índia.
A alternativa E está incorreta. Não se pode afirmar que não há meios de as pessoas conversarem
entre si, já que há meios de se comunicar de forma não presencial. Isso desqualifica a alternativa.
GABARITO: B

QUESTÃO 07 (AFA/INÉDITA) – Mark the alternative which has the sentence below correctly
reported.
Dr P K Mishra attributed the improvement to the lack of industrial waste polluting the waters.
The author
A) replied: “Dr P K Mishra attributed the improvement to the lack of industrial waste polluting
the waters.”
B) said that Dr P K Mishra had attributed the improvement to the lack of industrial waste
polluting the waters.
C) asked the readers if Dr P K Mishra attributed the improvement to the lack of industrial waste
polluting the waters.
D) asked if Dr P K Mishra attributed the improvement to the lack of industrial waste polluting
the waters.
E) said Dr P K Mishra attributed the improvement to the lack of industrial waste polluting the
waters.

AULA 10 – PASSIVE VOICE, TAG QUESTIONS AND IDIOMS 60


TEACHER ANDREA BELO

Comentário: A alternativa A está incorreta. No “reported speech”, não se usa dois pontos para
uma citação, mas sim a pessoa que está reportando dá a informação de maneira fluida. Exemplo:
“He said that he saw me in the restaurant yesterday” (ele disse que me viu no restaurante ontem).
A alternativa B está correta. A alternativa usa o reported speech de forma correta, pois “volta”
um tempo verbal em direção ao passado na hora de reportar aquilo que foi dito. A frase original
estava no Simple Past, logo a frase foi reportada usando o Past perfect (had attributed).
A alternativa C está incorreta. O erro da alternativa consiste no fato de que o autor não perguntou
nada aos leitores, mas sim disse. Portanto, o reported speech deveria começar com “said” ao
invés de “asked”.
A alternativa D está incorreta. Mais uma vez o erro está em dizer que o autor perguntou algo,
dessa vez ao professor Mishra. O autor não perguntou nada, apenas disse. Portanto, o reported
speech deveria começar com “said” ao invés de “asked”.
A alternativa E está incorreta. O erro da alternativa foi não alterar o tempo verbal na hora de
reportar a informação. No trecho original, verbo usado foi “attributed” (simple past), logo, no
reported speech deve-se usar “had attributed” (past perfect).
GABARITO: B

QUESTÃO 08 (AFA/INÉDITA) – In the third paragraph, the text DOESN’T


A) say how much industries activities affect the quality of Ganga river water.
B) say why the industries are shut and that the river water has been better since then.
C) say how better the river water is using numbers for that.
D) explain the process through which rainfall increases the cleaning capacity of the river water.
E) say rainfall boosted the river water improvement.
Comentário: A alternativa A está incorreta. O parágrafo realmente diz o quanto as atividades das
indústrias afetam a qualidade da água do rio. “One-tenth of the pollution in Ganga river comes
from industries”. O texto especifica quanto da poluição do rio é decorrente da atividade das
indústrias.
A alternativa B está incorreta. O terceiro parágrafo diz que as indústrias estão fechadas por causa
do “lockdown” imposto pelo governo e que a água do rio tem melhorado desde então. “As
industries are shut due to lockdown, situation has become better.”
A alternativa C está incorreta. O parágrafo usa números percentuais para quantificar a melhora
da água do rio Ganga. “We have seen 40-50 per cent improvement in the Ganga.”
A alternativa D está correta. O parágrafo diz que as chuvas aumentam o nível do rio assim como
sua capacidade de limpeza, mas o parágrafo não explica o processo através do qual as chuvas
aumentam a capacidade de limpeza da água do rio.
A alternativa E está incorreta. O parágrafo afirma que as chuvas aumentaram o nível do rio e, por
conseguinte, sua capacidade de limpeza da água.
GABARITO: D

AULA 10 – PASSIVE VOICE, TAG QUESTIONS AND IDIOMS 61


TEACHER ANDREA BELO

QUESTÃO 09 (AFA/INÉDITA) – In the sentence “Mumbai, too, has witnessed a similar reduction
in pollution” (second last paragraph), the underlined word means
A) produced.
B) was a victim of.
C) manufactured.
D) was responsible for.
E) saw an event take place.
Comentário: A alternativa A está incorreta. “produced” significa produziu, enquanto “witnessed”
significa testemunhou, presenciou. Portanto, não se pode dizer que há relação entre as duas
definições.
A alternativa B está incorreta. “Witness” é alguém que presencia algum evento ou
acontecimento, mas não é vítima na situação, e sim alguém que simplesmente observa o
desenrolar dos acontecimentos.
A alternativa C está incorreta. “Manufactured” tem a ver com produção de itens, não havendo
nenhuma relação de significação com “witnessed”.
A alternativa D está incorreta. “Witness” é alguém que simplesmente observa o desenrolar dos
fatos, não estando envolvido na situação. Portanto, não podemos dizer que alguém que foi
responsável por um acontecimento é um “witness”, pois não há relação de sentido entre eles.
A alternativa E está correta. “Saw an event take place” é exatamente a definição de “witness”,
nada mais é do que um observador de um acontecimento.
GABARITO: E

QUESTÃO 10 (AFA/INÉDITA) – “Due to the improvement in air quality following in India,


Dhauladhar range, which is part of a Himalayan chain of mountains in Himachal Pradesh, has
now become visible from Jalandhar in Punjab, The mountain rises from Kangra and Mandi.” (last
paragraph). The highlighted expression is closest in meaning to
A) otherwise.
B) thereafter.
C) beacause of.
D) moreover.
E) wherever.
Comentário: A alternativa A está incorreta. “otherwise” significa caso contrário, enquanto “due
to” significa por conta de. Não há relação de sentido entre as duas expressões.
A alternativa B está incorreta. “Thereafter” significa depois disso, enquanto “due to” significa por
conta de. Não há relação de sentido entre as duas expressões.
A alternativa C está correta. “because of” tem exatamente o mesmo significado de “due to”, por
conta de/por causa de.

AULA 10 – PASSIVE VOICE, TAG QUESTIONS AND IDIOMS 62


TEACHER ANDREA BELO

A alternativa D está incorreta. “moreover” é um sinônimo de “besides”, dando uma ideia de


adição, e não de causa como é o caso de “due to”.
A alternativa E está incorreta. “wherever” quer dizer em qualquer lugar e não possui nenhuma
relação de sentido com “due to”, que significa por conta de/por causa de.
GABARITO: C

QUESTÕES COLÉGIO NAVAL


Who was Steve Jobs?
Do you have an iPad, iPod, iPhone, a smartwatch or a Mac computer? If
you don’t, you probably know someone who has one (or wants one)!
Steve Jobs made the company – Apple – that created these things that are
now such an important part of the lives of millions of people.
He was born on February 24, 1955, in San Francisco, California. When he
was a boy, he had a special hobby: he liked to take apart televisions and
put them back together again. He was a very good student in school and
even skipped a grade. After he finished grade four, he went into grade six,
and in 1972, when he was seventeen, he graduated from high school. He
then began his studies in Reed College, in Portland, Oregon, but dropped
out after six months. Deciding to quit was not at all easy (...)
After Steve Jobs returned to the United States, he found a job as a video
game designer at a company called Atari, and two years after that, in 1976, when he was only
twenty-one years old, he created his own company – Apple Computer – with his friend, Steve
Wozniak. Steve Jobs later got married in 1991 and had four children. Unfortunately, doctors
discovered that he had cancer in 2003 and in 2011 he died at the age of only 56.
https://www.allthingstopics.com/uploads/2/3/2/9/23290220/7300865_orig.png

QUESTÃO 01 (COLÉGIO NAVAL/INÉDITA) – Which Read the extract from the text:
“If you don’t, you probably know someone who has one (or wants one)”
Mark the option that can replace the term one.
a) i-phone
b) computer
c) someone
d) you
e) company
Comentários: Para responder a questão pedida precisamos olhar para mais que apenas o trecho
mencionado.

AULA 10 – PASSIVE VOICE, TAG QUESTIONS AND IDIOMS 63


TEACHER ANDREA BELO

“Do you have an iPad, iPod, iPhone, a smartwatch or a Mac computer? If you don’t, you probably
know someone who has one (or wants one)!”
A primeira parte desse trecho pergunta se o leitor possui alguns dos produtos famosos da empresa
Apple. Então, o parte inicial da segunda frase é uma condicional que verifica as respostas
negativas. Transformando-a em uma oração condicional completa teríamos “if you don’t have an
iPad, iPod, iPhone, a smartwatch or a Mac computer, ...”. Por fim, em seguida, temos a oração
consecutiva caso a condicional anterior seja atendida.
Nessa última frase, temos: “you probably know someone who has one (or wants one)”, isto é,
“you probably know someone who has an iPad, iPod, iPhone, a smartwatch or a Mac computer
(or wants an iPad, iPod, iPhone, a smartwatch or a Mac computer)”.
Portanto, a resposta correta é alternative A.
A alternative B está incorreta, pois, especificamente no contexto, não se trata de qualquer
computer, mas de um Mac computer. Por isso, a alternative A está melhor que a B, já que Iphones
são produtos exclusivos da Apple.
A alternativa C está incorreta, pois não faria sentido a frase “you probably know someone who
has someone (or wnats someone)”.
O mesmo para a alternative D: “you probably know someone who has you (or wants you)”.
A alternative E está incorreta, pois a company (no caso, Apple) ainda nem havia sido mencionada,
portanto, não poderia estar sendo retomada.
GABARITO: A

QUESTÃO 02 (COLÉGIO NAVAL/INÉDITA) – What’s the correct question referring to the


underlined information?
“these things that are now such an important part of the lives of millions of people”
a) How important these things are for people’s lives?
b) How important are those things for people’s lives?
c) What’s the importance of the things mentioned?
d) What’s the importance of all the things at all?
e) How important are things and people’s lives?
Comentários: Vamos fazer essa questão por eliminação, uma das técnicas que ensino em meu
curso, facilitando e fazendo economia de tempo na hora da sua prova. Começando pela alternative
E, que está incorreta. Ela não pode ser nossa resposta, porque a frase entre aspas diz que “these
things” são uma “part of lives of million of people”. Assim, existe uma relação entre “things” e
“lives”, o que não existe na alternative E.
A alternative D também está incorreta. Aqui, temos um detalhe importante: a frase não fala de
qualquer things e sim de “these things”, ou seja, há determinação. A alternative D fala de “all the
things”, sem distinção, portanto, não pode estar correta. Temos um problema semelhante na

AULA 10 – PASSIVE VOICE, TAG QUESTIONS AND IDIOMS 64


TEACHER ANDREA BELO

alternative C, que fala “things mentioned” e, portanto, não se refere corretamente a quais
“things”.
Agora vamos analisar a alternativa A: aqui temos que ter atenção à gramática, pois em WH-
questions é necessário que nos lembremos de inverter o verb to be com o sujeito. Portanto, aqui
o correto seria: “How important are these things…”.
Por fim, então, concluímos que a alternative correta é B. Você pode estar se perguntar o motivo
da troca de “these” por “those” e, veja, podemos interpretar com simplicidade.
Vamos colocar as frase na ordem temporalem que mais naturalmente ocorrem:
Person 1 – “How important are those things for people’s lives?”
Person 2 – “these things that are now such an important part of the lives of millions of people”
A troca de “those” por “these” pode ser apenas uma adequação à distância que cada person está
das things. Person 1 está perguntando sobre things das quais está longe e person 2 está
respondendo sobre essas mesmas things e está perto delas.
GABARITO: B

QUESTÃO 03 (COLÉGIO NAVAL/INÉDITA) – Read the extract from the text


“He liked to take apart televisions and put them back together again”
The pronoun “them” refers to:
a) equipments in general
b) Apple equipments
c) Steve Jobs’ hobbies
d) televisions
e) things to take apart
Comentários: Aqui temos uma questão que requer o entendimento da função dos pronomes –
pronouns. Pronouns nos ajudam a evitar a repetição de palavras e à construção de frases curtas e
desconfortáveis de ler. Para encontrarmos a quem o pronome se refere, vamos fazer o processo
reverso e desacoplar as duas frase. Assim, teremos: “he liked to take apart televisions. And he liked
to put televisions back together again”. Assim, percebemos que “them” substitui “televisions” e
que a alternativa correta é D.
Outra alternativa rapidamente é descartada ao entendermos os sentidos dos verbos das frase.
“take apart” significa desmontar e “put together” significa remontar. Portanto, a alternativa
correta precisaria trazer um objeto que pudesse ser desmontado e remontado, o que não é o caso
da alternativa C.
As alternativas A, B e E são descartadas ao percebermos que nenhum dos substantivos – nouns
mencionados nelas já apareceram no texto anteriormente e, por isso, não poderiam estar sendo
retomados por um pronoun com o objetivo de substituir e evitar a repetição desse noun.
GABARITO: D

AULA 10 – PASSIVE VOICE, TAG QUESTIONS AND IDIOMS 65


TEACHER ANDREA BELO

QUESTÃO 04 (COLÉGIO NAVAL/INÉDITA) – Read the first sentence of the text


“Do you have an iPad, iPod, iPhone, a smartwatch or a Mac computer?”
What’s the plural form of the sentence?
a) Do you have any iPad, iPod, iPhone, smartwatches or any Mac computer?
b) Do you have iPads, iPods, iPhones, smartwatches or Mac computers?
c) Do you have any iPads, iPods, iPhones, smartwatches or Mac computers?
d) Do you have some iPads, iPods, iPhones, smartwatches or Mac computers?
e) Do you have iPads, iPods, iPhones, a smartwatchs or a Mac computers?
Comentários: Essa questão trabalha a transição de singular para plural sem mudança do sentido
envolvido no uso de definite articles e de indefinite articles. A frase original traz um indefinite
article, o “an”. Ao passarmos para o plural, precisamos manter a quantidade igualmente indifinida.
Como a frase é afirmativa, usaremos “some”. “Any” também representa quantidade indefinidas,
porém na interrogativa e na negativa. Assim, as alternatives A e C já estão descartadas. Como a
alternativa B não passou o indefinite article para o plural, embora tenha sentido, também não é
nossa resposta.
A alternativa E merece atenção, pois traz o erro num detalhe que pode passar despercebido se
olharmos rapidamente:” a smartwatchs or a Mac computers”. Não podemos usar o article no
singular e o noun no plural, eles devem concordar. Assim, nossa alternativa correta é D.
GABARITO: D

Texto para questões 05 a 08


FIVE WAYS TO GET MORE FIBRE IN YOUR DIET
Roughage helps reduce the risk of heart disease and bowel
cancer, yet few of us eat enough of it. Here’s how to up your
intake.
Fibre, or roughage, refers to indigestible carbohydrates. A fibre-
rich diet is linked to health benefits including a reduced risk of
heart disease and bowel cancer. While UK guidelines say adults
should get 30g a day, fewer than one in 10 meet this goal.
Popular low-carb diets may be a reason why. Understanding
what is in your food _____ help: a typical apple contains 2-3g of fibre, a sesame bagel about 4g.
Jo Greening, a spokesperson for the British Dietetic Association (BDA), says it is worth checking
the labels, as different brands have different levels of fibre.

AULA 10 – PASSIVE VOICE, TAG QUESTIONS AND IDIOMS 66


TEACHER ANDREA BELO

QUESTÃO 05 (COLÉGIO NAVAL/INÉDITA) – According to the text, a fibre diet


a) is linked to a lot of benefits
b) is linked to heart disease risk reduce
c) is linked to a few benefits
d) is linked to all health benefits
e) is linked to cancer disease prevention
Comentários: Essa questão analisa a frase “A fibre-rich diet is linked to health benefits including a
reduced risk of heart disease and bowel cancer.”
Vamos fazê-la por eleminação, começando pela alternativa D. Ela está descartada, pois afirma que
“fiber diet is linked to ALL health benefits”, o que não é coerente com a informação do texto. No
texto se afirma que existem benefícios advindos da fibe-rich diet, inclusive citam-se exemplos,
mas não há informação que nos permita concluir que esses são TODOS os helth benefits.
Agora vamos descartar a alternativa E. Novamente, temos um problema de generalização. Na
frase, cita-se “bowel cancer” como exemplo, mas não há informação que nos permita concluir que
há redução de risco para todos os cânceres. Por isso, a alternative E está incorreta.
As alternatives A e C estão descartadas pelo mesmo motivo: introduzem quantificação ao noun
benefits, porém a frase original não nos permite concluir se são “lots of” ou “few” benefits, já que
não essa quantificação.
Assim, concluímos que a alternativa correta é B, que traz corretamente um dos exemplos da frase,
sobre o benefício da redução do risco de doença cardíaca.
GABARITO: B

QUESTÃO 06 (COLÉGIO NAVAL/INÉDITA) – “… yet few of us eat enough of it. Here’s how to up
your intake.” The underlined word can be substituted for
a) even
b) but
c) besides
d) otherwise
e) then
Comentários: É necessário voltar ao texto e analisar mais do que apenas o trecho mostrado para
compreendermos o significado de “yet” nesse caso. O trecho em questão é: “Roughage helps
reduce the risk of heart disease and bowel cancer, yet few of us eat enough of it.”
A parte anterior à vírgula traz uma afirmação de que fiber, ou roughage, faz bem à saúde. Depois
do “yet” há uma segunda afirmação de que poucos de nós comem a quantidade suficiente de
fiber. Essas duas afirmações estabelecem entre si uma relação de oposição, pois deveríamos

AULA 10 – PASSIVE VOICE, TAG QUESTIONS AND IDIOMS 67


TEACHER ANDREA BELO

comer preferencialmente os alimentos que nos fazem bem. Assim sendo, essa relação e oposição
é justamente a função do “yet”.
Para encontrarmos a alternativa correta, precisamos de outra conjunção adversativa, que, no caso,
é “but”, alternative B. As alternatives A e C estão incorretas, pois “even” e “besides” trazem ideia
de inclusão.
A alternative D está incorreta, pois corresponderia à expressão “caso contrário”, o que não se
encaixaria no sentido a frase. A alternative E está incorreta, pois “then” traz ideia de conclusão.
GABARITO: B

QUESTÃO 07 (COLÉGIO NAVAL/INÉDITA) – The word bowel (line 2) means in the text
a) gut
b) chin
c) elbow
d) bladder
e) thigh
Comentários: Essa questão trabalha principalmente conhecimento de vocábulos. Guarde as
palavras que não conheça e assim, aumente seu vocabulário. “Bowel” é sinônimo de “gut” ou
“intestin”. Por isso, a alternativa correta é A.
Vou trazer o significado das outras alternativas para aumentarmos nosso vocabulário!
Lembre-se de anotar as novas palavras! “Chin” é queixo, “Elbow” é cotovelo, “Bladder” é bexiga
e “Thigh” é coxa.
GABARITO: A

QUESTÃO 08 (COLÉGIO NAVAL/INÉDITA) – The space in the text, “Understanding what is in your
food _____ help.” Can be completed with
a) should
b) could
c) can
d) ought to
e) must
Comentários: A alternativa correta deve indicar possibilidade, pois a ideia da frase é que o
entendimento do que a comida contém pode ajudar. Dessa forma, a alternativa A está descartada,
pois o modal verb “should” traz ideia de conselho.
As alternativas D e E também estão descartadas, pois “ought to” e “must” trazem ideia de
obrigação. Tanto B quanto C trazem ideia de possibilidade, sendo B no passado e C no presente.

AULA 10 – PASSIVE VOICE, TAG QUESTIONS AND IDIOMS 68


TEACHER ANDREA BELO

Devido ao tempo verbal do verb to be “is” da frase estar no presente, a opção mais adequada é C
para manter a coerência da frase no tempo.
GABARITO: A
Texto para questões de 09 e 10
A solicitor engaged by former members of the Supreme Horse Racing Club
said on Thursday evening he was “very optimistic” the 29 horses it has
hitherto owned, including the top-class chaser Kemboy, will be able to race
this season.
The Thurles-based Patrick Kennedy was speaking after the ruling body of
Irish horse racing in effect kicked SHRC out of the sport by declaring it was
no longer “permitted to be owners or part-owners of racehorses”.
The announcement from Horse Racing Ireland represents the moment
when its patience expired over the club’s repeated failure to provide answers to its questions.
Following complaints from club members about alleged irregularities in the way SHRC was run,
HRI got involved this summer and its concerns have not been allayed.

QUESTÃO 09 (COLÉGIO NAVAL/INÉDITA) – Mark the incorrect statement according to the text
a) Um advogado entrou para o Supreme Horse Racing Club.
b) Um membro do Supreme Horse Racing Club está muito engajado no projeto.
c) Foi incluído um advgado para o Supreme Horse Racing Club.
d) O Supreme Horse Racing Club incluiu um advogado.
e) Um advogado foi contratado por membros do Supreme Horse Racing Club.
Comentários: Essa questão trabalha o vocábulo “solicitor”, que é sinônimo de “lawyer”. É
necessário entender corretamente a palavra para entender corretamente o começo do texto.
As alternativas A, C e D estão incorretas, pois essa informação não consta no texto. O advogado
está defendendo membros/ex-membros da SHRC frente à acusações do Horse Racing Ireland.
A alternativa B está incorreta, pois o adjective “engaged” se refere ao “solicitor”, não a members
do SHRC.
A alternativa E está correta, pois essa é a afirmação inicial do texto: um solicitor engaged by former
members of SHRC is optimistic.
GABARITO: E

QUESTÃO 10 (COLÉGIO NAVAL/INÉDITA) – In the third paragraph“… Following complaints from


club members about alleged irregularities in the way SHRC was run, …”
The verb “run” refers to:
a) in a hurry irregularities
b) in a hurry action

AULA 10 – PASSIVE VOICE, TAG QUESTIONS AND IDIOMS 69


TEACHER ANDREA BELO

c) something urgent
d) a kind of function
e) the behave of something
Comentários: Aqui temos uma questão que busca interpretar a aplicação de um vocábulo em um
contexto. Geralmente, “run” significa “correr” , porém, aqui, temos o sentido de “gerenciar” - “...
in the way SHRC was run...” = a forma que SHRC era gerenciado. Assim, a alternativa correta é E.
As alternativas A e B estão incorretas, pois “run” não se refere aos nouns “irregularities” e “action”.
A alternativa C está incorreta, pois, nesse caso, o sentido de “run” não tem a ver com urgência.
A alternativa D está incorreta, pois o noun function remete à ideia do “gerente”, mas não do verb
“run”, nesse caso. Importante não cair nesse detalhe!
GABARITO: E

QUESTÕES EAM
QUESTÃO 01 (EAM/INÉDITA) – Use the verbs in parentheses to complete the following
statements.
I. __________ (go) there! It’s dangerous!
II. Are you ready __________ (go) home?
III. She never __________ (go) to the beach in the summer.
IV. I guess they __________ (go) away tomorrow.
V. Mr. Allen __________ (go) to a party two days ago.
VI. He __________ (go) to school right now.
Now mark the option which completes them respectively.
a) Not go / go / go / ’ll go / goes / going
b) Don’t go / to go / goes / ’ll go / went / ’s going
c) Don’t go / to go / gos / go / went / is going
d) Not go / to go / goes / will go / will go / is going
e) Don’t go / go / goes / went / went / going
Comentários: I. DON’T go there! It’s dangerous! Imperativo na negativa, basta usar o don’t antes
do verbo- não vá lá!
II. Are you ready TO GO home? Você está pronto para ir para casa? Após ready, usaremos o verbo
no infinitivo, com to.
III. She never GOES to the beach in the summer. O verbo go no Simple Present com a terceira
pessoa do singular (she) fica modificado – goes.
IV. I guess they’ll GO away tomorrow. Tomorrow (amanhã) nos mostra que é tempo futuro, por
isso o uso do will, indicativo de futuro.
V. Mr. Allen WENT to a party two days ago. Two days ago (dois dias atrás) indica que é tempo
passado, por isso o uso da forma do passado de go, went.

AULA 10 – PASSIVE VOICE, TAG QUESTIONS AND IDIOMS 70


TEACHER ANDREA BELO

VI. He’s GOING to school right now. A expressão right now (agora mesmo, neste exato momento)
indica que a ação está acontecendo no momento da fala, daí o uso do Present Continuous.
GABARITO: B
QUESTÃO 02 (EAM/INÉDITA) – Look at the picture below.

Which option correctly describes the picture above? Consider that the action takes place in the
future.
a) Mark and Chris went to the beach.
b) Mark and Chris will go skiing.
c) Mark and Chris are going to go to the beach.
d) Mark and Chris go to the beach.
e) Mark and Chris went skiing.
Comentários: Alternativa A: Considerando que a ação se passa no futuro, como manda a questã,
é incorreto descrevê-la dizendo que eles foram à praia (went to the beach). ERRADA.
Alternativa B: Essa alternativa trouxe uma opção correta de verbo, no futuro (will go), porém,
não descreve a imagem corretamente, pois eles não irão esquiar. Estão equipados para ir à praia.
ERRADA.
Alternativa C: Essa alternativa acertou quanto ao uso do verbo- usou outra opção de futuro, o
futuro com be going to, e acertou ao descrever a imagem, ao dizer que eles irão para a praia.
CORRETA.
Alternativa D: O verbo go, no caso, está sendo usado no Simple Present (eles vão), não
posicionando a ação no futuro. ERRADA.
Alternativa E: Essa alternativa errou quanto ao tempo verbal (usou passado- went) e errou quanto
à descrição da imagem (irão à praia, não esquiar). ERRADA.
GABARITO: C

QUESTÃO 03 (EAM/INÉDITA) – Fill in the blanks with the correct prepositions of time
respectively.
I. I never go to bed late __________ night.

AULA 10 – PASSIVE VOICE, TAG QUESTIONS AND IDIOMS 71


TEACHER ANDREA BELO

II. They have karate classes __________ Wednesdays.


III. My birthday is __________ April.
IV. My birthday is __________ April 7.
a) at / on / in / on
b) at / on / in / in
c) at / at / on / on
d) in / at / in / in
e) on / on / in / in
Comentários: I. I never go to bed late AT night. A preposição correta com a palavra night para
indicar à noite é sempre at- at night.
II. They have karate classes ON Wednesdays. A preposição correta para dias da semana é sempre
ON. Dica: lembre-se de MONDAY, que tem a preposição on “dentro”.
III. My birthday is IN April. A preposição correta para meses do ano é sempre IN.
IV. My birthday is ON April 7. Mas, se junto com o mês, eu falo também o dia, prevalece a
preposição de dia, que é ON.
GABARITO: A

QUESTÃO 04 (EAM/INÉDITA) – Read the dialog and mark the right option to fill the gaps
respectively.
A: Rick, look! Is __________ wallet here with me __________?
B: Uh... No, __________ is not __________. Maybe you should ask Lisa. It definitely looks like
__________ wallet.
a) this / your / this / mine / hers
b) these / yours / that / my / her
c) that / yours / that / mine / her
d) this / yours / this / my / hers
e) this / yours / that / mine / her
Comentários: A: Rick, look! Is THIS wallet here with me YOURS? Se a carteira está próxima de
quem fala (aqui comigo- here with me), o pronome demonstrativo correto no singular é this. Para
perguntar se é “sua”, temos que usar o pronome possessivo yours, que é o que usamos quando
o substantivo está implícito- observe que wallet não foi repetido – “esta carteira aqui comigo é
sua (carteira)?” Wallet está implícito.
B: Uh... No, THAT is not MINE. Maybe you should ask Lisa. It definitely looks like HER wallet.
Lembra que a carteira está com quem perguntou? Quem responde, então, responde com o
demonstrativo that, que é usado para quando o objeto está longe de quem fala. Para falar que “a
carteira não é minha”, vamos usar o mesmo raciocínio que fizemos acima- temos que usar o
pronome possessivo mine, que é o que usamos quando o substantivo está implícito – “não, essa

AULA 10 – PASSIVE VOICE, TAG QUESTIONS AND IDIOMS 72


TEACHER ANDREA BELO

carteira não é minha (carteira)”. Wallet está implícito. Por sua vez, para falar “a carteira dela”,
usamos o adjetivo possessivo her, pois nesse caso, o substantivo está explícito, foi mencionado-
her wallet.
GABARITO: E
QUESTÃO 05 (EAM/INÉDITA) – Look at the picture below.

Adapted from https://englishoutofthebox.blogspot.com/2019/01/what-do-you-know-about-modal-verbs.html

In the picture, “can” is used to express


a) an ability.
b) a request.
c) an order.
d) a possibility.
e) a prediction.
Comentários: Alternativa A: Um dos possíveis usos do modal verb CAN é para expressar
habilidade, falar de algo que você sabe ou não fazer. Exemplo: I can drive. I can’t cook. Na figura,
o menino está fazendo um pedido, e não falando de uma habilidade. ERRADA.
Alternativa B: Na imagem, o garoto está fazendo um pedido- a request. “Posso ir ao banheiro?”.
É um uso bastante frequente do CAN. CORRETA.
Alternativa C: CAN não é usado, de forma geral, para expressar uma ordem, um comando.
ERRADA
Alternativa D: CAN pode ser usado para indicar uma possibilidade. Exemplo: I can get home in 10
minutes. Porém, na imagem, o garoto está fazendo um pedido, pedindo permissão. ERRADA.
Alternativa E: CAN pode ser usado para fazer uma previsão, especialmente na sua forma negativa.
Exemplo: That can’t be true! (isso não pode ser verdade!). Como vimos, não é esse o sentido em
que CAN foi usado pelo menino. ERRADA.
GABARITO: B

AULA 10 – PASSIVE VOICE, TAG QUESTIONS AND IDIOMS 73


TEACHER ANDREA BELO

QUESTÕES EEAR
Texto para questões de 01 a 04
(Título omitido propositalmente)

QUESTÃO 01 (EEAR/INÉDITA) – Choose the most appropriate title for the text:
a) How to teach English.
b) Teaching English in Brazil.
c) Oportunities and flexibility.
d) Professional oportunities.
e) How to deal with English as a language.
Comentários: Na letra A, “Como ensinar inglês” não é o título mais apropriado para o texto. Afinal,
ele não se trata de como ensinar o inglês, mas tudo o que pode vir de bom quando se ensina
inglês – “Teaching English is a rewarding profession that can enable you...” (Ensinar inglês é uma
profissão gratificante que pode te permitir...). Alternativa incorreta.
Na letra B, “Ensinar inglês no Brasil” não é o título mais apropriado para o texto. Em nenhum
momento o texto afirma que o contexto se trata do ensino de inglês no Brasil, até porque o país
sequer é citado. Alternativa incorreta.
Na letra C, “Oportunidades e flexibilidade” não é o título mais apropriado para o texto. Apesar de
o texto conter “opportunity” e “flexibility”, isso é apenas uma ideia específica do trecho, e não a
ideia geral.
Em “It will give you the opportunity to learn valuable skills, such as problem solving,
communication and flexibility” (Isso [ensinar inglês] te dará a oportunidade de aprender
habilidades valiosas, como resolução de problemas, comunicação e flexibilidade). Note que uma
coisa é consequência da outra, mas os termos soltos não fariam sentido para o todo. Alternativa
incorreta.

AULA 10 – PASSIVE VOICE, TAG QUESTIONS AND IDIOMS 74


TEACHER ANDREA BELO

Na letra D, “Oportunidades profissionais” é o título mais apropriado para o texto. Afinal, ele se
trata das oportunidades que ensinar inglês pode te proporcionar, como trabalhar e viver em
lugares do mundo todo, aprender e desenvolver habilidades valiosas, e descobrir diferentes
países e culturas. Alternativa correta.
Na letra E, “Como lidar com o inglês como língua” não é o título mais apropriado para o texto.
Afinal, ele não se trata de lidar com o inglês como língua, mas das vantagens de se ensinar o inglês
– o que pode te dar muitas oportunidades, como trabalhar em lugares do mundo todo, aprender
e desenvolver habilidades e descobrir novas culturas. Alternativa incorreta.
GABARITO: D

QUESTÃO 02 (EEAR/INÉDITA) – In the sentence “Teaching English is a rewarding profession that


can enable…”, rewarding means all those words, except:
a) profitable.
b) rad.
c) fruitful.
d) tip top.
e) vain.
Comentários: O enunciado pede a exceção dentre as palavras que são sinônimas de “rewarding”,
ou seja, “gratificante”.
Na letra A, o termo “profitable” é “lucrativo” corresponde a “rewarding”. Portanto, alternativa
incorreta.
Na letra B, o termo “rad” é “bom, gíria maneiro, legal”” corresponde a “rewarding”. Portanto,
alternativa incorreta.
Na letra C, o termo “ fruitful” é “frutuoso” corresponde a “rewarding”. Portanto, alternativa
incorreta.
Na letra D, o termo “tip top” é “vantajoso, excelente” corresponde a “rewarding”. Portanto,
alternativa incorreta.
Na letra E, “vão” não corresponde a “rewarding”, pois é o mesmo que “inútil”. Portanto,
alternativa correta.
GABARITO: E

QUESTÃO 03 (EEAR/INÉDITA) – Read the extract from the text


“It will give you the opportunity to learn valuable skills…”
The pronoun “it” refers to the:
a) rewarding profession.
b) fascinating places.
c) problem solving.

AULA 10 – PASSIVE VOICE, TAG QUESTIONS AND IDIOMS 75


TEACHER ANDREA BELO

d) English.
e) the world.
Comentários: Antes do trecho citado, temos “Teaching English is a rewarding profession that can
enable you to live and work in fascinating places all over the world” (Ensinar inglês é uma profissão
gratificante que pode te permitir viver e trabalhar em lugares fascinantes ao redor do mundo). “It
will give you the opportunity to learn valuable skills…” (Ela te dará a oportunidade de aprender
habilidades valiosas).
Na letra A, “profissão gratificante” está correto, pois ela é o sujeito das duas orações. Ou seja,
além de ela poder “te permitir viver e trabalhar em lugares fascinantes ao redor do mundo”, ela
também “te dará a oportunidade de aprender habilidades valiosas”. Alternativa correta.
Na letra B, “lugares fascinantes” é incorreto, pois além de não corresponder ao contexto, os
termos em plural deveriam ser referenciados como “they”, e não “it”. Alternativa incorreta.
Na letra C, “resolução de problemas” é incorreto, pois esses termos são citados após “It”,
portanto, não há como o pronome se referir a eles. Alternativa incorreta.
Na letra D, “inglês” é incorreto, pois o contexto nos mostra que não é apenas o “inglês” que “te
dará a oportunidade de aprender habilidades valiosas”, mas a profissão gratificante como
professor de inglês. Alternativa incorreta.
Na letra E, “do mundo” é incorreto. Apesar de “the world” anteceder “It” e estarem muito
“próximos”, o contexto nos mostra que não é o mundo que “te dará a oportunidade de aprender
habilidades valiosas”, mas a profissão gratificante como professor de inglês. Alternativa incorreta.
GABARITO: A

QUESTÃO 04 (EEAR/INÉDITA) – According to the text, choose the correct statement


a) Flexibility is one of the advantages of teaching English.
b) Communication is one of the advantages of trading lore in English.
c) Problem solving is one of the advantages of trading lore in English.
d) Experience is one of the advantages of teaching English.
e) Flexibility is one of the advantages of learning English.
Comentários: Na letra A, “Flexibilidade é uma das vantagens de se ensinar inglês”, alternativa
correta, afirmada no texto.
Na letra B, “Comunicação é uma das vantagens de se aprender inglês” é incorreto, já que “trading
lore”é aprender, trocar tradições, trocar informações. O texto afirma que ensinar inglês (Teaching
English) te dará a oportunidade de aprender habilidades valiosas, como resolução de problemas,
comunicação e flexibilidade. Portanto, essa é uma das vantagens de se ensinar inglês, não
aprender. Alternativa incorreta.
Na letra C, “Resolução de problemas é uma das vantagens de se aprender inglês”, já que vimos,
anteriormente, que “trading lore” é aprender, trocar tradições, trocar informações, o que seria,
na verdade, vantagem de se ensinar Inglês. Incorreta.

AULA 10 – PASSIVE VOICE, TAG QUESTIONS AND IDIOMS 76


TEACHER ANDREA BELO

Na letra D, “Experiência é uma das vantagens de se ensinar inglês” é incorreto. O texto afirma que
assim como desenvolver habilidades profissionais, você também consegue conhecer (to
experience) diferentes países e culturas. Portanto, “experiência” por si só, segundo o texto, não é
uma das vantagens de se ensinar inglês (note que experience no texto não equivale a um
substantivo, mas a um verbo). Alternativa incorreta.
Na letra E, “Flexibilidade é uma das vantagens de se aprender inglês” é incorreto. O texto afirma
que ensinar inglês (Teaching English) te dará a oportunidade de aprender habilidades valiosas,
como resolução de problemas, comunicação e flexibilidade. Portanto, essa é uma das vantagens
de se ensinar inglês, não aprender. Alternativa incorreta.
GABARITO: A

Texto para questões 05 a 08


Taal: The 'very small but dangerous volcano'

Over the past few days, it's begun spewing lava, triggering earthquakes and emitting huge plumes
of ash that have spread across the island of Luzon and beyond.
Scientists fear a bigger "hazardous eruption" is imminent. Taal is tiny, as volcanoes go, but it has
been deadly before. And according to Renato Solidum, the head of the Philippines' Institute of
Volcanology and Seismology (Philvolcs), it is "very small but a dangerous volcano". "Taal volcano
is a baby volcano sitting within a much bigger caldera volcano," said Ben Kennedy, associate
professor of physical volcanology at the University of Canterbury in New Zealand. The entire
Volcano Island has been marked as a permanent danger zone by Phivolcs.
BBC News (Jan/2020)

QUESTÃO 05 (EEAR/INÉDITA) – According to the text,


a) Taal is small and very dangerous.
b) Taal is huge and menacing.
c) Taal is tiny and sheltered.
d) Taal is huge and hazardous.
e) Taal is huge and treacherous.

AULA 10 – PASSIVE VOICE, TAG QUESTIONS AND IDIOMS 77


TEACHER ANDREA BELO

Comentários: Na letra A, “O Taal é pequeno e muito perigoso” está correto. Segundo o texto
(linhas 5-6), “it is ‘very small but a dangerous volcano’” (um vulcão muito pequeno, mas perigoso).
Alternativa correta.
Na letra B, “O Taal é enorme e ameaçador” é incorreto. O texto se refere ao Taal como “tiny”
(pequeno) ou “very small” (muito pequeno), e não como “huge” (enorme). Alternativa incorreta.
Na letra C, “O Taal é pequeno e protegido” é incorreto. Segundo o texto, o Taal é pequeno (tiny;
small), mas não há a firmação de que ele seja protegido. Alternativa incorreta.
Na letra D, “O Taal é enorme e perigoso” é incorreto. Segundo o texto, o Taal é perigoso (“very
small but a dangerous volcano”, ou, “um vulcão muito pequeno, mas perigoso”), mas não é
enorme. Alternativa incorreta.
Na letra E, “O Taal é enorme e traiçoeiro” é incorreto. O texto se refere ao Taal como pequeno
(tiny; small) e perigoso (dangerous). Alternativa incorreta.
GABARITO: A

QUESTÃO 06 (EEAR/INÉDITA) – “… it's begun spewing lava, triggering earthquakes.”


The underlined words mean
a) deliberate and raze.
b) deliberate and defeat.
c) deliberate and ruin.
d) deliberate and tear down.
e) deliberate and incite.
Comentários: Na letra A, “deliberar e destruir” é incorreto; “destruir” não corresponde a
“triggering” (causando/causar). Alternativa incorreta.
Na letra B, “deliberar e derrotar” é incorreto; “derrotar” não corresponde a “triggering”
(causando/causar). Alternativa incorreta.
Na letra C, “deliberar e arruinar” é incorreto; “arruinar” não corresponde a “triggering”
(causando/causar). Alternativa incorreta.
Na letra D, “deliberar e derrubar” é incorreto; “derrubar” não corresponde a “triggering”
(causando/causar). Alternativa incorreta.
Na letra E, “deliberar e incitar” está correto; “incitar” corresponde a “triggering”
(causando/causar). Alternativa correta.
GABARITO: E

QUESTÃO 07 (EEAR/INÉDITA) – What is the question the author refers when he says: “… but it
has been deadly before”
a) Was it always dangerous?
b) Is it considered really dangerous?
c) Is it dangerous?

AULA 10 – PASSIVE VOICE, TAG QUESTIONS AND IDIOMS 78


TEACHER ANDREA BELO

d) Has it always been dangerous?


e) Has it caused any danger before?
Comentários: O enunciado diz: “... mas ele foi mortal antes”
Na letra A, “Ele sempre foi perigoso?” é incorreto. A pergunta deve equivaler ao mesmo tempo
verbal utilizado na resposta. Neste caso, temos a presença de present perfect continuous (has
been), e não do simple past (was). Alternativa incorreta.
Na letra B, “Ele é considerado periogoso?” é incorreto. A pergunta deve equivaler ao mesmo
tempo verbal utilizado na resposta. Neste caso, temos a presença de present perfect continuous
(has been), e não do simple presente (is). Alternativa incorreta.
Na letra C, “Ele é perigoso?” é incorreto. A pergunta deve equivaler ao mesmo tempo verbal
utilizado na resposta. Neste caso, temos a presença de present perfect continuous (has been), e
não do simple present (is). Alternativa incorreta.
Na letra D, “Ele sempre foi perigoso?” (forma verbal com has been) é incorreto. Apesar de o tempo
verbal da pergunta ser o mesmo da resposta, o advérbio “always” (sempre) não corresponde ao
fato apresentado como before (antes). Alternativa incorreta.
Na letra E, “Ele causou algum dano antes?” está correto. Tanto as formas verbais quanto a
indicação do advérbio de tempo são correspondentes (present perfect continuous e “before”).
Alternativa correta.
GABARITO: E

QUESTÃO 08 (EEAR/INÉDITA) – What kind of text is it?


a) Jornalistic biography.
b) Journalistic letter.
c) News nature tale.
d) Nature book review.
e) An article.
Comentários: A pergunta é “Que tipo de texto é este?”
Na letra A, “Biografia jornalística” é incorreto. Não há sinais de biografia, pois o texto fala sobre a
atividade de um vulcão. Alternativa incorreta.
Na letra B, “Carta do leitor” é incorreto. O texto não expressa a opinião de leitores em um jornal
– afinal, trata-se de fatos sobre o vulcão Taal. Alternativa incorreta.
Na letra C, “Conto sobre notícias da natureza” é incorreto. O texto não é um conto literário, e isso
fica claro pelos fatos apresentados sobre a atividade de um vulcão veiculado pela BBC News.
Alternativa incorreta.
Na letra D, “Resenha de livro sobre a natureza” é incorreto. O texto não se trata de uma resenha,
mas de fatos sobre o vulcão Taal. Alternativa incorreta.

AULA 10 – PASSIVE VOICE, TAG QUESTIONS AND IDIOMS 79


TEACHER ANDREA BELO

Na letra E, “Artigo” está correto. O título do texto indica a fonte: BBC News. Assim, podemos
concluir que é um jornal veiculando uma notícia sobre o vulcão Taal, “O ‘vulcão muito pequeno,
mas perigoso’”. Alternativa correta.
GABARITO: E
Texto para questões de 09 e 10
A solicitor engaged by former members of the Supreme Horse Racing
Club said on Thursday evening he was “very optimistic” the 29 horses
it has hitherto owned, including the top-class chaser Kemboy, will be
able to race this season.
The Thurles-based Patrick Kennedy was speaking after the ruling body
of Irish horse racing in effect kicked SHRC out of the sport by declaring
it was no longer “permitted to be owners or part-owners of
racehorses”.
The announcement from Horse Racing Ireland represents the moment when its patience expired
over the club’s repeated failure to provide answers to its questions.
The Guardian – November 14th

QUESTÃO 09 (EEAR/INÉDITA) – According to the text, the term solicitor means


a) A lawyer for the Supreme Horse Racing Club.
b) A new member to manage the Horse Racing Club.
c) Supreme Horse Racing Club’s new principal.
d) A candidate for the next Supreme Horse Racing Club elections.
e) A administrator for the Supreme Horse Racing Club.
Comentários: Na letra A, “Um advogado” está correto. O texto afirma que o solicitor foi “engaged
by former members of the Supreme Horse Racing Club” (integrado pelos membros antigos do
Clube Supremo de Corrida de Cavalos). Ou seja, o advogado dos antigos membros foi solicitado
para que os 29 cavalos até o momento pudessem correr. Alternativa correta.
Na letra B, “Um novo membro de gerenciamento” é incorreto. “Solicitor” corresponde a
“advogado”. Alternativa incorreta.
Na letra C, “O novo diretor” é incorreto. “Solicitor” corresponde a “advogado”. Alternativa
incorreta.
Na letra D, “Um candidato para as próximas eleições” é incorreto. “Solicitor” corresponde a
“advogado”. Alternativa incorreta.
Na letra E, “Um administrador” é incorreto. “Solicitor” corresponde a “advogado”. Alternativa
incorreta.
GABARITO: A

AULA 10 – PASSIVE VOICE, TAG QUESTIONS AND IDIOMS 80


TEACHER ANDREA BELO

QUESTÃO 10 (EEAR/INÉDITA) – In the text, the sentence “… was speaking after the ruling body
of Irish horse racing in effect, …” means
a) after confirming horses in force.
b) after making sure all horses were effective.
c) after getting new information about the horses.
d) after reading the horses legislation.
e) after making sure abou the racing effects.
Comentários: Aqui, os termos principais são “ruling body”, que correspondem a “corpo
legislativo”. Ou seja,
Na letra A, “depois de confirmar os cavalos em curso” é incorreto. O contexto não corresponde
aos cavalos em si. Alternativa incorreta.
Na letra B, “depois de ter certeza de que todos os cavalos eram eficazes” é incorreto. O contexto
não corresponde à eficácia dos cavalos. Alternativa incorreta.
Na letra C, “depois de conseguir novas informações sobre os cavalos” é incorreto. O contexto não
corresponde a novas informações, mas mais especificamente à legislação. Alternativa incorreta.
Na letra D, “depois de ler a legislação sobre cavalos” está correto. O trecho afirma que “... estava
falando depois que o corpo legislativo da corrida de cavalos irlandesa efetivamente. Alternativa
correta.
Na letra E, “depois de ter certeza sobre os efeitos da corrida” é incorreto. O contexto não
corresponde aos efeitos que a corrida pode ter. Alternativa incorreta.
GABARITO: D

QUESTÕES EFOMM
Read the text and answer questions 01, 02 and 03.
Netflix edits Squid Game phone number after woman deluged with calls
Popular drama’s use of real South Korean number leads to thousands of prank calls and texts
Netflix has edited out a phone number that appears ___ its hit series Squid Game after a South
Korean woman and others who use similar combinations were deluged with calls – with some
callers even asking to join the show’s life-or-death games.
The South Korea-made production has topped Netflix popularity charts in 90 countries since its
launch last month and is on track to become its most watched series ever.

AULA 10 – PASSIVE VOICE, TAG QUESTIONS AND IDIOMS 81


TEACHER ANDREA BELO

The nine-episode drama, which addresses widening economic inequality, involves hundreds of
cash-strapped people competing ___ children’s games to win the final reward of 45.6bn Korean
won (£28m), with the losers killed in uncompromising and violent ways.
To take part, contestants have to call a number on a business card printed with symbols. But while
film and television makers usually use fake numbers in such circumstances, adding 010 – the
standard prefix ___ South Korean mobiles – to the eight digits on the card generated a real phone
number.
It belongs to a South Korean woman who said she received thousands of calls and text messages
to her phone “to the point that it’s hard for me to go on with daily life”.
(Adapted from https://www.theguardian.com/media/2021/oct/07/netflix-edits-squid-game-phone-number-woman-deluged-calls)

QUESTÃO 01 (EFOMM/INÉDITA) – According to the text, which option is correct?


(A) Netflix edited the phone number that appears in Squid Game to be a real number
(B) The new South Korean production has achieved stratospheric success
(C) Usually, film and television producers use real phone numbers
(D) The owner of the number accidentally released in the series did not suffer any kind of impact
(E) Only one woman suffered from the exposure of her real phone number
Comentários: A alternativa A está incorreta. De acordo com o texto, não é correto afirmar que a
Netflix editou o número de telefone que aparece no Squid Game para ser um número real, mas
sim, que editou após descobrir que era um número real. Isso pode ser confirmado com o trecho
“Netflix has edited out a phone number that appears in its hit series Squid Game after a South
Korean woman and others who use similar combinations were deluged with calls…”.
A alternativa B está correta. De acordo com o texto, é correto afirmar que a nova produção sul-
coreana alcançou sucesso estratosférico. Isso pode ser confirmado com o trecho “The South
Korea-made production has topped Netflix popularity charts in 90 countries since its launch last
month and is on track to become its most watched series ever”.
A alternativa C está incorreta. De acordo com o texto, não é correto afirmar que normalmente,
os produtores de cinema e televisão usam números de telefone reais, pois utilizam-se,
normalmente, números falsos. Isso pode ser confirmado com o trecho “...film and television
makers usually use fake numbers in such circumstances…”.
A alternativa D está incorreta. De acordo com o texto, não é correto afirmar que o dono do
número lançado acidentalmente na série não sofreu nenhum tipo de impacto, mas sim, que
atrapalhou o cotidiano da dona pois eram muitas mensagens e ligações. Isso pode ser confirmado
com o trecho “to the point that it’s hard for me to go on with daily life”.
A alternativa E está incorreta. De acordo com o texto, não é correto afirmar que apenas uma
mulher sofreu com a exposição de seu número de telefone real, pois outras pessoas com
combinações de número semelhantes também sofreram. Isso pode ser confirmado com o trecho
“…after a South Korean woman and others who use similar combinations were deluged with
calls…”.

AULA 10 – PASSIVE VOICE, TAG QUESTIONS AND IDIOMS 82


TEACHER ANDREA BELO

GABARITO: B

QUESTÃO 02 (EFOMM/INÉDITA) – Which is the correct option to complete the gaps in the text?
(A) In / in / for
(B) In / on / for
(C) On / in / for
(D) In / in / at
(E) In / on / at
Comentários: A primeira lacuna deve ser preenchida com a preposição “in” pois se refere ao
número aparecer na série, ou seja, “Netflix has edited out a phone number that appears in its hit
series Squid Game...” (A Netflix editou um número de telefone que aparece em sua série de
sucesso Squid Game...).
A segunda lacuna deve ser preenchida com a preposição “in” pois se refere às pessoas competino
em jogos de crianças, ou seja, “...involves hundreds of cash-strapped people competing in
children’s games...” (...envolve centenas de pessoas sem dinheiro competindo em jogos
infantis...).
A terceira lacuna deve ser preenchida com a preposição “for” pois indica o prefixo para celulares
sul coreanos, ou seja, “...the standard prefix for South Korean mobiles...” (...o prefixo padrão para
celulares sul-coreanos...).
GABARITO: A

QUESTÃO 03 (EFOMM/INÉDITA) – What’s the meaning of the word “deluged” in the title?
(A) Deluded
(B) Shocked
(C) Overwhelmed
(D) Rested
(E) Empty
Comentários: A alternativa A está incorreta. A palavra “deluged” significa “inundado” e não pode
ser comparada com a palavra “deluded”, que significa “iludido”.
A alternativa B está incorreta. A palavra “deluged” significa “inundado” e não pode ser comparada
com a palavra “shocked”, que significa “chocado”.
A alternativa C está correta. A palavra “deluged” significa “inundado” e pode ser comparada com
a palavra “overwhelmed”, que significa “sobrecarregado”.

AULA 10 – PASSIVE VOICE, TAG QUESTIONS AND IDIOMS 83


TEACHER ANDREA BELO

A alternativa D está incorreta. A palavra “deluged” significa “inundado” e não pode ser comparada
com a palavra “rested”, que significa “descansado”.
A alternativa E está incorreta. A palavra “deluged” significa “inundado” e não pode ser comparada
com a palavra “empty”, que significa “vazio”.
GABARITO: C

Read the text and answer questions 04 and 05.


New tool identifies groups most at-risk from Covid after vaccination
While the risk of severe Covid-19 after vaccination remains low, some people remain more
vulnerable than others, research shows
The immunosuppressed and those with dementia, Parkinson’s or chronic disorders such as kidney
disease are still at a greater risk of hospitalisation or death from Covid after vaccination compared
to the rest of the population, new research shows.
The QCovid tool developed by scientists at the University of Oxford shows that while the risk of
severe Covid-19 after vaccination remains low, some people remain more vulnerable than others.
Age continues to be a “major risk factor”, the experts said, with older vaccinated individuals more
likely to end up ill from Covid compared to their younger counterparts.
Those from Indian and Pakistani backgrounds who are vaccinated face the highest risk among
ethnic groups.
People living with chronic conditions such as Down’s syndrome, kidney disease, sickle cell disease,
HIV/Aids and liver cirrhosis are also at more risk of severe disease or hospital admission after
vaccination.
The new algorithm behind the QCovid tool, which was first developed in 2020 and helped
influence policy on shielding, predicts those most at risk of serious Covid-19 outcomes from 14 or
more days after second vaccination dose, when substantial immunity is expected to have
developed. The findings do not take into account the different types of vaccines received by
people.
(Adapted from https://www.independent.co.uk/news/health/covid-vaccine-at-risk-latest-b1922196.html)

QUESTÃO 04 (EFOMM/INÉDITA) – It is possible to infer from the text that


(A) After complete vaccination, there are no more risk groups
(B) The risk of developing a severe response to Covid-19 after full vaccination is greater
(C) Indians and Pakistanis are at increased risk against Covid-19 because of their genetic
conditions
(D) The QCovid tool considers the post-vaccination immunization period to achieve its results
(E) The QCovid tool compares the effectiveness of different vaccines for Covid-19

AULA 10 – PASSIVE VOICE, TAG QUESTIONS AND IDIOMS 84


TEACHER ANDREA BELO

Comentários: A alternativa A está incorreta. Não é possível inferir do texto que após a vacinação
completa, não há mais grupos de risco, pois ainda existem. Isso pode ser confirmado com o trecho
“...immunosuppressed and those with dementia, Parkinson’s or chronic disorders such as kidney
disease are still at a greater risk of hospitalisation or death from Covid after vaccination compared
to the rest of the population…”.
A alternativa B está incorreta. Não é possível inferir do texto que o risco de desenvolver uma
resposta grave ao Covid-19 após a vacinação completa é maior, pois é menor. Isso pode ser
confirmado com o trecho “...while the risk of severe Covid-19 after vaccination remains low…”.
A alternativa C está incorreta. Não é possível inferir do texto que indianos e paquistaneses correm
maior risco contra a Covid-19 por causa de suas condições genéticas, pois esses grupos estão em
maior risco por questões étnicas, o texto não especifica condições genéticas. Isso pode ser
confirmado com o trecho “Those from Indian and Pakistani backgrounds who are vaccinated face
the highest risk among ethnic groups”.
A alternativa D está correta. É possível inferir do texto que a ferramenta QCovid considera o
período de imunização pós vacinação para atingir seus resultados. Isso pode ser confirmado com
o trecho “…predicts those most at risk of serious Covid-19 outcomes from 14 or more days after
second vaccination dose, when substantial immunity is expected to have developed”.
A alternativa E está incorreta. Não é possível inferir do texto que a ferramenta QCovid compara a
eficácia de diferentes vacinas para Covid-19, pois a ferramenta não considera o tipo de vacina
tomada pelo indivíduo. Isso pode ser confirmado com o trecho “The findings do not take into
account the different types of vaccines received by people”.
GABARITO: D

QUESTÃO 05 (EFOMM/INÉDITA) – In the excerpt “Those from Indian and Pakistani backgrounds
who are vaccinated face the highest risk among ethnic groups” (paragraph 4), the word in bold
means
(A) Hide
(B) Finish
(C) Encounter
(D) Avoid
(E) Support
Comentários: A alternativa A está incorreta. Nesse caso, a palavra “face” significa “enfrentar” e
não pode ser comparada com a palavra “hide”, que significa “ocultar”.
A alternativa B está incorreta. Nesse caso, a palavra “face” significa “enfrentar” e não pode ser
comparada com a palavra “finish”, que significa “finalizar”.
A alternativa C está correta. Nesse caso, a palavra “face” significa “enfrentar” e pode ser
comparada com a palavra “encounter”, que significa “encontrar”.
A alternativa D está incorreta. Nesse caso, a palavra “face” significa “enfrentar” e não pode ser
comparada com a palavra “avoid”, que significa “evitar”.

AULA 10 – PASSIVE VOICE, TAG QUESTIONS AND IDIOMS 85


TEACHER ANDREA BELO

A alternativa E está incorreta. Nesse caso, a palavra “face” significa “enfrentar” e não pode ser
comparada com a palavra “support”, que significa “apoiar”.
GABARITO: C

QUESTÃO 06 (EFOMM/INÉDITA) – Choose the correct alternative to complete the paragraph


below
“Across the UK, children are preparing to return to school after almost two months ___ summer
holidays. But experts have warned that the number of coronavirus cases may surge ___ the
coming weeks, as more children and adults mix inside the classroom. Professor Jason Leitch,
Scotland’s national clinical director, told Sky News on Tuesday that the UK could be facing a ‘fragile
moment’ ___ its response to the ongoing pandemic, as the Delta variant continues to spread, and
lockdown rules remain relaxed”.
(Adapted from https://www.independent.co.uk/life-style/health-and-families/coronavirus-children-school-testing-b1912550.html)

(A) At / in /on
(B) At / on / in
(C) Of / in / on
(D) Of / at / in
(E) Of / in / in
Comentários: A primeira lacuna deve ser preenchida com a preposição “of”, pois se refere aos
quase dois meses de férias de verão, ou seja, “...after almost two months of summer holidays”
(...depois de quase dois meses de férias de verão).
A segunda lacuna deve ser preenchida com a preposição “in”, pois se refere a alta de casos que
pode surgir nas próximas semanas no Reino Unido, ou seja, “...the number of coronavirus cases
may surge in the coming weeks...” (...o número de casos de coronavírus pode aumentar nas
próximas semanas...).
A terceira lacuna deve ser preenchida com a preposição “in”, pois se refere ao possível momento
delicado que o Reino Unido pode encontrar em sua resposta à pandemia, ou seja, “...the UK could
be facing a ‘fragile moment’ in its response to the ongoing pandemic...” (...o Reino Unido pode
estar enfrentando um ‘momento frágil’ em sua resposta à pandemia em curso...).
GABARITO: E

Read the text and answer questions 07, 08, 09 and 10.
Malaria vaccine: When will it be available?
Researchers and health professionals have been celebrating after the World Health
Organization (WHO) approved the widespread use of the world's first malaria vaccine.

AULA 10 – PASSIVE VOICE, TAG QUESTIONS AND IDIOMS 86


TEACHER ANDREA BELO

With more than 260,000 children under five dying from malaria each year in sub-Saharan Africa,
this development, decades in the making, could save tens of thousands of lives, the WHO says.
But when will people start benefitting from the vaccine known as RTS,S?
We have been looking at that and some other key questions.
How effective and safe is it?
The vaccine was proven effective six years ago, preventing 40% of malaria cases and 30% of severe
cases.
Since 2019, researchers have been carrying out wider pilot immunisation programmes in Ghana,
Kenya, and Malawi.
More than 800,000 children have received at least one dose and the WHO says there are no safety
concerns.
Does it matter that the protection is relatively low?
It would obviously be better if it was higher, but what many would say is you need to think about
the scale of the problem – with hundreds of millions of cases, a 40% reduction is still a huge
number of lives saved.
"This is a moderately efficacious vaccine… [but] saving, preserving, avoiding 30-40% of cases and
deaths can bring a major benefit to the population," the WHO's Pedro Alonso told BBC Focus on
Africa.
Health authorities are also keen to stress this is a new weapon in the fight against malaria to be
used alongside other preventative measures, such as treated bed nets and drugs that target the
malaria parasite.
How does the vaccine work?
Malaria is a parasite that invades and destroys our blood cells in order to reproduce, and it's
spread by the bite of blood-sucking mosquitoes.
The vaccine targets the most deadly and common parasite in Africa: Plasmodium falciparum.
It tries to deal with the form of the parasite which enters the victim's blood shortly after being
bitten, by partially blocking access into human cells and therefore preventing disease, Dr Alonso
said.
It needs four doses to be effective. The first three are given a month apart at the age of five, six
and seven months, and a final booster is needed at around 18 months.
Children are considered to be the most at risk from dying from malaria as, unlike adults, they have
not had a chance to build up immunity.
(Adapted from https://www.bbc.com/news/world-africa-58833382)

QUESTÃO 07 (EFOMM/INÉDITA) – In the phrase “…a 40% reduction is still a huge number of
lives saved” (paragraph 7), the underlined word is a synonym for
(A) Enormous

AULA 10 – PASSIVE VOICE, TAG QUESTIONS AND IDIOMS 87


TEACHER ANDREA BELO

(B) Limited
(C) Narrow
(D) Good
(E) Paltry
Comentários: A alternativa A está correta. A palavra “huge” significa “enorme” e é sinônimo da
palavra “enormous”, que significa “enormous”.
A alternativa B está incorreta. A palavra “huge” significa “enorme” e não é sinônimo da palavra
“limited”, que significa “limitado”.
A alternativa C está incorreta. A palavra “huge” significa “enorme” e não é sinônimo da palavra
“narrow”, que significa “estreito”.
A alternativa D está incorreta. A palavra “huge” significa “enorme” e não é sinônimo da palavra
“good”, que significa “bom”.
A alternativa E está incorreta. A palavra “huge” significa “enorme” e não é sinônimo da palavra
“paltry”, que significa “mesquinho”.
GABARITO: A

QUESTÃO 08 (EFOMM/INÉDITA) – The seventh paragraph of the text states that


(A) Malaria vaccine has achieved the highest rate of protection that a vaccine can provide
(B) The problem caused by Malaria is relatively small
(C) Despite the low rate of protection, the Malaria vaccine is extremely important and necessary
(D) 40% reduction in Malaria cases is not a significant number
(E) It is better that the malaria vaccine protection rate is low
Comentários: A alternativa A está incorreta. O sétimo parágrafo não afirma que a vacina da
malária atingiu a maior taxa de proteção que uma vacina pode fornecer, mas sim, que ela fornece
40% de proteção. Isso pode ser confirmado com o trecho “...preventing 40% of malaria cases... It
would obviously be better if it was higher…”.
A alternativa B está incorreta. O sétimo parágrafo não afirma que o problema causado pela
malária é relativamente pequeno, pois tem uma escala enorme e até uma pequena taxa de
proteção é significativa. Isso pode ser confirmado com o trecho “...you need to think about the
scale of the problem - with hundreds of millions of cases, a 40% reduction is still a huge number
of lives saved”.
A alternativa C está correta. O sétimo parágrafo afirma que apesar do baixo índice de proteção, a
vacina contra malária é extremamente importante e necessária. Isso pode ser confirmado com o
trecho “...you need to think about the scale of the problem – with hundreds of millions of cases,
a 40% reduction is still a huge number of lives saved”.
A alternativa D está incorreta. O sétimo parágrafo não afirma que redução de 40% nos casos de
malária não é um número significativo, pois é significativo. Isso pode ser confirmado com o trecho

AULA 10 – PASSIVE VOICE, TAG QUESTIONS AND IDIOMS 88


TEACHER ANDREA BELO

“...you need to think about the scale of the problem – with hundreds of millions of cases, a 40%
reduction is still a huge number of lives saved”.
A alternativa E está incorreta. O sétimo parágrafo não afirma que é melhor que a taxa de proteção
da vacina contra malária seja baixa, mas sim, que seria melhor se oferecesse maior proteção. Isso
pode ser confirmado com o trecho “It would obviously be better if it was higher…”.
GABARITO: C
QUESTÃO 09 (EFOMM/INÉDITA) – In the phrase “Health authorities are also keen to stress this
is a new weapon in the fight against malaria…” (paragraph 9), the underlined word is a synonym
for
(A) Apathetic
(B) Indifferent
(C) Reluctant
(D) Anxious
(E) Strong
Comentários: A alternativa A está incorreta. A palavra “keen” significa “ansioso” e não é sinônimo
da palavra “apathetic”, que significa “apático”.
A alternativa B está incorreta. A palavra “keen” significa “ansioso” e não é sinônimo da palavra
“indifferent”, que significa “indiferente”.
A alternativa C está incorreta. A palavra “keen” significa “ansioso” e não é sinônimo da palavra
“reluctant”, que significa “relutante”.
A alternativa D está correta. A palavra “keen” significa “ansioso” e é sinônimo da palavra
“anxious”, que significa “ansioso”.
A alternativa E está incorreta. A palavra “keen” significa “ansioso” e não é sinônimo da palavra
“strong”, que significa “forte”.
GABARITO: D

QUESTÃO 10 (EFOMM/INÉDITA) – The text


(A) Explains why the new malaria vaccine should not be distributed for use in the population
(B) Aims to lecture on the new vaccine against Malaria and highlight its importance
(C) Develops the idea that the Malaria vaccine should also be aimed at adults
(D) Focuses on advocating the non-use of Malaria vaccine in African children
(E) Criticizes the use of a vaccine that does not have a high protection rate
Comentários: A alternativa A está incorreta. O texto não explica por que a nova vacina contra
malária não deve ser distribuída para uso na população.
A alternativa B está correta. O texto visa dissertar sobre a nova vacina contra a malária e destacar
sua importância. Isso pode ser confirmado com o trecho “We have been looking at that and some

AULA 10 – PASSIVE VOICE, TAG QUESTIONS AND IDIOMS 89


TEACHER ANDREA BELO

other key questions … with hundreds of millions of cases, a 40% reduction is still a huge number
of lives saved”.
A alternativa C está incorreta. O texto não desenvolve a ideia de que a vacina contra malária
também deve ser destinada a adultos.
A alternativa D está incorreta. O texto não se concentra na defesa do não uso da vacina contra a
malária em crianças africanas, pois defende esse uso.
A alternativa E está incorreta. O texto não critica o uso de uma vacina que não tenha alto índice
de proteção, pois defende esse uso mesmo com a baixa taxa de proteção, devido a grande
dimensão do problema da Malária.
GABARITO: B

QUESTÕES EPCAR
Answer questions 01 to 10 according to the text.
The Brazilian army is turning into a de facto police force
Few places illustrate the modern role of the Brazilian army better than Tabatinga, a city of 62,000
on the shared border point between Brazil, Colombia, and Peru. The frontier has not budged since
the Portuguese built a now-ruined fort there in the 1700s. But Júlio Nagy, a local commander, has
his sights trained on unconventional threats. In February and March his troops intercepted 3.7
tonnes of cannabis. Last year they destroyed an airstrip built by illegal gold miners. Inside a small
army-run zoo, garish macaws rescued from animal traffickers squawk intermittently.
The country’s official defence review states that “at present, Brazil has no enemies”. Brazilian
strategists say that an insufficiency of military adversaries _______ _______ justify skimping on
defence. Criminal gangs operating in border areas can overwhelm civilian police, and in the future
Brazil hopes to deter foreigners covetous of its natural resources. New threats require new
responses. With 334,000 troops at its disposal, the government has had to find ways to deploy
them.
The army’s remit has expanded to police work. Most Brazilians seem unfazed by this trend. Unlike
politicians and police officers, servicemen are seen as honest, competent, and kind. Despite the
shadow of the dictatorship, confidence rankings of institutions often put the army at the top.
Soldiers are trying to adapt to their new role. At a training centre in Campinas, near São Paulo,
they are subjected to tear-gas and stun grenades, so they know what such weapons feel like
before unleashing them on civilians.
(Adapted from https://www.economist.com/the-americas/2017/07/06/the-brazilian-army-is-turning-into-a-de-facto-police-force)

QUESTÃO 01 (EPCAR/INÉDITA) – According to the text, it is incorrect to say that:


(A) The Brazilian army is performing police work.
(B) Soldiers are training to adapt to their new role.
(C) The Brazilian army deals with unusual threats.
(D) Politicians, police officers and servicemen are seen as honest.

AULA 10 – PASSIVE VOICE, TAG QUESTIONS AND IDIOMS 90


TEACHER ANDREA BELO

(E) Brazil does not have traditional military adversaries now.


Comentários: A questão pede a alternativa INCORRETA de acordo com o texto. Let’s go!
(A) The Brazilian army is performing police work > O exército brasileiro está realizando trabalho
policial- alternativa de acordo com o texto, como já nos informa o título do texto, logo “de cara”:
The Brazilian army is turning into a de facto police force = O exército brasileiro está se
transformando em uma força policial de fato. Como estamos buscando a alternativa incorreta,
ERRADA.
(B) Soldiers are training to adapt to their new role > Os soldados estão treinando para se adaptar
ao seu novo papel- alternativa de acordo com o texto: “Soldiers are trying to adapt to their new
role. At a training centre in Campinas, near São Paulo...” Como estamos buscando a alternativa
incorreta, ERRADA.
(C) The Brazilian army deals with unusual threats > O exército brasileiro lida com ameaças
incomuns- alternativa de acordo com o texto, como fica claro pelo seguinte trecho: “But Júlio
Nagy, a local commander, has his sights trained on unconventional threats. In February and March
his troops intercepted 3.7 tonnes of cannabis.” = Mas Júlio Nagy, comandante local, está de olho
em ameaças não convencionais. Em fevereiro e março, suas tropas interceptaram 3,7 toneladas
de cannabis. Como estamos buscando a alternativa incorreta, ERRADA.
(D) Politicians, police officers and servicemen are seen as honest > Políticos, policiais e militares
são vistos como honestos- essa alternativa não está de acordo com o texto, que afirma que,
diferentemente de (unlike) políticos e policiais, os militares são vistos como honestos,
competentes e gentis. “Unlike politicians and police officers, servicemen are seen as honest,
competent, and kind.” Como estamos buscando exatamente a alternativa que não está conforme
o texto, CORRETA.
(E) Brazil does not have traditional military adversaries now > O Brasil não tem adversários
militares tradicionais atualmente- alternativa de acordo com o texto: “The country’s oficial
defence review states that ‘at present, Brazil has no enemies.’” Como vimos acima, o exército
brasileiro lida com ameaças incomuns, como o tráfico de drogas. Como estamos buscando a
alternativa incorreta, ERRADA.
GABARITO: D

QUESTÃO 02 (EPCAR/INÉDITA) – the sentence “Few places illustrate the modern role of the
Brazilian army better than Tabatinga...” (paragraph 1), the underlined word indicates
(A) a large quantity.
(B) a huge quantity.
(C) an exact number.
(D) a definite amount.
(E) a small quantity.
Comentários: (A) a large quantity > Few é um quantificador que significa poucos, poucas,
indicando, portanto, pouca quantidade. Não grande (large), como indica a alternativa. ERRADA.

AULA 10 – PASSIVE VOICE, TAG QUESTIONS AND IDIOMS 91


TEACHER ANDREA BELO

(B) a huge quantity > Huge significa enorme, imenso- uma enorme quantidade. Como vimos
acima, Few é um quantificador que significa poucos, poucas, indicando, portanto, pouca
quantidade. ERRADA.
(C) an exact number > Few é um quantificador que significa poucos, poucas, indicando pouca
quantidade, sem especificar, contudo, a quantidade exata. Logo, não indica um número exato.
ERRADA.
(D) a definite amount > Aqui vale a mesma justificativa da alternativa anterior. Few é um
quantificador que significa poucos, poucas, indicando pouca quantidade, sem especificar,
contudo, a quantidade exata. Logo, não indica uma quantidade definida. ERRADA.
(E) a small quantity > Conforme estudamos, few é um quantificador que significa poucos, poucas,
indicando pouca quantidade- a small quantity. CORRETA.
GABARITO: E

QUESTÃO 03 (EPCAR/INÉDITA) – Brazilian strategists say that an insufficiency of military


adversaries _______ _______ justify skimping on defence. (paragraph 2)
To form a negative present statement, the correct words to respectively fill in the blanks are
(A) do not
(B) are not
(C) is not
(D) does not
(E) not does
Comentários: Nesse tipo de questão, é essencial identificar o sujeito, para saber qual auxiliar
vamos usar para formar a negativa no presente. No trecho, você deve se perguntar: o que não
justifica “skimping on defence” (reduzir a defesa)? A resposta é o sujeito que você procura. → an
insufficiency of military adversaries → uma insuficiência de adversários militares. É essa
insuficiência que não justifica “skimping on defence” (reduzir a defesa). Uma insuficiência é ela,
terceira pessoa do singularit.
Logo, vamos formar a negativa no presente usando does not.
(A) do not > Não usamos do not para terceira pessoa do singular- it. ERRADA.
(B) are not > Não usamos o verbo to be para formar a negativa no Simple Present. Usamos os
auxiliares do ou does e o not. ERRADA.
(C) is not > Não usamos o verbo to be para formar a negativa no Simple Present. Usamos os
auxiliares do ou does e o not. ERRADA.
(D) does not > Alternativa correta, conforme explicação acima. Brazilian strategists say that an
insufficiency of military adversaries DOES NOT justify skimping on defence. CORRETA.
(E) not does > A ordem dos termos está invertida. ERRADA.
GABARITO: D

AULA 10 – PASSIVE VOICE, TAG QUESTIONS AND IDIOMS 92


TEACHER ANDREA BELO

QUESTÃO 04 (EPCAR/INÉDITA) – In “... garish macaws rescued from animal traffickers squawk
intermittently.” (paragraph 1), what type of adverb is intermittently?
(A) an adverb of manner.
(B) an adverb of time.
(C) an adverb of place.
(D) an adverb of degree.
(E) an adverb of quantity.
Comentários: De um modo geral, advérbios terminados em -ly indicam o modo como algo é feito,
equivalendo à terminação -mente em Português. Observe que, com essa informação em mente,
seria possível acertar a questão, ainda que você não conhecesse o advérbio em si que a questão
exigiu. No trecho, o sentido é de que araras extravagantes resgatadas de traficantes de animais
gritam intermitentemente.
(A) an adverb of manner > Advérbio de modo, conforme explicado acima. CORRETA.
(B) an adverb of time > Intermitentemente indica o modo, não o momento em que a ação ocorria.
Exemplos de advérbios de tempo seriam now, tonight, early, late, soon, tomorrow etc.
INCORRETA.
(C) an adverb of place > Intermitentemente não indica o lugar da ação. Exemplos de advérbios de
lugar seriam here, there, outside, everywhere etc. INCORRETA.
(D) an adverb of degree > An adverb of degree indica a intensidade, ou grau de uma ação, de um
adjetivo ou de outro advérbio, como very, really, pretty... ERRADA.
(E) an adverb of quantity > Adverbs of quantity indicam quantidades, como too many, fewer,
more, enough, too much, less... ERRADA.
GABARITO: A

QUESTÃO 05 (EPCAR/INÉDITA) – Which sentence expresses something that is happening at the


moment of speaking?
(A) ... so they know what such weapons feel like before unleashing them on civilians.
(B) ... at present, Brazil has no enemies.
(C) Soldiers are trying to adapt to their new role.
(D) ... in the future Brazil hopes to deter foreigners covetous ...
(E) ... confidence rankings of institutions often put the army at the top.
Comentários: O tempo verbal que é usado para expressar uma ação que acontece no momento
da fala é o Present Continuous. A estrutura do Present Continuous tem como característica
marcante o uso verbo to be + verbo com -ing.
(A) ... so they know what such weapons feel like before unleashing them on civilians > Watch out:
esse trecho tem um verbo com -ing (unleashing), mas observe que ele não foi usado com o verbo
to be antes. O sentido dele é diferente, indica um infinitivo (soltar): para que eles saibam como
são essas armas antes de soltá-las nos civis. ERRADA.

AULA 10 – PASSIVE VOICE, TAG QUESTIONS AND IDIOMS 93


TEACHER ANDREA BELO

(B) ... at present, Brazil has no enemies > Temos aí o Presente Simples, o Brasil não tem inimigos
no momento. ERRADA.
(C) Soldiers are trying to adapt to their new role > Os soldados estão tentando se adaptar ao seu
novo papel- CORRETA, indica uma ação que está acontecendo no momento. Temos aí a estrutura
do Present Continuous = are trying. CORRETA.
(D) ... in the future Brazil hopes to deter foreigners covetous ... > No futuro, o Brasil espera... Não
indica uma ação que está acontecendo no momento. ERRADA.
(E) ... confidence rankings of institutions often put the army at the top > Watch out: aqui também
o -ing não está compondo o Present Continuous, assim como aconteceu na letra A. Rankings é
um substantivo- Os rankings (os índices) de confiança das instituições geralmente colocam o
exército no topo. ERRADA.
GABARITO: C

QUESTÃO 06 (EPCAR/INÉDITA) – Which noun below conveys a plural form?


(A) centre.
(B) dictatorship.
(C) airstrip.
(D) servicemen.
(E) commander.
Comentários: Qual substantivo abaixo transmite uma forma plural?
(A) centre > Centre, ou center em Inglês americano, significa centro, e está no singular. O plural
seria centres ou centers. ERRADA.
(B) dictatorship > Dictatorship significa ditadura e está no singular. O plural seria dictatorships.
ERRADA.
(C) airstrip > Airstrip significa pista de pouso e está no singular. O plural seria airstrips. ERRADA.
(D) servicemen > Servicemen significa militares, e está, sim, no plural. Não é um plural típico, pois
não termina em -s, mas é plural. O singular é serviceman, com a. CORRETA.
(E) commander > Commander significa comandante e está no singular. O plural seria
commanders. ERRADA.
GABARITO: D

QUESTÃO 07 (EPCAR/INÉDITA) – Criminal gangs operating in border áreas can overwhelm civilian
police, and in the future Brazil hopes to deter foreigners covetous of its natural resources.
With 334,000 troops at its disposal, the government has had to find ways to deploy them.
In paragraph 2, the underlined words refer, respectively, to
(A) civilian police / troops.

AULA 10 – PASSIVE VOICE, TAG QUESTIONS AND IDIOMS 94


TEACHER ANDREA BELO

(B) civilian police / the government.


(C) Brazil / troops.
(D) natural resources / disposal.
(E) Brazil / the government.
Comentários: Criminal gangs operating in border areas can overwhelm civilian police, and in the
future Brazil hopes to deter foreigners covetous of its natural resources. → Gangues criminosas
que operam em áreas de fronteira podem sobrecarregar a polícia civil e, no futuro, o Brasil espera
deter estrangeiros que cobiçam seus recursos naturais. → recursos naturais do Brasil → its é um
possesive adjective, estabele uma relação de posse, indicando a quem pertence algo e se refere,
no caso, a Brazil.
With 334,000 troops at its disposal, the government has had to find ways to deploy them. → Com
334.000 soldados à sua disposição, o governo teve que encontrar maneiras de implantá-los. → à
disposição do governo → its é um possesive adjective, estabele uma relação de posse, indicando
a quem pertence algo e se refere, no caso, a the government.
(A) civilian police / troops > Como vimos acima, os termos não se referem a essas expressões.
ERRADA.
(B) civilian police / the government > Como vimos acima, os termos não se referem a essas
expressões. ERRADA.
(C) Brazil / troops > Conforme estudamos acima, os termos não se referem a essas expressões.
Muita atenção, pois its jamais vai se referir a um termo no plural. Para indicar que algo pertence
às tropas, teríamos que usar o possessive adjective de plural, their, delas, das tropas. ERRADA.
(D) natural resources / disposal > Conforme estudamos acima, os termos não se referem a essas
expressões. Muita atenção, pois its jamais vai se referir a um termo no plural. Para indicar que
algo pertence aos recursos naturais, teríamos que usar o possessive adjective de plural, their,
deles, dos recursos. ERRADA.
(E) Brazil / the government > Alternativa correta, conforme explicação acima. CORRETA.
GABARITO: E

QUESTÃO 08 (EPCAR/INÉDITA) – In “But Júlio Nagy, a local commander, has his sights trained
on unconventional threats. (paragraph 1), the opposite of unconventional is
(A) unusual.
(B) ordinary.
(C) atypical.
(D) funny.
(E) strange.
Comentários: Unconventional significa incomum, não convencional. O prefixo -un significa
negação, não = not conventional. A questão pede que você identifique o oposto! Let’s go!

AULA 10 – PASSIVE VOICE, TAG QUESTIONS AND IDIOMS 95


TEACHER ANDREA BELO

(A) unusual > Muita atenção: unsual não é oposto de unconventional, mas, sim, sinônimo.
Significa não usual, incomum. ERRADA.
(B) ordinary > Ordinary significa comum. Tem sentido oposto ao de unconventional. CORRETA.
(C) atypical > Atypical significa atípico, ou seja, incomum. É sinônimo de unconventional, não
oposto. ERRADA.
(D) funny > Funny significa engraçado. Não tem sentido oposto ao de unconventional. ERRADA.
(E) strange > Strange significa estranho. Não tem sentido oposto ao de unconventional. ERRADA.
GABARITO: B
QUESTÃO 09 (EPCAR/INÉDITA) – What is the interrogative form of the sentence “New threats
require new responses.”?
(A) Are new threats require new responses?
(B) Does new threats require new responses?
(C) Do new threats require new responses?
(D) Do new threats requires new responses?
(E) Is new threats require new responses?
Comentários: A frase está no Simple Present. → New threats require new responses. = Novas
ameaças exigem novas respostas. O que exige novas respostas? Novas ameaças, certo? Assim,
new threats é o sujeito. Equivale a they, elas, as novas ameaças.
Vamos, então, usar, na interrogativa, o auxiliar do e o verbo permanece como está, na sua forma
base (require).
• Do new threats require new responses?
(A) Are new threats require new responses? > Não usamos o verbo to be para formar a
interrogativa no Simple Present. Usamos os auxiliares do ou does. ERRADA.
(B) Does new threats require new responses? > Não usamos does para terceira pessoa do plural-
they. ERRADA.
(C) Do new threats require new responses? > Alternativa correta, conforme explicado acima.
CORRETA.
(D) Do new threats requires new responses? > O erro aqui está no -s no verbo. Como vimos, o
verbo fica em sua forma base. ERRADA.
(E) Is new threats require new responses? > Não usamos o verbo to be para formar a interrogativa
no Simple Present. Usamos os auxiliares do ou does. ERRADA.
GABARITO: C

QUESTÃO 10 (EPCAR/INÉDITA) – “_______ illegal gold miners”


All the following terms can be used to correctly fill in the blank, except
(A) an.

AULA 10 – PASSIVE VOICE, TAG QUESTIONS AND IDIOMS 96


TEACHER ANDREA BELO

(B) the.
(C) some.
(D) many.
(E) a lot of.
Comentários: (A) an > An, de fato, não pode preencher a lacuna, porque significa um ou uma, e
sempre será usado com substantivos no singular. No trecho, temos uma expressão no plural-
illegal gold miners- garimpeiros ilegais. CORRETA.
(B) the > The é artigo definido, e pode ser usado tanto com substantivos no singular quanto no
plural, significando o, a, os, as. Poderia, assim, preencher a lacuna. ERRADA.
(C) some > Some significa algum, alguma, alguns, algumas, e pode ser usado tanto com
substantivos contáveis no plural quanto com substantivos incontáveis. ERRADA.
(D) many > Many significa muitos, muitas, e é usado justamente com substantivos contáveis no
plural, o que é o caso do substantivo miners na expressão illegal gold miners. ERRADA.
(E) a lot of > A lot of também significa muitos, muitas, e é usado tanto com substantivos contáveis
no plural, o que é o caso do substantivo miners na expressão illegal gold miners, quanto com
substantivos incontáveis. ERRADA.
GABARITO: A

QUESTÕES ESA
Dogs are just like ‘play, eat, sleep’ – they bring me back into the moment. I think we can all learn
something from that,” says Carole Henderson, who has been taking her “furry backup” to the
office for the last few months.
They are not so good at making the tea and things get a bit rowdy when the delivery man comes
round, but Henderson’s labradors Barney and Rusty, and labradoodle Lily, have been her sidekicks
for the last decade. As well as being excellent foot-warmers, they have helped her emotionally
with getting through solo months in the office.
After the latest lockdown left her with no choice but to part-time furlough the rest of her team,
including her husband, the empty office felt like a cavern. “I felt a bit sorry for myself and thought:
‘I’m taking the dogs with me, because at least then I can have a doggy cuddle when things aren’t
going to plan,’” she says.
As a teacher of grief recovery method, Henderson’s work has felt more important than ever,
particularly with reaching out to those grieving in isolation, and without her colleagues there in-
person, her dogs have provided immense emotional support. “Things do get to me. I’m human,
doing this work. Someone will tell me a really sad story and there’s nothing better than getting a
cuddle from one of my dogs,” she says. “Stroking a lovely bit of fur brings you right back down,
then I’m ready to help the next person.”
(https://www.theguardian.com/lifeandstyle/2021/apr/06/the-dogs-keeping-office-workers-company-through-lockdown)

QUESTÃO 01 (ESA/INÉDITA) – According to the text, it is correct to say that:

AULA 10 – PASSIVE VOICE, TAG QUESTIONS AND IDIOMS 97


TEACHER ANDREA BELO

(A) We are going to learn with pets.


(B) We learn every day with all kind of pets.
(C) We can learn with our pets.
(D) We are supposed to learn as much as possible.
(E) We are supposed to learn im the future.
Comentários: O enunciado pede o que podemos afirmar de acordo com o texto e percebemos na
primeira linha que começa com a palavra dogs e mais a frente diz "I think we can all learn
something from that", ou seja, todos temos que aprender algo com esses cães, portanto o nosso
gabarito é a letra C que diz "We can learn with our pets" (Podemos aprender com os nossos
animais de estimação).
As demais alternativas contêm erros ou informações não comprovadas no texto como na letra A
"We are going to learn with pets" dizendo que aprenderemos no futuro e o texto afirma que
podemos usando o verbo modal "Can". A letra B informa que aprenderemos todos os dias e não
temos essa confirmação, nem com todos os tipos de Pets e na letra D diz que vamos aprender
quanto mais for possível e não há essa informação no texto também.
GABARITO: C

QUESTÃO 02 (ESA/INÉDITA) – The sentence says "who has been taking her “furry backup” to
the office for the last few months." means that
(A) Pets are at the office.
(B) Pets sometimes go to the office.
(C) Pets go to the office with their owners every day.
(D) Pets seldom go to the office with theirs owners.
(E) Pets are everywhere.
Comentários: O enunciado traz a frase "quem está cuidado de “furry backup”", ou seja, pelo
fazendo backup, querendo dizer que os cachorros, os animais de estimação estão envolvidos no
escritório, então podemos confirmar que o gabarito é a letra A "Pets are at the office", ou seja,
animais de estimação no escritório.
As letras B, C, D e E carregam informações que não podemos comprovar ou estão incorretas. Na
letra B está escrito que às vezes eles vão para o escritório mas nao estamos falando de "às vezes"
(sometimes) mas sim de uma situação de lockdown citada no texto, Na letra C diz que vão todos
os dias para o escritório mas não temos a confirmação pela expressão "every day" e sim nesse
período de pandemia. Na letra D informa que o os animais de estimação raramente vão com a
palavra "seldom" mas isso também não é verdade nem dizer que animais de estimação estão em
todos os lugares (evrywhere)... e portanto o gabarito é a letra A.
GABARITO: A

QUESTÃO 03 (ESA/INÉDITA) – The word foot-warmers is

AULA 10 – PASSIVE VOICE, TAG QUESTIONS AND IDIOMS 98


TEACHER ANDREA BELO

(A) Uncaring.
(B) Stormy.
(C) Aloof.
(D) Balmy.
(E) Comfortable.
Comentários: O enunciado pede qual palavra podemos utilizar com o mesmo sentido da palavra
sublinhada "foot-warmers", que significa "aquecedor de pé", portanto o nosso gabarito é a letra
D, "Balmy" que é a palavra que mais se aproxima de algo carinhoso e aconchegante.
As palavras das opções A, B, C e E: "Uncaring", "Stormy” "Aloof" e “comfortable” significam o
contrário de aquecer e sim algo molhado ou frio e confortável, comprovando o nosso gabarito
letra D.
GABARITO: D

QUESTÃO 04 (ESA/INÉDITA) – In this part "I felt a bit sorry for myself and thought: I’m taking
the dogs with me, because at least then I can have ...". We can say that
(A) Having a dog at home assists the work at home office.
(B) Pets have helped a lot during lockdown period.
(C) Having a dog at home develops the work of their owners.
(D) Dogs are always used to aid in grieving recovery.
(E) Dogs are always used in tests.
Comentários: O enunciado traz uma passagem que quer dizer que as pessoas estão levando os
cachorros com ela "I’m taking the dogs with me" e isso significa que a pessoa do texto está levando
os animais para o escritório para não se sentirem sós nesse perído de pandemia enquanto
trabalham. Como o texto não fala de "home office" (trabalho em casa), inclusive com a foto da
pessoa no escritório a letra A está errada pois diz que o trabalho é em casa.
A letra B é o nosso gabarito pois cães têm ajudado durante o lockdown ("Pets have helped a lot
during lockdown period.").
As letras C, D e E têm informações incorretas como "... develops the work" (como se os cães
desenvolvessem o trabalho), na letra C diz que eles são usados para ajudar na recuperação de
situação de luto ("grieving recovery") e na E que são usados em testes e isso não é verdade de
acordo com o texto.
GABARITO: B

QUESTÃO 05 (ESA/INÉDITA) – The word cuddle in "because at least then I can have a doggy
cuddle when things aren’t going to plan" can be replaced for:
(A) Push.
(B) Release.

AULA 10 – PASSIVE VOICE, TAG QUESTIONS AND IDIOMS 99


TEACHER ANDREA BELO

(C) Impact.
(D) Hug.
(E) Stop.
Comentários: O enunciado pede para substituir a palavra "cuddle" que aparece na frase em
destaque e o nosso gabarito é a letra D "Hug". O motivo é simples, "hug" é sínonimo de "cuddle",
ambos significam abraço. As demais palavras: "Push", "Release", "Impact" significam algo que nao
tem a ver com aconchego ou abraço, seria empurrar, afastar, impactar e parar, portanto o
gabarito é a letra D.
GABARITO: D

QUESTÕES ESCOLA NAVAL


Limits on coronavirus testing in Brazil are hiding the true dimensions of Latin America’s largest
outbreak
Atop a shaded hill at the edge of São Paulo, the gravedigger thinks he knows the truth. No matter
how bad it appears in Brazil — the country hit hardest by the coronavirus in the Southern
Hemisphere — the reality is significantly worse.
Manoel Norberto Pereira watched another body being wheeled in, accompanied by what has by
now become a familiar set of details. Sex: female. Age: 77 years. Cause of death: insufficient
respiration.
Every day brings more. The cemetery now receives around 50 bodies every day — double the
average in normal times. Many are marked as confirmed cases of covid-19, the disease caused by
the virus. But many more cite only an unidentified respiratory ailment. To Pereira, they’re the
unseen toll of the coronavirus in Brazil, which has officially infected 51,000 people and killed 3,400
— but unofficially many times more than that.
Imprecise and insufficient testing is a global problem, but in Brazil, it's on an entirely different
scale. Latin America's largest country is testing people at a rate far lower than any other nation
with at least 40,000 cases. It tests 12 times fewer people than Iran, and 32 times fewer than the
United States. Hospitalized patients aren't being tested. Some medical professionals aren't being
tested. People are dying in their homes without being tested.
Researchers at the Federal University of Minas Gerais have suggested Brazil has eight times more
coronavirus cases than the official numbers indicate. A research team at the University of São
Paulo thinks it has 16 times more — more than 800,000 cases.
According to government statistics, nearly 37,300 people have been hospitalized this year with
respiratory ailments — four times the number at this point last year — but only half have received
test results. In São Paulo, nearly 1,300 people have died of unidentified respiratory problems.
Amazonas state has reported 193 deaths from covid-19. But its capital city is burying so many
people — three times the average — that it’s digging trenches for mass graves.
“The big conclusion is that we don’t know what the real scenario is,” said Leonardo Costa Ribeiro,
an economist at the Federal University of Minas Gerais. “It gives the sensation that it is more
controlled, when the reality is very different.”

AULA 10 – PASSIVE VOICE, TAG QUESTIONS AND IDIOMS 100


TEACHER ANDREA BELO

Domingos Alves, a data scientist at the University of São Paulo, did little to hide his anger at the
chasm between how government officials talk about the virus and what the data shows.
“As a researcher, I look at the data and make analyses for the government,” he said. “But as a
citizen, I’m frustrated. The government is trying to control the epidemic without the elements of
how to control it, because of their lack of knowledge about how serious it is.
“It will get much worse. There will be lines at the hospitals. There will be lines at the cemeteries.
The next few weeks will be very dark.”
The new health minister, Nelson Teich, named by Bolsonaro to replace Mandetta, has made
increased testing a pillar of the country’s response to the pandemic. He said this week that the
government intends to purchase 46 million tests, enough for just over a fifth of the population.
He hasn’t specified when the tests will arrive or when they’ll be used.
“We are not talking about testing the entire country,” Teich said. “We are going to use the tests so
that people tested will reflect the Brazilian population.”
But the obstacles will be enormous — and could portend the struggles that await much of the
developing world as the coronavirus moves deeper into Latin America and Africa. A variable as
basic as reliable data could become a distinguishing factor between poor and wealthy countries.
Analysts say Brazil has neither the manufacturing nor the purchasing capacity to meet the demand
for tests. Its laboratories, already overwhelmed by a testing backlog, aren’t equipped to process
them at scale. And in the international scrum for supplies and testing equipment, the country is
losing out to wealthier nations that can pay more or leverage closer ties with China.
“Brazil could be thought of as a barometer of what could be expected in other countries,” said
Marcelo Gomes, a researcher at the Oswaldo Cruz Foundation, a scientific institution.
“Information is paramount. The more we have, the more we can make the population aware of
what it is exactly that we are facing.”
A gravedigger at São Paulo’s Vila Formosa Cemetery, one of Latin America’s largest, said he started
noticing strange deaths in early March — weeks before Brazil’s first confirmed coronavirus death.
The causes of death — pneumonia, respiratory failure — were what he would expect to see during
the Southern Hemisphere’s winter, not peak summer.
“One day, I saw on the news that in all of São Paulo there had been 20 burials of coronavirus
victims,” said the man, who spoke on the condition of anonymity because he didn’t have
permission to give an interview. “But I knew at my cemetery alone, there had been 27.”
Pereira feels the same frustration. Some days, he can’t believe the number of people out in the
street. If they were seeing what he saw, no way would they be outside. No way would there be a
national debate over reopening the economy. No one would be listening to claims that the
pandemic had been overblown.
Bodies don’t lie, he said. And here came another one.
Adapted from (https://www.washingtonpost.com/world/the_americas/coronavirus-brazil-testing-bolsonaro-cemetery-gravedigger/2020/04/22/fe757ee4-83cc-11ea-878a-86477a724bdb_story.html)

QUESTÃO 01 (ESCOLA NAVAL/INÉDITA) – What’s the meaning of the word “overblown” in the
second last paragraph?

AULA 10 – PASSIVE VOICE, TAG QUESTIONS AND IDIOMS 101


TEACHER ANDREA BELO

A) Blown up
B) Risen
C) Made up
D) Exaggerated
E) Invented
Comentários: A alternativa A está incorreta. “Blown up” significa explodir. Já “overblown” significa
exagerado(a). Portanto, não se pode dizer que “blown up” é o significado de “overblown”.
A alternativa B está incorreta. Usar “risen” nesse contexto seria como dizer que a pandemia subiu,
o que não faz sentido, pois “overblown” significa exagerado(a).
A alternativa C está incorreta. “Made up” é uma expressão que significa inventado, e “overblown”
não significa inventado, mas sim exagerado(a).
A alternativa D está correta. “Exaggerated” significa exatamente o mesmo que “overblown”, ou
seja, exagerado(a).
A alternativa E está incorreta. “Invented” significa inventado, e “overblown” significa
exagerado(a). São dois conceitos bem diferentes.
GABARITO: D

QUESTÃO 02 (ESCOLA NAVAL/INÉDITA) – According to the text, which option is correct?


A) The figures of coronavirus in Brazil are as close to reality as they can possibly get.
B) The cemetery used to receive around 40 bodies every day in normal times.
C) People dying due to an unidentified respiratory ailment are the unseen toll of COVID-19 in
Brazil.
D) Brazil has around 60,000 people unofficially infected by the coronavirus.
E) Researchers at Federal University of Minas Gerais suggested Brazil has more than 800,000
cases.
Comentários: A alternativa A está incorreta. Segundo o texto, os números reais de infectados e
mortos pelo coronavírus no Brasil estão muito longe da realidade, e não o mais próximo possível
da realidade, como afirma a alternativa.
A alternativa B está incorreta. O texto afirma que o cemitério tem recebido por volta de 50 corpos
por dia, o que equivale ao dobro da média diária em tempos normais. Portanto, o número médio
de corpos recebidos diariamente em tempos normais é de aproximadamente 25, e não 40 como
afirma a alternativa.
A alternativa C está correta. O texto diz que um coveiro afirma que há várias pessoas morrendo
por problemas respiratórios e, segundo esse coveiro, essas pessoas representam o verdadeiro
quantitativo de mortes causadas pelo coronavírus.
A alternativa D está incorreta. O texto apenas afirma que o número não oficial de pessoas
infectadas pode ser muitas vezes maior que o real, mas não estabelece o número de 60.000 casos
como o possível número não oficial.

AULA 10 – PASSIVE VOICE, TAG QUESTIONS AND IDIOMS 102


TEACHER ANDREA BELO

A alternativa E está incorreta. No texto, quem sugere que o Brasil tem mais de 800.000 casos da
doença é a Universidade de São Paulo, não a Universidade Federal de Minas Gerais.
GABARITO: C

QUESTÃO 03 (ESCOLA NAVAL/INÉDITA) – According to the text, which option completes the
sentence below correctly?
The text says that gravediggers in Brazil _____________.
A) See a much worse reality than it is disclosed officially.
B) Are working as they usually do, with no changes in cemetery routine.
C) Are responsible for informing the government what are the real numbers related to the
pandemic.
D) Like the fact that there are more people dying, because this way they can secure their jobs.
E) Suggest that people watch the news, since it is a reliable source of accurate information.
Comentários: A alternativa A está correta. O texto afirma que não importa o quão ruim a situação
pareça, pois na realidade ela é muito pior.
A alternativa B está incorreta. O texto não fala que os coveiros estão trabalhando normalmente.
Na verdade, eles relatam grandes mudanças na rotina do cemitério por causa da pandemia.
A alternativa C está incorreta. O texto não fala nada sobre os coveiros serem responsáveis por
informar ao governo quais são os números reais relacionados à pandemia.
A alternativa D está incorreta. O texto não diz que os coveiros estão felizes com o fato de haver
muitas pessoas morrendo, nem fala nada sobre eles estarem preocupados em manter seus
empregos.
A alternativa E está incorreta. Os coveiros não recomendam que as pessoas assistam noticiários,
nem afirmam que os noticiários sejam uma fonte confiável de informações precisas.
GABARITO: A

QUESTÃO 04 (ESCOLA NAVAL/INÉDITA) – “It tests 12 times fewer people than Iran, and 32 times
fewer than the United States.” in paragraph 4, the word in bold refers to
A) Imprecise and insuficient testing
B) Global problem
C) Brazil
D) People
E) Nation
Comentários: O parágrafo inicia falando que testagem imprecisa e insuficiente é um problema
global, mas que no Brasil esse problema ocorre em uma escala inteiramente diferente. O texto
continua, dizendo que o maior país da América Latina está testando pessoas em uma proporção
muito menor que qualquer outro país que tenha pelo menos 40.000 casos. Nesse momento, o

AULA 10 – PASSIVE VOICE, TAG QUESTIONS AND IDIOMS 103


TEACHER ANDREA BELO

pronome “It” retoma a palavra “Brazil”, dizendo que o Brasil testa 12 vezes menos pessoas que o
Iran e 32 vezes menos pessoas que os Estados Unidos. Portanto, é correto dizer que “it” se refere
a “Brazil”. Isso nos leva ao gabarito de letra C.
GABARITO: C
QUESTÃO 05 (ESCOLA NAVAL/INÉDITA) – Which option completes the paragraph below
correctly?
Brazil ______(become) one of the global epicenters of the pandemic, recently surpassing China in
the number of reported cases. President Jair Bolsonaro ______(be) notoriously dismissive of the
virus, ______(say) earlier this week the number of deaths ______(be) declining, when in fact it
was increasing.
A) becomes / has been / saying / were
B) had become / was / said / were
C) has become / has been / said / were
D) becomes / was / saying / was
E) has become / has been / saying / was
Comentários: A primeira lacuna deve ser preenchida por “has become”. O texto continua após o
trecho da lacuna dizendo que o Brasil ultrapassou recentemente a China em número de casos
relatados. Isso nos indica que a lacuna deve ser preenchida pelo verbo no present perfect, já que
é um fato passado que aconteceu recentemente, mas sem uma data específica.
A segunda lacuna deve ser preenchida por “has been”. O texto coloca uma situação que acontece
no passado, mas vem se desenrolando até o momento. Por isso o uso do present perfect é
indicado.
A terceira lacuna deve ser preenchida por “saying”. Na continuação do trecho com o uso do
present perfect, não seria adequado usar o past simple, pois a frase perderia coerência e teria seu
sentido prejudicado. Por isso, o uso do gerúndio é indicado para dar continuidade ao raciocínio
no texto.
A quarta lacuna deve ser preenchida por “was”. Devemos usar “was” ao invés de “were”, pois o
verbo deve concordar com “number”, e não com “deaths”.
Temos a sequência: has become / has been / saying / was
GABARITO: E

QUESTÃO 06 (ESCOLA NAVAL/INÉDITA) – Mark the CORRECT alternative.


“Its laboratories, already overwhelmed by a testing backlog, aren’t equipped to process them at
scale.” (paragraph 14)
If laboratories are overwhelmed, it means that
A) They are comfortable with the demand.
B) The demand is over their capacity.

AULA 10 – PASSIVE VOICE, TAG QUESTIONS AND IDIOMS 104


TEACHER ANDREA BELO

C) The demand matches their capacity.


D) They are experienced in this kind of situation.
E) The demand is not a concern for them.
Comentários: A alternativa A está incorreta. A alternativa diz que os laboratórios estão
confortáveis com a demanda de testagens. Mas a palavra “overwhelmed” quer dizer
sobrecarregado , não confortável.
A alternativa B está correta. Se os laboratórios estão sobrecarregados (overwhelmed), então é
correto afirmar que a demanda é maior que a capacidade desses laboratórios de fazer os testes.
A alternativa C está incorreta. Essa alternativa diz que a demanda equivale à capacidade dos
laboratórios de fazer os testes, mas a palavra “overwhelmed” quer dizer sobrecarregados.
Portanto, a alternativa não explica o termo corretamente.
A alternativa D está incorreta. A palavra “overwhelmed” significa sobrecarregado, e nada tem a
ver com o nível de experiência que os laboratórios têm com esse tipo de situação.
A alternativa E está incorreta. A palavra “overwhelmed” significa sobrecarregado, e não tem
relação com o nível de preocupação dos laboratórios com a demanda.
GABARITO: B

QUESTÃO 07 (ESCOLA NAVAL/INÉDITA) – Mark the alternative which has the sentence below
correctly reported.
Analysts say Brazil has neither the manufacturing nor the purchasing capacity to meet the
demand for tests.
The author
A) replied: “Analysts say Brazil has neither the manufacturing nor the purchasing capacity to
meet the demand for tests.”
B) said that Analysts said Brazil had neither the manufacturing nor the purchasing capacity to
meet the demand for tests.
C) asked the readers if analysts say Brazil has neither the manufacturing nor the purchasing
capacity to meet the demand for tests.
D) asked analysts if they say Brazil has neither the manufacturing nor the purchasing capacity to
meet the demand for tests.
E) said analysts say Brazil has neither the manufacturing nor the purchasing capacity to meet the
demand for tests.
Comentários: A alternativa A está incorreta. No “reported speech”, não se usa dois pontos para
uma citação, mas sim a pessoa que está reportando dá a informação de maneira fluida. Exemplo:
“He said that he saw me in the restaurant yesterday” (ele disse que me viu no restaurante ontem).
A alternativa B está correta. A alternativa usa o reported speech de forma correta, pois “volta” um
tempo verbal em direção ao passado na hora de reportar aquilo que foi dito. A frase original estava
no present simple, logo a frase foi reportada usando o Past Simple (said/had).

AULA 10 – PASSIVE VOICE, TAG QUESTIONS AND IDIOMS 105


TEACHER ANDREA BELO

A alternativa C está incorreta. O erro da alternativa consiste no fato de que o autor não perguntou
nada aos leitores, mas sim disse. Portanto, o reported speech deveria começar com “said” ao invés
de “asked”.
A alternativa D está incorreta. Mais uma vez o erro está em dizer que o autor perguntou algo,
dessa vez aos analistas. O autor não perguntou nada, apenas disse. Portanto, o reported speech
deveria começar com “said” ao invés de “asked”.
A alternativa E está incorreta. O erro da alternativa foi não alterar o tempo verbal na hora de
reportar a informação. No trecho original, verbo usado foi “say/has” (simple present), logo, no
reported speech deve-se usar “said/had” (past simple).
GABARITO: B

QUESTÃO 08 (ESCOLA NAVAL/INÉDITA) – In the fourth paragraph, the text DOESN’T


A) explain that testing has been a problem for most countries in the world.
B) say that Brazil’s testing rate is far from ideal.
C) say testing rates in Iran and U.S. are examples of countries that test many more people than
Brazil.
D) say that Brazil tests a lot of people compared to countries with less than 40,000 cases.
E) say that Brazil is not testing health care professionals and that people are dying without being
tested.
Comentários: A alternativa A está incorreta. O parágrafo explica que a testagem insuficiente e
imprecisa tem sido um problema global. Portanto, é correto dizer que tem sido um problemas
para a maioria dos países do mundo.
A alternativa B está incorreta. De fato, é dito no parágrafo que a taxa de testagem no Brasil está
muito longe do ideal.
A alternativa C está incorreta. O texto estabelece Iran e Estados Unidos como países que testam
muito mais pessoas que o Brasil.
A alternativa D está correta. O parágrafo diz o Brasil é o país que menos testa dentre aqueles que
têm mais de 40.000 casos confirmados. Mas isso não significa que o Brasil teste muito quando
comparado a países que possuem menos de 40.000 casos confirmados.
A alternativa E está incorreta. O parágrafo afirma que há profissionais da área da saúde que não
estão sendo testados e que pessoas estão morrendo em suas casas sem terem sido testadas.
GABARITO: D

QUESTÃO 09 (ESCOLA NAVAL/INÉDITA) – In the sentence “Information is paramount.”


(paragraph 15), the underlined word means
A) production.
B) power.

AULA 10 – PASSIVE VOICE, TAG QUESTIONS AND IDIOMS 106


TEACHER ANDREA BELO

C) cure.
D) immunity.
E) primordial.
Comentários: A alternativa A está incorreta. “Production” significa produção, enquanto
“paramount” significa que algo é essencial.
A alternativa B está incorreta. “Power” significa poder, enquanto “paramount” significa que algo
é essencial.
A alternativa C está incorreta. “Cure” significa cura, enquanto “paramount” significa que algo é
essencial.
A alternativa D está incorreta. “Immunity” significa imunidade, enquanto “paramount” significa
que algo é essencial.
A alternativa E está correta. “Primordial” significa que algo é primordial, essencial e, esse é
exatamente o sentido da palavra “paramount”.
GABARITO: E

QUESTÃO 10 (ESCOLA NAVAL/INÉDITA) – “because of their lack of knowledge about how serious
it is.” (paragraph 9). The highlighted expression is closest in meaning to
A) otherwise.
B) thereafter.
C) due to.
D) moreover.
E) wherever.
Comentários: A alternativa A está incorreta. “otherwise” significa caso contrário, enquanto
“because of” significa por causa de. Não há relação de sentido entre as duas expressões.
A alternativa B está incorreta. “Thereafter” significa depois disso, enquanto “because of” significa
por conta de. Não há relação de sentido entre as duas expressões.
A alternativa C está correta. “because of” tem exatamente o mesmo significado de “due to”, por
conta de/por causa de.
A alternativa D está incorreta. “moreover” é um sinônimo de “besides”, dando uma ideia de
adição, e não de causa como é o caso de “because of”.
A alternativa E está incorreta. “wherever” quer dizer em qualquer lugar e não possui nenhuma
relação de sentido com “because of”, que significa por conta de/por causa de.
GABARITO: C

QUESTÕES EsPCEx
Leia o texto a seguir e responda às questões 01, 02 e 03.

AULA 10 – PASSIVE VOICE, TAG QUESTIONS AND IDIOMS 107


TEACHER ANDREA BELO

Afghanistan: US takes control of Kabul airport to evacuate staff


There have been scenes of panic at Kabul airport as desperate residents try to flee following the
seizure of the Afghan capital by the Taliban.
The US army say soldiers shot two armed men, while three people are reported to have died after
falling ___ (1) the underside of a plane they were clinging to shortly after take-off.
The airport was closed earlier for soldiers to try and clear the runways.
US military planes are now landing, including one carrying US marines.
Marines are being flown in to help the evacuation effort. The US and other countries are rushing
to remove staff and allies from the country.
A German evacuation plane also landed at Kabul airport on Monday, sources told Reuters news
agency.
___ (2) Sunday the Taliban declared victory after Afghan President Ashraf Ghani fled abroad and
his government collapsed.
The militants' return to rule brings an end to almost 20 years of a US-led coalition's presence in
the country.
Kabul was the last major city in Afghanistan to fall to a Taliban offensive that began months ago
but accelerated in recent days as they gained control of territories, shocking many observers.
The Islamist group was able to seize control after most foreign troops pulled out.
Following the Taliban's seizure ___ (3) Kabul, many people headed to the airport.
Evacuations of foreigners and some Afghans with links to foreign governments and organisations
have been taking place, but passengers said rumours spread that even those without visas were
being allowed to travel.
(Adapted from https://www.bbc.com/news/world-asia-58227029)

QUESTÃO 01 (EsPCEx/INÉDITA) – Choose the alternative containing the correct words to


respectively complete gaps (1), (2) and (3)
A) From, on, of
B) At, on, of
C) From, on, at
D) From, in, at
E) At, on, of
Comentários: A primeira lacuna deve ser preenchida com “from”, pois a frase nos indica de onde
(origem) as pessoas caíram, ou seja, “...enquanto três pessoas morreram depois de cair da parte
inferior de um avião...”.
A segunda lacuna deve ser preenchida com “on”, pois a frase nos indica um dia da semana, ou
seja, “No domingo, o Talibã declarou vitória...”.

AULA 10 – PASSIVE VOICE, TAG QUESTIONS AND IDIOMS 108


TEACHER ANDREA BELO

A terceira lacuna deve ser preenchida com “of”, pois a frase nos indica a tomada de algum lugar,
ou seja, “Após a tomada de Kabul pelo Talibã...”.
GABARITO: A

QUESTÃO 02 (EsPCEx/INÉDITA) – According to the text, is correct to state that


A) The Taliban's victory was peacefully accepted
B) Only the US is withdrawing its troops from Kabul
C) With the return of the Taliban, the US seized power from Kabul
D) Lack of foreign troops facilitated Taliban victory
E) No one is allowed to leave Kabul
Comentários: A alternativa A está incorreta. De acordo com o texto, não é correto afirmar que a
vitória do Talibã foi aceita de forma pacífica, mas sim, que houve cenas de pânico após a vitória.
Isso pode ser confirmado com o trecho “There have been scenes of panic at Kabul airport as
desperate residents try to flee following the seizure of the Afghan capital by the Taliban”.
A alternativa B está incorreta. De acordo com o texto, não é correto afirmar que apenas os EUA
estão retirando suas tropas de Kabul, mas sim, que outros países também estão fazendo isso. Isso
pode ser confirmado com o trecho “The US and other countries are rushing to remove staff and
allies from the country”.
A alternativa C está incorreta. De acordo com o texto, não é correto afirmar que com o retorno do
Talibã, os EUA tomaram o poder de Kabul, mas sim, que agora o Talibã está no poder. Isso pode
ser confirmado com o trecho “The Islamist group was able to seize control…”.
A alternativa D está correta. De acordo com o texto, é correto afirmar que a falta de tropas
estrangeiras facilitou a vitória do Talibã, assim como esta opção indica. Isso pode ser confirmado
com o trecho “The Islamist group was able to seize control after most foreign troops pulled out”.
A alternativa E está incorreta. De acordo com o texto, não é correto afirmar que ninguém está
autorizado a sair de Kabul, mas sim, que está havendo evacuação de pessoas do país. Isso pode
ser confirmado com o trecho “Evacuations of foreigners and some Afghans with links to foreign
governments and organisations have been taking place, but passengers said rumours spread that
even those without visas were being allowed to travel”.
GABARITO: D

QUESTÃO 03 (EsPCEx/INÉDITA) – In the sentence “The Islamist group was able to seize control
after most foreign troops pulled out” (paragraph 10), the word “seize” means to
A) Steal
B) Take
C) Give
D) Release

AULA 10 – PASSIVE VOICE, TAG QUESTIONS AND IDIOMS 109


TEACHER ANDREA BELO

E) Remove
Comentários: A alternativa A está incorreta. A palavra “seize” significa “pegar/tomar” e não pode
ser comparada com a palavra “steal”, que significa “roubar”.
A alternativa B está correta. A palavra “seize” significa “pegar/tomar” e pode ser comparada com
a palavra “take”, que significa “pegar”.
A alternativa C está incorreta. A palavra “seize” significa “pegar/tomar” e não pode ser comparada
com a palavra “give”, que significa “dar”.
A alternativa D está incorreta. A palavra “seize” significa “pegar/tomar” e não pode ser comparada
com a palavra “release”, que significa “liberar”.
A alternativa E está incorreta. A palavra “seize” significa “pegar/tomar” e não pode ser comparada
com a palavra “remove”, que significa “remover”.
GABARITO: B

Leia o texto a seguir e responda às questões 04, 05 e 06


New Zealand enters nationwide lockdown over one Covid case
New Zealand has announced a snap lockdown after a man tested positive for Covid, the first
case in six months.
The case ___ (1) detected in Auckland, which will be in lockdown for a week, while the rest of the
country will be in lockdown for three days.
Authorities say they are working on the assumption that the new case was the Delta variant.
Just around 20% of its population has been fully vaccinated.
Coromandel, a coastal town where the infected person had visited, will be in lockdown for seven
days too.
Prime Minister Jacinda Ardern ___ (2) the toughest "level 4" rules were required - closing schools,
offices and all businesses with only essential services remaining operational.
"I want to assure New Zealand that we have planned for this eventuality. Going hard and early has
worked for us before," she said.
The patient is a 58-year-old man, who is believed to have been infectious since last Thursday.
There ___ (3) reportedly a rush at supermarkets in Auckland, as locals anticipated a snap
lockdown.
Officials said there was a need for strong response because of the fear of the Delta variant, and
because there was no clear link between the new case and the border or quarantine facilities.
Data released by New Zealand's Ministry of Health on Monday showed that all Covid-19 cases
detected at the country's border in recent weeks had been Delta.
"We have seen what can happen elsewhere if we fail to get on top of it. We only get one chance,"
Ms Ardern said in a televised national address, calling the Delta strain "a game changer".

AULA 10 – PASSIVE VOICE, TAG QUESTIONS AND IDIOMS 110


TEACHER ANDREA BELO

New Zealand has been successful in eliminating the virus from within its borders, although its
international borders remain largely closed.
However, its vaccination programme has rolled out at a slow pace, with only around 20% of people
fully vaccinated and 33% of people having received one dose, according to Our World in Data.
(Adapted from https://www.bbc.com/news/world-asia-58241619)

QUESTÃO 04 (EsPCEx/INÉDITA) – In the sentence “...there was no clear link between the new
case and the border or quarantine facilities” (paragraph 9), the word “clear” means
A) Blocked
B) Free
C) Evident
D) Complicated
E) Hidden
Comentários: A alternativa A está incorreta. A palavra “clear” significa “clara” e não pode ser
comparada com a palavra “blocked”, que significa “bloqueado”.
A alternativa B está incorreta. A palavra “clear” significa “clara” e não pode ser comparada com a
palavra “free”, que significa “livre”.
A alternativa C está correta. A palavra “clear” significa “clara” e pode ser comparada com a palavra
“evident”, que significa “evidente”.
A alternativa D está incorreta. A palavra “clear” significa “clara” e não pode ser comparada com a
palavra “complicated”, que significa “complicado”.
A alternativa E está incorreta. A palavra “clear” significa “clara” e não pode ser comparada com a
palavra “hidden”, que significa “escondido”.
GABARITO: C

QUESTÃO 05 (EsPCEx/INÉDITA) – Choose the alternative containing the correct verb forms to
complete the gaps (1), (2) and (3) in paragraphs 1, 5 and 8 respectively
A) Will be, said, was
B) Was, says, was
C) Will be, said, were
D) Was, said, were
E) Was, said, was
Comentários: A primeira lacuna deve ser preenchida com o verbo “to be” (ser ou estar) no simple
past referindo-se ao pronome “it”, ou seja, “was” (foi); pois a frase indica algo que já aconteceu,
no passado.

AULA 10 – PASSIVE VOICE, TAG QUESTIONS AND IDIOMS 111


TEACHER ANDREA BELO

A segunda lacuna deve ser preenchida com o verbo “to say” (dizer) no simple past referindo-se ao
pronome “she”, ou seja, “said” (disse); pois a frase indica algo que já aconteceu, no passado.
A terceira lacuna deve ser preenchida com o verbo “to be” (ser ou estar) no simple past referindo-
se ao pronome “it”, ou seja, “was” (foi); pois a frase indica algo que já aconteceu, no passado.
GABARITO: E

QUESTÃO 06 (EsPCEx/INÉDITA) – According to the text, choose the correct statement


A) All of New Zealand will be in lockdown for 7 days
B) Despite coping well with the pandemic, New Zealand is not promoting rapid vaccination
C) Fortunately, the infected person has only been to one city
D) New Zealand's approach to the pandemic has never been effective
E) New Zealand has already provided vaccine for all its inhabitants
Comentários: A alternativa A está incorreta. De acordo com o texto, não é correto afirmar que
toda a Nova Zelândia ficará em lockdown por 7 dias, mas sim, que apenas duas cidades ficarão em
lockdown por 7 dias, o resto ficará por apenas 3 dias. Isso pode ser confirmado com o trecho
“...Auckland, which will be in lockdown for a week, while the rest of the country will be in lockdown
for three days … Coromandel, a coastal town where the infected person had visited, will be in
lockdown for seven days too”.
A alternativa B está correta. De acordo com o texto, é correto afirmar que apesar de lidar bem
com a pandemia, a Nova Zelândia não está promovendo a vacinação rápida, assim como esta
opção indica. Isso pode ser confirmado com os trechos “Going hard and early has worked for us
before” “…its vaccination programme has rolled out at a slow pace, with only around 20% of
people fully vaccinated and 33% of people having received one dose…”.
A alternativa C está incorreta. De acordo com o texto, não é correto afirmar que felizmente, a
pessoa infectada só esteve em uma cidade, mas sim, que ela esteve em, pelo menos, duas cidades.
Isso pode ser confirmado com o trecho “The case ___ (1) detected in Auckland ... Coromandel, a
coastal town where the infected person had visited…”.
A alternativa D está incorreta. De acordo com o texto, não é correto afirmar que a abordagem da
Nova Zelândia à pandemia nunca foi eficaz, mas sim, que foi. Isso pode ser confirmado com os
trechos “Going hard and early has worked for us before”.
A alternativa E está incorreta. De acordo com o texto, não é correto afirmar que a Nova Zelândia
já forneceu vacina para todos os seus habitantes, mas sim, que apenas 33% da população recebeu
a primeira dose. Isso pode ser confirmado com o trecho “...its vaccination programme has rolled
out at a slow pace, with only around 20% of people fully vaccinated and 33% of people having
received one dose…”.
GABARITO: B

AULA 10 – PASSIVE VOICE, TAG QUESTIONS AND IDIOMS 112


TEACHER ANDREA BELO

Leia o texto a seguir e responda às questões 07, 08 e 09


(Título omitido propositalmente)
Born in 1981, Britney Spears is an American singer, songwriter, and actress who influenced the
revival of teen pop during the 1990s and 2000s. People often referred to her as the Princess of
Pop.
In 2005 and 2006, she gave birth to her sons Sean and Federline; however, in 2006, she filed for
divorce from her husband Kevin Federline. They reached an agreement to share joint custody of
their sons.
Then, Spears was admitted to several drug rehabilitation facilities, and in 2007, she lost custody
of her sons to Federline. In 2008, a court placed Spears under conservatorship of her father James
Spears, giving him complete control of her life and assets for 13 years.
Now, Britney´s father is stepping down from the conservatorship. According to his lawyer, there
are no actual reasons to suspend or remove him; however, James Spears is responding to the
public battle and a petition for his suspension.
(Adapted from https://www.newsinlevels.com/products/britney-spears-free-again-level-3/)

QUESTÃO 07 (EsPCEx/INÉDITA) – Choose the most appropriate title for the text
A) Britney Spears free again
B) Britney Spears Maternity
C) How Rehab Affected Britney Spears
D) What was the role of Britney Spears' father in her rehabilitation?
E) How are Britney Spears' kids?
Comentários: O texto descreve brevemente a vida de Britney Spears e termina dando foco ao fato
de que seu pai abdicou de sua tutela e ela se torna livre novamente, ou seja, o título apropriado
é “Britney Spears free again” (Britney Spears livre novamente).
GABARITO: A

QUESTÃO 08 (EsPCEx/INÉDITA) – What kind of text is this?


A) Opinion article
B) News report
C) Letter
D) Essay
E) Textbook
Comentários: A alternativa A está incorreta. O texto dado não é um artigo de opinião, mas sim,
um relatório de notícia (news report).
A alternativa B está correta. O texto dado é um relatório de notícia, assim como esta opção indica.

AULA 10 – PASSIVE VOICE, TAG QUESTIONS AND IDIOMS 113


TEACHER ANDREA BELO

A alternativa C está incorreta. O texto dado não é uma carta, mas sim, um relatório de notícia
(news report).
A alternativa D está incorreta. O texto dado não é uma redação, mas sim, um relatório de notícia
(news report).
A alternativa E está incorreta. O texto dado não é um livro didático, mas sim, um relatório de
notícia (news report).
GABARITO: B

QUESTÃO 09 (EsPCEx/INÉDITA) – In the sentence “According to his lawyer, there are no actual
reasons to suspend or remove him…” (paragraph 4), the word “actual” means
A) Confused
B) Hypothetical
C) Doubtful
D) Problematic
E) Real
Comentários: A alternativa A está incorreta. A palavra “actual” significa “real” e não pode ser
comparada com a palavra “confused”, que significa “confuso”.
A alternativa B está incorreta. A palavra “actual” significa “real” e não pode ser comparada com a
palavra “hypothetical”, que significa “hipotético”.
A alternativa C está incorreta. A palavra “actual” significa “real” e não pode ser comparada com a
palavra “doubtful”, que significa “duvidoso”.
A alternativa D está incorreta. A palavra “actual” significa “real” e não pode ser comparada com a
palavra “problematic”, que significa “problemático”.
A alternativa E está correta. A palavra “actual” significa “real” e pode ser comparada com a palavra
“real”, que significa “real”.
GABARITO: E

Leia o texto a seguir e responda à questão 10


Afghan women to have rights within Islamic law, Taliban say
The rights of women in Afghanistan will be respected "within the framework of Islamic law",
the Taliban say.
In the group's first news conference since taking control of the country ___ (1) Sunday, a
spokesman said women would be free to work but gave little detail about other rules and
restrictions.
Zabihullah Mujahid repeated that all Afghans must live "within the framework of Islam".
Rights groups fear women's freedoms could be eroded under the Taliban.

AULA 10 – PASSIVE VOICE, TAG QUESTIONS AND IDIOMS 114


TEACHER ANDREA BELO

The militant group introduced or supported punishments in line with their strict interpretation of
Islam's legal system, Sharia law, when they controlled Afghanistan between 1996 and 2001.
Women had to wear the all-covering burka, and the Taliban also disapproved of girls aged 10 and
over going to school.
In the news briefing on Tuesday, Mr Mujahid fielded several questions from the international
media about what women's rights could look like under a Taliban government.
"We are going to allow women to work and study within our frameworks," he said. "Women are
going to be very active within our society."
But he did not expand when asked ___ (2) dress codes and what roles women would be able to
have within ___ (3) country's workforce.
(Adapted from https://www.bbc.com/news/world-asia-58249952)

QUESTÃO 10 (EsPCEx/INÉDITA) – Choose the alternative containing the correct words to


complete the gaps (1), (2) and (3) in paragraphs 1, 7 and 7 respectively
A) On, about, X
B) In, about, the
C) In, from, the
D) On, about, the
E) On, about, a
Comentários: A primeira lacuna deve ser preenchida com “on”, pois a preposição antecede um
dia da semena, ou seja, “Na primeira coletiva de imprensa do grupo desde que assumiu o controle
do país no domingo...”.
A segunda lacuna deve ser preenchida com “about”, pois a frase se refere ao assunto perguntado
ao Talibã, ou seja, “Mas ele não se expandiu quando questionado sobre os códigos de
vestimenta...”.
A terceira lacuna deve ser preenchida com “the”, pois a frase se refere a uma força de trabalho
específica, ou seja, “...e quais funções as mulheres seriam capazes de desempenhar na força de
trabalho do país”.
GABARITO: D

AULA 10 – PASSIVE VOICE, TAG QUESTIONS AND IDIOMS 115


TEACHER ANDREA BELO

CONSIDERAÇÕES FINAIS
Última aula alcançada com sucesso – passo enorme para sua aprovação!
E, dia após dia, os tópicos aprendidos aumentam, seu conhecimento fica mais amplo, o
vocabulário que você conhece se estende e a tendência é melhorar e ser capaz de alcançar a
aprovação de fato.
Nota-se o avanço em seus estudos e, provavelmente, uma maior tranquilidade para
enfrentar os exercícios que surgem. E você vai se acostumando a equilibrar seus estudos de forma
sistematizada, estudando cada vez mais e com mais dedicação.
Outro detalhe importante para seu sucesso nos estudos, é continuar fazendo aquelas listas
de vocabulário que aconselhei você, com palavras, verbos variados e termos que você considere
importante de ser anotado, de ser revisto, estudado.
Isso te ajudará nas questões futuras e torna você, como eu disse antes, um candidato mais bem
preparado e confiante para realizar uma excelente prova de vestibular.
É importante lembrar também do nosso Fórum de dúvidas, exclusivo do Estratégia
Militares. Será minha forma de responder você, esclarecer o que mais você precise saber para
que os conteúdos fiquem ainda mais claros em seus estudos, certo?
E, caso queira, acesse minhas redes sociais para aprender mais palavras e contar com dicas
importantes, que colaboram diretamente com seus estudos dia após dia.

AULA 10 – PASSIVE VOICE, TAG QUESTIONS AND IDIOMS 116


TEACHER ANDREA BELO

REFERÊNCIAS BIBLIOGRÁFICAS
BARRETO, Tania Pedroza; GARRIDO, Maria Line; SILVA, João Antenor de C., Inglês Instrumental.
Leitura e compreensão de textos. Salvador, Ba UFBA, 1995, p. 64.
BROWN. H. Douglas. Principles of Language Learning and Teaching. Prentice Hall International,
1988.
COMPEDELLI, Samira Yousseff. Português, Literatura, Produção de texto & Gramática – São
Paulo: Ed. Saraiva, 2002.
CORREIA, Clese Mary P. Reading Specific Purposes. Salvador/ Ba: UFBA, 1999.
COSTEIRA, Adriana Araújo de M. Reading Comprehension Skills. João Pessoa/PB: ETFP, 1998.
CRYSTAL David. Cambridge University Press 1997. The Cambridge Encyclopedia of Language.
Cambridge University Press 1997
FREEMAN. Diane Larsen. MURCIA. Marianne Celce. The Grammar Book, 1999.
DYE, Joan., FRANFORT, Nancy. Spectrum II, III A Communicative Course in English. USA, Prentice
Hall, 1994.
FAVERO, Maria de Lourdes Albuquerque (org.). Dicionário de educadores no Brasil: da colônia
aos dias atuais. Rio de Janeiro: UFRJ, MEC, INEP, 1999.
FRANKPORT, Nancy & Dye Hoab. Spectrum II, III Prentice Hall Regents Englewood Cliffs, New
Jersy, 1994.
GADELHA, Isabel Maria B. Inglês Instrumental: Leitura, Conscientização e Prática. Teresina:
EDUFFI, 2000.
GUANDALINI, Eiter Otávio. Técnicas de Leitura em Inglês: ESP – English For Specific Purposes:
estágio 1. São Paulo: Texto novo, 2002.
GRELLET, Françoise. Developing Reading Skills. Cambridge University Press, 1995
HOLAENDER, Arnon & Sanders Sidney. A complete English Course. São Paulo. Ed. Moderna,
1995.
HUTCHINSON, Tom & WATERS, Alan. English for Specific Purposes. Cambridge: Cambridge
University Press, 1996
KRASHEN. Stephen D. Second Language Acquisition and Second Language Learning, Prentice-Hall
International, 1988.
LAENG, Mauro. Dicionário de pedagogia. Lisboa: Dom Quixote, 1973.

AULA 10 – PASSIVE VOICE, TAG QUESTIONS AND IDIOMS 117


TEACHER ANDREA BELO

LEFFA, Vilson J. Metodologia do ensino de línguas. In: BOHN, H.; VANDRESEN, P. (org.). Tópicos
de linguística aplicada: o ensino de línguas estrangeiras. Florianópolis: Editora da UFSC, 1988. p.
211-231.
LIBERATO, Wilson. Compact English Book Inglês Ensino Médio. São Paulo: FTD, Vol. Único, 1998
Mc ARTHUR. The Oxford Companion to the English Language. Oxford University Press 1992
Fromkin. Victoria. An Introduction to Language
MARQUES, Amadeu. Inglês Série Brasil. ed. Atica. São Paulo: 2004. Vol. Único.
MURPHY, Raymond: Essencial Grammar in Use Oxford. New York Ed. Oxford University, 1997.
OLIVEIRA, Luciano Amaral. English For Tourism Students. Inglês para Estudantes de Turismo: São
Paulo, Rocca, 2001.
OLIVEIRA, Sara Rejane de F. Estratégias de leitura para Inglês Instrumental. Brasília: UNB, 1994.
QUINTANA, et alli. First Certificate. Master Class Oxford. New York, 2004: Ed. Oxford University.
PAULINO, Berenice F. et all. Leitura em textos em Inglês – Uma Abordagem Instrumental. Belo
Horizonte: Ed. Dos Autores, 1992.
PEREIRA, Edilberto Coelho. Inglês Instrumental. Teresina: ETFPI, 1998.
RODGES, Theodore. Jack C. Richards. Approaches and Methods in Language Teaching. Cambridge
University Press, 2001.
RODMAN Robert. Harcourt Brace 1993. English as a Global Language
STEWART, B., HAINES S. First Certificate, MasterClass. UK – Oxford 2004.
SILVA, João Antenor de C., GARRIDO, Maria Lina, BARRETO, Tânia Pedrosa. Inglês Instrumental:
Leitura e Compreensão de Textos. Salvador: Centro Editorial e Didático, UFBA. 1994
SOARES, Moacir Bretãs. Dicionário de legislação do ensino. 19.ed. Rio de Janeiro: FGV, 1981.
SOUZA, Adriana Srade F. Leitura em Língua Inglesa: Uma abordagem Instrumental. São Paulo:
Disal, 2005.
TUCK, Michael. Oxford Dictionary of Computing for Learners of English. Oxford: Oxford
University Press, 1996.
TOTIS, Verônica Pakrauskas. Língua Inglesa: leitura. São Paulo: Cortez, 1991.

Livros eletrônicos:
Dicionário Houaiss da Língua Portuguesa, Editora Objetiva, 2001.
MOURãO, Janaína Pereira. "Skimming x Scanning"; Brasil Escola. Disponível em
<https://brasilescola.uol.com.br/ingles/skimming-x-scanning.htm>. Acesso em 20 de março de
2019.
www.newsweek.com - Acesso em 18 de março de 2019.
http://www.galaor.com.br/tecnicas-de-leitura/ - Acesso em 19 de março de 2019.

AULA 10 – PASSIVE VOICE, TAG QUESTIONS AND IDIOMS 118


TEACHER ANDREA BELO

Expressões Idiomáticas (continuação)" em Só Língua Inglesa. Virtuous Tecnologia da


Informação,2008-2019. Consultado em 03/04/2019 às 22:09. Disponível na Internet em
http://www.solinguainglesa.com.br/conteudo/Expressoes5.php

TRADUÇÕES
Industries shut, Ganga water quality improves
With industries shutdown and people confined to their homes for days following the imposition
of the 21-day lockdown, the water quality of Ganga river has significantly improved, according to
a IITBHU professor.
Dr P K Mishra, a professor at the institute's Chemical Engineering department, attributed the
improvement to the lack of industrial waste polluting the waters.
"One-tenth of the pollution in Ganga river comes from industries. As industries are shut due to
lockdown, situation has become better. We have seen 40-50 per cent improvement in the Ganga.
It is a significant development," Mishra told news agency ANI. With rainfall on March 15 and 16,
the Ganga's water level has also increased, he said, adding that this meant the river's cleaning
capacity has also increased.
"There is a lot of difference between when we see the water of the Ganga river today and what
used to be earlier. Today, the water looks clean," ANI quoted a local as saying. He added that
people are also not taking bath at the ghats — a common ritual among locals there. "If this is the
condition in 10 days, then I believe Ganga river will be like it used to be earlier," he added. Another
local said nobody would have thought that the lockdown would have such an impact on
environment.
With several countries under lockdown to contain the spread of coronavirus, the global pollution
levels have also dropped significantly. In India, pollution levels have dipped as the lockdown in
megacities has kept cars off the road and closed factories. New Delhi, which regularly has
unhealthy air conditions, has now seen its AQI falling below 95 — a big reduction from its monthly
average of 161 from March 2019. Mumbai, too, has witnessed a similar reduction in pollution.
Due to the improvement in air quality following in India, Dhauladhar range, which is part of a
Himalayan chain of mountains in Himachal Pradesh, has now become visible from Jalandhar in
Punjab, The mountain rises from Kangra and Mandi.
Adapted from (https://www.msn.com/en-in/news/environment/industries-shut-ganga-water-quality-improves/ar-BB12beCN)

Indústrias fechadas, qualidade da água do Ganga melhora


Com o fechamento de indústrias e as pessoas confinadas em suas casas por dias após a imposição
do bloqueio de 21 dias, a qualidade da água do rio Ganga melhorou significativamente, de acordo
com um professor do IITBHU.

AULA 10 – PASSIVE VOICE, TAG QUESTIONS AND IDIOMS 119


TEACHER ANDREA BELO

O Dr. P K Mishra, professor do departamento de Engenharia Química do instituto, atribuiu a


melhora à falta de resíduos industriais poluindo as águas.
"Um décimo da poluição no rio Ganga vem das indústrias. Como as indústrias estão fechadas
devido ao bloqueio, a situação melhorou. Vimos uma melhoria de 40-50 por cento no Ganga. É
um desenvolvimento significativo", disse Mishra à notícia. agência ANI. Com as chuvas nos dias 15
e 16 de março, o nível da água do Ganga também aumentou, disse ele, acrescentando que isso
significa que a capacidade de limpeza do rio também aumentou.
“Há muita diferença entre quando vemos a água do rio Ganga hoje e o que costumava ser antes.
Hoje, a água parece limpa”, disse um morador da ANI. Ele acrescentou que as pessoas também
não estão tomando banho nos ghats - um ritual comum entre os moradores locais. "Se esta for a
condição em 10 dias, então acredito que o rio Ganga estará como era antes", acrescentou. Outro
local disse que ninguém pensaria que o bloqueio teria tanto impacto sobre o meio ambiente.
Com vários países bloqueados para conter a disseminação do coronavírus, os níveis globais de
poluição também caíram significativamente. Na Índia, os níveis de poluição caíram à medida que
o bloqueio em megacidades manteve os carros fora das estradas e fechou fábricas. Nova Delhi,
que regularmente tem condições de ar insalubres, agora viu seu AQI cair abaixo de 95 - uma
grande redução em relação à média mensal de 161 de março de 2019. Mumbai também
testemunhou uma redução semelhante na poluição.
Devido à melhoria da qualidade do ar na Índia, a cordilheira Dhauladhar, que faz parte de uma
cadeia de montanhas do Himalaia em Himachal Pradesh, agora se tornou visível a partir de
Jalandhar, no Punjab. A montanha sobe de Kangra e Mandi.

Around the 18th and 19th centuries two main ideas about dreams _______ (become) popular.
One was the idea that the things we see in our dreams are things our conscious mind is hiding
from us. However, the opposite idea _______ (say) that while we’re asleep, the brain _______
(organise) memories and events from the day. Dreams are just a random collection of these
thoughts, but we _______(try) to make a story from them when we wake up.

Por volta dos séculos 18 e 19, duas ideias principais sobre os sonhos _______ (se tornaram)
populares. Uma era a ideia de que as coisas que vemos em nossos sonhos são coisas que nossa
mente consciente está escondendo de nós. No entanto, a ideia oposta _______ (dizer) que
enquanto estamos dormindo, o cérebro _______ (organiza) memórias e eventos do dia. Os sonhos
são apenas uma coleção aleatória desses pensamentos, mas nós _______ (tentamos) fazer uma
história com eles quando acordamos.

Who was Steve Jobs?

AULA 10 – PASSIVE VOICE, TAG QUESTIONS AND IDIOMS 120


TEACHER ANDREA BELO

Do you have an iPad, iPod, iPhone, a smartwatch or a Mac computer? If you


don’t, you probably know someone who has one (or wants one)!
Steve Jobs made the company – Apple – that created these things that are now
such an important part of the lives of millions of people.
He was born on February 24, 1955, in San Francisco, California. When he was a
boy, he had a special hobby: he liked to take apart televisions and put them
back together again. He was a very good student in school and even skipped a
grade. After he finished grade four, he went into grade six, and in 1972, when
he was seventeen, he graduated from high school. He then began his studies in
Reed College, in Portland, Oregon, but dropped out after six months. Deciding
to quit was not at all easy (...)
After Steve Jobs returned to the United States, he found a job as a video game designer at a
company called Atari, and two years after that, in 1976, when he was only twenty-one years old,
he created his own company – Apple Computer – with his friend, Steve Wozniak. Steve Jobs later
got married in 1991 and had four children. Unfortunately, doctors discovered that he had cancer
in 2003 and in 2011 he died at the age of only 56.
https://www.allthingstopics.com/uploads/2/3/2/9/23290220/7300865_orig.png

Quem foi Steve Jobs?


Você tem um iPad, iPod, iPhone, smartwatch ou um computador Mac? Se não,
provavelmente você conhece alguém que tem um (ou quer um)!
Steve Jobs criou a empresa - a Apple - que criou essas coisas que agora são uma
parte tão importante da vida de milhões de pessoas.
Ele nasceu em 24 de fevereiro de 1955, em San Francisco, Califórnia. Quando
era menino, ele tinha um hobby especial: gostava de desmontar televisores e
montá-los novamente. Ele era um aluno muito bom na escola e até pulou uma
série. Depois de terminar a quarta série, ele foi para a sexta série e, em 1972,
quando tinha dezessete anos, concluiu o ensino médio. Ele então começou
seus estudos no Reed College, em Portland, Oregon, mas desistiu depois de
seis meses. Decidir desistir não foi nada fácil (...)
Depois que Steve Jobs voltou aos Estados Unidos, ele encontrou um emprego como designer de
videogame em uma empresa chamada Atari, e dois anos depois, em 1976, quando tinha apenas
21 anos, ele criou sua própria empresa - a Apple Computador - com seu amigo Steve Wozniak.
Steve Jobs mais tarde se casou em 1991 e teve quatro filhos. Infelizmente, os médicos descobriram
que ele tinha câncer em 2003 e em 2011 morreu com apenas 56 anos.

FIVE WAYS TO GET MORE FIBRE IN YOUR DIET

AULA 10 – PASSIVE VOICE, TAG QUESTIONS AND IDIOMS 121


TEACHER ANDREA BELO

Roughage helps reduce the risk of heart disease and bowel


cancer, yet few of us eat enough of it. Here’s how to up your
intake.
Fibre, or roughage, refers to indigestible carbohydrates. A fibre-
rich diet is linked to health benefits including a reduced risk of
heart disease and bowel cancer. While UK guidelines say adults
should get 30g a day, fewer than one in 10 meet this goal.
Popular low-carb diets may be a reason why. Understanding
what is in your food _____ help: a typical apple contains 2-3g of fibre, a sesame bagel about 4g.
Jo Greening, a spokesperson for the British Dietetic Association (BDA), says it is worth checking
the labels, as different brands have different levels of fibre.
CINCO MANEIRAS DE OBTER MAIS FIBRA NA SUA DIETA
A forragem ajuda a reduzir o risco de doenças cardíacas e câncer
de intestino, mas poucos de nós comemos o suficiente. Veja
como aumentar sua ingestão.
Fibra, ou volumoso, refere-se a carboidratos indigeríveis. Uma
dieta rica em fibras está associada a benefícios para a saúde,
incluindo redução do risco de doenças cardíacas e câncer de
intestino. Embora as diretrizes do Reino Unido digam que os
adultos devem ingerir 30g por dia, menos de um em cada 10
atinge essa meta. As dietas populares com baixo teor de carboidratos podem ser o motivo.
Entender o que está em sua comida _____ ajuda: uma maçã típica contém 2-3g de fibra, um bagel
de gergelim cerca de 4g. Jo Greening, porta-voz da British Dietetic Association (BDA), diz que vale
a pena conferir os rótulos, pois diferentes marcas têm diferentes níveis de fibra.

A solicitor engaged by former members of the Supreme Horse Racing Club


said on Thursday evening he was “very optimistic” the 29 horses it has
hitherto owned, including the top-class chaser Kemboy, will be able to race
this season.
The Thurles-based Patrick Kennedy was speaking after the ruling body of
Irish horse racing in effect kicked SHRC out of the sport by declaring it was
no longer “permitted to be owners or part-owners of racehorses”.
The announcement from Horse Racing Ireland represents the moment
when its patience expired over the club’s repeated failure to provide answers to its questions.
Following complaints from club members about alleged irregularities in the way SHRC was run,
HRI got involved this summer and its concerns have not been allayed.

AULA 10 – PASSIVE VOICE, TAG QUESTIONS AND IDIOMS 122


TEACHER ANDREA BELO

Um advogado contratado por ex-membros do Supreme Horse Racing Club


disse na noite de quinta-feira que estava "muito otimista" com os 29 cavalos
que possuía até agora, incluindo o caçador de primeira classe Kemboy, que
poderão correr nesta temporada.
Patrick Kennedy, de Thurles, falava depois que o corpo dirigente das
corridas de cavalos irlandesas expulsou o SHRC do esporte ao declarar que
não era mais “permitido ser proprietários ou co-proprietários de cavalos de
corrida”.
O anúncio da Horse Racing Ireland representa o momento em que sua paciência se esgotou com
o repetido fracasso do clube em fornecer respostas às suas perguntas.
Após reclamações de membros do clube sobre supostas irregularidades na forma como o SHRC
era administrado, o HRI se envolveu neste verão e suas preocupações não foram dissipadas.

por Cambridge English, 22/10/2019


Ensinar inglês é uma profissão gratificante que pode
permitir que você more e trabalhe em lugares
fascinantes em todo o mundo. Isso lhe dará a
oportunidade de aprender habilidades valiosas,
como resolução de problemas, comunicação e
flexibilidade. Além de desenvolver suas habilidades
profissionais, você também terá a oportunidade de
vivenciar diferentes países e culturas. Você pode até
ter a chance de aprender um novo idioma também!

Taal: The 'very small but dangerous volcano'

AULA 10 – PASSIVE VOICE, TAG QUESTIONS AND IDIOMS 123


TEACHER ANDREA BELO

Over the past few days, it's begun spewing lava, triggering earthquakes and emitting huge plumes
of ash that have spread across the island of Luzon and beyond.
Scientists fear a bigger "hazardous eruption" is imminent. Taal is tiny, as volcanoes go, but it has
been deadly before. And according to Renato Solidum, the head of the Philippines' Institute of
Volcanology and Seismology (Philvolcs), it is "very small but a dangerous volcano". "Taal volcano
is a baby volcano sitting within a much bigger caldera volcano," said Ben Kennedy, associate
professor of physical volcanology at the University of Canterbury in New Zealand. The entire
Volcano Island has been marked as a permanent danger zone by Phivolcs.
BBC News (Jan/2020)

Taal: O 'vulcão muito pequeno, mas perigoso'

Nos últimos dias, começou a cuspir lava, provocando terremotos e emitindo enormes nuvens de
cinzas que se espalharam pela ilha de Luzon e além.
Os cientistas temem que uma "erupção perigosa" maior seja iminente. Taal é minúsculo, como os
vulcões, mas já foi mortal antes. E de acordo com Renato Solidum, chefe do Instituto de
Vulcanologia e Sismologia das Filipinas (Philvolcs), é "muito pequeno, mas um vulcão perigoso".
"O vulcão Taal é um vulcão bebê localizado dentro de uma caldeira vulcânica muito maior", disse
Ben Kennedy, professor associado de vulcanologia física da Universidade de Canterbury, na Nova
Zelândia. Toda a Ilha do Vulcão foi marcada como zona de perigo permanente por Phivolcs.
BBC News (janeiro / 2020)

AULA 10 – PASSIVE VOICE, TAG QUESTIONS AND IDIOMS 124


TEACHER ANDREA BELO

A solicitor engaged by former members of the Supreme Horse Racing


Club said on Thursday evening he was “very optimistic” the 29 horses
it has hitherto owned, including the top-class chaser Kemboy, will be
able to race this season.
The Thurles-based Patrick Kennedy was speaking after the ruling body
of Irish horse racing in effect kicked SHRC out of the sport by declaring
it was no longer “permitted to be owners or part-owners of
racehorses”.
The announcement from Horse Racing Ireland represents the moment when its patience expired
over the club’s repeated failure to provide answers to its questions.
The Guardian – November 14th

Um advogado contratado por ex-membros do Supreme Horse Racing


Club disse na noite de quinta-feira que estava "muito otimista" com os
29 cavalos que possuía até agora, incluindo o caçador de primeira
classe Kemboy, que poderão correr nesta temporada.
Patrick Kennedy, de Thurles, falava depois que o corpo dirigente das
corridas de cavalos irlandesas expulsou o SHRC do esporte ao declarar
que não era mais “permitido ser proprietários ou co-proprietários de
cavalos de corrida”.
O anúncio da Horse Racing Ireland representa o momento em que sua paciência se esgotou com
o repetido fracasso do clube em fornecer respostas às suas perguntas.

Netflix edits Squid Game phone number after woman deluged with calls
Popular drama’s use of real South Korean number leads to thousands of prank calls and texts
Netflix has edited out a phone number that appears ___ its hit series Squid Game after a South
Korean woman and others who use similar combinations were deluged with calls – with some
callers even asking to join the show’s life-or-death games.
The South Korea-made production has topped Netflix popularity charts in 90 countries since its
launch last month and is on track to become its most watched series ever.
The nine-episode drama, which addresses widening economic inequality, involves hundreds of
cash-strapped people competing ___ children’s games to win the final reward of 45.6bn Korean
won (£28m), with the losers killed in uncompromising and violent ways.
To take part, contestants have to call a number on a business card printed with symbols. But while
film and television makers usually use fake numbers in such circumstances, adding 010 – the
standard prefix ___ South Korean mobiles – to the eight digits on the card generated a real phone
number.

AULA 10 – PASSIVE VOICE, TAG QUESTIONS AND IDIOMS 125


TEACHER ANDREA BELO

It belongs to a South Korean woman who said she received thousands of calls and text messages
to her phone “to the point that it’s hard for me to go on with daily life”.
(Adapted from https://www.theguardian.com/media/2021/oct/07/netflix-edits-squid-game-phone-number-woman-deluged-calls)

A Netflix edita o número de telefone do Squid Game depois que uma mulher recebe muitas
ligações
O uso de números sul-coreanos reais em dramas populares leva a milhares de trotes e
mensagens de texto
A Netflix editou um número de telefone que parece ___ sua série de sucesso Squid Game depois
que uma mulher sul-coreana e outras que usam combinações semelhantes foram inundadas com
chamadas – com alguns chamadores até pedindo para entrar nos jogos de vida ou morte do
programa.
A produção feita na Coreia do Sul liderou as paradas de popularidade da Netflix em 90 países
desde seu lançamento no mês passado e está a caminho de se tornar a série mais assistida de
todos os tempos.
O drama de nove episódios, que aborda o aumento da desigualdade econômica, envolve centenas
de pessoas sem dinheiro competindo ___ jogos infantis para ganhar a recompensa final de 45,6
bilhões de won coreanos (£ 28 milhões), com os perdedores mortos de forma intransigente e
violenta.
Para participar, os competidores devem ligar para um número em um cartão de visita impresso
com símbolos. Mas embora os produtores de filmes e televisão usem números falsos em tais
circunstâncias, adicionar 010 - o prefixo padrão ___ celulares sul-coreanos - aos oito dígitos no
cartão gerou um número de telefone real.
Pertence a uma mulher sul-coreana que disse ter recebido milhares de ligações e mensagens de
texto em seu telefone "a ponto de ser difícil para mim continuar com a vida diária".

New tool identifies groups most at-risk from Covid after vaccination
While the risk of severe Covid-19 after vaccination remains low, some people remain more
vulnerable than others, research shows
The immunosuppressed and those with dementia, Parkinson’s or chronic disorders such as kidney
disease are still at a greater risk of hospitalisation or death from Covid after vaccination compared
to the rest of the population, new research shows.
The QCovid tool developed by scientists at the University of Oxford shows that while the risk of
severe Covid-19 after vaccination remains low, some people remain more vulnerable than others.
Age continues to be a “major risk factor”, the experts said, with older vaccinated individuals more
likely to end up ill from Covid compared to their younger counterparts.
Those from Indian and Pakistani backgrounds who are vaccinated face the highest risk among
ethnic groups.

AULA 10 – PASSIVE VOICE, TAG QUESTIONS AND IDIOMS 126


TEACHER ANDREA BELO

People living with chronic conditions such as Down’s syndrome, kidney disease, sickle cell disease,
HIV/Aids and liver cirrhosis are also at more risk of severe disease or hospital admission after
vaccination.
The new algorithm behind the QCovid tool, which was first developed in 2020 and helped
influence policy on shielding, predicts those most at risk of serious Covid-19 outcomes from 14 or
more days after second vaccination dose, when substantial immunity is expected to have
developed. The findings do not take into account the different types of vaccines received by
people.
(Adapted from https://www.independent.co.uk/news/health/covid-vaccine-at-risk-latest-b1922196.html)

Nova ferramenta identifica os grupos de maior risco da Covid após a vacinação


Embora o risco de Covid-19 grave após a vacinação permaneça baixo, algumas pessoas
permanecem mais vulneráveis do que outras, mostram pesquisas
Os imunossuprimidos e aqueles com demência, Parkinson ou doenças crônicas, como doença
renal, ainda correm um risco maior de hospitalização ou morte por Covid após a vacinação em
comparação com o resto da população, mostra uma nova pesquisa.
A ferramenta QCovid desenvolvida por cientistas da Universidade de Oxford mostra que, embora
o risco de Covid-19 grave após a vacinação permaneça baixo, algumas pessoas permanecem mais
vulneráveis do que outras.
A idade continua a ser um “fator de risco importante”, disseram os especialistas, com indivíduos
vacinados mais velhos mais propensos a adoecer por causa da Covid em comparação com seus
colegas mais jovens.
Pessoas de origens indianas e paquistanesas vacinadas enfrentam o maior risco entre os grupos
étnicos.
Pessoas que vivem com doenças crônicas, como síndrome de Down, doença renal, doença
falciforme, HIV / Aids e cirrose hepática também correm mais risco de doença grave ou internação
hospitalar após a vacinação.
O novo algoritmo por trás da ferramenta QCovid, que foi desenvolvido pela primeira vez em 2020
e ajudou a influenciar a política de blindagem, prevê aqueles com maior risco de resultados graves
de Covid-19 a partir de 14 ou mais dias após a segunda dose de vacinação, quando se espera que
uma imunidade substancial tenha se desenvolvido . Os resultados não levam em consideração os
diferentes tipos de vacinas recebidas pelas pessoas.

“Across the UK, children are preparing to return to school after almost two months ___ summer
holidays. But experts have warned that the number of coronavirus cases may surge ___ the
coming weeks, as more children and adults mix inside the classroom. Professor Jason Leitch,

AULA 10 – PASSIVE VOICE, TAG QUESTIONS AND IDIOMS 127


TEACHER ANDREA BELO

Scotland’s national clinical director, told Sky News on Tuesday that the UK could be facing a ‘fragile
moment’ ___ its response to the ongoing pandemic, as the Delta variant continues to spread, and
lockdown rules remain relaxed”.
(Adapted from https://www.independent.co.uk/life-style/health-and-families/coronavirus-children-school-testing-b1912550.html)

“Em todo o Reino Unido, as crianças estão se preparando para voltar à escola após quase dois
meses de ____ férias de verão. Mas os especialistas alertaram que o número de casos de
coronavírus pode aumentar ___ nas próximas semanas, à medida que mais crianças e adultos se
misturam na sala de aula. O professor Jason Leitch, diretor clínico nacional da Escócia, disse à Sky
News na terça-feira que o Reino Unido pode estar enfrentando um ‘momento frágil’ ___ sua
resposta à pandemia em curso, à medida que a variante Delta continua a se espalhar e as regras
de bloqueio permanecem relaxadas ”.

Malaria vaccine: When will it be available?


Researchers and health professionals have been celebrating after the World Health
Organization (WHO) approved the widespread use of the world's first malaria vaccine.
With more than 260,000 children under five dying from malaria each year in sub-Saharan Africa,
this development, decades in the making, could save tens of thousands of lives, the WHO says.
But when will people start benefitting from the vaccine known as RTS,S?
We have been looking at that and some other key questions.
How effective and safe is it?
The vaccine was proven effective six years ago, preventing 40% of malaria cases and 30% of severe
cases.
Since 2019, researchers have been carrying out wider pilot immunisation programmes in Ghana,
Kenya, and Malawi.
More than 800,000 children have received at least one dose and the WHO says there are no safety
concerns.
Does it matter that the protection is relatively low?
It would obviously be better if it was higher, but what many would say is you need to think about
the scale of the problem – with hundreds of millions of cases, a 40% reduction is still a huge
number of lives saved.
"This is a moderately efficacious vaccine… [but] saving, preserving, avoiding 30-40% of cases and
deaths can bring a major benefit to the population," the WHO's Pedro Alonso told BBC Focus on
Africa.
Health authorities are also keen to stress this is a new weapon in the fight against malaria to be
used alongside other preventative measures, such as treated bed nets and drugs that target the
malaria parasite.
How does the vaccine work?

AULA 10 – PASSIVE VOICE, TAG QUESTIONS AND IDIOMS 128


TEACHER ANDREA BELO

Malaria is a parasite that invades and destroys our blood cells in order to reproduce, and it's spread
by the bite of blood-sucking mosquitoes.
The vaccine targets the most deadly and common parasite in Africa: Plasmodium falciparum.
It tries to deal with the form of the parasite which enters the victim's blood shortly after being
bitten, by partially blocking access into human cells and therefore preventing disease, Dr Alonso
said.
It needs four doses to be effective. The first three are given a month apart at the age of five, six
and seven months, and a final booster is needed at around 18 months.
Children are considered to be the most at risk from dying from malaria as, unlike adults, they have
not had a chance to build up immunity.
(Adapted from https://www.bbc.com/news/world-africa-58833382)

Vacina contra a malária: quando estará disponível?


Pesquisadores e profissionais de saúde têm comemorado depois que a Organização Mundial
da Saúde (OMS) aprovou o uso generalizado da primeira vacina contra malária do mundo.
Com mais de 260.000 crianças menores de cinco anos morrendo de malária a cada ano na África
Subsaariana, esse desenvolvimento, que está ocorrendo há décadas, pode salvar dezenas de
milhares de vidas, afirma a OMS.
Mas quando as pessoas começarão a se beneficiar com a vacina conhecida como RTS, S?
Temos analisado isso e algumas outras questões-chave.
Quão eficaz e seguro é?
A vacina se mostrou eficaz há seis anos, prevenindo 40% dos casos de malária e 30% dos casos
graves.
Desde 2019, os pesquisadores têm conduzido programas-piloto de vacinação mais amplos em
Gana, Quênia e Malaui.
Mais de 800.000 crianças receberam pelo menos uma dose e a OMS diz que não há preocupações
com a segurança.
Faz diferença que a proteção seja relativamente baixa?
Obviamente seria melhor se fosse maior, mas o que muitos diriam é que você precisa pensar na
escala do problema - com centenas de milhões de casos, uma redução de 40% ainda é um grande
número de vidas salvas.
"Esta é uma vacina moderadamente eficaz ... [mas] salvar, preservar e evitar 30-40% dos casos e
mortes pode trazer um grande benefício para a população", disse Pedro Alonso, da OMS, à BBC
Focus on Africa.
As autoridades de saúde também destacam que se trata de uma nova arma na luta contra a
malária, a ser usada em conjunto com outras medidas preventivas, como mosquiteiros tratados e
medicamentos que visam o parasita da malária.
Como funciona a vacina?

AULA 10 – PASSIVE VOICE, TAG QUESTIONS AND IDIOMS 129


TEACHER ANDREA BELO

A malária é um parasita que invade e destrói nossas células sanguíneas para se reproduzir e se
espalha pela picada de mosquitos sugadores de sangue.
A vacina tem como alvo o parasita mais mortal e comum na África: Plasmodium falciparum.
Ele tenta lidar com a forma do parasita que entra no sangue da vítima logo após ser picada,
bloqueando parcialmente o acesso às células humanas e, portanto, evitando doenças, disse o Dr.
Alonso.
São necessárias quatro doses para ser eficaz. Os três primeiros são administrados com um mês de
intervalo na idade de cinco, seis e sete meses, e um reforço final é necessário por volta dos 18
meses.
As crianças são consideradas as que mais correm o risco de morrer de malária, pois, ao contrário
dos adultos, não tiveram a oportunidade de desenvolver imunidade.

The Brazilian army is turning into a de facto police force


Few places illustrate the modern role of the Brazilian army better than Tabatinga, a city of 62,000
on the shared border point between Brazil, Colombia, and Peru. The frontier has not budged since
the Portuguese built a now-ruined fort there in the 1700s. But Júlio Nagy, a local commander, has
his sights trained on unconventional threats. In February and March his troops intercepted 3.7
tonnes of cannabis. Last year they destroyed an airstrip built by illegal gold miners. Inside a small
army-run zoo, garish macaws rescued from animal traffickers squawk intermittently.
The country’s official defence review states that “at present, Brazil has no enemies”. Brazilian
strategists say that an insufficiency of military adversaries _______ _______ justify skimping on
defence. Criminal gangs operating in border areas can overwhelm civilian police, and in the future
Brazil hopes to deter foreigners covetous of its natural resources. New threats require new
responses. With 334,000 troops at its disposal, the government has had to find ways to deploy
them.
The army’s remit has expanded to police work. Most Brazilians seem unfazed by this trend. Unlike
politicians and police officers, servicemen are seen as honest, competent, and kind. Despite the
shadow of the dictatorship, confidence rankings of institutions often put the army at the top.
Soldiers are trying to adapt to their new role. At a training centre in Campinas, near São Paulo,
they are subjected to tear-gas and stun grenades, so they know what such weapons feel like before
unleashing them on civilians.
(Adapted from https://www.economist.com/the-americas/2017/07/06/the-brazilian-army-is-turning-into-a-de-facto-police-force)

Exército brasileiro está se transformando em uma força policial de fato


Poucos lugares ilustram melhor o papel moderno do exército brasileiro do que Tabatinga, uma
cidade de 62.000 habitantes na fronteira comum entre Brasil, Colômbia e Peru. A fronteira não
mudou desde que os portugueses construíram lá um forte agora em ruínas no século XVIII. Mas
Júlio Nagy, um comandante local, está de olho nas ameaças não convencionais. Em fevereiro e
março, suas tropas interceptaram 3,7 toneladas de cannabis. No ano passado, eles destruíram
uma pista de pouso construída por garimpeiros ilegais. Dentro de um pequeno zoológico
administrado pelo exército, araras berrantes resgatadas de traficantes de animais grasnam
intermitentemente.

AULA 10 – PASSIVE VOICE, TAG QUESTIONS AND IDIOMS 130


TEACHER ANDREA BELO

A avaliação oficial da defesa do país afirma que “atualmente o Brasil não tem inimigos”. Os
estrategistas brasileiros dizem que uma insuficiência de adversários militares _______ _______
justifica economizar na defesa. Gangues de criminosos que operam em áreas de fronteira podem
oprimir a polícia civil e, no futuro, o Brasil espera dissuadir os estrangeiros ambiciosos de seus
recursos naturais. Novas ameaças exigem novas respostas. Com 334.000 soldados à sua
disposição, o governo teve que encontrar maneiras de implantá-los.
As atribuições do exército se expandiram para o trabalho policial. A maioria dos brasileiros parece
não se incomodar com essa tendência. Ao contrário de políticos e policiais, os militares são vistos
como honestos, competentes e gentis. Apesar da sombra da ditadura, as classificações de
confiança das instituições costumam colocar o exército no topo. Os soldados estão tentando se
adaptar ao seu novo papel. Em um centro de treinamento em Campinas, próximo a São Paulo,
eles são submetidos a gás lacrimogêneo e granadas de atordoamento, para que saibam como são
essas armas antes de dispará-las contra civis.
Dogs are just like ‘play, eat, sleep’ – they bring me back into the moment. I think we can all learn
something from that,” says Carole Henderson, who has been taking her “furry backup” to the
office for the last few months.
They are not so good at making the tea and things get a bit rowdy when the delivery man comes
round, but Henderson’s labradors Barney and Rusty, and labradoodle Lily, have been her sidekicks
for the last decade. As well as being excellent foot-warmers, they have helped her emotionally
with getting through solo months in the office.
After the latest lockdown left her with no choice but to part-time furlough the rest of her team,
including her husband, the empty office felt like a cavern. “I felt a bit sorry for myself and thought:
‘I’m taking the dogs with me, because at least then I can have a doggy cuddle when things aren’t
going to plan,’” she says.
As a teacher of grief recovery method, Henderson’s work has felt more important than ever,
particularly with reaching out to those grieving in isolation, and without her colleagues there in-
person, her dogs have provided immense emotional support. “Things do get to me. I’m human,
doing this work. Someone will tell me a really sad story and there’s nothing better than getting a
cuddle from one of my dogs,” she says. “Stroking a lovely bit of fur brings you right back down,
then I’m ready to help the next person.”
(https://www.theguardian.com/lifeandstyle/2021/apr/06/the-dogs-keeping-office-workers-company-through-lockdown)

Os cães são como ‘brincar, comer, dormir’ - eles me trazem de volta ao momento. Acho que todos
nós podemos aprender algo com isso ”, diz Carole Henderson, que tem levado seu“ backup
peludo ”para o escritório nos últimos meses.
Eles não são tão bons em fazer chá e as coisas ficam um pouco turbulentas quando o entregador
chega, mas os labradores de Henderson, Barney e Rusty, e a labradoodle Lily, têm sido seus
ajudantes na última década. Além de serem excelentes aquecedores de pés, eles a ajudaram
emocionalmente com os meses sozinhas no escritório.
Depois que o último bloqueio a deixou sem escolha a não ser a licença de meio período para o
resto de sua equipe, incluindo seu marido, o escritório vazio parecia uma caverna. “Senti um
pouco de pena de mim mesma e pensei:‘ Vou levar os cachorros comigo, porque pelo menos assim

AULA 10 – PASSIVE VOICE, TAG QUESTIONS AND IDIOMS 131


TEACHER ANDREA BELO

poderei dar um abraço de cachorro quando as coisas não estiverem indo como planejado ’”, diz
ela.
Como professora do método de recuperação do luto, o trabalho de Henderson se sentiu mais
importante do que nunca, especialmente com a ajuda àqueles que estão sofrendo isoladamente,
e sem seus colegas pessoalmente, seus cães forneceram um imenso apoio emocional. “As coisas
me afetam. Eu sou humano, fazendo este trabalho. Alguém vai me contar uma história muito triste
e não há nada melhor do que receber o carinho de um dos meus cachorros ”, diz ela. "Afagar um
pedaço adorável de pelo traz você de volta para baixo, então estou pronto para ajudar a próxima
pessoa."

Limits on coronavirus testing in Brazil are hiding the true dimensions of Latin America’s largest
outbreak
Atop a shaded hill at the edge of São Paulo, the gravedigger thinks he knows the truth. No matter
how bad it appears in Brazil — the country hit hardest by the coronavirus in the Southern
Hemisphere — the reality is significantly worse.
Manoel Norberto Pereira watched another body being wheeled in, accompanied by what has by
now become a familiar set of details. Sex: female. Age: 77 years. Cause of death: insufficient
respiration.
Every day brings more. The cemetery now receives around 50 bodies every day — double the
average in normal times. Many are marked as confirmed cases of covid-19, the disease caused by
the virus. But many more cite only an unidentified respiratory ailment. To Pereira, they’re the
unseen toll of the coronavirus in Brazil, which has officially infected 51,000 people and killed 3,400
— but unofficially many times more than that.
Imprecise and insufficient testing is a global problem, but in Brazil, it's on an entirely different
scale. Latin America's largest country is testing people at a rate far lower than any other nation
with at least 40,000 cases. It tests 12 times fewer people than Iran, and 32 times fewer than the
United States. Hospitalized patients aren't being tested. Some medical professionals aren't being
tested. People are dying in their homes without being tested.
Researchers at the Federal University of Minas Gerais have suggested Brazil has eight times more
coronavirus cases than the official numbers indicate. A research team at the University of São
Paulo thinks it has 16 times more — more than 800,000 cases.
According to government statistics, nearly 37,300 people have been hospitalized this year with
respiratory ailments — four times the number at this point last year — but only half have received
test results. In São Paulo, nearly 1,300 people have died of unidentified respiratory problems.
Amazonas state has reported 193 deaths from covid-19. But its capital city is burying so many
people — three times the average — that it’s digging trenches for mass graves.
“The big conclusion is that we don’t know what the real scenario is,” said Leonardo Costa Ribeiro,
an economist at the Federal University of Minas Gerais. “It gives the sensation that it is more
controlled, when the reality is very different.”

AULA 10 – PASSIVE VOICE, TAG QUESTIONS AND IDIOMS 132


TEACHER ANDREA BELO

Domingos Alves, a data scientist at the University of São Paulo, did little to hide his anger at the
chasm between how government officials talk about the virus and what the data shows.
“As a researcher, I look at the data and make analyses for the government,” he said. “But as a
citizen, I’m frustrated. The government is trying to control the epidemic without the elements of
how to control it, because of their lack of knowledge about how serious it is.
“It will get much worse. There will be lines at the hospitals. There will be lines at the cemeteries.
The next few weeks will be very dark.”
The new health minister, Nelson Teich, named by Bolsonaro to replace Mandetta, has made
increased testing a pillar of the country’s response to the pandemic. He said this week that the
government intends to purchase 46 million tests, enough for just over a fifth of the population.
He hasn’t specified when the tests will arrive or when they’ll be used.
“We are not talking about testing the entire country,” Teich said. “We are going to use the tests so
that people tested will reflect the Brazilian population.”
But the obstacles will be enormous — and could portend the struggles that await much of the
developing world as the coronavirus moves deeper into Latin America and Africa. A variable as
basic as reliable data could become a distinguishing factor between poor and wealthy countries.
Analysts say Brazil has neither the manufacturing nor the purchasing capacity to meet the demand
for tests. Its laboratories, already overwhelmed by a testing backlog, aren’t equipped to process
them at scale. And in the international scrum for supplies and testing equipment, the country is
losing out to wealthier nations that can pay more or leverage closer ties with China.
“Brazil could be thought of as a barometer of what could be expected in other countries,” said
Marcelo Gomes, a researcher at the Oswaldo Cruz Foundation, a scientific institution.
“Information is paramount. The more we have, the more we can make the population aware of
what it is exactly that we are facing.”
A gravedigger at São Paulo’s Vila Formosa Cemetery, one of Latin America’s largest, said he started
noticing strange deaths in early March — weeks before Brazil’s first confirmed coronavirus death.
The causes of death — pneumonia, respiratory failure — were what he would expect to see during
the Southern Hemisphere’s winter, not peak summer.
“One day, I saw on the news that in all of São Paulo there had been 20 burials of coronavirus
victims,” said the man, who spoke on the condition of anonymity because he didn’t have
permission to give an interview. “But I knew at my cemetery alone, there had been 27.”
Pereira feels the same frustration. Some days, he can’t believe the number of people out in the
street. If they were seeing what he saw, no way would they be outside. No way would there be a
national debate over reopening the economy. No one would be listening to claims that the
pandemic had been overblown.
Bodies don’t lie, he said. And here came another one.
Adapted from (https://www.washingtonpost.com/world/the_americas/coronavirus-brazil-testing-bolsonaro-cemetery-gravedigger/2020/04/22/fe757ee4-83cc-11ea-878a-86477a724bdb_story.html)

Os limites dos testes de coronavírus no Brasil estão escondendo as verdadeiras dimensões do


maior surto da América Latina

AULA 10 – PASSIVE VOICE, TAG QUESTIONS AND IDIOMS 133


TEACHER ANDREA BELO

No topo de uma colina sombreada na orla de São Paulo, o coveiro acha que sabe a verdade. Por
pior que pareça no Brasil - o país mais atingido pelo coronavírus no Hemisfério Sul - a realidade é
bem pior.
Manoel Norberto Pereira viu mais uma carroceria sendo transportada, acompanhada do que já se
tornou um conjunto de detalhes familiares. Sexo: feminino. Idade: 77 anos. Causa da morte:
respiração insuficiente.
Cada dia traz mais. O cemitério agora recebe cerca de 50 corpos todos os dias - o dobro da média
em tempos normais. Muitos são marcados como casos confirmados de covid-19, a doença
causada pelo vírus. Mas muitos outros citam apenas uma doença respiratória não identificada.
Para Pereira, eles são o pedágio invisível do coronavírus no Brasil, que oficialmente infectou
51.000 pessoas e matou 3.400 - mas, não oficialmente, muitas vezes mais do que isso.
Testes imprecisos e insuficientes são um problema global, mas no Brasil, em uma escala
totalmente diferente. O maior país da América Latina está testando pessoas a uma taxa muito
menor do que qualquer outra nação com pelo menos 40.000 casos. Ele testa 12 vezes menos
pessoas do que o Irã e 32 vezes menos do que os Estados Unidos. Pacientes hospitalizados não
estão sendo testados. Alguns profissionais médicos não estão sendo testados. Pessoas estão
morrendo em suas casas sem serem testadas.
Pesquisadores da Universidade Federal de Minas Gerais sugeriram que o Brasil tem oito vezes mais
casos de coronavírus do que indicam os números oficiais. Uma equipe de pesquisa da
Universidade de São Paulo avalia que são 16 vezes mais - mais de 800 mil casos.
De acordo com estatísticas do governo, quase 37.300 pessoas foram hospitalizadas este ano com
doenças respiratórias - quatro vezes o número neste momento no ano passado - mas apenas
metade recebeu os resultados dos testes. Em São Paulo, cerca de 1.300 pessoas morreram de
problemas respiratórios não identificados. O estado do Amazonas relatou 193 mortes por covid-
19. Mas sua capital está enterrando tantas pessoas - três vezes a média - que está cavando
trincheiras para valas comuns.
“A grande conclusão é que não sabemos qual é o verdadeiro cenário”, disse Leonardo Costa
Ribeiro, economista da Universidade Federal de Minas Gerais. “Dá a sensação de estar mais
controlado, quando a realidade é muito diferente.”
Domingos Alves, um cientista de dados da Universidade de São Paulo, pouco fez para esconder
sua raiva pelo abismo entre como os governantes falam sobre o vírus e o que os dados mostram.
“Como pesquisador, eu olho os dados e faço análises para o governo”, disse. “Mas, como cidadão,
estou frustrado. O governo está tentando controlar a epidemia sem os elementos de como
controlá-la, por causa de sua falta de conhecimento sobre a sua gravidade.
“Vai ficar muito pior. Haverá filas nos hospitais. Haverá filas nos cemitérios. As próximas semanas
serão muito escuras. ”
O novo ministro da saúde, Nelson Teich, nomeado por Bolsonaro para substituir Mandetta, fez do
aumento dos testes um pilar da resposta do país à pandemia. Ele disse esta semana que o
governo pretende adquirir 46 milhões de exames, o suficiente para pouco mais de um quinto da
população. Ele não especificou quando os testes chegarão ou quando serão usados.

AULA 10 – PASSIVE VOICE, TAG QUESTIONS AND IDIOMS 134


TEACHER ANDREA BELO

“Não estamos falando sobre testar o país inteiro”, disse Teich. “Vamos usar os testes para que as
pessoas testadas reflitam a população brasileira.”
Mas os obstáculos serão enormes - e podem prenunciar as lutas que aguardam grande parte do
mundo em desenvolvimento à medida que o coronavírus avança cada vez mais na América Latina
e na África. Uma variável tão básica quanto dados confiáveis pode se tornar um fator de distinção
entre países pobres e ricos.
Analistas afirmam que o Brasil não tem capacidade fabril e nem capacidade de compra para
atender à demanda por testes. Seus laboratórios, já sobrecarregados por uma carteira de testes,
não estão equipados para processá-los em escala. E na disputa internacional por suprimentos e
equipamentos de teste, o país está perdendo para as nações mais ricas que podem pagar mais ou
alavancar laços mais estreitos com a China.
“O Brasil poderia ser pensado como um barômetro do que se poderia esperar de outros países”,
disse Marcelo Gomes, pesquisador da Fundação Oswaldo Cruz, instituição científica. “A
informação é fundamental. Quanto mais temos, mais podemos conscientizar a população sobre o
que exatamente estamos enfrentando ”.
Um coveiro no cemitério de Vila Formosa, em São Paulo, um dos maiores da América Latina, disse
que começou a notar mortes estranhas no início de março - semanas antes da primeira morte
confirmada de coronavírus no Brasil. As causas da morte - pneumonia, insuficiência respiratória -
eram o que ele esperava ver durante o inverno do hemisfério sul, não o pico do verão.
“Um dia, vi no noticiário que em toda São Paulo haviam ocorrido 20 sepultamentos de vítimas do
coronavírus”, disse o homem, que falou sob condição de anonimato por não ter permissão para
dar entrevista. “Mas eu sabia que só no meu cemitério havia 27.”
Pereira sente a mesma frustração. Alguns dias, ele não consegue acreditar na quantidade de
pessoas na rua. Se eles estivessem vendo o que ele via, de jeito nenhum estariam do lado de fora.
De jeito nenhum haveria um debate nacional sobre a reabertura da economia. Ninguém ouviria
as alegações de que a pandemia havia sido exagerada.
Corpos não mentem, disse ele. E aí veio outro.

Brazil ______(become) one of the global epicenters of the pandemic, recently surpassing China in
the number of reported cases. President Jair Bolsonaro ______(be) notoriously dismissive of the
virus, ______(say) earlier this week the number of deaths ______(be) declining, when in fact it
was increasing.

O Brasil ______ (tornou-se) um dos epicentros globais da pandemia, ultrapassando recentemente


a China em número de casos notificados. O presidente Jair Bolsonaro ______ (ser) notoriamente
desdenhoso do vírus, ______ (dizer) no início desta semana o número de mortes ______ (estar)
diminuindo, quando na verdade estava aumentando.

AULA 10 – PASSIVE VOICE, TAG QUESTIONS AND IDIOMS 135


TEACHER ANDREA BELO

Afghanistan: US takes control of Kabul airport to evacuate staff


There have been scenes of panic at Kabul airport as desperate residents try to flee following the
seizure of the Afghan capital by the Taliban.
The US army say soldiers shot two armed men, while three people are reported to have died after
falling ___ (1) the underside of a plane they were clinging to shortly after take-off.
The airport was closed earlier for soldiers to try and clear the runways.
US military planes are now landing, including one carrying US marines.
Marines are being flown in to help the evacuation effort. The US and other countries are rushing
to remove staff and allies from the country.
A German evacuation plane also landed at Kabul airport on Monday, sources told Reuters news
agency.
___ (2) Sunday the Taliban declared victory after Afghan President Ashraf Ghani fled abroad and
his government collapsed.
The militants' return to rule brings an end to almost 20 years of a US-led coalition's presence in
the country.
Kabul was the last major city in Afghanistan to fall to a Taliban offensive that began months ago
but accelerated in recent days as they gained control of territories, shocking many observers.
The Islamist group was able to seize control after most foreign troops pulled out.
Following the Taliban's seizure ___ (3) Kabul, many people headed to the airport.
Evacuations of foreigners and some Afghans with links to foreign governments and organisations
have been taking place, but passengers said rumours spread that even those without visas were
being allowed to travel.
(Adapted from https://www.bbc.com/news/world-asia-58227029)

Afeganistão: EUA assumem o controle do aeroporto de Cabul para evacuar a equipe


Houve cenas de pânico no aeroporto de Cabul enquanto moradores desesperados tentavam fugir
após a tomada da capital afegã pelo Taleban.
O Exército dos EUA disse que os soldados atiraram em dois homens armados, enquanto três
pessoas morreram após cair ___ (1) a parte de baixo de um avião ao qual estavam se agarrando
logo após a decolagem.
O aeroporto foi fechado mais cedo para os soldados tentarem limpar as pistas.
Aviões militares dos EUA estão pousando agora, incluindo um transportando fuzileiros navais dos
EUA.
Fuzileiros navais estão chegando para ajudar no esforço de evacuação. Os EUA e outros países
estão correndo para remover funcionários e aliados do país.
Um avião de evacuação alemão também pousou no aeroporto de Cabul na segunda-feira,
disseram fontes à agência de notícias Reuters.
___ (2) No domingo, o Taleban declarou vitória depois que o presidente afegão Ashraf Ghani fugiu
para o exterior e seu governo entrou em colapso.
O retorno dos militantes ao governo põe fim a quase 20 anos de presença de uma coalizão liderada
pelos EUA no país.

AULA 10 – PASSIVE VOICE, TAG QUESTIONS AND IDIOMS 136


TEACHER ANDREA BELO

Cabul foi a última grande cidade do Afeganistão a cair em uma ofensiva do Taleban que começou
meses atrás, mas se acelerou nos últimos dias quando eles ganharam o controle de territórios,
chocando muitos observadores.
O grupo islâmico conseguiu assumir o controle após a retirada da maioria das tropas estrangeiras.
Após a apreensão do Taleban ___ (3) Cabul, muitas pessoas se dirigiram ao aeroporto.
Evacuações de estrangeiros e alguns afegãos com ligações com governos e organizações
estrangeiras estão ocorrendo, mas os passageiros disseram que rumores se espalharam que
mesmo aqueles sem visto estavam sendo autorizados a viajar.

New Zealand enters nationwide lockdown over one Covid case


New Zealand has announced a snap lockdown after a man tested positive for Covid, the first
case in six months.
The case ___ (1) detected in Auckland, which will be in lockdown for a week, while the rest of the
country will be in lockdown for three days.
Authorities say they are working on the assumption that the new case was the Delta variant.
Just around 20% of its population has been fully vaccinated.
Coromandel, a coastal town where the infected person had visited, will be in lockdown for seven
days too.
Prime Minister Jacinda Ardern ___ (2) the toughest "level 4" rules were required - closing schools,
offices and all businesses with only essential services remaining operational.
"I want to assure New Zealand that we have planned for this eventuality. Going hard and early has
worked for us before," she said.
The patient is a 58-year-old man, who is believed to have been infectious since last Thursday.
There ___ (3) reportedly a rush at supermarkets in Auckland, as locals anticipated a snap
lockdown.
Officials said there was a need for strong response because of the fear of the Delta variant, and
because there was no clear link between the new case and the border or quarantine facilities.
Data released by New Zealand's Ministry of Health on Monday showed that all Covid-19 cases
detected at the country's border in recent weeks had been Delta.
"We have seen what can happen elsewhere if we fail to get on top of it. We only get one chance,"
Ms Ardern said in a televised national address, calling the Delta strain "a game changer".
New Zealand has been successful in eliminating the virus from within its borders, although its
international borders remain largely closed.
However, its vaccination programme has rolled out at a slow pace, with only around 20% of people
fully vaccinated and 33% of people having received one dose, according to Our World in Data.
(Adapted from https://www.bbc.com/news/world-asia-58241619)

AULA 10 – PASSIVE VOICE, TAG QUESTIONS AND IDIOMS 137


TEACHER ANDREA BELO

Nova Zelândia entra em bloqueio nacional por causa de um caso da Covid


A Nova Zelândia anunciou um bloqueio instantâneo depois que um homem testou positivo para
Covid, o primeiro caso em seis meses.
O caso ___ (1) detectado em Auckland, que ficará em bloqueio por uma semana, enquanto o resto
do país ficará em bloqueio por três dias.
As autoridades afirmam que estão trabalhando com o pressuposto de que o novo caso era a
variante Delta.
Apenas cerca de 20% de sua população foi totalmente vacinada.
Coromandel, uma cidade costeira onde a pessoa infectada visitou, também ficará fechada por sete
dias.
A Primeira Ministra Jacinda Ardern ___ (2) as regras de "nível 4" mais rígidas foram exigidas -
fechamento de escolas, escritórios e todas as empresas com apenas os serviços essenciais
permanecendo operacionais.
"Quero garantir à Nova Zelândia que planejamos essa eventualidade. Trabalhar duro e cedo já
funcionou para nós antes", disse ela.
O paciente é um homem de 58 anos, que supostamente está infectado desde a última quinta-
feira.
Há ___ (3) supostamente uma corrida nos supermercados em Auckland, já que os moradores
anteciparam um bloqueio repentino.
As autoridades disseram que havia uma necessidade de uma resposta forte por causa do medo da
variante Delta e porque não havia uma ligação clara entre o novo caso e a fronteira ou instalações
de quarentena.
Dados divulgados pelo Ministério da Saúde da Nova Zelândia na segunda-feira mostraram que
todos os casos de Covid-19 detectados na fronteira do país nas últimas semanas foram de Delta.
"Vimos o que pode acontecer em outros lugares se não conseguirmos superá-lo. Temos apenas
uma chance", disse Ardern em um pronunciamento nacional pela televisão, chamando a
variedade Delta de "uma virada de jogo".
A Nova Zelândia tem conseguido eliminar o vírus de dentro de suas fronteiras, embora suas
fronteiras internacionais permaneçam praticamente fechadas.
No entanto, seu programa de vacinação foi desenvolvido em um ritmo lento, com apenas cerca
de 20% das pessoas totalmente vacinadas e 33% das pessoas tendo recebido uma dose, de acordo
com Our World in Data.

(Título omitido propositalmente)


Born in 1981, Britney Spears is an American singer, songwriter, and actress who influenced the
revival of teen pop during the 1990s and 2000s. People often referred to her as the Princess of
Pop.

AULA 10 – PASSIVE VOICE, TAG QUESTIONS AND IDIOMS 138


TEACHER ANDREA BELO

In 2005 and 2006, she gave birth to her sons Sean and Federline; however, in 2006, she filed for
divorce from her husband Kevin Federline. They reached an agreement to share joint custody of
their sons.
Then, Spears was admitted to several drug rehabilitation facilities, and in 2007, she lost custody
of her sons to Federline. In 2008, a court placed Spears under conservatorship of her father James
Spears, giving him complete control of her life and assets for 13 years.
Now, Britney´s father is stepping down from the conservatorship. According to his lawyer, there
are no actual reasons to suspend or remove him; however, James Spears is responding to the
public battle and a petition for his suspension.
(Adapted from https://www.newsinlevels.com/products/britney-spears-free-again-level-3/)

Nascida em 1981, Britney Spears é uma cantora, compositora e atriz americana que influenciou o
renascimento do pop adolescente durante os anos 1990 e 2000. As pessoas costumavam se referir
a ela como a princesa do pop.
Em 2005 e 2006, ela deu à luz seus filhos Sean e Federline; no entanto, em 2006, ela pediu o
divórcio de seu marido Kevin Federline. Eles chegaram a um acordo para compartilhar a guarda
conjunta de seus filhos.
Então, Spears foi internada em vários centros de reabilitação de drogas e, em 2007, perdeu a
custódia de seus filhos para Federline. Em 2008, um tribunal colocou Spears sob a tutela de seu
pai, James Spears, dando a ele o controle total de sua vida e bens por 13 anos.
Agora, o pai de Britney está saindo da tutela. Segundo seu advogado, não há motivos reais para
suspendê-lo ou destituí-lo; no entanto, James Spears está respondendo à batalha pública e a uma
petição para sua suspensão.

Afghan women to have rights within Islamic law, Taliban say


The rights of women in Afghanistan will be respected "within the framework of Islamic law",
the Taliban say.
In the group's first news conference since taking control of the country ___ (1) Sunday, a
spokesman said women would be free to work but gave little detail about other rules and
restrictions.
Zabihullah Mujahid repeated that all Afghans must live "within the framework of Islam".
Rights groups fear women's freedoms could be eroded under the Taliban.
The militant group introduced or supported punishments in line with their strict interpretation of
Islam's legal system, Sharia law, when they controlled Afghanistan between 1996 and 2001.
Women had to wear the all-covering burka, and the Taliban also disapproved of girls aged 10 and
over going to school.

AULA 10 – PASSIVE VOICE, TAG QUESTIONS AND IDIOMS 139


TEACHER ANDREA BELO

In the news briefing on Tuesday, Mr Mujahid fielded several questions from the international
media about what women's rights could look like under a Taliban government.
"We are going to allow women to work and study within our frameworks," he said. "Women are
going to be very active within our society."
But he did not expand when asked ___ (2) dress codes and what roles women would be able to
have within ___ (3) country's workforce.
(Adapted from https://www.bbc.com/news/world-asia-58249952)

Mulheres afegãs devem ter direitos dentro da lei islâmica, dizem o Talibã
Os direitos das mulheres no Afeganistão serão respeitados "dentro da estrutura da lei
islâmica", afirma o Taleban.
Na primeira entrevista coletiva do grupo desde que assumiu o controle do país ___ (1) domingo,
um porta-voz disse que as mulheres estariam livres para trabalhar, mas deu poucos detalhes sobre
outras regras e restrições.
Zabihullah Mujahid repetiu que todos os afegãos devem viver "dentro da estrutura do Islã".
Grupos de direitos humanos temem que a liberdade das mulheres possa ser corroída sob o
Taleban.
O grupo militante introduziu ou apoiou punições de acordo com sua interpretação estrita do
sistema legal do Islã, a lei Sharia, quando controlou o Afeganistão entre 1996 e 2001.
As mulheres tinham de usar a burca que cobrisse tudo, e o Talibã também desaprovava que
meninas de 10 anos ou mais fossem à escola.
Na coletiva de imprensa na terça-feira, Mujahid respondeu a várias perguntas da mídia
internacional sobre como seriam os direitos das mulheres sob o governo do Taleban.
“Vamos permitir que as mulheres trabalhem e estudem dentro de nossas estruturas”, disse ele.
"As mulheres serão muito ativas em nossa sociedade."
Mas ele não se expandiu quando questionado sobre os códigos de vestimenta de ___ (2) e quais
funções as mulheres poderiam desempenhar dentro da força de trabalho do país ___ (3).

AULA 10 – PASSIVE VOICE, TAG QUESTIONS AND IDIOMS 140

Você também pode gostar